You are on page 1of 1881

TRIUMPH’S

COMPLETE
REVIEW OF
DENTISTRY

https://t.me/DentalBooksWorld
https://t.me/DentalBooksWorld
TRIUMPH’S

COMPLETE
REVIEW OF
DENTISTRY
K. Rajkumar B.Sc., M.D.S., Ph.D. R. Ramya M.D.S.
Vice Principal Professor
Professor & Head Department of Oral Pathology
Department of Oral Pathology SRM Dental College – Ramapuram
SRM Dental College – Ramapuram Chennai, Tamil Nadu
Chennai, Tamil Nadu

https://t.me/DentalBooksWorld
Sr. Manager - Publishing: Sangeetha P
Manager-Production Editorial: Pooja Chauhan
Asstt. Manager Manufacturing: Sumit Johry

Copyright © 2018 by Wolters Kluwer Health (India)

10th Floor, Tower C


Building No. 10
Phase – II
DLF Cyber City
Gurgaon
Haryana - 122002

All rights reserved. This product, consisting of the printed book, is protected by copyright. No part of this book may be reproduced
in any form or by any means, including photocopying, or utilized by any information storage and retrieval system without written
permission from the copyright owner.

The publisher is not responsible (as a matter of product liability, negligence, or otherwise) for any injury resulting from any material
contained herein. This publication contains information relating to dentistry that should not be construed as specific instructions
for individual patients. Manufacturers’ product information and package inserts should be reviewed for current information, in-
cluding contraindications, dosages, and precautions. All products/brands/names/processes cited in this book are the properties of
their respective owners. Reference herein to any specific commercial products, processes, or services by trade name, trademark,
manufacturer, or otherwise is purely for academic purposes and does not constitute or imply endorsement, recommendation,
or favoring by the publisher. The views and opinions of authors expressed herein do not necessarily state or reflect those of the
publisher, and shall not be used for advertising or product endorsement purposes.

Care has been taken to confirm the accuracy of the information presented and to describe generally accepted practices. However,
the authors, editors, and publishers are not responsible for errors or omissions or for any consequences from application of the
information in this book and make no warranty, expressed or implied, with respect to the currency, completeness, or accuracy of
the contents of the publication. Application of this information in a particular situation remains the professional responsibility
of  the practitioner. Readers are urged to confirm that the information, especially with regard to drug dose/usage, complies with
current legislation and standards of practice.

Please consult full prescribing information before issuing prescription for any product mentioned in the publication.

The publishers have made every effort to trace copyright holders for borrowed material. If they have inadvertently overlooked any, they
will be pleased to make the necessary arrangements at the first opportunity.

ISBN: 978-93-88313-21-6

Published by Wolters Kluwer (India) Pvt. Ltd., New Delhi


Compositor: Chennai Publishing Services, Chennai
Printed and bound at

For product enquiry, please contact– Marketing Department (marketing@wolterskluwerindia.co.in) or


log on to our website www.wolterskluwerindia.co.in.

https://t.me/DentalBooksWorld
Foreword

Postgraduate entrance exam preparation has become highly competitive over the years. It was unimperative earlier but has
become compulsive after the advent of the National Eligibility and Entrance Test. Conduct and implementation of NEET is
a landmark achievement in Medical and Dental education. It has helped make the education process more transparent and
standardized.
Intense preparation is required to cover the vast ocean of the subject – Dentistry. Embracing this vastness during preparation
is a challenge.
A never-ending list of books is always suggested by peers and teachers. Finding a single source reference during entrance
exam preparation is highly desirable and this preparatory manual has made all efforts to consolidate both the clinical and basic
science subjects so effectively.

I consider the following features as highlights:


• Well-conceived and structured synopsis
• Quick review facts for rapid review
• Easy-to-comprehend illustrations
• Multitude of questions in the form of image-based questions, multiple-choice questions, and short-answer questions.

This book will serve as an indispensable source of reference to the entrance exam aspirants. It can also be used for quick
revision during university entrance exam preparations.

Dr. Dibyendu Mazumder


President
Dental Council of India
New Delhi

https://t.me/DentalBooksWorld
https://t.me/DentalBooksWorld
Foreword

Preparation for postgraduate entrance examinations is considered as a herculean task as the students have to prepare from a vast
array of books, which he/she has to master for getting a successful rank at the examinations. Over the years, few preparatory
manuals have evolved to becoming a single source reference and have been widely successful. However, those books could not
achieve complete subject consolidation and most of the students use other supplementary references to make their preparation
complete. Further, a mandatory national level common entrance examination for admission to undergraduate and postgraduate
dental education has made dental examination highly competitive and challenging. Dentistry still retains as one of the best
preferred career options.
Dr. K. Rajkumar, Professor and Head, Department of Oral Pathology, and Dr. R. Ramya, Professor, Department of Oral
Pathology, SRM Dental College, Chennai, after analyzing the postgraduate examination scenario over years and with rich
experience in teaching and authoring textbooks, have now brought Triumph’s Complete Review of Dentistry. Salient features of
the book are in depth consolidation of all subjects of dentistry to make it a single source reference; structured and comprehensive
synopsis; quick learning facts for rapid revision during examinations; easy-to-comprehend illustrations; image-based questions
which is the latest trend in NEET, AIIMS, and PGI examinations; short-answer questions to assess memory retention; and
multiple-choice questions to assess memory, intuitiveness, and critical thinking skills.
This book is written keeping in mind the needs of the struggling postgraduate examination aspirants. These two authors
have already brought a book namely “Textbook of Oral Anatomy, Histology, Physiology and Tooth Morphology” in 2012 which
received an overwhelming response from the dental students.
I am confident that this book will be very much useful to postgraduate students appearing for the postgraduate common
entrance examination. I congratulate the attempts of authors of the book and I wish them a grand success in their career.

Dr. J. Radhakrishnan, i.a.s.


Principal Secretary to Government,
Health and Family Welfare Department
Government of Tamil Nadu
Secretariat, Chennai

https://t.me/DentalBooksWorld
https://t.me/DentalBooksWorld
Preface

The field of dentistry involves evaluation, diagnosis, prevention, and treatment of diseases of the oral cavity, maxillofacial area,
associated structures, and their impact on the human body. Specialization in the field of dentistry requires an undergraduate
degree in general dentistry and a postgraduate degree in at least one of the nine dental specialties—Orthodontia, Prosthodontia,
Conservative dentistry, Oral surgery, Periodontia, Pedodontia, Public health dentistry, Oral medicine, and Oral Pathology.
Admission to undergraduate and postgraduate dental education requires taking up a mandatory national level common
entrance examination. Dentistry still retains its spot as one of the best preferred career options, which makes the exams highly
competitive. Ratio of postgraduate seats to undergraduate seats in dentistry is approximately 1:4 making postgraduate exams
even more challenging.
Trending inputs from toppers of medical and dental examinations have repeatedly suggested that preparation for postgraduate
examinations starts earlier during the undergraduate days itself. Very often the student is left with a wide array of books with
which he has to master the subject in depth for getting a successful rank at the examinations. Over the years, few preparatory
manuals have evolved to becoming a single source reference and have been widely successful. However, those books could not
achieve complete subject consolidation and most of the students use other supplementary references to make their preparation
complete.
Analyzing the postgraduate examination scenario over years and with rich experience in teaching and authoring textbooks,
the lacunae that still lingers in postgraduate entrance exam training was overcome by establishing Triumph Academy which
strives as a center for excellence in dentistry. The dream of bringing out a preparatory manual that will be comprehensive in both
basic and clinical dental sciences has evolved as Triumph’s Complete Review of Dentistry (Vol. I & II).

Salient Features
• In-depth consolidation of all subjects of dentistry to make it a single source reference
• Structured and comprehensive synopsis
• Quick learning facts for rapid revision during examinations
• Over 400 photographs and illustrations
• Image-based questions which is the latest trend in NEET, AIIMS, and PGI examinations
• Short-answer questions to help assess memory retention
• Multiple-choice questions to help assess memory, intuitiveness, and critical thinking skills

It is a sincere endeavor keeping in mind the needs of the struggling postgraduate exam aspirants. The preparation of this text
took 3 years; however, we understand that there is always scope for further improvement. Please send your comments and
suggestions to triumphdentalorg@gmail.com.

K. Rajkumar
R. Ramya

https://t.me/DentalBooksWorld
https://t.me/DentalBooksWorld
Acknowledgments

The journey of authoring textbooks has been a source of enormous contentment. It has been a decade since the urge of creating
learning material that would be immensely beneficial to students arose. From then it has been a whirlwind of activities resulting
in authoring three books consecutively. Although it required efforts in gargantuan proportions, the final outcome was euphoric.
Unlike the previous textbooks which involved fewer subjects, Triumph’s Complete Review of Dentistry (Vol. I & II) involved all
the specialties of dentistry and required a robust team work.
At this momentous juncture, we submit to the divine for granting us the energy to tread this challenging path. With absolute
benediction, we extend our warmest thanks to Chancellor Dr. T.R. Pachamuthu, SRM University, Chairman Dr. R. Shivakumar,
and Dr. K. Ravi, Dean, SRM Dental College, for a providing a vibrant academic platform to nurture our dreams.
It is with profound sense of gratitude we extend our deepest thanks to Dr. Dibyendu Mazumder, President, Dental Council
of India, for accepting to write the Foreword. This unstinted support from the highest governing body of Dentistry gives us a
strong sense of accomplishment. Our earnest thanks to Dr. Radhakrishnan, IAS, Principal Secretary, Department of Health &
Family Welfare, Government of Tamil Nadu, for agreeing to give the Foreword.
We would like to express our sincere thanks to all the contributors who made this possible. Our fervent thanks to
Dr. H. Murali Rao who has been a source of tremendous support in all our endeavors. His contribution and willingness to help
to get the foreword stands unmatched.
Heartfelt thanks to the principal contributor Dr. Muthalagappan for his dedication and commitment in developing the book.
His perseverance and hard work knows no boundaries. He stands as a strong source of inspiration amidst life’s uncertainties.
We sincerely thank Dr. Anitha Srinivasan, Dr. Anupama Tadepalli, Dr. Ashwini Ragao, Dr. Barathi Prasad,
Dr.  Dhivakaran, Dr. Chitra Poornima, Dr. Devapriya Appukuttan, Dr. R. Divya Sangeetha, Dr. J.V. Deepthi, Dr.  Eapen
Cherian, Dr. N. Gopichander, Dr. Harini Priya, Dr. Kalpana Devi, Dr. R. Karthikesh, Dr. Ramya Mahalingam,
Dr.  Ravalika Singarappu, Dr. Rega, Dr. Roger Renaldo, Dr. Sangeetha Subramaniyan, Dr. Scindia, Dr. Shanthi Rajkumar,
Dr. K. Sudheerkanth, Dr. Vallirajan for their untiring efforts in giving their valuable contributions.
We would like to sincerely appreciate the team at Wolters Kluwer India, Ms. P. Sangeetha, Sr. Manager-Publishing, who has
been rendering unconditional support to us for more than a decade in achieving our dreams in print. We would also like to
acknowledge Pooja Chauhan, Production Editor, for her guidance and suggestions.

K. Rajkumar
R. Ramya

https://t.me/DentalBooksWorld
https://t.me/DentalBooksWorld
Contributors

Contributor & Reviewer Principal Contributor


Murali H. Rao Muthalagappan P.L.
Professor & Head Faculty
Department of Prosthodontics Triumph Academy
DA Pandu Memorial R.V. Dental College Chennai, Tamil Nadu (AIIMS May 2018 – Rank 36;
Bangalore, Karnataka PGIMER June 2017 & 2018 – Rank 27)

Anila Charles J.V. Deepti


Associate Professor Junior Resident
Department of Orthodontics Department of Conservative Dentistry & Endodontics
SRM Dental College – Ramapuram Chettinad Dental College and Hospital
Chennai, Tamil Nadu Chennai, Tamil Nadu

Anitha Srinivasan Devapriya Appukuttan


Associate Professor Associate Professor
Department of Prosthodontics Department of Periodontics
SRM Dental College – Ramapuram SRM Dental College – Ramapuram
Chennai, Tamil Nadu Chennai, Tamil Nadu

Anupama Tadepalli Dhivakaran R.


Associate Professor Coordinator
Department of Periodontics Triumph Academy
SRM Dental College – Ramapuram Chennai, Tamil Nadu
Chennai, Tamil Nadu
R. Divya Sangeetha
Ashwini Ragao
Junior Resident
Junior Resident
Department of Operative Dentistry and Endodontics
Department of Orthodontics & Dentofacial Orthopedics
JSS Dental College
Bapuji Dental College
Mysuru, Karnataka
Davanagere, Karnataka
(NEET 2016 – AIR 193)
(NEET 2018 – AIR 353)

Bharathi Prasad Eapen Cherian


Junior Resident Professor
Department of Orthodontics & Dentofacial Orthopedics Department of Oral Pathology
Saveetha University St. Gregorios Dental College
Chennai, Tamil Nadu Ernakulam, Kerala

Chitra Poornima N. Gopichander


Junior Resident Professor
Department of Prosthodontics and Crown & Bridge Department of Prosthodontics
Thai Mookambika Dental College SRM Dental College – Ramapuram
Chennai, Tamil Nadu Chennai, Tamil Nadu

https://t.me/DentalBooksWorld
xiv Contributors

Harini Priya Sri Rajiv Gandhi College of Dental Sciences


Assistant Professor Bangalore, Karnataka
Department of Oral Pathology (NEET 2018 – AIR 834)
Chettinad Dental College
Chennai, Tamil Nadu Rega K.
Junior Resident
Kalpana Devi Department of Operative Dentistry and Endodontics
Infertility Consultant Maulana Azad Institute of Dental Sciences
Director New Delhi
KSK Hospital (NEET 2017 – AIR 5;
Theni, Tamil Nadu AIIMS May 2017 – Rank 6)

R. Karthikesh Roger Renaldo A.


Intern Junior Resident
SRM Dental College – Ramapuram Department of Oral and Maxillofacial Surgery
SRM University KLE Institute of Dental Sciences
Chennai, Tamil Nadu Bengaluru, Karnataka
(Indian Army Dental Corps Selection List – 21st Rank)
Pavithra Karthikeyan
Junior Resident Sangeetha Subramaniyan
Department of Pedodontics and Preventive Surgery Associate Professor
PGIMER Department of Periodontics
Chandigarh SRM Dental College – Ramapuram
(PGIMER June 2017 – Rank 2 (Cat.)) Chennai, Tamil Nadu

K. Rajkumar Scindia N.
Vice Principal Junior Resident
Professor & Head Department of Prosthodontics and Crown & Bridge
Department of Oral Pathology The Tamil Nadu Government Dental College and Hospital
SRM Dental College – Ramapuram Chennai, Tamil Nadu
Chennai, Tamil Nadu (NEET 2018 – AIR 184)
(TNPGEE – Rank 1)
Shanthi Rajkumar
R. Ramya Director
Professor Nathan Dental Clinic
Department of Oral Pathology Chennai, Tamil Nadu
SRM Dental College – Ramapuram
Chennai, Tamil Nadu K. Sudheerkanth
Professor & Head
Ramya Mahalingam Department of Oral Pathology
Junior Resident GSL Dental College
Department of Oral & Maxillofacial Pathology Rajahmundry, Andhra Pradesh
SRM University
Chennai, Tamil Nadu Vallirajan
Emergency Medicine Consultant
Ravalika Singarappu Director
Junior Resident KSK Hospital
Department of Oral and Maxillofacial Surgery Theni, Tamil Nadu

https://t.me/DentalBooksWorld
Contents

Foreword by Dr. Dibyendu Mazumder v


Foreword by Dr. J. Radhakrishnan vii
Prefaceix
Acknowledgments xi
Contributors xiii

Volume I

 1. Orthodontics1
  2. Pedodontics  78
  3. General Pathology 126
  4. Oral Pathology and Oral Medicine 225
  5. General Surgery 383
 6. Embryology, Head and Neck Anatomy, and General Histology 460
  7. Oral Surgery  581
  8. Pharmacology 671
  9. Oral Radiology 749
10. Oral Anatomy and Histology 827

Volume II

11. Dental Materials 941


12. Conservative Dentistry 1092
13. Endodontics 1121
14. Prosthodontics 1155
15. Physiology 1264
16. General Medicine 1344
17. Periodontics 1440
18. Microbiology 1571
19. Community Dentistry and Biostatistics 1673
20. Biochemistry1762

https://t.me/DentalBooksWorld
https://t.me/DentalBooksWorld
1 Orthodontics

SYNOPSIS

DEVELOPMENT OF DENTITION

Eruption

Pre-emergent Post-emergent
eruption eruption

1. Resorpon of
1. Post-emergent
bone and primary
teeth spurt
2. Juvenile occlusal
and adult occlusal
2. Guidance of
equilibrium.
tooth

PRE-EMERGENT ERUPTION
• Resorption is the rate-limiting factor in the pre-emergent eruption.
• The erupting tooth has to be guided into the path created by resorption of bone.
• In the condition called primary eruption syndrome, resorption takes place properly whereas the erupting tooth does not
follow the path that has been created.

POST-EMERGENT ERUPTION
• Post-emergent spurt: This is the phase in which there is rapid movement of tooth after the tooth has penetrated the gingiva
till it reaches the occlusal level.
• Juvenile occlusal equilibrium: Here, the teeth erupt to compensate the vertical growth of the mandibular ramus. It is a slow
process.
• Significance of juvenile occlusal equilibrium is best understood when a tooth gets ankylosed.
• Adult occlusal equilibrium: This final phase of tooth eruption occurs after the pubertal growth spurt ends. The tooth
continues to erupt when the antagonist is lost and also because of wear of the tooth structure.

https://t.me/DentalBooksWorld
2 Triumph’s Complete Review of Dentistry

STAGES OF TOOTH DEVELOPMENT


• Nolla has divided the development of tooth into 10 stages.

1 2 3 4 5 6 7 8

0th Stage – Absence of crypt


1st Stage – Presence of crypt
2nd Stage – Initial calcification
3rd Stage – One-third of crown completion
4th Stage – Two-thirds of crown completion
5th Stage – Crown almost completed
6th Stage – Crown formation completed and tooth eruptive movements begins
7th Stage – One-third of root completed
8th Stage – Two-thirds of root completed (after 2–3 years) and the tooth erupts into the oral cavity
9th Stage – Root almost completed
10th Stage – Root completed

STAGES OF OCCLUSAL DEVELOPMENT

Stages of Occlusal Development

Predental Deciduous Mixed Permanent


dentition dentition dentition

1. Gum pads 1. Spaced anterior teeth 1. Erupon of permanent 1. Molar relation


first molars
2. Relationships 2. Primate spaces
2. Replacement of
3. Deep overbite primary incisors by
permanent incisors
4. Shallow overbite • Incisor liability
• Ugly duckling stage
5. Terminal plane • Early mesial shi
relation
3. Replacement of CDE
6. Vertical • Leeway space
inclination of • Late mesial shi
lower incisors • Change of terminal
planes

https://t.me/DentalBooksWorld
Chapter 1 • Orthodontics 3

1. PRE-DENTAL JAW RELATIONSHIPS (0–6 months)

Incisal papilla
Elevaons
Transverse grooves
Lateral sulcus

Gingival groove
Dental groove

MAXILLARY GUM PADS


• The alveolar arches at the time of birth are called gum pads.
• They are firm and pink.
• Maxillary gum pads develop in two parts namely, labio-buccal and lingual.
• Labio-buccal portion grows fast. It is divided into 10 segments by transverse grooves which correspond to the deciduous
tooth sac.
• The groove between the canine and the deciduous first molar is known as lateral sulcus.
• Labio-buccal and lingual parts are demarcated by the dental groove. The dental groove passes from the incisive papilla,
runs laterally, and joins with the gingival groove at the lateral sulcus area.
• Gingival groove demarcates the palate from the gum pads.

Dental groove
Gingival groove
Lateral sulcus
Elevaons

LOWER GUM PADS


• Lower gum pads are U-shaped.
• Anteriorly, the lower gum pad is everted.
• Like maxillary gum pad, lower gum pad is also divided into 10 segments by the transverse groove but is less defined.

Maxillary gum pad


Maxillary lateral sulcus anterior
Contact in posterior area
Mandibular lateral sulcus
Mandibular gum pad

RELATIONSHIPS OF GUM PADS


• Gum pads do not have a definite relationship.
• The upper gum pad is wider than the lower gum pad and there is total overlapping of maxillary gum pads anteriorly and
posteriorly.
• Lower sulcus is distal to the upper lateral sulcus.
• The gum pads grow rapidly during the first year of life and the amount of growth is more in the transverse direction.
• Length of gum pads also increases, mostly posteriorly to accommodate the deciduous first and second molars.

https://t.me/DentalBooksWorld
4 Triumph’s Complete Review of Dentistry

2. DECIDUOUS DENTITION PERIOD (6 months–6 years)


a. Sequence of eruption of primary teeth:
• Eruption of primary teeth starts 6–7 months after birth.
• Delay in eruption by 4–10 months is considered normal.
• Mandibular teeth usually precede the maxillary arch, in the following order: A – B – D – C – E.

b. Chronology of primary dentition:

Deciduous teeth Age of eruption (in months)


Lower central incisor 7
Upper central incisor 9
Upper and lower lateral incisors 11
First molars 15
Canines 18
Second molars 26

c. Features of deciduous dentition:


i. Spaced dentition or open dentition:
• Primary dentition that has interdental spaces is called spaced dentition or open dentition.

There are two types of spaces:


1. Developmental spaces:
• Also known as physiologic or generalized spacing.
• Present throughout primary dentition.
• Occurs due to anteroposterior growth of jaws.
• This space is on an average of 4 mm in maxilla and 3 mm in mandible.

2. Primate spaces:
• Also known as simian space or anthropoid space or Baume space.
• In the maxilla, primate space is seen between deciduous lateral incisor and canine.
• In the mandible, primate space is found between deciduous canine and first molar.
• These spaces are used in early mesial shift.

ii. Closed dentition or nonspaced dentition:


• Primary teeth without any spaces are called closed dentition.
• Closed dentition would eventually lead to crowding in the permanent dentition.

iii.  Deep bite:


• Once the primary incisors are erupted, the overbite is deep due to their vertical inclination.
• This deep bite reduces over a period of time because of the eruption of primary molars and rapid attrition of incisors.

iv.  Overbite:
• The average overbite in primary dentition is 1–2 mm.

v. Terminal plane relationships:


• Baume classified the relationships of primary teeth into three categories, based on the relationship of upper and lower
primary second molars in the sagittal direction.
• Straight or flush terminal plane – seen in 76%
• Mesial step – seen in 14%
• Distal step – seen in 10%

https://t.me/DentalBooksWorld
Chapter 1 • Orthodontics 5

General features of deciduous dentition


• Dental arches are normally ovoid in shape.
• Deep bite present initially which converts to an edge–edge relationship
• Presence of primate/developmental spaces
• Shallow intercuspal contact
• Vertical inclination of incisors
• Flat curve of Spee
• Minimal Overjet
• Absence of crowding

3. MIXED DENTITION PERIOD (6–12 years)


• Transition from primary dentition to permanent dentition begins at 6 years of age with the eruption of permanent first
molars and permanent incisors.

Mixed dentition

Eruption of molars Replacement of


Intertransitional
+ replacement canine and
period
of incisors deciduous molars

First transitional No significant Second transitional


period changes take place period

Eruption of first permanent molar:

Early mesial shift:


• In patients with spaced primary dentition and flush terminal plane, when the permanent mandibular first molar emerges
at 6 years, they close the primate space distal to canine.
• Thus, the flush terminal plane is shifted into the mesial step.
• This will allow the permanent maxillary first molars to erupt in a class I molar relationship.
• This mechanism is called early mesial shift.

Replacement of incisors/Incisal liability:


• Usually, the mesiodistal width of the permanent incisors is larger than the mesiodistal width of the primary incisors.
• This difference between the amount of space needed for the permanent incisors to align themselves and the amount of
space available for them is called Incisor liability.
• This incisor liability was explained by Warren Mayne in 1969.
• Incisor liability becomes favorable when the primary dentition is a spaced dentition.
• The incisor liability is about 7.6 mm in the maxillary arch and 7 mm in the mandibular arch.
• The space discrepancy is compensated by the following three mechanisms:

https://t.me/DentalBooksWorld
6 Triumph’s Complete Review of Dentistry

1. Increased intercanine width: It is about 3–4 mm.


2. Utilization of interdental spacing.
3. Labial eruption of incisors.

UGLY DUCKLING STAGE


• Also known as Broadbent’s phenomenon, Physiologic median spacing.
• It is a form of transient malocclusion, wherein midline diastema is present between the maxillary central incisors.
• During eruption of maxillary canine, the canine will be impinging on the roots of lateral incisors.
• This effect of canine will cause the lateral incisor to erupt into the oral cavity with divergence of crown distally.
• The persistent pressure of canine is transmitted to central incisors also, which causes the crowns to diverge and roots to
converge toward midline.
• This bilateral effects of canine causes midline diastema, which is temporary.
• This temporary spacing will be closed automatically when the canine comes into occlusion.
• As it represents a metamorphosis from an unaesthetic phase to an aesthetic phase, this stage is called the ugly duckling
stage.
Clinical importance:
• As is rule, maxillary midline diastema of around 2 mm closes spontaneously.
• Any attempt to close midline diastema during ugly duckling stage will be dangerous.
–– Apex of lateral incisors will be damaged.
–– Canine may be deflected from its normal path of eruption.

Replacement of canine and primary molars as second transitional stage:

Leeway space of Nance:


• The combined mesiodistal width of C, D, and E is greater than the combined mesiodistal width of the permanent canine,
first, and second molars.
• This difference in space is called Leeway space of Nance.
• In maxillary arch, leeway space is about 0.9 mm in each segment, with a total of 1.8 mm.
• In mandibular arch, it calculates about 1.7 mm in each segment, with a total of 3.4 mm.

Transition of molar occlusal relationship from mixed dentition to permanent dentition:


The two major contributions to this molar transition are:
• Late mesial shift of molar: After shedding of second primary molar, the first permanent molar shifts mesially.
–– This mesial shift is more in case of lower molars as compared to upper molars because of the more amount of leeway
space.
• Differential growth of mandible relative to maxilla: Because of the cephalocaudal growth, mandible grows more than
maxilla. This is the second contributor to this molar transition.

Some features of normal occlusion in permanent dentition:


• Antero-posterior curvature in the mandibular arch is called the Curve of Spee.
• Corresponding curve in the maxillary arch is called the Compensating curve.
• Buccolingual curvature from one side to the other side is called Monson’s curve.
• Overbite: Normal is 10–30%
• Overjet: 1–3 mm
• With respect to molar relationship (Class I molar), mesiobuccal cusp of maxillary first permanent molar occludes in the
mesiobuccal groove of the mandibular first permanent molar.

https://t.me/DentalBooksWorld
Chapter 1 • Orthodontics 7

TRANSIENT MALOCCLUSIONS
• Transient malocclusions, otherwise called Self-correcting malocclusions are those conditions which look like
malocclusions at some particular time. But with continuous normal growth, these conditions get corrected on their own
without any treatment.
• Transient malocclusions are seen in predental, primary, and mixed dentition.

PREDENTAL JAW RELATION STAGE


S. No. Transient malocclusion Reasons for correction
1. Retrognathic mandible Corrects with cephalocaudal growth and differential growth of mandible
2. Complete overlap of maxillary gum pad Transverse and sagittal growth of mandibular gum pad
3. Anterior open bite Eruption of primary incisors
4. Infantile swallow With initiation of function at about 18 months of age

PRIMARY DENTITION
S. No. Transient malocclusion Reason for correction
1. Deep bite –– Eruption of primary molar
–– Attrition of incisal edges
2. Increased overbite More forward growth of mandible
3. Flush terminal plane Early mesial shift
4. Spacing Closes with eruption of permanent successors
5. Flush terminal plane Shedding of primary incisors and eruption of permanent incisors

MIXED DENTITION
S. No. Transient malocclusion Reason for correction
1. Deep bite –– Physiologic bite raisers at 6, 12 years with the eruption of first and
second permanent molars.
–– The overlying gingival pad of tissue will act as a bite raiser.
2. Ugly duckling stage Eruption of maxillary canine
3. Lower anterior crowding Increase in intercanine width
4. End on molar relationship Late mesial shift
5. Edge to edge at the age of 6 years, flush Both late mesial shift and differential jaw growth
terminal plane relationship

Andrew’s keys of static occlusion

Keys of occlusion

Roth’s keys of functional occlusion

https://t.me/DentalBooksWorld
8 Triumph’s Complete Review of Dentistry

ANDREW’S KEYS OF STATIC OCCLUSION


Andrew’s six keys of static occlusion (1972) is based on 120 nonorthodontic normal.
KEY 1: MOLAR RELATION
• The mesiobuccal cusp of the upper first molar rests in the mesiobuccal groove of the lower first molar.
• Distal surface of the distobuccal cusp of the upper first molar should occlude with the mesial surface of the mesiobuccal
cusp of the lower second molar.
• The mesiolingual cusp of the upper first molar should occlude in the central fossa of the lower first molar.
KEY 2: CROWN ANGULATIONS (TIP)
• The gingival portion of long axis of each crown should be distal to the incisal portion.
KEY 3: CROWN INCLINATION (TORQUE)
• Refers to the buccolingual inclination of the long axis of the crown, not to the long axis of the entire tooth.
• It is determined by the resulting angle between a line perpendicular to the occlusal plane and one tangent to the middle of
the labial or buccal clinical crown.
• Negative crown inclination (lingual inclination) occurs in the maxillary and mandibular posteriors.
• Positive crown inclination (labial inclination) occurs in the maxillary incisors.
KEY 4: ROTATION
• Absence of rotation in the arch indicates a normal static occlusion.
• A rotated molar occupies more mesiodistal space than a rotated incisor.
KEY 5: INTERPROXIMAL CONTACT
• Proximal contacts should be tight with devoid of spacing.
KEY 6: CURVE OF SPEE
• Flat curve of Spee is most receptive for normal occlusion.
• Deep curve of Spee results in more confined area (crowding).
• Reverse curve of Spee results in excessive Spacing.

ROTH’S KEYS OF FUNCTIONAL OCCLUSION


Key 1: Coincidence of intercuspal position (ICP) and retruded contact position (RCP)
Key 2: Maximum and stable cusp to fossa contacts throughout the buccal segments.
Key 3: On mandibular protrusion, there should be disclusion of posterior teeth with even contacts on the incisors.
Key 4: Lateral mandibular movements are guided by the working side canine contacts, with disclusion of all the other teeth
on both working and nonworking sides.

ANGLE’S CONCEPT OF NORMAL OCCLUSION


CLASS I NORMAL OCCLUSION
• Molars are in class I relation.
• Lines of occlusion are intact in both the arches.
• Presence of full complement of teeth.

LINE OF OCCLUSION
Maxillary line of occlusion: The smooth curve passing through the central fossa of the upper molars and along the cingulum
of the upper canines and incisors.
Mandibular line of occlusion: It runs along the buccal cusps of posterior and incisal edges of the anteriors.

https://t.me/DentalBooksWorld
Chapter 1 • Orthodontics 9

ETIOLOGY AND CLASSIFICATION OF MALOCCLUSION


ETIOLOGY

Graber’s Classification:
1. Hereditary
2. Congenital – Cleft lip and palate, cleidocranial dysplasias
3. Environmental
• Prenatal (trauma, maternal diet, German measles, maternal metabolism, etc.)
• Postnatal (birth injury, cerebral palsy, TMJ injury)
4. Predisposing metabolic and climate diseases
• Endocrine imbalances – hypothyroidism and hyperthyroidism
• Metabolic disturbances
• Infectious diseases
5. Dietary problems (nutritional deficiency)
6. Abnormal pressure habits and functional aberrations
• Abnormal sucking
• Thumb and finger sucking
• Tongue thrust
• Lip and nail biting
• Abnormal swallowing habits (improper deglutition)
• Speech defects
• Respiratory abnormalities (mouth breathing, etc.)
• Tonsils and adenoids
• Psychogenetics and bruxism
7. Posture
8. Trauma and accidents
Local Factors
1. Anomalies in number
2. Anomalies of tooth size
3. Anomalies of tooth shape
4. Abnormal labial frenum:mucosal barriers
5. Premature loss of deciduous teeth
6. Prolonged retention of deciduous teeth
7. Delayed eruption of permanent teeth
8. Abnormal eruptive path
9. Ankylosis

HISTORY REVIEW
John Hunter was the first to describe about normal occlusion.
1829 – The earliest scientific description of malocclusion was given by Samuel S Fitch, in his book “A System of Dental
Surgery”, 1829. He was the first to classify malocclusion into four states of irregularity
1842 – Georg Carabelli coined the terms “edge-to-edge bite” and “overbite”
His classification was based on the positions of incisors and canines which he termed as:
• Mordex normalis: normal occlusion
• Mordex rectus: edge to edge
• Mordex apertus: open occlusion
• Mordex prorsus: protruding occlusion
• Mordex retrosus: retruding occlusion
• Mordex tortuosus: zig-zag occlusion

https://t.me/DentalBooksWorld
10 Triumph’s Complete Review of Dentistry

1880 – Norman Kingsley – Classified malocclusion into two broad categories based on etiology
• Developmental malocclusion
• Accidental malocclusion

1899, 1900 – Edward H Angle – Detailed description of malocclusion and classified them into three categories
1912 – Lischer – Termed disto-occlusion and mesio-occlusion
1915 – Martin Dewey – Modified Angle’s classification
1920 – Paul Simon – Based on Gnathostatic and canine law
1964 – Ballard and Wayman – British classification based on incisor Overjet
1969 – Ackerman and Profitt classification – Based on Venn diagram
1992 – Katz classification – Based on Premolar

Normal Occlusion:
Molar relationship: The mesiobuccal cusp of the maxillary first molar is aligned with the buccal groove of the mandibular
first molar. There is alignment of the teeth, normal overbite, and overjet and coincident maxillary and mandibular midlines.
Canine relationship: The maxillary canine occludes with the distal half of the mandibular canine and the mesial half of the
mandibular first premolar.
Line of occlusion: The teeth all fit on a line of occlusion which, in the upper arch, is a smooth curve through the central
fossae of the posterior teeth and cingulum of the canines and incisors, and in the lower arch, is a smooth curve through the
buccal cusps of the posterior teeth and incisal edges of the anterior teeth.

ANDREW’S SIX KEYS TO NORMAL OCCLUSION


Key 1 Molar interarch relationship
Key 2 Crown angulation
Key 3 Crown inclination
Key 4 Absence of rotation
Key 5 Tight contacts
Key 6 Flat curve of Spee

ANGLE’S CLASSIFICATION (1899)


Edward H. Angle published the first classification for malocclusion.
The classifications are based on the relationship of the mesiobuccal cusp of the maxillary first molar and the buccal groove of
the mandibular first molar.

ANGLE CLASS I MALOCCLUSION


Molar relationship: (Same as normal occlusion exits but there is crowding, misalignment of teeth, crossbite etc.) The
mesiobuccal cusp of the maxillary first permanent molar occludes with the mesiobuccal groove of the mandibular first
permanent molar.
Canine relationship: The mesial incline of the maxillary canine occludes with the distal incline of the mandibular canine.
The distal incline of the maxillary canine occludes with the mesial incline of the mandibular first premolar.
Line of occlusion: Altered in the maxillary and mandibular arches
–– Individual tooth irregularities (crowding/spacing/other localized tooth problems)
–– Interarch problems (open bite/deep bite/crossbite)

https://t.me/DentalBooksWorld
Chapter 1 • Orthodontics 11

ANGLE CLASS II MALOCCLUSION


Molar relationship: The molar relationship shows that the mesiobuccal groove of the mandibular first molar is distally
(posteriorly) positioned when in occlusion with the mesiobuccal cusp of the maxillary first molar. Usually the mesiobuccal
cusp of maxillary first molar rests in between the first mandibular molar and the second premolar.
Canine relationship: The mesial incline of the maxillary canine occludes anteriorly with the distal incline of the mandibular
canine. 
The distal surface of the mandibular canine is posterior to the mesial surface of the maxillary canine by at least the
width of a premolar.
Line of occlusion is not specified but irregular, depending on facial pattern, overcrowded teeth, and space needs.
Class II Malocclusion has two subtypes to describe the position of anterior teeth:
• Class II division 1: The molar relationships are like that of Class II and the maxillary anterior teeth are protruded. Teeth
are proclaimed and a large overjet is present.
• Class II division 2: The molar relationships are Class II where the maxillary central incisors are retroclined. The maxillary
lateral incisor teeth may be proclaimed or normally inclined. Retroclined and a deep overbite exists.
• Class II sub-division: Class II molar relationship exists on one side and the other side has a normal Class I molar
relationship

Feature Class II division 1 Class II division 2


Profile Convex Convex/Straight
Lips Incompetent Competent
Lower facial height Increased/Normal Decreased
Arch form V-shaped Squared/U-shaped
Palate Deep Normal
Incisors Proclined Central incisors are retroclined and Lateral incisors are proclined
Overjet Increased Decreased
Overbite Deep Closed
Path of closure Normal Backward
Mentalis muscle activity Hyperactive Normal

ANGLE CLASS III MALOCCLUSION


Molar relationship: The mesiobuccal cusp of the maxillary first permanent molar occludes distally (posteriorly) to the
mesiobuccal groove of the mandibular first molar.
Canine relationship: Distal surface of the mandibular canines are mesial to the mesial surface of the maxillary canines by at
least the width of a premolar. Mandibular incisors are in complete crossbite
Line of occlusion is not specified but irregular, depending on facial pattern, overcrowded teeth and space needs
Class III malocclusion has two subdivisions:
1. True class III malocclusion (skeletal) which is genetic in origin due to excessively large mandible or smaller than normal
maxilla.
2. Pseudo class III malocclusion (false or postural) which occurs when mandible shifts anteriorly during the final stages of
closure due to premature contact of incisors or the canines. Forward movement of the mandible during jaw closure can
also result from premature loss of deciduous posterior teeth.
Class III sub-division: Class III molar relationship exists on one side and the other side as a normal Class I molar
relationship.

https://t.me/DentalBooksWorld
12 Triumph’s Complete Review of Dentistry

Normal occulusion Class I malocclusion

Class II malocclusion Class III malocclusion

INCIDENCE
Type of malocclusion Incidence
Class 1 50–60%
Class 2 20–30%
Class 3 10–15%

DEWEY’S MODIFICATION OF ANGLES CLASS-1 RELATION


Type 1 – Anterior teeth crowding
Type 2 – Maxillary incisor proclination
Type 3 – Anterior crossbite
Type 4 – Posterior crossbite
Type 5 – Permanent molar drifted mesially
Class III modifications of Dewey:
Type 1: The upper and lower dental arches when viewed separately are in normal alignment. But when the arches are made
to occlude the patient shows an edge to edge incisor alignment, suggestive of a forwardly moved mandibular dental arch.
Type 2: The mandibular incisors are crowded and are in lingual relation to the maxillary incisors.
Type 3: The maxillary incisors are crowded and are in crossbite in relation to the mandibular anteriors.
Anterior Tooth Positions
Overjet is a term used to describe the distance between the labial surfaces of the mandibular incisors and the incisal edge of
the maxillary incisors.
Anterior crossbite is a malrelation between the maxillary and mandibular teeth when they occlude with the antagonistic
tooth in the opposite relation to normal.
A posterior crossbite is present when posterior teeth occlude in an abnormal buccolingual relation with the antagonistic teeth.
Posterior crossbites can be the result of either malposition of a tooth or teeth, and/or the skeleton.
Examining the transverse dimension allows us to evaluate the intermolar and intercanine widths and determine which arch
is the offending unit.
Posterior crossbites can be unilateral or bilateral.
A functional crossbite results from an occlusal interference that requires the mandible to shift either anteriorly and/or
laterally in order to achieve maximum occlusion.
Overbite is the amount of overlap of the mandibular anterior teeth by the maxillary anterior teeth measured perpendicular
to the occlusal plane.
Open bite is present when there is no vertical overlap of the maxillary and mandibular anterior teeth or no contact between
the maxillary and mandibular posterior teeth.

https://t.me/DentalBooksWorld
Chapter 1 • Orthodontics 13

Drawbacks of Angle’s classification of malocclusion


Angle’s classification suffers from many limitations:
1. Angle grouped all the possible malocclusions in three classes of anteroposterior deviations. The range of discrimination
from class I to class II (disto) to class III (mesio) which was initially full cusp width of maxillary first molar (1899), was
further modified to half cusp width (1904, 1907)
2. Angle considered the evaluation of all teeth at harmony line and initially used maxillary first molars and canines as
reference points (1899), which later was modified with greater emphasis on maxillary first molar (1904, 1907)
3. Angle’s conviction of maxillary first molar being the most stable landmark, was later found not to be consistently true.
There can be anatomical variations of location of the maxillary first molar in the maxillary arch and the jawbones 24’25
4. Angle presupposed that all malocclusions are predominantly exhibited in anteroposterior direction and accordingly
classified them on the basis of sagittal deviations. Vertical and transverse dimensions were not considered
5. Angle’s classification is based upon criterion of dentition alone and it does not take into account the underlying craniofacial
relationship
6. It does not indicate the severity and complexity of malocclusion and hence, does not point out the need for treatment
7. This classification also does not draw attention to the etiological factors associated with malocclusion
8. This classification cannot be applied to deciduous dentition and requires considerable experience for its correct evaluation
in transition and deciduous dentition stages
9. Not suitable for measuring the orthodontic treatment needs of the society
10. Angle’s classification has difficulties in its application when there is an associated discrepancy between right and left sides
or where tooth movements have occurred because of factors such as crowding and premature loss of deciduous teeth
11. Interexaminer and intraexaminer errors in categorizing Angle class II, div. 2 malocclusions are relatively high
12. He did not consider bimaxillary/bidental malocclusions

SIMON’S CLASSIFICATION OF MALOCCLUSION


Malocclusion can occur in antero-posterior, transverse, and in the vertical planes. Simon had put forward a craniometric
classification of malocclusion that related the dental arches in all these three planes.
Simon’s system of classification of malocclusion made use of three anthropometric planes, that is, the Frankfort horizontal
plane, the orbital plane, and the midsagittal plane. The classification of malocclusion was based on abnormal deviations of the
dental arches from their normal position in relation to these three planes.
Frankfort horizontal plane
This is a plane that connects the upper margin of the external auditory meatus to the infraorbital margin. This plane is used
to classify malocclusions in a vertical plane. Two terms are used to describe any abnormal relation of the teeth to this plane.
When the dental arch or part of it is closer than normal to the Frankfort plane, it is called attraction. When the dental arch or
part of it is farther away from the Frankfort horizontal plane, it is called abstraction.
Orbital plane
This plane is perpendicular to the Frankfort horizontal plane, dropped down from the bony orbital margin directly under the
pupil of the eye. According to Simon, this plane should pass through the distal third of the upper canine. This plane is used to
describe malocclusion in a sagittal or antero-posterior direction. When the dental arch or part of it is farther from the orbital
plane, it is called protraction. When the arch or part of it is closer or more posteriorly placed in relation to this plane, it is
called retraction.
Midsagittal plane
The midsagittal plane is used to describe malocclusion in the transverse direction. When a part or whole of the arch is away
from the midsagittal plane, it is called distraction. When the arch of part of it is closer to the midsagittal plane, it is called
contraction.

https://t.me/DentalBooksWorld
14 Triumph’s Complete Review of Dentistry

A B C

A – Horizontal – Attraction/Abstraction
B – Orbital – Protraction/Retraction
C – Midsagittal – Contraction/Distraction

ACKERMAN–PROFFIT SYSTEM OF CLASSIFICATION OF MALOCCLUSION


Ackerman and Proffit in 1960 proposed a diagrammatic classification of malocclusion to overcome the limitations of the
Angle’s classification of malocclusion. Salient features of the classification include:
a. Transverse as well as vertical discrepancies can be considered in addition to antero-posterior malrelations.
b. Crowding and arch asymmetry can be evaluated.
This system of classification of malocclusion is based on the Venn symbolic diagram that identifies five major characteristics
to be considered and described in the classification.
Step 1 (Alignment)
The first step involves assessment of the alignment and symmetry of the dental arch. It is classified as ideal/crowded/spaced.
Step 2 (Profile)
It involves the consideration of the profile. The profile is described as convex/straight/concave. The facial divergence is also
considered, i.e., anterior or posterior divergence.
Step 3 (Type)
The transverse skeletal and dental relationships are evaluated. Buccal and palatal crossbites if any are noted. The crossbite is
further sub-classified as unilateral or bilateral. In addition, differentiation is made between skeletal and dental crossbite.
Step 4 (Class)
This involves the assessment of the sagittal relationships. It is referred to as Angle’s Class I/Class II/Class III classification of
malocclusion. Differentiation is made between skeletal and dental malocclusion.
Step 5 (Bite depth)
Malocclusions in the vertical plane are noted. They are described as anterior or posterior open bite, anterior deep bite or
posterior collapsed bite. A mention is made whether the malocclusion is skeletal or dental.
Strengths and Advantages:
1. Cases with only arch length problems are recognized
2. Influence of dentition on the profile is considered
3. Malocclusion can be recorded in all the three planes of space
4. Skeletal and dental problems can be segregated at appropriate levels
5. Diagnosis is inherent in this methodology
6. The classification can be modified to be used on computers for large surveys and data analysis
7. Computer compatibility makes it amenable to data storage, retrieval, and processing
8. Quantification and assessment of severity of malocclusion can be done in this system
9. Can serve as an aid in treatment planning
10. Useful teaching tool

https://t.me/DentalBooksWorld
Chapter 1 • Orthodontics 15

Limitations:
1. Very detailed and therefore time consuming and tedious
2. Does not include etiology
3. Only static view of occlusion considered
4. Communication is not easy without thorough knowledge of the system

VENN DIAGRAM

Sagial
Transverse
Deviation
Deviaon
• Class I
• Buccal Trans- • Class II Division 1
• Palatal Sagial • Class II Division 2
• Class III
• Unilateral • Dental
• Bilateral Trans- • Skeletal
Sagio-
• Dental Vertical
• Skeletal
Vertico- Sagio-
Transverse Vercal

Vercal Deviations • Anterior Divergent


• Open bite anterior • Posterior Divergent
Profile
• Open bite posterior • Convex
• Deep bite anterior • Concave
• Collapsed bite, posterior • Straight
• Dental
• Skeletal

• Ideal
• Crowding
• Spacing
Intra Arch Alignment

PREMOLAR CLASSIFICATION BY KATZ


Premolar class I: It is identified when the most anterior upper premolar fits exactly into the embrasure created by the distal
contact of the most anterior lower premolar
Premolar class II: Here, the most anterior upper premolar is occluding mesial of the embrasure created by the distal contact of
the most anterior lower premolar
Premolar class III: Here, the most anterior upper premolars occluding distal of the embrasure created by the distal contact of
the most anterior lower premolar
For deciduous and mixed dentition cases: In class I situation, the center axis of the upper first deciduous molar should split the
spell between both lower deciduous molars. In the event that an upper first deciduous molar is prematurely lost, a line drawn
through the center axis of the edentulous space should bisect the embrasure between the two lower deciduous molars

https://t.me/DentalBooksWorld
16 Triumph’s Complete Review of Dentistry

CEPHALOMETRICS
Landmarks in Cephalometric Analysis
Subspinale/point A Downs
Orbitale A Bjork
Rhinion Chaconas
Midpoint of entrance of sella (se) Schwartz
Pterygomandibular fissure Moyers
Basion Coben
Bolton point Broadbent
Opisthion Graber
Articulare Bjork and Coben
Gonion Bilmer
Machine porion Moyers
Point B/subspinale Downs
Key ridge Sassouni

• Discovery of X-rays in 1895 – by Sir William Conrad Roentgen proved to be a boon.


• Pacini in 1922 – presented the concept of standardized radiographic head images.
• **In 1931 – Hofrath in Germany and Broadbent in the United States – individually and simultaneously refined the
technique and applied these principles to orthodontics – Cephalostat.
• During these time period – Simon’s system of gnathostatics was in use – (a method for orienting orthodontic casts to the
Frankfort horizontal plane)

Uses of cephalometrics:
1. Study of craniofacial growth
2. Diagnosis of craniofacial deformity
3. Treatment planning
4. Evaluation of treated cases
5. Study of relapse in orthodontics
**Distance from the X-ray source to the subject midsagittal plane is kept at 5 feet.
**The distance from the midsagittal plane to the Cassette can vary in different machines – but must be the same for each
patient every time.

Obtaining the Cephalogram:

SE N
S

Co
ANS
Ba PNS
A

PAS B
Go

H Gn
Me

https://t.me/DentalBooksWorld
Chapter 1 • Orthodontics 17

Midsagittal plane
Posion “B”
of the film

Posion “A” X-ray source


of the film

Collimator
X-ray beam

Central X-ray beam

152.4cm

1. Cephalometric equipment
a. X-ray source
b. Cassette holder
c. Two types of cephalostat:
(i) Broadbent–Bolton method: Utilizes two sources and two film holders, which helps in unnecessary movement of the
subject between lateral and posteroanterior exposures.
Also helps in making precise 3-dimensional studies possible. Precludes oblique projections.
(ii) Higley method: Used in modern cephalostats.
• One X-ray and One film holder.
• Cephalostat capable of being rotated.
• Patient is repositioned during various projections.
2. Lateral projection
a. Midsagittal plane
b. Posteroanterior projection
c. Oblique projection
(i) Midsagittal plane of patients head is conventionally placed at 60 inches from the target of X-ray tube with left side
(European convention is right side of subject).
(ii) Posteroanterior projection: Head rotated by 90 degrees so that the central ray perpendicularly bisects the transmeatal
axis. All vertical displacements measured are altered when head is tilted.
(iii) Oblique direction: The right and the left oblique cephalograms are taken at 45° to lateral projection. Central ray
entering behind one ramus to obviate superimposition of halves of the mandible. Frankfort plane must stay horizontal –
any tipping will alter the measurements. Particularly useful for patients in the mixed dentition.

https://t.me/DentalBooksWorld
18 Triumph’s Complete Review of Dentistry

CEPHALOMETRIC LANDMARKS
Hard tissue landmarks

N
S

Or
Po Co
Bo Ar PTM
Ba ANS
PNS
A
Pr

Id
Go B
Pog
Me Gn

Soft tissue landmarks

G
N′

P
Sn
SLS
Stm Ls
Stm
Li
ILS
Pog′

Me′

1. Cranial landmarks: nasion, basion, Bolton, sella


2. Maxillary landmarks: point A, ANS, PNS, PTM, key ridge
3. Mandibular landmarks: point B, gonion, menton, pogonion, gnathion, articulare, condylion

1. Glabella: Most prominent point on the forehead in the midsagittal plane


2. Nasion: Most anterior point midway between the frontal and nasal bone on the frontonasal suture.
3. Orbitale: Lowest point on the inferior bony margin of the orbit
4. Porion: Highest bony point on the upper margin of external auditory meatus
5. Sella: Point representing the mid point of pituitary fossa or sella turcica
6. Basion: Median point of anterior margin of foramen magnum
7. Anterior nasal spine: Anterior tip of sharp bony process of maxilla in the midline of lower margin of anterior nasal spine
8. Point A: Deepest point in the midline between the anterior nasal spine and the alveolar crest between two central incisors

https://t.me/DentalBooksWorld
Chapter 1 • Orthodontics 19

9. Prosthion: Lowest and the most anterior point on the alveolar bone in the midline between the upper central incisors,
also known as Supra Dentale
10. Key ridge: Lower most point on the contour of the anterior wall of the infratemporal fossa.

11. Posterior nasal spine: Intersection of continuation of anterior wall of pterygopalatine fossa and the floor of the nose,
marking the distal limit of maxilla.
12. Broadbent registration point: Midpoint of the perpendicular from the center of the sella turcica to the Bolton point.
13. PTM point: Intersection of inferior border of foramen rotundum with the posterior wall of pterygomaxillary fissure.
14. Chelion: Lateral terminus of the oral slit on the outer corner of the mouth.

15. Subnasal: The point where the lower border of the nose meets the outer contour of the upper lip.

S G Glabella

N Nasion
Sella

Po Co
Orbitale
Or ANO
Porion Anterior
Ptm Nasal Spine
Bn PN A Point A
Basion

Pterygomaxillare
Gonion Go B Point B
Pog Pogonion
Posterior Nasal
Spine Gn Gnathion
Me
Menton

https://t.me/DentalBooksWorld
20 Triumph’s Complete Review of Dentistry

BIOMECHANICS IN ORTHODONTICS
BASICS OF TOOTH MOVEMENT

PHYSIOLOGIC TOOTH MOVEMENT


Physiologic tooth movement comprises primarily the slight tipping of the functioning tooth in its socket and secondarily, the
changes in the position of tooth in a young person during and after tooth eruption. It has three types:
1. Movement during mastication:
• It depends upon the location of neutral axis of the functioning tooth.
• For younger patients, the neutral axis is either located in the marginal region or closer to the middle of the root if the
root is fully developed.
• For adults, it is located between the middle and apical regions of the roots.

Centre of resistance:
Point in the object at which the resistance to movement is at the maximum.
If a force is applied to the center of resistance – there will be bodily movement.
Centre of resistance of tooth is variable:
1. Root morphology
2. No. of roots
3. Alveolar bone height
4. Root length

Types of tooth movements:


1. Translation
2. Pure rotation
3. Combination of both rotation and translation
Translation: Bodily movement – All points of tooth move in an equal distance. Pure translatory movements – During
orthodontic extrusion or intrusion or bodily movement during retraction or protraction of teeth

Structure Centre of resistance


Single rooted tooth One-third to one-fourth the distance from alveolar crest to apex
Molars At furcation
Maxillary dentition Apical to and between the roots of premolars
Maxilla Postero superior to zygomaxillary suture or slightly inferior to orbitale
For intrusion of maxillary molars Distal to lateral incisor roots
Mandibular dentition Apical and between the roots of premolars

Type of tooth movement Force Centre of rotation


Tipping (uncontrolled) 25–60 B/w center of resistance and apex
Controlled tipping 25–60 At the root apex
Translation 70–120 At infinity
Root movement 50–100 At incisal edge
Rotation 35–60 No net force acts at crest

https://t.me/DentalBooksWorld
Chapter 1 • Orthodontics 21

• Optimum orthodontic force according to Schwartz = 20–25 g/cm2.


• In orthodontic treatment, in order to avoid injuries to the tissues, the forces applied generally should not exceed capillary
blood pressure.
• Tipping of anterior tooth requires a force of 20–50 g.
• Heavy forces on PDL causes Hyalinization.
• Both intrusion and translation are difficult to achieve.

REMOVABLE ORTHODONTIC APPLIANCE

Definition
• Appliances that are designed to be taken from the mouth by the patient
• Attached removable appliances maintain a fixed relationship to the dentition through clasps or other attachments

Classification
1. ATTACHED
A. Active
Headgears
Facemask
Chin cups
Lip Bumper
Active plate
Hawley’s appliance
Space regainers
Schwarz expansion plates
Anterior spring aligners (Barrer appliance)
Crozat appliance
Vacuum formed appliances (invisible appliances)
B. Passive
Space maintainers
Bite planes
Retainers
Occlusal splints
Posterior bite blocks
2. LOOSE REMOVABLE APPLIANCE/FUNCTIONAL APPLIANCE
Activator
Bionator
Frankel regulator
Twin block

Advantages of removable appliance


• Malocclusion which require tipping can be efficiently treated by removable appliance
• Small in size
• Simple in design
• Light in weight

https://t.me/DentalBooksWorld
22 Triumph’s Complete Review of Dentistry

• Easy to clean
• Low cost
• More esthetic
• No need for special equipment

Disadvantages/Limitations of removable appliance


• Not effective with uncooperative patient
• Treat the minor cases of malocclusion
• Allow the simple movements
• Difficult to correct ectopic teeth
• Appliances may get damaged or broken if they are not worn
• Not suitable for closure of extraction spaces by mesial movement of posterior teeth
• Rotation correction cannot be achieved

COMPONENTS OF REMOVABLE APPLIANCE


1. Active components
Springs
Labial bows
Screws
Elastics
2. Passive components
Clasps
3. Acrylic base plate
4. Anchorage
Headgears
Intermaxillary elastics

Important concept
• Force delivered for a given deflection depends on the wire length (L), radius (r), and elastic modulus (E)
• F directly proportional to Er4/L3
• Increasing the size or diameter by two times increases the stiffness by 16 times
• Increasing the length by two times reduces the stiffness by eight times

TOOTH MOVEMENTS WITH REMOVABLE APPLIANCES


Variety of tooth movements can be achieved with removable appliances, either individually or on groups of teeth
• Tipping
• Overbite reduction
• Crossbite correction
• Extrusion
• Intrusion

Removable appliances are also useful in maintaining tooth positions during retention

https://t.me/DentalBooksWorld
Chapter 1 • Orthodontics 23

Retentive components
The retentive components of a removable appliance are concerned primarily with seating it in the correct position, but they
can also contribute toward anchorage.
Adam’s clasp
• Adam’s clasps are constructed in a 0.7-mm stainless steel wire and most commonly used on the first molars, although they
can be used on premolars and anterior teeth
• The arrowheads of the clasp engage undercuts at the mesial and distal corners of the buccal tooth surface and can easily be
adjusted at the chair side to increase retention
• The bridge of an Adam’s clasp can also be used by the patient to remove the appliance from the mouth, while the
orthodontist can use it to attach auxiliary springs or tubes for headgear
• Components of Adam’s clasp

1. Arrow heads
2. Bridge
3. Tags
4. Retentive parts


Southend clasp
• The Southend clasp is also constructed in a 0.7-mm stainless steel wire, but is used for retention on the incisor teeth
• This clasp is activated by bending the U-loop toward the baseplate, which carries the clasp back into the labial undercut of
the tooth


Ball-ended clasp
Ball-ended clasps engage into interproximal undercuts between the teeth and are activated by bending the ball toward the
contact point

https://t.me/DentalBooksWorld
24 Triumph’s Complete Review of Dentistry

Plint clasp
• Plint clasps are useful when using a removable appliance in combination with a fixed appliance
• These clasps are constructed in a 0.7-mm stainless steel and engage the undercuts on a maxillary molar band


Labial bow
• A labial bow is constructed from a 0.7-mm stainless steel wire and can provide retention from the labial surface of the
incisor teeth, which can be increased by contouring the wire around these teeth in a fitted labial bow or by placing an
acrylic facing on the wire of the bow
• The labial bow is afforded flexibility by incorporating U-loops at each end, which allow activation by compression


Active components
The active components of a removable appliance are responsible for producing the desired tooth movement. They can be
categorized as springs, bows, screws, and auxiliary elastics.
Springs
Mechanical principles should be considered when applying a force to any tooth with a spring
• It should be delivered at right angles to the long axis of the tooth and through a surface parallel to it; otherwise, a vertical
force is introduced, which will tend to displace the appliance or intrude the tooth
• It should pass as close to the center of resistance as possible to reduce rotation
• The force (F) delivered by a spring is related to the length (L) and thickness or radius of the wire (R), as well as the
deflection (D), such that
F ∝ DR4/L3
Palatal finger springs
• Palatal finger springs are constructed in a 0.5- or 0.6-mm stainless steel wire and used to move teeth mesially or distally
along the dental arch
• The incorporation of a helix increases the length of the wire and allows the delivery of lighter forces whilst a guard wire
will protect the spring from distortion

https://t.me/DentalBooksWorld
Chapter 1 • Orthodontics 25

Buccal canine retractor


• Buccal canine retractors are constructed in a 0.7-mm stainless steel, reduced to 0.5-mm if sheathed
• These springs can be used to retract buccally placed maxillary canines; however, when activated it is mechanically difficult
to apply force directly to the mesial surface of the tooth


Z-spring
• The Z-spring is constructed in a 0.5-mm stainless steel wire and generally used to move one or two teeth labially
• Activation is achieved by pulling the spring away from the baseplate at an angle of approximately 45 degrees, which will
tend to displace the appliance away from the palate; good anterior retention is therefore important


T-spring
• T-springs are constructed in a 0.5-mm stainless steel wire and used to move individual teeth either labially or buccally
• Activation is again produced by pulling the spring away from the baseplate and therefore retention also needs to be good


Coffin spring
• A coffin spring provides a useful alternative to a screw for expansion
• This heavy spring is constructed in a 1.25-mm wire and activated by pulling the two halves of the appliance apart manually
or flattening the spring with a plier
• Coffin springs deliver high forces that will tend to displace the appliance and good retention is important

https://t.me/DentalBooksWorld
26 Triumph’s Complete Review of Dentistry

Active labial bows


• An active labial bow can be used to reduce an increased overjet by tipping the teeth palatally if the upper labial segment is
proclined and spaced
• However, a normal labial bow will only allow a small range of activation and this can be improved either by increasing the
amount of wire in the bow, as in a Mills bow, or by constructing it in a lighter wire, such as a Roberts retractor
• The Roberts retractor is constructed in a 0.5-mm stainless steel with buccal arms sheathed in stainless steel tubing
• Activation occurs by bending the vertical arms of the bow toward the palate and trimming the acrylic behind the upper
incisors to allow palatal movement


Roberts retractor
Screws
• Screws can be embedded into the baseplate of an appliance and activated by the patient progressively turning a key
• Screws can be effective for expansion to correct a posterior dental crossbite, or for distal movement of the buccal segments,
often supported by headgear
• Each quarter turn of the screw activates it by approximately 0.2 mm and, therefore, should be done by the patient once or
twice a week
• 1 mm expansion per one full turn
• 0.25 mm per quarter turn
• Used for moving one tooth or group of teeth
• For simultaneous expansion of maxillary incisors anteriorly and posteriors laterally → y plate
• Y plate can be modified for treatment of unilateral crossbite
• Maxillary split plate → does not split mid palatal suture
• Active plate → to correct maxillary anterior crossbite almost totally by tipping posterior teeth
• Active plate not indicated for skeletal crossbites, also not for dental expansion greater than 2 mm

Elastics
• Elastomeric forces can also be applied from a removable appliance and these can be useful in providing light force, which
can be reactivated regularly by the patient
• Intra-arch elastics can be used to retract the upper incisors as well as applying an intrusive force in patients with reduced
periodontal support
• Interarch application of elastics from removable appliances requires good retention to avoid displacement and is generally
avoided

https://t.me/DentalBooksWorld
Chapter 1 • Orthodontics 27

Hawley’s appliance
• Hawley’s appliance is the most basic type of removable appliance
• It is called dental crutch
• It is useful for closing of spaces in anterior region (Hawley’s retainer)
• It is used as a retainer appliance
• Used in deep bite correction
• Used as a habit breaking appliance

Difference between Hawley’s appliance and Hawley’s retainer


Hawley’s appliance Hawley’s retainer
Active appliance Passive appliance
Used to correct spacing and retraction of minor proclination Retention appliance
The acrylic plate behind the incisor is trimmed to facilitate The acrylic plate extends up to the palatal surfaces in form of
palatal movement of incisor collars extending interdentally also
Labial bow is activated Labial bow should not be activated

RECOMMENDED WIRE THICKNESS


Component Wire diameter
Adam’s clasp for molars 0.7 mm
Adam’s clasp for premolars 0.7 mm
Adam’s clasp for deciduous canines and molars 0.6 mm
Southend clasp 0.7 mm
C clasp 0.9 mm
Triangular clasp 0.7 mm
Jackson’s clasp 0.9 mm
Delta clasp 0.7 mm
Labial bow 0.7 mm
Reverse labial bow 0.7 mm
Roberts retractor 0.5 mm
Mills retractor 0.7 mm
U-loop canine retractor 0.7 mm (all canine retractors – 0.7 mm except palatal canine
retractor – 0.5 mm)
Finger spring 0.5 mm
Coffin spring 1.25 mm

SPECIFIC COMPONENTS AND THEIR APPLICATIONS


Components Fabrication Use
Labial movement Double 0.5–0.6 mm diameter SS Proclination of 1 or 2 incisors (good range)
cantilever spring (Z-spring) (Activate by pulling away from baseplate at 45°)

https://t.me/DentalBooksWorld
28 Triumph’s Complete Review of Dentistry

T-spring 0.5–0.6 mm diameter SS Proclination of an incisor/premolar/molar (poor


range) (Activate by pulling away from baseplate
at 45)
Recurved spring 0.8 mm diameter SS Proclination of all four incisors
Cross-over wires 0.7–03 mm diameter SS Proclination of all four incisors
Screws – Proclination of multiple incisors where retention
is poor
Palatal movement Roberts’ 0.5 mm diameter SS (sleeved) Retraction of prolined and spaced maxillary
retractor incisors
Buccal canine retractor 0.5 mm (sleeved) or 0.7 mm Palatal and distal movement of mesially
diameter SS (unsleeved) angulated canines
Elastics Different diameters available Force dependent on the root surface area of teeth
to be moved
Distal movement palatal finger 0.5–0.7 mm diameter SS (guarded) Distal movement of canines, premolars and
spring molars
Headgear – Distal movement of molars, J-hook headgear for
incisor retraction
Expansion screw – Correction of buccal crossbites
Coffin spring 1.25 mm diameter SS Buccal crossbite correction
Rotation whip spring 0.5 mm diameter SS Correction of mild rotations
Extrusion and intrusion elastics Different diameters available Force dependent on the root surface area of teeth
to be moved

FIXED APPLIANCES

DEFINITION
Fixed appliances are devices or equipment attached to teeth, which cannot be removed by the patient and are capable of
causing tooth movement.

COMPONENTS OF FIXED APPLIANCES


Active components Passive components
Separators Brackets
Arch Wires Bands
Elastics Accessories (Lock pins, Ligature wires, Modules)
Springs Molar tube

SEPARATORS
They are used to create space for banding teeth. They are of two types – Metal separators and elastic separators
Metal separators:
• 0.020 brass wire
• Kesling separating spring

https://t.me/DentalBooksWorld
Chapter 1 • Orthodontics 29

Elastic separators:
• Elastic thread
• Maxian elastic separator
• Elastomeric rings (or) dough nut

Arch wires:
Arch Wire exerts force to the teeth through the brackets and is used for achieving all types of tooth movements

Elastics:
Elastics are used to move the teeth, to fix arch wire to the teeth, for separation of teeth. It is available in the form of bands,
threads, modules, and rotational wedges

Springs:
Coil spring are of two types Open and close coil springs and are used to open or close spaces
Uprighting springs Used for root movement to correct mesial/distal tipping
Rotation springs Used to correct rotation

BANDS
Bands are thin strips of stainless steel, which are adapted to the contours of the tooth to which attachments are welded or
soldered
BAND MATERIAL – TYPES
Teeth Size
Molar band 0.005 × 0.20
0.005 × 0.18
Premolars 0.004 × 0.150
Incisors 0.003 × 0.125

ATTACHMENT TO BANDS
Buccal tubes – It holds the arch wires and inner bow of the face bow attachment
Lingual sheaths – Receive and attach lingual arch wires
Molar hooks, lingual buttons, and cleats – These are used for engaging elastic bands and modules
Orthodontic brackets – The force required for orthodontic tooth movement is transmitted from the active components
through the brackets

https://t.me/DentalBooksWorld
30 Triumph’s Complete Review of Dentistry

Accessories:
• Lock pins – they are used to connect or engage the arch wire and they are made from brass
• Ligature wires – these are used to connect or engage the arch wire
• Modules – are used to fix the arch wire to the bracket slot. These are elastomeric rings which are used in preadjusted
edgewise technique

EVOLUTION OF FIXED APPLIANCE


E (Expansion) Arch – Angle’s first appliance

Pin and tube appliance (Angle)

Edge wise appliance by Angle

Preadjusted edgewise appliance by Lawrence Andrews

Ribbon arch appliance by Angle

Modified Ribbon arch appliance by Raymond Begg

Tip edge appliances by Peter Kesling

EDGEWISE APPLIANCE – ANGLE (1928)


• In this technique, arch wire is inserted through the narrowest portion into the slot. Hence it is called Edgewise technique.
• This technique allowed excellent control of crown and root in all the three planes
• The dimension of the slot is 0.022 × 0.028 inches
• The three bends used to accomplish tooth movement are:
1. First-order bends – in and out or labiolingual corrections
2. Second-order bends – mesiodistal corrections
3. Third-order bends – buccolingual corrections

STRAIGHT WIRE APPLIANCE OR PREADJUSTED EDGEWISE APPLIANCE


• Andrews introduced the straight wire concept in 1970s
• It eliminated the difficult wire bending procedures by modifying the brackets
• In this technique, different brackets are used for the teeth
• The type of tooth movement achieved is bodily movement
• Hence anchorage preparation is vital in preadjusted appliance technique
• The angulations and torque values built into the preadjusted bracket are called an appliance prescription
• The dimensions of slot is 0.022 × 0.028” or 0.018 × 0.025”

https://t.me/DentalBooksWorld
Chapter 1 • Orthodontics 31

BEGG TECHNIQUE OR MODIFIED RIBBON ARCH TECHNIQUE


• Raymond P. Begg introduced this technique in 1950s
• He introduced the concept of differential force technique where the bodily movements of anchor molars were pitted
against the tipping movement of anterior teeth. Hence, light forces are used in this technique. The type of tooth movement
achieved in this technique is tipping.
• Anchorage preparation is not very critical
Stages in Begg Treatment
There are three stages:
Stage 1 – Corrections achieved during stage 1 are:
Overjet reduction
Overbite correction
Correction of crowding
Correction of rotation
Correction of spacing
Crossbite correction
Stage 2 – Involves root following treatment objectives:
Maintaining correction achieved in stage 1
Space closure
Molar correction
Stage 3 – Involves movement by auxiliary arch wires:
Root uprighting
Root torquing

TIP EDGE APPLIANCE


• Peter Kesling introduced tip edge in 1980s
• This is a preadjusted type of Begg appliance
• Early stages of treatment resemble Begg technique
• Treatment is finished in readjusted technique method

SELF-LIGATING BRACKETS
A self-ligating bracket is defined as “a bracket, which utilizes a permanently installed, movable component to entrap the arch
wire”
Types
They are divided into two types – active and passive
Active brackets – use a flexible component to entrap the arch wire. This flexible component constrains the arch wire in the
slot. Hence there is precise and controlled movement with active self-ligating brackets
Passive brackets – use a rigid, movable component to entrap the arch wire. Hence, tooth control is determined entirely by the
fit between bracket slot and arch wire

https://t.me/DentalBooksWorld
32 Triumph’s Complete Review of Dentistry

LINGUAL BRACKETS
• Lingual brackets involve the placement of brackets and other attachments on the lingual surface of the teeth
• The appliance is not visible and hence does not affect the aesthetics of the patient. This technique is called invisible
orthodontics. Craven Kurz is credited with the development of lingual appliances
• First suggestion of lingual appliance was given by Pierre Fauchard in 1726
• Kinya Fujita submitted the concept of Lingual orthodontics in 1967

USE OF ELASTICS IN DIFFERENT CLINICAL SITUATION

Name Application Picture


Class I intramaxillary Space closure

Class II Used in class II malocclusion for retraction of


anterior teeth in the upper arch and correction of
molar relation by mesial movement of lower molar

Class III Used in class III malocclusion for retraction of


anterior teeth in lower arch and correction of
molar relation by mesial movement of upper molar

Crossbite elastics To correct crossbite when both the teeth are out of
position

Zig-zag To establish proper interdigitation

https://t.me/DentalBooksWorld
Chapter 1 • Orthodontics 33

Triangular Open bite correction (lateral)

Box elastics Open bite correction

Settling elastics Are used at the end of the treatment for final
posterior settling

Asymmetric elastics They are used to correct dental asymmetries


usually with class II on one side and class III on
other side. Also helps in midline correction
Cross palatal Cross palate elastics may be used to correct
undesired expansion of the upper molars during
the third stage of Begg technique

Lingual elastics Used in retraction and correction of rotated tooth

Diagonal elastics To correct midline discrepancy

https://t.me/DentalBooksWorld
34 Triumph’s Complete Review of Dentistry

Check elastics For bite opening and retraction of upper anterior


teeth

Extra oral elastics 8,12,14,16 Oz elastics are available and used along
with face mask for orthopedic effects

EVOLUTION OF BONDING
For the orthodontic treatment to carry out, force is to be applied to the teeth; to apply force we need some form of attachment
over the teeth, so this can be done in two ways
1. Banding
2. Bonding
BANDS – These bands were introduced by W. E. Magill in 1871 and have been in existence for more than 100 years
Disadvantages of banding
• Laborious, time consuming
• Skilled work is required
• Difficulty in banding partially erupted teeth
• Decalcification/discoloration with loose or uncontoured bands
• Gingival irritation
• Unaesthetic
• Need of separators
• Closure of band spaces after completion of treatment

HISTORY
Three major developments that made bonding of attachments to teeth possible
1. Buonocore (1955) – Improved retention of methyl methacrylate to enamel
–– 85% phosphoric acid for 30 seconds
2. Bowen (1962) – Bis glycidyl methacrylate – more stable and greater strength
3. Newman (1965) – First to acid etch and bond orthodontic brackets with epoxy resin
Fujio Miura et al. in 1971 – Described an acrylic resin Orthomite using a modified trialkyl borane catalyst, that proved to be
particularly successful for bonding plastic brackets and for enhanced adhesion in presence of moisture
In 1975 – Silverstone – Three patterns of enamel etching
In 1979 – Major R and Smith DC – Introduced an alternative to acid etching. The crystal growth on enamel surface

BONDING
Process of joining two materials by means of an adhesive agent that solidifies during bonding process
Types
1.  Physical bonding
Involves van der Waal/electrostatic interactions that are relatively weak. It is the type of bonding seen when surfaces smooth
and chemically dissimilar.

https://t.me/DentalBooksWorld
Chapter 1 • Orthodontics 35

2.  Chemical bonding


Involves bonds between atoms, are formed across the interface from adhesive and adherent. Since materials are dissimilar, the
extent to which bonding is possible is limited, overall contribution to bond strength low.
3.  Mechanical bonding
Result of an interface that involves undercuts and other irregularities that produce interlocking of the material.
Almost every case of dental adhesion is based primarily on mechanical bonding.

Advantages of Bonding
• Esthetically superior
• Faster and simpler
• There is less discomfort for patient
• Arch length not increased by band material
• Allows more precise bracket placement
• Improved gingival condition is possible and there is better access for cleaning
• Partially erupted or fractured teeth can be controlled
• Mesiodistal enamel reduction is possible during treatment
• Interproximal areas are accessible for composite buildup
• Caries under loose bands is eliminated. Interproximal caries can be detected and treated
• No band spaces to close at end of treatment
• No large supply of bands needed
• Brackets may be recycled further reducing the cost
• Lingual brackets – Invisible braces may be used when esthetics important
• Improved appearance, decreased discomfort for patient, and ease of application for clinician

Disadvantages of Bonding
• A bonded bracket has weaker attachment than a cemented band
• Few bracket adhesives are not strong
• Better access for cleaning does not necessarily guarantee better oral hygiene and improved gingival condition, especially if
excess adhesive extends beyond bracket base
• Protection against interproximal caries of well-contoured cemented band is absent
• Bonding is not indicated on teeth where lingual auxiliaries are required or where headgear are attached
• Rebonding a loose bracket requires more preparation than rebanding a loose band
• Debonding is more consuming than debanding since removal of adhesive is more time consuming

PATTERNS OF ETCHING
By Gwinnett and Silverstone
Type I – Core etching
Honeycomb pattern – Initially periphery of prism head is delineated by microclefts (0.1–0.2 mm) continued action of acid
leads to loss of substance predominantly in area of prism cores with simultaneous conservation of marginal areas. Least
amount of enamel is lost in this etch pattern.
Type II – Periphery etching
Peripheral etching pattern is an advanced stage in which fragile prism peripheries break off. Max enamel loss takes place in
this stage.
Type III – Mixed pattern
As action of acid proceeds, there is dissolution of crest like marginal ridges, while marginal clefts continue to widen. This
transitional zone of central and peripheral etching pattern in which existing marginal ridges are elevated to 3 µm.

https://t.me/DentalBooksWorld
36 Triumph’s Complete Review of Dentistry

Galil and Wright described Types IV and V


Type IV
–– Etch pattern commonly seen in cervical areas. It shows irregular pattern and displays no rod or prism pattern.

Type V
–– Shows no prism outline. Enamel surface is extremely flat and smooth and they lack micro-irregularities for resin
penetration

EXPANSION APPLIANCES
Expansion appliances can be broadly classified into maxillary and mandibular expansion appliances.

MAXILLARY EXPANSION APPLIANCES


Slow maxillary expansion appliances
Removable Fixed
– Active plate with screws – W arch
– Coffin spring – Quad helix
– Active plate with Z-spring – Expansion screws
Rapid maxillary expansion appliances
Banded RME Bonded RME
– Haas – Acrylic
– Isaacson Minne expander – Cast metal
– Hyrax
– Derichsweiler
– Biederman
Mandibular expansion appliances
Surgically assisted rapid palatal expansion

TYPES OF EXPANSION
1. ORTHODONTIC EXPANSION
• This kind of expansion is brought about by conventional fixed appliances and various removable appliances.
• In orthodontic expansion, there is lateral movement of the buccal segments which results in mainly dentoalveolar
expansion.
• There is buccal tipping of the crowns and lingual tipping of the roots

2. PASSIVE EXPANSION
• Results from the intrinsic forces exerted by the tongue.
• With the use of buccal shields, the forces from the labial and buccal musculature are prevented from acting on the
dentition.
• Passive expansion is achieved by the vestibular or lip shields.

https://t.me/DentalBooksWorld
Chapter 1 • Orthodontics 37

3. ORTHOPEDIC EXPANSION
• Here the changes are mainly produced in the skeletal structures.
• Rapid Maxillary Expansion (RME) appliances are the classic examples.
• RME causes separation of midpalatal suture. It also affects the circumzygomatic and circumaxillary sutures.

RAPID MAXILLARY EXPANSION

HISTORY OF EXPANSION APPLIANCES


• Expansion appliances have been used from olden days, e.g., Bandelette by Fauchard, E arch by Angle. (These were Sloe
expansion appliances.)
• Emerson C Angell (1860) is the pioneer of rapid maxillary expansions.
• Coffin spring (slow expansion) was introduced by Walter coffin.

BIOMECHANICS OF RME

Application of force to widen the maxilla

Causes opening of the midpalatal suture

New bone formation is induced

Space created in the midline is filled with tissue fluids and blood

After 3–4 months, new bone fills the space.

INDICATIONS OF RME
• Used in unilateral or bilateral posterior skeletal crossbite
• Narrow or constricted maxilla in case of class II cases can be treated using RME
• Used in class III cases also
• Treatment along with reverse pulls headgear to loosen the sutures
• Used in anterior crossbite to gain space
• Bonded RME can be used in high angle cases

https://t.me/DentalBooksWorld
38 Triumph’s Complete Review of Dentistry

RME – APPLIANCE MANAGEMENT


BEFORE 15 YEARS
Activated twice a day 90-degree activation each time
Total 180-degree activation every day 0.5 mm per day
Review: After 1 week
AT 15–20 YEARS
Activated 4 times a day 45-degree activation
Total 180-degree activation 0.5 mm/day
Review after 1 week
ABOVE 20 YEARS OF AGE
Activated 2 times a day First activation 90 degrees
Then 45-degree activation Total 90 degrees
Review after 3–4 days

CLINICAL IMPLICATIONS OF EXPANSION


• Midpalatal suture opens in V fashion
• The broad end of V is faced toward the anterior region and apex of V is placed toward posterior region
• There will be an appearance of midline diastema
• Occlusal and frontal radiographs show the suture opening
• Treatment duration is usually 2 weeks
• As relapse is higher after RME, overcorrection is advised
• Force exerted during RME is in the range of 10–20 pounds

TISSUE CHANGES OBSERVED WITH RME


BONE CHANGES
• Maxilla moves laterally due to expansion
• Maxilla also rotates with its fulcrum at frontonasal suture
• Increase in nasal airway, reduction in airway resistance
• Downward and backward rotation of mandible
• Increase in mandibular angle

SUTURAL CHANGES
• After initial hyperemia, sutural space is invaded by osteoblasts
• New bone is deposited at the edges of palatal process
• Gradual bone filling in the space
DENTAL CHANGES
• Initially teeth move labially by translation
• There is increased buccal inclination of the posterior teeth
• Slight extrusion of posterior teeth
• Appearance of midline diastema
• Later, midline diastema closes due to the pull of transseptal fibers

https://t.me/DentalBooksWorld
Chapter 1 • Orthodontics 39

TISSUE REACTION
• At the end of active expansion procedures, 80% skeletal and 20% dental expansion occur
• After 4 months, 50% skeletal and 50% dental changes are noted
• Relapse is highest during the first 6 weeks after expansion
RETENTION SCHEDULE
• The same fixed RME appliance is used as retainer for first 3 months
• Removable retainers are given from the 4th month
• Removable retainers are worn full time for about 9 months after expansion

SLOW EXPANSION APPLIANCES


• Slow expansion appliances are designed primarily to produce dentoalveolar changes
INDICATIONS
• To relieve crowding (<4 mm) in minimal space discrepancy cases
• Posterior dental crossbite in one or two teeth
• Cleft palate cases with collapsed maxilla
• Constricted maxillary arch
CLINICAL MANAGEMENT OF SLOW EXPANSION
• The expansion screw delivers controlled and equal force to both sides
• RME screws can be used to produce slow expansion
• Tooth movement is about 1 mm/month
• Activation is done once or twice a week
• For each activation quarter turn is given
• Quarter turn will produce 0.25 mm activation
• This corresponds to the average width of the periodontal ligament
COFFIN SPRINGS
• Introduced by Walter Coffin
• Wire used: 1.25 mm heavy stainless steel wire
• Indicated for expansion of constricted maxillary arch
• Used for correction of crossbite
• For conditions requiring differential expansion
QUAD HELIX
• Introduced by Ricketts
• The appliance is constructed using 0.038 inch Elgiloy or SS wire
• The appliance brings about both orthodontic and orthopedic changes in the posterior segment
• Quad helix can be activated in 4 positions
• In the anterior bridge: Results in expansion in molar region
• In the palatal bridge: Derotation and expansion of molar on the same side and distalization of molar on the opposite side
• Outer arms are activated to expand canines and premolars
• Opening of posterior helix expands the buccal arm
• An initial expansion of 8 mm will produce 14 ounces of force
• Average force is 200–400 g
• Quad helix can also be used as a space maintainer and reinforce anchorage

https://t.me/DentalBooksWorld
40 Triumph’s Complete Review of Dentistry

S. No. Factors Rapid expansion Slow expansion


1. Nature of expansion Skeletal and Dental Dentoalveolar changes
2. Final expansion 50 skeletal: 50 dental Only dental
3. Force level 10–20 pounds 2–4 pounds
4. Activation 0.5–1 mm per day 1 mm/month
5. Effect on mid palatal suture Opening of mid palatal suture Rare
6. Occurrence of midline diastema Yes No
7. Duration of treatment 2–3 weeks 2–3 months
8. Indication Skeletal crossbite cases in class II and class III Minor space discrepancy
9. Retention 9 months 3 months

ORTHOPEDIC APPLIANCES

HISTORY OF EXTRAORAL ORTHOPEDIC APPLIANCES


• Earliest use of headgear for occipital anchorage: Gunnel (1822)
• Use of headgear for retracting incisor teeth: Kingsley (1892)
• Use of headgear for extraoral anchorage: Calvin case (1900)
• Use of headgear for both orthodontic and orthopedic practice: Angle
• Revived the use of headgear in orthodontic practice after death of Angle: Oppenheim.
• Introduced and popularized cervical headgear: Kloehn (1940)

Orthopedics may be defined as any manipulation that alters the skeletal systems and associated motor organs (Ricketts)

HEADGEARS
• Headgear is an extra oral orthopedic appliance used to restrain the downward and forward growth of maxilla
COMPONENTS OF HEADGEAR
FACE BOW
• The force of headgear is transmitted to jaws through the handle
• Face bow is the force delivery unit
• It acts as a splint to hold the teeth and jaw together
• It is made up of stainless steel wire and consists of two parts:

OUTER BOW
• Outer bow is made of heavy stainless steel wire of gauge 0.059 inch or 1.5 mm.
• The outer bow is about 5–10 mm away from the cheek tissue to avoid discomfort to the patients.

INNER BOW
• The measurement of inner bow is about, 0.045 or 0.050 inch (1.12–1.25 mm)
• The inner bow is in contact with the maxillary first molar buccal tubes
• The bow should be of 3–4 mm away from all the teeth and the bow should not be in contact with the teeth at any point

Position of the bows: The position of inner and outer bow when engaged should be between the two lips and should be
passive. It should not lift the lips.
ANCHORAGE SOURCE OR HEADGEAR STRAP
• Headgear or head cap is wrapped around the occipital region, neck, or junction of parietal or occipital region.
• They form the base for deriving anchorage from bones.

https://t.me/DentalBooksWorld
Chapter 1 • Orthodontics 41

BIOMECHANICS OF HEADGEAR
• The effect of the headgear depends on the direction of application of force.

Force applied

Maxillary molar Whole maxilla

1. Through C-Res 1. Through C-Res


• Causes translation - Translation

2. Below C-Res 2. Below C-Res


• Distal crown tipping - Clockwise rotation
3. Above C-Res 3. Above C-Res
• Distal root tipping - Counter clockwise

CENTRE OF RESISTANCE (C-Res):


• The center of resistance of the maxillary first molars is in the mid root region between the roots slightly apical to the
furcation.
• Centre of resistance of maxilla: The center of resistance of maxilla is between and above the apices of the upper premolar
teeth and between the lower margin of orbitale and the distal apex of the first molar vertically in the sagittal plane.

POINTS ON FORCE PARAMETERS OF HEADGEARS


• Minimum value of force to impede the forward movement of maxilla appears to be 250 g/side.
• The type of force produced by headgear is heavy intermittent.
• Duration of wear is usually 12–16 hours per day.
• Duration of orthopedic treatment with headgear is usually 12–18 months.
USES OF HEADGEAR
• Orthopedic extra oral appliance in preadolescent patients with skeletal class II base due to prognathic maxilla. Force value
is 350–450 g/side.
• Reinforcement of anchorage during fixed appliance treatment (250–350 g/side).
• Distalization of maxillary first molar.
• Retention of molar correction.
• Space maintenance and regaining.
• Overjet reduction: Asher’s face bow is used to retract upper and lower anteriors simultaneously.
• Intrusion of molars and incisors using high pull headgear.
• Expansion or contraction of the arch by adjusting the inner bow.
• Uprighting of mesially tipped molars.

DISADVANTAGES OF CERVICAL HEADGEAR


• Extrusion of maxillary molars and increase in facial height.

REVERSE PULL HEADGEARS OR FACE MASKS


• Face mask is an extraoral traction appliance used to correct skeletal class III malocclusion.
• Face mask is a method by which anterior directed forces to the teeth or skeletal structures are delivered from an extraoral
source.

https://t.me/DentalBooksWorld
42 Triumph’s Complete Review of Dentistry

FACE MASK
• Hickam was the first to use the reverse pull headgear to correct the class III malocclusion.
• Jean Delaire (1972) improvised the design of facemask.
• Henri Petit modified Delaire’s facemask. The face mask used currently is a modification of the Petit’s facemask.
• McNamara advocates the use of a banded rapid maxillary expansion or bonded RME with hooks at premolar region for
engagement of elastics.
INDICATIONS OF FACEMASK
• It is primarily used to correct class III skeletal malocclusion due to maxillary retrognathism in young children.
• In pseudo-class III malocclusion, facemask can correct centric relation – centric occlusion discrepancy.
• After surgical correction of the skeletal class III malocclusion, facemask is used as a retaining device.

COMPONENTS OF A FACEMASK
• Chin cup
• Forehead cap/strap
• Framework
• Intraoral splint/device
• Elastics

BIOMECHANICS
• The elastic portion pulls the maxilla forward.
• The purpose of using RME in skeletal class III is that it helps in relieving the posterior crossbite often associated with class
III malocclusion. It also helps to facilitate protraction.
EFFECTS OF FACEMASK
• Forward movement of maxilla
• Forward movement of maxillary teeth
• Proclination of maxillary incisors
• Correction of anterior and posterior crossbite
• Downward and Backward rotation of the mandible

FORCE PARAMETER VALUES FOR THE USE OF FACEMASK


• Initial force of 300 g/side, 2 weeks later 450–500 g/side
• Force applied 20 degrees downward to the occlusal plane produces translation of maxilla forward.
• Elastic parallel to the occlusal plane produces an upward rotation along with forward movement of maxilla.
• Time of wear is 20 hours/day but after 6 months it is reduced to night time wear for retention.
• Active treatment takes about 4–6 months.
• Early mixed dentition is the ideal time to start the treatment.
• Verdon (1989) recommends forward posturing of mandible before the age of 8 years.
• Treatment completion is indicated by:
–– Attaining positive overjet
–– Improvement from the concave profile
• Facemask is the only extraoral traction device for correction of maxillary deficiency with rapid improvement.

CHIN CUP
• Chin cup is an extraoral device designed to exert an upward and backward force on the mandible by applying pressure to
the chin, thereby preventing forward growth.
Types and Indications of Chin Cup:
• Occipital pull chin cup: Skeletal class III with mild to moderate mandibular prognathism.
• Vertical pull chin cup: To correct anterior open bite cases.
• Chin cup can be used in patients with increased anterior facial height.

https://t.me/DentalBooksWorld
Chapter 1 • Orthodontics 43

Biomechanics:
• At the start of the treatment 150–300 g/side of force is applied.
• Two months later, it is increased to 450–700 g/side.
• Less amount of force is enough, if the line of force is below the condyle.
• Appliance is worn for the duration of 14 hours a day.
• Age of treatment is less than 8 years in the mixed dentition.
• Treatment completion is indicated by the correction of anterior crossbite.

Side Effects:
• Lingual tipping of lower incisors and crowding
• TMJ symptoms may develop, then treatment should be continued immediately.

QUICK FACTS
• Primary spacing: Normal development space present in deciduous dentition
• Secondary spacing: Spacing occurring in closed dentition wherein erupting lower incisors push the primary canine laterally
and creates space
• Tertiary spacing: Spacing due to extraction, proximal stripping
• Sequence of tooth formation:
–– Initiation
–– Proliferation
–– Histodifferentiation
–– Morphodifferentiation
–– Apposition
• Jaw radiograph of a newborn will show 24 teeth
• Early mesial shift occurs at the age of 6–7 years
• Late mesial shift occurs at the age of 10–11 years
• Mixed dentition period is otherwise known as Space age
• Primary teeth in boys are usually larger than girls
• Ankylosis is more common in primary dentition
• In primary dentition, lower primary teeth are more prone to ankylosis
• Arch length decreases in permanent dentition (2–3 mm)
• Arch length is usually greatest before the eruption of permanent first molar
• Total eruption path of the first permanent molar is about 2.5 cm
• Each permanent tooth occludes with two teeth except for maxillary third molars and mandibular central incisors
• Bolton’s tooth ratio is considered to be the seventh key of occlusion
• Step child of dentition – Third molars
• Cornerstone of dentition – First molars
• Corner tooth of dentition – Canines
• Servant of the tooth – Alveolar bone
• Slave of the orthodontist – Alveolar bone
• Bread of the orthodontist – Periodontal ligament
• Maxillary and mandibular laterals are generally displaced – lingual direction
• Maxillary canines are generally displaced – labial direction

CEPHALOMETRICS
• X-rays were discovered by Roentgen in 1895.
• Broadbent–Bolton cephalometer was devised in 1931.
• A magnification which occurs in the cephalostat will be in the range of 5–7% = considered normal.

https://t.me/DentalBooksWorld
44 Triumph’s Complete Review of Dentistry

• In lateral cephalometrics the distance at which the film is placed from the midsagittal plane = 18 cm or 7 inches.
• In a cephalostat, the distance between the X-ray source and the midsagittal plane of the patient is fixed at 60 inches.
• The distance between the X-ray tube and the midsagittal plane of patient’s head is 5 feet/60 inches/152.4 cm.
• Occipital condyle is represented by the following cephalometric point – Bolton’s Point.
• Gonion, menton, and pogonion are cephalometric landmarks located on the mandible.
• Unilateral landmark/landmarks present in midline: Nasion, menton, gnathion, basion, pogonion
• Gnathion – derived cephalometric landmark.
• Registration point – a point half way on the perpendicular from sella to Bolton’s plane.
• Cranial base length is measured from the nasion to Bolton – Bolton plane (a plane that connects nasion and Bolton).
• De coster’s line – the outline of the internal surface of the anterior cranial base.
• Nasion is situated at the frontonasal suture. Most anterior point of frontonasal suture in the median plane.
• Deepest point between the ANS and the superior prosthion is Subspine.
• Most anterior point of chin is pogonion.
• Most inferior point of chin is menton.
• Most anti-inferior point of chin is gnathion.

TOOTH MOVEMENT
• Center of resistance: Point around which the mass of the body is concentrated. The body translates when force applied
passes through this point. The location of center of resistance is different for single root tooth and molars.
• Tip about a point one-third the length of root from the apex.
• A tooth will translate when its center of rotation is of infinity.
• Tooth movement easily accomplished by removable appliances – tipping.
• Centre of rotation shifts to bracket slot during – torque.
• Centre of rotation for controlled tipping movement is at the apex.
• Torque in orthodontic refers to change in labiolingual inclination of the teeth.
• Intrusion requires very gentle and mild forces (utmost control is required).
• Easiest movement during orthodontic treatment is extrusion.
• Frontal resorption is caused due to Light continuous force.
• Gingival fibers take 236 days for reorganization after rotation.
• Heavy forces on the periodontal ligament causes hyalinization.
• High orthodontic forces cause: Resorption of cementum, Resorption of dentin, Nonvital teeth.
• First tissue to react when orthodontic forces are applied is PDL.
• Most of biochemical change occur in PDL.
• Bed of orthodontics – PDL.
• Enzyme which decreases the orthodontic movement – alkaline phosphatase.
• Lag phase of tooth movement is due to time taken for the removal of hyalinized tissue.
• Progress of tooth movements after wearing an appliance may be described as slow initially – delay – rapid finally.
• Tipping of tooth results in the fibers of the PDL to be half compressed, half stretched.
• Osteoclast appear within 48 hours of pressure application.
• According to Wolf ’s law, bone trabeculae line up in response to mechanical stresses.
• Resorption of cementum as compared to bone when orthodontic forces applied is less intense.
• Root loss inevitably follows orthodontic movement.
• In orthodontic movement NSAIDs are contraindicated, because it slows down the tooth movement.
• Prostaglandins which are involved in tooth movement are inhibited by NSAIDs.
• Tooth movements in pulpless teeth compared to vital teeth is SAME.
• Average shortening of tooth length of premolar in cases after orthodontic treatment is 1.5 mm.

https://t.me/DentalBooksWorld
Chapter 1 • Orthodontics 45

EXPANSION APPLIANCES
• The type of expansion achieved through Frankel appliance is passive expansion.
• Pain felt during RME treatment is due to buildup of forces.
• Appearance of midline diastema is a classic sign during RME.
• Relapse after rapid maxillary treatment is highest during the first 6 weeks after expansion.
• Bonded RME is used in the high angle case.
• Quad helix, introduced by Ricketts, produce skeletal expansion in young children.

ORTHOPEDIC APPLIANCE
• Kloehn face bow is used in class II malocclusion.
• The maximum extent of forward displacement of the maxilla in maxillary deficiency with extraoral traction is 3 mm.
• Rule of thumb in headgear therapy: If more than half of extraction space is required to align teeth and achieve class I canine
relationship, then extraoral anchorage will be required.
• Force applied for chin cup is about 450–700 g per side.
• Delaire facemask is used for treatment of maxillary deficiency.

MULTIPLE CHOICE QUESTIONS

HISTORY OF ORTHODONTICS
1. Who was the first to classify malocclusion?
A. Pierre Fauchard B. John Hunter
C. Joseph Fox D. Gunnell JS
2. Who was the first to describe the growth of jaws, on a scientific investigation?
A. Angell B. Kingsley
C. William Magill D. John Hunter
3. Who is regarded as, “Father of Orthodontics”?
A. Anderson B. Edward Angle
C. Kinsley D. Proffit
4. Who is known as, “Father of Modern Orthodontics”?
A. Calvin Case B. Edward Angel
C. Pierre Fauchard D. Kingsley
5. Who is called, “Father of Modern Dentistry”?
A. Pierre Fauchard B. Calvin Case
C. Le Foulon D. Emerson Angell
6. The first person to advocate the use of Finger pressure to align the irregular teeth is
A. Celsus B. Hippocrates
C. Calvin Case D. Ricketts
7. Which one of the following is not the contribution of E. H. Angel?
A. Edgewise appliance B. Pin and tube appliance
C. Ribbon arch appliance D. Straight wire appliance
8. Pre-adjusted Edgewise (Straight wire) appliance was introduced by
A. Pierre Fauchard B. Lawrence F Andrew
C. Edward Angel D. Craven Kurz
9. A modification of Ribbon Arch Technique was given by
A. Frankel B. Anderson
C. Begg D. Fujita

https://t.me/DentalBooksWorld
46 Triumph’s Complete Review of Dentistry

10. Indian Board of Orthodontics was formed in the year of


A. 1996 B. 1997
C. 1998 D. 1999
11. Who is considered as the First Qualified Orthodontist in India?
A. Dr. HD Merchant B. Dr. HS Shaikh
C. Dr. Naishadh D. Dr. AB Modi
12. Which of the following is not a contribution of Ricketts?
A. Utility arch B. Ricketts analysis
C. Computer tracing D. Tip edge appliance
13. Tip Edge Appliance was introduced by
A. Fujita B. Kesling
C. Ricketts D. Guilford
14. First person to cement the Bands is
A. Farrar B. Wilkinson
C. Magill D. Mistri
15. Sutural Opening of Palate was introduced by
A. Angell B. PR Begg
C. Edward Angel D. Kurz
16. First book on Malocclusion was written by
A. P Fauchard B. Calvin Case
C. Kneisel D. Kingsley
17. Who was the first to suggest the term Dentofacial Orthopedics?
A. Nutting Farrar B. Norman Bennett
C. Norman Kingsley D. BF Dewel
18. Who popularized Premolar extraction treatment?
A. John Hunter B. Peter Kesling
C. Charles Tweed D. Norman Kingsley
19. Dr. E. H. Angle was born in the year
A. 1855 B. 1856
C. 1875 D. 1853
20. Indian Orthodontic Society was established in the year
A. 1955 B. 1965
C. 1975 D. 1980
21. Which branch of Orthodontics is also known as Secondary Prevention in Orthodontics?
A. Surgical orthodontics B. Preventive orthodontics
C. Interceptive orthodontics D. Corrective orthodontics
22. Who developed Orthometer?
A. Korkhaus B. Peck and Peck
C. HD Keslings D. Moyer
23. Gnathostatic model was developed by
A. Martin Dewey B. Bolton
C. Simon D. Moyer
24. Intermaxillary elastics was introduced by Baker in the year
A. 1938 B. 1983
C. 1899 D. 1893
25. The first orthodontic appliance developed by Angle was
A. E arch B. Bandelette
C. Pin and tube D. Monoblock

https://t.me/DentalBooksWorld
Chapter 1 • Orthodontics 47

26. The concept of extraction was introduced in orthodontics by


A. Dewel B. Tweed
C. Calvin Case D. Nance
27. The term Anchorage was coined by
A. Guilford B. Alexis Schange
C. William Magill D. Baker
28. Who was the first person to introduce Occipital Anchorage?
A. Guilford B. Farrar
C. Gunnell D. Kurz
29. Who was the first to classify malocclusion using Premolar relationship?
A. Kurz B. Katz
C. Kingsley D. Kesling
30. Smile Index was developed by
A. Tanaka–Johnson B. Peck and Peck
C. Hixon–Oldfather D. Ackerman and Ackerman
31. RME was reintroduced in Orthodontics by
A. Korkhaus and Andrew B. Korkhaus and Farrar
C. Magill and Angell D. Farrar and Andrew

GROWTH AND DEVELOPMENT


1. Bone and cartilage grows respectively by
A. Interstitial and apposition B. Apposition and interstitial
C. Both by apposition D. Both by interstitial growth
2. The movement of bone as a whole unit is called
A. Drift B. Remodeling
C. Displacement D. Relocation
3. The sequence in which growth of jaws occur is by
A. Width–Length–Height B. Height–Width–Length
C. Length–Height–Width D. Width–Height–Length
4. The three main vertical pillars of trajectories of force arising from the alveolar process and ending in the base of skull
are all except
A. Canine pillar B. Condylar pillar
C. Pterygoid pillar D. Zygomatic pillar
5. According to functional matrix theory, the determinant growth of skeletal tissues resides in
A. Skeletal tissues B. Sutures
C. Nonskeletal tissues D. Cartilages
6. How many growth spurts are present in females?
A. One B. Two
C. Three D. Four
7. Which of the following parts of the cranial base shows purely endochondral ossification?
A. Occipital B. Sphenoid
C. Temporal D. Ethmoid
8. One of the following implants was used by Bjork for studying growth. Pick it up
A. Tantalum metal B. Hydroxyapatite
C. Stainless steel D. Carbon
9. The major growth sites in mandible are: 1. Posterior ramus, 2. Anterior ramus, 3. Condylar process, 4. Anterior part
of chin
A. 1, 2, 3 B. 1, 3
C. 2, 3, 4 D. 1, 4

https://t.me/DentalBooksWorld
48 Triumph’s Complete Review of Dentistry

10. Which of the following serves as an adaptation of immunological mechanisms to protect children against infection?
A. Lymphoid tissue B. Neural tissue
C. Genital tissue D. Visceral tissue
11. Cartilaginous theory was put forward by
A. Moss B. Sicher
C. Scott D. Limborgh
12. Petrovic and Chartier introduced a theory of growth called
A. Functional matrix theory B. Neurotropism theory
C. Multifactorial theory D. Servo-system theory
13. The original concept of Functional matrix theory was given by
A. Van Limborgh B. Van der Klaus
C. Brodie D. Behrents
14. Which of the following is the counterpart of Corpus of mandible?
A. Maxillary arch B. Bony maxilla
C. Middle cranial fossa D. Max. tuberosity
15. Which is regarded as the pacemaker of mandibular growth?
A. Condylar growth B. Coronoid growth
C. Posterior ramal growth D. Anterior ramal growth
16. Intervening disc of TMJ develops at
A. 10th week B. 12th week
C. 14th week D. 16th week
17. Intramatrix rotation refers to
A. Rotation of mandibular core relative to cranial base
B. Rotation of mandibular plane relative to cranial base
C. Rotation of mandibular plane relative to core of the mandible
D. Both A and B
18. Which of the following system shows the first evident of puberty?
A. Muscle B. Immune system
C. Brain D. Reproductive system
19. Number of ossification centers in Maxilla
A. 2 B. 3–4
C. 4–5 D. 6–7
20. Prepubertal growth spurt in girls is at the age of
A. 10–11 years B. 11–12 years
C. 11–13 years D. 13–14 years
21. Frontal bone shows?
A. Intramembranous ossification only B. Endochondral ossification only
C. Both of above D. None of the above
22. Which type of collagen is found in sutural tissues?
A. Type I B. Type II
C. Type III D. Type IV
23. The most critical period in the development of the palate is
A. 5–8 weeks B. 6–7 weeks
C. 4–7 weeks D. 6–9 weeks
24. The most stable area to evaluate craniofacial growth is the
A. Nasal floor B. Occlusal plane
C. Nasomaxillary complex D. Anterior cranial base
25. In a child with a history of generalized growth failure in the first 6 months of life, which of the following dental
sequelae is most likely to occur?
A. Enamel hypoplasia B. Retrusive maxilla
C. Retrusive mandible D. Dentinogenesis imperfect

https://t.me/DentalBooksWorld
Chapter 1 • Orthodontics 49

26. Calcification of teeth begins


A. At the first trimester B. At the second trimester
C. At the third trimester D. After birth

ANCHORAGE
1. Which of the following is related to occipital anchorage?
A. Intraoral force B. Tipping of tooth
C. Extra oral force D. Translation of tooth
2. Which of the following appliances takes anchorage from muscle?
A. Oral screen B. Bionator
C. Lip bumper D. Twin block
3. Which of the following is known as Baker’s anchorage?
A. Intramaxillary B. Compound
C. Stationary D. Intermaxillary
4. Inclined bite plane is the best example for
A. Simple anchorage B. Reciprocal anchorage
C. Reinforced anchorage D. Extra oral anchorage
5. The term “burning of anchorage” means
A. Unwanted movement of posterior teeth during fixed treatment
B. Unwanted movement of anterior teeth during fixed treatment
C. Unwanted movement of anchor unit
D. None of the above
6. SVED appliance is
A. Upper anterior inclined plane
B. Lower anterior inclined plane
C. Upper anterior bite plane
D. Upper anterior inclined plane that has an additional incisal capping
7. The most effective anchorage is in the
A. Triangular root B. Conical root
C. Flat root D. Round root
8. Which of the following is not used for reinforcing anchorage?
A. Upper anterior inclined plane B. Cervical head gear
C. Reverse pull head gear D. Nance holding arch
9. Elastic traction that can be used for correction of class 2 malocclusion is termed
A. Intramaxillary anchorage B. Intermaxillary anchorage
C. Bakers anchorage D. Both B and C
10. Which of the following is not a source for intraoral anchorage?
A. Musculature B. Teeth
C. Soft palate D. Cortical bone
11. Which of the following is an example for reinforced anchorage?
A. Lip strap B. Class 2 elastics
C. Trans palatal arch D. Finger springs
12. Tooth that has maximum anchorage value is
A. Mandibular 1st molar B. Maxillary 1st molar
C. Maxillary canine D. Mandibular 2nd molar
13. Simple anchorage means
A. Closure by intrusion B. Closure by bodily movements
C. Closure by tipping D. Closure by rotation

https://t.me/DentalBooksWorld
50 Triumph’s Complete Review of Dentistry

14. Intraoral anchorage can be obtained by


A. Tissue B. Teeth
C. Both of the above D. None of the above
15. Anchorage bends can also be termed
A. Bite opening bends B. Tip back bends
C. Both a and b D. Bayonet bend
16. Orthodontic mini implant provides
A. Relative anchorage B. Absolute anchorage
C. Floating anchorage D. None of the above
17. Anchorage that is derived from nape of neck is
A. Occipital anchorage B. Cervical anchorage
C. Parietal anchorage D. Facial anchorage
18. In which of the following planes can anchorage preservation be done?
A. Sagittal B. Vertical
C. Transverse D. All of the above
19. Anchorage for face mask is derived from
A. Parietal bone B. Nape of neck
C. Occipital region D. None of the above
20. Utility arch utilizes which type of anchorage when used in anchorage augmentation?
A. Cortical anchorage B. Muscular anchorage
C. Palatal anchorage D. Intermaxillary anchorage

BIOMECHANICS AND BIOLOGY OF TOOTH MOVEMENT


1. On comparing the orthodontic force responses between bone and cementum, the cementum resorbs
A. Less readily B. More readily
C. Both at same rate D. Does not resorb
2. What type of tooth movement occurs when Mc/Mf ratio is more than 1?
A. Controlled tipping B. Uncontrolled tipping
C. Translation D. Torque
3. The M-D displacement of the root tips comes under
A. First-order torque B. Second-order torque
C. Third-order torque D. Both A and B
4. Narrow bracket results in
A. Increase in flexibility B. Decrease in flexibility
C. Increases interbracket-arch span D. Both A and C
5. During extrusion of tooth, the periodontal ligament is subjected to an
A. Area of torsion B. Area of shear
C. Area of tension D. Area of compression
6. Who was the first to discuss about the root resorption of permanent teeth?
A. Bates B. Ottolengui
C. Bein D. Becker
7. Bone remodeling theory was proposed by
A. Bein B. Basset
C. Farrar D. Burstone
8. Prostaglandin was discovered by
A. Paul Erlich B. Von Euler
C. Ketcham D. Frost
9. Which of the following hormone is the primary regulator of frequency of bone remodeling?
A. Thyroxine B. Calcitonin
C. Vitamin D D. PTH/Parathormone

https://t.me/DentalBooksWorld
Chapter 1 • Orthodontics 51

10. The bone related activity of Osteoprotegerin (OPG) is


A. Osteoblast differentiation B. Bone resorption
C. Prevention of bone resorption D. Bone remodeling
11. Bone contains highest concentration of
A. Calcium phosphate B. Calcium carbonate
C. Calcium fluoride D. Sodium chloride
12. On applying a light orthodontic pressure, what is the minimum time taken for tooth movement to begin?
A. Occurs immediately B. 2 days
C. 3 days D. 4 days
13. Life span of an osteoclastic cell is
A. 2–7 days B. 2–10 days
C. 10–15 days D. 15–20 days
14. The force that declines to zero between the activations is called
A. Continuous force B. Interrupted force
C. Intermittent force D. Both A and C
15. An example of two couple systems
A. Trans-palatal arches B. Utility arches
C. Both the above D. Nance palatal arches
16. An example for Interrupted force is
A. Removable appliances B. Removable appliances
C. Orthopedic appliances D. Both B and C
17. During the physiological response to heavy pressure against a tooth, the minimum time required to generate
piezoelectric signal is
A. Less than 1 second B. 2–3 seconds
C. 3–5 seconds D. More than 5 seconds
18. Which of the following orthodontic forces is used to correct the effects of uncontrolled tipping?
A. Torquing force B. Couple
C. Moment of force D. Both B and C
19. During Torquing, the center of rotation lies at
A. C-r lies at the apex B. C-r lies at the infinity
C. C-r lies towards the incisal edge D. C-r lies close to C-res
20. Amount of force (in grams) required for tipping of canines is
A. 20–50 B. 50–75
C. 40–50 D. 70–100
21. The threshold force for orthodontic tooth movement is
A. 1–2 g/sq.cm B. 5–10 g/sq.cm
C. 10–15 g/sq.cm D. 20–26 g/sq.cm
22. Duration of lag phase in tooth movement corresponding to removal of hyalinized tissue is
A. 2 weeks B. 4 weeks
C. 6 weeks D. 8 weeks
23. All are horizontal trajectories of maxilla, except
A. Hard palate B. Orbital ridges
C. Lesser wing of sphenoid D. Pterygoid buttress
24. Pick out the WRONG statement regarding the universal sign convention for forces
A. Lateral forces are negative and medial forces are positive
B. Medial forces are positive and distal forces are negative
C. Anterior forces are positive and posterior forces are negative
D. Buccal forces are positive and lingual forces are negative

https://t.me/DentalBooksWorld
52 Triumph’s Complete Review of Dentistry

25. The C-res of maxilla, in transverse plane lies


A. Between the roots of premolars B. Through the center of maxillary sinus
C. Below the orbit D. Lateral to lateral incisors for intrusive movements

CEPHALOMETRICS
1. With age gonial angle tends to
A. Increase in size B. Decrease in size
C. First increase then decrease D. First decrease then increase
2. Maxilla ante inclination means
A. There is an anterior open bite B. Patient will exhibit horizontal growth pattern
C. Forward rotation of the maxilla D. There is a deep bite
3. ANB angle in severe class 2 malocclusion is
A. Negative B. Small
C. Normal D. Large
4. Most stable and commonly used plane for super imposition of lateral cephalogram in the study of growth of a child is
A. S-N plane B. Frankfort plane
C. Mandibular plane D. Occlusal plane
5. Which of the following reference plane is used in McNamara’s analysis?
A. FH plane B. Basion–nasion plane
C. Both of the above D. None of the above
6. Most suitable method for analyzing pre- and post-myofunctional therapy is
A. Pitchfork analysis B. Burstone analysis
C. Rickets growth prediction D. Using template analysis
7. Which of the following is a postero-anterior cephalometric analysis?
A. Harvold B. Grummon
C. Sassouni D. Margolis
8. Orthodontic/orthopedic therapy influences
A. Saddle angle B. Gonial angle
C. Articular angle D. Base plane angle
9. Growth estimation, orthodontic treatment planning, and surgical prediction can be assessed by
A. Quick cephalometrics B. Dentofacial planner
C. Both a and b D. None
10. Cephalometric analysis that emphasize vertical and horizontal proportions first is
A. Harvold analysis B. Down analysis
C. Sassouni analysis D. Template analysis
11. Clinically examined plane is
A. SN plane B. Palatal plane
C. Frankfort plane D. Bolton’s plane
12. Airway analysis is an integral part of
A. TOMAC analysis B. Ricketts analysis
C. McNamara’s analysis D. Moore’s proportional analysis
13. Centroid of mandibular symphysis
A. Point B B. Point D
C. All of the above D. None of the above
14. Which of the following relationships can be assessed by cephalometrics?
A. Tooth to tooth B. Bone to bone
C. Tooth to bone D. All of the above
15. Superimposition in longitudinal cephalometric studies best demonstrates the growth of
A. Plane and point B. Structures farthest from plane and point
C. Structures nearest to plane and point D. All of the above

https://t.me/DentalBooksWorld
Chapter 1 • Orthodontics 53

16. Cephalometrics is used for


A. Treat malocclusions B. Study growth changes
C. Aid in diagnosis and caste analysis D. Both B and C
17. What does ANB angle of 2 degrees indicate?
A. Favorable relationship of maxillary alveolar base to mandibular alveolar base
B. Favorable relationship of mandible to cranium
C. Poor cranial growth
D. Upright incisors
18. Cephalometric study reveals
A. Maxilla during growth is translated in a downward and forward direction
B. Mandibular growth stops after maxillary growth
C. Both A and B
D. Cranial base growth determines mandibular growth
19. In cephalometry
A. Frankfort plane passes horizontally from nasion through porion
B. Frankfort plane passes horizontally from nasion to Bolton point
C. Frankfort plane passes horizontally from orbitale to the superior aspect of external auditory meatus
D. Frankfort plane passes vertically from orbitale through the maxillary canine
20. Parameters used to diagnose a long face syndrome patient in cephalometrics is
A. Jarabak ratio B. Saddle angle
C. SNA D. SNB
21. Angle formed between occlusal plane and F-H plane is
A. 7 B. 9.3
C. 20 D. 0
22. Following cephalometric landmarks is not a midline structure
A. Nasion B. Menton
C. Sella D. Gonion
23. Horizontal plane that is most commonly used for cephalometric orientation is
A. SN plane B. FH plane
C. Occlusal plane D. Palatal plane
24. Mid-point amidst sella to Bolton point is
A. Condylion B. Nasion
C. Basion D. Registration point

CLEFT LIP AND PALATE


1. The incidence of cleft palate in India is approximately
A. 1:500 B. 1:700
C. 1:1,000 D. 1:2,000
2. The incidence of cleft lip and palate in India is approximately
A. 1:200 B. 1:400
C. 1:600 D. 1:800
3. Cleft palate patients are always evaluated for
A. Stickler syndrome B. Seckel syndrome
C. Streiff syndrome D. Crouzon syndrome
4. In Kernahan striped Y Classification, no. 2 and 5 denotes
A. Lip B. Alveolus
C. Hard palate anterior to incisive foramen D. Hard palate posterior to incisive foramen
5. Millard rule of over 10, in 1967 for cleft lip surgery includes all of the following except
A. Weight >10 pounds B. Hemoglobin >10 g%
C. Age >10 weeks D. Total WBC count 10,000/mm3

https://t.me/DentalBooksWorld
54 Triumph’s Complete Review of Dentistry

6. Rule of 10, recommended by Wilhelmsen and Musgrave (1966) includes all of the following except
A. 10 pounds weight B. Total WBC count 10,000/mm3
C. Hemoglobin 10 g% D. Age >10 weeks
7. Statement A – Unilateral clefts are more common than bilateral clefts
Statement B – Cleft palate is more often associated with bilateral cleft lip than unilateral cleft
A. Both the statements are true B. Both the statements are false
C. Statement A is true and Statement B is false D. Statement A is false and Statement B is true
8. The incidence of isolated cleft palate is more in
A. Males B. Females
C. Both the gender has equal predilection D. None
9. In Millard’s classification, No. 3 and 7 represents
A. Hard palate B. Soft palate
C. Alveolus D. Nasal floor
10. American cleft palate association classification (Internationally approved) was established in the year
A. 1962 B. 1931
C. 1958 D. 1987
11. In Veau’s classification, cleft of soft palate only falls under which group/class?
A. Class 1 B. Class 2
C. Class 3 D. Class 4
12. What is the effect of glucocorticoids on palatal growth?
A. It inhibits the growth of palatal mesenchyme
B. It potentiates the growth of palatal mesenchyme
C. Alters the terminal differentiation of medial palatal epithelial cells
D. It has no action of palatal growth
13. Risk of the second child being affected with cleft lip/palate when either one of the parent has cleft palate defect is
A. 5% B. 10%
C. 25% D. 30%
14. Secondary alveolar bone grafting for cleft palate is usually done by
A. 3–5 years of age B. 9–12 years of age
C. 6–7 years of age D. 7–10 years of age
15. Conventional orthodontic treatment for cleft palate patients is done by
A. 10 years of age B. 15 years of age
C. 18 years of age D. 3 years after the cleft palate surgery
16. All of the following are features of unoperated cleft subjects except
A. Tendency for normal maxillary development
B. Smaller sized mandible than noncleft patients
C. Maxilla is placed posteriorly compared to noncleft patients
D. Supernumerary teeth
17. The clefts of the lip and alveolus may have bands of soft tissue bridging across the two sides called
A. Simonart’s bands B. Bands of Büngner
C. Both of the above D. None of the above
18. Fusion of palatal shelves begins at
A. 8th week B. 9th week
C. 10th week D. 7th week
19. Primary ABG is usually done between the
A. First few days of life to 2.5 years age B. At birth
C. 3–6 years of age D. 5–8 years of age
20. Early secondary alveolar bone grafting is done between
A. 2 and 5 years old B. 5 and 7 years old
C. 8 and 10 years old D. Any time after cleft palate repair

https://t.me/DentalBooksWorld
Chapter 1 • Orthodontics 55

21. Late alveolar bone grafting is done


A. After 13 years of age B. After 8 years of age
C. After 16 years of age D. After 10 years of age
22. Cleft lip and palate patient often requires expansion. Appliance of choice in such cases is
A. Hyrax appliance B. Hass appliance
C. Cap splint type of expansion appliance D. SARPE

MODEL ANALYSIS
1. All of the following are mixed dentition analysis except
A. Nance–Carey’s analysis B. Moyer’s mixed dentition analysis
C. Total space analysis D. Linder–Harth index
2. The most accurate cast analysis among the following is
A. Moyer’s B. Stanley–Kerber
C. Kerby D. Tanaka–Johnston
3. If Carey’s arch perimeter analysis shows a discrepancy of more than 5 mm, then
A. Proceed with extraction of first premolar B. Proceed with extraction of second premolar
C. Proceed with extraction of first molar D. Do proximal stripping
4. Which of the following is true regarding Moyer’s mixed dentition analysis?
A. Can be done for both mandibular and maxillary arch
B. Estimation of size of the mandibular incisor on cast
C. Its analysis is based on radiograph
D. All of the above
5. In case of severe arch length discrepancy, which posterior teeth is usually displaced?
A. Mandibular second molar displaced buccally B. Mandibular second premolar displaced lingually
C. Mandibular second premolar displaced buccally D. Mandibular first premolar displaced buccally
6. Total space analysis was given by
A. Korkhaus B. L. Merrifield
C. Pont D. Nance–Carey
7. Which one among the following is false regarding Stanley–Kerber analysis?
A. This method uses radiograph and cast for the estimation of space
B. It is the most accurate method
C. This is applicable to both the arches
D. Canine width can be read directly from the sum of premolar and incisor widths
8. Unfavorable mixed dentition analysis is an indication for
A. Space maintenance and space regaining B. Space maintenance and space supervision
C. Space regaining and gross discrepancy cases D. Space supervision and gross discrepancy cases
9. Moyer mixed analysis give more accurate value for unerupted teeth at
A. 75% probability B. 50% probability
C. 70% probability D. 85% probability
10. The ratio between the anatomic portion and the artistic portion should be
A. 2:1 B. 1:2
C. 3:1 D. 1:3
11. Anterior Bolton ratio is
A. 91.3% B. 100%
C. 82.3% D. 77.2%
12. Excess in lower anterior Bolton is an indication for
A. Build up in upper anteriors B. Stripping in lower molars
C. Stripping in lower premolars D. Extraction of lower incisors

https://t.me/DentalBooksWorld
56 Triumph’s Complete Review of Dentistry

APPLIANCES
1. All of the following can be classified as myofunctional appliance except
A. An anterior bite plane B. Activator
C. Begg’s appliance D. Oral screen
2. Oral screens are used for all of the following purposes except
A. Mouth breathing B. Tongue thrusting
C. Lip biting D. Retraction of upper protruded teeth
3. At which stage is myofunctional appliance given?
A. Primary dentition B. Mixed dentition
C. Permanent dentition D. Adulthood
4. Which appliance is not suitable for skeletal C–III malocclusion at the age of 8 years?
A. Frankel III B. Activator III
C. Chin cap D. Maxillary splint appliance
5. All of the following appliances were introduced by Edward H Angle except
A. E-arch appliance B. Pin and tube appliance
C. Edgewise appliance D. Straight wire appliance
6. Edge wise appliance are used
A. On incisal edge of anterior teeth
B. With rectangular arch wire
C. With labial arch at incisal and gingival arch of each bracket
D. To obtain edge to edge occlusion
7. Cetlin appliance is used for
A. Molar distalization B. Anterior cross bite
C. Posterior cross bite D. For midline diastema
8. Whip spring is
A. Fixed B. Semifixed
C. Removable D. Functional
9. Example of a fixed functional appliance is
A. Churro jumper B. Mandibular anterior reposition appliance (MARA)
C. Sabbagh universal spring (SUS) D. All of above
10. Orthodontic appliances should not be reactivated more frequently than at
A. 1 week interval B. 2 week interval
C. 3 week interval D. 4 week interval
11. A chin cap should be worn by the patient for at least
A. 6 hours a day B. 10 hours a day
C. 14 hours a day D. 20 hours a day
12. The jasper jumper differs from Herbst appliance in?
A. Effect B. Rigidity
C. Indications D. All of the above
13. Functional regulator used in the treatment for correction of open bite is
A. FR 1c B. FR 5
C. FR 3 D. FR 4
14. Pterygoid response to activator is seen at
A. 2–3 weeks B. 4–6 weeks
C. 6–8 weeks D. 10–12 weeks
15. Wunderers modification of an Activator is used for correction of which malocclusion?
A. Class 2 div 1 B. Class 3
C. Class 2 div 2 D. Bimaxillary protrusion

https://t.me/DentalBooksWorld
Chapter 1 • Orthodontics 57

16. What type of force is exhibited by a removable functional appliance?


A. Intermittent B. Interrupted
C. Continuous D. All of the above
17. Identify the clasp shown in the picture


A. Adam’s clasp B. South end clasp
C. Delta clasp D. Adam’s clasp with modification
18. Identify the following picture


A. MARA B. SAIF spring
C. Jasper jumper D. Forsus appliance
19. Identify the following appliance


A. Functional regulator 2 B. Activator
C. Twin block D. Posterior bite plane
20. Identify the following picture


A. Mills retractor B. Robert’s retractor
C. Labial bow with Anderson loop D. U loop canine retractor

https://t.me/DentalBooksWorld
58 Triumph’s Complete Review of Dentistry

21. Identify the clasp in this image


A. Triangular clasp B. Double eyelet clasp
C. Arrowhead clasp D. South end clasp
22. Which of the following is an example for Group 1 myofunctional appliance?
A. Activator B. Bionator
C. Twin block D. Vestibular screen
23. One of the following is wrong about Herbst appliance?
A. Patience compliance is acceptable B. It can be worn for 24 hours a day
C. It is fixed D. Treatment period is long
24. An activator can be used as a passive appliance if
A. If more than 3 mm of occlusal correction is needed B. If less than 3 mm of occlusal correction is needed
C. An activator is always active D. It should not be used for occlusal correction
25. Which one of the following is the flexible fixed functional appliance?
A. Jasper jumper B. Herbst appliance
C. Twin block D. Activator
26. Churro jumper was introduced by
A. Castanon R et al. B. Jaraback
C. John J Stiffler et al. D. Rickets
27. The term Monoblock was given by
A. Dr. A Sauver B. Robin
C. Anderson D. Rickets
28. Activator is used to correct a retruded mandible by
A. Inducing growth at symphyseal region
B. Inducing growth at condylar region
C. Selective trimming
D. Mandibular teeth are guided to erupt more in distal direction
29. All of the following are contraindications of an activator except
A. Adult patient B. Vertical facial growth pattern
C. Growing patient with class 2 skeletal malocclusion D. Severe inclination of lower anterior
30. Normal inclination for slopes of twin block appliance is
A. 70 degrees B. 90 degrees
C. 45 degrees D. 60 degrees

SPACE MAINTAINERS AND SERIAL EXTRACTION


1. A dentist finds an 8-year-old boy with all primary molars present, which exhibits a cusp to cusp relation of permanent
maxillary and mandibular first molars. What should be the treatment plan for this case?
A. Regular recalls
B. Referral to an orthodontist for consultation
C. Plan for a serial extraction, for more normal adjustments of occlusion
D. Use of cervical headgear to reposition maxillary molars
E. Proximal disking of distal surfaces of primary mandibular second molars to permit normal adjustment of permanent
molars

https://t.me/DentalBooksWorld
Chapter 1 • Orthodontics 59

2. Which one of the following conditions is usually seen in a class II division 2 malocclusion?
A. Open bite B. Steep mandibular plane
C. Mesio-occlusion of permanent first molar D. Lingual inclination of maxillary central incisors
3. Wilkinson’s extraction is carried out at the age of
A. 7½–8½ years B. 8½–9½ years
C. 9½–10½ years D. 10½–11½ years
4. A 9-year-old boy has been observed with the following findings:
– Crowding of mandibular anterior teeth
– An end-to-end molar relationship
– Class II facial profile
– The mixed dentition analysis shows a 4.4 mm TSALD in the mandible.
Which of the following is the ideal management of this patient at this time?
A. Referral to an orthodontist for consultation
B. Start a serial extraction program to create room for posterior teeth
C. Removal of mandibular lateral incisor to allow crowded anterior teeth to align themselves
D. Do nothing because the probability is good that the child’s apparent malocclusion is a transitional stage and future
growth spurts will correct the problem.
5. A space maintainer is least indicated, when there is a premature loss of a
A. Permanent max. C.I B. Primary max. first molar
C. Primary max. C.I D. Primary man. First molar.
6. A 9-year-old patient shows a lingually locked permanent maxillary left central incisor. Supporting bone is intact
and in harmony with tooth size. Other relationships are normal without any basic discrepancies. What could be the
possible etiological feature?
A. Premature extraction of the primary maxillary right central incisor
B. Prolonged retention of the primary maxillary left central incisor
C. Premature extraction of the primary maxillary left central incisor with space loss in the area
D. Pronounced facioversion on eruption of the permanent mandibular left central incisor
7. A 10-year-old girl comes for examination immediately after losing a primary mandibular second molar with otherwise
clinically normal occlusion. What should the dentist do?
A. Observe the patient B. Place a lingual arch space maintainer
C. Place a functional space maintainer D. Base his choice of treatment upon radiographic findings
8. In a 5-year-old child, how will the extraction of a primary maxillary central incisor with incisal spacing affect the size
of the intercanine space?
A. The intercanine space will increase in size B. The intercanine space will decrease in size
C. The intercanine space will not change
9. Common cause of severe mandibular incisor crowding is
A. Mesial migration of teeth B. Premature loss of primary teeth
C. Presence of supernumerary teeth D. Tooth size arch length discrepancy
10. A distal shoe space maintainer is indicated when a primary
A. Incisor is avulsed
B. First molar is prematurely lost
C. Second molar is lost after eruption of a permanent first molar
D. Second molar is lost before eruption of a permanent first molar
11. Following an accident, an 8-year-old girl lost both maxillary central incisors. What would be the ideal treatment
option?
A. Observe the patient
B. Move the L.I into C.I position
C. Construct and place a space maintainer with bands cemented on L.I
D. Place a temporary prosthesis supplying artificial crowns for the lost central incisors

https://t.me/DentalBooksWorld
60 Triumph’s Complete Review of Dentistry

12. Premature exfoliation of a primary canine may indicate


A. An arch length excess B. An arch length deficiency
C. A skeletal malocclusion D. None of the above
13. The best space maintainer among the following is
A. A Nance holding arch B. A fixed lingual arch
C. A pulpectomized primary molar D. A Band and loop appliance
14. A mixed dentition 10-year-old patient has a cusp to cusp molar relationship. This will probably become a Class I
molar relation by
A. The maxillary molar drifting posteriorly 0.9 mm
B. The mandibular molar drifting forward 3.4 mm
C. Both molars drifting forward with the mandibular molar drifting about twice as far as the maxillary molar
D. Orthodontic intervention only
15. A substantial increase in width of maxillary arch is usually best achieved by placing
A. Lingual arch wires B. A sutural-expansion fixed appliance
C. Posterior intermaxillary cross elastic bands D. A face–bow headgear with an expanded inner bow
16. Space maintainers are usually needed in which area?
A. Mandibular primary incisor teeth area B. Maxillary primary incisor teeth area
C. Mandibular primary second molar teeth area D. Mandibular primary canine teeth area
17. Indication of serial extraction
A. Arch perimeter–tooth material discrepancy B. Spaced dentition
C. Skeletal abnormalities D. None of the above
18. The Dewel’s (usual) regimen followed in serial extraction is
A. C D 4 B. 4 D C
C. C 4 D D. D C 4
19. The Tweed’s method followed in serial extraction is
A. C D 4 B. D 4 C
C. C 4 D D. 4 C D
20. The Nance method followed in serial extraction is
A. D 4 C B. C D 4
C. 4 C D D. C D 4
21. During serial extraction procedures, the most critical observation is during the eruption of
A. Primary second molar B. Permanent maxillary molar
C. Permanent mandibular canine and first premolar D. Permanent mandibular incisors and canines
22. In which arch is the modified Dewel’s method of serial extraction usually carried out?
A. Maxillary arch B. Mandibular arch
C. Upper anterior region D. None of above
23. Balancing extraction refers to removal of tooth in
A. Opposing arch B. Opposite side of the same arch
C. Extraction in both upper and lower arches D. None of the above
24. Which of the following is a contraindication of Distal shoe space maintainer?
A. Patients with SABE B. Immunocompromised patients
C. Congenitally missing first molar D. All of above
25. Which of the following is a modification of band and loop space maintainer?
A. Holtz appliance B. Mayne appliance
C. Herbst appliance D. Gerber appliance
26. Who termed serial extraction as, “Active supervision of teeth by extraction”?
A. Bunon B. Kjellgren
C. Hayes Nance D. Rudolph Holtz
27. Nance palatal arch is a
A. Bilateral, nonfunctional B. Unilateral, functional
C. Bilateral, functional D. Unilateral, nonfunctional

https://t.me/DentalBooksWorld
Chapter 1 • Orthodontics 61

28. The term “Timely extraction” was given by


A. Stemm B. Willet
C. Mayne D. Wilkinson
29. In case of primary dentition, which of the following problems require most active tooth movement?
A. Transverse B. Vertical
C. Sagittal D. All of above
30. Which of the following orthodontic categories involves occlusal adjustment of teeth through judicious grinding?
A. Corrective orthodontics B. Interceptive orthodontics
C. Preventive orthodontics D. Surgical orthodontics

SURGICAL ORTHODONTICS
1. Best treatment for long face syndrome is
A. Impaction of maxilla B. Impaction of maxilla followed by mandibular surgery
C. Body osteotomy D. BSSO
2. Condition in which vertical section of ramus of mandible is done is
A. Mandibular protrusion B. Mandibular retrusion
C. Maxillary prognathism D. Maxillary retrognathism
3. What is the sequelae of bilateral sagittal split osteotomy?
A. TMD B. Tinnitus
C. Altered sensation postoperatively D. None of the above
4. Preferred procedure for mandibular advancement is
A. Bilateral sagittal split osteotomy B. Le fort osteotomy
C. Trans oral vertical ramus osteotomy D. Genioplasty
5. Skeletal deep bite is best treated surgically by
A. Bilateral sagittal split osteotomy B. Vertical ramus osteotomy
C. Le fort 1 superior repositioning of maxilla D. Le fort 1 inferior repositioning of maxilla
6. Preferred surgical procedure for maxillary advancement is
A. Vertical oblique ramus osteotomy B. Bilateral sagittal split osteotomy
C. Le fort 1 osteotomy D. None of the above
7. Widening the maxilla surgically is
A. Stable B. Least stable
C. Moderately stable D. Stability cannot be predicted
8. Wassmund procedure is a treatment for
A. Deep bite B. Open bite
C. Skeletal class 3 D. Maxillary prognathism
9. Corticotomy involves
A. Sectioning of abnormal frenum
B. Sectioning of supracrestal fibers
C. Sectioning of dentoalveolar bone into multiple small units to hasten orthodontic tooth movements
D. Cutting of maxillary tuberosity
10. Le fort 1 osteotomy is done for
A. Maxillary retrognathism B. Maxillary prognathism
C. Both of the above D. None of the above
11. Type of reduction genioplasty in which incidence of ptosis is maximum is
A. Osteoplasty of chin protuberance
B. Vertical reduction osteotomy with wedge ostectomy
C. Lateral reduction with midline ostectomy for broad based chins
D. None of the above

https://t.me/DentalBooksWorld
62 Triumph’s Complete Review of Dentistry

12. What are the objectives of orthognathic surgery?


A. Function B. Aesthetics
C. Stability D. All of the above
13. Cosmetic surgery of chin is known as
A. Rhinoplasty B. Chinoplasty
C. Symphysioplasty D. Genioplasty
14. Which categorization of age is most reliable for surgical orthodontics?
A. Dental B. Physiological
C. Skeletal D. Biological
15. Dental compensations to skeletal malocclusions are treated
A. Prior to surgery B. After surgery
C. During surgery D. None of the above
16. Which of the following is the major indication for orthognathic surgery before puberty is?
A. Transverse deficiency of maxilla B. Ankylosis of mandible
C. Prognathic mandible D. Prognathic maxilla
17. Crossbite to narrow maxilla is best treated surgically by
A. Only slow maxillary expansion B. Only rapid maxillary expansion
C. Surgically assisted rapid maxillary expansion D. Surgically assisted slow maxillary expansion
18. The best material for alveolar bone grafting is
A. Cancellous bone B. Autogenous bone graft
C. Heterogenous bone graft D. Hydroxyapatite crystal
19. Impaired growth after surgical correction of cleft lip and palate is due to
A. Periosteal stripping B. Scar formation
C. Interferences with blood supply D. Combination of the above
20. Ideal time for grafting alveolar bone is
A. During lip repair B. At the age of 5 years
C. During palatal repair D. At the age of 8–9 years
21. A minor surgical procedure that is most commonly undertaken in conjunction with orthodontic treatment is
A. Extraction B. Frenectomy
C. Pericision D. Edward procedure

ANSWERS
HISTORY OF ORTHODONTICS
1. Answer: C (Ref. History of Orthodontics, By Basavaraj Subhashchandra Phulari, 2013, page no. 1870)
Joseph Fox (1776–1816) wrote his “Natural History of the Teeth.” His major interest was dedicated toward the etiology of
dental irregularities and malpositioning. He classified types of anomalies and described various treatment devices.
2. Answer: D (Ref. History of Orthodontics, By Basavaraj Subhashchandra Phulari, 2013, page no. 1870)
3. Answer: C
4. Answer: B (Ref. History of Orthodontics, By Basavaraj Subhashchandra Phulari, 2013, page no. 1850)
5. Answer: A (Ref. Phillips’ Science of Dental Materials (e-book), By Kenneth J. Anusavice, Chiayi Shen, H. Ralph Rawls, 2014,
page no. 7)
6. Answer: A. Aulius Cornelius Celsus was the first person to advocate the use of finger pressure to align irregular teeth.
7. Answer: D (Ref. History of Orthodontics, By Basavaraj Subhashchandra Phulari, 2013, page no. 1884)
8. Answer: B (Ref. Textbook of Orthodontics (e-book), By Sridhar Premkumar, 2015, page no. 359)
9. Answer: C (Ref. Textbook of Orthodontics, By Gurkeerat Singh, 2015, page no. 477)
10. Answer: D

https://t.me/DentalBooksWorld
Chapter 1 • Orthodontics 63

11. Answer: A
12. Answer: D (Ref. Orthodontics: Diagnosis and Management of Malocclusion, By Om Prakash Kharbanda, 2009)
13. Answer: B (Ref. History of Orthodontics, By Basavaraj Subhashchandra Phulari, 2013, page no. 88)
14. Answer: C
15. Answer: A
16. Answer: C
17. Answer: D (Ref. Textbook of Orthodontics (e-book), By Sridhar Premkumar, 2015, page no. 2)
18. Answer: C (Ref. Textbook of Orthodontics, By Gurkeerat Singh, 2015, page no. 261)
19. Answer: A (Ref. History of Orthodontics, By Basavaraj Subhashchandra Phulari, 2013, page no. 1884)
20. Answer: B (Ref. History of Orthodontics, By Basavaraj Subhashchandra Phulari, 2013, page no. 1814)
21. Answer: C
22. Answer: A (Ref. Orthodontics: Diagnosis and Management of Malocclusion, By Om Prakash Kharbanda, 2009)
23. Answer: C (Ref. Orthodontics: Principles and Practice, By Basavaraj Subhashchandra Phulari, 2011, page no. 612)
24. Answer: D (Ref. History of Orthodontics, By Basavaraj Subhashchandra Phulari, 2013, page no. 1857)
25. Answer: A (Ref. History of Orthodontics, By Basavaraj Subhashchandra Phulari, 2013, page no. 173)
26. Answer: C (Ref. Orthodontics, By Kumar, 2008, page no. 482)
27. Answer: B (Ref. Textbook of Orthodontics (e-book), By Sridhar Premkumar, 2015, page no. 338)
28. Answer: C (Ref. History of Orthodontics, By Basavaraj Subhashchandra Phulari, 2013, page no. 1855)
29. Answer: B (Ref. Textbook of Orthodontics, By Gurkeerat Singh, 2015, page no. 176)
30. Answer: D (Ref. Textbook of Orthodontics (e-book), By Sridhar Premkumar, 2015, page no. 262)
31. Answer: A (Ref. Textbook of Orthodontics (e-book), By Sridhar Premkumar, 2015, page no. 581)

GROWTH AND DEVELOPMENT


1. Answer: B (Ref. Human Anatomy & Physiology, By Elaine N. Marieb, 2006, page no. 187)
• Bone usually grows by appositional growth, whereas soft tissues and cartilages grow by interstitial growth.
• At cellular level, growth in the body involves the following three mechanisms:
1. Hypertrophy – Increase in the size of cell
2. Hyperplasia – Increase in number of cells
3. Secretion of extracellular matrix.
• These three mechanisms contribute to growth of all tissues in the body.
2. Answer: C (Ref. Orthodontics, By Kumar, 2008, page no. 54)
• The movement of bone as a unit is called displacement. This displacement can be primary or secondary.
• In case of primary displacement, the bone gets displaced as a result of its own growth.
• In secondary displacement, the bone is displaced as a result of growth and enlargement of surrounding bone.
3. Answer: A (Ref. Handbook of Orthodontics, By Martyn T. Cobourne, Andrew T. DiBiase, 2015, page no. 120)
• There is a definite sequence in which growth of face (maxilla and mandible) is completed.
• According to this, the width (transverse plane) of face is completed first, followed by depth/length (sagittal plane) and the
last one to get completed is the height (vertical plane) of face.
• In the face, height shows the greatest incremental change. It means, postnatally the height of the face keeps growing and
it is the last to be completed.
4. Answer: B (Ref. Textbook Of Orthodontics (e-book), By Sridhar Premkumar, 2015, page no. 106)
5. Answer: C (Ref. Textbook of Orthodontics (e-book), By Sridhar Premkumar, 2015, page no. 33)
• Melvin Moss (1962) introduced the doctrine of functional matrix depending on the original concept of functional cranial
component by Van der Klaus (1952).
• This functional matrix theory attempts to associate the relationship between form and function.
• This theory emphasizes that the determinants of skeletal components mainly depend on nonskeletal tissues.
6. Answer: D (Ref. Nutrition, Health and Disease: A Lifespan Approach, By Simon Langley-Evans, 2015, page no. 190)
7. Answer: D (Ref. Human Embryology and Developmental Biology (e-book), By Bruce M. Carlson, 2008, page no. 194)

https://t.me/DentalBooksWorld
64 Triumph’s Complete Review of Dentistry

Parts of Head Purely Intramembranous Purely Endochondral Both


Bones Bones
Cranial vault Parietal and Frontal - -
Cranial base - Ethmoid bone Occipital, temporal, sphenoid
Face Maxilla and nasal bones - Mandible

8. Answer: A (Ref. Textbook of Orthodontics (e-book), By Sridhar Premkumar, 2015, page no. 66)
• Bjork (1969) was the first person to use implants to study the growth of bones.
• Tantalum implants were embedded in certain areas of maxilla and mandible to determine the growth of skull.
• Various radio-isotopes such as Technetium-33, Calcium-45, and Potassium-32 were also used to study the growth.
9. Answer: B (Ref. Modern Pediatric Dentistry, By Vinay Kumar Srivastava, 2011, page no. 87)
Postnatal growth of Mandible:
• Ramus: Resorption of anterior border and deposition along the posterior border of ramus. Ramus shifts posteriorly and
uprighting.
• Condyle: Growth posteriorly and superiorly when mandible is translated forward by capsular matrix condylar cartilage
fill-in.
• Condylar growth takes place in postero-superior direction in order to preserve the functionally important TMJ.
10. Answer: A (Ref. Robbins and Cotran Review of Pathology, By Edward C. Klatt and Vinay Kumar, 2015)
• Scammon’s growth curve demonstrates that lymphoid tissue proliferates rapidly from late childhood till puberty.
• These tissues complete 100% of growth at the age of 7 years and by the time of puberty (14 years), it almost doubles its
growth reaching up to 200%.
• This is an adaptation of immunological mechanisms to protect children against infection.
• At the age of 18 or more, lymphoid tissue undergoes regression. This is referred as negative growth or regressive growth
or involution.
11. Answer: C (Ref. Textbook of Orthodontics (e-book), By Sridhar Premkumar, 2015, page no. 32)

Various theories of growth Name of the scientists proposed


Functional matrix theory Melvin Moss
Cartilaginous theory James Scott
Sutural theory Sicher
Servo system theory/Cybernetics Alexander Petrovic and Chartier
Multifactorial theory Van Limborgh
Genetic theory Brodie
Neurotrophism Behrents
Bone remodeling theory Brash
12. Answer: D (Ref. International Journal of Orthodontics, Volume 16, page no. 19)
13. Answer: B (Ref. Textbook of Orthodontics, By Gurkeerat Singh, 2015, page no. 15)
14. Answer: B (Ref. Textbook of Craniofacial Growth, By Sridhar Premkumar, 2011, page no. 103)
• According to Enlow’s Counterpart Principle, the growth of certain skeletal parts of the craniofacial region are related
specifically to other structural and geometric counterparts in the face and cranium.
• Few of the counterparts situated in craniofacial region are:
1.  Nasomaxillary complex Anterior cranial fossa
2.  Middle cranial fossa Breadth of ramus
3.  Maxillary tuberosity Lingual tuberosity
4.  Bony maxilla Corpus of mandible
5.  Maxillary arch Mandibular arch

https://t.me/DentalBooksWorld
Chapter 1 • Orthodontics 65

15. Answer: A (Ref. Orthodontic Functional Appliances: Theory and Practice, By Padhraig S. Fleming and Robert T. Lee, 2016,
page no. 26)
• Condylar growth is a secondary cartilage and is peculiar. It shows interstitial growth pattern like cartilage as well as
appositional growth like bone.
• Hence condylar growth is considered as the “Pacemaker of mandibular growth.”
16. Answer: B (Ref. Oral Anatomy, Histology and Embryology (e-book), By Barry K. B Berkovitz, 2017, page no. 342)
17. Answer: C (Ref. Handbook of Orthodontics, By Martyn T. Cobourne and Andrew T. DiBiase, 2015, page no. 96)
• According to Bjork, three types of rotation are seen in the mandible.
1. Total rotation: Rotation of the mandibular core relative to cranial base.
2. Matrix rotation: Rotation of mandibular plane relative to cranial base.
3. Intramatrix rotation: Rotation of mandibular plane relative to core of mandible.
4. Matrix rotation = Total rotation – Intramatrix rotation.
18. Answer: C (Ref. Child Development: Understanding A Cultural Perspective, By Martin J. Packer, 2017, page no. 450)
19. Answer: B (Ref. Textbook of Orthodontics, By Gurkeerat Singh, 2015, page no. 25)
• Ossification centers:
1. Maxilla: One above the canine fossa and two for premaxilla (1+2 = 3)
2. Mandible: One each on the lateral side in the area of future mental foramen (2)
20. Answer: C (Ref. Nutrition, Health and Disease: A Lifespan Approach, By Simon Langley-Evans, 2015, page no. 190)
GROWTH SPURTS:
• Just before birth
• One year after birth
• Mixed dentition growth spurt
1. Boys: 8–11 years
2. Girls: 7–9 years
• Pre-pubertal growth spurt
1. Boys: 14–16 years
2. Girls: 11–13 years
21. Answer: A (Ref. Orthodontics (e-book): Current Principles and Techniques, By Lee W. Graber, Robert L. Vanarsdall,
Katherine W. L. Vig, 2016, page no. 10)
22. Answer: C (Ref. Stem Cells in Development and Disease, page no. 99)
23. Answer: D (Ref. Human Embryology: The Ultimate USMLE Step 1 Review, By Philip R. Brauer, 2003, page no. 53)
• The development of palate begins in 6th week of IUL and it will be completed by the 12th week of IUL.
• The most critical period in the development of palate is the end of the 6th week to the beginning of the 9th week.
24. Answer: D (Ref. Textbook of Craniofacial Growth, By Sridhar Premkumar, 2011, page no. 187)
25. Answer: A (Ref. Textbook of Orthodontics, By Samir E. Bishara, 2001)
26. Answer: B (Ref. Woelfel’s Dental Anatomy, By Rickne C. Scheid, 2012, page no. 168)

ANCHORAGE
1. Answer: C (Ref. Mosby’s Dental Dictionary (e-book), page no. 32)
2. Answer: C (Ref. Textbook of Orthodontics (e-book), By Sridhar Premkumar, page no. 342)
3. Answer: D (Ref. Textbook of Orthodontics, By Gurkeerat Singh, page no. 269)
4. Answer: C (Ref. Orthodontic Functional Appliances: Theory and Practice, By Padhraig S. Fleming and Robert T. Lee, 2016,
page no. 116)
5. Answer: C (Ref. Bioengineering: Analysis of Orthodontic Mechanics, By Robert J. Nikolai, 1985, page no. 416)
6. Answer: D (Ref. Textbook of Orthodontics (e-book), By Sridhar Premkumar, 2015, page no. 517)
7. Answer: A (Ref. Fundamentals of Fixed Prosthodontics, By Herbert T. Shillingburg and David A. Sather, 2012)
8. Answer: C (Ref. Mosby’s Orthodontic Review (e-book), By Jeryl D. English, Sercan Akyalcin, Timo Peltomaki, 2014)
9. Answer: D (Ref. Textbook of Orthodontics, By Gurkeerat Singh, 2015)
10. Answer: C (Ref. Textbook of Orthodontics, By Gurkeerat Singh, 2015, page no. 265)

https://t.me/DentalBooksWorld
66 Triumph’s Complete Review of Dentistry

11. Answer: C (Ref. Textbook of Orthodontics (e-book), By Sridhar Premkumar, 2015, page no. 342)
12. Answer: B (Ref. Orthodontics in the Vertical Dimension: A Case-Based Review, Thomas E. Southard, Steven D. Marshall,
Laura L. Bonner, 2015, page no. 87)
13. Answer: C (Ref. Textbook of Orthodontics (e-book), By Sridhar Premkumar, 2015, page no. 338)
14. Answer: C (Ref. Textbook of Orthodontics (e-book), By Sridhar Premkumar, 2015, page no. 340)
15. Answer: C (Ref. History of Orthodontics, By Basavaraj Subhashchandra Phulari, page no. 177)
16. Answer: B (Ref. Orthodontics: Diagnosis and Management of Malocclusion, By Om Prakash Kharbanda, 2009)
17. Answer: B (Ref. Textbook of Orthodontics, By Gurkeerat Singh, 2015, page no. 508)
18. Answer: D (Ref. Handbook of Orthodontics, By Martyn T. Cobourne and Andrew T. DiBiase, 2015, page no. 152)
19. Answer: D (Ref. An Introduction to Orthodontics, By Laura Mitchell, 2013, page no. 189)
20. Answer: A (Ref. Dental Implants (e-book): The Art and Science, By Charles A. Babbush, Jack A. Hahn, Jack T. Krauser, 2010,
page no. 293)

BIOMECHANICS AND BIOLOGY OF TOOTH MOVEMENT


1. Answer: A (Ref. Textbook of Orthodontics (e-book), By Sridhar Premkumar, 2015, page no. 333)
Cementum is highly resistant to resorption than bone. As we all know, cementum is avascular and bone is richly vascular; all
the degenerative processes would be easily affected by interference with blood circulation in bone. Thus cementum resorbs
less readily than bone.
2. Answer: D (Ref. Contemporary Orthodontics, By William R. Proffit and Henry W. Fields, 2000, page no. 342)
M/F ratio determines the center of rotation and thereby the type of tooth movement.
M/F Ratio Tooth movement
0:1–5:1 Uncontrolled tipping
7:1 Controlled tipping
10:1 Translation
12:1 Torque
3. Answer: B (Ref. Tip-edge Orthodontics and the Plus Bracket, By Richard Parkhouse, 2009, page no. 37)
Tip or the mesiodistal angulation of a tooth is also known as second-order torque. Second-order torque is a force system
used to alter a long axis orientation in a faciolingual plane resulting in mesiodistal displacement of root tips.
4. Answer: D (Ref. Orthodontic Applications of Biomaterials: A Clinical Guide, By Theodore Eliades and William A. Brantley,
2016, page no. 102)
A narrow bracket results in increased interbracket-arch span eventually resulting in increasing the flexibility of arch wire.
5. Answer: C (Ref. Carranza’s Clinical Periodontology (e-book): Expert Consult: Online, By Michael G. Newman, Henry Takei,
Perry R. Klokkevold, 2014)
Extrusion of the tooth is also one of the types of bodily movements. Here the tooth moves outside the socket, in the vertical
direction toward the occlusal plane. During this extrusion process, an area of tension is created in the periodontal ligament.
6. Answer: A (Ref. Orthodontic Treatment of Impacted Teeth, By Adrian Becker, 2012, page no. 175)
Bates, in 1856, was the first person to discuss about the root resorption of permanent teeth.
7. Answer: C (Ref. Craniofacial and Dental Developmental Defects: Diagnosis and Management, By J Timothy Wright, 2015)
Bone remodeling theory was proposed by Farrar in 1888.
8. Answer: B (Ref. Anatomy and Physiology (e-book), By Kevin T. Patton, 2015, page no. 573)
Von Euler discovered prostaglandins. They are the group of chemical mediators which belongs to a hormonal family called
eicosanoids.
9. Answer: D (Ref. Recent Progress in Hormone Research: Proceedings of the 1987, By James H. Clark, 2013)
10. Answer: C (Ref. Bone Formation, By Felix Bronner, Mary C. Farach-Carson, Mary Farach-Carson, 2004, page no. 52)
OPG is secreted by osteoblasts and blocks the effects of RANKL. This bone active substance mainly prevents bone resorption
by binding to RANKL extracellularly.
11. Answer: A (Ref. Dental Caries: The Disease and Its Clinical Management, By Ole Fejerskov and Edwina Kidd, 2009,
page no. 294)

https://t.me/DentalBooksWorld
Chapter 1 • Orthodontics 67

The main inorganic components present in bone are:


1. Calcium phosphate 51%
2. Calcium carbonate 11%
3. Calcium fluoride 2%
4. Sodium chloride 1%
5. Magnesium phosphate 1%
12. Answer: B (Ref. Orthodontics: Principles & Practice, By Thomas M. Graber, 1972, page no. 502)
The minimum time taken by the tooth movement to begin when light orthodontic pressure is applied is 2 days. This movement
is due to bone resorption and deposition (bone remodeling) carried out by osteoclasts and osteoblasts, respectively.
13. Answer: D (Ref. Principles of Molecular Rheumatology, By George C. Tsokos, 2000, page no. 289)
14. Answer: B (Contemporary Orthodontics (e-book), By William R. Proffit, Henry W. Fields, David M. Sarver, 2014)
• Continuous force: Here the force is maintained at the same appreciable level of the original force between two visits of the
patient.
• Interrupted force: It is the force in which the levels decline to zero between the activations.
• Intermittent force: Here the force value declines abruptly to zero when the patient removes the appliance or when a fixed
appliance is temporarily deactivated.
15. Answer: C (Ref. Textbook of Orthodontics, By Samir E. Bishara, 2001, page no. 213)
• When an arch wire is inserted into two consecutive brackets, activation of wire produces a couple at each bracket. The
couple at each bracket has an associated equilibrium. This is called two couple system.
• This is also known as, statistically indeterminate system.
16. Answer: D (Ref. Text book of orthodontics, By S Gowri Sankar, 2011, page no. 293)
17. Answer: A (Ref. Textbook of Orthodontics, By Gurkeerat Singh, 2015, page no. 230)

Time (in seconds) Events


<1 PDL fluid is compressible, Alveolar bone bends, Piezoelectric effect
1–2 PDL fluid expression, tooth moves in PDL space
3–5 PDL fluid squeezed out, immediate pain if pressure is heavy
18. Answer: A (Ref. Textbook of Orthodontics (e-book), By Sridhar Premkumar, 2015, page no. 338)
• Torquing force results in the movement of root without any significant crown movement.
• This is used to correct the effects of uncontrolled tipping.
19. Answer: C (Ref. Textbook of Orthodontics, By Gurkeerat Singh, 2015, page no. 509)

Location of crot Tooth movement


At infinity Translation/Bodily movement
At the tip of root apex Controlled tipping
Between the apical one-third to C-res of the tooth Uncontrolled tipping
At the bracket slot or crown of the tooth/incisal edges Torquing
20. Answer: B (Ref. Orthodontics: Diagnosis and Management of Malocclusion, By Om Prakash Kharbanda, 2009,)

Type of tooth movement Amount of force (g)


Optimum orthodontic force 20–26 g/sq. cm of root
Tipping of U/L anteriors 20–50
Tipping of canines 50–70
Translation of anteriors 40–50
Translation of posteriors 100–150
Rotation, Extrusion, and Torquing of a tooth 50–75
Uprighting of teeth 75–125
Maximum upper limit of tooth movement 400

https://t.me/DentalBooksWorld
68 Triumph’s Complete Review of Dentistry

21. Answer: B (Ref. Orthodontics: Diagnosis and Management of Malocclusion, By Om Prakash Kharbanda, 2009)
The threshold force for orthodontic tooth movement is 5–10 g/sq.cm, acting for a minimum period of 6 hours continuously
in one day.
22. Answer: C (Ref. Textbook of Orthodontics, By Gurkeerat Singh, 2015, page no. 229)
• Clinically it is not possible to bring the tooth movement only with the help of frontal resorption. Unavoidably,
undermining resorption also occurs.
• Lag phase in tooth movement corresponds to removal of hyalinized tissue produced by undermining resorption.
• The minimum duration of lag phase is about 6 weeks.
23. Answer: D (Ref. Textbook of Orthodontics (e-book), By Sridhar Premkumar, 2015, page no. 106)
• Horizontal trajectories of maxilla include hard palate, orbital ridges, zygomatic arches, palatal bones, and lesser wing of
sphenoid.
• Vertical trajectories of maxilla include the malar-zygomatic buttress, the frontonasal buttress, and the pterygoid buttress.
24. Answer: A (Ref. Orthodontics: Current Principles and Techniques (e-book), By Lee W. Graber, Robert L. Vanarsdall,
Katherine W. L. Vig, 2016, page no. 155)
The universal sign convention for orthodontic forces are:
• Anterior forces are positive and posterior forces are negative
• Lateral forces are positive and medial forces are negative
• Forces on medial direction are positive and forces on distal direction are negative
• Forces in buccal direction are positive and forces in lingual direction are negative
25. Answer: B (Ref. Textbook of Orthodontics, By Samir E. Bishara, 2001, page no. 226)
Centre of resistance of maxilla:
In vertical direction In transverse direction C-res of maxilla as such
Below the orbit and lateral to Passes through the center of maxillary Between the roots of the premolars
the lateral incisors for intrusive sinus
movements of the maxillary teeth

CEPHALOMETRICS
1. Answer: B (Ref. Orthodontic Treatment of Class III Malocclusion, By Peter W. Ngan, Toshio Deguchi, Eugene W. Roberts,
2014, page no. 68)
2. Answer: C (Ref. Textbook of Craniofacial Growth, By Sridhar Premkumar, 2011, page no. 229)
3. Answer: D (Ref. Esthetics and Biomechanics in Orthodontics (e-book), By Ravindra Nanda, 2012, page no. 202)
4. Answer: A (Ref. Craniofacial Identification, By Caroline Wilkinson and Christopher Rynn, 2012, page no. 155)
5. Answer: C (Ref. Textbook of Orthodontics (e-book), By Sridhar Premkumar, 2015, page no. 273)
6. Answer: A (Ref. Australian Orthodontic Journal, Volume 15, page no. 199)
7. Answer: B (Ref. Esthetics and Biomechanics in Orthodontics (e-book), By Ravindra Nanda, 2012, page no. 28)
8. Answer: C (Ref. Essential Orthodontics, By Birgit Thilander, Krister Bjerklin, Lars Bondemark, 2017, page no. 133)
9. Answer: C (Ref. Textbook of Oral Radiology (e-book), By Anil Govindrao Ghom, 2017, page no. 353)
10. Answer: C (Ref. Contemporary Orthodontics, By William R. Proffit and Henry W. Fields, 2000, page no. 178)
11. Answer: C (Ref. Textbook of Orthodontics (e-book), By Sridhar Premkumar, 2015, page no. 150)
12. Answer: C (Ref. Textbook of Orthodontics (e-book), By Sridhar Premkumar, 2015, page no. 286)
13. Answer: B (Re. Orthodontics: Diagnosis and Management of Malocclusion, By Om Prakash Kharbanda, 2009)
14. Answer: D (Ref. Orthodontic and Dentofacial Orthopedic Treatment, By Thomas Rakosi and Thomas M. Graber, 2011,
page no. 25) 
15. Answer: B (Ref. Surgical-orthodontic treatment, By William R. Proffit and Raymond P. White, 1991, page no. 125)
16. Answer: D (Ref. Orthodontics: Principles and Practice, By Basavaraj Subhashchandra Phulari, 2011, page no. 182)
17. Answer: A (Ref. Orthodontics: Current Principles and Techniques (e-book), By Lee W. Graber, Robert L. Vanarsdall,
Katherine W. L. Vig, 2016, page no. 997)
18. Answer: C
19. Answer: C (Ref. General Anatomy and Osteology of Head and Neck, By Mahdi Hasan, 2009, page no. 89)
20. Answer: A (Ref. Mosby’s Orthodontic Review (e-book), By Jeryl D. English, Sercan Akyalcin, Timo Peltomaki, 2014,
page no. 12)

https://t.me/DentalBooksWorld
Chapter 1 • Orthodontics 69

21. Answer: B (Ref. Orthodontics: Principles and Practice, By Basavaraj Subhashchandra Phulari, 2011, page no. 191)
22. Answer: D (Ref. Textbook of Oral Radiology (e-book), By Anil Govindrao Ghom, 2017, page no. 352)
23. Answer: B (Ref. Orthodontics: Diagnosis and Management of Malocclusion, By Om Prakash Kharbanda, 2009)
24. Answer: D (Ref. Textbook of Orthodontics (e-book), By Sridhar Premkumar, 2015, page no. 269)

CLEFT LIP AND PALATE


1. Answer: A (Ref. Textbook of Oral and Maxillofacial Surgery, By Neelima Anil Malik, 2012, page no. 616)
Race Incidence
Generally 1:700 live birth
Asia 1:500 live birth
India 5 per 1,000 live birth (combined CLP)
2 per 1,000 (CP alone)
Caucasian 1:500–600 live births
Pakistan 1.91:1,000 live births
2. Answer: C (Ref. Textbook of Oral and Maxillofacial Surgery – Page 616, By Neelima Anil Malik, 2012, page no. 616)
3. Answer: A (Ref. Syndromes of the Head and Neck, By Robert J. Gorlin, M. Michael Cohen Jr, Raoul C.M. Hennekam, 2001,
page no. 351)
Syndromes associated with cleft palate
Several syndromes are associated with cleft lip and cleft palate. Apart from the several syndromes reported in cleft lip and
palate, certain syndromes which require special attention are Velocardiofacial syndrome, Van der Woude syndrome (VWS),
and Stickler syndrome. Fetal alcohol syndrome, Holzgreve syndrome, Marfan syndrome, Myotonic dystrophy, Klippel–Feil
syndrome, Patau syndrome, Potter sequence, and Pierre Robin sequence are also some of the syndromes which have been
associated with cleft lip and palate.
4. Answer: A (Ref. Textbook of Oral and Maxillofacial Surgery, By Balaji, 2009, page no. 496)

KERNAHAN STRIPED Y CLASSIFICATION (1971)


R L

1 4
LIP

2 5 ALVEOLUS

3 6
INCISIVE PRIMARY
FORAMEN PALATE
7
HARD PALATE
8

9 SOFT PALATE

Number Structures
1 and 4 Lip
2 and 5 Alveolus
3 and 6 Hard palate anterior to incisive foramen
7 and 8 Hard palate posterior to incisive foramen
9 Soft palate
5. Answer: D (Ref. Neonatal Surgery of the Cleft Lip and Palate, By S.N. Desai, 1997, page no. 9)
“Millard rule of over 10” by Millard (1967) for lip surgery includes
i. Weight >10 pounds
ii. Hemoglobin >10 g%
iii. Age >10 weeks

https://t.me/DentalBooksWorld
70 Triumph’s Complete Review of Dentistry

“Rule of 10” was recommended by Wilhelmsen and Musgrave (1965)


i. Weight of 10 pounds
ii. Hemoglobin – 10 g%
iii. Total WBCs count of 10,000/mm3
6. Answer: D (Ref. Neonatal Surgery of the Cleft Lip and Palate, By S.N. Desai, 1997, page no. 9)
7. Answer: A (Ref. Orthodontics: Principles and Practice, By Basavaraj Subhashchandra Phulari, 2011, page no. 561)
8. Answer: B (Ref. Craniofacial Surgery for the Facial Plastic Surgeon, By Lisa M. Morris and Sherard A. Tatum, 2016)
9. Answer: C (Ref. Textbook of Prosthodontics, By Deepak Nallaswamy, 2017)
Nose
1 Nasal floor 5

2 Lip 6

Alveolus
3 7

Hard palate
4 8

9
Hard

palate
10

Soft
11
palate

Number Representing structure


Inverted triangles Nose
1 and 5 Nasal floor
2 and 6 Lip
3 and 7 Alveolus
4 and 8 Hard palate anterior to incisive foramen
9 and 10 Hard palate posterior to incisive foramen
11 Soft palate
10. Answer: A (Ref. Textbook of Orthodontics, By Gurkeerat Singh, 2015, page no. 691)
11. Answer: A (Ref. Shafer’s Textbook of Oral Pathology, By Arya Rajendran and B Sivapathasundharam, 2014, page no. 19)
Veau’s Classification
Veau proposed the following classification in 1931.
Group I (A) – Defects of the soft palate only
Group II (B) – Defects involving the hard palate and soft palate extending not further than the incisive foramen, thus
involving the secondary palate alone
Group III (C) – Complete unilateral cleft, extending from the soft palate to the alveolus, usually involving the lip
Group IV (D) – Complete bilateral clefts, resembles Group III but is bilateral. When cleft is bilateral, premaxilla is suspended
from the nasal septum
12. Answer: A (Ref. Teratogenic Mechanisms, By T.V.N. Persaud, 2012, page no. 76)
13. Answer: A (Ref. Cleft Palate & Craniofacial Anomalies: Effects on Speech and Resonance, By Ann W. Kummer, 2013)

https://t.me/DentalBooksWorld
Chapter 1 • Orthodontics 71

14. Answer: D (Ref. Textbook of Oral and Maxillofacial Surgery, By Balaji, 2009, page no. 510)

Primary alveolar bone grafting Birth–2.5 years of age


Early secondary alveolar bone grafting 3–5 years of age
Secondary alveolar bone grafting 7–10 years
Late secondary alveolar bone grafting Above 13 years of age
15. Answer: B (Ref. Orthodontic Treatment of Class III Malocclusion, By Peter W. Ngan, Toshio Deguchi, Eugene W. Roberts,
2014, page no. 418)
16. Answer: C (Ref. Cleft Lip and Palate: Diagnosis and Management, By Samuel Berkowitz, 2013, page no. 258)
17. Answer: A (Treatment of Facial Cleft Deformities: An Illustrated Guide (https://books.google.co.in/books?isbn=1563860325)
By Kurt-W. Bütow, 1995, page no. 45)
Simonart’s band – Soft tissue bridge located at the base of the nostril.
Effects of Simonart’s band
• Mild but positive influence on the final facial pattern
• Reduce the need for second surgeries of the lip and nose
• Long-term influence on the morphology of maxillary dental teeth
18. Answer: A (Ref. Textbook of Craniofacial Growth, By Sridhar Premkumar, 2011, page no. 41)
19. Answer: A (Ref. Textbook of Oral and Maxillofacial Surgery, By Balaji, 2009, page no. 510)
20. Answer: A (Ref. Cleft Lip and Palate, By M.T. Cobourne, 2012, page no. 110)
21. Answer: A (Ref. Cleft Lip and Palate, By M.T. Cobourne, 2012, page no. 110)
22. Answer: C (Ref. Orthognathic Surgery: Principles, Planning and Practice, By Farhad B. Naini and Daljit S. Gill, 2017,
page no. 311)

MODEL ANALYSIS
1. Answer: D (Ref. Textbook of Orthodontics (e-book), By Sridhar Premkumar, 2015, page no. 244)
Various analyses to study the relationship of tooth size and available space during mixed dentition are as follows:
• Moyer’s Mixed Dentition Analysis
• Tanaka and Johnston Analysis
• Radiographic Analysis
• Ballard and Willie Analysis
• Huckaba’s Analysis
• Staley–Kerber Analysis
• Hixon and Oldfather Analysis
Various analyses used in maxillary and mandibular arch:
• Maxillary dentition (upper teeth)
–– Pont’s Analysis
–– Linder–Harth Index
–– Korkhaus Analysis
–– Arch Perimeter Analysis
• Mandibular dentition (lower teeth)
–– Ashley Howe’s Analysis
–– Carey’s Analysis
• Both arches (upper and lower teeth)
–– Bolton Analysis
2. Answer: B (Ref. Textbook of Orthodontics (e-book) – Page 243, By Sridhar Premkumar, 2015, page no. 243)
3. Answer: A (Ref. Orthodontics: Principles and Practice, By Basavaraj Subhashchandra Phulari, 2011, page no. 172)
CAREY’S ANALYSIS
• The arch-length–tooth material discrepancy is the main cause for most malocclusion and can be calculated with the help
of Carey’s analysis.

https://t.me/DentalBooksWorld
72 Triumph’s Complete Review of Dentistry

PROCEDURE
• The arch length is measured anterior to the first permanent molar using a soft brass wire. The wire is placed touching
the mesial aspect of lower first permanent molar, then passed along the buccal cusps of premolars, incisal edges of the
anteriors and finally continued the some way up to mesial of the first molar of the contralateral side.
• The brass wire should be passed along the cingulum of anterior teeth if anteriors are proclined and along the labial surface
if anteriors are retroclined. The mesiodistal width of teeth anterior to first molar are measured and summed up as total
tooth material. The difference between the arch length and the actual measured tooth material gives the discrepancy.
INTERPRETATION
• If the arch length discrepancy is
–– 0–2.5 mm – proximal stripping can be carried out to reduce the total tooth material
–– 2.5–5 mm – extraction of second premolar is indicated
–– Greater than 5 mm – extraction of first premolar is usually required
4. Answer: A (Ref. Textbook of Orthodontics (e-book), By Sridhar Premkumar, 2015, page no. 244)
It is a nonradiographic method.
5. Answer: C (Ref. Textbook of Orthodontics (e-book), By Sridhar Premkumar, 2015, page no. 582)
Because, mandibular second molar erupts after the second premolar and also buccal displacement is more common than
lingual displacement.
6. Answer: B (Ref. Orthodontics: Current Principles and Techniques (e-book), By Lee W. Graber, Robert L. Vanarsdall,
Katherine W. L. Vig, 2016, page no. 492)
7. Answer: C (Ref. Orthodontics: Diagnosis and Management of Malocclusion, By Om Prakash Kharbanda, 2009)
It is done only in the mandibular arch.
8. Answer: D (Ref. Textbook of Orthodontics (e-book), By Sridhar Premkumar, 2015, page no. xvii)
9. Answer: B or A? (Ref. Textbook of Orthodontics (e-book), By Sridhar Premkumar, 2015, page no. 244)
10. Answer: A (Ref. Orthodontics: Diagnosis and Management of Malocclusion, By Om Prakash Kharbanda, 2009)
The ratio between anatomic to artistic portion should be 2:1 in a well-trimmed study cast. The tooth portion, soft tissue
portion, and the artistic are related in 1:1:1 fashion.
11. Answer: D (Ref. Orthodontics: Diagnosis and Management of Malocclusion, By Om Prakash Kharbanda, 2009)
An Overall Analysis measures the sum of mesiodistal width of all 12 (first molar to first molar) mandibular teeth and
compares them to the 12 maxillary teeth. The overall ratio known to be 91.3%.
The Anterior analysis measures the sum of mesiodistal width of front 6 mandibular teeth and compares them to maxillary

teeth. The anterior ratio is known to be 77.2%.


An overall ratio of more than 91.3% means that the mandibular teeth are bigger compared to normal. A ratio smaller than
91.3% would mean the mandibular teeth are smaller than normal.
Anterior analysis follows the same principle. Having a different ratio than normal is referred to as Bolton Discrepancy.
A standard deviation of more than 2 yields a significant discrepancy.
12. Answer: D (Ref. Orthodontics: Diagnosis and Management of Malocclusion, By Om Prakash Kharbanda, 2009)
If the ratio is more than the mean value than the mandibular tooth material is excess.
If the ratio is less than the mean value then the maxillary tooth material is excess.
Bolton preferred to do proximal stripping on the upper arch if the upper anterior tooth material is in excess and extraction
of lower incisor, if necessary to reduce tooth material in lower arch.
Permanent Dentition Mixed Dentition
Model Analysis Model Analysis
Pont’s analysis Moyer’s Mixed dentition analysis
Carey’s analysis Tanaka and johnston analysis
Ashley Howe’s analysis Nance mixed dentition analysis
Arch perimeter analysis Radiographic method (Huckaba’s mixed dentition analysis)
Linder Harth analysis
Korkhaus analysis
Bolton’s analysis
Peck and Peck index

https://t.me/DentalBooksWorld
Chapter 1 • Orthodontics 73

APPLIANCES
1. Answer: C (Ref. An Introduction to Orthodontics, By Laura Mitchell, 2013, page no. 121)
Myofunctional appliances in orthodontics are those appliances which take help from the muscles to act on the desired
treatment plan, as the name suggests the muscles are used to bring out the desired function.
Tooth-borne passive appliances Activator, bionator, Herbst appliance
Tooth-borne active appliances Expansion screws, springs
Tissue-borne passive appliances Frankel regulator (only tissue born passive
appliance)
Fixed functional appliances Herbst appliance, jasper jumper, MARS
Removable functional appliances Activator, bionator, Frankel appliances
Removable fixed functional appliance or Lip bumper
semifixed appliance
GROUP OF MYOFUNCTIONAL APPLIANCES BASED ON THEIR FUNCTION

Group 1 Appliances which transmit muscle force Oral screen, Inclined Plane
to teeth
Group 2 Appliances which reposition mandible Activator, bionator
Group 3 Appliances which reposition mandible by Frankel appliance, vestibular appliance
acting on the vestibule
2. Answer: None or B (by exclusion) (Ref. Orthodontics: Principles and Practice, By Basavaraj Subhashchandra Phulari, 2011,
page no. 445)
The oral screen is a functional appliance, suitable for the treatment of developing malocclusion associated with aberrant
muscular patterns. The better muscle balance between the tongue and the buccinator mechanism can be established, and the
reestablishment of normal growth and development can be achieved. The oral screen can be used for the correction of the
following conditions:
(1)  thumb sucking, tongue thrusting, and lip biting
(2)  mouth breathing
(3)  mild distocclusion with premaxillary protrusion
(4)  open bites in deciduous and mixed dentition
(5)  incompetent lips
The patient should wear the oral screen every night and also during the day whenever possible. The effects of oral screen can
be elevated through lip seal exercise: the lips should be kept in contact all the time to improve the lip seal. In the presented
two cases, the patients were considered mouth breathers having incompetent lips, one patient with maxillary incisal
protrusion and the other with open bite. They were instructed to wear the oral screen with lip seal exercise. After wearing
the appliance for 1 and 2 years, respectively, mouth breathing was decreased and lip length and strength were increased, the
maxillary incisors were retruded and open bite reduced.
3. Answer: B (Ref. Orthodontic Management of the Developing Dentition: An Evidence-based Guide, By Martyn T. Cobourne,
2017, page no. 49) 
The timing of treatment varies according to the type of malocclusion. Researchers have stated that early myofunctional
treatment during mixed dentition contributes, to a certain extent, to treat malocclusion. It has been indicated that the most
ideal time to use a pre-orthodontic trainer is when lower–upper incisors erupt. This is the right time to guide newly erupted
teeth and for myofunctional training. A pre-orthodontic trainer can be used up to the permanent dentition stage.
4. Answer: D (Ref. Esthetics and Biomechanics in Orthodontics (e-book), By Ravindra Nanda, 2012, page no. 257)
Treatment of skeletal class III malocclusion is difficult when compared with a nonskeletal class III malocclusion. Functional
orthopedic appliances affect the facial skeletal complex of children, activate orthodontic force in teeth and alveolar areas,
create a more normal skeletal development, and achieve a clinically acceptable esthetic facial profile. These appliances are
effective only in growing children. Maxillary splint appliance is not used in this case.
The use of a maxillary splint with a high-pull extraoral traction assembly has been shown to be most effective in reducing
Class II skeletal dysplasias through a combination of dentoalveolar and basal bone changes. Force delivery to the maxillary
complex in Class II skeletal jaw disharmonies is through the teeth. The philosophy behind the use of the maxillary splint
is that if the force delivered to the upper jaw involved the use of all the upper teeth (and hard palate) rather than only the

https://t.me/DentalBooksWorld
74 Triumph’s Complete Review of Dentistry

maxillary first molars, as in conventional extraoral orthodontic therapy, the effect on the jaws would be more orthopedic
than orthodontic in nature. The advantages of the use of the maxillary splint in the younger patient with a severe Class
II malocclusion are that it reduces the vulnerability of the maxillary incisors to accidental fracture, while concomitantly
reducing the Class II dysplasia, thereby effectively shortening the later-stage multiband corrective time and procedures.
Further advantages of the maxillary splint described are ease of construction and clinical application, which makes it an
attractive appliance for use in dental clinics or institutions in which patient volume, infrequent visits, and economic factors
are major considerations. This preliminary report on the philosophy of treatment procedure and description of the appliance
design is to be followed by a further cephalometric and clinical evaluation of results achieved with its use.
5. Answer: D (Ref. Orthodontics: Principles and Practice, By Basavaraj Subhashchandra Phulari, 2011, page no. 13)
It was introduced by Andrew.
6. Answer: B (Ref. Orthodontics: Current Principles and Techniques (e-book), By Lee W. Graber, Robert L. Vanarsdall,
Katherine W. L. Vig, 2016, page no. 495)
7. Answer: A (Ref. Textbook of Orthodontics (e-book), By Sridhar Premkumar, 2015, page no. 435)
Removable distalizing Cetlin appliance
The Cetlin appliance is used to distalize the upper first molars. This is achieved with 5 mm helical springs centered with the
upper first molars. The springs are made of 0.028 spring wire. An anterior bite plane is added to dislodge the occlusion. A
lateral to lateral labial bow with acrylic added and Adam’s clasp make up the retention.
Elements: Distal spring 0.7 mm spring hard, modified labial bow 17 × 24 rectangular stainless steel wire
Function: Distal movement of the last molars using a headgear
Description: Simple plate to move the last molars distally with fixed bands for extraoral traction
Shammy

The Shammy appliance is used to distalize the upper first molars. This is achieved with 3 mm helical springs centered with
the upper first molars. The springs are made of 0.022 × 0.028 wire. An anterior bite plane is added to dislodge the occlusion.
A lateral to lateral labial bow with acrylic added and Adam’s clasp make up the retention.
8. Answer: C (Ref. Textbook of Orthodontics (e-book), By Sridhar Premkumar, 2015, page no. 485)
One of the simplest fixed appliances for anterior cross bite purpose is a maxillary lingual arch with finger springs (sometimes
referred to as whip springs). This appliance is indicated for a very young child or preadolescent with whom compliance
problems are anticipated. It consists of a 36 mil maxillary lingual arch to which 22 mil springs are soldered. The springs are
usually soldered on the opposite side of the arch from the tooth to be corrected, in order to increase the length of the spring
and are most effective if they are approximately 15 mm long. This length provides exceptional flexibility and range, but
occasionally a spur is needed to serve as a guide wire to keep the wires from slipping over the incisal edge of the incisors.
9. Answer: D (Ref. Orthodontics: Principles and Practice, By Basavaraj Subhashchandra Phulari, 2011, page no. 433)
10. Answer: C (Ref. Contemporary Orthodontics, By William R. Proffit and Henry W. Fields, 2000, page no. 306)
11. Answer: C (Ref. Orthodontics: Current Principles and Techniques (e-book), By Lee W. Graber, Robert L. Vanarsdall,
Katherine W. L. Vig, 2016, page no. 431)
12. Answer: B (Ref. Orthodontics: Current Principles and Techniques (e-book), By Lee W. Graber, Robert L. Vanarsdall,
Katherine W. L. Vig, 2016, page no. 997)
13. Answer: D (Ref. Esthetics and Biomechanics in Orthodontics (e-book), By Ravindra Nanda, 2012, page no. 153)

Functional Indication
Regulator
Functional • Angle class I and crowding, especially when the apical base is underdeveloped
regulator type 1 • Angle class II/1 (mandibular retrognathism) with normal overbite or open bite or distortion of the
mandible (vertical growth type)
Functional • Angle class I and overbite, especially when the apical base is underdeveloped
regulator type 2 • Angle class II/2 (mandibular retrognathism) with overbite or vertical overlap, especially when the
apical base is underdeveloped
Functional • Angle class III: mandibular prognathism, maxillary retrognathism, and combinations
regulator type 3
Functional • Historical (no longer used)
regulator type 4 • Used in the past in vertical growth type with anterior open bite

https://t.me/DentalBooksWorld
Chapter 1 • Orthodontics 75

14. Answer: C (Ref. Orthodontics, By Kumar, 2008, page no. 386)


15. Answer: B (Ref. Orthodontics: Principles and Practice, By Basavaraj Subhashchandra Phulari, 2011, page no. 18)
16. Answer: B (Ref. Contemporary Orthodontics, By William R. Proffit and Henry W. Fields, 2000, page no. 491)
Ideal Spring – Continuous force
Removable functional appliances – Interrupted
Removable appliances – Intermittent
17. Answer: C
The Delta clasp is a derivation of the Adam’s clasp. Its name comes from the shape of its retentive portion. It is commonly
used with the Clark Twin block because, with this appliance, occlusal interferences are not an issue and an extremely retentive
clasp is needed. Like the Adam’s clasp, the tooth to be clasped must be erupted enough to expose the buccal undercuts.
Delta clasps are constructed from a 0.028 wire. They were first introduced in the United States by Dr. William Clarke,
inventor of the Twin Block Appliance. Delta clasps function much like Adam’s clasps, but they utilize a helix on the mesial
and distal undercuts for retention.
18. Answer: D
This is a fixed appliance that moves the upper molars back and the lower arch forward. The appliance is usually placed when
a patient is wearing braces and may be farther along in treatment. This addition to the braces is designed to correct the
overbite.
19. Answer: C
20. Answer: B
Roberts retractors are a type of removable functional orthodontic appliance. The special part of this appliance is the labial
arch which is a flexible bow in tubing to allow for the adjustment/retraction of central and lateral incisors. The activation is
enhanced by the transversal mesial tags of the labial arch through the placement of small tubes of corresponding size which
are pulled over the wire. This way the thinner active part of the labial arch, with additional loops in the U-loops, can serve
specifically as an active element.
21. Answer: A
Function – Retention for first molar if second molar is missing.
22. Answer: D (Ref. Textbook of Orthodontics (e-book), By Sridhar Premkumar, 2015, page no. 511)
23. Answer: D (Ref. Orthodontics: Current Principles and Techniques (e-book), By Lee W. Graber, Robert L. Vanarsdall,
Katherine W. L. Vig, 2016, page no. 997)
24. Answer: A (Ref. Contemporary Orthodontics, By William R. Proffit, Henry W. Fields, David M. Sarver, 2007)
25. Answer: A (Ref. Orthodontics: Current Principles and Techniques (e-book), By Lee W. Graber, Robert L. Vanarsdall,
Katherine W. L. Vig, 2016, page no. 997)
26. Answer: A (Ref. Textbook of Orthodontics, By Gurkeerat Singh, 2015, page no. 517)
27. Answer: B (Ref. Textbook of Orthodontics (e-book), By Sridhar Premkumar, 2015, page no. 356)
28. Answer: B
29. Answer: C (Ref. Orthodontics, By Kumar, 2008, page no. 361)
Contraindications of an activator
• Class I crowding, due to tooth size jaw discrepancy
• Increased lower facial height
• Extreme vertical mandibular growth
• Severely proclines lower incisors
• Nasal stenosis
• Nongrowing individuals
30. Answer: A (Ref. Twin Block Functional Therapy, By William Clark, William J. Clark, 2014, page no. 27)

SPACE MAINTAINERS AND SERIAL EXTRACTION


1. Answer: A
2. Answer: D
3. Answer: B (Ref. Orthodontics: Principles and Practice – Page xviii, By Basavaraj Subhashchandra Phulari, 2011, page no. xviii)
• Wilkinson’s procedure involves the extraction of first permanent molars during the mixed dentition period of 8½ to 9½
years. The concept behind this extraction procedure is that permanent first molar is prone to caries and following the
extraction of these teeth the incidence of dental caries can be decreased.
• Also second permanent molar will assume a normal relationship allowing enough space for the third molars to erupt.

https://t.me/DentalBooksWorld
76 Triumph’s Complete Review of Dentistry

4. Answer: A
5. Answer: C
6. Answer: B
7. Answer: D
8. Answer: C
9. Answer: D
10. Answer: D
• Distal shoe space maintainer is a cantilever type of space maintainer. It is also known as intraalveolar appliance or
eruption guiding appliance.
• It is indicated in cases where there is early loss of primary second molar prior to the eruption of permanent first molar.
• This appliance holds greater control of the path of eruption of permanent first molar.
11. Answer: D
12. Answer: B
13. Answer: C
• As a rule of thumb in preventive orthodontics, “a well restored primary tooth acts as a best space maintainer for the
upcoming permanent dentition.”
• Therefore, in order to maintain the integrity of dental arches, carious tooth should be timely managed and properly
restored.
14. Answer: C
15. Answer: B
• Adequate amount of arch expansion is brought about by the sutural expansion of the palatal bone. This active type of
expansion increases the width of the maxillary arch.
16. Answer: C
• Posterior teeth usually have the tendency to shift mesially when there is no proximal contact.
• When the first primary molar is missed before the eruption of the first permanent molar, strong eruptive forces of
permanent molar will cause the mesial tipping of primary second molar into the space. This leads to the arch discrepancy
which can be prevented by use of appropriate space maintainers.
17. Answer: A
Indications for serial extraction:
• Class I molar relationship bilaterally with normal neuromuscular balance
• Tooth size arch length discrepancy should be more than 10 mm in each arch
• Dental and skeletal midline should coincide with each other
• There should be no open bite or close bite
18. Answer: A
19. Answer: B
20. Answer: A
Various regimens of serial extraction are as follows:
Methods Extraction regimen
Dewel’s method CD4
Tweed’s method D4C
Nance’s method D4C
21. Answer: C
22. Answer: B
23. Answer: B
24. Answer: D
25. Answer: B
• Mayne’s appliance is the modification of band and loop space maintainer.
• It consists of only a lingual guiding wire, with no buccal wire.
26. Answer: D
• Bunon (1743) put forward the concept of serial extraction.
• Kjellgren (1929) introduced the term serial extraction.

https://t.me/DentalBooksWorld
Chapter 1 • Orthodontics 77

• Hayes Nance (1940) popularized the serial extraction technique. He is called the “father of serial extraction” philosophy
in the United States.
• Rudolph Holtz (1970) gave the term, “active supervision of teeth by extraction.”
27. Answer: A
• Nance palate arch is a bilateral, nonfunctional appliance mainly indicated when there is a bilateral premature loss of
primary molars and canines in the upper arch during mixed dentition period.
28. Answer: A
• The term “Timely extraction” was given by Stemm.
• This is equal to serial extraction where there is a sequential removal of primary teeth without the removal of any
permanent teeth.
29. Answer: A
• The growth of maxilla and mandible jaws are completed in the following directions:
1. Transverse (Width)
2. Sagittal (Depth)
3. Vertical (Height)
• Therefore, the growth in the transverse direction will start first and complete first. So it is of prime importance to prevent,
intercept, and correct the problem in transverse direction first which requires an active tooth movement.
30. Answer: C
• Occlusal adjustment of teeth through judicious grinding is included in preventive orthodontics.
• Functional occlusal pre-maturities are commonly found in primary dentition. So after the eruption of primary teeth to
full occlusal contact, they should be reviewed for any occlusal prematurity and should be adjusted accordingly.

SURGICAL ORTHODONTICS
1. Answer: B (Ref. Textbook of Orthodontics (e-book), By Sridhar Premkumar, 2015, page no. 736)
2. Answer: B (Ref. Textbook of Oral and Maxillofacial Surgery, By Balaji, 2009, page no. 484)
3. Answer: C (Ref. Peterson’s Principles of Oral and Maxillofacial Surgery, By Michael Miloro, 2012, page no. 1362)
4. Answer: A (Ref. Atlas of Oral and Maxillofacial Surgery, By Deepak Kademani and Paul Tiwana, 2015, page no. 324)
5. Answer: D (Ref. Textbook of Oral and Maxillofacial Surgery, By Neelima Anil Malik, 2012, page no. 313)
6. Answer: C (Ref. Oral and Maxillofacial Surgery (e-book): 3-Volume Set, By Raymond J. Fonseca, 2017, page no. 616)
7. Answer: B (Ref. Distraction Osteogenesis of the Facial Skeleton, By William H. Bell and César A. Guerrero, 2007,
page no. 234)
8. Answer: D (Ref. Textbook of Orthodontics, By Gurkeerat Singh, 2015, page no. 303)
9. Answer: C (Ref. Atlas of Oral and Maxillofacial Surgery (e-book), By Deepak Kademani and Paul Tiwana, 2015,
page no. 336)
10. Answer: C (Ref. Oral and Maxillofacial Surgery (e-book): 3-Volume Set, By Raymond J. Fonseca, 2017, page no. 171)
11. Answer: A (Ref. Plastic Surgery – Aesthetic, By Peter C. Neligan, Richard J. Warren, Allen Van Beek, 2012, page no. 181)
12. Answer: D (Ref. Peterson’s Principles of Oral and Maxillofacial Surgery, By Michael Miloro, G. E. Ghali, Peter Larsen, 2004)
13. Answer: D (Ref. Oral and Maxillofacial Surgery (e-book): 3-Volume Set, By Raymond J. Fonseca, 2017, page no. 96)
14. Answer: B
15. Answer: A (Ref. Surgical Correction of Facial Deformities, By Varghese Mani, 2010, page no. 79)
16. Answer: D (Ref. Surgical Correction of Facial Deformities, By Varghese Mani, 2010, page no. 19)
17. Answer: C (Ref. Textbook of Orthodontics (e-book), By Sridhar Premkumar, 2015, page no. 581)
18. Answer: B (Ref. Evidence-Based Implant Dentistry, By Oreste Iocca, 2016, page no. 178)
19. Answer: D (Ref. Cleft Lip and Palate: Diagnosis and Management, By Samuel Berkowitz, 2013, page no. 323)
20. Answer: D (Ref. Craniofacial and Dental Developmental Defects: Diagnosis and Management, By J Timothy Wright, 2015)
21. Answer: A (Ref. Orthodontics, By Kumar, 2008, page no. 530)

https://t.me/DentalBooksWorld
2 Pedodontics

SYNOPSIS

CHILD PSYCHOLOGY AND BEHAVIOR MANAGEMENT


Definition
It is the science or study of child’s mind and how it functions. It is also the science that deals with the mental power or an
interaction between the conscious and subconscious elements in a child.

Theories of Child Psychology


1. Psychodynamic theory
Theory Author
Psychosexual theory or psychoanalytical theory Sigmund Freud (1905)
Psychosocial theory or model of personality development Erik Erikson (1963)
Cognitive theory Jean Piaget (1952)
2. Theories of learning and development of behavior
Theory Author
Hierarchy of needs Maslow (1954)
Social learning theory Bandura (1963)
Classical conditioning Pavlov (1927)
Operant conditioning Skinner (1938)

Psychic Model/Psychic Triad


1. Id:
• It is the basic structure of the personality, which serves as a reservoir of instincts.
• It acts as a reservoir of mental representative.
• It is a pleasure principle.
2. Ego:
• It develops out of id in the 2nd and 6th month of life when an infant begins to distinguish between itself and the outside world.
• It is a mediation between id and superego.
• Ego is governed by reality principle.
• It is concerned with memory and judgment.
3. Superego:
• It is the prohibition learned from environment (more from parents and authorities).
• It acts as a censor of acceptability of thoughts, feelings, and behavior.
• It is the internalized control which produces the feeling of shame and guilt.

https://t.me/DentalBooksWorld
Chapter 2 • Pedodontics 79

Ego Defense Mechanisms


• Displacement
• Projection
• Reaction formation
• Regression
• Repression
• Rationalization
• Denial
• Identification

Sigmund Freud’s Stages of Development


1. Oral stage:
• In infants, the oral cavity is the site for identifying needs.
• Oral cavity serves as an erogenous zone.
• Age: Birth to 18 months.
• If child’s needs are not adequately met in this stage, the following traits develop:
–– Excessive optimism
–– Narcissism
–– Pessimism
–– Demandingness
–– Envy
–– Jealousy
2. Anal stage:
• Maturation of neuromuscular control occurs during this stage.
• Age: 18 months to 3–4 years.
• Development of personal autonomy and independence.
• Anal eroticism and defenses against it result in fixation on anal function.
3. Urethral stage:
• It is a transition between the anal stage and the phallic stage.
4. Phallic stage:
• This stage begins during the third year of life till the fifth year.
• It is characterized by:
–– Oedipus complex
–– Castration anxiety
–– Penis envy
–– Electra complex
5. Latency stage:
• Age: 6–12 years
• Maturation of ego takes place
• Greater degree of control over instinctual impulses
• Child starts adapting to adverse environment.

Frankel’s Behavior Rating Scale


Definitely negative (− −) Refuses treatment, cried forcefully, associated with fear
Negative (−) Reluctant to accept treatment and displays evidence of slight negativism

https://t.me/DentalBooksWorld
80 Triumph's Complete Review of Dentistry

Positive (+) Accepts treatment but during treatment may become uncooperative.
He had a bad experience
Definitely positive (++) Unique behavior, looks forward to and understands the importance of good
preventive care.

Wright’s Classification of Child Behavior


Cooperative (Positive behavior) 1. Cooperative behavior
2. Lacking cooperative ability
3. Potentially cooperative
Uncooperative (Negative behavior) 1. Hysterical/uncontrolled
2. Defiant/obstinate
3. Timid/shy
4. Stoic
5. Whining type
6. Tense cooperative

Types of Cry
Obstinate cry Loud high pitched sound
Characterized by siren-like wail
Represents child’s external response to anxiety
Accompanied by small whimper
Hurt cry Single tear running from the corner of the eye and runs along the child’s cheeks without creating
any resistance toward dental treatment
Frightened cry Torrents of tears
Breath-catching sobs
Child is overwhelmed by the treatment situation
Compensatory cry Not a cry, sound is slow and monotonous
Sort of coping mechanism to unpleasant auditory stimuli

Types of Fear
Innate fear Without stimuli or previous experience
Fear of unknown or anxiety
Subjective fear Fears transmitted (suggested) to the individual by family, friends etc.
Objective fear Fears due to own experience and direct stimulation of sense organs

Operant Conditioning by Skinner in 1938


Individual response is changed by reinforcement or extinction of previous responses.
1. Positive reinforcement: Occurs if a pleasant result follows the response, e.g., Child rewarded for good behavior following
dental treatment.
2. Negative reinforcement: Removal of unpleasant stimulus following a response.
3. Omission: Removal of pleasant response after a particular response.
4. Punishment: Introduction of an aversive stimulus into a situation to decrease the undesirable behavior.

https://t.me/DentalBooksWorld
Chapter 2 • Pedodontics 81

Principles Involved in Conditioning Process


• Acquisition
• Generalization
• Extinction
• Discrimination

Tell–Show–Do (TSD) Addleson (1959)


Modeling Bandura (1969)
Desensitization Joseph Wolpe (1975)
Hand–Over–Mouth technique Jordan

HOME is the Last Resort in Behavior Management


Indications of HOME:
• 3–6 years of age
• A child who can understand simple verbal commands.
• Healthy child displaying uncontrollable behavior
Contraindications of HOME:
• Child under 3 years of age
• Handicapped/immature/frightened child
• Physical/mental/emotionally handicap

For pediatric dentists, euphemisms or word substitutes are like a second language.
Words Substitutes
Rubber dam Raincoat for tooth
Rubber dam clamp Button
Sealant Tooth paint
Topical fluoride gel Cavity fighter
Air syringe Wind gun
Water syringe Water gun
Alginate Pudding
Suction Vacuum cleaner
High speed hand piece Whistle
Low speed hand piece Motor cycle

Intelligent Quotient was measured by Alfred Binet by examining the memory, spatial relations, and reasoning.
IQ = Mental age/Chronological age × 100
Mental IQ range (Stanford– IQ range (American Training/treatment
retardation Binet test) association of mental
Severity deficiency)
Mild 55–69 52–68 Can be educated in special cases to gain
elementary school level academic skills
Moderate 40–54 36–51 Trainable child
Severe 25–39 20–35 Child with limited trainability
Profound <24 <19 Not trainable

https://t.me/DentalBooksWorld
82 Triumph's Complete Review of Dentistry

EARLY CHILDHOOD CARIES


Terminology Author and Year
Nursing caries Winter et al., 1966
Nursing bottle mouth Kroll et al., 1967
Nursing bottle syndrome Shelton et al., 1977
Bottle propping caries
Labial caries
Baby bottle mouth Croll, 1984
Nursing mouth decay
Night bottle syndrome Diley et al., 1980
Baby bottle caries
Nursing mouth
Nursing bottle caries Tsamtsouris, 1986
Tooth cleaning neglect Moss, 1996
Rampant infant and early childhood dental decay Horowitz, 1998
(RIECDD)
Early childhood caries Davies, 1998

Definition (AAPD)
• The disease of early childhood caries (ECC) is the presence of one or more decayed (noncavitated or cavitated lesions),
missing (due to caries), or filled tooth surfaces in any primary tooth in a child of 71 months of age or younger.
• In a child younger than 3 years of age, any sign of smooth-surface caries is indicative of Severe Early Childhood Caries
(S-ECC).
• Filled score of >4 (age 3), >5 (age 4), >6 (age 5) surfaces constitutes S-ECC.

Classification of ECC
Type I ECC • Carious lesion involving the molars and incisors
(mild–moderate) • Seen in 2–5 years of age
Type II ECC • Labiolingual carious lesion affecting the maxillary incisors with or without molar caries depending
(moderate–severe) on age
• Seen soon after the first tooth erupts
• Unaffected mandibular incisors
Type III ECC • Carious lesion involving almost all the teeth, including mandibular incisors
(severe) • Usually seen in 3–5 years of age
• Rampant in nature and involves immune tooth surfaces

Etiology of ECC
• Bovine milk, milk formulas, and human milk are implicated in the etiology of ECC because of their lactose contents (Bovine
milk – 4.5%, breast milk – 7.2%, milk powder – 7%)
• Pathogenic microorganism: Streptococcus mutans is the principle organism which colonizes the tooth after it erupts into
the oral cavity.
• S. mutans is more commonly evident in rapid and smooth surface caries and less common in pit and fissure caries.
• It is seen that a child’s infection is nine times greater when maternal salivary count of S. mutans is greater than 100,000
colony forming units per ml.
• Carbohydrates are utilized by microorganism to form dextrans.
• Time is an important factor that determines caries activity.

https://t.me/DentalBooksWorld
Chapter 2 • Pedodontics 83

Clinical Features
• Thin enamel in the primary teeth is one of the reasons for early spread of lesions.
• The intraoral decay pattern of nursing caries is characteristic and pathognomonic of the condition. The sequence is as
follows.
1. Maxillary central incisors: Facial, Lingual, Mesial, and Distal.
2. Maxillary lateral incisors: Facial, Lingual, Mesial, and Distal.
3. Maxillary first molars: Facial, Lingual and Occlusal, Proximal.
4. Maxillary canine and second molars: Facial, Lingual, and Proximal surfaces.
5. Mandibular molars: At later stage.
Mandibular anterior teeth are usually spared because of:
• Protection by the tongue.
• Cleansing action of saliva due to presence of the orifice of the duct of the sublingual glands very close to lower incisors.

Rampant Caries
• Acute widespread caries with early pulpal involvement of teeth which are usually immune to decay.
• The term rampant caries applies to a mouth having 10 or more lesions per year.
• Seen at all ages, including adolescence.
• Affects both the primary and permanent dentition.
• Surfaces considered immune to decay are involved.
• Thus mandibular incisors are also involved.

Window of Infectivity
• Designated by Caufield (1993).
• He monitored the oral cavity levels from birth to 5 years of age.
• First window of infectivity is the window period in deciduous teeth, the MS is established, i.e., 7–31 months of age.
• Second window of infectivity (Klock & Kroshke, 1977) is present in permanent dentition between 6 and 12 years of age.

Management of ECC and Rampant Caries


First visit:
• All lesions should be excavated and restored with ZOE
• Abscess drainage
• Radiographs and its evaluation
• Diet charts to be given to record diet for 1 week
• Parent counseling
• Topical fluoride application
Second visit:
• Scheduled 1 week after first visit
• Analysis of diet chart and explanation of disease process of child’s teeth
• Isolate sugar factors from diet chart and sugar control (Diet instructions)
• Caries activity tests
Third visit:
• Restoring all grossly decayed teeth
• Endodontic treatment
• Extraction of unrestorable teeth
• Crowns
• Review and recall after every 3 months

https://t.me/DentalBooksWorld
84 Triumph's Complete Review of Dentistry

MOLAR–INCISOR HYPOMINERALIZATION (MIH)


Definition
Recently, Weerheijm et al. in 2001 suggested the term Molar Incisor Hypomineralization (MIH) and defined it as
“hypomineralization of systemic origin of one to four permanent first molars frequently associated with affected incisors”

Other terminologies of MIH


• Hypomineralized permanent first molars
• Non-fluoride hypomineralization
• Opaque spots
• Internal enamel hypoplasia
• Idiopathic enamel hypomineralization
• Nonendemic mottling of enamel
• Cheese molars
• Nonfluoride enamel opacities

Etiology
• Enamel formation is a sensitive process, which can be divided into several stages. A disturbance occurring during the
maturation phase will be clinically visible as enamel opacity, which suggests that in the case of MIH the ameloblasts are
affected in the (early) maturation stage.
• Ameloblasts are irreversibly damaged:
–– Clinically these appear as yellow or yellow/brown opacities.
–– These opacities are more porous.
• Ameloblasts have the potential to recover after the disturbance:
–– These defects appear creamy yellow or whitish cream demarcated opacities.
• Associations have been made between the presence of polychlorinated dibenzo-p-dioxins (PCDDs) in breast milk and
enamel hypomineralization in both clinical and laboratory studies.
• Children with poor health during the first 3 years of life are most likely affected by MIH.

Classification
• Mild MIH – when 30% of affected teeth are present
• Moderate MIH – 30–40%
• Severe MIH – 50% or more

Clinical Features
• MIH is a hypomineralized defect of the first permanent molars, frequently associated with affected incisors.
• Primary teeth is not usually affected.
• In some cases, apart from defects in the first permanent molars, opacities may be found in the upper and sometimes the
lower incisors.
• The risk of defects to the upper incisors appears to increase when more first permanent molars have been affected.
• The defects of incisors are usually without loss of enamel substance.
• Clinically, the hypomineralized enamel can be soft, porous, and look like discolored chalk or old Dutch cheese. Hence the
condition has been termed “cheesy molar”.
• The enamel defects can vary from white to yellow or brownish but they always show a sharp demarcation between the
affected and sound enamel.
• The porous, brittle enamel can easily chip off under the masticatory forces. Sometimes, the loss of enamel (posteruptive
enamel breakdown) can occur so rapidly after eruption that it seems as if the enamel was not formed initially.

https://t.me/DentalBooksWorld
Chapter 2 • Pedodontics 85

• After occurrence of the posteruptive enamel breakdown, the clinical pictures can resemble hypoplasia. In hypoplasia,
however, the borders to the normal enamel are smooth, whilst in posteruptive enamel breakdown the borders to the
normal enamel are irregular.
• MIH can sometimes be confused with fluorosis. It can be differentiated from fluorosis as its opacities are demarcated,
unlike the diffuse opacities that are typical of fluorosis and by the structure of the enamel
• Fluorosis is caries resistant and MIH is caries prone.

Diagnostic Criteria
The following diagnostic criteria and clinical appearance of the defects have been agreed upon (By European Academy of
Pediatric Dentistry):
1. P
 ermanent first molars and incisors. One to all four permanent first molars (FPM) shows hypomineralization of the
enamel. Simultaneously, the permanent incisors can be affected. To diagnose MIH, at least one FPM has to be affected.
Defects can also be seen in second primary molars, incisors, and the tip of the canines.
2. D
 emarcated opacities. The affected teeth show clearly demarcated opacities at the occlusal and buccal part of the crown.
The defects vary in color and size. The color can be white, creamy, or yellow to brownish. It is recommended that defects
less than 1 mm are not to be reported.
3. E
 namel disintegration. The degree of porosity of the hypomineralized opaque areas varies. Severely affected enamel
subjected to masticatory forces soon breaks down, leading to unprotected dentine and rapid caries development.
4. A
 typical restorations. FPM and incisors with restorations revealing similar extensions as MIH are recommended to be
judged as affected.
5. T
 ooth sensitivity. The affected teeth may be reported by frequent as sensitive, ranging from a mild response to external
stimuli to spontaneous hypersensitivity.
These teeth are usually difficult to anaesthetize.
6. E
 xtracted teeth. Extracted teeth can be defined as having MIH only in cases where there are notes in the records or
demarcated opacities on the other FPM.
7. R
 ecording the severity of the defects. Severity should be recorded as mild or severe in order to help the clinician. In mild
cases there are demarcated enamel opacities without enamel breakdown.
In severe cases there are demarcated enamel opacities with breakdown, caries, persistent/spontaneous hypersensitivity
affecting function.

Treatment Approaches
Prevention
• Toothpaste with a fluoride level of at least 1,000 ppm F should be recommended.
• Recently, Casein Phosphopeptide–Amorphous Calcium Phosphate (CPP-ACP), which provides a super saturated
environment of calcium and phosphate on enamel surface, has been shown to enhance remineralization.
• For patients with spontaneous hypersensitivity, professional application of fluoride varnish (e.g., Duraphat 22,600 ppm F)
and possibly 0.4% stannous fluoride gel may be helpful.
• Microabrasion, bleach, and sealant for anterior teeth: bleaching with carbamide peroxide and microabrasion with 18%
hydrochloric acid or 37.5% phosphoric acid and abrasive paste.
Cavity design
• Two empirical approaches to where the margins of the restoration should extend have been proposed:
a) Removal of all defective enamel is reached and
b) Removal of only the porous enamel, until resistance to the bur or to probe is felt.
Glass ionomer restorations
• This modality of treatment is important in early posteruptive stages because it can be used as an intermediate treatment in
less-than-ideal conditions of moisture control.
• In late posteruptive stage it might help as a sublayer beneath composite restorations.

https://t.me/DentalBooksWorld
86 Triumph's Complete Review of Dentistry

Extractions
• This modality of treatment should be considered in late mixed dentition when radiographically the second premolar is in
the crypt of the second primary molar and the second permanent molar’s bifurcation starts to form.
• Too early extraction will result the second premolar drifting distally, inhibiting the second permanent molar’s eruption into
the FPM’s space.
• Late extraction has less chance for spontaneous closure, resulting in excess residual space between the second premolar and
second permanent molar, especially in the lower jaw.

PIT AND FISSURE SEALANTS


Thin plastic coatings placed on the occlusal surfaces of posterior teeth to form a mechanical barrier between tooth structure and
the oral environment.

Types of Fissures
Fissures extend from enamel toward dentinoenamel junction in different shapes.
Four different types of fissures based on the alphabetical description of shape.
They are I, K, U, and V types.
U and V = shallow and wide, self-cleansable, and are caries resistant.
I and K = fissures are susceptible to caries.
I shaped ones are narrow and deep constricted like bottle neck.
Has a narrow slit like opening and a large base toward the dentinoenamel junction

Types of Pit and Fissure Sealants


1. Based on curing method
2. Based on presence of filler
3. Based on color

1. Based on Curing Method


1st gen:
Polymerized with UV light of 350 nm.
Absorbs UV light excessively and prevents complete polymerization of sealant.
2nd gen:
Self-cured or chemically cured. Most of them were unfilled.
Can be transparent, opaque, or tinted.
Filled ones show increased wear and abrasion resistance than unfilled.

https://t.me/DentalBooksWorld
Chapter 2 • Pedodontics 87

3rd gen:
Visible light cured at 430–490 nm.
May be unfilled or filled.
4th gen:
With addition of fluoride for added benefit.
2. Based on Presence of Filler
Unfilled → better flow
Filled → strong and resistant
3. Based on Color
Tinted for easy identification
Clear → difficult to detect
Opaque → for easy identification
Pink → better fluoride release

Materials Used as Sealants


A. Cyanoacrylates:
• Discovered in late 1950s.
• Used as surgical adhesive and tooth sealants.
• In presence of traces of moisture, they polymerize rapidly to hard and brittle polymers on etched tooth surface.
• Mechanical durability is not satisfactory and they are not biodegradable.
• Hydrolysis to potentially toxic materials occurred, as initially methyl cyanoacrylate was used.
B. Polyurethanes:
• E.g., Epoxylite → contains 10% sodium monofluorophosphate with liquid polyurethane and utilizes citric acid as etchant.
C. Dimethacrylate:
• Methyl methacrylate is highly volatile and lacks penetration.
• Enamite → a new sealant utilizes MMA–PMMA system è initiated by butyl boron.
• Binds better and is less affected by immersion in water.
• BISGMA is a viscous amber liquid of low volatility diluted with MMA for use as sealant.
Ratio → 3:1
D. Glass ionomer:
• Developed by Mclean and Wilson
• Hydrophilic, good adhesion, biocompatible
• Has fluoride releasing property
• Used for fissure whose orifice exceeds 100 µm.

Procedure of Sealant
1. Polish the tooth surface
2. Isolate and dry the tooth surface
3. Acid etching
30–50% orthophosphoric acid is used in liquid form for 30–60 seconds using stable hairbrush
4. Rinse the tooth
5. Isolate and dry the tooth
6. Material application
7. Evaluate the sealant
8. Check occlusion
9. Retention and periodic maintenance

https://t.me/DentalBooksWorld
88 Triumph's Complete Review of Dentistry

Indications of Pit and Fissure Sealant


1. Children and young adults with high risk of caries
2. Teeth showing signs of incipient caries
3. Adult patients with potentially susceptible area like deep fissures
4. Young permanent teeth having deep retentive pits and fissures
Contraindications
1. Children and young adults having low risk for caries
2. Teeth that have self-cleansing pits and fissures
3. Stained occlusal pits and fissures
4. Cases in which isolation of tooth is difficult
5. Teeth showing resistance to caries 3–4 years after eruption

Preventive Resin Restoration


• Are a natural extension of use of occlusal sealants
• Conservative step to conventional “extension for prevention”
• Simonsen classified in 1978
Type A:
• Suspicious pits and fissures where caries removal is limited to enamel.
• LA not needed.
• A slow speed ½ or ¼ round bur is used to remove decalcified enamel.
• Sealant is placed.
Type B:
• Incipient lesion in dentin that is small and confined.
• No LA.
• Appropriate base should be placed in areas of dentin exposure.
• Then composite resin is placed.
• The remaining pit and fissure are covered with a sealant.
Type C:
• More extensive dentinal involvement and requires restorations with posterior composite material.
• Appropriate base is placed over dentin.
• Pits and fissures are covered with sealant.
• LA required.

TRAUMA
Children in the age group of 1–2½ years sustain injuries to the primary dentition most frequently. The children sustain injuries
to the permanent dentition most commonly during the age group of 8–11 years. In permanent dentition, Class II malocclusion
cases are more prone to fractures due to proclined upper incisors.

Ellis and Dewey Classification of Fracture of Teeth


Class 1 Fracture involving enamel
Class 2 Fracture involving enamel and dentin
Class 3 Fracture involving dental pulp
Class 4 Nonvital tooth
Class 5 Avulsion
Class 6 Root fracture with or without crown fracture

https://t.me/DentalBooksWorld
Chapter 2 • Pedodontics 89

Class 7 Displacement of tooth without fracture of crown or root


Class 8 Fracture of crown en masse
Class 9 Fracture of primary teeth

WHO Classification (Anderson – 1983)


873.60 Incomplete fracture (crack) of enamel without loss of tooth substance
873.60 Fracture involving enamel only
873.61 Fracture of crown involving enamel and dentin but not pulp
873.62 Fracture of crown involving enamel, dentin, and pulp
873.63 Fracture of root only involving dentin, cementum, and pulp
873.64 Fracture of root and crown involving enamel, cementum, dentin, but not the pulp
873.66 Fracture of both crown and root involving enamel, dentin, cementum, and pulp
873.67 Concussion, subluxation, extrusive luxation or partial avulsion or peripheral dislocation, lateral luxation
873.68 Intrusion or central dislocation
873.69 Complete avulsion or exarticulation

Classification of Injuries to Periodontal Tissues


1. Concussion:
• Injury to the supporting tissues without loosening or displacement but with marked reaction to percussion
2. Subluxation:
• Abnormal loosening but without displacement of tooth
• Sulcular/gingival bleeding is noted
3. Intrusion:
• Also known as central dislocation
• Displacement of the tooth into the alveolar bone accompanied by the fracture of the alveolar socket
4. Extrusion:
• Also known as peripheral dislocation
• Partial displacement of the tooth out of its socket
5. Ex-Articulation:
• Also known as complete avulsion
• Complete displacement of tooth out of its socket
6. Laceration:
• Shallow or deep wound in the mucosa caused by a sharp object
7. Contusion:
• Bruise produced by impact of blunt object
8. Abrasion:
• Superficial wound produced by rubbing or scrapping

Sequelae of Trauma to the Tooth


Internal hemorrhage Rupture of capillaries lead to escape of RBC and subsequent breakdown resulting in pigments
called biliverdin and bilirubin which discolored tooth.
Hyperemia Congestion of blood within pulp chamber leads to color change after several weeks.
Internal resorption The tooth appears pink as the pulp shows through the enamel
Pulpal necrosis Pulp undergoes autolysis and necrosis with deposition of blood pigments
Canal calcification Partial or complete obliteration of the pulp chamber and canal may occur. The crowns of teeth
may have a yellowish opaque color.

https://t.me/DentalBooksWorld
90 Triumph's Complete Review of Dentistry

Treatment Modalities of the Fracture of Permanent Teeth Involving Pulp


Treatment Indications
Pulp capping • Exposure of less than 1 mm
• Exposure not over 24 hours
• Minimal hemorrhage
• Open apex
Pulpotomy • Relatively large exposure
• Patient seen within 72 hours
• Infection confined to crown portion
• Moderate hemorrhage
• Open apex
Apexification • Nonvital permanent teeth with immature open apex
Apexogenesis • Vital permanent teeth with immature open apex
Pulpectomy • Degenerated pulp
• Pulp exposure greater than 72 hours

Management of Avulsed Teeth


The two main factors responsible for the success of reimplantation are:
1. Revascularization of the pulp
2. Maintain the integrity of PDL ligament and prevent root resorption
Extraoral Condition of Prognosis Treatment
time the apex
30 minutes Open Revascularization of the No need of pulp treatment
pulp and continued root
development are possible.
Close PDL healing is excellent No need of pulp treatment
30–60 Open Chances of revascularization Initiate apexification in the second appointment.
minutes is poor. The teeth should be soaked in Doxycycline (1 mg/20 ml
physiologic saline) prior to re-implantation.
This increases the chances of revascularization.
Close Root canal treatment has to The teeth should be soaked in Doxycycline (1 mg/20 ml
be initiated within 7–10 days physiologic saline) prior to re-implantation.
This increases the chances of revascularization.
More than Open and Soaking is ineffective as all Here the main aim is to prevent root resorption.
60 minutes close the cells might have died. Root canal treatment has to be initiated.
The teeth should be soaked in citric acid for 5 minutes followed
by a 5-minute soak in 2% stannous fluoride, followed by another
5 minutes in doxycycline, after which they are reimplanted.

Time Period for Splinting of Various Injuries


Injuries Duration of splinting
Avulsion 7–10 days
Extrusion 2–3 weeks
Luxation 2–8 weeks
Root fracture 12 weeks
Alveolar bone fracture with replanted teeth 3–4 weeks

https://t.me/DentalBooksWorld
Chapter 2 • Pedodontics 91

Storage Media for Avulsed Teeth


Viaspan (Best storage medium): Hank’s Balanced Salt Solution (HBSS):
The pH is 7.4 The pH is 7.2
320 mOsm 320 mOsm
Vital cells are present for 7 days 24 hours protection

PEDIATRIC ENDODONTICS
Direct pulp capping • Direct pulp capping is carried out when a healthy pulp is inadvertently exposed during
an operative procedure
• It is recommended for small mechanical or traumatic exposures (less than 1 mm)
which is surrounded by sound dentin
Indications of DPC • Small traumatic exposure (<1 mm)
• Light red bleeding from the exposure site that can be controlled by a cotton pellet
• Traumatic exposures with clean field, reported within 24 hours
Contraindications of DPC • Pain at night
• Spontaneous pain
• Excessive and uncontrolled bleeding
• Periapical or Intraradicular radiolucency
• Mobile tooth primary tooth
Direct pulp capping and primary DPC in primary tooth
tooth Ø
High cellular content of primary pulp
Ø
Undifferentiated mesenchymal cells
Ø
Odontoclasts
Ø
Internal resorption
Indirect pulp capping • The procedure involving a tooth with a deep carious lesion where carious dentin
removal is left incomplete, and the decay process is treated with a biocompatible
material for some time, in order to avoid pulp tissue exposure
Indications of IPC • Minimal pulpal inflammation, where complete removal of pulp would cause a pulp
exposure
Contraindications of IPC • Any signs of pulpal or periapical pathology
• Soft leathery dentin covering a very large area of the cavity
Few important points about IPC • The treated tooth is re-entered after 6–8 weeks
• The rate of reparative dentin deposition is average 1.4–1.5 microns/day
• The rate of reparative dentin formation decreases markedly after 48 days
Partial pulpotomy • Cvek’s pulpotomy
• When pulp exposure is about 1–3 mm (<3 mm)
• Mainly indicated in vital traumatically exposed young permanent teeth with
incompletely formed apex
• Placement of calcium hydroxide or MTA
Garcia Godoy pulpotomy One minute pulpotomy
Apexogenesis • Procedure involving treatment of vital pulp by capping or pulpotomy, in order to
permit continued growth of the root and closure of open apex
  Note:
–– Vital tooth
–– Immatured with open apex

https://t.me/DentalBooksWorld
92 Triumph's Complete Review of Dentistry

Apexification • Procedure that induces development of root apex of an immature nonvital tooth by
formation of osteocementum/bone-like tissue
Note:
–– Nonvital tooth
–– Immatured open apex
Evidence of root apical closure Described by Frank:
in Apexification • Continues closure of canal and apex to a normal configuration
• Dome-shaped apical closure but the canal remains with blunderbuss configuration
• No radiographic change, but a thin osteoid-like barrier provides a definite stop at the
apex
• Radiographic evidence of a barrier short of apex
Revascularization of pulp • Revascularization is defined “as the procedure to re-establish the vitality in a nonvital
tooth to allow repair and regeneration of tissues”
• Revascularization, per se, is not new. It was introduced by Ostby in 1961.
Indications:
–– Nonvital tooth
–– Immatured tooth with open apex
–– High regenerative capacity
Steps in revascularization Tooth is anesthetized and isolated
first appointment Access opening
Ø
Irrigation of root canal with 20 ml of NaOCl followed by 20 ml of 0.12–2% CHX
Ø
Dry the canal with paper points and place the TAP (Triple Antibiotic Paste)
Ø
Mixture of Ciprofloxacin, Metronidazole, and Minocycline paste as described by
Hoshino et al. is placed with a lentulo spiral instrument to a depth of 8 mm into the
canal (1:1:1, 20 µg/ml)
Ø
Seal access cavity with 4 mm Cavit and dismiss patient for 3–4 weeks
Steps in revascularization Tooth is anaesthetized (LA without vasoconstrictor)
second appointment Ø
Isolated and access opened
Ø
Removal of TAP then irrigate canal with 20 ml of NaOCl followed by 20 ml of 0.12–2%
CHX
Ø
Dry the canal with paper points and induce apical bleeding
Ø
Create apical bleeding by overinstrumenting (#15 or #20 K endo file, endo explorer) up
to 3 mm from CEJ
Ø
A small piece of CollaPlug may be inserted into the root canal system as absorbable
scaffold matrix to restrict the positioning of the MTA
Ø
Place 3–4 mm MTA (along with wet cotton ball over it) and reinforced glass ionomer as
temporary filling

https://t.me/DentalBooksWorld
Chapter 2 • Pedodontics 93

Important points on • A 12–18 month recall should be considered the earliest time point to conduct clinical
revascularization examination
• Blood clot is formed within 12–15 minutes
• The ratio of TAP is 1:1:1 (20 µg/ml each)
• The disadvantage of using minocycline in TAP is, it causes tooth discoloration (so TAP
is placed 2–3 mm below CEJ)
• Replaced by Cefaclor but it is as significant as minocycline
Why revascularization over • Continued root development with thicker dentinal walls and prevention of root
apexification? fracture
Permanent tooth • Immature apex
Vital tooth –– Calcium hydroxide pulpotomy
–– Apexogenesis
Permanent tooth • Matured tooth/closed apex
Vital tooth –– Pin-point exposure : DPC
–– Large exposure: Pulpotomy
Permanent tooth • Immature tooth with open apex
Nonvital tooth –– Apexification
–– Revascularization (Best)
Permanent tooth • Matured tooth/closed apex
Nonvital tooth –– Root canal treatment
Primary tooth • Deep Caries
Vital tooth –– Indirect pulp capping
–– DPC – Not indicated
Primary tooth • Large pulpal exposure or carious pulpal exposure
Vital tooth –– Formocresol pulpotomy
–– CaOH2 pulpotomy – Not indicated
Primary tooth • With two-thirds root length available or minimal bifurcation radiolucency
Nonvital tooth –– Pulpectomy (RCT)
Primary tooth • Vertical # or Luxation injuries
Nonvital tooth –– Extraction

GENETICS IN PEDIATRIC DENTISTRY


Phenotype
• The numerous features by which we recognize an organism form its phenotype.
• In humans, e.g., height, color of skin etc., constitute its phenotype.
Genotype
• It represents the genetic identity card of an organism.
• It is the material that is transmitted from one generation to the next.
• The expression of the phenotype is attributable not only to the genotype but to the environmental conditions as well.
• Environmental alterations of the phenotype do not reflect alterations in the genotype.

Basic Concepts in Genetics


• Codon is a set of three adjacent bases in DNA/RNA that specify an amino acid.
• Housekeeping genes encode proteins required for basic functions.
• Anti-oncogene (or tumor suppressor) is involved in controlling cellular growth.

https://t.me/DentalBooksWorld
94 Triumph's Complete Review of Dentistry

• Inactivation of antioncogene causes cancer.


• Telomeres are the only regions of a chromosome that comprise a single helical DNA present as hairpin-like structure.
• Telomeres may also serve as “mitotic clocks”, i.e., its length can indicate the number of divisions undergone by the cell.

Chromosomal Abnormalities
• Mutations as stated by Vries, 1901 is a term used for sudden genetic changes which were first noted by him in the plants,
Oenothera lamarckiana.
• The term mutation refers to a Genotype.
• The familiar term mutant refers to an unusual phenotype.
Germ line mutation: The change occurs during the DNA replication that precedes meiosis.
Somatic mutation: The change occurs during DNA replication that precedes mitosis (this is responsible for certain cancers).
Genetic Basis of Dental Disorders
• The X chromosome appears to mainly regulate enamel thickness.
• On the other hand Y chromosome seems to affect both enamel and dentin.
• Cephalometric analysis of a sample of 47, XXY males indicates pronounced facial prognathism in Klinefelter males,
especially in the mandible.
• It is suggested that the X chromosome may alter morphology of the cranial base by affecting growth at the synchondroses,
that is, cartilaginous joints and it also has a direct effect on mandibular shape.
• Human dental enamel amelogenin gene is located on both X and Y chromosomes although the gene on the X-chromosome
is predominant.
• The amelogenin gene has been localized to the distal portion of the short arm of the X chromosome and to the pericentromeric
region of Y chromosome.
Periodontal Problems
• In an infectious disease such as periodontal disease, the association between the HLA antigens and various forms of the
disease has been of interest with several studies reporting the incidence of Class I and II HLA antigens in patients with early
onset periodontal disease.

DNA Probes
The DNA library includes probes for
• A. Actinomycetemcomitans
• P. gingivalis
• B. intermedius
• C. rectus
• E. corrodens
• F. nucleatum
• T. denticola

Dental Caries Vaccine


• Recently a vaccine has been discovered, a protein called p1025.
• This protein tricks S. mutans, the main cause of decay to believe that there are no vacant sites on tooth for it to attach as it
mimics the protein of the bacterium and occupies all clocking points.
Few Facts to Remember
• Syndromes warranting a high resolution analysis of prophase or prometaphase chromosomes are called microdeletion
syndromes.
• When a chromosome suffers two terminal deletions followed by the fusion of broken ends, a ring chromosome is formed.
• Evidence from experimental studies suggests 50% genetic contribution to the development of caries.
• BRAX-1 gene is responsible for control of enamel growth.

https://t.me/DentalBooksWorld
Chapter 2 • Pedodontics 95

Genomic Imprinting
• The most clear example of genomic imprinting in humans comes from the study of two clinically distinct disorders – The
Angelman and Prader–Willi syndromes.
• In both the cases, chromosome 15 region q11–q13 is affected.
• In Angelman syndrome, deletion arises on maternally derived chromosome 15.
• In Prader-Willi syndrome, deletion arises on the paternally derived chromosome 15.

Prenatal Diagnosis of Genetic Disorders


• The cardiac activity of fetus is detectable at 7–8 weeks using ultrasound.
• Optimum time for amniocentesis is 16–18 weeks.
• Optimum time for chorionic villus sampling is 9–12 weeks.
• The fetal cells are present in maternal blood even as early as 33 days of gestation.
• Fetal skin biopsy: Definitive prenatal diagnosis requires that the histological appearance of the skin be pathognomonic at
20 weeks of gestation.

Gene Therapy
• Gene transfer in vivo can be done using intraperitoneal, intravenous, intra-arterial, intrahepatic, intramuscular, and
intratracheal route.
Viral delivery systems and their benefits:
Retroviruses • 100% transduction is possible
• Infection does not lead to cell lysis
Adenoviruses • Minor pathogens in humans
• Not associated with malignancies
• Can infect nondividing cells
Herpes S virus • Ideal for treating nervous systems
Vaccinia virus • Can infect all cell types

Latest Advances in Genetics


• Recombinant DNA technology involves isolation and manipulation of DNA to make chimeric molecules (e.g., molecules
containing both human and bacterial DNA sequences in a sequence-independent fashion)
• Endonucleases – Enzymes that cut DNA at specific DNA sequences within the molecule
• Exonucleases – Enzymes that digest from the ends of DNA molecules
Stem cell therapy:
1. Dental Pulp Stem Cells (DPSC):
• DPSC are the stem cells which have the ability to regenerate the dentin–pulp complex.
• DPSC like osteoblasts, express bone markers such as bone sialoprotein, alkaline phosphatase, type I collagen, and
osteocalcin.
2. Stem Cells from Human Exfoliated Primary Teeth (SHED):
• The SHED have been identified to form a part of highly proliferative, clonogenic cell capable of differentiating into
variety of cells types including neural cells, adipocytes, and odontoblasts.
• The SHEDs have the osteoinductive capacity in vivo but have failed to reconstitute a dentin–pulp complex.
• These DPSC and SHED are being utilized to a great extent in regenerative endodontics.
Valiant venture Ltd. offers a service called Clonaid to provide assistance to would-be parents willing to have child cloned from
one of them.

https://t.me/DentalBooksWorld
96 Triumph's Complete Review of Dentistry

QUICK FACTS

History of Pedodontics
• Gerauldy (1737) – Wrote about theories regarding tooth eruption and exfoliation
• Joseph Hurlock (1763) – Published 1st book on child dentistry
• Robert Bunon (1764) – “Father of Pedodontics” reiterated the importance of deciduous dentition
• Dr. Samuel D Harris (1926) – Father of Children’s Dentistry Organizations
• American Academy of Pedodontics (AAP) was found in 1947
• AAP was renamed the American Academy of Pediatric Dentistry in 1984
• Pedodontic treatment triangle was given by Wright, 1975
Pedodontics in India
• Dr. Rafiuddin Ahmed – Father of Dentistry in India
• Dr. BR Vacher – Father of Pedodontics in India
• Calcutta Dental College (1920) – 1st Dental College started by Dr. Rafiuddin Ahmed
• Government Dental College, Amritsar, starts Pedodontics as a specialty in 1950
• In 1988, the subject was given independent status by the Dental Council of India

Radiographic Examination of Pediatric Patients


• Size 0 is used for bitewing and periapical radiographs of small children.
• OPG is used in the evaluation of traumatic injuries as well as dental age estimation as it gives good information about the
eruption status of the children.
• Sialography is contraindicated in acute infection of the salivary glands.
• TACT (Tuned Aperture Computed Tomography) is used for the diagnosis of the external root resorption.

Radiation Protection
1. Before exposure:
• Filtration to remove low-energy radiation. Aluminum filtration of 1.5 mm at voltages below 70 kVp and 2.5 mm at
voltages above 70 kVp.
• Round or rectangular collimation to restrict the size of the beam.
2. During exposure:
• Intraoral – Use E or F speed films
• Extraoral – Use of intensifying screens
• Increase in focal spot to film distance
• Decrease of time of exposure by increasing voltage (kVp) and current (mA)
• Principle of radiation protection: ALARA – As Low As Reasonably Achievable, should be followed.
• Use of lead aprons, thyroid collar, and lead gloves
• Position and Distance Rule – If the lead barrier is not present the personnel should be standing at a distance of 6 feet
from the patient and in the area coming at 90–35 degrees angulation from the object

Child Appointments
• Morning appointments are preferable in a young patient because the child will be fresh and active.
• The length of the appointment should be less than 30 minutes.
• The concept of “Four-Handed-Dentistry” implies that the assistant’s hands are constantly employed in the treatment of the
child.

https://t.me/DentalBooksWorld
Chapter 2 • Pedodontics 97

Pedologic Anatomy and Growth


• Cardiovascular system starts functioning by the end of the third week.
• By the end of the 24th day, 3 pairs of branchial arches are present.
• By the end of the 5th week, 42–44 pairs of somites are formed.
• Fetal period begins at 9th week.
• Sucking reflex develops by 17–20th week.
• Grasp reflex is well developed by 28th week.
• During 36–38th week, the following changes are seen:
–– Hand forms one-third of the body length
–– Cranium to face ratio is 8:1
–– Length is 45–50 cm (30 inches)
–– Head circumference 35 cm

Reflexes Present at Birth


1. Moro reflex
• Any sudden movement of the neck initiates this reflex.
• The reflex consists of a rapid abduction and extension of the arms with the opening of hands.
• This reflex gives an importance to muscle tone.
• This reflex usually disappears in 2–3 months.
2. Startle reflex
• It is similar to Moro reflex, but it is initiated by a sudden noise or any other stimulus.
• In this reflex, the elbows are flexed and the hands remain closed.
3. Palmar/grasp reflex
• When the baby’s palm is stimulated, the hand closes. There is also a corresponding plantar reflex.
• Both normally disappear by 24 months.
• Clinical significance is that, an exceptionally strong grasp reflex may be found in the spastic form of cerebral palsy and
in kernicterus.
4. Babinski’s reflex
• Stroking of the lateral surface of the plantar surface of the foot from the heel to the toe results in flexion of the toe.
5. Parachute reflex
• This reflex is elicited by holding the child in ventral suspension and suddenly lowering him to the couch.
• The arms extend as a defensive reaction.
• In children with cerebral palsy, the reflex may be absent or abnormal.
• This reflex appears at about 6–9 months.
6. Landau reflex
• It is seen in vertical suspension, with the head, spine, and legs extended.
• It is normally present from 3 months and is difficult to elicit after 1 year.
• Absence of reflex occurs in hypotonia, hypertonia, or severe mental abnormality.
7. Rooting reflex
• Onset – 28 weeks IU
• Well established – 32–34 weeks
• Disappears – 3–4 months
8. Sucking reflex
• Onset – ~28 weeks IU
• Well established – 32–34 weeks
• Disappears – around 12 months

https://t.me/DentalBooksWorld
98 Triumph's Complete Review of Dentistry

9. Swallowing
• Begins around 12½ weeks of IU life
• Full swallowing and sucking is established by 32–36 weeks of IU life
10. Infantile swallow
• Until the primary molars erupt, infant swallows with the jaws separated and the tongue thrust forward using facial
muscles (Orbicularis oris and Buccinator).
• This is a nonconditional congenital reflex.
11. Gag reflex
• It is seen in 18½ weeks IU life.
• In the buccal cavity and pharynx, the ectoderm/endoderm zone in toward the posterior third of the tongue.
• Touching here elicits a gag reflex, a protective reflex.
12. Cry
• It is a nonconditioned reflex which accounts for its lack of individual character and it is of sporadic nature.
• It starts as early as 21–29 weeks IU life.

Important Features in Growth and Development


• End of first month enamel organs developed from dental lamina.
• The ratio between the calvarial and facial portion is 8:1 at birth whereas it is 2.5:1 in an adult.
• Skull bones in the neonate are 45 (due to incomplete ossification) and in the adult it is 22.
• The neurocranium consists of two parts: Desmocranium and Chondrocranium.
• Head: At birth circumference is around 35 cm (13.75).
• Mandible: The two halves of the mandible fuse into a single bone by the age of 1–2 years.
• Buccal pad of fat is also known as corpus adiposum or Bichat’s fat pad.
• Full swallowing and sucking is established around 32–36 weeks of IU life.
• Growth of eyes is completed by 5 years.
• Growth of brain is completed by 10 years.
• Growth of heart is completed by 20 years.
• “Growth spurt” is sudden increase in growth.
• From birth to 7 years of age, height increases by 2.5 times.
• From birth to 2 years of age, weight increases by 4.5 times.
• From 2–10 years of age, weight increases by 5 lb/year.

Important Features in Development of Dentition


• Eruption is derived from the Latin word “erumpere” – To break through.
• Teeth develop from the epithelial primary germ cell.
• In humans, odontogenic epithelium which is the analogue of the dentitions, can be identified in 28–30 day (ovulation age)
embryo.
• Mineralization commences in the deciduous dentition during the 14th IU week on average and begins with central incisors.
• Mineralization of the permanent teeth is usually initiated around the time of birth on average beginning with the permanent
first molar.
• The mean daily eruption velocity was seen to be 71 µm/day.
• Eruption of primary dentition usually begins in the 5th or 6th month of a child’s life.
• The first sign of root resorption is seen in the deciduous mandibular incisors and the first molars by the age of 4–5 years.
• Resorption of the deciduous incisors takes place more rapidly (lasting 1.5–2 years on average) than that of the canines and
molars (2.5–5.7 years).
• Gum pads are horse-shoe shaped in maxilla and U shaped in the mandible.

https://t.me/DentalBooksWorld
Chapter 2 • Pedodontics 99

• Average overjet in primary dentition is 1–2 mm with a normal range of 2–6 mm.
• The incisors may assume an edge-to-edge inter relationship by the age of 6 years.
• Tooth crowns pierce the bony alveolar crest when approximately two-thirds of root development is complete.
• The most common sequence of eruption of permanent lateral teeth in the maxilla is 4-3-5 and in the mandible is 3-4-5.

Periodontal Diseases in Children


• The gingivitis in primary dentition tends to remain confined to marginal gingiva.
• Two unique characteristics of the attached gingiva in children are interdental clefts and retrocuspid papilla.
• Saddle areas are present in primary dentition due to interdental spacing.
• The primary infection of herpetic gingivostomatitis occurs in a child under 5 years of age who has no contact with HSV-1
and therefore no neutralizing antibodies.
• The most common form of leukemia in children is acute lymphocytic leukemia.
• The most striking feature of juvenile periodontitis is lack of clinical inflammation despite presence of deep pockets.
• Premature exfoliation of primary teeth is seen in hypophosphatasia.
• Floating teeth is a feature of histiocytosis X.
• Prepubertal periodontitis may begin around the age of 4 years but may not be diagnosed until 7–9 years of age.

Hard and Soft Tissue Lesions


• Submucous cyst is a painless swelling caused due to obstruction of duct of mucous glands.
• Mucocele arises as a result of obstruction of minor salivary gland ducts as a result of trauma.
• Hemangioma is not a true tumor; it is a hamartoma.
• Sublingual dermoid is a type of congenital sequestration dermoid.
• Fordyce’s granules are developmental disturbances and not a disease.
• Herpes zoster causes unilateral pain, rash and oral vesicles followed by ulceration.
• Epulis is more common in females.
• Cherubism may regress as the child approaches puberty
• The mandible is more affected by tumors than maxilla.
• Most of the odontogenic tumors occurring in children are benign.
• Typical radiographic appearance of ameloblastoma is termed “soap bubble.”
• An odontoma is a tumor that is derived from mesenchymal elements of dental follicle and is most common during childhood.
• The characteristic feature of Ewing’s sarcoma is onion skin appearance.
• Starry sky appearance is a characteristic histological feature of Burkitt’s lymphoma.

Causes of Tooth Discoloration


Intrinsic stains
Generalized staining of single or complete dentition
White Fluorotic/Nonfluorotic
Red/brown Congenital porphyria
Opalescent teeth (Blue/brown) Dentinogenesis imperfecta
Green/blue Hyperbilirubinemia
Yellow/brown to dark yellow Amelogenesis imperfecta
Grey/Brown Nonvitality
Localized staining on one or several teeth
Grey/black Amalgam staining
Pink Internal resorption
White Developmental defect
Yellow/brown Developmental defect

https://t.me/DentalBooksWorld
100 Triumph's Complete Review of Dentistry

Chronological staining of dentition


Bright yellow Tetracycline
Yellow/grey brown Tetracycline
Extrinsic stains
Black Silver fluoride treatment, Ferrous sulfate
Brown Chromogenic bacteria
Yellow Bile pigments from GCF
Green Chromogenic bacteria

Effects of Different Fluoride Concentration (Plasma Peak) on Enamel Formation


Cell proliferation 1 µmol/L and above Cell inhibition
Cell death
3 µmol/L and above Delayed differentiation
Epithelial and mesenchymal interactions affected
0.2 µmol/L Cell vacuolation and matrix accumulation
26.5 µmol/L Slight elevation of existing mineral
53 µmol/L New matrix fails to mineralize
Transition Up to 159 µmol/L Accumulation of F with increasing dose in tissues
Maturation 10.6 µmol/L Increased protein content
Increase in magnesium content
Incomplete maturation and crystal growth inhibited

Slow Releasing Fluorides


Amalgam Release fluoride, decreased with time
Acrylic paste Release fluoride, but frequent application required
Composite resins Variable rate of fluoride release, short term
Fissure sealants Release for 7 days
Glass ionomer sealants Release up to 1 year

Various Fluoride Preparations


Type of fluoride F salt F ppm
Professional gel 2.72% APF 12,300
2.0% NaF 9,200
F varnish 5.0% NaF 26,000
Daily rinse 0.05% NaF 230
0.02% APF 200
Weekly rinse 0.2% NaF 910
Home gel 0.4% SnF2 970
1.1% NaF 5,000
F tablets 2.2% NaF 1,000
1.1% NaF 500
0.55% NaF 250
F drops 1.1% NaF 500
Dentifrice 0.22% NaF 1,000
0.76% MFP 1,000

https://t.me/DentalBooksWorld
Chapter 2 • Pedodontics 101

Laboratory Tests in Bleeding Disorders


Test Normal value Increased in Normal in Reduced in
Bleeding time 1–6 minutes Von Willebrand disease Hemophilia A and B
APTT 25–35 seconds Hemophilia A and
B, DIC, liver disease,
Vitamin K deficiency,
coumarins
PT 11–15 seconds DIC, liver disease, Hemophilia A and B
Vitamin K deficiency,
coumarins
Factor VIII 60–150% Liver disease Liver disease, Vitamin Hemophilia A, VwB
K deficiency disease, DIC
Factor IX 60–100% Hemophilia B
Factor VII 60–100% Liver disease, Vitamin K
deficiency, DIC
Factor V 60–100% Vitamin K deficiency Liver disease, DIC

Oral Cavity–Mirror of Child’s Body


Disease Characteristic feature Oral manifestation
Mineral deficiency disorders
Calcium Carpopedal spasm Defective calcification of teeth
Copper Steely–kinky hair syndrome Pale mucosa
Zinc Acrodermatitis enteropathica Stomatitis, glossitis
Disorders of protein metabolism
Amyloidosis Macroglossia
Porphyria Excretion of red urine Discolored teeth with red fluorescence
under UV light
Disorders of carbohydrate metabolism
Hurler’s syndrome Progressive corneal clouding Short and broad mandible, spacing of
teeth
Lipoid proteinosis Infants unable to cry at birth Thick lips and tongue bound to floor of
mouth
Disorders of lipid metabolism
Histiocytosis X Punched out bone lesion in skull, Loose teeth, halitosis, bad taste (R/F:
exophthalmos, Diabetes insipidus Flying tooth)
Avitaminoses
Hypophosphatasia Severe rickets Early loss of deciduous teeth
(R/F: Spotty or streaky ossification of
long bones)
Vitamin D deficient rickets Cessation of calcification Delayed eruption, enamel hypoplasia
Vitamin D resistant rickets Bowing of legs, decrease in height Defective dentin and cementum
formation
Scurvy Boggy and bleeding marginal gingiva,
loose teeth
Riboflavin deficiency Dermatitis, corneal vascularization Glossitis, cheilosis
Pellagra (Niacin) Dermatitis, diarrhea, dementia Painful red oral mucosa, profuse
salivation, Sandwith’s bald tongue

https://t.me/DentalBooksWorld
102 Triumph's Complete Review of Dentistry

Hormonal disorders
Progeria Dwarfism, premature senility Accelerated secondary dentin formation,
delayed teeth eruption
Addison’s disease Bronze skin, general debility Brownish discoloration of oral mucosa
Cushing’s disease Moon face, buffalo hump
Disorders of blood
Pernicious anemia Weakness, tingling of extremities Beefy red tongue, Hunter’s or Moeller’s
glossitis, glossodynia
Aplastic anemia Weakness, pallor of skin, decreased resistance Spontaneous gingival hemorrhage,
to infection ulcerative lesions
Thalassemia Hepato- and splenomegaly Prominent premaxilla
Sickle cell anemia Weakness, cardiomegaly, pains in joints Loss of trabeculation in alveolar bone
Erythroblastosis fetalis Jaundice, Pallor, and anemia Pigmented enamel and Rh Hump
Plummer-Vinson Syndrome Predisposition to carcinoma of upper Atrophy of tongue and mucous
alimentary tract membrane
Leukemia Weakness, fever, generalized Gingival hyperplasia, hemorrhage,
lymphadenopathy ulceration
Hemophilia Bleeding from slight trauma, bleeding into Severe gingival hemorrhage
joints
Bacterial infections
Scarlet fever Scarlet skin rash, tonsillitis, and pharyngitis Stomatitis scarlatina, strawberry tongue,
raspberry tongue, Quinsy
Diphtheria Fever, sore throat, cervical lymphadenopathy Diphtheric membrane on tonsils
Tuberculosis Cough, fever, chills, malaise Irregular, painful ulcers on the tongue
Leprosy Erythematous macules, facial paralysis Tumor-like masses on tongue, lips, hard
palate, gingival hyperplasia, loose teeth
Actinomycosis Abscesses discharging sulfur granules Multiple abscesses, osteomyelitis
Syphilis Chancre, mucus patches, gumma Hutchinson’s triad, mulberry molars,
saddle nose
Viral infections
Herpes simplex Fever, headache, regional lymphadenopathy Yellowish fluid-filled vesicles, painful
mouth and gingiva
Measles Fever, conjunctivitis, tiny red macules on skin Koplik’s spot

Measures of Dental Anxiety


Anxiety scale Author
Behavior profile rating scale Melamed
Venham picture scale Venham L (1979)
Venham anxiety and behavior rating scale Venham (1980)
Children’s fear Survey Schedule Dental Subscale (CFSS-DS) Cuthbert and Melamed (1982)
Adolescents’ fear of dental treatment cognitive inventory Gauthier
Physiologic fear rating scale Sowjanya V, Tandon S, Tharian E (1995)
Smiley faces program Bauchanan H (2005)

https://t.me/DentalBooksWorld
Chapter 2 • Pedodontics 103

Important Facts in Pedo-Dentoalveolar Injuries and Their Management


• Clinically a root fracture can be seen as an extruded tooth displaced lingually.
• In case of root fractures, the time duration for rigid splinting is 12 weeks.
• Formation of new bone and PDL occurs in 7 weeks.
• Calcium hydroxide should be placed in avulsion or intrusion or in any luxation injuries, in which the tooth is displaced
more than 3–5 mm.
• Children with 7–10 years are more prone to avulsion injuries because of unsettled PDL due to continuous tooth eruption.
• Maxillary central incisor is the most frequently avulsed tooth.
• Antibiotic coverage for an avulsion injury is maintained for 7–10 days.
• Primary teeth that are vital with injuries involving pulp are treated with formocresol pulpotomy with full coverage restoration.
• Primary teeth that are nonvital with injuries involving pulp are treated with zinc oxide eugenol pulpotomy.
• In case of primary teeth, the common root fracture occurs at the cervical third.
• Removal of primary incisor before 3 years generally causes delay in the eruption of permanent teeth.
• Removal of primary incisors, 1–2 years before its normal exfoliation causes acceleration in eruption of permanent teeth.
• Ideally splinting duration is 7–10 days.
• Gingival fibers take around 1 week to heal.
• In case of acid etching splint, the resin should be removed after 2–3 months.
• Orthodontic brackets acts as the excellent splint.
• Splinting is usually removed with tapered fissure but with low speed hand piece.
• First reaction of teeth to trauma is hyperemia.
• RCT after re-implantation should be performed after 2 weeks.
• Optimal time for transplantation of a tooth is when the root is 1/2 to 3/4th formed.
• Vehicle of LA: Ringer’s Solution.
• Acetylcholine theory was given by Dett Barn.
• Calcium displacement theory was given by Goldman.
• Specific receptor theory was given by Strichartz in 1987 (most accepted theory).
• Membrane expansion theory was given by Lee.
• Surface repulsion theory was given by Wei.
• Laterally deviated greenstick fractures are most common in pedo patients.
• Half-life of a teeth with reattachment procedures is 2.5 years.

MULTIPLE CHOICE QUESTIONS

INTRODUCTION, CHILD PSYCHOLOGY, AND BEHAVIORAL MANAGEMENT


1. Fear to strangers is known as
A. Xenophobia B. Clostrophobia
C. Arthrophobia D. Claustrophobia
2. Compared to permanent dentition, primary dentition
A. Enamel and dentin is more mineralized B. Dentinoenamel junction is scalloped
C. Dentine is uniformly calcified D. Enamel and dentin thickness is more
3. Earnest Klein has classified habits into
A. Compulsive and noncompulsive habits B. Intentional and nonintentional habits
C. Primary and secondary habits D. Pressure and nonpressure habits
4. All of the following are behavior modification (shaping) techniques in a normal child, except
A. Desensitization B. Contingency management
C. Modeling D. Aversive conditioning

https://t.me/DentalBooksWorld
104 Triumph's Complete Review of Dentistry

5. During what stage of development is peer group identity strongest?


A. Toddler B. Latency
C. Teenager D. Prepuberty
6. According to the intelligence quotient classification, a score of 120–139 is
A. Very superior B. Superior
C. High average D. Average
7. Which of the following is commonly used for immobilization of the head in a noncompliant patient?
A. Papoose board B. Pedi-wrap
C. Forearm–body support D. All of the above
8. Wright’s clinical classification of children’s cooperative behavior has _______ categories
A. 3 B. 4
C. 5 D. 2
9. Universally used method of behavior management in pediatric dentistry for both co-operative and uncooperative
children is
A. Voice control B. Behavior shaping
C. Implosion therapy D. Communication
10. Learning disability is otherwise known as
A. Infantile psychosis B. Minimal brain dysfunction
C. Childhood schizophrenia D. Attention deficit disorder
11. Child of an overprotective overindulgent mother shows
A. Shy, submissive behavior B. Evasive and dawdling behavior
C. Demanding and aggressive behavior D. Stoic behavior
12. The window of infectivity during which most children acquire the cariogenic organisms is between the ages of
A. 15 and 16 months B. 17 and 18 months
C. 19 and 33 months D. 36 and 40 months
13. Fear of closed spaces like “dental clinics” is known as
A. Nyctophobia B. Zoophobia
C. Xenophobia D. Claustrophobia
14. Film size -0 for bitewing radiograph is
A. 22 × 35 B. 24 × 40
C. 32 × 41 D. 50 × 60
15. Form and function originally given by
A. Balters B. Grabbers
C. Van Der Klaauw D. Melvin Moss
16. EGO
A. Reality B. Instinct
C. Moral value D. Conscience
17. Anxiety
A. Fear of unknown B. Fear to stranger
C. Fear to pain D. Fear to object
18. Wechsler test is for all of the following, except
A. Adoptive social behavior B. Intelligence
C. Memory D. IQ
19. To predict the occurrence of caries in a 10-year-old child, which of the following gives best result?
A. Sibling caries B. Mother’s caries
C. Previous caries experience D. Snyder’s test
20. Growth of children of different age groups is evaluated on different parameters at same point of time
A. A longitudinal studies B. Cross-sectional
C. Cross-longitudinal D. Interventional

https://t.me/DentalBooksWorld
Chapter 2 • Pedodontics 105

21. Neonatal teeth are


A. Teeth that erupt at birth or within 30 days after birth B. Teeth that erupt after the 120 days
C. Teeth that erupt within 45 days D. Teeth appearing in 180 days
22. Egocentrism and animism are seen in which stage?
A. Sensorimotor B. Preoperational
C. Concrete operations D. Formal operational stage
23. Discontinuation of nursing bottle should be done at
A. 6 months B. 9 months
C. 12 months D. 18 months
24. Pulse-oximeter is used to determine
A. Rate of blood flow B. Oxygen saturation
C. Blood volume D. Blood coefficient
25. According to Piaget, period of concrete operations is
A. Birth to 18 months B. 12 years and onward
C. 18 months to 7 years D. 6–12 years
26. The nonpharmacologic technique that has remained a corner stone of behavior management is
A. Tell-show-do B. Averse conditioning
C. Hand over mouth exercise D. Voice control
27. Father of Pedodontics in India
A. Dr. BR Vacher B. Dr. Rafiuddin Ahmed
C. Robert Bunon D. Bowen
28. The theory of “Hierarchy of need” is given by
A. Freud B. Maslow
C. Pavlov D. Skinner
29. Frankel’s Behavior Rating Scale includes how many classes?
A. 2 B. 3
C. 4 D. 5
30. By what age a child can sit without support?
A. 4 months B. 6 months
C. 12 months D. 15 months
31. A child visiting a dentist for the first time has basic fear of
A. Unknown B. Equipment
C. Tooth extraction D. Pain
32. Projection and introjection are explained by which theory?
A. Freud B. Maslow
C. Erikson D. Maslow
33. Animism corresponds to
A. Sensorimotor stage B. Formal operation stage
C. Concrete operation stage D. Preoperational phase
34. The suckling reflex and infantile swallow normally disappear by the
A. First year B. Second year
C. Sixth year D. Eighth year
35. A child patient demonstrating resistance in dental office is usually manifesting
A. Anxiety B. Anger
C. Inborn fear D. Immaturity
36. According to Frankel Behavioral Rating Scale, a child who accepts dental treatment but cautious at times is rated as
(Positive)
A. Rating 1 B. Rating 2
C. Rating 3 D. Rating 4

https://t.me/DentalBooksWorld
106 Triumph's Complete Review of Dentistry

37. Classical conditioning occurs when two stimuli are paired together
A. When child’s action are reinforced or rewarded B. When two stimuli are paired together
C. When multiple stimuli are paired together D. When child’s actions are bribed and praised
38. Ontology means
A. Our way of being B. Study of ear
C. Both A and B D. None of these
39. “The silent epidemic” is
A. Dental disease B. Swine flu
C. Cholera D. None of these
40. CMCP stands for
A. Calcium mono-parachlorophenol B. Camphorated mono-chlorobenzophenone
C. Camphorated mono-meta chlorophenol D. Camphorated mono-parachlorophenol
41. The primary reason for aggressive behavior of a 5-year-old child in a dental office is
A. Fear B. Separation from parents
C. Pain D. Unknown
42. The most common type of epilepsy in children is
A. Grand mal epilepsy B. Infantile spasm
C. Petit mal D. None of the above
43. The fear of a 6-year-old related to dentistry is primarily
A. Subjective B. Objective
C. Subjective and objective D. Psychological
44. The approximate age at which the child begins to walk is at
A. 0–2 months B. 3–6 months
C. 7–8 months D. 9–12 months
45. The most common congenital defect of the face and jaws is
A. Macrosomia B. Fetal alcohol syndrome
C. Cleft lip and palate D. Ectodermal dysplasia
46. According to Stanford–Binet, nontrainable type of mental retardation has an IQ of
A. Below 20 B. Between 36–51
C. Above 51 D. 130
47. Which of the following theories of child psychology was proposed by Pavlov?
A. Operant conditioning B. Classical conditioning
C. Social learning theory D. Cognitive theory
48. Radiographs of the jaws of a newborn child ordinarily indicate calcification of
A. 12 teeth B. 16 teeth
C. 20 teeth D. 24 teeth
49. The best time to begin the counseling of parents and establishing a child’s preventive program actually starts
A. Before the birth of the child B. Infants (0–2 years)
C. Toddlers (1–3 years) D. Preschool (3–6 years)
50. In case of conscious sedation for children aged 6 months to 3 years, clear liquids should be stopped
A. 4 hours before the procedure B. 6 hours before the procedure
C. 8 hours before the procedure D. 10 hours before the procedure
51. Best time to see a 3-year-old child in the dental office is
A. Early in the morning B. Just before lunch
C. Just after lunch D. Any time
52. The ratio of Head:Body at 2–3 months IU life is
A. 1:2 B. 1:24
C. 1:8 D. 1:12
53. By what age does the startle and grasp reflex disappear?
A. 1 year B. 1 ½ year
C. 2 years D. 3 years

https://t.me/DentalBooksWorld
Chapter 2 • Pedodontics 107

54. Pacifier sucking is an example of


A. Digit sucking B. Nonnutritive sucking
C. Implicative sucking D. Nutritive sucking
55. For a 3-year-old child, a removable space maintainer is fabricated. What is the biggest disadvantage of this appliance?
A. Irritation to supporting tissues B. Difficult to clean
C. Child may not tolerate wearing D. Function will be compromised
56. “Whistle” is a word substitute for explaining which of the following instruments in children?
A. High-speed hand piece B. Suction apparatus
C. Low-speed hand piece D. Air syringe
57. Joseph Wolpe developed the following behavior management technique
A. Modeling B. Tell-show-do
C. Desensitization D. Voice control
58. Bean bag is used in dental chair for the following purpose
A. Restraining body B. Restraining head
C. Comfort of patient D. In spondylitic patients
59. What is to be done in the initial stage of behavior modification in children?
A. Adaption B. Awareness
C. Evaluation D. Motivation
60. Number of arms in rubber dam retainer is
A. 4 prongs + 4 jaws B. 2 prongs + 4 jaws
C. 4 prongs + 2 jaws D. 2 prongs + 2 jaws
61. The first ADA-approved fluoride toothpaste for OTC (Over-the-counter) was in the year
A. 1945 B. 1955
C. 1964 D. 1971
62. Fear of Strangers and Fear of Separation from parents for the children will be diminished by
A. First birthday B. Fifth birthday
C. Third birthday D. Seventh birthday
63. During development of speech function, the first speech sounds acquired are
A. Bilabials B. Consonants
C. Sibilants D. Nasolabials
64. The following is not a communicative management technique according to American Academy of Pediatric dentistry’s
Standards
A. Voice control B. Positive reinforcement
C. Physical Restraint D. Distraction
65. According to Piaget, the period of concrete operations is
A. Birth to 18 months B. 12 years and onward
C. 18 months to 7 years D. 7–12 years
66. Behavior management techniques were labeled as “Embarrassment of Riches” in 1977 by
A. Jimmy R Pinkham B. Gerald Z Wright
C. David W Chambers D. J C Brauer
67. According to the Freudian psychosexual stages of development the stage which corresponds with development of
mixed dentition and character formation is
A. Concrete operational B. Latency
C. Phallic D. Genital
68. In behavior modification, what is the most common route for premedication?
A. Oral B. Intramuscular
C. Subdermal D. Intravenous
69. In an uncooperative child, the last resort is
A. Physical restraints B. Modeling
C. Desensitization D. HOME

https://t.me/DentalBooksWorld
108 Triumph's Complete Review of Dentistry

70. In a 3½-year-old uncooperative child, which of the following should be best for dental procedures?
A. Modeling B. Physical restraints
C. HOME D. Desensitization
71. If the use of a technique on a child is objectionable enough that the child will co-operate in order to avoid it, that
technique will come under
A. Pharmacological domain B. Physical domain
C. Aversive domain D. Reward-oriented domain
72. In a 9-year-old child, permanent teeth with highest prevalence of caries is
A. Labial surface of canine B. Mesial surface of central incisors
C. Occlusal surface of first molar D. Lingual surface of lateral incisor

DENTAL CARIES AND ANOMALIES, ENDO, SURGICAL, AND PROSTHODONTIC THERAPY


1. Retention in a stainless steel crown is achieved primarily by
A. Chamfer B. Cement
C. Parallel distal and mesial walls D. Crimping
2. The term “atraumatic restorative treatment (ART)” was introduced by
A. McClean B. McDonald
C. McKay D. Mclevan
3. One hour after injury of a 9-year-old child who came with class 3 fracture and small pulp exposure, mode of treatment
is (an hour-old Ellis class 3 injury to 21 in a 9-year-old boy is treated with)
A. Pulpotomy B. Pulpectomy
C. Apexification D. Pulp capping
4. The commonly used filling material in deciduous teeth for RCT
A. Gutta-percha B. Calcium hydroxide
C. Zinc oxide eugenol D. Amalgam
5. A definite method to determine the vitality of primary tooth after trauma is
A. Electric pulp testing B. Transillumination
C. Temperature pulp testing D. None of the above
6. Which of the following is not a space maintainer?
A. Distal shoe B. Lingual holding arch
C. Transpalatal arch D. E-arch
7. Which of the following is false about apexogenesis?
A. Done in a nonvital tooth B. More physiologic process
C. Pulp tissue with mild inflammation D. Root development continues
8. “Chicken-wire” appearance of enlarged bone marrow spaces is seen in
A. Fetal alcohol syndrome B. Sickle cell anemia
C. Hemophilia A D. Beta thalassemia major
9. Cvek pulpotomy refers to
A. Partial pulpotomy B. Cervical pulpotomy
C. Glutaraldehyde pulpotomy D. Formocresol pulpotomy
10. Primary alveolar grafting done in
A. 6 months–1 year B. <2 years
C. 2–4 years D. 4–15 years
11. With all the factors present in the mouth, caries progression is
A. Slow B. Intermittent
C. Continuous D. Rapid
12. A patient comes with history of trauma to anterior teeth. There is metallic sound on percussion of the affected tooth.
The diagnosis based on clinical examination is
A. Lateral luxation B. Subluxation
C. Concussion D. Extrusive luxation

https://t.me/DentalBooksWorld
Chapter 2 • Pedodontics 109

13. Which of the following is not a consideration while selecting stainless steel crown for deciduous teeth?
A. Occlusal morphology B. MD width
C. Occlusogingival height D. Minimal resistance fit
14. All are true for Bruxism, except
A. It is forceful grinding of teeth B. Child is aware of it
C. Flattened molars and lingual wear of max anteriors D. Also known as night grinding
15. N2O/O2 sedation can be effectively used in all, except
A. Mild-to-moderately anxious patient B. Patients not having medical contraindications
C. Patients with gag reflex D. Patients with severe emotional disturbance
16. The term “canker” sore refers to
A. Herpes ulcers B. Vincent infection
C. Recurrent aphthous ulcer D. Candida patch
17. The first sign of root resorption seen in deciduous central incisor and first primary molar is by the age of
A. 2–3 years B. 1–2 years
C. 4–5 years D. 6 years
18. In a mouth breather, tonicity of upper lip
A. Increases B. Decreases
C. Remain same D. There is no relation between them
19. In glutaraldehyde pulpotomy procedure, cotton pellet is kept on pulp stumps for
A. 2 minutes B. 30 seconds
C. 4 minutes D. 5 seconds
20. Fluoride absorption takes place through
A. Stomach and small intestine B. Large intestine
C. Duodenum D. Small intestine
21. Dental fluorosis takes place when
A. >1 mg/l B. >1.5 mg/l
C. >2.5 mg/l D. >3 mg/l
22. The most susceptible area of caries in primary teeth is
A. Mesial surface of 2nd primary molar B. Distal surface of 1st primary molar
C. Distal surface of 2nd primary molar D. Mesial surface of 1st primary molar
23. Full mouth radiographic survey for a child 6–12 years, recommended number of radiographs is
A. 12 B. 14
C. 16 D. 18
24. Fissure eradication in tooth was advocated by
A. Hyatt B. Bunocore
C. Bodecker D. Bowen
25. BIS GMA was advocated by
A. Bowen B. Hyatt
C. Newman D. Dean
26. SnF was advocated by
A. Knutson B. Bibby
C. Muhler D. Dean
27. Yellowish discolorations due to trauma to primary teeth indicates
A. Degeneration of pulp B. Calcific reaction of pulp
C. Necrotic pulp D. Infected pulp
28. Mechanical retention of sealants is the direct result of resin penetration into the porous etched enamel forming tags of
A. 8 microns B. 40 microns
C. 1,000 microns D. 30 microns
29. In a 9-year-old child, permanent tooth surface with highest prevalence of caries is
A. Labial surface of the canines B. Mesial surface of the central incisors
C. Occlusal surface of the first molars D. Lingual surface of the lateral incisors

https://t.me/DentalBooksWorld
110 Triumph's Complete Review of Dentistry

30. A more accurate method of determining delayed or accelerated eruption of permanent teeth is by
A. Arch length analysis
B. Measuring mesial–distal width and cervico-occlusal height of primary teeth
C. Measuring amount of root development and alveolar bone overlying the unerupted permanent tooth
D. Chronologic age of the patient
31. An avulsed immature tooth with open apex with extra oral dry time less than 60 minutes should be
A. Replanted after the root is rinsed off debris with water or saline
B. Replanted after soaking the tooth in doxycycline for 5 minutes
C. Discarded because the success is poor after replantation
D. Replanted after root canal treatment and apical scaling
32. Which among the following is referred to as “Fluoride Bombs”?
A. Nursing caries B. Rampant caries
C. Occult caries D. Radiation caries
33. Prophylactic odontotomy was proposed by
A. Bunocore B. Hyatt
C. Newbrun D. Bowen
34. Which of the pulp vitality tests measures velocity of red blood cells in capillaries? (Repeat)
A. Odontometer B. Dual wavelength spectrometer
C. Hughes Probeye camera D. Laser Doppler flowmetry
35. Simian crease is a characteristic feature of child with
A. Cerebral palsy B. Down’s syndrome
C. Mental retardation D. Epilepsy
36. Before the application of pit and fissure sealants to primary teeth, the enamel etched with a 30–50% phosphoric
acid for
A. 30 seconds B. 60 seconds
C. 90 seconds D. 120 seconds
37. “Nursing bottle caries” is an example for
A. Caries of the primary molar teeth B. Rampant caries
C. Interproximal caries D. Caries affecting only the primary incisors
38. Calcium hydroxide is supplied in
A. Catalyst paste B. Base paste
C. Both A and B D. Gel form
39. The first step in the traditional preparation of a class II cavity in a primary tooth involves
A. Placing the mesio-occlusal depth cut B. Placing the disto-occlusal depth cut
C. Opening the marginal ridge area D. Removing any unsupported enamel
40. The incidence of dental caries in individuals with cerebral palsy, compared to the general population is
A. Controversial B. No difference
C. Always decreased D. Always increased
41. The maximum allowable dosage of 2% lidocaine for a 20-pound child is
A. 4 mg/lb body weight B. 3 mg/lb body weight
C. 2 mg/lb body weight D. 1 mg/lb body weight
42. The last sensation to be lost following local anesthesia administration is
A. Pain B. Touch
C. Temperature sense D. Deep pressure
43. Which is the best medium to transport an avulsed tooth, which is not to be reimplanted immediately?
A. Milk B. Coconut water
C. HBSS D. Viaspan
44. Syncope in a child during dental treatment
A. Situational syncope B. Cerebral syncope
C. Cardioneural syncope D. Chronic fatigue syncope

https://t.me/DentalBooksWorld
Chapter 2 • Pedodontics 111

45. One of the following is a reliable means of quantifying the extent of mouth breathing
A. Cephalometry B. Butterfly or cotton test
C. Water holding test D. Rhinomanometry
46. A simple method of motivating a child patient to control dental caries is
A. Snyder’s test B. Enamel solubility test
C. The use of a disclosing agent D. Using the phase contrast microscope to examine plaque
47. Which of the following type of matrices is frequently used on pediatric patients?
A. Tofflemire B. Graimferential
C. Custom-F band D. Ivory
48. After how many days following concussion should the tooth be checked again for pulpal vitality tests?
A. 24 hours B. 10–12 days
C. 1 month D. 3–6 months
49. The major consideration involved in performing pulpectomy in primary teeth is
A. Root resorption B. Large pulp chamber
C. Tortuous course of the pulp canal D. All of the above
50. Best material for pulp capping is
A. Calcium hydroxide B. Zinc phosphate cement
C. ZOE D. Glass ionomer cement
51. The term “Anaesthesia by copper” was used to
A. Indicate extraction where copper was used as LA B. Indicate the extraction done by Dr. Copper
C. Indicate the incentive of a penny for good behavior D. Indicated surgical instrument made of Copper
52. Rubber dam was introduced by
A. Tomy Hanks B. SC Barnum
C. Hedstorm D. GC Black
53. S-ECC in a child 2, 3, or 4 years of age is characterized by
A. Early involvement of maxillary and the mandibular canines
B. Involvement of mandibular incisors only
C. Early involvement of mandibular incisors and maxillary incisors
D. Mandibular and maxillary molars only
54. Foolproof method to know the efficiency of diet counseling is
A. Lactobacillus test B. Albans test
C. Vinegar test D. None
55. The ratio of formalin to cresol in Buckley’s formaldehyde is
A. 3:2 B. 2:3
C. 1:2 D. 2:1
56. Restoration of choice for obturation in deciduous tooth
A. Zinc oxide eugenol B. Reinforced zinc oxide eugenol
C. Gutta-percha D. Silver point
57. Apexogenesis is
A. Physiological process of root development B. Inducing the root development
C. Arresting the root development D. It is a type of pulpectomy procedure
58. Use of pit and fissure sealants in deciduous teeth is
A. Never indicated B. Limited to buccal pits
C. Indicated even though retention is limited D. Indicated only in rampant caries
59. When a dentist suspects a case of child abuse or neglect attending his dental office, his first step should be oriented
toward
A. Referral B. Reporting
C. Documentation D. Treatment
60. The range of time for the crowns of primary dentition to fully erupt after piercing the gum pads is
A. 2–20 months B. 2 years
C. 7–30 months D. 6–13 years

https://t.me/DentalBooksWorld
112 Triumph's Complete Review of Dentistry

61. Cariogram is given by


A. Avery B. Brathall
C. Mc Donald D. Klevin Allen
62. Partial pulpotomy technique is given by
A. Bowen in 1974 B. Clark in 1976
C. Cvek in 1978 D. Willet in 1980
63. A 7-year-old child comes to your clinic with ulcers with indurated gingiva in the oral cavity and fever with crusting of
lips. Your line of treatment
A. No active treatment. Symptomatic relief and observation
B. Prescribe broad-spectrum antibiotic and mouth-wash
C. Prescribe corticosteroids as this is an allergic reaction
D. Clean the mouth with a gentle mouthwash
64. In a 3-year-old child patient, X-ray film is held in patient’s mouth by
A. Dentist B. Auxiliary personnel
C. Parents D. Child only
65. Most common route for conscious sedation in pedodontic patients is
A. Intravenous B. Intradermal
C. Submucosal D. Subdermal
66. In primary molars
A. The cervical ridges are more pronounced specially on the buccal aspect
B. Markedly constricted neck is present compared to permanent
C. Enamel rods at the cervix slope occlusally
D. All of the above
67. Gingival stripping in children
A. Is due to narrow attached gingiva B. Is treated with gingivectomy
C. Is due to increase in width of attached gingiva D. Is treated by gingivoplasty
68. After avulsion of the tooth, 60 minutes have elapsed; it is kept in doxycycline for 5 minutes for the purpose of
A. Revascularization B. Blocks Ca2+ channels
C. Conditioning of the cementum D. Revitalization of periodontal ligament
69. Stainless steel crowns should extend intragingivally
A. 0.5–1.0 mm B. 1.0–1.5 mm
C. 1.5–2.0 mm D. 0.0–0.5 mm
70. In Basket crown technique
A. Window is prepared on lingual aspect for aesthetics B. Window is prepared on labial aspect for aesthetics
C. Window is prepared on occlusal aspect for aesthetics D. Window is prepared in cervical region for aesthetics
71. Fluoride supplement for a child above 3 years, living in an area with fluoride content of 0.25 ppm is
A. 0.25 mg B. 0.5 mg
C. 0.75 mg D. 1 mg
72. Radiograph of traumatized tooth is mainly necessary
A. To assess stage of root development B. To rule out root fractures
C. To have a baseline comparison with future radiograph D. To rule out hyalinization of pulp
73. Most periodontal damage occurs in the following conditions
A. Intrusion B. Extrusion
C. Infarction D. Luxation
74. If the lactobacillus count is 9,500, the reading will be referred as
A. Little or none B. Slight
C. Moderate D. Marked
75. Fluoride concentration in 0.05% mouth rinses is
A. 250 ppm B. 225 ppm
C. 900 ppm D. 1,000 ppm

https://t.me/DentalBooksWorld
Chapter 2 • Pedodontics 113

76. Demineralization of enamel is due to imbalance between


A. Calcium and phosphate in saliva B. pH in saliva and dental plaque
C. pH of dental plaque and tooth surface D. Calcium and phosphate in dental plaque
77. Caries excavation using Carisolv
A. Conventional preparation design B. Clinical separation of sound and carious dentin
C. Clinical separation of primary and secondary dentin D. Conventional rotary method of cavity preparation
78. Multiple retained deciduous teeth are seen in
A. Amelogenesis imperfecta B. Craniofacial dysostosis
C. Cleidocranial dysostosis D. Osteogenesis imperfecta
79. Diagnosis of incipient carious lesion is best done by
A. D speed film B. C speed film
C. E speed film D. B speed film
80. Which of the following is most commonly used for incipient caries detection?
A. D speed film B. C speed film
C. E speed film D. B speed film
81. The primary teeth that are most often observed to be ankylosed are
A. Maxillary canines B. Mandibular molars
C. Mandibular canines D. Maxillary molars
82. Premature bone loss in primary teeth could be seen in conditions like
A. Hypophosphatasia B. Fibrous dysplasia
C. Papillon–Lefevre syndrome D. All of the above

OCCLUSION/MALOCCLUSION AND ORTHODONTIC THERAPY


1. Tongue blade therapy is carried out for
A. Tongue thrusting B. Thumb cramming
C. Anterior crossbite D. Lip biting habit
2. In patients with surgically corrected cleft lip and palate, what is the most common anomaly seen?
A. Anterior open bite B. Rotated maxillary central incisor
C. Anterior deep bite D. Unilateral/bilateral posterior crossbite
3. A 5-year and 4-month-old child having distal step deciduous molar relationship. There is maxillary constriction with
normal SNA (Sella, Nasion, point A) and submandibular retrusion with decreased SNB. Patient is having low FMA.
What will be the treatment of choice?
A. Wait and review at 6 years of age B. Kloehn headgear for class II maxillary correction
C. Twin block D. FR-II appliance
4. Which of the following serves as a safety valve for the basal discrepancy between forward bodily mandibular thrust
and maxillary horizontal growth changes?
A. Downward growth of maxilla B. Downward growth of mandible
C. Maxillary intercanine width D. Mandibular intercanine width
5. The disadvantage noted while using a buccal coil spring to gain space for mandibular 2nd premolar
A. Rotation of mandibular 1st premolar B. Flaring of mandibular incisors
C. Extrusion of molars D. Rotation of maxillary 2nd premolar
6. Which is the most common occlusion pattern seen in primary dentition?
A. Mesial step B. Distal step
C. Flush terminal D. None of the above
7. Blue grass appliance is used to treat
A. Mouth breathing B. Tongue thrusting
C. Pacifier habit D. Thumb sucking
8. During mature swallowing, the mandible is stabilized by the
A. 4th cranial nerve B. 5th cranial nerve
C. 6th cranial nerve D. 9th cranial nerve

https://t.me/DentalBooksWorld
114 Triumph's Complete Review of Dentistry

9. Distal shoe guiding appliance is


A. Habit breaking appliance B. Space regainer
C. Helps in eruption of first permanent molar D. It is a type of pulpectomy procedure
10. The type of space maintainer advised when there is bilateral loss of teeth same arch
A. Distal shoe B. Band and loop
C. Band and bar D. Lingual arch holding appliance
11. Space maintainer is most commonly given in case of early loss of
A. Primary mandibular second molar B. Primary mandibular incisor
C. Primary maxillary incisor D. Permanent maxillary lateral
12. Space maintainers are
A. Not indicated in mandibular arch as it is contained arch
B. Not usually indicated in maxillary anterior teeth but helps in esthetics
C. Not indicated in maxillary arch as there is no physiological mesial drift
D. Are indicated only to maintain the Leeway space of Nance
13. First to advocate distal shoe in 1929
A. Wilson B. William
C. Willet D. Roche
14. In an 8-year-old child, the first permanent molar erupts ectopically in mesial direction in an intact arch without
excessive resorption of the primary molar roots and does not retract itself. The treatment of choice is to
A. Extract the primary second molar B. Disking the distal surface of the primary second molar
C. Use brass clip to distalize the molar D. No need to treatment, only observation
15. The distal shoe space maintainer is usually placed
A. At the time of extraction of primary second molar B. Just before eruption of permanent molar
C. Before extraction of primary second molar D. To guide a permanent second molar
16. The faciolingual dimension of the Band and loop space maintainer should be approximately
A. 8 mm B. 10 mm
C. 4 mm D. 5 mm
17. The most important determining factors in the placement of a space maintainer after the untimely loss of a primary
tooth is
A. The stage of development of the successor tooth
B. The amount of bone covering the successor
C. The chronological age of the child
18. Best space maintainer for premature loss of second deciduous molar tooth at the age of 5–6 years
A. Distal shoe B. Band and loop
C. Crown and loop D. Removable partial denture
19. The chronological age of a child is
A. Related to the physiological age B. Closely related to the dental age
C. Closely related to the maturity D. Independent of the dental and skeletal age of the child
20. Which of the following shows lowest frequency of cleft lip and palate?
A. Negroes B. Afganis
C. American Indian D. Indian
21. Which of the following is not a feature of Down’s syndrome?
A. Late eruption of deciduous teeth B. Early shedding of deciduous teeth
C. Periodontal disease D. Retrognathism
22. Inclined plane is used for the correction of which of the following conditions?
A. Developing crossbite B. Deep bite
C. Open bite D. Developed crossbite
23. The mandibular foramen in a child is
A. Below the occlusal level B. At the occlusal level
C. Above the occlusal level D. Above or below the occlusal level

https://t.me/DentalBooksWorld
Chapter 2 • Pedodontics 115

24. What is the amount of time required by erupting premolar to move through 1 mm of bone as measured on a bite-wing
radiograph?
A. 2–3 months B. 4–5 months
C. 6–7 months D. 7–8 months
25. Which of the following is an indication for serial extraction?
A. Class II malocclusion with anterior deep bite
B. Class I malocclusion with severe mandibular anterior crowding
C. Class II malocclusion with anterior open bite and arch length deficiency
D. Class I malocclusion with well-spaced arch
26. Mayne space maintainer represents which of the following type of space maintainers?
A. Nonfunctional B. Functional
C. Cantilever D. Crown and crib
27. Nickel–Titanium alloy was first introduced as an orthodontic wire by
A. Andreason B. Buehler
C. Burrstone D. Bishara
28. In serial extraction procedure if maxillary 1st premolar is extracted then maxillary canine erupts
A. Downward B. Downward-backward
C. Downward – forward D. Forward
29. Distal shoe space maintainer extends intragingivally
A. 0.0–0.5 mm B. 0.5–1.0 mm
C. 1.0–1.5 mm D. 1.5–2.0 mm
30. Thumb sucking habit is prevented by
A. Elbow bandage B. Finger bandage
C. Lingual crib D. All of the above
31. Rate of space loss following extraction of mandibular permanent first molar is?
A. 1 mm/year B. 1.5 mm/year
C. 2 mm/year D. 2 mm/year
32. The common cause of anterior crossbite in mixed dentition is due to
A. Prolonged retention of deciduous teeth B. Jaw discrepancy
C. Thumb sucking D. Mouth breathing
33. Which of the following is not an indication for serial extraction?
A. Arch length deficiency in comparison to total tooth material
B. Patients with straight profile
C. Absence of sufficient growth to overcome the jaw base-tooth size discrepancy
D. Class II and Class III malocclusion with skeletal abnormality
34. Band and loop space maintainer is contraindicated in all, except
A. Single tooth missing in the posterior region B. Lower anterior crowding
C. Moderate to severe space loss D. High caries susceptibility
35. Which of the following technique is not used in cleft lip repair?
A. Tennison–Randall flap B. Le-Mesurier flap
C. Miliard’s technique D. Von Langenback flap technique
36. Inca bone, Goethe ossicle in skull presents an independent bone in suture
A. Pterion B. Lambda
C. Bregma D. Asterion
37. Arch length from deciduous to permanent dentition
A. Decreases B. Increases
C. Remains same D. First increases then decreases

https://t.me/DentalBooksWorld
116 Triumph's Complete Review of Dentistry

38. The suckling reflex and infantile swallow normally disappear by the
A. First year B. Second year
C. Sixth year D. Eighth year
39. Narrow, high arched palate, prolonged retention of deciduous teeth and failure in the eruption of permanent teeth is
a characteristic feature of
A. Paget’s disease B. Cherubism
C. Cleidocranial dysplasia D. Osteogenesis imperfecta
40. In a preadolescent child, the maximum midline diastema that will be closed spontaneously after canine eruption is
A. 1 mm B. 2 mm
C. 4 mm D. 5 mm
41. What is the ideal time for repair of cleft lip?
A. At birth B. Between 3 and 6 months of age
C. At 1 year 6 months D. At 1.5–2 years of age
42. The most common post anesthetic complication seen in child is
A. Pain B. Extraoral swelling
C. Hematoma D. Lip biting
43. Most likely diagnosis of familial developmental abnormality causing enlargement of jaws in the children?
A. Fibrous dysplasia B. Hyperparathyroidism
C. Cherubism D. Central giant cell granuloma
44. A more accurate method of determining delayed or accelerated eruption of permanent teeth is by
A. Arch length analysis
B. Measuring mesial distal width and cervico-occlusal height of primary teeth
C. Measuring amount of root development and alveolar bone overlying the unerupted permanent tooth
D. Chronological age of the patient
45. Primate spaces are present as primary dentition between
A. Maxillary primary canine and central incisor and mandibular primary canine and mandibular second molar
B. Maxillary primary central incisor and lateral incisor and mandibular primary lateral incisor and mandibular primary
canine
C. Maxillary primary lateral incisor and primary canine and between mandibular primary canine and a mandibular first
molar
D. Maxillary primary lateral incisor and primary first molar between primary first molar and mandibular second molar
46. The functional regulator designed by Frankel is
A. Active tooth borne appliance B. Passive tooth borne appliance
C. Tissue borne appliance D. Hybrid appliance
47. Eruption time of permanent mandibular canine is
A. 9–10 years B. 11–12 years
C. 6–7 years D. 17–25 years
48. In the FDI tooth numbering system 51 denotes
A. Maxillary right primary central incisor B. Maxillary right primary 2nd molar
C. Maxillary right permanent central incisor D. Maxillary right second premolar
49. Which of the following is not a type of mixed dentition analysis?
A. Moyer’s B. Tanaka Johnston
C. Pont and Linder Harth D. Hixon old father
50. The maxillary intercanine dimensions serves to control the mandibular growth, which is known as
A. Mechano transduction B. Safety valve mechanism
C. Pterygoid response D. Buccinator mechanism

https://t.me/DentalBooksWorld
Chapter 2 • Pedodontics 117

ANSWERS

INTRODUCTION, CHILD PSYCHOLOGY, AND BEHAVIORAL MANAGEMENT


1. Answer: A
2. Answer: C
3. Answer: B
4. Answer: D
Aversive conditioning is a type of behavior conditioning in which noxious stimuli are associated with undesirable or
unwanted behavior that is to be modified or abolished.
5. Answer: C
6. Answer: B
7. Answer: C
8. Answer: A
9. Answer: D
10. Answer: B
11. Answer: C
12. Answer: C
13. Answer: D
14. Answer: A
15. Answer: C
16. Answer: A
17. Answer: A
18. Answer: A
The Wechsler Adult Intelligence Scale (WAIS) is an IQ test designed to measure intelligence and cognitive ability in adults
and older adolescents.
19. Answer: C
20. Answer: B
21. Answer: A
22. Answer: B
The preoperational stage is the second stage in Piaget’s theory of cognitive development. This stage begins around age 2 as
children start to talk and last until approximately age 7. During this stage, children begin to engage in symbolic play and
learn to manipulate symbols.
23. Answer: C
24. Answer: B
25. Answer: D
The concrete operational stage is the third in Piaget’s theory of cognitive development. This period spans the time of middle
childhood and is characterized by the development of logical thought. While kids at this age become more logical about
concrete and specific things, they still struggle with abstract ideas.
26. Answer: A
27. Answer: A
28. Answer: B
29. Answer: C
Pediatric Dentistry Module
Rating 1 – Definitely negative
Refusal of treatment; crying forcefully, fearful, or any other evidence of extreme negativism
Rating 2 – Negative
Reluctance to accept treatment; uncooperative; some evidence of negative attitude but not pronounced, i.e., sudden
withdrawal

https://t.me/DentalBooksWorld
118 Triumph's Complete Review of Dentistry

Rating 3 – Positive
Acceptance of treatment; at time of cautious; willingness to comply with the dentist, at time with reservation, but patient
follows the dentist’s directions cooperatively
Rating 4 – Definitely positive
Good rapport with dentist; interested in the dental procedures; laughing and enjoying the situation
30. Answer: C
31. Answer: A
32. Answer: A
Sigmund Freud. Freud developed the psychoanalytic theory of personality development, which argued that personality is
formed through conflicts among three fundamental structures of the human mind: the id, ego, and superego.
33. Answer: A
The sensorimotor stage is the first of the four stages. Piaget used to define cognitive development. Piaget designated the
first 2 years of an infant’s life as the sensorimotor stage. During this period, infants are busy discovering relationships
between their bodies and the environment.
34. Answer: A
The suck-swallow reflex is one of the first reflexes demonstrated by the developing infant. This behavior can be observed
during gestation. It typically begins to appear around the 12th–13th week of pregnancy. During this period, the fetus may
demonstrate the beginnings of this reflex by sucking its thumb, yawning, or making swallowing motions. By 36 weeks, the
reflex is usually fully developed. When born, the infant should be able to suck and swallow immediately.
35. Answer: A
36. Answer: C
37. Answer: B
38. Answer: A
39. Answer: A
40. Answer: D
41. Answer: A
42. Answer: C
Absence seizures are one of several kinds of seizures. These seizures are sometimes referred to as petit mal seizures (from
the French for “little illness,” a term dating from the late 18th century). Absence seizures are characterized by a brief loss
and return of consciousness, generally not followed by a period of lethargy (without a notable postictal state).
43. Answer: A
44. Answer: D
45. Answer: C
46. Answer: A
Stanford–Binet Fifth Edition (SB5) classification
IQ Range (“deviation IQ”) IQ Classification
145–160 Very gifted or highly advanced
130–144 Gifted or very advanced
120–129 Superior
110–119 High average
90–109 Average
80–89 Low average
70–79 Borderline impaired or delayed
55–69 Mildly impaired or delayed
40–54 Moderately impaired or delayed

47. Answer: B
Classical conditioning (also known as Pavlovian or respondent conditioning) refers to a learning procedure in which a
biologically potent stimulus (e.g., food) is paired with a previously neutral stimulus (e.g., a bell).
48. Answer: D
49. Answer: A

https://t.me/DentalBooksWorld
Chapter 2 • Pedodontics 119

50. Answer: A
51. Answer: A
52. Answer: A
53. Answer: C
Rooting reflex
This reflex begins when the corner of the baby’s mouth is stroked or touched. The baby will turn his or her head and open
his or her mouth to follow and root in the direction of the stroking. This helps the baby find the breast or bottle to begin
feeding. This reflex lasts about 4 months.
Suck reflex
Rooting helps the baby become ready to suck. When the roof of the baby’s mouth is touched, the baby will begin to
suck. This reflex does not begin until about the 32nd week of pregnancy and is not fully developed until about 36 weeks.
Premature babies may have a weak or immature sucking ability because of this. Because babies also have a hand-to-mouth
reflex that goes with rooting and sucking, they may suck on their fingers or hands.
Moro reflex
The Moro reflex is often called a startle reflex. That is because it usually occurs when a baby is startled by a loud sound or
movement. In response to the sound, the baby throws back his or her head, extends out his or her arms and legs, cries, then
pulls the arms and legs back in. A baby’s own cry can startle him or her and trigger this reflex. This reflex lasts until the
baby is about 2 months old.
Tonic neck reflex
When a baby’s head is turned to one side, the arm on that side stretches out and the opposite arm bends up at the elbow.
This is often called the fencing position. This reflex lasts until the baby is about 5–7 months old.
Grasp reflex
Stroking the palm of a baby’s hand causes the baby to close his or her fingers in a grasp. The grasp reflex lasts until the baby
is about 5–6 months old. A similar reflex in the toes lasts until 9–12 months.
Stepping reflex
This reflex is also called the walking or dance reflex because a baby appears to take steps or dance when held upright with
his or her feet touching a solid surface. This reflex lasts about 2 months.
54. Answer: B
55. Answer: C
56. Answer: A
57. Answer: C
Systematic desensitization, also known as graduated exposure therapy is a type of behavior therapy used in the field of
psychology to help effectively overcome phobias and other anxiety disorders. More specifically, it is a form of counter
conditioning, a type of Pavlovian therapy developed by South African psychiatrist, Joseph Wolpe. In the 1947, Wolpe
discovered that the cats of Wits University could overcome their fears through gradual and systematic exposure. The
process of systematic desensitization occurs in three steps. The first step of systematic desensitization is the identification
of an anxiety inducing stimulus hierarchy. The second step is the learning of relaxation or coping techniques. When the
individual has been taught these skills, he or she must use them in the third step to react toward and overcome situations
in the established hierarchy of fears. The goal of this process is for the individual to learn how to cope with, and overcome
the fear in each step of the hierarchy.
58. Answer: A
59. Answer: B
60. Answer: C
61. Answer: C
62. Answer: C
63. Answer: A
64. Answer: C
65. Answer: D
66. Answer: C
67. Answer: B

https://t.me/DentalBooksWorld
120 Triumph's Complete Review of Dentistry

68. Answer: C
69. Answer: A
70. Answer: A
71. Answer: C
72. Answer: C

DENTAL CARIES AND ANOMALIES, ENDO, SURGICAL, AND PROSTHODONTIC THERAPY


1. Answer: C
Properties of SS crown
• Heating does not increase strength
• Work hardens
• High chromium reduces corrosion
• Soldering with flux reduces corrosion resistance
2. Answer: A
3. Answer: D
Direct Pulp Capping (DPC)
• Indications: small mechanical or traumatic exposures in primary teeth, tooth is restorable – doubtful prognosis
• Contraindications: carious exposure in primary tooth, persistent inflammation, internal resorption, calcific
metamorphosis
• Objectives: preserve pulp vitality under tertiary dentin bridge
• Ca(OH)2 may produce internal resorption
• Pulpotomy is preferred due to predictable outcomes
Pulpotomy
• Indications: carious/iatrogenic pulp exposure, coronal pulp affected/infected, radicular tissue vital (or affected but vital)
as judged by clinical and radiographic means, tooth is restorable
• Objectives: to maintain tooth in symptomless state until tooth is not strategic, healthy supporting tissues, no harm to
succedaneous tooth
• Technique: excavate caries, amputate coronal pulp, achieve hemostasis, treat radicular pulp with medicament/
technique, restore with permanent restoration
4. Answer: C
5. Answer: D
6. Answer: D
7. Answer: A
Apexogenesis
• Remove coronal portion of vital pulp
• Place agent to preserve radicular vitality
• Encourage continued root development
• Emergency procedure for future RCT
• Promote tertiary dentin formation
• No evidence of inflammatory resorption
• No evidence of root and periradicular pathosis
Nonvital pulp therapy for young permanent teeth
Objectives
• Promote continued apical root development
• Achieve apical closure
Apexification (Ca(OH)2: Frank technique)
• Necrotic tissue removal short of the apex
• Place agent (Ca(OH)2) in canals
• May be necessary to replace Ca(OH)2 q3–6 months

https://t.me/DentalBooksWorld
Chapter 2 • Pedodontics 121

Action of Ca(OH)2
• Bactericidal
• Low-grade irritation induces hard tissue formation
• Dissolves necrotic debris
8. Answer: D
9. Answer: A
Partial pulpotomy (Cvek)
• Preservation of cell rich coronal pulp
• Increased healing potential due to preserved pulp
• Physiologic apposition of cervical dentin
• Obviate need for RCT
• Natural color and translucency preserved
• Maintenance of pulp test responses
10. Answer: B
11. Answer: B
12. Answer: A
13. Answer: C
14. Answer: B
15. Answer: D
16. Answer: C
17. Answer: C
18. Answer: B
19. Answer: C
20. Answer: A
21. Answer: B
The fluorosis issue:
• Fluorosis is a permanent intrinsic white-to-brown discoloration of enamel
• Occurs during tooth formation during the first few years of life
• Increase in prevalence due to increased ambient fluoride
• Sources of ingested fluoride
–– Diet/des of ingested fl due to increased ambient fldiscoloration of
–– Dentifrice consumption
–– Previous supplementation schedules based on presumed lower fluoride intake
–– Inappropriate Rxentation schedules based on presumed lower fluoride
• Measured by Deanntation schedules based on presumed lower flion of enamelionosis (TSIF), Fluorosis Risk Index
(FRI), and Thylstrup–Fejerskov Index (TF)
Acute fluoride toxicity
• Symptoms of overdose – GI, CNS; death in 4 hours
• Probably toxic dose 5 mg F/kg
• Certainly lethal dose 16–32 mg F/kg (Hodge and Smith) 15 mg F/kg Whitford)
• Treatment – Induce vomiting or bind F
–– <8 mg F/kg: milk, observe > 6 hours, refer if symptoms develop
–– ≥8 mg F/kg: syrup of ipecac, followed by milk; refer immediately
–– Unknown dose – asymptomatic: treat as <8 mg F/kg symptomatic: give milk, refer immediately
–– Poison control center: gastric lavage, IV calcium gluconate
22. Answer: A
23. Answer: B
24. Answer: C
25. Answer: A
26. Answer: C

https://t.me/DentalBooksWorld
122 Triumph's Complete Review of Dentistry

27. Answer: B
28. Answer: B
29. Answer: B
30. Answer: C
31. Answer: B
32. Answer: C
33. Answer: B
34. Answer: D
35. Answer: B
36. Answer: D
37. Answer: B
38. Answer: A
39. Answer: C
40. Answer: A
41. Answer: C
42. Answer: B
43. Answer: D
44. Answer: C
45. Answer: D
46. Answer: C
47. Answer: C
48. Answer: B
49. Answer: C
50. Answer: A
Calcium hydroxide (Ca(OH)2)
Method of action – Mineralization
Acceptable outcomes – 31–87%
Demonstrated worse outcome than FC (FORMO CRESOL) in randomized control trial
51. Answer: C
52. Answer: B
53. Answer: A
54. Answer: A
55. Answer: C
Formocresol (FC) Full-strength or Buckley’s
Method of action: Tissue fixation
Acceptable outcomes: 62–97%
Gold standard, safety concerns, distribution to viscera with multiple pulpotomies, 3–5 minutes application
56. Answer: A
Zinc oxide eugenol (ZOE) pulpotomy
Method of action: Palliative
Acceptable outcomes: 57%
NOTE: Internal resorption common outcome
57. Answer: A
58. Answer: C
59. Answer: C
60. Answer: C
61. Answer: B
62. Answer: C
63. Answer: A

https://t.me/DentalBooksWorld
Chapter 2 • Pedodontics 123

64. Answer: C
65. Answer: C
66. Answer: D
67. Answer: A
68. Answer: A
69. Answer: A
70. Answer: B
71. Answer: D
Fluoridated water
• Issues
–– Controlled/natural water fluoridation
–– Optimal range is 0.7–1.2 mg/l (0.7–1.2 ppm)
–– Diet: g/range is 0.7ater flbial as
–– Bottled water (variable F-content but usually low F-concentrations; may vary seasonally and with manufacturer)
–– Filtration systems: reverse osmosis and distillation reduce F to very low levels
72. Answer: C
73. Answer: D
74. Answer: C
75. Answer: B
Fluoride rinses
• OTC products: 0.05% NaF=0.022% F ion = 220 ppm ~ 1 mg/5 mL (daily use)
• Rx products: NaF 0.2% (weekly use)
• Indications
–– orthodontics appliances
–– radiation therapy to head, face or neck
–– prosthetic appliances
–– high sucrose diet - either liquid or solids
–– high risk patients with history of caries, poor oral hygiene
• Risks: F ingestion and alcohol (some products)
76. Answer: C
77. Answer: B
78. Answer: C
79. Answer: A
80. Answer: C
81. Answer: B
Ankylosed Teeth
• Lower first primary molars most commonly affected, followed by upper first primary molars, lower second primary
molars, and upper second primary molars
–– Also occurs secondary to traumatized primary and permanent anterior teeth
–– Resorption of ankylosed molars usually proceeds in normal mode with 95% of premolars erupting into proper
occlusion with normal periodontal health and alveolar bone height
–– Most common sequelae is simply delayed transition as to timing
• Static retention of ankylosed tooth often results in clinical “submersion,” supraeruption of opposing tooth and tipping
of adjacent teeth with accompanying loss of space. Severity related to how early ankylosis occurred, which tooth
involved, which arch
–– Second primary molars of much greater significance to arch integrity than first primary molars
–– Particularly true when ankylosis occurs prior to eruption of first permanent molar
–– Greater vertical development of maxillary processes can effectively “bury” ankylosed upper second primary molar
with more severe consequences. Often requires complicated surgical removal
82. Answer: D

https://t.me/DentalBooksWorld
124 Triumph's Complete Review of Dentistry

OCCLUSION/MALOCCLUSION AND ORTHODONTIC THERAPY


1. Answer: C
Anterior Crossbites in the Primary Dentition
• True Class III versus Pseudo-Class III: Anterior crossbites typically involve full incisor segment as opposed to individual
teeth. Must distinguish between a pseudo-Class III and a true Class III
–– Pseudo-Class III: Incisal and canine interference produces anterior shift of the mandible on closure
–– Treatment directed at advancement of incisor segment to eliminate interference.
–– Fixed or removable maxillary appliances with finger or sweep springs to advance incisors
–– True Class III: In primary dentition presents classic skeletal and dental patterns with retruded maxilla, prognathic
mandible, “adult” concave profile, retroclined lower incisors
–– Treatment directed at dentofacial orthopedic changes to correct skeletal malocclusion
–– Reverse pull headgear/facemask, chincup.
–– Both pseudo- and true Class III may require concurrent maxillary expansion
2. Answer: D
• Bilateral Posterior Crossbite: Represent true maxillary skeletal constriction with bilateral buccal segment crossbite,
midline symmetry, and no notable shift of mandible
–– Reported at 2 to 3% of posterior crossbites in children, often associated with dolichofacial skeletal vertical growth,
openbite malocclusion, compromised airways, and mouth breathing patterns
–– Maxillary expansion with emphasis toward sutural separation indicated for significant transverse discrepancy (e.g.,
fixed palatal expanders – RPE of Haas, Hyrax). Given etiological factors with skeletal growth and airway issues, while
early expansion desirable; long-term management generally requires a multiphased comprehensive treatment plan
3. Answer: A
4. Answer: C
5. Answer: A
6. Answer: A
Permanent Molar Relationships in the Mixed Dentition
• Class I: Maxillary first molar mesial cusp in mandibular molar buccal groove – “Ideal”
• End-on Class II: Majority of mixed dentition occlusions. Corrects to Class I with late mesial shift in late transition
stages in most cases; but may stay end-on or even shift to full Class II
• Full Class II: Maxillary first molar mesial cusp forward in embrasure between lower first molar and second primary
molar or second premolar
7. Answer: D
Nonnutritive Digit Sucking Habits (NNS)
• Normal at early age: 50% of children with NNS habit will discontinue between 24 and 28 months of age
–– Digit habits can last longer than pacifier habits; both produce similar affects if persist past 4 years
–– Incidence rate of 10–15% at age 5 years
• May result in anterior openbite, distorted incisor eruption, increased overjet, proclined upper incisors, linguoversion of
lower incisors, posterior crossbite with constricted maxilla, possibly Class II relations
• Consider intervention prior to eruption of permanent anterior teeth approximating age 5 to 6 years if NNS habit
persists and patient-parent indicate understanding of need to stop
–– Use “gentle persuasion” as beginning treatment; behavior modification can be successful
–– Cribs, rakes, “bluegrass appliance” are choices for fixed therapy to “help” child quit
8. Answer: B
9. Answer: C
10. Answer: D
11. Answer: A
12. Answer: B
13. Answer: C
14. Answer: C
15. Answer: C

https://t.me/DentalBooksWorld
Chapter 2 • Pedodontics 125

16. Answer: A
17. Answer: A
18. Answer: A
19. Answer: D
20. Answer: A
21. Answer: D
22. Answer: A
23. Answer: A
24. Answer: A
25. Answer: B
Lower anterior crowding considered normal as the average crowding is 1.6 ± 1.0 mm. after incisor eruption is complete.
This means the vast majority of children express 0–4 mm. of crowding at 8–9 years of age and importantly, there are
no future arch dimensional changes to compensate for any degree of crowding and malalignment in the lower anterior
segment.
• Approach during incisor transition is to allow “wedging” effect of eruption to optimize width
• After lateral incisor eruption, what you see is what you get
• Arch length analysis
26. Answer: A
27. Answer: A
28. Answer: B
29. Answer: B
30. Answer: D
31. Answer: A
32. Answer: A
33. Answer: D
34. Answer: A
35. Answer: D
36. Answer: B
37. Answer: A
38. Answer: A
39. Answer: C
40. Answer: B
41. Answer: B
42. Answer: D
43. Answer: C
44. Answer: C
45. Answer: C
46. Answer: C
47. Answer: A
48. Answer: A
49. Answer: C
50. Answer: B

https://t.me/DentalBooksWorld
3 General Pathology

SYNOPSIS

BASIC PATHOLOGY
PATHOLOGY

It is the “scientific study of disease.” Scientific study of the molecular, cellular, tissue, or organ system
response to injurious agents.
STALWARTS IN PATHOLOGY
1. Father of Clinical pathology – Paul Ehrlich
2. Father of Cellular pathology – Rudolf Virchow
3. Father of Blood Transfusion – Carl Landsteiner
4. Father of Exfoliative Cytology – George Papanicolaou
5. Father of Museum in Pathology – John Hunter

CELLULAR HOUSEKEEPING
1. Mitochondria
• Oxidative phosphorylation
• Intermediate for heme synthesis
• Intrinsic pathway for apoptosis
2. Plasma membrane proteins
• Phosphatidylinositol serves as a scaffold for intracellular proteins.
• Phosphatidylserine is needed for apoptosis. It acts as co-factor in clotting of blood.
• Glycolipids are important in cell-to-cell interactions.
3. Smooth endoplasmic reticulum (SER)
• Found abundant in gonads and liver
• Used for steroid hormone synthesis
• Sequestration of calcium
4. Peroxisomes
• Breakdown of fatty acids
5. Proteasomes
• Required for selective chewing of denatured proteins using ubiquitin
6. Lysosomes
• Most cytosolic enzymes prefer to work at a pH of 7.4 whereas lysosomal enzymes function best at pH of 5
7. Golgi apparatus
• Mannose-6-phosphate is the marker
8. The human genome contains roughly 3.2 billion DNA base pairs and only 2% is used for coding of proteins

https://t.me/DentalBooksWorld
Chapter 3 • General Pathology 127

GROWTH FACTORS AND TRANSCRIPTION FACTORS


• MYC and JUN are the transcription growth factors that regulate the expression of genes that are needed for growth.
• p53 is a transcription factor that triggers the expression of genes that lead to growth arrest.
• Hepatocyte growth factor is also known as Scatter factor.
• Vascular endothelial growth factor-A (VEGF-A) is the major angiogenic factor.
• Hypoxia is the most important inducer of VEGF production.
• Fibroblast growth factor (FGF-7) is known as keratinocyte growth factor.

EXTRACELLULAR MATRIX
• Laminin is the most abundant glycoprotein in the basement membrane.
• The major constituents of basement membrane are nonfibrillar type IV collagen and laminin.
• Collagens are typically made of triple helix.

CELL INJURY
Cell injury is a sequence of events that occur if the limits of adaptive capability are exceeded or no adaptive response is possible.
• Hypoxia is the most common cause of cell injury.
• Ischemia is the most common cause of hypoxia.
Mechanisms of Cell Injury
• Depletion of ATP
• Mitochondrial damage
• Influx of intracellular calcium and loss of calcium homeostasis
• Accumulation of oxygen-derived free radical (Oxidative stress)
• Defects in membrane permeability

Types of cell injury


• Reversible cell injury
• Irreversible cell injury

REVERSIBLE CELL INJURY


• Hypoxia is the most common cause.
• Hypoxia will result in earliest involvement of mitochondria.
• ATP depletion is the most critical event in both ischemic and toxic cell injury.

Morphology of Reversible Cell Injury


1. Cellular swelling
• Hydropic change or swelling of cell due to increased water entry is the earliest change seen in reversible cell injury
• Swelling of organelles such as mitochondrial swelling and phospholipid rich amorphous densities
• Loss of microvilli and bleb formation of plasma membrane
• Formation of myelin figures due to breakdown of membranes of cellular organelles like endoplasmic reticulum
2. Fatty degeneration
• Fatty degeneration or fatty metamorphosis, steatosis is the abnormal appearance of fat within parenchymal cells.
3. Hyaline change
• Zenker’s degeneration – Rectus abdominalis muscle in typhoid fever.
• Mallory’s degeneration – Alcoholic liver cell injury.
• Nuclear or cytoplasmic degeneration – Viral infections.
• Russell’s bodies – Immunoglobulins in RER.

https://t.me/DentalBooksWorld
128 Triumph's Complete Review of Dentistry

IRREVERSIBLE CELL INJURY


• Persistence of ischemia or hypoxia results in irreversible damage to the cell (cell death).
• Inability to reverse mitochondrial dysfunction mainly characterizes irreversible cell injury.

Features of Irreversible Cell Injury


1. Damage to cell membrane
• Damage to the cell membrane results in massive influx of calcium
• Calcium influx results in formation of large flocculent mitochondrial densities and activation of enzymes
2. Nuclear changes
These are the most specific microscopic features of irreversible cell injury
• Pyknosis: Nuclear condensation
• Karyorrhexis: Fragmentation of nucleus
• Karyolysis: Nuclear dissolution

Irreversible cell injury is of two types


1. Necrosis
2. Apoptosis

NECROSIS
Definition: Necrosis is defined as focal death along with degradation of tissue by hydrolytic enzymes liberated by cells.
It is invariably accompanied by inflammation.
The different type of necrosis are explained as follows.
1. Coagulative necrosis
• Most common type of necrosis
• Associated with ischemia
• Seen in organs such as heart, liver, kidney, and spleen, except brain
• Coagulative necrosis is associated with “tomb stone” appearance of the affected tissue
2. Liquefactive necrosis
• Also called colliquative necrosis
• Hydrolytic enzymatic destruction of cells
• Associated with bacterial and fungal infections
• Abscess formation
• Seen in brain
• E.g., Infarct brain and abscess cavity
3. Caseous necrosis
• Combination of coagulative and liquefactive necrosis.
• Cheese-like appearance of the necrotic material.
• Characteristics of tuberculosis.
• Also seen in syphilis, histoplasmosis, and coccidioidomycosis.
4. Fat necrosis
• Action of lipases on fatty tissues
• Seen in breast, omentum, and pancreatitis
5. Fibrinoid necrosis
• Complexes of antigens and antibodies are deposited in vessel walls with leakage of fibrinogen out of vessels.
• Seen in PAN, rheumatic heart disease, malignant hypertension, autoimmune diseases.

https://t.me/DentalBooksWorld
Chapter 3 • General Pathology 129

6.  Gangrenous necrosis


• Necrosis of tissues with superadded putrefaction
• Dry gangrene is similar to coagulative necrosis
• Wet gangrene is similar to liquefactive necrosis
• Noma is the gangrenous lesion seen in mouth
• Fournier’s gangrene is seen in scrotum.

APOPTOSIS
Definition: Apoptosis is a form of “coordinated and internally programmed cell death,” which is of significance in a variety
of physiologic and pathologic conditions.
Features of Apoptosis
• The process may be either pathological or physiological.
• Absence of inflammation is the characteristics of apoptosis.
• Involvement of a single cell or small clusters of cells.
Pro-Apoptotic Factors
These proteins helps in regulation of apoptosis
• BAX
• BAD
• p53
• Puma
• Noxa
• BCL-XS
Antiapoptotic Factors
These proteins inhibits apoptosis
• BCL-2
• BCL-XL
• MCL-1
Apoptosis in Physiological Conditions
• Endometrial cells (menstruation)
• Cell removal during the development of embryo
• Virus infected cells and neoplastic cells by cytotoxic T cells
• Thymus involution at early age
Apoptosis in Pathological Process
• Councilman bodies: Viral hepatitis
• Duct obliteration in cystic fibrosis
• Damage to DNA by radiation
• Corticosteroid destruction of lymphocytes
• Graft versus host disease (GVHD)

Difference Between Necrosis and Apoptosis


Necrosis Apoptosis
Passive process Active process
Always pathological Both physiological and pathological
Death of many contiguous cells Death of a single cell
Plasma membrane is disrupted Plasma membrane is intact
Cell size increases Cell size decreases (cell shrinkage)
Presence of inflammation Absence of inflammation
Lysosomes breakdown and release hydrolases Lysosomes are intact
Electrophoresis: SMEAR Pattern Electrophoresis: STEP-LADDER Pattern

https://t.me/DentalBooksWorld
130 Triumph's Complete Review of Dentistry

CELLULAR ADAPTATION
Cells may show adaptation to injury by various processes like atrophy, hypertrophy, hyperplasia, metaplasia, dysplasia, and so on.
Atrophy
• Reduced size of an organ or tissue resulting from a decrease in cell size and number.
Hypertrophy
• Increase in cell size without increase in number
• DNA content is more than normal cells
• Hypertrophy results due to increase in growth factors
Hyperplasia
• Increase in cell number without increase in cell size
• DNA content is same as normal cells
Metaplasia
• Replacement of one differentiated cell type by another cell type
• Reversible process
• No loss of polarity
• Results from “reprogramming of stem cells”
• Pleomorphism is absent
• Most common metaplasia is columnar to squamous
Anaplasia
• Loss of differentiation
• Irreversible process
• It is the hallmark of malignancy
• Increased atypical mitosis
• Tumor giant cells
Dysplasia
• Carcinoma in situ
• Disordered arrangement of epithelial cells
• Basement membrane is intact
• Reversible in early stages
• Typical mitotic figures are present

Few Examples of Cellular Adaptation


• Presence of goblet cells in the nonrespiratory terminal bronchioles (where they are usually absent) of a chronic smoker is
an example of metaplasia.
• Increase in goblet cells in main stem bronchus is an example of hyperplasia.
• The most common example of epithelial metaplasia is columnar to squamous.
• Deficiency of vitamin A induces squamous metaplasia in the respiratory epithelium.
• Metaplasia from squamous to intestinal columnar type occurs in Barrett’s esophagus.
• Epithelial metaplasia – Barrett’s esophagus
• Connective tissue metaplasia – Myositis ossificans

PATHOLOGIC CALCIFICATION
Pathologic calcification is the abnormal tissue deposition of calcium salts together with other mineral salts.
Calcification begins in mitochondria of all the organs except kidneys (begins in basement membrane).
It is of two types:
1. Dystrophic calcification
2. Metastatic calcification

https://t.me/DentalBooksWorld
Chapter 3 • General Pathology 131

Dystrophic Calcification
• Seen in dead and degenerated tissues
• Serum calcium is normal
• Seen at the sites of necrosis
• Psammoma bodies are seen
• Often causes organ dysfunction
Dystrophic calcification in dead tissues
• Caseous necrosis
• Liquefaction necrosis
• Fat necrosis
• Infarcts
• Thrombi
• Hematomas
• Dead parasites
• Calcifications in breast cancer
• Congenital toxoplasmosis
Dystrophic calcification in degenerated tissues
• Dense old scars
• Atheromas
• Monckeberg’s disease
• Cysts
• Calcinosis cutis

Metastatic Calcification
• Seen in living tissues
• Associated with elevated serum calcium
• Does not cause clinical dysfunction
• Lung is the most common site for metastatic calcification
It occurs by two mechanisms:
1. Excessive mobilization of calcium from bone
• Hyperparathyroidism
• Bony destructive lesions such as multiple myeloma
• Prolonged immobilization of the patient
2. Excessive absorption of calcium from the gut
• Hypervitaminosis D
• Milk alkali syndrome
• Hypercalcemia of infancy

MISCELLANEOUS
Antioxidants
Antioxidants may act by inhibiting the generation of free radicals or scavenging the already present free radicals.
This may be divided into enzymatic and nonenzymatic.
Enzymatic antioxidants
• Superoxide dismutase
• Catalase
• Glutathione peroxidase
Nonenzymatic antioxidants
• Vitamin E
• Cysteine and glutathione
• Albumin, ceruloplasmin, and transferrin.

https://t.me/DentalBooksWorld
132 Triumph's Complete Review of Dentistry

Chemical Fixatives
• The most common fixative used for light microscopy is 10% neutral buffered formalin (4% formaldehyde in phosphate
buffered saline).
• For electron microscopy, the most commonly used fixative is 4% glutaraldehyde.

STAINS COMMONLY USED


Substances Stain used
Glycogen Carmine (best), PAS
Lipids Sudan black, oil red “O”
Amyloid Congo red, thioflavin T and S
Calcium Alizarin red, von Kossa
Hemosiderin Perl’s stain
Trichrome Collagen appears blue, smooth muscle appears red
Apoptotic cell Annexin V
DNA and RNA Acridine orange, methyl green-pyronin
Melanin Masson Fontana

STAINS FOR MICROORGANISMS


Leprosy bacilli Wade-Fite
Fungi Grocott’s methenamine silver
Hepatitis B surface Shikata orcein
antigen

WOUND HEALING
Definition: Healing is the body’s response to injury in an attempt to restore normal structure and function.
The process of healing involves two distinct mechanisms:
1. Repair: When the healing takes place by proliferation of connective tissue elements resulting in fibrosis and scarring.
2. Regeneration: When healing occurs by proliferation of parenchymal cells and usually results in complete restoration of the original tissues.

Regeneration
• In order to maintain proper structure of tissues, these cells are under the constant regulatory control of their cell cycle.
• Cell cycle is defined as the period between two successive cell divisions and is divided into four unequal phases:
–– M phase: Phase of Mitosis
–– G1 phase (Gap 1 phase)
–– S phase: Synthesis phase where the synthesis of nuclear DNA takes place
–– G2 phase (Gap 2 phase)
–– G0 phase: Resting phase
• Some mature cells do not undergo division, while other cells complete one cell cycle in 16–24 hours.
• Depending on the capacity to divide, the cells are categorized as follows:
–– Labile cells: Under normal physiological conditions, these cells continue to multiply throughout life. E.g., Surface epithelial cells
of epidermis, alimentary tract, respiratory tract, vagina, cervix, hematopoietic cells of BM, and cells of lymph nodes and spleen.
–– Stable cells: These cells decrease or lose their ability to proliferate after adolescence but retain their capacity to divide
in response to stimuli throughout the life. E.g., Parenchymal cells of liver, pancreas, kidney, adrenal, and thyroid;
Mesenchymal cells like smooth muscle cells, fibroblasts, vascular endothelium, etc.
–– Permanent cells: These cells never multiply. They lack the ability to regenerate. E.g., Neurons of nervous system, skeletal
muscle, and cardiac muscle cells.

https://t.me/DentalBooksWorld
Chapter 3 • General Pathology 133

Repair
• Repair is the replacement of injured tissue by fibrous tissue which involves the following mechanisms.
– Granulation tissue formation
– Contraction of wounds.
• Repair consists of a combination of regeneration and scar formation by the deposition of collagen.

WOUND HEALING AND ITS MECHANISMS


• Wound: A wound is a break in the integrity of the skin or tissues, which may often be associated with disruption of the
structure and function.

Classification of Wound
Rank and Wakefield classification
1.  Tidy wounds
2.  Untidy wounds
Classification of surgical wounds
1.  Clean wound
2.  Clean contaminated wound
3.  Contaminated wound
4.  Dirty infected wound

Healing of skin wounds provides a classical example of regeneration and repair. Wound healing is accompanied by the following
mechanisms:
1.  Healing by first intention (primary union)
2.  Healing by second intention (secondary union)

Healing by Primary Union


This is regarded as the healing of wound which has the following characteristics:
• Clean and uninfected
• Surgically incised
• Without much loss of cells and tissues
• Edges of wound are approximated by surgical sutures
The sequence of events in primary union are as follows:
• Initial hemorrhage
• Acute inflammatory response
• Epithelial changes
• Organization
• Suture tracks

Mechanism of Primary Healing


Day 0 (when wound is formed)
• Presence of blood clot in the incision
Day 1 (within 24 hours)
• Neutrophilic infiltration + Blood clot
Day 2 (24–48 hours)
• Neutrophilic infiltration
• Blood clot
• Continuous thin epithelial layer

https://t.me/DentalBooksWorld
134 Triumph's Complete Review of Dentistry

Day 3
• Macrophages replace neutrophils
• Appearance of granulation tissue
• Type III collagen deposition begins
Day 5
• Abundant granulation tissue
• Collagen fibrils bridge the incision
• Neovascularization is maximum
• Full epithelial thickness with surface keratinization
End of Second Week
• Accumulation of collagen
• Fibroblast proliferation
1 month
• Replacement of collagen type III with collagen type I due to action of collagenase enzyme
• Type I collagen has the greater tensile strength
Vitamin C is required for the conversion of tropocollagen to collagen due to hydroxylation of lysine and proline residues
providing stability to collagen molecules.

Healing by Secondary Intention


• This is regarded as the healing of a wound that has the following characteristics:
–– Open wound with a large tissue defect, sometimes infected
–– Wound with extensive loss of cells and tissues
–– Wound that is not approximated and left as such opened
• The sequence of events in secondary union are as follows:
–– Initial hemorrhage
–– Inflammatory phase
–– Epithelial changes
–– Granulation tissue
–– Wound contraction
–– Presence of infection
• Repair process in secondary intention involves a combination of regeneration and scarring.
–– There is abundant granulation tissue formation.
–– At first matrix containing fibrin, plasma fibronectin, and type III collagen is formed.
–– In about 2 weeks that is replaced by a matrix composed mainly of type I collagen.
–– Wound contraction is a very important feature of secondary intention.
–– Wound contraction requires myofibroblasts.

Complications of Wound Healing


• Infection
• Implantation (epidermal) cyst
• Pigmentation
• Deficient scar formation
• Incisional hernia
• Hypertrophied scars and keloid formation
• Excessive contraction
• Neoplasia (SCC arising from Marjolin’s ulcer, which is a scar formed following burns on skin)

Wound Strength
• Wound strength is 10% after 1 week.
• It increases rapidly during next 4 weeks.
• Becomes 70% at the end of the third month.

https://t.me/DentalBooksWorld
Chapter 3 • General Pathology 135

Factors Influencing Wound Healing


Local factors
• Infection
• Poor blood supply
• Foreign bodies
• Movement
• Ionizing radiation
• Exposure to UV rays
Systemic factors
• Age
• Nutrition: Deficiency of zinc delays wound healing
• Systemic infection
• Uncontrolled diabetes
• Blood abnormalities

Scar Formation
Scar – formed as part of healing process following damage to skin as body lays down collagen fibers.
A scar localized to a wound is called a hypertrophic scar.
Features of hypertrophic scar
• Scar is confined within the wound
• Usually develops within 4 weeks after trauma
A scar that extends beyond injury site is called a keloid.
Features of keloid
• Scar grows extensively beyond wound margins
• Genetically predisposed condition
• Seen mostly in Afro-Americans
• Sites: Sternum (most common), ear lobe, face, and neck
• Usually develops after 3 months of injury

TYPES OF COLLAGEN
Collagen type Tissue distribution Genetic condition
Type I Adult hard and soft tissues, bones. Osteogenesis imperfecta
(has greatest tensile strength) Mainly in adult skin type. EDS
Type II Cartilage, intervertebral discs, vitreous Achondroplasia type II
Type III Hollow organs and soft tissues Vascular EDS
(initial collagen in wound repair)
Type IV Basement membrane collagen Alport syndrome
Type V Blood vessels and soft tissues Classic EDS
Type VI Microfibrils Bethlem myopathy
Type VII Anchoring fibrils Dystrophic epidermolysis bullosa
Type VIII Endothelial tissues Osteogenesis imperfecta
Type IX Cartilage, vitreous Stickler syndrome

https://t.me/DentalBooksWorld
136 Triumph's Complete Review of Dentistry

HEALING OF SPECIALIZED TISSUES


Healing of a Fracture
• Fracture heals either by primary union or secondary union.
Primary union: It occurs in few special situations when the ends of fracture are approximated by application of compression
clamps.
In this case, bony union takes place with the formation of medullary callus without periosteal callus formation.
Secondary union: It is the common process of fracture healing, which has the following three processes:
1. Procallus formation
2. Osseous callus formation
3. Remodeling

Complications of Fracture Healing


1. Fibrous union: A false joint may develop at the fracture site (pseudarthrosis).
2. Nonunion: It occurs if some soft tissue is interposed between the fracture ends.
3. Delayed union: Occurs in general conditions where there is infection, inadequate wound supply, poor nutrition, etc.

Wound Healing After Pulpectomy


• The healing pattern following pulpectomy is characterized by an initial inflammatory reaction in the apical tissue due to
the trauma induced by the cutting procedure.
• In the absence of wound infection, reorganization soon occurs. This involves replacement of the injured tissue by a
connective tissue derived from the periapical region.
• Materials used to fill root canals may compromise the normal healing pattern, owing to their irritating capacity, and result
in a long-standing inflammatory lesion.
• Inflammatory cells accumulate close to the root filling material and remain for as long as toxic components are released.
Eventually the material will be lined off by fibrous connective tissue.
• When overfilling occurs, the process of phagocytosis may eliminate the excess root filling material and occasionally also
the material inside the canal.

Wound Healing of Apical Periodontitis


• During periapical wound healing, the osteoblasts or mesenchymal cells line the surfaces of endosteum.
• If both cortical bone plates are destroyed by large apical periodontitis lesions, it is possible that the lesion may be repaired
with fibrous scar tissue because of extensive destruction of the periosteum beneath the oral mucosa.
• Guided tissue regeneration procedure and bone grafts is recommended to prevent ingrowth of fibroblasts from periosteum
or submucosa into the bony defect and to enhance periapical wound healing if periapical surgery is necessary.

Healing of Root Fracture


Healing of root fracture depends upon:
• Site of the fracture
• Status of the pulp
Two directions of wound healing response are expected:
• At the pulpal side
• At periodontal ligament side
• If pulp is intact at the fracture site, the odontoblastic progenitor cells will create a hard tissue bridge uniting the fractures
fragments.

https://t.me/DentalBooksWorld
Chapter 3 • General Pathology 137

The pattern of healing of root fracture are:


• Healing with calcified tissue
• Healing with interproximal calcified tissue
• Healing with interposition of bone and connective tissue
• Interposition of granulation tissue

STEM CELLS
The most widely accepted stem cell definition is a cell with a unique capacity to produce unaltered daughter cells (self-renewal)
and to generate specialized cell types (potency).
• Stem cells are located in special sites called niches.
Name of stem cell Function
Oval cells Forming hepatocytes and biliary cells
Satellite cells Differentiate into myocytes after injury
Limbus cells Stem cells of the cornea
Ito cells (stem cells of liver) Store vitamin A
Paneth cells (crypts of small intestine) Host defense against microbes

SYSTEMIC PATHOLOGY
The pancreas consists of the islets of Langerhans, which contain four major cells types – beta, alpha, delta, and PP (pancreatic
polypeptide) cells.
Cells Hormone secreted
Beta cell Insulin, amylin
Alpha cell Glucagon
Delta cells Somatostatin
PP cells Pancreatic polypeptide (vasoactive intestinal peptide, VIP)

DIABETES MELLITUS
Diabetes mellitus is a group of metabolic disorders having the feature of hyperglycemia which results from either defect in
insulin secretion, insulin action, or both.
Type 1 diabetes is characterized by an absolute deficiency of insulin secretion caused by pancreatic beta cell destruction, usually
resulting from an autoimmune attack.
Type 2 diabetes is caused by relative insulin deficiency due to combination of peripheral resistance to insulin action and an
inadequate compensatory response of insulin secretion by the pancreatic beta cell.

Pathogenesis
Type 1 DM
Genetic factors
• HLA genes (commonest locus being affected is on chromosome 6p21 (HLA D) like HLA DR3/DR4 with DQ8 haplotype)
• The non-HLA genes like that for insulin or polymorphism in CD25 (normally regulate the function of T cells)
Environmental factors
• Viral infections like Coxsackie B, mumps, rubella, or cytomegalovirus

https://t.me/DentalBooksWorld
138 Triumph's Complete Review of Dentistry

The failure of self-tolerance in T cells is the main defect in type 1 DM

The autoreactive T cells (TH1 and CD8+ cytotoxic T cells) get activated and cause beta cell injury resulting in the reduction
of beta cell mass

Type 2 DM
Insulin resistance
• It is defined as resistance to the effects of insulin on glucose uptake, metabolism, or storage. It is a characteristic feature of
most individuals with type 2 DM.
• There is no autoimmune basis of type 2 DM. The insulin is being contributed maximally by the loss of sensitivity in the
hepatocytes.
• Obesity is the most important risk factor in type 2 DM.
Clinical features of DM
1.  Insulin deficiency

Decreased cellular glucose uptake

Hyperglycemia

Glycosuria

Polyuria (decreased intracellular water – stimulation of osmotic receptors – intense thirst – polydipsia)

2.  Insulin deficiency

Increased lipolysis and proteolysis

Negative energy balance (weight loss)

Increased appetite

Polyphagia

Diagnosis
• Random plasma glucose concentration of 200 mg/dL or higher with classical signs and symptoms
• Fasting glucose >126 mg/dL or higher on more than one occasion
• An abnormal Oral Glucose Tolerance Test (OGTT), in which the glucose concentration is 200 mg/dL or higher 2 hours after a
standard carbohydrate load (75 g of glucose)
• A level of glycated hemoglobin (HbAc1c) >6.5 g/dL (additional criteria for diagnosis of DM by American Diabetic association)

https://t.me/DentalBooksWorld
Chapter 3 • General Pathology 139

THYROID GLAND
It is a gland weighing 15–20 g responsible for the secretion of the thyroid hormones (T3 and T4) and calcitonin.

Clinical Features
Common symptoms of hypothyroidism include
• Fatigue
• Weight gain
• Inability to lose weight with diet and exercise 
• Constipation 
• Infertility
• Feeling cold
• Hair loss (including the outer edge of the eyebrows) 
• Brain fog (lack of mental clarity)
• Muscle and joint pains/aches
Common symptoms of hyperthyroidism include
• Anxiety
• Insomnia
• Panicky feeling 
• Tremors
• Exaggerated reflexes
• Elevated heart rate
• Diarrhea or loose stools
• Feeling overheated 
• Unexplained weight loss

Assessment of Thyroid Gland


Disorder T4 T3 TSH
Primary hypothyroidism D D I
Primary hyperthyroidism I I D
Subclinical hypothyroidism N N I
Subclinical hyperthyroidism N N D
TSH secreting pituitary tumor I I I
Sick euthyroid/secondary hypothyroidism D D D

*I – Increase
*D – Decrease

Hashimoto thyroiditis
• Autoimmune thyroiditis
• Most common type of thyroiditis and the most common cause of hypothyroidism in areas having sufficient iodine levels
• The fibrosis does not extend beyond capsule (which is the main differentiating feature from Riedel thyroiditis)
• Hurthle cells – Cells with abundant eosinophilic and granular cytoplasm

https://t.me/DentalBooksWorld
140 Triumph's Complete Review of Dentistry

Subacute painless lymphocytic thyroiditis


• Subacute painless lymphocytic thyroiditis – develops postpartum and progression to hypothyroidism
• There is absence of Hurthle cells and fibrosis (Differentiating feature from Hashimoto thyroiditis)

Subacute/granulomatous/De Quervain thyroiditis


• HLA-B5
• Most common cause of painful thyroiditis
• Virus induced
• No cervical lymphadenopathy

Riedel’s Thyroiditis
• Also known as fibrous thyroiditis or invasive thyroiditis
• Fibrous tissue replacement of gland and surrounding tissue

• Thyroid acropachy is digital swelling and clubbing of fingers in Graves disease


• In Graves disease, there is crowding of cells with papillae formation without fibrovascular core (presence of the latter is the
differentiating feature of papillary thyroid cancer)
• Exophthalmos and pretibial myxedema are not seen in toxic multinodular goiter
• Presence of a hyperfunctioning nodule developing in MNG is known as Plummer syndrome

Thyroid Carcinomas
Most common thyroid carcinoma Papillary
Least common thyroid carcinoma Anaplastic
Least malignant Papillary
Most malignant Anaplastic
Most common cancer after radiation Papillary
Cancer developing in Hashimoto’s thyroiditis Lymphoma
Thyroid cancer developing in long standing multinodular goiter Follicular, anaplastic (rare)
Type of thyroid carcinoma in MEN syndrome Medullary
Thyroid cancer associated with amyloidosis Medullary
Psammoma bodies seen in Papillary
Orphan – Annie eyed nuclei seen in Papillary
Thyroid carcinoma associated with dystrophic calcification Papillary
Carcinoma derived from “C” cell of thyroid Medullary
Carcinoma developing in thyroglossal tract Papillary
Spread by lymphatic route Papillary carcinoma
Excellent prognosis Papillary
Positive Iodine 131 uptake Papillary
Caused due to postradiation in head and neck Papillary
Mutation in tyrosine kinase receptors RET or NTRK1 (Neurotrophic tyrosine kinase Papillary
receptor 1) BRAF oncogene
Mutation in RAS oncogene particularly - N-RAS, specific translocation associated with Follicular
follicular cancer t(2;3)
Mutation in RET proto-oncogene Medullary
Mutation in the p53 tumor suppressor gene Anaplastic
Hurtle cells seen in Follicular
Presence of capsular invasion and vascular invasion Follicular
Arises from parafollicular cells/C cells and secretes calcitonin Medullary
Worst prognosis Anaplastic carcinoma
Undifferentiated thyroid cancer Anaplastic carcinoma

https://t.me/DentalBooksWorld
Chapter 3 • General Pathology 141

PARATHYROID GLAND
These are four glands situated near the thyroid gland and are composed of chief cells (containing PTH granules) and oxyphil
cells (containing glycogen).
Hyperparathyroidism
Most common cause – parathyroid adenoma
Adenomas are more commonly located in the inferior parathyroid gland
Three Types:
Primary
• PRAD 1 proto oncogene on chromosome 11 causes overexpression of cyclin D1 resulting in proliferation of the parathyroid cells
• MEN 1 suppressor gene on 11 q 13
Secondary
• It is seen in renal failure (most common cause), vitamin D deficiency, steatorrhea, and nutritional deficiency
Tertiary
• Autonomous excessive parathyroid activity even when serum calcium is increased is known as tertiary hyperparathyroidism
which is usually managed by parathyroidectomy
Clinical Features (Bone, Serum, and X-ray)
• Recurrent nephrolithiasis
• Peptic ulceration
• Extensive bone resorption
• Mental changes
• Elevated PTH
• Asymptomatic hypercalcemia
• Osteitis fibrosa cystica
• Absence of lamina dura
• Pinhead stippling of skull
• Subperiosteal bone resorption of pharyngeal clefts
• Brown tumor – Osteoporosis/deformity/pathological fracture
• Salt pepper/pepper pot appearance
• Rugger-jersey spine: Band like osteosclerosis of superior/inferior margins of vertical body. Seen in CRF due to secondary
hyperparathyroidism or renal osteodystrophy
Hyperparathyroidism Serum calcium Serum phosphate Alkaline phosphatase PTH
Primary I – Increase D – Decrease I – Increase I – Increase
Secondary D – Decrease I – Increase or D I – Increase I – Increase
Tertiary I – Increase D – Decrease I – Increase I – Increase

Hypoparathyroidism
• Low calcium/high phosphate (hyperphosphatemia)
25 hydroxycholecalciferol

25(OH)-D-1 Hydroxylase
1,25 Dihydroxycholecalciferol (active vitamin D)
• So if PTH is decreased, there will be no conversion
• Albrights osteodystrophy is not seen here
• If PTH is decreased, then there will be decreased mobilization of calcium from bone, resulting in hypocalcemia
• Decreased PTH/normal PTH infusion response

https://t.me/DentalBooksWorld
142 Triumph's Complete Review of Dentistry

Pseudohypoparathyroidism
Hereditary disorder associated with signs and symptoms of hypoparathyroidism (i.e., decreased calcium + increased
phosphate) but with elevated PTH levels
Clinical Features:
• Increased PTH
• Decreased calcium
• Increased phosphate
• Decreased response of urinary cAMP to PTH
• Round face
• Short stature
• Cataract
• Brachydactyly
• Short 4th, 5th metacarpals
• Exostosis
• Impairment in olfaction and taste
• Obesity

Pseudopseudohypoparathyroidism (PPHP)
It refers to the subset of patients who carry the abnormal GNAS1 mutation (GS ALPHA Subunit deficiency, with Albright
hereditary osteodystrophy, but no endocrine/biochemical changes or altercations)

HEPATOLOGY HEPATITIS – AT A GLANCE


Hepatitis A Hepatitis B Hepatitis C Hepatitis D Hepatitis E
What is hepatitis virus disease?
HAV mainly affects HBV is the second HCV is the most HDV occurs as a HEV is the most
children. It does not most common cause common cause of co-infectious disease common hepatitis
have a fatal course. It of chronic hepatitis. chronic hepatitis. It along with HBV and it all over the world in
is the most common It causes liver cell causes cirrhosis and causes superinfection adults and it has no
hepatitis in children all damage, leading to cancer. of those patients who chronicity.
over the world. It does cirrhosis and cancer. were infected with
Most common cause of
not lead to chronic HBV.
Most common cause of fulminant hepatitis in
disease.
fulminant hepatitis. Highest incidence of pregnancy.
fulminant hepatic
failure.
Most common
hepatitis all over India
is Hepatitis A.
Chronic hepatitis
and carriers is seen in
all except hepatitis
A and E.
Single stranded RNA Double stranded DNA RNA RNA RNA
42 nm size
What is its incubation period?
15–50 days. 3–6 months. 2–25 weeks. 2–8 weeks. 2–9 weeks.
Approximately 30 Approximately 120 Approximately 7–9 Approximately 40
days. days. weeks. days.

https://t.me/DentalBooksWorld
Chapter 3 • General Pathology 143

How is it spread?
Transmitted by fecal/ Mainly via hetero Contact with infected Contact with infected Transmitted through
oral route, through sexual. Contact blood, contaminated blood, contaminated fecal/oral route.
close person-to-person with infected blood, IV needles, razors, and needles. Sexual contact Outbreaks associated
contact, or ingestion seminal fluid, tattoo/body piercing with HDV-infected with contaminated
of contaminated food vaginal secretions, tools. Infected mother person. water supply in other
and water. contaminated needles, to newborn. Not easily countries. Not spread
including tattoo/body spread through sex via blood transfusion
piercing tools. Infected and via breast milk. and sexual contact.
mother to newborn.
Symptoms
May have none. Adults May have none. Some Even fewer acute cases Same as HBV. Same as HBV.
may have light stools, persons have mild seen than any other
dark urine, fatigue, flu-like symptoms, hepatitis. Otherwise
fever, and jaundice dark urine, light stools, same as HBV.
jaundice, fatigue, and
fever.
Treatment of chronic disease
No treatment. Antivirals with varying Interferon and Interferon with No treatment.
success. combination therapies varying success.
with varying success.
Vaccine
Two doses of vaccine Three doses may be None. HBV vaccine prevents None.
to anyone over the age given to persons of any HDV infection.
of 2. age.
Who is at risk?
Household or sexual Infant born to infected Anyone who had a IV drug users, Travelers to developing
contact with an mother, having sex blood transfusion homosexual men and countries, especially
infected person or with infected person or before 1992; health those having sex with pregnant women.
living in an area multiple partners, IV care workers, IV drug an HDV infected
with HAV outbreak. drug users, emergency users, hemodialysis person.
Travelers to developing responders, health care patients, infants born
countries, homosexual workers, homosexual to infected mother,
men, and IV drug men, and hemodialysis and multiple sex
users. patients. partners.
Prevention other than vaccine
Immune globulin Immune globulin Safe sex. Clean up Hepatitis B vaccine to Avoid drinking or
within 2 weeks of within 2 weeks of spilled blood with prevent HBV infection. using potentially
exposure. Vaccination. exposure. Vaccination bleach. Wear gloves Safe sex. contaminated water.
Washing hands with provides protection when touching blood.
soap and water after for 18 years. Safe sex. Do not share razors or
going to the toilet. Clean up infected toothbrushes.
Use household bleach blood with bleach and
to clean surfaces wear protective gloves.
contaminated with Do not share razors,
feces, such as changing toothbrushes, needles.
tables. Safe sex.

https://t.me/DentalBooksWorld
144 Triumph's Complete Review of Dentistry

Prognosis
Excellent Poor Poor Good Poor

Sequential Appearance of Hepatitis B Markers and Significance


HBsAg Present in acute disease
Continuous presence indicates chronic disease or carrier state
HBeAg Seen with active viral replication and denotes high infectivity
HBV DNA
DNA polymerase
IgM anti-HBc Antibody detectable shortly before onset of symptoms
Marker of window period
IgM anti-HBc is an indicator of recent disease whereas IgG anti-HBc is seen with chronic or prior infection
Anti-HBeAb Detected after HBeAg disappears and denotes low infectivity
IgG anti-HBs Appears after disappearance of HBsAg
Provides protection against Hepatitis B and indicates immunity
Seen in prior infection and in a vaccinated person

Hepatitis B Serology
HBsAg IgM anti-HBc IgG anti-HBc IgG anti-HBs
Acute HBV infection + + − −
Window period − + − −
Chronic infection + +/− + −
Prior infection − − + +
Immunization − − − +

Crohn’s Disease
• It is a chronic granulomatous disease which can affect any part of the gut from the esophagus to the large intestine, but the
most commonly affected part is the small intestine, particularly the ileum
• So it is called “terminal ileitis” or “granulomatous colitis”
• It is associated with HLA-DR 1/DQw5 and NOD2 genes and an abnormal T-cell response particularly, CD4+ T cells (TH1 cells)
• Important features of Crohn’s disease
–– Skip lesions
–– Ileum
–– Saccharomyces cerevisiae antibody present
–– Transmural involvement
–– Extra fibrosis and fistula formation (as compared to ulcerative colitis)
• Radiological sign – String sign of Kantor, Rectum is usually spared

Ulcerative Colitis
It is a chronic inflammatory condition affecting the colon. It most commonly starts from the rectum and affects the superficial
layers, the mucosa, and the submucosa (muscularis propria is rarely affected). It is associated with HLA-DR2, polymorphism
in IL-10 gene, and an abnormal T-cell response particularly of CD4+ cell (TH2 cells).
Important features
• Ulcers in mucosa and submucosa (muscle layer not affected)
• Continuous retrograde involvement (no skip lesions)
• Originates in the rectum
• Lead pipe appearance
• Increased chances of cancer (more than that of Crohn’s disease)
• Toxic megacolon (due to involvement of transverse colon)
• Increased growth from the mucosa (Pseudopolyps)
• Symptoms are severe (as compared to Crohn’s disease)

https://t.me/DentalBooksWorld
Chapter 3 • General Pathology 145

Nephrotic Syndrome
Clinical presentation that is characterized by a triad of heavy proteinuria (>3.5 g/day) – edema and hypoalbuminemia

Nephritic Syndrome
Nephritic syndrome is a clinical presentation that is characterized by hematuria, proteinuria, oliguria, and hypertension
Etiology
• Postinfection with nephritogenic strains of Group A beta-hemolytic streptococcus (typically occurs in children).
• Any of the other causes of glomerulonephritis:
–– Other bacterial infections – e.g., typhoid, secondary syphilis, methicillin-resistant Staphylococcus aureus (MRSA)
infection, pneumococcal pneumonia, infective endocarditis
–– Viral infections – e.g., hepatitis B, mumps, measles, infectious mononucleosis, varicella, Coxsackievirus
–– Parasitic infections – e.g., malaria, toxoplasmosis
–– Multisystem systemic diseases – e.g., systemic lupus erythematosus (SLE), vasculitis, Henoch–Schönlein purpura,
Goodpasture’s syndrome, Wegener’s granulomatosis
–– Primary glomerular diseases – e.g., Berger’s disease (IgA nephropathy), membranoproliferative glomerulonephritis
–– Guillain–Barré syndrome
–– Diphtheria–pertussis–tetanus vaccine
Clinical Features
The key clinical features of acute nephritic syndrome are:
• Hematuria
• Reduced urine output
• Fluid retention and edema (including periorbital, pedal, and pulmonary edema)
• Proteinuria (usually <3.5 g/day)
• Hypertension
• Uremic symptoms (including anorexia, pruritus, lethargy, nausea)
• Deteriorating renal function

Acute Glomerulonephritis (GN)


• Acute glomerulonephritis following streptococcal infection is characterized by the sudden appearance of hematuria,
proteinuria, red blood cell casts in the urine, edema, and hypertension with or without oliguria
• Poststreptococcal GN is a form of glomerulonephritis. It is caused by an infection with a type of streptococcus bacteria. The
infection does not occur in the kidneys, but in a different part of the body, such as the skin or throat.
• The strep bacterial infection causes the tiny blood vessels in the filtering units of the kidneys (glomeruli) to become
inflamed. This makes the kidneys less able to filter the urine.
• Poststreptococcal GN is uncommon today because infections that can lead to the disorder are commonly treated with
antibiotics. The disorder may develop 1–2 weeks after an untreated throat infection, or 3–4 weeks after a skin infection.
• It may occur in people of any age, but it most often occurs in children of ages 6 through 10. Although skin and throat
infections are common in children, poststreptococcal GN is a rare complication of these infections.
Risk factors include:
• Strep throat
• Streptococcal skin infections (such as impetigo)
Symptoms
Symptoms may include any of the following:
• Decreased urine output
• Rust-colored urine
• Swelling (edema), general swelling, swelling of the abdomen, swelling of the face or eyes, swelling of the feet, ankles, hands
• Visible blood in the urine
• Joint pain
• Joint stiffness or swelling

https://t.me/DentalBooksWorld
146 Triumph's Complete Review of Dentistry

RESPIRATORY TRACT
• Acinus is the functional unit of lung whereas alveoli are the chief sites of gaseous exchange.
• Lobule is composed of 3–5 terminal bronchioles with their acini.
• Alveoli are lined by type I pneumocytes (forming 95% of alveolar surface) and type II pneumocytes (responsible for secretion
of surfactant and repair of alveoli after type I pneumocyte destruction).
• The alveolar wall has the presence of pores of Kohn for allowing the passage of bacteria and exudate between adjacent alveoli.
• The entire respiratory tract is lined by pseudostratified, tall, ciliated columnar epithelial cells except vocal cords (these have
stratified squamous epithelium).
• Broadly, the diseases of lung may be divided into infectious, obstructive, restrictive, vascular, and neoplastic etiology.

Pneumonia
• Pneumonia is an infection of the lungs that can be caused by viruses, bacteria, and fungi. A common cause of bacterial
pneumonia is Streptococcus pneumoniae.
• Other bacteria can cause pneumonia, including Mycoplasma pneumoniae, Chlamydia pneumoniae, Chlamydia psittaci, and
Legionella pneumophila.
• These bacteria are referred to as “atypical” because pneumonia caused by these organisms might have slightly different
symptoms, appear different on a chest X-ray, or respond to different antibiotics than the typical bacteria that cause
pneumonia. Even though these infections are called “atypical,” they are not uncommon.

TWO TYPES OF TYPICAL PNEUMONIA


Lobar Pneumonia
Consolidation of entire lobe usually caused by Streptococcus pneumoniae.
Following four stages of inflammation are present:
1. Congestion: It is due to vasodilation. There is bacteria rich intra-alveolar fluid.
2. Red hepatization: Exudate is rich in RBC, neutrophils, and fibrin.
3. Gray hepatization: Degradation of RBC and fibrinosuppurative exudates.
4. Resolution: Enzymatic degradation of exudate and healing. Chest X-ray shows opacification of the entire lobe.

Atypical Pneumonia
• Infection caused by intracellular organisms like Mycoplasma, Chlamydia pneumonia, and viruses like RSV, influenza virus,
rhinovirus.
• Characterized by lymphocytic infiltration and presence of alveolar septal and interstitial inflammation with absence of
alveolar exudates.
• Clinical features include fever, headache, dry cough, and myalgia.
• Productive cough and pleural involvement is uncommon.
• Viral pneumonia result in interstitial infiltrates (therefore called interstitial pneumonia) and may result in variety of
cytopathic effects; e.g., RSV shows bronchiolitis and multinucleate giant cells and CMV and herpes show inclusion bodies.
Common Types of Pneumonia Caused by Atypical Bacteria
• Chlamydia pneumoniae infection
This illness, caused by Chlamydia pneumoniae, is most common in school-aged children and usually develops into mild
pneumonia or bronchitis.
• Legionnaires’ disease
Legionnaires’ disease is a severe type of pneumonia that is caused by a type of bacteria called Legionella.
• Mycoplasma pneumoniae infection
This generally mild illness is a common cause of “walking pneumonia.”
• Psittacosis
People get this sometimes serious illness, caused by Chlamydia psittaci, from infected pet birds (parrots, parakeets) and
poultry (turkeys, ducks).

https://t.me/DentalBooksWorld
Chapter 3 • General Pathology 147

TUBERCULOSIS (KOCH’S DISEASE)


• Pulmonary tuberculosis is caused by droplet infection (coughing, sneezing, etc.) due to Mycobacterium tuberculosis.
• It is a strict aerobic bacteria for which the reservoir of infection is a human being with active tuberculosis.
• However, certain clinical conditions can increase the risk of tuberculosis like diabetes mellitus, Hodgkin’s lymphoma,
chronic lung disease (particularly silicosis), chronic renal failure, malnutrition, alcoholism, and immunosuppression.
Concept
• Infection with M. tuberculosis is different from disease.
• Infection is the presence of organisms, which may or may not cause clinically significant disease.
• In most of the people, primary tuberculosis is asymptomatic though it may be associated with fever and pleural effusion.
• Infection with M. tuberculosis typically leads to the development of delayed hypersensitivity to M. tuberculosis antigens,
which can be detected by the tuberculin (Mantoux) test.
• A positive tuberculin test result signifies cell-mediated hypersensitivity to tubercular antigens but does not differentiate
between infection and disease.
Pathogenesis
• The bacteria enter macrophages by endocytosis and multiply there.
• The bacterial cell wall glycolipid lipoarabinomannan blocks the fusion of the phagosome and lysosome.
• After about 3 weeks of infection, the TH1 cells are stimulated by mycobacterial antigens and these cells differentiate into
mature TH1 cells by the action of IL-12.
• The mature THI cells in the lymph nodes and lung produce IFN-7 which activates macrophages leading to oxidative
damage to the mycobacteria.
• Activated macrophages produce TNF and recruit monocytes which then differentiate into the “epithelioid histiocytes,”
a characteristic feature of granulomatous inflammation.
• The immune response is usually accompanied by hypersensitivity and tissue destruction.
Clinical Features
Primary tuberculosis
• It develops in a previously unexposed and unsensitized individual.
• The source of the organism is usually exogenous.
• Most patients with primary tuberculosis develop latent disease while a minority develops progressive infection.
• Primary tuberculosis almost always begins in the lungs leading to formation of a subpleural lesion called Ghon’s focus.
During the first few weeks, there is also lymphatic and hematogenous dissemination to other parts of the body.
• At times, occult hematogenous spread occurs in primary TB where the focus is then called Simon focus.
• In majority of the people, development of cell-mediated immunity controls the infection.
• The Ghon’s complex undergoes progressive fibrosis and calcification (detected radiologically and called Ranke complex).
Secondary tuberculosis
• It is the pattern of disease that arises in a previously sensitized host.
• It usually results from a reactivation of latent primary lesions after many years of an initial infection, particularly when host
immunity is decreased or uncommonly may follow primary tuberculosis.
• The pre-existence of hypersensitivity contributes to localization of the infection and increased incidence of cavitation.
• This can lead to erosion into an airway (leading to spread of bacilli during coughing).
Miliary tuberculosis
• It occurs when organisms drain through lymphatics and blood vessels to the different organs of the body resulting in small
yellow–white consolidated lesions.
• Miliary tuberculosis is most prominent in the liver, bone marrow, spleen, adrenals, meninges, kidneys, fallopian tubes, and
epididymis.

https://t.me/DentalBooksWorld
148 Triumph's Complete Review of Dentistry

Chronic Bronchitis
• It is defined clinically as the presence of productive cough for at least 3 months in at least 2 consecutive years in the absence
of any other identifiable cause.
• The most important initiating agent is smoking resulting in airway irritation leading to mucous hypersecretion; the latter
may cause airway obstruction.
• Infection plays a secondary role particularly in maintaining chronic bronchitis and is also responsible for the acute
exacerbations.
Histologic features:
• Chronic lymphocytic infiltration of the airways and submucosal gland hypertrophy.
• There is also increase in Reid index.
• The bronchial epithelium may also have squamous metaplasia and dysplasia.
Clinical features:
• Late onset of dyspnea with productive cough (copious sputum), recurrent infections, hypoxemia, and mild cyanosis
(blue bloaters). Long-standing chronic bronchitis can cause cor pulmonale (right-sided heart failure due to pulmonary
hypertension).

Emphysema
It is abnormal permanent enlargement of the airspace distal to terminal bronchioles and is associated with destruction of
their walls.
Characteristically, there is loss or reduction of elastic recoil of the lung.
• Most important etiological agent for emphysema is smoking which causes inflammation in airways resulting in increased
neutrophils and macrophages.
These inflammatory cells release elastase responsible for destruction of lung tissue resulting in emphysema.
• Normally, the pulmonary tissue destruction by elastase is prevented by the presence of anti-elastase activity which is
primarily due to cq-antitrypsin; UI-AT (mainly) and serum (11-macroglobulin).
So, any increase in neutrophils (usually in smokers) or deficiency of alpha-AT would contribute to the development of
emphysema.
Clinical features: Progressively increasing dyspnea, weight loss, late onset of cough with scanty sputum.
The patient is noncyanotic, uses accessory muscle of respiration, and shows pursed lip breathing (pink puffers).
Management: Cessation of smoking and use of bronchodilators is the mainstay of the management.

Asthma
Hyperactivity of the airways resulting in reversible bronchoconstriction and air flow obstruction on exposure to some
external stimuli is called asthma.
Pathogenesis: Primary exposure of an allergen causes T-H2 cell dominated inflammatory response resulting in IgE production
and eosinophil recruitment (called sensitization).
• Exposure to the same allergen causes cross-linking of IgE bound to IgE receptors on mast cells in the airways which cause
opening up of epithelial cells due to released mediators.
• Antigens then cause activation of mucosal mast cells and eosinophils and this along with neuronal reflexes (subepithelial
vagal receptors) cause bronchospasm, increased vascular permeability and mucus production (acute or immediate response).
• Later on, leucocytic infiltration causes release of more mediators and damage to the epithelium (late phase reaction).
Clinical features: Acute asthmatic attack is characterized by wheezing, cough, and severe dyspnea.
Morphology: The most striking macroscopic finding is occlusion of the bronchi and bronchioles by mucus plugs.

Histology of asthma
There are numerous eosinophils, Charcot–Leyden crystals (composed of eosinophil membrane protein called galectin-10),
and Curschmann spirals (whorls of shed airway epithelium).

https://t.me/DentalBooksWorld
Chapter 3 • General Pathology 149

Structural changes in the bronchial wall called “airway remodeling” is characterized by presence of eosinophilic
inflammation and edema of bronchial walls, increased size of submucosal glands, hypertrophy of bronchial wall smooth
muscle, and deposition of subepithelial collagen in the bronchial wall.
Individual epithelial cells present in the sputum of the patients are called Creola bodies.
Important facts in asthma
• IL-13 gene polymorphism is strongly associated with bronchial asthma.
• ADAM-33 is another gene causing proliferation of smooth muscle cells and fibroblasts in bronchi resulting in bronchial
hyperreactivity and subepithelial fibrosis.

QUICK FACTS

• Eosin is acidophilic and stains positively charged structures in the cell (e.g., mitochondria)
• Hematoxylin is basic and stains negatively charged structures (DNA and RNA)
• Caspase-dependent programmed cell death is called apoptosis
• Caspase-independent programmed cell death is called necroptosis
• Fatty change in myocardium is known as tigered effect
• Lipofuscin – Wear and tear pigment
• Presence of lipofuscin is a tell-tale sign of free radical injury
• Lung is the most common site for metastatic calcification
• Calcification begins in mitochondria in all the organs except kidney
• In kidney, calcification begins from basement membrane
• Most destructive free radical is hydroxyl (OH–)
• Vitamin C is the best neutralizer of hydroxyl free radicals
• Zone III hepatocytes in liver are most susceptible to hypoxic injury
• Prions – Misfolded proteins
• Chaperons – Prevent protein misfolding
• Decreased basophilia is one of the features of irreversible cell injury
• CD 95 is the molecular marker for apoptosis
• Sex steroids are the inhibitors of apoptosis
• Breast at puberty comes under hyperplasia
• Breast during lactation comes under – Hypertrophy
• Neurons are the most sensitive cell to hypoxic injury in the brain
• Regulation gene (BCL-2) for apoptosis is located on chromosome 18
• A defect in DNA helicase enzyme results in premature ageing, known as Werner syndrome
• Excessive activity of telomerase enzyme is associated with cancers
• Chromatin condensation is the hall mark feature of apoptosis
• Gamma Gandy bodies are seen in congestive splenomegaly
• Melanin is an endogenous brown and black pigment

WOUND HEALING
• Neutrophilic infiltration occurs within 24 hours of injury.
• Continuous thin epithelial layer is formed in day 2.
• Neutrophils are replaced by macrophages on day 3.
• Neovascularization is maximum on day 5.
• The predominant collagen in adult skin type is type I.
• In early granulation tissue, the predominant collagen are of types III and I.

https://t.me/DentalBooksWorld
150 Triumph's Complete Review of Dentistry

DIABETES MELLITUS
• Glycosylated hemoglobin A1C (HbA1C) is formed due to nonenzymatic attachment of glucose with globin component of
hemoglobin
• It is used for diagnosis as well as a marker of glucose control in diabetes
• Its target level during the treatment of DM < 7%
• The most important stimulus that triggers insulin synthesis and release is glucose itself
• Since both insulin and C-peptide are secreted in equal amounts, equimolar quantities after physiologic stimulation, C-peptide
levels are used as a marker for endogenous insulin secretion
• The presence of islet cell antibodies is used as a predictive marker for type 1 DM
• The genetic factors are much more important in type 2 DM than in type 1 DM
• The presence of islet amyloid protein (amylin) is a characteristic feature of long-standing type 2 DM. There is no insulitis in
type 2 DM (which is characteristically seen in type 1 DM)
• A family of proteins called sirtuins, identified to be involved in aging are now implicated in diabetes. Sirt-1 improves glucose
tolerance, enhance beta cell insulin secretion, and increase production of adiponectin
• Honeymoon period is the symptom-free interval period in a patient of DM in which the individual is asymptomatic. It is due
to the beta reserve cell mass in the pancreatic islets
• Diabetic ketoacidosis is an important complication seen in type 1 DM. Type 2 DM patients do not develop diabetic ketoacidosis
– the fat sparing effect of insulin prevents the formation of ketone bodies by inhibiting the fatty oxidation in the liver
• Diabetic nephropathy – Most characteristic lesion – Nodular glomerulosclerosis or Kimmelstiel–Wilson lesion – PAS positive
nodules
• Most common lesion – Diffuse glomerulosclerosis
• Dawn phenomenon – Is an early morning rise in plasma glucose requiring increased amounts of insulin to maintain
euglycemia

Somogyi effect – Is rebound hyperglycemia in the morning because of counterregulatory hormone release after an episode of
hypoglycemia in the middle of the night.

ENDOCRINE DISORDERS
Thyroid disorders
• Graves disease is the commonest cause of thyrotoxicosis
• Thyrotoxicosis factitia is an exogenous thyroid hormone induced hyperthyroidism
• The cardiac manifestations are the earliest and most consistent feature of hyperthyroidism
• Serum TSH is the best screening test for thyroid dysfunction
• Autoimmune hypothyroidism is the commonest cause of hypothyroidism in iodine sufficient areas of the world

Adrenal disorders
• Layers of adrenal cortex from outside to inside: glomerulosa, fasciculata, and reticularis
• Administration of exogenous corticosteroids is the commonest cause of Cushing’s syndrome
• Pituitary Cushing – Increased ACTH, increased cortisol
• Adrenal Cushing – Decreased ACTH, increased cortisol
• Ectopic Cushing – Increased ACTH, increased cortisol
• In Cushing’s syndrome, there is presence of light basophilic material due to accumulation of intermediate keratin filaments in
the cytoplasm called Crooke hyaline change in the pituitary
• Cushing’s syndrome
• Primary hypersecretion due to increased ACTH (also called Cushing disease), seen in women of 20–30 years due to an ACTH
producing microadenoma
• Adrenal oversecretion due to adenomas or carcinomas (adrenal Cushing’s syndrome)

https://t.me/DentalBooksWorld
Chapter 3 • General Pathology 151

• Primary aldosteronism – Diastolic hypertension is present and there is decreased renin secretion
• Secondary aldosteronism – Diastolic hypertension is absent and there is increased renin secretion
• Tuberculosis is the most common cause of Addison’s disease in India

INFECTIONS
• Microscopically, “HBsAg” is responsible for “ground glass” hepatocytes whereas “HBcAg” gives “sanded nuclei” appearance
• Most useful indicator of prior infection with HBV is Anti HBcAg
• Mallory–Weiss tears: These are mucosal tears in the esophagogastric junction or the gastric cardiac mucosa caused due to
vigorous vomiting usually seen in alcoholics
• In the most of the cases (90%), the tear is present immediately below the squamocolumnar junction at the cardia whereas in
10%, it is present in esophagus
• Barrett’s esophagus is the most important risk factor for the development of esophageal adenocarcinoma
• Barrett’s esophagus is the metaplastic change in the esophageal lining in which normal squamous epithelium is changed to
columnar epithelium due to prolonged gastroesophageal reflux disease (GERD). It is classified as long segment (if >3 cm is
involved) or short segment (if <3 cm is involved)
• Microscopically, esophageal squamous epithelium is replaced by columnar epithelium
• Definite diagnosis is made only when columnar mucosa contains the intestinal goblet cells
• Barrett’s ulcer is the ulcer in the columnar lined portion of the esophagus
• Triad of Plummer–Vinson syndrome = iron deficiency anemia + esophageal webs + glossitis
• The investigation of choice in esophageal cancer is endoscopy and biopsy
• Gold standard – Staining of H. pylori with silver stain or Warthin–Starry stain
• Most specific investigation – culture of bacteria (done on Skirrow’s medium)
• Humans are the only known host of H. pylori
• H. pylori gastritis causes involvement of the antrum
• Autoimmune gastritis causes the involvement of the fundus and the body
• Histology remains the gold standard for detection of H. pylori
• The special stains used for H. pylori include nonsilver stains (like Giemsa, Diff-Quik, Gimenez, Acridine orange) and silver
stains (like Warthin–Starry, Steiner)
• H. pylori is the most important cause of peptic ulcer
• Gastroduodenal artery is the source of the bleeding in duodenal ulcer whereas left gastric artery bleeds in gastric ulcer
• Urea breath test is used to ensure the efficacy of the treatment for peptic ulcer disease
• Menetrier disease – It is characterized by diffuse foveolar cell hyperplasia due to excessive secretion of TGF-alpha. It is
associated with enlarged gastric rugae and protein losing enteropathy
• Infection with H. pylori is associated with distal intestinal type and not with diffuse, proximal carcinoma
• Diffuse involvement of the stomach in cancer is called linitis plastica or “leather bottle” appearance of the stomach. It is also
seen in metastasis from cancers of breast and lung
• Metastasis to anterior left axillary lymph node is called Irish nodes
• Cushing ulcer is seen in esophagus, stomach, or the duodenum and is associated with intracranial disease or head injury. It is
caused by intracranial disease or head injury
• It is caused by gastric acid hypersecretion due to vagal nuclei stimulation
• Curling ulcer is seen in proximal duodenum and is associated with burns or trauma
• It is caused due to reduced blood supply and systemic acidosis in burns or trauma

WOUND HEALING
• Wound strength will never reach 100%.
• Zinc is a co-factor in collagenase.
• Zinc deficiency is associated with impaired wound healing.
• Infections are the most common cause of impaired wound healing.

https://t.me/DentalBooksWorld
152 Triumph's Complete Review of Dentistry

• Granulation tissue is the hall mark of the fibrogenic repair.


• The chief cell responsible for scar contraction is a myofibroblast.
• Sternum is the most common site for keloid formation.
• Intralesional steroids (triamcinolone) are the usual drugs for the management of a keloid.
• Stem cells appear in human embryo at about the third week.
• Glucocorticoids delay wound healing by inhibiting collagen synthesis and anti-inflammatory effect.
• Neuronal stem cells are oligopotent stem cells.
• Blanching at wound site occurs during the second week.
• Remodeling of connective tissues is carried out by matrix metalloproteinases (MMPs).

MULTIPLE CHOICE QUESTIONS

CELL INJURY AND ADAPTATION


1. Stem cell research consists of
A. Human cells grown in vitro B. Plant cells grown in vitro
C. Cadaver cells grown in vitro D. Synonymous with PCR
2. Human genome project was completed in
A. 2004 B. 2003
C. 2002 D. 2001
3. Methods of cell proliferation analysis except
A. Flow cytometry B. Microspectrophotometry
C. Immunohistochemistry D. PCR
4. In the cell besides nucleus having DNA is
A. Ribosome B. Golgi apparatus
C. Mitochondria D. Endoplasmic reticulum
5. Actin and myosin proteins are found in
A. Microtubules B. Microfilaments
C. Intermediate filaments D. Ribosomes of muscle
6. Which of the following is true about ischemic–reperfusion cell injury?
A. Increased Ca ions in the extracellular fluid B. Increased Ca ions in the cytosol
C. Ca ions are equal in the cytosol and in extracellular fluid D. Ca ion equilibrium is unaffected
7. Major mechanism of plasma membrane damage in ischemia is
A. Reduced intracellular pH B. Increased intracellular accumulation of Na
C. Increased calcium ions in the cytosol D. Reduced aerobic respiration
8. Most reactive free radical species is
A. O2’ B. H2O2

C. OH D. NO
9. Which one of the following is not involved in fatty liver due to chronic alcoholism?
A. Increased free fatty acid synthesis B. Decreased triglyceride utilization
C. Increased α-glycerophosphate D. Block in lipoprotein excretion
10. Enzymatic digestion is the predominant event in
A. Coagulative necrosis B. Liquefactive necrosis
C. Caseous necrosis D. Fat necrosis
11. The protein that plays an important role in the mechanism of apoptosis is
A. Receptor for TNF B. BCL-2
C. TP53 D. CED-9

https://t.me/DentalBooksWorld
Chapter 3 • General Pathology 153

12. Diabetic foot evolves as


A. Dry gangrene B. Wet gangrene
C. Gas gangrene D. Necrotizing inflammation
13. Which one of the following is a type of idiopathic calcinosis cutis?
A. Necrotizing inflammation B. Dystrophic calcification
C. Metastatic calcification D. Calcified thrombi in veins
14. Cells in atrophy are
A. Dead cells B. Shrunken cells
C. Irreversibly injured cells D. Reversibly injured cells
15. Which of the following is true about metaplasia?
A. It is a disordered growth B. It affects only epithelial tissues
C. It is a reversible change D. It is an irreversible and progressive change
16. In cell cycle, activator of signal transduction system is
A. G protein receptors B. Selectins
C. Cadherins D. Integrins
17. Which one among the following activates the immune system?
A. Cell adhesion molecules B. Cytokines
C. G-protein receptors D. Ion channels
18. Choose the proapoptotic protein
A. p53 B. Bcl-2
C. CRMA D. Bax
19. Annexin V is a marker for
A. Necrosis B. Fatty change
C. Apoptosis D. Gangrene
20. Enzyme to prevent aging is
A. Catalase B. Superoxide dismutase
C. Metalloproteinase D. Telomerase
21. Hyperplasia is primarily operative in
A. Renovascular hypertension
B. Thickened bladder wall in a patient with urethral obstruction
C. Barrett’s esophagus in a patient with esophageal reflux
D. Galactorrhea in a woman with a prolactinoma
22. Psammoma bodies are seen in which condition?
A. Multiple myeloma of bone B. Papillary thyroid carcinoma
C. Parathyroid adenoma D. Paget’s disease
23. Example of coagulation necrosis is
A. Lobar pneumonia in an alcoholic
B. Hepatic abscess in a patient with amebiasis
C. Pseudomembranous colitis in a patient on ampicillin
D. Embolus to the superior mesenteric artery leading to bowel infarction
24. The eosinophilic cytoplasmic inclusions seen in alcoholic liver disease are
A. Amyloid B. Mallory bodies
C. Russel bodies D. Kimura bodies
25. Which of the following is a feature of necrosis?
A. Reduced cell size B. Intact cellular contents
C. Intact plasma membrane D. Karyolysis
26. Which of the following is true about the characteristic feature of apoptosis?
A. Absence of inflammation B. Affects numerous cells at a time
C. Cell swelling D. Leaking of enzymes

https://t.me/DentalBooksWorld
154 Triumph's Complete Review of Dentistry

27. The characteristic feature of irreversible injury is


A. Cell swelling B. Fatty change
C. Surface blebs D. Cell membrane defects
28. The substances essential for procollagen fibers to transform into collagen fibers after cell injury is
A. Cortisone B. Carotene
C. Ascorbic acid D. Prothrombin
29. Monocytes give rise to
A. Macrophages B. Epithelioid cells
C. Giant cells D. All the above
30. Which is the most susceptible tissue to liquefactive necrosis following ischemic injury?
A. Brain B. Intestine
C. Liver D. Pancreas
31. Type of necrosis seen in myocardial infarction of a geriatric patient is
A. Caseous necrosis B. Coagulation necrosis
C. Enzymatic fat necrosis D. Gangrenous necrosis
32. The reduction in the size of the nucleus and a condensation of nuclear material is known as
A. Pyknosis B. Karyolysis
C. Karyorrhexis D. Metachromasia
33. Main enzymes responsible for the liquefaction of tissue in an abscess is
A. Lymphocytes B. Macrophages
C. Mast cells D. Neutrophils
34. Type of necrosis seen in hemorrhagic infarction of the lung is
A. Caseous necrosis B. Coagulation necrosis
C. Fibrinoid necrosis D. Gangrenous necrosis
35. Caseous necrosis is associated with
A. Pulmonary tuberculosis B. Soft tissue abscesses
C. Sore throat D. Thrombophlebitis
36. First cells to aggregate at the site of acute tissue injury are
A. B-lymphocytes B. Macrophages
C. Neutrophils D. Plasma cells
37. Statements that best defines the term “labile cells” is
A. Cells in a continuous cycle of cell division
B. Cells incapable of mitotic division
C. Cells which have a constant but irregular mitotic rate
D. Cells which rarely divide but proliferate in response to injury
38. Which one of the following is not a sign of necrosis?
A. Lipofuscin B. Pyknosis
C. Karyolysis D. Karyorrhexis
39. Enzymatic necrosis affecting the pancreas is referred to as
A. Coagulative necrosis B. Liquefactive necrosis
C. Fat necrosis D. Caseous necrosis
40. Metastatic calcification is seen in
A. Vitamin A deficiency. B. Vitamin D deficiency
C. Hyperparathyroidism D. Hypothyroidism
41. The replacement of dead cells cannot occur in tissues composed of
A. Labile cells B. Stable cells
C. Permanent cells D. Stem cells
42. Regeneration typically do not occur in
A. Epidermis B. Liver
C. Adrenal glands D. Lens

https://t.me/DentalBooksWorld
Chapter 3 • General Pathology 155

43. The metastatic calcification results from


A. Fat necrosis B. Fracture of a bone
C. Hypercalcemia D. Malignant neoplasia
44. Which of the following is the true statement about necrosis?
A. It is a physiological response to injury B. It maintains plasma membrane integrity
C. It may be stimulated by oxygen-free radicals D. It does not result in an inflammatory response
45. Which of the following is true of pure apoptosis?
A. Is only a pathological process B. Occurs in groups of cells
C. Results in cell shrinkage and fragmentation D. DNA is cleaved by proteases
46. CD 95 is a marker of
A. Intrinsic pathway of apoptosis B. Extrinsic pathway of apoptosis
C. Necrosis of cell D. Cellular adaptation
47. Which of the following is a characteristic feature of irreversible injury on electron microscopy?
A. Disruption of ribosomes B. Amorphous densities in mitochondria
C. Swelling of endoplasmic reticulum D. Cell swelling
48. Caspases is associated with
A. Hydropic degeneration B. Collagen hyalinization
C. Embryogenesis D. Fatty degeneration
49. Caspases are seen in
A. Cell division B. Apoptosis
C. Necrosis D. Inflammation
50. Light microscopic characteristic feature of apoptosis is
A. Intact cell membrane B. Eosinophilic cytoplasm
C. Both A and B D. Condensation of the nucleus
51. Which organelle plays a pivotal role in apoptosis?
A. All the below B. Golgi complex
C. Mitochondria D. Nucleus
52. Fibrinoid necrosis is not observed in
A. Diabetic glomerulosclerosis B. Aschoff ’s nodule
C. Malignant hypertension D. Polyarteritis nodosa
53. Choose the antiapoptotic gene
A. C-myc B. p53
C. bcl-2 D. bax
54. Annexin V in a nonpermeable cell indicates
A. Apoptosis B. Necrosis
C. Cell entering replication phase D. Cell cycle arrest
55. Caspases is involved in which of the following?
A. Necrosis B. Apoptosis
C. Atherosclerosis D. Inflammation
56. Which of the following is false about apoptosis?
A. Inflammation is present B. Chromosomal breakage
C. Clumping of chromatin D. Cell shrinkage
57. Characteristic feature of internucleosomal cleavage of DNA is
A. Reversible cell injury B. Irreversible cell injury
C. Necrosis D. Apoptosis
58. Ladder pattern of DNA electrophoresis in apoptosis is caused by which enzyme?
A. Endonuclease B. Transglutaminase
C. Caspase D. Cytochrome

https://t.me/DentalBooksWorld
156 Triumph's Complete Review of Dentistry

59. Which of the following indicates irreversible cell injury?


A. Dilatation of endoplasmic reticulum
B. Dissociation of ribosomes from rough endoplasmic reticulum
C. Flocculent densities in the mitochondria
60. Liquefaction necrosis is typically seen in
A. Ischemic necrosis of the heart B. Ischemic necrosis of the brain
C. Ischemic necrosis of the intestine D. Tuberculosis
61. Which of the following is false about coagulative necrosis as a primary event?
A. Kidneys B. CNS
C. Spleen D. Liver
62. Liquefactive necrosis is seen in
A. Heart B. Brain
C. Lung D. Spleen
63. Which of the following is true about pyogenic infection and brain infarction?
A. Coagulative necrosis B. Liquefactive necrosis
C. Caseous necrosis D. Fat necrosis
64. First cellular change in hypoxia is
A. Decreased oxidative phosphorylation in mitochondria B. Cellular swelling
C. Alteration in cellular membrane permeability D. All of the above
65. Cytochrome C acts through which of the following during apoptosis?
A. Apaf 1 B. Bcl-2
C. FADD D. TNF
66. Most sensitive cell to hypoxia is
A. Myocardial cells B. Neurons
C. Hepatocytes D. Renal tubular epithelial cells
67. Myelin figures are derived from which of the following after cell death?
A. Nucleus B. Cell membrane
C. Cytoplasm D. Mitochondria
68. Intracellular calcification begins in
A. Mitochondria B. Golgi body
C. Lysosome D. Endoplasmic reticulum
69. Inducer of apoptosis in a cell is
A. Oleic acid B. Isoprenoids
C. Myristic acid D. Glucocorticoids
70. Which of the following is true about apoptosis and necrosis?
A. Both may be physiological B. Both may be pathological
C. Inflammation D. Intact cell membrane
71. Psammoma bodies do not occur in
A. Follicular carcinoma of thyroid B. Papillary carcinoma of thyroid
C. Serous cystadenoma of ovary D. Meningioma
72. Which of the following is true about Fenton reaction?
A. Hydrogen peroxide is formed by myeloperoxidase B. Radiant energy is absorbed by water
C. Nitric oxide is converted to peroxynitrite anion D. Ferrous ions are converted to ferric ions
73. Heterotopic calcification occurs in
A. Reiter’s syndrome B. Forrestier’s disease
C. Rheumatoid arthritis D. Gouty arthritis
74. Wear and tear pigment in body refers to
A. Lipochrome B. Melanin
C. Anthracotic pigment D. Hemosiderin

https://t.me/DentalBooksWorld
Chapter 3 • General Pathology 157

75. “Russell’s body” is an accumulation of


A. Phospholipids B. Cholesterol
C. Lipoproteins D. Immunoglobulins
76. Brown atrophy is due to
A. Lipofuscin B. Ceruloplasmin
C. Fatty necrosis D. Hemosiderin
77. Which of the following is true about coagulative necrosis?
A. Characteristic of focal bacterial infections B. Characteristic of hypoxic death
C. Characterized by loss of tissue architecture D. None of the above
78. Diabetic gangrene is caused by
A. Vasospasm B. Peripheral neuritis
C. Atherosclerosis D. None of the above
79. Metastatic calcifications are seen in
A. Hypoparathyroidism B. Vitamin D deficiency
C. Hypercalcemia D. All the above
80. Pick the odd one out
A. Coagulation necrosis – Tuberculosis B. Caseation – Yellow fever
C. Fat necrosis – Pancreatitis D. Gumma – Infarction
81. Example of cellular swelling and fatty change is
A. Reversible injury B. Irreversible injury
C. Both A and B D. None of the above
82. Pyknosis is characterized by
A. Nuclear basophilia B. Nuclear shrinkage
C. Nucleus disintegration D. Nucleolus disintegration
83. Gangrene is accompanied by
A. Suppuration B. Putrefaction
C. Calcification D. Coagulation
84. Example of hormone dependent shedding of endometrium is
A. Necrosis B. Autolysis
C. Apoptosis D. None of the above
85. Phase at which synthesis of DNA occurs is
A. G1 B. S
C. G2 D. M
86. Apoptosis is
A. Single cell necrosis B. Intracytoplasmic accumulation
C. Degenerative change D. Neoplastic change in the cell
87. Severe generalized edema is called
A. Myxedema B. Pitting edema
C. Anasarca D. Dependent edema
88. Tigered effect of heart is seen in
A. Fatty change B. Hyaline change
C. Amy D. Atrophy
89. The combination of rest pain, color changes, hyperesthesia, and edema is seen in
A. Gangrene B. Pregangrene
C. Necrosis D. None of the above
90. Dominant histologic feature of infarction is
A. Liquefactive necrosis B. Coagulative necrosis
C. Chronic inflammation D. Scar tissue
91. Which of the following is false about dystrophic calcification?
A. Occurs in damaged tissue B. Tuberculosis lesions
C. Atherosclerotic lesions D. Serum calcium levels are high

https://t.me/DentalBooksWorld
158 Triumph's Complete Review of Dentistry

92. Which one among the following is associated with defective apoptosis and increased cell survival?
A. Neurodegenerative diseases B. Autoimmune disorders
C. Myocardial infarction D. Stroke
93. Which phase is prone to ionizing radiation effect?
A. G2 S B. G1 G2
C. G2 M D. G0 G1
94. The correct sequence of cell cycle is
A. G0–G1–S–G2–M B. G0–G1–G2–S–M
C. G0–M–G2–S–G1 D. G0–G1–S–M–G2
95. The cellular content of DNA is doubled at which phase of the cell cycle?
A. Mitotic phase B. G1 phase
C. G2 phase D. S phase
96. p53 induces cell cycle arrest at the level of
A. G2–M phase B. S–G2 phase
C. G1–S phase D. G0 phase
97. Which of the following controls G2 to M phase transition of the cell cycle?
A. Retinoblastoma gene product B. p53 protein
C. Cyclin E D. Cyclin B
98. Fixed time is required for which steps of the cell cycle?
A. S B. M
C. G1 D. G2

INFLAMMATION, IMMUNITY, AND HYPERSENSITIVITY


1. Epithelioid cells are seen in all of the following except
A. Tuberculosis B. Granulation tissue
C. Syphilis D. Sarcoidosis
2. Which of the following statement about fibrinous exudate is false?
A. It is associated with many types of severe inflammation B. It has low protein content
C. It has fibrin precipitates D. It induces connective tissue organization
3. Some microorganisms produce a diffuse spreading inflammatory reaction due to the elaboration of
A. Coagulase B. Peroxidase
C. Bradykinin D. Hyaluronidase
4. Bradykinin causes all the following, except
A. Smooth muscle contraction B. Dilatation of blood vessels
C. Pain D. Opsonization
5. Component of tubercle bacilli which produces granuloma is
A. Surface glycolipids B. Sulfatide
C. Hetero polysaccharide D. Sulfatase
6. The vasoactive amine that causes vasodilatation, released during inflammation, is found in which of the following cells?
A. Plasma cells B. Lymphocytes
C. Monocytes D. Mast cells
7. Difference in transudate and exudate is that the former has
A. Low protein B. Cloudy appearance
C. Increased specific gravity D. High protein
8. An acute inflammatory focus would attract
A. Monocytes B. Plasma cells
C. Neutrophils D. Basophils
9. Granuloma is characterized by all of the following, except
A. A specific type of chronic inflammation
B. Accumulation of modified macrophages

https://t.me/DentalBooksWorld
Chapter 3 • General Pathology 159

C. Initiated in number of infections and noninfectious agents


D. A reaction of acute inflammation
10. Which cell releases vasoactive amine so as to increase vascular permeability?
A. Leukocyte B. Macrophage
C. Mast cell D. Fibroblast
11. Prostaglandins are synthesized from
A. RNA template B. Rough endoplasmic reticulum
C. Polyunsaturated fatty acids D. None of the above
12. Transudate is characterized by
A. Associated inflammatory conditions B. Low protein content
C. Tendency to clot D. Specific gravity of above 1.018
13. Edema occurs due to
A. Increased capillary permeability B. Decreased capillary permeability
C. Decreased interstitial fluid D. Decreased blood flow
14. Following are functions of prostaglandins except
A. Increase capillary permeability B. Uterine contraction
C. Lowers blood pressure D. Elevates blood pressure
15. The host tissue response in acute inflammation is all, except
A. Exudative B. Necrotizing
C. Granulomatous D. Cytopathic
16. In chronic granulomatous inflammation, which of the following processes is most likely to predominate?
A. Exudation B. Congestion
C. Transudation D. Proliferation
17. Ag–Ab (antigen–antibody) reaction due to the presence of antibody at the surface of cell is
A. Type I hypersensitivity B. Type II hypersensitivity
C. Type III hypersensitivity D. Type IV hypersensitivity
18. Which of the following cells are more abundant in chronic inflammation than in acute inflammation?
A. Plasma cells B. Eosinophils
C. Neutrophils D. Normoblasts
19. The process by which red blood cells move out of vessels through widened inter endothelial junction is referred to as
A. Pavementing B. Diapedesis
C. Rouleaux formation D. Chemotaxis migration
20. IL-1 helps in
A. Stimulation of T-lymphocytes
B. Inhibits of B-lymphocytes
C. Inhibits the chemotaxis of neutrophils and macrophages
D. Decreases fibroblast and bone resorption activity
21. Lepra cells are seen in abundance in
A. Tuberculoid leprosy B. Lepromatous leprosy
C. Histoid leprosy D. Intermediate leprosy
22. Which of the following is not a mediator of inflammation?
A. Interferon B. Prostaglandins
C. TNF D. Myeloperoxidase
23. Caseating granuloma is commonly seen in
A. Viral infections B. Tuberculosis
C. Typhoid D. Amebiasis
24. Virchow’s cells are seen in
A. Lepromatous leprosy B. Tuberculoid leprosy
C. Borderline lepromatous leprosy D. Intermediate leprosy

https://t.me/DentalBooksWorld
160 Triumph's Complete Review of Dentistry

25. All of the following about prostaglandins and leukotriene are correct except
A. Arachidonic acid is metabolized to form prostaglandins and leukotrienes
B. COX-1 is inducible in many tissues
C. COX is induced by cytokines at the site of inflammation
D. Leukotrienes cause bronchoconstriction
26. An acute inflammation would attract
A. Monocytes B. Plasma cells
C. Neutrophils D. Eosinophils
27. In granuloma, epithelial and giant cells are derived from
A. T cells B. B cells
C. Plasma cells D. Monocyte
28. All of the following are true for exudates, except
A. It has a specific gravity of 1.018 B. It has a low fibrin content
C. It has more than 3% proteins D. It is mucinous in consistency
29. Inflammation is characterized by
A. Transudation -> Exudation -> Edema B. Edema -> Exudation
C. Exudation -> Transudation -> Edema D. Only by exudation -> Edema
30. The characteristic cells of chronic inflammation are all, except
A. Plasma cells B. Macrophages
C. Lymphocytes D. PMNs
31. The predominant cells after 48 hours of inflammation are
A. Monocytes B. Macrophages
C. Neutrophils D. A and B
32. Chronic inflammation is characterized by
A. Presence of macrophages B. Tissue destruction
C. Proliferation of fibroblasts and endothelial cells D. All of the above
33. Example of granulomatous inflammation
A. Sarcoidosis B. Leprosy
C. Tuberculosis D. All of the above
34. The type of immunity activated in tuberculosis is
A. Cell mediated B. Humoral
C. Foreign body reaction D. None of the above
35. Lymphocytosis is seen in
A. Fungal infections B. Viral infections
C. Bacterial infections D. Protozoal infections
36. The characteristic feature of macrophage is
A. High capacity to divide B. Limited capacity to divide
C. Long life span as compared to lymphocytes D. Both B and C
37. Highly infective stage of syphilis
A. Primary B. Secondary
C. Tertiary D. Congenital
38. Early positive reactive in lepromin test is
A. Fernandez reaction B. Mitsuda reaction
C. Wasserman reaction D. None of the above
39. Kveim’s test is diagnostic test for
A. Actinomycosis B. Diphtheria
C. Tuberculosis D. Sarcoidosis
40. Which of the following statements is correct?
A. Hard tubercle – absence of caseous necrosis B. Ghon’s complex – primary tuberculosis
C. Miliary tuberculosis – extra pulmonary TB D. All of the above

https://t.me/DentalBooksWorld
Chapter 3 • General Pathology 161

41. Which of the following bacteria resembles fungus?


A. Mycobacterium leprae B. Actinomyces israelii
C. Mycobacterium bovis D. All of the above
42. Which of the following is incorrect?
A. Bacterial infections – neutrophilia B. Viral infections – lymphocytosis
C. Parasitic infections – eosinophilia D. None of the above
43. Enzymes responsible for suppuration are derived chiefly from
A. PMNs B. Lymphocytes
C. Monocytes D. Eosinophils
44. Tissue macrophages are called
A. Kupffer cells in liver B. Microglial cells in nervous system
C. Histiocytes in connective tissues D. All of the above
45. Anaphylatoxins are
A. C3a, C5a B. C3b, C5a
C. C2, C3 D. C3b, C5b
46. Serum sickness syndrome is
A. A transparent immunity B. An anaphylactic shock
C. Systemic Arthus reaction D. None of the above
47. All of the following vascular changes are observed in acute inflammation, except
A. Vasodilation B. Stasis of blood
C. Increased vascular permeability D. Decreased hydrostatic pressure
48. Lepra cells seen in leprosy are
A. Lymphocytes B. Plasma cells
C. Vacuolated histiocytes D. Neutrophils
49. Virchow Lepra cells are seen in
A. Tuberculoid leprosy B. Indeterminate leprosy
C. Borderline tuberculoid leprosy D. Lepromatous leprosy
50. Function of hepatic Kupffer cells is
A. Formation of sinusoids B. Vitamin – A storage
C. Increases blood perfusion D. Phagocytosis
51. Cold abscess formation is due to
A. Presence of acute inflammation
B. Formation of caseous pus with signs of acute inflammation
C. Formation of caseous pus without signs of acute inflammation
D. Infected cyst
52. The process of “Phagocytosis” was discovered by
A. Celsus B. Elie Metchnikoff
C. Virchow D. None of the above
53. Lipofuscin is an insoluble endogenous pigment, also known as
A. Lipochrome B. Wear and tear pigment
C. Aging pigment D. All of the above
54. The following chemical mediator is a product of arachidonic acid metabolite by cyclooxygenase pathway
A. LxA4 B. LxB4
C. 5-HETE D. PGH2
55. Earliest transient change following tissue injury
A. Neutrophilia B. Neutropenia
C. Monocytosis D. Lymphocytosis
56. Ghon’s complex of the lung usually
A. Undergoes cavitation B. Undergoes calcification
C. Progresses to tuberculous pneumonia D. Progresses to military tuberculosis

https://t.me/DentalBooksWorld
162 Triumph's Complete Review of Dentistry

57. Which of the following is not a constituent of Virchow’s triad?


A. Damage to endothelium due to injury or inflammation B. Diminished rate of blood flow
C. Increased coagulability of blood D. Increased venous blood pressure
58. “Ghon’s focus” is associated with
A. Gonorrhea B. Syphilis
C. AIDS D. Tuberculosis
59. The specific gravity of a transudate is
A. Below 1.010 B. Below 1.012
C. Between 1.012 and 1.020 D. Above 1.020
60. In acute inflammation, immediate transient permeability in cells is increased by
A. Direct injury and necrosis B. Endothelial gaps by histamine
C. Leukocyte mediated endothelial injury D. Increased tissue oncotic pressure
61. Delayed prolonged increase in vascular permeability in inflammation is due to
A. Interleukin 1 mediated endothelial retraction B. Increased transcytosis
C. Leakage from newly formed blood vessels D. Release of histamine causing endothelial contraction
62. Most commonly used treponemal test in diagnosis of syphilis is
A. TPI (Treponema Pallidum Immobilization)
B. TPIA (Treponema Pallidum Immune Adherence)
C. TPHA (Treponema Pallidum Hemagglutination)
D. FTA ABS (Fluorescent Treponemal Antibody Absorption Test)
63. The explosive and widespread form of secondary syphilis in immunocompromised individual is known as
A. Condylomata lata B. Mucous patches
C. Lues maligna D. Lupus vulgaris
64. Syphilitic gumma is seen in
A. Primary syphilis B. Secondary syphilis
C. Tertiary syphilis D. Quaternary syphilis
65. Binding of complement to bacterial cell surface is called
A. Opsonization B. Complement activation
C. Complement stabilization D. None of the above
66. Which inflammatory mediator involved in intracellular killing of microbes?
A. Catalase B. Oxidase
C. Lysozyme D. IL6
67. All of the following are hallmarks of tuberculous inflammation, except
A. Caseous necrosis B. Langerhans cells
C. Epithelioid cell granuloma D. Liquefactive necrosis
68. Which is anti-inflammatory?
A. Thromboxane B. Lipoxins
C. Prostacyclin D. Prostaglandin
69. Syphilitic arterial aneurysms typically involve which of the following processes?
A. Cystic medial sclerosis B. Fatty streaks
C. Circumferential calcification D. Endarteritis of the vasa vasorum
70. IL-1 and THF-alpha during inflammation are secreted by
A. Plasma cells B. Lymphocytes
C. Activated macrophages D. Platelets
71. Which of the following substances may cause edema during inflammation?
A. Histamine B. C5a
C. Interleukin 1 D. TNF
72. Which of the following is known as the membrane attack complex?
A. C3b B. C67
C. C5b9 D. C1b3

https://t.me/DentalBooksWorld
Chapter 3 • General Pathology 163

73. Which of the following white blood cells predominate within 6–24 hours after tissue injury?
A. Macrophage B. Basophils
C. Neutrophils D. Lymphocytes
74. Type of inflammation induced by indigestible foreign bodies is
A. Serous B. Granulomatous
C. Suppurative D. Fibrinous
75. Central actors in chronic inflammation are
A. Neutrophils B. Lymphocytes
C. Macrophages D. Eosinophils
76. The hallmark of acute inflammation is
A. Transient vasoconstriction B. Vasodilation
C. Increased vascular permeability D. Slowing of circulation
77. Which of the complement components act as chemokines?
A. C3b B. C4b
C. C5a D. C4a
78. All of the following are types of tissue macrophages, except
A. Littoral cells B. Hoffbauer cells
C. Osteoclasts D. Osteoblasts
79. Formation of granuloma is
A. Type I hypersensitivity reaction B. Type II hypersensitivity reaction
C. Type III hypersensitivity reaction D. Type IV hypersensitivity reaction
80. Which of the following is atypical mycobacteria?
A. Mycobacterium microti B. Mycobacterium canettii
C. Mycobacterium africanum D. Mycobacterium ulcerans
81. IgM antibody against PGL-1 antigen is used for the diagnosis of
A. Leprosy B. Tuberculosis
C. Syphilis D. Brucellosis
82. Which category of leprosy is not included in Ridley–Jopling classification?
A. Mid borderline leprosy B. Borderline tuberculoid leprosy
C. Indeterminate leprosy D. Tuberculoid polar leprosy
83. Hepar lobatum is seen in
A. Primary syphilis B. Secondary syphilis
C. Tertiary syphilis D. Congenital syphilis
84. Main cytokines acting as mediators of inflammation are as under, except
A. Interleukin-1 B. Tumor necrosis factor-α
C. Nitric oxide D. Interferon-γ
85. Receptor for IgE is present on
A. Polymorphs B. Eosinophil
C. Basophil D. Plasma cell
86. Typhoid fever is an example of
A. Acute inflammation B. Chronic nonspecific inflammation
C. Chronic granulomatous inflammation D. Chronic suppurative inflammation
87. Tubercle bacilli cause lesions by the following mechanisms
A. Elaboration of endotoxin B. Elaboration of exotoxin
C. Type IV hypersensitivity D. Direct cytotoxicity
88. Tubercle bacilli in caseous lesions are best demonstrated in
A. Caseous center B. Margin of necrosis with viable tissue
C. Epithelioid cells D. Langhans’ giant cells
89. Lepromin test is always positive in
A. Lepromatous leprosy B. Borderline lepromatous leprosy
C. Tuberculoid leprosy D. Indeterminate leprosy

https://t.me/DentalBooksWorld
164 Triumph's Complete Review of Dentistry

90. Spirochaetes are most difficult to demonstrate in


A. Primary syphilis B. Secondary syphilis
C. Tertiary syphilis D. Congenital syphilis
91. Actinomycosis is caused by
A. Fungus B. Gram-negative bacteria
C. Anaerobic bacteria D. Acid fast bacteria
92. Typically, sarcoid granuloma has the following features except
A. Noncaseating granuloma B. Giant cells have cytoplasmic inclusions
C. Peripheral mantle of lymphocytes D. Fibroblastic proliferation at the periphery of a granuloma
93. The following holds true for stable cells in a cell cycle
A. They remain in cell cycle from one mitosis to the next
B. They are in resting phase but can be stimulated to enter the cell cycle
C. They have left the cell cycle
D. They do not have capacity to multiply in response to stimuli throughout adult life
94. Connective tissue in scar is formed by which of the following types of fibrillar collagen
A. Types II, III, IV B. Types I, III, V
C. Types I, II, V D. Types III, V, VII
95. Basement membrane consists of
A. Type I collagen B. Type II collagen
C. Type III collagen D. Type IV collagen
96. Which of the following is nonfibrillar collagen?
A. Type V B. Type I
C. Type III D. Type VI
97. Which is false about primary union?
A. Exuberant granulation tissue to fill the gap B. Clear margins
C. Uninfected D. Lead to neat linear scar.
98. In acute inflammation endothelial retraction leads to
A. Delayed transient increase in permeability B. Immediate transient increase in permeability
C. Delayed prolonged increase in permeability D. Immediate transient decrease in permeability
99. After binding of complement and antibody on the surface of encapsulated bacteria, the process of phagocytosis by
polymorph nuclear leukocytes involves which of the following?
A. Fc and C3b B. Receptor-mediated endocytosis
C. Respiratory burst D. Pseudopod extension
100. Free radicals are generated by all except
A. Superoxide dismutase B. NADPH oxidase
C. Myeloperoxidase D. NO synthase
101. Which among the following is the hallmark of acute inflammation?
A. Vasoconstriction B. Stasis
C. Vasodilation and increase in permeability D. Leukocyte margination
102. The main feature of chemotaxis is
A. Increased random movement of neutrophils B. Increase adhesiveness to intima
C. Increased phagocytosis D. Unidirectional locomotion of neutrophils
103. Characteristic of acute inflammation is
A. Vasodilation and increased vascular permeability B. Vasoconstriction
C. Platelet aggregation D. Infiltration by neutrophils
104. Which of the following helps in generating reactive O2 intermediates in the neutrophils?
A. NADPH oxidase B. SOD (superoxide dismutase)
C. Catalase D. Glutathione peroxidase
105. Basement membrane degeneration is mediated by
A. Metalloproteinases B. Oxidases
C. Elastases D. Hydroxylases

https://t.me/DentalBooksWorld
Chapter 3 • General Pathology 165

106. Delayed prolonged bleeding is caused by


A. Histamine B. Endothelial retraction
C. IL-1 D. Direct injury to endothelial cells
107a. Earliest transient change following tissue injury will be
A. Neutropenia B. Neutrophilia
C. Monocytosis D. Lymphocytosis
107b. All of the following vascular changes are observed in acute inflammation, except
A. Vasodilation B. Stasis of blood
C. Increased vascular permeability D. Decreased hydrostatic pressure
108. The following host tissue responses can be seen in acute infection, except
A. Exudation B. Vasodilation
C. Margination D. Granuloma formation
109. Oxygen-dependent killing is done through
A. NADPH oxidase B. Superoxide dismutase
C. Catalase D. Glutathione peroxidase
110. Which of the following is not true?
A. NADPH oxidase generates superoxide ion
B. MPO kills by OCl
C. Chediak–Higashi syndrome is due to defective phagolysosome formation
D. In Bruton’s disease there is normal opsonization
111. Nitroblue tetrazolium test is used for
A. Phagocytes B. Complement
C. T cell D. B cell
112. In acute inflammation due to the contraction of endothelial cell cytoskeleton, which of the following results?
A. Delayed transient increase in permeability B. Early transient increase
C. Delayed permanent increase D. Early permanent increase
113. Endothelium leukocyte interaction during inflammation is mediated by/due to
A. Selectins B. Endothelin
C. Defensins D. None of the above
114. In genetic deficiency of MPO the increased susceptibility to infection is due to
A. Defective production of prostaglandins B. Defective rolling of neutrophils
C. Inability to produce hydroxy-halide radicals D. Inability to produce hydrogen peroxide
115. After extravasation, leukocytes emigrate in the tissue toward the site of injury. It is known as
A. Margination B. Chemotaxis
C. Diapedesis D. Pavementing
116. The complex process of leukocyte movements through the blood vessels are all except
A. Rolling B. Adhesion
C. Migration D. Phagocytosis
117. All are true about exudate except
A. More protein B. Less protein
C. More specific gravity D. All
118. All of the following are signs of inflammation except
A. Pain B. Swelling
C. Redness D. Absence of functional loss
119. Endogenous chemoattractant is
A. C5a B. Bacterial products
C. Lipopolysaccharide A D. C8
120. A patient with myasthenia gravis developed progressive muscle weakness. Which type of hypersensitivity reaction is
this?
A. Type I B. Type II
C. Type III D. Type IV

https://t.me/DentalBooksWorld
166 Triumph's Complete Review of Dentistry

121. Leucocytes leave the blood vessels and move toward the site of bacteria. Which of the following is likely to mediate this
movement of the bacteria?
A. Histamine B. C3b
C. C3a D. C5a
122. All of the following are a family of selectin except
A. P selectin B. L selectin
C. A selectin D. E selectin
123. Which of the following is the most important for diapedesis?
A. PECAM B. Selectin
C. Integrin D. Mucin-like glycoprotein
124. In acute inflammation the tissue response consists of all except
A. Vasodilatation B. Exudation
C. Neutrophilic response D. Granuloma formation
125. The function common to neutrophils, monocytes, and macrophages is
A. Immune response is reduced
B. Phagocytosis
C. Liberation of histamine
D. Destruction of old erythrocytes – chemical mediators of inflammation
126. The role of bradykinin in process of inflammation is
A. Vasoconstriction B. Bronchodilation
C. Pain D. Increased vascular permeability
127. Which of the following is not a pyrogenic cytokine?
A. IL-1 B. TNF
C. IFN-α D. IL-18
128. Which of the following complement component can be activated in both common as well as alternative
pathways?
A. C1 B. C2
C. C3 D. C4
129. Which of the following is not an inflammatory mediator?
A. Tumor necrosis factor B. Myeloperoxidase
C. Interferons D. Interleukin
130. Nephrocalcinosis in a systemic granulomatous disease is due to
A. Overproduction of 1,25 dihydroxy vitamin D B. Dystrophic calcification
C. Mutation in calcium sensing receptors D. Increased reabsorption of calcium
131. Most important bactericidal agent is
A. Cationic basic protein B. Lactoferrin
C. Lysozyme D. Reactive O2 species
132. Bradykinin causes
A. Vasoconstriction B. Pain at the site of inflammation
C. Bronchodilation D. Decreased vascular permeability
133. Lewis triple response is caused due to
A. Histamine B. Axon reflex
C. Injury to endothelium D. Increased permeability
134. Factor present in the final common terminal complement pathway is
A. C4 B. C3
C. C5 D. Protein B
135. To which of the following family of chemical mediators of inflammation, the lipoxins belong?
A. Kinin system B. Cytokines
C. Chemokines D. Arachidonic acid metabolites

https://t.me/DentalBooksWorld
Chapter 3 • General Pathology 167

136. Both antibody dependent and independent complement pathway converge on which complement component?
A. C3 B. C5
C. C1q D. C8
137. C-C beta chemokines include
A. IL-8 B. Eotaxin
C. Lymphotactin D. Fractalkine
138. All of the following are mediators of acute inflammation except
A. Angiotensin B. Prostaglandin E2
C. Kallikrein D. C3a
139. All of the following are mediators of inflammation except
A. Tumor necrosis factor-a (TNF-a) B. Interleukin-1
C. Myeloperoxidase D. Prostaglandins
140. Interleukin secreted by macrophages, stimulating lymphocytes is
A. IFN alpha B. TNF alpha
C. IL-1 D. IL-6
141. Febrile response in CNS is mediated by
A. Bacterial toxin B. IL-l
C. IL-6 D. Interferon
E. Tumor necrosis factor (TNF)
142. Cytokines
A. Include interleukins B. Produced only in sepsis
C. Are polypeptide (complex proteins) D. Have highly specific action
143. Conversion of prothrombin to thrombin requires
A. V only B. V and Ca++
C. XII D. X and Ca++
144. Which complement fragments are called “anaphylatoxins”?
A. C3a and C3b B. C3b and C5b
C. C5a and C3b D. C3a and C5a
145. Cryoprecipitate is rich in which of the following clotting factors?
A. Factor II B. Factor V
C. Factor VII D. Factor VIII
146. Most important mediator of chemotaxis is
A. C3b B. C5a
C. C5-7 D. C2
147. Histamine causes
A. Hypertension B. Vasoconstriction
C. Vasodilation D. Tachycardia
148. Which of the following is found in secondary granules of neutrophils?
A. Catalase B. Gangliosides
C. Proteolytic enzyme D. Lactoferrin
149. All are mediators of neutrophils except
A. Elastase B. Cathepsin
C. Nitric oxide D. None of the above
150. Ultrastructurally, endothelial cells contain
A. Weibel–Palade bodies B. Langerhan’s granules
C. Abundant glycogen D. Kallikrein
151. Partial thromboplastin time correlates with
A. Intrinsic and common pathway B. Extrinsic and common pathway
C. Vessel wall integrity and intrinsic pathway D. Platelet functions and common pathway

https://t.me/DentalBooksWorld
168 Triumph's Complete Review of Dentistry

152. Bleeding time assesses


A. Extrinsic clotting pathway B. Intrinsic clotting pathway
C. Fibrinogen level D. Function of platelets
153. The estimation of the prothrombin level is useful in the following clotting factor deficiency, except
A. II B. V
C. VII D. IX
154. Which of the following is the secondary mediator of anaphylaxis?
A. Histamine B. Proteases
C. Eosinophilic chemotactic factor D. Leukotriene B4
155. Birbeck’s granules in the cytoplasm are seen in
A. Langerhans cells B. Mast cells
C. Myelocytes D. Thrombocytes
156. The eosinophils secrete all except
A. Major basic protein B. Hydrolytic enzyme
C. Reactive form of O2 D. Eosinophilic chemotactic factor
157. In lipoxygenase pathway of the arachidonic acid metabolism, which of the following products helps to promote the
platelet aggregation and vasoconstriction?
A. C5a B. Thromboxane A2
C. Leukotriene B4 D. C1 activators
158. Chemotactic complement components are
A. C3a B. C5a
C. Both D. C3b
159. In inflammatory process, the prostaglandin E1and E2 cause
A. Vasodilatation B. Increased gastric output
C. Decreased body temperature D. Vasoconstriction
160. Opsonins are
A. C3a B. IgM
C. Carbohydrate binding proteins D. Selectins
161. Inflammatory mediator of generalized systemic inflammation is
A. IL-1 B. IL-2
C. Interferon alpha D. TNF
162. All are cytokines except
A. Monoclonal antibody B. Interleukin
C. Chemokine D. TNF
163. Fever occurs due to
A. IL 1 B. Endorphin
C. Enkephalin D. Histamine
164. E cadherin gene deficiency is seen in
A. Gastric cancer B. Intestinal cancer
C. Thyroid cancer D. Pancreatic cancer
165. Cell-matrix adhesions are mediated by
A. Cadherins B. Integrins
C. Selectins D. Calmodulin
166. Pro-inflammatory cytokines include all of the following except
A. Interleukin 1 B. Interleukin-10
C. Interleukin 6 D. TNF-Alpha
167. The most important source of histamine
A. Mast cells B. Eosinophil
C. Neutrophil D. Macrophages

https://t.me/DentalBooksWorld
Chapter 3 • General Pathology 169

168. Following injury to a blood vessel, immediate hemostasis is achieved by which of the following?
A. Fibrin deposition B. Vasoconstriction
C. Platelet adhesion D. Thrombosis
169. PAF causes all except
A. Bronchoconstriction B. Vasoconstriction
C. Decreased vascular permeability D. Vasodilation
170. Eosinophils are activated by
A. IL1 B. IL5
C. IL4 D. IL6
171. Both antibody-dependent and -independent complement pathway converge on which complement component?
A. C3 B. C5
C. C1q D. C8
172. Cryoprecipitate is rich in which of the following clotting factors?
A. Factor II B. Factor V
C. Factor VII D. Factor VIII
173. Prostaglandins are synthesized from
A. Linoleic acid B. Linolenic acid
C. Arachidonic acid D. Butyric acid
174. Which chemical mediator is an arachidonic acid metabolite produced by cyclo-oxygenase pathway?
A. LXA4 B. LXB4
C. 5HETE D. PGH2
175. Procalcitonin is used as marker of
A. Cardiac dysfunction in acute coronary syndrome B. Menstrual periodicity
C. Pituitary function D. Sepsis
176. The epithelioid cell and multinucleated giant cells of granulomatous inflammation are derived from
A. Basophils B. Eosinophils
C. CD4-T lymphocytes D. Monocytes–macrophages
177. Granuloma is pathological feature of all, except
A. Giant cell arteritis B. Microscopic polyangiitis
C. Wegener’s granulomatosis D. Churg–Strauss disease
178. Granulomatous inflammatory reaction is caused by all, except
A. M. tuberculosis B. M. leprae
C. Yersinia pestis D. Mycoplasma
179. Noncaseating granulomas are seen in all of the following except
A. Byssinosis B. Hodgkin’s lymphoma
C. Metastatic carcinoma of lung D. Tuberculosis
180. Epithelioid granuloma is caused by
A. Neutrophil B. Cytotoxic T-cells
C. Helper T-cells D. NK cells
181. Caseous necrosis in granuloma are not found in
A. Tuberculosis B. Leprosy
C. Histoplasmosis D. CMV
182. The most important function of epithelioid cells in tuberculosis is
A. Phagocytosis B. Secretory
C. Antigenic D. Healing
183. Necrotizing epithelioid cell granulomas are seen in all, except
A. Tuberculosis B. Wegener’s granulomatosis
C. Cat Scratch disease D. Leprosy
184. Epithelioid granulomatous lesions are found in all of the following diseases, except
A. Tuberculosis B. Sarcoidosis
C. Berylliosis D. Pneumocystis carinii

https://t.me/DentalBooksWorld
170 Triumph's Complete Review of Dentistry

185. Caseous granuloma is seen in


A. Histoplasmosis B. Silicosis
C. Sarcoidosis D. Foreign body
186. Noncaseating granuloma is characteristically seen in
A. Syphilis B. Sarcoidosis
C. Tuberculosis D. Histoplasmosis
187. All are granulomatous diseases except
A. Syphilis B. Sarcoidosis
C. Schistosomiasis D. P. carinii
188. Which of the following is most characteristic of granuloma?
A. Epithelioid cell B. Giant cell
C. Fibroblasts D. Endothelial cell
189. Caseating granuloma are seen in
A. Histoplasmosis B. Sarcoidosis
C. Coccidioidomycosis D. All
190. In a granuloma, epithelioid cells and giant cells are derived from
A. T-lymphocytes B. Monocyte–macrophages
C. B-lymphocytes D. Mast cells
191. In a lymph node showing nonnecrotizing and noncaseating granuloma, which of the following is suspected?
A. Toxoplasmosis B. Lymphogranuloma venereum
C. Cat scratch disease D. Kikuchis lymphadenitis
192. Which of these is not a granulomatous disease?
A. Leprosy B. Tuberculosis
C. Sarcoidosis D. Amebiasis
193. Which one of the following statements is not correct regarding “Stem cell”?
A. Developmental elasticity B. Transdifferentiation
C. Can be harvested from embryo D. “Knockout mice” was made possible because of it
194. Which of the following is absolutely essential for wound healing?
A. Vitamin D B. Carbohydrates
C. Vitamin C D. Balanced diet
195. Chronic granulomatous disease is
A. Associated with formation of multiple granulomas B. A benign neoplastic process
C. A parasitic disease D. Acquired leukocyte function defect
196. In regeneration
A. Granulation tissue
B. Repairing by same type of tissue
C. Repairing by different type of tissue
D. Cellular proliferation is largely regulated by biochemical factors
197. Wound contraction is mediated by
A. Epithelial cells B. Myofibroblasts
C. Collagen D. Elastin
198. Which of the following adhesion molecules is involved in morphogenesis?
A. Osteopontin B. Osteonectin SPARC
C. Tenascin D. Thrombospondins
199. When a cell transforms itself into different lineage, the ability is known as
A. De-differentiation B. Re-differentiation
C. Trans-differentiation D. Sub-differentiation
200. Prion disease is caused by
A. Misfolding of protein B. Denaturation of proteins
C. Reduced formation of proteins D. Excess formation of proteins

https://t.me/DentalBooksWorld
Chapter 3 • General Pathology 171

201. Maximum collagen in wound healing is seen at which stage of healing?


A. End of first week B. End of second week
C. End of third week D. End of 2 months
202. First sign of wound injury is
A. Epithelialization B. Dilatation of capillaries
C. Leucocytic infiltration D. Localized edema
203. Which of these cells can lyse tumor cells or virus infected cells without prior sensitization?
A. B cells B. T cells
C. NK cells D. Macrophages
204. Vitamin used for posttranslational modification of glutamic acid to gamma carboxy glutamate is
A. A B. D
C. E D. K
205. Which one of the following statements about hematopoietic stem cell is false?
A. Stem cells have self-renewal property
B. Subset of stem cells normally circulate in peripheral blood
C. Marrow-derived stem cells can seed other tissues and develop into non-hematopoietic cells as well
D. Stem cells resemble lymphoblasts morphologically
206. Synthesis of DNA occurs in which phase of the cell cycle?
A. Mitosis–M phase B. Gap–G2 phase
C. Gap–G1 phase D. Synthesis–S phase
207. Which of the following cells have the capacity to multiply throughout their life?
A. Stable cells B. Permanent cells
C. Labile cells D. None of the above
208. Which of the following has least capacity for regeneration?
A. Cardiac muscle B. Skeletal muscle
C. Neurons D. All of the above
209. Granulation tissue contains
A. Giant cells B. Fibroblasts
C. Endothelial cells D. B and C
210. The cells which do not undergo mitotic divisions
A. Smooth muscle cells B. Endothelial cells
C. Bone marrow cells D. Neurons
211. The first event in primary wound healing
A. Epithelial changes B. Organization
C. Formation of blood clot D. Acute inflammatory response
212. Wounds which are clean uninfected and surgically incised, with edge of wounds approximated by sutures heal by
A. Primary intention B. Secondary intention
C. Cicatrisation D. All of the above
213. Large open wounds that are characterized by tissue loss and repaired by formation of granulation tissue in the floor
of the wound is characteristic of
A. Secondary healing B. Primary healing
C. Cicatrisation D. Regeneration
214. All of the following events are common to primary and secondary wound healing except
A. Formation of blood clot B. Inflammatory response
C. Epithelial changes D. Wound contraction
215. The persistence of epithelial cells after wound healing give rise to which of the following?
A. Implantation cyst B. Cicatrisation
C. Keloid D. Wound dehiscence
216. The factor which gives strength in wound healing is
A. Collagen B. Blood supply
C. Growth factors D. Hormones

https://t.me/DentalBooksWorld
172 Triumph's Complete Review of Dentistry

217. Factors inhibiting wound healing are


A. Infection B. Cortisol
C. Foreign bodies D. All of the above
218. Incomplete fractures of the bone are called
A. Comminuted fracture B. Compound fracture
C. Simple fracture D. Greenstick fracture
219. Bony union taking place with formation of medullary callus without periosteal callus formation is
A. Primary union B. Secondary union
C. Both of the above D. None of the above
220. All are desirable in the union of the fracture site except
A. Formation of fibrous joint or pseudoarthrosis B. Remodeling of bone
C. Formation of blood clot D. Formation of procallus or osseous callus
221. Systemic lupus erythematosus can be confirmed by testing for
A. SSA B. dsDNA
C. Centromere D. Anti GBM
222. For wound healing which mineral is helpful?
A. Calcium B. Selenium
C. Magnesium D. Copper
223. All of the following promotes wound healing, except
A. Protein B. Steroids
C. Vitamin C D. Adequate oxygen supply
224. Correct sequence of cell cycle
A. G0–G1–S–G2–M B. G0–G1–G2–S–M
C. G0–M–G2–S–G1 D. G0–G1–S–M–G2
225. Essential granulation tissue constituents include all, except
A. Fibroblast B. Macrophages
C. Polymorphs D. Budding blood vessels
226. Important components of collagen synthesis, wound strength, and contraction include all of the following,
except
A. Fibroblasts B. Myofibroblasts
C. Vitamin C D. Vitamin D
227. Factors influencing wound healing are the following
A. Nutrition B. Adequate blood supply
C. Size and location of wound D. All of the above
228. All cells do not divide at the same rate. Events in which phase of the cell cycle determine when a cell is going to
replicate?
A. M phase B. G1 phase
C. S phase D. G2 phase
229. Fibroblast in healing wound contains large quantities of which of the cell parts?
A. Nucleoli B. Rough endoplasmic reticulum
C. Free nonmembranous bound ribosomes D. Smooth endoplasmic reticulum
230. Which of the following is not true about fibroblasts?
A. Secrete and synthesize collagen B. Derived from blood precursor cells
C. Migrate to wound along fibers used as scaffold D. Large fibrin clot acts as barrier to fibroblast penetration
231. Least ability to regenerate is seen in
A. Liver B. Bone
C. Collagen D. Striated muscle

https://t.me/DentalBooksWorld
Chapter 3 • General Pathology 173

HEMODYNAMICS
1. Difference between plasma and interstitial fluid compartment is
A. Glucose is higher in the former B. Urea is higher in the former
C. Protein content is higher in the former D. Potassium is higher in the former
2. Which one of the following do not have the feature of osmotic pressure exerted by the chemical constituents of the
body fluids?
A. Crystalloid osmotic pressure comprises minor portion of total osmotic pressure
B. Oncotic pressure constitutes minor portion of total osmotic pressure
C. Oncotic pressure of plasma is higher
D. Oncotic pressure of interstitial fluid is lower
3. Most important for causation of edema by decreased osmotic pressure is
A. Fall in albumin as well as globulin B. Fall in globulin level
C. Fall in albumin level D. Fall in fibrinogen level
4. Transudate differs from exudate, except
A. No inflammatory cells B. Low glucose content
C. Low protein content D. Low specific gravity
5. Nephritic edema differs from nephrotic edema, except
A. Mild edema B. Distributed on face, eyes
C. Heavy proteinuria D. Occurs in acute glomerulonephritis
6. Edema that is characteristically dependent is
A. Nephrotic edema B. Nephritic edema
C. Pulmonary edema D. Cardiac edema
7. Pulmonary edema appears due to elevated pulmonary hydrostatic pressure when the fluid accumulation is
A. Two-fold B. Four-fold
C. Eight-fold D. Ten-fold
8. Active hyperemia is the result of which of the following?
A. Dilatation of capillaries B. Dilatation of arterioles
C. Venous engorgement D. Lymphatic obstruction
9. Brown induration, which is seen in sectioned surface of lung, shows
A. Pulmonary embolism B. Pulmonary hemorrhage
C. Pulmonary infarction D. CVC lung
10. The pathogenesis of endothelial cell injury in septic shock involves the following mechanisms except
A. Lipopolysaccharide from lysed bacteria injures the endothelium
B. Interleukin-1 causes endothelial cell injury
C. TNF-α causes direct cytotoxicity
D. Adherence of PMNs to endothelium causes endothelial cell injury
11. An intact endothelium elaborates the following anti-thrombotic factors except
A. Thrombomodulin B. ADPase
C. Tissue plasminogen activator D. Thromboplastin
12. The most common cause of arterial thromboemboli is
A. Cardiac thrombi B. Aortic aneurysm
C. Pulmonary veins D. Aortic atherosclerotic plaques
13. Venous emboli are most often lodged in
A. Intestines B. Kidneys
C. Lungs D. Heart
14. Pathologic changes between sudden decompression from high pressure to normal levels and decompression from
low  pressure to normal levels are
A. More marked in the former B. More marked in the latter
C. No difference between the two D. Acute form is more marked in the latter

https://t.me/DentalBooksWorld
174 Triumph's Complete Review of Dentistry

15. The infarct of which of the following organs is invariably hemorrhagic?


A. Infarct kidney B. Infarct spleen
C. Infarct lung D. Infarct heart
16. Milroy’s disease is
A. Cerebral edema B. Pulmonary edema
C. Hereditary lymphedema D. Postural edema
17. Pick the correct sequence
A. Renin–Angiotensin II–Angiotensin I–Angiotensinogen–Aldosterone
B. Angiotensinogen–Renin–Angiotensin II–Angiotensin I–Aldosterone
C. Renin–Angiotensinogen–Angiotensin I–Angiotensin II–Aldosterone
D. Aldosterone–Renin–Angiotensinogen–Angiotensin II–Angiotensin I
18. Which of the following is true?
A. Arterial thrombi are white and occlusive B. Venous thrombi are white and occlusive
C. Arterial thrombi are white and mural D. Venous thrombi are red and mural
19. Edema may be caused by any of the following, except
A. An increase in the plasma protein concentration B. An increase in the capillary hydrostatic pressure
C. An increase in the capillary permeability D. Lymphatic obstruction
20. The most common site of origin for venous thrombi leading to pulmonary embolism is
A. Ascending aorta B. Portal vein
C. Deep leg veins D. Right atrium
21. Edema is due to
A. Increased albumin in blood and decreased globin B. Decreased albumin concentration in blood
C. Increased osmotic pressure D. None of the above
22. Anasarca means
A. Abnormal inflammatory process
B. Severe generalized swelling
C. Absence of proliferation of vessels following inflammation
D. Presence of pus
23. All of the following are typically associated with the loss of 40% of the circulating blood volume, except
A. A decrease in the blood pressure B. A decrease in the central venous pressure
C. A decrease in the heart rate D. A decrease in the urine output
24. Which of the following is common in all forms of shock?
A. Sepsis B. Hypovolemia
C. Vasoconstriction D. Impaired tissue perfusion
25. In hypovolemic shock
A. The central venous pressure is high B. The extremities are pale, cold and sweating
C. There is always a site of bleeding D. Urine output is unaffected
26. Shock is a circulatory disturbance characterized by
A. Increased blood pressure B. Decreased volume of circulating blood
C. Elevated body temperature D. Decreased volume of interstitial fluid
27. Hypovolemic shock develops after loss of
A. 10% blood B. 20% blood
C. 30% blood D. 40% blood
28. Which of the following is the most important factor in the management of shock?
A. Blood pressure B. Cardiac output
C. CVP to 8 cm of water D. Deficiency of effective circulating blood volume
29. Heart failure cells are
A. Fibrocytes in myocardium B. Aschoff ’s giant cells
C. Hemosiderin laden macrophages in alveoli D. Hypertrophic myocardial fibers

https://t.me/DentalBooksWorld
Chapter 3 • General Pathology 175

30. “Heart failure cells” are seen in


A. Heart B. Lungs
C. Liver D. Kidney
31. Nutmeg liver occurs in
A. Jaundice B. Chronic venous congestion
C. Cirrhosis D. Hepatocellular carcinoma
32. In hemorrhage lost plasma is replaced by
A. Bone marrow B. Kidney
C. Spleen D. Muscle
33. In a thrombus, the dark lines of Zahn are due to
A. Coagulated fibrin B. Aggregated proteins
C. Aggregated platelets D. Aggregated RBC
34. The type of embolism seen in fractures of long bones is
A. Thromboembolism B. Air embolism
C. Fat embolism D. Amniotic fluid embolism
35. Caisson’s disease is caused by
A. Amniotic fluid embolism B. Hyper coagulability
C. Air or gas embolism D. Tumor embolism
36. The commonest site of thrombosis is
A. Arteries B. Veins
C. Capillaries D. Heart
37. Partial or complete obstruction of some part of the CVS by a foreign body transported by the blood stream is
termed
A. Thrombosis B. Coagulation
C. Ischemia D. Embolism
38. Which of the following being is the most frequent site of thrombus?
A. Veins of lower extremities B. Portal vein
C. Pulmonary vein D. Hepatic vein
39. An infarct is most frequently is characterized by what type of necrosis?
A. Fatty B. Caseous
C. Gangrenous D. Coagulative
40. Active hyperemia is seen in
A. Muscles during exercise B. Inflammation
C. Blushing D. All of the above
41. In left ventricular cardiac failure there is
A. Passive congestion of lungs B. Passive congestion in liver
C. Active hyperemia in lungs D. None of the above
42. Which of the following predisposes to thrombogenesis?
A. Endothelial injury B. Stasis of blood
C. Turbulence of blood D. All of the above
43. Thrombosis due to hypercoagulability is seen in
A. Women taking oral contraceptives B. Severe trauma or burns
C. Cardiac failure D. All of the above
44. Extravasation of blood into the tissues with resulting swelling is called
A. Hemothorax B. Hemoperitoneum
C. Hemopericardium D. Hematoma
45. Which of the following is correctly matched with the forms of extravasation hemorrhages?
A. Ecchymosis–large extravasation of blood in to the skin and mucous membrane
B. Purpura–small areas (1 cm) of hemorrhage into the skin and mucous membrane
C. Petechiae–small pin point hemorrhages
D. All of the above

https://t.me/DentalBooksWorld
176 Triumph's Complete Review of Dentistry

46. Which of the following is not an indicative of hemorrhage?


A. Ecchymosis B. Malena
C. Melanosis D. Petechiae
47. The process of formation of solid mass in circulation from the constituents of normal flowing blood is called
A. Clot formation B. Thrombosis
C. Embolism D. Infarction
48. All the following differs antemortem thrombus from a postmortem intravascular clots, except
A. Chicken fat appearance
B. Lines of Zahn
C. Dry and friable
D. Firm adherence to endothelium whereas absent in postmortem clots
49. Major and frequent influence for thrombus formation is
A. Alteration in blood flow B. Hypercoagulability
C. Endothelial damage D. Fatty streak
50. The major factors predisposing to thrombogenesis include all of the following except
A. Endothelial injury B. Hyper coagulability of blood
C. Altered blood flow D. Pattern of blood flow
51. Which of the following statements is true?
A. Arterial thrombi produces ischemia and infarction whereas cardiac and venous thrombi cause embolism
B. Arterial thrombi are white and nonocclusive (mural) whereas venous thrombi are red and occlusive
C. Venous thrombi are soft, red, and gelatinous whereas arterial thrombi are firm and white
D. All of the above
52. Ascites seen in alcoholic liver cirrhosis result form
A. Decreased protein production by liver B. Due to increased portal hypertension
C. Due to obstruction of bile duct D. All of the above
53. Left-sided heart failure results in
A. Edema of lungs B. Edema of spleen
C. Edema of legs D. Edema of liver
54. The most reliable postmortem feature of left-sided cardiac failure is
A. Systemic venous congestion B. Edematous ankles
C. Chronic venous congestion of lungs D. Enlargement of spleen and liver
55. The common cause of sudden death in myocardial infarction is
A. Pulmonary embolism B. Arrhythmias
C. Congestive heart failure D. None of the above
56. After myocardial infarction, all the following enzyme levels are high, except
A. Serum glutamic oxaloacetic transaminase (SGOT) B. Lactic dehydrogenase (LDH)
C. Creatine phosphokinase (CPK) D. Serum ornithine carbamoyltransferase (SOCT)
57. Three days after the onset of myocardial infarction, which enzyme level has the best predictive value?
A. Serum CPK B. Serum LDH
C. Serum SGOT D. Serum SGPT
58. Infarcts are not common in
A. Liver B. Lung
C. Kidney D. Both A and B
59. Pale infarcts are seen in all of the following except
A. Lung B. Liver
C. Kidney D. Spleen
60. Both pale or red infarcts are seen in
A. Lung B. Brain
C. Kidney D. Spleen

https://t.me/DentalBooksWorld
Chapter 3 • General Pathology 177

61. Earliest lesion in atherosclerosis is


A. Atheroma B. Fatty streak
C. Fibrous plaque D. None of the above
62. Major site of atherosclerosis is
A. Coronary artery B. Abdominal aorta
C. Thoracic aorta D. Internal carotid artery
63. Which of the following statements is incorrect?
A. Low-density lipoproteins has maximum association with atherosclerosis
B. High-density lipoproteins have protective role in atherosclerosis
C. Poly unsaturated fats lowers plasma cholesterol levels
D. HDL contains the highest cholesterol content
64. Risk factor for atherosclerosis is
A. Smoking B. Hypertension
C. Diabetes D. All of the above
65. Which of the following symptomatic atherosclerosis disease is correct?
A. Aorta – Aneurysm, thromboembolism B. Heart – MI, ischemic heart disease
C. Lower extremities – claudication gangrene D. All of the above
66. White infarcts occurs in one of the following organs?
A. Ovary B. Lung
C. Intestine D. Heart
67. In which one of the following organs, the venous emboli are most often lodged?
A. Lungs B. Kidneys
C. Intestines D. Heart
68. The necrotic tissue and deposits of immune complexes, complement, and plasma protein produce a smudgy eosinophilic
deposit termed
A. Coagulative necrosis B. Fibrinoid necrosis
C. Caseous necrosis D. Fatty necrosis
69. The infarct of which of the following organs is invariably hemorrhagic?
A. Kidney B. Lung
C. Spleen D. Heart
70. Venous emboli are most often lodged in
A. Intestines B. Lungs
C. Kidneys D. Heart
71. All of the following are true about blood coagulation, except
A. Factor X is a part of both intrinsic and extrinsic pathway.
B. Extrinsic pathway is activated by contact of plasma with negatively charged surfaces
C. Calcium is very important for coagulation
D. Intrinsic pathway can be activated in vitro
72. Among the following, Vitamin K helps in carboxylation of which amino acid in the clotting factors?
A. Aspartate B. Glutamate
C. Proline D. All the above
73. Due to which of the following, edema in nephrotic syndrome occurs?
A. Na+ and water restriction B. Increased venous pressure
C. Decreased serum albumin D. Decreased fibrinogen
74. The reason for thrombomodulin–thrombin complex preventing clotting is
A. Thrombomodulin inhibits prothrombin activator
B. The complex activates antithrombin III
C. Thrombomodulin–thrombin complex activates heparin
D. The complex removes thrombin and also activates protein C which inactivates the activated factors V and VIII
75. Clotting factors associated with Vitamin K is
A. IX, X B. I, V
C. VII, VIII D. I, VIII
https://t.me/DentalBooksWorld
178 Triumph's Complete Review of Dentistry

76. The endothelial cells produce thrombomodulin except


A. Hepatic circulation B. Cutaneous circulation
C. Cerebral microcirculation D. Renal circulation
77. Mark the procoagulation protein in the following
A. Thrombomodulin B. Protein C
C. Protein S D. Thrombin
78. Which of the following is false about Thromboxane A2?
A. Low-dose aspirin inhibits its synthesis B Causes vasoconstriction in blood vessels
C. Causes bronchoconstriction D. Secreted by WBC
79. The coagulation defects associated with increased coagulation are seen in which of the following?
A. Increased Protein C B. Increased Protein S
C. Increased Anti-thrombin III D. Protein C resistance
80. All of the following are anticoagulant substances, except
A. Antithrombin III B. Protein S
C. vWF D. Nitric oxide
81. Cause of edema can be described as
A. Decreased plasma protein concentration B. Increased lymph flow
C. Increased ECF volume D. Increased plasma protein concentration
82. The endothelium derived relaxing factor (EDRF) is associated with
A. Ras B. C-myc
C. Bcl D. N NOS
83. Which of the following is not involved in local hemostasis?
A. Fibrinogen B. Calcium
C. Vitamin K D. Collagen
84. Which one of the following is not synthesized in the liver?
A. Factor II B. Factor VII
C. Factor IX D. Factor VIII
85. Which of the following does tissue thromboplastin activate?
A. Factor VII B. Factor IV
C. Factor VI D. Factor XII
86. Platelet adhesion to collagen is mediated by
A. Factor VIII B. Factor IX
C. Von Willebrand factor D. Fibronectin
87. Gandy gamma body is typically seen in chronic venous congestion of
A. Lung B. Kidney
C. Spleen D. Liver
88. The extrinsic pathway of a clotting factors can be measured by
A. Prothrombin time B. Activated partial thromboplastin time
C. Bleeding time D. Clotting time
89. Antiphospholipid syndrome is not associated with
A. Recurrent abortion B. Venous thrombosis
C. Pancytopenia D. Antibody to lupus
90. The pale infarct is not seen in
A. Lungs B. Spleen
C. Kidney D. Heart
91. Which of the following conditions does not relate with congenital hypercoagulability state?
A. Protein C deficiency B. Protein S deficiency
C. Anti-phospholipid antibody syndrome D. MTHFR gene mutation
92. In which of the following is fat embolism commonly seen?
A. Head injuries B. Long bone fractures
C. Drowning D. Hanging

https://t.me/DentalBooksWorld
Chapter 3 • General Pathology 179

93. Virchow’s triad do not include


A. Injury to vein B. Venous thrombosis
C. Venous stasis D. Hypercoagulability of blood
94. Hypercoagulability due to defective factor V gene is called
A. Lisbon mutation B. Leiden mutation
C. Antiphospholipid syndrome D. Inducible thrombocytopenia syndrome
95. Arterial thrombosis is seen in
A. Homocysteinemia B. Phospholipid syndrome
C. Protein S deficiency D. Protein C deficiency
96. The hemorrhagic infarction is seen in which of the following?
A. Venous thrombosis B. Thrombosis
C. Embolism D. All the above
97. Hyperviscosity is seen in which of the following?
A. Cryoglobulinemia B. Multiple myeloma
C. MGUS D. All the above
98. Inherited coagulation disorders are seen in
A. Protein C deficiency B. Protein S deficiency
C. Leiden factor mutation D. All the above
99. Which of the following statements about pulmonary emboli is not true?
A. 60–80% pulmonary emboli are clinically silent
B. In more than 95% cases venous emboli originate from deep leg veins
C. Embolic obstruction of pulmonary vessels almost always cause pulmonary infarction
D. Embolic obstruction of medium sized arteries may result in pulmonary infarction
100. Which one of the following inherited disorders produces arterial thrombosis?
A. Factor V Leiden mutation B. Antithrombin deficiency
C. Homocysteinemia D. Protein S deficiency
101. The heart failure cells are seen in which of the following?
A. Chronic venous congestion of liver B. Chronic venous congestion of lung
C. Acute venous congestion of lung D. Acute venous congestion of liver
102. The necrosis with putrefaction is called
A. Desiccation B. Gangrene
C. Liquefaction D. Coagulative necrosis
103. Lines of Zahn are found in
A. Thrombus B. Infarct tissue
C. Postmortem clot D. All the above
104. Chicken fat clot is
A. Postmortem clot B. Thrombus
C. Infarct D. All
105. The five stages of hemostasis are given below in random order. Put them into their correct order.
A. Clot dissolution B. Blood coagulation
C. Vessel spasm D. Clot retraction
106. White infarct is seen in which of the following?
A. Lung B. Intestine
C. Heart D. Ovary
107. Which one of the following characterizes early endotoxic shock rather than hypovolemic or cardiogenic shock?
A. Warm skin B. Decreased cardiac output
C. Low mixed venous oxygen content D. Increased total peripheral resistance
108. White infarcts are not seen in
A. Liver B. Kidney
C. Spleen D. Heart

https://t.me/DentalBooksWorld
180 Triumph's Complete Review of Dentistry

109. Which of the following is False about DIC?


A. Increased fibrinogen B. Increased activated partial thromboplastin time
C. Decreased prothrombin time D. Increased fibrin degradation products
110. Edema conditions that represent a transudate secondary to an decrease in oncotic pressure is
A. Patient with pneumonia who has a pleural effusion
B. Patient with cirrhosis who has dependent pitting edema
C. Patient with edema of the arm post-modified radical mastectomy
D. Patient with a pulmonary infarction who has a left pleural effusion
111. Choose the initiating mechanism in endotoxic shock
A. Peripheral vasodilatation B. Endothelial injury
C. Increased vascular permeability D. Cytokine release
112. D-Dimer is the most sensitive diagnostic test for which one of the following?
A. Pulmonary embolism B. Acute pulmonary edema
C. Cardiac tamponade D. Acute myocardial infarction
113. Shock lung is characterized by which of the following?
A. Alveolar proteinosis B. Bronchiolitis obliterans
C. Diffuse pulmonary hemorrhage D. Diffuse alveolar damage
114. Histological features of shock include one of the following
A. ATN B. Pulmonary congestion
C. Depletion of lipids in adrenal cortex D. Hepatic necrosis
115. Following conditions associated with incoagulable state is
A. Abruption placentae B. Acute promyelocytic leukemia
C. Severe falciparum malaria D. Snake envenomation
116. Feature of disseminated intravascular coagulation is
A. Normal prothrombin time B. Reduced plasma fibrinogen
C. Normal platelet count D. Normal clotting time
117. Virchow’s triad with following factors favoring thrombosis includes
A. Leukocytosis, thrombocytosis, and erythrocytosis B. Occult cancer, heart failure, and jaundice
C. Hyperglycemia, hyperlipidemia, and hypertension D. Stasis, vascular injury, and hypercoagulability

NEOPLASIA
1. Malignant neoplasms show all the following features except
A. Disorganized cell structure B. Encapsulation
C. Invasion of blood vessels D. Rapid, erratic growth
2. The term metaplasia refers to
A. Irregular, atypical proliferative changes in epithelial or mesenchymal cells
B. Loss of cell substance producing shrinkage of the cells
C. Replacement of one type of adult cell by another type of adult cell
D. None of the above
3. Exfoliative cytology is indicated in
A. Heavily keratinized lesions of oral cavity B. Precancerous lesions of oral cavity
C. Cigarette smokers D. Mass screening of cervical cancer
4. Change in structure and functions of a tissue is called
A. Dysplasia B. Metaplasia
C. Anaplasia D. Aplasia
5. The most definite feature of a malignant tumor is
A. Hemorrhage B. Increased mitoses
C. Metastasis D. Necrosis
6. Neoplasia of blood vessel is called
A. Angioma B. Hematoma
C. Lymphosarcoma D. Papilloma

https://t.me/DentalBooksWorld
Chapter 3 • General Pathology 181

7. The increase in size of individual cells is referred as


A. Hypertrophy B. Hyperplasia
C. Hypodontia D. Differentiation
8. Increased proliferation of cells is called
A. Hypertrophy B. Atrophy
C. Hyperplasia D. Metaplasia
9. Burkitt’s lymphoma is associated with the proliferation of
A. T-cell B. B-cell
C. Lymph vessels D. Lymph nodes
10. Incidence of most common malignant tumors in men in India is in
A. Lungs B. Oral cavity
C. Prostate D. Liver
11. Incidence of most common malignant tumors in women is
A. Breast B. Lung
C. Cervix D. Ovary
12. Carcinogens which induce tumors are
A. Chemicals B. Radiation
C. Viruses D. All of the above
13. Oncogenic viruses in human are
A. HPV B. EBV
C. Hepatitis-B virus D. All of the above
14. Most tumors are
A. Biclonal in origin B. Monoclonal in origin
C. Multiclonal in origin D. None of the above
15. The wasting syndrome associated in cancer is
A. Achalasia B. Cachexia
C. Atelectasis D. Cacogeusia
16. Rodent ulcer is most commonly seen on
A. Face below ala-tragus line B. Face above ala-tragus line
C. Anywhere on hairless skin D. All of the above
17. Basal cell carcinoma occurs on
A. Skin and pilosebaceous adnexa B. Skin and mucosa
C. Skin, lips, and tongue D. All of the above
18. The predisposing factor for squamous cell carcinoma is
A. Solar keratosis B. Chronic ulcers
C. Leukoplakia D. All of the above
19. Keratin pearls with downward proliferation of epithelial masses is characteristic of
A. Squamous cell carcinoma B. Basal cell carcinoma
C. Epidermoid carcinoma D. Melanoma
20. Nevus cells are
A. Modified melanocytes B. Cells containing excessive melanin pigment
C. Melanocytes which are functionally more active D. Large melanocytes
21. Malignant tumor of skeletal muscle is
A. Rhabdomyoma B. Rhabdomyosarcoma
C. Leiomyoma D. Leiomyosarcoma
22. Tumors arising from secretory and glandular epithelium are
A. Adenomas B. Adenosarcomas
C. Angiomas D. Both A and B
23. Epstein–Barr virus is associated with
A. Nasopharyngeal carcinoma B. Burkitt’s lymphoma
C. Infectious mononucleosis D. All of the above

https://t.me/DentalBooksWorld
182 Triumph's Complete Review of Dentistry

24. Immune response to tumor is mediated by


A. Cytotoxic T-lymphocytes B. Natural killer cells
C. Humoral mechanism D. All of the above
25. Pap smear is used to detect dysplasia, carcinoma in situ, and invasive carcinoma of uterine cervix. This is based on the
principle of
A. Neoplastic cells are cohesive B. Neoplastic cells are noncohesive
C. Hyperchromatic and pleomorphism of tumor cells D. All of the above
26. Tumor which shows origin from more than one germ layers is known as
A. Teratoma B. Pleomorphic tumors
C. Choristoma D. Hamartoma
27. Malignant tumor of epithelial origin is
A. Carcinoma B. Sarcomas
C. Papilloma D. All of the above
28. Which of the following statement is correct?
A. Direct acting carcinogens does not undergo any prior metabolic activation
B. Indirect acting carcinogens induce carcinogenicity after chemical transformation
C. Both of the above
D. None of the above
29. In dysplasia mitotic figures are seen in
A. In basal layers of epithelium B. In surface layers only
C. From basal layer to surface D. None of the above
30. Anaplasia is
A. Lack of functional differentiation B. Lack of structural differentiation
C. Is a characteristic feature of malignancy D. All of the above
31. Nuclear cytoplasmic ratio in malignant cells is
A. 1:6 B. 1:5
C. 1:1 D. 6:4
32. Which of the following is a correct statement?
A. Grading is the degree of macroscopic and microscopic differentiation of tumor
B. Staging is the extent of spread of tumor
C. TNM and AJC systems can be used for staging malignant tumors
D. All of the above
33. Onion skin appearance is seen in
A. Ewing’s sarcoma B. Osteosarcoma
C. Osteoma D. Fibro sarcoma
34. Sunburst appearance and Codman’s triangle are seen in
A. Osteosarcoma B. Osteoma
C. Chondrosarcoma D. None of the above
35. The most common malignant lesion of bone
A. Osteosarcoma B. Osteochondroma
C. Ewing’s sarcoma D. Metastatic carcinoma
36. Lymphocytic infiltrate is frequently present in the following tumors indicative of host immune response except
A. Seminoma testis B. Medullary carcinoma breast
C. Papillary carcinoma thyroid D. Malignant melanoma
37. Antibody-stain that is used in immunohistochemistry to identify epithelial cells is
A. Desmin B. Vimentin
C. Cytokeratin D. Neurofilaments
38. Name the viral infection that is not known to produce any human tumor
A. Polyoma virus B. EBV
C. HSV D. HTLV

https://t.me/DentalBooksWorld
Chapter 3 • General Pathology 183

39. The increase in size of uterus during pregnancy is an example of


A. Hyperplasia B. Hypertrophy
C. Both of the above D. None of the above
40. The increase in the size of left kidney following the removal of right kidney (vicarious hypertrophy) is due to
A. Hyperplasia B. Hypertrophy
C. Both of the above D. None of the above
41. The most characteristic feature of a neoplasm as opposed to inflammatory over growth is
A. Abnormal mitosis B. Size of the lesion
C. Progressive growth after removal of causative stimuli D. Tendency to grow rapidly
42. A simple bacterial test for mutagenic carcinogens is
A. Ame’s test B. Redox test
C. Bacteriophage D. Gene splicing
43. All of the following statements are true regarding reversible cell injury, except
A. Formation of amorphous densities in the mitochondrial matrix
B. Diminished generation of adenosine triphosphate (ATP)
C. Formation of blebs in the plasma membrane
D. Detachment of ribosomes from the granular endoplasmic reticulum
44. The cell with increased mitotic rate and resembling the undifferentiated mesenchymal cells of the same origin is
A. Anaplastic B. Dysplastic
C. Metaplastic D. Hyperplastic
45. Which one of the following is not a malignant feature?
A. Aplasia B. Anaplasia
C. Abnormal mitosis D. Pleomorphism
46. Features of a benign tumor are all, except
A. Absence of hyperchromasia of nucleus B. Retained basal polarity
C. Slow growth D. Presence of local invasion
47. Philadelphia chromosome is consistently found in
A. AML B. CLL
C. CML D. All of the above
48. The most common site of metastases of osteosarcoma is
A. Liver B. Spleen
C. Lymph nodes D. Lung
49. All of the following viruses are capable of causing human cancers, except
A. Papilloma virus B. Hepatitis B virus
C. Epstein Barr virus D. Parvovirus B-19
50. Somatic mutation of PTEN is seen in
A. Retinoblastoma B. Osteosarcoma
C. Carcinoma breast D. Endometrial carcinoma
51. Hypercalcemia as a paraneoplastic syndrome is observed in the following tumors except
A. Squamous cell carcinoma lung B. Small cell carcinoma lung
C. Renal cell carcinoma D. Breast cancer
52. 45 chromosomes are seen in
A. Turner’s syndrome B. Down’s syndrome
C. Dentinogenesis imperfecta D. Treacher Collins syndrome
53. Name the important examples of tumor suppressor genes implicated in human cancers include the following except
A. RB gene B. TP53
C. APC D. ERB-B
54. In a 31-year female patient, left supraclavicular lymph node biopsy histologically shows acidophilic owl eye nucleoli lying
freely in empty spaces showing immuno histocompatibility with CD 15 and CD 30; it is suggestive of which of the following
A. Nodular sclerosis Hodgkin’s lymphoma B. Lymphocytic predominant Hodgkin’s lymphoma
C. Reactive nodular hyperplasia D. Large cell lymphoma

https://t.me/DentalBooksWorld
184 Triumph's Complete Review of Dentistry

55. The most important factor associated with causation of head and neck carcinoma is
A. Intravenous drug abuse B. Exposure to nickel
C. History of syphilis D. Tobacco use
56. The most common benign tumor of the lung is
A. Hamartoma B. Alveolar adenoma
C. Teratoma D. Fibroma
57. The most common variety of soft tissue sarcoma is
A. Rhabdomyosarcoma B. Liposarcoma
C. Malignant fibrous histiocytoma D. Synovial sarcoma
58. Invasive squamous cell carcinoma is differentiated from carcinoma in situ by
A. Penetration of basement membrane B. Number of mitotic figures
C. Increased in size of cell D. Nuclear pleomorphism
59. Test used to differentiate between chromosomal pattern of normal cell and cancer cell is
A. FISH B. PCR
C. Karyotyping D. Comparative genomic hybridization
60. Retinoblastoma spreads through
A. Vascular invasion B. Direct invasion
C. Optic nerve D. Lymphatic
61. Fibrous histiocytoma involves most commonly
A. Eyelid B. Epibulbar
C. Intraocular D. Orbit
62. Chemical carcinogen implicated in the causation of mesothelioma is
A. Arsenic B. Vinyl chloride
C. Asbestos D. Nickel
63. Human papilloma virus type responsible for cervical carcinoma
A. 2 and 4 B. 16 and 18
C. 20 and 40 D. 6 and 11
64. Most common paraneoplastic syndrome associated with squamous cell carcinoma of the lung is
A. Hypercalcemia B. Cushing syndrome
C. SIADH D. Carcinoid syndrome
65. Which of the following is not an anti-angiogenesis factor?
A. Thrombospondin-1 B. Basic fibroblast growth factor (bFGF)
C. Endostatin D. Angiostatin
66. Which of the following is characteristic of a malignant tumor?
A. Well differentiated B. Slow growing
C. Cohesive and well demarcated D. Metastasis
67. Differentiation of follicular carcinoma from follicular adenoma of thyroid gland is via
A. Hurthle cell change B. Lining of tall columnar and cuboidal cells
C. Vascular invasion D. Increased mitoses
68. Which of the following is not a malignant tumor?
A. Chloroma B. Fibromatosis
C. Askin’s tumor D. Liposarcoma
69. Which of the following is not a feature of malignant transformation by cultured cells?
A. Increased cell density B. Increased requirement for growth factors
C. Alterations of cytoskeletal structures D. Loss of anchorage
70. Carcinoma that is most frequently metastasizes to brain is
A. Small cell carcinoma lung B. Prostate cancer
C. Rectal carcinoma D. Endometrial cancer
71. Chemotherapeutic drugs can cause
A. Only necrosis B. Only apoptosis
C. Both necrosis and apoptosis D. Anoikis

https://t.me/DentalBooksWorld
Chapter 3 • General Pathology 185

72. Reversible loss of polarity with abnormality in size and shape of cells is known as
A. Metaplasia B. Dysplasia
C. Hyperplasia D. Anaplasia
73. Predisposing factors for skin cancer is
A. Smoking B. UV light
C. Chronic ulcer D. All the above
74. In which of the following the strong propensity for vascular invasion is seen?
A. Prostatic carcinoma B. Hepatocellular carcinoma
C. Bronchogenic carcinoma D. Gastric carcinoma
75. Earliest changes of neoplastic transformation as seen at a microscopic level is called
A. Hyperplasia B. Metaplasia
C. Dysplasia D. Carcinoma in situ
76. The spread of squamous cell carcinoma is through
A. Hematogenous route B. Lymphatic route
C. Direct invasion D. All
77. The following are hereditary diseases that have higher incidence of cancers due to inherited defect in DNA repair
mechanism, except
A. Ataxia telangiectasia B. Xeroderma pigmentosum
C. Familial polyposis coli D. Bloom’s syndrome
78. Hamartoma is
A. Proliferation of cells in foreign sit B. Proliferation of native cells in tissue
C. Malignant condition D. Acquired condition
79. Sure sign of malignancy is
A. Mitoses B. Polychromasia
C. Nuclear pleomorphism D. Metastasis
80. Test for mutagenicity is
A. Kveim’s test B. Ame’s test
C. Schilling’s test D. Mantoux test
81. Malignancy is typically associated with disordered differentiation and maturation. Which of the following mentioned
options best describe anaplasia?
A. Hepatic tumor cells synthesizing bile B. Skin tumor cells producing keratin pearl dysplasia
C. Bronchial epithelial cells producing keratin pearls D. Muscle tumor cells forming giant cells
82. The criteria used to determine whether pheochromocytoma lesion is benign or malignant is
A. Blood vessel invasion B. Cannot be determined by microscopic examination
C. Hemorrhage and necrosis D. Nuclear pleomorphism
83. Most reliable feature of malignant transformation of pheochromocytoma is
A. Presence of mitotic figures B. Presence of metastasis to other organs
C. Vascular/capsular invasion D. All of the above
84. At a localized region overgrowth of a skin structure can be
A. Hamartoma B. Malignant tumor
C. Choristoma D. Polyp
85. Which of the following mediates cell–matrix adhesions?
A. Cadherins B. Integrins
C. Selectins D. Calmodulin
86. Which of the following is not a precancerous condition?
A. Crohn’s disease B. Ulcerative colitis
C. Leukoplakia D. Xeroderma pigmentosum
87. The feature that differentiates invasive carcinoma from carcinoma in situ
A. Anaplasia B. Number of mitosis
C. Basement membrane invasion D. Pleomorphism

https://t.me/DentalBooksWorld
186 Triumph's Complete Review of Dentistry

88. Which one of the following is the sure sign of malignancy?


A. Mitoses B. Polychromasia
C. Nuclear pleomorphism D. Metastasis
89. True regarding oncogenesis is
A. Proto-oncogenes are activated by chromosomal translocation
B. Malignant transformations involves accumulation of mutations in proto-oncogenes and tumor suppressor genes
C. Point mutation of somatic cells
D. Increase in telomerase activity causes anti-tumor effects
90. The phase at which tumor suppressor gene P53 induces cell arrest
A. G2–M phase B. S–G2 phase
C. G1–S phase D. G0–phase
91. Which of the following is not a premalignant condition?
A. Fragile X syndrome B. Down’s syndrome
C. Blount’s syndrome D. Fanconi’s syndrome
92. Important cyclins in cell cycle include the following except:
A. Cyclin A B. Cyclin B
C. Cyclin C D. Cyclin D
93. In which of the following E cadherin gene deficiency is seen
A. Gastric cancer B. Intestinal cancer
C. Thyroid cancer D. Pancreatic cancer
94. Mutation of which gene causes Li Fraumeni syndrome
A. p21 B. p53
C. p41 D. p43
95. Which of the following is not a cyclin-dependent kinase (CDK) inhibitor?
A. p21 B. p27
C. p53 D. p57
96. The phase at which cells are most radiosensitive
A. S-phase B. M-phase
C. G1-phase D. G0-phase
97. Which of the following is false about Fanconi anemia?
A. Defect in DNA repair B. Bone marrow hyper function
C. Congenital anomaly present D. Increased chances of cancer
98. The role of HER2/neu receptor
A. Predicting therapeutic response B. Diagnosis of breast cancer
C. Screening of breast cancer D. Recurrence of tumor
99. Which of the following is false about Fanconi anemia?
A. Autosomal dominant B. Bone marrow show pancytopenia
C. Usually aplastic anemia D. It is due to defective DNA repair
100. Growth factor oncogene is
A. Myc B. Fos
C. Sis D. Jun
101. Choose a tumor suppressor gene
A. myc B. fos
C. ras D. Rb
102. Choose a nontumor suppressor gene
A. WT-1 B. Rb
C. p53 D. ras
103. Which of the following is the “guardian of the genome”?
A. p53 B. Mdm2
C. p14 D. ATM

https://t.me/DentalBooksWorld
Chapter 3 • General Pathology 187

104. True about tumor suppressor gene p53, except


A. It regulates certain genes involved in cell cycle regulation
B. Its increased levels can induce apoptosis
C. Its activity in the cells decreases following UV irradiation and stimulates cell cycle
D. Mutations of the p53 gene are the most common genetic alteration seen in human cancer
105. Which of the following is true regarding proto-oncogenes?
A. Important for normal cell growth B. May get converted into oncogenes
C. C-myc overexpression causes lymphoma D. Their mutation causes retinoblastoma
106. Which of the following is true regarding oncogene?
A. Present in normal cell
B. They are of viral origin
C. They are transduced from virus infected cells
D. P53 is most common oncogene mutation causing malignancy
107. Which of the following enhances cancer cell survival?
A. Suppression of p53 protein B. Over expression of p53 gene
C. bcl-2 D. bax
108. Xeroderma pigmentosum is caused due to an abnormalities in
A. Mismatch repair B. Base excision repair
C. Nucleotide excision repair D. SOS repair
109. Which of the following causes oncogenesis due to its increased expression?
A. IGF receptor B. EGF receptor
C. GH receptor D. Aldosterone receptor
110. Tumor suppressor genes are all, except
A. APC B. p53
C. Rb D. C-myc
111. Angiogenesis is
A. Formation of the new blood vessels B. Repair by connective tissues
C. Formation of the blood clot D. All of the above
112. APC gene is located on chromosome
A. Chromosome 5 B. Chromosome 6
C. Chromosome 9 D. Chromosome 11
113. p53 induces cell cycle arrest at
A. G2–M phase B. S–G2 phase
C. G1–S phase D. G0 phase
114. MYC gene is
A. Protein kinase inhibitor B. Growth factor inhibitor
C. GTPase D. Transcription activator
115. Choose the condition with DNA repair defect
A. Retinoblastoma B. Neurofibromatosis
C. Xeroderma pigmentosum D. MEN-I
116. Which of the following is not a tumor marker?
A. Beta-2 macroglobulin B. HCG
C. Alpha-fetoprotein D. CEA
117. Which of the following causes posttransplant lymphoma?
A. CMV B. EBV
C. Herpes simplex D. HHV-6
118. Choose the essential condition for tumor metastasis
A. Angiogenesis B. Tumorigenesis
C. Apoptosis D. Inhibition of tyrosine kinase activity

https://t.me/DentalBooksWorld
188 Triumph's Complete Review of Dentistry

119. The following statements about carcinogenesis is true, except


A. Asbestos exposure increases the incidence of lung cancer
B. Papilloma viruses produce tumors in animals but not in humans
C. Exposure to aniline dyes predisposes to cancer of the urinary bladder
D. Hepatitis B virus has been implicated in hepatocellular carcinoma
120. Name an oncogenic RNA virus
A. Hepatitis B virus B. Human papilloma virus
C. Epstein Barr virus D. Hepatitis C virus
121. Smoking is not a risk factor for
A. Oral B. Bronchial
C. Bladder D. Thyroid
122. Radiation exposure during infancy has been linked to which carcinoma?
A. Breast B. Melanoma
C. Thyroid D. Lung
123. UV radiation affects cells by
A. Preventing formation of pyrimidine dimers B. Stimulating formation of pyrimidine dimers
C. Preventing formation of purine dimers D. All of the above
124. Most radiosensitive cells among the following are
A. Neutrophils B. Lymphocytes
C. Erythrocytes D. Megakaryocytes
125. Which of the following is a BCL2 marker?
A Follicular lymphoma B. Mycosis fungoides
C B-cell lymphoma D. Mantle cell lymphoma
126. Diagnostic tumor marker of liver carcinoma is
A. CEA B. AFP
C. CA-125 D. All of the above
127. Hamartoma refers to
A. Tumor differentiating toward more than one cell line
B. Tumor arising from totipotent cells
C. Mass of disorganized but mature cells indigenous to the part
D. Mass of ectopic rests of normal tissue
128. Increased number of normal mitoses may not be present in
A. Bone marrow cells B. Nails
C. Hepatocytes D. Intestinal epithelium
129. A tumor is termed medullary when it is most entirely composed of
A. Amyloid stroma B. Large areas of necrosis
C. Abundant lymphoid tissue D. Parenchymal cells
130. Malignant tumors that do not metastasize is
A. Synovial sarcoma B. Malignant mesothelioma
C. Glioma D. Neuroblastoma
131. The form of ionizing radiation exposure associated with highest risk of cancer is
A. α-Rays B. β-Rays
C. γ-Rays D. X-rays
132. Degradation of ECM is not brought about by
A. Proteases B. Metalloproteinases
C. Free radicals D. Cathepsin D
133. Grading of tumors do not depend on
A. Degree of anaplasia B. Metastatic spread
C. Rate of growth of cells D. Degree of differentiation

https://t.me/DentalBooksWorld
Chapter 3 • General Pathology 189

134. Xeroderma pigmentosum patients are prone to develop cancers but not
A. Basal cell carcinoma B. Sweat gland carcinoma
C. Malignant melanoma D. Squamous cell carcinoma
135. Under which of the following is the primary target of reactive electrophiles?
A. Cytochrome P-450 B. RNA
C. DNA D. Mitochondria
136. Carcinogenic influence of radiation appears after
A. <2 years B. 2–5 years
C. 5–10 years D. >10 years

BLOOD AND LYMPHATICS


1. A reduction in the total leucocyte count is called
A. Leucocytosis B. Leucopenia
C. Leucorrhoea D. Leukemia
2. Reduced number of platelets is found in all the conditions except
A. Disseminated intravascular coagulation B. Aplastic anemia
C. Acute myelocytic leukemia D. Von Willebrand disease
3. A hemophilic patient should not be given with which of the following?
A. Factor VIII concentrate B. Cryoprecipitate
C. EACA D. Platelet factor
4. Normal adult hemoglobin contains
A. One alpha chain and one beta chain B. One alpha chains and two beta chains
C. One beta chains and two alpha chains D. Two alpha chains and two beta chains
5. In sickle cell anemia there is substitution of
A. Valine for glutamic acid at the sixth position of beta chain
B. Phenylalanine for glutamic acid
C. Tyrosine for valine at the sixth position at beta chain
D. All of the above
6. Which of the following is not true about hemophilia A?
A. Hemophilia A is due to deficiency of factor VIII
B. In hemophilia A females are carriers
C. Desmopressin is useful in the management of hemophilia A
D. Levels of factor VIII less than 50% are associated with spontaneous hemorrhage
7. In megaloblastic anemia the cells are
A. Macrocytic hyperchromic B. Macrocytic hypochromic
C. Macrocytic normochromic D. None of the above
8. A 15-year boy complains of gingival bleeding, oral ulcers, anemia with hepatomegaly but no lymphadenopathy, total
leukocyte count of 1 lakh cells per mm3 is suffering from which one of the following?
A. Acute myeloid leukemia B. Chronic monocytic leukemia
C. Chronic lymphoblastic leukemia D. Chronic myeloid leukemia
9. Pernicious anemia is associated with the deficiency of
A. Folic acid B. Vitamin B1
C. Vitamin B6 D. Vitamin B12
10. The protein that carries iron form the gut to the body tissues is called
A. Ferritin B. Hemosiderin
C. Myoglobin D. Transferrin
11. A patient with dyspnea on slight exertion has multiple petechial spots and spontaneous hemorrhage. His RBC count
is less than 1 lakh/mm3. His hematocrit and hemoglobin is low. Most probable diagnosis is
A. Pernicious anemia B. Thalassemia
C. Aplastic anemia D. Sprue

https://t.me/DentalBooksWorld
190 Triumph's Complete Review of Dentistry

12. A patient with a bleeding disorder with increased bleeding time and normal clotting time is suffering from
A. Classic hemophilia B. Christmas disease
C. Vitamin K deficiency D. Idiopathic thrombocytopenic purpura
13. Gingiva are enlarged in leukemia because of
A. Capillary dilation B. Erythrocyte engorgement
C. Edema D. WBC infiltration
14. Glucose-6-phosphate dehydrogenase deficiency causes
A. Hemophilia B. Hemolytic anemia
C. Aplastic anemia D. Megaloblastic anemia
15. In leucopenia, which cell type is predominantly involved?
A. Erythrocytes B. Granulocytes
C. Eosinophils D. Monocytes
16. Reed–Sternberg cells are seen in histopathological examination of
A. African jaw lymphoma B. Hodgkin’s disease
C. Burkitt’s lymphoma D. Infectious mononucleosis
17. Hemophilia
A. Affects males and females equally B. Increased clotting time in all patients
C. Nerve blocks can be given safely D. Is a congenital disorder
18. Hemophiliacs show
A. Increased bleeding time and clotting time B. Decreased bleeding time and clotting time
C. Decreased bleeding time and increased clotting time D. Normal bleeding time and increased clotting time
19. In iron deficiency anemia there is
A. Decrease in hemoglobin B. Increase in hemoglobin
C. Increase in platelets D. Decrease in platelets
20. Schilling test is performed to find out
A. Folic acid level B. B12 malabsorption
C. Pancreatic enzyme deficiency D. Coronary artery disease
21. Both beta chains of hemoglobin are abnormal in
A. Heterozygous sickle cell trait B. Thalassemia major
C. Homozygous sickle cell anemia D. Megaloblastic anemia
22. Target cells are seen in peripheral blood in
A. Thalassemia B. Pernicious anemia
C. Aplastic anemia D. Sickle cell anemia
23. Christmas disease is due to deficiency of
A. Factor IX B. Factor X
C. Factor VIII D. Factor V
24. Which of the following is not an example of massive splenomegaly?
A. Chronic myeloid leukemia B. Chronic malaria
C. Tropical splenomegaly D. Acute lymphoblastic leukemia
25. Hypersegmented neutrophils are seen in
A. Megaloblastic anemia B. Iron deficiency anemia
C. Thalassemia D. Idiopathic thrombocytopenic purpura
26. Sideroblastic anemia is caused due to
A. Mercury B. Lead
C. Arsenic D. Iron
27. Which of the following is commonly involved in multiple myeloma?
A. Clavicle B. Vertebrae
C. Pelvis D. Lungs

https://t.me/DentalBooksWorld
Chapter 3 • General Pathology 191

28. Hemophilia A is caused due to deficiency of


A. Factor X B. Factor XI
C. Factor IX D. Factor VIII
29. In Hodgkin’s disease, the lymphoma cells seen are
A. Eosinophils B. Plasma cells
C. Reed–Sternberg cells D. Giant cells
30. In Fanconi anemia there is a
A. Deficiency of copper B. Mutation of DNA repair gene
C. No increased risk for squamous cell carcinoma D. Purely nutritional disorder etiology
31. Bence Jones proteins are associated with
A. Hodgkin’s lymphoma B. Multiple myeloma
C. Burkitt’s lymphoma D. Infectious mononucleosis
32. Which of the following is a nutritional deficiency anemia?
A. Aplastic anemia B. Sickle cell anemia
C. Megaloblastic anemia D. Hemolytic anemia
33. Which of the following is a hemolytic anemia?
A. Sickle cell anemia B. Thalassemia
C. Hereditary spherocytosis D. All of the above
34. Increased iron binding capacity and decreased serum iron is seen in which anemia?
A. Iron deficiency B. Aplastic anemia
C. Sickle cell anemia D. Chronic infections
35. Normocytic, normochromic anemia is seen in
A. Hemolytic anemia B. Chronic diseases
C. Acute blood loss D. All of the above
36. Prothrombin time indicates the integrity of
A. Intrinsic pathway B. Extrinsic pathway
C. Capillary function D. All of the above
37. Reliable screening test for hemophilia is
A. APTT B. BT
C. PT D. CBP
38. The following are recognized causes of neutropenia except
A. Corticosteroid therapy B. Typhoid fever
C. Aplastic anemia D. Viral infection
39. Which of the following is not seen in classical triad of megaloblastic anemia?
A. Howell–Jolly bodies B. Reticulocytosis
C. Hypersegmented neutrophil D. Macro-ovalocytes
40. Increased bleeding time is seen in all, except
A. Thrombocytopenia B. Von Willebrand disease
C. Hemophilia D. Ehlers–Danlos syndrome
41. Hemophilia is best treated by
A. Dried freeze plasma B. Factor VIII concentrate
C. Whole blood D. Cryoprecipitate
42. A shift to left indicates an increase in number of
A. Immature neutrophils B. Erythrocytes
C. Mature neutrophils D. Platelets
43. Increase in number of RBC is seen in
A. Neutropenia B. Leukemia
C. Polycythemia D. Anemia
44. Deposition of bile pigment in the basal ganglia is called
A. Jaundice B. Kernicterus
C. Hemosiderosis D. All of the above

https://t.me/DentalBooksWorld
192 Triumph's Complete Review of Dentistry

45. Pallor, spoon-shaped nails, atrophic glossitis, with accompanied dysphagia is a typical picture of
A. Plummer–Vinson syndrome B. Trotter’s syndrome
C. Vincent stomatitis D. None of the above
46. Neurologic abnormalities occur with
A. Foliate deficiency anemia B. Pernicious anemia
C. Vitamin B12 deficiency anemia D. Both B and C
47. Intrinsic factor for absorption of Vit B12 is secreted in
A. Duodenum B. Stomach
C. Intestine D. Ileum
48. Sickle cell anemia precipitates when
A. Oxygen tension goes down B. Increased viscosity of blood
C. There is dehydration D. All of the above
49. Aplastic anemia results from
A. Cytotoxic drugs B. Whole body irradiation
C. HIV infection D. All of the above
50. The characteristic finding in chronic myeloid leukemia is
A. Reduced score of alkaline phosphatase in granulocytes B. Reduced score of acid phosphatase in granulocytes
C. Total lack of platelets D. Total lack of neutrophils
51. Erythroblastosis fetalis is a condition seen when there is
A. Rh –ve mother and Rh –ve fetus B. Rh –ve mother and Rh +ve fetus
C. Rh +ve mother and Rh –ve fetus D. Rh +ve mother and Rh +ve fetus
52. Hodgkin’s lymphoma can be distinguished from non–Hodgkin’s lymphomas by
A. Reed–Sternberg cells B. Systemic manifestations
C. Occurrence in young adults D. All of the above
53. All the following can be transmitted through blood transfusion, except
A. Malaria B. Leukemia
C. Hepatitis D. AIDS
54. The patient’s urine with a particular protein appears normal, but on standing it becomes dark and the dark color
disappears on heating. Which of the following is the cause?
A. Osteoporosis B. Multiple myeloma
C. Infectious mononucleosis D. T-cell tumor
55. All of the following causes excessive bleeding during tooth extraction, except
A. Hemophilia A B. Hemophilia B
C. Anti-thrombin III deficiency D. Von Willebrand disease
56. Autoimmune hemolytic anemia is seen in
A. AML B. CML
C. CLL D. ALL
57. Osteomalacia is associated with
A. Decrease in osteoid volume B. Decrease in osteoid surface
C. Increase in osteoid maturation time D. Increase in mineral apposition rate
58. The fading of cellular chromatin is
A. Karyolysis B. Karyorrhexis
C. Pyknosis D. Cytolysis
59. Amyloid material is best diagnosed by which of the following?
A. Polarized microscopy B. Electron microscopy
C. X-ray crystallography D. Scanning electron microscopy
60. In sickle cell anemia there is
A. 75–100% hemoglobins B. 10–20% hemoglobins
C. 20–30% hemoglobins D. 50–60% hemoglobins

https://t.me/DentalBooksWorld
Chapter 3 • General Pathology 193

61. Lymphatic leukemoid reaction is usually seen in the following, except


A. Tuberculosis B. Pertussis
C. Lobar pneumonia D. Viral infections
62. Increased levels of hemoglobin A2 is characteristic of
A. Sickle cell trait B. β-Thalassemia trait
C. Glucose-6-phosphate dehydrogenase deficiency D. α-Thalassemia
63. Type of acute myelogenous leukemia (AML) associated with a high incidence of disseminated intravascular
coagulation is
A. Acute erythroleukemia B. Acute promyelocytic leukemia
C. Acute megakaryocytic leukemia D. Acute myelomonocytic leukemia
64. A 4-year-old child presented with progressive anemia, jaundice, and failure to thrive. O/E: Pallor, splenomegaly are
seen. Peripheral smear showed normoblasts and small round intensely stained red cells. Which of the following is
the cause?
A. Hereditary spherocytosis B. Thalassemia
C. Sickle cell anemia D. Vitamin B12 deficiency anemia
65. Thrombocytosis is seen in
A. Osteoporosis B. Disseminated intravascular coagulation
C. Hemolytic anemia D. Thiazide therapy
66. Primary hemostasis is disturbed in
A. Platelet disorder B. Lupus anticoagulant
C. Hemophilia D. Liver disease
67. Reed–Sternberg cells in Hodgkin’s lymphoma are derived from
A. Macrophages B. NK cell
C. T cell D. B cell
68. Thrombocytopenia due to increased platelet destruction is seen in
A. Aplastic anemia B. Cancer chemotherapy
C. Acute leukemia D. Systemic lupus erythematosus
69. Cooley’s anemia is also called
A. Mediterranean anemia B. β-Thalassemia major
C. Erythroblastic anemia D. All of the above
70. A 21-year male complains of recurrent attacks of sore throat since 2 years. The total leukocyte count was 1,000/µL.
A differential count revealed severe neutropenia. The diagnosis is
A. Subleukemic leukemia B. Infectious mononucleosis
C. Agranulocytosis D. Leukoerythroblastic anemia
71. Syndrome associated with increased risk of leukemia is
A. Plummer–Vinson syndrome B. Klinefelter syndrome
C. Sturge–Weber syndrome D. Multiple hamartoma syndrome
72. “Chicken-wire” appearance of enlarged bone marrow space is seen in
A. Fetal alcohol syndrome B. Sickle cell anemia
C. Hemophilia A D. β-Thalassemia major
73. Which is responsible for adhesion of platelet on exposed collagen fibril after an injury?
A. Von Willebrand factor B. Factor 8
C. Factor 9 D. Fibronectin
74. Which of the following condition is associated with brown skin pigmentation, hypoplasia of kidney and spleen, absent
or hypoplastic thumb or radius, microcephaly, and mental and sexual retardation?
A. Aplastic anemia B. Pernicious anemia
C. Sickle cell anemia D. Megaloblastic anemia
75. The presence of Heinz bodies in erythrocytes indicates
A. Malarial infection B. Hemoglobin abnormalities
C. Oxidative stress D. Degraded bacterial forms

https://t.me/DentalBooksWorld
194 Triumph's Complete Review of Dentistry

76. Direct Coombs test is positive for


A. IgA B. IgE
C. IgM D. IgG
77. The Schilling test is used for detecting
A. Presence of intrinsic factor B. Deficiency of riboflavin
C. Excretion of folic acid D. Capillary fragility
78. A rare disorder characterized by benign growth in the lymph node tissue throughout the body is called
A. Castleman’s disease B. Heck’s disease
C. Darier’s disease D. Crohn’s disease
79. A serious coagulopathy characterized by severe thrombocytopenia and hemorrhage due to platelet trapping within
the tumor is called
A. PHACE(S) syndrome B. Kasabach–Merritt phenomenon
C. Kaposi’s sarcoma D. Hemangioma
80. The mode of inheritance in hemophilia is
A. Autosomal dominant B. Autosomal recessive
C. X linked dominant D. X linked recessive
81. When they consulted a dentist for a decayed tooth, he advised for checking of prothrombin time (PT) and plasma
thromboplastin time (PTT), which would be as follows
A. PT is normal, PTT is normal B. PT is prolonged, PTT is prolonged
C. PT is normal, PTT is prolonged D. PT is prolonged, PTT is normal
82. The patient could be suffering from
A. Hemophilia A B. Anemia
C. Attention deficit hyperactive disorder D. Disseminated intravascular coagulation
83. His blood picture will show
A. Bleeding time is prolonged, clotting time is normal B. Bleeding time is normal, clotting time is normal
C. Bleeding time is normal, clotting time is prolonged D. None

SYSTEMIC PATHOLOGY AND MISCELLANEOUS


1. Disappearance of nuclear chromatin is known as
A. Pyknosis B. Karyolysis
C. Karyorrhexis D. None
2. A patient has increased number of columnar cells in the lower esophagus. He has which of the following change?
A. Dysplasia B. Anaplasia
C. Metaplasia D. Normal histology
3. Saddle embolus causes sudden death by blocking
A. Coronary arteries B. Cerebral arteries
C. Pulmonary arteries D. Renal arteries
4. Which of the following is not sequelae of cellular events in atherosclerotic infarction?
A. Neutrophilic infiltration B. Astrocytes
C. Ingress of macrophages D. Intense eosinophilia
5. Syndrome associated with increased risk of leukemia is
A. Plummer–Vinson syndrome B. Klinefelter syndrome
C. Sturge–Weber syndrome D. Multiple hamartoma syndrome
6. Phagocytes kill bacteria through which mechanism?
A. Membrane attack complex killing B. Oxidative and nonoxidative killing
C. Zipper killing D. Chemotaxis
7. Leukopenia is characteristic of
A. Appendicitis B. Enteric fever
C. Meningitis D. Myocardial infarction

https://t.me/DentalBooksWorld
Chapter 3 • General Pathology 195

8. Egg shell calcification of hilar lymph node is associated with


A. Silicosis B. Asbestosis
C. Byssinosis D. Anthracosis
9. Barr body is associated with
A. Metaphase B. Interphase
C. Anaphase D. Prophase
10. Which one of the following does not present antigens?
A. NK cells (Natural Killer cells) B. Dendritic cells
C. Langerhans cells D. Macrophages
11. The gene for breast cancer (BRCA) is present on chromosome
A. 21 B. 17
C. 15 D. 9
12. Which of the following is used to stain fungi?
A. PAS B. Fontana stain
C. Ferrous trichrome D. Pearls Prussian blue
13. Which of the following is not present in lungs?
A. Clara cells B. Langerhans cells
C. Brush cells D. Anitschkow cells
14. An infant with cleft lip, palate, polydactyly, microcephaly with holoprosencephaly, and ectodermal scalp defect is
suffering from
A. Trisomy 21 B. Trisomy 18
C. Trisomy 13 D. Turner syndrome
15. Why fetal cells continue to divide but terminally differentiated adult cells do not divide?
A. There are many cyclin inhibitors which prevent cell to enter into S phase in adult
B. Phosphatase is absent in fetal cells
C. Proteinase is absent in fetus
D. Absence of CD kinase
16. Which if the most common cause of pulmonary embolism?
A. Thrombophlebitis B. Endarteritis
C. Atherosclerosis D. Lymphangitis
17. Which of the following is most common location of intracranial neurocysticercosis?
A. Brain parenchyma B. Spinal cord
C. Basal cisternae D. Medulla oblongata
18. Syndrome which is characterized by 2X chromosomes and 1Y chromosome is
A. Klinefelter’s syndrome B. Down syndrome
C. Turner syndrome D. Marfan syndrome
19. All of the following types of collagen except one are present in cartilage
A. 2 B. 4
C. 6 D. 9
20. Fibrin degradation product help in detection of
A. Hemophilia B. DIC
C. Thrombocytopenic purpura D. Thrombasthenia
21. Which is associated with defect in DNA repair?
A. Xeroderma pigmentosum B. Albinism
C. Ichthyosis D. Sickle cell anemia
22. HOX gene is responsible for which malformation?
A. Polysyndactyly B. Holoprosencephaly
C. Mayer–Rokitansky syndrome D. Gorlin syndrome
23. Prostate specific antigens is used as
A. Tumor marker B. Proto oncogene
C. Oncogene D. Bacterial antigen

https://t.me/DentalBooksWorld
196 Triumph's Complete Review of Dentistry

24. Genes for sex determination is


A. Shh B. SRY
C. AZF D. HOX
25. Which of the following procedures is used as a routine technique in karyotyping using light microscopy?
A. G banding B. C banding
C. O banding D. Brd V-staining
26. Daily loss of iron per day in a healthy adult male is
A. 0.06 mg B. 0.6 mg
C. 60 mg D. 600 mg
27. Which is true about multiple myeloma?
A. M spike increase
B. Direct plasma cell deposition in glomerular capillaries leads to kidney damage
C. IL 8 is a marker for bone lesions
D. Invariably increase in alkaline phosphatase
28. T-Helper cells (TH2) initiate which type of immunity?
A. Induces cell mediated immunity B. Induces humoral immunity
C. Induces immunity by activation of cytotoxic T-cells D. None of the above
29. Bone marrow stem cells differ from differentiated progenitor stem cells in what respect?
A. Provide differentiated terminal cells B. Reconstitution of Bone Marrow
C. Formation of the ovum D. Act as a repair system for the body
30. The correct nomenclature for Down’s syndrome with translocation of chromosome 14 to chromosome 21 is depicted as
A. 47 X &, (+2) B. 47 X Y, –14 t(14;21)
C. 46 X Y, – 14 t(14;21) D. 46 X Y, t(14;21)
31. A child’s CSF was examined; it contains slightly raised or normal PMN cells, raised lymphocytes, absence of mycotic
growth, value of protein is normal or above normal, glucose level is normal. The most probable diagnosis is
A. Pyogenic bacterial meningitis B. Viral meningitis
C. Fungal meningitis D. Tuberculous meningitis
32. Heterozygous sickle cell anemia gives protection against
A. G6PD B. Malaria
C. Thalassemia D. Dengue fever
33. DiGeorge’s syndrome is due to
A. Congenital thymic aplasia B. Deficiency of complement factors
C. Inborn error of metabolism D. Chromosomal anomaly
34. Mural thrombi are thrombi in
A. Heart chamber B. Valve cusp
C. Vein D. Arteries
35. Endotoxic shock is propagated
A. Endothelial injury B. Peripheral vasodilation
C. Increased vascular permeability D. Cytokine action
36. All the following statements are true regarding cervicofacial actinomycosis, except
A. 10–20% of actinomycosis cases occur in the cervicofacial region
B. The discharging pus contains visible “Sulfur granules”
C. Cultures on blood agar often produce a typical “molar tooth” morphology
D. There is usually history of trauma such as tooth extraction or a blow to the jaw
37. A 13-year-old boy presents with a large mixed lytic and blastic mass in the metaphyseal region of the lower end of
femur. The most likely diagnosis is
A. Ewing’s sarcoma B. Osteosarcoma
C. Chondrosarcoma D. Giant cell tumor
38. A patient with amenorrhea, short stature, less pubic hair can be in all of the following except
A. 47 XXY B. 46 XY
C. 45 X D. 46 XXY

https://t.me/DentalBooksWorld
Chapter 3 • General Pathology 197

39. If a hemophilia person has been married to a normal woman, then his
A. Daughters are carriers B. Sons are infected
C. Daughters are infected D. All are normal
40. Down syndrome is caused by all, except
A. Trisomy of 21 chromosome B. Mosaicism of 21 chromosome
C. Robertsonian translocation of 21.21, 21.18 D. Deletion of 21
41. Acute pyelonephritis is diagnosed by
A. Focal scar in renal cortex B. Septicemia
C. Altered renal function D. Chills, fever, flank pain
42. NK cells are effective against viral infected cells only if the cells with infection
A. Express MHC class I proteins B. Unable to express MHC class I proteins
C. Express MHC class II proteins D. Unable to express MHC class II proteins
43. A 40-year-old female with fullness in the upper right quadrant of abdomen with diabetes type II, hyperlipidemia, and
biopsy shows
A. Biliary cirrhosis B. Luminal thrombosis
C. Non-alcoholic steatohepatitis D. Autoimmune hepatitis
44. A 4-year-old girl child suffering from vomiting, cerebral edema is being treated for viral infection from the past 5
days. Liver biopsy in this patient may show
A. Central hemorrhagic necrosis B. Nonalcoholic steatohepatitis
C. Autoimmune hepatitis D. Microvascular steatohepatitis
45. A lady complains of headache, nausea, and tenderness in temporal region with migraine. On microscopic investigation
what will be seen?
A. Temporal aneurysm B. Giant cell arteritis
C. Granulomatous giant cell lesions D. Luminal thrombosis
46. In a 60-year-old hypertensive male with renal failure, renal biopsy shows onion skin appearance. The most likely
diagnosis is
A. Hyaline arteriosclerosis B. Thrombophlebitis obliterans
C. Hyperplastic arteriosclerosis D. Arteriosclerosis obliterans
47. Which of the following statement is not true regarding chlamydia trachomatis?
A. Elementary body is metabolically active B. It is biphasic
C. Reticulocyte body divides by binary fission D. Inside the cell it evades phagolysosome
48. Which is not true about phenotype?
A. It is modified with the passage of time B. It is the appearance of an individual
C. It is genetic sequence of an individual D. It is influenced by genotype
49. Air forced or sucked into the connective tissue and facial spaces are known as
A. Empyema B. Asphyxia
C. Emphysema D. Aspiration

ANSWERS

CELL INJURY AND ADAPTATION


1. Answer: A (Ref. Robbins Basic Pathology, By Vinay Kumar, Abul K. Abbas, Jon C. Aster, 10th edition, page no. 27)
2. Answer: B (Ref. Robbins Basic Pathology, By Vinay Kumar, Abul K. Abbas, Jon C. Aster, 10th edition, page no. 295)
3. Answer: D (Ref. Robbins Basic Pathology, By Vinay Kumar, Abul K. Abbas, Jon C. Aster, 10th edition, page no. 294)
4. Answer: C (Ref. Robbins Basic Pathology, By Vinay Kumar, Abul K. Abbas, Jon C. Aster, 10th edition, page no. 47)
Mitochondria performs the important metabolic function of oxidative phosphorylation, and in the process generate free
radicals injurious to membranes.
5. Answer: B (Ref. Robbins Basic Pathology, By Vinay Kumar, Abul K. Abbas, Jon C. Aster, 10th edition, page no. 11)
• Microfilaments are long filamentous structures having a diameter of 6–8 nm. They are composed of contractile proteins,
actin and myosin, and diverse materials like parts of microtubules and ribonucleoprotein fibers.

https://t.me/DentalBooksWorld
198 Triumph's Complete Review of Dentistry

• Intermediate filaments are filamentous structures, 10 nm in diameter, and are cytoplasmic constituent of a number of
cell types.
They are composed of proteins. There are five principal types of intermediate filaments:
1. Cytokeratin (found in epithelial cells)
2. Desmin (found in skeletal, smooth, and cardiac muscle)
3. Vimentin (found in cells of mesenchymal origin)
4. Glial fibrillary acidic protein (present in astrocytes and ependymal cells)
5. Neurofilaments (seen in neurons of central and peripheral nervous system)
• Microtubules are long hollow tubular structures about 25 nm in diameter. They are composed of the protein,
tubulin. Cilia and flagella which project from the surface of cell are composed of microtubules enclosed by plasma
membrane.
6. Answer: B (Ref. Robbins Basic Pathology, By Vinay Kumar, Abul K. Abbas, Jon C. Aster, 10th edition, page no. 43)
Mechanism of reperfusion injury and free radical mediated injury involves three main components:
• Calcium overload
• Generation of reactive oxygen radicals
• Subsequent inflammatory reaction
7. Answer: C (Ref. “previous question”)
Calcium overload. Upon restoration of blood supply, the ischemic cell is further bathed by the blood fluid that has more
calcium ions at a time when the ATP stores of the cell are low → results in further calcium overload on the already injured
cells, triggering lipid peroxidation of the membrane causing further membrane damage.
8. Answer: C (Ref. Robbins Basic Pathology, By Vinay Kumar, Abul K. Abbas, Jon C. Aster, 10th edition, page no. 44)
Mechanism of oxygen-free radical generation
• The reaction of O2 to H2O involves “four electron donation” in four steps involving transfer of one electron at each step.
• Oxygen-free radicals are the intermediate chemical species having an unpaired oxygen in their outer orbit.
• Three intermediate molecules of partially reduced species of oxygen are generated depending upon the number of
electrons transferred
− Superoxide oxygen (O’2): one electron
− Hydrogen peroxide (H2O2): two electrons
− Hydroxyl radical (OH–): three electrons
• Other-oxygen free radicals: Nitric oxide (NO), hypochlorous acid (HOCl)
9. Answer: C (Ref. Robbins Basic Pathology, By Vinay Kumar, Abul K. Abbas, Jon C. Aster, 10th edition, page no. 312)
• Liver is the commonest site for accumulation of fat because it plays a central role in fat metabolism.
• Lipids as free acids enter the liver cell from either of the following two sources.
− From diet as chylomicrons (containing triglycerides and phospholipids) and as free fatty acids
− From adipose tissue as free fatty acids
• Normally, besides above two sources, small part of fatty acids is also synthesized from acetate in the liver cells. Most
of free fatty acid is esterified to triglycerides by the action of α-glycerophosphate and only a small part is changed into
cholesterol, phospholipids, and ketone bodies.
• While cholesterol, phospholipids, and ketones are used in the body, intracellular triglycerides are converted into
lipoproteins, which requires “lipid acceptor protein.” Lipoproteins are released from the liver cells into circulation as
plasma lipoproteins.
10. Answer: B (Ref. Robbins Basic Pathology, By Vinay Kumar, Abul K. Abbas, Jon C. Aster, 10th edition, page no. 36)
11. Answer: B (Ref. Robbins Basic Pathology, By Vinay Kumar, Abul K. Abbas, Jon C. Aster, 10th edition, page no. 218)
• Activation of growth controlling genes (BCL-2 and p53)
• BCL-2 gene is a human counterpart of CED-9 (cell death) gene
• Net effect on the mitochondrial membrane is based on pro-apoptotic and anti-apoptotic actions of BCL-2 gene family
12. Answer: B (Ref. Robbins and Cotran Review of Pathology, By Edward C. Klatt, Vinay Kumar, 4th edition, page no. 388)
• Diabetic foot is another example of wet gangrene due to high sugar content in the necrosed tissue which favors growth
of bacteria.
• Bed sores occurring in a bed-ridden patient due to pressure on sites like the sacrum, buttocks, and heels are the other
important clinical conditions included in wet gangrene.
• Line of demarcation between gangrenous segment and viable bowel is generally not clear.

https://t.me/DentalBooksWorld
Chapter 3 • General Pathology 199

13. Answer: B (Ref. Robbins Basic Pathology, By Vinay Kumar, Abul K. Abbas, Jon C. Aster, 10th edition, page no. 53)
Calcinosis cutis → Senile degenerative changes such as in costal cartilages, tracheal or bronchial cartilages, and pineal
gland in the brain etc.
14. Answer: B (Ref. Robbins Basic Pathology, By Vinay Kumar, Abul K. Abbas, Jon C. Aster, 10th edition, page no. 50)
In atrophic condition
• The organ is small, often shrunken.
• The cells become smaller in size, but are not dead cells.
• Shrinkage in cell size is due to reduction in cell organelles, chiefly mitochondria, myofilaments, and endoplasmic
reticulum.
• There is often increase in the number of autophagic vacuoles containing cell debris.
15. Answer: C (Ref. Robbins Basic Pathology, By Vinay Kumar, Abul K. Abbas, Jon C. Aster, 10th edition, page no. 51)
• Metaplasia is defined as a reversible change of one type of epithelial or mesenchymal adult cells to another type of adult
epithelial or mesenchymal cells, usually in response to abnormal stimuli, and often reverts back to normal on removal of
stimulus.
16. Answer: A (Ref. Robbins Basic Pathology, By Vinay Kumar, Abul K. Abbas, Jon C. Aster, 10th edition, page no. 18)
17. Answer: B (Ref. Robbins Basic Pathology, By Vinay Kumar, Abul K. Abbas, Jon C. Aster, 10th edition, page no. 130)
18. Answer: D (Ref. Robbins Basic Pathology, By Vinay Kumar, Abul K. Abbas, Jon C. Aster, 10th edition, page no. 39)
19. Answer: C (Ref. Robbins Basic Pathology, By Vinay Kumar, Abul K. Abbas, Jon C. Aster, 10th edition, page nos. 38 and 39)
20. Answer: D (Ref. Robbins Basic Pathology, By Vinay Kumar, Abul K. Abbas, Jon C. Aster, 10th edition, page no. 219)
21. Answer: D (Ref. Robbins Basic Pathology, By Vinay Kumar, Abul K. Abbas, Jon C. Aster, 9th edition, page no. 31)
• Endocrine stimulation of target tissues is invariably hyperplasia. A is atrophy, B is hypertrophy C, and is metaplasia.
22. Answer: B (Ref. Robbins Basic Pathology, By Vinay Kumar, Abul K. Abbas, Jon C. Aster, 9th edition, page no. 5315)
23. Answer: D (Ref. Robbins Basic Pathology, By Vinay Kumar, Abul K. Abbas, Jon C. Aster, 10th edition, page no. 36)
Option A, B, and C are liquefactive necrosis.
Option D is hemorrhagic infarction.
24. Answer: B (Ref. Robbins Basic Pathology, By Vinay Kumar, Abul K. Abbas, Jon C. Aster, 10th edition, page no. 654)
25. Answer: D (Ref. Lippincott’s Pocket Pathology, by Donna E. Hansel, Renee Z. Dintzis, page no. 12)
26. Answer: A (Ref. Lippincott’s Pocket Pathology, by Donna E. Hansel, Renee Z. Dintzis, page no. 16)
27. Answer: D (Ref. Robbins Basic Pathology, By Vinay Kumar, Abul K. Abbas, Jon C. Aster, 10th edition, page no. 34)
28. Answer: C (Ref. Robbins Basic Pathology, By Vinay Kumar, Abul K. Abbas, Jon C. Aster, 10th edition, page no. 332)
29. Answer: D (Ref. Robbins Basic Pathology, By Vinay Kumar, Abul K. Abbas, Jon C. Aster, 10th edition, page no. 82)
30. Answer: A (Ref. Lippincott’s Pocket Pathology, by Donna E. Hansel, Renee Z. Dintzis, page no. 16)
31. Answer: B (Ref. Lippincott’s Pocket Pathology, by Donna E. Hansel, Renee Z. Dintzis, page no. 12)
32. Answer: A (Ref. Lippincott’s Pocket Pathology, by Donna E. Hansel, Renee Z. Dintzis, page no. 10)
33. Answer: D (Ref. Lippincott’s Pocket Pathology, by Donna E. Hansel, Renee Z. Dintzis, page no. 16)
34. Answer: B (Ref. Lippincott’s Pocket Pathology, by Donna E. Hansel, Renee Z. Dintzis, page no. 20)
35. Answer: A (Ref. Robbins Basic Pathology, By Vinay Kumar, Abul K. Abbas, Jon C. Aster, 10th edition, page no. 530)
36. Answer: C (Ref. Robbins Basic Pathology, By Vinay Kumar, Abul K. Abbas, Jon C. Aster, 10th edition, page no. 33)
37. Answer: A (Ref. Robbins Basic Pathology, By Vinay Kumar, Abul K. Abbas, Jon C. Aster, 10th edition, page no. 25)
38. Answer: A (Ref. Robbins Basic Pathology, By Vinay Kumar, Abul K. Abbas, Jon C. Aster, 10th edition, page no. 52)
39. Answer: C (Ref. Robbins Basic Pathology, By Vinay Kumar, Abul K. Abbas, Jon C. Aster, 9th edition, page no. 44)
40. Answer: C (Ref. Robbins and Cotran Review of Pathology, By Edward C. Klatt, Vinay Kumar, 4th edition, page no. 386)
41. Answer: C (Ref. Review of Pathology, by Ivan Damjanov, Emanuel Rubin. 2000, page no. 27)
42. Answer: D (Ref. The Journal of Cell Biology, Vol. 105, Issue 4, Part 2, 1987)
43. Answer: C (Ref. Pathology: Review for New National Boards, page no. 6)
44. Answer: C (Ref. Lippincott’s Pocket Pathology, by Donna E. Hansel, Renee Z. Dintzis, page no. 20)
• Necrosis is a pathological response to injury.
• Necrosis does not maintain plasma membrane Integrity.
• Oxygen-free radicals may stimulate necrosis.
• Necrosis is associated with a local inflammatory response.
• Necrosis is an energy-independent process.

https://t.me/DentalBooksWorld
200 Triumph's Complete Review of Dentistry

45. Answer: C (Ref. “previous question”)


• Apoptosis can be either a pathological or physiological process.
• Single cells are usually involved.
• Cells appear fragmented and shrunk (apoptotic bodies).
• DNA fragmentation occurs in an energy-dependent process, and is cleaved by endonucleases.
• There is no inflammatory response.
46. Answer: B (Ref. “previous question”)
• Activation of extrinsic pathway of apoptosis → binding of Fas ligand takes place to CD95/binding of TRAIL (TNF related
apoptosis inducing ligand) attaches to death receptors DR4 and DR5 → induces the association of FADD (Fas-associated
death domain) and procaspase-8 → activation of caspase 8 (in humans caspase 10) → activates caspases
• *Fas member of TNF receptor family*
47. Answer: B (Ref. “previous question”)
Two phenomena consistently characterize irreversibility
The first is the inability to reverse mitochondrial dysfunction (lack of oxidative phosphorylation and ATP generation)
even after resolution of the original injury.
The second is the development of profound disturbances in membrane function KEY.
Features of reversible cell injury
• Cellular swelling
• Loss of microvilli
• Formation of cytoplasmic blebs
• Endoplasmic reticulum swelling
• Detachment of ribosomes
• Myelin figures
• Clumping of nuclear chromatin
Features of irreversible cell injury
• Large flocculent, amorphous densities in swollen mitochondria due to increased calcium influx
• Swelling and disruption of lysosomes and leakage of lysosomal enzymes in cytoplasm
• Decreased basophilia
• Severe damage to plasma membranes
• Nuclear changes include
− Pyknosis (nuclear condensation)
− Karyorrhexis (nuclear fragmentation)
− Karyolysis (nuclear dissolution)
48. Answer: C (Ref. Robbins Basic Pathology, By Vinay Kumar, Abul K. Abbas, Jon C. Aster, 10th edition, page no. 39)
• Caspases are cysteine proteases and are critical for the process of apoptosis.
• Apoptosis is required at the time of different processes in embryogenesis, such as, implantation, organogenesis,
developmental involution, and metamorphosis.
49. Answer: B (Ref. “previous question”)
50. Answer: D (Ref. “previous question”)
The morphologic features characteristic of apoptosis includes
• Cell shrinkage: The cell is smaller in size having dense cytoplasm and the organelles are tightly packed.
• Chromatin condensation: This is the most characteristic feature of apoptosis.
• Formation of cytoplasmic blebs and apoptotic bodies.
NOTE:
• Plasma membranes are thought to remain intact till late stage of apoptosis, as well as is a normal cell.
• Eosinophilic cytoplasm, it is a common feature of necrosis and apoptosis.
• Nuclear molding is a characteristic of malignant cell, defined as the “The shape of one nucleus conforming around the
shape of an adjacent nucleus.”
51. Answer: C (Ref. “previous question”)
52. Answer: A (Ref. Robbins Basic Pathology, By Vinay Kumar, Abul K. Abbas, Jon C. Aster, 10th edition, page no. 781)
Fibrinoid necrosis is seen in

https://t.me/DentalBooksWorld
Chapter 3 • General Pathology 201

• Malignant hypertension
• Vasculitis like PAN
• Acute rheumatic fever
53. Answer: C (Ref. Robbins Basic Pathology, By Vinay Kumar, Abul K. Abbas, Jon C. Aster, 10th edition, page no. 26)
54. Answer: A (Ref. “previous question”)
Apoptotic cells express phosphatidylserine, which moves out from the inner layers → recognized by a number of receptors
on the phagocytes and helps in binding of a protein called Annexin V (marker for apoptosis).
55. Answer: A (Ref. “previous question”)
Activated caspases cleave proteins and induce apoptosis.
56. Answer: A (Ref. “previous question”)
57. Answer: D (Ref. “previous question”)
Inter-nucleosomal cleavage of DNA into oligonucleosomes is brought about by Ca2+ and Mg2+ dependent endonucleases
and is characteristic of apoptosis.
58. Answer: A (Ref. “previous question”)
• Apoptosis – Endonucleases are enzymes → cause internucleosomal cleavage of DNA into oligonucleosomes, the latter
being visualized as DNA ladders.
• Necrosis – smeared pattern is commonly seen.
59. Answer: D (Ref. Robbins Basic Pathology, By Vinay Kumar, Abul K. Abbas, Jon C. Aster, 10th edition, page no. 34)
60. Answer: B (Ref. Lippincott’s Pocket Pathology, by Donna E. Hansel, Renee Z. Dintzis, page no. 16)
61. Answer: B (Ref. Lippincott’s Pocket Pathology, by Donna E. Hansel, Renee Z. Dintzis, page no. 12)
62. Answer: B (Ref. Lippincott’s Pocket Pathology, by Donna E. Hansel, Renee Z. Dintzis, page no. 17)
63. Answer: B (Ref. Lippincott’s Pocket Pathology, by Donna E. Hansel, Renee Z. Dintzis, page no. 20)
64. Answer: A (Ref. Robbins Basic Pathology, By Vinay Kumar, Abul K. Abbas, Jon C. Aster, 10th edition, page no. 42)
65. Answer: A (Ref. Robbins Basic Pathology, By Vinay Kumar, Abul K. Abbas, Jon C. Aster, 10th edition, page no. 217)
• Cytochrome c binds to a protein called Apaf-1 (apoptosis-activating factor-1 which is responsible for formation of a
complex called apoptosome.
• This complex binds to caspase-9; is a critical initiator caspase of the mitochondrial pathway of apoptosis.
66. Answer: B (Ref. “previous question”)
67. Answer: B (Ref. “previous question”)
Dead cells may be replaced by large, whorled phospholipid masses called myelin figures that are derived from damaged
cell membranes.
68. Answer: A (Ref. “previous question”)
69. Answer: D (Ref. “previous question”)
Inducers of apoptosis
• Withdrawal of growth factor
• Loss of matrix attachment
• Glucocorticoids
• Free radicals
• Some viruses
• Ionizing radiation
• DNA damage
Inhibitors of apoptosis
• Growth factors
• Extracellular matrix
• Steroids
• Some viral proteins
70. Answer: B (Ref. “previous question”)
71. Answer: A (Ref. “previous question”)
Some tumors show characteristic spherules of calcification called psammoma bodies or calcospherites such as in
meningioma, papillary serous cystadenocarcinoma of the ovary, and papillary carcinoma of the thyroid and glucagonoma.
72. Answer: D (Ref. Robbins Basic Pathology, By Vinay Kumar, Abul K. Abbas, Jon C. Aster, 10th edition, page no. 44)
• Free radicals are generated through Fenton’s reaction which is (H2O2 + Fe2+ → Fe3+ + OH+ + OH–)

https://t.me/DentalBooksWorld
202 Triumph's Complete Review of Dentistry

• In this reaction iron is converted from its ferrous to ferric form and a radical is generated.
• The effects of these reactive species relevant to cell injury include: Lipid peroxidation of membranes, oxidative
modification of proteins, and lesions in DNA.
73. Answer: B (Ref. Robbins Basic Pathology, By Vinay Kumar, Abul K. Abbas, Jon C. Aster, 10th edition, page no. 54)
Pathologic calcification (Heterotopic calcification) is the abnormal tissue deposition of calcium salts together with small
amounts of iron, manganese, and other mineral salts.
• Types: Dystrophic calcification or metastatic calcification
74. Answer: A (Ref. Robbins Basic Pathology, By Vinay Kumar, Abul K. Abbas, Jon C. Aster, 10th edition, page no. 52)
75. Answer: D (Ref. Robbins Basic Pathology, By Vinay Kumar, Abul K. Abbas, Jon C. Aster, 10th edition, page no. 52)
76. Answer: A (Ref. Robbins Basic Pathology, By Vinay Kumar, Abul K. Abbas, Jon C. Aster, 10th edition, page no. 52)
Lipofuscin (Wear and tear pigment)
• Lipofuscin or lipochrome is a yellowish-brown intracellular lipid pigment (lipo = fat, fuscus = brown).
• The pigment is often found in atrophied cells of old age and hence the name “wear and tear pigment.”
• It is seen in the myocardial fibers, hepatocytes, Leydig cells of the testes, and in neurons in senile dementia.
77. Answer: B (Ref. Lippincott’s Pocket Pathology, by Donna E. Hansel, Renee Z. Dintzis, page no. 20)
78. Answer: C (Ref. Robbins Basic Pathology, By Vinay Kumar, Abul K. Abbas, Jon C. Aster, 10th edition, page no. 376)
79. Answer: C (Ref. Essential Pathology For Dental Students, by Harsh Mohan, Sugandha Mohan, page no. 47)
80. Answer: C (Ref. Lippincott’s Pocket Pathology, by Donna E. Hansel, Renee Z. Dintzis, page no. 20)
81. Answer: A (Ref. Lippincott’s Pocket Pathology, by Donna E. Hansel, Renee Z. Dintzis, page no. 9)
82. Answer: B (Ref. Robbins Basic Pathology, by Vinay Kumar, Abul K. Abbas, Jon C. Aster, 2017, page no. 35)
83. Answer: B (Ref. Essential Pathology For Dental Students, by Harsh Mohan, Sugandha Mohan, page no. 77)
84. Answer: C (Ref. The Endometrium, by Stanley R. Glasser, John D. Aplin, Linda C. Giudice, 2004, page no. 169)
85. Answer: B (Ref. Robbins Basic Pathology, By Vinay Kumar, Abul K. Abbas, Jon C. Aster, 9th edition, page no. 645)
86. Answer: A (Ref. “previous question”)
87. Answer: C (Ref. Robbins Basic Pathology, By Vinay Kumar, Abul K. Abbas, Jon C. Aster, 10th edition, page no. 98)
88. Answer: A (Ref. Robbins Basic Pathology, By Vinay Kumar, Abul K. Abbas, Jon C. Aster, 9th edition, page no. 62)
89. Answer: B
Pre-gangrene (is the penultimate stage of vascular insufficiency before gangrene sets in; the term is usually applied to
ischemia of the lower limb)
90. Answer: B (Ref. “previous question”)
91. Answer: D (Ref. “previous question”)
92. Answer: B (Ref. “previous question”)
93. Answer: C (Ref. “previous question”)
• The G2/M checkpoint monitors the completion of the DNA replication and checks whether the cell can safely initiate
the mitosis and separate sister chromatids.
• This checkpoint is particularly important in cells exposed to ionizing radiation.
• Cells damaged by ionizing radiation activate G2/M checkpoint and arrest in G2.
94. Answer: A (Ref. Robbins Basic Pathology, By Vinay Kumar, Abul K. Abbas, Jon C. Aster, 9th edition, page no. 25)
G1/S check-point is controlled by p53 whereas G2/M check-point has both p53 dependent as well independent
mechanisms.
95. Answer: D (Ref. “previous question”)
96. Answer: C (Ref. “previous question”)
G1/S check-point is controlled by p53 whereas G2/M check-point has both p53 dependent as well as independent
mechanisms. p53 induces the synthesis of p21 which inhibits cyclin D/Cdk4. This results in stoppage no. of activation of
Rb and cell cycle is arrested in G1/S phase.
97. Answer: D (Ref. “previous question”)
Cyclin/CDK complexes controlling the cell cycle
• Regulates the transition from G2 to M phase → Cyclin B/CDK1
• Regulates the transition from G1-S → Cyclin D/CDK4, Cyclin D/CDK6, Cyclin E/CDK2
• Active in S phase → Cyclin A/CDK2 and cyclin B/CDK1
98. Answer: A (Ref. “previous question”)
The time taken for S, G2, and M phases are similar for most cell types, occupying about 6, 4, and 2 hours, respectively.

https://t.me/DentalBooksWorld
Chapter 3 • General Pathology 203

• Duration of G1 shows considerable variation. It can be as short as 2 hours in rapidly dividing cells like embryonic tissues
or as long as 12 hours in some adult tissues.
• G1 phase is most variable because, in this phase cells are not committed to DNA replication. They can either enter
resting state or progress to next cell division.

INFLAMMATION, IMMUNITY, AND HYPERSENSITIVITY


1. Answer: B (Ref. Robbins Basic Pathology, By Vinay Kumar, Abul K. Abbas, Jon C. Aster, 10th edition, page no. 90)
Granulation tissue has capillaries, fibroblasts, and a variable amount of inflammatory cells (mostly mononuclear, but with
the possibility of some PMNs still being present).
2. Answer: B (Ref. Robbins Basic Pathology, By Vinay Kumar, Abul K. Abbas, Jon C. Aster, 10th edition, page no. 57)
Exudation: Fluid, proteins, red blood cells, and white blood cells escape from the intravascular space as a result of increased
osmotic pressure extravascularly and increased hydrostatic pressure intravascularly.
3. Answer: D (Ref. Robbins Basic Pathology, By Vinay Kumar, Abul K. Abbas, Jon C. Aster, 10th edition, page no. 354)
Hyaluronidase is a protein enzyme. It works by causing rapid spreading of injected fluid into the body, which increases the
absorption of injected fluids.
4. Answer: D (Ref. Robbins Basic Pathology, By Vinay Kumar, Abul K. Abbas, Jon C. Aster, 10th edition, page no. 67)
5. Answer: A (Ref. Robbins Basic Pathology, By Vinay Kumar, Abul K. Abbas, Jon C. Aster, 10th edition, page no. 85)
6. Answer: D (Ref. Robbins Basic Pathology, By Vinay Kumar, Abul K. Abbas, Jon C. Aster, 10th edition, page no. 137)
7. Answer: A (Ref. Robbins Basic Pathology, By Vinay Kumar, Abul K. Abbas, Jon C. Aster, 10th edition, page no. 544)
Transudate Exudate
↑ Hydrostatic pressure, Inflammation-Increased 
Main causes
↓ colloid osmotic pressure vascular permeability
Appearance Clear Cloudy
Specific gravity <1.012 – Low >1.020 – High
Protein content <2.5 g/dL – Low >2.9 g/dL – High
Fluid protein/serum protein <0.5 – Low >0.5 – High
SAAG = Serum [albumin] − Effusion [albumin] >1.2 g/dL – High <1.2 g/dL – Low
Fluid LDHupper limit for serum <0.6 or < 2⁄3 – Low > 0.6 [2] or > 2⁄3 – High
Cholesterol content <45 mg/dL – Low >45 mg/dL – High
8. Answer: C (Ref. Robbins Basic Pathology, By Vinay Kumar, Abul K. Abbas, Jon C. Aster, 10th edition, page no. 60)
9. Answer: D (Ref. “previous question”)
10. Answer: C (Ref. Robbins Basic Pathology, By Vinay Kumar, Abul K. Abbas, Jon C. Aster, 10th edition, page no. 137)
11. Answer: C (Ref. Robbins Basic Pathology, By Vinay Kumar, Abul K. Abbas, Jon C. Aster, 10th edition, page no. 73)
Prostaglandins are found in most tissues and organs. They are produced by almost all nucleated cells. They are autocrine
and paracrine lipid mediators that act upon platelets, endothelium, uterine, and mast cells. They are synthesized in the cell
from the essential fatty acids (EFAs).
12. Answer: B (Ref. “previous question”)
13. Answer: A (Ref. Robbins Basic Pathology, By Vinay Kumar, Abul K. Abbas, Jon C. Aster, 10th edition, page no. 61)
14. Answer: D (Ref. Robbins Basic Pathology, By Vinay Kumar, Abul K. Abbas, Jon C. Aster, 10th edition, page no. 73)
15. Answer: C (Ref. Robbins Basic Pathology, By Vinay Kumar, Abul K. Abbas, Jon C. Aster, 10th edition, page no. 85)
16. Answer: D (Ref. Robbins Basic Pathology, By Vinay Kumar, Abul K. Abbas, Jon C. Aster, 10th edition, page no. 85)
17. Answer: A (Ref. Robbins Basic Pathology, By Vinay Kumar, Abul K. Abbas, Jon C. Aster, 10th edition, page no. 136)
NOTE:
Type I – Allergic
Type II – Cytotoxic
Type III – Immune complex deposition
Type IV – Delayed
• Type I is IgE-mediated and occurs very quickly after exposure. It is associated with allergens such as bee stings, peanuts,
and certain medications, to name a few. This can lead to a life-threatening condition called anaphylaxis.
• Type II hypersensitivity reactions are cytotoxic/antibody-mediated. Some examples in this category are hemolytic
reactions, Goodpasture syndrome, and hyperacute graft rejection.

https://t.me/DentalBooksWorld
204 Triumph's Complete Review of Dentistry

• Type III is known as immune complex/IgG/IgM mediated and includes certain diagnoses like hypersensitivity
pneumonitis, systemic lupus erythematosus, polyarteritis nodosa, and serum sickness.
• Type IV is known as delayed or cell-mediated hypersensitivity reaction. Examples include chronic graft rejections,
purified protein derivative (PPD), latex, nickel, and poison ivy.
18. Answer: A (Ref. “previous question”)
19. Answer: B (Ref. Robbins Basic Pathology, By Vinay Kumar, Abul K. Abbas, Jon C. Aster, 10th edition, page no. 64)
20. Answer: A (Ref. Robbins Basic Pathology, By Vinay Kumar, Abul K. Abbas, Jon C. Aster, 10th edition, page no. 74)
21. Answer: B (Ref. Robbins Basic Pathology, By Vinay Kumar, Abul K. Abbas, Jon C. Aster, 10th edition, page no. 344)
22. Answer: A (Ref. Robbins Basic Pathology, By Vinay Kumar, Abul K. Abbas, Jon C. Aster, 10th edition, page no. 60)
23. Answer: B (Ref. Robbins Basic Pathology, By Vinay Kumar, Abul K. Abbas, Jon C. Aster, 10th edition, page no. 86)
24. Answer: A (Ref. “previous question”)
• Virchow’s cell – a macrophage in Hansen’s disease (Leprosy)
• Virchow’s cell theory – every living cell comes from another Virchow’s angle – the angle between the nasobasilar line
and the subnasal lining cell
• Virchow’s disease, leontiasis ossea – now recognized as a symptom rather than a disease
• Virchow’s line – a line from the root of the nose to the lambda
• Virchow’s metamorphosis – lipomatosis in the heart and salivary glands
• Virchow’s method of autopsy – a method of autopsy where each organ is taken out one by one
• Virchow’s law, during craniosynostosis, skull growth is restricted to a plane perpendicular to the affected, prematurely
fused suture and is enhanced in a plane parallel to it
• Virchow–Robin spaces, enlarged perivascular spaces (EPVS) (often only potential) that surround blood vessels for a
short distance as they enter the brain 
• Virchow–Seckel syndrome, a very rare disease also known as “bird-headed dwarfism”
• Virchow’s triad, the classic factors which precipitate venous thrombus formation: endothelial dysfunction or injury,
hemodynamic changes, and hypercoagulability
25. Answer: B (Ref. Robbins Basic Pathology, By Vinay Kumar, Abul K. Abbas, Jon C. Aster, 10th edition, page no. 73)
26. Answer: C (Ref. Robbins Basic Pathology, By Vinay Kumar, Abul K. Abbas, Jon C. Aster, 10th edition, page no. 78)
27. Answer: D (Ref. Diagnostic Pathology of Infectious Disease, Richard L. Kradin, 2010, page no. 417)
28. Answer: B (Ref. Essential Pathology For Dental Students, by Harsh Mohan, Sugandha Mohan, 2011, page no. 157)
29. Answer: A (Ref. British Medical Journal, Vol. 1872, page no. 229)
30. Answer: D (Ref. Langman’s Medical Embryology, by Thomas W. Sadler, 2011, page no. 85)
31. Answer: D (Ref. Robbins Basic Pathology, By Vinay Kumar, Abul K. Abbas, Jon C. Aster, 10th edition, page no. 75)
32. Answer: D (Ref. “previous question”)
33. Answer: D (Ref. “previous question”)
34. Answer: A (Ref. Robbins Basic Pathology, By Vinay Kumar, Abul K. Abbas, Jon C. Aster, 10th edition, page no. 526)
35. Answer: B (Ref. Robbins Basic Pathology, By Vinay Kumar, Abul K. Abbas, Jon C. Aster, 10th edition, page no. 460)
36. Answer: D (Ref. Robbins Basic Pathology, By Vinay Kumar, Abul K. Abbas, Jon C. Aster, 10th edition, page no. 462)
37. Answer: B (Ref. Robbins and Cotran Pathologic Basis of Disease, By Vinay Kumar, Abul K. Abbas, Jon C. Aster, 9th
edition, page no. 379)
38. Answer: A (Ref. Robbins and Cotran Pathologic Basis of Disease, By Vinay Kumar, Abul K. Abbas, Jon C. Aster, 9th
edition, page no. 377)
• The Fernandez reaction is a reaction that occurs to signal a positive result in the lepromin skin test for leprosy.
• The reaction occurs in the skin at the site of injection if the body possesses antibodies to the Dharmendra antigen, one
of the antigens found in Mycobacterium leprae, the bacteria that causes leprosy.
• This reaction occurs as delayed-type hypersensitivity mechanism.
• This reaction occurs within 48 hours of injection of lepromin and is seen in both lepromatous and tuberculoid forms of
leprosy.
• In contrast, the Mitsuda reaction occurs 5–6 weeks after injection of lepromin and is only seen in patients with the
tuberculoid form of leprosy (not the lepromatous form, in which the body does not mount a strong response against
the bacterium). In terms of mechanism of action and appearance, the reaction is similar to the tuberculin reaction of a
positive Mantoux test for tuberculosis.
39. Answer: D (Ref. Robbins Basic Pathology, By Vinay Kumar, Abul K. Abbas, Jon C. Aster, 10th edition, page no. 513)

https://t.me/DentalBooksWorld
Chapter 3 • General Pathology 205

• The Kveim test, Nickerson–Kveim, or Kveim–Siltzbach test is a skin test used to detect sarcoidosis, where part of a
spleen from a patient with known sarcoidosis is injected into the skin of a patient suspected to have the disease.
• If noncaseating granulomas are found (4–6 weeks later), the test is positive.
• If the patient has been on treatment (e.g., glucocorticoids), the test may be false negative.
40. Answer: D (Ref. Robbins Basic Pathology, By Vinay Kumar, Abul K. Abbas, Jon C. Aster, 10th edition, page no.526)
41. Answer: B (Ref. Robbins Basic Pathology, By Vinay Kumar, Abul K. Abbas, Jon C. Aster, 10th edition, page no. 344)
42. Answer: D
43. Answer: A (Ref. Robbins Basic Pathology, By Vinay Kumar, Abul K. Abbas, Jon C. Aster, 9th edition, page no. 91)
44. Answer: D (Ref. “previous question”)
45. Answer: A (Ref. Robbins Basic Pathology, By Vinay Kumar, Abul K. Abbas, Jon C. Aster, 10th edition, page no. 76)
46. Answer: C (Ref. Robbins Basic Pathology, By Vinay Kumar, Abul K. Abbas, Jon C. Aster, 10th edition, page no. 141)
47. Answer: D (Ref. “previous question”)
48. Answer: C (Ref. “previous question”)
49. Answer: D (Ref. Robbins Basic Pathology, By Vinay Kumar, Abul K. Abbas, Jon C. Aster, 9th edition, page no. 377)
50. Answer: D (Ref. Robbins Basic Pathology, By Vinay Kumar, Abul K. Abbas, Jon C. Aster, 10th edition, page no. 67)
Kupffer cells, also known as stellate macrophages and Kupffer–Browicz cells, are specialized macrophages located in the
liver, lining the walls of the sinusoids that form part of the mononuclear phagocyte system.
51. Answer: C (Ref. Robbins Basic Pathology, By Vinay Kumar, Abul K. Abbas, Jon C. Aster, 10th edition, page no. 545)
52. Answer: B (Ref. “previous question”)
53. Answer: D (Ref. Robbins Basic Pathology, By Vinay Kumar, Abul K. Abbas, Jon C. Aster, 10th edition, page no. 52)
54. Answer: D (Ref. Robbins Basic Pathology, By Vinay Kumar, Abul K. Abbas, Jon C. Aster, 10th edition, page no. 71)
55. Answer: A (Ref. Robbins Basic Pathology, By Vinay Kumar, Abul K. Abbas, Jon C. Aster, 10th edition, page no. 710)
56. Answer: B (Ref. Robbins Basic Pathology, by Vinay Kumar, Abul K. Abbas, Jon C. Aster, 10th edition, page no. 404)
57. Answer: D (Ref. Robbins Basic Pathology, by Vinay Kumar, Abul K. Abbas, Jon C. Aster, 10th edition, page no. 297)
58. Answer: D (Ref. Robbins Basic Pathology, by Vinay Kumar, Abul K. Abbas, Jon C. Aster, 10th edition, page no. 405)
59. Answer: B (Ref. Robbins Basic Pathology, by Vinay Kumar, Abul K. Abbas, Jon C. Aster, 10th edition, page no. 407)
60. Answer: B (Ref. Robbins Basic Pathology, by Vinay Kumar, Abul K. Abbas, Jon C. Aster, 10th edition, page no. 409)
61. Answer: A (Ref. Robbins Basic Pathology, By Vinay Kumar, Abul K. Abbas, Jon C. Aster, 10th edition, page no. 61)
62. Answer: C (Ref. Robbins and Cotran Pathologic Basis of Disease, By Vinay Kumar, Abul K. Abbas, Jon C. Aster, 9th
edition, page no. 379)
63. Answer: C (Ref. Robbins and Cotran Pathologic Basis of Disease, By Vinay Kumar, Abul K. Abbas, Jon C. Aster, 9th
edition, page no. 378)
64. Answer: C (Ref. “previous questions”)
65. Answer: A (Ref. “previous questions”)
66. Answer: C (Ref. Robbins Basic Pathology, By Vinay Kumar, Abul K. Abbas, Jon C. Aster, 10th edition, page no. 70)
67. Answer: D (Ref. “previous questions”)
68. Answer: B (Ref. Robbins Basic Pathology, By Vinay Kumar, Abul K. Abbas, Jon C. Aster, 10th edition, page no. 73)
69. Answer: D (Ref. Robbins Basic Pathology, By Vinay Kumar, Abul K. Abbas, Jon C. Aster, 10th edition, page no. 379)
70. Answer: C (Ref. Robbins Basic Pathology, By Vinay Kumar, Abul K. Abbas, Jon C. Aster, 10th edition, page no. 83)
71. Answer: A (Ref. Robbins Basic Pathology, By Vinay Kumar, Abul K. Abbas, Jon C. Aster, 10th edition, page no. 71)
72. Answer: C (Ref. Robbins Basic Pathology, By Vinay Kumar, Abul K. Abbas, Jon C. Aster, 10th edition, page no. 76)
73. Answer: C (Ref. Robbins Basic Pathology, By Vinay Kumar, Abul K. Abbas, Jon C. Aster, 10th edition, page no. 69)
74. Answer: B (Ref. “previous question”)
75. Answer: C (Ref. “previous question”)
76. Answer: C (Ref. “previous question”)
77. Answer: C (Ref. “previous question”)
78. Answer: D (Ref. Robbins Basic Pathology, By Vinay Kumar, Abul K. Abbas, Jon C. Aster, 10th edition, page no. 82)
79. Answer: D (Ref. “previous question”)
80. Answer: D (Ref. Robbins and Cotran Pathologic Basis of Disease, By Vinay Kumar, Abul K. Abbas, Jon C. Aster, 9th
edition, page no. 253)
81. Answer: A (Ref. “previous question”)
82. Answer: C (Ref. Conn’s Current Therapy, By Edward T. Bope, Rick D. Kellerman, 2017, page no. 547)

https://t.me/DentalBooksWorld
206 Triumph's Complete Review of Dentistry

83. Answer: C (Ref. Robbins Basic Pathology, By Vinay Kumar, Abul K. Abbas, Jon C. Aster, 10th edition, page no. 707)
84. Answer: C (Ref. “previous question”)
85. Answer: C (Ref. Robbins Basic Pathology, By Vinay Kumar, Abul K. Abbas, Jon C. Aster, 10th edition, page no. 504)
86. Answer: A (Ref. Robbins Basic Pathology, By Vinay Kumar, Abul K. Abbas, Jon C. Aster, 10th edition, page no. 618)
87. Answer: C (Ref. “previous question”)
88. Answer: B (Ref. Essential Pathology For Dental Students By Harsh Mohan, Sugandha Mohan, 4th edition, page no. 114)
89. Answer: C (Ref. “previous question”)
90. Answer: C (Ref. “previous question”)
91. Answer: C (Ref. Robbins Basic Pathology, By Vinay Kumar, Abul K. Abbas, Jon C. Aster, 10th edition, page no. 344)
92. Answer: C (Ref. Robbins Basic Pathology, By Vinay Kumar, Abul K. Abbas, Jon C. Aster, 10th edition, page no. 513)
93. Answer: B (Ref. Robbins and Cotran Pathologic Basis of Disease, By Vinay Kumar, Abul K. Abbas, Jon C. Aster, 9th
edition, page no. 25)
94. Answer: B (Ref. Robbins Basic Pathology, By Vinay Kumar, Abul K. Abbas, Jon C. Aster, 10th edition, page no. 92)
95. Answer: D (Ref. Robbins Basic Pathology, By Vinay Kumar, Abul K. Abbas, Jon C. Aster, 10th edition, page no. 22)
96. Answer: D (Ref. “previous question”)
97. Answer: A (Ref. Robbins Basic Pathology, By Vinay Kumar, Abul K. Abbas, Jon C. Aster, 10th edition, page no. 90)
98. Answer: C (Ref. Robbins Basic Pathology, By Vinay Kumar, Abul K. Abbas, Jon C. Aster, 10th edition, page no. 61)
99. Answer: D (Ref. Robbins Basic Pathology, By Vinay Kumar, Abul K. Abbas, Jon C. Aster, 10th edition, page no. 76)
100. Answer: A (Ref. Robbins Basic Pathology, By Vinay Kumar, Abul K. Abbas, Jon C. Aster, 10th edition, page no. 44)
• Superoxide dismutase (SOD) is an antioxidant enzyme.
• Nitric oxide (NO), an important chemical mediator generated by endothelial cells, macrophages, neurons, and other cell
types, can act as a free radical and can also be converted to a highly reactive form.
101. Answer: C (Ref. “previous question”)
102. Answer: D (Ref. “previous question”)
103. Answer: A (Ref. “previous question”)
104. Answer: A (Ref. Robbins Basic Pathology, By Vinay Kumar, Abul K. Abbas, Jon C. Aster, 10th edition, page no. 44)
• The initiating enzyme for this process is NADPH oxidase (also called respiratory burst oxidase).
• Glutathione peroxidase, glutathione reductase, and superoxide dismutase are examples of antioxidants. They reduce free
radical formation.
105. Answer: A (Ref. Robbins Basic Pathology, By Vinay Kumar, Abul K. Abbas, Jon C. Aster, 10th edition, page no. 92)
• Extracellular Matrix (ECM) comprises of interstitial matrix and basement membrane. The degradation of collagen and other
ECM proteins is achieved by a family of matrix metalloproteinases (MMPs) which are dependent on zinc ions for their activity.
• MMP8 and MMP2 are collagenases which cleave type IV collagen of basement membranes.
• MMPs also have a role in tumor cell invasion.
106. Answer: D (Ref. Robbins Basic Pathology, By Vinay Kumar, Abul K. Abbas, Jon C. Aster, 10th edition, page no. 62)
107a. Answer: B (Ref. “previous question”)
107b. Answer: D (Ref. “previous question”)
With acute inflammation, hydrostatic pressure is increased (due to increased blood flow from vasodilation) and at the
same time osmotic pressure is reduced because of protein leakage (due to increased permeability)
108. Answer: D (Ref. “previous question”)
109. Answer: A (Ref. “previous question”)
110. Answer: D (Ref. Robbins Basic Pathology, By Vinay Kumar, Abul K. Abbas, Jon C. Aster, 9th edition, page no. 140)
• X-linked agammaglobulinemia (XLA), or Bruton agammaglobulinemia, is an inherited immunodeficiency disease
caused by mutations in the gene coding for Bruton tyrosine kinase (BTK).
• BTK is necessary for the proliferation and the differentiation of B lymphocytes.
111. Answer: A (Ref. Robbins and Cotran Atlas of Pathology, By Edward C. Klatt, 3rd edition, page no. 66)
112. Answer: B (Ref. “previous question”)
113. Answer: A (Ref. Robbins Basic Pathology, By Vinay Kumar, Abul K. Abbas, Jon C. Aster, 10th edition, page no. 63)
114. Answer: C (Ref. Robbins Basic Pathology, By Vinay Kumar, Abul K. Abbas, Jon C. Aster, 10th edition, page no. 384)
115. Answer: B (Ref. “previous question”)
116. Answer: D (Ref. Robbins and Cotran Review of Pathology, By Edward C. Klatt, Vinay Kumar, 2015, page no. 19)
117. Answer: B (Ref. Robbins Basic Pathology, By Vinay Kumar, Abul K. Abbas, Jon C. Aster, 10th edition, page no. 384)

https://t.me/DentalBooksWorld
Chapter 3 • General Pathology 207

118. Answer: D (Ref. Robbins Basic Pathology, By Vinay Kumar, Abul K. Abbas, Jon C. Aster, 10th edition, page no. 384)
119. Answer: A (Ref. Essential Microbiology for Dentistry, By Lakshman Samaranayake)
120. Answer: D (Ref. Immunobiology: The Immune System in Health and Disease, By Charles Janeway, Paul Travers, Mark
Walport, 2001)
121. Answer: D (Ref. Robbins and Cotran Review of Pathology, By Edward C. Klatt, Vinay Kumar, 2015)
122. Answer: C (Ref. Inflammation and the Microcirculation, By D. Neil Granger, Elena Senchenkova, 2010)
123. Answer: A (Ref. Robbins Basic Pathology, By Vinay Kumar, Abul K. Abbas, Jon C. Aster, 10th edition, page no. 64)
124. Answer: D (Ref. “previous question”)
125. Answer: B (Ref. “previous question”)
126. Answer: D (Ref. Robbins Basic Pathology, By Vinay Kumar, Abul K. Abbas, Jon C. Aster, 10th edition, page no. 77)
127. Answer: D (Ref. Robbins Basic Pathology, By Vinay Kumar, Abul K. Abbas, Jon C. Aster, 10th edition, page no. 116)
128. Answer: C (Ref. “previous question”)
129. Answer: B (Ref. “previous question”)
130. Answer: A (Ref. Robbins Basic Pathology, By Vinay Kumar, Abul K. Abbas, Jon C. Aster, 10th edition, page no. 329)
131. Answer: D (Ref. “previous question”)
132. Answer: D (Ref. “previous question”)
133. Answer: A (Ref. A Rubin’s Pathology: Clinicopathologic Foundations of Medicine, Edited by Raphael Rubin, David S.
Strayer, Emanuel Rubin, 6th edition, page no. 52)
134. Answer: C (Ref. “previous question”)
135. Answer: D (Ref. “previous question”)
136. Answer: A (Ref. “previous question”)
137. Answer: B (Ref. Robbins Basic Pathology, By Vinay Kumar, Abul K. Abbas, Jon C. Aster, 10th edition, page no. 74)
138. Answer: A (Ref. “previous question”)
139. Answer: C (Ref. “previous question”)
140. Answer: C (Ref. Robbins Basic Pathology, By Vinay Kumar, Abul K. Abbas, Jon C. Aster, 10th edition, page no. 74)
141. Answer: A (Ref. Robbins Basic Pathology, By Vinay Kumar, Abul K. Abbas, Jon C. Aster, 10th edition, page no. 354)
142. Answer: A (Ref. Robbins Basic Pathology, By Vinay Kumar, Abul K. Abbas, Jon C. Aster, 10th edition, page no. 130)
143. Answer: D (Ref. Robbins Basic Pathology, By Vinay Kumar, Abul K. Abbas, Jon C. Aster, 10th edition, page no. 104)
144. Answer: D (Ref. “previous question”)
145. Answer: D (Ref. “previous question”)
146. Answer: B (Ref. “previous question”)
147. Answer: C (Ref. “previous question”)
148. Answer: D (Ref. Robbins Basic Pathology, By Vinay Kumar, Abul K. Abbas, Jon C. Aster, 10th edition, page no. 68)
149. Answer: D (Ref. “previous question”)
150. Answer: A (Ref. A Rubin’s Pathology: Clinicopathologic Foundations of Medicine, Edited by Raphael Rubin, David S.
Strayer, Emanuel Rubin, 6th edition, page no. 982)
• Weibel–Palade bodies are the storage granules of endothelial cells, it forms the inner lining of the blood vessels and
heart.
• They store and release two principal molecules: von Willebrand factor and P-selectin; thus plays a dual role in
hemostasis and inflammation.
151. Answer: A (Ref. “previous question”)
152. Answer: D (Ref. “previous question”)
153. Answer: D (Ref. “previous question”)
154. Answer: D (Ref. Robbins Basic Pathology, By Vinay Kumar, Abul K. Abbas, Jon C. Aster, 10th edition, page no. 72)
155. Answer: A (Ref. Robbins Basic Pathology, By Vinay Kumar, Abul K. Abbas, Jon C. Aster, 10th edition, page no. 484)
156. Answer: B (Ref. Robbins Basic Pathology, By Vinay Kumar, Abul K. Abbas, Jon C. Aster, 10th edition, page no. 504)
157. Answer: B (Ref. Robbins Basic Pathology, By Vinay Kumar, Abul K. Abbas, Jon C. Aster, 10th edition, page no. 72)
158. Answer: B (Ref. “previous question”)
159. Answer: A (Ref. “previous question”)
160. Answer: C (Ref. “previous question”)
161. Answer: A (Ref. “previous question”)
162. Answer: A (Ref. Robbins Basic Pathology, By Vinay Kumar, Abul K. Abbas, Jon C. Aster, 9th edition, page no. 312)

https://t.me/DentalBooksWorld
208 Triumph's Complete Review of Dentistry

163. Answer: A (Ref. “previous question”)


164. Answer: A (Ref. Robbins Basic Pathology, By Vinay Kumar, Abul K. Abbas, Jon C. Aster, 9th edition, page no. 274)
165. Answer: B (Ref. Robbins Basic Pathology, By Vinay Kumar, Abul K. Abbas, Jon C. Aster, 10th edition, page no. 24)
166. Answer: B (Ref. Robbins Basic Pathology, By Vinay Kumar, Abul K. Abbas, Jon C. Aster, 10th edition, page no. 117)
167. Answer: A (Ref. “previous question”)
168. Answer: B (Ref. Robbins Basic Pathology, By Vinay Kumar, Abul K. Abbas, Jon C. Aster, 9th edition, page no. 501)
169. Answer: C (Ref. Robbins Basic Pathology, By Vinay Kumar, Abul K. Abbas, Jon C. Aster, 10th edition, page no. 77)
170. Answer: B (Ref. “previous question”)
171. Answer: A (Ref. “previous question”)
172. Answer: D (Ref. “previous question”)
173. Answer: C (Ref. “previous question”)
174. Answer: D (Ref. “previous question”)
175. Answer: D (Ref. Robbins Basic Pathology, By Vinay Kumar, Abul K. Abbas, Jon C. Aster, 10th edition, page no. 116)
176. Answer: D (Ref. Robbins Basic Pathology, By Vinay Kumar, Abul K. Abbas, Jon C. Aster, 9th edition, page no. 465)
177. Answer: B (Ref. Robbins Basic Pathology, By Vinay Kumar, Abul K. Abbas, Jon C. Aster, 9th edition, page no. 271)
178. Answer: D (Ref. Robbins Basic Pathology, By Vinay Kumar, Abul K. Abbas, Jon C. Aster, 10th edition, page no. 522)
179. Answer: C (Ref. Robbins Basic Pathology, By Vinay Kumar, Abul K. Abbas, Jon C. Aster, 9th edition, page no. 98)
180. Answer: C (Ref. Robbins Basic Pathology, By Vinay Kumar, Abul K. Abbas, Jon C. Aster, 9th edition, page no. 210
181. Answer: B (Ref. Robbins Basic Pathology, By Vinay Kumar, Abul K. Abbas, Jon C. Aster, 10th edition, page no. 37)
182. Answer: A (Ref. “previous question”)
183. Answer: D (Ref. “previous question”)
184. Answer: D (Ref. Robbins Basic Pathology, By Vinay Kumar, Abul K. Abbas, Jon C. Aster, 10th edition, page no. 345)
185. Answer: A (Ref. Robbins and Cotran Atlas of Pathology, By Edward C. Klatt, 3rd edition, page no. 139)
186. Answer: B (Ref. “previous question”)
187. Answer: D (Ref. “previous question”)
188. Answer: A (Ref. “previous question”)
189. Answer: A (Ref. “previous question”)
190. Answer: B (Ref. “previous question”)
191. Answer: A (Ref. Robbins Basic Pathology, By Vinay Kumar, Abul K. Abbas, Jon C. Aster, 10th edition, page no. 868)
192. Answer: D (Ref. Robbins Basic Pathology, By Vinay Kumar, Abul K. Abbas, Jon C. Aster, 10th edition, page no. 868)
193. Answer: A (Ref. Robbins Basic Pathology, By Vinay Kumar, Abul K. Abbas, Jon C. Aster, 10th edition, page no.495)
194. Answer: C (Ref. Robbins Basic Pathology, By Vinay Kumar, Abul K. Abbas, Jon C. Aster, 10th edition, page no. 332)
195. Answer: A (Ref. “previous question”)
196. Answer: B (Ref. “previous question”)
197. Answer: B (Ref. Robbins Basic Pathology, By Vinay Kumar, Abul K. Abbas, Jon C. Aster, 10th edition, page no. 92)
198. Answer: C (Ref. Robbins and Cotran Review of Pathology, By Edward C. Klatt, Vinay Kumar, 4th edition, page no. 45)
Tenascin C (TN-C) is a glycoprotein that in humans is encoded by the TNC gene. It is expressed in the extracellular matrix
of various tissues during development, disease, or injury, and in restricted neurogenic areas of the central nervous system.
199. Answer: C (Ref. Robbins Basic Pathology, By Vinay Kumar, Abul K. Abbas, Jon C. Aster, 10th edition, page no. 51)
200. Answer: A (Ref. Robbins Basic Pathology, By Vinay Kumar, Abul K. Abbas, Jon C. Aster, 10th edition, page no. 869)
201. Answer: C (Ref. “previous question”)
202. Answer: B (Ref. “previous question”)
203. Answer: C (Ref. Robbins Basic Pathology, By Vinay Kumar, Abul K. Abbas, Jon C. Aster, 10th edition, page no. 128)
204. Answer: D (Ref. Robbins Basic Pathology, By Vinay Kumar, Abul K. Abbas, Jon C. Aster, 10th edition, page no. 4)
205. Answer: B (Ref. Robbins Basic Pathology, By Vinay Kumar, Abul K. Abbas, Jon C. Aster, 10th edition, page no. 458)
206. Answer: D (Ref. Robbins Basic Pathology, By Vinay Kumar, Abul K. Abbas, Jon C. Aster, 10th edition, page no. 210)
207. Answer: C (Ref. “previous question”)
208. Answer: D (Ref. “previous question”)
209. Answer: D (Ref. “previous question”)
210. Answer: D (Ref. “previous question”)
211. Answer: C (Ref. “previous question”)

https://t.me/DentalBooksWorld
Chapter 3 • General Pathology 209

212. Answer: A (Ref. “previous question”)


213. Answer: A (Ref. “previous question”)
214. Answer: D (Ref. “previous question”)
215. Answer: A
216. Answer: A (Ref. “previous question”)
217. Answer: D (Ref. Robbins Basic Pathology, By Vinay Kumar, Abul K. Abbas, Jon C. Aster, 10th edition, page no. 93)
218. Answer: D (Ref. Essential Pathology For Dental Students, By Harsh Mohan, Sugandha Mohan, 4th edition, page no. 135)
219. Answer: A (Ref. “previous question”)
220. Answer: A (Ref. “previous question”)
221. Answer: B (Ref. Essential Pathology For Dental Students, By Harsh Mohan, Sugandha Mohan, 4th edition, page no. 155)
222. Answer: D (Ref. Robbins Basic Pathology, By Vinay Kumar, Abul K. Abbas, Jon C. Aster, 10th edition, page no. 333)
The copper compound escalates the amount of protein synthesis in collagen and elastin, which aids in wound healing.
223. Answer: B (Ref. “previous question”)
224. Answer: A (Ref. “previous question”)
225. Answer: C (Ref. “previous question”)
226. Answer: D (Ref. “previous question”)
227. Answer: D (Ref. “previous question”)
228. Answer: B (Ref. “previous question”)
229. Answer: B (Ref. “previous question”)
230. Answer: B (Ref. Robbins Basic Pathology, By Vinay Kumar, Abul K. Abbas, Jon C. Aster, 10th edition, page no. 90)
231. Answer: D (Ref. “previous question”)

HEMODYNAMICS
1. Answer: C (Ref. Robbins and Cotran Pathologic Basis of Disease, By Vinay Kumar, Abul K. Abbas, Jon C. Aster,
9th edition, page no. 98)
• Plasma can be found within the vascular system of the body while body cells are surrounded by interstitial fluid.
• The protein concentration in the plasma is higher than in interstitial fluid.
• Hormones and plasma proteins exist within the plasma, whereas some hormones and proteins secreted by cells can be
seen in interstitial fluid.
• Erythrocytes, leucocytes, and platelets are the major cell types found in blood plasma. Unlike in plasma, only some
leucocytes can be found in interstitial fluid.
• The glucose concentration is higher in plasma than interstitial fluid.
• Amino acid and oxygen concentration are also higher in plasma than in interstitial fluid.
• One-fourth of the extracellular fluid is composed of water in plasma. The remaining three-fourths is composed of water
in interstitial fluid and lymph.
• The concentrations of diffusible cations are higher in plasma than that of interstitial fluid.
• The diffusible anion concentrations are lower in plasma than interstitial fluid.
• The concentrations of bound and free Ca2+ and Mg2+ are higher in plasma than in interstitial fluid.
2. Answer: A (Ref. Robbins and Cotran Pathologic Basis of Disease, By Vinay Kumar, Abul K. Abbas, Jon C. Aster,
9th edition, page no. 99)
3. Answer: C (Ref. “Previous question”)
Hypoalbuminemia (or hypoalbuminaemia) is a medical sign in which the level of albumin in the blood is abnormally low.
Low albumin levels can be an indicator of chronic malnutrition or protein losing enteropathy. Hypoalbuminemia may
cause generalized edema (swelling) via a decrease in oncotic pressure.
4. Answer: B (Ref. Robbins and Cotran Pathologic Basis of Disease, By Vinay Kumar, Abul K. Abbas, Jon C. Aster,
3rd edition, page no. 73)
5. Answer: C (Ref. Robbins and Cotran Pathologic Basis of Disease, By Vinay Kumar, Abul K. Abbas, Jon C. Aster,
9th edition, page no. 550)
6. Answer: D (Ref. “Previous question”)
7. Answer: D (Ref. Robbins and Cotran Pathologic Basis of Disease, By Vinay Kumar, Abul K. Abbas, Jon C. Aster,
3rd edition, page no. 671)

https://t.me/DentalBooksWorld
210 Triumph's Complete Review of Dentistry

8. Answer: B (Ref. Robbins and Cotran Pathologic Basis of Disease, By Vinay Kumar, Abul K. Abbas, Jon C. Aster,
9th edition, page no. 392)
9. Answer: D (Ref. Robbins and Cotran Pathologic Basis of Disease, By Vinay Kumar, Abul K. Abbas, Jon C. Aster,
3rd edition, page no. 671)
10. Answer: A (Ref. Robbins and Cotran Pathologic Basis of Disease, By Vinay Kumar, Abul K. Abbas, Jon C. Aster,
9th edition, page no. 116)
11. Answer: D (Ref. Robbins and Cotran Pathologic Basis of Disease, By Vinay Kumar, Abul K. Abbas, Jon C. Aster,
3rd edition, page no. 656)
12. Answer: A (Ref. Robbins and Cotran Pathologic Basis of Disease, By Vinay Kumar, Abul K. Abbas, Jon C. Aster,
9th edition, page no. 112)
13. Answer: C (Ref. Robbins and Cotran Pathologic Basis of Disease, By Vinay Kumar, Abul K. Abbas, Jon C. Aster,
9th edition, page no. 112)
14. Answer: A (Ref. Robbins and Cotran Pathologic Basis of Disease, By Vinay Kumar, Abul K. Abbas, Jon C. Aster,
9th edition, page no. 114)
15. Answer: C (Ref. Robbins and Cotran Pathologic Basis of Disease, By Vinay Kumar, Abul K. Abbas, Jon C. Aster,
3rd edition, page no. 698)
16. Answer: C (Ref. Robbins and Cotran Pathologic Basis of Disease, By Vinay Kumar, Abul K. Abbas, Jon C. Aster,
9th edition, page no. 926)
17. Answer: C
18. Answer: C (Ref. Robbins and Cotran Pathologic Basis of Disease, By Vinay Kumar, Abul K. Abbas, Jon C. Aster,
3rd edition, page no.125)
19. Answer: A (Ref. “Previous question”)
20. Answer: C (Ref. “Previous question”)
21. Answer: B (Ref. “Previous question”)
22. Answer: B (Ref. Robbins and Cotran Review of Pathology, By Edward C. Klatt, Vinay Kumar, 3rd edition, page no. 36)
23. Answer: C (Ref. Robbins and Cotran Pathologic Basis of Disease, By Vinay Kumar, Abul K. Abbas, Jon C. Aster,
9th edition, page no. 319)
24. Answer: D (Ref. Robbins and Cotran Pathologic Basis of Disease, By Vinay Kumar, Abul K. Abbas, Jon C. Aster,
9th edition, page no. 116)
25. Answer: B (Ref. “Previous question”)
26. Answer: B (Ref. “Previous question”)
27. Answer: D (Ref. “Previous question”)
28. Answer: D (Ref. Robbins and Cotran Pathologic Basis of Disease, By Vinay Kumar, Abul K. Abbas, Jon C. Aster,
9th edition, page no. 119)
29. Answer: C (Ref. Robbins and Cotran Pathologic Basis of Disease, By Vinay Kumar, Abul K. Abbas, Jon C. Aster,
9th edition, page no. 519)
30. Answer: B (Ref. Robbins and Cotran Pathologic Basis of Disease, By Vinay Kumar, Abul K. Abbas, Jon C. Aster,
9th edition, page no. 402)
31. Answer: B (Ref. Robbins and Cotran Atlas of Pathology, By Edward C. Klatt, 3rd edition, page no. 238)
32. Answer: B (Ref. Robbins and Cotran Review of Pathology, By Edward C. Klatt, Vinay Kumar, 4th edition, page no. 651)
33. Answer: D (Ref. Robbins and Cotran Pathologic Basis of Disease, By Vinay Kumar, Abul K. Abbas, Jon C. Aster,
9th edition, page no. 110)
34. Answer: C (Ref. Robbins and Cotran Pathologic Basis of Disease, By Vinay Kumar, Abul K. Abbas, Jon C. Aster,
9th edition, page no. 113)
35. Answer: C (Ref. Robbins and Cotran Pathologic Basis of Disease, By Vinay Kumar, Abul K. Abbas, Jon C. Aster,
9th edition, page no. 114)
36. Answer: B (Ref. Robbins and Cotran Pathologic Basis of Disease, By Vinay Kumar, Abul K. Abbas, Jon C. Aster,
9th edition, page no. 105)
37. Answer: D (Ref. Robbins and Cotran Pathologic Basis of Disease, By Vinay Kumar, Abul K. Abbas, Jon C. Aster,
9th edition, page no. 113)
38. Answer: A (Ref. Robbins and Cotran Review of Pathology, By Edward C. Klatt, Vinay Kumar, 4th edition, page no. 514)
39. Answer: D (Ref. Robbins and Cotran Review of Pathology, By Edward C. Klatt, Vinay Kumar, 4th edition, page no. 43)

https://t.me/DentalBooksWorld
Chapter 3 • General Pathology 211

40. Answer: D (Ref. Robbins and Cotran Pathologic Basis of Disease, By Vinay Kumar, Abul K. Abbas, Jon C. Aster,
9th edition, page no. 97)
41. Answer: A (Ref. Robbins and Cotran Pathologic Basis of Disease, By Vinay Kumar, Abul K. Abbas, Jon C. Aster,
9th edition, page no. 401)
42. Answer: D (Ref. Robbins and Cotran Pathologic Basis of Disease, By Vinay Kumar, Abul K. Abbas, Jon C. Aster,
9th edition, page no. 108)
43. Answer: D (Ref. Robbins and Cotran Pathologic Basis of Disease, By Vinay Kumar, Abul K. Abbas, Jon C. Aster,
9th edition, page no. 109)
44. Answer: D
45. Answer: D (Ref. Robbins and Cotran Review of Pathology, By Edward C. Klatt, Vinay Kumar, 4th edition, page no. 159)
46. Answer: C (Ref. Robbins and Cotran Review of Pathology, By Edward C. Klatt, Vinay Kumar, 4th edition, page no. 488)
47. Answer: B (Ref. Robbins and Cotran Pathologic Basis of Disease, By Vinay Kumar, Abul K. Abbas, Jon C. Aster,
9th edition, page no. 111)
48. Answer: A (Ref. Robbins and Cotran Pathologic Basis of Disease, By Vinay Kumar, Abul K. Abbas, Jon C. Aster,
9th edition, page no. 110)
49. Answer: C (Ref. Robbins and Cotran Pathologic Basis of Disease, By Vinay Kumar, Abul K. Abbas, Jon C. Aster,
9th edition, page no. 164)
50. Answer: D (Ref. “Previous question”)
51. Answer: D (Ref. “Previous question”)
52. Answer: A (Ref. Robbins and Cotran Pathologic Basis of Disease, By Vinay Kumar, Abul K. Abbas, Jon C. Aster,
4th edition, page no. 854)
53. Answer: A (Ref. Robbins and Cotran Pathologic Basis of Disease, By Vinay Kumar, Abul K. Abbas, Jon C. Aster,
4th edition, page no. 530)
54. Answer: C (Ref. Robbins and Cotran Pathologic Basis of Disease, By Vinay Kumar, Abul K. Abbas, Jon C. Aster,
4th edition, page no. 530)
55. Answer: B
56. Answer: D (Ref. Robbins and Cotran Pathologic Basis of Disease, By Vinay Kumar, Abul K. Abbas, Jon C. Aster,
9th edition, page no. 419)
57. Answer: B (Ref. Robbins and Cotran Pathologic Basis of Disease, By Vinay Kumar, Abul K. Abbas, Jon C. Aster,
9th edition, page no. 638)
58. Answer: D (Ref. Robbins and Cotran Pathologic Basis of Disease, By Vinay Kumar, Abul K. Abbas, Jon C. Aster,
9th edition, page no. 414)
59. Answer: A (Ref. Robbins and Cotran Pathologic Basis of Disease, By Vinay Kumar, Abul K. Abbas, Jon C. Aster,
4th edition, page no. 698)
60. Answer: B (Ref. Robbins and Cotran Pathologic Basis of Disease, By Vinay Kumar, Abul K. Abbas, Jon C. Aster,
4th edition, page no. 1266)
61. Answer: B (Ref. Robbins and Cotran Pathologic Basis of Disease, By Vinay Kumar, Abul K. Abbas, Jon C. Aster,
4th edition, page no. 491)
62. Answer: B (Ref. Robbins and Cotran Pathologic Basis of Disease, By Vinay Kumar, Abul K. Abbas, Jon C. Aster,
9th edition, page no. 379)
63. Answer: D (Ref. Robbins and Cotran Pathologic Basis of Disease, By Vinay Kumar, Abul K. Abbas, Jon C. Aster,
4th edition, page no. 448)
64. Answer: D (Ref. “Previous question”)
65. Answer: D (Ref. “Previous question”)
66. Answer: D (Ref. “Previous question”)
67. Answer: A (Ref. “Previous question”)
68. Answer: B (Ref. Robbins and Cotran Pathologic Basis of Disease, By Vinay Kumar, Abul K. Abbas, Jon C. Aster,
9th edition, page no. 386)
69. Answer: B (Ref. “Previous question”)
70. Answer: B (Ref. “Previous question”)

https://t.me/DentalBooksWorld
212 Triumph's Complete Review of Dentistry

71. Answer: B (Ref. Robbins and Cotran Pathologic Basis of Disease, By Vinay Kumar, Abul K. Abbas, Jon C. Aster,
9th edition, page no. 97)
Contact of plasma with negative charged surface activates intrinsic pathways (not extrinsic)
72. Answer: B (Ref. Robbins and Cotran Pathologic Basis of Disease, By Vinay Kumar, Abul K. Abbas, Jon C. Aster,
4th edition, page no. 442)
The binding of clotting factors II, VII, IX, and X to calcium depends on the addition of γ-carboxylation of glutamic acid
residues on these proteins. This step requires vitamin K as a cofactor.
• Increasing clotting
− Clotting factors II, VII, IX and X
− Inhibiting clotting
• Protein C and protein S
73. Answer: C (Ref. Robbins and Cotran Pathologic Basis of Disease, By Vinay Kumar, Abul K. Abbas, Jon C. Aster,
9th edition, page no. 556)
Na+ and water retention is now the more important cause of edema in nephrotic syndrome.
74. Answer: D (Ref. Robbins and Cotran Pathologic Basis of Disease, By Vinay Kumar, Abul K. Abbas, Jon C. Aster,
9th edition, page no. 106)
75. Answer: A (Ref. “Previous question”)
76. Answer: C (Ref. “Previous question”)
“All endothelial cells except those in the cerebral microcirculation produce thrombomodulin, a thrombin protein, and
express it on their surface.”
77. Answer: D (Ref. “Previous question”)
• Thrombin is clotting factor IIa which participates in coagulation cascade by converting factor I (soluble protein
fibrinogen) to factor Ia (insoluble fibrin).
• Protein C and protein S are Vitamin-K-dependent anticlotting proteins (remember factors II, VII, IX, and X are
Vitamin-K-dependent clotting factors). These act by inactivating factor Va and VIIIa.
• Thrombomodulin–thrombin complex activates protein C and thus the complex acts as an anticoagulant.
78. Answer: D (Ref. Robbins and Cotran Pathologic Basis of Disease, By Vinay Kumar, Abul K. Abbas, Jon C. Aster,
9th edition, page no. 72)
• TXA2 is also a powerful vasoconstrictor and bronchoconstrictor.
• Low-dose aspirin (50–325 mg) is used as antiplatelet drug because it inhibits COX irreversibly and decreases formation
of TXA2 by platelets.
79. Answer: D (Ref. Robbins and Cotran Pathologic Basis of Disease, By Vinay Kumar, Abul K. Abbas, Jon C. Aster,
9th edition, page no. 108)
80. Answer: C (Ref. Robbins and Cotran Pathologic Basis of Disease, By Vinay Kumar, Abul K. Abbas, Jon C. Aster,
9th edition, page no. 486)
VWF (von Willebrand Factor) is produced by endothelial cells and it is required for platelet binding to collagen and other
substances. So, it is a procoagulant factor.
81. Answer: A (Ref. Robbins and Cotran Pathologic Basis of Disease, By Vinay Kumar, Abul K. Abbas, Jon C. Aster,
9th edition, page no. 99)
82. Answer: D (Ref. Robbins and Cotran Pathologic Basis of Disease, By Vinay Kumar, Abul K. Abbas, Jon C. Aster,
9th edition, page no. 68)
83. Answer: C (Ref. Robbins and Cotran Pathologic Basis of Disease, By Vinay Kumar, Abul K. Abbas, Jon C. Aster,
4th edition, page no. 442)
84. Answer: D (Ref. Robbins and Cotran Pathologic Basis of Disease, By Vinay Kumar, Abul K. Abbas, Jon C. Aster,
4th edition, page no. 663)
85. Answer: A (Ref. Robbins and Cotran Pathologic Basis of Disease, By Vinay Kumar, Abul K. Abbas, Jon C. Aster,
9th edition, page no. 486)
86. Answer: C (Ref. “Previous question”)
87. Answer: C (Ref. “Previous question”)
Gamna–Gandy bodies in chronic venous congestion (CVC) of the spleen is characterized by calcific deposits admixed
with hemosiderin on fibrous tissue.
88. Answer: A (Ref. “Previous question”)

https://t.me/DentalBooksWorld
Chapter 3 • General Pathology 213

• Prothrombin time and extrinsic pathway – Factor 5/7


• Activated partial thromboplastin time and Intrinsic pathway – Factor 8
• Bleeding time and platelet function and platelet count – Platelet function and count
89. Answer: C (Ref. Robbins and Cotran Pathologic Basis of Disease, By Vinay Kumar, Abul K. Abbas, Jon C. Aster,
9th edition, page no. 109)
• Antiphospholipid antibody syndrome is characterized by antibodies against plasma proteins in complex with
phospholipid.
• In primary antiphospholipid antibody syndrome there is hypercoagulable state without evidence of autoimmune
disorders.
• In association with SLE or lupus, the name given is secondary antiphospholipid antibody syndrome.
• There is formation of antibody against phospholipid beta– 2-glycoprotein 1 complex.
• It also binds to cardiolipin antigen and lead to false positive test for syphilis.
• It also interferes with in vitro clotting time and so is known as lupus anticoagulant.
• In vivo, these patients have hypercoagulable state resulting in arterial and venous thrombosis resulting spontaneous
recurrent miscarriage and focal or cerebral ischemia.
90. Answer: A (Ref. “Previous question”)
The lungs have dual blood supply and so, they exhibit red infarct. The infarcts may be either red (hemorrhagic) or white
(anemic) and may be either septic or bland. All infarcts tend to be wedge shaped with the occluded vessel at the apex and
the periphery of the organ forming the base. The infarct microscopically has features of ischemic coagulative necrosis.
91. Answer: C (Ref. Robbins and Cotran Pathologic Basis of Disease, By Vinay Kumar, Abul K. Abbas, Jon C. Aster,
9th edition, page no. 109)
Primary (Genetics) Hypercoagulable state
• Mutations in factor V (Most common)
• Antithrombin III deficiency
• Protein C or S deficiency
• Fibrinolysis defects
• Homocysteinemia
• Allelic variations in prothrombin levels
• Mutations in the methyl tetra hydro folate (MTHF) gene
Secondary (Acquired) Hypercoagulable state
• Prolonged bed rest or immobilization
• Homocysteinemia
• Tissue damage (surgery, fracture, burns)
• Cancer
• MI, Prosthetic cardiac valves
• Disseminated intravascular coagulation (DIC)
• Heparin-induced thrombocytopenia
• Antiphospholipid antibody syndrome
92. Answer: B (Ref. “Previous question”)
93. Answer: B (Ref. Robbins Basic Pathology, By Vinay Kumar, Abul K. Abbas, Jon C. Aster, 9th edition, page no. 86)
The factors that predispose to venous thrombosis were initially described by Virchow in 1856 and are known as Virchow’s
triad. These include
• Stasis
• Vascular damage
• Hypercoagulability
94. Answer: B (Ref. Robbins and Cotran Pathologic Basis of Disease, By Vinay Kumar, Abul K. Abbas, Jon C. Aster,
9th edition, page no. 108)
• Mutation in factor V gene is caused by the substitution of glutamine for the normal arginine residue at position 506.
• It is known as Leiden mutation and it is the most common inherited cause of hypercoagulability.
• Lisbon mutation is associated with a mutation in thyroid peroxidase gene.
95. Answer: A (Ref. Robbins and Cotran Pathologic Basis of Disease, By Vinay Kumar, Abul K. Abbas, Jon C. Aster,
9th edition, page no. 109)

https://t.me/DentalBooksWorld
214 Triumph's Complete Review of Dentistry

96. Answer: D (Ref. Robbins and Cotran Pathologic Basis of Disease, By Vinay Kumar, Abul K. Abbas, Jon C. Aster,
4th edition, page no. 974)
97. Answer: D (Ref. Robbins and Cotran Pathologic Basis of Disease, By Vinay Kumar, Abul K. Abbas, Jon C. Aster,
4th edition, page no. 602)
Hyperviscosity is seen in
• Multiple myeloma
• Lymphoplasmacytic lymphoma (Waldenstrom’s macroglobulinemia)
• Cryoglobulinemia
• Myeloproliferative disorders
Monoclonal gammopathy of uncertain significance (MGUS): Here, M Protein can be identified in the serum of 1% of
healthy individual >50 years of age and 3% in older than 70 years. It is the most common form of monoclonal gammopathy.
In MGUS, less than 3g/dL of monoclonal protein is present in serum and there is no Bence Jones proteinuria.
98. Answer: D (Ref. Robbins and Cotran Pathologic Basis of Disease, By Vinay Kumar, Abul K. Abbas, Jon C. Aster,
9th edition, page no. 486)
99. Answer: C (Ref. “Previous question”)
100. Answer: C (Ref. Robbins and Cotran Pathologic Basis of Disease, By Vinay Kumar, Abul K. Abbas, Jon C. Aster,
9th edition, page no. 109)
Hyperhomocysteinemia is a mixed disorder (inherited as well as acquired) which can cause both venous and arterial
thrombosis.
101. Answer: B (Ref. Robbins and Cotran Pathologic Basis of Disease, By Vinay Kumar, Abul K. Abbas, Jon C. Aster,
9th edition, page no. 402)
102. Answer: B (Ref. Robbins and Cotran Pathologic Basis of Disease, By Vinay Kumar, Abul K. Abbas, Jon C. Aster,
9th edition, page no. 319)
103. Answer: A (Ref. “Previous question”)
104. Answer: A (Ref. “Previous question”)
105. Answer: C (Ref. Robbins and Cotran Pathologic Basis of Disease, By Vinay Kumar, Abul K. Abbas, Jon C. Aster,
9th edition, page no. 101)
106. Answer: C (Ref. Robbins and Cotran Pathologic Basis of Disease, By Vinay Kumar, Abul K. Abbas, Jon C. Aster,
9th edition, page no. 114)
107. Answer: A (Ref. “Previous question”)
Due to the decrease in TPR from vasodilatation (histamine, bradykinin, nitric oxide, release of anaphylatoxins).
108. Answer: A (Ref. “Previous question”)
109. Answer: C (Ref. “Previous question”)
110. Answer: B (Ref. “Previous question”)
In cirrhosis, there are two alterations in Starling’s forces—an increase in hydrostatic pressure.
111. Answer: D (Ref. “Previous question”)
112. Answer: A (Ref. “Previous question”)
113. Answer: D (Ref. Robbins and Cotran Pathologic Basis of Disease, By Vinay Kumar, Abul K. Abbas, Jon C. Aster,
9th edition, page no. 119)
114. Answer: A (Ref. “Previous question”)
115. Answer: A (Ref. Robbins and Cotran Atlas of Pathology E-Book, By Edward C. Klatt, 3rd edition, page no. 361)
116. Answer: B (Ref. Robbins and Cotran Pathologic Basis of Disease, By Vinay Kumar, Abul K. Abbas, Jon C. Aster,
9th edition, page no. 78)
117. Answer: D

NEOPLASIA
1. Answer: B (Ref. Robbins Basic Pathology, By Vinay Kumar, Abul K. Abbas, Jon C. Aster, 10th edition, page no. 190)
2. Answer: C (Ref. Robbins Basic Pathology, By Vinay Kumar, Abul K. Abbas, Jon C. Aster, 10th edition, page no. 50)
3. Answer: D (Ref. Rubin’s Pathology: Clinicopathologic Foundations of Medicine, Edited by Raphael Rubin, David S.
Strayer, Emanuel Rubin, Jay M. McDonald, 5th edition, page no. 1267)
4. Answer: B (Ref. “previous question”)
5. Answer: C (Ref. “previous question”)

https://t.me/DentalBooksWorld
Chapter 3 • General Pathology 215

6. Answer: A (Ref. Robbins Basic Pathology, By Vinay Kumar, Abul K. Abbas, Jon C. Aster, 10th edition, page no. 190)
7. Answer: A (Ref. Robbins Basic Pathology, By Vinay Kumar, Abul K. Abbas, Jon C. Aster, 10th edition, page no. 48)
8. Answer: C (Ref. Robbins Basic Pathology, By Vinay Kumar, Abul K. Abbas, Jon C. Aster, 10th edition, page no. 49)
9. Answer: B (Ref. Robbins and Cotran Pathologic Basis of Disease, By Vinay Kumar, Abul K. Abbas, Jon C. Aster,
9th edition, page no. 597)
10. Answer: B
11. Answer: A
12. Answer: D (Ref. Robbins Basic Pathology, By Vinay Kumar, Abul K. Abbas, Jon C. Aster, 10th edition, page no. 204)
13. Answer: D (Ref. Robbins Basic Pathology, By Vinay Kumar, Abul K. Abbas, Jon C. Aster, 10th edition, page no. 181)
14. Answer: B (Ref. Robbins Basic Pathology, By Vinay Kumar, Abul K. Abbas, Jon C. Aster, 10th edition, page no. 235)
15. Answer: B (Ref. Robbins Basic Pathology, By Vinay Kumar, Abul K. Abbas, Jon C. Aster, 10th edition, page no. 235)
16. Answer: B (Ref. Robbins and Cotran Atlas of Pathology, By Edward C. Klatt, 3rd edition, page no. 422)
17. Answer: A (Ref. Robbins and Cotran Atlas of Pathology, By Edward C. Klatt, 3rd edition, page no. 422)
18. Answer: D (Ref. Robbins and Cotran Atlas of Pathology, By Edward C. Klatt, 3rd edition, page no. 145)
19. Answer: A (Ref. “previous question”)
20. Answer: A (Ref. Robbins and Cotran Atlas of Pathology, By Edward C. Klatt, 3rd edition, page no. 414)
21. Answer: B (Ref. Robbins Basic Pathology, By Vinay Kumar, Abul K. Abbas, Jon C. Aster, 10th edition, page no. 830)
22. Answer: D (Ref. Robbins and Cotran Atlas of Pathology, By Edward C. Klatt, 3rd edition, page no. 214)
23. Answer: D (Ref. Robbins Basic Pathology, By Vinay Kumar, Abul K. Abbas, Jon C. Aster, 10th edition, page no. 233)
24. Answer: D (Ref. Robbins Basic Pathology, By Vinay Kumar, Abul K. Abbas, Jon C. Aster, 10th edition, page no. 224)
25. Answer: B (Ref. Robbins and Cotran Atlas of Pathology, By Edward C. Klatt, 3rd edition, page no. 333)
26. Answer: A (Ref. Robbins and Cotran Atlas of Pathology, By Edward C. Klatt, 3rd edition, page no. 316)
27. Answer: A (Ref. Robbins Basic Pathology, By Vinay Kumar, Abul K. Abbas, Jon C. Aster, 10th edition, page no. 199)
28. Answer: C
29. Answer: C (Ref. Robbins Basic Pathology, By Vinay Kumar, Abul K. Abbas, Jon C. Aster, 10th edition, page no. 194)
30. Answer: D (Ref. Robbins and Cotran Atlas of Pathology, By Edward C. Klatt, 3rd edition, page no. 77)
31. Answer: C (Ref. Robbins Basic Pathology, By Vinay Kumar, Abul K. Abbas, Jon C. Aster, 10th edition, page no. 193)
32. Answer: D (Ref. Robbins Basic Pathology, By Vinay Kumar, Abul K. Abbas, Jon C. Aster, 10th edition, page no. 236)
33. Answer: A (Ref. Robbins and Cotran Atlas of Pathology, By Edward C. Klatt, 3rd edition, page no. 464)
34. Answer: A (Ref. Robbins Basic Pathology, By Vinay Kumar, Abul K. Abbas, Jon C. Aster, 9th edition, page no. 417)
35. Answer: D (Ref. Robbins Basic Pathology, By Vinay Kumar, Abul K. Abbas, Jon C. Aster, 10th edition, page no. 195)
36. Answer: C (Ref. Robbins and Cotran Atlas of Pathology, By Edward C. Klatt, 3rd edition, page no. 396)
37. Answer: C (Ref. Robbins and Cotran Atlas of Pathology, By Edward C. Klatt, 3rd edition, page no. 62)
38. Answer: A (Ref. Robbins Basic Pathology, By Vinay Kumar, Abul K. Abbas, Jon C. Aster, 10th edition, page no. 867)
39. Answer: C (Ref. “previous question”)
40. Answer: C (Ref. “previous question”)
41. Answer: C (Ref. “previous question”)
42. Answer: A (Ref. Essential Pathology For Dental Students, By Harsh Mohan, Sugandha Mohan, 4th edition, page no. 256)
43. Answer: A (Ref. Essential Pathology For Dental Students, By Harsh Mohan, Sugandha Mohan, 4th edition, page no. 35)
44. Answer: A (Ref. “previous question”)
45. Answer: A (Ref. Robbins Basic Pathology, By Vinay Kumar, Abul K. Abbas, Jon C. Aster, 10th edition, page no. 323)
46. Answer: D (Ref. “previous question”)
47. Answer: C (Ref. Robbins Basic Pathology, By Vinay Kumar, Abul K. Abbas, Jon C. Aster, 10th edition, page no. 202)
48. Answer: D (Ref. Robbins and Cotran Review of Pathology, By Edward C. Klatt, Vinay Kumar, 4th edition, page no. 418)
49. Answer: D (Ref. Robbins and Cotran Review of Pathology, By Edward C. Klatt, Vinay Kumar, 4th edition, page no. 134)
50. Answer: D (Ref. Robbins and Cotran Atlas of Pathology, By Edward C. Klatt, 3rd edition, page no. 343)
51. Answer: B (Ref. Robbins Basic Pathology, By Vinay Kumar, Abul K. Abbas, Jon C. Aster, 10th edition, page no. 53)
52. Answer: A (Ref. Robbins Basic Pathology, By Vinay Kumar, Abul K. Abbas, Jon C. Aster, 10th edition, page no. 268)
53. Answer: D (Ref. Robbins and Cotran Atlas of Pathology, By Edward C. Klatt, 3rd edition, page no. 386)
54. Answer: A (Ref. Robbins Basic Pathology, By Vinay Kumar, Abul K. Abbas, Jon C. Aster, 10th edition, page no. 474)
55. Answer: D

https://t.me/DentalBooksWorld
216 Triumph's Complete Review of Dentistry

56. Answer: A (Ref. Robbins Basic Pathology, By Vinay Kumar, Abul K. Abbas, Jon C. Aster, 10th edition, page no. 886)
57. Answer: C (Ref. Robbins Basic Pathology, By Vinay Kumar, Abul K. Abbas, Jon C. Aster, 10th edition, page no. 833)
58. Answer: A (Ref. “previous question”)
59. Answer: D (Ref. Robbins Basic Pathology, By Vinay Kumar, Abul K. Abbas, Jon C. Aster, 10th edition, page no. 293)
60. Answer: C (Ref. Robbins Basic Pathology, By Vinay Kumar, Abul K. Abbas, Jon C. Aster, 10th edition, page no. 209)
61. Answer: D (Ref. Robbins Basic Pathology, By Vinay Kumar, Abul K. Abbas, Jon C. Aster, 10th edition, page no. 833)
62. Answer: D (Ref. “previous question”)
63. Answer: B (Ref. Robbins Basic Pathology, By Vinay Kumar, Abul K. Abbas, Jon C. Aster, 10th edition, page no. 921)
64. Answer: A (Ref. “previous question”)
65. Answer: B (Ref. Robbins Basic Pathology, By Vinay Kumar, Abul K. Abbas, Jon C. Aster, 10th edition, page no. 220)
66. Answer: D (Ref. “previous question”)
67. Answer: C (Ref. Robbins and Cotran Review of Pathology, By Edward C. Klatt, Vinay Kumar, 4th edition, page no. 395)
68. Answer: B (Ref. Robbins and Cotran Atlas of Pathology, By Edward C. Klatt, 3rd edition, page no. 395)
69. Answer: B (Ref. Robbins Basic Pathology, By Vinay Kumar, Abul K. Abbas, Jon C. Aster, 10th edition, page no. 199)
70. Answer: A (Ref. Robbins Basic Pathology, By Vinay Kumar, Abul K. Abbas, Jon C. Aster, 10th edition, page no. 786)
Small cell carcinoma of lung most commonly metastasize to the brain. It accounts for about 40% of brain metastases.
71. Answer: C (Ref. Robbins Basic Pathology, By Vinay Kumar, Abul K. Abbas, Jon C. Aster, 10th edition, page no. 474)
• Chemotherapeutic drugs can cause both necrosis and apoptosis, but it is apoptosis which is the basis of action of
chemotherapeutic drugs.
• Anoikis refers to death of epithelial cells after removal from the normal milieu of substrate, particularly from cell to cell
contact.
72. Answer: B (Ref. Robbins Basic Pathology, By Vinay Kumar, Abul K. Abbas, Jon C. Aster, 10th edition, page no. 904)
73. Answer: D (Ref. Robbins Basic Pathology, By Vinay Kumar, Abul K. Abbas, Jon C. Aster, 10th edition, page no. 231)
74. Answer: B (Ref. Robbins Basic Pathology, By Vinay Kumar, Abul K. Abbas, Jon C. Aster, 10th edition, page no. 669)
75. Answer: C (Ref. Robbins Basic Pathology, By Vinay Kumar, Abul K. Abbas, Jon C. Aster, 10th edition, page no. 595)
• Dysplasia is the loss of uniformity of individual cells as well as their architectural orientation.
• Carcinoma in situ (dysplastic changes are marked but lesion remains confined to normal tissue: pre-invasive neoplasm).
Basement membrane is intact.
• Anaplasia is complete lack of differentiation of cells both morphologically and functionally (Invasive Ca).
76. Answer: B (Ref. “previous question”)
77. Answer: C (Ref. Robbins and Cotran Atlas of Pathology, By Edward C. Klatt, 3rd edition, page no.89)
78. Answer: B (Ref. Robbins Basic Pathology, By Vinay Kumar, Abul K. Abbas, Jon C. Aster, 10th edition, page no. 886)
79. Answer: D (Ref. “previous question”)
80. Answer: B (Ref. “previous question”)
81. Answer: D (Ref. “previous question”)
82. Answer: B (Ref. Robbins and Cotran Atlas of Pathology, By Edward C. Klatt, 3rd edition, page no. 405)
• Pheochromocytomas, and their related counterparts in extra-adrenal sites called paragangliomas, are notorious because
the only reliable indicator of metastatic potential is the presence of distant metastases. Very malignant-appearing tumors
may not metastasize and benign-appearing tumors may produce metastases. These tumors should all be considered
“potentially malignant.”
83. Answer: B (Ref. “previous question”)
84. Answer: A (Ref. “previous question”)
85. Answer: B (Ref. Robbins Basic Pathology, By Vinay Kumar, Abul K. Abbas, Jon C. Aster, 10th edition, page no. 24)
86. Answer: A (Ref. Robbins Basic Pathology, By Vinay Kumar, Abul K. Abbas, Jon C. Aster, 10th edition, page no. 623)
87. Answer: C (Ref. Robbins Basic Pathology, By Vinay Kumar, Abul K. Abbas, Jon C. Aster, 10th edition, page no. 742)
88. Answer: D (Ref. “previous question”)
89. Answer: A (Ref. Robbins Basic Pathology, By Vinay Kumar, Abul K. Abbas, Jon C. Aster, 10th edition, page no. 232)
90. Answer: C (Ref. Robbins Basic Pathology, By Vinay Kumar, Abul K. Abbas, Jon C. Aster, 10th edition, page no. 211)
91. Answer: A (Ref. Robbins Basic Pathology, By Vinay Kumar, Abul K. Abbas, Jon C. Aster, 10th edition, page no. 269)
92. Answer: C (Ref. Robbins Basic Pathology, By Vinay Kumar, Abul K. Abbas, Jon C. Aster, 10th edition, page no. 25)
93. Answer: A (Ref. Robbins and Cotran Atlas of Pathology, By Edward C. Klatt, 3rd edition, Page no. 191)
94. Answer: B (Ref. Robbins Basic Pathology, By Vinay Kumar, Abul K. Abbas, Jon C. Aster, 10th edition, page no. 212)

https://t.me/DentalBooksWorld
Chapter 3 • General Pathology 217

95. Answer: C (Ref. “previous question”)


96. Answer: B (Ref. “previous question”)
97. Answer: B (Ref. Robbins Basic Pathology, By Vinay Kumar, Abul K. Abbas, Jon C. Aster, 10th edition, page no. 227)
98. Answer: A (Ref. Robbins and Cotran Review of Pathology, By Edward C. Klatt, Vinay Kumar, 4th edition, page no. 365)
99. Answer: A (Ref. “previous question”)
• Fanconi’s anemia: Autosomal recessive disease associated with BRCA gene
Characteristics: Progressive pancytopenia, increased risk of malignancy (solid tumors and AML) and congenital
developmental anomalies like short stature, café au lait spots, abnormalities affecting thumb, radius and genitourinary
tract.
100. Answer: C (Ref. Robbins and Cotran Review of Pathology, By Edward C. Klatt, Vinay Kumar, 4th edition, page no. 373)
• SIS oncogene is one of a growth factor oncogene.
• Its overexpression is seen in cancers like astrocytoma and osteosarcoma
• A number of nuclear transcription factors are the products of oncogenes like myc, fos, jun, myb, and rel
• Myc is most commonly involved in tumors
101. Answer: D (Ref. Robbins Basic Pathology, By Vinay Kumar, Abul K. Abbas, Jon C. Aster, 10th edition, page no. 209)
Tumor suppressor genes are the genes whose products downregulate the cell cycle, and thus apply brakes to cellular
proliferation.
• Rb gene is a tumor suppressor gene whereas Myc, fos, and ras are all examples of proto-oncogenes.
102. Answer: D (Ref. Robbins Basic Pathology, By Vinay Kumar, Abul K. Abbas, Jon C. Aster, 10th edition, page no. 206)
103. Answer: A (Ref. “previous question”)
• p53 is a tumor suppressor gene, located on chromosome 17. It is also known as “Guardian of the genome.”
• At the time of DNA injury following irradiation, its level increases and it acts to cause cell cycle arrest (G1/S).
• The cell cycle arrest is to allow time for DNA repair. If repair is unsuccessful, p53 causes apoptosis of the cell by activating
bax (apoptosis inducing gene). So, any exposure to UV irradiation would cause increased activity of p53 gene resulting
in apoptosis and cell death.
104. Answer: C (Ref. “previous question”)
105. Answer: A (Ref. Robbins Basic Pathology, By Vinay Kumar, Abul K. Abbas, Jon C. Aster, 10th edition, page no. 202)
Genes that promote autonomous cell growth in cancer cells are called oncogenes and their normal cellular counterparts
are called proto-oncogenes.
• Proto-oncogenes are important for cellular function related to growth and proliferation.
• Proto-oncogenes may be converted into cellular oncogenes that are involved in oncogenesis.
• C-myc proto-oncogenes is expressed in virtually all eukaryotic cells and its persistent expression or overexpression is
commonly found in tumors. Dysregulation of c-myc expression resulting from translocation of gene occurs in Burkitt’s
lymphoma.
106. Answer: C (Ref. “previous question”)
• Viral oncogenes (V-onc) are commonly known as “cancer genes” which encode proteins triggering transformation of
normal cells into cancer cells.
• Proto-oncogenes are the normal cellular genes that promote normal growth and differentiation.
• Oncogenes isolated from cancer cells are called cellular oncogenes (C-onc).
• Proto-oncogenes are converted to oncogenes and cause cancer by:
− Transduction into retrovirus (V-oncs) or
− Changes in situ that affect their expression and function thereby converting them into cellular oncogene (Concs).
• The transduction of oncogenes by the virus (e.g. retrovirus) is through recombination with DNA of a (normal) host
cell that had been infected by the virus. Thus, they are of host cell origin. The virus acts as a transducing agent, carrying
oncogenes from one cell to another.
107. Answer: A (Ref. “previous question”)
108. Answer: C (Ref. Robbins Basic Pathology, By Vinay Kumar, Abul K. Abbas, Jon C. Aster, 10th edition, page no. 227)
109. Answer: B (Ref. Robbins Basic Pathology, By Vinay Kumar, Abul K. Abbas, Jon C. Aster, 10th edition, page no. 206)
110. Answer: D (Ref. Robbins and Cotran Review of Pathology, By Edward C. Klatt, Vinay Kumar, 4th edition, page no. 80)
111. Answer: A (Ref. Robbins Basic Pathology, By Vinay Kumar, Abul K. Abbas, Jon C. Aster, 10th edition, page no. 220)
112. Answer: A (Ref. Robbins Basic Pathology, By Vinay Kumar, Abul K. Abbas, Jon C. Aster, 10th edition, page no. 214)

https://t.me/DentalBooksWorld
218 Triumph's Complete Review of Dentistry

113. Answer: C (Ref. “previous question”)


The cell cycle has its own internal controls, called checkpoints. There are two main checkpoints: one at the G1/S transition
and another at G2/M.
• In the G1/S checkpoint, cell-cycle arrest is mostly mediated through p53, which induces the cell-cycle inhibitor p21.
• Arrest of the cell cycle by the G2/M checkpoint involves both p53-dependent (via cyclin A/cdK-2) and independent
(via cdc 25) mechanisms.
114. Answer: D (Ref. “previous question”)
MYC gene is associated with “Conflict model” in carcinogenesis.
MYC is most commonly involved in human tumors.
Tumors associated with different subtypes of MYC.
C-MYC → Translocation → Burkitt’s lymphoma
N-MYC → Amplification → Neuroblastoma, small-cell carcinoma of lung
L-MYC → Amplification → Small-cell carcinoma of lung
115. Answer: C (Ref. “previous question”)

Defects in DNA repair

Nucleotide excision repair Mismatch repair Recombination repair

• Hereditary Non polyposis • Bloom syndrome


• Xeroderma
colorectal carcinoma • Ataxia telangiectasia
pigmentosum
syndrome • Fanconi’s anemia

116. Answer: A
Note: Beta-2 macroglobulin is not a tumor marker but Beta-2 microglobulin (B2M) is a tumor marker
• Beta-2 microglobulin (B2M) is a protein that is found on the surface of almost all cells in the body and is shed by cells
into the blood, particularly by B lymphocytes and tumor cells. It is present in most body fluids and its level rises with
conditions that increase cell production and/or destruction, or that activate the immune system. This test measures
B2M in the blood, urine, or rarely in the cerebrospinal fluid (CSF).
• Increased levels of B2M in the blood and/or urine indicate that there is a problem, but they are not diagnostic of a specific
disease or condition. They do, however, reflect disease activity and the amount of cancer present. When someone has
been diagnosed with multiple myeloma or lymphoma, that person is likely to have a poorer prognosis if the B2M level is
significantly elevated.
117. Answer: B (Ref. “previous question”)
Lymphoid Malignancy of Epstein–Barr virus
• Burkitt’s lymphoma
• Postorgan transplant lymphoma
• Primary CNS diffuse large B cell lymphoma
• Hodgkin’s disease
• Extranodal NK/T cell lymphoma, nasal type
LMP-1 gene plays a role in oncogenesis induced by EBV
118. Answer: A (Ref. “previous question”)
One of the critical events required for metastasis is the growth of a new network of blood vessels, called tumor
angiogenesis.
• Without vascularization or angiogenesis, the tumor can grow only 1–2 mm. Vessels are also required for nutrition.
• Vascularization is promoted by VEGF and bFGF and inhibited by angiostatin, endostatin, and tumstatin.
• It has been found that angiogenesis inhibitors would therefore prevent the growth of metastases.
119. Answer: B (Ref. Robbins and Cotran Atlas of Pathology, By Edward C. Klatt, 3rd edition, page no. 574)
120. Answer: D (Ref. Robbins and Cotran Atlas of Pathology, By Edward C. Klatt, 3rd edition, page no. 483)
121. Answer: D (Ref. Robbins Basic Pathology, By Vinay Kumar, Abul K. Abbas, Jon C. Aster, 10th edition, page no. 371)

https://t.me/DentalBooksWorld
Chapter 3 • General Pathology 219

122. Answer: C (Ref. “previous question”)


Most radiosensitive organ sites in children in order of sensitivity are the thyroid gland, breasts, bone marrow, brain, and
skin.
123. Answer: B (Ref. Robbins Basic Pathology, By Vinay Kumar, Abul K. Abbas, Jon C. Aster, 10th edition, page no. 231)
“The carcinogenicity of UV-B light is attributed to its formation of pyrimidine dimers in DNA.” This type of DNA
damage is repaired by the nucleotide excision repair pathway. The importance of the nucleotide excision repair pathway
of DNA repair is illustrated by the high frequency of cancers in individuals with the hereditary disorder xeroderma
pigmentosum.
124. Answer: B (Ref. “previous question”)
• The most radiosensitive cell in the blood is the lymphocytes.
• The least radiosensitive cell in the blood is the platelets.
• DNA is the most sensitive intracellular organelle to radiation.
125. Answer: A (Ref. Robbins Basic Pathology, By Vinay Kumar, Abul K. Abbas, Jon C. Aster, 10th edition, page no. 467)
Hallmark of follicular lymphoma is a (14; 18) translocation, which leads to the juxtaposition of the IgH locus on
chromosome 14 and BCL 2 locus on chromosome 18.
126. Answer: B (Ref. “previous question”)
127. Answer: C (Ref. “previous question”)
128. Answer: B (Ref. “previous question”)
129. Answer: D (Ref. “previous question”)
130. Answer: C (Ref. Robbins and Cotran Atlas of Pathology, By Edward C. Klatt, 3rd edition, page no. 454)
131. Answer: A (Ref. “previous question”)
132. Answer: C (Ref. Robbins and Cotran Atlas of Pathology, By Edward C. Klatt, 3rd edition, page no. 158)
133. Answer: B (Ref. “previous question”)
134. Answer: B (Ref. Robbins Basic Pathology, By Vinay Kumar, Abul K. Abbas, Jon C. Aster, 9th edition, page no. 999)
135. Answer: C (Ref. Robbins Basic Pathology, By Vinay Kumar, Abul K. Abbas, Jon C. Aster, 10th edition, page no. 230)
136. Answer: D (Ref. Robbins Basic Pathology, By Vinay Kumar, Abul K. Abbas, Jon C. Aster, 10th edition, page no. 231)

BLOOD AND LYMPHATICS


1. Answer: B (Ref. Robbins and Cotran Pathologic Basis of Disease, By Vinay Kumar, Abul K. Abbas, Jon C. Aster,
9th edition, page no. 582)
2. Answer: D (Ref. Robbins and Cotran Pathologic Basis of Disease, By Vinay Kumar, Abul K. Abbas, Jon C. Aster,
9th edition, page no. 662)
Von Willebrand Disease was discovered by Erik Adolf von Willebrand. It is a multimeric protein that is required for
platelet adhesion.
The three types of hereditary vWD are vWD type 1 (most common with severe symptoms), vWD type 2 (second most
common type of the disorder and has mild to moderate symptoms), and vWD type 3.
Conditions that lead to reduced production of platelets in the bone marrow
• Leukemia and other cancers that can replace the bone marrow, so fewer platelets are made
• Some types of a blood condition called anemia
• Infections with viruses, including HIV and hepatitis C
• Sepsis
• Acute respiratory distress syndrome
3. Answer: D (Ref. Robbins and Cotran Pathologic Basis of Disease, By Vinay Kumar, Abul K. Abbas, Jon C. Aster,
9th edition, page no. 663)
4. Answer: D (Ref. Robbins and Cotran Pathologic Basis of Disease, By Vinay Kumar, Abul K. Abbas, Jon C. Aster,
9th edition, page nos. 95 and 660)
5. Answer: A (Ref. Robbins and Cotran Pathologic Basis of Disease, By Vinay Kumar, Abul K. Abbas, Jon C. Aster,
9th edition, page no. 635)

https://t.me/DentalBooksWorld
220 Triumph's Complete Review of Dentistry

Normal Sickle

Continuation of
Amino Acids

The chain of colored boxes represent the first eight amino acids in the beta chain of hemoglobin. The sixth position in
the normal beta chain has glutamic acid, while sickle beta chain has valine.
The mutation causing sickle cell anemia is a single nucleotide substitution (A to T) in the codon for amino acid 6. The
mutation converts a glutamic acid codon (GAG) to a valine codon (GTG). The form of hemoglobin in persons with sickle
cell anemia is referred to as HbS.
6. Answer: D (Ref. Robbins and Cotran Pathologic Basis of Disease, By Vinay Kumar, Abul K. Abbas, Jon C. Aster,
9th edition, page no. 663)
7. Answer: C (Ref. Robbins and Cotran Pathologic Basis of Disease, By Vinay Kumar, Abul K. Abbas, Jon C. Aster,
9th edition, page no. 645)
8. Answer: A (Ref. Robbins and Cotran Pathologic Basis of Disease, By Vinay Kumar, Abul K. Abbas, Jon C. Aster,
9th edition, page no. 613)
Acute myeloid leukemia (AML) starts in the bone marrow of the spongy tissue inside the bones where blood cells are
produced. The disease prevents immature blood cells from growing into healthy blood cells.
Usually acute leukemias are common in children and young age groups.
ALL is primarily a disease, of children and young adults, whereas AML occurs at all ages and increases with the age.
Syndromes associated with an increased incidence of AML: syndromes with somatic cell chromosome aneuploidy,
e.g., Down (chromosome 21 trisomy), Klinefelter (XXY and variants), and Patau (chromosome 13 trisomy).
Chronic leukemia are found around and above 40 years. CML is found in middle age and CLL tends to occur in the elderly.
NOTE:
Lymphadenopathy is characteristic of both ALL and CLL.
Lymphadenopathy is mild in acute myeloid leukemia but may or may not be present in chronic myeloid leukemia.
Splenomegaly of moderate grade is seen in acute leukemia, while massive splenomegaly is seen in chronic leukemia.
In acute leukemias white cell count exceeds 1 lakh per mm3, whereas in chronic leukemias white cell count may exceed
more than 2 lakhs/mm3.
Gum hypertrophy occurs commonly in acute monocytic (monocytes) and acute myelomonocytic (granulocytes)
leukemia (FAB—M4 type).
9. Answer: D (Ref. Robbins and Cotran Pathologic Basis of Disease, By Vinay Kumar, Abul K. Abbas, Jon C. Aster,
9th edition, page no. 648)
Pernicious anemia is due to the deficiency of intrinsic factor of Castle. Extrinsic factor is B12. All pernicious anemia are
megaloblastic anemias. But all megaloblastic anemias are not pernicious.
10. Answer: D (Ref. Robbins and Cotran Pathologic Basis of Disease, By Vinay Kumar, Abul K. Abbas, Jon C. Aster,
9th edition, page no. 650)
11. Answer: C (Ref. Robbins and Cotran Pathologic Basis of Disease, By Vinay Kumar, Abul K. Abbas, Jon C. Aster,
9th edition, page no. 653)
Low levels of hemoglobin is due to hypochromic and microcytic anemia and massive hemolytic anemia (petechiae and
spontaneous hemorrhage) causes hepatosplenomegaly, leg ulcers, gallstones, and high-output congestive heart failure
(dyspnea on exertion).

https://t.me/DentalBooksWorld
Chapter 3 • General Pathology 221

12. Answer: D (Ref. Robbins and Cotran Review of Pathology, By Edward C. Klatt, Vinay Kumar, 4th edition, page no. 213)
13. Answer: D (Ref. Robbins and Cotran Review of Pathology, By Edward C. Klatt, Vinay Kumar, 4th edition, page no. 198)
14. Answer: B (Ref. Robbins and Cotran Pathologic Basis of Disease, By Vinay Kumar, Abul K. Abbas, Jon C. Aster,
9th edition, page no. 634)
Glucose-6-phosphate dehydrogenase deficiency causes red blood cells to break down prematurely (hemolysis) -> cause
hemolytic anemia -> which can lead to symptoms of paleness, yellowing of the skin and whites of the eyes (jaundice), dark
urine, fatigue, shortness of breath, and a rapid heart rate.
15. Answer: B (Ref. “previous question”)
16. Answer: B (Ref. Robbins and Cotran Pathologic Basis of Disease, By Vinay Kumar, Abul K. Abbas, Jon C. Aster,
9th edition, page no. 611)
17. Answer: B (Ref. “previous question”)
18. Answer: D (Ref. “previous question”)
19. Answer: A (Ref. Robbins and Cotran Pathologic Basis of Disease, By Vinay Kumar, Abul K. Abbas, Jon C. Aster,
9th edition, page no. 455)
20. Answer: B (Ref. Robbins and Cotran Pathologic Basis of Disease, By Vinay Kumar, Abul K. Abbas, Jon C. Aster,
9th edition, page no. 220)
21. Answer: B (Ref. “previous question”)
22. Answer: A (Ref. Robbins and Cotran Pathologic Basis of Disease, By Vinay Kumar, Abul K. Abbas, Jon C. Aster,
9th edition, page no. 448)
23. Answer: A
24. Answer: D (Ref. Robbins Basic Pathology, By Vinay Kumar, Abul K. Abbas, Jon C. Aster, 10th edition, page no. 465)
25. Answer: A (Ref. “previous question”)
26. Answer: D (Ref. Robbins Basic Pathology, By Vinay Kumar, Abul K. Abbas, Jon C. Aster, 10th edition, page no. 442)
27. Answer: B (Ref. Robbins Basic Pathology, By Vinay Kumar, Abul K. Abbas, Jon C. Aster, 10th edition, page no. 472)
28. Answer: D (Ref. “previous question”)
29. Answer: C (Ref. “previous question”)
30. Answer: B (Ref. Robbins Basic Pathology, By Vinay Kumar, Abul K. Abbas, Jon C. Aster, 10th edition, page no. 227)
31. Answer: B (Ref. “previous question”)
32. Answer: C (Ref. “previous question”)
33. Answer: D (Ref. “previous question”)
34. Answer: A (Ref. “previous question”)
35. Answer: D
36. Answer: B
37. Answer: A (Ref. “previous question”)
38. Answer: A (Ref. Robbins Basic Pathology, By Vinay Kumar, Abul K. Abbas, Jon C. Aster, 10th edition, page no. 459)
39. Answer: B (Ref. Robbins Basic Pathology, By Vinay Kumar, Abul K. Abbas, Jon C. Aster, 10th edition, page no. 456)
40. Answer: C (Ref. “previous question”)
41. Answer: B (Ref. “previous question”)
42. Answer: A (Ref. Hematology, By Susan Cotter, page no. 65)
43. Answer: C (Ref. “previous question”)
44. Answer: B (Ref. Robbins and Cotran Pathologic Basis of Disease, By Vinay Kumar, Abul K. Abbas, Jon C. Aster,
9th edition, page no. 226)
45. Answer: A (Ref. Robbins and Cotran Pathologic Basis of Disease, By Vinay Kumar, Abul K. Abbas, Jon C. Aster,
9th edition, page no. 276)
46. Answer: D (Ref. “previous question”)
47. Answer: B (Ref. “previous question”)
48. Answer: D (Ref. “previous question”)
49. Answer: D (Ref. “previous question”)
50. Answer: A (Ref. “previous question”)
51. Answer: B (Ref. Robbins Basic Pathology, By Vinay Kumar, Abul K. Abbas, Jon C. Aster, 10th edition, page no. 284)
52. Answer: D (Ref. “previous question”)

https://t.me/DentalBooksWorld
222 Triumph's Complete Review of Dentistry

53. Answer: B (Ref. Robbins and Cotran Pathologic Basis of Disease, By Vinay Kumar, Abul K. Abbas, Jon C. Aster,
9th edition, page no. 221)
54. Answer: B (Ref. “previous question”)
55. Answer: C (Ref. Robbins Basic Pathology, By Vinay Kumar, Abul K. Abbas, Jon C. Aster, 10th edition, page no. 108)
56. Answer: C (Ref. “previous question”)
57. Answer: C (Ref. Robbins and Cotran Pathologic Basis of Disease, By Vinay Kumar, Abul K. Abbas, Jon C. Aster,
9th edition, page no. 1191)
58. Answer: (Ref. A Robbins and Cotran Atlas of Pathology, By Edward C. Klatt, 3rd edition, page no. 38)
59. Answer: A (Ref. Robbins and Cotran Pathologic Basis of Disease, By Vinay Kumar, Abul K. Abbas, Jon C. Aster,
9th edition, page no. 262)
60. Answer: A (Ref. “previous question”)
61. Answer: C (Ref. Robbins Basic Pathology, By Vinay Kumar, Abul K. Abbas, Jon C. Aster, 10th edition, page no. 924)
62. Answer: B (Ref. Robbins Basic Pathology, By Vinay Kumar, Abul K. Abbas, Jon C. Aster, 9th edition, page no. 414)
63. Answer: B (Ref. Robbins and Cotran Pathologic Basis of Disease, By Vinay Kumar, Abul K. Abbas, Jon C. Aster,
9th edition, page no. 613)
64. Answer: A (Ref. Robbins Basic Pathology, By Vinay Kumar, Abul K. Abbas, Jon C. Aster, 10th edition, page no. 444)
65. Answer: C (Ref. “previous question”)
66. Answer: A (Ref. Robbins Basic Pathology, By Vinay Kumar, Abul K. Abbas, Jon C. Aster, 10th edition, page no. 101)
67. Answer: D (Ref. “previous question”)
68. Answer: D (Ref. Robbins Basic Pathology, By Vinay Kumar, Abul K. Abbas, Jon C. Aster, 10th edition, page no. 488)
69. Answer: D (Ref. Robbins Basic Pathology, By Vinay Kumar, Abul K. Abbas, Jon C. Aster, 10th edition, page no. 455)
70. Answer: C (Ref. Robbins Basic Pathology, By Vinay Kumar, Abul K. Abbas, Jon C. Aster, 10th edition, page no. 459)
71. Answer: B (Ref. Robbins Basic Pathology, By Vinay Kumar, Abul K. Abbas, Jon C. Aster, 10th edition, page no. 267)
72. Answer: D (Ref. Robbins Basic Pathology, By Vinay Kumar, Abul K. Abbas, Jon C. Aster, 10th edition, page no. 448)
73. Answer: A (Ref. “previous question”)
74. Answer: A (Ref. “previous question”)
75. Answer: C (Ref. Robbins Basic Pathology, By Vinay Kumar, Abul K. Abbas, Jon C. Aster, 10th edition, page no. 450)
76. Answer: D (Ref. Robbins Basic Pathology, By Vinay Kumar, Abul K. Abbas, Jon C. Aster, 10th edition, page no. 451)
77. Answer: A (Ref. Robbins and Cotran Pathologic Basis of Disease, By Vinay Kumar, Abul K. Abbas, Jon C. Aster,
9th edition, page no. 648)
78. Answer: A (Ref. Robbins and Cotran Pathologic Basis of Disease, By Vinay Kumar, Abul K. Abbas, Jon C. Aster,
9th edition, page no. 254)
79. Answer: B (Ref. Oral and Maxillofacial Pathology, By Brad W. Neville, Douglas D. Damm, Angela C. Chi, Carl M. Allen,
4th edition, page no. 912)
80. Answer: D (Ref. “previous question”)
81. Answer: C (Ref. “previous question”)
82. Answer: A (Ref. “previous question”)
83. Answer: C

SYSTEMIC PATHOLOGY AND MISCELLANEOUS


1. Answer: B (Ref. Robbins and Cotran Pathologic Basis of Disease, By Vinay Kumar, Abul K. Abbas, Jon C. Aster,
9th edition, page no. 42)
2. Answer: C (Ref. Robbins and Cotran Pathologic Basis of Disease, By Vinay Kumar, Abul K. Abbas, Jon C. Aster,
9th edition, page no. 38)
3. Answer: C (Ref. Robbins and Cotran Pathologic Basis of Disease, By Vinay Kumar, Abul K. Abbas, Jon C. Aster,
9th edition, page no. 698)
4. Answer: B (Ref. Robbins Basic Pathology, By Vinay Kumar, Abul K. Abbas, Jon C. Aster, 10th edition page no. 850)
5. Answer: B (Ref. Robbins Basic Pathology, By Vinay Kumar, Abul K. Abbas, Jon C. Aster, 10th edition page no. 915)
6. Answer: B (Ref. Robbins Basic Pathology, By Vinay Kumar, Abul K. Abbas, Jon C. Aster, 10th edition, page no. 68)
7. Answer: B
8. Answer: A (Ref. Robbins Basic Pathology, By Vinay Kumar, Abul K. Abbas, Jon C. Aster, 10th edition, page no. 510)

https://t.me/DentalBooksWorld
Chapter 3 • General Pathology 223

9. Answer: B (Ref. Robbins Basic Pathology, By Vinay Kumar, Abul K. Abbas, Jon C. Aster, 10th edition, page no. 291)
10. Answer: A (Ref. Robbins Basic Pathology, By Vinay Kumar, Abul K. Abbas, Jon C. Aster, 10th edition, page no. 128)
11. Answer: B (Ref. Robbins and Cotran Atlas of Pathology, By Edward C. Klatt, 3rd edition, page no. 386)
12. Answer: A (Ref. Robbins and Cotran Atlas of Pathology, By Edward C. Klatt, 3rd edition, page no. 278)
13. Answer: B (Ref. Robbins and Cotran Atlas of Pathology, By Edward C. Klatt, 3rd edition, page no. 84)
14. Answer: C (Ref. Robbins and Cotran Review of Pathology, By Edward C. Klatt, Vinay Kumar, 4th edition, page no. 54)
15. Answer: A (Ref. Robbins Basic Pathology, By Vinay Kumar, Abul K. Abbas, Jon C. Aster, 10th edition, page no. 210)
16. Answer: A (Ref. Robbins and Cotran Atlas of Pathology, By Edward C. Klatt, 3rd edition, page no. 127)
17. Answer: A (Ref. Robbins and Cotran Atlas of Pathology, By Edward C. Klatt, 3rd edition, page no. 1313)
18. Answer: A (Ref. “Previous question”)
19. Answer: B (Ref. Robbins and Cotran Pathologic Basis of Disease, By Vinay Kumar, Abul K. Abbas, Jon C. Aster,
9th edition, page no. 22)
20. Answer: B (Ref. Robbins and Cotran Pathologic Basis of Disease, By Vinay Kumar, Abul K. Abbas, Jon C. Aster,
9th edition, page no. 486)
21. Answer: A (Ref. Robbins and Cotran Pathologic Basis of Disease, By Vinay Kumar, Abul K. Abbas, Jon C. Aster,
9th edition, page no. 227)
22. Answer: A (Ref. Robbins and Cotran Pathologic Basis of Disease, By Vinay Kumar, Abul K. Abbas, Jon C. Aster,
9th edition, page no. 276)
23. Answer: A (Ref. Robbins and Cotran Atlas of Pathology, By Edward C. Klatt, 3rd edition, page no. 337)
24. Answer: B (Ref. Robbins and Cotran Pathologic Basis of Disease, By Vinay Kumar, Abul K. Abbas, Jon C. Aster,
9th edition, page no. 267)
25. Answer: A (Ref. Robbins and Cotran Pathologic Basis of Disease, By Vinay Kumar, Abul K. Abbas, Jon C. Aster,
9th edition, page no. 268)
26. Answer: B (Ref. Robbins and Cotran Pathologic Basis of Disease, By Vinay Kumar, Abul K. Abbas, Jon C. Aster,
9th edition, page no. 454)
27. Answer: A (Ref. Robbins and Cotran Pathologic Basis of Disease, By Vinay Kumar, Abul K. Abbas, Jon C. Aster,
9th edition, page no. 472)
28. Answer: B (Ref. Robbins and Cotran Pathologic Basis of Disease, By Vinay Kumar, Abul K. Abbas, Jon C. Aster,
9th edition, page no. 132)
29. Answer: B (Ref. Robbins and Cotran Atlas of Pathology, By Edward C. Klatt, 3rd edition, page no. 658)
30. Answer: D (Ref. Robbins and Cotran Atlas of Pathology, By Edward C. Klatt, 3rd edition, page no. 163)
31. Answer: B (Ref. Robbins and Cotran Pathologic Basis of Disease, By Vinay Kumar, Abul K. Abbas, Jon C. Aster,
9th edition, page no. 863)
32. Answer: B (Ref. Robbins and Cotran Pathologic Basis of Disease, By Vinay Kumar, Abul K. Abbas, Jon C. Aster,
9th edition, page no. 445)
33. Answer: A (Ref. Robbins and Cotran Pathologic Basis of Disease, By Vinay Kumar, Abul K. Abbas, Jon C. Aster,
9th edition, page no. 170)
34. Answer: A (Ref. Robbins and Cotran Pathologic Basis of Disease, By Vinay Kumar, Abul K. Abbas, Jon C. Aster,
9th edition, page no. 418)
35. Answer: D (Ref. Robbins and Cotran Pathologic Basis of Disease, By Vinay Kumar, Abul K. Abbas, Jon C. Aster,
9th edition, page no. 116)
36. Answer: A (Ref. Robbins and Cotran Pathologic Basis of Disease, By Vinay Kumar, Abul K. Abbas, Jon C. Aster,
9th edition, page no. 344)
37. Answer: B (Ref. Robbins and Cotran Atlas of Pathology, By Edward C. Klatt, 3rd edition, page no. 1198)
38. Answer: A (Ref. Robbins and Cotran Pathologic Basis of Disease, By Vinay Kumar, Abul K. Abbas, Jon C. Aster,
3rd edition, page no. 165)
39. Answer: A (Ref. Robbins and Cotran Pathologic Basis of Disease, By Vinay Kumar, Abul K. Abbas, Jon C. Aster,
3rd edition, page no. 663)
40. Answer: D (Ref. “Previous question”)
41. Answer: D (Ref. Robbins and Cotran Atlas of Pathology, By Edward C. Klatt, 3rd edition, page no. 281)
42. Answer: B (Ref. “Previous question”)
43. Answer: C (Ref. Robbins and Cotran Pathologic Basis of Disease, By Vinay Kumar, Abul K. Abbas, Jon C. Aster,
3rd edition, page no. 845)

https://t.me/DentalBooksWorld
224 Triumph's Complete Review of Dentistry

44. Answer: D (Ref. Robbins and Cotran Review of Pathology, By Edward C. Klatt, Vinay Kumar, 4th edition, page no. 300)
45. Answer: B (Ref. Robbins and Cotran Pathologic Basis of Disease, By Vinay Kumar, Abul K. Abbas, Jon C. Aster,
3rd edition, page no. 508)
46. Answer: C (Ref. Robbins and Cotran Pathologic Basis of Disease, By Vinay Kumar, Abul K. Abbas, Jon C. Aster,
3rd edition, page no. 491)
47. Answer: A (Ref. Robbins and Cotran Pathologic Basis of Disease, By Vinay Kumar, Abul K. Abbas, Jon C. Aster,
3rd edition, page no. 351)
48. Answer: C (Ref. Robbins and Cotran Pathologic Basis of Disease, By Vinay Kumar, Abul K. Abbas, Jon C. Aster,
3rd edition, page no. 468)
49. Answer: C (Ref. Robbins and Cotran Atlas of Pathology, By Edward C. Klatt, 3rd edition, page no. 675)

https://t.me/DentalBooksWorld
4 Oral Pathology and
Oral Medicine

SYNOPSIS

DEVELOPMENTAL DISTURBANCES
1. Anomaly Irregularity
2. Developmental disorder Disturbances occurring during the process of development
3. Congenital disorder A condition present at birth, inherited or caused by the environment, especially
the uterine environment
4. Genotype Genetic constitution of an individual
5. Phenotype Final outcome of combination of genetic and environmental influence
6. Types of inheritance • Autosomal recessive
• Autosomal dominant
• X-linked recessive
• X-linked dominant
• Codominant
7. Homozygous Identical genes (alleles)
8. Heterozygous Nonidentical genes (alleles)
9. X-linked dominant Both males and females can be affected although males may be more severely
affected because they carry only one copy of genes found on the X chromosome.
Some X-linked dominant disorders are lethal in males
10. X-linked recessive X-linked recessive conditions, genetic carrier females will not show symptoms
themselves because for this gene, you only need one working copy and the X
with the working copy is switched on in most or all of their cells
11. Dominant If a trait is expressed with a heterozygous genes
12. Recessive If a trait is expressed with a homozygous genes
13. Polygenic inheritance More number of genes
14. Monogenic inheritance Single-gene disorder
15. Atavism Atavism is the tendency to revert to ancestral type. In biology, an atavism is
an evolutionary throwback, such as traits reappearing that had disappeared
generations before
16. Craniosynostosis Premature closure of skull sutures – Skull and facial asymmetry
17. Hemifacial microsomia Unilateral underdevelopment of face (Goldenhar syndrome, brachial arch
syndrome, facio-auriculovertebral syndrome, oculo-auriculovertebral
spectrum or lateral facial dysplasia
18. Vascular malformations Hemangioma, lymphangioma, arteriovenous aneurysm
19. Plagiocephaly Obliquely asymmetric cranium
20. Syngnathia Congenital adhesion of jaws
21. Cleft lip (CL) and palate (CP) 1:800 (1:500–1:2500)
CL+CP-males/isolated CP-females
Rule of three 10’s–10 lb, 10 mg/l, 10 weeks
https://t.me/DentalBooksWorld
226 Triumph's Complete Review of Dentistry

22. Median cleft-face syndrome Hypertelorism, median cleft of the premaxilla and palate, cranium bifidum
occultum
23. Median cleft of the upper lip Lack of fusion of median nasal process
(Orofacial digital syndrome and Ellis–van Creveld syndrome)
24. Lateral facial cleft Lack of fusion of maxillary and mandibular process
(mandibulofacial dysostosis, oculo-auriculo-vertebral spectrum,
Nager acrofacial dysostosis, amniotic rupture sequence)
25. Oblique facial cleft Failure of fusion of lateral nasal process and maxillary process
26. Submucous palatal cleft Intact surface mucosa with a notch in the bone due to defect in musculature
27. Common congenital Nasal, auricular, mandibular
deformations of head and neck
28. Agnathia (otocephaly, Hypoplasia or absence of mandible or part maxilla
holoprosencephaly)
29. Micrognathia True, acquired type

30. Pierre Robin syndrome Micro/retrognathia, glossoptosis, cleft palate


(Robin sequence, Pierre
robin anomalad, Pierre robin
malformation)
31. Pierre Robin malformation Associated with Stickler syndrome, velocardiofacial syndrome, fetal alcohol
syndrome
32. Macrognathia Large jaws occur in pituitary gigantism, acromegaly, Paget’s disease of bone,
acromegaly, leontiasis ossea
Excessive condylar growth results in macrognathia
33. Mandibulofacial dysostosis TCOF1 gene, bird or fish facies, antimongoloid palpebral fissures, hypoplasia
(Treacher Collins syndrome, of facial bones, malformed external ear, macrostomia, blind fistulas, atypical
Franceschetti–Zwahlen–Klein) hair growth, facial clefts, salivary gland aplasia, sleep apnea syndrome
34. Facial hemihypertrophy Beckwith–Wiedemann, neurofibromatosis, Klippel–Trenaunay–Weber,
Proteus, McCune–Albright, epidermal nevus, Langer–Giedion, triploid–
diploid mixoploidy, Maffucci, Ollier syndromes
35. Facial hemiatrophy (Parry Common in females, unilateral atrophy of skin, cartilage, bone, muscle,
Romberg, progressive facial contralateral Jacksonian epilepsy, trigeminal neuralgia, Coup de sabre, bluish
hemiatrophy) localized hue of skin
scleroderma
36. Partial lipodystrophy Bilateral facial atrophy
37. Congenital lip, pits and fistulas Van der Woude syndrome (paramedian lip pits and cleft lip, palate) Intraoral –
Maxillary hypodontia, syngnathia, high arched palate, ankyloglossia. Extraoral –
Limb anomalies, popliteal web, brain abnormalities, accessory nipples
38. Paramedian lip pits Pits on either side of midline in lower lip. (Van der Woude syndrome, popliteal
pterygium syndrome, Kabuki syndrome (everted lower lips)
39. Van der Woude syndrome (Paramedian lip pits and cleft lip, palate) most common form of syndromic
clefting. Mutation of interferon regulatory factor (IRF6)
40. Double lip Congenital – Persistence of sulcus between pars glabrosa and pars villosa
Ascher’s syndrome – Acquired double lip, blepharochalasis, nontoxic
thyroid goiter
41. Cheilitis glandularis Superficial type (Baelz type)
(actinic cheilitis) Deep suppurative type (Myxadenitis labialis)
Important differential diagnosis: Sarcoidosis, squamous cell carcinoma

https://t.me/DentalBooksWorld
Chapter 4 • Oral Pathology and Oral Medicine 227

42. Tests to differentiate Melkersson Serum angiotensin converting enzyme test, chest radiography, PET
and sarcoidosis
43. Microglossia Abnormally small tongue (Oromandibular – Limb hypogenesis syndromes –
Hypodactylia, hypomelia, cleft palate, intraoral bands, situs inversus)
44. Macroglossia Enlarged tongue (Beckwith–Wiedeman syndrome, Down syndrome, Duchenne
muscular dystrophy)
45. Beckwith–Wiedeman syndrome Omphalocele, visceromegaly, gigantism, neonatal hypoglycemia, macroglossia
46. Cheilitis granulomatosa Syndrome-fissured or plicated tongue, facial palsy, Miescher cheilitis
(Miescher–Melkersson– Noncaseating granulomas
Rosenthal)
47. Differentiation of true and Retrognathia of maxilla and mandible, checking tongue tone and mobility
pseudomacroglossia
48. Fissured tongue (lingual plicata) Seen in Melkersson–Rosenthal and Down syndrome

49. Black hairy tongue (coated Increased accumulation of keratin in filiform papilla
tongue)
50. Geographic tongue (psoriasiform Changing pattern of serpiginous white lines in smooth depapillated mucosa on
mucositis of tongue) dorsum of tongue
51. Caliber persistent artery Main arterial branch near the superficial mucosa without reduction in diameter.
Seen in lip
52. Castleman tumor ALHE, lymphoid hamartoma
HHV-8 infection group, HHV-negative group
• Hyaline vascular type – Asymptomatic
• Plasma cell type-fever, hemolytic anemia, hypergammaglobulinemia
• Mixed-hyaline + plasma cell type
• Plasmablastic, multicentric or generalized-hepatosplenomegaly
53. Twinning Division into one normal and one supernumerary tooth
54. Complete fusion Occurs if union occurs before calcification
55. Rubinstein–Taybi syndrome Developmental retardation, broad thumbs, great toes, delayed or incomplete
descent of testis, head circumference and bone age below fiftieth percentile
56. Radicular type of dens in dente Infolding of Hertwig’s epithelial root sheath
57. Dens evaginatus (occlusal Accessory cusp in occlusal surface of premolars
tuberculated premolar, Leong’s
premolar, evaginated odontome,
occlusal enamel pearl
58. Types of taurodontism Hypotaurodont, mesotaurodont, hypertaurodont
59. Syndrome associated with Klinefelter’s syndrome (XXY), amelogenesis imperfecta
taurodontism
60. Supernumerary roots Mandibular cuspids and bicuspids, mandibular third molars
61. Anodontia True partial anodontia-third molars, maxillary lateral incisors, maxillary and
mandibular second molars
Deciduous teeth – Maxillary lateral, mandibular lateral, mandibular cuspid
62. Supernumerary teeth Associated syndromes and conditions – Cleft lip and palate, cleidocranial
(hyperdontia) dysplasia, Gardner’s syndrome
63. Supernumerary teeth Mesiodens, distomolar/distodens (accessory fourth molar), paramolar
(supernumerary situated lingually or buccally to a molar)
64. Gardener’s syndrome Supernumerary teeth, multiple polyposis of intestine, osteomas of the jaw
bones, epidermoid or sebaceous cysts of the bones, desmoid tumors

https://t.me/DentalBooksWorld
228 Triumph's Complete Review of Dentistry

65. Microdontia Relative – Normal sized teeth in large jaws


True – Small teeth (Down’s syndrome)
66. Macrodontia (megalodontia, Relative macrodontia – Normal sized tooth with smaller jaw
megadontia) True macrodontia in association with pituitary gigantism, otodental syndrome,
XYY males, and pineal hyperplasia with hyperinsulinism
67. Natal teeth Predeciduous dentition – Present at the time of birth
68. Neonatal teeth Teeth that erupt during the first month after birth
69. Dilaceration Abnormal angulation of the root. Common in maxillary central incisor. (Smith–
Magenis syndrome, Ehlers–Danlos syndrome, Axenfeld–Rieger syndrome and
congenital Ichthyosis)
70. Globodontia Gigantic globe-shaped (otodental syndrome)
71. Lobodontia Teeth resembling a lobe of a carnivore
72. Amelogenesis imperfecta Witkop classification (hypoplastic, hypocalcified, hypomaturation,
(hereditary brown enamel, hypomaturation with taurodontism)
enamel dysplasia, brown GENES-DXS85-Xp22 (amelogenin), ENAM (enamelin)
opalescent teeth)
73. Environmental enamel Nutritional, exanthematous diseases, congenital syphilis, hypocalcemia, birth
hypoplasia injury, Rh hemolytic disease, local trauma, fluoride toxicity
74. Congenital syphilis Hutchinson’s teeth (anterior teeth)
Mulberry molars, Moon’s molars, Fournier’s molars (molars)
75. Turner’s teeth Maxillary or mandibular premolars, maxillary central incisors
76. Dentinogenesis imperfecta Chromosome 4 (DSPP-4q21.3) – Autosomal dominant inheritance
77. Revised dentinogenesis DI type 1 – Dentinogenesis imperfecta without osteogenesis imperfecta
imperfecta classification (shields type II)
DI type 2 – Dentinogenesis imperfecta without osteogenesis imperfecta
(shields type III)
78. Dentin dysplasia (Rootless Shields classification (type I – Dentin dysplasia and type II – Anomalous
dentin) dysplasia of dentin)
Witkop classification – Radicular dentin dysplasia (type I), coronal dentin
dysplasia (type II)
79. Radicular dentin dysplasia Lava flowing around boulders, crescent-shaped pulpal remnant
80. Coronal dentin dysplasia Thistle tube or flame-shaped pulp chamber
81. Regional odontodysplasia Thin enamel and dentin, large pulp. Fuzzy appearance of the coronal
(odontodysplasia, ghost teeth) region, short roots with open apex. Enameloid conglomerates are seen in
histology
82. Pulpal dysplasia Similar to coronal dentin dysplasia with enlarged pulpal chambers. Thistle tube
pulp chambers and multiple pulp stones are seen
83. Eruption sequestrum Irregular spicule of bone overlying an erupting molar
84. Delayed eruption Rickets, cretinism, cleidocranial dysplasia
85. Ankylosed teeth (submerged Deciduous mandibular second molar, dull/metallic sound on percussion
teeth)

https://t.me/DentalBooksWorld
Chapter 4 • Oral Pathology and Oral Medicine 229

ODONTOGENIC CYSTS AND TUMORS


Developmental Odontogenic Cyst
1. Odontogenic keratocyst (KCOT) • Philipsen (1956) sPTCH mutation in 85% syndromic and 30%
nonsyndromic cases
• 3–11% of odontogenic cysts
• Polarized basal layer (picket fence, tombstone appearance), satellite or
daughter cysts in cyst wall. Protein content of cyst fluid is <4 g/100 ml
• Associated with nevoid basal cell carcinoma syndrome (cutaneous,
dental, osseous, ophthalmologic, neurologic, and sexual abnormalities)
2. Orthokeratinized odontogenic cyst Thinner lining than OKC, orthokeratinized, onion skin like luminal
surface keratinization, prominent stratum granulosum
• 7–17% of odontogenic cysts
3. Dentigerous cyst (follicular cyst) Most common type of developmental odontogenic cyst. 10% of impacted
teeth develop into dentigerous cyst. Radiological types – Central,
lateral, and circumferential. Complications – Ameloblastoma, SCC,
mucoepidermoid carcinoma
Multiple cysts are associated with cleidocranial dysplasia and Maroteaux–
Lamy syndrome
4. Lateral periodontal cyst Cyst in the lateral periodontal location. Lining with focal thickenings
(plaques). Seen commonly in mandibular bicuspid
5. Botryoid odontogenic cyst Polycystic variant of lateral periodontal cyst
6. Glandular odontogenic Lining exhibits epithelial spherules, multiple cystic spaces, papillary
cyst (sialoodontogenic cyst, projections of epithelium projecting into cystic lumen, surface eosinophilic
mucoepidermoid odontogenic cyst) cuboidal cells (hobnail cells)
7. Calcifying odontogenic cyst Included as an odontogenic tumor. Presence of ghost cells in the epithelial
(keratinizing or calcifying epithelial lining
odontogenic cyst, Gorlin cyst, cystic
keratinizing tumor)
8. Eruption cyst (eruption hematoma) Soft tissue counter part of dentigerous cyst overlying an erupting tooth
9. Gingival cyst of new born (dental Epstein’s pearls – Cystic keratin-filled nodules in midpalatine raphe
lamina cyst of newborn, Epstein’s Bohn’s nodules – Keratin-filled cysts scattered over the palate
pearls, Bohn’s nodules)
Inflammatory Odontogenic Cysts
10. Apical periodontal cyst (radicular True cyst (cystic cavities completely enclosed in epithelial lining
cyst, periapical cyst, root end cyst) Bay cyst or pocket cyst (epithelial lined cavities that are open to the affected
root canals)
Rushton bodies – Arc-shaped hyaline bodies
11. Residual cyst Inflammatory odontogenic cyst in the edentulous alveolar ridge
12. Inflammatory collateral cyst or Inflammatory cyst related to the lateral/accessory canal of the root
lateral radicular cyst
13. Paradental cyst Inflammatory cyst found on the distal and facial aspect of vital mandibular
third molar
14. Buccal bifurcation cyst (Juvenile Cyst on the buccal aspect of permanent mandibular first and second molar
paradental cyst, mandibular which is partly or completely erupted and vital
infected buccal cyst)

https://t.me/DentalBooksWorld
230 Triumph's Complete Review of Dentistry

Developmental Nonodontogenic Cysts


15. Naso palatine cyst (nasopalatine Occurs in the midline of the anterior maxilla from the epithelial remnants
cyst, incisive canal cyst, median inside the nasopalatine ducts
anterior maxillary cyst
16. Nasoalveolar cyst (nasolabial cyst, Occurs from the epithelial remnants in the junction of the globular process,
Klestadt’s cyst) lateral nasal process, and maxillary process
17. Median palatal cyst Arises from the epithelial remnants in the midline of palate in the fusion
line of palatal process
18. Globulomaxillary cyst Occurs at the junction of globular portion of the medial nasal process and
maxillary process. Inverted pear-shaped radiolucency between the roots of
lateral incisor and canine
19. Median mandibular cyst Arises from epithelial remnants at the junction of the fusion of mandibular
process
20. Thyroglossal duct cyst Occurs at the midline of the neck from the remnants of the thyroid
diverticulum
21. Oral lymphoepithelial cyst Appears as yellowish nodule, arises from the lymphoid aggregates of the
oral and pharyngeal mucosa
22. Epidermoid cyst (epidermal cyst, • Occurs due to implantation of epithelial cells
epidermal inclusion cyst, epithelial • Associated with syndromes – Gardner syndrome, basal cell nevus
cyst, keratin cyst, milia) syndrome, pachyonychia congenita
• Milia – Miniature epidermoid cysts
23. Dermoid cyst (dermoid cystic Implantation of the skin epidermis along embryonic fusion lines
tumor, cystic teratoma, tumors of
omentum, spinal dermoid cysts)
Inflammatory Nonodontogenic Cysts
24. Mucous extravasation cyst Extravasation of salivary secretion from a traumatized salivary duct within
(mucocele, mucous extravasation/ the connective tissue
escape phenomenon
25. Ranula Extravasation cyst in the floor of the mouth associated with Bartholin and
Wharton’s duct
• Plunging Ranula – Suprahyoid type which has herniated through the
mylohyoid muscle
26. Mucous retention cyst (salivary True cyst – Arises because of ductal dilatation due to retention of saliva
duct cyst, sialocyst) within the duct
27. Retention cyst of the maxillary Occurs due to blockage of seromucous glands associated with lining of
sinus maxillary sinus
28. Surgical ciliated cyst of maxilla Arises due to implantation of the sinus epithelium along surgical incision
(sinus mucocele, postoperative planes
maxillary cyst)
29. Mucopyocele Infected mucocele in the maxillary sinus
30. Traumatic bone cyst (unicameral Pseudocyst with an empty cavity
bone cyst, Idiopathic bone cavity)
31. Aneurysmal bone cyst Pseudocyst, resembles a blood-soaked sponge, occurs below 20 years
of  age, four phases of growth (osteolytic, active growth, mature stage,
healing phase), honeycomb, soap bubble radiographic appearance
32. Parasitic cysts Hydatid cyst, cysticercosis, trichinosis presents as pseudocyst

https://t.me/DentalBooksWorld
Chapter 4 • Oral Pathology and Oral Medicine 231

BENIGN ODONTOGENIC TUMORS


Tumors Arising from Odontogenic Epithelium
1. Ameloblastoma (adamantinoma) Coined by Churchill (1934), second most common odontogenic
tumor, molar ramus area of mandible affected 80%, soap bubble
appearance in radiograph, six histopathologic subtypes (follicular,
plexiform, acanthomatous, granular cell, basal cell, desmoplastic)
rare variants (clear cell, papilliferous, keratoameloblastoma,
hemangiomatous ameloblastoma)
Follicular presents with a high recurrence rate. Granular type is
more aggressive (granules – lysosomes). Desmoplastic (increased
collagenized stroma due to increased TGF-β)
2. Unicystic ameloblastoma Lining epithelium with Vickers Gorlin criteria (hyperchromatic
basal cells arranged in a polarized and palisaded pattern,
cytoplasmic vacuolization with intercellular spacing)
Types – Luminal, intraluminal/plexiform, mural
3. Squamous odontogenic tumor (Benign Consists of islands of epithelium without peripheral palisading
epithelial odontogenic tumor) • Premolar/molar area of mandible and incisor – Canine area in
maxilla
• Important differential diagnosis – Acanthomatous
ameloblastoma and squamous cell carcinoma
4. Calcifying epithelial odontogenic tumor • Driven snow appearance in radiograph
(Pindborg tumor) • Hypointense in T1/mixed hyper intense in T2-CT image
• Liesegang rings – Histology
• Amyloid like eosinophilic material exhibiting apple green
birefringence in polarized light (Unique protein – ODAM –
Odontogenic ameloblast associated protein)
5. Adenomatoid odontogenic tumor • Odontogenic tumor with scattered duct-like structures
(adenoameloblastoma) • Ducts are surrounded by hyaline ring
• Occurs below 20 years of age and in females
• Common in anterior region of maxilla and mandible
Tumors Arising from Odontogenic Epithelium with Ectomesenchyme with or without
Hard Tissue Formation
6. Ameloblastic fibroma (soft mixed Mixed tumor with epithelial and mesenchymal tissue without
odontoma, fibroadamantoblastoma) hard tissue formation
7. Ameloblastic fibroodontoma Consists of epithelial and mesenchymal tissue without hard tissue
formation
8. Odontoma Hamartomatous proliferation of epithelial and mesenchymal tissue
that undergoes complete differentiation that results in functional
ameloblasts and odontoblasts forming enamel and dentin
Types: Compound (resembles teeth) complex (irregular mass)
Ghost cells are seen in histology
9. Ameloblastic odontoma Tumor characteristic of ameloblastoma and a composite odontoma
(odontoameloblastoma)
10. Calcifying cystic odontogenic tumor and Benign cystic neoplasm of odontogenic origin, characterized by
dentinogenic ghost cell tumor (calcifying an ameloblastoma like epithelium with ghost cells that may calcify
odontogenic cyst) Dermal variant: Calcifying epithelioma of Malherbe

https://t.me/DentalBooksWorld
232 Triumph's Complete Review of Dentistry

Tumors Arising from Odontogenic Ectomesenchyme with or without Included Odontogenic Epithelium
11. Peripheral odontogenic fibroma Occurs as a solid gingival mass. Cuffing calcifications are present.
Calcified areas are present as dysplastic dentin
12. Central odontogenic fibroma Fibroblastic neoplasm with odontogenic epithelium and dysplastic
dentin or cementum-like material
13. Odontogenic myxoma (odontogenic Mesenchymal origin with myxomatous degeneration of fibrous
fibromyxoma, myxofibroma) stroma. Tennis Racquet, stepladder, sunray or sunburst appearance
in radiograph
• Increased fibronectin, tenascin, chondroitin sulfate and
hyaluronic acid
• Increased alkaline phosphatase and lactate dehydrogenase
14. Cementoblastoma (true cementoma) True neoplasm of functional cementoblasts. Occurs below 25 years
of age, mandibular first molar is commonly affected. Affected
tooth is vital
15. Granular cell odontogenic tumor (granular Odontogenic epithelium exhibiting granular change with
ameloblastic fibroma) cementum/dysplastic dentin

MALIGNANT ODONTOGENIC TUMORS


Odontogenic Carcinomas
16. Malignant ameloblastoma (metastasizing Tumor that shows typical ameloblastoma histologic features but
ameloblastoma) exhibits metastasis
17. Ameloblastic carcinoma Tumor with malignant epithelial proliferation that is associated
with an ameloblastoma (carcinoma ex ameloblastoma)
18. Ghost cell odontogenic carcinoma Ameloblastic carcinoma with evidence of ghost cell keratinization
19. Clear cell odontogenic carcinoma Tumor is composed of neoplastic ameloblastomatous proliferation
with clear cell differentiation
Odontogenic Sarcomas
20. Odontogenic sarcoma (ameloblastic Malignant proliferation of connective tissue cells with benign
fibrosarcoma) odontogenic epithelium

EPITHELIAL PATHOLOGY
Benign Tumors of Epithelial Origin
1. Squamous papilloma
• Human papillomavirus (HPV), double-stranded DNA viruses of the papovavirus subgroup A
• Age: 30–50, Site: Soft palate
• Soft painless, exophytic, pedunculated, cauliflower-like projections
• The papilloma is characterized by a proliferation of keratinized stratified squamous epithelium arrayed in finger-like
projections with fibrovascular connective tissue cores
–– Koilocytes: Virus-altered epithelial clear cells with small dark (pyknotic) nuclei are sometimes seen high in the
prickle cell layer
• Conservative surgical excision, including the base of the lesion is adequate treatment
• HPV 6 and 11 → RRP (recurrent respiratory papillomatosis)
2. Verruca vulgaris (common wart)
One or more of the associated human papillomavirus (HPV) types 2, 4, 6, and 40 are found in virtually all examples
• Age – Children (more common), Site – The skin of the hands is usually the site of infection, the vermilion border,
labial mucosa, or anterior tongue
• Painless papule or nodule with papillary projections or a rough pebbly surface (It may be pedunculated or sessile,
cutaneous lesions may be pink, yellow, or white; oral lesions are almost always white

https://t.me/DentalBooksWorld
Chapter 4 • Oral Pathology and Oral Medicine 233

• Verruca vulgaris enlarges rapidly to its maximum size (usually less than 5 mrnl, and the size remains constant for
months or years thereafter unless the lesion is irritated. Multiple or clustered lesions are common
–– Extreme accumulation of compact keratin may result in a hard surface projection several millimeters in height,
termed a cutaneous horn or keratin horn
• Elongated rete ridges tend to converge toward the center of the lesion, producing a “cupping” effect. A prominent
granular cell layer (hypergranulosis) exhibits coarse, clumped keratohyaline granules. Abundant koilocytes are often
seen in the superficial spinous layer
• Skin verrucae are treated effectively by liquid nitrogen cryotherapy, conservative surgical excision or curettage, or
topical application at keratinolytic agents (usually containing salicylic acid and lactic acid
3. Condyloma acuminatum
• HPV types 2, 6, 11, 53, and 54
• Age: Teenagers and young adults, Site: Labial mucosa, soft palate, and lingual frenum
• Sessile, pink, well-demarcated, nontender exophytic mass with short, blunted surface projections
• Larger than the papilloma and is characteristically clustered with other condylomata. Size: 1.0–1.5 cm, but oral lesions
as large as 3 cm have been reported
• Condyloma acuminatum appears as a benign proliferation of acanthotic stratified squamous epithelium with mildly
keratotic papillary surface projections
• The covering epithelium is mature and differentiated, but the prickle cells often demonstrate pyknotic nuclei
surrounded by clear zones (koilocytes), a microscopic feature of HPV infection
• Treatment: The oral condyloma is usually treated by conservative surgical excision. Laser ablation also has been used
4. Sinonasal papilloma
• Papillomas of the sinonasal tract are benign. localized proliferations of the respiratory mucosa of this region
• This mucosa gives rise to three histomorphologically distinct papillomas:
–– Fungiform (septate)
–– Inverted (inverted schneiderian)
–– Cylindrical cell (oncocytic schneiderian)
5. Molluscum contagiosum
DNA poxvirus
• Children and young adults
• The papules almost always are multiple and occur predominantly on the skin of the neck, face (particularly eyelids),
trunk, and genitalia
• Usually on the lips, buccal mucosa, or palate. Lesions are pink, smooth-surfaced, sessile, nontender, and nonhemorrhagic
papules that are 2–4 mm in diameter. Many show a small central indentation or keratin-like plug from which a curd
like substance can be expressed. Some are surrounded by a mild inflammatory erythema and may be slightly tender
• The central portion of each lobule is filled with bloated keratinocytes that contain large, intranuclear, basophilic viral
inclusions called molluscum bodies (Henderson–Paterson bodies). These bodies begin as small eosinophilic structures
in cells just above the basal layer
• Treatment: Spontaneous remission occurs within 6–9 months; however, curettage and cryotherapy
6. Verruciform xanthoma
• Unusual reaction or immune response to localized epithelial trauma or damage
• Typically seen in whites, 40–70 years of age. There is a strong female predilection
• Gingiva and alveolar mucosa
• Well-demarcated, soft, painless, sessile, slightly elevated mass with a white, yellow–white, or red color and a papillary
or roughened (verruciform) surface. Important diagnostic feature is the accumulation of numerous large macrophages
with foamy cytoplasm
–– Foam cells also known as xanthoma cells. contain lipid and periodic acid–Schiff (PAS)positive, diastase-resistant
granules
7. Ephelis
• Common small hyperpigmented macule of the skin that represents a region of increased melanin production
• Mutations in MC1R gene (melanocortin 1 receptor gene)
• First decade, becomes less prominent in adult life
• Macule, less than 3 mm in diameter, uniform light brown color discoloration, intensity increases on sun exposure
• Increased melanin pigmentation and number of melanocytes are normal

https://t.me/DentalBooksWorld
234 Triumph's Complete Review of Dentistry

8. Actinic lentigo (lentigo solaris, solar lentigo, age spot, liver spot, senile lentigo)
• Chronic ultraviolet light damage to the skin
• 90% of whites older than 70 years of age and rarely is seen before age 40
–– Dorsa of the hands, on the face, and on the arms of elderly whites
–– Multiple, individual lesions appear as uniformly pigmented brown to tan macules with well-demarcated but irregular
borders
• Lesion may reach more than 1 cm in diameter. Unlike ephelides, no change in color intensity is seen after exposure to
ultraviolet light
• Rete ridges are elongated and club-shaped in actinic lentigines, with thinning of the epithelium above the connective
tissue papillae
• The ridges sometimes seem to coalesce with one another. Within each rete ridge, melanin-laden basilar cells are
intermingled with excessive numbers of heavily pigmented melanocytes
9. Lentigo simplex
• Lentigo simplex is one of several forms of benign cutaneous melanocytic hyperplasia of unknown origin
• Occurs in children but may occur at any age
• Sharply demarcated macule smaller than 5 mm in diameter, with a uniformly tan to dark-brown color
• Clinically, individual lesions of lentigo simplex are indistinguishable from the nonelevated melanocytic nevus. With
multiple lesions, conditions such as lentiginosis profusa, Peutz–Jegher’s syndrome and the multiple lentigines or
Leopard' syndrome must be considered
• Lentigo simplex shows an increased number of benign melanocytes within the basal layer of the epidermis, and these
often are clustered at the tips of the rete ridges
• Abundant melanin is distributed among the melanocytes and basal keratinocytes, as well as within the papillary dermis
in association with melanophages (melanin in continence)
10. Melasma
• Mask of pregnancy
• The cause is unknown, but it is strongly associated with pregnancy and the use of oral contraceptives that contain both
estrogen and progesterone
• Dark-complexioned persons are more likely to develop melisma – Adult women
• Melasma appears as bilateral light to dark-brown cutaneous macules that vary in size from a few millimeters to more
than 2 cm in diameter. Lesions develop slowly with sun exposure and occur primarily on the midface, forehead, upper
lip, chin, and (rarely) the arms
• Melasma is characterized by increased melanin deposition within an otherwise unremarkable epidermis
• Pigment also may be seen within numerous melanophages in the dermis (melanin-laden macrophages)
• Treatment: 3% hydroquinone, triretinoic acid
11. Oral melanotic macule
• Melanotic macule is not dependent on sun exposure
–– Appears as a solitary (17% are multiple), well-demarcated, uniformly tan to dark-brown, asymptomatic, round or
oval macule
• Discoloration produced by a focal increase in melanin deposition and increase in the number of melanocytes in the
basal and parabasal lavers of an otherwise normal stratified squamous epithelium
• Melanin may also be seen free or within melanophages in the subepithelial connective tissue (melanin incontinence).
The lesion does not show elongated rete ridges like actinic lentigo
12. Oral melanoacanthoma
• Melanoacanthosis
• Oral melanoacanthoma is a benign and uncommon acquired pigmentation of the oral mucosa characterized by
dendritic melanocytes dispersed throughout the epithelium
13. Acquired melanocytic nevus (nevocellular nevus, mole)
• Congenital or developmental in nature
• Before 35 years of age. Both men and women, although women usually have a few more than men
Congenital melanocytic nevus
Present in 1% of new borns, two types – Type 1 – Small (<20 cm), Type 2 – Large (>20 cm), hypertrichosis within
the lesion is seen, Increases in prominence with age (giant hairy nevus). Large nevus is called bathing trunk nevus/
garment nevus

https://t.me/DentalBooksWorld
Chapter 4 • Oral Pathology and Oral Medicine 235

Halo nevus
Melanocytic nevus with a hypopigmented border. Due to destruction of the nevus cells by the immune system, Spitz
nevus (benign juvenile melanoma/spindle and epithelioid cell nevus). Similar to melanoma in histopathology
Blue nevus (dermal melanocytoma: Jadassohn–Tieche nevus)
• Benign proliferation of dermal melanocytes
• Blue color is produced due to TYNDALL effect – Interaction of light with particles in a colloidal suspension
Combined nevus – Blue nevus with an overlying melanocytic nevus
• Mutation in GNAQ gene (encodes G-protein alpha subunit important for signal transduction from cell surface
receptor)
• Junctional – Nevus cells are found along the basal layer of the epithelium
• Compound – Proliferation of nevus cells and drops into the connective tissue (lamina propria)
• Intradermal/Intramucosal – Found only in the connective tissue. Most common mucosal type of oral nevi
• The acquired melanocytic nevus is characterized by a benign, unencapsulated proliferation of small, ovoid cells (nevus
cells). Nevus cells typically lack the dendritic processes that melanocytes possess
• A characteristic microscopic feature is the presence of superficial nevus cells which tend to be organized into small
round aggregates called theques.
• Type A-epithelioid, Type B-lymphocyte like, Type C-Spindle-shaped
Premalignant Lesions
14. Leukoplakia (leukokeratosis/erythroleukoplakia)
Tobacco, alcohol, sanguinaria, UV radiation, microorganisms, trauma
Risk factors – Loss of heterogeneity in 3p and 9p arms/4q, 8p, 11q, 13q, 17p, alterations in p53 (tumor suppressor),
alterations in p16 (cell cycle marker)
Age – 40, Sex – Males, Site – Vermilion border of lip, buccal mucosa, gingiva. Lesions on tongue, lip vermilion, oral
floor – 90% shows dysplasia
• Types – Homogenous leukoplakia, granular or nodular, verrucous or verruciform, proliferative verrucous
• Hyperkeratosis (ortho/para) – hyperorthokeratosis presents with granular cell layer, acanthosis
• Dysplastic features – (Enlarged nuclei, large and prominent nucleoli, increased nuclear-to- cytoplasmic ratio,
hyperchromatic nuclei, pleomorphic nuclei, dyskeratosis, increased mitotic activity, abnormal mitotic figures, bulbous
or tear-shaped rete pegs, loss of polarity, keratin pearls, loss of typical epithelial cell cohesiveness) – Ductal dysplasia
• Mild dysplasia – Alterations limited to basal and parabasal layer
• Moderate dysplasia – Basal layer to midportion of spinous layer
• Severe dysplasia – Basal layer to above midportion of epithelium
• Carcinoma in situ – Top to bottom change
• Malignant transformation – For homogenous leukoplakia – 1–7%, granular and verruciform – 4–15%, erythroplakia
– 28%, lesions in the floor of the mouth, ventral surface of tongue, nonsmokers – 16–39%, female patients (47%)
Treatment: Surgery, laser ablation – pharmacological agents – Iso tretinoin, betacarotene, bleomycin, lycopene, COX2
inhibitors
15. Erythroplakia (erythroplasia, erythroplasia of Queyrat)
• Age: 4th–6th decade, Site: Floor of the mouth, tongue, soft palate
• 90% severe epithelial dysplasia, carcinoma in situ, invasive squamous cell carcinoma
• Epithelium is atrophic allowing the underlying microvasculature to show through
• Surgery and pharmacological management, long-term follow-up
16. Smokeless tobacco keratosis (snuff pouch, snuff dipper’s lesion, tobacco pouch keratosis, spit tobacco keratosis)
Smokeless tobacco–chewing tobacco, moist snuff, dry snuff
• Gingival recession, painless loss of gingival tissues – Destruction of facial surface of alveolar bone
• Increased prevalence of dental caries, Localized or generalized occlusal or incisal wear
• Smokeless tobacco keratosis – White plaque on mucosa – Snuff pouch, tobacco pouch-stretched mucosa appears
fissured or rippled (sand on a beach after an ebbing tide)
• Hyper keratinized squamous epithelium, acanthosis, intracellular vacuolization or edema of glycogen-rich superficial cells
• Parakeratin – Chevrons – Pointed projections above or within superficial epithelial layers
• Mild epithelial dysplasia

https://t.me/DentalBooksWorld
236 Triumph's Complete Review of Dentistry

17. Oral submucous fibrosis


Site: Buccal mucosa, retromolar area, soft palate
• Vesicles, petechiae, melanosis, xerostomia, stomatopyrosis while eating spicy foods, marble-like progressive stiffness
of subepithelial tissues, Trismus, interincisal width of 20 mm is considered severe
• Submucosal deposition of dense and hypovascular collagenous connective tissue with variable numbers of chronic
inflammatory cells
• Subepithelial vesicles in early lesions, hyperkeratosis with marked epithelial atrophy in older lesion, epithelial dysplasia
is seen in 10–15% of cases, and carcinoma is found in 6% of sampled cases
• Does not regress with cessation of habit
• Intralesional corticosteroids, interferon gamma, surgical stripping/splitting of fibrous band, lycopene and pentoxifylline
• Malignant transformation: 8–20%
18. Nicotine stomatitis (nicotine palatinus, smoker’s palate)
• Due to cigar and pipe smoking, develops due to heat rather than the chemicals of the tobacco smoke
• Reverse smoking produces – Reverse smokers palate, age: above 45 years, sex: male
• Diffuse gray or while change with slightly elevated papules with punctate red centers representing inflamed minor
salivary glands and their ductal orifices, palatal keratin becomes so thick and fissured giving a dried mud appearance
• Heavy brown or black stain on the teeth may be seen
19. Actinic keratosis (solar keratosis)
• Cumulative UV radiation
• Occurs in persons older than 40 years, face, neck, dorsum of hands, forearms, scalp of bald headed men
• Palpation – Sand paper-like texture
• May present with a horn arising from the central area-Keratin or cutaneous horn
• Scaly plaques – White, gray, or brown, parakeratinosis and acanthosis, teardrop-shaped retepegs
20. Actinic cheilitis (actinic cheilosis; solar cheilosis)
• Strong male predilections/45 years
• Common premalignant alteration of the lower lip vermilion, chronic UV light exposure
• Farmers lip, sailors lip
• Common premalignant alteration of the lower lip vermilion that results from long-term exposure of UV rays,
immunocompromised state
• Ulceration may develop → lasts for months and progress to squamous cell carcinoma, hyperkeratosis, epithelium –
Atropic or acanthotic
• Vermilionectomy, laser ablation, electrodessication, chemo exfoliation
21. Keratoacanthoma (self-healing carcinoma, molluscum pseudocarcinomatosum, molluscum sebaceum, verrucoma)
• Sunlight, exposure to coal tar, trauma, HPV, genetic factors, occurs at all ages, incidence increases with age, common
in men
• Immunocompromised state, chromosomal aberrations-3p.9p, 19p, 19q
• Solitary, firm, round, skin-colored or reddish papules which progress to nodules. The nodules have a central crater
form ulceration or keratin plug that may project like a horn, size does not exceed 1.0–1.5 cm, painful and regional
lymphadenopathy is present, undergoes spontaneous regression after a period of six to eight weeks
• The most characteristic feature is found at the margins where the normal adjacent epithelium is elevated toward the
central portion of the crater. (acute angle or buttress)
• Drugs – BRAF inhibitors and tyrosine kinase inhibitor
Phases:
• Growth phase – Rapid enlargement of about 1–2 cm within 6 weeks
• Stationary phase – Stabilization of the lesion; similar duration as in growth phase
• Most lesions regress spontaneously within 6–12 months of onset; leaving behind a depressed scar)
• Involution phase – Related to cytotoxic immune response to the tumor or mechanisms similar to those controlling
normal cycling of the hair follicles
• Multiple lesions occur in Ferguson Smith type and Witten-Zak syndrome
• Multiple papules in skin and Upper GIT- Eruptive Grzybowski type, Muir-Torre syndrome (sebaceous neoplasms,
keratoacanthoma, and gastrointestinal carcinoma), Xeroderma pigmentation
• Treatment – Surgical excision, prognosis is excellent. Recurrent tumors need aggressive therapy, regular follow-up is
necessary

https://t.me/DentalBooksWorld
Chapter 4 • Oral Pathology and Oral Medicine 237

Malignant Epithelial Tumors


Incidence rate in India – Age standardized incidence rate of oral cancer is reported at 12.6 per 100,000 people
22. Basal cell carcinoma (basal cell epithelioma rodent ulcer)
Results mainly from cumulative (including chronic and intermittent) UV radiation exposure, frequent sun burns,
additional risk factors – Psoriasis, PUVA treatment, tanning device, ionizing radiation exposure, immunosuppression,
and arsenic ingestion
• Dysregulation of hedgehog signaling pathway → BCC, mutation of PTCH gene → nevoid BCC, mutation of TP53 →
50% of sporadic BCC
• Nodular, begins as firm and painless papule growing slowly with central depression or umbilication, destruction of
surrounding bone, cartilage occur and extremely rare occurrence of metastasis
• Nodular, pigmented, cystic, superficial, micronodular, keratotic, adenoid, infiltrative, sclerosing (morphea form)and
micronodular. (Nodular is most common, micronodular, morpheaform, and infiltrative are more aggressive)
• Syndrome-related BCC → uniform, ovoid, dark staining basaloid cells with medium-sized nuclei and little cytoplasm
• Actinic damage in the form of solar elastosis of adjacent stroma
• BCC admixed with an independent primary SCC of the skin → “COLLISON” tumor is called Basosquamous
Carcinoma
• Immunohistochemically expression of Ber-EP4 (a cell surface glycoprotein preferentially expressed in cutaneous
BCC) – Helps to distinguish extreme rare cases of intraoral BCC from Peripheral ameloblastoma
• Head and neck lesions with a high risk for recurrence → Mohs microscopic surgery, radiotherapy – Patients who can’t
tolerate surgery
• Alternative treatments → 5-fluorouracil, topical imiquimod, photodynamic therapy, vigorous cryotherapy (for low
risk superficial BCC)
23. Merkel cell carcinoma (merkel cell tumor, neuroendocrine carcinoma of skin; small cell carcinoma of skin, trabecular
carcinoma of skin)
• Aggressive malignancy with neuroendocrine features
• Merkel cells – Mechanoreceptor cells found primarily in skin and keratinized oral mucosa
• Most common in white adults
• Causes – Sun exposure, immunosuppression, old age
• Extracutaneous lesion are rare, most often affect salivary glands. Asymptomatic, rapidly enlarging, smooth, dome-
shaped nodule with Telangiectasias (acronym – AEIOU)
• Approximately 27% of the cases demonstrate regional lymph node metastasis
• 14% of the cases – Nodal disease of unknown primary origin ; presumably due to regression of the primary tumor
• Sheets and anastamosing strands of small to moderately sized, round, basophilic cells; infiltrates dermis and
subcutaneous fat
• Pseudoglandular, trabecular, cribriform (“swiss cheese”), and sheet-like pattern
• Surgery (Mohs micrographic surgery), chemotherapy – For cases with distant metastasis, approximately 25% of Merkel
cell carcinoma develops additional malignancy (SCC, hematological malignancy, adenocarcinoma of breast and ovary
24. Oral squamous cell carcinoma
• Tobacco smoking – 70 carcinogens (nitrosamines, arsenic, benzo[a]pyrene and benzene
• Smoking produces free radicals and oxidants → promotes destruction of endogenous antioxidants (glutathione-S-
transferase, glutathione reductase and superoxide dismutase)
Smokeless tobacco, alcohol (ethanol metabolized into acetaldehyde-known carcinogen), carcinogenic impurities –
Polycyclic aromatic hydrocarbons and nitrosamines may be present in some beverages, occupational exposure and
environmental pollutants, high risk for workers in Wood products industries chronically exposed to certain chemicals
such as phenoxyacetic acids. heavy metal pollutants – Nickel, chromium, arsenic concentration in blood, radiation,
vitamin/mineral deficiency and dietary factors – Plummer–Vinson or Paterson–Kelly syndrome → SCC of esophagus,
oropharynx, and posterior mouth, vit A deficiency (producing excess keratinization of skin), vit C and E, folate,
flavonoids, fibers, lycopene and phytosterols
Bacteria – “Streptococcus species, Neisseria species, Candida and other bacteria associated with production of high level
of acetaldehyde” Immuno compromised state – Candida, HPV, EBV, HCV
Other factors – An impaired ability to repair DNA damaged by mutagens, impaired ability to metabolize carcinogens
• Deficiencies of vitamins A, E, or C, or trace elements, Poor oro-dental hygiene, chronic irritation – Ill-fitting denture
• The age-standardized incidence rate per 100,000 population 12.8 in males and 7.5 in female

https://t.me/DentalBooksWorld
238 Triumph's Complete Review of Dentistry

• Age: Elderly – After 4th decade. Oral SCC affects – The tongue in 20–40% of cases, the floor of the mouth in 15–20%
of the cases, and together these sites account for about 50% of all cases of oral SCC
• The gingivae, palate, retromolar area, and the buccal and labial mucosa are oral sites less frequently affected
• An indurated lump/ulcer (i.e., a firm infiltration beneath the mucosa), nonhealing extraction socket
• A lesion fixed to deeper tissues or to overlying skin or mucosa, cervical lymph node enlargement
• SCCs of the posterior part of the oral cavity are much more likely to metastasize to regional lymph nodes than are
comparable SCCs of the anterior part of the oral cavity
• Variants of oral squamous cell carcinoma – Spindle cell carcinoma, adenosquamous carcinoma, basaloid carcinoma
• Radiography: The local destruction of bone produces the “teeth floating in space” appearance similar to that of
histiocytosis X
• Preoperative imaging needs to comment on absence or presence of mandibular erosion, whether erosion is subtle or
gross (buccal, occlusal, and/or lingual cortices and if the marrow and inferior alveolar canal are invaded
• CT or MRI, size of tumor and tumor thickness
• MRI: MRI is excellent at identifying the extent of tumor infiltration and is especially useful in patients with significant
dental amalgam which causes artifact on CT – In larger lesions that abut the mandible, MRI is more sensitive than CT
at identifying early marrow change but less sensitive at visualizing cortical erosion
• Metastasis:
–– The different types of metastasis of oral squamous cell carcinoma include:
–– Lymphatic metastasis, distant metastasis, perineural spread. About 8% of patients with oral SCC will have distant
metastases at the time of diagnosis, most frequently to the lungs
25. Broder’s histopathological grading
Tumors were graded as follows
Well differentiated (Grade) = <25% undifferentiated cells
Moderately differentiated (Grade II) = <50% undifferentiated cells
Poorly differentiated (Grade III) = <75% undifferentiated cells
Anaplastic/pleomorphic (Grade IV) = >75% undifferentiated cells
26. Verrucous carcinoma (snuff dippers cancer, Ackerman’s tumor)
• Smokeless tobacco, snuff, human papilloma virus 16, 11, 16 and 18
• Develop from high-risk premalignant state- proliferative verrucous leukoplakia
• Tobacco pouch keratosis may be seen in adjacent mucosal surfaces
• Age – 65–70 years, sex – Males, incidence – 1–10% of all oral squamous cell carcinoma
• Appears as diffuse, well-demarcated, painless, thick plaque with papillary or verruciform surface projections
• Enlarged cervical lymph nodes usually represent inflammatory reactive changes rather than nodal metastases
• Verrucous hyperplasia – Exophytic overgrowth of well-differentiated keratinizing epithelium but without pushing
borders
• Characterized by wide, elongated rete ridges that appear to push into the underlying connective tissue, lesions usually
show abundant keratin – Usually parakeratin, parakeratin typically fills the numerous clefts or crypts-parakeratin
plugs-appear as surface projections, lesional epithelial cells show a normal maturation pattern with no significant
atypia
• Surgical excision, inoperable cases – Chemotherapy, photodynamic therapy
27. Malignant melanoma
• Neoplasm of the epithelial melanocytes. 3% of all malignancies and third most common cancer of skin
• Sun exposure, artificial UV sources
• Phases of growth – Radial growth phase (flat or macular), vertical growth phase (mass, nodule, elevation), and vertical
growth phase–melanoma with metastasis

https://t.me/DentalBooksWorld
Chapter 4 • Oral Pathology and Oral Medicine 239

Types
• Superficial spreading melanoma: Common type, has a radial growth phase. Melanoma in situ
• Nodular melanoma: Exists in a vertical growth phase
• Lentigo maligna melanoma: Exists in a radial growth phase
• Acral lentiginous melanoma: On palms and soles. Rapidly progresses from radial to vertical growth phase
• Mucosal lentiginous melanoma: Aggressive
• Amelanotic melanoma: Erythematous pink nodular presentation
Clinical diagnosis ABCDE Rule
A-Asymmetry
B-Irregular borders
C-Irregular color
D-Diameter more than 6 mm
E-Elevation surface
Clark histopathological Clark scale has five levels:
grading Level 1 also called melanoma in situ
Level 2 melanoma cells in the papillary dermis
Level 3 melanoma cells in the reticular dermis
Level 4 melanoma has spread into the reticular or deep dermis
Level 5 melanoma has grown into the subcutaneous fat
Breslow tumor thickness • 0–0.76 mm, 0.76–1.49 mm, 1.50–3.99 mm, and greater than 4.00 mm
grading • 10-year survival:
–– 92% for melanoma <1.00 mm thick;
–– 80% if 1.01–2.00 mm thick;
–– 63% if 2.01–4.00 mm thick;
–– 50% if >4.00 mm thick

BENIGN CONNECTIVE TISSUE TUMORS

1. Fibroma • Most common benign fibrous tumor of connective tissue


• Most often on the buccal mucosa along the plane of occlusion of the maxillary and
mandibular teeth. It is a round-to-ovoid, asymptomatic, smooth-surfaced, and firm
sessile or pedunculated mass. The diameter may vary from 1 mm to 2 cm.
The surface is usually similar in color to the surrounding mucosa but may be
hyperkeratotic or ulcerated, owing to repeated trauma
• Epithelium appears to be stretched and the underlying connective tissue shows dense
collagen fibers
• Treatment: Excision
2. Myofibroma • Well-circumscribed but not encapsulated nodules
• Biphasic
• Paucicellular nodules of spindled myofibroblasts with elongated nuclei
• Cellular areas between nodules composed of smaller primitive round or spindle
basaloid cells with round to oval nuclei and scant cytoplasm. Cellular areas contain
branching blood vessels

https://t.me/DentalBooksWorld
240 Triumph's Complete Review of Dentistry

3. Central giant cell granuloma • Young age – Below 30 years, females, mandible, common in anterior segments
• Nonaggressive/aggressive – Expansion of cortex, perforation, mobility, displacement,
root resorption of associated teeth, pain
• Radiographic features: Destructive lesion, Radiolucent area
• Loose fibrillar connective tissue stroma, interspersed with proliferating fibroblasts and
small capillaries, multinucleated giant cells are prominent in the connective tissue,
numerous foci of old extravasated blood and associated hemosiderin pigment, foci of
new trabeculae of osteoid or bone are seen
• Treatment – Curettage/surgical excision
4. Peripheral giant cell • Local irritation due to dental plaque or calculus, periodontal disease, poor dental
granuloma (peripheral giant restorations, ill-fitting dental appliances, dental extractions, 4th–6th decade, females
cell epulis peripheral giant • Asymptomatic, occurs on the gingiva or alveolar process, anterior to the molars,
cell reparative granuloma) pedunculated/sessile
• Originates from the periodontium or mucoperiosteum, 0.5–1.5 cm
• Dark red, vascular, hemorrhagic
• Nonencapsulated mass composed of reticular and fibrillar connective tissue, ovoid
spindle-shaped young connective tissue cells and multinucleated giant cells, numerous
capillaries, foci of hemorrhage with liberation of hemosiderin pigment, spicules of
newly formed osteoid or bone are often found scattered throughout the vascular and
cellular fibrous lesion
5. Osteoclastoma • Females, swelling in the affected region, pain, weakness
• Limitation of motion of the joint, pathologic fracture
• Round to oval or even spindle-shaped nucleus, mitotic figures can be found
• Presence of giant cells – 40–60 nucleus, areas of infarctile necrosis, foam cells
• Curettage, secondary malignant change
6. Aneurysmal bone cyst • Arises from a traumatic event, vascular malformation or neoplasm that disrupts the
normal osseous hemodynamics and leads to an enlarging hemorrhagic extravasation
• May form when an area of hemorrhage maintains connection with the disrupted
feeding vessels, subsequently
• Giant cell granuloma-like areas can develop after loss of connection with the original
vascular source
• Swelling, pain, paresthesia compressibility, and crepitus. Malocclusion, mobility,
migration, or resorption of involved teeth may be present
• Maxillary lesions often bulge into the adjacent sinus; nasal obstruction, nasal bleeding,
proptosis, and diplopia
• Radiography – Unilocular or multilocular radiolucent with marked cortical expansion
and thinning. ballooning or "blow-out" distention of the contour of the affected bone
• Filled with unclotted blood surrounded by cellular fibroblastic tissue containing
multinucleated giant cells and trabeculae of osteoid and woven bone. On occasion, the
wall contains an unusual lacelike pattern of calcification that is uncommon in other
intraosseous lesions
• The appearance at surgery “blood-soaked sponge”
7. Lipoma • The lipoma is a benign tumor of fat
• Twice as common in females as in males, 40 years of age or older;
• Soft, smooth-surfaced nodular masses that can be sessile or pedunculated,
asymptomatic, less than 3 cm in size
• Yellow hue often is detected clinically, deeper examples may appear pink
• Buccal vestibules account for 50% of all cases. Less common sites include the tongue,
floor of the mouth, and lips
• Composed of mature fat cells, a thin fibrous capsule, rare occasions, central
cartilaginous or osseous metaplasia may occur
• Treatment: Conservative local excision

https://t.me/DentalBooksWorld
Chapter 4 • Oral Pathology and Oral Medicine 241

8. Verruciform xanthoma • Hyperplastic condition of the epithelium of the mouth, skin, and genitalia, whites –
40–70 years of age, strong female predilection
• Gingiva and alveolar mucosa, well-demarcated, soft, painless, sessile, slightly
elevated mass with a white, yellow–white, or red color and a papillary or roughened
(verruciform) surface
• Verruciform xanthoma demonstrates papillary, acanthotic surface epithelium covered
by a thickened layer of parakeratin. Distinctive clefts or crypts between the epithelial
projections are filled with parakeratin and rete ridges are elongated to a uniform depth
• Accumulation of numerous large macrophages with foamy cytoplasm which typically
are confined to the connective tissue papillae. These foam cells also known as xanthoma
cells contain lipid and periodic acid–Schiff (PAS) positive diastase-resistant granules
• Treatment: Conservative surgical excision
9. Hemangioma • Characterized by a rapid growth phase with endothelial cell proliferation, followed by
gradual involution
• Tumors of infancy, occurring in 5–10% of 1-year-old children. They are much more
common in females
• Superficial tumors of the skin appear raised and bosselated with a bright-red color
(Strawberry Hemangioma)
• Firm and rubbery to palpation. Deeper tumors may appear only slightly raised with
a bluish hue
• Kasabach–Merrit syndrome – Large or extensive hemangioma
• Early hemangiomas are characterized by numerous plump endothelial cells and often-
indistinct vascular lumina
• Because of their cellular nature, these lesions also have been called juvenile
hemangioendothelioma
• The proliferative phase usually lasts for 6–10 months after which the tumor slows in
growth and begins to involute
• Complete resolution by 5 years of age with 90% resolving by age 9
• Because most hemangiomas undergo involution, management often consists of
“watchful neglect”
• The treatment of arteriovenous malformations is more challenging and also depends on the
size of the lesion and degree of involvement of vital structures. Radiographic embolization
is often performed 24–48 hours before surgery in order to minimize blood loss
10. Vascular malformations • Present at birth and persist throughout life
• 10–20 years of age. more common in females than males and occur twice as often in
the mandible
• Intrabony vascular malformations – Asymptomatic or associated with pain and
swelling. Mobility of teeth or bleeding from the gingival sulcus may occur. A bruit or
pulsation may be apparent on auscultation and palpation
• Radiography – Honeycomb appearance or large (soap bubble appearance)
• Vascular malformations do not show active endothelial cell proliferation – Capillary
malformation – Similar to the capillary stage of hemangioma, venous malformations
may show dilated vessels that resemble the cavernous stage of hemangioma
• Vascular malformations of the jaws – Risk of severe bleeding which may occur
spontaneously or during surgical manipulation
11. Sturge–Weber angiomatosis • Nonhereditary developmental condition that is characterized by a hamartomatous
(encephalotrigeminal vascular proliferation involving the tissues of the brain and face
Angiomatosis) • Mutation of GNAQ gene on chromosome 9q21
• Belongs to the group of diseases ‘mother spot diseases’
• Port-wine stain, intracranial convolutional calcifications-leptomeningeal angioma,
congenital glaucoma, exophthalmus, angioma of the choroid, convulsive disorders,
spastic hemiplegia
• Tramline calcifications in radiograph
• Roach scale classification (Type 1 – All features, Type 2 – Leptomeningeal angioma
absent, Type 3 – Port-wine stain and glaucoma absent)

https://t.me/DentalBooksWorld
242 Triumph's Complete Review of Dentistry

12. Hereditary hemorrhagic • Autosomal dominant disorder – Intrinsic defect of the endothelial cells permitting their
telangiectasia detachment or a defect in the perivascular supportive tissue which weakens the vessels
(Osler–Rendu–Weber • Triad – Telangiectasia, recurrent epistaxis, positive family history
syndrome) • Mortality – Multiorgan arteriovenous malformations
• Spider-like telangiectasias present shortly after birth, mostly manifests at puberty
• Increases with age, involvement of the skin occurs on the face, neck and chest, GI tract,
pulmonary vasculature, brain
• Oral cavity – Lips, gingiva, buccal mucosa and palate, floor of the mouth and tongue
• BT and CT are normal – Severe bleeding – Anemia thrombocytopenia
• Treatment: Epistaxis – Pressure packs cautery, X-ray radiation, surgical excision
13. Nasopharyngeal • Vascular malformation rather than a true neoplasm
angiofibroma • Nasopharyngeal angiofibromas occur almost exclusively in males
• Shows a striking predilection for adolescents between the ages of 10 and 17 and often
has been called the juvenile nasopharyngeal angiofibroma
14. Lymphangioma • Lymphangioma simplex (capillary lymphangioma), which consists of small, capillary-
size vessels
• Cavernous lymphangioma, which is composed of larger, dilated lymphatic vessels
• Cystic lymphangioma (cystic hygroma), which exhibits large, macroscopic cystic
spaces
• Lymphangiomas have a marked predilection for the head and neck, which accounts
for 50–75% of all cases
15. Myxoma • Deeply situated, occurs in skin, GI tract, liver, spleen
• Intra oral soft tissue myxoma is rare
• Nerve sheath myxoma – Arises from perineural cells of peripheral nerves and
characterized by occurrence of stellate cells in a mucoid matrix
• Oral focal mucinosis – Oral counter part of cutaneous focal mucinosis
• Loosely arranged tissue with delicate reticulin fibers and mucoid material probably
hyaluronic acid
• Treatment: Excision
16. Chondroma • Benign tumors composed of mature hyaline cartilage
• Chondromas usually arise in the third and fourth decades without a significant sex
predilection
• Chondromas are painless and slowly growing tumors. Tooth mobility and root
resorption are noted occasionally
• Radiographically, chondromas typically appear as radiolucencies with central areas of
radiopacity been found in the condyle
• Multiple chondromas widespread involvement with a tendency to be unilateral is
termed Oilier disease
• Maffucci syndrome is seen in association with soft tissue angiomas
• Benign chondroblastoma (Codman tumor)
17. Chondromyxoid fibroma • Uncommon benign neoplasm accounting for less than 1% of all primary bone tumors
• It is located most commonly in the metaphyseal region of the long bones. Rarely
involves the jaws
• Radiographically, the lesion is a circumscribed radiolucent defect with sclerotic or
scalloped margins
• Lobulated areas of spindle-shaped or stellate cells with abundant myxoid or chondroid
intercellular substance with varying numbers of multinucleated giant cells, focal areas
of calcification, and spicules of residual bone may also be present within the tumor
• Treatment: Curettage; recurrence is uncommon
18. Osteoma • Osteomas are benign tumors composed of mature compact or cancellous bone
• Osteomas of the jaws may arise on the surface of the bone, as a polypoid or sessile
mass (periosteal osteoma) or they may be located in the medullary bone (endosteal
osteoma) detected in young adults and are generally asymptomatic, solitary lesions

https://t.me/DentalBooksWorld
Chapter 4 • Oral Pathology and Oral Medicine 243

• Osteoma in mandibular condyle may cause a slowly progressing shift in the patient's
occlusion, with deviation of the midline of the chin toward the unaffected side. Other
signs and symptoms include facial swelling, pain, and limited mouth opening
• Radiography – Circumscribed sclerotic masses. Periosteal osteomas may show a
uniform sclerotic pattern or may demonstrate a sclerotic periphery with a central
trabecular pattern
• Compact osteomas are composed of normal-appearing dense bone showing minimal
marrow tissue
• Cancellous osteomas are composed of trabeculae of bone and fibrofatty marrow.
Osteoblastic activity may be fairly prominent
• Treatment: Local resection

MALIGNANT CONNECTIVE TISSUE TUMORS


1. Fibrosarcoma • No specific cause, men are more affected, 35–55 years of age, often arises in soft tissue
of thigh and posterior knee
• Sarcomas of bone present with pain and swelling, soft tissue sarcomas are painless
• Types: Well-differentiated. Plump fibroblasts with pale eosinophilic cytoplasm,
intermediate grade
• Tumors are cellular and have the typical herringbone pattern showing the diagnostic
parallel sheets of cells arranged in intertwining whorls, high-grade lesions are very
cellular with marked cellular atypia and mitotic activity), sclerosing epitheloid variant-
uncommon tumor of deep soft tissues
• Treatment: Tumors require radical surgery, including removal of potentially invaded
muscle and bone
2. Miscellaneous locally • Nodular fasciitis: “A benign and probably reactive fibroblastic growth extending as a
aggressive fibrous lesions solitary nodule from the superficial fascia into the subcutaneous fat, or less frequently,
(nodular fasciitis) into the subjacent muscle. Confusion with a sarcoma is possible because of its
pseudosarcomatous cellularity, its mitotic activity, its richly mucoid stroma, and its rapid growth
fibromatosis, aggressive • Aggressive fibromatosis: “A nonmetastasizing tumor-like fibroblastic growth of
fibromatosis (extra unknown pathogenesis involving voluntary muscle as well as aponeurotic and fascial
abdominal desmoid), structures. Histologically, it is indistinguishable from an abdominal fibromatosis. The
proliferative myositis, lesion has a strong tendency to local recurrence and aggressive, infiltrating growth
fibrous histiocytoma • Proliferative myositis: “A rapidly growing, poorly circumscribed, probably reactive
(fibroxanthoma), proliferation of fibroblasts and ganglion cell-like giant cells involving chiefly the
atypical fibroxanthoma, connective tissue framework of striated muscle tissue
desmoplastic fibroma of • Fibrous histiocytoma: “A benign, unencapsulated and often richly vascular growth
bone made up of histiocytes and collagen-producing fibroblast-like cells, which are arranged
in a whorled or cartwheel pattern. Frequently, the growth contains lipid-carrying
macrophages. It may occur anywhere but is most common in the dermis”
• Atypical fibroxanthoma: “A probably benign growth, which is closely related to
fibroxanthoma but shows a much greater degree of cellular pleomorphism with
multinucleated giant cells and occasional giant cells of the Touton type as well as
numerous mitotic figures, including atypical forms
• Desmoplastic fibroma of bone: This is a lesion of bone, including the jaws, which is
histologically indistinguishable from aggressive fibromatosis or the extra abdominal
desmoid. Although there is a wide spread in the age of occurrence, the vast majority of
cases have occurred in the second decade
3. Malignant fibrous • MFH occurs in adults, 50–70 years of age. Men are affected almost twice as frequently
histiocytoma as women
• Moderately firm submucosal mass expanding slowly or moderately fast, with or
without pain and surface ulceration
• The cellular differentiation and density vary markedly, even within the same tumor

https://t.me/DentalBooksWorld
244 Triumph's Complete Review of Dentistry

• The classic histopathologic features, however, include at least mild-cellular and nuclear
pleomorphism, an admixture of fibroblastic and histiocytic elements, and focal areas
with a storiform or cartwheel pattern of streaming spindle cells
• MFH of the oral region is usually treated by radical surgical resection
4. Synovial sarcoma • Resemblance to developing synovial tissue under light microscope
• Arises from pluripotential mesenchymal cells near joint surfaces, tendons, tendon
sheaths, juxta-articular membranes, and fascial aponeuroses
• Third-to-fifth decades of life. Slowly enlarging, deep-seated mass, which is painful in
slightly more than one half of patients, dyspnea, dysphagia, hoarseness, and headache
• Biphasic (epithelioid and spindle cell), monophasic spindle cell, or monophasic
epithelioid
• Radiography – Spotty calcification (snowstorm) within the matrix of the soft-tissue
tumor
• Treatment is wide resection
5. Liposarcoma • Liposarcoma of the head and neck region – Adults
• World Health Organization classification – Five categories of liposarcomas: Well
differentiated, which includes the adipocytic, sclerosing, and inflammatory subtypes,
dedifferentiated, myxoid, round cell, pleomorphic
• The recognition of lipoblasts is the key finding in the diagnosis of liposarcoma.
A  lipoblast has the ability to produce and accumulate nonmembrane-bound lipid
within its cytoplasm
6. Hemangioendothelioma • Chromosomal translocation involving chromosomes 1 and 3 [t(1;3) (p36.3;q25)],
second and third decades, females appear to be affected
• The malignant hemangioendothelioma is similar to the hemangioma in appearance
• Kaposiform hemangioendothelioma, histopathologic admixture of tissues similar to
both capillary hemangioma and Kaposi’s sarcoma
• Polymorphous hemangioendothelioma vascular and angiomatous endothelial areas
• Biphasic proliferation of venous or capillary vessels. There are dilated and congested
veins with inactive endothelial cell nuclei and with occasional thrombi or phleboliths
• Lesional cells of the spindle-cell hemangioendothelioma are rather bland, bipolar
mesenchymal fibroblast-like cells which may contain vacuoles, presumed to be
abortive or primitive vascular lumina
• Treatment – Wide surgical excision
7. Hemangiopericytoma • Hemangiopericytoma is a tumor thought to be derived from pericytes
• Chromosomal translocations t(12;19) and t(13;22) have been observed in lesional
cells – Adults
• Rapidly enlarging red or bluish mass
• Soft or rubbery, sessile or pedunculated, surface lobularity or telangiectasia
• Infantile hemangiopericytoma – Usually multiple and congenital, and often
demonstrates an alarmingly rapid rate of enlargement after birth
• The branching vascular channels of varying sizes is often described as a ‘staghorn’
pattern
• Treatment of hemangiopericytoma is dependent on cellular dysplasia and mitotic
activity
8. Multiple idiopathic • Kaposi’s sarcoma has four major clinical presentations: classic (chronic), endemic
hemorrhagic sarcoma of (lymph-adenopathic; African), immunosuppression-associated (transplant), and
Kaposi (Kaposi’s sarcoma, AIDS-related
angioreticuloendothelioma) • Lymphadenopathic Kaposi’s sarcoma is endemic to young African children and
presents as a localized or generalized enlargement of lymph node chains, including
the cervical nodes
• Transplantation-associated Kaposi’s sarcoma is seen in 1–4% of renal transplant
patients, usually becoming manifested 1 or 2 years after transplantation
• AIDS-related Kaposi’s sarcoma. Approximately 40% of homosexual AIDS patients will
develop Kaposi’s sarcoma, often as an early sign of the disease

https://t.me/DentalBooksWorld
Chapter 4 • Oral Pathology and Oral Medicine 245

• Early oral mucosal sarcomas are flat and slightly blue, red or purple plaques, either
focal or diffuse, become deeply discolored and surface papules and soft nodules
develop, or may become exophytic and ulcerated, and may bleed, usually remaining
less than 2 cm in size
• Proliferation of small veins and capillaries around one or more preexisting dilated
Vessels. Advanced lesions (plaque stage) are nodular and show increased numbers
of small capillaries or dilated vascular channels interspersed with proliferating sheets
of sarcomatous or atypical spindle cells, often with large numbers of extravasated
erythrocytes and abundant hemosiderin deposition
• Treatment: Smaller lesions – Surgically excision, low-dose irradiation, intralesional
chemotherapy and sclerosing solutions, larger and multifocal lesions, systemic
chemotherapy is often effective
9. Ewing’s sarcoma • Ewing’s sarcoma is a small round cell sarcoma of the bone
(endothelial myeloma, • Children and young adults – 5–25 years
‘round cell’ sarcoma) • Pain, usually of an intermittent nature, and swelling of the involved bone are often
the earliest clinical signs and symptoms of Ewing’s sarcoma. Facial neuralgia and lip
paresthesia have been reported in cases of jaw involvement
• Radiography – Destructive, irregular, diffuse radiolucency
• Extremely cellular neoplasm composed of solid sheets or masses of small round cells
with very little stroma, although a few connective tissue septa may be present. Sarcoma
cells are arranged in Filigree pattern. Mitotic figures are common. The cells are positive
for glycogen and are diastase resistant
• Treatment – Radical surgical excision has been done, alone and coupled with X-ray
radiation
10. Chondrosarcoma • Malignant counterpart of the chondroma
• High-grade, fast-growing tumor- excruciating pain. A low-grade, more indolent
• Tumor is likely to be present in an older patient complaining of pain and swelling.
Chondrosarcoma usually has a slow clinical evolution. Metastasis is relatively rare and
often occurs late
• Mesenchymal chondrosarcoma is a characteristic and distinctive type of
chondrosarcoma
• Clear cell chondrosarcoma is a recognized variant of the usual chondrosarcoma –
Slow growth pattern and favorable clinical course with low metastatic potential and
high probability of cure
• Dedifferentiated chondrosarcoma is the most malignant form of chondrosarcoma
• Radiography – Long-standing lesions exhibit considerable destruction of bone
• Cells with plump nuclei, more than an occasional cell with two such nuclei, and
especially, giant cartilage cells with large single or multiple nuclei or with clumps of
chromatin sheets of chondrocytes, which have a lobulated growth pattern under low
power
• Treatment – Surgery
11. Osteosarcoma osteogenic • It is thought to arise from a primitive mesenchymal bone-forming cell and is
sarcoma characterized by production of osteoid
• Higher in males than in females (1.25:1). Osteosarcoma occurs chiefly in young
persons 10–25 years
• Variants of osteosarcoma are: conventional types (i.e., osteoblastic, chondroblastic,
fibroblastic); multifocal; telangiectatic; small cell; intraosseous well-differentiated;
intracortical; periosteal; parosteal; high-grade surface; and extraosseous
• Parosteal (juxtacortical) osteosarcoma is a very uncommon form – Slow growth lower
tendency for metastasis
• Periosteal osteosarcoma – Aggressive variant of parosteal osteosarcoma
• Extraosseous osteosarcoma involving extraskeletal osteosarcoma of soft tissue in the
absence of a primary skeletal tumor occasionally occurs but is rare. Cumulus cloud
densities form within the intramedullary and soft tissue components caused by
mineralizing tumor osteoid

https://t.me/DentalBooksWorld
246 Triumph's Complete Review of Dentistry

• Small streaks of bone radiate outward from approximately 25% of these tumors. This
produces a sunray (sunburst) pattern
• Codman’s triangle – Long bones affected with osteosarcoma, the periosteum is elevated
over the expanding tumor mass in a tent-like fashion. At the point on the bone where
the periosteum begins to merge (edge of the tent), an acute angle between the bone
surface and the periosteum is created
• Proliferation of both atypical osteoblasts and their less differentiated precursors.
Characteristic feature of osteosarcoma is the presence of osteoid formed by malignant
osteoblasts
• Treatment – Radical resection
12. Non-Hodgkin’s Lymphoma • Lymphoproliferative malignancy – Older than 50 years
• Lymphadenopathy is the most common manifestation of lymphoma. Systemic
symptoms like fevers, night sweats, weight loss, and fatigue, pruritus are noticed
• Oral lesions are characterized by swellings which may grow rapidly and then ulcerate
to become large, fungating, necrotic, foul-smelling masses
–– Histologic pattern which is described as either nodular or diffuse.
• Follicular lymphoma (nodular):
–– Two principal cell types are observed in varying proportions: small cells with irregular
or cleaved nuclear contours and scant cytoplasm that are referred to as centrocytes
(small cleaved cells) and larger cells with open nuclear chromatin, several nucleoli,
and modest amounts of cytoplasm that are referred to as centroblasts
–– Treatment – Radiotherapy, chemotherapy (forms the cornerstone of therapy
in lymphoma and has a curative, as well as a palliative role) or biologic therapy
(comprised of interferon therapy and monoclonal antibodies).
13. Primary lymphoma of The etiology of is unknown. Viral agents and immunosuppression are implicated in
bone (primary lymphocytic some cases
lymphoma, reticulum cell • Adults – Male-to-female ratio ranges from 1.5 to 2:1
sarcoma of bone) • The diagnostic criteria (Coley et al., 1950) by WHO are: A primary focus in a single
bone, histologic confirmation, at the time of diagnosis, no evidence of distant soft
tissue or distant lymph node involvement
• More frequent in the mandible than in the maxilla. oral mucosa shows minor change
in the texture or hue, sometimes, appearing diffusely inflamed. The teeth often become
exceedingly loose, owing to destruction of bone
• Laboratory findings – Elevated lactate dehydrogenase (LDH) levels and erythrocyte
sedimentation rate (ESR)
• Since the oral tissues frequently exhibit considerable inflammatory cell infiltration
• Treatment – Radiation, surgical ablation
14. African jaw lymphoma • It is a high-grade B-cell neoplasm and has two major forms: the endemic (African)
(Burkitt’s lymphoma) form and the nonendemic (sporadic) form
• Childhood tumor but it is observed in adult patients too rapid growth may manifest
significant metabolic derangement and renal function impairment. Less common
presentations of include an epidural mass, skin nodules, CNS symptoms, and bone
marrow involvement. Rare cases of Burkitt’s lymphoma can present as acute leukemia
(L3-ALL) with fever, anemia, bleeding, and adenopathy
• Burkitt’s lymphoma is a monoclonal proliferation of B lymphocytes characterized
by small noncleaved cells that are uniform in appearance and that produce a diffuse
pattern of tissue involvement
• Characteristic starry sky appearance is imparted by scattered macrophages with an
abundant clear cytoplasm, often containing phagocytic cellular debris
• Treatment – Chemotherapy and CNS prophylaxis (intrathecal chemotherapy)
15. Hodgkin’s disease • The etiology of HD is unknown. Infectious agents, Epstein–Barr virus (EBV)
(Hodgkin’s lymphoma, • Painless enlargement of one or more cervical lymph nodes
malignant lymphoma) • Types – Nodular sclerosis, mixed-cellularity, lymphocyte-depleted, lymphocyte-rich,
nodular lymphocyte-predominant

https://t.me/DentalBooksWorld
Chapter 4 • Oral Pathology and Oral Medicine 247

• Nodular sclerosis (NS) Hodgkin’s disease comprises 60–80% of all cases. The
morphology shows a nodular pattern
• Mixed-cellularity Hodgkin’s disease comprises 15–30%. Histologically, the infiltrate is
usually diffuse. Reed Sternberg cells (large, bilobate, double or multiple nuclei, large
nucleolus)
• Lymphocyte-depleted Hodgkin’s disease makes up less than 1%. The infiltrate
in lymphocyte-depleted Hodgkin disease (LDHD) is diffuse, hypocellular. Large
numbers of RS cells and bizarre sarcomatous variants
• Lymphocyte-rich classic Hodgkin’s disease comprises 5%. RS cells of the classic or
lacunar type, with a background infiltrate of lymphocytes
• Nodular lymphocyte-predominant Hodgkin’s disease constitutes 5%. In contrast to the
other histological subtypes, the typical RS cells in nodular lymphocyte-predominant
Hodgkin disease are not observed or appear infrequently
• Variant of RS cells, the lymphocytic and histiocytic cells (L and H), or popcorn cells
(their nuclei resemble an exploded kernel of corn)
• Treatment – Radiation therapy and combination chemotherapy
16. Multiple myeloma • Monoclonal malignancy of plasma cells. Expansion of a single line of plasma cells that
replace normal bone marrow and produce monoclonal immunoglobulins
• Older people. Common in men
• Axial skeleton and include the vertebral column, ribs, skull, pelvis, and femur bone.
Lytic foci or diffuse demineralization at the time of diagnosis
• Bone pain, especially from compression fractures of vertebrae or ribs, is the most common
symptom. Lytic bone lesions, anemia, azotemia, hypercalcemia, and recurrent infections
• Extramedullary plasmacytoma. Patients with isolated plasma cell tumors of soft
tissues, most commonly occurring in the tonsils, nasopharynx, or paranasal sinuses,
should have skeletal X-rays and bone marrow biopsy
• Radiographic features – Sharply punched-out areas in vertebrae, ribs, skull, jaws, and
ends of long bones
• Composed of sheets of closely packed cells resembling plasma cells. Round or ovoid
cells with eccentrically placed nuclei exhibiting chromatin clumping in a ‘cartwheel’ or
‘checkerboard’ pattern
• Russell bodies are common as in chronic inflammatory lesions with numerous typical
plasma cells
• A Bence Jones protein is a monoclonal immunoglobulin light chain found in the urine,
molecular weight of 22–24 kDa. Present in multiple myeloma and Waldenström’s
macroglobulinemia
17. Leiomyosarcoma • The leiomyosarcoma is a malignant tumor of smooth muscle origin
• Middle-aged or older individual
• Painful, lobulated, fixed mass of the submucosal tissues, fascicles of interlacing spindle-
shaped cells with abundant, eosinophilic cytoplasm and moderately large, centrally
located, cigar-shaped, or blunt-ended nuclei, often with mild atypia
• Epithelioid variant, called malignant leiomyoblastoma or epithelioid leiomyosarcoma,
is most prevalent in the gastrointestinal and genitourinary tracts
• Treatment – Radical surgery with adjunctive chemotherapy or radiotherapy
18. Rhabdomyosarcoma • Four types of rhabdomyosarcoma based pleomorphic, alveolar, embryonal, botryoid
• Embryonal rhabdomyosarcoma – Most common subtype observed in children
• Botryoid rhabdomyosarcoma (sarcoma botryoides) malignant tumor of the vagina,
prostate, and bladder in young children, maxillary sinus, nasopharynx, common bile
duct and middle ear. Characteristic feature is a peripheral zone of increased cellularity-
‘cambium layer’
• Alveolar rhabdomyosarcoma: extremities, approximately 18% were found in the head
and neck region
• Pleomorphic rhabdomyosarcoma (least common of all rhabdomyosarcoma): seen
in older individuals, racquet’ cell. ‘Strap’ and ‘ribbon’ cells with processes of long
streaming cytoplasm

https://t.me/DentalBooksWorld
248 Triumph's Complete Review of Dentistry

• Divergence of an eye, abnormal phonation, dysphagia, cough, aural discharge or


deviation of the jaw
• Ulcerated and invade underlying bone and develop distant metastases
• Eosinophilic spindle cells, usually arranged in interlacing fascicles. Round eosinophilic
cells, large and intermediate in size, with a small nucleus and a granular eosinophilic
cytoplasm
19. Alveolar soft-part sarcoma • Alveolar soft-part sarcomas are myogenic, females, early 20s
(malignant granular cell • Muscles of the extremities, tongue, floor of the mouth
myoblastoma) • Composed of large cells with a finely granular cytoplasm, large and polygonal with
a distinct cell border, a vesicular nucleus, dense, abundant granular, eosinophilic, or
vacuolated cytoplasm
• Uniform pseudoalveolar or organoid pattern, numerous delicate endothelium-lined
vascular channels and septa
• Treatment – Radical surgical excision
20. Malignant peripheral nerve • Malignant peripheral nerve sheath tumor (MPNST) is spindle cell malignancy of
sheath tumor peripheral nerve Schwann cells
(malignant schwannoma, • 20–50 years of age
malignant neurilemmoma, • Tongue or soft palate, lip, gingiva, palate and buccal mucosa, central tumors – Mandible
neurogenic • Diffuse radiolucency characteristic of a malignant infiltrating neoplasm
sarcoma, • Spindle-shaped lesional cells, wavy or comma-shaped outline, Spindle cells are
neurofibrosarcoma) arranged in sweeping
• Fascicles interspersed with hypocellular and myxoid regions
• Epithelioid, mesenchymal, or glandular
• Triton tumor – MPNST lesions show rhabdomyoblastic differentiation
• Treatment – Wide surgical excision, resistant to radiotherapy and chemotherapy
21. Olfactory neuroblastoma • Rare tumor originating from the olfactory apparatus
(esthesioneuroblastoma, • Occurs in adults rather than children
esthesioneuroepithelioma) • Painful swelling in the area of the nasal fossa, invasive, destructive tumor
• Densely packed masses of small darkly staining cells each with a poorly defined
eosinophilic cytoplasm and a regular round vesicular nucleus, sometimes with stippled
chromatin. Rosette formation is common
• Pseudoglandular structure lined by a single layer of nonciliated columnar cells with
a basal nucleus and a cuticular border at the apex of the cell – Sustentacular and
olfactory cells of the olfactory mucosa. Pseudorosettes also occur. The stroma has a
fibrillar neuroid pattern
• Treatment is surgery, radiation, or both in combination
22. Metastatic tumors of jaws • Metastatic tumors to the oral region are uncommon 40–70 years
• The most common primary sources of metastatic tumors to the oral region are cancers
in the breast, lung, kidney, bone, or colorectum
• The breast is the most common primary site for tumors that metastasize to the
jawbones, whereas the lung is the most common source for cancers that metastasize
to the oral soft tissues
• Gingival metastasis resembles hyperplastic or reactive lesions (e.g., pyogenic granuloma,
peripheral giant cell granuloma, fibrous epulis) and in other oral soft tissue locations,
especially in the tongue, the metastatic lesion manifests as a submucosal mass
• Most of such metastases produce osteolytic lesions
• Clue to the diagnosis is the resemblance of the metastasis to the primary tumor
• Treatment modalities are limited to palliation

DENTAL CARIES
S. No. Category Classification
1. Based on location • Pit and fissure
• Smooth surface
• Root surface

https://t.me/DentalBooksWorld
Chapter 4 • Oral Pathology and Oral Medicine 249

2. Based on severity • Incipient


• Rampant
• Arrested
• Recurrentw
• Radiation
3. Age pattern • Nursing bottle caries
4. Rapidity • Acute
• Chronic

S. No. Name of the theory Person and year Concept


1. Chemico-Parasitic Miller 1882 Caries is a chemicoparasitic process consisting of decalcification of
theory – Acidogenic enamel and dentin followed by dissolution of the softened residue
theory
2. The Proteolytic Gottlieb, 1944; Caries formation is a proteolytic process involving acid production
theory Diamond and of microbes and subsequent proteolysis
Applebaum, 1946
3. The Proteolysis – Schatz et al., 1955 Microbial degradation of organic components and dissolution of
Chelation theory inorganic components by chelation
4. Sucrose – Chelation Eggars and Lura, Sucrose acts a chelating agent to form calcium saccharate causing
theory 1948, 1968 removal of calcium from enamel
5. Sulfatase theory Pincus, 1950 Microbes produce sulfatase that hydrolyses the mucoitin sulfate of
enamel and chondroitin sulfate of dentin resulting in formation of
sulfuric acid and decalcification
6. Phosphate Eggars and Lura, 1967 Utilization of phosphate from enamel by bacteria
Sequestration theory
7. Bacterial Kreitzman et al., 1969 Alkaline phosphatase from bacteria acts on enamel to release
Phosphatase theory phosphate from enamel
8. Autoimmune theory Jackson and Burch, Autoimmune reaction in specific sites of odontoblasts makes dentin
1970 vulnerable to damage
9. Genetic theory Bachrach and Young, Genetic predisposition to dental caries
1927; Horowitz, 1958

Factors that Influence in Caries Etiology

Host factors Components Features


Microbial flora Pit and fissure Streptococcus mutans, Streptococcus sanguis, Lactobacillus sp., Actinomyces sp.
Types Smooth surface Streptococcus mutans, Streptococcus salivarius
Root surface Actinomyces viscosus, Actinomyces naeslundii, Streptococcus mutans, Streptococcus
sanguis
Deep dentinal Lactobacillus sp., Actinomyces naeslundii and filamentous rods
caries
Microbial Streptococcus Forms extracellular polymers glucans and fructans from glucose and fructose by
metabolism mutans the action of enzyme glucosyl transferase and fructosyl transferase
• Glucans have two homopolymers – Dextran and mutan
• Fructan homopolymer – Levan
Lactobacilli Acidogenic and aciduric, present in deeper lesions
Actinomyces Form extracellular levans and heteropolysaccharides hexosamine and hexose
Veillonella Utilizes lactic acid and converts it into propionic acid

https://t.me/DentalBooksWorld
250 Triumph's Complete Review of Dentistry

Plaque Supragingival plaque gives rise to caries


Bacterial adherence – Adhesins to saccharide receptors
Lectins bind to specific sugars
Tooth Composition The enamel of sound teeth contains 0.0111 ± 0.0020% fluoride, while that of carious
Morphologic teeth contains 0.0069 ± 0.0011% fluoride
characteristics Deep, narrow occlusal fissures or buccal or lingual pits – Tends to trap food – Leads
Position to caries
The most susceptible permanent teeth are the mandibular first molars, closely
followed by the maxillary first molars and mandibular and maxillary second molars
Mandibular first molar – Surface involved in decay in descending order – Occlusal–
Buccal–Mesial–Distal–Lingual areas
The order in maxillary first molar is – Occlusal–Mesial–Lingual–Buccal–Distal
Malaligned, out of position, rotated teeth tend to develop caries
Saliva Composition The phosphate concentration in saliva tends to fall as the flow rate of saliva increases,
Inorganic while the calcium concentration falls initially but then rises to higher flow rates
Organic
pH This is due to the associated increase in pH at high flow rates
Quantity Despite the continuous flow of the saliva, dental plaque can accumulate at a rapid
rate of 10–20 mg/day in the absence of oral hygiene procedures, but the rate of
plaque accumulation appears to be even more rapid in patients with xerostomia
Viscosity Increase viscosity increases caries susceptibility
Antibacterial Lysozyme (N-acetylmuramide glycanohydrolase)
factors Lysozyme activity is significantly greater in a group of caries – Free preschool
children than caries – Susceptible group
Salivary peroxidase system
This antibacterial system is known to be inhibitory towards L. acidophilus and
S. cremoris, by preventing cells from accumulating lysine and glutamic acid, which
are essential for growth
Immunoglobulins
The predominant immunoglobulin class in saliva is secretory IgA or sIgA
Diet Physical factors Those foods that lacks roughness – Softer foods tends to stick to tooth surface and
cause caries
Quality of diet Physical form of the diet determines caries prevalence. Increased fibers in the diet
lead to decreased retention on tooth surface
Carbohydrate Carbohydrate is primary factor for caries in which it includes sucrose and fructose,
content and xylitol is not metabolized by plaque microorganisms and it does not cause caries
Vitamin content Vit K is an anticaries agent because of its enzyme-inhibiting activity in carbohydrate
degradation cycle
Vit B6 (pyridoxine) has been proposed as an anti caries agent because is selectively
alters the oral flora by promoting the growth of noncariogenic organisms
There is no relation between vit C deficiency and incidence of caries
There is no relation between vit K deficiency and incidence of dental caries
Fluoride content Drinking water fluoride is more metabolically active than dietary fluoride
Systemic conditions Hereditary There is still no indisputable evidence that hereditary per se has a definite relation
Pregnancy and to dental caries incidence
lactation The possibility exists that if there is any such relation, it may be mediated through
inheritance of tooth form or structure, which predisposes to caries immunity or
susceptibility
There is no relation between pregnancy and caries activity, though in the later stages
of pregnancy or after delivery, there will be a significant increase in caries activity

https://t.me/DentalBooksWorld
Chapter 4 • Oral Pathology and Oral Medicine 251

International Caries Detection and Assessment System (ICDAS) Criteria


• For use on coronal and root surfaces, as well as caries adjacent to restorations and sealants
• Unifying, predominantly visual, criteria code a range of the characteristics of clean, dry teeth in a consistent way that promotes
the valid comparison of results between studies, settings, and locations
• ICDAS criteria record both enamel and dentine caries and explore the measurement of caries activity

Detection Codes:
0 Sound
1 First visual change in enamel (seen only after prolonged air drying or restricted to within the confines of a pit or fissure)
2 Distinct visual change in enamel
3 Localized enamel breakdown (without clinical visual signs of dentinal involvement)
4 Underlying dark shadow from dentin
5 Distinct cavity with visible dentin
6 Extensive distinct cavity with visible dentin

Histopathologic Zones of Enamel Caries


The following four histological zones of an enamel lesion clearly explain the development of enamel caries:
Zones Features
Translucent zone • First recognizable zone of alteration
(TZ) • Advancing front of the lesion
• Half of the lesions demonstrate TZ, not always present
• TZ appears structureless
• Pore volume – 1% (compared to 0.1% of sound enamel)
Dark zone • Lies adjacent and superficial to the translucent zone
• Positive zone
• Shows positive birefringence Pore volume – 2–4% (polarized light)
• Presence of small pores; large molecules of quinoline are unable to penetrate
• Micropore system – Gets filled with air and becomes dark
• Mediums like water may penetrate
Body of the lesion • Between unaffected, surface and dark zone
• Area of greatest demineralization
• Pore volume – 5% in periphery and 25% in center
• Quinoline imbibition – Body appears transparent and helps to detect the translucent zone
(quinolone has a refractive index similar to that of enamel)
• Water imbibition – Positive birefringence compared to sound enamel
• Striae of retzius – Prominent
Surface zone • Quantitative studies – Partial demineralization of 1–10%
• Pore volume – Less than 5% of the spaces
• Negative birefringence – Water imbibition
• Positive birefringence – Porous subsurface
• All the four zones of enamel caries cannot be seen with the same immersion medium

Histopathologic Zones of Dentinal Caries


Zone 1 Zone of fatty degeneration of Tome’s fibers (next to pulp)
Zone 2 Zone of dentinal sclerosis, deposition of Ca salts in the tubules
Zone 3 Zone of decalcification of dentin
Zone 4 Zone of bacterial invasion
Zone 5 Zone of decomposed dentin due to acids and enzymes

https://t.me/DentalBooksWorld
252 Triumph's Complete Review of Dentistry

Advanced dentinal caries • Decalcification of the walls of the tubules leading to confluence of the tubules
• Thickening of sheath of Neumann
• Accumulation of microbes leads to increase in diameter of dentinal tubules
• Liquefaction foci
• Necrotic mass of dentin with a leather-like consistency
• Transverse cleft formation
• Stripping of carious dentin

PULPITIS
1. Focal reversible • Symptoms: Sensitive to cold
pulpitis • Sign: Application of ice to the tooth results in pain, pain disappears after removal of thermal
stimuli, responds to low levels of current from a pulp tester
• Dilatation of the pulp vessels, edema fluid that collects during the inflammatory process causes
damage to the capillary walls. This leads to extravasation of RBCs and diapedesis of WBCs
• Mostly reversible, removal of irritant before the pulp is severely damaged
2. Acute pulpitis • Symptom: Severe pain with thermal changes like taking ice cold drinks
• Persistence of pain after removal of thermal stimuli, poor localization of pain, and may be felt
on the upper or lower jaw of the affected side – As pulp of individual tooth is not represented
precisely in the sensory cortex, pain may become severe as larger proportion of pulp becomes
involved, pain lasts for 10–15 minutes and increases when the patient lies down
• Vascular dilatation seen in focal reversible pulpitis along with accumulation of edema fluid in
the connective tissue. Layer of odontoblasts is usually lost. Rise in pressure due to inflammatory
exudate leads to local collapse of the venous system part of the circulation
• Local tissue hypoxia and anoxia leads to localized destruction and formation of intrapulpal
abscess containing pus
• Pus arises from breakdown of leucocytes and tissue digestion. This necrotic zone contains
leucocytes and histiocytes
• Treatment total or partial pulpectomy
3. Chronic pulpitis • Sequel of acute pulpitis – When the irritant associated with acute pulpitis is not severe (low
virulence)
• Anachoretic pulpitis (in normal teeth not affected by caries)
• Pain is not prominent, mild, dull ache which is intermittent. Reaction to thermal changes is
reduced because of degeneration of nerves. Response to pulp vitality tester is reduced
• Wide open carious lesion and with exposure of pulp cause relatively little pain
• Infiltration of mononuclear cells, lymphocytes and plasma cells, with vigorous connective tissue
reaction. Capillaries are prominent; fibroblastic activity and collagen fibers in bundles
• Pulpal reaction vacillating between an acute and chronic phase causes pulp abscess formation,
which is surrounded by fibrous CT wall, which is called pyogenic membrane
• Treatment: Root canal therapy, extraction of tooth
4. Chronic • Children and young adults with high degree of tissue resistance and reactivity respond to
hyperplastic proliferative lesions
pulpitis • Pinkish red mass protruding from chamber extends beyond caries
(pulp polyp) • Most commonly affected are deciduous molar and first permanent molars
• Pulp is relatively insensitive because few nerves in hyperplastic tissue. Lesion bleeds profusely
upon provocation
• Hyperplastic tissue is basically granulation tissue, consisting delicate CT fibers and young blood
capillaries. Stratified squamous type epithelial lining resembles oral mucosa with well-formed rete pegs
• Grafted epithelial cells are believed to be desquamated epithelial cells, which are carried by saliva
• Treatment: Extraction of tooth or pulp extirpation

https://t.me/DentalBooksWorld
Chapter 4 • Oral Pathology and Oral Medicine 253

PERIAPICAL LESIONS
1. Apical • The products of bacterial invasion and pulp necrosis initiate most periapical diseases
periodontitis • Pain is the presenting symptom, it may be provoked by percussion. affected tooth is higher than
nearby one, pain diffuses
• Radiography: Appears normal except for widening of PDL space
• Localized acute inflammatory exudate with dilated capillaries, and neutrophils
2. Chronic apical • Low-grade pulpitis, infection through deep periodontal pocket, trauma, hematogenous infection,
periodontitis deep-seated restoration
(periapical • Tooth involved is nonvital/slightly tender on percussion. Percussion may produce dull sound
granuloma) instead of metallic due to granulation tissue at apex. Mild pain on chewing on solid food. Tooth
may be slightly elongated in socket
• Sensitivity is due to hyperemia, edema, and inflammation of PDL
• Thickening of PDL at root apex. Bone resorption and proliferation of granulation tissue appears
to be radiolucent area. Thin radiopaque line or zone of sclerotic bone sometimes seen outlining
lesion. Long-standing lesion may show varying degrees of root resorption
• Early – Slight widening of the periodontal ligament space, Late – Well circumscribed radiolucency
of varying size
• Granulation tissue mass – Proliferating fibroblasts, immature blood capillaries with bone
resorption, swollen endothelial cells, chronic inflammatory cells (plasma cells and lymphocytes).
Lymphocytes produce IgG, IgA, IgM, and IgE
• T lymphocytes produce cytotoxic lymphokines, collagenase, and destructive lymphokines
• Scattered hyaline bodies (pulse granuloma giant cell hyaline angiopathy), which appear as small
circumscribed pools of eosinophilic material that exhibit a corrugated periphery of condensed
collagen, often surrounded by lymphocytes and multinucleated giant cells. Russell bodies or
pyronine bodies (clusters of lightly basophilic particles) may be associated with the plasmacytic
infiltrate; both are plasma cell products but are not specific for periapical granuloma. Cholesterol
clefts with multinucleated giant cells, red blood cells, and areas of hemosiderin pigmentation
• Treatment: Extraction and RCT with/without apicoectomy
3. Periapical abscess • Acute exacerbation of chronic lesion – Phoenix abscess
(alveolar abscess, • Due to pulp infection, traumatic injury, pulp necrosis, irritation of periapical tissues (endo procedures),
alveolar abscess) chemicals during root canal treatment, area of chronic infection on periapical granuloma
• Features of acute inflammation, tenderness of tooth, which relives after pressure application.
Extreme painful tooth extrudes from socket. Systemic manifestations like lymphadenitis and
fever may present
• Extension to bone marrow spaces produces osteomyelitis, but clinically considered as dento-
alveolar abscess – Swelling of tissues may penetrate the buccal or lingual cortical plates and
expand into the surrounding soft tissues
• Radiography: Slight thickening of PDL space. Radiolucent area at apex of root if the abscess
develops from a granuloma
• Disintegrating polymorphonuclear neutrophils surrounded by viable leukocytes, lymphocytes,
cellular debris, necrotic materials and bacterial colonies. Dilation of the blood vessels in the
periodontal ligament and adjacent marrow spaces of the bone
• Treatment: Drainage must be established by opening the pulp chamber or extracting the tooth

BACTERIAL INFECTIONS
1. Syphilis (lues) • Causative agent – Spirochete – Treponema pallidum
• Primary syphilis – Chancre, 3–90 days after exposure, penis and vulva-oral chancre is
painless and covered by greyish pseudo membrane but may become painful because of
secondary infection
• Secondary syphilis: (Metastatic stage) Begins 6 weeks after primary lesion, multiple on skin
and mucus membranes. Mucus patches are painless and highly infectious. Patches fuse and
form a serpentine or snail-track ulcers. Split papules – Mucous patches in commissures
of lips. Condyloma lata – Papillary lesions that resemble viral papillomas – Arise in the
genital or anal regions in immunocompromised patients, it is present as lues maligna

https://t.me/DentalBooksWorld
254 Triumph's Complete Review of Dentistry

• Tertiary syphilis: After a latent period of 1–30 years, involves CVS and CNS. Gumma,
causes perforation of the palate. Gumma occurs as a result of hypersensitivity reaction
between host and the breakdown products of the organism. Tongue and palate are
usually involved intraorally. Interstitial glossitis – Result of contracture of lingual
musculature after healing of gummas. Leutic glossitis or syphilitic glossitis is atrophic
or interstitial glossitis due to tongue involvement by treponema. It has potential for
malignant transformation. It is due to endarteritis obliterans
• Congenital syphilis is Hutchinson’s triad, which includes Mulberry molars and notched
incisors, interstitial keratitis, 8th nerve deafness
• Treatment: Penicillin is the drug of choice
2. Sarcoidosis • Causative agent – Mycobacterium and Propionibacterium
• Young adults, common in lungs, skin, lymph nodes, spleen, salivary glands and bones,
depressed type IV hypersensitivity. Multiple, raised red patches that occur in groups,
grow slowly and do not tend to ulcerate or crust
• Presence of nests of epitheloid cells with multinucleated giant cells in noncaseating
granulomas. The granulomas transforms into a solid, amorphous, eosinophilic hyaline
mass as it ages
• Kveim–Siltzbach intracutaneous test is an important aid in the early and accurate
diagnosis of the disease
3. Actinomycosis • Causative agent – Gram-positive anaerobic, microaerophilic, nonacid fast, branched
(lumpy jaw) filamentous organisms, mainly A. israelii
• Multiple abscess which drain through skin and the pus contains typical sulfur granules.
Wooden indurated area of fibrosis. Actinomycotic osteomyelitis occurs in mandible and
maxilla – Colonies show peripheral radiating filaments because of which actinomycosis
is also known as sulfur granules. Peculiar form of colonies, with the peripheral radiating
filaments, is the basis for the often used term “Ray Fungus”
• Treatment: Draining of abscess with sinus tract excision along with high-dose antibiotic
4. Tetanus (lock jaw) • Causative agent – Clostridium tetani
• Generalized tetanus: Lock jaw or trismus due to spasm of masseter which is an initial
symptom, dysphagia, stiffness or pain in the neck, shoulder or back muscles appears
concurrently. Sustained contraction of facial muscles results in the grimace or sneer
called risus sardonicus. The contraction of muscles of the back produces an arched back
called opisthotonus
• Localized tetanus: Spasm of muscles near the wound, cephalic tetanus characterized by
trismus and facial palsy after head injury
• Treatment: Antitoxin is injected to neutralize the circulating toxins – Human tetanus
immunoglobulin (TIG) 3000–6000 units IM
• For prophylaxis, anti-tetanus serum 1500 units or TIG 250 units should be given
5. Impetigo • Causative agent – S. pyogenes and S. aureus
• Scabbing eruption. Nonbullous impetigo- (impetigo contagiosa)-macules/papules with
subsequent development of fragile vesicles –rupture to form thick amber crusts – Corn
flakes glued to surface, pruritis, lymphangitis, cellulitis. Bullous impetigo – Superficial
vesicles enlarge to form large bullae-rupture to form a thin brown crust called – Lacquer
6. Erysipelas (Saint • Causative agent – Beta hemolytic streptococci (group A S. pyrogens, occasionally by
Anthony’s fire) group C, B G)
• Facial erysipelas produce butterfly-like rash resembling lupus erythematosus, eyelids are
edematous, affected skin – Peau d’orange, high fever with lymphadenopathy, increased
vasculature
• Treatment of choice – Penicillin, alternatively macrolides, cephalosporins, and
fluoroquinones
7. Streptococcal • Causative agent – Group A, beta hemolytic streptococci, adenovirus, influenza,
tonsillitis and parainfluenza, EBV
pharyngitis • Group A streptococci are responsible for 20–30% of acute pharyngitis in children and
5–15% in adults

https://t.me/DentalBooksWorld
Chapter 4 • Oral Pathology and Oral Medicine 255

• Tonsillitis, pharyngitis, dysphagia, tonsillar hyperplasia, fever, cervical lymphadenopathy,


beefy red and swollen uvula, conjunctivitis, coryza, cough, hoarseness, absence of fever,
viral exanthema
• Sudden onset of sore throat, temperature of 101°–104°F, dysphagia, tonsillar
hyperplasia, redness of oropharynx and tonsils, palatal petechiae, cervical
lymphadenopathy, yellowish tonsillar exudate, “Beefy red”, swollen uvula,
scarlatiniform rash
• Treatment – Penicillin V or amoxicillin. In penicillin allergy – Azithromycin,
clindamycin, cephalosporins and macrolides
8. Scarlet fever • Causative agent – Beta hemolytic streptococci
(scarlatina) • Initial lesion – White strawberry tongue
• 4–5th day – Red strawberry tongue
• Body temperature peaks approximately 103°F and returns normal in 6 days, classic rash of
scarlet fever-sunburn with goose pimples, Pastia’s Lines – Transverse red streaks in the skin
folds secondary to capillary fragility, skin of trunk and extremities – Sandpaper texture
• Treatment – Penicillin V or amoxicillin. In penicillin allergy cases – Azithromycin,
clindamycin, cephalosporins, and macrolides
9. Tonsillar • Convoluted crypts of tonsil are filled with desquamated keratin and debris which are
concretions and secondarily infected with bacteria – Tonsillar concretions. Condensed necrotic debris
tonsillolithiasis undergoes dystrophic calcifications-Tonsillolith
10. Cat-scratch • Causative agent – Bartonella henselae
disease (benign • Disease arises after a traumatic break in skin due to scratch or bite of a household cat
lymphoreticulosis, • Characterized by papule, pustule, and vesicle at the site of injury followed by regional
benign nonbacterial lymphadenitis
regional • Epitheloid cells and multinucleated giant cells are occasionally seen
lymphadenitis) • Oculoglandular syndrome of Parinaud is unusual manifestation of this disease. It
consists of localized granuloma of the eye with peri-auricular lymphadenopathy. The
lymph nodes gradually become soft and fluctuant, owing to necrosis and suppuration
11. Diphtheria • Causative agent – Klebs-Löffler Bacillus – C. diphtheriae
• Diptheric membrane which usually begins on tonsils – Yellow–white, thin film that
thickens to form an adherent gray covering. Peculiar nasal twang and nasal regurgitation
of liquids. Palatal perforation temporary paralysis of soft palate during third and fifth
weeks of disease
• Evidence of diphtheritic membrane
12. Gonorrhea • Causative agent – N. gonorrhoeae
• Sexually transmitted disease
• Urethral infections, purulent discharge, dysuria, increased vaginal discharge,
intermenstrual bleeding, pelvic inflammatory disease. Myalgia, arthralgia, polyarthritis,
dermatitis. Necrotizing ulcerative gingivitis, fetor oris, mandibular or cervical
lymphadenopathy, gonococcal opthalmia neonatorum – Blindness
• NAATs (nucleic acid amplification test, amplifies and detect N. gonorrhoeae-specific
DNA or RNA sequence
• Prophylactic opthalmic erythromycin, tetracycline or silver nitrate is applied to newborn’s
eye to prevent occurrence of gonococcal ophthalmia neonatorum
13. Tuberculosis • Causative agent – Mycobacterium tuberculosis
• Primary TB of skin is called lupus vulgaris, regional lymphadenopathy involving
submandibular and cervical lymph nodes. Scrofula is another name for tuberculous
lymphadenopathy
• Primary oral TB usually involves gingiva and is present as diffuse, nodular or papillary
proliferation of gingival tissues
• Secondary TB – Tongue is most commonly affected followed by palate. Superficial or
deep, irregular painful ulcer. Tuberculous osteomyelitis may occur
• Miliary TB or extrapulmonary TB-blood borne spread of primary TB

https://t.me/DentalBooksWorld
256 Triumph's Complete Review of Dentistry

• Oral lesions of tuberculosis are called lepromas which develop on tongue, lip, and hard
palate. They show a tendency to breakdown and ulcerate
• Potts disease – Tuberculous infection resulting in spinal curvature
• Foci of caseation necrosis, surrounded by epithelioid cells, lymphocytes, multinucleated
giant cells
14. Leprosy (Hansen • Causative agent – Mycobacterium leprae
disease) • Tuberculoid – In patients with high immune reaction, nerve involvement → anesthesia
of affected skin, loss of sweating
• Lepromatous – In patients with reduced immunity, commonly occurs in face, →
Leonine facies (facial destruction)
• Multibacillary – Numerous, ill-defined, hypopigmented macules or papules on the skin
– Leonine facies, Facies leprosa
• Lepra cells – Improper granulomas with sheets of lymphocytes with histiocytes

FUNGAL INFECTIONS
Candidiasis (moniliasis) • Causative agent – C. albicans
• Other Candida spp.: C. tropicalis, C. parapsilosis, C. lusitaniae, C. krusei, C. glabrata,
C. guilliermondii, and C. dubliniensis
• Acute, angular cheilitis, cheilocandidiasis
• Chronic mucocutaneous, diffuse, familial, endocrinal
• Candidosis associated with immunodeficiency
• Treatment – Nystatin oral suspension 100,000 units/ml – Amphotericin B 100 mg/ml
Ketoconazole 200 mg tabs (1st day 2/day; later 1/day)
Acute pseudomembranous • White curd‐like or cottage cheese lesions
candidiasis (oral thrush) • Can be easily wiped off, underlying mucosa normal or erythematous ± bleeding, buccal
mucosa, tongue, soft plate and floor of the mouth, symptoms: burning sensation,
unpleasant taste
• Concomitant involvement of oral cavity and esophagus is common in HIV patients
Acute atrophic (erythematous) • Red macules with burning sensation, due to prolonged or continuous antibiotic therapy
candidiasis (antibiotic stomatitis) xerostomia, immunosuppression
• Location: Posterior hard palate, buccal mucosa, dorsal tongue
Chronic atrophic candidiasis • Common in females, associated with diabetes mellitus
(denture sore mouth) • Usually confined to the denture‐bearing areas
• Appears as a red lesion in the hard palate area and granular; whitish nodules may be present
Median rhomboid glossitis • Central papillary atrophy of the tongue/posterior atrophic candidiasis
(kissing Lesion) • Well‐demarcated erythematous zone in the midline of dorsal part of the tongue, posteriorly
• Asymptomatic, predisposing cause: immunosuppression
Chronic multifocal candidiasis • Oral mucosal candidal infection at multiple sites
• Erythematous areas + white plaques
• Burning sensation
• Posterior palate, posterior dorsal tongue, angles of the mouth
Chronic hyperplastic candidiasis • Buccal mucosa, dorsal tongue, commissures
leukoplakia type of candidiasis • Well‐defined, nonscrapable thick indurated lesion
• May undergo dysplastic changes if left untreated (pre‐malignant lesion) – mainly
speckled leukoplakia
• Associated with iron and folate deficiency and defective CMI
Angular cheilitis • Common in elderly with decreased vertical dimension of occlusion and accentuated
(perlèche) folds at the corners of the mouth
• Miconazole –2% cream

https://t.me/DentalBooksWorld
Chapter 4 • Oral Pathology and Oral Medicine 257

Deep Fungal Infections

1. Histoplasmosis (Darling • Causative agent – Histoplasma capsulatum – Airborne infection


disease) • Acute histoplasmosis: fever, headache, myalgia, nonproductive cough, anorexia.
Calcification of the hilar lymph nodes may be present
• Chronic histoplasmosis – Less common, similar to TB, chest roentgenograms show upper
lobe infiltrates and cavitation. Disseminated histoplasmosis-spread to extrapulmonary
sites. Common in HIV patients. May produce hypoadrenocorticism (Addison’s disease)
• Oral lesions common – Tongue, palate, and buccal mucosa. Ulcerated lesions have firm,
rolled margins and they may be indistinguishable clinically from malignancy
• Self-limiting no specific treatment
2. North American • Causative agent – Blastomyces dermatitidis
blastomycosis • Male predilection, acute – resembles pneumonia, life-threatening adult respiratory
Gilchrist Disease distress syndrome
• Chronic-more common, mimic TB
• Symptoms – low-grade fever, night sweats, weight loss and productive cough. Oral lesions
may result from extrapulmonary dissemination or local inoculation with the organism
• Identification of the organisms is especially important because it induces a benign reaction
of the overlying epithelium in mucosal or skin lesions called pseudoepitheliomatous
(pseudocarcinomatous) hyperplasia. Because this benign elongation of the rete ridges
resemble squamous cell carcinoma. Resembles chronic granulomatous infection showing
occasional giant cells and macrophages
• No treatment required. In severe cases systemic amphotericin B
3. South American • Causative agent – Paracoccidioides brasiliensis
blastomycosis Male predilection, may cause Addison’s disease
(Lutz disease, • Oral lesions appear as mulberry-like ulcerations that affect the alveolar mucosa, gingiva,
paracoccidioidomycosis) and palate
• Organisms often show multiple daughter buds on the parent cell which resemble Mickey
Mouse ears or mariner’s wheel
• Treatment – Sulfonamide derivatives, IV amphotericin B
4. Cryptococcosis • Causative agent – Cryptococcus neoformans and C. bacillispora
(European • Most frequent site of involvement is meninges followed by skin, bone, and the
blastomycosis, torulosis) prostate gland
• Soap bubble lesion in gray matter in immunosuppressed patients
• Cryptococcal meningitis is characterized by head ache, fever, vomiting, and neck stiffness
• Meningoencephalitis is another clinical presentation of cryptococcosis
• Oral lesions rarely appear as crater-like nonhealing ulcers may disseminate to salivary
glands
• Gram-positive, budding, yeast-like with thick, gelatinous capsule
• In tissue sections, it appears as, “tissue microcyst”
• Multinucleated giant cells are common with their characteristic halos
• Amphotericin B and flucytosine
5. Zygomycosis • Causative agent – Zygomycetes including genre Absidia, Mucor, Rhizomucor and
(mucormycosis, Rhizopus
phycomycosis) • Superficial – It involves external ear, fingernails, and skin
• Visceral, pulmonary, gastrointestinal
• Rhinocerebral or rhinomaxillary form – It is important from a dental point of view.
Infection of the head by these organisms are characterized by the classical syndrome
of uncontrolled diabetes, cellulitis, ophthalmoplegia, and meningoencephalitis. The
infection apparently enters the tissue through the nasal mucosa and extends to paranasal
sinuses, palate, pharynx, orbit, and brain. One early clinical manifestation of the disease
is the appearance of a reddish black nasal turbinate and septum with a nasal discharge
• Cases of phycomycosis involving the maxillary sinus may present clinically as a mass in
the maxilla, resembling the carcinoma of maxillary antrum

https://t.me/DentalBooksWorld
258 Triumph's Complete Review of Dentistry

• The tissue involved by mucormycosis shows necrosis and chronic inflammatory infiltrate
• The organism appears as large, nonseptate hyphae with branching at obtuse angle.
Round and ovoid sporangia are also seen. Extensive tissue destruction which disturb
normal blood flow resulting in infarction
• Treatment – Surgical debridement is the treatment of choice. Systemic amphotericin can
be given
6. Rhinosporidiosis • Caused by a fungus called Rhinosporidium seeberi
• Nasal mucosa is the most common site involved. Lesion may appear as small verrucate
or warts, which ultimately become pedunculated
• Oral manifestations: The lesion may be of a mucoid discharge and appear as soft red
polypoid tumor-like growths
7. Aspergillosis • Causative agent – Aspergillus fumigatus
• Bronchopulmonary aspergillosis: Fever, breathlessness, productive cough
• Disseminated aspergillosis: Abscesses in brain, kidney, heart, GIT, and bone
• Aspergillosis of paranasal sinuses (invasive and noninvasive): Invasive form is usually
seen in immunocompromised patients
• Cutaneous aspergillosis – Erythematous macules or papules
• Oral manifestations: Paranasal aspergillosis involves hard palate. Pulmonary aspergillosis
involves soft palate. Painful ulcer with progressive necrosis. Bleeding from lesion with
foul odor
• Smears are stained with, KOH and Parker ink or calcofluor stain
• Aspergillus hyphae appear as septate and dichotomously branched. Immunodiffusion
test for the detection of antibodies to Aspergillus species also proved to be a valuable
diagnostic tool
• Treatment – Amphotericin B along with surgical debridement. Combination therapy
along with caspofungin, flucytosine or itraconazole
8. Toxoplasmosis • Causative agent – Toxoplasma gondii
• Low-grade fever, cervical lymphadenopathy, fatigue, and resemble infectious mononucleosis
• Manifestations of infection can include necrotizing encephalitis, pneumonia, and myocarditis.
CNS infection is very serious patient complaint of headache, lethargy, distortion, and
hemiparesis. Congenital type from mother to fetus more severe in first trimester of pregnancy
• Treatment during pregnancy – Combination of sulfadiazine and pyrimethamine along
with folinic acid
9. Coccidioidomycosis • Causative agent – Coccidioides immitis
(‘valley fever’ ‘San • Primary nondisseminated coccidioidomycosis
Joaquin Valley fever’ • Progressive disseminated coccidioidomycosis
‘desert fever’ or ‘coccidial • General: It is common in all age groups and predominately seen in males. Symptoms occur
granuloma’) usually 14 days after the inhalation of fungus. Infection is common in summer months,
especially after periods of dust storm. It is self-limiting and runs its course within 10–14 days
• Oral manifestations: Proliferative, granulomatous and ulcerated lesions, heal by
hyalinization and scar formation. Lytic lesions of bones of jaw may develop
• Large mononuclear cells, lymphocytes and plasma cells, foci of coagulation necrosis,
multinucleated giant cells are scattered throughout the lesion. Organism is found within
the cytoplasm of giant cells, or lies free in the tissue
• Treatment – Amphotericin B
10. Geotrichosis • Causative agent – Geotrichum candidum
• Commonly found in lungs and oral mucosa. pneumonitis or bronchitis, expectoration
tinted with blood
• Oral manifestations – White velvety patch, mucosa, isolated or diffuse in distribution,
tonsillar lesions are common in association with oral lesions
• Small, rectangular shaped; spores measuring approximately 4–8 microns, with rounded ends
• The tissue reaction is nonspecific and of acute inflammatory type
• Treatment – It includes topical and systemic application of nystatin and amphotericin B

https://t.me/DentalBooksWorld
Chapter 4 • Oral Pathology and Oral Medicine 259

11. Leishmaniasis • Causative genus – Leishmania


• Visceral leishmaniasis—It is also called kala azar caused by Leishmania donovani. Cough
and diarrhea can also develop, spleen enlargement, anemia and muscle wasting, increase
pigmentation of face, spontaneous bleeding, edematous gingiva, and loose teeth
• Cutaneous leishmaniasis—caused by Leishmania braziliensis caused by bite of an
infected sandfly, which heals with depressed scar. Nasal mucosa becomes congested and
ulcerates. Destruction of nasal soft tissue. Lips, soft palate, and larynx may be involved.
The mucosal lesions are long standing, destructive, granulating ulcers causing severe
mutilation of structure involved
• Treatment – Amphotericin is the drug of choice for visceral leishmaniasis
12. Trichinosis • Causative agent – Trichinella spiralis
• Common sites involved are striated muscle, masseter, neck muscle and diaphragm, fever,
facial and periorbital edema, muscle pain and eosinophilia
• Oral manifestations: Tongue is the most common site involved, muscles attached to
mandible, in mandibular alveolar process and in gingival tissues. Trismus, muscular
cramps of the facial muscle, jaw, and tongue can occur, monotony of speech, petechiae
of buccal mucosa, palate and floor of mouth, bleeding from gingiva, lips and nose
• There is no specific treatment for trichinosis and in severe cases, prognosis is poor

VIRAL INFECTIONS
1. Herpes simplex • Causative agent – Herpes simplex virus
infection (primary • Early childhood, preschool period is more prone due to frequent exchange of salivary and nasal
herpes simplex secretions
infection. It • HSV I – Infections above the waist
is also called • HSV II – Infections below the waist
‘acute herpetic • Both HSV I and II can be transmitted sexually
gingivostomatitis,’ • Fever, headache, malaise, nausea, vomiting, irritability, pain upon swallowing and regional
'herpes labialis,’ lymphadenopathy. Vesicles quickly rupture leaving small, shallow, oval-shaped discrete ulcers
‘fever blister,’ • Base of the ulcer is covered with grayish white or yellow plaque. Margins of the sloughed lesions
‘cold sore,’ and are uneven and are accentuated by bright red-rimmed, well-demarcated, inflammatory halos.
‘infectious Size 2–6 mm. Entire gingiva is edematous, swollen with small gingival ulcers
stomatitis’) • Intraepithelial blisters filled with fluid. Intranuclear inclusions: Lipschutz bodies which are
eosinophilic, ovoid, homogenous structure within the nucleus, tend to displace the nucleolus and
nuclear chromatin peripherally. Displacement of chromatin often produces a peri-inclusion halo
• Treatment-topical anesthetics, topical anti-infective agents or specific antiviral drugs acyclovir,
idoxuridine, cytosine arabinoside, and adenine arabinoside
2. Measles (rubeola, • Causative agent – Rubeola virus
morbilli) • Incubation period is 8–10 days. Onset of fever, malaise, cough, conjunctivitis, photophobia,
lacrimation, and eruptive lesions of skin and oral mucosa occur. Otitis media and sore throat.
Skin eruption begins on face, in the hair line and behind the ear and spread to neck, chest, back,
and extremities. Tiny red macules or papules enlarge and coalesce to form blotchy discolored
irregular lesions, which blanch on pressure
• Oral manifestations: Precede 2–3 days before cutaneous rash and are pathognomonic of this disease.
Most common site is on buccal mucosa. Intraoral lesions are called Koplik’s spots and occur in 97%
of cases. They are small, irregularly shaped flecks which appear as bluish white specks surrounded by
bright red margins. Generalized inflammation, congestion, swelling and focal ulceration of gingiva,
palate, throat may occur. Koplik’s spots represent areas of focal hyperparakeratosis which exhibits
spongiosis, intercellular edema, dyskeratosis, and epithelial syncytial giant cells in epithelium
• There is pink staining inclusion in the nuclei or less commonly in the cytoplasm. Inclusion
represents microtubular aggregates characteristic of the paramyxovirus, microabscess formation,
epithelial necrosis, and ulceration
• Treatment: Antiviral drug and vitamin A

https://t.me/DentalBooksWorld
260 Triumph's Complete Review of Dentistry

3. Chickenpox • Causative agent – Varicella-zoster virus 


varicella • Incubation period is 2 weeks and transmitted by airborne droplet or direct contact with infected
persons, with the probable port of entry being respiratory tract. Eruptions usually begin on
the trunk and spread to involve the face and extremities. Occur in successive crops so many
vesicles in different stages of formation or resorption may be found. Skin eventually ruptures,
forming a superficial crust and heals by desquamation. Secondary infection of vesicle results in
the formation of pustules which may leave small pitting scar upon healing
• Small blister-like lesions occasionally involve the oral mucosa chiefly buccal mucosa, tongue,
gingiva, palate, as well as the mucosa of pharynx
• Cause acantholysis with formation of numerous free-floating Tzanck cells. There is also nuclear
margination of chromatin and occasional multinucleation
• No treatment is required in majority of the cases.
4. Molluscum • Causative agent – Pox group
contagiosum • More common in children and young adults. Papules with depressed centers, which may be
keratinized, normal or slightly red in color
• Common sites are lips, tongue, and buccal mucosa; lesions are similar to skin lesions
• Thickening and down growth of epithelium with the formation of large eosinophilic
intracytoplasmic inclusion bodies known as Henderson–Paterson inclusion or simply
molluscum bodies measuring approximately 25 microns in diameter are accumulated in the
crater formed by the distinctive central umbilicated or the dome-shaped lesion
• Treatment: Curettage, followed by local cautery, cryotherapy. Topical application of caustic acid
and irritants such as phenol, TCA, podophyllin, and cantharidin
5. Cytomegalovirus • Causative agent – Cytomegalovirus
infection • Adult population, mononucleosis-like illness, or severe illness with neurologic abnormalities.
Hepatosplenomegaly, jaundice, petechial hemorrhages
• Pneumonia, microcephaly, cerebral calcification, and hearing defect can also occur
• Oral manifestations: Patient may suffer from gingivitis and gingival hyperplasia. Scattered
infected cells are swollen showing intracytoplasmic and intranuclear inclusion and prominent
nucleoli. Salivary ductal cells are also affected and form owl eye cells
• Prevention by passive immunization with hyperimmune gamma globulin can be successful
6. Varicella zoster • Common in adult life
shingles and zona • Equal sex predilection, fever and malaise. Pain and tenderness along the course of the sensory
herpes zoster nerves involved. Involvement is unilateral and dermatomic. Linear papular or vesicular
eruption of the skin or mucosa in areas supplied by the affected nerve. Rupture of the vesicles is
complicated with secondary infection
• Oral manifestations: Involvement of face – if trigeminal nerve is involved, unilateral involvement
of the skin and mucosa in areas supplied by ophthalmic, maxillary, and mandibular nerves.
Extremely painful vesicles are seen in buccal mucosa, tongue, uvula, pharynx, and larynx
7. Infectious • Causative agent – Epstein–Barr Virus
mononucleosis • 15–20-year age group, equal sex predilection, fever, sore throat, headache, chills, cough, nausea
(glandular fever • Lymphadenopathy, splenomegaly, hepatomegaly, cervical lymph nodes show enlargement,
and kissing pharyngitis and tonsillitis
disease) • Oral manifestations – Acute gingivitis and stomatitis, white grayish membrane on the mucosa
• Palatal petechiae – Present in the junction of the soft palate and hard palate, oral ulcers
• Paul Bunnell Test – Normal titer between agglutinins and hemolysins in human blood against
sheep red blood cells – 1:8 (raised to 1:4096 in IM)
• Monospot Test – Agglutination of horse RBCs on exposure to EB virus heterophil antibodies
• Raised ESR leukocytosis
• Symptomatic treatment, adequate rest, short-term corticosteroid therapy
8. Hepatitis B • Causative agent – Hepatitis B virus (HBV)
• It can cause both acute and chronic infections, cirrhosis and liver cancer

https://t.me/DentalBooksWorld
Chapter 4 • Oral Pathology and Oral Medicine 261

9. Herpangina • A4 causes majority of the cases. A4 to A10 and A16 to A22 have also been implicated
aphthous • Young children aged 3–10 years, fever, chills, headache, anorexia, prostration, abdominal
pharyngitis, pain and sometimes vomiting. Sore throat, dysphagia and occasionally, sore mouth. Occurs
vesicular on posterior pharynx, tonsil, faucial pillars and soft palate. Starts as punctuate macule which
pharyngitis evolves into papules and vesicles
• Affected epithelium exhibits intracellular and intercellular edema, which leads to spongiosis
and formation of intraepithelial vesicle. Rupture of vesicle with formation of subepithelial
vesicle. Epithelial necrosis and ulceration
• Treatment – Self-limiting and supportive treatment by proper hydration and topical anesthetic,
when eating or swallowing is difficult
10. Acute • Caused by A10 and is the same as herpangina
lymphonodular • Predominately children and young adults, occasionally older adults can also be affected
pharyngitis • Appears on uvula, soft palate, anterior pillars and posterior oropharynx, sore throat, 41°C
temperature, mild headache, anorexia, and loss of appetite. Raised, discrete, whitish to yellowish
solid papules 3–6 mm in diameter, surrounded by narrow well-defined zone of erythema
• Lesion is nonvascular, nonulcerated, tender, superficial, and bilateral. Papular lesion consists of
densely packed nodules of lymphocytes. Intra-nuclear and cytoplasmic inclusion bodies
11. Hand–foot–mouth • Causative agent – Coxsackie A16, A5, A7, A9, A10, B2 and B5
disease • Children between the age of 6 months and 5 years. Maculo-papular, exanthematous and vesicular
lesions of skin, particularly involving the hands, feet, legs, arms, and occasionally buttocks
• Oral manifestations: Common sites hard palate, tongue, and buccal mucosa. Sore mouth with
refusal to eat. Tongue appears red and edematous. Rise in acute or convalescent serum antibody
titer to coxsackie A16
• Intracytoplasmic viral inclusion can be seen in vesicular scrapping of the lesion
• No specific treatment is necessary since the disease is self-limiting and generally regresses
within 1–2 weeks
12. Foot and mouth • Causative agent – Coxsackie virus
disease • Fever, nausea, vomiting, malaise, and appearance of ulcerative lesions of oral mucosa and
pharynx. Vesicle on palms of hands and soles of feet
• Oral manifestation – Lips, tongue, palate, and oro-pharynx appear to be affected. Small vesicles
rapidly rupture, but heal within 2 weeks
13. Smallpox (Variola) • Causative agent – Variola virus
• High fever, nausea, vomiting, chills, and headache. Patient is extremely ill and may become
comatose during this period. Skin lesions begin as small macules and papules which first appear
on face but rapidly spread to cover much of the body surface, within few days
• Papules develop into vesicles. Pustules appear small elevated and yellowish green, with inflamed
border. They are secondarily infected and occasionally become hemorrhagic
• Oral manifestations: Ulceration of oral mucosa and pharynx. Multiple vesicles appear and rupture
to form ulcers of nonspecific nature. Tongue is swollen and painful, making swallowing difficult
• Treatment – Symptomatic and immunization is essential
14. Acute • Causative agent – Human immuno deficiency virus
immunodeficiency • Asymptomatic infection to severe clinical illness and AIDS
syndrome • Patients generally remain asymptomatic until their CD4 count falls below 500 cells/mm
(Symptomatic HIV infection)
• AIDS is diagnosed when the CD4 count falls below 200 cells/mm
• The immune function ceases, resulting in advanced HIV infection, with a characteristic CD4
cell count of <50 cells/mm

https://t.me/DentalBooksWorld
262 Triumph's Complete Review of Dentistry

Oral Manifestations of HIV


Revised Classification of HIV Infection (1993): EC Clearinghouse Classification
Lesions strongly associated with Lesions LESS commonly associated Lesions seen in HIV
HIV with HIV
• Candidiasis • Bacterial infections • Bacterial infections
• Hairy leukoplakia –– Mycobacterium avium complex –– Actinomyces israelii
• Kaposi sarcoma –– M. tuberculosis –– E. coli
• Non-Hodgkin disease ■■ Melanotic hyperpigmentation –– Klebsiella
• Periodontal diseases ■■ NUS –– Cat Scratch disease
–– LGE • Drug reactions
–– NUG • Fungal infections other than candidiasis:
–– NUP –– C. neoformans
–– Geotrichum
–– Histoplasma
–– Mucormycosis
–– Aspergillus flavis
• Neurological disturbances
• Recurrent aphthous stomatitis (RAS)
• CMV infections
• Molluscum
Diagnosis of HIV
Virus-Based Tests
1. Viral culture:
• It directly detects the virus and highly specific
• It is positive even during the Window period
2. Polymerase chain reaction (PCR): Two types:
• PCR for DNA of provirus present in infected host cells
• PCR for HIV RNA from plasma and is highly sensitive and specific
• PCR can detect as low as 20 copies/mm of plasma
• Results of PCR will be available in 24–48 hours, whereas for viral load it takes about 2–4weeks
3. P24 antigen:
• It is detectable during the window period
• It is positive during late phase of infection and when the virus is replicating fast
• This test is not sensitive
Anti-HIV Antibody Tests
1. ELISA
• Enzyme-linked immunosorbent assay
• More widely done test
• Preferably, three ELISA tests are positive, a person is confirmed HIV positive
• ELISA cannot help in early diagnosis hence once has to wait for 18 months to diagnosis HIV
• As it is cheaper, ELISA remains the mainstay of HIV diagnosis in older children and adults in developing countries
2. Western blot analysis:
• This detects specific antibodies and shows them as separate bands on gels
• More specific than ELISA
• It is regarded as positive if at least two to three bands are positive
• If none is positive, the test is negative
• If only one band is positive, it is regarded as intermediate
• Similar to ELISA, western blot is also negative during window period and is not useful for early diagnosis
• Salivary glands are positive and qualitative xerostomia

https://t.me/DentalBooksWorld
Chapter 4 • Oral Pathology and Oral Medicine 263

DISEASES OF THE BONE


1. Osteogenesis imperfecta • Autosomal dominant
(Brittle bones, osteopsathyrosis, • Mutation in COL1A1 (17q21, COL1A2 (7q22.1)
Lobstein’s disease) • Failure of fetal collagen to be transformed into mature collagen
• Four types (Sillence classification)
Type I – Mild
Type II – Severe (stillborn or death within 4 weeks)
Type III – Moderate–severe (opalescent dentin)
Type IV – Mild–moderate
Type V – IFITM5 mutation, moderate
Type VI – SERPINF1 mutation, moderate mineralization defect
Type VII – CRTAP mutation, severe (popcorn calcifications of femoral head,
coxa vara)
Type VIII – LEPRE1 mutation (bowing of long bones, Wormian bones, bulbous
metaphyses)
Type IX – PPIB mutation, moderate
Type X – SERPINH1 mutation, chaperone defects-severe
Type XI – FKBP10 mutation, moderate
Type XII – BMP mutation, severe
• Extreme fragility and porosity of bone
• Pale blue sclera (thin sclera showing pigmented choroid)
• Hypoacusis, laxity of joints, capillary bleeding, triangular facies, relative
macrocephaly, flattened vertex, and skull base
• Oral manifestations (frontal and temporal bossing, Class III malocclusion,
anterior and posterior cross bites, open bites
• Radiography (Wormian bones, bowing and angulation of long bones, premature
pulpal obliteration of affected teeth)
2. Bruck syndrome Severe osteogenesis imperfecta with congenital contractures
3. Blue sclera Osteopetrosis, fetal rickets, Turner syndrome, Paget’s disease, Marfan syndrome,
Ehlers–Danlos syndrome
4. Osteopetrosis (Albers– • Hereditary
Schonberg, marble bone • Failure of osteoclast to remove bone, increased sclerotic bone mass
disease) • Three types
–– Type 1 (infantile/malignant)
–– Type 2 (adult/benign)
–– Type 3 (intermediate)
• Infantile – Nasal stuffiness, macrocephaly, hypertelorism, short/snub nose,
easy bruising, enamel hypoplasia, microdentinal defect
• Adult type – Osteomyelitis of mandible, bone pain
• Autosomal recessive (seven subtypes – TCIRG1, RANKL, CAII, CLCN7, QSTM1,
PLEKHM1, RANK), Autosomal dominant (two subtypes-LRP5, CLCN7)
• Radiography (sandwich vertebrae, Rugger–Jersey sign (alternating focal
sclerosis), Erlenmeyer flask deformity (thickening and defective bone
remodeling, bone – In-bone of vertebrae and phalanges
• Lab findings [increased creatine kinase BB isoenzyme, tartrate resistant acid
phosphatase (TRAP), PTH]
• Treatment by stem cell therapy, interferon – Gamma 1b
5. Diseases causing osteosclerosis Osteopetrosis, Worth type endosteal hyperostosis, sclerosteosis, van Buchem disease
6. Fibrous dysplasia • Nonhereditary, bone replaced by fibrous tissue leading to metaplastic bone formation
• Mutation GNAS (guanine nucleotide binding protein) gene
• Types
–– Polyostotic type (more bones, bowing of long bones, leg-length deformity,
shepherd-crook deformity, café-au-lait spots)

https://t.me/DentalBooksWorld
264 Triumph's Complete Review of Dentistry

McCune-Albright syndrome-(polyostotic +endocrine disturbances +café au


lait)
–– Jaffe’s type – (polyostotic + café au lait)
–– Monostotic type (less severe form) – Leontiasis Ossea if maxilla and mandible
are affected
• Radiography (ground glass or orange peel appearance, rind sign)
• Histology – Chinese letter pattern
• Treatment – Surgery, vitamin D, and bisphosphonates therapy
7. Mazabraud syndrome Fibrous dysplasia with multiple intramuscular myxoma
8. Fibro-osseous lesions of jaws • Fibrous dysplasia
• Cementoosseous dysplasia (focal, periapical, florid)
• Ossifying fibroma
9. Cemento-osseous dysplasia • Fibro-osseous lesion occurring in the tooth bearing area that form cementoid
matrix
• Types
–– Focal cemento-osseous dysplasia (females, mandibular molar region)
–– Periapical cemento-osseous dysplasia (mandibular anterior region, females)
–– Florid cemento-osseous dysplasia (multifocal in maxilla and mandible
posterior region)
10. Gigantiform cementoma • Familial deposition of excessive cementum that can cause impaired vascularity
leading to complications like osteomyelitis after extraction
• Appear as lobular radioopacities in maxilla and mandible
11. Cherubism (familial fibrous • Autosomal dominant
dysplasia) • Mutations in SH3BP2 (4p16.3)
• Cherubic face – Upward gaze (eye to haven), V-shaped palate
• Grading of Cherubism (Grade I – Bilateral mandibular rami, Grade II –
Bilateral maxillary tuberosity and mandibular rami, Grade III – Whole maxilla
and mandible except coronoid and condyle)
• Radiography – Multilocular radiolucency, displacement of inferior alveolar
nerve, ‘hard palate sign,’ floating tooth syndrome, ground glass appearance
• Histology-vascular connective tissue with TRAP positive giant cells,
eosinophilic perivascular cuffing
• Associated with Noonan syndrome, Ramon syndrome and type 1
neurofibromatosis
12. Paget’s Disease (osteitis • Chronic progressive bone disease
deformans) • Abnormal excessive remodeling of bone with increased osteoclastic activity
• Mutations of SQSTM1 (sequestome) gene, hyperresponsiveness of osteoclasts
to RANKL
• Bone pain, bone deformity (Simian stance), jaw bones (Leontiasis ossea),
platybasia (softening of base of skull
• Radiography – Cotton wool appearance, large circumscribed defect in skull
(osteoporosis circumscripta)
• Increased bone specific alkaline phosphatase, urinary hydroxy proline.
Excretion of pyridinium cross links, N-terminal telopeptide, alpha collagen
C-telopeptide
• Histology – Jigsaw puzzle or mosaic pattern, hematoxyphilic resting and
reversal lines in bone due to repeated resorption and deposition
• Complication – Osteomyelitis, difficulty wearing dentures, osteosarcoma
13. Massive osteolysis (vanishing • Progressive resorption and proliferation of vascular connective tissue – Total
bone, Phantom bone, Gorham disappearance of affected bone
syndrome) • Swelling of affected bone, pathologic fractures

https://t.me/DentalBooksWorld
Chapter 4 • Oral Pathology and Oral Medicine 265

• Radiography-intramedullary and subcortical radiolucency


• Histology – Highly vascular connective tissue –Multinucleated osteoclasts
• Treatment – Vitamin D, interferon alfa-2b, bisphosphonates
14. Infantile cortical hyperostosis • Unusual cortical thickening of bones
(Caffey Silverman syndrome) • Triad – Irritability, swelling of soft tissue and associated bones
• Two types (familial and sporadic)
• Radiography – Periosteal new bone ‘cloaking’ is restricted to the diaphysis
• Histology – Thick periosteum, hyperplasia of lamellar bone
• Increased ESR and alkaline phosphatase
15. Marfan syndrome • Heritable, autosomal dominant
(Arachnodactyly) • Reduction in microfibrils leads to excess TGF-Beta, decreased elasticity
• Dolichostenomelia (disproportionate long limbs), hyperextensibility of joints,
Arachnodactyly, thoracolumbar scoliosis, dilatation of aorta, rupture of mitral
and tricuspid valve, myopia, cataract, glaucoma, retinal detachment
• High arched palate, bifid uvula, severe malocclusion, multiple odontogenic
cysts of the maxilla and mandible
• Diagnosis is based on revised ghent nosology
• Radiography – High arched palate, increased skull height, enlarged frontal sinus
16. Craniosynostosis • Premature fusion of cranial sutures, suture ridging, early closure of fontanels
• Primary (primary defect in ossification)
• Secondary (failure of brain growth) – Vitamin D deficiency, renal osteodystrophy,
hypercalcemia, rickets, Mutation in FGFR
• Craniosynostosis syndromes (Apert, Crouzon, Chotzen, Pfeiffer, Carpenter)
17. Craniofacial dysostosis • Most common craniosynostosis
(Crouzon syndrome) • Mutations in FGFR-2 and 3
• Brachycephaly (short and broad), scaphocephaly (boat shaped), trigonocephaly
(triangular), kleeblattschadel (cloverleaf-shaped)
• Deviated nasal septum and obliterated anterior nares-parrot beak nose,
hypertelorism, strabismus, antimongoloid facies, ‘frog face,’ crowding of teeth,
ectopic eruption of maxillary first molars, anterior open bite, lateral palatal
swellings, V-shaped palate, dysphagia
• Radiography – Beaten metal appearance, obliteration of coronal and sagittal sutures
• Spina bifida, fusion of C2-C3, C5-C6, Butterfly vertebrae
18. Apert syndrome • Craniosynostosis, hypoplastic middle third and syndactyly of hands and feet
(acrocephalosyndactyly) • Mutation of FGFR-2
• Acrocephaly or brachycephaly, parrot beak appearance (sunken bridge and
bulbous tip), hyperhidrosis, patent ductus arteriosus, pulmonary stenosis
• Malocclusion, shovel-shaped incisors, supernumerary teeth
19. Achondrogenesis • Defective endochondral ossification with prominent abdomen and anasarca
• Type 1A (Houston–Harris) – Autosomal dominant
• Type 1B (Fraccaro type) – Autosomal recessive (DDST – diastrophic dysplasia
sulfate transporter mutation)
• Type 2 (Langer saldino type) – Autosomal dominant COL1A1
–– Dwarfism, short limb and trunk, narrow thorax, prominent abdomen,
large cranium
20. Achondroplasia • Disturbance in cartilage mediated bone formation
• Mutation in FGFR-3 gene
• Dwarfism, frontal bossing, hypoplasia, short limbs, bullet-shaped vertebral
bodies, metaphysical flaring, trident hands
21. Chondro-ectodermal dysplasia • Affects structures derived from mesoderm and ectoderm
(Ellis-Van Creveld syndrome) • Four components – Chondrodysplasia, polydactyly, ectodermal dysplasia,
congenital heart failure

https://t.me/DentalBooksWorld
266 Triumph's Complete Review of Dentistry

22. Cleidocranial dysplasia • Autosomal dominant


(Marie and Sainton’s disease, • Mutation in Cbfa1 (core binding alfa) gene (RUNX2)S
mutational dysostosis) • Hypoplasia or agenesis of clavicle-unusual mobility of shoulders, delayed
ossification of fontanels of skull, brachycephaly, Wormian bones, Genus
Valgus (knock knees), Pes planus (flat feet)
• Narrow high arched palate, cleft palate, prolonged retention of deciduous teeth,
delayed eruption of permanent teeth, absence or paucity of cellular cementum,
supernumerary teeth, Dentigerous cyst
• Radiography – Absence or thinning of clavicle, spina bifida, shortening or
broadening of digits
23. Tricho-dento-osseous • Hereditary condition that involves hair, teeth and bones
syndrome • Autosomal dominant
• Mutation in DLX gene (17q21.3-q22)
• Kinky or thick curly hair, thin nails, thick bones, frontal bossing, square jaw,
mandibular prognathism, dolichocephaly
• Yellow brown pigmentation of teeth, enamel hypoplasia, severe attrition of
enamel, taurodontism in molars
24. Down syndrome (trisomy 21, • Genetic disease
congenital acromicria • Three types (trisomy 21, chromosomal translocation, chromosomal mosaicism)
syndrome, mongolism) • Mental retardation, mongoloid appearance, brachycephaly, hypertelorism,
upward and outward slant of palpebral fissures, medial epicanthal folds,
strabismus, cataract, retinal detachment
• Skeletal anomalies – Short and broad digits, short curved fifth finger – Dysplasia
of mid-phalanx, clinodactyly, dysplasia of pelvis, joint laxity, wide gap between
first and second toes, atlanto-occipital instability
• Protuberant abdomen, umbilical hernia, congenital defect of heart, hypogenitalism,
delayed or incomplete puberty, duodenal atresia, Hirschsprung disease, recurrent
respiratory infections, epilepsy, hypothyroidism, presenile dementia
• Oral manifestations – microstomia, macroglossia, fissured/scrotal tongue,
geographic tongue, thickening of lips, angular cheilitis, delayed eruption of tooth,
partial anodontia, enamel hypoplasia, juvenile periodontitis, cleft lip or palate
25. Langerhans cell histiocytosis • Affects bone and other organ systems
(histiocytosis X, Langerhans • Arises from the Langerhans cell
cell granulomatosis, type II • Three types
histiocytosis and nonlipid –– Type 1 – Acute fulminant disseminated – Letterer–Siwe
reticuloendotheliosis) –– Type 2 – Indolent, chronic, eosinophilic granuloma
–– Type 3 – Intermediate form – Hand–Schuller–Christian disease
• Presence of histiocytosis X bodies (Birbeck granules with a terminal tennis
racquet appearance
• Loosening and precocious exfoliation of teeth
26. Focal osteoporotic marrow Hematopoietic marrow due to aberrant bone regeneration after tooth
defect extraction, persistence of fetal marrow, marrow hyperplasia due to increased
erythropoietic demand

Diseases of Temporomandibular Joint


27. Condylar hypoplasia/aplasia • Failure of development of condyle
• Type 1 – Congenital or primary – Treacher Collins syndrome, oculo-auriculo-
vertebral syndrome, hemifacial macrosomia, Pierre Robin syndrome, Hurler
syndrome
• Type 2 – Acquired or Secondary – Trauma, infection, therapeutic radiation
• Other co-existing defects – defective ear, underdeveloped mandibular ramus
or macrostomia
• Unilateral – Mandible shifts to affected side with a noticeable midline shift
• Bilateral results in micrognathia

https://t.me/DentalBooksWorld
Chapter 4 • Oral Pathology and Oral Medicine 267

28. Condylar hyperplasia • Unilateral enlargement of condyle


• Mild chronic inflammatory process leads to proliferative osteomyelitis
–– Type A – Hemimandibular hyperplasia with vertical asymmetry
–– Type B – Hemimandibular hyperplasia with transverse asymmetry
–– Type C – Combination of A and C
• Deviation of chin away from the affected side
29. Bifid condyle • Groove or depression in the midline of condylar head due to defective blood
supply during embryogenesis or trauma
30. Dislocation of condyle • Luxation – Complete dislocation (non-self-reducing)
• Subluxation – Partial or incomplete dislocation (self-reducing)
31. Ankylosis of TMJ • Adhesion of joint components with fibrous or bony union
(hypomobility) –– Type 1: Intra-articular
–– Type 2: Extra-articular
• Displacement of the chin is toward the affected side
32. Internal derangement of TMJ • Abnormality of the internal components of the joint
• Four clinical stages
–– Stage I: Articular disc displacement in closed mouth and reduces to normal
contact in open mouth
–– Stage II: Disc displacement in closed mouth and locks intermittently in open
mouth
–– Stage III: Disc displacement in closed mouth and does not reduce to normal
contact in open mouth
–– Stage IV: Disc is displaced without reduction and with perforation of the disc
33. Osteoarthritis • Occurs above 40 years of age
• Clicking or snapping of joints
• Pain and limitation of motion is rare
• Radiography – Diagnostic criteria – Condyle has osteophytes, condyle and
fossa has subcortical erosion, condyle or fossa has subcortical pseudocysts
34. Rheumatoid arthritis • Hypersensitivity to bacterial toxins
• Involvement of TMJ makes the joint irregular, flat articular surface, osteophytes
• Juvenile idiopathic arthritis (Still’s disease) – In children – Asymmetry of
mandible and class II malocclusion
35. Septic arthritis • Acute infectious arthritis due to Staphylococcus aureus
36. Loose joint bodies (Synovial • Benign condition with nodular cartilaginous proliferation in the joint synovial
Chondromatosis) space
• Three cardinal signs-pain in the preauricular area, swelling, facial asymmetry,
joint deformity, limited joint function
37. Temporomandibular disorder • Common cause of facial pain next to tooth ache
(TMD) • Pain without any noticeable destructive change in the TMJ radiograph
• Occurs due to bruxism or day time clenching of jaws
• Multifactorial etiology – Malocclusion, jaw clenching, bruxism, personality
disorder, increased pain sensitivity, stress and anxiety
• Four cardinal signs – Pain, muscle tenderness, clicking or popping noise in
TMJ, limitation of jaw movement

DISEASES OF SKIN
1. Ectodermal dysplasia • Structures affected – Ectodermal derivatives – Hair, nail, salivary gland, and teeth
(ectodermal dysplasia • Types: Hypohidrotic/anhidrotic (Christ Siemens–Touraine syndrome) and
syndrome) Hidrotic (Clouston syndrome)
• Typical facies – Frontal bossing, sunken cheeks, saddle nose, everted lips,
wrinkled hyperpigmented skin around eyes, large low set ears)
• Truncated cone-shaped teeth

https://t.me/DentalBooksWorld
268 Triumph's Complete Review of Dentistry

2. Onychodysplasia Distortion of fingernail or lunula


3. Cannon disease Hereditary white sponge nevus
• Autosomal dominance
• Hyperparakeratosis and acanthosis, vacuolated cells show pyknotic nuclei
with perinuclear condensation
4. Hereditary benign Hereditary diseases with superficial foamy, gelatinous white plaques,
intraepithelial dyskeratosis dyskeratotic cells in epithelium
5. Dyskeratosis congenital, Cutaneous reticular hyperpigmentation
(Zinsser–Cole–Engleman Nail dystrophy, premalignant leukoplakia, progressive pancytopenia
syndrome, Hoyeraal– Increased incidence of oral cancer
Hreidarsson syndrome) • Mainly x-linked recessive – males affected more commonly
–– Grayish brown pigmentation
• Increased incidence of dental caries, shortened roots, mild taurodontism,
tooth loss
• Thrombocytopenia
6. Xeroderma pigmentosum • Defect in DNA repair, autosomal recessive
• Severe sunburn with limited sun exposure, depigmentation or hyperpigmentation
• Complications – SCC, melanoma, BCC, fibrosarcoma
7. Hereditary mucoepithelial –– Autosomal dominant trait affecting skin and mucosa
dysplasia –– Sparse, coarse hair with nonscarring alopecia, severe photophobia, corneal
vascularization, keratitis, nystagmus
–– Demarcated fiery red erythema of palate
–– Histology – Dysplastic features in epithelium, cytoplasmic vacuoles in
epithelial cells, lesional cells have reduced number of desmosomes

8. Ehler’s Danler syndrome (cutis • Mutation of COL5A1, COLSA2


hyperelastica, Tenascin – X • Rubber man and Contortionist
deficiency) • Joint hypermobility, cutaneous fragility, hyperextensibility, unusual healing
with papyraceous scarring
• Joint hypermobility is seen in types 1, 3, 6
• Lack of normal scalloping of DEJ, passage of dentinal tubules into enamel,
formation of irregular dentin, increased tendency to form pulp stones
9. Ehler’s Danler syndrome Type 1 – Severe – Autosomal dominant – Joint hypermobility, dystrophic scars
classification Type 2 – Moderate – Autosomal dominant
Type 3 – Familial hypermobility – Autosomal dominant – Joint hypermobility,
multiple joint dislocations
Type 4 – Vascular (Sack Barabas) – Autosomal dominant – Vascular fragility,
perforation of hollow organs
Type 5 – X-linked – Similar to type II and III
Type 6 – Ocular – Detached retina, scoliosis
Type 7 – Arthrochalasis multiplex congenita – Hyperelasticity of skin, small stature
Type 8 – Periodontal – Autosomal dominant – Generalized early onset
periodontitis, skin ecchymoses
10. Tuberous sclerosis (epiloia, • Autosomal dominant. Mutation in TSC 1 and 2 gene
Bournville–Pringle syndrome) • Intellectual instability, seizures, angiofibroma of skin
• Tuberous proliferation in CNS (10% subependymal giant cell astrocytoma)
• Cardiac rhabdomyoma in heart muscle
• Renal angiomyolipoma
• Shagreen patches (connective tissue hamartomas), ash leaf spots (ovoid areas
of hypopigmentation)
11. Kawasaki disease Acute abnormal inflammatory response triggered by a neoantigen, febrile
(mucocutaneous lymph node illness with systemic vasculitis, fever of unknown origin for 5 days, generalized
syndrome) erythema and desquamation of skin, cervical nonsuppurative lymphadenopathy

https://t.me/DentalBooksWorld
Chapter 4 • Oral Pathology and Oral Medicine 269

12. Focal dermal hypoplasia X-linked dominant, mnemonic – FOCAL female, osteopathia striata,
syndrome (Goltz–Gorlin coloboma, absent ectodermis, mesodermis, neurodermitis derivatives and
syndrome) lobster claw deformity
13. Oral lichen planus • Common mucocutaneous disease, (prevalence worldwide 0.5–1%,
prevalence in India – 2.6%)
• Bilateral, Wickhams striae (Grayish white intersecting lines)
• Types – Bullous, erosive, atrophic, hypertrophic reticular, annular
• Typical histological findings (granular layer, acanthosis with intracellular
edema of the spinous cells in some instances, ‘saw tooth’ appearance of the
rete pegs, subepithelial Band-like T-cells and histiocytes; intraepithelial
T-cells, degenerating basal keratinocytes colloid (civatte, hyaline, cytoid)
bodies – Homogenous eosinophilic globules
• Max Joseph spaces (histologic clefts due to weakening of epithelial and
connective tissue interface)
• Fibrinogen and fibrin in linear pattern I immunofluorescence)
• Breaks, branches, and duplications of basement membrane is seen
• Higher risk of malignant transformation 0.3–3% (erosive and atrophic)
14. Lichenoid reaction • Occurs after systemic drug therapy, dental restoration, metallic allergy or as
graft-versus-host disease
• Unilateral, inflammation is diffuse and consists of plasma cells and
eosinophils in addition to lymphocytes, perivascular inflammatory infiltrate
15. Lichenoid dysplasia Proliferative verrucous leukoplakia, exhibits epithelial dysplasia with a band-
like inflammatory infiltrate can mimic lichen planus
16. Psoriasis • Skin lesion with dry papules covered by silvery scale
–– Associated with HLA Cw6 and B57
• Increased turnover rate of dermal epithelial cells (3–5 days)
• Removal of deep scales leave tiny bleeding spots – Auspitz sign
• Hyperkeratosis – Mainly parakeratosis, some orthokeratosis,
neutrophils in stratum corneum and squamous cell layer, hypogranulosis
• Intraepithelial microabscess (Monro’s abscesses)
• Basket weave pattern of stratum corneum
• Sandwich sign (plasma cells infiltrate below the orthokeratotic layer)
17. Psoriasiform lesions Reiter’s syndrome, benign migratory glossitis
18. Pityriasis rosea • Benign papulosquamous disease
• Primary lesion (Herald spot)
19. Erythema multiforme • Erythematous discrete macules, papules or vesicles in symmetrical pattern
• Appear as target, iris or bull’s eye
20. Steven–Johnson syndrome Severe bullous form of erythema multiforme. Characterized by fever, malaise,
photophobia, and eruptions of the oral mucosa, genitalia, and skin
21. Toxic epidermal necrolysis Fatal bullous drug eruption with widespread peeling of skin
(Lyell’s disease)
22. Pachyonychia congenital • Palmoplantar keratoderma in nails, hyperhidrosis
(Jadassohn–Lewandowsky • Type 1 – Jadassohn–Lewandowsky
syndrome, polykeratosis • Type 2 – Jackson–Lawler
congenita) • Mutations in keratins K6a, K6b, K16, and K17
23. Keratosis follicularis (Darier • Genodermatoses with hyperkeratotic papules
disease) • Mutation in ATP2A2 gene
• Cobble stone appearance white flat-topped papules
• Corps ronds (large squamous cells with basophilic nucleus, dark eosinophilic
cytoplasm, distinct cell membrane) and grains (small elongated parakeratotic
cell), test tube-shaped reteridges
• Leafing out pattern in parabasal cells

https://t.me/DentalBooksWorld
270 Triumph's Complete Review of Dentistry

24. Warty dyskeratoma (isolated Single isolated lesion resembling Darier’s disease
Darier’s disease, follicular
dyskeratoma)
25. Incontinentia pigmenti (Bloch • X-linked dominant single-gene disorder with neurologic, ophthalmologic,
Sulzberger syndrome) and dental involvement, as well as cutaneous findings
• Hallmark – melanin pigmentation of the epithelium dropping down as
clusters in upper dermis
• Four classic stages (vesicular, verrucous, hyperpigmentation, atrophy or
depigmentation)
• Oligodontia, delayed eruption, hypoplasia of teeth
26. Porokeratosis of Mibelli • Faulty keratinization and atrophy of skin
• Cornoid lamella – Epithelial atrophy with dyskeratosis
27. Acanthosis nigricans • Dermatoses with symmetric, mild hyperpigmentation, mild papillary
hypertrophy
• Presence of pseudohorn cyst
• Associated with Crouzon syndrome or drug ingestion (corticosteroid and
oral contraceptives)
28. Pemphigus • Autoimmune, intraepithelial blistering disease
• Occurs at 50–60 years of age
• Antibodies against desmoglein 1 and 3
• Nikolsky sign (loss of epithelium with oblique pressure in unaffected area)
• Suprabasilar split
• Basal layer appears as ‘Row of tombstones’
• Tzanck cells (epithelial cells separated due to acantholysis)
• Immunofluorescence demonstrates IgG, C3, IgA and IgM
29. Pemphigus vegetans • Uncommon variant of pemphigus vulgaris
• Cerebriform tongue (sulci and gyri pattern in dorsum of tongue)
• Type 1 – Flaccid bullae and erosions (Neumann)
• Type 2 – Pustules (Hallopeau)
30. Pemphigus foliaceus (fogo • Superficial type of pemphigus
selvagem) • Six types (pemphigus erythematosus, pemphigus herpetiformis, endemic
pemphigus foliaceus, IgA pemphigus foliaceus, paraneoplastic pemphigus
foliaceus, drug-induced pemphigus foliaceus)
• Brazilian wildfire type – Mild form occurs in children in Brazil
31. Paraneoplastic pemphigus • Associated with other malignant conditions (non-Hodgkin’s lymphoma,
chronic lymphocytic leukemia, Castleman tumor, giant cell lymphoma,
poorly differentiated sarcoma)
• Autoantibodies to intercellular Adhesins (periplakins and envoplakins)
32. Familial benign pemphigus • Autosomal dominant
(Hailey–Hailey disease) • Histologically resembles pemphigus and Darier’s disease
• Dilapidated brick wall effect – persistence of few intercellular bridges and
amidst acantholysis
33. Cicatricial pemphigoid (ocular • Autoimmune blistering disease affecting the oral cavity, nasal and
pemphigus, Benign mucous oropharynx, conjunctiva, genitals
membrane pemphigoid) • Healing occurs by scarring and pigmentation
• Complications – Blindness, supraglottic stenosis with hoarseness, airway
obstruction, dyspareunia (pain during intercourse)
• Ocular lesions – Starts with subconjunctival fibrosis, heals with adhesions
called Symblepharon’s, inward turning of eyelid (entropion), trichiasis
(rubbing of eyelashes on cornea
• Autoantibodies of IgG4 against BPAG2 and epiligrin (laminin - Five
antigens)
• Subepidermal bullae – With basement membrane attached to epithelium

https://t.me/DentalBooksWorld
Chapter 4 • Oral Pathology and Oral Medicine 271

34. Bullous pemphigoid • Autoantibodies of IgG against BP230 (BPAg1) and BP180 (BPAg2)
(Parapemphigus) • Subepidermal bullae with basement membrane attached to connective tissue
• Viable roof over new blister, necrotic over an old blister
• Variable perivascular infiltrate (lymphocytes, histiocytes, eosinophils)
• Eosinophil exocytosis (eosinophilic spongiosis)
35. Epidermolysis bullosa Three types
Type 1 – Epidermolysis bullosa simplex (intraepithelial)
Type 2 – Junctional epidermolysis bullosa (between lamina lucida and lamina
densa of basement membrane)
Type 3 – Dystrophic epidermolysis bullosa (sublamina densa of basement
membrane)
Type 4 – Kindler syndrome (subepithelial clefting)
36. Epidermolysis bullosa simplex Types
1. Generalized
2. Localized (Weber Cockayne syndrome)
37. Junctional epidermolysis Mutation of LAMB3 gene
bullosa
38. Dystrophic epidermolysis Types
bullosa 1. Dystrophic epidermolysis bullosa dominant
2. Dystrophic epidermolysis bullosa, Recessive–Scarring–Functional club fist.
Complications-SCC
39. Kindler syndrome Mutation in Hemidesmosomal attachment protein (Kindlin-1)
Subepithelial clefting just below the basal layer above the lamina lucida
40. Systemic lupus erythematosus • Autoimmune disease with immune complex against the kidney, skin, blood
cells, and the CNS
• Butterfly distribution on face
• Fibrinoid thickening of glomerular capillaries – Wire loops
• Warty vegetations in heart valves (Libman–Sacks endocarditis)
• Lupus cheilitis (involvement of vermilion zone of lower lip)
• Lab findings (increased ESR and CRP, gammaglobulin, creatinine
phosphokinase, decreased albumin)
• Tests (positive ANA, anti-DNA, anti-Sm, decreased C3 and C4)
41. Discoid lupus erythematosus • Chronic scarring atrophy
• Removal of scales present with Carpet tack extensions
42. Systemic sclerosis • Connective tissue disease with vasomotor disturbances, fibrosis, atrophy of
(scleroderma, hidebound skin, and subcutaneous tissue
disease) • CREST syndrome (Calcinosis cutis, Raynaud’s phenomenon, esophageal
dysfunction, sclerodactyly, telangiectasia)
• Circumscribed scleroderma – Morphea
• Linear scleroderma – Coup de sabre
43. Linear pattern of LP and lupus erythematosus
Immunofluorescence
44. Patchy linear pattern of Pemphigoid and erythema multiforme
immunofluorescence
45. Acrodermatitis enteropathica • Autosomal recessive disorder
• Zinc deficiency
• Perioral, acral dermatitis, alopecia
46. Solar elastosis (actinic elastosis) • Degenerative skin disease due to aging and sun exposure
• Sailors skin, farmers skin
• Grenz zone – Hematoxyphilic elastic fibers separated from epidermis by a
band of normal collagen

https://t.me/DentalBooksWorld
272 Triumph's Complete Review of Dentistry

47. CANDLE syndrome • Chronic atypical neutrophilic dermatosis with lipodystrophy and
elevated temperature (CANDLE) syndrome is a newly characterized
autoinflammatory disorder
• Mutations in PSMB8
48. Apoptotic keratinocytes Colloid bodies (civatte, hyaline, cytoid)
49. Nikolsky’s sign is seen in • Pemphigus, toxic epidermal necrolysis, Steven–Johnson syndrome,
epidermolysis bullosa dystrophic recessive, herpes, and leukemia
• "Marginal Nikolsky's sign" describes the extension of the erosion on the
surrounding normal-appearing skin by rubbing the skin surrounding
existing lesions
• "Direct Nikolsky's sign" is the induction of an erosion on normal-appearing
skin, distant from the lesions
50. Asboe-Hansen sign The Asboe-Hansen sign (also known as “indirect Nikolsky sign” or “Nikolsky
II sign”) refers to the extension of a blister to adjacent unblistered skin when
pressure is put on the top of the bulla
51. Sheklakov sign False Nikolsky’s sign or Sheklakov’s sign is positive in sub-epidermal blistering
disorders

DISEASES OF THE BLOOD


Diseases of Red Blood Cells
1. Anemia Etiologic classification
Loss of blood
Trauma (acute), GIT lesions, gynecological (chronic)
Decreased red cell production
Nutritional deficiency, inflammation mediated, erythropoietin deficiency,
immune mediated, endocrine failures
Increased red blood cell destruction
Membrane alterations – Genetic (hereditary elliptocytosis, spherocytosis),
membrane alterations – Acquired – (hypophosphatemia, paroxysmal
nocturnal hemoglobinuria)
Enzyme deficiency-G6PD, pyruvate kinase, hexokinase,
Hemoglobin abnormalities (thalassemia syndromes, sickle cell disease,
hemoglobinopathy), immunologic (Hemolytic disease, transfusion reactions,
drug induced), mechanical trauma (DIC, TTP, defective cardiac valves, physical
trauma)
Infections (malaria), splenic sequestration (reticuloendothelial hyperactivity)
Morphological classification
Normocytic normochromic
Normal MCV, MCH, Hemorrhage, hemolysis, lack of blood
MCHC formation
Macrocytic normochromic
Increased MCV, MCH: Lack of erythrocyte-maturing factors (extrinsic
Normal MCHC and intrinsic factors)
Microcytic normochromic
Reduced MCV, MCH: Thalassemia syndrome
normal MCHC
Microcytic hypochromic
Reduced MCV, MCH and Iron deficiency
MCHC

https://t.me/DentalBooksWorld
Chapter 4 • Oral Pathology and Oral Medicine 273

2. Iron deficiency anemia • Anemia due to depletion of iron stores


• Reduced oxygen carrying capacity, pallor of mucosa and skin, fatigue,
pagophagia (craving for ice cold food), poor mental ability, intolerance to
cold, repeated infections, brittle spoon-shaped nails (koilonychias), atrophic
glossitis
• Angular cheilitis, burning and tenderness of tongue
• Lab findings – Reduced MCV, MCH and MCHC, pencil-like erythrocytes,
reduced reticulocyte hemoglobin
3. Plummer–Vinson syndrome Esophageal webs, dysphagia, premalignant condition
4. Megaloblastic anemia Large erythrocytes due to vitamin B12 and folate deficiency
5. Vitamin B12 deficiency • Vitamin B12 deficiency – Neurological manifestations like peripheral
polyneuritis with numbness, pain, tingling, burning, unsteady gait and ataxia
• Hunter’s or Moeller’s glossitis, glossodynia, lingual paresthesia
• Cheilitis, recurrent ulcers, xerostomia, oral candidiasis
• Lab findings: Increased MCV, MCH: Normal MCHC, RBCs exhibit
anisocytosis, poikilocytosis, decreased reticulocyte count, Howell–Jowell
bodies (residual fragment of nucleus in RBC), hypersegmented nuclei,
hypercellular bone marrow, giant metamyelocytes and band forms in
granulocytic cell lines
• Treatment: Parenteral and oral administration of B12
6. Folate deficiency anemia • Anemia due to inadequate intake, impaired absorption and increased
requirement of vitamin B9 (folate)
• Neurological changes, hyperpigmentation of skin and mucosa
7. Pernicious anemia Autoimmune form of B12 deficiency anemia, antibody to intrinsic factor (IF),
type I and II
8. Dimorphic anemia Combination of microcytic and macrocytic
9. Aplastic anemia • Pancytopenia due to hypocellular bone marrow due to immune-mediated
destruction of hematopoietic progenitors
• Increased incidence in Asian population
• Anemia-pallor, breathlessness, palpitation
• Thrombocytopenia-petechiae, ecchymosis, and bleeding
• Neutropenia – Recurrent infections, chills, and fever
• Spontaneous gingival bleeding, mucosal ulcerations, gingival hyperplasia
• Lab findings: Diagnostic criteria – Hb count less than 10 g/dl, platelet count
less than 20,000/µL, neutrophil count less than 500/mm3
10. Fanconi’s anemia • Inherited bone marrow failure syndrome
• Abnormal thumbs and/or radial bones, skin pigmentation, small heads, small
eyes, abnormal kidney structures, and cardiac and skeletal anomalies
Hemolytic Anemia
11. Sickle cell anemia Most common hereditary disease of blood
• Point mutation of the beta globin gene – HbS replaces normal adult HbA
• Low oxygenation leads to aggregation of HbS (sickling) – Vasoocclusive
crisis – Obstruction of microvasculature – Tissue hypoxia and infarction –
Severe pain and organ damage
• Fatigue, exercise intolerance, jaundice, pain in bones and joints, frequent
infections, retinal ischemia, pulmonary hypertension, stroke, osteopenia,
osteomyelitis, osteonecrosis
• Acute chest syndrome – Severe dyspnea, chest pain – Dangerous manifestation
of sickle cell anemia
• Sequestration crisis – Sudden spleen enlargement and hypovolemic shock

https://t.me/DentalBooksWorld
274 Triumph's Complete Review of Dentistry

• Prominent cheek bones and maxillary overgrowth due to marrow hyperplasia,


increased overjet, over bite, diastemata, delayed eruption of teeth, gingival
hyperplasia
• Radiography – ‘Step ladder’ image of mandible due to increased intertrabecular
spacing. Hair on end pattern due to thickening of diploe
• Lab findings: Hb-5–9 g/dl, reduced RBC count, hematocrit 17–29%, reduced
ESR
• Treatment – Prophylactic transfusion
12. Erythroblastosis fetalis Congenital hemolytic anemia due to Rh incompatibility
• Destruction of fetal blood due to reaction of maternal and fetal blood factors
• Anemia, pallor, jaundice, compensatory erythropoiesis, fetal hydrops
• Ground sections show positive test for bilirubin, enamel hypoplasia, in
incisal edges of anterior teeth and middle portion of deciduous cuspid and
first molar, ring-like defect – Rh Hump
• Lab findings – RBC count less than 100,000 cells/mm3
13. Thalassemia Inherited defect with reduced and defective hemoglobin production
• Beta thalassemia-reduced beta chains in hemoglobin-severe form (minor
– One defective gene, major – Two defective genes-Cooley’s anemia or
Mediterranean anemia)
• Alpha thalassemia-reduced alpha chains-(Alpha thalassemia trait – Two
genes defective, Hemoglobin H disease (HbH) – Three genes defective, four
genes defective homozygous is called hydrops fetalis
• Pallor, breathlessness, extramedullary hemopoiesis leading to
hepatosplenomegaly, mongoloid facies with prominent cheek bones, enlarged
maxillary antrum, increased maxillary overbite
• Repeated transfusions lead to iron overload
• Protruded maxilla, spacing, rotation of teeth, malocclusion, increased risk of
caries due to reduced salivation – Deposition of iron in salivary glands due to
secondary hemochromatosis-
• Lab finding – Basophilic stippling, fragmented cells, target cells
• Radiography – Typical hair on end pattern, osteoporosis
14. Sideroblastic anemia/ • Anemia in which the bone marrow produces ringed sideroblasts rather than
sideroachrestic anemia healthy red blood cells 
• SF3B1 gene, ALAS gene, PSU1, ABC7gene mutation, DIDMOAD syndrome
• Defective synthesis of aminolevulinic acid synthetase
15. Polycythemia Abnormal increase in number of erythrocytes
1. Relative polycythemia – Apparent increase in number of RBCs due to
hemoconcentration of blood
2. Primary or polycythemia Rubra Vera (Vaquez disease) – true idiopathic
increase – Mutation in JAK2 (Janus kinase 2)
3. Secondary polycythemia – Absolute increase due to reduced oxygen
concentration
• Panmyelosis – Concurrent increase in WBCs, platelets along with RBCs
• Headache, dizziness, lassitude, tinnitus, visual disturbances, hypertension,
excessive sweating, and intense itching of skin
• Increased risk of thrombotic events
• Erythromelalgia – Peculiar peripheral vascular event in extremities –
Erythema and warmth
• Submucosal petechiae
• Lab findings – RBC level increases to 10,00,000 cells/mm3, Hb 14–28 mg/dl,
reduced serum erythropoietin level
• Treatment: Phlebotomy, myelosuppressive therapy

https://t.me/DentalBooksWorld
Chapter 4 • Oral Pathology and Oral Medicine 275

Diseases of White Blood Cells


16. Leukopenia • Abnormal reduction in number of white blood cells
–– Acquired: Intrinsic bone marrow disease, immune-mediated, increased
utilization in infections, nutritional deficiency, drugs and chemicals
–– Congenital: Chronic, syndrome-associated neutropenia in children is
commonly associated with congenital or genetic abnormality (Schwachman–
Diamond syndrome, dyskeratosis congenita, cartilage-hair syndrome)
17. Cyclic neutropenia • Periodic neutropenia in a regular 21-day cycle
• Due to bone marrow maturation arrest
• Mutation in ELANE or ELA-2 gene (neutrophil elastase gene)
• Headache, arthritis, cutaneous infection, infection of GIT
• Severe gingivitis, premature periodontal lesions
18. Agranulocytosis • Serious form of neutropenia characterized by near absence of neutrophils as
low as 100 cells/µL
–– Primary – Idiopathic
–– Secondary – Drug induced, drug acts as a hapten and induces antibody
formation
• Lab findings – WBC count below 2,000 cells/mm3
19. Kostmann syndrome Severe congenital neutropenia
20. Leukocytosis Increase in number of circulating leukocytes greater than 11,000 cells/mm3
21. Leukemia • Neoplastic proliferation of WBCs
–– Lymphoid – Lymphoblastic and lymphocytic (B cell, Tcel, NK cell)
–– Myeloid – Myelogenous (erythrocytes, granulocytes, monocytes)
• Etiology – Radiation, alkylating agents, benzene, formaldehyde, viruses
(HTLV), genetic syndromes (Down’s syndrome, Fanconi’s anemia, ataxia
telangiectasia, Bloom syndrome, neurofibromatosis type I, Shwachman sy
ndrome, Klinefelter’s syndrome, Wiskott–Aldrich syndrome)
• Lab findings – Increase in blast cells, Bone marrow aspiration, chromosome
changes, DNA markers
22. Acute myeloid leukemia • Adults, rapidly progressing infections, anemia, bleeding disorder, fever,
headache, infections, delayed healing
• Lymphadenopathy, swollen and bleeding gingiva, abdominal pain
23. Chronic myeloid leukemia Adults above 65 years, mutation in Philadelphia chromosome
• Increased fatigability, moderate anemia, weakness, weight loss
• Splenomegaly, enlargement of salivary gland and tonsils, petechiae,
ecchymoses, leukemids (papules, pustules, bullae), Blast crisis (change to an
acute phase)
• Gingival infiltration (most common in acute monocytic type), gingival
hyperplasia, petechiae, ulceration of mucosa, rapid loosening of teeth due to
necrosis
• WBC predominantly neutrophils, megakaryocytes, seablue histiocytes or
macrophages with abundant green-blue cytoplasm
24. Myelophthisic anemia Crowding of normal hematopoietic cells by malignant proliferation
25. Myeloid sarcoma Tumor-like collection of leukemic cells in extramedullary sites
(Granulocytic sarcoma,
extramedullary myeloid
tumor, chloroma)
Diseases of Blood Platelets
26. Thrombocytopenia Reduced blood platelet count below 1,50,000 cells/mm3
• Occurs due to reduced production, increased destruction, increased
sequestration in spleen
• Bleeding into the tissues, skin, and mucosa

https://t.me/DentalBooksWorld
276 Triumph's Complete Review of Dentistry

27. Thrombocytopenic purpura • Primary – Idiopathic


• Common in children, purpura, hemorrhagic lesions of skin, pinpoint
petechiae to massive hematomas, spontaneous bleeding from nose, urinary
tract, and GIT
• Palatal petechiae, severe gingival hemorrhage
28. Thrombocytopathia • Qualitative defects of platelet function
• Disorders of platelet adhesion – Bernard–Soulier syndrome, deficiency of
platelet membrane glycoprotein complex Ib-IX, Von Willebrand disease
(defect in vWF)
• Disorders of platelet aggregation – Glanzmann thrombasthenia, defect in
glycoprotein IIb/IIIa complex
• Disorders of platelet secretion/activation – Defective release of thromboxane
and granule bound ADP
29. Thrombocythemia Increase in number of platelets
(Thrombocytosis) Primary – Idiopathic
Secondary – In response to other diseases like iron deficiency anemia, hemolytic
anemia, splenectomy, tuberculosis, malignant tumors, and drugs
Diseases of Specific Blood Factors
30. Hemophilia X-linked recessive trait
• Hemophilia A – Deficiency of factor VIII
• Hemophilia B – Deficiency of factor IX (Christmas disease)
• Persistent bleeding, internal bleeding, spontaneous bleeding into joints and
muscles (hemarthrosis)
• Prolonged gingival bleeding, mandibular pseudotumor
• Lab findings: Prolonged clotting time, prolonged aPTT
• Treatment: Factor VIII concentrate or IX, intraligamentary/infiltration/
intra-papillary injection for extraction
31. Von Willebrand disease • vWF factor deficiency, most common inherited bleeding disorder
(pseudohemophilia, vascular • vWF helps to form platelet plug
hemophilia)
32. Plasminogen deficiency Mutation in PLG gene
• Defective plasminogen leads to improper breakdown of fibrin
• Ligneous conjunctivitis (wood-like thick firm plaques) leading to scarring
and loss of vision
• Oral and nasal mucous membrane develop nodular ulcerations and gingival
hyperplasia
33. Factor I Afibrinogenemia, hyperfibrinogenemia, dysfibrinogenemia
Miscellaneous
34. Lymphoid hyperplasia Enlargement of lymphoid tissue
• ‘Kissing tonsils’ enlargement of tonsils in Waldeyer’s ring leading to occlusion
of airway
• ‘Tingible bodies’ presence of phagocytozed material in macrophages

DISEASES OF NERVE
1. Trigeminal neuralgia • Peripheral injury or disease of the trigeminal nerve, compression of trigeminal nerve by tumors
(Tic Doloureau, or vascular aneurysms, abnormal vessels, aneurysms, tumors, chronic meningeal inflammation
trifacial neuralgia, or other lesions may irritate trigeminal nerve roots along the pons, demyelination due to
Fothergill’s disease) multiple sclerosis especially in younger individuals, lesion within the central nervous system
• Rarely dental fillings composed of dissimilar metals (galvanism) can trigger attacks,
periodontal disease, traumatogenic occlusion have also been suggested as causes
• Slight female predominance: 1.74 :1, peak incidence 60–70
• ‘Trigger zone’ on the face – vermilion border of lip, alae of the nose, cheeks, and around the eyes

https://t.me/DentalBooksWorld
Chapter 4 • Oral Pathology and Oral Medicine 277

• Spasmodic contractions of the facial muscles, excruciating pain appears for a few seconds and
lasts for several minutes
• Focal areas of myelin degeneration may be present. Multiple sclerosis patients tends to show
unique amorphous plaques
• Symptomatic treatment
Medical treatment:
• Carbamazepine should be the initial Rx of choice for classical Trigeminal Neuralgia
• Baclofen: 10–80 mg daily, Dilantin, Lamictal, Neurontin, Topamax, Klonopin
Surgical management:
• Both percutaneous and open techniques, glycerol injection, balloon compression, radio
rhizotomy, gamma knife, partial rhizotomy, microvascular, and decompression
2. Trigeminal neuritis • Dental surgical procedure: Pressure of a denture on the dental nerve, tumors of the head and
(trigeminal neck, intracranial aneurysms
neuropathy) • Presents with ache which is burning, boring, pulling in nature
• Continues over a period of hours, days, or weeks
3. Sphenopalatine Periodicity due to hypothalamic hormonal influences
neuralgia (Cluster • Pain is generated at the level of the pericarotid/cavernous sinus complex
head ache, Horton’s • Sympathetic and parasympathetic input from the brainstem mediating an autonomic
syndrome, Vidian phenomenon
nerve neuralgia, • Termed alarm clock head ache
lower half head ache, • Males are more affected, over the age of 40 years
histamine cephalgia, • Rapid onset, persists for 15 minutes, no trigger zone
atypical facial • Sneezing, swelling of the nasal mucosa and severe nasal discharge, epiphora (watering of
neuralgia eyes), blood shot eyes
• Paresthetic sensations over the skin over the lower half of the face
• Treatment – Cocainization of the sphenopalatine ganglion-ergotamine
• Combination with antiserotonin agent – Methysergide – exerts synergistic action
4. Glossopharyngeal Neural ischemia
neuralgia • Trigger zone in the posterior oropharynx or tonsillar fossa
• Unilateral
• Trigger zones stimulation happens with swallowing, talking, yawning, or coughing
• Sharp, shooting pain in the ear, pharynx, nasopharynx, tonsil, or the posterior portion of the tongue
• Pain is paroxysmal, rapidly subsiding type of pain
• Resection of the extracranial portion or intracranial portion of the nerve
5. Bells palsy • Idiopathic, HSV infection, inflammation of the facial nerve while it passes through the
(seventh nerve temporal bone. Inflammatory, demyelinating, ischemic, or compressive processes may impair
paralysis, facial neural conduction at this unique anatomic site
paralysis) • Females are more affected, middle-aged adults
• Dropping of the corner of the mouth, dribbling of saliva, watering of the eye, inability to close
or wink the eye (leads to infection), mask-like or expressionless appearance, difficulty in
speech and eating, rarely taste sensation of the anterior two-third is lost or altered
• Treatment: Use of vasodilator drugs-histamine-physiologic flushing doses of nicotinic acid if
started within a week of onset
6. Auriculotemporal • Damage to the auriculotemporal nerve and subsequent reinnervation of sweat glands by
nerve syndrome parasympathetic salivary fibers
(Frey’s syndrome, • Surgery resulting in damage of auriculotemporal nerve
gustatory swelling) • Nerve regenerates with parasympathetic fibers innervating the salivary gland
• Parotitis, parotid abscess, parotid tumor, ramus resection
• Exhibits flushing and sweating on the involved side of faces, especially in the temporal region
during eating
• profuse sweating may be induced by pilocarpine and eliminated by atropine
• Crocodile tears – Salivary lacrimal reflex arc
• Intracranial division of auriculotemporal nerve

https://t.me/DentalBooksWorld
278 Triumph's Complete Review of Dentistry

7. Burning mouth • Local causes – Dry mouth, geographic tongue, Lichen planus, tongue thrusting, GERD,
syndrome sensory nerve damage
• Systemic causes – Vit B12 deficiency, ACE inhibitor, Sjogren’s syndrome, psychogenic
disorders, psychosocial stresses, diabetes mellitus
• Burning sensation continuous or intermittent discomfort, increased thirst, altered taste
sensation, reduction in taste perception, associated anxiety, and depression
• Continuous or intermittent disturbance in lip commonly the tongue followed by lips and
palate, sensation of dryness may also be present
• Associated anxiety or depression may be present
• Treatment: Antidepressants, vitamins, dietary mouthwashes, salivary substituents
8. Orolingual paresthesia • Local and systemic factor, deficiency states-pellagra, diabetes, gastric disturbances,
(Glossodynia, psychogenic factors, trigeminal neuralgia, periodontal disease, xerostomia, hypothyroidism,
Glossopyrosis) referred pain, mercurialism, Moeller’s glossitis, excessive use of tobacco, spices, local dental
causes such as dentures, irritating clasps, newly fixed bridges, galvanic current between two
dissimilar metals, TMJ disorders
• Psychogenic factors
• Affects women more commonly, tongue is commonly affected, paresthetic sensations are
evident
• Treatment: Topical anesthetics, analgesics, smooth and skeletal muscle relaxants, sedatives,
vitamins, salivary stimulants
9. Motor system disease • Corticospinal and anterior horn cell degeneration
• Males are more affected, mostly during childhood
• Motor neuron disease constitutes three closely related conditions such as progressive
muscular dystrophy, amyotrophic lateral sclerosis, progressive bulbar palsy
• Progressive muscular atrophy-foot drop, steppage gait, stork legs
• Amyotrophic lateral sclerosis-atrophy and fasciculations of tongue (a brief spontaneous
contraction affecting a small number of muscle fibers, often causing a flicker of movement
under the skin/mucosa)
• Amyotrophic lateral sclerosis-weakness, spasticity of limbs, difficulty in swallowing, talking
with indistinct speech and hoarseness,
• Progressive bulbar palsy-difficulty in phonation, hoarseness, facial weakness, weakness
during mastication
• No specific treatment, The disease is often fatal, sometimes presenting with temporary remissions
10. Multiple sclerosis • Autoimmune reaction, HHV-6, Chlamydia pneumonia
(disseminated • Affects 20–40 years of age, females are more affected
sclerosis) • Visual impairment due to retrobulbar neuritis, nystagmus, diplopia, bladder/rectal
incontinence or retention
• Charcot’s Triad – Intentional tremor, nystagmus, dysarthria or scanning speck, imperfect
speech articulation, staccato type of speech, Bell’s palsy, trigeminal neuralgia
• Fatigability, stiffness of extremities, ataxia, superficial or deep paresthesia
• No treatment
11. Migraine • Changes in the brainstem and its interactions with the trigeminal nerve, a major pain pathway,
imbalances in brain chemicals , including serotonin, which helps regulate pain
• Hormonal changes in females
• Prodrome – One or two days before a migraine, you may notice subtle changes that signify
an oncoming migraine, including constipation
• Depression, food cravings, hyperactivity, irritability, neck stiffness, uncontrollable yawning
• Aura – Aura may occur before or during migraine headaches. Visual phenomena, such as
seeing various shapes, bright spots or flashes of light, vision loss, pins and needles sensations
in an arm or leg, speech or language problems (aphasia), less commonly, an aura may be
associated with limb weakness (hemiplegic migraine)
• Attack – A migraine usually lasts from four to 72 hours, pain that has a pulsating, throbbing
quality, sensitivity to light, sounds and sometimes smells, nausea and vomiting, blurred
vision, lightheadedness, sometimes followed by fainting

https://t.me/DentalBooksWorld
Chapter 4 • Oral Pathology and Oral Medicine 279

• Postdrome – During this time you may feel drained and washed out, though some people
report feeling mildly euphoric
• Treatment – Aspirin, or nonsteroidal anti-inflammatory drugs (NSAIDs) Drugs marketed
specifically for migraines, such as the combination of acetaminophen, aspirin and caffeine
(Excedrin Migraine), also may ease moderate migraine pain, but aren't effective alone for
severe migraines
12. Giant cell arteritis • Common form of systemic vasculitis in adults, GCA and  polymyalgia rheumatica  to be
(temporal arteritis) different manifestations of the same disease process
• Adults are affected, visual disturbances, headache, jaw claudication, neck pain, and scalp
tenderness. Constitutional manifestations, such as fatigue, malaise, and fever, may also be
present. Intimal proliferation with resulting luminal stenosis, disruption of internal elastic
lumina by mononuclear infiltrate, invasion, and necrosis of the media progressing to
panarteritic involvement by mononuclear cells, giant cell formation along with granulomata
• Treatment – Corticosteroid therapy
13. Complex regional pain • Partial peripheral nerve injuries, caused by brachial plexus injuries. Injury – An abnormal
syndrome (causalgia, sympathetic response, predisposing personality, extraction of a multirooted tooth
reflex sympathetic • Severe cases are called major causalgia. Minor causalgia describes less severe forms, similar
dystrophy syndrome) to reflex sympathetic dystrophy (RSD). RSD includes muscular and joint pain symptoms, and
changes in bone density
• Pain at the site of injury, attack may be initiated by actual touch and also by emotional
disturbances
• Treatment: Injection of procaine, alcohol nerve block, phenol cauterization, surgical curettage
of bone
14. Atypical facial pain • No specific cause, injury of any peripheral or proximal branch of trigeminal nerve
(atypical facial • Vague, deep, poorly localized pain in the regions supplied by the fifth and ninth cranial nerves
neuralgia, facial • Lacks a trigger zone, constant and persists for weeks, months, or even years
causalgia) • Pain in trigeminal territory
• Treatment: Nonnarcotic drug, antidepressant
15. Eagle’s syndrome • Elongated styloid process or ossification of the stylohyoid ligament
• Dysphagia, sore throat, otalgia, glossodynia
• Surgical management
16. Horner’s syndrome • Miosis
(sympathetic • Ptosis
ophthalmoplegia) • Anhidrosis
• Vasodilatation
17. Marcus Gunn Jaw • Aberrant connection exists between the motor branches of the trigeminal nerve innervating
Winking syndrome the external pterygoid muscle and the fibers of the superior division of the oculomotor nerve
(Trigeminal that innervates the levator superioris muscle of the upper eyelid
oculomotor • Congenital unilateral ptosis with rapid elevation of the ptotic eyelid on movement of mandible
synkinesis) to the contralateral side
• Synkinetic ptosis (synkinesis refers to the abnormal involuntary facial movement that occurs
with voluntary movement of a different facial muscle group)
18. Inverted Marcus– • Seen after peripheral facial nerve paralysis
Gunn Phenomenon • Eye closes when patient opens mouth forcefully
– Marin–Amat • Occurs after peripheral facial paralysis
syndrome
DISEASES OF THE MUSCLE
Dystrophies
19. Severe generalized • Autosomal dominant
familial muscular • Occurs in males (female carriers)
dystrophy • Inability to walk or run, frequent fall due to muscle weakness, muscular enlargement leading
(Pseudohypertrophic to atrophy, limbs appear flaccid
muscular dystrophy of
Duchenne)

https://t.me/DentalBooksWorld
280 Triumph's Complete Review of Dentistry

• Types: Severe generalized familial muscular dystrophy, mild restricted muscular dystrophy,
Facioscapulohumeral dystrophy of Landouzy and Dejerine
• Slowly progressive proximal myopathy involving muscle of face and shoulder, inability to
raise arms above the head and inability to close the eyes, tapir lips, myopathic facies inability
to whistle or smile, cardiac abnormalities, including cardiomegaly, tachycardia, death due to
cardiac failure
• No treatment
20. Mild restricted • Inherited disorder, autosomal dominant inheritance
muscular dystrophy • Inability to raise the arms and close the eyes, lips have developed looseness, tapir lips, cardiac
(Facioscapulohumeral abnormalities
dystrophy of • No treatment
Landouzy and
Dejerine)
Myotonia
21. Dystrophic myotonia • Steadily progressive familial
(myotonic dystrophy, • Third decade of life
dystrophia myotonica) • Distal myopathy with associated weakness of the muscles of the face, jaw and neck, and
levators of the eyelids, testicular atrophy
• Ptosis of eyelids, atrophy of masseter and sternocleidomastoid muscles, Myopathic Facies,
Swan Neck, nasal type of voice due to weakening of pharyngeal and laryngeal muscles,
recurrent dislocation of jaws, hypothyroidism, slow pulse, loss of hair
• Enlargement of scattered muscle fibers and presence of centrally placed muscle nuclei in long
rows, isolated fibers show degenerative changes
• No treatment
22. Congenital myotonia • Inherited disorder, onset in early childhood, difficulty to stand and walk
(Thomsen’s disease, • Muscle contraction induces severe, painless muscular spasms with a delay in relaxation
myotonia congenital) • Percussion contraction – Prolonged contraction with electrical or physical stimulation
• Herculean Appearance – Muscles of the leg, arm, shoulder, neck, and masseter (tongue is not
involved) – Strabismus (convergent)-spasm of the extraocular muscles, prolonged spasms of
the facial muscles with sneezing
• No specific alterations except hypertrophy of all muscle fibers
• No specific treatment
23. Acquired myotonia • Intense spasms of the muscles
• Intermittent spasms – Clonus (myoclonic)
• Constant spasms – Trismus (myotonic)
• Occur in pericoronitis, infectious myositis, hysterical trismus
24. Hemifacial spasm • Unknown etiology, adult females are affected
(facial myoclonus, • Repeated, rapid, painless, irregular, nonrhythmic, uncontrollable, unilateral contractures of
facial dystonia) the facial muscles
• Compression of the facial nerve in the facial canal, brief transitory twitching, may progress
to sustained spasms
• Treatment: Decompression of facial nerve can give relief
25. Periodic paralyses • Hereditary, alteration in serum phosphatase
(paramyotonia) • Episodes of flaccid muscle weakness, eyelids are closed, face – mask like, tongue cramping
after drinking cold drinks, tongue myotonia induced by percussion
• No treatment
26. Hypotonia • Congenital causes like CNS defects, glycogen storage diseases, cretinism, mongolism, neuromas
• Reduced tendon reflexes, muscular weakness
• Floppy infant syndrome

https://t.me/DentalBooksWorld
Chapter 4 • Oral Pathology and Oral Medicine 281

Myasthenias
27. Myasthenia gravis • Acquired autoimmune disorder
• Middle-aged females
• Weakness of voluntary muscle, speech is slow and slurred, disturbance in taste sensation,
diplopia and ptosis, focal collections of small lymphocytes or “lymphorrhages”
• Treatment: Physostigmine (anticholinesterase)
Myositis
28. Dermatomyositis • Idiopathic inflammatory myositis
(Juvenile • Fifth decade of life
dermatomyositis, • Progressive proximal symmetrical muscle weakness, calcinosis universalis. Stomatitis,
childhood pharyngitis
dermatomyositis, • Muscle fibers demonstrate degeneration and hyalinization
polymyositis) • Symptomatic treatment
Heterotopic Ossification
29. Myositis ossificans • Unknown etiology, young children and adolescents
progressiva • Skeletal muscle is affected, petrified man, muscle tissue is replaced by connective tissue which
undergoes osteoid formation
• No treatment
30. Traumatic myositis • Traumatization of periosteum, activation of periosteal implants
ossificans • Metaplasia of pluripotential intermuscular connective tissue into bone, metaplasia of
fibrocartilage
• Firm painful mass in injured males
• Radiograph shows feathery time of calcification
• Hemorrhage, degeneration of muscle, connective tissue hyperplasia, osteoid and trabeculae
formation
• Surgical excision
31. Neurogenic • Heterotopic calcification in patients with spinal cord injury
heterotopic • Severe neurological disorders including stroke, encephalitis, polio, tetanus, tabes dorsalis
ossification
32. Proliferative myositis • Pseudosarcomatous proliferation of muscle
• 50 years of median age
• Firm solitary nodule which is seldom tender or painful
• Fibroblastic proliferation, involving epimysium, perimysium and endomysium
• Treatment: Local excision
33. Congenital facial • Nonfamilial, deficient development of cranial muscles, defective flow in basilar artery
diplegia (Möbius • Difficulty in mastication, associated congenital deformities, prominent everted lips
syndrome) • No treatment

FORENSIC ODONTOLOGY
Terminologies
1. Forensic science Refers to all the legal means which tend to prove or disprove any fact, the truth of which is
submitted to judicial investigation
2. Forensic odontology Odontology refers to the study of teeth, and in effect denotes dentistry. Forensic odontology,
therefore, has been defined by the Federation Dentaire Internationale (FDI) as ‘that branch of
dentistry which, in the interest of justice, deals with the proper handling and examination
of dental evidence, and with the proper evaluation and presentation of dental findings’
3. Daubert standard The Daubert standard is a rule of evidence regarding the admissibility of expert witnesses’
testimony during the United States federal legal proceedings

https://t.me/DentalBooksWorld
282 Triumph's Complete Review of Dentistry

4. Evidence The law of evidence governs the use of testimony (e.g., oral or written statements, such as an
affidavit) and exhibits (e.g., physical objects) or other documentary material which is admissible
(i.e., allowed to be considered by the Trier of fact, such as jury) in a judicial or administrative
proceeding (e.g., a court of law)
5. Expert witness Are those whose training, qualifications, or experience enables them to give an opinion on a
relevant matter where the ordinary person is not so enabled professional, unbiased, ethical, and
truthful
6. Forensic odontology Forensic dentistry or forensic odontology is the proper handling, examination, and evaluation
of dental evidence, which will be then presented in the interest of justice. The evidence that may
be derived from teeth is the age (in children) and identification of the person to whom the teeth
belong. This is done using dental records or ante-mortem (prior to death) photographs

Roles of a Forensic Odontologist


1. Identification of found human remains
2. Identification in mass fatalities
3. Assessment of bite mark injuries
4. Assessment of cases of abuse (child, spousal, elder)
5. Civil cases involving malpractice
6. Age estimation
7. Dental identification
8. Morphology and arrangement of teeth vary from one person to another
9. Teeth are resistant to environmental insults and susceptible to physiologic and pathological changes – During life

Relevance of Forensic Dentistry


1. Dental restorations Dental restorative product contains inorganic materials, trace element,
and fillers that are unique to that product and can be detected by X-ray
fluorescence (XRF)
2. Uniqueness of morphology of teeth The total of all characteristics of the teeth and their associated structures
which, while not individually unique, when considered together provide
a unique totality

Dental identification procedures


1.  Comparative identification
2.  Reconstructive identification
Comparative identification
• Oral autopsy
–– Autopsy (necropsy or postmortem examination) is done usually to determine the cause of death. It involves examining
the deceased usually with dissection to expose the organs
–– Additional challenges faced in cases of incinerated remains are – They need to be reinforced with cyanoacrylate glue prior
to examination since teeth may be brittle following exposure to prolonged heat
–– According to Griffiths and Bellamy, access for radiography in incinerated bodies can be obtained by removing the tongue
and contents of the floor of the mouth in a ‘tunneling’ fashion from beneath the chin
Obtaining dental records – Comparing post and ante-mortem dental date. Comparative identification involves four steps
namely:
1. Positive identification: This indicates that the ante-mortem and post-mortem data match each other
2. P
 robable identification: There is a high level of concordance between the two sets of date but may lack radiographic
support

https://t.me/DentalBooksWorld
Chapter 4 • Oral Pathology and Oral Medicine 283

3. P
 ossible identification: The post and the ante mortem records are in agreement but the available information is insufficient,
usually in terms of quality
4. Excludes identification: Both the recorded data are inconsistent

High Yield Facts


Modified interpol postmortem odontogram Pink in color (NEET 2017 QUESTION)
Modified interpol antemortem odontogram Yellow in color

Dental Identification Methods


Ameloglyphics • Tooth prints are the pattern formed by the enamel rod ends at the crown surface of the tooth
• Recorded the enamel rod end pattern using acetate peel technique
• Doubts regarding its forensic value are raised since enamel undergoes regressive changes and the
courses taken by the enamel rods vary at different levels of the enamel (Manjunath et al.)
(NEET 2018 QUESTION)
Palatal rugae • Ridges on the anterior part of the palatal mucosa on each side of the mid palatine raphe behind the
incisive papilla
• Well-protected in fire accidents and high-impact trauma
• Identification in edentulous individuals
• They do not change shape with age and reappear after trauma or surgical procedures
Classification – Primary >5 mm/Secondary 3–5 mm/Fragmentary 2–3 mm
Types – Branched, unified, crosslinked, annular, papillary
Analysis of Rugae Patterns
• Trace the patterns on a clear acetate sheet
• Superimpose these tracings on photographs of plaster models
• Computerized analysis – RUGFP-ID-using digitized images of the palate – 93% accuracy
• Points are placed on the medial and the lateral extremities of all the rugae
• Plotted points are processed by the software and stored
• Comparison of the ante and postmortem rugae is done
Lip prints Imprint of the wrinkles and grooves in the lips
Individualistic similar to fingerprints
I-Classification
1. Simple wrinkles – Straight line/curved line/angled line sine-shaped line
2. Compound wrinkles – Bifurcated/trifurcated/anomalous
II-Classification
Type I – Vertical grooves run across the entire lip
Type II – Similar to type I but do not cover the entire lip
Type III – Intersected grooves
Type IV – Reticular grooves
Type V – Grooves that cannot be morphologically differentiated
Revealing latent lip prints
Invisible lip prints can be made visible by applying aluminum powder and magnetic powder
Reliability
• Scarring due to surgical procedures
• Differs with differences with pressure and its direction

https://t.me/DentalBooksWorld
284 Triumph's Complete Review of Dentistry

Type I Type I’

Type II Type III

Type IV Type V

Bite marks Bite marks has been defined by Macdonald as “a mark caused by teeth either alone or in combination
with other mouth parts”
Macdonald’s classification – Tooth pressure mark/tongue pressure marks/tooth scrape marks
Webster’s –
• Type I – The food item fractures readily with limited depth of tooth penetration (hard chocolates)
• Type II – Fracture of fragment of food item with considered penetration of teeth (bite on apples and
fruits)
• Type III – Complete penetration of the food items with slide marks (cheese, banana)
Bite marks evidence collection from the victim:
• Visual examination
• Photography
• Saliva swab
• Impressions
Bite mark analysis may have one of the concluding statements suggested by levine and the ABFO
Definite biter: Characteristics match between the bite mark dimensions
Probable biter: It shows some degree of specificity to suspects teeth
Possible biter: There is similarity of class characteristics
Not the biter: Bite mark and suspect teeth are not consistent
(*SELF – Inflicted bite marks are observed in Lesch–Nyhan syndrome – X-linked)
Difference between Feature Human Carnivores
human bite and Arch size and Shape Broad, U-shaped Narrow anterior aspect, V-shaped
animal bite
Teeth Broad centrals and narrow laterals Broad lateral and narrow centrals
Injury pattern Commonly bruising Severe laceration

Dental Profiling
Shoveling Shoveling refers to the presence of mesial and distal marginal ridges on the lingual surface of the
maxillary and mandibular anterior teeth
The maxillary central incisors are the recommended teeth for observing the trait in assessing
population differences
Carabelli’s trait The Carabelli’s cusp or tubercle of carabelli is a cingular derivative expressed on the mesiopalatal
or palatal aspect of the mesiopalatal cusp of maxillary molars
In Indians it is reported to be present in 26% of the population

https://t.me/DentalBooksWorld
Chapter 4 • Oral Pathology and Oral Medicine 285

Cusp 5 This is characterized by the presence of occlusal tubercles on the distal marginal ridge of maxillary
ridge of maxillary molars, particularly first molar
An incidence of 75% is observed in Indians
Sex determinants The use of morphological features of the skull and mandible is a common approach used by
anthropologists in sex identification

Morphological Differences in Skull


Skull features Male Female
Size/architecture Big/rugged Small/smooth
Frontal/parietal eminence Small Large
Forehead Sloping Vertical
Orbits Squarish Rounded
Occipital condyles Large Small
Mandible Large, Broad ascending ramus Small, narrow ascending ramus
Condyles Large Small
Gonial angle Less obtuse More obtuse

Dental Age Estimation Facts


1. The primary teeth begin to calcify at approximately 12–14 weeks in utero
2. The enamel formation of all deciduous teeth is usually completed by the first year
3. Among permanent teeth, first molar commences calcification around the time of birth
4. The neonatal line is considered as an indicator of live birth and is formed due to the slowing down of enamel
prism growth rate thus creating an apparent line of demarcation
5. The neonatal line may take up to 3 weeks after birth to form
Age Estimation Methods in Children and Adolescents
Schour and The chart of Schour and Massler’s method was probably the first attempt at dental age estimation. It describes
Massler’s 20 chronological stages of tooth development starting from 5 months IU until 21 years of age
method
Demirjian’s • This method analysis involves the mandibular left-sided teeth. (NEET 2017 QUESTION)
method • This is the most widely used technique for assessing age in children and adolescents
• A modification was made in this method to include third molar tooth
• In the modified method, then calcification of the teeth is divided into 10 stages and numbered from 0 to 9
–– Stage 0 – Calcification yet to begin
–– Stage 5 – Crown completed
–– Stage 9 – Completion of tooth calcification
Gustafson’s • In 1950, Gustafson proposed a method for age estimation based on morphological and histological changes
method of the teeth
• The method assessed changes such as, attrition (A), secondary dentin deposition (S), apical migration of
periodontal attachment (P), cementum apposition at the root apex (C), root resorption at the apex (R), root
dentin translucency (T)
• Gustafson assigned four grades ranging from 0 to 3
• Adding the allotted grade for each variable, a total score X is estimated
• Age estimation was done using the formula, age = 11.43 + (4.56 × X)
• Root dentin translucency (T) is considered as the standard tool among various parameters of Gustafson
method
• Root dentin is considered to start becoming translucent in the third decade of life, beginning at the apex,
advancing toward the cemento enamel junction

https://t.me/DentalBooksWorld
286 Triumph's Complete Review of Dentistry

Sex determination
Methods:
• Data from morphology of skull and mandible
• Tooth measurements
• DNA analysis of teeth
Dental index Mandibular canine index (MCI) = Mesiodistal crown width of mandibular canine
Mandibular intercanine arch width
Standard MCI = (mean female MCI-SD) + (mean female MCI + SD) 2
DNA analysis of • Highly accurate
teeth • Determination can be done with minute quantities
• Amelogenin gene located on X and Y chromosomes
Objective:
• To extract DNA from pulp of teeth subjected to different environmental conditions
• To quantitatively estimate the amount of DNA that can be extracted from dental pulp
• To determine sex of the individual from dental pulp

QUICK FACTS

S. No. Term Explanation


1. Plagiocephaly Asymmetric oblique-shaped head
2. Pastia lines Rash in skin folds – Scarlet fever
3. Koplik’s spots Intraoral lesions of measles
4. Drusenfieber Glandular fever (infectious mononucleosis)
5. Platybasia Softening of the base of the skull
6. Kleeblattschadel Clover leaf-shaped skull (craniosynostosis)
7. Acrobrachycephaly Tower-shaped skull (craniosynostosis)
8. Rh hump Ring like defect in teeth in erythroblastosis fetalis
9. Cerebriform tongue Sulci and gyri pattern in tongue (pemphigus vegetans)
10. Asboe-Hansen sign Blister spread sign – Enlarging a small bulla with finger pressure
11. Auspitz sign Pinpoint bleeding on removal of scales
12. Barnett’s sign (scleroderma neck sign) Ridging and tightening of the skin of the neck on extending the
head with a visible and palpable tight band over platysma in the
hyperextended neck
13. Buschke–Ollendorff sign Deep dermal tenderness on pressing the lesion of secondary
syphilis and cutaneous vasculitis
14. Button hole sign Invagination of the neurofibroma – Type 1 neurofibromatosis
15. Carpet tack sign/tin tack sign, cat tongue Exposure of horny plugs on removal of scales in discoid lupus
sign erythematosus
16. Crowe sign Axillary freckling in neurofibromatosis type I
17. Flag sign Alternating pigmented and hypopigmented bands in Kwashiorkor
and marasmus
18. Gorlin sign Ability to touch the tip of the nose with the tongue – Ehlers–
Danlos sign
19. Hanging curtain sign Pityriasis rosea
20. Herald patch Pityriasis rosea

https://t.me/DentalBooksWorld
Chapter 4 • Oral Pathology and Oral Medicine 287

21. Hertoghe sign (Queen Anne’s sign) Loss of lateral one-third of eyebrow – Leprosy, ectodermal
dysplasia, DLE
22. Higoumenaki sign Enlargement of the sternal end of the (right) clavicle – Congenital
syphilis
23. Hypopyon sign Secondary infection of vesicles in vesiculobullous lesions
24. Ingram’s sign Inability to retract the lower eye-lid in patients of progressive
systemic sclerosis 
25. Jellinek sign (Rasin sign) Pigmentation of the eyelids in hyperthyroidism
26. Meffert’s sign Fordyce disease – Lipstick-like mark on the mug of hot beverage
in case of lip lesions
27. Millian’s ear sign Involvement of ear in erysipelas
28 Stafne’s sign Widening of the periodontal ligament space secondary to increase
in the collagen synthesis in progressive systemic sclerosis
29. Steinberg sign Protrusion of thumb beyond ulnar border in Marfan syndrome
30. Sand on a beach after an ebbing tide Smokeless tobacco keratosis
31. Dried mud appearance Nicotine palatinus
32. Sand paper-like texture Actinic keratosis, scarlet fever
33. Chocolate-colored cystic fluid Warthin’s tumor
34. Straw berry and raspberry tongue Scarlet fever
35. Hebra nose Rhinoscleroma
36. Cobble stone appearance Pyostomatitis vegetans
37. Table salt-like appearance (Koplik’s Measles
spots)
38. Saddle back fever Chikungunya
39. Lilly of the valley Candidiasis
40. Butterfly vertebrae Craniofacial dysostosis (Crouzon syndrome)
41. Mitten hands, sock feet Apert syndrome
42. Pencil-shaped small elongated Iron deficiency anemia
erythrocytes
43. Brazilian wild fire Pemphigus foliaceus
44. Coup de sabre Linear scleroderma
45. Herculean appearance Congenital myotonia
46. Coast of Maine Café au lait spot irregular margin in fibrous dysplasia
47. Coast of California Café au lait spot smooth margin in neurofibromatosis
48. POEMS HHV-negative lymphoid hamartoma, polyneuropathy,
organomegaly, endocrinopathy, monoclonal plasma cell disorder,
and skin changes. POEMS is a paraneoplastic syndrome
49. Corn flakes glued to the skin surface Impetigo
50. Saint Anthony’s fire Erysipelas
51. Peau d’orange (orange peel) in skin Erysipelas
52. Bacillary peliosis hepatis Hepatosplenic Bartonella disease (cat scratch disease)
53. Stork legs/steppage gait/foot drop Progressive muscular atrophy
54. Charcot triad Multiple sclerosis, intentional tremor, nystagmus, dysarthria
55. Shepherd crook deformity/hockey stick Polyostotic fibrous dysplasia
deformity, coxa vara

https://t.me/DentalBooksWorld
288 Triumph's Complete Review of Dentistry

56. Parulis (fistula granuloma/gum boil) Exophytic granulation tissue mass found at the opening of the sinus 
tract of a peri-apical or periodontal fistula
57. Forscheimer sign Rubella
58. Pseudocleft palate Apert syndrome
59. Perimolysis Erosion of teeth due to gastric secretion
60. Attrition Postdevelopmental loss of tooth structure due to physiological
wearing away
61. Abrasion Postdevelopmental loss of tooth structure due to pathological
wearing away
62. Erosion Loss of tooth structure due to nonbacterial chemical action
63. Abfraction Loss of tooth structure due to repeated tooth flexure
64. Demastication Increase in tooth wear due to chewing of an abrasive substance
produces a combined effect of attrition and abrasion
65. Pink tooth of Mummery Pink tooth due to internal resorption
66. Parasitosis/formication Neurosis (drug addicts) which feels like snake or insect crawling
under skin
67. Argyria Chronic silver intoxication
68. Acrodynia/Pink disease/Swift-Feer Chronic mercury exposure in infants and children
disease
69. Erethism Neurologic symptoms in mercury poisoning
70. Burton line Bluish line in Gingiva-lead poisoning
71. Chyriasis Slate blue discoloration of sun-exposed skin – Chronic gold
therapy
72. Hamman’s crunch Mediastinal involvement in emphysema
73. Myospherulosis Foreign body reaction to a topical antibiotic placed in surgical site
74. Sutton’s disease (periadenitis mucosa Major aphthous ulcers
necrotica recurrens)
75. Lupus pernio Violaceous indurated lesions of skin in sarcoidosis
76. Erythema nodosum Tender erythematous nodules in sarcoidosis
77. Keratin horn Verruca vulgaris
78. Leser–Trelat sign Multiple sudden seborrheic keratosis with pruritus as a feature of
internal malignancy
79. Ackerman tumor Snuff dippers cancer, verrucous carcinoma
80. AEIOU Acronym (Merkel cell Asymptomatic, expanding, immunosuppression, old age,
carcinoma) ultraviolet exposed
81. Kuttner tumor Marginal zone B-cell lymphoma arising in chronic sclerosing
sialadenitis
82. Frenal tag Most common fibrous hyperplasia
83. Bag of worms Neurofibroma plexiform variant
84. Lisch nodules Brown-pigmented spots in iris (iris hamartomas) in
neurofibromatosis (NF1)
85. Triton tumor Aggressive tumor made up of both malignant schwannoma cells
and malignant rhabdomyoblasts
86. Ligneous conjunctivitis Plasminogen deficiency
87. Eyes turned to heaven appearance Cherubism

https://t.me/DentalBooksWorld
Chapter 4 • Oral Pathology and Oral Medicine 289

88. ODAM protein Odontogenic ameloblast associated protein (Pindborg tumor,


central odontogenic fibroma)
89. Shagreen patches. Ash leaf spots Tuberous sclerosis
90. Bronchiolitis obliterans Paraneoplastic pemphigus
91. Erlenmeyer flask deformity of long bones Gaucher disease
92. Dagger-shaped pulpal calcifications Pseudohypoparathyroidism
93. Bolt of lightening, stabbing with ice pack Trigeminal neuralgia
94. Neumann tumor Congenital epulis of newborn
95. Leaf-like denture fibroma Epulis fissuratum
96. Ghost teeth Regional odontodysplasia
97. Riga-fede disease Sublingual ulcerations due to newly erupted tooth in infants
98. Petrified man Generalized myositis ossificans
99. Rule of three 10’s 10 lb of weight, 10 mg/l of hemoglobin, age 10 weeks
100. Hutchinson’s teeth Anterior teeth affected in congenital syphilis
101. Mulberry molars Congenital syphilis
102. Tulip teeth Dentinogenesis imperfecta
103. Conditions associated with premature Ehlers–Danlos syndrome, Papillon–Lefevre syndrome,
exfoliation of primary teeth and delayed Cherubism, Hajdu–Cheney syndrome
eruption of permanent teeth
104. Conditions associated with premature Hyperthyroidism, hypophosphatasia, precocious puberty, Proteus
eruption of permanent teeth syndrome
105. Greenspan lesion Hairy leukoplakia
106. Abrikossoff tumor Granular cell myoblastoma
107. Marfanoid body, bumpy lip MEN syndrome
108. Leonine facies Paget’s disease, lepromatous leprosy
109. Washleather appearance Diphtheria
110. Iodine mumps Chemical mumps after administration of inorganic or organic iodine
111. Reddish black turbinate Mucormycosis
112. Parrot beak-like nose, frog face Crouzon syndrome
113. Target, iris, bull’s eye appearance Erythema multiforme
114. Acrodermatitis enteropathica Periorofacial and acral dermatitis, alopecia, diarrhea

PIGMENTATIONS IN ORAL CAVITY


Discolorations in tooth – Intrinsic
Cause Color
Metabolic
Erythropoietic porphyria (Gunther’s disease) Purple/brown
Hyperbilirubinemia Yellow, green (chlorodontia)
Inherited
Amelogenesis imperfecta Yellow brown to dark yellow
Dentinogenesis imperfecta Blue-brown opalescence
Iatrogenic
Dental fluorosis White, yellow, gray, or black

https://t.me/DentalBooksWorld
290 Triumph's Complete Review of Dentistry

Tetracycline Banding appearance, yellow, brown, blue, black, or gray


Ochronosis Yellow, blue-black
Trauma
Localized red blood cell breakdown in pulp Gray brown, black
Internal resorption Pink
Idiopathic
Molar incisal hypomineralization Whitish-cream, yellow-brown
Discolorations in tooth – Extrinsic
Bacterial stains Black-Actinomyces
Green-Penicillium, Aspergillus
Orange-Flavobacterium lutescens
Iron, manganese, silver Black
Mercury, lead Blue green
Copper Green
Tobacco Brown to black
Foods and beverages Brown to black
Restorative materials Amalgam – Gray-black
Composites, glass ionomer, acrylic-gray
Mouth washes • Potassium permanganate – Violet-black
• Silver nitrate – Black
• Stannous fluoride – Brown
Oral Mucosal Pigmentation
Focal
Hemangioma, varix Red-blue-purple
thrombus, hematoma
Amalgam tattoo, blue nevus Blue-gray
Melanotic macule, pigmented nevus, melanoacanthoma, Brown
melanoma
Diffuse
Endocrinopathies (hyperadrenocorticism, Cushing disease)
Genetic dysfunction – Peutz–Jeghers syndrome
Laugier–Hunziker syndrome, Cronkite -Canada syndrome, Bloom syndrome, Dunnigan syndrome, Oculo-cerebro-cutaneous
syndrome, Rothmund-Thomson syndrome
AIDS-related medications
Graves’ disease
Primary biliary cirrhosis
Vitamin B12 insufficiency

UNIQUE HISTOPATHOLOGICAL APPEARANCE


S. No. Disease Histopathological facts
1. Actinic keratosis Tear drop-shaped rete pegs
2. Keratoacanthoma Acute angle or buttress of margins of lesion with adjacent
normal epithelium
3. Basosquamous carcinoma Collision tumor

https://t.me/DentalBooksWorld
Chapter 4 • Oral Pathology and Oral Medicine 291

4. Merkel cell carcinoma Pseudoglandular, trabecular, cribriform (“swiss cheese”) and


sheet-like pattern
5. Sturge–Weber syndrome Vascular steal phenomenon
6. Fibrous histiocytoma Whorled or cartwheel pattern
Storiform or crisscross pattern
7. Myxoid liposarcoma Delicate plexiform capillaries (Chicken wire configuration)
8. Hemangiopericytoma Staghorn or antler pattern of branching blood vessels
9. Hemangiopericytoma Patternless pattern (disorganized type)
10. Ewing’s sarcoma Filigree pattern
11. Non-Hodgkin’s lymphoma Centrocytes and centroblasts
12. Hodgkin’s lymphoma Reed–Sternberg cells (owl-eye nuclei) Pennies in a plate –
multinucleated RS
13. Burkitt’s lymphoma Starry sky appearance
14. Leiomyosarcoma Cigar-shaped nuclei
15. Rhabdomyosarcoma ‘Racquet’ cell. ‘Strap’ and ‘ribbon’
16. Alveolar Soft-Part Sarcoma Pseudoalveolar or organoid pattern
17. Multiple myeloma Russell bodies, cartwheel or checker board pattern of plasma cells
18. Fibrosarcoma Herringbone pattern
19. Actinomycosis and botryomycosis Sulfur granules
20. Adenomatoid odontogenic tumor Hyaline ring
21. Calcifying cystic odontogenic tumor, odontome Ghost cells
22. Pindborg tumor Liesegang calcification
23. Pleomorphic adenoma Hyaline cells
24. Basal cell adenoma (membranous type) Reduplicated basement membrane
25. Canalicular adenoma Party wall appearance
26. Adenoid cystic carcinoma Swiss cheese or honey comb appearance
27. Polymorphous low-grade adenocarcinoma Single file appearance
28. Paget’s disease Jig saw or mosaic pattern
29. Familial benign pemphigus Dilapidated brick wall effect
30. Kaposi’s sarcoma Promontory sign – small vessel protruding into an abnormal
vascular space 
31. Munro’s abscess Intraepithelial microabscesses in psoriasis, geographic tongue
32. Grenz zone Solar elastosis
33. Schaumann bodies Degenerated lysosomes in sarcoidosis
34. Asteroid bodies Entrapped fragments of collagen
35. String of pearl Annular fluorescent pattern
36. Mitosoid cell, club-shaped rete pegs Heck’s disease (focal epithelial hyperplasia)
37. Henderson Paterson bodies (molluscum bodies) Molluscum contagiosum
38. Xanthoma cells Verruciform xanthoma, xanthogranuloma, histiocytosis
39. Pseudohorn cysts (horn cysts) Seborrheic keratosis, Acanthosis nigricans
40. Squamous eddies Squamous metaplasia of lesional cells in a whorled
appearance-Seborrheic keratosis
41. Melanophages Melasma, postinflammatory pigmentation
42. Melanin incontinence Oral melanotic macule, incontinentia pigmenti

https://t.me/DentalBooksWorld
292 Triumph's Complete Review of Dentistry

43. Kamino bodies Eosinophilic globules in the epithelium/epidermis-Spitz nevus


44. Parakeratin chevrons Tobacco pouch keratosis
45. Perinuclear dot pattern Merkel cell carcinoma – immunohistochemical expression
of cytokeratin 20
46. Zellballen Large epithelioid nests in paraganglioma
47. Syncytial appearance Granular cell tumor
48. Mycosis cells/Sezary cells, Pautrier Mycosis fungoides (plaque stage)
microabscess, cerebriform nucleus
49. Ginger root, curvilinear trabeculae Cemento-osseous dysplasia
50. Loose bodies (joint mice) Synovial chondromatosis
51. Cell-within-a-cell appearance Hereditary benign intraepithelial dyskeratosis
52. Test tube rete pegs/corps, ronds and grains Darier’s disease
53. Sea blue histiocyte Niemann–Pick disease
54. Chondral bodies, subchondral cyst Osteoarthritis
55. Rice bodies Rheumatoid arthritis of TMJ
56. Birbeck’s granules Rod-shaped or “tennis-racket” cytoplasmic organelles
with a central linear density and a striated appearance in
Langerhans cells
57. Foam cells Fat-laden macrophages
58. Howell jolly bodies RBCs with residual nuclear fragments in megaloblastic anemia
59. Rushton bodies Linear or curved hyaline bodies present in cyst exhibiting
inflammation
60. Lipschutz bodies Recurrent aphthous stomatitis – Intranuclear inclusions in
infected cells
61. Ballooning degeneration Herpetic stomatitis – Swollen infected cells with pale
eosinophilic cytoplasm and large vesiculated nuclei
62. Nikolsky sign Extension of a bulla due to application of tangential pressure
to the lateral aspect of the lesion
63. Tzanck cells Degenerated epithelial cell with pyknotic nuclei. (herpes
simplex, pemphigus vulgaris)
64. Warthin–Finkeldey giant cells Measles, lymphoma, Kimura disease, AIDS, Lupus
erythematosus
65. Pappenheimer bodies Nonheme granules containing siderocytes in sideroblastic anemia
66. Tissue microcyst Cryptococcosis – Organism with a clear halo
67. Bay cyst Not completely enclosed in the epithelial lining, but open to
the root canals
68. Cambium layer (Nicholson cambium layer) Hypercellular zone beneath the epithelium in alveolar
rhabdomyosarcoma
69. Lava flowing around boulders in ground Type I dentin dysplasia (radicular)
sections of tooth
70. Russell bodies Small, discrete, variably sized, spheric, intracytoplasmic,
acidophilic, hyaline bodies that stain deeply with fuchsine;
occur in plasma cells in chronic inflammation and
malignant disorders, and consist of immunoglobulin
71. Mott cell Plasma cells with multiple Russell bodies in their cytoplasm
72. Cornoid lamella Porokeratosis of Mibelli

https://t.me/DentalBooksWorld
Chapter 4 • Oral Pathology and Oral Medicine 293

73. Abtropfung or dropping off effect Junctional nevus


74. Munro’s abscess (epithelial microabscess – Psoriasis, benign migratory glossitis, Reiter’s syndrome
Seen in stratum corneum)
75. Eye liner sign Carcinoma in situ – A typical keratinocytes fill the entire
epithelium sparing the basal layer
76. Checker board pattern in epithelium Psoriasiform lesions, inflammatory hyperplasia (Prominent
orthokeratosis and parakeratosis)
77. Checker board pattern Spindle cell carcinoma (infiltration of tumor cells between
muscle bundles without myocyte necrosis)
78. Tombstone or Picket fence appearance Odontogenic keratocyst
79. Vickers Gorlin criteria Columnar palisaded basal cells, polarization of basal cells,
subnuclear vacuolization in cytoplasm of basal cells
80. Koilocytes HPV-altered cells with perinuclear halo and nuclear pyknosis
81. Herringbone pattern Fibrosarcoma
82. Neurofibroma Shredded carrot-like or wire loop appearance of wavy
elongated cells
83. Schwannoma Antoni A tissue – Parallel spindle cells with palisaded nuclei
(Verocay bodies)
Antoni B tissue-disorderly arrangement of spindle cells
84. Organisms branching at obtuse angles Mucormycosis
85. Organisms branching at acute angle Actinomycosis

SYNDROMES AND ASSOCIATED DISEASES


S. No. Syndrome Associated diseases
1. Parry–Romberg syndrome Hemifacial atrophy, contralateral Jacksonian epilepsy, trigeminal
neuralgia, bluish hue of skin, painless early cleft (coup de sabre)
2. Van der Woude syndrome Cleft lip, cleft palate with lip pits
3. Ascher’s syndrome Acquired double lip. Blepharochalasis, nontoxic thyroid
enlargement
4. Popliteal pterygium syndrome Paramedian lip pits, cleft lip and palate, hypodontia, syngnathia,
syndactyly, popliteal web, hypoplasia of labia majora, bifid scrotum
5. Kabuki syndrome Paramedian lip pits, cleft palate, high arched palate, arched
eyebrows, long eyelashes, long palpebral fissures, broad tip of
the nose, microcephaly, hypotonia, nystagmus, strabismus,
scoliosis, fetal finger pads, otitis media
6. Median cleft–face syndrome Hypertelorism, median cleft of the premaxilla and palate,
cranium bifidum occultum
7. Miescher–Melkersson–Rosenthal syndrome Cheilitis granulomatosa, fissured tongue (plicated tongue),
facial nerve palsy
8. Peutz–Jeghers syndrome Polyp hamartomas in intestine and melanocytic macules
9. Aglossia – microglossia syndrome Malformation of hands, fee, cleft palate, dental agenesis,
rudimentary tongue
10. Beckwith–Wiedemann syndrome Macroglossia, omphalocele, umbilical hernia, visceromegaly,
hypoglycemia
11. Gardner syndrome Intestinal polyps, osteomas, epidermoid tumors,
supernumerary teeth

https://t.me/DentalBooksWorld
294 Triumph's Complete Review of Dentistry

12. Reiter’s syndrome Triad of conjunctivitis, urethritis, arthritis


13. Behcet’s syndrome Recurrent aphthous ulcers, genital ulcers, uveitis
14. Sjogren’s syndrome Dry mouth, dry eyes, rheumatoid arthritis
15. Rubinstein–Taybi syndrome Developmental retardation, broad thumbs, great toes,
characteristic facial features, delayed or incomplete descent of
testis, stature and head circumference below fiftieth percentile,
talon cusp
16. Cowden’s syndrome Multiple hamartoma and neoplasia syndrome, facial
trichilemmomas in GIT, thyroid, CNS, musculoskeletal
abnormalities. Cutaneous marker of breast cancer
17. Muir–Torre syndrome Sebaceous adenomas, Keratoacanthoma and gastrointestinal
carcinoma
18. Ferguson Smith type and Witten–Zak syndrome Multiple keratoacanthoma, not seen in mucosal surfaces
19. Grzybowski syndrome Multiple keratoacanthoma with military appearance and seen
in mucosal surfaces
20. Laugier–Hunziker syndrome Macular mucocutaneous hyperpigmentation and
melanonychia
21. Crest syndrome Calcinosis cutis, Raynaud phenomenon, esophageal
dysfunction, sclerodactyly, telangiectasia
22. Sturge–Weber syndrome Port-wine stain, intracranial convolutional calcifications,
congenital glaucoma, exophthalmus, angioma of the choroid,
convulsive disorders, spastic hemiplegia
23. Osler–Rendu–Weber syndrome Telangiectasia, recurrent epistaxis, positive family history
24. PHACE(S) syndrome Posterior fossa brain anomalies (Dandy–Walker malformation),
hemangioma, aterial anomalies, cardiac defects, eye anomalies,
sternal cleft or supraumbilical raphe
25. Kasabach–Merritt syndrome Thrombocytopenic purpura, hemangioma, coagulopathy,
microangiopathic hemolysis, intralesional fibrinolysis
26. Maffucci syndrome Hemangiomas of the mucous membranes, dyschondroplasia
27. Von Hippel-Lindau syndrome Hemangiomas of the cerebellum and retina, angiomatosis of
the extremities
28. Klippel–Trenaunay–Weber syndrome Port wine stain, angiomatosis of the extremities
29. Oculoglandular syndrome of Parinaud Cat scratch disease with granuloma of the eye with peri-
auricular lymphadenopathy
30. Nevoid basal cell carcinoma syndrome Multiple basal cell carinomas, keratocystic odontogenic
(Gorlin–Goltz syndrome) tumors, bifid ribs
31. Maroteaux–Lamy syndrome Mucopolysaccharidosis (deficiency in arylsulfatase B) coarse
facial features, corneal clouding, joint abnormalities, various
skeletal malformations, hepatosplenomegaly, hearing loss.
Cardiac disease and restrictive pulmonary disease. Multiple
dentigerous cyst
32. CHILD syndrome Verruciform xanthoma, lymphedema, epidermal nevi, discoid
lupus erythematosus, congenital hemidysplasia, ichthyosiform
erythroderma and limb defect
33. Mazabraud syndrome Fibrous dysplasia with intramuscular myxomas
34. Gorham–Stout syndrome Massive osteolysis, phantom bone
35. Congenital acromicria syndrome, Down Trisomy 21
syndrome

https://t.me/DentalBooksWorld
Chapter 4 • Oral Pathology and Oral Medicine 295

36. Marie Sainton’s syndrome Cleidocranial dysplasia


37. Goldenhar syndrome (oculo auriculo Cleft palate, coloboma of eye, dermoid cyst in eye,
vertebral syndrome) underdeveloped kidneys, microtia or anotia
38. Plummer–Vinson syndrome (Paterson– Triad of microcytic hypochromic anemia (iron deficiency),
Brown–Kelly)sideropenic dysphagia atrophic glossitis, and esophageal webs or strictures
39. Bernard–Soulier syndrome Defective adhesion of platelets due to deficiency of platelet
membrane glycoprotein complex Ib-IX
40. Kostmann syndrome Severe congenital neutropenia
41. Chediak–Higashi syndrome Aplastic anemia, oculocutaneous albinism, pigmentation
42. Christ–Siemens–Touraine syndrome Hypohidrotic ectodermal dysplasia
43. Clouston syndrome Hidrotic ectodermal dysplasia
44. Zinsser–Cole–Englman syndrome/ Dyskeratosis congenital – Triad – nail dystrophy, reticular skin
Hoyeraal–Hreidarsson syndrome pigmentation, oral leukoplakia
45. Tenascin-X-deficiency syndrome Ehlers–Danlos syndrome
46. Mucocutaneous lymph node syndrome Kawasaki disease – systemic vasculitis
47. Goltz–Gorlin syndrome Focal dermal hypoplasia. FOCAL-females, osteopathia striata,
coloboma, absent ectodermal, mesodermal, neurodermal
derivatives, lobster claw deformity
48. Grinspan syndrome Lichen planus, diabetes mellitus, vascular hypertension
49. Weber–Cockayne syndrome Localized form of epidermolysis bullosa simplex
50. Steven–Johnson syndrome Severe bullous form of erythema multiforme
51. Trotter’s syndrome Nasopharyngeal tumor causing neuralgic pain
52. Raeder’s syndrome Paratrigeminal neuralgia
53. Horton syndrome Sphenopalatine neuralgia
54. Frey’s syndrome Auriculotemporal syndrome
55. Eagle’s syndrome Elongation of styloid process causing dysphagia, sore throat,
otalgia, glossodynia, orofacial pain
56. Carotid artery syndrome Ossified styloid process impinging on carotid arteries
57. Horner syndrome (sympathetic Miosis, ptosis, anhidrosis
ophthalmoplegia)
58. Floppy infant syndrome Infantile hypotonia
59. Marcus Gunn jaw-winking syndrome Trigemino-oculomotor synkinesis
60. Marin-Amat syndrome or inverted Marcus Closure of eyes when mouth opens forcefully
Gunn phenomenon
61. Mobius syndrome Congenital facial diplegia
62. Sjogren syndrome (a) Primary – Dry eyes + dry mouth
(b) Secondary – Dry eyes  +  dry mouth  + connective tissue
disorders like SLE, rheumatoid arthritis
63. TAFRO syndrome Lymphoid hamartoma, Thrombocytopenia, anasarca,
myelofibrosis, renal dysfunction, and organomegaly 
64. Fanconi syndrome Familial and congenital aplastic anemia
65. DIDMOAD syndrome Diabetes insipidus, diabetes mellitus, optic atrophy, and
deafness. Associated with sideroblastic anemia
66. APECED syndrome Autoimmune polyendocrinopathy-candidiasis-ectodermal
dystrophy syndrome
67. Congenital rubella syndrome Neonatal rubella

https://t.me/DentalBooksWorld
296 Triumph's Complete Review of Dentistry

68. Acute retroviral syndrome After initial HIV exposure (infectious mononucleosis-
lymphadenopathy, sore throat, fever, maculopapular rash,
headache, myalgia, arthralgia, diarrhea, photophobia,
peripheral neuropathies)
69. DILS syndrome (diffuse infiltrative HIV-associated salivary gland disease, CD8+ lymphocytosis,
lymphocytosis syndrome) interstitial pneumonia
70. Immune reconstitution syndrome Worsening of symptoms with antiretroviral therapy
administered during advanced stages
71. Laugier–Hunziker syndrome Hyperpigmented macules of the lips and buccal mucosa,
longitudinal melanonychia
72. Ramsay–Hunt syndrome Reactivation of Varicella zoster virus – Facial paralysis, hearing
deficits, vertigo
73. Granulomatous angiitis syndrome Fatal ischemic stroke during resolution of zoster rash
74. Lofgren syndrome Erythema nodosum, bilateral hilar lymphadenopathy,
arthralgia
75. Heerfordt syndrome Parotid enlargement, anterior uveitis, facial paralysis, fever
76. Leopard syndrome Lentigines, electrocardiographic conduction defects, ocular
hypertelorism, pulmonary stenosis abnormalities of the
genitals, retarded growth, deafness
77. Xeroderma pigmentosum, Rasmussen Basal cell carcinoma-associated syndromes
syndrome, Rombo syndrome, Bazex–
Christol–Dupre syndrome, Dowling meara
subtype of epidermolysis bullosa simplex,
NBCCSa
78. LADD syndrome Salivary gland aplasia or hypoplasia, cup-shaped ears, hearing
loss, dental, digital anomalies
79. Proteus syndrome overgrowth of the bones, skin, and other tissues
80. Sezary syndrome Aggressive expression of mycosis fungoides. Generalized
exfoliative erythroderma, lymphadenopathy, hepatomegaly,
splenomegaly
81. Hyperparathyroidism-jaw tumor syndrome Parathyroid adenoma, ossifying fibromas of jaw, renal cysts,
Wilm’s tumor
82. Numb chin syndrome Mental nerve paresthesia due to mandibular metastasis
83. Urbach–Wiethe syndrome Lipoid proteinosis
84. Gilbert syndrome Constitutional hepatic dysfunction and familial nonhemolytic
jaundice 
85. Di-George syndrome 22q11.2 deletion syndrome, rheumatoid arthritis, Graves’
disease, thrombocytopenia, hypoparathyroidism
86. Guillain–Barre syndrome Bilateral facial palsy in sarcoidosis
87. Treacher Collins Franceschetti syndrome Antimongoloid features, hypoplasia of facial bones, malformed
ear, macrostomia, high arched palate, blind fistulas, atypical
hair growth, facial clefts, salivary gland aplasia
88. Rett syndrome Sporadic mutation of MECP2 gene, onset 2 years, acquired
microcephaly, stopped development, motor and speech
regression, autism-like behavior, self-mutilating behavior,
inconsolable crying/screaming fits, emotional inversion,
hypotonia, dystonia, chorea, bruxism, scoliosis, long QT
89. Van der Woude syndrome Cleft lip and palate, congenital lip pits, delayed language
development, learning disabilities, or other mild cognitive
problem

https://t.me/DentalBooksWorld
Chapter 4 • Oral Pathology and Oral Medicine 297

90. Papillon–Lefevre syndrome Palmar plantar hyperkeratosis, periodontitis, nail dystrophy,


hyperhidrosis, premature exfoliation of deciduous and
permanent teeth
91. Hajdu–Cheney syndrome Joint laxity, short stature, scoliosis, kyphosis, multiple osteolytic
lesions, periodontitis, wide sella turcica, dolichocephaly
92. Syndromes associated with dilaceration Smith–Magenis syndrome, Ehlers–Danlos syndrome,
Axenfeld–Rieger syndrome, congenital ichthyosis

RADIOGRAPHIC PATHOLOGY
Analogy in Radiographic Appearances
S. No. Radiographic appearance Lesion
1. Teeth floating in space Histiocytosis-X, oral squamous cell carcinoma (if bones are involved)
2. Ballooning or blow out distension Aneurysmal bone cyst
of bone
3. spotty calcification (snowstorm) Synovial sarcoma
4. Cumulus cloud densities Extraosseous osteosarcoma
5. Codman triangle and Sunburst Osteosarcoma
pattern
6. Soap bubble appearance Ameloblastoma
7. Driven snow appearance Pindborg tumor
8. Honey comb appearance and Hemangioma
sunburst appearance
9. Orange peel, ground glass Fibrous dysplasia
appearance
10. ground glass appearance Hyperparathyroidism
11. Hard palate sign Cherubism
12. Floating tooth syndrome Cherubism
appearance
13. Cotton wool appearance Paget’s disease, cementoosseous dysplasia, Gardner’s syndrome,
gigantiform cementoma, chronic diffuse sclerosing osteomyelitis
14. Cloaking bone appearance Periosteal new bone formation in Infantile cortical hyperostosis
15. Beaten metal appearance Craniofacial dysostosis (Crouzon syndrome)
(increased digital markings)
16. Multiple radiolucencies of jaw Eosinophilic granuloma
and skull
17. Step ladder image/bone in a Sickle cell anemia
bone appearance/hair on end
appearance
18. Hair on end appearance/crew cut Thalassemia
appearance/salt and pepper effect
19. Tramline calcifications Sturge weber syndrome
20. Onion skin appearance Ewing’s sarcoma, histiocytosis-X, proliferative periostitis
21. Cob web trabeculation Odontogenic myxoma
22. Crescent-/half-moon-shaped Radicular dentin dysplasia (type I)
pulp chamber
23. Thistle tube pulp chamber Coronal dentin dysplasia (type II)

https://t.me/DentalBooksWorld
298 Triumph's Complete Review of Dentistry

24. Tennis racquet appearance Odontogenic myxoma


25. Cup-shaped or dish-shaped Primary Intraosseous carcinoma
pattern radiolucency/moth eaten
radiolucency
Radiolucencies
26. Unilocular pericoronal Hyperplastic dental follicle (<5 mm), dentigerous cyst (>5 mm), eruption
radiolucency cyst, unicystic ameloblastoma, ameloblastic fibroma, adenomatoid
odontogenic tumor, calcifying odontogenic tumor
27. Unilocular periapical Periapical granuloma, periapical cyst, periapical cementoosseous
radiolucency dysplasia, periapical scar, dentin dysplasia type 1
28. Unilocular radiolucencies in Developing tooth bud, lateral radicular cyst, nasopalatine duct cyst,
other sites lateral periodontal cyst, residual cyst, odontogenic keratocyst, central
giant cell granuloma, stafne bone cyst, cementoosseous dysplasia, central
ossifying fibroma, unicystic ameloblastoma, buccal bifurcation cyst,
histiocytosis-X, median palatal cyst
29. Multilocular radiolucencies Ameloblastoma, central giant cell granuloma, ameloblastic fibroma,
odontogenic fibroma, calcifying epithelial odontogenic cyst,
orthokeratinized odontogenic cyst, lateral periodontal cyst, calcifying
odontogenic cyst, central hemangioma, aneurysmal bone cyst,
cherubism, hyperparathyroidism, fibrous dysplasia, intraosseous
mucoepidermoid carcinoma
30. Poorly defined radiolucencies Periapical granuloma, focal osteoporotic defect, osteomyelitis, simple
bone cyst, metastatic tumors, osteoradionecrosis, multiple myeloma,
primary intraosseous carcinoma, osteosarcoma, chondrosarcoma,
Ewing’s sarcoma, fibrosarcoma, lymphoma, massive osteolysis
31. Multifocal radiolucencies Cementoosseous dysplasia, nevoid basal cell carcinoma syndrome,
multiple myeloma, cherubism, hyperparathyroidism, histiocytosis -X
32. Radiolucency below the inferior Stafne’s or static bone cyst
alveolar canal
33. Radiolucency above the inferior Traumatic bone cyst
alveolar canal
Radiopacities
34. Well-demarcated radioopacities Torus, idiopathic osteosclerosis, pseudocyst of maxillary sinus, condensing
osteitis, odontoma, cementoosseous dysplasia, osteoma, enamel pearl,
osteoblastoma, cementoblastoma
35. Poorly demarcated radioopacities cementoosseous dysplasia, osteoradionecrosis, condensing osteitis,
sclerosing osteomyelitis, fibrous dysplasia, Paget’s disease of bone,
osteosarcoma, chondrosarcoma, proliferative periostitis
36. Multifocal radioopacities Florid cementoosseous dysplasia, osteonecrosis of jaws, idiopathic
osteosclerosis, Paget’s disease of bone, Gardner syndrome, polyostotic
fibrous dysplasia, osteopetrosis
37. Well-demarcated mixed Cementoosseous dysplasia, odontoma, central ossifying fibroma,
radiolucent/radiopaque lesions ameloblastic fibroodontoma, adenomatoid odontogenic tumor, CEOT,
osteoblastoma, osteoid osteoma, cementoblastoma
38. Poorly demarcated mixed osteonecrosis of jaws, osteomyelitis, metastatic carcinoma, osteosarcoma,
radiolucent/radiopaque lesions chondrosarcoma
39. Multifocal mixed radiolucent/ Florid cementoosseous dysplasia, osteonecrosis of jaws, Paget’s disease
radiopaque lesions

https://t.me/DentalBooksWorld
Chapter 4 • Oral Pathology and Oral Medicine 299

Soft Tissue Radiopacities


40. Soft tissue radiopacities Amalgam tattoo, sialolith, tonsillitis, phlebolith, calcified lymph nodes,
calcinosis cutis, myositis ossificans, osteo and chondro choristoma
Widening of Periodontal Ligament Space
41. Heavy occlusal trauma, periodontitis, osteomyelitis, osteosarcoma, chondrosarcoma, non-Hodgkin’s
lymphoma, systemic sclerosis, osteonecrosis

DIAGNOSTIC TESTS
Disease Name of the test Method and inference
Psoriasis Grattage test Eliciting Auspitz sign
Hypoparathyroidism Ellsworth–Howard test Parathyroid hormone stimulation test.
To differentiate hypoparathyroidism and
pseudohypoparathyroidism
Sjogren syndrome Rose Bengal test or ocular dye test To test ocular epithelial damage. Also done in
brucellosis
Schirmer test Paper strip inside the lower eyelid of each eye to
check tear production
Phenol red thread test Phenol red-stained strips to assess tear production
Anti SSA (Ro) and anti SSB (La), Antibodies
ANA, rheumatoid factor
Salivary sialography Diffuse sialectasia
Salivary scintigraphy Delayed uptake and reduced concentration
Brown–Hopp’s method of gram Bartonella henselae
staining
Pathergy test Behcet’s disease Exaggerated skin injury occurring after minor
trauma such as bump, bruise, needle stick injury
Kveim’s test Sarcoidosis Intradermal injection of sterilized suspension of
human sarcoid tissue
Vanillylmandelic acid test Neuroblastoma, neuroendocrine Measures the amount of VMA that is passed into
tumors the urine, typically over a 24-hour period, to
detect excess epinephrine and norepinephrine
Specialized marker of bone Paget’s disease N-terminal propeptide of type 1 collagen
formation
Specialized marker of bone Paget’s disease N-terminal telopeptide of type 1 collagen
resorption
Minor starch iodine test Auriculotemporal syndrome Helps to detect sweating
Plasma cell dyscrasia diagnostic Protein and immunofixation Detects abnormal immunoglobulin protein
tests electrophoresis
Urine free light chains (Bence Jones Tested in urine voided over a 24-hour period. For
protein) diagnosis and monitoring treatment
Serum-free light chains Done for treatment monitoring
Bone marrow aspiration and biopsy Confirmatory test for multiple myeloma
Paul Bunnell test Infectious mononucleosis Sheep RBCs agglutinate in the presence of
heterophile antibodies in Ebstein–Barr Virus
infection

https://t.me/DentalBooksWorld
300 Triumph's Complete Review of Dentistry

Mono spot test Infectious mononucleosis Agglutination of horse RBCs on exposure to


heterophile antibodies in Ebstein–Barr Virus
infection
Schick test Diphtheria Detects disease susceptibility by an intradermal
injection of diphtheria toxin 
Wasserman test Syphilis A complement-fixation test for the diagnosis
of syphilis by determining the presence of
syphilitic antibodies in the blood serum

MODE OF INHERITANCE OF GENETIC DISORDERS


Autosomal dominant Autosomal recessive X-linked dominant Sex-linked recessive
• Osteogenesis • Thalassemia • Incontinentia pigmenti • Hemophilia
imperfecta
• Amelogenesis • Agnathia • Orofacial digital • Duchenne muscular dystrophy
imperfecta syndrome
• Dentinogenesis • Sickle cell anemia • Hypophosphatemic • Color blindness
imperfecta rickets
• Von Willebrand • Cystic fibrosis • Fragile X syndrome • Fabry disease
disease
• White sponge nevus • Phenylketonuria • Rett syndrome • X-linked agammaglobulinemia
• Van der Woude • Lysosomal storage • Focal dermal hypoplasia • Lesch–Nyhan syndrome
syndrome disease
• White sponge nevus • Ehler–Danlos • Craniofrontonasal • Wiskott–Aldrich syndrome
syndrome dysplasia
• Ehler–Danlos • Hemochromatosis • Alport syndrome • Menkes disease (kinky hair
syndrome syndrome)
• Cleidocranial • Wilsons disease
dysplasia
• Marfan syndrome • Fanconi anemia
• Achondroplasia • Xeroderma
• Familial polyposis pigmentosum
coli
• Cherubism
• Neurofibromatosis
• Tuberous sclerosis
• Van der Woude
syndrome
• Elephantiasis
gingivae
• Peutz–Jeghers
syndrome

GENE MUTATIONS
Disease Gene Gene loci
Peutz–Jeghers syndrome STK11 (LKB1) Chromosome 19
Dentinogenesis imperfecta DSPP Chromosome 4

https://t.me/DentalBooksWorld
Chapter 4 • Oral Pathology and Oral Medicine 301

Amelogenesis imperfecta • Amelogenin (AMELX)-(smooth hypoplastic Chromosome Xp22 (AMELX)


X-linked, hypomaturative)
• Enamelin (ENAM)-(local hypoplastic, smooth Chromosome 4q13.3 (ENAM)
hypoplastic AD)
• MMP 20 Pigmented hypomaturative (AR)
• Kallikrein-4 – Hypomaturative types
• FAM83H – Hypocalcified (AD)
• WDR72 – Hypomaturative (AR)
• C4orf26 – Hypomineralized (AR)
• DLX3 – Hypoplastic-hypomaturative with
taurodontism
Seborrheic keratosis FGFR3 (fibroblast growth factor receptor 3), Chromosome 4p16.3/chromosome
PIK3CA (phosphatidyl inositol 3-kinase catalytic 3q26.32
subunit alpha)
Actinic lentigo FGFR3 (fibroblast growth factor receptor 3), Chromosome 4p16.3/chromosome
PIK3CA (phosphatidyl inositol 3-kinase catalytic 3q26.32
subunit alpha)
Ephelis MC1R (melanocortin 1 receptor) Chromosome 16q24.3
Acquired melanocytic BRAF (B-RAF – Proto oncogene, serine/ Chromosome 7q34
nevus threonine kinase)
Spitz nevus HRAS (HRas proto-oncogene) Chromosome 11p15.5
Blue nevus (Jadassohn– GNAQ (G-protein alpha subunit Q) Chromosome 9q21
Tieche nevus)
Sturge Weber syndrome GNAQ (G-protein alpha subunit Q) Chromosome 9q21
Leukoplakia Loss of heterozygosity (3p, 9p, 4q, 8p, 11q, 13q, 17p),
Chromosome 17p13.1 (P53), 7p11.2
increased expression of Telomerase, p53, EGFR, (EGFR), 11q22.2 (MMP), 4q34.3
MMP, VEGF (VEGF)
Squamous cell carcinoma Ras, myc, EGFR, p53, pRb, p16, E-cadherin Chromosome 3q23 (ras), 8q24.21
(myc), 7p11.2 (EGFR), 17p13.1 (P53),
13q14.2 (pRB)
9p21.3 (p16)
16q22.1 (E-cadherin)
Basal cell carcinoma MC1R (melanocortin 1 Receptor), PTCH (patched), Chromosome 16q24.39 (MC1R),
SMO (smoothened frizzled class receptor), p53 9q22.32 (PTCH), 7q32.1 (SMO),
17p13.1 (P53),
Malignant melanoma CDKN2A (cyclin-dependent kinase inhibitor), Chromosome 9p21.3 (CDKN2A),
CDK4, MC1R (melanocortin 1 Receptor), BRAF 12q14.1 (CDK4), 16q24.3 (MC1R),
(B-RAF-proto oncogene, serine/threonine kinase) 7q34 (BRAF)
LADD syndrome FGF10 (fibroblast growth factor) Chromosome 5p12
Pleomorphic adenoma PLAG1 (pleomorphic adenoma gene 1) Chromosome 8q12
Warthin’s tumor CRTC1-MAML2 Chromosome t(11;19) (q21;p13)
Mucoepidermoid Translocation of (11;19)-CRTC1-MAML2 Chromosome 11;19
carcinoma
Mammary analog secretory t(12;15) (p13;q25)-ETV6-NTRK3 Chromosome (12;15) (p13;q25)
carcinoma
Schwannomatosis SMARCB1 Chromosome 22q11.23
Neurofibromatosis NF1 Chromosome 17q11.2
Proteus syndrome AKT1 Chromosome 14q32.33
Synovial sarcoma t(X;18) (P11.2;Q11.2)- SS18-SSX Chromosome (X;18) (P11.2;Q11.2)
Alveolar soft part sarcoma Der(17) t(X;17) (P11.2;2q5)-ASPL;TFE3A Chromosome (17) t(X;17) (P11.2;q25)

https://t.me/DentalBooksWorld
302 Triumph's Complete Review of Dentistry

Cyclic neutropenia ELANE/ELA-2 Chromosome 19p13.3


Polycythemia vera JAK2 (Janus kinase 2) Chromosome 9p24.1
Leukemia (CML) t(22;9)-BCR-ABL Philadelphia Chromosome (22;9)
Burkitt lymphoma t(8;14) (q24;q32)-c-myc Chromosome (8;14) (q24;q32)
Osteogenesis imperfecta COL1A1 and COL1A2, IFITM5, SERPINF1, Chromosome 17q21.33 (COL1A1),
CRTAP, LEPRE1, PPIB, BMP1, FKBP10 7q21.3 (COL1A1), 11p15.5
(IFITM5), 17p13.3 (SERPINF1), 3p22.3
(CRTAP), 1p31.3
(LEPRE1), 15q22.31 (PPIB), 8p21.3
(BMP1), 17q21.2 (FKBP10)
Osteopetrosis TCIRG1, RANKL, CAII, CLCN7, OSTM1, Chromosome 11q13.2 (TCIRG1),
PLEKHM1, LRP5, CLCN7 8q24.12 (RANKL), 8q21.2 (CAII), 6q21
(OSTM1), 17q21.31
(PLEKHM1), 11q13.2
(LRP5), 16p13.3 (CLCN7)
Cleidocranial dysplasia RUNX2 (CBFA1) Chromosome 6p21
Paget’s disease SQSTM1 (sequestome 1 gene) Chromosome 5q35.3
Cherubism SH3BP2 gene Chromosome 4p16.3
Marfan syndrome Fibrillin gene (FBN1) FBN1-15q21.1
Transforming growth factor beta receptor (TGFBR2) TGFBR2-3p24.1
Crouzon syndrome FGFR-3-fibroblast growth receptor 3 Chromosome 4p16.3
Apert syndrome FGFR-2 -fibroblast growth receptor 2 Chromosome 10q26.13
Achondroplasia FGFR-3-fibroblast growth receptor 3 Chromosome 4p16.3
Aneurysmal bone cyst USP6 (TRE17) (ubiquitin specific protease 6) Chromosome 15q21.2
Fibrous dysplasia GNAS (guanine nucleotide-binding protein-Alpha Chromosome 20q13.32
stimulating)
Familian gigantiform GDD1 (TMEM16E) Chromosome 11p14.3
cementoma
Ossifying fibroma HRPT2 (Parafibromin protein) Chromosome 1q31.2
Juvenile ossifying fibroma Xq26, 2q33 (X;2) Chromosome Xq26, 2q33
Gardner syndrome APC (adenomatous polyposis coli) Chromosome 5q21
Chondroma IDH1 (isocitrate dehydrogenase 1) Chromosome 2q34
Desmoplastic fibroma CTNNB1 Chromosome 3p22.1
Osteosarcoma P53, RB1 Chromosome 17p13.1 (P53), 13q14.2
(RB1)
Chondrosarcoma IDH1 (isocitrate dehydrogenase 1) and IDH2 Chromosome 2q34
Ewing’s sarcoma t(11:22) (q24;q12)-EWS-FL11 Chromosome t(11:22) (q24;q12)
Keratocystic odontogenic PTCH1 (patched), p16, p53, MCC, TSLC, LATS2, Chromosome 9q22.32 (PTCH), 9p21.3
tumor FHIT (p16), 17p13.1 (P53), 3q27.1 (MCC)
Ectodermal dysplasia Xq12-q13.1 Xq12-q13.1
White sponge nevus KRT4 and 13 Chromosome 12q13.139KRT4),
17q21.2 (KRT13)
Pachyonychia congenita KRT6A (keratin 6A) Chromosome 12q13.13
Dyskeratosis congenita DKC1 Chromosome Xq28
Incontinentia pigmenti XQ28-related to NEMO Chromosome XQ28
Darier’s disease ATP2A2 encodes for SERCA2 Chromosome 12q24.11

https://t.me/DentalBooksWorld
Chapter 4 • Oral Pathology and Oral Medicine 303

Hereditary hemorrhagic Type 1-ENGgene Chromosome 9q34.11 (ENG), 12q13.13


telangiectasia Type 2-ALK1, ACVRL1 (ALK1)
Tuberous sclerosis TSC1, TSC-2 Chromosome TSC1(9), TSC-2(16)
Multiple hamartoma PTEN Chromosome 10q23.31
syndrome
Epidermolysis bullosa KRT5 and 14, laminin 332, type XVII collagen, α6β4 Chromosome 12q13.13 (KRT
Integrin, LAMB3 gene 5), 17q21.2 (KRT14), 18q11.2
(laminin332), 10q25.1
(XVII), 17q25.1 (α6β4 Integrin), 1q32.2
(LAMB3)
Niemann–Pick disease NPC-1 and 2 Chromosome 18q11.2
Lipoid proteinosis ECM1 (extracellular matrix protein 1) Chromosome 1q21.2, 
Treacher Collins syndrome TCOF1 (Treacher Collins Franceschetti 1 gene) Chromosome 5q32-q33.1
Van der Woude syndrome IRF6 1q32

CAUSATIVE ORGANISMS OF ORAL INFECTIVE DISEASES


S. No. Disease Causative organism
Bacterial Diseases
1. Impetigo Staphylococcus aureus, Streptococcus pyogenes
2. Erysipelas Streptococcus pyogenes
3. Tuberculosis Mycobacterium tuberculosis
4. Leprosy Mycobacterium leprae
5. Actinomycosis Actinomyces israelii
6. Tetanus Clostridium tetani
7. Syphilis Treponema pallidum
8. Gonorrhea Neisseria gonorrhoeae
9. Granuloma inguinale Calymmatobacterium granulomatis
10. Scarlet fever Streptococcus pyogenes
11. Diphtheria Corynebacterium diphtheriae
12. Botryomycosis Staphylococcus, Streptococcus, Escherichia, Pseudomonas
13. Tularemia Francisella tularensis
14. Rhinoscleroma Klebsiella rhinoscleromatis
15. Cat scratch disease Bartonella henselae
16. Streptococcal tonsillitis, β-hemolytic streptococci, adenovirus, enterovirus, influenza, parainfluenza, and
pharyngitis EBV
17. Noma Fusobacterium necrophorum, Prevotella intermedia
18. Sinusitis Streptococcus pneumoniae, Haemophilus influenza, Moraxella catarrhalis
Viral Infections
19. Measles (Rubeola, Morbilli) Paramyxo virus (RNA),(morbillivirus)
20. Mumps (epidemic parotitis) Paramyxo virus (RNA)
21. Rubella (German measles) Rubella virus (RNA)
22. Poliomyelitis Poliovirus (RNA)
23. Herpangina Cox sackie virus A-1,6,8,10,16,2

https://t.me/DentalBooksWorld
304 Triumph's Complete Review of Dentistry

24. Acute lymphonodular Cox sackie A10


pharyngitis
25. Hand foot mouth disease Cox sackie A16, A5, 6, B2, B5 OR Enterovirus 71
26. Foot and mouth disease Cox sackie virus
(aphthous fever, hoof and
mouth disease)
27. Chikungunya CHIKV-Chikungunya virus
28. Herpes simplex (acute Herpes simplex virus-1
herpetic gingivostomatitis,
herpes labialis)
29. Herpes genitalis Herpes simplex virus-2
30. Primary herpetic stomatitis Herpes simplex virus-1
31. Secondary herpetic Herpes simplex virus-1 or 2
stomatitis
32. Small pox Variola virus
33. Chicken pox Varicella zoster virus (DNA)
34. Herpes zoster Varicella zoster virus (DNA)
35. Cytomegalovirus inclusion Cytomegalovirus
disease
36. Infectious mononucleosis Ebstein–Barr virus
37. Molluscum contagiosum Molluscum contagiosum virus (Pox virus)
38. Condyloma acuminatum Human papilloma virus (Type 6, a11, a30, b42, 43, 46b, 51, b54, 55, 70)
39. Acquired immunodeficiency Human immunodeficiency virus (HIV)
syndrome
Fungal Infections
40. Candidiasis Candida albicans (other sp. C. tropicalis, C. parapsilosis, C. stellatoidea, C. krusei,
C. guillermondi, C. dubiliensis, C. rugosa, C. viswanathii, C. glabrata)
41. Mucormycosis Mucormycetes
42. Rhinosporidiosis Rhinosporidium seeberi
43. Aspergillosis Aspergillus fumigatus
44. Histoplasmosis Histoplasma capsulatum
45. North American Blastomyces dermatitidis
blastomycosis
46. South American Paracoccidioides brasiliensis (brasiliensis Blastomyces brasiliensis)
blastomycosis
47. Coccidioidomycosis Coccidioides immitis
48. Cryptococcosis Cryptococcus neoformans
49. Geotrichosis Geotrichum candidum
50. Sporotrichosis Sporotrichum schenckii
51. Toxoplasmosis Toxoplasma gondii
Parasitic Infections
52. Cysticercosis Cysticercosis cellulosae
53. Trichinosis Trichinella spiralis
54. Hydatid cyst Echinococcus granulosus, E. multilocularis, E. vogeli
55. Myiasis Myia

https://t.me/DentalBooksWorld
Chapter 4 • Oral Pathology and Oral Medicine 305

MOST COMMON’S IN ORAL PATHOLOGY


Common feature Disease
Common malignant tumor of bone in children Ewing’s sarcoma
Common malignancy of oral cavity Oral squamous cell carcinoma
Common malignancy of skin Basal cell carcinoma
Common intraoral nevus Intramucosal nevus
Common benign tumor arising from epithelium Papilloma
Common benign tumor arising from connective tissue Fibroma
Common salivary gland malignancy in children Mucoepidermoid carcinoma
Common premalignant lesion of the oral mucosa Leukoplakia
Common congenital craniofacial anomaly Cleft lip and palate
Common congenital anomaly to be associated with Cleft palate
syndromes
Common congenital deformation of the craniofacial Nasal, auricular, and mandibular deformities
region
Common genetic disorder Thalassemia
Common malocclusion Class 1
Common contagious oral papillary lesion Focal gingival hyperplasia
Common congenital cause of macroglossia Down’s syndrome and Beckwith–Wiedemann syndrome
Common developmental odontogenic cyst Dentigerous cyst
Common inflammatory odontogenic cyst/most Radicular cyst (apical periodontal cyst)
common odontogenic cyst
Common soft tissue sarcoma of late adult life Malignant fibrous histiocytoma
Common soft tissue sarcoma below 15 years of age Rhabdomyosarcoma
Common benign salivary gland tumor Pleomorphic adenoma
Second common benign salivary gland tumor Warthin’s tumor
Common malignant salivary gland tumor Mucoepidermoid carcinoma
Common malignant salivary gland tumor in children Mucoepidermoid carcinoma
Second common malignant salivary gland tumor Adenocarcinoma
Common autoimmune disease Rheumatoid arthritis
Second common autoimmune disease Mikulicz syndrome
Common intraoral opportunistic infection in HIV Candidiasis
Common bleeding disorder Von Willebrand disease (pseudohemophilia)
Common genodermatoses Ichthyosis>epidermolysis bullosa>ectodermal dysplasia
Common cranial neuralgia Trigeminal neuralgia

AGE PREDILECTION
Developmental Disturbances
Focal epithelial hyperplasia Children
Odontogenic Cysts and Tumors
Odontogenic keratocyst 2nd and 3rd decades
Orthokeratinized odontogenic cyst 3rd and 4th decades
Dentigerous cyst 2nd and 3rd decades

https://t.me/DentalBooksWorld
306 Triumph's Complete Review of Dentistry

Lateral periodontal cyst Adults


Gingival cyst of adults Above 40 years of age
Radicular cyst (apical periodontal cyst) 20–60 years
Nasopalatine cyst 4th–6th decade
Mucous extravasation cyst First three decades
Traumatic bone cyst 2nd decade
Aneurysmal bone cyst 1st and 2nd decades
Stafne bone cyst Adults
Ameloblastoma Mean age – 3rd decade
Squamous odontogenic tumor 2nd–6th decade
Calcifying epithelial odontogenic tumor Mean age - 40 years
Adenomatoid odontogenic tumor Below 20 years
Ameloblastic fibroma Younger age group Odontogenic myxoma
Odontogenic myxoma Mean age 23–30 years
Malignant ameloblastoma Mean age 20–30 years
Epithelial Pathology
Leukoplakia 30–50 years
Carcinoma in situ Elderly age group
Oral submucous fibrosis 20–40 years
Basal cell carcinoma 4th decade
Squamous cell carcinoma 4th–6th decade
Verrucous carcinoma 6th and 7th decades
Malignant melanoma 4th–7th decade
Connective Tissue Tumors
Fibroma 3rd–5th decade
Giant cell Fibroma 10–30 years
Peripheral ossifying fibroma Children and young adults
Central ossifying fibroma Young adults
Peripheral giant cell granuloma 4th–6th decade
Central giant cell granuloma Young age – Below 30 years
Giant cell tumor of bone 3rd decade
Lipoma 3rd decade
Verruciform xanthoma 4th and 5th decades
Hemangioma 2nd decade
Lymphangioma 1st decade
Chondroblastoma 1st–3rd decade
Osteoid osteoma 1st–3rd decade
Osteoblastoma 1st and 2nd decades
Torus 1st–3rd decade
Fibrosarcoma 4th decade
Malignant fibrous histiocytoma 5th–7th decade
Liposarcoma 4th and 6th decades
Ewing’s sarcoma 1st–2nd decade
Chondrosarcoma 4th–6th decade

https://t.me/DentalBooksWorld
Chapter 4 • Oral Pathology and Oral Medicine 307

Osteosarcoma 1st and 2nd decades


Non-Hodgkin lymphoma 5th decade
Hodgkin’s lymphoma Bimodal (2–3rd decade) and (>55 years)
Leiomyoma 6th decade
Rhabdomyoma Middle age
Granular cell myoblastoma 3rd–6th decade
Congenital epulis At birth
Leiomyosarcoma 6th decade
Rhabdomyosarcoma 1st and 2nd decades
Neurofibroma 2nd and 3rd decades
Melanotic neuroectodemal tumor of 1–6 months
infancy
Malignant Schwannoma (MPNST) 2nd–5th decade
Salivary Gland Pathology
Pleomorphic adenoma 4th–6th decade
Warthin’s tumor 5th–6th decade
Mucoepidermoid carcinoma 3rd and 5th decades
Adenoid cystic carcinoma 5th and 6th decades
Necrotizing sialometaplasia Mean age – 47.9 years
Mikulicz syndrome 4th and 5th decades
Sjogren’s syndrome Above 4th decade
Bacterial Infections
Tuberculosis Primary – Children
Secondary – Adults
Pulp and Periapical Diseases
Chronic focal sclerosing osteomyelitis Children and young adults
Bone and Joint Diseases
Osteogenesis imperfecta Infants
Fibrous dysplasia 3–15 years
Cherubism 14 months–3 years
Paget’s disease Above 50 years of age
Blood Disorders
Acute lymphocytic leukemia <20 years
Acute myeloid leukemia Above 65 years
Chronic lymphocytic Above 65 years
Chronic myeloid leukemia Above 65 years
Skin Disorders
Lichen planus Above 40 years
Pemphigus vulgaris 5th–6th decade
Cicatricial pemphigoid 4th and 5th decades
Bullous pemphigoid Above 60 years
Epidermolysis bullosa At birth
Erythema multiforme 2nd–4th decade

https://t.me/DentalBooksWorld
308 Triumph's Complete Review of Dentistry

Systemic lupus erythematosus 3rd–4th decade


Systemic sclerosis 3rd–5th decade
Psoriasis 2nd and 3rd decades
Acanthosis nigricans Adults
Pityriasis rosea Children and young adults
Dermatitis herpetiformis 20–55 years
Acrodermatitis enteropathica 1st few weeks or months of life
Nerve and Muscle Diseases
Trigeminal neuralgia Above 50 years
Sphenopalatine neuralgia Before 4th decade
Burning mouth syndrome Above 50 years
Bell’s palsy 2nd–6th decade
Glossopharyngeal neuralgia Above 3rd decade
Multiple sclerosis 2nd–4th decade
Generalized familial muscular dystrophy 1st–2nd decade
Myasthenia gravis 3rd–5th decade

GENDER PREDILECTION
Developmental Disturbances
Cleft lip and palate Males
Isolated cleft palate Females
Facial hemihypertrophy Females
Facial hemiatrophy Females
Oral nevi Females
Labial oral melanotic macule Females
Fissured tongue (scrotal tongue) Males
Median rhomboid glossitis Males
Hairy tongue Males
Lingual thyroid nodule Females
Angiolymphoid hyperplasia with eosinophilia (ALHE) Females (males in Asia region)
Supernumerary teeth Males
Odontogenic Cysts and Tumors
Odontogenic keratocyst Males
Orthokeratinized odontogenic cyst Males
Dentigerous cyst Males
Lateral periodontal cyst Males
Glandular odontogenic cyst Males
Nasopalatine cyst Males
Traumatic bone cyst Males
Stafne bone cyst Males
Squamous odontogenic tumor Males
Adenomatoid odontogenic tumor Females
Odontoma Males

https://t.me/DentalBooksWorld
Chapter 4 • Oral Pathology and Oral Medicine 309

Epithelial Pathology
Keratoacanthoma Males
Leukoplakia Males
Proliferative verrucous leukoplakia Females
Carcinoma in situ Males
Oral submucous fibrosis Males
Basal cell carcinoma Males
Squamous cell carcinoma Males
Verrucous carcinoma Males
Malignant melanoma Males
Connective Tissue Tumors
Fibroma Females
Peripheral ossifying fibroma Females
Peripheral giant cell granuloma Females
Giant cell tumor of bone Females
Hemangioma Females
Pyogenic granuloma Females
Chondroblastoma Males
Osteoid osteoma Males
Osteoblastoma Males
Torus Females
Fibrosarcoma Males
Malignant fibrous histiocytoma Males
Liposarcoma Males
Ewing’s sarcoma Males
Chondrosarcoma Males
Osteosarcoma Males
Non-Hodgkin lymphoma Males
Hodgkin’s lymphoma Males
Rhabdomyoma Males
Granular cell myoblastoma Females
Congenital epulis Females
Leiomyosarcoma Males
Rhabdomyosarcoma Females
Traumatic neuroma Females
Schwannoma Females
Malignant Schwannoma (MPNST) Males
Salivary Gland Pathology
Pleomorphic adenoma Females
Warthin’s tumor Males
Mucoepidermoid carcinoma Females
Necrotizing sialometaplasia Males
Mikulicz syndrome Females
Sjogren’s syndrome Females

https://t.me/DentalBooksWorld
310 Triumph's Complete Review of Dentistry

Bone and Joint Diseases


Cherubism Males
Blood Disorders
Polycythemia Males
Hemophilia A Males
Von Willebrand disease Females (more symptomatic due to bleeding episodes)
Skin Disorders
Hypohidrotic Ectodermal dysplasia Males
Dyskeratosis congenita Males
Lichen planus Females
Cicatricial pemphigoid Females
Erythema multiforme Males
Systemic lupus erythematosus Females
Systemic sclerosis Females
Psoriasis Females
Pityriasis rosea Females
Dermatitis herpetiformis Males
Nerve and Muscle Diseases
Trigeminal neuralgia Females
Sphenopalatine neuralgia Males
Burning mouth syndrome Females
Bell’s palsy Females
Multiple sclerosis Females
Generalized familial muscular dystrophy Males
Myasthenia gravis Females

SITE PREDILECTION
Developmental Disturbances
Agnathia Mandible
Congenital lip pits Lower lip
Double lip Upper lip
Cheilitis glandularis Lower lip
Oral nevi Hard palate
Labial oral melanotic macule Lower lip – Vermilion border
Mucous membrane pigmentation in Peutz–Jeghers Buccal mucosa
syndrome
Fordyce spots Buccal mucosa opposite molar teeth
Focal epithelial hyperplasia Buccal mucosa
Odontogenic Cysts and Tumors
Odontogenic keratocyst Mandible third molar region
Orthokeratinized odontogenic cyst Mandible third molar region
Dentigerous cyst Mandible third molar region and maxillary cuspid region

https://t.me/DentalBooksWorld
Chapter 4 • Oral Pathology and Oral Medicine 311

Lateral periodontal cyst Mandibular bicuspid/cuspid/incisor region


Glandular odontogenic cyst Anterior mandible
Bohn’s nodules Gingival cysts scattered over the palate
Epstein’s pearls Gingival cysts on the median palatal raphe
Gingival cyst of adult Mandibular bicuspid and cuspid region
Radicular cyst (apical periodontal cyst) Maxillary anteriors>mandibular premolars and molars
Stafne bone cyst Below the inferior alveolar canal in mandible
Ameloblastoma Mandibular molar ramus region
Squamous odontogenic tumor Mandible molar cuspid region
Calcifying epithelial odontogenic tumor Mandible molar cuspid region
Adenomatoid odontogenic tumor Maxilla
Ameloblastic fibroma Molar region of mandible
Odontoma Anterior maxilla – Compound odontoma
Posterior maxilla – Complex odontoma
Peripheral odontogenic fibroma Mandible
Odontogenic myxoma Mandible
Epithelial Pathology
Keratoacanthoma Sun – exposed areas – lips
Squamous papilloma Tongue
Leukoplakia Buccal mucosa
Proliferative verrucous leukoplakia Gingiva
Carcinoma in situ Floor of the mouth
Oral submucous fibrosis Buccal mucosa
Basal cell carcinoma Middle third of the face
Squamous cell carcinoma Buccal mucosa and retromolar area
Verrucous carcinoma Buccal mucosa
Malignant melanoma Palate and maxillary gingiva
Connective Tissue Tumors
Fibroma Buccal mucosa
Fibromatoses Submandibular region
Giant cell fibroma Mandibular gingiva
Central ossifying fibroma Mandible
Peripheral giant cell granuloma Mandible
Giant cell tumor of bone Sphenoid, ethmoid, and temporal bone
Lipoma Tongue
Verruciform xanthoma Masticatory mucosa
Oral hemangiomas Mandible
Pyogenic granuloma Gingiva (facial aspect)
Lymphangioma Tongue
Chondroma Anterior maxilla
Chondroblastoma Mandibular condyle
Osteoid osteoma Femur, mandible
Osteoblastoma Vertebra, equal predilection in jaw bones

https://t.me/DentalBooksWorld
312 Triumph's Complete Review of Dentistry

Torus Palate and mandible


Fibrosarcoma Femur and tibia
Liposarcoma Buccal mucosa
Ewing’s sarcoma Mandible
Osteosarcoma Femur, mandible
Hodgkin’s lymphoma Cervical lymph nodes
Multiple myeloma Bone marrow – Vertebra
Leiomyoma Tongue
Rhabdomyoma Oral floor
Granular cell myoblastoma Dorsum of tongue
Congenital epulis Maxillary gingiva
Leiomyosarcoma Maxilla
Rhabdomyosarcoma Soft palate and tongue
Traumatic neuroma Near mental foramen
Schwannoma Tongue
Malignant Schwannoma (MPNST) Tongue and soft palate
Salivary Gland Pathology
Pleomorphic adenoma Parotid gland
Mucoepidermoid carcinoma Parotid gland
Adenoid cystic carcinoma Parotid gland
Necrotizing sialometaplasia Minor salivary glands in palate
Bacterial Infections
Tuberculosis Lung/tongue (oral site )
Leprosy Skin and peripheral nervous system/tongue, gingiva
Actinomycosis Mandible
Tetanus Masseter
Syphilis Genitals and lips
Gonorrhea Genitals and lips
Granuloma inguinale Genitals and lips
Noma Gingiva
Scarlet fever Pharynx
Diphtheria Tonsils
Rhinoscleroma Nose/soft palate
Viral Infections
Measles Skin of face
Mumps Parotid
Herpangina Oropharynx
Herpes simplex HSV 1 – Face, lips, oral cavity
HSV 2 – Genitals, skin of lower body
Primary herpetic stomatitis Gingiva
Variola (Chicken pox) Trunk/buccal mucosa
Herpes Zoster Trunk, palate

https://t.me/DentalBooksWorld
Chapter 4 • Oral Pathology and Oral Medicine 313

Infectious mononucleosis Cervical lymph nodes, pharynx, tonsils


Molluscum contagiosum Extremities, lips
Fungal Infections
Candidiasis Buccal mucosa
Rhinosporidiosis Nasal mucosa
Aspergillosis Hard palate (paranasal aspergillosis)
Soft palate (pulmonary aspergillosis)
Histoplasmosis Spleen/buccal mucosa
South American blastomycosis Periodontal tissues
Pulp and Periapical Diseases
Acute osteomyelitis Maxilla (localized )
Mandible (diffuse)
Bone and Joint Diseases
Fibrous dysplasia Femur/mandible
Cherubism Maxilla and mandible
Paget’s disease Maxilla
Infantile cortical hyperostosis Angle of the mandible and ramus
Cleidocranial dysplasia Skull and shoulder girdle
Blood Disorders
Acute myeloid leukemia Gingiva (extramedullary manifestation)
Skin Disorders
Lichen planus Flexor aspects of wrist, forearms, inner aspect of knees, thighs and
trunk
Cicatricial pemphigoid Oral mucosa and conjunctiva
Bullous pemphigoid Hands and feet
Epidermolysis bullosa Hands and feet
Erythema multiforme Hands and feet
Systemic lupus erythematosus Face
Systemic sclerosis Face, hands, and trunk
Psoriasis Extensor surfaces of extremities
Acanthosis nigricans Axilla, palms/tongue and lips
Pityriasis rosea Trunk, thighs/buccal mucosa
Dermatitis herpetiformis Extremities, trunk
Acrodermatitis enteropathica Periorofacial region and fingers
Nerve and Muscle Diseases
Trigeminal neuralgia Right side of face
Sphenopalatine neuralgia Eyes, maxilla, ear, and mastoid
Burning mouth syndrome Tongue, lips, palate
Bell’s palsy Facial muscles
Glossopharyngeal neuralgia Ear, pharynx, nasopharynx, tonsil
Multiple sclerosis Central nervous symptoms
Generalized familial muscular dystrophy Muscles of the extremities
Myasthenia gravis Voluntary muscles

https://t.me/DentalBooksWorld
314 Triumph's Complete Review of Dentistry

TOOTH PREDILECTION
Developmental Disturbances
Facial hemihypertrophy Cuspid, premolars, first molar
Retrocuspid papilla Mandibular cuspid -lingual mucosa
Microdontia Maxillary lateral incisor and third molar
Congenitally missing Deciduous teeth Deciduous maxillary and mandibular lateral incisors
Congenitally missing permanent teeth Permanent maxillary third molars, lateral incisor, and mandibular second
premolar
Fusion Deciduous dentition
Talon’s cusp Maxillary and mandibular incisors
Dens in dente Maxillary lateral incisors
Dens evaginatus (Leong’s premolar) Premolar
Enamel pearl Roots of maxillary third molar and mandibular third molar
Taurodontism Molars
Dilaceration Maxillary premolars (Ref. 303, Ch-19, 6th edition, White and Pharoah)
Supernumerary roots Mandibular bicuspids and cuspids
Supernumerary teeth Mesiodens>distomolar>paramolar
Turners teeth Permanent maxillary central incisor>mandibular premolar>maxillary premolar
Ghost teeth Permanent maxillary central incisor> maxillary lateral incisor> maxillary cuspid
Neonatal teeth Deciduous mandibular central incisors
Submerged teeth (ankylozed teeth) Mandibular second molar
Impacted teeth (embedded teeth) Maxillary third molar>mandibular third molar>maxillary cuspid
Globodontia Gigantic globe-shaped teeth – features of otodental syndrome
Lobodontia Teeth resembling lobed teeth of carnivores

CLINICAL STAGING SYSTEMS


TNM staging Malignancy T – Size of tumor (1–4)
T0 – No evidence of primary tumor
T1 – <2 cm (minimally invasive carcinoma
T1a – <1 cm
T1b – 1–2 cm
T1c – 2–3 cm)
T2 – 3–5 cm (T2a – 3–4 cm, T2b – 4–5 cm)
T3 – 5–7 cm
T4 – >7 cm
N – Affected lymph nodes (0–3)
N1 – Single ipsilateral node (<3 cm)
N2a – Single ipsilateral node (>3 cm)
N2b – Multiple ipsilateral node (<6 cm)
N2c – Contralateral node (<6 cm)
N3 – >6 cm
M – Metastasis
M0 – No metastasis
M1 – Distant metastasis
MX – Cannot be assessed
Lugano staging (based on Old Ann Non-Hodgkin’s Stage 1 – lymph node or lymphoid organ
Arbor system) lymphoma Stage 2 – More groups of LN on same side
Stage 3 – More groups of LN on both sides
Stage 4 – Extra nodal spread to other organ systems

https://t.me/DentalBooksWorld
Chapter 4 • Oral Pathology and Oral Medicine 315

Kadish clinical staging Olfactory Stage A – Confined to nasal cavity


neuroblastoma Stage B – Paranasal sinus
Stage C – Beyond nasal and paranasal sinus
Stage D – Lymph node or distant metastasis
Histological Grading
Anneroth grading system Squamous cell 1. Degree of keratinization
carcinoma 2. Nuclear polymorphism
3. Number of mitosis per high power field
4. Pattern of invasion
5. Stage of invasion
6. Lymphoplasmacytic infiltration

TUMOR-ASSOCIATED ANTIGENS FOR DIAGNOSIS


S. No. Neoplasm Antigens
1. Carcinoma Keratins
2. Adenocarcinoma Keratins
3. Salivary gland tumors S-100, actins, calponin
4. Rhabdomyosarcoma Desmin, myoglobin, actin, myogenin, muscle-specific actin
5. Leiomyosarcoma Smooth muscle actin
6. Neurosarcoma Neurofilaments, S-100
7. Angiosarcoma and Kaposi’s sarcoma CD-31, CD-34, factor VIII-related antigen
8. Melanoma HMB-45, S-100, MART-1 (Melan-A)
9. Langerhans cell disease CD1a
10. Lymphomas CD45, CD45RB
11. T-cell lymphomas CD3, CD43, CD45RO
12. B-cell lymphomas CD-20, CD791, CD45RA
13. Anaplastic large cell CD30, ALK-1
14. Hodgkin’s disease (Reed–Sternberg cell) CD15, CD30
15. Plasma cell myeloma κ/λ light chain
16. Leukemic infiltrates TdT, myeloperoxidase
17. Paraganglioma and neuroendocrine carcinoma Synaptophysin, chromogranin, neurofilaments
18. Olfactory neuroblastoma Synaptophysin, chromogranin
19. Merkel cell tumor Synaptophysin, chromogranin
20. Ewing’s sarcoma, and PNETs CD99
21. Solitary fibrous tumor CD34, CD99, Bcl-2

MULTIPLE CHOICE QUESTIONS

DEVELOPMENTAL ANOMALIES
1. The least common type of cleft lip is
A. Unilateral incomplete B. Unilateral complete
C. Bilateral incomplete D. Bilateral complete
2. Most common type of inheritance seen in cleft lip/palate is
A. Monogenic B. Both monogenic and polygenic
C. Syndromic D. Multifactorial

https://t.me/DentalBooksWorld
316 Triumph's Complete Review of Dentistry

3. Which is the etiologically and embryologically different identity from other clefts?
A. Isolated cleft palate B. Cleft lip with or without cleft palate
C. Cleft palate with or without cleft lip D. Cleft lip
4. Defects involving the lip comes under which group in Veau’s classification?
A. Group I B. Group II
C. Group III D. Group IV
5. Cheilitisglandularis affects the
A. Upper lip B. Lower lip
C. Both lips but upper lip slightly more D. Both lips but lower lip slightly more
6. Macroglossia is seen in all the following except
A. Carcinoma B. Ascher’s syndrome
C. Down’s syndrome D. Beckwith–Wiedemann syndrome
7. A child was brought to the dentist with c/o grey brown discoloration of teeth. There is a history of multiple bone
fractures. The radiograph shows pulp obliteration with short roots. Which gene is affected?
A. COL1A1 B. AMEL – X
C. APC tumor suppressor D. DSSP
8. Turner’s teeth is
A. Permanent maxillary incisors B. Mandibular premolar
C. Permanent mandibular first molar D. Primary molars
9. In case of dentinogenesis imperfecta, the gene maps to chromosome number
A. 8 B. 4
C. 9 D. 1
10. Delayed eruption of teeth with little formation of dentin and large pulp chambers is a characteristic of
A. Dentinogenesis imperfecta B. Regional odontodysplasia
C. Dentin dysplasia D. Dentin hypoplasia
11. Cysts found along the median palatine raphae of infants due to entrapment of epithelium during fusion of palatine
processes are
A. Epstein’s pearls B. Bohn’s nodules
C. Bay cysts D. Cystic hygroma
12. Teeth showing dentin dysplasia characteristically have
A. Long crowns B. Long roots
C. Short crowns D. Short roots
13. Chronic alcoholism could sometimes lead to
A. Greenspan’s lesion B. Sialosis
C. Static bone cavity D. Strawberry tongue
14. The O.M.E.N.S. classification system is for
A. Cleft lip/palate B. Hemifacial microsomia
C. Skin pathology D. Ameloblastoma
15. O.M.E.N.S classification system – stands for all except
A. Eye B. Ear
C. Mandible D. Facial nerve
16. Malformation syndromes associated with hemihyperplasia are all except
A. Proteus syndrome B. Maffucci’s syndrome
C. Beckwith–Wiedemann syndrome D. Parry–Romberg syndrome
17. The most common nonodontogenic cyst of the oral cavity is
A. Nasolabial cyst B. Globulomaxillary cyst
C. Nasopalatine cyst D. Median mandibular cyst
18. Nonsyndromic multiple supernumerary teeth occur most frequently in the
A. Maxillary anterior region B. Mandibular premolar region
C. Region distal to maxillary third molar D. Mandibular molar region

https://t.me/DentalBooksWorld
Chapter 4 • Oral Pathology and Oral Medicine 317

19. Identify the anomaly


A. Gemination B. Fusion
C. Concrescence D. Taurodontism
20. Identify the anomaly
A. Talons cusp B. Supernumerary teeth
C. Dens in dente D. Fusion
21. Identify the anomaly
A. Fusion B. Concrescence
C. Taurodontism D. Normal tooth
22. Identify the pathology in the given OPG
A. Right-sided hemi hyperplasia B. Right-sided atrophy
C. Left-sided hemi hyperplasia D. Left-sided condylar hyperplasia
23. Dens invaginatus that extends through the root and perforates in the apical area without any immediate
communication with the pulp belongs to
A. Type I B. Type II
C. Type III D. Type IV
24. Lyonization effect in amelogenesis imperfecta is seen in
A. Autosomal dominant pattern B. Autosomal recessive pattern
C. X-linked pattern D. Y-linked pattern
25. Ptyalism is seen in
A. Sialometaplasia B. Agenesis
C. Cerebral palsy D. Mental retardation
26. Odontodysplasia is most common in
A. Mandibular premolar B. Mandibular canine
C. Mandibular third molar D. Maxillary central incisor
27. Most commonly submerged tooth is
A. Mandibular primary 1st molar B. Mandibular primary 2nd molar
C. Maxillary primary 1st molar D. Maxillary primary 2nd molar
28. The most common site in the oral cavity for a lymphangioma is the
A. Tongue B. Palate
C. Tonsils D. Floor of the mouth
29. All are associated with cleft lip/palate except
A. Treacher Collins syndrome B. Down’s syndrome
C. Marfan’s syndrome D. Gardner’s syndrome

ODONTOGENIC CYSTS AND TUMORS


1. The most potentially aggressive and destructive cyst is
A. Dentigerous cyst B. Incisive canal cyst
C. Globulomaxillary cyst D. Periapical cyst
2. Difference between the epithelial lining of the cyst and the oral cavity is
A. Stratum germinativum B. Stratum corneum
C. Stratum spinosum D. Stratum lucidum
3. Most aggressive form of ameloblastoma
A. Follicular B. Plexiform
C. Acanthomatous D. Granulomatous
4. Maximum amount of recurrence is seen in
A. Odontogenic keratocyst B. Dentigerous cyst
C. Calcifying odontogenic epithelial cyst D. Follicular cyst
5. Most common odontogenic tumor is
A. Odontoma B. Ameloblastoma
C. Pleomorphic adenoma D. Dentigerous cyst
https://t.me/DentalBooksWorld
318 Triumph's Complete Review of Dentistry

6. Identify the lesion

A. OKC B. Ameloblastoma
C. Dentigerous cyst D. Stafne cyst
7. Picket fence appearance is seen in
A. OKC B. Ameloblastoma
C. Dentigerous cyst D. Stafne cyst
8. All of the following are derived from dental lamina except
A. Odontogenic keratocyst B. Lateral periodontal cyst
C. Gingival cyst of adult D. Dentigerous cyst
9. All of the following are features of OKC except
A. Lined by stratified squamous epithelium
B. Rete ridges are present
C. Thickness of the epithelium, usually ranging from 6 to 10 cells thick
D. Lumen filled with thin straw colored fluid
10. A distinct type of developmental cyst characterized by a thin, nonkeratinized epithelium usually one to five cell layers
thick, which resembles the reduced enamel epithelium is seen in
A. Dentigerous cyst B. Lateral periodontal cyst
C. Both of the above D. None of the above
11. Most common type of ameloblastoma is
A. Follicular B. Plexiform
C. Acanthomatous D. Basal cell
12. Ameloblastoma according to Robinson– false is
A. Multicentric B. Nonfunctional
C. Benign lesion D. Intermittent in growth
13. The highest rate of recurrence is for
A. Follicular B. Plexiform
C. Acanthomatous D. Granulomatous
14. All are true for plexiform ameloblastoma except
A. The ameloblast-like tumor cells are arranged in irregular masses, or more frequently, as a network of interconnecting
strands of cells
B. Stellate reticulum like tissue is more prominent
C. Areas of cystic degeneration of stroma are common
D. Peripheral palisading arrangement
15. Pindborg tumor arises form
A. Cells in the stratum intermedium layer of the enamel organ
B. Cells in the stellate reticulum layer of the enamel organ
C. Cell rests of Malassez
D. Cell rests of Serres

https://t.me/DentalBooksWorld
Chapter 4 • Oral Pathology and Oral Medicine 319

16. Peripheral AOT occurs in


A. Maxillary gingiva, females B. Maxillary gingiva, males
C. Mandibular gingiva, females D. Mandibulargingiva, males
17. Histopathology of AOT – all are true except
A. The tumor is made up of a multinodular proliferation of spindle, cuboidal, and columnarcells in a variety of patterns
comprising of scattered duct-like structures
B. Contain dentinoid-like structures and osteodentin
C. Cystic component is present
D. The supporting stroma is loose, hypercellular, and fibrovascular that may not show aprominent vascular component
18. The radiographic feature of SOT is
A. Radiolucent, semicircular, apical portion of root B. Radiolucent, pear shaped, cervical portion of root
C. Radiopaque, semicircular, apical portion of root D. Radiopaque, pear shaped, cervical portion of root
19. Odontoma is a
A. Hamartoma B. Choristoma
C. Teratoma D. None of the above
20. Compound odontoma
A. Anterior maxilla B. Anterior mandible
C. Posterior maxilla D. Posterior mandible
21. The treatment for odontoma is
A. Surgical excision and does not recur B. No treatment and does not recur
C. Surgical excision and it does recur D. No treatment and it does recur
22. One of the following is not a variety of dentigerous cyst
A. Central B. Lateral
C. Medial D. Circumferential
23. The most common cyst is
A. Dental cyst B. Dentigerous cyst
C. OKC D. CEOT
24. CEOT is often seen along with
A. Odontoma B. Ameloblastoma
C. OKC D. Dentigerous cyst
25. Tennis racket appearance is seen in
A. OKC B. Odontogenic myxoma
C. Ameloblastic fibroma D. Ameloblastic carcinoma

BENIGN AND MALIGNANT LESIONS


1. HPV subtypes associated with malignancies are
A. HPV 6 and 8 B. HPV 16 and 18
C. HPV 6 and 11 D. HPV 7
2. The difference between papilloma and common wart is
A. Papilloma is contagious and wart is not B. Common wart is contagious and papilloma is not
C. Autoinoculation is not seen in wart D. Wart is associated with Cowden’s syndrome
3. Verrucoma is also known as
A. Verrucous carcinoma B. Verruca vulgaris
C. Keratoacanthoma D. Papilloma
4. _______ is a true mesodermal structure composed of dermal melanocytes which only rarely undergo malignant
transformation.
A. Spindle cell nevus B. Spitz nevus
C. Blue nevus D. Intradermal nevus

https://t.me/DentalBooksWorld
320 Triumph's Complete Review of Dentistry

5. Distribution of intramucosal nevi in oral cavity:


A. Hard palate > buccal mucosa > gingiva > lip B. Hard palate > buccal mucosa > lip > gingiva
C. Hard palate > lip > buccal mucosa > gingiva D. Hard palate > lip > gingiva >buccal mucosa
6. Which has high malignant potential?
A. Speckled leukoplakia B. Nodular leukoplakia
C. PVL D. Verrucous carcinoma
7. Normal nuclear cytoplasmic ratio in case of dysplasia is
A. 1:1 B. 1:2
C. 1:5 D. 1:8
8. In Oral leukoplakia (OLEP) staging, stage 2 is
A. L1 P1 B. L2 P0
C. L2 P1 D. L1 P2
9. Differentiation of erythroplakia with malignant change and other early squamous cell carcinomas from benign
inflammatory lesions of the oral mucosa can be enhanced by use of ________ solution applied topically with a
swab or as an oral rinse.
A. Silver nitrite B. Acetic acid
C. Methylene blue D. Tolonium chloride
10. Palatal changes comprise several components in Reverse Smoking except
A. Keratosis and excrescences B. Pigmented areas
C. Nonpigmented areas D. Patches
11. The triad of lesions seen in palatal erythema are all except
A. Palatal papillary hyperplasia B. Central papillary atrophy of the tongue
C. Bilateral commissural leukoplakias D. Candidiasis
12. All of the following are the biochemical alterations seen in OSMF patients except
A. Increased ESR B. Decrease in serum iron
C. Reduced LDH isoenzyme D. Reduced antistreptolysin “O” titer
13. Counting of AgNORs may be useful as a predictor of the biological behavior of
A. Leukoplakia B. OSMF
C. SCC D. BCC
14. Precancerous lesions are all except
A. Leukoplakia B. Solar keratosis
C. Dysplasia D. Xeroderma pigmentosum
15. T1 N1 MO lesion is
A. Stage I B. Stage II
C. Stage III D. Stage IV
16. Highly sensitive marker for malignant melanoma is
A. HMB45 B. S – 100
C. Cyclin D D. CDKN
17. Giant cell fibroma is most commonly seen in
A. Palate B. Tongue
C. Buccal mucosa D. Mandibular gingiva
18. All of the following are true about peripheral ossifying fibroma except
A. Peak incidence is 45 years of age
B. Female predilection
C. Originates from interdental papilla
D. The lesions are approximately equally divided between the maxilla and the mandible
19. A 12-year-old child came with soft tissue growth on lingual surface of anterior teeth as shown in the figure. Few areas
of calculus deposition were also seen. He had no systemic illness. How will you proceed?
A. Scaling followed by excisional biopsy B. Refer to oncologist
C. Excision with laser D. Give antibiotics andreexamine after 2 weeks

https://t.me/DentalBooksWorld
Chapter 4 • Oral Pathology and Oral Medicine 321

20. A 21-year-old female reported with a painless hard expansile growth extending from gingiva of 23 to 26 which slowly
grew in size in the past 7 months. Histologic examination revealed speckled calcified deposition with little stroma
andosteoblastic rimming. Most likely diagnosis is
A. Calcifying odontogenic cyst B. Ameloblastoma
C. Cemento-ossifying fibroma D. Giant cell granuloma
21. A 25-year-old female patient came for RCT. On routine radiograph, a swelling in the anterior mandible region was
found andon examination the lesion was present from 13 to 23. Microscopic findings reveal loose fibrillar connective
tissue stroma with many interspersed proliferating fibroblasts and small capillaries and fibers will often present a
whorled appearance. What can be the diagnosis?
A. Peripheral giant cell granuloma B. Central giant cell granuloma
C. Peripheral ossifying fibroma D. Central ossifying fibroma
22. Central giant cell granuloma is differentiated from giant cell tumor of bone by
A. Size of nuclei B. Number of nuclei
C. Origin of giant cells D. Number of giant cells
23. Intramuscular hemangiomas in the oral region are most commonly seen in the
A. Lateral pterygoid B. Medial pterygoid
C. Masseter D. Buccinator
24. Osteosarcoma occurs chiefly in?
A 10–25 years, males B. 10–25 years, females
C. 35–55 years, males D. 35–55 years, females
25. Microscopic findings of osteosarcoma includes all except
A. The chondroblastic elements appear astan colored and fibroblastic elements appear as translucent lobules
B. Characterized by the proliferation of bothatypical osteoblasts and their less differentiated precursors
C. Hemorrhage and necrosis are common
D. The characteristic feature of osteosarcoma is thepresence of osteoid formed by malignant osteoblasts in the lesion
26. The most common chromosomal abnormality associated with NHL is the
A. t(14;18) B. t(18;14)
C. t(14;21) D. t(21;14)
27. Involvement of two or more lymph node regions on the same side of the diaphragm with unexplained fever with
temperatures above 38°C denotes
A. Stage II subclass B B. Stage 1 subclass B
C. Stage II subclass C D. Stage II subclass A
28. Centrocytes and centroblasts are seen in
A. Hodgkin’s lymphoma B. Non-Hodgkin’s lymphoma
C. Burkitt’s lymphoma D. Malignant melanoma
29. Most common source for cancers that metastasize to the oral soft tissues:
A. Breast B. Prostate
C. Lung D. Liver

https://t.me/DentalBooksWorld
322 Triumph's Complete Review of Dentistry

BONE PATHOLOGY
1. A 60-year-old male patient reports of painless swelling of maxilla since 4 years. On examination the maxilla was
expanded bilaterally. Laboratory diagnosis reveal that the patient had increased alkaline phosphatase levels. What can
be the diagnosis?
A. Fibrous dysplasia B. Cherubism
C. Paget’s disease D. Osteosarcoma
2. The following cannot be the differential diagnosis for the given image

A. Fibrous dysplasia B. Osteosarcoma


C. Paget’s disease D. Hyperparathyroidism
3. One of the following is false for the given image (AIIMS, November 2016)

A. Core-binding factor alpha-1 is involved B. Open fontanels


C. Enlarged frontal sinus D. Mandibular prognathism
4. Identify the syndrome from the given image

A. Treacher Collins syndrome B. Apert’s syndrome


C. Down’s syndrome D. Ellis–van Creveldsyndrome
5. Identify the possible syndrome for the given image

A. Chondroectodermal dysplasia B. Ectodermal dysplasia


C. Cleidocranial dysostosis D. Mandibulofacial dysostosis

https://t.me/DentalBooksWorld
Chapter 4 • Oral Pathology and Oral Medicine 323

6. Identify the syndrome from the given image

A. Treacher Collins syndrome B. Apert’s syndrome


C. Down’s syndrome D. Ellis–van Creveld syndrome
7. Most common oral anomaly of Down’s syndrome:
A. Cleft lip or palate B. Geographic tongue
C. Enamel hypoplasia D. Fissured tongue
8. All of the following are true about Paget’s disease except
A. Males are more commonly affected B. Viral infection is a possible etiology
C. Autosomal dominant inheritance D. serum calcium levels are normal
9. Bilateral mandibular involvement is seen in
A. Cherubism B. Lichen planus
C. Osteoma D. Central giant cell granuloma
10. Ground glass appearance is seen in
A. Hyperparathyroidism B. Fibrous dysplasia
C. Osteopetrosis D. Condensing osteitis

BLOOD DISORDERS
1. “Bald tongue of Sandwith” is seen in the deficiency of
A. Niacin B. Riboflavin
C. Vit. B12 D. Folic acid
2. All of the following are seen in pernicious anemia except
A. Elevated indirect bilirubin B. Increased size of leukocytes
C. Howell–Jolly bodies D. Decreased serum lactic dehydrogenase
3. All of the following are found in thalassemia except
A. Safety pin cells B. Target cells
C. Rib–within a rib appearance D. Enamel hypoplasia
4. Cells increased in infectious mononucleosis are
A. Monocytes B. Eosinophils
C. Lymphocytes D. Macrophages
5. One of the following is true about Wiskott–Aldrich syndrome?
A. Almost exclusively found in boys B. Immunoglobulin G (IgG) deficiency
C. Qualitative defect in blood platelets D. Does not lead to occurrence of malignant lymphoma
6. Disease of Hapsburg is a
A. Class III malocclusion B. Von Willebrand’s disease
C. Hemophilia D. ITP
7. Laboratory findings in patients with severe aplastic anemia may show
A. Normal bleeding time and prolonged clotting time B. Prolonged bleeding time and normal clotting time
C. Prolonged bleeding and clotting times D. Decreased bleeding time and normal clotting times
8. Oral manifestations are reported to be the maximum times in which form of leukemia?
A. Monocytic leukemia B. Lymphocytic leukemia
C. Myelogenous leukemia D. Megakaryocytic leukemia

https://t.me/DentalBooksWorld
324 Triumph's Complete Review of Dentistry

9. Parahemophilia is the disorder characterized by


A. Deficiency of multiple clotting factors B. Deficiency of factors VIII and X
C. Deficiency of factor V D. Deficiency of factor II
10. Fessas bodies seen within the erythrocytes of patients with beta thalassemia are
A. Insoluble precipitates of excess alpha chains B. Soluble precipitates of beta chains
C. Precipitated out iron component of hemoglobin D. Conjugated DNA of RBCs

ALLERGIC AND IMMUNOLOGIC DISORDERS


1. Bacterial infection causing recurrentaphthous stomatitis is
A. Streptococci B. Staphylococci
C. Lactobacillus D. Actinomyces
2. Antibiotics used to treat recurrent aphthous stomatitis is
A. Amoxicilline B. Ciprofloxacin
C. Tetracycline D. Acyclovir
3. Recurrent aphthous stomatitis in certain aspects of the clinical appearance, duration of lesions and chronic recurrence,
and general failure of response to any form of therapy mimics closely
A. Hand, foot and mouth disease B. Herpangina
C. Herpes simplex infection D. Any of the above
4. The endocrinal hormones which are regarded as precipitating factors for recurrent aphthous stomatitis are
A. Testosterone B. Growth hormone
C. Thyroxine D. Progesterone
5. The characteristic cells containing changes in the nuclei of epithelial cells taken by cytologic smears from around
recurrent aphthous ulcers are known as
A. Tzanck cells B. Lipschutzbodies
C. Hyaline cells D. Anitschkow cells
6. Reiter’s syndrome clinically mimics
A. Behcet’s syndrome B. Sutton’s disease
C. Gonorrhea D. Syphilis
7. The test used to diagnose sarcoidosis is
A. Tuberculin test B. Mantoux reaction
C. Kveim–Siltzbach test D. Tzanck smear test
8. Histopathologically, lesions of sarcoidosis and tuberculosis can be differentiated by the
A. Presence of nests of epithelioid cells with multinucleated giant cells in tuberculosis
B. Presence of granulomatous nodules in sarcoidosis
C. Fact that no acid fast organism can be demonstrated in sarcoidosis
D. Presence of caseous necrosis in tuberculosis
9. Quincke’s edema was also referred to, in the past, as angioneurotic edema because
A. Most patients were also suffering from hemangiomas B. Most patients presented with psychological problems
C. Most patients reported with brain tumors D. None of the above
10. Stomatitis medicamentosa is
A. An allergic reaction of the oral mucosa to the systemic administration of a drug
B. A toxic reaction of the oral mucosa to the systemic administration of a drug
C. A caustic reaction of the oral mucosa to the local application of a drug, such as aspirin burns
D. Oral ulcerations induced as a result of pharmacologic effects of drugs on systemic administration

BACTERIAL, VIRAL, AND FUNGAL INFECTIONS


1. Entry of the microorganism into the body in scarlet fever is primarily via
A. Nose B. Skin
C. Pharynx D. Mouth

https://t.me/DentalBooksWorld
Chapter 4 • Oral Pathology and Oral Medicine 325

2. Corynebacterium diphtheriae is
A. Anaerobic Gram-positive organism B. Anaerobic Gram-negative organism
C. Aerobic Gram-positive organism D. Aerobic Gram-negative organism
3. The earliest sensation to be affected in leprosy is
A. Pain B. Temperature
C. Touch D. Proprioception
4. Botryomycosis is caused by
A. Actinobacillus B. Staphylococcus
C. Streptococcus D. Pseudomonas
5. One of the following is correct about “mucous patches” seen in secondary syphilis
A. Painful B. Usually occurs as single patch
C. Most commonly seen in tongue D. They are uncommon in oral cavity
6. A 5-year-old child complains of fever and pain upon swallowing. The mouth was painful and the gingiva which is
intensely inflamed appeared erythematous and edematous. The lips, tongue, buccal mucosa, palate, pharynx, and
tonsils may also be involved. Yellowish, fluid-filled vesicles were also seen. What can be the diagnosis?
A. Herpangina B. Aphthous ulcer
C. Primary herpetic stomatitis D. Recurrent herpes
7. Herpangina is
A. Winter disease B. Spring disease
C. Summer disease D. Autumn disease
8. An otherwise healthy patient reports to you with rashes as given in the image. What can be the diagnosis?

A. Verruca vulgaris B. Mollascum contagiosum


C. Condyloma acuminatum D. Chicken pox
9. EC Clearing house classified
A. Oral manifestations of HIV B. Types of HIV
C. Oral manifestations of candidiasis D. Types of candidiasis
10. The evidence of HIV infection was first documented in India in
A. Kolkata B. Mumbai
C. Chennai D. Delhi
11. A patient complains of fever and breathlessness, and it is found to be an inflammatory granulomatous condition. On
microscopic examination, acute angulating branched septate hyphae can be seen. What can be the diagnosis?
A. Aspergillosis B. Candidiasis
C. Mucormycosis D. Histoplasmosis
12. The lesion of pyogenic granuloma are
A. Essentially pyogenic that is pus producing B. Granulomatous lesion, with specific microbial infection
C. Response of the tissue to a nonspecific infection D. A benign tumor of gingival tissue
13. The very first oral manifestation of tetanus may include
A. Facial paralysis B. Trismus due to spasm of buccinators muscle
C. Trismus due to spasm of masseter muscle D. Dysphagia

https://t.me/DentalBooksWorld
326 Triumph's Complete Review of Dentistry

14. Which of the following statements made regarding herpangina are true?
A. Herpangina is chiefly a winter disease
B. The ulcers of herpangina are extremely painful and heals with scarring
C. It is a specific viral infection caused by herpes family of viruses
D. None of the above
15. Measles is basically known for
A. Acute, noncontagious, neurotrophic viral infection B. Acute, contagious, dermotropic viral infection
C. Chronic, noncontagious viral infection D. sexually transmitted viral infection
16. The inclusion body seen in the disease Molluscum contagiosum is known as
A. Lipschutz body B. Cowdry type A
C. Cowdry type B D. Anitschkow body
17. In India, the chief organization concerned with only the health of the people affected with HIV/AIDS is
A. WHO B. NLEP
C. NACO D. NRHM
18. On histopathological examination of a tissue infected fungal infection, it shows typical round organisms, often
budding, measuring between 5 and 15 µm in diameter and have a characteristic doubly refractile capsule. The most
suggested diagnosis would be
A. Paracoccidioidomycosis B. Darling’s disease
C. Coccidioidomycosis D. North American blastomycosis
19. The candidiasis infection usually associated with a denture wearer patient is
A. Chronic hyperplastic candidiasis B. Pseudomembranous candidiasis
C. Chronic atrophic candidiasis D. Erythematous candidiasis
20. The most common intraoral site involved by rhinosporidiosis is
A. Gingiva B. Soft palate
C. Tongue D. Buccal mucosa

ORAL MANIFESTATIONS OF METABOLIC DISEASE


1. Which of the following viral disease presents with oral manifestations?
A. Scarlet fever B. Herpangina
C. Candidiasis D. Cryptococcosis
2. Urbach–Wiethe disease – all are oral manifestations except
A. Macroglossia B. Supernumerary teeth
C. Stenosis of the parotid duct opening D. Enamel hypoplasia
3. Premature loss of tooth and hypocalcification of teeth with large pulp chambers, loss of alveolar bone, and absence of
cementumindicated the diagnosis of
A. Amelogenesis imperfecta B. Dentinogenesis imperfecta
C. Hypophosphatasia D. Down’s syndrome
4. One of the following statements true for hypophosphatasia is
A. Both B and C B. Excretion of phosphoethanolamine in the urine
C. Deficiency of the enzyme alkaline phosphatase in serum D. Only C
5. Hypothyroidism in children is
A. Acromegaly B. Cretinism
C. Myxedema D. Gigantism
6. In hyperthyroidism
A. Shedding of the deciduous teeth occurs slower than normal, and eruption of the permanent teeth is greatly accelerated
B. Shedding of the deciduous teeth occurs earlier than normal, and eruption of the permanent teeth is also slower
C. Shedding of the deciduous teeth occurs earlier than normal, and eruption of the permanent teeth is greatly
accelerated
D. Shedding of the deciduous teeth occurs earlier than normal, and eruption of the permanent teeth is greatly retarded

https://t.me/DentalBooksWorld
Chapter 4 • Oral Pathology and Oral Medicine 327

PERIAPICAL INFECTIONS
1. The best technique to differentiate between periapical cemental dysplasia and periapical granuloma in a woman with
history of cardiac valve replacement is
A. Culture and sensitivity of RCT washings B. Serum chemistry
C. Tooth vitality D. Radiographic appearance
2. Phlegmon is
A. An STD B. A type of cellulitis
C. A type of osteomyelitis D. A venereal disease
3. Periapical cyst can be differentiated from periapical granuloma by
A. Radiograph B. Vitality testing
C. Histopathology D. Transillumination
4. Which is the most difficult to diagnose radiographically?
A. Necrosed pulp B. Internal resorption
C. External resorption D. Acute apical abscess
5. A patient comes to you with an ice jar on his right side face and says the pain gets relieved when ice jar is placed,
diagnosis?
A. Reversible pulpitis B. Irreversible pulpitis
C. Abscess D. Apical periodontitis
6. Treatment of a pulp polyp is
A. Condition is reversible and do RCT B. Condition is not reversible as tooth is extracted
C. No treatment D. Antibiotic and review after a week
7. Earliest periodontal ligament change in case of periapical granuloma is
A. Thickening of the ligament at the root apex B. Loss of ligament at the root apex
C. Widening of ligament at the root apex D. None of the above
8. The most commonly involved teeth in radicular cyst is
A. Maxillary anteriors B. Mandibular anteriors
C. Maxillary posteriors D. Mandibular posteriors
9. The epithelium lining the apical periodontal cyst is usually
A. Pseudostratified ciliated columnar B. Respiratory type of epithelium
C. Stratified squamous keratinized epithelium D. Stratified squamous non keratinized
10. Garre’s chronic nonsuppuratives clerosing osteitis – Which among the following is false?
A. Affects young persons
B. Most frequently involves anterior surface of tibia
C. Surgical intervention is needed
D. Duplication of the cortical layer of bone is seen radiographically

SALIVARY GLAND
1. Pleomorphic adenoma commonly occurs in the
A. Upper pole of the superficial lobe B. Lower pole of the superficial lobe
C. Upper and middle poles of the superficial lobe D. Upper and lower poles of the superficial lobe
2. All are true about pleomorphic adenoma except
A. Does not show fixation either to the deeper tissues or to the overlying skin
B. Facial nerve involvement manifested by facial paralysis is common
C. Parotid gland in minor salivary glands are unencapsulated
D. Radiation therapy is contraindicated in pleomorphic adenoma
3. The tumor that can be divided on the basis of their morphologic appearances into four subtypes: solid, tubular,
trabecular, membranous is
A. Basal cell adenoma B. Canalicular adenoma
C. Mucoepidermoid carcinoma D. Adenoid cystic carcinoma

https://t.me/DentalBooksWorld
328 Triumph's Complete Review of Dentistry

4. A 70-year-old patient complains of swelling below the ear. Swelling was bilaterally present in both the parotid glands.
Macroscopic appearance consists of a variable number of confluent cystic spaces that contains clear fluid. What can
be the diagnosis?
A. Sjogren’s syndrome B. Pleomorphic adenoma
C. Warthin’s tumor D. Sialadenosis
5. The following tumor has male predilection
A. Pleomorphic adenoma B. Inverted ductal papilloma
C. Sialadenomapa pilliferum D. Intra ductal papilloma
6. Inverted ductal papilloma was first described by
A. White et al. in 1982 B. Black et al. in 1982
C. Silver et al. in 1984 D. Rose et al. in 1984
7. Grade the prevalence of salivary gland neoplasms according to AFIP data of salivary gland neoplasms
A. Mucoepidermoid carcinoma, adenocarcinoma, acinic cell carcinoma, PLGA, adenoid cystic carcinoma
B. Adenocarcinoma, acinic cell carcinoma, mucoepidermoid carcinoma, adenoid cystic carcinoma, PLGA
C. Mucoepidermoid carcinoma, acinic cell carcinoma, adenocarcinoma, adenoid cystic carcinoma, PLGA
D. Mucoepidermoid carcinoma, PLGA, acinic cell carcinoma, adenocarcinoma, adenoid cystic carcinoma
8. Which malignant salivary gland neoplasm closely resembles pleomorphic adenoma?
A. Acinic cell carcinoma B. Pleomorphic adenoma ex carcinoma
C. Mucoepidermoid carcinoma D. Cylindroma
9. One of the following is false about mucoepidermoid carcinoma
A. The mucoepidermoid carcinoma is not encapsulated
B. Prior exposure to ionizing radiation appears to substantially increase the risk of developing mucoepidermoid carcinoma
C. They resemble mucocele
D. Cyst formation is seen in high grade tumors
10. Dedifferentiated carcinomas are a variant of
A. Mucoepidermoid carcinoma B. Acinic cell carcinoma
C. Cylindroma D. PLGA
11. True malignant mixed tumor is
A. Mucoepidermoid carcinoma B. Carcinoma ex pleomorphic adenoma
C. Carcinosarcoma D. Metastasizing mixed tumor
12. Sialadenosis – true among the following is
A. Noninflammatory, nonneoplastic, bilateral enlargement with recurrence and pain
B. Noninflammatory, neoplastic, unilateral enlargement with recurrence and pain
C. Noninflammatory, nonneoplastic, bilateral enlargement without recurrence and pain
D. Noninflammatory, neoplastic, unilateral enlargement without recurrence and pain
13. False about necrotizing ulceritis is
A. Lesions are usually painless
B. Male predilection
C. Anonneoplastic condition
D. The microscopic features of NS include caseous necrosis of glandular acini and squamous metaplasia of its ducts
14. Which HLA system is associated with both forms of Sjogren’s syndrome?
A. HLA-DRw3 B. HLA-B8
C. HLA-DRw52 D. HLA-DRw43
15. Histologically, Sjogren’s syndrome and Mikulicz disease are differentiated by
A. Lymphocytic infiltration B. Epimyoepithelial islands
C. Atrophy of glands D. All of the above
16. The most common site for occurrence for polymorphous low-grade adenocarcinoma is
A. Parotid salivary glands B. Minor salivary glands
C. All major salivary glands D. Only submandibular salivary glands

https://t.me/DentalBooksWorld
Chapter 4 • Oral Pathology and Oral Medicine 329

17. This appearance is classically described as

A. Snow storm B. Tree in winter


C. Leaf less fruit laden pattern D. Ball in hand
18. Lesions similar to necrotizing sialometaplasia occurring on skin are known as
A. Syringometaplasia B. Necrotizing fasciitis
C. Tricho metaplasia D. Lupus necrotizing disease
19. If there is an abnormal enlargement of salivary glands along with lacrimal glands and the etiology is known, then the
disease may be characterized as
A. Mikulicz syndrome B. Mikulicz disease
C. Any of the above D. Primary Sjogren’s syndrome
20. Kuttner tumor is also known as
A. Acute inflammatory sialadenitis of submandibular gland
B. Chronic inflammatory sialadentis of submandibular gland
C. Chronic sclerosing sialadentis of submandibular gland
D. Chronic sclerosing sialadentis of parotid gland

SKIN PATHOLOGY
1. Saw tooth rete pegs are seen in
A. Lichen planus B. Psoriasis
C. Pemphigus D. Erythema multiforme
2. Ehlers–Danlos syndrome is
A. Autosomal dominant B. Autosomal recessive
C. Sex-linked dominant D. Sex-linked recessive
3. Auspitz’s sign is seen in
A. Pemphigus B. Lichen planus
C. Psoriasis D. Leukoplakia
4. Test tube shaped rete pegs are seen in
A. Psoriasis B. OKC
C. Pemphigus D. Radicular cyst
5. Which type of epidermolysis bullosa is positive for Nikolsky?
A. Epidermolysis bullosa dystrophic, dominant B. Epidermolysis bullosa dystrophic, recessive
C. Junctional epidermolysis bullosa D. Epidermolysis bullosa simplex
6. All are true for white sponge nevus except
A. Congenital abnormality B. Can change into malignancy
C. Autosomal dominant trait D. No definite sex predilection
7. Widening of the periodontal ligament is seen in all except
A. Scleroderma B. Osteosarcoma
C. Lupus erythematosus D. Leukemia

https://t.me/DentalBooksWorld
330 Triumph's Complete Review of Dentistry

8. Gene involved in ectodermal dysplasia with cleft lip/palate is


A. GJB6 B. ED1
C. ED2 D. PVRL1
9. The given histopathology is
A. Pemphigus B. Pemphigoid
C. Lichen planus D. Psoriasis
10. Farmer’s skin – False statement among the following is
A. Damaged by prolonged exposure to chemicals
B. Also known as sailor’s skin
C. Basophilic degeneration
D. Damaged by prolonged exposure to elements of the weather

DISEASES OF NERVES AND MUSCLES


1. Sphenopalatine neuralgia differs from that of trigeminal neuralgia in that
A. It occurs bilaterally B. Females are affected more commonly than men
C. Onset of paroxysm occurs at different times of the day D. There is no trigger zone
2. Uhthoff ’s sign is seen in
A. Bell’s palsy B. Trigeminal neuralgia
C. Cerebral palsy D. Multiple sclerosis
3. Severe pain which arises after injury to orsectioning of a peripheral sensory nerve is
A. Hyperalgesia B. Neuralgia
C. Allodynia D. Causalgia
4. Enlargement of scattered muscle fibers and the presence of centrally placed muscle nuclei inlong rows have been
described as being the characteristic of
A. Paramyotonia B. Myasthenias
C. Myotonia D. Thomsen’s disease
5. The muscle that is spared in myositis ossificans progressiva is
A. Masseter B. Rectus abdominis
C. Genioglossus D. Risorius
6. Horner’s syndrome involves all except
A. Ptosis B. Miosis
C. Mydriasis D. Anhidrosis
7. Statement A: Atypical facial pain is not associated with diseases of the teeth, throat, nose, sinuses,eyes, or ears.
Statement B: This pain, which lacks a triggerzone, is constant and persists for weeks, months, or even years.
A. Both the statements are true B. Both the statements are false
C. A is true and B is false D. B is true and A is false
8. Crocodile tears in pathology literature refers to
A. Fake tears in psychologically disturbed persons to gain attention
B. The tears tinged with red color because of associated hemorrhage in lacrimal ducts
C. Patients exhibiting profuse lacrimation when having hot and spicy foods
D. Spontaneous lacrimation without any apparent physical or psychological stimulus
9. The atrophy of the masseter muscle along with ptosis and generalized weakness of the facial musculature as seen in
dystrophic myotonia imparts a characteristic appearance to patient known as
A. Bull’s neck appearance B. Swan neck appearance
C. Mouse facies appearance D. Mongoloid facies
10. Myotonia refers to
A. Muscular atrophy
B. Muscular hyperplasia
C. Failure of muscle relaxation after cessation of voluntary contraction
D. Failure of muscle contraction after cessation of voluntary relaxation

https://t.me/DentalBooksWorld
Chapter 4 • Oral Pathology and Oral Medicine 331

FORENSIC ODONTOLOGY
1. Access for radiography in incinerated bodies can be obtained by removing the
A. Teeth B. Tongue
C. Cheeks D. Alveolar bone
2. Enamel rod end pattern using acetate peel technique – ameloglyphics – was recorded by
A. Manjunath K et al. B. Acharya andVimi
C. Rajendren D. Muller et al.
3. Bilateral labial rotation of the distal margins of maxillary central incisors is
A. Shoveling B. Rotating
C. Winging D. None
4. Demirjian’s method uses which quadrant for age assessment?
A. Mandibular right B. Maxillary right
C. Mandibular left D. Maxillary left
5. All of the following authors modified the original Gustafson’s method except
A. Bang and Ramm B. Johanson
C. Acharya and Vimi D. Schourand Massler
6. Which tooth was used to identify the age by using the pulp-to-tooth area ratio?
A. Incisors B. Canines
C. Premolars D. Molars
7. Lip prints was classified by
A. Webster B. Santos
C. Johanson D. Lee et al.
8. Cheiloscopy is
A. Examination of lip print B. Cleft lip surgery
C. Lip reshaping D. Lip print anomalies
9. Gustafson’s method assessed the following regressive changes except
A. Dentin deposition B. Cementum resorption
C. Root resorption D. Dentin translucency
10. Dental report in postmortem is recorded as
A. Odontogram B. Case files
C. Case reports D. Dental reports

MISCELLANEOUS
1. Canalicular adenoma is common in
A. Lower lip B. Upper lip
C. Parotid gland D. Submandibular gland
2. Gustatory sweating occurs in
A. Trotter’s syndrome B. Auriculotemporal syndrome
C. Costen’s syndrome D. Myofascial pain dysfunction syndrome
3. The other name for factor XI is
A. Stuart–Power factor B. Calcium
C. Plasma thromboplastin component D. Plasma thromboplastin antecedent
4. “Onion skin appearance” in the radiograph is suggestive of
A. Osteosarcoma B. Chondrosarcoma
C. Ewing’s sarcoma D. Multiple myeloma
5. Koplik’s spots are pathognomonic of
A. Rubeola B. Rubella
C. Mumps D. Infectious mononucleosis

https://t.me/DentalBooksWorld
332 Triumph's Complete Review of Dentistry

6. The inclusion bodies seen in molluscumcontagiosumare


A. Lipschutz bodies B. Negri bodies
C. Henderson–Patterson bodies D. Colloid bodies
7. The most common type of actinomycosis is
A. Pulmonary B. Cervicofacial
C. Abdominal D. Hepatic
8. Which of the following is true about pyogenic granuloma?
A. A true granuloma B. Reactive lesion
C. Caused by pyogenic organisms D. Neoplastic lesions
9. Marcus Gunn phenomenon – all are true except
A. Also known as jaw winking syndrome
B. Commonly seen in adults
C. Consists of unilateral ptosis
D. Aberrant connection between trigeminal nerve and oculomotor nerve
10. The most common type of muscular dystrophy is
A. Mild restricted muscular dystrophy B. Myotonic dystrophy
C. Duchenne’s muscular dystrophy D. Ophthalmoplegic dystrophy

MISCELLANEOUS
1. Which of all following is commonly associated with vital adjacent tooth?
A. Dentigerous cyst B. Globulomaxillary cyst
C. Lateral cyst D. Periapical cyst
2. Bilateral mandibular involvement seen in
A. Cherubism B. Osteoma
C. Central giant cell granuloma D. Lichen planus
3. A 40-year-old lady presents with ameloblastoma in the mandibular molar region. Histopathological features reveal
A. Central loose tissue with mitotic figures
B. Peripheral palisading cells with central loosestellate reticulum
C. Peripheral palisading cells with artifact stroma
D. Central palisading cells which fails to differentiate
4. Which of the following is the true microdontia of a lateral incisor?
A. Peg lateral B. Natal teeth
C. Mulberry teeth D. Hutchinson’s teeth
5. Patients having extra X chromosomes exhibit
A. Dilacerations B. Taurodontism
C. Cheesy molars D. Mulberry molars
6. The treatment for a tooth with the epithelial structure at the level of the alveolar gingiva in a 2-month-old child
without any radiographic evidence of rootsis
A. Surgically remove it under LA
B. Let it remain as it is the future tooth and root formation occurs at around 6 months
C. Preserve it and reassure the parents
D. Extract if mobile under GA
7. How can you distinguish between unilocular fibrous dysplasia and hemifacial hypertrophy?
A. Changes in face size
B. Size of tooth and rate of eruption
C. Difference in the distance between anatomical soft tissue landmarks relative to midline
D. Change in palatal structures
8. Cysts present along the median raphe of hard palate and appeared to arise from entrapped epithelial remnants are
A. Epstein’s pearls B. Dental lamina
C. Bohn’s nodules D. Gingival cyst of adult

https://t.me/DentalBooksWorld
Chapter 4 • Oral Pathology and Oral Medicine 333

9. Cysts arising due to degeneration of stellate reticulum in place of tooth are


A. Primordial cyst B. Dentigerous cyst
C. Lateral periodontal cyst D. Globulomaxillary cyst
10. Supernumerary tooth, tooth within a tooth, is most commonly seen in
A. Maxillary lateral incisor B. Mandibular lateral and central incisor
C. Mandibular second premolar D. Mandibular incisors
11. A 30-year-old patient having chronic pain of long duration with radiographic appearance of endosteal bone formation,
sclerosed bone is likely to be
A. Ewing’s sarcoma B. Garre’s osteomyelitis
C. Chronic focal sclerosing osteomyelitis D. Acute osteomyelitis
12. Which form of osteomyelitis have a negative microbiological culture and does not respond to antibiotic treatment?
A. Acute suppurative osteomyelitis B. Bidder osteomyelitis
C. SAPHO syndrome D. Garre’s osteomyelitis
13. Which of the following disorder is associated with increase in dental caries?
A. Epidermolysis bullosa B. Down’s syndrome
C. Turner’s syndrome D. Hereditary fructose intolerance 
14. Enamel loss over molars and incisors is described as
A. Qualitative B. Quantitative
C. Posteruptive deformity D. Quantitative and qualitative
15. Supernumerary teeth is seen in all except
A. Gardner’s syndrome B. Seckel’s syndrome
C. Apert’s syndrome D. Down’s syndrome
16. Correct sequence among the following is
A. Inoculation, abscess, cellulitis B. Abscess, inoculation, cellulitis
C. Cellulitis, abscess, inoculation D. Inoculation, cellulitis, abscess
17. Which of the following is true about lichen planus?
A. Common on extensor surface of skin B. Most common in females in ratio of female:male= 3:2
C. Hepatitis C is most commonly associated D. Prevalence in general population is 3.7–5.8%
18. Gene associated with amelogenesis imperfecta
A. ENAM B. DSSP
C. MSX-1/MSH-1 D. PAX-9
19. Which one of the following is NOT a lichenoid reaction?
A. Graft versus host disease B. Fixed drug eruption
C. Pemphigus vulgaris D. Secondary syphilis 
20. Narrow, high-arched palate, prolonged retention of deciduous teeth and failure in the eruption of permanent teeth is
a characteristic feature of
A. Paget’s disease B. Cherubism
C. Cleidocranial dysplasia D. Osteogenesis imperfecta

RANDOM REPEATS
1. Which of the following is not a component of Melkersson–Rosenthal syndrome?
A. Facial paralysis B. Fissured tongue
C. Cheilitis granulomatosa D. Cheilitis glandularis
2. Which of the following is true about xerostomia?
A. pH of saliva increases
B. pH of saliva decreases
C. pH increases in the morning and decreases in the evening
D. No change in the pH of saliva
3. Acinic cell carcinoma occurs most commonly in
A. Parotid gland B. Submandibular gland
C. Sublingual gland D. Minor salivary gland

https://t.me/DentalBooksWorld
334 Triumph's Complete Review of Dentistry

4. Mixed tumor of the salivary gland is


A. Usually malignant
B. Associated with calculi formation
C. Most common in submandibular gland
D. Made up of both ductal and myoepithelial cell proliferation
5. The major difference between primary and secondary Sjogren’s syndrome is
A. Increased salivation B. Decreased salivation
C. Alteration in pH of saliva D. Presence/absence of connective tissue disorders
6. Xerostomia is seen in all except
A. Anticholinergic drugs B. Sjogren’s syndrome
C. Dental caries D. Dehydration
7. Marcus Gunn phenomenon – all are true except:
A. Also known as jaw winking syndrome
B. Commonly seen in adults
C. Consists of unilateral ptosis
D. Aberrant connection between trigeminal nerve and oculomotor nerve
8. The most common muscle involved in myositis ossificans progressive is?
A. Lateral pterygoid B. Medial pterygoid
C. Masseter D. Platysma
9. Which of the following is present in the jaw winking syndrome?
A. Unilateral ptosis B. Bilateral ptosis
C. Exophthalmos D. Nystagmus
10. Gustatory sweating occurs in
A. Trotter’s syndrome B. Auriculotemporal syndrome
C. Costen’s syndrome D. Myofascial pain dysfunction syndrome
11. The most common type of muscular dystrophy is
A. Mild restricted muscular dystrophy B. Myotonic dystrophy
C. Duchenne’s muscular dystrophy D. Ophthalmoplegic dystrophy
12. Lymphorrhages are seen in the histology of
A. Myotonia B. Hypotonia
C. Myasthenia gravis D. Dermatomyositis
13. “Petrified man” appearance is seen in
A. Myositis ossificans B. Proliferative myositis
C. Focal myositis D. Dermatomyositis
14. Parahemophilia is due to the deficiency of
A. Factor 8 B. Factor 9
C. Factor 5 D. Factor 7
15. The cycle of neutropenia occurring in cyclic neutropenia is
A. Every 14 days B. Every 21 days
C. Every 4 weeks D. Every week alternatively
16. Abnormal lysosomic granules resembling Dohle bodies are seen in
A. Cyclic neutropenia B. Subleukemic leukemia
C. Megaloblastic anemia D. Chediak–Higashi syndrome
17. The other name for factor XI is
A. Stuart–Power factor B. Calcium
C. Plasma thromboplastin component D. Plasma thromboplastin antecedent
18. In a leukemic patient bleeding does not stop due to
A. Decreased platelet count B. Decreased coagulation factors
C. Decreased calcium levels D. Increased WBC count

https://t.me/DentalBooksWorld
Chapter 4 • Oral Pathology and Oral Medicine 335

19. Hair on end appearance is the radiographic feature of


A. Aplastic anemia B. Megaloblastic anemia
C. Thalassemia D. Iron deficiency anemia
20. Rh hump is seen in
A. Sickle cell anemia B. Thalassemia
C. Erythroblastosis fetalis D. Megaloblastic anemia
21. Bald tongue of Sandwith is seen in
A. Iron deficiency anemia B. Megaloblastic anemia
C. Pellagra D. Pernicious anemia
22. Cooley’s anemia is also known as
A. Sickle cell anemia B. Thalassemia
C. Megaloblastic anemia D. Iron deficiency anemia
23. Chloroma refers to extramedullary manifestation of
A. Leukemia B. Hodgkin’s lymphoma
C. Non-Hodgkin’s lymphoma D. Multiple myeloma
24. The following is a form of scleroderma
A. Goldenhar syndrome B. Parry–Romberg syndrome
C. Pierre-Robin syndrome D. Down’s syndrome
25. The mildest form of cleft palate is observed as
A. Bifid uvula B. Cleft of soft palate
C. Cleft of primary palate D. Cleft of philtrum
26. Median rhomboid glossitis is recently considered to be a
A. Developmental disorder B. Reactive lesion
C. Herpetic infection D. Candidiasis
27. Turner’s teeth
A. Is more common in mandibular canines B. Is a form of enamel hypoplasia
C. Affects the root D. Occurs due to high fluoride content in water
28. “Lava flowing around boulders” is the histologic feature of
A. Dentin dysplasia type I B. Dentin dysplasia type II
C. Dentinogenesis imperfecta D. Shell teeth
29. Ghost teeth refers to abnormally in
A. Enamel and dentin B. Dentin and pulp
C. Enamel and pulp D. Enamel, dentin, and pulp
30. Deficiency of _____________ in pregnant women can lead to cleft lip or cleft palate
A. Vitamin K B. Amino acids
C. Vitamin C D. Folic acid
31. All of the following are potentially malignant disorders except
A. Oral submucous fibrosis B. Hairy leukoplakia
C. Erythroplakia D. Erosive lichen planus
32. Speckled leukoplakia clinically appears as
A. Red lesion B. White lesion
C. Both red and white lesions D. Grayish white lesion
33. Which of the following is not a feature of OSF?
A. Red discoloration of the mucosa B. Limitation in mouth opening
C. Deviation of the uvula D. Burning sensation of the mouth
34. The primary difference between hemangioma and vascular malformation is
A. Color of the lesion B. Size
C. Endothelial proliferation D. Type of vessel involved
35. Kaposi sarcoma is caused by
A. Herpes simplex virus B. Human herpes virus 8
C. Varicella zoster virus D. Cytomegalovirus

https://t.me/DentalBooksWorld
336 Triumph's Complete Review of Dentistry

36. Which of the following is not true about Sturge–Weber syndrome?


A. Facial angiomatosis B. Leptomeningeal calcifications
C. Absence of seizures D. Type of phakomatoses
37. Multiple myeloma is
A. Malignant tumor of plasma cells B. Malignant tumor of lymphocytes
C. Benign tumor of plasma cells D. Benign tumor of lymphocytes
38. The cell of origin for granular cell myoblastoma is
A. Skeletal muscle B. Smooth muscle
C. Nerve D. Blood vessel
39. Verocay bodies are characteristic feature of
A. Neurofibroma B. Neurilemmoma
C. Neuroma D. Neuroectodermal tumor
40. All are true about herpes simplex infection except
A. Latency in trigeminal ganglion B. Tzanck cells seen in cytological smear
C. Commonly affects the lining mucosa D. Secondary infection is called herpes zoster
41. All of the following are associated human papilloma virus except
A. Condyloma acuminatum B. Condyloma lata
C. Heck’s disease D. Verruca vulgaris
42. Strawberry gingivitis is characteristic of
A. Tuberculous granuloma B. Gumma
C. Wegener’s granulomatosis D. Scarlet fever
43. Which of the following is incorrect about koilocytes?
A. HPV altered cells B. Seen in connective tissue
C. Have pyknotic nuclei D. Clear cells
44. Cysticercosis is caused by
A. Taeniasolium B. Tinea capitis
C. Tinea versicolor D. Ascaris lumbricoides
45. Which of the following is a feature of congenital syphilis?
A. Hutchinson’s incisor B. Interstitial keratitis
C. Eighth nerve deafness D. All of the above
46. Mucormycosis is associated with
A. Uncontrolled diabetes B. Hypertension
C. Hypothyroidism D. All the above
47. All of the following are manifestations of HIV infection except
A. Candidiasis B. Oral hairy leukoplakia
C. Hypopigmentation D. Hyperpigmentation
48. Auto inoculation of herpes simplex virus in fingers cause
A. Herpetic gladiatorum B. Herpetic whitlow
C. Eczema herpeticum D. Scrumpox
49. The most common complication of mumps is
A. Orchitis B. Hepatitis
C. Pulmonary fibrosis D. Arthritis
50. HIV virus attaches itself to the surface of CD4 lymphocytes by
A. Gp41 B. Gp120
C. CCR3 D. CXCR5
51. Which of the following fungal organism has a mucopolysaccharide capsule? (Similar question came in NEET
2018)
A. Coccidioides immitis B. Paracoccidiodes
C. Cryptococcus neoformans D. Blastomyces dermatitidis

https://t.me/DentalBooksWorld
Chapter 4 • Oral Pathology and Oral Medicine 337

52. The causative organism for cat scratch disease is


A. Borreliavincenti B. Blastomycosis
C. Bartonella henselae D. Botryomycosis
53. Interstitial glossitis and luetic glossitis are features of
A. Primary syphilis B. Secondary syphilis
C. Tertiary syphilis D. Congenital syphilis
54. Dull, muffled sound on percussion is a characteristic clinical feature of
A. Subluxation B. Tooth ankylosis
C. Concussion D. None of the above
55. The main factor responsible for the destruction of clot leading to dry socket is
A. Fibrinogen B. Plasminogen
C. Fibrin D. Actin
56. Which of the following is not a feature of Reiter’s syndrome?
A. Arthritis B. Mucocutaneous lesions
C. Keratitis D. Urethritis
57. Riga Fede disease refers to
A. Abfraction B. Erosion
C. White patch D. Traumatic ulcer related to neonatal teeth
58. Which of the following is true about pyogenic granuloma?
A. A true granuloma B. Reactive lesion
C. Caused by pyogenic organisms D. Neoplastic lesions
59. Which space infection is not a component of Ludwig’s angina?
A. Submandibular B. Submental
C. Sublingual D. Submasseteric
60. Which of the following denture-related injuries is considered as atrophic candidiasis?
A. Epulis fissuratum B. Denture papillomatosis
C. Denture stomatitis D. Denture base intolerance
61. Cervicofacial emphysema is differentiated from angioedema by
A. Size of the swelling B. Radiography
C. Extent of the lesion D. Presence of crepitus
62. Sutton’s disease refers to
A. Mercury poisoning B. Recurrent aphthous ulcer
C. Lead poisoning D. Recurrent herpetic ulcer
63. Which of the following is not a cause of angioedema?
A. Virus B. Food allergy
C. ACE inhibitors D. C1 inhibitor deficiency
64. Id reaction refers to hypersensitivity to
A. Bacterial antigen B. Candidal antigen
C. Viral antigen D. Cryptococcal antigen
65. Pathologic loss of tooth structure due to biomechanical loading forces is called
A. Attrition B. Abrasion
C. Abfraction D. Erosion
66. Alveolar osteitis occurs due to
A. Destruction of blood clot B. Bacterial infection
C. Sequelae of caries D. Viral infection
67. Plasma cell gingivitis is a type of
A. Conditioned enlargement B. Allergic enlargement
C. Inflammatory enlargement D. Idiopathic enlargement

https://t.me/DentalBooksWorld
338 Triumph's Complete Review of Dentistry

68. All of the following causes tooth discoloration except


A. Cyclosporin B. Minocycline
C. Tetracycline D. Fluoride
69. Inverted pear-shaped radiolucency between the roots of maxillary lateral incisor and canine is most probably
A. Midpalatal cyst B. Globulomaxillary cyst
C. Nasopalatine cyst D. Nasolabial cyst
70. “Blood soaked sponge” appearance surgically is a characteristic of
A. Aneurismal bone cyst B. Central giant cell granuloma
C. Peripheral giant cell granuloma D. Giant cell tumor
71. Which of the following is not a feature of Osler–Weber–Rendu syndrome?
A. Telangiectasia B. Positive family history
C. Recurrent epistaxis D. Leptomeningeal calcifications
72. Collection of foamy macrophages in the connective tissue papilla is suggestive of
A. Myxoma B. Verruciform xanthoma
C. Chondromyxoid fibroma D. Giant cell fibroma
73. The differentiating feature between osteoid osteomas and osteoblastoma is
A. Age of the patient B. Gender
C. Presence or absence of pain D. Pain relief by salicylates
74. The most common malignancy occurring in bone affected with Paget’s disease is
A. Osteosarcoma B. Chondrosarcoma
C. Ewing’s sarcoma D. Metastatic carcinoma
75. Codman’s triangle representing an acute angle between the bone surface and periosteum is suggestive of
A. Chondrosarcoma B. Osteosarcoma
C. Ewing’s sarcoma D. Osteoid osteoma
76. The chromosomal abnormality noted in Ewing’s sarcoma is
A. t(8, 14) B. t(11, 22)
C. Inversion of 11q D. Amplification of 11q
77. Malignant ameloblastoma is
A. Characterized by cytologic atypia B. Malignant tumor
C. Seen in anterior maxilla D. Metastasize to distant site
78. Multiple OKC occurs in
A. Goltz–Gorlin syndrome B. Gorlin–Goltz syndrome
C. Gardner’s syndrome D. Goldenhar syndrome
79. Which of the following is not true about Rushton bodies?
A. Arc-shaped bodies B. Secreted by plasma cells
C. Also called hyaline bodies D. Amorphous in nature

TRIUMPH MCQ BANK


1. Interferon regulatory factor 6 is related to
A. Gardner’s syndrome B. Treacher Collins syndrome
C. Crouzon’s syndrome D. Van der Woude syndrome
2. Incidence of isolated cleft palate
A. 1 in 700 B. 1 in 1,000
C. 1 in 2,000 D. 1 in 3,000
3. of lip arises due to failure in fusion of
A. FNP, MNP, LNP, maxillary, mandibular processes B. FNP, MNP, maxillary, mandibular processes
C. MNP, maxillary, mandibular processes D. Maxillary and mandibular processes
4. Dilated composite odontome is
A. Dens invaginatus B. Dens evaginatus
C. Odontoma D. Enamel pearl

https://t.me/DentalBooksWorld
Chapter 4 • Oral Pathology and Oral Medicine 339

5. Tuberculate supernumeraries are commonly located on the


A. Palatal aspect of the central incisors B. Mesial aspect of the central incisors
C. Palatal aspect of molars D. Distal to molars
6. Supernumerary teeth is associated with all except
A. Unilateral cleft lip B. Cleidocranial dysplasia
C. Gardner’s syndrome D. Down’s syndrome
7. The radiographic and histologic features of cementoblastoma closely resembles
A. Osteoblastoma B. Osteoma
C. Osteoid osteoma D. Osteoclastoma
8. The most common site of metastasis in metastasizing ameloblastoma is
A. Liver B. Bone
C. Lung D. Lymph nodes
9. The histologic features of gingival cyst of adult is similar to
A. Gingival cyst of newborn B. Lateral periodontal cyst
C. Apical periodontal cyst D. Periapical cyst
10. Increased hyaluronic acid and chondroitin sulfate is seen in
A. Odontogenic fibroma B. Odontogenic myxoma
C. Complex odontoma D. Compound odontoma
11. The most common site for glandular odontogenic cyst is
A. Anterior maxilla B. Posterior maxilla
C. Anterior mandible D. Posterior mandible
12. The direct immunofluorescent studies of lichen planus show deposition of _____________ in the basement membrane
zone
A. Laminin B. Fibronectin
C. Fibrinogen D. Keratin
13. The basic disturbance in osteopetrosis
A. Excessive osteoblastic activity B. Excessive osteoclastic activity
C. Less osteoblastic activity D. Less osteoclastic activity
14. The triad of diabetes insipidus, exophthalmos, and osteolytic lesions is seen in
A. Eosinophilic granuloma B. Sculler’s disease
C. Hand–Schuller–Christian disease D. Letterer–Siwe disease
15. The serum calcium level is decreased by
A. Vitamin E B. Vitamin D
C. Parathormone D. Calcitonin
16. Which of the following is a feature of Hutchinson–Gilford syndrome?
A. Increased formation of secondary dentin B. Premature senility
C. Both A and B D. None of the above
17. This condition is seen in all of the following syndromes except
A. Down’s syndrome B. Lymphangioma
C. Pierre robin syndrome D. Carcinoma of tongue
18. Deposition of IgG in the epidermis with characteristic fish net appearance is seen in which of the following disease?
A. Bullous pemphigoid B. Pemphigus
C. Lichen planus D. Cicatricial pemphigoid
19. A 64-year-old male patient reported to our dental OP with a chief complaint of painless swelling in the right side
middle and lower third of the face since 30 years. The patient was apparently normal 30 years back. Patient gave a
history of swelling which was gradual in onset, initially small in size as size of the lesion was 2 cm ¥ 3 cm. Patient also
gave no relevant history of associated pain and fever. Diagnosis based on the clinical features is
A. Paget’s disease B. Fibrous dysplasia
C. Condensing osteitis D. Osseous fibroma

https://t.me/DentalBooksWorld
340 Triumph's Complete Review of Dentistry

ANSWERS

DEVELOPMENTAL ANOMALIES
1. Answer: C (Ref. Shafer’s Textbook of Oral Pathology, 7th edition, page no. 10)
• (Tabulation – classification of cleft lip)
• Unilateral incomplete – 33%
• Unilateral complete – 48%
• Bilateral incomplete – 7%
• Bilateral complete – 12%
2. Answer: D (Ref.Oral Pathology – E-Book: Clinical Pathologic Correlations, By Joseph A. Regezi, James J. Sciubba, and
Richard C. K.Jordan, 2012, page no. 368)
• The incidence of cleft lip and cleft palate has been reported to be 1 in 700–1,000 births, with variable racial predilection.
Isolated cleft palate is less common, with an incidence of 1 in 1,500–3,000 births. Cleft lip with or without cleft palate
is more common in males, and cleft palate alone is more common in females. Most cases of cleft lip and/or cleft
palate can be explained by the multifactorial threshold hypothesis. The multifactorial inheritance theory implies
many contributory risk.
• Genes interact with one another and the environment and collectively determine whether a threshold of abnormality is
breached, resulting in a defect in the developing fetus. Multifactorial or polygenic inheritance explains the transmission
of isolated cleft lip or palate, and it is extremely useful in predicting occurrence risks of this anomaly among family
members of an affected individual.
(Ref. Shafer’s Textbook of Oral Pathology, 7th edition, page no. 18)
• Bixler more recently has expanded upon this concept and reiterated that there are two forms of clefts. The most common
is hereditary, its nature being most probably polygenic (determined by several different genes acting together). In other
words, when the total genetic liability of an individual reaches a certain minimum level, the threshold for expression is
reached and a cleft occurs.
3. Answer: A (Ref. Shafer’s Textbook of Oral Pathology, 7th edition, page no. 19)
• An isolated cleft palate is etiologically and embryologically different from the cleft lip with or without cleft palate.
Several subtypes of isolated cleft palate can be diagnosed based on severity. The uvula is the place where the minimal
form of clefting of the palate is observed.
4. Answer: C (Ref. Shafer’s Textbook of Oral Pathology, 7th edition, page no. 19)
Veau classification
The Veau classification system is illustrated in:
• Group I (A): Defects of the soft palate only
• Group II (B): Defects involving the hard palate and soft palate
• Group III (C): Defects involving the soft palate to the alveolus, usually involving the lip
• Group IV (D): Complete bilateral clefts
5. Answer: B (Ref. Shafer’s Textbook of Oral Pathology, 7th edition, page nos. 21 and 22)
• Cheilitis glandularis – exclusively lower lip
• Cheilitis granulomatosa – both lips but upper lip slightly more common
6. Answer: B (Ref. Shafer’s Textbook of Oral Pathology- Developmental Disturbances of Oral and Paraoral Structures, 7th
edition, page no. 29)
7. Answer: A
• A case of osteogenesis imperfecta…
• Brown discoloration + Pulp obliteration + Bone Fractures…
8. Answer: B (Ref. Shafer’s Textbook of Oral Pathology, 7th edition, page no. 53)
• A type of hypoplasia occasionally seen is unusual in that only a single tooth is involved, most commonly one of the
permanent maxillary incisors or a maxillary or mandibular premolar. There may be any degree of hypoplasia, ranging
from a mild, brownish discoloration of the enamel to a severe pitting and irregularity of the tooth crown. These single
teeth are frequently referred to as “Turner’s teeth,” and the condition is called “Turner’s hypoplasia.”
• Answer will be premolars as primary molars are more commonly affected by caries than incisors.
9. Answer: B
10. Answer: A

https://t.me/DentalBooksWorld
Chapter 4 • Oral Pathology and Oral Medicine 341

11. Answer: A
12. Answer: D
13. Answer: B
14. Answer: B (Ref. Shafer’s Textbook of Oral Pathology, 2014, page no. 36)
• The wide spectrum of anomalies associated with hemifacial microsomia (HFM) has made systematic and inclusive
classification difficult. We propose a nosologic system in which each letter of the acronym O.M.E.N.S. indicates one of
the five major manifestations of HFM. O for orbital distortion; M for mandibular hypoplasia; E for ear anomaly; N for
nerve involvement; and S for soft tissue deficiency. The O.M.E.N.S. system is easily adapted for data storage, retrieval,
and statistical analysis. A retrospective study of 154 patients with HFM classified according to the O.M.E.N.S. system
confirmed the concept that the mandibular deformity is the cornerstone of the anomaly.
• Statistical analysis demonstrated a positive association between mandibular hypoplasia and the severity of orbital,
auricular, neural, and soft tissue involvement. This study did not confirm a previously reported predominance of gender or
sidedness. Analysis of statistical correlations failed to substantiate a Goldenhar variant as a syndromic entity. Our analysis
showed that palatal deviation is probably caused by muscular hypoplasia and not by weakness of a particular cranial nerve.
15. Answer: A
Table 1.  OMENS Classification System*
Orbit Facial nerve†
00  Normal orbital size, position N0  No facial nerve involvement
01  Abnormal orbital size N1 Upper facial nerve involvement
02  Abnormal orbital position (temporal or zygomatic branches)
03  Abnormal orbital size, position N2  Lower facial nerve involvement
N3  All branches affected
Mandible Soft tissue
M0 Normal S0  No obvious tissue or muscle deficiency
M1  Small mandible and glenoid fossa with ramus S1  Minimal soft tissue or muscle deficiency
M2 Ramus short and abnormally shaped S2 Moderate soft tissue or muscle deficiency
Subdivision A and B are based on relative positions of the condyle S3  Serve soft tissue or muscle deficiency
and temporomandibular joint (TMJ)
   2A Glenoid fossa in anatomically acceptable position
   2B TMJ inferiorly, medially, and anteriorly displaced, with
severely hypoplastic condyle
M3 Complete absence of ramus, glenoid fossa, and TMJ
Ear
E0 Normal ear
E1 Mincr hypoplasia and cupping with all structures present
E2 Absence of external auditory canal with variable hypoplasia of
concha
E3 Malpositioned lobule with absent auricle, lobular remnant usually
inferior anteriorly displaced
* OMENS indicates the following: O, orbital asymmetry; M, mandible hypoplasia; E, auricular deformity; N, facial nerve
involvement; and S, soft tissue deficiency (table adapted from vento et al11).
†  Other involved nerves were analyzed, ie, the trigeminal nerve and hypoglossal nerve.

16. Answer: A (Ref. Shafer’s Textbook of Oral Pathology, 7th edition, page no. 14)
Malformation syndromes associated with hemihyperplasia
• Beckwith–Wiedemann syndrome
• Neurofibromatosis
• Klippel–Trenaunay–Weber syndrome
• Proteus syndrome
• McCune–Albright syndrome
• Epidermal nevus syndrome

https://t.me/DentalBooksWorld
342 Triumph's Complete Review of Dentistry

• Triploid/diploid mixoploidy
• Langer–Giedion syndrome
• Multiple exostoses syndrome
• Maffucci’s syndrome
• Ollier syndrome
• Segmental odontomaxillary dysplasia
• Parry–Romberg is hemiatrophy.
17. Answer: C
18. Answer: B
19. Answer: B
• Gross examination of the tooth showed two fully erupted crowns in that one crown was of the normal mandibular
third molar and the other was a supernumerary tooth fused distally (a) and (b). The supernumerary tooth had the
morphology similar to the mandibular second premolar with two cusps placed lingually and buccally.
20. Answer: A
21. Answer: C
22. Answer: C
(Ref. Shafer’s Textbook of Oral Pathology, 2014, page no. 14)
• Hemifacial hyperplasia is a rare developmental anomaly exhibiting asymmetric growth of one or more body parts.
It is characterized by hyperplasia of tissues rather than a hypertrophy, so it is better to call it hyperplasia rather than
hypertrophy. Hemifacial hyperplasia may be associated with deformities of the skeletal system, including macrodactyly,
polydactyly, syndactyly, scoliosis, tilting of the pelvis, and clubfoot.
• The differential diagnosis of condylar hyperplasia should be considered (which is an uncommon malformation of the
mandible caused by excessive growth of condyle). It is difficult to differentiate between hemifacial hyperplasia and
condylar hyperplasia clinically, but it can be easily distinguished radiographically owing to the involvement of soft
tissue and other bones in hemifacial hyperplasia.
• Panoramic radiography displayed hyperplasia (increase in size) of maxilla and mandible including condylar process,
ramus, and body on the left side. The size of the mandibular canal on the left side was also increased. To better define
the facial deformity, CT scan was performed which showed the left maxilla, mandible, and sphenoid to be enlarged
including the palatal and zygomatic bones. An increased size of maxillary sinus was also realized on the affected side.
23. Answer: C
24. Answer: C
• X-inactivation (also called lyonization) is a process by which one of the copies of the X chromosome present in
female mammals is inactivated. The inactive X chromosome is silenced by it being packaged in such a way that it has a
transcriptionally inactive structure called heterochromatin.
  X Chromosome Inactivation (Lyonization)
• During fertilization, both X chromosomes are active. Very soon, however, one of the X chromosomes in a cell,
apparently taken at random, is inactivated and forms a Barr body. All other cells derived from the initial cell have the
SAME X chromosome inactivated. Genes on the inactive X chromosome are not expressed in humans, though, a few
genes are expressed.
• Lyonization theory states that in the somatic cells of female mammals, one of the two X chromosomes is randomly
inactivated early in development. Therefore, females who are heterozygous for a given X-linked gene will be mosaic
with varying proportions of cells in which only one of a particular pair of alleles is active. This mosaicism produced by
lyonization ensures considerable phenotypic variability in the clinical expression of X-linked disorders.
25. Answer: C
26. Answer: D
27. Answer: B
28. Answer: A
29. Answer: D

ODONTOGENIC CYSTS AND TUMORS


1. Answer: A
2. Answer: A
3. Answer: D (Ref. Shafer’s Textbook of Oral Pathology, 4th edition, page no. 284)

https://t.me/DentalBooksWorld
Chapter 4 • Oral Pathology and Oral Medicine 343

4. Answer: A
5. Answer: A
6. Answer: C
7. Answer: A
8. Answer: D
Classification by tissue of origin
Derived from rests of Malassez
• Periapical cyst
• Residual cyst
Derived from reduced enamel epithelium
• Dentigerous cyst
• Eruption cyst
Derived from dental lamina (rests of Serres)
• Odontogenic keratocyst
• Gingival cyst of newborn
• Gingival cyst of adult
• Lateral periodontal cyst
• Glandular odontogenic cyst
Unclassified
• Paradental cyst
• Calcifying odontogenic cyst
9. Answer: B
10. Answer: C (Ref. Shafer’s Textbook of Oral Pathology, 7th edition, page no. 270)
• Histologically, the lateral periodontal cyst is a distinct type of developmental cyst characterized by a thin, nonkeratinized
epithelium usually one to five cell layers thick, which resembles the reduced enamel epithelium. Cuboidal or even
columnar cells may be found composing the lining. Many of the lining cells have a clear, vacuolated, glycogen-rich
cytoplasm. This lining is incomplete and easily sloughs away. Focal thickened plaques of proliferating lining cells often
project into the lumen in areas. These are especially prominent in the botryoid odontogenic cyst.
11. Answer: A
12. Answer: A
• Ameloblastoma is defined as “usually unicentric, nonfunctional, intermittent in growth, anatomically benign and
clinically persistant” by Robinson.
13. Answer: A
14. Answer: B (Ref. Shafer’s Textbook of Oral Pathology, 2014, page no. 280)
• In the plexiform ameloblastoma, the ameloblast-like tumor cells are arranged in irregular masses, or more frequently, as
a network of interconnecting strands of cells. Each of these masses or strands is bounded by a layer of columnar cells,
and between these layers may be found stellate reticulum-like cells. Sometimes double rows of columnar cells are lined
up back to back. However, the stellate reticulum like tissue is much less prominent in the plexiform type than in the
follicular type of ameloblastoma. Areas of cystic degeneration of stroma are also common.
15. Answer: A (Ref. Shafer’s Textbook of Oral Pathology, 2014, page no. 283)
• The Pindborg tumor is classified as an uncommon, benign, odontogenic neoplasm that is exclusively epithelial in
origin. Some have suggested that the epithelial cells of the Pindborg tumor are reminiscent of the cells in the stratum
intermedium layer of the enamel organ in tooth development. Some hypothesize that the Pindborg tumor arises from
remnants of the primitive dental lamina found in the initial stage of odontogenesis, and these epithelial rests are the
more likely true progenitor cell. The definite etiology of this neoplasm still remains enigmatic.
16. Answer: A
Peripheral AOT – maxillary gingiva
Female to male ratio – 14:1
17. Answer: D (Ref. Shafer’s Textbook of Oral Pathology, 7th edition, page no. 287)
18. Answer: B

https://t.me/DentalBooksWorld
344 Triumph's Complete Review of Dentistry

• There are no radiographic features sufficiently characteristic to suggest the diagnosis of this condition. It presents as a
semicircular or roughly triangular radiolucent area, with or without a sclerotic border, usually in association with the
cervical portion of the tooth root.
19. Answer: A
• Hamarotma: Abnormal tissue at normal site
• Choristoma: Normal tissue at abnormal site
20. Answer: A
• Of all odontomas combined, 67% occurred in the maxilla and 33% in the mandible. The compound odontoma had
a predilection, in this study, for the anterior maxilla (61%), whereas only 34% of complex odontomas occurred here.
In general, complex odontomas had a predilection for the posterior jaws (59%) and lastly the premolar area (7%).
Interestingly, both types of odontomas occurred more frequently on the right side of the jaw than on the left (compound,
62%; complex, 68%).
21. Answer: A
Treatment and Prognosis
The treatment of odontomais surgical removal, and there is no expectancy of recurrence.
22. Answer: C
23. Answer: A
24. Answer: A (Ref. Shafer’s Textbook of Oral Pathology, 7th edition, page no. 296)
• The calcifying odontogenic cyst is often encountered in association with an odontoma which may be identified in
juxtaposition to the proliferative lining epithelium or intermixed with the ghost cells. This has been called “dentinoid.”
When this material is formed in abundance and the lesion is “solid” rather than “cystic,” the lesion may be termed a
“dentinogenic ghost cell tumor.” Dystrophic calcification of the ghost cells may be seen; however, it is one of the less
common and least important of the histologic features.
25. Answer: B
• The radiograph may present a mottled or honeycombed appearance of bone in some cases, while others may appear as
a destructive, expanding radiolucency which sometimes has a multilocular pattern. Displacement of teeth by the tumor
mass is a relatively common finding, but root resorption is less frequent. The tumor is often extensive before being
discovered. Invasion of the antrum occurs frequently in lesions of the maxilla.

BENIGN AND MALIGNANT LESIONS


1. Answer: B (Ref. Shafer’s Textbook of Oral Pathology, 2014, page no. 83)
• The HPV types 6 and 11, commonly associated with squamous papillomas, failed to be demonstrated in oral
malignancies as well as potentially malignant oral lesions
• Associated with malignancies – 16 and 18
• Common Wart or Verruca vulgaris – The associated viruses in verruca are the subtypes HPV-2, HPV-4, and HPV-40
2. Answer: B (Ref. Shafer’s Textbook of Oral Pathology, 2014, page no. 81)
• Clinically it shows pointed or verruciform surface projections, a very narrow stalk, appears white due to considerable
surface keratin, and presents as multiple or clustered individual lesions. It enlarges rapidly to its maximum size, seldom
achieving more than 5 mm in greatest diameter.Verruca vulgaris is contagious and capable of spreading toother
parts of an affected person’s skin or membranes by way of autoinoculation. Lesions that are histologically identical
to the verruca vulgaris of the skin are frequently found on the lips and occasionally intraorally. These are often seen in
patients with verrucae on the hands or fingers, and the oral lesions appear to arise through autoinoculation by finger
sucking or fingernail biting.
3. Answer: C (Ref. Shafer’s Textbook of Oral Pathology, 2014, page no. 83)
Keratoacanthoma
(Self-healing carcinoma, molluscum pseudocarcinomatosum, molluscum sebaceum)
• A lesion which clinically and pathologically resembles squamous cell carcinoma, keratoacanthoma is a relatively
common low-grade malignancy that originates in the pilosebaceous glands. It is considered to be a variant of invasive
squamous cell carcinoma.
4. Answer: C (Ref. Shafer’s Textbook of Oral Pathology, 2014, page no. 86)
• The junctional nevus may appear clinically similar to the intradermal nevus, the distinction being chiefly histologic.
It is extremely important. The compound nevus is a lesion composed of two elements:an intradermal nevus and an
overlying junctional nevus.

https://t.me/DentalBooksWorld
Chapter 4 • Oral Pathology and Oral Medicine 345

• The spindle cell and/or epithelioid cell nevus (Spitz nevus) occurs chiefly in children, only about 15% appearing in
adults, and may appear histologically similar to malignant melanoma in the adult. The blue nevus is a true mesodermal
structure composed of dermal melanocytes which only rarely undergo malignant transformation. It occurs chiefly on
the buttocks, on the dorsum of the feet and hands, on the face, and occasionally on other areas.
5. Answer: A (Ref. Shafer’s Textbook of Oral Pathology, 2014, page no. 84)
• The anatomic distribution of nevi closely follows its histologic type. Almost two-thirds of blue nevi occur in hard
palate. Intramucosal nevi are distributed almost equally between the hard palate and buccal mucosa, with almost 25%
in each location. Approximately 17% of intramucosal nevi are on the gingiva, 12% on the vermilion border of the lip,
and almost 9% on the labial mucosa.
6. Answer: C (Ref. Shafer’s Textbook of Oral Pathology, 2014, page no. 91)
• PVL: First described by Hansen et al., in 1985, PVL continues to be recognized as a particularly aggressive form
of oral idiopathic leukoplakia that has a considerable morbidity and a strong potential for malignant transformation.
Diagnosis is often made late in the protracted course of PVL with the disease in an advanced stage when it is especially
refractory to treatment. The histologic spectrum that is seen in PVL are:
–– Verrucous hyperplasia (VH), a histologically defined lesion
–– Varying degrees of dysplasia
–– Three forms of squamous cell carcinoma: verrucous, conventional, and according to some, papillary squamous cell
carcinoma
• VH: This is a forerunner of verrucous carcinoma and the transition is so consistent that the hyperplasia, once
diagnosed, should be treated like verrucous carcinoma.
7. Answer: A (Ref. Shafer’s Textbook of Oral Pathology, 2014, page no. 89)
• Normal: 1:4–6
• In case of dysplasia: increased to 1:1
8. Answer: B (Ref. Shafer’s Textbook of Oral Pathology, 2009, page no. 89)
• Any P1 lesion is Stage III
OLEP Staging System
• Stage I — L1 P0
• State II — L2 P0
• Stage III — L3 P0 or L1 L2 P1
• Stage IV — L3 P1
9. Answer: D (Ref. Shafer’s Textbook of Oral Pathology, 2014, page no. 95)
• The epithelium shows lack of keratin production and is often atrophic, but it may be hyperplastic. This lack of
keratinization, especially when combined with epithelial thinness, allows the underlying microvasculature to show
through, thereby causing the red color. The underlying connective tissue often demonstrates chronic inflammation.
Differentiation of erythroplakia with malignant change and other early squamous cell carcinomas from benign
inflammatory lesions of the oral mucosa can be enhanced by the use of 1% toluidine blue (tolonium chloride) solution
applied topically with a swab or as an oral rinse. This technique gives excellent results in detecting epithelial dysplasia
with false-negative (underdiagnosis) and false positive (overdiagnosis) rates of well below 10%.
10. Answer: B (Ref. Shafer’s Textbook of Oral Pathology, 2014, page no. 96)
Clinical Aspect:
Palatal changes comprise several components: Keratosis—diffuse whitening of the entire palatal mucosa
• Excrescences—1–3 mm elevated nodules, often with central red spots
• Patches—well-defined, elevated white plaques
• Red areas—well-defined reddening of the palatal mucosa
• Ulcerated areas—crater-like areas covered by fibrin
• Nonpigmented areas—areas of palatal mucosa that are devoid of pigmentation
11. Answer: D (Ref. Shafer’s Textbook of Oral Pathology, 2014, page no. 96)
Palatal erythema
• This lesion is marked by a diffused erythematous hard palate, occasionally extending to the soft palate.

https://t.me/DentalBooksWorld
346 Triumph's Complete Review of Dentistry

Epidemiology of the 69 lesions observed among 7,216 tobacco users: 87% occurred among smokers, especially
bidi smokers.
Clinical Aspects
• This lesion occurs either independently or sometimes with other lesions. About 10% of the lesions were associated
with palatal papillary hyperplasia and 25% with central papillary atrophy of the tongue and bilateral commissural
leukoplakias. This triad of lesions is comparable to the multifocal candidiasis described in western literature.
12. Answer: D (Ref. Shafer’s Textbook of Oral Pathology, 2014, page no. 98)
• Biological Studies on Individuals and Tissues from OSF
Blood chemistry and hematological variations.
• Deficiency of vitamin B12, folate, and iron can affect the integrity of the oral mucosa. Significant hematological
abnormalities have been reported in OSF, including an increased erythrocyte sedimentation rate (ESR), anemia and
eosinophilia, increased gammaglobulin, a decrease in serum iron, and an increase in total iron binding capacity (TIBC).
The percentage saturation of transferrin also decreased and a significant reduction in total serum iron and albumin
was found. The role of iron deficiency anemia as the cause or the effect of the disorder is doubtful. A rise in serum
mucoproteins, mucopolysaccharides, and antistreptolysin titer “O” (measured in Todd’s unit) has also been reported. A
significant depression of the lactate dehydrogenase isoenzyme ratio (LDH IV/LDH II) is reported at the tissue level in
OSF. A significant alteration in the serum copper and zinc ratio is also reported with a reduction in zinc content.
13. Answer: B (Ref. Shafer’s Textbook of Oral Pathology, 2014, page no. 98)
• AgNOR. Silver-binding nucleolar organizer region Proteins (AgNORs) comprise a simple and reproducible cytological
test indicative of the proliferative status of cells, particularly of epithelial and hematopoietic origin. It was found that
the pooled mean AgNOR count in clinically advanced OSF was higher than in moderately advanced cases. Counting of
AgNORs may be useful as a predictor of the biological behavior of OSF.
14. Answer: D (Ref. Shafer’s Textbook of Oral Pathology, 2014, page no. 111)
Precancerous lesions (clinical classification)
• Leukoplakia
• Erythroplakia
• Palatal keratosis associated with reverse smoking
Precancerous lesions (histological classification)
• Squamous epithelial dysplasia
• Squamous cell carcinoma in situ
• Solar keratosis
Benign lesions capable of resembling oral precancerous lesions
• White lesions resembling leukoplakia
• Red lesions resembling erythroplakia
• Focal epithelial hyperplasia
• Reactive and regenerative atypia
Precancerous conditions
• Sideropenic dysphagia
• Lichen planus
• Oral submucous fibrosis
• Syphilis
• Discoid lupus erythematosus
• Xeroderma pigmentosum
• Epidermolysis bullosa
15. Answer: C
• Any N1 lesion is stage 3
16. Answer: B (Ref. Shafer’s Textbook of Oral Pathology, 2014, page no. 144)
• Although immunohistochemical stains are usually not necessary for diagnosis, they are generally performed for
confirmation. Both S-100 and homatropine methylbromide (HMB45) stains are positive in melanoma. The S-100 is
highly sensitive, although not specific, for melanoma, while the HMB45 is highly specific and moderately sensitive
for melanoma. The two stains, in concert, can be useful in diagnosing poorly differentiated melanomas. Vimentin is
positive in most cases. Recently, microphthalmic transcription factor, tyrosinase, and melanoma immunostains have
been used to highlight melanocytes.
https://t.me/DentalBooksWorld
Chapter 4 • Oral Pathology and Oral Medicine 347

17. Answer: D (Ref. Shafer’s Textbook of Oral Pathology, 2014, page no. 133)
Giant Cell Fibroma
• Giant cell fibroma is an oral tumor first described in 1974 by Weathers and Callihan as a distinctive entity. The
distinctive histologic appearance sets it apart from a conventional fibroma.
Clinical Features
• It appears as an asymptomatic sessile or pedunculated nodule that is less than 1 cm in diameter. Often, it has
a bosselated or somewhat papillary surface. Most cases are diagnosed in persons aged 10–30 years, and no gender
predilection exists. The most common site is the mandibular gingiva, followed by the maxillary gingiva, the tongue, and
the palate. The clinical differential diagnoses include squamous papilloma, irritation fibroma, pyogenic granuloma, and
peripheral giant cell granuloma.
Histologic Features
• Microscopically, a giant cell fibroma is an unencapsulated mass of loose fibrous connective tissue that contains
numerous characteristic large, plump, spindle-shaped and stellate fibroblasts, some of which are multinucleated. These
cells are easily observed in the peripheral areas of the lesion, whereas the more central Areas contain typical fusiform
fibroblasts. The surface epithelium is corrugated and atrophic; in contrast to an irritation fibroma, a giant cell fibroma
has thin, elongated rete ridges.
• The origin of stellate and multinucleate cells is not well known. Few studies showed positive immunostaining for
vimentin. This suggests that the stellate and multinucleate cells of GCF have a fibroblast phenotype.
Treatment and Prognosis
Conservative excisional biopsy is curative, and its findings are diagnostic. Recurrence is rare.
18. Answer: A (Ref. Shafer’s Textbook of Oral Pathology, 2014, page no. 133)
• The peripheral ossifying fibroma can occur at any age, although it appears to be somewhat more common in children
and young adults. In a study of 365 cases by Cundiff, 50% of the lesions occurred between the ages of 5 and 25 years
with the peak incidence at 13 years, while the mean age was 29 years. Most reported series of cases show a predilection
for occurrence in females by a ratio ranging from 2:1 to 3:2. In addition, the lesions are approximately equally divided
between the maxilla and the mandible. In the series reported by Cundiff, over 80% of the lesions in both jaws occurred
anterior to the molar area. A series of 185 cases of “peripheral fibroma with calcification” were also reported by Bhaskar
and Jacoway with very similar clinical data.
• The clinical appearance of the lesion is characteristic but not pathognomonic. It is a well-demarcated focal mass of
tissue on the gingiva, with a sessile or pedunculated base. It is of the same color as normal mucosa or slightly reddened.
The surface may be intact or ulcerated. It most commonly appears to originate from an interdental papilla.
19. Answer: A
20. Answer: C
21. Answer: B (Ref. Shafer’s Textbook of Oral Pathology, 2014, page no. 138)
• Typical lesion crosses – midline (that too anterior region), whorled appearance, young age (below 30 years), where as
peripheral giant cell granuloma occurs anterior to molar, mean age – 38–42 years and does not crosses midline often.
Note: Both has female predilection

22. Answer: B (Ref. Shafer’s Textbook of Oral Pathology, 2014, page no. 138)
• Central giant cell granuloma:
• Multinucleated giant cells are prominent throughout the connective tissue, but not necessarily abundant. These
giant cells vary in size from case to case and may contain only a few or several dozen nuclei. In addition, there are
usually numerous foci of old extravasated blood and associated hemosiderin pigment, some of it phagocytized by the
macrophage.
• There is a debate whether the giant cells are of fibroblast origin or from monocytes/macrophages. Recent study by
Itonaga et al., indicate that the giant cells in CGCG of the jaw are osteoclast like and formed from monocyte/macrophage
precursors which differentiate into osteoclasts.
Central giant cell tumor of bone:
Histologic Features
• The basic proliferating cell has round-to-oval or even spindle-shaped nucleus in the field that is diagnostic of a true
giant cell tumor. The nucleus is surrounded by an ill-defined cytoplasmic zone, and discernible intercellular substance
is absent. Mitotic figures can be found, sometimes numerous. Mitotic activity has no prognostic significance. The giant
cells are usually scattered uniformly throughout the lesion. They usually contain 40–60 nuclei. Areas of infarct-like

https://t.me/DentalBooksWorld
348 Triumph's Complete Review of Dentistry

necrosis are common in giant cell tumors. Some tumors are almost completely necrotic. The necrosis is not associated
with an inflammatory response. Small collections of foam cells are common. Grading of giant cell tumors has no
prognostic significance.
23. Answer: C (Ref. Shafer’s Textbook of Oral Pathology, 2014, page no. 145)
• Hemangioma affects as many as 12% of infants in Whites, but it rarely occurs in darker-skinned individuals. Vascular
malformations are also more common in Whites. Hemangiomas are about three times more common in females than
in males. But for venous malformations gender ratio is reported to be 1:1.
• The most commonly affected facial bones are the mandible, the maxilla, and the nasal bones. Intraosseous lesions
affect the mandible more often than the maxilla, with a ratio of 2:1. Involvement of the zygoma is rare. Intramuscular
hemangiomas in the oral region are most commonly seen in the masseter, comprising 5% of all intramuscular
hemangiomas.
24. Answer: D (Ref. Shafer’s Textbook of Oral Pathology, 2014, page no. 149)
• Sturge–Weber syndrome is a rather uncommon congenital condition. It consists of congenital hamartomatous
malformations that may affect the eye, the skin, and the central nervous system at different times, characterized by the
combination of a venous angioma of the leptomeninges over the cerebral cortex with ipsilateral angiomatous lesions of
the face, and sometimes, of the skull, jaws, and oral soft tissues.
• The clinical manifestations of SWS have a common embryological basis. The primary defect is a developmental insult
affecting precursors of tissues that originate in the promesencephalic and mesencephalic neural crest. Then, these
affected precursors give rise to vascular and other tissue malformations in the meninges, the eye, and the dermis.
• The facial cutaneous capillary venous angiomas (or port-wine nevi) are usually the first component of the syndrome to
be observed at birth, and are confined almost exclusively to the skin area supplied by the trigeminal nerve. A second
common feature is the presence of typical intracranial convolutional calcifications discernible in cranial radiographs.
Ocular involvement occurs in some patients, consisting generally of glaucoma, exophthalmos, angioma of the choroid,
or other abnormalities.
• Neurologic manifestations are among the most characteristic features of the disease and consist of convulsive
disorders and spastic hemiplegia with or without mental retardation. These manifestations are directly related to the
leptomeningeal angioma and calcifications, the latter being also related to the vascular disturbance.
25. Answer: A (Ref. Shafer’s Textbook of Oral Pathology, 2014, page no. 174)
• Osteosarcoma is a bone tumor that can occur in any bone. It most commonly occurs in the long bones of the extremities
near metaphyseal growth plates. The most common sites are femur (42%, with 75% of tumors in the distal femur), tibia
(19%, with 80% of tumors in the proximal tibia), and humerus (10%, with 90% of tumors in the proximal humerus).
Other significant locations are the skull or jaw (8%) and pelvis (8%).
• Incidence is slightly higher in males than in females (1.25: 1). Osteosarcoma occurs chiefly in young persons, the
majority between 10 and 25 years with decreasing incidence as the age advances. It is very rare in young children and
the incidence increases steadily with age; a more dramatic increase in adolescence corresponds with the growth spurt.
26. Answer: A (Ref. Shafer’s Textbook of Oral Pathology, 2014, page no. 175)
Histologic Features
Gross tissue of osteoblastic osteosarcoma show white-tan, yellow in color and firm in consistency. The chondroblastic
elements appear as translucent lobules and fibroblastic elements appear as tan colored, with soft, or firm consistency.
Hemorrhage and necrosis are common.
• Osteosarcoma is characterized by the proliferation of both atypical osteoblasts and their less differentiated precursors.
In general, the characteristic feature of osteosarcoma is the presence of osteoid formed by malignant osteoblasts in the
lesion, even at sites distant from bone (e.g., the lung). Stromal cells may be spindle shaped and atypical with irregularly
shaped nuclei.
• A number of distinct histologic types of osteosarcoma exist. The conventional type is the most common in childhood
and adolescence, and has been subdivided on the basis of the predominant features of the cells (i.e., osteoblastic,
chondroblastic, fibroblastic), though the subtypes are clinically indistinguishable.
27. Answer: A (Ref. Shafer’s Textbook of Oral Pathology, 2014, page no. 180)
• The most common chromosomal abnormality associated with NHL is the t(14;18)(q32;q21) translocation that
is found in 85% of follicular lymphomas and 25–30% of intermediate-grade NHLs. This translocation results in the
juxtaposition of the bcl-2 apoptotic inhibitor oncogene at chromosome band 18q21 to the heavy-chain region of the

https://t.me/DentalBooksWorld
Chapter 4 • Oral Pathology and Oral Medicine 349

immunoglobulin (Ig) locus within chromosome band 14q32, resulting in its overexpression. The t(11;14)(q13;q32)
translocation results in over expression of bcl-1 (cyclin-D1/PRAD1), a cell cycle control gene on chromosome band
11q13, and is diagnostic of mantle cell lymphoma.
28. Answer: A

Stage Definition

I Involvement of a single lymph node (LN) region (I) or of a single extranodal organ or site (IE)

II Involvement of two or more LN regions, on the same side of the diaphragm (II) or localized
involvement of an extralymphatic organ or site and one or more LN region on the same side of the
diaphragm (IIE)

III Involvement of LN regions on both sides of the diaphragm (III), which may be accompanied by
involvement of the spleen (III S) or by localized involvement of an extralymphatic organ (III E) or
both (IIISE)

IV Noncontiguous involvement of one or more extralymphatic site with or without LN involvement

Annotation Definition

A No B symptoms

B At least one of the following within the last 6 months:


a. Weight loss > 10%
b. Unexplained persistent or recurrent fever
c. Drenching night sweats

X Bulky disease (> 6 cm in diameter or mass > 1/3 of mediastinal) diameter

E Extension to a single extralymphatic organ adjacent to a known involved site

29. Answer: B (Ref. Shafer’s Textbook of Oral Pathology, 2014, page no. 178)
• The histologic findings in B-cell NHL are varied. The salient features of the most common subtypes are as follows.
• Follicular lymphoma (nodular): Represents 22% of all non-Hodgkin’s lymphomas. At low magnification, a
predominantly nodular growth pattern is observed in lymph nodes. Two principal cell types are observed in varying
proportions: small cells with irregular or cleaved nuclear contours and scant cytoplasm that are referred to as
centrocytes (small cleaved cells) and larger cells with open nuclear chromatin, several nucleoli, and modest amounts of
cytoplasm that are referred to as centroblasts. In most follicular lymphomas, small cleaved cells comprise the majority
of the cellularity. Peripheral blood involvement sufficient to produce lymphocytosis (usually <20,000/dl) is observed
in about 10% of patients. Bone marrow involvement occurs in 65% of patients and characteristically takes the form of
paratrabecular lymphoid aggregates.

BONE PATHOLOGY
1. Answer: C
2. Answer: B (Ref. Shafer’s Textbook of Oral Pathology, 2014, page no. 968)
Leontiasis ossea, also known as leontiasis or lion face, is a rare medical condition, characterized by an overgrowth of the facial
and cranial bones. It is not a disease in itself, but a symptom of other diseases, including Paget’s disease, fibrous dysplasia,
hyperparathyroidism, and renal osteodystrophy. The word “leontiasis” has been used to describe the leonine appearance of
some patients with facial leprosy. Virchow added the word “ossea” to describe the leonine appearance in bilateral.

https://t.me/DentalBooksWorld
350 Triumph's Complete Review of Dentistry

3. Answer: C (Ref. Shafer’s Textbook of Oral Pathology, 2014, page no. 726)
The syndrome is familial and transmitted as an autosomal dominant trait. Several chromosome abnormalities have been
reported to be associated with this syndrome, including rearrangement of the long arm of chromosome 8 (8q22) and the
long arm of chromosome 6. Mutations in the core-binding factor alpha-1 (CBFA1) gene, located on chromosome 6p21,
have been shown to be the cause of cleidocranial dysplasia.
Delayed ossification of the skull, excessively large fontanels, and delayed closing of the sutures are prominent features
of this disorder. The fontanels may remain open until adulthood, but the sutures often close with interposition of wormian
bones. Bossing of the frontal, parietal, and occipital regions gives the skull a large globular shape with a small face. The
characteristic skull abnormalities are sometimes referred to as the Arnold head named after the descendants of a Chinese
who settled in South Africa and changed his name to Arnold.
Cleidocranial dysplasia is characterized by abnormalities of the skull, teeth, jaws, and shoulder girdle as well as by
occasional stunting of the long bones. In the skull the fontanels often remain open or at least exhibit delayed closing, and
for this reason tend to be rather large. The sutures also may remain open and wormian bones are common. The sagittal
suture is characteristically sunken, giving the skull a flat appearance. Frontal, parietal, and occipital bones are prominent
and the paranasal sinuses are underdeveloped and narrow. Based on the cephalic index, the head is brachycephalic, or wide
and short, with the transverse diameter of the skull being increased.
4. Answer: C (Ref. Shafer’s Textbook of Oral Pathology, 2014, page no. 728)
• Down’s syndrome is a genetic disorder caused when abnormal cell division results in extra genetic material from
chromosome 21.
• Down’s syndrome causes a distinct facial appearance, intellectual disability, and developmental delays. It may be
associated with thyroid or heart disease.
• The appearance of a mentally retarded or less in IQ but always a happy face (kind and humble)
5. Answer: A (Ref. Shafer’s Textbook of Oral Pathology, 2014, page no. 798)
• It is characterized by four components: Chondrodysplasia; polydactyly; ectodermal dysplasia affecting the hair, teeth,
and nails; and congenital heart failure
Only one features for MCQ point of view if clinical picture is given:
• The most constant oral finding is a fusion of the middle portion of the upper lip to the maxillary gingival margin
eliminating the normal mucolateral sulcus. Thus, the middle portion of the upper lip appears hypoplastic.
6. Answer: A (Ref. Shafer’s Textbook of Oral Pathology, 2014, page no. 716)
Only two identification features:
• Absence of ears
• Underdeveloped malar bones
• Then coloboma (eye slant – antimongoloid)
Note – In Down’s syndrome it is mongoloid
7. Answer: D (Ref. Shafer’s Textbook of Oral Pathology, 7th edition, page no. 729)
Oral Manifestations
Small mouth with protrusion of the tongue (macroglossia) with difficulty in eating and speaking, scrotal tongue,
hypoplasia of the maxilla, delayed tooth eruption, partial anodontia, enamel hypoplasia, juvenile periodontitis, and cleft
lip or palate (rare) are noticed commonly. Fissuring and thickening of the lips and angular cheilitis are frequent and gets
increased in incidence and severity with age. Cheilitis occurs with greater frequency in children with Down’s syndrome
than in unaffected persons. It is explained by mechanical factors, trauma, actinic influence, atopy, avitaminosis, or low-
grade infections (candidiasis). A fissured tongue (plicated or scrotal) occurs in as many as 80% of children with Down’s
syndrome, but it affects about 5% of the general population. Geographic tongue occurs in 11.3% of patients with Down’s
syndrome. Juvenile periodontitis is a feature of Down’s syndrome, and its incidence among the various age groups parallels
the occurrence of cheilitis, but without significant correlation.
8. Answer: A
The prevalence of Paget’s disease increases with age. Paget’s disease is recognized most commonly after age 50 years and
is rarely diagnosed in people younger than 20 years. By the ninth decade of life, prevalence reaches nearly 10% of the peer
group. The male-to-female ratio is approximately 1:1.The etiology of Paget’s disease is still unknown.
  Evidence exists of a genetic link, as a 7-fold to 10-fold increase in incidence of Paget’s disease was observed in relatives
of patients diagnosed with the condition. The overall pattern of apparent transmission suggests an autosomal dominant
inheritance. Another possible etiology is related to viral infection. Some studies have shown the presence of viral
inclusion particles in Pagetic osteoclasts.

https://t.me/DentalBooksWorld
Chapter 4 • Oral Pathology and Oral Medicine 351

  The serum calcium and serum phosphorus levels are usually within normal limits, even in cases of advanced osteitis
deformans. The serum alkaline phosphatase level may be elevated, however, to extreme limits. Values as high as over 250
Bodansky units have been reported, particularly inpatients in the osteoblastic phase of the disease, when there is rapid
formation of new bone and when there is a polyostotic involvement. In fact, there is no other disease of the bone in which
the serum alkaline phosphatase level may be as high as in Paget’s disease. In the monostotic form of the disease, the alkaline
phosphatase level seldom exceeds 50 Bodansky units.
9. Answer: A
10. Correct Answer: A and B
• Both are answers
• For safer side choose fibrous dysplasia
• Ground glass appearance is also seen in cherubism

BLOOD DISORDERS
1. Answer: A (Ref. Shafer’s Textbook of Oral Pathology, 2014, page no. 645)
• Small and shallow ulcers — resembling aphthous ulcers — occur on the tongue. Characteristically, with the glossitis,
glossodynia, and glossopyrosis, there is gradual atrophy of the papillae of the tongue that eventuate in a smooth or
“bald” tongue which is often referred to as Hunter’s glossitis or Moeller’s glossitis and is similar to the “bald tongue of
Sandwith” seen in pellagra.
2. Answer: D (Ref. Shafer’s Textbook of Oral Pathology, 2014, page no. 762)
• The indirect bilirubin may be elevated because pernicious anemia is a hemolytic disorder associated with increased
turnover of bilirubin. The serum lactic dehydrogenase is usually markedly increased. The serum potassium, cholesterol,
and skeletal alkaline phosphatase often are decreased. Serum antibodies for IF are highly specific.
• Other abnormalities have been described, particularly in advanced cases of anemia, including polychromatophilic cells,
stippled cells, nucleated cells, Howell–Jolly bodies and Cabot’s Rings punctate basophilia. Leukocytes are also often
remarkably reduced in number, but are increased in average size, in number of lobes to the nucleus (becoming the so-called
macropolycytes) and anisopoikilocytosis. Mild to moderate thrombocytopenia is noticed. Coexistent iron deficiency
is common because achlorhydria prevents solubilization of dietary ferric iron from foodstuffs. Striking reticulocyte
response and improvement in hematocrit values after parenteral administration of cobalamin is characteristic.
3. Answer: D (Ref. Shafer’s Textbook of Oral Pathology, 2009, page no. 761)
• The pronounced anemia is of a hypochromic microcytic type, the red cells exhibiting poikilocytosis and anisocytosis.
These cells are extremely pale, but in some instances appear as “target” cells with a condensation of coloring matter
in the center of the cell. The presence of typical safety-pin cells and of normoblasts or nucleated red blood cells in
the circulating blood is also a characteristic feature. The white blood cell count is frequently elevated, often as high as
10,000–25,000 or more per cubic millimeter.
• The skeletal changes in thalassemia are most striking and have been thoroughly described by Caffey. A frequent finding
in rib has been referred to as the rib-within-a-rib appearance and is noted particularly in the middle and anterior
portions of the ribs. The finding consists of a long linear density within or overlapping the medullary space of the rib
and running parallel to its long axis.
Enamel hypoplasia is seen in erythroblastosis fetalis.
4. Answer: C (Ref. Shafer’s Textbook of Oral Pathology, 2009, page no. 774)
• An increase in the white blood cell count is also common, and this is almost invariably a lymphocytosis. In fact,
infectious mononucleosis is defined partly on the basis that the patient has more than a 50% lymphocytosis, of which
10% or more are the “atypical” forms. These “atypical” forms consist ofeither oval, horseshoe-shaped, or indented nuclei
with dense, irregular nuclear chromatin and a basophilic, foamy, or vacuolated cytoplasm.
5. Answer: A (Ref. Shafer’s Textbook of Oral Pathology, 2009, page no. 783)
Glanzmann thrombasthenia is an abnormality in the quality of the platelet membrane glycoprotein GpIIb-IIIa. The genes
of these proteins are located on chromosome 17. The disease is transmitted as autosomal recessive disorder. Normal
platelet count and morphology is observed. Three types are there in which type I is most severe and type III is least severe.
6. Answer: C
• Also known as disease of Kings
7. Answer: B
8. Answer: A
9. Answer: C
10. Answer: A

https://t.me/DentalBooksWorld
352 Triumph's Complete Review of Dentistry

ALLERGIC AND IMMUNOLOGIC DISORDERS


1. Answer: A (Ref. Shafer’s Textbook of Oral Pathology, 2014, page no. 665)
• The work of Barile, Graykowski, and Stanley very strongly implicated a pleomorphic, transitionalL-form of ahemolytic
Streptococcus, Streptococcus sanguinis, as the causative agent of the disease. This organism has been consistently
isolated from the lesions of patients with typical aphthous ulcers, and micro organisms morphologically consistent with
the L-form Streptococcus have been found histologically in the vast majority of aphthous lesions.
2. Answer: C (Ref. Shafer’s Textbook of Oral Pathology, 2009, page no. 666)
• Graykowski and his coworkers found that a Tetracycline mouthwash (250 mg per 5 ml), used four times daily for
5–7 days, produced a good response in nearly 70% of the patients tested, by relieving the pain, reducing the size of the
lesions, and reducing the healing time.
3. Answer: C (Ref. Shafer’s Textbook of Oral Pathology, 2014, page no. 341)
• Recurrent aphthous stomatitis is an unfortunately common disease characterized by the development of painful,
recurring solitary or multiple ulcerations of the oral mucosa. There is similarity between recurrent aphthous stomatitis
and herpes simplex infection, with respect to precipitating factors leading to development of the lesions, certain aspect
of clinical appearance of the lesions, duration of the lesion, chronic recurrence, and general failure of response to any
form of therapy.
4. Answer: D (Ref. Shafer’s Textbook of Oral Pathology, 2014, page no. 666)
• Relationship exists between the occurrence of the menstrual period and the development of the ulcers. Incidence of
aphthae is greatest during the premenstrual period. Dolby has shown that similarity between that ulceration is maximal
in the postovulation period and has related this to blood level of progesterone.
5. Answer: D (Ref. Shafer’s Textbook of Oral Pathology, 2009, page no. 666)
• Wood and his associates have described characteristic changes in the nuclei of the epithelial cells taken by the cytologic
smears from around recurrent aphthous ulcers. These have been referred to as Anitschkow cells and consists of cells
with elongated nuclei containing a linear bar of chromatin with radiating processes of chromatin extending toward the
nuclear membrane.
6. Answer: C (Ref. Shafer’s Textbook of Oral Pathology, 2014, page no. 671)
• Reiter’s syndrome is associated with urethritis, balanitis, conjunctivitis, and mucocutaneous lesions. It is the most
common complication of nonspecific urethritis, and in fact, clinically mimics gonorrhea, although urethral discharge is
negative for Neisseria.
7. Answer: C (Ref. Shafer’s Textbook of Oral Pathology, 2014, page no. 672)
• An intracutaneous test for diagnosis of the sarcoidosis, the Kveim–Siltzbach test, has been devised utilizing a suspension
of human known sarcoidal tissue as the test agent.
8. Answer: C (Ref. Shafer’s Textbook of Oral Pathology, 2014, page no. 672)
• Sarcoid lesion closely resembles proliferative noncaseating nodules of tuberculosis,and the differential diagnosis
is frequently difficult to establish; however, no acid fast organisms can be demonstrated in the tissue sections of
sarcoidosis.
9. Answer: B (Ref. Shafer’s Textbook of Oral Pathology, 2014, page no. 675)
• Quinke, describing this disorder as early as in 1882, related the changes observed to an alteration in vascular
permeability considering the fact that many angioedema patients present with psychological problems; it has been
erroneously referred to as “angioneurotic edema” in the past.
10. Answer: A (Ref. Shafer’s Textbook of Oral Pathology, 2014, page no. 673)
• Drug allergy (drug idiosyncrasy, drug sensitivity, and stomatitis or dermatitis medicamentosa) includes a variety
of sensitivity reactions following exposure to any one of a great many drugs and chemicals but is not related to any
pharmacologic activity or toxicity of these materials. The common allergic reactions to systemic administration of a
drug include skin lesions, arthralgia, fever, lymphadenopathy, and rarely agranulocytosis.

BACTERIAL, VIRAL, AND FUNGAL INFECTIONS


1. Answer: C (Ref. Shafer’s Textbook of Oral Pathology, 2014, page no. 317)
• After the entry of the microorganisms into the body, which is believed to occur usually through the pharynx.
2. Answer: A
3. Answer: C (Ref. Shafer’s Textbook of Oral Pathology, 2014, page no. 324)

https://t.me/DentalBooksWorld
Chapter 4 • Oral Pathology and Oral Medicine 353

• General features of leprosy are hypopigmented patches, partial or total loss of cutaneous sensation in the affected areas.
The thickening of nerves and the presence of acid-fast bacilli in the skin or nasal smear are common. Tuberculoid lesions
are characterized by single or multiple macular, erythematous eruptions, with dermal nerve and peripheral nerve trunk
involvement resulting in the loss of sensation, often accompanied by the loss of sweating of the affected skin.
4. Answer: A (Ref. Shafer’s Textbook of Oral Pathology, 2014, page no. 326)
• Botryomycosis is a chronic granulomatous infection which was recognized over 130 years ago when it was first found to
affect horses. Since that time, approximately more than 50 cases occurring in humans have been reported in the literature,
and the first case involving the oral cavity was reported by Small and Kobernick. There is some confusion as to the
actual causative organism in this disease, although an Actinobacillus has been thought to be the one involved. However,
Actinomycosis is known to occur from a “pure culture” of Actinomycetes. Whether “pure cultures” of Actinobacilli can
produce botryomycosis is not known, but many workers believe that a number of common bacteria such as Staphylococcus,
Streptococcus, Escherichia, Pseudomonas, and probably many others may serve as etiologic agents of the disease.
5. Answer: C (Ref. Shafer’s Textbook of Oral Pathology, 2014, page no. 330)
• The secondary or metastatic stage, usually commencing about 6 weeks after the primary lesion, is characterized
by diffuse eruptions of the skin and mucous membranes. In contrast to the solitary lesion in the primary stage,
lesions of the secondary stage are typically multiple. On the skin, the lesions often appear as macules or papules
which are painless. The oral lesions, called “mucous patches,” are usually multiple, painless, grayish-white plaques
overlying an ulcerated surface. They occur most frequently on the tongue, gingiva, or buccal mucosa. They are
often ovoid or irregular in shape and are surrounded by an erythematous zone. Mucous patches are also highly
infectious, since they contain vast numbers of microorganisms. In the secondary stage the serologic reaction is
always positive.
6. Answer: C (Ref. Shafer’s Textbook of Oral Pathology, 2014, page no. 342)
• Herpetic stomatitis is a common oral disease transmitted by droplet spread or contact with the lesions. It affects children
and young adults. However, it has been suggested by Sheridan and Herrmann that the primary form of the disease is
probably more common in older adults than was once thought. It rarely occurs before the age of 6 months, apparently
because of the presence of circulating antibodies in the infant derived from the mother. The disease occurring in
children is frequently the primary attack and is characterized by the development of fever, irritability, headache, pain
upon swallowing,and regional lymphadenopathy. Within a few days, the mouth becomes painful and the gingiva which
is intensely inflamed appears erythematous and edematous. The lips, tongue, buccal mucosa, palate, pharynx, and
tonsils may also be involved. Shortly, yellowish, fluid-filled vesicles develop. These vesicles rupture and form shallow,
ragged, extremely painful ulcers covered by a gray membrane and surrounded by an erythematous halo.
7. Answer: C (Ref. Shafer’s Textbook of Oral Pathology, 2014, page no. 345)
• Herpangina is a specific viral infection, which was described by Zahorsky in 1920 and later named by him. Studies
by Huebner and coworkers proved that Coxsackie group A viruses are the cause of the disease, with types 1 through
6, 8, 10, 16, and 2, as well as other enteroviruses, being isolated at various times. Infection occurs through ingestion,
direct contact, or through droplet spread and multiple cases in a single household are common.The incubation period
is probably 2–10days. It is most commonly seen in young children; older children and adults are only occasionally
affected. Herpangina is chiefly a summer disease, and many children may actually harbor the virus at this time without
exhibiting clinical manifestations of the disease.
8. Answer: B (Ref. Shafer’s Textbook of Oral Pathology, 2014, page no. 361)
• Molluscumcontagiosum is a disease caused by a virus of the pox group. The lesions, which only occur on the skin or
mucosal surfaces, are often considered to be tumor like in nature because of the typical localized epithelial proliferation
caused by the virus.
• The infection is more common in children and young adults and manifests itself as single, or more frequently, multiple
discrete elevated nodules, usually occurring on the arms and legs, trunk and face, particularly the eyelids. However, it is
now recognized that the disease can be sexually transmitted, and lesions of the genitalia and pubo-abdominal area also
occur with some frequency.
• These lesions are hemispheric in shape, usually about 5 mm in diameter with a central umbilication which may be
keratinized, and are normal or slightly red in color. The disease appears to be spread by autoinoculation, direct contact
with an infected individual, or fomites with a reported incubation period of 14–50 days. Lack of inflammation and
necrosis differentiates these proliferative lesions from other poxvirus lesions. Lesions can occur anywhere in the body,
other than palms and soles, and may be associated with eczematous rash.
• Condyloma acuminatum is an infectious disease caused by a virus which belongs to the same group of human
papillomaviruses (HPV) as those associated with common and plantar warts, flat warts, cervical flat warts,

https://t.me/DentalBooksWorld
354 Triumph's Complete Review of Dentistry

pityriasis-like lesions in patients with epidermodysplasia verruciformis and juvenile laryngeal papillomas. It is one of
the most common sexually transmitted diseases in the world. Incidence among Children and adults is high, but low in
early childhood. It reaches its peak between 12 and 16 years of age and then declines sharply at the age of 20 or above.
Transmission is mainly by close contact with infected persons, autoinoculation, and orogenital sexual practice.
• This transmissible and autoinoculable viral disease presents as soft pink nodules which proliferate and coalesce rapidly
to form diffuse papillomatous clusters of varying size. They occur most frequently on the anogenital skin or other
warm, moist intertriginous areas.
9. Answer: A
Group 1 – Lesions most commonly associated with HIV infection:
• Oral candidiasis
• Oral hairy leukoplakia
• Linear gingival erythema
• Necrotizing gingivitis and periodontitis
• Non-Hodgkin’s lymphoma
Group 2 – Lesions less commonly associated with HIV infection:
• Melanotic hyperpigmentation
• Ulcer not otherwise specific
• Herpes simplex viral infection
• Herpes zoster
• Decreased salivary flow rate
Group 3 – Lesions associated with HIV infection:
• Recurrent aphthous ulcers
• Molluscum contagiosum
• Lichenoid reaction
• Facial palsy
• Erythma multiforme
10. Answer: C
• The evidence of HIV infection was first documented in Chennai in southern India in 1986. The heterosexual route is
the predominant mode of transmission, followed by Intravenous drug use.
11. Answer: A (Ref. Shafer’s Textbook of Oral Pathology, 2014, page no. 547)
• The organisms appear as large, nonseptate hyphae with branching at obtuse angles. Round or ovoid sporangia are also
frequently seen in the tissue section. The organisms can be cultured. Histopathologically, mucormycosis should be
differentiated from aspergillosis in which the former has an acute angulating branched hyphae of smaller width and
latter has a septate branched hyphae.
Sporotrichosis:
• The fungus is a small, ovoid branching organism with septate hyphae, showing budding forms. It is only 3–5 mm in
diameter and because of the small size, is seldom recognized in the routine tissue sections. However, it can be cultured
on Sabouraud’s medium.
12. Answer: C
13. Answer: C
14. Answer: D (Ref. Shafer’s Textbook of Oral Pathology, 2014, page no. 345)
• Herpangina is a specific viral infection, described by Zahorsky. Studies by Huebner and coworkers proved that Coxsackie
group A viruses are the cause of the disease, with type 1 through 6,8,10,16, and 2 as well as other enteroviruses, being
isolated at various times. It is chiefly a summer disease. Clinical manifestations of herpangina are comparatively mild
and of short duration.
• It begins with a sore throat, cough, rhinorrhea, low-grade fever, headache, sometimes vomiting, prostration, and
abdominal pain. Vesicles preceding ulcers are small and of short duration. The ulcers do not tend to be extremely
painful although dysphagia may occur.
15. Answer: B (Ref. Shafer’s Textbook of Oral Pathology, 2014, page no. 348)
• Measles is an acute, contagious, dermatropic viral infection, primarily affecting children and occurring many times in
epidemic form. It is caused by paramyxoviruses belonging to the family paramyxoviridae, which is an RNA virus.
16. Answer: B (Ref. Shafer’s Textbook of Oral Pathology, 2014, page no. 361)

https://t.me/DentalBooksWorld
Chapter 4 • Oral Pathology and Oral Medicine 355

• Molluscum contagiosum is a disease caused by a virus of the pox group. The lesions, which only occur on the skin
or mucosal surfaces, are often considered tumor like in nature because of the typical localized epithelial proliferation
caused by the viruses. The virus replicate in the stratum spinosum and forms inclusion bodies which are characteristic
and pathognomonic of poxvirus infection Cowdry type A inclusion bodies.
17. Answer: C (Ref. Shafer’s Textbook of Oral Pathology, 2014, page no. 356)
• The evidence of HIV infection was first documented in Chennai in Southern India in 1986. The heterosexual route is
the predominate mode of transmission, followed by intravenous drug use. India’s prevalence estimates are based on
sentinel surveillance conducted at a public site, according to the National AIDS Control Organization (NACO).
18. Answer: D (Ref. Shafer’s Textbook of Oral Pathology, 2014, page no. 368)
• Microscopic features of North American blastomycosis are similar to those of chronic granulomatous infections. The
inflamed connective tissue shows occasional giant cells and macrophage. Microabscesses are frequently found. If the
lesion are not ulcerated, overlying pseudoepitheliomatous hyperplasia may be prominent.
19. Answer: C (Ref. Shafer’s Textbook of Oral Pathology, 2014, page no. 374)
• Chronic atrophic candidiasis is also known as denture stomatitis, denture sore mouth, a diffuse erythema, and edema
of the denture bearing area.
• Chronic hyperplastic candidiasis: This is often spoken as the leukoplakia type of candidiasis because of the definite
relationship of the chronic candidiasis with true leukoplakia. Some cases are associated with folate deficiency and
defective cell-mediated immunity.
• Erythematous candidiasis is also known as antibiotic sore mouth.
• Pseudo membranous candidiasis is especially prone to occur in debilitated or the chronically ill patients or in infants.
20. Answer: B (Ref. Shafer’s Textbook of Oral Pathology, 2014, page no. 378)
• Rhinosporidiosis is a chronic granulomatous disease caused by a fungus called Rhinosporidium seeberi. Nasal mucosa is
the most common site involved whereas the most frequent intraoral site involved is the soft palate.

ORAL MANIFESTATIONS OF METABOLIC DISEASE


1. Answer: A
2. Answer: B (Ref. Shafer’s Textbook of Oral Pathology, 7th edition, page no. 632)
Lipoid Proteinosis
(Hyalinosis cutis et mucosae, Urbach–Wiethe disease)
• Lipoid proteinosis is a rare, autosomal recessive disorder typified by generalized thickening of skin, mucosae, and
certain viscera.
• Classical features include beaded eyelid papules and laryngeal infiltration leading to the hoarseness of the voice. One
of the characteristic features of the disease is the inability of infants to cry at birth as the hoarseness of the voice is
also present from birth. These features are due to the yellowish-white plaques in the epiglottis, aryepiglottic folds, and
interarytenoid region.
• The exact pathogenesis of this disease is not known but has been postulated to be the result of either a lysosomal storage
disorder involving multiple enzyme defects or from a disturbance in collagen synthesis, as evidenced bya decrease in the
ratio of type I to type III collagen associated with a decrease in mRNA for type I procollagen. There is also an increase in
mRNA for type IV procollagen resulting in underproduction of fibrous collagens and an overproduction of basement
membrane collagens, which tend to deposit in the skin and various organs, which form the hallmark of the disease.
• The oral cavity is usually severely affected in this disease with much of the oral mucous membrane developing the
characteristic yellowish-white papular plaques which become increasingly more prevalent and prominent from
childhood into adult life. The lips become thickened and nodular while the tongue becomes thickened, enlarged, very
firm on palpation, and sometimes bound to the floor of the mouth.
• Recurrent painful parotitis may occur as a result of involvement of the buccal mucosa, with stenosis of the parotid duct
opening. Congenital absence of teeth and severe enamel hypoplasia have also been reported.
• Biochemically, this material is characterized by a decrease in type I collagen with overproduction of type IV or
basement-membrane collagen. The hyaline deposits in the biopsies examined consist of a carbohydrate–protein
complex containing hyaluronic acid and probably chondroitin sulfate, plus large amounts of lipids.
3. Answer: C (Ref. Shafer’s Textbook of Oral Pathology, 7th edition, page no. 640)
• Hypophosphatasia, a hereditary disease first recognized as an entity by Rathbun in 1948, is transmitted as a recessive
autosomal characteristic. Since then many cases have been reported and several reviews of the disease presented.

https://t.me/DentalBooksWorld
356 Triumph's Complete Review of Dentistry

The basic disorder is a deficiency of the enzyme alkaline phosphatase in serum or tissues and excretion of

phosphoethanolamine in the urine. The severity of the disease is not directly related to serum alkaline phosphatase
levels. There is an interesting similarity of many aspects of this disease to the condition known as “vitamin D resistant
rickets with familial hypophosphatemia.”
• On the basis of clinical manifestations and chronology of the appearance of bone disease, hypophosphatasia is
divided into three clinical forms: infantile, childhood, and adult. The infantile form is manifested by severe rickets,
hypercalcemia, bone abnormalities, and failure to thrive. Most of these cases are lethal. Hypophosphatasia of childhood
is characterized by premature exfoliation of deciduous teeth, increased infection, growth retardation and rachitic-like
deformities, including deformed extremities, costochondral junction enlargement (rachitic rosary), and failure of the
calvarium to calcify.
• The earliest manifestation of the disease may be loosening and premature loss of deciduous teeth, chiefly the incisors.
There are varying reports of gingivitis. The metaphyses of long bones have been described as showing “spotty,” “streaky,”
or “irregular ossification.” Dental radiographs generally reveal hypocalcification of teeth and the presence of large pulp
chambers, as well as alveolar bone loss.
• The long bones characteristically exhibit an increased width of proliferating cartilage with widening of the hypertrophic
cell zone irregularity of cell columns, irregular penetration of the cartilage by marrow with persistence of numerous
cartilage islands in the marrow, and formation of large amounts of osteoid which is inadequately calcified.
• The teeth present a unique appearance characterized by the absence of cementum, presumably as a result of failure of
cementogenesis, so that there is no sound functional attachment of the tooth to bone by a periodontal ligament. This
lack of attachment is thought to account for the early spontaneous exfoliation of the deciduous teeth.
Treatment:
• Therapeutic measures are generally unsuccessful. Vitamin D in high doses has resulted in partial improvement in
some cases, but this may lead to deposition of calcium in many tissues, including the kidney.
4. Answer: A
5. Answer: B
6. Answer: C

PERIAPICAL INFECTIONS
1. Answer: C
2. Answer: B
3. Answer: C
4. Answer: A
• Pulp necrosis is a histological diagnosis and therefore cannot be diagnosed radiographically
• Internal and external resorptions are mainly diagnosed radiographically
• Acute apical abscess manifests radiographically as apical periodontal ligament widening
5. Answer: B
• Pain relieved on cold stuffs indicates irreversible pulpitis.
6. Answer: B (Ref. Shafer’s Textbook of Oral Pathology, 7th edition, page no. 880)
• Chronic hyperplastic pulpitis may persist as such for many months or several days. The conditionis not reversible and
may be treated by extraction of the tooth or by pulp extirpation.
7. Answer: A (Ref. Shafer’s Textbook of Oral Pathology, 7th edition, page no. 483)
• The earliest periapical change in the periodontal ligament appears as a thickening of the ligamentat the root apex. As
proliferation of granulation tissue and concomitant resorption of bone continues, the periapical granuloma appears as a
radiolucent area of variable size seemingly attached to the root apex.
8. Answer: A
9. Answer: D (Ref. Shafer’s Textbook of Oral Pathology, 7th edition, page no. 489)
• The apical periodontal cyst is histologically identical with the periapical granuloma, from which it is actually derived,
except for the presence of the epithelium-linedlumen. The epithelium lining the apical periodontal cyst is usually
stratified squamous in type. The only exception to this is in those rare cases of periapical lesions of maxillary teeth that
involve the maxillary sinus.
• In occasional instances, the cyst may then be lined with a pseudostratified ciliated columnar or respiratory type of
epithelium. The usual squamous epithelium seldom exhibits keratin formation. This lining epithelium varies remarkably

https://t.me/DentalBooksWorld
Chapter 4 • Oral Pathology and Oral Medicine 357

in thickness. In newly formed cysts, the epithelial thickness is uneven and often shows hyperplasia while in established
cysts it is of regular appearance and of fairly even thickness. It may be only a few cells thick, or exceedingly thick with a
great deal of proliferation into the adjacent connective tissue. Actual rete ridge formation sometimes occurs.
10. Answer: C (Ref. Shafer’s Textbook of Oral Pathology, 7th edition, page no. 498)
• This is a distinctive type of chronic osteomyelitis in which there is focal gross thickening of the periosteum, with
peripheral reactive bone formation resulting from mild irritation or infection. It is essentially a periosteal osteosclerosis
analogous to the endosteal sclerosis of chronic focal and diffuse sclerosing osteomyelitis. The synonym “Garrè’s
osteomyelitis” for this lesion is unfortunate as Garrè, in his original publication, neither described the periostitis nor
the classical onion skin appearance in the radiograph produced by the cortical duplication.
• This sclerosing osteomyelitis occurs almost entirely in young persons before the age of 25 years and most frequently
involves the anterior surface of the tibia. The lesion in this location has been recognized for many years by orthopedic
surgeons and pathologists.
• Since there is probably greater opportunity for infection to enter the bone of the maxilla and the mandible than any
other bone of the body, because of the peculiar anatomic arrangement of the teeth situated in and protruding from the
bone, it is surprising that the disease has not been described more frequently as a dental complication.
• Intraoral radiographs will often reveal a carious tooth opposite the hard bony mass. Anocclusal radiograph shows a
focal overgrowth of bone on the outer surface of the cortex, which may be described as duplication of the cortical layer
of bone.This mass of bone is smooth and rather well calcified and may itself show a thin but definite cortical layer.
• Chronic osteomyelitis with a proliferative periostitis is treated endodontically or removal of the carious infected tooth,
with no surgical intervention for the periosteal lesion except for biopsy to confirm the diagnosis.

SALIVARY GLAND
1. Answer: B (Ref. Shafer’s Textbook of Oral Pathology, 2014, page no. 225)
• Pleomorphic adenoma is the most common tumor of the salivary glands. The parotid gland is the most common site
of the pleomorphic adenoma; 90% of a group of nearly 1,900 such tumors have been reported by Eneroth. It may occur
in any of the major glands or in the widely distributed intraoral accessory salivary glands; however, its occurrence in
the sublingual gland is rare. In the parotid this tumor most often presents in the lower pole of the superficial lobe of the
gland; about 10% of the tumors arise in the deeper portions of the gland. Approximately 8% of pleomorphic adenomas
involve the minor salivary glands, the palate is the most common site (60–65%) of minor salivary gland involvement.
2. Answer: B (Ref. Shafer’s Textbook of Oral Pathology, 2014, page no. 225)
Option A – is correct
• Option B – slow growing, soft or rubber consistency, 85% of parotid tumors are benign, do not ulcerate, and no
associated nerve signs (facial palsy is rare)
• Option C – major gland tumors – completely or incompletely capsulated where as minor gland tumors are
unencapsulated
• Option D – The accepted treatment for this tumor is surgical excision. The intraoral lesions can be treated somewhat
more conservatively by extracapsular excision. Since these tumors are radio resistant, the use of radiation therapy is of
little benefit and is therefore contraindicated
3. Answer: A (Ref. Shafer’s Textbook of Oral Pathology, 2014, page no. 229)
Basal cell adenomas can be divided on the basis of their morphologic appearances into four subtypes:
• Solid
• Tubular
• Trabecular
• Membranous
• Solid type. The most common type of basal cell adenoma is the solid variant. The basaloid cells form islands and cords
that have a broad, rounded, lobular pattern. These cells are sharply demarcated from the connective tissue stroma by
basement membrane. This feature contrasts with the melting type of growth characteristic of pleomorphic adenoma.
• Tubular type. This pattern exhibits multiple small, round duct-like structures. These tubules are lined by two distinct
layers of cells, with inner cuboidal ductal cells surrounded by an outer layer of basaloid cells. The tubular variant is
the least common; however, tubule formation either alone or with basal cell masses, can be found in most basal cell
adenomas, at least focally.
• Trabecular type. This subtype has the same cytologic features as the solid type, but the epithelial islands are narrower
and cord like and are interconnected with one another, producing a reticular pattern.

https://t.me/DentalBooksWorld
358 Triumph's Complete Review of Dentistry

• Membranous type. This is a distinct subtype of basal cell adenoma characterized by the presence of abundant, thick,
eosinophilic hyaline layer that surrounds and separates the epithelial islands. Electron microscopy has shown that this
hyaline material is a reduplicated basement membrane.The epithelial islands are arranged in large lobules and appear to
mould to the shape of other lobules to resemble a jigsaw puzzle pattern.
NOTE:
Acinic cell carcinoma:
• Abrams and his associates havedescribed four growth patterns: (1) solid, (2) papillary-cystic, (3) follicular, and (4)
microcystic.
NOTE:
• Adenoid cystic carcinoma: described three growth patterns: (1) cribriform, (2) tubular, and (3) solid
4. Answer: C (Ref. Shafer’s Textbook of Oral Pathology, 2014, page no. 257)
• Bilateral swelling points toward both Sjogren’s syndrome andWarthin’s tumor.
• Coming to macroscopic and microscopic appearance of Warthin’s tumor:
• Macroscopic features. It is a smooth, somewhat soft parotid mass and is well encapsulated when located in the parotid.
The tumor contains variable number of cysts that contain a clear fluid. Areas of focal hemorrhage may also be seen.
• Microscopic features. This tumor is made up of two histologic components: epithelial and lymphoid tissue. As the
name would indicate, the lesion is essentially an adenoma exhibiting cyst formation, with papillary projections into
the cystic spaces and a lymphoid matrix showing germinal centers. The cysts are lined by papillary proliferations of
bilayered oncocytic epithelium. The inner layer cells are tall columnar with finely granular and eosinophilic cytoplasm
due to the presence of mitochondria and slightly hyperchromatic nuclei. The outer layer cells are oncocytic triangular
and occasionally fusiform basaloid.
5. Answer: C
• Salivary gland tumors are seen in females except Warthin’s tumor, sebaceous adenoma, and sialadenoma papilliferum
• Inverted and intraductal papilloma – No gender predilection has been noted
6. Answer: A (Ref. Shafer’s Textbook of Oral Pathology, 2014, page no. 233)
• Inverted Ductal Papilloma. Inverted ductal papilloma was first described by White et al., in 1982. It is a very rare
tumor and has been described only in minor salivary glands of adults. The lower lip is the most frequently involved
site followed by buccal vestibular mucosa. Inverted ductal papillomas appear to arise from the excretory ducts near
the mucosal surface. Clinically, these tumors are seen as submucosal nodules which may have a pit or indentation in
the overlying surface mucosa. These tumors do not show any gender predilection. Histologically, it consists of basaloid
and squamous cells arranged in thick, bulbous papillary proliferations that project into the ductal lumen. The lumen
of the tumor is often narrow and in some tumors communicates with the exterior of the mucosal surface through a
constricted opening.
7. Answer: A
• In AFIP data of salivary gland neoplasms, acinic cell carcinoma is the third most common malignant salivary gland
epithelial neoplasm after mucoepidermoid carcinoma and adenocarcinoma. The AFIP found adenoid cystic carcinoma
to be the fifth most common malignant epithelial tumor of the salivary glands after mucoepidermoid carcinoma;
adenocarcinoma; acinic cell carcinoma; and polymorphous low-grade adenocarcinoma (PLGA). In this data, acinic
cell carcinoma comprised 17% of primary malignant salivary gland tumors or about 6% of all salivary gland neoplasms.
8. Answer: A (Ref. Shafer’s Textbook of Oral Pathology, 2014, page no. 234)
• The acinic cell carcinoma closely resembles the pleomorphic adenoma in gross appearance tending to be encapsulated
and lobulated.
9. Answer: D
• Option A – is correct
• Option B – is correct
• Option C – In addition to palate, intraoral tumors occur on the buccal mucosa, tongue, and retromolar areas. Because
of their tendency to develop cystic areas, these intraoral lesions may bear close clinical resemblance to the mucous
retention phenomenon or mucocele, especially those in the retromolar area.
• Option D (Ref. Shafer’s Textbook of Oral Pathology, 2014, page no. 236)
• Mucoepidermoid carcinomas are graded as lowgrade, intermediate grade, and high grade.
• Low-grade tumors show well-formed glandular structures and prominent mucin-filled cystic spaces, minimal
cellular atypia, and a high proportion of mucous cells.

https://t.me/DentalBooksWorld
Chapter 4 • Oral Pathology and Oral Medicine 359

• Intermediate-grade tumors have solid areas of epidermoid cells or squamous cells with intermediate basaloid cells.
Cyst formation is seen but is less prominent than that observed in low-grade tumors. All cell types are present, but
intermediate cells predominate.
• High-grade tumors consist of cells present as solid nests and cords of intermediate basaloid cells and epidermoid
cells. Prominent nuclear pleomorphism and mitotic activity is noted. Cystic component is usually very less (<20%).
Glandular component is rare although occasionally it may predominate. Necrosis and perineural invasion may be
present.
10. Answer: C (Ref. Shafer’s Textbook of Oral Pathology, 2014, page no. 238)
Dedifferentiation of adenoid cystic carcinoma:
• Dedifferentiated adenoid cystic carcinomas are a recently defined, rare variant of adenoid cystic carcinoma
characterized histologically by two components: conventional low-grade adenoid cystic carcinoma and high-grade
“dedifferentiated” carcinoma. Because of frequent recurrence and metastasis, the clinical course is short, similar to that
of adenoid cystic carcinomas with a predominant solid growth pattern.
11. Answer: C (Ref. Shafer’s Textbook of Oral Pathology, 2014, page no. 244)
Carcinoma in Pleomorphic Adenoma
(Malignant mixed tumor)
• Malignant mixed tumors include three distinct clinicopathologic entities: carcinoma ex pleomorphic adenoma,
carcinosarcoma, and metastasizing mixed tumor.
• Carcinoma ex pleomorphic adenoma (most common) constitutes the vast majority of cases, whereas carcinosarcoma
(true malignant mixed tumor) and metastasizing mixed tumor are extremely rare.
12. Answer: A (Ref. Shafer’s Textbook of Oral Pathology, 2014, page no. 248)
• Sialadenosis is the name given to nonneoplastic, noninflammatory enlargement of salivary glands, particularly the
parotid gland. The enlargement is usually bilateral and may manifest recurrence or pain, or both. The condition is almost
always found in association with systemic disorders; this association forms the basis for classification of sialadenosis.
13. Answer: D
• The microscopic features of NS include coagulative necrosis of glandular acini and squamous metaplasia of its ducts.
• All other options are correct.
14. Answer: C (Ref. Shafer’s Textbook of Oral Pathology, 2014, page no. 250)
• Bertram has reported that 75% of a series of 35 patients with Sjögren’s syndrome had in their sera anti salivary duct
antibody. Similar antibody was found in the sera of 24% of a group of 29 patients with systemic lupus erythematosus,
a documented autoimmune disease. In addition, the sicca complex and Sjogren’s syndrome have been found to be
associated with the HLA system, specifically HLA-DR3 and HLA-B8, which are associated with primary form of
the disease and HLA-DRw52 seen to be associated with both the forms of Sjögren’s syndrome. Cytomegalovirus,
paramyxovirus, and Epstein–Barr virus have all been implicated in the pathogenesis of this condition but have not been
proven conclusively.
15. Answer: C (Ref. Shafer’s Textbook of Oral Pathology, 2014, page no. 250)
• Three types of histologic alterations in the major salivary glands have been described. In one case, there may be intense
lymphocytic infiltration of the gland replacing all acinar structures although the lobular architecture is preserved. In
another, there may be a proliferation of ductal epithelium and myoepithelium to form “epimyoepithelial islands.”
• Both of these histologic changes are identical with those occurring in the benign lymphoepithelial lesion in
Mikulicz’s disease. The third alteration may be simply an atrophy of the glands sequential to the lymphocytic
infiltration.
16. Answer: B
17. Answer: A
• Sialography (Sjogren’s syndrome) – classically shows a “snow storm” or “cherry blossom” appearance due to sialectasia
• Ball in hand – tumor
• Snow storm/cherry blossom/branchless fruit laden pattern – Sjogren’s syndrome
• Leafless pattern – sialadenosis
• Tree in winter – normal parotid gland
• Bush in winter – normal submandibular gland
18. Answer: A
19. Answer: A (Ref. Shafer’s Textbook of Oral Pathology, 2014, page no. 249

https://t.me/DentalBooksWorld
360 Triumph's Complete Review of Dentistry

• Mikulicz’s disease is a chronic condition characterized by abnormal enlargement of salivary and lacrimal glands; tonsil
and other glands in the soft tissue of the face and neck may also be involved. But most of these cases were caused by
tuberculosis, sarcoidosis, or lymphoma; this led to confusion regarding the terminology related to Mikulicz’s disease.
Hence, the proposed term Mikulicz’s disease should be used in case of unknown etiology and Mikulicz’s syndrome
in the cases if enlargement is associated with known disease.
20. Answer: C

SKIN PATHOLOGY
1. Answer: A
2. Answer: A
3. Answer: C
4. Answer: A
5. Answer: B
Nikolsky sign is seen in:
• Pemphigus vulgaris
• Epidermolysis bullosa (except recessive type)
• Staphylococcal Scladed Skin Syndrome (SSSS)
• Toxic Epidermolysis Necrosis (TEN)
6. Answer: B (Ref. Shafer’s Textbook of Oral Pathology, 7th edition, page no. 821)
• Familial white folded dysplasia is a relatively uncommon condition of the oral mucosa described by Cannon in 1935.
The disease appears to follow a hereditary pattern as an autosomal dominant trait but with irregular penetrance and no
definite sex predilection.
7. Answer: C
8. Answer: D (Ref.Shafer’s Textbook of Oral Pathology, 2014, page no. 806)
• X-linked hypohidrotic ED has been mapped in the proximal area of the long arm of band Xq12-q13.1. Decreased
expression of the epidermal growth factor receptor has been proposed playing a causal role in this condition’s
phenotype. The gene ED1 responsible for the disorder has been identified.
• The gene that causes hidrotic ED (Clouston’s syndrome) has been identified to be GJB6, which encodes for connexin-30.
GJB6 has been mapped to the pericentromeric region of chromosome 13q. Mutations of the gene PVRL1, encoding
acell-to-cell adhesion molecule/herpesvirus receptor, have been reported in those with cleft lip/palate ED.
9. Answer: A
• Histology of oral mucosa of pemphigus vulgaris shows acantholysis in the lower spinous cell layers. Basal layer cells
are attached to the connective tissue and suprabasal cleft are seen at the tips of the epithelial rete ridges.
10. Answer: D (Ref. Shafer’s Textbook of Oral Pathology, 7th edition, page no. 844)
• Solar elastosis is a dermatologic disease which is essentially a degenerative condition of skin associated with the general
process of aging which itself may be influenced by hereditary factors including skin coloration or pigmentation or its
absence, and exposure to the elements, especially sunlight and wind.
• Such skin, damaged by prolonged exposure to elements of the weather, has often been termed sailor’s skin or farmer’s
skin. This disturbance seldom occurs on the oral mucous membranes, but does involve the lip with considerable
frequency. Although not confined to elderly patients, it is most common in this age group. The affected skin is wrinkled
and appears dry, atrophic, and flaccid. On the lip there may be mild keratosis and subtle blending of the vermilion with
the skin surface.
• The chief microscopic characteristic is the apparent increase in the amount of elastic connective tissue fibers, a
phenomenon that is best observed by special stains. In routine hematoxylin and eosin-stained sections, the connective
tissue may appear hyalinized, but it stains with hematoxylin rather than with eosin, and this has been termed basophilic
degeneration.

DISEASES OF NERVES AND MUSCLES


1. Answer: D (Ref. Shafer’s Textbook of Oral Pathology, 2014, page no. 855)
• Option A – Sphenopalatine ganglion neuralgia, or periodic migrainous neuralgia is characterized by unilateral
paroxysms of intense pain in the region of the eyes, the maxilla, the ear and mastoid, base of the nose, and beneath the
zygoma.

https://t.me/DentalBooksWorld
Chapter 4 • Oral Pathology and Oral Medicine 361

• Option B – Men are affected more commonly than women (5:1) and the majority of patients experience their first
manifestations of the disease before the age of 40 years.
• Option C – Interestingly, in some patients the onset of the paroxysm occurs at exactly the same time of the day, and for
this reason, the disease has been referred to as alarm clock headache.
• Option D – These paroxysms of pain have a rapid onset, persist for about 15 minutes to several hours, and then
disappear as rapidly as they began. There is no “trigger zone.”
2. Answer: D
Uhthoff ’s phenomenon:
• Transient worsening of function with increased body temperature
• Occurs due to a drop below the saftey threshold for conduction because of physiological changes involving the partially
demyelinated axon
• Seen in multiple sclerosis
3. Answer: D
• Causalgia is the definition (question).

Allodynia Lowered threshold Stimulus and response mode differ

Hyperalgesia Increased response Stimulus and response rate are the same

Hyperpathia Raised threshold Stimulus and response rate may be the same or
Increased response different

4. Answer: C (Ref. Shafer’s Textbook of Oral Pathology, 2014, page no. 865)
• Myotonia is a failure of muscle relaxation after cessation of voluntary contraction. It occurs in three chief forms:
dystrophic, congenital, and acquired myotonia. Though each presents the same basic defect, there are sufficient
differences between the three types to warrant their separation. Paramyotonia is a disorder related to the other
myotonias, but differing from them in several important aspects.
Histologic Features:
• Enlargement of scattered muscle fibers and the presence of a centrally placed muscle nuclei in long rows have
been described as being characteristic of atrophy. True hypertrophy of some fibers is almost invariably found, as
well as isolated fibers which show extreme degenerative changes, including nuclear proliferation, intense basophilic
cytoplasmic staining, and phagocytosis. In advanced muscular atrophy, fibers appear small, and there may be interstitial
fatty infiltration.
5. Answer: C
• Any skeletal muscle may be affected, but those of the trunk and proximal limbs are most frequently involved.
Interestingly, certain muscles tend to escape involvement: the tongue, larynx, diaphragm, and perineal muscles.
Ultimately, entire groups of muscles become transformed into bone with resulting limitation of movement. The
masseter muscle is often involved so that fixation of the jaw occurs. The patient becomes transformed into a rigid
organism sometimes encountered in circuses as the “petrified man.”
6. Answer: C (Ref. Shafer’s Textbook of Oral Pathology, 2014, page no. 863)
Horner’s syndrome is a condition characterized by:
• Miosis, or contraction of the pupil of the eye due to paresis of the dilator of the pupil
• Ptosis, or drooping of the eyelid due to paresis of the smooth muscle elevator of the upper lid
• Anhidrosis and vasodilatation over the face due to interruption of sudomotor and vasomotor control
• Its chief significance lies in the fact that it indicates the presence of a primary disease. The exact features of the syndrome
depend upon the degree of damage of sympathetic pathways to the head and the site of this damage. Thus, lesions in the
brainstem, chiefly tumors or infections, or in the cervical or high thoracic cord occasionally will produce this syndrome.
Preganglionic fibers in the anterior spinal roots to the sympathetic chain in the low cervical and high thoracic area are
rather commonly involved by infection, trauma, or pressure as by aneurysm or tumor to produce Horner’s syndrome.
• This pain, which lacks a trigger zone, is constant and persists for weeks, months, or even years. Atypical facial pain
occurs in the territory of the trigeminal nerve, but the discomfort is not typical of trigeminal neuralgia. It may be
as severe as trigeminal neuralgia, but its pattern and quality are different. The distinction is important for making

https://t.me/DentalBooksWorld
362 Triumph's Complete Review of Dentistry

treatment decisions, because surgery, usually rhizotomy or vascular decompression, is highly effective for trigeminal
neuralgia, whereas surgery is not appropriate for atypical facial pain.
7. Answer: A (Ref. Shafer’s Textbook of Oral Pathology, 2014, page no. 862)
Atypical Facial Pain
(Atypical facial neuralgia, facial causalgia)
• Atypical facial pain constitutes a group of conditions in which there is a vague, deep, poorly localized pain in the
regions supplied by the 5th and 9th cranial nerves and the 2nd and 3rd cervical nerves. The pain is not associated with
trigeminal neuralgia; glossopharyngeal neuralgia; postherpetic neuralgia; or with diseases of the teeth, throat, nose,
sinuses, eyes, or ears. The distribution of this pain is unanatomic, since it involves portions of the sensory supply of two
or more nerves and may cross the midline.
• This pain, which lacks a trigger zone, is constant and persists for weeks, months, or even years. Atypical facial pain
occurs in the territory of the trigeminal nerve, but the discomfort is not typical of trigeminal neuralgia. It may be
as severe as trigeminal neuralgia, but its pattern and quality are different. The distinction is important for making
treatment decisions, because surgery, usually rhizotomy or vascular decompression, is highly effective for trigeminal
neuralgia, whereas surgery is not appropriate for atypical facial pain.
8. Answer: C
9. Answer: B
• Bull neck is seen in Diphtheria
10. Answer: C

FORENSIC ODONTOLOGY
1. Answer: B (Ref. Shafer’s Textbook of Oral Pathology, 2014, page no. 880)
• In cases of incinerated remains, additional challenges are faced—since teeth may be brittle following exposure to
prolonged heat, they need to be reinforced with cyanoacrylate glue prior to examination. According to Griffiths and
Bellamy, access for radiography in incinerated bodies can be obtained by removing the tongue and contents of the floor
of the mouth in a “tunneling” fashion from beneath the chin.
2. Answer: A (Ref. Shafer’s Textbook of Oral Pathology, 2014, page no. 884)
• Tooth prints are the pattern formed by the enamel rod ends at the crown surface of the tooth. Manjunath and coworkers
recorded the enamel rod end pattern using acetate peel technique (a technique used to study the texture and surface
details of rocks and fossils). Based on their recent study that examined 60 subjects and 120 teeth, they have categorized
tooth prints into eight different patterns and demonstrated that no two teeth have a similar pattern and coined the
term ameloglyphics. However, they have raised doubts regarding its forensic value since enamel undergoes regressive
changes and the course taken by the enamel rods vary at different levels of the enamel.
3. Answer: C (Ref. Shafer’s Textbook of Oral Pathology, 2009, page no. 880)
• Shoveling: Shoveling refers to the presence of mesial and distal marginal ridges on the lingual surface of the maxillary
and mandibular anterior teeth. The marginal ridges may be absent, slightly developed, or very prominent. The lingual
fossa is a secondary reflection of marginal ridge development.
• Winging: This is an indirect crown trait. It ischaracterized by the bilateral labial rotation of the distal margins of
maxillary central incisors. The incisal edge of the central incisors, taken together, appears “V” shaped from the occlusal
aspect. Winging was observed in 16% of the Indian population.
4. Answer: C
• Demirjian and coworkers have developed an age estimation method that assesses the mandibular left side teeth. The
method is the most widely used technique for assessing age in children and adolescents, probably due to the detailed
description and radiographic illustrations of tooth developmental stages, as well as its relative simplicity.
5. Answer: D
6. Answer: B (Ref. Shafer’s Textbook of Oral Pathology, 2009, page no. 896)
• Age estimation from pulp-to-tooth area ratio: Cameriere and associates suggested measuring the area of the pulp
chamber/root canal and the tooth area of canines on radiographs and calculate their ratio. This was named the pulp-to-
tooth area ratio. The method is based on the principle of age-related secondary dentin deposition and, as age increases,
the area of the pulp chamber/root canal reduces, which is reflected in the decrease in pulp-to-tooth area ratio.
7. Answer: B (Ref. Shafer’s Textbook of Oral Pathology, 2009, page no. 896)
Lip prints were first classified by Santos into two categories:

https://t.me/DentalBooksWorld
Chapter 4 • Oral Pathology and Oral Medicine 363

Simple wrinkles
• Straight line
• Curved line
• Angled line
• Sine-shaped curve.
Compound wrinkles
• Bifurcated
• Trifurcated
• Anomalous
8. Answer: A (Ref. Shafer’s Textbook of Oral Pathology, 2009, page no. 896)
Lip Prints
• The wrinkles and grooves visible on the lips have been named by Tsuchihashi as “sulci labiorumrubrum.” The imprint
produced by these grooves is termed “lip print,” the examination of which is referred to as “cheiloscopy.” These grooves
are heritable and are supposed to be individualistic. Lip prints, therefore, can constitute material evidence left at a crime
scene, similar to fingerprints.
9. Answer: B
10. Answer: A

MISCELLANEOUS
1. Answer: B
2. Answer: B
3. Answer: D
4. Answer: C
5. Answer: A
6. Answer: C
7. Answer: B
8. Answer: B
9. Answer: B
10. Answer: C

MISCELLANEOUS
1. Answer: B (Ref. Shafer’s Textbook of Oral Pathology, 2014, page no. 85)
Globulomaxillary cyst
• The globulomaxillary cyst has traditionally been described as a fissural cyst found within the bone between the
maxillary lateral incisor and canine teeth. Radiographically, it is a well-defined radiolucency which “frequently” causes
the roots of the adjacent teeth to diverge. While there can be no doubt that cysts do occur in this region and that the
pulps of the adjacent teeth may give positive vitality responses, there is now a considerable body of opinion against
the idea that they are fissural cysts. The evidence against their being fissural cysts is, in fact, more substantial than the
evidence in favor (Shear, 1996).
2. Answer: A (Ref. Shafer’s Textbook of Oral Pathology, 2014, page no. 715)
Cherubism:
• Autosomal dominant fibro-osseous lesion of jaws that stabilizes after growth period, usually leaving some facial
deformity and malocclusion
• It is a nonneoplastic hereditary bone lesion that is histologically similar to central giant cell granuloma, affects the jaws
of children bilaterally and symmetrically producing the so called cherubic look.
Exam Crackers:
• Associated with Noonan’s syndrome Q
• Producing a radiographic appearance known as floating tooth syndrome Q and ground glass appearance Q

https://t.me/DentalBooksWorld
364 Triumph's Complete Review of Dentistry

Points to remember
–– Autosomal dominant
–– High penetrance but variable expressivity
–– Chromosome mapped 4p 16
Clinical features
–– Bilateral involvement of posterior mandible
–– Eyes upturned to heaven (appearance is due to wide rim of exposed sclera noted below iris)
–– Painless lesion
–– Distortion of alveolar ridges
–– Impaired mastication (in severe case)
Radiological features
–– Multilocular expansible radiolucency
–– Floating tooth
3. Answer: B (Ref. Shafer’s Textbook of Oral Pathology, 2014, page no. 280)
• The follicular type will have an outer arrangement of columnar or palisaded ameloblast-like cells and an inner zone of
triangular shaped cells resembling stellate reticulum in the “bell stage.”
• Histopathology will show cells that have the tendency to move the nucleus away from the basement membrane. This
process is referred to as “Reverse Polarization.”
• The central cells sometimes degenerate to form central microcysts. The plexiform type has an epithelium that
proliferates in a “Fish Net Pattern.”
• The plexiform ameloblastoma shows epithelium proliferating in a “cord-like fashion,” hence the name “plexiform.”
There are layers of cells in between the proliferating epithelium with well-formed desmosomal junctions, simulating
spindle cell layers.
4. Answer: A (Ref. Shafer’s Textbook of Oral Pathology, 2014, page no. 40)
• One of the common forms of localized microdontia is that which affects the maxillary lateral incisor, a condition that
has been called the “peg lateral.” Instead of exhibiting parallel or diverging mesial and distal surfaces, the sides converge
or taper together incisally, forming a peg-shaped or cone-shaped crown. The root of such a tooth is frequently shorter.
When maxillary lateral incisors are involved, the teeth often appear “cone” or “peg” shaped and are often designated as
peg laterals. These teeth often give a peculiar facial expression of the patient.
• Microdontia is the condition in which one or more teeth are smaller than normal in size. When all teeth are involved,
it is called generalized microdontia and when only a few teeth are involved it is called localized or focal microdontia.
Types of Microdontia
True Generalized Microdontia
• When all the teeth in both arches are uniformly and measurably smaller than normal, the condition is known as true
generalized microdontia.
• This is an extremely uncommon condition and can be seen in pituitary dwarfism.
• True generalized microdontia can also be associated with other congenital defects like Down’s syndrome and congenital
heart disease.
Relative Generalized Microdontia
• Relative generalized microdontia is the condition in which teeth of normal size may look smaller, if they are placed in
an abnormally large maxilla or mandible.
• In such cases larger size of the jaw give an illusion of microdontia although the teeth are not really small.
Focal Microdontia
• When one or two teeth in the jaw are measurably smaller in size, while rest of the teeth are normal, the condition is
called focal microdontia.
• Maxillary lateral incisors and maxillary third molars are the most frequently involved teeth in focal microdontia.
• When maxillary lateral incisors are involved, the teeth often appear “cone” or “peg” shaped and are often designated as
peg laterals. These teeth often give a peculiar facial expression of the patient.
• These peg laterals carry an autosomal dominant type.
• Microdontia involving only a single tooth is a rather common condition. It affects most often the maxillary lateral
incisor and the third molar. These two teeth are among those that are most often congenitally missing.

https://t.me/DentalBooksWorld
Chapter 4 • Oral Pathology and Oral Medicine 365

• It is of interest to note, however, that other teeth which are often congenitally absent, the maxillary and mandibular
second premolars, seldom exhibit microdontia. Supernumerary teeth, however, are frequently small in size.
5. Answer: B
Syndromes associated with taurodontism
–– Amelogenesis imperfecta, Hypoplastic, type IE
–– Amelogenesis imperfect, Taurodontism, type IV
–– Cranioectodermal
–– Ectodermal dysplasia
–– Hyperphosphatasia oligophrenia, taurodontism
–– Hypophosphatasia
–– Klinefelter
–– Microdontia–taurodontia–dens invaginatus
–– Microcephalic dwarfism, taurodontism
–– Oculodentodigital dysplasia
–– Oral–facial–digital, type II
–– Rapp–Hodgkin
–– Scanty hair–oligodontia–taurodontia
–– Sex chromosomal aberrations (e.g., XXX, XYY)
–– Down
–– “Tricho-dento-osseous, types I, II and III
(Ref. Neville Oral and Maxillofacial Pathology, 2nd edition)
(Ref. Shafer’s Textbook of Oral Pathology, 6th edition, page no. 48)
6. Answer: C
• Natal teeth must be approached individually with sound clinical judgment guiding appropriate therapy. As stated, the
erupted teeth in most cases represent the deciduous dentition and removal should not be performed hastily. If the teeth
are mobile and at risk for aspiration, removal is indicated. If mobility is not a problem and the teeth are stable, they
should be retained. (Neville)
• Such retained remnants may subsequently develop atypical tooth-like structures that require additional treatment, as
reported by Nedly, Stanley, and Cohen. The preferable approach, however, is to leave the tooth in place and to explain
to the parents the desirability of maintaining this tooth in the mouth because of its importance in the growth and
uncomplicated eruption of the adjacent teeth. (Mc Donald, page 183)
• The predeciduous teeth have been described as hornified epithelial structures without roots, occurring on the gingiva
over the crest of the ridge, which may be easily removed. Prematurely erupted true deciduous teeth, of course, are not
to be extracted.
• These predeciduous teeth have been thought to arise either from an accessory bud of the dental lamina ahead of the
deciduous bud or from the bud of an accessory dental lamina.
• However, the concept of predeciduous teeth has been questioned by Spouge and Feasby. They are probably correct
in believing that considering predeciduous teeth as an entity is a misinterpretation and that such structures, present
at birth, undoubtedly represent only the dental lamina cyst of the newborn (Q.V). This cyst does commonly project
above the crest of the ridge, is white in color and is packed with keratin, so that it appears “hornified” and can be easily
removed.
• Spouge and Feasby have pointed out that prematurely erupted teeth are often well formed and normal in all respects
except that they may be somewhat mobile. These teeth should be retained even though nursing difficulties may be
experienced.
7. Answer: B
–– Because most of the striking signs of congenital hemihypertrophy are usually manifested in the orofacial region, Gorlin
and Meskin (1962) suggested them help in the differentiation of this condition from other entities that may simulate
hypertrophy.
–– Among the conditions that closely mimic CHH as suggested by various authors are fibrous dysplasia, dyschondroplasia,
congenital lymphedema, arteriovenous aneurysm, hemangioma, lymphangioma, Klippel–Trenaunay syndrome, and
Von Recklinghausen’s neurofibromatosis, malignant conditions such as osteosarcoma and chondrosarcoma.
–– All these conditions, however, exhibit sufficient clinical differences with CHH and should be distinguished on the basis
of specific radiographs, clinical and laboratory findings.

https://t.me/DentalBooksWorld
366 Triumph's Complete Review of Dentistry

–– To the unfamiliar clinician, hemihypertrophy that is localized to orofacial region can constitute a diagnostic problem.
–– Deformities of the teeth and their related hard tissues in the jaw are key findings for correct diagnosis, particularly in
hemihypertrophy limited to the face.
(Ref. ContempClin Dent, 2011 Jul–Sep; 2(3): 261264)
8. Answer: A
• Epstein’s pearls: These small creamy colored cystic lesions are found linearly along the mid palatine raphe and
are probably derived from the epithelium, entrapped along the line of fusion of the palate during embryogenesis.
(Shafer)
Dental Lamina Cyst (Gingival Cyst) of the Newborn
Definition
• Gingival cysts of the newborn are multiple small, nodular, keratin-filled, cystic lesions seen in the oral cavity of
newborns or very young infants (from birth upto 3 months of age).
• Depending upon their locations in the oral cavity, these cysts are divided into several types.
• Cysts of the dental lamina: These lesions are mostly found along the alveolar ridge and are odontogenic in origin
(arising from the remnants of dental lamina).
• Epstein’s pearls: These small creamy colored cystic lesions are found linearly along the mid palatine raphe and are
probably derived from the epithelium, entrapped along the line of fusion of the palate during embryogenesis.
• Bohn’s nodules: In this case, small cysts are usually found along the junction of the hard and soft palate and on the
buccal and lingual aspects of alveolar ridge. These types of cysts are derived from remnants of the mucous glands.
Clinical Features
• All these types of cysts in the newborn usually appear as multiple, asymptomatic, small discrete, white nodules, which
develop in several parts of the oral cavity.
• Once formed, the dental lamina cysts may discharge the contents by fusion with the overlying alveolar mucosa or they
may undergo spontaneous regression.
• The size of these cysts are very small and do not exceed 2–3 mm in maximum diameter.
• The gingival cysts of newborn involve the maxillary arch more often than mandibular arch.
Histopathology
• Microscopic section exhibits a small keratin-filled cystic cavity, which is lined by a thin and flattened squamous
epithelium.
Treatment
• No treatment is required
(Ref. Shafer’s Textbook of Oral Pathology, 6th edition, page no. 48)
9. Answer: A
• It originates due to cystic degeneration and liquefaction of stellate reticulum in an enamel organ before calcification.
(Shafer’s)
• The term “primordial cyst” was first used by Robinson (1945) to describe a cyst of the jaw that he suggested was derived
from the enamel organ in its early stages of development by degenerationof the stellate reticulum before any calcified
structures has been laid down.
• He stated that primordial cysts may occur in single or multiple forms arising either from an enamel organ of a single
tooth of the regular series or from numerous aberrant dental anlage which become cystic.
• Primordial cyst is relatively quite uncommon.
• It originates due to cystic degeneration and liquefaction of stellate reticulum in an enamel organ before calcification.
• Thus, primordial cyst occurs in the place of a tooth and is not associated with any tooth.
• Sometimes, it can also occur in the place of a supernumerary tooth. This results in normal complement of teeth.
• It must be emphasized that if a cystic lesion is present in the region of an edentulous area, it is more appropriate to call
it as residual cyst rather than primordial cyst. Most of the residual cyst are periapical cysts or dentigerous cysts.
(Ref. Shafer’s Textbook of Oral Pathology, 6th edition, page no. 64)
10. Answer: A
• Dens-in-dente refers to a folding or invagination of the surface of the tooth toward pulp; which begins before the
calcification of the tooth and eventually after calcification; the defect produces a typical appearance of a “tooth within a
tooth.” (Shafer’s)

https://t.me/DentalBooksWorld
Chapter 4 • Oral Pathology and Oral Medicine 367

Definition
• Dens-in-dente refers to a folding or invagination of the surface of the tooth toward pulp, which begins before the
calcification of the tooth and eventually after calcification the defect produces a typical appearance of a “tooth within a
tooth.”
• The defect in generally localized to a single tooth and interestingly maxillary lateral incisors are more often affected
than any other tooth in the dental arch.
• Bilateral involvement (of the same tooth on either side of jaw) is often seen and sometimes the defect can involve
multiple teeth including the supernumeraries.
Types
• Dens-in-dente is often broadly divided into two types- coronal type and radicular type.
• Coronal type: Coronal type of dens-in-dente occurs when the invagination or folding occurs on the crown portion of
the tooth. The coronal type is further divided into three subtypes, which are as follows.
• Type I – The invagination within the crown of the tooth.
• Type II – The invagination extends below the cementoenamel junction (CEJ) of the tooth but it may or may not
communicate with the pulp.
• Type III – The invagination extends through the root and perforates in the apical or lateral radicular area. Radicular
type: In case of dens-in-dente if the invagination occurs in the root portion of the tooth, it is called radicular type and
the condition presumably occurs due to folding of the Hertwig’s sheath during the development of root.
Clinical Forms of Dens-in-Dente
• Depending upon the extent or depth of the invagination toward the pulp, the dens-in-dente presents several clinical
forms and these are mostly determined by radiographs.
• Mild form: This form of dens-in-dente is characterized by the presence of a deeply invaginated or accentuated lingual
pit area. Such external pits can be clinically inconspicuous but are clearly visible with the periapical radiographs.
• Intermediate form: Intermediate form of dens-in-dente radiographically reveals a small, pear-shaped invagination of
enamel and dentine into the pulp chamber; this produces a typical appearance of “tooth within a tooth.”
• Extreme form: In this dens-in-dente, the invagination extends beyond the pulp chamber in the root of the affected
tooth. This condition is sometimes known as “dilated odontomes.”
(Ref. Shafer’s Textbook of Oral Pathology, 6th edition, page no. 46)
11. Answer: C
• Chronic focal sclerosing osteomyelitis or condensing osteitis is a rare nonsuppurative inflammatory condition of bone
characterized by sclerotic bone formation around the root apex of a nonvital tooth.
Pathogenesis
• The condition develops as a result of chronic persistent inflammation in the bone, where resistance of the tissue against
infection is very high or where the virulence of the infective organisms is low.
• A low-grade inflammation in the jaw bone causes stimulation of the osteoblast cells, which results in the formation of
dense trabecular bone in the area and this process is known as osteosclerosis.
• Osteosclerosis with additional bone formation may sometimes result in decreased marrow spaces.
Etiology
• The disease often occurs in young individuals, having low-grade sustained inflammation in the bone. The common
conditions, which can precipitate chronic focal sclerosing osteomyelitis, include the following:
• Chronic pulpitis
• Traumatic malocclusion
Clinical Features
• The disease frequently develops in children or young adults before the age of 20 years.
• Mandibular first molars are mostly involved with this condition.
• The condition is mostly asymptomatic and there is no bony expansion seen.
• Majority of the lesions are discovered incidentally during routine radiographic examination of the jawbone.
• The associated tooth is nonvital and usually presents a large carious lesion; it is mostly asymptomatic or is associated
with occasional mild pain.
Radiological Features
The lesion radiologically presents the following features:
• Well-circumscribed radiopaque mass with uniform radiodensity; seen around the root apex of a nonvital tooth.

https://t.me/DentalBooksWorld
368 Triumph's Complete Review of Dentistry

• There is no radiolucent border around the lesion as may be seen in cemento-osseous dysplasia.
• The affected tooth exhibits an apical inflammatory process with widening of periodontal ligament space.
• A residual area of condensing osteitis that is seen after resolution of the inflammatory focus is known as “bone-scar.”
Histological Features
• There is usually the presence of a dense mass of sclerotic bone in the lesion with little or no interstitial marrow tissue.
• Wherever the bone marrow is present it is usually fibrotic and is often infiltrated by chronic inflammatory cells.
Treatment
• The affected tooth should be treated endodontically or it should be removed
• No treatment required for the bony lesion
• Biopsy may be necessary to rule out metastatic malignancy
(Ref. Shafer’s Textbook of Oral Pathology, 6th edition, page no. 497)
12. Answer: C
–– Osteomyelitis sclerosansGarre is still not completely understood. Inspite of the fact that it is caused by bacterial
infection, in most cases no bacterial growth can be discovered in the culture.
–– Furthermore, it remains open whether the chronic process is maintained by low-grade persistent infection or is
maintained by itself after sterilization of the infection. Newer studies that may prove the presence of bacteria, e.g., by
PCR, may elucidate the causes in the future.
–– Sclerosing bone disease, sometimes multifocal, which present the same clinical, radiographic, and histological features
as chronic sclerosing osteomyelitis may be associated with diseases such as palmoplantar pustulosis, colitis ulcerosa,
Crohn’s disease, etc. and are reported as SAPHO (Synovitis Acne Pustulosis Hyperostosis Osteitis) syndrome. In these
diseases bacterial growth is almost always negative. However, temporary improvement under antibiotic therapy may
be observed.
–– A relationship between typical isolated chronic sclerosing osteomyelitis and SAPHO syndrome may exist, but the
diseases should at present be differentiated. Neoplasms such as osteoid osteoma, Ewing’s sarcoma, or eosinophilic
granuloma may simulate primary sclerosing osteomyelitis and pathological examination of peripherally taken
specimens may appear to be compatible with the diagnosis of sclerosing osteomyelitis. It should be appreciated that
malignant tumors– even though this is the exception – can present a chronic, mild picture.
–– Treatment of osteomyelitis sclerosansGarre needs to be determined on an individual basis. A conservative approach
with antibiotics usually leads only to temporary pain relief. The biopsy with opening of the medullary canal is
sometimes sufficient for healing, while in extreme situations only segmental or even complete resection of the diseased
bone case result in a permanent cure.
• Syndrome-associated Primary Chronic Osteomyelitis (SAPHO syndrome)
• The term SAPHO syndrome describes a chronic disorder that involves the skin, bones, and joints. SAPHO
is an acronym that stands for morbid alteration of the dermato skeletal system: synovitis, acne, and pustulosis;
hyperostosis; and osteitis. The clinical picture is determined by chronic inflammation of one tissue or a combination
of any of these tissues. According to Kahb et al. (1994) three diagnostic criteria characterize SAPHO syndrome:
–– Multifactorial osteomyelitis with or without skin manifestations
–– Sterile acute or chronic joint inflammation associated with either pustular psoriasis or palmoplantar pustulosis,
acne, or hidradenitis
–– Sterile osteitis in the presence of one of the skin manifestations
(Ref. Osteomyelitis of the Jaws, By Marc M. Baltensperger, page no. 10)
13. Answer: A
Etiopathogenesis
• The molecular defect occurs in laminin 5 which is present in anchoring filaments associated with hemidesmosomes.
There is mutation in the gene encoding the protein, e.g., defects in two subunits of Laminin 5 involve LAMA3 gene. The
mutations in the genetic codes for type XVII collagen and bullous pemphigoid antigen have also been observed.
• The most severe subtype of JEB is Herlitz syndrome which appears at or soon after birth as extensive blisters and
erosions. Skin is very fragile and separates with insignificant mechanical trauma.
• Oral mucosa is involved and teeth exhibit a highly dysplastic enamel defect that gives teeth a pitted or cobble stone
appearance. Infants with this variant do not survive beyond 2 years of age.
• The other oral manifestations may include anodontia, neonatal teeth, and dental caries.
(Ref. Clinical Dermatology - Epidermolysis Bullosa. Vol. 2, By Fine JD, Wright JT, 1995, page no. 135)

https://t.me/DentalBooksWorld
Chapter 4 • Oral Pathology and Oral Medicine 369

14. Answer: B
• Anomalies in the enamel structure may arise during enamel matrix formation or its resorption and subsequent
calcification.
• Hypoplasia is a quantitative developmental defect caused by failure of matrix production or insufficient deposition of
proteins on the outside of the developing enamel surface, whereby the normally smooth enamel surface becomes pitted
or lacks in substance in large parts – it may be very thin or totally absent.
• Hypocalcification is a qualitative developmental deficiency that arises due to interruption of the resorption of the
organic enamel matrix or a deficiency in the active calcium transport through the ameloblasts and/or failure of
maturation.
15. Answer: B
Syndromes associated with hyperdontia
• Apert
• Angio-osteohypertrophy
• Cleidocranial dysplasia
• Cleido-metaphyseal dysplasia
• Crouzon
• Curtis
• Down
• Ehlers–Danlos
• Fabry–Anderson
• Fucosidosis
• Gardner
• Hallermann–Streiff
• Klippel–Trenaunay–Weber
• Laband
• Nance–Horan
• Oral–facial–digital, types I and III
• Sturge–Weber
• Tricho-rhino-phalangeal
(Ref. Neville, 2nd edition, page no. 70)
16. Answer: D
17. Answer: B (Ref. Shafer’s Textbook of Oral Pathology, 2014, page no. 810)
Lichen planus
Definition
• Lichen planus is a rather common chronic mucocutaneous disease, which probably arises due to an abnormal
immunological reaction and the disease have some tendency to undergo malignant transformation.
Clinical Features
Incidence: Lichen planus is a common skin disease and it occurs in about 1% of the population. The cutaneous lesions
alone occur in about 35% cases, the mucosal lesions alone occur in about 25% cases; however, 40% patients exhibit both
mucosal and cutaneous lesions together. In India, the average incidence rate of lichen planus is about 2.1 per 1,000 men
and 2.5 per 1,000 women.
• Age: Lichen planus occurs among the middle aged or elderly people. Rarely, it can affect children.
• Sex: Both sexes are affected but there is often a slight predilection for females.
• Site: Lichen planus can involve several areas of the body and important among those areas or sites are as follows:
–– Cutaneous lesions: Lichen planus of the skin usually involves (a) flexor surface of the wrist and forearms; (b) inner
aspect of the knee and thigh; (c) upper part of the trunk; (d) scalp, nail beds, and genitalia etc.
–– Oral lesions: Oral lesion of lichen planus commonly occurs on the mucosal surfaces of the buccal mucosa, vestibule,
tongue, lips, and gingival, etc. Palate and floor of the mouth are the least affected sites. In many cases, oral lesions
develop bilaterally.

https://t.me/DentalBooksWorld
370 Triumph's Complete Review of Dentistry

Presentation
Cutaneous lesions of lichen planus:
• The cutaneous lesions of lichen planus clinically appear as clusters or diffuse areas of raised, purplish or reddish papules,
which are covered by a white glistening scale (or a white keratotic “cap”).
• These lesions often occur in a bilaterally symmetrical pattern.
• Lichen planus lesions increase in size, if it is subjected to some irritation.
• As the skin lesions produce an itching sensation, patients often produce linear excoriations, which result in the
development of linear pattern of additional lesions along the scratch marks.
• Koebner phenomenon: It refers to the development of skin lesions of lichen planus, which are extending along the areas
of injury or irritation.
• Cutaneous lesions of lichen planus sometimes exhibit periods of regression and recurrence.
Oral lesions of lichen planus
• The classic form of oral lichen planus clinically exhibits numerous interlacing white keratotic lines, which often produce
a typical “lace-like” or “annular” pattern, against an erythematous base.
• A tiny white elevated dot-like structure is frequently present at the point of intersection of the white lines, which is
known as “striae of Wickham.”
• Oral lesions are generally asymptomatic, although few lesions can cause pain and burning sensation while taking hot
or spicy foods.
18. Answer: A (Ref. Shafer’s Textbook of Oral Pathology, 2014, page no. 50)
Amelogenesis imperfecta
Definition
• Amelogenesis imperfectais a heterogeneous group of hereditary disorders of enamel formation, affecting both
deciduous and the permanent dentition.
• The disease involves only the ectodermal component of the tooth (i.e., enamel) while the mesodermal structures of
tooth (e.g., dentin, cementum, and pulp) always remain normal.
• Levels two clinically distinct forms of autosomal dominant amelogenesis imperfecta – smooth hypoplastic amelogenesis
imperfecta and local hypoplastic amelogenesis imperfect, which are associated with mutations in the enamelin (ENAM)
gene located at 4q21.
• In addition, autosomal dominant amelogenesis imperfecta can be associated with mutation in the Kallikrein-4 (KLK-4)
gene and autosomal recessive pigmented hypomaturation amelogenesis imperfecta with an enamelysin (MMP-20) gene
mutation, illustrating the heterogeneity of the condition. An X-linked form (AIHI) has been found to be associated
with as many as 14 mutations in the amelogenin (AMELX) gene, located at Xp21, 92; however, at least one family has
had the trait linked to another location on the chromosome Xq22-q28.93.
Types
Normally, the process of enamel formation progresses through three stages:
• Stage of enamel matrix formation
• Stage of early mineralization
• Stage of enamel maturation
Amelogenesis imperfecta may set in during any stage of enamel formation. Four basic types of the disease have been
identified, which corresponds with three developmental stages of enamel.
• Type I – Hypoplastic type of amelogenesis imperfecta
• Type II – Hypomaturation type of amelogenesis imperfecta
• Type III –Hypocalcification type of amelogenesis imperfecta
• Type IV – Hypomaturation Hypoplastic type with taurodontism
Hypoplastic Type:
• The enamel thickness is usually far below normal in hypoplastic type of amelogenesis imperfecta since the disease
affects the stage of matrix formation. The teeth exhibit either complete absence of enamel from the crown surface or
there may be a very thin layer of enamel on some focal areas of the crown.
Hypomaturation Type:
• This type occurs due to interruption in the process of maturation of enamel. Here the enamel is of normal thickness but
it does not have the normal hardness and translucency (snow-capped tooth).
• The enamel can be pierced with an explorer tip with firm pressure.

https://t.me/DentalBooksWorld
Chapter 4 • Oral Pathology and Oral Medicine 371

Hypocalcification Type:
• Hypocalcification type of amelogenesis imperfecta represents the disturbance in the process of early mineralization
of the enamel. In this type of amelogenesis imperfecta, the enamel is of normal thickness but is soft and can be easily
removed with a blunt instrument.
Hypomaturation – Hypoplastic Type with Taurodontism
• This is a rare condition where taurodontism is reported in association with amelogenesis imperfecta.
19. Answer: C
20. Answer: C

RANDOM REPEATS
1. Answer: D
• The Melkersson–Rosenthal syndrome is a rare disorder of unknown etiology characterized by a triad of recurrent
orofacial swelling, relapsing facial paralysis, and fissured tongue. Exacerbations and recurrences are common.
The orofacial swelling is characterized by fissured, reddish-brown, swollen, nonpruritic lips or firm edema of the
face. The facial palsy is indistinguishable from Bell’s palsy. The fissured tongue is seen in one-third to one-half of
patients and, although the least common manifestation, its presence assists in diagnosis. The classic triad is not seen
frequently in its complete form; therefore, diagnosis is difficult. This is particularly true because monosymptomatic
and oligosymptomatic variants are seen more commonly. Cheilitis granulomatosa of Miescher is an example of a
monosymptomatic variant of the Melkersson–Rosenthal syndrome.
• Cheilitis glandularis (CG) is a clinically descriptive diagnosis that refers to an uncommon, poorly understood
inflammatory disorder of the lower lip. Its etiology remains obscure. Cheilitis glandularis is characterized by progressive
enlargement and eversion of the lower labial mucosa that results in obliteration of the mucosal–vermilion interface.
With externalization and chronic exposure, the delicate lower labial mucous membrane is secondarily altered by
environmental influences, leading to erosion, ulceration, crusting, and, occasionally, infection.
2. Answer: B
3. Answer: B
4. Answer: D
• Mixed tumor of parotid gland is usually benign (e.g., Pleomorphic adenoma) – so option A is wrong.
• Calculi are not usually seen – so option B is wrong.
• Associated most common with parotid gland – so option C is wrong.
5. Answer: D
• Sjögren’s syndrome is a systemic chronic inflammatory disorder characterized by lymphocytic infiltrates in exocrine
organs. The disorder most often affects women, and the median age of onset is around 50–60 years. Most individuals
with Sjögren’s syndrome present with sicca symptoms, such as xerophthalmia (dry eyes), xerostomia (dry mouth), and
parotid gland enlargement.
6. Answer: C
Diseases that increase risk of xerostomia:
• AIDS
• Systemic lupus erythematosus
• Thyroid dysfunction
• Parkinson’s disease
• Cerebral palsy
• Depression
• Anxiety
• Posttraumatic stress disorder
• Dehydration
• Eaton–Lambert syndrome
• Trauma to salivary glands
• Anorexia and bulimia

https://t.me/DentalBooksWorld
372 Triumph's Complete Review of Dentistry

Common causes of reduced salivary flow:


Medications Opioid
Respiratory inhalers
Antipsychotics
Sedatives/Antianxiety
Antidepressants
Antihistamines
Antihypertensives
Systemic diseases Sjogren’s syndrome
Diabetes mellitus
Rheumatoid arthritis
Systemic lupus erythematosus
Cystic fibrosis
Hormonal imbalance
Radiation therapy
Radioactive iodine therapy
Electrolyte loss Dehydration
Vomiting
Diarrhea
Lifestyle factors Smoking
Excessive abuse intake
Alcohol consumption
7. Answer: B
8. Answer: C
9. Answer: A
10. Answer: B
11. Answer: C
• Duchenne’s muscular dystrophy and Becker’s muscular dystrophy are X-linked recessive disorders characterized
by progressive proximal muscle weakness caused by muscle fiber degeneration. Becker’s dystrophy has a later onset
and causes milder symptoms. Diagnosis is suggested clinically and is confirmed by the analysis of the protein
product (dystrophin) of the mutated gene.
The role of dystrophin in muscle structure and integrity:
• Dystrophin is an important structural protein found in skeletal (striated) muscle which connects the cytoskeleton of a
muscle fiber to the extracellular matrix (ECM). It is a cytoplasmic protein, and links actin filaments to other structural
proteins outside of the cell. Dystrophin is key in maintaining the mechanical stability of skeletal muscle; a lack of it
causes these cells to be increasingly fragile and prone to damage. The result of this damage is a steady leak of calcium
into the sarcolemma, disrupting signaling pathways and ultimately causing mitochondria to burst. Damaged muscle
tissue is progressively replaced by scar tissue and fat.

Membrane

Muscle Muscle fiber

Bundle of
muscle fibers
ECM
Membrane
protein complex Outside the cell

Inside the cell


Dystrophin

Actin filaments

https://t.me/DentalBooksWorld
Chapter 4 • Oral Pathology and Oral Medicine 373

12. Answer: C
13. Answer: A
• Petrified meaning – so frightened that one is unable to move; terrified.
14. Answer: C
15. Answer: B
• Cyclic neutropenia is a form of agranulocytosis characterized by a periodic decrease of neutrophilic leukocytes in the
peripheral blood.
• Cycles have an interval of approximately 21–27 days and occasionally several months.
• An autosomal dominant mode of inheritance is likely.
• Oral manifestations include severe ulcerative gingivitis or stomatitis or both.
• Pain is the constant feature of these lesions.
• Neutropenia usually lasts from 2–5 days.
16. Answer: D
17. Answer: D
18. Answer: A
19. Answer: C
• Normal adult hemoglobin (HbA) consists of a tetramer made up of two alpha-globin and two beta-globin subunits.
The alpha globin gene is found on chromosome 16 and is duplicated, which means that each somatic cell with its pair
of homologous chromosomes contains four copies of the alpha chain gene. The gene encoding beta globin only has two
copies, one present on each of the pair of chromosome 11. 
• Thalassemia results when mutations affecting the genes involved in Hb biosynthesis lead to decreased Hb production.
The clinical phenotype results from both the diminished amount of the particular globin chain as well as from the
resultant chain imbalance that occurs because of normal production of the other globin chain.
• Beta thalassemia
• Genetics/etiology
• Upward of 100 mutations have been described that decrease beta chain synthesis. Most of these are point mutations, and
interfere with processes such as splicing, chain termination, and promoter sites resulting in defective gene transcription
or translation.
• Mutations fall into two classes:
–– B0 refers to mutations that cause no beta globulin to be produced
–– B+ describes mutations that result in a diminished but not absent quantity of beta globulin. The severity of these
mutations can vary depending on the amount of normal beta globulin that is produced
• Depending on the class of mutation present and the gene dosage (i.e., heterozygous or homozygous) patients can
present with differing severity of disease
–– Beta thalassemia major: refers to a severe clinical phenotype that occurs when patients are homozygous or
compound heterozygous for more severe beta chain mutations (e.g., severe B+/B+ mutations, B+/B0, B0/B0)
–– Beta thalassemia intermedia: An in-between clinical phenotype with heterogeneous genetic mutations that still
allow for some beta chain production (e.g., B+/B0, B+/B+). Some rare cases also exist in which both beta and alpha
mutations coexist
–– Beta thalassemia minor/thalassemia trait: a mild clinical phenotype when one normal copy of the beta globulin
gene is present (e.g., B+/B, B0/B)
• Alpha thalassemia
• Genetics/etiology
• Many mutations can affect the alpha globin gene, but the most common are gene deletions
• As mentioned previously, there are four copies of the alpha gene in each somatic cell. Thus, phenotypes increase in
severity as the number of functional alpha genes decreases
–– Silent carrier: refers to patients with one alpha gene deletion, they are clinically asymptomatic (e.g., a-/aa).
–– Alpha thalassemia trait: these patients have two alpha gene deletions, and very mild phenotypes. Gene deletions
can both be present on the same chromosome, or divided between the two chromosomes which has relevance for
the patient’s offspring (e.g., aa/– or a-/a-).
• Hemoglobin H disease: named reflecting the presence of the beta tetramer HbH (B4) found in red cells. Causes
moderately severe anemia. Occurs with three alpha chain deletions (e.g., a-/–).

https://t.me/DentalBooksWorld
374 Triumph's Complete Review of Dentistry

• Hydropsfetalis: the most severe form, caused by four alpha gene deletions (e.g., –/–). Becomes manifest later in fetal
development, when the fetus transitions from using early embryonic globin alleles (gamma2/zappa2) to later fetal alleles
(gamma2/alpha2). Red cells contain gamma-globin tetramers (Hb Bart) which are ineffective at delivering oxygen to
tissues, causing anoxia, edema, hepatosplenomegaly. Historically, was not compatible with life, but aggressive inutero
and lifelong transfusions may save individuals with this condition.
Signs and Symptoms:
1. Shortness of breath, fatigue, and weakness (anemia)
Mechanism:
• Secondary to anemia
• Deficient synthesis of the beta chain of hemoglobin causes red cells have low HbA levels, thus explaining their
phenotype and impaired ability to transport oxygen.
• Imbalance between alpha and beta globin production leads to a precipitation of the relatively overabundant alpha
globin chain within the RBCs their precursors.
2. Hepatomegaly
Mechanism:
• Ineffective erythropoiesis leads to activation of extramedullary erythropoiesis in areas such as the spleen, liver,
lymph nodes, and the thorax.
• Hepatomegaly can result from a number of mechanisms; extramedullary erythropoiesis, hepatitis due to chronic
transfusion associated infections, and iron overload.
3. Splenomegaly
Mechanism:
• Secondary to extramedullary hematopoiesis
• Can also be due to extravascular hemolysis causing a hypertrophic response in the spleen.
4. Bone pain and fragility fractures
Mechanism:
• Caused by two major mechanisms: 
• Destruction of the cortex leading to weakening of the bone 
• Secondary to osteopenia/osteoporosis (mechanism above)
5. Malnutrition
Mechanism:
• Rapidly growing erythrocyte precursors compete for nutrients and can cause malnutrition
20. Answer: C
21. Answer: C
• The tongue is coated centrally, but its edges are smooth and slimy. Papillae are pronounced and often injected.
Sometimes the epithelium of the tongue is shed and we have the “bald” tongue of Sandwith, sometimes called the
“cardinal” tongue.
22. Answer: B
23. Answer: A
24. Answer: B
25. Answer: A
• A bifid uvula is sometimes referred to as the mildest form of a cleft palate. A cleft palate is one of the most common
birth defects and occurs when there is a hole in the roof of a baby’s mouth. Bifid uvula affects 2% of the population.
26. Answer: D
• Median rhomboid glossitis (MRG, also known as central papillary atrophy, or glossal central papillary atrophy) is a
condition characterized by an area of redness and loss of lingual papillae, situated on the dorsum of the tongue in the
midline immediately in front of the circumvallate papillae.
27. Answer: B
28. Answer: A

https://t.me/DentalBooksWorld
Chapter 4 • Oral Pathology and Oral Medicine 375

Dentin Dysplasia Dentin Dysplasia Dentinogenesis Regional odontodysplaia


type 1 type II Imperfecta
Mode of inheritance Autosomal dominant Autosomal Autosomal dominant Non-hereditary
dominant
Dentition affected Primary and Primary teeth- Primary and Primary and permanent
permanent teeth- discolored permanent teeth teech discolored - yellow to
normal Permanent teeth- discolored brown surface
normal
Mobility Present Absent Absent Present
Taurodontism Present Absent Absent Absent
Radiographic Small or no roots Thistle tube shaped Bulbous crowns, Thin enamel and dentin
appearance and obliteration of or flame-shaped pulp cervical constriction, surrounding enlarged
the pulp chambers- chambers thin roots and early radiolucent pulp-ghost teeth
rootless teeth obliteration of pulp appearance
chamber
Histopathologic Central portion Coronal dentin Coronal dentin Follicular tissue contains
features of the root forms - Primary teeth- exhibiting short scattered collections of
whorls of tubular irregular tubules misshapen tubules enameloid conglomerates
dentin giving the Permanent teeth- within atypical and islands of odontogenic
appearance of normal tubules True granular dentin epithelium
"stream flowing denticles, excessive matrix
around boulders." dentin remodelling
29. Answer: D
30. Answer: D

Genetics:
• IRF6
• Ch8q24
• VAX1
• FGFR2
• BMP4
Maternal risk factors:
• Smoking
• Alcoholism
• Pregestational and gestational diabetes
• Age > 40 years
• Folate deficiency
• Zinc deficiency
Teratogens:
• Valproic acid
• Phenytoin
• Retinoic acid
• Chemical solvents
• Pesticides
• Occupation related (leather, shoe making, healthcare)

31. Answer: B
32. Answer: C
33. Answer: A
34. Answer: C
35. Answer: B

https://t.me/DentalBooksWorld
376 Triumph's Complete Review of Dentistry

36. Answer: C
37. Answer: A
38. Answer: C
39. Answer: B
40. Answer: D
41. Answer: B
42. Answer: C
43. Answer: B
44. Answer: A
45. Answer: D
46. Answer: A

PHYCOMYCOSIS (Mucormycosis, zygomycosis):


• Phycomycosis is a fungal infection caused by the order mucorales.
• Zygomycosis refers to the infection caused by both the orders of fungus.
• Mucorales and Entomophthorales. It is worldwide in distribution andthe organisms normally occur in soil, manure
fruits, and in decaying matter. These organisms are present in the nasal passages and oralcavities of normal persons.
• This is an opportunistic infection associated with debilitation and isbecoming more frequently recognized as a secondary
occurrence incancer patients, especially those with any of the malignant lymphomasand in patients having renal failure,
organ transplant, AIDS, and cirrhosis. It is also especially common in patients with diabetes mellitus, especially those
with diabetic ketoacidosis; fully 75% of the patients with the rhinocerebral form of mucormycosis have the ketoacidosis.
• This disease may actually be caused by numerous Phycomycetes organisms of the Eumycetes (true fungi) class
characterized by lack of septation (coenocytic). The three most important types causing infection in man are Rhizopus,
Mucor, and Absidia.
Clinical Features:
• Two main types of phycomycosis infection occur in human beings: (1) superficial and (2) visceral, although it is
sometimes also classified as localized and disseminated. The superficial infection includes involvement of the external
ear, the fingernails, and the skin. The visceral forms of phycomycosis are of three main types: (a) pulmonary, (b)
gastrointestinal, and (c) rhinocerebral.
• One early clinical manifestation of the disease is the appearance of areddish-black nasal turbinate and septum with
a nasal discharge. The necrosis may extend to the paranasal sinuses and orbital cavity, with the development of sinus
tracts and sloughing of tissue.
Histologic Features:
• The tissue involved by this infection shows a variable amount of necrosis, some of which may be related to infarction
brought about by thrombi consisting of the organisms. This fungus has an apparent predilection for blood vessels; it is
able to penetrate their walls and thereby produce thrombosis. The organisms appear as large, nonseptate hyphae with
branching at obtuse angles. Round or ovoid sporangia are also frequently seen in the tissue section. The organisms
can be cultured. Histopathologically mucormycosis should be differentiated from aspergillosis in which the former
has an acute angulating branched hyphae of smaller width and latter has septate branched hyphae. A special stain like
Grocott’s silver methenamine stain may use to confirm the diagnosis.
Treatment and Prognosis:
• Treatment of the disease consists of control of the predisposing factors such as diabetes, surgical excision if the lesion
is localized, and administration of amphotericin B, since it is the only drug with proven efficacy.
47. Answer: C
EEC clearing house classification (2002
GROUP I Lesions strongly associated with HIV infection
a)  Candidiasis – Erythematous, Pseudomembranous
b)  Hairy leukoplakia
c)  Kaposi’s sarcoma
d)  Non-Hodgkin’s lymphoma
e) Periodontal disease linear gingival erythema Necrotizing (ulcerative) gingivitis
Necrotizing (ulcerative) Periodontitis

https://t.me/DentalBooksWorld
Chapter 4 • Oral Pathology and Oral Medicine 377

GROUP II Lesions less commonly associated with HIV infection


a) Bacterial infections
Mycobacterium avium intracellulare
Mycobacterium tuberculosis
b)  Melanotic hyperpigmentation
c)  Necrotizing (ulcerative) stomatitis
d)  Salivary gland disease
 Dry mouth due to decreased salivary flow rate Unilateral or Bilateral swelling of
major salivary glands
e)  Thrombocytopenic purpura
f)  Ulceration NOS (not otherwise specific)
g)  Viral infections
  Herpes simplex virus
  Human papilloma virus (warty-like lesions)
–– Condylomaacuminatum
–– Focal epithelial hyperplasia
–– Verucca vulgaris Varicella
–– Zoster virus
–– Herpes zoster
–– Varicella
GROUP III Lesions seen in HIV infection
a) Bacterial infection
Actinomyces israelii
Escherichia coli
Klebsiella pneumoniae
b)  Cat-scratch disease
c)  Drug reactions (Ulceration, Erythema multiforme, Lichenoid, Toxic epidermolysis)
d) Fungal infections other than candidiasis Cryptococcus neoformans
Geotrichum candidum
Histoplasma capsulatum
Mucoraceae (Mucormycosis/Zygomycosis) Aspergillus flavus
e) Neurologic disturbances
Facial palsy trigeminal neuralgia
f)  Recurrent aphthous stomatitis
g) Viral infections
Cytomegalovirus
Molluscum contagiosum
Revised classification of oral lesions associated with HIV infection in adults.
GROUP I Seven cardinal lesions that are strongly associated withHIV infection
Oral candidosis
Hairy leukoplakia
Kaposi sarcoma
Linear gingival erythema
Necrotizing ulcerative gingivitis
Necrotizing ulcerative periodontitis
Non-Hodgkin lymphoma

https://t.me/DentalBooksWorld
378 Triumph's Complete Review of Dentistry

GROUP II Atypical ulcers


Salivary gland diseases
Viral infection such as cytomegalovírus (CMV), herpes simplex virus (HSV),
papillomavirus (HPV), and herpes zoster virus (HZV).
GROUP III Lesion rarer than those on groups 1 and 2
Diffuse osteomyelitis
Squamous cell carcinoma

48. Answer: B
49. Answer: A
50. Answer: B
51. Answer: C
52. Answer: C
53. Answer: C
54. Answer: B
55. Answer: B
56. Answer: C
57. Answer: D
58. Answer: B
59. Answer: D
60. Answer: C
61. Answer: D
• Cervicofacial emphysema is an infrequently reported sequela to dental surgery. Most cases result from the accidental
introduction of air into the soft tissues during the use of air-driven, high-speed handpieces or air/water syringes.
Surgical procedures, in particular, removal of lower third molars, predispose to the development of an emphysema.
The clinical presentation is usually a facial or cervicofacial swelling coincident with the dental treatment. The use of
air instruments, immediate onset, CREPITUS, and often a radiographically discernible enlarged facial space are the
diagnostic features. Pain is not usually a feature. The possibility of mediastinal involvement should be recognized and
the patient monitored appropriately. Active treatment requirements are minimal. Reassurance of the patient, antibiotic
prophylaxis, and analgesics, if required, are generally sufficient.
62. Answer: B
63. Answer: A
What is C1 esterase and why its deficiency causes angioedema?
• C1 esterase inhibitor (C1-INH) is a protein found in the fluid part of blood. It controls a protein called C1, which is part
of the complement system. This system is a group of proteins that move freely through the bloodstream.

C1 Plasminogen

C1 esterase inhibitor

C1* Plasmin
C2b

C2 C2 Kinin

Angioedema

https://t.me/DentalBooksWorld
Chapter 4 • Oral Pathology and Oral Medicine 379

Complement

(−) C1 esterase inhibitor (negave modulator)

Bradykinin Angiodema

(+) Acetylcholinesterase (ACE) (positive modulater)

Bradykinin breakdown products

64. Answer: B
65. Answer: C
66. Answer: A
67. Answer: B
68. Answer: A
69. Answer: B
70. Answer: A
71. Answer: D
72. Answer: B
73. Answer: D
Osteoid Osteoma:
• The osteoid osteoma is a benign tumor of bone which has seldom been described in the jaws. The true nature of this
lesion is unknown.
Clinical Features:
• The osteoid osteoma usually occurs in young persons, seldom developing after the age of 30 years. Young children
under the age of 10 years or even 5 years are frequently affected. In most series, males predominate over females by a
ratio of at least 2 to 1. It has been reported most frequently in the femur or in the tibia, although other bones throughout
the body have occasionally been involved.
Oral Manifestations:
• Greene and his associates have reviewed the literature and added one more cases, bringing the total number of cases
of osteoid osteoma of the jaws to seven. Of these, four have occurred in the mandible and three in the maxilla. Of the
mandibular lesions, three were in the body and one in the condyle, while one maxillary lesion involved the antrum.
Radiographic Features:
• Radiographically, the osteoid osteoma presents a pathognomonic picture characterized by a small ovoid or round
radiolucent area surrounded by a rim of sclerotic bone. The central radiolucency may exhibit some calcification. The
lesion seldom is larger than 1 cm indiameter, but the overlying cortex does become thickened by subperiosteal new
bone formation.
Histologic Features:
• The microscopic appearance of the osteoid osteoma is characteristicand consists of a central nidus composed of
compact osteoid tissue, varying in degree of calcification, interspersed by a vascular connective tissue. Formation of
definite trabeculae occurs, particularly in olderlesions, outlined by active osteoblasts. Osteoclasts and foci of bone
resorption are also usually evident. The overlying periosteum exhibits new bone formation, and in this interstitial tissue
collections of lymphocytes may be noted.
• The osteoblasts of a benign osteoblastoma were studied and were found to be essentially identical, including the
atypical mitochondria.The author concluded that his observations supported the idea that the osteoid osteoma and the
osteoblastoma are closely related lesions.

https://t.me/DentalBooksWorld
380 Triumph's Complete Review of Dentistry

Treatment:
• The treatment of the osteoid osteoma consists of surgical removal of the lesion. If the lesion is completely excised,
recurrence is not to be expected. There is fairly good circumstantial evidence that spontaneous regression may occur in
at least some untreated cases.
Benign Osteoblastoma(Giant osteoid osteoma):
• This central bone tumor occurs most frequently in young persons, approximately 75% of the patients being under 20
years of age and 90% under 30 years of age. However, it does occur even in elderly adults. In most reported series, there
is a definite predilection for occurrence in males. The lesion is characterized clinically by pain and swelling at the tumor
site, the duration being just a few weeks to a year or more. Unlike osteoid osteoma, the pain of osteoblastoma is more
generalized and less likely to be relieved by salicylates.
74. Answer: A
75. Answer: B
Codman Triangle

Periosteal reaction
Codman
Triangle
Advancing tumor margin
destroys periosteal new
bone before it ossifies

Tumor

76. Answer: B
77. Answer: D
78. Answer: B
79. Answer: B
• Rushton bodies (RBs) are one of the characteristic features seen in the epithelial lining of odontogenic cysts mainly
radicular, dentigerous, and odontogenic keratocyst. It has two different histomorphological appearances: granular and
homogeneous. Although widely investigated, the exact pathogenesis and histogenesis of RBs is still an enigma. Many
hypotheses were made in the literature but none has explained conceivably the two histomorphological appearances
of RBs and their association with inflammation. In the present paper the various pathogenesis for the formation of
RBs proposed till date are discussed along with proposal for a novel hypothesis. The given hypothesis is mainly related
to inflammation and its effect on pore size of basement membrane of odontogenic cystic epithelium. It explains RBs
association with inflammation as well as existence of two histomorphological appearances.
Pathogenesis of Rushton bodies
• The given hypothesis is mainly related to inflammation and its effect on pore size of basement membrane of odontogenic
cystic epithelium. It explains RBs association with inflammation as well as existence of two histomorphological
appearances. The proposed hypothesis also justifies the RB’s presence inside the lining epithelium of odontogenic cyst
despite its hematogenous origin.

TRIUMPH MCQ BANK


1. Answer: D
Van der Woude syndrome
(Cleft lip syndrome, lip pit syndrome, dimpled papillae of the lip)
• Van der Woude syndrome is an autosomal dominant syndrome typically consisting of a cleft lip or cleft palate and
distinctive pits of the lower lips.
• The degree to which individuals who carry the gene are affected widely varies, even within families. These variable
manifestations include lower lip pits alone, absent teeth, or isolated cleft lip and cleft palate of varying severity.
Hypodontia (absent teeth) has been increasingly recognized as a frequently associated anomaly.
• The most prominent and consistent features of van der Woude syndrome are orofacial anomalies caused by an abnormal
fusion of the palate and lips at 30–50 days post conception. Most cases of van der Woude syndrome have been linked

https://t.me/DentalBooksWorld
Chapter 4 • Oral Pathology and Oral Medicine 381

to a deletion in chromosome 1q32-q41; however, a second chromosomal locus at 1p34 has also been identified. The
responsible mutation has been identified in the interferon regulatory factor-6 (IRF -6) gene, but the exact mechanism of
this mutation on craniofacial development is uncertain.
• The hallmark of the van der Woude syndrome is the association of cleft lip and/or palate with distinctive lower lip pits.
This combination is seen in about 70% of those who are overtly affected but in less than half of those who carry the
gene. The cleft lip and palate may be isolated. They may take any degree of severity and may be unilateral or bilateral.
Submucous cleft palate is common and may be easily missed on physical examination. Hypernasal voice and cleft or
bifid uvula are clues to this diagnosis.
2. Answer: C
• Option A – cleft lip
• Option B – combined cleft lip and palate
3. Answer: C

Prominence Structures formed


Frontonasal One in number
Forms: Forehead, bridge of nose, lateral, and medial nasal prominences
Maxillary (Two) One on each side. Merge with the lateral nasal process, then with the medial
nasal process to form:
Parts of cheek, upper lip except the median part (philtrum), definitive palate
Two medial nasal process (from Two arise from lower border of frontonasal process; one on each side medially
frontonasal process) Forms: Philtrum of upper lip, primitive palate, crest, and tip of the nose
Two lateral nasal process (from Two arise from lower border of frontonasal process; one on each side laterally.
frontonasal process) Separated from the maxillary process by a cleft where the nasolacrimal duct lies
Forms: Ala of nose
Mandibular (Two) One on each side below the maxillary processes. Merge with its fellow of other
side to form: Whole of lower lip, whole of lower jaw, most of cheek

4. Answer: A
Dens evaginatus (Occlusal tuberculated premolar, Leong’s premolar, evaginatedodontome, occlusal enamel pearl)
Dens in dente (Dens invaginatus, dilated composite odontome)
5. Answer: A
6. Answer: D
7. Answer: A
8. Answer: C
9. Answer: B
Gingival cyst:
• It is located in the gingival soft tissues outside of the bone, and is derived from the rests of the dental lamina.
Two types of gingival cyst:
One – Gingival cyst of the adult:
• Occurs as a firm but compressible swelling on the mandible or maxilla, facial gingiva in the premolar, canine, incisor
region
• The cyst does not appear on a radiograph because it is confined to the gingival soft tissue
• Histopathologically, it include a thin lining of non keratinized epithelium about 1–3 cell thickness with a number of
clear cells
Two – Gingival cyst of the newborn:
• Generally seen on the alveolar ridges of newborn infants as small, often multiple swelling, and appear whitish in color
10. Answer: B
11. Answer: C
12. Answer: C
13. Answer: D

https://t.me/DentalBooksWorld
382 Triumph's Complete Review of Dentistry

14. Answer: C
15. Answer: D
16. Answer: C
17. Answer: C
18. Answer: B
• Direct immunofluorescence (DIF) of skin in patients with pemphigus vulgaris (PV), bullous pemphigoid (BP), and
dermatitis herpetiformis (DH).
• A -  Intercellular deposits of IgG within the lower epidermis in a chicken-wire pattern (PV) 
• B  - Linear deposits of C3 at the basement membrane zone (BP); a similar pattern of IgA deposition is seen in linear IgA
bullous dermatosis
• C  - Granular deposits of IgA within the dermal papillae (DH)
19. Answer: A

https://t.me/DentalBooksWorld
5 General Surgery

SYNOPSIS

WOUND HEALING
Definition
Wound: A wound is a break in the integrity of the skin or tissues, often which may be associated with disruption of the
structure and function.
Healing: Healing is the body’s response to injury in an attempt to restore its normal structure and function.
The process of healing involves two distinct mechanisms:
1. Repair: When healing takes place by proliferation of connective tissue elements, resulting in fibrosis and scarring
2. Regeneration: When healing occurs by proliferation of parenchymal cells, usually resulting in complete restoration of the
original tissues
Regeneration
• In order to maintain the proper structure of tissues, these cells are under the constant regulatory control of their cell cycle.
• Cell cycle is defined as the period between two successive cell divisions and is divided into four unequal phases:
1. M phase: Phase of Mitosis
2. G1 phase (Gap 1 phase)
3. S phase: Synthesis phase where the synthesis of nuclear DNA takes place
4. G2 phase (Gap 2 phase)
5. G0 phase: Resting phase

• Some mature cells do not undergo division, while other cells complete one cell cycle in 16–24 hours.
• Depending on the capacity to divide, the cells are categorized as follows:
–– Labile cells: Under normal physiological conditions, these cells continue to multiply throughout life. Examples: Surface
epithelial cells of epidermis, alimentary tract, respiratory tract, vagina, cervix, hematopoietic cells of BM, and cells of
lymph nodes and spleen.
–– Stable cells: These cells decrease or lose their ability to proliferate after adolescence but retain their capacity to divide
in response to stimuli throughout the life. Examples: Parenchymal cells of liver, pancreas, kidney, adrenal, and thyroid;
mesenchymal cells like smooth muscle cells, fibroblasts, vascular endothelium, etc.
–– Permanent cells: These cells never multiply. They lack the ability to regenerate. Examples: Neurons of nervous system,
skeletal muscle, and cardiac muscle cells.

Repair
• Repair is the replacement of injured tissue by fibrous tissue which involves the following mechanisms:
–– Granulation tissue formation
–– Contraction of wounds
• Repair consists of a combination of regeneration and scar formation by the deposition of collagen

https://t.me/DentalBooksWorld
384 Triumph’s Complete Review of Dentistry

Classification of Wound
Rank and Wakefield classification
1.  Tidy wounds
2.  Untidy wounds
Classification of surgical wounds
1. Clean wound
2. Clean contaminated wound
3. Contaminated wound
4. Dirty infected wound

Healing of skin wounds provides a classical example of regeneration and repair. Wound healing is accompanied by the
following mechanisms:
1.  Healing by First intention (primary union)
2.  Healing by Second intention (secondary union)

Healing by primary union


This is regarded as the healing of wound which has the following characteristics:
• Clean and uninfected
• Surgically incised
• Without much loss of cells and tissues
• Edges of wound are approximated by surgical sutures
The sequence of events in primary union are as follows:
• Initial hemorrhage
• Acute inflammatory response
• Epithelial changes
• Organization
• Suture tracks

Mechanism of primary healing

Day 0 (When wound is formed):


• Presence of blood clot in the incision

Day 1 (Within 24 hours):


• Neutrophilic infiltration + Blood clot
Day 2 (24–48 hours):
• Neutrophilic infiltration
• Blood clot
• Continuous thin epithelial layer
Day 3:
• Macrophages replaces neutrophils
• Appearance of granulation tissue
• Type III collagen deposition begins
Day 5:
• Abundant granulation tissue
• Collagen fibrils bridge the incision
• Neovascularization is maximum
• Full epithelial thickness with surface keratinization

https://t.me/DentalBooksWorld
Chapter 5 • General Surgery 385

End of second week:


• Accumulation of collagen
• Fibroblast proliferation
1 Month:
• Replacement of collagen type III with collagen type I due to action of collagenase enzyme
• Type I collagen has the greater tensile strength
Vitamin C is required for the conversion of tropocollagen to collagen due to hydroxylation of lysine and
proline residues providing stability to collagen molecules

Healing by secondary intention


This is regarded as the healing of a wound that has the following characteristics:
• Open wound with a large tissue defect, sometimes infected
• Wound with extensive loss of cells and tissues
• Wound that is not approximated and left as such opened
The sequence of events in secondary union are as follows:
• Initial hemorrhage
• Inflammatory phase
• Epithelial changes
• Granulation tissue
• Wound contraction
• Presence of infection

Repair process in secondary intention involves a combination of regeneration and scarring


• There is abundant granulation tissue formation
• At first matrix containing fibrin, plasma fibronectin and type III collagen is formed
• In about 2 weeks, it is replaced by a matrix composed mainly of type I collagen
• Wound contraction is a very important feature of secondary intention
• Wound contraction requires myofibroblasts

Complications of wound healing


• Infection
• Implantation (epidermal) cyst
• Pigmentation
• Deficient scar formation
• Incisional hernia
• Hypertrophied scars and keloid formation
• Excessive contraction
• Neoplasia (SCC arising from Marjolin’s ulcer)

Wound strength
• Wound strength is 10% after 1 week
• It increases rapidly during next 4 weeks
• Becomes 70% at the end of third month

https://t.me/DentalBooksWorld
386 Triumph’s Complete Review of Dentistry

Factors influencing wound healing

Local factors:
• Infection
• Poor blood supply
• Foreign bodies
• Movement
• Ionizing radiation
• Exposure to UV rays

Systemic factors:
• Age
• Nutrition: Deficiency of zinc delays wound healing
• Systemic infection
• Uncontrolled diabetes
• Blood abnormalities

Scar formation

Scar: formed as part of healing process following damage to skin as body lays down collagen fibers

A scar localized to a wound is known as hypertrophic scar


Features of hypertrophic scar:
• Scar is confined within the wound
• Usually develops within 4 weeks after trauma

A scar that extends beyond injury site is known as keloid


Features of keloid:
• Scar grows extensively beyond wound margins
• Genetically predisposed condition
• Seen mostly in Afro-Americans
• Sites: Sternum (Most common), ear lobe, face, and neck
• Usually develops after 3 months of injury

TYPES OF COLLAGEN
Collagen type Tissue distribution Genetic condition
Type I Adult hard and soft tissues, bones Osteogenesis imperfecta
Mainly in adult skin type EDS
(Has greatest tensile
strength)
Type II Cartilage, intervertebral discs, vitreous Achondroplasia type II
Type III Hollow organs and soft tissues Vascular EDS

(Initial collagen in wound


repair)
Type IV Basement membrane collagen Alport syndrome
Type V Blood vessels and soft tissues Classic EDS

https://t.me/DentalBooksWorld
Chapter 5 • General Surgery 387

Type VI Microfibrils Bethlem myopathy


Type VII Anchoring fibrils Dystrophic epidermolysis bullosa
Type VIII structural Descemet’s membrane of the cornea Osteogenesis imperfecta
Type IX Cartilage, vitreous Stickler syndrome

Healing of Specialized Tissues


Healing of a fracture
• Fracture heals either by primary union or secondary union
Primary union: It occurs in few special situations when the ends of fracture are approximated by application of
compression clamps. In this case, a bony union takes place with the formation of medullary callus without
periosteal callus formation.
Secondary union: It is the common process of fracture healing, which has the following three processes:
1.  Procallus formation
2.  Osseous callus formation
3. Remodeling
Complications of fracture healing
1.  Fibrous union: A false joint may develop at the fracture site (pseudoarthritis)
2.  Non-Union: It occurs if some soft tissue is interposed between the fracture ends
3.  Delayed union: Occurs in general conditions where there is infection, inadequate wound supply, poor nutrition, etc.

Wound healing after pulpectomy


• The healing pattern following pulpectomy is characterized by an initial inflammatory reaction in the apical tissue due to the
trauma induced by the cutting procedure.
• In the absence of wound infection, reorganization soon occurs. This involves replacement of the injured tissue by connective
tissue derived from the periapical region.
• Materials used to fill root canals may compromise the normal healing pattern, owing to their irritating capacity, and result
in a longstanding inflammatory lesion.
• Inflammatory cells accumulate close to the root filling material and remain for as long as toxic components are released.
Eventually the material will be lined off by fibrous connective tissue.
• When overfilling occurs, the process of phagocytosis may eliminate the excess root filling material and occasionally also
material inside the canal.

Wound healing of apical periodontitis


• During periapical wound healing, the osteoblasts or mesenchymal cells line the surfaces of endosteum.
• If both cortical bone plates are destroyed by large apical periodontitis lesions, it is possible that the lesion may be repaired
with fibrous scar tissue because of extensive destruction of the periosteum beneath the oral mucosa.
• Guided tissue regeneration procedure and bone grafts is recommended to prevent ingrowth of fibroblasts from periosteum
or submucosa into the bony defect and to enhance periapical wound healing if periapical surgery is necessary.

Healing of root fracture


• Healing of root fracture depends upon:
a. Site of the fracture
b. Status of the pulp
• Two directions of wound healing response are expected:
a. At the pulpal side
b. At periodontal ligament side
• If pulp is intact at the fracture site, the odontoblastic progenitor cells will create a hard tissue bridge uniting the fractures fragments.

https://t.me/DentalBooksWorld
388 Triumph’s Complete Review of Dentistry

The pattern of healing of root fracture are:


• Healing with calcified tissue
• Healing with interproximal calcified tissue
• Healing with interposition of bone and connective tissue
• Interposition of granulation tissue

Stem cells
• The most widely accepted stem cell definition is a cell with a unique capacity to produce unaltered daughter cells (self-
renewal) and to generate specialized cell types (potency)
• Stem cells are located in special sites called niches
Name of stem cell Function
Oval cells Forming hepatocytes and biliary cells
Satellite cells Differentiate into myocytes after injury
Limbus cells Stem cells of the cornea
Ito cells Store vitamin A
Paneth cells Host defense against microbes

SHOCK AND BLOOD TRANSFUSION


The metabolic response to injury
“Homeostasis: the coordinated physiological process which maintains most of the steady states of the organism.” (Definition
given by, Walter Cannon)
Neuroendocrine response to injury/critical illness
The neuroendocrine response to severe injury/critical illness is biphasic:
• Acute phase characterized by an actively secreting pituitary and elevated counter-regulatory hormones (cortisol, glucagon,
adrenaline)
• Chronic phase associated with hypothalamic suppression and low serum levels of the respective target organ hormones
Systemic inflammatory response syndrome (SIRS) following major injury
• Is driven initially by pro-inflammatory cytokines (e.g., IL-1, IL-6, and TNFα)
• Is followed rapidly by increased plasma levels of cytokine antagonists and soluble receptors (e.g., IL-1Ra, TNF-sR)
• If prolonged or excessive may evolve into a counter-inflammatory response syndrome (CARS)
The diagnostic findings of SIRS include
• Body temperature less than 36°C or greater than 38°C
• Heart rate/tachycardia >90 beats/minute
• Tachypnea >20/minute
• WBC count, less than 4,000 or more than 12000 cells/mm3
• Immature neutrophils more than 10% of total WBCs
Changes in the body composition following injury
• The average 70-kg male can be considered to consist of fat (13 kg) and fat-free mass (or lean body mass: 57 kg).
• In such an individual, the lean tissue is composed primarily of protein (12 kg), water (42 kg), and minerals (3 kg).
• The protein mass can be considered as two basic compartments: skeletal muscle (4 kg) and nonskeletal muscle (8 kg), which
includes the visceral protein mass.
• The water mass (42 l) is divided into intercellular (28 l) and extracellular (14 l) spaces.
• The main labile energy reserve in the body is fat, and the main labile protein reserve is skeletal muscle.

https://t.me/DentalBooksWorld
Chapter 5 • General Surgery 389

• Within lean tissue, each 1 g of nitrogen is contained within 6.25 g of protein.


• Protein turnover in the whole body is of the order of 150–200 g per day.
• A normal human ingests about 70–100 g protein per day, which is metabolized and excreted in urine as ammonia and urea
(i.e., approximately 14 g N per day).
• As with total starvation, once loss of body protein mass has reached 30–40% of the total, survival is unlikely.

Starvation
• During starvation, the body is faced with an obligate need to generate glucose to sustain cerebral energy metabolism
(100 g of glucose per day).
• Provision of at least 2 l of intravenous 5% dextrose as intravenous fluids for surgical patients who are fasted provides
100 g of glucose per day and has a significant protein-sparing effect.

Shock
Shock is a systemic state of low tissue perfusion, which is inadequate for normal cellular respiration
Hypovolemic shock
• Hypovolemic shock is caused by a reduced circulating volume.
• Hypovolemia is probably the most common form of shock and is, to some degree, a component of all other forms of shock.
• It is clinically manifested by
–– Low cardiac output
–– Tachycardia
–– Low BP
–– Vasoconstriction
Cardiogenic shock
• Cardiogenic shock is due to primary failure of the heart (mainly left ventricle) to pump blood to the tissues.
Neurogenic shock
• Such shock is primarily due to blockade of sympathetic nervous system resulting in loss of arterial and venous tone with
pooling of blood in the dilated peripheral nervous system.
• Neurogenic shock is probably the only form of shock that can be safely treated with a vasoconstrictor drug.
Septic shock
• This is most often due to Gram-negative septicemia (Endotoxins).
• It may occur in cases of severe septicemia, cholangitis, peritonitis, or meningitis.
• In the later phases of septic shock, there is hypovolemia from fluid loss into the interstitial spaces and there may be
concomitant myocardial depression, which complicates the clinical picture.
Anaphylactic shock
• It occurs due to increased release of histamine and slow release substance (SRS) of anaphylaxis by combination of antigen
with IgE on the mast cells and basophils.
• It is usually characterized by
–– Bronchospasm
–– Laryngeal edema
–– Respiratory distress
–– Hypoxia

Clinical stages of hypovolemic shock


Mild shock • Loss of less than 20% of blood volume
• Urinary output, pulse rate, and BP remain normal

https://t.me/DentalBooksWorld
390 Triumph’s Complete Review of Dentistry

Moderate shock • Loss of blood volume from 20% to 40%


• There will be oliguria, urine output dips below 0.5 ml kg–1h–1
• Pulse is increased but usually less than 100 beats/min
• Blood pressure may remain normal but may fall in later stages
Severe shock • Loss of blood more than 40%
• There will be pallor, low urinary output, rapid pulse, and low blood pressure

Endocrine shock
• Causes of endocrine shock include hypo- and hyperthyroidism and adrenal insufficiency
• Hypothyroidism causes a shock state similar to that of neurogenic shock as a result of disordered vascular and cardiac
responsiveness to circulating catecholamines
• Adrenal insufficiency leads to shock as a result of hypovolemia and a poor response to circulating and exogenous
catecholamines

Hemorrhage
• Hemorrhage leads to a state of hypovolemic shock

Types of hemorrhage
1.  Hemorrhage may be revealed or concealed
2.  It may be primary, secondary, or reactionary
• Revealed hemorrhage is obvious external hemorrhage, such as exsanguination from an open arterial wound or from
massive hematemesis from a duodenal ulcer.
• Concealed hemorrhage is contained within the body cavity and must be suspected, actively investigated, and controlled.
• Primary hemorrhage is hemorrhage occurring immediately as a result of an injury (or surgery).
• Reactionary hemorrhage is delayed hemorrhage (within 24 hours) and is usually caused by dislodgement of clot by
resuscitation, normalization of blood pressure, and vasodilatation. Reactionary hemorrhage may also result from technical
failure such as slippage of a ligature.
• Secondary hemorrhage is caused by sloughing of the wall of a vessel. It usually occurs 7–14 days after injury and is
precipitated by factors such as infection, pressure necrosis (such as from a drain), or malignancy.

Degree of hemorrhage
• The adult human has approximately 5 l of blood (70 ml/kg – children and adults, 80 ml/kg – neonates).
• Weighing of swabs is the best method of detecting blood loss.
• Total blood clot of the size of the clenched fist of a patient is equal to 500 ml of blood.
• The hemoglobin level is a poor indicator of the degree of hemorrhage as it represents a concentration and not an absolute
amount.
The degree of hemorrhage can be classified into classes 1–4 based on the estimated blood loss required to produce certain
physiological compensatory changes.

Class 1 Class 2 Class 3 Class 4


Blood volume lost as <15% 15–30% 30–40% >40%
percentage of total

Blood Transfusion
• Blood from the donor is collected in a sterile bag which already contains 75 ml of anticoagulant solution.
• About 410 ml of blood is taken in a single bag.

https://t.me/DentalBooksWorld
Chapter 5 • General Surgery 391

Blood and blood products


• Blood is stored in a blood bank in a special refrigerator at controlled temperature of 4°C (2–6°C)
Packed red cells
• Each unit is approximately 330 ml and has a hematocrit of 50–70%. Packed cells are stored in a SAG-M solution (saline–
adenine–glucose–mannitol) to increase their shelf-life to 5 weeks at 2–6°C.
• Older storage regimens included storage in CPD (citrate–phosphate–dextrose solutions) giving cells a shelf-life of
2–3 weeks.
Fresh-frozen plasma
• Fresh-frozen plasma (FFP) is rich in coagulation factors; it is removed from fresh blood and stored at –40 to –50°C with a
2-year shelf-life.
• It is the first-line therapy in the treatment of coagulopathic hemorrhage.
• Rhesus D-positive FFP may be given to a Rhesus D-negative woman.
Cryoprecipitate
• Cryoprecipitate is a supernatant precipitate of FFP and is rich in factor VIII and fibrinogen. It is stored at –30°C with a
2-year shelf-life.
• It is given in low-fibrinogen states or in cases of factor VIII deficiency.
Platelets
• Platelets are supplied as a pooled platelet concentrate containing about 250 × 109 cells per liter.
• Platelets are stored on a special agitator at 20–24°C and have a shelf-life of only 5 days.
Prothrombin complex concentrates
• Prothrombin complex concentrates (PCCs) are highly purified concentrates prepared from pooled plasma.
• They contain factors II, IX, and X; factor VII may be included or produced separately.
• PCCs are indicated for the emergency reversal of anti-coagulant (warfarin) therapy in uncontrolled hemorrhage.

Indications of blood transfusion


• Acute blood loss, to replace circulating volume and maintain oxygen delivery
• Perioperative anemia, to ensure adequate oxygen delivery during the perioperative phase; symptomatic chronic anemia
without hemorrhage or impending surgery

Transfusion trigger
• Historically, patients were transfused to achieve a hemoglobin level of >10 g dl–1.
• A hemoglobin level of 6 g dl–1 is acceptable in patients who are not bleeding, not about to undergo major surgery, and not
symptomatic.

Blood groups
Major blood group systems are ABO & Rh, while minor blood groups are MNS, Duffy, Kell, and Kidd.
ABO system
• Introduced by Karl Landsteiner in 1900
• These are strongly antigenic and are associated with naturally occurring antibodies in the serum
• The system consists of three allelic genes – A, B, and O
• Genetic loci of ABO is on chromosome 9 while Rh on chromosome 1
• The system allows for six possible genotypes although there are only four phenotypes
• Blood group O is the universal donor type as it contains no antigens to provoke a reaction
• Conversely, group AB individuals are “universal recipients” and can receive any ABO blood type as they have no circulating
antibodies

https://t.me/DentalBooksWorld
392 Triumph’s Complete Review of Dentistry

ABO AND Rh BLOOD GROUP SYSTEMS


Properties ABO system Rh system
Blood groups Four main blood groups: A, B, AB, and O Rh positive and negative
Genetic loci On chromosome 9 On chromosome 1
Antigen and antibodies Group Antigen Antibody C, c, D, E, e antigens
O H Anti-A and Anti-B d indicates the absence of D
A A Anti-B Anti-D antibody is the most
important
B B Anti-A
AB A and B None
Antigens also seen on Endothelial cells, epithelial cells, plasma, saliva, and No other cells
semen
Not seen in CSF
Type of Ab IgM, naturally occurring antibodies IgG, do not occur naturally
Clinical importance Anti-A and Anti-B ab can cause severe intravascular Rh− individuals make anti-D Ab if:
hemolysis after incompatible transfusion Transfused with Rh+ blood, Rh−ve
ABO-Matching is required before transplantation of pregnant women, is exposed to Rh
solid organs +ve fetal RBCs that have crossed the
placenta

Perioperative red blood cell transfusion criteria


Hemoglobin level Indication
<6 g/dl Probably will benefit from transfusion
6–8 g/dl Transfusion unlikely to be of benefit in the absence of bleeding or impending surgery
>8 g/dl No indication for transfusion

Transfusion reactions
• Complications from a single transfusion include:
• Incompatibility hemolytic transfusion reaction
• Febrile transfusion reaction
• Allergic reaction
• Infection
–– Bacterial infection (usually as a result of faulty storage)
–– Hepatitis
–– HIV
–– Malaria
• Air embolism
• Thrombophlebitis
• Transfusion-related acute lung injury (usually from FFP)
Complications from massive transfusion
Complications from massive transfusion include
• Coagulopathy
• Hypocalcemia
• Hyperkalemia
• Hypokalemia
• Hypothermia

https://t.me/DentalBooksWorld
Chapter 5 • General Surgery 393

Management of coagulopathy
Standard guidelines are as follows:
• FFP if prothrombin time (PT) or partial thromboplastin time (PTT) >1.5 × normal
• Cryoprecipitate if fibrinogen <0.8 g/l
• Platelets if platelet count <50 × 109/ml
–– In case of nonsurgical hemorrhage, anti-fibrinolytics such as tranexamic acid and aprotinin are the most commonly
administered.
–– Recombinant factor VIIa is also under investigation for the treatment of nonsurgical hemorrhage.

Additive solutions used for storage of blood products


Name Shelf-life
ACD Acid citrate dextrose 21 days
CPD Citrate phosphate dextrose 21 days
CPD–A Citrate phosphate dextrose with adenine 35 days
SAGM Sodium adenine glucose mannitol (also contains CPD as anticoagulant) 42 days

Action of ingredients of additive/anticoagulant solution


Ingredient Action
Glucose ATP generation by glycolysis
Adenine Synthesis of ATP, increases shelf-life of RBCs to 42 days
Citrate Prevents coagulation by chelating calcium
Sodium di phosphate Prevents fall in pH

Adverse effects of blood transfusion


Immediate Delayed Infections

Immune 1. Hemolytic transfusion 1. Delayed hemolytic-transfusion Viral:


reactions reactions 1. Hepatitis A, B, C, G
2. Febrile nonhemolytic 2. Allo-immunization to RBC/HLA/ 2. HIV, CMV, EBV
transfusion reactions Platelet/Neutrophil antigens 3. HTLV I/II
(FNHTR) 3. Graft-versus-Host disease (GVHD) 4. Parvo B19
3. Allergic transfusion 4. Post-transfusion purpura Bacterial:
reactions (Anaphylaxis/ 5. Immunodeficiency effects 1. Syphilis, Sepsis
Urticaria) Parasites:
4. Transfusion related Acute 1. Malaria, Babesia
Lung Injury (TRALI)
Others:
1. Creutzfeldt–Jakob disease
Nonimmune 1. Bacterial contamination 1. Pulmonary microembolization
2. Volume overload 2. Transfusion hemosiderosis
3. Massive transfusion

Massive transfusion
Definition 1. Replacement of >1 time the total blood volume, within 24 hours
2. Replacement of more than 50% of the blood volume in 3 hours in an adult

https://t.me/DentalBooksWorld
394 Triumph’s Complete Review of Dentistry

Complications –– Metabolic alkalosis > acidosis


–– Hyperkalemia
–– Hypocalcemia
–– Citrate toxicity
–– Hypomagnesemia
–– Depletion of coagulation factors
–– Increased risk of DIC
–– Dilutional thrombocytopenia
–– Hypothermia

TRAUMA
Trauma is recognized as a serious public health problem. In fact, it is the leading cause of death and disability in the first four
decades of life and is the third most common cause of death overall.
Mechanism of injury
• Low-velocity bullets behave like knife injuries
• High-velocity bullets cause cavitation
• The temporary cavity is large and draws in foreign materials
• The permanent cavity is smaller and gives no clue to the extent of damage

Triage
Triage is an important concept in modern health-care systems, and three essential phases have developed:
1.  Prehospital triage – in order to dispatch ambulance and prehospital care resources
2.  At the scene of trauma
3.  On arrival at the receiving hospital
In trauma, two types of triage situation usually exist:

1.  Multiple casualties: Here, the number and severity of injuries do not exceed the ability of the facility to render care.
Priority is given to the life-threatening injuries followed by those with polytrauma.

2.  Mass casualties: The number and severity of the injuries exceed the capability and facilities available to the staff.
In this situation, those with the greatest chance of survival and the least expenditure of time, equipment and supplies are
prioritized.

Energy and injury prevention


• A 10% increase in speed of impact increases pedestrian fatality risk by 40%
• Seatbelts reduce the risk of injury in a vehicle by 45%

TRIAGE CATEGORIES
Priority Colour Medical need Clinical status Examples
First (I) Red Immediate Critical but likely to survive Severe facial trauma, tension
if treatment given early pneumothorax, profuse external bleeding,
haemothorax, flail chest, major intra-
abdominal bleed, extradural haematomas
Second (II) Yellow Urgent Critical, likely to survive if Compound fractures, degloving injuries,
treatment given within hours ruptured abdominal viscus, pelvic
fractures, spinal injuries

https://t.me/DentalBooksWorld
Chapter 5 • General Surgery 395

Third (III) Green Non-urgent Stable, likely to survive even Simple fractures, sprains, minor
if treatment is delayed for lacerations
hours to days
Last (0) Black Unsalvageable Not breathing, pulseless, Severe brain damage, very extensive
so severely injured that no burns, major disruption/loss of chest or
medical care is likely to help abdominal wall structures

Crush injury
• Muscle cells die. If reperfused, they release myoglobin
• Injured tissue sequesters fluid
• Renal shutdown results
• Treatment is fluid loading with monitoring of renal output to maintain diuresis

Primary survey
This is the heartbeat core of the ATLS system and constitutes the ABCDE of trauma care.
ABCDE of trauma care
• A – Airway maintenance and cervical spine protection
• B – Breathing and ventilation
• C – Circulation with hemorrhage control
• D – Disability: neurological status
• E – Exposure: completely undress the patient and assess for other injuries
Airway assessment
• Check verbal response
• Clear mouth and airway with large-bore sucker
• If GCS 8, consider a definitive airway; otherwise use jaw thrust or chin lift
Breathing
• Give 100% oxygen at high flow
• Check for tension pneumothorax
• Decompress at once if tension pneumothorax is suspected (needle in the second intercostal space)
Secondary survey
This starts after completion of the primary survey and once initial resuscitative measures have commenced. The purpose of
the secondary survey is to identify all injuries and perform a more thorough “head to toe” examination.
Again, the AMPLE mnemonic from the ATLS group is helpful here.

Review of patient’s history (AMPLE)


• Allergy
• Medication including tetanus status
• Past medical history
• Last meal
• Events of the incident

HEAD INJURY
Normal metabolism of brain
• Brain oxygen consumption (CMRO2, cerebral metabolic rate for oxygen) is about 3.5 ml/100 g/min.
• The brain relies on blood borne glucose for 90% of its energy requirements.
• Normal cerebral blood flow is approximately 55 ml 100 g–1min–1 and is usually maintained at a constant level via mechanisms
termed cerebral autoregulation

https://t.me/DentalBooksWorld
396 Triumph’s Complete Review of Dentistry

Primary vs. secondary brain injury


• Primary brain injury occurs at the time of impact and includes injuries such as brainstem and hemispheric contusions,
diffuse axonal injury, and cortical lacerations.
• Secondary brain injury occurs at some time after the moment of impact and is often preventable.
The principle causes of secondary brain injury are:
1.  Hypoxia: PO2 <8 kPa
2.  Hypotension: systolic blood pressure (SBP) <90 mmHg
3.  Raised intracranial pressure (ICP): ICP >20 mmHg
4.  Low cerebral perfusion pressure (CPP): CPP <65 mmHg
5. Pyrexia
6. Seizures
7.  Metabolic disturbance

Glasgow coma scale


Severity of head injury is classified according to the Glasgow Coma Score (GCS). The GCS – and in particular the motor
score – is the best predictor of neurological outcome.
• Minor head injury: GCS 15 with no loss of consciousness (LOC)
• Mild head injury: GCS 14 or 15 with LOC
• Moderate head injury: GCS 9–13
• Severe head injury: GCS 3–8

GLASGOW COMA SCALE


Eyes open Spontaneously 4
To verbal command 3
To painful stimulus 2
Do not open 1
Verbal Normal oriented conversation 5
Confused 4
Inappropriate/words only 3
Sounds only 2
No sounds 1
Intubated patient T
Motor Obeys commands 6
Localises to pain 5
Withdrawal/flexion 4
Abnormal flexion (decorticate) 3
Extension (decerebrate) 2
No motor response 1
•  For intubated patients, only “E” (eye opening) + “M” (best motor response) and suffix “T” is given.
•  So for intubated patients, the maximal GCS score is 10 “T” and the minimal score is 1 “T”.

NICE (National Clinical Guideline Center, UK) guidelines for computerized tomography (CT) in head
injury
• Glasgow Coma Score (GCS) <13 at any point
• GCS 13 or 14 at 2 hours
• Focal neurological deficit
• Suspected open, depressed, or basal skull fracture
• Seizure
• Vomiting >one episode

https://t.me/DentalBooksWorld
Chapter 5 • General Surgery 397

Urgent CT head scan if none of the above but


• Age >65 years
• Coagulopathy (e.g., on warfarin)
• Dangerous mechanism of injury (CT within 8 hours)
• Antegrade amnesia >30 minutes (CT within 8 hours)

Extradural hematoma
• An extradural hematoma (EDH) is a neurosurgical emergency.
• An EDH is nearly always associated with a skull fracture.
• The skull fracture is associated with tearing of a meningeal artery and a hematoma accumulates in the space between the
bone and the dura.
• The most common site is temporal, as the pterion is not only the thinnest part of the skull but also overlies the largest
meningeal artery – the middle meningeal.
• The classical presentation of an EDH, occurring in less than one-third of cases, is initial injury followed by a lucid interval
when the patient complains of a headache but is fully alert and orientated with no focal deficit.
• After minutes or hours a rapid deterioration occurs, with contralateral hemiparesis, reduced conscious level and ipsilateral
pupillary dilatation as a result of brain compression and herniation.
• The features of an EDH on a CT scan are a lentiform (lens-shaped or biconvex) hyperdense lesion between the skull and
the brain.
Acute subdural hematoma
• An acute subdural hematoma (ASDH) accumulates in the space between the dura and the arachnoid.
• The CT appearance of an ASDH is also hyperdense (acute blood) but the hematoma spreads across the surface of the brain
giving it a rather diffuse and concave appearance.
Chronic subdural hematoma
• Chronic subdural hematomas (CSDH) usually occur in the elderly and are more common in those on anti-coagulant or
antiplatelet agents.
• The CT appearance of a CSDH is variable. Acute blood (0–10 days) is hyperdense whereas subacute blood (10 days to
2 weeks) is isodense relative to brain; chronic blood (>2 weeks) is hypodense.

Intracranial pressure monitoring


A sustained ICP of >20 mmHg is associated with a worse outcome. The ICP can also be used to calculate the cerebral
perfusion pressure.
Medical management of raised intracranial pressure
• Position head up 30 degrees
• Avoid obstruction of venous drainage from head
• Sedation +/– muscle relaxant
• Normocapnia 4.5–5.0 kPa
• Diuretics: furosemide, mannitol
• Seizure control
• Normothermia
• Sodium balance
• Barbiturates
• Hypotension sits alongside hypoxia as a major cause of secondary brain injury.
• A single episode of hypotension with a systolic blood pressure of <90 mmHg is associated with a worse outcome in traumatic
coma.
• Cerebral perfusion pressure should be maintained at >65 mmHg in severely head-injured patients.

https://t.me/DentalBooksWorld
398 Triumph’s Complete Review of Dentistry

Cerebral perfusion pressure = mean arterial pressure − intracranial pressure


CPP = MAP – ICP
If the ICP is 20 mmHg, it follows that the MAP should be ≥85 mmHg.

FRACTURES OF HEAD AND NECK


Radiological Views for Specific Fractures
Site of fracture Radiographic views
Mandible; body and ramus OPT, Lateral obliques, lower occlusal, PA mandible
Mandible; condyles OPT, PA mandible with mouth open, Toller transpharyngeal views
Maxilla OM 15 and 30 degrees, lateral facial bones
Zygomatic complex OM 15 and 30 degrees
Orbital blow-outs OM 15 and 30 degrees and tomograms
Nasal bones Lateral nasal bones, occipitofrontal
Frontal bones Lateral skull, occipitofrontal

Fractures of middle third


In 1911, René Le Fort classified fractures according to patterns which he created on cadavers using varying degrees of force.
Le Fort I fracture:
• The Le Fort I fracture effectively separates the alveolus and palate from the facial skeleton above.
• The fracture line runs through points of weakness from the nasal pyriform aperture through the lateral and medial walls of
the maxillary sinus, running posteriorly to include the lower part of the pterygoid plates.
Le Fort II fracture:
• The Le Fort II fracture is pyramidal in shape. The fracture involves the orbit, running through the bridge of the nose and
the ethmoids, whose cribriform plate may be fractured, leading to a dural tear and CSF rhinorrhea.
• It continues to the medial part of the infraorbital rim and often through the infraorbital foramen.
Le Fort III fracture:
• The Le Fort III fracture effectively separates the facial skeleton from the base of the skull – the fracture lines run high
through the nasal bridge, septum, and ethmoids, again with the potential for dural tear and CSF leak, and irregularly
through the bones of the orbit to the frontozygomatic suture.

The zygomatic complex fractures


• This is the most common fracture of the middle third of the face, apart from the nose. The fractures occur through points
of weakness – the infraorbital margin, the frontozygomatic suture, the zygomatic arch, and the anterior and lateral wall of
the maxillary sinus.
• Damage to the infraorbital nerve is common, causing numbness on the cheek.

Blow-out fractures
• Direct trauma to the globe of the eye may push it back within the orbit.
• The weakest plate of bone, most commonly the orbital floor, fractures, and the orbital contents herniate down into the
maxillary antrum.
• This soft-tissue herniation may lead to muscular dysfunction, particularly the inferior oblique and inferior rectus, leading
to failure of the eye to rotate upwards.
• Enophthalmos and diplopia can follow, although both may initially be concealed by edema.
• Paresthesia in the distribution of the infraorbital nerve may be an important clue to the blow-out fracture.

https://t.me/DentalBooksWorld
Chapter 5 • General Surgery 399

Fractures of the mandible


The condylar neck is the weakest part of the mandible and is the most frequent site of fracture.
• Guardsman fracture: A blow to the chin point may cause a fracture of the symphysis or parasymphysis of the lower jaw, and
indirect transmission of the kinetic energy causes a unilateral or bilateral fracture of the mandibular condyles.
• Butterfly fracture: A segment of mandible is detached from the rest of the mandible in the canine regions. The segment of
bone may include the anterior insertion of the tongue (genioglossus and geniohyoid).

Management of Facial Injuries


Soft tissue injuries
• Facial soft tissues have an excellent blood supply and heal well with contours.
• Hemostasis is important. Muscle and underlying tissues should be brought together with absorbable sutures so that the
edges of the wound lie passively within 2 mm of their final position.
• Then fine monofilament sutures (5/0 or 6/0) are used to bring the wound edges together.
Parotid duct
Parotid duct transection
• Cannulate from the mouth and anastomose over the stent.
Fractured nasal bones
• The nasal bones are the most commonly fractured bones of the facial skeleton. Best results are obtained when soft-tissue
edema has been allowed to settle so that accurate reduction can be achieved.
• Surgery should ideally be carried out within a week of the injury.
• Reduction should be directed first to repositioning the nasal bones, disimpacting with Walsham’s forceps.
• The nasal bones are first moved laterally to disimpact them and then medially to reposition them. The septum is then
grasped with Asch’s forceps, manipulated until it is straight and then positioned in the groove of the nasal crest and vomer.
• It should be remembered, however, that the nasal septum often cannot be adequately manipulated into position and may
require formal septoplasty at a later date.
Fractures of maxilla
• The principle of reducing and stabilizing fractures of the frontal and facial bones is that the surgeon starts at the top and
works down.
• When the stabilization of the upper part of the face is complete, attention may be turned to the midface.
• Incisions in the lower eyelid (blepharoplasty incision), lower conjunctival sac, or infraorbital region are used to explore
fractures of the infraorbital rim.
• These also give access to the orbital floor and are used to treat orbital blow-out fractures.
• The lower part of the maxilla is approached through a gingival sulcus incision above the maxillary teeth as far back as the
second molar.
• The principle of treatment is to restore the fragments to their original position. To achieve this, it is usually necessary to
reduce the maxilla first with Rowe’s disimpaction forceps, which grasp the palate between the nasal and palatal mucosa.
Fractures of mandible
• Fractures of the mandible were frequently reduced indirectly and then fixed with intermaxillary fixation (IMF).
• Prior to the introduction of these plating systems, patients would often have their jaws “wired together” for a period of up
to 6 weeks.
• Open reduction and fixation of the fractured mandibular condyle within 7–10 days of the original injury is indicated.
• If an anterior open bite (vertical pull of, muscles of mastication, shortening the ramus height) is evident in a unilateral
condylar fracture with significant displacement, or in a bilateral condylar fracture.
Fractures of zygomatic complex
• Second to the fractured nasal bone, this is the most common fracture of the middle third of the facial skeleton.
• Displacement is usually posteriorly.
• Most fractures may be reduced by the Gillies temporal approach.
• This entails an incision in the hair line, superficial to the temporal fossa, about 15 mm long, at 45 degrees to the vertical.
• It is deepened down to and through the temporalis fascia.

https://t.me/DentalBooksWorld
400 Triumph’s Complete Review of Dentistry

Orbital blow out fractures


• These fractures are ideally treated within 10–14 days of the original injury.
• The floor of the orbit is approached either through a blepharoplasty incision in the lower eyelid or through the inferior fornix.
• If the fragments are very unstable owing to comminution or the size of the blow-out is excessively large, packing the antrum
via a Caldwell–Luc approach may be necessary.
• When the orbital floor has been adequately reduced and periorbital soft tissue freed from the fracture, the antrum may be
packed from above downwards with ribbon gauze soaked in Whitehead’s varnish, taking care not to overpack the antrum
and so displace the orbital contents.
• The pack should be removed at 3 weeks.
Intraoral injuries
Management of avulsed teeth
• Ideally, immediate reimplantation is the treatment of choice (should be reimplanted within 60 minutes of the injury).
• Once out of the socket, periodontal ligament cells can survive a dry period of up to 30 minutes without significant harm.
• Wet storage can increase periodontal ligament cell vitality, and such storage may increase the chances of successful reimplantation.
• The patient’s own saliva or fresh milk is the best transport medium available to the lay person.
• Water should not be used because of its osmotic effects on the periodontal ligament cells.
• The socket should be gently irrigated with normal saline to remove any clot/debris.
• The tooth should be held by the crown, and the root gently irrigated with saline to remove debris before the tooth is firmly
reimplanted in the socket.
• Semirigid fixation for a period of 7–10 days is necessary, and oral antibiotics are required for a period of 5–7 days following
reimplantation.

BURNS
Burns cause damage in a number of different ways, but by far the most common organ affected is the skin.
Airway injuries occur when the face and neck are burned.
Dangers of smoke, hot gas, or steam inhalation
• Inhaled hot gases can cause supraglottic airway burns and laryngeal edema
• Inhaled steam can cause subglottic burns and loss of respiratory epithelium.
Burns may be classified as
• Ordinary burns caused by dry heat
• Scalds caused by moist heat
• Chemical burns caused by strong acids or base
• Electric burns
• Radiation burns
• Cold burns

The shock reaction after burns


• Burns produce an inflammatory reaction
• This leads to vastly increased vascular permeability
• Water, solutes, and proteins move from the intra- to the extravascular space
• The volume of fluid lost is directly proportional to the area of the burn
• Above 15% of surface area, the loss of fluid produces shock.

Assessing the area of burn


The patient’s whole hand is 1% TBSA, and is a useful guide in small burns.
• The Lund and Browder chart is useful in larger burns.
• The rule of nines is adequate for a first approximation only.

https://t.me/DentalBooksWorld
Chapter 5 • General Surgery 401

Rule of Nines
Anatomic area % of body surface
Head, face, and neck 9
Right upper extremity 9
Left upper extremity 9
Right lower extremity 18
Left lower extremity 18
Anterior trunk 18
Posterior trunk 18
External genitalia 1

Assessing the depth of burn


The history is important – temperature, time, and burning material
• Superficial burns have capillary filling
• Deep partial-thickness burns do not blanch but have some sensation
• Full-thickness burns feel leathery and have no sensation
It takes 6 hours for skin maintained at 44°C to suffer irreversible changes, but a surface temperature of 70°C for 1 second is all
that is needed to produce epidermal destruction.
Superficial partial-thickness burns
• The damage in these burns goes no deeper than the papillary dermis.
• The clinical features are blistering and/or loss of the epidermis.
• The underlying dermis is pink and moist. The capillary return is clearly visible when blanched.
• There is little or no fixed capillary staining. Pinprick sensation is normal.
• Superficial partial-thickness burns heal without residual scarring in 2 weeks.
Deep partial-thickness burns
• These burns involve damage to the deeper parts of the reticular dermis. Clinically, the epidermis is usually lost. The exposed
dermis is not as moist as that in a superficial burn. There is often abundant fixed capillary staining, especially if examined
after 48 hours.
• Pinprick test is positive.
• Deep dermal burns take 3 or more weeks to heal without surgery and usually lead to hypertrophic scarring.
Full-thickness burns
• The whole of the dermis is destroyed in these burns.
• Clinically, they have a hard, leathery feel.
• The appearance can vary from that similar to the patient’s normal skin to charred black, depending upon the intensity of
the heat.
• There is no capillary return. Often, thrombosed vessels can be seen under the skin.
• These burns are completely anesthetized: a needle can be stuck deep into the dermis without any pain or bleeding.

Fluid resuscitation
In children with burns over 10% TBSA and adults with burns over 15% TBSA, consider the need for intravenous fluid resuscitation
• If oral fluids are to be used, salt must be added
• Fluids needed can be calculated from a standard formula
• The key is to monitor urine output.
Hyponatremia and water intoxication can be fatal. It is therefore appropriate to give oral rehydration with a solution such as
Dioralyte.
Three types of fluids are used. The most common is Ringer’s lactate or Hartmann’s solution.
Perhaps the simplest and most widely used formula is the Parkland formula. This calculates the fluid to be replaced in the first
24 hours by the following formula.

https://t.me/DentalBooksWorld
402 Triumph’s Complete Review of Dentistry

Total percentage body surface area × weight (kg) × 4 = volume (ml)


Half of this volume is given in the first 8 hours, and the second half is given in the subsequent 16 hours.

Crystalloid resuscitation
Ringer’s lactate is the most commonly used crystalloid. Crystalloids are said to be as effective as colloids for maintaining
intravascular volume.
Another reason for the use of crystalloids is that even large protein molecules leak out of capillaries following burn injury.
In children, maintenance fluid must also be given. This is normally dextrose–saline given as follows:
• 100 ml kg–1 for 24 hours for the first 10 kg
• 50 ml kg–1 for the next 10 kg
• 20 ml kg–1 for 24 hours for each kilogram over 20 kg body weight
Colloid resuscitation
Proteins should be given after the first 12 hours of burn because, before this time, the massive fluid shifts cause proteins to
leak out of the cells.
The commonest colloid-based formula is the Muir and Barclay formula:
• 0.5 × percentage body surface area burnt × weight = one portion
• Periods of 4/4/4, 6/6, and 12 hours, respectively
• One portion to be given in each period

Monitoring of resuscitation
• The key to monitoring of resuscitation is urine output.
• Urine output should be between 0.5 and 1.0 ml kg–1 body weight per hour.
• If the urine output is below this, the infusion rate should be increased by 50%.
• It is important that patients are not over resuscitated, and urine output in excess of 2 ml kg–1 body weight per hour should
signal a decrease in the rate of infusion.

Options for topical treatment of deep burns


• 1% silver sulfadiazine cream
• 0.5% silver nitrate solution
• Mafenide acetate cream
• Serum nitrate, silver sulfadiazine, and cerium nitrate

Reconstruction
Graft anatomy
Split-thickness graft (Thiersch graft)
Split-thickness skin grafts are harvested by taking all of the epidermis together with some dermis, leaving the remaining
dermis behind to heal the donor site.
Thicker knife-gap settings recognized give rise to fewer but brisker bleeding points on the donor site.
• Thicker grafts heal with less contracture and are more durable
• Thinner donor sites heal better
• Grafts are hairless and do not sweat (these structures are not transferred)
Full-thickness grafts (Wolfe grafts)
Full-thickness grafts are harvested to incorporate the whole dermis, with the underlying fat trimmed away – unless elements
of fat (or even cartilage as well) are deliberately left attached to form a composite graft.

https://t.me/DentalBooksWorld
Chapter 5 • General Surgery 403

Metabolic response to starvation


• Low plasma insulin
• High plasma glucagon
• Hepatic glycogenolysis
• Protein catabolism
• Hepatic gluconeogenesis
• Lipolysis: mobilization of fat stores
• Adaptive ketogenesis
• Reduction in resting energy expenditure (15–20 kcal kg–1 day–1)

Metabolic response to trauma and sepsis


• Increased counter-regulatory hormones: adrenaline, noradrenaline, cortisol, glucagon, and growth hormone
• Increased energy requirements (up to 40 kcal kg–1day–1)
• Increased nitrogen requirements
• Insulin resistance and glucose intolerance
• Preferential oxidation of lipids
• Increased gluconeogenesis and protein catabolism
• Loss of adaptive ketogenesis
• Fluid retention with associated hypoalbuminemia

FLUID AND ELECTROLYTES


Fluid intake is derived from both exogenous (consumed liquids) and endogenous (released during oxidation of solid
foodstuffs) fluids.
Fluid losses occur by four routes
Lungs: About 400 ml of water is lost in expired air each 24 hours.
Skin: In a temperate climate, skin (i.e., sweat) losses are between 600 and 1,000 ml day–1.
Feces: Between 60 and 150 ml of water are lost daily in patients with normal bowel function.
Urine: The normal urine output is approximately 1,500 ml day–1.
A minimum urine output of 400 ml day–1 is required to excrete the end products of protein metabolism.
The following are the approximate daily requirements of some electrolytes in adults:
• Sodium: 50–90 mM day–1
• Potassium: 50 mM day–1
• Calcium: 5 mM day–1
• Magnesium: 1Mm day–1

Composition of crystalloid and colloidal solutions


5% Dextrose
• Isotonic solution that supplies calories but not electrolytes
• Used when the patient requires replenishment of his blood volume along with some nutrition but no electrolytes
• Particularly used in the immediate postoperative period when Na excretion is considerably diminished by renal conservation
Isotonic (0.9%) sodium chloride solution
• It is isotonic and contains Na and Cl in the concentration almost similar to that in plasma.
• It should not be used in first 24 hours after operation due to natural sodium conservation.
• One important fact is that it contains a high concentration of Cl as compared to plasma and imposes appreciable load of
excess Cl on the kidneys that cannot be readily excreted.

https://t.me/DentalBooksWorld
404 Triumph’s Complete Review of Dentistry

Ringer’s lactate solution


• Its main advantage is that it has almost similar electrolyte concentration as ECF and the pH remains normal even if infused
in large quantities.
• This solution is best to be used in hypovolemic shock while awaiting for blood.
• Its main disadvantage is its slight hypo-osmolarity with respect to Na.
Darrow’s solution
• This is the only solution which contains more potassium than available in the plasma or ECF.
• This is the best solution to combat hypokalemia.
• The rate of infusion should be slower than other solutions to avoid state.

Parenteral nutrition
• Total parenteral nutrition (TPN) is defined as the provision of all nutritional requirements by means of the intravenous
route and without the use of the gastrointestinal tract.
• The most frequent clinical indications relate to those patients who have undergone massive resection of the small intestine,
who have intestinal fistula.
Peripheral feeding
• Peripheral feeding is appropriate for short-term feeding of up to 2 weeks.
• Access can be achieved either by means of a dedicated catheter inserted into a peripheral vein and maneuvered into the
central venous system (Peripherally Inserted Central venous Catheter (PICC) line).
• These PICC lines have a mean duration of survival of 7 days.
• Their disadvantage is that when thrombophlebitis occurs the vein is irrevocably destroyed.
Central
• When the central venous route is chosen, the catheter can be inserted via the subclavian or internal or external jugular vein.
Complications of TPN
• Common metabolic complications include fluid overload, hyperglycemia, abnormalities of liver function, and vitamin
deficiencies.
• A weight change of >1 kg day–1 normally indicates fluid retention.
• Hyperglycemia is common because of insulin resistance in critically ill patients.
• Abnormalities of liver enzymes are common in patients receiving TPN.
• Catheter related sepsis are also more common.

Stenosis or occlusion
• Stenosis or occlusion produces symptoms and signs that are related to the organ supplied by the artery, e.g., lower limb –
claudication, rest pain, and gangrene.
• Features of arterial stenosis or occlusion in the leg.
Intermittent claudication
Intermittent claudication is a cramp-like pain felt in the muscles that is
• brought on by walking
• not present on taking the first step (unlike osteoarthrosis) relieved by standing still
The pain of claudication is most commonly felt in the calf
Boyd’s classification
Grade I: Pain disappears if the patient continues to walk.
Grade II: Pain continues but the patient can still walk with effort.
Grade III: Pain compels the patient to take rest.

https://t.me/DentalBooksWorld
Chapter 5 • General Surgery 405

Rest pain
Rest pain occurs with the limb (usually the leg) at rest; it is exacerbated by lying down or elevation of the foot.
Characteristically, the pain is worse at night and it may be lessened by hanging the foot out of bed or by sleeping in a chair.
Temperature sensation
A severely ischemic foot is usually cold, but an ischemic limb tends to equilibrate with the temperature of its surroundings and
may feel quite warm under the bedclothes.

Gangrene
It often affects the distal part of a limb because of arterial obstruction (from thrombosis, embolus, or arteritis).
Clinical features
• The color of the part changes through a variety of shades according to circumstances (pallor, dusky grey, mottled,
purple) until finally taking on the characteristic dark-brown, greenish-black, or black appearance, which is caused by the
disintegration of hemoglobin and the formation of iron sulfide.
• Dry gangrene occurs when the tissues are desiccated by gradual slowing of the bloodstream; it is typically the result of
atheromatous occlusion of arteries.
• Moist gangrene occurs when infection and putrefaction are present.
• Crepitus may be palpated as a result of infection by gas-forming organisms. This situation is quite common in the feet of
diabetics.
• A zone of demarcation between the truly viable and the dead or dying tissue will eventually appear.

Bed sores
A bedsore is gangrene caused by local pressure.
Bedsores are predisposed to by five factors:
• Pressure
• Injury
• Anemia
• Malnutrition
• Moisture
A bedsore can be expected if erythema appears that does not change color on pressure.
Indications for amputation
Dead limb
• Gangrene

Deadly limb
• Wet gangrene
• Spreading cellulitis
• Arteriovenous fistula
• Other (e.g., malignancy)

“Dead loss” limb


• Severe rest pain
• Paralysis
• Other (e.g., contracture, trauma)

https://t.me/DentalBooksWorld
406 Triumph’s Complete Review of Dentistry

ARTERITIS AND VASOSPASTIC CONDITIONS


Thromboangiitis obliterans (Buerger’s disease)
• This is characterized by occlusive disease of the small- and medium-sized arteries (plantar, tibial, radial, etc.), thrombophlebitis
of the superficial or deep veins, and Raynaud’s syndrome.
• It occurs in male smokers, usually under the age of 30 years.
• Temporal arteritis is a disease in which localized infiltration with inflammatory and giant cells leads to arterial occlusion,
ischemic headache, and tender, palpable, pulseless (thrombosed) arteries in the scalp.
• Irreversible blindness occurs if the ophthalmic artery becomes occluded.
• The surgeon may be required to perform a temporal artery biopsy, but this should not delay immediate steroid therapy to
arrest and reverse the process before the ophthalmic artery is involved.
Takayasu’s disease is an arteritis that obstructs major arteries, particularly the large vessels coming off the aorta. It usually
pursues a relentless course.
Buerger’s test
• The legs of the normal individuals remain pink when elevated, but in case of ischemic limb, elevation causes marked pallor.
• The angle of the limb at which such pallor appears is known as Buerger’s point.
• An angle less than 30 degrees indicates severe arterial occlusion.
Raynaud’s disease
• This idiopathic condition usually occurs in young women and affects the hands more than the feet.
• There is abnormal sensitivity in the arteriolar response to cold.
• These vessels constrict and the digits (usually the fingers) turn white and become incapable of fine movements.
• The condition is recognized by the characteristic sequence of blanching, dusky cyanosis, and red engorgement, often
accompanied by pain.
Raynaud’s syndrome
• Raynaud’s syndrome is most often used for a peripheral arterial manifestation of a collagen disease such as systemic lupus
erythematosus or rheumatoid arthritis.
• Raynaud’s syndrome may also follow the use of vibrating tools. In this context it is a recognized industrial disease and is
known as “vibration white finger.”
• Nifedipine, steroids, and vasospastic antagonists may all have a role in treatment. Patients with vibration white finger should
avoid vibrating tools.
Clinical presentation of acute arterial occlusion can be described by 5 P’s
• Pain
• Paralysis
• Paresthesia
• Pallor
• Absent pulses

Venous diseases
Venous ulcers
• Venous ulcers are the commonest ulcers of the leg.
• Usually occur in the lower one-third of the leg.
Varicose veins
• These are defined as tortuous dilated veins.
• There is often a clear family history of the disorder, with some patients inheriting abnormalities in the FOXC2 gene.
• Varicose veins may develop secondarily in patients with post-thrombotic limbs and in patients with congenital abnormalities
such as the Klippel–Trenaunay syndrome or multiple arteriovenous fistulae.
• Left side is more predominantly affected than right side.

https://t.me/DentalBooksWorld
Chapter 5 • General Surgery 407

Venous thrombosis
• A venous thrombus is the formation of a semi-solid coagulum within flowing blood in the venous system. Venous thrombosis
of the deep veins of the leg is complicated by the immediate risk of pulmonary embolus and sudden death.
• The three factors described by Virchow over a century ago are still considered important in the development of venous
thrombosis.
• These are changes in the vessel wall (endothelial damage); stasis, which is diminished blood flow through the veins;
coagulability of blood (thrombophilia).
• Deficiencies of anti-thrombin, activated protein C, and protein S have all been shown to predispose to venous thrombosis
in young patients.
• Activated protein C deficiency is associated with inheritance of the factor V Leiden gene and may account for the higher
incidence of venous thrombosis in Caucasian populations.
• Recently the term “e-thrombosis” has been used to describe blood clots occurring in people sitting at their computers for
long periods of time.
Diagnosis
• The most common presentation of a deep vein thrombosis is pain and swelling, especially in the calf of one lower
limb.
• Homans’ sign – resistance (not pain) of the calf muscles to forcible dorsiflexion – is not discriminatory and should be
abandoned.

Management
• Warfarin is usually started at a dose of 10 mg on day 1, 10 mg on day 2, and 5 mg on day 3.
• A prothrombin time taken on day 3 guides the maintenance dose of warfarin.

Lymphatic diseases
Lymphedema
Lymphedema may be defined as abnormal limb swelling caused by the accumulation of increased amounts of high
protein ISF (Interstitial fluid) secondary to defective lymphatic drainage in the presence of (near) normal net capillary
filtration.
Symptoms frequently experienced by patients with lymphedema
• Constant dull ache, even severe pain
• Burning and bursting sensations
• General tiredness and debility
• Sensitivity to heat
• “Pins and needles”
• Cramp
• Skin problems including flakiness, weeping, excoriation, and breakdown
• Immobility, leading to obesity and muscle wasting
• Athlete’s foot
• Acute infective episodes
Two main types of lymphedema are recognized:
1.  Primary lymphedema, in which the cause is unknown (or at least uncertain and unproven); it is thought to be caused by
“congenital lymphatic dysplasia”;
2.  Secondary or acquired lymphedema, in which there is a clear underlying cause.

https://t.me/DentalBooksWorld
408 Triumph’s Complete Review of Dentistry

QUICK FACTS

Wound healing
• Neutrophilic infiltration occurs within 24 hours of injury
• Continuous thin epithelial layer is formed in day 2
• Neutrophils are replaced by macrophages on day 3
• Neovascularization is maximum on day 5
• The predominant collagen in adult skin type is type I
• In early granulation tissue, the predominant collagen are types III and I
• Wound strength will never reach 100%
• Zinc is a co-factor in collagenase
• Zinc deficiency is associated with impaired wound healing
• Infections are the most common cause of impaired wound healing
• Granulation tissue is the hall mark of the fibrogenic repair
• The chief cell responsible for scar contraction is myofibroblast
• Sternum is the most common site for keloid formation
• Intralesional steroids (triamcinolone) are the usual drugs for the management of a keloid
• Stem cells appear in human embryo at about third week
• Glucocorticoids delay wound healing by inhibiting collagen synthesis and anti-inflammatory effect
• Neuronal stem cells are oligopotent stem cells
• Blanching at wound site occurs during second week
• Remodeling of connective tissues is carried out by matrix metalloproteinases (MMPs)

Shock
• Hyperventilation is an important indicator of shock
• The diastolic pressure is the main indicator of the degree of vasoconstriction
• Multiple organ failure currently carries a mortality rate of 60%
• Adrenergic discharge starts within 60 seconds after blood loss
• Release of vasoactive hormones usually takes place after 1–2 minutes of hemorrhage
• Swan–Ganz catheter is used in sophisticated centers to get valuable information about the precise diagnosis and circular
derangements of shock
• This catheter is introduced into the CVS mostly through right internal jugular vein
• Steroids have been used sometimes in the treatment of septic shock
• Cardiac compressive shock occurs due to the compression of heart from outside leading to failure of the pumping
mechanism, though the heart itself is normal
• Hypovolemic shock develops after the loss of 40% of blood
• Tachycardia is the earliest sign of hemorrhagic shock
• In a victim of maxillofacial trauma, hypovolemic shock is the most common type of shock
• Dopamine is the choice of vasopressor in cardiogenic shock
• During the management of shock, the JVP should be maintained in the range of 10–15 mmHg
• The common condition seen in all forms of shock is inadequate tissue perfusion

Blood transfusion
• Fresh frozen plasma contains all coagulation factors
• Most frequent transfusion reaction is FNHTR
• TRALI is caused by Ab against patient’s HLA type II and HNA (human neutrophilic antigen)
• A total of 30 blood group systems have been classified
• Cryoprecipitate is not useful in Hemophilia B
• FFP is relatively deficient in factors V and VIII

https://t.me/DentalBooksWorld
Chapter 5 • General Surgery 409

• To prevent hyperkalemia due to blood transfusion, it is preferable to use blood <7 days old
• Hepatitis C is the most common cause of transfusion-associated viral hepatitis
• Acute hemolytic transfusion reactions are type II hypersensitivity reactions caused by complement-mediated hemolysis
• FNHTR is caused by antibodies against donor lymphocytes and HLA antigens

Fluid replacement therapy


• Intravenous route is the most common route of fluid replacement
• Mostly the median cubital vein in the cubital fossa is used for venepuncture
• Dextrose solutions provide water replacement without any electrolytes and with modest calorie supplements (1 l of 5%
dextrose contains 400 kcal)
• A typical daily maintenance fluid regimen would consist of a combination of 5% dextrose with either Hartmann’s or normal
saline to a volume of 2 l
• If the hematocrit is below 30%, blood transfusion may be required

MULTIPLE CHOICE QUESTIONS

INTRODUCTION AND FLUID DYNAMICS


1. In medicines management, the extent to which patients take medication as prescribed by healthcare professionals is termed
A. Concordance B. Adherence
C. Holistic care D. Paternalism
2. What percentage of the body weight of a normal person is intracellular fluid? Select the one best answer
A. 20% B. 30%
C. 40% D. 50%
3. Insensible loss of fluid from skin and lungs for 24 hours is normally in the span of
A. 100–250 ml B. 250–500 ml
C. 500–750 ml D. 750–1,000 ml
4. Insensible water loss from lungs is
A. 200 ml B. 500 ml
C. 1,000 ml D. 1,500 ml
5. The intravascular and interstitial fluids are different only in
A. pH B. Osmolarity
C. Protein concentration D. Electrolyte composition
E. Nonionizable solutes
6. The extracellular fluid differs from the intracellular fluid in all of the following except
A. Protein content B. Electrolyte composition
C. Volume D. Osmolarity
E. pH
7. Concerning the sodium ion, which statement is untrue among the following?
A. Diffuses readily through cell membranes B. Is the major cation of the extracellular fluid
C. Is the chief regulator of body water D. Its urinary excretion is reduced after trauma
8. Potassium concentration is highest in
A. Bile B. Gastric juice
C. Succus entericus D. Blood
9. What percentage (by body weight) of the extracellular volume in a normal person is extravascular or interstitial?
A. 5% B. 10%
C. 15% D. 20%

https://t.me/DentalBooksWorld
410 Triumph’s Complete Review of Dentistry

10. Which one of the following statements about homeostasis is false?


A. It is defined as a stable state of the normal body
B. The central nervous system, heart, lungs, kidneys, and spleen are the essential organs that maintain normal homeostasis
C. Elective surgery should cause little disturbance to homeostasis
D. Emergency surgery should cause little disturbance to homeostasis
11. Changes in body metabolism that occur in response to trauma are the following except
A. Lipolysis B. Gluconeogenesis
C. Protein breakdown D. Hypoglycemia
12. You are called to see the same patient 7 days postoperatively as he has become unwell and pyrexial with a temperature
of 39.0° C. The patient has generalized abdominal discomfort. The abdomen is tender with generalized guarding and
rebound. The chest is clear to auscultation. The patient’s catheter and epidural were removed 2 days ago. The most
likely explanation for the patient’s pyrexia is
A. Deep vein thrombosis B. Infective exacerbation of chronic airways disease
C. Pulmonary embolus D. Anastomotic leakage
13. A 21-year-old man with O blood group sustained a splenic injury in a road traffic accident. He is undergoing a
transfusion of 4 units prior to surgery. You are asked to review the patient 10 minutes into the transfusion as he has
become unwell and agitated. He has pyrexia (39.5°C) with associated tachycardia (120 beats/minute) and hypotension
(80/50 mmHg). Which of the following is the most likely cause?
A. Nonhemolytic febrile transfusion reaction B. Transfusion-related acute lung injury
C. Bacterial contamination D. Hemolytic transfusion reaction (ABO incompatibility)
14. A 35-year-old man is about to undergo a left hemicolectomy for colorectal carcinoma. He is an insulin-dependent
diabetic. The most appropriate perioperative management is
A. Preoperatively commence 0.9% normal saline (3 l in 3 hours), along with 20 units of intramuscular Actrapid insulin
with 6 units per hour thereafter along with potassium supplementation
B. Preoperatively start 50 units of insulin in 500 ml of normal saline and continue through to postoperative period, then
restart normal subcutaneous insulin when the patient is eating and drinking normally
C. Start intravenous infusion of 5 or 10% dextrose (500 ml bags) over 4–6 hours and add insulin and potassium chloride
to each bag, titrated to blood glucose and potassium levels
D. Continue usual subcutaneous insulin until and including the day of surgery. Place first on the list and monitor blood
glucose preoperatively, intraoperatively, and in recovery
15. Perioperative glycemic control–A 55-year-old man is about to undergo a diagnostic knee arthroscopy as a day case.
He has type 2 diabetes mellitus for which he takes metformin 850 mg/day. The most appropriate perioperative
management is
A. Start intravenous infusion of 5% or 10% dextrose (500 ml bags) over 4–6 hours and add insulin and potassium chloride
to each bag, titrated to blood glucose and potassium levels
B. Continue oral hypoglycemic agents until and including the day of surgery
C. Provided that blood glucose <10 mmol/l, continue oral hypoglycemic agents until day of surgery, then omit morning
dose, restart oral hypoglycemics with first meal
D. Provided that blood glucose <10 mmol/l, preoperatively start 50 units of insulin in 50 ml of normal saline and continue
through to postoperative period, then restart oral hypoglycemics with first meal
16. A 38-year-old man is admitted with abdominal pain and treated for pancreatitis; 48 hours following his admission you
are asked to assess the patient as he has become increasingly confused and aggressive. Observations are not possible but
you note he appears to be breathing hard, he is tremulous, and has pruritus. Choose an appropriate management strategy
A. Septic screen; urine dip, chest radiograph, and blood cultures
B. Chlordiazepoxide 20 mg intravenously, four times daily for 1 week
C. Haloperidol 2 mg intramuscularly and confine to side room
D. Oral chlordiazepoxide-reducing regimen with 48 hours intravenous thiamine
17. A 72-kg patient is 1 day following total hip replacement. He has not started eating and drinking. He is being rehydrated
with dextrose/saline (4% dextrose and 0.18% saline). Which one of the following best describes this type of fluid therapy?
A. It is an inappropriate fluid therapy for a postoperative patient
B. Its osmolality is almost isotonic with plasma (286 mOsm/kg)
C. It contains 120 mmol of Na+ ions
D. Potassium supplementation is not required

https://t.me/DentalBooksWorld
Chapter 5 • General Surgery 411

18. Hypovolemic shock leads to


A. Metabolic acidosis B. Respiratory acidosis
C. Respiratory alkalosis D. Metabolic alkalosis
19. A patient is commenced on total parenteral nutrition. You are asked by the nutrition team to ensure that adequate
monitoring takes place. Which one of the following statements regarding monitoring of total parenteral nutrition is
most correct?
A. Daily liver function tests B. Weekly blood capillary glucose
C. Daily blood glucose D. Monthly full blood count
20. 1 unit of blood should be transfused at a rate of
A. 4–6 hours B. 1–2 hours
C. 3 hours D. 7 hours
21. A 34-year-old man is on the surgical ward following an assault resulting in severe head injury. The speech and
language therapist is unhappy with the patient’s swallow as he regurgitates fluid and is at risk of aspiration. Which of
the following is the best long-term strategy for addressing this patient’s nutritional requirements?
A. Nasogastric feeding B. Nasojejunal feeding
C. Percutaneous gastrostomy tube D. Total parenteral nutrition
22. Platelets survive in stored blood for
A. 1 day B. 2 days
C. 5 days D. 7 days
23. The most serious complication of blood transfusion is
A. Pyrogenic reactions B. Thrombophlebitis of recipient vein
C. Circulatory overloading D. Incompatibility reactions
24. The most important clue to mismatched transfusion in an anesthetized patient is
A. ECG changes B. Hypotension
C. Bleeding D. Tachycardia
25. Common indications for nutritional support
A. Ileus more than 4 days B. Massive bowel resection
C. Intestinal fistula D. All of the above
26. Regarding the role of the gut in shock and sepsis, all are true except
A. Selective decontamination of the digestive tract with the use of oral antibiotics has been shown to reduce nosocomial
pneumonias and to improve mortality rates
B. Enteral nutrition preserves the villus architecture of the gut
C. Gut dysfunction may contribute to the development of MODS by bacterial translocation
D. As compared with parenteral nutrition, enteral nutrition is associated with a reduction in septic morbidity
27. Blood group mismatching can be accepted for which transplant?
A. Kidney B. Liver
C. Heart D. None of the above
28. Indication of liver transplantation include all the following except
A. Primary sclerosing cholangitis
B. Biliary atresia
C. Fulminant hepatic failure secondary to acetaminophen overdose
D. Small central hepatoma with extrahepatic disease
29. Regarding immunology in liver transplantation, all are true except
A. Good human leukocyte antigen (HLA) matching between recipient and donor is mandatory for a good outcome for
liver transplantation
B. Hyperacute rejection is almost nonexistent following liver transplantation
C. Acute rejection occurs in 30–50% of patients and is reversible in most patients with large doses of steroids
D. Chronic rejection, is usually irreversible, and often requires retransplantation
30. The earliest manifestation of hypocalcaemia is
A. Carpopedal spasms B. Positive Chvostek’s sign
C. Positive Trousseau’s sign D. Tingling of fingers and circumoral region

https://t.me/DentalBooksWorld
412 Triumph’s Complete Review of Dentistry

31. Regarding the prothrombin time, all of the following are true except
A. Measures the activity of the extrinsic coagulation pathway
B. Is not usually prolonged in liver disease
C. Is normal in hemophilia A
D. Can be expressed as the INR when monitoring warfarin dosage
E. Is prolonged in vitamin K malabsorption (obstructive jaundice)
32. Regarding low-molecular-weight heparins, choose the correct answer
A. Have a longer half-life than unfractionated heparins
B. Act predominantly on factor Xa
C. In spite of adequate anticoagulation, APTT remains within the normal limits
D. All are true
33. Regarding nonclostridial diffuse necrotizing fasciitis
A. Caused by synergistic action of microaerophilic streptococci, staphylococci, aerobic Grain-negative bacteria and anaerobes
B. Fasciitis begins in a localized area, e.g., a puncture wound, leg ulcer, or surgical wound
C. The infection spreads along the relatively ischemic fascial planes leading to thrombosis of penetrating vessels.
D. All the above
34. The patient with tension pneumothorax will exhibit the following
A. Tachypnea B. Agitation
C. Decreased percussion note on the injured side D. A and B is true
35. Regarding acute extradural hematoma
A. Patients usually have ipsilateral papillary dilatation B. Patients usually have contralateral hemiplegia
C. CT demonstrates a biconvex lesion D. All the above

ENDOCRINE AND THYROID


1. Which of the following is incorrect regarding the thyroid?
A. The gland develops from the third pharyngeal pouch
B. The C cells are developed from the ultimobranchial body
C. The weight of normal gland is 20–25 g
D. The resting follicle contains a colloid in which iodine is stored
2. In median ectopic thyroid, which of the following statements is true?
A. Forms a swelling in the upper part of the neck B. Usually mistaken as thyroglossal cyst
C. May be the only normal thyroid tissue present D. All of the above
3. Which of the following statements regarding lingual thyroid is false?
A. This forms a rounded swelling at the foramen caecum
B. It may represent the only thyroid tissue
C. It may cause impairment of speech, respiratory obstruction, or hemorrhage
D. It is common in males
4. Which of the following statements regarding thyroglossal cyst is true?
A. This may be present in any part of the thyroglossal tract
B. Develops from 1st and 2nd pharyngeal arches
C. The swelling involves upward on protrusion of the tongue as well as on swallowing
D. All of the above
5. Which of the following statements regarding thyroglossal fistula is false?
A. It is congenital
B. Follows infection
C. Follows inadequate removal of a thyroglossal cyst
D. Long-standing fistulas are inclined to be situated low down in the neck
6. Synthesis and liberation of thyroid hormone from thyroid is controlled by
A. Hypothalamus B. Hippocampus
C. Anterior pituitary D. Posterior pituitary

https://t.me/DentalBooksWorld
Chapter 5 • General Surgery 413

7. The distinct steps of T3 and T4 synthesis are the following except


A. Trapping of inorganic iodide from the blood
B. Oxidation of iodide to iodine
C. Binding of iodine with tyrosine to form iodotyrosines
D. No hormone remains free in the serum
8. The correct sequence of events for the metabolism of iodine and synthesis of thyroid hormone is
A. Trapping, organification, coupling release, oxidation
B. Oxidation, trapping, coupling, organification, release
C. Coupling, organification, trapping, oxidation, release
D. Trapping, oxidation, organification, coupling, release
9. The thyroid stimulating antibody is
A. IgG B. IgM
C. lgA D. lgD
10. Which of the following statements regarding thyroid function test is true?
A. T3 estimation is conclusive B. T4 estimation is conclusive
C. Serum TSH level is conclusive D. No single test is conclusive
11. Simple goiter commonly develops as a result of
A. Stimulation of thyroid gland by TSH-as a result of inappropriate secretion from a microadenoma in anterior
pituitary
B. Stimulation of thyroid gland by TSH to chronically low level of circulating thyroid hormone
C. The most important factor is dietary deficiency of iodine
D. Defective hormone synthesis probably accounts for many sporadic goiter
12. All of the following are goitrogens except
A. Vegetables of Brassica family B. Para-aminosalicylic (PAS) and calcium
C. Thyroxine D. Thiocyanate and perchlorates
13. Which of following statements regarding diffuse hyperplastic goiter is untrue?
A. The goiter appears in childhood
B. If TSH stimulation ceases, the goiter may regress
C. Goiter is soft, diffuse, and may become large enough to cause discomfort
D. T3 level is increased
14. Which of the following statements regarding multinodular goiter is true?
A. Nodules are usually multiple forming a multinodular goiter
B. Occasionally only macroscopic nodule is found, but microscopic changes will be present throughout the gland
C. This is one form of a clinically solitary nodule
D. All of the above
15. All the following are complications in nodular goiter except
A. Tracheal obstruction B. Secondary thyrotoxicosis
C. Carcinoma D. Enophthalmos
16. Which of the following statements regarding isotope thyroid scan is true?
A. Cold nodules are categorized as underactive nodules
B. Warm nodules are categorized as active nodules
C. Hot nodules are categorized as overactive nodules
D. All of the above
17. For the diagnosis of which of the following condition, thyroid scan is most useful?
A. Goiter B. Toxic adenoma of thyroid
C. Cysts of thyroid D. Thyroid malignancies
18. Which type of thyroid is more prone to for malignant transformation?
A. Nodular B. Simple goiter
C. Toxic goiter D. Colloid goiter
19. Wayne’s clinical diagnostic index is helpful for detecting
A. Thyroid tumor B. Thyrotoxicosis
C. Exophthalmos D. Hypothyroidism

https://t.me/DentalBooksWorld
414 Triumph’s Complete Review of Dentistry

20. Which of the following statements regarding retrosternal goiter is untrue?


A. Often symptomless and is discovered on a routine chest radiograph
B. Many of these patients attend chest clinic with diagnosis of asthma
C. May present as dysphagia
D. Recurrent laryngeal nerve paralysis is very common
21. The symptoms of hyperthyroidism are the following except
A. Emotional liability B. Heat intolerance
C. Weight gain D. Excessive appetite
22. Which of the following statements is true about primary thyrotoxicosis?
A. CVS manifestation is common B. Eye signs are never seen
C. CNS manifestations are common D. Usually manifests around 35 years of age
23. True regarding Von Graefe’s sign
A. Lagging behind of the upper eyelid
B. Retraction of the upper eyelid with infrequent wrinkling
C. Absence of wrinkling of the forehead
D. Convergence of the eyes is difficult
24. In pediatric Graves’ disease, why is radioiodine not preferred?
A. Radioiodine is potentially carcinogenic B. Antithyroid drugs are without side effects
C. Radioiodine is expensive D. None of the above
25. Regarding treatment of thyrotoxicosis during pregnancy, all are true except
A. Treatment of thyrotoxicosis during pregnancy is predominantly with antithyroid drugs
B. Antithyroid drugs should be used with the highest possible doses
C. Propylthiouracil (PTUI) is the drug of choice, predominantly because of its protein binding which leads to less
transplacental transfer
D. Radioiodine treatment is contraindicated during pregnancy and lactation
26. Which of the following treatment schedule for diffuse toxic goiter is true?
A. Over 45 years: radioactive iodine B. Under 45 years, with large goiter: surgery
C. None of the above D. All of the above
27. Surgery is the treatment of choice in toxic nodular goiter because of the following except
A. Does not respond to antithyroid drugs rapidly
B. Does not respond to radioiodine rapidly
C. Goiter itself is often large and uncomfortable
D. Enlargement continues to occur even with antithyroid drugs
E. None of the above
28. Which of the following statements regarding tension hematoma deep to cervical fascia following subtotal
thyroidectomy is true?
A. Slippage of a ligature on the superior thyroid artery
B. Occasionally hemorrhage from a thyroid remnant
C. Bleeding thyroid veins
D. It is necessary to open the wound in the ward to relieve tension
E. All of the above
29. Which of the following statement regarding thyrotoxic crisis (storm) is untrue?
A. Is an acute exacerbation of hyperthyroidism
B. If a thyrotoxic patient is inadequately prepared for thyroidectomy
C. May follow an unrelated operation
D. Symptomatic and supportive treatment for dehydration, hyperpyrexia, and restlessness is essential
E. None of the above
30. Which of the following statements regarding follicular adenoma is true?
A. It presents clinically as a solitary nodule
B. Distinction between follicular adenoma and carcinoma can only be made by histological examination
C. In adenoma there is no invasion of the capsule or of pericapsular blood vessels
D. Preferable treatment is lobectomy
E. All of the above

https://t.me/DentalBooksWorld
Chapter 5 • General Surgery 415

31. Most common primary malignant thyroid tumor is


A. Papillary B. Follicular
C. Anaplastic D. Medullary
32. Which of the following is not correct in the presentation of carcinoma of the thyroid?
A. Earache B. Hoarseness of the voice
C. A pulsating bone tumor D. The sex ratio is 3 males to 1 female
33. All of the following are true about papillary carcinoma of the thyroid except
A. It is the slowest growing malignant tumor of the thyroid
B. It may change into the anaplastic variety
C. It metastasizes primarily by hematogenous spread
D. It is dependent on thyroid-stimulating hormone (TSH) stimulation
34. Advantages of total thyroidectomy for management of papillary carcinomas of the thyroid, include all the following except
A. Possibility of using radioactive iodine postoperatively to identify and treat metastases
B. The ability to use thyroglobulin levels as a marker for recurrence
C. Lower overall recurrence rate
D. Lower risk of hypoparathyroidism
35. The term lateral aberrant thyroid implies
A. Congenital aberrant thyroid tissue lateral to the thyroid
B. A metastasis in a cervical lymph node from an occult thyroid carcinoma
C. A metastasis from carcinoma of the larynx
D. A type of branchial cyst
36. In which of the following carcinoma of thyroid, psammoma bodies are seen?
A. Follicular B. Medullary
C. Papillary D. Anaplastic
37. Thyroid cancer with multiple bone metastasis is most likely to be of which type?
A. Medullary B. Papillary
C. Anaplastic D. Follicular
38. Which of the following statements regarding medullary carcinoma is untrue?
A. Tumors of C cells derived from neural crest B. Tumors of cells of thyroid follicle
C. There is a characteristic amyloid stroma D. High levels of serum calcitonin are produced
39. In thyroid carcinoma, mediastinal node involvement is a feature of which type?
A. Follicular B. Anaplastic
C. Papillary D. Medullary
40. Diarrhea in medullary carcinoma of thyroid is due to
A. Serotonin B. Calcitonin
C. Thyroxine D. Hypercalcemia
41. Which of the following is not a part of multiple endocrine neoplasia Type IIa?
A. Medullary thyroid carcinoma B. Pheochromocytoma
C. Parathyroid adenoma D. None of the above
42. Regarding medullary carcinoma of the thyroid, all are true except
A. Is a tumor of the parafollicular C cells B. Produce thyroxine as the principle hormone
C. The majority of cases are sporadic D. Can occur as part of the MEN type II syndrome
43. Which of the following is of viral origin?
A. Autoimmune thyroiditis B. de Quervain’s thyroiditis
C. Riedel’s thyroiditis D. Colloid goiter
44. The classical presentation of Hashimoto’s thyroiditis is
A. Presents as a multinodular goiter
B. Thyroid failure is common
C. Primary myxedema without detectable thyroid enlargement represents the end stage of pathological process
D. All of the above

https://t.me/DentalBooksWorld
416 Triumph’s Complete Review of Dentistry

45. Ideal treatment of Hashimoto’s thyroiditis is


A. Subtotal thyroidectomy B. Steroids
C. Thyroxin D. TSH
46. Which of the statements is false regarding Hashimoto’s thyroiditis?
A. Autoimmune thyroiditis B. Women commonly affected
C. Hypothyroid state D. Always requires thyroidectomy
47. Ligation of the superior thyroid artery above the superior pole of the thyroid gland carries with it the risk of
A. Injury to the recurrent nerve B. Injury to the vagus nerve
C. Devascularization of the inferior parathyroid gland D. Injury to the external branch of the superior laryngeal nerve
48. Regarding the parathyroid glands, they
A. Are normally two in number B. Lie inside the capsule of the thyroid
C. Are reddish-blue in color D. Are not the secretors of calcitonin
49. Which of the following clinical features is not related to hyperparathyroidism?
A. Risus sardonicus B. Psychic Moans
C. Abdominal groans D. Pain from renal stones
50. In primary hyperparathyroidism, choose the wrong answer
A. Result from single or multiple adenomas B. Associated with multiple bone cysts
C. Can present with renal stones D. Tetany is one of the feature
51. Chvostek’s sign is
A. Carpal spasm induced by sphygmomanometer cuff pressure on the upper arm above the systolic blood pressure for not
more than two minutes in the normal person
B. As above option A, but the person has tetanus
C. Twitching of the facial muscles produced by tapping over the branches of the facial nerve in front of the tragus of the
ear in the normal person
D. As above option C, but the person has tetany
52. In a patient presenting with Graves’ disease, the following serum parameters would be raised except
A. T4 B. Thyrocalcitonin
C. Total protein bound iodine (PBI) D. Thyroid stimulating antibodies
53. Thyrotoxicosis is characterized by all except
A. Weight loss B. Proximal myopathy
C. Enophthalmos D. Tachycardia during sleep
54. Papillary carcinoma of the thyroid
A. Is associated with hoarseness B. Presents with bony metastases
C. Spreads to cervical lymph nodes D. Is associated with a bruit over the thyroid gland
55. A 42-year-old woman with a medullary carcinoma of the thyroid gland may present with all of the following except
A. Toxic symptoms B. Earache
C. Hoarseness D. Stridor
E. Enlarged cervical lymph nodes
56. Regarding retrosternal goiter, all of the following are true except
A. Usually arises from ectopic thyroid tissue B. Can be of the plunging type
C. Can cause a “scabbard” trachea D. May have to be removed piece meal
57. Regarding pretibial myxedema, all of the following are true Except
A. Is a thickening of the skin by a mucin-like deposit B. May be cyanotic when cold
C. Is an occasional feature of myxedema D. May be associated with clubbing of the fingers and toes
58. Total thyroidectomy
A. Is indicated for papillary carcinoma
B. Is indicated for medullary carcinoma
C. Is often impossible for anaplastic carcinoma
D. Includes the removal of all the parathyroid glands as there is usually an ectopic gland in the mediastinum
E. Option A, B, and C is true

https://t.me/DentalBooksWorld
Chapter 5 • General Surgery 417

59. All of the following are true about Riedel’s thyroiditis, except
A. The thyroid tissue is replaced by fibrous tissue B. A scan shows no uptake over the swelling
C. An anaplastic carcinoma is in fact present D. There may be mediastinal fibrosis
60. Regarding parathormone (PTH), all of the following are true except
A. Is a peptide hormone
B. Increases urinary phosphate excretion
C. Serum levels are raised in chronic renal failure
D. Requires vitamin D as a precursor
E. Stimulates osteoblastic activity
61. Bones, stones, groans, and moans are related to
A. Hyperparathyroidism B. hypoparathyroidism
C. Hyperthyroidism D. Hypothyroidism
62. The biochemical findings which support a diagnosis of hyperparathyroidism include
A. Diminution of the serum calcium
B. Elevation of the serum alkaline phosphatase
C. Increased excretion of calcium in the urine
D. Elevated serum T4
E. Both B and C
63. Which of the following statements regarding primary hyperparathyroidism are true?
A. Adenoma is the commonest cause of hyperparathyroidism
B. Multiple adenomas may be found in small percentage of cases
C. Carcinoma is a rare cause of hyperparathyroidism
D. Radionuclide scan is the best way of preoperative localization of parathyroid adenoma
E. All the above
64. Which of the following statements are true regarding secondary hyperparathyroidism?
A. Associated with chronic renal failure
B. The stimulus for hyperplasia is chronic hypocalcemia
C. All four glands are involved
D. Most patients with secondary hyperparathyroidism are treated medically
E. All the above
65. Which of the following statements are true for MEN type I, except
A. There is hyperplasia of parathyroid glands
B. Chromophobe adenoma of pituitary gland may result in acromegaly
C. Pancreatic tumors may produce insulin, gastrin, somatostatin, glucagons
D. Treatment is surgical excision
66. Which of the following statements are true for MEN type2 a?
A. 50% of patients have parathyroid hyperplasia
B. Medullary carcinoma of thyroid may be associated
C. Pheochromocytoma may be associated
D. Pheochromocytoma should be excluded before exploration of the neck
E. All the above
67. MEN type ll b include
A. Medullary carcinoma of thyroid B. Pheochromocytoma
C. Thyroid hyperplasia D. Mucosal neuromas and Marfanoid facial appearance
68. Pheochromocytoma
A. Represent tumors of the adrenal cortex B. Are associated with MEN I syndrome
C. May be found at aortic bifurcation D. Are frequently bilateral
69. “Rule of ten” for pheochromocytoma include all of the following, except
A. 10% bilateral B. 10% benign
C. 10% extra-adrenal D. 10% multiple
70. Which of the following statements are true for neuroblastoma, except
A. Over 50% occur in children below 2 years B. 50% arise from adrenal gland
C. 50% are malignant D. 50% show stippled calcification
https://t.me/DentalBooksWorld
418 Triumph’s Complete Review of Dentistry

71. Neuroblastoma commonly metastasizes to


A. Lung B. Bone
C. Liver D. Brain
72. Thyroid stimulating hormone (TSH) causes all of the following, except
A. Activation of thyroid follicular cells B. Increased iodide trapping in thyroid follicles
C. Increased thyroglobulin synthesis D. Increased release of T3/T4
E. Increased release of calcitonin
73. All of the following are true regarding medullary carcinoma of the thyroid, except
A. Arises from the cells of the thyroid follicle
B. Associated with increased level of calcitonin
C. Some tumors are familial
D. Can present with diarrhea
E. Metastasis to cervical nodes occurs in 50–60% of cases
74. In hyperparathyroidism
A. Result from single or multiple adenomas B. Associated with multiple bone cysts
C. Can present with renal stones D. Pancreatitis is a recognized complication
E. All the above
75. All of the following are true regarding papillary carcinoma of the thyroid, except
A. May follow radiation exposure in childhood B. Is a rapid growing tumor
C. Usually metastasizes to lymph nodes D. Can be multifocal tumor
76. Regarding Hashimoto’s thyroiditis
A. Characterized by diffuse lymphocytic infiltration
B. Diffuse enlargement of the thyroid gland in a middle-aged woman
C. Autoantibodies against thyroid peroxidase, thyroglobulin
D. Increase the risk of thyroid lymphoma
E. All the above
77. When progressive enlargement of a multinodular goiter causes symptomatic tracheal compression, the preferred
management in otherwise good-risk patients is
A. Iodine treatment B. Thyroid hormone treatment
C. Surgical resection of the abnormal thyroid D. Radioactive iodine treatment
78. The most precise diagnostic screening procedure for differentiating benign thyroid nodules from malignant
ones is
A. Thyroid ultrasonography B. Thyroid scintiscan
C. Fine-needle-aspiration biopsy (FNAB) D. Thyroid hormone suppression
79. The preferred operation for initial management of a thyroid nodule that is considered suspicious for malignancy by
FNAB is
A. Excision B. Partial lobectomy
C. Total lobectomy and isthmusectomy D. Total thyroidectomy
80. At the age of 45, an accountant has developed hoarseness due to an inoperable cancer of the left upper lung lobe. He
has smoked heavily since the age of 14. Which of the following features of cancer of the lung indicates distant spread?
A. Hypercalcemia
B. Cushing-like syndrome
C. Gynecomastia
D. Syndrome of inappropriate secretion of antidiuretic hormone (SIADH)
E. Brachial plexus lesion (Pancoast’s syndrome)

BURNS, WOUND HEALING, GRAFT, AND RECONSTRUCTIVE SURGERY


1. Healing by first intention means
A. Using catgut
B. Union between two edges of an incision without subsequent breakdown
C. Subsequent breakdown
D. The immediate use of a plastic dressing

https://t.me/DentalBooksWorld
Chapter 5 • General Surgery 419

2. Regarding hypertrophic scar and keloid spot, the wrong statement among the following is
A. Hypertrophic scar is common on flexor surfaces B. Keloid is often familial
C. Hypertrophic scar outgrows wound area D. Keloid is common on sternum, shoulder and face
3. A hypertrophic scar
A. Is the same as a keloid scar
B. Requires radiotherapy for a cure
C. Raised red scar which persists for a year or two before becoming white
D. Only occurs on the abdomen
4. Keloids are characterized by the following except
A. Consist of dense overgrowth of scar tissue
B. Develop after wounds, burns, and vaccination marks
C. Are particularly common in Negroes and pregnant females
D. Occur most often on the face, neck, and front of the chest
E. May turn malignant
5. Which topical agent is preferred in electrical burns?
A. Mafenide acetate B. Silver nitrate
C. Cerium nitrate D. All of the above
6. A Wolfe graft is
A. A partial thickness skin graft B. A pinch skin graft
C. A small full thickness skin graft D. A pedicle graft
7. Split-thickness skin grafts differ from full-thickness grafts in all of the following except that they
A. Contain part of the dermis
B. Take even when the conditions of the recipient bed are suboptimal
C. Are less cosmetically acceptable
D. Develop little pigmentation after transfer
8. Which of the following skin grafts has least contraction?
A. Full thickness B. Split thickness
C. Both have equal contraction D. All of the above
9. The aim of treatment of an infant with a cleft lip is to
A. Improve appearance B. Make feeding possible
C. Achieve adequate speech D. Achieve adequate dentition
10. Cleft palate repair is ideal at
A. 6 months B. 6–18 months
C. 12–24 months D. 2.5 years
11. In carcinoma of the tongue, which of the following is incorrect?
A. It is virtually symptomless in its early stages B. One form of presentation is a fissure
C. It may cause earache D. The usual site is the back of the tongue
12. In the rule of nine for calculation of (body surface area) BSA, the entire back accounts for
A. 9% B. 18%
C. 24% D. 36%
13. Using the “rule of nines,” what percent of this patient’s body is burned?
A. 45% B. 18%
C. 27% D. 36%
14. At what initial rate using the Parkland formula, should intravenous lactated Ringer’s solution be administered?
A. 340 cc/hr B. 420 cc/hr
C. 550 cc/hr D. 630 cc/hr
E. 710 cc/hr
15. In extensive burns, which of the following statements is incorrect concerning fluid replacement during the first 24 hours?
A. May consist of colloid
B. Should be controlled by the hourly urine output
C. Should be supplemented by blood transfusion to maintain the hematocrit around 35%
D. Is calculated according to the body weight and the surface area

https://t.me/DentalBooksWorld
420 Triumph’s Complete Review of Dentistry

16. The best solution to combat hypokalemia is


A. Ringer’s lactate solution B. Isotonic sodium chloride solution
C. Darrow’s solution D. 5% Dextrose
17. The most common site of infection in a burn patient is
A. Lungs B. Heart
C. Kidney D. Liver
18. Which of the following influences wound dehiscence?
A. Type of incision B. Type of closure
C. Anemia–hypoproteinemia D. All of the above
19. Steroids are responsible for all of the following except
A. Decreased wound tensile strength B. GI ulceration
C. Gall stone formation D. Avascular necrosis of hip
20. Factors that decrease collagen synthesis include all of the following except
A. Hypoproteinemia B. Infection
C. Anemia D. Advanced age
E. Hypoxia
21. Which of the following parameters is characteristic of burn patients in the first 12 hours?
A. Rise in eosinophil count B. Rise in leucocytes
C. Fall in eosinophil count D. Ulceration of stomach
22. Which is the commonest cause of death in burn patients?
A. Bacteremic shock B. Oligemic shock
C. Fluid loss D. All of the above
23. Which is the primary mediator of inflammatory vascular responses in inflammation?
A. Monocytes B. Serotonin
C. Histamine D. A and C

SALIVARY GLANDS AND SWELLING OF THE NECK


1. Acute bacterial sialadenitis most commonly affects
A. Parotid gland B. Submandibular gland
C. Submaxillary gland D. All equally
2. 80% of all salivary stones occur in
A. Parotid gland B. Submandibular gland
C. Submaxillary gland D. Minor salivary glands
3. Spot the wrong statement
A. 75% of salivary neoplasms arise in parotid
B. 80% of parotid tumors are benign
C. 80% of benign parotid tumors are pleomorphic adenoma
D. All are correct
4. Malignant parotid tumor is characterized by
A. Pain B. Hard and fixed mass
C. Paralysis of facial muscles D. All of the above
5. Salivary carcinoma with poor prognosis is
A. Adenoid cystic carcinoma B. Adenocarcinoma
C. Squamous carcinoma D. Mucoepidermoid carcinoma
6. Salivary carcinoma with perineural spread to brain is
A. Adenocarcinoma B. Adenoid cystic carcinoma
C. Squamous carcinoma D. Acinic cell carcinoma
7. The most common causative organism in acute suppurative parotitis is
A. Streptococcus pyogenes B. Staphylococcus aureus
C. Viral origin D. Streptococcus mitis

https://t.me/DentalBooksWorld
Chapter 5 • General Surgery 421

8. The tumor that commonly metastatize to the parotid gland is


A. Mucoepidermoid carcinoma B. Adenocarcinoma
C. Malignant melanoma D. Adenoid cystic carcinoma
9. Which indicates carcinomatous change in pleomorphic adenoma?
A. Involvement of facial nerve B. Pain
C. Swelling D. All of the above

NECK SWELLINGS
10. Pharyngeal pouch should be suspected in the presence of
A. Dysphagia B. Regurgitation of undigested food
C. Aspiration pneumonitis D. All of the above
11. Regarding a pharyngeal pouch which statement is incorrect?
A. It protrudes through Killian’s dehiscence B. It usually turns to the left side of the neck
C. It may be visible in the neck D. It is twice as common in males as in females
12. Branchial cyst commonly arises from remnant of
A. First branchial cleft B. Second branchial cleft
C. Third branchial cleft D. All of the above
13. Regarding branchial cyst, which statement is incorrect?
A. Arises from the second branchial cleft
B. Usually appears between the ages of 20 and 25 years
C. Protrudes beneath the anterior border of the sternomastoid
D. All are correct
14. Branchial cyst is best differentiated from cold abscess by
A. Fluctuation B. Transillumination
C. Contains cholesterol crystals D. None of the above
15. Characteristics of cystic hygroma include all except
A. Develops from jugular lymph sacs B. Brilliantly translucent
C. Typically occupies the middle third of neck D. Enlarges when the child cries
16. Which of the following is inappropriate to cystic hygroma?
A. It is a type of cavernous hemangioma B. It can be the earliest swelling of the neck to appear in life
C. Obstruct labor D. Brilliant translucent
17. A ranula is a
A. Cystic swelling in the floor of the mouth B. Forked uvula
C. Sublingual thyroid D. Thyroglossal cyst
18. Laryngocele is common to
A. Trumpet blowers B. Glass blowers
C. Patients of chronic cough D. All of the above
19. Tubercular cervical lymphadenitis commonly affects
A. Posterior triangle nodes B. Upper deep cervical nodes
C. Lower deep cervical nodes D. Submaxillary nodes
20. Of the 800 lymph nodes of the body, lymph nodes in the neck is around
A. 100 B. 200
C. 300 D. 400
21. Cervical rib may present by which of the following?
A. Numbness in fingers B. Bruit around clavicle
C. Lump in lower neck D. All of the above
22. Pain in the arm due to cervical rib is caused by
A. Compression of T1 B. Compression of C7
C. Muscle ischemia D. All of the above

https://t.me/DentalBooksWorld
422 Triumph’s Complete Review of Dentistry

23. Potato tumor is a


A. Carotid body tumor B. Sternomastoid tumor
C. Cystic hygroma D. Branchial cyst
24. Which of following is brilliantly translucent?
A. Sebaceous cyst B. Dermoid cyst
C. Cystic hygroma D. Branchial cyst
25. In Hodgkin’s lymphoma, spot the wrong statement
A. It has a better prognosis than NHL
B. It is characterized by the presence of few malignant cells outnumbered by inflammatory cells
C. It is characterized by the presence of Reed–Sternberg cells
D. Diagnostic laparotomy is essential in all patients
26. Most common site of enlargement of lymph nodes in Hodgkin’s lymphoma is
A. Abdominal B. Cervical
C. Axillary D. Mediastinal
27. Most common presentation of Hodgkin’s lymphoma is
A. Leukocytosis B. Fever
C. Painless enlargement of lymph nodes D. Pruritus
28. Hodgkin’s lymphoma with right-sided neck nodes and left inguinal node without fever is of
A. Stage la B. Stage IIIa
C. Stage IIa D. Stage IVa
29. The symptoms of Hodgkin’s lymphoma include all the following except
A. Pruritus B. Headache
C. Weight loss D. Fever
30. In a patient with Hodgkin’s lymphoma with unilateral lymph node involvement, treatment of choice is
A. Irradiation B. Single drug chemotherapy
C. Neck dissection D. Radiotherapy plus chemotherapy
31. Worst prognosis in Hodgkin’s lymphoma is
A. Lymphocytic depletion B. Mixed cellularity
C. Lymphocyte predominance D. Nodular sclerosis

WOUND INFECTION
1. Positive risk factors for wound infection include all except
A. Obesity B. Foreign body
C. Poor surgical technique D. Immunocompromised patients
2. Which of the following has increased rate of wound infection?
A. Clean wounds B. Clean contaminated wounds
C. Contaminated wounds D. None of the above
3. A boil is
A. Any abscess of the skin B. Carbuncle
C. An acute infection of a hair follicle D. An infection of subcutaneous tissue
4. Fournier’s gangrene of the scrotum is caused by all except
A. Clostridia B. Bacteroides
C. Coliforms D. Peptostreptococci
5. Debridement of a wound means
A. Excising 1 mm skin from the edges of a wound
B. Not excising skin but excising all damaged muscle
C. Laying open all layers of a wound and excision of devitalized tissue
D. Delayed primary suture

https://t.me/DentalBooksWorld
Chapter 5 • General Surgery 423

6. The most common organism in abdominal abscess is


A. Staphylococcus B. Pseudomonas
C. E. coli D. Proteus
7. The most frequent organism associated with neck abscess in childhood is
A. Mononucleosis B. Cat scratch
C. Staphylococcus aureus D. Mycobacteria
8. Which statement is wrong concerning Clostridium tetani?
A. Is a Gram-negative bacillus B. Has a drum-stick appearance
C. Is a strict anaerobe D. Produces highly resistant spores
9. The tetanospasmin acts by
A. Direct stimulation of motor endplate B. Stimulation of anterior horn cells
C. Inhibition of cholinesterase D. Inhibits GABA
10. The main organism of endotoxin release in multiple organ dysfunction syndrome is
A. Proteus vulgaris B. E. coli
C. Pseudomonas D. Acinetobacter
11. The most important factor influencing prognosis in multiple organ failure is
A. Patient’s age B. Number of organs that have failed
C. The initiating process D. Sequence in which organs failed
12. Which of the following is most rewarding in management of contaminated wounds?
A. Adequate debridement B. Broad spectrum antibiotic
C. Skin grafting D. Vasodilators
13. The prodromal symptoms of tetanus include the following except
A. Restlessness and sleeplessness B. Rigidity or muscular twitches in the region of the wound
C. Stiffness and twitches of the jaw muscles D. High fever
14. The poorest prognosis in tetanus is when
A. Spasms are violent and continuous B. Spasms appearing within 48 hours of trismus
C. Contamination of large wound D. Spasms brought about by skin stimulus
15. In a patient of tetanus with severe spasms and cyanosis, the first thing to be done is
A. Thorough wound debridement and heavy dose of penicillin
B. Human tetanus immunoglobulin
C. Tracheostomy, muscle relaxant, and ventilator
D. Sedation and nasogastric suction
16. Following active immunization with tetanus toxoid injection, when should a tetanus toxoid booster be given?
A. Every year B. Every 2 years
C. Every 5 years D. Every 10 years
17. A patient who has recovered from tetanus
A. Should be given tetanus toxoid B. May be given tetanus toxoid
C. Toxoid not required as infection itself induces immunity D. None of the above
18. Gas gangrene can be caused by which of the following?
A. Clostridium welchii (perfringens) B. Clostridium histolyticum
C. Clostridium novyi D. All of the above
19. The most lethal exotoxin of C. welchii is
A. Hyaluronidase B. Collagenase
C. Hemolysin D. Lecithinase (A. toxin)
20. Indicate the wrong statement about gas gangrene
A. Is caused by anaerobic spore-bearing clostridia B. Has a very short incubation period (1–2 days)
C. Is associated with high fever D. May be associated with anemia and jaundice
21. Which type of tetanus is most fatal?
A. Acute tetanus B. Cephalic tetanus
C. Bulbar tetanus D. Latent tetanus

https://t.me/DentalBooksWorld
424 Triumph’s Complete Review of Dentistry

22. Predisposing factors for gas gangrene


A. Diabetes B. Occlusive arterial diseases
C. Traumatic injuries D. All of the above
23. Regarding gas gangrene, choose the correct answer
A. It is due to Clostridium botulinum infection
B. Clostridial species are Gram-negative spore forming anaerobes
C. The clinical features are due to the release of protein endotoxin
D. Surgical debridement and antibiotics are an essential part of treatment
24. Most common infection of the hand is
A. Acute paronychia B. Web space infection
C. Palmar abscess D. Felon
25. Gas gangrene is not associated with which of the following?
A. Deep wounds of the thigh and buttock B. Short incubation period
C. Slight fever D. High leucocytosis
26. The hand infection which carries the highest risk of developing osteomyelitis
A. Paronychia B. Intrathecal whitlow
C. Distal pulp space infection (felon) D. Web space infection
27. Kanavel’s sign is
A. Swelling above the flexor retinaculum
B. Flexion of the thumb when the radial bursa is infected
C. Flexion of the fingers in a compound palmar ganglion
D. Tenderness over an infected ulnar bursa between the transverse palmar creases
28. Which of the following is false regarding carbuncle?
A. Infective gangrene of subcutaneous tissue B. Caused by staphylococcus
C. Diabetics are more prone D. Caused by Streptococcus
29. False regarding erysipelas is
A. Streptococcal infection B. Usually extends to deeper soft tissues
C. Margins are raised D. Flaccid bullae may develop
30. Regarding surgical antimicrobial prophylaxis, all the following are true except
A. It is necessary in clean wound
B. It should be given 30 minutes before skin incision
C. Discontinuation of the antibiotic within 24 hours after surgery is recommended
D. Indiscriminate use of antimicrobials can lead to the development of antibiotic resistant microorganisms
31. The following measures are important contributing factors in controlling bacterial wound contamination in the
operating room, except
A. Appropriately timed preoperative antibiotic prophylaxis
B. Scrubbing, gowning, and gloving, refined and aseptic techniques
C. Surgical site, hair shaving in the evening prior operation
D. Antimicrobial skin preparation
32. Which is the distinguishing feature between a true erysipela and a cellulitis? In erysipelas
A. Rosy red rash disappears on pressure with a sharply defined margin
B. Vesicles containing serum
C. Spread into pinna
D. All of the above

TUMOR, CYST, ULCER, AND SINUSES


1. Which of the following is a sequestration dermoid?
A. Dermoid of testes B. Epidermoid
C. External angular D. Retroperitoneal dermoid

https://t.me/DentalBooksWorld
Chapter 5 • General Surgery 425

2. A sequestration dermoid cyst is


A. Due to squamous cells being driven in by a needle B. Due to cells being buried during development
C. May affect limbs D. A variety of sebaceous cyst
3. The wall of a True cyst can be lined by all except
A. Epithelium B. Granulation tissue
C. Endothelium D. All of the above
4. A decubitus ulcer is
A. A venous ulcer B. An ulcer in the region of the elbow
C. A pressure sore D. An ulcer of the tongue
5. The following statements about bed-sores (Decubitus ulcers) are correct except
A. Occur in bed-ridden patients B. Usually involve pressure points on the back
C. Result from pressure ischemia and sloughing D. Never penetrate deeply to involve muscle and bone
6. Marjolin’s ulcer is
A. Tuberculous ulcer B. Fungal ulcer
C. Scar carcinoma D. Syphilitic ulcer
7. All are features of ulcer except
A. Slow growth B. Pain
C. Associated with chronib scar D. Lymph node involvement is a late feature
8. A sinus or fistula will be persistent in presence of
A. Inefficient drainage B. Foreign body or necrotic tissue
C. Epithelialization of the wall D. All of the above
9. What is not true of hamartomas?
A. Overgrowth of tissue at abnormal location B. Overgrowth of normal tissue at normal location
C. Virtually benign D. Common example is benign mole
10. Characteristic features of lipomas are all except
A. Slowly growing B. Universal tumor
C. No definite edge D. Most are painless
11. Lipoma which undergo malignant degeneration is
A. Retroperitoneal B. Subserosal
C. Subfascial D. Submucosal
12. All of the following are locally invasive tumors except
A. Basal cell cancer B. Mixed salivary tumors
C. Melanoma D. Bronchial adenoma
13. A genetic predisposition to cancer is related to
A. Increased susceptibility to carcinogens B. Increased chromosomal fragility
C. Defective tumor suppressor gene D. All of the above
14. Cancers commonly metastasizing by blood are all except
A. Thyroid B. Breast
C. Lung D. Brain
15. The treatment of a primary malignant melanoma of the skin is
A. Wide excision B. Radiotherapy
C. Cytotoxic therapy D. Immunotherapy
16. Which statement is untrue concerning malignant melanoma?
A. Is common in children and Negroes B. Usually occurs between the ages of 50 and 60 years
C. May arise de novo or in a benign pigmented nevus D. Always carries a bad prognosis
17. Malignant melanoma is characterized by the following except
A. Rarely arises from hair-bearing nevi B. Frequently arises from junctional nevi
C. Is rare in the black races D. Is radiosensitive

https://t.me/DentalBooksWorld
426 Triumph’s Complete Review of Dentistry

18. The prognosis of patients with malignant melanoma depends on


A. Depth of invasion B. Clinical stage of the disease
C. Location of the tumor D. All of the above
19. Nerve commonly affected by plexiform neurofibromatosis is
A. Glossopharypngeal B. Peripheral
C. Trigeminal D. Facial
20. Carcinoma arising from sebaceous cyst is
A. Squamous cell carcinoma B. Basal cell carcinoma
C. Carcinoma in situ D. None of the above
21. Ulcer due to infection by vincents organisms is
A. Trophic ulcer B. Rodent ulcer
C. Tropical ulcer D. Marjolins ulcer
22. Ulcer with shelving edge
A. Tuberculous ulcer B. Non- specific ulcer
C. Syphilitic ulcer D. All of the above
23. Which type of melanoma is least malignant?
A. Lentigo maligna B. Nodular
C. Acral D. Amelanotic

NEOPLASIA
1. The clinical situations which are most likely to cause death in the terminal stages of carcinoma of the tongue
include
A. Hemorrhage B. Bronchopneumonia
C. Malnutrition D. Edema of the glottis
2. Regarding carcinoma of the tongue, all of the following are True except
A. Ankyloglossia occurs when a carcinoma begins on the dorsum of the tongue
B. It may simply present with a lump in the neck
C. Alteration of the voice is an early feature of carcinoma of the back of the tongue
D. In many instances the lymphatics draining the anterior two-thirds of the tongue and the floor of the mouth traverse
the periosteum of the mandible
3. Regarding carcinoma of the lip, all are True except
A. If occurring at the angle of the mouth, tends to be more malignant in behavior than carcinoma of the upper or
lower lip
B. May be confused with a keratoacanthoma
C. Is curable by surgery
D. Carries a 40% 5-year survival rate if seen in its early stages
4. Regarding cleft lip and cleft palate, all of the following are True except
A. The condition is familial in about 12% of cases
B. Clefts on the left greatly outnumber those on the right
C. Cleft lip interferes with feeding
D. 50% of children with cleft palate have some degree of deafness
5. Regarding carcinoma of the lip all are true except
A. It is most frequently squamous cell carcinoma
B. It occurs most frequently in the lower lip
C. It is radioresistant
D. It is usually low grade and well differentiated
E. It metastasizes via lymphatics
6. Which of the following is true regarding carcinoma of the tongue?
A. Is usually an adenocarcinoma
B. Is more common in females
C. Most commonly presents as an indolent ulcer
D. Metastasis via the blood stream at an early stage of the disease

https://t.me/DentalBooksWorld
Chapter 5 • General Surgery 427

7. The following factors increase the risk of developing gastric cancer


A. Male sex
B. High socioeconomic group
C. A diet high in vegetables and fruits
D. Japanese ethnicity
8. All of the following are true of colorectal cancer, except
A. There is an association with high red meat consumption
B. There is an association with low intake of saturated fatty acids
C. There is an association with low dietary fiber
D. The majority of tumors arise in the recto-sigmoid region
E. Patients with familial polyposis will usually present with carcinoma before the age of 40
9. In malignant melanoma
A. Breslow’s thickness is a better prognostic indicator than Clark’s levels
B. May not arise in a pre-existing nevus
C. The most common type is nodular
D. If thicker than 1 mm should be excised with a margin of 1 cm
10. Hodgkin’s disease
A. Has a unimodal age distribution
B. Commonly presents with lymphadenopathy below the diaphragm
C. Systemic or symptoms such as weight loss, fever, and night sweats are seen in 30% of cases
D. The mixed cellularity type has better prognosis than lymphocytic predominance
11. In non-Hodgkin’s lymphoma
A. It is more localized at the time of presentation than Hodgkin’s disease
B. Serum lactate dehydrogenase is a prognostic factor
C. Few patients have bone marrow involvement
D. The stomach is not the most common extranodal site of presentation
12. Regarding primary gastrointestinal lymphoma, all of the following are true except
A. Is usually of non-Hodgkin’s type
B. Primary gastrointestinal lymphoma is rare but is the most common extranodal presentation of the disease
C. Is usually of low grade
D. The stomach is the most common site of affection followed by small intestine
13. Origin of bone is from
A. Ectoderm B. Mesoderm
C. Endoderm D. All of the above
14. Acute osteomyelitis is commonly caused by
A. Staphylococcus aureus B. S. pyogenes
C. H. influenzae D. Salmonella
15. Acute osteomyelitis usually begins at
A. Epiphysis B. Metaphysis
C. Diaphysis D. Any of the above
16. What is not true of acute pyogenic osteomyelitis?
A. Trauma is a predisposing factor B. Common infecting agent is Staphylococcus aureus
C. Infection is usually blood borne D. All are true
17. What is not true of Brodie’s abscess?
A. A form of chronic osteomyelitis B. Intermittent pain and swelling
C. Common to diaphysis D. Excision is very often required
18. Tuberculosis of the spine most likely originates from
A. Intervertebral disc B. Cancellous vertebral body
C. Ligamentous structures D. Paravertebral soft tissue
19. In Pott’s spine, the disease starts in the
A. Intervertebral disc B. Anterior vertebral margin
C. Posterior vertebral margin D. Paravertebral soft tissue

https://t.me/DentalBooksWorld
428 Triumph’s Complete Review of Dentistry

20. Melon seed bodies in joint fluid are characteristic of


A. Rheumatoid arthritis B. Tuberculous arthritis
C. Septic arthritis D. None of the above
21. The earliest sign of TB hip in X-ray is
A. Narrow joint space B. Irregular moth eaten femoral head
C. Periarticular osteoporosis D. Dislocation
22. Healing of tuberculous arthritis can lead to
A. Calcification B. Fibrous ankylosis
C. Boney ankylosis D. None of the above
23. Osteoid osteoma originates from
A. Periosteum B. Cortex
C. Medullary cavity D. All of the above
24. What is not true of osteoclastoma?
A. Arise from epiphysis B. Soap bubble appearance
C. Common in men around 40 D. Pathological fracture is common
25. What is true of giant cell tumor?
A. Rare in third decade B. Best treated by local excision
C. Soap bubble appearance in X-ray D. All of the above
26. What is not true of osteogenic sarcoma?
A. X-ray is diagnostic in 50% of cases B. New bone formation is universal
C. Usually metastasizes by blood stream D. Usually affects teenagers
27. Osteogenic sarcoma may occur in
A. Fibrous dysplasia B. Nonossifying fibroma
C. Paget’s disease D. None of the above
28. Sun ray appearance of osteosarcoma is because of
A. Periosteal reaction B. Osteonecrosis
C. Calcification along vessels D. None of the above
29. Which of these statements are incorrect concerning chondrosarcoma?
A. It occurs mainly in middle-aged persons B. It is rare in children
C. It has a predilection for flat bones D. It does not metastasize to the lung
30. The most common site of Ewing’s sarcoma is
A. Femur B. Tibia
C. Radius D. Ribs
31. The characteristic X-ray feature of Ewing’s sarcoma is
A. Sunburst appearance B. Prominent Codman’s triangle
C. Onion layering D. Osteosclerosis
32. Ewing’s sarcoma arises from which of the following?
A. Squamous cells B. Mesothelial cells
C. Endothelial cells D. None of the above
33. Ewing’s sarcoma can be confused histologically with which of the following?
A. Osteomyelitis B. Osteosarcolma
C. Giant cell tumor D. Myeloma
34. Bone metastasis can be best evaluated by
A. X-ray B. 99mTC bone scan
C. 111-Indium scan D. Calcium-alkaline phosphatase elevation
35. The synonym for Paget’s disease is
A. Osteitis fibrosa B. Osteitis proliferans
C. Osteitis deformans D. None of the above

https://t.me/DentalBooksWorld
Chapter 5 • General Surgery 429

VASCULAR DISEASES
1. Claudication is
A. Pain at rest B. Pain relieved by rest
C. Constant pain D. Pain not relieved by rest
2. Rest pain refers to pain in
A. Anywhere in the body at rest B. In the thigh of the patient with Buerger’s disease
C. In the back D. In the foot of a patient of severe vascular disease
3. Patients with chronic ischemia can present clinically in four different grades. Which of the following statements is
not correct?
A. In grade I, the patient is asymptomatic because of well-developed collaterals
B. In grade II, the patient develops intermittent muscle pain only with exercise
C. In grade Ill, severe diabetic peripheral neuropathy results in rest pain
D. In grade IV, the blood flow is critically low and cannot keep tissues alive
4. By definition, which of the following patients is having critical limb ischemia?
A. A patient presenting with acute limb ischemia and impending gangrene
B. A patient presenting with a chronic ischemic foot ulcer and rest pain
C. A patient presenting with calf claudication progressing over the past 3 months
D. A patient presenting with infective gangrene of his toe and intact pedal pulse
5. All the following statements describe ischemic rest pain except
A. Continuous severe aching, or burning pain that becomes worth at night
B. Sudden cramping pain in the calf that awakens the patient from sleep
C. Partially relieved by putting the leg below the level of the heart
D. Increases if there is super added infection in the ischemic foot
6. Which one of the following statements describes an ischemic ulcer?
A. Superficial painful ulcer above the ankle surrounded with pigmented eczematous skin
B. Deep painless ulcer in the sole reaching down to the bone with intact pedal pulse
C. Superficial painful ulcer on the heal of a diabetic patient with absent pedal pulse
D. Deep painless ulcer between the toes of a diabetic patient with gangrenous floor and intact pedal pulse
7. Normally, the ankle or brachial index is above 1. In the absence of arterial calcification, the diagnosis of critical limb
ischemia is made when the ankle or brachial index is
A. Less than or equal to 1 B. Less than or equal to 0.7
C. Less than or equal to 0.5 D. Less than or equal to 0.2
8. In diabetic patients with critical limb ischemia, the ankle/brachial index may be falsely high because
A. Diabetic patients may develop good arterial collaterals
B. Diabetic patients may develop calcification of leg arteries
C. Diabetic patients may develop early atherosclerosis
D. Diabetic patients may develop concomitant DVT
9. All of the following statements concerning popliteal artery aneurysms are true except
A. Approximately 50% are associated with aneurysms at other sites
B. Rupture into the popliteal space is a frequent complication
C. Associated thrombosis carries a high risk of amputation
D. Associated distal embolization may result in tissue loss
10. A diabetic patient presented with advanced ischemic gangrene of the foot and rest pain. On examination, the pedal
pulses were absent but the popliteal pulse was felt. Angiography showed occlusion of the tibial arteries with no distal
run-off. The classical treatment for this patient is
A. Popliteal-to-distal bypass B. Syme’s amputation
C. Below knee amputation D. Above knee amputation
11. In lower limb acute embolic ischemia, the embolus may originate from all of the following sites except
A. The heart over recent myocardial infarction
B. The deep veins from extensive lower limb DVT
C. The heart with valve disease and atrial fibrillation
D. The abdominal aorta with aortic aneurysm

https://t.me/DentalBooksWorld
430 Triumph’s Complete Review of Dentistry

12. Diabetic patients are more prone to develop foot ulcers. All the following are important contributing causes
except
A. Diabetic patients usually have peripheral neuropathy affecting their feet
B. Diabetic foot deformities render the foot more susceptible to trauma
C. Diabetic patients have exaggerated inflammatory response to infection
D. Diabetic patients may have concomitant chronic ischemia
13. If a diabetic patient presents with gangrene of one of his toes, the first thing that should be done is to
A. Amputate the gangrenous toe to prevent spread of gangrene
B. Ask for fasting blood sugar to control blood sugar level
C. Ask for serum lipid profile to correct possible hyperlipidemia
D. Look for pedal pulses to evaluate foot circulation
14. Once you diagnose acute lower limb ischemia, the first thing to do is
A. Angiography to differentiate between embolic and thrombotic ischemia
B. Catheter-directed embolectomy under local anesthesia
C. Give the patient heparin to avoid clot propagation
D. Transfer the patient to a vascular surgery center
15. Which one of the following is not a component of “Leriche Syndrome”?
A. Bilateral absent femoral pulse B. Bilateral feet paresthesia
C. Buttock and thigh claudication D. Impotence
16. In acute ischemia, catheter-directed thrombolysis can be used in all of the following conditions except
A. Recent acute thrombosis of less than 3 days duration
B. Viable limbs with lax muscles and intact sensations
C. Patient with history of major surgery one and a half months ago
D. Patient with history of cerebral stroke one and a half months ago
17. All the following can result in acute limb ischemia, except
A. Embolism originating from the heart with chronic atrial fibrillation
B. Acute hemolysis of RBCs in a patient with known spherocytosis
C. Thrombosis of a diseased artery on top of chronic lower limb ischemia
D. Traumatic fracture of bones with injury to nearby arteries
18. In acute ischemia, one of the following is a sign of irreversible ischemia that will need amputation
A. Marked delay in the capillary refilling time
B. Marked swelling and turgidity of calf muscles
C. Loss of foot superficial and deep sensation
D. Paralysis of the small muscles of the foot
19. Reconstructive arterial surgery is recommended for patients with following manifestations of ischemia except
A. Ischemic neuropathy B. Trophic ulceration
C. Toe gangrene D. Claudication
20. The most satisfactory graft material for femoropopliteal bypass grafting is
A. Autogenous vein graft B. Woven dacron
C. Knitted dacron D. Gor-Tex
21. In arterial bypass surgery the best vein to use is
A. Cephalic vein B. Femoral vein
C. Long saphenous vein D. Short saphenous vein
22. The most common site at which arterial emboli lodge is the
A. Aortic bifurcation B. Common iliac bifurcation
C. Common femoral bifurcation D. Cerebral circulation
23. Fogarty catheter is used for
A. IV nutrition B. Ureteric catheterization
C. Arteriography D. Arterial embolectomy
24. In acute embolic occlusion, heparin is given to
A. Dissolve embolus B. Reduce extension
C. Maintain patency of distal vessels D. All of the above

https://t.me/DentalBooksWorld
Chapter 5 • General Surgery 431

25. The most important prognostic sign of acute ischemia of a limb is


A. Pallor B. Cold skin
C. Cutaneous anesthesia D. Muscular paralysis
26. The most frequent cause of arterial embolism is
A. Mitral valve disease B. Arterial fibrillation
C. Myocardial infarction D. Aortic aneurysm
27. Indicate the Incorrect statement about arterial embolism
A. It results in acute ischemia
B. Is always due to a detectable site of thrombosis
C. Is often due to lodgement of an embolism at the bifurcation of a main artery
D. Is associated with a much higher incidence of gangrene than simple ligation of the same artery
E. Tends to induce reflex spasm and secondary thrombosis in the distal arterial tree
28. Intra-arterial thrombolysis is best achieved with
A. Streptokinase B. Urokinase
C. tPA infusion D. Pulse-spray tPA (tissue plasminogen activator)
29. In fat embolism, the fat is most likely to arise from
A. Bone marrow B. Adipose tissue
C. Chylomicrons D. None of the above
30. Therapeutic embolization is used for
A. Arrest of hemorrhage from GI and urinary tract B. Treatment of A-V malformation
C. Shrinkage of tumor growth D. All of the above
31. Moist gangrene occurs i the presence of
A. Venous and arterial occlusion B. Embolism
C. Diabetes D. All of the above
32. Bowel strangulation is an example of
A. Dry gangrene B. Moist gangrene
C. Mixture of both D. None of the above
33. Peripheral aneurysm most commonly involves
A. Femoral artery B. Axillary artery
C. Popliteal artery D. Brachial artery
34. In subclavian steal syndrome, blood is drawn from which artery?
A. Internal caroti B. External carotid
C. Vertebral D. Subclavian
35. Lumbar sympathectomy is of value in the management of
A. Intermittent claudication B. A-V fistula
C. Diabetic neuropathy D. Distal ischemia affecting skin of toes
36. Sympathectomy is not advised for
A. Raynaud’s disease of the fingers B. Hyperhidrosis of the feet
C. Intermittent claudication D. Acrocyanosis
37. Today the commonest cause of a true aneurysm is
A. Congenital B. Syphilitic
C. Atherosclerosis D. A gunshot wound
38. An overdose of Heparin is treated by
A. Prostaglandins B. Phenindione
C. Protamine sulfate D. Prostigmine
39. Which statement is incorrect concerning Buerger’s disease?
A. Occurs most often in young males B. Is particularly common in heavy smokers
C. Is a segmental occlusive disease of both arteries and veins D. Affects large arteries only
40. In Buerger’s disease, the following statements are correct except
A. Phlebitis migrans may occur B. Intermittent claudication is the most common symptoms
C. Rest pain is a grave symptom D. Only the lower limbs are affected

https://t.me/DentalBooksWorld
432 Triumph’s Complete Review of Dentistry

41. Which is the earliest symptom in Buerger’s disease?


A. Phlebitis migrans B. Intermittent claudication
C. Rest pain is a grave symptom D. There is increased susceptibility to cold
42. In Buerger’s disease, the following statements are true except
A. Severe pain never occurs
B. Recurrent exacerbations and remissions are characteristic
C. May end in gangrene
D. The arteriographic findings are often diagnostic
43. Which statement is untrue concerning Raynaud’s disease?
A. Occurs equally in both sexes
B. Usually manifests itself in the third decade
C. Affects the fingers and hands and rarely the feet
D. Is characterized by recurrent episodes initiated by cold or emotional stress
44. Concerning abdominal aortic aneurysms, which of the following statements is untrue?
A. Are commonest in the sixth and seventh decades
B. have a much higher incidence in males than females
C. Characteristically originate below the renal arteries
D. Never extend beyond the aortic bifurcation
E. Are often associated with hypertension
45. The cardinal signs and symptoms of sudden peripheral ischemia include
A. Pain B. Pallor
C. Paresthesia D. Paralysis
E. All of the above
46. Cause of A-V fistula
A. Congenital B. Blunt trauma
C. Penetrating trauma D. All of the above
47. The hemodynamic effects of an arteriovenous fistula in the thigh include the following except
A. Decreased peripheral resistance B. Reduced cardiac output
C. Increased venous pressure D. Reduced diastolic pressure
48. Branham’s sign refers to
A. Bruit heard over an arteriovenous fistula
B. The increased growth of the limb involved with the arteriovenous fistula in a child
C. The occurrence of heart failure where there is a large arteriovenous fistula present
D. A sharp decrease in the pulse rate when the arteriovenous fistula is closed by digital pressure
E. All of the above
49. The Adson maneuver for thoracic outlet syndrome is positive with
A. Numbness and tingling in the hand B. Pallor of the hand
C. Disappearance of radial pulse D. Acrocyanosis in the hand
50. A Seldinger needle is used for
A. Liver biopsy B. Suturing skin
C. Breast biopsy D. Arteriography
E. Lymphangiography
51. Regarding acute limb ischemia, all are true except
A. Arterial occlusion has less dramatic effects in those with underlying occlusive disease
B. Occurs more often in patients with AF
C. Objective sensory loss requires urgent treatment
D. Fixed skin staining is an early sign
52. The term venous pump refers to
A. A part of autotransfusion apparatus
B. The left atrium
C. The apparatus for rapid transfusion of blood
D. Presence of valves in the IVC
E. Musculofascial anatomy and physiology of calf
https://t.me/DentalBooksWorld
Chapter 5 • General Surgery 433

53. Pressure in superficial veins in leg during standing is


A. 20 mmHg B. 40 mmHg
C. 80 mmHg D. 120 mmHg
54. What happens to superficial venous pressure during exercise?
A. Rises B. Falls
C. Unchanged D. None of the above
55. Regarding varicose veins all statements are true except
A. Occurs only in the legs B. Are dilated lengthened veins
C. Are results of valvular incompetence D. May follow venous thrombosis
56. All the following are venous symptoms except
A. Aching B. Leg cramps
C. Palpitation D. Tiredness
57. The best operation for varicose vein is
A. Multiple subcutaneous ligations
B. Injecting sclerosants throughout
C. Sub-fascial ligatures
D. Division or ligation at sites of communication from deep to superficial system
58. The Brodie–Trendelenburg test is used to detect
A. The presence of deep femoral vein thrombosis
B. The integrity of the long saphenous vein
C. The presence of an incompetent valve at the saphenofemoral junction
D. The presence of valves in the inferior vena cava
59. A patient is receiving anticoagulation for treatment of deep venous thrombosis. Which of the following findings
suggest that he is given warfarin?
A. Anticoagulation is monitored by measuring the prothrombin time
B. Anticoagulation can be reversed by giving protamine sulfate
C. Anticoagulation is achieved within 1 hour of drug administration
D. The anticoagulant is administered by subcutaneous injection
60. A patient with primary varicose veins and normal deep system can present by any of the following except
A. Discomfort on prolonged standing B. Lower limb muscle cramps
C. Severe lower limb edema D. Venous ulcer
61. A patient with dilate tortuous veins over the lower abdomen filling predominantly from below upward. This clinical
finding can be associated with
A. Chronic superior vena cava obstruction B. Chronic inferior vena cava obstruction
C. Chronic portal hypertension D. Chronic pelvic congestion
62. In a patient with chronic venous insufficiency due to residual deep venous obstruction, which of the following
describes the clinical picture of his limb?
A. Soft pitting edema with pigmentation and eczema B. Hard pitting edema without pigmentation and eczema
C. Soft pitting edema without pigmentation and eczema D. Hard pitting edema with pigmentation and eczema
63. Which of the following statements describes infected venous ulcer?
A. Painless superficial ulcer with skin pigmentation around it
B. Painful superficial ulcer with absent peripheral pulsations
C. Painless deep ulcer with absent peripheral pulsations
D. Painful superficial ulcer with skin pigmentation around it
64. Severe prolonged venous congestion of the lower limbs may result in an acute inflammatory condition known as acute
lipodermatosclerosis. The main line of treatment in this condition is
A. Anti-inflammatory drugs and external compression B. Broad spectrum antibiotic and external compression
C. Anti-inflammatory drugs and broad spectrum antibiotic D. Venotonic drugs and broad spectrum antibiotic
65. Which of the following best describes a healing ulcer?
A. Small ulcer with undermined edge and red granulating floor
B. Large ulcer with sloping edge and red granulating floor

https://t.me/DentalBooksWorld
434 Triumph’s Complete Review of Dentistry

C. Small ulcer with raged edge and bleeding floor


D. Large ulcer with raised edge and necrotic floor
66. One of the complications of Iv varicose veins is acute superficial thrombophlebitis. The main line of treatment for this
condition is
A. Systemic anti-inflammatory drugs and external compression
B. Local anti-inflammatory ointments and external compression
C. Systemic broad spectrum antibiotic and external compression
D. Local antibiotic ointments and external compression
67. The main aim of investigations in a patient with chronic venous insufficiency is to
A. Identify coagulation defects and hypercoagulable states
B. Identify major systemic problems that may alter the operative decision
C. Identify venous reflux and confirm deep venous patency
D. Identify congenital arteriovenous fistula to avoid operating on them
68. The accurate dynamic investigation used to evaluate superficial and deep venous reflex is
A. Pocket Doppler B. Ascending venography
C. Duplex ultrasound D. Descending venography
69. A 33-year-old healthy male presented with symptomatic varicosity of the long saphenous vein. Duplex ultrasound
showed severe saphenofemoral reflux normal deep venous system and competent leg perforators. What treatment
would you advise?
A. Saphenofemoral disconnection (Trendelenburg operation)
B. Stripping of the long saphenous in the thigh only
C. Stripping of the long saphenous in the leg and thigh
D. Elastic compression stocking
70. Which of the following clinical presentations is not compatible with acute lower limb deep venous thrombosis?
A. Swollen, painful, cold, and deeply cyanosed limb B. Swollen, cyanosed painful limb with fever
C. Swollen, pale, and painful limb D. Painful, cold, and cyanosed limb
71. In acute deep venous thrombosis, the recommended treatment protocol is
A. Low-molecular-weight heparin for the whole duration of treatment
B. Initial heparin anticoagulation followed by warfarins
C. Oral anticoagulants for the whole duration of treatment
D. Initial heparin anticoagulation followed by low dose salicylates
72. Thrombolytic therapy is effective in all of the following patients except
A. Chronic iliofemoral deep venous thrombosis with severe edema
B. Massive pulmonary embolism with acute pulmonary hypertension
C. Acute deep venous thrombosis with impending venous gangrene
D. Acute arterial thrombosis with a viable non critical limb ischemia
73. Spontaneous superficial thrombophlebitis occurs in
A. Polycythemia B. Polyarteritis
C. Buerger’s disease D. Internal cancer
E. All of the above
74. The appropriate management of thrombophlebitis of superficial veins is
A. Supportive bandages and ambulation B. Supportive bandages and strict bed rest
C. Anticoagulants and bed rest D. Anticoagulants and ambulation
75. High-risk patient for DVT include
A. Neoplasia B. Pelvic surgery
C. Congestive heart failure D. Lower extremity trauma
E. All of the above
76. Postoperative deep venous thrombosis is suspected from the following except
A. Unexplained postoperative fever or tachycardia
B. Pain in the sole or calf
C. Swelling or edema of the calf or leg
D. Pain on plantar flexion of the foot
E. Color changes in the skin of the leg

https://t.me/DentalBooksWorld
Chapter 5 • General Surgery 435

77. Phlegmasia alba dolens or white leg is due to


A. Lymphatic obstruction
B. Femoral deep vein thrombosis
C. Thrombosis of I VC and lymphatic obstruction
D. Iliofemoral vein thrombosis plus lymphatic obstruction
E. None of the above
78. Phlegmasia cerulea dolens or painful purple swelling is characterized by all the following except
A. Extensive iliofemoral thrombosis B. Propagation of the thrombosis proximally and distally
C. Reduced arterial inflow D. Treated by anticoagulants
79. The treatment of phlegmasia cerulea dolens is
A. Nearly always surgical B. Anticoagulation
C. Low-molecular-weight dextran D. Supportive
80. The commonest cause of fatal pulmonary embolism is
A. Iliac vein thrombosis B. Thrombophlebitis of the femoral vein
C. Calf vein thrombosis D. Axillary vein thrombosis
E. Phlebitis migrans
81. Raynaud’s disease
A. Is caused by an abnormal sensitivity of skin vessels to cold
B. Is marked by a characteristic pallor of the hands after cold stimulation followed by blue and then red color changes
C. May be associated with scleroderma
D. Is permanently relieved by sympathectomy in the vast majority of cases
E. Both A and B
82. Percutaneous transluminal angioplasty (PTA)
A. Ideal fort short segment stenosis
B. Causes a higher bleeding complication rate than angiography
C. Dissection of the artery can occur
D. Arterial thrombosis may occur with PTA
E. All the above
83. What is true of intermittent claudication?
A. Will progress to gangrene rapidly if not treated quickly
B. May improve with exercise
C. Claudication distance remains the same irrespective of walking, climbing, or load carrying
D. Caused by muscle ischemia
E. Both B and D
84. Following conditions are associated with Raynaud’s phenomenon
A. Systemic lupus erythematosus and thoracic outlet syndrome
B. Thromboangiitis obliterans
C. Occupation, e.g., vibrating tools
D. Drugs, e.g., ergotamine
E. All the above
85. Buerger’s disease
A. Affects women more than men B. Has a strong association with cigarette smoking
C. Affects lower limbs only D. Can affect nerves and veins
E. Both B and D
86. The following are characteristic of the feet of diabetic patients
A. Diabetic foot disease can be caused by presence of arteriopathy, neuropathy, infection, or any combination of these
three factors
B. Gangrene occurs even in the presence of palpable foot pulses
C. Ulcers are usually painless
D. Ulcers usually occur around the gaiter area
E. All the above

https://t.me/DentalBooksWorld
436 Triumph’s Complete Review of Dentistry

87. Venous ulcers


A. Are painful B. Occur in the gaiter area
C. Can occur in absence of varicose veins D. Caused by lipodermatosclerosis
E. All the above

GALL BLADDER AND BILE DUCTS


1. Causes of choledochal cyst include all the following except
A. Due to both intrinsic defect and obstruction B. Congenital anomaly of bile duct
C. Inspissated bile D. Pancreaticobiliary anatomical anomaly
2. Predisposing factors for cholelithiasis include
A. Stasis B. Obstruction
C. Inflammation D. All of the above
3. Pigment rather than cholesterol gallstones are more common in the following situations except
A. Morbid obesity B. Cirrhosis
C. Hereditary spherocytosis D. Biliary tract infection
4. In hemolytic anemia, gallstones are
A. Oxalate stone B. Mixed stone
C. Cholesterol stone D. Pigment stone
5. The “Fs” for predilection to cholelithiasis are all except
A. Fertile B. Flatulent
C. Family history D. Fat
6. The common type of gall stone is
A. Cholesterol stone B. Mixed stone
C. Pigment stone D. All of the above
7. Regarding pure cholesterol stones all are true except
A. Single B. Multiple
C. Light in weight D. Radio-opaque
8. Regarding stones in the gallbladder
A. Pigment stones are the most common
B. Are not a risk factor for the development of gallbladder carcinoma
C. 90% of gallstones are radio-opaque
D. A mucocele of the gallbladder is caused by a stone impacted in Hartmann’s pouch
9. The rarest type of gall stone is
A. Cholesterol stone C. Mixed stones
B. Pigment stone D. Calcium carbonate stone
10. Constituents of Saint’s triad are all except
A. Gall stones B. Crohn’s disease
C. Colonic diverticulosis D. Hiatus hernia
11. The primary investigation of choice in gall stones is
A. Plain X-ray B. Oral cholecystography
C. Ultrasonography D. IV cholangiography
12. The investigation of choice in acute cholecystitis is
A. US B. Oral cholecystography
C. Radionuclide scan (HIDA scan) D. None of the above
13. Gallstones are radio-opaque in
A. 5% B. 10%
C. 20% D. 40%
14. Approach to asymptomatic gall stone disease is
A. Immediate surgery B. Elective surgery
C. Wait and watch D. Ursodeoxycholic acid

https://t.me/DentalBooksWorld
Chapter 5 • General Surgery 437

15. Positive Boas’ sign is suggestive of


A. Acute appendicitis B. Acute pancreatitis
C. Acute cholecystitis D. All of the above
16. Regarding acalculous cholecystitis, all the following are true except
A. The disease is often accompanied by or associated with other conditions
B. The diagnosis is often difficult
C. The mortality rate is less than that for acute calculous cholecystitis
D. The disease has been treated successfully by percutaneous cholecystostomy
17. In the treatment of acute cholecystitis, most patients are best treated with
A. Early cholecystectomy (within 2 days of onset of symptoms)
B. IV antibiotics and cholecystectomy in 6–8 weeks
C. Percutaneous drainage of the gall bladder
D. Endoscopic sphincterotomy
18. Which of the following is a possibility if symptoms persist after cholecystectomy?
A. Stone in CBD B. Long stump of cystic duct remnant
C. Damage to CBD from consequent stricture D. All of the above
19. Postcholecystectomy obstructive jaundice is due to
A. CBD stone B. Ischemic stricture of CBD
C. Missed malignancy D. All of the above
20. The common bile duct is said to be dilated when its size exceeds
A. 6 mm B. 8 mm
C. 10 mm D. 12 mm
21. Benign stricture in common bile duct is most commonly caused by
A. Stones B. Malignancy
C. Postoperative D. Cholangitis
22. Regarding stones in the common bile duct, all are true except
A. Are found in 10–20% of patients undergoing cholecystectomy
B. Can present with Charcot’s Triad
C. Are suggested by bile duct diameter >8 mm on ultrasound
D. Primary closure of the common bile duct after exploration is the standard treatment
23. In obstructive jaundice all are true except
A. Serum conjugated bilirubin is increased B. Serum unconjugated bilirubin is increased
C. Urinary urobilinogen is reduced D. Urinary conjugated bilirubin is increased

THE PANCREAS
24. Regarding both endocrine and exocrine tissues of the pancreas, all are true except
A. There are about one million islets of Langerhans in a healthy adult human pancreas, which are distributed throughout
the organ
B. Alpha cells produce glucagon
C. Beta cells produce insulin
D. Epsilon cells produce somatostatin
25. Pancreatitis occurs due to
A. Biliary tract disease B. Alcoholism
C. Drugs D. All of the above
26. The most common feature of acute pancreatitis is
A. Severe acute epigastric pain radiating to back B. Jaundice
C. Cullen’s sign and Grey Turner’s sign D. Abdominal guarding and loss of bowel sounds
27. What is not true of pancreatic pseudocyst?
A. Presents in epigastrium as a fixed swelling
B. Mimics aneurysm of aorta if very tense
C. Pseudocysts less than 6 cm invariably have spontaneous resolution
D. All are true

https://t.me/DentalBooksWorld
438 Triumph’s Complete Review of Dentistry

28. Indications for drainage of pseudocyst include


A. Infection B. Persists beyond 3 months
C. Larger than 6 cm D. All of the above
29. Pancreatic pseudocyst is usually drained to
A. Abdominal wall B. Stomach
C. Duodenum D. Ileum
30. The most accepted surgical procedure used to treat a chronic pancreatic pseudocyst is
A. Percutaneous drainage B. Internal drainage to the gastrointestinal tract
C. Pancreatectomy D. Excision of the pseudocyst
31. The most significant presentation of carcinoma of body of pancreas is
A. Pain B. Weight loss
C. Anorexia D. Jaundice
32. Zollinger–Ellison syndrome should be suspected in the presence of
A. Recurrent and resistant peptic ulceration
B. Multiple ulcers
C. Ulcers in jejunum, Meckel’s diverticulum or any abnormal site
D. All of the above
33. Which of the following statements about gastrinoma (Zollinger–Ellison syndrome) is/are correct?
A. Occur sporadic or part of multiple endocrine neoplasia type I (MEN l)
B. Extrapancreatic may arise in duodenum and peripancreatic lymph nodes
C. Diarrhea may be a prominent presenting feature
D. Before elective operation, acid-reducing medications such as omeprazole should be administered
E. All are true
34. Whipple’s triad in insulinoma includes
A. Hypoglycemia below 45 mg% B. An attack of hypoglycemia in fasting stage
C. Symptoms relieved by glucose D. All of the above
35. Most common pancreatic malignancy is
A. Squamous cell carcinoma B. Adenosquamous carcinoma
C. Insulinoma D. Ductal adenocarcinoma

THE LIVER
1. Normal portal venous pressure is
A. 5–7 mmHg B. 8–12 mmHg
C. 10–15 mmHg D. 15–20 mmHg
2. The recognized four complications of cirrhosis are all except
A. Ascites B. Portal hypertension
C. Hepatic failure D. Hepatic rupture
3. In a long-standing case of compensated cirrhosis, sudden deterioration can occur due to
A. Hepatoma B. Portosystemic encephalopathy
C. Spontaneous bacterial peritonitis D. All of the above
4. Toxic products responsible for portosystemic encephalopathy include
A. Ammonia B. Methionine
C. Short chain fatty acids D. All of the above
5. Anastomosis between portal and systemic venous systems occurs at all of the following except
A. Lower end of esophagus B. Around umbilicus
C. Peritoneal covering of liver D. Anal canal
6. In splenic vein thrombosis, collaterals develop around
A. Lower end of esophagus B. Fundus of stomach
C. Umbilicus D. Anus

https://t.me/DentalBooksWorld
Chapter 5 • General Surgery 439

7. A portal venous pressure of 30 mmHg (elevated) and a hepatic venous wedge pressure of 5 mmHg (normal) may be
associated with the following causes of portal hypertension except
A. Portal vein thrombosis
B. Alcoholic cirrhosis
C. Schistosomiasis
8. Which of the following treatments most effectively preserves hepatic portal perfusion?
A. Distal splenorenal shunt B. Conventional splenorenal shunt
C. Endoscopic sclerotherapy D. Side-to-side portacaval shunt
9. Which of the following shunts is associated with lowest risk of hepatic encephalopathy?
A. Mesocaval shunt B. Proximal splenorenal shunt
C. Distal splenorenal shunt (Warren shunt) D. Side-to-side portacaval shunt
10. All of the following problems commonly occur with the use of balloon tamponade for control of variceal bleeding except
A. Perforation with mediastinitis B. Aspiration of nasopharyngeal secretions
C. Rebleeding following removal of the tube D. Gastritis
11. Variceal bleeding not responding to drug and sclerotherapy is treated by
A. Embolization B. Surgical ligation
C. TIPSS D. Liver transplant
12. Before adopting TIPS (Transjugular intrahepatic portosystemic shunt), _________ must be excluded
A. Cardiovascular disease B. Portal vein occlusion
C. IV C obstruction D. All of the above
13. Variceal bleeding secondary to portal vein thrombosis is best treated by
A. TIPSS B. Sclerotherapy
C. Gastroesophageal devascularization D. Octreotide
14. Intrahepatic biliary lakes with stone characterize
A. Hepatic polycystic disease B. Primary biliary cirrhosis
C. Caroli’s disease D. Cholangitis
15. Regarding hepatic hemangioma, all are true except
A. Males are affected more than females
B. Steroids, estrogen, and pregnancy can increase the size of an already existing hemangioma
C. Usually are small and asymptomatic
D. At ultrasonography, hemangiomas appear as hyperechoic lesions
16. Regarding benign liver tumors, all are true except
A. Hemangiomas are the most common form of benign liver tumors
B. Focal nodular hyperplasia is the second most common form of benign liver tumor, appears as a well-circumscribed
solid lesion with central scar
C. Hepatocellular adenomas are less common benign liver tumors and may be multiple
D. Hepatic adenomas do not turn malignant
17. Concerning focal nodular hyperplasia (FNH) all are true except
A. The lesion predominantly affects young women
B. There is no clear relationship between oral contraceptives and the development of FNH
C. Radionucleotide scanning can be useful in the specific diagnosis of FNH
D. Excisional biopsy is indicated in almost all cases because of the risk of bleeding
18. Worldwide, the most important predisposing factor for HCC is
A. Alcoholic cirrhosis B. Hepatitis B infection
C. Hepatitis C infection D. Chronic liver disease of any etiology
19. Regarding hepatocellular carcinoma all are true, except
A. May complicate hepatitis B infections B. May present with rupture and peritoneal bleeding
C. Alpha-fetoprotein is a useful tumor marker D. Never complicates alcoholic cirrhosis
20. To exclude onset of hepatoma in a cirrhotic patient, the test of choice is
A. US B. CT
C. Alpha fetoprotein D. None of the above

https://t.me/DentalBooksWorld
440 Triumph’s Complete Review of Dentistry

21. A 52-year-old chronic alcoholic with known cirrhosis is noted to have a mass in the right lobe of the liver and an
elevated alpha-fetoprotein level. What is the most likely diagnosis?
A. Hepatocellular adenoma B. Hepatocellular carcinoma
C. Metastatic carcinoma of the colon D. Regenerating nodule of cirrhosis
22. Most primary and metastatic tumors to the liver derive nearly all their blood supply from
A. Portal vein B. Hepatic artery
C. Collateral circulation D. Celiac axis
23. With regard to treatment of HCC (Hepatocellular Carcinoma), choose the correct answer
A. Liver resection for solitary FICC in patients with preserved liver function
B. Liver transplantation is indicated in cirrhotic patients with a small (5 cm or less single nodule or up to three lesions of
3 cm or less) HCC
C. Percutaneous ethanol injections (PEI), cryotherapy, and radiofrequency ablation are used when the above measures
are not feasible
D. Traditional chemotherapy is generally ineffective, causes many side effects that may severely impair the patient’s quality of life
E. All are true
24. With regard to treatment of HCC, all are true except
A. Liver resection is the treatment of choice in patients with normal livers and Child A cirrhosis when the disease is
localized to the liver
B. Liver transplantation is the treatment of choice in Child B/C patients with limited tumor involvement, when the
disease is localized to the liver
C. Radiofrequency is successful in tumors up to 10 cm
D. Transarterial chemoembolization (TACE) in patients with preserved liver functions depends on the fact that HCC
receive almost 100% of its blood supply from the artery
25. Pringle maneuver refers to
A. Clamp over IVC B. Clamp over hepatic vein
C. Clamp across foramen of Winslow D. Clamp across splenic artery
26. Refractory ascites of cirrhosis can be treated by
A. TIPS B. IV infusion of salt poor albumin
C. Peritoneal–jugular shunt D. All of the above
27. The most feared complication of Denver shunt is
A. Infection B. Malfunction
C. DIC D. Rupture
28. In Budd–Chiari syndrome, the occlusion is at the
A. IVC B. Renal vein
C. Hepatic vein D. Splenic vein
29. All the following statements regarding Budd–Chiari syndrome are true except
A. Can occur due to thrombosis of portal vein
B. It presents with the classical triad of abdominal pain, ascites, and hepatomegaly
C. Liver transplantation is an effective treatment
D. Caudate lobe hypertrophy is often present
30. A small cirrhotic liver with grossly enlarged caudate lobe demands exclusion of
A. Portal vein thrombosis B. Budd–Chiari syndrome
C. Hepatoma D. Primary sclerosing cholangitis

THE SPLEEN
1. Common sites of accessory spleen are all of the following, except
A. Splenic hilum B. Tail of pancreas
C. Head of pancreas D. Transverse mesocolon
2. Kehr’s sign in splenic trauma refers to
A. Pain and hyperesthesia in left shoulder B. Pain and hyperesthesia in right shoulder
C. Bruising around left 10th and 11th ribs D. Hiccup and hemoptysis on leg elevation

https://t.me/DentalBooksWorld
Chapter 5 • General Surgery 441

3. Radiographic signs of splenic rupture include


A. Obliteration of psoas shadow B. Indentation of gastric air bubble
C. Elevation of left hemidiaphragm D. All of the above
4. Purpura is due to
A. Reduced platelets B. Thrombocytopathies
C. Increased capillary fragility D. All of the above
5. The spleen is in contact with
A. Diaphragm B. Upper pole of kidney
C. Pancreas D. All of the above
6. What is not true about the function of spleen?
A. Essential to life B. Destruction of aged RBCs by pitting and culling
C. Platelet reservoir D. Erythrocyte production in fetal spleen
7. Splenunculi refer to
A. Calculi within spleen B. Accessory spleen
C. Atrophic spleen D. None of the above
8. Thrombocytopenia can be due to all of the following except
A. Aplastic anemia B. DIC
C. Hypersplenism D. Postsplenectomy
9. During acute ITP, treatment options are all except
A. Splenectomy B. Steroids
C. lgG D. Platelet concentrate
10. A young child having anemia and gallstones should be investigated for
A. Cystic fibrosis B. Congenital spherocytosis
C. Malaria D. Primary sclerosing cholangitis
11. RBC destruction in hereditary spherocytosis is due to
A. RBC antibody B. Complement activation
C. Increase in permeability of RBC membrane to sodium D. All of the above
12. Which of the following does not go with diagnosis of congenital spherocytosis?
A. Gall stones B. Reticulocytosis
C. Decreased urobilinogen D. RBC hemolysis occur at 0.6% normal saline or above
13. The commonest cause of neoplastic enlargement of spleen is
A. Lymphoma B. Secondary deposit
C. Hemangiosarcoma D. Leukemia
14. Splenomegaly along with lymphadenopathy characterize all the following except
A. Hodgkin’s disease B. Acute leukemia
C. Cirrhosis D. Felty’s syndrome
15. Indication for elective splenectomy include all except
A. Hypersplenism B. Staging of Hodgkin’s disease
C. To treat splenic cyst, tumor, or abscess D. Sickle cell anemia

ANSWERS

INTRODUCTION AND FLUID DYNAMICS


1. Answer: B (Ref. Essentials of General Surgery, By Peter F. Lawrence, Richard M. Bell, Merril T. Dayton, 5th edition, page
no. 8)
2. Answer: C (Ref. Essentials of General Surgery, By Peter F. Lawrence, Richard M. Bell, Merril T. Dayton, 5th edition, page
no. 34)

https://t.me/DentalBooksWorld
442 Triumph’s Complete Review of Dentistry

3. Answer: D (Ref. Essentials of General Surgery, By Peter F. Lawrence, Richard M. Bell, Merril T. Dayton, 5th edition, page
no. 36)
4. Answer: B (Ref. Essentials of General Surgery, By Peter F. Lawrence, Richard M. Bell, Merril T. Dayton, 5th edition, page
no. 36)
5. Answer: C (Ref. Essentials of General Surgery, By Peter F. Lawrence, Richard M. Bell, Merril T. Dayton, 5th edition, page
no. 38)
6. Answer: D (Ref. Essentials of General Surgery, By Peter F. Lawrence, Richard M. Bell, Merril T. Dayton, 5th edition, page
no. 34)
7. Answer: A (Ref. Essentials of General Surgery, By Peter F. Lawrence, Richard M. Bell, Merril T. Dayton, 5th edition, page
no. 100)
8. Answer: C (Ref. Essentials of General Surgery, By Peter F. Lawrence, Richard M. Bell, Merril T. Dayton, 5th edition, page
no. 35)
9. Answer: C (Ref. “Previous question”)
10. Answer: D (Ref. Essentials of General Surgery, By Peter F. Lawrence, Richard M. Bell, Merril T. Dayton, 5th edition, page
no. 263)
11. Answer: D (Ref. Essentials of General Surgery, By Peter F. Lawrence, Richard M. Bell, Merril T. Dayton, 5th edition, page
no. 195)
It is hyperglycemia and not hypoglycemia that occurs in stress response. There is an increase in glucagon and reduction of
insulin producing the “diabetes of stress.”
Marked changes occur in the body metabolism – lipolysis, hepatic gluconeogenesis, skeletal muscle protein breakdown,
hyperglycemia, and hypermetabolism. The changes that occur in body metabolism in the stress response have been divided
into the “ebb and flow” phases, a term that Sir David Cuthbertson first used in 1932. The initial ebb phase (a holding
pattern), which lasts for a few hours, is characterized by the classical features of shock – hypovolemia, hypothermia,
reduced cardiac output, and lactic acidosis.
12. Answer: D (Ref. Essentials of General Surgery, By Peter F. Lawrence, Richard M. Bell, Merril T. Dayton, 5th edition, page
no. 267)
The patient is 7 days following an anterior resection without a defunctioning stoma. The patient has generalized peritonitis.
The most likely answer is an anastomotic leakage. Pulmonary embolus and deep vein thrombosis are recognized causes of
postoperative pyrexia and should be excluded but are less likely. Pre-existing chest infection and infective exacerbations
are common in patients with COPD but are less likely to be responsible for increasing abdominal pain and peritonitis.
13. Answer: D (Ref. Essentials of General Surgery, By Peter F. Lawrence, Richard M. Bell, Merril T. Dayton, 5th edition, page
no. 30)
Patients becoming unwell within minutes of starting a blood transfusion should arouse the suspicion of a hemolytic
transfusion reaction (ABO incompatibility), especially in a patient with O group status. In this scenario we are not told
whether the blood was cross-matched or type specific. Nonhemolytic febrile transfusion reactions are more likely to occur
>30 minutes following transfusion, and generally the patient remains well. Bacterial contamination is a possibility and
should be excluded but is less likely than ABO incompatibility. Air embolus and transfusion-related acute lung injury are
recognized complications of blood transfusions but are less likely given the patient’s symptoms.
14. Answer: C (Ref. Essentials of General Surgery, By Peter F. Lawrence, Richard M. Bell, Merril T. Dayton, 5th edition, page
no. 317)
Insulin-dependent diabetic patients undergoing major elective surgery should continue their normal subcutaneous insulin
until nil by mouth the night before surgery. Two preoperative regimens are commonly used: PIG (= potassium, insulin,
and glucose) as per option C or 50 units of insulin in 50 ml normal saline (i.e., 1:1 regimen) administered according to a
sliding scale, run with fluids supplemented with KCl. Option B cannot be considered as the correct answer as it does not
take into consideration the need to supplement potassium when infusing insulin. Option A is the appropriate therapy for
diabetic ketoacidosis. Note that insulin-dependent diabetic patients undergoing minor surgery may not require additional
insulin, and only require close monitoring of blood glucose in the perioperative period.
15. Answer: C (Ref. Essentials of General Surgery, By Peter F. Lawrence, Richard M. Bell, Merril T. Dayton, 5th edition, page
no. 148)
The management of type 2 diabetic patients undergoing minor to intermediate surgery is to continue oral hypoglycemic
agents until the day of surgery, then omit the morning dose, restarting oral hypoglycemics with the first meal. If the
patient’s blood glucose is >10 mmol/l or they are undergoing major surgery, one of the two commonly used regimens
for insulin-dependent diabetic patients should be followed, i.e., either PIG (potassium, insulin and glucose) or an insulin
sliding scale. Perioperative management does not depend on HbA1c levels but this is a reasonably good marker of long-
term diabetic control.

https://t.me/DentalBooksWorld
Chapter 5 • General Surgery 443

16. Answer: D (Ref. Essentials of General Surgery, By Peter F. Lawrence, Richard M. Bell, Merril T. Dayton, 5th edition, page
no. 65)
17. Answer: B (Ref. Essentials of General Surgery, By Peter F. Lawrence, Richard M. Bell, Merril T. Dayton, 5th edition, page
no. 107)
Dextrose/saline is a useful fluid therapy in the early postoperative period because it does not cause salt and water
overload and provides some energy to the patient. Dextrose/saline solution, otherwise known as one-fifth normal saline,
has an osmolality that is nearly isotonic with plasma because of the 4% content of dextrose. It has a slightly alkaline
pH and contains approximately 30 mmol of sodium and chloride ions. It does not contain K+ ions and so potassium
supplementation is important if the patient is not yet established on oral intake. It predominantly replaces pure water
losses that are common following surgery. It is less useful in hypovolemic resuscitation as it is a less effective plasma
expander than colloid or normal saline and in patients who are losing excess salts.
18. Answer: A
19. Answer: C (Ref. Essentials of General Surgery, By Peter F. Lawrence, Richard M. Bell, Merril T. Dayton, 5th edition, page
no. 573)
Blood glucose should be monitored daily as hyper/hypoglycemia is common on total parenteral nutrition regimens.
Electrolyte disturbance is also common, and therefore urea, creatinine, potassium, sodium, magnesium, and phosphate
levels must also be checked daily. Hypophosphatemia is a particular problem with TPN and additional supplementation
is almost always required. There is significant risk of sepsis, therefore daily FBC is also required. Daily weights should be
taken, along with meticulous fluid balance charting. Liver function tests should be performed twice a week to monitor any
sign of cholestatic jaundice and fatty infiltration.
20. Answer: A
21. Answer: C (Ref. Essentials of General Surgery, By Peter F. Lawrence, Richard M. Bell, Merril T. Dayton, 5th edition, page
no. 274)
This patient’s gastrointestinal tract is functioning normally but the oral route is not an option. This is an ideal patient for a
percutaneous gastrostomy tube. Nasogastric and nasojejunal feeding, and total parenteral nutrition are not suitable long-
term strategies.
22. Answer: A (Ref. Essentials of General Surgery, By Peter F. Lawrence, Richard M. Bell, Merril T. Dayton, 5th edition, page
no. 85)
23. Answer: D (Ref. Essentials of General Surgery, By Peter F. Lawrence, Richard M. Bell, Merril T. Dayton, 5th edition, page
no. 81)
24. Answer: C (Ref. Essentials of General Surgery, By Peter F. Lawrence, Richard M. Bell, Merril T. Dayton, 5th edition, page
no. 86)
25. Answer: D (Ref. Essentials of General Surgery, By Peter F. Lawrence, Richard M. Bell, Merril T. Dayton, 5th edition, page
no. 57)
26. Answer: A (Ref. Essentials of General Surgery, By Peter F. Lawrence, Richard M. Bell, Merril T. Dayton, 5th edition, page
no. 114)
27. Answer: B (Ref. Essentials of General Surgery, By Peter F. Lawrence, Richard M. Bell, Merril T. Dayton, 5th edition, page
no. 89)
28. Answer: D (Ref. Essentials of General Surgery, By Peter F. Lawrence, Richard M. Bell, Merril T. Dayton, 5th edition, page
no. 386)
29. Answer: A (Ref. “Previous question”)
30. Answer: D (Ref. Essentials of General Surgery, By Peter F. Lawrence, Richard M. Bell, Merril T. Dayton, 5th edition, page
no. 47)
31. Answer: B (Ref. Essentials of General Surgery, By Peter F. Lawrence, Richard M. Bell, Merril T. Dayton, 5th edition, page
no. 78)
32. Answer: D (Ref. Essentials of General Surgery, By Peter F. Lawrence, Richard M. Bell, Merril T. Dayton, 5th edition, page
no. 25)
33. Answer: D (Ref. Essentials of General Surgery, By Peter F. Lawrence, Richard M. Bell, Merril T. Dayton, 5th edition,
page47)
34. Answer: D (Ref. Essentials of General Surgery, By Peter F. Lawrence, Richard M. Bell, Merril T. Dayton, 5th edition, page
no. 169)
35. Answer: D (Ref. Essentials of General Surgery, By Peter F. Lawrence, Richard M. Bell, Merril T. Dayton, 5th edition, page
no. 166,167)

https://t.me/DentalBooksWorld
444 Triumph’s Complete Review of Dentistry

ENDOCRINE AND THYROID


1. Answer: A (Ref. Essentials of General Surgery, By Peter F. Lawrence, Richard M. Bell, Merril T. Dayton, 5th edition, page
no. 408)
2. Answer: D (Ref. Essentials of General Surgery, By Peter F. Lawrence, Richard M. Bell, Merril T. Dayton, 5th edition, page
no. 408)
3. Answer: D (Ref. Essentials of General Surgery, By Peter F. Lawrence, Richard M. Bell, Merril T. Dayton, 5th edition, page
no. 409)
4. Answer: D (Ref. Essentials of General Surgery, By Peter F. Lawrence, Richard M. Bell, Merril T. Dayton, 5th edition, page
no. 407)
5. Answer: A (Ref. Essentials of General Surgery, By Peter F. Lawrence, Richard M. Bell, Merril T. Dayton, 4th edition, page
no. 465)
6. Answer: C
7. Answer: D (Ref. “Previous question”)
8. Answer: D (Ref. Essentials of General Surgery, By Peter F. Lawrence, Richard M. Bell, Merril T. Dayton, 5th edition, page
no. 575)
9. Answer: A (Ref. Essentials of General Surgery, By Peter F. Lawrence, Richard M. Bell, Merril T. Dayton, 5th edition, page
no. 428)
10. Answer: D (Ref. Essentials of General Surgery, By Peter F. Lawrence, Richard M. Bell, Merril T. Dayton, 5th edition, page
no. 408)
11. Answer: C (Ref. Essentials of General Surgery, By Peter F. Lawrence, Richard M. Bell, Merril T. Dayton, 5th edition, page
no. 558)
12. Answer: C (Ref. Essentials of General Surgery, By Peter F. Lawrence, Richard M. Bell, Merril T. Dayton, 5th edition, page
no. 558)
13. Answer: D (Ref. Essentials of General Surgery, By Peter F. Lawrence, Richard M. Bell, Merril T. Dayton, 5th edition, page
no. 551)
14. Answer: D (Ref. Essentials of General Surgery, By Peter F. Lawrence, Richard M. Bell, Merril T. Dayton, 5th edition, page
no. 553)
15. Answer: D (Ref. “Previous question”)
16. Answer: D (Ref. “Previous question”)
17. Answer: B (Ref. “Previous question”)
18. Answer: A (Ref. Essentials of General Surgery, By Peter F. Lawrence, Richard M. Bell, Merril T. Dayton, 4th edition, page
no. 391)
19. Answer: B (Ref. Essentials of General Surgery, By Peter F. Lawrence, Richard M. Bell, Merril T. Dayton, 5th edition, page
no. 403)
20. Answer: D (Ref. Essentials of General Surgery, By Peter F. Lawrence, Richard M. Bell, Merril T. Dayton, 4th edition, page
no. 405)
21. Answer: C (Ref. Essentials of General Surgery, By Peter F. Lawrence, Richard M. Bell, Merril T. Dayton, 5th edition, page
no. 410)
22. Answer: C (Ref. “Previous question”)
23. Answer: A (Ref. Essentials of General Surgery, By Peter F. Lawrence, Richard M. Bell, Merril T. Dayton, 5th edition, page
no. 410)
Von Graefe’s sign is the lagging of the upper eyelid on downward rotation of the eye, indicating exophthalmic goiter
(Graves’ Disease). It is a dynamic sign, whereas lid lag is a static sign which may also be present in cicatricial eyelid
retraction or congenital ptosis.
Some clinical signs of thyrotoxicosis:
Dalrymple’s sign – Unnatural degree of separation between the margins of two lids.
Mobius’ sign – Imperfect power of convergence.
Stellwag’s sign – Diminished frequency of blinking and imperfect closure of lids during the act.
Von Graefe’s sign – The failure of the eyelid to move downward promptly with the eyeball and the lid moves tardily and
jerkily.
24. Answer: A (Ref. Essentials of General Surgery, By Peter F. Lawrence, Richard M. Bell, Merril T. Dayton, 5th edition, page
no. 584)

https://t.me/DentalBooksWorld
Chapter 5 • General Surgery 445

25. Answer: B (Ref. “Previous question”)


• Anti-thyroid drugs are safe and should be used with the lowest possible doses to maintain the free T4 and T3 in the
upper end of the normal range
• Both carbimazole and PTU cross the placenta and may cause fetal hypothyroidism, but PTU has less transplacental
transfer
26. Answer: D (Ref. “Previous question”)
27. Answer: E (Ref. “Previous question”)
28. Answer: E (Ref. Essentials of General Surgery, By Peter F. Lawrence, Richard M. Bell, Merril T. Dayton, 4th edition, page
no. 403)
29. Answer: E (Ref. Essentials of General Surgery, By Peter F. Lawrence, Richard M. Bell, Merril T. Dayton, 4th edition, page
no. 135)
30. Answer: E (Ref. “Previous question”)
31. Answer: A (Ref. Essentials of General Surgery, By Peter F. Lawrence, Richard M. Bell, Merril T. Dayton, 4th edition, page
no. 425)
32. Answer: D (Ref. “Previous question”)
33. Answer: C (Ref. Essentials of General Surgery, By Peter F. Lawrence, Richard M. Bell, Merril T. Dayton, 5th edition, page
no. 412)
34. Answer: D (Ref. Essentials of General Surgery, By Peter F. Lawrence, Richard M. Bell, Merril T. Dayton, 5th edition, page
no. 412)
Following total thyroidectomy, iodine 131 can be used more efficiently because of the absence of normal thyroid tissue,
which has greater affinity for iodine than papillary carcinoma tissue. When all normal thyroid tissue is removed, serum
thyroglobulin, which is produced by normal and malignant thyroid tissue, becomes a more effective marker for recurrence.
The overall recurrence rate is lower for patients undergoing total thyroidectomy, but the risk of hypoparathyroidism is
higher for patients who have total thyroidectomy instead of unilateral lobectomy.
35. Answer: B (Ref. Essentials of General Surgery, By Peter F. Lawrence, Richard M. Bell, Merril T. Dayton, 5th edition, page
no. 412)
36. Answer: C (Ref. Essentials of General Surgery, By Peter F. Lawrence, Richard M. Bell, Merril T. Dayton, 5th edition, page
no. 412)
37. Answer: D (Ref. “Previous question”)
38. Answer: B (Ref. Essentials of General Surgery, By Peter F. Lawrence, Richard M. Bell, Merril T. Dayton, 5th edition, page
no. 412)
39. Answer: D (Ref. “Previous question”)
40. Answer: A (Ref. “Previous question”)
41. Answer: C (Ref. Essentials of General Surgery, By Peter F. Lawrence, Richard M. Bell, Merril T. Dayton, 5th edition, page
no. 425)
42. Answer: B (Ref. “Previous question”)
43. Answer: B (Ref. Essentials of General Surgery, By Peter F. Lawrence, Richard M. Bell, Merril T. Dayton, 5th edition, page
no. 558)
De Quervain’s thyroiditis, also known as subacute granulomatous thyroiditis or giant cell thyroiditis, is a member of the
group of thyroiditis conditions known as resolving thyroiditis.
44. Answer: D (Ref. Essentials of General Surgery, By Peter F. Lawrence, Richard M. Bell, Merril T. Dayton, 5th edition, page
no. 408)
45. Answer: C (Ref. “Previous question”)
46. Answer: D (Ref. “Previous question”)
47. Answer: D (Ref. “Previous question”)
48. Answer: D (Ref. Essentials of General Surgery, By Peter F. Lawrence, Richard M. Bell, Merril T. Dayton, 5th edition, page
no. 416)
49. Answer: A (Ref. Essentials of General Surgery, By Peter F. Lawrence, Richard M. Bell, Merril T. Dayton, 5th edition, page
no. 414)
Risus sardonicus or rictus grin is a highly characteristic, abnormal, sustained spasm of the facial muscles that appears to
produce grinning.
50. Answer: D (Ref. “Previous question”)

https://t.me/DentalBooksWorld
446 Triumph’s Complete Review of Dentistry

51. Answer: D (Ref. Essentials of General Surgery, By Peter F. Lawrence, Richard M. Bell, Merril T. Dayton, 5th edition, page
nos. 70, 154)
52. Answer: B (Ref. “Previous question”)
53. Answer: C (Ref. “Previous question”)
54. Answer: C (Ref. “Previous question”)
55. Answer: A (Ref. “Previous question”)
56. Answer: A (Ref. “Previous question”)
57. Answer: C (Ref. Essentials of General Surgery, By Peter F. Lawrence, Richard M. Bell, Merril T. Dayton, 5th edition, page
no. 410)
58. Answer: E (Ref. Essentials of General Surgery, By Peter F. Lawrence, Richard M. Bell, Merril T. Dayton, 5th edition, page
no. 426)
59. Answer: C (Ref. Essentials of General Surgery, By Peter F. Lawrence, Richard M. Bell, Merril T. Dayton, 5th edition, page
no. 558)
Riedel thyroiditis, or Riedel’s thyroiditis (RT), is a rare, chronic inflammatory disease of the thyroid gland characterized by
a dense fibrosis that replaces normal thyroid parenchyma. The fibrotic process invades adjacent structures of the neck and
extends beyond the thyroid capsule.
60. Answer: E (Ref. “Previous question”)
61. Answer: A (Ref. Essentials of General Surgery, By Peter F. Lawrence, Richard M. Bell, Merril T. Dayton, 5th edition, page
no. 414)
62. Answer: E (Ref. Essentials of General Surgery, By Peter F. Lawrence, Richard M. Bell, Merril T. Dayton, 5th edition, page
no. 414)
63. Answer: E (Ref. “Previous question”)
64. Answer: E (Ref. “Previous question”)
65. Answer: B (Ref. Essentials of General Surgery, By Peter F. Lawrence, Richard M. Bell, Merril T. Dayton, 5th edition, page
no. 423)
66. Answer: E (Ref. Essentials of General Surgery, By Peter F. Lawrence, Richard M. Bell, Merril T. Dayton, 5th edition, page
no. 424)
67. Answer: D (Ref. Essentials of General Surgery, By Peter F. Lawrence, Richard M. Bell, Merril T. Dayton, 5th edition, page
no. 424)
68. Answer: C (Ref. Essentials of General Surgery, By Peter F. Lawrence, Richard M. Bell, Merril T. Dayton, 5th edition, page
no. 425)
Multiple endocrine neoplasia (MEN)
Is characterized by tumors involving two or more endocrine glands.
MEN inherited as autosomal dominant disorders.
First-degree relatives have about a 50% risk of developing the disease.
Tumors may be benign (the majority of insulinoma and pheochromocytomas) or
malignant (medullary carcinoma of thyroid and majority of gastrinomas)
MEN 1 Syndrome (Wermer’s syndrome)
Parathyroid hyperplasia
Pancreatic islet cell tumors (gastrinoma, insulinoma, glucagonoma)
Anterior pituitary tumor
MEN 2a Syndrome (Sipple’s Syndrome)
Medullary thyroid carcinoma
Pheochromocytoma
Parathyroid hyperplasia
MEN 2b Syndrome
Medullary thyroid carcinoma
Pheochromocytoma
Multiple mucosal neuromas
69. Answer: B (Ref. “Previous question”)
70. Answer: C (Ref. Essentials of General Surgery, By Peter F. Lawrence, Richard M. Bell, Merril T. Dayton, 5th edition, page no. 533)
71. Answer: B (Ref. “Previous question”)
72. Answer: E (Ref. “Previous question”)

https://t.me/DentalBooksWorld
Chapter 5 • General Surgery 447

73. Answer: A (Ref. “Previous question”)


Medullary carcinoma arises from parafollicular cell.
74. Answer: E (Ref. “Previous question”)
75. Answer: B (Ref. “Previous question”)
76. Answer: E (Ref. Essentials of General Surgery, By Peter F. Lawrence, Richard M. Bell, Merril T. Dayton, 5th edition, page
no. 408)
77. Answer: C (Ref. “Previous question”)
78. Answer: C (Ref. Essentials of General Surgery, By Peter F. Lawrence, Richard M. Bell, Merril T. Dayton, 5th edition, page
no. 506)
79. Answer: C (Ref. Essentials of General Surgery, By Peter F. Lawrence, Richard M. Bell, Merril T. Dayton, 4th edition, page
no. 403)
80. Answer: E (Ref. Essentials of General Surgery, By Peter F. Lawrence, Richard M. Bell, Merril T. Dayton, 4th edition, page
no. 469)
In apical lung cancers, the malignant tumor may extend above the thoracic inlet, penetrate the suprapleural membrane,
and infiltrate the structures found at the root of the neck. The first thoracic nerve and lower trunk of the brachial plexus are
most likely to be involved initially, as T1 passes along the inner border of the first rib to reach the neck. If the sympathetic
nerve is involved, pupil constriction and ptosis may be evident (Horner syndrome). The other listed items are all features of
the paraneoplastic syndrome associated with lung cancer and do not necessarily indicate extranodal metastasis. Cushing’s
syndrome in lung cancer occurs more frequently in men and in an older age group and has a more rapid downhill course
than typical Cushing’s syndrome. SIADH should be suspected if the patient with a lung lesion develops unexplained
mental changes and an extremely low serum sodium level. Fluid restriction is required. Urine osmolarity is low.

BURNS, WOUND HEALING, GRAFT AND RECONSTRUCTIVE SURGERY


1. Answer: B (Ref. Essentials of General Surgery, By Peter F. Lawrence, Richard M. Bell, Merril T. Dayton, 5th edition, page
no. 133)
2. Answer: C (Ref. Essentials of General Surgery, By Peter F. Lawrence, Richard M. Bell, Merril T. Dayton, 5th edition, page
no. 95)
3. Answer: C
4. Answer: E
5. Answer: A
6. Answer: C (Ref. Essentials of General Surgery, By Peter F. Lawrence, Richard M. Bell, Merril T. Dayton, 5th edition, page
no. 188)
7. Answer: D (Ref. Essentials of General Surgery, By Peter F. Lawrence, Richard M. Bell, Merril T. Dayton, 5th edition, page
no. 193)
8. Answer: A (Ref. Essentials of General Surgery, By Peter F. Lawrence, Richard M. Bell, Merril T. Dayton, 5th edition, page
no. 188)
9. Answer: A (Ref. Essentials of General Surgery, By Peter F. Lawrence, Richard M. Bell, Merril T. Dayton, 5th edition, page
no. 140)
10. Answer: B
11. Answer: D (Ref. Essentials of General Surgery, By Peter F. Lawrence, Richard M. Bell, Merril T. Dayton, 5th edition, page
no. 406)
12. Answer: B (Ref. Essentials of General Surgery, By Peter F. Lawrence, Richard M. Bell, Merril T. Dayton, 5th edition, page
no. 90)
13. Answer: D
14. Answer: D (Ref. Burn Care for General Surgeons and General Practitioners, By David G. Greenhalgh, page no. 66)
15. Answer: A (Ref. Essentials of General Surgery, By Peter F. Lawrence, Richard M. Bell, Merril T. Dayton, 5th edition, page
no. 538)
16. Answer: C
17. Answer: A (Ref. Essentials of General Surgery, By Peter F. Lawrence, Richard M. Bell, Merril T. Dayton, 5th edition, page
no. 90)
18. Answer: D (Ref. Essentials of General Surgery, By Peter F. Lawrence, Richard M. Bell, Merril T. Dayton, 5th edition,
page no. 26)

https://t.me/DentalBooksWorld
448 Triumph’s Complete Review of Dentistry

19. Answer: C (Ref. Essentials of General Surgery, By Peter F. Lawrence, Richard M. Bell, Merril T. Dayton, 4th edition,
page no. 253)
20. Answer: C (Ref. Essentials of General Surgery, By Peter F. Lawrence, Richard M. Bell, Merril T. Dayton, 5th edition, page
no. 134)
Anemia was once believed to be a significant cause of wound disruption; studies have shown that in the absence of
malnutrition or hypovolemia, anemia with a hematocrit greater than 15% does not interfere with healing.
21. Answer: C
22. Answer: B
Bacteremia and bacteremic shock are the second most common cause of death in burn.
23. Answer: C

SALIVARY GLANDS AND SWELLING OF THE NECK


1. Answer: A (Ref. Burket’s Oral Medicine, By Lester William Burket, Martin S. Greenberg, Michael Glick, Jonathan A. Ship,
11th edition, page no. 203)
2. Answer: B (Ref. Burket’s Oral Medicine, By Lester William Burket, Martin S. Greenberg, Michael Glick, Jonathan A. Ship,
11th edition, page no. 201)
3. Answer: D (Ref. Burket’s Oral Medicine, By Lester William Burket, Martin S. Greenberg, Michael Glick, Jonathan A. Ship,
11th edition, page no. 217)
4. Answer: D (Ref. Essentials of General Surgery edited, By Peter F. Lawrence, Richard M. Bell, Merril T. Dayton, 5th edition,
page no. 421)
5. Answer: C (Ref. Essentials of General Surgery, By Peter F. Lawrence, Richard M. Bell, Merril T. Dayton, 5th edition, page
no. 512)
6. Answer: B (Ref. Essentials of General Surgery edited, By Peter F. Lawrence, Richard M. Bell, Merril T. Dayton, 4th edition,
page no. 211)
7. Answer: B
8. Answer: C
Epidermoid carcinoma of skin and malignant melanoma commonly metastatize to the parotid gland.
9. Answer: A

NECK SWELLINGS
10. Answer: D (Ref. Essentials of General Surgery edited, By Peter F. Lawrence, Richard M. Bell, Merril T. Dayton, 5th edition,
page no. 407)
11. Answer: D (Ref. “Previous question”)
12. Answer: B (Ref. Essentials of General Surgery edited, By Peter F. Lawrence, Richard M. Bell, Merril T. Dayton, 5th edition,
page no. 548)
13. Answer: D (Ref. Essentials of General Surgery edited, By Peter F. Lawrence, Richard M. Bell, Merril T. Dayton, 5th edition,
page no. 548)
14. Answer: C (Ref. Essentials of General Surgery edited, By Peter F. Lawrence, Richard M. Bell, Merril T. Dayton, 5th edition,
page no. 548)
15. Answer: C (Ref. Essentials of General Surgery edited, By Peter F. Lawrence, Richard M. Bell, Merril T. Dayton, 5th edition,
page no. 194)
16. Answer: A (Ref. Essentials of General Surgery edited, By Peter F. Lawrence, Richard M. Bell, Merril T. Dayton, 5th edition,
page no. 194)
17. Answer: A (Ref. Essentials of General Surgery edited, By Peter F. Lawrence, Richard M. Bell, Merril T. Dayton, 5th edition,
page no. 352)
18. Answer: D (Ref: Ballenger’s Otorhinolaryngology 17: Head and Neck Surgery,
By James Byron Snow, John Jacob Ballenger, page no. 82)
19. Answer: B (Ref. Essentials of General Surgery edited, By Peter F. Lawrence, Richard M. Bell, Merril T. Dayton, 4th edition,
page no. 506)
20. Answer: C (Ref. Essentials of General Surgery edited, By Peter F. Lawrence, Richard M. Bell, Merril T. Dayton, 5th edition,
page no. 303)

https://t.me/DentalBooksWorld
Chapter 5 • General Surgery 449

21. Answer: D (Ref. Essentials of General Surgery edited, By Peter F. Lawrence, Richard M. Bell, Merril T. Dayton, 4th edition,
page no. 469)
22. Answer: C (Ref. Essentials of General Surgery edited, By Peter F. Lawrence, Richard M. Bell, Merril T. Dayton, 4th edition,
page no. 469)
23. Answer: A (Ref. Mastery of Surgery edited, By Josef E. Fischer, Kirby I. Bland, Mark P. Callery, page no. 2007)
Carotid body is situated at the bifurcation of the carotid artery. It presents with a long history of lump at carotid bifurcation
which moves from side 10 side but not vertically and a pulsating vessel is present at list outer surface.
24. Answer: C (Ref. “Previous question”)
25. Answer: D (Ref. Essentials of General Surgery edited, By Peter F. Lawrence, Richard M. Bell, Merril T. Dayton, 5th edition,
page no. 429)
26. Answer: B (Ref. “Previous question”)
27. Answer: C (Ref. “Previous question”)
28. Answer: B (Ref. “Previous question”)
29. Answer: B (Ref. “Previous question”)
30. Answer: A (Ref. “Previous question”)
31. Answer: A (Ref. “Previous question”)

WOUND INFECTION
1. Answer: A
2. Answer: C (Ref. Essentials of General Surgery, By Peter F. Lawrence, Richard M. Bell, Merril T. Dayton, 4th edition, page
no. 95)
Clean wounds: 1–2%
Clean contaminated wounds: <10%
Contaminated wounds: 15–20%
3. Answer: C (Ref. Essentials of General Surgery, By Peter F. Lawrence, Richard M. Bell, Merril T. Dayton, 5th edition, page
no. 165)
4. Answer: A (Ref. Essentials of General Surgery, By Peter F. Lawrence, Richard M. Bell, Merril T. Dayton, 5th edition, page
no. 153)
5. Answer: C
6. Answer: C (Ref. Essentials of General Surgery, By Peter F. Lawrence, Richard M. Bell, Merril T. Dayton, 5th edition, page
no. 322)
7. Answer: C (Ref. “Previous question”)
Peritonsillar abscess is most common neck abscess to occur in childhood.
8. Answer: A (Ref. Essentials of General Surgery, By Peter F. Lawrence, Richard M. Bell, Merril T. Dayton, 5th edition, page
no. 153)
9. Answer: D (Ref. Essentials of General Surgery, By Peter F. Lawrence, Richard M. Bell, Merril T. Dayton, 5th edition, page
no. 154)
10. Answer: B (Ref. Essentials of General Surgery, By Peter F. Lawrence, Richard M. Bell, Merril T. Dayton, 5th edition, page
no. 124)
11. Answer: B (Ref. “Previous question”)
12. Answer: A (Ref. Essentials of General Surgery, By Peter F. Lawrence, Richard M. Bell, Merril T. Dayton, 4th edition, page no. 94)
13. Answer: D (Ref. Essentials of General Surgery, By Peter F. Lawrence, Richard M. Bell, Merril T. Dayton, 5th edition, page
no. 154)
14. Answer: B (Ref. “Previous question”)
15. Answer: C (Ref. “Previous question”)
16. Answer: D (Ref. “Previous question”)
17. Answer: A (Ref. “Previous question”)
Active immunization is indicated in all case of tetanus, as prior infection does not confer immunity. (Ref. Bacterial
infections of the central nervous system, By Karen L.Roos, Allan R. Tunkel)
18. Answer: D (Ref. Essentials of General Surgery, By Peter F. Lawrence, Richard M. Bell, Merril T. Dayton, 5th edition, page
no. 153)

https://t.me/DentalBooksWorld
450 Triumph’s Complete Review of Dentistry

19. Answer: D (Ref. “Previous question”)


20. Answer: C
21. CORRECT: C
Extensive spasm of muscles of deglutition and respiration is often dangerous in bulbar tetanus as death is caused by spasm
of muscles of respiration.
22. Answer: D (Ref. “Previous question”)
23. Answer: D (Ref. “Previous question”)
24. Answer: A (Ref. Essentials of General Surgery, By Peter F. Lawrence, Richard M. Bell, Merril T. Dayton, 5th edition, page
no. 154)
25. Answer: D (Ref. “Previous question”)
26. Answer: C (Ref. Essentials of General Surgery, By Peter F. Lawrence, Richard M. Bell, Merril T. Dayton, 5th edition, page
no. 155)
27. Answer: D (Ref. Essentials of General Surgery, By Peter F. Lawrence, Richard M. Bell, Merril T. Dayton, 4th edition, page
no. 135)
28. Answer: D (Ref. Essentials of General Surgery, By Peter F. Lawrence, Richard M. Bell, Merril T. Dayton, 5th edition, page
no. 152)
29. Answer: B (Ref. Essentials of General Surgery, By Peter F. Lawrence, Richard M. Bell, Merril T. Dayton, 5th edition, page
no. 152)
Erysipelas is caused by Streptococcus pyogenes and is characterized by a sudden onset of red swelling of the face of
extremities. The distinctive features of erysipelas are:
•  Well-defined indurated margins, particularly along the nasolabial fold
•  Rapid progression
•  Flaccid bullae develop during the second or third day of illness, but extension to deeper soft tissues is rare
30. Answer: A (Ref. Essentials of General Surgery, By Peter F. Lawrence, Richard M. Bell, Merril T. Dayton, 5th edition, page
no. 8)
31. Answer: C (Ref. “Previous question”)
Shaving the evening before operation may increase the incidence of SSIs. Clipping shortly before surgery is better.
32. Answer: D

TUMOR, CYST, ULCER, AND SINUSES


1. Answer: C (Ref. General Surgery for BDS Students, By Sukumar, page no. 40)
2. Answer: B (Ref. “Previous question”)
3. Answer: B (Ref. Essentials of General Surgery, By Peter F. Lawrence, Richard M. Bell, Merril T. Dayton, 5th edition, page
no. 408)
4. Answer: C (Ref. Essentials of General Surgery, By Peter F. Lawrence, Richard M. Bell, Merril T. Dayton, 5th edition, page
no. 139)
5. Answer: D (Ref. Essentials of General Surgery, By Peter F. Lawrence, Richard M. Bell, Merril T. Dayton, 5th edition, page
no. 139)
6. Answer: C (Ref. Essentials of General Surgery, By Peter F. Lawrence, Richard M. Bell, Merril T. Dayton, 5th edition, page
no. 512)
7. Answer: B (Ref. Essentials of General Surgery, By Peter F. Lawrence, Richard M. Bell, Merril T. Dayton, 5th edition, page
no. 256)
8. Answer: D (Ref. Essentials of General Surgery, By Peter F. Lawrence, Richard M. Bell, Merril T. Dayton, 5th edition, page
no. 292)
9. Answer: A (Ref. Essentials of General Surgery, By Peter F. Lawrence, Richard M. Bell, Merril T. Dayton, 5th edition, page
nos. 314,249)
Hamartoma is mostly benign focal malformation, that resembles a neoplasm in the tissue of its origin.
10. Answer: C (Ref. Essentials of General Surgery, By Peter F. Lawrence, Richard M. Bell, Merril T. Dayton, 5th edition, page
no. 423)
11. Answer: A (Ref. “Previous question”)
12. Answer: C (Ref. Essentials of General Surgery, By Peter F. Lawrence, Richard M. Bell, Merril T. Dayton, 5th edition, page
no. 518)

https://t.me/DentalBooksWorld
Chapter 5 • General Surgery 451

13. Answer: D (Ref. Essentials of General Surgery, By Peter F. Lawrence, Richard M. Bell, Merril T. Dayton, 4th edition, page
no. 499)
14. Answer: D (Ref. “Previous question”)
15. Answer: A (Ref. “Previous question”)
16. Answer: A (Ref. “Previous question”)
17. Answer: D (Ref. “Previous question”)
18. Answer: D (Ref. “Previous question”)
19. Answer: C (Ref. Essentials of General Surgery, By Peter F. Lawrence, Richard M. Bell, Merril T. Dayton, 4th edition, page
no. 333)
20. Answer: B
21. Answer: C
22. Answer: B
23. Answer: A
Nodular type is most malignant. Lentigo maligna is also known as Hutchinson’s melanotic freckle.

NEOPLASIA
1. Answer: D (Ref. Essentials of General Surgery, By Peter F. Lawrence, Richard M. Bell, Merril T. Dayton, 4th edition, page
no. 211)
2. Answer: A (Ref. “Previous question”)
3. Answer: D (Ref. Essentials of General Surgery, By Peter F. Lawrence, Richard M. Bell, Merril T. Dayton, 5th edition, page
no. 512)
4. Answer: C (Ref. Essentials of General Surgery, By Peter F. Lawrence, Richard M. Bell, Merril T. Dayton, 4th edition, page
no. 140)
5. Answer: C (Ref. Essentials of General Surgery, By Peter F. Lawrence, Richard M. Bell, Merril T. Dayton, 5th edition, page
no. 512)
6. Answer: C (Ref. “Previous question”)
7. Answer: C (Ref. Essentials of General Surgery, By Peter F. Lawrence, Richard M. Bell, Merril T. Dayton, 5th edition, page
no. 252)
8. Answer: B (Ref. Essentials of General Surgery, By Peter F. Lawrence, Richard M. Bell, Merril T. Dayton, 5th edition, page
no. 316)
9. Answer: A (Ref. Essentials of General Surgery, By Peter F. Lawrence, Richard M. Bell, Merril T. Dayton, 5th edition, page
no. 518)
10. Answer: C (Ref. Essentials of General Surgery, By Peter F. Lawrence, Richard M. Bell, Merril T. Dayton, 5th edition, page
no. 429)
11. Answer: B (Ref. Essentials of General Surgery, By Peter F. Lawrence, Richard M. Bell, Merril T. Dayton, 5th edition, page
no. 429)
12. Answer: C (Ref. Essentials of General Surgery, By Peter F. Lawrence, Richard M. Bell, Merril T. Dayton, 5th edition, page
no. 291)
13. Answer: B
14. Answer: A (Ref. Essentials of General Surgery, By Peter F. Lawrence, Richard M. Bell, Merril T. Dayton, 5th edition, page
no. 155)
15. Answer: B (Ref. Essentials of General Surgery, By Peter F. Lawrence, Richard M. Bell, Merril T. Dayton, 5th edition, page
no. 155)
16. Answer: D (Ref. Essentials of General Surgery, By Peter F. Lawrence, Richard M. Bell, Merril T. Dayton, 4th edition, page
no. 267)
17. Answer: C (Ref. Essentials of General Surgery, By Peter F. Lawrence, Richard M. Bell, Merril T. Dayton, 5th edition, page
no. 155)
A Brodie abscess is a subacute osteomyelitis, which may persist for years before converting to a frank osteomyelitis.
18. Answer: B (Ref. Essentials of General Surgery, By Peter F. Lawrence, Richard M. Bell, Merril T. Dayton, 4th edition, page
no. 277)
19. Answer: B (Ref. Essentials of General Surgery, By Peter F. Lawrence, Richard M. Bell, Merril T. Dayton, 4th edition, page
no. 283)

https://t.me/DentalBooksWorld
452 Triumph’s Complete Review of Dentistry

20. Answer: B (Ref. Essentials of General Surgery, By Peter F. Lawrence, Richard M. Bell, Merril T. Dayton, 4th edition, page
no. 290)
21. Answer: C (Ref. Essentials of General Surgery, By Peter F. Lawrence, Richard M. Bell, Merril T. Dayton, 4th edition, page
no. 277)
22. Answer: B (Ref. Essentials of General Surgery, By Peter F. Lawrence, Richard M. Bell, Merril T. Dayton, 4th edition, page
no. 290)
23. Answer: B (Ref. Essentials of General Surgery, By Peter F. Lawrence, Richard M. Bell, Merril T. Dayton, 4th edition, page
no. 277)
24. Answer: C
25. Answer: D (Ref. Essentials of General Surgery, By Peter F. Lawrence, Richard M. Bell, Merril T. Dayton, 4th edition, page
no. 564)
26. Answer: B (Ref. Essentials of General Surgery, By Peter F. Lawrence, Richard M. Bell, Merril T. Dayton, 5th edition, page
no. 522)
27. Answer: C (Ref. Essentials of General Surgery, By Peter F. Lawrence, Richard M. Bell, Merril T. Dayton, 5th edition, page
no. 522)
28. Answer: C (Ref. “Previous question”)
29. Answer: D (Ref. Essentials of General Surgery, By Peter F. Lawrence, Richard M. Bell, Merril T. Dayton, 5th edition, page
no. 524)
30. Answer: B (Ref. Essentials of General Surgery, By Peter F. Lawrence, Richard M. Bell, Merril T. Dayton, 5th edition, page
no. 522)
31. Answer: C (Ref. “Previous question”)
32. Answer: C (Ref. “Previous question”)
33. Answer: A (Ref. “Previous question”)
34. Answer: B (Ref. Essentials of General Surgery, By Peter F. Lawrence, Richard M. Bell, Merril T. Dayton, 5th edition, page
no. 520)
35. Answer: C (Ref. Essentials of General Surgery, By Peter F. Lawrence, Richard M. Bell, Merril T. Dayton, 4th edition, page
no. 279)

VASCULAR DISEASES
1. Answer: B (Ref. Essentials of General Surgery, By Peter F. Lawrence, Richard M. Bell, Merril T. Dayton, 5th edition, page
no. 459)
2. Answer: D (Ref. Essentials of General Surgery, By Peter F. Lawrence, Richard M. Bell, Merril T. Dayton, 5th edition, page
no. 288)
3. Answer: C (Ref. Essentials of General Surgery, By Peter F. Lawrence, Richard M. Bell, Merril T. Dayton, 5th edition, page
no. 352)
Diabetic neuropathy can occur in patients with normal arterial circulation. Ischemic rest pain results from either ischemic
neuropathy in severe ischemia or superadded infection in a previously painless chronic ischemic limb.
4. Answer: B (Ref. Essentials of General Surgery, By Peter F. Lawrence, Richard M. Bell, Merril T. Dayton, 5th edition, page
no. 463)
BY definition, critical ischemia is severe chronic ischemia grade 1/1 and IV. In literature acute ischemia is not described as
critical. Infective gangrene with a palpable is due to the local effect of infection.
5. Answer: B (Ref. Essentials of General Surgery, By Peter F. Lawrence, Richard M. Bell, Merril T. Dayton, 5th edition, page
no. 459)
*This type of pain usually result from muscle fatigue or chronic venous congestion
6. Answer: C (Ref. Essentials of General Surgery, By Peter F. Lawrence, Richard M. Bell, Merril T. Dayton, 5th edition, page
no. 140)
Patients with ischemic ulcers should have absent pedal pulses. They start superficial with minor trauma and gradually
become deep
7. Answer: C (Ref. Essentials of General Surgery, By Peter F. Lawrence, Richard M. Bell, Merril T. Dayton, 5th edition, page
no. 180)
The cut-off level for the diagnosis of critical limb ischemia is less than 0.5
8. Answer: B (Ref. “Previous question”)

https://t.me/DentalBooksWorld
Chapter 5 • General Surgery 453

9. Answer: B (Ref. Essentials of General Surgery, By Peter F. Lawrence, Richard M. Bell, Merril T. Dayton, 5th edition, page
no. 458)
10. Answer: C (Ref. Essentials of General Surgery, By Peter F. Lawrence, Richard M. Bell, Merril T. Dayton, 5th edition, page
no. 460)
No bypass is possible without an angiography showing a distal run-off classically amputation done at a level below the
distal palpable pulse.
11. Answer: B (Ref. Essentials of General Surgery, By Peter F. Lawrence, Richard M. Bell, Merril T. Dayton, 5th edition, page
no. 441)
Detached emboli from DVT causes pulmonary embolism.
12. Answer: C (Ref. Essentials of General Surgery, By Peter F. Lawrence, Richard M. Bell, Merril T. Dayton, 5th edition, page
no. 155)
Diabetics are more prone to develop infection because of impaired immune response to infection. This is the reason why
signs of inflammation are not well apparent except late.
13. Answer: D (Ref. “Previous question”)
14. Answer: C (Ref. Essentials of General Surgery, By Peter F. Lawrence, Richard M. Bell, Merril T. Dayton, 5th edition, page
no. 463)
15. Answer: B (Ref. Essentials of General Surgery, By Peter F. Lawrence, Richard M. Bell, Merril T. Dayton, 5th edition, page
no. 451)
16. Answer: D (Ref. Essentials of General Surgery, By Peter F. Lawrence, Richard M. Bell, Merril T. Dayton, 5th edition, page
no. 459)
Because of risk of bleeding, thrombolysis is contraindicated in patients with history of stroke within the previous 2 months
or history of major surgery within the previous 10 days; thrombolysis is effective in fresh thrombosis. Avoid thrombolysis
in limbs with signs of late ischemia as it may need up to 72 hours to complete the treatment.
17. Answer: B (Ref. Essentials of General Surgery, By Peter F. Lawrence, Richard M. Bell, Merril T. Dayton, 5th edition, page
no. 377)
18. Answer: B (Ref. Essentials of General Surgery, By Peter F. Lawrence, Richard M. Bell, Merril T. Dayton, 5th edition, page
no. 181)
• Marked muscle turgidity and swelling is a very late and irreversible sign.
• Delayed capillary refilling is a sign of severe ischemia, but it also denotes that the capillary circulation is still intact.
• Sensory loss is a late sign that starts with loss of touch and progresses to loss deep sensations. However, the patient can
gradually regain sensations over months after revascularization.
• Paralysis of small foot muscles is difficult to elicit clinically because movements can be done by leg muscles. This is the
reason why it is usually passed unnoticed until leg muscles are paralyzed in a later Stage.
19. Answer: D (Ref. “Previous question”)
20. Answer: A (Ref. Essentials of General Surgery, By Peter F. Lawrence, Richard M. Bell, Merril T. Dayton, 5th edition, page
no. 465)
21. Answer: C (Ref. Essentials of General Surgery, By Peter F. Lawrence, Richard M. Bell, Merril T. Dayton, 5th edition, page
no. 465)
22. Answer: C (Ref. Essentials of General Surgery, By Peter F. Lawrence, Richard M. Bell, Merril T. Dayton, 5th edition, page
no. 463)
23. Answer: D (Ref. Essentials of General Surgery, By Peter F. Lawrence, Richard M. Bell, Merril T. Dayton, 5th edition, page
no. 460)
24. Answer: B (Ref. Essentials of General Surgery, By Peter F. Lawrence, Richard M. Bell, Merril T. Dayton, 5th edition, page
no. 460)
25. Answer: C (Ref. Essentials of General Surgery, By Peter F. Lawrence, Richard M. Bell, Merril T. Dayton, 5th edition, page
no. 449)
26. Answer: B (Ref. Essentials of General Surgery, By Peter F. Lawrence, Richard M. Bell, Merril T. Dayton, 5th edition, page
no. 459)
27. Answer: B (Ref. “Previous question”)
28. Answer: D (Ref. Essentials of General Surgery, By Peter F. Lawrence, Richard M. Bell, Merril T. Dayton, 5th edition, page
no. 354)
29. Answer: C (Ref. “Previous question”)

https://t.me/DentalBooksWorld
454 Triumph’s Complete Review of Dentistry

30. Answer: D (Ref. Essentials of General Surgery, By Peter F. Lawrence, Richard M. Bell, Merril T. Dayton, 5th edition, page
no. 252)
31. Answer: D (Ref. “Previous question”)
32. Answer: B (Ref. Essentials of General Surgery, By Peter F. Lawrence, Richard M. Bell, Merril T. Dayton, 5th edition, page
no. 278)
33. Answer: C (Ref. Essentials of General Surgery, By Peter F. Lawrence, Richard M. Bell, Merril T. Dayton, 5th edition, page
no. 458)
34. Answer: C (Ref. Essentials of General Surgery, By Peter F. Lawrence, Richard M. Bell, Merril T. Dayton, 5th edition, page
no. 472)
Proximal occlusion of the left subclavian artery cause retrograde flow of blood through the left vertebral artery “stealing”
blood from the basilar circulation and causing transient dizziness and syncope with arm exercise.
35. Answer: D (Ref. Essentials of General Surgery, By Peter F. Lawrence, Richard M. Bell, Merril T. Dayton, 5th edition, page
no. 459)
36. Answer: C (Ref. Essentials of General Surgery, By Peter F. Lawrence, Richard M. Bell, Merril T. Dayton, 5th edition, page
no. 469)
37. Answer: C (Ref. Essentials of General Surgery, By Peter F. Lawrence, Richard M. Bell, Merril T. Dayton, 5th edition, page
no. 452)
38. Answer: C
39. Answer: D (Ref. Essentials of General Surgery, By Peter F. Lawrence, Richard M. Bell, Merril T. Dayton, 5th edition, page
no. 565)
Thromboangiitis obliterans (Buerger’s disease) is an occlusive disease of small and medium-sized arteries of both upper
and lower limbs.
40. Answer: D (Ref. “Previous question”)
41. Answer: B (Ref. “Previous question”)
42. Answer: A (Ref. “Previous question”)
43. Answer: A (Ref. Essentials of General Surgery, By Peter F. Lawrence, Richard M. Bell, Merril T. Dayton, 5th edition, page
no. 469)
It is more common in women with a ratio of 5:1, 90% of patients are below 40 years of age.
44. Answer: D (Ref. Essentials of General Surgery, By Peter F. Lawrence, Richard M. Bell, Merril T. Dayton, 5th edition, page
no. 457)
45. Answer: E (Ref. Essentials of General Surgery, By Peter F. Lawrence, Richard M. Bell, Merril T. Dayton, 5th edition, page
no. 459)
46. Answer: D (Ref. Essentials of General Surgery, By Peter F. Lawrence, Richard M. Bell, Merril T. Dayton, 5th edition, page
nos. 322, 383)
47. Answer: B (Ref. Essentials of General Surgery, By Peter F. Lawrence, Richard M. Bell, Merril T. Dayton, 5th edition, page
no. 383)
48. Answer: D (Ref. “Previous question”)
49. Answer: C (Ref. Essentials of General Surgery, By Peter F. Lawrence, Richard M. Bell, Merril T. Dayton, 5th edition, page
no. 469)
Adson’s test is a test for thoracic outflow syndrome. Positive sign of numbness or tingling in the hand indicates the brachial
plexus are compressed at the site of scalene muscle.
50. Answer: D (Ref. Essentials of General Surgery, By Peter F. Lawrence, Richard M. Bell, Merril T. Dayton, 5th edition, page
no. 461)
51. Answer: D (Ref. Essentials of General Surgery, By Peter F. Lawrence, Richard M. Bell, Merril T. Dayton, 5th edition, page
no. 377)
52. Answer: E (Ref. Essentials of General Surgery, By Peter F. Lawrence, Richard M. Bell, Merril T. Dayton, 5th edition, page
no. 94)
53. Answer: C (Ref. Essentials of General Surgery, By Peter F. Lawrence, Richard M. Bell, Merril T. Dayton, 5th edition, page
no. 474)
54. Answer: B (Ref. “Previous question”)
55. Answer: A (Ref. Essentials of General Surgery, By Peter F. Lawrence, Richard M. Bell, Merril T. Dayton, 5th edition, page
no. 477)
56. Answer: C

https://t.me/DentalBooksWorld
Chapter 5 • General Surgery 455

57. Answer: D (Ref. Essentials of General Surgery, By Peter F. Lawrence, Richard M. Bell, Merril T. Dayton, 5th edition, page
no. 477)
58. Answer: C (Ref. Essentials of General Surgery, By Peter F. Lawrence, Richard M. Bell, Merril T. Dayton, 5th edition, page
no. 8)
59. Answer: A (Ref. Essentials of General Surgery, By Peter F. Lawrence, Richard M. Bell, Merril T. Dayton, 5th edition, page
no. 450)
60. Answer: C (Ref. Essentials of General Surgery, By Peter F. Lawrence, Richard M. Bell, Merril T. Dayton, 5th edition, page
no. 140)
61. Answer: B (Ref. Essentials of General Surgery, By Peter F. Lawrence, Richard M. Bell, Merril T. Dayton, 5th edition, page
no. 493)
62. Answer: D (Ref. Essentials of General Surgery, By Peter F. Lawrence, Richard M. Bell, Merril T. Dayton, 5th edition, page
no. 479)
This is the clinical picture of the postphlebitic limb with lower leg pigmentation and eczema. Edema is always hard pitting
due to the high protein content. It is never pitting.
63. Answer: D (Ref. Essentials of General Surgery, By Peter F. Lawrence, Richard M. Bell, Merril T. Dayton, 5th edition, page
no. 139)
Venous ulcers are painful. They are usually superficial.
64. Answer: A (Ref. Essentials of General Surgery, By Peter F. Lawrence, Richard M. Bell, Merril T. Dayton, 5th edition, page
no. 479)
Severe prolonged venous congestion results in white cell trapping and neutrophile activation causing skin and SC tissue
damage. Such tissue damage has all the features of acute inflammation (nonbacterial acute inflammation). The basic
treatment of acute Iipodermatosclerosis is elevation with external compression to relieve venous hypertension, and anti-
inflammatory drugs to suppress the acute inflammatory condition. Antibiotics have no role.
65. Answer: B (Ref. “Previous question”)
66. Answer: A (Ref. Essentials of General Surgery, By Peter F. Lawrence, Richard M. Bell, Merril T. Dayton, 5th edition, page
no. 463)
Acute superficial thrombophlebitis is a nonbacterial inflammatory condition, treated by systemic anti- inflammatory drugs
and external compression. There is no role for antibiotics in the treatment (not even for prophylaxis), local treatment is useless.
67. Answer: C (Ref. Essentials of General Surgery, By Peter F. Lawrence, Richard M. Bell, Merril T. Dayton, 5th edition, page
no. 477)
68. Answer: C (Ref. Essentials of General Surgery, By Peter F. Lawrence, Richard M. Bell, Merril T. Dayton, 5th edition, page
no. 484)
69. Answer: B (Ref. Essentials of General Surgery, By Peter F. Lawrence, Richard M. Bell, Merril T. Dayton, 5th edition, page
no. 479)
The incidence of recurrence after Trendelenburg operation alone is very high. Stripping of leg long saphenous may cause
saphenous nerve injury and is not needed because leg perforators are connected to the posterior arch vein and not the long
saphenous.
70. Answer: D (Ref. “Previous question”)
Swelling is a common feature in acute DVT. Phlegmasia cerulea dolens presents with swelling and deep cyanosis, and may
be cold in the stage of impending venous gangrene. Fever is a common finding in DVT. Phlegmasia alba presents with a
swollen, pale, and painful limb. A painful cold cyanosed limb without swelling is possibly ischemic.
71. Answer: B (Ref. Essentials of General Surgery, By Peter F. Lawrence, Richard M. Bell, Merril T. Dayton, 5th edition, page
no. 450)
The classical treatment of acute DVT is to start with heparin anticoagulation (whether low molecular weight or
unfractionated) for 5 days overlap and continue with oral anticoagulation (warfarin) for the treatment duration which
is usually from 3 to 6 months. Giving low-molecular-weight heparin all through the treatment period is very expensive,
inconvenient to the patient, and not needed except in very special situations (e.g., during pregnancy). Oral anticoagulants
are not suitable to start with as they take several days to give therapeutic anticoagulation; during this period there is an
unaccepted high incidence of PE. Low-dose aspirin is an antiplatelet and not anticoagulant, and it is not the treatment of
acute DVT.
72. Answer: A (Ref. “Previous question”)
Chronic organized thrombi are not affected by thrombolytic drugs. Massive PE with cardiopulmonary compromise (acute
pulmonary hypertension) is a classical indication of thrombolysis. The classical treatment of DVT is anticoagulation.

https://t.me/DentalBooksWorld
456 Triumph’s Complete Review of Dentistry

Thrombolysis is only used with impending venous gangrene. Thrombolysis takes several hours to act. So it can be used in
acute thrombotic ischemia if the limb is not critically ischemic. Acute ischemia with threatened sensory or motor loss (or
starting muscle turgidity) is not suitable for thrombolytic therapy.
73. Answer: E (Ref. Essentials of General Surgery, By Peter F. Lawrence, Richard M. Bell, Merril T. Dayton, 5th edition, page
no. 354)
74. Answer: A (Ref. Essentials of General Surgery, By Peter F. Lawrence, Richard M. Bell, Merril T. Dayton, 5th edition, page
no. 450)
75. Answer: E (Ref. “Previous question”)
76. Answer: D (Ref. “Previous question”)
77. Answer: D (Ref. Essentials of General Surgery, By Peter F. Lawrence, Richard M. Bell, Merril T. Dayton, 5th edition, page
no. 437)
Phlegmasia cerulea dolens is an uncommon severe form of deep vein thrombosis which results from extensive thrombotic
occlusion of the major and the collateral veins of an extremity.
78. Answer: D (Ref. Essentials of General Surgery, By Peter F. Lawrence, Richard M. Bell, Merril T. Dayton, 5th edition, page
no. 474)
79. Answer: A (Ref. “Previous question”)
80. Answer: A (Ref. Essentials of General Surgery, By Peter F. Lawrence, Richard M. Bell, Merril T. Dayton, 5th edition, page
no. 126)
81. Answer: E (Ref. “Previous question”)
82. Answer: E (Ref. Essentials of General Surgery, By Peter F. Lawrence, Richard M. Bell, Merril T. Dayton, 5th edition, page
no. 462)
83. Answer: E (Ref. “Previous question”)
84. Answer: E (Ref. “Previous question”)
85. Answer: E (Ref. “Previous question”)
86. Answer: E (Ref. “Previous question”)
87. Answer: E (Ref. “Previous question”)

GALL BLADDER AND BILE DUCTS


1. Answer: C (Ref. Essentials of General Surgery, By Peter F. Lawrence, Richard M. Bell, Merril T. Dayton, 5th edition, page no. 338)
Choledochal cysts are congenital anomalies of the biliary tract with varying degrees of cystic dilation occurring at various
segments of the biliary tree (extrahepatic or intrahepatic). Choledochal cysts are predominantly a disorder of children and
young adults. (Ref. Walkers pediatric gastrointestinal disease by Ronald)
2. Answer: D (Ref. Essentials of General Surgery, By Peter F. Lawrence, Richard M. Bell, Merril T. Dayton, 5th edition, page
no. 340)
Cholelithiasis is the term used for formation of gallstones. Mostly gallstones are asymptomatic. They are classified on the basis
of their chemical components as cholesterol or pigment stones. (Ref. Chemical Pharmacy, By NA Hughes, Jeffery D. Hughes)
3. Answer: A (Ref. Essentials of General Surgery, By Peter F. Lawrence, Richard M. Bell, Merril T. Dayton, 5th edition, page
no. 263)
4. Answer: D (Ref. Essentials of General Surgery, By Peter F. Lawrence, Richard M. Bell, Merril T. Dayton, 5th edition, page
no. 438)
5. Answer: C (Ref. Essentials of General Surgery, By Peter F. Lawrence, Richard M. Bell, Merril T. Dayton, 5th edition, page
no. 329)
6. Answer: B (Ref. Essentials of General Surgery, By Peter F. Lawrence, Richard M. Bell, Merril T. Dayton, 5th edition, page
no. 367)
7. Answer: D (Ref. Essentials of General Surgery, By Peter F. Lawrence, Richard M. Bell, Merril T. Dayton, 5th edition, page
no. 329)
8. Answer: D (Ref. Essentials of General Surgery, By Peter F. Lawrence, Richard M. Bell, Merril T. Dayton, 5th edition, page
no. 297)
9. Answer: D
10. Answer: B (Ref. Essentials of General Surgery, By Peter F. Lawrence, Richard M. Bell, Merril T. Dayton, 5th edition, page
no. 564)
11. Answer: C

https://t.me/DentalBooksWorld
Chapter 5 • General Surgery 457

12. Answer: C
13. Answer: B
14. Answer: C
15. Answer: C
16. Answer: C
17. Answer: A
18. Answer: D
19. Answer: D (Ref. Essentials of General Surgery, By Peter F. Lawrence, Richard M. Bell, Merril T. Dayton, 5th edition, page
no. 336)
20. Answer: B
21. Answer: C (Ref. Essentials of General Surgery, By Peter F. Lawrence, Richard M. Bell, Merril T. Dayton, 5th edition, page
no. 327)
22. Answer: D
23. Answer: B

The Pancreas
24. Answer: D
Delta cells produce somatostatin.
25. Answer: D
26. Answer: A
27. Answer: D (Ref. Essentials of General Surgery, By Peter F. Lawrence, Richard M. Bell, Merril T. Dayton, 5th edition, page
no. 356)
Pancreatic pseudocyst results from disruption of the main duct. They can cause pain, duodenal and biliary obstruction,
can erode into adjacent structures more commonly the colon or duodenum. (Ref. Atlas of Gastrointestinal Surgery, Vol 1,
By John L. Cameron, Corinne Sandone)
28. Answer: D
29. Answer: D (Ref. Essentials of General Surgery, By Peter F. Lawrence, Richard M. Bell, Merril T. Dayton, 5th edition, page
no. 356)
30. Answer: B (Ref. Essentials of General Surgery, By Peter F. Lawrence, Richard M. Bell, Merril T. Dayton, 5th edition, page
no. 353)
31. Answer: A
32. Answer: D (Ref. Essentials of General Surgery, By Peter F. Lawrence, Richard M. Bell, Merril T. Dayton, 5th edition, page
no. 259)
33. Answer: E (Ref. Essentials of General Surgery, By Peter F. Lawrence, Richard M. Bell, Merril T. Dayton, 5th edition, page
no. 259)
34. Answer: D (Ref. Essentials of General Surgery, By Peter F. Lawrence, Richard M. Bell, Merril T. Dayton, 5th edition, page
no. 586)
35. Answer: D (Ref. Essentials of General Surgery, By Peter F. Lawrence, Richard M. Bell, Merril T. Dayton, 4th edition, page
no. 362)

THE LIVER
1. Answer: B (Ref. Essentials of General Surgery, By Peter F. Lawrence, Richard M. Bell, Merril T. Dayton, 5th edition, page
no. 367)
2. Answer: D (Ref. Essentials of General Surgery, By Peter F. Lawrence, Richard M. Bell, Merril T. Dayton, 5th edition, page
no. 374)
3. Answer: D (Ref. Essentials of General Surgery, By Peter F. Lawrence, Richard M. Bell, Merril T. Dayton, 5th edition, page
no. 374)
4. Answer: D (Ref. Essentials of General Surgery, By Peter F. Lawrence, Richard M. Bell, Merril T. Dayton, 5th edition, page
no. 370)
5. Answer: C (Ref. Essentials of General Surgery, By Peter F. Lawrence, Richard M. Bell, Merril T. Dayton, 5th edition, page
no. 379)

https://t.me/DentalBooksWorld
458 Triumph’s Complete Review of Dentistry

6. Answer: C (Ref. Essentials of General Surgery, By Peter F. Lawrence, Richard M. Bell, Merril T. Dayton, 4th edition, page
no. 374)
7. Answer: B (Ref. “Previous question”)
8. Answer: C (Ref. Essentials of General Surgery, By Peter F. Lawrence, Richard M. Bell, Merril T. Dayton, 4th edition, page
no. 380)
9. Answer: C (Ref. “Previous question”)
10. Answer: D (Ref. Essentials of General Surgery, By Peter F. Lawrence, Richard M. Bell, Merril T. Dayton, 4th edition, page
no. 367)
11. Answer: C (Ref. “Previous question”)
12. Answer: B (Ref. Essentials of General Surgery, By Peter F. Lawrence, Richard M. Bell, Merril T. Dayton, 4th edition, page
no. 379)
It is a procedure that may be used to reduce portal hypertension and its complications, especially variceal bleeding and is
done with a small wire-mesh coil (stent) into a liver vein by a radiologist.
13. Answer: C (Ref. “Previous question”)
14. Answer: C (Ref. Essentials of General Surgery, By Peter F. Lawrence, Richard M. Bell, Merril T. Dayton, 4th edition, page no. 383)
Caroli’s disease is caused by nonobstructive saccular or fusiform dilatation of the intrahepatic bile ducts. It can be
associated with stone formation and recurrent cholangitis.
15. Answer: A (Ref. Essentials of General Surgery, By Peter F. Lawrence, Richard M. Bell, Merril T. Dayton, 4th edition, page
no. 369)
16. Answer: D (Ref. Essentials of General Surgery, By Peter F. Lawrence, Richard M. Bell, Merril T. Dayton, 5th edition, page
no. 367)
17. Answer: D (Ref. Essentials of General Surgery, By Peter F. Lawrence, Richard M. Bell, Merril T. Dayton, 5th edition, page
no. 372)
18. Answer: D (Ref. Essentials of General Surgery, By Peter F. Lawrence, Richard M. Bell, Merril T. Dayton, 5th edition, page
no. 372, 374)
19. Answer: D (Ref. Essentials of General Surgery, By Peter F. Lawrence, Richard M. Bell, Merril T. Dayton, 5th edition, page
no. 372, 374)
20. Answer: C (Ref. Essentials of General Surgery, By Peter F. Lawrence, Richard M. Bell, Merril T. Dayton, 4th edition, page no. 370)
21. Answer: B (Ref. “Previous question”)
22. Answer: B (Ref. Essentials of General Surgery, By Peter F. Lawrence, Richard M. Bell, Merril T. Dayton, 5th edition, page no. 367)
23. Answer: E (Ref. “Previous question”)
24. Answer: C (Ref. “Previous question”)
25. Answer: C (Ref. Essentials of General Surgery, By Peter F. Lawrence, Richard M. Bell, Merril T. Dayton, 4th edition, page
no. 198)
Pringle maneuver is compression of the portal triad structures with a noncrushing vascular clamp for hepatic inflow
control.
26. Answer: D (Ref. Essentials of General Surgery, By Peter F. Lawrence, Richard M. Bell, Merril T. Dayton, 5th edition, page
no. 119)
27. Answer: C (Ref. Essentials of General Surgery, By Peter F. Lawrence, Richard M. Bell, Merril T. Dayton, 5th edition, page no. 82)
The Denver shunt is used to treat both cirrhotic and malignant effusions in the peritoneal and pleural cavities.
28. Answer: C (Ref. Essentials of General Surgery, By Peter F. Lawrence, Richard M. Bell, Merril T. Dayton, 4th edition, page
no. 374)
Budd–Chiari syndrome is caused by hepatic venous outflow obstruction. It presents with classical triad of abdominal pain,
ascites, and liver enlargement.
29. Answer: A (Ref. “Previous question”)
30. Answer: B (Ref. “Previous question”)

THE SPLEEN
1. Answer: C (Ref. Essentials of General Surgery, By Peter F. Lawrence, Richard M. Bell, Merril T. Dayton, 5th edition, page
no. 430)
2. Answer: A (Ref. Essentials of General Surgery, By Peter F. Lawrence, Richard M. Bell, Merril T. Dayton, 5th edition, page
no. 436)

https://t.me/DentalBooksWorld
Chapter 5 • General Surgery 459

Splenic injury may cause peritoneal irritation which includes left upper quadrant tenderness to palpation and pain medial
to the top of the left shoulder joint which is referred as Kehr sign or omalgia.
3. Answer: D (Ref. “Previous question”)
4. Answer: D (Ref. “Previous question”)
5. Answer: D (Ref. “Previous question”)
6. Answer: A (Ref. Essentials of General Surgery, By Peter F. Lawrence, Richard M. Bell, Merril T. Dayton, 5th edition, page no. 429)
7. Answer: B (Ref. Essentials of General Surgery, By Peter F. Lawrence, Richard M. Bell, Merril T. Dayton, 5th edition, page
no. 430)
8. Answer: D (Ref. Essentials of General Surgery, By Peter F. Lawrence, Richard M. Bell, Merril T. Dayton, 5th edition, page
no. 439)
9. Answer: A (Ref. Essentials of General Surgery, By Peter F. Lawrence, Richard M. Bell, Merril T. Dayton, 5th edition, page
no. 439)
10. Answer: B (Ref. Essentials of General Surgery, By Peter F. Lawrence, Richard M. Bell, Merril T. Dayton, 5th edition, page
no. 437)
11. Answer: C (Ref. “Previous question”)
12. Answer: C (Ref. “Previous question”)
13. Answer: A (Ref. Essentials of General Surgery, By Peter F. Lawrence, Richard M. Bell, Merril T. Dayton, 5th edition, page
no. 429)
14. Answer: C (Ref. Essentials of General Surgery, By Peter F. Lawrence, Richard M. Bell, Merril T. Dayton, 5th edition, page
no. 429)
15. Answer: D (Ref. Essentials of General Surgery, By Peter F. Lawrence, Richard M. Bell, Merril T. Dayton, 5th edition, page
no. 428)

https://t.me/DentalBooksWorld
6 Embryology, Head and Neck
Anatomy, and General Histology

SYNOPSIS
nn Embryology

BASICS OF CELL CYCLE


Eukaryotic cells pass through a series of stages collectively known as the cell cycle: The resting phase, two gap phases (G1 and
G2) and S (synthesis) phase, and the M phase. Here G1, S, and G2 are collectively known as the interphase.
• The period between mitotic divisions (i.e., G1, S, and G2) is known as the interphase.
• Interphase of the cell cycle—the longest phase or the phase where the cell spends its most of its time, it includes the G₁, S,
and G₂ phases.
• The cell replicates its DNA before mitosis. The final stage is cell division or the M phase (mitosis).

G₁ - Growth
G₂
S - DNA synthesis
S M
G₂ - Growth and
preparaon for
mitosis

G₁ M - Mitosis
(cell division)

CELL CYCLE
Resting—G0 or gap phase The cell leaves the cycle and stops dividing
Interphase G1 Synthesis of proteins occurs
Interphase S DNA replication occurs (the genetic material is duplicated)
Interphase G2 The cell continues to grow
M phase Phase of cell division (mitosis partitions the genetic material and the cell divides)
In the mitotic (M) phase, the cell splits itself into two distinct cells, often called “daughter cells.”
In the final phase, cytokinesis, the new cell is completely divided

https://t.me/DentalBooksWorld
Chapter 6 • Embryology, Head and Neck Anatomy, and General Histology 461

The cell cycle

Interphase
S
(growth and DNA
replicaon)
G₂
G₁ (growth and final
(growth) preparaons for
G₀ M
M ito sis division)

se
s
esi
se

Metaphase

ha
se
ha

se
kin
p

ap
o

a
ha
t Pro
Cy

Anaph

et
op

m
Tel

ro P
Cell division Cell division is the process by which a parent cell divides into two or more daughter cells.
Cell division is usually a small segment of a larger cell cycle. This type of cell division in
eukaryotes is known as mitosis, and leaves the daughter cells capable of dividing again.
Meiosis A type of cell division that results in four daughter cells each with half the number of
chromosomes of the parent cell, as in the production of gametes.

MITOSIS
• Mitosis is a form of eukaryotic cell division that produces two daughter cells with the same genetic composition as the
parent cell.
• The replicated chromosomes are attached to a “mitotic apparatus” that aligns them and then separates the sister chromatids
to produce an even partitioning of the genetic material.
• This separation of the genetic material in a mitotic nuclear division (karyokinesis) is followed by a separation of the cell
cytoplasm in a cellular division (cytokinesis) to produce two daughter cells.
• Mitosis, although a continuous process, is conventionally divided into five stages: prophase, prometaphase, metaphase,
anaphase, and telophase.
PHASES OF MITOSIS
Prophase • Over half of mitosis is occupied by prophase
• In this phase:
–– The nuclear membrane breaks down to form a number of small vesicles
–– The nucleolus disintegrates
–– Centrosome duplicates itself to form two daughter centrosomes that migrate to opposite ends of
the cell (NEET 2017)
–– The centrosomes organize the production of microtubules that form the spindle fibers that constitute
the mitotic spindle
–– Each replicated chromosome can now be seen to consist of two identical  chromatids  (sister
chromatids) held together by a structure known as the centromere
Prometaphase • The chromosomes, led by their centromeres, migrate to the equatorial plane in the mid-line of the
cell—at right angles to the axis formed by the centrosomes. This region of the mitotic spindle is known
as the metaphase plate
Metaphase • The chromosomes align themselves along the metaphase plate of the spindle apparatus

https://t.me/DentalBooksWorld
462 Triumph’s Complete Review of Dentistry

Anaphase • The shortest stage of mitosis


• The centromeres divide and the sister chromatids of each chromosome are pulled apart—or “disjoin”—
and move to the opposite ends of the cell, pulled by spindle fibers attached to the kinetochore regions
• The separated sister chromatids are now referred to as daughter chromosomes
Telophase • The final stage of mitosis, and a reversal of many of the processes observed during prophase
• The nuclear membrane reforms around the chromosomes grouped at either pole of the cell, the
chromosomes uncoil and become diffuse, and the spindle fibers disappear
Cytokinesis • The final cellular division to form two new cells
• The cell then enters the interphase—the interval between mitotic divisions

MEIOSIS
Meiosis is a special type of cell division necessary for sexual reproduction. In animals, meiosis produces gametes like sperm
and egg cells, while in other organisms like fungi, it generates spores. Meiosis begins with one diploid cell containing two
copies of each chromosome—one from the organism’s mother and one from its father—and produces four haploid cells
containing one copy of each chromosome.

Meiosis
• Meiosis is the form of eukaryotic cell division that produces haploid sex cells or gametes (which contain a single copy of
each chromosome) from diploid cells (which contain two copies of each chromosome).
• The process takes the form of one DNA replication followed by two successive nuclear and cellular divisions (meiosis I
and meiosis II).
• As in mitosis, meiosis is preceded by a process of DNA replication that converts each chromosome into two sister
chromatids.

Meiosis I
• Meiosis I separates the pairs of homologous chromosomes.
• In meiosis I, a special cell division reduces the cell from diploid to haploid.

Prophase I
The homologous chromosomes pair and exchange DNA to form recombinant chromosomes.
Prophase I is Divided into Five Phases:
Leptotene: Chromosomes start to condense
Zygotene: Homologous chromosomes become closely associated (synapsis) to form pairs of chromosomes (bivalents)
consisting of four chromatids (tetrads)
Pachytene: Crossing over between pairs of homologous chromosomes to form chiasmata
Diplotene: Homologous chromosomes start to separate but remain attached by chiasmata
Diakinesis: Homologous chromosomes continue to separate, and chiasmata moves to the ends of the chromosomes

Prometaphase I Spindle apparatus is formed and chromosomes attach to spindle fibers by kinetochores (NEET 2017)
Metaphase I Homologous pairs of chromosomes (bivalents) are arranged as a double row along the metaphase
plate. The arrangement of the paired chromosomes with respect to the poles of the spindle apparatus
is random along the metaphase plate
Anaphase I The homologous chromosomes in each bivalent are separated and move to the opposite poles of the
cell

https://t.me/DentalBooksWorld
Chapter 6 • Embryology, Head and Neck Anatomy, and General Histology 463

Telophase I The chromosomes become diffuse and the nuclear membrane reforms
Cytokinesis
The final cellular division to form two new cells, followed by meiosis II. Meiosis I is a reduction division: The original diploid
cell had two copies of each chromosome; the newly formed haploid cells have one copy of each chromosome.
Meiosis II • Meiosis II separates each chromosome into two chromatids
• The events of meiosis II are analogous to those of a mitotic division, although the number of
chromosomes involved has been halved

Outcome of Meiosis
Generates genetic diversity through:
• The exchange of genetic material between homologous chromosomes during meiosis I.
• The random alignment of maternal and paternal chromosomes in meiosis I.
• The random alignment of the sister chromatids at meiosis II.

Mitosis Parent cell Meiosis Parent cell

DNA replicates
DNA replicates

2 daughter
cells
2 daughter
cells
4 daughter
cells

Gametogenesis—Spermatogenesis and Oogenesis


• The origin and development of gametes is called gametogenesis.
• This may be divided into spermatogenesis and oogenesis.
• Production of sperms occurs in the testes in seminiferous tubules.
• Spermatogenesis is initiated through hormonal controls in the hypothalamus.
• Spermatogenesis deals with the development of male sex cells called sperms in the male gonad or testis.
• The process of differentiation of a simple diploid spermatogonium into a spermatid is known as spermatogenesis.
• Spermatogenesis can be divided into three phases: (1) proliferation and differentiation of spermatogonia, (2) meiosis, and
(3) spermiogenesis, a complex process that transforms round spermatids after meiosis into a complex structure called the
spermatozoon.
• It involves both mitotic and meiotic divisions and extensive cellular remodeling.
• In humans, the process of spermatogenesis starts at puberty and continues throughout the entire life span of the individual.

https://t.me/DentalBooksWorld
464 Triumph’s Complete Review of Dentistry

Spermatogenesis and Major Events in the Life Cycle of a Sperm


• The process of differentiation of a simple diploid spermatogonium into a spermatid is known as spermatogenesis.
• It involves both mitotic and meiotic divisions and extensive cellular remodeling.
Spermatogenesis
Spermatogonium
• Mitosis
• Meiosis
• Spermiogenesis
–– Head
–– Midpiece Primary spermatocyte
(diploid number) 46
–– Tail

Capacitation
Secondary spermatocytes
23 23

Spermatids 23 23 23 23

Head

Middle piece

Sperm

Lifespan of spermatozoa • Puberty through life


• 30 × 10 to the power of six per day
• 60–75 days for sperm production (approximately 64 days)
• 10–14 days for transport (epididymis)
• The volume of ejaculation ranges from 2 to 6 ml (3.5 ml approximately) usually and
contains 20–100 (NEET 2017) million per ml of ejaculate (approximately 40 million per
ejaculate)
• Stored in vas deferens and epididymis before ejaculation
The evolution process takes approximately 74 days. Of these 74 days, the seminiferous tubule is involved for about
50 days (NEET 2017).
Each stage of spermatogenesis has a specific time frame:
• 16–18 days for spermatogonia
• 23 days for primary spermatocytes
• 1 day for secondary spermatocytes
• About 23 days for spermatids

Primary spermatocyte Produces four sperm cells with a haploid number of chromosomes (23 chromosomes)
Infertility/sterile Less than 10 million sperms/ml (NEET 2017)
Sperm becomes fully motile only after entering female reproductive tract

https://t.me/DentalBooksWorld
Chapter 6 • Embryology, Head and Neck Anatomy, and General Histology 465

• Semen provides fructose for energy and an alkaline pH for protection against the acid environment of the vagina and
it dilutes the sperm to improve motility. Sperm moves 2–3 mm per minute.
• Motility is slower in the acidic vaginal environment and faster in the alkaline uterine environment.
• Failure of sperm to achieve motility is a cause of male infertility; for potential fertility, at least 40% should be motile
by 2 hours after ejaculation.
• Mitochondria provides adenosine triphosphate (ATP) for independent sperm motility.
• As phagocytosis by leukocytes begins within few hours, most sperms do not survive for more than 48 hours; however,
the maximum survival time is 80 hours.
• Fertilization begins with contact between the sperm and secondary oocyte, arrested in the metaphase of the second
meiotic division.
• Fertilization usually occurs in the upper third of the fallopian tube, usually in the ampulla.
• Before fertilization, the sperm must undergo two final maturational changes—capacitation and the acrosome
reaction.
Capacitation • It involves the removal of the glycoprotein coat and seminal plasma proteins from the
plasma membrane over the acrosome (head of the sperm), which allows the acrosomal
reaction to occur
• Capacitation takes about 7 hours (PGI 2015) and usually occurs in the fallopian tubule
while the sperm is attached to the tubal epithelial lining
• It is a transient process that lasts 50–240 minutes
Acrosome reaction • The acrosome is a sac-like structure containing many enzymes—acid glycohydrases,
proteases, phosphatases, esterases, and hyaluronidase. Acrosin, a serine protease, is the
most important
• The capacitated sperm binds to the zona pellucida of the ovum, initiating the acrosome
reaction (sperm activation)

OOGENESIS
Oogenesis is the development of female sex cells called ova or eggs in the female gonad or ovary.

Oogonium
2n

2n 2n Mitosis

Oogonia
2n 2n 2n 2n

cell growth

2n Primary Oocyte

Reduction
Meiosis 1
division

n
n Secondary Oocyte
and first polar body

Meiosis 2

Ovum and second polar body


n n

https://t.me/DentalBooksWorld
466 Triumph’s Complete Review of Dentistry

2n
Mitosis
2n 2n

Oogonia 2n 2n 2n 2n

Primary Oocyte 2n

1st meiotic division n 1st polar body

Secondary Oocyte n
n 2nd polar body
2nd meiotic division

Ovum/egg cell n

Pharyngeal Arches

Pharyngeal Arch Derivatives


Pharyngeal Nerve Artery Neural crest Muscles Ligaments
arch (skeletal structures)
1 Trigeminal (V) Maxillary artery Mandible, maxilla, Muscles of Anterior ligament
(Maxillary/ (terminal branches) malleus, incus mastication, of malleus,
mandibular) mylohyoid, tensor sphenomandibular
tympanic, ant. belly ligament
digastric
2 Facial (VII) Stapedial Stapes, styloid Muscles of facial Stylohyoid
(Hyoid) (embryonic) process, lesser cornu expression, ligament
caroticotympanic of hyoid, upper part stapedius,
(adult) of body of hyoid stylohyoid, post.
bone belly digastric
3 Glossopharyngeal Common carotid, Greater cornu of Stylopharyngeus
(IX) internal carotid hyoid, lower part of
arteries body of hyoid bone
4 Vagus (X) superior Part of aortic arch Thyroid, cricoid, Cricothyroid, soft
laryngeal branch (left), part right arytenoid, palate levator veli
subclavian artery corniculate, and palatini (not tensor
(right) cuneiform cartilages veli palatini)
5 Vagus (X) recurrent Part of left Thyroid, cricoid, Larynx intrinsic
laryngeal branch pulmonary artery arytenoid, (NEET muscles (not
(left), part of right 2018) corniculate cricothyroid
pulmonary artery and cuneiform muscle)
(right) cartilages

https://t.me/DentalBooksWorld
Chapter 6 • Embryology, Head and Neck Anatomy, and General Histology 467

Pharyngeal Pouch

Pharyngeal pouch 1  Develops into the auditory tube and middle ear cavity
Pharyngeal pouch 2 Becomes the crypt of the palatine tonsil; later to establish the definitive palatine tonsil
Pharyngeal pouch 3 Divides into a superior (or dorsal) and inferior (or ventral) portion:
• Superior (dorsal) portion of pouch 3: Forms the inferior parathyroid glands—the
chief (or principal) and oxyphil cells are derived from the endodermal lining of the
pouch (AIPG 2002)
• Inferior (ventral) portion of pouch 3: Forms the thymus
Pharyngeal pouch 4 Also divides into a superior (or dorsal) and inferior (or ventral) portion:
• Superior (dorsal) portion of pouch 4: Forms the superior parathyroid glands—the
chief (or principal) and oxyphil cells are derived from the endodermal lining of the pouch
• Inferior (ventral) portion of pouch 4: Forms a diverticulum called the ultimobranchial
body and the cells differentiate into parafollicular (C) cells of the thyroid gland
Anomalous development of the Symptoms and signs of DiGeorge often include:
derivatives of pouches 3 and/ • Hypoplasia of the hyoid
or 4 can result in ectopic or • Thymic hypoplasia (immunodeficiency due to a lack of T-cells)
absent parathyroid, thymic, or
• Hypoparathyroidism (missing or hypoplastic inferior parathyroid glands)
parafollicular thyroid tissue.
The most common disorder in • Outflow tract defects (neural crest in this area also contributes to conotruncal cushions of
which this occurs is DiGeorge the outflow tract)
syndrome, caused by a deletion Interestingly, the hypoplasia of the second and third arches can also disrupt the first and
in the long (or “q”) arm of second arch, leading to the following additional findings:
chromosome 22, leading to a • Micrognathia (reduced jaw)
hypoplasia of second and third • Cleft palate
pharyngeal pouch derivative • Hearing loss

DERIVATIVES OF THE THREE GERM LAYERS

Ectodermal derivatives Mesodermal derivatives Endodermal derivatives


• Epidermis of the skin along with glands • Musculoskeletal system • Epithelial lining of respiratory passages
(subcutaneous and mammary glands), hair, • Connective tissue of and digestive tract
and nails • gastrointestinal tract • Endocrine glands and organs (liver
• Epithelium of mouth and nasal cavity • Dermis of the skin and pancreas)
• Sensory epithelium of ear, nose, and eye • Cardiovascular system
• Neuroectoderm cells • Peritoneum
• Lens, cornea, and retina of the eye • Reproductive system
• Nervous system • Excretory system
• Posterior pituitary
• Neural crest cells

Derivatives of Neural Crest Cells

Derivative Cell type or structure derived


Peripheral nervous system • Neurons, including sensory ganglia, sympathetic and parasympathetic ganglia, and
(PNS) plexuses
• Neuroglial cells
• Schwann cells
Endocrine and paraendocrine • Adrenal medulla
derivatives • Calcitonin-secreting cells
• Carotid body type I cells

https://t.me/DentalBooksWorld
468 Triumph’s Complete Review of Dentistry

Pigment cells • Melanocytes


Facial cartilage and bone • Facial and anterior ventral skull cartilage and bones
Connective tissue • Corneal endothelium and stroma
• Dental papillae
• Dermis, smooth muscle, and adipose tissue of skin of head and neck
• Connective tissue of salivary, lachrymal, thymus, thyroid, and pituitary glands
• Connective tissue and smooth muscle in arteries of aortic arch origin
• Odontoblasts (AIPG 2010)

nn THE HEAD
OSTEOLOGY

Development of Skull
• The base of the skull develops by endochondral ossification.
• The brain and cranial nerves develop before the development of skull, so when the chondrocranium develops, its
components form around the nerves and form the foramina.
• The bones of the calvarium ossify by intramembranous ossification.
• The bones of the face are partly basal and partly calvarial bones, so they ossify both by intramembranous and endochondral
ossification.
Bones of the Skull
The human skull is generally considered to consist of 22 bones—8 cranial and 14 facial skeletons (note: 28 bones if three
paired ear ossicles are included).

Skull Bones
Bones of Cranium Bones of Face
Paired Unpaired Paired Unpaired
Parietal Frontal Zygomatic Mandible
Temporal Occipital Maxilla Vomer
Sphenoid Nasal
Ethmoid Lacrimal
Palatine
Inferior conchae

Condition Description
Dolichocephaly/scaphocephaly Premature closure of the sagittal suture (anteroposterior widening)
Brachycephaly Premature closure of the bilateral coronal suture (transverse suture)
Trigonocephaly Premature closure of metopic suture (posterosuperior widening)
Metopic suture is present between two frontal bones. It usually closes
by 9 months and disappears at 7 years of age. Metopic suture persists
in approximately 7% of the people
Oxycephaly/acrocephaly/turricephaly Premature closure of the coronal suture plus lambdoid suture
Anterior plagiocephaly (flat head syndrome) Unilateral coronal synostosis
Posterior plagiocephaly Lambdoid synostosis

https://t.me/DentalBooksWorld
Chapter 6 • Embryology, Head and Neck Anatomy, and General Histology 469

Fontanelles (AIPG December 2015)

The posterior fontanelle Between two parietal bones and Generally closes 2–3 months after birth
occipital bone
The sphenoidal fontanelle Between the frontal, parietal, temporal, Is the next to close around 6 months
and sphenoid after birth
The mastoid fontanelle Between parietal, occipital, and the Closes next from 6 to 18 months after
temporal bone birth
The anterior fontanelle Between the two parietal bones and the Is generally the last to close between 18
frontal bone and 24 months
Related Points
• At birth, the anterior fontanelle is a diamond-shaped area between the two frontal bones and the two parietal bones.
• It pulsates and bulges when the baby cries.
• It closes by 18 months to 2 years and is then known as the bregma.
• The parietal bones and the occipital bone meet at the posterior which becomes lambda, along the lambdoid suture.
• The side wall of the skull is completed by the squamous part of the temporal bone and the greater wing of the sphenoid
bone, at the pterion.
• This is located 4 cm above the midpoint of the zygomatic arch and is the site of surgical exploration for the middle
meningeal artery.
• The flat bones of the skull (frontal, parietal, temporal, and occipital) are formed by a sandwich of diploë (cancellous bone
containing red bone marrow) between two layers of compact bone.
• The bones are drained by diploic veins; there are usually four on each side: Frontal, anterior temporal, posterior temporal,
and occipital which open into the nearest convenient venous sinus.
• The flat bones of the skull are also pierced by emissary foramina transmitting emissary veins connecting veins of the scalp
with the dural venous sinuses inside the skull (this is a possible route for the spread of infection). They may be seen in the
parietal bone or in the temporal bone posterior to the external auditory meatus.

The Cranial Fossae


From anterior to posterior, three fossae to form three successive steps leading down to the foramen magnum.
1. Anterior cranial fossa
It contains:
• Orbital plate of the frontal bone
• Lesser wing of the sphenoid bone
• The cribriform plate of the ethmoid with the crista galli transmits olfactory nerves from the upper part of the nasal cavity;
this is a possible route for infection or escape route for CSF in skull fractures
• Optic canal for transmission of the optic nerve
• Anterior clinoid processes projecting posteriorly to the posterior clinoid processes of the sphenoid bone

2. Middle cranial fossa


• It is formed by the greater wing of the sphenoid bone and the temporal bone. It is occupied by the temporal lobe of the
brain.
• The most posterior tip of the greater wing contains the foramen spinosum.
• Anterior to the foramen spinosum is the foramen ovale and then the foramen rotundum. The foramen rotundum leads to
the pterygopalatine fossa.
• The superior orbital fissure is a gap between the lesser wing and the greater wing of the sphenoid leading to the orbit, just
lateral to and below the optic nerve.
–– The body of the sphenoid bone contains the hypophyseal fossa for the pituitary gland. This fossa is also called the sella
turcica and it is shaped like a four-poster bed.

https://t.me/DentalBooksWorld
470 Triumph’s Complete Review of Dentistry

––Its maximum length is 14 mm and its depth is 8 mm. Measurements are important because pituitary tumors cause
ballooning of the sella.
• The foramen lacerum is located lateroposterior to the sella.
3. Posterior cranial fossa
• It is formed by the temporal bone and the occipital bones
• It houses the cerebellum

The posterior cranial fossa contains:


• The foramen magnum for the lower end of the medulla
• The clivus, anterior to the foramen magnum, which leads up to the body of the sphenoid bone
• The transverse sinus
• The sigmoid sinus which ends at the jugular foramen
• The hypoglossal canal (anterior condylar) canal and the posterior condylar canal
• The internal auditory meatus on the posterior aspect of the petrous temporal bone
• The arcuate eminence on the petrous portion of the temporal bone, marking the position of the superior semicircular
canal
• The spine of the sphenoid bone is near the foramen spinosum
• The jugular foramen is occupied by the jugular bulb (for expansion of the internal jugular vein) in life
• The styloid process, the mastoid process, and the stylomastoid foramen between the two processes
• Anterior to the jugular foramen and in the petrous portion of the temporal bone lies the carotid canal

Related Key Points


• The middle meningeal artery enters the skull through the foramen spinosum and its groove can be traced laterally and
anteriorly on the squamous portion of the temporal bone before dividing into anterior and posterior (frontal and parietal)
branches.
• The anterior branch crosses the greater wing at the pterion and then breaks up into branches that run superiorly and
posteriorly.
• The posterior branch runs posteriorly supplying bone and meninges.

STRUCTURES PASSING THROUGH FORAMINA

Foramen ovale (male) Mandibular nerve


Accessory meningeal artery
Anterior trunk of the middle meningeal vein
Lessor petrosal nerve
Emissary vein
Foramen spinosum (men) Middle meningeal artery and posterior trunk of the middle meningeal vein
Emissary vein
Nervous spinosus (meningeal branch of the mandibular nerve)
Foramen lacerum Meningeal branch of the ascending pharyngeal artery
Emissary vein
Internal carotid artery with sympathetic plexus
Greater petrosal nerve unites with the deep petrosal nerve to form the nerve of the
pterygoid canal

https://t.me/DentalBooksWorld
Chapter 6 • Embryology, Head and Neck Anatomy, and General Histology 471

Carotid canal Internal carotid artery with sympathetic plexus


Emissary vein

The internal carotid artery enters the carotid canal, runs anteromedially to the foramen
lacerum, and then runs superoanteriorly to enter the cranium through the internal
orifice of the foramen lacerum
It then turns anteriorly and lies on the side of the sella turcica. It now lies in the
cavernous sinus.
It then turns superoposteriorly, medial to the anterior clinoid process, and breaks up
into three branches
Foramen rotundum Maxillary nerve
Jugular foramen Anterior part: Inferior petrosal sinus
Middle part: 9th, 10th, and 11th cranial nerves
Posterior part: Sigmoid sinus continues as internal jugular vein and meningeal branch
of the occipital artery
Stylomastoid foramen Facial nerve, stylomastoid branch of the posterior auricular artery
Foramen magnum Medulla, tonsils of the cerebellum, meninges
Through subarachnoid space—spinal accessory nerve, vertebral artery, posterior
spinal artery, anterior spinal artery
Anterior part: Ligament of dens, membrana tectoria
Optic canal Optic nerve and ophthalmic artery
Superior orbital fissure Middle part (within the ring): Upper and lower divisions of the oculomotor nerve
(third nerve), nasociliary nerve (ophthalmic division of fifth nerve), and abducent
nerve (sixth nerve)
Lateral part (above the ring): Trochlear nerve (fourth nerve), frontal nerve, and
lacrimal nerve (ophthalmic division of fifth nerve), superior ophthalmic vein, recurrent
meningeal branch of the lacrimal artery, orbital branch of the middle meningeal artery,
and sometimes meningeal branch of the ophthalmic artery
Medial part (below the ring): Inferior ophthalmic vein, sympathetic nerves around the
internal carotid artery
Inferior orbital fissure Zygomatic branch of the maxillary nerve, infraorbital nerve and vessels, rami of the
pterygoid ganglion, communicating vein between the inferior ophthalmic vein, and
pterygoid plexus of veins
Hypoglossal canal (anterior condylar Hypoglossal nerve, meningeal branch of the ascending pharyngeal artery, and emissary
canal) vein connecting the sigmoid sinus to the internal jugular vein
Posterior condylar canal Emissary vein connecting the sigmoid vein with the suboccipital venous plexus
Internal acoustic meatus Vestibulocochlear and facial nerves (nervus intermedius of Wrisberg), labyrinthine
vessels. The internal acoustic meatus is divided by the transverse crest (or falciform
crest) into a superior and an inferior part. The superior part is divided by the vertical
crest or Bill’s bar into anterior and posterior parts
Pterygoid canal Vidian nerve (nerve of the pterygoid canal), vidian artery (artery of the pterygoid
canal)
Mastoid canaliculus Auricular branch of the vagus nerve (entry) (Arnold’s or Alderman’s nerve)
Tympanomastoid fissure Auricular branch of the vagus nerve (exit)
Tympanic canaliculus Tympanic branch of the glossopharyngeal nerve (Jacobson nerve)
Note:
Anterior ethmoidal and posterior ethmoidal nerve branches of nasociliary nerve pass through the anterior and posterior
ethmoidal canal, respectively, whereas the nasopalatine nerve passes through the incisive foramen.

https://t.me/DentalBooksWorld
472 Triumph’s Complete Review of Dentistry

SUPERFICIAL FACE AND SCALP


The scalp is attached to the zygomatic arch laterally
• S = Skin
• C = Connective tissue (blood vessels are present on this layer)
• A = Aponeurosis referring to the Galea aponeurotica (epicranial aponeurosis). This aponeurosis belongs to
the occipitofrontalis muscle located antero- and posteriorly. The posterior attachment of this muscle is to the
occipital bone above the superior nuchal line.
• L = Space, this is a plane of cleavage for injury and the spread of blood which can travel as far as the zygomatic
arch and into the upper eyelids. The first three layers of scalp remain attached to each other in case of cleavage.
• P = Periosteum

Blood vessels and nerves of the scalp:

Anastomoses in the scalp are formed by the following arteries from anterior to posterior:
• Supratrochlear artery
• Supraorbital artery
• Superficial temporal artery
• Posterior auricular artery
• Occipital artery

• The scalp is extremely vascular and bleeds profusely when cut. Due to anastomoses, there is no single vessel to
compress but bleeding may stop by direct pressure on or around the wound.
• Emissary veins (valveless) may spread infections from the scalp to the intracranial cavity. Normal blood flow is
from inside to outside of the skull.
• The following nerves innervate the scalp from anterior to posterior.
• Two branches from the ophthalmic division of the trigeminal nerve (cranial nerve V): The supratrochlear and
supraorbital nerves.
• One branch from the maxillary division of the trigeminal nerve: The zygomaticotemporal nerve.
• One branch from the mandibular division of the trigeminal nerve: The auriculotemporal nerve.

FACIAL MUSCULATURE
Muscles of Face

Frontalis

Procerus

Temporalis Orbicularis oculi

Levator labii superioris Levator labii superioris


alaeque nasi
Zygomacus minor Compressor naris
Zygomacus major
Buccinator
Orbicularis oris
Masseter
Depressor anguli oris
Depressor labii inferioris
Mentalis
Platysma

Frontalis
https://t.me/DentalBooksWorld
Temporalis

Orbicularis oculi
Levator labii superioris
Zygomacus major
Buccinator
Orbicularis oris
Masseter
Depressor anguli oris
Depressor labii inferioris
Mentalis
Platysma
Chapter 6 • Embryology, Head and Neck Anatomy, and General Histology 473

Frontalis
Temporalis

Orbicularis oculi
Levator labii superioris
alaeque nasi
Compressor naris
Depressor sep
Zygomacus major
Zygomacus minor Masseter
Orbicularis oris
Mentalis

Muscle Origin Insertion Action Cranial nerve


Orbicularis oris Extrinsic fibers—from Extrinsic fibers pass Compresses lips against Seventh nerve—
insertions of circum oral around mouth within anterior teeth, closes zygomatic, buccal, and
muscles the lips as a sphincter mouth, protrudes lips mandibular branches
Intrinsic fibers—from Intrinsic fibers pass (AIIMS November
incisive fossae of the obliquely forward and 2017)
mandible and maxilla insert into the skin of
the lip
Depressor Oblique line of the Angle of the mouth Depresses angle of the Seventh nerve—buccal
anguli oris mandible mouth and mandibular
branches
Levator anguli Canine fossa of the Angle of the mouth Elevates angle of the Seventh nerve—
oris maxilla, below the mouth zygomatic and buccal
infraorbital foramen branches
Zygomaticus Zygomatic bone Angle of the mouth Draws angle of the Seventh nerve—
major mouth up and back zygomatic and buccal
branches
Risorius Fascia of the parotid and Angle of the mouth Draws angle of the Seventh nerve—
masseteric regions mouth up and laterally zygomatic and buccal
branches
Levator labii Angular head—frontal Alar cartilage and skin Elevates upper lip, flares Seventh nerve—
superioris process of the maxilla of the nose nostrils zygomatic and buccal
Infraorbital head— branches
inferior margin of orbit Upper lip
Zygomatic head
(zygomaticus minor)— Nasolabial groove and
zygomatic bone upper lip
Depressor labii Oblique line of the Lower lip Depresses lower lip Seventh nerve—
inferioris mandible mandibular branches

https://t.me/DentalBooksWorld
474 Triumph’s Complete Review of Dentistry

Buccinator Pterygomandibular Upper fibers cross to Compresses cheeks Seventh nerve—buccal


raphe, buccal alveolar insert into the lower lip against molar teeth; branches
process of the maxilla and lower fibers cross to sucking and blowing
and mandible insert into the upper lip
Mentalis Incisive fossa mandible Skin of the chin Puckers skin of the chin Seventh nerve—
and protrudes the lower mandibular branches
lip
Nasalis Compressor nares— Midline aponeurosis Compresses nostrils Seventh nerve—
canine eminence of the overlying lateral nasal zygomatic and buccal
maxilla cartilages Dilates or flares nostrils branches
Dilator nares—nasal Skin of margin of the
notch of the maxilla nostrils
Orbicularis Orbital: Bone of the Fibers encircle margins Closes eyes forcefully Seventh nerve—
oculi upper medial orbital of orbit and insert into temporal and zygomatic
margin the medial palpebral branches
ligament
Closes eye gently
Palpebral: Medial Fibers arch laterally
palpebral ligament through lids and
interdigitate laterally in Squeezes lubricating
a raphe tears against the eyeball
Lacrimal: Lacrimal bone
behind lacrimal sac Medial aspect of lids

Muscles of Mastication

Muscles Origin and insertion Action Innervation


Temporalis Origin: Temporal Fossa insertion: Bilateral: Elevation of the Trigeminal nerve
Coronoid process and ramus of the mandible (closing the mouth), (Anterior branch—
mandible retrusion of the mandible Cranial n. V)
Unilateral: Ipsilateral lateral
The temporalis muscle attaches from deviation
the lateral side of the skull below the
temporal line to coronoid process and
anterior border of the ramus of the
mandible almost as far as the third
molar tooth
Masseter The masseter muscle attaches to the Bilateral: Elevation of the Trigeminal nerve
zygomatic arch and the outer surface mandible (closing the mouth) (Anterior branch—
of the mandible near the angle. It is Unilateral: Ipsilateral lateral Cranial n. V)
composed of a superficial and deep part deviation
Medial pterygoid The medial pterygoid is attached Bilateral: Elevation of the Trigeminal nerve
from the medial surface of the lateral mandible (closing the mouth) (Direct branch
pterygoid plate to the deep surface and protrusion of the mandible nervous meningous—
of the mandible (at the angle of the Unilateral: Contralateral lateral Cranial n. V)
mandible) opposite to the attachment deviation
of the masseter muscle. The angle of the
mandible thus lies between these two
muscles

https://t.me/DentalBooksWorld
Chapter 6 • Embryology, Head and Neck Anatomy, and General Histology 475

Lateral pterygoid The lateral pterygoid muscle attaches Bilateral: Depression of the Trigeminal nerve
from the lateral surface of the lateral mandible (opening the mouth) (Anterior branch—
pterygoid plate to the neck of mandible and protrusion of the mandible Cranial n. V)
and the intraarticular disc of the Unilateral: Contralateral lateral
temporomandibular joint deviation
Blood Supply
The facial artery provides the main blood supply
• It passes over the lower border of mandible at the anterior border of the masseter (feel the pulse).
• It has a tortuous course to allow for movement of the face, first to the angle of the mouth and then up at the side of the
nose to the medial angle of the eye.

Parotid Gland
This major salivary gland becomes inflamed with the mumps. It is enclosed in a split layer of deep cervical fascia so
that the swelling of the gland leads to an increase in pressure and pain. The medial thickening in this capsule forms the
stylomandibular ligament.
The duct leaves the anterior border, crosses the masseter muscle, turns around the anterior border of the muscle, and pierces
the buccinator muscle to enter the mouth opposite the second upper molar tooth. It can be rolled under the fingers when the
masseter is contracted by clenching the teeth. It is in line with the tragus of the ear.
The gland occupies the space between the sternocleidomastoid and the back of the mandible and molds itself to all adjacent
structures.
Three important structures pass through the parotid gland from superficial to deep:
• The facial nerve
• The retromandibular vein
• The external carotid artery

The retromandibular vein


• Is formed in the gland by the union of the maxillary and superficial temporal veins.
• Emerges from the gland near the angle of the mandible.
• Divides into two with the anterior branch joining the facial vein and draining into the internal jugular vein.
• The posterior branch joins with the small posterior auricular vein to form the external jugular vein.
• The external carotid artery divides in the substance of the parotid gland into the maxillary and superficial temporal
arteries.
• The maxillary artery runs deep to the neck of mandible to enter the infratemporal region.
• The superficial temporal artery lies in front of the ear and divides into anterior (frontal—side of the forehead) and posterior
(parietal—side of the scalp) branches.
The PAROTID BED is defined
• Posteriorly by the mastoid process with the origins of the sternocleidomastoideus and the posterior belly of the digastric
• Medially by the styloid process of the temporal bone and the stylohyoid muscle. Styloglossus and stylopharyngeus are also
medial to the parotid bed within the lateral pharyngeal space
• Anteriorly by the sphenomandibular and stylomandibular ligaments as well as the fasciae of the medial pterygoid and
masseter muscles
• Superiorly by the zygomatic arch
• Inferiorly by the posterior belly of the digastric

https://t.me/DentalBooksWorld
476 Triumph’s Complete Review of Dentistry

Important Points About Nerve Supply of the Parotid Gland

Secretomotor—Parasympathetic Auriculotemporal nerve


Vasomotor—Sympathetic Plexus around the middle meningeal artery
Sensory nerves Auriculotemporal nerve
Parotid fascia Greater auricular nerve—C2, C3

GANGLION

Nerve Sensory ganglion Parasympathetic ganglion


Oculomotor – Ciliary
Trigeminal Semilunar (Gasserian) –
Facial Geniculate Pterygopalatine, submandibular
Glossopharyngeal Superior and inferior ganglion Otic
Vagus – In the wall of the viscera

INFRATEMPORAL REGION

The infratemporal region inferior to the temporal fossa and zygomatic arch and deep to the ramus of the mandible. It stretches
from the parotid fascia posterior to the mandibular ramus to the tuberosity of the maxilla.
The lateral wall is formed by the medial aspect of ramus of the mandible.
The anterior wall is formed by
• Body and tuberosity of the maxilla, deep to zygoma and zygomatic process of the maxilla.
• The pterygomaxillary fissure or sphenopalatine foramen may be seen in the medial aspect of this anterior wall, opening
into the more medial pterygopalatine fossa.
• The inferior orbital fissure may also be seen.
• Inferior to the pterygomaxillary fissure is the hamulus serving as attachment point for the pterygomandibular raphe. It
serves as the common site of origin for the buccinator and the superior constrictor muscle and runs from the hamulus to
the upper one-fifth of the mylohyoid line.
The medial wall is formed by the
• Lateral pterygoid plate
• Superior constrictor muscle
• Levator and tensor palati muscles

The roof of the infratemporal fossa is formed by:


• The greater wing of the sphenoid anteriorly
• And the squamous portion of the temporal bone posteriorly

The infratemporal crest is on the anterior aspect of the undersurface of the greater wing of the sphenoid and serves as an
attachment site for the upper head of the lateral pterygoid. Posterior to this infratemporal crest are:
• The foramen ovale for transmission of V3 and the lesser petrosal nerve (from ninth) from the middle cranial fossa to the
infratemporal fossa.
• The foramen spinosum for transmission of the middle meningeal artery from the infratemporal fossa to the middle cranial
fossa.

https://t.me/DentalBooksWorld
Chapter 6 • Embryology, Head and Neck Anatomy, and General Histology 477

Contents of the Infratemporal Fossa


The key structure for orientation is the lateral pterygoid muscle.
This muscle has two heads:
• From the infratemporal crest to the capsule of the interarticulating disc of the TMJ.
• From the lateral aspect of the lateral pterygoid plate to the neck of the mandible.

Protrusive actions of the lateral pterygoid muscle are used to test V3: Deviation is toward the side of the lesion.

MAXILLARY ARTERY

The maxillary artery lies lateral to the lateral pterygoid muscle.


From the external carotid artery in the parotid gland, the artery enters the posterior aspect of the infratemporal fossa by
passing deep to the neck of the mandibular condyle.
• It crosses the lateral side of the lateral pterygoid muscle and enters the pterygomaxillary fissure.
• It is divided into a first or mandibular part, second or pterygoid part, and third or pterygopalatine part.
• The mandibular and pterygoid parts are associated with the infratemporal fossa and the pterygopalatine part is associated
with the deep face and the nasal region.
The mandibular division of the maxillary artery has five branches, all entering a canal:
• The middle meningeal artery is the principal artery to periosteal dura of the cranial cavity.
• The inferior alveolar artery runs into the mandibular foramen and supplies the teeth and the mandible. The angle of the
mandible is poorly supplied and may suffer from the alveolar osteitis (dry socket).
• The deep auricular artery supplies the auditory meatus
• The anterior tympanic artery accompanies the chorda tympani through the petrotympanic fissure to reach the middle ear.
• The accessory meningeal branch (inconsistent) enters the foramen ovale and supplies the trigeminal ganglion and the
surrounding dura.
The pterygoid portion of the maxillary artery has five branches supplying muscles of mastication in the infratemporal fossa:
• Two deep temporal branches
• A masseteric branch
• A pterygoid branch
• A buccal branch

The pterygoid plexus of veins follows the maxillary artery in the infratemporal fossa, lying mostly lateral to the artery.
• This is a route for infection: The veins have connections with the cavernous sinus via the deep facial, inferior ophthalmic,
and emissary veins in the sphenoid bone.
• Veins of the head have no valves.

Branches of V3 (MANDIBULAR DIVISION OF THE TRIGEMINAL NERVE)


I. Anterior Division
• Masseteric branches
• Posterior and anterior temporal branches to the temporalis muscle
• The nerve to the medial pterygoid
• The nerve to the lateral pterygoid
• The buccal nerve

https://t.me/DentalBooksWorld
478 Triumph’s Complete Review of Dentistry

II. Posterior Division


Auriculotemporal Nerve
• Leaves V3 just inferior to the foramen ovale and projects posteriorly in the infratemporal fossa parallel to the roof.
• The initial segment encircles the middle meningeal artery as the artery ascends to enter the foramen spinosum and
receives postganglionic parasympathetic fibers from the otic ganglion which are secretomotor to the parotid gland.
• Passes medial to the head of the mandibular condyle and sends a sensory branch to the TMJ.
• Enters the deep portion of the parotid gland giving sensory branches as well as parasympathetic postganglionic fibers
from the otic ganglion.
• Its terminal portion accompanies the superficial temporal artery and innervates the upper half of pinna of the ear and part
of the temporal region (pain and general sensation).
Inferior Alveolar (Dental) Nerve
• From the foramen ovale to the mandibular foramen on the medial aspect of the ramus of the mandible, lying between the
medial and lateral pterygoid muscles and just posterior to the lingual nerve.
• The branch to mylohyoid and to the anterior belly of the digastric is the only branch in the infratemporal fossa. It first
lies in the mylohyoid groove and then on the inferior aspect of the mylohyoid to reach the anterior belly of the digastric
muscle.
• The portion of the inferior alveolar nerve in the ramus of the mandible is entirely sensory to lower teeth, lower gums, and
the mucosa of the lower lips.
• It exits the mandible as the mental nerve to innervate the mucosa and gum adjacent to the lower lip.

Lingual Nerve
• Lies anterior to the inferior alveolar nerve and remains medial to the mandible.
• Receives the chorda tympani in the infratemporal fossa. The chorda tympani reaches the infratemporal fossa via the
petrotympanic fissure.
• The chorda tympani contains preganglionic parasympathetic secretomotor fibers of VII from the tympanic plexus and
special sensory fibers for taste from the anterior two-third of the tongue. The taste fibers have their cell bodies in the
geniculate ganglion of VII.
• Terminal distribution of the lingual nerve and associated fibers which mediate general sensation (pain, touch, temperature,
and pressure) is to the floor of the mouth and the anterior two-third of the tongue.
The Buccal Nerve of V3
• Passes between the two heads of the lateral pterygoid muscle (PGI December 2008)
• Continues into the cheek on the lateral surface of the buccinator muscle
• Is the terminal branch of the anterior division
• Is sensory to the mucosa of the inside of the cheek and the lower gums around the molar teeth
• Does not supply the motor innervation of the buccinator

TEMPOROMANDIBULAR JOINT
• Head of the mandible
• Mandibular fossa and articular tubercle of the temporal bone
• Synovial joint with intraarticular disc dividing joint into a lower compartment (hinge rotation for mandibular head) and
upper compartment (sliding joint for protrusion)
Minor supportive elements of the TMJ:
• Lateral temporomandibular ligament (thickening of the joint capsule)
• Stylomandibular ligament (between parotid and submandibular glands)
• Sphenomandibular ligament

https://t.me/DentalBooksWorld
Chapter 6 • Embryology, Head and Neck Anatomy, and General Histology 479

MENINGES
• Dura mater (pachymeninx)
• Arachnoid
• The subarachnoid space continues through the foramen magnum, around the spinal cord
• Pia mater (closely adherent to the brain)
Arachnoid and pia mater are also called leptomeninges
DURA MATER
Two layers:
• An outer fibrous layer
• An inner serous layer, which parts from the fibrous layer to form the venous sinuses

The dura is supplied by small arteries and the middle meningeal artery. The vein runs with the artery.
Intracranial partitions of the dura mater:
• The falx cerebri with the superior sagittal sinus starts at the crista galli.
• The tentorium cerebelli incompletely roofs over the posterior cranial fossa.
• At this level, the midbrain runs superiorly through the opening to join with the diencephalon. The sharp edges of dura
may have fatal consequences when the brain is displaced by force or a space-occupying lesion.
• The flow of the cerebrospinal fluid in the venous sinuses is from the superior sagittal sinus to the right transverse sinus to
the sigmoid sinus to the internal jugular vein.
• The arachnoid villi drain CSF from the subarachnoid space to the venous sinuses. With age, the arachnoid villi become
clumped together to form the arachnoid granulations.
• The inferior sagittal sinus is in the free edge of the falx cerebri and receives part of the drainage of the great cerebral vein
(of Galen) and becomes the straight sinus. This passes to the left and forms the left transverse sinus, left sigmoid sinus, and
left internal jugular vein.
• The flow from the larger superior sagittal sinus tends to go to the right transverse sinus, the right jugular foramen is
usually bigger than the left. But, at other times, the ends of the superior sagittal sinus and the straight sinus join together to
form the confluence of the sinuses and the jugular foramina will be even in size.

BLOOD–BRAIN BARRIER IS ABSENT IN CIRCUMVENTRICULAR ORGANS


• Pituitary—Neurohypophysis
• Area postrema
• Organum vasculosum of the lamina terminalis
• Subfornical organ
• Ventral part of the median eminence of the hypothalamus
• Anterior pituitary
• Pineal gland

BLOOD SUPPLY OF THE BRAIN


After passing through the cavernous sinus, the internal carotid artery turns superiorly:
• It then gives out the ophthalmic artery (which runs into the optic canal with the optic nerve).
• And terminates as the anterior and middle cerebral arteries and the posterior communicating artery.
• Before it breaks up into terminal branches, it gives out small branches to the pituitary gland and adjacent structures.

https://t.me/DentalBooksWorld
480 Triumph’s Complete Review of Dentistry

The vertebral artery goes through the foramen magnum and gives off
• The anterior spinal artery
• The posterior spinal artery
• The posterior inferior cerebellar artery
• And finally joins with the opposite artery to form the basilar artery

The basilar artery lies ventral to the pons of the brainstem, on the clivus. The basilar artery sends out.
• Branches to the brainstem
• The anterior inferior cerebellar artery
• And terminates as the superior cerebellar and posterior cerebral arteries

Terminal branches of the internal carotid and the vertebral arteries form the circle of Willis

Schematic Diagram of Circle of Willis

Central branches Anterior cerebral


(anteromedial group) artery
Anterior communicang
artery
Internal carod
artery

Central branches
(anterolateral group)
Posterior
communicang artery
Central branches
(anteromedial group)
Posterior cerebral
Basilar artery artery
Superior cerebellar
Ponne branches artery

Cranial Nerves Within the Cranium

Cranial Nerve Type Function


Olfactory Sensory Perceives smell
Optic Sensory Vision
Oculomotor Motor Supplies levator palpebrae superioris, superior rectus, medial rectus, inferior rectus,
inferior oblique, supplies sphincter pupillae, and ciliary muscle with parasympathetic
fibers
Trochlear Motor Supplies superior oblique
The cranial nerves all leave the brain on its ventral surface except for the fourth cranial
(trochlear) nerve

https://t.me/DentalBooksWorld
Chapter 6 • Embryology, Head and Neck Anatomy, and General Histology 481

Trigeminal Mixed Supplies sensory fibers to the skin of the scalp, face, mouth, teeth, nasal cavity, and
paranasal sinuses
Supplies motor fibers to the muscles of mastication, tensor veli palatini, and tensor
tympani
Abducent Motor Supplies lateral rectus
Facial Mixed Supplies motor fibers to the muscles of face, scalp and auricle, buccinator, platysma,
stapedius, stylohyoid, and posterior belly of digastric.
Gives sensory supply to the anterior two-third of the tongue, floor of mouth, and the
palate
Gives parasympathetic secretomotor fibers to the submandibular and sublingual
salivary glands, lacrimal glands, and the glands of the nose and palate
Vestibulocochlear Sensory Position and movement of the head
Hearing
Glossopharyngeal nerve Mixed Supplies stylopharyngeus muscle
Supplies parasympathetic secretomotor fibers to the parotid gland, sensory and taste
fibers to the posterior third of the tongue and pharynx
Vagus Mixed Supplies constrictor muscles of the pharynx and intrinsic muscles of the larynx,
involuntary muscles of the trachea and bronchi, heart, alimentary tract from the
pharynx to the splenic flexure of the colon, liver, and pancreas
Afferent fibers form the above structures, taste from epiglottis, and valleculae
Spinal accessory Motor Supplies sternocleidomastoid and trapezius
Hypoglossal Motor Supplies muscles of the tongue, except palatoglossus

Eye and Orbit

Walls Formation
Roof (superior wall) • Orbital plate of frontal (mainly)
• Lesser wing of the sphenoid
Lateral wall—Thickest wall • Anterior surface of the greater wing of the sphenoid
• Orbital surface of frontal process of the zygomatic bone
Floor (inferior wall) • Orbital surface of the maxilla (mainly)
• Orbital surface of the zygomatic bone
• Orbital process of the palatine bone
Medial wall—Weakest wall • Frontal process of the maxilla
• Lacrimal bone
• Orbital plate of the ethmoid (including lamina papyracea)
• Body of the sphenoid
Note:
• Inferior orbital fissure occupies between lateral wall and the floor.
• Superior orbital fissure occupies between the junction of the roof and the lateral wall.
• Optic canal lies at the junction of the roof and medial wall, and is enclosed by lesser wing and body of the sphenoid.
• The medial walls of the orbit are parallel, whereas the lateral walls diverge: The long axis of the orbit is at an angle to
the long axis of the eyeball.

https://t.me/DentalBooksWorld
482 Triumph’s Complete Review of Dentistry

The levator palpebrae superioris muscle:


• Superior to the cone, outside of it, lies the levator palpebrae superioris muscle attaching anteriorly to the upper
eyelid.
• From the back of the orbit, above the fibrous ring, it lies under the roof of the orbit.
• It is supplied both by the III cranial nerve and sympathetic nerves (loss of either innervation will lead to ptosis).

Muscles of the Eyeball

Muscle Nerve supply Function


Medial rectus Oculomotor nerve Moves eyeball medially
Lateral rectus Abducent Moves eyeball laterally
Superior oblique Trochlear Moves eyeball downward and laterally
Inferior oblique Oculomotor nerve Moves eyeball upward and laterally
Orbicularis oculi Facial nerve Closes eyelids and dilates lacrimal sac
Levator palpebrae superioris Oculomotor nerve and sympathetic supply Raises upper lid
Sphincter pupillae Parasympathetic supply by the oculomotor nerve Constricts pupil
Dilator pupillae Sympathetic supply Dilates pupil
Ciliary muscle Parasympathetic supply by the oculomotor nerve Controls shape of eye and helps in
accommodation
Superior rectus Oculomotor nerve Moves eyeball upward and medially
Inferior rectus Oculomotor nerve Moves eyeball downward and
medially

Inferior oblique Superior rectus

Lateral rectus Medial rectus

Superior oblique Inferior rectus

https://t.me/DentalBooksWorld
Chapter 6 • Embryology, Head and Neck Anatomy, and General Histology 483

Nose, Nasal Cavities, and Paranasal Sinuses


Cutaneous Innervation
• The nerve supply to the skin of the nose is by V1 (infratrochlear nerve) and V2.
• The innervation of the tip of the nose is by the external nasal nerve, an anterior ethmoidal branch of the nasociliary nerve
(V1) running from the root of the nose to the tip of the nose.
Loss of sensation to the tip of the nose may be due to an intracranial, intraorbital, or ethmoidal air sinus disorder affecting V1
pathway from the trigeminal ganglion.
Bony Landmarks
• The paired nasal bones articulate with the frontal bone and frontal processes of the maxillary bones
• The central septal cartilage connects with the superior lateral cartilages articulating with the nasal bones
• Alar cartilages are supported by the septal cartilage

Nasal Cavities
• The nares (nostrils) open into the right and left nasal cavities separated by the septum.

The choanae are the posterior apertures leading to the nasopharynx.


Floor of the nasal cavity • Palatine processes of the maxilla and horizontal plates of the palatine bone
Roof of nasal cavity • Anterior part: Slope of the nasal bones
• Intermediate part: Cribriform plate of the ethmoid bone
• Posterior part: Anterior and inferior aspects of the body of the sphenoid bone
Nasal septum • The perpendicular plate of the ethmoid bone and the vomer bone may articulate
with each other posteriorly
• The septal cartilage intervenes between bony elements of the septum, forming
support for the midline ridge of the nose, tip, and columella (between nares from
tip of the nose to anterior nasal spine of the maxilla)
• Deviation of the nasal septum is most frequent between the vomer and the septal
cartilage
Lateral wall • Superior, middle, and inferior nasal conchae and their corresponding meatuses
• Sphenoethmoid recess lies superior to the superior concha
• Vestibule, just inside each naris, is lined by skin
• Atrium, inferior to nasal bones, is lined by mucoperiosteum
The inferior concha
• It is a separate bone articulating with maxilla, lacrimal, and palatine bones on the lateral wall of the nasal cavity.
• Its meatus contains the opening of the nasolacrimal duct draining tears from medial aspect of the orbit into the nasal
cavity.
The posterior extent of the nasal cavity is adjacent to the opening of the auditory tube in the nasopharynx.
The middle concha is a process of the ethmoid bone and it overlies the middle meatus.
The superior concha is also a process of the ethmoid bone
• It overlies the superior meatus receiving the opening of the posterior ethmoidal air cells.
• Superior to the superior concha is the sphenoethmoidal recess where the sphenoid air cells drain into the nasal cavity.

Nerve Supply to the Nasal Cavity


The mucosa of upper nasal cavity is innervated by the olfactory (I) and trigeminal (V1; anterior ethmoidal) nerves.
The anterior ethmoidal nerve carries general sensation (pain, temperature, touch, and pressure).

https://t.me/DentalBooksWorld
484 Triumph’s Complete Review of Dentistry

Most of the general sensation of the lateral wall and nasal septum is mediated by V2, which is associated with the
pterygopalatine ganglion.
• The pterygopalatine ganglion receives the preganglionic parasympathetic fibers of the superficial (greater) petrosal
nerve.
• The postganglionic parasympathetic neurons send secretomotor fibers to glands above the floor of the mouth.

Anterosuperior part Anterior ethmoidal nerve


Posterosuperior part Posterior ethmoidal nerve
Anteroinferior part Nasopalatine nerve
Posteroinferior part Greater palatine nerve

Frontal bone
Crista galli

Frontal air sinus Sinus groove for Sphenoidal air sinus


ethmoidal nerve
Arculaon of nasal bone Cribriform plate Anterior clinoid
with septum of nose Olfactory tract process
Nasal bone Hypophyseal
fossa
Perpendicular
plate of ethmoid Posterior clinoid
Upper lateral carlage
process
Sphenoidal
Septal carlage rostrum
Lower lateral carlage

Vomer bone Clivus of skull

Lower lateral Lateral crus Posterior nasal


carlage Medial crus aperture (choanae)
Bony (osseous)
palate
Nasal crest
Maxillary tooth
Sphenoidal process
of septal carlage

BLOOD SUPPLY OF THE NASAL CAVITY


The arterial supply of the different parts of the lateral wall is as follows:
• Anterosuperior quadrant, by the anterior ethmoidal artery—a branch of the ophthalmic artery
• Anteroinferior quadrant, by branches of the facial and greater palatine arteries
• Posterosuperior quadrant, by the sphenopalatine artery—a branch of the maxillary artery
• Posteroinferior quadrant, by branches of the greater palatine artery, which pierces the perpendicular plate of the palatine
The nasal septum is supplied by the following arteries:
• Septal branch of the anterior ethmoidal artery (a branch of the ophthalmic artery)
• Septal branch of the posterior ethmoidal artery (a branch of the ophthalmic artery)
• Septal branch of the sphenopalatine artery (a branch of the maxillary artery)
• Septal branch of the greater palatine artery (a branch of the maxillary artery)
• Septal branch of the superior labial artery (a branch of the facial artery)

https://t.me/DentalBooksWorld
Chapter 6 • Embryology, Head and Neck Anatomy, and General Histology 485

Olfactory bulb Anterior ethmoidal nerve


with filaments

Olfactory bulb
with filaments
Anterior
ethmoidal Pterygopalane
nerve ganglion

Greater
palane nerve

Nasopalane
nerve

(a) (b)

PARANASAL AIR SINUSES


Paranasal air sinuses open into this meatus: The hiatus semilunaris opens on the wall of the middle meatus between the
unciform process of the ethmoid bone and the ethmoid bulla.
• Frontal sinus drains into the superior aspect of the hiatus semilunaris.
• Anterior and middle ethmoidal air sinuses drain through openings of the ethmoidal bulla on superoposterior aspect of the
hiatus semilunaris.
• Maxillary air sinus has its ostium directly inferior to the ethmoid bulla within the hiatus semilunaris.
• The sphenoid, ethmoid, frontal, and maxillary are paired but asymmetrical.
• They are lined with respiratory epithelium which becomes converted to the stratified squamous epithelium with chronic
respiratory irritation. This may lead to chronic sinusitis.

DRAINAGE OF PARANASAL AIR SINUSES


Meatuses Sinus drainage
Middle meatus (hiatus semilunaris) Maxillary sinus, anterior ethmoidal sinus, frontal sinus
Superior meatus Posterior ethmoidal sinus
Sphenoethmoidal recess Sphenoidal air sinus
Inferior meatus Nasolacrimal duct
Mastoid air sinuses drain into the nasal cavity via the middle ear and auditory tube

AUDITORY TUBE

Blood supply • Ascending pharyngeal artery


• Middle meningeal artery
• Artery of the pterygoid canal
Nerve supply • Glossopharyngeal nerve

https://t.me/DentalBooksWorld
486 Triumph’s Complete Review of Dentistry

Epithelium lined • Ciliated columnar epithelium


Opens at • Lower opening: anterior wall of the tympanic cavity
• Upper opening: posterior end of the inferior turbinate
• Pharyngeal opening: lateral wall of the pharynx

In infants • Tube is shorter, wider, and horizontal (in adults, it is vertical)


Develops from • First and second pharyngeal arch
Parts • Inner two-third: is cartilaginous
• Outer one-third: is bony
Length • Total: 36 cm
• 24 cartilaginous + 12 bony part
• In infants: 13–18 cm

The nasal septum mucosa is innervated by the nasopalatine nerve.


• It enters the nasal cavity from the pterygopalatine fossa through the sphenopalatine foramen.
• And descends on the median nasal septum
• The terminal branch leaves the nasal cavity via the incisive foramen.
The sympathetic innervation of the nasal cavities comes from the superior cervical ganglion. These postganglionic fibers reach
the nose via the nerve of the internal carotid artery and the deep petrosal nerve of the pterygoid canal. In the pterygopalatine
fossa, they join with terminal branches of the maxillary artery and are vasomotor to blood vessels in the nasal cavity and
palate.

THE ORAL CAVITY AND CONTENTS

THE VESTIBULE
• Is bounded by lips and cheeks.
• Is lined with the nonkeratinized stratified squamous epithelium.
• The parotid papilla is in the superior vestibule, opposite the second upper molar tooth.
• Is vascularized by the superior and inferior labial arteries from the facial artery. These arteries anastomose freely with their
contralateral counterparts. Because of these profuse anastomoses, lip bleeding is controlled by grasping the injured lip
between the fingers to stop the blood flow.
Nerve Supply of the Vestibule
• The orbicularis oris and buccinator muscles are innervated by cranial nerve VII (facial nerve).
• The skin and mucosa of the upper lip, cheek, and vestibule are innervated by the anterior, middle, and posterior superior
alveolar nerves from V2.
• The skin and mucosa of the lower lip and adjacent anterior vestibule are innervated by the mental nerve (V3).
• The mucosa of the inferior vestibule adjacent to the cheek is innervated by the long buccal nerve from the anterior division
of V3.

https://t.me/DentalBooksWorld
Chapter 6 • Embryology, Head and Neck Anatomy, and General Histology 487

ORAL CAVITY
• The roof is formed by the hard and soft palates with the midline uvula.
• The posterior border is formed by the pillars of the fauces.
• The floor is formed by the tongue divided into anterior two-third and posterior one-third by the palatoglossal arch, the
V-shaped sulcus terminalis, and circumvallate papillae (lying anterior to the sulcus).
• The lingual frenulum is found on the undersurface of the tongue with openings of the ducts of the submandibular
gland.
In examination of the tongue, grasp the tip of the tongue with gauze and pull the tongue out of the mouth. Examine the
lateral aspects of the anterior two-third of the tongue. This is a common site for cancer of the tongue.

FLOOR OF THE MOUTH (SUBLINGUAL REGION)


The Sublingual Gland
• It lies on the lingual aspect of the body of the mandible, deep to the plica sublingualis (sublingual fold), which is the
posterolateral continuation of the lingual frenulum.
• It has a row of 15 or 16 (“middle-teens”) ducts that empty into the floor of the mouth on the plica sublingualis.
• The duct of the submandibular gland and the lingual nerve lie on the medial surface of the sublingual gland.
• The mylohyoid muscle lies inferior to the sublingual gland.
• The sublingual gland is innervated by postganglionic parasympathetic fibers reaching the gland via its sensory nerve,
the lingual nerve (V3). Preganglionic parasympathetic fibers run with the chorda tympani (VII) synapsing in the
submandibular ganglion.
The Lingual Nerve
• It provides the general sensory (pain, touch, and temperature) modality to the anterior two-third of the tongue and the
floor of the mouth.
• It also carries chorda tympani which has special taste fibers and secretomotor fibers of VII.
• It enters the floor of the mouth on the medial mandible next to the third molar tooth. It is thus vulnerable in extraction of
the wisdom teeth.
• Preganglionic parasympathetic fibers leave the lingual nerve to synapse in the submandibular ganglion. Postganglionic
parasympathetic fibers rejoin the lingual nerve to reach the sublingual salivary gland. The submandibular ganglion is thus
suspended from the main trunk of the lingual nerve.
The lingual nerve is
• Superior to the mylohyoid muscle in the floor of the mouth
• Lateral to the submandibular duct
• And medial to the sublingual gland
• Subsequently, it passes inferior and then medial to the submandibular duct to ascend into the body of the tongue

• The chorda tympani provides taste fibers which supply the anterior two-third of the tongue. The cell bodies are in the
geniculate ganglion in the middle ear.
• The hypoglossal nerve enters the floor of the mouth on the lateral aspect of the hyoglossus muscle, above the hyoid bone
and the mylohyoid muscle. Cranial nerve XII lies inferior to the lingual nerve and is purely motor to the muscles of the
tongue.
• Test cranial nerve XII by protrusion of the tongue. Deviation is toward the side of the lesion.

https://t.me/DentalBooksWorld
488 Triumph’s Complete Review of Dentistry

THE TONGUE
SENSORY INNERVATION OF THE TONGUE

Region General sensation Taste sensation


Anterior two-third (including Lingual nerve Chorda tympani branch of the facial
circumvallate papillae for taste nerve
sensation)
Posterior one-third (except Glossopharyngeal nerve Glossopharyngeal nerve
circumvallate papillae for taste
sensation)

• The anterior two-third (body or oral part) is derived from the ectodermal stomodeum
• The posterior one-third (pharyngeal part or root) is derived from the endodermal foregut
• These two parts are separated by the sulcus terminalis posterior to the circumvallate papillae
• The sulcus terminalis is oriented posteriorly and the foramen cecum can be found at the tip of the V-shaped sulcus
terminalis. This is the point of origin of the thyroid gland
• Lingual tonsils are located posterior to the sulcus terminalis

Median glossoepigloc

Palatopharyngeal arch
Lingual tonsil
Sulcus
Foramen cecum
terminalis
Circumvallate
papillae
Fungiform
papillae

Fungiform
papillae
Filiform papillae

Mucous Membrane of the Tongue


• The papillae (filiform, fungiform, circumvallate, and foliate) are innervated by cranial nerve VII via the chorda tympani
(anterior two-third) and by IX (posterior one-third)
• The taste buds in the epiglottis and the pharyngeal walls are innervated by X
• The taste buds in the palate are innervated by cranial nerve VII via the greater petrosal nerve. Branches from the latter are
distributed by the greater and lesser palatine nerves

https://t.me/DentalBooksWorld
Chapter 6 • Embryology, Head and Neck Anatomy, and General Histology 489

Muscles of the Tongue


1.  The three extrinsic muscles of the tongue change the position of the tongue.
• The genioglossus attaches to the superior genial tubercles and protrudes the tongue
• The hyoglossus depresses the tongue
• The styloglossus retracts the tongue

2.  The intrinsic (longitudinal, transverse, and vertical) muscles change the shape of the tongue.

The Lingual Artery


• Is a branch from the external carotid artery and supplies the tongue
• Courses anteriorly on the middle constrictor parallel with cranial nerve XII
• The hyoglossus muscle intervenes between cranial nerve XII (lateral) and the lingual artery (medial). The lingual artery is
the only major structure medial to the hyoglossus muscle
• Dorsal lingual branches from the lingual artery are given to the dorsum of the tongue
• And deep lingual arteries are given to the body of the tongue
• Other terminal branches supply the genioglossus muscle and the sublingual gland
• Lymphatics follow arteries and drain to both right and left jugular lymphatic trunks of the neck

TONSILS

Main blood supply Tonsillar branch of facial artery


Main nerve supply Glossopharyngeal nerve/lesser palatine nerve
Lie over Superior constrictor
Developed from Second branchial pouch
Epithelium Stratified squamous, nonkeratinized

TEETH
In each adult jaw
• Four incisors
• Two canines
• Four premolars
• Six molars

Nerve Supply of Teeth and Gums


V2 supplies the teeth and gums of the maxillary arch
• The molar teeth are supplied by the posterior superior alveolar nerve from the pterygopalatine fossa.
• The bicuspids (premolars) are innervated by the middle superior alveolar nerve from the infraorbital nerve.
• The canines and incisors are innervated by the anterior superior alveolar nerve from the infraorbital nerve.
• The gums on the palatal surface are innervated by the nasopalatine nerve (incisors) and greater palatine nerve (bicuspids
and molars).
V3 supplies teeth and gums of mandibular arch
• The inferior alveolar (dental) nerve innervates all the teeth in the mandible.
• The gums of the molars and bicuspids are innervated by the long buccal nerve.
• The gums of the incisors are innervated by the mental nerve.
• The lingual gums are innervated by the lingual nerve.

https://t.me/DentalBooksWorld
490 Triumph’s Complete Review of Dentistry

Unilateral Facial Muscles Paralysis


Test
1. For loss of taste in the anterior two-third of the tongue for the integrity of the chorda tympani
2. For hyperacusis to test the integrity of the nerve to stapedius.
3. For lack of lacrimation on one side for the integrity of the greater (superficial) petrosal nerve. If this is present, it will
result in desiccation of the cornea, ulceration, and blindness. Desiccation of the cornea will result in pain sensation carried
by V1.
If all three signs are present, then the lesion is between the brainstem and the geniculate ganglion.
Bell’s palsy usually affects only branches of the facial nerve (VII) below the stylomastoid foramen.
Bell’s Palsy: Lesions of the facial nerve (cranial nerve VII)
• Drainage of tears and dribbling of saliva due to paralysis of the two main orbicularis muscles. Paralysis of the buccinator
will lead to accumulation of food in the vestibule.
• Test by asking patients to screw up the eye (loss of muscle tone causes the normal skin folds to disappear on the side of the
lesion), to smile, or to whistle.
• Muscles must be supported during recovery or they will stretch under gravity and cause a permanent asymmetry of the
face.
• The facial vein has a straighter path and communicates with deeper veins such as veins of the orbit (leading to the
cavernous sinus within the skull at the medial angle of the eye and the pterygoid venous plexus). The central face area is
thus a “danger area” for an infection on the face to travel into the skull or into the deep face.

LYMPHATIC DRAINAGE OF THE HEAD AND NECK


Lymph node sublevels of the neck

Infraorbital
Mastoid nodes lymph nodes
Mastoid process Facial vein
Parod nodes Buccal nodes
Occipital nodes Facial nodes
Digastric
Jugulo-digastric nodes Submandibular nodes

Hyoid
Internal jugular vein
Deep cervical nodes Trachea
Jugulo-omohyoid nodes
Pretracheal nodes
Trapezius Thyroid
Omohyoid
Clavicle

Probable Source of Nodal Metastasis


Level 1: Oral cavity, submandibular gland
Level 2: Nasopharynx, oropharynx, parotid, supraglottic larynx
Level 3: Oropharynx, hypopharynx, supraglottic larynx
Level 4: Subglottic larynx, hypopharynx, esophagus, thyroid

https://t.me/DentalBooksWorld
Chapter 6 • Embryology, Head and Neck Anatomy, and General Histology 491

Level 5: Nasopharynx, oropharynx


Levels 6 and 7: Thyroid, larynx, lung
Radical and Modified Radical Neck Dissection: Levels I–V

Lymph nodes in the head and neck are organized into two groups:
1.  A terminal (collecting) group which
• Is related to the carotid sheath
• Is also named the deep cervical group
• All lymph vessels from the head and neck drain directly to this group or indirectly via the superficial lymph nodes

2.  Intermediary, outlying groups


The jugular trunk:
• Is formed by efferents of the deep cervical group
• Drains on the right into the right lymphatic duct or at the junction between the internal jugular and subclavian vein
• Drains on the left into the thoracic duct or joins the internal jugular or subclavian vein

The deep cervical lymphatic nodes are organized into the following:
1.  Superior deep cervical nodes
• Can be found next to the upper portion of the internal jugular vein
• Most lie deep to the sternocleidomastoid muscle
• Drain to the lower inferior group or directly to the jugular trunk

The jugulodigastric node is responsible for lymphatic drainage of the tongue. It can be found in a triangle bordered by the
posterior belly of the digastric muscle and the facial and internal jugular veins.
2.  Inferior deep cervical lymph nodes are related to
• The deep surface of the sternocleidomastoid muscle
• The lower portion of the internal jugular vein
• The brachial plexus and subclavian vessels

The jugulo-omohyoid node at the level of the intermediate tendon of the omohyoid muscle is concerned with lymphatic
drainage of the tongue.
The inferior deep cervical lymph nodes drain into the jugular lymph trunk.
Lymphatic drainage of superficial tissues of the head and neck: Two types of drainage exist:
• Drainage by vessels afferent to local groups of nodes, which in turn drain to the deep cervical nodes
• Or, direct drainage to deep cervical nodes

Lymphatic Drainage of the Scalp and Ear


Submandibular nodes receive drainage from the frontal region above the root of the nose.
Superficial parotid nodes (anterior to the tragus, superficial and deep to the parotid fascia) receive drainage from the:
• Rest of the forehead
• Temporal region
• Upper half of the lateral auricular aspect
• Anterior wall of the external acoustic meatus
• Lateral vessels from the eyelids and skin of the zygomatic region
Efferent vessels drain to the upper deep cervical nodes
Upper deep cervical nodes and retroauricular (mastoid) nodes receive drainage from:
• A strip of scalp above the auricle
• The upper half of the cranial aspect and margin of the auricle
• The posterior wall of the external acoustic meatus

https://t.me/DentalBooksWorld
492 Triumph’s Complete Review of Dentistry

The retroauricular nodes are found:


• Superficial to the mastoid attachment of the sternocleidomastoid muscle
• Deep to the auricularis posterior
• They drain to the upper deep cervical nodes

Submandibular Nodes
• Lie deep to the cervical fascia, in the submandibular triangle
• Are usually three in number
–– One at the anterior pole of the submandibular gland
–– Two on either side of the facial artery as it reaches the mandible
–– Other nodes may be embedded in the submandibular gland or deep to it
• Drain a wide area from the
–– Submental nodes
–– Buccal (buccinator) nodes
–– Lingual nodes

• Drain to the upper and lower deep cervical nodes


• Drain directly the:
–– External nose
–– Cheeks
–– Upper lip and lateral part of the lower lips
–– The mucosa of lips and cheeks
–– A few buccal (buccinator) nodes may be present near the facial vein and they also drain to the submandibular nodes

The skin over the root of the nose and central forehead drains partly to the parotid nodes and partly to the submandibular
nodes.
The lateral part of the cheek drains to the parotid nodes.
Submental Nodes
• Are located on the mylohyoid, between the anterior bellies of the digastric muscles
• Receive bilateral afferents
• Have efferents running to the submandibular and jugulo-omohyoid nodes

Lymphatic Drainage of the Thyroid Gland


Lymphatic vessels from the thyroid gland communicate with
• Prelaryngeal nodes (above thyroid isthmus) via the tracheal plexus
• Pretracheal nodes
• Paratracheal nodes
• Brachiocephalic nodes (in superior mediastinum)
• Deep cervical nodes via the superior thyroid vessels
• And directly to the thoracic duct

Lymphatic Drainage of the Mouth


• Gingiva drains to the submandibular nodes.
• Soft and hard palate drain to the superior deep cervical nodes and the retropharyngeal nodes.
• Anterior part of the floor of the mouth drains via the submental nodes or directly to the superior deep cervical nodes.
• Rest of the floor of the mouth drains to the submandibular and superior deep cervical nodes.
Lymphatic Drainage of the Teeth
Lymphatics from the teeth pass to the submandibular and deep cervical nodes.

https://t.me/DentalBooksWorld
Chapter 6 • Embryology, Head and Neck Anatomy, and General Histology 493

Lymphatic Drainage of the Tonsil


The lymphatic drainage of the tonsil drains to the superior deep cervical nodes:
• Most to the jugulodigastric node
• Some to the small nodes on the lateral aspect of the internal jugular vein

Lymphatic Drainage of the Tongue


The lingual mucosal plexus is continuous with the intramuscular plexus.
• The anterior two-third of the tongue drains into the marginal and central vessels (AIPG 2014).
• The posterior one-third of the tongue drains into the dorsal lymph vessels.

Marginal vessels of the tongue


• Arise from the tip of the tongue and frenulum
• Drain bilaterally to the
–– Submental nodes
–– Jugulo-omohyoid node
–– Anterior or middle submandibular node
–– Jugulo-digastric nodes

Central vessels of the tongue follow the lingual vein to drain to:
• Deep cervical nodes (jugulodigastric and jugulo-omohyoid nodes)
• Submandibular nodes

Dorsal vessels of the tongue


• Join with the marginal vessels
• Drain into the jugulodigastric node or jugulo-omohyoid node

Lymphatic Drainage of the Pharynx and Cervical Part of the Esophagus


• Through the retropharyngeal or paratracheal nodes
• Or, directly to the deep cervical nodes

The epiglottis drains to the infrahyoid nodes.

QUICK FACTS

Father of anatomy Herophilus


Father of modern anatomy Andreas Vesalius

Metopic suture Separates the frontal bone into two halves


Present in 3–8% of individuals
Closes at 3–9 months of age
Obliterates at 7 years
Vertex Highest point in the sagittal suture
Bregma Meeting point between the coronal and sagittal suture
Lambda Meeting point between the sagittal and lambdoid suture
Obelion It is a point on the sagittal suture between two parietal formanien
Pterion The pterion is the region where the frontal, parietal, temporal, and sphenoid join together. It is
located on the side of the skull, just behind the temple
Asterion A point on the skull where the lambdoid, occipitotemporal, and parietotemporal sutures meet

https://t.me/DentalBooksWorld
494 Triumph’s Complete Review of Dentistry

Anatomical parameter Landmark


Beginning of the trachea C6
Bifurcation of the trachea T4–T5
Thyroid gland C5–T1
Sternal angle T4–T5
Beginning of the esophagus C6
Termination of the esophagus T11
Opening of the esophagus T10
Level of the larynx in adults C3–C6
Level of the larynx in infants C2–C3

Embryonic structure Adult structure


Frontonasal process Forehead, bridge of the nose
Medial nasal process Midline of the nose and philtrum of the lip
Lateral nasal process Ala of the nose
Nasolacrimal groove Nasolacrimal duct, lacrimal sac
Maxillary prominence Cheeks, lateral upper lip
Mandibular prominence Lower lip, lower jaw

Cranial nerve Columns


Olfactory SSA
Optic SSE
Oculomotor SSE and GVE
Trochlear GSE
Trigeminal GSA and SVE
Abducent SSE
Facial SVE, GVE, SVA/GVA, and GSA
Vestibulocochlear SSA
Glossopharyngeal SVE, GVE, GVA, and SVA
Accessory SVE
Hypoglossal GSA and SSE
GVE, general visceral efferent; GVA, general visceral afferent; SVE, special visceral efferent; SVA, special visceral afferent;
SSE, special somatic efferent; GSA, general somatic afferent; SSA, special somatic afferent.

https://t.me/DentalBooksWorld
Chapter 6 • Embryology, Head and Neck Anatomy, and General Histology 495

Spinal nerves Numbers


Cervical 8
Thoracic 12
Lumbar 5
Sacral 5
Coccygeal 1
Total 31

CRANIAL NERVE REFLEXES

Reflexes Afferent limb Efferent limb


Corneal Ophthalmic nerve (fifth cranial nerve) Seventh cranial nerve
Conjunctival Ophthalmic nerve (fifth cranial nerve) Seventh cranial nerve
Lacrimation Ophthalmic nerve (fifth cranial nerve) Seventh cranial nerve
Sneezing Maxillary nerve (fifth cranial nerve) Tenth cranial nerve
Jaw—jerk (masseteric reflex) Mandibular nerve (fifth cranial nerve) Mandibular nerve (fifth cranial nerve)
Oculomotor reflex Ophthalmic nerve (fifth cranial nerve) Tenth cranial nerve
Gag reflex Ninth cranial nerve Tenth cranial nerve
Carotid sinus reflex Ninth cranial nerve Tenth cranial nerve

Muscle Primary action Secondary action


Superior rectus Elevation Adduction and intorsion
Inferior rectus Depression Adduction and extorsion
Medial rectus Adduction –
Lateral rectus Abduction –
Superior oblique Intorsion Abduction and depression
Inferior oblique Extorsion Abduction and elevation

Lymphatic drainage of specific sites in head and neck region:


Skin of the head and neck Superficial cervical nodes
Eyelids Parotid and deep nodes
External nose and cheeks Buccal nodes and submandibular nodes
Lateral part of the cheek Parotid nodes
Buccal floor and lingual apex Submental node
Deeper tissue of head and neck Deep cervical nodes
Anterior part of the floor of the mouth Upper deep cervical nodes via submental nodes

https://t.me/DentalBooksWorld
496 Triumph’s Complete Review of Dentistry

Lateral part of the floor of the mouth Submandibular and superior deep cervical nodes
Tip of the tongue Submental lymph nodes
Anterior two-third of the tongue Submental and submandibular nodes and then to lower deep cervical
nodes
Posterior one-third of the tongue Upper deep cervical nodes
Hard palate Retropharyngeal and superior deep cervical nodes
Soft palate Retropharyngeal and superior deep cervical nodes
Tonsils Superior deep cervical nodes and then to jugulodigastric nodes
Rich submucous lymphatic plexus of mouth and Deep cervical lymph nodes
pharynx
Teeth Submandibular and deep cervical nodes
Gingiva Submandibular nodes
Maxillary gingiva (buccal) Submandibular lymph nodes
Maxillary gingiva (palatal) Superior deep cervical lymph nodes
Maxillary teeth Submandibular nodes
Hard palate Superior deep cervical nodes and retropharyngeal nodes
Soft palate Superior deep cervical nodes and retropharyngeal nodes
Tongue—tip of the tongue Submental nodes
Anterior 2/3rd Submandibular nodes and then to lower deep cervical nodes
Posterior 1/3rd Upper deep cervical nodes
Mandibular teeth—Incisors Submental nodes
Canines and posterior teeth Submandibular nodes
Mandibular gingiva—Buccal Submandibular
Mandibular gingiva—Lingual anterior Submandibular nodes
Mandibular gingiva—Lingual posterior Deep cervical nodes

Bony opening Bony location Nerves and vessels


Carotid canal Temporal Internal carotid artery
Cribriform plate with Ethmoid Olfactory nerves
foramina
External acoustic meatus Temporal Opening to tympanic cavity
Foramen lacerum Sphenoid, occipital, Cartilage
temporal
Foramen magnum Occipital Spinal cord, vertebral arteries, 11th cranial nerve
Foramen ovale Sphenoid Mandibular division of the fifth cranial nerve

https://t.me/DentalBooksWorld
Chapter 6 • Embryology, Head and Neck Anatomy, and General Histology 497

Foramen rotundum Sphenoid Fifth cranial nerve


Foramen spinosum Sphenoid Middle meningeal artery
Greater palatine foramen Palatine Greater palatine nerve and vessels
Hypoglossal canal Occipital Ninth cranial nerve
Incisive foramen Maxilla Nasopalatine nerve and branches of the sphenopalatine artery
Inferior orbital fissure Sphenoid and maxilla Infraorbital and zygomatic nerves, infraorbital artery,
ophthalmic vein
Infraorbital foramen and Maxilla Infraorbital nerve and vessels
canal
Internal acoustic meatus Temporal Seventh and eighth cranial nerves
Jugular foramen Occipital and temporal Internal jugular vein and ninth, tenth, and eleventh cranial
nerves
Lesser palatine foramen Palatine Lesser palatine nerve and vessels
Mandibular foramen Mandible Inferior alveolar nerve and vessels
Mental foramen Mandible Mental nerve and vessels
Optic canal and foramen Sphenoid Optic nerve and ophthalmic artery
Petrotympanic fissure Temporal Chorda tympani nerve
Pterygoid canal Sphenoid Area nerves and vessels
Stylomastoid foramen Temporal Seventh cranial nerve
Superior orbital fissure Sphenoid Third, fourth, and sixth cranial nerves, and ophthalmic nerve
and vein

Glandular Tissue
Lacrimal glands • Paired exocrine glands that secrete lacrimal fluid (tears) for lubrication of the conjunctiva, which
leaves the gland through tubules
• After passing over the eyeball, drained through a hole in each eyelid, gland terminates in the
nasolacrimal sac, structure behind medial canthus
Salivary glands • Produce saliva, which lubricates and cleanses oral cavity and aids digestion; include both major
and minor glands, defined by their size; exocrine glands with ducts that drain saliva directly into
the oral cavity where it is used; controlled by ANS
• With the connective tissue of the gland divided into the capsule, which surrounds the outer part,
and septa (plural of septum), each septum helps divide the inner part of the gland into larger lobes
and smaller lobules
• Major glands are large paired glands; ducts are named after them; include parotid, submandibular,
and sublingual
Parotid • Largest encapsulated gland; provides only 25% of the total volume; has purely serous secretion;
divided into two lobes: superficial and deep
• Parotid duct (Stensen’s), which emerges from the anterior border of the gland, pierces buccinator,
then opens into the oral cavity at the parotid papilla; occupies parotid fascial space, posterior to the
ramus, anterior and inferior to the ear; extends irregularly from zygomatic arch to the angle of the
mandible

https://t.me/DentalBooksWorld
498 Triumph’s Complete Review of Dentistry

• Innervated by parasympathetic nerves of the otic ganglion of the ninth cranial nerve
(glossopharyngeal), as well as afferent nerves from the auriculotemporal branch of the fifth cranial
nerve (trigeminal); drains into deep parotid nodes; supplied by the branches of the external carotid
artery
• Becomes enlarged and tender with mumps (unilateral or bilateral parotitis, inflammation of the
gland), viral disease that because of introduction of a vaccine is not a common childhood disease;
the salivary gland most commonly involved in tumorous growth, which can change consistency
and cause unilateral facial pain on involved side (seventh [VII] cranial nerve [facial] travels
through gland)
• Trauma can also occur to nerve from accidental overreaching of needle during inferior alveolar
nerve block, causing unilateral transient facial paralysis, temporary loss of movement of muscles
of facial expression on affected side; patient cannot close one eye, smiles asymmetrically, has
drooping lip on that side
Submandibular • Second largest encapsulated gland; provides 60–65% of the total volume with mixed secretion
• Submandibular duct (Wharton’s), which arises from deep lobe and remains inside mylohyoid,
travels along anterior floor of the mouth and then opens into oral cavity at the sublingual
caruncle; tortuous travel may be the reason that it is the gland most often involved in stone
formation
• Occupies submandibular fossa in submandibular fascial space, mostly superficial to the mylohyoid;
deep lobe wraps around the posterior part and is posterior to the sublingual
• Innervated by parasympathetic fibers of chorda tympani and submandibular ganglion of the
seventh (VII) cranial nerve (facial); drains into submandibular nodes; supplied by branches of
facial and lingual arteries
Sublingual • Smallest and only unencapsulated gland; provides only 10% of total volume with mixed
secretion
• Not just one major duct; sublingual ducts (Bartholin’s) open directly into oral cavity through the
gland and have other ducts that open along the sublingual fold; located in sublingual fossa in
sublingual fascial space at the floor of the mouth; superior to the mylohyoid, medial to the body of
the mandible, anterior to the submandibular
• Innervated by parasympathetic fibers of the chorda tympani and submandibular ganglion of the
seventh (VII) cranial nerve (facial); drains into submandibular nodes; supplied by sublingual and
submental arteries
Minor salivary • Smaller than major glands but more numerous, exocrine glands with unnamed ducts that are
glands shorter than major glands; scattered in buccal, labial, and lingual mucosa, soft palate, lateral parts
of the hard palate, floor of the mouth; include von Ebner’s glands (associated with circumvallate
lingual papillae)
• Secrete mainly mucous saliva, except von Ebner’s glands, which secrete only serous secretions
• Innervated by the seventh (VII) cranial nerve (facial); drain into various lymph nodes; supplied by
various arteries
Histology of • Secretory cells: Produce saliva; two types of epithelial cells in glands:
salivary glands –– Mucous cells: Cloudier-looking cytoplasm; produce mucous secretory product
–– Serous cells: Clearer cytoplasm; produce serous secretory product
• Acinus: Single layer of cuboidal epithelial cells surrounding the lumen, where saliva is deposited
after being produced; most acini match with type of cell, but mucoserous acini have mucous cells
surrounding lumen, with serous demilune located superficially
• Myoepithelial cells: On the surface of some acini to help flow of saliva (squeeze play)
• Duct system: Intercalated duct is associated with acinus, connected to striated duct, then
excretory duct

https://t.me/DentalBooksWorld
Chapter 6 • Embryology, Head and Neck Anatomy, and General Histology 499

SOME “Excepts”
• All muscles of the pharynx are supplied by the pharyngeal plexus except stylopharyngeus which is supplied by the
glossopharyngeal nerve.
• All muscles of the soft palate are supplied by the pharyngeal plexus except tensor veli palatini which is supplied by the
mandibular nerve (via nerve to the medial pterygoid).
• All muscles of the larynx are supplied by the recurrent laryngeal nerve except cricothyroid which is supplied by the
external laryngeal nerve.
• All muscles of the tongue are supplied by the hypoglossal nerve except palatoglossus which is supplied by the pharyngeal
plexus.

Unpaired sinuses Paired sinuses


Superior sagittal sinus Transverse sinus
Inferior sagittal sinus Sigmoid sinus
Straight sinus Superior petrosal sinus
Occipital sinus Inferior petrosal sinus
Intercavernous sinus Cavernous sinus
Note:
Eagleton’s criteria are for cavernous sinus thrombosis
Abducent nerve and ICA pass through the cavernous sinus
Neural tube derivatives of the brain:
Prosencephalon (forebrain) Telencephalon (cerebral hemispheres, lateral ventricle)
and diencephalon (thalamus, hypothalamus, and lateral
ventricle)
Mesencephalon (midbrain) Midbrain
Rhombencephalon (hind brain) Metencephalon (pons, upper half of the fourth ventricle)
and myelencephalon (medulla, caudal half of the fourth
ventricle)
Myelon Spinal cord

Vagus nerve
Superior laryngeal nerve
1. Internal laryngeal nerve 1. Anesthesia of the larynx
2. External laryngeal nerve 2. Loss of timber of voice/weakness in
phonation (AIPG 2008)
Recurrent laryngeal nerve Hoarseness of voice

nn HISTOLOGY

SOME ONE LINERS


• Epithelium has a high regeneration capacity
• Epithelium is derived from all three layers of germ layers
• Serosa is absent in esophagus

https://t.me/DentalBooksWorld
500 Triumph’s Complete Review of Dentistry

• Clavicle is the only long bone which does not contain the medullary cavity
• Clavicle is the only long bone that lies horizontally
• Clavicle is the only long bone which ossifies mainly in the membrane (intramembranous ossification)
• Metaphysis is the most vascular part of the bone
• Example of traction epiphysis is mastoid process
• Epiphysis is the growing end of the bone
• Diaphysis the longest and strongest part of a long bone
• Metaphysis is relatively weak
• Articular cartilage is a type of hyaline cartilage
• All hyaline cartilages are covered by perichondrium except articular cartilages
• Hyaline cartilage has the tendency to calcify except for the articular cartilage
• Articular cartilage lacks the ability to repair and regenerate itself
• Elastic cartilage is covered by the perichondrium
• Pubic symphysis is composed of fibrocartilage
• Gonial angle in children and old age: 140° (obtuse)
• Gonial angle in adults: 120°

Epithelium Tissue/structure
Simple cuboidal epithelium Thyroid follicles (inactive state), in case of active state—tall columnar cells
Pseudostratified columnar epithelium Larynx except vocal cord, trachea, bronchi, olfactory epithelium
Stratified squamous keratinized Epidermis of skin, ducts of sebaceous gland
Stratified squamous nonkeratinized Tonsils, tongue, laryngopharynx, epithelium of oral cavity, cornea,
conjunctiva, esophagus
Stratified cuboidal epithelium Duct of sweat glands, salivary glands, pancreas, ovarian follicle (the ovary is
simple cuboidal)

FASCICULAR ARCHITECTURE OF THE SKELETAL MUSCLE


Parallel fasciculi Examples
• Quadrilateral • Thyrohyoid, pronator quadratus
• Strake-like • Infrahyoid muscle, sartorius
• Strap-like with tendinous intersections • Rectus abdominis
• Fusiform • Biceps, digastric

Convergent fasciculi
• Triangular • Adductor longus
• Fan-shaped • Temporalis
Spiral or twisted fasciculi • Trapezius, latissimus dorsi, pectoralis major, supinator
Sphincter fasciculi • Orbicularis oculi, orbicularis oris
Pennate fasciculi
• Unipennate • Flexor pollicis longus
• Bipennate • Flexor hallucis longus
• Multipennate • Deltoid
• Circumpennate • Tibialis anterior

https://t.me/DentalBooksWorld
Chapter 6 • Embryology, Head and Neck Anatomy, and General Histology 501

STRUCTURAL CLASSIFICATION OF JOINTS


Fibrous • Sutures
• Syndesmosis (e.g., tympanostapedial joint)
• Gomphoses (peg and socket joint) (e.g., dentoalveolar peg)
Cartilaginous • Primary cartilaginous joints (synchondrosis or hyaline cartilage joints) (e.g.,
synostosis – spheno-occipital joint, costochondral joints)
• Secondary cartilaginous joints (symphyses or fibrocartilaginous joints) (e.g.,
symphysis pubis, intervertebral disc, symphysis menti (not true symphysis))
Synovial (diarthrodial joints) • Plane synovial joints
• Hinge joints (ginglymus joints) (e.g., elbow and ankle joints)
• Pivot or trochoid joints – Median atlanto-axial joints
• Condylar joints (bicondylar joints) (e.g., knee joint and TMJ)
• Ellipsoid joints (e.g., wrist joint and atlanto-occipital joint)
• Saddle (sellar) joints (e.g., sternoclavicular joint, incudomalleolar joints)
• Ball and socket (spheroidal joints) (e.g., hip joint (largest ball and socket joint),
shoulder joint, incudostapedial joint)
Schindylesis is a type of suture, a ridge of bone fits into groove of the other one, for example, vomer–sphenoid rostrum
junction

Laryngeal Cartilages
According to BDC
Total 9: Three paired and three unpaired (corniculate, cuneiform, and arytenoid are paired… thyroid, cricoid, and epiglottis
are unpaired)
According to Grey’s
Total 11: Four paired and three unpaired (corniculate, cuneiform, arytenoid, and tritiate are paired… thyroid, cricoid, and
epiglottis are unpaired)

Maxillary artery: Branches


The maxillary artery is divided into three • First (mandibular) part: Five branches, each of which enters a bony foramen:
portions by its relation to the lateral –– Deep auricular artery (enters the squamotympanic fissure)
pterygoid muscle: –– Anterior tympanic artery (enters the squamotympanic fissure)
• First (mandibular) part: Posterior –– Middle meningeal artery (enters the foramen spinosum)
to the lateral pterygoid muscle (five –– Accessory meningeal artery (enters the foramen ovale)
branches) –– Inferior alveolar artery (enters the mandibular foramen)
–– Artery to the mylohyoid
• Second (pterygoid or muscular)
part: Within lateral pterygoid muscle • Second (pterygoid or muscular) part: Five branches although pterygoid
(five branches) branches are irregular in their number and origin:
–– Anterior deep temporal branches
–– Posterior deep temporal branches

–– Pterygoid branches
–– Masseteric artery
–– Buccinator artery

https://t.me/DentalBooksWorld
502 Triumph’s Complete Review of Dentistry

• Third (pterygopalatine) • Third (pterygopalatine) part: Six branches including the terminal branch:
part: Anterior to lateral pterygoid –– Posterior superior alveolar artery
muscle (six branches including –– Infraorbital artery (enters the inferior orbital fissure)
terminal branch) (NEET 2018) –– Artery of the pterygoid canal
–– Pharyngeal artery (enters the palatovaginal canal)
–– Greater (descending) palatine artery (enters the greater palatine foramen)
–– Sphenopalatine artery—terminal branch (enters the sphenopalatine
foramen)
Branches of external carotid artery S: Superior thyroid artery. (Note: Inferior thyroid artery is a branch of
thyrocervical trunk which is in turn a branch of vertebral artery)
A: Ascending pharyngeal artery (only medial branch)
L: Lingual artery
F: Facial artery
O: Occipital artery
P: Posterior auricular artery
M: Maxillary artery
S: Superficial temporal artery
Branches of internal carotid artery — C1: Cervical segment
seven segments of internal carotid artery C2: Petrous (horizontal) segment
C3: Lacerum segment
C4: Cavernous segment
C5: Clinoid segment
C6: Ophthalmic (supraclinoid) segment
C7: Communicating (terminal) segment
C1: Cervical segment No branches
C2: Petrous (horizontal) segment • Vidian artery
• Caroticotympanic artery
C3: Lacerum segment –
C4: Cavernous segment • Inferolateral trunk: Supplies trigeminal ganglion
• Meningohypophyseal trunk: Supplies meninges and pituitary (through the
inferior hypophyseal artery)
C5: Clinoid segment –
C6: Ophthalmic (supraclinoid) segment • Superior hypophyseal artery
• Ophthalmic artery
C7: Communicating (terminal) segment • Posterior communicating artery
• Anterior choroidal artery
• Terminal branches: Middle cerebral
• artery and anterior cerebral artery

https://t.me/DentalBooksWorld
Chapter 6 • Embryology, Head and Neck Anatomy, and General Histology 503

MULTIPLE CHOICE QUESTIONS

OSTEOLOGY
1. Which of the following structures is inferior to sphenopetrosal synchondrosis?
A. Osseous part of auditory tube B. Abducens nerve
C. Petrosquamous sinus D. Cartilaginous part of auditory tube
2. All of the following are pneumatic bones, except
A. Ethmoid B. Frontal
C. Maxilla D. Mandible
3. Which is sustained in relation to hyoid bone?
A. Thyroid gland B. Thyroglossal cyst
C. Inferior thyroid artery D. Cricoid bone
4. Trachea bifurcates at
A. T6-T7 B. T7-T8
C. T4-T5 D. T5-T6
5. Name the paired cartilages
A. Cricoid B. Thyroid
C. Hyoid D. Corniculate
6. Which of the following is true?
A. Tracheobronchial cartilage is fibrous B. Thyroid cartilage is hyaline
C. Corniculate cartilage is elastic D. All of the above
7. Paired bones are
A. Ethmoid plate B. Vomer
C. Inferior concha D. Frontal bone
8. The adult larynx extends from cervical spine
A. C3-C4 B. C2-C2
C. C3-C6 D. C7-T1
9. Veins communicating the cavernous sinus to pterygoid plexus pass through the fossa/foramen of
A. Ovale B. Scarpa
C. Deltoid D. Langer
10. The larynx extends from
A. C5-C6 B. C3-C6
C. C2-C7 D. C1-C4
11. In the temporomandibular joint, least vascularity is seen in which of the following?
A. Articular cartilage B. Anterior part of articular disc
C. Posterior part of articular disc D. Central part of articular disc
12. Which of the following statements about orbital articulation is true?
A. The lateral wall of orbit is formed by the frontal bone, zygomatic bone, and greater wing of sphenoid
B. The inferior orbital fissure is formed between the medial wall and the floor of orbit
C. The medial wall of orbit is formed by maxilla, sphenoid, ethmoid, and the lacrimal bone
D. The floor is formed by maxilla, zygomatic, and ethmoid
13. The medial wall of orbit is formed by
A. Greater wing of sphenoid B. Anterior and posterior ethmoidal canals
C. Body of sphenoid D. Lesser wing of sphenoid
14. Structures passing through foramen magnum include all except
A. Spinal cord B. Spinal accessory nerve
C. Vertebral venous plexus D. Vertebral artery

https://t.me/DentalBooksWorld
504 Triumph’s Complete Review of Dentistry

15. Which laryngeal cartilage is above the glottis?


A. Epiglottis B. Arytenoid
C. Thyroid D. Cricoid
16. Mental foramen is located near
A. First premolar of mandible B. Second molar of mandible
C. Canine of mandible D. Canine of maxilla
17. Isthmus of thyroid is at the level of
A. First and second rings B. Second and third rings
C. Third and fourth tracheal rings D. Fifth and sixth rings
18. All of the following statements about Sternberg canal are true, except
A. Located posterior and lateral to foramen rotundum B. Located anterior and medial to foramen rotundum
C. Cause of intrasphenoidal meningocele D. Represents persistent craniopharyngeal canal
19. Which of the following bones does not contribute to the nasal septum?
A. Lacrimal B. Sphenoid
C. Ethmoid D. Palatine
20. The Laryngeal skeleton has
A. Three paired, three unpaired B. Three paired, two unpaired
C. Three paired, two unpaired D. Two paired, three unpaired
21. The commonest cartilage to ossify is
A. Fibrous B. Fibroelastic
C. Hyaline D. Elastic
22. The movement at the following joint permits a person to look toward the right or left
A. Atlanto-axial joint B. Atlanto-occipital joint
C. C3-C4 joint D. C2-C3 joint
23. All cervical vertebra have a
A. Body B. Spine
C. Bifid spinous process D. Transverse foramen
24. Which statement is correct?
A. The crista galli is a part of the ethmoid bone
B. The perpendicular plate of the ethmoid bone lies directly behind the vomer
C. The crista galli lies above the perpendicular plate of the ethmoid bone
D. The squamous part of the temporal bone lies directly behind and articulates with the lesser wing of the sphenoid
25. All synovial joints possess
A. A synovial lining B. A joint space
C. A joint capsule D. An articular disc
26. Which of the following is true?
A. Intervertebral discs are found between all cervical vertebrae
B. Intervertebral discs are found between all thoracic vertebrae
C. Intervertebral discs are found between all sacral vertebrae
D. Herniation of an intervertebral disc usually occurs on the posterior side of the disc
27. Motions of the head include nodding (as in the affirmative) and turning the head from side to side (as in the negative)
A. Nodding the head involves the atlanto-axial joint
B. Turning the head side to side involves the atlanto-occipital joints
C. The dens is part of the atlas
D. The atlas is able to rotate around the dens
28. Which statement concerning the paranasal sinuses is true?
A. The frontal sinuses open into the superior meatus
B. The paranasal sinuses closest to the hypophyseal fossa are the posterior ethmoidal cells
C. The middle ethmoid air cells drain into the hiatus semilunaris
D. General sensation is carried in fibers of the ophthalmic and maxillary nerve

https://t.me/DentalBooksWorld
Chapter 6 • Embryology, Head and Neck Anatomy, and General Histology 505

29. The following sinuses drain into these spaces respectively


A. Frontal sinus–superior meatus B. Posterior ethmoidal air spaces–sphenoethmoidal recess
C. Maxillary sinus–middle meatus D. A and C above
30. The superior orbital fissure
A. Lies directly inferior to the greater wing of the sphenoid
B. Lies directly above the posterior portion of the maxilla
C. Lies directly between the orbit and the infratemporal fossa
D. Transmits the III, IV, and VI cranial nerves

NERVES
1. Ext branch of sup. laryngeal nerve supplies
A. Posterior cricoarytenoid B. Cricoarytenoid
C. Cricothyroid D. Thyroarytenoid
2. The secretomotor fiber to the parotid gland passes through
A. Geniculate ganglion B. Lesser ganglion
C. Otic ganglion D. Sphenopalatine ganglion
3. All of the following are supplied by the facial nerve except
A. Nasal glands B. Parotid gland
C. Lacrimal gland D. Submandibular gland
4. The internal laryngeal nerve supplies
A. Cricothyroid membrane B. Vocalis
C. Mucous membrane below vocal fold D. None of the above
5. Which of the following is not a sign of stellate ganglion block?
A. Exophthalmos B. Miosis
C. Conjunctival redness D. Nasal congestion
6. Structures passing through the foramen ovale
A. Emissary B. Mandibular nerve
C. Trigeminal nerve D. All of the above
7. All the following branches of the Vth nerve supply the dura mater except
A. Anterior ethmoidal B. Posterior ethmoidal
C. Auriculotemporal D. Mandibular
8. Which of the following is concerned with the auditory pathway?
A. Trapezoid body B. Medial geniculate body
C. Genu of internal capsule D. Lateral lemniscus
9. Injury of which of these nerves causes vocal cord paralysis
A. Internal laryngeal B. Superior laryngeal
C. Recurrent laryngeal D. External laryngeal
10. Tympanic plexus is found by
A. Facial nerve B. Mandibular nerve
C. Tympanic branch of glossopharyngeal D. Vagus nerve
11. Which of the following nerves supplies the tip of the nose?
A. Ophthalmic nerve B. Maxillary nerve
C. Mandibular branch of Vth nerve D. Facial nerve
12. Which of the following muscles is supplied by the mandibular nerve?
A. Buccinators B. Masseter
C. Posterior belly of digastric D. Tensor veli palatini
13. Nerve supply of the tympanic membrane is by the
A. Auriculotemporal B. Lesser occipital
C. Greater occipital D. Parasympathetic ganglion

https://t.me/DentalBooksWorld
506 Triumph’s Complete Review of Dentistry

14. Which is not a feature of oculomotor nerve palsy?


A. Superior gaze palsy B. Diplopia
C. Miosis D. Difficulty in accommodation
15. The sensory supply for the palate is through all of the following, except
A. Hypoglossal nerve B. Facial nerve
C. Maxillary division of trigeminal nerve D. Glossopharyngeal nerve
16. Regarding trochlear nerve all are true except
A. Arises from dorsum of brainstem
B. Longest intracranial course
C. Enters orbit through sup. orbital fissure outside annulus of Zinn
D. Supplies ipsilateral superior oblique muscle
17. A patient has loss of sensation at the angle of mandible (jaw) and parotid area. Which of the following nerves is most
likely to be injured?
A. Lesser occipital nerve B. Greater auricular nerve
C. Mandibular nerve D. Auriculotemporal nerve
18. Which of the following cranial nerves presents in the posterior fossa?
A. 5th to 12th B. 6th to 12th
C. 3rd to 12th D. 4th to 12th
19. Adductors of arytenoid cartilage
A. Cricothyroid B. Posterior cricoarytenoid
C. Vocalis D. Transverse arytenoids
20. The facial nerve
A. Traverses through parotid gland B. Arises from the medulla oblongata
C. Carries no taste fibers D. Supplies muscles of mastication
21. Main nerve supply of the palatine tonsils is
A. Greater palatine nerve B. Lesser palatine nerve
C. Facial nerve D. Glossopharyngeal nerve
22. A 44-year-old woman came with a large abscess in the middle of the right posterior triangle of the neck and the
physician incised and drained the abscess. Five days later, the patient noticed that she could not extend her right hand
above her head to brush her hair. Which of the following are the signs and symptoms of additional harm?
A. Injury to suprascapular nerve B. Damage to scalenus medius
C. Spread of infection to shoulder joint D. Cut to the spinal part of the accessory nerve
23. All of the following structures are related to cavernous sinus except
A. Sixth nerve B. Mandibular division of trigeminal nerve
C. Optic nerve D. Fourth nerve
24. Which of the cranial nerves is related to the apex of the petrous temporal bone?
A. VII B. VIII
C. IX D. VI
25. The largest ganglion in the neck is
A. Stellate ganglion B. Second thoracic ganglion
C. Superior ganglion D. Middle ganglion
26. The superior oblique muscle is supplied by
A. Fourth cranial nerve B. Third cranial nerve
C. Sixth cranial nerve D. Fifth cranial nerve
27. During testing of facial nerve stimulation, which of the following is indicated by its contraction
A. Masseter B. Orbicularis oris
C. Temporalis D. Sternocleidomastoid
28. The following structures are in the lateral wall of the cavernous sinus except
A. Trochlear B. Oculomotor
C. Ophthalmic D. Trigeminal

https://t.me/DentalBooksWorld
Chapter 6 • Embryology, Head and Neck Anatomy, and General Histology 507

29. Lacrimal gland is supplied by which of the following ganglia?


A. Ciliary ganglion B. Otic ganglia
C. Submandibular D. Sphenopalatine ganglion
30. In the fracture of the middle cranial fossa, absence of tears would be due to lesion in
A. Ciliary ganglion B. Trigeminal ganglion
C. Cervical ganglion D. Greater petrosal nerve
31. All are supplied by ansa cervicalis except
A. Sternohyoid B. Inferior belly of omohyoid
C. Thyrohyoid D. Sternothyroid
32. The middle superior alveolar nerve is a branch of
A. Maxillary nerve B. Mandibular nerve
C. Facial nerve D. Lingual nerve
33. U/L injury to hypoglossal nerve leads to all except
A. Deviation of tongue toward the same side B. Hemiatrophy of the side involved
C. Fasciculation of the tongue D. Loss of taste sensation in one half of the tongue
34. Muscle supplied by glossopharyngeal nerve is
A. Styloglossus B. Thyrohyoid
C. Stylopharyngeus D. Stylohyoid
35. Dilator pupillae are supplied by
A. Oculomotor (III) cranial nerve
B. Sympathetic fibers from the V.C.N.
C. Postganglionic parasympathetic fibers from Edinger–Westphal nucleus
D. Postganglionic sympathetic fibers from cervical ganglion
36. The following statements concerning chorda tympani nerve are true except that it
A. Is a branch of facial nerve
B. Contains postganglionic parasympathetic fibers
C. Carries secretomotor fibers to submandibular gland
D. Joins the lingual nerve in infratemporal fossa
37. Which of the following is not supplied by the anterior division of mandibular nerve (V3)?
A. Lateral pterygoid B. Medial pterygoid
C. Temporalis D. Masseter
38. Recurrent laryngeal nerve has the following course in the neck
A. It passes posterior to inferior thyroid vessels
B. It passes anterior to the inferior thyroid artery
C. It passes between branches of inferior thyroid vessels
D. All of the above
39. Which is true about pharyngotympanic tube?
A. Supplied by glossopharyngeal N B. It opens into oropharynx
C. It is cartilaginous throughout its length D. Lined by stratified squamous epithelium
40. Lacrimal secretion is diminished when which site of nerve is damaged?
A. Parotid nerve B. Gasserian ganglia
C. Ciliary ganglia D. Pterygopalatine ganglion
41. Which structure passes through the infraorbital fissure?
A. Trochlear nerve B. Zygomatic nerve
C. Superior ophthalmic vein D. Ophthalmic artery
42. Alderman’s nerve is a branch of
A. Inferior cervical ganglion B. Superior cervical ganglion
C. None D. Auricular branch of the vagus nerve
43. The only pharyngeal muscle innervated by the glossopharyngeal nerve is
A. Superior constrictor B. Middle constrictor
C. Stylopharyngeus D. Palatopharyngeus

https://t.me/DentalBooksWorld
508 Triumph’s Complete Review of Dentistry

44. Which is not a content of tympanic cavity?


A. Chorda tympani B. Post. auricular nerve
C. Maleus D. Stapedius
45. General sensation to the posterior 1/3rd of the tongue is mediated by the
A. Vagus nerve B. Hypoglossal nerve
C. Lingual nerve D. Glossopharyngeal nerve
46. A person has inability to look downward and laterally. The nerve injured is
A. Trochlear B. Oculomotor
C. Trigeminal D. Abducent
47. Which nerve passes through Meckel’s cave?
A. Facial B. Abducens
C. Trochlear D. Trigeminal
48. True about sympathetic ganglion
A. Extends from the base of the skull to coccyx
B. Postganglionic fibers are myelinated
C. Cervicofacial area supplied from C2-4 craniospinal nerve
D. Consists of five ganglia
49. Nerve of pterygoid canal is
A. Sympathetic + greater superficial petrosal nerve B. Sympathetic + lesser petrosal nerve
C. Jacobson’s nerve D. Chorda tympani
50. Which nerve is in close relation with the root of the lower third molar?
A. Chorda tympani nerve B. Inferior alveolar nerve
C. Mylohyoid nerve D. Lingual nerve
51. Sensory fibers from the taste buds in the hard and soft palate travel along
A. Trigeminal nerve B. Facial nerve
C. Glossopharyngeal nerve D. Vagus nerve
52. Cricothyroid is supplied by
A. Internal laryngeal nerve B. Recurrent laryngeal nerve
C. Hypoglossal nerve D. External laryngeal nerve
53. The paralysis of third, fourth, sixth nerves, with involvement of ophthalmic division of fifth nerve, localizes the lesion
to
A. Brainstem B. Base of skull
C. Cavernous sinus D. Apex of orbit
54. Dorello’s canal transmits in the tip of temporal bone
A. Superior alveolar branch of maxillary B. Abducent nerve
C. Middle meningeal artery D. Mandibular nerve
55. The Eustachian tube is supplied by
A. Tympanic plexus B. Caroticotympanic nerve
C. Glossopharyngeal nerve D. All of the above
56. The nerve supplying the submandibular gland is
A. IX B. V
C. XII D. VII
57. Enophthalmos is due to palsy of
A. Superior tarsal muscle B. Levator palpebrae superioris
C. Inferior tarsal muscle D. Orbitalis muscle
58. Which of the following results from damage to the internal laryngeal nerve?
A. Loss of timbre of voice B. Hoarseness of voice
C. Breathing difficulty D. Anesthesia of the larynx
59. Sensory supply of external auditory meatus
A. Facial nerve nucleus B. Auriculotemporal nerve
C. Pterygomandibular ganglion D. Geniculate ganglion

https://t.me/DentalBooksWorld
Chapter 6 • Embryology, Head and Neck Anatomy, and General Histology 509

60. To give inferior alveolar nerve block, the nerve is approached lateral to pterygomandibular raphe between the
buccinator and the
A. Temporalis B. Superior constrictor
C. Middle constrictor D. Medial pterygoid
61. Which of the following ventral spinal rootlets is more prone to injury during decompressive operations because they
are shorter and exit in a more horizontal direction?
A. C5 B. C6
C. C7 D. T1
62. Posterior belly of digastric is supplied by
A. Accessory nerve B. Facial nerve
C. Mandibular nerve D. Hypoglossal nerve
63. The structure superficial to mylohyoid in anterior digastric DELTA is
A. Part of parotid gland B. Mylohyoid artery and nerve
C. Deep part of submandibular gland D. Hypoglossal nerve
64. Which of the following passes through the foramen magnum?
A. XIth cranial nerve B. Internal carotid artery
C. Vertebral artery D. Sympathetic chain
65. Which of the following is not true about the trochlear nerve?
A. Supplies the ipsilateral superior oblique muscle
B. Has the longest intracranial course
C. Enters orbit through the superior orbital fissure outside the annulus of Zinn
D. The only cranial nerve that arises from the dorsal aspect of the brainstem
66. The oculomotor nerve supplies the muscle of the eye except
A. Superior rectus B. Inferior rectus
C. Lateral rectus D. Inferior oblique
67. Sphenoplatine ganglion does not supply
A. Nasal mucosa B. Sublingual gland
C. Palate D. All of the above
68. The mandibular nerve passes through which of the following foramen?
A. Foramen spinosum B. Foramen lacerum
C. Foramen ovale D. Foramen rotundum
69. In carcinoma of tongue, pain is referred to the ear through
A. Vagus nerve B. Hypoglossal nerve
C. Lingual nerve D. Glossopharyngeal nerve
70. Cranial part of accessory nerve supplies all palatal muscles except
A. Palatopharyngeus B. Palatoglossus
C. Levator palate D. Tensor veli palate
71. The structure passing through superior orbital fissure
A. Cranial nerve II B. Ophthalmic nerve
C. Cranial nerve VI D. Cranial nerve I
72. Muscles spared by the accessory nerve
A. Cricopharyngeus B. Palatopharyngeus
C. Stylopharyngeus D. Salpingopharyngeus
73. Regarding palatine tonsil, which of the following is an incorrect statement?
A. Lymph from it drains into the deep cervical nodes B. Lies on the superior constrictor
C. Is a derivative of the second pharyngeal pouch D. Has sensory innervation from vagus
74. The hypoglossal nerve is related to
A. Digastric triangle B. Carotid triangle
C. Both of the above D. None of the above

https://t.me/DentalBooksWorld
510 Triumph’s Complete Review of Dentistry

75. The smallest muscles of head are supplied by


A. C4 B. C8
C. C5 D. C6
76. The stapedius muscle is supplied by which of the following nerves?
A. Glossopharyngeal B. Trochlear
C. Facial D. Vagus
77. Upper premolars are supplied by
A. Middle superior alveolar B. Anterior superior alveolar
C. Inferior alveolar D. Posterior superior alveolar
78. The septum of the nasal cavity is innervated by
A. Nasopalatine nerve from cranial nerve V2 B. Posterior ethmoidal nerve from V1
C. Greater palatine nerve from V2 D. Lesser palatine nerve from V2
79. All of the following are supplied by mandibular nerve except
A. Masseter B. Tensor tympani
C. Tensor palatini D. Buccinators
80. The structure passing through the internal auditory canal
A. Nerve of Wrisberg B. Superior vestibular nerve
C. Cochlear nerve D. All of the above
81. Which is direct content of cavernous sinus?
A. Trochlear nerve B. Ophthalmic division of trigeminal nerve
C. Oculomotor nerve D. Abducent nerve
82. The middle superior alveolar nerve is a branch of
A. Mandibular division of trigeminal nerve B. Platine division of maxillary nerve
C. Anterior nasal division of maxillary nerve D. None of the above
83. Dilator pupillae iridis is supplied by
A. Postganglionic parasympathetic fibers from ciliary ganglion
B. Postganglionic parasympathetic fibers from Edinger–Westphal nucleus
C. Postganglionic sympathetic fibers in the frontotemporal branch of CN V
D. Postganglionic sympathetic fibers from superior cervical ganglion
84. Sensory nerve supply of the middle ear cavity is provided by
A. Facial B. Glossopharyngeal
C. Trigeminal D. Vagus
85. The parasympathetic secretomotor fibers to parotid traverse through the following, except
A. Tympanic plexus B. Otic ganglion
C. Auriculotemporal nerve D. Greater petrosal nerve
86. The mandibular nerve does not supply
A. Tensor tympani B. Temporalis
C. Buccinators D. Masseter
87. Lacrimation is lost in the lesion of
A. Greater petrosal nerve B. Nasociliary nerve
C. Supraorbital nerve D. Ant. ethmoidal nerve
88. Sensory nerve supply of pinna is
A. Greater occipital B. Abducent nerve
C. Mandibular nerve D. Maxillary nerve
89. Major part of skin of pinna is supplied by
A. Posterior auricular nerve B. Great auricular nerve
C. Auriculotemporal nerve D. Auricular branch of the vagus
90. Fascia around nerve bundle of brachial plexus is derived from
A. Investing layer B. Superficial cervical fascia
C. Prevertebral fascia D. Pretracheal fascia

https://t.me/DentalBooksWorld
Chapter 6 • Embryology, Head and Neck Anatomy, and General Histology 511

91. Inferior oblique is supplied by the cranial nerve


A. Fifth B. Sixth
C. Third D. Fourth
92. The anterior ethmoidal nerve supplies all except
A. Dural sheath of anterior cranial fossa B. Ethmoidal air cells
C. Maxillary sinus D. Interior of nasal cavity
93. Which of the following does not give sensory supply to the pinna?
A. Lesser occipital nerve B. Tympanic branch of glossopharyngeal nerve
C. Auriculotemporal nerve D. Great auricular nerve
94. Muscle that is supplied by the facial nerve is
A. Risorius B. Anterior digastric
C. Stylohyoid D. All of the above
95. Tensor tympani is supplied by
A. Glossopharyngeal nerve B. Vagus nerve
C. Trigeminal nerve D. Facial nerve
96. Which nerve supplies the muscles of the palate?
A. Vagus nerve B. Greater palatine nerve
C. Hypoglossal nerve D. Glossopharyngeal nerve
97. Which of the following arteries does not accompany its corresponding named nerve throughout most of its course?
A. Infraorbital arteries B. Greater palatine arteries
C. Inferior alveolar arteries D. Lingual arteries
98. The tissues of the hard and soft palate receive innervation that is described by all of the following except
A. Parasympathetic fibers arrive by way of the lesser petrosal nerve
B. Sympathetic fibers arrive by way of the deep petrosal nerve
C. Afferent neuron cell bodies are in the trigeminal ganglion
D. The greater and lesser palatine nerves descend in the palatine canal
E. The anterior portion of the hard palate is supplied by the nasopalatine nerves
99. Which of the following is not formed from the cervical plexus?
A. Greater occipital nerve B. Ansa cervicalis
C. Supraclavicular nerve D. Transverse cervical nerve
E. Greater auricular nerve
100. Which statement is correct?
A. The posterior ethmoidal nerve is a branch of the nasociliary nerve
B. The lacrimal nerve lies medial to the frontal nerve
C. The external nasal nerve is a continuation or a branch of the nasociliary nerve
D. The greater occipital nerve is a branch of the ansa cervicalis
101. Which statement is correct?
A. The chorda tympani hitch-hikes a ride on the inferior alveolar nerve
B. The superior alveolar nerves are branches of the infraorbital nerve or of the maxillary nerve
C. The stylomandibular ligament is attached to the lingula of the mandible
D. Cranial nerve IX contains some taste fibers
102. The cranial nerve attaching to the medulla is/are
A. Hypoglossal B. Glossopharyngeal
C. Vagus D. Facial
103. Pick the odd one out
A. The facial nerve, as it descends through the facial canal, is entirely motor in function
B. The facial nerve leaves the skull through the stylomastoid foramen
C. The foramen spinosum is so called because it is near the styloid process
D. The zygomatic bone articulates with four other bones in the skull

https://t.me/DentalBooksWorld
512 Triumph’s Complete Review of Dentistry

104. Choose the best response


A. Autonomic motor nerves always contain a two neuron chain
B. Nerves III, V, VII, IX, and X contain parasympathetic preganglionic fibers where they are leaving the brain
C. The pterygopalatine ganglion has some sensory nerve fibers running through it
D. The optic nerve (II) contains parasympathetic fibers
105. Which of the following muscles is not innervated by the pharyngeal plexus?
A. Middle constrictor B. Tensor veli palatini
C. Salpingopharyngeus D. Internal (medial) pterygoid
106. Structures passing through the stylomastoid foramen are
A. The chorda tympani B. The lesser petrosal nerve
C. The hypoglossal nerve D. The main motor portion of the facial nerve
107. Choose the best statement with regard to the spinal accessory nerve
A. No fibers of this nerve branch from it before it crosses the posterior triangle of the neck
B. The spinal accessory nerve is strictly a sensory nerve
C. The spinal accessory nerve enters the cranial cavity through the foramen magnum
D. The spinal accessory nerve enters the cranial cavity through the foramen rotundum
108. The ansa cervicalis is found in this triangle of the neck
A. Occipital B. Supraclavicular
C. Digastric D. Carotid
109. The spinal accessory nerve passes through the posterior cervical triangle
A. Superficial to the platysma muscle
B. In the superficial cervical fascia
C. Between the investing fascia and the prevertebral fascia
D. Within the carotid sheath
110. The superior root of the ansa cervicalis is formed directly by a branch of the
A. Ventral primary ramus of C1 B. Dorsal primary ramus of C2
C. Dorsal primary ramus of C3 D. Hypoglossal
111. The skin over the angle of the mandible and over most, if not all of the parotid gland, is supplied by which cutaneous
nerve?
A. Buccal and mandibular branches of facial nerve B. Auriculotemporal nerve
C. Great auricular nerve D. Lesser occipital nerve
112. The patient’s left cornea is in an abducted and depressed position and the right cornea appears to be in the normal
position. What nerve is injured on the left side?
A. Oculomotor B. Trochlear
C. Abducens D. Nasociliary
113. A person has a drooping right eyelid and the superior division of the oculomotor nerve is normal, the person could
have a lesion in
A. Superior cervical sympathetic ganglion B. Optic nerve
C. Abducent nerve D. Trochlear nerve
114. A patient is asked to elevate his cornea when the eye is in an adducted position, he would use this muscle to perform
the requested movement
A. Superior rectus B. Inferior rectus
C. Levator palpebrae superioris D. Inferior oblique
115. Motor nerve that enters the orbit above and lateral to the common tendinous ring (annulus) is
A. Superior division of the oculomotor nerve B. Inferior division of the oculomotor nerve
C. Trochlear nerve D. Abducens nerve
116. Destruction of nerve cell bodies found in pterygopalatine fossa would result in
A. Constricted pupils B. Dilated pupils
C. Dry eye D. Loss of taste in the anterior 2/3 of the tongue
117. True statement concerning the nerve supply to the nasal cavity is
A. General sensory nerve fibers have their cell bodies in the geniculate ganglion
B. Postganglionic parasympathetic nerve fibers have their cell bodies in the pterygopalatine ganglion

https://t.me/DentalBooksWorld
Chapter 6 • Embryology, Head and Neck Anatomy, and General Histology 513

C. Postganglionic sympathetic nerve fibers have their cell bodies in the otic ganglion
D. General sensation from the posterior inferior part of the nasal cavity is by ethmoidal nerves
118. The pterygopalatine ganglion
A. Is a parasympathetic ganglion
B. Provides postganglionic fibers that distribute with branches of the maxillary division of the trigeminal nerve
C. Receives preganglionic parasympathetic fibers from IX
D. A and B
119. The parasympathetic ganglion that sends postganglionic fibers to the lacrimal gland is the
A. Lacrimal B. Ciliary
C. Pterygopalatine D. Otic
120. When a physician asks a patient to say “ah,” the movement of the soft palate indicates that this cranial nerve is
functioning
A. Trigeminal B. Facial
C. Glossopharyngeal D. Vagus
121. Smooth muscle of the upper eyelid is important in maintaining an open eye OR. Which of the following might be
damaged when the patient cannot keep his upper lid elevated due to paralysis of the smooth muscle that attaches to
the superior tarsal plate?
A. Short ciliary nerves or the nerve to the inferior oblique muscle
B. Oculomotor nerve—specifically, the portion from the Edinger–Westphal nucleus
C. Facial nerve
D. Superior cervical sympathetic ganglion
122. The glands of the mucous membranes of the nose and palate receive their parasympathetic postganglionic innervation
from the
A. Pterygopalatine ganglion B. Otic ganglion
C. Geniculate ganglion D. Trigeminal ganglion
123. The order in which the three branches of the facial nerve come off within the petrous portion of the temporal bone is
as follows
A. Lesser petrosal, greater petrosal, nerve to the stapedius muscle
B. Greater petrosal, chorda tympani, nerve to the stapedius muscle
C. Greater petrosal, nerve to the stapedius muscle, chorda tympani
D. Nerve to the stapedius, chorda tympani, greater petrosal
124. General sensation from the anterior part of the nasal cavity above the vestibule is carried by branches of the
A. Olfactory nerve B. Infraorbital nerve
C. Anterior ethmoidal nerve D. Nasopalatine nerve
125. If a person was unable to close the eyelids tightly due to nerve damage, the damaged nerve(s) would be the
A. Sympathetic fibers on the ophthalmic artery B. Superior division of the oculomotor
C. Inferior division of the oculomotor D. Facial
126. The cell bodies of postganglionic parasympathetic axons supplying the lacrimal gland are in the
A. Geniculate ganglion B. Pterygopalatine ganglion
C. Otic ganglion D. Trigeminal ganglion
127. Regarding the abducent nerve (CN V1)
A. The nucleus lies in the medulla B. Supplies superior oblique muscle
C. Enters the orbit via superior orbital fissure D. Has no relation to the cavernous sinus
128. Which nerve does not supply the skin of the upper eyelid?
A. Lacrimal B. Supraorbital
C. Supratrochlear D. Infraorbital
129. The nerve supplying general sensation to the floor of the mouth proper is the
A. Buccal nerve B. Glossopharyngeal
C. Lingual nerve D. Inferior alveolar
130. The tissues of the hard and soft palate receive innervation that is described by all of the following except
A. Parasympathetic fibers arrive by way of the lesser petrosal nerve
B. Sympathetic fibers arrive by way of the deep petrosal nerve

https://t.me/DentalBooksWorld
514 Triumph’s Complete Review of Dentistry

C. Afferent neuron cell bodies are in the trigeminal ganglion


D. The greater and lesser palatine nerves descend in the palatine canal
131. Which of the following is not formed from the cervical plexus?
A. Greater occipital nerve B. Ansa cervicalis
C. Supraclavicular nerves D. Transverse cervical nerve
132. If the facial nerve (VII) is injured just proximal to the origin of the chorda tympani nerve, the patient would experience
which of the following symptoms
A. Loss of taste sensation to the anterior 2/3rd of the tongue
B. Decreased saliva from the submandibular salivary gland
C. Loss of lacrimation (tearing) from the lacrimal gland
D. Both A and B
133. The preganglionic parasympathetic fibers to the otic ganglion enter through the _______ nerve
A. Greater petrosal B. Deep petrosal
C. Lesser petrosal D. Chorda tympani
134. Which of the following is/are true of the parasympathetic division of the ANS?
A. Originated from cranial nerves II, VI, XI
B. Is also referred to as the craniosacral division
C. Has long postganglionic fibers and short preganglionic fibers
D. Both A and B
135. Which of the following ganglia does not contain synapses?
A. Submandibular ganglion B. Geniculate ganglion
C. Superior cervical sympathetic ganglion D. Otic ganglion
136. Mandibular division of the trigeminal nerve (V3)
A. Provides general cutaneous fibers to the forehead above the orbit
B. Provides motor fibers to the masseter and temporalis muscle
C. Provides general sensation to the posterior 1/3rd of the mouth
D. Provides general cutaneous sensory fibers to that portion of the neck that develops from the second pharyngeal (hyoid)
arch in the embryo
137. The facial nerve (VII)
A. Exits from the skull via the external auditory meatus
B. Contains only sensory nerve fibers
C. Provides motor fibers to the platysma and buccinator muscles
D. Provides parasympathetic secretomotor fibers to the parotid gland
138. The following cranial nerve(s) are considered to have a parasympathetic function
A. III, IV, and V B. III, V, and VI
C. III, V, and VII D. III, VII, and IX
139. The nerve of the pterygoid canal
A. Receives its contributions from a branch of the glossopharyngeal nerve
B. Contains only sympathetic fibers
C. Enters the pterygopalatine fossa
D. Is formed by the joining of the greater petrosal and lesser petrosal nerves
140. The phrenic nerve
A. Is part of the cervical plexus B. Is derived from spinal segments C3, C4, and C5
C. Lies on the belly of the anterior scalene muscle D. A, B, and C
141. The lesser occipital nerve is
A. Derived from posterior rami of C2-C3 B. A motor nerve to part of the scalp
C. A part of the cervical plexus D. A mixed nerve
142. The suboccipital nerve
A. Is sensory to the back of the neck
B. Emerges between the rectus capitis major and minor muscles
C. Supplies motor innervation to the splenius capitis
D. Supplies motor innervation to the muscles forming the suboccipital triangle

https://t.me/DentalBooksWorld
Chapter 6 • Embryology, Head and Neck Anatomy, and General Histology 515

143. The only cranial nerve to emerge from the dorsal side of the brainstem is the
A. Oculomotor B. Facial
C. Trigeminal D. Trochlear
144. The sensory innervation of the auricle of the ear includes
A. Auriculotemporal nerve (V3) B. Great auricular nerve (C2)
C. Facial nerve via tympanic plexus (VII) D. All of the above
145. Sensation (heat, cold, touch) of the gingiva of the hard palate is carried to the brain through
A. The nerve of the pterygoid canal B. Nerves in the greater palatine canals
C. The nasociliary branch of V-1 D. The greater superficial branch of the vagus n.
146. The pterygopalatine ganglion is suspended from which of the following nerves?
A. The nerve of the pterygoid canal B. The main trunk of V-1
C. The main trunk of V-2 D. The main trunk of V-3
147. The deep petrosal nerve
A. Contains parasympathetic fibers from VII
B. Carries preganglionic sympathetic fibers from the sympathetic plexus on the internal carotid artery
C. Carries postganglionic parasympathetic fibers from cranial nerve IX
D. Carries postganglionic sympathetic fibers from the superior cervical ganglion
148. The greater superficial petrosal nerve contains autonomic fibers from which of the following nerves?
A. V-3 B. V-2
C. VII D. IX
149. The cranial nerve that supplies the sensory innervation to the dura above the level of the tentorium cerebelli is the
A. Trigeminal B. Facial
C. Glossopharyngeal D. Vagus
150. This muscle receives motor fibers derived from the cervical plexus
A. Respiratory diaphragm B. Levator scapulae
C. Omohyoid D. All of the above
151. The cranial nerve that is sensory to the carotid body and sinus is the
A. Trigeminal B. Facial
C. Hypoglossal D. Glossopharyngeal
152. Muscle that receives its motor supply from two cranial nerves
A. Buccinator B. Trapezius
C. Digastric D. Omohyoid
153. Auriculotemporal nerve of the scalp is a branch of the
A. Glossopharyngeal nerve B. Ophthalmic division of V
C. Maxillary division of V D. Mandibular division of V
154. The following nerves are found in the posterior triangle of the neck
A. Lesser occipital nerve B. Recurrent laryngeal nerve
C. Spinal accessory nerve D. Only A and C
155. The dura matter of the posterior cranial fossa is supplied by the meningeal branch of this nerve
A. Mandibular B. Ophthalmic
C. Maxillary D. DPR of C2-C3
156. The nerve supplying the carotid artery and sinus is a branch of cranial nerve
A. V B. VII
C. IX D. X
157. The mucous membrane of the larynx below the vocal folds is supplied by this nerve
A. Pharyngeal B. Recurrent laryngeal
C. Internal laryngeal D. External laryngeal
158. The sensory nerve supply to the skin overlying the angle of the mandible is by the
A. Greater occipital nerve (C2)
B. Lesser occipital nerve (C2)
C. Buccal branch of mandibular division of the trigeminal nerve
D. Great auricular nerve (C2-C3)
https://t.me/DentalBooksWorld
516 Triumph’s Complete Review of Dentistry

159. Which statement is correct?


A. The facial nerve (VII), being completely motor, possesses no ganglion
B. The trigeminal ganglion contains some parasympathetic cells supplying structures in the hand
C. A portion of the facial nerve has a ganglion containing cells whose fibers bring taste information from the anterior
2/3rd of the tongue
D. The foramen rotundum is located in the posterior part of the frontal bone
E. None of the above are correct
160. Which statement is incorrect?
A. Parasympathetic outflow in the vagus nerve causes a decrease in the rate of heartbeat
B. The pterygopalatine ganglion contains both parasympathetic fibers of the seventh nerve as well as sensory fibers whose
cells of origin are in the trigeminal ganglion
C. The lacrimal gland receives secretomotor fibers from cranial nerve III
D. The zygomatic branch of the facial nerve innervates the orbicularis oculi muscle
161. Which statement is incorrect?
A. Two cranial nerves (VII and VIII) leave the posterior cranial fossa through the internal acoustic meatus
B. Three cranial nerves pass through the jugular foramen
C. The trigeminal nerve is exclusively sensory
162. Which is incorrect?
A. In a fourth cranial nerve paralysis, the affected eye lies slightly inturned
B. In a sixth cranial nerve palsy, the lateral rectus is affected
C. In a third nerve palsy, levator palpebrae is affected
D. In an oculomotor nerve palsy, the eye looks down and out

MUSCLES
1. Which ocular muscle does not arise from the apex of the orbit?
A. Superior rectus B. Inferior rectus
C. Superior oblique D. Inferior oblique
2. Following are the TMJ joint ligaments except
A. Stylomandibular B. Tympanomandibular
C. Temporomandibular D. Sphenomandibular
3. Anterior triangles of the neck are all, except
A. Carotid triangle B. Submental triangle
C. Muscular triangle D. Supraclavicular triangle
4. All are true about scalenus anterior except
A. It is pierced by the phrenic nerve
B. It separates the subclavian vein from the subclavian artery
C. It is attached to the tubercle of second rib
D. It is anterior to the transverse cervical artery
5. Nerve supply of the mucosa of the larynx is
A. Internal laryngeal B. Hoarseness
C. Superior laryngeal D. External laryngeal
6. The following ligaments are present in temporomandibular joint, except
A. Stylomandibular ligament B. Alar ligament
C. Lateral temporomandibular ligament D. Sphenomandibular ligament
7. Muscles of the tongue are supplied by
A. Chorda tympani B. Hypoglossal
C. Lingual nerve D. Glossopharyngeal nerve
8. Ipsilateral deviation of tongue is due to the unaltered action of
A. Hyoglossus B. Lateral pterygoid
C. Genioglossus D. Medial pterygoid

https://t.me/DentalBooksWorld
Chapter 6 • Embryology, Head and Neck Anatomy, and General Histology 517

9. Subcutaneous muscle extending from the clavicle to the mandible


A. Omohyoid B. Sternohyoid
C. Platysma D. Sternocleidomastoid
10. Which of the following is a prime mover in head flexion?
A. Occipitofrontalis B. Corrugator supercilii
C. Sternocleidomastoid D. Masseter
11. The ansa cervicalis innervates which muscle?
A. Stylohyoid B. Sternohyoid
C. Mylohyoid D. Cricothyroid
12. The larynx has
A. Nine cartilages
B. Six cartilages
C. All intrinsic muscles of larynx including cricothyroid are supplied by recurrent laryngeal nerve
D. Two paired, two unpaired
13. Toynbee’s muscle is
A. Tensor tympani B. Stapedius
C. Levator ani D. Scalenus minimus
14. In dislocation of the jaw, displacement of the articular disc beyond the articular tubercle of the temporomandibular
joint results from spasm or excessive contraction of the following muscle
A. Buccinator B. Lateral pterygoid
C. Masseter D. Temporalis
15. The isthmus of the thyroid gland is across tracheal ring
A. Fifth to sixth B. Fourth only
C. Second to fourth D. Third to fifth
16. The antagonistic muscle to superior rectus is
A. Inferior rectus B. Inferior oblique
C. Lateral rectus D. Superior oblique
17. All of the following structures pierce the buccinator muscle except
A. Buccal branch of facial nerve B. Buccal branch of the mandibular nerve
C. Parotid duct D. Molar glands of the cheek
18. Abductors of vocal cord
A. Cricothyroid B. Posterior cricoarytenoids
C. Vocalis D. Transverse arytenoids
19. All are extrinsic membranes of the larynx except
A. Hyoepiglottic B. Cricotracheal
C. Cricothyroid D. Thyrohyoid
20. Posterior boundary of the carotid triangle is
A. Digastric posterior bally B. Sternocleidomastoid anterior margin
C. Omohyoid superior bally D. Sternocleidomastoid posterior margin
21. Oral diaphragm is formed by
A. Hyoglossus B. Buccinator
C. Mylohyoid D. Genioglossus
22. Floor of the digastric triangle is formed by
A. Digastric posterior belly B. Digastric anterior belly
C. Stylohyoid D. Mylohyoid
23. Which is true about opening of the jaw?
A. Articular disc moves backward B. Head of the mandible moves forward
C. Lateral pterygoid muscle contracts D. Movement in vertical axis
24. Intrinsic muscles and extrinsic muscles of the tongue are derived from
A. Pharyngeal arch mesenchyme B. 2nd branchial cleft
C. Cervical somites D. Occipital myotome

https://t.me/DentalBooksWorld
518 Triumph’s Complete Review of Dentistry

25. Safety muscle of the tongue is


A. Genioglossus B. Hyoglossus
C. Styloglossus D. Palatoglossus
26. Infrahyoid ribbon muscles are contained in
A. Anterior digastric triangle B. Carotid triangle
C. Muscular triangle D. Submental triangle
27. The smallest muscle in the body is
A. Stapedius B. Interarytenoid
C. Superior oblique D. Corrugator supercilli
28. On both sides, submental triangle is bounded by
A. Anterior digastric B. Hyoid bone
C. Mylohyoid D. Posterior digastric
29. Depressor of mandible is
A. Masseter B. Median pterygoid
C. Temporalis D. Lateral pterygoid
30. Opening of the jaw is done by
A. Masseter B. Median pterygoid
C. Temporalis D. Lateral pterygoid
31. Opening of the nasolacrimal duct is in
A. Inferior meatus B. Superior meatus
C. Middle meatus D. Sphenoethmoidal recess
32. All of the following are digastric muscles, except
A. Occipitofrontalis B. Sternocleidomastoid
C. Muscle fibers in the ligament of Treitz D. Omohyoid
33. Lockwood ligament is found in
A. Larynx B. TMJ
C. Orbit D. Pharynx
34. Abductor of the vocal cord is
A. Cricothyroid B. Thyroepiglottics
C. Posterior cricoarytenoid D. Lateral cricoarytenoid
35. Superficial incision on the posterior triangle of neck leads to
A. Difficulty in shrugging of shoulder B. Difficulty in start of abduction
C. Difficulty in rotation of face toward the same side D. Difficulty in flexion
36. All of the following muscle are grouped together as “muscles of mastication” except
A. Temporalis B. Pterygoids
C. Buccinator D. Masseter
37. Which one of the following statements are true regarding nasopharynx
A. Passavant’s is formed by stylopharyngeus
B. Fossa of Rosenmuller corresponds to the ICA
C. Lower border of the nasopharynx corresponds to the upper faucial pillar
D. None of the above
38. Posterior boundary of a carotid triangle is
A. Sternohyoid B. Sternocleidomastoid
C. Sup. belly of omohyoid D. Post. belly of digastric
39. All are suprahyoid muscles except
A. Geniohyoid B. Mylohyoid
C. Digastric D. Omohyoid
40. The recurrent laryngeal nerve supplies
A. Stylopharyngeus B. Omohyoid
C. Vocalis muscle D. Cricothyroid

https://t.me/DentalBooksWorld
Chapter 6 • Embryology, Head and Neck Anatomy, and General Histology 519

41. Palsy of right genioglossus causes


A. Deviation of soft palate to right B. Deviation of soft palate to left
C. Deviation of tongue to right D. Deviation of tongue to left
42. The action of superior oblique muscle is A/E
A. Extorsion B. Depression
C. Abduction D. Intorsion
43. Superior malleolar ligament connects
A. Head of malleus to roof of epitympanum B. Malleus to incus
C. Malleus to fossa incudis D. Incus to fossa incudis
44. Which of the following anatomic landmark would be useful in defining the borders of the posterior triangle of the
neck A/E?
A. The posterior border of the sternocleidomastoid muscle
B. The posterior border of the anterior scalene muscle
C. Difficulty in shrugging of shoulder
D. The anterior border of the trapezius muscle
45. Five pairs of muscles contribute to the structure of the soft palate. Which statement is incorrect?
A. The fan-shaped tendons of the tensor veli palatini form palatal aponeurosis
B. The tendons of the tensor veli palatini hook around the pterygoid hamulus on each side
C. The levator veli palatini passes between the base of the skull and the superior fibers of the superior pharyngeal
constrictors
D. The palatopharyngeus muscles attach to the hyoid bone
46. The following statements concerning the falx cerebri are true except
A. It arises from the crista galli of the ethmoid bone
B. It lies between the right and left cerebral hemispheres
C. It contains the superior sagittal sinus
D. It fuses with the tentorium cerebelli in the region of the inferior sagittal dural venous sinus
47. The pterygomandibular raphe is essentially a vertical line of connective tissue indicating the plane of interconnection
of the buccinator and the
A. Palatoglossal muscle B. Palatopharyngeal muscle
C. Superior pharyngeal constrictor muscle D. Middle pharyngeal constrictor muscle
48. The following strap muscles of the neck are innervated by the ansa cervicalis
A. Omohyoid B. Sternohyoid
C. Sternothyroid D. Thyrohyoid
49. Which of the following muscles cannot be found in the floor of the posterior triangle of the neck?
A. Scalenus medius B. Scalenus anterior
C. Teres minor D. Levator scapulae
50. The platysma muscle is innervated by which motor nerve
A. Ansa cervicalis B. Transverse cervical
C. Supraclavicular D. Facial
51. The medial pterygoid muscle
A. Originates from the medial pterygoid plate of the sphenoid bone
B. Inserts into the medial surface of the neck of the mandible
C. Has the lingual nerve passing forward on its deep surface
D. Elevates the mandible
52. The muscles that extort the cornea of the eyeball are
A. Inferior rectus and superior oblique B. Inferior rectus and inferior oblique
C. Superior rectus and inferior oblique D. Superior rectus and superior oblique
53. The structure that occupies the fenestra vestibuli is the
A. Facial nerve B. Base of the stapes
C. Stapedius muscle D. Tensor tympani muscle

https://t.me/DentalBooksWorld
520 Triumph’s Complete Review of Dentistry

54. The pyramid of the middle ear contains the


A. Stapedius muscle B. Tensor tympani muscle
C. First turn of the cochlea (basal turn) D. Chorda tympani nerve
55. The vocal ligament is the thickened upper margin of the lateral portion of the
A. Quadrangular membrane B. Hyoepiglottic ligament
C. Thyroepiglottic D. Cricothyroid ligament
56. Retraction of the tongue is produced mainly by the contraction of these two muscles
A. Right and left genioglossus B. Styloglossus and hyoglossus
C. Palatoglossus and genioglossus D. Palatoglossus and stylopharyngeus
57. If all extraocular muscles were paralyzed except for the inferior oblique muscle, the cornea would be in this position
A. Adducted and elevated B. Abducted and elevated
C. Adducted and depressed D. Abducted and depressed
58. The muscle of the orbit that attaches to the eyeball and does not arise from the common tendinous ring is the
A. Inferior rectus B. Superior rectus
C. Superior oblique D. Medial rectus
59. The following muscle is an elevator of the larynx
A. Omohyoid B. Sternothyroid
C. Thyrohyoid D. Digastric
60. The muscle that is most important for narrowing the anterior part of the rima glottidis is the
A. Thyroarytenoid B. Cricothyroid
C. Oblique arytenoid D. Lateral cricoarytenoid
61. The upper free edge of this membrane forms the vocal ligament
A. Quadrangular B. Thyrohyoid
C. Cricothyroid D. Hyoepiglottic
62. The depressions on each side of the median glossoepiglottic fold are the
A. Piriform sinuses B. Pharyngeal recesses
C. Ventricles D. Valleculae
63. The vestibular ligament of the larynx is the lower margin of the
A. Median thyrohyoid ligament B. Lateral thyrohyoid ligament
C. Cricothyroid ligament D. Quadrangular membrane
64. The muscle that abducts the membranous (ligamentous) part of the vocal folds is the
A. Transverse arytenoid B. Oblique arytenoid
C. Lateral cricoarytenoid D. Posterior cricoarytenoid
65. Five pairs of muscles contribute to the structure of the soft palate. Which statement is incorrect?
A. The fan-shaped tendons of the veli palatini form the palatal aponeurosis
B. The tendons of the tensor veli palatini hook around the pterygoid hamulus on each side
C. The levator veli palatini passes between the base of the skull and the superior fibers of the superior pharyngeal
constrictors
D. The palatopharyngeus muscles attach to the hyoid
66. This portion of this muscle retracts the mandible
A. Superficial head of the medial pterygoid muscle B. Posterior portion of the temporalis
C. Upper head of the lateral pterygoid D. Anterior portion of the temporalis
67. The periosteum of the skull is called
A. Arachnoid B. Leptomeninx
C. Pia mater D. Pericranium
68. The following muscles are innervated by the ansa cervicalis
A. Sternohyoid B. Geniohyoid
C. Thyrohyoid D. Only A and B
69. Which of the following muscles is attached to the spinous process of the axis?
A. Splenius capitis B. Semisplenius capitis
C. Rectus capitis posterior major D. All of the above

https://t.me/DentalBooksWorld
Chapter 6 • Embryology, Head and Neck Anatomy, and General Histology 521

70. The infrahyoid muscles (strap muscles of the neck)


A. Are the sternohyoid, sternothyroid, omohyoid, and thyrohyoid
B. Are all innervated by the ansa cervicalis
C. Include one muscle that attaches to the scapula
D. Only A and C
71. The ligament that is the most important for preventing the posterior displacement of the odontoid process from the
anterior arch of the atlas is the
A. Posterior atlanto-occipital membrane B. Alar ligament
C. Cruciate ligament D. Posterior longitudinal ligament
72. The two important structures found in the suboccipital triangle are the vertebral artery and the
A. Lesser occipital nerve B. Greater occipital nerve
C. Suboccipital nerve D. Spinal accessory nerve
73. Regarding the extraocular muscles
A. They all arise from a common tendinous ring around the superior orbital fissure
B. The rectus muscles all pass laterally to their point of insertion because of the angle of the orbital apex within the skull
C. The superior oblique muscle passes through the trochlear, a fibrocartilage loop attached to the frontal bone lateral
margin, just behind the orbital margin
D. Except for superior oblique, all nerve supply to muscles enters from their optic side
74. With regard to the eye movement, which is incorrect?
A. The medial and lateral rectus evoke simple horizontal movements only
B. The inferior oblique turns the eye down and out
C. The superior rectus turns the eye up and in
D. The inferior rectors and superior oblique together produce vertical down gaze
75. Which is an intrinsic muscle of the tongue?
A. Superior longitudinal muscle B. Genioglossus muscle
C. Styloglossus muscle D. Palatoglossus muscle
76. Which structure passes through the tendinous ring of the orbit?
A. Ophthalmic artery B. Inferior rectus muscle
C. Lacrimal nerve D. Trochlear nerve
77. Protection of the airway during swallowing is not facilitated by
A. The sphincteric action of the aryepiglottic muscles B. Closure of the glottis
C. Elevation of the larynx D. Contraction of the cricothyroid muscle
78. Regarding triangles of the neck, all of the following structures are boundaries of the triangle named except
A. Submental triangle, mandible B. Anterior triangle, mandible
C. Digastric triangle, mandible D. Carotid triangle, sternocleidomastoid
79. The alar ligaments connect the
A. Bodies of the axis to the foramen magnum B. Dens to the foramen magnum
C. Adjacent vertebral bodies posteriorly D. Tips of adjacent spinous processes
80. Muscles for smiling and frowning are derived from
A. First arch B. Second arch
C. Third arch D. Fourth arch
81. An example of dense regular connective tissue is
A. Deep fascia B. Dermis
C. Scar D. Tendon
82. Proprioceptive impulses from muscles of mastication are projected at
A. Mesencephalic nucleus B. Spinal nucleus
C. Nucleus gracilis D. Nucleus of tractus solitarius
83. Fibers of palatopharyngeus constitute
A. Killian’s dehiscence B. Passavant’s ridge
C. Zenker’s diverticulum D. Killian’s diverticulum

https://t.me/DentalBooksWorld
522 Triumph’s Complete Review of Dentistry

84. The little finger is supplied by which dermatome


A. C6 B. C7
C. C8 D. T1

VASCULAR SYSTEM
1. Which structure can be felt at the lower part of the medial border of sternocleidomastoid?
A. Subclavian artery B. Common carotid artery
C. Internal mammary artery D. Maxillary artery
2. Posterior superior alveolar artery is a branch of
A. Palatal branch of the maxillary artery B. Nasal branch of the maxillary artery
C. Inferior alveolar artery D. Mandibular artery
3. The bifurcation of common carotid occurs at
A. Upper border of the thyroid cartilage B. Upper border of the cricoid cartilage
C. Upper border of the cricothyroid membrane D. Level of hyoid bone
4. Superior cerebral veins drain into
A. Vein of Galen B. Great cerebral vein
C. Inferior sagittal sinus D. Superior sagittal sinus
5. All of the following statements about diploic veins are true except
A. Present in cranial bones
B. Have a thin wall lined by a single layer of endothelium
C. Develop around the 8th week of gestation
D. These have no valves (valveless)
6. Which of these is not a tributary of the cavernous sinus?
A. Sphenoparietal sinus B. Superior petrosal sinus
C. Superficial middle cerebral vein D. Inferior ophthalmic vein
7. Which structure can be felt at the lower part of the medial border of sternocleidomastoid?
A. Common carotid artery B. Subclavian artery
C. Maxillary artery D. Internal mammary artery
8. Middle meningeal artery is a branch of artery
A. ECA B. Vertebral
C. Maxillary D. Posterior cerebellar
9. All are branches of the maxillary artery except
A. Posterior ethmoidal artery B. Anterior tympanic artery
C. Infraorbital artery D. Middle meningeal artery
10. Lymph from the tonsils drains into the
A. Jugulodigastric node B. Jugulo-omohyoid node
C. Upper deep cervical node D. Submental node
11. The middle thyroid vein drains into …… vein
A. Anterior jugular B. External jugular
C. Brachiocephalic D. Internal jugular
12. The palatine tonsil receives its arterial supply from all of the following except
A. Facial B. Ascending palatine
C. Sphenopalatine D. Dorsal lingual
13. Which of the following is not a branch of cavernous segment of internal carotid artery?
A. Cavernous branch B. Inferior hypophyseal branch
C. Meningeal branch D. Ophthalmic branch
14. The only medial branch of external carotid artery is
A. Lingual B. Maxillary
C. Superior thyroid D. Ascending pharyngeal

https://t.me/DentalBooksWorld
Chapter 6 • Embryology, Head and Neck Anatomy, and General Histology 523

15. Common carotid artery divides at the level of


A. Cricoid cartilage B. Hyoid bone
C. Inferior border of the thyroid cartilage D. Superior border of the thyroid cartilage
16. The vertebral artery traverses all of the following except
A. Foramen magnum B. Subarachnoid space
C. Intervertebral foramen D. Foramen transversarium
17. The artery palpable at the anterior border of the masseter is
A. Maxillary artery B. Superficial temporal artery
C. Lingual artery D. Facial artery
18. After exposure of the left subclavian artery by supraclavicular approach, which of the following muscle does not
require cutting?
A. Scalenus anterior B. Sternocleidomastoid
C. Omohyoid D. Scalenus medius
19. Which of the following is not true about facial artery?
A. Is a branch of internal carotid artery
B. Main source of oxygenated blood to the palatine tonsil
C. Conveys postganglionic sympathetic nerve fibers
D. Supplies branches to both upper and lower lips
20. Sternocleidomastoid muscle is supplied by all of the following arteries, except
A. Posterior auricular artery B. Occipital artery
C. Superior thyroid artery D. Thyrocervical trunk artery
21. The level of branching of the common carotid artery is
A. Upper border of the cricoid B. Hyoid
C. Upper border of the thyroid cartilage D. Lower border of the cricoid
22. Sternocleidomastoid is supplied by all except
A. Superior thyroid artery B. Occipital artery
C. Thyrocervical trunk D. Postauricular artery
23. Ophthalmic artery is the branch of which part of internal carotid artery
A. Intrapetrous B. Intercavernous
C. Extracranial D. Intracerebral
24. Tip of the tongue drains into
A. Deep cervical lymph nodes B. Tonsillar lymph nodes
C. Occipital lymph node D. Submental lymph node
25. Which of the following passes through the foramen magnum?
A. XIth cranial nerve B. Internal carotid artery
C. Vertebral artery D. Sympathetic chain
26. Internal carotid artery at the bifurcation from the common carotid is
A. Post to external carotid B. Anterior to external carotid
C. Lateral to the external carotid D. Medial to external carotid
27. Endolymphatic duct drains into
A. Subarachnoid space B. Succulus
C. Subdural space D. External space
28. All of the following are branches of the external carotid artery except
A. Anterior ethmoidal artery B. Superior thyroid artery
C. Posterior auricular artery D. Occipital artery
29. The only medial branch of the external carotid artery is
A. Superior thyroid B. Ascending pharyngeal
C. Lingual D. Maxillary
30. Structure passing through the foramen spinosum is
A. Middle meningeal artery B. Accessory meningeal artery
C. Maxillary nerve D. Mandibular nerve

https://t.me/DentalBooksWorld
524 Triumph’s Complete Review of Dentistry

31. Excessive hemorrhage in tonsillectomy results from injury to


A. Venous commitants of the facial ovary B. Ascending palatine artery
C. Internal carotid artery D. Palatine vein
32. The number of branches of the internal carotid artery in the neck is
A. 3 B. None
C. 1 D. 2
33. Structures lying deep into the posterior belly of the digastric are all, except
A. Hyoglossus muscle B. Occipital artery
C. Retromandibular vein D. Hypoglossal nerve
34. Branchial sinus passes between
A. Superficial to ECA B. Posterior mediastinum
C. Two carotids D. IJV and carotid
35. Which of the following are draining channels of the cavernous sinus?
A. Sphenoparietal sinus B. Middle cerebral vein
C. Inferior petrosal sinus D. Inferior ophthalmic vein
36. The blood supply to the inner ear is derived from
A. Middle cerebellar artery B. Anterior inferior cerebellar artery
C. Superior cerebellar artery D. Posterior inferior cerebellar artery
37. Tributary of the cavernous sinus includes all of the following, except
A. Superficial middle cerebral vein B. Deep middle cerebral vein
C. Superior petrosal sinus D. Inferior petrosal sinus
38. The danger area of the face is so called because of the connection of the facial veins to the cavernous sinus through the
A. Transverse facial vein B. Superior ophthalmic vein
C. Maxillary vein D. Ethmoidal vein
39. In emergency tracheostomy, the following structures are damaged except
A. Inferior thyroid artery B. Isthmus of the thyroid
C. Inferior thyroid vein D. Thyroid ima
40. Foramen transversarium transmits
A. Sigmoid sinus B. Vertebral artery
C. Inferior jugular vein D. Inferior petrosal sinus
41. All are branches of the maxillary artery except
A. Anterior tympanic artery B. Posterior ethmoidal artery
C. Middle meningeal artery D. Infraorbital artery
42. Inferior thyroid artery arises from
A. Thyrocervical trunk of the first part of the subclavian artery
B. First part of the subclavian artery
C. Internal carotid artery
D. Third part of the subclavian artery
43. Which of the following does not supply the palate?
A. Descending palatine artery B. Ascending pharyngeal artery
C. Tonsillar branch of the facial artery D. Ascending palatine artery
44. Parathyroid glands are supplied by which artery
A. Middle thyroid B. Inferior thyroid
C. Common carotid D. Superior thyroid
45. Common carotid artery is palpated at
A. Hyoid bone B. Cricothyroid membrane
C. Upper border of the cricoid cartilage D. Upper border of the thyroid cartilage
46. All are true except
A. Inferior thyroid artery is a branch of thyrocervical trunk
B. Thyroidea ima artery is invariably a branch of arch of aorta
C. Superior thyroid is a branch of external carotid artery
D. Posterior branch of the superior thyroid artery supplies parathyroid glands
https://t.me/DentalBooksWorld
Chapter 6 • Embryology, Head and Neck Anatomy, and General Histology 525

47. Which is true of tonsils?


A. Developed from the fourth branchial cleft B. Lies on the inferior constrictor
C. Crypts lined by the stratified columnar epithelium D. Drained by jugulodigastric lymph nodes
48. Portal circulation is seen in which of the following endocrine gland
A. Hypophysis cerebri B. Pineal gland
C. Ovary D. Pancreas
49. Middle meningeal artery is a direct branch of
A. Internal maxillary artery B. External carotid artery
C. Middle cerebral artery D. Superficial temporal artery
50. The great vein of Galen drains into the
A. IJV B. FJV
C. Straight sinus D. Inferior sagittal sinus
51. The following statements are true of the transverse sinuses except
A. They drain into sigmoid sinuses
B. The right often drains primarily the superior sagittal sinus
C. The confluence of sinuses joins these with the straight sinus
D. They contain valves that help direct the blood flow
52. The following veins and/or dual venous sinuses may drain directly into the cavernous sinus except
A. Ophthalmic veins B. Superior petrosal sinus
C. Superior sagittal sinus D. Pterygoid plexus of veins
53. The retromandibular vein is formed at the junction of the maxillary vein and the
A. Facial vein B. Middle temporal vein
C. Transverse facial vein D. Superficial temporal vein
54. The internal jugular vein directly receives the following tributaries
A. Facial vein (common facial vein) B. Superficial temporal vein
C. Inferior petrosal sinus D. Superior petrosal sinus
55. Lymph drainage from the whole vestibule of the nose goes first to this group of nodes
A. Submental B. Submandibular
C. Parotid D. Retropharyngeal
56. The posterior part of the nose receives a great portion of its arterial supply from this artery
A. Infraorbital B. Facial
C. Superior labial D. Sphenopalatine
57. The external jugular vein is formed just behind the angle of the mandible by the union of the
A. Facial vein and the anterior division of the retromandibular vein
B. Facial vein and the posterior division of the retromandibular vein
C. Occipital vein and superficial temporal vein
D. Posterior auricular vein and the posterior division of the retromandibular vein
58. Lymph draining from the cervical portion of the esophagus goes first to this group of nodes
A. Pretracheal B. Paratracheal
C. Retropharyngeal D. Deep cervical
59. The external jugular vein usually begins (is formed by the union of two veins) just
A. Posterior to the neck of the mandible
B. Posterior to the angle of the mandible
C. Posterior to the middle third of the sternocleidomastoid muscle
D. Just superior to the middle third of the clavicle
60. In the root of the neck, the subclavian artery and subclavian vein are separated by the
A. Anterior scalene muscle B. Trunks of the brachial plexus
C. Middle scalene muscle D. All of the above
61. The lymphatic drainage from the medial portion of the upper lip goes first to this group of regional nodes
A. Submental B. Submandibular
C. Anterior cervical D. Jugulodigastric

https://t.me/DentalBooksWorld
526 Triumph’s Complete Review of Dentistry

62. If epistaxis (serious nose bleeds) could not be controlled by packing the nasal cavity with gauze, and if the source of
the bleeding appeared to be in the posterior part of the nasal cavity, then ligating this artery would stop the bleeding
A. Septal branch of the superior labial artery B. Nasal branch of the infraorbital artery
C. Superior nasal branch of the ophthalmic artery D. Sphenopalatine artery
63. The following statement(s) is/are true of the vessels of the nasal and palatine mucosa
A. They exhibit extensive anastomosis
B. The sphenopalatine artery runs on the nasal septum and anastomoses with the greater palatine artery in the incisive
foramen
C. The greater and lesser palatine arteries are terminal branches of the maxillary artery
D. The sphenopalatine artery passes through the inferior orbital fissure to reach the nasal septum
64. Lymph drainage from the lateral parts of the eyelids goes first to this group of lymph nodes
A. Parotid B. Buccal
C. Submandibular D. Occipital
65. The following lymph node(s) would be classified as belonging to the terminal group of nodes (deep cervical nodes)
A. Jugulodigastric B. Jugulo-omohyoid
C. Retropharyngeal D. A and B, but not C
66. On the right side of the root of the neck, the costocervical trunk usually arises from the
A. First part of the subclavian artery B. Second part of the subclavian artery
C. Third part of the subclavian artery D. Brachiocephalic artery
67. The lymphatic drainage of the larynx first goes to this group of nodes
A. Submandibular B. Submental
C. Superficial cervical D. Deep cervical
68. The thyrohyoid membrane is pierced by this artery
A. Inferior thyroid B. Superior thyroid
C. Inferior laryngeal D. Superior laryngeal
69. The lymphatic drainage of the maxillary teeth usually goes first to this group of nodes
A. Sublingual B. Parotid
C. Submandibular D. Retropharyngeal
70. This artery has branches that supply most of the blood to the anterior part of the nasal cavity
A. Sphenopalatine B. Facial artery
C. Anterior ethmoidal artery D. Descending palatine artery
71. The artery supplying to the inferior part of the larynx is a branch of the
A. Internal carotid B. Ascending pharyngeal
C. Ascending cervical D. Inferior thyroid
72. Which of the following arteries does not accompany its correspondingly named nerve throughout most of its course?
A. Infraorbital B. Greater palatine
C. Inferior alveolar D. Posterior superior alveolar
73. The following veins and/or dural venous sinuses may drain directly into the cavernous sinus except
A. Ophthalmic veins B. Superior petrosal sinus
C. Superior sagittal sinus D. Pterygoid plexus of veins
74. The following statements are true of the transverse sinuses except
A. They drain into sigmoid sinuses
B. The right often drains primarily the superior sagittal sinus
C. The confluence of sinuses joins these with the straight sinus
D. They contain valves that help direct blood flow
75. The retromandibular vein is formed by the junction of the maxillary vein and the
A. Facial vein B. Middle temporal vein
C. Transverse facial vein D. Superficial temporal vein
76. The carotid sheath and its contents may be safely retracted as a unit during surgical procedures of the neck. The
contents of the carotid sheath include all of the following structures except the
A. Common carotid artery B. Internal carotid artery
C. Internal jugular vein D. Sympathetic trunk

https://t.me/DentalBooksWorld
Chapter 6 • Embryology, Head and Neck Anatomy, and General Histology 527

77. The following are characteristics of the facial artery


A. It follows a convoluted (“zigzag” or “tortuous”) course over the body of the mandible
B. It passes just anterior to the masseter muscle
C. It is a branch of the internal carotid artery
D. Only A and B
78. The lymph drainage of the upper central incisor teeth goes first to this group of nodes
A. Submental B. Submandibular
C. Jugulodigastric D. Jugulo-omohyoid
79. The vertebral artery
A. Does not pass through the foramen transversarium of the first cervical vertebrae
B. Is found deep in the occipital triangle
C. Is a branch of the maxillary artery
D. Enters the vertebral foramen at the C6 level
80. At the C5 vertebral level, the carotid sheath encloses the
A. Common carotid artery B. Vagus nerve
C. Internal jugular vein D. A, B, and C
81. The following artery usually does not arise from the external carotid
A. Superior thyroid B. Lingual
C. Superficial temporal D. Middle meningeal
82. A branch of the external carotid artery in the neck is the
A. Superficial temporal artery B. Vertebral artery
C. Subclavian artery D. Occipital artery
83. The first lymph node group receiving lymphatic drainage from the most posterior portion of the tongue is
A. Submental B. Jugulo-omohyoid
C. Parotid D. Jugulodigastric
84. The veins that form a plexus in the spongy bone of the skull are
A. Venous lacunae B. Diploic veins
C. Meningeal veins D. Emissary veins
85. The veins and arteries of the scalp lie mainly in the
A. Aponeurotic layer B. Periosteal later
C. Loose connective tissue later (subaponeurotic) D. Subcutaneous connective tissue later
86. The blood vessel supplying most of the medial surface of the cerebrum is the
A. Anterior cerebral artery B. Middle cerebral artery
C. Posterior cerebral artery D. Middle meningeal artery
87. The sinus that directly connects the cavernous sinus to the internal jugular vein is the
A. Occipital B. Inferior sagittal
C. Inferior petrosal D. Transverse
88. The largest structure passing through the cavernous sinus is the
A. Internal carotid artery B. Internal jugular vein
C. Abducens nerve D. Trigeminal (mandibular division)
89. Which one of the following vessels is most likely to be cut when doing a tracheostomy (opening the trachea from
the front)?
A. Superior thyroid vein B. Inferior thyroid artery
C. Thyroidea ima artery D. Middle thyroid vein
90. This arterial branch of the external carotid artery originates at the level of the hyoid bone and forms a loop before
passing deep to the hyoglossus muscle to enter the submandibular region
A. Ascending pharyngeal B. Lingual
C. Facial D. Occipital
91. The common carotid artery usually divides into the external carotid and internal carotid arteries within this cervical
triangle
A. Digastric B. Occipital
C. Carotid D. Muscular
https://t.me/DentalBooksWorld
528 Triumph’s Complete Review of Dentistry

92. The superficial veins of the scalp are


A. Connected to the dural venous sinuses by valveless emissary veins
B. Found mainly in the subcutaneous connective tissue layer
C. Named according to the nerve they accompany in the region above and behind the external ear
D. Only A and B
93. Most of the lateral surface of the cerebrum is supplied by this large artery’s branches
A. Anterior cerebral B. Middle cerebral
C. Posterior cerebral D. Vertebral
94. The blood supply of Little’s area (anteroinferior part of the septum) is best described by
A. Anterior and posterior ethmoidal arteries from the ophthalmic artery
B. The sphenopalatine branch of the maxillary artery
C. Sphenopalatine artery and septal branch of the superior labial branch of the facial artery
D. All of the above
95. Regarding the basilar artery, all the below are true except
A. Gives off branches to the anterior spinal artery
B. Divides to give off both posterior cerebral arteries
C. Supplies branches to the pons
D. Formed by the union of the vertebral arteries
E. Lies on the ventral aspect of the cerebral peduncle
96. In the circle of Willis
A. The basilar artery divides into right and left posterior cerebellar arteries
B. The middle cerebral arteries branch off and supply the motor but not sensory cortex
C. The anterior communicating artery unites the middle cerebral artery to the internal cerebral artery
D. The internal carotid artery gives off the ophthalmic artery before dividing into anterior and middle cerebral arteries
97. Regarding the facial vein, all of the following are true except:
A. It drains into the internal jugular vein
B. It has no valves
C. It communicates with the cavernous sinus via the ophthalmic vein
D. It runs inferoposteriorly anterior to the facial artery
98. The internal jugular vein
A. Is surrounded by the thickest portion of the carotid sheath
B. Receives drainage from all the parathyroid glands
C. Is crossed posteriorly by the accessory nerve
D. Is crossed anteriorly by the thoracic duct
99. Venous drainage of the face
A. Is both deep and superficial
B. Empties ultimately into the internal jugular vein alone
C. Communicates indirectly with the cavernous sinus via the deep facial vein
D. Communicates directly with the cavernous sinus via the supraorbital veins
100. Which artery arises from the anterior aortic sinus?
A. Right coronary artery B. Left coronary artery
C. Brachiocephalic artery D. Subclavian artery

MISCELLANEOUS
1. Waldeyer’s lymphate chain is formed by all except
A. Tubal tonsils B. Postauricular nodes
C. Palatine tonsils D. Pharyngeal tonsils
2. Regarding the face, which is incorrect
A. There is no deep fascia on the face
B. The bulk of the orbicularis oris muscle comes from the buccinator
C. The eyelid muscles are completely supplied by CN VII
D. The parotid duct pierces the buccinator opposite the third upper molar tooth

https://t.me/DentalBooksWorld
Chapter 6 • Embryology, Head and Neck Anatomy, and General Histology 529

3. Which of the following is true about internal ear anatomy?


A. The angle between the anterior and the posterior SCC is 180°
B. Three semicircular canals here six openings into the vestibule
C. Spiral canal makes two turns
D. Vestibule is the central chamber
4. Tympanic plexus is present in
A. Medial projection of the middle ear cavity B. Tunica adventitia of the internal jugular vein
C. Petrous part of the temporal bone D. Part of the tympanic membrane
5. Lamina cribrosa is a modification of
A. Optic nerve sheath B. Retina
C. Sclera D. Choroid
6. The carotid sheath and its contents may be safely retracted as a unit during surgical procedures of the neck. The
contents of the carotid sheath include all of the following structures except
A. Common carotid artery B. Internal carotid artery
C. Internal jugular vein D. Sympathetic trunk
7. Which of the following about trachea is correct?
A. Commences below the cricoid at the level of C5
B. Enters the thoracic inlet slightly to the left
C. Is marked at its lower end by the level of the sternal angle
D. Is supplied by the glossopharyngeal nerve
8. Which of the following statements is correct?
A. The masseter muscle is innervated by a branch of V-3
B. The internal (medial) pterygoid muscle arises from the medial pterygoid plate
C. The styloglossus muscle serves in retracting the tongue
D. All muscles whose names end in -glossus are innervated by cranial nerve XII
9. Which of the following statements is correct?
A. The anterior clinoid processes are parts of the lesser wing of the sphenoid bone
B. The anterior end of the falx cerebri lies just in front of the crista galli
C. The posterior clinoid processes are connected to the tentorium cerebelli
D. Of the meningeal arteries, the middle meningeal artery supplies the greatest territory
10. Cerebrospinal fluid is found normally in the
A. Epidural space B. Subarachnoid space
C. Subdural space D. Ventricles of the vein
11. At the level of C5 or C6, the carotid sheath contains
A. The internal jugular vein B. Common carotid artery
C. Vagus nerve D. Sympathetic trunk
12. The isthmus of the thyroid gland lies directly anterior to the
A. Thyrohyoid membrane B. Lamina of the thyroid cartilage
C. Cricothyroid ligament D. second, third, and fourth tracheal rings
13. The following part(s) of the internal ear is/are filled with perilymph
A. Saccule B. Utricle
C. Cochlear duct D. Semicircular ducts
14. Which statement about the nasal cavity is true?
A. When the middle conchae is raised, a rounded elevation called the torus tubarius is visible
B. The sphenopalatine foramen permits the neurovascular structures to pass between the pterygopalatine fossa and the
nasal cavity
C. The sphenoethmoidal recess is located inferior to the superior conchae
D. The frontal recess is located inferior to the inferior meatus
15. When removing the thyroid gland the most easily damaged structure that the surgeon needs to avoid ligating or
cutting is
A. Recurrent laryngeal artery B. Inferior thyroid artery
C. Superior thyroid artery D. Vertebral artery

https://t.me/DentalBooksWorld
530 Triumph’s Complete Review of Dentistry

16. On the left side of the root of the neck, the thoracic duct turns laterally to pass between
A. Phrenic nerve and the prevertebral fascia B. Anterior and middle scalene muscles
C. Anterior primary rami of C8 and T1 D. Common carotid artery and vertebral artery
17. A large mass in the posterior portion of the tongue which is not a tumor might be the
A. Thyroid gland B. Submandibular gland
C. Sublingual gland D. Parotid gland
18. The tonsillar fossa
A. Lies posterior to the palatopharyngeal arch
B. Is also known as the piriform fossa
C. Has lymph drainage from its walls that enters the jugulodigastric node
D. Contains the pharyngeal tonsil because it is in the oral pharynx
19. The parotid duct opens into the _____opposite the _____tooth
A. Mouth proper – second lower premolar B. Vestibule of mouth – second upper molar
C. Mouth proper – first lower incisor D. Mouth proper – third lower molar
20. The following statements concerning the falx cerebri are true except
A. It arises from the crista galli of the ethmoid bone
B. It lies between the right and left cerebral hemispheres
C. It contains the superior sagittal sinus
D. It fuses with the tentorium cerebelli in the region of the inferior sagittal dural venous sinus
21. Which statement is incorrect?
A. The zygomaticofacial nerve is a branch of the trigeminal nerve
B. The chorda tympani joins the lingual nerve within the infratemporal region
C. The maxillary artery is one of the two terminal branches of the external carotid artery
D. The otic ganglion is a sympathetic ganglion
22. Which statement is correct?
A. The internal acoustic meatus has two nerves passing through it
B. The facial artery is a branch of the internal carotid artery
C. The facial artery passes superficial to the submandibular gland
D. The chorda tympani, a branch of the facial nerve, enters the temporal bone through the sphenopalatine foramen
23. Regarding the carotid sheath
A. Ansa cervicalis lies behind the IJV B. Is free to move in the neck
C. Contains the vagus and phrenic D. The sympathetic trunk lies outside the sheath
E. The CCA lies lateral to the IJV
24. Which nerve is known as “pinched nerve”?
A. Radial nerve B. Ulnar nerve
C. Median nerve D. Trigeminal nerve
25. Which artery is known as “widow maker artery”?
A. Left anterior descending B. Right coronary artery
C. Sinoatrial nodal artery D. Left marginal artery
26. Bilateral paralysis of the recurrent laryngeal nerve causes
A. Stridor B. Timbre of voice
C. Hoarseness of voice D. Dysphonia
27. The most appropriate localization of the internal carotid artery is
A. Mastoid region B. Tonsillar region
C. Pterygomandibular space D. Digastric triangle
28. Characteristic feature of Weber syndrome is
A. Paraplegia B. Bell’s palsy
C. Alternative hemiplegia D. Parkinsonism
29. Malpighian layer is also known as
A. Str. basale B. Str. corneum
C. Str. spinosum D. Str. granulosum

https://t.me/DentalBooksWorld
Chapter 6 • Embryology, Head and Neck Anatomy, and General Histology 531

30. Type of joint found in the left and right jaw point
A. Condylar B. Pivot
C. Hinge D. Ball and socket
31. Down syndrome is an example of
A. Disomy B. Monosomy
C. Aneuploidy D. Ring chromosome
32. Normal sperm count that is considered to be fertile is
A. 10 million/ml B. 15 million/ml
C. 20 million/ml D. 5 million/ml
33. Main energy substrate for sperms
A. Glucose B. Fructose
C. Sucrose D. Lactose
34. Chiasmata formation is seen in
A. Diakinesis B. Zygotene
C. Pachytene D. Leptotene

ANSWERS

OSTEOLOGY
1. Answer: D (Ref. Netter’s Head and Neck Anatomy for Dentistry, 3rd edition, By Neil S. Norton, page no. 432)
2. Answer: D (Ref. Textbook of Anatomy Head, Neck, and Brain; Volume 3, By Vishram Singh, page no. 29)
3. Answer: B (Ref. Shafer’s Textbook of Oral Pathology, 6th edition, By R. Rajendran, page no. 67)
4. Answer: C (Ref. Netter’s Clinical Anatomy, 3rd edition, By John T. Hansen, page no. 108)
5. Answer: D (Ref. Clemente’s Anatomy Dissector: Guides to Individual Dissections in Human, 3rd edition, By Carmine D.
Clemente, page no. 354)
The three unpaired cartilages of larynx are the epiglottis, thyroid, and cricoid cartilages.
The three paired cartilages of larynx are the arytenoid, corniculate, and cuneiform.
6. Answer: D (Ref. “previous question”)

Bone/cartilage Description Notes


Corniculate cartilage A small cartilage located on Corniculate cartilage is found in the base of the aryepiglottic fold; it
the apex of the arytenoid is a yellow elastic cartilage
cartilage
Arytenoid cartilage A pyramid-shaped cartilage Paired; each is connected to the epiglottis above via the aryepiglottic
located on the superior margin m. and to the thyroid cartilage anteriorly via the vocal ligament;
of the cricoid lamina paired arytenoid cartilages are pulled together (adducted) by the
arytenoid m.
Cuneiform cartilage Small cartilaginous nodule Cuneiform cartilage is a yellow elastic cartilage
located in the aryepiglottic
fold
Cricoid cartilage The inferior and posterior Connected: Above to the thyroid cartilage via the inferior horn of
cartilage of the larynx; it forms the thyroid cartilage, to the conus elasticus, and to the arytenoid
a complete cartilaginous ring; cartilages which sit atop the lamina; connected below to the first
its arch projects anteriorly and tracheal ring via the cricotracheal ligament
its lamina is broad and flat
posteriorly
Epiglottis The superior part of the larynx Epiglottic cartilage is covered by a mucous membrane

https://t.me/DentalBooksWorld
532 Triumph’s Complete Review of Dentistry

7. Answer: C (Ref. Clemente’s Anatomy Dissector: Guides to Individual Dissections in Human, 3rd edition, By Carmine D.
Clemente, page no. 359)
The cranium consists of the following bones:
• Frontal bone (1)
• Parietal bones (2)*
• Occipital bone (1)
• Temporal bones (2)*
• Sphenoid bone (1)
• Ethmoid bone (1)
*Only two paired bones in the cranium.
Facial bones consist of the following:
• Zygomatic bones (2)
• Maxillae (2)
• Nasal bones (2)
• Lacrimal bones (2)
• Vomer (1)*
• Palatine bones (2)
• Inferior conchae (2)
• Mandible (1)*
*Only two unpaired bones in the face.
8. Answer: C (Ref. Gray’s Anatomy, 41st edition: The Anatomical Basis of Clinical Practice, edited by Susan Standring, page
no. 619)
9. Answer: A (Ref. Netter’s Clinical Anatomy, 3rd edition, By John T. Hansen, page no. 134)
• This plexus communicates freely with the anterior facial vein; it also communicates with the cavernous sinus, by
branches through the foramen Vesalii, foramen ovale, and foramen lacerum.
• The pterygoid plexus of veins becomes the maxillary vein.
10. Answer: B (Ref. “previous question”)
11. Answer: D (Ref. Netter’s Clinical Anatomy, 3rd edition, By John T. Hansen, page no. 525)
12. Answer: C (Ref. Gray’s Anatomy for Students, 3rd edition, By Richard Drake, A. Wayne Vogl, Adam W. M. Mitchell, page
no. 927)
The borders and anatomical relations of the bony orbit are as follows:
• Roof (superior wall): Formed by the frontal bone and the lesser wing of the sphenoid. The frontal bone separates the
orbit from the anterior cranial fossa.
• Floor (inferior wall): Formed by the maxilla, palatine, and zygomatic bones. The maxilla separates the orbit from the
underlying maxillary sinus.
• Medial wall: Formed by the ethmoid, maxilla, lacrimal, and sphenoid bones. The ethmoid bone separates the orbit from
the ethmoid sinus.
• Lateral wall: Formed by the zygomatic bone and greater wing of the sphenoid.
• Apex: Located at the opening to the optic canal, the optic foramen.
• Base: Opens out into the face, and is bounded by the eyelids. It is also known as the orbital rim.
13. Answer: C (Ref. “previous question”)
14. Answer: A (Ref. Gray’s Anatomy for Students, 3rd edition, By Richard Drake, A. Wayne Vogl, Adam W. M. Mitchell, page
no. 856)
15. Answer: A (Ref. Gray’s Anatomy for Students, 3rd edition, By Richard Drake, A. Wayne Vogl, Adam W. M. Mitchell, page
no. 998)
16. Answer: A (Ref. Netter’s Head and Neck Anatomy for Dentistry, 3rd edition, By Neil S. Norton, page no. 127)
17. Answer: B (Ref. “previous question”)
18. Answer: A (Ref. Gray’s Anatomy, 41st edition: The Anatomical Basis of Clinical Practice, edited by Susan Standring, page
no. 556)
19. Answer: A (Ref. Netter’s Clinical Anatomy, 3rd edition, By John T. Hansen, page no. 466)
• The vomer and the perpendicular plate of the ethmoid bone.

https://t.me/DentalBooksWorld
Chapter 6 • Embryology, Head and Neck Anatomy, and General Histology 533

• The vomer contributes to the inferior portion of the nasal septum; the perpendicular plate of the ethmoid bone
contributes to the superior portion.
• The hard palate is a thin horizontal bony plate of the skull, located in the roof of the mouth and forms floor of the nasal
septum. It is formed by the palatine process of the maxilla and horizontal plate of the palatine bone, and spans the arch
formed by the upper teeth.
20. Answer: A (Ref. “previous question”)
21. Answer: C (Ref. Netter’s Essential Histology, 2nd edition, By William K. Ovalle, Patrick C. Nahirney, page no. 133)
Hyaline cartilage:
• Dominant component of extracellular
• Matrix is collagen
• Bluish-white in life
• Translucent
• Important in the formation and growth of long bones
• In adults, mainly found lining the outer wall of the respiratory system and on surfaces of bone joints where it is called
articular cartilage
• Undergoes calcification in bone formation and also as part of aging process
22. Answer: A (Ref. Netter’s Clinical Anatomy, 3rd edition, By John T. Hansen, page no. 56)
23. Answer: D (Ref. Netter’s Head and Neck Anatomy for Dentistry, 3rd edition, By Neil S. Norton, page no. 56)
24. Answer: A (Ref. Gray’s Anatomy for Students, 3rd edition, By Richard Drake, A. Wayne Vogl, Adam W. M. Mitchell, page no. 823)
Parts of the ethmoid bone: Crista galli, labyrinth process, superior and middle nasal concha, uncinate process,
perpendicular plate of the ethmoid bone.
25. Answer: A (Ref. Netter’s Clinical Anatomy, 3rd edition, By John T. Hansen, page no. 314)
26. Answer: C (Ref. Clemente’s Anatomy Dissector: Guides to Individual Dissections in Human, 3rd edition, By Carmine D.
Clemente, page no. 255)
27. Answer: A (Ref. “previous question”)
28. Answer: D (Ref. Netter’s Head and Neck Anatomy for Dentistry, 3rd edition, By Neil S. Norton, page no. 315)
There are four pairs of sinuses (named for the skull bones in which they are located):
1. Frontal sinuses: The right and left frontal sinuses are located in the center of the forehead (frontal bone) just above each
eye.
2. Maxillary sinuses: These are the largest of the sinuses and are located behind the cheekbones near the maxillae, or upper
jaws.
“Achilles Heel”—the only sinus whose opening is superior to the sinus itself, such that it cannot easily drain into the
nose due to gravity itself.
This is why, when you lie down on one side of your body with a cold, you tend to clog the “downhill” side of your nose.
3. Sphenoid sinuses: The sphenoid sinuses are located in the sphenoid bone near the optic nerve and the pituitary gland
on the side of the skull.
4. Ethmoid sinuses: The ethmoid sinuses are located in the ethmoid bone, which separates the nasal cavity from the brain.
These sinuses are not single sacs but a collection of 6–12 small air cells that open independently into the nasal cavity. They
are divided into front, middle, and rear groups.
29. Answer: D (Ref. “previous question”)
30. Answer: D (Ref. Netter’s Clinical Anatomy, 3rd edition, By John T. Hansen, page no. 442)
The order of the nerves passing through the superior orbital fissure from superior to inferior:
• Lacrimal nerve (branch of CN V1)
• Frontal nerve (branch of CN V1)
• Trochlear nerve (CN IV)
• Superior division of the oculomotor nerve (CN III)
• Nasociliary nerve (branch of CN V1)
Structures passing through the superior orbital fissure:
Mnemonic: LOT of FANs for opthalmic men
L: Lacrimal nerve
O: Oculomotor nerve
T: Trochlear nerve
F: Frontal nerve

https://t.me/DentalBooksWorld
534 Triumph’s Complete Review of Dentistry

A: Abducens nerve
N: Nasociliary nerve
OPHTHALMIC: ophthalmic veins (superior and inferior)
MEN: MENingeal branch of the lacrimal artery
Middle MENingeal anas. branch

NERVES
1. Answer: C (Ref. Atlas of Human Anatomy, 6th edition, By Frank H. Nette, page no. e-22)
All the muscles which play any role in the movement of the vocal cord are supplied by the recurrent laryngeal nerve except
the cricothyroid muscle which receives its innervation from the external laryngeal nerve—a branch of superior laryngeal
nerve.
2. Answer: C (Ref. Netter’s Clinical Anatomy, 3rd edition, By Frank H. Netter, John T. Hansen)
Ganglions in Head and Neck:

Nerve Ganglion Source Branches Motor Sensory Notes


Otic ganglion Preganglionic Postganglionic Secretomotor None A parasympathetic
parasympathetic parasympathetic to the parotid ganglion; the otic
via the lesser axons distribute gland ganglion hangs off of
petrosal, from the with the the mandibular division
tympanic n. of parotid brs. of of the trigeminal nerve.
glossopharyngeal auriculotemporal (V3) Inferomedial to the
n. (IX) nerve (from V3) foramen ovale
Ciliary ganglion Preganglionic Postganglionic Sphincter None A parasympathetic
parasympathetic parasympathetic pupillae m., ganglion; the ciliary
axons arrive axons which ciliary m. of the ganglion is located on the
via the inferior distribute via short eye lateral side of the optic n.
division of the ciliary n. near the apex of the orbit;
oculomotor n. sensory and sympathetic
(III) axons pass through the
ciliary ganglion without
synapse—the sensory
root is carried via the
nasociliary n. and the
sympathetic root arrives
in the orbit via the
internal carotid a.
Sympathetic Preganglionic Postganglionic Dilator pupillae, Pain from Located lateral to the
chain ganglia sympathetic sympathetic vascular smooth viscera vertebral bodies in
fibers arrive fibers depart muscle, arrector the neck, thorax, and
via white rami via gray rami pili muscles, abdominopelvic cavity;
communicans of communicans to sweat glands, the ganglia plus their
ventral primary all spinal nerves; suprarenal interconnecting fibers
rami of spinal internal and medulla, heart, are also known as the
nerves T1-L2 external carotid lungs, and gut sympathetic trunk;
n.; cervical cardiac preganglionic cell
branches; thoracic bodies are located in the
direct visceral intermediolateral gray
brs.; greater, lesser, matter of spinal cord
and least thoracic levels T1-L2
splanchnic nerve;
lumbar splanchnic
nn.; sacral
splanchnic nn.

https://t.me/DentalBooksWorld
Chapter 6 • Embryology, Head and Neck Anatomy, and General Histology 535

Geniculate Facial nerve (VII) Nervus None Taste (SVA) A sensory ganglion
(chorda tympani intermedius (SVA from the equivalent in histological
branch) sensory root of anterior 2/3 of structure and function
facial n.) the tongue to a dorsal root ganglion;
some taste from the
palate travels through
the greater petrosal n. to
the geniculate ganglion;
located in the facial canal
within the petrous portion
of the temporal bone
Pterygopalatine Preganglionic Postganglionic Secretomotor to None A parasympathetic
parasympathetic parasympathetic mucous glands ganglion; the
axons arrive axons distribute of the palate, pterygopalatine ganglion
via the n. of the via the greater and nasal cavity, hangs off of the maxillary
pterygoid canal lesser palatine nn., lacrimal gland division of the trigeminal
from greater nasopalatine n., n. (V2) within the
petrosal n. of the sphenopalatine n., pterygopalatine fossa;
facial n (VII); and zygomatic n. preganglionic axons of the
postganglionic greater petrosal n. synapse
sympathetic axons here; postganglionic
arrive via the n. sympathetic axons of
of the pterygoid the deep petrosal n. pass
canal from the through the pterygopalatine
deep petrosal n. ganglion without
synapsing (they synapse
in the superior cervical
sympathetic ganglion)
Semilunar Ophthalmic (V1), Sensory fibers None Skin of the A sensory ganglion
maxillary (V2), depart via the face, mucous equivalent in histological
and mandibular trigeminal n. (V) membranes of structure and function
(V3) divisions of the nasal and to a dorsal root ganglion;
the trigeminal n. oral cavities, also known as trigeminal
mucous or Gasserian ganglion
membrane of
the anterior
2/3rds of the
tongue (GSA
only)
Spiral Fibers of the Cochlear n. None Hearing (SSA) Sensory ganglion of the
cochlear hair cells portion of the cochlear part of CN VIII
vestibulocochlear
n. (VIII)
Stellate Neurons in the Gray rami Vascular smooth Pain from lungs A sympathetic ganglion;
intermediolateral communicans muscle, arrector stellate ganglion is
cell column of the to spinal nerves pili muscle, formed by the fusion
spinal cord level C8 and T1 sweat glands of the inferior cervical
T1 (postganglionic of the C8 and sympathetic ganglion and
sympathetic); T1 cutaneous the T1 ganglion of the
thoracic visceral distribution on sympathetic trunk
br. chest and upper
limb (C8 and T1
dermatomes);
vascular smooth
muscle of the
lungs

https://t.me/DentalBooksWorld
536 Triumph’s Complete Review of Dentistry

Submandibular Preganglionic Postganglionic Secretomotor None Parasympathetic ganglion;


parasympathetic parasympathetic to the submandibular ganglion
axons from the axons distribute submandibular is suspended from the
chorda tympani either directly or and sublingual lingual n. near the deep
(accompanying with branches of glands and the part of the submandibular
the lingual n. from the lingual n. to small glands gland
the mandibular the submandibular of the lingual
division of the and sublingual mucosa
trigeminal n.) glands
Pterygopalatine Parasympathetic The ganglion None Its sensory It is largely innervated
ganglion ganglion found in also consists root is derived by the greater petrosal
(Meckel’s pterygopalatine of sympathetic from two nerve (a branch of the
Ganglion/nasal fossa efferent sphenopalatine facial nerve); and its axons
(postganglionic) branches of project to the lacrimal
ganglion or
fibers from the the maxillary glands and nasal mucosa
spheno palatine
superior cervical nerve; their
ganglion)
ganglion. These fibers, for the
fibers, from the most part, pass
superior cervical directly into
ganglion, travel palatine nerves
through the
carotid plexus,
and then through
the deep petrosal
nerve

3. Answer: B (Ref. Gray’s Anatomy for students, 2nd edition, By Richard Drake, A. Wayne Vogl, Adam W. M. Mitchell, page
no. 869)

Nerve Source Branches Motor Sensory Notes


Facial n. Pons and medulla: Greater petrosal Stapedius m., Taste (SVA) from the Also known as:
Nucleus solitarius n. (preganglionic stylohyoid m., anterior 2/3rd of the CN VII, seventh
of medulla via parasympathetic posterior belly tongue; part of the cranial nerve;
nervus intermedius to pterygopalatine of digastric m., skin of the external exits the posterior
(SVA sensory root) ganglion, postganglionic muscles of facial auditory meatus cranial fossa by
from geniculate parasympathetic travels expression; passing into the
ganglion; superior with brs. of maxillary secretomotor internal acoustic
salivatory nucleus division of V), chorda to lacrimal, meatus and goes
(GVE preganglionic tympani (SVA taste from submandibular, through the facial
parasympathetic) anterior 2/3rd of the sublingual, and canal; motor to
of pons via nervus tongue; preganglionic mucous glands of muscles of facial
intermedius; facial parasympathetic to the nasal and oral expression exits
motor nucleus of the submandibular cavities the skull at the
pons via motor root ganglion, postganglionic stylomastoid
parasympathetic to the foramen
submandibular and
sublingual glands), n.
to stapedius, posterior
auricular n., intraparotid
plexus with temporal,
zygomatic, buccal,
marginal mandibular,
and cervical brs.

https://t.me/DentalBooksWorld
Chapter 6 • Embryology, Head and Neck Anatomy, and General Histology 537

4. Answer: D (Ref. Cunningham’s Textbook of Anatomy, 12th edition, page no. 764)
The internal laryngeal nerve is a branch of the vagus nerve. It forms from the division of the superior laryngeal nerve into
external and internal laryngeal branches at the level of the hyoid bone.
The internal laryngeal nerve descends within the carotid sheath posterior to the internal carotid artery and then passes
anteromedially at the level of thyrohyoid membrane. It pierces the thyrohyoid membrane to emerge within the laryngeal
part of the pharynx in the piriform recess. From the piriform recess, the nerve fibers distribute as, according to their type:
• Somatic sensory fibers from the mucosae of
–– Larynx above the vocal cords
–– Epiglottis
–– Valleculae
• Special visceral sensory fibers that carry the sensation of taste from the region of the valleculae
5. Answer: A (Ref. Essential Clinical Anesthesia, edited by Linda S. Aglio, Robert W. Lekowski, Richard D. Urman,
page no. 459)
Complications associated with a stellate ganglion block include Horner’s syndrome, intra-arterial or intravenous injection,
difficulty in swallowing, vocal cord paralysis, and epidural spread of local anesthetic and pneumothorax.
Signs of Horner’s syndrome: Drooping of the face on the side that was injected, constriction of the pupil, redness, and
watering of the eye, flushing of the face, nasal congestion, and hoarseness.
These side effects are temporary and last a few hours.
6. Answer: D (Ref. Netter’s Head and Neck Anatomy for Dentistry, By Neil S. Norton, page no. 220)
Structures passing through the foramen ovale:
• Mandibular nerve
• Motor root of the trigeminal nerve
• Accessory meningeal artery (small meningeal or paradural branch, sometimes derived from the middle
meningeal artery)
• Lesser petrosal nerve, a branch of the glossopharyngeal nerve
• An emissary vein connecting the cavernous sinus with the pterygoid plexus of veins
• Occasionally the anterior trunk of the middle meningeal vein
–– The foramen ovale is used as the entry point into the skull when conducting a percutaneous stereotactic rhizotomy,
a type of radiofrequency ablation performed to treat trigeminal neuralgia
Exits of cranial nerves from the skull.

Location Nerve
Cribriform plate Olfactory nerve (I)
Optic foramen Optic nerve (II)
Oculomotor (III)
Trochlear (IV)
Superior orbital fissure Abducens (VI)
Trigeminal V1
(ophthalmic)
Trigeminal V2
Foramen rotundum
(maxillary)
Trigeminal V3
Foramen ovale
(mandibular)
Facial (VII)
Internal auditory canal
Vestibulocochlear (VIII)
Glossopharyngeal (IX)
Jugular foramen Vagus (X)
Accessory (XI)
Hypoglossal canal Hypoglossal (XII)

7. Answer: C (Ref. Clinically Oriented Anatomy, 7th edition, By Keith L. Moore, Arthur F. Dalley, A. M. R. Agur,
page no. 1067)

https://t.me/DentalBooksWorld
538 Triumph’s Complete Review of Dentistry

Nerve Source Branches Motor Sensory Notes


Ethmoidal, Nasociliary Internal and None Mucous membrane Anterior ethmoidal
anterior n. external nasal lining the anterior n. passes from the
brs. ethmoid air cells orbit into the anterior
and the upper ethmoidal foramen,
anterior part of the passes through the
nasal cavity; skin of cribriform plate, passes
the lower half of the anteriorly on cribriform
nose plate, and then exits the
cranial cavity through
the ethmoid fissure into
the nasal cavity
Ethmoidal, Nasociliary No named None Mucous membrane Posterior ethmoidal
posterior n. branches lining the posterior n. exits the orbit by
ethmoid air cells passing through the
and sphenoid sinus posterior ethmoid
foramen
Auriculotemporal Mandibular Parotid brs., Secretomotor Skin of the anterior Two roots of the
n. division articular brs., to the parotid ear and the skin auriculotemporal n.
of the and anterior gland by carrying anterosuperior encircle the middle
trigeminal auricular brs. postganglionic to the ear; part meningeal a.
n. (V3) parasympathetic fibers of the external
from the otic ganglion; auditory meatus;
[preganglionic temporomandibular
parasympathetic joint
fibers originate in the
lesser petrosal br. of
the glossopharyngeal
n. (IX)]
Mandibular Trigeminal Meningeal SVE: Mylohyoid GSA: Skin of the Also known as: V3;
division of the ganglion; br., medial m., anterior belly lower lip and passes through the
trigeminal n. motor root pterygoid and of the digastric m.; jaw extending foramen ovale to exit
arises from lateral pterygoid tensor tympani superiorly above the middle cranial
the pons nn., masseteric m., tensor veli the level of the ear; fossa; the otic ganglion
n., anterior palatini m.; muscles mucous membrane is associated with the
and posterior of mastication of the tongue and medial side of V3 below
deep temporal (temporalis, floor of the mouth; the foramen ovale; the
nn., buccal n., masseter, medial lower teeth and auriculotemporal n.
auriculotemporal pterygoid, and lateral gingiva of the carries postganglionic
n., lingual n., pterygoid) mandibular alveolar parasympathetic axons
inferior alveolar arch to the parotid gland;
n. the submandibular
ganglion is associated
with the lingual n. near
the submandibular
gland; postganglionic
parasympathetics from
the submandibular
ganglion supply the
submandibular gland
and the sublingual
gland

https://t.me/DentalBooksWorld
Chapter 6 • Embryology, Head and Neck Anatomy, and General Histology 539

8. Answer: A (Ref. Gray’s Clinical Neuroanatomy, 2010, By Elliott L. Mancall, David G. Brock, page no. 171)
The trapezoid body is part of the auditory pathway where some of the axons coming from the cochlear nucleus decussate or
cross-over to the other side before traveling on to the superior olivary nucleus. This is believed to help with localization of
sound.
9. Answer: C (Ref. Cummings Otolaryngology, 6th edition, Head and Neck Surgery E-Book, By Paul W. Flint, Bruce H.
Haughey, K. Thomas Robbins, J. Regan Thomas, John K. Niparko, Valerie J. Lund, Marci M. Lesperance, page no. 950)

Nerve Source Branches Motor Sensory Notes


Superior laryngeal Vagus n. (X) Internal br. Cricothyroid m., Mucous membrane of External br. supplies
n. and external inferior pharyngeal the larynx above the the cricothyroid m.;
br. constrictor muscle.; vocal folds all other intrinsic
secretomotor to muscles of the larynx
mucosal glands of are supplied by the
the larynx above recurrent laryngeal
the vocal folds nerve
Inferior laryngeal Recurrent No named All intrinsic Mucous membrane of Inferior laryngeal n.
n. laryngeal br. of branches muscles of the the larynx below the is the continuation
the vagus n. (X) larynx except vocal fold of the recurrent
the cricothyroid; laryngeal n., the
those muscles are: name change occurs
thyroarytenoid, at the cricothyroid
oblique and articulation
transverse
arytenoid, posterior
and lateral
cricoarytenoid,
aryepiglottic,
thyroepiglottic,
vocalis;
secretomotor to the
mucous membrane
of the larynx below
the vocal fold
Recurrent Vagus n. (X) Esophageal Upper esophagus, Upper esophagus, Laryngeal n. loops
laryngeal nerve brs., tracheal lower pharynx, lower pharynx, posteriorly around
brs., cardiac laryngeal larynx below the the right subclavian
brs., mm. (except vocal folds, GVA a.; left recurrent
pharyngeal cricothyroid); from the heart laryngeal n. loops
brs., inferior smooth muscle posteriorly around
laryngeal n. of the trachea; the aortic arch
secretomotor to and ligamentum
mucosal glands arteriosum; the
in the upper inferior laryngeal br.
esophagus, lower supplies all intrinsic
pharynx, larynx muscles of the
below the vocal larynx EXCEPT the
fold, trachea; cricothyroid m.
cardiac muscle of
the heart (slows
heart rate and
decreases force of
contraction)

https://t.me/DentalBooksWorld
540 Triumph’s Complete Review of Dentistry

10. Answer: C (Ref. Netter’s Head and Neck Anatomy for Dentistry, 3rd edition, page no. 18)

Nerve Source Branches Motor Sensory Notes


Glossopharyngeal Medulla: Tympanic nerve SVE: GVA: Carotid Also known as:
n. Spinal trigeminal to the tympanic Stylopharyngeus; body, carotid CN IX, ninth
nucleus from plexus and lesser GVE: sinus, pharynx, cranial nerve; the
the superior petrosal n., secretomotor to middle ear; glossopharyngeal
ganglion (GVA); carotid sinus n., the parotid gland GSA: skin of the n. exits the
nucleus solitarius stylopharyngeus (preganglionic external ear; SVA: posterior cranial
from the inferior brs., pharyngeal parasympathetic Taste from the fossa by passing
ganglion brs via the tympanic posterior 1/3rd of through the
(SVA); nucleus n. to the lesser the tongue jugular foramen; it
ambiguus (SVE); petrosal n. to the may penetrate the
inferior salivatory otic ganglion; stylopharyngeus
nucleus postganglionic m.
(GVE - parasympathetic
preganglionic via the
parasympathetic) auriculotemporal n.)

11. Answer: A (Ref. Gray’s Anatomy for students, 2nd edition, By Richard Drake, A. Wayne Vogl, Adam W. M. Mitchell,
page no. 1029)

Nerve Source Branches Motor Sensory Notes


Ophthalmic Trigeminal Meningeal None (GSA) Skin of Also known as V1; the
division of the ganglion br., lacrimal the forehead, ophthalmic division
trigeminal n. n., frontal n., upper eyelid and of the trigeminal n.
nasociliary n. nose; mucous passes through the
membrane of the superior orbital fissure
upper nasal cavity, to exit the middle cranial
frontal sinus, fossa; the lacrimal n.
ethmoid air cells, receives postganglionic
and sphenoid parasympathetic axons to
sinuses the lacrimal gland from
the zygomaticotemporal
br. of the zygomatic n.

12. Answer: D (Ref. Clinical Anatomy: An Illustrated Review with Questions and Explanations, By Richard S. Snell, 4th edition,
page no. 240)
13. Answer: A (Ref. Gray’s Basic Anatomy, International edition, By Richard Drake, Richard Lee Drake, Wayne Vogl, Adam W.
M. Mitchell, page no. 504)
14. Answer: C (Ref. Textbook of Ophthalmology, 6th edition, By H. V. Nema, Nitin Nema, page no. 487)
The eye will be displaced downward, because the superior oblique (innervated by the fourth cranial or trochlear nerve) is
unantagonized by the paralyzed superior rectus, inferior rectus, and inferior oblique. The affected individual will also have
a ptosis, or drooping of the eyelid, and mydriasis (pupil dilation).
15. Answer: A (Ref. Netter’s Head and Neck Anatomy for Dentistry, 2nd edition, By Neil S. Norton, page no. 346)

Nerve Source Branches Motor Sensory Notes


Hypoglossal n. Medulla: No named branches; Intrinsic and None Also known as: CN XII, 12th
Hypoglossal branches of the ventral extrinsic cranial nerve; the hypoglossal n.
nucleus primary ramus of spinal muscles of exits the posterior cranial fossa by
nerve C1 are carried by the tongue passing through the hypoglossal
this nerve and are not (except the canal; the superior root of the
considered to be branches palatoglossus ansa cervicalis travels with the
of the hypoglossal nerve m.) hypoglossal n. for a short distance

https://t.me/DentalBooksWorld
Chapter 6 • Embryology, Head and Neck Anatomy, and General Histology 541

16. Answer: D (Ref. McMinn’s Color Atlas of Head and Neck Anatomy, 5th edition, By Bari M. Logan, Patricia Reynolds, Scott
Rice, Ralph T. Hutchings, page no. 149)

Nerve Source Branches Motor Sensory Notes


Trochlear n. Trochlear No named Superior oblique m. None Also known as CN IV, the fourth
nucleus branches of the eye (GSE) cranial nerve; the trochlear n.
of the passes through the superior
midbrain orbital fissure to exit the middle
cranial fossa; it is the smallest
cranial nerve and the only cranial
nerve to arise from the dorsum of
the brainstem

Famous mnemonic: SO4, superior oblique is supplied by the fourth cranial nerve.
LR6: Lateral rectus is supplied by sixth cranial nerve.
17. Answer: B (Ref. Nerves and Nerve Injuries: Volume 2: Pain, Treatment, Injury, Disease and further Directions, edited by R.
Shane Tubbs, Elias Rizk, Mohammadali M. Shoja, Marios Loukas, Nicholas Barbaro, Robert J. Spinner, page no. 497)

Nerve Source Branches Motor Sensory Notes


Great auricular n. Cervical plexus Mastoid n., None Skin of the ear The great auricular n. crosses
(contributions from auricular n. and skin below the superficial surface of the
the ventral primary the ear sternocleidomastoid m.
rami of spinal
nerves C2 and C3)
Lesser occipital n. Ventral primary No named None Skin behind the Lesser occipital n. arises from
ramus of spinal branches ear the cervical plexus
nerve C2

18. Answer: C (Ref. Gray’s Clinical Neuroanatomy: The Anatomic Basis for Clinical Neuroscience, By Elliott L. Mancall, David
G. Brock, 2010, page no. 155)
19. Answer: D (Ref. Gray’s Anatomy for Students, 2nd edition, By Richard Drake, A. Wayne Vogl, Adam W. M. Mitchell, page
no. 1006)
20. Answer: A (Ref. Salivary Gland Disorders, edited by Eugene N. Myers, Robert L. Ferris, page no. 4)
21. Answer: D (Ref. Gray’s Clinical Neuroanatomy E-Book, By Elliott L. Mancall, David G. Brock, page no. 201)
The nerves supplying the palatine tonsils come from the maxillary division of the trigeminal nerve via the lesser palatine
nerves, and from the tonsillar branches of the  glossopharyngeal nerve. The glossopharyngeal nerve continues past the
palatine tonsil and innervates the posterior 1/3rd of the tongue to provide general and taste sensation. This nerve is most
likely to be damaged during a tonsillectomy, which leads to reduced or lost general sensation and taste sensation to the
posterior third of the tongue.
22. Answer: D (Ref. Netter’s Cranial Nerve Collection, 3rd edition, By Frank H. Netter, page no. 102)

Nerve Source Branches Motor Sensory Notes


Accessory Cranial root: No named GSE: None Also known as CN XI, the 11th cranial nerve;
Medulla branches Sternocleidomastoid spinal root enters cranial cavity by passing
- nucleus and trapezius mm. through the foramen magnum; exits the skull
ambiguus; by passing through the jugular foramen;
spinal root: accessory n. is motor only; the subtrapezial
spinal nucleus plexus of nerves receives proprioceptive
of the upper fibers: for the sternocleidomastoid m. from
cervical spinal the ventral primary rami of spinal nn. C2 and
cord C3—for trapezius via ventral primary rami of
C3 and C4

https://t.me/DentalBooksWorld
542 Triumph’s Complete Review of Dentistry

Spinal nerve injury:


Symptoms:
• Shoulder pain (the most common presenting symptom)
• Pain can radiate to the neck and upper back and occasionally to the ipsilateral arm.
• Pain may worsen when the weight of the involved shoulder is not supported, placing strain upon the shoulder joint.
• The etiology of pain is multifactorial and can include the straining of supporting muscles (rhomboids and levator
scapulae) with subsequent traction on the brachial plexus.
• Diminished strength when performing regular daily activities (e.g., placing dishes in overhead shelves and exercises that
involve bearing weight on the shoulders).
Signs:
• Limited or loss of sustained abduction of the shoulder is the most common sign.
• A full passive range of motion may eventually progress to decreased passive range of motion due to adhesive capsulitis
(frozen shoulder).
• The ipsilateral shoulder may droop.
• Scapular winging or prominence of the medial border of the scapula and protraction may be found.
• Internal rotation of the humeral head may be found.
• Atrophy of the trapezius muscle may be found.
• Sternoclavicular joint hypertrophy or subluxation may be caused by abnormal stress on the medial clavicular head after
the loss of the trapezius muscle support.
23. Answer: C (Ref. Caplan’s Stroke, 4th edition: A Clinical Approach, By Louis Caplan, page no. 561)

Nerve Source Branches Motor Sensory Notes


Optic n. Ganglion layer None None Vision (SSA) Also known as CN II, the second
of the retina to cranial nerve; the course of the
the forebrain optic nerve is through the optic
canal to the optic chiasma, then the
optic tract to the lateral geniculate
body and optic radiation

Structures within the lateral wall of the cavernous sinus:


• Oculomotor nerve
• Ophthalmic nerve
• Trochlear nerve
• Maxillary nerve
• Trigeminal ganglion
Structures passing through the medial portion of the cavernous sinus:
• Abducent nerve
• Internal carotid artery accompanied by the internal carotid plexus
24. Answer: D (Ref. Textbook of Anatomy: Volume 3, 5th edition: Head and Neck, Central Nervous System, By Inderbir Singh,
page no. 804)

Nerve Source Branches Motor Sensory Notes


Abducent Pons: No named GSE: Lateral None CN VI, the sixth cranial nerve;
Abducens branches rectus m. passes through the superior orbital
nucleus fissure

25. Answer: C (Ref. Gray’s Basic Anatomy, 2nd edition, By Richard Drake, A. Wayne Vogl, Adam W. M. Mitchell, page no. 542)
The superior ganglion
It is very small, and is usually regarded as a detached portion of the petrous ganglion.
The inferior ganglion of the glossopharyngeal nerve (petrous ganglion) is larger than the superior ganglion and is situated
in a depression in the lower border of the petrous portion of the temporal bone which is named fossula petrosa.
• It also contains the neurons that innervate the carotid sinus baroreceptors.

https://t.me/DentalBooksWorld
Chapter 6 • Embryology, Head and Neck Anatomy, and General Histology 543

The middle cervical ganglion is the smallest of the three cervical ganglia, and is occasionally absent.
• It is placed opposite the sixth cervical vertebra, usually in front of, or close to, the inferior thyroid artery.
• It sends gray rami communicantes to the fifth and sixth cervical nerves, and gives off the middle cardiac nerve.
• It is probably formed by the coalescence of two ganglia corresponding to the fifth and sixth cervical nerves.
The stellate ganglion (or cervicothoracic ganglion) is a sympathetic ganglion formed by the fusion of the inferior cervical
ganglion and the first thoracic ganglion.
• Sometimes the second and the third thoracic ganglia are included in this fusion.
• Stellate ganglion is relatively big compared to much smaller thoracic, lumbar, and sacral ganglia and it is polygonal in
shape (lat. stellatum meaning star-shaped).
• Stellate ganglion is located at the level of C7, anterior to the transverse process of C7 and the neck of the first rib, superior
to the cervical pleura, and just below the subclavian artery.
26. Answer: A (Ref. Netter’s Cranial Nerve Collection E-Book, By Frank H. Netter, page no. 3)
27. Answer: B (Ref. Cummings Otolaryngology, 5th edition: Head and Neck Surgery E-Book: Head and Neck, By Paul W. Flint,
Bruce H. Haughey, John K. Niparko, Mark A. Richardson, Valerie J. Lund, K. Thomas Robbins, Marci M. Lesperance, J.
Regan Thomas, page no. 2387)
Only orbicularis oris is supplied by the facial nerve.
28. Answer: D (Ref. Gray’s Basic Anatomy, 2nd edition, By Richard Drake, A. Wayne Vogl, Adam W. M. Mitchell, page no. 439)
29. Answer: D (Ref. Netter’s Head and Neck Anatomy for Dentistry, 2nd edition, By Neil S. Norton, page no. 86)
The  pterygopalatine ganglion  (Meckel’s  ganglion, nasal  ganglion,  or sphenopalatine  ganglion) is a
parasympathetic ganglion found in the pterygopalatine fossa. It is largely innervated by the greater petrosal nerve (a branch
of the facial nerve); and its axons project to the lacrimal glands and nasal mucosa.
30. Answer: D (Ref. Netter’s Cranial Nerve Collection, By Frank H. Netter, page no. 30)
Intrapetrous collateral branches
Within the facial canal, the facial nerve gives rise to five intrapetrous branches
• The greater petrosal nerve
• The communicating branch with the lesser petrosal nerve
• The nerve to stapedius (stirrup muscle)
• The chorda tympani nerve
• The auricular branch (vagal anastomosis)

Nerve Source Branches Motor Sensory Notes


Greater petrosal n. Facial (VII) No named Secretomotor None Greater petrosal n. joins the
branches (preganglionic deep petrosal n. to form the n. of
parasympathetic) the pterygoid canal; the greater
to: lacrimal gland, petrosal n. contains preganglionic
mucous glands of the parasympathetic axons bound for
lower nasal cavity, the pterygopalatine ganglion where
maxillary sinus, and they will synapse; it passes through
palate the hiatus of the canal of the greater
petrosal n. in the petrous part of the
temporal bone

31. Answer: C (Ref. Cummings Otolaryngology—Head and Neck Surgery, 5th edition, First Volume, By Paul W. Flint, Bruce
H. Haughey, John K. Niparko, Mark A. Richardson, Valerie J. Lund, K. Thomas Robbins, Marci M. Lesperance, J. Regan
Thomas, page no. 2581)
32. Answer: A (Ref. Netter’s Cranial Nerve Collection, By Frank H. Netter, page no. 22)
33. Answer: D (Ref. Gray’s Anatomy Review, By Marios Loukas, R. Shane Tubbs, Peter H. Abrahams, Stephen W. Carmichael,
Gene L. Colborn, page no. 249)
Cranial Nerve Injury
• Olfactory nerve (cranial nerve I)
–– Anosmia (loss of the sense of smell) and hyposmia (a decreased sense of smell)
–– Parosmia (a perversion of the sense of smell), or cacosmia

https://t.me/DentalBooksWorld
544 Triumph’s Complete Review of Dentistry

• Optic nerve (cranial nerve II)


–– Immediate monocular blindness (partial or complete)
–– Visual field deficits, blurring, scotomata, and monocular diplopia
• Oculomotor nerve (cranial nerve III)
–– Oculomotor nerve palsy, including isolated and bilateral oculomotor nerve palsies
–– Signs:
■■ Outward and downward deviation of the eye
■■ Ptosis of the eyelid
■■ Dilation of the ipsilateral pupil in complete palsy
(Parinaud syndrome—paralysis of upward gaze—is caused by an injury to the dorsal midbrain and not the peripheral
oculomotor nerves)
• Trochlear nerve (cranial nerve IV)
–– Vertical diplopia on looking downward which improves with contralateral head tilt and worsens with ipsilateral
head tilt
• Abducent nerve (cranial nerve VI)
–– Complete injury—affected eye is turned medially
–– Incomplete injury—affected eye is seen at midline at rest, but the patient cannot deviate the eye laterally.
–– Isolated sixth-nerve palsy, which can be either unilateral or bilateral, can resolve spontaneously
Combined injuries of the III, IV, and/or V nerves are common and can result in the loss of depth perception and reading
and visual scanning problems.
Trigeminal nerve (cranial nerve V)
Corneal drying, abrasions, and/or pain, decreased salivation, and, especially, anesthesia of the forehead, eyebrow, and/or
nose.
Facial nerve (cranial nerve VII)
• Complete or partial paralysis of the face, hyperacusis, and/or an unusual or impaired sense of taste
• The “disinhibition syndrome,” in which there is an increase in cochlear amplifier gain, can occur subsequent to head
injury
Vestibulocochlear nerve (cranial nerve VIII)
Positional vertigo is the most common problem, although tinnitus, hearing loss, and deafness can also occur
Glossopharyngeal nerve (cranial nerve IX)
Vagus nerve (cranial nerve X)
Spinal accessory nerve (cranial nerve XI)
Causes: Injury to any or all of these nerves usually causes weakness on the ipsilateral side and other problems
• Dysphagia and dysarthria are caused by injury to the nuclei of the glossopharyngeal and vagus nerves. Symptoms
of neurogenic dysphagia include drooling, difficulty initiating swallowing, nasal regurgitation, difficulty managing
secretions, choke/cough episodes while feeding, and food sticking in the throat.
• Aphonia or weak/hoarse voice is caused by an injury to the vagus nerve.
• Locked-in syndrome (severely decreased bowel sounds, intact response to suppository, and elevated but unchanging
pulse) can result from injury to the nucleus of the vagus nerve and nerve tract.
Hypoglossal nerve (cranial nerve XII): Weakness or wasting (atrophy) of the tongue on the affected side, dysarthria, and
swallowing difficulties
34. Answer: C (Ref. Netter’s Cranial Nerve Collection, By Frank H. Netter, page no. 42)
35. Answer: D (Ref. Clinical Anatomy by Regions, By Richard S. Snell, 8th edition, page no. 749)
36. Answer: B (Ref. Gray’s Anatomy for Students, 2nd edition, By Richard Drake, A. Wayne Vogl, Adam W. M. Mitchell, page
no. 935)

Nerve Source Branches Motor Sensory Notes


Chorda tympani Facial No named Secretomotor to the Taste to the anterior Chorda tympani joins
(VII) branches submandibular and sublingual 2/3rd of the tongue the lingual nerve in the
glands (it carries preganglionic infratemporal fossa and
parasympathetic axons to the continues with it to the
submandibular ganglion) tongue

https://t.me/DentalBooksWorld
Chapter 6 • Embryology, Head and Neck Anatomy, and General Histology 545

37. Answer: B (Ref. Atlas of Human Anatomy, 6th edition, By Frank H. Netter, page no. 8)
38. Answer: B (Ref. Netter’s Clinical Anatomy, 3rd edition, By John T. Hansen, page no. 486)
39. Answer: A (Ref. Netter’s Head and Neck Anatomy for Dentistry, 3rd edition, By Neil S. Norton, page no. 505)
40. Answer: D (Ref. Netter’s Head and Neck Anatomy for Dentistry, 2nd edition, By Neil S. Norton, page no. 505)
41. Answer: B (Ref. Gray’s Basic Anatomy, By Richard Drake, A. Wayne Vogl, Adam W. M. Mitchell, page no. 470)

Nerve Source Branches Motor Sensory Notes


Zygomatic Maxillary Zygomaticofacial Carries Skin of the Zygomatic n. carries
division of the and secretomotor face lateral and postganglionic
trigeminal (V2) zygomaticotemporal fibers superior to the parasympathetic fibers
orbit from the pterygopalatine
ganglion to the
lacrimal gland (via the
communicating br. of the
zygomaticotemporal n.
which joins the lacrimal n.)
Zygomaticofacial Zygomatic No named branches None Skin of face Zygomaticofacial n.
from the lateral to the passes through the
maxillary orbit zygomaticofacial foramen
division of the
trigeminal n.
(V2)
Zygomaticotemporal Zygomatic Communicating br. Carries Skin of face Zygomaticotemporal n.
from the secretomotor superolateral carries postganglionic
maxillary fibers to the orbit parasympathetic fibers
division of the from the pterygopalatine
trigeminal n. ganglion to the
(V2) lacrimal gland (via the
communicating br. which
joins the lacrimal n.)

42. Answer: D (Ref. Netter’s Cranial Nerve Collection, By Frank H. Netter, page no. 44)
• Auricular branch (also known as the mastoid branch) of the vagus also known as Arnold’s nerve
• Named after Friedrich Arnold, German anatomist.
Also known as the Alderman’s nerve on the belief that stimulating the external auditory canal will stimulate gastric
emptying; the Aldermen who ate too much for lunch would wriggle their fingers in the external canal to relieve their
epigastric discomfort.
43. Answer: C (Ref. Netter’s Head and Neck Anatomy for Dentistry, 2nd edition, By Neil S. Norton, page no. 97)
44. Answer: B (Ref. Gray’s Anatomy, 41st edition: The Anatomical Basis of Clinical Practice, edited by Susan Standring, page
no. 632)
45. Answer: D (Ref. Gray’s Basic Anatomy, By Richard Drake, A. Wayne Vogl, Adam W. M. Mitchell, page no. 579)
46. Answer: A (Ref. Clinically Oriented Anatomy, 7th edition, By Keith L. Moore, Arthur F. Dalley, A. M. R. Agur, page no.
1064)
47. Answer: D (Ref. Anesthesiology and Otolaryngology, edited by Adam I. Levine, Satish Govindaraj, Samuel DeMaria, Jr.,
page no. 24)
• Meckel’s cave, also known as trigeminal cave or Meckel’s cavity or cavum trigeminale, is an arachnoidal pouch and is a
cerebrospinal fluid-containing arachnoidal pouch protruding from the posterior cranial fossa and houses the trigeminal
ganglion.
• The trigeminal cave is formed by two layers of  dura mater  which are part of an  evagination  of the  tentorium
cerebelli near the apex of the petrous part of the temporal bone. It envelops the trigeminal ganglion.
It is bounded by the dura overlying four structures:
• The cerebellar tentorium superolaterally
• The lateral wall of the cavernous sinus superomedially

https://t.me/DentalBooksWorld
546 Triumph’s Complete Review of Dentistry

• The clivus medially
• The posterior petrous face inferolaterally
48. Answer: A (Ref. Textbook of Anatomy: Volume 2, 5th edition: Thorax, Abdomen and Pelvis, By Inderbir Singh, page no. 458)
49. Answer: A (Ref. Gray’s Anatomy for Students, 3rd edition, Richard Drake, A. Wayne Vogl, Adam W. M. Mitchell, page no.
997)
50. Answer: D (Ref. Anatomy for Dental Students, 4th edition, By Martin E. Atkinson, page no. 252)
51. Answer: B (Ref. Gray’s Anatomy, 41st edition: The Anatomical Basis of Clinical Practice, edited by Susan Standring)
52. Answer: D (Ref. Clinically Oriented Anatomy, 7th edition, By Keith L. Moore, Arthur F. Dalley, A. M. R. Agur, page no.
1039)
53. Answer: C (Ref. Nerves and Nerve Injuries: Volume 1: History, Embryology, Anatomy, Imaging, and Diagnostics, page no.
184; By R. Shane Tubbs, Elias Rizk, Mohammadali M. Shoja, Marios Loukas, Nicholas Barbaro, Robert J. Spinner Academic
Press, 20-Apr-2015)
54. Answer: B (Ref. Netter’s Neurology, 2nd edition, By H. Royden Jones, Jr., Jayashri Srinivasan, Gregory J. Allam, Richard A.
Baker, page no. 84)
Dorello’s canal is the bow-shaped bony enclosure surrounding the abducens nerve and the inferior petrosal sinus as the
two structures merge with the cavernous sinus. It is sometimes found at the tip of the temporal bone.
This canal is named after the famous Italian anatomist Primo Dorello, who proved the existence of this canal after a series of
meticulous dissections.
55. Answer: D (Ref. Clinically Oriented Anatomy, 7th edition, by Keith L. Moore, Arthur F. Dalley, A. M. R. Agur, page no. 970)
The  Eustachian tube,  also known as the  auditory tube  or  pharyngotympanic tube, links the  nasopharynx  to the
middle ear.
Functions:
1. Pressure equalization
2. Mucus drainage
56. Answer: D (Ref. Netter’s Head and Neck Anatomy for Dentistry, 2nd edition, By Neil S. Norton, page no. 525)
57. Answer: D (Ref. Gray’s Anatomy for Students, 3rd edition, By Richard Drake, A. Wayne Vogl, Adam W. M. Mitchell, page
no.931)
The orbitalis muscle is a vestigial or rudimentary nonstriated muscle (smooth muscle) that crosses from the infraorbital
groove and sphenomaxillary fissure and is intimately united with the periosteum of the orbit and is often called Müller’s
muscle.
It lies at the back of the orbit and spans the infraorbital fissure.
It is a thin layer of the smooth muscle that bridges the inferior orbital fissure.
• Horner’s syndrome causes paralysis of the structures of the eye and orbit that receive sympathetic innervation. The signs
of Horner’s syndrome are ptosis, miosis, anhydrosis, and (apparent) enophthalmos. It is supplied by sympathetic nerves.
58. Answer: D (Ref. Gray’s Anatomy E-Book: The Anatomical Basis of Clinical Practice, 41st edition, edited by Susan Standring,
page no. 599)
59. Answer: B (Ref. Gray’s Basic Anatomy, 2nd edition, By Richard Drake, A. Wayne Vogl, Adam W. M. Mitchell, page no. 489)
60. Answer: B (Ref. Handbook of Local Anesthesia, 6th edition, By Stanley F. Malamed, page no. 226)
61. Answer: D (Ref. A Spine Surgery 2-Vol Set: Techniques, Complication Avoidance, and Management, by Edward C. Benzel,
page no. 738)

Nerve Source Branches Motor Sensory Notes


C5 ventral C5 spinal n. Contributes to: Respiratory Skin of the Joins the ventral primary ramus
primary ramus phrenic n., long diaphragm, ventral arm and of C6 to form the superior
thoracic n. scalene mm., ventral forearm trunk of the brachial plexus
muscles of the
shoulder and
upper arm
C6 ventral C6 spinal n. Contributes to: Scalene mm., Skin of the Joins the ventral primary ramus
primary ramus long thoracic n. muscles of the lateral side of of C5 to form the superior
lower shoulder the upper limb trunk of the brachial plexus
and arm

https://t.me/DentalBooksWorld
Chapter 6 • Embryology, Head and Neck Anatomy, and General Histology 547

C7 ventral C7 spinal n. Contributes to: Muscles of the Skin of the Continues as the middle trunk
primary ramus long thoracic n. lower shoulder, posterior side of of the brachial plexus
arm forearm the upper limb

62. Answer: B (Ref. Gray’s Anatomy: The Anatomical Basis of Clinical Practice, 41st edition, edited by Susan Standring,
page no. 449)

Muscle Origin Insertion Action Innervation Artery Note


Digastric Anterior belly: Body of the Elevates the Anterior belly: Anterior The digastric m.
digastric fossa hyoid via hyoid bone; Mylohyoid belly: forms two sides of
of the mandible; a fibrous depresses the nerve, from Submental the submandibular
posterior belly: loop over an mandible the mandibular a.; posterior triangle; it is formed
mastoid notch intermediate division of the belly: from mesenchyme
of the temporal tendon trigeminal nerve Occipital a. derived from the
bone (V)Posterior first two pharyngeal
belly: Facial arches, hence its dual
nerve (VII) innervation

63. Answer: B (Ref. Gray’s Anatomy for Students, 3rd edition, By Richard Drake, A. Wayne Vogl, Adam W. M. Mitchell,
page no. 1008)

Muscle Origin Insertion Action Innervation Artery Note


Mylohyoid Mylohyoid Midline raphe Elevates the Mylohyoid nerve Mylohyoid The nerve to the
line of the and body of the hyoid bone and from the inferior branch of mylohyoid also
mandible hyoid bone the tongue; alveolar nerve, the inferior innervates the anterior
depresses the a branch of the alveolar a. belly of digastric m.;
mandible mandibular both muscles are
division of the derivatives of the
trigeminal nerve second pharyngeal
(V) arch

64. Answer: C (Ref. Textbook of Anatomy: Volume 3, 5th edition: Head and Neck, Central Nervous System, By Inderbir Singh,
page no. 735)
Structures passing through the foramen magnum:
• Accessory nerves (spinal roots)
• Meningeal lymphatics
• Spinal cord
• Spinal meninges
• Sympathetic plexus of vertebral arteries
• Vertebral arteries
• Vertebral artery spinal branches
65. Answer: A
Trochlear nerve—Only and the smallest nerve arises from the dorsum of the brain stem. It passes through the superior
orbital fissure to exit the middle cranial fossa and supplies superior oblique muscle of the eye. (GSE)
66. Answer: C (Ref. Netter’s Cranial Nerve Collection, By Frank H. Netter)
Lateral rectus muscle is supplied by the abducens nerve (VI)
67. Answer: B (Ref. Netter’s Head and Neck Anatomy for Dentistry, 3rd edition, By Neil S. Norton, page no. 355)
The sublingual gland
• Lies on the lingual aspect of the body of the mandible, deep to the  plica  sublingualis (sublingual fold), which is the
posterolateral continuation of the lingual frenulum.
• It has a row of 15 or 16 (“middle-teens”) ducts that empty into the floor of the mouth on the plica sublingualis.

https://t.me/DentalBooksWorld
548 Triumph’s Complete Review of Dentistry

• The duct of the submandibular gland and the lingual nerve lie on the medial surface of the sublingual gland.
• The mylohyoid muscle lies inferior to the sublingual gland.
• The sublingual gland is innervated by postganglionic parasympathetic fibers reaching the gland via its sensory nerve,
the lingual nerve (V3).
• Preganglionic parasympathetic fibers run with the chorda tympani (VII) synapsing in the submandibular ganglion
68. Answer: C (Ref. Netter’s Cranial Nerve Collection E-Book, By Frank H. Netter, page no. 3)
O – Otic ganglion (does not pass through it, but lies inferior to it)
V – V3 cranial nerve
A – Accessory meningeal artery
L – Lesser petrosal nerve
E – Emissary vein
69. Answer: D (Ref. Diseases of Ear, Nose and Throat, By Mohan Bansal, 2012, page no. 136)
70. Answer: D (Ref. Gray’s Anatomy, 41st edition: The Anatomical Basis of Clinical Practice, edited by Susan Standring,
page no. 579)
Tensor veli palatini is innervated by the medial pterygoid nerve, a branch of mandibular nerve, the third branch of the
trigeminal nerve (CN V3) and is the only muscle of the palate not innervated by the pharyngeal plexus, which is formed by
the vagal and glossopharyngeal nerves.
71. Answer: C (Ref. Gray’s Basic Anatomy, 2012, By Richard Drake, A. Wayne Vogl, Adam W. M. Mitchell,
page no. 470)
The order of the nerves passing through the superior orbital fissure from superior to inferior:
• Lacrimal nerve (branch of CN V1)
• Frontal nerve (branch of CN V1)
• Trochlear nerve (CN IV)
• Superior division of the oculomotor nerve (CN III)
• Nasociliary nerve (branch of CN V1)
• Inferior division of the oculomotor nerve (CN III)
• Abducens nerve (CN VI)
72. Answer: C (Ref. Lippincott’s Concise Illustrated Anatomy: Head & Neck, Vol. 3, By Ben Pansky, Thomas R. Gest,
page no. 334)
Stylopharyngeus is the only muscle innervated by IX, is the only muscle of the pharyngeal wall NOT innervated by the
vagus (X) nerve; it is a derivative of the third pharyngeal arch.
73. Answer: D (Ref. Textbook of Anatomy: Volume 3: Head and Neck, Central Nervous System, By Inderbir Singh, 5th edition,
page no. 996)
The nerves supplying the palatine tonsils come from the maxillary division of the trigeminal nerve via the lesser palatine
nerves and from the tonsillar branches of the glossopharyngeal nerve.
74. Answer: C (Ref. McMinn’s Color Atlas of Head and Neck Anatomy, 5th edition, By Bari M. Logan, Patricia Reynolds, Scott
Rice, Ralph T. Hutchings)

Nerve Source Branches Motor Sensory Notes


Hypoglossal Medulla: No named branches; Intrinsic and None Also known as: CN XII, 12th
n. Hypoglossal branches of the extrinsic muscles cranial nerve; the hypoglossal
nucleus ventral primary of the tongue n. exits the posterior cranial
ramus of spinal (except the fossa by passing through
nerve C1 are carried palatoglossus m.) the hypoglossal canal; the
by this nerve and are superior root of the ansa
not considered to cervicalis travels with the
be branches of the hypoglossal n. for a short
hypoglossal nerve distance

https://t.me/DentalBooksWorld
Chapter 6 • Embryology, Head and Neck Anatomy, and General Histology 549

75. Answer: B (Ref. Gray’s Anatomy: The Anatomical Basis of Clinical Practice, 41st edition, edited by Susan Standring,
page no. 781)

Nerve Source Branches Motor Sensory Notes


C4 ventral C4 spinal Contributes to: Longus colli, Skin of the C4 contributes to the cervical
primary ramus n. supraclavicular scalenus medius, root of the plexus
nn., phrenic n.; levator scapulae, neck and the
brs. to: longus colli, respiratory upper shoulder;
scalenus medius, diaphragm proprioception
levator scapulae, from the
trapezius trapezius m.
C8 ventral C8 spinal No named branches Muscles of the Skin of the Joins the ventral primary ramus
primary ramus n. forearm and medial side of of T1 to form the inferior trunk
hand the upper limb of the brachial plexus

76. Answer: C (Ref. Gray’s Anatomy Review, 2nd edition, By Marios Loukas, R. Shane Tubbs, Peter H. Abrahams, Stephen W.
Carmichael, page no. 448)
77. Answer: A (Ref. Gray’s Anatomy: The Anatomical Basis of Clinical Practice, 41st edition, edited by Susan Standring, page no. 628)
78. Answer: A (Ref. Netter’s Head and Neck Anatomy for Dentistry, 3rd edition, By Neil S. Norton, page no. 89)
Nasal innervation
• The sphenopalatine ganglion (V2) is located at the posterior end of the middle turbinate and  innervates  the
posterior nasal cavity.
• The anterior and posterior ethmoid nerves (V1) and the sphenopalatine ganglion (through the nasopalatine nerve)
provide sensation to most of the septum.
79. Answer: D (Ref. Gray’s Anatomy for Students, 3rd edition, By Richard Drake, A. Wayne Vogl, Adam W. M. Mitchell, page
no. 1092)
Although the buccinator is important in mastication, it is innervated by the buccal branch of the facial nerve and not by the
buccal nerve from V3 (a sensory nerve).
80. Answer: D (Ref. Netter’s Head and Neck Anatomy for Dentistry, 3rd edition, By Neil S. Norton, page no. 200)
81. Answer: D (Ref. Netter’s Head and Neck Anatomy for Dentistry, 3rd edition, By Neil S. Norton, page no. 184)
82. Answer: D (Ref. Netter’s Head and Neck Anatomy for Dentistry, 3rd edition, By Neil S. Norton, page no. 326)
83. Answer: D (Ref. Netter’s Head and Neck Anatomy for Dentistry, 3rd edition, By Neil S. Norton, page no. 551)
84. Answer: B (Ref. Netter’s Head and Neck Anatomy for Dentistry, 3rd edition, By Neil S. Norton, page no. 497)
85. Answer: D (Ref. Gray’s Anatomy, 41st edition: The Anatomical Basis of Clinical Practice, edited by Susan Standring, page no. 638)
86. Answer: C (Ref. Netter’s Head and Neck Anatomy for Dentistry, 3rd edition, By Neil S. Norton, page no. 179)
The mandibular nerve innervates:
Anterior division
(Motor innervation – Muscles of mastication)
• Masseteric nerve
• Masseter
• Medial pterygoid nerve
• Medial pterygoid
• Tensor tympani
• Tensor veli palatini nerve
• Tensor veli palatini
• Lateral pterygoid nerve
• Lateral pterygoid
• Deep temporal nerve
• Temporalis
(Sensory innervation)
• Buccal nerve
• Inside of the cheek (buccal mucosa)

https://t.me/DentalBooksWorld
550 Triumph’s Complete Review of Dentistry

Posterior division
Lingual split
(Sensory innervation—not taste)
• Anterior 2/3rd of the tongue (mucous membrane)
• Inferior alveolar split (motor innervation)
• Mylohyoid
• Digastric (anterior belly)
(Sensory innervation)
• Teeth and mucoperiosteum of mandibular teeth
• Chin and lower lip
Auriculotemporal split
• Scalp (auricula/temporal region)
87. Answer: A (Ref. Lippincott’s Concise Illustrated Anatomy: Head & Neck, By Ben Pansky, Thomas R. Gest, page no. 352)
88. Answer: C (Ref. Gray’s Anatomy for Students, 2nd edition, By Richard Drake, A. Wayne Vogl, Adam W. M. Mitchell,
page no. 903)
Sensory innervation of the auricle

Nerve Derived from Region supplied


Greater auricular Cervical plexus C2 C3 Medial surface and posterior part of the lateral surface of the pinna
Lesser occipital Cervical plexus C2 Superior portion of the medial surface
Auricular Vagus Concha and antihelix
Auriculotemporal Mandibular division of Tragus, crus of the helix, and adjacent helix
trigeminal nerve
Facial Supplies root of concha

89. Answer: B
90. Answer: C (Ref. Netter’s Head and Neck Anatomy for Dentistry, 3rd edition, By Neil S. Norton, page no. 469)
• Fascia around the brachial plexus is called axillary sheath and is a derivative of prevertebral fascia (a).
• Prevertebral fascia (PVF) covers the anterior vertebral muscles and lies on the anterior aspect of the scalenus anterior
and the medius, thus forming the floor of the posterior triangle of the neck.
• Brachial plexus emerges between the scalenus and the medius in the neck and pass behind the clavicle along with the
subclavian artery to reach the axilla.
a. In the process, they carry an extension of PVF over them as a cover (the axillary sheath) toward the axilla.
b. Subclavian/axillary veins lie outside the axillary sheath and therefore can distend freely.
Applied anatomy:
Neck infections behind the PVF are usually due to tuberculosis of the cervical vertebra and may form  chronic
retropharyngeal abscess—a bulging in the posterior wall of the pharynx.
• The pus may track into the axilla via the axillary sheath and point in the posterior/lateral wall of the axilla.
• The pus may also extend into the superior mediastinum but does not reach the posterior mediastinum, since the PVF
fuses to the fourth thoracic vertebra.
PVF is separated from the pharynx/buccopharyngeal fascia by the retropharyngeal space.
• Neck infections in front of PVF in the retropharyngeal space form acute retropharyngeal abscess which bulges forward
in the paramedian position.
• This is due to the attachment of PVF with buccopharyngeal fascia in the median plane.
• This infection may spread to the posterior mediastinum via the superior mediastinum.
Most of the nerves in the neck are behind the PVF but the spinal accessory nerve lies superficial to it and may get
damaged quite often.
• The spinal accessory nerve is the most common nerve damaged iatrogenically. This happens mostly during I&D (incision
and drainage) procedures in the neck.
91. Answer: C (Ref. Netter’s Head and Neck Anatomy for Dentistry, 3rd edition, By Neil S. Norton, page no. 518)
92. Answer: C (Ref. Netter’s Clinical Anatomy, 3rd edition, By John T. Hansen, page no. 467)

https://t.me/DentalBooksWorld
Chapter 6 • Embryology, Head and Neck Anatomy, and General Histology 551

Neurovascular Supply of Maxillary Sinus


Arterial supply:
• The maxillary sinus is unique in that it is not supplied by the anterior or posterior ethmoidal arteries.
• Instead, it receives supply from the superior alveolar arteries (anterior, middle, and posterior) as well as the infraorbital
and greater palatine arteries.
Venous drainage:
• Veins drain posteriorly into the pterygopalatine venous plexus or laterally into the facial vein.
Lymphatics:
• Lymph originating in the maxillary sinus drains inferiorly to submandibular nodes.
Innervation:
• The maxillary sinus receives general sensory supply from branches of the maxillary nerve (V2), including the superior
alveolar nerves and the infraorbital nerve.
The anterior ethmoidal nerve is a nerve which provides sensory branches to the nasal cavity. The nerve is the continuation
of the nasociliary nerve after it enters the anterior ethmoidal foramen into the anterior ethmoidal air cells.
93. Answer: B (Ref. Netter’s Head and Neck Anatomy for Dentistry, 3rd edition, By Neil S. Norton, page no. 416)
94. Answer: A (Ref. Netter’s Head and Neck Anatomy for Dentistry, 3rd edition, By Neil S. Norton, page no. 676)

Muscle Origin Insertion Action Innervation Artery Note


Risorius Fascia of the Skin of the angle Draws the Buccal branches Transverse Risorius is active
lateral cheek (corner) of the corner of the of the facial facial a., in expressions of
mouth mouth laterally nerve (VII) facial a. mirth (Latin, risorius =
to laugh)

95. Answer: C (Ref. Netter’s Head and Neck Anatomy for Dentistry, 3rd edition, By Neil S. Norton, page no. 490)
96. Answer: A (Ref. Netter’s Head and Neck Anatomy for Dentistry, 3rd edition, By Neil S. Norton, page no. 686)
• These muscles are innervated by the pharyngeal plexus via the vagus nerve, with the exception of the tensor veli palatini.
• The tensor veli palatini is innervated by cranial nerve 5 branch V3 (which is the mandibular division of the trigeminal
cranial nerve).
97. Answer: D (Ref. Gray’s Anatomy, 41st edition: The Anatomical Basis of Clinical Practice, edited by Susan Standring,
page no. 513)
98. Answer: A (Ref. Clemente’s Anatomy Dissector: Guides to Individual Dissections in Human, 3rd edition, By Carmine D.
Clemente, page no. 406)
99. Answer: A (Ref. Netter’s Clinical Anatomy, 3rd edition, By John T. Hansen, page no. 485)

Nerve Source Branches Motor Sensory Notes


Greater occipital Dorsal No named Posterior neck Skin of the Muscles innervated by this
n. primary ramus branches muscles posterior surface nerve develop from epimeres in
of spinal nerve of the scalp the embryo
C2 (medial br.)

100. Answer: B (Ref. Gray’s Anatomy, 41st edition: The Anatomical Basis of Clinical Practice, edited by Susan Standring, page
no. 679)

Nerve Source Branches Motor Sensory Notes


Lacrimal n. Ophthalmic No named Carries Portion of the upper eye Lacrimal n. carries
division of the branches secretomotor lid and its associated the postganglionic
trigeminal n. axons to conjunctiva parasympathetic
(V1) the lacrimal axons from the
gland zygomaticotemporal
br. of the maxillary
n. that originates in
the pterygopalatine
ganglion

https://t.me/DentalBooksWorld
552 Triumph’s Complete Review of Dentistry

Frontal Ophthalmic Supraorbital n., None Skin of the forehead and The most superior
division of the supratrochlear n. the medial part of the linear structure within
trigeminal n. upper eyelid; mucous the orbit
(V1) membrane of the frontal
sinus
Anterior ethmoidal Nasociliary n. Internal and None Mucous membrane lining Anterior ethmoidal
external nasal the anterior ethmoid n. passes from
brs. air cells and the upper the orbit into the
anterior part of the nasal anterior ethmoidal
cavity; skin of the lower foramen, passes
half of the nose through the
cribriform plate,
passes anteriorly on
the cribriform plate,
then exits the cranial
cavity through the
ethmoid fissure into
the nasal cavity
Posteriorethmoidal Nasociliary n. No named None Mucous membrane lining Posterior ethmoidal
branches the posterior ethmoid air n. exits the orbit by
cells and sphenoid sinus passing through the
posterior ethmoid
foramen
Nasociliary n. Ophthalmic Communicating None Eyeball, skin of the nose The distribution of
division of the br. to the ciliary and medial sides of the this nerve is indicated
trigeminal n. ganglion, long eyelids; conjunctiva by its name—
(V1) ciliary n., anterior of the medial sides of nasociliary
and posterior the eyelids; mucous
ethmoidal nn., membranes of the upper
infratrochlear n. nasal cavity, ethmoid and
sphenoid sinuses

101. Answer: C (Ref. Textbook of Anatomy: Volume 3, 5th edition: Head and Neck, Central Nervous System, By Inderbir Singh,
page no. 776)
102. Answer: A (Ref. Netter’s Cranial Nerve Collection by Frank H. Netter, page no. 5)
103. Answer: C (Ref. Textbook of Anatomy: Volume 3, 5th edition: Head and Neck, Central Nervous System, By Inderbir Singh,
page no. 730)
It is so called because it lies just in front of a downward projection called the spine of sphenoid.
104. Answer: B (Ref. Gray’s Anatomy for Students, 3rd edition, By Richard Drake, A. Wayne Vogl, Adam W. M. Mitchell, page
no. 46)
105. Answer: B (Ref. Gray’s Anatomy, 41st edition: The Anatomical Basis of Clinical Practice, edited by Susan Standring, page
no. 579)
The tensor veli palatini is innervated by the medial pterygoid nerve, a branch of the mandibular nerve, the third branch of
the trigeminal nerve (CN V3)—the only muscle of the palate not innervated by the pharyngeal plexus, which is formed by
the vagal and glossopharyngeal nerves.
106. Answer: D (Ref. Gray’s Anatomy for Students, 3rd edition, By Richard Drake, A. Wayne Vogl, Adam W. M. Mitchell, page
no. 868)
Structures passing through the stylomastoid foramen
• Facial nerve
• Posterior auricular artery (stylomastoid branch)
107. Answer: C (Ref. Gray’s Anatomy for Students, 3rd edition, By Richard Drake, A. Wayne Vogl, Adam W. M. Mitchell, page
no. 868)

https://t.me/DentalBooksWorld
Chapter 6 • Embryology, Head and Neck Anatomy, and General Histology 553

108. Answer: D (Ref. Gray’s Anatomy for Students, 2nd edition, By Richard Drake, A. Wayne Vogl, Adam W. M. Mitchell, page
no. 964)
109. Answer: C (Ref. Netter’s Cranial Nerve Collection, By Frank H. Netter, page no. 47)
110. Answer: A (Ref. Gray’s Anatomy for Students, 2nd edition, By Richard Drake, A. Wayne Vogl, Adam W. M. Mitchell, page
no. 964)
111. Answer: C (Ref. Netter’s Head and Neck Anatomy for Dentistry, 3rd edition, By Neil S. Norton, page no. 195)
112. Answer: A (Ref. Clinically Oriented Anatomy, 7th edition, By Keith L. Moore, Arthur F. Dalley, A. M. R. Agur, page no.
1053)
113. Answer: A (Ref. Lippincott’s Concise Illustrated Anatomy: Head & Neck, Volume 3, By Ben Pansky, Thomas R. Gest, page
no. 352)
114. Answer: D (Ref. Gray’s Anatomy, 41st edition: The Anatomical Basis of Clinical Practice, edited by Susan Standring, page
no. 671)
115. Answer: C (Ref. Netter’s Cranial Nerve Collection, By Frank H. Nette, page no. 17)
116. Answer: C (Ref. Gray’s Anatomy for Students, 2nd edition, By Richard Drake, A. Wayne Vogl, Adam W. M. Mitchell, page
no. 945)
117. Answer: B (Ref. Gray’s Anatomy for Students, 2nd edition, By Richard Drake, A. Wayne Vogl, Adam W. M. Mitchell. page
no. 945)
118. Answer: D (Ref. Gray’s Anatomy for Students, 2nd edition, By Richard Drake, A. Wayne Vogl, Adam W. M. Mitchell, page
no. 945)
119. Answer: C
120. Answer: D (Ref. Textbook of Anatomy: Volume 3, 5th edition: Head and Neck, Central Nervous System, By Inderbir Singh,
page no. 926)
121. Answer: D (Ref. Gray’s Anatomy Review, 2nd edition, By Marios Loukas, R. Shane Tubbs, Peter H. Abrahams, Stephen W.
Carmichael, page no. 436)
122. Answer: A
123. Answer: C (Ref. Gray’s Anatomy for Students, 2nd edition, By Richard Drake, A. Wayne Vogl, Adam W. M. Mitchell, page
no. 917)
124. Answer: C (Ref. Gray’s Anatomy for Students, 2nd edition, By Richard Drake, A. Wayne Vogl, Adam W. M. Mitchell, page
no. 1029)
125. Answer: D (Ref. Cummings Otolaryngology—Head and Neck Surgery E-Book: Head and Neck, 5th edition, By Paul W.
Flint, Bruce H. Haughey, John K. Niparko, Mark A. Richardson, Valerie J. Lund, K. Thomas Robbins, Marci M. Lesperance,
J. Regan Thomas, page no. 2417)
The eyelid movements are mediated mainly by the orbicularis oculi and the levator palpebrae superioris (LPS) muscles.

Muscle Origin Insertion Action Innervation Artery Note


Levator Apex of the Skin and fascia Elevates Oculomotor Ophthalmic a. Fibers inserting
palpebrae orbit above of the upper the upper nerve (III) and into the superior
superioris the optic eyelid and the eyelid(opens sympathetics tarsal plate are called
canal superior tarsal eye lid) (to the superior superior tarsal muscle
plate tarsal portion) and are smooth
muscles; lesion of the
sympathetic supply
causes slight ptosis
(drooping of the
eyelid)
Orbicularis Skin and Skin and fascia Purses the Buccal branch Superior and The “kissing” muscle
oris fascia of of the lips lips(closes of the facial inferior labial
the lips and eye lid) nerve (VII) branches of the
the area facial a., mental
surrounding a., infraorbital a.
the lips

https://t.me/DentalBooksWorld
554 Triumph’s Complete Review of Dentistry

126. Answer: B (Ref. “previous question no. 120”)


127. Answer: C (Ref. Gray’s Clinical Neuroanatomy, By Elliott L. Mancall, David G. Brock, page no. 187)
128. Answer: D (Ref. Clemente’s Anatomy Dissector: Guides to Individual Dissections in Human, 3rd edition, By Carmine D.
Clemente, page no. 232)
129. Answer: C (Ref. Anatomy for Dental Students, 4th edition, By Martin E. Atkinson, page no. 265)
130. Answer: A (Ref. Clemente’s Anatomy Dissector: Guides to Individual Dissections in Human, 3rd edition, By Carmine D.
Clemente, page no. 359)
131. Answer: A (Ref. Netter’s Head and Neck Anatomy for Dentistry, 3rd edition, By Neil S. Norton, page no. 144)
132. Answer: D (Ref. Gray’s Anatomy for Students, 2nd edition, By Richard Drake, A. Wayne Vogl, Adam W. M. Mitchell, page
no. 935)
133. Answer: C (Ref. Gray’s Basic Anatomy, 2nd edition, By Richard Drake, A. Wayne Vogl, Adam W. M. Mitchell, page no. 511)
134. Answer: B (Ref. Netter’s Clinical Anatomy, 3rd edition, By John T. Hansen, page no. 31)
The sympathetic division of the ANS is described as having “thoracolumbar outflow” due to the origin of its preganglionic
neurons in the thoracic and upper lumbar spinal cord; the parasympathetic  division  of the ANS is described as having
“craniosacral outflow” due to the origin of its preganglionic neurons.
135. Answer: B (Ref. Gray’s Anatomy, 41st edition: The Anatomical Basis of Clinical Practice, edited by Susan Standring, page
no. 67)
Synapse is a structure that permits a neuron (or nerve cell) to pass an electrical or chemical signal to another neuron.
136. Answer: B (Ref. Gray’s Anatomy, 41st edition: The Anatomical Basis of Clinical Practice, edited by Susan Standring, page
no. 321)
137. Answer: C (Ref. “previous question 134”)
138. Answer: D (Ref. Gray’s Anatomy for Students, 2nd edition, By Richard Drake, A. Wayne Vogl, Adam W. M. Mitchell, page
no. 49)
Preganglionic parasympathetic nerves (CN III, CN VII, and CN IX) usually arise from specific nuclei in the central nervous
system (CNS) and synapse at one of the four parasympathetic ganglia: Ciliary, pterygopalatine, otic, or submandibular.
From these four ganglia the parasympathetic nerves complete their journey to target tissues via  trigeminal  branches
(ophthalmic nerve, maxillary nerve, mandibular nerve).
139. Answer: C (Ref. Gray’s Anatomy for Students, 3rd edition, By Richard Drake, A. Wayne Vogl, Adam W. M. Mitchell, page
no. 997)
140. Answer: D (Ref. Gray’s Anatomy for Students, 2nd edition, By Richard Drake, A. Wayne Vogl, Adam W. M. Mitchell, page
no. 214)

Nerve Source Branches Motor Sensory Notes


Phrenic n. Ventral No named Skeletal muscle Diaphragmatic Phrenic n. crosses the anterior
primary rami branches of the respiratory pleura; surface of the anterior scalene
of spinal diaphragm some fibers muscle
nerves C3– contributed to
C5 (cervical the pericardium
plexus) and to the
adjacent
mediastinal and
costal pleurae

141. Answer: C (Ref. Gray’s Basic Anatomy, By Richard Drake, A. Wayne Vogl, Adam W. M. Mitchell, page no. 481)
142. Answer: D (Ref. Netter’s Head and Neck Anatomy for Dentistry, 3rd edition, By Neil S. Norton, page no. 122)
143. Answer: D
144. Answer: D (Ref. Gray’s Anatomy for Students, 2nd edition, By Richard Drake, A. Wayne Vogl, Adam W. M. Mitchell, page
no. 903)
145. Answer: B
146. Answer: C (Ref. Netter’s Head and Neck Anatomy for Dentistry, 2nd edition, By Neil S. Norton, page no. 626)
147. Answer: D (Ref. Gray’s Basic Anatomy, 2nd edition, By Richard Drake, A. Wayne Vogl, Adam W. M. Mitchell, page no. 642)

https://t.me/DentalBooksWorld
Chapter 6 • Embryology, Head and Neck Anatomy, and General Histology 555

Nerve Source Branches Motor Sensory Notes


Deep petrosal Internal No named Vascular smooth None Deep petrosal n. joins the greater
carotid branches muscle of the mucous petrosal n. to form the n. of the
plexus membranes of the pterygoid canal; the deep petrosal n.
lower nasal cavity, contains postganglionic sympathetic
maxillary sinus, and axons (synapse occurred in the
palate superior cervical sympathetic
ganglion)

148. Answer: C (Ref. Gray’s Anatomy, 41st edition: The Anatomical Basis of Clinical Practice, edited by Susan Standring, page
no. 833)
149. Answer: A (Ref. Gray’s Anatomy, 41st edition: The Anatomical Basis of Clinical Practice, edited by Susan Standring, page
no. 467)
150. Answer: D (Ref. Netter’s Head and Neck Anatomy for Dentistry, 2nd edition, By Neil S. Norton, page no. 146)
151. Answer: D (Ref. Netter’s Cranial Nerve Collection, By Frank H. Netter, page no. 42)
152. Answer: C (Ref. Anatomy and Physiology, 9th edition, By Kevin T. Patton, page no. 500)
153. Answer: D (Ref. Gray’s Basic Anatomy, 2nd edition, By Richard Drake, A. Wayne Vogl, Adam W. M. Mitchell, page no. 464)
154. Answer: D (Ref. Gray’s Basic Anatomy E-Book, By Richard Drake, A. Wayne Vogl, Adam W. M. Mitchell,
page no. 530)
155. Answer: D (Ref. Gray’s Anatomy for Students, 3rd edition, By Richard Drake, A. Wayne Vogl, Adam W. M. Mitchell, page
no. 876)
The dorsal rami of the first three cervical nerves, the ventral rami of the first two cervical nerves, the hypoglossal nerve
and recurrent branches of the vagus nerve that follow the posterior meningeal artery provide innervation to the posterior
cranial fossa dura mater.
156. Answer: C (Ref. Gray’s Anatomy, 41st edition: The Anatomical Basis of Clinical Practice, edited by Susan Standring, page
no. 466)
157. Answer: B (Ref. Gray’s Anatomy for Students, 3rd edition, By Richard Drake, A. Wayne Vogl, Adam W. M. Mitchell, page
no. 1034)
158. Answer: D
159. Answer: C (Ref. Clemente’s Anatomy Dissector: Guides to Individual Dissections in Human, 3rd edition, By Carmine D.
Clemente, page no. 206)
160. Answer: C (Ref. Gray’s Basic Anatomy, 2nd edition, By Richard Drake, A. Wayne Vogl, Adam W. M. Mitchell, page no. 446)
161. Answer: C (Ref. Netter’s Head and Neck Anatomy for Dentistry, 2nd edition, By Neil S. Norton, page no. 75)
162. Answer: A (Ref. McMinn’s Color Atlas of Head and Neck Anatomy, 5th edition, By Bari M. Logan, Patricia Reynolds, Scott
Rice, Ralph T. Hutchings, page no. 149).

MUSCLES
1. Answer: D (Ref. Gray’s Anatomy, 41st edition: The Anatomical Basis of Clinical Practice, edited by Susan Standring, page
no. 670)

Muscle Movement Contralateral Target Origin Insertion Nerve supply


compensatory motion
action direction
Superior Moves eyes up Inferior oblique Superior Common Superior surface Oculomotor
rectus and toward the (elevates); tendinous ring of the eyeball
nose; rotates eyes medial (ring attaches to
from 1 o’clock to (adducts) optic foramen)
3 o’clock
Inferior Moves eyes down Superior oblique Inferior Common Inferior surface Oculomotor
rectus and toward the (depresses); tendinous ring of the eyeball
nose; rotates eyes medial (ring attaches to
from 6 o’clock to (adducts) optic foramen)
3 o’clock

https://t.me/DentalBooksWorld
556 Triumph’s Complete Review of Dentistry

Muscle Movement Contralateral Target Origin Insertion Nerve supply


compensatory motion
action direction
Lateral Moves eyes away Medial rectus Lateral Common Lateral surface of Abducent
rectus from the nose (abducts) tendinous ring the eyeball
(ring attaches to
optic foramen)
Medial Moves eyes Lateral rectus Medial Common Medial surface of Oculomotor
rectus toward the nose (adducts) tendinous ring the eyeball
(ring attaches to
optic foramen)
Inferior Moves eyes up Superior oblique Superior Floor of orbit Surface of the Oculomotor
oblique and away from (elevates); (maxilla) eyeball between
the nose; rotates lateral the inferior
the eyeball from (abducts) rectus and the
12 o’clock to 9 lateral rectus
o’clock
Superior Moves eyes down Inferior rectus Superior Sphenoid bone Surface of eyeball Trochlear
oblique and away from (elevates); between the
the nose; rotates lateral superior rectus
the eyeball from 6 (abducts) and the lateral
o’clock to 9 o’clock rectus
Levator Opens eyes – Roof of orbit Skin of upper – Oculomotor
palpebrae (sphenoid eyelids
superioris bone)
Orbicularis Closes eyelids – Medial bones Circumference of – Buccal branch
occuli composing orbit of the facial
the orbit nerve

2. Answer: B (Ref. Netter’s Head and Neck Anatomy for Dentistry, 3rd edition, By Neil S. Norton, page no. 126)
The TMJ is supported by the following ligaments:
There are three ligaments associated with the TMJ: two of these ligaments are functional, whereas one is a fascia collection
that plays no functional role in limiting the mandible’s motion.
• The temporomandibular ligament is located on the lateral aspect of the capsule and its function includes preventing
the lateral or the posterior displacement of the condyle.
• The stylomandibular ligament arises from the styloid process and attaches to the mandibular angle. It is responsible for
allowing the mandible to protrude.
• The sphenomandibular ligament stretches between the spine of the sphenoid bone and the lingula of the mandible. It
contributes to the limitation of extensive protrusive movements and jaw opening.
• The medial and lateral collateral ligaments (also known as the discal ligaments) help connect the medial and lateral
sides of the articular disc to the same side of the condyle.
3. Answer: D (Ref. Netter’s Head and Neck Anatomy for Dentistry, 3rd edition, By Neil S. Norton, page no. 126)
Borders of the anterior triangle of the neck are formed by the following:
• Median line of the neck from the chin to the manubrium
• Anterior margin of the sternocleidomastoid
• Horizontal plane formed by the lower margin of the mandible
The anterior triangle of the neck can be further subdivided into the following:
• The submandibular (digastric) triangle  between the posterior and anterior bellies of the digastric muscle and the
inferior border of the mandible
• The carotid triangle  between the  posterior belly of the digastric, the superior belly of the omohyoid, and the
sternocleidomastoid muscle

https://t.me/DentalBooksWorld
Chapter 6 • Embryology, Head and Neck Anatomy, and General Histology 557

• The muscular triangle between the superior belly of the omohyoid, the lower anterior margin of the sternocleidomastoid,
and the median line of the neck
• The submental triangle between the anterior bellies of the digastric muscle and above the hyoid bone
4. Answer: B (Ref. Netter’s Head and Neck Anatomy for Dentistry, 3rd edition, By Neil S. Norton, page no. 128)
Scalenus anterior muscle:
• Runs from the anterior tubercula of the transverse processes of the third to sixth cervical vertebrae to the first rib.
• Originates at the posterior tubercula of the transverse processes of the third to seventh cervical vertebrae and inserts at
the first rib dorsally to the scalenus anterior muscle.
Scalenus posterior muscle:
• Has its origin at the posterior tubercula of the transverse processes of the fifth to seventh cervical vertebrae and its
insertion at the second rib.
• The triangle between the scalenus anterior muscle, the scalenus medius muscle, and the first rib forms the interscalene
triangle.
• The subclavian artery and the brachial plexus pass through this gap. In contrary, the subclavian vein runs ventrally from
the scalenus anterior muscle.
Function:
Elevate the ribs, and therefore the thorax → considered as accessory muscles of inspiration.
5. Answer: A (Ref. Clinically Oriented Anatomy, 7th edition, By Keith L. Moore, Arthur F. Dalley, A. M. R. Agur, page no.
1002)
The larynx is the organ for vocalization and lies between the levels of C4 and C6 vertebrae.
Nerves of the larynx
• Superior laryngeal nerves – arise from the inferior ganglia of the vagus nerve and receive a branch from the superior
cervical sympathetic ganglion on each side of the upper neck. They descend adjacent to the pharynx on either side,
behind the internal carotid artery, and divide into internal and external branches.
–– The external branch (external laryngeal nerve) descends beneath the sternothyroid muscle and supplies the
cricothyroid muscle. Injury to this nerve during thyroidectomy or cricothyrotomy causes hoarseness of the voice
and an inability to produce high-pitched sounds.
–– The internal branch (internal laryngeal nerve) pierces the thyrohyoid membrane and supplies sensory innervation
to the laryngeal cavity down to the level of the vocal folds. It is responsible for the cough reflex.
• Recurrent laryngeal branch of the vagus nerve (CN X)
–– The recurrent laryngeal branches of the vagus nerves ascend into the larynx within the groove between the esophagus
and the trachea.
–– The left recurrent laryngeal nerve originates in the thorax, looping under the aortic arch before ascending, while the
right recurrent laryngeal nerve originates in the neck.
–– These nerves are responsible for supplying sensory innervation to the laryngeal cavity below the level of the vocal
folds, as well as motor innervation to all laryngeal muscles except the cricothyroid.
–– Since the nerves run immediately posterior to the thyroid gland, they are at risk of injury during thyroidectomies.
Unilateral nerve damage presents with voice changes, including hoarseness. Bilateral nerve damage may result in
aphonia (inability to speak) and breathing difficulties.
6. Answer: B (Ref. Gray’s Anatomy, 41st edition: The Anatomical Basis of Clinical Practice, edited by Susan Standring, page
no. 546)
Cervical spine ligaments
• Anterior atlanto-occipital membrane
• Apical ligament
• Alar ligaments
• Cruciate ligament of the atlas
• Tectorial membrane
• Posterior atlanto-occipital membrane
• Ligamentum nuchae
• Intertransverse ligaments
The alar ligaments join the lateral margins of the sloping upper margin of the dens of C2 to the lateral margins of the foramen
magnum (adjacent to the occipital condyles) and lie on either side of the apical ligament. 
They are paired ligaments that are very strong and limit the rotation of the head.

https://t.me/DentalBooksWorld
558 Triumph’s Complete Review of Dentistry

7. Answer: B (Ref. Netter’s Head and Neck Anatomy for Dentistry, 3rd edition, By Neil S. Norton, page no. 14)
• Genioglossus: Helps to protrude the tongue, depress the central part of the tongue making it concave, and move the
tongue to the opposite side
• Hyoglossus: Helps to depress the tongue
• Styloglossus: Helps to pull the tongue upward and backward to aid swallowing
• Palatoglossus: Pulls the soft palate onto the tongue while swallowing
• Intrinsic muscles: Help in the widening, flattening, thickening, lengthening, and rolling of the tongue
Motor supply for all intrinsic and extrinsic  muscles  of the  tongue  is supplied by efferent motor nerve fibers from the
hypoglossal nerve (CN XII), with the exception of the palatoglossus. The palatoglossus is innervated by the vagus nerve
(CN X).
8. Answer: C (Ref. Netter’s Head and Neck Anatomy for Dentistry, 3rd edition, By Neil S. Norton, page no. 14)
• Genioglossus is the fan-shaped extrinsic tongue muscle that forms the majority of the body of the tongue.
• It arises from the mental spine of the mandible and its insertions are the hyoid bone and the bottom of the tongue.
• The genioglossus is innervated by the hypoglossal nerve, as are all muscles of the tongue except for the palatoglossus.
• The canine genioglossus muscle has been divided into horizontal and oblique compartments.
Functions:
• The left and right genioglossus muscles protrude the tongue and deviate it toward the opposite side. When acting
together, the muscles depress the center of the tongue at its back.
• Unilateral tongue weakness causes the tongue to deviate toward the weak side.
• Tongue weakness can result from lesions in the tongue muscles, the neuromuscular junction, the lower motor neurons
of the hypoglossal nerve (CN XII), or the upper motor neurons originating in the motor cortex causing the deviation of
the tongue.
9. Answer: C (Ref. Clinically Oriented Anatomy, 7th edition, By Keith L. Moore, Arthur F. Dalley, A. M. R. Agur, page no.
1002)
10. Answer: C (Ref. Netter’s Head and Neck Anatomy for Dentistry, 3rd edition, By Neil S. Norton, page no. 126)
11. Answer: B (Ref. Netter’s Head and Neck Anatomy for Dentistry, 3rd edition, By Neil S. Norton, page no. 127)
Ansa cervicalis
• Lying superficial to the internal jugular vein and the carotid sheath, the ansa cervicalis  forms a loop of nerve fibers
whose principal function is to supply motor innervation to the strap muscles (except the thyrohyoid and the geniohyoid).
It is formed by two components:
–– Superior loop (descending hypoglossi) anterior and lateral on the carotid sheath
–– Inferior loop from the cervical plexus: Loops from posterior to lateral of carotid sheath
Leaves the hypoglossal as it turns anteriorly around the occipital artery.
• C1 hitchhikers  travel with the hypoglossal nerve: Some of these fibers leave the hypoglossal nerve in the neck and
descend down (superior root) and join other nerves of anterior rami of C2 and C3 (inferior root) to innervate the neck
muscles (descendens hypoglossi)
• Other fibers of C1 travel further with the hypoglossal nerve and jump off to innervate the thyrohyoid and, later in the
floor of the mouth, to the geniohyoid muscles
• Remember: Only hitch-hiking fibers from C1–C3 actually innervate the neck muscles—not the hypoglossal nerve itself
• Produce sensory nerves: The  great auricular nerve  and  the transverse cervical nerves  (anterior cutaneous nerve of
neck) for C2 and C3 dermatomes
The five motor branches of the ansa cervicalis loop are as follows:
• Geniohyoid nerve (C1)
• Thyrohyoid nerve (C1)
• Omohyoid nerve (C1–C3)
• Sternohyoid nerve (C1–C3)
• Sternothyroid nerve (C1–C3)
12. Answer: A (Ref. Clinically Oriented Anatomy, 7th edition, By Keith L. Moore, Arthur F. Dalley, A. M. R. Agur, page no. 1002)
There are nine cartilages located within the larynx: three unpaired and six paired. 
Unpaired cartilages
• Thyroid cartilage
• Cricoid cartilage
• Epiglottis

https://t.me/DentalBooksWorld
Chapter 6 • Embryology, Head and Neck Anatomy, and General Histology 559

Paired cartilages (3 ´ 3)


• Arytenoid
• Corniculate
• Cuneiform
They are situated bilaterally in the larynx.
13. Answer: A (Ref. Netter’s Head and Neck Anatomy for Dentistry, 3rd edition, By Neil S. Norton, page no. 351)
14. Answer: B (Ref. Netter’s Head and Neck Anatomy for Dentistry, 3rd edition, By Neil S. Norton, page no. 239)

Muscle Origin Insertion Action Innervation Artery Notes


Buccinator Pterygomandibular Angle of mouth and Pulls the Buccal Facial Although the
raphe, mandible, the lateral portion corner of branches of the buccinator is
and the maxilla of the upper and mouth facial nerve important in
lateral to the molar lower lips laterally; (VII) mastication, it is
teeth presses innervated by the
the cheek buccal branch of
against the the facial nerve and
teeth NOT by the buccal
nerve from V3 (a
sensory nerve)
(Latin, buccinator =
trumpeter)
Lateral Superior head: Superior head: Protracts Lateral Pterygoid The only one of
pterygoid Greater wing of Capsule and the pterygoid branch the muscles of
the sphenoid bone; articular disc of the mandible; branch of the of the mastication that
inferior head: temporomandibular opens the mandibular maxillary a. opens the mouth;
Lateral surface joint; inferior mouth; division of the superior head
of the lateral head: Neck of the active in the trigeminal of lateral pterygoid
pterygoid plate mandible grinding nerve (V) is sometimes called
actions of sphenomeniscus
chewing due to its insertion
into the disc of the
temporomandibular
joint (Greek,
pterygoid = wing-
shaped)
Masseter Zygomatic arch Lateral surface of Elevates the Nerve to the Masseteric (Greek, masseter =
and zygomatic the ramus and angle mandible masseter, branch the chewer)
bone of the mandible from the of the
mandibular maxillary a.
division of
the trigeminal
nerve (V)
Temporalis  Temporal fossa and Coronoid process Elevates the Anterior and Anterior A powerful chewing
the temporal fascia  of the mandible and mandible; posterior and muscle; a derivative
the anterior surface retracts the deep temporal posterior of the first
of the ramus of the mandible nerves deep pharyngeal arch 
mandible  (posterior from the temporal
fibers)  mandibular aa. 
division of
the trigeminal
nerve (V) 

https://t.me/DentalBooksWorld
560 Triumph’s Complete Review of Dentistry

15. Answer: C (Ref. Clinically Oriented Anatomy, 7th edition, By Keith L. Moore, Arthur F. Dalley, A. M. R. Agur, page no.
1040)
• The thyroid is a highly vascular, brownish-red gland located anteriorly in the lower neck, extending from the level of the
fifth cervical (C5) vertebra down to the first thoracic (T1).
• The gland varies from an H to a U shape and is formed by two elongated lateral lobes with the superior and inferior poles
connected by a median isthmus, with an average height of 12–15 mm, overlying the second to fourth tracheal rings.
16. Answer: A (Ref. Gray’s Anatomy, 41st edition: The Anatomical Basis of Clinical Practice, edited by Susan Standring, page
no. 670)
17. Answer: B (Ref. Gray’s Anatomy for Students, 2nd edition, By Richard Drake, A. Wayne Vogl, Adam W. M. Mitchell, page
no. 1034)
18. Answer: B (Ref. Clinically Oriented Anatomy, 7th edition, By Keith L. Moore, Arthur F. Dalley, A. M. R. Agur, page no.
1002)
Muscles of larynx

Muscle Origin Insertion Action Innervation Artery Notes


Cricothyroid Arch of Inferior Draws the External Cricothyroid This is the only intrinsic
the cricoid border of thyroid cartilage branch of branch of muscle of the larynx that
cartilage the thyroid forward, superior the superior is not supplied by the
cartilage lengthening the laryngeal thyroid a. recurrent laryngeal n.; all
vocal ligaments nerve, a other laryngeal muscles are
branch of the innervated by the recurrent
vagus nerve laryngeal nerve, via its
(X) inferior laryngeal branch
Posterior Posterior Muscular Draws the Inferior Superior The posterior
cricoarytenoid surface process muscular process laryngeal laryngeal cricoarytenoid is the only
of the of the posteriorly, nerve, from a., the abductor of the vocal folds;
lamina of arytenoid which pivots the recurrent cricothyroid the arteries that supply the
the cricoid cartilage the arytenoid laryngeal branch of larynx anastomose within
cartilage cartilage and nerve, a the superior the larynx to supply the
abducts the vocal branch of the thyroid a. mucous membranes and
folds vagus nerve muscles
(X)
Vocalis Surface of Vocal Relaxes segments Inferior Laryngeal Represents the innermost
the thyroid ligament of the vocal laryngeal br. of the fibers of the thyroarytenoid
cartilage, ligament, thereby nerve, from superior muscle; the cricothyroid
vocal adjusting the the recurrent thyroid a. m. grossly tenses the
process pitch laryngeal vocal ligaments, while the
of the nerve, a vocalis muscle provides the
arytenoid branch of the mechanism for fine tuning
cartilage vagus nerve the vibrations of the vocal
(X) folds
Transverse Posterior Posterior Draws arytenoid Inferior Laryngeal The transverse arytenoid
arytenoid surface surface cartilages laryngeal br. of the m. lies anterior to the
of the of the together, nerve, from superior oblique arytenoid m.;
arytenoid contralateral adducting the the recurrent thyroid a. the arytenoideus is often
cartilage arytenoid vocal folds laryngeal considered to be one muscle
cartilage nerve, a with oblique and transverse
branch of the parts
vagus nerve
(X)

https://t.me/DentalBooksWorld
Chapter 6 • Embryology, Head and Neck Anatomy, and General Histology 561

Lateral Arch of Muscular Draws the Inferior Superior An antagonist of the


cricoarytenoid the cricoid process muscular process laryngeal laryngeal posterior cricoarytenoid m.;
cartilage of the of the arytenoid nerve, from a., the the arteries that supply the
arytenoid cartilage the recurrent cricothyroid larynx anastomose within
cartilage anteriorly, laryngeal branch of the larynx to supply the
which pivots nerve, a the superior mucous membranes and
the arytenoid branch of the thyroid a. muscles
cartilage and vagus nerve
adducts the vocal (X)
folds

19. Answer: C (Ref. Clinically Oriented Anatomy 7th edition, By Keith L. Moore, Arthur F. Dalley, A. M. R. Agur, page no. 1002)
20. Answer: B (Ref. Netter’s Head and Neck Anatomy for Dentistry, 3rd edition, By Neil S. Norton, page no. 126)
The carotid triangle of the neck has the following boundaries:
• Superior: Posterior belly of the digastric muscle
• Lateral: Medial border of the sternocleidomastoid muscle
• Inferior: Superior belly of the omohyoid muscle
The main contents of the carotid triangle are the common carotid artery (which bifurcates within the carotid triangle into
the external and internal carotid arteries), the internal jugular vein, and the hypoglossal and vagus nerves.
21. Answer: C (Ref. Netter’s Head and Neck Anatomy for Dentistry, 3rd edition, By Neil S. Norton, page no. 127)
• All suprahyoid muscles contribute to the floor of the mouth but the actual muscle plate which bridges between the two
rami of the mandible is formed by the mylohyoid muscles (oral diaphragm).
• From above, the mouth floor is reinforced by the geniohyoid muscles and from below by the anterior bellies of the
digastric muscles.

Muscle Origin Insertion Action Innervation Artery Notes


Mylohyoid Mylohyoid line of Midline raphe Elevates Mylohyoid Mylohyoid The nerve to mylohyoid
the mandible and body of the hyoid nerve from branch also innervates the
the hyoid bone bone and the inferior of the anterior belly of the
the tongue; alveolar inferior digastric m.; both
depresses the nerve, a alveolar a. muscles are derivatives
mandible branch of the of the second
mandibular pharyngeal arch
division
of the
trigeminal
nerve (V)
Hyoglossus Upper border of Spreads out Depresses Hypoglossal Lingual a. An extrinsic muscle of
the greater horn of into the the sides of nerve (XII) the tongue
the hyoid and body intrinsic the tongue;
of the hyoid bone muscles of the retracts the
tongue tongue
Genioglossus Mental spine on Fans out to Protrudes Hypoglossal Lingual a. An extrinsic muscle
the inner aspect insert into the the tongue nerve (XII) of the tongue;
of the mental tongue from (inferior XII innervates all
symphysis the tip to the fibers); tongue muscles
base depresses except palatoglossus
the tongue (innervated by vagus
(middle (X), as are most of the
fibers) palate and pharynx
muscles); genioglossus
is used to test the
function of cranial
nerve XII

https://t.me/DentalBooksWorld
562 Triumph’s Complete Review of Dentistry

Buccinator Pterygomandibular Angle of the Pulls the Buccal Facial a. Although the
raphe, mandible, mouth and the corner of branches of buccinator is important
and the maxilla lateral portion the mouth the facial in mastication, it is
lateral to the molar of the upper laterally; nerve (VII) innervated by the
teeth and lower lips presses the buccal branch of
cheek against the facial nerve and
the teeth NOT by the buccal
nerve from V3 (a
sensory nerve) (Latin,
buccinator = trumpeter)

22. Answer: D (Ref. Netter’s Head and Neck Anatomy for Dentistry, 3rd edition, By Neil S. Norton, page no. 126)
Submandibular (digastric) triangle:
• Between the posterior and anterior bellies of the digastric muscle and the inferior border of the mandible
• Its floor is formed by the mylohyoid, hyoglossus, and middle constrictor muscles
• Continuous with the fossa for the parotid gland
• Mylohyoid muscle lies superior to the anterior belly of the digastric
• Forms a sling passing from side to side from its attachment to the internal surface of the mandible (mylohyoid line)
• Forms the floor of mouth: It is attached from the mylohyoid line to the superior aspect of body of hyoid bone and the
midline raphe
• Around the free edge of this muscle lies the duct of the submandibular salivary gland which occupies a significant part
of the triangle
• Associated with the anterior belly of digastric, as both are derived from the first branchial arch and therefore share the
same innervation: Mylohyoid Br. of the inferior alveolar n. of V3
• Hypoglossal nerve  (CN XII) also passes into the triangle as it goes to the tongue between the hyoglossus and the
mylohyoid close to the hyoid bone
• Facial artery, arising from the external carotid, passes superiorly deep to the posterior belly of digastric, follows the floor
of the triangle, and winds posteriorly over the submandibular gland and “grooves” the inferior edge of the mandible at
the anterior–inferior angle of the masseter muscle to reach the face
Posterior belly of the digastric:
• Originates from the digastric fossa medial to the mastoid process
• Attaches to the anterior belly of digastric by the intermediate tendon which is tied down by a fascial sling to the body of
the hyoid
• Associated with the  stylohyoid, which arises from the lateral surface of the styloid process, and it splits around the
common tendon of the digastric to insert into the hyoid bone
• Both muscles are derived from the second branchial arch and therefore share the same innervation: Facial n. (CN VII)
23. Answer: C (Ref. Netter’s Head and Neck Anatomy for Dentistry, 3rd edition, By Neil S. Norton, page no. 239)

Muscle Origin Insertion Action Innervation Artery Notes


Lateral Superior head: Superior head: Protracts the Lateral Pterygoid The only one of the
pterygoid Greater wing Capsule and mandible; pterygoid branch muscles of mastication
of the sphenoid articular disc of the opens the branch of the that opens the mouth;
bone; inferior temporomandibular mouth; of the maxillary a. the superior head
head: Lateral joint; inferior active in mandibular of lateral pterygoid
surface of the head: Neck of the grinding division is sometimes called
lateral pterygoid mandible actions of of the sphenomeniscus
plate chewing trigeminal due to its insertion
nerve (V) into the disc of the
temporomandibular
joint

https://t.me/DentalBooksWorld
Chapter 6 • Embryology, Head and Neck Anatomy, and General Histology 563

Medial Medial surface Medial surface of Elevates and Medial Pterygoid This muscle mirrors the
pterygoid of the lateral the ramus and angle protracts the pterygoid branch masseter m. in position
pterygoid plate, of the mandible mandible branch of the and action with the
pyramidal of the maxillary a. ramus of the mandible
process of the mandibular between the two mm.
palatine bone, division
tuberosity of the of the
maxilla trigeminal
nerve (V)
Masseter Zygomatic arch Lateral surface of Elevates the Nerve to the Masseteric
and zygomatic the ramus and angle mandible masseter, branch
bone of the mandible from the of the
mandibular maxillary a.
division
of the
trigeminal
nerve (V)
Temporalis Temporal fossa Coronoid process Elevates the Anterior and Anterior A powerful chewing
and the temporal of the mandible and mandible; posterior and muscle; a derivative
fascia the anterior surface retracts the deep posterior of the first pharyngeal
of the ramus of the mandible temporal deep arch
mandible (posterior nerves temporal
fibers) from the aa.
mandibular
division
of the
trigeminal n.

24. Answer: D (Ref. Netter’s Head and Neck Anatomy for Dentistry, 3rd edition, By Neil S. Norton, page no. 14)
25. Answer: A (Ref. Netter’s Head and Neck Anatomy for Dentistry, 3rd edition, By Neil S. Norton, page no. 14)
26. Answer: C (Ref. Netter’s Head and Neck Anatomy for Dentistry, 3rd edition, By Neil S. Norton, page no. 128)
Muscular triangle: Between the superior belly of the omohyoid, lower anterior margin of the sternocleidomastoid, and the
median line of the neck.
• Medially contains the infrahyoid muscles
• As stated, these strap muscles lie between the investing deep fascia and the visceral fascia covering the thyroid gland,
trachea, and the esophagus
• Are depressors of the larynx and the hyoid bone
• Except for thyrohyoid, they are all innervated by the ansa cervicalis (a motor plexus from the ventral rami of C1, 2, 3,
and 4) discussed in the Carotid Triangle Section in detail
• Deep in the muscular triangle it contains visceral structures of the neck including the thyroid gland, larynx, trachea, and
esophagus.
• Also includes the  recurrent laryngeal nerve, inferior laryngeal artery, and external laryngeal nerve parallel to the
superior laryngeal nerve
27. Answer: A
28. Answer: A (Ref. Netter’s Head and Neck Anatomy for Dentistry, 3rd edition, By Neil S. Norton, page no. 127)
Submental triangle: Between the anterior belly of the digastric, superior to the hyoid bone, and the midline of the neck
• Floor is formed by the mylohyoid muscle.
• Most noted for the presence of several submental lymph nodes which drain the floor of the oral cavity, tip of the tongue
and the middle lower lip, and central incisors.
• Anterior jugular veins: Lying in the midline, running from the submental triangle, they pierce the deep fascia above the
manubrium.

https://t.me/DentalBooksWorld
564 Triumph’s Complete Review of Dentistry

They pass between the posterior border of the sternocleidomastoid muscle and the upper border of the clavicle to drain into
the external jugular veins in the posterior triangle of the neck.
29. Answer: D (Ref. Netter’s Head and Neck Anatomy for Dentistry, 3rd edition, By Neil S. Norton, page no. 239)
30. Answer: D (Ref. Netter’s Head and Neck Anatomy for Dentistry, 3rd edition, By Neil S. Norton, page no. 239)
31. Answer: A (Ref. Clinically Oriented Anatomy, By Keith L. Moore, Arthur F. Dalley, A. M. R. Agur, page no. 962)
32. Answer: B (Ref. Netter’s Head and Neck Anatomy for Dentistry, 3rd edition, By Neil S. Norton, page no. 126)
33. Answer: C (Ref. Gray’s Anatomy, 41st edition: The Anatomical Basis of Clinical Practice, edited by Susan Standring, page
no. 670)
34. Answer: C (Ref. Clinically Oriented Anatomy, 7th edition, By Keith L. Moore, Arthur F. Dalley, A. M. R. Agur, page no.
1002)
35. Answer: A (Ref. Clemente’s Anatomy Dissector: Guides to Individual Dissections in Human, 3rd edition, By Carmine D.
Clemente, page no. 328)
36. Answer: C (Ref. Netter’s Head and Neck Anatomy for Dentistry, 3rd edition, By Neil S. Norton, page no. 239)
37. Answer: B (Ref. Clinically Oriented Anatomy, 7th edition, By Keith L. Moore, Arthur F. Dalley, A. M. R. Agur, page no.
1037)
38. Answer: B (Ref. Netter’s Head and Neck Anatomy for Dentistry, 3rd edition, By Neil S. Norton, page no. 127)
39. Answer: D (Ref. Netter’s Head and Neck Anatomy for Dentistry, 3rd edition, By Neil S. Norton, page no. 127)

Muscle Origin Insertion Action Innervation Artery Notes


Omohyoid Inferior belly: Upper Inferior belly: Depresses/ Ansa Transverse The intermediate
border of the scapula Intermediate stabilizes the cervicalis cervical a. tendon of omohyoid
medial to the scapular tendon; superior hyoid bone is tethered to the
notch; superior belly: belly: Lower clavicle by a fascial
Intermediate tendon border of the sling
hyoid bone lateral
to the sternohyoid
insertion

40. Answer: C (Ref. Clinically Oriented Anatomy, 7th edition, By Keith L. Moore, Arthur F. Dalley, A. M. R. Agur, page no.
1002)
41. Answer: C (Ref. Netter’s Head and Neck Anatomy for Dentistry, 3rd edition, By Neil S. Norton, page no. 14)
42. Answer: C (Ref. Gray’s Anatomy, 41st edition: The Anatomical Basis of Clinical Practice, edited by Susan Standring, page
no. 670)
43. Answer: A (Ref. Gray’s Anatomy, 41st edition: The Anatomical Basis of Clinical Practice, edited by Susan Standring, page
no. 636)
The ligaments of malleus are three ligaments that attach the malleus in the inner ear. They are the anterior, lateral, and
superior ligaments.
• The anterior ligament of the malleus is a fibrous band that extends from the neck of the malleus just above its anterior
process to the anterior wall of the tympanic cavity close to the petrotympanic fissure. Some of the fibers also pass through
the fissure to the spine of sphenoid bone.
• The lateral ligament of the malleus is a triangular fibrous band that crosses from the posterior aspect of the tympanic notch
to the head or neck of the malleus.
• The superior ligament of the malleus is a delicate fibrous strand that crosses from the roof of the tympanic cavity to the
head of the malleus.
Prussak’s space is the small middle ear recess, bordered laterally by the flaccid part of Shrapnell’s membrane, superiorly
by the scutum (a sharp bony spur that is formed by the superior wall of the external auditory canal) and the lateral malleal
ligament, inferiorly by the lateral process of the malleus, and medially by the neck of the malleus.
44. Answer: B (Ref. Netter’s Head and Neck Anatomy for Dentistry, 3rd edition, By Neil S. Norton, page no. 127)
45. Answer: D (Ref. Netter’s Head and Neck Anatomy for Dentistry, 3rd edition, By Neil S. Norton, page no. 351)

https://t.me/DentalBooksWorld
Chapter 6 • Embryology, Head and Neck Anatomy, and General Histology 565

Muscle Origin Insertion Action Innervation Artery Notes


Tensor veli palatini Scaphoid Palatine Opens the Mandibular Ascending Remember: V3
fossa, lateral aponeurosis auditory tube; division pharyngeal innervates both tensor
wall of the tenses the of the a. muscles (tympani and
auditory tube soft palate trigeminal veli palatini); all other
cartilage and involved nerve (V) palatal muscles are
in swallowing innervated by vagus
Palatoglossus Palatine Side of the Elevates and Vagus nerve Tonsillar Although palatoglossus
aponeurosis tongue, retracts the (X) via the branch of sounds like a tongue
entering it tongue and pharyngeal the facial a., muscle (and would
from above involved in plexus ascending therefore be innervated
swallowing pharyngeal by XII), it is a palatal
a. muscle innervated by X
Palatopharyngeus Posterior Posterior Elevates the Vagus nerve Ascending Palatopharyngeus
margin of wall of the larynx and (X) via pharyngeal is part of the inner
the bony pharynx and involved in pharyngeal a. longitudinal muscle
palate and the posterior breathing plexus layer of the pharynx
the palatine margin of
aponeurosis the thyroid
cartilage
Musculus uvulae Posterior Mucosa of Shortens Vagus nerve Lesser The soft palate and the
nasal spine the uvula and moves (X) via the palatine a. uvula resist the upward
the uvula pharyngeal pressure of the tongue
plexus during swallowing
Levator veli Apex of Muscles and Elevates the Vagus nerve Ascending A derivative of the
palatini the petrous fascia of the soft palate and (X) via the pharyngeal fourth pharyngeal arch
part of the soft palate; involved in pharyngeal a.
temporal palatine swallowing plexus
bone and aponeurosis
the medial
surface of the
auditory tube
cartilage

46. Answer: D (Ref. Gray’s Anatomy for Students, 2nd edition, By Richard Drake, A. Wayne Vogl, Adam W. M. Mitchell, page
no. 843)
The falx cerebelli is a small sickle-shaped fold of dura mater, projecting forward into the posterior cerebellar notch as well
as projecting into the vallecula of the cerebellum between the two cerebellar hemispheres.
47. Answer: C (Ref. Gray’s Anatomy for Students, 2nd edition, By Richard Drake, A. Wayne Vogl, Adam W. M. Mitchell, page
no. 1034)
48. Answer: A (Ref. Netter’s Head and Neck Anatomy for Dentistry, 3rd edition, By Neil S. Norton, page no. 127)
49. Answer: C (Ref. Netter’s Head and Neck Anatomy for Dentistry, 3rd edition, By Neil S. Norton, page no. 128)
50. Answer: D (Ref. Clinically Oriented Anatomy, 7th edition, By Keith L. Moore, Arthur F. Dalley, A. M. R. Agur, page no.
985)
51. Answer: D (Ref. Netter’s Head and Neck Anatomy for Dentistry, 3rd edition, By Neil S. Norton, page no. 239)
52. Answer: B (Ref. Gray’s Anatomy, 41st edition: The Anatomical Basis of Clinical Practice, edited by Susan Standring, page
no. 670)
53. Answer: B (Ref. Gray’s Anatomy, 41st edition: The Anatomical Basis of Clinical Practice, edited by Susan Standring, page
no. 634)
The vestibule is the central part of the bony labyrinth, and is situated medial to the eardrum (tympanic cavity), behind
the cochlea, and in front of the semicircular canals.
The vestibule is somewhat oval in shape, but flattened transversely; it measures about 5 mm from front to back, the same
from top to bottom, and about 3 mm across.

https://t.me/DentalBooksWorld
566 Triumph’s Complete Review of Dentistry

In its lateral or tympanic wall is the oval window (fenestra vestibuli), closed, in the fresh state, by the base of the stapes and the
annular ligament.
54. Answer: A (Ref. Gray’s Anatomy, 41st edition: The Anatomical Basis of Clinical Practice, edited by Susan Standring, page
no. 634)

Muscle Origin Insertion Action Innervation Artery Notes


Stapedius Walls of the Neck of the Dampens Facial nerve Anterior Entirely enclosed in bone, except
pyramidal stapes vibration of (VII) tympanic a. for its tendon; a derivative of the
eminence the stapes second pharyngeal arch

55. Answer: D (Ref. Clinically Oriented Anatomy, 7th edition, By Keith L. Moore, Arthur F. Dalley, A. M. R. Agur, page no.
1037)
56. Answer: B (Ref. Netter’s Head and Neck Anatomy for Dentistry, 3rd edition, By Neil S. Norton, page no. 127)
57. Answer: B (Ref. Gray’s Anatomy, 41st edition: The Anatomical Basis of Clinical Practice, edited by Susan Standring, page
no. 670)
58. Answer: C (Ref. Gray’s Anatomy, 41st edition: The Anatomical Basis of Clinical Practice, edited by Susan Standring, page
no. 670)
59. Answer: C (Ref. Netter’s Head and Neck Anatomy for Dentistry, 3rd edition, By Neil S. Norton, page no. 127)
60. Answer: D (Ref. Clinically Oriented Anatomy, 7th edition, By Keith L. Moore, Arthur F. Dalley, A. M. R. Agur, page no.
1037)
61. Answer: C (Ref. Clinically Oriented Anatomy, 7th edition, By Keith L. Moore, Arthur F. Dalley, A. M. R. Agur, page no.
1037)
62. Answer: D (Ref. Clinically Oriented Anatomy, 7th edition, By Keith L. Moore, Arthur F. Dalley, A. M. R. Agur, page no.
1002)
63. Answer: D (Ref. Clinically Oriented Anatomy, 7th edition, By Keith L. Moore, Arthur F. Dalley, A. M. R. Agur, page no.
1037)
64. Answer: D (Ref. Clinically Oriented Anatomy, 7th edition, By Keith L. Moore, Arthur F. Dalley, A. M. R. Agur, page no.
1037)
65. Answer: D (Ref. Netter’s Head and Neck Anatomy for Dentistry, 3rd edition, By Neil S. Norton, page no. 351)
66. Answer: B (Ref. Netter’s Head and Neck Anatomy for Dentistry, 3rd edition, By Neil S. Norton, page no. 239)
67. Answer: D (Ref. Netter’s Head and Neck Anatomy for Dentistry, 3rd edition, By Neil S. Norton, page no. 156)
68. Answer: A (Ref. Netter’s Head and Neck Anatomy for Dentistry, 3rd edition, By Neil S. Norton, page no. 127)
69. Answer: C (Ref. Netter’s Head and Neck Anatomy for Dentistry, 3rd edition, By Neil S. Norton, page no. 122)
The rectus capitis posterior major (or rectus capitis posticus major, both being Latin for larger posterior straight muscle
of the head) arises by a pointed tendon from the spinous process of the axis, and, becoming broader as it ascends, is
inserted into the lateral part of the inferior nuchal line of the occipital bone and the surface of the bone immediately
below the line.
70. Answer: D (Ref. Netter’s Head and Neck Anatomy for Dentistry, 3rd edition, By Neil S. Norton, page no. 45–46)
71. Answer: C (Ref. Clemente’s Anatomy Dissector: Guides to Individual Dissections in Human, 3rd edition, By Carmine D.
Clemente, page no. 270)
72. Answer: C (Ref. Netter’s Head and Neck Anatomy for Dentistry, 3rd edition, By Neil S. Norton, page no. 128)
73. Answer: D (Ref. Gray’s Anatomy, 41st edition: The Anatomical Basis of Clinical Practice, edited by Susan Standring, page
no. 670)
74. Answer: B (Ref. Gray’s Anatomy, 41st edition: The Anatomical Basis of Clinical Practice, edited by Susan Standring, page
no. 670)
75. Answer: A (Ref. Netter’s Head and Neck Anatomy for Dentistry, 3rd edition, By Neil S. Norton, page no. 15)
76. Answer: A (Ref. Gray’s Anatomy, 41st edition: The Anatomical Basis of Clinical Practice, edited by Susan Standring, page
no. 670)
77. Answer: D (Ref. Clinically Oriented Anatomy, 7th edition, By Keith L. Moore, Arthur F. Dalley, A. M. R. Agur, page no.
1037)
78. Answer: A (Ref. Netter’s Head and Neck Anatomy for Dentistry, 3rd edition, By Neil S. Norton, page no. 125)
79. Answer: B (Ref. Netter’s Head and Neck Anatomy for Dentistry, 3rd edition, By Neil S. Norton, page no. 24)

https://t.me/DentalBooksWorld
Chapter 6 • Embryology, Head and Neck Anatomy, and General Histology 567

80. Answer: B
81. Answer: D
• Deep fascia, dermis, scars, capsules of kidney, spleen are examples of dense irregular connective tissue.
• Fascia, tendon, aponeurosis, and ligaments are example of dense regular connective tissue.
82. Answer: A
Nucleus of trigeminal nerve: four in number

Mesencephalic nucleus Proprioception from muscles of mastication and periodontal ligament


Motor nucleus Motor supply to muscles of mastication, mylohyoid, ant. belly of digastric, tensor
tympani, tensor veli palatine
Trigeminal nucleus and spinal Sensory supply to scalp, ant. 2/3rd of dura, cornea, conjunctiva, face, paranasal sinus,
nucleus (main/chief) teeth, gingiva, and ant. 2/3rd of the tongue

83. Answer: B
• Weakest area in the posterior pharyngeal wall between the thyropharyngeus and the cricopharyngeus—Killian’s
dehiscence.
• Pharyngeal pouch at the weakest area of the pharynx due to Killian’s dehiscence—Zenker’s diverticulum.
• Passavant’s ridge is formed by the fibers of the palatopharyngeus and is present on the posterior wall of the pharynx.
84. Answer: C
Dermatome and the area supplied

Supraclavicular fossa at the midclavicular line C3


Over the acromioclavicular joint C4
Lateral side of the antecubital fossa C5
Dorsal surface of the proximal phalanx of the thumb C6
Dorsal surface of the proximal phalanx of the middle finger C7
Dorsal surface of the proximal phalanx of the little finger C8
Medial side of the antecubital fossa T1
Apex of the axilla T2

VASCULAR SYSTEM
1. Answer: B (Ref. Gray’s Anatomy for Students, 2nd edition, By Richard Drake, A. Wayne Vogl, Adam W. M. Mitchell, page
no. 958)

Artery Source Branches Supply to Notes


Common Brachiocephalic trunk External carotid a., Most of the Common carotid a. bifurcates at the
carotid (right), aortic arch (left) internal carotid a. head and the level of the superior border of the
upper neck thyroid cartilage; the internal carotid
a. and the external carotid a. are
its terminal brs.; the carotid sinus
and carotid body are located at the
bifurcation

2. Answer: A (Ref. Gray’s Basic Anatomy, 2nd edition, By Richard Drake, A. Wayne Vogl, Adam W. M. Mitchell, page no. 588)

Artery Source Branches Supply to Notes


Posterior superior Maxillary a. Dental brs., Maxillary molar teeth; Posterior superior alveolar a. enters
alveola mucosal brs. part of the maxillary the maxilla in the infratemporal fossa
sinus

https://t.me/DentalBooksWorld
568 Triumph’s Complete Review of Dentistry

3. Answer: A (Ref. Netter’s Head and Neck Anatomy for Dentistry, 3rd edition, By Neil S. Norton, page no. 132)
4. Answer: D (Ref. Gray’s Anatomy for Students, 2nd edition, By Richard Drake, A. Wayne Vogl, Adam W. M. Mitchell, page
no. 843)

Vein Source Branches Supply to Notes


Superior sagittal V. of the foramen Confluence of Cerebral Superior sagittal sinus occupies the
sinus cecum; superior sinuses hemispheres superior part of the falx cerebri;
cerebral vein lateral lacunae receive grossly visible
arachnoid granulations

5. Answer: C (Ref. Lippincott’s Concise Illustrated Anatomy: Head & Neck, Volume 3, By Ben Pansky, Thomas R. Gest, page
no. 124)

Vein Source Branches Supply to Notes


Diploic Multiple veins draining Meningeal vv.; dural Bones of the cranial vault It connects the venous
the diploe of the skull: venous sinuses; veins of system of the scalp with
Frontal diploic, anterior the scalp the dural sinuses and
temporal diploic, the meningeal vein. It is
posterior temporal absent at birth
diploic, occipital diploic

6. Answer: B (Ref. Gray’s Anatomy, 41st edition: The Anatomical Basis of Clinical Practice, edited by Susan Standring, page
no. 437)

Vein Source Branches Supply to Notes


Superior petrosal Cavernous sinus Sigmoid sinus, at All regions drained Superior petrosal sinus lies on the
sinus its proximal end by the cavernous petrous ridge within the dura mater at
sinus, including the line of attachment of the tentorium
the orbit a. cerebelli

7. Answer: A (Ref. Gray’s Anatomy for Students, 2nd edition, By Richard Drake, A. Wayne Vogl, Adam W. M. Mitchell, page
no. 958)
8. Answer: C (Ref. Gray’s Anatomy, 41st edition: The Anatomical Basis of Clinical Practice, edited by Susan Standring, page
no. 649)

Artery Source Branches Supply to Notes


Middle meningeal Maxillary artery Frontal br., parietal Most of the Middle meningeal a. passes through
br., petrous br., dura mater the foramen spinosum; it may be
superior tympanic (approximately torn by a fracture at the pterion; it
br. 80%), bones of the is encircled by the auriculotemporal
cranial vault nerve

9. Answer: A (Ref. Gray’s Anatomy, 41st edition: The Anatomical Basis of Clinical Practice, edited by Susan Standring, page
no. 662)

Artery Source Branches Supply to Notes


Posterior Ophthalmic artery No named Posterior Posterior ethmoidal a. leaves the
ethmoidal branches ethmoidal air cells; orbit through the posterior ethmoidal
olfactory nerves foramen

10. Answer: A (Ref. Netter’s Head and Neck Anatomy for Dentistry, 2nd edition, By Neil S. Norton, page no. 868)

https://t.me/DentalBooksWorld
Chapter 6 • Embryology, Head and Neck Anatomy, and General Histology 569

Structure Location Afferents from Efferents to Regions drained Notes


Jugulodigastric Anterolateral to Superior deep Inferior deep Oral cavity, A member of the superior
node internal jugular v. cervical nodes cervical nodes tongue, palatine deep cervical node group;
where it is crossed tonsil important node to palpate
by the posterior during physical examination
belly of the digastric

11. Answer: D (Ref. Netter’s Head and Neck Anatomy for Dentistry, 2nd edition, By Neil S. Norton, page no. 138)

Vein Source Branches Supply to Notes


Middle thyroid Numerous Internal jugular Lateral aspect of Middle thyroid vein crosses superficial
tributaries from vein the thyroid gland to the common carotid artery
the thyroid gland

12. Answer: C (Ref. Textbook of Anatomy: Volume 3: Head and Neck, Central Nervous System, 5th edition, By Inderbir Singh,
page no. 996)

Artery Source Branches Supply to Notes


Sphenopalatine Maxillary a. Posterior lateral nasal brs., Posterolateral nasal wall, Sphenopalatine a.
posterior septal brs. posteroinferior nasal septum, accompanies the
hard palate behind the maxillary nasopalatine n.
incisor teeth

13. Answer: D (Ref. Gray’s Anatomy, 41st edition: The Anatomical Basis of Clinical Practice, edited by Susan Standring, page
no. 453)

Artery Source Branches Supply to Notes


Ophthalmic Internal carotid a. Central retinal a., Optic nerve, optic Ophthalmic a. provides the only
lacrimal a., muscular chiasm optic tract, artery to the retina (central retinal
brs., anterior ethmoidal retina, extraocular a.)
a., posterior ethmoidal mm., eyelids,
a., medial palpebral forehead, ethmoidal
a., supraorbital a., air cells, lateral nasal
supratrochlear a., dorsal wall, dorsum of the
nasal a. nose
Internal Common carotid a. None in the neck; in Brain; eye and orbit; Internal carotid a. is the primary
carotid the head: Ophthalmic forehead blood supply to the brain; it
a., posterior anastomoses with the vertebral
communicating a., aa. and the contralateral internal
anterior cerebral a., carotid a. in the cerebral arterial
middle cerebral a. circle (of Willis); anterior and
middle cerebral aa. are the terminal
brs. of the internal carotid a.

14. Answer: D (Ref. Clinically Oriented Anatomy, 7th edition, By Keith L. Moore, Arthur F. Dalley, A. M. R. Agur, page no.
1003)

https://t.me/DentalBooksWorld
570 Triumph’s Complete Review of Dentistry

Artery Source Branches Supply to Notes


External carotid Common carotid Superior thyroid a., Upper neck, face, External carotid a. is the primary
ascending pharyngeal a., and scalp blood supply to the face and
lingual a., facial a., occipital superficial head; the maxillary a.
a., posterior auricular a., and superficial temporal a. are
maxillary a., superficial its terminal branches
temporal a.
Ascending External carotid a. Pharyngeal brs., inferior Pharynx, meninges Ascending pharyngeal a. arises
pharyngeal tympanic a., posterior from the medial side of the
meningeal a. external carotid a. close to the
bifurcation

15. Answer: D (Ref. Gray’s Anatomy for Students, 2nd edition, By Richard Drake, A. Wayne Vogl, Adam W. M. Mitchell, page
no. 963)
16. Answer: C (Whiplash: Evidence Base for Clinical Practice, By Michele Sterling, Justin Kenardy, page no. 33)

Artery Source Branches Supply to Notes


Vertebral Subclavian a. (first Spinal brs., muscular brs., Deep neck, cervical spinal Vertebral a. anastomoses
part) anterior spinal a., posterior cord, spinal cord; medulla with the internal carotid
inferior cerebellar a., (dorsal motor nucleus of a. in the cerebral arterial
medullary brs., meningeal cranial nerve X, nucleus circle (of Willis); it courses
brs., basilar a. ambiguus, spinal accessory through the transverse
nucleus, and hypoglossal foramina of vertebrae
nucleus) C1–C6

17. Answer: D (Ref. Gray’s Anatomy, 41st edition: The Anatomical Basis of Clinical Practice, edited by Susan Standring, page no. 411)

Artery Source Branches Supply to Notes


Facial External carotid a. Ascending palatine a., Lower part of the palatine The angular branch of
tonsillar br., submental a., tonsil, submandibular the facial a. anastomoses
superior labial a., inferior gland, facial muscles, and with the ophthalmic a.
labial a., lateral nasal a., fascia
angular a.
Facial, transverse Superficial No named branches Parotid gland, masseter Transverse facial a.
temporal a. m., facial muscles, and anastomoses with
skin branches of the buccal,
infraorbital, and facial
artery

18. Answer: D (Ref. Gray’s Anatomy, 41st edition: The Anatomical Basis of Clinical Practice, edited by Susan Standring, page no. 451)
Because scalenus medius is located posterior to the subclavian artery. So there is no need to cut this to expose the subclavian
artery.

Artery Source Branches Supply to Notes


Subclavian Brachiocephalic a. First part: Vertebral a., Neck, brain, spinal cord, Subclavian a. is continuous
(right), aortic arch thyrocervical trunk, thyroid gland, larynx, with the axillary a., the
(left) internal thoracic a.; Second shoulder, chest muscles, name change occurs at
part: Costocervical trunk; upper limb the lateral border of the
Third part: Dorsal scapular first rib; anterior scalene
a. (70%) muscle passes anterior to
the subclavian a., dividing it
into three parts

https://t.me/DentalBooksWorld
Chapter 6 • Embryology, Head and Neck Anatomy, and General Histology 571

19. Answer: A (Ref. Gray’s Basic Anatomy, 2nd edition, By Richard Drake, A. Wayne Vogl, Adam W. M. Mitchell, page no. 461)
20. Answer: A (Ref. Gray’s Basic Anatomy, 2nd edition, By Richard Drake, A. Wayne Vogl, Adam W. M. Mitchell, page no. 462)

Artery Source Branches Supply to Notes


Posterior External carotid a. Stylomastoid br., auricular Part of the external Posterior auricular and greater
auricular br., occipital br. ear, scalp, and deeper occipital share their region of
structures posterior distribution
to the ear
Occipital External carotid a. Sternocleidomastoid brs., Lateral neck, Occipital a. anastomoses with the
auricular br., mastoid br., posterior neck, deep cervical a.; it courses with
descending br., occipital posterior scalp the greater occipital nerve on the
brs. posterior surface of the head
Superior External carotid a. Infrahyoid br., Thyroid gland, upper Superior thyroid a. is usually
thyroid sternocleidomastoid br., part of the larynx, the first branch of the external
superior laryngeal a., infrahyoid mm., carotid a.
cricothyroid br., glandular sternocleidomastoid
brs. m.
Thyrocervical Subclavian a., first Inferior thyroid a., Lower neck, posterior Thyrocervical trunk does
trunk part transverse cervical a., shoulder, thyroid not give rise to the ascending
suprascapular a. gland cervical a.; it is a branch of the
inferior thyroid a.

21. Answer: C (Ref. “previous question no. 3”)


22. Answer: D (Ref. Gray’s Basic Anatomy, By Richard Drake, Richard Lee Drake, Wayne Vogl, Adam W. M. Mitchell, page no.
530)
23. Answer: D (Ref. “previous question no. 13”)
24. Answer: D (Ref. Gray’s Anatomy for Students, 2nd edition, By Richard Drake, A. Wayne Vogl, Adam W. M. Mitchell, page
no. 984)

Part of tongue Drainage


Tip of tongue Submental lymph nodes
Anterior 2/3rd Submandibular lymph nodes
Posterior 1/3rd Jugulo-omohyoid lymph nodes
Posterior most Retropharyngeal

Structure Location Afferents from Efferents to Regions drained Notes


Deep cervical In and around Lymphatic vessels Jugular trunk Head and neck Superior and inferior
nodes the carotid and numerous subdivisions of deep cervical
sheath posterior node groups from nodes are delineated by the
and lateral to the the head and crossing of the omohyoid m.
internal jugular v. neck
Lingual tonsil Superior surface Lymphatic vessels Superior deep “Guards” the Lingual tonsil is part of the
of the root of the of the tongue cervical nodes entrance of the tonsillar ring (of Waldeyer)
tongue oropharynx
Palatine tonsil Lateral wall of Lymphatic vessels Superior deep “Guards” the Palatine tonsil is part of the
the oropharynx of the posterior cervical nodes, entrance of the tonsillar ring (of Waldeyer)
between the tongue and especially the oropharynx
palatoglossal and palatoglossal/ jugulodigastric
palatopharyngeal palatopharyngeal node
arches arch region

https://t.me/DentalBooksWorld
572 Triumph’s Complete Review of Dentistry

Occipital nodes Superior nuchal Lymphatic Accessory Occipital part of Occipital nodes are two or
line, along the vessels from the nodes the scalp and the three nodes located between
course of the posterior head superior neck the attachments of the
occipital a. and v. and neck sternocleidomastoid m. and
the trapezius m.
Submental Under the Lymphatic vessels Submandibular Tip of the tongue, Submental nodes are
nodes mandible on the from the lower nodes, jugulo- lower lip, floor of important nodes to examine
mylohyoid m. face and chin omohyoid node the mouth, chin, in cases of oral cancer
gums, and lower caused by the use of tobacco
incisor teeth products

25. Answer: C (Ref. Netter’s Clinical Anatomy, 3rd edition, By John T. Hansen, page no. 71)
26. Answer: C (Ref. Gray’s Anatomy, 41st edition: The Anatomical Basis of Clinical Practice, edited by Susan Standring, page
no. 453)
27. Answer: C (Ref. Gray’s Anatomy for Students, 2nd edition, By Richard Drake, A. Wayne Vogl, Adam W. M. Mitchell, page
no. 916)
Endolymphatic duct: It is a part of membranous labyrinth (scala media)
• It is formed by union of saccule and utricle
• It connects scala media to subdural space
• Its terminal part is dilated to form the endolymphatic sac
• Endolymphatic sac lies between the two layers of dura on the posterior surface of the petrous bone
• Surgical importance: Endolymphatic sac is exposed for drainage or shunt operation in Meniere’s disease
Also know:
• Ductus reuniens—Connects cochlear duct to saccule
• Aqueduct of cochlea—Connects scala tympani to subarachnoid space
28. Answer: A (Ref. Gray’s Anatomy, 41st edition: The Anatomical Basis of Clinical Practice, edited by Susan Standring, page
no. 556)

Artery Source Branches Supply to Notes


Anterior ethmoidal Ophthalmic a. Anterior meningeal a., Anterior ethmoidal air Anterior ethmoidal a.
anterior septal br., anterior cells, frontal paranasal leaves the orbit through
lateral nasal br. sinus, dura mater in the the anterior ethmoidal
anterior cranial fossa, nasal foramen, then runs
mucosa, olfactory nerves forward on the cribriform
plate

29. Answer: B (Ref. “previous question no. 13”)

Artery Source Branches Supply to Notes


Ascending External carotid a. Pharyngeal brs., Pharynx, meninges Ascending pharyngeal a. arises from
pharyngeal inferior tympanic a., the medial side of the external carotid
posterior meningeal a. a. close to the bifurcation

30. Answer: A (Ref. Gray’s Anatomy for Students, 2nd edition, By Richard Drake, A. Wayne Vogl, Adam W. M. Mitchell, page
no. 820)
31. Answer: D (Ref. Textbook of Anatomy: Volume 3: Head and Neck, Central Nervous System, 5th edition, By Inderbir Singh,
page no. 996)
Facial artery
The arterial supply to the tonsil derives mainly from the tonsillar branch of the facial artery and the descending palatine
artery; veins drain into the pharyngeal plexus and the lymphatic drainage from the tonsil into the upper deep cervical
lymph nodes, which may enlarge during tonsillar infection.

https://t.me/DentalBooksWorld
Chapter 6 • Embryology, Head and Neck Anatomy, and General Histology 573

The tonsil is supplied by five arteries


1. Tonsillar branch of facial—this is the main artery
2. Ascending pharyngeal artery from external carotid
3. Ascending palatine—a branch of facial artery
4. Dorsal linguae branches of lingual artery
5. Descending palatine branch of the maxillary artery
32. Answer: B (Ref. Netter’s Head and Neck Anatomy for Dentistry, 2nd edition, By Neil S. Norton, page no. 170–171)
33. Answer: C (Ref. Netter’s Head and Neck Anatomy for Dentistry, 2nd edition, By Neil S. Norton, page no. 172)

Vein Source Branches Supply to Notes


Retromandibular v. Formed by the Anterior and posterior Side of the head Retromandibular v. is
union of the super- divisions of the retro- and scalp; deep contained within the parotid
ficial temporal v. mandibular v. face gland; anterior division
and maxillary v. unites with the facial v. to
form the common facial v.;
posterior division unites
with the posterior auricular
v. to form the external
jugular v.

34. Answer: C (Ref. “previous questions”)


35. Answer: C (Ref. Netter’s Head and Neck Anatomy for Dentistry, 2nd edition, By Neil S. Norton, page no. 174)

Vein Source Branches Supply to Notes


Inferior petrosal Cavernous Sigmoid sinus, at All regions drained by the Inferior petrosal sinus lies within
sinus sinus its distal end cavernous sinus, including the the dura mater along the inferior
orbit and the brain portion of the petrous part of the
temporal bone
Superior Cavernous Sigmoid sinus, at All regions drained by the Superior petrosal sinus lies on
petrosal sinus sinus its proximal end cavernous sinus, including the the petrous ridge within the dura
orbit and the brain mater at the line of attachment of
the tentorium cerebelli

36. Answer: B

Artery Source Branches Supply to Notes


Anterior inferior Basilar artery Labyrinthine artery Pons (motor nucleus of Anterior inferior cerebellar
cerebellar (usually) the cranial nerve V, chief a. shares its region of supply
sensory nucleus of the with branches of the basilar
cranial nerve V, abducens a.
nucleus, facial nucleus,
superior salivatory nucleus);
cerebellum; inner ear
Posterior inferior Vertebral Posterior spinal a. Part of cerebellum; Posterior inferior cerebellar
cerebellar artery medulla (cochlear nucleus, a. shares its region of supply
vestibular nucleus, dorsal with the vertebral a. and
motor nucleus of cranial anterior spinal a. (watershed
nerve X, nucleus ambiguus) region)
Superior Basilar a. No named branches Upper cerebellum; trochlear There may be more than
cerebellar nucleus one superior cerebellar a.
arising from the basilar a.
on each side

https://t.me/DentalBooksWorld
574 Triumph’s Complete Review of Dentistry

Middle cerebral Internal Lateral frontobasal a.; Frontal, parietal, and The middle cerebral a. is the
carotid artery prefrontal sulcal a.; temporal lobes, especially direct continuation of the
precentral sulcal a.; central on their lateral surfaces internal carotid a.
sulcal a.; anterior parietal
a.; posterior parietal a.;
anterior, middle, and
posterior temporal aa.

37. Answer: B (Ref. Netter’s Head and Neck Anatomy for Dentistry, 2nd edition, By Neil S. Norton, page no. 174)

Vein Source Branches Supply to Notes


Cavernous sinus Superior ophthalmic Superior petrosal Orbit, brain Cavernous sinus lies between the
v., cerebral vv., sinus, inferior cranial and meningeal layers of the
sphenoparietal sinus petrosal sinus dura mater beside the body of the
sphenoid bone; cranial nn. III, IV and
V1 are in its lateral wall; the internal
carotid a. and cranial n. VI are in its
lumen

38. Answer: B (Ref. Netter’s Head and Neck Anatomy for Dentistry, 2nd edition, By Neil S. Norton, page no. 174)

Vein Source Branches Supply to Notes


Ophthalmic, Ciliary vv., veins from the Superior Inferior portion of Valveless; inferior ophthalmic v.
inferior muscles of the inferior part ophthalmic v. the orbit connects with the pterygoid plexus
of the orbit through the inferior orbital fissure
Ophthalmic, Nasofrontal v., anterior Cavernous sinus Eyeball, superior Valveless; superior ophthalmic v.
superior ethmoidal v., posterior portion of the communicates with the angular v.
ethmoidal v., ciliary vv., orbit, ethmoidal
central retinal v., lacrimal v., air cells, forehead
inferior ophthalmic v.

39. Answer: A (Ref. Netter’s Clinical Anatomy, 3rd edition, By John T. Hansen, page no. 504)

Artery Source Branches Supply to Notes


Inferior thyroid Thyrocervical Ascending cervical a., Thyroid gland, lower Inferior thyroid a. gives
trunk inferior laryngeal a., larynx, upper trachea, rise to the ascending
esophageal brs., tracheal upper esophagus, deep cervical a. as it arches
brs., glandular brs. neck muscles medially

40. Answer: B (Ref. Clinically Oriented Anatomy, 7th edition, By Keith L. Moore, Arthur F. Dalley, A. M. R. Agur, page no. 882)
41. Answer: B (Ref. Clinically Oriented Anatomy, 7th edition, By Keith L. Moore, Arthur F. Dalley, A. M. R. Agur, page no. 845)
42. Answer: A (Ref. Gray’s Anatomy for Students, 2nd edition, By Richard Drake, A. Wayne Vogl, Adam W. M. Mitchell, page
no. 958)
43. Answer: C (Ref. Netter’s Clinical Anatomy, 3rd edition, By John T. Hansen, page no. 441)
44. Answer: B (Ref. Gray’s Anatomy for Students, 2nd edition, By Richard Drake, A. Wayne Vogl, Adam W. M. Mitchell, page
no. 965)
45. Answer: C (Ref. Netter’s Clinical Anatomy, 3rd edition, By John T. Hansen, page no. 504)
46. Answer: B (Ref. Gray’s Anatomy for Students, 2nd edition, By Richard Drake, A. Wayne Vogl, Adam W. M. Mitchell, page
no. 958)

https://t.me/DentalBooksWorld
Chapter 6 • Embryology, Head and Neck Anatomy, and General Histology 575

Artery Source Branches Supply to Notes


Thyroidea ima Brachiocephalic a. No named Isthmus of the Thyroidea ima a. is present in 10% of
or aortic arch branches thyroid gland, individuals; it is clinically significant
upper trachea because it is a midline artery in the region of
tracheostomy; ima means “lowest”

47. Answer: D (Ref. Netter’s Clinical Anatomy, 3rd edition, By John T. Hansen, page no. 503)
48. Answer: A (Ref. A Textbook of Anatomy: Volume 3: Head and Neck, Central Nervous System, 5th edition, By Inderbir
Singh, page no. 795)
Portal circulatory system
Circulation of blood to the liver from the small intestine, the right half of the colon, and the spleen through the portal vein;
sometimes specified as the hepatic portal circulation
49. Answer: A (Ref. Clinically Oriented Anatomy, 7th edition, By Keith L. Moore, Arthur F. Dalley, A. M. R. Agur,
page no. 882)
50. Answer: C (Ref. Clinically Oriented Anatomy, 7th edition, By Keith L. Moore, Arthur F. Dalley, A. M. R. Agur,
page no. 883)

Vein Source Branches Supply to Notes


Straight sinus Inferior sagittal sinus, Confluence of Deep parts of Straight sinus lies within the junction
great cerebral vein, sinuses the cerebrum, of the falx cerebri and tentorium
superior cerebellar vv. cerebellum cerebelli; also known as sinus rectus

51. Answer: D (Ref. Clinically Oriented Anatomy, 7th edition, By Keith L. Moore, Arthur F. Dalley, A. M. R. Agur,
page no. 883)
52. Answer: C (Ref. Clinically Oriented Anatomy, 7th edition, By Keith L. Moore, Arthur F. Dalley, A. M. R. Agur,
page no. 883)

Vein Source Branches Supply to Notes


Superior sagittal V. of the foramen Confluence of Cerebral Superior sagittal sinus occupies the
sinus cecum; superior sinuses hemispheres superior part of the falx cerebri;
cerebral vv. lateral lacunae receive grossly visible
arachnoid granulations

53. Answer: D (Ref. Clinically Oriented Anatomy, 7th edition, By Keith L. Moore, Arthur F. Dalley, A. M. R. Agur,
page no. 856)
54. Answer: B (Ref. Clinically Oriented Anatomy, 7th edition, By Keith L. Moore, Arthur F. Dalley, A. M. R. Agur, page no. 857)
55. Answer: B (Ref. Netter’s Clinical Anatomy, 3rd edition, By John T. Hansen, page no. 503)

Structure Location Afferents from Efferents to Regions drained Notes


Lymphatic duct Along the Submental Superior Anterior part of the Submandibular
andsubmandibular inferior nodes; facial deep cervical tongue, lower lip, nodes may be within
nodes border of the nodes; nodes; jugulo- floor of the mouth, the sheath of the
submandibular lymphatic omohyoid node nose, cheeks, chin, submandibular gland;
gland vessels gums and lower this group consists of
from the incisor teeth, lower from three to six nodes
submandibular surface of the palate
and sublingual
regions

https://t.me/DentalBooksWorld
576 Triumph’s Complete Review of Dentistry

Superficial parotid Superficial to Anterior Superior deep Anterior surface Superficial parotid
nodes the parotid auricular nodes cervical nodes of the ear and nodes are up to 10 in
gland and also external acoustic number and may be
deep to the meatus; temporal located superficial or
parotid fascia and frontal regions; deep to the parotid fasci
eyelids, lacrimal
gland, cheek, and
nose
Deep parotid nodes On the lateral Lymphatic Superior deep External acoustic Deep parotid nodes are
side of the vessels from cervical nodes meatus, auditory part of the deep cervical
pharyngeal the ear tube, middle ear chain of nodes
wall, deep to the
parotid gland
Retropharyngeal Posterior to the Lymphatic Superior deep Nasal fossae, Retropharyngeal nodes
nodes pharynx in the vessels from cervical nodes paranasal sinuses, are one or two in
retropharyngeal the nasal and hard palate, soft number; they are part
space pharyngeal palate, middle ear, of the deep cervical
regions oropharynx chain of nodes
Superficial cervical In superficial Lymphatic Varies by Head and neck Several groups are
nodes fascia and along vessels from group; ultimate designated by location:
superficial superficial destination occipital, retroauricular,
vessels of the structures in is the jugular anterior auricular,
head and neck head and neck trunk superficial parotid,
facial, submental,
submandibular, external
jugular, anterior jugular

56. Answer: D (Ref. Netter’s Clinical Anatomy, 3rd edition, By John T. Hansen, page no. 467)
Kiesselbach’s plexus, which lies in Kiesselbach’s area, Kiesselbach’s triangle, or Little’s area, is a region in the anteroinferior
part of the nasal septum where four arteries anastomose to form a vascular plexus.
The arteries are:
• Anterior ethmoidal artery and posterior ethmoidal artery (both from the ophthalmic artery)
• Sphenopalatine artery (terminal branch of the maxillary artery)
• Greater palatine artery (from the maxillary artery)
• Septal branch of the superior labial artery (from the facial artery)
57. Answer: D (Ref. Clinically Oriented Anatomy, 7th edition, By Keith L. Moore, Arthur F. Dalley, A. M. R. Agur, page no. 857)
58. Answer: D (Ref. Netter’s Clinical Anatomy, 3rd edition, By John T. Hansen, page no. 503)
59. Answer: B (Ref. Clinically Oriented Anatomy, 7th edition, By Keith L. Moore, Arthur F. Dalley, A. M. R. Agur, page no. 857)
60. Answer: A (Ref. Netter’s Clinical Anatomy, 3rd edition, By John T. Hansen, page no. 493)
61. Answer: B (Ref. Netter’s Clinical Anatomy, 3rd edition, By John T. Hansen, page no. 503)
62. Answer: D (Ref. Netter’s Clinical Anatomy, 3rd edition, By John T. Hansen, page no. 467)
63. Answer: A (Ref. Netter’s Head and Neck Anatomy for Dentistry, 2nd edition, By Neil S. Norton, page no. 88)
64. Answer: A (Ref. Netter’s Clinical Anatomy, 3rd edition, By John T. Hansen, page no. 503)
65. Answer: D (Ref. Netter’s Clinical Anatomy, 3rd edition, By John T. Hansen, page no. 503)

Structure Location Afferents from Efferents to Regions drained Notes


Jugulo- Lateral to internal Superior deep Inferior deep Submental region An important node to
omohyoid node jugular v. where it cervical nodes cervical and tip of tongue; examine in cases of oral
is crossed by the nodes head and neck cancer caused by use of
superior belly of the above this level tobacco products
omohyoid

https://t.me/DentalBooksWorld
Chapter 6 • Embryology, Head and Neck Anatomy, and General Histology 577

66. Answer: D (Ref. Netter’s Clinical Anatomy, 3rd edition, By John T. Hansen, page no. 503)
67. Answer: D (Ref. Netter’s Clinical Anatomy, 3rd edition, By John T. Hansen, page no. 503)
68. Answer: D (Ref. Gray’s Anatomy for Students, 2nd edition, By Richard Drake, A. Wayne Vogl, Adam W. M. Mitchell, page
no. 958)

Artery Source Branches Supply to Notes


Superior laryngeal Superior thyroid a. No named Internal aspect of Superior laryngeal a. penetrates the
branches the superior larynx thyrohyoid membrane in company
with the internal br. of the superior
laryngeal n.

69. Answer: C (Ref. Netter’s Clinical Anatomy, 3rd edition, By John T. Hansen, page no. 32)
70. Answer: C (Ref. Netter’s Clinical Anatomy, 3rd edition, By John T. Hansen, page no. 464)
71. Answer: D (Ref. Gray’s Anatomy for Students, 2nd edition, By Richard Drake, A. Wayne Vogl, Adam W. M. Mitchell, page
no. 958)
72. Answer: D (Ref. Netter’s Clinical Anatomy, 3rd edition, By John T. Hansen, page no. 462)
73. Answer: C (Ref. Clinically Oriented Anatomy, 7th edition, By Keith L. Moore, Arthur F. Dalley, A. M. R. Agur, page no. 857)
74. Answer: D (Ref. Clinically Oriented Anatomy, 7th edition, By Keith L. Moore, Arthur F. Dalley, A. M. R. Agur, page no. 857)
75. Answer: D (Ref. Clinically Oriented Anatomy, 7th edition, By Keith L. Moore, Arthur F. Dalley, A. M. R. Agur, page no. 856)
76. Answer: D (Ref. “previous question”)
77. Answer: D (Ref. Clinically Oriented Anatomy, 7th edition, By Keith L. Moore, Arthur F. Dalley, A. M. R. Agur, page no. 856)
78. Answer: B (Ref. Clemente’s Anatomy Dissector: Guides to Individual Dissections in Human, 3rd edition, By Carmine D.
Clements, page no. 406)
79. Answer: B (Ref. Clinically Oriented Anatomy, 7th edition, By Keith L. Moore, Arthur F. Dalley, A. M. R. Agur, page no. 882)
80. Answer: D (Ref. Clinically Oriented Anatomy, 7th edition, By Keith L. Moore, Arthur F. Dalley, A. M. R. Agur, page no. 882)
81. Answer: D (Ref. Clinically Oriented Anatomy, 7th edition, By Keith L. Moore, Arthur F. Dalley, A. M. R. Agur, page no. 882)
82. Answer: D (Ref. Clinically Oriented Anatomy, 7th edition, By Keith L. Moore, Arthur F. Dalley, A. M. R. Agur, page no. 882)
83. Answer: D (Ref. Netter’s Clinical Anatomy, 3rd edition, By John T. Hansen, page no. 74–75)
84. Answer: B (Ref. Clinically Oriented Anatomy, 7th edition, By Keith L. Moore, Arthur F. Dalley, A. M. R. Agur, page no. 883)

Vein Source Branches Supply to Notes


Diploic vv. Multiple veins draining the diploe Meningeal vv.; Bones of the Diploic vv. connect the
of the skull: frontal diploic, anterior dural venous cranial vault venous system of the scalp
temporal diploic, posterior temporal sinuses; veins of the with the dural sinuses and
diploic, occipital diploic scalp meningeal vv.

85. Answer: D (Ref. Clinically Oriented Anatomy, 7th edition, By Keith L. Moore, Arthur F. Dalley, A. M. R. Agur, page no.
856–857)
86. Answer: A (Ref. Clinically Oriented Anatomy, 7th edition, By Keith L. Moore, Arthur F. Dalley, A. M. R. Agur, page no.
882)

Artery Source Branches Supply to Notes


Anterior cerebral Internal carotid a. Anterior communicating Medial and inferior The anterior
a., medial frontobasal portions of the frontal communicating a. unites
a., polar frontal a., lobe; medial side of the the two anterior cerebral
callosomarginal a., parietal lobe; corpus aa. across the midline
precuneal a. callosum and part of the
limbic lobe; olfactory
bulb and tract; optic
nerve, optic chiasm and
optic tract

https://t.me/DentalBooksWorld
578 Triumph’s Complete Review of Dentistry

87. Answer: C (Ref. Netter’s Head and Neck Anatomy for Dentistry, 2nd edition, By Neil S. Norton, page no. 174)
88. Answer: A (Ref. Clinically Oriented Anatomy, 7th edition, By Keith L. Moore, Arthur F. Dalley, A. M. R. Agur, page no. 882)
89. Answer: C (Ref. Gray’s Anatomy for Students, 2nd edition, By Richard Drake, A. Wayne Vogl, Adam W. M. Mitchell, page
no. 963)
90. Answer: B (Ref. Netter’s Clinical Anatomy, 3rd edition, By John T. Hansen, page no. 504)
91. Answer: C (Ref. “previous question”)
92. Answer: D (Ref. Gray’s Anatomy, 41st edition: The Anatomical Basis of Clinical Practice, edited by Susan Standring, page
no. 498)
93. Answer: B (Ref. Clinically Oriented Anatomy, 7th edition, By Keith L. Moore, Arthur F. Dalley, A. M. R. Agur, page no. 882)
94. Answer: C (Ref. Netter’s Head and Neck Anatomy for Dentistry, 2nd edition, By Neil S. Norton, page no. 169)
95. Answer: A (Ref. Clinically Oriented Anatomy, 7th edition, By Keith L. Moore, Arthur F. Dalley, A. M. R. Agur, page no. 882)

Artery Source Branches Supply to Notes


Basilar Formed by the Pontine brs., anterior Pons (motor nucleus of cranial Basilar a. contributes
joining of the two inferior cerebellar a., nerve V, chief sensory nucleus of blood to the cerebral
vertebral aa. superior cerebellar a., cranial nerve V, abducens nucleus, arterial circle
two posterior cerebral aa. facial nucleus, superior salivatory
(terminal brs.) nucleus); oculomotor nucleus; nucleus
of Edinger–Westphal; cerebellum;
posterior cerebrum

96. Answer: D (Ref. Clinically Oriented Anatomy, 7th edition, By Keith L. Moore, Arthur F. Dalley, A. M. R. Agur, page no.
882)

Artery Source Branches Supply to Notes


Ophthalmic Internal carotid a. Central retinal a., lacrimal Optic nerve, optic chiasm, optic Ophthalmic a.
a., muscular brs., anterior tract, retina, extraocular mm., provides the only
ethmoidal a., posterior eyelids, forehead, ethmoidal air artery to the retina
ethmoidal a., medial cells, lateral nasal wall, dorsum (central retinal a.)
palpebral a., supraorbital a., of the nose
supratrochlear a., dorsal nasal a.

97. Answer: D (Ref. Netter’s Head and Neck Anatomy for Dentistry, 2nd edition, By Neil S. Norton, page no. 173)

Vein Source Branches Supply to Notes


Deep facial Facial v. Pterygoid venous Face Deep facial v. is valveless
plexus
Common facial Formed by the union of Internal jugular v. Face and anterior Common facial v. may receive
the anterior division of the scalp drainage from the lingual v.
retromandibular v. and the
facial v.

98. Answer: C (Ref. Netter’s Clinical Anatomy, 3rd edition, By John T. Hansen, page no. 505)
The position of the SAN was determined to be oriented lateral to the IJV, medial to the IJV, posterior to the IJV, or directly
through the IJV at the level of the posterior belly of the digastric muscle.
99. Answer: C (Ref. Netter’s Head and Neck Anatomy for Dentistry, 2nd edition, By Neil S. Norton, page no. 173)
100. Answer: A

MISCELLANEOUS
1. Answer: B (Ref. Netter’s Essential Histology, 2nd edition, By William K. Ovalle, Patrick C. Nahirney, page no. 203)
• Waldeyer’s tonsillar ring  (pharyngeal lymphoid ring  or  Waldeyer’s lymphatic ring) is an anatomical term collectively
describing the annular arrangement of lymphoid tissue in the pharynx.

https://t.me/DentalBooksWorld
Chapter 6 • Embryology, Head and Neck Anatomy, and General Histology 579

• Waldeyer’s ring circumscribes the naso- and oropharynx, with some of its tonsillar tissue located above and some below
the soft palate (and to the back of the oral cavity).
The ring consists of the (from superior to inferior):
• Pharyngeal tonsils  (or nasopharyngeal tonsil, due to the location; also known as adenoid(s) when inflamed/swollen.
They are located on the roof of the nasopharynx, under the sphenoid bone.
• Tubal tonsil  (bilaterally, where each  Eustachian tube  opens into the nasopharynx) (Usually develops from an
accumulation of lymphoid tissue in the pharyngeal tonsil)
• Two palatine tonsils (commonly called “the tonsils” in the vernacular, less commonly termed “faucial tonsils”; located in
the oropharynx; also see tonsillitis and tonsillectomy)
• One or many lingual tonsils (on the posterior tongue)
–– There normally is a good amount of mucosa-associated lymphoid tissue (MALT) present between all these tonsils
(intertonsillar) around the ring, and more of this lymphoid tissue can variably be found more or less throughout at
least the nasopharynx and the oropharynx.
2. Answer: C (Ref. Netter’s Clinical Anatomy, 3rd edition, By John T. Hansen, page no. 449)
Motor supply of eyelid:
• Eyelid muscle innervation is achieved by the facial nerve (VII), oculomotor nerve (III), and sympathetic nerve fibers.
• The facial nerve (CN VII) innervates the orbicularis oculi, frontalis, procerus, and corrugator supercilii muscles, and
supports eyelid protraction.
• Temporal and zygomatic branches of the facial nerve supply the orbicularis oculi, the main eyelid protractor.
• The facial nerve also supplies the corrugator supercilii and the procerus, both of which secondarily contribute to the
upper eyelid protraction.
• Oculomotor nerve (CN III) innervates the main upper eyelid retractor, the levator palpebrae superioris, via its superior
branch. Sympathetic fibers contribute to the upper eyelid retraction by innervation of the superior tarsal muscle, also
known as Miller’s muscle. Sympathetic fibers also innervate the inferior tarsal muscle, contributing to the lower lid
retraction.
Sensory supply of eyelid:
• Trigeminal nerve (V) supplies somatosensory innervation to the eyelid via its ophthalmic (V1) and maxillary (V2)
divisions.
• Terminal branches of the ophthalmic division supply the upper eyelid as the lacrimal, supraorbital, and supratrochlear
nerves (lateral to medial), and the medial aspect of both upper and lower lids as the infratrochlear nerve.
• Terminal branches of the maxillary division supply the lower eyelid as the zygomaticofacial and infraorbital nerves.
• The zygomaticofacial nerve supplies the lateral lower lid and the infraorbital nerve supplies the lower eyelid proper.
3. Answer: D (Ref. Netter’s Clinical Anatomy, 3rd edition, By John T. Hansen, page no. 471)
4. Answer: A (Ref. “previous questions”)
5. Answer: C (Ref. Netter’s Head and Neck Anatomy for Dentistry, 2nd edition, By Neil S. Norton, page no. 521)
• The nerve fibers forming the optic nerve exit the eye posteriorly through a hole in the sclera that is occupied by a mesh-
like structure called the lamina cribrosa.
• It is formed by a multilayered network of collagen fibers that insert into the scleral canal wall.
• The nerve fibers that comprise the optic nerve run through pores formed by these collagen beams.
6. Answer: D (Ref. Netter’s Head and Neck Anatomy for Dentistry, 2nd edition, By Neil S. Norton, page no. 132)
7. Answer: C (Ref. Netter’s Clinical Anatomy, 3rd edition, By John T. Hansen, page no. 108)
8. Answer: B (Ref. Netter’s Head and Neck Anatomy for Dentistry, 2nd edition, By Neil S. Norton, page no. 432)
9. Answer: C (Ref. Netter’s Clinical Anatomy, 3rd edition, By John T. Hansen, page no. 420)
10. Answer: B (Ref. Netter’s Clinical Anatomy, 3rd edition, By John T. Hansen, page no. 76)
11. Answer: A (Ref. “previous question”)
12. Answer: D (Ref. Netter’s Head and Neck Anatomy for Dentistry, 2nd edition, By Neil S. Norton, page no. 123)
13. Answer: D (Ref. “previous question”)
• Three small membranous tubes of the vestibular labyrinth within the bony semicircular canals of the bony labyrinth that
form loops of about two-thirds of a circle.
• The three semicircular ducts: anterior semicircular duct [TA] (ductussemicircularis anterior [TA]), lateral semicircular
duct [TA] (ductus semicircularis lateralis [TA]), and posteriorsemicircular duct [TA] (ductus semicircularis posterior
[TA] lie in planes at right angles to each other and open intothe vestibule by five openings of which one is common to
the anterior and lateral ducts.

https://t.me/DentalBooksWorld
580 Triumph’s Complete Review of Dentistry

14. Answer: B (Ref. Netter’s Head and Neck Anatomy for Dentistry, 2nd edition, By Neil S. Norton, page no. 368)
15. Answer: C (Ref. Netter’s Clinical Anatomy, 3rd edition, By John T. Hansen, page no. 504)
16. Answer: D (Ref. Netter’s Clinical Anatomy, 3rd edition, By John T. Hansen, page no. 504)
17. Answer: A (Ref. Netter’s Clinical Anatomy, 3rd edition, By John T. Hansen, page no. 477)
18. Answer: C (Ref. Netter’s Head and Neck Anatomy for Dentistry, 2nd edition, By Neil S. Norton, page no. 661)
19. Answer: B (Ref. Netter’s Head and Neck Anatomy for Dentistry, 2nd edition, By Neil S. Norton, page no. 190)
20. Answer: D (Ref. Netter’s Clinical Anatomy, 3rd edition, By John T. Hansen, page no. 418)
21. Answer: D (Ref. Netter’s Clinical Anatomy, 3rd edition, By John T. Hansen, page no. 508)
• The otic ganglion is a small parasympathetic ganglion located immediately below the foramen ovale in the infratemporal
fossa and on the medial surface of the mandibular nerve.
• It is functionally associated with the glossopharyngeal nerve and innervates the parotid gland for salivation.
• It is one of the four parasympathetic ganglia of the head and neck.
• The others are the ciliary ganglion, the submandibular ganglion, and the pterygopalatine ganglion.
22. Answer: A (Ref. “previous question”)
23. Answer: D (Ref. “previous question”)
24. Answer: B (Ref. ACSM’s Primary Care Sports Medicine, By Douglas McKeag, James L. Moeller, 2007, page no. 381)
• Ulnar nerve is called pinched nerve as it is most vulnerable to compression and entrapment
• Cubital tunnel syndrome—Compression of ulnar nerve “characteristic ulnar claw”
• Carpal tunnel syndrome—Compression of median nerve
25. Answer: A (Ref. Human Anatomy: A Clinically-Orientated Approach, By Sam Jacob, 2007, page no. 58)
Left anterior descending (LAD)—coronary artery—is named so because it gets occluded frequently and is most lethal.
26. Answer: A (Ref. Clinical Anatomy by Regions, By Richard S. Snell, 2008, page no. 808)
• Unilateral paralysis of recurrent laryngeal nerve causes dysphonia and hoarseness.
• Bilateral paralysis of recurrent laryngeal nerve causes airway collapse, stridor, aphonia, and laryngeal obstruction.
27. Answer: B (Ref. Stroke: A Practical Approach, 2009, page no. 7)
28. Answer: C (Ref. Color Atlas of Oral Diseases, By George Laskaris, 2003, page no. 42)
29. Answer: C
30. Answer: A (Ref. Fundamentals of Oral Histology and Physiology, By Arthur R. Hand, Marion E. Frank, 2014,
page no. 38)
31. Answer: C (Ref. Aneuploidy: Etiology and Mechanisms, By Vicki Dellarco, 2012, page no. 56)
32. Answer: C (Ref. Male Reproductive Dysfunction, By SC. Basu, 2011, page no. 203)
33. Answer: B
34. Answer: C (Ref. Cell Division Machinery and Disease, By Monica Gotta, Patrick Meraldi, 2017, page no. 5)

https://t.me/DentalBooksWorld
7 Oral Surgery

SYNOPSIS

ARMAMENTARIUM
1. Cheatle’s Forceps: Instrument used for picking up sterile instruments

• Long angulated instrument


• Stored in a container containing an antiseptic solution like savlon
• Solution must be changed every day

2. Swab Holder: Instrument used for preparing surgical field

Instrument with long blades, expanded at the ends, forming an oblong tip.
The blades have a central fenestration and a transverse serrations.
Uses
• Used to hold the swab and clean the area of operation
• Swab the throat when there are profuse secretions
• Press on the tonsillar bed to arrest hemorrhage
• Hold the tongue and give anterior traction → preventing tongue fall and airway obstruction in an unconscious patient

3. Scalpel: Instruments used for making an incision

Scalpel has two parts:


1. Blade
2. Blade handle

https://t.me/DentalBooksWorld
582 Triumph's Complete Review of Dentistry

Bard parker blade handle:


Most commonly used handle is no. 3.
Handle: Has a receiving slot for the blade.
While fitting, the blade is held with help of needle holder or an artery forceps to prevent injury to the operator.
The blade is then pushed into the slot till it fits in snugly.
Scalpel is always used in the pen grip.
Blades

NO. 10 NO. 11 NO. 12

NO. 15 NO. 20 NO. 21

NO. 22 NO. 23 NO. 24

No. 10 → for making skin incisions—cricothyrotomy


No. 11 → for making stab incisions, for draining abscess, for precision (supracrestal fibrotomy)
No. 12 → for mucogingival procedures
No. 15 → for intraoral surgery for abscess drainage

3. Towel Clips: Instrument used for holding the drapes


Two types of towel clips:
i. Pincher type → forceps type
ii. BECKHAUS towel clip
Tips of these instruments are pointed, curved toward each other and overlap one another.
i. Pincher type → box joint

ii. Beckhaus type → spring joint

Dissecting Scissors: Used for incising the tissues


Used to perform soft tissue dissection in the deeper layers.
Scissors have a blunt nose for undermining the tissues
A side cutting edge for cutting the tissues.

https://t.me/DentalBooksWorld
Chapter 7 • Oral Surgery 583

INSTRUMENTS USED FOR RETRACTING TISSUES


1. Langenbacks retractor
2. C-shaped retractor
3. Austins retractor
4. Obwegesers ramus retractor
5. Condyle retractors
6. Weiders retractor
7. Cat paw retractor
8. Tongue depressor
9. Seldins retractor

1. Langenbeck’s Retractor: Used to separate the edges of a surgical incision or wound

2. C-Shaped Retractor
Has a long handle but the blade is c shaped. More commonly used in abdominal surgeries.

3. Austin’s Retractor
*Short right-angled retractor
*Used for retracting cheek, tongue, and mucoperiosteal flaps

https://t.me/DentalBooksWorld
584 Triumph's Complete Review of Dentistry

4. Obwegeser’s Ramus Retractor


Similar to Langenbeck’s retractor except that the edge of retracting blade is forked. Forming a “v” shaped notch. To engage the
anterior border of ramus of mandible and aid in good tissue retraction.

Uses
Retract soft tissues along anterior border of ramus during sagittal split or ramus osteotomy
Retract the tissues from anterior border of ramus during coronoidectomy procedures

5. Condyle Retractors
Special retractors that have an appearance similar to the tongue depressor, but are narrower
And tip of the blade has a “C” shaped hook that is slipped under the ankylosed mass to retract and protect the medial soft
tissues during release of the ankylosis.

Instrument for retracting soft tissue

Sigmiod retractor

Condylar neck retractor

6. Weider’s Retractor
Broad, heart-shaped retractor that is serrated on one side.
So that it can engage the tongue and firmly retract it medially and anteriorly.

https://t.me/DentalBooksWorld
Chapter 7 • Oral Surgery 585

7. Cat Paw Retractor


Instrument resembles a cat’s paw → as the name suggests

blade has prongs that are curved at the tip. Used to retract small amounts of soft tissue.
Excessive force during retraction may lead to perforation or tear in the flap.

8. Tongue Depressor
Tongue depressor is an “l” shaped instrument with a broad smooth blade for depressing or retracting the tongue.

Used→Depress the tongue for visualization of the tonsils and pharyngeal wall during inspection. Depress the tongue during
endotracheal intubation and extubation. Depress the tongue and move it anteriorly to check for airway obstruction.
Prevent tongue fall in an unconscious patient.
To retract the tongue and cheek during surgical procedures.

9. Seldin’s Retractor
Instrument for retracting soft tissue

Seldin retractor

Minnesota retractor

SIMILAR TO PERIOSTEAL ELEVATOR


Here the leading edge is dull and not sharp.
Used only for retracting soft tissues and not for elevating the mucoperiosteal flap.

INSTRUMENTS USED FOR REFLECTING MUCOPERIOSTEAL FLAP


Moon’s Probe:
Thin, flat instrument that has a small working tip at right angles to the handle.
Tip is narrow and sharp.

Moon’s Probe

https://t.me/DentalBooksWorld
586 Triumph's Complete Review of Dentistry

PERIOSTEAL ELEVATORS
MOLT NO.9, HOWARTH’S, DIAL’S
These instruments are used for reflecting the mucoperiosteum.
Most of the periosteal elevators have a broad end on one side and a pointed or triangular end on the other.

USES
• Pointed end is used to release the interdental papillae.
• Broad end is used for elevating the mucoperiosteal flap from the bone.
• Broad end can also be used as a soft tissue retractor.

PERIOSTEAL ELEVATOR

2 Hourigan

9 Molt

Buser

CLEFT PALATE RASPATORY


Broad, flat handled elevator, especially used to elevate palatal mucoperiosteum while mobilizing the flaps for cleft palate repair.

HEMOSTATIC FORCEPS
• Spencer Wells, Kellys, Halstead.
• They used for catching both arteries and veins. They are used to catch hold of bleeding vessels.
• Unidirectional, transverse serrations on the blades of the hemostat prevent the vessel from slipping. Small bleeders may be
controlled by just crushing the vessel, whereas bigger vessels may be cauterized or ligated.
TYPES OF HEMOSTATS
• Large
• Medium
• Straight
• Curved
• Small
• A small curved artery forceps known as mosquito forceps

https://t.me/DentalBooksWorld
Chapter 7 • Oral Surgery 587

INSTRUMENTS USED FOR HOLDING THE SOFT TISSUES


1. Allis tissue holding forceps
2. Babcock’s tissue holding forceps
3. Lanes tissue holding forceps

DENTAL ELEVATOR
Elevators are used to loosen and “elevate” the teeth in their sockets prior to extraction. Care needs to be taken when using
elevators to avoid causing trauma to the adjacent teeth.
Parts of an Elevator
Working end
The working end is the functional, elevating and retracting end of the elevator
Single-ended
The working end is adapted to the function of the instrument

Shank
The area between the working end and the handle
The shank may be straight or angled for easy access to some areas in the mouth
The shank may also have a finger rest to enable the operator to get a better grip and apply more force
Handle
The part of the instrument that the operator holds
Designed for stability and leverage
The handle can be of many different varieties (i.e., serrated, smooth, hollow, solid, octagonal, round, large)

Elevator Based on Type of Blades Examples


Straight elevator
Straight elevator Coupland elevator
Straight apexo elevator
Triangular type Cryer elevator
Apex, crane pick, apical
Pick type
Pick

STRAIGHT/GOUGE TYPE
Most commonly used to luxate teeth
Blade—Concave surface on one side (which faces the tooth/concave on its working side). Blade at an angle from shank used
for posterior teeth—e.g., Miller’s and Pott’s elevator

https://t.me/DentalBooksWorld
588 Triumph's Complete Review of Dentistry

TRIANGULAR/PENNATE SHAPED
Second most common
Pairs—Right and left
Indication—When a broken root remains in socket and adjacent socket is empty; e.g., Cryer’s elevator
PICK TYPE
Indications—used to remove root
Hole is drilled 3 mm deep into the root, pick is inserted into the hole, root is elevated using buccal plate as fulcrum
e.g., Root tip pick/Apex elevator

THREE PRINCIPLES OF ELEVATORS


1. Lever principle
2. Wedge principle
3. Wheel and Axle principle
or combinations of the above
Lever principle
Most commonly used principle elevator is lever of the first-order fulcrum is between effort and resistance
In order to get mechanical advantage, effort arm must be longer than resistance arm, e.g., straight elevator, Cryer’s elevator,
Apexo elevator
Wedge principle
Wedge is a movable inclined plane which overcomes large resistance at right angles to the applied effort Wedge elevators is
forced between the root and the bone parallel to the long axis of the tooth.
e.g., Apexo elevator, Cryer’s elevator
Wheel and Axle principle
It is a modified form of lever principle
The effort is applied to the circumference of the wheel which turns the axle so as to raise a weight, e.g., Crossbar elevators
Warwick James Elevator/Hockey Stick Pattern
Blade is straight and at an angle to the shank Blade has convex and flat surface
Flat surface is working end and has transverse serrations on it for better contact with root stumps

Mechanical advantage calculation


Apexo elevator Lever and Wedge principle
Cryer’s elevator Lever, Wedge, and Wheel and Axle principle
Crossbar elevator Wheel and Axle principle

Principle Mechanical Advantage


Lever 3
Wedge principle 2.5
Wheel and Axle 4.6

https://t.me/DentalBooksWorld
Chapter 7 • Oral Surgery 589

SUTURE MATERIALS
Suture materials are used to bring together tissue or skin together in a wound or surgical laceration to aid in healing of the
wound.
Suture materials are divided into various types based on
• Absorbability
• Source
• Unifilament or Multifilament type

Advantages and Disadvantages of Unifilament and Multifilament Sutures


• Unifilament Suture helps in having a smooth surface which makes it easy to handle and reduces tissue trauma.
• Multifilament suture makes it easy to give knots because of their flexibility but they often act as a source of infection and
their braided nature of the suture can attract bacteria.

Absorbable Suture Material


Absorbable suture materials get absorbed by:
1. Digestion by proteolytic enzymes released from the polymorphonuclear cells
2. Hydrolysis whereby the action of water on the suture causes the breakdown of the suture material
In hydrolysis the action increases with rise in temperature or with a change in pH. The increase in absorption of the suture
material makes it loose its tensile strength
Natural Absorbable Suture Material
• Catgut
• Collagen
• Cargile membrane
• Kangaroo Tendon
• Fascia lata
Synthetic Absorbable Suture Material
• Dexon—Polyglycolic Acid
• Vicryl—Polyglactic Acid
• PDS—Polydioxanone
• Maxon—Polytrimethylene Carbonate

Nonabsorbable Suture Material


These are made up of materials which resist enzymatic digestion and therefore need removal when applied on any site on the
body. The nonabsorbable suture material is left in place for defined periods of time which is sufficient to heal the injury and
until the tension lasts for the specific material.
When nonabsorbable suture material is left in place without removing it can lead to:
• Excessive scarring
• It acts as a focus of infection
• Formation of stitch abscess
• Formation of stitch sinus
Natural Nonabsorbable Suture Material
• Silk
• Linen
• Cotton

https://t.me/DentalBooksWorld
590 Triumph's Complete Review of Dentistry

Synthetic Nonabsorbable Suture Material


• Nylon
• Polypropylene (Prolen, Surgilene)
• Braided Polyesters (Ethibond, Etiflex, Mersilene, Dacron)
• Polybutester (Novafil)

NONABSORBABLE SUTURES
Perma—Hand Surgical Silk
• Derived from cocoon of the silk worm larvae.
• Basically a protein like a keratin of hair and skin and is covered initially by an albuminous layer.
• This albuminous layer is removed by process of degumming prior to making of sutures.
• Suture is braided round a core and coated with wax to reduce capillary action.
• Material has high tensile strength → which gets totally lost after 2 years.
• Perma hand surgical silk is available as eyeless needled sterile suture in sizes 7–0 to 1–0.
• Reels as nonsterile sutures are available in sizes 5–0 to 3–0.
LINEN
• Made from flax and is cellulose material.
• It is a natural cellulose polymer.
• Twisted to form a fiber to make a suture.
• Tissue reaction is similar to silk. Material handles and knots well.
• **Gains 10% in tensile strength when wet.
• Very extensively used for tying pedicles and as ligatures.
COTTON
Derived from hair of seed of cotton plant.
**Like linen → it is twisted to form a suture.
• Tissue reaction is like silk and linen
• Tends to be a polymorphonuclear cellular type
• Weaker as compared to linen
• Handling is not as good as silk
• Polyamides
• Better known as nylon
• Chemically extruded and generally available in monofilament form
• Passage through the tissue is easy because of low coefficient of friction and tissue reaction is minimal
• Tensile strength loss after 1 year implantation is approximately 25%
• Nylon has a memory and hence knot security is lower than terylene
• Can be available in braided from also
Monofilament sutures are available in sizes ranging from 10–0 to 2–0.
POLYESTERS
• Fibers are better known as terylene or dacron
• Chemically extruded from polymer and braided to from sutures
• Has extremely high tensile strength, tends to be retained indefinitely
**Has become the suture of choice in cardiovascular surgical procedures**
• Teflon or e-PTFE coating to these polyester threads reduces the tendency to cut through the tissues
• E-PTFE → Expanded poly tetrafluoroethylene**
• This coating causes flaking in the tissues
• So, ethicon→introduced ethibond sutures
• Ethibond → polyester coated with polybutylate
• Provides excellent bonding and **does not flake **
• Does not increase the suture diameter

https://t.me/DentalBooksWorld
Chapter 7 • Oral Surgery 591

OTHER CLASSIFICATION
1. Absorbable
• Sutures that can be digested by body enzymes or hydrolyzed by tissue fluids
• Further classified into natural and synthetic
2. Nonabsorbable
• Sutures that cannot be digested by tissue enzymes and are encapsulated and walled off are nonabsorbable.
• Natural, metallic, and synthetic
a) Monofilament
b) Multifilament
3. Coated or noncoated
→ Coated: Example polyester sutures usually coated with biologically inert non resorbable compound.
→ This coating however makes knot security an issue
4. Based on thread diameter:
*3–0 → Usually used to secure flaps
*4–0 → Used closer to the flap edges to co-apt tension free flap edges

REQUISITES FOR SUTURE MATERIALS


1. Tensile strength
2. Tissue biocompatibility, low tissue irritation, and reaction
3. Low capillarity
4. Good handling and knotting properties
5. Sterilization without deterioration of properties

PRINCIPLES OF SUTURE MATERIAL


1. Rate of healing of tissues:
• Surgeon should select a suture that will lose its tensile strength at about the same rate that the tissues gain strength.
• Tissues that ordinarily heal slowly such as skin, fascia, and tendons should usually be closed with nonabsorbable sutures.
• Tissues that heal rapidly such as muscles, periosteum may be closed with absorbable sutures.
2. Tissue contamination:
Monofilament absorbable or nonabsorbable sutures are used in potentially contaminated wounds.
3. Cosmetic results:
Close and prolonged apposition of wounds and avoidance of irritants will produce the best results.
4. Microsurgical procedures:
Tissues most commonly approximated under microscope are arteries, veins, nerves, tendons, etc.
Most commonly used suture is 10-0 polyamide monofilament.
5. Cancer patients:
• Hypoproteinemia and chemotherapy can breakdown the wound.
• Synthetic nonabsorbable sutures are used.
• If the patient is to be irradiated in the postoperative period, monofilament polypropylene should not be used.
• Instead, polyester should be used.
6. Wound repair in patients following irradiation
7. Nutritional status
8. Suture size
Biological response of the body to suture materials

https://t.me/DentalBooksWorld
592 Triumph's Complete Review of Dentistry

SUTURE NEEDLES
Two types
1. Eyed
2. Eyeless
Suture needles usually are also classified according to their curvature, radius, and shape.
Surgical needle comprises of three parts:
1. Needle eye or swagged end → press fit
2. Needle body
3. Needle point

LOCAL ANESTHETICS
Local anesthetics are drugs which upon topical application or local injection cause reversible loss of sensory perception,
especially of pain in a localized area of the body
• These drugs act by blocking the conduction of nerve impulse along the axon. Small diameter and myelinated fibers are
blocked first whereas unmyelinated an d thick fibers are blocked at last
• The order of blockade of fibers is B, C, Aδ, and then Aα, β, and γ.
• Autonomic fibers are blocked first, then sensory (cold temperature sensation is lost first followed by heat, pain, and
proprioception) and finally motor are blocked at last. Order of recovery is in the reverse order.

Mechanism of Action
Entry of Na+ is essential for action potential
Two things happen:
• Rate and rise of AP and maximum depolarization decreases—slowing of conduction. Finally, local depolarization fails to
reach threshold potential—conduction block.
• LAs interact with a receptor within the voltage sensitive Na+ channel and raise the threshold of opening the channel—Na+
permeability decreased and ultimately stopped in response to stimulus or impulse—Impulse conduction is interrupted
when a critical length of fiber is blocked (2–3 nodes of Ranvier)

Vasoconstrictor Function
• Decrease blood flow to the site of injection
• Absorption of local anesthetic into the cardiovascular system is solved
• Decrease the risk of local toxicity
• Higher volume of local anesthetic agent remain in and around the nerve for longer period
• Thereby increasing the duration of action
• Vasoconstrictor decreases bleeding at the site of their administration

Classification of Local Anesthetic Agents


1. Esters
a) Esters of benzoic acid Cocaine (topical only)
Butacaine
Ethyl aminobenzoate
Benzocaine (topical only)
Piperocaine
Isobucaine
Meprylcaine

https://t.me/DentalBooksWorld
Chapter 7 • Oral Surgery 593

b) Esters of paba 2-Chloroprocaine (Nesacaine)


Procaine (Novocain)
Propoxycaine (Ravocaine)
Butethamine
Tetracaine (Pontocaine)
c) Esters Of Meta Amino Benzoic Acid Metabuthamine
Primacaine
2. Amides Lidocaine (Xylocaine)
Bupivacaine (Marcaine)
Mepivacaine (Carbocaine)
Dibucaine
Etidocaine (Duranest)
Articaine
Prilocaine
3. Quinolones Centbundine
4. Injectable
a) Low potency, short duration Procaine
Chloroprocaine
b) Intermediate potency and duration Lidocaine
Prilocaine
c) High potency, long duration Tetracaine
Bupivacaine
Dibucaine
5. Surface anesthetic
a) Soluble Cocaine
Lidocaine
Tetracaine
b) Insoluble Benzocaine
Butyl amino benzoate (butamben)
Oxethazaine
Note: All amide LA’s has two “i’s” to its name. the only LA that has two “i’s” but falls under ester group is “piperocaine”

Techniques of Mandibular Anesthesia:


Nerve block Other common Nerves Technique/Important landmarks Complications
names anesthetized
Inferior alveolar Mandibular Inferior alveolar 25-gauge long needle recommended • Hematoma is rare
nerve block block nerve Landmarks: • Transient facial
Incisive • Coronoid notch paralysis due to
Mental deposition of LA
• Pterygomandibular raphae solution into parotid
Lingual • Occlusal plane of mandibular gland
posterior teeth • Trismus
The needle approaches the IAN
approximately at right angle
Positive aspiration: 10–15%

https://t.me/DentalBooksWorld
594 Triumph's Complete Review of Dentistry

Buccal nerve Long buccal Buccal 25-gauge needle is recommended Hematoma


block nerve block, Soft tissue buccal to mandibular molar
buccinator teeth is anesthetized
nerve block
Landmarks:
• Mandibular molars and mucobuccal
fold
Penetrate distal and buccal to last molar
Syringe aligned parallel to occlusal
plane
Positive aspiration: 0.7%
Gow gates Third division Inferior alveolar 25-gauge needle is recommended Hematoma and
technique nerve block nerve Target area: trismus—Rare
Incisive Lateral side of the condylar neck, just Temporary paralysis
Mental below the insertion of lateral pterygoid of cranial nerves—3,4,
muscle and 6—diplopia,
Lingual blepharoptosis, and
Mylohyoid Landmarks: complete paralysis of
Auriculotemporal • Extraoral: Lower border of tragus eye persists

Buccal (in 75% of • Intraoral: Mesiolingual cusp of


patients) maxillary second molar
Penetration of soft tissue just distal
to the soft tissues of maxillary second
molar
Position:
For right GGMNB—8’ o clock position
facing the patient
For left GGMNB—10’ o clock position
facing the same direction as the patient
Positive aspiration: 2%
Vazirani Closed-mouth Inferior alveolar 25-gauge needle is recommended Hematoma and
Akinosi: mandibular nerve Target area: trismus—rare
Closed mouth nerve block Incisive Soft tissue on the medial (lingual) border Temporary paralysis of
mandibular Tuberosity Mental of the ramus of the region of the inferior facial nerve
nerve block technique alveolar, lingual, and mylohyoid nerves
Lingual
Mylohyoid Landmarks:
• Mucogingival junction of maxillary
3rd molar (or second molar)
• Maxillary tuberosity
• Coronoid notch on the mandibular
ramus
Orientation of bevel is very important:
The bevel must be oriented away from
the bone of mandibular ramus (bevel
faces towards the midline)
Position:
8’ o clock position facing the patient

https://t.me/DentalBooksWorld
Chapter 7 • Oral Surgery 595

Mental nerve - Mental nerve 25- or 27-guage needle is recommended Hematoma


block Soft tissue anesthesia (while incisive
nerve block gives pulpal anesthesia for
incisors, canine, and premolars)
Landmarks:
• Premolars and mucobuccal fold
Orientation of bevel—toward the bone
during the injection
Positive aspiration—5.7%

Recommended volume of local anesthetic solution:


Technique Volume (mL)
Inferior alveolar 1.5
Buccal 0.3
Gow-Gates 1.8
Vazirani-Akinosi 1.5–1.8
Mental 0.6
Incisive 0.6–0.9

Newer Techniques of LA:


• Iontophoresis: Lidocaine-soaked sponges
• Liposomes: Liposomal Bupivacaine Formulation

Duration of action of various Local anesthetic agents used in dental practice:


• Etidocaine: 200 minutes
• Bupivacaine: 175 minutes
• Tetracaine: 175 minutes
• Lignocaine: 175 minutes (commonly used for tooth extraction and also as a topical local anesthetic agent)
• Procaine: 50 minutes (used in children because of the short duration of action, which prevents self-inflicting injuries to the
lip under anesthesia)

Calculation of dosage in LA:


Local Anesthesia: Maximum dose of LA with adrenaline—7.7 mg/kg (for normal adult considering 60 to
70 kg weight, 500 mg max)
Maximum dose of LA without adrenaline—4.4 mg/kg (here its 300 mg max)
What is meant by 2% lignox? 2 g in 100 mL.
(Convert units—2,000 mg in 100 mL. So 20 mg in 1 mL)
How many cartridges are safe to We give 1.5 mL (according to Stanley F Malamad) of solution in normal IANB block.
administer? So if 1 mL contains 20 mg, 1.5 mL contains 30 mg. So when we give an IANB block,
we give 30 mg of LA.
We have already saw Max dose is 500 mg.
So 30 × 16 is 480. So for an adult we can give 14–16 cartridge of 1.5 mL IANB block
(till this level it is safe)

https://t.me/DentalBooksWorld
596 Triumph's Complete Review of Dentistry

Adrenaline What is meant by 1:80,000?


1 g of adrenaline in 80,000 parts diluted
So 1,000 mg:80,000 mL
So 1,000/80,000 is 0.0125,mg of adrenaline is present in LA
Safety Dose of Adrenaline for Dental Use in normal patients is 0.2 mg, which means 20
mL of LA can be given to normal patients containing Adrenaline
For cardiac patients the safety dose of Adrenaline is 0.04 mg – Which means 4 ml of LA
can be given to patients with cardiac problems containing Adrenaline
If the concentration of LA is 1:50,000 – 10 mL of LA can be given safely

Complications/Adverse Effects may be further divided into 2 groups:


1. Those attributed to the solutions used:
This group includes those complications that result from the absorption of the anesthetic solution:
• Toxicity
• Idiosyncrasy
• Allergy
• Anaphylactoid reactions
• Infections caused by contaminated solutions
• Local irritations or tissue reactions caused by the solution
2. Those attributed to the insertion of the needle:
This group includes those complications attributed to the insertion of the needle:
• Syncope (fainting)
• Muscle trismus
• Pain or hyperalgesia
• Edema
• Infectious
• Broken needles
• Prolonged anesthesia other than from the anesthetic solution
• Hematoma
• Sloughing
• Neurological complications

Complications based on Local and Systemic Effects:


1. Local Complications:
• Needle Breakage
• Pain on injection
• Burning on injection
• Persistent anesthesia (paresthesia)
• Trismus
• Hematoma
• Infection
• Edema
• Slugging of tissue
• Soft tissue injury (lip, cheek, tongue, palate)
• Facial nerve paralysis
• Postoperative intraoral lesions—Recurrent Aphthous Stomatitis, Herpes Simplex
• Visual disturbance

https://t.me/DentalBooksWorld
Chapter 7 • Oral Surgery 597

• Intravascular injection
• Failure to obtain anesthesia
2. Systemic Complications:
• Toxicity due to overdose
• Allergy
• Idiosyncrasy
• Syncope
• Drug interaction
• Serum hepatitis
• Occupational dermatitis
• Respiratory arrest
• Cardiac arrest
• Hyperventilation

EXODONTIA
• Extraction of teeth is a minor surgical procedure involving the bony and soft tissues of the oral cavity. Extraction is one of
the most common surgical procedure performed in dental office.
• It involves severing of periodontal attachment of tooth followed by elevation of tooth out of the socket by making use of
elasticity of bone.
Indications:
Usually dental caries and periodontal pathology account for 85–90% of the extractions.
1. Periodontal disease:
2. Dental caries and its sequelae
3. Nonvital teeth with acute or chronic pulpitis
4. Teeth with infected pulp that has led to periapical disease and are not treated by endodontic procedures with or without
apicoectomy
5. Teeth mechanically interfering with placement of partial dentures and bridges
6. Overretained deciduous teeth that may deflect or prevent normal eruption of permanent teeth
7. Therapeutic extraction
8. Malposed teeth whose realignment not possible
9. Serial extractions
10. Retention of impacted and unerupted tooth
11. Supernumerary teeth
12. Teeth in line of fracture
13. Teeth with fractured roots especially coronal half
14. Potentially infected roots and root fragments are indicated for extraction even if asymptomatic
15. Teeth causing bony pathology
16. Tooth involved in cyst formation
17. Extraction of decayed first and second molars → in order to permit eruption of erupting third molar in selected cases
18. Patients of oral malignancy where radiation therapy is to be given
19. Teeth responsible for focal sepsis causing systemic disorders
Contraindications:
Local factors
1. Acute infection with uncontrolled cellulitis
2. Acute pericoronitis
3. Acute infections like gingivitis and stomatitis

https://t.me/DentalBooksWorld
598 Triumph's Complete Review of Dentistry

4. Extraction of maxillary premolars and molars is contraindicated in acute maxillary sinusitis


5. Tooth embedded in malignant growth is not extracted → leaving anon healing growth
6. Extraction of tooth from irradiated jaw may develop osteoradionecrosis → due to low vascularity
Systemic factors
Absolute and relative contraindications
1. Uncontrolled diabetes mellitus
2. Cardiac problems
3. Pregnancy
4. Bleeding disorders
5. Medically compromised patients
6. Patients on steroid therapy
7. Renal failure
8. Psychosis and neurosis
9. Patients on anticoagulant therapy
10. Patients with liver disorders
11. Patients with toxic goiter

PRINCIPLES OF ELEVATION: These are for luxation**


1. Wedge principle: Example → straight elevator
Wedging between the bony socket and the root
2. Principle of lever of first order: Example → forceps
**Weight × weight arm → power × power arm
3. Wheel and axle: Example → cross bar
Represented by pulley mechanism
**Greater the diameter of wheel more is the mechanical advantage
This principle results in bodily rotation of tooth → easy to deliver the tooth out of the socket

First Class Levers

Effort = 2
Load = 4 Effort = 4 Load = 4 This end of the lever moves twice
as far as the other end moves.

Fulrcrum Center Fulcrum OffCenter

Second Class Levers

Effort applied 3 times one third


the force further from fulcrum:
three mes the distance Effort 2

Third Class Levers

Load will move 3 mes the distance that the


Load = 2 effort is applied moves at a greater speed

Effort 2

https://t.me/DentalBooksWorld
Chapter 7 • Oral Surgery 599

Following forces hold good for forces used for extraction


1. Apical pressure
2. Buccal force
3. Lingual pressure
4. Rotational force
5. Traction force

EXTRACTION TECHNIQUE
Can be either closed or open extraction technique
1. INTRAALVEOLAR EXTRACTION: CLOSED EXTRACTION→forceps extraction**
• Correct elevators and forceps must be chosen
• No injudicious force should be applied
• Do not grasp the forceps near the beaks
• Hold the forceps firmly with the full palm grip
• **Long axis of the forceps beaks should be parallel to the long axis of tooth
• Forceps should be placed on sound root and never on enamel of the crown.
Beaks of forceps should engage only the tooth to be extracted → else the luxation of adjoining tooth is endangered
2. OPEN EXTRACTION
a. Separate the gingiva with NO.12 B.P. blade or a fine probe using gingival margin as the incising guide.
b. Carry this incision interproximally to the crest of inter dental papilla
c. Then the beaks of the forceps are inserted under the gingival margin reaching the neck of the tooth to rest on cementum
using an apical force
d. Then the pressure is applied which differs in location and its kind based on the tooth
Open extraction sequence:
1. Anesthesia
2. Elevation of mucoperiosteal flap
3. Removal of bone
4. Division of tooth, if required
5. Removal of tooth and roots
6. Control of bleeding
7. Alveoloplasty → if required
8. Toilet of the alveolar socket
9. Suturing the flap
3. EXTRACTION FOR DECIDUOUS TEETH:
• Generally deciduous teeth are easier to extract than the permanent teeth.
• Because of their root resorption.
• But, there is danger of damage to the underlying permanent tooth or tooth crypt.
• SPLIT TECHNIQUE should always be considered for extraction of deciduous molars.

EXTRACTION TECHNIQUE FOR INDIVIDUAL TEETH


Type of teeth Maxillary incisor Mandibular incisor
Central incisor Slight labial and palatal pressure with mesial Labial and lingual rocking and slight mesial
rotation followed by slight traction. and distal rotation followed by removal to
labial side.
Lateral incisor Slight labio palatal rocking with mesial rotation Similar to mandibular central incisor
is used followed by traction.

https://t.me/DentalBooksWorld
600 Triumph's Complete Review of Dentistry

Canines Difficult to remove→ longer root Labial pressure with mesial rotation and vertical
Labial pressure →lingual pressure → then pull and vertical pull yields desired result
again labial pressure with mesial rotation
followed by traction
First premolar Buccal pressure → palatal pressure → and then
extract out to buccal side. Buccal and lingual rocking with removal to
**no torsion is to be used buccal side
Second premolar Buccal pressure and lingual pressure with slight Buccal pressure with slight mesiodistal rotation
rotational force is used
Buccal beak of forceps has got a projecting tip
that fits between buccal roots
First molar Buccal and lingual rocking is done prior to Buccal and lingual rocking with removal to
removal on buccal side buccal side
Second molar Removal is similar to first molar Similar to first molar
Third molar Buccal pressure along with distal and Buccal and lingual rocking with removal to
downward movement is used to luxate tooth buccal or lingual side
out of socket

Primary molars Primary anteriors


Best removed by buccolingual rocking and removal to Removed by labial pressure and mesial rotation with removal to
lingual side labial side

ROOT EXTRACTIONS:
Roots not decayed deep into bone → easily extracted by using forceps only.
Bayonet forceps → used for roots in upper jaw.
Forceps used for incisors or premolars → used for roots in lower jaw.
**roots should be grasped as far as possible under the gum margins without injuring the alveolar bone and gingiva.
Deeply embedded roots should be removed by open method.
TRANSALVEOLAR EXTRACTION: Open extraction → surgical extraction.
Surgical extraction of tooth by reflection of an adequate mucoperiosteal flap and removal of the bone followed by tooth
removal.
Advantages:
Removal of tooth lying in difficult positions without damaging neurovascular bundle.
Fracture of bone is avoided.
Less danger of creating an oroantral fistula → which means less chances of tearing soft tissues and fracture of large pieces of
alveolar bone.
Indications:
1. Tooth that resist extraction by closed method
2. Unerupted tooth that cannot be removed by closed method
3. Fractured tooth or roots below the level of epithelial attachment
4. Hypercementosed
5. Fused, dilacerated or locked roots
6. Widely divergent roots especially the permanent molars
7. Teeth with complicated and unfavorable root curvature
8. Teeth with post crowns or very large root canal fillings
9. Teeth whose roots are dipping in to maxillary antrum and has no intervening bone
10. Ankylosed roots → mostly in elderly patients

https://t.me/DentalBooksWorld
Chapter 7 • Oral Surgery 601

IMPACTION
Impaction is cessation of eruption of a tooth caused by a physical barrier or ectopic positioning of a tooth
LOCAL CAUSES
• Lack of space between second molar and ramus
• Overretained deciduous teeth
• Ectopic position
• Dilaceration of roots
• Inclination of tooth
• Associated soft tissue or bony pathology
• Heredity
• Endocrinal causes
SYSTEMIC CAUSES
Prenatal
• Heredity
• Miscegenation
Postnatal Causes
• Rickets
• Anemia
• Congenital syphilis
• Tuberculosis
• Endocrine dysfunctions
• Malnutrition

THEORIES BEHIND IMPACTED TEETH


Phylogenic → Jaw size becoming smaller based on evolution
Mendelian → Genetically receives small jaw from parents

REFERENCE LINE DURING RADIOLOGICAL EXAMINATION


W-White line

R-Red line

A-Amber line

White line → Corresponds to occlusal plane


**This line is drawn touching the occlusal surfaces of first and second molar
Then extended posteriorly over third molar region.
Indicates the difference in occlusal level of second and third molars.
Amber line → Represents the bone level.
Line is drawn from the crest of interdental septum between the molars and extended posteriorly distal to third molar or to the
ascending ramus.
Red line → Drawn perpendicular from amber line to an imaginary point of application of elevator.
Indicates the amount of bone that will have to be removed before elevation.

https://t.me/DentalBooksWorld
602 Triumph's Complete Review of Dentistry

CLASSIFICATION OF IMPACTED TEETH


1. Classification describes the general position of impacted tooth
2. Aids in estimating the difficulty in removal
Difficulty index:
Very difficult → 7–10
Moderately difficult → 5–7
Minimally difficult → 3–4
WINTERS CLASSIFICATION
ANGULATION: Based on the position of impacted third molar to the long axis of second molar
Winter’s Classification (1926):
• Based on angulations: According to the position of the impacted third molar to the long axis of second molar
–– Mesioangular: 45%
–– Horizontal: 10%
–– Vertical: 40%
–– Distoangular: 5%
BASED ON ANGULATION
1. Mesioangular
Most Common Finding
43% Cases In Mandibular Impaction
63% Cases In Maxillary Impaction
Easiest To Remove
2. Horizontal/Transverse/Inverted
3. Vertical
4. Distoangular
5. Buccoangular
6. Linguoangular
DEPTH: Based on the relationship to the occlusal surface of adjoining second molar of impacted maxillary
or mandibular third molar.
2nd Molar

3rd Molar/
Wisdom Tooth
Coronal 1/3 Easy
Middle 1/3 Moderate
Point of
Apical 1/3 Difficult
Applicaon

Diagram illustrang the method of relang the point of applicaon of


an elevator to the roots of the lower 2nd molar to assess the depth
of the 3rd molar in the alveolar bone.
Position A:
Highest position of tooth is on a level with or above occlusal line.
Position B:
Highest position is below the occlusal plane, but above the cervical level of second molar.
Position C:
Highest position of tooth is below the cervical level of second molar.

https://t.me/DentalBooksWorld
Chapter 7 • Oral Surgery 603

Pell and Gregory classification (Impacted 3rd molar)

Mesial Horizontal Distal


Impaction Impaction Impaction

Gum layer Gum layer

Vertical So Tissue Vertical Bony Vertical


Impaction Impaction Impaction

• Based on the amount of tooth covered by the anterior border of the ramus (1, 2, or 3).
• The depth of the impaction relative to the adjacent tooth (A, B, or C).
Helpful in predicting surgical difficulty
3rd molar impaction can be:
Class I
Class I 3rd molar impaction: Situated anterior to the anterior border of the ramus.

Class II
Class II  3rd molar impaction:  Crown ½ covered by the anterior border of the
ramus.

Class III
Class III 3rd molar impaction: Crown fully covered by the anterior border of the
ramus.

Class A
Class A maxilla 3rd molar impaction: The occlusal plane of the impacted tooth is A
at the same level as the adjacent tooth.

https://t.me/DentalBooksWorld
604 Triumph's Complete Review of Dentistry

Class A
Class A mandible 3rd molar impaction: The occlusal plane of the impacted tooth
is at the same level as the adjacent tooth.

Class B
Class B maxilla 3rd molar impaction: The occlusal plane of the impacted tooth is B
between the occlusal plane and the cervical line of the adjacent tooth.

Class B
Class B mandible 3rd molar impaction: The occlusal plane of the impacted tooth
is between the occlusal plane and the cervical line of the adjacent tooth.

Class C
Class C maxilla 3rd molar impaction: The occlusal plane of the impacted tooth is C
apical to the cervical line of the adjacent tooth.

Class C
Class C mandible 3rd molar impaction: The occlusal plane of the impacted tooth is
apical to the cervical line of the adjacent tooth.

MAXILLARY THIRD MOLARS CLASSIFICATION


**Angulation and depth classification is same as mandibular third molars.
Based On The Relationship With Floor Of Sinus:
1. Sa → Sinus Approximation → No Bone Or A Thin Bony Partition Present Between Impacted Maxillary Third Molar And The
Floor Of The Maxillary Sinus.
2. Nsa → 2 mm Or More Bone Is Present Between The Sinus Floor And The Impacted Maxillary Third Molar

COMMONLY IMPACTED TEETH


1. Mandibular third molars
2. Maxillary third molars
3. Maxillary canine
4. Mandibular premolar
5. Maxillary premolar
6. Mandibular canine
7. Maxillary central incisors
8. Maxillary lateral incisors

https://t.me/DentalBooksWorld
Chapter 7 • Oral Surgery 605

MID FACE FRACTURE


BONES CONSTITUTING THE MIDDLE THIRD OF THE FACE
8 Paired And 2 Unpaired Bones → In Mid Face Region.
2 Maxillae
2 Palatine bones
2 Zygomatic bones and their temporal processes
2 Zygomatic processes of temporal bones
Two nasal bones
Two lacrimal bones
2 Inferior conchae
2 Pterygoid plates of sphenoid
Vomer Unpaired
Ethmoid and its Attached Conchae Unpaired

PHYSICAL PROPERTIES OF MID FACE


Fragile bones
Articulated in a complex fashion
Acts as a cushion for trauma directed toward the cranium
Fractures of this area are generally comminuted.
Crushed inward → causing “typical dish face deformity”

VERTICAL AND HORIZONTAL PILLARS OF MAXILLARY SKELETON


Vertical Pillars:
1. Anterior or Canine Pillars
**Starts in the region of alveolar process of canine
2. Middle or Zygomatic Pillars
Starts in the region of first molar
3. Posterior or Pterygoid Pillar
Is the pterygoid process of sphenoid bone to which the pyramidal process of palatine bone is anchored.
++ These vertical pillars are braced against each other by superior and inferior orbital rims and the zygomatic arch.
Horizontal Pillars:
1. Supraorbital rims with frontal bone
2. Infraorbital rims
3. Alveolar process

CLASSIFICATION:
Le Fort via his experimental studies, discovered the complex fracture patterns.
And broadly subdivided it into three groups:
Le Fort 1
Le Fort 2
Le Fort 3
Erichs 1942, as per the direction of fracture lines,
1. Horizontal fracture
2. Pyramidal fracture
3. Transverse fracture

https://t.me/DentalBooksWorld
606 Triumph's Complete Review of Dentistry

Depending on the relationship of fracture line to zygomatic bone


Subzygomatic
Suprazygomatic
Level of fracture:
Low level
Mid level
High level

Le Fort 1
Low level, subzygomatic fracture.
Aka => Guerin’s fracture, horizontal fracture
Floating fracture =→ due to complete pterygomaxillary dysjunction
**Fractured fragment is freely mobile and the resultant displacement will depend on the direction of the force
Depending on the displacement of a fragment, variety of occlusal disharmony can be seen in this type of fracture
Signs and symptoms:
• Slight swelling and edema of lower part of the face, with upper lip swelling
• Ecchymosis in the labial and buccal vestibule
• Contusion/laceration in upper lip
BILATERAL EPISTAXIS OR NASAL BLEEDING OBSERVED
Percussion of maxillary teeth produces dull “cracked cup” sound
Clinical Anterior Open Bite
Impacted Or Telescopic Fracture

Le Fort II: Pyramidal or subzygomatic fracture


Signs and Symptoms:
• Dish face
• Gross edema of middle third of face → ballooning or moon face
• Presence of bilateral circumorbital edema and ecchymosis → Black eye
• Bilateral subconjunctival hemorrhage → confined to medial half of eye
• Nasal disfigurement→ bridge of the nose will be depressed →flat face
• CSF leak may present
• “Step deformity”** at infraorbital margins
• Anesthesia or paresthesia of cheek is noted

Le Fort III:
• High level fracture
• **Line of fracture extends above the zygomatic bones on both sides
• Here it is “craniofacial dysjunction”
• Initial impact is taken by zygomatic bone
• Entire middle third will hinge about fragile ethmoid bone
• And then the impact transmitted on the contralateral side resulting in laterally displaced zygomatic fracture of opposite side

https://t.me/DentalBooksWorld
Chapter 7 • Oral Surgery 607

SIGNS AND SYMPTOMS


• Panda facies
• Raccoon eyes
• Bilateral subconjunctival hemorrhage → “without posterior limit”
• Epistaxis, CSF rhinorrhea
• Enophthalmos, diplopia or impairment of vision, temporary blindness
UNILATERAL/BILATERAL HOODING OF EYES
Due to separation of frontozygomatic sutures.

Lengthening of face

Lowering of ocular level

Hooding of eyes

MANDIBULAR FRACTURES
Fracture of mandible occurs more frequently than any other fracture of facial skeleton
Broadly divided into two main groups:
1. Fractures with no gross comminution of bone and without significant loss of hard or soft tissue
2. Fractures with gross comminution of bone and with extensive loss of both hard and soft tissue

Classification:
1. Type of fracture
2. Site of fracture
3. Cause of fracture
S. No. Classification Types Description
Based on
1. Type of Simple Includes closed linear fractures of coronoid, ramus, condyle, and edentulous
fracture body of mandible
Green stick fracture is a closed variant of simple fracture
Found exclusively in children
Compound Fractures of tooth bearing portions of mandible, via periodontal membrane
Some severe injuries are compound through the overlying skin
Comminuted Comminution is invariably the result of considerable direct violence at the site
fractures of fracture
Degree of comminution is most common in symphysis and parasymphyseal
regions

https://t.me/DentalBooksWorld
608 Triumph's Complete Review of Dentistry

Pathological Direct violence to the mandible from penetrating sharp objects and missiles
These fractures are usually compound and further complicated by bone and
soft tissue loss
Fracture result from minimal trauma to a mandible already weakened by a
pathological condition
Example:
Osteomyelitis
Neoplasms
Generalized skeletal disease
Unilateral Usually single
But occasionally more than one fracture may be present on one side of
mandible
If this occurs often there is gross displacement of fragments
Unilateral fracture of body of mandible is most frequently caused by direct
violence
But in the case of weak condylar neck an indirect force may cause fracture,
while the site of direct impact remains intact
Bilateral Most often occur from a combination of direct and indirect violence
Example:
Most commonly involving angle and opposite condylar neck
or canine region and opposite angle
However, every possible combination and variation of linear fractures already
mentioned can occur bilaterally
Multiple Same association of direct with indirect violence may give rise to multiple
fractures
Most common multiple fracture is fall on midpoint of resulting in fractures of
symphysis and both condyles
“Guardsman fracture”
*Derived its name from the fracture combination seen on soldiers who faint
on parade
These fractures are commonly seen in:
Epileptics
Elderly patients losing consciousness
2. Site of fracture 1. Dentoalveolar Signs and symptoms vary according to the site of fracture so does the treatment
Most useful 2. Condyle
classification 3. Coronoid
for practical 4. Ramus
purposes 5. Angle
is based on
6. Body
anatomical
location of the 7. Parasymphysis
injury 8. Symphysis
3. Cause of a) Direct violence
fracture b) Indirect violence
c) Excessive muscular contraction

https://t.me/DentalBooksWorld
Chapter 7 • Oral Surgery 609

FRACTURE DUE TO EXCESSIVE MUSCULAR CONTRACTURE


1. Occasionally fracture of coronoid process occurs because of sudden reflex contracture of temporalis muscle
2. Sudden muscular contracture may also be a factor in some fractures of condylar neck
Clinical examination
Includes three stages
1. Immediate assessment and treatment of any condition constituting a threat to life
2. General clinical examination of the patient
3. Local examination of the mandibular fracture

Local examination of mandibular fracture


• Preparation for examination
• Prior to detailed examination of mandible, face must be gently cleaned with warm water or swabs
• For example, removing caked blood, road dirt, etc., which enhances an accurate evaluation of soft tissue injuries
• Similarly should be examined for loose or broken teeth or dentures
Extraoral examination
• Most of the signs are regarding → extravasation of blood from the damaged bone ends
• Results in:
–– Rapid early swelling from the accumulation of blood within the tissues
–– Later increase in swelling resulting from increased capillary permeability and edema
–– Swelling and ecchymosis indicates site of mandibular fracture
• Sometimes obvious deformity in the bony contour of mandible observed
• If considerable displacement has occurred patient is unable to close the anterior teeth together and mouth hangs open
• Particularly in recent fractures → blood stained saliva is frequently observed dribbling from the corners of the mouth
Palpation
• Should begin bilaterally in the condylar region
• Then continue downward along the lower border of mandible
• Bone tenderness is almost pathognomonic of fracture → *even an undisplaced crack*
• Possibility of palpating deformity or eliciting bony crepitus → when there is more displacement
• Reduced or absent of sensation with fractures of body of mandible are often associated with injury to inferior dental nerve →
one or both sides of lower lip
Intraoral examination
• Conscious cooperative patients
• Pathognomonic feature → Corman’s sign → ecchymosis in the lingual sulcus
• Bucker handle fracture → fracture of edentulous mandible
• Bilateral fracture of body of edentulous mandible near the posterior attachment of mylohyoid at the molar area
• Guardsman fracture →
–– fracture occurring in epileptics, elderly patients, and soldiers
–– multiple, tripartite fracture
–– combination of bilateral condylar fracture combined with symphyseal fracture

FRACTURES AT ANGLE OF MANDIBLE


Influenced by pull of medial pterygoid, masseter, and temporalis muscles
Tends to displace the ramus in an upward and medial direction

https://t.me/DentalBooksWorld
610 Triumph's Complete Review of Dentistry

Further classified into:


A. Horizontally favorable fracture:
Fracture line runs from lower border of mandible extending upward and backward to meet the upper border
Upward displacement of posterior fragment is prevented by anterior fragment
If the horizontal direction of fracture line favors the unopposed action of masseter and medial pterygoid muscle in an upward
direction, posterior fragment will be displaced upward
B. Horizontally unfavorable fracture:
Fracture line extends from the lower border in an upward and forward direction to meet the upper border
Posterior fragment will move in upward direction due to unopposed action of the masseter and medial pterygoid muscle
C. Vertically favorable fracture: Occlusal view
Fracture line runs from buccal plate obliquely backward toward the lingual plate
It resists the medial displacement of posterior segment
Such a fracture is known as vertically favorable fracture
D. Vertically unfavorable fracture:
Vertical direction of fracture line favors the unopposed action of medial pterygoid muscle
Posterior fragment will be pulled lingually
Medial pterygoid is the muscle that determines the vertical favorability of the mandibular angle fracture

DISPLACEMENT OF FRACTURE SEGMENTS


Fracture site in mandible Displacement direction Involved muscles
Horizontally unfavorable fracture Posterior fragment is displaced Masseter, medial pterygoid
of angle of mandible upward
Vertically unfavorable fracture of Posterior fragment is pulled lingually Medial pterygoid
angle of mandible
Fracture in the midline of mandible Minimal displacement occurs Geniohyoid, mylohyoid
Fracture lateral to the midline in Fragment with the genial tubercle is Geniohyoid, genioglossus, anterior belly
the incisor area of mandible displaced lingually digastric
Bilateral fracture of mandible in the Anterior segment is displaced These fractures remove the attachment of
canine region backward tongue to the mandible
Bilateral fracture of mandible in the Anterior segment is displaced Also allows the tongue to fall back and obstruct
molar region backward and downward oropharynx
Subcondylar fracture Condyle is displaced anteromedially Digastric, mylohyoid Lateral pterygoid
Fracture of coronoid process Fragment is displaced upward toward Temporalis muscle
the infratemporal fossa

MANAGEMENT OF MANDIBULAR FRACTURES


• Bone plating
• Simple
• Non compression plates
• Mini plates
• Compression plates
• Intermaxillary fixation
• Transosseous wiring
• Benthrust splint—bone clamp form of external fixation
• Transfixation— wires
• Circumferential wiring—oblique fractures of body of mandible
• Gunning splints—Edentulous mandible

https://t.me/DentalBooksWorld
Chapter 7 • Oral Surgery 611

METHODS OF IMMOBILIZATION OF MANDIBULAR FRACTURE


a. Osteosynthesis without IMF:
1. Noncompression small plates
2. Compression plates
3. Miniplates
4. Lag screws
b. Intermaxillary fixation:
1. Bonded brackets
2. Dental wiring: a) Direct, b) Eyelet
3. Arch bars
4. Cap splints
c. IMF with osteosynthesis:
1. Transosseous wiring
2. Circumferential wiring
3. External pin fixation
4. Bone clamps
5. Transfixation with Kirschner wires

ORTHOGNATHIC SURGERY
S. No. Osteotomy Indication
Mandibular body osteotomies (intraoral)
I Mandibular body osteotomy
Anterior body 1. Mandibular prognathism with functional posterior occlusion
2. Class II malocclusion with or without anterior open bite
Posterior body 1. Missing posterior teeth
2. Class III deformity
3. For the correction of cross bite
Mid symphysis Done along with anterior subapical mandibular body osteotomy
The complete vestibular incision can be planned if it is combined with posterior or
anterior body osteotomy
II Segmental subapical mandibular surgeries
Anterior subapical According to Bell and Legan and Wolford and Moenning, the mandibular anterior
mandibular osteotomy subapical osteotomy may be indicated to
1. level the occlusion,
2. produce anteroposterior changes of the osteotomized segment,
3. correct crowding in the lower anterior arch,
4. correct anterior dentoalveolar asymmetries,
5. alter the axial inclination of the anterior teeth,
6. reduce treatment time, and
7. improve treatment stability.
Posterior subapical 1. Uprighting the posterior segment which is in extreme linguoversion or
mandibular osteotomy buccoversion
2. Closing a premolar or molar space
3. Leveling supraerupted posterior teeth

https://t.me/DentalBooksWorld
612 Triumph's Complete Review of Dentistry

Total subapical -
mandibular osteotomy
III Genioplasties—Horizontal osteotomy in the chin region
Augmentation Indications
genioplasty—to increase Indications for functional genioplasty are as follows:
chin projection I. Vertical excess of the lower anterior facial height in which the following clinical
Reduction signs and radiographic findings are present:
genioplasty—to reduce A. Clinical signs
chin region 1. Lip incompetence when the lips are in repose and a normal relationship
Straightening of the maxillary incisor tooth to the upper lip exists
genioplasty - 2. Open mouth posture with interposition of the tongue between the teeth
1. In facial asymmetry, 3. Elevation of the mental soft tissues to obtain lip closure
where complete
4. Thinning of the alveolar bone overlying the facial surfaces of the roots of
correction of the
the anterior mandibular teeth
asymmetry cannot
be achieved by 5. Flattening of the contour of the anterior surface of the soft tissue profile of
appropriate jaw the chin
osteotomies, e.g., B. Radiographic findings
TMJ joint ankylosis 1. ANS-Me distance, which constitutes more than 55% of the total anterior
2. The horizontal facial height, in the absence of vertical maxillary excess
osteotomy is done 2. ANS-Me distance, which persists in constituting more than 55% of the
and segment is total anterior facial height after predicted or actual appropriate correction
shifted laterally and of vertical maxillary excess, open bite, or anteroposterior dentofacial
then contoured to deformities
get desired result II.  Residual and associated deformities or cleft lip/cleft palate, particularly in the
Lengthening case of insufficient cheiloplasty in which the upper lip is too short. Superior
genioplasty repositioning of the mental osseous and the myocutaneous tissues can aid in
achievement of lip competence and development of the upper lip
III. Requirement for a complimentary procedure to adjust lower anterior facial
height when maxillary and mandibular osteotomies are performed for
correction of anteroposterior and transverse maxillofacial deformities
IV.  As an aid in stabilization of orthodontic treatment results by normalization of
the osseous and myocutaneous elements of the lower face
V.  As a complementary procedure with partial anterior glossectomy to aid in the
management of dental open bite
Mandibular ramus osteotomies
I Subcondylar ramus 1. Major setback of mandible more than 10mm
osteotomy 2. Asymmetric setback of the mandible
3. Reoperation of previously operated case
Vertical Extraoral subcondylar Indicated for setback cases (mandibular prognathism) (where as inverted L
Subsigmoid ramus osteotomy osteotomy is used for advancement cases)
Osteotomy (subsigmoid)
(VSSO)—also Intraoral subcondylar
known as ramus osteotomy
subsigmoid (subsigmoid)
osteotomy

https://t.me/DentalBooksWorld
Chapter 7 • Oral Surgery 613

Arching ramus -
osteotomy
Inverted L Mandibular Indications for the Use of the Procedure:
Osteotomy The inverted L osteotomy may be the operation of choice for large advancements
(greater than 12 mm) with counterclockwise rotation or for large setbacks (greater
than 10 mm). It is also a good choice for reoperations resulting from altered ramal
morphology and in patients with masseter hypertrophy with dense underlying
cortical bone
Intraoral Vertical Indications for the Use of the Procedure:
Ramus Osteotomy Intraoral vertical ramus osteotomy is indicated for the management of horizontal
mandibular excess. Additionally, small distal segment advancement (less than
2 mm) is compatible with IVRO. Intraoral vertical ramus osteotomy is also ideally
suited to the management of mandibular asymmetry with planned rotation about
one ramus.
For symptomatic temporomandibular disorder, IVRO may be preferred over SSO
because the condyle is passively positioned, with little opportunity to place the
condyle in an unphysiologic and/or loaded position. Furthermore, experience with
modified mandibular condylotomy suggests that IVRO may actually improve joint
symptoms.
II Intraoral modified Indication
sagittal split osteotomy 1. The correction of congenital dentofacial deformities, including mandibular
deficiency, hyperplasia, asymmetry, and dysgnathia
2. The correction of acquired dentofacial deformities resulting from facial
trauma, tumor ablative surgery, and temporomandibular joint asymmetries and
deformities
Contraindications
1. Distorted ramus anatomy (thin or abnormal shape)
2. Excessive counterclockwise rotation (greater than 2 cm apertognathia requires
two-jaw surgery)
3. Mandibular advancements greater than 12 mm
4. Neurosensory concerns
5. Previous head and neck radiation
Maxillary osteotomy procedures (intraoral)
I Segmental maxillary Divided into anterior/posterior/single tooth and horse shoe shaped
osteotomy
Single tooth dento- 1. Indicated in tooth mal position
osseous osteotomy 2. Dental ankylosis
3. Closure of diastema
Interdental osteotomy -
Surgically assisted 1. Skeletal discrepancy greater than 5 mm associated with wide mandible
maxillary expansion 2. Failed orthodontic expansion
(SAME) 3. Extremely thin delicate gingival tissue
4. Significant nasal stenosis

https://t.me/DentalBooksWorld
614 Triumph's Complete Review of Dentistry

Anterior maxillary The anterior segment of the maxilla can be osteotomized separately using the
osteotomy Wassmund or the Wunderer approach. The following are the main indications for
this osteotomy:
1. Maxillary prognathism where posterior movement of the maxilla is not required
2. Anterior vertical maxillary excess with excessive gingival show
3. Protrusion of the anterior segment of the maxilla with proclined upper incisors
but a satisfactory posterior occlusion
4. Anterior open bite caused by anterior vertical deficiency of the maxilla with
reduced tooth show
Posterior maxillary 1. Posterior maxillary hyperplasia
osteotomy 2. Total maxillary hyperplasia (When combined with AMO)
3. Distal repositioning of the maxillary alveolar fragment to provide proper space
for eruption of an impacted canine or bicuspid tooth
4. Spacing in the dentition that can be closed by anterior repositioning of the
posterior segment
5. Transverse excess or deficiency
6. Posterior open bite
7. Posterior cross bite
II Total maxillary 1. Correction of the hypoplastic and retrognathic maxilla. Due to the anatomical
surgery—Le Fort I position of the pterygoid plates, a maxillary posterior shift is limited to 2–3 mm
osteotomy and removal of bone in the tuberosity area may be required 2. Correction of
vertical maxillary excess and deficiency
3. The correction of occlusal canting by impacting the longer side or down grafting
the shorter side or a combination of both
4. Correction of an anterior open bite. This is usually achieved by impaction of the
posterior part of the maxilla to allow the mandible to auto rotate and close the
anterior open bite
5. Correction of a narrow maxilla and narrow dental arch
6. Correction of a prominent anterior maxillary segment with or without
extractions
Le Fort I segmental Indications:
osteotomies The main indication of this surgical procedure is the correction of a narrow maxilla
and narrow dental arch
Two-piece maxillary The main indication of this surgical procedure is the correction of an anterior open
osteotomy bite. This is usually achieved by impaction of the posterior part of the maxilla to
allow the mandible to auto rotate and close the anterior open bite.
Three-piece maxillary The main indication of this surgical procedure is the correction of a narrow
osteotomy (also refer posterior maxilla and allowing differential vertical movements of the anterior and
horse shoe shaped posterior segments of the maxilla
osteotomy)
Four-piece maxillary The main indication of this surgical procedure is the correction of a transverse
osteotomy maxillary deficiency which involves both the intercanine and intermolar width. It
also allows differential vertical movements of the anterior and posterior segments
of the maxilla. The procedure should be considered for correction of anterior open
bite which is associated with a narrow maxillary dental arch at the canine and
molar regions

https://t.me/DentalBooksWorld
Chapter 7 • Oral Surgery 615

Superior repositioning Anterior Open bite


of the maxilla
Superior repositioning 1. Maxillary alveolar hyperplasia with or without an anterior open bite deformity
of the maxilla 2. Transverse hypoplasia without a vertical component
leaving nasal floor This procedure creates a three piece maxilla, with the central nasal portion left
intact: Horseshoe- undisturbed, through the use of palatal parasagittal osteotomies
shaped osteotomy
(combination of
anterior and posterior
maxillary osteotomy)
Advancement of Hypoplastic maxilla
maxilla
Simultaneous Cross bite conditions/cleft palate
expansion of maxilla
Simultaneous -
narrowing of maxilla
Inferior repositioning Anterior deep bite
of maxilla
Levelling of maxilla Maxilla malalignment

PREPROSTHETIC SURGERY
PREPROSTHETIC SURGERY:
Alveolectomy → surgical removal or trimming of alveolar process
Trimmed with bone rongeur or round bur
Smoothened with bone file.
Alveoloplasty → surgical recontouring of alveolar process

Types of Alveoloplasty
Alveolar compression → easiest and quickest method
• Compression of cortical plates with fingers
• Reduction in socket width
Simple alveoloplasty → reduction of buccal/labial plate
• Extraction of single or multiple plate
• Labial and buccal cortical
Deans intraseptal → crush technique
• Reduction of labial or alveolar prominences
• Preserves cortical bone
Obwegesr’s technique → indicated in premaxillary protrusion
• Indicated in immediate dentures and in quadrant extraction

https://t.me/DentalBooksWorld
616 Triumph's Complete Review of Dentistry

Vestibuloplasty
Soft tissue procedures:
Frenectomy
Z plasty→ this procedure is used when the frenum is short and vestibule is broad
V-Y→ these incisions are used for lengthening localized area
Semi lunar → these incisions are used for broad premolar and molar region freni
Clarkes Technique
• Flap is reflected from alveolar crest till vermilion border of the lip
• Supraperiosteal dissection is done till desired vestibular depth and edge of the mobilized flap is pushed into vestibular depth
• This flap is held in position with sutures passed through the chin area extraorally and tied around the rubber catheter
• And alveolar bone here is covered by periosteum which heals quickly by granulation
Lingual vestibuloplasty
Trauner’s technique:
• Procedure used to increase the depth of floor of mouth in mylohyoid region
• Incision over lingual side of alveolar ridge bilaterally in posterior region (near second molars)
• Supraperiosteal dissection is done to identify mylohyoid muscle
• Only to separate its attachment and sutured to new desired vestibular depth
Visor osteotomy
Used where insufficient vertical mandibular bone height is present for the horizontal osteotomy technique but adequate
bone width.

TEMPOROMANDIBULAR JOINT DISORDER MANAGEMENT


TMJ BASICS
• Also known as craniomandibular joint
• Articulation between squamous part of temporal bone and the head of mandibular condyle
• This articulation is labeled as “bilateral diarthrodial or freely movable joint”
• Also considered as “complex joints” → because of involving two separate synovial joints
• Right and left
TMJ Articulation Consists of
1. Mandibular or Glenoid Fossa
2. Articular Eminence
3. A Condyle
4. Separating Disk
5. A Joint Fibrous Capsule
6. An Extra Capsular Cheek Ligaments.
Articulatory System
Comprises of:
1. TMJ
2. Masticatory and accessory muscles
3. Occlusion of the teeth

https://t.me/DentalBooksWorld
Chapter 7 • Oral Surgery 617

TMJ DISORDERS CLASSIFICATION


1. Intrinsic disorders → Intra articular origin
2. Extrinsic disorders → Extra articular origin

Disorders due to extrinsic factors: false ankylosis Disorders due to intrinsic factors: true ankylosis
Masticatory Muscle Disorders: 1. Trauma
a. Protective muscle splinting A. Dislocation, subluxation
b. Masticatory muscle spasm (MPD syndrome) B. Hemarthrosis
c. Masticatory muscle inflammation (myositis) C. Intracapsular fracture, extracapsular fracture
Problems That Result From Extrinsic Trauma: 2. Internal Disk Displacement
a. Traumatic arthritis A. Anterior disk displacement with reduction
b. Fracture B. Anterior disk displacement without reduction.
c. Internal disk derangement 3. Arthritis
d. Myositis, myospasm A. Osteoarthrosis
e. Tendonitis B. Rheumatoid arthritis
f. Contracture of elevator muscle → myofibrotic C. Juvenile rheumatoid arthritis
contracture D. Infectious arthritis
4. Developmental Defects
A. Condylar agenesis or aplasia → unilateral/bilateral
B. Bifid condyle
C. Condylar hypoplasia
D. Condylar hyperplasia
5. Ankylosis
6. Neoplasms
A. Benign tumors: Osteoma, osteochondroma, chondroma
B. Malignant tumors:
Chondrosarcoma, fibrosarcoma, synovial sarcoma

CAUSES OF TRISMUS
1. Due to infection:
Examples:
Pericoronitis
Ludwig’s angina
Submasseteric and infratemporal abscess
2. Trauma
3. Inflammation
4. Myositis ossificans
5. Tetany
6. Tetanus
7. Neurological disorders
8. Psychosomatic trismus
9. Drug-induced trismus
10. Mechanical blockage
11. Extraarticular fibrosis

https://t.me/DentalBooksWorld
618 Triumph's Complete Review of Dentistry

MANAGEMENT OF TMJ ANKYLOSIS


**Treatment of TMJ ankylosis is always surgical
Early surgical correction of the ankylosed joint is highly desirable, if satisfactory function is to be regained
Factors that influence the surgical therapy:
a. Age of onset of ankylosis
b. Extent of ankylosis
c. Whether there is unilateral or bilateral involvement
d. Associated facial deformity

SURGICAL TECHNIQUES
Three Basic Methods
1. Condylectomy
2. Gap arthroplasty
3. Interpositional arthroplasty
*Most surgical procedures can be done through a preauricular incision
Popowich’s incision is chosen for the obvious advantages
Whenever required, additional submandibular incision can be used for fixation of the graft

Incisions

Blairs’s Inverted Thoma’s Dingman’s Endaural


Hockey Angulated Incision Incision
Sck Incision Incision

Post ramal

Submandibular

Popowich & Crane Posterior


Incision Auricular Incision

CONDYLECTOMY
• Advocated in cases of fibrous ankylosis
• Where the joint space is obliterated with deposition of fibrous bands but there is not much deformity of condylar head
• Vital structures on the medial side of the condylar neck should be protected →using special condylar retractor → inserted
prior to the bony cut
• Horizontal osteotomy cut is carried out with help of surgical bur at the level of condylar neck
• Radiologically and clinically after surgical exposure one can see the demarcation between the roof of glenoid fossa and
the head of condyle
Unilateral condylectomy → tends to cause deviation of mandible toward operated side, on oral opening
Bilateral condylectomy → anterior open bite will be caused as a result of loss of height in vertical ramus

https://t.me/DentalBooksWorld
Chapter 7 • Oral Surgery 619

A B C

D E F

F:condylectomy-surgical: (A) Exposure of condylar head via preauricular incision, (B) Sectioning of condylar head,
(C) Breaking the fibrous adhesions, (D) Condylectomy complete, (E) Surturing the capsule, (F) Final skin suturing.

GAP ARTHROPLASTY: Minimum Gap of At least 1 cm to Prevent Reankylosis


Above term describes the operation in which “level of section is below that of previous joint space”
And in which no substance is interposed between two cut bony surfaces
Width of the bone is considered crucial
**Important to create a gap of equal dimension both laterally and medially
So that possibility of medial reankylosis due to bone contact is avoided
SECTIONS
Two horizontal osteotomy cuts
Removal of a bony wedge for creation of gap
INTERPOSITIONAL ARTHROPLASTY
“Recurrence of ankylosis is less likely when something is interposed between the two cut bony surfaces”
So, interpositional arthroplasty involves the creation of a gap but in addition a barrier is inserted between a cut bony
surfaces
Barrier is inserted → only to minimize the risk of recurrence and to maintain the vertical height of the ramus

Regimens for a dental procedure


Situation Agent Regimen: Single Dose 30–60 Minutes Before
Procedure
Adults Children
Oral Amoxicillin 2 grams 50 milligrams per kilogram
Unable to Take Oral Amoxicillin 2 g IM* or IV† 50 mg/kg IM or IV
Medication OR
Cefazolin or ceftriaxone 1 g IM or IV 50 mg/kg IM or IV
Allergic to Penicillins or Cephalexin ठ2g 50 mg/kg
Ampicillin Oral OR
Clindamyin 600 mg 20 mg/kg
OR
Azithromycin or clarithromycin 500 mg 15 mg/kg

https://t.me/DentalBooksWorld
620 Triumph's Complete Review of Dentistry

Regimens for a dental procedure


Allergic to Penicillins or Cefazolin or ceftriaxone § 1 g IM or IV 50 mg/kg IM or IV
Ampicilliin and Unable to OR
Take Oral Medication clarithromycin 600 mg IM or IV 20 mg/kg IM or IV
* IM: Intramuscular.
† IV: Intravenous.
‡ Or other first- or second-generation oral cephalosporin in equivalent adult or pediatric dosage.
§ Cephalosporins should not be used in a person with a history oa anaphylaxis, angioedema or urticaria with penicillins or
ampicillin.

COMMON MEDICAL EMERGENCIES AND CPR


Warning Signs of a Medical Emergency
• Change in mental status (e.g., unusual behavior, confusion, difficulty arousing, aggression, collapse)
• Chest pain
• Continuous bleeding
• Coughing up or vomiting blood
• Difficulty breathing, shortness of breath or choking
• Fainting or loss of consciousness
• Severe or persistent vomiting
• Sudden dizziness, weakness or change in vision
• Sudden, severe pain anywhere in the body

Factors Influencing the Effect of Healthcare Procedures on a Patient (the Outcome)


• Health of patient
• Type of procedure
• Duration of procedure
• Degree of trauma and stress
• Degree of urgency of procedure
• Skill and experience of operator
• Skill and experience of anesthetist/sedationist

VARIOUS MEDICAL EMERGENCIES


Adrenal insufficiency
Any patient who has been taking corticosteroids for any length of time, or has stopped taking them, can suffer adrenal
insufficiency due to physiological stress, as this results in hypotension. As patients can become anxious at the thought of and
actually receiving treatment there is a possibility that they may collapse because of this insufficiency. It is possible that routine
dental treatment does not affect them so other reasons should not be ruled out when an assessment is made. A few patients
have warning cards and by updating a patient’s medical history it can be established whether they would require a prophylactic
administration of steroids prior to treatment to prevent its occurrence.

https://t.me/DentalBooksWorld
Chapter 7 • Oral Surgery 621

Signs and symptoms of adrenal insufficiency


• Pale
• A fall in blood pressure
• A rapid loss of consciousness
• Low blood glucose levels
Management of adrenal insufficiency
• Patients should be laid flat with their legs higher than their head
• Oxygen administered at 10–15 L/min
• Emergency services called
• Monitored and reassured
• Hydrocortisone administered

Asthma attack
• Asthma is a very common condition, which many sufferers make light of despite it affecting a large number of the population.
However, it must be remembered that it can be life threatening.
• It is a chest condition which occurs due to narrowing of the airways where the lining of the walls swell and become inflamed.
• Occasionally, sticky mucus and phlegm can attach to the airways making the tubes even narrower.
• Asthma attacks can occur because of stress, emotion, anxiety, exercise, being exposed to an allergen, colds or chest infections
and laughter.
• Many people with asthma suffer from eczema and hay fever, with their condition being worsened during the hay fever season.
Signs and symptoms of an asthma attack
• Breathlessness
• Inability to complete a sentence
• Wheezing on exhalation
• Accessory muscles of respiration in action
• Increased respiratory rate (more than 25/minute)
• Tachycardia, a fast pulse rate (more than 110/minute)
• Anxiety
Life-threatening signs and symptoms
• Bradycardia, a slow pulse rate (less than 8/minute)
• Decreased respiratory rate (less than 50/minute)
• Cyanosis, blueness of the lips, and/or extremities
• Exhaustion, confusion, and a decreased level of consciousness
Management of an asthma attack
• Reassure the patient and sit them up. Do not lay the patient flat as this will increase their breathlessness.
• Patients normally carry their salbutamol (ventolin) inhaler (100 µg per activation) with them. They should be encouraged
to take a few activations as this is usually all that is required. If they do not have their medication with them obtain it from
the emergency drugs box. To eliminate the spread of infections the inhaler can be either given to the patient or discarded in
the waste drugs box to be disposed of in the normal way.
• If a patient is unable to use their inhaler effectively, then additional doses should be given through a large volume spacer
device.
• Call the emergency services if the patient does not improve or they exhibit life threatening signs and symptoms.
• If the patient’s nebuliser is unavailable a large volume spacer device should be used with 4–6 activations of salbutamol being
given and repeated every 10 minutes, as needed, until the emergency services arrive.
• While waiting for the emergency services maintain a patient’s airway and administer oxygen at 10–15 L/minute.

https://t.me/DentalBooksWorld
622 Triumph's Complete Review of Dentistry

• If a patient becomes unresponsive you should check for breathing and signs of life and if necessary undertake cardiopulmonary
resuscitation, ignoring the occasional gasp.
• At all times patients must be monitored and reassured.
• Any sick, cyanosed patient with respiratory difficulty should be administered a high flow of oxygen until the ambulance
arrives as this is of benefit to them, even in the case of a patient who has chronic obstructive pulmonary disease. The benefit
would outweigh any risks of causing respiratory depression.

Anaphylaxis
An anaphylactic shock is a type of hypersensitive reaction to otherwise unknown antigen (i.e., antibiotics, nuts). In dentistry,
anaphylactic reactions may follow the administration of a drug or exposure to latex. It is caused by the release of histamine
following an exposure to an antigen in a person who has previously been sensitized to that allergen. Anaphylactic reactions
can also be attributed to additives and recipients in medicines, so it is vital to check the full contents of any which may contain
fats and oils. Generally the more rapid the onset of the anaphylactic reaction the more serious the condition will be.
Signs and symptoms of anaphylaxis
• Symptoms of an anaphylactic reaction can develop within minutes of exposure and early, effective management of this
condition could be life saving. Unfortunately, as there are a huge range of possible signs and symptoms it can make the
condition very difficult to diagnose:
• Urticaria (an itchy skin eruption which is characterized by wheals that have pale interiors with well-defined red margins).
• Rhinitis (an inflammation of the mucous membrane lining the nose).
• Conjunctivitis (inflammation of the conjunctiva of the eye).
• Nausea, vomiting, diarrhea and abdominal pains.
• Patients experience a sense of unease and impending doom.
• Flushing is very common—however a pale complexion may also occur.
• Marked upper airway (laryngeal).
• Edema (swelling) of the tongue and upper airway.
• Bronchospasms may develop, causing strider (a whistling noise on inspiration and wheezing).
• Peripheral coldness and cold clammy skin.
• Rapid/weak impalpable pulse, tachycardia with a rapid drop in blood pressure.
• Vasodilation leading to a drop in the blood pressure and collapse.
• Respiratory arrest (breathing has stopped, but circulation is still present).
• Loss of consciousness and cardiac arrest.
Management of anaphylaxis
First-line treatment
• Remove the item that has caused the reaction and if a drug was being administered stop its use immediately.
• Immediately place the patient in the supine position to restore their blood pressure.
• Maintain the patient’s airway and administer oxygen at 10–15 L/minute.
• Severe reaction
• Call the emergency services.
• A semi-conscious patient or one presenting severe bronchospasms and a widespread rash should have a 0.5ml adrenaline
injection 1:1000 administered intramuscularly (IM) in either their outer arm or thigh.
• An auto-injector (epipen) preparation of adrenaline is available as a 0.3ml injection, 1:1,000 for self-administration by a
patient who is aware that they will have a severe reaction. If the patient has his/her epipen and it is immediately available
then it is acceptable to use it.
• The dose of adrenaline should be repeated every 5 minutes according to the patient’s blood pressure, respiratory and pulse rates.
• At all times monitor and reassure the patient.
• If the patient loses consciousness they should be assessed for signs of life and breathing and, if necessary, undertake
cardiopulmonary resuscitation, ignoring the occasional gasp.

https://t.me/DentalBooksWorld
Chapter 7 • Oral Surgery 623

• All patients should be transferred to hospital for further assessment, irrespective of their initial recovery.
• An antihistamine drug, chlorpheniramine maleate and steroid, hydrocortisone succinate (Solu-cortef) are useful in the
management of an allergic reaction but they are not first-line drugs and will be administered by the emergency services if
necessary
Children
• The dose of intramuscular adrenaline 1:1,000 is based on the approximate age of the child or their weight:
–– 12 years—500 µg IM (0.5 mL)
–– If child is small or prepubertal—250 µg
–– 6–12 years—250 µg IM (0.25 mL)
• 6 months to 6 years—120 µg IM (0.12 mL)
• 6 months—50 µg (0.05 mL)

The “ABCDE” approach to assessing any patient who is unwell


A – Airway
B – Breathing
C – Circulation
D – Disability
E – Exposure
The underlying principles of managing medical emergencies
1. Conduct a complete “ABCDE” assessment and re-assess regularly
2. Treat life-threatening problems before moving to the next part of the assessment
3. Reassess to determine the success of treatment
4. Recognize when to call for help—call for help early
5. Use all members of the clinical team, ideally to be working simultaneously and in a coordinated manner
6. Communicate effectively—use the Situation, Background, Assessment, Recommendation (SBAR) approach
7. The aim of the initial treatment is to keep the patient alive until help arrives

BASIC LIFE SUPPORT


The standard, well-tested approach to BLS (called the primary survey) includes:
1. Rapidly assessing any Dangers, and the patient’s Responsiveness and Airways, Breathing and Circulation (DRABC)
2. Carrying out cardiopulmonary resuscitation (CPR), to help the person start to breathe again and to help the person’s heart
pump blood around the body.
ADULT BLS SEQUENCE
1. Check to see if the victim is breathing by looking at the chest. If they are not breathing, or not breathing normally (only
gasping). Call for emergency services
2. Check the carotid pulse on the side of the neck closest to you for 5–10 seconds
3. If no pulse, start with 30 chest compressions on the lower half of the breastbone.
4. The rate should be at least 100 compressions per minute, but not more than 120 (Chest compression: Kneel by the side of
the person—Place the heel of one hand in the center of the person’s chest—Place the heel of your other hand on top of the
first hand—Interlock the fingers of your hands and ensure that you are not applying pressure on the person’s ribs, the upper
abdomen or the bottom end of their sternum—With your arms straight, press down 4–5 cm on the sternum—Then release
all the pressure on the chest but do not lift your hands from the sternum)
5. Open the airway and give 2 breaths using a pocket mask or bag valve mask.
• Continue 5 sets of 30 compressions and 2 breaths until the ambulance arrives to take over

https://t.me/DentalBooksWorld
624 Triumph's Complete Review of Dentistry

CHILD BLS SEQUENCE


1. Check to see if the child is breathing by looking at their chest. If they are not breathing, or not breathing normally (only
gasping), Call for emergency services
2. Check the carotid pulse on the side of the neck closest to you for 5–10 seconds
3. If no pulse, begin 30 chest compressions on the lower half of the breastbone with one or two hands.
4. The rate should be at least 100 compressions per minute
5. Open the airway and give 2 breaths using a pocket mask or bag valve mask.
6. Continue 5 sets of 30 compressions and 2 breaths until the ambulance arrives to take over

ASSESSMENT IN EMERGENCIES
Airway Identify foreign body obstruction and stridor
Breathing Document respiratory rate, use of accessory muscles, presence of wheeze or cyanosis
Circulation Assess skin color and temperature, estimate capillary refill time (normally, 2 seconds with hand above heart),
assess rate of pulse (normal is 70 beats/minute)
Disability Assess conscious level using acronym AVPU:
• Alert
• responds to Voice
• responds to Painful stimulus
• Unresponsive
• Blood glucose
Exposure Respecting the patient’s dignity, try to elicit the cause of acute deterioration (e.g. rash, signs of recreational drug use)

QUICK FACTS

Carnoy’s solution:
Carnoy’s solution described by Voorsmit (1981) contains 100% ethanol, chloroform, and glacial acetic acid in a 6:3:1 ratio with
added ferric chloride.

Absolute ethanol 6 mL
Chloroform 3 mL
Glacial acetic acid 1 mL
Ferric chloride 1g

Color coding for local anesthesia (LA)


Articaine 4% + epinephrine 1:100,000 Gold
Articaine 4% + epinephrine 1:200,000 Silver
Bupivacaine 0.5% + epinephrine 1:200,000 Blue
Lidocaine Hcl 2% Light blue
Lidocaine 2% + epinephrine 1:50,000 Green
Lidocaine 2% + epinephrine 1:100,000 Red
Mepivacaine 3% Tan
Mepivacaine 3% + Levonordefrin 1:20,000 Brown
Prilocaine 4% Black
Prilocaine 4% + epinephrine 1:200,000 Yellow

https://t.me/DentalBooksWorld
Chapter 7 • Oral Surgery 625

LA t1/2 and clearance in various systemic diseases


Lidocaine group Half-life (minutes) Clearance (mL/kg/min)
Normal 1.8 10
Heart failure 1.9 6.3
Hepatic failure 4.9 6
Renal disease 1.3 13.7

LA composition and function


Composition of Local Anesthetic agent
Description Component
Local anesthetic agent Lignocaine HCL—2% (20 mg/mL)
Vasoconstrictor Adrenaline (Epinephrine)—1:80,000
Reducing agent Sodium metabisulfite—0.5 mg
Preservative Methylparaben—0.1% (1 mg)
Isotonic solution Sodium chloride—6 mg
Fungicide Thymol
Diluting agent Distilled water
To adjust pH Sodium hydroxide
Nitrogen bubble 1–2 mm in diameter and is present to prevent oxygen from being trapped in the cartridge
and potentially destroying the vasopressor or vasoconstrictor

Distraction osteogenesis
Location Latency period Rate (total mm/day) Rhythm Consolidation period
(days) (or 2× activation period)
Mandible or 5–7 1.0 BID 8 weeks
maxilla—adult
Alveolar ridge 5–7 0.5–1.0 BID–TID 4 weeks
Mandible neonatal 0 2.0–4.0 BID–QID 2 weeks
Mandible child 2–3 2.0 BID 3–4 weeks
Mandible elderly 7–10 0.5–1.0 QID–BID 10–12 weeks

Hendrickson palatal classification system based on anatomic location of fracture


Palatal fracture Picture
Type I: Alveolar fracture
Type Ia: Anterior alveolus; contains only incisor teeth and
associated alveolus

https://t.me/DentalBooksWorld
626 Triumph's Complete Review of Dentistry

Type Ib: Posterolateral; contains premolars, molars, and


associated alveolus

Type II: Sagittal fracture, a split of the palatal midline; typically


occurs in second or third decade because of a lack of ossification
of the midline palatal suture

Type III: Parasagittal fracture; most common fracture pattern in


adults (63%) because of thin bone parasagittally; fracture pattern
differs from type Ia fracture by inclusion of maxillary canine

Type IV: Para-alveolar fracture; occurs palatal to the maxillary


alveolus and incisors

Type V: Complex comminuted fracture; multiple fractured


segments

Type VI: Transverse fracture, rare; involves a division in the


coronal plane

https://t.me/DentalBooksWorld
Chapter 7 • Oral Surgery 627

BASIC EMERGENCY DRUGS FOR THE DENTAL OFFICE


Drug Indication Action Administration
Epinephrine Bronchospasm (severe allergic Alpha and Beta 1:1,000 solution, subcutaneously,
reaction, severe asthma) adrenergic receptor intramuscularly, or sublingually dose 0.3
agonist (bronchodilator) mg adults, 0.15 mg children
Diphenhydramine Mild allergic reaction Antihistamine 50 mg intramuscularly, 25–50 mg orally
every 3–4 hours
Nitroglycerine Angina Vasodilator Sublingual spray: 0.4 mg per metered dose,
one spray every 5 minutes up to three times
Sublingual tablet: One every 5 minutes up to
three doses
Albuterol Bronchospasm (mild asthma) Selective Beta 2 Inhaler: Two to three inhalations every 1–2
adrenergic receptor minutes, up to three times
agonist
Aspirin Myocardial infarction Antiplatelet One full-strength 325 mg tablet (nonenteric
coated) chewed and swallowed
Glucose orange Hypoglycemia (insulin shock) Antihypoglycemic If the patient is conscious, ingest
juice, sugar packet,
glucagon
Aromatic Syncope Respiratory stimulant Inhalant crushed and held 4–6 in under the
ammonia nose
Note: IM injections are generally easier and safer to give. IM injections are best given into the side of the thigh.

Epidemiology of mandibular fractures according to Kelly and Kay


Ramus 4%
Angle 12.1%
Body 30.9%
Alveolar process 7.4%
Condylar process 16.1%
Symphysis 27.5%

According to WHO/1997, 2003//3/ the international classification of mandibular fractures is


S 02.6 Fractura mandibulae
S 02.60 Fractura processus alveolaris
S 02.61 Fractura corpus mandibulae
S 06.62 Fractura processus articularis/condylaris
S 06.63 Fractura processus muscularis/coronoideus
S 02.64 Fractura ramus mandibulae
S 02.05 Fractura symphysis
S 02.66 Fractura angulus mandibulae
S 02.67 Fractura mandibulae multiplex
S 02.68 Unspecified mandibular fractures
The term “unspecified mandibular fractures” having in mind the contemporary apparatus is not correct

https://t.me/DentalBooksWorld
628 Triumph's Complete Review of Dentistry

Fracture reduction for Le Fort


A Rowe disimpaction forceps is used when
attempting to reduce a displaced maxilla

Positioning of the Hayton-Williams forceps


when attempting to reduce a fractured
maxilla

Treatment for fractures


Symphysis Two plates one at upper and one at lower border
Angle One plate at superior border
Body One plate at upper border
Condylar Two plates positioned parallel to each other
Le Fort I Four point fixation
Champy’s plates Mini plate, nonrigid
Lag screws Rigid, load-sharing fixation
Atrophic edentulous mandible, comminuted fractures Reconstruction plate, load bearing, rigid

MULTIPLE CHOICE QUESTIONS

LOCAL ANESTHESIA AND GENERAL ANESTHESIA


1. Local anesthetic agent preferred in prolonged surgical procedures is
A. Bupivacaine B. Cocaine
C. Lignocaine D. Prilocaine
2. The theory for local anesthetic action includes the following
A. Membrane expansion theory B. Calcium displacement theory
C. Electrical potential theory D. Specific receptor theory
E. All of the above
3. In the extraoral technique for mandibular nerve block, the needle is directed _____ after contacting the pterygoid
plate.
A. Anteriorly B. Posteriorly
C. Superiorly D. Inferiorly

https://t.me/DentalBooksWorld
Chapter 7 • Oral Surgery 629

4. For hemophiliac patients, the local anesthetic technique followed is


A. Nerve block B. Supraperiosteal
C. Intraligamentary D. Field block
5. Which LA has intrinsic vasoconstrictive action?
A. Cocaine B. Procaine
C. Xylocaine D. Bupivacaine
6. Why is it difficult to obtain local infiltration anesthesia in the presence of inflammation?
A. Decreased PH B. Increased vascularity
C. Edema D. Pain
7. What happens when patient is administered with 40% nitrous oxide?
A. Paresthesia B. Floating sensation
C. Sweating D. None of the above
8. Why are alpha adrenergic agonists used in combination with local anesthetics?
A. Increase the rate of liver metabolism of local anesthetic
B. Increase the concentration of LA at receptor site
C. Stimulate myocardial contraction
D. Increases vascular absorption of LA
9. Toxic doses of local anesthesia on CNS can be described as
A. First stimulating the CNS followed by depression B. First depressing it followed by CNS stimulation
C. Only depression of the CNS D. Only stimulation of the CNS
10. LA should be deposited near _____ to give field block
A. Main trunk B. Large branch of peripheral nerve
C. Small nerve endings D. Periodontal ligament
11. _____ theory demonstrates pain conduction
A. Gate control theory B. Specificity theory
C. Membrane stabilization theory D. None of the above
12. All the following are indicated in anesthetizing lower anteriors except
A. Fischer 123 technique B. Classical inferior alveolar block
C. Mental nerve block D. Incisive block
13. All of the following are affected by the action of lignocaine except
A. PH at the site of injection B. Blood flow at the site of injection
C. Vasoconstrictor in the LA solution D. Action of cholinesterase at the site of injection
14. Which of the following about syncope is not true?
A. Another term for fainting B. Is transient reversible loss of consciousness
C. Caused by altered circulation D. None of the above
15. Hyaluronidase ______ when added to a to local anesthetic solution
A. Increase the duration of anesthesia B. Limit the area of anesthesia
C. Reduce bleeding D. Enhance diffusion of local anesthetic
16. Which of the following when added can increase the effect of local anesthesia?
A. Adrenaline B. Isoprenaline
C. Dopamine D. Felypressin (synthetic vasopressin)
17. Where do amide type of local anesthetic agents undergo biotransformation primarily?
A. Kidney B. Liver
C. Plasma D. Excreted in unaltered form
18. The maximum dose of lignocaine without adrenaline that can be admitted to a patient is
A. 6 mg/kg body weight B. 5 mg/kg body weight
C. 7 mg/kg body weight D. 9 mg/kg body weight
19. Surgery can be carried out in which stage of general anesthesia?
A. Plane I B. Plane II
C. Plane III D. Plane IV

https://t.me/DentalBooksWorld
630 Triumph's Complete Review of Dentistry

20. The amount of vasoconstrictor in 3 mL of 2% lignocaine solution with 1:2,00,000 adrenaline is


A. 0.5 mg B. 0.015 mg
C. 0.005 mg D. 0.0005 mg
21. When is inferior alveolar nerve block used?
A. Pulpotomy of third molar B. Apicoectomy of third molar
C. Extraction of first molar D. Root resection of first molar
22. _____ is pierced by a needle while giving an inferior alveolar nerve block.
A. Medial pterygoid B. Superior constrictor
C. Temporalis D. Buccinator
23. Which of the following is true regarding bilateral mandibular block?
A. Is dangerous because patient may swallow tongue B. Will lead to space infection
C. Is not contraindicated D. Should rarely be performed
24. Which general anesthetic technique is used for anesthesia in oral surgery?
A. Open drop method B. Anesthesia with nasopharyngeal airway
C. Nasoendotracheal tube with throat pack D. IV anesthesia with nitrous oxide and oxygen
25. ______ is the longest-acting, most potent, and most toxic local anesthesia?
A. Lidocaine B. Dibucaine
C. Bupivacaine D. Tetracaine
26. Which of the following is used to prevent laryngospasm due to GA?
A. Atropine B. Epinephrine
C. Diazepam D. Succinylcholine
27. Anaesthesia given for extraction of mandibular molar is
A. Inferior alveolar nerve B. Buccal nerve
C. Lingual nerve D. Masseteric nerve
28. Contraindication to the use of local anesthetic agent is
A. Parkinson’s disease B. Liver damage
C. Pregnancy (third trimester) D. Hypersensitivity to the drug
29. Absolute contraindication of inferior alveolar nerve block is
A. Thrombocytopenia B. Hemophilia
C. Hypoprothrombinemia D. Von Willebrand’s disease
30. All of the following are features of myelinated nerve fibers except
A. Conduction is slower in myelinated than in nonmyelinated fibers
B. Current discharges at nodes of Ranvier
C. Outer layer is of lipids
D. Depolarization occurs only at nodes of Ranvier
31. Profound analgesia is produced by which parenteral anesthetic?
A. Halothane B. Propofol
C. Ketamine D. Etomidate
32. An agent added to local anesthetics to speed the onset of action is
A. Methylparaben B. Bicarbonate
C. Fentanyl D. Adrenaline
33. The following statements about bupivacaine are true except
A. Must never be injected into a vein B. More cardiotoxic than lignocaine
C. 0.5% is effective for sensory block D. It produces methemoglobinemia
34. Adrenaline is added to lignocaine to prolong its effect and decrease its absorption into blood stream in a ratio of
A. 1:50,000 B. 1:100,000
C. 1:200,000 D. 1:500,000

https://t.me/DentalBooksWorld
Chapter 7 • Oral Surgery 631

35. A patient with ruptured spleen is taken for laparotomy. His blood pressure is 80/50 and heart rate is 125/minute.
Induction agent of choice for this patient is
A. Sodium thiopentone B. Fentanyl
C. Ketamine D. Halothane
36. All of the following regarding xenon anesthesia are true except
A. Slow induction and recovery B. Nonexplosive
C. Minimal cardiovascular side-effects D. Low blood solubility
37. Anesthetic having epileptogenic potential is
A. Desflurane B. Sevoflurane
C. Ether D. Halothane
38. A patient with mitral stenosis had to undergo surgery. Preanesthetic checkup revealed the increased liver enzymes.
Which of the following inhalational agent should be preferred in this patient?
A. Xenon B. Enflurane
C. Halothane D. Sevoflurane
39. Propofol false is
A. Antiemetic B. Antipruritic
C. Injection not painful D. Choice for day care surgery
40. A 5-year-old child is suffering from cyanotic heart disease. He is planned for corrective surgery. The induction agent
of choice would be
A. Thiopentone B. Ketamine
C. Halothane D. Midazolam
41. Which of the following general anesthetics has poor muscle relaxant action?
A. Ether B. Nitrous oxide
C. Halothane D. Isoflurane
42. “Second gas effect” is exerted by which of the following gas when coadministered with halothane?
A. Nitrous oxide B. Cyclopropane
C. Nitrogen D. Helium
43. A young man having pheochromocytoma has BP of 188/92 mmHg and a hematocrit of around 50%. Pulmonary
function tests and renal functions are normal. His catecholamines are elevated. Which of the following drugs should
not be included in the anesthesia protocol?
A. Desflurane B. Fentanyl
C. Halothane D. Midazolam
44. While injecting LA, you mistakenly inject the solution into parotid gland and thus paralysis of face occurs. How do
you correct it?
A. Reposition the syringe barrel more posteriorly B. Reposition the syringe barrel more anteriorly
C. Remove, discard the solution, and reload D. Redirect the syringe barrel more laterally

https://t.me/DentalBooksWorld
632 Triumph's Complete Review of Dentistry

45. 5 minutes after injecting a local anesthetic, a patient experiences generalized warmth of the face, mouth, and upper
chest. What is the most likely cause?
A. Vagal shock B. Nervousness
C. Hypertension D. Anaphylactic reaction
46. Dose of epinephrine in case of anaphylaxis
A. 0.5 mg, 1:1,000, subcutaneous B. 0.5 mg, 1:1,000, IV
C. 0.5 mg, 1:10,000, subcutaneous D. 0.5 mg, 1:10,000, IV
47. The height of injection of Vazirani–Akinosi technique is
A. Below that of the Gow-gates but above that of the IANB B. Below that of the Gow-gates and the IANB
C. Above that of the Gow-gates and the IANB D. Above that of the Gow-gates but below that of the IANB
48. From which of the following routes, absorption of local anesthetic is maximum?
A. Intercostal B. Epidural
C. Brachial D. Caudal
49. Blockade of nerve conduction by a local anesthetic is characterized by
A. Greater potential to block a resting nerve as compared to a stimulated nerve
B. Need to cross the cell membrane to produce the block
C. Large myelinated fibers are blocked before the unmyelinated fibers
D. Cause consistent change of resting membrane potential
50. Which of the following induction agents produces cardiac stability?
A. Ketamine B. Etomidate
C. Propofol D. Midazolam
51. Anesthetic that has a smooth induction is
A. Diethyl ether B. Isoflurane
C. N2O D. Halothane
52. “MAC” of desflurane is
A. 1.15 B. 2
C. 4 D. 6
53. How many cartridges contains the maximum dose of epinephrine?
A. 6 cartridges B. 13 cartridges
C. 9 cartridges D. 16 cartridges

EXODONTIA AND IMPACTIONS


1. Which tooth needs rotatory movement during extraction?
A. Mandibular canine B. Maxillary central incisor
C. Maxillary lateral incisor D. All of the above
2. Bull’s eye appearance is in the radiograph of _______
A. Lingually placed mandibular III molar B. Buccally placed mandibular III molar
C. Inverted mandibular III molar D. Distoangular impaction
3. During extraction, pregnant lady in second trimester falls into syncope. Which position has to be followed?
A. Trendelenburg position B. Head down toward her feet
C. Right lateral position D. Left lateral position
4. Why does reactionary hemorrhage happen?
A. Broken roots B. High blood pressure
C. Sharp interdental septum D. Gingival laceration
5. The mechanical advantage obtained from the straight elevator is
A. 2.5 B. 3
C. 46 D. 6
6. In maxillary arch, which tooth poses difficulty during extraction?
A. Lateral incisor B. First premolar
C. Second premolar D. Canine

https://t.me/DentalBooksWorld
Chapter 7 • Oral Surgery 633

7. The displacement of a root into the maxillary sinus is most likely to take place during the extraction of _____
A. Canine B. Deciduous first molar
C. First molar D. Second molar
8. Most difficult tooth to extract among the following is
A. Maxillary canine B. Maxillary central incisor
C. Mandibular premolar D. Mandibular canine
9. The most valuable laboratory test used to assess the surgical risk of a patient who is on Dicoumarol therapy is
A. Clotting time B. Bleeding time
C. Prothrombin time D. Complete blood cell count
10. The dose of corticosteroids to be administered to the patient (who takes 60 mg of hydrocortisone daily) on the day
before extraction should be ____
A. 60 mg B. 30 mg
C. 120 mg D. 40 mg
11. What are the complications that may arise when an airotor of more than 40,000 rpm is used?
A. Necrosis of bone B. Tissue laceration
C. Tissue necrosis D. Emphysema
12. Medical history in oral surgery is necessary to
A. In medicolegal cases B. To assess the growth stage of a patient
C. To determine bleeding disorder D. To determine communicable disease
13. What are the complications expected during extraction for a patient of liver disease?
A. Dry socket B. Fascial space infection
C. Bleeding D. Loss of clot
14. Dry socket
A. Results from loss of blood clot in the socket B. Is treated with reinducing bleeding into the socket
C. Is a form of osteomyelitis D. Is common in extraction of anterior teeth
15. A patient develops trismus after 4 weeks of extraction. Probable cause is
A. Breakage of needle in pterygomandibular space B. Hematoma of TMJ
C. Submasseteric space abscess D. Root stump in the socket
16. Time duration of reactionary hemorrhage is
A. Within 24 hours B. After 24 hours
C. After 72 hours D. After 7 days
17. Which elevator fits well in the operator’s hand and can be rotated between the thumb and the finger?
A. Coupland elevator B. Hospital pattern elevator
C. Winter’s elevator D. Warwick-James elevator
18. Which of the following is an absolute contraindication for extraction of teeth?
A. Hypertension B. Myocardial infarction
C. Thyrotoxicosis D. Central hemangioma
19. A patient had myocardial infarction 2 months back. What are the precautions to be taken prior to extraction?
A. Performed under oral vacuum sedation
B. Performed using an epinephrine-free local anesthetic
C. Performed using both of the above
D. Postponed until at least 6 months have elapsed
20. The principle utilized by Apex elevator is
A. Wedge B. Pulley
C. Wheel and axle D. None of the above
21. While extracting a mandibular third molar, it is noted that the distal root is missing. The root tip is most likely in the
A. Submental space B. Submandibular space
C. Parapharyngeal space D. Pterygomandibular space

https://t.me/DentalBooksWorld
634 Triumph's Complete Review of Dentistry

22. The blue black spots seen on the neck of the patients after mouth extraction indicate
A. Thrombocytopenic purpura B. Postoperative ecchymosis
C. Impaired blood circulation D. Cellulitis
23. When can we administer analgesics during extraction?
A. Before anesthesia wears off B. Prior to extraction
C. When pain is moderate to severe D. After anesthesia wears off
24. What happens when tooth is extracted during acute infection?
A. Can cause extensive spread of infection
B. Helps drainage and relieves pain if proper antibiotic is given and its adequate blood level is reached
C. Can cause sudden death due to pulmonary embolism
D. Can lead to trigeminal neuralgia in postoperative period
25. Most important principle during extraction
A. Least trauma to bone white extracting whole tooth out
B. Least trauma to mucosa while extracting the whole tooth out
C. Least trauma to both bone and mucosa while extracting the tooth in pieces
D. None of the above
26. The force first directed when a forceps is to be utilized for removal of a tooth is
A. Occlusally B. Buccally
C. Lingually D. Apically
27. Alveolar osteitis after dental extraction involves the following treatment
A. Topical antibiotics B. Systemic antibiotics
C. Debridement of socket and sedative dressing D. Curettage to induce fresh bleeding
28. The elevator can be used to advantage when
A. The tooth to be extracted is isolated B. The interdental bone is used as a fulcrum
C. The adjacent tooth is not to be extracted D. Multiple adjacent teeth are to be extracted
29. During extraction, it is seen that the tuberosity is fractured but remains attached to the mucoperiosteum with an
intact blood supply. The treatment is
A. Remove the tuberosity
B. Refer to an oral surgeon
C. Elevate a flap and do transosseous wiring
D. Reposition the fragments and stabilize with sutures
30. Contraindication for coronectomy is
A. Horizontal B. Vertical
C. Mesioangular D. Distoangular
31. Carnoy’s solution is composed of all except
A. Acetic acid B. Ferric chloride
C. Alcohol D. Glycerin
32. “Postage Stamp” method is
A. A method of bone removal in transalveolar extraction
B. A method of extraction of maxillary canines by intraalveolar method extraction
C. A method of bone grafting
D. None of the above
33. “Odontectomy” is synonymous to
A. Transalveolar extraction B. Intraalveolar extraction
C. Extraction under local anesthesia D. Extraction under general anesthesia
34. “Line of withdrawal” of a tooth is mainly determined by
A. The size of the crown B. The root pattern
C. Shape of the crown D. Size of the roots
35. The maxillary first molar is extracted by forceps method. The healing of the socket can be described as
A. Healing by primary intention B. Granulomatosis
C. Healing by secondary intention D. Epithelialization

https://t.me/DentalBooksWorld
Chapter 7 • Oral Surgery 635

36. Ten teeth have been removed for a patient who was premedicated. The proper position of the patient in the recovery
room is
A. Head elevation 30 degrees with patient on his back
B. Trendelenburg position, patient head is almost parallel to the floor
C. Reverse Trendelenburg position
D. Supine position
37. In vasovagal syncope, which of the following does not occur?
A. Hypotension and tachycardia B. Constriction of pupil
C. Vomiting D. Cold extremities
38. During the extraction of maxillary first molar, the palatal root tip of molar is slipped into the maxillary sinus. Proper
way to approach to recovery is through the
A. Same socket by enlarging the opening through which the root entered the sinus
B. Maxillary incisive fossa
C. Maxillary incisive fossa medial to canine
D. Lateral nasal wall in the middle meatus of the nose
39. Composition of Talbot’s solution is
A. Iodine, zinc iodine, glycerin, and water B. Chlorine, zinc chloride, glycerin, and water
C. Fluorine, zinc fluoride, glycerin, and water D. Chlorates, zinc chlorate, glycerin, and water
40. In modified Ward’s incision, the incision commences at the
A. Distobuccal corner of mandibular first molar B. Distobuccal corner of mandibular second molar
C. Mesiobuccal corner of mandibular first molar D. Mesiobuccal corner of mandibular second molar
41. Orthodontic theory of impaction was given by
A. Burstone B. Wall
C. Durbeck D. Willis
42. The scoring criteria for mesioangular, position B, and class 2 impaction is
A. 4 B. 5
C. 6 D. 2
43. Apical notch in radiograph is commonly is seen in the following impactions
A. Mesioangular and distoangular impaction B. Vertical and horizontal impaction
C. Mesioangular and horizontal impaction D. Horizontal and distoangular impaction
44. Which maxillary impactions are most likely to be displaced into the antrum or the infratemporal fossa with incorrect
technique during the extraction?
A. Distoangular B. Mesioangular
C. Vertical D. Faciolingual
45. During the dental procedure, suddenly patient became “light headed, diaphoretic,” and then became unconscious.
The diagnosis is
A. Shock B. Syncope
C. CVA D. Hyperventilation
46. Incidence of dry socket after surgical removal of impacted mandibular third molar is approximately
A. 25% B. 50%
C. 1% D. 10%

MANDIBULAR FRACTURES
1. All are true about mandibular fracture except
A. Fractures of the mandible are common at the angle of the mandible
B. Fractures of the mandible are effected by the muscle pull
C. Fractures of the mandible are usually characterized by sublingual hematoma
D. CSF rhinorrhea is a common finding

https://t.me/DentalBooksWorld
636 Triumph's Complete Review of Dentistry

2. Treatment for fracture of angle of mandible is


A. Transosseous wiring B. Intermaxillary fixation
C. Plating on the lateral side of the body of the mandible D. Plating at the inferior border of the mandible
3. A child has left subcondylar region fracture with occlusion undisturbed. Treatment would be
A. Immobilization for 7 days
B. Immobilization for 14 days with intermittent active opening
C. No immobilization with restricted mouth opening for 10 days
D. No immobilization and active treatment
4. Mandibular fracture must be immobilized for
A. 3 weeks B. 6 weeks
C. 9 weeks D. 12 weeks
5. Most common complications of condylar fractures in children are
A. Pain B. Ankylosis
C. Osteoarthritis D. Fracture of glenoid fossa
6. The condyle moves in _______ case of subcondylar fracture
A. Anterior—lateral direction B. Posterior—medial direction
C. Posterior—lateral direction D. Anterior—medial direction
7. Fracture that occurs in the tooth-bearing segment or runs along the periodontal ligament is known as
A. Simple B. Complex
C. Compound D. Comminuted
8. The proximal segment of mandibular angle fracture is usually displaced in which direction?
A. Anterior and superior B. Posterior and interior
C. Interior only D. Posterior and superior
9. Management of displaced mandibular fracture in a child is
A. Circummandibular wiring B. Early mobilization
C. Intermaxillary fixation D. Transosseous wiring
10. Management of fracture of the mandible in the canine region of a 6-year-old child should be
A. Cap splint fixation B. Intermaxillary fixation
C. Risdon wiring D. Transosseous wiring
11. In mandible the most common site to be fractured is the
A. Body B. Angle
C. Symphysis D. Condyle
12. Unfavorable fracture of the angle of mandible is treated by
A. Closed reduction with intermaxillary fixation B. Closed reduction with cap splints
C. Open reduction with interosseous wiring D. Open reduction with rigid bone fixation
13. Bucket handle type of fractures is seen in
A. Children B. Soldiers
C. Edentulous persons D. Young adults
14. _______ displaces mandibular condyle forward and medially
A. Temporalis B. External pterygoid
C. Internal pterygoid D. Masseter
15. Primary healing of a mandibular fracture is seen following fixation with
A. Gunning splints B. Compression plates
C. Transosseous wires D. Clampy plates
16. Eburnation is seen in
A. Malunion B. Nonunion
C. Osteomyelitis D. Osteoradionecrosis

https://t.me/DentalBooksWorld
Chapter 7 • Oral Surgery 637

17. Paresthesia of lower lip occurs in


A. Body fracture B. Symphysis fracture
C. Coronoid fracture D. Condyle fracture
18. Green stick fractures are commonly seen in
A. Older people B. Adult
C. Children D. Soldiers
19. Treatment of mandibular fracture between the incisors is
A. Risdon wiring B. Essig wiring
C. Cap splint with circummandibular wiring D. Transosseous wiring
20. Compression osteosynthesis heals mandibular fracture by
A. Primary union without callus formation B. Secondary union without callus formation
C. Compression union D. All of the above
21. Indication for open reduction and fixation of mandibular fracture is
A. Shortening of ramal height is more than 5 mm B. Superior 5 mm condyle is fractured
C. Green stick fracture in children D. Condylar fracture which is not dislocated
22. ______ is the most common pathognomonic sign of mandibular fracture
A. Malocclusion B. Sublingual hematoma
C. Deviation of the jaw on opening D. Paresthesia of the mental nerve
23. Other name for direct interdental wiring
A. Risdon’s wiring B. Gilmer’s wiring
C. Eyelet wiring D. Col Stouts wiring
24. Clinical sign of an adult patient who sustained a subcondylar fracture on the left side is
A. Moderate intraoral bleeding B. Trismus and bilateral crepitus
C. Deviation of the mandible to the right on protrusion D. Inability to deviate the mandible to the right
25. The weakest point of the mandible where fracture occurs is
A. Neck of the mandible B. Angle of the mandible
C. Symphysis menti D. Oblique ridge near mental foramen
26. Clinical persistent sign in bone fracture is
A. Crepitus B. Tenderness
C. Abnormal mobility D. All of the above
27. Mandibular fracture can be viewed by which of the following extraoral radiograph?
A. Submentovertex B. Posteroanterior
C. Water’s D. Towne’s
28. Postauricular ecchymosis seen in cases of the basal skull fracture of skull is called
A. Battle’s sign B. Tinel’s sign
C. Trousseau’s sign D. Nikolsky’s sign
29. Direct impact on the bone will result in
A. Transverse fracture B. Oblique fracture
C. Spiral fracture D. Comminuted fracture
30. Right side deviation of mandible may suggest
A. Fracture of left condyle B. Hyperplasia of right condyle
C. Hypoplasia of left condyle D. Fracture of right condyle
31. Linear fracture of mandible can be best treated by
A. Transosseous wiring B. Champy’s plate
C. Reconstruction plate D. IMF
32. All are contraindications of lag screw except
A. Oblique fractures
B. Comminuted fractures
C. Patients with diminished or poor bone quality
D. Fractures within the symphysis and parasymphysis region

https://t.me/DentalBooksWorld
638 Triumph's Complete Review of Dentistry

33. Most common region of dentate mandible to fracture is


A. Body B. Angle
C. Condyle D. Symphysis
34. Management for the fracture shown in image


A. Extraction of 38 and 48 followed by superior border plating
B. Retain 38 and 48, superior border plating
C. Extract 38 and 48, inferior and superior border plating
D. Retain 38 and 48, inferior border plating
35. All are true regarding lag screw fixation except
A. Compression osteosynthesis B. Rigid fixation
C. Load bearing D. Primary healing
36. All are true regarding Dolan’s and Jacoby fracture lines are true except
A. Palatal line B. Orbital line
C. Maxillary line D. Zygomatic line
37. Coleman sign is seen in fracture of
A. Mandible body B. Mandible condyle
C. Le Fort I D. Le Fort II and III

MIDDLE THIRD FRACTURES


1. Le Fort III resembles
A. Craniofacial dysjunction B. Guerin’s fracture
C. Pyramidal fracture D. None of the above
2. A 19-year-old woman collapses and develops tonic–clonic seizure lasting 2 minutes. On examination, she utters few
words which were meaningless. When the examiner applied pressure on her finger, she opened her eyes and prevented
the examiner from doing that. What is her Glasgow coma scale (GCS) score?
A. 12 B. 11
C. 10 D. 9
3. In which of the following is CSF rhinorrhea seen?
A. Frontal bone structure B. Zygomatico maxillary fracture
C. Nasoethmoidal fracture D. Condylar fracture
4. Damage of _____ causes epiphora
A. Ciliary ganglion B. Greater palatine nerve
C. Infraorbital nerve D. None of the above
5. The first step in head injury management is
A. Secure airway B. IV mannitol
C. IV dexamethasone D. Blood transfusion
6. The following are features of Le Fort II fracture except
A. Enophthalmos B. Malocclusion
C. Paresthesia D. CSF rhinorrhea

https://t.me/DentalBooksWorld
Chapter 7 • Oral Surgery 639

7. Guerin fracture is
A. Maxillary fracture B. Maxillary and zygomatic fracture
C. Maxillary and nasal bone fracture D. Nasal bone fracture only
8. A ping ball causes _____ fracture of eye
A. Blow out fracture B. Orbital fracture
C. Blow in fracture D. Compound fracture
9. “Panda fades” is a common feature of
A. Le Fort I fractures B. Le Fort II fractures
C. Mandible fractures D. None of the above
10. Walsham’s forceps is used to
A. Remove teeth B. Remove root
C. Clamp blood vessels D. Reduce nasal bone fractures
11. Treatment for a patient who has gross comminuted fracture and shock is
A. Normal saline B. Ringer’s lactate solution
C. Whole blood D. Plasma expanders
12. ____ forceps is used for maxillary disimpaction
A. Rowe’s B. Bristow’s
C. Ash’s D. Walsham’s
13. _____ fractures show paresthesia
A. Subcondylar B. Zygomatico maxillary
C. Coronoid process D. Symphyseal.
14. Diplopia is most common with
A. Mandibular fracture B. Craniofacial dysjunction
C. Nasal fractures D. Zygomatico maxillary complex
15. Facial injuries should be sutured within
A. 2 hours B. 6 hours
C. 4 hours D. 8 hours
16. The “hanging drop appearance” in the maxillary sinus radiograph indicates
A. A nasal polyp B. A blow out of the orbit
C. A radiograph artifact D. An antrolith
17. Which poses as a potential danger to a patient with severe facial injuries?
A. Bleeding B. Infection
C. Associated fracture spine D. Respiratory obstruction
18. In order to open up airway in a patient with maxillofacial trauma, the best approach is
A. Head tilt-chin tilt B. Jaw thrust technique
C. Head lift–neck lift D. Heimlich procedure
19. ______ causes difficulty in opening mouth by impingement in depressed zygomatic arch fracture
A. Condyles B. Ramus
C. Petrous temporal D. Coronoid process
20. Which of the following causes true open bite?
A. Horizontal fracture of the maxilla
B. Unilateral fracture of mandibular angle
C. Fracture of the coronoid process of left side of mandible
D. Fracture of mandibular symphysis
21. All are features of nasal fractures except
A. Even if minor, they may be followed by bilateral ecchymosis and facial edema
B. They may need to be reduced for a few weeks
C. They need not be complicated by traumatic telecanthus
D. They may lead to the telescoping of the nasal complex into the frontal sinus

https://t.me/DentalBooksWorld
640 Triumph's Complete Review of Dentistry

22. The rationale behind fixing a pack in maxillary sinus is


A. To support comminuted fracture of the body of zygomatic complex
B. To support and reconstitute comminuted orbital floor fracture
C. To protect mucosat covering of maxillary sinus
D. (A) and (B) are correct
23. Floating maxilla is typically seen in
A. Le Fort I or Guerin fractures B. Le Fort II or pyramidal fractures
C. Craniomandibular dysjunction D. All of the above
24. First thing to be examined in a patient with head injury is
A. Pupillary light reflex B. Pupillary size
C. Corneal reflex D. Ability to open eye
25. Airway obstruction is diagnosed by which of the following sign?
A. Slow pounding pulse B. Strenuous breathing
C. Increase in pulse rate D. Decrease in blood pressure
26. “Moon face” appearance is seen in
A. Isolated Le Fort I fracture B. Le Fort II and Le Fort III fractures
C. Mandibular fractures D. Unilateral zygomatic complex fractures
27. Bleeding from ______ is arrested by placing whitehead varnish in gauze
A. Gingival crest B. Bleeding from pulp
C. Bleeding from bone D. Bleeding from capillaries
28. Identify the intubation tube


A. North pole tube B. South pole tube
C. Endotracheal tube D. Combitube
29. Choice of intubation in a patient with Le Fort II, Le Fort III, and nasoethmoidal fracture would be
A. Oral B. Oral and nasal
C. Nasal D. Submental
30. Choice of intubation in TMJ ankylosis is
A. Fiberoptic B. Flexometallic
C. Endotracheal D. Nasal
31. Intubation contraindicated in basal skull fracture is
A. Endotracheal B. Nasal
C. Oral D. Fiberoptic
32. Merits of nasotracheal intubation
A. Good oral hygiene B. Less infection
C. Less mucosal damage and bleeding D. More movements or displacement of endotracheal tube
33. Which of the following is used for controlling bleeding in maxillary osteotomy?
A. Tranexamic acid B. Heparin
C. Acetoacetic acid D. Citric acid
34. Blood supply after Le Fort I osteotomy is preserved by
A. Greater palatine artery B. Ascending pharyngeal artery
C. Posterior superior alveolar artery D. Nasopalatine artery

https://t.me/DentalBooksWorld
Chapter 7 • Oral Surgery 641

CYSTS AND TUMORS


1. Which of the following is a disadvantage of marsupialization?
A. Exposure of very little bone B. Preservation of vital structure
C. Rapid healing D. Conserve surgical structures
2. The rationale behind needle aspiration done in case of a central bone lesion is
A. To rule out a vascular lesion B. To determine thickness of buccal plate
C. To diagnose traumatic bone cyst D. To feel root surfaces
3. There is 3-cm ameloblastoma involving the inferior border of the mandible. What would be the treatment plan?
A. Enucleation B. Fulguration
C. Cryosurgery D. Resection
4. OKC has which of the characteristic aspirate feature?
A. A low-soluble protein content B. A high-soluble protein content
C. Cholesterol crystal D. Inflammatory cells
5. An empty cavity in the mandible with no lining is mostly likely to be
A. Aneurysmal bone cyst B. Idiopathic bone cavity
C. Dentigerous cyst D. Keratocyst
6. Identify the cyst


A. Dentigerous B. Lateral periodontal
C. Odontogenic keratocyst D. Radicular
7. Surgical procedure of choice for odontogenic cysts is
A. Incision and drainage B. Sclerosing solution
C. Marsupialization D. Enucleation
8. The procedure in which cyst is deroofed and surrounding periosteum is sutured to margins in cyst wall is
A. Decortication B. Marsupialization
C. Saucerization D. Enucleation
9. How can we differentiate between ameloblastoma and dentigerous cyst?
A. Radiographic examination B. Aspiration cytology
C. Microscopic examination D. Clinical features
10. Management of fibrous dysplasia is
A. Radical resection of lesion
B. Radiation therapy
C. If the lesion is small, dissection is done; if lesion is large, cosmetic surgery has to be carried out
D. Cryosurgery
11. Which odontogenic tumor recurs when treated with simple curettage?
A. Complex odontoma B. Compound odontoma
C. Odontogenic myxoma D. Ameloblastic fibroma

https://t.me/DentalBooksWorld
642 Triumph's Complete Review of Dentistry

12. Treatment of choice for odontoma


A. Excision B. Resection
C. Curettage D. Radiotherapy
13. Ameloblastoma is treated by
A. Excision of tumor B. Resection of the mandible along with the tumor
C. Incisional biopsy and marsupialization D. No active treatment is necessary
14. Treatment of large cysts is
A. Enucleation B. Marsupialization
C. Marsupialization followed by enucleation D. Enucleation followed by marsupialization
15. A 5-cm suspicious looking lesion of oral mucosa should be
A. Incised and sent for biopsy B. Excised and sent for biopsy
C. Irradiated D. Offered palliative treatment
16. Biopsy specimens are immediately placed in ______ for examinations
A. 10% ethanol B. 10% formalin
C. Hydrogen peroxide D. 1% formalin
17. Protein content of <4 mg/mL is seen in
A. Dentigerous B. Periapical
C. Keratocyst D. Periodontal cyst
18. What will happen when palatine tumor is enucleated?
A. Excessive bleeding from nasopalatine vessels B. Tearing of nasal mucosa
C. Damage to nasopalatine nerve D. Alteration of speech
19. Adenoameloblastoma is best treated by
A. En bloc resection of maxilla B. Marsupialization
C. Enucleation D. No treatment
20. Odontogenic tumors just 1 cm away from lower border is treated by
A. En bloc resection B. Hemi mandibulectomy
C. Enucleation D. None
21. Which among the following includes marsupialization procedure?
A. Large cyst B. Dental cyst
C. Aneurysmal bone cyst D. Stafne cyst
22. Treatment for fibrous dysplasia in a young 25-year-old patient involving maxilla is best treated by
A. En bloc resection B. Cosmetic contouring
C. Maxillary resection D. Radiation therapy
23. Submandibular calculus can be removed by
A. Dilatation of the duct B. Excision of the opening of duct
C. Removal of the gland D. Incision of the duct and removal of calculus
24. Which structure is more likely to be damaged during surgical excision of the parotid gland?
A. Lesser occipital nerve, hypoglossal nerve, chorda tympani
B. Facial nerve and auriculotemporal nerve
C. Submandibular duct
D. Cervical fascia
25. Salivary calculus is more common in
A. Submandibular gland B. Sublingual gland
C. Parotid gland D. Minor salivary gland
26. Which cyst shows arcading pattern?
A. Odontogenic keratocyst B. Dentigerous cyst
C. Radicular cyst D. Calcifying odontogenic cyst
27. During excision of the submandibular gland for calculus or tumors, incision is given below angle of the jaw.
What is the rationale behind it?
A. Ansa cervicalis B. Mandibular branch of facial nerve
C. Posterior auricular nerve D. Submandibular ganglion

https://t.me/DentalBooksWorld
Chapter 7 • Oral Surgery 643

28. Squamous cell carcinoma of lateral border of tongue. T2N0M0 treatment?


A. SOHND B. Level 1–2 lymph node excision
C. Level 3–4 lymph node excision D. Level 1–4 lymph node excision
29. T1N0M1 staging
A. Stage 1 B. Stage 2
C. Stage 3 D. Stage 4
30. A 45-year-old patient presents with a 4 × 5 cm mass in the neck. Histopathological examination showed metastasis of
squamous cell origin. Patient was thoroughly examined using laryngoscope, whole body CT, and PET CT. No primary
is found and the diagnosis of carcinoma of unknown primary is made. TNM staging as per AJCC is
A. T1N1M0 B. T1N2M0
C. T0N1M0 D. T0N2M0
31. Treatment for carcinoma tongue is
A. Surgery + radiotherapy B. Surgery + chemotherapy
C. Only surgery D. Only radiotherapy
32. The procedure done here is

A. Commando’s operation B. Hemimandibulectomy


C. Segmental resection D. Marginal resection

ODONTOGENIC INFECTIONS
1. Lateral pharyngeal space infection cause trismus because of the irritation of
A. Buccinator B. Masseter
C. Lateral pterygoid D. Medial pterygoid
2. Garre’s osteomyelitis is
A. Chronic focal sclerosis and nonsupportive osteomyelitis
B. Chronic focal sclerosis and suppurative osteomyelitis
C. Characterized by suppuration and acute pain
D. Chronic diffuse sclerosing osteomyelitis
3. Death occurs in Ludwig’s angina when there is
A. Sepsis B. Respiratory obstruction
C. Cavernous sinus thrombosis D. Carotid blow-out
4. Which muscle forms the roof of pterygomandibular space?
A. Temporalis muscle B. Medial pterygoid muscle
C. Cranial base D. Lateral pterygoid
5. Which of the following can cause retropharyngeal space infection?
A. Cervical tuberculosis B. Meningoencephalitis
C. Mumps D. Odontogenic infections

https://t.me/DentalBooksWorld
644 Triumph's Complete Review of Dentistry

6. Masticatory space is distinguished by


A. Pain B. Dysphagia
C. Trismus D. Swelling
7. Which one among the following acts as a barrier?
A. Connective tissue B. Epithelium
C. Muscle D. Fascia
8. Osteoradionecrosis is because of
A. Infection B. Endarteritis of blood vessels
C. Sepsis D. None of the above
9. A mandibular dental infection which exists in the buccal cortical plate above the muscle attachment has
affected ______
A. Buccal space B. Masseter space
C. Vestibular sulcus D. Masticatory space
10. All of the following are primary mandibular space except
A. Buccal B. Sublingual
C. Submandibular D. Pterygomandibular
11. Treatment for Garre’s osteomyelitis is
A. Incision and drainage B. Sequestrectomy
C. Saucerization D. Surgical recontouring
12. The most common cause of odontogenic infections are
A. Mixed bacteria B. Anaerobic bacteria
C. Aerobic bacteria D. Streptococci
13. Infection from pericoronal area of lower third molar spreads mostly to ___ space
A. Submandibular B. Submental space
C. Pterygomandibular D. Buccal space
14. Involucrum is
A. Dead bone B. New live bone surrounding dead bone
C. Previous live bone D. Sclerotic bone
15. Styloid process divides ____ space into anterior and posterior compartments
A. Pterygomandibular B. Lateral pharyngeal
C. Retropharyngeal D. Infratemporal
16. Important clinical sign of masticatory space infection is
A. Trismus B. Xerostomia
C. Difficulty in swallowing D. Swelling in submental area
17. Which of the following suggest postoperative infection except
A. Increase in temperature B. Swelling
C. Pain D. Pitting edema
18. Characteristic feature of Ludwig’s angina is
A. Raised tongue B. Elevation of ear lobe
C. Trismus D. Unilateral swelling
19. A 19-year-old patient reported with swelling over the left angle of the mandible, temperature of 38°C and
negative history of trauma. Probable diagnosis is
A. Spontaneous fracture of the mandible B. Pericoronal infection
C. Mumps D. Sjogren’s syndrome
20. Osteomyelitis
A. Never occurs in infants B. In acute cases fracture mandible is very common
C. Of mandible can show symptoms of lip paresthesia D. Produces no lymphadenopathy
21. How does sequestrum show up in radiograph?
A. More radiolucent B. More radiopaque
C. With osteophytes growing out D. Soap-bubble appearance

https://t.me/DentalBooksWorld
Chapter 7 • Oral Surgery 645

22. Spread of infection from apical abscess to ______ possess danger


A. Infratemporal fossa B. Pterygoid
C. Parapharyngeal space D. Submandibular space
23. Among the following, for which disease is hyperbaric oxygen indicated?
A. Obstructive lung diseases B. Osteoradionecrosis
C. Cardiac failure D. Renal diseases
24. ______ spreads infection from anterior maxillary teeth and causes cavernous sinus thrombosis.
A. Facial artery B. Angular artery
C. Ophthalmic vein D. Pterygoid plexus
25. A tender swelling in submandibular triangle is mostly likely diagnosed as
A. Lymphadenopathy B. Ludwig’s angina
C. Phlegmon D. None of the above
26. A patient has an infectious process associated with maxillary right lateral incisor; the tooth is deeply carious and
nonrestorable. His temperature is 102°F. The patient has not been able to chew for the past 24 hours. The right side
of the face is swollen. The patient cannot open his right eye, on palpation the area is soft, painful to touch and tissue
rebound when palpated; treatment for the patient should be
A. Aspiration B. Antibiotics and heat
C. Incision and drainage D. Antibiotics, heat, and fluid
27. Infection of masticatory space is usually associated with
A. Tonsillar abscess B. Mandibular molar
C. Lateral pharyngeal space D. Parotid space infection
28. A diagnosis of cavernous sinus thrombosis is made on the basis of
1. Known site of infection 2. Septicemia
3. Venous obstruction in retina conjunctiva or eyelid 4. Paresis of third, fourth, and sixth nerves
5. Abscess formation in neighboring soft tissues 6. Nuchal rigidity
A. 1, 2, 3, 4, 5 B. 3, 4, 5
C. 1, 2, 5, 6 D. All the above correct
29. Treatment procedure for osteomyelitis of jaw is
A. Resection B. Physiotherapy
C. Sequestrectomy with antibiotic treatment D. Drainage
30. All are connected directly to the masticatory space infection except
A. Buccal space B. Sublingual space
C. Submandibular space D. Retropharyngeal space
31. Which muscle divides the superior part of submasseteric space into two portions?
A. Temporalis B. Masseter
C. Pterygoid D. Buccinator

TMJ AND MAXILLARY SINUS


1. Bony fusion of condylar head to the glenoid fossa is due to
A. Trauma B. Atrophy
C. Development disturbances D. Microorganisms
2. Frequent dislocation of TMJ is caused due to
A. spasm of masseter B. The articular eminence being smaller
C. Bony ankylosis D. All of the above
3. Dautery procedure, which is done by intentional fracture of zygomatic process, is used to treat?
A. MPDS B. TMJ ankylosis
C. TMJ dislocation D. TMJ internal derangement
4. Surgical procedure to treat TMJ ankylosis is
A. Capsulorrhaphy B. Discoplasty
C. Condylectomy D. Arthroplasty

https://t.me/DentalBooksWorld
646 Triumph's Complete Review of Dentistry

5. Double contrast arthroscopy in TMJ is used for


A. Clicking of TMJ B. Deviation of TMJ
C. Joint space D. Disc
6. Maxillary sinusitis shows which of the following symptoms?
A. Tenderness over the involved area B. Postnasal drip
C. Change in phonation D. All of the above
7. All are features of TMJ dislocation except
A. Anterior displacement of condyles
B. Loss of hearing
C. Apertognathia
D. Can be reduced by applying pressure on the mandible in downward and backward direction
8. We use Berger flap to treat?
A. TMJ dislocation B. TMJ bony ankylosis
C. Oroantral communication. D. TMJ fibrous ankylosis
9. Condylar dislocation can be treated by
A. Forcing mandible upward and backward B. Forcing mandible upward and forward
C. Forcing mandible downward and forward D. Forcing mandible downward and backward
10. Trismus results due to the inflammation of
A. Buccinator B. Facial nerve irritation
C. Superior constrictor D. medial pterygoid
11. Eminectomy is used to treat the condition
A. TMJ ankylosis B. TMJ dislocation
C. Coronoid fracture D. All of the above
12. On giving submandibular incision, ____ nerve is commonly affected
A. Marginal mandibular branch of the facial nerve B. Hypoglossal nerve
C. Mylohyoid nerve D. Lingual nerve
13. All of the following are features are unilateral TMJ ankylosis except
A. Multiple carious teeth
B. Facial asymmetry with fullness on the normal side of mandible
C. Chin deviated toward the affected side
D. Prominent antegonial notch on the affected side
14. Treatment for an 8-year-old child with TMJ ankylosis would be
A. Treatment not required B. Condylectomy
C. Gap arthroplasty with costochondral grafting D. Gap arthroplasty alone
15. TMJ dislocates most commonly in ______ direction
A. Lateral B. Posterior
C. Medial D. Anterior
16. Examination of joint cavity without surgical exposure is known as
A. Sialography B. Arthroscopy
C. Biopsy D. Endoscopy
17. The treatment of choice when a maxillary root tip is left behind in the maxillary sinus during extraction is
A. Perform Caldwell Luc to remove the tip B. Perform hemi maxillectomy
C. Enlarge opening in the socket area D. No treatment indicated
18. During Caldwell procedure, _______ is entered to gain access to the sinus.
A. Malar eminence B. Canine fossa
C. Tuberosity D. Zygomatic ridge
19. Oroantral fistula most commonly occurs during extraction of
A. Maxillary first premolar B. Maxillary second premolar
C. Maxillary first molar D. Maxillary third molar

https://t.me/DentalBooksWorld
Chapter 7 • Oral Surgery 647

20. An oroantral communication has been established during extraction


A. Does not require any treatment
B. Immediate prolapse of antral lining occurs into mouth
C. Reflux of fluids into the nasal cavity while drinking can be demonstrated
D. Close it immediately
21. The maxillary sinus drains into the
A. Middle meatus B. Inferior meatus
C. Superior meatus D. Sphenoethmoidal recess
22. What happens when corticosteroids are injected into a painful joint?
A. Anesthetize the nerve supply B. Lubricate the synovial joint
C. Increase the blood supply D. Decrease the inflammatory response
23. ______ forms the entrance for performing nasal antrostomy
A. Middle concha B. Inferior concha
C. Middle meatus D. Inferior meatus
24. During maxillary first molar extraction, a perforation of about 0.5 mm was made into maxillary sinus. What will be
the next step to treat the perforation?
A. Caldwell luc procedure should be performed immediately
B. Do alveoloplasty
C. Pack the opening with iodoform gauge and allow to heal by secondary intention
D. No treatment necessary
25. Above which of the following teeth does maxillary sinus lie?
A. Incisors B. Canine
C. Premolars D. Molars
26. The most commonly involved tooth which dislodges materials into the maxillary sinus due to overfilling is
A. Maxillary second premolar B. Maxillary first molar
C. Maxillary first premolar D. Facial root maxillary first premolar
27. How will maxillary sinusitis present on radiograph?
A. Fluid levels B. Clouding of antra
C. Erosion of bone D. Clouding and fluid level
28. Best flap used for oroantral fistula in maxillary third molar area is
A. Buccal mucoperiosteal flap B. Bridge flap
C. Palatal island flap D. Palatal mucoperiosteal flap
29. If a healthy person has oroantral fistula of 1 cm since 1 month, what will be the treatment?
A. Obturator for an indefinite period B. Refresh margins and primary closure immediately
C. Immediate raise flap for closure D. Before closure, treat for chronic sinusitis
30. In dislocation of the jaw, displacement of the articular disc beyond the articular tubercle of the temporomandibular
joint results from spasm or excessive contraction of which of the following muscle?
A. Buccinator B. Lateral pterygoid
C. Masseter D. Temporalis
31. Costochondral graft has the disadvantage of
A. Overgrowth B. Very less growth
C. No growth D. Delayed growth
32. The tear of intracapsular articular disc of TMJ causes
A. Dislocation B. Ankylosis
C. Chronic pain D. Arthritis

PREPROSTHETIC AND ORTHOGNATHIC SURGERY


1. Purpose of genioplasty is
A. To change the attachment of genioglossus muscle in preprosthetic procedure
B. To change the position of genial tubercies

https://t.me/DentalBooksWorld
648 Triumph's Complete Review of Dentistry

C. To modify the position of the chin


D. To modify the attachment of anterior belly of digastric
2. Which of the following is false for a flap required for maxillary tuberosity except
A. Incision is given over the alveolar crest B. A wedge-shaped incision may be first tried
C. Incision extends to the bicuspid cuspid region D. None the above
3. Treatment of choice for bimaxillary protrusion is
A. Removal of four premolars and anterior alveolar segment repositioning
B. Mandibular body ostectomy and posterior maxillary osteotomy
C. Step osteotomy of the mandibular body
D. Subcondylar osteotomy
4. Sagittal split osteotomy is a procedure done in the mandibular
A. Ramus B. Body
C. Symphysis D. Angle
5. Long-term results have been proved by which type of genioplasty?
A. Pedicled horizontal sliding osteotomy B. Injection of silastic gel
C. Insertion of silastic rubber implant D. Onlay bone graft
6. Why are myoplasty and sulcus extension procedures commonly performed?
A. It increases retention B. It increases stability
C. It increases support D. It increases retention and stability
7. The procedure wherein the alveolar ridges are recontoured is known as
A. Alveoloplasty B. Fiberectomy
C. Mucogingivectomy D. Alveolectomy
8. VY plasty procedure is used for
A. Mucogingival surgery B. Ridge augmentation
C. Frenectomy D. Vestibuloplasty
9. Which procedure is done in augmented genioplasty?
A. Sliding horizontal osteotomy B. Bone grafts
C. Silicone implants D. Temporalis muscle
10. What would happen when palatal bone is fractured during torus palatinus removal?
A. An opening into the nasal cavity B. An opening into the maxillary antrum
C. Vertical fracture of maxilla D. Horizontal fracture of maxilla
11. What should be done if retromolar pad and tuberosity contact with each other?
A. Surgical reduction of tuberosity B. Do not extend dentures to retromolar area
C. Denture fabrication not possible D. Has no effect on denture
12. While performing skin graft, vestibuloplasty relapse is prevented by
A. Forming a barrier to reattachment of muscle to periosteum
B. Promoting osteogenesis to establish a bone barrier to loss of sulcus depth
C. Causing an inhibitory effect on fibroblasts in the underlying tissue
D. None of the above
13. Surgery of choice for mandibular prognathism
A. Sagittal spilt osteotomy B. Le fort I osteotomy
C. Vertical ramus osteotomy D. Genioplasty
14. The surgical procedure of choice for skeletal class II malocclusion due to retrognathic mandible is
A. Le Fort l osteotomy B. Advancement genioplasty
C. BSSO set back of the mandible D. BSSO advancement of the mandible
15. Which type of genioplasty would result in a patient with class III facial profile?
A. Reduction genioplasty B. Advancement genioplasty
C. Straightening genioplasty D. Rotational genioplasty

https://t.me/DentalBooksWorld
Chapter 7 • Oral Surgery 649

16. Severe bony undercuts should be treated by


A. Remove undercut on both sides so that no undercut exists
B. Remove undercut on one side
C. Alveolar ridge contouring
D. Alveoloplasty
17. The drawback when heat is used in sterilization is
A. Time consuming B. Rusts the instruments
C. Dulls the sharp instruments D. Improper sterilization
18. Poor accessibility is the main disadvantage of which flap?
A. Envelope B. Trapezoid
C. Semilunar D. Any of the above
19. Which of the following is disinfect dental units and most commonly used to hand pieces?
A. Chlorex B. Glutaraldehyde
C. Betadine D. Ethyl alcohol
20. How can we sterilize a metal impression tray which has a melting point of 175oC?
A. Filtration B. Moist heat sterilization
C. Dry heat sterilization D. Soaking in 2% glutaraldehyde
21. Cidex contains
A. Glutaraldehyde B. Alcohol
C. A phenolic compound D. A caustic agent
22. Which gas is used in cryotherapy?
A. Nitrous oxide B. Carbon dioxide
C. Liquid oxygen D. Nitric dioxide
23. Most commonly used scalpel in oral surgery is
A. No: 22 B. 10
C. No: 11 D. 15
24. Polyglycolic acid suture material is
A. An absorbable natural suture B. An absorbable synthetic suture
C. A nonabsorbable natural suture D. A nonabsorbable synthetic suture
25. In oral surgical procedures, which suture is most commonly used?
A. Chromic gut B. Polyglactin
C. Cotton D. Black silk
26. Ethyl alcohol is effective
A. 70% B. 60%
C. 50% D. 100%

MISCELLANEOUS
1. A direct connection between living bone and load-bearing endosseous implant at the light microscopic level
A. Osteosynthesis B. Osteogenesis
C. Osseointegration D. Osteoinduction
2. Which of the following statement about the bone quality is untrue?
A. D4 bone is the densest
B. D1 bone is the densest
C. There is a direct correlation between bone density and implant survival
D. Bone quality is determined precisely based on Hounsfield numbers

https://t.me/DentalBooksWorld
650 Triumph's Complete Review of Dentistry

D1 D2 D3 D4

3. Which of the following is true in regard to reconstruction of an edentulous mandible with implants?
A. Subperiosteal implant requires only single surgery
B. Staple implant is most useful for the posterior mandible
C. Both blade and the osteointegrated cylinder are useful as posterior abutments in patients with high mental foramen
D. Osteointegrated concept of implant. Stabilization has the best documentation of long-term success.
4. Midazolam can be administered via all routes except
A. IV B. IM
C. Inhalational D. Intranasal
5. During extraction patient becomes unconscious; patient is breathing but not responsive. What will be your first step?
A. Immediate CPR and chest compression B. Recline chair, give oxygen, and shout for help
C. Check blood pressure, pulse, and prepare for intubation D. ECG
6. The primary cause of the needle breakage is
A. Sudden unexpected movement by the patient
B. Improper technique/handling of needle while anesthetizing
C. Weakening of needle by bending it before its insertion
D. Needle that are defective in manufacture
7. After administering LA, the patient appears tensed, restless, and complains of headache and dizziness and he is
conscious. There is a sharp rise in blood pressure. How will you manage this condition?
A. Position the patient in supine position B. Labetalol prescription
C. Monitor vital sign and administer oxygen D. Reassure the patient
8. A known diabetic patient came for extraction of tooth, after LA administration experienced double vision. What
could be the reason (AIIMS NOV 2017)?
A. Hypoglycemia B. Syncope
C. Hyperglycemia D. LA diffusion into inferior orbital fissure

ANSWERS

LOCAL ANESTHESIA AND GENERAL ANESTHESIA


1. Answer: A (Ref. Roberts and Hedges’ Clinical Procedures in Emergency Medicine, By James R. Roberts, 2017)
Etidocaine (200 minutes), followed by Bupivacaine (180 minutes) are long-acting anesthetics and this is due to their
increased protein binding property.
2. Answer: E (Ref. Handbook of Local Anesthesia - E-Book, By Stanley F. Malamed, 2014, page no. 11)
3. Answer: B (Ref. Textbook of Oral and Maxillofacial Surgery, 2nd edition, By S. M. Balaji, 2013, page no. 191)
In extraoral technique for mandibular nerve block (Gow gates and Akinosi techniques are for intraoral mandibular
anesthesia), the needle is inserted from below the zygomatic arch and is directed posterior to the lateral pterygoid plate
below the foramen ovale. In extraoral technique for maxillary nerve block, the needle is directed anterior to lateral
pterygoid plate into the pterygopalatine fossa.

https://t.me/DentalBooksWorld
Chapter 7 • Oral Surgery 651

4. Answer: C (Ref. Dental Care of the Hemophilia Patient, By Karen J. Ridley, Lynn Bergero, 1987)
In hemophiliacs to avoid complications, intraligamentary (an intraosseous technique) method of local infiltration
technique is preferred. In this technique, the solution is deposited in depth of gingival sulcus. The needle should be
inserted apically into the bone until resistance is met.
5. Answer: A (Ref. Essentials of Medical Pharmacology, By KD Tripathi, 2013, page no. 365)
All local anesthetics except cocaine are synthetic compounds and they are vasodilating in nature. Cocaine is a natural local
anesthetic agent and causes local vasoconstriction. (Cocaine increases the vasoconstrictive action of adrenaline. If LA with
adrenaline is given risk of death is more in cocaine abusers. So they are known as walking time bombs.)
6. Answer: A (Ref. Instant Pharmacology, By Kourosh Saeb-Parsy, 1999, page no. 123)
The reaction occurring in the submucosa at normal tissue pH is RNH -> RN + H. RN is lipophilic and is responsible for
penetration into the nerve. If the tissue PH is decreased due to infection or pus, it results in abundance of H ions outside
the nerve sheath, and the equilibrium of reaction in formation of lipophilic molecule (RN) is shifted to left. Therefore, RN
fails to enter the nerve and cannot block the conduction of impulses.
7. Answer: B (Ref. Textbook of Pediatric Emergency Procedures, By Christopher King, Fred M. Henretig, 2008, page no. 433)
5–25% of nitrous oxide
• Diminution of fear and anxiety
• Marked relaxation
• Dissociation sedation and analgesia
25–45% of nitrous oxide
• Floating sensation
• Reducing blink rate
• Euphoric state (laughing gas)
• Total anesthesia
45–65% of nitrous oxide
• Analgesia is complete
• Marked amnesia
8. Answer: B (Ref. Nurse Anesthesia, By John J. Nagelhout, Karen L. Plaus, 2014, page no. 1251)
Adrenaline is the most potent and commonly used alpha adrenergic agonist or vasoconstrictor. In small doses it causes
vasoconstriction of arterioles in immediate area of injection.
Advantages of adding vasoconstrictor agents:
• Prevents rapid absorption of LA agents and thus increases the concentration LA at receptor site
• Prolongs the duration of action
• The reduced absorption rate and increased efficacy in small doses of LA results in reduced toxicity of the LA agent
• Bleeding in the area of operation is minimized
9. Answer: A (Ref. Handbook of Local Anesthesia - E-Book, By Stanley F. Malamed, 2014, page no. 323)
10. Answer: B (Ref. Essentials of Medical Pharmacology, By KD Tripathi, 2013, page no. 368)

Nerve block LA is deposited near main trunk


Field block LA is deposited near Large branch of peripheral nerve
Infiltration LA is deposited near small nerve endings
11. Answer: A (Ref. Current Therapy in Pain, By Howard S. Smith, 2009, page no. 547)
Gate control theory was proposed by Melzack and Wall (AIIMS, 2012) in 1965.
12. Answer: C (Ref. Textbook of Oral and Maxillofacial Surgery, 2nd edition, By S. M. Balaji, 2013, page no. 191)
The mental nerve passes through the mental foramen on the lateral surface of mandible and is sensory to lower lip and skin
of chin. During mental nerve block, the LA solution is developed anterior to apex of the bicuspid near mental foramen.
During incisive nerve block both the incisive nerve and mental nerves are anaesthetized. The incisive nerve continues
anteriorly in the inferior dental canal to midline and it is sensory to mandibular anterior teeth and labial gingival. To block
the incisive nerve, the needle should penetrate the mental foramen.

https://t.me/DentalBooksWorld
652 Triumph's Complete Review of Dentistry

13. Answer: D
14. Answer: D (Ref. Vasovagal Syncope, By Paolo Alboni, Raffaello Furlan, 2014, page no. 229)
15. Answer: D (Ref. Textbook of Anesthesia for Postgraduates, By TK Agasti, 2010, page no. 333)
16. Answer: A (Ref. Textbook of Anesthesia for Postgraduates, By TK Agasti, 2010, page no. 332)
Epinephrine stimulates both a and b receptors. In small doses it causes vasoconstriction and thus it increases the efficiency
of local anesthetic agent. Norepinephrine is not recommended in dental practice because of its predominant action
on a-receptors. Norepinephrine causes intense vascular constriction because of which blood supply to a local area is
compromised and necrosis may result.
17. Answer: B (Ref. Handbook of Local Anesthesia - E-Book, By Stanley F. Malamed, 2014, page no. 139)
The amide type of LA undergoes biotransformation primarily in the liver by microsomal enzymes. The ester type of LA is
inactivated by hydrolysis in the plasma and is catalyzed by enzyme plasma cholinesterase. Some hydrolysis also occurs in
liver. So Ester LA is contraindicated in patients with cholinesterase deficiency.
18. Answer: A (Ref. Textbook of Anesthesia for Postgraduates, By TK Agasti, 2010, page no. 337)
Without vasoconstrictor the maximum acceptable dose is 4.4 mg/kg body weight and not to exceed 300 mg. When used
with vasoconstrictor, the maximum dose is 7 mg/kg body weight (COMEDK-14) and not to exceed 500 mg.
19. Answer: C (Ref. Pharmacology for Dentistry, By Tara Shanbhag, 2017, page no. 152)
Minor surgeries like extractions are carried out in Stage I or stage of analgesia. Major surgeries are carried on in Stage Ill
and Plane Ill of general anesthesia or stage of surgical anesthesia.

Breathing Eye Pupil Eye Muscle tone


movement diameter reflexes
Stage 1 Normal Normal Normal Normal Normal
Analgesia

Stage 2 Increased Decreasing Dilated Losing lid Increased involuntary


Excitement variability reflex movement
Stage 3 Plane 1 Decreasing Decreasing Losing Decreasing response to
Surgical anaesthesia corneal surgical stimulation
reflex
Plane 2 Absent Constricted
Losing
Plane 3 Absent Normal
light reflex
Plane 4 Absent dilated
Stage 4 Imminent Apnoea Absent Maximally No
death dilated reflexes

20. Answer: B (Ref. Local Anesthesia for Dental Professionals, By Kathy B. Bassett, Arthur C. DiMarco, Doreen K. Naughton,
2010, page no. 99)
A solution that contain 1:200000 epinephrine in 1,000 mg in 200,000 mL of solution, i.e., 1:200,000 = 1,000 mg/200,000
mL = 1 mg/200 mL = 0.005 mg/mL (so for 3 mL, it is 0.015 mg)
Since the maximum safe dose is 0.2 mg, this concentration permits the use of 40 mL (0.2/0.005) dose in a healthy patient
to reach the maximum dose of 0.2 mg.
21. Answer: A (Ref. Handbook of Local Anesthesia - E-Book, By Stanley F. Malamed, 2014, page no. 226)
For options “B,” “C,” and “D,” inferior alveolar block should be accompanied by long buccal nerve to anaesthetize the
buccal mucosa of the mandibular molars.
22. Answer: D (Ref. Handbook of Local Anesthesia - E-Book, By Stanley F. Malamed, 2014, page no. 226)
23. Answer: C (Ref. Contemporary Implant Dentistry, By Carl E. Misch, 2008, page no. 685)
24. Answer: C (Ref. Atlas of Oral and Maxillofacial Surgery- E-Book, By Deepak Kademani, Paul Tiwana, 2015, 514)
25. Answer: B (Ref. Essentials of Medical Pharmacology, By KD Tripathi, 2013, page no. 367)
Dibucaine is amide type of local anesthetic agent. It is used as a surface anesthetic on delicate mucous membranes such as
anal canal and occasionally for spinal anesthesia.

https://t.me/DentalBooksWorld
Chapter 7 • Oral Surgery 653

26. Answer: D (Ref. Practical Ambulatory Anesthesia, By Johan Raeder, Richard D. Urman, 2015, page no. 118)
27. Answer: A (Ref. Handbook of Local Anesthesia - E-Book, By Stanley F. Malamed, 2014, page no. 226)
28. Answer: D (Ref. Pharmacology for Dentistry, By Tara Shanbhag, 2017, page no. 161)
29. Answer: B (Ref. Handbook of Local Anesthesia - E-Book, By Stanley F. Malamed, 2014, page no. 390)
Field blocks and nerve blocks are contraindicated in hemophilic patients. Inferior alveolar nerve block and posterior
alveolar nerve blocks should be administered only after replacement therapy of factor VIII, because of possibility of
dissecting hematoma. Intraligamentary injection technique is the preferred one.
30. Answer: A (Ref. Textbook Of Physiology For Dental Students, By Tripathi, 2006, page no. 31)
Conduction is faster in the myelinated nerves than in unmyelinated fibers. The effect in myelinated nerves is at the “nodes
of Ranvier” as the LA do not penetrate the myelin sheath.
31. Answer: C (Ref. Mosby’s Dental Dictionary - E-Book, By Elsevier, Mosby, 2007, page no. 371)
32. Answer: B (Ref. Introduction to Anesthesia, By Robert Dunning Dripps, James E. Eckenhoff, Leroy D. Vandam, 1997,
page no. 209)
LAs are weak bases. These require penetration inside the neuron for their action. For entry in the neuron, LAs have to
cross the neuronal membrane.
Unionized drugs (lipid soluble) can easily cross the membrane; therefore, addition of NaHCO3 in the local anesthetic
solution (weak bases are unionized in the alkaline medium) makes them rapid acting.
Adrenaline increases the duration of action by causing vasoconstriction.
Methylparaben is the preservative added in LA solution.
33. Answer: D (Ref. Clinical Anesthesia, By Paul G. Barash, 2009, page no. 543)
• Methemoglobinemia is caused by prilocaine and not by bupivacaine.
34. Answer: C (Ref. Textbook of Anesthesia for Postgraduates, By TK Agasti, 2010, page no. 331)
35. Answer: C (Ref. Nurse Anaesthesia Secrets, By Mary Karlet, 2005, page no. 89)
Ketamine increases all pressures (blood pressure, intracranial tension, intraocular pressure) in the body. It is therefore an
intravenous anesthetic of choice for shock (increases blood pressure).
36. Answer: A (Ref. Textbook of Anesthesia for Postgraduates, By TK Agasti, 2010, page no. 425)
Xenon is very close to the “ideal agent.”
Advantages of Xenon Anesthesia
• Inert (probably nontoxic to liver and kidney with no metabolism)
• Minimal effect on CVS function
• Lowest blood solubility (lowest blood gas partition coefficient); therefore rapid induction and recovery
• Does not trigger malignant hyperthermia
• Environmental friendly
• Nonexplosive
37. Answer: B (Ref. Handbook of Ambulatory Anesthesia, By Rebecca S. Twersky, Beverly K. Philip, 2010, page no. 186)
Sevoflurane, enflurane, and isoflurane have epileptic potential.
38. Answer: A (Ref. Pharmacology, 2nd edition, By Bhattacharya, 2003, page no. 230)
• Halothane is hepatotoxic and all fluorinated anesthetic agents can cause dose dependent decrease in arterial BP and
depression of heart. Xenon has minimal effect on CVS function.
39. Answer: C (Ref. Clinical Anesthesia, By Paul G. Barash, 2009, page no. 452)
40. Answer: C
41. Answer: B (Ref. Pharmacology, 2nd edition, By Bhattacharya, 2003, page no. 229)
• Nitrous oxide is not a complete anesthetic (MAC 104%)
• It is a good analgesic but poor muscle relaxant42.
42. Answer: A (Ref. Clinical Anesthesia, By Paul G. Barash, 2009, page no. 419)
• Concentration effect, second gas effect, and diffusion hypoxia are seen with inhalational agents used in high
concentrations (like N2O).
43. Answer: C (Ref. Pharmacology and Pharmacotherapeutics - E-Book, By RS Satoskar, Nirmala Rege, SD Bhandarkar, 2015,
page no. 102)
Halothane sensitizes the heart to arrhythmogenic action of catecholamines.
• In pheochromocytoma, there are elevated levels of catecholamines.
• Therefore, halothane should not be used in patients with pheochromocytoma.

https://t.me/DentalBooksWorld
654 Triumph's Complete Review of Dentistry

44. Answer: A
If bone is contacted too soon (less than half the length of a long dental needle), the needle tip is usually located too far
anteriorly (laterally) on the ramus.
To correct:
i)   Withdraw the needle slightly but do not remove it from the tissue.
ii)  Bring the syringe barrel around toward the front of the mouth, over the canine or lateral incisor on the contralateral
side.
iii) Redirect the needle until a more appropriate depth of insertion is obtained. The needle tip is now located posteriorly
in the mandibular sulcus.
If bone is not contacted, the needle tip is usually located too far posterior (medial).
To correct:
i) Withdraw it slightly in tissue (leaving approximately one-fourth its length in tissue) and reposition the syringe barrel
more posteriorly (over the mandibular molars)
ii)  Continue the insertion until contact with bone is made at an appropriate depth (20–25 mm)
45. Answer: D (Ref. Clinical Anesthesia, By Paul G. Barash, 2009, page no. 265)
46. Answer: D (Ref. Principles of Ambulatory Medicine, By Nicholas H. Fiebach, Lee Randol Barker, John Russell Burton,
2007, page no. 449)
The route of epinephrine administration depends on the severity of the clinical situation. Epinephrine may be given
through the subcutaneous (SC) route when the reaction is mild and the patient normotensive. However, when generalized
urticaria or hypotension exists, SC absorption may be variable and slow; IM administration is preferred. When epinephrine
is needed in a faster action, IV route can be preferred.
47. Answer: A (ref. Handbook of Local Anesthesia - E-Book, By Stanley F. Malamed, 2014, page no. 417)
48. Answer: A (Ref. Miller’s Anesthesia - E-Book, By Ronald D. Miller, Lars I. Eriksson, Lee A Fleisher, 2014, page no. 1046)
The greater the blood supply to the area injected, the greater is the systemic absorption. Sites of absorption from
greatest to least include:
Interpleural > Intercostal > Pudendal > Caudal > Epidural > Brachial plexus > Infiltration
49. Answer: B (Ref. Pharmacology and Therapeutics for Dentistry - E-Book, By Frank J. Dowd, Bart Johnson, Angelo Mariotti,
2016, page no. 209)
Local anesthetics cross the neuronal membrane in unionized state and become ionized again in the neuron to block
sodium channels. Due to this reason, sodium bicarbonate increases the rapidity of onset of action (LA are weak bases and
in alkaline medium easily cross the membrane). Unmyelinated and weakly myelinated fibers are blocked first and then the
myelinated ones. These do not affect the resting membrane potential rather inhibit the depolarization. Resting nerves are
less sensitive to block by LA than the stimulated nerves.
50. Answer: B
Major advantage of etomidate over other intravenous anesthetics is that it causes minimum cardiovascular and respiratory
depression.
Propofol has greatest negative inotropic action among all intravenous anesthetics.
Ketamine has cardiostimulatory properties and can cause hypertension.
Midazolam is not used as an inducing agent.
51. Answer: D (Ref. Fundamentals of Pediatric Anaesthesia, 2nd edition, By Paul, 2006, page no. 180)
52. Answer: D (Ref. Clinical Anesthesia, By Paul G. Barash, 2009, page no. 426)
Minimum alveolar concentration or MAC is the concentration of a vapor in the lungs that is needed to prevent movement
(motor response) in 50% of subjects in response to surgical (pain) stimulus. MAC is used to compare the strengths, or
potency, of anesthetic vapors. MAC was introduced in 1965.
53. Answer: D
In a normal, healthy patient: 0.2 mg per appointment
Concentrations of vasoconstrictor in local anesthetics
• 1:50,000–0.02 mg/mL
• 1:100,000–0.01 mg/mL
• 1:200,000–0.005 mg/mL
2% lidocaine = 20 mg/mL × (20 mg/mL) × (1.8 mL/cartridge) = 36 mg lidocaine/cartridge
Epi 1/80,000 = 1 g in 80,000 dilution. So 1,000 mg in 80,000 dilution. 1000/80,000 or 1/80 is = 0.0125 mg epinephrine/cartridge

https://t.me/DentalBooksWorld
Chapter 7 • Oral Surgery 655

• So, there is 0.0125 mg of epinephrine per cartridge containing epi 1/80,000


• Therefore, 0.2 mg/0.0125 mg per cartridge = 16 cartridges contains the maximum dose of epinephrine

EXODONTIA AND IMPACTIONS


1. Answer: B (Ref. Textbook of Oral and Maxillofacial Surgery, By Balaji, 2009, page no. 292)
The two teeth which can be removed by rotation only are maxillary central incisors and mandibular second premolars.
2. Answer: A
3. Answer: D (Ref. Medical Emergencies in Dentistry, By Morton B. Rosenberg, 2002, page no. 495)
4. Answer: B (Ref. Textbook of Surgery for Dental Students, By Sanjay Marwah, 2018, page no. 282)
5. Answer: B (Ref. Textbook of Oral and Maxillofacial Surgery, By Balaji, 2009, 218)
6. Answer: B (Ref. Textbook of Oral and Maxillofacial Surgery, By Balaji, 2009, page no. 235)
Since maxillary first premolar has two roots which may be curved and divergent, fracture occurs readily during extraction.
7. Answer: C (Ref. Oral and Maxillofacial Surgery Secrets, By A. Omar Abubaker, Din Lam, Kenneth J. Benson, 2016, page
no. 268)
Usually the palatal root of maxillary first molar is present closer to maxillary sinus followed by second premolar, first
premolar and last the maxillary second molar.
8. Answer: A
9. Answer: C (Ref. Oral Surgery for the General Dentist, By Harry Dym, Orrett E. Ogle, 2011, page no. 26)
Prothrombin time is increased in patients on anticoagulant therapy or salicylate therapy. It is also prolonged in case of liver
diseases, in deficiency of factors I, II, V. VII, and X. Normal prothrombin is 11–15 seconds. Extraction can be carried out,
When prothrombin time is within 1½ times the normal prothrombin time, that is, about 20–23 seconds.
10. Answer: A (Ref. Contemporary Oral and Maxillofacial Surgery, 6th edition, By James R. Hupp, 2013, page no. 15)
The patients under corticosteroids therapy will have concomitant decreased adrenal reserve and will decompensate when
exposed to additional stress. In order to prevent adrenal crisis the dose should not be missed.
11. Answer: D
12. Answer: C
13. Answer: C (Ref. Hazards and Errors in Anaesthesia, By D. A. B. Hopkin, 2012, page no. 16)
14. Answer: A (Ref. Shafer’s Textbook Of Oral Pathology, 6th edition, By R. Rajendran, 2009, page no. 601)
Dry socket is a form of localized osteitis characterized by an acutely painful tooth socket containing bare bone and broken
down clot. The patients complain of continuous, severe throbbing pain that usually starts 3–4 after extraction. Management
consists of irrigation of socket with warm saline followed by obtundent dressing.
15. Answer: A (Ref. Contemporary Oral and Maxillofacial Surgery - E-Book, By James R. Hupp, Myron R. Tucker, Edward
Ellis, 2013, page no. 172)
Trismus is inability to open mouth due to muscle spasm. It is caused by injection into medial pterygoid muscle resulting in
hematoma formation or infection. The application of heat and administration of antibiotics relieve the symptoms.
16. Answer: A (Ref. Textbook of Surgery for Dental Students, By Sanjay Marwah, 2018, page no. 282)
17. Answer: D (Ref. Principles of Oral and Maxillofacial Surgery, By U. J. Moore, 2011, page no. 120)
The deciduous roots which cannot be grasped with forceps are displaced toward the permanent tooth with a Warwick
James elevator using bony socket wall as the fulcrum.
18. Answer: D (Ref. Textbook of Oral and Maxillofacial Surgery, By Balaji, 2009, page no. 213)
Extraction of a tooth associated with central hemangioma results in profuse bleeding and death of the patient.
19. Answer: D (Ref. When, Why and Where in Oral and Maxillofacial Surgery: Part II - Part 2, By KC Gupta, 2014)
20. Answer: B (Ref. Textbook of Oral and Maxillofacial Surgery, By Rajiv M Borle, 2014, page no. 203)

Elevator Principle
Straight elevator Lever and wedge
Cryer elevator Lever
Apex elevator Lever and wedge
Crossbar elevator Wheel and axle

https://t.me/DentalBooksWorld
656 Triumph's Complete Review of Dentistry

21. Answer: B (Ref. Textbook of Oral and Maxillofacial Surgery, By Balaji, 2009, page no. 125)
The location of root tip will depend on the position of third molar. If the tooth is vertically positioned, the root tip
is most likely in the submandibular space. If the tooth is mesioangular or horizontal in position, it will be located in
pterygomandibular space. Maxillary third molar is usually pushed into the infratemporal space.
22. Answer: B (Ref. Manual of Minor Oral Surgery for the General Dentist, By Pushkar Mehra, Richard D’Innocenzo, 2015,
page no. 222)
Ecchymosis are large extravasations of blood into subcutaneous tissues with facial discoloration caused by breakdown of
hemoglobin. The common areas of postoperative ecchymosis are circumorbital and submandibular regions, lower lip, and
floor of mouth. Management consists of immediate application of cold followed by heat. In severe cases, antibiotics are
given along with proteolytic enzymes which causes breakdown of coagulated blood.
23. Answer: A
24. Answer: B (Ref. Perry’s The Chemotherapy Source Book, By Michael C. Perry, Donald C. Doll, Carl E. Freter, 2012,
page no. 130)
It is clear that the most rapid resolution of an infection secondary to pulpal necrosis is obtained when the tooth is removed
as early as possible. Therefore, acute infection should not be a contraindication to extraction. If access and anesthetic
considerations can be met, the tooth should be removed as early as possible after maintaining proper antibiotic level.
25. Answer: C (Ref. Principles of Oral Surgery, By J. R. Moore, G. V. Gillbe, 1981, page no. 109)
26. Answer: D (Ref. Master Dentistry, Volume 1: Oral and Maxillofacial Surgery, By Paul Coulthard, Keith Horner, Philip
Sloan, 2013, page no. 119)
The first direction of force is apically so that the tip is placed on the root surface as far as apically possible and then the
other movements are given.
In maxilla: The first movement is always apically. Except lateral incisors, all other teeth are moved buccally, lateral incisors
are moved apically and palatally because of its palatal inclination.
In mandible: The first movement is apical followed by labial movement for anteriors and lingual movement for posterior teeth.
27. Answer: C
28. Answer: D (Ref. Greenberg’s Text-Atlas of Emergency Medicine, By Michael I. Greenberg, 2005, page n. 172)
Elevators are used for extraction of distal most teeth in the arch and for luxation of adjacent teeth. In case of extractions,
usually the interdental bone is used as fulcrum. The use of an adjacent tooth as a fulcrum is only permissible if that tooth
is to be extracted at the same visit.
29. Answer: D (Ref. Oral Surgery, By Fragiskos D. Fragiskos, 2007, page no. 183)
30. Answer: A (Ref. Current Therapy In Oral and Maxillofacial Surgery, By Shahrokh C. Bagheri, Bryan Bell, Husain Ali
Khan, 2011, page no. 130)
31. Answer: D (Ref. Textbook of Oral and Maxillofacial Surgery, By Rajiv M Borle, 2014, page no. 646)
32. Answer: A (Ref. The Extraction of Teeth, By Geoffrey Leslie Howe, 1971, page no. 85)
33. Answer: A (Ref. Textbook of Oral and Maxillofacial Surgery, By Rajiv M Borle, 2014, page no. 233)
34. Answer: B (Ref. The Extraction of Teeth, By Geoffrey Leslie Howe, 1974, page no. 54)
35. Answer: C (Ref. Oral Wound Healing: Cell Biology and Clinical Management, By Hannu Larjava, 2012, page no. 195)
36. Answer: A
37. Answer: B (Ref. Contemporary Oral and Maxillofacial Surgery - E-Book, By James R. Hupp, Myron R. Tucker, Edward
Ellis, 2013, page no. 29)
38. Answer: B
39. Answer: A (Ref. Dental Journal of Australia - Volume 8, 1936, page no. 155)
40. Answer: A (Ref. Textbook of Oral and Maxillofacial Surgery, By Rajiv M Borle, 2014, page no. 231)
41. Answer: C (Ref. Current Therapy In Oral and Maxillofacial Surgery, By Shahrokh C. Bagheri, Bryan Bell, Husain Ali Khan,
2011, page no. 135)
42. Answer: B (Ref. Oral and Maxillofacial Surgery, By Lars Andersson, Karl-Erik Kahnberg, M. Anthony Pogrel, 2012, page
no. 230)
43. Answer: A (Ref. Principles of Oral Surgery, By J. R. Moore, 1976, page no. 107)
44. Answer: A (Ref. Contemporary Oral and Maxillofacial Surgery - E-Book, By James R. Hupp, Myron R. Tucker, Edward
Ellis, 2013, page no. 158)
45. Answer: B (Ref. Medical Emergencies in Dentistry, By Morton B. Rosenberg, 2002, page no. 495)
46. Answer: A (Ref. Peterson’s Principles of Oral and Maxillofacial Surgery, 2012, page no. 115)

https://t.me/DentalBooksWorld
Chapter 7 • Oral Surgery 657

MANDIBULAR FRACTURES
1. Answer: D (Ref. Textbook of Oral and Maxillofacial Surgery, By Balaji, 2009, page no. xiv)
CSF rhinorrhea is seen in Le Fort II, III, and in cases of severe nasoethmoidal fractures.
2. Answer: D (Ref. Textbook of Oral and Maxillofacial Surgery, By Balaji, 2009, page no. 569)
Compression plates are used at inferior border of mandible below the inferior dental canal. If there is opening of the upper
border, it is necessary to apply a tension band in the form of arch bar or miniplates at the upper border.
3. Answer: D (Ref. Textbook of Oral and Maxillofacial Surgery, By Balaji, 2009, page no. 593)
In case of subcondylar fracture in children below 10 years:
• No immobilization and active treatment is required if the occlusion is undisturbed
• If the occlusion is grossly deranged, IMF is indicated for 7–10 days with intermittent active mouth openings
4. Answer: B (Ref. Principles of Surgical Patient, By C. J. Mieny, 2003, page no. 545)
Period of immobilization for fractures of tooth-bearing areas of mandible.
a)  Young adult with fracture of the angle receiving early treatment 3 weeks in which tooth removed from J fracture line
b)  If tooth retained in fracture Line — 1 week is added. (3+1 = 4 weeks)
c)  If fracture occurs at the symphysis — 1 week is added. (4+1 = 5 weeks)
d)  If the patient age is 40 years and over — 1 or 2 weeks are added.
e)  If fracture occurs in children and adolescents, subtract 1 week.
5. Answer: B (Ref. Surgical Correction of Facial Deformities, By Varghese Mani, 2010, page no. 271)
Hence, early mobilization is advised to avoid complications in children.
6. Answer: D (Ref. Textbook of Oral and Maxillofacial Surgery, By Neelima Anil Malik, 2012, page no. 455)
The condyle is moved in anteromedial direction under the influence of lateral pterygoid or external pterygoid muscle.
7. Answer: C (Ref. Peterson’s Principles of Oral and Maxillofacial Surgery, 2012, page no. 408)
• Simple fracture includes linear if of condyle, coronoid ramus, and edentulous body of mandible.
• Compound fracture include fractures of tooth-bearing portions of mandible.
• Green stick fracture is a rare type of simple if and is found exclusively in children.
• Comminuted fracture are due to direct violence to mandible from penetrating sharp objects and missiles.
8. Answer: A (Ref. Oral and Maxillofacial Trauma - E-Book, By Raymond J. Fonseca, H. Dexter Barber, Michael P. Powers,
2013, page no. 189)
Fractures of angle of mandible are influenced by the pull of medial pterygoid, masseter and temporalis muscles, which
displace the ramus in superior and anterior direction. This is a horizontally unfavorable fracture.
9. Answer: B (Peterson’s Principles of Oral and Maxillofacial Surgery, 2012, page no. 432)
10. Answer: A (Ref. Plastic surgery in infancy and childhood, By John Clark Mustardé, Ian T. Jackson, 1988, page no. 347)
The use of acrylic cap splint with circumferential wiring is the best method to treat mandibular fractures in children.
11. Answer: B (Ref. Fundamentals of Diagnostic Radiology, By William E. Brant, Clyde A. Helms, 2007, page no. 84)
Because of sudden change in angulation, angle is considered as weakest part of mandible.
12. Answer: D (Ref. Textbook of Oral and Maxillofacial Surgery, By Balaji, 2009, page no. 570)
13. Answer: C (Ref. Textbook of Oral and Maxillofacial Surgery, By Neelima Anil Malik, 2012, page no. 448)
In edentulous mandible, the molar areas are weakened following alveolar resorption and become the site for bilateral
fracture of edentulous mandible. There is downward and backward movement of anterior part of mandible under the
influence of digastric and mylohyoid muscles.
14. Answer: B (Ref. Oral and Maxillofacial Surgery, By Lars Andersson, Karl-Erik Kahnberg, M. Anthony Pogrel, 2010, page
no. 904)
15. Answer: B (Ref. Oral and Maxillofacial Surgery - E-Book: 3-Volume Set, By Raymond J. Fonseca, 2017, page no. 151)
Bone plating is of three types — simple, noncompression plates, mini plates and compression plates.
Advantages:
• Ensures sufficient rigidity across the fracture site to obviate the need for IMF.
• Enables the patient to enjoy a relatively normal diet and to maintain oral hygiene more easily.
• Useful in mentally challenged and physically handicapped patients.
When compression plates are used, they provide extremely rigid fixation and the bone healing takes place without the
formation of intermediate callus.

https://t.me/DentalBooksWorld
658 Triumph's Complete Review of Dentistry

Wherever the compression plates are applied to the convex surface of the mandible at its lower border, there is tendency
for the upper border and lingual plate to open up with the final tightening of the screw. This leads to distortion of occlusion
as well as opening of the fracture line on the other side. To avoid this, a tension band in the form of arch bar or mini plates
is applied at the upper border. The optimum length of screw, for fixation of plate in mandible is 4 mm.
16. Answer: B (Ref. Fractures of the Facial Skeleton, By Michael Perry, Andrew Brown, Peter Banks, 2015, page no. 155)
Eburnation is seen in the case of nonunion and the radiographs show rounding off and sclerosis of bone ends.
17. Answer: A (Ref. Bailey & Love’s Short Practice of Surgery, 26th edition, By Norman Williams, P Ronan O’Connell, 2013,
page no. 343)
• Damage to the inferior alveolar nerve after fracture, results in the paresthesia or anesthesia of the lower lip on the
affected side.
• Although changes in sensation in the lower lip and chin may be related to chin and lip lacerations and blunt trauma,
numbness in the distribution of inferior alveolar nerve after trauma is almost pathognomonic of a fracture distal to the
mandibular foramen.
18. Answer: C (Ref. The Closed Treatment of Common Fractures, By John Charnley, 2003, page no. 123)
Green stick fracture is a variant of simple number and is found exclusively in children.
19. Answer: A (Ref. Textbook of Oral and Maxillofacial Surgery, By Balaji, 2009, page no. 570)
Risdon wiring is indicated for symphysis, when all the teeth are present in the arch.
20. Answer: A (Ref. Oral and Maxillofacial Trauma - E-Book, By Raymond J. Fonseca, H. Dexter Barber, Michael P. Powers,
2013, page no. 812)
21. Answer: A (Ref. Textbook of Oral and Maxillofacial Surgery, By Balaji, 2009, page no. 570)
Greatly displaced and dislocated fractures (separation of fragments exceeding 5 mm and displacements exceeding 30
degrees) increasingly are being treated surgically, a functional conservative treatment is worth considering in less severely
dislocated fractures (Rowe and Williams’s maxillofacial injuries 2nd edition, page no. 428).
Zide and Kent (1983) have defined absolute and relative indication for open reduction of condylar fractures.
Absolute indications for open reduction:
• Fracture dislocation of condyle in to middle cranial fossa
• Impossibility of obtaining adequate occlusion by closed reduction due to locking by the condylar fragment
• Lateral fracture dislocation of the condyle
• Invasion by a foreign body (gunshot wound)
22. Answer: B (Ref. Textbook of Oral and Maxillofacial Surgery, By Rajiv M Borle, 2014, page no. 406)
Sublingual ecchymosis is considered as pathognomonic sign of mandibular fracture.
23. Answer: B (Ref. Textbook of Oral and Maxillofacial Surgery, By Balaji, 2009, page no. 571)
In presence of sufficient numbers of teeth, simple fractures of tooth-bearing part of the mandible is adequately immobilized
by IMF alone.
Gilmer’s direct method of wiring is simplest and rapid method of immobilizing the jaws. In this method, the wires are directly
attached to the teeth. It is therefore difficult to release the intermaxillary connection without stripping off the entire fixation.
24. Answer: D (Ref. Textbook of Oral and Maxillofacial Surgery, By Balaji, 2009, page no. 588)
In case of unilateral condylar, the mandible deviates toward the side of fracture, the patient cannot deviate the mandible to
the opposite side because of ineffective action of lateral pterygoid on the fractured side.
25. Answer: B (Ref. Bailey & Love’s Short Practice of Surgery, 26th edition, By
Norman Williams, P Ronan O’Connell, 2013, page no. 346. https://books.google.co.in/books?isbn=144416502X)
26. Answer: B (Ref. Mercer’s Textbook of Orthopaedics and Trauma, 10th edition, By Suresh Sivananthan, Eugene Sherry,
Patrick Warnke, 2012, page no. 1027)
27. Answer: B (Ref. Dental Radiography - E-Book: Principles and Techniques, By Joen Iannucci, Laura Jansen Howerton,
2013, page no. 289)
28. Answer: A (Ref. Oral and Maxillofacial Surgery, By Carrie Newlands, Cyrus Kerawala, 2014, page no. 478)
29. Answer: A (Ref. The Human Skeleton in Forensic Medicine, 3rd edition, By Mehmet Yasar Iscan, Maryan Steyn, 2013,
page no. 322)
• A direct blow usually causes a transverse fracture and damage to the overlying skin
• Crushing is more likely to cause a comminuted fracture
• Twisting causes spiral fracture
• Compression causes a short oblique fracture
• Bending results in fracture with a triangular butterfly fragment

https://t.me/DentalBooksWorld
Chapter 7 • Oral Surgery 659

30. Answer: D (Ref. Textbook of Oral Medicine, By Anil Govindrao Ghom, Savita Anil (Lodam) Ghom, 2014, page no. 592)
31. Answer: B (Ref. Textbook of Oral and Maxillofacial Surgery, By Balaji, 2009, page no. 581)
32. Answer: D (Ref. Atlas of Oral and Maxillofacial Surgery- E-Book, By Deepak Kademani, Paul Tiwana, 2015, page no. 681)
33. Answer: C (Ref. Oral and Maxillofacial Surgery, By Lars Andersson, Karl-Erik Kahnberg, M. Anthony Pogrel, 2010, page
no. 890)
34. Answer: A
35. Answer: C
36. Answer: A
37. Answer: A

MIDDLE THIRD FRACTURES


1. Answer: A (Ref. Atlas of Head & Neck Surgery–otolaryngology - Volume 2, By Byron J. Bailey, 2001, page no. 578)
Le fort III level (High transverse/suprazygomatic fracture/craniofacial (dysjunction). A fracture in which the facial
bones separate from the cranial bones is known as craniofacial dysjunction fracture. This is also known as Le Fort HI
fracture.
• Fracture line runs from near the frontonasal suture transversely backward parallel with the base of the skull and involves
the full depth of the ethmoid bone, including the cribriform plate.
• Within the orbit, it passes below the optic foramen into posterior limit of the inferior orbital fissure.
• From the base of the inferior orbital fissure the line extends in two directions.
• Backward across the pterygomaxillary fissure to U the roots of pterygoid laminae (upper one-third region).
• Laterally across the lateral wall of orbit separating the zygomatic bone from the frontal bone.
2. Answer: D
Glasgow coma scale is used to know the level of consciousness of patient by ascertaining motor response, verbal
performance and eye opening.
Lowest score—3; highest score—15
Score of ≤8—Coma
In this case, E—2 (responds to pain), V—2 (incomprehensible sounds), and M—5 (localizes pain)GCS—9/15
3. Answer: C (Ref. Otorhinolaryngology, Head and Neck Surgery, By Matti Anniko, Manuel Bernal-Sprekelsen, Victor
Bonkowsky, 2010, page no. 276)
CSF rhinorrhea is seen in nasoethmoidal, Le Fort II, and Le Fort III fractures, associated with corn minuted cribriform
plate of ethmoid. Tramline effect and halo on pillow effect are diagnostic signs of CSF rhinorrhea.
4. Answer: D (Ref. Oral and Maxillofacial Surgery - E-Book: 3-Volume Set, By Raymond J. Fonseca, 2017, page no. 226)
Epiphora is seen in Le Fort II, Le Fort III, and severe nasal complex injuries due to partial or complete obstruction of
nasolacrimal duct (MCET-07). The patient complains of epiphora and may develop infected mucocele, a condition termed
as dacryocystitis.
5. Answer: A (Ref. Strategy in Head Injury Management, By Richard H. Simon, James T. Sayre, 1987, page no. 122)
If the patient is unconscious, the patient should be carried in lateral position. This allows the clearing of blood and mucus
from the mouth and nasopharynx and escape of further secretions.
6. Answer: A (Ref. Text book of Oral Medicine, Oral Diagnosis and Oral Radiology, By Ravikiran Ongole, 2009)
Enophthalmos are seen in zygomatic complex fracture, orbital fracture, and Le Fort III fracture. It is caused either by
escape of orbital contents like fat or by an increase in the volume of the bony orbit.
7. Answer: A (Ref. SRB’s Manual of Surgery, By Sriram Bhat M, 2016, page no. 352)
• Guerin fracture is other name for Le Fort I or low-level fracture.
• Le Fort II involves maxilla, nasal, lacrimal bones.
• Le Fort III involves maxilla, lacrimal, nasal, and ethmoidal bones.
8. Answer: A (Ref. Oral and Maxillofacial Trauma - E-Book, By Raymond J. Fonseca, H. Dexter Barber, Michael P. Powers,
2013, page no. 434)
Orbital blow-out fracture occurs when a rounded object strikes the protruding eyeball resulting in fracture of orbital
floor.

https://t.me/DentalBooksWorld
660 Triumph's Complete Review of Dentistry

“Blow-in” fracture is due to inward buckling of orbital floor. It usually occurs in children and results from trauma to
inferior orbital rim.
9. Answer: B
“Panda fades” is due to edema and ecchymosis around the eyes. Because of this the patient develops black circles around
the eyes (Raccoon eyes).
10. Answer: D (Ref. Textbook of Oral and Maxillofacial Surgery, By Rajiv M Borle, 2014, page no. 466)
11. Answer: B (Ref. Diagnostic Imaging: Spine E-Book, By Jeffrey S. Ross, Kevin R. Moore, 2015, page no. 280)
Usually after trauma, hypovolemic shock is developed due to severe blood loss. Ringer lactate solution because of its high
osmotic value maintains the fluid in vascular compartment.
12. Answer: A (Ref. Textbook of Oral and Maxillofacial Surgery, By Balaji, 2009, page no. 611)
13. Answer: B (Ref. The Trauma Manual: Trauma and Acute Care Surgery, By Andrew B. Peitzman, Michael Rhodes,
C. William Schwab, 2008, page no. 184)
14. Answer: D (Ref. Atlas of Operative Maxillofacial Trauma Surgery: Primary Repair, By Michael Perry, Simon Holmes,
2014)
Diplopia is due to interference with activity of ocular muscles. It is seen following fractures of zygomatic complex, Le Fort
II, and Le Fort III fractures when the fracture line passes above Whitnall’s tubercle. The degree of diplopia in maxillofacial
trauma can be accurately recorded by Hess Chart.
15. Answer: B (Ref. Mastery of Surgery, By Josef E. Fischer, Kirby I. Bland, Mark P. Callery, 2006, page no. 380)
16. Answer: B (Ref. Textbook of Oral and Maxillofacial Surgery, By Neelima Anil Malik, 2012, page no. 415)
Hanging drop sign is best seen in the Waters projection of the face.
17. Answer: D (Ref. Fractures of the Facial Skeleton, By Michael Perry, Andrew Brown, Peter Banks, 2015, page no. 10)
18. Answer: A (Ref. Emergency Procedures and Techniques, By Robert Rutha Simon, Barry E. Brenner, 2002, page no. 45)
19. Answer: D (Ref. Textbook of Oral and Maxillofacial Surgery, By Balaji, 2009, page no. 618)
20. Answer: A (Ref. SRB’s Surgery for Dental Students, By Sriram M Bhat, 2014, page no. 428)
21. Answer: B (Ref. Facial Trauma, By Seth Thaller, W. Scott McDonald, 2004, page no. 266)
“Walsham’s forceps” and “Asch’s forceps” are used for reduction of fractured segments. These fractures should be repaired
within 7–10 days.
22. Answer: D (Ref. Textbook of Oral and Maxillofacial Surgery, By Balaji, 2009, page no. 636)
23. Answer: A (Ref. Plastic and Reconstructive Surgery, By Maria Z. Siemionow, Marita Eisenmann-Klein, 2010, page no. 291)
24. Answer: D
25. Answer: B
26. Answer: B (Ref. Textbook of Oral Medicine, By Anil Govindrao Ghom, Savita Anil (Lodam) Ghom, 2014, page no. 673)
Patient with Le Fort II and III fractures have gross edema of the soft tissues overlying the middle third of the facial skeleton,
giving rise to the characteristic “moon face” appearance. This ballooning of the features is not seen in isolated Le Fort I fractures.
27. Answer: C (Ref. Textbook of Oral and Maxillofacial Surgery, By Balaji, 2009, page no. 636)
Composition of Whitehead’s varnish:
• Benzoin 10 g
• Iodoform 10 g
• Storax 7.5 g
• Tolu balsam 5 g
• Ether 100 mL
28. Answer: A
29. Answer: D (Ref. Atlas of Operative Oral and Maxillofacial Surgery, By Christopher J. Haggerty, Robert M. Laughlin, 2015,
page no. 103)
Formal tracheostomy/submental intubation is performed in treatment of complex panfacial fractures, i.e, Le Fort II/
III fractures. The submental route involves pulling the free end of a tracheal tube through a submental incision, after a
conventional orotracheal intubation has been performed. It gives unimpeded access to the facial region.
30. Answer: B
(Ref. Clinical Anesthesia, By Paul G. Barash, 2009, page no. 1317)
Infraglottis airway devices are placed below glottis and are further classified as definitive and emergency airway
management devices.
• Definitive airway management includes endotracheal tube and tracheostomy
• Infraglottis emergency include cricothyroidotomy

https://t.me/DentalBooksWorld
Chapter 7 • Oral Surgery 661

31. Answer: B (Ref. Manual of Emergency Airway Management, By Ron Walls, Michael Murphy, 2012, page no. 186)
32. Answer: A (Ref. Anesthesiologist’s Manual of Surgical Procedures, By Richard A. Jaffe, Brenda Golianu, Clifford A.
Schmiesing, 2014, page no. 275)
Advantage—Facilitates better oral hygiene
Disadvantage—Have greater airway resistance, are more difficult to suction through, and may cause purulent sinusitis or
otitis media
33. Answer: A (Ref. Oral Surgery for the General Dentist, By Harry Dym, Orrett E. Ogle, 2011, page no. 40)
34. Answer: B (Ref. Cleft Lip and Palate: Diagnosis and Management, By Samuel Berkowitz, 2006, page no. 577)

CYSTS AND TUMORS


1. Answer: C (Ref. Contemporary Oral and Maxillofacial Surgery - E-Book, By James R. Hupp, Myron R. Tucker, Edward
Ellis, 2013, page no. 454)
“Marsupialization” or “Partsch operation” or “decompression” involves deroofing of cyst along with adjoining bone and
mucoperiosteum so that the cystic lining becomes continuous with oral epithelium. This process decreases intracystic
pressure and promotes shrinkage of cyst and bone fill. It is usually indicated in excessively large cysts.
2. Answer: A (Ref. Textbook of Oral and Maxillofacial Surgery, By Neelima Anil Malik, 2012, page no. 529)
Aspiration helps to distinguish between a cyst and a tumor and between aneurysmal bone cyst and cavernous hemangioma.
3. Answer: D (Ref. Shafer’s Textbook of Oral Pathology, By Arya Rajendran, B Sivapathasundharam, 2014, page no. 282)
4. Answer: A (Ref. Burket’s Oral Medicine, By Lester William Burket, Martin S. Greenberg, Michael Glick, 2008, page no. 148)
A low protein content of less than of 4 g/dL is typical for all keratocysts. All, other cysts will have a protein content of 5–11
g/dL which is equal to mean protein level of serum.
5. Answer: B (Ref. Textbook of Oral Medicine, By Anil Govindrao Ghom, Savita Anil (Lodam) Ghom, 2014, page no. 607)
Stafne’s idiopathic bone cavity is located below the mandibular canal and yields air on aspiration.

Pus or brownish fluid with paste-like or caseous consistency Infected cyst


Mucus Mucocele, Ranula
Traumatic bone cavity or solitary bone
Air on aspiration/Golden yellow fluid
cyst
• Vascular malformation in jaw
Blood on aspiration • Aneurysmal bone cysts
• Central giant cell granulomas
Clear pale, straw-colored fluid with total protein in excess of
Dentigerous cyst
4 g/100 mL
Dirty, creamy white viscoid suspension of keratin with the protein
Odontogenic keratocyst
level <4 g/100 mL
Traumatic or hemorrhagic or solitary bone cyst is located above mandibular canal. It is usually filled with brick red-colored
fluid though some cysts are empty in nature.
6. Answer: A (Ref. Shafer’s Textbook of Oral Pathology, By Arya Rajendran, B Sivapathasundharam, 2014, page no. 260)
7. Answer: D (Ref. Peterson’s Principles of Oral and Maxillofacial Surgery, 2012, page no. 630)
8. Answer: B (Ref. Textbook of Oral and Maxillofacial Surgery, By Balaji, 2009, page no. 347)
9. Answer: C (Ref. Burket’s Oral Medicine: Diagnosis & Treatment, By Lester William Burket, Martin S. Greenberg, Michael
Glick, 2003, page no. 160)
10. Answer: C (Ref. Burket’s Oral Medicine: Diagnosis & Treatment, By Lester William Burket, Martin S. Greenberg, Michael
Glick, 2003, page no. 150)
11. Answer: C (Ref. Textbook of Oral and Maxillofacial Surgery, By Balaji, 2009, page no. 377)
The extension of odontogenic myxoma occurs beyond the radiological limits of main tumor and it is infiltrative in nature.
So curettage is unlikely to cure and results in recurrence of lesion. Excision of the lesion including a perimeter margin of
tumor-free bone is the treatment of choice.
12. Answer: A (Ref. Textbook of Oral and Maxillofacial Surgery, By Rajiv M Borle, 2014, page no. 662)
13. Answer: B (Ref. Peterson’s Principles of Oral and Maxillofacial Surgery, 2012, page no. 651)

https://t.me/DentalBooksWorld
662 Triumph's Complete Review of Dentistry

14. Answer: C (Ref. Textbook of Oral and Maxillofacial Surgery, By Balaji, 2009, page no. 347)
“Waldron’s technique” is a combination of marsupialization followed by enucleation (B HU-07). First, marsupialization is
performed and cavity is allowed to shrink. When the cavity becomes smaller enucleation is performed and the cyst lining
is completely eliminated.
15. Answer: A
16. Answer: B (Ref. Contemporary Oral and Maxillofacial Surgery - E-Book, By James R. Hupp, Myron R. Tucker, Edward
Ellis, 2013, page no. 436)
17. Answer: C (Ref. Textbook of Oral Radiology, By Ghom, 2009, page no. 493)
18. Answer: B (Ref. Textbook of Oral and Maxillofacial Surgery, By Balaji, 2009, page no. 689)
In these tumors, the palatal bone is very thin and there is chances of tear of very thin and nasal mucosa during
enucleation. Bleeding is encountered from greater palatine artery. Alteration of speech occurs only when the flap is not
sutured back.
19. Answer: C (Ref. Oral Pathology - E-Book: Clinical Pathologic Correlations, By Joseph A. Regezi, James J. Sciubba, Richard
C. K. Jordan, 2012, page no. 279)
Tumors like adenoameloblastoma (adenomatoid odontogenic tumor), ossifying fibroma are encapsulated and they can be
managed by simple enucleation.
20. Answer: A (Ref. Textbook of Oral and Maxillofacial Surgery, By Balaji, 2009, page no. 375)
When the tumor does not involve the tower border, en bloc resection is the treatment of choice. It helps in maintaining the
continuity of the bone at the level of lower border of mandible.
When the lower border is involved, segmental resection or hemimandibulectomy is performed depending on the extent
of tumor.
21. Answer: A (Ref. Textbook of Oral and Maxillofacial Surgery, By Balaji, 2009, page no. 347)
22. Answer: B (Ref. Basic Clinical Radiobiology, 5th Edition, By
Michael C. Joiner, Albert van der Kogel, 2016, page no. 1940)
Cosmetic recontouring is generally undertaken after active growth stage.
23. Answer: D (Ref. Textbook of Oral and Maxillofacial Surgery, By Balaji, 2009, page no. 410)
• Small stones in distal portion of the duct can be removed by manipulation known as milking the gland
• Large sialoliths can be removed surgically by transoral sialolithotomy during which incision is given in the mucous
membrane and the salivary duct and the stone is removed
• Piezoelectric lithotripsy is a noninvasive method of disintegrating sialoliths. But this procedure is complicated by
destruction of amalgam restoration
24. Answer: B (Ref. Textbook of Oral and Maxillofacial Surgery, By Rajiv M Borle, 2014, page no. 310)
Structures passing through parotid gland:
• Superficial zone contains facial nerve and its five terminal branches, auriculotemporal nerve, and greater auricular nerve
• Intermediate zone contains retromandibular vein
• Deep zone contains external carotid, transverse facial and posterior auricular arteries
25. Answer: A (Ref. Textbook of Oral and Maxillofacial Surgery, By Rajiv M Borle, 2014, page no. 310)
About 90% sialoliths are formed in the submandibular gland. This is due to:
• High viscous secretion of submandibular gland
• The presence of gland in dependent position
• Tortuous course of submandibular duct
• Its alkaline secretion with higher concentration of calcium and phosphate ions
26. Answer: C (Ref. Essentials of Pediatric Oral Pathology, By Mayur Chaudhary, Schweta Dixit Chaudhary, 2011,
page no. 187)
27. Answer: B (Ref. Textbook of Oral and Maxillofacial Surgery, By Balaji, 2009, page no. 411)
Excision of submandibular gland is done by submandibular incision 2 cm below the inferior border to prevent damage to
mandibular branch of facial nerve. Excision of parotid gland can be done by preauricular or submandibular or combination
of two approaches and care should be taken to prevent damage to zygomatic branch of facial nerve.
28. Answer: A (Ref. Shafer’s Textbook of Oral Pathology, By Arya Rajendran, B Sivapathasundharam, 2014, page no. 225)

https://t.me/DentalBooksWorld
Chapter 7 • Oral Surgery 663

Procedure Nodal levels removed Structures preserved Indications


Supraomohyoid neck Levels I–III SAN, SCM, IJV N0 neck for SCC oral cavity and
dissection oropharynx
N0 neck malignant melanoma where
primary site is anterior to ear (include
parotidectomy for face and scalp)
Lateral neck dissection Levels II–IV SAN, SCM, IJV N0 neck for SCC larynx and
hypopharynx
Posterolateral neck Levels II–V SAN, SCM, IJV N0 primary site posterior to neck
dissection Suboccipital and
retroauricular nodes
SOHND – Lymph nodes in levels I–III and the SMSG
Extended supraomohyoid neck dissection – LN levels I–IV and SMSG
ALND – LN in levels II–IV are removed
PLND – LN in levels II–V and suboccipital and retroauricular lymph nodes are removed
MRND – Modified radial neck dissection
• In Type 1 MRND – Nodes are removed along with sternocleidomastoid and internal jugular vein (accessory nerve is
preserved); indicated for SCC in upper aerodigestive tract with clinically positive lymph nodes
• In Type 2 MRND – Nodes are removed along with internal jugular vein (sternocleidomastoid muscle and accessory
nerve is preserved)
• In Type 3 MRND (functional neck dissection) – Nodes are removed. Sternocleidomastoid muscle, internal jugular vein,
and accessory nerve is preserved; indicated for thyroid cancers.
In short, Type 1 – N preserved type
Type 2 – NM preserved
Type 3 – NMV preserved
29. Answer: D (Ref. Shafer’s Textbook of Oral Pathology, By Arya Rajendran, B Sivapathasundharam, 2014, page no. 225)

Staging System T stage N stage M stage


AJCC Fifth edition
I T1 or T2 N0 M0
II T3 or T4 N0 M0
III Any T N1 M0
IV Any T Any N M1
AJCC sixth edition
I T1 or T2 N0 M0
IIa T3 N0 M0
IIb T4 N0 M0
IIIa T1 or T2 N1 M0
IIIb T3 or T4 N1 M0
IIIc Any T N2 M0
IV Any T Any N M1
30. Answer: D (Ref. Shafer’s Textbook of Oral Pathology, By Arya Rajendran, B Sivapathasundharam, 2014, page no. 225)
31. Answer: A
32. Answer: B (Ref. Head and Neck Surgery and Oncology, By Jatin P. Shah, Snehal G. Patel, Bhuvanesh Singh, 2012, page no. 281)
The Commando Operation or Commando Procedure is a complicated operation for first-degree malignancy of the tongue.
It comprises glossectomy and hemimandibulectomy together with block dissection of the cervical nodes.

https://t.me/DentalBooksWorld
664 Triumph's Complete Review of Dentistry

ODONTOGENIC INFECTIONS
1. Answer: D (Ref. Textbook of Oral and Maxillofacial Surgery, By Balaji, 2009, page no. 130)
Trismus is due to collection of pus between medial pterygoid and superior constrictor. Infections from lateral pharyngeal
space are life threatening because of its intimate relationship with carotid sheath which contains internal jugular vein,
vagus nerve, internal carotid artery, and common carotid artery.
2. Answer: A (Ref. Shafer’s Textbook of Oral Pathology, 6th edition, By R. Rajendran, 2009, page no. 497)
Garre’s osteomyelitis characteristically shows proliferative periostitis where there is peripheral subperiosteal bone
deposition caused by mild irritation and infection.
3. Answer: B (Ref. Surgical Emergencies in Clinical Practice, By Iqbal Shergill, Manit Arya, Tahwinder Upile, 2012,
page no. 125)
In Ludwig’s angina, there is progressive dyspnea due to backward spread of infection. If not treated, it results in edema of
glottis and causes complete respiratory obstruction.
4. Answer: D (Ref. Textbook of Oral and Maxillofacial Surgery, By Balaji, 2009, page no. 127)
Boundaries of pterygomandibular space:
Posteriorly—Deep portion of parotid gland
Anteriorly—Pterygomandibular raphae
Roof—Lateral pterygoid muscle
5. Answer: D (Ref. Textbook of Oral and Maxillofacial Surgery, By Balaji, 2009, page no. 130)
Retropharyngeal space is involved due to extension of odontogenic infections from the lateral pharyngeal space.
6. Answer: C (Ref. Textbook of Oral and Maxillofacial Surgery, By Rajiv M Borle, 2014, page no. 337)
Masticatory space comprises of the following spaces:
• Pterygomandibular
• Sub masseteric
• Superficial temporal
• Deep temporal spaces
Though options “A,” “B,” and “D” are seen with other space infections, trismus due to infection of masseter and medial
pterygoid is the characteristic feature of masticatory space infection.

Laterally Medial surface of ramus of mandible


Medially Lateral surface of medial pterygoid muscle
7. Answer: D
8. Answer: B (Ref. Textbook of Oral and Maxillofacial Surgery, By Balaji, 2009, page no. 141)
Endarteritis of blood vessels leads to decreased microcirculation and this results in hypocellularity, hypoxia, and
hypovascularity of the irradiated tissues.
9. Answer: C (Ref. Shafer’s Textbook of Oral Pathology, By Arya Rajendran, B Sivapathasundharam, 2014, page no. 505)
10. Answer: D (Ref. Textbook of Oral and Maxillofacial Surgery, By Neelima Anil Malik, 2012, page no. 680)
Primary mandibular spaces:
• Submental space
• Sublingual space—Spaces involved in Ludwig’s angina
• Submandibular space
• Buccal space
Primary maxillary spaces (CBI):
• Canine space
• Buccal space
• Infratemporal space
All the remaining spaces are secondary spaces.
11. Answer: D (Ref. Shafer’s Textbook Of Oral Pathology, 6th edition, By R. Rajendran, 2009, page no. 497)
Surgical recontouring is done to recontour the cortical expansion of the jaw.
12. Answer: A (Ref. Textbook of Oral and Maxillofacial Surgery, By Balaji, 2009, page no. 116)
13. Answer: C (Ref. Shafer’s Textbook of Oral Pathology, By Arya Rajendran, B Sivapathasundharam, 2014, page no. 507)

https://t.me/DentalBooksWorld
Chapter 7 • Oral Surgery 665

14. Answer: B
Option A is sequestrum.
15. Answer: B (Ref. Shafer’s Textbook Of Oral Pathology, 6th edition, R. Rajendran, 2009, page no. 505)
16. Answer: A (Ref. Textbook of Oral and Maxillofacial Surgery, By Rajiv M Borle, 2014, page no. 337;
https://books.google.co.in/books?isbn=9351520099
Rajiv M Borle – 2014)
17. Answer: D (Ref. Oral Surgery, By Fragiskos D. Fragiskos, 2007, page no. 358)
18. Answer: A (Ref. Surgical Emergencies in Clinical Practice, By Iqbal Shergill, Manit Arya, Tahwinder Upile, 2012, page no. 125)
Ludwig’s angina is the name given to massive, firm, brawny cellulites involving simultaneously, the submandibular,
sublingual, and submental spaces bilaterally.
• Intraorally, the swelling develops rapidly, which involves the sublingual tissues, and distends or raises the floor of
mouth, woody edema of the floor of mouth and tongue.
• Tongue may be raised against palate, increased salivation, stiffness of tongue movements, difficulty in swallowing,
backward spread of infection leads to edema of glottis, resulting in respiratory obstruction and embarrassment.
• There is reduced control of muscles and jaw posture—Saliva is excessive and saliva may be seen drooling. Oral opening
and jaw movements may be reduced.
19. Answer: B (Ref. Textbook of Oral Medicine, By Anil Govindrao Ghom, Savita Anil (Lodam) Ghom, 2014, page no. 420)
20. Answer: C (Ref. Textbook of Oral and Maxillofacial Surgery, By Rajiv M Borle, 2014, page no. 382)
21. Answer: B (Ref. Maxillofacial Imaging, By Tore A. Larheim, Per-Lennart A. Westesson, 2008, page no. 119)
22. Answer: C (Ref. Orofacial Pain and Headache, By Yair Sharav, Rafael Benoliel, 2008, page no. 105)
Parapharyngeal space include lateral pharyngeal and retropharyngeal spaces. The infection of this space is dangerous because
• The lateral pharyngeal space is intimately related with carotid sheath (which consists of internal jugular rein, vagus
nerve, internal and common carotid arteries).
• Infections from these spaces spreads directly into neck and mediastinum.
23. Answer: B (Ref. Head and Neck Cancer: A Multidisciplinary Approach, By Louis B. Harrison, Roy B. Sessions, Waun Ki
Hong, 2009, page no. 122)
Hyperbaric oxygen therapy involves intermittent, daily inhalation of 100% oxygen at 2–3 atmospheric pressure.
Advantages are
• Increased vascular supply
• Increased O2 perfusion to ischemic areas of infection
• Increased bactericidal and bacteriostatic action of increased O2 concentration
24. Answer: C (Ref. Clinical Anatomy of the Eye, By Richard S. Snell, Michael A. Lemp, 2013, page no. 289)
Cavernous thrombosis (CST) is the infectious thrombosis of the cavernous sinus, which is a dural venous space present in
the middle cranial fossa on either side of the sella turcica.
• It is a paired sinus—Anterior and posterior. Infections to cavernous sinus may spread by two path ways.
• The anterior route composed of ophthalmic veins and their anastomosis with the facial vein; the angular vein; the
infraorbital vein; and the inferior palpebral vein; readily allows the invasion of the cavernous sinus. Spread of infection
by this pathway presents the classic picture of a fulminating cavernous sinus thrombosis and CST through this route is
more common than posterior route.
• The pterygoid venous plexus (NEET-2013), which constitutes the posterior route, provide a connection between
cavernous sinus and the retromandibular vein.
25. Answer: A (Ref. Clinical Outline of Oral Pathology: Diagnosis and Treatment, By Lewis R. Eversole, 2001, page no. 412)
26. Answer: C
27. Answer: B (Ref. Textbook of Oral and Maxillofacial Surgery, By Rajiv M Borle, 2014, page no. 337)
The masticator spaces—masseteric, pterygoid, and temporal—are well differentiated but communicate with each other
and with the buccal, submandibular, and parapharyngeal spaces.
• Infection of masticator space occurs most frequently from molar teeth, and the infections of the third molars are
implicated most commonly as the cause.
• Pericoronitis of the gingival flap of the third molars or caries-induced dental abscesses usually can be found in case of
masticator space infection.
• Infection of this space also has been reported as a result of contaminated mandibular block anesthetic injections, or
infection may spread to this space from nearby contiguous spaces.

https://t.me/DentalBooksWorld
666 Triumph's Complete Review of Dentistry

• Infection may also result from direct trauma to or through the muscles of mastication or surgery in the area. It has also
been reported as a complication of circumzygomatic wiring for midfacial trauma.
• Clinically the hallmark of masticator space infection is trismus.
• Swelling may not be a prominent sign of masticator space infection, especially in the masseteric compartment.
28. Answer: D
29. Answer: C (Ref. Cambridge Textbook of Accident and Emergency Medicine, By David V. Skinner, 1997, page no. 471)
Treatment of osteomyelitis include
• Antibiotic therapy
• Hyperbaric oxygen
• Incision and drainage
• Sequestrectomy (removal of dead bone)
• Saucerization (excision of margins of necrotic bone)
30. Answer: A (Ref. Textbook of Oral and Maxillofacial Surgery, By Rajiv M Borle, 2014, page no. 337)
The parapharyngeal spaces communicate with submandibular and sublingual space anteroinferiorly and communicate
with retromandibular space posteriorly.
31. Answer: A (Ref. Textbook of Oral and Maxillofacial Surgery, By Balaji, 2009, page no. 128)

Submasseteric Space
Causes • Mandibular 3rd molars
Contents • Masseteric artery & vein
Location • Anterior – buccal space, parotidomasseteric fascia
• Posterior – parotid gland and its fascia
• Medial – ramus of the mandible
• Lateral – masseter muscle
• Superior – zygomatic arch
• Inferior – inferior border of mandible

TMJ AND MAXILLARY SINUS


1. Answer: A (Ref. Textbook of Oral and Maxillofacial Surgery, By Balaji, 2009, page no. 448)
Trauma results in extravasation of blood into joint space (hemarthrosis). This predisposes to calcification and finally
results in ankylosis.
2. Answer: B (Ref. Temporomandibular Joint Disorders, By Dr. Mohammad Akheel, 2014, page no. 261)
Spasm of muscle is responsible for acute dislocation. For recurrent frequent dislocation ligament and capsular flaccidity,
flattening of articular eminence and trauma are the causative triad.
3. Answer: C (Ref. Textbook of Oral and Maxillofacial Surgery, By Balaji, 2009, page no. 460)
4. Answer: D (Ref. Textbook of Oral and Maxillofacial Surgery, By Neelima Anil Malik, 2012, page no. 266)
Condylectomy Indicated in case of fibrous ankylosis
• Indicated in cases of bony ankylosis
Gap arthroplasty • Osteotomy is performed to remove about 1–1.5 cm of bone and the lower stump of bone
is reshaped Like a condyle
Interpositional In this method, materials like tantalum, steel, acrylic are interposed between the bony
gap arthroplasty fragments for preventing union
5. Answer: C
6. Answer: D (Ref. Shafer’s Textbook of Oral Pathology, By Arya Rajendran, B Sivapathasundharam, 2014, page no. 511)
7. Answer: B (Ref. Textbook of Oral and Maxillofacial Surgery, By Balaji, 2009, page no. 460)
8. Answer: C (Ref. Excerpta medica. Section 34: Plastic Surgery - Volume 17, 1986, page no. 172)
Modified Rehrmann’s buccal advancement flap is also known as Berger’s flap which is used as a method of closing oroantral
fistula.
9. Answer: D (Ref. Burket’s Oral Medicine, 12th edition, By Michael Glick, 2015, page no. 307)
10. Answer: D (Ref. Oral and Maxillofacial Surgery - E-Book: 3-Volume Set, By Raymond J. Fonseca, 2017, page no. 304)

https://t.me/DentalBooksWorld
Chapter 7 • Oral Surgery 667

The most common cause of trismus is due to infection adjacent to muscles of jaw closure, i.e., masseter, medial pterygoid,
temporalis mostly due to involvement of pericoronal, submasseteric, and pterygomandibular spaces.
11. Answer: B (Ref. Peterson’s Principles of Oral and Maxillofacial Surgery, 2012, page no. 1168)
Eminectomy involves excision of articular eminence and thus allows the condylar head to move anteroposteriorly free of
obstruction.
12. Answer: A (Ref. Textbook of Oral and Maxillofacial Surgery, By Rajiv Borle, 2014, page no. 579)
Submandibular incision is given about 1 cm below the angle of mandible. This is used for surgery on ramus and neck of
condyle and this is a good approach for the placement and fixing of costochondral graft.
13. Answer: B (Ref. Principles and Practice of Orthognathic Surgery, By Jeffrey C. Posnick, 2013, page no. 1472)
In unilateral ankylosis the face is asymmetrical with fullness on the affected side of mandible and flattening on the
unaffected side. The patients with bilateral ankylosis will have typical “bird face” appearance.
14. Answer: C (Ref. Operative Oral and Maxillofacial Surgery, 3rd edition, By John D. Langdon, Mohan F. Patel, Robert Ord,
2017, page no. 62–9)
Autogenous costochondral graft acts as a growth center to the TMJ area to provide future growth of mandible.
A minimum of 1.5 cm of chondral graft is harvested through the inframammary region. Either sixth or seventh rib is
harvested.
15. Answer: D (Ref. TMJ and Craniofacial Pain: Diagnosis and Management, By James R. Fricton, Richard J. Kroening, Kate
M. Hathaway, 1988, page no. 90)
16. Answer: B (Ref. Orofacial Pain and Headache, By Yair Sharav, Rafael Benoliel, 2008, page no. 181)
17. Answer: A
18. Answer: B (Ref. Head & Neck Surgery–Otolaryngology - Volume 1, By Byron J. Bailey, Jonas T. Johnson, Shawn D.
Newlands, 2006, page no. 365)
A semilunar incision is made in the canine fossa from canine to second molar area well above the apices.
19. Answer: C (Ref. Oral and Maxillofacial Surgery, By Cyrus Kerawala, Carrie Newlands, 2014, page no. 419)
The palatal root of maxillary first molar, which is broken during extraction, is the most common cause of oroantral
communication. Sometimes a conical maxillary third molar slips into antrum along with the fracture of tuberosity and
creates oroantral communication.
20. Answer: D (Ref. Textbook of Oral and Maxillofacial Surgery, By Balaji, 2009, page no. 334)
21. Answer: A
22. Answer: D (Ref. Pharmacology and Therapeutics for Dentistry - E-Book, By John A. Yagiela, Frank J. Dowd, Bart Johnson,
2010, page no. 385)
Hydrocortisone compounds—intraarticular injection of the hydrocortisone reduces the inflammatory process within the
joint.
23. Answer: D (Ref. Textbook of Oral and Maxillofacial Surgery, By Balaji, 2009, page no. 336)
The main aim of nasal antrostomy is to allow drainage from sinus to occur into the nose. If nasal antrostomy is not done in
inferior meatus, it will result in incomplete drainage.
24. Answer: D (Ref. Oral Surgery for the General Dentist, By Harry Dym, Orrett E. Ogle, 2011, page no. 231)
If the opening is small (≤0.5 mm), a good clot is formed and normal healing will occur without any complication.
If the opening is large (>0.5 mm), immediate closure should be done to reduce the chance of contamination and formation
of an oroantral fistula.
25. Answer: D (Ref. Oral Anatomy, Histology and Embryology E-Book, By Barry K. B Berkovitz, 2017, page no. 10)
26. Answer: B (Ref. Textbook of Oral Radiology, By Ghom, 2009, page no. 546)
27. Answer: D (Ref. Textbook of Oral Radiology - E-Book, By Anil Govindrao Ghom, 2017, page no. 713)
Radiographic Features of Maxillary Sinusitis
Water’s view/Occipitomental 15° is the most valuable radiograph
Odontogenic sinusitis:
• Either totally opaque sinus or a fluid level
Acute maxillary sinusitis
• Shows uniform opacity. Sometimes a fluid level is discernible.
Chronic maxillary sinusitis:
• Shows pansinusitis
• Presence of fluid level

https://t.me/DentalBooksWorld
668 Triumph's Complete Review of Dentistry

• Thickened lining membrane


• Opaque air space may enclose polyps associated with mucosal thickening
• In case of presence of tooth or root, the characteristic outline is seen within the sinus
28. Answer: C (Ref. Operative Oral and Maxillofacial Surgery, 2nd edition, By John Langdon, Mohan Patel, Peter Brennan,
2010, page no. 313)
29. Answer: D (Ref. Textbook of Oral and Maxillofacial Surgery, By Balaji, 2009, page no. 334)
<24 hours—Immediate closure, supportive measures
>24 hours—Defer the treatment until the gingival edges of the fistula has healed properly
>1 month—Re-establish sinus drainage, supportive treatment, when acute condition subsides, fistula should be surgically
repaired
30. Answer: B (Ref. Illustrated Anatomy of the Head and Neck - E-Book, By Margaret J. Fehrenbach, Susan W. Herring, 2015,
page no. 129)
31. Answer: A (Ref. Peterson’s Principles of Oral and Maxillofacial Surgery, 2012, page no. 1176)
32. Answer: C (Ref. Burket’s Oral Medicine, By Lester William Burket, Martin S. Greenberg, Michael Glick, 2008,
page no. 248)

PREPROSTHETIC AND ORTHOGNATHIC SURGERY


1. Answer: B (Ref. Rhinologic and Sleep Apnea Surgical Techniques, By Stilianos E. Kountakis, T. Metin Önerci, 2007,
page no. 361)
Genioplasty is done to correct the deformities of chin without altering the denture-bearing part or occlusion of the patient.

Augmentation genioplasty To increase the chin projection


To reduce the symphysis region in class III patients
Reduction genioplasty
To correct the facial asymmetry
Straightening genioplasty
Indicated in patients with short vertical facial height with class I and class II deep bite
Lengthening genioplasty
cases
2. Answer: A (Ref. Textbook of Complete Dentures, By Arthur O. Rahn, John R. Ivanhoe, Kevin D. Plummer, 2009,
page no. 68)
3. Answer: A (Ref. Plastic, Reconstructive and Aesthetic Surgery, By Kiyonori Harii, 1995, page no. 260)
Anterior maxillary osteotomy (to correct maxillary prognathism) is combined with anterior sub apical mandibular
osteotomy (to correct mandibular prognathism) to correct bimaxillary protrusion.
4. Answer: B (Ref. Peterson’s Principles of Oral and Maxillofacial Surgery, 2012, page no. 1362)
The osteotomy splits the ramus (mainly) and the posterior body of mandible sagittally which allows either setback or
advancement.
Sagittal split osteotomy is useful in correction of:
• Mandibular retrognathism
• Mandibular prognathism
• Open bite or apertognathia
5. Answer: A (Ref. Orthognathic Surgery: Principles, Planning and Practice, By Farhad B. Naini, Daljit S. Gill, 2017,
page no. 878)
6. Answer: D (Ref. Prosthodontic treatment for edentulous patients, By Carl O. Boucher, Judson C. Hickey, George Albert
Zarb, 1975, page no. 100)
7. Answer: A (Ref. Contemporary Oral and Maxillofacial Surgery - E-Book, By James R. Hupp, Myron R. Tucker, Edward
Ellis, 2013, page no. 204)
8. Answer: C (Ref. Textbook of Oral and Maxillofacial Surgery, By Balaji, 2009, page no. 259)
VY plasty procedure or Z plasty procedure is used for frenectomy procedure to increase the stability of denture.
9. Answer: A (Ref. Textbook of Oral and Maxillofacial Surgery, By Neelima Anil Malik, 2012, page no. 332)
10. Answer: A (Ref. Textbook of Oral and Maxillofacial Surgery, By Balaji, 2009, page no. 262)
The palatal torus should be excised with the help of burs and rongeurs by splitting into small segments. Nasal perforation
occurs only when the torus palatinus is excised en mass with the help of chisel.

https://t.me/DentalBooksWorld
Chapter 7 • Oral Surgery 669

11. Answer: A (Ref. Textbook of Oral and Maxillofacial Surgery, By Rajiv M Borle, 2014, page no. 258)
Oro-antral fistula is a common complication following surgical reduction of maxillary tuberosity area.
12. Answer: A (Ref. Textbook of Oral and Maxillofacial Surgery, By Rajiv M Borle, 2014, page no. 252)
13. Answer: A (Ref. Craniofacial Surgery, By Seth Thaller, James P. Bradley, Joe I. Garri, 2007, page no. 210)
Bilateral sagittal split osteotomy (BSSO) is a very popular, most versatile procedure performed on the mandibular ramus
and body.
• The osteotomy splits the ramus and the posterior body of the mandible sagittally, which allows either setback or
advancement.
• The surgical procedure of choice for skeletal class II malocclusion due to retrognathic mandible is
14. Answer: A
In reduction genioplasty, the symphysis part of mandible is reduced so that the chin will attain a straight profile.
15. Answer: A (Ref. Contemporary Oral and Maxillofacial Surgery - E-Book, By James R. Hupp, Myron R. Tucker, Edward
Ellis, 2013, page no. 202)
Treatment of choice for bilateral soft tissue tuberosity undercuts is removal of tissue undercut on one side so that the
undercut on the other side helps in retention. Treatment of choice for bilateral bony tuberosity undercuts is removal of
both the undercuts such that no bony undercut exists.
16. Answer: B (Ref. Principles of Oral Surgery, By J. R. Moore, G. V. Gillbe, 1981, page no. 52)
Dry heat sterilization or hot air oven
• The killing effect of dry heat is due to protein denaturation, oxidative damage, and toxic effect of elevated levels of
electrolytes.
• Usually 160°C for 2 hours or 120°C for 6 hours is widely employed for sterilization of cutting instruments.
• The spores of nontoxigenic strain of Clostridium tetani are used as test organism.
Moist heat sterilization or auto clave
• The lethal effect of moist heat is due to denaturation and coagulation of protein.
• Usually at temperature of 121°C at 15 pounds of pressure is recommended for 15 minutes or 134°C at 30 lb of pressure
is recommended for 3 minutes.
17. Answer: C (Ref. Oral Surgery, By Fragiskos D. Fragiskos, 2007, page no. 36)
18. Answer: B (Ref. Essential Microbiology for Dentistry E-Book, By Lakshman Samaranayake, 2011, page no. 345)
19. Answer: B (Ref. Textbook of Oral and Maxillofacial Surgery, By Balaji, 2009, page no. 82)
20. Answer: A (Ref. Oral Surgery in Dental Practice, By Eberhard Krüger, Philip Worthington, 1981, page no. 157)
21. Answer: A (Ref. Cryotherapy in Chest Medicine, By Jean-Paul Homasson, 2012, page no. 76)
22. Answer: D (Ref. Textbook of Oral and Maxillofacial Surgery, By Balaji, 2009, page no. 66)
23. Answer: B (Ref. Contemporary Oral and Maxillofacial Surgery - E-Book, By James R. Hupp, Myron R. Tucker, Edward
Ellis, 2013, page no. 74)
24. Answer: D (Ref. Textbook of Oral and Maxillofacial Surgery, By Neelima Anil Malik, 2012, page no. 49)

Nonabsorbable sutures Absorbable sutures


Surgical steel • Surgical gut or catgut
Silk • Polyglycolic acid (Dexon; Syneture)
Linen • Polyglactin 910 (Vicryl; Ethicon)
Nylon • Polydioxanone (PDS; Ethicon)
Polypropylene • Polytrimethylene carbonate (Maxon; Syneture)
Polybutester • Glycomer 631 (Biosyn; Syneture)
coton • Polyglytone 6211 (Caprosyn; Syneture)
• Poliglecaprone (Monocryl; Ethicon)
25. Answer: A (Ref. Bureau of Standards Journal of Research - Volume 10, 1933, page no. 693)
26. Answer: A

https://t.me/DentalBooksWorld
670 Triumph's Complete Review of Dentistry

MISCELLANEOUS
1. Answer: C
2. Answer: A

D1 D2 D3 D4

3. Answer: D
4. Answer: C
5. Answer: B
6. Answer: C
• Smaller needles bend more than longer ones
• Previously bent needles have more chances of breaking since it has a weakened structure
• Manufacturing defect
7. Answer: C
8. Answer: A

https://t.me/DentalBooksWorld
8 Pharmacology

SYNOPSIS

General Pharmacology
Routes of administration:
• Routes of administration can be of two broad categories: Local or systemic.
• Local route: Local routes include topical application on the skin and mucous membranes as well as the routes like intra-articular.
• Systemic route: Systemic routes include oral, sublingual, transdermal, nasal, inhalational, rectal, and other parenteral routes
(intravenous, intramuscular, intradermal, and subcutaneous).
Note that Intra-articular is the local route and subcutaneous is the systemic route

Routes Advantages Disadvantages


Oral Safer, economical High first pass metabolism
Sublingual Avoids first pass metabolism, self- –
administered, used in emergencies
Transdermal Constant release of the drug and administered Only for highly lipid soluble drugs
less frequently
Inhalational Rate of drug delivery can be controlled –
Rectal Avoids first pass metabolism –
Parenteral (IV, IM, Intradermal, IV – can be given as bolus or as infusion Systemic toxicity maximum
Subcutaneous)

Pharmacokinetics: Effect of body on the drug


Pharmacodynamics: Effect of drug on the body
• Pharmacokinetics is also called ADME study as it deals with absorption, distribution, metabolism, and excretion of a drug.

Absorption Acidic drugs can cross the acidic medium easily and basic drugs can cross the basic medium easily.
Which means “When medium is same, drugs can cross the membrane”
So acidic drugs will be absorbed from stomach (as it has acidic pH) and it will be in unionized form
Note:
Nonionized form is lipid soluble and ionized form is nonlipid soluble
So if pH is high (means alkaline) then ionized form increases and nonionized form decreases (in case
of acidic drugs)
So if pH is low (means acidic) then ionized form decreases and nonionized form increases (in case of
basic drugs)
Distribution After absorption the drug has to be distributed to various tissues. This can be determined by the volume of
distribution (Vd) (see below)
Metabolism Metabolism occurs in liver
Metabolism may occur with the help of microsomal (present in smooth endoplasmic reticulum) or
nonmicrosomal enzymes. (see below)
Excretion The major route of excretion is kidney. Excretion through kidneys occurs by glomerular filtration, tubular
reabsorption, and tubular secretion (see below)
https://t.me/DentalBooksWorld
672 Triumph's Complete Review of Dentistry

Concept with Volume of Distribution and Plasma Protein Binding


Drugs which are highly lipid soluble can cross the blood Hence high volume of distribution
vessel wall (the more the drugs cross the blood vessel
wall, the more it can be distributed to tissues)
Drugs highly bound to plasma protein (stays in plasma – Hence low volume of distribution
less will go to the tissues)
So the drugs which is not plasma protein bound is Responsible for the action of the drug by distribution
So the drugs which has high plasma binding protein Will have longer duration of action
Vd = Dose administrated (IV)/plasma concentration (C)

Concept with Plasma Protein Binding


For a high plasma protein binding (PPB)
Duration of action Longer duration of action
Volume of distribution Low volume of distribution as drug stays in plasma

Metabolism
Phase I (Both microsomal as well as Phase II
nonmicrosomal)
Oxidation Microsomal Nonmicrosomal
• Hydroxylation Glucuronide conjugation Glutathione conjugation
• Dealkylation (Most common phase II reaction) Acetylation
• Deamination Methylation
Reduction sulfation
Hydrolysis

Microsomal enzymes Nonmicrosomal enzymes


Oxidations All phase II except glucuronidation
–– Cytochrome P450 Oxidation
–– Flavin Monooxygenases Reduction
Glucuronide conjugation Hydrolysis
Reduction
Hydrolysis

Phase I reactions Phase II reactions


Oxidation Glucuronidation
Reduction Acetylation
Hydrolysis Methylation
Cyclization Sulfation
Decyclization Glycine conjugation

Excretion
Glomerular filtration Depends on the plasma protein binding and renal blood flow. It does not depend on the lipid
solubility as all substances can cross the membrane.
Tubular reabsorption Depends on the lipid solubility
Tubular secretion Does not depend on lipid solubility or plasma protein binding.

https://t.me/DentalBooksWorld
Chapter 8 • Pharmacology 673

Order of kinetics

First order kinetics - most drugs


Plasma Drug is readily metabolished in the liver
Conc

0 1 2 3 4 5 6 7 8 9 10 11 12 13
Time (h)

Zero order kinetics


Metabolism is saturable e.g. alcohol
Metabolism is saturable in drug excess
e.g.aspirin and phenytoin
Plasma
Conc

Time (h)

First-order kinetics (first picture in With most drugs, there is a rapid fall in drug levels, as most drugs are readily
the graph) metabolized, and there is an excess of enzyme available for the metabolism. Thus,
the enzyme never becomes saturated with the drug. This is known as the first-order
kinetics. In first-order kinetics, increasing the concentration of the drug increases
the metabolism of the drug. First-order kinetics is also observed with drugs that are
eliminated unchanged.
Zero-order kinetics (second picture With some drugs there is a limited amount of enzyme available to metabolize the
in the graph) drug, and when that limit is reached, metabolism occurs at a constant rate. Thus, the
enzyme becomes saturated with drug. This is known as zero-order kinetics, and is seen
as a straight line on the graph. In zero-order kinetics, increasing the concentration
of drug above a certain point does not increase the rate of metabolism. The best
known example of zero-order kinetics is alcohol. There are no notable examples of
therapeutic drugs that have saturable metabolism and zero-order kinetics. However,
some therapeutic drugs taken in excess can have saturable kinetics. Examples include
aspirin and the anti-epileptic drug phenytoin.

Half-Life (t1/2)
It is the time required to reduce the plasma concentration to half (50%) of the original value.
Elimination of the drug from plasma is 50% in one half-life, 75% (50 + 25) in two half-lives, 87.5% (50 + 25 + 12.5) in three
half-lives, and so on.

1 half-life 50% of drug eliminated


2 half-life 25
3 half-life 12.5
4 half-life 6.25
5 half-life 3.10 (steady state concentration)

Therapeutic index
Median Effective Dose (ED50): It is the dose that will produce the half of the maximum (50%) response. The more ED50, the
lower the potency and vice versa.
Median Lethal Dose (LD50): It is the dose that will result in the death of 50% of the animals receiving the drug. The more LD50,
the safer the drug.

https://t.me/DentalBooksWorld
674 Triumph's Complete Review of Dentistry

Therapeutic Index (T.I.): It is a measure of the safety of a drug. It is calculated as a ratio of LD50 to ED50. Drugs having high
T.I. are safer whereas those having low T.I. are more likely to be toxic.
Therapeutic Index (T.I.) = LD 50/ED 50

Dose response curve


The farther the curve determines potency (farthest – least potent), the higher the curve determines efficacy
(highest – more efficacious)
The curve is usually a hyperbola

The difference between ‘potency’


(affinity) and ‘efficacy’ (activity)

Efficacy B
difference
A
Response

Potency
difference

Log drug dose (concentration)

In simple terms the height of the graph represents efficacy and the difference between two drug curves (width) represents
potency. So here drug A is more potent and drug B is more efficacious

Another example

More potent
Less efficacious
% Response

fentanyl morphine aspirin

0.01 0.1 1 10 100 1000


Dose (mg)
log scale

In the above example, fentanyl is more potent than morphine and aspirin but morphine is more efficacious, while aspirin is least
potent and least efficacious.

Summary of Clinical Trials


Phase Name Conducted on Purpose
I Human pharmacology and safety Healthy volunteers • To know maximum tolerable dose (MTD)
• Safety and tolerability
II Therapeutic exploratory Patients • To establish therapeutic efficacy
III Therapeutic Up to 5,000 patients • To confirm therapeutic efficacy
confirmatory from several centers
IV Postmarketing surveillance Large number of patients • To know rare long-term adverse effects
0 (Zero) Microdosing studies Healthy volunteers • Very low dose (1/100th of human dose)

https://t.me/DentalBooksWorld
Chapter 8 • Pharmacology 675

Clinical trial Other name


Phase 0 Microdosing studies
Phase I Human pharmacology and safety
Phase II Therapeutic exploratory
Phase III Therapeutic confirmatory
Phase IV Postmarketing surveillance

CLASSIFICATION OF DRUGS
Autonomic Nervous System
Cholinergic Drugs
(Cholinomimetic, Parasympathomimetic)
Cholinergic Agonists
1. Choline esters: Acetylcholine, Methacholine, Carbachol, Bethanechol
2. Alkaloids: Muscarine, Pilocarpine, Arecoline
ANTICHOLINESTERASES
1. Reversible
(a) Carbamates: Physostigmine (Eserine), Neostigmine, Pyridostigmine, Edrophonium, Rivastigmine*, Donepezil*,
Galantamine*
(b) Acridine: Tacrine*
* They act as cholinesterase inhibitors.
2. Irreversible
(a) Organophosphates: Dyflos (DFP), Echothiophate, Malathion**, Diazinon**, Tabun***, Sarin***, Soman***
(b) Carbamates: Carbaryl**, Propoxur**
(i) Cerebroselective; used for Alzheimer’s disease
** Insecticides
*** Nerve gases for chemical warfare
Anticholinergic Drugs
(Muscarinic receptor antagonists, Parasympatholytic, Atropinic)
1. Natural alkaloids
• Atropine, Hyoscine (Scopolamine)
2. Semisynthetic derivatives
• Homatropine, Atropine methonitrate, Hyoscine butyl bromide, Ipratropium bromide, Tiotropium bromide
3. Synthetic compounds
(a) Mydriatics: Cyclopentolate, Tropicamide
(b) Antisecretory antispasmodics:
(i) Quaternary compounds: Propantheline, Oxyphenonium, Clidinium, Pipenzolate methylbromide, Isopropamide,
Glycopyrrolate
(ii) Tertiary amines: Dicyclomine, Valethamate, Pirenzepine
(c) Vasicoselective: Oxybutynin, Flavoxate, Tolterodine
(d) Antiparkinsonian: Trihexyphenidyl (Benzhexol), Procyclidine, Biperiden
GANGLION BLOCKING AGENTS
A. Competitive blockers
• Quaternary ammonium compounds
–   Hexamethonium, Pentolinium
• Amines (secondary/tertiary)
–   Mecamylamine, Pempidine

https://t.me/DentalBooksWorld
676 Triumph's Complete Review of Dentistry

• Monosulfonium compound
–   Trimethaphan camphorsulfonate
B. Persistent depolarizing blockers
• Nicotine (large dose), anticholinesterases (large dose)
Adrenergic Drugs (Sympathomimetics)
1. Directly acting
• Adrenaline, Noradrenaline, Isoprenaline, Dopamine, Phenylephrine, Methoxamine, Xylometazoline, Salbutamol
2. Indirectly acting
• Tyramine
3. Mixed action
• Ephedrine, Amphetamine, Mephentermine
Adrenergic Blocking Agents
I. Nonequilibrium type
(i)  α-Haloalkylamines: Phenoxybenzamine
II. Equilibrium type (competitive)
A. Nonselective
(i) Ergot alkaloids: Ergotamine, Ergotoxine
(ii) Hydrogenated ergot alkaloids: Dihydroergotamine (DHE), Dihydroergotoxine
(iii) Imidazolines: Tolazoline, Phentolamine
(iv) Miscellaneous: Chlorpromazine, Ketanserin
B. α1 selective: Prazosin, Terazosin, Doxazosin, Alfuzosin, Tamsulosin
C. α2 selective: Yohimbine
Adrenergic Blocking Agents
Nonselective (α1 and α2)
(a) Without intrinsic sympathomimetic activity
• Propranolol, Sotalol, Timolol
(b) With intrinsic sympathomimetic activity
• Pindolol
(c) With additional α blocking property
• Labetalol, Carvedilol
Cardioselective (α1)
• Metoprolol, Atenolol, Acebutolol, Bisoprolol, Esmolol, Betaxolol, Celiprolol, Nebivolol
GENERATION-WISE CLASSIFICATION
1. First Generation (older, nonselective)
• Propranolol, Timolol, Sotalol, Pindolol
2. Second Generation (α2 selective)
• Metoprolol, Atenolol, Acebutolol, Bisoprolol, Esmolol
3. Third Generation (with additional α blocking and/or vasodilator property)
• Labetalol, Carvedilol, Celiprolol, Nebivolol
Histaminergic Agonists
1. Nonselective (H1+H2+H3) agonists
• Histamine
• Betahistine: 4–8 mg 6 hourly; Vertin 8 mg tab
2. Selective H1 agonists
• 2-Methylhistamine, 2-Pyridyl ethylamine, 2-Thiazolyl ethylamine

https://t.me/DentalBooksWorld
Chapter 8 • Pharmacology 677

3. Selective H2 agonists
• 4-Methylhistamine, Dimaprit, Impromidine
4. Selective H3 agonists
• Methylhistamine, Imetit
H1-Antagonists
(Conventional Antihistaminics)
1. Highly sedative
• Diphenhydramine, Dimenhydrinate, Promethazine, Hydroxyzine
2. Moderately sedative
• Pheniramine, Cyproheptadine, Meclozine (Meclizine), Buclizine, Cinnarizine
3. Mildly sedative
• Chlorpheniramine, Dexchlorpheniramine, Dimethindine, Triprolidine, Mebhydroline, Cyclizine, Clemastine
4. Second generation (Nonsedating)
• Terfenadine, Fexofenadine, Astemizole, Loratadine, Desloratadine, Cetirizine, Levocetirizine, Azelastine, Mizolastine,
Ebastine, Rupatadine

Antipyretic-Analgesics and Nonsteroidal Anti-Inflammatory Drugs (NSAIDs)


A. Nonselective COX inhibitors (conventional NSAIDs)
• Salicylates: Aspirin
• Propionic acid derivatives: Ibuprofen, Naproxen, Ketoprofen, Flurbiprofen
• Anthranilic acid derivative: Mefenamic acid
• Aryl-acetic acid derivatives: Diclofenac, Aceclofenac
• Oxicam derivatives: Piroxicam, Tenoxicam
• Pyrrolopyrrole derivative: Ketorolac
• Indole derivative: Indomethacin
• Pyrazolone derivatives: Phenylbutazone, Oxyphenbutazone
B. Preferential COX-2 inhibitors
• Nimesulide, Meloxicam, Nabumetone, Etodolac
C. Selective COX-2 inhibitors
• Celecoxib, Etoricoxib, Parecoxib
D. Analgesic-antipyretics with poor anti-inflammatory action
• Para aminophenol derivative: Paracetamol (Acetaminophen)
• Pyrazolone derivatives: Metamizole (Dipyrone), Propyphenazone
• Benzoxazocine derivative: Nefopam

Drugs for Bronchial Asthma


A. Bronchodilators
• Sympathomimetics: Salbutamol, Terbutaline, Bambuterol, Salmeterol, Formoterol, Ephedrine
• Methylxanthines: Theophylline (anhydrous), Aminophylline, Choline theophyllinate, Hydroxyethyl theophylline,
Theophylline ethanolate of piperazine, Doxophylline
• Anticholinergics: Ipratropium bromide, Tiotropium bromide
B. Leukotriene antagonists
• Montelukast, Zafirlukast
C. Mast cell stabilizers
• Sodium cromoglycate, Ketotifen

https://t.me/DentalBooksWorld
678 Triumph's Complete Review of Dentistry

D. Corticosteroids
• Systemic: Hydrocortisone, Prednisolone, and others
• Inhalational: Beclomethasone dipropionate, Budesonide, Fluticasone propionate, Flunisolide, Ciclesonide
E. Anti-IgE antibody
• Omalizumab

Thyroid Inhibitors
• Inhibit hormone synthesis (Antithyroid drugs)
–– Propylthiouracil, Methimazole, Carbimazole
• Inhibit iodide trapping (Ionic inhibitors)
–– Thiocyanates (–SCN), Perchlorates (–ClO4), Nitrates (–NO3)
• Inhibit hormone release
–– Iodine, Iodides of Na and K, Organic iodide
• Destroy thyroid tissue
–– Radioactive iodine (131I, 125I, 123I)

Antidiabetic Drugs (Hypoglycemic Drugs)


Insulin
A. Rapid Acting
• Insulin lispro
• Insulin aspart
• Insulin glulisine
B. Short Acting
• Regular (soluble) Insulin
• Prompt insulin zinc suspension (amorphous) or Semilente
C. Intermediate Acting
• Insulin zinc suspension or Lente (Ultra: Semi: 7:3)
• Neutral protamine hagedorn (NPH) or Insulin isophane
D. Long Acting
• Extended Insulin Zinc Suspension (crystalline) or Ultralente
• Protamine Zinc Insulin (PZI)
• Insulin glargine

Oral Hypoglycemic Drugs


1. Sulfonylureas
First generation Second generation
Tolbutamide Glibenclamide (Glyburide)
Chlorpropamide Glipizide
Gliclazide
Glimepiride
2. Biguanide
Metformin
3. Meglitinide/Phenylalanine analogues
Repaglinide Nateglinide
4. Thiazolidinediones
Rosiglitazone Pioglitazone

https://t.me/DentalBooksWorld
Chapter 8 • Pharmacology 679

5. α Glucosidase inhibitors
Acarbose Miglitol
6. Dipeptidyl peptidase-4 (DPP-4) inhibitor
Sitagliptin Vildagliptin

Corticosteroids
I. Glucocorticoids
• Short acting
–– Hydrocortisone (cortisol), Cortisone
• Intermediate acting
–– Prednisolone, Methyl-prednisolone, Triamcinolone
• Long acting
–– Dexamethasone, Betamethasone, Deflazacort
II. MINERALOCORTICOIDS
• Desoxycorticosterone acetate (DOCA), Fludrocortisone, Aldosterone
Skeletal Muscle Relaxants
Peripherally Acting Muscle Relaxants
I. Neuromuscular blocking agents
A. Nondepolarizing (Competitive) blockers
1. Long acting: d-Tubocurarine, Pancuronium, Doxacurium, Pipecuronium
2. Intermediate acting: Vecuronium, Atracurium, Cisatracurium, Rocuronium, Rapacuronium
3. Short acting: Mivacurium
B. Depolarizing blockers: Succinylcholine (SCh., Suxamethonium), Decamethonium (C-10)
II. Directly acting agents
• Dantrolene sodium
• Quinine
Centrally Acting Muscle Relaxants
1. Mephenesin congeners: Mephenesin, Carisoprodol, Chlorzoxazone, Chlormezanone, Methocarbamol
2. Benzodiazepines: Diazepam and others
3. GABA derivative: Baclofen
4. Central α2 agonist: Tizanidine

Local Anesthetics
A. Injectable anesthetic
1. Low potency, short duration
• Procaine
• Chloroprocaine
2. Intermediate potency and duration
• Lidocaine (Lignocaine)
• Prilocaine
3. High potency, long duration
• Tetracaine (Amethocaine)
• Bupivacaine
• Ropivacaine
• Dibucaine (Cinchocaine)

https://t.me/DentalBooksWorld
680 Triumph's Complete Review of Dentistry

B. Surface anesthetic
Soluble Insoluble
Cocaine Benzocaine
Lidocaine Butyl aminobenzoate
Tetracaine (Butamben)
Benoxinate Oxethazaine

Sedative-Hypnotics
1. Barbiturates
Long acting Short acting Ultra short acting
Phenobarbitone Butobarbitone Thiopentone
Pentobarbitone Methohexitone
2. Benzodiazepines
Hypnotic Antianxiety Anticonvulsant
Diazepam Diazepam Diazepam
Flurazepam Chlordiazepoxide Lorazepam
Nitrazepam Oxazepam Clonazepam
Alprazolam Lorazepam Clobazam
Temazepam Alprazolam
Triazolam
3. Newer nonbenzodiazepine hypnotics
• Zopiclone, Zolpidem, Zaleplon
4. Other CNS depressants
• Chloral hydrate, Triclofos, Paraldehyde, Glutethimide, Methyprylon, Methaqualone, Meprobamate, Promethazine,
Chlorpromazine, Amitriptyline, Morphine, Hyoscine

Opioid Analgesics
1. Natural opium alkaloids: Morphine, Codeine
2. Semisynthetic opiates: Diacetylmorphine (Heroin), Pholcodine, Oxymorphone, Hydromorphone, Oxycodone
3. Synthetic opioids: Pethidine (Meperidine), Fentanyl, Alfentanil, Sufentanil, Methadone, Dextropropoxyphene, Tramadol
Complex Action Opioids and Opioid Antagonists
1. Agonist-antagonists (α-analgesics)
• Nalorphine, Pentazocine, Nalbuphine, Butorphanol
2. Partial/weak ì agonist + α antagonist
• Buprenorphine
3. Pure antagonists
• Naloxone, Naltrexone, Nalmefene

Antihypertensive Drugs
1. Angiotensin converting enzyme (ACE) inhibitors
• Captopril, Enalapril, Lisinopril, Perindopril, Ramipril, Benazepril, Trandolapril, Fosinopril, Imidapril
2. Angiotensin (AT1 receptor) antagonists
• Losartan, Candesartan, Irbesartan, Valsartan, Telmisartan, Olmesartan
3. Calcium channel blockers
• Verapamil, Diltiazem, Nifedipine, Felodipine, Amlodipine, S(-) Amlodipine, Nitrendipine, Lacidipine, Benidipine, Lercanidipine

https://t.me/DentalBooksWorld
Chapter 8 • Pharmacology 681

4. Diuretics
• Thiazides: Hydrochlorothiazide, Chlorthalidone, Indapamide
• High ceiling: Furosemide, etc.
• K+ sparing: Spironolactone, Triamterene, Amiloride
5. Adrenergic blockers
• Propranolol, Metoprolol, Atenolol, etc.
6. Adrenergic blockers
• Labetalol, Carvedilol
7. Adrenergic blockers
• Prazosin, Terazosin, Doxazosin,
• Phentolamine, Phenoxybenzamine
8. Central sympatholytics
• Clonidine, Methyldopa
9. Vasodilators
Arteriolar: Hydralazine, Minoxidil, Diazoxide
Arteriolar + venous: Sodium Nitroprusside

Diuretics
1. High efficacy diuretics (Inhibitors of Na+–K+–2Cl– cotransport)
(a) Sulfamoyl derivatives: Furosemide, Bumetanide, Torasemide
(b) Phenoxyacetic acid derivative: Ethacrynic acid.
2. Medium efficacy diuretics (Inhibitors of Na+–Cl– symport)
(a) Benzothiadiazines (thiazides): Hydrochlorothiazide, Benzthiazide, Hydroflumethiazide, Clopamide
(b) Thiazide like (related heterocyclics): Chlorthalidone, Metolazone, Xipamide, Indapamide
3. Weak or adjunctive diuretics
(a) Carbonic anhydrase inhibitors: Acetazolamide
(b) Potassium sparing diuretics
(i) Aldosterone antagonist: Spironolactone, Eplerenone
(ii)  Inhibitors of renal epithelial Na+ channel: Triamterene, Amiloride
(c) Osmotic diuretics: Mannitol, Isosorbide, Glycerol
(d) Xanthines: Theophylline
Antidiuretics
1. Antidiuretic hormone (ADH) and its analogues: Vasopressin, Desmopressin, Lypressin, Terlipressin
2. Diuretics: Thiazides, Amiloride
3. Miscellaneous: Chlorpropamide, Carbamazepine

Hematinics
1. Iron
(a) Oral: Ferrous sulfate, ferrous fumarate, ferrous gluconate, ferrous succinate, colloidal ferric hydroxide, iron hydroxy
polymaltose, ferric ammonium citrate, iron calcium complex, carbonyl iron
(b) Parenteral: Iron-dextran, Iron-sorbitol-citric acid
2. Maturation factors
(a) Vitamin B12: Cyanocobalamin. Hydroxocobalamin, Methylcobalamin
(b) Folic acid, Folinic acid (leucovorin, citrovorum factor, 5-formyl THFA)
3. Miscellaneous
Copper, Pyridoxine, Riboflavin

https://t.me/DentalBooksWorld
682 Triumph's Complete Review of Dentistry

Coagulants
1. Vitamin K
K1 (from plants: Phytonadione (Phylloquinone) fat soluble)
K2 (produced by: Menaquinones bacteria)
K3 (synthetic)
– Fat soluble: Menadione, Acetomenaphthone
– Water soluble: Menadione sodium bisulfite, Menadione sodium diphosphate
2. Miscellaneous: Fibrinogen (human)
• Antihemophilic factor
• Adrenochrome monosemicarbazone
• Rutin, Ethamsylate
Anticoagulants
1. Used in vivo
A. Parenteral anticoagulants
• Heparin, low-molecular-weight heparin
• Heparinoids–Heparan sulfate, Danaparoid, Lepirudin, Ancrod
B. Oral anticoagulants
(i) Coumarin derivatives: Bishydroxycoumarin (Dicumarol), Warfarin sodium, Acenocoumarol (Nicoumalone),
Ethyl biscoumacetate
(ii) Indandione derivative: Phenindione
2. Used in vitro
A. Heparin
B. Calcium complexing agents: Sodium citrate, Sodium oxalate, Sodium edetate

Drugs for Peptic Ulcer


1. Gastric acid secretion inhibitors
(a) H2 antihistamines: Cimetidine, Ranitidine, Famotidine, Roxatidine
(b) Proton pump inhibitors: Omeprazole, Esomeprazole, Lansoprazole, Pantoprazole, S(–) Pantoprazole, Rabeprazole,
Dexrabeprazole.
(c) Anticholinergics: Pirenzepine, Propantheline, Oxyphenonium
(d) Prostaglandin analogues: Misoprostol
2. Gastric acid neutralizers (Antacids)
(a) Systemic: Sodium bicarbonate, Sodium citrate
(b) Nonsystemic: Magnesium hydroxide, Mag. trisilicate, Aluminum hydroxide gel, Magaldrate, Calcium carbonate
3. Ulcer protectives
• Sucralfate, Colloidal bismuth subcitrate (CBS)
4. Anti-H. pylori drugs
• Amoxicillin, Clarithromycin, Metronidazole, Tinidazole, Tetracycline

Antimicrobials
A. Chemical classification
1. Sulfonamides and related drugs: Sulfadiazine and others, Sulfones—Dapsone (DDS), Para-aminosalicylic acid (PAS)
2. Diaminopyrimidines: Trimethoprim, Pyrimethamine
3. Quinolones: Nalidixic acid, Norfloxacin, Ciprofloxacin, Gatifloxacin, etc.
4. b-lactam antibiotics: Penicillins, Cephalosporins, Monobactams, Carbapenems, b-lactamase inhibitors – Clavulanic acid,
Sulbactam
5. Tetracyclines: Oxytetracycline, Doxycycline, etc.

https://t.me/DentalBooksWorld
Chapter 8 • Pharmacology 683

6. Nitrobenzene derivative: Chloramphenicol


7. Aminoglycosides: Streptomycin, Gentamicin, Amikacin, Neomycin, etc.
8. Macrolide antibiotics: Erythromycin, Roxithromycin, Clarithromycin, Azithromycin
9. Lincosamide antibiotics: Lincomycin, Clindamycin
10. Glycopeptides: Vancomycin, Teicoplanin
11. Oxazolidinone: Linezolid
12. Polypeptide antibiotics: Polymyxin-B, Colistin, Bacitracin, Tyrothricin
13. Nitrofuran derivatives: Nitrofurantoin, Furazolidone
14. Nitroimidazoles: Metronidazole, Tinidazole, and others
15. Nicotinic acid derivatives: Isoniazid, Pyrazinamide, Ethionamide
16. Others: Rifampin, Spectinomycin, Cycloserine, Viomycin, Ethambutol, Thiacetazone, Clofazimine
B. Classification on the basis of mechanism of action
1. Inhibit cell wall synthesis: Penicillins, Cephalosporins, Cycloserine, Vancomycin, Bacitracin
2. Cause leakage from cell membranes: Polypeptides—Polymyxins, Colistin, Bacitracin
3. Inhibit protein synthesis: Tetracyclines, Chloramphenicol, Erythromycin, Clindamycin, Linezolid
4. Cause misreading of m-RNA code and affect permeability: Aminoglycosides—Streptomycin, Gentamicin, etc.
5. Inhibit DNA gyrase: Fluoroquinolones—Ciprofloxacin, etc.
6. Interfere with DNA function: Rifampin, Metronidazole
7. Interfere with intermediary metabolism: Sulfonamides, Sulfones, PAS, Trimethoprim, Ethambutol
C. Classification according to spectrum of activity
Narrow and Broad spectrum
• Penicillin G Tetracyclines
• Streptomycin Chloramphenicol
• Erythromycin
Intermediate spectrum
• Aminopenicillins
• Cephalosporins (second/third Gen)
• Fluoroquinolones
• Newer macrolides
D. Classification according to type of action
Primarily bacteriostatic
• Sulfonamides, Ethambutol
• Tetracyclines, Clindamycin
• Chloramphenicol, Linezolid
• Erythromycin
Primarily bactericidal
• Penicillins, Cephalosporins
• Aminoglycosides, Vancomycin
• Polypeptides, Nalidixic acid
• Rifampin, Ciprofloxacin
• Co-trimoxazole, Metronidazole
• Isoniazid, Pyrazinamide
Sulfonamides
1. Short-acting (4–8 hours): Sulfadiazine
2. Intermediate-acting (8–12 hours): Sulfamethoxazole
3. Long-acting (~7 days): Sulfadoxine, sulfamethoxypyrazine
4. Special purpose sulfonamides: Sulfacetamide sodium, sulfasalazine, mafenide, silver sulfadiazine

https://t.me/DentalBooksWorld
684 Triumph's Complete Review of Dentistry

Quinolones
1. Nonfluorinated Quinolone
• Nalidixic acid
2. First-generation Fluoroquinolones
• Norfloxacin, Ofloxacin
• Ciprofloxacin, Pefloxacin
3. Second-generation Fluoroquinolones
• Lomefloxacin, Levofloxacin
• Sparfloxacin, Gatifloxacin, Moxifloxacin
4. Third-generation Fluoroquinolones
• Gemifloxacin, Prulifloxacin
Beta-Lactam Antibiotics
PENICILLINS
A. Natural Penicillin
• Benzyl penicillin (Penicillin G)
B. Semisynthetic Penicillins
1. Acid-resistant alternative to penicillin G
• Phenoxymethyl penicillin (Penicillin V)
2. Penicillinase-resistant penicillins
• Methicillin, Cloxacillin
3. Extended spectrum penicillins
(a) Aminopenicillins: Ampicillin, Bacampicillin, Amoxicillin
(b) Carboxypenicillins: Carbenicillin, Ticarcillin
(c) Ureidopenicillins: Piperacillin, Mezlocillin
C. β-lactamase inhibitors
• Clavulanic acid, Sulbactam, Tazobactam
CEPHALOSPORINS
First generation
Parenteral Oral
• Cephalothin Cephalexin
• Cefazolin Cephradine
• Cefadroxil
Second generation
Parenteral Oral
• Cefuroxime Cefaclor
• Cefoxitin Cefuroxime axetil
• Cefprozil
Third generation
Parenteral Oral
• Cefotaxime Cefixime
• Ceftizoxime Cefpodoxime proxetil
• Ceftriaxone Cefdinir
• Ceftazidime Ceftibuten
• Cefoperazone Cefetamet pivoxil
Fourth generation
Parenteral
• Cefepime
• Cefpirome

https://t.me/DentalBooksWorld
Chapter 8 • Pharmacology 685

Aminoglycoside Antibiotics
A. Systemic aminoglycosides
• Streptomycin, Gentamicin, Kanamycin, Tobramycin, Amikacin, Sisomicin, Netilmicin
B. Topical aminoglycosides
• Neomycin, Framycetin
Antitubercular Drugs
First-line drugs
1. Isoniazid (H)
2. Rifampin (R)
3. Pyrazinamide (Z)
4. Ethambutol (E)
5. Streptomycin (S)
Second-line drugs
1. Thiacetazone (Tzn) Newer drugs
2. Para-aminosalicylic 1.  Ciprofloxacin
acid (PAS) 2.  Ofloxacin
3. Ethionamide (Etm) 3.  Clarithromycin
4. Cycloserine (Cys) 4.  Azithromycin
5. Kanamycin (Kmc) 5.  Rifabutin
6. Amikacin (Am)
7. Capreomycin (Cpr)
Antifungal Drugs
1. Antibiotics
A.  Polyenes: Amphotericin B, Nystatin, Hamycin, Natamycin (Pimaricin)
B.  Heterocyclic benzofuran: Griseofulvin
2. Antimetabolite: Flucytosine (5-FC)
3. Azoles
A.  Imidazoles
Topical: Clotrimazole, Econazole, Miconazole, Oxiconazole
Systemic: Ketoconazole
B.  Triazoles (Systemic): Fluconazole, Itraconazole, Voriconazole
4. Allylamine: Terbinafine
5. Other topical agents: Tolnaftate, Undecylenic acid, Benzoic acid, Quiniodochlor, Ciclopirox olamine, Butenafine,
Sodium thiosulfate
Antiviral Drugs
1. Anti-Herpes virus
• Idoxuridine, Acyclovir, Valaciclovir, Famciclovir, Ganciclovir, Foscarnet
2. Anti-Retrovirus
(a) Nucleoside reverse transcriptase inhibitors (NRTIs):
• Zidovudine (AZT), Didanosine, Zalcitabine, Stavudine, Lamivudine, Abacavir, Tenofovir
(b) Nonnucleoside reverse transcriptase inhibitors (NNRTIs): Nevirapine, Efavirenz, Delavirdine
(c) Protease inhibitors: Ritonavir, Indinavir, Nelfinavir, Saquinavir, Amprenavir, Lopinavir
3. Anti-Influenza virus
• Amantadine, Rimantadine, Oseltamivir, Zanamivir
4. Nonselective Antiviral Drugs
• Ribavirin, Lamivudine, Adefovir dipivoxil, Interferon alpha

https://t.me/DentalBooksWorld
686 Triumph's Complete Review of Dentistry

Antimalarial Drugs
1. 4-Aminoquinolines: Chloroquine, Amodiaquine, Piperaquine
2. Quinoline-methanol: Mefloquine
3. Cinchona alkaloid: Quinine, Quinidine
4. Biguanide: Proguanil (Chloroguanide), Chlorproguanil
5. Diaminopyrimidine: Pyrimethamine
6. 8-Aminoquinolines: Primaquine, Bulaquine
7. Sulfonamides and sulfone: Sulfadoxine, Sulfamethoxypyrazine, Dapsone
8. Tetracyclines: Tetracycline, Doxycycline
9. Sesquiterpene lactones: Artesunate, Artemether, Arteether
10. Amino alcohols: Halofantrine, Lumefantrine
11. Mannich base: Pyronaridine
12. Naphthoquinone: Atovaquone
Antiamoebic Drugs
1. Tissue amebicides
(a) For both intestinal and extraintestinal amoebiasis:
(i) Nitroimidazoles: Metronidazole, Tinidazole, Secnidazole, Ornidazole, Satranidazole
(ii) Alkaloids: Emetine, Dehydroemetine
(b) For extraintestinal amoebiasis only: Chloroquine
2. Luminal amebicides
(a) Amide: Diloxanide furoate, Nitazoxanide
(b) 8-Hydroxyquinolines: Quiniodochlor (Iodochlorhydroxyquin, Clioquinol), Diiodohydroxyquin (Iodoquinol)
(c) Antibiotics: Tetracyclines
A. Drugs acting directly on cells (Cytotoxic drugs)
1. Alkylating agents: Mechlorethamine (Mustine, HCl)
Nitrogen mustards: Cyclophosphamide, Ifosfamide, Chlorambucil, Melphalan
Ethylenimine: Thio-TEPA
Alkyl sulfonate: Busulfan
Nitrosoureas: Carmustine (BCNU), Lomustine (CCNU)
Triazine: Dacarbazine (DTIC)
2. Antimetabolites
Folate antagonist: Methotrexate (Mtx)
Purine antagonist: 6-Mercaptopurine (6-MP), 6-Thioguanine (6-TG), Azathioprine, Fludarabine
Pyrimidine antagonist: 5-Fluorouracil (5-FU), Cytarabine (cytosine arabinoside)
3. Vinca alkaloids: Vincristine (Oncovin), Vinblastine
4. Taxanes: Paclitaxel, Docetaxel
5. Epipodophyllotoxin: Etoposide
6. Camptothecin analogues: Topotecan, Irinotecan
7. Antibiotics: Actinomycin D (Dactinomycin), Doxorubicin, Daunorubicin (Rubidomycin), Mitoxantrone, Bleomycins,
Mitomycin C
8. Miscellaneous: Hydroxyurea, Procarbazine, L-Asparaginase, Cisplatin, Carboplatin, Imatinib
B. Drugs altering hormonal milieu
1. Glucocorticoids: Prednisolone and others
2. Estrogens: Fosfestrol, Ethinylestradiol
3. Selective estrogen receptor modulators: Tamoxifen, Toremifene
4. Selective estrogen receptor downregulator: Fulvestrant

https://t.me/DentalBooksWorld
Chapter 8 • Pharmacology 687

5. Aromatase inhibitors: Letrozole, Anastrozole, Exemestane


6. Antiandrogen: Flutamide, Bicalutamide
7. 5-α reductase inhibitors: Finasteride, Dutasteride
8. GnRH analogues: Nafarelin, Triptorelin
9. Progestins: Hydroxyprogesterone caproate, etc.
Immunosuppressants
1. Calcineurin inhibitors (Specific T-cell inhibitors)
• Cyclosporine (Ciclosporin), Tacrolimus
2. Antiproliferative drugs (Cytotoxic drugs)
• Azathioprine, Cyclophosphamide, Methotrexate, Chlorambucil, Mycophenolate mofetil (MMF), Sirolimus
3. Glucocorticoids
• Prednisolone and others
4. Antibodies
• Muromonab CD3, Antithymocyte globulin (ATG), Rho(D) immunoglobulin

Locally Acting Drugs on Skin and Mucous Membranes


A. Demulcents
1. Gum Acacia: As 2–4% pseudosolution in water
2. Gum Tragacanth: As 2–4% pseudosolution in water
3. Glycyrrhiza: As glycyrrhiza dry extract 1–2 g or liquid extract 2–4 mL in lozenges and mixtures
4. Methylcellulose: 0.5% in nose drops and contact lens solution; Cadilose 0.5% drops in 10 mL bottle
5. Propylene glycol: 50% in water
6. Glycerine: 10–50% in water
B. Emollients
1. Vegetable oils: Olive oil, Arachis oil, Sesame oil, Cocoa butter
2. Animal products: Wool fat, Lard, Bees wax, Spermaceti
3. Petroleum products: Paraffin wax (soft/hard), Liquid paraffin
C. Adsorbents and Protectives
1. Dermal protectives: Magnesium stearate, Zinc stearate, Talc, Calamine, Zinc oxide, Bentonite, Starch, Boric acid, Aloe-vera gel
2. Occlusive protectives: Polyvinyl polymer, Feracrylum, Dimethicone, Sucralfate
D. Astringents
1. Vegetable astringents
(a)  Tannic acid: As glycerine of tannic acid 25%
(b)  Tannins: As tincture catechu, tea leaf infusion
2. Alcohols
• Ethanol, Methanol, Propanol
3. Mineral astringents
• Alum, Aluminum hydroxychloride, Zinc oxide, Zirconyl hydroxychloride
E. Counterirritants
1. Volatile oils: Turpentine oil, Eucalyptus oil, Clove oil
2. Stearoptenes: Camphor, Thymol, Menthol
3. Other counterirritants: Mustard seeds (as mustard plaster), Capsicum, Cantharidin, Methyl salicylate, Alcohol
Antiseptics and Disinfectants
1. Phenol derivatives: Phenol, Cresol, Hexylresorcinol, Chloroxylenol, Hexachlorophene
2. Oxidizing agents: Potassium permanganate, Hydrogen peroxide, Benzoyl peroxide
3. Halogens: Iodine, Ionophores, Chlorine, Chlorophores

https://t.me/DentalBooksWorld
688 Triumph's Complete Review of Dentistry

4. Biguanide: Chlorhexidine
5. Quaternary ammonium (Cationic): Cetrimide, Benzalkonium chloride (Zephiran), Dequalinium chloride
6. Soaps: of Sodium and Potassium
7. Alcohols: Ethanol, Isopropanol
8. Aldehydes: Formaldehyde, Glutaraldehyde
9. Acids: Boric acid, Acetic acid
10. Metallic salts: Merbromin, Silver nitrate, Silver sulfadiazine, Mild silver protein, Zinc sulfate, Calamine, Zinc oxide
11. Dyes: Gentian violet, Brilliant green, Acriflavine, Proflavine
12. Furan derivative: Nitrofurazone

Vaccines
Bacterial vaccines Live attenuated
Killed (Inactivated)
Typhoid-paratyphoid (TAB) Bacillus Calmette-Guérin (BCG)
Vi Typhoid polysaccharide
Cholera Typhoid-Ty 21a
Whooping cough (Pertussis)
Meningococcal
Haemophilus influenzae type b
Plague

Viral vaccines
Killed (Inactivated) Live attenuated
Poliomyelitis inactivated (IPV, Salk) Poliomyelitis oral live
Rabies (Chick embryo cell, PCEV) (OPV, Sabin)
Rabies (Human diploid cell, HDCV) Mumps (live attenuated)
Rabies (Vero cell, PVRV) Measles (live attenuated)
Influenza Rubella (live attenuated)
Hepatitis B Varicella (live attenuated)
Hepatitis A
Toxoids
• Tetanus (fluid/adsorbed)
• Diphtheria (adsorbed)
Combined vaccines
• Double antigen (DT-DA)
• Triple antigen (DPT)
• Measles, mumps, rubella (MMR)

Anesthesia
Local anesthetics
• These drugs act by blocking the conduction of nerve impulse along the axon.
• Small diameter and myelinated fibers are blocked first whereas unmyelinated and thick fibers are blocked at last.
• Thus, the order of blockade of fibers is B, C, Aδ and then Aα, β, and γ. Autonomic fibers are blocked first, then sensory
(cold temperature sensation is lost first followed by heat, pain, and proprioception), and finally motor are blocked at last.
Order of recovery is in the reverse order.
Adrenaline is added to LA:
To make them long acting whereas sodium bicarbonate makes them fast acting

https://t.me/DentalBooksWorld
Chapter 8 • Pharmacology 689

Other effects
• Injection becomes more painful
• Increases chances of local tissue edema and necrosis
• Delays the wound healing
• More bloodless field for surgery
• Decreases toxicity

Some other points:


• Small diameter axons are more susceptible to block than large diameter fibers.
• Myelinated fibers are more sensitive than non-myelinated.
• Sequence of block is type B > type C > type A.
• In functional terms: Autonomic > sensory > motor.
• Among sensory fibers sequence of block is pain > temperature (cold before heat) > touch > deep pressure > proprioception.
General anesthesia:
Classification

General anesthesia
Inhalational Agents Intravenous Agents
Gases Liquids Inducing agents Slower-acting agents
Nitrous Chloroform Thiopentone Benzodiazepines Opioids Neuroleptic agents
oxide Trilene Propofol Diazepam Fentanyl Droperidol
Entonox cyclopropane Etomidate Lorazepam Remifentanil
Xenon Ether Ketamine Midazolam, Alfentanil Sufentanil
Halothane Methohexitone
Enflurane, desflurane,
sevoflurane,
isoflurane,
methoxyflurane

Anesthesiologist Contribution
Alfred Einhorn Procaine
Archie Brain LMA
August Bier Father of spinal anesthesia
Karl Koller First used cocaine as LA
Ivan Magill First endotracheal intubation
Melzack and Wall Gate way theory of pain

Color of cylinders Gases


Black body with white shoulder Oxygen
Blue Nitrous oxide
Grey Carbon dioxide
Orange Cyclopropane
Brown Brown
Blue body with white shoulder Entonox

https://t.me/DentalBooksWorld
690 Triumph's Complete Review of Dentistry

Pin indices of cylinders (denotes safety)

Air 1, 5
Oxygen 2, 5
Nitrous oxide 3, 5
Nitrogen 1, 4
Entonox 7

Blood–gas partition coefficient (BGPC)


It denotes speed of induction and recovery. The higher the value, the lower the speed of onset and recovery (inversely proportional)
Xenon 0.14
Desflurane 0.42
Cyclopropane 0.44
Nitrous oxide 0.47
Sevoflurane 0.69
Isoflurane 1.38
Enflurane 1.8
Halothane 2.4
Ether 12
Methoxyflurane 15

So the BGPC is highest for methoxyflurane which means it has slowest induction and recovery. (Note: Among the agents used
now halothane has slowest induction and recovery.) While xenon has the highest induction and recovery. (Note: Among the
agents used now Desflurane has the highest induction and recovery.)

Minimum Alveolar Concentration (MCA)


It denotes potency. It is inversely proportional to MAC. Highest the MAC, least the potency
Xenon 70%
Methoxyflurane 0.16%
Trilene 0.2%
Halothane 0.74%
Isoflurane 1.15%
Enflurane 1.68%
Ether 1.92%
Sevoflurane 2.05%
Desflurane 6.0%
Nitrous oxide 104%

So the MAC for methoxyflurane is lowest which means it has the highest potency. And Nitrous oxide has the least potency.
(Note: Among the agents used now halothane has the highest potency.)

Some Important Points Regarding Various Drugs of General Anesthesia


Stress response to laryngoscopy and intubation Alfentanil/Fentanyl
Muscle relaxant (MR) of choice in: Atracurium
• Hepatic failure
• Renal failure
• Myasthenia gravis
• Newborns
• Old age

https://t.me/DentalBooksWorld
Chapter 8 • Pharmacology 691

Inhalational agent in shock Cyclopropane


Induction in children Desflurane/Sevoflurane
Reversal agent in mivacurium Edrophonium
Cardio stable IV agent Etomidate
Inhalational agent used in asthmatics Halothane
IV agent for shock Ketamine
Patients with full stomach Ketamine
Muscle relaxant for day care surgery Mivacurium
LA of choice in malignant hyperthermia Procaine
Combination in day care surgery Propofol + Alfentanil
TIVA Propofol
Opioid for day care surgery Remifentanil
Non depolarizing agent for intubation Rocuronium
Most common used IV agent Thiopentone
Agent of choice for cerebral protection Thiopentone
MR for cardiac patients Vecuronium

Side Effects
Muscle rigidity Alfentanil
Convulsions Atracurium
Methemoglobinemia Prilocaine, Benzocaine
Malignant hyperthermia Succinyl choline, Lignocaine
Hyperglycemia Chloroform, Ether
Depression of respiration Enflurane
Nausea and vomiting Etomidate
Inhibit to ventilatory response Halothane
Megaloblastic anemia/Subacute degeneration of spinal cord Nitrous oxide
Cardiac arrhythmias Pancuronium

Autonomic Nervous System


Autonomic nervous system consists of three main divisions:
• Parasympathetic nervous system (or cholinergic)
• Sympathetic nervous system (or adrenergic)
• Enteric nervous system

Parasympathetic nervous system Sympathetic nervous system


• Acetylcholine is the neurotransmitter at all autonomic • Noradrenaline is the major neurotransmitter in the peripheral
ganglionic synapse sympathetic nervous system
• Hypothalamus is the major controlling center for the • Adrenaline is the primary hormone secreted by adrenal
parasympathetic system medulla
• Acetylcholine receptors are muscarinic and nicotinic • Dopamine is the metabolic precursor of NAdr and Adr and
receptors third neurotransmitter in sympathetic system
• The neurotransmitter at the sympathetic ganglia is
Acetylcholine

https://t.me/DentalBooksWorld
692 Triumph's Complete Review of Dentistry

Receptors
Parasympathetic Sympathetic
Muscarinic receptors Nicotinic receptors Adrenergic receptors
• The receptors are located at the • These receptors are located in • Two types of adrenergic receptors – α
parasympathetic neuroeffector neuromuscular junctions and at all and β
junction at all smooth muscles and autonomic ganglia • See below for tabulation
glands • They play a facilitatory role in the Adrenaline – alpha 1 + alpha 2 + beta 1 +
• They are primarily divided into release of other transmitters like beta 2 and weak beta 3 action
5 subtypes (M1, M2, and M3 are the Dopamine and Glutamate Noradrenaline – alpha 1 + alpha 2 + beta 1
most important) • They are classified as muscle type + beta 3 and no beta 2 action
M1 Receptors (neuronal and gastric) (NM), neuronal type (NN), and Isoprenaline – beta 1 + beta 2 + beta 3 and
Location: Ganglia (autonomic and central nicotinic receptors no alpha action
enteric), gastric, paracrine cells, CNS
(cortex and hippocampus)
Function: Gastric acid secretion, GI
motility, CNS excitation
M2 Receptors (Cardiac)
Location: SA node, AV node, atrium,
ventricle, presynaptic terminals
Function: SA node: ↓rate of impulse
generation; AV node: ↓velocity of
conduction, ↓contractility, vagal
bradycardia
M3 Receptors (Glandular)
Location: Exocrine glands, smooth
muscles, vascular endothelium
Function: ↑exocrine secretions,
smooth muscle contraction

Alpha actions Beta actions


Constriction of arterioles and veins – alpha 1 and alpha Dilatation of arteries and veins – fall in BP – beta 2 action
2 – increase in BP
Alpha 1 alone – arrhythmia at high dose Beta 1 – cardiac stimulation
Contraction of radial muscles of iris – mydriasis, alpha 1 Slight relaxation of ciliary muscle – enhanced aqueous secretion
– decrease aqueous secretion
Increase Ach release ADH secretion from posterior pituitary – beta 1
Alpha 2 inhibits insulin secretion Insulin/glucagon – secretion
Liver – glycogenolysis Liver/muscle – glycogenolysis, Fat – lipolysis
Salivary gland – K and water retention Ptyalin secretion

Golden Points for Quick Revision


• First-generation beta-blockers: Propranolol
• Beta-blockers with alpha-blocking property: Labetalol
• Second-generation beta-blockers: Betaxolol
• Third-generation blockers: Celiprolol
• Cardioselective beta-blockers: Betaxolol
• Beta-blockers with maximum plasma binding protein: Carvedilol
• Most cardioselective: Nebivolol
• Cardioselective beta-blocker which releases Nitric Oxide: Nebivolol

https://t.me/DentalBooksWorld
Chapter 8 • Pharmacology 693

• Cardioselective beta-blocker which has both intrinsic and membrane selective activity: Acebutolol
• Cardioselective beta-blocker without intrinsic stabilizing activity: Bisoprolol and Esmolol
• Beta-blocker which is a potassium channel opener: Tilisolol (third generation)
• Beta-blocker which is a calcium channel blocker: Carvedilol
• First-generation, nonselective, and beta-blockers without ISA (intrinsic stabilizing activity) are: propranolol, sotalol, timolol
• First generation, nonselective with ISA: Pindolol
• Third generation with vasodilating and alpha blocking property are: labetalol, carvedilol, medroxalol, bucindolol

NSAIDs
Nonsteroidal Anti-Inflammatory Drugs (NSAIDs): NSAIDs act by inhibiting cyclooxygenase (COX) enzyme and thus
prostaglandin synthesis. These drugs act as antipyretics, analgesics, and anti-inflammatory agents. Prostaglandins play a
protective role in the stomach and nonselective COX inhibitors can cause GI toxicity (peptic ulcer) on long-term use.
Classification
• Nonselective COX inhibitors (inhibit both COX-1 and COX-2)
• Preferential COX-2 inhibitors (inhibitory activity on COX-2 is greater than COX-1)
• Selective COX-2 inhibitors

Mechanism of action
Aspirin Anti-inflammatory, Antipyretic action – Nonselective, only irreversible inhibitor of
COX
Antiplatelet action – TAX2 inhibitor
Nimesulide, Meloxicam, Nabumetone, Preferential COX-2 inhibitors
Etodolac, and Diclofenac These drugs have more inhibitory action on COX-2 than COX-1
Paracetamol COX-3 inhibitors

Some important points in NSAIDs


Aspirin • Irreversible inhibitor of COX enzyme
• Low doses (40–325 mg), it acts as an antiplatelet drug, anti-inflammatory dose –
6 g/day, antiplatelet dose – 75 mg/day, antipyretic dose – 350 mg/day
• Antiplatelet action – It acts by inhibiting cyclooxygenase enzyme and thus
decreasing the synthesis of TXA2 (platelet aggregator)
• Aspirin is used to inhibit niacin induced flushing
• Adverse effects
–– Metabolic acidosis
–– Prolong bleeding time
–– Hyperuricemia
• Contraindication:
Contraindicated in children (<12 years old) due to increased risk of Reye’s
syndrome
Paracetamol (Acetaminophen) • Does not possess anti-inflammatory activity
• Selective COX-3 inhibition in the brain
• It produces very little GI toxicity and can be administered in patients intolerant to
other NSAIDs
• It is metabolized to N-acetyl para aminobenzo quinoneimine (NAPQ) by
microsomal enzymes
• N-acetylcysteine is an antidote of choice for paracetamol poising
• Doses up to 10 g is tolerated
• Glutathione produced by the liver combines with NAPQ to detoxify the effects of
acetaminophen (PGI June 2018).

https://t.me/DentalBooksWorld
694 Triumph's Complete Review of Dentistry

NSAIDs that are pro drugs • Sulindac and nabumetone


NSAID used for sports injury • Nimesulide
Longest-acting NSAIDs • Piroxicam and tenoxicam
NSAID that has chondroprotective • Acetaminophen
property
NSAIDs withdrawn due to increased • Rofecoxib and valdecoxib
risk of thrombotic disorders like
myocardial infarction
Indomethacin • Possesses immunosuppressive properties apart from its COX inhibitory action
• It causes more GI upset than other NSAIDs
• It causes headache
Propionic acid derivatives include • Ibuprofen, ketoprofen, and flurbiprofen
Dose of ibuprofen • 400 mg
Longest-acting coxib • Etoricoxib
Selective COX-2 inhibitors • Very little GI toxicity (NEET 2018)
• Celecoxib, rofecoxib, and valdecoxib are sulfonamide derivatives, thus can cause
rash and hypersensitivity
All NSAIDs are weak acids except • Nabumetone
Dose of Diclofenac • 50 mg
NSAIDs causing agranulocytosis • Phenylbutazone
Only NSAID that can be used IV (also • Ketorolac
available as eye drops)
Paracetamol (acetaminophen) • Has no anti-inflammatory activity
Nimesulide and nefopam • Do not act by decreasing PG synthesis
Drugs of choice for the treatment of • Proton pump inhibitors
peptic ulcer due to chronic NSAID use
PGI2 • Inhibits platelet aggregation
TXA2 • Potent aggregator of platelets
COX-1 • Constitutive (always present in cells)
COX-2 • Inducible (synthesis simulated by endotoxins and other inflammatory mediators)
• Participates in inflammation
COX-3 • Involved in pain perception and fever
• Not involved in inflammation
• Paracetamol targets COX-3 (NEET 2017)
Prodrug of valdecoxib • Parecoxib

ANTIMICROBIALS
An antimicrobial is any substance of natural, semisynthetic, or synthetic origin that kills or inhibits the growth of microorganisms
but causes little or no damage to the host. All antibiotics are antimicrobials, but not all antimicrobials are antibiotics.
Drugs and Mechanism of Action
Antimicrobials:
Bacteriostatic Bactericidal
Protein Drugs Drugs Protein Drugs Polypeptide Cell wall First line ATT
synthesis affecting affecting synthesis affecting antibiotics synthesis drugs (except
inhibitors DNA metabolism inhibitors DNA inhibitors Ethambutol)
• Tetracyclines • Nitro- • Sulfon­ • Aminoglyco- • Quinolones • Polymyxin B • Fosfomycin • Rifampicin
• Tigecycline furantoin amides sides • Metronida- • Colistin • Cycloserine • Isoniazid
• Chloramphen- • Novo­ • Dapsone • Strepto- zole • Ampho- • Bacitracin • Pyrazinamide
icol biocin • PAS gramins tericin B • Vancomycin • Streptomycin
• Macrolides • Trimetho­ • Penicillins (aminoglyco-
• Lincosamides prim • Cephalo- side)
• Linezolid • Ethambutol sporins

https://t.me/DentalBooksWorld
Chapter 8 • Pharmacology 695

Anthelmintics
• The helminthes (worms) – macroscopic, multicellular organisms having own digestive, excretory, reproductive, and nervous
system
• They can be round bodied (Nemathelminthes) or flat bodied (Platyhelminthes)

Nemathelminthes (Nematodes) Platyhelminthes (two types – flukes, tapeworms)


1.  Round worms (Ascaris lumbricoides) 1.  Trematodes (flukes)
2. Hook worms (Necator americanus or Ancylostoma a. Blood flukes (Schistosomiasis or bilharziasis)
duodenale) b. Liver flukes (Clonorchiasis)
3.  Whip worms (Trichuris trichiura) c. Intestinal flukes (Fasciolopsiasis)
4.  Thread worms (Strongyloides stercoralis) d. Lung flukes (Paragonimiasis)
5.  Pin worms (Enterobius vermicularis) 2.  Cestodes (tapeworms)
6. Filariasis (Wuchereria bancrofti or Brugia malayi) a. Beef tapeworm (Taenia solium)
7.  Onchocerciasis (Onchocerca volvulus) b. Fish tapeworm (Diphyllobothrium)
8.  Guinea worms (Dracunculus medinensis) c. Dwarf tapeworm (Hymenolepis nana)

Some important points on antimicrobial drugs


Beta-lactam antibiotics include • Penicillins
• Cephalosporins
• Cephamycins
• Carbapenems
Beta-lactam antibiotics • MOA: Inhibits bacterial cell wall synthesis
• Five different groups of antibiotics: Penicillins, Cephalosporins,
Monobactams, Carbapenems, Carbacephems
Penicillins • Penicillin was initially obtained from the fungus Penicillium notatum,
but the high-yielding source is Penicillium chrysogenum
• Penicillin nucleus consists of fused thiazolidine and beta-lactam rings
to which side chains are attached through an amide linkage
Penicillin G • Narrow spectrum, activity is limited to Gram-positive resistance:
many bacteria are inherently insensitive to PnG – because target
enzymes and PBPs are located deeper under the lipoprotein barrier
Extended spectrum penicillins Diarrhea is frequent after oral administration – it is incompletely
Ampicillin absorbed – the unabsorbed drug irritates the lower intestines as well as
Amoxicillin causes marked alterations of bacterial flora
Similar to Ampicillin except
Oral absorption is better, food does not interfere, higher and more
sustained blood levels are produced and less incidence of diarrhea
Cephalosporin
First Generation: Cephalexin
Developed in 1960 Orally effective first generation
High activity against Gram-positive and weaker Less bound to plasma proteins, attains high concentration in bile % is
against Gram-negative excreted unchanged in urine; t1/2 ~60 min
Cefazolin: Preferred parenteral first-generation One of the most commonly used Cephalosporins
drug for surgical prophylaxis
Second Generation: Cefuroxime
Developed after first generation It is resistant to Gram-negative β-lactamases
More active against Gram-negative organisms It is well tolerated by IM and attains relatively higher CSF levels
Important use is meningitis caused by H. influenzae, meningococci,
pnemococci, and for single dose IM therapy of gonorrhea

https://t.me/DentalBooksWorld
696 Triumph's Complete Review of Dentistry

Third Generation: Cefotaxime:


Cefotaxime, Ceftriaxone, Cefoperazone It is the prototype of third generation
Introduced in 1980 Exerts potent action on aerobic Gram-negative and Gram-positive and
They have high activity against Gram-negative not so active on anaerobes
enterobacteriaceae and Pseudomonas Ceftriaxone
All are highly resistant to β-lactamases from It has longer duration of action (t1/2-8 hours) – so once or mostly twice
Gram-negative bacteria daily dosing
They are less active on Gram-positive cocci Good CSF penetration and eliminated equally in urine and bile
Cefoperazone
It is having stronger activity on Pseudomonas and weaker activity on
others
Excreted in bile, t1/2 ~2 hours
It has hypoprothrombinemic action but does not affect platelet function
Fourth Generation Cefepime
Developed in 1990
Spectrum same as third generation but highly resistant to β-lactamases
Cefpirome
It is used for the treatment of serious and resistant hospital-acquired
infections
Imipenem It is extremely potent and broad spectrum β-lactam It is resistant to
most β-lactamases and inhibits penicillinase producing staphylococci
It is rapidly hydrolyzed by the enzyme dehydropeptidase-I
It is combined with Cilastatin which is a reversible inhibitor of
dehydropeptidase-I
Imipenem has propensity to induce seizures at higher doses and in
predisposed patients
Loracarbef Synthetic β-lactam antibiotic
Similar to Cefaclor
Dose: 200-4 mg BD
Activity and uses same as second-generation Cephalosporins
Aminoglycosides They are highly water soluble and stable for longer times
They are not absorbed orally and distribute only extracellularly – does
not cross BBB
The route of administration is IM or IV
Post-antibiotic effect Aminoglycosides continue to suppress the bacterial growth even when
their serum concentration falls below their MIC – it is due to the
disruption in 30S ribosomal function – bacteria needs time to for the
synthesis of new ribosomes
Isoniazid It acts on extracellular and on intracellular tubercle bacilli and is
equally active in acidic and alkaline medium.
Resistance: Mutation in katG gene
Rifampicin Semisynthetic derivative of Rifamycin
Macrocyclic antibiotic produced by Streptomyces species

DOTS (Directly Observed Treatment Short-course)


• Because patient’s poor compliance is the main reason for therapeutic failure
• Health care worker watches patient swallow each dose of medication
• Consider DOTS for all the patients
• DOTS should be used with all intermittent regimens

https://t.me/DentalBooksWorld
Chapter 8 • Pharmacology 697

• DOTS can lead to reductions in relapse and acquired drug resistance


• Use DOTS with other measures to promote adherence
• For short course: 6–8 months
• Supervision for at least 2 months
• DOTS require continuous examination and supply of essential drugs and Government’s monitoring system to check
whether the patients are cured

Drugs and other facts Active against


First-generation cephalosporin Gram-positive cocci
Second-generation cephalosporin Gram-negative species, less active against Gram-positive
Third-generation cephalosporin Gram-negative organisms that are resistant to other beta-
lactam antibiotics, Gram-positive organisms
Fourth-generation cephalosporin Gram-positive, Gram-negative pseudomonas. Not active
against anaerobes
E. fecalis No cephalosporin is active
Ceftazidime + aminoglycoside Pseudomonas
Fifth-generation cephalosporin MRSA
Only parenteral in first-generation cephalosporin Cefazolin
Cephalosporin causing bleeding Cefoperazone, Moxalactam, Cefamandole, and Ceftotetan
Third-generation cephalosporin penetrates BBB except Cefoperazone and Cefixime
Monobactams and Carbapenems Active against pseudomonas
Concentration-dependent killing (used as single large dose) Aminoglycoside and Fluoroquinolones
Time-dependent killing (used as multiple doses) Beta-lactam, macrolides, vancomycin
Pseudomembrane colitis
MC organism causing Clostridium difficile
MC drug causing Third-generation cephalosporin > Clindamycin > Ampicillin
Drug of choice > Tetracycline
Drug of choice in severe cases Metronidazole
Vancomycin
Drug of choice MRSA Vancomycin/Teicoplanin
Drug of choice VRSA Linezolid/Streptogramins
Chloramphenicol Active against anaerobes
Aminoglycosides Not active against anaerobes and does not cross BBB
Nephrotoxicity Neomycin > Gentamicin
Ototoxicity Amikacin (auditory)/Streptomycin (Vestibular)
NM blockade Neomycin > Streptomycin
Clindamycin is not active against Gram-negative aerobes
The following represents MIC susceptibility data for a few Staphylococcus aureus: 0.016 μg/mL: >256 μg/mL
medically significant pathogens Streptococcus pneumoniae: 0.002 μg/mL: >256 μg/mL
Streptococcus pyogenes: <0.015 μg/mL: >64 μg/mL

SYSTEMIC PHARMACOLOGY
Renal Drugs: Diuretics
• Diuretic drugs increase the urinary output of electrolytes and water from the kidney by interfering with one or more
reabsorptive processes occurring at different segments of the nephron.

https://t.me/DentalBooksWorld
698 Triumph's Complete Review of Dentistry

Some important points


Diuretics mainly exert their effect on the inhibition of renal tubular reabsorption of sodium and water. These may be classified
according to their efficacy as high-ceiling (loop and osmotic diuretics), medium-ceiling (thiazides), and low-ceiling (carbonic
anhydrase inhibitors and potassium sparing) diuretics.
Diuretics acting on the proximal tubule (PT)
• These are noncompetitive but reversible inhibitors of carbonic anhydrase and act by inhibiting the reabsorption of
sodium in the proximal tubular portion of the nephron.
• Due to urinary excretion of bicarbonate, metabolic acidosis (and urinary alkalosis) ensues that result in reduced filtration
of HCO3– at the glomerulus, action of these diuretics is self-limiting.
• Acetazolamide, dorzolamide, and brinzolamide are noncompetitive but reversible inhibitors of carbonic anhydrase.
• Action of carbonic anhydrase inhibitors is self-limiting.
• Dorzolamide and brinzolamide are topically acting CA inhibitors for use in glaucoma as eye drops.
• Acetazolamide is a sulfonamide derivative and can result in bone marrow suppression and hypersensitivity reactions; other
adverse reactions include metabolic acidosis (urinary alkalosis) and hypokalemia.
Diuretics acting on the loop of Henle
INHIBITORS OF NA+–K+– ATPASE   These are also known as diuretics and act by causing inhibition of Na+k+ 2C1–
symporter present at the luminal membrane of the ascending limb of loop of Henle.
• Furosemide possesses vasodilation which is responsible for the quick relief in LVF and pulmonary edema (used IV)
• Bumetanide is the most potent loop diuretic (Ref. Goodman and Gilman, page no. 750) and produces less adverse effects
than furosemide
• Ethacrynic acid is highly ototoxic with steep DRC
• Mersal like organomercurials are used not due to the risk of kidney damage
• Torsemide has the longest half-life (Ref. Goodman and Gilman, page no. 750)
• At equally natriuretic doses, K+ excretion is maximum with carbonic anhydrase inhibitors
• Carbonic anhydrase inhibitors should not be used in the presence of liver disease due to the risk of precipitation of
hepatic coma
Uses of carbonic anhydrase inhibitors
– Glaucoma angle closure
– Alkalinization of urine
– Mountain sickness
– Epilepsy
• Loop diuretics decrease positive as well as negative free water clearance whereas thiazides decrease positive free water
clearance only
• Loop diuretics are the diuretics of choice in the presence of moderate to severe renal failure
• Adverse effects: Hypokalemia, hypomagnesemia, hyponatremia, alkalosis, hyperglycemia (C/I in DM), hyperuricemia
(C/I in gout), and dyslipidemia are seen with both thiazides as well as loop diuretics. Effect on Ca++ excretion is opposite
to thiazides (loop loses calcium)

OSMOTIC DIURETICS
Mannitol, glycerol, urea, and isosorbide are inert drugs that can cause osmotic diuresis
Along with water, excretion of all the cations and anions is increased. Properties for a substance to act as an ideal osmotic
diuretic are:
• It should exert osmotic effect
• It should be pharmacologically inert
• It should be freely filtered at the glomerulus
• It should not be reabsorbed
Mannitol is a low-molecular-weight compound possessing all these properties; it is used IV for the treatment of glaucoma and
cerebral edema. It can also be used to maintain GFR in the impending renal failure. It is contraindicated in acute renal failure
because ECF volume increases but it cannot be filtered; it is also contraindicated in cerebral hemorrhage (active bleeding).

https://t.me/DentalBooksWorld
Chapter 8 • Pharmacology 699

Diuretics Acting on the Distal Tubules and Collecting Ducts

THIAZIDES
• These drugs act by inhibiting Na+– C1– symporter at the luminal membrane of early DT
• Furosemide possesses vasodilatory action which is responsible for the quick relief in LVF and pulmonary edema
• Bumetanide is the most potent loop diuretic whereas Torsemide has the longest half life
• Ethacrynic acid is highly ototoxic diuretic
• Properties of an ideal osmotic diuretic are:
– Exerts osmotic effect
– Is pharmacologically inert
– Is freely filtered at the glomerulus
– Is not reabsorbed
• Mannitol can be used to maintain GFR in the impending renal failure
• Chlorothiazide has minimum potency and efficacy whereas other drugs differ only in potency (efficacy is similar)
• These drugs tend to reduce GFR; therefore, are not indicated in renal failure patients
– Polythiazide and trichlormethiazide are most potent thiazides
– Chlorthalidone is the longest-acting thiazide
– Metolazone is useful even in severe renal failure
– Indapamide has no CA inhibitory action; it has vasodilatory property because of which, its antihypertensive effect
precedes the natriuretic effect
• Thiazides are used as first-line antihypertensive drugs

POTASSIUM-SPARING DIURETICS
These diuretics act in the late DT and CD cells to preserve K+. Luminal membrane of these portions of renal tubule contains
epithelial Na+ channels responsible for reabsorption of Na+
1. Epithelial Na+ channel inhibitors
• Important members of this group are amiloride and triamterene
• Thiazides are used for the treatment of patients with recurrent Ca++ stones in the kidney
• Amiloride is the drug of choice for lithium-induced diabetes insipidus
• Amiloride is more potent and longer acting than triamterene
– Triamterene is less often used because of incomplete absorption, photosensitivity, and impairment of glucose tolerance;
it is also associated with interstitial nephritis and renal stones
2. Aldosterone antagonists: Spironolactone, canrenone, potassium canrenoate, and eplerenone antagonize the action of
aldosterone and produce effects similar to amiloride. These drugs act from the interstitial site of the tubular cell (all other
diuretics act from luminal side)
• Aldosterone antagonists act from the interstitial site of tubular cell whereas all other diuretics act from luminal side
• ADH increases the permeability of collecting ducts by its action on V2 receptors

Some Important Drugs Used in Cardiovascular System


Antiarrhythmic drugs
The five main classes in the Singh–Vaughan Williams classification of antiarrhythmic agents are:
• Class I agents interfere with the sodium (Na+) channel.
• Class II agents are anti-sympathetic nervous system agents. Most agents in this class are beta-blockers.
• Class III agents affect potassium (K+) efflux.
• Class IV agents affect calcium channels and the AV node.
• Class V agents work by other or unknown mechanisms.

https://t.me/DentalBooksWorld
700 Triumph's Complete Review of Dentistry

Class Known as Examples Mechanism Medical uses


Ia Fast- • Quinidine (Na+) channel block (intermediate • Ventricular arrhythmias
channel • Ajmaline association/dissociation) and K+ • Prevention of paroxysmal recurrent atrial
blockers • Procainamide channel blocking effect; affects fibrillation (triggered by vagal overactivity)
QRS complex • Procainamide in Wolff–Parkinson–White
• Disopyramide
syndrome
• Increases QT interval
Ib • Lidocaine Na+ channel block (fast • Treatment and prevention during and
• Phenytoin association/dissociation); can immediately after myocardial infarction,
• Mexiletine prolong QRS complex in overdose though this practice is now discouraged
given the increased risk of asystole
• Tocainide
• Ventricular tachycardia
Ic • Encainide Na+ channel block (slow • Prevents paroxysmal atrial fibrillation
• Flecainide association/dissociation) • Treats recurrent tachyarrhythmias of
• Propafenone abnormal conduction system
• Moricizine • Contraindicated immediately after
myocardial infarction
II Beta- • Carvedilol Beta blocking • Decrease myocardial infarction mortality
blockers • Propranolol Propranolol also shows some • Prevent recurrence of tachyarrhythmias
• Esmolol class I action • Propranolol has sodium channel-blocking
• Timolol effects
• Metoprolol
• Atenolol
• Bisoprolol
• Nebivolol
III • Amiodarone K+ channel blocker • In Wolff–Parkinson–White syndrome
• Sotalol Sotalol is also a beta-blocker • (Sotalol): ventricular tachycardias and atrial
• Ibutilide Amiodarone has Class I, II, III, fibrillation
• Dofetilide and IV activity • (Ibutilide): atrial flutter and atrial
• Dronedarone fibrillation
• E-4031 • (Amiodarone): hemodynamically stable
ventricular tachycardia
IV Slow- • Verapamil Ca2+ channel blocker • Prevent recurrence of paroxysmal
channel • Diltiazem supraventricular tachycardia
blockers • Reduce ventricular rate in patients with
atrial fibrillation
V • Adenosine Work by other or unknown Used in supraventricular arrhythmias,
• Digoxin mechanisms (direct nodal especially in heart failure with atrial
• Magnesium inhibition) fibrillation, contraindicated in ventricular
Sulfate arrhythmias, or in the case of magnesium
sulfate, used in torsades de pointes.

Antihypertensive Drugs:
ACE Inhibitors
• Captopril, Enalapril, Lisinopril, Benazepril, Ramipril, Perindopril, Quinapril, Cilazapril, Zofenopril, Fosinopril
• The first ACEI – Teprotide derived from Pit Viper Venom
Mechanism of Action
• ACE inhibitors inhibit the angiotensin-converting enzyme which helps in the formation of Angiotensin-II and III – the
actions of angiotensin-II and III are inhibited.

https://t.me/DentalBooksWorld
Chapter 8 • Pharmacology 701

• CVS – Vasodilatation and suppression of central sympathetic activity and also suppression of release and synthesis of
adrenaline and noradrenaline – decreases myocardial contraction by inhibiting Ca+2 influx, decreases heart rate by
suppressed sympathetic activity, increases cardiac output, decreases cardiac work
• Adrenal cortex: By inhibiting the ACE – stimulation of Angiotensin-II and Angiotensin-III are inhibited – decreases
synthesis and release of aldosterone
• Due to inhibition of Aldosterone – Na+2 and water reabsorption decreases – decrease in plasma volume
• By inhibiting the stimulation of Angiotensin-II and Angiotensin-III – it increases renal blood flow and result in Na+ and
water excretion
Captopril
• It is a sulfhydryl containing dipeptide surrogate of proline, which abolishes the pressor actions of Angiotensin-I
• It increases plasma kinin levels and potentiate the hypotensive action of exogenously administered bradykinin
• Elevated kinins are responsible for cough and angioedema induced by ACE inhibitors
Adverse Effects
• Hypotension: Decreases BP in diuretic-treated and CHF patients
• Hyperkalemia: Risk in patients with impaired renal function and in those taking K+ sparing diuretics, NSAIDs, or β-blockers
• Cough: Occurs within 1–8 weeks and subsides within 4–6 weeks after discontinuation
• Rashes, Urticaria
• Angioedema: Swelling of lips, mouth, nose, larynx, airway obstruction
• Dysgeusia: Loss or alteration of taste
• Fetopathic: Fetal growth retardation, hypoplasia of organs and fetal death – ACE inhibitors prescribed in later half of
pregnancy
• Headache, dizziness, nausea, bowel upset
• Granulocytopenia and proteinuria
• Acute renal failure
Enalapril
Advantages over Captopril (NEET 2017)
• More potent, effective dose 5–20 mg OD or BD
• Its absorption is not affected by food
• Onset of action is slower (due to need for conversion to active metabolites), less liable to cause abrupt first-dose hypotension
• Longer duration of action – can be used in all types
• Rashes and dysgeusia is rare

Some Important Points on Respiratory Drugs


Bronchodilators
• They relieve asthmatic symptoms and improve pulmonary functions by relaxing the bronchial smooth muscle – they
provide a rapid symptomatic relief but do not control the disease process – they are termed relievers
• These include nonselective sympathomimetics, selective β2 agonists, methylxanthines, and antimuscarinic drugs
Selective β2–Agonists
• Salbutamol (Albuterol), Terbutaline, Rimiterol, Fenoterol, Bitolterol, Pirbuterol – short acting
• Salmeterol, Formoterol, Bambuterol – highly lipid soluble (it is due to the binding of the drug with an exoreceptor in the
vicinity of β2 adrenoceptor on airway smooth muscle) – longer acting (half-life is 12 hours)
• They activate β2 adrenoreceptors present on airway muscle – enhance the release of cAMP by activating adenylyl cyclase
enzyme
Nonselective Sympathomimetics
• Epinephrine, Ephedrine, Isoproterenol, Orciprenaline
• Isoproterenol is a nonselective – but it is largely superseded by the more selective β2 agonists
• It is not used because besides usual side effects (tremors and tachycardia), it disturbs the ventilation:perfusion ratio of
lungs leading to hypoxemia and respiratory acidosis

https://t.me/DentalBooksWorld
702 Triumph's Complete Review of Dentistry

• Anticholinergics
• Aerosol preparations of Ipratropium, Oxitropium, and Tiotropium are used – these are less effective bronchodilators then
β2 agonists
• They cause bronchodilation by binding to M3 receptors on airway smooth muscle – block M3 receptors – decrease the
cGMP levels in bronchial muscle – prevents the action of acetylcholine release from parasympathetic nerves
• They do not cross BBB – no CNS side effects (NEET 2017)
Drugs used in Treatment of Status Asthmaticus
• It is a potentially life-threatening acute attack of severe asthma needing immediate treatment
• A high concentration (40–60%) of O2 is administered with high flow rate along with high doses of inhaled short acting
β2 agonists
• A high dose of systemic corticosteroids are given like Prednisolone 30–60 mg orally or Hydrocortisone 200 mL IV
• If situation is life threatening, Ipratropium 0.5 mg can also be added through inhalation

Some Important Points Regarding Coagulants – Anticoagulants


Heparin
• It is a mixture of sulfated mucopolysaccharides with a molecular weight ranging from 10,000 to 40,000
• It is a strong electronegatively charged acidic polymer – it has a number of anionic SO4− and COO− acidic groups in it
• It is present together with histamine in all tissues containing mast cells
• Richest source – Lung, liver, intestinal mucosa
• It is obtained from beef lung and pig intestinal mucosa
MOA of Heparin
• Antithrombin-III – blocks the activity of activated clotting factors XII, XI, X, IX, and II – this action is very slow – action
of Antithrombin-III is accelerated by Heparin against IIa, Xa
LMW Heparins
• Unfractionated heparin (5,000–40,000) on fractionation heparin and LMWH
• LMWHs have shorter polymer length – lesser effect on thrombin, platelet function, on coagulation
• Thrombocytopenia is less frequent, chances of hemorrhage are less
Warfarin
• Commercial Warfarin is a racemic mixture of R(+) and S(−) warfarin
• S(−) Warfarin is four times more potent then R(+)
• Oral bioavailability is 100%, plasma protein bound 99% – Low Vd (7.7 L), Longer half-life 36 hours, and several drug
displacement reactions

Differences between Heparin and Warfarin (Dicumarol)


Heparin Warfarin
• It is a parenteral anticoagulant • It is an oral anticoagulant
• Anticoagulation action is by accelerating AT-III and • Anticoagulant action is by inhibiting the Vit K
inactivating factors IIa and Xa reductase which indirectly inhibits the synthesis of
• Duration of action is shorter coagulant factors
• Continuous monitoring of coagulant parameters in blood • Onset of action is slower and duration of action is
after administration longer
• It is safer in pregnancy • It is not safer in pregnancy – it causes skeletal
• But it causes thrombocytopenia, hemorrhage abnormalities in fetus
• Heparin antagonist is Protamine Sulfate • It also causes necrosis of soft tissues
• They are given initially before the warfarin administration • Warfarin antagonist is Vit K1-Phytonadione
• Heparin should be given before the warfarin
administration to avoid necrosis of soft tissues

https://t.me/DentalBooksWorld
Chapter 8 • Pharmacology 703

QUICK FACTS

MECHANISM OF ACTION OF VARIOUS DRUGS


Inhibit cell wall synthesis • Penicillins
• Cephalosporins
• Glycopeptides
Inhibit protein synthesis • Aminoglycosides
• Macrolides
• Chloramphenicol
• Tetracyclines
Inhibit nucleic acid synthesis • Quinolones
• Sulfonamides
• Rifampicin
• Trimethoprim
• Metronidazole
Aminoglycoside • 30S ribosomal units of the bacteria – prevent the formation of “initiation complex” –
which is the prerequisite for peptide synthesis
Sulfonamides • Synthesis of active form of folic acid: Dihydropteridine + PABA – in presence of
dihydropteroic acid synthetase – dihydropteroic acid – in presence of glutamate –
dihydrofolic acid – in presence of dihydrofolate reductase – tetrahydrofolic acid – useful
for the synthesis of RNA, DNA, proteins, nucleic bases, amino acids
• Excess amount of PABA can antagonize the antimicrobial action of sulfonamides – local
anesthetic like procaine, which is PABA ester can antagonize the effects of sulfonamides
Isoniazid • Inhibits the synthesis of mycolic acid (essential components of M. cell wall)
• Mycolic acid is a waxy lipid content of cell wall – INH reduces the lipid content of cell wall
• Suppress the formation of DNA and RNA
Rifampicin • Binds strongly to the beta subunits of bacterial “DNA-dependent RNA polymerase” –
inhibits RNA synthesis of bacteria
Heparin • Antithrombin-III – blocks the activity of activated clotting factors XII, XI, X, IX, and II – this
action is very slow – action of antithrombin-III is accelerated by Heparin against IIa, Xa
Warfarin • They act by competitively inhibiting Vit K reductase – inhibit the synthesis of clotting
factors II, VII, IX, X by the liver
• The reduced form of Vit K (KH) – serves as cofactor in the γ-carboxylation of clotting
factors
• In this carboxylation, Vit K (KH) is oxidized to its epoxide (KO) – which is reduced
enzymatically by Vit-K reductase – all these drugs inhibit this reduction

ADVERSE EFFECTS OF VARIOUS DRUGS


Gingival hyperplasia Phenytoin, Lamotrigine, Calcium antagonists, Cyclosporine, Sirolimus
Pulmonary fibrosis Bleomycin, Mitomycin, Amiodarone, Busulfan, Chlorambucil, Cyclophosphamide,
Methysergide, Vinblastine
Altered taste ACE inhibitors, Acetazolamide, Biguanides, Griseofulvin, Lithium, Metronidazole, Rifampicin
Metallic taste Metronidazole, Acetazolamide, Disulfiram, Auranofin, Vincristine
Seizures INH, Nalidixic acid, Amphetamines, Imipenem, Local anesthetics, Pethidine, Penicillins,
Phenothiazines, TCA, Vincristine, Bupropion, Clozapine, Physostigmine (IV), Quinolones,
Theophylline

https://t.me/DentalBooksWorld
704 Triumph's Complete Review of Dentistry

Hypotension Theophylline, Adenosine, Morphine, Quinidine, Fosphenytoin (IV), Amiodarone, IL-2


Levodopa, Alpha-blockers, Guanethidine, Bretylium, β-blockers (IV), Glyceryl trinitrate,
Chlorpromazine, Diuretics, Clonidine, Calcium channel blockers
First dose phenomenon Prazosin, Muromonab CD3, Sargramostim, ACE inhibitors esp. captopril
Hyperglycemia Thiazides, Furosemide, Glucocorticoids, Oral contraceptives, Diazoxide, L-asparaginase,
Glucagon, Cyclosporine, Phenytoin, Propranolol, Tacrolimus, Protease inhibitors, Niacin,
Encainide, Pentamidine (late in therapy)
Hypoglycemia Oral hypoglycemics, Quinine, Insulin β-blockers, Ethanol, Octreotide, Salicylates (late in over
dose), Pentamidine (early in therapy)
Hyperkalemia NSAIDs, SCh, ACE inhibitors, Potassium sparing diuretics (spironolactone, amiloride, and
triamterene), Salt substitutes, ARBs (Losartan), Lithium, Pentamidine, Digoxin overdose,
Cyclosporine, Heparin β-blockers (initially), Cytotoxics, Trimethoprim
Hypokalemia Thiazides, Furosemide, Carbenoxolone, Lithium, Corticosteroids, Amphotericin B, Gentamicin,
Insulin, Mannitol, Theophylline, Carbonic anhydrase inhibitors
Prolonged QTc interval Terfenadine, Astemizole, Cisapride, Sparfloxacin, Gatifloxacin, Grepafloxacin, Amiodarone,
Bretylium, Disopyramide, Procainamide, Quinidine, Sotalol, Mefloquine, Pentamidine,
Thioridazine, Ziprasidone
SLE-like syndrome Hydralazine, Acebutolol, Asparaginase, Barbiturates, Bleomycin, Cephalosporins, Iodides,
Sulfonamides, Thiouracil, Methyldopa, Phenytoin, INH, Quinidine, Procainamide
Aplastic anemia Chloramphenicol, Phenytoin, Gold Salts, Carbamazepine, Phenylbutazone, Sulfonamides,
Zidovudine, Colchicine, Carbimazole, Quinacrine, Felbamate, Cytotoxics, Thioamides,
Trimethadione
Megaloblastic anemia Pentamidine, Methotrexate, Trimethoprim, Co-trimoxazole, N2O, Oral contraceptives,
Metformin, Primidone, Phenobarbitone, Phenytoin, Triamterene
Ototoxicity (Auditory) NSAIDs, Vancomycin, Ethacrynic acid, Aminoglycosides
Ototoxicity (Vestibular) Aminoglycosides, Mustine, Quinidine, Quinine, Chloroquine, Vancomycin, Furosemide,
Ethacrynic acid, Salicylates (high dose)
Hypothyroidism Lithium, Iodides, Sulfonamides, Amiodarone, Phenylbutazone, Carbimazole, Acetazolamide,
Phenytoin

ANTIBIOTIC DOSAGES
Antibiotic Dosages of Oral Antibiotics for Neonates
Antibiotic Daily dosage
Amoxicillin 20–40 mg/kg div. 8 hourly
Ampicillin 50–100 mg/kg div. 8 hourly
Cephalexin 50 mg/kg div. 6 hourly
Chloramphenicol <14 days: 25 mg/kg div. 8 hourly
>14 days: 50 mg/kg div. 6 hourly
Clindamycin  20 mg/kg div. 6 hourly
Cloxacillin  >2.5 kg: 50–100 mg/kg div. 6 hourly
<2.5 kg: 50 mg/kg div. 8 hourly
Erythromycin <7 days: 20 mg/kg div. 12 hourly
>7 days: 20–40 mg/kg div. 6 hourly
Metronidazole 25 mg/kg div. 12 hourly
Penicillin V 50,000 U/kg div. 8 hourly

https://t.me/DentalBooksWorld
Chapter 8 • Pharmacology 705

Parenteral Antibiotic Dosages for Serious Infections in Infants and Children


Antibiotic Daily dosage
Aminoglycosides 
Amikacin 22 mg/kg div. 8 hourly
Gentamicin 7.5 mg/kg div. 8 hourly
Kanamycin 30 mg/kg div. 8 hourly
Netilmicin 7.5 mg/kg div. 8 hourly
Streptomycin 20 mg/kg div. 12 hourly
Tobramycin 5 mg/kg div. 8 hourly
Cephalosporins
Cefoperazone >12 years: 150 mg/kg div. 8 hourly
Cefotaxime 200 mg/kg div. 6 hourly
Ceftazidime 150 mg/kg div. 8 hourly
Ceftriaxone 100 mg/kg once daily
Chloramphenicol 100 mg/kg div. 6 hourly
Clindamycin  40 mg/kg div. 6 hourly
Erythromycin 40 mg/kg div. 6 hourly
Imipenem 40–60 mg/kg div. 6 hourly
Metronidazole 30 mg/kg div. 6 hourly
Penicillins
Penicillin G 400,000 U/kg div. 6 hourly
Benzathine penicillin 50,000 U/kg single dose IM
Procaine penicillin 50,000 U/kg div. 12 hourly IM
Ampicillin 200 mg/kg div. 6 hourly
Cloxacillin 200 mg/kg div. 6 hourly
Piperacillin 200–300 mg/kg div. 6 hourly
Rifampicin 10–20 mg/kg div. 12 hourly
Trimethoprim-sulfamethoxazole (Co-trimoxazole) 20 mg TMP/100 mg SMX/kg div. 6 hourly
Vancomycin 40 mg/kg div. 6 hourly

Antibiotic Dosages of Oral Antibiotics for Infants and Children

Antibiotic Daily dosage


Azithromycin 10 mg/kg daily
Cephalosporins
Cefuroxime 30 mg/kg div. 12 hourly
Cephalexin 25–50 mg/kg div. 6 hourly
Cefaclor 20–50 mg/kg div. 8 hourly
Cefadroxil 30 mg/kg div. 12 hourly
Cephradine 25–50 mg/kg div. 12 hourly
Chloramphenicol 50–100 mg/kg div. 6 hourly
Clindamycin  25 mg/kg div. 6 hourly
Macrolides
Clarithromycin 15 mg/kg div. 12 hourly
Erythromycin 25–50 mg/kg div. 6 hourly

https://t.me/DentalBooksWorld
706 Triumph's Complete Review of Dentistry

Metronidazole 25 mg/kg div. 6 hourly


Nalidixic acid 50 mg/kg div. 6 hourly
Nitrofurantoin 7 mg/kg div. 6 hourly
2 mg/kg single dose daily (prophylaxis)
Penicillins
Penicillin V <10 kg: 125 mg 8 hourly; >10 kg: 250 mg 8 hourly
Amoxicillin 20–40 mg/kg div. 8 hourly
Ampicillin 50–100 mg/kg div. 6 hourly
Cloxacillin 50–100 mg/kg div. 6 hourly
Amoxicillin–clavulanate 20–40 mg/kg div. 8 hourly
Sultamicillin 25–50 mg/kg div. 12 hourly
Rifampicin 20 mg/kg div. 12 hourly
Trimethoprim–sulfamethoxazole (Co-trimoxazole) 6–20 mg TMP/30–100 mg SMX/kg div. 12 hourly

DRUG OF CHOICE
1. Hypertensive emergencies in pregnancy 1. Labetalol
2. Hypertensive emergencies 2. Nicardipine + labetalol
3. Iron toxicity 3. Desferrioxamine
4. Kala azar 4. Liposomal amphotericin B
5. Malaria (P. Vivax) 5. Chloroquine
6. Malaria (P. falciparum) 6. Artemisinin combination therapy
7. Malignant hyperthermia 7. Dantrolene
8. Methanol poisoning 8. Fomepizole
9. MRSA 9. Vancomycin
10. Multiple myeloma 10. Dexamethasone + Lenalidomide, and/or Bortezomib
11. NSAID-induced PUD 11. Proton pump inhibitors
12. Syphilis 12. Syphilis
– Primary – Benzathine Penicillin G
– Secondary – Benzathine Penicillin G
– Latent – Benzathine Penicillin G
– Tertiary (except neurosyphilis) – Benzathine Penicillin G
– Neurosyphilis – Penicillin G (Aqueous)
– In pregnancy – Penicillin G (Aqueous)
13. Partial seizures (Temporal lobe epilepsy) 13. Carbamazepine
14. Peptic ulcer disease 14. PPIs
15. Petit mal epilepsy in >3 years 15. Valproate
16. Petit mal epilepsy in children 16. Ethosuximide
17. VRSA 17. Linezolid/Streptogramins
18. Warfarin overdose 18. Vitamin K1
19. Wegener’s granulomatosis (now known as agranulomatosis 19. Cyclophosphamide
with polyangiitis)

https://t.me/DentalBooksWorld
Chapter 8 • Pharmacology 707

MULTIPLE CHOICE QUESTIONS

HISTORY OF PHARMACOLOGY
1. Who is the father of Pharmacology?
2. Who is the father of Indian Pharmacology?
3. Who is the father of modern Pharmacology?
4. Who discovered insulin in 1921?
5. Who worked out the chemical structure of insulin in 1956?
6. Who coined the term balanced anesthesia?
7. Who used N2O (laughing gas) in 1844 for dental anesthesia?
8. Who described the four stages of anesthesia with Ether?
9. Who found the classification of anti-arrhythmic drugs?
10. Who
• Coined the term chemotherapy?
• Used the idea that if certain dyes can selectively stain microbes, they can also be toxic to these microbes?
• Developed arsenic compounds (Salvarsan) for treatment of syphilis?
11. Who discovered penicillin?
12. Who discovered streptomycin?
13. Name the first local anesthetic?
14. Name the first IV anesthetic?
15. Name the first drug for Schizophrenia?
16. Name the first ACE inhibitor
17. Name the first oral ACE inhibitor
18. Name the first fibrinolytic
19. Name the first antibiotic
20. Name the first antitubercular drug

PHARMACOLOGICAL AGENTS AND AUTHORS


21. Morphine
22. Atropine
23. Histamine
24. Sulfanilamide
25. Oxytocin
26. Insulin
27. Penicillins
28. Sulfonamides
29. Cortisone
30. Streptomycin
31. Chloramphenicol
32. Tetracycline
33. Lithium

https://t.me/DentalBooksWorld
708 Triumph's Complete Review of Dentistry

GENERAL PHARMACOLOGY
1. All are false about route of drug administration, except
A. 80% bioavailability by IV injection
B. IM administration needs sterile technique
C. ID injection produces local tissue necrosis and irritation
D. Inhalation produces delayed systemic bioavailability
2. Volume of distribution of drugs is altered in which of the following conditions?
A. Obesity B. Athletes
C. Pregnancy D. Old age
E. Neonate
3. Which of the following are prodrugs?
A. Mercaptopurine B. Dipivefrine
C. Enalapril D. Phenytoin
E. Linezolid
4. First pass metabolism is high in
A. Lignocaine B. Propranolol
C. Salbutamol D. Dipyridamole
E. Erythromycin
5. Identify the phase II reactions
A. Dealkylation B. Sulfation
C. Methylation D. Glucuronidation
E. Deamination
6. What is the effect of a drug that is high plasma protein binding?
A. Short duration of action B. Less drug interactions
C. Lower volumes of distribution D. All of the above
7. All of the following induces microsomal enzymes except
A. Cimetidine B. Griseofulvin
C. Rifampicin D. Phenobarbitone
8. All of the following are not prodrugs except
A. Ampicillin B. Captopril
C. Levodopa D. Phenytoin
9. All of the following drugs have active metabolite except
A. Diazepam B. Propranolol
C. Allopurinol D. Lisinopril
10. Which one of the following drugs does not undergo hepatic first pass effect?
A. Propranolol B. Lidocaine
C. Insulin D. Morphine
11. Which of the following is a prodrug?
A. Captopril B. Cimetidine
C. Carbimazole D. Carbamazepine
12. High hepatic first pass metabolism is seen in all except
A. Insulin B. Propranolol
C. Lignocaine D. Nitroglycerine
13. Apparent volume of distribution (Vd) of a drug exceeds total body fluid volume, if a drug is
A. Sequestrated in body tissues B. Slowly eliminated from body
C. Poorly soluble in plasma D. Highly bound to plasma proteins
14. Which of the following drug acts as microsomal enzyme inhibitor?
A. Rifampicin B. Cimetidine
C. Phenobarbitone D. Phenytoin

https://t.me/DentalBooksWorld
Chapter 8 • Pharmacology 709

15. Which of the following is an inducer of microsomal enzymes?


A. Phenobarbitone B. Paracetamol
C. Digoxin D. Penicillin
16. Removal of acidic drugs from body is done by using
A. Ammonium chloride B. Sodium bicarbonate
C. Hydrochloric acid D. Citric acid
17. Very high first pass metabolism is seen in
A. Digoxin B. Dicoumarol
C. Propranolol D. Practolol
18. What is pharmacokinetics??
A. Study of absorption, distribution, binding storage/biotransformation, and excretion of the drug
B. Study of physiological and biochemical effects of drugs
C. Application of pharmacological information together with knowledge of the disease
D. Scientific study of drugs in humans
19. Which one of the following is a prodrug?
A. Dopamine B. Epinephrine
C. Levodopa D. Prednisolone
20. Which one of the following is a prodrug?
A. Dopamine B. Enalapril
C. Ampicillin D. Prednisolone
21. In which drug is redistribution phenomenon seen?
A. Halothane B. Ether
C. Thiopentone D. All
22. Sulfonamide is conjugated with
A. Acetylation B. Methylation
C. Hydroxylation D. None
23. Which of the following statements is correct?
A. Most drugs are absorbed in ionized form
B. Basic drugs are generally bound to plasma albumin
C. Microsomal enzymes are located in the mitochondria of hepatic cells
D. Blood brain barrier is deficient at the chemoreceptor trigger zone
24. Nonsynthetic phase I reaction for drug detoxification is
A. Glucuronidation B. Acetylation
C. Methylation D. Oxidation
25. Which of the following is not a prodrug?
A. Enalapril B. Imipramine
C. Sulfasalazine D. Cyclophosphamide
26. Why is loading dose of a drug given?
A. To achieve steady state concentration in short time B. For drugs with short t½
C. To reduce complications D. All of these
27. Time for peak plasma concentration (T max) indicates
A. The rate of elimination B. The rate of absorption
C. The duration of effect D. The intensity of effect
28. One of the potential microsomal enzymes inhibitor drug is
A. Phenobarbitone B. Griseofulvin
C. Sodium valproate D. Phenytoin
29. Least oral bioavailability is seen in
A. d-Tubocurarine B. Morphine
C. Ampicillin D. Phenytoin

https://t.me/DentalBooksWorld
710 Triumph's Complete Review of Dentistry

30. One of the potent microsomal enzyme inducer drug is


A. Captopril B. Erythromycin
C. Rifampicin
31. Therapeutic index is a measure of
A. Safety B. Potency
C. Efficacy D. Selectivity
32. Which drug does not require therapeutic drug monitoring?
A. Phenytoin B. Metformin
C. Tacrolimus D. Cyclosporin
33. Drugs having narrow therapeutic index are
A. Lithium B. Erythromycin
C. Phenytoin D. Propranolol
E. Tricyclic antidepressants
34. Therapeutic drug monitoring is required for
A. Prodrugs B. Levodopa
C. Lithium carbonate D. MAO inhibitors Cimetidine
35. Phase IV of clinical trials collect information specially about
A. Drug efficacy B. Drug potency
C. Drug toxicity D. Pharmacokinetics of the drug
36. Usually healthy human volunteers are taken in
A. Phase I of clinical trial B. Phase II of clinical trial
C. Phase III of clinical trial D. Phase IV of clinical trial
37. Orphan drugs are
A. Drugs with high therapeutic failure B. Drugs with high toxicity
C. Drugs having low therapeutic margin D. Drugs for rare disease
38. Hemodialysis is useful in all of the following except
A. Barbiturate poisoning B. Methanol poisoning
C. Salicylate poisoning D. Digoxin poisoning
39. Elimination after 3 half-lives in first-order kinetics is
A. 12.5% B. 75%
C. 87.5% D. 94%
40. Drug remaining in the body after 3 half-lives is
A. 12.5% B. 75%
C. 87.5% D. 94%
41. The bioavailability of the drug depends upon
A. First-pass metabolism B. Second-pass metabolism
C. Volume of distribution D. Excretion
42. Bioavailability is
A. Amount of drug that reaches the systemic circulation
B. Drug metabolized in liver before the drug reaches the systemic circulation
C. Drug metabolized in liver after the drug reaches the systemic circulation
D. Maximum by rectal route
43. Therapeutic index is a measure of
A. Drug safety B. Bioavailability
C. Potency D. Efficacy
44. Partial agonist possess
A. Maximum intrinsic activity and low affinity B. High intrinsic activity and no affinity
C. Low intrinsic activity and high affinity D. Low intrinsic activity and low affinity

https://t.me/DentalBooksWorld
Chapter 8 • Pharmacology 711

45. Which of the following is untrue about biotransformation?


A. Inactive metabolites are formed
B. Active metabolites are formed
C. More fat soluble metabolites are formed
D. More H2O soluble metabolites are formed

ANESTHESIA
1. An ideal anesthetic drug would
A. Induce anesthesia smoothly and rapidly and secure rapid recovery
B. Possess a wide margin of safety
C. Be devoid of adverse effects
D. All of the above
2. Which of the following general anesthetics belongs to inhalants?
A. Thiopental B. Desflurane
C. Ketamine D. Propofol
3. Indicate the anesthetic, which is used intravenously
A. Propofol B. Halothane
C. Desflurane D. Nitrous oxide
4. Which of the following inhalants is a gas anesthetic?
A. Halothane B. Isoflurane
C. Nitrous oxide D. Desflurane
5. Sevoflurane has largely replaced halothane and isoflurane as an inhalation anesthetic of choice because
A. Induction of anesthesia is achieved more rapidly and smoothly
B. Recovery is more rapid
C. It has low postanesthetic organ toxicity
D. All of the above
6. The limitation of sevoflurane is
A. High incidence of coughing and laryngospasm
B. Chemically unstable
C. Centrally mediated sympathetic activation leading to a rise of BP and HR
D. Hepatotoxicity
7. Which of the following inhalants lacks sufficient potency to produce surgical anesthesia by itself and therefore is
commonly used with another inhaled or intravenous anesthetic?
A. Halothane B. Sevoflurane
C. Nitrous oxide D. Desflurane
8. Which of the following inhaled anesthetics has rapid onset and recovery?
A. Nitrous oxide B. Desflurane
C. Sevoflurane D. All of the above
9. Indicate the inhaled anesthetic, which reduces arterial pressure and heart rate
A. Isoflurane B. Halothane
C. Desflurane D. Nitrous oxide
10. Which of the following inhaled anesthetics causes centrally mediated sympathetic activation leading to a rise in blood
pressure and heart rate?
A. Desflurane B. Sevoflurane
C. Nitrous oxide D. Isoflurane
11. Indicate the inhaled anesthetic, which decreases the ventilatory response to hypoxia?
A. Sevoflurane B. Nitrous oxide
C. Desflurane D. Halothane
12. Which of the following inhaled anesthetics is an induction agent of choice in patient with airway problems?
A. Desflurane B. Nitrous oxide
C. Halothane D. None of the above

https://t.me/DentalBooksWorld
712 Triumph's Complete Review of Dentistry

13. Indicate the inhaled anesthetic, which causes the airway irritation
A. Nitrous oxide B. Sevoflurane
C. Halothane D. Desflurane
14. Which of the following inhaled anesthetics increases cerebral blood flow least of all?
A. Sevoflurane B. Nitrous oxide
C. Isoflurane D. Desflurane
15. Indicate the inhaled anesthetic, which should be avoided in patients with a history of seizure disorders
A. Enflurane B. Nitrous oxide
C. Sevoflurane D. Desflurane
16. All of the following factors increase anesthetic requirements except
A. Infants B. Hypernatremia
C. Hyperthermia D. Pregnancy
17. Indicate the inhaled anesthetic, which may cause nephrotoxicity
A. Halothane B. Sevoflurane
C. Nitrous oxide D. Diethyl ether
18. Which of the following inhaled anesthetics decrease methionine synthase activity and causes megaloblastic anemia?
A. Desflurane B. Halothane
C. Nitrous oxide D. Sevoflurane
19. Unlike inhaled anesthetics, intravenous agents such as thiopental, etomidate, and propofol
A. Have a faster onset and rate of recovery
B. Provide a state of conscious sedation
C. Are commonly used for induction of anesthesia
D. All of the above
20. Indicate the intravenous anesthetic, which is an ultra-short-acting barbiturate
A. Fentanyl B. Thiopental
C. Midazolam D. Ketamine
21. Local anesthetics produce
A. Analgesia, amnesia, loss of consciousness
B. Blocking pain sensation without loss of consciousness
C. Alleviation of anxiety and pain with an altered level of consciousness
D. A stupor or somnolent state
22. A good local anesthetic agent should not cause
A. Local irritation and tissue damage B. Systemic toxicity
C. Fast onset and long duration of action D. Vasodilatation
23. Most local anesthetic agents consist of
A. Lipophilic group (frequently an aromatic ring)
B. Intermediate chain (commonly including an ester or amide)
C. Amino group
D. All of the above
24. Which one of the following groups is responsible for the duration of the local anesthetic action?
A. Intermediate chain B. Lipophilic group
C. Ionizable group D. All of the above
25. Indicate the local anesthetic agent, which has a shorter duration of action
A. Lidocaine B. Procaine
C. Bupivacaine D. Ropivacaine
26. Which one of the following groups is responsible for the potency and the toxicity of local anesthetics?
A. Ionizable group B. Intermediate chain
C. Lipophilic group D. All of the above
27. Indicate the drug, which has greater potency of the local anesthetic action
A. Lidocaine B. Bupivacaine
C. Procaine D. Mepivacaine

https://t.me/DentalBooksWorld
Chapter 8 • Pharmacology 713

28. Ionizable group is responsible for


A. The potency and the toxicity B. The duration of action
C. The ability to diffuse to the site of action D. All of the above
29. Which one of the following local anesthetics is an ester of benzoic acid?
A. Lidocaine B. Procaine
C. Ropivacaine D. Cocaine
30. Indicate the local anesthetic, which is an ester of para-aminobenzoic acid
A. Mepivacaine B. Cocaine
C. Procaine D. Lidocaine
31. Which of the following local anesthetics is an acetanilide derivative?
A. Tetracaine B. Lidocaine
C. Cocaine D. Procaine
32. Indicate the local anesthetic, which is a toluidine derivative
A. Lidocaine B. Bupivacaine
C. Prilocaine D. Procaine
33. Which of the following local anesthetics is a thiophene derivative?
A. Procaine B. Ultracaine
C. Lidocaine D. Mepivacaine
34. Local anesthetics are
A. Weak bases B. Weak acids
C. Salts D. None of the above
35. For therapeutic application local anesthetics are usually made available as salts for the reasons of
A. Less toxicity and higher potency B. Higher stability and greater lipid solubility
C. Less local tissue damage and more potency D. More stability and greater water solubility
36. Which of the following statements is not correct for local anesthetics?
A. In a tissue they exist either as an uncharged base or as a cation
B. A charged cationic form penetrates biologic membranes more readily than an uncharged form
C. Local anesthetics are much less effective in inflamed tissues
D. Low pH in inflamed tissues decreases the dissociation of nonionized molecules
37. Which one of the following statements about the metabolism of local anesthetics is incorrect?
A. Metabolism of local anesthetics occurs at the site of administration
B. Metabolism occurs in the plasma or liver but not at the site of administration
C. Ester group of anesthetics like procaine, are metabolized systemically by pseudocholinesterase
D. Amides such as lidocaine, are metabolized in the liver by microsomal mixed function oxidases
38. Indicate the anesthetic agent of choice in patient with a liver disease
A. Lidocaine B. Bupivacaine
C. Procaine D. Etidocaine
39. Which of the following local anesthetics is preferable in a patient with pseudocholinesterase deficiency?
A. Procaine B. Ropivacaine
C. Tetracaine D. Benzocaine
40. The primary mechanism of action of local anesthetics is
A. Activation of ligand-gated potassium channels
B. Blockade of voltage-gated sodium channels
C. Stimulation of voltage-gated N-type calcium channels
D. Blockade the GABA-gated chloride channels
41. Which of the following local anesthetics is more water-soluble?
A. Tetracaine B. Etidocaine
C. Procaine D. Bupivacaine
42. Indicate the local anesthetic, which is more lipid-soluble
A. Bupivacaine B. Lidocaine
C. Mepivacaine D. Procaine

https://t.me/DentalBooksWorld
714 Triumph's Complete Review of Dentistry

43. The more lipophilic drugs


A. Are more potent B. Have longer duration of action
C. Bind more extensively to proteins D. All of the above
44. Which of the following fibers is the first to be blocked?
A. Type A alpha fibers B. B and C fibers
C. Type A beta fibers D. Type A gamma fibers
45. Indicate the function, which is the last to be blocked
A. Pain, temperature B. Muscle spindles
C. Motor function D. Touch, pressure

ANTIMICROBIALS
1. Which of the following is not excreted in bile?
A. Erythromycin B. Ampicillin
C. Rifampicin D. Gentamicin
2. Multiple drug resistance is transferred through
A. Transduction B. Transformation
C. Conjugation D. Mutation
3. Most common mechanism for transfer of resistance in Staphylococcus aureus is
A. Conjugation B. Transduction
C. Transformation D. Mutation
4. Mechanism of drug resistance is via elaboration of inactivating enzymes among the following antibiotics except
A. Quinolones B. Penicillin
C. Chloramphenicol D. Aminoglycosides
5. Pneumococcal resistance to penicillin G is mainly acquired by
A. Conjugation B. Transduction
C. Transformation D. All of the above
6. Drug of choice of MRSA
A. Vancomycin B. Penicillin
C. Cephalosporin D. Clindamycin
7. Find the drug which acts by inhibiting cell wall synthesis
A. Erythromycin B. Cephalosporins
C. Chloramphenicol D. Sulfonamides
8. In staphylococci, plasmids encoding beta-lactamase are transmitted by
A. Conjugation B. Transduction
C. Transposon D. Transformation
9. Identify the drug that is bactericidal
A. Sulfonamides B. Erythromycin
C. Chloramphenicol D. Co-trimoxazole
10. Superinfection is common in
A. Narrow spectrum antibiotics B. Immunocompromised host
C. Low spectrum antibiotics D. Nutritional deficiency
11. Which of the following is a broad spectrum antibiotic?
A. Erythromycin B. Streptomycin
C. Tetracycline D. All of the above
12. Find the antibiotic that does not act by inhibiting protein synthesis
A. Vancomycin B. Tetracycline
C. Streptomycin D. Azithromycin
13. Antipseudomonal penicillin is
A. Cephalexin B. Cloxacillin
C. Piperacillin D. Dicloxacillin

https://t.me/DentalBooksWorld
Chapter 8 • Pharmacology 715

14. Which one of the following is not penicillinase susceptible?


A. Amoxicillin B. Penicillin G
C. Piperacillin D. Cloxacillin
15. Which of the following antimicrobials has antipseudomonal action?
A. Cefpodoxime proxetil B. Cephradine
C. Cefotetan D. Cefoperazone
16. Which of the following is a fourth-generation cephalosporin?
A. Ceftriaxone B. Cefaclor
C. Cefepime D. Cefuroxime
17. All of the following cephalosporins have good activity against Pseudomonas aeruginosa except
A. Cefadroxil B. Cefepime
C. Cefoperazone D. Ceftazidime
18. Which of the following statements are true regarding cefepime?
A. It is a fourth-generation cephalosporin
B. Once a day dose is sufficient
C. It possess antipseudomonal action
D. Its dose should not be reduced in renal pathology
E. It is a prodrug
19. Which one of the following is not true about cefepime?
A. Fourth-generation cephalosporin B. Useful in hospital-acquired infection
C. Inhibits transpeptidase D. Given twice daily orally
20. All of the following are antipseudomonals, except
A. Cephalexin B. Carbenicillin
C. Piperacillin D. Ceftazidime
21. Cilastatin is given along with
A. Imipenem B. Amoxicillin
C. Erythromycin D. Ampicillin
22. Which of the following cephalosporins is active against Pseudomonas aeruginosa?
A. Ceftriaxone B. Cephalothin
C. Ceftazidime D. Cefotaxime
23. Which of the following is not true about penicillins?
A. Penicillin V is absorbed orally B. Benzathine penicillin is a short-acting penicillin
C. Cloxacillin is β-lactamase and acid resistant D. Ampicillin is not resistant to β-lactamases
24. Mechanism of action of vancomycin is
A. Inhibition of cell wall synthesis B. Inhibition of protein synthesis
C. Leakage from cell membrane D. Inhibition of DNA gyrase
25. Carbenicillin
A. Is effective in pseudomonas infection B. Has no effect in Proteus infection
C. Is a macrolide antibiotic D. All the above
26. A potent inhibitor of beta-lactamase is
A. Carbenicillin B. Clavulanic acid
C. Cefamandole D. Idoxuridine
27. All are true about cefuroxime except
A. Inhibit cell wall synthesis B. Third-generation cephalosporin
C. Some acquired resistance with penicillin D. More active against Gram-negative organisms
28. Amoxicillin is better than ampicillin due to
A. Better bioavailability if taken with food B. Lesser bioavailability if taken with food
C. Incidence of diarrhea is higher D. More active against Shigella and H. influenzae
29. Mechanism of action of penicillins and cephalosporins is to inhibit
A. Cell wall synthesis B. Leakage from cell membrane
C. Protein synthesis D. DNA gyrase

https://t.me/DentalBooksWorld
716 Triumph's Complete Review of Dentistry

30. The following organisms are known to develop resistance to penicillin except
A. Staphylococcus B. Streptococcus
C. Pneumococcus D. Treponema
31. Ceftriaxone is
A. A second-generation short-acting cephalosporin B. Has activity against beta lactamase producing bacteria
C. A fourth-generation long-acting cephalosporin D. A third-generation long-acting cephalosporin
32. Acid susceptible penicillin is
A. Methicillin B. Ampicillin
C. Amoxicillin D. Cloxacillin
33. All are first-generation cephalosporins except:
A. Cefadroxil B. Cefazolin
C. Cephalexin D. Cefaclor
34. Which is not a beta-lactam antibiotic?
A. Penicillin B. Carbapenem
C. Monobactam D. Azithromycin
35. A second-generation cephalosporin that can be used orally is
A. Cefepime B. Cefalotin
C. Cefaclor D. Cefoperazone
36. Third-generation cephalosporin that can be given orally is
A. Cefixime B. Cefpirome
C. Cefaclor D. Cefadroxil
37. Ampicillin is not given in EB virus infection due to
A. Increased toxicity B. Skin rash
C. Blindness D. Convulsions
38. Which among the following is not a beta-lactamase inhibitor?
A. Sulbactam B. Clavulanic acid
C. Piperacillin D. None
39. Oral cephalosporin among these is
A. Cefotaxime B. Ceftriaxone
C. Cefaclor D. Ceftazidime
40. Beta lactam antibiotics act by inhibiting
A. Cell wall synthesis B. Protein synthesis
C. RNA synthesis D. DNA synthesis
41. Which one of the following is a fourth-generation cephalosporin?
A. Cefuroxime B. Ceftazidime
C. Cefepime D. Cefamandole
42. Neutropenia is associated with
A. Nafcillin B. Methicillin
C. Carbenicillin D. Ampicillin
43. Third-generation cephalosporins include all of the following except
A. Ceftizoxime B. Cefoperazone
C. Cefoxitin D. Cefixime
44. Which of the following drugs act by inhibiting bacterial protein synthesis?
A. Bacitracin B. Dapsone
C. Ethambutol D. Streptomycin
45. Tetracyclines are not useful for
A. Trichomonas B. Chlamydia
C. Syphilis D. Rickettsia
46. Which of the following drug interferes with translocation of protein synthesis?
A. Erythromycin B. Tetracycline
C. Chloramphenicol D. Penicillins

https://t.me/DentalBooksWorld
Chapter 8 • Pharmacology 717

47. Chloramphenicol acts through action on


A. 50S ribosome B. 30S ribosome
C. Nucleus D. Mitochondria
48. Tetracyclines can be given in all forms except
A. Oral B. Intravenous
C. Topical in eye D. Topical in open wound
49. Which of the following penicillins is effective against pseudomonas?
A. Piperacillin B. Amoxicillin
C. Ampicillin D. Oxacillin
50. Drug of choice for prophylaxis of diphtheria is
A. Tetracycline B. Erythromycin
C. Ciprofloxacin D. Amikacin
51. Which of the following is least nephrotoxic?
A. Streptomycin B. Gentamicin
C. Polymyxin B D. Doxycycline
52. All of the following are drugs commonly used against enteric fever, except
A. Amikacin B. Ciprofloxacin
C. Ceftriaxone D. Azithromycin
53. Most effective drug against extracellular mycobacteria is
A. Isoniazid B. Rifampicin
C. Pyrazinamide D. Ethambutol
54. Hypothyroidism is caused by which of the following antitubercular drug?
A. Streptomycin B. Ethionamide
C. Thioacetazone D. Ethambutol
55. ATT causing orange colored urine is
A. Rifampicin B. Isoniazid
C. Streptomycin D. Pyrazinamide
56. Which of the following antitubercular drug is not hepatotoxic?
A. Isoniazid B. Rifampicin
C. Ethionamide D. Streptomycin
57. Which of the following is active against atypical mycobacteria?
A. Clarithromycin B. Rifabutin
C. Ciprofloxacin D. All of the above
58. ATT most commonly implicated in causing peripheral neuropathy is
A. Rifampicin B. Pyrazinamide
C. INH D. Ethambutol
59. Arthralgia is commonly caused by which ATT drug?
A. INH B. Rifampicin
C. Pyrazinamide D. Ethambutol
60. Which of the following antitubercular drugs can be safely used in severe renal failure?
A. Streptomycin B. Ethambutol
C. Capreomycin D. Rifampicin
61. Common dose-dependent side effect of ethambutol is
A. Red-urine B. Optic neuritis
C. Nephropathy D. Peripheral neuropathy
62. Most common drug used in Leprosy is
A. Dapsone B. Clofazimine
C. Ethionamide D. Ofloxacin
63. The bacterial drug resistance in tuberculosis results from
A. Transduction B. Transformation
C. Plasmid mediated D. Mutation

https://t.me/DentalBooksWorld
718 Triumph's Complete Review of Dentistry

64. INH and pyridoxine are given together in antituberculous chemotherapy


A. To prevent peripheral neuritis B. To prevent emergence of INH resistance
C. As a nutrient supplement D. As a synergistic combination
65. Bacteriostatic antitubercular drug among the following is
A. Isoniazid B. Rifampin
C. Streptomycin D. Ethambutol
66. Dapsone is used in all except
A. Dermatitis herpetiformis B. Leprosy
C. Pneumocystis jiroveci pneumonia D. Tuberculosis
67. In Lepra reaction, the drug useful is
A. Penicillins B. Clofazimine
C. Dapsone D. None
68. Drug of choice for herpes simplex virus infection is
A. Acyclovir B. Zidovudine
C. Indinavir D. Ribavirin
69. All can be used for systematic fungal infections except
A. Ketoconazole B. Fluconazole
C. Amphotericin B D. Griseofulvin
70. Drug of choice for chronic hepatitis-B is
A. Lamivudine B. IFN-alpha
C. Ribavirin D. Zidovudine
71. All are effective against Tinea versicolor except
A. Fluconazole B. Clotrimazole
C. Ketoconazole D. Griseofulvin
72. Drugs that can be used to treat candida infection are all except
A. Ketoconazole B. Nystatin
C. Amphotericin D. Griseofulvin
73. Which of the following antifungal drugs has only topical action?
A. Fluconazole B. Ketoconazole
C. Itraconazole D. Clotrimazole
74. Which is the integrase inhibitor used in treatment of HIV?
A. Raltegravir B. Indinavir
C. Lopinavir D. Fosamprenavir
75. All of the following cause inhibition of CYP3A except
A. Saquinavir B. Ritonavir
C. Itraconazole D. Erythromycin
76. Which of the following drug is a reverse transcriptase inhibitor?
A. Indinavir B. Ritonavir
C. Nelfinavir D. Abacavir
77. Maximum risk of pancreatitis is present with
A. Didanosine B. Lamivudine
C. Zidovudine D. Abacavir

AUTONOMIC NERVOUS SYSTEM


1. Acetylcholine is not a specific neurotransmitter at
A. Sympathetic ganglia B. Sympathetic postganglionic nerve endings
C. Parasympathetic ganglia D. Parasympathetic postganglionic nerve endings
2. Muscarinic receptors are located in
A. Autonomic ganglia B. Skeletal muscle neuromuscular junctions
C. Autonomic effector cells D. Sensory carotid sinus baroreceptor zone

https://t.me/DentalBooksWorld
Chapter 8 • Pharmacology 719

3. Indicate the location of M2 cholinoceptor type


A. Heart B. Glands
C. Smooth muscle D. Endothelium
4. The symptoms of mushroom poisoning include all of the following, except
A. Salivation, lacrimation, nausea, vomiting B. Dryness of mouth, hyperpyrexia, hallucination
C. Headache, abdominal colic D. Bradycardia, hypotension, and shock
5. Which of the following cholinomimetics activates both muscarinic and nicotinic receptors?
A. Lobeline B. Pilocarpine
C. Nicotine D. Bethanechol
6. Indicate a cholinomimetic agent, which is related to direct-acting drugs
A. Edrophonium B. Physostigmine
C. Carbachol D. Isoflurophate
7. Characteristics of carbachol include all of the following, except
A. It decreases intraocular pressure B. It causes mydriasis
C. It exerts both nicotinic and muscarinic effects D. It is resistant to acetylcholinesterase
8. Acetylcholine is not used in clinical practice because
A. It is very toxic B. The doses required are very high
C. It is very rapidly hydrolyzed D. It is very costly
9. Parasympathomimetic drugs cause
A. Bronchodilation B. Mydriasis
C. Bradycardia D. Constipation
10. Which of the following direct-acting cholinomimetics is mainly muscarinic in action?
A. Bethanechol B. Carbachol
C. Acetylcholine D. None of the above
11. Which of the following direct-acting cholinomimetics has the shortest duration of action?
A. Acetylcholine B. Methacholine
C. Carbachol D. Bethanechol
12. Bethanechol has all of the following properties, except
A. It is extremely resistant to hydrolysis B. Purely muscarinic in its action
C. It is used for abdominal urinary bladder distention D. It exerts both nicotinic and muscarinic effects
13. A M-cholinomimetic agent is
A. Carbachol B. Pilocarpine
C. Acetylcholine D. Bethanechol
14. Characteristics of pilocarpine include all of the following, except
A. It is a tertiary amine alkaloid
B. It causes miosis and a decrease in intraocular pressure
C. Causes a decrease in secretory and motor activity of gut
D. It is useful in the treatment of glaucoma
15. Which of the following cholinomimetics is a plant derivative with lower potency than nicotine but with a similar
spectrum of action?
A. Lobeline B. Pilocarpine
C. Carbachol D. Acetylcholine
16. Which of the following cholinomimetics is indirect-acting?
A. Lobeline B. Edrophonium
C. Pilocarpine D. Carbachol
17. The mechanism of action of indirect-acting cholinomimetic agents is
A. Binding to and activation of muscarinic or nicotinic receptors
B. Inhibition of the hydrolysis of endogenous acetylcholine
C. Stimulation of the action of acetyl cholinesterase
D. Releasing acetylcholine from storage sites

https://t.me/DentalBooksWorld
720 Triumph's Complete Review of Dentistry

18. Indicate a reversible cholinesterase inhibitor


A. Isoflurophate B. Carbachol
C. Physostigmine D. Parathion
19. Which of the following cholinesterase inhibitors is irreversible?
A. Physostigmine B. Edrophonium
C. Neostigmine D. Isoflurophate
20. Indicate cholinesterase activator
A. Pralidoxime B. Edrophonium
C. Pilocarpine D. Isoflurophate
21. Isoflurophate increases all of the following effects except
A. Lacrimation B. Bronchodilation
C. Muscle twitching D. Salivation
22. Indicate a cholinesterase inhibitor, which has an additional direct nicotinic agonist effect
A. Edrophonium B. Carbachol
C. Neostigmine D. Lobeline
23. Сholinesterase inhibitors do not produce
A. Bradycardia, no change or modest fall in blood pressure
B. Increased strength of muscle contraction, especially in muscles weakened by myasthenia gravis
C. Miosis and reduction of intraocular pressure
D. Dramatic hypertension and tachycardia
24. Which of the following cholinomimetics is commonly used in the treatment of glaucoma?
A. Pilocarpine B. Lobeline
C. Acetylcholine D. Neostigmine
25. Indicate the organophosphate cholinesterase inhibitor, which can be made up in an aqueous solution for ophthalmic
use and retains its activity within a week
A. Physostigmine B. Edrophonium
C. Echothiophate D. Neostigmine
26. Which of the following cholinomimetics is most widely used for paralytic ileus and atony of the urinary bladder?
A. Lobeline B. Neostigmine
C. Pilocarpine D. Echothiophate
27. Chronic long-term therapy of myasthenia is usually accomplished with
A. Edrophonium B. Neostigmine
C. Echothiophate D. Carbachol
28. Which of the following cholinomimetics is a drug of choice for reversing the effects of nondepolarizing neuromuscular
relaxants?
A. Echothiophate B. Physostigmine
C. Edrophonium D. Pilocarpine
29. Indicate the reversible cholinesterase inhibitor, which penetrates the blood–brain barrier
A. Physostigmine B. Edrophonium
C. Neostigmine D. Pyridostigmine
30. Which of the following cholinomimetics is used in the treatment of atropine intoxication?
A. Neostigmine B. Carbachol
C. Physostigmine D. Lobeline
31. The symptoms of excessive stimulation of muscarinic receptors include all of the following except
A. Abdominal cramps, diarrhea B. Increased salivation, excessive bronchial secretion
C. Miosis, bradycardia D. Weakness of all skeletal muscles
32. The excessive stimulation of muscarinic receptors by pilocarpine and choline esters is blocked competitively by
A. Edrophonium B. Atropine
C. Pralidoxime D. Echothiophate

https://t.me/DentalBooksWorld
Chapter 8 • Pharmacology 721

33. The toxic effects of a large dose of nicotine include all of the following, except
A. Hypotension and bradycardia
B. Convulsions, coma, and respiratory arrest
C. Skeletal muscle depolarization blockade and respiratory paralysis
D. Hypertension and cardiac arrhythmias
34. The dominant initial sites of acute cholinesterase inhibitors intoxication include all of the following except
A. Salivation, sweating B. Mydriasis
C. Bronchial constriction D. Vomiting and diarrhea
35. Which of the following drugs is used for acute toxic effects of organophosphate cholinesterase inhibitors?
A. Atropine B. Pilocarpine
C. Pralidoxime D. Edrophonium

AUTACOIDS
1. Indication of Misoprostol is
A. Uterine relaxant B. Antiulcer
C. Bronchodilator D. Vasodilator
2. Which drug causes analgesic nephropathy?
A. Aspirin B. Ibuprofen
C. Phenacetin D. Phenylbutazone
3. Aspirin is
A. Methyl salicylate B. Para-aminobenzoic acid
C. Para-aminosalicylic acid D. Acetyl salicylic acid
4. False about COX-2 is
A. It is constitutionally expressed on some cell surfaces
B. Activation of COX-2 leads to ulceroprotective effect on gastric mucosa
C. Induced at the site of inflammation
D. It is utilized in generation of eicosanoids with a ring structure
5. An example of nonopioid analgesic and which does not inhibit prostaglandin synthesis is
A. Nefopam B. Tenoxicam
C. Ketorolac D. Piroxicam
6. Which of the following is false about Reye’s syndrome?
A. Hepatic encephalopathy B. Seen with ampicillin therapy
C. Fever and rash D. Viral associated
7. Ibuprofen acts on
A. Lipoxygenase pathway B. Cyclooxygenase pathway
C. Kinin system D. Serotonin system
8. Which of the following prostaglandin analogues is used in glaucoma?
A. Misoprostol B. Latanoprost
C. Enprostil D. Rioprostil
9. Cyclooxygenase enzyme is not inhibited by
A. Aspirin B. Warfarin
C. Phenylbutazone D. Diclofenac

HEMATOLOGY
1. In pregnancy which anticoagulant is given?
A. Heparin B. Warfarin
C. Dicoumarol D. Phenindione
2. All drugs cross placenta except
A. Heparin B. Warfarin
C. Dicoumarol D. Nicoumalone

https://t.me/DentalBooksWorld
722 Triumph's Complete Review of Dentistry

3. Which of the following is not an anticoagulant?


A. Phytonadione B. Warfarin
C. LMW heparin D. Lepirudin
4. Orally acting direct thrombin inhibitor is
A. Bivalirudin B. Ximelagatran
C. Melagatran D. Argatroban
5. Heparin does not cause
A. Osteoporosis B. Factor V inhibition
C. Thrombocytopenia D. Prolongation of aPTT
6. Which of the following is not seen with heparin therapy?
A. Skin necrosis B. Thrombosis and thrombocytopenia
C. Osteoporosis D. Alopecia
7. All of the following are adverse effects of heparin except
A. Bleeding B. Thrombocytopenia
C. Hypokalemia D. Osteoporosis
8. Vitamin K-dependent clotting factors are
A. Factor IX and X B. Factor IV
C. Factor XII D. Factor I
9. All are true about warfarin, except
A. It inhibits the activation of vitamin K dependent clotting factors
B. Its half-life is 36 hours
C. It can cross placenta
D. Its dose is increased in liver disease
10. Drug used in heparin overdose is
A. Protamine sulfate B. Phylloquinone
C. Ticlopidine D. Clopidogrel
11. As compared to unfractionated heparin, low molecular weight heparins
A. Are absorbed more uniformly when given subcutaneously
B. Require more frequent laboratory monitoring
C. Can be given to patients with heparin-induced thrombocytopenia
D. Predispose to a higher risk of osteopenia
12. All of the following are true statements about heparin, except
A. It prolongs aPTT B. Hyperkalemia is not seen
C. It can result in alopecia D. It can cause thrombocytopenia
13. Recent oral direct thrombin inhibitor which can be used for prevention of stroke is
A. Dabigatran B. Ximelagatran
C. Lepirudin D. Saxagliptin
14. Vitamin K is involved in the post-translational modification of
A. Glutamate B. Aspartate
C. Glycine D. GABA
15. Which of the following is a vitamin K-dependent clotting factor?
A. Factor VII B. Factor I
C. Factor XI D. Factor XII
16. Action of aspirin is due to
A. Decrease in thromboxane A2 B. Inhibition of adenylyl cyclase
C. GP IIb/IIIa inhibition D. ADP antagonism
17. Aspirin is contraindicated in a patient who is on treatment with
A. Prednisolone B. Warfarin
C. Theophylline D. Oral contraceptives

https://t.me/DentalBooksWorld
Chapter 8 • Pharmacology 723

18. What is the formula for parenteral iron therapy?


A. 4.4. × body weight (kg) × Hb deficit (g/dL) B. 3.3. × body weight (kg) × Hb deficit (g/dL)
C. 2.2 × body weight (kg) × Hb deficit (g/dL) D. 1.1 × body weight (kg) × Hb deficit (g/dL)
19. Cyanide poisoning can be treated by
A. Pyridoxine B. Vitamin B12
C. Hyperbaric oxygen D. Flumazenil
20. Mechanism of action of aspirin is
A. Inhibits COX-2 preferentially B. Inhibits COX-1 preferentially
C. Inhibits COX-1 and COX-2 reversibly D. Inhibits COX-1 and COX-2 irreversibly
21. All of the following are true about heparin-induced thrombocytopenia, except
A. Low-molecular-weight heparins should not be used for treatment
B. It causes both arterial and venous thrombosis
C. More common with fractionated heparin
D. Occurs after about a week of heparin therapy
22. Clopidogrel inhibit platelet aggregation by
A. Inhibit GpIIb/IIIa B. Inhibits phosphodiesterase
C. Inhibits ADP D. Inhibits cyclooxygenase
23. Protamine antagonism for heparin is
A. Competitive B. Chemical
C. Toxic D. Noncompetitive

RESPIRATORY SYSTEM
1. Which of the following drugs has been found to be useful in acute severe asthma?
A. Magnesium sulfate B. Anti-leukotriene
C. Cromolyn sodium D. Cyclosporine
2. Leukotriene receptor antagonist used for bronchial asthma is
A. Zafirlukast B. Zileuton
C. Cromolyn sodium D. Aminophylline
3. Which enzyme is inhibited by aminophylline?
A. Monoamine oxidase B. Alcohol dehydrogenase
C. Phosphodiesterase D. Cytochrome P450
4. The drug not used in acute asthma is
A. Salbutamol B. Ipratropium
C. Montelukast D. Hydrocortisone
5. Which of the following drugs cannot be administered by inhalation?
A. Theophylline B. Ipratropium bromide
C. Budesonide D. Terbutaline
6. Disodium cromoglycate is used by which of the following routes?
A. Inhalation B. Oral
C. IV D. IM
7. Which is a “soft steroid” used in bronchial asthma?
A. Budesonide B. Dexamethasone
C. Ciclesonide D. Flunisolide
8. Omalizumab is administered in bronchial asthma by which route?
A. Oral B. Intravenous
C. Subcutaneous D. Aerosol
9. Directly acting cough suppressant is
A. Dextromethorphan B. Bromhexine
C. Acetylcysteine D. Carbetapentane

https://t.me/DentalBooksWorld
724 Triumph's Complete Review of Dentistry

10. Longest acting β-agonist is


A. Salbutamol B. Terbutaline
C. Salmeterol D. Theophylline
11. Complications of aerosol steroids use include
A. Oral candidiasis B. Cushing’s syndrome
C. Decreased ACTH D. Systemic complications
12. The following drug is contraindicated in bronchial asthma
A. Propranolol B. Ipratropium bromide
C. Theophylline D. Ketotifen
13. Advantage of salmeterol over salbutamol is its
A. Shorter duration of action B. More potency
C. Longer duration of action D. Lesser cardiac effects
14. Interaction of theophylline with ciprofloxacin is
A. Ciprofloxacin increases theophylline metabolism B. Ciprofloxacin decreases theophylline metabolism
C. Theophylline increases ciprofloxacin metabolism D. Theophylline decreases ciprofloxacin metabolism
15. Theophylline overdose causes
A. Bradycardia B. Seizures
C. Drowsiness D. Bronchospasm
16. Therapeutic blood range of theophylline in microgram per milliliter is
A. 0–5 B. 5–10
C. 5–15 D. 5–20

CARDIOVASCULAR SYSTEM
1. Which of the following drug is not used in congestive heart failure?
A. Adrenaline B. Digoxin
C. Hydrochlorothiazide D. Enalapril
2. Which one of the following is not a contraindication for use of digitalis?
A. Acute rheumatic carditis B. Thyrotoxicosis
C. WPW syndrome D. Hyperkalemia
3. Therapeutic plasma level of digoxin is
A. 0.1–0.3 ng/mL B. 0.8–1.5 ng/mL
C. 1.2–2 ng/mL D. More than 2.4 ng/mL
4. Time taken for digitalization is
A. 36 hours B. 12 hours
C. 5 day D. 10 day
5. Which drug directly acts on blood vessels?
A. Hydralazine B. Verapamil
C. Propanolol D. Methyldopa
6. Digitalis toxicity can cause
A. Hyperkalemia B. Nausea
C. Arrhythmias D. All of the above
7. The drug that is used in left ventricular failure is
A. Propanolol B. Morphine
C. Amlodipine D. Epinephrine
8. Which one of the following is not a vasodilator?
A. Methyl dopa B. Nitroprusside
C. Hydralazine D. Diazoxide
9. Patient with hypertension and diabetes mellitus. What is the treatment of choice?
A. Beta-blockers B. Thiazides
C. ACE inhibitors D. Calcium channel blockers

https://t.me/DentalBooksWorld
Chapter 8 • Pharmacology 725

10. An antihypertensive that causes sedation is


A. Clonidine B. Hydralazine
C. Losartan D. Amlodipine
11. Propanolol is contraindicated in a patient of angina pectoris who is already receiving
A. Nifedipine B. Aspirin
C. Verapamil D. Isosorbide mononitrate
12. All of the following antiarrhythmic drugs are correctly matched to the group
A. Procainamide: class I B. Amiodarone: class III
C. Esmolol: class IV D. Diltiazem: class IV
13. Mechanism of action of lovastatin is
A. HMG CoA reductase inhibitor B. Decarboxylase inhibitor
C. Activate lipoprotein lipase D. Inhibits lipolysis
14. Most potent statin is
A. Simvastatin B. Pitavastatin
C. Atorvastatin D. Rosuvastatin
15. Mechanism of action of NO is
A. ↑ cAMP B. ↑ cGMP
C. ↑ PGE2 D. ↑ PGD4
16. Iodine content in amiodarone is
A. 10–20% B. 20–40%
C. 40–60% D. 60–80%
17. Which of the following is not a direct acting anti-platelet agent?
A. Aspirin B. Clopidogrel
C. Atorvastatin D. Alteplase
18. Which of the following is not a cardioselective β-blocker?
A. Acebutolol B. Atenolol
C. Pindolol D. Metoprolol

ENDOCRINOLOGY
1. Which one of the following is not an antithyroid drug?
A. Propylthiouracil B. Methimazole
C. Carbimazole D. Carbamazepine
2. Indication of L-thyroxine is
A. Thyroid storm B. Cretinism
C. Endemic goiter D. Grave’s disease
3. Management which is safe for pregnant women in case of hyperthyroidism is
A. Radioactive iodine B. Methimazole
C. Carbimazole D. Propylthiouracil
4. Which of the following drug does not cause hypothyroidism?
A. PAS B. Captopril
C. Lithium D. Amiodarone
5. Conversion of T4 to T3 inhibition is associated with
A. Propylthiouracil B. Ampicillin
C. Lithium D. Carbimazole
6. Insulin does not cause
A. Glycogenesis B. Glycolysis
C. Lipogenesis D. Ketogenesis
7. Consequence of rapid administration of insulin alone in diabetic ketoacidosis is
A. Hypokalemia B. Hypernatremia
C. Hyperkalemia D. Hypocalcemia

https://t.me/DentalBooksWorld
726 Triumph's Complete Review of Dentistry

8. Which of the following is not true about acarbose?


A. It acts by inhibiting the enzyme alpha-glucosidase
B. It reduces postprandial hyperglycemia
C. It decreases the progression of impaired glucose tolerance to overt diabetes mellitus
D. It can cause hypoglycemia
9. All of the following preparations of insulin are rapid and short acting, except
A. Lispro B. Aspart
C. Glargine D. NPH
10. Insulin acts by stimulation of
A. Ionotropic receptor B. Enzymatic receptor
C. Metabotropic receptor D. Nuclear receptor
11. The most common route of administration of insulin is
A. Intradermal B. Subcutaneous
C. Intramuscular D. Intravenous
12. Human insulin as compared to pork/beef insulin is
A. More potent B. Rapidly absorbed
C. Longer acting D. More antigenic
13. Glipizide differs from chlorpropamide in that it
A. Is more potent B. Is longer acting
C. Does not lower blood sugar in nondiabetic subjects D. Is less prone to cause hypoglycemic reaction
14. Sulfonylureas act by
A. Decreasing glucagon secretion from pancreas B. Decreasing insulin secretion from pancreas
C. Increasing gluconeogenesis D. Increasing insulin secretion from pancreas
15. Flushing is common in patients taking which of the following oral hypoglycemic drug with alcohol?
A. Chlorpropamide B. Phenformin
C. Glibenclamide D. Tolazamide
16. Antidiabetic effect of sulfonylureas is by reducing
A. Glucagon production B. Insulin secretion
C. Tissue sensitivity to insulin D. Tissue sensitivity to glycogen
17. Lactic acidosis is common in
A. Metformin B. Phenformin
C. Repaglinide D. Rosiglitazone
18. Tolbutamide acts by increasing
A. Insulin receptors B. Glucose entry
C. Glucose absorption D. Insulin secretion
19. Adverse effects of insulin include all of the following except
A. Edema B. Hyperglycemia
C. Lipodystrophy D. Allergy
20. Long-acting insulin is
A. Lente B. Semilente
C. Ultralente D. Lispro insulin
21. Insulin having longest duration of action is
A. Isophane insulin B. Protamine zinc insulin
C. Insulin zinc suspension D. Plain insulin

CENTRAL NERVOUS SYSTEM


1. Morphine cannot be used in which condition?
A. Head injury B. Asthma
C. Hypothyroidism D. Diabetes

https://t.me/DentalBooksWorld
Chapter 8 • Pharmacology 727

2. If the mother is on which of the following drug, naloxone will be contraindicated in neonatal resuscitation?
A. Cocaine B. Amphetamine
C. Methadone D. Phencyclidine
3. Opioid which has a maximum plasma protein binding capacity is
A. Morphine B. Sufentanil
C. Fentanyl D. Pethidine
4. The effect of μ-opioid receptor is
A. Miosis B. Tachycardia
C. Hyperthermia D. Bronchodilation
5. Which of the following drug is contraindicated for analgesia in a patient of head injury?
A. Morphine B. NSAIDs
C. Rofecoxib D. Acetaminophen
6. Antidote for ethylene glycol poisoning is
A. Methyl violet B. Fluconazole
C. Fomepizole D. Ethyl alcohol
7. The opioid receptor associated with dysphoria is
A. Mu B. Kappa
C. Delta D. None
8. Drug of choice in acute morphine poisoning is
A. Atropine B. Methadone
C. Naloxone D. Alcohol
9. For which of the following poisoning naltrexone is used?
A. Heroin B. Atropine
C. Cannabis D. Diazepam
10. Disulfiram like reaction is not seen with
A. Amoxicillin B. Metronidazole
C. Cefoperazone D. Disulfiram
11. Which of the following is true regarding opioid-induced seizures?
A. They usually occur at therapeutic doses B. Children are more susceptible
C. Seizures occur only with µ-opioid agonists D. Diazepam is the drug of choice in treatment
12. What is Antabuse?
A. Inhibits glucuronide conjugation B. Inhibits oxidation of alcohol
C. Inhibits excretion of alcohol through kidney D. None of the above
13. Site of action of opioid receptor is
A. Area postrema B. Dorsal horn
C. Injury site D. Brain
14. Opium is a derivative of
A. Solanum tuberosum B. Datura stramonium
C. Papaver somniferum D. Nicotiana tobacum
15. The most potent analgesic agent is
A. Fentanyl B. Sufentanil
C. Remifentanil D. Alfentanil
16. A nonsynthetic alkaloid compound acting similar to amphetamine is
A. Caffeine B. Cocaine
C. Nicotine D. All of the above
17. Pure opiate antagonists are all of the following except
A. Naloxone B. Nalorphine
C. Nalmefene D. Naltrexone
18. Endogenous opioid peptide includes
A. Encephalin B. Endorphins
C. Dynorphins D. All of the above

https://t.me/DentalBooksWorld
728 Triumph's Complete Review of Dentistry

19. Antidote of methyl alcohol poisoning is


A. Barbiturate B. Fomepizole
C. Phenytoin D. Lamotrigine
20. Which of the following opioid analgesic is suitable for hemodynamically unstable patients?
A. Morphine B. Meperidine
C. Fentanyl D. Pentazocine

DRUGS ACTING ON KIDNEY


1. Which of the following is not a use of aldosterone antagonists?
A. Hypertension B. Congestive heart failure
C. Gynecomastia D. Hirsutism
2. Contraindication of carbonic anhydrase inhibitor is
A. Sulfonamide hypersensitivity B. Glaucoma
C. High altitude sickness D. Metabolic acidosis
E. COPD
3. Identify the potassium sparing diuretics
A. Spironolactone B. Triamterene
C. Amiloride D. Ethacrynic acid
4. Diuretics that causes metabolic acidosis
A. Indapamide B. Furosemide
C. Hydrochlorothiazide D. Acetazolamide
5. Diuretic agent that cause gynecomastia if used for long term is
A. Amiloride B. Spironolactone
C. Triamterene D. Acetazolamide
6. Reason for cause of amiloride-induced hyperkalemia is
A. Electrogenic K+ channels B. Electrogenic Na+ channels
+ –
C. Nonelectrogenic Na –C1 symporter D. H+–K+–ATPase
7. Triamterene causes
A. Hypokalemia B. Muscle cramps
C. Decrease in urea level D. Better glucose tolerance
8. Which of the following is not a side effect of acetazolamide?
A. Hypokalemia B. Drowsiness
C. Diarrhea D. Paresthesia
9. Canrenone is a metabolite of
A. Ampicillin B. Spironolactone
C. Furosemide D. Acetazolamide
10. Acetazolamide can be used in all except
A. Epilepsy B. Acute mountain sickness
C. Cirrhosis D. Glaucoma
11. Furosemide causes all of the following, except
A. Hyperglycemia B. Hypomagnesemia
C. Hypokalemia D. Acidosis
12. Spironolactone is contraindicated with enalapril because it causes
A. Hyperkalemia B. Hypercalcemia
C. Hypernatremia D. Hypokalemia
13. Loop diuretics act on
A. PCT B. DCT
C. Thick ascending limb of loop of Henle D. Collecting duct

https://t.me/DentalBooksWorld
Chapter 8 • Pharmacology 729

14. Which of the following diuretics is contraindicated in the presence of cardiac failure?
A. Mannitol B. Spironolactone
C. Furosemide D. Hydrochlorothiazide
15. Site of action of ADH is
A. PCT B. DCT
C. Collecting tubule D. Ascending loop
16. K+ sparing diuretic is
A. Furosemide B. Spironolactone
C. Thiazide D. None
17. Hypercalcemia is caused by which drug?
A. Bumetanide B. Spironolactone
C. Thiazide D. Furosemide
18. Furosemide acts at
A. PCT B. DCT
C. Ascending limb of loop of Henle D. Descending limb of loop of Henle
19. Diuretics that can be used in renal failure is
A. Furosemide B. Chlorothiazide
C. Mannitol D. Chlorthalidone
20. Thiazides act on
A. PCT B. DCT
C. Glomerulus D. Ascending limb of loop of Henle

GASTROINTESTINAL TRACT
1. Drug of choice for the treatment of peptic ulcer caused due to chronic use of NSAIDs is
A. Pirenzepine B. Loxatidine
C. Misoprostol D. Esomeprazole
2. Most specific drug for the treatment of peptic ulcer disease due to chronic use of aspirin is
A. Omeprazole B. Misoprostol
C. Pirenzepine D. Ranitidine
3. M1 blocker used in peptic ulcer disease is
A. Pirenzepine B. Pyridostigmine
C. Atropine D. Oxybutynin
4. Cimetidine inhibits the metabolism of all of the following drugs, except
A. Phenytoin B. Warfarin
C. Ketoconazole D. Diazepam
5. Drug used in the treatment of gastric ulcer due to H. pylori is
A. Anticholinergics B. Carbenoxolone sodium
C. Bismuth sub citrate D. Corticosteroid
6. Which one of the following is not an antacid?
A. Magnesium sulfate B. Magaldrate
C. Magnesium carbonate D. Magnesium phosphate
7. NSAID-induced ulcers are treated by
A. Antacids B. H2 blockers
C. Misoprostol D. PPI (proton pump inhibitors)
8. Which of the following is not the effect of ranitidine as compared to cimetidine?
A. Action on H2 receptors B. Given orally
C. Used with proton pump blockers D. Anti-androgenic action

https://t.me/DentalBooksWorld
730 Triumph's Complete Review of Dentistry

9. Esomeprazole acts by inhibiting


A. H+K+ ATPase pump B. H+Na+ ATPase pump
C. H+ pump D. Any of the above
10. Antipeptic ulcer drug that can be given in patients with chronic renal failure (CRF) is
A. Aluminum hydroxide B. Magnesium hydroxide
C. Sucralfate D. None
11. Antacid drug that typically causes diarrhea
A. Sodium bicarbonate B. Magnesium hydroxide
C. Calcium bicarbonate D. Aluminum hydroxide
12. Ondansetron acts by
A. Acting on CTZ B. 5-HT3 antagonism
C. D1 and D2 receptor antagonism D. Increasing GIT motility
13. Ondansetron acts by inhibiting which of the following receptors?
A. 5-HT1 B. 5-HT2
C. 5-HT3 D. 5-HT4
14. Which of the following proton pump inhibitor has enzyme inhibitory activity?
A. Rabeprazole B. Lansoprazole
C. Pantoprazole D. Omeprazole

ANSWERS

HISTORY OF PHARMACOLOGY
1. Oswald Schmiedeberg
2. Col. Ram Nath Chopra
3. Sir James Black
4. Banting and Best
5. Sanger
6. Lundy
7. Horace Wells
8. Guedel
9. Vaughan Williams and Singh
10. Ehrlich
11. Fleming
12. Waksman
13. Cocaine (1884) for ocular anesthesia
14. Thiopentone
15. Chlorpromazine
16. Teprotide
17. Captopril
18. Streptokinase
19. Penicillin
20. PAS (followed by streptomycin)

PHARMACOLOGICAL AGENTS & AUTHORS


21. Friedrich Serturner (1805)
22. Grieger and Hessie (1833)
23. Vogt (1907)
24. P. Gleno (1908)

https://t.me/DentalBooksWorld
Chapter 8 • Pharmacology 731

25. Abel (1919)


26. Banting and Best (1922)
27. A. Fleming (1928)
28. Domagk (1932)
29. Edward C. Kendall
30. Waksman (1944)
31. Bartz (1948)
32. Duggar (1948)
33. Cade (1950)

GENERAL PHARMACOLOGY
1. Answer: B and C (Ref. Essentials of Medical Pharmacology, By Tripathi KD, 6th edition, page no. 9)
• 100% bioavailability is seen in case of IV route.
• Sterile technique is needed in case of IV and IM administration.
• Irritation and local tissue necrosis is seen in case of intradermal (ID) route.
• In inhalational route, absorption of drugs takes place from vast surfaces of alveoli; so bioavailability is high and action
is very rapid.
2. Answer: A, C, D, and E (Ref. Nutrition in Pediatrics: Basic Science, Clinical Applications, By Christopher Duggan, John B.
Watkins, W. Allan Walker, 2008, page no. 195)
• In elderly patients, the Vd is more because of increased total body fat content and decreased plasma protein binding of
drugs.
• In pediatric patients also, there is greater volume of extracellular fluid and this provides a larger volume of distribution
of highly ionized drugs. Therefore, a larger initial dose may be required to achieve the desired blood level.
• In obese patients because of greater than normal adipose content, Vd is increased.
• In pregnancy also blood volume increases about 30–40%. Although the total protein is increased, but plasma protein
concentration is decreased, thus altering Vd.
3. Answer: A, B, and C
Prodrug active form
• Enalapril
• Enalaprilat
• Dipivefrine
• Epinephrine
• Mercaptopurine
• Methylmercaptopurine
4. Answer: A, B, and C (Ref. Physiological Pharmaceutics: Barriers to Drug Absorption, By Neena Washington, Clive
Washington, Clive Wilson, 2000, page no. 166)
5. Answer: B, C, and D
6. Answer: C (Ref. Principles of Pharmacology: Workbook, By Susan E. Farrell, David E. Golan, 2008, page no. 24)
• The clinically significant implications of plasma protein binding are:
–– Plasma protein binding causes restriction of drugs in the vascular compartment and thus lower volume of distribution.
–– Longer duration of action – as the protein-bound fraction is not available for metabolism or excretion.
–– Plasma protein bound drugs tend to have more drug interactions due to displacement of a drug with lower affinity by
a drug with higher affinity for plasma proteins.
–– Hypoalbuminemia can lead to a high concentration of free drug and thus drug toxicity.
7. Answer: A
8. Answer: C
9. Answer: D
• Captopril and lisinopril are ACE inhibitors that are not prodrugs.
• Diazepam produce many active metabolites like oxazepam.
• Propranolol can produce 4-hydroxypropanolol which has β-antagonist activity.
• Allopurinol gives rise to oxypurinol which can inhibit xanthine oxidase.
10. Answer: C (Ref. Clinical Chemistry, By David White, Nigel Lawson, Paul Masters, 2016, page no. 28)

https://t.me/DentalBooksWorld
732 Triumph's Complete Review of Dentistry

11. Answer: C (Ref. Pharmaceutical Chemistry – E-Book, By David G. Watson, 2011)


12. Answer: A (Ref. Clinical Chemistry, By David White, Nigel Lawson, Paul Masters, 2016, page no. 28)
13. Answer: A
• Apparent volume of distribution (Vd) is more for drugs sequestered in tissues.
• Lipid-insoluble drugs do not enter cells, Vd approximates ECF volume.
14. Answer: B (Ref. Veterinary Toxicology: Basic and Clinical Principles, By Ramesh C. Gupta, 2011, page no. 385)
15. Answer: A
16. Answer: B (Ref. Forensic Nursing Science – E-Book, By Virginia A. Lynch, Janet Barber Duval, 2010, page no. 110)
17. Answer: C (Ref. Essentials of Medical Pharmacology, By KD Tripathi, 2013, page no. 146)
18. Answer: A (Ref. Pharmacology for Dentistry, By Surender, 2007, page no. 3)
19. Answer: C
20. Answer: B
21. Answer: C (Ref. Textbook of Anesthesia for Postgraduates, By TK Agasti, 2010, page no. 346)
22. Answer: A
23. Answer: D (Ref. Pharmacology, By Bhattacharya, 2003, 2nd edition, page no. 224)
24. Answer: D (Ref. Pharmacology and Pharmacotherapeutics – E-Book, By RS Satoskar, Nirmala Rege, SD Bhandarkar, 2015,
page no. 19)
25. Answer: B
26. Answer: A (Ref. Concepts in Clinical Pharmacokinetics, By Joseph T. DiPiro, 2010, page no. 50)
27. Answer: B
28. Answer: C
29. Answer: A
Tubocurarine is not absorbed orally whereas oral bioavailability of morphine, ampicillin, and phenytoin are 24%, 62%,
and 90%, respectively.
30. Answer: C
31. Answer: A (Ref. Modern Pharmacology with Clinical Applications, By Charles R. Craig, Robert E. Stitzel, 2004,
page no. 14)
• Therapeutic index is a measure of margin of safety of a drug.
• It is defined as the ratio of median lethal dose and median effective dose.
32. Answer: B
• We can easily monitor blood glucose levels as an effect of metformin; thus TDM is not required.
• TDM is required for lithium, digitalis, phenytoin, immunosuppressants, anti-arrhythmics, etc.
33. Answer: A, C, and E
34. Answer: C (Ref. The Science and Practice of Lithium Therapy, By Gin S. Malhi, Marc Masson, Frank Bellivier, 2016,
page no. 26)
35. Answer: C
36. Answer: A
37. Answer: D (Ref. Orphan Drugs and Rare Diseases, By David C Pryde, Michael J Palmer, 2014, page no. 15)
38. Answer: D (Ref. Small Animal Critical Care Medicine – E-Book, By Deborah Silverstein, Kate Hopper, 2008, page no. viii)
39. Answer: C
40. Answer: A
41. Answer: A
42. Answer: A
43. Answer: A
44. Answer: C (Ref. Medical Pharmacology and Therapeutics – E-Book, By Derek G. Waller, Tony Sampson, 2013,
page no. 17)
45. Answer: C

ANESTHESIA
1. Answer: D (Ref. Essentials of Medical Pharmacology, By KD Tripathi, 2013, page no. 377)
2. Answer: B (Ref. Handbook of Local Anesthesia – E-Book, By Stanley F. Malamed, 2014, page no. 25)

https://t.me/DentalBooksWorld
Chapter 8 • Pharmacology 733

Classification
• Inhalation
• Gases
–– Nitrous oxide
• Volatile liquid
–– Halothane
–– Enflurane
–– Desflurane
–– Methoxy fluorene trichloroethylene
–– Ethyl chloride
–– Ether
–– Chloroform
• Intravenous
• Ultra short Barbiturate
• Nonbarbiturate
–– Benzodiazepines
–– Propofol
–– Propanidid
–– Neuroleptanalgesia
–– Etomidate
–– Ketamine
Desflurane (1,2,2,2-tetrafluoroethyl difluoromethyl ether) is a highly fluorinated methyl ethyl ether used for maintenance
of general anesthesia. Like halothane, enflurane, and isoflurane, it is a racemic mixture of (R) and (S) optical isomers
(enantiomers).
3. Answer: A (Ref. Handbook of Local Anesthesia – E-Book, By Stanley F. Malamed, 2014, page no. 25)
4. Answer: C (Ref. Handbook of Local Anesthesia – E-Book, By Stanley F. Malamed, 2014, page no. 25)
5. Answer: D (Ref. Smith’s Anesthesia for Infants and Children – E-Book, By Peter J. Davis, Franklyn P. Cladis, 2016,
page no. 200)
Sevoflurane (1,1,1,3,3,3-hexafluoro-2-(fluoromethoxy) propane) is a colorless, volatile, and nonflammable liquid with
a characteristic smell. It is stable at room temperature and has a boiling point of 58.6°C and a vapor pressure of 157
mm Hg. Hence, in contrast to desflurane, it can be used in standard vaporizers. Sevoflurane has an oil/gas partition
coefficient of 47.2 and its minimal alveolar concentration (MAC), which is the percentage that is necessary to prevent
movement in 50% of patients during skin incision, is 2.05%. As a consequence, its potency is considerably lower than
that of the older inhalational agents such as halothane and isoflurane, but it is about three times more potent than
desflurane.
  Sevoflurane has an intermediate solubility in blood and tissues and it does not cause respiratory irritation, circulatory
stimulation, or hepatotoxicity. It is particularly useful for the induction of anesthesia, and it is environmental friendly (i.e.,
it does not deplete the ozone layer). However, sevoflurane may be associated with nephrotoxicity from physical degradation
to compound A, seizures, and postoperative agitation. There is a risk of explosion and fire in the respiratory circuit of the
anesthesia machine if sevoflurane is used with a desiccated carbon dioxide absorbent. Sevoflurane has a high acquisition
cost, and flow rate limitations required to minimize exposure to compound A add to the cost of using sevoflurane.
6. Answer: B (Ref. Pediatric Anesthesia, By Bruno Bissonnette, 2014, page no. 1296)
7. Answer: C (Ref. Clinical Anesthesia, By Paul G. Barash, 2009, page no. 419)
The ideal anesthetic agent produces anesthesia while allowing the use of a high concentration of oxygen. The minimum
alveolar concentration (MAC) of an anesthetic agent at one atmosphere that abolishes movement in response to a noxious
stimulus in 50% of subjects provides the standard definition of inhaled anesthetic potency. In 30–60-year-old patients,
MAC values for halothane, isoflurane, sevoflurane, and desflurane are 0.75%, 1.15%, 1.85%, and 6.0% at one atmosphere,
respectively, which indicates that they all are potent and can be given with a high concentration of oxygen. By contrast,
the MAC for nitrous oxide is 104% at one atmosphere, and it must be given in a pressurized chamber due to safety
considerations.
8. Answer: D (Ref. Medical Pharmacology and Therapeutics – E-Book, By Derek G. Waller, Tony Sampson, 2017,
page no. 251)

https://t.me/DentalBooksWorld
734 Triumph's Complete Review of Dentistry

Solubility of an anesthetic agent in blood is quantified as the blood:gas partition coefficient, which is the ratio of the
concentration of an anesthetic in the blood phase to the concentration of the anesthetic in the gas phase when the
anesthetic is in equilibrium between the two phases. For example, the partition coefficient is 0.5 if the concentration
of an anesthetic in arterial blood is 3% and the concentration in the lungs is 6%. A low blood:gas partition coefficient
reflects a low affinity of blood for the anesthetic, a desirable property because it predicts a more precise control over the
anesthetic state and a more rapid recovery from anesthesia. The blood:gas partition coefficients for inhaled anesthetics
vary from a low of about 0.45 for nitrous oxide and desflurane and 0.65 for sevoflurane to 1.4 for isoflurane and 2.4 for
halothane.
Key points:
• Higher partition coefficient = higher lipophilicity = higher potency = higher solubility
• High solubility = more anesthetic needs to be dissolved = slower onset
• MAC decreases as blood–gas partition coefficient increases, generally speaking

Anesthetic MAC (%) Blood/Gas


Desflurane 5.8 0.42
Halothane 0.74 2.4
Isoflurane 1.15 1.4
Nitrous oxide 105 0.47
Sevoflurane 2 0.65

9. Answer: B (Ref. Veterinary Anaesthesia: Principles to Practice, By Alexandra Dugdale, 2011, page no. 70)
• Advantages:
–– Noninflammable and nonexplosive
–– Nonirritant
–– Produces bronchodilatation
–– Produces controlled hypotension
• Disadvantage:
–– Weak analgesic
–– Weak skeletal muscle relaxants
–– Uterine relaxant
–– Cardiotoxic
■■ Stage 1: Bradycardia
■■ Stage 2: Sensitize the heart to catecholamine – arrhythmia
■■ Stage 3: Direct depressant
–– Hepatotoxic
–– Malignant hyperthermia
–– Expensive
–– Effect of CVS – Decreases blood pressure, vasodilation
–– Effect of RS – depresses ventilation
–– Solubility – high
10. Answer: A (Ref. Basic and Clinical Pharmacology, By Bertram G. Katzung, 2004, page no. 407)
Effects on CVS – Decrease blood pressure and increase heart rate
Effect on RS – Depresses ventilation
11. Answer: B (Ref. Clinical Anesthesia, By Paul G. Barash, 2009, page no. 434)
12. Answer: C (Ref. Basic and Clinical Pharmacology, By Bertram G. Katzung, 2017, 14th edition, page no. 448)
13. Answer: D (Ref. Miller’s Anesthesia – E-Book, By Ronald D. Miller, Lars I. Eriksson, Lee A Fleisher, 2014, page no. 696)
14. Answer: B (Ref. Miller’s Anesthesia Review, By Lorraine M. Sdrales, Ronald D. Miller, 2012, page no. 328)

https://t.me/DentalBooksWorld
Chapter 8 • Pharmacology 735

Drug Induction Speed of Duration Effects on Effects on RS Effects on Other Comments


dose induction of action CVS CNS side effects
(mg/kg) (s) (minutes)
Propofol 1.5–2.5 30–45 4–7 Hypotension, Apnea up to Decreases Pain on injec- Non-
worse 60 seconds, CBF and tion, involun- cumulative
if hypovolemic depression of ICP tary
or cardiac ventilation movement,
disease hiccoughs
Etomidate 0.2–0.3 30–40 3–6 Relatively less Depression of Decreases Pain on Emulsion
cardiovascular ventilation CBF and injection, available,
depression ICP, involuntary less painful
anticonvul- movement,
sant hiccoughs
Thiopentone 2–6 20–30 9–10 Dose- Apnea, Decreases Rare but Delayed
dependent depression of CBF and severe adverse recov-
hypotension, ventilation ICP, anti- reactions ery after
worse if convulsant repeated
hypovolemic doses
or cardiac
disease
Ketamine 1–2 50–70 10–12 It raises all Minimal CBF Vivid, halluci- Subanes-
pressures depression of maintained, nations thetic dose
(hypertension, ventilation, prolonged causes
increased IOP, bronchodilation, anesthesia analgesia
increased laryngeal
heart rate) reflexes better
preserved
Midazolam 0.1–0.3 40–70 10–15 Dose- Depression of Mildly anti- - Cause
dependent ventilation convulsant amnesia
hypotension

15. Answer: A (Ref. Drug-Induced Neurological Disorders, By K. K. Jain, 2011, page no. 124)
16. Answer: D (Ref. Smith and Aitkenhead’s Textbook of Anaesthesia – E-Book, By Alan R. Aitkenhead, Jonathan Thompson,
David J. Rowbotham, 2013, page no. 418)

Factors which increase anesthetic requirements Factors which decrease anesthetic requirements
• Chronic EtOH • Acute EtOH
• Infant (highest MAC at 6 months) • Elderly patients
• Red hair • Hyponatremia
• Hypernatremia • Hypothermia
• Hyperthermia • Anemia (Hgb < 5 g/dL)
• Hypercarbia
• Hypoxia
• Pregnancy

17. Answer: B (Ref. A Practical Approach to Pediatric Anesthesia, By Robert S. Holzman, Thomas J. Mancuso, David M.
Polaner, 2008, page no. 38)
18. Answer: C (Ref. Pharmacology and Therapeutics for Dentistry – E-Book, By John A. Yagiela, Frank J. Dowd, Bart Johnson,
2010, page no. 479)
19. Answer: D

https://t.me/DentalBooksWorld
736 Triumph's Complete Review of Dentistry

CNS Effects of Anesthetics


Tone CMRO2 CBF ICP
NO vasodil ↑ ↑
Sevo vasodil ↓* ↑* ↑*
Des vasodil ↓ ↑* ↑*
Iso vasodil ↓ ↑* ↑*
*Because of CMRO2, CBF and ICP do not increase until MAC 1.0

20. Answer: B (Ref. Essentials of Pharmacotherapeutics, By F S K Barar, 2000, page no. 84)
Classification of Barbiturates
Barbiturates are classified into four categories:
• Long acting – Phenobarbitone
• Short acting – Butobarbitone, Pentobarbitone
• Ultrashort acting – Thiopentone, Methohexitone
21. Answer: B (Ref. Handbook of Local Anesthesia – E-Book, By Stanley F. Malamed, 2014, page no. 2)
22. Answer: C (Ref. Handbook of Local Anesthesia – E-Book, By Stanley F. Malamed, 2014, page no. 2)
23. Answer: D (Ref. Handbook of Local Anesthesia – E-Book, By Stanley F. Malamed, 2014, page no. 2)
24. Answer: A
25. Answer: B (Ref. Miller’s Anesthesia – E-Book, By Ronald D. Miller, Lars I. Eriksson, Lee A Fleisher, 2014, page no. 1696)
Amide Ester
* Lignocaine * Cocaine
* Prilocaine * Procaine
* Bupivacaine * Chloroprocaine (Shortest acting)
* Dibucaine (Longest acting) * Tetracaine (Amethocaine)
* Mepivacaine * Benzocaine
* Etidocaine
* Ropivacaine
26. Answer: C (Ref. Nurse Anesthesia – E-Book, By John J. Nagelhout, Sass Elisha, Karen Plaus, 2013, page no. 129)
27. Answer: B (Ref. Clinical Anesthesia, By Paul G. Barash, 2009, page no. 544)
28. Answer: C (Ref. Handbook of Local Anesthesia – E-Book, By Stanley F. Malamed, 2014, page no. 14)
29. Answer: D (Ref. Principles of Forensic Toxicology, By Barry Levine, 2003, page no. 208)
30. Answer: C (Ref. Patty’s Toxicology, 6 Volume Set, By Eula Bingham, Barbara Cohrssen, 2012, page no. 183)
Esters of benzoic acid – cocaine, benzocaine, and butacaine
Esters of para-aminobenzoic acid – procaine, Chloroprocaine, and propoxycaine
31. Answer: B (Ref. Textbook of Medicinal Chemistry Vol I – E-Book - Volume 1, By V Alagarsamy, 2013, page no. 151)
Classification of local anesthetics
Natural
Cocaine
Synthetic Nitrogenous Compounds
• Derivatives of para-aminobenzoic acid
–– Freely soluble – Procaine and amethocaine
–– Poorly soluble – Benzocaine and orthocaine
• Derivatives of acetanilide
–– Lidocaine
• Quinoline derivatives
–– Cinchocaine (Nupercaine)
• Acridine derivatives
–– Bucricaine (Centbucridine)

https://t.me/DentalBooksWorld
Chapter 8 • Pharmacology 737

Synthetic Nonnitrogenous compounds


Benzyl alcohol, propanediol
32. Answer: C (Ref. Dentistry at a Glance, By Elizabeth Kay, 2016, page no. 137)
33. Answer: B (Ref. Manual of Local Anaesthesia in Dentistry, By AP Chitre, 2016, page no. 56)
34. Answer: A (Ref. Handbook of Local Anesthesia – E-Book, By Stanley F. Malamed, 2014, page no. 14)
35. Answer: D
36. Answer: B (Ref. Handbook of Local Anesthesia – E-Book, By Stanley F. Malamed, 2014, page no. 11)
37. Answer: A
38. Answer: C (Ref. Nurse Anesthesia – E-Book, By John J. Nagelhout, Karen Plaus, 2009, page no. 152)
Procaine as it is an ester and it is metabolized in plasma.
Lignocaine with adrenaline should not be used for
• Areas with end arteries, e.g., for ring block of fingers, toes, penis, pinna (absolute contraindication)
• When an inhalational agent, especially halothane, which sensitizes myocardium to adrenaline is used
• Myocardial ischemic patients
• Hyperthyroid patient
• Severe hypertensives
• Intravenous regional anesthesia (Bier’s block)
39. Answer: B (Ref. Nurse Anesthesia, By John J. Nagelhout, Karen L. Plaus, 2014, page no. 143)
40. Answer: B (Ref. Local Anesthesia for the Dental Hygienist – E-Book, By Demetra D. Logothetis, 2016, page no. 33)
41. Answer: C (Ref. Nurse Anesthesia – E-Book, By John J. Nagelhout, Karen Plaus, 2009, page no. 152)
42. Answer: A (Ref. Shnider and Levinson’s Anesthesia for Obstetrics, By Samuel C. Hughes, Gershon Levinson, Mark A.
Rosen, 2002, page no. 87)
43. Answer: D
44. Answer: B (Ref. Textbook of Anesthesia for Postgraduates, By TK Agasti, 2010, page no. 760)
45. Answer: C (Ref. Basic and Clinical Pharmacology, By Bertram Katzung, 2004, page no. 422)
These drugs act by blocking the conduction of nerve impulse along the axon. Small diameter and myelinated fibers are
blocked first whereas unmyelinated and thick fibers are blocked at last. Thus, the order of blockade of fibers is B, C, Aδ
and then Aα, β, and γ. Autonomic fibers are blocked first, then sensory (cold temperature sensation is lost first followed
by heat, pain, and proprioception) and finally motor are blocked at last. Order of recovery is in the reverse order.
• Small diameter axons are more susceptible to block than large diameter fibers.
• Myelinated fibers are more sensitive than non-myelinated.
• Sequence of block is type B > type C > type A.
• In functional terms: Autonomic > sensory > motor.
• Among sensory fibers, sequence of block is pain > temperature (cold before heat) >touch > deep pressure > proprioception.
• All LAs are vasodilators except cocaine (act as sympathomimetic due to inhibition of noradrenaline reuptake) which
is a vasoconstrictor. Therefore, all LAs decrease BP except cocaine (increases).
• Cocaine should never be given by intravenous route or with adrenaline.
• Cocaine is the only ester which is not metabolized by pseudocholinesterase. It is metabolized in the liver.
• Procaine is the local anesthetic of choice in malignant hyperthermia.
Chloroprocaine is the shortest-acting local anesthetic and is contraindicated in spinal anesthesia (it may cause
paraplegia due to the presence of sodium metabisulfite as preservative, which is neurotoxic).
• All LAs if absorbed in systemic circulation can cause CNS toxicity that manifests as excitation followed by depression.
Initial excitation is due to inhibition of inhibitory neurons. Thus, LAs may lead to seizures followed by coma at high
doses.
• Dibucaine is the most potent, longest-acting and most toxic LA whereas chloroprocaine is the shortest-acting LA.
• Lignocaine is the most commonly used LA and is the drug of choice for ventricular tachycardia. It can precipitate
malignant hyperthermia due to release of calcium. Dose is limited to 7 mg/kg with adrenaline or 4 mg/kg without
adrenaline.
• Bupivacaine is the best drug for regional block but it is also the most cardiotoxic LA. Due to cardiotoxic effect, it
should not be used for Bier’s block. It is more potent and longer acting than lignocaine. Addition of adrenaline does

https://t.me/DentalBooksWorld
738 Triumph's Complete Review of Dentistry

not significantly increase the duration of action of this drug. For spinal anesthesia, 0.5% solution is made hyperbaric
with 8.25% dextrose in water. Its maximum dose is 2 mg/kg. Most common ECG changes in bupivacaine toxicity
are slow idioventricular rhythm with broad QRS complex. Bretylium is the drug of choice for bupivacaine-induced
ventricular tachycardia.
• Ropivacaine is a less cardiotoxic congener of bupivacaine.
• Prilocaine produces a metabolite “O-toluidine” which is an oxidizing agent. Latter can oxidize hemoglobin to
methemoglobin that can cause methemoglobinemia.
It is the most suitable LA for Bier’s block.
• Oxethazaine (mucaine) can be used to provide symptomatic relief in gastritis (it remains unionized in the acidic pH
of stomach)

ANTIMICROBIALS
1. Answer: D
Gentamicin is an aminoglycoside and is excreted via renal route.
2. Answer: B (Ref. The Antimicrobial Drugs, By Eric M. Scholar, Eric Michael Scholar, William B. Pratt, 2000, page no. 41)
Beta lactamases are encoded by plasmids that can be transferred with the help of bacteriophage (transduction) in
Staphylococci and by transformation in Pneumococci.
3. Answer: B
4. Answer: C
5. Answer: B
6. Answer: A
7. Answer: B
8. Answer: C
Multiple drug resistance is transferred through plasmids, mostly by conjugation.
9. Answer: D
10. Answer: B
11. Answer: D
12. Answer: A
Bactericidal drugs kill the bacteria whereas bacteriostatic drugs only inhibit bacterial growth. Bacteriostatic activity is
adequate for the treatment of most infections, bactericidal activity may be necessary for cure in patients with altered
immune systems like: neutropenias, HIV, and other immunosuppressive conditions.
13. Answer: C
14. Answer: D
• Methicillin, cloxacillin, oxacillin, and nafcillin are penicillinase-resistant penicillins.
• Piperacillin, ticarcillin, ampicillin, amoxicillin, carbenicillin, etc. are broad spectrum penicillins but these are susceptible
to penicillinase.
15. Answer: D
16. Answer: C
17. Answer: A
First-generation cephalosporins like cefadroxil are mainly effective against Gram-positive organisms and possess little
activity against Pseudomonas.
18. Answer: A
It is a fourth-generation cephalosporin; C: It possess antipseudomonal action.
• Cefepime, a fourth-generation cephalosporin is more stable against plasmid-mediated β-lactamase. It is active against
Staph aureus enterobacter and citrobacter. It possesses anti-pseudomonal activity comparable to that of ceftazidime and
Gram-positive activity similar to that of ceftriaxone.
• Cephalosporins except cefoperazone and ceftriaxone are eliminated primarily by the kidney; thus dose adjustment is
required in renal insufficiency.
• Cefepime has a short t½ (2 hours) and needs to be given 8 hourly.
19. Answer: D
• Cefepime is a fourth-generation cephalosporin.

https://t.me/DentalBooksWorld
Chapter 8 • Pharmacology 739

• Due to high potency and extended spectrum, it is effective in many serious infections like hospital-acquired pneumonia,
febrile neutropenia, bacteremia, septicemia, etc.
• All β-lactam antibiotics act by inhibiting the enzyme transpeptidase.
• Cefepime is given by IV route as it is not effective orally.
20. Answer: A
Cephalexin is an orally effective first-generation cephalosporin active against Gram-positive but not against Gram-
negative organisms like pseudomonas.
21. Answer: A
22. Answer: C
23. Answer: B
Benzathine penicillin is the longest-acting penicillin.
24. Answer: A
25. Answer: A
• Carbenicillin is a penicillin congener effective against pseudomonas and indole positive proteus which are not inhibited
by penicillin G or ampicillin/amoxicillin.
• It is inactive orally and excreted rapidly in urine. It is sensitive to penicillinase and acid, so administered parenterally as
sodium salt.
26. Answer: B
27. Answer: B
28. Answer: A
29. Answer: A
30. Answer: D
31. Answer: D
32. Answer: A
33. Answer: D
34. Answer: D
35. Answer: C
36. Answer: A
37. Answer: B
38. Answer: C
39. Answer: C
40. Answer: A
41. Answer: C
42. Answer: A
43. Answer: C
44. Answer: D
45. Answer: A
46. Answer: A
47. Answer: A
48. Answer: D
49. Answer: A
50. Answer: B
Drug of choice for prophylaxis of diphtheria is penicillin or erythromycin.
51. Answer: D
• All aminoglycosides are nephrotoxic, ototoxic, and produce curare type neuromuscular blockade.
• Doxycycline with its longer half-life and lack of nephrotoxicity (due to biliary excretion) is a popular choice for patients
with pre-existing renal disease.
52. Answer: A
Antibiotic therapy for typhoid fever
• First-line Ciprofloxacin or Ceftriaxone
• Alternative (for Nalidixic acid-resistant S. typhi) Azithromycin

https://t.me/DentalBooksWorld
740 Triumph's Complete Review of Dentistry

53. Answer: B
• Ethambutol is a bacteriostatic drug.
• INH and rifampicin are equally effective against intra- as well as extracellular mycobacteria. INH requires
a  concentration of 0.025 μg/mL whereas rifampicin inhibits the growth of bacteria at a concentration of 0.005 μg/mL.
• Pyrazinamide acts more in acidic pH and it requires a concentration of 12.5 μg/mL.
• Thus, most active drug for extracellular bacteria is rifampicin.
54. Answer: B
PAS and ethionamide can lead to hypothyroidism.
55. Answer: A
56. Answer: D
Streptomycin and ethambutol are not hepatotoxic. Read carefully, option C is ethionamide not ethambutol.
57. Answer: D
• Most atypical Mycobacteria are resistant to the usual antitubercular drugs, though pulmonary disease caused by
M. avium complex or M. kansasii may respond to prolonged treatment with rifampicin, isoniazid, and ethambutol.
• Drugs that are used are:
–– Rifabutin
–– Clofazimine
–– Quinolones, e.g., ciprofloxacin
–– Newer macrolides like clarithromycin and azithromycin
58. Answer: C
• Peripheral neuritis and a variety of neurological manifestations (paresthesias, numbness, mental disturbances, rarely
convulsions) are the most important dose-dependent toxic effects of INH.
• These are due to interference with utilization of pyridoxine and its increased excretion in urine.
59. Answer: C
• Arthralgia is caused by pyrazinamide, which may be nongouty or due to hyperuricemia secondary to inhibition of uric
acid secretion in the kidney.
• Ethambutol also produces hyperuricemia due to interference with urate excretion.
60. Answer: D
Streptomycin and capreomycin are nephrotoxic whereas ethambutol accumulates in renal failure and thus should be
avoided in the presence of severe renal failure.
61. Answer: B
62. Answer: A
63. Answer: D
• Resistance to INH occurs due to point mutation in inhA or katG genes.
• Resistance to rifampicin occurs due to point mutation in rpoB genes.
• Resistance to ethambutol is due to mutations resulting in overexpression of embB gene.
64. Answer: A
65. Answer: D
66. Answer: D
67. Answer: B
68. Answer: A
69. Answer: D
70. Answer: A
Now, the drug of choice for hepatitis B is entecavir.
71. Answer: D
72. Answer: D
73. Answer: D
74. Answer: A
Raltegravir is an integrase inhibitor used in HIV.
75. Answer: A

https://t.me/DentalBooksWorld
Chapter 8 • Pharmacology 741

All the drugs given in the options are microsomal enzyme inhibitors. Among protease inhibitors, ritonavir is the strongest
inhibitor of CYP3A4 enzymes whereas saquinavir is the weakest.
76. Answer: D
All drugs ending with navir are protease inhibitors. Abacavir is an NRTI.
77. Answer: A
• All NRTIs can cause pancreatitis and peripheral neuropathy.
• Maximum risk of pancreatitis is associated with didanosine and maximum incidence of peripheral neuropathy is seen
with stavudine.
• Lamivudine is safest NRTI as it has a minimum risk of pancreatitis and peripheral neuropathy.

AUTONOMIC NERVOUS SYSTEM


1. Answer: B (Ref. Concise Textbook of Physiology for Dental Students, By Tripathi, 2011, page no. 508)
2. Answer: C (Ref. Pharmacology, By Gary C. Rosenfeld, David S. Loose, 2007, page no. 29)
3. Answer: A
4. Answer: B (Ref. Oski’s Pediatrics: Principles & Practice, By Julia A. McMillan, Ralph D. Feigin, Catherine DeAngelis, 2006,
page no. 766)
5. Answer: D (Ref. Brenner and Stevens’ Pharmacology – E-Book, By George M. Brenner, Craig Stevens, 2017, page no. 64)
6. Answer: C (Ref. Textbook of Pharmacology, By Seth, 2009, page no. 16)
7. Answer: B (Ref. Drill’s Pharmacology in medicine, By Victor Alexander Drill, Joseph R. DiPalma, page no. 590)
8. Answer: C
9.  Answer: C (Ref. Advanced Emergency Medical Technician Transition Manual, By AAOS (American Academy of
Orthopaedic Surgeons Staff), Rhonda Beck, 2013)
10. Answer: A (Ref. Elsevier Comprehensive Guide, 2009, page no. 446)
11. Answer: A (Ref. Anesthesiology and Otolaryngology, By Adam I. Levine, Satish Govindaraj, Samuel DeMaria, Jr., 2013,
page no. 42)
12. Answer: D (Ref. Xerostomia: New Insights for the Healthcare Professional, 2013)
13. Answer: B
14. Answer: C (Ref. Growth Factors and Epilepsy, By Devin K. Binder, Helen E. Scharfman, Brenda J. Anderson, 2005, page
no. 87)
15. Answer: A (Ref. Rau’s Respiratory Care Pharmacology – E-Book, By Douglas S. Gardenhire, 2015, page no. 277)
16. Answer: B (Ref. Rau’s Respiratory Care Pharmacology – E-Book, By Douglas S. Gardenhire, 2015, page no. 84)
17. Answer: B (Ref. Textbook Of Medicinal Chemistry – Volume 1, By Algar, 2010, page no. 379)
18. Answer: C (Ref. Foundations of Biochemical Psychiatry, By David S. Segal, Joel Yager, John L Sullivan, 2013, page no. 234)
19. Answer: D (Ref. Becker-Shaffer’s Diagnosis and Therapy of the Glaucomas, By Robert L. Stamper, Marc F. Lieberman,
Michael V. Drake, 2009, page no. 424)
20. Answer: A (Ref. Pharmacodynamic Basis of Herbal Medicine, By Manuchair Ebadi, 2010, page no. 430)
21. Answer: B
22. Answer: C (Ref. Basic and Clinical Pharmacology 12/E, By Bertram G. Katzung, Susan Masters, Anthony Trevor, 2012,
page no. 107)
23. Answer: D
24. Answer: A (Ref. Glaucoma E-Book, By Tarek M. Shaarawy, Mark B. Sherwood, Roger A. Hitchings, 2014, page no. 577)
25. Answer: C (Ref. Basic and Clinical Pharmacology, By Bertram G. Katzung, Susan B. Masters, Anthony J. Trevor, 2012)
26. Answer: B (Ref. Modern Pharmacology with Clinical Applications, By Charles R. Craig, Robert E. Stitzel, 2004,
page  no.  130)
27. Answer: B
28. Answer: C (Ref. Miller’s Anesthesia Review: Expert Consult – Online and Print, By Lorraine M. Sdrales, Ronald D. Miller,
2012, page no. 100)
29. Answer: A (Ref. Clinical Anesthesia, By Paul G. Barash, 2009, page no. 346)
30. Answer: C (Ref. Clinical Anesthesia, By Paul G. Barash, 2009, page no. 346)
31. Answer: D (Ref. Cardiac Functioning, Disorders, Challenges and Therapies, By Mahira Parveen, 2013, page no. 127)

https://t.me/DentalBooksWorld
742 Triumph's Complete Review of Dentistry

32. Answer: B
33. Answer: A
34. Answer: B
35. Answer: C (Ref. Principles of Pediatric and Neonatal Emergencies, By Panna Choudhury, Arvind Bagga, Krishan Chugh,
Siddharth Ramji, 2011, page no. 484)

AUTACOIDS
1. Answer: B (Ref. Saunders Comprehensive Review of the NAVLE – E-Book, By Patricia Schenck, 2009, page no. 98)
2. Answer: C (Ref. Oxford Textbook of Clinical Nephrology Volume 2, By Alex M. Davison, 2005, page no. 1037)
It is a prodrug of paracetamol and is commonly implicated in the causation of analgesic nephropathy.
3. Answer: D
4. Answer: B (Ref. COX-2 Inhibitor Research, By Maynard J. Howardell, 2006, page no. 51)
• COX-2 is constitutively active within kidney, endothelium, and brain. Recommended doses of COX-2 inhibitors cause
renal toxicities similar to those associated with other NSAIDs.
• COX-2 inhibitors have been shown to have less gastrointestinal side effects because COX-1 is mainly involved in
protection from gastric ulcers.
• Constitutive COX-1 isoform tends to be housekeeping in function while COX-2 is induced during inflammation.
• COX have a role in synthesis of PG’s from arachidonic acid, PGs have 20C fatty acids containing cyclopentane ring.
• Selective COX-2 inhibitors increase the risk of MI.
5. Answer: A (Ref. Clinical Pharmacology – E-Book, By Morris J. Brown, Pankaj Sharma, Fraz A. Mir, 2018, page no. 293)
6. Answer: B (Ref. Hepatic Encephalopathy: Pathophysiology and Treatment, Roger F. Butterworth, Gilles Pomier Layrargues,
2012, page no. 255)
7. Answer: B (Dementia Care: A Practical Approach, By Grahame Smith, 2016, page no. 97)
8. Answer: B
9. Answer: B (Ref. Radiology of Osteoporosis, By Stephan Grampp, 2013, page no. 31)

HEMATOLOGY
1. Answer: A (Ref. Manual of Obstetrics, By Arthur T. Evans, 2007, page no. 353)
2. Answer: A (Ref. Manual of Obstetrics, By Arthur T. Evans, 2007, page no. 353)
3. Answer: A
4. Answer: B (Ref. Rutherford’s Vascular Surgery – E-Book, By Jack L. Cronenwett, K. Wayne Johnston, 2014, page no. 589)
5. Answer: B (Ref. Handbook of Dialysis, By John T. Daugirdas, Peter G. Blake, Todd S. Ing, 2012, page no. 212)
6. Answer: A
7. Answer: C (Ref. Handbook of Dialysis, By John T. Daugirdas, Peter G. Blake, Todd S. Ing, 2012, page no. 212)
8. Answer: A (Ref. Anticoagulation, By Christian Doutremepuich, 2012, page no. 24)
9. Answer: D (Ref. The Harriet Lane Handbook – E-Book, By Johns Hopkins Hospital, Branden Engorn, Jamie Flerlage,
2014, page no. 980)
• Warfarin is an oral anticoagulant that acts by inhibiting the γ- carboxylation of glutamate residues in vitamin-K
dependent clotting factors (II, VII, IX, and X).
• It has 99% binding to albumin that result in
–– Long half-life (t1/2 = 36 hours)
–– Small volume of distribution
–– Lack of urinary excretion of unchanged drug
• It readily crosses the placenta. If given during pregnancy, it can result in “Contradi syndrome” in the fetus.
• Liver disease reduces the synthesis of clotting factors; thus increases the effect of warfarin. The dose of this drug,
therefore needs to be reduced in liver disease.
10. Answer: A (Ref. Pharmacology for Dentistry, By Tara Shanbhag, Smita Shenoy, Veena Nayak, 2017, page no. 250)
Protamine sulfate is antidote of heparin overdose whereas vitamin K is used as an antidote for warfarin toxicity.
11. Answer: A (Ref. Neurologic Aspects of Systemic Disease – Part 2, By José Biller, José M. Ferro, 2014, page no. 1131)
• Unlike unfractionated heparin, LMW heparins have more consistent SC bioavailability and thus do not require
monitoring.

https://t.me/DentalBooksWorld
Chapter 8 • Pharmacology 743

• Adverse effects of both type of heparins are similar.


• Both are contraindicated in heparin-induced thrombocytopenia where the agent of choice is direct thrombin inhibitors
like lepirudin.
12. Answer: B (Ref. Handbook of Dialysis, By John T. Daugirdas, Peter G. Blake, Todd S. Ing, 2012, page no. 212)
• Monitoring of anticoagulant effect of heparin is done by measuring aPTT
• Adverse effect of heparin include
–– Bleeding
–– Osteoporosis
–– Thrombocytopenia
–– Hypersensitivity Reactions
–– Alopecia
–– Hyperkalemia (because heparin inhibits aldosterone production in adrenal glands)
13. Answer: A (Ref. Hemostasis and Thrombosis, By Thomas G. DeLoughery, 2014, page no. 121)
Ximelagatran was the first oral direct thrombin inhibitor approved; however, it was later withdrawn because of
hepatotoxicity.
Recently a new direct thrombin inhibitor dabigatran has been approved for the prophylaxis of stroke and systemic
embolism in nonvalvular atrial fibrillation. It is administered as a prodrug – dabigatran etexilate. It is not metabolized by
CYP enzymes; however, dose adjustment is required in renal failure.
14. Answer: A (Ref. Advanced Human Nutrition, By Medeiros, Robert E. C. Wildman, 2018, page no. 300)
Vitamin K causes gamma carboxylation of glutamate residues in many clotting factors that result in their activation.
15. Answer: A (Ref. Anticoagulation, By Christian Doutremepuich, 2012, page no. 24)
Vitamin K-dependent factors are clotting factor II (prothrombin), VII, IX, and X and anticlotting factors protein C and
protein S.
16. Answer: A (Ref. Pharmacology, By Richard Finkel (PharmD.), Michelle Alexia Clark, Luigi X. Cubeddu, 2009,
page no. 233)
17. Answer: B (Ref. Applied Pharmacology for the Dental Hygienist – E-Book, By Elena Bablenis Haveles, 2015, page no. 51)
18. Answer: A (Ref. Concise Pharmacology for Physiotherapy Students 2.0, By Buch JG, page no. 347)
19. Answer: B (Ref. Mammalian Toxicology, By Mohamed Abou-Donia, 2015, page no. 223)
20. Answer: D (Ref. Essentials of Medical Pharmacology, By KD Tripathi, 2013, page no. 630)
21. Answer: C (Ref. Rutherford’s Vascular Surgery – E-Book, Ref. Jack L. Cronenwett, K. Wayne Johnston, 2014, page no. 610)
22. Answer: C (Ref. Drug Therapy in Nursing, By Diane S. Aschenbrenner, Samantha J. Venable, 2009, page no. 610)
23. Answer: B (Ref. Pharmacology for Dentistry, By Tara Shanbhag, 2017, page no. 250)

RESPIRATORY SYSTEM
1. Answer: A (Ref. Prescribing in Pregnancy, By Peter C. Rubin, Margaret Ramsey, 2008, page no. 179)
Magnesium sulfate by IV or inhalational route has been used for the treatment of acute severe asthma. All other drugs
mentioned in the options are used for prophylaxis of asthma.
2. Answer: A (Ref. Essentials of Medical Pharmacology, By KD Tripathi, 2013, page no. 228)
• Montelukast, zafirlukast, and idalukast are Cys-LT1 receptor antagonists.
• Zileuton inhibits the production of leukotrienes by inhibiting the enzyme 5-lipoxygenase.
3. Answer: C (Ref. Essentials of Medical Pharmacology, By KD Tripathi, 2013, page no. 226)
4. Answer: C (Ref. Essentials of Medical Pharmacology, By KD Tripathi, 2013, page no. 232)
• Only bronchodilator drugs are useful for the treatment of acute attack of asthma. Main drugs are:
–– Beta 2 agonists, e.g., salbutamol, terbutaline
–– Anticholinergics, e.g., ipratropium, tiotropium
–– Methylxanthines, e.g., theophylline
• In addition, steroids like hydrocortisone are used for the treatment of status asthmaticus.
• Other drugs used for asthma (like mast cell stabilizers, leukotriene receptor antagonists, and lipoxygenase inhibitors)
are indicated only for prophylaxis.
5. Answer: A (Ref. Essentials of Medical Pharmacology, By KD Tripathi, 2013, page no. 231)
• β2-Agonists like salbutamol and terbutaline can be administered by inhalational route.
• Ipratropium and tiotropium are inhalational anticholinergic agents.

https://t.me/DentalBooksWorld
744 Triumph's Complete Review of Dentistry

• Inhalational steroids include budesonide, fluticasone, beclomethasone, and flunisolide.


• Theophylline is given by oral route and is not used by inhalational route.
6. Answer: A (Ref. Drug Dosage in Renal Insufficiency, By G. Seyffart, 2012, page no. 532)
7. Answer: C (Ref. Asthma in Children, By Singh Meenu, 2011, page no. 25)
Ciclesonide has got high topical:systemic activity ratio.
8. Answer: C (Ref. Allergy Frontiers: Therapy and Prevention, By Ruby Pawankar, Stephen T. Holgate, Lanny J. Rosenwasser,
2010, page no. 238)
9. Answer: A (Ref. Nurse’s Handbook of Combination Drugs, By Jones and Bartlett, Jones and Bartlett Learning Staff, 2010,
page no. 782)
10. Answer: C (Ref. Essentials of Medical Pharmacology, By KD Tripathi, 2013, page no. 223)
11. Answer: A (Ref. Rau’s Respiratory Care Pharmacology – E-Book, By Douglas S. Gardenhire, 2015, page no. 195)
12. Answer: A (Ref. Roach’s Introductory Clinical Pharmacology, By Susan M. Ford, Sally S. Roach, 2013, page no. 328)
13. Answer: C (Ref. Essentials of Medical Pharmacology, By KD Tripathi, 2013, page no. 223)
14. Answer: B (Ref. A Manual of Adverse Drug Interactions, By J.P. Griffin, P.F. D’Arcy, 1997, page no. 251)
15. Answer: B (Ref. Textbook of Adult Emergency Medicine – E-Book, By Peter Cameron, George Jelinek, Anne-Maree Kelly,
2014, page no. 994)
16. Answer: D (Ref. Roach’s Introductory Clinical Pharmacology, By Susan M. Ford, Sally S. Roach, 2010, page no. 790)

CARDIOVASCULAR SYSTEM
1. Answer: A
2. Answer: D (Ref. Drugs for the Heart – E-Book, By Lionel H. Opie, Bernard J. Gersh, 2011, page no. 189)
3. Answer: B (Ref. Care of the Critically Ill Patient, By J. Tinker, M. Rapin, 2013, page no. 286)
Digitoxin: Therapeutic plasma conc. 15–30 ng/mL, 0.5–1.4 ng/mL
Digoxin: Toxic plasma conc. >35 ng/mL > 2.5 ng/mL
4. Answer: C (Ref. Franklin D. Roosevelt: The War Years, 1939–1945, By Roger Daniels, 2016, page no. 390)
5. Answer: A (Ref. Crush Step 1 – E-Book: The Ultimate USMLE Step 1 Review, By Theodore X. O’Connell, Ryan Pedigo,
Thomas Blair, 2017, page no. 277)
6. Answer: D (Ref. The Textbook of Emergency Cardiovascular Care and CPR, By John M. Fieldm, 2009, page no. 453)
7. Answer: B (Ref. Heart Failure and Palliative Care: A Team Approach, By Miriam Johnson, Richard Lehman, 2006,
page no. 64)
8. Answer: A (Ref. Adrenergic Activators and Inhibitors – Part 1, By Anden NE, Armstrong JM, Arnold A, Blasig J, Ellis S,
2012, page no. 202)
9. Answer: C
10. Answer: A (Ref. Moderate and Deep Sedation in Clinical Practice, By Richard D. Urman, Alan D. Kaye, 2012,
page no. 243)
11. Answer: C (Ref. Pharmacology of Antihypertensive Therapeutics, By Detlev Ganten, Patrick J. Mulrow, 2012,
page no. 336)
Both verapamil and propranolol decrease the conduction through AV node and their concomitant use can result in heart
block.
12. Answer: C (Ref. Antiarrhythmic Drugs: A Practical Guide, By Richard N. Fogoros, 2008, page no. 44)
Beta blockers are classified as class II anti-arrhythmics.
13. Answer: A (Ref. Foye’s Principles of Medicinal Chemistry, By Thomas L. Lemke, David A. Williams, 2008, page no. 808)
14. Answer: D (Ref. Handbook of Lipidology, By Tapan Ghose, 2016, page no. 109)
Rosuvastatin is the most potent statin followed by atorvastatin, pitavastatin, simvastatin, and pravastatin.
15. Answer: B (Ref. Pediatric Critical Care Medicine, By Anthony D. Slonim, Murray M. Pollack, 2006, page no. 25)
16. Answer: B (Ref. Thyroid Disorders and Diseases, An Issue of Medical Clinics – E-Book, By Kenneth Burman, 2012)
17. Answer: C (Ref. Textbook of Cardiovascular Medicine, By Eric J. Topol, Robert M. Califf, 2007, page no. 239)
18. Answer: C (Ref. Foye’s Principles of Medicinal Chemistry, By Thomas L. Lemke, David A. Williams, 2008, page no. 775)

https://t.me/DentalBooksWorld
Chapter 8 • Pharmacology 745

ENDOCRINOLOGY
1. Answer: D
2. Answer: B (Ref. Nelson Textbook of Pediatrics – Volume 2, By Robert M. Kliegman, Bonita Stanton, 2016, page no. 2678)
3. Answer: D (Ref. Pharmacology for Women’s Health, By Mary C. Brucker, Tekoa L. King, 2015, page no. 1055)
4. Answer: B
5. Answer: A (Ref. Pharmacology and Therapeutics for Dentistry – E-Book, John A. Yagiela, Frank J. Dowd, Bart Johnson,
2010, page no. 542)
6. Answer: D
Insulin inhibits the formation of ketone bodies; therefore, its deficiency can result in diabetic ketoacidosis.
7. Answer: A (Ref. Therapy of Renal Diseases and Related Disorders, By Wadi N. Suki, Shaul G. Massry, 2012, page no. 68)
Insulin results in shift of potassium into the cells and thus can result in hypokalemia.
8. Answer: D (Ref. Lehne’s Pharmacology for Nursing Care – E-Book, By Jacqueline Burchum, Laura Rosenthal, 2014,
page no. 690)
• Alpha-glucosidase inhibitors (acarbose and miglitol) reduce postprandial hyperglycemia by delaying glucose
absorption.
• This class of agents is unique because it reduces the postprandial glucose rise even in individuals with type 1 DM.
• Acarbose could be used, either as an alternative or in addition to changes in lifestyle, to delay development of type 2
diabetes in patients with impaired glucose tolerance.
• These drugs do not cause hypoglycemia.
9. Answer: C
• Insulin glargine and insulin detemir are ultralong-acting insulins.
• Insulin aspart and insulin lispro are ultrashort-acting insulins.
10. Answer: B (Ref. Endocrinology – E-Book: Adult and Pediatric, By J. Larry Jameson, Leslie J. De Groot, 2010, page no. 772)
Insulin acts by stimulation of tyrosine kinase receptors.
11. Answer: B (Ref. Closed-Loop Control of Blood Glucose, By Frederick Chee, Tyrone Fernando, 2007, page no. 45)
12. Answer: B
Human insulin has rapid absorption and shorter duration of action than pork or beef insulin.
13. Answer: A
Second-generation (like glipizide) sulfonylureas are more potent than first-generation agents (like chlorpropamide).
• Chlorpropamide is the longest-acting sulfonylurea.
• Sulfonylureas can cause hypoglycemia (even in nondiabetics) due to release of insulin.
14. Answer: D (Ref. Contemporary Medical-Surgical Nursing, By Rick Daniels, Leslie H. Nicoll, 2011, page no. 1604)
Sulfonylureas stimulate the release of insulin by the beta cells of the islets of Langerhans by blocking K+ channels.
Glucagon secretion is also reduced by sulfonylureas, but it is a minor action.
15. Answer: A (Ref. Krause’s Food and the Nutrition Care Process – E-Book, By L. Kathleen Mahan, Janice L Raymond, Sylvia
Escott-Stump, 2013, page no. 218)
• Intolerance to alcohol with flushing (disulfiram like reaction) occurs with chlorpropamide.
• Chlorpropamide, tolbutamide, tolazamide, and acetohexamide are first-generation sulfonylureas.
16. Answer: A (Ref. Contemporary Medical-Surgical Nursing, By Rick Daniels, Leslie H. Nicoll, 2011, page no. 1604)
Sulfonylureas act by
• Increasing insulin release from pancreas (not by decreasing insulin secretion), so “option B” ruled out.
• A minor action reducing glucagon and increasing somatostatin release has been demonstrated.
17. Answer: B (Ref. Pharmacology in Clinical Practice, By Richard Lancaster, 2013, page no. 398)
18. Answer: D
19. Answer: B
20. Answer: C (Ref. Pharmacology for Rehabilitation Professionals – E-Book, By Barbara Gladson, 2010, page no. 232)
21. Answer: B (Ref. Principles of Medicinal Chemistry – Volume 1, By Dr. S. S. Kadam, K. G. Bothara, 2013, page no. 202)

https://t.me/DentalBooksWorld
746 Triumph's Complete Review of Dentistry

CENTRAL NERVOUS SYSTEM


1. Answer: A (Ref. The Essence of Analgesia and Analgesics, By Raymond S. Sinatra, Jonathan S. Jahr, J. Michael Watkins-
Pitchford, 2010, page no. 182)
Head injury is an absolute contraindication to morphine use. Hypothyroidism and asthma are relative contraindications.
2. Answer: C
• Naloxone is used to reverse neonatal respiratory depression due to opioid use during labor.
• It should not be used in a patient who is dependent on opioids because it can result in withdrawal syndrome.
• When the mother is on opioids, fetus becomes opioid dependent in utero and the use of naloxone in respiratory
resuscitation may result in severe withdrawal symptoms.
• Methadone is an opioid.
3. Answer: B (Ref. Essential Clinical Anesthesia, By Charles Vacanti, Scott Segal, Pankaj Sikka, 2011, page no. 240)
Sufentanil has maximum plasma protein binding (90%) among opioids.
4. Answer: A (Ref. Essentials of Medical Pharmacology, By KD Tripathi, 2013, page no. 480)
5. Answer: A
Morphine is contraindicated in a patient with head injury.
6. Answer: C (Ref. Principles and Practice of Dialysis, By William L. Henrich, 2012, page no. 597)
• The antidote of ethylene glycol poisoning is ethanol or fomepizole.
Ethanol and fomepizole bind to alcohol dehydrogenase with higher affinity than ethylene glycol and blocks the production
of toxic metabolites.
7. Answer: B (Ref. Medical Toxicology of Drug Abuse: Synthesized Chemicals and Psychoactive Plants, Donald G. Barceloux,
2012)
8. Answer: C (Ref. Family Medicine: Principles and Practice, By Robert B. Taylor, 2013, page no. 558)
9. Answer: A (Ref. Pharmacology for Health Professionals, By Bronwen Jean Bryant, Kathleen Mary Knights, 2011,
page no. 415)
Naltrexone is an orally effective opioid antagonist. It is useful for the maintenance of the patient of opioid poisoning once
it has been treated with naloxone. Heroin is an opioid, whose poisoning can be treated by naloxone and naltrexone.
10. Answer: A (Ref. Drug Interactions in Infectious Diseases, By Stephen C. Piscitelli, Keith A. Rodvold, Manjunath P. Pai,
2011, page no. 115)
11. Answer: B (Ref. Smith’s Anesthesia for Infants and Children – E-Book, By Peter J. Davis, Franklyn P. Cladis, 2016,
page no. 459)
Opioid-induced seizures
• In animals high dose of morphine and related opioids can produce convulsions.
• Morphine excites hippocampal pyramidal cells.
• Selective δ-agonists produce similar effect.
• These action may contribute to seizures that are produced by some agents at doses only moderately higher than those
required for analgesia, especially in children. However, with most opioids seizures occur only at doses far in excess of
therapeutic dose.
• Seizures are not seen when potent μ-agonists are used.
• Naloxone is potent drug for treatment of opioid poisoning.
• Anticonvulsants (like diazepam) are not always effective in suppressing opioid-induced seizures.
So the best answer is B.
12. Answer: B (Ref. Illustrated Medical Pharmacology, By AK Kapoor, SM Raju, 2013, page no. 164)
Antabuse (disulfiram) inhibits the enzyme aldehyde dehydrogenase, which causes oxidation of aldehyde.
13. Answer: B (Ref. Essentials of Medical Pharmacology, By KD Tripathi, 2013, page no. 480)
14. Answer: C (Ref. Pain Management Secrets – E-Book, By Andrew Dubin, Julie Pilitsis, Charles E. Argoff, 2009, page no.
268)
15. Answer: B (Ref. Handbook of Biologically Active Peptides, By Abba Kastin, 2013, page no. 1283)
16. Answer: B (Ref. Emerging Pollutants: Origin, Structure, and Properties, By Francisco G. Calvo-Flores, Joaquin Isac-Garcia,
Jose A. Dobado, 2017, page no. 173)
Cocaine is a product of coca plant and it is also a stimulant like amphetamine. Both of these are addictive and
sympathomimetic substances.
17. Answer: B (Ref. The Pharmacologic Basis of Psychotherapeutics, By Louis A. Pagliaro, Ann M. Pagliaro, 2013, page no. 63)

https://t.me/DentalBooksWorld
Chapter 8 • Pharmacology 747

18. Answer: D (Ref. Essentials of Medical Pharmacology, By KD Tripathi, 2013, page no. 484)
19. Answer: B (Ref. Principles and Practice of Dialysis, By William L. Henrich, 2012, page no. 597)
20. Answer: C (Ref. Comprehensive Hospital Medicine, By Mark Williams, 2007, page no. 658)
In analgesic doses, fentanyl produces little cardiovascular effects. It has less propensity to release histamine.

DRUGS ACTING ON KIDNEY


1. Answer: C (Ref. Clinical Use of Calcium Channel Antagonist Drugs, By Lionel H. Opie, 2012, page no. 139)
Aldosterone antagonists cause gynecomastia as an adverse effect.
2. Answer: A, D and E (Ref. Pharmacology, By Gary C. Rosenfeld, David S. Loose, 2007, page no. 64)
• Carbonic anhydrase inhibitors (CAIs) are more likely to produce acidosis in patients of COPD, CRF, and DM. They are
contraindicated in COPD as they may precipitate respiratory acidosis.
• They are sulfonamides in structure, so should not be given in hypersensitive persons.
3. Answer: A, B, and C (Ref. Practical Pharmacology for the Surgical Technologist, By Teri Junge, 2016, page no. 132)
4. Answer: D (Ref. Meyler’s Side Effects of Endocrine and Metabolic Drugs, By Jeffrey K. Aronson, 2009, page no. 589)
5. Answer: B (Ref. Meyler’s Side Effects of Cardiovascular Drugs, By Jeffrey K. Aronson, 2009, page no. 255)
6. Answer: B (Ref. Meyler’s Side Effects of Cardiovascular Drugs, By Jeffrey K. Aronson, 2009, page no. 221)
• Amiloride and triamterene are K+ sparing diuretics that act by inhibiting epithelial Na+ channels.
In the distal tubules and collecting ducts, three separate channels are present (one for Na+, K+, and H+ each). Aldosterone
acts on DCT and CD to cause reabsorption of Na+. This generates a lumen negative potential difference across
the membrane of this part of the nephron. To maintain the electric neutrality, K+ and H+ are secreted in the lumen.
When amiloride and triamterene inhibits epithelial Na+ channels, transepithelial potential difference is not generated
and therefore K+ and H+ are not secreted in the lumen. Thus due to more retention of K+, amiloride may result in
hyperkalemia.
7. Answer: B (Ref. Meyler’s Side Effects of Cardiovascular Drugs, By Jeffrey K. Aronson, 2009, page no. 222)
8. Answer: C (Ref. Meyler’s Side Effects of Endocrine and Metabolic Drugs, By Jeffrey K. Aronson, 2009, page no. 589)
9. Answer: B (Ref. Current Cardiovascular Drugs, By William H. Frishman, Angela Cheng-Lai, James Nawarskas, 2005, page
no. 248)
10. Answer: C (Ref. Neuroimaging Pharmacopoeia, By Daniel Thomas Ginat, Juan E. Small, Pamela Whitney Schaefer, 2015,
page no. 309)
11. Answer: D (Ref. Meyler’s Side Effects of Endocrine and Metabolic Drugs, By Jeffrey K. Aronson, 2009, page no. 524)
12. Answer: A
13. Answer: C (Ref. Atlas of Heart Failure: Cardiac Function and Dysfunction, By Wilson S. Colucci, 2013, page no. 182)
14. Answer: A (Ref. Drug Therapy in Nursing, By Diane S. Aschenbrenner, Samantha J. Venable, 2009, page no. 486)
15. Answer: C (Ref. Renal Physiology: Principles, Structure, and Function, By Esmail Koushanpour, Wilhelm Kriz, 2013,
page no. 194)
16. Answer: B (Ref. Practical Pharmacology for the Surgical Technologist, By Teri Junge, 2016, page no. 132)
17. Answer: C (Ref. Therapy of Renal Diseases and Related Disorders, By Wadi N. Suki, Shaul G. Massry, 2012, page no. 116)
18. Answer: C (Ref. Pediatric Gastrointestinal and Liver Disease – E-Book, By Robert Wyllie, Jeffrey S. Hyams, 2010,
page no. 194)
19. Answer: A (Ref. Workbook in Practical Neonatology – E-Book, By Richard A. Polin, Mervin C. Yoder, 2014, page no. 307)
20. Answer: B (Ref. Primer on Kidney Diseases – Volume 953, By Arthur Greenberg, Alfred K. Cheung, 2005, page no. 139)

GASTROINTESTINAL TRACT
1. Answer: D (Ref. Goldman-Cecil Medicine – E-Book, By Lee Goldman, Andrew I. Schafer, 2015, page no. 253; https://
books.google.co.in/books?isbn=0323322859)
• Proton pump inhibitors are the drugs of choice for peptic ulcer disease due to any etiology.
• Misoprostol is the most specific drug for the treatment of PUD due to chronic NSAID use because it is a PGE1 analog.
2. Answer: B (Ref. Family Medicine: Principles and Practice, By Robert Taylor, 2002, page no. 519)
3. Answer: A (Ref. Pathogenesis of Functional Bowel Disease, By William J. Snape Jr., 2013, page no. 103)
4. Answer: C (Ref. Basic Pharmacology for Nurses – E-Book, By Michelle Willihnganz, Bruce D. Clayton, 2014,
page no. 526)

https://t.me/DentalBooksWorld
748 Triumph's Complete Review of Dentistry

Cimetidine is a potent inhibitor of microsomal enzymes. It prolongs the half-lives of warfarin, theophylline, phenytoin,
oral hypoglycemic agents, alcohol, and benzodiazepines.
5. Answer: C (Ref. Elsevier’s Integrated Review Pharmacology, By Mark Kester, Kelly D. Karpa, Kent E. Vrana, 2011,
page no. 176)
6. Answer: A
7. Answer: D (Ref. Yamada’s Textbook of Gastroenterology, By Daniel K. Podolsky, Michael Camilleri, J. Gregory Fitz, 2015,
page no. 1070)
8. Answer: D (Ref. Stephens’ Detection of New Adverse Drug Reactions, By John Talbot, Patrick Waller, 2004, page no. 632)
9. Answer: A (Ref. Drugs in Anaesthesia and Intensive Care, By Susan Smith, Edward Scarth, 2016, page no. 282)
10. Answer: C
11. Answer: B (Ref. Pharmacology for Nursing Care – E-Book, By Richard A. Lehne, Laura Rosenthal, 2014, page no. 996)
12. Answer: A
• Ondansetron blocks the depolarizing action of serotonin through 5-HT3 receptors on vagal afferents in the gut as well
as in NTS and CTZ.
• It do not block dopamine receptors (D1 and D2) or ACh receptors.
13. Answer: C (Ref. Pharmacology for Dentistry, By Tara Shanbhag, Smita Shenoy, Veena Nayak, 2017, page no. 228)
14. Answer: D
• Proton pump inhibitors (PPIs) are prodrugs that require activation in an acid environment.
• After absorption into the systemic circulation, the prodrug diffuses into the parietal cells of the stomach and accumulates
in the acidic secretory canaliculi. Here, it is activated by proton-catalyzed formation of a tetracyclic sulfenamide,
trapping the drug so that it cannot diffuse back across the canalicular membrane. This preferential accumulation in
areas of very low pH, such as occur uniquely in the secretory canaliculi of gastric parietal cells, means that PPI have a
specific effect on these cells.
• The activated form then binds covalently with sulfhydryl groups of cysteines in the H+, K+-ATPase, irreversibly
inactivating the pump molecule. Acid secretion resumes only after new pump molecules are synthesized and inserted
into the luminal membrane, providing a prolonged (up to 24- to 48-hour) suppression of acid secretion, despite the
much shorter plasma half-lives (0.5–2 hours) of the parent compounds.
• Because they block the final step in acid production, the proton pump inhibitors are effective in acid suppression
regardless of other stimulating factors.
• To prevent degradation of proton pump inhibitors by acid in the gastric lumen, oral dosage forms are supplied in
enteric-coated formulations. The enteric-coated tablets dissolve only at alkaline pH.
• Esomeprazole, pantoprazole, and lansoprazole are approved for intravenous administration.
• Because an acidic pH in the parietal cell acid canaliculi is required for drug activation and food stimulates acid
production, these drugs ideally should be given ~30 minutes before meals.
• Because not all pumps or all parietal cells are active simultaneously, maximal suppression of acid secretion requires
several doses of the proton pump inhibitors. For example, it may take 2–5 days of therapy with once daily dosing to
achieve the 70% inhibition of proton pumps that is seen at steady state.

https://t.me/DentalBooksWorld
9 Oral Radiology

SYNOPSIS

NORMAL RADIOGRAPHIC ANATOMY


The radiographic recognition of disease requires a sound knowledge of the radiographic appearance of normal structures. Here,
we will discuss about normal radiographic anatomical landmarks in the maxilla and mandible.

Anatomical Landmarks Common to Maxilla and Mandible


• Teeth
• Lamina dura
• Alveolar crest
• Periodontal ligament space
• Cancellous bone

Anatomical Landmarks in Maxilla


• Intermaxillary suture
• Anterior nasal spine
• Nasal aperture
• Incisive foramen
• Superior foramina of the nasopalatine canal
• Lateral fossa
• Nose
• Nasolacrimal canal
• Maxillary sinus
• Zygomatic process and zygomatic bone
• Nasolabial fold
• Pterygoid plates

Anatomical Landmarks in Mandible


• Symphysis
• Genial tubercles
• Lingual foramen
• Mental ridge
• Mental fossa
• Mental foramen
• Mandibular canal
• Nutrient canals
• Mylohyoid ridge
• Submandibular gland fossa
• External oblique ridge
• Inferior border of the mandible
• Coronoid process

https://t.me/DentalBooksWorld
750 Triumph's Complete Review of Dentistry

Anatomical Landmarks*

Structures Radiopacity or Radiolucency Diagram


Teeth Enamel – Radiopaque
Dentin – Less radiopaque than enamel
Cementum – It is not usually apparent
radiographically because the contrast
between it and dentin is so low and the
cementum layer is so thin
Lamina dura Thin radiopaque layer of dense bone

Alveolar crest Radiopaque line

Periodontal ligament Radiolucent space between tooth root and


space the lamina dura

Cancellous Bone Composed of thin radiopaque plates and


rods (trabeculae) surrounding many small
radiolucent pockets of marrow

Intermaxillary suture Thin radiolucent line

Anterior nasal spine Radiopaque

https://t.me/DentalBooksWorld
Chapter 9 • Oral Radiology 751

Nasal aperture Radiolucent

Incisive foramen Ovoid radiolucency

Superior foramina of Radiolucent


nasopalatine canal

Lateral fossa Diffusely radiolucent

Nose Slightly radiopaque appearance

Nasolacrimal canal Ovoid radiolucency

https://t.me/DentalBooksWorld
752 Triumph's Complete Review of Dentistry

Maxillary sinus Borders of maxillary sinus – Thin


radiopaque line
The sinus as such appears radiolucent
Thin radiolucent lines of uniform
width found within the maxillary sinus
are shadows of neurovascular canals or
grooves in the lateral sinus walls that
accommodate the posterior superior
alveolar canals
Consequently, on periapical radiographs of
the canine, the floors of the sinus and nasal
cavity are often superimposed and may
be seen crossing one another, forming an
inverted Y in the area – the Y line of Ennis
Y - line of Ennis
Zygomatic process and U-shaped radiopaque line with its open
zygomatic bone end directed superiorly

Nasolabial fold An oblique radiopaque line

Pterygoid plates Single radiopaque homogeneous shadow


without any evidence of trabeculation

Symphysis Radiolucent line through the midline of


the jaw between the images of forming
deciduous central incisors

https://t.me/DentalBooksWorld
Chapter 9 • Oral Radiology 753

Genial tubercles Radiopaque mass of 3–4 mm in size

Genial tubercles : radio-opaque area in the midline surrounding


the lingual foramen

Lingual foramen Single round radiolucent canal with a


well-defined opaque border lying in the
midline below the level of the apices of the
incisors

Mental ridge Occasionally can be seen as two


radiopaque lines sweeping bilaterally
forward and upward toward the midline

Mental fossa Radiolucent depression on the anterior


surface of the mandible

Mental foramen Oval radiolucency

https://t.me/DentalBooksWorld
754 Triumph's Complete Review of Dentistry

Mandibular canal Dark linear shadow (radiolucent) with


thin radiopaque superior and inferior
borders

Nutrient canals Radiolucent lines

Mylohyoid ridge Radiopaque

Submandibular gland Poorly defined radiolucency


fossa

External oblique ridge Radiopaque line

Inferior border of Broad radiopaque brand of bone


mandible

https://t.me/DentalBooksWorld
Chapter 9 • Oral Radiology 755

Coronoid process Triangular radiopacity, with its apex


directed superiorly and somewhat
anteriorly

*These can be asked in the exams as image-based questions.

Restorative Materials in Radiograph


• Restorative materials vary in their radiographic appearance, depending primarily on their thickness, density, and atomic
number. Of these atomic number is most influential.
• Silver amalgam – Completely radiopaque
• Gold – Radiopaque
• Calcium hydroxide – It is radiopaque but less radiopaque than amalgam; may be radiolucent (conventional materials –
recent ones radiopaque)
• Those materials that appear radiolucent include – Silicates, Composite, Porcelain
• Stainless steel crowns and orthopedic appliances around teeth are relatively radiopaque

RADIOPAQUE LESIONS AND THEIR INTERPRETATION


Definition
Normal radiopacity may be defined as the radiographic image of the normal anatomic structures of sufficient density, thickness,
or both to appear light or white on radiographs.
Radiopacities Common to Both Jaws
• Teeth
• Bone
• Cancellous bone
• Cortical plates
• Lamina dura
• Alveolar process

Radiolucent Structures of Maxilla


• Intermaxillary suture
• Nasal fossa
• Incisive foramen
• Lateral fossa
• Nose
• Nasolacrimal canal

Radiopaque Structures of Maxilla


• Anterior nasal spine
• Nasal septum
• Floor of nasal fossa
• Nasolabial fold
• Maxillary sinus
• Zygomatic process
• Maxillary tuberosity
• Hamular notch

https://t.me/DentalBooksWorld
756 Triumph's Complete Review of Dentistry

Radiolucent Structures of Mandible


• Symphysis
• Mental fossa
• Submandibular gland fossa
• Mental foramen
• Mandibular canal
• Nutrient canals

Radiopaque Structures of Mandible


• Genial tubercles
• Mental ridge
• Mylohyoid ridge
• External oblique ridge
• Inferior border
• Coronoid process

Radiopacities Peculiar to Maxilla


The commonly seen radiopacities of maxilla from anterior region to posterior region
1. Nasal septum and boundaries of the nasal fossae
• The nasal septum may be seen on films of the central incisors
• It is positioned superiorly to the apices of these teeth
• Appear as a wide vertical radiopaque shadow and frequently deviates slightly from the midline
• Nasal fossae are lined with compact cortical bone
• There floors may be seen extending bilaterally from the inferior limit of the septum
• They appear as linear radiopacities that curve superiorly when the lateral walls of the fossae are approached
2. Anterior nasal spine
• It is a projection of the maxilla at the lower borders of the nasal fossae
• It is seen as a small white, v-shaped, opaque shadow below the nasal septum
3. Walls and floor of the maxillary sinus
• Walls of maxillary sinus appear as white lines on the radiographs of the maxillary teeth
• Outline of sinus extends from area of canine to the tuberosity
• Floor of the sinus lies above the apices of maxillary teeth but varies widely as to extent and contour
• It is scalloped as it dips between roots to varying depths or it may be smoothly curved or flat especially in the edentulous jaws
4. Zygomatic process of maxilla and zygomatic bone
• It is seen as a U-shaped radiopaque shadow above the roots of max. 1st molar
• The inferior border of the zygomatic bone may appear on the superior aspect of maxillary molar as a dense, more or less
horizontal, extending from the zygomatic process posteriorly
5. Maxillary tuberosity
• It is a rounded projection of cancellous bone outlined by a thin layer of compact bone
• Cancellous bone may extend into the tuberosity causing this structure to appear on the radiograph as a thin shell of
cortical bone
6. Pterygoid plates and pterygoid hamulus
• Lateral pterygoid plate is wider than the medial plate and rarely seen on radiographs of max. third molar region
• Pterygoid hamular process varies in length, thickness, and density, and its tips may be seen lying above or below the level
of alveolar crest on periapical films

https://t.me/DentalBooksWorld
Chapter 9 • Oral Radiology 757

7. Coronoid process
• It is a mandibular structure that often appears on radiographs of max. third molar region
• It is cone shaped with its apex pointing upward and forward with varying contours and positions
• Sometime its radiopaque shadow has been mistaken for a root fragment in the maxilla

Radiopacities Peculiar to Mandible


1. External oblique ridge
• It is a continuation of anterior border of ramus clearly seen as a radiopaque line passing across the molar region
2. Mylohyoid ridge
• It originates on the medial portion of ramus over the lingual surface of mandible
• It is clearly seen in its posterior portion crossing retromolar and molar region inferior to and running approx. parallel to
the external oblique ridge
3. Mental ridge
• The term mental ridge is a misnomer
• Two bilateral radiopaque lines occasionally run anteriorly and superiorly from low in the premolar area toward the
midline where they meet
4. Genial tubercles
• They are located on the lingual side of the mandible slightly above the inferior border and in the midline
• They are about 3–4 mm in diameter in the midline below the incisors roots

Radiolucent Structures Peculiar to Maxilla


Intermaxillary suture
• It is also known as median suture, appears as a thin radiolucent line in the midline between the two portions of the premaxilla
• It extends from alveolar crest between the central incisors superiorly through the anterior nasal spine and continues
posteriorly between the maxillary process to the posterior aspect of the hard palate
Nasal aperture
• Because the air-filled nasal aperture (and cavity) lies just above the oral cavity, its radiolucent image is apparent in maxillary
anterior projections
• The inferior border of the nasal aperture appears as a radiopaque line extending bilaterally away from the base of anterior
nasal spine
Incisive foramen
• Also known as nasopalatine or anterior palatine foramen
• They appear as ovoid radiolucency between the roots of the central incisors
• The lateral walls of the nasopalatine canal extend from the incisive foramen to the floor of the nasal floor
Lateral fossa
• Also known an incisive fossa, it is a gentle depression in the maxilla near the apex of the lateral incisor
• It appears diffusely radiolucent
Nasolacrimal canals
• They are commonly seen as ovoid radiolucencies in the maxillary occlusal projections
• The nasal and maxillary bones form the nasolacrimal canal
• When steep vertical angulation is used, it is visible in the periapical radiograph above the region of canine

Radiolucent Structures Peculiar to Mandible


Symphysis
• It appears as a radiolucent line through the midline of the jaw between the images of forming deciduous central incisors
• This suture usually fuses by the end of first year of life, after which it is no longer radiographically apparent

https://t.me/DentalBooksWorld
758 Triumph's Complete Review of Dentistry

Lingual foramen
• They are seen in the lingual surface of the mandible, in the midline in the region of genial tubercles
• It is seen as single round radiolucency with radiopaque borders
Mental foramen
• It is the anterior limit of the inferior dental canal
• It may be round, oblong, slit-like, very irregular, and partially or completely corticated
• It is found usually in the region of the second premolar
Mandibular canal
• It is radiolucent (dark linear shadow) with superior and inferior radiopaque borders
• The canal is in close proximity to the apex of the third molar and its distance with other tooth roots increases as it passes anteriorly
Nutrient canals
• Nutrient canals carry a neurovascular bundle and appear as radiolucent lines of fairly uniform width
• They are most often seen on mandibular periapical radiographs running vertically from the inferior dental canal directly
to the apex of the tooth
• They are visible in about 5% of the patients and more frequent in blacks, males, older persons, and individuals with high
blood pressure or advanced periodontal disease

RADIATION PHYSICS
Atomic Structure
• X-rays and their ability to penetrate human tissues were discovered by Röntgen in 1895.
• He called them X-rays because their nature was then unknown.
• Atom is the fundamental unit of any particular element, i.e., the basic unit of an element.
• It is composed of a central nucleus and outer orbits which spaced at a definite distance from the nucleus and are identified
by letters – K, L, M, N, O, P, Q.
• Electrons are negatively charged particles that orbit shells.
• The central nucleus is composed of two kinds of particles: Proton – positive charged and neutrons – no charge.
• Since neutrons have no charge, the magnitude of the charge of the nucleus will depend on the number of protons (Atomic
number) which are equal to the number of electrons.

Atom States
• Ground state (stable): It is the normal or ground state of atom on which the atom is electrically neutral with equal numbers
of protons and electrons.
• Excitation state: It occurs when sufficient energy applied to the atom, results in removing of electron from its normal shell
to a higher energy shell.
• Ionization state: It is the process by which an atom loses its electrical neutrality and becomes ions by either addition or
removal of electrons. If electron is added or removed from the atom, the atom will be termed as ion. If the electron is
removed, the atom becomes a positive ion while the removed electron is called a negative ion.

MATTER AND ATOMIC STRUCTURE


• Matter is anything that has mass and occupies space. Matter occurs in three states: Solid, liquid, and gas. Atoms, the
fundamental units of matter, cannot be subdivided by chemical methods although they may be composed of many smaller
(subatomic) particles.
• There are 12 types of fundamental matter particles plus their corresponding antiparticles.
• These particles are considered to be fundamental because current experiments show that they have no inner structure and
cannot be divided.

https://t.me/DentalBooksWorld
Chapter 9 • Oral Radiology 759

Quarks
• A quark is a type of elementary particle and a fundamental constituent of matter. Quarks combine to form composite
particles called hadrons, the most stable of which are protons and neutrons, the components of atomic nuclei.
• There are six types of quarks, known as flavors: Up, down, strange, charm, top, and bottom.

Proton A proton is a subatomic particle, symbol p or p+, with a positive electric charge of +1e elementary charge and mass
slightly less than that of a neutron.
Neutron The neutron is a subatomic particle, symbol n or n0, with no net electric charge and a mass slightly larger than that
of a proton.
Lepton A lepton is an elementary, half-integer spin (spin 1​ ⁄2) particle that does not undergo strong interactions. Two
main classes of leptons exist: Charged leptons (also known as the electron-like leptons) and neutral leptons (better
known as neutrinos).

• These fundamental particles consist of six types of quarks and six types of leptons and their antiparticles (particles having an
opposite charge but otherwise identical to quarks and leptons).
• Quarks only exist in association with other quarks, never as solitary particles.
• Neutrons and protons are made of quarks.
• Unlike quarks, leptons exist only as solitary particles.
• The stable leptons are electrons and neutrinos.

ATOMIC STRUCTURE
Nucleus
• In all atoms except hydrogen, the nucleus consists of positively charged protons and neutral neutrons. A hydrogen nucleus
contains a single proton.
• Protons and neutrons in turn are made of quarks.

Protons (with a Consist of two up quarks (charge A proton is composed of 2 up A neutron is composed of 1 up
charge of 1) 2/3 each) and one down quark quarks (u) and 1 down quark (d). quark (u) and 2 down quarks (d).
(charge - 1/3).
Neutrons Are made of one up quark and
two down quarks and thus are
d u
neutral.

u u d d

Total charge: Total charge:


+ 2/3 + 2/3 - 1/3 = +1 + 2/3 - 1/3 - 1/3 = 0

According to the wavelengths, radiations can differ in their properties.


Radiation may be of short wavelength or long wave length
The short wavelength increases frequency, which increases the energy accompanied with it, and in turn increases the power
of penetration; these rays will be termed Hard radiation, which is characterized with low power of absorption into matter and
low ionization.

https://t.me/DentalBooksWorld
760 Triumph's Complete Review of Dentistry

The longer wavelength decreases frequency, which decreases the energy accompanied with it, and in turn decreases the
power of penetration; these rays are termed Soft radiation, which is characterized with high power of absorption into matter
and high ionization effects.

Examples of Electromagnetic Radiation Arranged in an Ascending Order According to Their wavelength


1. Cosmic rays.
2. Gamma ray
3. X-ray (Wavelength = 0.1−1Ao, Ao = 10-10 m
4. Ultraviolet rays
5. Visible light
6. Infrared
7. Microwaves
8. Radio, radar, TV waves

X-ray
• X-rays were first discovered in 1895 by Wilhelm Conrad Röntgen, Professor of Physics and Director of Physics Institute at
the University of Wurzburg in Bavaria.
• Hence, the term Roentgen Rays, often applied to mechanically generated X-rays. He won a Nobel prize for his discovery of X-ray.
• Roentgen called them X-rays after the mathematical symbol X for unknown.
Definition of X-ray
• It is a type of electromagnetic radiation characterized by wavelengths approximately between 1 A and 10–4 A.
• They are invisible, penetrative especially at higher photon energies, and travel with the same speed as visible light.
• They are usually produced by bombarding a target of high atomic number with fast electrons in a high vacuum.
In brief: X-rays are a form of pure energy units belonging to electromagnetic spectrum characterized by having a very short
wave length and have the ability of producing shadiness’ or images of the body tissues.

Properties of X-rays
1. They have a very short wave length: As the wavelength decreases, the power of penetration of the X-ray increase. The
power of penetration depends on several factors in addition to the wavelength such as atomic number of the object,
thickness of the object, and the density of the object.
2. They have a selective penetration, absorption power: When the X-ray hit an object, certain interactions occur; these
interactions may occur in either of three forms or possibilities: (a) Penetrate the object, (b) Absorbed by the object,
(c) Deflected from certain objects, e.g., heavy metals.
3. It affects photographic film’s emulsion: X-rays upon falling on the emulsion of a photographic film cause physical changes
producing what is termed Latent Image formation, which cannot be seen except after chemical application.
4. It causes certain substances to fluoresce: X-rays can cause certain fluorescing substances to fluoresce or emit “violet-blue visible
light” which is of a longer wave length than the X-rays so this was used in dentistry in the formation of intensifying screen.
5. They cause ionization of atoms: X-rays have the power of converting atoms into ions with the formation of ion pairs
which are electrically charged, unbalanced, nonfunctioning cells; thus will have a harmful effect later on the body cells and
fluids.
6. They have biological damaging effects: May be of somatic effects such as skin burns, erythema, or cancer or genetic
effects.
• Travel in straight lines in wave motion with the same speed of light – 3 × 108 m/sec.
• Short waves about 1/10,000 of that of light (0.1–0.001 nm)
• Invisible, cannot be felt, smelt, or heard.
• Weightless, massless, and changeless.
• They cannot be focused or collected by a lens.
• They cannot be reflected by a mirror or by fluids.
• They cannot be deviated by a magnet.
• They can deflect on heavy metals by deviated into a new linear trajectory.

https://t.me/DentalBooksWorld
Chapter 9 • Oral Radiology 761

X-ray Machine
Dental X-ray apparatus and how are X-rays created?
• When fast-moving electrons (minute particles each consisting of a negative electrical charge) collide with matter,
X-radiation is produced.
• The most efficient means of generating X-rays is an X-ray tube.
• In it, X-rays are produced by directing a high-speed stream of electrons against a metal target.
• As they strike the atoms of the target, the electrons are stopped.
• Most of their energy is transformed into heat, but a small proportion is transformed into X-rays.
X-ray machine consists of the following
The Tube
The tube is an evacuated glass tube with two arms or electrodes extending in two opposite directions, which are the cathode
and anode.
The tube is evacuated for two reasons
1. This will prevent collision of the moving electrons with the molecules of the air.
2. This evacuation will prevent oxidation and burn out of the filaments.
Cathode
It is the negative electrode of the tube, which serves as the source of electrons. It consists of two parts
a) Filament.
b) Focusing cup.
Filament
It is made of tungsten coil, which is 0.2 cm in diameter and 1 cm or less in length.
Tungsten is used because
1. It has a very high melting point so it can withstand the high temperature accompanied by the production of an X-ray.
2. It has a high atomic number, which denotes a high number of protons resulting in higher number of electrons.
Focusing cup
• It is a negatively charged concave reflector cup made of molybdenum, act as focusing the electrons to a narrow beam to fall
on the target.
• The high negative charge of the cathode repels the negatively charged electrons; thus this cup collects the electrons and
repels them till the anode attracts them.
Anode
It consisting of two main parts.
Target
It is made up of tungsten because
1. It has a very high atomic number (i.e., large number of protons and electrons).
2. It has a high melting point.
3. It has a very poor thermal conductivity.
Copper head
Due to the poor thermal conductivity of the tungsten target, it is embedded in a large block of copper, which is a good thermal
conductor so it allows proper dissipation of heat which accompanies the process of X-ray production

Production of X-rays
The principles of X-ray production
• When an electric current, composed of a steam of negatively charged electrons having kinetic energy, passes through a
filament or wire, it will be heated; so the orbiting electrons within its atoms will acquire sufficient energy to escape from
their shells. Finally, this electron cloud will be given from the heated wire of filament.
• If these electrons stopped suddenly, they will lose the accompanying kinetic energy and convert into heat and X radiation.

https://t.me/DentalBooksWorld
762 Triumph's Complete Review of Dentistry

Application of this principle on dental X-ray machine


• The step-down transformer will decrease the electric current to 8–12 V.
• This current is sufficient enough to heat the tungsten filament of the cathode and produce electrons according to the degree
of heating by thermo-ionic emission.
• These electrons will form a cloud around the cathode, which will be collected by the concave focusing cup, but they have
no velocity to move.
• The step-up transformer will raise the potential difference between the cathode and the anode by raising the voltage to 60–70 kV.
• This increase in potential difference will accelerate the electron cloud to move toward the anode, as there is a force of
attraction between the positive anode and the negative cathode.
• By the action of the focusing cup, the electrons will hit only the tungsten target of the anode, losing their kinetic energy in
a form of 99.8% heat and only 0.2% X-rays.
• The produced X-rays (primary beam) are conducted to get out from the tube housing through the filters and collimators
to be used as a useful beam.

Accessories
Filters
• A thin sheet of pure aluminum placed in the path of the X-ray beam at the end of the X-ray tube in order to improve the
quality of the beam.
• The X-ray beam is heterogeneous in characteristic, i.e., containing a ray of different energies and wavelengths. Because of
this, the filter is used in order to absorb unnecessary X-rays of the longer wavelengths being both useless in radiography and
dangerous to the patient and the dentist.
• The thickness of the filters varies according to the kVp of the machine being used – 2 mm. Aluminum thickness with up to
70 kVp and 2.5 mm over 70 kVp.
Types of filters:
• Added filters: They are external filters that can be removed or added by the clinician.
• Inherent filters: These include the glass wall of the X-ray tube, the insulating oil, and the metal housing.
Total filters = Inherent filters + Added filters
Collimator
It is a device used to limit or restrict the size of an X-ray beam just to cover the film to produce the desired image.
Types of collimators
Diaphragm collimator: A thin sheet of lead with an opening in the center.
• Tubular collimator: A tube of lead with one of its ends connected or in conjunction to the diaphragm collimator. This tube
helps in decreasing diverging rays and almost increase more parallel rays, which in turn helps to increase the quality of the
image and is more safe to the patient.
• Rectangular collimators: It provides a beam of rectangular shape that is larger than the size of periapical films.
Cones, Position Indicating device (PID)
It is a device used to
• Fix the target film distance
• Indicate the point of entry
• Determine the direction and distribution of the X-ray tube
• It is made of plastic, glass, or metal.
• The majority of cones are made of plastic because it is lightweight.
• In the case of metal one (lead), it may act as a collimator and a cone at the same time, but it is not practical as it is very heavy
on the tube itself and will cause a decrease in beam intensity as some of it will be absorbed by the lead.
• It may be of opened end or pointed end shape. The pointed end shape has harmful effects that it acts as a source of scattered
radiation as the rays hit the walls.
• It may be short— 8” or long—16”.

https://t.me/DentalBooksWorld
Chapter 9 • Oral Radiology 763

Types of X-ray Radiation


1. Primary radiation
2. Useful beam
3. Central ray
4. Secondary radiation
5. Scattered radiation
6. Stray radiation
7. Remnant radiation
8. Leakage radiation
9. Soft radiation
10. Hard radiation

Primary radiation: It is the radiation coming directly out of the target; most of it is absorbed by the tube housing except for
the useful beam.
Useful beam: It is that part of the primary radiation, which is not absorbed by the housing but passes through the apparatus
and affects the film.
Central ray: It is that part occupying the central portion of the useful beam on which the rays are relatively parallel to each
other.
Secondary radiation: It is that radiation generated from the patient’s surrounding objects due to passage, interaction of the
primary beam with these objects. They are of a long wavelength and so increased absorption and are more dangerous to the
patient.
Scattered radiation: It is a form of secondary radiation which has been deviated in direction during passage of the X-rays
through objects.
Stray radiation: This radiation occurs when the primary beam hits a metal heavier than aluminum, e.g., metallic eyeglasses.
Remnant radiation: It is that portion of radiation remaining or emerging from the object after the passage of the primary
beam through it, to expose the film and produce the image.
Leakage radiation: The radiation that escapes through the protective housing of the X-ray tube.
Soft radiation: Radiation produced by decreased kilo-voltage are of longer wavelength, decreased penetration, and increased
absorption and so have a more damaging effect.
Hard radiation: Radiation produced by increased kilo-voltage are of shorter wavelength, increased energy, increased
penetration, and decreased absorption and are the ones used to produce the image, i.e., of diagnostic value.

Terminology
Image
The representation or semblance of a structure or structures produced by passage of X-radiation, visible only when transmitted
onto a fluorescent screen or an X-ray film (in the latter case, visible only after processing the film).
Contrast
• It is the difference in density appearing on a radiograph.
• Is the differentiation between black, white, and gray shades on the radiograph.
Density
• It is the degree of darkening of exposed and processed photographic or X-ray film, expressed as the logarithm of the opacity
of a given area of the film.
Exposure
• A measure of the X-radiation to which a person or object, or a part of either, is exposed at a certain place, this measure being
based on its ability to produce ionization.

https://t.me/DentalBooksWorld
764 Triumph's Complete Review of Dentistry

Roentgen
• X-radiation has a property of causing ionization of the matter that passes through it.
• So, the unit of X-rays is Roentgen, which is the measurement of ionization.
• It is defined as the amount of radiation that passes in one c.c. air producing two billion ion pairs (negative and positive)
under standard conditions of temperature and atmospheric pressure.
Rad
It is the unit of absorbed dose; it is the amount of ionizing radiation absorbed dose by 1 g of the tissues.
Rem
• It is the unit of biological damaging effect of radiation (B.D.E)
• It is the amount of ionizing radiation that produces biologic damage effects (B.D.E) in 1 g of tissue.
• It is Roentgen equivalent mass, i.e., measurement unit denoting the amount of a radiation dose that produced biological
damaging effects equal to that in a person with one Roentgen of X-ray.

Factors Affecting the Quality of an Image


1. Kilo-voltage
2. Milliamperage
3. Collimation
4. Filtration
5. Distance
6. Atomic number and thickness (density) of the object

Kilo-voltage
• Kilo-voltage power of conventional dental X-ray machine ranges from 65 to 90 kVp.
• X-ray penetration power is controlled with kVp, i.e., the higher the kVp is, the shorter wavelength X-ray with high penetration
power.
• So kVp is the factor which determines the quality of the X-ray beam, and when the thickness of the structure is increased
we need a higher kVp.
• If the kVp is increased above the normal range it will affect the contrast of the image.
• In this case of very high kVp, the penetration power of the X-ray will increase resulting in nearly complete penetration of the
objects and finally blacking the film and the areas which should have been white (as metal) will appear gray.
• The end result of such image will be an image with black and gray shades with low contrast image (long gray scale).
• If the kVp will decrease than the normal, the penetration power of the beam will be decreased, resulting in image with white
color representing hard objects and few blacking or gray represent soft tissue objects, which is called high contrast image or
(low gray scale).
• Thus, an optimum contrast is required which is achieved by range of kVp between 65 and 90; any alterations in this range
either increase or decrease and will affect the quality of the image contrast.
Milliamperage
• The normal range of mA is about 5–15, which is affecting the quantity of the X-ray.
• By controlling mA and time, we can control the quantity of the beam and thus, control the density of the image.
• The higher mA (within normal range) will result in an increase in the quantity of the current, increase heating of the coil,
increase the amount of electrons emitted, increase the number of X-ray photons, increase the amount of X-ray reaching the
film, with final resultant of increasing the amount of blacking of the image resulting in an image with good density.
• If the mA is increased above the normal range, this will result in increasing the darkness of the image (high density), which
may controlled or avoided by decreasing the time of exposure.
• If the mA is lower than the normal range it will result in a very light image with low density, which may be controlled by
increasing the exposure time.

https://t.me/DentalBooksWorld
Chapter 9 • Oral Radiology 765

Collimation
• Collimators exert three main functions: the first, increase the safety to the patient; the second, increase the quality of the
image; and the third increase the sharpness of the image.
• It helps on reduction the amount of X-ray reached to the patient and in the same time increase the image quality by decreasing
the amount of scattered radiation.
• The image sharpness will also be increased by reduction of the beam size, leads to reduction of the more diverging rays and
increase the more parallel rays.
Filtration
• Proper filtration will provide X-rays with short wavelength, results in a good quality image.
• Over-filtration will result in decrease in the amount of X-ray photons and in decreased density image, while under-filtration
will give long wavelength X-rays with low penetration power and low contrast image.
Distance
• The distance between the source and the object may affect the image quality as follows:
–– If the distance is increased, the intensity of the beam will decrease leading to decrease in the quality of the rays and affect
the density, but at the same time if the distance is increased, it helps in the production of less diverging rays leading to an
increase in the quality of the beam and increase the sharpness.
–– If the distance is decreased, this will help in increasing the intensity of the beam and increase the density, while at the same
time it will increase the divergent rays leading to decrease in the sharpness.
Atomic number and thickness (density) of the object
• As the atomic number, density, and thickness of the object increase, the need for more powerful x-radiation will increase
to produce a good image. So the kVp should increase, but within limits, in order to not alter the contrast. So this may
compensate with not only the increase in exposure time, but also within limits in order not to affect the density.

Dental X-ray Film


• It is a thin, transparent sheet of plastic material coated on both sides with an emulsion sensitive to radiation and light.
• Radiographic films closely resemble the films used by photographers to produce black and white negatives.
• There are differences in that photographic films only carry an emulsion on one side of the film base, whereas both sides of
radiographic films are coated, to double the response to an X-ray exposure.
The X-ray film basically consists of four components:
• Film base
• Adhesive layer
• Film emulsion
• Protective layer
Film base
• It is a flexible piece of plastic, about 0.008 in thick, to provide the desired degree of stiffness and flatness for handling.
• It is transparent and has a slight blue tint to make it easier to visualize the image.
• Film base serve as a stable support for the emulsion.
Adhesive layer
• It is a thin layer of adhesive material to act as an attachment between the base and emulsion from both sides.
Film emulsion
• This is the most important constituent of the film.
• It is a homogeneous mixture of gelatin and silver halide crystals coated on both sides of the film base to provide maximum
speed to the film (sensitivity).
• Gelatin is used to suspend and evenly disperse the silver halide crystals.
• Halide crystal is a chemical compound that is sensitive to radiation or light.
• The halides used in a dental X-ray film are silver bromide and to a lesser extent silver iodide.
• On exposure to the X-ray, this silver bromide absorbs the rays and physical changes take place in the emulsion. This change
is called the latent image.

https://t.me/DentalBooksWorld
766 Triumph's Complete Review of Dentistry

Protective layer
• It is a thin, transparent, clear layer of gelatin which covers the emulsion to protect it from mechanical damage.

What is the Latent Image and How is it Formed?


• Silver bromide crystals absorb X-radiation, and store the energy of the radiation as a certain pattern to an extent depending
on the density of objects.
• This pattern of energy on the exposed film cannot be seen and is referred to as latent image.
• The latent image remains invisible within the emulsion until the film undergoes chemical processing; then it becomes
visible.
• When the X-ray hits the surface of emulsion, the silver bromide crystals that are exposed to the rays ionized are separated
to silver and bromide atoms.
• However, when the exposed film is treated with a solution called a developer, a chemical reaction takes place, and the
exposed grains of silver compound are transformed to tiny masses of black metallic silver.
• The unexposed grains are essentially unaffected. It is this silver suspended in the gelatin that constitutes the visible image
on the radiograph.

Types of Dental X-Ray Films


• Films used in dental radiography come in a variety of sizes and packaging.
• Those of the smaller sizes suitable for intraoral use, ranging from 22 to 31 mm across and from 35 to 54 mm in length, come
individually enclosed in light-tight envelopes of thin plastic or paper.
• Other films of large size are used for extraoral exposure in dental radiography. They are positioned outside the oral cavity
in a special light protected holder (cassette) that is loaded within the selected film inside the dark room.

Intraoral films
Intraoral films are usually supplied inside special film packets. The film packet consists of:
Outer protective plastic cover – open from behind – lead foil – protective black paper – within found is X-ray film – front
cover

Outer packet wrapping or envelope


• It is a soft plastic wrapper to protect the film completely from light and saliva.
• It has two sides: a white smooth side (tube side), which has a raised bump on one corner, corresponds to the identification
dot on the film.
• The other side (the label side) has a flap used to open the film packet during processing.
• It contains data about the number of films per packet, and the film speed.
• It also contains a circle of concave dots that represent the identification dot of the film.
Black paper film wrapping
• Two black papers enclose the film between them and further protect it from light.
The film
• It is a double emulsion film; the packet may contain one or two films.
• At one of the film corners there is a small raised dot (identification dot). It is used after film processing to distinguish
between the left and right sides of the patient’s mouth during reading of radiograph (interpretation).
• This identification dot or bump has convex and concave surfaces – the convex surface should face the rays while the concave
side being back to the film during exposure.
• Also, it should always be away from any anatomical landmarks to avoid being misdiagnosed as any pathologic lesion, so it
should be occlusal or incisal during exposure.

https://t.me/DentalBooksWorld
Chapter 9 • Oral Radiology 767

Lead foil sheet


• Placed back to the film away from the smooth side of the film packet (back side).
• Its function is to absorb the back-scattered radiation and thus protects the film from fogging.
• It also adds to the rigidity of the film packets.
• It has a special pattern (herring bone) stamped on the exposed finished radiograph if the film is exposed from the wrong
side (back side film).

Types of intraoral films


• Intraoral films
Intraoral films can be classified mainly according to their usage into
–– Periapical films
–– Bitewing films
–– Occlusal films
Also, intraoral films can be classified according to their
• Speed
• Size
• Number of films per packet
• Whether the film packet is lead backed or not
• Periapical radiograph

Periapical radiograph
It is the most frequently used intraoral view radiograph, which shows the entire tooth and surrounding structures on the film.
Need for prescribing periapical dental radiograph
• Extent of carious involvement in the tooth
• Interproximal decay under the contact point
• Periapical pathological changes
• Traumatic injuries to dentoalveolar process
• Periodontal diseases
• Dental anomalies
• Occult diseases
• Prognostic assessment during treatment planning
• Postobturation assessment of endodontic therapy
• Working length measurement during root canal therapy
• Implants

Radiographic Interpretation
Interpretation
A step-by-step analytical process that provides an exact idea of the clinical problem and helps to achieve the final diagnosis
of any particular lesion.
The importance of interpretation
Radiographic interpretation is an essential part of the diagnostic process. The ability to evaluate and recognize what is revealed
by a radiograph enables us to detect diseases, lesions, and conditions which cannot be identified clinically.
Steps of interpretation
• Localization
• Observation
• General consideration
• Interpretation
• Correlation
https://t.me/DentalBooksWorld
768 Triumph's Complete Review of Dentistry

Localization
• Localized or generalized
• Position in the jaw
• Single or multiple
• Size
Observation
All shadows, other than the localized shadows of the normal landmarks must be observed.
For example, shadows in crowns, cervical area, roots, restorations, size of root canals, periodontal membrane space, periapical
area, alveolar crest, foreign bodies, integrity of bone
General consideration
A radiograph shows only two dimensions of a 3-dimensional object (width and height but not the depth)
Cervical burnout: Usually appears as cervical radiolucency and misinterpreted by caries; this occurs due to less density and
more penetration of rays.
Pulp exposure: Never to be determined from a radiograph but only the proximity to the pulp.
Interpretation – features of teeth and bone
Teeth
Study the whole tooth (crown, root, enamel, pulp), number of teeth, and finally supporting structures (Periodontal membrane
space, lamina dura, alveolar crest)
Bone
Changes in bone may include:
1. Changes in density
2. Changes in the margin
3. Changes inside the lesion
4. Effect on surrounding tissues
5. Changes in structure
Correlation
The final step is to correlate all of the radiographic features to reach a radiographic differential diagnosis.
Then to draw a final diagnosis, we have to correlate other data as case history, clinical examination, and other diagnostic aids
with the radiographic differential diagnosis.

Classification of Intraoral Films According to Use


Periapical films
It is the most frequently used intraoral view, which shows the entire tooth and surrounding structure on the film.
There are three basic sizes for Periapical films:
No. 0 or child film – 22 × 35 mm
No. 1 or narrow adult – 27 × 54 mm
No. 2 or standard adult film – 31 × 41 mm
Periapical films are used to examine the following
I-Enamel:
• Normally appears as a radiopaque structure.
• Caries of the enamel: Which appears as a radiolucent area.
• Enamel hypoplasia: Appears as a radiolucent area surrounded with radiopaque margin.
• Amelogenesis imperfecta: All the enamel appears as radiolucent area.
• Congenital syphilis: Hutchinson’s incisors; appears as v-shaped radiolucent area surrounded by radiopacity.

https://t.me/DentalBooksWorld
Chapter 9 • Oral Radiology 769

II-Dentin:
• Normally appears as a radiopaque structure
• Caries of the dentin: Appears as a v-shaped radiolucent area.
• Dentinogenesis imperfecta: Dentin appears as a radiolucent area surrounded by faint radiopaque margin.
• Dense in dente: Appears as a radiopaque structure within the tooth surrounded by radiolucent margin.
• Internal resorption: Radiolucent lines on the apex or lateral side of the root dentin.
III-Pulp:
• Normally appears as a radiolucent area within the tooth.
• Calcification of the pulp: Appears as a localized area of radiopacity = pulp stone. If it is generalized it appears as a generalized
radiopacity of the pulp chamber.
• Shell tooth: Appears as a wide pulp chamber.
IV-Cementum:
• Normally it cannot be differentiated from the dentin.
• Hypercementosis: Appears as radiopaque areas cover the cementum line.
• Cementoma: Appears at the apex of the tooth as a radiolucent area in its early stages and converted into a radiopaque area
at its terminal stages.
V-Periodontal ligament space:
• Normally appears as a radiolucent line surrounding the root surface
• Narrowing of it as a result of an osteoblastic process, e.g., scleroderma
• Widening of the space as a result of osteolytic process, e.g., osteolytic osteoma
VI-Lamina dura:
• Appears as radiopaque clear continuous band that covers the alveolar bone, i.e., lining the socket and cover the crest of the
crest of alveolar bone (crestal lamina dura).
• Discontinuity of lamina dura indicate pathological changes.
VII-Alveolar bone:
• Bone resorption – either horizontal or vertical.

Bitewing films
These films often have a paper tab projecting from the middle of the film, on which the patient bites to support the film.
This tab is not visualized and does not interfere with the diagnostic quality of the image.
It is used to record the coronal portions of maxillary and mandibular teeth in one image.
The apices of the teeth are not shown.
Size Indications
31 × 41 mm Adult size commonly used for bitewing view
24 × 40 mm Views for anterior teeth
22 × 35 mm For small children
57 × 76 mm Larger films used for occlusal view
Uses of Bitewing Films
• Overhanging amalgam filling
• Detection of initial proximal caries
• Detection proximal overhanging margins of fillings and crowns
• Approximate estimation of the size of the pulp chamber and pulp horns
• Detection of initial interproximal crestal alveolar bone resorption indicating periodontal disease
• Determination of the position of permanent forming teeth in relation to deciduous ones
• Determination of any proximal calculus formation

https://t.me/DentalBooksWorld
770 Triumph's Complete Review of Dentistry

Occlusal Films
Occlusal films are use to radiographically clarify the anatomical structures and the pathological conditions of the maxilla or
mandible in the buccolingual dimension.
Occlusal films may be used for the following purposes
• Obtaining gross views for the jaws in the buccolingual dimension.
• Detection location and extent of fractures.
• Detection of the buccolingual direction of impactions and supernumerary teeth.
• Detection of buccolingual direction of displaced fracture.
• Detection of salivary gland or duct stone especially in the mandible.
• Localization of foreign bodies such as broken needle.
• Determination of the shape of dental arches.

Classification of Intraoral Films According to the Speed


• Film speed (sensitivity) can be defined as the efficiency by which a film can respond to an X-ray exposure, i.e., a fast film
requires low exposure time to produce a standard density image, while a slow film requires longer time of exposure to
produce the same standard quality.
• It refers to the amount of radiation required to produce a radiograph of standard density.
• Intraoral films vary in speed; fast films need less X-radiation and using such films routinely plays a major role in the field
of radiation protection.
Factors Affecting Film Speed
• Whether the film is coated only on one side with the silver halide grains (slow films) or on both sides (medium and fast films)
• The size of the silver halide grains – the larger the size, the more sensitive the film.
• The speed of dental X-ray films is expressed in a letter form.
• Speed groups are A, B, C, D, E, and F.
• A being the slowest film and each subsequent group being approximately twice as fast as the preceding group to give a final
image of the same object with the same density.
• This mean that, for example, E-speed film requires one-half the exposure time of D-speed film.
• Groups A and B are called slow films (regular).
• Group C is called medium speed (radiatized).
• Groups D and E are called high speed films (ultra-speed and ecta-speed, respectively).
• Groups D-speed films and E-speed films are the most common intraoral film in everyday use.
• Kodak introduced E-speed plus film; this film provides the superior image quality of D-speed film at a reduced radiation exposure.

Classification of Intraoral Films According to Size


Periapical and bitewing film comes in three sizes:
• For small children (about 22 × 35mm)
• Which is relatively narrow and used for anterior projections (about 24 × 40 mm)
• The standard film used for adults (about 32 × 41 mm)

Classification of Intraoral Films According to Number of Films per Packets


• Usually intraoral films supply in packets containing one film. Some film packets include two films instead of one.
• This may be helpful for record keeping, research purposes, teaching purposes, medico-legal aspects, or if it is meant to
control the density of each of the two films in a different way during processing.

PROJECTION GEOMETRY
When X-rays are produced at the target in an X-ray tube, they originate from all points within the area of the focal spot. Because these
rays originate from different points and travel in straight lines, their projections of a feature of an object do not occur at exactly the
same location on an image receptor. As a result, the image of the edge of an object is slightly blurred rather than sharp and distinct.

https://t.me/DentalBooksWorld
Chapter 9 • Oral Radiology 771

Three Methods to Maximize Image Sharpness


1. Use as small an effective focal spot as practical.
• Dental X-ray machines preferably should have an effective focal spot size of 0.4 mm because this greatly adds to image
clarity.
• The size of the effective focal spot is a function of the angle of the target with respect to the long axis of the electron
beam.
• A large angle distributes the electron beam over a larger surface and decreases the heat generated per unit of target area,
thus prolonging tube life; however, this results in a larger effective focal spot and loss of image clarity.
• A small angle has a greater wearing effect on the target but results in a smaller effective focal spot and increased image
sharpness.
2. Increase the distance between the focal spot and the object by using a long, open-ended cylinder.
• A longer focal spot-to-object distance minimizes blurring by using photons whose paths are almost parallel.
• The benefits of using a long focal spot-to-object distance support the use of long, open-ended cylinders as aiming devices
on dental X-ray machines.
3. Minimize the distance between the object and the image receptor.
• As the object-to-image receptor distance is reduced, the zone of unsharpness decreases, resulting in enhanced image
clarity. This is the result of minimizing the divergence of the X-ray photons.

Image Size Distortion


• Image size distortion (magnification) is the increase in size of the image on the radiograph compared with the actual size
of the object.
• The divergent paths of photons in an X-ray beam cause enlargement of the image on a radiograph. Image size distortion
results from the relative distances of the focal spot-to-image receptor and an object-to-image receptor.
• Increasing the focal spot-to-image receptor distance and decreasing the object-to-image receptor distance minimizes image
magnification.
• The use of a long, open-ended cylinder as an aiming device on an X-ray machine thus reduces the magnification of images
on a periapical view. As previously mentioned, this technique also improves image sharpness by increasing the distance
between the focal spot and the object.

Image Shape Distortion


Image shape distortion is the result of unequal magnification of different parts of the same object. This situation arises when
not all the parts of an object are at the same focal spot-to-object distance.
The physical shape of the object may often prevent its optimal orientation, resulting in some shape distortion. Such a
phenomenon is seen by the differences in appearance of the image on a radiograph compared to the true shape. To minimize
shape distortion, the practitioner should make an effort to align the tube, object, and image receptor carefully according to
the following guidelines:
1. Position the image receptor parallel to the long axis of the object. Image shape distortion is minimized when the long axes
of the image receptor and tooth are parallel. The central ray of the X-ray beam is perpendicular to the image receptor, but
the object is not parallel to the image receptor. The resultant image is distorted because of the unequal distances of the
various parts of the object from the image receptor. This type of shape distortion is called foreshortening because it causes
the radiographic image to be shorter than the object. When the X-ray beam is oriented at right angles to the object but
not to the image receptor, this results in elongation, with the object appearing longer on the image receptor than its actual
length.
2. Orient the central ray perpendicular to the object and image receptor. Image shape distortion occurs if the object and
image receptor are parallel, but the central ray is not directed at right angles to each other. This is most evident on maxillary
molar projections.

https://t.me/DentalBooksWorld
772 Triumph's Complete Review of Dentistry

Object Localization
Tube shift technique
This technique is also called Buccal object rule or Clarks rule.
• This rule governs the orientation of structure portrayed in two radiographs exposed at different angulations.
• One periapical or bitewing film is exposed using proper technique and angulations.
• A second periapical or bitewing film is then exposed after changing the direction of the X-ray beam – a different horizontal
or vertical angulations is used.
These relationships are easily remembered by the acronym – SLOB – Same Lingual Opposite Buccal. Thus if the object in
question appears to move in the same direction with respect to the reference structures as does the X-ray tube, it is lingual aspect of
the reference object; if it appears to move in the opposite direction of the X-ray tube, it is on the buccal aspect. If it does not move
with respect to the reference object, it lies at the same depth (in same vertical plane) as the reference object.
Peripheral egg shell effect
Peripheral egg shell effect in radiographs is due to difference in the travel path of photons. It accounts for why lamina dura,
the border of maxillary sinus and nasal fossa, and numerous other structures are well demonstrated on projection images. The
soft tissue masses do not show peripheral egg shell effect because they are uniform rather than being composed of dense layer
surrounding a more radiolucent interior.

IMAGE RECEPTORS AND IMAGE PROCESSING


Image Receptors: They are used in dentistry to detect X-rays.
These include:
• Radiographic film
–– Direct action or packet film
–– Indirect action film used in conjunction with intensifying screens in a cassette
• Digital receptors
–– Solid-state sensors
–– Phosphor plates

Radiographic Film
There are two basic types:
• Direct-action or nonscreen film (sometimes referred to as wrapped or packet film).
–– This type of film is sensitive primarily to X-ray photons.
• Indirect-action or screen film, so-called because it is used in combination with intensifying screens in a cassette.
–– This type of film is sensitive primarily to light photons, which are emitted by the adjacent intensifying screens
–– They respond to shorter exposure of X-rays, enabling a lower dose of radiation to be given to the patient.
Different emulsions are manufactured which are sensitive to the different colors of light emitted by different types of
intensifying screens:
• Standard silver halide emulsion sensitive to BLUE light
• Modified silver halide emulsion with ultraviolet sensitizers sensitive to ULTRAVIOLET light
• Orthochromatic emulsion sensitive to GREEN light
• Panchromatic emulsion sensitive to RED light

Characteristics of a Radiographic Film


Optical density (OD)
OD log = Incident light intensity/Transmitted light intensity
• Optical density is the term used for describing the degree of film blackening and can be measured directly using a
densitometer.
• In diagnostic radiology the range of optical densities is usually 0.25–2.5.
• There are no units for optical density
Characteristic curve
• The characteristic curve is a graph showing the variation in optical density (degree of blackening) with different exposures.

https://t.me/DentalBooksWorld
Chapter 9 • Oral Radiology 773

Background fog density


• This is the small degree of blackening evident even with zero exposure.
This is due to:
• The color/density of the plastic base
• The development of some unexposed silver halide crystals.
• If the film has been stored correctly ,background fog density should be less than 0.2
Film speed
• This is the exposure required to produce an optical density of 1.0 above background fog
Film sensitivity
• This is the reciprocal of the exposure required to produce an optical density of 1.0 above background fog.
• Thus, a fast film has a high sensitivity.
Film latitude
• This is a measure of the range of exposures that produces distinguishable differences in optical density, i.e., the linear
portion of the characteristic curve.
• The wider the film latitude the greater the range of object densities that may be seen.
Film contrast
• This is the difference in optical density between two points on a film that have received different exposures.
Film gamma and average gradient
• Film gamma is the maximum gradient or slope of the linear portion of the characteristic curve, the maximum slope
(steepest) portion of the characteristic curve is usually very short.
• Average gradient is a more useful measurement and is usually calculated between density 0.25 and 2.0 above background fog.
• Thus the film gamma or average gradient measurement determines both film latitude and film contrast as follows:
–– If the gamma or average gradient is high (i.e., a steep gradient), that film will show good contrast, but will have less latitude.
–– If the film gamma or average gradient is low (i.e., a shallow gradient), that film will show poor contrast but will have a
wider latitude.
Resolution
• Resolution, or resolving power, is a measure of the radiograph’s ability to differentiate between different structures that are
close together.
• Factors that can affect resolution include
–– Penumbra effect (image sharpness)
–– Silver halide crystal size and contrast
• It is measured in line pairs (lp) per mm.
• Direct-action film has a resolution of approximately 10 lp per mm and indirect-action film has solution of about 5 lp per mm.

Intensifying Screens
• Intensifying screens consist of fluorescent phosphors, which emit light when excited by X-rays, embedded in a plastic matrix
• Action – Two intensifying screens are used – one in front of the film and the other at the back
• The front screen absorbs the low-energy X-ray photons and the back screen absorbs the high-energy photons.
• The two screens are therefore efficient at stopping the transmitted X-ray beam, which they convert into visible light by the
photoelectric effect. The ultraviolet system was developed to improve resolution by reducing light diffusion and having
virtually no light crossover through the plastic film base.
The following terms are used to describe intensifying screens
• Conversion efficiency – the efficiency with which the phosphor converts X-rays into light
• Absorption efficiency – the ability of the phosphor material to absorb X-rays
• Screen efficiency – the ability of the light emitted by the phosphor to escape from the screen and expose the film
• Intensification factor (IF)
IF = Exposure required when screens are not used/Exposure required with screens
• Screen speed – the time taken for the screen to emit light following exposure to X-rays.
• The faster the screen, the lower the radiation dose to the patient.
• Packing density – the ability of the phosphor to pack closely together resulting in thin screens and less light divergence.

https://t.me/DentalBooksWorld
774 Triumph's Complete Review of Dentistry

Fluorescent Materials
Three main phosphor materials are, or have been, used in intensifying screens
• Rare earth phosphors including gadolinium and lanthanum
• Yttrium (a non-rare earth phosphor but having similar properties)
• Calcium tungstate (CaWO4) phosphors only fluoresce properly when they contain impurities of other phosphors,
e.g., gadolinium plus 0.3% terbium.
Typical screens include:
• Terbium-activated gadolinium oxysulfide (Gd2O1S:Tb)
• Thulium-activated lanthanum oxybromide (LaOBr:Tm)
• Terbium-activated screens emit GREEN light, while thulium-activated screens emit BLUE light
• Yttrium (Z = 39), the rare earth related phosphor, in the form of pure yttrium tantalate (YtaO4) emits ULTRAVIOLET
light
• Rare earth and related screens are approximately five times faster than calcium tungstate
Calcium tungstate screens
• This was the original material used but it is no longer recommended.
The speed of these screens depends upon:
• The thickness of the phosphor layer
• The size of the phosphor crystals
• The presence or absence of light-absorbing dyes within the screen
• The conversion efficiency of the crystals
• The faster the screen, the lower the radiation dose to the patient but the less the detail of the final image
• All calcium tungstate screens emit blue light and must be used with blue-light-sensitive monochromatic radiographic film
• Slower than rare earth screens

Digital Receptors
• There are two types of direct digital image receptors available, namely:
–– Solid state
–– Photostimulable phosphor storage plates
Uses
• Both types of sensors can be used for intraoral (periapical and bitewing radiography) and extraoral radiography including
panoramic and skull radiography.
• Only phosphor storage plates are available for occlusal and oblique lateral radiography as it is currently too expensive to
manufacture sufficiently large solid-state sensors.

IMAGE PROCESSING
Exposure Latent image created
Development Converts latent image to black metallic silver
Wash Removes excess developer
Fixing and Hardening Dissolves out unexposed silver halide crystals
Washing Removes products of processing
Dry Removes water

https://t.me/DentalBooksWorld
Chapter 9 • Oral Radiology 775

Chemical Processing
Stage 1: Development
• The sensitized silver halide crystals in the emulsion are converted to black metallic silver to produce the black/gray parts of
the image.
Stage 2: Washing
• The film is washed in water to remove residual developer solution.
Stage 3: Fixation
• The unsensitized silver halide crystals in the emulsion are removed to reveal the transparent or white parts of the image and
the emulsion is hardened.
Stage 4: Washing
• The film is washed thoroughly in running water to remove residual fixer solution.
Stage 5: Drying
• The resultant black/white/gray radiograph is dried.
Chemical Processing Methods
• Manual or wet processing
• Automatic processing
• Using self-developing films
Manual processing
Visual method
• The visual method of manual processing is carried out in a darkroom with safe lighting conditions.
• In this method, an exposed X-ray film is immersed in the developing solution and periodically viewed under the safelight
for the emergence of a clear image.
• When the image appears, the film is washed and immersed in the fixing solution.
Time–temperature method
• Time–temperature method is a type of manual processing method in which effective standardization may be achieved
without any automatic aids.
• It is a simple technique of immersing the film in the developer kept at a constant temperature for a fixed duration of time.
• The time–temperature chart is as follows:
Temperature Development time
65°F 6 minutes
68°F 5 minutes
70–72°F 4 minutes
76°F 3 minutes
• The advantage of manual processing is that the action of development is under the direct control of the operator.
• Disadvantages: Handling wet film, the requirement of a darkroom, and time consuming.
Automatic processing
• In automatic processing machines, the exposed film is fed at one end and it passes successively through the developer, fixer,
water, and drier.
• The roller system has a squeezing action; the developing solution absorbed by the gelatin of the emulsion will be less as it is
transported from the developer to the fixer.
• The automatic processing machines make use of roller system for the transport of film. The film comes out through the
other end of the processor, processed, dry, and ready for viewing.

https://t.me/DentalBooksWorld
776 Triumph's Complete Review of Dentistry

Facts to be remembered during processing


• Safelights – positioned 1.2 m from the work surfaces with 25 W bulbs and filters suitable for the type of film being used.
• The film is immersed in the developer and agitated several times in the solution to remove air bubbles and left for about
5 minutes at 20°C.
• The residual developer is rinsed off in water for about 10 seconds.
• The film is immersed in fixer for about 8–10 minutes.
• The film is washed in running water for about 10–20 minutes to remove any residual fixer.
• The developer solution is oxidized by air and its effectiveness decreased. Solutions should be used for no more than 10–14 days.
• Development time (in fresh solutions) is dependent on the temperature of the solution. The usual value recommended is
5 minutes at 20°C.

Developing Solution
Ingredient Chemical Function
Developing agent i. Hydroquinone i. Converts exposed silver halide crystals to black metallic silver. Slowly generates
ii. Elon the black tones and contrast in the image.
ii. Converts exposed silver halide crystals to black metallic silver. Quickly generates
the gray tones in the image.
Preservative Sodium sulfite Prevents rapid oxidation of the developing agents.
Accelerator Sodium carbonate Activates developer agents. Provides alkaline environment for developing agents.
Softens gelatin of the film emulsion.
Restrainer Potassium bromide Prevents the developer from developing the unexposed silver halide crystals.
Hardener Glutaraldehyde Used in automatic processing, to prevent emulsion from softening and sticking to
the rollers.
Antibacterial Fungicide It prevents bacterial growth.
Solvent Water It dissolves chemicals.

Fixing Solution
Ingredient Chemical Function
Fixing agent Sodium thiosulfate; ammonium Removes all unexposed undeveloped silver halide crystals from the
thiosulfate emulsion
Preservative Sodium sulfite Prevents deterioration of fixing agent
Hardening agent Potassium alum Shrinks and hardens the gelatin in the emulsion
Acidifier Acetic acid; sulfuric acid Neutralizes the alkaline developer and stops further development
Solvent Water It dissolves chemicals

COMPUTER DIGITAL PROCESSING


• The digital image is captured in pixels (tiny squares), by two different types of sensor – solid-state or photostimulable
phosphor plates.
• In digital imaging, each 2D pixel represents a 3D cuboid or voxel of the patient.
• The depth of the cuboid is dependent on the thickness of the part of the body being X-rayed.
• Each pixel measures the total X-ray absorption throughout the whole of each voxel.
• This 2D limitation has been overcome with the development of cone beam computed tomography.

https://t.me/DentalBooksWorld
Chapter 9 • Oral Radiology 777

Phosphor plates
• Phosphor plates are not directly connected to the computer and therefore an intermediary stage is required when the plate is read.
• The time taken to read the plate depends on the particular system being used, and the size of the plate, but typically varies
between 5 and 100 seconds.
Computer processing
• Each pixel has an x and y coordinate and is allocated a number.
• Typically using the gray-scale, there are 256 numbers to select.
• These range from 0, when the voltage received is at its maximum (no X-ray attenuation in the patient), to 255 when there is
no voltage (total X-ray attenuation in the patient).
• 256 shades of gray from black through to white, to each pixel (0 = black, 255 =white) to create the visual image on the monitor.
Advantages
• No need for chemical processing, thus avoiding all conventional processing faults and the hazards associated with handling
chemical solutions.
• Easy storage and archiving of patient information and incorporation into patient records.
• Easy transfer of images electronically.
• Image enhancement and manipulation.
• Phosphor plates have a wide latitude producing an acceptable image whether underexposed or overexposed.
Disadvantages
• Large pixels result in poor resolution and structures may not be represented accurately.
• Conventional PC screens/monitors reduce or limit image quality.
• Diagnostic image quality screens/monitors are required for optimal viewing.
• Images need to be backed up to a separate storage area remote from the image-capture computer in case this computer fails.
• Over-exposure and overloading of CCD sensors creating the phenomenon of blooming.
• Loss of image quality and resolution on hard copy printouts when using thermal, laser, or ink-jet printers.
• Image enhancement and manipulation:
–– Operators need to understand how the image is created and being altered to avoid being misled
• Time-consuming
• Magnification is achieved by enlarging the pixels, but resolution is lost

https://t.me/DentalBooksWorld
778 Triumph's Complete Review of Dentistry

RADIOBIOLOGY
Radiobiology is the study of the effects of ionizing radiations on living systems. The initial interaction between ionizing radiation
and matter occurs at the level of electron within the first 10 to the power of minus 13 second after exposure.
Radiation injury to organisms results from either the killing of large numbers of cells (deterministic effects) or sublethal damage
to individual cells that results in cancer formation or heritable mutation (Stochastic effects).

DETERMINISTIC EFFECTS
E.g., Mucositis resulting from radiation therapy to oral cavity
Radiation-induced cataract formation
Deterministic effects are characterized by:
• A threshold dose below which no effect is seen
• Worsening of the effect as dose increases over the threshold
• Always occurring once the threshold dose is reached
• Different effects, tissues, and people have different threshold doses for deterministic effects
• All early effects, and most normal tissue late effects are deterministic

STOCHASTIC EFFECTS
E.g., Radiation-induced cancer
Heritable effects
Stochastic effects account for the remaining late effects:
• They have no threshold dose
• They increase in likelihood as dose increase
• Their severity is not dose related
• There is no dose above which stochastic effects are certain to occur

Deterministic Effects on Tissues and Organs


The radiosensitivity of a tissue or organ is measured by its response to irradiation. The severity of this change depends on the
dose and thus the amount of cell loss.
Relative Radiosensitivity of Various Organs
High Intermediate Low
Lymphoid organs Fine vasculature Optic lens
Bone marrow Growing cartilage Muscle
Testes Growing bone
Intestines Salivary glands
Mucous membrane Lungs
Kidney
Liver

Radiation Effects on Oral Tissues

Tissues Effects
Oral mucous • Mucositis
membrane • White to yellow pseudomembrane – Desquamated epithelial layer
• Candida infection
• Healing of mucosa is usually complete by 2 months

https://t.me/DentalBooksWorld
Chapter 9 • Oral Radiology 779

Taste buds • Loss of taste acuity in the second or third week of radiotherapy
• Bitter and acid flavors are more severely affected when the posterior two-thirds are irradiated and salt and
sweet when the anterior two-thirds are irradiated
• Taste loss is reversible and recovery takes 60–120 days
Salivary glands • The parenchymal component of salivary gland is rather radiosensitive (parotid gland is more radiosensitive than
submandibular or sublingual glands) – Why? Because serous cells are more radiosensitive than mucous cells
• Hyposalivation is seen
• Saliva secretion is 0 at 60 Gy
• Buffering capacity of saliva falls to as much as 44% during radiation therapy
Teeth • Retarded root development
• Dwarfed teeth
• Failure to form one or more teeth
• Adult teeth are resistant to direct effects of radiation exposure
• Radiation has no discernible effects on enamel, dentin, or cementum, and radiation does not increase their
solubility
Radiation caries • Radiation caries is a rampant form of dental decay that may occur in patients who receive a course of
radiotherapy
• Increase in S. mutans, Lactobacillus, and Candida is seen
• The best method to reduce radiation caries is to apply topical 1% neutral sodium fluoride gel for 5 minutes daily
Bone • Treatment of cancers in oral region include irradiation of the mandible or maxilla
• The marrow tissue becomes hypovascular, hypoxic, and hypocellular
• Endosteum becomes atrophic, showing lack of osteoblastic and osteoclastic activity, and some lacunae of the
compact bone, an indication of necrosis
• The degree of mineralization may be reduced, leading to brittleness, or little altered from normal bone;
when these changes become more severe, they lead to bone death and the bone is exposed – the condition is
known as osteoradionecrosis
Musculature • Cause inflammation and fibrosis resulting in contracture and trismus in the muscles of mastication
• Usually the masseter or pterygoid muscles are involved
• Restriction in mouth opening usually starts about 2 months after radiotherapy

Acute Radiation Syndrome


The acute radiation syndrome is the collection of signs and symptoms experienced by persons after whole-body exposure
to radiation.
Individually, the clinical symptoms are not unique to radiation exposure, but taken as a whole, the pattern constitutes a
distinct entity.
The four stages of ARS are:
• Prodromal stage (N-V-D stage): The classic symptoms for this stage are nausea, vomiting, as well as anorexia and possibly
diarrhea (depending on dose), which occur from minutes to days following exposure. The symptoms may last (episodically)
for minutes up to several days.
• Latent stage: In this stage, the patient looks and feels generally healthy for a few hours or even up to a few weeks.
• Manifest illness stage: In this stage the symptoms depend on the specific syndrome and last from hours up to several
months.
• Recovery or death: Most patients who do not recover will die within several months of exposure. The recovery process lasts
from several weeks up to 2 years.

https://t.me/DentalBooksWorld
780 Triumph's Complete Review of Dentistry

Acute Radiation Syndrome


Dose (Gy) Manifestation
1–2 Prodromal symptoms
2–4 Mild hematopoietic symptoms
4–7 Severe hematopoietic symptoms
7–15 Gastrointestinal symptoms
50 Cardiovascular and central nervous systems symptoms

Syndrome Dose Prodromal stage Latent stage Manifest illness Recovery


stage
Hematopoietic >0.7 Gy • Symptoms are • Stem cells in • Symptoms are • In most cases,
(Bone marrow) (>70 rads) anorexia, nausea, bone marrow anorexia, fever, bone marrow
(mild and vomiting are dying, and malaise. cells will begin
symptoms • Onset occurs although • Drop in all blood to repopulate
may occur as 1 hour to 2 days patient may cell counts occurs the marrow.
low as 0.3 Gy after exposure• appear and feel for several weeks. • There should
or 30 rads) Stage lasts for well. • Primary cause of be full recovery
minutes to days • Stage lasts 1–6 death is infection for a large
weeks. and hemorrhage. percentage of
• Survival individuals from
decreases with a few weeks up
increasing dose. to 2 years after
exposure.
• Most deaths
occur within a • Death may
few months after occur in some
exposure. individuals at
1.2 Gy
(120 rads).
• The LD50/60 is
about 2.5 to 5 Gy
(250–500 rads)
Gastrointestinal (GI) >10 Gy • Symptoms are • Stem cells in • Symptoms • The LD100‡
(>1,000 rads) anorexia, severe bone marrow are malaise, is about 10 Gy
(some nausea, vomiting, and cells lining anorexia, severe (1,000 rads)
symptoms cramps, and GI tract are diarrhea, fever,
may occur as diarrhea. dying, although dehydration,
low as 6 Gy • Onset occurs patient may and electrolyte
or 600 rads) within a few hours appear and feel imbalance.
after exposure. well. • Death is due
• Stage lasts about • Stage lasts less to infection,
2 days. than 1 week. dehydration,
and electrolyte
imbalance.
• Death occurs
within 2 weeks of
exposure.

https://t.me/DentalBooksWorld
Chapter 9 • Oral Radiology 781

Cardiovascular >50 Gy • Symptoms • Patient may • Symptoms • No recovery is


(CV)/Central (5,000 rads) are extreme return to partial are return of expected.
nervous system (some nervousness and functionality. watery diarrhea,
(CNS) symptoms confusion; severe • Stage may last convulsions, and
may occur nausea, vomiting, for hours but coma.
as low as 20 and watery often is less. • Onset occurs
Gy or 2,000 diarrhea; loss of 5–6 hours after
rads) consciousness; and exposure.
burning sensations • Death occurs
of the skin. within 3 days of
• Onset occurs exposure.
within minutes of
exposure.
• Stage lasts for
minutes to hours.

Susceptibility of Different Organs to Radiation-Induced Cancer


High Intermediate Low
Colon Bladder Bone surface
Stomach Liver Brain
Lung Thyroid Salivary glands
Bone marrow (leukemia) Skin
Female breast

INTRAORAL RADIOGRAPHIC TECHNIQUES AND EXTRAORAL


RADIOGRAPHIC TECHNIQUES
INTRAORAL TECHNIQUES

Intraoral radiographic examinations are the backbone of imaging for the general dentist. Intraoral periapical radiography can
be divided into three categories:
• Techniques for Periapical radiographs
• Techniques for Bitewing radiographs
• Techniques for Occlusal radiographs

Periapical radiographic survey and examination coverage


Typical 14-Periapical film survey for adults – PERIAPICAL AND PERIODONTAL
PERIAPICAL SURVEY
This survey includes
1. Maxillary and mandibular anterior occlusal radiographs
2. Four molar periapical radiographs
3. Two posterior bitewings
PERIODONTAL SURVEY
Complete periodontal survey requires utilization of 14 periapical films.
Periapical 14-film survey for adults
The central rays is targeted onto the apex; depiction of the alveolar crest is of only secondary importance.
Periodontal 14-film survey for adults

https://t.me/DentalBooksWorld
782 Triumph's Complete Review of Dentistry

The central rays is targeted onto the alveolar crest; depiction of the root apices is only of secondary importance.

Primary Dentition (3–6 years)


• Maxillary anterior occlusal projection
• Mandibular anterior occlusal projection
• Bitewing projection
• Deciduous maxillary molar periapical projection
• Deciduous mandibular molar projection

Mixed Dentition (7–12 years)


• Maxillary anterior periapical projection
• Mandibular anterior periapical projection
• Canine periapical projection
• Deciduous and permanent molar periapical projection
• Posterior bitewing projection

Techniques for Periapical Radiography


i. Paralleling technique
ii. Bisecting angle technique
Paralleling Technique
Right angle technique – Long cone technique

Head Position
Head position for the paralleling technique is
• In the paralleling technique, the film is positioned in the mouth so that the long axis of the film and the long axis of the
tooth are parallel.
• We cannot see the long axes of the teeth but, in general, all the teeth incline toward the middle of the head.
• Thus the film/instrument will almost always be tipped slightly (up or down, depending on the arch).
• In the illustration above right, the film is placed straight up and down and is not parallel.
• The patient is unable to close completely on the bite block and the apices of the teeth would not appear on the film.

Why Long Cone Technique?


• To prevent the magnification of the image and the unsharpness of the film due to increasing the film object distance.
• A parallel nondiverging X-ray beam is required; this is achieved by increasing target film distance by using a long cone (16
inches)

https://t.me/DentalBooksWorld
Chapter 9 • Oral Radiology 783

SHARPNESS
• Measures how well the details (boundaries) of an object are reproduced on a radiograph
Increased by:
• Source–object distance
• Object–film distance
• Film crystal size
• Motion will decrease sharpness
Decreased by:
• Source–object distance
• Object–film distance
Some Important Equations
• Magnification = Target−film distance/Object−film distance
• Object Size = d (source–object distance) × i (image length)/D (source–film distance)
• Increasing mAs = increase in primary signal intensity
• Decreasing mAs = decrease in the primary signal intensity
• Increasing kVp increases the number of photons produced and also the penetration of the X-ray beam. This increase may
also cause the image to be over-exposed and appear too dark/black.
• Decreasing kVp decreases the number of photons and decreases the penetrability of the X-ray beam, causing fewer photons
to reach the target, or the receptor, and may cause the image to be under-exposed or too light.
• Distance between the source of X-ray production (which is at the target on the anode inside the tube head) and the
image receptor – Target–image receptor distance
• Distance between the object being radiographed (the teeth) and the dental X-ray image receptor (film or
digital sensor) – Object–image receptor distance

Source to Image Distance (SID)


• Source to image distance (SID) is the distance from the X-ray tube to the image receptor or focal-film distance (FFD) from
the focal spot on the anode to the receptor/film
• Increasing the distance = decreasing the beam intensity at the receptor
• Decreasing the distance = increasing the beam intensity at the receptor

Target–Film Distance
• Distance between the source of X-ray production and the film. PID is used to establish the target film distance. PID is long
or short. The shorter the target distance the more divergent the X-ray beam.
• Long target film distance has the X-rays in the center of the beam that are nearly parallel; therefore, a sharper image is
produced but also results in less magnification.
Object–Film Distance
• Object being X-rayed and the film.
• Film should always be placed as close to the teeth as possible. The closer the film to the object the sharper the image and less
magnification. The image will become fuzzy and magnified as the object–film distance is increased.
• Radiograph’s unsharpness can be minimized by reducing the size of focal spot, increasing the source–object distance, and
reducing the object–film distance.

https://t.me/DentalBooksWorld
784 Triumph's Complete Review of Dentistry

Inverse Square Law


• The intensity of an X-ray beam (the number of photons per cross-sectional area per unit of exposure time) depends on the
distance of the measuring device from the focal spot. For a given beam the intensity is inversely proportional to the square
of the distance from the source. The relationship is as follows:
• I 1/I 2 = D2 Square/D1 Square, where I is intensity and D is distance
• I 1 – Original intensity
• I 2 – New intensity
• D 1 – Original distance
• D 2 – New distance
• Therefore, if a dose of 1 Gy is measured at a distance of 2 m, a dose of 4 Gy will be found at 1 m and 0.25 Gy at 4 m.

Maxillary central incisors

Centered on contact between central and lateral incisors.

Maxillary canine

film placed far back in


patient’s mouth

Film centered on canine

https://t.me/DentalBooksWorld
Chapter 9 • Oral Radiology 785

Maxillary Premolar

Front edge of film anterior to middle of canine; approximately centered on the second premolar

Film equidistant from lingual surfaces of teeth (red arrows); this opens contacts between the teeth

Maxillary Molar

Film centered on second molar

Film equidistant from lingual surfaces of teeth (red arrows); this opens contacts between the teeth

Mandibular Incisor
Film centered on midline

https://t.me/DentalBooksWorld
786 Triumph's Complete Review of Dentistry

Film positioned away from teeth, pushing tongue back slightly

Mandibular Canine → film centered on canine

Film positioned away from teeth, pushing tongue back slightly

Mandibular Premolar

Front edge of film anterior to middle of canine; approximately centered on second premolar

Film equidistant from lingual surface of teeth (red arrows); film placed toward center of mouth,
displacing tongue

https://t.me/DentalBooksWorld
Chapter 9 • Oral Radiology 787

Mandibular Molar

centered on second molar

Film equidistant from lingual surface of teeth


In this case, the film will usually contact lingual of molars.

Bisecting Angle Technique


Bisecting line

X-ray beam

Film

X-ray Beam Perpendicular to Bisecting Line


The image on the film is equal to the length of the tooth when the central ray is directed at 90 degrees to the imaginary bisector.
A tooth and radiographic image will be equal in length when two equal triangles are formed that share a common side.
• The film is positioned with the long axis vertical and the dot-end of film extending ¼ inch beyond the incisal edge or
occlusal surface.
• With the all-white side of film facing the teeth, the finger pressure is applied at the cervical portion of the crown to avoid
film bending.
Bisecting angle film placement
Incisors → centered on midline
Cuspid → centered on cuspid
Premolar → centered on second premolar
Molar → centered on second molar
Film placement, as indicated above, is the same for maxilla or mandible.
The film is placed vertically for anterior teeth (canine to canine) and horizontal for posterior teeth.
The film is held in the proper position using the thumb (maxillary anterior, above left).
Index finger of opposite hand (all other areas, above right).

Vertical angulations during bisecting angle technique


Teeth Maxilla Mandible
Incisors 45–55 20–25

https://t.me/DentalBooksWorld
788 Triumph's Complete Review of Dentistry

Canines 45–50 15–20


Premolars 35–40 5–15
Molars 25–30 0–5
Horizontal angulations during bisecting angle technique
Central rays should be parallel to interproximal surface of the teeth
Central rays should pass through the contact area of the teeth

Anatomical Variations
Anatomical situations which might require using the bisecting angle technique are:
• A shallow palate
• A large palatal tour
• A shallow or tender floor of the mouth

Advantages of Bisecting Angle Technique


• More comfortable: Because the film is placed in the mouth at an angle to the long axis of the teeth, the film does not impinge
on the tissues as much.
• A film holder, although available, is not needed. Patients can hold the film in position using a finger.
• No anatomical restrictions: The film can be angled to accommodate different anatomical situations using this technique
Disadvantages of Bisecting Angle Technique
• More distortion: Because the film and teeth are at an angle to each other (not parallel) the images will be distorted.
• Difficult to position X-ray beam: Because a film holder is often not used it is difficult to visualize where the X-ray beam
should be directed.
• Film less stable: Using finger retention, the film has more chance of moving during placement.
• Distortion
• In the bisecting technique, the long axis of the tooth is not parallel with the long axis of the film.
• This results in a distortion of the image produced using this technique.
• In the left radiograph below, the buccal roots appear much shorter than the palatal root, even though in the actual tooth the
lengths are not that much different.
• In the other radiograph taken with the paralleling technique, the lengths are projected in their proper relationship
(minimal distortion).

Occlusal radiography
Occlusal radiography is defined as those intraoral radiographic techniques taken using a dental X-ray set where the film
packet (57 × 76 mm) or a small intraoral cassette is placed in the occlusal plane.
Maxillary occlusal projections
• Upper standard occlusal
• Upper oblique occlusal
• Vertex occlusal
Mandibular occlusal projections
• Lower 90-degree occlusal (true occlusal)
• Lower 45-degree occlusal (standard occlusal)
• Lower oblique occlusal (oblique occlusal)

https://t.me/DentalBooksWorld
Chapter 9 • Oral Radiology 789

Lower 90-degree projection


Technique and positioning
• The film packet, with the white (pebbly) surface facing downward, is placed centrally into the mouth, on to the occlusal
surfaces of the lower teeth, with its long axis crossways. The patient is asked to bite together gently.
• The patient then leans forward and then tips the head backward as far as is comfortable, where it is supported.
• The X-ray tubehead, with circular collimator fitted, is placed on an imaginary line joining the first molars, at an angle of 90
degrees to the film.
• Variation of a technique – To show a particular part of the mandible, the film packet is placed in the mouth with its long
axis anteroposteriorly over the area of interest. The X-ray tubehead, still aimed at 90 degrees to the film, is centered below
the body of the mandible in that area.
Indications
• Presence and position of radio opaque calculi in the submandibular salivary ducts
• Assessment of the buccolingual position of unerupted mandibular teeth
• Evaluation of buccolingual expansion of the body of the mandible because of cysts or tumors
• Assessment of displacement fractures of the anterior body of the mandible in the horizontal plane

90

Lower 45-degree occlusal


Technique and Positioning
• The patient is seated with the head supported and with the occlusal plane horizontal and parallel to the floor
• The film packet, with the white (pebbly) surface facing downward, is placed centrally into the mouth, on to the occlusal
surfaces of the lower teeth, with its long axis anteroposteriorly. The patient is asked to bite together gently.
• The X-ray tubehead is positioned in the midline, centering through the chin point, at an angle of 45 degrees to the film.
Indications
• Periapical assessment of the lower incisor teeth, especially useful in adults and children unable to tolerate periapical films
• Evaluation of the size and extent of lesions in the anterior region of mandible because of tumors
• Assessment of displacement of fractures of the anterior mandible in the vertical plane

45

https://t.me/DentalBooksWorld
790 Triumph's Complete Review of Dentistry

Lower oblique occlusal


• The film packet, with the white (pebbly) surface facing downward, is placed inside the mouth, on to the occlusal surfaces of
the lower teeth, with its long axis anteroposteriorly. The patient is asked to bite together gently.
• The patient’s head is supported, then rotated away from the side under investigation and the chin is raised. This rotated
positioning allows the subsequent positioning of the X-ray tubehead.
• The X-ray tubehead with a circular collimator is aimed upward and forward toward the film, from below and behind the
angle of the mandible and parallel to the lingual surface of the mandible.
Indications
• Detection of radiopaque calculi in a submandibular salivary gland
• Assessment of the buccolingual position of the lower wisdom teeth
• Evaluation of extent of tumors in the posterior part of the mandible

Extraoral radiographs
Adjuncts in extraoral radiography
• Intensifying screens
• Grids
• Digital radiography
Extraoral radiographic techniques
• Panoramic radiographs
Skull views
• PA skull
• AP skull
• PA cephalogram
• Towne’s view
• Submentovertex (base of the skull)
• Lateral skull
• Lateral cephalogram
Maxillary sinus
• PA Water’s view
• Modifications – Grenger’s view
• Caldwell’s projection
Mandible
• PA Mandible
Lateral oblique views
• Body
• Ramus
TMJ views
• Transcranial
• Transpharyngeal
• Transorbital
• Reverse Towne’s view

https://t.me/DentalBooksWorld
Chapter 9 • Oral Radiology 791

Introduction
Extraoral radiographs are taken when large areas of the skull or jaw must be examined or when patients are unable to open their
mouths for film placement.
Extraoral radiographs are very useful for evaluating large areas of the skull and jaws but are not adequate for detection of subtle
changes such as the early stages of dental caries or periodontal disease.

Intensifying Screens
Intensifying screen is used in the cassette to intensify the effect of the X-ray photon by producing a larger number of light
photons. It decreases the mAs required to produce a particular density and hence decrease the patient dose significantly.
Layers
• Base
• Reflecting layer/absorptive layer
• Phosphor: Absorbs the X-ray photon and convert it to visible light that is recorded by the film
–– Calcium tungstate (CaWO4): blue light
–– Lanthanum oxybromide (LaOBr): blue light
–– Gadolinium oxysulfide (Gd2O2S): green light
• Protective layer
• Rare earth elements are used in present day screens as they are faster and have higher absorption and conversion efficiency:
–– Gadolinium
–– Lanthanum
–– Yttrium
8 × 10 inch image receptor required for cephalometric and skull views
5 × 7 inch for oblique lateral projections
Will not exhibit details as precise as intraoral radiographs

Extraoral Radiographic Techniques


Panoramic Radiograph
• An orthopantomogram (OPG), also known as an “orthopantogram” or “panorex,” is a panoramic scanning dental X-ray of
the upper and lower jaw. It shows a two-dimensional view of a half-circle from ear to ear.
• Hudson D and Kumpula invented it.
• Patero and Numata described the principles of OPG.
• Dental panoramic radiography equipment consists of a horizontal rotating arm which holds an X-ray source and a moving
film mechanism (carrying a film) arranged at opposed extremities. The patient’s skull sits between the X-ray generator
and the film. The X-ray source is collimated toward the film, to give a beam shaped as a vertical blade having a width of
4–7 mm when arriving on the film, after crossing the patient’s skull. Also the height of that beam covers the mandibles and
the maxilla regions. The arm moves and its movement may be described as a rotation around an instant center which shifts
on a dedicated trajectory.
• Curvilinear variant of conventional tomography.
• Technique for producing a single tomographic image of facial structures that includes both maxillary and mandibular
arches and their supporting structures.
• Used on the principle of reciprocal movement of an X-ray source and an image receptor.
Indications
• Trauma
• Location of third molars
• Extensive dental or osseous disease
• Known or suspected large lesions
• Tooth development
• Retained teeth or root tips
• TMJ pain
• Dental anomalies
https://t.me/DentalBooksWorld
792 Triumph's Complete Review of Dentistry

Towne’s View
The Towne view is an angled AP radiograph of the skull.
Patient position
• The patient’s nuchal ridge is placed against the image detector
• Dorsum sella overlies the foramen magnum
• Image size: 24 × 30 cm
• Anteroposterior view with the back of patients head touching the film
• Canthomeatal line is perpendicular to the film
• Central ray is directed at 30 degrees to the canthomeatal line
X-ray beam features
• The beam travels in the anterior to posterior (AP) direction, with ~30–40 degrees of angulation from ~5 cm above the level
of the nasion, toward the foramen magnum
• Source-to-image distance: 40 inch (100 cm)
–– Cassette placed perpendicular to the floor
–– Long axis of the cassette is positioned vertically
Indications
Primarily used to observe the occipital area of skull
Necks of condylar process can also be viewed

30

Submentovertex
For this projection the neck is maximally extended and the film cassette touches the top of the head. The X-ray beam enters
the head under the chin (near the mental tubercle of the mandible) and exits at the vertex. The direction of the beam is
perpendicular to the canthomeatal line. This view is used in conjunction with other projections, and allows direct visualization
of the base of the skull.
Patient Position
• Image receptor positioned parallel to patient’s transverse plane and perpendicular to the midsagittal and coronal planes.
• Canthomeatal line forms a 10-degree angle with the receptor.
• Central beam is perpendicular to the image receptor.
• Directed from below the mandible toward the vertex of the skull.
• Centered about 2 cm anterior to a line connecting the right and left condyles.

https://t.me/DentalBooksWorld
Chapter 9 • Oral Radiology 793

5

B
A

Jug Handle View


• Similar to submentovertex.
• Exposure time for the zygomatic arch is reduced to approximately one-third the normal exposure time for submentovertex
projection.

Lateral Oblique
• Cassette is positioned against the patients cheek (its lower border parallel with the inferior border of mandible).
• Overlying the ascending ramus and the posterior aspect of the condyle under investigation.
• Cassette lower border lies 2 cm below it.
• Positioning achieves a 10-degree angle of separation between the median sagittal plane and the film.
• Mandible extended as far as possible.

True lateral

Oblique lateral

Waters’ View
• The occipitomental (OM) or Waters’ view is an angled PA radiograph of the skull, and can be used to assess for facial
fractures as well as the ethmoid and maxillary sinuses.
• Image receptor placed in front of the patient and perpendicular to the midsagittal plane.
• Central beam is perpendicular to the image receptor and centered in the area of maxillary sinus.
• If patient’s mouth is kept open, sphenoid sinus will be seen superimposed over the palate.
• Patient’s head tilted upward so that the canthomeatal line forms a 37-degree angle with the image receptor.

https://t.me/DentalBooksWorld
794 Triumph's Complete Review of Dentistry

Or
bit
om
45

en
t al
Radiographic plate

lin
e
Direction of X-ray beam

Water’s radiographic view of skull

TMJ Views
Transcranial view →
Cassette is placed flat against the patient’s ear and centered over the place of interest.
Patient’s head is adjusted so that the sagittal plane is vertical.
Ala-tragus line is parallel to the floor.
View is taken in both open and closed position.
Lindblom technic →
Central ray entered half inch behind and 2 inches above external auditory meatus.
Grewcock technic →
Central ray entry point is 2 inches above external auditory meatus perpendicular to occlusal plane.
Gillis technic →
Central entry point is half inch in front and 2 inches above external auditory meatus parallel and perpendicular to the occlusal
plane
Depicts the lateral aspect of TMJ.
Helps to evaluate the joints bony relationship.
Detecting arthritic changes on articular surfaces.

Transpharyngeal View
• Cassette is placed flat against patient’s ear at the place of interest
• Facial skin parallel to sagittal plane.
• Patient is positioned so that the sagittal plane is vertical and parallel to the film.
• Patient should open mouth
• Central ray is directed from the opposite side cranially at an angle of 5 and 10 degrees posteriorly
• Directed through mandibular notch

Transorbital View
• Film behind patient’s head at an angle of 45 degrees to the sagittal plane
• Patient’s position should allow sagittal plane to be vertical
• Canthomeatal line should be 10 degrees to the horizontal with the head tipped downward
• Mouth should be kept open
• Tube should be placed in front of the patient
• Central ray is directed at the joint of interest
• At an angle of 20 degree to strike the cassette at right angles

https://t.me/DentalBooksWorld
Chapter 9 • Oral Radiology 795

• Point of entry
–– Pupil of the same eye, patient looking straight
–– Medial canthus of the same eye
–– Medial canthus of the opposite eye
• Used to view
–– Anterior view of TMJ
–– Medial displacement of fractured condyle
–– Fracture of neck of condyle

Reverse Towne’s View


• Image receptor placed in front of the patient
• Perpendicular to midsagittal plane
• Parallel to coronal plane.
• Patient’s head tilted downward
• Canthomeatal line forms 25–30-degree angle with image receptor
• Patient’s mouth kept open to improve visualization of condyle
• On kept open, condylar heads are located inferior to articular eminence
• Central beam
–– Perpendicular to image receptor
–– Parallel to midsagittal plane
–– Centered at the level of condyles
• Used to view
–– Condylar neck and head
–– High fractures of condylar neck
–– Intracapsular fractures of TMJ
–– Condylar hypoplasia or hypertrophy

Computed Tomography
• Computed tomography (CT) scanning, also known as computerized axial tomography (CAT) scanning, is a diagnostic
imaging procedure that uses X-rays in order to present cross-sectional images (“slices”) of the body. Cross-sections are
reconstructed from the measurements of attenuation coefficients of X-ray beams in the volume of the object studied.
• CT is based on the fundamental principle that the density of the tissue passed by the X-ray beam can be measured from the
calculation of the attenuation coefficient.
• So, CT allows the reconstruction of the density of the body, by a two-dimensional section perpendicular to the axis of the
acquisition system.
• The emitter of X-rays (typically with energy levels between 20 and 150 keV), emits N photons (monochromatic) per unit
of time. The beam passes through the layer of biological material of thickness delta x. A detector placed at the exit of the
sample, measures N + delta N photons, delta N smaller than 0. So, the X-rays interacted with the object and the beam have
been attenuated.
• The conventional rescaling was made into CT numbers, expressed in Hounsfield Units (HU), as mentioned before. CT
numbers based on measurements with the EMI scanner were invented by Sir Godfrey Hounsfield.
Hounsfield chose a scale that affects the four basic densities, with the following values:
• Air = −1,000
• Fat = −60 to −120
• Water = 0
• Compact bone = +1,000

https://t.me/DentalBooksWorld
796 Triumph's Complete Review of Dentistry

Indicated in
• Variety of infections
• Osteomyelitis
• Cysts
• Benign and malignant tumors
• Trauma in maxillofacial region
• Lesions involving the bone
• 3D CT has been applied to trauma and craniofacial reconstructive surgery
• Used for treatment of congenital and acquired deformities

Magnetic Resonance Imaging (MRI)


• Magnetic resonance (MR) imaging is an important tool in the diagnosis and evaluation of diseases. In the early 1970s,
Paul Lauterbur and Raymond Damadian applied nuclear magnetic resonance (NMR) technology to the imaging of living
organisms, generating images referred to as zeugmatographs. Subsequent refinements in image acquisition and processing,
developed by Sir Peter Mansfield and others.
• The technique uses a very powerful magnet to align the nuclei of atoms inside the body, and a variable magnetic field that
causes the atoms to resonate, a phenomenon called nuclear magnetic resonance. The nuclei produce their own rotating
magnetic fields that a scanner detects and uses to create an image.
Indicated in
• Evaluate the position and integrity of disk in the TMJ
• Neoplasia involving the soft tissues, such as tongue, cheek, salivary gland, and neck
• Determining malignant involvement of lymph nodes
• Determining perineural invasion by malignant neoplasms
• With contrast enhances the image resolution of neoplasia

Ultrasonography
• Ultrasound is based on the use of high-frequency sound to aid in the diagnosis and treatment of patients. Ultrasound
frequencies range from 2 to approximately 15 MHz, although even higher frequencies may be used in some situations.
• The ultrasound beam originates from mechanical oscillations of numerous crystals in a transducer, which is excited by electrical
pulses (piezoelectric effect). The transducer converts one type of energy into another (electrical ↔ mechanical/sound).
• The ultrasound waves (pulses of sound) are sent from the transducer, propagate through different tissues, and then return
to the transducer as reflected echoes. The returned echoes are converted back into electrical impulses by the transducer
crystals and are further processed to form the ultrasound image presented on the screen.
• Ultrasound transducers contain a range of ultrasound frequencies, termed bandwidth. For example, 2.5–3.5 MHz for
general abdominal imaging and 5.0–7.5 MHz for superficial imaging.
Indicated for the evaluation of
• Neoplasms in the thyroid, parathyroid, or salivary glands or lymph nodes
• Stones in salivary glands or ducts
• Vessels of neck
• To guide fine-needle aspiration in the neck

Cone Beam Computed Tomography (CBCT)


• CBCT is a variation on traditional computed tomography (CT) that is on the rise. Unlike traditional CT scanners, in CBCT
an X-ray tube and detector panel rotate around the patient capturing data with a cone-shaped X-ray beam instead of the
“slices” CTs are typically known for.
• Constitutes 2-dimensional digital array providing an area detector fixed on a rotating gantry.
• A three-dimensional cone-shaped X-ray beam.
• Resultant beam is in the shape of a cone; hence termed “cone beam.”

https://t.me/DentalBooksWorld
Chapter 9 • Oral Radiology 797

Difference between CT and CBCT


• Cone Beam Computed Tomography (CBCT) or Dental Volume Tomography (DVT) scanners (such as the i-CAT) utilize a
cone beam, which radiates from the X-ray source in a cone shape, encompassing a large volume with a single rotation about
the patient. Images are then reconstructed using algorithms to produce 3-dimensional images at high resolution.
• Conventional CT scanners make use of a fan-beam. Transmitted radiation takes the form of a helix or spiral. The data are
then interpolated or rebinned by the scanner into a set of slices making up a volume. Large anatomical regions of the body
can be imaged during a single breath hold, reducing the possibility of artifacts caused by patient movement.
• Traditional CT uses a high-output, rotating anode X-ray tube. Cone beam tomography utilizes a low-power, medical
fluoroscopy tube that provides continuous imaging throughout the scan.
• Traditional computerized tomography records data with a fan-shaped X-ray beam onto image detectors arranged in an arc around
the patient, producing a single slice image per scan. Each slice must overlap slightly in order to properly reconstruct the images.
• The advanced cone beam technology uses a cone-shaped X-ray beam that transmits onto a solid-state area sensor for image
capture, producing the complete volume image in a single rotation.

How the Image Acquisition Occurs?


2D transaxial, multiplanar reformatted
3D techniques such as surface reconstruction and volume rendering

CBCT TMJ View


• CT scans are now widely used for the examination of pathological conditions and trauma in the maxillofacial region, in
presurgical implant treatment planning and the assessment of the temporomandibular joint. One of the reasons for this
is the development of new techniques, such as spiral CT and cone beam CT (CBCT). These techniques allow the use of a
shorter scanning time while the radiation dose is up to six times lower compared with conventional CT scans.
• CBCT, also known as volumetric CT (VCT), uses a cone-shaped X-ray beam instead of the collimated fan beam used with
spiral CT. The tube-detector system performs a 360-degree rotation around the head of the patient using a constant beam
angle. This rotation produces the initial data, the so-called raw data, which are presented as a lateral tomogram. The raw
data are used for primary reconstruction. The options for the thickness of the layers to be reconstructed are 0.3 mm, 1.0 mm,
and 3.0 mm, and the reconstruction angles are determined by the clinician. The primary images can be used for further
secondary reconstructions in all planes and for three-dimensional (3D) reconstructions.
Advantages
• Superior images to panoramic
• High speed scanning
• Low heat load
• Low cost
• Compact design
Disadvantages
• Poor soft tissue contrast
• Worse low contrast detectability
• Slightly inferior to conventional CT in terms of quality

Structures and Best View


Structures Best view
Fractures of the base of zygomatic arch Submentovertex
Base of skull
Fracture of zygoma Waters’ view
Maxillary sinus
Nasal septum

https://t.me/DentalBooksWorld
798 Triumph's Complete Review of Dentistry

Condylar neck fractures Reverse Towne’s view


Medially displaced condylar fractures PA view
Mid mandibular ramus to condylar apex Transpharyngeal view
Coronoid process of mandible PA view of skull
Fractures of ramus and body of mandible Lateral oblique 15 degrees
Horizontal favorable and unfavorable fractures of mandible Lateral oblique 30 degrees
Bony ankylosis of TMJ CT scan
Fibrous ankylosis of TMJ MRI
Internal derangement of the disk
Disk perforation Arthrography

CAUSES OF FAULTY RADIOGRAPH


Radiographic faults can occur at any stage in the radiographic process, right from handling and storing of films to the
processing. The causes for radiographic faults can be categorized as:
• Errors in film storage and handling
• Errors in film placement and projection technique
• Errors in exposure parameters and processing technique
• Artifacts

Errors in Film Storage and Handling


• Film fog
• Emulsion peel and scratched film
• Dark spots or lines
• Static electricity artifact
• Nail marks or kink marks

Errors in Film Placement and Projection Technique


• Herring bone effect, tyre mark pattern, raised diamond markings/knurled effect
• Cone cut
• Slanting of the occlusal plane
• Apical ends of the teeth not imaged (Partial image)
• Crown portion of teeth not imaged (Partial image)
• Overlapped image
• Foreshortened image
• Elongated image (Total)
• Elongated image (Partial)
• Blurred image

https://t.me/DentalBooksWorld
Chapter 9 • Oral Radiology 799

Errors in Exposure Parameters and Processing Technique


• Blank radiograph
• Dark radiograph
• Light radiographs
• Film fog
• Insufficient contrast
• Yellow or brown stains
• Partial image
• Blisters on the film
• White spots
• Dark spots on the radiograph
• Light spots on the radiograph
• Emulsion wash away
• Reticulation of emulsion
• Hyporetention
• Dichroic fog
• No image/blank radiograph

Radiographic Errors and Artifacts on the Film


1. Distorted images – from improper alignment of the tube, object, or film
Vertical elongation – vertical angulation too small
Vertical foreshortening – vertical angulation too great
Horizontal overlapping – incorrect horizontal angulation
2. Finger marks – from improper handling with hands
Dark finger marks
• Developer on fingers
• Fluoride – particularly stannous
• Dirt (grease)
Clear finger marks
• Fixer on fingers
3. Blurred images – from movement of the patient, film, or tube during exposure
4. With a bent film, part of the film will be in focus (usually the crowns) and part will be blurred.
When the film is completely bent over the lead foil from the back of the film appears on the front of the film and causes
whites line areas, usually in the corner of the film closest to the roots.
5. Dark or light films where images are visible – error in any one of the factors controlling density or distance.
With the three exposure or three processing errors, the whole film will be affected.
6. Completely clear film
• Machine not switched on
• Malfunction of machine
• Placing film in fixer before developer solution
• Film not taken/exposed

https://t.me/DentalBooksWorld
800 Triumph's Complete Review of Dentistry

7. Cone cutting
• Beam of radiation did not cover film
• Improper alignment (vertical or horizontal)/or
• Long axis of rectangular cone placed horizontal for anterior film or vice versa, or improper setup of aligning instruments
8. Herring bone pattern
Tire Track – film placed wrong way round in mouth. Film will have reduced density and marks/pattern on one side of the
film.
9. Double exposure – same film is exposed twice.
Often this results in another film not being exposed, thus another film will appear clear. The images may appear
superimposed, parallel at 90-degree angles to each other or upside-down.
Static electricity – films forcibly unwrapped or excessive flexing of film
10. 
Seen more often in dry, hot environment.
11. Crescent-shaped black lines
• Fingernail pressure on the film
• Excessive bending the film
Crescent-shaped white lines
• Cracked intensifying screen
12. Reticulation
• The emulsion contracts with time when subjected to great changes (difference of at least 15 degrees) in temperature
between the different processing solutions.
13. Undeveloped/clear area on film
• Incomplete immersion of film in developer – linear gray levels along edge – common board question
• Films overlapping during processing – outline of film
• Fixer on operators hands – clear finger prints
• Cone cutting sharply delineated round or straight area
• Film not exposed
14. Scratched film
• Emulsion is soft during processing. Long fingernails, careless handling during manual processing, wet films touching
other films while being processed or drying.
15. Black borders
• Wet or leaking packets allow light to enter a poorly sealed edge of film packet.
• Dry films on removal from mouth.
• Light due to opening day light loader too soon
16. Black spots
• dirt in the duplicating machine (developer drops will be dark)
17. Streaks
• Improper washing of film hanger
• Dirty rollers
• Heating pad in automatic processor not functioning

https://t.me/DentalBooksWorld
Chapter 9 • Oral Radiology 801

18. Radiolucent spots


• Developer drops (dark)
• Powder from the gloves
• Developer chemicals not properly dissolved
19. Clear spots
• Air bubbles sticking to film during processing
• Fixer splashed on film prior to developing
• Dirt in the intensifying screens
20. Brown film
• With time the film will go brown if not left in fixer solution or water bath (final wash) for the required amount of time
with manual processing.
• Also with exhausted fixer solution with automatic processing. When the radiograph is initially processed it will appear
“normal.”
21. Small, round, irregular, dark dots similar to static electricity – marks due to powder from gloves.
22. Black film
• Completely exposed to light
Black on one side of the film – hands taken out of automatic processor too soon
23. Ear rings, nose rings, (metal) dentures, eye glasses, etc. will all create radiographic artifacts (double or ghost images) and
must be removed before radiographs are taken.
24. Gray film with loss of detail
• Film fogged
• Exhausted fixer
• Insufficient time in fixer solution
25. Film fog
• Improper safe lightening
• Light leaks
• Over development
• Contaminated solutions
• Deteriorated film
26. Yellow or brown stains
• Depleted developer
• Depleted fixer
• Insufficient washing
• Contaminated solutions
27. Insufficient contrast
• Under development
• Under exposure
• Excessive kVp
• Excessive film fog

https://t.me/DentalBooksWorld
802 Triumph's Complete Review of Dentistry

AMERICAN DENTAL ASSOCIATION RECOMMENDATIONS FOR PRESCRIBING


DENTAL RADIOGRAPHS
Type of Child with Child with Adolescent with Adult, dentate, Adult
encounter primary dentition transitional permanent dentition and partially edentulous
(prior to eruption dentition (after (prior to eruption of edentulous
of that permanent eruption of first third molars)
tooth) permanent tooth)
New Individualized Individualized Individualized Individualized Individualized
Patient being radiographic radiographic radiographic radiographic radiographic
evaluated for oral exam consisting of exam consisting exam consisting of exam consisting exam, based on
diseases selected periapical/ of posterior posterior bitewings of posterior clinical signs and
occlusal views bitewings with with panoramic bitewings with symptoms.
and/or posterior panoramic exam exam or posterior panoramic exam
bitewings or posterior bitewings and or posterior
if proximal bitewings and selected periapical bitewings
surfaces cannot selected periapical images. A full and selected
be visualized or images. mouth intraoral periapical
probed. Patients radiographic exam images. A full
without evidence is preferred when mouth intraoral
of disease and with the patient has radiographic
open proximal clinical evidence exam is preferred
contacts may of generalized oral when the patient
not require a disease or a history has clinical
radiographic exam of extensive dental evidence of
at this time treatment. generalized
oral disease
or a history of
extensive dental
treatment.
Recall Posterior Posterior Posterior bitewing Posterior Not applicable
Patient with bitewing exam bitewing exam exam at 6–12-month bitewing
clinical caries or at 6–12-month at 6–12-month intervals if proximal exam at
at increased risk intervals if intervals if surfaces cannot be 6–18-month
for caries proximal surfaces proximal surfaces examined visually or intervals
cannot be cannot be with a probe
examined visually examined visually
or with a probe. or with a probe.
Recall Posterior Posterior Posterior Posterior Not applicable
Patient with no bitewing exam bitewing exam bitewing exam at bitewing exam at
clinical caries and at 12–24-month at 12–24-month 18–36-month 24–36-month
not at increased intervals if intervals if intervals intervals
risk for caries proximal surfaces proximal surfaces
cannot be cannot be
examined visually examined visually
or with a probe or with a probe

https://t.me/DentalBooksWorld
Chapter 9 • Oral Radiology 803

Recall Clinical judgment Clinical judgment Clinical judgment Clinical Not applicable
Patient with as to the need as to the need as to the need judgment as
periodontal for and type of for and type of for and type of to the need for
disease radiographic radiographic radiographic images and type of
images for the images for the for the evaluation of radiographic
evaluation of evaluation of periodontal disease. images for the
periodontal periodontal Imaging may consist evaluation of
disease. disease. of, but is not limited periodontal
Imaging may Imaging may to, selected bitewing disease.
consist of, but is consist of, but is and/or periapical Imaging may
not limited to, not limited to, images of areas consist of, but
selected bitewing selected bitewing where periodontal is not limited
and/or periapical and/or periapical disease (other to, selected
images of areas images of areas than nonspecific bitewing and/or
where periodontal where periodontal gingivitis) can be periapical images
disease (other disease (other of areas where
than nonspecific than nonspecific periodontal
gingivitis) can be gingivitis) can be disease (other
than nonspecific
gingivitis) can be
Patient (new Clinical judgment Clinical judgment Clinical judgment Usually not Usually not
and recall) for as to need for as to need for as to need for and indicated for indicated for
monitoring and type of and type of type of radiographic monitoring of monitoring of
of dentofacial radiographic radiographic images for evaluation growth and growth and
growth and images for images for and/or monitoring development. development.
development, evaluation and/ evaluation and/ of dentofacial growth Clinical Clinical
and/or assessment or monitoring of or monitoring of and development, judgment as judgment as
of dental/skeletal dentofacial growth dentofacial growth or assessment of to the need for to the need for
relationships and development and development dental and skeletal and type of and type of
or assessment of or assessment of relationships. radiographic radiographic
dental and skeletal dental and skeletal Panoramic or image for image for
relationships relationships periapical exam to evaluation evaluation
assess developing of dental of dental
third molars and skeletal and skeletal
relationships. relationships.
Patient with other Clinical judgment Clinical judgment Clinical judgment Clinical Clinical
circumstances as to need for as to need for as to need for and judgment as to judgment as to
including, but and type of and type of type of radiographic need for and type need for and type
not limited to, radiographic radiographic images for evaluation of radiographic of radiographic
proposed or images for images for and/or monitoring of images for images for
existing implants, evaluation and/ evaluation and/ these conditions evaluation and/ evaluation and/
other dental or monitoring of or monitoring of or monitoring of or monitoring of
and craniofacial these conditions these conditions these conditions these conditions
pathoses,
restorative/
endodontic
needs, treated
periodontal
disease, and caries
remineralization

https://t.me/DentalBooksWorld
804 Triumph's Complete Review of Dentistry

Clinical situations for which radiographs may be indicated include, but are not limited to:

A. Positive Historical Findings


1. Previous periodontal or endodontic treatment
2. History of pain or trauma
3. Familial history of dental anomalies
4. Postoperative evaluation of healing
5. Remineralization monitoring
6. Presence of implants, previous implant-related pathosis or evaluation for implant placement
B. Positive Clinical Signs/Symptoms
1. Clinical evidence of periodontal disease
2. Large or deep restorations
3. Deep carious lesions
4. Malposed or clinically impacted teeth
5. Swelling
6. Evidence of dental/facial trauma
7. Mobility of teeth
8. Sinus tract (“fistula”)
9. Clinically suspected sinus pathosis
10. Growth abnormalities
11. Oral involvement in known or suspected systemic disease
12. Positive neurologic findings in the head and neck
13. Evidence of foreign objects
14. Pain and/or dysfunction of the temporomandibular joint
15. Facial asymmetry
16. Abutment teeth for fixed or removable partial prosthesis
17. Unexplained bleeding
18. Unexplained sensitivity of teeth
19. Unusual eruption, spacing, or migration of teeth
20. Unusual tooth morphology, calcification, or color
21. Unexplained absence of teeth
22. Clinical tooth erosions
23. Peri-implantitis

QUICK FACTS
Roentgen discovered X-rays in November 1895.
Ionization → conversion of an atom into ion.
Types of radiation: Two
1. Particulate radiation
• Another form of radiation, that do not travel as a wave, but as particles of matter (have mass) released from unstable atoms
• Very high energy
What are the two types of particulate radiation?
• Alpha particles and beta particles
–– Alpha particles: Released by the nucleus of an unstable atom, consists of two protons and two neutrons
–– Beta particles:
• High speed electron emitted from an unstable nucleus
• Smaller/lighter than an alpha particle
• Travels farther than alpha particles and can penetrate through paper and aluminum but not lead

https://t.me/DentalBooksWorld
Chapter 9 • Oral Radiology 805

2. Nonparticulate/Electromagnetic radiation
X-rays
• Electromagnetic radiation
• Produced when velocity of an electrically charged particle is altered
Properties
• Travel at the speed of light
• Invisible
• Cannot be focused
• Cannot be reflected or refracted
• But can be affected by magnetic and electric fields
• Affect photographic plates
• They cast shadows of the objects in their paths

IMPORTANT FACTS
Hard X-rays → shorter wavelength and higher penetration → used in diagnostic
Produced by increased kVp
    ↓
Soft X-rays → long wavelength and less penetrating power → therapeutic use
Produced by Decreasing kVp
    ↓
Grenz rays → soft X-rays → wavelength of 2 AU
Stray radiation → produced from tube other than focal spot
Secondary radiation → produced by primary beam incident in any matter
X-ray machine:
X-ray tube has → anode and cathode
Cathode → tungsten coil/filament → source of electrons
Molybdenum focusing cup → focuses the electron toward focal spot
Anode → tungsten target and copper stem
Target is inclined about → 20 degrees, with respect to central x-ray beam
This inclination decreases actual focal spot (1×3 mm) → (1×1 mm) effective focal spot → based on line focus principle
Tungsten target → converts kinetic energy of electrons emitted from cathode source into X-ray photons
Why tungsten??
• High atomic number
• High melting point
• Low vapor pressure
Copper stem → dissipates heat → reducing the risk of target melting

Bremsstrahlung Characteristic radiation


Breaking or primary radiation Discrete radiation
Primary source of radiation Minor source of radiation
Results from transfer of kinetic energy of electrons into X-rays Results from transfer of energy from ionization of target
material into X-rays

https://t.me/DentalBooksWorld
806 Triumph's Complete Review of Dentistry

Power supply:
Supply the X-ray tube→ to maintain potential difference between anode and cathode
Auto transformer → controls voltage between anode and cathode
Coolidge transformer/filament step-down transformer

Maintains the current flow through filament

Thus, tube current maintained.
Tube current → 0.2–0.8% of cathode rays are transformed into X-rays after striking the anode target.
X-rays with high intensity are found toward cathode side of central ray, whereas low intensity are found toward anode.
Half value layer: Penetrating quality of X-ray beam
Nothing but thickness of aluminum required to reduce half the number of X-ray photons passing through it
Aluminum filter selectively removes the less penetrating photons that contributes only to patient exposure and does not
have enough energy to reach film.
Its thickness is 1.5 mm for 70 kVp.
Linear energy transfer: Rate of loss of energy from a particle as it moves through the irradiated material is known as
LET.
Alpha rays have higher LET → more damaging to biologic system than X-rays
Erythema dose → amount of radiation necessary to produce noticeable skin reaction
ALARA → as low as reasonably achievable
Penumbra → zone of unsharpness or blurred zone on an image.
Umbra → zone of image clarity with proper sharpness and resolution
Three mechanisms that explain the interactions of X-rays with matter are:
• Coherent scattering
• Photoelectric scattering
• Compton scattering
Thompson effect:
• Also known as classical scattering or coherent scattering.
• Occurs when lo er energy incident photon passes near an outer electron of an atom.
• Incident photon is not absorbed but scattered without loss of energy.
• Energy of scattered photon = energy of incident photon.
• 8% of the total number of interactions.
Photoelectric absorption:
• Occurs when an incident photon collides with a bound electron in an atom.
• Incident photon is absorbed and the electron is expelled from its shell.
• Expelled electron becomes photoelectron.
• Energy of photoelectron → energy of incident photon-binding energy of electron.
• 30% of total number of interactions.
Compton scattering:
• When a photon interacts with an outer electron of an atom.
• Electron receives kinetic energy and recoils from the point of impact.
• Incident photon is scattered from the site of collision, making the atom ionized.
• 62% of photons undergo Compton scattering.
• Compton scattering is a major source of secondary radiation.

https://t.me/DentalBooksWorld
Chapter 9 • Oral Radiology 807

Action of radiation can either be direct or indirect.


• Direct effect:
–– Energy of a photon is transferred directly to biologic molecules.
• Indirect effect:
–– Photon is absorbed by water and form free radicals, which in turn reacts with biologic macromolecules.
–– About two-thirds of radiation-induced biologic damage results from indirect effects.
–– Sensitive sites within nucleus → DNA and Chromosomes.
Order of cell cycle
• G1 → S → G2 → M
• G1 or early S → Before DNA Replication → damage occurs in both arms
• G2 or late S → After DNA Synthesis → only one arm of affected chromosome will be broken
• G2 → most sensitive phase
• Early S → least sensitive phase
• Most Sensitive Period → organogenesis → 18–45 days of gestation
• Exposure in this period leads to developmental abnormalities

MULTIPLE CHOICE QUESTIONS

GENERAL PROPERTIES OF X-RAY PRODUCTION AND DOSE


1. A film badge is a type of
A. Identification plate B. Sonometer
C. Dosimeter D. Tachometer
2. Magnification can be reduced in X-ray by
A. Long cone B. Short cone
C. Place film far from object and parallel D. Reduce exposure time
3. The best radiographic view of lateral condyle is
A. Towne’s view B. Reverse Towne’s view
C. Water’s view D. Caldwell view
4. Most common cause of break in canal seen on IOPA radiograph is
A. Calcification of canal B. Merging of canal
C. Bifurcation of canal D. Extra canal
5. Proximal caries best detected by
A. Bitewing B. Occlusal
C. IOPA D. Panoramic
6. The size and shape of the X-ray beam is restricted by
A. Filter B. Collimator
C. Film badge D. E speed film
7. Use of F speed films reduces patient exposure by
A. 50% compared to E speed films B. 75% compared to E speed films
C. 25% compared to E speed films D. No reduction in exposure
8. The primary source of X-ray photons
A. Bremsstrahlung radiation B. Characteristic radiation
C. Coherent scattering D. Compton scattering
9. Which of the following is used to estimate risk in human organs?
A. Effective dose B. Exposure
C. Absorbed dose D. Equivalent dose

https://t.me/DentalBooksWorld
808 Triumph's Complete Review of Dentistry

10. Radiographic technique to detect monitor salivary gland disease is called


A. Angiography B. Computed tomography
C. Electrocardiogram D. Sialography
11. Radiation caries is characteristically seen in
A. Occlusal fissures B. Proximal surface
C. Incisal edges D. Cervical third
12. The best radiograph for viewing of maxillary sinus is
A. Lateral oblique maxilla B. Submentovertex
C. Water’s view D. Lateral skull
13. The standard size of adult IOPA film is
A. 22 × 30 mm B. 31 × 41 mm
C. 31 × 40 mm D. 22 × 40 mm
14. The zygomatic arches are best visualized on the
A. Submentovertex view B. Water’s view
C. Reverse Towne’s view D. PA view
15. Least radiosensitive cells are
A. WBC, bone, cells, cartilage B. Muscle cell, nerve cell, bone cell
C. Connective tissue cell, endothelial cell, muscle cell D. RBC, muscle cell, bone cell
16. Halo effect around root is seen in
A. Widened PDL B. Horizontal root fracture
C. Periapical periostitis D. Acute periapical abscess
17. A 30-year-old patient having chronic pain of long duration with radiographic appearance of endosteal bone formation.
Diagnosis is
A. Acute osteomyelitis B. Ewing’s sarcoma
C. Garre’s osteomyelitis D. Chronic focal sclerosing osteomyelitis
18. Recommended dose of prednisolone for the treatment of desquamative gingivitis is
A. Daily dose of 40–50 mg reduced to daily maintenance dose of 5–10 mg
B. Daily dose of 30–40 mg reduced to daily maintenance dose of 10–20 mg
C. Daily dose of 30–40 mg reduced to daily maintenance dose of 5–10 mg
D. Daily dose of 50–60 mg reduced to daily maintenance dose of 10–20 mg
19. Medical linear accelerators produce
A. Neutrons + Electrons B. Protons + Neutrons
C. Photons + Protons D. Photons + Electrons
20. Filter used in biologically safety cabinet is
A. High-efficiency particulate filter (HEPA) B. Berkfield
C. Millipore D. Seitz filter
21. Unit to measure radiation in radiotherapy is
A. Gray B. Sievert
C. Celsius D. Radons
22. Effective dose of X-ray exposure for an adult in an IOPA
A. 200 μSv B. 5,000 μSv
C. 26 μSv D. 5 μSv
23. Which of the following is true about radiation?
A. More rapidly dividing cells are less susceptible B. Effects of X-rays are temporary and reversible
C. X-rays do not damage all kinds of biologic forms D. Only primary central beam is potentially hazardous
24. Digital radiography requires less radiation than conventional radiography because
A. The sensor is larger B. The sensor is more sensitive to X-rays
C. The exposure time is increased D. The pixels sense transmitted light quickly
25. Diagnosis of incipient carious lesion is best done by
A. D speed film B. C speed film
C. E speed film D. B speed film

https://t.me/DentalBooksWorld
Chapter 9 • Oral Radiology 809

26. Diameter and length of filament in X-ray tube, respectively, are


A. 2 mm and 1 cm B. 3 mm and 2 cm
C. 4 mm and 1 cm D. 1 cm and 2 mm
27. Pantomograph–chin placed in front of focal trough; the effect on image will be
A. Blurred and magnified horizontally B. Blurred and magnified vertically
C. Clear and magnified D. Blurred and diminished
28. Osteogenic sarcoma, radiographic appearance
A. Sunburst appearance B. Snow driven appearance
C. Onion skin appearance D. Cotton wool appearance
29. Multilocular radiolucency of cyst is associated with
A. OKC B. Ameloblastoma
C. Dentigerous cyst D. Periapical cyst
30. Ground glass appearance of bone in the radiograph is suggestive of
A. Hyperparathyroidism B. Fibrous dysplasia
C. Condensing osteitis D. Osteopetrosis
31. Too light, thin faded image on radiographic film is due to
A. Overdevelopment B. Underdevelopment
C. Exhausted developer solution D. Pre-exposure of radiograph
32. The metallic lead back in a film packet helps in which of the following?
A. To absorb secondary radiation B. To reduce the exposure time
C. To reduce the developing time D. All of the above
33. Hardener used in fixing solutions is
A. Sodium thiosulfate B. Alkaline potassium sulfate
C. Potassium bromide D. Sodium sulfide
34. The following kVp setting produces greatest contrast in radiographic images
A. 70 kVp B. 80 kVp
C. 90 kVp D. 100 kVp

RADIOBIOLOGY
1. Example of deterministic effects are
A. Mucositis resulting from radiation therapy to oral cavity B. Radiation-induced cataract formation
C. Both the above D. None of the above
2. Stochastic effects caused by
A. Sublethal damage to DNA B. Killing of many cells
C. Both of the above D. None of the above
3. Severity of clinical effects is proportional to dose in case of
A. Deterministic effects B. Stochastic effects
C. Both D. None
4. Deterministic effect is
A. Probability of effect is independent of dose B. Frequency of effect is proportional to dose
C. Severity of clinical effects is independent of dose D. Severity of clinical effects is not proportional to dose
5. If radiation exposure occurs in G2 phase, then it results in
A. Chromatid aberration B. Chromosome aberration
C. Chromosome translocation D. Chromatid translocation
6. Different cells from various organs of the same individual may respond to irradiation quite differently. This variation
was recognized as early as 1906 by the French radiobiologists ________ and _____________.
7. Which of the following cell is highly radiosensitive?
A. Endothelial cells B. Basal cells of oral mucous
C. Vascular endothelial cells D. Fibroblasts

https://t.me/DentalBooksWorld
810 Triumph's Complete Review of Dentistry

8. All are radiosensitive organs except


A. Lymphoid B. Mucous membrane
C. Bone marrow D. Muscle
9. Dose rate indicates
A. Amount of dose given B. Rate of exposure
C. Actual dose D. Dose required
10. Which of the following is false regarding taste loss after radiation exposure?
A. Taste acuity decreases by a factor of 100–1,000 B. It is reversible
C. Recovery takes 2–4 months D. Seen during second or third week of radiotherapy
11. Acute radiation syndrome – severe hematopoietic symptoms seen at a dose of
A. 1–2 Gy B. >50 Gy
C. 4–7 Gy D. 7–15 Gy
12. Radiation effect on blood cells – which cell has least duration?
A. Lymphocyte B. Granulocyte
C. Platelets D. Erythrocytes
13. All are low susceptible organs except
A. Salivary glands B. Skin
C. Brain D. Stomach

RADIOLOGICAL INTERPRETATION
1. Supernumerary teeth occur in _______ of the population
A. 1–4% B. 5–8%
C. 12% D. Less than 1%
2. Multiple supernumerary teeth occur most frequently in the ______ regions, usually in the _______
A. Molar, maxilla B. Molar, mandible
C. Premolar, mandible D. Anterior region, maxilla
3. Supernumerary teeth occur mostly in
A. Females B. Males
C. Equal gender predilection D. None of the above
4. Radiographs may reveal supernumerary teeth in the deciduous dentition after __________ years of age
A. 2–3 years B. 3–4 years
C. Younger than 1 year D. Older than 4 years
5. Radiographs may reveal supernumerary teeth in the permanent dentition after __________ years of age
A. 6–8 years B. 8–9 years
C. Older than 9–12 years D. Older than 12 years
6. Which of the following is false about pyknodysostosis?
A. Obtuse mandibular gonial angle often with relative prognathism
B. Osteopetrosis
C. Premature closure of sutures in skull
D. Autosomal recessive and also known as Toulouse-Lautrec syndrome
7. Hypodontia in the permanent dentition, excluding third molars, is found in _______ of the population
A. 10% B. 1%
C. 15% D. 20%
8. Peridens refers to
A. Supernumerary teeth that occur in the premolar region
B. Supernumerary teeth that occur in molar region
C. Both of the above
D. Supernumerary teeth distal to premolar
9. Although missing primary teeth are relatively uncommon, when one tooth is missing, it is usually a _________
A. Maxillary incisor B. Mandibular incisor
C. Maxillary/mandibular first molars D. Maxillary/mandibular second molars

https://t.me/DentalBooksWorld
Chapter 9 • Oral Radiology 811

10. Hemangioma is associated with


A. Macrodontia B. Microdontia
C. Hypodontia D. Synodontia
11. What is true concrescence?
A. If the condition occurs during development it is called true concrescence
B. If there is complete fusion of root of tooth it is called true concrescence
C. If it occurs in molar region it is called true concrescence
D. All of the above
12. Although gemination may occur in both the deciduous and permanent dentitions, it more frequently affects the
________, usually in the incisor region
A. Primary teeth B. Permanent teeth
C. Equal in both D. None of the above
13. Statements regarding Gemination A – The occurrence in males and females is about equal.B – The enamel or dentin
of geminated teeth may be hypoplastic or hypocalcified.
A. Both are true B. Both are false
C. Statement A is true and statement B is false D. Statement A is false and statement B is true
14. Gestant odontoma is also known as
A. Dens in dente B. Dens evaginatus
C. Regional odontodysplasia D. Enamel pearl
15. Incidence of dentinogenesis imperfecta
A. 1:2,000 B. 1:4,000
C. 1:6,000 D. 1:8,000
16. Which of the following is true about osteogenesis imperfecta?
A. Autosomal recessive
B. Increased incidence of impacted first and second molars
C. Class 2 malocclusion
D. Dentinogenesis imperfecta is found in 45 % of the patients
17. Most common tooth involved in case of dens evaginatus is
A. Mandibular premolar B. Maxillary premolar
C. Lateral incisors D. Canines
18. Bull’s eye appearance of the root apex produced by the root canal, tooth root, and PDL space seen in case of
A. Dilaceration B. Dilated odontome (severe form of dens invaginatus)
C. Dentinogenesis imperfecta D. Regional odontodysplasia

MISCELLANEOUS
1. The area from which X-rays emanate is called the
A. Target B. Focal spot
C. Intensifying screen D. Cone
2. The image of the coronoid process of the mandible often appears in periapical X-rays of
A. The incisor region of the mandible B. The molar region of the mandible
C. The incisor region of the maxilla D. The molar region of the maxilla
3. X-ray developer contains all of the following except
A. A developing agent B. An antioxidant preservative
C. A clearing agent D. An accelerator
E. A restrainer
4. X-ray fixer contains all of the following except:
A. A clearing agent B. An antioxidant preservative
C. An accelerator D. An acidifier
E. A hardener

https://t.me/DentalBooksWorld
812 Triumph's Complete Review of Dentistry

5. After processing a film, you notice that it appears brown in color. What is the most likely cause?
A. Solutions are too strong B. Solutions are too weak
C. Fixing time was not long enough D. Fixing time was too long
E. Film was underdeveloped
6. The unit for measuring the absorption of X-rays is termed
A. REM B. RAD
C. Roentgen D. QF
7. Which of the following is considered radioresistant?
A. Immature reproductive cells B. Young bone cells
C. Mature bone cells D. Epithelial cells
8. It is recommended that the operator stand at least how many feet away from the patient when taking radiographs?
A. Two feet B. Four feet
C. Six feet D. Eight feet
9. The inverse square law formula is
A. New intensity Original distance 2/Original intensity New distance 2
B. New intensity New distance 2/Original intensity Original distance 2
C. Original intensity New distance 2/New intensity Original distance 2
D. None of the above
10. Increasing the kilovoltage (kVp) causes the resultant X-ray to have
A. Decreased density B. More latitude
C. A shorter scale of contrast D. A longer scale of contrast
11. The amount of material that is required to reduce the intensity of an X-ray beam to half is called the
A. Value layer B. Intensity value layer
C. Half-value layer D. Full-value layer
12. The radiation generated at the anode of the X-ray tube is called
A. Primary radiation B. Scattered radiation
C. Potential radiation D. Reverse radiation
13. The removal of parts of the X-ray spectrum using absorbing materials in the X-ray beam is called
A. Elimination B. Filtration
C. Collimation D. Reduction
14. The most effective means in reducing the time of exposure, the amount of radiation reaching the patient, and the
amount of radiation scattered to the dentist is
A. A lead apron B. Ekta-speed film
C. Lead diaphragms D. Increasing target–JHm distance
15. The use of metal plates, slots, bars, etc., to confine a direct radiation (e.g., X-rays or gamma-rays) to a specific region
and I to discriminate against radiation from unwanted directions (e.g., scattered radiation) is called
A. Discrimination B. Collimation
C. Filtration D. Coning
16. Image magnification may be minimized by
A. Using a short cone B. Placing the film as far from the tooth as possible
C. Using a long cone D. Shortening the exposure time
17. Foreshortening and elongation are produced by
A. Incorrect horizontal angulation
B. Incorrect vertical angulation
C. Either of the above
18. Which of the following positioning errors is the most likely cause of the reverse occlusal plane curve in a panorex
projection?
A. Chin tilted too far upward
B. Chin tilted too far downward
C. Head turned slightly

https://t.me/DentalBooksWorld
Chapter 9 • Oral Radiology 813

19. Which of the following is a major disadvantage of the paralleling technique?


A. The image formed on the film will not have dimensional accuracy
B. Due to the amount of distortion, periodontal bone height cannot be accurately diagnosed
C. An increase in exposure time is necessary due to the use of a long cone
D. An increase in exposure time is necessary due to the use of a short cone
20. Which of the following is not a disadvantage of the bisecting technique?
A. Image on X-ray film may be dimensionally distorted (amount may vary)
B. Increased exposure time
C. Due to the use of a short cone (which results in divergent rays), the image is not a true reproduction of the object
D. May not be able to judge the correct alveolar bone height
21. Which of the following projections is best for examination of fractures of the zygomatic arch?
A. Waters projection B. Submentovertex projection
C. Reverse Towne projection D. Lateral cephalometric projection
22. Which of the following projections is best for examination of the maxillary sinus?
A. Lateral jaw projection B. Reverse Towne projection
C. Waters’ projection D. Submentovertex projection
23. Which of the following projections is best for examination of the condyles and neck of the mandible (from an anterior-
posterior projection)?
A. Waters projection B. Transcranial projection
C. Townes’ projection D. Submentovertex projection
24. All of the following are advantages of a panoramic radiograph except
A. It shows areas that may not be visible on a full mouth series
B. It shows both arches on the same film
C. It gives better detail and definition than periapical radiographs
D. It is more comfortable for the patient (eliminates gagging)
E. It requires less time than a full mouth series
25. A phenomenon caused by a relatively lower X-ray absorption on the mesial or distal aspect of teeth, between the edge
of the enamel, and the adjacent crest of the alveolar ridge is called
A. Apical burnout B. Cervical burnout
C. Coronal burnout D. Root burnout
26. Which of the following types of intraoral radiographs are most useful in detecting interproximal caries?
A. Periapical radiographs B. Bitewing radiographs
C. Occlusal radiographs
27. The period between radiation exposure and the onset of symptoms is called the
A. Latent period B. Period of cell injury
C. Recovery period
28. Which type of digital image receptor is most common at this time?
A. CID (Charge Injection Device)
B. CMOS I APS (Complimentary Metal Oxide Semiconductor/Active Pixel Sensor)
C. CCD (Charge-Coupled Device)
29. All of the following are advantages of digital radiography except
A. Digital subtraction B. The ability to enhance the image
C. Size of the intraoral sensor D. Patient education
30. The method of obtaining a digital image similar to scanning a photograph to a computer screen is termed
A. Indirect digital imaging B. Direct digital imaging
C. Storage phosphor imaging D. CMOS/APS
31. Digital radiography requires less radiation than conventional radiography because
A. The sensor is larger B. The exposure time is increased
C. The sensor is more sensitive to X-rays D. The pixel’s sense transmitted light quickly

https://t.me/DentalBooksWorld
814 Triumph's Complete Review of Dentistry

32. Which of the following errors in radiographic technique is the most likely reason that an image on a radiograph would
appear elongated?
A. Too much vertical angulation B. Too little vertical angulation
C. Incorrect horizontal angulation D. Beam not aimed at center of film
33. A film badge is a type of
A. Identification plate B. Sonometer
C. Dosimeter D. Tachometer
34. Magnification can be reduced in X-ray by
A. Long cone B. Short cone
C. Place film far from object and parallel D. Reduce exposure time
35. The best radiographic view of lateral condyle is
A. Towne’s view B. Reverse Towne’s view
C. Waters’ view D. Caldwell’s view
36. Most common cause of break in canal seen on IOPA radiograph is
A. Calcification of canal B. Merging of canal
C. Bifurcation of canal D. Extra canal
37. Proximal caries are best detected by
A. Bitewing B. Occlusal
C. IOPA D. Panoramic
38. The size and shape of the X-ray beam is restricted by
A. Filter B. Collimator
C. Film badge D. E speed film
39. Use of F speed films reduces patient exposure by
A. 50% compared to E speed films B. 75% compared to E speed films
C. 25% compared to E speed films D. No reduction in exposure
40. The primary source of X-ray photons is
A. Bremsstrahlung radiation B. Characteristic radiation
C. Coherent scattering D. Compton scattering
41. Which of the following is used to estimate risk in human organs?
A. Effective dose B. Exposure
C. Absorbed dose D. Equivalent dose
42. Radiographic technique to detect salivary gland disease is called
A. Angiography B. Computed tomography
C. Electrocardiogram D. Sialography
43. Radiation caries is characteristically seen in
A. Occlusal fissures B. Proximal surface
C. Incisal edges D. Cervical third
44. The best radiograph for viewing of maxillary sinus is
A. Lateral oblique maxilla B. Submentovertex
C. Waters’ view D. Lateral skull
45. The standard size of adult IOPA film is
A. 22 x 30 mm B. 31 x 41 mm
C. 31 x 40 mm D. 22 x 40 mm
46. The zygomatic arches are best visualized on the
A. Submentovertex view B. Waters’ view
C. Reverse Towne’s view D. PA view
47. Least radiosensitive cells are
A. WBC, bone, cells, cartilage B. Muscle cell, nerve cell, bone cell
C. Connective tissue cell, endothelial cell, muscle cell D. RBC, muscle cell, bone cell

https://t.me/DentalBooksWorld
Chapter 9 • Oral Radiology 815

48. Halo effect around root is seen in


A. Widened PDL B. Horizontal root fracture
C. Periapical periostitis D. Acute periapical abscess
49. A 30-year-old patient having chronic pain of long duration with radiographic appearance of endosteal bone formation.
Diagnosis is
A. Acute osteomyelitis B. Ewing’s sarcoma
C. Garre’s osteomyelitis D. Chronic focal sclerosing osteomyelitis

ANSWERS

GENERAL PROPERTIES OF X-RAY PRODUCTION AND DOSE


1. Answer: C
2. Answer: A
3. Answer: B
4. Answer: C
5. Answer: A
6. Answer: B
7. Answer: A
8. Answer: A
9. Answer: A
10. Answer: D
11. Answer: D
12. Answer: C
13. Answer: B
14. Answer: A
15. Answer: D
16. Answer: C
17. Answer: D
18. Answer: C
19. Answer: D
20. Answer: A
21. Answer: A
22. Answer: D
23. Answer: D
24. Answer: B
25. Answer: A
26. Answer: A
27. Answer: D
28. Answer: A
29. Answer: A
30. Answer: A
31. Answer: B and C
32. Answer: A
33. Answer: B
34. Answer: A

https://t.me/DentalBooksWorld
816 Triumph's Complete Review of Dentistry

RADIOBIOLOGY
1. Answer: C
Deterministic effects
• Mucositis resulting from radiation therapy to oral cavity
• Radiation-induced cataract formation
Stochastic effects
• Radiation-induced cancer
• Heritable effects
2. Answer: A
Stochastic effects caused by sublethal damage to DNA.
Deterministic effects caused by killing of many cells.
3. Answer: A
Deterministic effects – Severity of clinical effects is proportional to dose. The greater the dose the greater the effect.
Stochastic effects – Severity of clinical effects is independent of dose. All or none response; an individual either has effect
or does not.
4. Answers: A and B
Probability of effect is independent of dose. All individuals show effect when dose is above threshold.
Frequency of effect is proportional to dose. The greater the dose the greater the chance of having the effect.
5. Answer: A
If radiation exposure occurs after DNA synthesis (i.e., in G2 or mid and late S), only one arm of the affected chromosome
is broken (chromatid aberration). However, if the radiation-induced break occurs before the DNA has replicated (i.e., in
G1 or early S), the damage manifests as a break in both arms (chromosome aberration).
6. Answer: Bergonié and Tribondeau
7. Answer: B
High
• Spermatogenic and erythroblastic stem cells
• Basal cells of oral mucous membrane
Intermediate
• Vascular endothelial cells
• Fibroblasts acinar and ductal salivary gland cells
• Parenchymal cells of liver, kidney, and thyroid
Low
• Neurons
• Striated muscle cells
• Squamous epithelial cells
• Erythrocytes
8. Answer: D
High
• Lymphoid organs
• Bone marrow
• Testes
• Intestines
• Mucous membranes
Intermediate
• Fine vasculature
• Growing cartilage
• Growing bone
• Salivary glands
• Lungs
• Kidney
• Liver

https://t.me/DentalBooksWorld
Chapter 9 • Oral Radiology 817

Low
• Optic lens
• Muscle
9. Answer: B
The term dose rate indicates the rate of exposure. For example, a total dose of 5 Gy may be given at a high dose rate (5 Gy/
min) or a low dose rate (5 mGy/min). Exposure of biologic systems to a given dose at a high dose rate causes more damage
than exposure to the same total dose given at a lower dose rate.
10. Answer: A
Taste buds are sensitive to radiation. Doses in the therapeutic range cause extensive degeneration of the normal histologic
architecture of taste buds. Patients often notice a loss of taste acuity during the second or third week of radiotherapy. Bitter
and acid flavors are more severely affected when the posterior two-thirds of the tongue is irradiated and salt and sweet
when the anterior third of the tongue is irradiated. Taste acuity usually decreases by a factor of 1,000–10,000 during the
course of radiotherapy. Alterations in the saliva may partly account for this reduction, which may proceed to a state of
virtual insensitivity. Taste loss is reversible and recovery takes 60–120 days.
11. Answer: C
Dose (Gy) Manifestation
1–2 Prodromal symptoms
2–4 Mild hematopoietic symptoms
4–7 Severe hematopoietic symptoms
7–15 Gastrointestinal symptoms
50 Cardiovascular and central nervous system symptoms
12. Answer: A
The rate of fall in the circulating levels of a cell depends on the life span of that cell in the peripheral blood. Granulocytes,
with short lives in circulation, fall off in a few days, whereas red blood cells, with long lives in circulation, fall off slowly.
13. Answer: D
High
• Colon
• Stomach
• Lung
• Bone marrow (leukemia)
• Female breast
Intermediate
• Bladder
• Liver
• Thyroid
Low
• Bone surface
• Brain
• Salivary glands
• Skin

RADIOLOGICAL INTERPRETATION
1. Answer: A
2. Answer: C
3. Answer: A
4. Answer: B
5. Answer: C
6. Answer: C
7. Answer: A
8. Answer: A
9. Answer: A

https://t.me/DentalBooksWorld
818 Triumph's Complete Review of Dentistry

10. Answer: A
11. Answer: A
12. Answer: A
13. Answer: A
14. Answer: A
Dens invaginatus, Dens in dente, and dilated odontome
Synonyms
Gestant odontome and “tooth within a tooth”
Definition
All the three entities result from varying degrees of invagination or infolding of the enamel surface into the interior of a
tooth. The least severe form of this infolding is dens invaginatus, and the most severe form is the dilated odontome. The
invagination can occur in either the cingulum area (dens invaginatus) or incisal edge (dens in dente) of the crown or in the
root during tooth development. It may also involve the pulp chamber or a root canal system. This may result in a deformity
of either the crown or the root, although these anomalies are seen most often in tooth crowns. Coronal invaginations
usually originate from an anomalous infolding of the enamel organ into the dental papilla. In a mature tooth the result
is a fold of hard tissue within the tooth characterized by the enamel lining the fold. When the abnormality involves the
root, it may be the result of an invagination of Hertwig’s epithelial root sheath and produce an accentuation of the normal
longitudinal root groove. In contrast to the coronal type, which is lined with enamel, the radicular type defect is lined with
cementum. If the invagination retracts and is cut off, it leaves a longitudinal structure of cementum, bone, and remnants
of PDL within the pulp canal. The structure often extends for most of the root length. In other cases the root sheath may
bud off a sac-like invagination that produces a circumscribed cementum defect in the root. Mandibular first premolars and
second molars are especially prone to develop the radicular variety of this invagination anomaly.
15. Answer: D
Dentin dysplasia is rarer than dentinogenesis imperfecta (1:100,000 compared with 1:8,000)
16. Answer: B
Osteogenesis imperfecta is a hereditary disorder characterized by osseous fractures. The pathogenesis is thought to be an
inborn error in the synthesis of type I collagen, which results in brittle bones. It is usually transmitted as an autosomal
dominant trait. Patients may have blue sclera, Wormian bones (bones in skull sutures), skeletal deformities, and progressive
osteopenia. Dentinogenesis imperfecta is found in approximately 25% of cases. In addition, oral findings may include class
III malocclusions and an increased incidence of impacted first and second molars.
17. Answer: C
Dens evaginatus
Synonym
Leong’s premolar
Definition
In contrast to dens invaginatus or dens in dente, dens evaginatus is the result of an outpouching of the enamel organ. The
resultant enamel-covered tubercle usually occurs in or near the middle of the occlusal surface of a premolar or occasionally
a molar. Lateral incisors are most commonly involved, whereas canines are rarely affected. The frequency of occurrence of
dens evaginatus is highest in Asians and Native Americans.
18. Answer: A
Dilaceration
Radiographs provide the best means of detecting a radicular dilaceration. The condition occurs most often in maxillary
premolars. One or more teeth may be affected. If the roots dilacerate mesially or distally, the condition is clearly apparent
on a periapical radiograph. However, when the roots are dilacerated buccally (labially) or lingually, the central X-ray passes
approximately parallel with the deflected portion of the root and the apical end of the root may have the appearance of a
circular or oval radiopaque area with a central radiolucency (the apical foramen and root canal), giving the appearance of a
bull’s eye. The PDL space around this dilacerated portion may be seen as a radiolucent halo encircling the radiopaque area.
In some cases, especially in the maxilla, the geometry of the projections may preclude the recognition of a dilaceration.

MISCELLANEOUS
1. Answer: B
The focal spot is the area of tungsten on the anode that receives the impact of the speeding electrons and converts them
into X-ray photons. It is one of three factors influencing image sharpness (the others are film composition and movement).

https://t.me/DentalBooksWorld
Chapter 9 • Oral Radiology 819

The target (also called the tungsten target) is a wafer of tungsten embedded in the face of the anode at the point of electron
bombardment. Tungsten is used due to its high atomic number (X-ray production), high melting point, high thermal
conductivity (dissipates heat), and low vapor pressure (maintains vacuum, at high temperature) actually a small area of the
target. Remember: Intensifying screens are devices used in extraoral radiography that convert X-ray energy into visible
light. The light, in turn, exposes the screen film. Therefore, the radiation that a patient receives is decreased. A cassette
holder is a light-tight device used in extraoral radiography to hold film and intensifying screens.
Important: Target film distance (also called source-to-film distance) is the distance from the source of X-rays (focal spot
on the tungsten target) to the film. It is determined by the length of the position-indicating device (also called PID). Two
standard target–film distances are used in intraoral radiography:
• 20 cm (8 inches) – + is called the short cone, exposes more tissue by producing a more divergent beam.
• 41 cm (16 inches) – ÷ is called the long cone, reduces the amount of exposed tissue by producing a less divergent beam
and a sharper image.
2. Answer: D
As the mouth is opened, the process moves forward, and therefore it comes into view most often when the mouth is
opened to its fullest extent at the time the exposure is made. It is evidenced by a tapered or triangular radiopacity, which
may be seen below, or in some instances, superimposed on the molar teeth and maxilla.
3. Answer: C
• A developing agent, such as hydroquinone, is a chemical compound that is capable of changing the exposed silver
halide crystals to black metallic silver. At the same time, it produces no appreciable effect on the unexposed silver halide
crystals in the emulsion. Gives detail to the X-ray image. Note: Also Elon which quickly generates gray tones.
• An antioxidant preservative, for example, sodium sulfite, prevents the developer solution from oxidizing in the presence
of air.
• An accelerator – an alkali (sodium carbonate) – activates the developing agents and maintains the alkalinity of the
developer at the correct value. It softens gelatin of emulsion.
• A restrainer, such as potassium bromide, is added to developers to control the action of the developing agent so that it
does not develop the unexposed silver halide crystals to produce fog.
Remember: Developer is a chemical solution that converts the invisible image on a film into a visible one composed of
minute masses of black metallic silver.
Important: The function of developing solution is to reduce silver halide crystals to black metallic silver, while the function
of fixing solution is to stop development and remove remaining unexposed crystals.
Film processing involves the following steps: Immerse film in developer – rinse film in water bath – immerse film in
fixer – wash film in water bath – dry film
4. Answer: C
• A clearing agent, such as sodium or ammonium thiosulfate, commonly called hypo, dissolves and removes the
underdeveloped silver halide crystals from the emulsion (Note: this is one of the main functions of fixing solutions.)
The chemical “clears” the film so that the black silver image produced by the developer becomes distinctly perceptible.
When the film is improperly cleared, the remaining unexposed silver halide crystals darken upon exposure to light and
obscure the image.
• An antioxidant preservative, for example sodium sulfite, prevents the decomposition of the fixer chemical.
• An acidifier such as acetic acid that is necessary for the correct action of the other chemicals and also neutralizes any
alkaline developer that may be carried over by the film or hanger.
• A hardener such as potassium alum that shrinks and hardens the gelatin in the emulsion, it shortens drying time and
protects the emulsion from abrasion.
5. Answer: C
A film will appear brown when it is not completely fixed.
Some common errors made in the darkroom
• Mounted films are improperly labeled (wrong patient name) – + racks not labeled properly
• Fogged film (gray/lack of contrast) – + faulty safelight in darkroom; white light leaking into darkroom
• Lost films –÷ films not secured properly on rack
• Static marks (multiple black lines) – friction when opening film packets causes static electricity
• Overdeveloped film (dark) – incorrect time (too long) and temperature (too hot)
• Underdeveloped film (light) – incorrect time (too short) and temperature (too cold); weak solutions (too old or diluted)
• Torn emulsion films were allowed to touch or overlap while they were drying

https://t.me/DentalBooksWorld
820 Triumph's Complete Review of Dentistry

• Stained film (dark/white spots) — dirty work surfaces; person developing film was sloppy
• Scratched films (white lines) — film emulsion removed by sharp object (fingernails/rack touching)
• Clear films (emulsion washed away) films left in water (wash) for over 24 hours
• Air bubbles (white spots) – + air trapped on film surface while being placed in processing
6. Answer: B
RAD — radiation absorbed dose
The rad is a unit used to measure a quantity called absorbed dose. This relates to the amount of energy actually absorbed
in some material, and is used for any type of radiation and any material. One rad is defined as the absorption of 100 ergs
per gram of material. The unit rad can be used for any type of radiation, but it does not describe the biological effects of the
different radiations.
The rem (Roentgen equivalent man) is a unit used to derive a quantity called equivalent dose. This relates the absorbed dose in
human tissue to the effective biological damage of the radiation. Not all radiation has the same biological effect, even for the
same amount of absorbed dose. Equivalent dose is often expressed in terms of thousandths of a rem, or mrem. To determine
equivalent dose (rem), you multiply absorbed dose (rad) by a quality factor (QF) that is unique to the type of incident radiation.
The QF is a factor used for radiation protection purposes that accounts for the exposure effects of different types of radiation.
For X-rays QF 1. The roentgen is a unit used to measure a quantity called exposure. This can only be used to describe an
amount of gamma and X-rays, and only in air. Exposure is a measure of radiation quantity, the capacity of the radiation to
ionize air. Equivalent dose is used to compare the biologic effects of different types of radiation to a tissue or organ.
Effective dose is used to estimate the risk in humans.
7. Answer: C
Cells in the body have different sensitivities to radiation than others.
Radiosensitive cells:
• Small lymphocytes (immature blood cells)
• Bone marrow
• Reproductive cells (sperm and ova)
• Immature bone cells
Radioresistant cells:
• Mature bone
• Muscle
• Neurons
In general, the greater the rate or potential for mitosis and the more immature the cells and tissues are, the greater the
sensitivity or susceptibility to radiation.
8. Answer: C
Radiation exposure to the operator can be reduced by standing at least 6 feet away, behind a lead shield, or both when
exposing radiographs. The operator should never remain in the room holding the X-ray packet in place for the patient.
If a film must be held in place by someone else (for a child), drape the parent and have him or her hold the film. All
dental personnel should wear film badges that monitor exposure dosages. The operator must avoid the primary beam by
positioning themselves at a 90–135-degree angle to the beam.
Note: Regarding the taking and processing of dental radiographs, always remember to maintain proper infection control
at all times!!!
9. Answer: C
Important point: For a given beam of radiation the intensity is inversely proportional to the square of the distance from the
source of radiation. The intensity of an X-ray beam is the total energy of the X-ray beam; it is the product of the quantity
(number of X-ray photons) and quality (energy of each photon) per unit of area per time of exposure. The intensity of an X-ray
beam at a given point is dependent on the distance of the measuring device from the focal spot. The reason for this decrease
in intensity (why it is inversely proportional) is that the X-ray beam spreads out as it moves from the source. The “spread out”
beam is less intense. For example, when the PID length is changed from 8 to 16 inches, the source-to-film distance is doubled.
According to the Inverse Square Law, the resultant beam is one-fourth as intense. When the PID length is changed from 16
to 8 inches, the source-to-film distance is reduced by one-half. According to the Inverse Square Law, the resultant beam is
four times as intense. Remember: The intensity of the radiation is inversely proportional to the square of the distance.
10. Answer: D
Remember: Kilovoltage controls the speed of electrons. One effect of a change in kilovoltage is a change in the penetrating
power of the X-rays. Increasing kilovoltage reduces subject contrast (and the longer the scale of contrast); decreasing
kilovoltage increases subject contrast (and the shorter the scale of contrast). A second effect of an increase in kilovoltage is

https://t.me/DentalBooksWorld
Chapter 9 • Oral Radiology 821

that not only are new, more penetrating X-rays produced, but more of the less penetrating rays which were also produced
at the lower kilovoltage are omitted. Conclusion: kilovoltage influences the X-ray beam and radiograph by:
• Altering contrast quality (for patients with thick jaws, increase kilo voltage)
• Determining the quality of the X-rays produced
• Determining the velocity of the electrons to the anode
11. Answer: C
For X-ray beams, this is normally expressed in aluminum or copper thickness, but can also be expressed in other materials or
media, such as water. Strictly, the half value layer is defined for different quantities: photon fluence (number of photons/cm2),
energy fluence (number of photons × photon energy/cm2) or absorbed dose. The term intensity is commonly used but is too
vague and should therefore be avoided. Due to the spectral nature of X-rays, the half-value layer (HVL) is not constant. When
measuring multiple half-value layers, the second HVL is greater than the first. This is due to the fact that the mean energy of
the X-ray spectrum is increased following passage of the first HVL, which results in the X-rays becoming more penetrating.
In oral diagnostic radiography, the half-value layer of the beam of radiation is approximately 2 mm of aluminum. This means
that half of the X-rays exiting the vacuum tube are absorbed by 2 mm of aluminum. It should be noted that doubling the
thickness of aluminum will not absorb all of the X-rays, but only half of the remaining X-rays. Important point to remember:
The half-value layer is an indicator of the penetrating quality of an X-ray beam. The higher the half-value layer, the more
penetrating the beam. Note: X-rays and gamma rays are examples of nonparticulate radiation energy.
12. Answer: A
It is attenuated by the filter and the object. The amount of primary radiation follows the inverse square law measured from
the focal spot. The attenuation of primary radiation should be measured with a narrow beam geometry to exclude all
secondary radiation (i.e., scattered radiation). Secondary radiation (scattered radiation) which arises from interactions of
the primary radiation beam with the atoms in the object being imaged. Because the scattered radiation deviates from the
straight line path between the X-ray focus and the image receptor, scattered radiation is a major source of image degradation
in both X-ray and nuclear medicine imaging techniques. When X-ray radiation passes through a patient, three types of
interactions can occur, including coherent scattering (coherent scatter), photoelectric absorption, and Compton scattering.
Of these three events, the great majority of scattered X-rays in diagnostic X-ray imaging arise from Compton scattering.
13. Answer: B
The X-ray spectrum reaching the patient is filtered by attenuating material in its path. Filtering of the beam is used in
order to modify the spectral or spatial distribution of X-rays, or both. Filtration is in principal divided in two parts:
1. Inherent filtration: The filtration of an X-ray beam by any part of the X-ray tube or tube shield through which the beam
must pass. The parts include the glass envelope of the X-ray tube, the oil cooling the tube, and the exit window in the
tube housings. The inherent filtration corresponds to approximately 0.51 mm of aluminum.
2. Added filtration: Obtained by placing thin sheets of aluminum in the cone to filter the useful beam further. The total
filtration of the X-ray beam before it reaches the patient consists of the inherent filtration plus the added filtration.
Recommended total: equivalent of 0.5 mm (below 50 kVp) and 2.5 mm (over 70 kVp) of aluminum.
14. Answer: B
All of the following reduce the amount of radiation to the patient:
• A lead apron
• Increased filtration using an aluminum disk
• Ekta-speed film
• Lead diaphragms placed within the cone of an X-ray tubehead
• Collimating an X-ray beam
• Increasing source–film distance
• Intensifying screens (used for all extraoral radiography — f panoramic, cephs.)
Note: The Committee on Radiation Protection of the National Bureau of Standards recommends that a person who works
near radiation be exposed in 1 year to a maximum dose of 5 REM (0.1 REM per week). Secondary radiation (scatter
radiation) pose the greatest hazard to the dental team. Important: Carcinogenesis and genetic mutation are important and
serious effects of repeated exposure to low doses of X-radiation. The mechanisms involved may be frameshift mutations,
synergism with chemical carcinogens, and altered DNA repair enzyme functions.
15. Answer: B
Collimation refers to the control of the size and shape of X-ray beam. In X-ray imaging systems, a collimator
mounted to the X-ray tube is used to define the dimensions of the beam which is to be incident on the subject and the
detector. To minimize radiation dose and to comply with government regulations, a certain level of precision must

https://t.me/DentalBooksWorld
822 Triumph's Complete Review of Dentistry

be maintained. It is a basic rule of radiation hygiene that the radiation beam be as small as practical. For intraoral
radiography, by state law, the diameter of a circular beam of radiation at the patient’s skin can be no greater than
2.75 inches. One can use a diaphragm or metal cylinders, cones, or tubes to collimate the beam. These devices do not
reduce the amount of radiation received by the exposed tissues, but reduce the radiation to surrounding tissues due to
X-ray beam divergence.
16. Answer: C
Five rules for accurate image formation when taking X-rays:
1. Use the smallest focal spot that is practical. Note: The size of the focal spot influences radiographic definition or
sharpness. They are inversely proportional. The operator cannot control the size of the focal spot.
2. Use the longest source–film distance that is practical in that particular situation.
3. Place the film as close as possible to the structure being radiographed.
4. Direct the central ray at as close to a right angle to the film as anatomical structures will allow.
5. As far as is practical, keep the film parallel to the structure being radiographed.
17. Answer: B
Vertical angulation is directing X-rays so that they pass vertically through the part being examined. This is accomplished
by positioning the tubehead and direction of the central ray in an up-and-down (vertical) plane. Important: Foreshortening
(See Figure 1) refers to a shortened image and elongation (Figure 2) refers to an elongated image. Both are produced
by an incorrect vertical angulation. Excessive vertical angulation causes foreshortened images, while insufficient vertical
angulation causes elongated images.
Horizontal angulation is maintaining the central ray at 0 degrees as the tube is moved around the head. This is accomplished
by positioning the tubehead and direction of the central ray in a side-to-side (horizontal) plane. Note: The general rule for
horizontal angulation is that the central ray should be perpendicular to the mean anteroposterior plane of the teeth being
X-rayed. Important: Incorrect horizontal tube angulation causes overlapping (teeth images are superimposed on each other). The
central ray is said to be at 0 degrees when the X-ray tube is adjusted so that the central ray is parallel to the floor. If the tubehead
is directed at the floor, it is called positive angulation; if it is directed toward the ceiling, it is called negative angulation.
18. Answer: A
Mandibular structures look narrower and maxillary structures look wider (looks like a “frown”). Chin tilted too far\upward.
I. Occlusal plane shows an excessive upward curve (looks like a “big smile”). See figure below.
Severe interproximal overlapping, anterior teeth appear very distorted.
O. Overbent film — cracked emulsion.
Some other common errors made when taking dental radiographs cause:
• Light films (underexposed/image NOT dense enough) – k incorrect milliamperage (too low) or exposure (too short);
incorrect focal film distance; cone too far from patient’s face, film placed backward. See Figure 1.
• Dark films (overexposed/image too dense)- ÷ incorrect milliamperage (too high), exposure (too long), incorrect kVp
(too high). See figure 2.
• Double exposure – k film was used twice.
• Fogged films – exposed to radiation other than primary beam. See figure 3.
• O Artifacts –÷ patient did not remove eyeglasses, earrings, or removable prosthetic appliances.
• Poor contrast –k incorrect kVp (too high).
• Blurred image –÷ patient movement or drifting of X-ray arm.
• Clear films – were not exposed to radiation.
19. Answer: C
The paralleling technique is based on the concept of parallelism. Other names for this technique
include XCP (extension cone paralleling technique), right-angle technique, and long-cone technique.
Basic Principles:
• Film is placed parallel to the long axis of the tooth being X-rayed.
• Central X-ray is directed perpendicular to both the film and the long axis of the tooth.
• A film holder (XCP) must be used to keep the film parallel to the long axis of the tooth.
• The object–film distance must be increased to keep the film parallel. This results in image magnification and loss of
definition.

https://t.me/DentalBooksWorld
Chapter 9 • Oral Radiology 823

• The source–film distance must also be increased to compensate for the image magnification and to make sure that only
the most parallel rays will be aimed at the tooth and the film. Using a long cone (16 inch target–film distance) results in
greater definition and less image magnification.
20. Answer: B
The exposure time is actually decreased. The bisecting angle technique is based on the geometric principal known as the
rule of isometry. The rule states that two triangles are equal if they have two equal angles and share a common side.
The following best describes the bisecting technique:
• The dental X-ray film is placed along the lingual surface of the tooth.
• At the point where the film contacts the tooth, an angle is formed by the plane of the film and the long axis of the tooth.
• The person taking the X-ray needs to visualize a plane that bisects this angle. This plane is called the imaginary bisector;
this creates two equal angles and provides a common side for the two imaginary equal triangles.
• The central ray is positioned perpendicular to the imaginary bisector.
Important: When this technique is followed strictly, the image of the tooth produced is accurate.
21. Answer: B
For this projection the neck is maximally extended and the film cassette touches the top of the head. The X-ray beam
enters the head under the chin (near the mental tubercle of the mandible) and exits at the vertex. This view is used in
conjunction with other project ions, and allows direct visualization of the base of the skull. The zygomatic arches stand out
like the handles of a jug on this view.
22. Answer: C
This is a posterior–anterior projection with the patient’s face lying against the film and the X-ray source behind the patent’s
head. Waters projection is the most useful conventional radiographic technique to image the maxillary sinuses. In this
projection, the radiographic densities of normal maxillary sinuses are the same on both sides and equal to those of the
orbits. If one of the sinuses is diseased, Waters projection will exhibit either a radiopaque (fluid) level, a sinus opacification,
mucosal hyperplasia, a radiopaque growth, or a loss of cortical borders of sinus. Other useful projections include
periapical, panoramic, occlusal, lateral head, and Caldwell. It is also one of the best films for radiographic diagnosis of
mid-facial fractures.
23. Answer: C
The patient lies on his back with the film under his head. The X-ray source is from the front, but rotated 30 degrees
from the Frankfort plane and is directed right at the condyles. Towne’s projection is often of value in assessing the status
of the condyles, condylar neck, and rami because superimposition of the mastoid and zygoma over the condylar neck
region in the straight posteroanterior projection often makes interpretation difficult. Towne’s projection eliminates this
superimposition, thus giving good visualization of the condylar area and rami.
24. Answer: C
This statement is false; a panoramic radiograph gives less detail and definition than periapical radiographs due to
intensifying screens, movement of the X-ray tube and film and increased object–film distance.
Indications for a panoramic radiograph:
• Diagnosis of oral pathology that may not be visible on periapical radiographs
• Treatment planning (especially orthodontic cases)
• Evaluation of anomalies
• As one part of the follow-up evaluation in surgical and trauma cases
• Edentulous patients (prior to constructing full dentures)
• Patients that are unable to tolerate intraoral X-rays
• Disadvantages of panoramic radiograph
• The drawback of a panoramic radiograph is that there is a loss of image detail (it is hard to diagnose early carious
lesions). Bitewing X-rays are required for the diagnosis of carious lesions.
• Other disadvantages of a panoramic radiograph:
• Distortion of image due to increased object–film distance
• Inadequate for interproximal caries detection or for detecting periodontal breakdown (bone loss)
• Proximal overlapping (especially in premolar and molar areas)
• Added exposure to a large area of body tissue, in addition to the oral tissues
25. Answer: B

https://t.me/DentalBooksWorld
824 Triumph's Complete Review of Dentistry

Because of the relative diminished X-ray absorption, these areas appear relatively radiolucent with ill-defined margins. It
is caused by the normal configuration of the affected teeth (the cementoenamel junction), which results in decreased X-ray
absorption in those areas. Important: These radiolucencies should be anticipated when viewing X-rays of almost any tooth
and should not be mistaken for a carious lesion.
26. Answer: B
These X-rays show the crowns of both Max. and Mand. teeth; not root apices. The primary reason for taking bitewing
radiographs is to detect interproximal caries They are also useful in monitoring the progression of periodontal disease.
These films show crestal bone levels as well as interproximal areas of both arches. In order for the film to be of diagnostic
use, the quality of the following must be excellent: dimensional accuracy, open contacts, and optimum contrast and clarity
of the image. When taking bitewing radiographs, the film must be placed in either a horizontal or vertical posit ion.
Vertical bitewings provide more periodontal information, such as bony defects and function involvement. A fuzzy or
indistinct image of crestal bone is often associated with early periodontitis. Two bitewings are usually taken on a child, one
on each side. If the child has primary dentition only, number “0” film is used. If the child has mixed dentition, number “1”
film is utilized. Once the individual has second molars, two to four number “2” films are conventionally utilized. If using
four films, one film images the premolar area, while the other images the molar area. Sometimes two, long, number “3”
films are utilized (one for each side) instead of two number “2” films on each side. This practice is not recommended due to
the curvature of the arch making it difficult to open all contacts on one film.
27. Answer: A
The latent period is the period of time between radiation exposure and the onset of symptoms. It may be short or long,
depending on the total dose of radiation received and the amount of time it took to receive the dose. The period of cell
injury follows the latent period. Cellular injury may result in cell death, changes in cell function, or abnormal mitosis of
cells. The recovery period is the last event in the sequence of radiation injury. Some cells recover from the radiation injury,
especially if the radiation is “low level.”Note: The
effects of radiation exposure are additive and the damage that remains, cumulative effects of repeated radiation unrepaired,
accumulates in the tissues. The exposure can lead to various serious health ,leads to various carcinomas, genetic mutations
of leukemia and cataracts).
28. Answer: C
To produce a direct digital X-ray image, three components are necessary: an X-radiation source, a sensor, and a
computer. The images are captured using a solid-state detector or sensor such as a charge-coupled device (CCD), a
complementary metal oxide semiconductor I active pixel sensor (CMOS/APS), or a charge injection device (CID). Most
direct digital systems use a CCD device. CCD, CMOS, and CID sensors are referred to as “wired” because they are
linked by a fiberoptic cable to the computer. The sensor itself is basically a silicon chip with an electronic circuit on it.
The CCD is the most common device used today. Its sensor is about the same size as a #2 intraoral film and is connected
to the computer by an optic fiber wire in most cases. Microwave technology has been used with CCDs, eliminating the
need for the wire tether, but requires additional electronic components. The CCD consists of a silicon chip with an active
array of rows and columns called pixels (picture elements). These pixels are analogous to silver crystals in conventional
films but are 80% more sensitive to radiation; thus the reduction in radiation dose to patients. Smaller pixel sizes mean
more pixels fit onto the sensor, which decreases the size of the receptor but increases the cost. CCDs are available in
large enough sizes to accommodate panoramic films, and manufacturers have made CCDs that can be retrofitted to
existing units to ease the cost of equipment conversion. With cephalometric imaging, a CCD large enough to capture
an entire skull is too expensive to manufacture and a different technology has been employed that involves a CCD array
and a scan lasting several seconds. The main advantage of digital radiography over traditional film is that digital images
are available immediately. This saves time in treatment requiring progress films or if a shot needs to be retaken. Patient
education is improved by an image displayed immediately on a chairside monitor, rather than making the patient leave
the chair to squint into a viewbox at a miniscule shadow. The ability to enhance an image, particularly by increasing
brightness or contrast, is a tremendous benefit to the practitioner, since often a film is not viewed until after the patient
has left the office.
29. Answer: C
Many advantages have been ascribed to digital radiography. First, they allow a reduction in the amount of radiation
reaching the patient, which is always a concern. With direct systems, images are displayed immediately on the computer
monitor, so less chair time is required during diagnosis and treatment appointments. This is a particularly attractive
feature for clinicians performing endodontic treatment where several images are usually made during an appointment.
Clinicians also find it useful to be able to manipulate the image because it enhances diagnosis. Image storage and electronic
transmission are possible, and patients can be educated about their diagnosis and treatment using the images. Many users

https://t.me/DentalBooksWorld
Chapter 9 • Oral Radiology 825

appreciate the fact that there is a lot less mess associated with producing digital images than conventional ones, because
there is no need for film, film processors, processing chemicals, darkrooms, or film mounts. Of course, no technology is
without some disadvantages. Commonly mentioned ones for digital radiography include the high initial setup cost, the
need for staff training, and the bulkiness (i.e., thickness) of the sensors. Note: The sensor itself is basically a silicon chip
with an electronic circuit on it. Sensors range in thickness from 3.2 to 8.8 mm.
30. Answer: A
There are three options available for capturing a digital X-ray image: indirect, direct, and storage phosphor imaging.
Indirect digital X-ray images are produced by placing a conventional X-ray film on a desktop scanner and allowing
a transparency adapter to shine light through the image as it is scanned into the computer. This converts the original
analog image (i.e., dental radiograph) into a digital image by scanning. Once digitized, the image can be processed like
any other digital image. To produce a direct digital X-ray image, three components are necessary: an X-radiation source,
a sensor, and a computer. The images are captured using a solid-state detector or sensor such as a charge-coupled device
(CCD), a complementary metal oxide semiconductor I active pixel sensor (CMOS IAPS), or a charge injection device
(CID). A third method of obtaining a digital image is storage phosphor imaging, a wireless digital radiography system.
In this system, a reusable imaging plate coated with phosphors is used instead of a sensor with a fiber optic cable.
The plates are described as “wireless” because they are not connected via cable or wire to the computer. The plates are
similar in every way to conventional intraoral film, including size, thickness, rigidity, and placement. These plates store
the energy from incoming X-rays, and are then placed in a scanning device. The scanner stimulates the stored X-ray
information by subjecting the plate to a laser light. When the light strikes the phosphor material, energy is released
as a light signal in an electronic waveform and is converted to a digital image by the computer. The image cannot
instantaneously be viewed on the monitor, but takes from 30 seconds to 5.5 minutes depending upon the system and
certain variables.
31. Answer: C
One of the positive features of digital radiography is that it requires less radiation than conventional radiography, because
the sensor is more sensitive to X-rays than dental film. Exposure times for digital radiography are from 50 to 80% shorter
than those for E-speed film. This translates into less radiation exposure for the patient.
Notes
1. All direct and PSP digital radiography systems use a conventional dental X-ray unit. The literature emphasizes that the
X-ray unit must have the ability to reduce exposure times to 0.01 seconds to reduce the likelihood of oversaturating
the sensor.
2. In digital radiography, a sensor, or small detector is placed inside the mouth of the patient to capture the radiographic
image. The sensor is used instead of an intraoral film. As in conventional radiography the X-ray beam is aimed to
strike the sensor. An electronic charge is produced on the surface of the sensor; this electronic signal is digitized, or
converted into “digital” form.
3. Digital radiography systems are not limited to intraoral images: panoramic and cephalometric images may also be
obtained.
32. Answer: B
Some errors often made when taking dental radiographs:
• Elongation (most common error) –+ teeth appear too long – may be caused by too little vertical angulation, the film not
parallel to the long axis of the teeth or the occlusal plane not being parallel to the floor.
• Foreshortening –+ teeth appear too short – may be caused by too much vertical angulation or poor chair position.
• Cone cutting –+ portion of film will appear clear with a curved line – the beam was not aimed at the center of the film.
See figure 1.
• Herringbone effect –+ zigzagged pattern appears on the film – the film was placed backward in the mouth.
• Poor film placement –+ the film was not placed far enough back or not forward enough in the mouth. See figure 2.
• Overlapping –+ interproximal areas are overlapped, reduces diagnostic quality of film – due to incorrect horizontal
angulation (the central X-ray was not directed perpendicular to the curvature of the arch and through the contacts). See figure 3.
33. Answer: C
34. Answer: A
35. Answer: B
36. Answer: C
37. Answer: A
38. Answer: B

https://t.me/DentalBooksWorld
826 Triumph's Complete Review of Dentistry

39. Answer: A
40. Answer: A
41. Answer: A
42. Answer: D
43. Answer: D
44. Answer: C
45. Answer: B
46. Answer: A
47. Answer: D
48. Answer: C
49. Answer: D

https://t.me/DentalBooksWorld
10 Oral Anatomy and Histology

SYNOPSIS

TERMINOLOGIES IN TOOTH MORPHOLOGY


PARTS OF A TOOTH
Crown – Crown is the portion of the tooth which is covered by enamel
Anatomical crown Crown (entire) which is covered by enamel, regardless of whether it is completely erupted or not.
The size of the anatomical crown remains constant throughout the lifetime of the tooth, except in
cases of attrition or any other physical wearing of tooth surface
Clinical crown Crown which is visible clinically, and it is what one sees when looking into the mouth. The height
of the clinical crown is determined by the position of the gingival margin. The clinical crown may
be smaller than the anatomical crown, such as in a newly erupted tooth where only a part of the
anatomical crown is visible and the cervical portion of the crown is still covered by the gingiva.
The clinical crown can also be larger than the anatomical crown in cases where there has been
recession of the gingiva with root exposure
Root – The portion of the tooth covered by cementum is known as the root
Anatomical root Root that is below the CEJ and is covered with cementum, a bone-like substance that facilitates
anchorage of the tooth in its bony socket (the alveolus). The size of the anatomical root remains
constant throughout the lifetime of the tooth, except in cases of resorption of the root.
Clinical root Part of tooth that is under the gingiva and not exposed to the oral cavity. The height of the
clinical root is determined by the position of the gingival margin. It may be longer than the
anatomical root such as in newly erupted teeth where the part of the crown that is not erupted
is considered part of the clinical root. It may also be shorter than the anatomical root in
elderly persons, where there is gingival recession and part of the anatomical root is exposed
to the oral cavity.
Cervical line – This line separates the anatomical crown from the anatomical root. It is the junction between two tissues of
the tooth, the enamel and the cementum, and hence is known as the cementoenamel junction.
Enamel – Enamel is the hard, highly mineralized, protective, white, shiny structure covering the outermost layer of the
anatomical crown. It is the hardest substance in the human body.
Dentin – Dentin is the hard, mineralized, yellowish tissue present in both the crown and the root. It makes up the major bulk
of the tooth, is present beneath the enamel and cementum, and lines the pulp cavity.
Cementum – Cementum is the mineralized, dull, yellowish tissue covering the outermost layer of the anatomical root
extending from the cervical line.
Pulp – Pulp is the soft tissue present in the central, innermost portion of the tooth called the pulp cavity. It is completely
covered by the dentin layer except near the apical foramen.

https://t.me/DentalBooksWorld
828 Triumph’s Complete Review of Dentistry

SUPPORTING STRUCTURES OF THE TOOTH

Alveolar bone – The portion of the jaw (maxillary or mandibular) which anchors and supports the teeth is known as the
alveolar process or the alveolar bone.
Periodontal ligament – Periodontal ligament develops from the dental sac (mesoderm) and is composed of periodontal
ligament fibers attached to the alveolar bone on one side and the cementum of the tooth on the other side.
Gingiva – The soft tissue that partly covers the crown portion, and sometimes part of the cervical cementum.

SURFACES OF THE TEETH

Facial surface – The surface of the tooth nearest to the cheeks or lips is referred to as the facial surface. The facial surface can
be subdivided into buccal surface and labial surface.
Palatal surface – The surface of the tooth closest to the palate is termed palatal surface.
Lingual surface – The surface of the tooth closest to the tongue is termed lingual surface.
Proximal surface – The surface of a tooth that is toward another tooth in the dental arch is termed proximal surface. Proximal
surface can be subdivided into two surfaces on the basis of position in relation to the median line of the face. Mesial surface
is the surface that is toward or closer to the median line of the face. Distal surface is the surface that is away or distant from
the median line of the face.
Masticatory surface – The surface that aids in chewing is known as the masticatory surface. The masticatory surface is
subdivided into the occlusal surface in posterior teeth and the incisal surface in anterior teeth.

DIVISION INTO THIRDS

Division of the Crown into Thirds


Cervico-occlusally/cervico-incisally – Occlusal or incisal third/Middle third/Cervical third
Faciolingually – Facial (labial/buccal) third/Middle third/Palatal or lingual third
Mesiodistally – Mesial third, Middle third, Distal third
Division of the Root into Thirds
Cervico-apically – Cervical third, Middle third, Apical third
Faciolingually – Facial (labial/buccal) third, Middle third, Palatal/lingual third
Mesiodistally – Mesial third, Middle third, Distal third

LINE ANGLES

A line or a point where two or more surfaces or borders meet is known as an angle. A line angle is an angle formed by
the  junction or union of two surfaces and the name is derived from both the surfaces. For example, the junction between
the labial and distal surface is termed distolabial line angle. Anterior teeth have six line angles and posterior teeth have eight
line angles

https://t.me/DentalBooksWorld
Chapter 10 • Oral Anatomy and Histology 829

Line angles of anterior teetha Line angles of posterior teeth


• Mesiolabial • Mesiobuccal
• Mesiopalatal/mesiolingual • Mesiopalatal/mesiolingual
• Distolabial • Distobuccal
• Distopalatal/distolingual • Distopalatal/distolingual
• Labioincisal • Bucco-occlusal
• Linguoincisal • Linguo-occlusal
• Mesio-occlusal
• Disto-occlusal

aSince the mesioincisal and distoincisal line angles are rounded, they are not considered as line angles.

Linguo-occlusal line angle


a l Occlusal Disto-occlusal line angle
Incis Labio-incisal line angle
Bucco-occlusal line angle
Linguo-incisal line angle
Mesio-occlusal line angle
Mesiolabial line angle Mesiolingual line angle
Distolabial line angle Distolingual line angle
Distolingual line angle Mesiobuccal line angle
Mesiolingual line angle Distobuccal line angle
al
Dist

Di
ual

sta
Ling
Lin

l
gu
al
La

cal
M

sial Buc
bia

es

Me
l

ial

POINT ANGLES

A point angle is an angle formed by the junction or union of three surfaces, and the name is derived from the three surfaces.
For example, the junction between labial, mesial, and incisal surface is termed mesio-labio-incisal point angle. Anterior and
posterior teeth have four point angles each.
Table: Point Angles
Point angles of anterior teeth Point angles of posterior teeth
• Mesio-labio-incisal • Mesio-bucco-occlusal
• Mesio-palato-incisal/mesio-linguo-incisal • Mesio-palato-occlusal/mesio-linguo-occlusal
• Disto-labio-incisal • Disto-bucco-occlusal
• Disto-palato-incisal/disto-linguo-incisal • Disto-palato-occlusal/disto-linguo-occlusal

Occlusal
al
Incis Mesio-linguo-occlusal point angle
Mesio-labio-incisal point angle Disto-linguo-occlusal point angle
Mesio-linguo-incisal point angle Disto-bucco-occlusal point angle
Disto-linguo-incisal point angle Mesio-bucco-occlusal point angle
Disto-labio-incisal point angle

al ual
Di

Dist
Lin

Ling
sta
gu

l
al

sial cal
La

Buc
M

Me
bia

es
ial
l

https://t.me/DentalBooksWorld
830 Triumph’s Complete Review of Dentistry

ANATOMICAL LANDMARKS ON CROWN SURFACE

Landmark Description
Cusp A cusp is an elevation on an occlusal surface of posterior teeth and canines. It contributes to a significant
portion of the tooth surface and divides the occlusal surface of posterior teeth
Tubercle Tubercle is a small, rounded prominence on the surface of tooth resulting from the extra formation of
enamel
Cingulum Cingulum is a bulge or elevation on the lingual surface of incisors or canines
Lobe Developmental lobe is the growth center in the development of the crown of the tooth. Each tooth begins
to develop from four or more developmental lobes, and they appear as cusps and mamelons on the tooth
surface
Mamelons Mamelons are the protrusions seen in the incisal ridges of newly erupted permanent incisor teeth
Ridges Ridges are any linear, flat elevations on teeth, and they are named according to their location or form
(Types of ridges – Labial, Buccal, Incisal, Linguoincisal, Cervical, Marginal, cuspal, triangular, transverse,
oblique)
Fossa Fossa is an irregular, rounded depression or concavity found on the lingual surface of anterior teeth and
occlusal surface of posterior teeth (Types of fossa – lingual, central, distal, triangular)
Sulcus Sulcus is a long linear depression or valley in the occlusal surface of posterior teeth, the inclines of which
meet at an angle to form a developmental groove
Developmental The developmental groove is a sharply defined, narrow and linear depression formed during tooth
groove development, usually separating the lobes or the major portions of a tooth
Pits Pits are small, pinpoint depressions usually seen in the fossa of posterior teeth. They are found at the ends
or cross-sections of the developmental grooves
Maxillary teeth

Central pit
Mesial pit
Third molar Second molar First molar Distal pit

Third molar Second molar First molar

Mandibular teeth

Fissures A fissure is a sharp crevice between cusp and ridges, formed because of the faulty union of the enamel of
the different lobes; it is formed at the bottom of the developmental grooves
Contact area Contact areas/points are the crests of curvature on the proximal surface of two adjacent teeth in the same
dental arch that come in contact with each other. In newly erupted teeth, the contact is small in size and
is termed contact point. As age advances, because of constant rubbing of the proximal surface, the contact
becomes broad and is termed contact area

https://t.me/DentalBooksWorld
Chapter 10 • Oral Anatomy and Histology 831

Interproximal Interproximal space is the triangular or V-shaped space cervical to the contact area. The proximal surface
space of the adjacent teeth form the sides of the triangle, the contact area forms the apex, and the alveolar bone
forms the base of the triangle
Maxillary teeth

Mandibular teeth

Embrasures When two teeth in the same arch are in contact, their curvatures adjacent to the contact areas form
V-shaped spillway spaces or triangular spaces known as embrasures (Types – Facial, lingual/palatal,
occlusal, gingival)
Labial embrasure
e
asur

Maxillary teeth
mbr

Interproximal space Interproximal space


Bucc
cal e

Palatal
al em
Buc

embrasure
bras
ure

Occlusal embrasure Incisal embrasure Occlusal embrasure

Lingual
e
Buc

sur

embrasure
bra
cal

l em
em

Interproximal space Interproximal space


bra

cca

Mandibular teeth
su

Bu
re

Labial embrasure

ANATOMICAL LANDMARKS ON ROOT SURFACE

Apex The terminal end of the root portion of the tooth is termed the apex.
Apical foramen The apical foramen is the opening of the pulp canal at the apical end of the root through which blood
and accessory and nerve supply pass into the pulp canal. The accessory foramen is a channel leading from the root
foramen pulp laterally through the dentin to the periodontal tissue; it may be found anywhere in the tooth root
but is more common in the apical third of the root.
Root trunk In multirooted teeth, the undivided cervical portion of the root is termed root trunk.

https://t.me/DentalBooksWorld
832 Triumph’s Complete Review of Dentistry

Furcation The division of the root is termed furcation. It is called bifurcation if the root is divided into two parts
and trifurcation if the root is divided into three parts.

Root trunk

Root furcaon

EVOLUTION OF TEETH AND COMPARATIVE DENTAL ANATOMY


ORIGIN AND EVOLUTION OF MOLARS IN MAMMALS
Concrescence Theory Differentiation Theory
Kukenthal and Rose postulated this theory in the year Cope and Osborn postulated this theory in the year 1880
1890. This theory urges that the posterior teeth originated and it was further updated by Gregory in 1934. This theory
by the fusion of two or more conical shaped teeth. This was suggests that conical tooth additional projections or buds
supported with an evidence in the animal dugong were develop giving rise to the triconodont shape. These cones
several enamel organs fuse to form the molar teeth. However, further shift to give rise to separate tubercles or cusps arranged
there were no embryological evidences and the condition in a triangle called tritubercular position. This later forms
found in dugong is considered as an exception, so this theory additional cusps or folds, giving rise to the varied types of
was not considered. mammalian posterior teeth. Due to abundant embryological
and paleontological evidences, this theory has been in favor.

SEQUENCE OF EVOLUTION
1. Maxillary Teeth: Protocone – mesial to protocone paracone – distal to protocone – metacone – (on mesial and distal aspect/ –
hypocone–protoconule–metaconule) – cusps along the buccal margin – (on buccal aspect/ – parastyle, mesostyle, metastyle)
2. Mandibular Teeth: Protoconid – paraconid – metaconid – distal talonid (hypoconid–entoconid–hypoconulid)

Five-cusped first molar teeth in humans (due to loss of paraconid)


Four-cusped second molar teeth in humans (due to loss of hypoconulid)

Classification Based on Succession or Replacement of Teeth


S. No. Type Description
1. Monophyodont Teeth appear only once in lifetime. If lost, they are never again replaced by new ones.
2. Diphyodont Has two sets of dentition – the first temporary set of teeth is called deciduous teeth or milk
teeth or lacteal teeth; they are lost and replaced by a second permanent teeth.
3. Polyphyodont Teeth that are replaced continuously throughout life.

Classification Based on Kinds of Teeth – Form and Function of Dentition


S. No. Type Description
1. Homodont or Teeth which are functionally and anatomically of the same type, although their size may be
isodont variable depending on the location. Example: Fishes, amphibians, reptiles
2. Specialized Occurs in homodont dentition where a functional specialization is required like a venomous
homodont fang in the anterior teeth. Example: Venomous Snake
3. Heterodont Dentition occurs in mammals in which there are four functionally different types of teeth,
namely, incisors for cutting, canines for tearing flesh, and large and broad premolars and
molars with flat grinding surface.

https://t.me/DentalBooksWorld
Chapter 10 • Oral Anatomy and Histology 833

Classification Based on the Type of Attachment of Teeth on the Jaw Bone


S. No. Type Description
1. Acrodont Teeth are attached on the top surface of the jaw bone
2. Pleurodont Teeth are attached on the inner side and upper side of the jaw bone
3. Thecodont Tooth is firmly fixed in a socket of the jaw bone

Acrodont Pleurodont Thecodont

Classification Based on Structure of Cusps in Posterior Teeth


S. No. Type Description
1. Tribosphenic type Triangular pattern with three cusps – the Paracone and metacone on the buccal aspect and
the protocone
2. Quadrate type Quadritubercular and has an additional fourth cusp on the lingual side called the hypocone
3. Secodont Teeth having sharp cutting edges that acts like scissors
4. Bunodont Teeth with smaller cusps or tubercles
5. Brachydont Small and low crowned with crescent-shaped cusps
6. Hypsodont Teeth with large crown and short open roots
7. Selenodont Teeth have cusps in the form of crescent shape
8. Lophodont Teeth has ridges on the occlusal surface

DENTAL FORMULA – COMPARATIVE CHART


S. No. Species Formula
1. Man I – 2/2   C – 1/1   P – 2/2   M – 3/3 = 16 × 2 = 32
2. Dog I – 3/3   C – 1/1   P – 4/4   M – 2/3 = 21 × 2 = 42
3. Cat I – 3/3  C – 1/1  P – 3/2  M – 1/1 = 15 × 2 = 30
4. Horse I – 3/3  C – 1/1   P – 3 or 4/3  M – 3/3 = 20 (or 21) × 2 = 40 (or 42)
5. Cattle I – 0/4  C – 0/0  P – 3/3   M – 3/3 = 16 × 2 = 32
6. Sheep I – 0/4  C – 0/0  P – 3/3  M – 3/3 = 16 × 2 = 32
7. Pig I – 3/3  C – 1/1  P – 4/4  M – 3/3 = 22 × 2 = 44

NOMENCLATURE OF HUMAN TEETH

ESSENTIAL FUNCTIONS OF TEETH


• For acquiring and chewing food and helping in mastication and digestion
• For protecting the oral cavity by acting as a hard physical barrier
• For phonation and speech
• For aesthetic purposes
• For self-protection

https://t.me/DentalBooksWorld
834 Triumph’s Complete Review of Dentistry

Dentition is the term used to describe all the teeth present in the upper and lower jaw bones. In their lifetime, humans develop
two sets of dentitions – primary and permanent dentition.

Anterior teeth

Central incisors
On the basis of the number and type of teeth present, the dental formula
Lateral incisors
Canines
for primary dentition is
First molars I 2–₂ C −| M 2–₂ = –⁵₅ = 10
Second molars
In this formula, I represents incisors, C represents canine, and M
Posterior teeth

Posterior teeth
represents molars, and the formula is read as incisors, 2 in maxillary
Second molars
and 2 in mandibular; canine, 1 in maxillary and 1 in mandibular; and
First molars molars, 2 in maxillary and 2 in mandibular; so a total of 10 teeth are
Canines present on one side whether right or left.
Lateral incisors
Central incisors

Anterior teeth

Anterior teeth

Central incisors
Lateral incisors On the basis of the number and type of teeth present, the dental formula
Canines
First premolars
for permanent dentition is:
Second premolars
First molars I 2–₂ C −| PM 2–₂ M –₃3 = –₈8 = 16
Second molars
In this formula, I represents incisors, C represents canine, PM represents
Posterior teeth

Posterior teeth

Third molars
premolars, and M represents molars, and the formula is read as incisors,
Third molars
2 in maxillary and 2 in mandibular; canine, 1 in maxillary and 1 in
Second molars
First molars
mandibular; premolars, 2 in maxillary and 2 in mandibular; and molars,
Second premolars 3 in maxillary and 3 in mandibular; so a total of 16 teeth are present on
First premolars
Canines
one side whether right or left.
Lateral incisors
Central incisors

Anterior teeth

TOOTH IDENTIFICATION SYSTEMS/TOOTH NUMBERING SYSTEMS

Human dentition is classified into primary and permanent dentition on the basis of eruption sequence and time. Since human
dentition includes 20 primary teeth and 32 permanent teeth it is mandatory to have a specific nomenclature for each tooth
which is based on the type of dentition (primary/permanent), arch (maxillary/mandibular), class (incisor/canine/premolar/
molar), side (right/left), position (central/lateral/first/second/third).

TIMELINE OF TOOTH NUMBERING SYSTEMS


1861 Palmer Notation – Palmer notation (named after Ohio dentist Dr. Corydon Palmer) is a system used
by dentists to associate information to a specific tooth. Also known as the Military System. Although
supposedly superseded by the FDI World Dental Federation notation, it overwhelmingly continues to be the
preferred method used by orthodontists, dental students, and practitioners in the United Kingdom.
It was originally termed the Zsigmondy system after the Hungarian dentist Adolf Zsigmondy who
developed the idea in 1861, using a Zsigmondy cross to record quadrants of tooth positions.

https://t.me/DentalBooksWorld
Chapter 10 • Oral Anatomy and Histology 835

1885 The Hillischer system (Hillischer, 1885) of notation refers to the tooth type; it is more logical but very
confusing because of the use of colons and semi colons to distinguish primary and permanent teeth.
1890 The change found in the Mons Dubois system (How, 1890) is the use of even numbers and odd numbers,
which represent the right and left quadrant, respectively.
1882, 1883 The universal numbering system (Parreidt, 1882; Cunningham, 1883) is the official tooth designation
system in the United States, adopted by the American Dental Association since 1975.
1891 The Victor Haderup system (Haderup, 1891) is similar to the Zsigmondy–Palmer notation
1962 The Frykholm and Lysell system (Frykholm and Lysell, 1962)
1966 The Federation Dentaire Internationale (FDI) system (Viohl, 1966)
2007 The Woelfel system is similar to the universal numbering system
2011 The MICAP Notation (Akram et al., 2011) is a recently developed system
2015 Novel “Havale’s Alphanumeric dental notation” system for primary teeth

For routine usage and easy communication a simplified numbering system came into force. There are three universally recognized
tooth numbering systems namely Zsigmondy–Palmer system (1861), Universal system (1882), and FDI system (1971).

System Primary dentition Permanent dentition Advantages


Universal system It is easy
Maxillary right second premolar
Maxillary right central incisor
Maxillary right second molar

Maxillary right lateral incisor

Maxillary le central incisor

Maxillary le second premolar


Maxillary le lateral incisor

Maxillary le second molar

Maxillary right central incisor


to use with
Maxillary right first premolar
Maxillary right second molar

Maxillary right lateral incisor


Maxillary right first molar

Parreidt, 1882

Maxillary le central incisor

Maxillary le first premolar

Maxillary le second molar


Maxillary le lateral incisor
Maxillary le first molar

Maxillary right third molar

Maxillary right first molar

computing

Maxillary le third molar


Maxillary right canine

Maxillary le first molar


Maxillary le canine

Maxillary right canine data on

Maxillary le canine
–Each tooth is computers.
assigned a specific It can be
alphabet or used in word
number. processing
software.

A B C D E F G H I J
T S R Q P O N M L K 1 2 3 4 5 6 7 8 9 10 11 12 13 14 15 16

32 31 30 29 28 2726 25 242322 21 20 19 18 17
Mandibular le central incisor
Mandibular right central incisor

Mandibular le lateral incisor


Mandibular le canine
Mandibular le first molar

Mandibular le second molar


Mandibular right second molar

Mandibular right first molar


Mandibular right canine
Mandibular right lateral incisor

Mandibular right third molar


Mandibular right second molar
Mandibular right first molar
Mandibular right second premolar
Mandibular right first premolar
Mandibular right canine

Mandibular le canine
Mandibular le first premolar
Mandibular le second premolar
Mandibular le first molar
Mandibular le second molar
Mandibular right lateral incisor
Mandibular right central incisor
Mandibular le central incisor
Mandibular le lateral incisor

Mandibular le third molar

https://t.me/DentalBooksWorld
836

system
Palmer

software

Dentaire
and word

FDI, 1971
processing

Federation

– First digit
Modified by
Zsigmondy/

is assigned a

indicates the

(maxillary or
(FDI) system
or grid system
Palmer system

Internationale
of the alphabet

– International

quadrant (right
with computers

Standardization

the second digit


or left) and arch
– Each quadrant
Zsigmondy, 1861

Organization for

mandibular) and
which surrounds
quadrant bracket

– Not compatible

(ISO) designation
specific symbol or

a number or letter

indicates the tooth


denoting the tooth
– Quadrant system

– Two-digit system
E
E
Mandibular right second molar Maxillary right second molar Mandibular right second molar Maxillary right second molar

85
55
Maxillary right first molar

D
Mandibular right first molar Maxillary right first molar

D
Mandibular right first molar
Maxillary right canine

C
Mandibular right canine Maxillary right canine

C
Mandibular right canine
Maxillary right lateral incisor Maxillary right lateral incisor
B
Mandibular right lateral incisor

B
Mandibular right lateral incisor
Mandibular right central incisor Maxillary right central incisor Maxillary right central incisor

54 53 52 51
84 83 82 81
A
A
Mandibular right central incisor
Triumph’s Complete Review of Dentistry

A
A

Mandibular le central incisor Maxillary le central incisor Mandibular le central incisor Maxillary le central incisor
Mandibular le lateral incisor Mandibular le lateral incisor
B

Maxillary le lateral incisor Maxillary le lateral incisor


B C

Mandibular le canine Maxillary le canine Mandibular le canine


C

Maxillary le canine


Mandibular le first molar Maxillary le first molar
D

Mandibular le first molar

71 72 73 74
61 62 63 64
Maxillary le first molar
D

Mandibular le second molar Maxillary le second molar


E
E

Mandibular le second molar

75
65
Maxillary le second molar

Mandibular right third molar Mandibular right third molar


Maxillary right third molar Maxillary right third molar
Mandibular right second molar Mandibular right second molar
Maxillary right second molar Maxillary right second molar
Mandibular right first molar Maxillary right first molar
Mandibular right first molar Maxillary right first molar
Mandibular right second premolar Mandibular right second premolar Maxillary right second premolar
Maxillary right second premolar Mandibular right first premolar
Mandibular right first premolar Maxillary right first premolar Maxillary right first premolar
Mandibular right canine Maxillary right canine Mandibular right canine Maxillary right canine

https://t.me/DentalBooksWorld
Mandibular right lateral incisor Maxillary right lateral incisor Mandibular right lateral incisor Maxillary right lateral incisor
8 7 6 5 4 3 2 1

8 7 6 5 4 3 2 1

Mandibular right central incisor Maxillary right central incisor Mandibular right central incisor Maxillary right central incisor
Mandibular le central incisor Mandibular le central incisor
Maxillary le central incisor Maxillary le central incisor
Mandibular le lateral incisor Mandibular le lateral incisor Maxillary le lateral incisor
Maxillary le lateral incisor Mandibular le canine
Mandibular le canine Maxillary le canine
Maxillary le canine Mandibular le first premolar
Mandibular le first premolar Maxillary le first premolar Maxillary le first premolar
Mandibular le second premolar Maxillary le second premolar Mandibular le second premolar Maxillary le second premolar
Mandibular le first molar Maxillary le first molar Mandibular le first molar Maxillary le first molar
Mandibular le second molar Maxillary le second molar Mandibular le second molar Maxillary le second molar
1 2 3 4 5 6 7 8
1 2 3 4 5 6 7 8

Maxillary le third molar Mandibular le third molar Maxillary le third molar

48 47 46 45 44 43 42 41 3132 33 34 35 36 37 38
18 17 16 15 14 13 12 11 21 22 232425 26 27 28
Mandibular le third molar
system

with all
unique.
use this

because
because

requires
it allows

numbers

software.
tooth are
are easily

and word
given to a
particular
tooth that

treatment.

computers

processing
compatible
this system

the tooth is
of denoting
the symbol.

Dentists use
identified by

This method
discussion of
the particular
Orthodontists

The quadrants
Chapter 10 • Oral Anatomy and Histology 837

Other Tooth Numbering Systems


• Hillischer tooth numbering system
• Mons Dubois system
• Victor Haderup system
• Woelfel system
• Frykholm and Lysell system
• MICAP Notation
• Havale’s Alphanumeric dental notation

Hillischer System

7:, 6:, 3:, 2:, ,1: :1, :2, :3, :4, :5,

7:, 6:, 3:, 2:, ,1: :1, :2, :3, :4, :5,
• The Hillischer system (Hillischer,1885) of notation refers to the tooth type; it is more logical but confusing due to the use
of colons and semi colons to distinguish primary and permanent teeth
• It requires keen observation to differentiate the right side and the left side, drawing horizontal lines above the number and
below the number which represents upper teeth and lower teeth, respectively.
• 43. 7:, 6:, 3:, 2:, ,1: :1, :2, :3, :4, :5, 7:, 6:, 3:, 2:, ,1: :1, :2, :3, :4, :5,
• This system is only designed for deciduous and mixed dentition. Therefore, it is more cumbersome and difficult to
communicate, write and type, and requires much concentration and patience.
• It is confusing using the same teeth numbers 6 and 7 in primary as well as in permanent teeth in a mixed dentition stage.
Mons Dubois System

.10 .8 .6 .4 .2 .1 .3 .5 .7 .9

.10 .8 .6 .4 .2 .1 .3 .5 .7 .9
• The Mons Dubois system (How, 1890) is the use of even numbers and odd numbers, it represents the even number for
right side and odd number for left side
• However, it creates confusion regarding the use of horizontal lines in the lower arch and not in the upper one
• Moreover, the use of decimals requires concentration and time to dictate, write, type .10 .8 .6 .4 .2 .1 .3 .5 .7 .9 45
Victor Haderup System

Upper right Upper left


05+ 04+ 03+ 02+ 01+ +01 +02 +03 +04 +05
05– 04– 03– 02– 01– –01 –02 –03 –04 –05
Lower right Lower left

• The Victor Haderup system (Haderup, 1891) is similar to the Zsigmondy–Palmer notation
• Here for primary dentition, zero is added (prefix) to the left side of the numerals
• The upper and lower quadrants are denoted with plus (+) and minus (−) signs, For the right and left quadrants “+” or
“–” sign placed on the right side for the right quadrant “+” or “–” sign placed on the left side for the left quadrant,
respectively. 46
• For permanent dentition 8+ 7+ 6+ 5+ 4+ 3+ 2+ 1+ +1 +2 +3 +4 +5 +6 +7 +8 8- 7- 6- 5- 4- 3- 2- 1- -1 -2 -3 -4 -5 -6 -7 -8 47
Advantages
• This system is computer friendly
• Easy to understand
• Disadvantages
• It can be very confusing for recording mixed dentitions
• It is very difficult to dictate, so it cannot be used routinely

https://t.me/DentalBooksWorld
838 Triumph’s Complete Review of Dentistry

Woelfel System

Upper right Upper left


05+ 04+ 03+ 02+ 01+ +01 +02 +03 +04 +05
05– 04– 03– 02– 01– –01 –02 –03 –04 –05
Lower right Lower left

• The Woelfel system is similar to the universal numbering system


• In this system instead of continuous sequencing of upper case letters to denote primary teeth, suffix upper case letter “D”
alone to sequential numerals
• There is no anatomic reference in this system. It is difficult to learn and match the specific teeth, there can be confusion of
quadrants
• It requires much concentration while recording the mixed dentition stage
• Upper right Upper left 1D 2D 3D 4D 5D 6D 7D 8D 9D 10D 20D 19D 18D 17D 16D 15D 14D 13D 12D 11D Lower right
Lower left 50
Frykholm and Lysell System

Upper right Upper left


5m, 4m, 3m, 2m, 1m, 1m, 2m, 3m, 4m, 5m,
5m, 4m, 3m, 2m, 1m, 1m, 2m, 3m, 4m, 5m,
Lower right Lower left

• The Frykholm and Lysell system (Frykholm and Lysell, 1962) is a variant of the Palmer notation
• The suffix “m” (lower case letter) is added to the numerals to denote milk teeth
• This will not be confusing while recording the mixed dentition 5m, 4m, 3m, 2m, 1m 1m 2m, 3m, 4m, 5m, 5m, 4m, 3m,
2m, 1m 1m 2m, 3m, 4m, 5m, Upper right Upper left Lower right Lower left
MICAP Notation System
Maxillary permanent teeth
21 12 1 1 21 12 321 123
# I # C # #
21 12   1 1    21 12   321 123
P M
Mandibular permanent teeth

Deciduous Maxillary teeth


21 12 1 1 21 12
# d I # d

C
21 12   1 1    21 12
# d M
Deciduous Mandibular teeth
• The MICAP Notation (Akram et al., 2011) is a recently developed system
• It denotes the Deciduous Dentition, with the lower case letter “d” always written on the left side, after the sign #, of each
ANAASEA letter (dI, dC, dM)

https://t.me/DentalBooksWorld
Chapter 10 • Oral Anatomy and Histology 839

• The TOT digits (1,2,3) are mentioned on both sides of a particular ANAASEA letter with a superscript (upper corner) and
subscript (lower corner) number
• The right/left and upper/lower corner numbers (ToT digits) indicate quadrants, tooth type, and arches, respectively
• This system is based on the first letter of each tooth class, M – molar, I – incisor, c – canine, and P – premolar termed as
ANAASEA letters and digits (1,2,3) termed as TOT digits
• The letter “d” is written along with the ANAASEA letters used for deciduous class
• For permanent dentition the letters used are I C P M And for deciduous dentition letters used are I C M
Advantages
• The identification of and communication about human teeth by MICAP system is very simple
• This system is error free and user friendly

Disadvantages
• The mid-sagittal line is difficult to enter in the computer without a special dictation
• Is time-consuming software
• The symbols are very difficult to write on paper
• It looks more complicated to understand for a beginner

Havale’s Alphanumeric Dental Notation

Upper right Upper left


1e 1d 1c 1b 1a 2a 2b 2c 2d 2e
4e 4d 4c 4b 4a 3a 3b 3c 3d 3e
Lower right Lower left

• This system is designed only for Deciduous Teeth


• Currently great importance is given to preservation of primary teeth for a pleasant smile, in order to preserve the space for
the permanent dentition, and also for general health benefits for the child
• The novel dental notation for primary dentition is an alphanumeric system
• In this system numbers indicate the quadrant, while low case letters designate the tooth type
• The first digit indicates the quadrant (1 through 4) and the alphabets (lowercase) indicate the tooth type (a through e) 1 –
maxillary right, 2 – maxillary left, 3 – mandibular left, 4 – mandibular right 1e 1d 1c 1b 1a 2a 2b 2c 2d 2e 4e 4d 4c 4b 4a
3a 3b 3c 3d 3e 57
• In mixed dentition stage, this alphanumeric dental notation can be used together with FDI notation
• An example of this can be …. 11, 12, 1c, 1d, 1e, 16, 21 2b, 2c, 2d, 2e, 26. This combination is easy to use. and does not
generate confusion regarding quadrant and permanent and primary teeth
• It is also early speed of recall, and entering data in a computer is an easy and fast process
• It is also compatible with the new proposed Leo Toureno system (Toureno, 2012) for designation of supernumerary teeth
Notation of Numerical Abnormalities by an Addition to the FDI System
Fused Normal Teeth
• The first digit specifies the quadrant in accordance with the present FDI system
• The second digit indicate the mesial tooth that participate in the fusion
• And the third digit indicates the distal tooth of the fusion
• Example: The fusion of each primary mandibular lateral incisor with its adjacent canine…this tooth will be counted as 723
(left) and 823 (right); and for permanent teeth that is 323 (left) and 423 (right)
Geminated Teeth
• Gemination results from the fusion of normal tooth and a supernumerary tooth
• This tooth is represented as 9
• The first digit indicates the quadrant
• The second number indicates the tooth participating in gemination
• Example: 129 – Maxillary right lateral incisor fused with a supernumerary incisor

https://t.me/DentalBooksWorld
840 Triumph’s Complete Review of Dentistry

Supernumerary Teeth
• Supernumerary tooth are noted by a letter of alphabet
• A capital letter is used if the supernumerary tooth is a normal one
• A lower case letter is used, if the tooth is abnormal one
• Supernumerary tooth present outside the dental arch (buccally/lingually)
• A V (a v in case of abnormal supernumerary teeth) is used, if supernumerary tooth present buccally
• If more than one tooth present buccally from mesial to distal, then V, E, S, T are used, respectively
• An L (l) is used for lingual positioning of supernumerary teeth, And for more teeth L, I, N, G are used for mesiodistal
direction
• Example: For two maxillary left supernumerary molars positioned buccally to the dental arch, it becomes 2v8 (mesial) and
2e8 (distal) 63
• Supernumerary teeth placed within the dental arch (proximal to a normal tooth or a tooth of reference). The letters P, R,
O, X are used
• The letter is placed as a third digit, when the supernumerary tooth is distal to the tooth of reference
• The letters P R O X are used respectively in mesiodistal direction
• The appropriate is placed as a first (digit) when the supernumerary tooth is mesial to the tooth of reference
• Example: For maxillary mesiodens….the tooth named as P11 or P21 and p11 or p21 is used, when the mesiodens is
conical shape 64
• Example: For maxillary right hypoplastic molar which is called tooth 18p and for left 28p
• If two maxillary right hypoplastic molars are present then it called 18p, 18r

CHRONOLOGY OF TEETH
Deciduous teeth First evidence Crown Eruption Root completion Shedding (years)
of calcification completion (months) (years)
(weeks in utero) (months)
Maxillary teeth
Central incisor 14 (13–16) 1½ 10 (8–12) 1½ 7
Lateral incisor 16 (14–17) 2½ 11 (9–13) 2 8
Canine 17 (15–18) 9 19 (16–22) 3¼ 11
First molar 15 (14–17) 6 16 (13–19) 2½ 10
Second molar 19 (16–24) 11 29 (25–33) 3 10
Mandibular teeth
Central incisor 14 (13–16) 2½ 8 (6–10) 1½ 7
Lateral incisor 16 (14–17) 2½ 13 (10–16) 1½ 8
Canine 17 (16–18) 9 20 (17–23) 3¼ 9
First molar 15 (14–17) 5½ 16 (14–18) 2¼ 9
Second molar 18 (17–19) 10 27 (23–31) 3 10

Permanent teeth First evidence of Crown completion Eruption (years) Root completion
calcification (years) (years)
Maxillary teeth
Central incisor 3–4 months 4–5 7–8 10
Lateral incisor 10–12 months 4–5 8–9 11
Canine 4–5 months 6–7 11–12 13–15
First premolar 1½–1¾ years 5–6 10–11 12–13
Second premolar 2–2¼ years 6–7 10–12 12–14

https://t.me/DentalBooksWorld
Chapter 10 • Oral Anatomy and Histology 841

First molar At birth 2½–3 6–7 9–10


Second molar 2½–3 years 7–8 12–13 14–16
Third molar 7–9 years 12–16 17–21 18–25
Mandibular teeth
Central incisor 3–4 months 4–5 6–7 9
Lateral incisor 3–4 months 4–5 7–8 10
Canine 4–5 months 6–7 9–10 12–14
First premolar 1¾–2 years 5–6 10–12 12–13
Second premolar 2¼–2½ years 6–7 11–12 13–14
First molar At birth 2½–3 6–7 9–10
Second molar 2½–3 years 7–8 11–13 14–15
Third molar 8–10 years 12–16 17–21 18–25

DIFFERENCES BETWEEN DECIDUOUS AND PERMANENT DENTITION


Basis Components Deciduous tooth Permanent tooth
General Number 20 32
features Type of teeth Central incisor, lateral incisor, canine and Central incisor, lateral incisor, canine, first
first and second molars. and second premolars and first, second,
and third molars
Dental formula I 2–₂ C −| M 2–₂ = –⁵₅ = 10 I 2–₂ C −| PM 2–₂ M –₃3 = –₈8 = 16
Color White Yellowish white or grey or brown
Placement in Jaws Placed perpendicular in relation to the Placed oblique in relation to the jaws
jaws
Eruption At 8 months after birth and is completed Starts at 6 years after birth and is
by 30 months of age completed by 25 years of age
Root formation 1 year of eruption of the particular tooth 3 years for completion
Shedding 6–10 years –
Crown Shape More bulbous Less bulbous
morphology Dimensions Crowns are wider mesiodistally Length of the crown of permanent teeth is
more
Surface In anterior teeth: The facial surface is flat Convex
In molars: The lingual/palatal surfaces are
also flat
Mamelons Absent Present in incisors
Cingulum More prominent Less prominent
Cusps Short, sharp, and pointed Less sharp
Occlusal surface Shallow occlusal surface Deeper occlusal surface
Occlusal Area The occlusal area/table appears narrow The occlusal area/table appears wider
buccolingually buccolingually
Contact area Small contact area placed more gingivally Large contact area placed either in the
middle third or at the junction of middle
and incisal third
Cervical line Less curved More curved
Cervical ridge Prominent Less pronounced
Cervical Marked Less pronounced
constriction

https://t.me/DentalBooksWorld
842 Triumph’s Complete Review of Dentistry

Root Root length Shorter and slender Long and strong


morphology Crown:Root ratio Roots are proportionally longer Roots are not as long
Inclination of root 10 degree labial inclination No labial inclination
Furcation Near the cervix 3 or 4 mm from the cervix
Root trunk Since the level of furcation is very close to The root trunk is long and distinct as the
the cervix, the root trunk is not distinct level of furcation is not close to the cervix
Root flare Diverged and flared Not pronounced
Apical foramen Large and wide Narrow and constricted
Pulp Pulp chamber Large Relatively small
morphology Pulp horn Placed close to the enamel Placed at a high level
Pulp canal Wider Narrow
Accessory pulp More in number, situated mostly in the Less common, mostly in the apical region
canal furcation area
Histology Enamel Thin Thick
Uniform thickness Varying thickness
Less calcified More calcified
More permeable Less permeable.
In the cervical region, the enamel rods In the cervical region, the enamel rods
incline either incisally/occlusally/ incline apically or cervically toward the
horizontally toward the enamel surface enamel surface
Dentin Thin and is of variable thickness Thick and is of uniform thickness
Neonatal line Seen in all teeth Seen only in the permanent first molars
Dentinoenamel Scalloped Smooth
junction
Cementum Relatively thin Relatively thick

DECIDUOUS MAXILLARY LATERAL INCISOR

Labial aspect Palatal aspect Mesial aspect Distal aspect Incisal aspect
• Roughly Square • Similar to labial aspect • Triangular or wedge • Similar to mesial • Roughly triangular in
in shape with few exceptions as shaped aspect except shape
• Mesial and follows: • Labial outline is that the depth of • Incisal ridge is
distal sides • Cingulum is not convex, height of curvature is less prominent
diverge from the as prominent as in convexity is at the prominent • The mesial and distal
cervical aspect maxillary central incisor cervical third • Contact area: Incisal marginal ridges
• Mesial outline • Gradually the tooth • Lingual outline third converge toward the
is straight and tapers palatally when is concavoconvex cingulum
merges with the compared to the labial with the presence of
mesioincisal aspect cingulum
edge at right
angle

https://t.me/DentalBooksWorld
Chapter 10 • Oral Anatomy and Histology 843

• Distal outline • The incisal ridge


is convex and forms the apex of
distoincisal edge the triangular shape
is rounded of the mesial aspect
• Cervical line is of the crown and in
semicircular and line with the root
curvature is seen axis
apically • Contact area: Incisal
• Incisal outline third
is straight with
a prominent
incisal ridge
• Labial cingulum,
is not that
prominent as in
the maxillary
central incisor
Root • Single root, conical with
pointed apex

DECIDUOUS MANDIBULAR CENTRAL INCISOR


Labial aspect Lingual aspect Mesial aspect Distal aspect Incisal aspect
• Roughly square • Similar to labial • Triangular in shape • Similar to mesial • Round or ovoid in
shaped aspect with few • Labial outline is aspect except shape
• Narrower exceptions as convex that the depth of • Incisal ridge is
mesiodistally than follows: • Height of convexity curvature is less straight
the deciduous • The lingual fossa is is at the cervical prominent • Labial outline is flat
mandibular lateral not as prominent third • Contact area: Incisal and slightly convex
incisor and is as in the maxillary • Lingual outline third mesiodistally
symmetrical in incisors is concavoconvex • Lingual outline
outline • Incisal ridge is with the presence of is narrower than
• Mesial and distal perpendicular to the cingulum the labial outline
outline of the crown root axis • Contact area: Incisal because of the
tapers evenly from third tapering of the
the mesial contact lingual surface
area up to the toward the cingulum
narrow cervix
• Mesioincisal angle is
a right angle
• Distoincisal angle is
rounded
• Highest point of
convexity in the
cervical third is due
to the prominent
cervical ridge
Root • Single root, conical with blunt apex
• Labial inclination of as much as 10 degrees in the apical third of the root is evident
• Distal surface demonstrate a prominent developmental depression

https://t.me/DentalBooksWorld
844 Triumph’s Complete Review of Dentistry

DECIDUOUS MANDIBULAR LATERAL INCISOR


Labial aspect Lingual aspect Mesial aspect Distal aspect Incisal aspect
• Roughly square shaped • Similar to • Triangular in shape • Similar to mesial • Round or ovoid in
• Mesiodistal width is larger deciduous • Labial outline is aspect except shape
than in the deciduous mandibular central convex that the depth of • Incisal ridge is
mandibular central incisor incisor with few • Height of convexity curvature is less straight
• Not bilaterally symmetrical exceptions as is at the cervical prominent • Labial outline
follows: third • Contact area: is flat and
• Mesial and distal outline
of the crown tapers evenly • Cingulum is more • Lingual outline Middle third slightly convex
from the mesial contact prominent than is concavoconvex mesiodistally
area up to the narrow deciduous central with the presence • Lingual outline
cervix incisor of cingulum is narrower than
• Mesioincisal angle is a • Incisal ridge slopes • Contact area: the labial outline
right angle towards the distal Incisal third because of the
surface making the tapering of the
• Distoincisal angle is more
distoincisal angle lingual surface
rounded
more rounded toward the
• Highest point of convexity
cingulum
in the cervical third is due
to the prominent cervical
ridge
Root • Single root, conical with blunt apex, longer and narrower than in the deciduous
mandibular central incisor.
• Labial inclination of as much as 10 degrees in the apical third of the root is evident
• Mesial and distal surfaces demonstrate developmental depressions

DECIDUOUS MAXILLARY CANINE


Labial aspect Palatal aspect Mesial aspect Distal aspect Incisal aspect
• Roughly angular or • Similar to labial • Triangular in shape • Triangular in shape • Diamond shaped
diamond shaped aspect with few • Labial outline is • Labial outline is • Labiopalatal width
• Mesial outline of the exceptions as convex convex of the mesial half of
crown is convex from the follows: • Height of convexity • Height of convexity the crown is more
cervical line up to the • Mesiodistal width is at the cervical is at the cervical than that of the
mesial contact area and it is comparatively third third distal half
is shorter than the distal less • Palatal outline is • Palatal outline is • Tip of the canine
outline • Surface is irregular concavoconvex concavoconvex cusp is placed distal
• Distal outline is more with palatal fossa with the presence with the presence to the long axis of
rounded and prominent of cingulum of cingulum the tooth
• Incisally, a well- enamel ridges – the • Contact area: • Contact area:
developed, sharp cusp is mesial marginal Middle of the Middle of the
seen ridge, the distal middle third middle third
• Mesial cusp ridge is marginal ridge,
longer than the distal the cingulum, the
cusp ridge canine cusp ridge
and the palatal
• Cusp is distally placed to
ridge
the long axis of the tooth
• Surface exhibits a distinct
labial ridge called the
canine ridge
Root • Single, long, and slender root with a blunt apex
• Slight labial inclination of the root at the apical third
• Distal surface exhibit a prominent developmental depression

https://t.me/DentalBooksWorld
Chapter 10 • Oral Anatomy and Histology 845

DECIDUOUS MANDIBULAR CANINE


Labial aspect Lingual aspect Mesial aspect Distal aspect Incisal aspect
• Roughly angular or diamond • Similar to • Triangular in • Triangular in • Diamond shaped
shaped deciduous shape shape • Labiopalatal width
• Mesial outline of the crown maxillary • Labial outline is • Labial outline is of the mesial half
is convex from the cervical canine with few convex convex of the crown is
line up to the mesial contact exceptions as • Height of • Height of more than that of
area and it is shorter than the follows: convexity is at the convexity is at the the distal half
distal outline • Cingulum is not cervical third cervical third • Tip of the canine
• Distal outline is more as well developed • Lingual outline • Lingual outline cusp is placed
rounded • Distal marginal is concavoconvex is concavoconvex mesial to the long
• Incisally, a well-developed, ridge is shorter with the presence with the presence axis of the tooth
sharp cusp is seen than the mesial of cingulum of cingulum
• Mesial cusp ridge is shorter marginal ridge • Cusp tip is • Contact area:
than the distal cusp ridge • Indistinct lingual situated lingual to Middle of the
• Cusp is mesially placed to the ridge is mesially the root axis line middle third
long axis of the tooth placed and • Contact area:
extends from the Middle of the
• Surface exhibits a distinct
cusp tip to the middle third
labial ridge called the canine
cingulum and
ridge
divides the lingual
• On either side of the canine fossa into shallow
ridge, developmental mesial and distal
depressions are seen lingual fossae
demarcating the facial
developmental lobes
Root • Single, long, and slender root with a pointed apex
• Slight distal inclination of the root at the apical third
• Distal surface exhibit a prominent developmental depression

DECIDUOUS MAXILLARY FIRST MOLAR


Buccal aspect Palatal aspect Mesial aspect Distal aspect Occlusal aspect
• Roughly • Similar to buccal • Roughly • Similar to mesial • Three-cusp pattern
trapezoidal in aspect with trapezoidal in aspect with • Cusp: Mesiobuccal,
shape the following shape the following distobuccal, and palatal
• Mesial outline variations: • Buccal outline variations: cusps
of the crown is • A large is convex in • Distobuccal cusp • Ridges: Triangular ridge,
straight up to the mesiopalatal cusp the cervical and distopalatal transverse ridges for all
mesial contact and an indistinct third cusp, are seen with cusps, oblique ridge, mesial
area distopalatal • The height of a prominent distal and distal marginal ridge
• Distal outline cusp – separated contour is at the marginal ridge • Oblique ridge: Union of
of the crown is by a palatal cervical third of • Contact area: the triangular ridge of the
convex up to the developmental the crown Middle third of distobuccal cusp and the
distal contact area groove • Palatal outline the crown distal cusp ridge of the
• Large mesiobuccal • Cervical ridge is is convex in palatal cusp.
cusp and an not as prominent the cervical • Fossae: Central fossa, mesial
indistinct and the middle triangular fossa
distobuccal third • Pits: Central pit and mesial
cusp separated • The height of pit
by a buccal contour is at the • Developmental grooves:
developmental middle third of Central groove, buccal
groove is seen the crown groove

https://t.me/DentalBooksWorld
846 Triumph’s Complete Review of Dentistry

• Surface • Two cusps • Supplemental grooves –


demonstrates a are seen – mesiobuccal triangular
prominent buccal mesiobuccal and groove, mesiopalatal
cervical ridge/ mesiopalatal triangular groove, and mesial
buccal cingulum cusps – with a marginal developmental
prominent mesial groove.
marginal ridge • Four-cusp pattern
• Mesial marginal • Cusps mesiobuccal,
developmental distobuccal, mesiopalatal,
groove is and distopalatal cusps
occasionally seen • Ridges: Cuspal triangular
• Contact area: ridges, transverse ridge,
Occlusal third of oblique ridge, mesial and
the crown distal marginal
• Transverse ridge: Between
the mesiobuccal and the
mesiopalatal cusps
• Oblique ridge: Distobuccal
cusp and mesiopalatal cusp
• Fossae: Central fossa, mesial
and distal triangular fossae
• Pits: Central pit, mesial and
distal pit
• Developmental grooves:
Central, mesiobuccal
triangular, mesiopalatal
triangular, buccal
developmental and
distopalatal grooves.
• Supplemental grooves:
Mesiobuccal triangular
groove, mesiopalatal
triangular groove, and mesial
marginal developmental
groove
Root • Three roots – Mesiobuccal, distobuccal, and palatal (divergent roots), prominent buccal inclination
of the palatal root is seen.
• Level of trifurcation of the three roots is near the cervix and the root trunk is either small or
indistinct

DECIDUOUS MAXILLARY SECOND MOLAR


Buccal aspect Palatal aspect Mesial aspect Distal aspect Occlusal aspect
• Roughly • Similar to buccal • Roughly • Similar to mesial • Cusps mesiobuccal,
trapezoidal in aspect with trapezoidal in aspect with distobuccal, mesiopalatal,
shape the following shape the following distopalatal cusps and Cusp
• Mesial outline variations: • Buccal outline variations: of Carabelli
of the crown is • Palatal is convex in the • Distobuccal cusp • Ridges: Cuspal triangular
straight up to the convergence is cervical third and distopalatal ridges, transverse ridge,
mesial contact seen • The height of cusp, are seen oblique ridge, mesial and
area contour is at the with a prominent distal marginal
cervical third of distal marginal • Oblique ridge: Distobuccal
the crown. ridge cusp and mesiopalatal cusp

https://t.me/DentalBooksWorld
Chapter 10 • Oral Anatomy and Histology 847

• Distal outline • Large mesiopalatal • Palatal outline • Distal marginal • Fossae: Central fossa, distal
of the crown is cusp and a small is convex in the ridge is at a lower fossa, mesial and distal
convex up to the distopalatal cusp cervical and the level than mesial triangular fossae
distal contact area are seen in this middle third marginal ridge • Pits: Central pit, mesial and
• Large mesiobuccal aspect, separated • The height of • Contact area: distal pit
cusp, small by a palatal contour is at the Middle third of • Developmental
distobuccal developmental middle third of the crown grooves: Central, buccal
cusp separated groove the crown developmental and
by buccal • Cusp or tubercle • Two cusps distopalatal grooves.
developmental of Carabelli is are seen – • Supplemental grooves:
groove is seen situated cervical to mesiobuccal and Mesiobuccal triangular
• Cervical ridge is the mesiopalatal mesiopalatal groove, mesiopalatal
not as prominent cusp cusps – with a triangular groove, mesial
as first molar prominent mesial marginal developmental
marginal ridge groove, distobuccal triangular
• Mesial marginal groove, distal marginal
developmental developmental groove
groove is
occasionally seen
• Contact area:
Middle third of
the crown
Root • Three roots – Mesiobuccal, distobuccal, and palatal diverging roots
• Long and slender with blunt apex

DECIDUOUS MANDIBULAR FIRST MOLAR


Buccal aspect Lingual aspect Mesial aspect Distal aspect Occlusal aspect
• Roughly • Similar to • Roughly trapezoidal in • Distobuccal • Cusps: Mesiobuccal,
trapezoidal in buccal aspect shape and distobuccal, mesiolingual,
shape with the • Buccal outline is convex distolingual and distolingual cusps
• Mesial outline following in the cervical third cusps, with a • Ridges: Cuspal triangular
of the crown is variations: • The height of contour is distal marginal ridges, mesial and distal
straight up to the • Large at the cervical third of the ridge are seen marginal ridge, transverse
mesial contact mesiolingual crown • Contact area: ridge
area cusp and a small • Lingual outline is convex Middle third • Transverse ridge: Between
• Distal outline distolingual in the cervical and the of the crown the mesiobuccal and the
of the crown is cusp separated middle third mesiolingual cusps
convex up to the by a lingual • Fossae: Central fossa, mesial
• The height of contour is
distal contact area developmental and distal triangular fossae
at the middle third of the
• Large mesiobuccal groove is seen
crown • Developmental grooves:
cusp and a small • Buccal cusp tips Central, buccal, lingual and
• Two cusps are seen
distobuccal cusp, are also seen distal developmental groove
– mesiobuccal and
separated by a mesiolingual cusps – • Supplementary grooves:
developmental with a prominent mesial Mesiobuccal triangular
depression instead marginal ridge groove, distobuccal
of a groove triangular groove,
• Mesial marginal
• Cervical ridge/ developmental groove is mesiolingual triangular
buccal cingulum occasionally seen groove, distolingual
is prominent triangular groove
• Contact area: Cervical
third of the crown • Pits: Central pit, mesial and
distal pit
Root • Two roots – Mesial and distal root flaring roots to accommodate permanent tooth bud
• Distal inclination is noted at the apical third of both the roots
https://t.me/DentalBooksWorld
848 Triumph’s Complete Review of Dentistry

DECIDUOUS MANDIBULAR SECOND MOLAR

Buccal aspect Lingual aspect Mesial aspect Distal aspect Occlusal aspect
• Trapezoidal in shape • Lingual • Rhomboidal in • Similar to the • Pentagonal in shape
• Mesial outline is convergence is shape mesial aspect • Cusps: Mesiobuccal
straight and distal evident • Buccal outline is • Distal marginal > Mesiolingual >
outline is convex • Mesiolingual and convex and the ridge is not that Distolingual > Distobuccal
from the occlusal distolingual cusps crest of curvature prominent and is > distal
surface to the mesial are seen is located at the thin and curves • Ridges: Mesial and distal
contact area then • Lingual cervical third of cervically and marginal ridge, cuspal
straightens developmental the crown so part of the ridges, triangular ridges,
• Mesiobuccal and groove is evident • Buccal cervical occlusal surface transverse ridge
distobuccal and ridge is evident can be seen • Transverse ridge: Between
distal cusps are seen • Lingual outline • Contact area: the mesiobuccal and the
• Mesiobuccal cusp is is convex and the Junction of the mesiolingual cusps
the largest and distal crest of curvature occlusal and • Fossae: Mesial and distal
cusp is the smallest is located at the middle thirds of triangular fossa, central
• Mesiobuccal middle third of the crown fossa
and distobuccal the crown • Developmental grooves:
developmental • Mesiobuccal cusp Mesiobuccal, distobuccal,
grooves are seen and mesiolingual and lingual groove, central
• Cervical ridge/ cusp are seen from groove
buccal cingulum is this aspect. • Pit: Mesial pit, distal pit,
prominent • Mesial marginal central pit
ridge is
prominent.
• Contact area:
Middle third of
the crown
Root • Two roots – Mesial and distal root diverging roots
• Level of bifurcation of the roots is near the cervix and the root trunk is either small or
indistinct

PULP MORPHOLOGY OF DECIDUOUS INCISORS AND CANINES


• The morphology of the pulp chambers and canals follows the outline of the tooth
• The deciduous incisors and canines have a single root canal
• The chamber is larger than in permanent teeth, with pointed pulp horns
• The root canals are wider in the coronal aspect

PULP MORPHOLOGY OF DECIDUOUS MOLARS


• Deciduous molars have four root canals in each tooth
• The maxillary deciduous molars have two canals in the mesial root, accounting for four in total
• Mandibular deciduous molars have two canals in each of the two roots
• The pulp chamber is very close to the coronal surface and the furcation area

https://t.me/DentalBooksWorld
Chapter 10 • Oral Anatomy and Histology 849

PERMANENT MAXILLARY AND MANDIBULAR INCISORS


PERMANENT MAXILLARY LATERAL INCISOR
Labial aspect Lingual aspect Mesial aspect Distal aspect Incisal aspect
• Roughly ovoid in • Similar to labial • Triangular in shape • Similar to mesial • Round or ovoid in
shape aspect with few • Labial outline is aspect except shape
• Mesial outline is exceptions as more convex than that the depth of • The mesial and
straight follows: maxillary central curvature is less distal marginal
• Mesioincisal edge is • The lingual fossa incisor prominent ridges converge
more rounded is deeper when • Height of convexity • Contact area: toward the cingulum
• Distal outline compared with the is at the cervical Middle third • The cingulum
is convex and maxillary central third is centered
distoincisal edge is incisor • Lingual outline mesiodistally unlike
more rounded than • Cingulum is more is concavoconvex its position in the
mesioincisal edge prominent with the presence of maxillary central
• Incisal outline is • Deep developmental cingulum incisors
rounded grooves at the side of • The incisal ridge is
• labial surface is the cingulum called in line with the root
more convex than the palatogingival axis
maxillary central groove or the • Contact area:
incisor palatoradicular Junction of the
groove is evident incisal and middle
third
Root • Single root
• Conical with blunt apex
• Apically root demonstrates distal curvature
Pulp morphology • The anatomy is similar to that of a central incisor but of smaller dimensions
• Pulp horns may or may not be present
• Single root canal
• Cross-section of root canal:
–– Cervical: Wider in the labiopalatal dimension
–– Midroot: Ovoid
–– Apical third: Generally round and gradually curved

PERMANENT MANDIBULAR CENTRAL INCISOR


Labial aspect Lingual aspect Mesial aspect Distal aspect Incisal aspect
• Trapezoidal in shape • Similar to labial • Triangular in shape • Similar to mesial • Round or ovoid in
• Mesial outline is straight aspect with few • Labial outline is aspect except shape
• Mesioincisal edge is at exceptions as convex that the depth of • The labial outline
right angles to the root follows: • Height of convexity curvature is less is straight and
axis • The lingual fossa is is at the cervical prominent is at right angles
• Distal incisal angle is at not as prominent third • Contact area: to a line drawn
right angles to the long as in the maxillary • Lingual outline Incisal third labiolingually.
axis of the tooth but is incisors is concavoconvex • The lingual outline
more rounded than the with the presence is convex, with the
mesial incisal angle of cingulum mesial and distal
• Incisal outline is straight • The incisal ridge marginal ridges
is placed lingual to converging toward
• Labial surface is smooth
the root axis the cingulum
and flat unlike the
maxillary incisors • Contact area:
Incisal third

https://t.me/DentalBooksWorld
850 Triumph’s Complete Review of Dentistry

Root • Single root


• Conical with blunt apex
• Distal tilt in the apical third is evident
Pulp morphology • Has a small pulp chamber with three distinct pulp horns at birth
• The pulp chamber is wide labiolingually. The cross-section in the cervical third is ovoid and
tapers incisally
• Single root canal
• The root is straight in 60% of the cases. Nearly 70% of the cases have one canal and one
apical foramen, whereas 22% of the cases show one canal which bifurcates into two and
exits into one apical foramen
• Cross-section of root canal:
–– Cervical: Wider labiolingually
–– Midroot: Ovoid
–– Apical third: Round

PERMANENT MAXILLARY LATERAL INCISOR


Labial aspect Lingual aspect Mesial aspect Distal aspect Incisal aspect
• Trapezoidal in shape • Similar to labial • Triangular in shape • Similar to mesial • Ovoid in shape
• Mesial outline is straight aspect with few • Labial outline is aspect except • Labial outline
• Mesioincisal edge is at exceptions as convex that the depth of curves toward the
right angles to the root follows: • Height of convexity curvature is less distal side because
axis • Cingulum and is at the cervical prominent of the distal
• Distal incisal angle is at lingual fossa are third • Contact area: developmental
right angles to the long more prominent • Lingual outline Incisal third lobe and is not
axis of the tooth but is is concavoconvex straight as in the
more rounded than the with the presence mandibular central
mesial incisal angle of cingulum incisor
• The incisal outline is • The incisal ridge • The lingual outline
slightly curved as a result is placed lingual to is convex
of a prominent distal the root axis • The marginal
developmental lobe • Contact area: ridges are more
• The labial surface is more Incisal third prominent, and
rounded mesiodistally the lingual fossa
than the mandibular is slightly deeper
central incisors than that of the
mandibular central
• Developmental grooves
incisors
are present on the labial
surface
Root • Single root
• Long and conical
• Narrow mesiodistally
• Flattened labiolingually
Pulp morphology • Similar to central incisor but with larger dimensions
• Cross-section of root canal:
–– Cervical: Wider in the labiolingual direction
–– Midroot: Ovoid
–– Apical third: Round

https://t.me/DentalBooksWorld
Chapter 10 • Oral Anatomy and Histology 851

PERMANENT MAXILLARY AND MANDIBULAR CANINES

PERMANENT MAXILLARY CANINE


Labial aspect Lingual aspect Mesial aspect Distal aspect Incisal aspect
• Trapezoidal • Similar to labial • Similar to the • Similar to mesial • Roughly ovoid in
• Resembles an incisor aspect with few incisors, but they aspect except that shape
from this aspect but exceptions as are more bulky and the outlines are • Increased
is smaller than the follows: prominent than the more convex than in labiolingual
maxillary central • Cingulum is present incisors the mesial surface dimensions is
incisors • Lingual ridge • Triangular or wedge • Contact area: Center evident from this
• Mesial outline from extending from shaped of the middle third aspect
the cusp tip takes a the cusp tip to the • Crown is bulkier • Lingual fossae and
slope to the contact cingulum divide the labiolingually lingual ridge are
point and is concave lingual surface into • Labial outline evident from this
from the contact two lingual fossae is convex, aspect
point to the cervical • Cingulum may lingual outline is
line exhibit tubercles concavoconvex
• Distal outline is s • Lingual pit is • Height of contour in
shaped. It is convex associated with the the labial aspect is at
toward the contact tubercles the cervical third
area and gradually • Cusp tip is placed
becomes concave labial to the long
toward the cervical axis of the tooth
line
• Contact area-
• Incisal edge and is junction of the
divided into two middle and incisal
cuspal slopes: the third
mesial slope and
the distal slope. The
slopes meet at right
angle
• Labial surface is
convex, with a
vertical and centrally
placed labial ridge
extending from the
cervical line to the
cusp tip
Root • Cone shaped and has a blunt apex
• It shows a distal tilt
• Longest of all the teeth
• Bulkier labiolingually
• Demonstrates a developmental depressions mesially and distally
Pulp morphology • Single canal
• Pulp horns are absent
• Cross-section of root canal:
–– Cervical: Wider in the mesiodistal direction
–– Midroot: Ovoid
–– Apical third: Round

https://t.me/DentalBooksWorld
852 Triumph’s Complete Review of Dentistry

PERMANENT MANDIBULAR CANINE


Labial aspect Lingual aspect Mesial aspect Distal aspect Incisal aspect
• Trapezoidal • Outlines are • Triangular or • Similar to mesial • Roughly ovoid in
• Mesial outline is straighter similar to the wedge shaped aspect shape
than in the maxillary canine labial aspect • Labial outline • Contact area: • The incisal aspect
• Distal outline is slightly • Exhibits a is convex, Junction of the is asymmetrical
rounded near the contact cingulum and lingual outline is incisal and middle with the mesial
area and is straight from the a lingual fossa concavoconvex thirds portion of
contact area to the cervical line bordered by • The cusp tip is the crown
• The distal slope is longer marginal ridges, either in line with having greater
and more rounded than the which are less or lingual to the labiolingual width
mesial slope prominent and long axis of the • Lingual fossae and
smoother than tooth lingual ridge and
• The contact areas on the
in the maxillary • Contact area: cingulum are less
mesial and distal sides
canines Incisal third prominent than
are located at different
levels. This feature helps to maxillary canine
differentiate between the right
and left mandibular canines
• Labial surface is convex;
the convexity is less when
compared with maxillary
canines
Root • Cone shaped
• Slightly more pointed than in the maxillary canines
Pulp morphology • Single canal
• Pulp chamber is wide labiolingually
• Has a prominent pulp horn
• Cross-section of root canal:
–– Cervical: Wider in the labiolingual direction
–– Midroot: Ovoid
–– Apical third: Round

PERMANENT MAXILLARY AND MANDIBULAR PREMOLARS


PERMANENT MAXILLARY FIRST PREMOLAR
Buccal aspect Lingual aspect Mesial aspect Distal aspect Occlusal aspect
• Trapezoidal • Lingual aspect • Mesial marginal • Similar to the mesial • Roughly hexagonal
• Resembles a maxillary is rounded in all groove extends from aspect • Sides: Mesiobuccal,
canine directions but is the mesial marginal • But the mesial,
• Mesial outline is smaller than the ridge and runs from developmental mesiolingual,
slightly concave from buccal surface the occlusal third to depression and the distolingual, distal
the occlusal outline to • Lingual ridge is the middle third of groove is absent on and distobuccal
the contact area and evident the crown the distal aspect • Buccal outline is
becomes convex near • Lingual cusp • Buccal outline is • Contact area: convex and makes
the contact area and is is sharp and convex from the Middle third the crown appear
almost straight till the is narrower cuspal tip to the bulkier from this
cervical line and more mesiodistally than cervical line and aspect
rounded the buccal cusp crest of contour is • Lingual dimensions
located near the are smaller
cervical third

https://t.me/DentalBooksWorld
Chapter 10 • Oral Anatomy and Histology 853

• Distal outline is • Lingual outline is • Ridges: Mesial and


slightly straighter than convex and is more distal marginal
the mesial outline rounded and crest ridge, Buccal and
• Occlusal outline is of contour is located lingual triangular
divided into mesial within the middle ridge, Transverse
slope (longer) and the third of the crown ridge
distal slope • Lingual cusp is • Fossae: Mesial and
• Cervical line is almost shorter than the distal triangular
straight buccal cusp fossae
• Buccal surface • Cusp tips are in • Developmental
is convex with line within the root grooves: Central
prominent buccal trunk groove, mesiobuccal
ridge • Contact area: and distobuccal
Middle third developmental
grooves, few
supplementary
grooves and
characteristic mesial
marginal groove
Root • Two roots – Buccal and lingual, same length
• Deep mesial developmental depression evident
Pulp morphology • Two root canals: Buccal and palatal (lingual)
• Buccal pulp horn is higher than the palatal pulp horn
• Palatal canal is relatively larger in dimension than the buccal canal
• Cross-section of root canal:
–– Cervical: Very wide buccolingually
–– Midroot: Slightly ovoid
–– Apical third: Round

PERMANENT MAXILLARY SECOND PREMOLAR


Buccal aspect Lingual aspect Mesial aspect Distal aspect Occlusal aspect
• Similar to that • Rounded in all • Buccal outline is • Similar to the • Similar to maxillary first
of maxillary directions convex from the mesial aspect molar with the following
first molar with • Lingual cusp cuspal tip to the • But the variations
the following is of the same cervical line and developmental • Ovoid and more rounded
variations length as the crest of contour is depression and the • Buccal outline is more convex
• Smaller buccal cusp, located near the groove is absent • Lingual dimensions are
mesiodistally unlike the first cervical third on the distal similar to buccal dimensions
and cervico- premolar • Lingual outline is aspect
• Ridges: Mesial and distal
occlusally convex and is more • Contact area: marginal ridge, Buccal, and
• More rounded rounded and crest Junction of the lingual triangular ridge,
• Mesial slope is of contour is located occlusal and Transverse ridge
shorter than the within the middle middle thirds of
third of the crown • Fossae: Mesial and distal
distal slope the crown
triangular fossae
• Buccal ridge is • Cusps are of same
length • Developmental Grooves:
less prominent Central groove, mesiobuccal
• Contact area:
and distobuccal
Junction of the
developmental grooves, many
occlusal and middle
supplementary grooves
thirds of the crown
• Pits: Mesial and Distal pits

https://t.me/DentalBooksWorld
854 Triumph’s Complete Review of Dentistry

Root • Single root


• Broad buccolingually, narrow mesiodistally
• Blunt apex
• Distal inclination at the apex
Pulp morphology • Single or two root canals
–– Cervical: Very wide mesiodistal dimension
–– Midroot: Ovoid
–– Apical third: Round

PERMANENT MANDIBULAR FIRST PREMOLAR


Buccal aspect Lingual aspect Mesial aspect Distal aspect Occlusal aspect
• Trapezoidal • Similar to buccal • Rhomboidal in • Similar to mesial • Diamond shaped
• Mesial outline is aspect with the shape aspect • Much of the buccal
straight from the following variations • Buccal outline shows • Distal marginal aspect is evident as
cervical line up to • Lingual cusp is prominent convexity ridge is better the lingual cusp is
the contact areas smaller than the and crest of developed than the smaller
while distal outline buccal cusp curvature is present mesial marginal • Ridges: Mesial and
demonstrates a • Hence most of the in the cervical third ridge distal marginal
slight concavity occlusal surface is of the crown • Developmental ridge, Buccal and
• Distal slope is longer seen • Lingual outline is grooves similar to lingual triangular
than the mesial • Prominent more convex and mesiolingual groove ridge, mesiobuccal
slope developmental crest of curvature is not present and distobuccal cusp
• Buccal surface is groove called the is present in the ridge, mesiolingual
more convex than mesiolingual groove middle third of the cusp ridge
maxillary premolar is present over the crown • Fossae: Mesial and
mesial marginal • Buccal cusp tip lies distal triangular
ridge in line with the root fossae
apex, Lingual cusp • Developmental
is either in line with grooves: Central
the root tip or might groove, mesiolingual
extend lingually groove, mesiobuccal
outside the confines and distobuccal
of the root tip triangular groove
• Mesiolingual • Pits: Mesial and
developmental Distal pits
groove is evident
• Contact area:
Junction of the
occlusal and middle
thirds of the crown
Root • Single root
• Cone shaped
• Mild mesial inclination evident at the apex
Pulp morphology • Single root canal
• Prominent buccal pulp horn is present under the well-developed buccal cusp
• In young tooth, a smaller lingual pulp horn is present
–– Cervical: Wider in the buccolingual dimension
–– Midroot: Ovoid
–– Apical third: Round

https://t.me/DentalBooksWorld
Chapter 10 • Oral Anatomy and Histology 855

PERMANENT MANDIBULAR SECOND PREMOLAR


Buccal aspect Lingual aspect Mesial aspect Distal aspect Occlusal aspect
• Trapezoidal • Similar to buccal • Rhomboidal in shape • Similar to mesial • Roughly squarish
• Mesial and aspect with the • Buccal outline shows aspect shaped
distal outline is following variations prominent convexity • Distal marginal • Much of the buccal
convex from the • More bulbous and and crest of curvature ridge is at a lower aspect is evident as the
cervical line up spheroidal is present in the middle level than the lingual cusp is smaller
to the contact • Lingual cusp is third of the crown mesial marginal • Ridges: Mesial and
areas prominent • Lingual outline is more ridge distal marginal ridge,
• Distal slope is • Three cusp pattern convex and crest of • Contact area: cuspal triangular ridge
longer than the (Y): Mesiolingual, curvature is present in Junction of the • Fossae: Mesial and
mesial slope distolingual and the middle third of the occlusal and distal triangular fossae,
• Buccal surface buccal cusp crown middle thirds of central fossa
is more convex • Two cusp pattern • Buccal cusp (more the crown • Developmental
than maxillary (U/H): Buccal and rounded) tip lies in grooves: Central
premolar lingual cusp line with the root apex, groove, mesiobuccal
• Lingual Lingual cusp extend and distobuccal groove,
developmental lingually outside the mesiolingual and
groove is evident (Y) confines of the root tip distolingual groove,
• Marginal ridges are at Lingual groove (Y)
right angles to the tooth • Pits: Mesial and Distal
• Contact area: Junction pits
of the occlusal and • Y (three cusps) pattern:
middle thirds of the Central pit is present
crown
Root • Single root
• Cone shaped and tapers apically
Pulp morphology • Single root canal
• Prominent buccal pulp horn is present under the well-developed buccal cusp
• Lingual pulp horn is larger than that of the mandibular first premolar
–– Cervical: Wider buccolingually
–– Midroot: Long ovoid
–– Apical third: Round

PERMANENT MAXILLARY AND MANDIBULAR MOLARS


PERMANENT MAXILLARY FIRST MOLAR
Buccal aspect Lingual aspect Mesial aspect Distal aspect Occlusal aspect
• Trapezoidal in • More convex and • Trapezoidal in shape • Similar to the • Rhomboidal in shape
shape rounded than • Broad faciolingually mesial aspect • Cusps: Mesiolingual >
• Mesial and distal the buccal aspect • Buccal outline is • Distobuccal and Mesiobuccal > Distolingual
outline is convex • Mesiolingual convex and the the distolingual > Distobuccal > Cusp Of
from the occlusal and distolingual crest of curvature cusps as well as Carabelli
surface to the cusps are seen is located at the the mesial cusp • Ridges: Mesial and
mesial contact area • Mesiolingual cervical third of the tips are seen Distal Marginal Ridge,
then straightens cusp is the crown from this aspect Cuspal Ridges, Oblique
• Mesiobuccal and largest • Lingual outline • Contact area: Ridge, Transverse Ridge
distobuccal cusps • Distolingual is convex and the Middle thirds of • Oblique Ridge:
are seen cusp is the crest of curvature is the crown Mesiolingual to the
smallest located at the middle Distobuccal Cusp
third of the crown
https://t.me/DentalBooksWorld
856 Triumph’s Complete Review of Dentistry

• Mesial slope of the • Lingual • Mesial aspect of the • Transverse Ridge: Triangular
mesiobuccal cusp developmental mesiobuccal cusp Ridge of the Mesiolingual
is longer than the groove is evident and the mesiolingual Cusp and Mesiobuccal Cusp
distal slope of the cusp with the cusp • Fossae: Mesial and Distal
mesiobuccal cusp of Carabelli are Triangular Fossa, Distal Fossa
and they meet at evident • Developmental grooves:
an obtuse angle • Buccal slope and the Buccal and lingual groove,
• A buccal groove lingual slope of the Distal oblique groove, Central
between the buccal cusps meet at right groove, Transverse groove,
cusps is seen angles Fifth cusp groove
• Lingual cusp is • Distal oblique groove:
longer than the Distolingual cusp and the
buccal cusp Mesiolingual cusp
• Contact area: • Central groove: Central fossa
Junction of the to the transverse ridge and
occlusal and middle ends in the mesial triangular
thirds of the crown fossa
• Transverse groove: Runs
distally across the oblique
ridge to the distal triangular
fossa.
• Fifth cusp groove: Separates
the fifth cusp from the
mesiolingual cusp
Root • Three roots – Mesiobuccal, distobuccal, and palatal
• Mesiobuccal root demonstrates a distal inclination at the apex
• Distobuccal root demonstrates a mesial inclination at the apex
• Palatal root is conical
Pulp morphology • Mesiobuccal root – 2 canals
• Distobuccal – 1 canal
• Palatal/lingual – 1 canal (largest)
• C.S: Cervical: Triangular; Apical third: Round

PERMANENT MAXILLARY SECOND MOLAR

Buccal aspect Lingual aspect Mesial aspect Distal aspect Occlusal aspect
• Crown is shorter in • Cusp of Carabelli is • Trapezoidal in shape • Similar to the mesial • Similar to maxillary
the occlusocervical absent • Broad faciolingually aspect first molar
aspect and narrower • Mesiodistal width • Buccal outline is • Distobuccal and the • Rhomboidal in
mesiodistally than in is lesser than that of convex and the distolingual cusps shape
the first molars the first molars crest of curvature as well as the mesial • Pronounced lingual
• More rounded than • Outlines are similar is located at the cusp tips are seen convergence than
in the first molars to the buccal aspect cervical third of the from this aspect first molars
• Surface is smaller mesiolingual and the crown • Contact area: • Less prominent
than that of the first distolingual cusps • Lingual outline Middle thirds of the oblique ridge than
molar are seen is convex and the crown first molar
• Buccal groove is • The mesiolingual crest of curvature is • Absence of fifth
evident cusp is longer than located at the middle cusp and fifth cusp
the distolingual cusp third of the crown groove

https://t.me/DentalBooksWorld
Chapter 10 • Oral Anatomy and Histology 857

• The distolingual • Buccal slope and the


cusp is small and lingual slope of the
less developed when cusps meet at right
compared with that angles
in the first molars • Lingual cusp and
buccal cusp are of
same length
• Contact area:
Middle thirds of the
crown
Root • Three roots – Mesiobuccal, distobuccal, and palatal
• Mesiobuccal root demonstrates a distal inclination at the apex
• Distobuccal root demonstrates a mesial inclination at the apex
• Palatal/Lingual root is conical, apex lies in line with the distolingual cusp tip
Pulp morphology • Root converges more and hence canals demonstrate more curvature than first molars
• Mesiobuccal root – 2 canals
• Distobuccal – 1 canal
• Palatal/Lingual – 1canal (largest)
• C.S: Cervical: Triangular; apical third: Round

PERMANENT MAXILLARY THIRD MOLAR


Buccal aspect Lingual aspect Mesial aspect Distal aspect Occlusal aspect
• Crown is shorter in • The distolingual • Buccal outline is • Similar to the mesial • Similar to maxillary
the occlusocervical cusp is absent in convex and the aspect first molar but one
aspect and narrower most of the cases crest of curvature • Distobuccal and the large lingual cusp
mesiodistally than in • One large lingual is located at the distolingual cusps is usually present
the second molars cusp is present cervical third of the are seen from this rather than two
• Surface is smaller crown aspect lingual cusps
than that of the • Lingual outline • Contact area: • Heart shaped
second molar is convex and the Middle thirds of the • Pronounced lingual
• Buccal groove is crest of curvature is crown convergence than
evident located at the middle first and second
third of the crown molars
• Contact area: • Supplemental
Middle thirds of the grooves are
crown numerous, making
it appear more
wrinkled
• The oblique ridge is
poorly developed or
even absent in some
cases
Root • Roots are usually fused to form a single root
• They have a distal tilt
Pulp morphology • Three roots and three canals is a common occurrence
• The root can exhibit curvature of different nature
• The intracoronal and intraradicular anatomy of maxillary third molar cannot be generalized

https://t.me/DentalBooksWorld
858 Triumph’s Complete Review of Dentistry

PERMANENT MANDIBULAR FIRST MOLAR


Buccal aspect Lingual aspect Mesial aspect Distal aspect Occlusal aspect
• Trapezoidal in shape • Mesiolingual • Rhomboidal in shape • Similar to the • Pentagonal in shape
• Mesial and distal and distolingual • Buccal outline is convex mesial aspect • Cusps: Mesiobuccal
outline is convex cusps are seen and the crest of curvature is • All the five cusps > Mesiolingual
from the occlusal • The located at the cervical third are seen from this > Distolingual >
surface to the mesial mesiolingual of the crown aspect because Distobuccal > distal
contact area then cusp is larger • Buccal cervical ridge is of the distal • Ridges: Mesial and
straightens than the evident convergence of the distal marginal
• Mesiobuccal an distolingual • Lingual outline is convex crown and shorter ridge, cuspal ridges,
distobuccal and cusp and the crest of curvature is distal marginal triangular ridges
distal cusps are seen • Lingual cusps located at the middle third ridge • Fossae: Mesial and
• Mesiobuccal cusp is are sharper of the crown • The distal distal triangular
the largest and distal • Lingual • Mesiobuccal cusp and marginal ridge has fossa, Central fossa
cusp is the smallest developmental mesiolingual cusp are seen a distal marginal • Developmental
• Mesiobuccal groove is from this aspect groove which is grooves:
and distobuccal evident placed lingually Mesiobuccal,
• Mesiolingual cusp is higher
developmental than the mesiobuccal cusp • Contact area: distobuccal, and
grooves are seen Junction of the lingual groove,
• Mesial marginal ridge
• Mesiobuccal groove occlusal and Central groove
is higher than the distal
usually ends in a middle thirds of • Pit: Mesial pit, distal
marginal ridge
buccal pit the crown pit, central pit
• Mesial marginal ridge has
a mesial marginal groove
positioned lingual to the
center of the crown
• Contact area: Junction of
the occlusal and middle
thirds of the crown
Root • Two roots – Mesial and distal root
• Root demonstrate a distal inclination at the apex
• Deep developmental depression is present in the distal root
Pulp morphology • Mesial root – 2 canals
• Distal root – 1 canal
• Has four pulp horns, namely mesiobuccal, mesiolingual, distobuccal, and distolingual
• C.S: Cervical: Cervical: Oval, Midroot: Ovoid, Apical third: Round

PERMANENT MANDIBULAR SECOND MOLAR


Buccal aspect Lingual aspect Mesial aspect Distal aspect Occlusal aspect
• Trapezoidal/ • Mesiolingual and • Rhomboidal in shape • Similar to the mesial • Rectangular in shape
Rectangular in distolingual cusps • Buccal outline is convex aspect • Cusps: Mesiobuccal,
shape are seen and the crest of curvature • All the four cusps mesiolingual,
• Mesiobuccal • Higher than the is located at the cervical are seen from this distolingual,
and distobuccal buccal cusps third of the crown aspect because of the distobuccal
cusps are seen • Mesiolingual cusp • Buccal cervical ridge is short distal marginal • Ridges: Mesial and
• Buccal is almost similar in less prominent ridge distal marginal
developmental dimensions to the • Lingual outline is convex • The distal marginal ridge, cuspal ridges,
groove is seen distolingual cusp and the crest of curvature ridge does not have Triangular ridges
is located at the middle a distal marginal • Fossae: Mesial and
third of the crown groove distal triangular
fossa, Central fossa

https://t.me/DentalBooksWorld
Chapter 10 • Oral Anatomy and Histology 859

• Buccal groove • Cuspal slopes meet • Mesial marginal ridge is • Contact area: • Developmental
usually ends in a at an obtuse angle not intervened by mesial Middle thirds of the grooves: Buccal
buccal pit • Lingual marginal groove crown and lingual groove,
developmental • Contact area: Junction of Central groove
groove runs between the occlusal and middle • Pit: Mesial pit, distal
the two lingual thirds of the crown pit, central pit
cusps
Root • Two roots – Mesial and distal root
• Shallow developmental depression is present in the distal root
• Mesial root demonstrate an apical distal inclination while distal root demonstrate a mild mesial
inclination
Pulp morphology • Mesial root – 2 canals
• Distal root – 1 canal
• Pulp horns are prominent
• C.S: Cervical: Cervical: Oval, Midroot: Ovoid, Apical third: Round

PERMANENT MANDIBULAR THIRD MOLAR


Buccal aspect Lingual aspect Mesial aspect Distal aspect Occlusal aspect
• Roughly ovoid in • Mesiolingual and • Rhomboidal in • Similar to the mesial • Roughly oval in
shape distolingual cusps shape aspect shape
• Mesiobuccal and are seen • Buccal outline is • All the four cusps • Similar to
distobuccal cusps • Lingual convex and the are seen from this mandibular second
are seen developmental crest of curvature aspect because of the molar
• Buccal groove runs between is located at the short distal marginal • Occlusal surface has
developmental the two lingual cervical third of the ridge a much wrinkled
groove is seen cusps crown • The distal marginal appearance because
• Buccal groove • Lingual outline ridge does not have of the presence
usually ends in a is convex and the a distal marginal of supplemental
buccal pit crest of curvature is groove grooves and occlusal
located at the middle • Contact area: pits
third of the crown Middle thirds of the
• Contact area: crown
Middle thirds of the
crown
Root • Two roots – Mesial and distal root
• Roots are short and placed close to each other. Can be fused also
• Mesial root demonstrate an apical distal inclination while distal root demonstrate a mesial
inclination
Pulp morphology • Intraradicular anatomy is unpredictable
• The tooth may have one to four roots and one to six canals
• The root canals are large and short
• C-shaped canals occur due to the fusion of mesial or distal root on either the buccal or lingual
surface

https://t.me/DentalBooksWorld
860 Triumph’s Complete Review of Dentistry

DEVELOPMENT OF FACE
1. Development of Cranium
• 110 ossification centers; 45 bones in newborn; 32 bones in young adults
• Bones of the skull are grouped into neurocranium (protective covering of brain)
• and Viscerocranium (bones of the face)
• Neurocranium is divided into Membranous desmocranium and Cartilaginous chondrocranium

Membranous desmocranium:
• Develops in the 4th week of IUL
• Formed by condensation of mesenchyme around developing brain. Bone is formed by membranous ossification
• Has inner layer of endomeninx of neural crest origin. Differentiates into arachnoid and pia mater
• Outer layer ectomeninx is of mesodermal origin. Differentiates into dura mater and skull bone

Cartilaginous chondrocranium:
• Cartilage eventually undergoes ossification
• Chondrification occurs at the region of notochord (parachordal chondrocranium) and in front of notochord (prechordal
chondrocranium)
• Parachordal chondrocranium is derived from occipital myotomes. Mastoid process, petrous part of temporal bone,
occipital bone and nasal bones are formed from parachordal chondrocranium
• Prechordal chondrocranium is derived from neural crest cells. Sphenoid bone and ethmoid bones are formed from
prechordal chondrocranium
2. Development of Mandible
• Meckel’s cartilage is the primary cartilage. Condylar cartilage, coronoid cartilage, and symphyseal cartilage are secondary
cartilages
• 6th week IUL – Meckel’s cartilage on either side extends as a solid rod from otic capsule to the midline. The inferior
alveolar nerve and lingual nerve run along the medial and lateral aspect of the cartilage
• At 7th week IUL ossification commences extending anteriorly and posteriorly forming the body of the mandible hosting
the inferior alveolar nerve within
• Ramus is formed by the spread of ossification posteriorly into the mesenchyme of the first arch
• The mandible formation is almost complete by the end of the 10th week, formed almost entirely by intramembranous
ossification
• Endochondral ossification begins at 14th week of IUL in the condylar cartilage and the process ends by 20th week of IUL
forming the condyle. Remnants of the cartilage remain as growth cartilage and articular cartilage
• Coronoid cartilage appears at 14th week IUL and ossifies forming coronoid process
• At 7th month of IUL symphyseal cartilage ossifies
• The union of the two separate centers of ossification occurs in the midline between the 4th and 12th month postnatally

3. Development of TMJ
• Primitive joint between malleus and incus briefly function as jaw joint from 8th week IUL till the formation of TMJ
• At 12th week IUL temporomandibular joint cavity appears as a cleft in the vascular fibrous tissue formed by the
mesenchymal differentiation between condylar and temporal blastema. The cavity is later invaded by synovial membrane
• The developing articular disc assumes its biconvex shape. The disc is continuous with the tendon of lateral pterygoid
muscle anteriorly and attached to malleus posteriorly
4. Development of Face and Nose
• The face develops from frontonasal process, paired maxillary and mandibular processes.
• At the 5th week of IUL, the ectodermal lining of the frontonasal process forms a thickening on both anterior and
inferolateral borders to form the optic and olfactory placodes.
• The medial arm end of the proliferating olfactory placode is called the median nasal process (MNP) and the lateral arm
is called the lateral nasal process (LNP)
• As the olfactory placodes proliferate and enlarge, a depression (olfactory pit or the nasal pit) appears in the center. The nasal
pit further enlarges towards the developing brain, producing the nasal sac. The nasal sacs later develop into the nasal cavities
• Facial skeleton develops intramembraneously from ossification centers in the embryonic facial process. Bony fusion of
the various units occurs at the 5th month of IUL.

https://t.me/DentalBooksWorld
Chapter 10 • Oral Anatomy and Histology 861

Frontonasal process
(forehead, bridge of the nose)

Lateral nasal
Olfactory pit or process
nasal pit (alae of the nose)
Median nasal
process
(crest and p of
the nose, philtrum)

5. Development of Oral Cavity
• The maxillary process grows in the ventral direction and fuses with the LNP to establish continuity between the future
cheek and nose
• The extensions of the mesenchyme from the MNP bulges into the stomodeum in the anterior region form the
premaxillary process or globular process, which later gives rise to the primary palate and philtrum
• The right and left MNPs fuse with each other and fuse with the maxillary processes on the corresponding side to form the
upper lip
• The mandibular processes grow medially toward each other and fuse to establish the continuity of lower lip
• The surface ectoderm proliferates into the underlying mesenchyme to form a band of epithelial cells called vestibular
lamina which undergo autolysis to establish the labial and buccal sulcus

Frontonasal process

Maxillary process
Mandibular process
Hyoid arch

6. Development of Tongue

Parts Origin General sensory Special sensory (taste)


innervations innervation
Anterior two-thirds of body Two lateral lingual swellings Lingual branch of Chorda tympani branch of
of the tongue (first pharyngeal arch) mandibular branch of facial nerve
trigeminal nerve
Posterior one-third of root of Copula of the second, third, Glossopharyngeal nerve Glossopharyngeal nerve
the tongue and anterior parts of the
fourth pharyngeal arch
Epiglottis Hypobranchial eminence Superior laryngeal branch of Vagus nerve
(fourth pharyngeal arch) vagus nerve
• The intrinsic muscles of the tongue, except the palatoglossus, arise from the occipital myotome and are supplied by the
hypoglossal nerve. The palatoglossus is supplied by the pharyngeal plexus.
7. Development of Palate
• The primary palate is derived from the globular process or the premaxillary segment of the MNP

https://t.me/DentalBooksWorld
862 Triumph’s Complete Review of Dentistry

• In the 6th week of IUL, two outgrowths develop from the maxillary process towards the midline, called palatal shelves
which are vertically oriented
• With the development of the tongue, the palatal shelves ascend to a horizontal position and fuse with each other at 9th
week of IUL to form the secondary palate and fuse with primary palate anteriorly
• The closure of secondary palate proceeds gradually from the anterior to the posterior direction

DEVELOPMENT OF TEETH

1. Enamel Organ, Tooth Germ, and Dental Lamina


• The primitive oral cavity is lined by stratified squamous epithelium called the oral ectoderm
• Oral ectoderm is made of two to three layers of epithelium and covers the embryonic connective tissue which is termed
ectomesenchyme

Vesbular Dental lamina


lamina

• At 5–6 week IUL, at some areas of oral ectoderm basal cells proliferate to form the primary epithelial band (future
dental arches)
• Primary epithelial band gives rise to dental lamina and vestibular lamina
• At 10 areas in each arch corresponding to the 10 deciduous teeth, the cells undergo further proliferation to form enamel
organ (gives rise to enamel) which protrude into the underlying ectomesenchyme
• The portion of the condensed ectomesenchyme immediately under the enamel organ is the dental papilla (gives rise to
dentin and pulp), and the portion surrounding the enamel organ and the dental papilla is the dental follicle or the dental
sac (gives rise to cementum, PDL, and alveolar bone)
• The enamel organ, dental papilla and dental follicle constitute the tooth germ. The development of the tooth germ is
divided into bud stage, cap stage, and bell stage based on the shape of the enamel organ
• Histophysiologic process: Initiation
2. Development of Enamel Organ of Permanent Teeth
• The dental lamina of the deciduous teeth undergoes lingual and distal extensions to give rise to the enamel organs of the
permanent teeth.
• The permanent central and lateral incisors, canines, and the first and second premolars are formed by the lingual
extensions of dental lamina (succedaneous teeth) while permanent first, second, and third molars are developed from
the distal extension of dental lamina (nonsuccedaneous teeth).
• The activity of the dental lamina extends for 5 years, after which it begins to degenerate because of mesenchymal invasion.
In some areas the remnants persist as epithelial islands known as cell rests of Serres in the jaw or in the gingiva.
3. Bud Stage
• The enamel organ, which looks like a bud, consists of low columnar cells in the periphery and polygonal cells in the center.
• Dental papilla (condensation of ectomesenchyme immediately adjacent to the enamel organ) and dental follicle can be
delineated at this stage
• Histophysiologic process: Proliferation

https://t.me/DentalBooksWorld
Chapter 10 • Oral Anatomy and Histology 863

Oral epithelium

Dental lamina

Peripherally placed
low columnar cells
Dental follicle
Centrally placed
polygonal cells
Dental papilla

4. Cap Stage
• The enamel organ shows unequal rate of proliferation in different parts which leads to a stage where the enamel organ
looks like a cap
• The cells of the enamel organ in the convex portion of the cap are cuboidal in shape and form the outer enamel epithelium
• The cells in the concavity of the cap are columnar in shape and form the inner enamel epithelium
• The polygonal cells in the center of the enamel organ synthesize glycosaminoglycans which pull water inside, forcing the
cells to move apart. The cells retain attachment through cytoplasmic processes. Thus, star-shaped cellular network called
stellate reticulum is formed.
• The dental papilla becomes more vascular with budding capillaries, cells are crowded and mitotic figures are evidenced.
• The dental sac appears more condensed and fibrous
• Histophysiologic process: Proliferation

Outer enamel
epithelium
Stellate Dental follicle
reculum

Inner enamel
epithelium
Dental papilla

5. Bell Stage
• The enamel organ further invaginates with growth in the margins and takes the shape of a bell.
• The columnar inner enamel epithelial layer undergoes histodifferentiation to form tall columnar cells called
ameloblasts. These cells are responsible for enamel formation.
• The inner enamel epithelial layer also demonstrates in folding determining the crown pattern of the tooth (morpho
differentiation). This occurs due to differential rates of mitotic divisions within the cell layer.

https://t.me/DentalBooksWorld
864 Triumph’s Complete Review of Dentistry

Rests of dental lamina/cell


rests of Serres

Outer enamel epithelium


Dental follicle

Stellate reculum

Stratum intermedium
Odontoblasts

Inner enamel epithelium Dental papilla

• Simultaneously, the peripherally placed undifferentiated ectomesenchymal cells of the dental papilla differentiate to
form columnar cells called odontoblasts. These cells are responsible for dentin formation.
• The basement membrane which separates the enamel organ and dental papilla just before dentin formation is called
membrana preformativa and this membrane develops in to future dentinoenamel junction.
• The junction between the inner and outer enamel epithelia is known as the zone of reflexion or cervical loop.
• After a layer of dentin is laid down, the inner enamel epithelial cells are deprived of their nutritional supply from the
dental papilla.
• This is compensated by collapse of the stellate reticulum cells and folding of the inner enamel epithelial layer bringing
the dental follicle capillaries closer to the inner enamel epithelial cells.
• Few layers of squamous epithelial cells called stratum intermedium is seen between inner enamel epithelial cell layer and
stellate reticulum cells.
• The dental follicle becomes more fibrous. Future periodontal ligament fibers are differentiated from these fibers.
• Histophysiologic process: Histodifferentiation and Morphodifferentiation
6. Advanced Bell Stage
• Advanced bell stage is characterized by commencement of mineralization of hard tissues and root formation.
• Odontoblast elaborate organic matrix of dentin along the dentinoenamel junction, initially in the region of future cusp.
This matrix proceeds pulpally and apically and mineralizes later.
• Ameloblasts produce organic matrix of enamel which mineralizes almost immediately. The matrix is laid from the
dentinoenamel junction toward the outer surface, consequently the ameloblasts move coronally and cervically.
• Histophysiologic process: Histodifferentiation, Morphodifferentiation, and Apposition (Formation of organic matrix)

Enamel

Dental follicle
Ameloblasts
Denn matrix
Outer enamel
Odontoblasts epithelium

Stratum intermedium

Stellate reculum Dental papilla

https://t.me/DentalBooksWorld
Chapter 10 • Oral Anatomy and Histology 865

7. Reciprocal Induction

INNER ENAMEL AMELOBLAST


EPITHELIAL CELLS differentiation

INDUCTION

Enamel formation
RECIPROCAL
INDUCTION

Differentiation of dental papilla


undifferentiated mesenchymal cells to
ODONTOBLASTS
First layer of dentin
laid down

8. Transient Structures
• The cells in the center of the concavity of the “cap” of the enamel organ are densely packed and form a knob-like
enlargement projecting toward the underlying dental papilla called primary enamel knot.

ENAMEL KNOT

Vercally extend towards


center - ENAMEL CORD

If enamel cord meets


outer enamel
epithelium - ENAMEL SEPTUM

So formed depression
in outer enamel
epithelium - ENAMEL NAVEL

9. HERS, Radicular Dentin, and Cementum Formation


• Hertwig’s epithelial root sheath (HERS) arises from the cervical portion of the enamel organ. This sheath plays an
important role in the determination of the shape, length, size, and number of roots.
• HERS is a double-layered structure composed of inner enamel epithelium and outer enamel epithelium cell layers.
• HERS induce adjacent dental papilla cells differentiation into odontoblasts which in turn produces radicular dentin.
• With the formation of radicular dentin, the HERS loses the continuity allowing dental follicle cells to come in contact
with radicular dentin. This induces cementoblasts differentiation in the dental follicle eventually forming cementum.
• Remnants of HERS persist as cords or islands and are known as Epithelial cell rests of Malassez.

10.  Root Formation


• HERS bend at the future cementoenamel junction onto a horizontal plane forming the epithelial diaphragm. The
diaphragm narrows the wide cervical opening of the tooth germ.

https://t.me/DentalBooksWorld
866 Triumph’s Complete Review of Dentistry

• Proliferation of the cells of the epithelial diaphragm is accompanied by the proliferation of the dental papilla resulting in
the lengthening of the root sheath.
• During the last stages of root development, the proliferation of cells of the epithelial diaphragm (HERS) lags behind that
of the cells of the dental papilla narrowing the cervical opening. Dentin and cementum deposition further narrows the
opening. In this way, the apical third of the root becomes conical.
• In case of multirooted teeth, tongue-like extensions develop on the horizontal diaphragm due to differential growth of
the diaphragm, free ends of which grow toward each other and fuse.

QUICK FACTS

Shapes of the Occlusal Surfaces of Some Teeth


Permanent maxillary 1st premolar Hexagonal
Permanent mandibular 1st premolar Diamond
Permanent mandibular 2nd premolar Square
Permanent maxillary 1st molar Rhomboidal
Primary maxillary 2nd molar
Permanent maxillary 2nd molar Rhomboidal with more obtuse angles
Permanent maxillary 3rd molar Heart shape
Permanent mandibular 1st molar Hexagonal/Trapezoidal
Primary maxillary 1st molar Rectangular

Longest tooth in oral cavity Maxillary canine (10 + 17 mm)


Mandibular canine (11 + 16 mm)
Longest crown in maxillary arch Central incisor (10.5 mm)
Longest crown in mandibular arch or longest crown in oral cavity Mandibular canine (11 mm)
Maximum root length in maxillary arch/longest root in oral cavity Maxillary canine (17 mm)
Maximum root in length in mandibular arch Mandibular canine (16 mm)
Shortest tooth of the dentition or shortest tooth in maxillary arch Maxillary second molar (6.5 + 13 mm)
Shortest tooth in mandibular arch Second molar (7 + 13 mm)
Shortest crown in the dentition or shortest crown in maxillary arch Maxillary second molar (6.5 mm)
Shortest crown in mandibular arch Second molar (7 mm)
Shortest root in maxillary arch or shortest root in oral cavity Maxillary second molar (11 mm)
Shortest root in mandibular arch Central incisor (12.5 mm)
Largest mesiodistal diameter of the crown in anteriors Maxillary central incisor
Largest mesiodistal diameter of the crown in oral cavity Mandibular 1st molar
Largest labiolingual diameter of the crown in anteriors Maxillary canine
Largest buccolingual diameter of the crown in oral cavity Maxillary 1st molar
Posterior tooth with longest root Palatal root of maxillary 1st molar
Largest mesiodistal diameter in primary dentition Mandibular 2nd molar
Largest buccolingual diameter in primary dentition Maxillary 2nd molar
Maxillary lateral incisor Palatogingival groove
Mandibular 1st premolar Mesiolingual developmental groove
Maxillary 1st premolar Mesial marginal developmental groove

https://t.me/DentalBooksWorld
Chapter 10 • Oral Anatomy and Histology 867

The primary tooth that has the most distinctly prominent facial cervical Mandibular 1st molar
ridge is
The lingual cusp of the maxillary premolars is offset to The mesial the first more than the second
The primary second molar generally exhibits Cusp of Carabelli
Mamelons that remain beyond the age of 10 generally Indicate an open bite
Mandibular central incisors and Maxillary third molars generally Only one opposing tooth
occlude with
The developmental groove between the DF cusp and the DL cusp of the Distofacial
mandibular 1st molar is
The roots of the maxillary second molar tend to be Less divergent and have greater distal
inclinations
The teeth whose function is primarily biting are Incisors and canines
The groove pattern for the mandibular first molar is Y or Dryopithecus pattern
The groove pattern for the mandibular second molar is A cross (+) pattern.
The cusp pattern for the mandibular second pre molar is H, U, and Y (not V) – NEET 2018
When compared to a maxillary canine the mandibular canine has contact More incisally
areas located
The mandibular 1st premolar the mesial marginal ridge located more Cervical than the distal
The oblique ridge of maxillary molar Forms the distal boundary of the central fossa
A transverse ridge results from The union of the facial and lingual triangular
ridges
Maxillary incisors are the only anterior teeth that are Wider mesiodistally than faciolingually
Mandibular molars are the only posterior teeth That are wider mesiodistally than faciolingually
The primary maxillary 2nd molar is the primary tooth that generally has An oblique ridge
The mesiolingual cusp of the Maxillary molars occludes in the Central fossa of the mandibular molars
The distobuccal cusp of the mandibular molars occludes in the Central fossa of the maxillary molars.
The primary second molar exhibits more cusps Than the primary first molar.
The contact between a max central and lateral incisor makes the Lingual embrasure larger than the facial
The nonmolar tooth that most frequently has a mesial and distal pulp Max central incisor
horn is the
The nonworking condyle moves Downward, forward, and medial
The nonmolar tooth that most frequently exhibits three roots is The maxillary 1st premolar
The mesiofacial and distolingual angles from the occlusal outline Acute angles maxillary molars
tend to be
This mesiolingual and distofacial angles from the occlusal outline Obtuse angles maxillary molars
tend to be
The obtuse corners coincide with The direction of the oblique ridge
Cingulum present in 12 teeth in permanent dentition
The mandibular canine is the anterior tooth that most frequently Bifurcated root that is facial and lingual
exhibits a
The cross-section of the mandibular canine at the CEJ is OVOID but wider mesiodistally at the labial
The nonmolar that is least likely to have a bifurcated root is the Maxillary central incisor
Most prominent marginal ridges of all anterior teeth is present in Maxillary lateral incisor
Distinct and deepest lingual fossae of all anterior teeth is present in Maxillary lateral incisor
The occlusal outline for the mandibular 1st premolar occlusal view is Diamond shaped
All premolars are wider Faciolingually than mesiodistally

https://t.me/DentalBooksWorld
868 Triumph’s Complete Review of Dentistry

The maxillary 2nd premolar has Two cusps that are of equal height
When a 4th pulp canal is present in a maxillary first molar it is located Mesiofacial canal
in the
The crown form of canines from a facial view is Pentagonal
Mandibular central incisors have proximal contacts at approximately The same levels on mesial and distal
Viewed from the occlusal the 4 posterior teeth in the mandibular arch are Aligned in a straight line
Only primary tooth to have oblique and transverse ridges and DL groove Primary 2nd molar 
The occlusal table of a posterior tooth makes up 55–65% of the total faciolingual dimension
Facial view of a primary mandibular 1st molar the CEJ is Apically positioned toward the mesial one-
third
The mesiodistal width of the mandibular lateral incisor is Wider than the mandibular central incisor
The mesiodistal width of the maxillary lateral incisor is Narrower than the maxillary central incisor
The premolar that has a longer mesiofacial cusp ridge than distofacial cusp Maxillary 1st
ridge is the
The facial cusp of the maxillary 1st premolar is offset to the Distal
A common trait of maxillary premolars is that their lingual cusps are off Mesial
set to the
The premolar with the steepest cusp inclines is the Maxillary 1st premolar
The primary mandibular 1st molar usually exhibits a Distal triangular fossa
The largest cusp of the mandibular first molar is the Mesiofacial
The mandibular 1st premolar has a uniquely Prominent triangular ridge
The cervical cross-section of the maxillary 1st premolar exhibits a Kidney-shaped root outline

MULTIPLE CHOICE QUESTIONS

TOOTH NUMBERING SYSTEM


1. The universal numbering system given by
2. Dental Formula for Permanent Teeth
3. Dental Formula for Primary/Deciduous Teeth
4. The MICAP Notation is a recently developed system given by
5. FDI system by
6. Identify the system

7:, 6:, 3:, 2:, ,1: :1, :2, :3, :4, :5,

7:, 6:, 3:, 2:, ,1: :1, :2, :3, :4, :5,
7. Identify the system

21 12 1 1 21 12 321 123
Bleibende Zähne # I
21 12
# C
1 1
# P
21 12
# M
321 123

https://t.me/DentalBooksWorld
Chapter 10 • Oral Anatomy and Histology 869

21 12 1 1 – 21 12
Milchzähne

#d I
21 12
C
#d
1 1
M
#d
21 12
8. Identify the system (this question was asked in November 2016 AIIMS for medical students)

Upper right Upper left


05+ 04+ 03+ 02+ 01+ +01 +02 +03 +04 +05
05– 04– 03– 02– 01– –01 –02 –03 –04 –05
Lower right Lower left
9. Identify the system

Upper right Upper left


05+ 04+ 03+ 02+ 01+ +01 +02 +03 +04 +05
05– 04– 03– 02– 01– –01 –02 –03 –04 –05
Lower right Lower left
10. Identify the system

Upper right Upper left


1e 1d 1c 1b 1a 2a 2b 2c 2d 2e
4e 4d 4c 4b 4a 3a 3b 3c 3d 3e
Lower right Lower left
11. Identify the system

.10 .8 .6 .4 .2 .1 .3 .5 .7 .9

.10 .8 .6 .4 .2 .1 .3 .5 .7 .9
12. Identify the system

16 15 14 13 12 11 10 9 8 7 6 5 4 3 2 1

17 18 19 20 21 22 23 24 25 26 27 28 29 30 31 32

molars molars
premolars incisors premolars
canines canines

https://t.me/DentalBooksWorld
870 Triumph’s Complete Review of Dentistry

13. Identify the system

18 17 16 15 14 13 12 11 21 22 23 24 25 26 27 28

48 47 46 45 44 43 42 41 31 32 33 34 35 36 37 38

molars premolars molars


incisors premolars
canines canines

14. Universal System for Deciduous Teeth “T” denotes


A. Maxillary right second molar B. Maxillary left second molar
C. Mandibular left second molar D. Mandibular right second molar
15. Identify the system

87654321 12345678

87654321 12345678
16. Which system cannot be coded by computer?
A. Zsigmondy And Palmer Tooth Numbering System B. Universal system for Permanent teeth
C. FDI tooth numbering system D. All the above
17. There is no differentiation between right upper, right lower, left upper and left lower in which system?
A. Zsigmondy and Palmer Tooth Numbering System B. Universal system for Permanent teeth
C. FDI tooth numbering system D. All the above
18. In Universal system of tooth numbering, if wisdom tooth is missing, then what number will be assigned to the adjacent
2nd molar tooth?
A. Assigned as number 1 since in this case second molar is the 1st tooth in the arch
B. Assigned as number 2 even if third molar is missing
C. Assigned as 7 (counting from central incisor)
D. Wisdom teeth are never taken into consideration in case of universal system of tooth numbering and so it is assigned
as number 1
19. Zsigmondy And Palmar Notation was adopted in the year?
A. 1861 for permanent dentition and 1874 for deciduous dentition
B. 1874 for permanent dentition and 1861 for deciduous dentition
C. Both dentition 1874
D. Both dentition in 1861

ENAMEL, DENTIN, PULP

1. Enamel is permeable to
A. Bacteria B. Bacterial products
C. Peroxides D. Retrovirus
2. Network of nerves located adjacent to the cell rich zone is known as
A. Plexus of Raschkow B. Myelinated nerve sheath
C. Nonmyelinated nerve sheath D. Brachial plexus

https://t.me/DentalBooksWorld
Chapter 10 • Oral Anatomy and Histology 871

3. Accentuated incremental lines seen in dentin due to the disturbances in the matrix and mineralization process is
known as
A. Incremental lines of Von Ebner B. Contour lines of Owen
C. Incremental lines of Salter D. Incremental lines of Retzius
4. Alternating dark and light strips of varying width that can be viewed in the ground section of enamel under reflected
light is
A. Enamel rods B. Hunter-Schreger bands
C. Gnarled enamel D. Incremental lines
5. Type II enamel etching pattern
A. Involves the dissolution of prism peripheries B. Involves the dissolution of prism cores
C. Not related with enamel prism D. Involves dentin and enamel
6. False statement regarding interglobular dentin is
A. The term “interglobular dentin” refers to organic matrix that remains unmineralized because the mineralizing globules
fail to coalesce
B. This occurs most often in the circumpulpal dentin just below the mantle dentin
C. Where the pattern of mineralization is likely to be oppositional than globular
D. Large area of interglobular dentin is a characteristic feature of childhood hypophosphatasia
7. Matrix vesicles play an important role in the mineralization of
A. Dentin B. Enamel
C. Pulp D. Periodontal ligament
8. The most common cause of dental hypersensitivity
A. Movement of fluid in dentinal tubules B. Transduction of odontoblasts and other pulpal cells
C. Direct stimulation of nerve endings at outer dentin D. Direct stimulation of pulpal nerves
9. Number of enamel rods ranges on an average in the order of
A. 2–5 million B. 5–12 million
C. 7–15 million D. 10–19 million
10. Cells occurring in greatest number in the pulp are (COMED-08)
A. Cementoblasts B. Fibroblasts
C. Osteoblasts D. Ameloblasts
11. Dentinal sensitivity is attributed for
A. Neural stimulation of dentinal tubules
B. Craze lines in dentin
C. Dentinoenamel junction
D. Pain transmission through movements of fluid in dentinal tubules
12. The most accepted theory for dentine sensitivity is
A. Transduction theory B. Direct neural stimulation
C. Hydrodynamic theory D. Hydrostatic theory
13. Odontoblastic processes are also known as
A. Tome’s process B. Tome’s granular layer
C. Tome’s fibers D. Intratubular dentin
14. Fibroblasts in the pulp produce collagen of
A. Type I only B. Type II and III
C. Types I and III D. Types I and II
15. The largest portion of the tooth structure is formed by
A. Pulp cavity B. Cementum
C. Periodontal membrane D. Dentine
16. The enamel is thickest at the
A. Cervical margin B. Incisal and occlusal areas
C. Contact area D. Middle third

https://t.me/DentalBooksWorld
872 Triumph’s Complete Review of Dentistry

17. Which of the following does not have calcified tissue?


A. Enamel B. Dentine
C. Pulp D. Cementum
18. Dentinal tubule is wide at
A. Enamel end B. Pulpal end
C. Cemental end D. None of the above
19. Incipient caries consists of opaque, chalky white areas that appear when the tooth surface is dried; this is referred to as
A. White spot B. Hot spot
C. Translucent zone D. Body of lesion
20. The diameter of the largest arterial vessels in the pulp range from
A. 50–100 µm B. 101–150 µm
C. 151–200 µm D. 201–250 µm
21. The compressive strength of dentine is approximately
A. 468 MPa B. 162 MPa
C. 350 MPa D. 266 MPa
22. Interglobular dentine occurs due to defective
A. Formation B. Histodifferentiation
C. Mineralization D. None of the above
23. Obliterated pulp chambers with continuous deposition of dentine is a characteristic feature of
A. Amelogenesis imperfecta B. Regional odontodysplasia
C. Dentinogenesis imperfecta D. Dentinal dysplasia
24. In the formation of interglobular dentin and entrapment of odontoblasts within ossification can be compared
A. Endochondral ossification B. Intramembranous ossification
C. Osteoid D. Both endochondral and intramembranous ossification
25. Which of the following are hypomineralized structures in enamel?
A. Enamel lamellae and incremental lines B. Enamel lamellae and gnarled enamel
C. Enamel lamellae and perikymata D. Enamel lamellae and enamel prism
26. Disturbances occurring during calcification stage of development results in
A. Mantle dentin B. Interglobular dentin
C. Transparent dentin D. Granular dentin
27. The “Key hole” pattern appearance in the cross-section is a feature of
A. Aprismatic enamel B. Prismatic enamel
C. Intertubular dentin D. Intratubular dentin
28. Each one of the following is an intrinsic form of tooth discoloration, except
A. Endemic fluorosis B. Hereditary opalescent dentin
C. Tetracycline staining D. Peridex staining
29. The shallow furrows on the enamel surface, where striae of Retzius end are known as
A. Cracks B. Pellicle
C. Perikymata D. Enamel lamellae
30. In adult tooth, the type of dentin present underlying dentin–enamel junction is
A. Predentin B. Mantle dentin
C. Interglobular dentin D. Tertiary dentin
31. The most accepted theory of dentin sensitivity is
A. Transduction theory B. Direct neural stimulation
C. Hydrodynamic theory D. Hydrostatic theory
32. The histological appearance of “lava following around boulders in dentin dysplasia” suggests
A. Attempt to repair the defective dentin
B. Abrupt arrest to dentin formation in crown
C. Abnormal dentin formation in a disorganized fashion
D. Cascades of dentin to form root

https://t.me/DentalBooksWorld
Chapter 10 • Oral Anatomy and Histology 873

33. The term “Dead tract” in dentin was coined by


A. Robinson B. Sir Wilfred Fish
C. Silver man D. Silverstone
34. The number of enamel rods in a maxillary molar is
A. 15 million B. 12 million
C. 7 million D. 9 million
35. Water content in dentine is
A. 10% B. 5%
C. 7% D. 15%
36. Which type of dentin is most regular in structure?
A. Primary dentin B. Secondary dentin
C. Reparative dentin D. All have the same regular structure
37. Dentinal tubules which enter into enamel are known as
A. Enamel spindles B. Enamel tufts
C. Enamel lamellae D. None of the above
38. The most highly mineralized oral or dental tissue is
A. Enamel B. Dentin
C. Bone D. Cementum
39. Odontoblasts laying down dentin matrix would be expected to contain large numbers of well-developed
A. Nuclei B. Lysosomes
C. Rough ER D. Smooth ER
E. Mitochondria
40. Hypocalcified enamel rods which enter into enamel from the DEJ are known as
A. Enamel spindles B. Enamel tufts
C. Enamel lamellae D. None of the above
41. The most numerous cell type of the dental pulp is the
A. Neuron B. Fibroblast
C. Leukocyte D. Macrophage
E. Ameloblast
42. Which two tissues are formed by the same part of the tooth bud?
A. Enamel and dentin B. Dentin and cementum
C. Dentin and pulp D. Pulp and cementum
E. None of the above
43. The Golgi apparatus in odontoblasts producing dentin can be expected to be
A. Absent
B. Less developed than in an average cell
C. Developed to the same degree as that of an average cell
D. More highly developed than that of an average cell

PDL, CEMENTUM, BONE

1. Which of the following fiber groups are not attached to alveolar bone?
A. Transseptal B. Oblique
C. Horizontal D. Dentoperiosteal
2. Cementicles are
A. Calcified thrombosed blood vessels B. Epithelial rests
C. Calcified Sharpey’s fibers D. All of the above
3. Cartilage differs from bone in that, the cartilage can increase in size by
A. Apposition B. Interstitial growth
C. Selective resorption D. Endosteal remodeling

https://t.me/DentalBooksWorld
874 Triumph’s Complete Review of Dentistry

4. The type of cementum located in the furcation areas of multirooted teeth is


A. Cellular extrinsic fiber cementum B. Cellular intrinsic fiber cementum
C. Cellular mixed stratified cementum D. Acellular intrinsic fiber cementum
5. Remnants of osteons left behind during remodeling are called
A. Concentric lamellae B. Circumferential lamella
C. Interstitial lamellae D. Volkmann’s canals
6. Incremental lines seen in cementum are called
A. Incremental lines of Retzius B. Incremental lines of Von Ebner
C. Incremental lines of Salter D. Resting lines
7. Sharpey’s fibers are seen in
A. Acellular extrinsic fiber cementum B. Cellular intrinsic fiber cementum
C. Acellular fibrillar cementum D. None of the above
8. Premature bone loss in primary teeth could be seen in conditions like
A. Hypophosphatasia B. Fibrous dysplasia
C. Papillon–Lefevre syndrome D. All of the above
9. Osteoblast covering the periodontal surface of the alveolar bone constitute a
A. Modified periosteum B. Modified endosteum
C. Periosteum D. Endosteum
10. Degeneration of periodontal tissue without inflammation is known as
A. Desquamative gingivitis B. Chronic periodontitis
C. Periodontosis D. Pericoronitis
11. Calculus embedded deep in the cementum is termed
A. Subgingival calculus B. Altered cementum
C. Calculo cementum D. Ankylosis
12. Periodontal fibers are thinnest at
A. Cervical B. Middle
C. Apical D. Fulcrum of tooth
13. Traumatic occlusion does not affect
A. Bone B. Cementum
C. Periodontal ligament D. Epithelial attachment
14. Bone formed after successful osseous regeneration
A. Bundle bone B. Cancellous bone
C. Woven bone D. Lamellar bone
15. Periodontal fibers not attached to the bone
A. Transseptal B. Oblique
C. Apical D. Horizontal
16. Elastic fiber content of periodontal ligament
A. Remains same B. Increases
C. Decreases D. Either increases or decreases with age
17. Elaunin fibers are found in
A. Gingiva B. Cementum
C. Alveolar bone D. Periodontal ligament
18. Which type of collagen fibers demonstrates “chicken wire” configuration?
A. Type I B. Type II
C. Type III D. Type IV
19. The most common clinical sign of trauma to the periodontium is
A. Gingival inflammation B. Increased attachment loss
C. Increased tooth mobility D. Pocket formation
20. When osteoblast lay new bone from outside to inside in a osteon, the active formation area is called
A. Haversian cone B. Filling cone
C. Cutting cone D. Howship’s cone

https://t.me/DentalBooksWorld
Chapter 10 • Oral Anatomy and Histology 875

21. Osteoclasts show an intense activity of which of the following enzymes?


A. Alkaline phosphatase B. Acid phosphatase
C. Sterase D. Carbonic anhydrase
22. Gingivectomy of which of the fibers can reduce the relapse of orthodontically moved teeth?
A. Gingival fibers B. Alveolar crest fibers
C. Transseptal D. None
23. Absence of cementum is a feature of
A. Vitamin D resistant rickets B. Hypocalcaemia
C. Hypophosphatasia D. Osteoporosis
24. Collagen fibers are absent in
A. Acellular extrinsic fiber cementum B. Acellular afibrillar cementum
C. Mixed stratified cementum D. Intermediate cementum
25. Type of Collagen found in principal fibers is
A. Type I B. Type II
C. Type III D. Type IV
26. The apical group of periodontal fibers flares from
A. End root B. Cervical part of the root
C. Apical third of the root D. The pivot region of the roots
27. Inorganic content of cementum is
A. 45–50% B. 50–55%
C. 55–60% D. 60–65%
28. Cementicles are calcified masses sometimes seen in the
A. Pulp B. Bone
C. Periodontal ligament D. Cementum
29. Canaliculi and Haversian systems
A. Are found in acellular cementum B. Are found in cellular cementum
C. Are found in both types of cementum D. Are not found in cementum
30. The marrow space inside alveolar bone surrounding teeth
A. Does not exist B. Is usually red marrow
C. Is usually hematopoietic marrow D. Is usually yellow marrow
31. Collagen fibers inserting into compact bone of the alveolar socket are called
A. Tomes processes
B. Sharpey’s fibers
C. Von Ebner lines
D. Contour lines of Owen
E. None of the above
32. Alveolar bone is composed of
A. Cortical bone only B. Cancellous bone only
C. Cortical bone surrounding cancellous bone D. Cancellous bone surrounding cortical bone
33. Sharpey’s fibers in the periodontal ligament
A. Enter only alveolar bone B. Enter only cementum
C. Are collagenous in composition D. Enter cancellous bone to form bundle bone

PRIMARY AND PERMANENT TOOTH CALCIFICATION AND ERUPTION,


SHEDDING AND FORMS
1. Primary Begin to calcify about _______ in utero
A. 3 months B. 2 months
C. 4 months D. 10 months

https://t.me/DentalBooksWorld
876 Triumph’s Complete Review of Dentistry

2. The last permanent anterior tooth of the Mx to initiate calcification is the


A. Lateral incisor B. Second molar
C. Canine D. Central incisor
3. Active eruption of teeth occurs after _______ of the root is formed
A. One-half B. Three-fourths
C. One-third D. Root completion
4. First succedaneous to erupt is the ___________ at 6–7 years old
A. Mandibular 1st molar B. Maxillary 1st molar
C. Mandibular lateral incisor D. Mandibular central incisor
5. Eruption age of lower canine is
A. 9–10 years B. 8–9 years
C. 10–11 years D. 11–12 years
6. On a panoramic radiograph of a 7-year old, the dentist will expect to see all of the following except
A. Primary Mx laterals w/ partially resorbed roots
B. Partially erupted Mx centrals w/ incomplete root closure at the apex
C. Partially erupted Mn laterals w/ incomplete root closure at the apex
D. Fully erupted Mn M1s w/ incomplete root formation & non-closure at the apex
7. Identify the tooth


A. Maxillary right central incisor B. Maxillary right lateral incisor
C. Maxillary left central incisor D. Maxillary left lateral incisor
8. Identify the tooth


A. Maxillary right 1st premolar B. Maxillary right 2nd premolar
C. Maxillary left 1st premolar D. Maxillary left 2nd premolar
9. Identify the tooth


A. Maxillary right 1st premolar mesial side B. Maxillary right 2nd premolar distal side
C. Maxillary left 1st premolar mesial side D. Maxillary left 2nd premolar mesial side

https://t.me/DentalBooksWorld
Chapter 10 • Oral Anatomy and Histology 877

10. Identify the tooth


A. Mandibular left canine B. Mandibular right canine
C. Maxillary right canine D. Maxillary left canine
11. Identify the tooth


A. Mandibular 1st premolar B. Mandibular 2nd premolar
C. Mandibular canine right side D. Mandibular canine left side
12. Identify the side of mandibular 1st premolar


A. Right and mesial B. Right and distal
C. Left and mesial D. Left and distal
13. Identify the tooth

  
A. Mandibular right 1st premolar B. Mandibular left 1st premolar
C. Mandibular right 2nd premolar D. Maxillary left 1st premolar
14. Identify the cross-section of the tooth
1 2 3 4 5 6

D

A. Maxillary central incisor B. Primary maxillary central incisor
C. Maxillary canine D. Primary maxillary canine

https://t.me/DentalBooksWorld
878 Triumph’s Complete Review of Dentistry

15. Identify the cross-section of the tooth


1 2 3 4 5 6

D
  
A. Maxillary central incisor B. Primary maxillary central incisor
C. Maxillary canine D. Maxillary lateral incisor
16. Occlusal table makes up ______% of the BL dimension of the tooth
A. 50 B. 40
C. 60 D. 70
17. Statement A – all maxillary incisors – Embrasures – L > F
Statement B – all mandibular incisors – Embrasures – F > L
A. Statement A is correct and Statement B is wrong B. Statement B is correct and Statement A is wrong
C. Both are correct D. Both are wrong
18. Find the false statement
A. Maxillary premolars are wider BL than MD
B. All premolars (max and mand) have same MD dimensions
C. Maxillary premolar are wider BL than mandibular PM
D. Maxillary 2nd PM is larger than 1st premolar
19. Mucin is
A. Glycoprotein B. Nucleoprotein
C. Phosphoprotein D. Chromoprotein
20. ________________ are the population of cells that develop from the ectoderm during the development of the lateral
margins of the neural plate
A. Neural crest cells B. Ameloblasts
C. Fibroblasts D. Odontoblasts
21. Minimum lobes from which a tooth develops
A. 2 B. 3
C. 4 D. 5
22. Number of developmental lobes in human permanent anterior teeth?
A. 1 B. 2
C. 3 D. 4
23. Single conical crown with a single root is called
A. Acrodont B. Haplodont
C. Diphyodont D. Polyphyodont
24. Facial muscles are derived from
A. 1st branchial arch B. 2nd branchial arch
C. 3rd branchial arch D. 4th branchial arch
25. Human beings are
A. Monophyodonts B. Polyphyodonts
C. Homodonts D. Diphyodonts

https://t.me/DentalBooksWorld
Chapter 10 • Oral Anatomy and Histology 879

Practice Questions

26. Smallest premolar is


A. Maxillary first B. Maxillary second
C. Mandibular first D. Mandibular second
27. Longest teeth of permanent dentition are
A. Canines B. Incisors
C. Molars D. Premolars
28. Centric holding cusp is
A. Mesiobuccal cusp of maxillary 1st molar B. Mesiolingual cusp of mandibular 1st molar
C. Distolingual cusp of mandibular 1st molar D. Mesiopalatal cusp of maxillary 1st molar
29. Calcification of permanent teeth begins at
A. 1 year B. 4 months IU
C. At birth D. 1 month after birth
30. The first primary tooth to erupt is the
A. Mandibular central incisor B. Maxillary central incisor
C. Mandibular first molar D. Maxillary first molar
31. Which of the following grooves separates cusp ridges from marginal ridges?
A. Supplemental B. Developmental
C. Mesiomarginal developmental D. Marginal ridge developmental
32. The occlusal outline of the permanent mandibular first molar is usually similar to a
A. Circle B. Square
C. Trapezoid D. Parallelogram
33. Most compatible tooth numbering system for feeding in the computer?
A. FDI B. Grid
C. Zsigmondy D. Palmar
34. Permanent mandibular canine erupts __________ in relation to primary mandibular canine
A. Lingually B. Facially
C. Distally D. Mesially
35. According to FDI tooth numbering system, the number 45 refers to
A. Deciduous mandibular right second molar B. Mandibular right second premolar
C. Maxillary right second premolar D. Deciduous maxillary right second molar
36. Which of the following is generally the longest root canal on the maxillary first molar?
A. Mesiobuccal B. Distobuccal
C. Palatal D. Distolingual
37. At what age, does calcification of the primary dentition begin?
A. 6–10 weeks in utero B. 13–16 weeks in utero
C. 22–26 weeks in utero D. 30–34 weeks in utero
38. Which of the following is likely to have bifurcated roots?
A. Mandibular canine B. Maxillary canine
C. Mandibular incisor D. Maxillary incisor
39. Primary dentition period is
A. Birth to 11 years B. 6 months to 11 years
C. 6 months to 6 years D. 6 years to 11 years
40. How many pulpal horns are present in mandibular 2nd primary molars?
A. 3 B. 6
C. 4 D. 5
41. Which of the following has a bifid root?
A. 32, 42 B. 31, 41
C. 34, 44 D. 13, 23

https://t.me/DentalBooksWorld
880 Triumph’s Complete Review of Dentistry

42. Steepest cusp is seen in


A. Maxillary first premolar B. Maxillary 2nd premolar
C. Mandibular 2nd molar D. Mandibular 1st premolar
43. Permanent mandibular 1st molar is developed from __________ lobes
A. 3 B. 4
C. 5 D. 6
44. The roots seen in permanent mandibular first molar are
A. Buccal root and lingual root B. Mesial root and distal root
C. Mesiobuccal, distobuccal, and lingual roots D. Mesiolingual, distolingual, and buccal roots
45. Which of the following teeth are referred to as cornerstone of the dental arches?
A. Incisors B. Canines
C. Premolars D. Molars
46. In the FDI tooth numbering system 51 denotes
A. Maxillary right primary central incisor B. Maxillary right primary second molar
C. Maxillary right permanent central incisor D. Maxillary right second premolar
47. The number of line angles in anterior teeth are
A. Three B. Four
C. Five D. Six
48. Eruption time of permanent mandibular canine is
A. 9–10 years B. 11–12 years
C. 6–7 years D. 17–25 years
49. At the age of 9 years, the mandibular permanent first molar has its distal contact with which of the following teeth?
A. First premolar B. Second premolar
C. Deciduous 2nd molar D. None
50. The position of maximum intercuspation of upper and lower teeth is referred to as centric
A. Occlusion B. Relation
C. Position D. Bite
51. Centric holding cusp is (Repeat AIPG 2014)
A. Mesiobuccal cusp of maxillary 1st molar B. Mesiolingual cusp of mandibular 1st molar
C. Distolingual cusp of mandibular 1st molar D. Mesiopalatal cusp of maxillary 1st molar
52. The enamel of permanent incisor forms between
A. Birth to 5 years B. Birth to 2 years
C. Birth to 10 years D. Birth to 7 years
53. The primary teeth differs from the permanent teeth in the following except
A. Cervical prominence is less in primary than in permanent
B. Primary teeth are lighter than permanent
C. Roots are very long and thin in primary teeth
D. Mesiodistal dimension is more than cervico incisal length
54. The “Y”-shaped occlusal pattern appears in
A. Mandibular 1st premolar B. Maxillary 2nd premolar
C. Maxillary 1st premolar D. Mandibular 2nd premolar
55. The highest and sharpest cusp on the deciduous mandibular first molar is
A. Mesiolingual B. Distolingual
C. Mesiobuccal D. Distobuccal
56. In the absence of permanent second premolar the roots of primary second molar will most likely (AP-2007)
A. Resorb rapidly than normal B. Resorb slowly than normal
C. No resorption at all D. Resorb at normal rate
57. Proximal view of which of the following tooth resembles rhomboid shape
A. Mandibular posterior teeth B. Maxillary posterior teeth
C. Mandibular anterior teeth D. Maxillary anterior teeth

https://t.me/DentalBooksWorld
Chapter 10 • Oral Anatomy and Histology 881

58. The term diphyodont means


A. Second dentition B. Two same sets of dentition
C. Two separate sets of dentition D. More than two sets of dentition
59. Space between 52 and 53 is called
A. Bolton space B. Freeway space
C. Leeway space D. Primate space
60. The first evidence of calcification of primary anterior teeth begins approximately between
A. 14 and 17 weeks in utero B. 9 and 12 weeks in utero
C. 6 and 9 weeks in utero D. 17 and 20 weeks in utero
61. According to Schour and Massler the crown completion of both permanent maxillary and mandibular first molar is
A. 1–2 years B. 2–3 years
C. 3–4 years D. 4–5 years
62. The maxillary lateral incisor is smaller than the maxillary central incisor on all aspects, except which one of the
following?
A. Crown length B. Mesiodistal crown width
C. Faciolingual crown width D. Root length
63. “Ugly duckling” stage of the transitional dentition is characterized by
A. Distoangular axial inclination of the maxillary incisors
B. Distoangular axial inclination of the mandibular incisors with spacing
C. Mandibular lateral incisors erupting lingual to the central incisors
D. Maxillary lateral incisors erupting lingual to the central incisors
64. Ugly duckling stage of dentition in children is corrected by eruption of which tooth?
A. Central incisor B. Lateral incisor
C. Canine D. Second molar
65. The tooth that is the most vulnerable to injury is the
A. Mandibular central incisor B. Maxillary central incisor
C. Mandibular lateral incisor D. Maxillary lateral incisor
66. Anatomic teeth should have a cusp angle of
A. 30° B. 31°
C. 32° D. 33°
67. Which of the following molar relationship is the most common in primary dentition?
A. Mesial step B. Distal step
C. Flush terminal plane D. Cannot say
68. Transposition of teeth is most commonly seen in the region of
A. Maxillary central and lateral incisors B. Mandibular canine and first premolar
C. Maxillary lateral incisor, canine and premolar D. Mandibular first and second premolars
69. The biting forces at molar region during mastication is
A. 20–55 pounds B. 30–75 pounds
C. 50–75 pounds D. 90–200 pounds
70. Roots of the permanent mandibular 1st molar are
A. Mesiobuccal, distobuccal, and lingual B. Mesiolingual, distolingual, and buccal
C. Mesial and distal D. Buccal and lingual
71. Mixed dentition lasts from
A. 6–9 years B. 8–9 years
C. 6–12 years D. 2–12 years
72. Longest root in the permanent maxillary molar is
A. Buccal B. Lingual
C. Mesiobuccal D. Distobuccal
73. Eruption of maxillary permanent canine takes place at the age of
A. 9–10 years B. 11–12 years
C. 13–14 years D. 8–9 years

https://t.me/DentalBooksWorld
882 Triumph’s Complete Review of Dentistry

74. Two roots are commonly seen in


A. Maxillary first premolar B. Mandibular first premolar
C. Mandibular second premolar D. Mandibular first molar
75. The first primary molar tooth contains
A. Four pulp horns and three root canals B. Three pulp horns and three root canals
C. Two pulp horns and two root canals D. Two pulp horns and three root canals
76. The number of point angles in a permanent mandibular first molar is
A. Four B. Six
C. Eight D. Ten
77. Following eruption, the root of the maxillary central incisor completes development at what age?
A. 7 years B. 8 years
C. 9 years D. 10 years
78. Two pulp canals are usually found in
A. Mesial root of permanent mandibular first molar B. Distal root of permanent mandibular first molar
C. Palatal root of permanent maxillary first molar D. Distal root of permanent mandibular second molar
79. Oblique ridge of permanent maxillary first molar connects
A. Mesiobuccal cusp and Distobuccal cusp B. Distobuccal and Mesiolingual cusp
C. Mesiolingual cusp and Distolingual cusp D. Mesiolingual cusp and Distolingual cusp
80. Occlusal aspect of the maxillary first premolar is roughly
A. Octagonal B. Trapezoidal
C. Hexagonal D. Rhomboidal
81. Number of line angles in anterior teeth
A. 6 line angles B. 8 line angles
C. 4 line angles D. 10 line angles

PRIMARY AND PERMANENT COMPARISON, VARIOUS TOOTH MORPHOLOGY


1. Which cusp on the primary mandibular first molar is the highest and sharpest?
A. Mesiobuccal B. Distobuccal
C. Mesiolingual D. Distolingual
2. The anterior tooth with the most pronounced lingual ridge is the
A. Maxillary central incisor B. Mandibular lateral incisor
C. Maxillary canine D. Mandibular canine
3. In comparison to those of permanent molars, the common root trunks of primary molars are proportionately
A. Larger B. Smaller
C. Relatively equal D. Variable, depending on the tooth
4. Which premolar is considered to be most molar-like in form?
A. Maxillary first B. Maxillary second
C. Mandibular first D. Mandibular second
5. The distal inclination in roots of the permanent mandibular first molar is
A. More pronounced in either root, depending on the tooth
B. Usually more pronounced in the mesial root
C. Usually more pronounced in the distal root
D. Usually equal in both roots
6. The greatest curvature of the cervical line interproximally is found on the
A. Mesial of the maxillary central B. Mesial of the maxillary lateral
C. Distal of the maxillary central D. Distal of the maxillary lateral
7. Which incisor commonly exhibits the most variety in anatomy in both crown form and root form?
A. Maxillary central B. Maxillary lateral
C. Mandibular central D. Mandibular lateral

https://t.me/DentalBooksWorld
Chapter 10 • Oral Anatomy and Histology 883

8. The most prominent cingulum is found on which incisor?


A. Maxillary central B. Maxillary lateral
C. Mandibular central D. Mandibular lateral
9. Which premolar has a lingually inclined occlusal table?
A. Maxillary first B. Maxillary second
C. Mandibular first D. Mandibular second
10. When viewed from the buccal, the tooth closest to vertical in the mesiodistal angulation of its main axis is the
A. Maxillary central incisor B. Maxillary lateral incisor
C. Maxillary canine D. Maxillary first premolar
11. At the cervical line, a cross-section of the maxillary canine would show
A. A round shape
B. An oval shape wider on the lingual than the labial
C. A triangular shape with a distinctly wider labial surface
D. An oval shape with a slightly wider labial surface
12. Which incisor is most commonly congenitally missing?
A. Maxillary central B. Maxillary lateral
C. Mandibular central D. Mandibular lateral
13. A patient is congenitally missing the maxillary lateral incisors and the mandibular second premolars. This individual
is said to have the condition known as
A. Anodontia B. Oligodontia
C. Hypodontia D. Hyperdontia
14. Cervical bulges are found on which surfaces of primary molar teeth?
A. Mesial and distal B. Buccal and lingual
C. Lingual only D. Buccal only
15. In rare cases, a permanent mandibular first molar may have a complete supernumerary third root. This root is usually
A. A second mesial root B. A second distal root
C. A buccal root D. A lingual root
16. At the cervical line, a cross-section of the maxillary lateral incisor would show
A. A round shape
B. An oval shape wider on the lingual than the labial
C. A triangular shape with a distinctly wider labial surface
D. An oval shape with a slightly wider labial surface
E. Mesiodistal flattening
17. The largest cusp of a maxillary first molar is the
A. Mesiobuccal B. Mesiolingual
C. Distobuccal D. Distolingual
E. Carabelli
18. Tooth #B is replaced by tooth #
A. 2 B. 3
C. 4 D. 5
E. 6
19. Which two molars can be considered to be five-cusped teeth?
A. Maxillary first and second B. Mandibular first and second
C. Maxillary and mandibular first D. Maxillary and mandibular second
20. The shape of the pulp chamber in a cross-section cut at the cervical line in the maxillary canine is
A. Oval (flattened mesiodistally) B. Oval (flattened buccolingually)
C. Round D. Figure eight
21. In a distal view of a permanent maxillary first molar, how many roots should be visible?
A. 1 B. 2
C. 3 D. 4

https://t.me/DentalBooksWorld
884 Triumph’s Complete Review of Dentistry

22. Dens-in-dente is most commonly noted in which tooth?


A. Maxillary central B. Maxillary lateral
C. Maxillary canine D. Mandibular central
E. Mandibular lateral
23. The maxillary molar most likely to have fused roots is the
A. First B. Second
C. Third D. Fused roots are rare for all three molars
24. Oblique ridges are found on which molar teeth?
A. Maxillary first and second B. Mandibular first and second
C. Maxillary and mandibular first D. Maxillary and mandibular second
E. First and second molars in both arches
25. When a fourth canal is found in the permanent maxillary first molar, it is found in the
A. Mesiobuccal root B. Distobuccal root
C. Palatal root D. Mesiolingual root
26. A mandibular permanent first molar is likely to have a deep root concavity running apically down
A. Both roots B. The mesial root
C. The distal root D. Neither root
27. Based on root morphology, the tooth most likely to be successfully rotated during exodontia is the
A. Maxillary central B. Mandibular central
C. Mandibular lateral D. Mandibular canine
28. As compared to the cross-section of the maxillary central incisor at the cervical line, the cross-section of the mandibular
central is
A. More flattened mesiodistally B. More flattened faciolingually
C. Rounder D. Almost identical
29. When compared to the buccal roots of the maxillary first molar, the buccal roots of the maxillary second molar are
A. More parallel, with a pliers-like appearance
B. Less parallel, lacking a pliers-like appearance
C. More parallel, with both having a mesial tip orientation
D. More parallel, with both having a distal tip orientation
30. Which of the following permanent teeth is most likely to be missing a distolingual cusp?
A. Mandibular first molar B. Maxillary first molar
C. Mandibular second premolar D. Maxillary second molar
31. As compared to those of the permanent mandibular first molar, the roots of the second molar are
A. More mesially inclined B. Straighter
C. More distally inclined D. Very similar in inclination to the first molar
32. The shape of the pulp chamber in a cross-section cut at the cervical line in the maxillary central incisor is
A. Oval (flattened mesiodistally) B. Oval (flattened buccolingually)
C. Round D. Figure of eight
33. The appearance of a second root in a canine is
A. Never found
B. Found more often in the maxillary than the mandibular
C. Found more often in the mandibular than the maxillary
D. Found about equally in both arches
34. The most common arrangement of canals in the roots of a permanent maxi. first molar is
A. ML, MB, DL, DB B. ML, MB, and DB only
C. ML, MB, and palatal D. MB, DB, and palatal
E. None of the above
35. From a labial view, which incisors are bilaterally symmetrical?
A. Both mandibular central and lateral B. Neither mandibular central nor lateral
C. Mandibular central only D. Mandibular lateral only

https://t.me/DentalBooksWorld
Chapter 10 • Oral Anatomy and Histology 885

36. The longest tooth in the mouth (apico-incisally) is the


A. Maxillary central B. Maxillary canine
C. Mandibular canine D. Maxillary second premolar
37. The least likely area to find caries on teeth is the
A. Lingual surface of maxillary molars
B. Occlusal surface of mandibular molars
C. Lingual surface of mandibular molars
D. Buccal surface of mandibular molars
E. Occlusal surface of mandibular premolars
38. When extracting maxillary teeth, the root tip most likely to be forced into the maxillary sinus is that of a
A. Canine B. First premolar
C. Second premolar D. First molar
E. Second molar
39. The class of teeth considered to have a long axis which is most vertical in a buccolingual direction (as viewed from the
mesial or distal) is the
A. Incisors B. Canines
C. Premolars D. Molars
40. The lingual surfaces of most maxillary teeth of a patient are worn down so that dentin is visible through the remaining
enamel on the lingual side. This is due to the dissolving of the mineral of the enamel by strong acid. This loss of tooth
structure can be termed
A. Attrition B. Abrasion
C. Erosion D. Internal resorption
41. The formation of dual teeth with combined enamel and dentin crowns but with only one root is known as
A. Gemination B. Dilaceration
C. Concrescence D. Dens-in-dente
E. Taurodont
42. Which pair of angles of a maxillary first molar are acute when the tooth is viewed from the occlusal?
A. MB and ML B. DB and DL
C. MB and DB D. ML and DL
E. None of the above
43. Which characteristic most distinguishes permanent mandibular central incisors from lateral incisors?
A. Mesiodistal length
B. Buccolingual length
C. Root length
D. Rotation of the crown around the root axis
E. None of the above
44. When primary mandibular incisors are retained too long and the permanent incisors erupt with the primaries still in
place, the permanents usually erupt in what position relative to the primaries?
A. Mesially B. Distally
C. Lingually D. Buccally
45. Which of the following is not typical of masticatory mucosa?
A. Simple squamous epithelium B. Highly keratinized
C. Poorly developed submucosa D. Thick epithelium
E. Covers the hard palate
46. The earliest succedaneous tooth in the mouth to erupt is the
A. Primary mandibular central incisor
B. Primary mandibular lateral incisor
C. Permanent mandibular first molar
D. Permanent maxillary first molar
E. Permanent mandibular central incisor

https://t.me/DentalBooksWorld
886 Triumph’s Complete Review of Dentistry

47. The time between eruption of a tooth crown and root completion is often closest to
A. 6 months B. 1 year
C. 2–3 years D. 4–5 years
E. 10 years or more
48. A mandibular permanent first molar is more likely to have a partially bifurcated
A. Mesial root B. Distal root
C. Both roots are equally likely to be bifurcated D. Neither root is bifurcated
49. The mesial and distal heights of contour of the maxillary canine are, respectively
A. Incisal third, incisal third
B. Junction of incisal/middle third, junction of incisal/middle third
C. Junction of incisal/middle third, middle third
D. Middle third, middle third
E. Middle third, cervical third
50. The most common arrangement of canals in the roots of a permanent mandibular first molar is
A. Two mesial, two distal B. Two mesial, one distal
C. One mesial, two distal D. One mesial, one distal
E. None of the above
51. The lingual fossa is normally deepest on which incisor?
A. Maxillary central B. Maxillary lateral
C. Mandibular central D. Mandibular lateral
52. Which primary teeth most differ from their permanent counterparts?
A. Incisors B. Canines
C. First molars D. Second molars
53. The premolar with the buccal cusp most distally placed relative to the lingual is the
A. Maxillary first B. Maxillary second
C. Mandibular first D. Mandibular second
54. Which tooth is least likely to have a divided pulp canal?
A. Mandibular canine B. Maxillary first premolar
C. Maxillary central incisor D. Maxillary lateral incisor
55. Synovial fluid-producing tissue in the TMJ
A. Is found throughout all surfaces B. Is not found within the joint
C. Is found only on articular surfaces D. Is found only on nonarticular surfaces
56. The primary maxillary first molar is often said to have a premolar shaped crown. This tooth will invariably have how
many roots?
A. One B. Two
C. Three D. Four
57. Mesial-occlusal-distal (MOD) cavity preparations are most difficult in which of the following teeth?
A. Permanent mandibular first premolar B. Permanent maxillary second molar
C. Primary mandibular second molar D. Primary mandibular first molar
58. The primary second maxillary molar is usually exfoliated between which ages?
A. 4–5 years B. 6–7 years
C. 8–9 years D. 10–11 years
E. 12–13 years
59. In a mandibular canine, which surface is usually most parallel to the long axis of the tooth?
A. Buccal B. Lingual
C. Mesial D. Distal
60. Which muscle of mastication controls movement of the articular disc of the TMJ?
A. Medial pterygoid B. Lateral pterygoid
C. Temporalis D. Masseter

https://t.me/DentalBooksWorld
Chapter 10 • Oral Anatomy and Histology 887

61. The inclinations of the root tips of the mesiobuccal and distobuccal roots of the maxillary first molar are, respectively
A. Mesial, mesial B. Mesial, distal
C. Distal, mesial D. Distal, distal
E. Straight, straight
62. Which premolar exhibits H, Y, and U occlusal patterns?
A. Maxillary first B. Maxillary second
C. Mandibular first D. Mandibular second
63. The primary crown most unlike any permanent tooth is the
A. Central incisor B. Lateral incisor
C. Canine D. First molar
E. Second molar
64. When compared to that of permanent teeth, the ratio of mesiodistal diameter to crown height of primary teeth is
A. Greater B. Less
C. The same D. Variable, depending on the individual tooth
65. When compared to that of permanent teeth, primary teeth color tends to be
A. Whiter B. Darker
C. Similar in shade D. Variable, depending on the individual tooth
66. Which is the narrowest anterior tooth in a mesiodistal direction?
A. Maxillary lateral B. Mandibular central
C. Mandibular lateral D. Mandibular canine
67. Which of the following angles in a maxillary incisor is sharpest?
A. Mesioincisal of central B. Mesioincisal of lateral
C. Distoincisal of central D. Distoincisal of lateral
68. As you go from mandibular first to second to third molars, mesiodistal crown length
A. Increases, then decreases B. Increases
C. Decreases D. Decreases, then increases
69. Which of the following permanent teeth is most likely to be missing a distolingual cusp?
A. Mandibular first molar B. Maxillary first molar
C. Maxillary third molar D. Maxillary second molar
70. The protein matrix of enamel is
A. Composed of collagen
B. Composed of keratin
C. Composed of protein which is not collagen or keratin
D. Absent, there is no protein matrix
71. Cervical bulges are found on which surfaces of primary anterior teeth?
A. Mesial and distal B. Buccal and lingual
C. Lingual only D. Buccal only
72. In the permanent mandibular third molar, when viewed from the occlusal
A. The mesial end is larger than the distal B. The distal end is larger than the mesial
C. The mesial and distal ends are the same size D. It is impossible to generalize about mandibular third molars
73. When viewed from the mesial, the tooth closest to vertical in the buccolingual angulation of its main axis is the
A. Maxillary central incisor B. Maxillary lateral incisor
C. Maxillary canine D. Maxillary first premolar
74. Tooth #23 replaces which primary tooth?
A. K B. L
C. M D. N
E. O
75. In the primary mandibular first molar, the most noticeable cusps are
A. MB and DB B. MB and ML
C. DB and DL D. MB and DL

https://t.me/DentalBooksWorld
888 Triumph’s Complete Review of Dentistry

76. Which position is achieved entirely by soft tissue, without guidance from teeth?
A. Postural (physiologic rest) B. Centric occlusion
C. Centric relation D. Protruded contact
77. The faciolingual dimension of a mandibular first molar, when compared to the mesiodistal dimension, is
A. Smaller B. Larger
C. The same D. Larger or smaller, depending on the tooth
78. The premolar which is most often double rooted is the
A. Maxillary first B. Maxillary second
C. Mandibular first D. Mandibular second
79. In centric occlusion, the mesiolingual cusp of the maxillary first molar will contact
A. The mesial marginal ridge of the mandibular first premolar and the distal marginal ridge of the second premolar
B. The distal marginal ridge of the mandibular first molar and the distal marginal ridge of the second molar
C. The distal marginal ridge of the mandibular second premolar and the mesial marginal ridge of the first molar
D. The central fossa of the mandibular first molar
80. The crowns of the incisors when viewed from the incisal
A. Are wider mesiodistally than buccolingually in both arches
B. Are wider buccolingually than mesiodistally in both arches
C. Are wider mesiodistally in the maxilla, and wider buccolingually in the mandible
D. Are wider mesiodistally in the mandible and wider mesiodistally in the maxilla
81. The largest root of the maxillary first molar is the
A. Mesiobuccal B. Distobuccal
C. Palatal D. Varies depending on the individual tooth
82. On which premolar can you see more of the occlusal table from the mesial than from the distal?
A. Maxillary first B. Maxillary second
C. Mandibular first D. Mandibular second
83. At the cervical line, a cross-section of the mandibular canine would show
A. A round shape
B. An oval shape wider on the lingual than the labial
C. A triangular shape with a distinctly wider labial surface
D. An oval shape with a slightly wider labial surface
84. In a mandibular first molar, which pulp horn is likely to be smallest?
A. Mesiobuccal B. Mesiolingual
C. Distobuccal D. Distolingual
E. Distal
85. The premolar most likely to have a three-cusped appearance when viewed from the occlusal is the
A. Maxillary first B. Maxillary second
C. Mandibular first D. Mandibular second
86. At age 6 1/2, an average child is most likely to have how many erupted permanent teeth? (Assume that you average the
teeth present in a sample of 100 children)
A. 4–8 B. 9–12
C. 13–16 D. 17–20
E. 21–24
87. The major function of contact points in the dentition is to
A. Protect the incisal surface
B. Protect the periodontium
C. Protect the alveolar mucosa
D. Protect restorations from poor retention
E. None of the above
88. The maxillary canine has
A. A mesial marginal ridge only B. A distal marginal ridge only
C. Neither a mesial nor distal marginal ridge D. Both a mesial and distal marginal ridge

https://t.me/DentalBooksWorld
Chapter 10 • Oral Anatomy and Histology 889

TMJ AND PDL

1. Which structure is involved in pulling the disc of the TMJ forward?


A. Stylomandibular ligament B. Medial pterygoid muscle
C. Mandibular condyle D. TMJ meniscus
E. Lateral pterygoid muscle
2. Which ligament of the temporomandibular joint originates from a thin, pointed extension of the temporal bone?
A. Temporomandibular B. Stylomandibular
C. Sphenomandibular D. Lateral
3. Which of the following is the most coronally located periodontal fiber group?
A. Alveolar crestal B. Apical
C. Horizontal D. Oblique
4. The major blood supply of the PDL is from
A. Pulpal vessels B. Gingival vessels
C. Periosteal vessels D. Periapical vessels
5. Which of the following is NOT a periodontal fiber?
A. Oblique B. Transseptal
C. Apical D. Horizontal
6. Which of the following fiber sets does NOT attach to the cementum?
A. Alveolar crestal B. Dentogingival
C. Circular D. Oblique
7. Which ligament of the temporomandibular joint inserts into the lingula of the mandible?
A. Temporomandibular B. Stylomandibular
C. Sphenomandibular D. Lateral
8. In cases of TMJ dislodgement, where the condyle becomes locked due to movement beyond the articular (temporal)
fossa, the dentist should move the condyle in which direction initially to return the joint to its normal position?
A. Anteriorly B. Posteriorly
C. Inferiorly D. Superiorly
9. The TMJ structure consists of
A. One synovial compartment B. An upper synovial and lower nonsynovial compartment
C. Two synovial compartments D. Two nonsynovial compartments
10. When assigning types of mandibular motion to the compartments of the TMJ, it is generally agreed that which
compartment is associated with which motion type?
A. Anterior: rotation, posterior: translation B. Anterior: translation, posterior: rotation
C. Superior: rotation, inferior: translation D. Superior: translation, inferior: rotation
11. Functions attributed to the ligaments of the temporomandibular joint include all of the following except
A. Increasing stability of the joint
B. Aiding in forward movement of the condyle
C. Increasing the strength of the joint
D. Limiting extreme motion of the joint
E. All of the above are ligament functions
12. Which cell type is NOT found within the periodontal membrane?
A. Cementoblasts B. Osteoblasts
C. Odontoblasts D. Osteoclasts

ORAL MUCOUS MEMBRANE


1. Gingival “Col” is lined by
A. Orthokeratinized stratified squamous epithelium B. Thin nonkeratinized epithelium
C. Parakeratinized stratified squamous epithelium D. Thick nonkeratinized epithelium
2. False statement about melanocytes is
A. Melanocytes arise embryologically from the neural crest epithelium
B. Melanocytes lack desmosomes and tonofilaments
https://t.me/DentalBooksWorld
890 Triumph’s Complete Review of Dentistry

C. Melanocytes lack dendritic processes


D. Melanocytes synthesized within the melanocytes
3. Which of the following are stratification-specific cytokeratins?
A. K6 and K16 B. K5 and K14
C. K1 and K2 D. K4 and K13
4. Which of the following parts of the hard palate is devoid of submucosa?
A. Anterolateral B. Posterolateral
C. Raphe D. Incisive papilla
5. Lamina densa of basement membrane is made up of
A. Type I collagen B. Type II collagen
C. Type III collagen D. Type IV collagen
6. Merkel cells is found in which of the following tissue?
A. Epidermis B. Papillary layer of dermis
C. Reticular layer of dermis D. Hypodermis
7. Which of the following does not show taste buds?
A. Filiform papillae B. Fungiform papillae
C. Circumvallate papillae D. Soft palate
8. Taste buds are absent in which of the following papillae of the tongue?
A. Circumvallate B. Filiform
C. Foliate D. Fungal form
9. Collagen seen in basement membrane is
A. Type IV B. Type VI
C. Type VIII D. Type IX
10. The tissue macrophages are derived from the
A. Neutrophil B. Eosinophil
C. Monocyte D. All of the above
11. Junctional epithelium is
A. Keratinized B. Nonkeratinized
C. Hyperkeratinized D. Parakeratinized
12. Plasma cells are mostly seen in one of the stages of gingivitis
A. Initial lesion B. Early lesion
C. Established lesion D. Advanced lesion
13. Biological width means
A. Sum of the connective tissue and junctional epithelium
B. Distance from the tip of papilla to the base of the sulcus
C. Distance from the marginal gingiva to the muco-gingival junction
D. Distance from the incisal edge of the tooth to the muco-gingival junction
14. Melanocytes are present in
A. Stratum spinosum B. Stratum granulosum
C. Stratum intermedium D. Stratum basale
15. The epithelium of the mucous membrane of the mouth is
A. Simple squamous B. Stratified squamous
C. Simple columnar D. Stratified columnar
E. Cuboidal
16. The submucosa of gingival masticatory mucosa is
A. Similar in thickness to that of lining mucosa
B. Much thicker than that of lining mucosa
C. Found between the epithelium and the lamina propria
D. Thin or absent

https://t.me/DentalBooksWorld
Chapter 10 • Oral Anatomy and Histology 891

17. The mucosa found on the surface of the hard palate is known as
A. Lining mucosa B. Masticatory mucosa
C. Specialized mucosa D. None of the above

OCCLUSION
1. Which cusp becomes smaller and less conspicuous as you go from maxillary first to second to third molar?
A. Mesiobuccal B. Mesiolingual
C. Distobuccal D. Distolingual
2. Which of the following is not considered an important reason for restoring proximal contact in dental restorations?
A. Contact protects the periodontal tissue B. Contact prevents food impaction
C. Contact increases retention of the restorations D. Contact prevents tooth drifting
3. In centric occlusion, the mesiolingual cusp of the maxillary third molar will contact
A. The central fossa of the mandibular second molar
B. The central fossa of the mandibular third molar
C. The distal marginal ridge of mandibular first molar and mesial marginal ridge of the second molar
D. The distal marginal ridge of mandibular second molar and mesial marginal ridge of the third molar
4. The attrition at the contact point between teeth is known to cause
A. Reduced embrasures interproximally B. Diastemas
C. Longer clinical crowns D. Shorter clinical crowns
5. The glenoid (articular) fossa in which the mandibular condyle articulates is a depression within which cranial bone?
A. Sphenoid B. Zygomatic
C. Temporal D. Parietal
E. A combination of more than one bone
6. What traction is used for extraction of maxillary first premolars?
A. Buccal–lingual luxation B. Mesiodistal luxation
C. Rotation D. All of the above
7. How many teeth (out of 32) in the normal dentition oppose only one other tooth?
A. None B. 1
C. 2 D. 4
E. 6
8. The masseteric sling is composed of the masseter and the
A. Medial pterygoid B. Lateral pterygoid
C. Anterior digastric D. Temporalis
9. The mesiobuccal cusp of the mandibular second molar occludes with which maxillary tooth surfaces?
A. The mesial marginal ridge of the second molar and the distal marginal ridge of the first molar
B. The distal marginal ridge of the second molar and the mesial marginal ridge of the third molar
C. The embrasure between the first and the second molars
D. The central fossa of the maxillary second molar
10. The distobuccal cusp of the mandibular first molar occludes with which maxillary tooth surfaces?
A. The mesial marginal ridge of the second molar and distal marginal ridge of the first molar
B. The central fossa of the first molar
C. The mesial marginal ridge of the first molar and the distal marginal ridge of the second premolar
D. The embrasure between the first and second molars
11. An example of a guiding (nonsupporting) cusp is
A. Distolingual of #30 B. Mesiobuccal of #18
C. Distolingual of #14 D. Lingual of #5
E. Palatal of #13
12. In centric occlusion, the distolingual cusp of the maxillary first molar will contact
A. Mesial marginal ridge of the mandibular first premolar and the distal marginal ridge of the second premolar
B. The distal marginal ridge of the mandibular second premolar and the mesial marginal ridge of the first molar

https://t.me/DentalBooksWorld
892 Triumph’s Complete Review of Dentistry

C. The distal marginal ridge of the mandibular first molar and the mesial marginal ridge of the second molar
D. The distal marginal ridge of the mandibular second molar and the mesial marginal ridge of the first molar
13. The mesiolingual cusp of the mandibular second molar contacts
A. The lingual embrasure between the maxillary first molar and the second molar
B. The distal marginal ridge of the maxillary second molar
C. The central fossa of the maxillary second molar
D. The lingual groove of the maxillary second molar
E. The lingual embrasure between the maxillary second molar and the third molar
14. In the maxillary arch, the narrowest incisal or occlusal embrasure is located
A. Between the first premolar and the canine B. Between the canine and the lateral incisor
C. Between the lateral incisor and the central incisor D. Between the central incisors
15. The inner enamel cuticle, found on the surface of enamel of an erupting tooth crown, is
A. Also known as Nasmyth’s membrane B. Also known as the stratum intermedium
C. Cellular D. Formed by odontoblasts
16. The heights of contour of the mandibular first premolar, when viewed from the mesial, are located within which third?
A. Buccal in occlusal third, lingual in middle third
B. Both in middle third
C. Both in occlusal third
D. Buccal in cervical third, lingual in occlusal third
E. Both in cervical third
17. Which of the following oral muscles is not innervated by CN V?
A. Masseter B. Buccinator
C. Medial pterygoid D. Lateral pterygoid
E. Mylohyoid
18. The contact of the permanent mandibular incisor with the canine occurs at which level of the lateral?
A. Incisal third B. Junction of incisal and middle thirds
C. Middle third D. Gingival third
19. Guiding cusps normally contact
A. Mesial marginal ridges B. Distal marginal ridges
C. Central fossae D. Embrasures
20. Which premolar, when viewed from the facial, has a longer mesial cusp ridge than distal cusp ridge?
A. Maxillary first B. Maxillary second
C. Mandibular first D. Mandibular second
21. In centric occlusion, the distolingual cusp of the maxillary second molar will contact
A. The central fossa of the mandibular second molar
B. The distal marginal ridge of the mandibular first molar and the mesial marginal ridge of the second molar
C. The distal marginal ridge of the mandibular second molar and the mesial marginal ridge of the first molar
D. The distal marginal ridge of the mandibular second molar and the mesial marginal ridge of the third molar
22. Of the following premolars, the one which can never be rotated during extraction is the
A. Maxillary first B. Maxillary second
C. Mandibular first D. Mandibular second
23. In centric occlusion, the lingual cusp of the maxillary second premolar contacts
A. The mesial marginal ridge of the mandibular first premolar and the distal marginal ridge of the second premolar
B. The distal marginal ridge of the mandibular first premolar and the mesial marginal ridge of the second premolar
C. The mesial marginal ridge of the mandibular first molar and the distal marginal ridge of the second premolar
D. The distal marginal ridge of the mandibular second premolar and the mesial marginal ridge of the first molar
24. The distobuccal cusp of the mandibular second molar occludes with which maxillary tooth surfaces?
A. The mesial marginal ridge of the second molar and the distal marginal ridge of the first molar
B. The distal marginal ridge of the second molar and the mesial marginal ridge of the third molar
C. The embrasure between the first and second molars
D. The central fossa of the maxillary second molar

https://t.me/DentalBooksWorld
Chapter 10 • Oral Anatomy and Histology 893

25. A patient presents for examination with both mandibular first molars distal to the maxillary first molars. This
occlusion is known as
A. Class I, distocclusion B. Class II, mesiocclusion
C. Class II, distocclusion D. Class III, mesiocclusion
E. Class III, distocclusion
26. The embrasures which surround the contact points of anterior teeth are
A. Mesial, distal, lingual, buccal
B. Mesial and distal only
C. Buccal and lingual only
D. Buccal, lingual, cervical, gingival
E. Incisal, cervical, buccal, lingual
27. The distolingual cusp of the mandibular second molar contacts
A. The lingual embrasure between the maxillary first molar and the second molar
B. The lingual groove of the maxillary second molar
C. The central fossa of the maxillary second molar
D. The lingual groove of the maxillary first molar
E. The lingual embrasure between the maxillary second molar and the third molar
28. The distobuccal cusp of the maxillary second molar occludes in
A. The mesial marginal ridge of the mandibular second molar
B. The buccal groove of the mandibular second molar
C. The central groove of the mandibular third molar
D. The facial embrasure between the mandibular first and second molars
E. The facial embrasure between the mandibular second and third molars
29. The mandibular incisors of a patient are worn down so that dentin is visible on the incisal edge. This is due to grinding
of these edges against the natural maxillary incisors. This loss of tooth structure can be termed
A. Attrition B. Abrasion
C. Erosion D. Internal resorption
30. In centric occlusion, the lingual cusp of the maxillary first premolar contacts
A. The mesial marginal ridge of the mandibular first premolar and the distal marginal ridge of the second premolar
B. The distal marginal ridge of the mandibular first premolar and the mesial marginal ridge of the second premolar
C. The mesial marginal ridge of the mandibular first molar and the distal marginal ridge of the second premolar
D. The distal marginal ridge of the mandibular second premolar and the mesial marginal ridge of the first molar
31. The distobuccal cusp of the mandibular third molar occludes with which maxillary tooth surfaces?
A. The mesial marginal ridge of the third molar and the distal marginal ridge of the second molar
B. The central fossa of the third molar
C. The mesial marginal ridge of the third molar only
D. The distal marginal ridge of the third molar only
32. Damage to the right lateral pterygoid results in
A. The mandible turning right on protrusion B. Inability to elevate the mandible
C. The mandible turning left on protrusion D. None of the above
33. The distal cusp of the mandibular first molar occludes with which maxillary tooth surfaces?
A. The mesial marginal ridge of the first molar and the distal marginal ridge of the second molar
B. The distal marginal ridge of the second molar and the mesial marginal ridge of the first molar
C. The embrasure between the first and second molars
D. The distal triangular fossa of the first molar
34. The mesiolingual cusp of the mandibular third molar contacts
A. The lingual embrasure between the maxillary second molar and the third molar
B. The distal marginal ridge of the maxillary second molar
C. The central fossa of the maxillary second molar
D. The lingual groove of the maxillary third molar
E. The buccal embrasure between the maxillary second molar and the third molar

https://t.me/DentalBooksWorld
894 Triumph’s Complete Review of Dentistry

35. The buccal cusp of the maxillary second premolar occludes in


A. The mesial marginal ridge of the mandibular first premolar
B. The distal marginal ridge of the mandibular second premolar
C. The mesial marginal ridge of the mandibular second premolar
D. The facial embrasure between the mandibular first and second premolars
E. The facial embrasure between the mandibular second premolar and the first molar
36. In occlusion, guiding cusps are defined as
A. Lingual cusps of maxillary and mandibular arches
B. Buccal cusps of maxillary and mandibular arches
C. Buccal cusps of the maxillary arch and lingual cusps of the mandibular arch
D. Lingual cusps of the maxillary arch and buccal cusps of the mandibular arch
37. Protrusive movement causes the condyle of the mandible to move
A. Backward and downward B. Backward and upward
C. Forward and downward D. Forward and upward
38. The mesiobuccal cusp of the maxillary second molar occludes in
A. The buccal groove of the second mandibular molar
B. The facial embrasure between the mandibular first and second molars
C. The central groove of the mandibular second molar
D. The distal marginal ridge of the mandibular first molar
E. The buccal groove of the mandibular third molar
39. In centric occlusion, the distolingual cusp of the maxillary third molar will contact
A. The central fossa of the mandibular second molar
B. The central fossa of the mandibular third molar
C. The distal marginal ridge of the mandibular first molar and the mesial marginal ridge of the second molar
D. The distal marginal ridge of the mandibular second molar and the mesial marginal ridge of the first molar
E. None of the above
40. When you move the mandible from centric occlusion (maximum intercuspation) to protruded contact (edge-to-edge)
position
A. Vertical dimension decreases B. Horizontal overlap increases
C. Vertical overlap increases D. None of the above
41. The buccal cusp of the mandibular first premolar contacts which surfaces on maxillary teeth?
A. The mesial marginal ridge of the first premolar and the distal marginal ridge of the canine
B. The mesial marginal ridge of the second premolar and the distal marginal ridge of the first premolar
C. The distal marginal ridge of the first premolar and the mesial marginal ridge of the canine
D. None of the above
42. The buccal cusp of the mandibular canine occludes with which maxillary tooth surfaces?
A. The mesial marginal ridge of the first premolar and the distal marginal ridge of the canine
B. The mesial marginal ridge of the canine and the distal marginal ridge of the lateral incisor
C. The embrasure between the canine and the first premolar
D. None of the above
43. The lamina propria is
A. Epithelial tissue B. Connective tissue
C. Adipose tissue D. Muscle tissue
44. Which of the following is not normally found in cases of occlusal trauma?
A. Resorption of alveolar bone B. Increased mobility
C. Thicker cementum (hypercementosis) D. Widened periodontal ligament space
45. The buccal cusp of the maxillary first premolar occludes in
A. The mesial marginal ridge of the mandibular first premolar
B. The distal marginal ridge of the mandibular second premolar
C. The mesial marginal ridge of the mandibular second premolar
D. The facial embrasure between the mandibular first and second premolars
E. The facial embrasure between the mandibular first premolar and canine

https://t.me/DentalBooksWorld
Chapter 10 • Oral Anatomy and Histology 895

46. In restoring a mandibular first molar, lingual cusps are important for which movement?
A. Centric occlusion B. Protrusive
C. Retrusive D. Working
E. Nonworking
47. The distal contact point and height of contour of the maxillary lateral incisor is located
A. At the incisal edge B. In the incisal third
C. In the middle third D. In the cervical third
48. The distobuccal cusp of the maxillary first molar occludes in
A. The mesial marginal ridge of the mandibular second molar
B. The distal marginal ridge of the mandibular first molar
C. The central groove of the mandibular first molar
D. The facial embrasure between the mandibular first and second molars
E. The distobuccal groove of the first mandibular molar
49. Which teeth exhibit isomorphy with each other?
A. Primary maxillary first and second molars
B. Primary maxillary second and primary mandibular second molars
C. Primary mandibular first molars and primary maxillary first molars
D. Primary maxillary second molars and permanent maxillary first molars
E. None of the above
50. The mesiolingual cusp of the mandibular first molar contacts
A. The lingual embrasure between the maxillary first molar and the second premolar
B. The distal marginal ridge of the maxillary second molar
C. The central fossa of the maxillary first molar
D. The lingual groove of the maxillary first molar
E. The lingual embrasure between the maxillary first molar and the second molar
51. In protrusive movements, the lingual surface of the maxillary lateral incisor will contact which mandibular teeth?
A. Central incisor only B. Central and lateral
C. Lateral incisor only D. Lateral and canine
E. Canine only
52. The Carabelli cusp (trait) is found attached to which cusp?
A. Mesiobuccal B. Mesiolingual
C. Distobuccal D. Distolingual
53. If vertical dimension of occlusion is VDO, vertical dimension of rest is VDR, and freeway space is FS, then
A. VDO + VDR = FS B. VDO + FS = VDR
C. VDR + FS = VDO D. None of the above
54. Which muscle of mastication both strongly elevates and strongly retrudes the mandible?
A. Masseter B. Medial pterygoid
C. Lateral pterygoid D. Temporalis
E. Anterior digastric
55. In centric occlusion, the mesiolingual cusp of the maxillary second molar will contact
A. The central fossa of the mandibular second molar
B. The central fossa of the mandibular first molar
C. The distal marginal ridge of the mandibular first molar and the mesial marginal ridge of the second molar
D. The distal marginal ridge of the mandibular second premolar and the mesial marginal ridge of the first molar
56. The mesial height of contour of the mandibular central incisor is
A. Near the incisal edge B. At the junction of the incisal and middle thirds
C. In the middle third D. At the junction of the middle and cervical thirds
57. Which permanent mandibular molar is most likely to be marked by supplemental occlusal grooves, crenulations, and
unpredictable placement of pits?
A. First molar B. Second molar
C. Third molar D. All molars about equally

https://t.me/DentalBooksWorld
896 Triumph’s Complete Review of Dentistry

58. In a left lateral (working movement)


A. The left mandibular teeth are the working side and move to the left
B. The left mandibular teeth are the working side and move to the right
C. The left mandibular teeth are the balancing side and move to the left
D. The left mandibular teeth are the balancing side and move to the right
59. The mesiobuccal cusp of the mandibular third molar occludes with which maxillary tooth surfaces?
A. The mesial marginal ridge of the third molar and the distal marginal ridge of the second molar
B. The distal marginal ridge of the third molar and the mesial marginal ridge of the second molar
C. The mesial marginal ridge of the third molar only
D. The central fossa of the maxillary third molar
60. The central fossa of a maxillary first molar is not bordered by which cusp?
A. Mesiobuccal B. Mesiolingual
C. Distobuccal D. Distolingual
61. In a left working movement
A. The left side moves laterally, and the right side medially
B. The left side moves medially, and the right side laterally
C. Both sides move medially
D. Both sides move laterally
62. In performing a left working movement
A. Only the left lateral pterygoid contracts B. Only the right lateral pterygoid contracts
C. Both lateral pterygoids contract D. Neither lateral pterygoid contracts
63. The distolingual cusp of the mandibular first molar contacts
A. The lingual embrasure between the maxillary first molar and the second premolar
B. The lingual groove of the maxillary second molar
C. The central fossa of the maxillary first molar
D. The lingual groove of the maxillary first molar
E. The lingual embrasure between the maxillary first molar and the second molar
64. Which of the premolars is most likely to have a pronounced mesial marginal groove extending into a pronounced
mesial root concavity?
A. Maxillary first B. Maxillary second
C. Mandibular first D. Mandibular second
65. The mesiobuccal cusp of the mandibular first molar occludes with which maxillary tooth surfaces?
A. The mesial marginal ridge of the first molar and the distal marginal ridge of the second premolar
B. The distal marginal ridge of the first molar and the mesial marginal ridge of the second molar
C. The embrasure between the first and second molars
D. The distal triangular fossa of the first molar
66. In the Posselt’s envelope of motion, protruded contact position (PCP) is
A. Achieved with the posterior teeth in occlusion B. An extreme border position
C. The same as edge-to-edge D. The same as an Angle Class II relationship
67. The buccal cusp of the mandibular second premolar occludes with which maxillary tooth surfaces?
A. The mesial marginal ridge of the first molar and the distal marginal ridge of the second premolar
B. The mesial marginal ridge of the second premolar and the distal marginal ridge of the first premolar
C. The embrasure between the first and second molars
D. The embrasure between the second premolar and the first molar
68. The lingual cusp of the mandibular first premolar contacts
A. The mesial marginal ridge of the maxillary first premolar
B. The distal marginal ridge of the maxillary second premolar
C. The central fossa of the maxillary second premolar
D. The lingual embrasure between the maxillary first and second premolars
E. The lingual embrasure between the maxillary first premolar and canine

https://t.me/DentalBooksWorld
Chapter 10 • Oral Anatomy and Histology 897

GENES IN TOOTH DEVELOPMENT


1. Genes that regulate oral–aboral axis are
2. The position of the tooth germ in the established oral–aboral axis is determined by the expression of
A. FGF-8 B. PITX-2
C. BMP-4 (in oral epithelium) D. PAX-9 (in tooth mesenchyme)
3. Gene expression seen in dental ectoderm is _____________
4. Tooth type determination is regulated through which subfamily of homeobox genes __________
5. Tooth type determination is regulated through which subfamily of homeobox genes in the posterior region ________
6. Tooth type determination is regulated through which subfamily of homeobox genes in the anterior region __________
7. Gene responsible for ectopic tooth formation is __________
8. Abnormal initiation is due to __________
9. Abnormalities in morphodifferentiation result in __________
10. Abnormalities in apposition result in __________

SHORT-ANSWER QUESTIONS
1. All teeth develop from __________ lobes except permanent first molars and mandibular 2nd premolar
2. Anterior tooth most likely to have a bifurcated root __________
3. Best developed lingual anatomy __________
4. The CEJ dips deeper on anterior than posterior __________
5. Contact points for mandibular teeth __________
6. Contact points for maxillary teeth __________
7. Crowns of teeth tend to get shorter from __________ to __________
8. The distal side of a tooth is typically bigger than the mesial side (True/False)
9. Enamel is the hardest substance in the human body. It is about __________ calcified
10. Facial embrasures are narrower than lingual embrasures except for __________
11. Facial heights of contour are least prominent on __________
12. Facial heights of contour are most prominent on __________
13. From a facial view, all teeth have a crown shaped like a __________
14. From a proximal view, what is the shape of anterior and posterior teeth?
15. Greatest constriction from pulp chamber to canals __________
16. Height of contour for teeth (facial and lingual) __________
17. If one root has 2 canals, they will be oriented __________ and __________
18. Largest incisal embrasure is between __________ and __________
19. Largest overall occlusal embrasure is between __________ and __________
20. Longest root incisogingivally __________
21. Mandibular canines have incisal edges that are __________
22. Mandibular incisors have incisal edges that are __________
23. Maxillary canines have incisal edges that are __________
24. Maxillary incisors have incisal edges that are __________
25. Maxillary teeth usually erupt before mandibular teeth (True/False)
26. Most congenitally malformed or missing __________

https://t.me/DentalBooksWorld
898 Triumph’s Complete Review of Dentistry

27. Most likely to have dens in dente __________


28. Most symmetrical __________
29. Narrowest and smallest rooted pre-molar __________
30. Narrowest M-D diameter __________
31. Only tooth that is narrower facially than lingually ___________
32. Only tooth with 2 triangular ridges on 1 cusp ___________
33. Only tooth with an M-L groove ___________
34. Only tooth with a pulp wider M-D than F-L ___________
35. Only tooth with a root that is inclined mesially ___________
36. Only tooth with crown concavities ___________
37. Permanent anterior shows the first evidence of calcification before age __________
38. Permanent Pms show the first evidence of calcification around age __________
39. Premolar most likely to be congenitally missing ___________
40. Premolar most likely to have 1 root and 1 canal ___________
41. Primary teeth show evidence of calcification in utero in __________ trimester
42. The roots of all teeth are inclined __________, except for __________
43. The roots of primary teeth will complete about ___ years after eruption; the roots of permanent teeth will complete
about __________ years after eruption
44. Second largest incisal embrasure is between ___________
45. Smallest tooth ___________
46. Tallest crown incisogingivally ___________
47. Tallest tooth incisogingivally ___________
48. Teeth erupt with as many pulp horns as they have cusps (True/False)
49. When a tooth erupts, its root is about __________% formed
50. Which tooth shows the first evidence of calcification at birth?
51. Widest F-L Permanent
52. Widest mesiodistally
53. Wisdom teeth show the first evidence of calcification around age __________

ANSWERS

TOOTH NUMBERING SYSTEM


1. Parreidt, 1882; Cunningham, 1883
2. I2/2 C1/1 P2/2 M3/3
3. I2/2 C1/1 M2/2
4. Akram et al., 2011
5. Viohl in 1966
6. The Hillischer system (Hillischer, 1885) of notation
7. The MICAP Notation by Akram et al.
8. Victor Haderup system by Haderup in 1891
9. The Woelfel system
10. Havel’s Alphanumeric Dental Notation
11. Mons Dubois system
12. Universal tooth numbering system

https://t.me/DentalBooksWorld
Chapter 10 • Oral Anatomy and Histology 899

13. FDI World Dental Federation notation


14. Answer: D
Universal System For Deciduous Teeth
A – Maxillary right second molar
J – Maxillary left second molar
K – Mandibular left second molar
T – Mandibular right second molar
15. Zsigmondy and Palmer tooth numbering system
16. Answer: B (Ref. Wheeler’s Dental Anatomy, Physiology and Occlusion, By Stanley Nelson, 2014, page no. 4)
Advantages
• Separate number/alphabet is given for individual tooth
• Easy to visualize
Disadvantages
• Confusing when comparing with palmar notation system
• Cannot be coded by computer
• Confusing and difficult to remember
• It does not consider the jaw quadrant clearly, thus similar teeth are given various numerous in different quadrant
17. Answer: A (Ref. Wheeler’s Dental Anatomy, Physiology and Occlusion, By Stanley Nelson, 2014, page no. 4)
Advantages
• System is simple to use
• Easier for beginner due to less confusion as permanent teeth and deciduous teeth are indicated differently
Disadvantages
• There is no differentiation between right upper, right lower, left upper, and left lower
• Segments has only one number and used to designate particular tooth
• No provision to identify supernumerary tooth
• Difficult for verbal transmission
18. Answer: B
• In this system, the teeth that should be there are numbered
• If the wisdom teeth is missing, then first number will be 2 instead of 1, acknowledging the missing tooth
• If the teeth had removed or teeth are missing, the missing teeth will be numbered as well
19. Answer: A (Ref. Wheeler’s Dental Anatomy, Physiology and Occlusion, By Stanley Nelson, 2014, page no. 5)
Zsigmondy and Palmar Notation
• In 1947 a committee at the American Dental Association (ADA) recommended the symbolic(Zsigmondy/Palmar) system
as the numbering method of choice
• Originally called the Zsigmondy system by an Austrian dentist Adolf Zsigmondy who developed the idea in 1861
• The Zsigmondy–Palmer system (Zsigmondy, 1861; Palmer, 1891),called “eight numerical quadrant system” (1 through 8),
is meant for permanent dentition only
Initially it was not designed for primary teeth, but in 1874 it was adopted for the primary dentition.

ENAMEL, DENTIN, PULP


1. Answer: C (Ref. Orban’s Oral Histology and Embryology, By GS Kumar, page no. 49)
Enamel is translucent and varies in color from light yellow to whitish. It varies in thickness, with maximum over cusps
(2.5 mm) to a feather edge at the cervical line. Thickness of enamel in primary teeth is nearly half than that in the permanent
teeth.
Although enamel is an extremely hard tissue it is partially permeable to some fluids, bacteria, and other products of the
oral cavity The permeability of enamel is due to the presence of cracks and microscopic spaces on the surface of the enamel
which allows penetration of fluids. The permeability of enamel decreases and hardness increases with age.
2. Answer: A (Ref. Orban’s Oral Histology and Embryology, By GS Kumar, page no. 163)
A plexus of myelinated nerve fibers located between the core of the pulp of the tooth and the cell-rich zone; axons of
Raschkow plexus lose their myelin sheath (but not their Schwann cells) as they penetrate the cell-rich and cell-free zones
to make synaptic contact with the odontoblast cell body in the pulp or odontoblastic process within the dentinal tubule;
Raschkow plexus is responsible for transmitting pain sensation from the pulp of the tooth

https://t.me/DentalBooksWorld
900 Triumph’s Complete Review of Dentistry

Predenn

Odontoblasts

Odontogenic
zone Cell-free zone

Parietal layer
Plexus of nerves

3. Answer: B (Ref. Orban’s Oral Histology and Embryology, By GS Kumar, 2014, page no. 419)

4. Answer: B (Ref. Orban’s Oral Histology and Embryology, By GS Kumar, 2014, page no. 57)

5. Answer: A (Ref. Orban’s Oral Histology and Embryology, By GS Kumar, 2014, page no. 68)
Types of Acid Etching
According to Silverstone et al. there are five types of acid etching:
Type 1: Preferential dissolution of prism cores, resulting in a honey comb-like appearance
Type II: Preferential dissolution of prism peripheries, giving a cobblestone-like appearance
Type III: A mixture of type I and II pattern

https://t.me/DentalBooksWorld
Chapter 10 • Oral Anatomy and Histology 901

Type VI: Pitted enamel surfaces as well as structures that look like unfinished puzzle, maps, or networks
Type V: Flat, smooth surface
6. Answer: C (Ref. Orban’s Oral Histology and Embryology, By GS Kumar, page no. 104)
− Mantle Dentin
§ Peripheral or first layer of dentin adjacent to enamel or cementum
§ Consists of more coarse fibers (Korff ’s) – Think Dorff on Golf
− Peritubular Dentin (Intratubular Dentin)
§ Lines each dentinal tubule
§ More mineralized than intertubular dentin
− Intertubular Dentin
§ This is the main bulk of dentin
§ Surrounds peritubular dentin
§ Less mineralized
− Interglobular Dentin
§ Imperfectly calcified matrix of dentin situated between the calcified globules near the periphery of the dentin
− Dead tracts are tubules with dead cytoplasmic fibers in them
• Primary dentin
− Layed down before apical foramen closure
− Dentin laid down before birth
• Secondary dentin
− Formed after foramen closure
− Slower forming than primary, as functional stresses are placed on tooth
− Following the initial period of functional activity, an appreciable alteration in the size of the pulp chamber is a
direct result of deposition of secondary dentin
− Regular and uniform layer
§ **There is a sharp change in the direction of tubules at junction of primary and secondary dentins
− The tubules of secondary dentin are wavy
• Reparative dentin
− What indicates Trauma during Dentin Formation????
− Forms in response to stimuli produced by carious penetration of a tooth
− Formed very rapidly in response to irritants like attrition, abrasion, erosion, caries, etc.
− The tubules of reparative dentin are twisted
• Sclerotic dentin
− From aging and slowly advancing dental caries
− Tubules become obliterated, which blocks access of irritants
• All the following are seen in dentin except
− Tomes granular layer, Odontoblastic processes, Striae of Retzius, and contour lines of Owen
7. Answer: A (Ref. Orban’s Oral Histology and Embryology, By GS Kumar, 2014, page no. 219)
8. Answer: A (Ref. Orban’s Oral Histology and Embryology, By GS Kumar, 2014, page no. 106)
Theories of dentin hypersensitivity
1. Direct innervation theory
2. Odontoblast deformation theory/transducer theory
3. Hydrodynamic theory
• Direct innervation theory
• First theory to be put forward
• Nerve fibers present within dentinal tubules initiate impulses when they are injured and causes dentinal hypersensitivity
• Direct innervation theory disputes about this theory
• Nerve fibers are present only in the predentin and inner dentinal zones
• When pain inducing substances like potassium chloride and acetylcholine are applied to exposed dentin, they fail to elicit
a painful response
• Odontoblast deformation theory

https://t.me/DentalBooksWorld
902 Triumph’s Complete Review of Dentistry

• Odontoblast or their processes are damaged when external stimuli are applied to exposed dentin
• They conduct impulses to the nerves in the predentin and underlying pulp and then to CNS
• Disfavored as the odontoblastic processes extend only partly through the dentin and not up to DEJ
• Odontoblastic membrane potential is too low to permit transduction
• There are no demonstrable neurotransmitters in the neural transmission of the pulp
• Hydrodynamic theory is the most widely accepted mechanism of action of dentin hypersensitivity, the hydrodynamic
theory which was proposed by Gysi in 1900 and validated by Brannstrom in 1996
9. Answer: B (Ref. Orban’s Oral Histology and Embryology, By GS Kumar, 2014, page no. 53)
An enamel rod is the basic unit of tooth enamel. Measuring 4 μm wide to 8 μm high, an enamel rod is a tightly packed, highly
organized mass of hydroxyapatite crystals, which are hexagonal in shape and provide rigidity to the rods and strengthen the
enamel. In cross-section, it is best compared to a keyhole with the top, or head, oriented toward the crown of the tooth and
the bottom, or tail, oriented toward the root of the tooth.
They range from 5 million to 12 million in number (5 million in mandibular lateral incisor and 12 million in maxillary first
molars).
Enamel rods are found in rows along the tooth. Within each row, the long axis of the enamel rod generally is perpendicular
to the underlying enamel–dentine junction. In permanent teeth, the enamel rods near the cementoenamel junction (CEJ)
tilt slightly more toward the root of the tooth than would be expected. Knowing the orientation of enamel is very important
in restorative dentistry because enamel unsupported by underlying dentin is prone to fracture and usually is avoided.
Submicroscopic Structure of Enamel Rods
• Keyhole or paddle-shaped
• Separated by interrod substance
• About 5 µm in breadth and 9 µm in length
• The bodies are near the occlusal or incisal surface
• The tails point cervically
• The crystals, parallel to the long axis of the prism heads
• Deviate about 65 degrees from the tails
10. Answer: B
11. Answer: D
12. Answer: C
13. Answer: C (Ref. Orban’s Oral Histology and Embryology, By GS Kumar, 2014, page no. 140)
Dentin is made up of many structures which combine to form the layer Dentin:
• Odontoblastic Processes (Tomes’ process): These are cytoplasmic extensions of odontoblasts, which extend into the
dentinal tubules as odontogenic processes. These are larger in diameter at the pulp when compared toward the dentin.
These processes sometimes extend up to the dentin–enamel junction. They are composed of microtubules, filaments,
mitochondria, lysosomes, microvesicles, etc.
• Peritubular Dentin: The dentin that immediately surrounds the dentinal tubules. It is more mineralized than the
intertubular dentin.
• Intertubular Dentin: It is the main body of dentin. It is located between the dentin tubules or between zones of
peritubular dentin.
• Mantle Dentin: It is a type of dentin which is present under the dentin–enamel junction.
• Pre Dentin: It is the first formed dentin as seen in the name and it is not mineralized. Pre-dentin is located adjacent to
the pulp tissues.
• Primary Dentin: Mantle dentin is the first formed dentin which is mineralized. Circumpulpal dentin forms the remaining
primary dentin or the bulk of the tooth. It represents dentin formed before root completion.
• Secondary Dentin: It represents the dentin formed after root completion. It contains fewer tubules than primary dentin.
Secondary dentin is also known as adventitious dentin.
• Tertiary Dentin: It is the reparative, response, or reactive dentin which is formed in response to trauma such as caries
and restorative procedures. It is characterized by irregular and few tubules. Sometimes it contains cellular inclusions
within the matrix; tertiary dentin is also known as osteodentin.

https://t.me/DentalBooksWorld
Chapter 10 • Oral Anatomy and Histology 903

• Incremental Lines of Von Ebner: These are also known as Imbrication Lines. They appear as fine lines or striations in
dentin. They are at right angles to dentinal tubules and represent rhythmic, recurrent daily deposition of dentin.
• Contour Lines of Owen: Some of the incremental lines of Von Ebner are accentuated due to disturbance in matrix and
mineralization process.
• Interglobular Dentin: Mineralization of dentin begins in small globular masses that fail to coalesce. So the areas of
hypomineralization between these are known as interglobular dentin or spaces.
• Tomes’ Granular Layer: This is the granular layer adjacent to the cementum and visualized by observing ground sections
in transmitted light. This increases toward the root apex and is due to coalescing and looping of terminal portions of
dentinal tubules.
• Tomes’ Fibers: These are odontoblastic processes within the dentinal tubules.
• Korff ’s Fibers: These are fibers seen in the mantle dentin which is seen at the DEJ.
14. Answer: C
15. Answer: D
16. Answer: B
17. Answer: C
18. Answer: B
19. Answer: A
20. Answer: B
21. Answer: D
22. Answer: C
23. Answer: C (Ref. Orban’s Oral Histology and Embryology, By GS Kumar, 2014, page no. 403)
1. Type 1
• Associated with osteogenesis imperfecta
• Deciduous teeth are mainly involved
• Dentinogenesis Imperfecta Radiological Features (Type 1): Obliteration of pulp chamber and reduce radiolucency of
root canal.
2. Type 2
• Not associated with osteogenesis imperfecta
• Both deciduous and permanent dentition are equally involved
• Radiological features: Same as Type 1
3. Type 3
• Unusual form of disease
• Commonly show multiple pulp exposure and periapical lesion
• Both deciduous and permanent dentition are affected
• Radiological features: Pulp chamber and root canal are extremely large. Main feature of all types is short stunted root
24. Answer: B
25. Answer: A (Ref. Orban’s Oral Histology and Embryology, By GS Kumar, 2014, page no. 65)
Enamel lamellae
• In this ground cross-section of tooth, you can see enamel lamellae and enamel tufts and the neonatal line.
• What do all three of these structures have in common? They are all hypocalcified.
26. Answer: B
27. Answer: B
28. Answer: D
29. Answer: D
30. Answer: B
31. Answer: C
32. Answer: D

https://t.me/DentalBooksWorld
904 Triumph’s Complete Review of Dentistry

Dentin dysplasia Dentin dysplasia Dentinogenesis Regional


type I type 2 imperfecta odontodysplasia
Mode of inheritance Autosomal dominant Autosomal dominant Autosomal dominant Nonhereditary
Dentition affected Primary and Primary teeth- Primary and Primary and
permanent teeth- discolored Permanent permanent teeth permanent teeth
normal teeth-normal discolored discolored-yellow to
brown surface teeth
Mobility Present Absent Absent Present
Taurodontism Present Absent Absent Absent
Radiographic Small or no roots Thistle tube shaped Bulbous crowns. Thin enamel and
appearance and obliteration of or flame-shaped pulp cervical constriction, dentin surrounding
the pulp chambers- chambers thin roots, and early enlarged radiolucent
rootless teeth obliteration of pulp pulp-ghost teeth
chamber appearance
Histopathologic Central portion of the Coronal dentin. Coronal dentin Follicular tissue
features root forms whorls of Primary teeth exhibiting short contains scattered
tubular dentin giving irregular tubules misshapen tubules collections
the appearance of Permanent teeth- within atypical of enameloid
stream flowing around normal tubules granular dentin conglomerates
boulders matrix and islands of
True Denticks. odontogenic
Excessive dentin epithelium
remodeling
Conditions Associated with Dentin Dysplasia

Calcinosis universalis
Deposition of calcium salts in skin, subcutaneous tissue, tendons, and muscles. Clinically, patient may have arthralgia to
movement limitation.
Rheumatoid and Vitaminosis D
It is an autoimmune disease characterized by arthritis and symmetric, inflammatory arthritis of small and large joints with
constitutional symptoms including fatigue, weight loss, morning stiffness, low grade fever, and anemia.

Sclerotic Bone and Skeletal Anomalies


Patients with teeth showing all features of skeletal radicular dentine dysplasia (type I) have been anomalies found to have
dense sclerotic of the wrists and hand bones.

Tumoral calcinosis (Ref. Shafer’s Textbook of Oral Pathology, By B Sivapathasundharam, 2014, page no. 58)
Calcium deposition in the soft tissue in periarticular location, that is, around joints. Frequently seen in patients undergoing
renal dialysis.
33. Answer: B
34. Answer: B (Ref. Orban’s Oral Histology and Embryology, By GS Kumar, 2014, page no. 53)
• Enamel Rods or Prism
− Fundamental morphologic primary unit
− Aligned perpendicularly to the DEJ (except in cervical regions of primary teeth)
• A chamfer or long bevel is commonly used as a gingival finish line in permanent tooth preparations because the direction
of the enamel rods in the region of the CEJ is such that the rods deviate from the horizontal in an apical direction
• The direction of enamel rods in primary teeth is inclined in an occlusal direction in the cervical one-third of the crown
− 5–12 million per crown
− Rods increase in diameter as they flare outward “Tails” (from 4 to 8 microns)
− Begin at the future cusp and spread down the cusp slope
− Oldest enamel is at DEJ under cusp or Cingulum
− Good thermal insulator

https://t.me/DentalBooksWorld
Chapter 10 • Oral Anatomy and Histology 905

35. Answer: A (Ref. Orban’s Oral Histology and Embryology, By GS Kumar, 2014, page no. 162)
• Dentin
− Composition
§ 70% inorganic, 20% organic, and 10% water
§ Organic
s Mostly of collagen fibers, Type I
§ Inorganic
s More mineralized than cementum or bone, but less than enamel
¡  Hard→soft: enamel→dentin→cementum
s Calcium Hydroxyapatite
s Main cell is odontoblast – derived from ectomesenchyme
§ Avascular
− More flexible than enamel
− Forms the greatest amount of tooth structure
− Function
§ Nutritive
§ Sensory
§ Protective
− Along with pulp tissue, is formed by the dental papilla
36. Answer: A (Ref. Orban’s Oral Histology and Embryology, By GS Kumar, 2014, page no. 100)
All dentin is formed by odontoblasts located at the border of the pulp and dentin. It consists of tubules surrounding
odontoblastic processes, as well as intertubular dentin. In primary dentin, the tubules are regular in pattern. In
secondary dentin, formed later in life, tubules are less regular, less numerous, and more wavy. Reparative dentin
(sclerotic dentin) forms in response to caries, heat, deep fillings, etc. It is least regular and most wavy, and tubules are
least numerous.
37. Answer: A (Ref. Orban’s Oral Histology and Embryology, By GS Kumar, 2014, page no. 65)
There are a number of somewhat confusing anomalies at the dentinoenamel junction. Enamel spindles are odontoblastic
processes and dentinal tubules which extend a short way into enamel. Enamel lamellae, on the other hand, extend from
enamel into dentin. They are pieces of uncalcified organic enamel material. Enamel tufts are hypocalcified enamel rods. They
are individual entities; however, they appear as groups (tufts) when a group of them are viewed laterally.
38. Answer: A
Enamel is the most mineralized and hardest tissue in the human body. Most estimates place the mineral content of enamel
at about 95–98%. Dentin, though highly mineralized, still has more organic content than does enamel. Most estimates place
the mineral content of dentin at about 80%. Cementum is least mineralized, and is most similar to bone. There are varying
estimates of its mineral content, but most place it at about 60–65%.
39. Answer: C (Ref. Orban’s Oral Histology and Embryology, By GS Kumar, 2014, page no. 125)
This is a case of a best answer with a reasonable second-best answer. The best answer is rough endoplasmic reticulum. The
rough ER contains numerous ribosomes, and is known for producing proteins for export. Collagen fibers of the dentin matrix
would be good examples of protein made for export (outside of the cell). The second-best answer is probably mitochondria,
in that the synthesis of collagen would be energy-consuming. Odontoblasts are single nuclei cells (unlike osteoclasts, which
are multinucleated). Note that smooth ER lacks ribosomes, and that the digestive enzyme activity of lysosomes would not be
needed for this process.
40. Answer: B (Ref. Orban’s Oral Histology and Embryology, By GS Kumar, 2014, page no. 162)
There are a number of somewhat confusing anomalies at the dentinoenamel junction. Enamel spindles are odontoblastic
processes and dentinal tubules which extend a short way into the enamel. Enamel lamellae, on the other hand, extend from
the enamel into dentin.
They are pieces of uncalcified organic enamel material. Enamel tufts are hypocalcified enamel rods. They are individual
entities; however, they appear as groups (tufts) when a group of them are viewed laterally.
41. Answer: B (Ref. Orban’s Oral Histology and Embryology, By GS Kumar, 2014, page no. 180)
The dental pulp is a connective tissue characterized by multiple collagen fibers running in all directions, and large numbers
of fibroblasts, which produce the fibers. There are also capillaries, neurons, lymphatic channels, and different types of
leukocytes, including macrophages, neutrophils, and eosinophils. However, the predominant cell type is the fibroblast. Note
that the pulp is lined by the dentin-forming odontoblasts, but there are no ameloblasts, which form enamel.
42. Answer: C (Ref. Orban’s Oral Histology and Embryology, By GS Kumar, 2014, page no. 30)

https://t.me/DentalBooksWorld
906 Triumph’s Complete Review of Dentistry

The three sections of the tooth bud are the enamel organ, the dental sac, and the dental papilla. The enamel organ, not
surprisingly, produces enamel and contains the inner enamel epithelium, stratum intermedium, stellate reticulum, and outer
enamel epithelium. The dental papilla, a mesodermal derivative, produces both dental pulp and dentin. The dental sac, also
mesodermal, produces both cementum and the periodontal ligament.
43. Answer: D (Ref. Orban’s Oral Histology and Embryology, By GS Kumar, 2014, page no. 125)
Remember that the function of Golgi apparatus is to modify and package proteins produced for export by the cell. In this
way, an odontoblast would be similar to any other protein-producing secretory cell. Proteins produced by ribosomes on
the rough ER will enter the ER and travel to the Golgi apparatus. Here, within the flattened sacs of the Golgi, the collagen
and other fibers will be modified and packaged to be sent to the edge of the cell for secretion as dentin matrix. So the large
production demands of producing collagen and other proteins for predentin should result in odontoblasts having large,
numerous, and active Golgi bodies.

PDL, CEMENTUM, BONE


1. Answer: A (Ref. Orban’s Oral Histology & Embryology, By GS Kumar, 2014, page no. 341)

AC
IR
H

OBL

PA


• The principal fibers of the periodontal ligament are arranged in six groups that develop sequentially in the developing
root: the transseptal, alveolar crest, horizontal, oblique, apical, and interradicular fibers
• Transseptal fibers extend interproximally over the alveolar bone crest and are embedded in the cementum of adjacent
teeth
• They are reconstructed even after destruction of the alveolar bone that results from periodontal disease. These fibers may
be considered as belonging to the gingiva, because they do not have osseous attachment.
• Alveolar crest fibers extend obliquely from the cementum just beneath the junctional epithelium to the alveolar crest
Fibers also run from the cementum over the alveolar crest and to the fibrous layer of the periosteum that covers the
alveolar bone. The alveolar crest fibers prevent the extrusion of the tooth and resist lateral tooth movements. The incision
of these fibers during periodontal surgery does not increase tooth mobility unless significant attachment loss has occurred
• Horizontal fibers extend at right angles to the long axis of the tooth from the cementum to the alveolar bone
• Oblique fibers, which comprise the largest group in the periodontal ligament, extend from the cementum in a coronal
direction obliquely to the bone
• They bear the brunt of vertical masticatory stresses and transform such stresses into tension on the alveolar bone
• The apical fibers radiate in a rather irregular manner from the cementum to the bone at the apical region of the socket.
They do not occur on incompletely formed roots
• The interradicular fibers fan out from the cementum to the tooth in the furcation areas of multi-rooted teeth
2. Answer: D (Ref. Orban’s Oral Histology & Embryology, By GS Kumar, 2014, page no. 196)
Cementicles are small, spherical particles of cementum that may lie free in the periodontal ligament adjacent to the
cementum surface. These are classified as free cementicles. They can also be attached to the cementum surface (attached or
sessile cementicles), or incorporated into the cementum layer (imbedded cementicles). Cementicles may be composed of
fibrillar or afibrillar cementum, or a mixture of the two. They are usually acellular. Their etiology is unknown.
3. Answer: B
4. Answer: C (Ref. Orban’s Oral Histology & Embryology, By GS Kumar, 2014, page no. 157)
Classified according to three factors
1. Based on time of formation
• Primary cementum – formed before eruption
• Secondary cementum – formed after eruption

https://t.me/DentalBooksWorld
Chapter 10 • Oral Anatomy and Histology 907

2. Based on presence or absence of cells in the matrix


• Acellular cementum
• Cellular cementum
3. Based on origin of collagen fibers in the matrix
• Intrinsic fiber cementum – fibers formed by cementoblasts
• Extrinsic fiber cementum – fibers resulting from incorporation of periodontal ligament fibers
Five types of cementum according to this classification
1. Primary acellular intrinsic fiber cementum
2. Primary cellular extrinsic fiber cementum
3. Secondary cellular intrinsic fiber cementum
4. Secondary cellular extrinsic fiber cementum
5. Acellular afibrillar cementum
On the basis of these findings, Schroeder has classified cementum as follows:
• Acellular afibrillar cementum contains neither cells nor extrinsic or intrinsic collagen fibers, except for a mineralized
ground substance. Acellular afibrillar cementum is a product of cementoblasts and found as coronal cementum in
humans, with a thickness of 1–15 μm.
• Acellular extrinsic fiber cementum is composed almost entirely of densely packed bundles of Sharpey’s fibers and lacks
cells. Acellular extrinsic fiber cementum is a product of fibroblasts and cementoblasts. It is found in the cervical third of
roots in humans, but it may extend farther apically. Its thickness is between 30 and 230 μm.
• Cellular mixed stratified cementum is composed of extrinsic (Sharpey) and intrinsic fibers, and it may contain cells.
Cellular mixed stratified cementum is a co-product of fibroblasts and cementoblasts. In humans, it appears primarily in
the apical third of the roots and apices and in furcation areas. Its thickness ranges from 100 to 1000 μm.
• Cellular intrinsic fiber cementum contains cells but no extrinsic collagen fibers. Cellular intrinsic fiber cementum is
formed by cementoblasts, and, in humans, it fills the resorption lacunae. Intermediate cementum is a poorly defined
zone near the cementodentinal junction of certain teeth that appears to contain cellular remnants of the Hertwig sheath
embedded in calcified ground substance.
5. Answer: C
6. Answer: C (Ref. Orban’s Oral Histology & Embryology, By GS Kumar, 2014, page no. 153)
Incremental lines of Salter
• These lines represent the rest periods in cementum formation and are more mineralized than the adjacent cementum
• Both cellular and acellular are arranged in lamellae separated by incremental lines
• These lines are parallel to the long axis of the root
7. Answer: C
8. Answer: D
9. Answer: B
10. Answer: C
11. Answer: C
12. Answer: D
13. Answer: D
Trauma from occlusion does not leads to pocket formation. It does not affects the epithelial attachment apparatus.
14. Answer: D
15. Answer: A
16. Answer: B (Ref. Orban’s Oral Histology & Embryology, By GS Kumar, 2014, page no. 200)
Periodontal changes associated with aging Gingiva
• Thinning and decreased keratinization of gingival epithelium
• Increase in the width of the attached gingiva, with constant location of the mucogingival junction throughout the life
• More dense and coarse connective tissue
• Increase (or) no change in mitotic index of gingival epithelium
• Reduced (or) no change in stippling
Periodontal changes associated with Periodontal Ligament
• Greater no. of elastic fibers
• Decrease in vascularity

https://t.me/DentalBooksWorld
908 Triumph’s Complete Review of Dentistry

• Decrease in mitotic activity


• Decrease in fibroplasia
• Decrease in number of collagen fibers
• Width of periodontal ligament increased or decreased
The tissues that support the teeth are called the periodontium, which consists of gingiva, periodontal ligament, cementum,
and alveolar bone. Anatomical and functional changes in periodontal tissues have been reported as being associated with the
ageing process.
Thinning of epithelium and diminished keratinization
Increased epithelial permeability to pathogens
Decreased resistance to functional trauma
Conflicting results have been reported regarding the shape of the rete pegs. A flattening of rete pegs and an increase in the
height of the epithelial ridges associated with ageing were both demonstrated.
Number of cellular elements decreases as age increases.
The fibroblasts are the main cells in the synthesis of periodontal connective tissue.
17. Answer: D (Ref. Orban’s Oral Histology & Embryology, By GS Kumar, 2014, page no. 191)
Elaunin. Elaunin (Greek verb ἐλαύνω “I steer”) is a component of elastic fibers formed from a deposition of elastin between
oxytalan fibers. It is found in the periodontal ligament and in the connective tissue of the dermis, particularly in association
with sweat glands
18. Answer: D (Ref. Orban’s Oral Histology & Embryology, By GS Kumar, 2014, page no. 242)
Basement Membrane Collagen [type IV]
Collagen IV is similar in size to type I collagen but does not assemble as fibers in the extracellular compartment.
It contains frequent nonhelical sequences and aggregates in a sheet-like, chicken wire configuration.
Type IV collagen is a major component of the basal lamina and is a product of epithelial cells.
FACIT Collagens [types IX, XII, and XIV]
Fibril associated collagen with interrupted helices [FACIT]
Consists of chains that have different lengths and contain a variety of noncollagenous domains. They exhibit several
interruptions in triple helix and are found in various locations in different tissues. This collagen forms heterotypic fibrils
with type II collagen in cartilage and Type XII vitreous humor and XIV are believed to modulate interactions between
adjacent fibrils.
Meshwork-Forming Collagens [types VIII and Type VIII is associated with the basal laminae X] of endothelial cells and
smooth muscle cells and is present in Descemet’s membrane of cornea. It imparts strength while providing an open porous
meshwork. Type X collagen is found almost exclusively in the hypertrophic zone of the epiphyseal cartilage growth plate.
19. Answer: C (Ref. Orban’s Oral Histology & Embryology, By GS Kumar, 2014, page no. 200)
Usually, acute occlusal trauma presents signs and symptoms such as tooth pain, sensitivity to percussion, and increased
tooth mobility. When occlusal forces are dissipated by a shift in the position of the tooth, occlusal wearing, or adjustment,
the injury heals and signs and symptoms usually subside.
20. Answer: B
21. Answer: A
22. Answer: A (Ref. Orban’s Oral Histology & Embryology, By GS Kumar, 2014, page no. 341)
The transseptal fibers come under a group of supra-alveolar fibers, which constitute the principal fibers of the periodontium.
They play a very important role in maintaining the position and orientation of the teeth in the arch. These fibers are
embedded firmly in the cementum of adjacent teeth which maintain the mesiodistal contacts of the teeth. There are a
continuous renewal and self-regeneration of transseptal fibers, even after destruction. They have a role in midline diastema
and relapse of orthodontically treated rotated teeth.
23. Answer: C
24. Answer: B
25. Answer: A
26. Answer: A
27. Answer: A (Ref. Orban’s Oral Histology & Embryology, By GS Kumar, 2014, page no. 156)
• Cementum
− Formed by cementoblasts from PDL, not from odontoblasts from pulp
− Slightly softer and yellower

https://t.me/DentalBooksWorld
Chapter 10 • Oral Anatomy and Histology 909

− Most closely resembles bone, except no Haversian systems or blood vessels


− Avascular, no innervation
− Fxn
§ Compensates for tooth loss
§ Protects from resorption
§ Reparative fxn
− 50% Inorganic, 40% Organic, and 10% Water
− Has the Highest Organic Content
§ Organic
s Collagen and protein
− More resistant to resorption than alveolar bone (permits ortho movement of teeth w/o resorption)
§ 2 Types
s Acellular
¡ No cells

¡ Coronal two-thirds

s Cellular
¡ Contains cementoblasts, cementocytes, fibroblasts from PDL and cementoclasts

¡ Apical one-third

¡ Thickest to compensate for attritional wear of the occlusal/incisal surface and passive eruption

§ Cementoid
s Peripheral layer of developing cementum that is not calcified
28. Answer: C
29. Answer: D (Ref. Orban’s Oral Histology & Embryology, By GS Kumar, 2014, page no. 156)
Cementum is often described as the dental tissue which most resembles bone. It has a similar mineral content, and has an
organic matrix primarily composed of collagen fibers. However, the Haversian systems, concentric tissue layers, lacunae,
canaliculi, etc., found in bone are not found in cementum. Remember that these features are not always found in bone. Only
compact bone (rather than trabecular bone) has these features. Remember also that acellular cementum could not have such
features as Haversian systems, because Haversian systems contain osteocytes (cells) within bony lacunae. However, neither
acellular nor cellular cementum actually has these features.
30. Answer: D (Ref. Orban’s Oral Histology & Embryology, By GS Kumar, 2014, page no. 235)
Two types of marrow space are found within bone, red (hematopoietic), which is the source of blood cells; and yellow (fatty),
which does not produce blood cells. The alveolar bone is similar to most bone in the body in having a compact outer layer of
lamellar bone and an inner layer of spongy bone. This spongy bone contains marrow space, usually of the yellow (fatty) type,
although some red marrow exists, especially in the ramus and condyle of the mandible.
31. Answer: B (Ref. Orban’s Oral Histology & Embryology, By GS Kumar, 2014, page no. 156)
The fibers of the periodontal ligament are collagenous and insert into the compact, lamellar outer layer of the tooth socket.
This is similar to fibers of a tendon inserting into bone. The appearance of the bone with the collagen fibers causes it to
be named bundle bone. The fibers themselves are called Sharpey’s fibers. Tomes processes (choice A) are projections of
ameloblasts into developing enamel. Von Ebner lines (choice C) are found in dentin and show the incremental laying down
of dentin tissue. Lines of Owen (choice D) are found in dentin as well, and are exaggerated Von Ebner lines found in areas of
trauma during dentin formation.
32. Answer: C (Ref. Orban’s Oral Histology & Embryology, By GS Kumar, 2014, page no. 235)
Alveolar bone is a combination of cortical (compact) bone and trabecular (spongy) bone. The cortical bone is on the outside
surface, both next to the tooth root and also continuous with the cortical bone of the maxilla and the mandible. The cortical
bone adjacent to the tooth root is sometimes referred to as lamina dura. Nerves and blood vessels travel primarily through
the more porous cancellous bone between the two cortical plates. Note that alveolar bone can be lost rapidly either in
periodontal disease or following tooth extraction.
33. Answer: C (Ref. Orban’s Oral Histology & Embryology, By GS Kumar, 2014, page no. 156)
Sharpey’s fibers is an older name for the collagenous bundles of the periodontal ligament which connect the tooth to the
tooth socket. The two tissues entered by these fibers are cementum on the tooth, and the cortical bone plate of the alveolar
bone. When they enter the cortical plate, the resulting bone, when viewed under the microscope, is termed bundle bone. The
fibers do not penetrate the bone enough to reach the cancellous inner layers of the alveolar bone.

https://t.me/DentalBooksWorld
910 Triumph’s Complete Review of Dentistry

PRIMARY AND PERMANENT TOOTH CALCIFICATION AND ERUPTION, SHEDDING AND FORMS
Answer: C (Ref. Wheeler’s Dental Anatomy, Physiology and Occlusion, By Stanley Nelson, 2014, page no. 35)
1.
• Primary Teeth
• All 20 primary teeth – in utero
− Begin to form about 6 weeks in utero
− Begin to calcify about 4–6 months in utero
§ Primary teeth show calcification in utero during the 2nd trimester
§ 1st and 2nd molars show calcification during 5–6 months and completed by 3 years
− Primary roots are completed
§ 14 months after emergence for Mn
s Mn teeth erupt from 6–7 months to 20 months
§ 15 months after emergence for Mx
s Mx teeth erupt from 7 months to 24 months
− Calcification of the roots is normally completed by 3–4 years old
§ Remember last tooth comes in at 24 months Primary Mx M2
At 1.5 years, roots are completed for Mn centrals and laterals and Mx Centrals
• Primary Calcification Initiation Sequence
− Mx (in weeks)
§ Centrals 14
§ Laterals 16
§ Canines 17
§ M1 15.5
§ M2 19
− Mn (in weeks)
§ Centrals 14
§ Laterals 16
§ Canines 17
§ M1 15.5
§ M2 18
− Hypoplasia of primary teeth limited to the incisal thirds of incisors, incisal tips of canines, and occlusal portion of
molars indicates a metabolic disturbance during the prenatal period
− If a women took tetracycline during the second trimester, what teeth would be affected???? (Week 13–27)
§ Primary teeth only
s Note Tetracycline affects teeth erupting 1–2 years after taking it
• Eruption Sequence
− Primary Eruption Sequence From the Tooth Bible
§ Mn central (6)
§ Mn laterals (7)
§ Mx central (7.5)
§ Mx lateral (9)
§ Mn M1 (12)
s 1 year, you should have 10 teeth…Unless you are a girl, then @13 months you will have 12 teeth!!!
§ Mx M1 (14)
§ Mn canine (16)
§ Mx canine (19)
§ Mn M2 (20)
§ Mx M2 (24)
− Deciduous eruption sequence: Central, Lateral, M1, Canine, M2
§ The last primary tooth to erupt is the Mx M2
s Also last to start and finish calcifying

https://t.me/DentalBooksWorld
Chapter 10 • Oral Anatomy and Histology 911

− The first primary teeth to erupt are the Mn R and L central incisors
− At 1 year, a child is expected to have erupted prim Mx and Mn incisors and M1s
§ A parent notices a new primary tooth at 12 months, most likely a Mn M1 (12 month Molar)
− Prim M2s are expected to erupt shortly after the child’s 2nd birthday
− Last anterior tooth to calcify
− Most dramatic change to the Oral Flora occurs à when primary teeth erupt
• Exfoliation sequence for Primary Teeth
− Centrals 6–8 years
− Laterals 7–9 years
− 1st Molars 10–12 years
− Canines 9–12 years (10–11 for Mx canine was the correct answer option)
− 2nd Molars 10–12 years
2. Answer: A
• Permanent teeth
• Begin to form at 4 months in utero
• Calcification Initiation Sequence
− Mx
§§ M1 Birth
§§ Centrals 3–4 months
§§ Canines 4–5 months
§§ Laterals 10–12 months
s The Last Permanent Anterior Tooth of the Mx to initiate calcification is the Lateral @10 months
§§ PM1 18–21 months
§§ PM2 2–2.5 years
§§ M2 2.5–3 years
§§ M3 7–9 years
− Mn
§§ M1 Birth
§§ Centrals 3–4 months
§§ Laterals 3–4 months
§§ Canines 4–5 months
s Do not get clowned, here it is the Canine as the last anterior to initiate calcification
§§ PM1 21–24 months
§§ PM2 2–2.5 years
§§ M2 2.5–3 years
§§ M3 8–10 years
• Mx and Mn M1s begin to calcify at birth
− In development of the human permanent dentition, the first teeth to begin calcification are the Mn M1s
§ Initiation of calcification for the mandibular central incisors is 3–4 months
• The incisal ridge is the 1st structure to begin to calcify in an anterior tooth
• Perm M3s begin to calcify at 8–10 years of age
• Active eruption of teeth occurs after one-half of the root is formed (perm or primary)
− 50% of root calcification is complete at the time of eruption
• The apex is usually fully developed by 2–3 years after eruption
3. Answer: A
4. Answer: D (Ref. Wheeler’s Dental Anatomy, Physiology and Occlusion, By Stanley Nelson, 2014, page no. 38)
• 1st succedaneous to erupt is the Mn central incisor at 6–7 years old (do not get clowned – Mn M1 is not succedaneous)
− Permanent max centrals erupt just after at 7–8 years
− Permanent max laterals erupt at 8–9 years
• 6 year old

https://t.me/DentalBooksWorld
912 Triumph’s Complete Review of Dentistry

Presents with all 20 primary and 4 perm 1st molars



First sign of mixed dentition à Mn Molars at age 6

• Ordinarily, a 6-year-old would have the following teeth clinically visible:
− All primary teeth and permanent M1s
• 7 year old (another Q said 6½ year old)
5. Answer: A
Permanent Dentition
Typical Range Maxillary Mandibular Calcification Root Enamel
eruption age completion completion
(years)
6 6–7   1st molar Birth 9–10 years 2.5–3 years
6 6–7 1st molar   Birth 9–10 years 3–4 years
6 6–7   central incisor 3–4 months 9 years 4–5 years
7 7–8 central incisor   3–4 months 10 years 4–5 years
7 7–8   lateral incisor 3–4 months 10 years 4–5 years
8 8–9 lateral incisor   10 months 11 years 4–5 years
10 9–10   Canine 4–5 months 12–14 years 6–7 years
10 10–12   1st premolar 1.75–2 years 12–13 years 5–6 years
10 10–12 1st premolar   1.5–1.75 years 12–13 years 5–6 years
11 10–12   2nd premolar 2.25–2.5 years 13–14 years 6–7 years
11 10–12 2nd premolar   2–2.5 years 12–14 years 6–7 years
11 11–12 canine   4–5 months 13–15 years 6–7 years
12 11–13   2nd molar 2.5–3 years 14–15 years 7–8 years
12 12–13 2nd molar   2.5 years 14–16 years 7–8 years
20 17–21   3rd molar 8–10 years 18–25 years 12–16 years
20 17–21 3rd molar   7–8 years 18–25 years 12–16 years
6. Answer: D
• At 8 years of age
− The teeth normally present are perm central and lateral incisors, primary canines and molars, permanent first molars
− The Perm Mx M1 has no distal contact (it does have a mesial contact with the primary Mx M2)
• At 9 years of age
− 12 primary teeth remain in the mouth (Primary C9 and 2 Molars per quadrant)
• At 10 years of age
− The perm teeth expected are central and lateral incisors, first premolars, and first
− One would expect the root of the perm M1s to be finished forming and calcifying
− One would expect primary tooth “H” (primary Mx canine) to be mobile due to the erupting succedaneous tooth
• The first perm tooth to erupt is generally the Mn M1
− The earliest indication of a mixed dentition consists of the primary dentition and the Mn M1s
s The earliest indication is not the exfoliation of any primary tooth
• The last perm incisors to erupt are the Mx laterals
• Perm PM1 replaces primary M1
• The earliest age by which the roots of the Mx PM1 are completely formed is 12–13 years
− Typically eruption is 10, then 2–3 years for root completion
• At age 26, the third molars are fully erupted with a complete root structure
• The perm Mn arch is the only arch (prim or perm) in which the canine erupts before the tooth immediately distal
to it
− In Primary, Remember M1 is the distal tooth and it erupts at 12 months!!!!, canines in Mn at 16, Mx at 19 months
• 3 Cardinal Eruption Rules:

https://t.me/DentalBooksWorld
Chapter 10 • Oral Anatomy and Histology 913

− Girls’ teeth erupt before Boys


− Mn erupt before Mx
− Teeth of slender kids erupt before fat kids
• Teeth usually erupt in pairs – one on the left and one on the right
• The follicles of the developing permanent incisors are in a position lingual to the deciduous roots
• Eruption problems:
– In cases of delayed resorption of primary incisors, the permanent incisors may be expected to erupt lingual to the
normal arch form
7. Answer: A
8. Answer: C
9. Answer: D
10. Answer: B
11. Answer: A
12. Answer: A
13. Answer: A
14. Answer: A
15. Answer: D
16. Answer: C
• All Posterior Teeth:
− 2 or more cusps
− Occlusal table with ridges and grooves
− Occlusal table makes up 55–65% of the BL dimension of the tooth
− Marginal ridges perpendicular to long axis of the tooth
− Facial height of contour in cervical third
§ Maybe except for Mandibular Molars which have them at the jxn of cervical and middle
− Lingual height of contour in middle third
§ Except for the Mn PM2s, which have their height of contour in the occlusal one-third
− Mesial marginal ridge more occlusal than distal except for Mn PM1s
− Contacts in middle one-third
− Contacts in the faciolingual direction are located in the Buccal side of the center
− Are mesially inclined
− All posterior teeth have rectangular occlusal tables, except PM1s, which have trapezoidal occlusal tables
This is not the same as Occlusal outline (aka “crown profile”)
17. Answer: C
• All Maxillary Incisors:
− Mesioincisal edge is sharp
− Distoincisal edge is rounder
− Distinguish Mx incisors from Mn incisors
− Both larger than Mn incisors
− Central is larger than lateral – except root length which could be quite similar
− Mesioincisal angle is sharper
− Incisal edge is centered labiolingually
− Embrasures: L > F
− Have distinct lingual anatomy and may contain pits
− Both Mx centrals and laterals are wider MD than LL
− Mx incisor roots more frequently contain a single root canal than Mn incisors, Mx PM1s, MxMs (MB root), Mn Ms
(M root)
§ Roots are more rounded than on Mn incisors
• All Mandibular Incisors:
− Laterals are larger than centrals; Centrals are smallest of all teeth
− Incisal edges are lingual to root axis line (Lingual to the LL midpoint)

https://t.me/DentalBooksWorld
914 Triumph’s Complete Review of Dentistry

− Indistinct cingula w/o grooves and pits


− Smooth, continuous convexity incisoapically
− Wider LL than MD
− Embrasures: F > L
− Alveolar process is thinnest facial to Mn central incisors (good for infiltration injection)
§ In anesthesia of the Mn arch, local infiltration is likely to be effective in the incisor area
− Proximal Contact
§ Contact occurs in the incisal third
§ Tends to occur equidistant from facial and lingual surfaces
18. Answer: D
• All Premolars:
− 1 buccal cusp
− 1 or 2 lingual cusps
− Transitional between canines and molars in function
− Enamel is thickest in the occlusal third
− Have their long axis most perpendicular to the horizontal plane → Mx and Mn PMs are the most nearly vertically
aligned teeth
− The union of the F and L triangular ridges forms a transverse ridge
• All Maxillary Premolars:
− B and L cusps of nearly equal heights (but on Mx PM1, the B cusp is slightly larger)
− The L cusp tip is located more mesially than the B cusp tip
− Wider BL than MD (wider BL than Mn PMs, although MD dimension of all PMs is about the same)
− Trapezoidal outline viewed from M or D
− From occlusal view, more rectangle
− Lingual height of contour is ~ midway between the CEJ and the L cusp tip (Dumb way of saying Middle one-third)
− 1st is larger than 2nd
− Roots are more flattened MD than Mn PMs
§§ Roots have mesial concavities
− In development, Mx PMs show crown completion at the same time
• All Mandibular Premolars:
− Large buccal cusp and very small lingual cusp(s)
− Nearly equal MD and BL
− 1st PM is smaller than 2nd PM
− From proximal view, crown tilts lingually in relation to the long axis of the tooth
§§ Occlusal table is lingually displaced (Mx PM table is centered FL)
− Rhomboid when viewed from the interproximal
§§ Mn PM B cusps are more toward the FL midpoint than are Mx PM B cusps
− From occlusal view, more square
− Mn PMs compared to Mx PMs:
§§ Mn have more rounded roots and seldom bifurcated
§§ Mn crowns are tilted to the lingual
§§ Have crowns much more rounded
§§ Mn have less developed lingual cusps
− In development, crown completion of Mn PM1 is 5–6 years; for Mn PM2, crown completion at 6–7 years
19. Answer: A
20. Answer: A
21. Answer: C
• Minimum lobes for permanent tooth – 4
• Mandibular 1st permanent molar – 5 lobes
• Primary mandibular 1st molar – 4 lobes

https://t.me/DentalBooksWorld
Chapter 10 • Oral Anatomy and Histology 915

• Primary mandibular 2nd molar – 5 lobes


• Permanent mandibular 2nd molar – 4 lobes
22. Answer: D
23. Answer: B
24. Answer: B
25. Answer: D

Practice Questions
26. Answer: C
27. Answer: A
28. Answer: D
29. Answer: C
30. Answer: A
31. Answer: B
32. Answer: C
33. Answer: A
34. Answer: A
35. Answer: B
36. Answer: C
37. Answer: B
38. Answer: A
39. Answer: C
40. Answer: D
41. Answer: C
42. Answer: A
43. Answer: C
44. Answer: B
45. Answer: B
46. Answer: A
47. Answer: D
48. Answer: A
49. Answer: D
50. Answer: A
51. Answer: D
52. Answer: D
53. Answer: A
54. Answer: D
55. Answer: A
56. Answer: B
57. Answer: A
58. Answer: C
59. Answer: D
60. Answer: A
61. Answer: B
62. Answer: D
63. Answer: A
64. Answer: C
65. Answer: B
66. Answer: D
67. Answer: A
68. Answer: C

https://t.me/DentalBooksWorld
916 Triumph’s Complete Review of Dentistry

69. Answer: D
70. Answer: C
71. Answer: C
72. Answer: B
73. Answer: A
74. Answer: D
75. Answer: A
76. Answer: A
77. Answer: D
78. Answer: A
79. Answer: B
80. Answer: C
81. Answer: C

PRIMARY AND PERMANENT COMPARISON, VARIOUS TOOTH MORPHOLOGY


1. Answer: C (Ref. Wheeler’s Dental Anatomy, Physiology and Occlusion, By Major M Ash and Stanley Nelson, 2003,
page no. 263)
The primary first mandibular molar is a highly unusual tooth. It is molar-like in form, with two roots (mesial and distal),
but has a very pronounced mesial development of the crown. The mesial section comprises two-thirds of the crown, and
the distal portion is much reduced. The mesiolingual cusp is the highest and sharpest, although the mesiobuccal is overall
largest as viewed from the occlusal. It is followed in size by the ML, DB, and DL cusps.
2. Answer: C (Ref. Wheeler’s Dental Anatomy, Physiology and Occlusion, By Major M Ash and Stanley Nelson, 2003,
page no. 77)
A lingual ridge runs from the incisal edge down the center of the lingual surface of the crown, toward the cervical end of
the crown. It splits the lingual surface in half, and usually has a depression on either side (lingual fossae) which separate the
lingual ridge from the mesial and distal marginal ridges. Lingual ridges are found on canines, but not on incisors (choices A
and B). The lingual ridge is more pronounced on the maxillary canine than on the mandibular (choice D). This is in keeping
with a general rule that the lingual anatomy of the maxillary canine is much more pronounced than that of the mandibular.
3. Answer: B
The common root trunk refers to that area of root apical to the crown but coronal to the bifurcation or trifurcation of the
roots. It can be viewed as the combined root area from which the individual roots emerge. It is characteristic of primary
molars that they have little or no common root trunk. That is, the roots appear to furcate almost immediately below the
crown. This is due to the fact that the primary molar roots must flare widely in order to leave room for the developing
crowns of the succedaneous premolars.
4. Answer: D (Ref. Wheeler’s Dental Anatomy, Physiology and Occlusion, By Stanley Nelson, 2014, page no. 137)
The maxillary premolars are considered to be much more alike than are the mandibular premolars. They are most premolar-
like in form, with fairly similar sized buccal and lingual cusps and greater overall symmetry. The mandibular premolars,
by contrast, are significantly different from each other. The first premolar is very canine-like with a small lingual cusp and
relatively large buccal cusp. The mandibular second premolar, while more premolar-like than the first, has attributes of
molars as well. The chief molar-like attribute of the mandibular second is the tendency toward three cusps rather than two.
The most common form of this tooth has a large buccal cusp with two smaller lingual cusps making up the rest of the crown.
The two lingual cusps are the mesiolingual and distolingual and are separated by a lingual groove. There is a less common
two-cusped variety which is less molar-like in form.
5. Answer: B (Ref. Wheeler’s Dental Anatomy, Physiology and Occlusion, By Stanley Nelson, 2014, page nos. 9 and 12)
In the mandibular first molar, one of the major distinctions between roots is the greater distal curvature of the mesial root.
The distal root, by contrast, is usually more straight, without significant curvature. Another distinction is the deep root
concavity seen running the length of the mesial surface of the mesial root. The distal root, by comparison, will have no
similar concavity or a very slight depression. This concavity will sometimes express itself as a partial split (bifurcation) of
some small section of the apical end of the mesial root into two roots. This feature is rare to unknown on the distal root.
6. Answer: A (Ref. Wheeler’s Dental Anatomy, Physiology and Occlusion, By Stanley Nelson, 2014, page no. 93)
When viewed from the proximal, all incisors show a distinct curvature of the cervical line as the line curves markedly
upward (toward the incisal). It then returns to a much lower (apical) level on both the facial and lingual. It is most noticeable

https://t.me/DentalBooksWorld
Chapter 10 • Oral Anatomy and Histology 917

on the mesial of the central incisor. This is one of those most, biggest, longest, shortest type questions common in this
section.
7. Answer: B
It is commonly said that the third molars and the maxillary lateral incisor are the most variable teeth in the mouth. The
common forms of maxillary lateral variation include the peg lateral, with an ice cream cone-shaped crown; the deep lingual
pit, with a deep (often carious) invagination; and the dens-in-dente, a deep lingual invagination giving a “tooth-within-a-
tooth” appearance on x-ray. By contrast, variations in the other incisors are minor and less pronounced.
8. Answer: B (Ref. Wheeler’s Dental Anatomy, Physiology and Occlusion, By Stanley Nelson, 2014, page no. 129)
The maxillary lateral is often the most distinct in almost any anatomical form. Its cingulum is usually more notable relative
to its size than that of the central. It should be noted that maxillary cingula are almost always more prominent than their
mandibular counterparts. In addition to the prominent cingulum, the maxillary lateral often features a deep lingual fossa,
deep lingual pits, and tubercles on the cingulum.
9. Answer: C (Ref. Wheeler’s Dental Anatomy, Physiology and Occlusion, By Stanley Nelson, 2014, page no. 146)
The mandibular first premolar is very canine-like in form, with a much reduced lingual cusp, almost resembling a cingulum.
The occlusal table thus inclines apically from buccal to lingual, as in a downward slope. This is important in operative
dentistry, as the occlusal preparation for this tooth also is slanted to the lingual. All other premolars are essentially flat in
their occlusal table, or possibly very slightly lingually leaning if the buccal cusp is slightly higher than the lingual. However,
only the mandibular first premolar has this trait so pronounced that it is very different from the other three.
10. Answer: A (Ref. Wheeler’s Dental Anatomy, Physiology and Occlusion, By Stanley Nelson, 2014, page no. 93)
Another way of asking the question is: which tooth does not lean toward the mesial or distal? The maxillary central incisor is
almost straight vertically, estimated to be about 2 degrees from vertical. The lateral tends to lean slightly mesially, the canine
even more in a distal direction, and the first premolar slightly in a distal direction. Do not confuse this leaning with the
buccolingual leaning (when viewed from the mesial or distal). These measurements differ significantly for the teeth listed,
with the premolar being most vertical.
11. Answer: D (Ref. Wheeler’s Dental Anatomy, Physiology and Occlusion, By Stanley Nelson, 2014, page no. 169)
The maxillary canine, at the cervix, will have a cross-section which is oval and flattened mesiodistally. It is generally slightly
wider at the labial end than at the lingual. The pulp cavity at this point is lens-shaped, double convex. It is similar in cross-
sectional shape to the mandibular canine.
12. Answer: B
The maxillary lateral and the third molars show more variability than any other teeth. Besides a wide variety of unusual
morphologies (peg lateral, etc.), the maxillary lateral incisor is fairly commonly congenitally missing. In these patients, the
canines drift mesially toward the centrals. Also commonly congenitally missing are third molars and premolars.
13. Answer: C
14. Answer: D (Ref. Wheeler’s Dental Anatomy, Physiology and Occlusion, By Stanley Nelson, 2014, page no. 53)
Cervical bulges are overhangs of the crown sticking out above the cervical line of the primary teeth. In primary anteriors,
the cervical bulge is found on both the buccal and lingual surfaces. In the molars, it is found only on the buccal. The most
conspicuous cervical bulge in the primary molars is that of the primary mandibular first molar. It is often described as being
potbelly when seen from the proximal view. The other molars are not as exaggerated in this area, but still show a pronounced
buccal cervical bulge.
15. Answer: B (Ref. Wheeler’s Dental Anatomy, Physiology and Occlusion, By Stanley Nelson, 2014, page no. 183)
Despite the fact that the mandibular first molar has a distinct mesial root concavity on the mesial side of the mesial root, and
that this root sometimes furcates partially upward from the mesial, complete additional whole roots are usually second distal
roots. Buccal and/or lingual additional roots are not found.
16. Answer: A (Ref. Wheeler’s Dental Anatomy, Physiology and Occlusion, By Stanley Nelson, 2014, page no. 93)
The maxillary lateral incisor exhibits an almost round root cross-section shape at the cervical line. It is not flattened
mesiodistally or labiolingually. The pulp chamber mimics the root shape by being round at this point as well. It is very
similar in both root shape and pulp shape to those of the maxillary central. Note that as the pulp enters the coronal section,
it will begin to widen mesiodistally (flatten buccolingually) in keeping with the crown shape.
17. Answer: B (Ref. Wheeler’s Dental Anatomy, Physiology and Occlusion, By Stanley Nelson, 2014, page no. 216)
The three largest cusps (trigon, or primitive cusp triangle) of the maxillary first molar are the MB, ML, and DB cusps. The
DL cusp is smallest, and is known as the talon cusp (or talon). This cusp is also the one which is even less conspicuous as you
go from first to second to third molar. Note also that the DB cusp decreases in relative size as you go from the first to second
and third molars. The order of size of the cusps in this tooth, then, is from largest to smallest: ML, MB, DB, DL, Carabelli.
18. Answer: D

https://t.me/DentalBooksWorld
918 Triumph’s Complete Review of Dentistry

Remember that primary molars are succeeded by permanent premolars. The permanent molars erupt distal to all primary
teeth and do not replace them. So the maxillary replacements are 4 for A, 5 for B, 6 for C, 7 for D, 8 for E, 9 for F, 10 for G, 11
for H, 12 for I, and 13 for J. 1, 2, 3, 14, 15, and 16 do not replace primary teeth.
19. Answer: C (Ref. Wheeler’s Dental Anatomy, Physiology and Occlusion, By Stanley Nelson, 2014, page no. 16)
Most maxillary molars are considered four-cusped teeth. However, in some cases, the Carabelli cusp (trait) can be large
enough to be considered a fifth cusp on a maxillary first molar. The maxillary second molar is usually four cusped. The
mandibular first molar is normally five cusped with an ML, MB, DL, and DB cusp and a fifth distal cusp. The mandibular
second lacks the distal cusp and is normally a four-cusp tooth.
20. Answer: A (Ref. Wheeler’s Dental Anatomy, Physiology and Occlusion, By Stanley Nelson, 2014, page no. 191)
Only maxillary incisors have roundish pulp chamber cross-sections when cut at the cervical line. This is true for both
maxillary centrals and laterals. In general, mandibular centrals and both arch canines will have ovalish pulp chambers,
flattened mesiodistally. (Imagine squashing a round pulp from both the mesial and distal sides at once, resulting in a
squashed oval, pointing to the buccal and lingual.) Remember that the shape of a pulp chamber is usually related to the
shape of the surrounding root.
21. Answer: C (Ref. Wheeler’s Dental Anatomy, Physiology and Occlusion, By Stanley Nelson, 2014, page no. 216)
This may not be as obvious as it seems. Rule out 4, as the molar only has three roots: the palatal, mesiobuccal, and distobuccal.
However, how should they look in a distal view? The palatal root will clearly be visible as the only root on the palatal side. It
is massive, and often hooked. On the buccal end, a distal view should easily show the full length of the distobuccal root, so
we have two roots so far. What about the mesiobuccal? In this case we can see this root mostly because it is larger than the
distobuccal, so some, but not all, of the mesiobuccal root is blocked by the distobuccal. We should, however, see some small
sections of it outside of the profile of the distobuccal root.
22. Answer: B
The maxillary lateral is the most highly variable tooth in the mouth. It can be normal incisor-shaped, peg-shaped, or
congenitally missing, or have a wide variety of invaginations, from small pits to deep pits to dens-in-dente, or tooth-within-
a-tooth. This is the case when the invagination is so deep that the radiographic appearance appears to show a small tooth
inside the maxillary lateral.
23. Answer: C (Ref. Wheeler’s Dental Anatomy, Physiology and Occlusion, By Stanley Nelson, 2014, page no. 179)
While all three maxillary molars commonly have three roots, the roots and their arrangement differ distinctively from
one to the other. Most notable in the first molar is the pliers-like appearance of the two buccal roots. The mesiobuccal
hooks distally and the distobuccal hooks mesially. The second molar does not have this curvature, and the two buccal
roots are more parallel, with both roots inclined distally. The third molar often has three fused roots forming a cone-like
structure, or sometimes all distally inclined. These factors are important in exodontia of maxillary molars. Maxillary third
molars, in particular, can sometimes be elevated out distally in a rotating manner because of the fused cone-like distally
facing roots.
24. Answer: A (Ref. Wheeler’s Dental Anatomy, Physiology and Occlusion, By Stanley Nelson, 2014, page no. 216)
Oblique ridges are characteristic of maxillary molar teeth. They connect the mesiolingual and distobuccal cusps. The oblique
ridge is also considered to be the distal end of the trigon, the major cusp area of the maxillary molar, composed of the ML,
MB, and DB cusps. The DL cusp is often reduced and is referred to as the talon (as opposed to the trigon). The trigon is
evolutionarily the primitive cusp triangle of the molar crown. Oblique ridges are not found on mandibular molars or on any
other teeth.
25. Answer: A (Ref. Wheeler’s Dental Anatomy, Physiology and Occlusion, By Stanley Nelson, 2014, page no. 349)
Note that the three-rooted maxillary first molar does not have a mesiolingual root. The maxillary first molar most often
has three roots: mesiobuccal, distobuccal, and palatal. The palatal root is largest, and its canal is largest and widest. In the
case of a fourth canal, it is invariably found in the mesiobuccal root. The distobuccal and palatal roots are invariably one
canaled.
26. Answer: B (Ref. Wheeler’s Dental Anatomy, Physiology and Occlusion, By Major M Ash and Stanley Nelson, 2003, page
no. 67)
One of the distinctions between the mandibular first-molar mesial and distal roots is the deep root concavity seen running
the length of the mesial surface of the mesial root. The distal root, by comparison, will have no similar concavity or a very
slight depression. Other differences include the greater likelihood of the mesial root having a distal curvature, and the greater
likelihood of the mesial root being partly split by a partial furcation running partway up from the apical end.
27. Answer: A
During extraction, rotation can be used on teeth with rounded conical roots. Two primary examples are maxillary centrals
(especially) and maxillary laterals. Rotation may also be used somewhat with maxillary canines. If roots are flattened

https://t.me/DentalBooksWorld
Chapter 10 • Oral Anatomy and Histology 919

mesiodistally, then buccal–lingual movement is used instead of rotation, as rotation may cause crown fracture. Mandibular
centrals, laterals, and canines (choices B, C, and D) are all flattened mesiodistally, and therefore are usually not rotated during
extraction. Mandibular canines also may occasionally have two roots (buccal and lingual), which will prohibit rotation.
28. Answer: A (Ref. Wheeler’s Dental Anatomy, Physiology and Occlusion, By Stanley Nelson, 2014, page no. 216)
As a general rule, maxillary incisors have round root sections and round pulp chamber cross-sections. Mandibular incisors
have more oval ones, and are flattened mesiodistally. This is in keeping with the general root shape of each. Maxillary incisor
roots are round, and mandibular ones are flattened to a greater degree. Note that in both arches, as the pulp moves coronally
it will flatten buccolingually, in keeping with the coronal shape of all incisors.
29. Answer: D (Ref. Wheeler’s Dental Anatomy, Physiology and Occlusion, By Stanley Nelson, 2014, page no. 215)
A characteristic curve of the two buccal root tips of the maxillary first molar toward each other is sometimes referred to as a
pliers-handled appearance or pincer appearance. In effect, the roots grow apically and then turn toward each other, forming
a U shape. This shape helps explain the stability of the tooth, especially when combined with the tripod-like arrangement of
the palatal root. Note that the same roots of the second maxillary molar often both point distally, distinguishing one molar
from the other. These roots in the second molar are also more parallel than those in the first molar. In buccal view they seem
to sway distally, with the palatal root in the midline and slightly mesially tilted.
30. Answer: C (Ref. Wheeler’s Dental Anatomy, Physiology and Occlusion, By Major M Ash and Stanley Nelson, 2003, page no.
508)
The mandibular first molar is invariably a five-cusped tooth, with MB, ML, DB, DL, and distal cusps. Maxillary first and
second molars are normally four-cusped teeth. The distolingual cusp is usually less pronounced on the second molar. It is
often absent on the maxillary third molar, but that answer is not listed. The mandibular second premolar is either a two-
or three-cusped tooth. The two-cusped variety has a single buccal cusp and a single lingual cusp, while the three-cusped
version has a buccal cusp and paired mesiolingual and distolingual cusps. However, both varieties are common, so it is
common to find this tooth lacking the distolingual cusp.
31. Answer: C (Ref. Wheeler’s Dental Anatomy, Physiology and Occlusion, By Major M Ash and Stanley Nelson, 2003, page no.
508)
In the permanent mandibular first molar, the mesial root is usually slightly curved distally, and the distal root is straighter
but pointing toward the distal. In the second molar, both roots are usually significantly inclined toward the distal and curved
distally at the end. They tend to resemble each other more than the two roots of the first molar do. They are also closer
together. The roots in the first molar are more widely spread.
32. Answer: C (Ref. Wheeler’s Dental Anatomy, Physiology and Occlusion, By Major M Ash and Stanley Nelson, 2003,
page no. 161)
Only maxillary incisors have roundish pulp chamber cross-sections when cut at the cervical line. Laterals are egg-shaped
with the widest portion labial, not round. This is true for both maxillary centrals and laterals, which are egg-shaped with
the widest portion of the labial not round. In general, mandibular incisors and both arch canines will have oval-shaped pulp
chambers, flattened mesiodistally. (Imagine squashing a round pulp from both the mesial and distal sides at once, resulting
in a squashed oval, pointing to the buccal and lingual.) Remember that the shape of a pulp chamber is usually related to the
shape of the surrounding root.
33. Answer: C (Ref. Wheeler’s Dental Anatomy, Physiology and Occlusion, By Major M Ash and Stanley Nelson, 2003, page no.
512)
Two roots are a fairly rare occurrence in canines, but are occasionally found in the mandibular canine. They are almost
unknown in the maxillary. If they are found in the mandibular canine, they will be buccal and lingual, similar to those found
in premolars.
34. Answer: D
Most often, the maxillary first molar has three canals: a mesiobuccal, a distobuccal, and a palatal. The palatal is usually
largest and widest. Occasionally (30%) the first molar may have a fourth canal, located within the mesiobuccal root, slightly
lingual to the mesiobuccal canal. When found, it is the smallest and thinnest of all canals in the tooth.
35. Answer: C (Ref. Wheeler’s Dental Anatomy, Physiology and Occlusion, By Stanley Nelson, 2014, page no. 117)
The mandibular central incisor is often described as the most symmetrical tooth when viewed from the labial. The incisal
edge is horizontal, the mesial and distal contacts are at the same height, and the mesioincisal and distoincisal line angles are
equal. In addition, the tooth is not rotated. The mandibular lateral incisor crown is rotated, and its distal contact point is
more apical than the mesial.
36. Answer: B (Ref. Wheeler’s Dental Anatomy, Physiology and Occlusion, By Stanley Nelson, 2014, page no. 149)

https://t.me/DentalBooksWorld
920 Triumph’s Complete Review of Dentistry

The maxillary canine is the longest tooth in the mouth, measured cusp tip to root tip. It is about 3 mm longer than the
maxillary central (choice A), 1 mm longer than the mandibular canine (choice C), and 4 mm longer than the maxillary
second premolar (choice D). Note that the mandibular central is the narrowest tooth in the mouth.
37. Answer: C (Ref. Wheeler’s Dental Anatomy, Physiology and Occlusion, By Stanley Nelson, 2014, page no. 236)
Caries is most commonly found in grooves, pits, and fissures of teeth. Sucrose and other substrates collect and S. mutans
secretes lactic acid to dissolve enamel. All posterior occlusal surfaces are susceptible to caries, as they contain pits, fissures,
and grooves. Maxillary molars have distinct lingual pits or grooves which may become carious. Likewise, buccal pits are
common on mandibular molars. Note that lingual surfaces of mandibular molars are generally smooth, without pits and
fissures.
38. Answer: D
Anatomically, roots of the maxillary first molar are closest to the sinus, which dips downward in this area. Periapical x-rays
of the area sometimes reveal that the roots of the first molar border right on the sinus margin. Although the first molar is
the most likely tooth to have a root enter the sinus, roots of the second premolar and second molar can occasionally end up
there as well.
39. Answer: C
Premolars do not vary far from a vertical line when viewed from the mesial or distal. Both maxillary and mandibular incisors
tip their roots greatly toward the lingual, and canines less so. Mandibular molar roots lean buccally while maxillary molar
roots lean palatally.
40. Answer: C
Erosion is the chemical dissolving of tooth structure. It can be caused by a number of factors, including sucking on lemons
and other acidic fruit, excessive intake of acidic beverages (cola), or excessive vomiting, as in bulimia. Bulimics often exhibit
normal facial surfaces with severely eroded lingual surfaces. This is due to the effect of strong mineral acid from the stomach
(hydrochloric acid at pH 2). Attrition (choice A) is the mechanical wearing of teeth due to physiologic processes, including
chewing and bruxism. Abrasion (choice B) is the mechanical wearing away of tooth structure due to some outside object,
such as toothbrush abrasion at the cervical lines of teeth due to hard sideways brushing, or from habitually holding bobby
pins or nails with the teeth. Internal resorption (choice D) is the loss of dentin from pathologic dissolving by processes
inside the pulp chamber. It can follow tooth trauma.
41. Answer: A (Ref. Shafer’s Textbook of Oral Pathology, By B Sivapathasundharam, 2014, page no. 41)
Gemination is a form of fusion, where a single root attempts to form two crowns, and these crowns share dentin and enamel.
A dilaceration is a sharply bent root, especially near the apex. Concrescence is the joining of two teeth by cementum union.
Taurodonts are teeth, usually molars, with short roots relative to crown size, and large pulpal chambers, giving rise to a bull-
like appearance. Dens-in-dente refers to a tooth-within-a-tooth appearance and is due to invagination, usually in a maxillary
lateral incisor.
42. Answer: E (Ref. Wheeler’s Dental Anatomy, Physiology and Occlusion, By Major M Ash and Stanley Nelson, 2003,
page no. 508)
If you view the maxillary first molar from the occlusal, its shape is that of a rhombus, or equal-sided parallelogram. In any
parallelogram, one pair of opposite corners will be obtuse, while the other pair of opposite corners will be acute. In the
maxillary first molar, the opposite pair of MB and DL are acute, while the pair ML and DB are obtuse.
43. Answer: D (Ref. Wheeler’s Dental Anatomy, Physiology and Occlusion, By Major M Ash and Stanley Nelson, 2003, page no. 520)
The mandibular central and lateral incisors can be viewed as almost twins. They are similar in dimension both mesiodistally
and buccolingually. (The lateral may be slightly, but not significantly, larger.) Their roots are similar in length, although
the lateral may have a more pronounced distal inclination. The crowns are of similar anatomy. However, the lateral has
a twisted or rotated crown, as if you took a central and then slightly rotated the crown around the long axis of the root.
The root may appear to face forward, while the incisal edge is curved in keeping with the normal curve of cusp tips of the
mandibular arch.
44. Answer: C
This is a fairly common occurrence. A child of about 6 years will often present to the dental clinic with a parent, and the
parent will be concerned about a double row of teeth. In these cases, the permanent mandibular incisors are erupting
before the primaries have completely exfoliated. An x-ray usually confirms partial resorption of the primary roots, and
the primaries are allowed to exfoliate. No further treatment is usually indicated. In rare cases where the permanents
erupt buccally, extraction of the primaries and tooth movement of the permanents is in order; however, this is highly
unusual.
45. Answer: A (Ref. Orban’s Oral Histology & Embryology, By GS Kumar, 2014, page no. 284)

https://t.me/DentalBooksWorld
Chapter 10 • Oral Anatomy and Histology 921

Both types of oral epithelium (masticatory and lining) are stratified, not simple, squamous epithelium. In masticatory
epithelium it is highly keratinized and thickened. This type of mucosa covers the hard palate and gingival areas. The
submucosa of these tissues is usually poorly developed or absent. Lining mucosa is generally thin and nonkeratinized, and
has a glandular lamina propria and a well-developed submucosa.
46. Answer: E (Ref. Wheeler’s Dental Anatomy, Physiology and Occlusion, By Stanley Nelson, 2014, page no. 1)
The question depends entirely on the definition of succedaneous. A succedaneous tooth succeeds (replaces) another tooth.
So only permanent teeth can be succedaneous. Note, however, that permanent molars are never succedaneous, as they erupt
distal to all primary teeth (choices C and D). Only permanent incisors, canines, and premolars can be succedaneous (refer to
choices A and B). Of all succedaneous teeth, the permanent mandibular central incisor is usually first, appearing at about age
6, close to the eruption of the nonsuccedaneous permanent mandibular first molar.
47. Answer: C (Ref. Wheeler’s Dental Anatomy, Physiology and Occlusion, By Stanley Nelson, 2014, page no. 157)
Although the time varies, 2–3 years is a common average time. Remember that as a crown erupts, the root is still forming,
and X-rays of newly erupted teeth show varying levels of root formation. This has important implications in endodontics,
when we may try to cause a tooth to complete its root formation in order to better manage it for restoration. A few examples
of typical time lags are: maxillary central incisor, eruption 7–8 years, root completion 10 years; maxillary canine, eruption
11–12 years, root completion 14 years; mandibular second molar, eruption 11–13 years, root completion 14–15 years.
48. Answer: A (Ref. Wheeler’s Dental Anatomy, Physiology and Occlusion, By Stanley Nelson, 2014, page no. 537)
One of the distinctions between the mandibular first-molar mesial and distal roots is the deep root concavity seen running
the length of the mesial surface of the mesial root. The distal root, by comparison, will have no similar concavity or a very
slight depression. This concavity will sometimes express itself as a partial split (bifurcation) of some small section of the
apical end of the mesial root into two roots. This feature is rare to unknown on the distal root. Other differences include the
greater likelihood of the mesial root having a pronounced distal curvature.
49. Answer: C
One way to eliminate answers is to note that the labial view of the maxillary canine is not symmetrical. The mesial cusp
ridge is shorter than the distal cusp ridge. This means that the distance from the cusp tip to the mesial contact is shorter than
that from the cusp tip to the distal contact. Another way to look at it is that the distal cusp ridge dips lower (more cervical).
Therefore, the distal height of contour will be more cervical than the mesial. The mesial height of contour is at about the
junction of incisal and middle thirds, while the distal is at the middle of the middle third. Do not consider choice E, even
though it follows the pattern.
50. Answer: B (Ref. Wheeler’s Dental Anatomy, Physiology and Occlusion, By Major M Ash and Stanley Nelson, 2003, page no. 331)
Although some variation exists, the most common arrangement of canals in the mandibular first molar is two canals in
the mesial root (MB and ML), and one in the distal. The largest is normally the distal, followed by the MB, then the ML.
Approximately 25% of mandibular first molars will have four canals: MB, ML, DB, and DL.
51. Answer: B (Ref. Wheeler’s Dental Anatomy, Physiology and Occlusion, By Major M Ash and Stanley Nelson, 2003, page no. 130)
The maxillary lateral is noted for its variability, and one area of great variety is the lingual fossa. It is normally more
pronounced than that of any other incisor and often contains a pit, from narrow and shallow to deep and extensive.
Extremely deep pits may give a dens-in-dente appearance. In operative dentistry the maxillary lateral is the only incisor in
which you will occasionally see lingual pit amalgams. The lingual surface is often marked by a lingual marginal groove and
tubercles on the cingulum. In general, expect any type of unusual variety to be most commonly found on maxillary laterals.
52. Answer: C (Ref. Wheeler’s Dental Anatomy, Physiology and Occlusion, By Stanley Nelson, 2014, page no. 44)
For the most part, despite an overall size difference and some differences in ratio (crown:root, crown height:mesiodistal
length), the primary teeth generally resemble the permanents for all teeth except the first molars. Primary central and
lateral incisors and canines are fairly close to small versions of the permanents. Primary second molars very closely resemble
permanent first molars. However, primary first molars are unique. The primary maxillary first is premolar-like in crown
form but possesses three roots. The primary mandibular molar is molar-like in form, but with an unusually pronounced
mesial section and a much less developed distal section. It does not resemble any permanent tooth.
53. Answer: A (Ref. Wheeler’s Dental Anatomy, Physiology and Occlusion, By Stanley Nelson, 2014, page no. 121)
An unusual aspect of the maxillary first premolar is the mesial–distal placement of the cusps. The buccal cusp is off center
toward the distal, while the lingual cusp is off center toward the mesial. If you look at the tooth from the lingual, therefore,
you will always see two distinct cusp tips, one slightly to the mesial or distal of the other. This placement, when viewed
occlusally, is said to give the tooth a twisted appearance. This situation is not found in the second premolar, where the cusps
are almost equal in size and straight in alignment.
54. Answer: C (Ref. Wheeler’s Dental Anatomy, Physiology and Occlusion, By Major M Ash and Stanley Nelson, 2003, page no. 333)

https://t.me/DentalBooksWorld
922 Triumph’s Complete Review of Dentistry

Some teeth almost invariably have a divided pulp canal, such as the maxillary first premolar. This tooth normally has two
roots, although the level at which the division occurs can vary. The other three teeth listed are normally single rooted with
single undivided pulp canals. However, the maxillary central incisor almost never varies from a single rounded root. In
contrast, you can sometimes, although rarely, find divided roots, especially near the apex, for the mandibular lateral and
canine.
55. Answer: D (Ref. Orban’s Oral Histology & Embryology, By GS Kumar, 2014, page no. 336)
The TMJ is a synovial joint, similar to most other movable joints. It is different in having a fibrous articular disc. The surface
of the disc is fibrous connective tissue, and the surface of the articulating surfaces of the glenoid fossa are fibrous connective
tissue overlying hyaline cartilage. These surfaces are not covered with synovial tissue; however, their smooth functioning is
dependent on the production of synovial fluid elsewhere in the joint. This production occurs in all nonarticulating surfaces
of the joint, which are lined with synovial tissue.
56. Answer: C
Despite a somewhat premolar-shaped crown, the primary maxillary first molar is consistent in root form with all maxillary
molars, primary or permanent, in having three roots. They are the mesiobuccal, distobuccal, and palatal. Premolars will
routinely have either one or two roots. Primary molars lack a common root trunk, and have widespread roots which allow
space for the developing premolar crown underneath.
57. Answer: D
MOD preparations can be difficult because tooth structure must be removed from three tooth surfaces. The remaining tooth
structure can be weak, and pulp horns may be exposed. They are especially hard in small teeth with large pulp horns. The
primary mandibular first molar is a good example. It has a very narrow distal end, is small overall, and has very high mesial
pulp horns. In addition, little tooth structure remains after the MOD is cut. Permanent mandibular first premolars are small,
but not like the primaries. They should have a slanted occlusal floor, but an MOD is possible. Permanent maxillary second
molars are generally large and would be the best of this group for making an MOD preparation. The primary mandibular
second molar, while small, is larger than the first molar, and has the same shape as a permanent first molar, making it not as
difficult to perform an MOD preparation.
58. Answer: D (Ref. Wheeler’s Dental Anatomy, Physiology and Occlusion, By Stanley Nelson, 2014, page no. 21)
Eruption and exfoliation questions are common NBDE Part 1 topics. Answers are approximate, of course, and subject to
variation. However, you should know the most common average ages for all teeth. For maxillary primaries, the centrals
exfoliate at years 7–8, laterals at 8–9, canines at 11–12, first molars at 10–11, and second molars at 11–12. Remember that
these are averages. Always eliminate wildly wrong answers first.
59. Answer: C
Reviewing the surfaces, the lingual surface of the canine contains the bulbous cingulum. The surface of the lingual is thus
slanted at roughly 45 degrees from the long axis (as represented by the root). Likewise, the facial surface slants inward as
you go from cervical to incisal. The distal surface slants lingually as the tooth narrows coronally. Our answer is mesial. The
mesial surface is vertical overall, with no large bulge. Notice that the distal is always more rounded and bulbous than the
mesial, which is relatively straight and, if extended, would roughly parallel the root.
60. Answer: B
The lateral pterygoid muscle has two heads. The superior head arises from the greater wing of the sphenoid bone, and the
inferior head from the lateral plate of the lateral pterygoid bone. They extend posteriorly to the mandibular condyle, TMJ
capsule, and TMJ disc. Fibers from the upper head enter the capsule and merge with the fibrous connective tissue of both
the capsule and disc. This is related to the function of the muscle in protruding the mandible, and pulling the disc with the
condyle during protrusion, depressing (opening), and lateral movements. No other muscle enters the TMJ.
61. Answer: C (Ref. Wheeler’s Dental Anatomy, Physiology and Occlusion, By Major M Ash and Stanley Nelson, 2003, page no. 508)
A characteristic curve of the two buccal root tips of the maxillary first molar toward each other is sometimes referred to as a
pliers-handled appearance or pincer appearance. In effect, the roots grow apically and then turn toward each other, forming
a U shape. This shape helps explain the stability of the tooth, especially when combined with the tripod-like arrangement of
the palatal root. Note that the same roots of the second maxillary molar often both point distally, distinguishing one molar
from the other. These roots in the second molar are also more parallel than those in the first molar.
62. Answer: D (Ref. Wheeler’s Dental Anatomy, Physiology and Occlusion, By Major M Ash and Stanley Nelson, 2003, page no. 508)
The mandibular second premolar has a variety of occlusal appearances due mostly to its two-cusped or three-cusped nature.
There is a slightly larger buccal cusp area, and then either an approximately equal-sized lingual cusp, or a pair of lingual
cusps, known as the mesiolingual and distolingual cusps. Together, these two form an area slightly smaller than the buccal
cusp. In the event of the three cusps, an occlusal Y groove pattern is common. On the two-cusped variety, either an H (more
common) or U (less common) pattern may be found.
63. Answer: D

https://t.me/DentalBooksWorld
Chapter 10 • Oral Anatomy and Histology 923

The primary first molars, both maxillary and mandibular, are unique in morphology and bear little resemblance to any
permanent teeth. Primary incisors and canines resemble their permanent counterparts closely, except in certain proportional
ways. Likewise, primary second molars are notable for closely resembling their permanent FIRST molar counterparts. The
primary maxillary first molar is a little like a premolar in shape, but generally has one minor and two major cusps. It is both
small and squat in appearance. The mandibular primary first molar is four cusped but has a distinctly larger development of
the mesial end. The MB and ML cusps generally comprise two-thirds of the area of the crown.
64. Answer: A
Primary teeth are often said to have a short or squat appearance of their crowns compared to those of permanent teeth.
This is because these teeth are often relatively wide mesiodistally and short incisocervically. This results in a large ratio
of mesiodistal diameter to incisocervical crown height. This ratio is smaller in the permanent teeth, which lack the short,
squat appearance of primaries. One example: for maxillary central incisors, the primary tooth ratio is 1.083 (the tooth is
just slightly bigger mesioincisally than in crown height), while for the corresponding permanent incisor it is 0.809 (the
mesiodistal diameter is only 80% of the crown height).
65. Answer: A (Ref. Wheeler’s Dental Anatomy, Physiology and Occlusion, By Stanley Nelson, 2014, page no. 1)
As a general rule, primary teeth are whiter than their permanent counterparts. This shade difference is often noticed by
parents. It is common for parents to notice, for example, that newly erupted permanent mandibular incisors are a few shades
darker or more yellowish than the primary maxillary incisors that may remain as the mandibular permanent incisors erupt.
In the mixed dentition (normal, unstained, not heavily carious, and no tetracycline effects), you can often pick out the
primary versus permanent teeth by color alone.
66. Answer: B (Ref. Wheeler’s Dental Anatomy, Physiology and Occlusion, By Stanley Nelson, 2014, page no. 119)
In general, canines are wider mesiodistally than lateral incisors (both arches), so we should eliminate the canine (choice
D). In general, maxillary incisors are wider mesiodistally than their mandibular counterparts, so we should eliminate the
maxillary lateral (choice A). So it should only be a question of mandibular lateral versus central. On the average, the central
is slightly smaller, about 0.5 mm smaller, and this gives it the distinction of narrowest anterior tooth mesiodistally. The
Dental Board examination really loves longest, shortest, roundest, fattest, etc., type questions. Note that the maxillary canine
is the longest tooth in the mouth.
67. Answer: A (Ref. Wheeler’s Dental Anatomy, Physiology and Occlusion, By Stanley Nelson, 2014, page no. 86)
In looking at a maxillary central incisor from the facial, you will see that the mesioincisal angle is a sharp, nearly right angle.
This is especially noticeable where the two centrals meet and a very small embrasure is present. The distoincisal angle,
by contrast, is more rounded (choice C). On the lateral, the mesioincisal (choice B) is again sharper than the distoincisal
(choice D), which is very rounded. However, the angle on the lateral does not approach the sharpness of that on the central.
68. Answer: C
While the mandibular third molar is often so variable that it is hard to make useful predictions about it, these generalities
apply to mandibular first, second, and third molars as you move distal in the arch: (1) they decrease in mesiodistal length,
and (2) their roots decrease in length. Third-molar crowns may resemble normal molars or may be missing whole cusps. In
rare cases they are roundish or ovoid, but they are always smaller than their second-molar counterparts, which are always
smaller than the first molars. Note that buccolingual length is always shortest in the third molar but about equal in the first
and second.
69. Answer: C (Ref. Wheeler’s Dental Anatomy, Physiology and Occlusion, By Major M Ash and Stanley Nelson, 2003, page no. 65)
The mandibular first molar is invariably a five-cusped tooth, with MB, ML, DB, DL, and distal cusps. Maxillary first and
second molars are normally four-cusped teeth. The distolingual cusp is usually less pronounced on the second molar. It is
often absent on the maxillary third molar. This is an arch trait: the gradual diminishment in size of the distolingual cusp as
you go posteriorly in the maxilla. In the lower arch, the mandibular second premolar can also be viewed as possibly lacking
a distolingual cusp. This tooth is either two- or three-cusped. The two-cusped variety has a single buccal cusp and a single
lingual cusp, while the three-cusped version has a buccal cusp and paired mesiolingual and distolingual cusps. However,
both varieties are common, so it is common to find this tooth lacking the distolingual cusp as well, although that answer is
not listed here.
70. Answer: C (Ref. Orban’s Oral Histology & Embryology, By GS Kumar, 2014, page no. 89)
Chemical analysis of the enamel matrix has not adequately identified the protein involved, but it is clear that it is neither
keratin, as was once thought, nor collagen, which is the matrix of both dentin and bone. The protein is extremely high in
proline, but makes up less than 1% of the enamel mass.
71. Answer: B (Ref. Wheeler’s Dental Anatomy, Physiology and Occlusion, By Major M Ash and Stanley Nelson, 2003, page no. 70)
Primary teeth are notable for cervical bulges; that is, extension of the crown at the cervical line. In the primary anteriors,
these bulges extend over both the buccal and lingual surfaces, as though the cervical line was a tightened belt and was

https://t.me/DentalBooksWorld
924 Triumph’s Complete Review of Dentistry

tightened further to make the crown overhang in both front and back. In the primary molars, by contrast, the cervical bulge
is buccal only.
72. Answer: A (Ref. Wheeler’s Dental Anatomy, Physiology and Occlusion, By Stanley Nelson, 2014, page no. 198)
Beware of choice D. Although mandibular third molars are notoriously variable in both crown and root form, there are
some good generalizations. Almost invariably, the mesial (trigonid) end of the crown is wider, larger, and more distinct than
the distal (talonid). The most common pattern is a four-cusp tooth, with MB and ML being the largest cusps and the whole
occlusal surface tapering toward the distal.
73. Answer: D
Another way of asking the question is: which tooth does not lean toward the buccal or lingual? The maxillary first premolar
is almost straight vertically, estimated to be about 5 degrees from vertical. Premolars, as a class, have the least buccolingual
angulation. The central tends to lean greatly toward the lingual (28 degrees), the lateral about the same (26 degrees), and the
canine slightly less. Do not bother to remember degree measures, which are mentioned only for comparison. Do not confuse
this leaning with the mesiodistal leaning (when viewed from the buccal or lingual). These measurements differ significantly
for the teeth listed, with the central incisor being most vertical.
74. Answer: D
Both 23 and N are mandibular left incisors. Remember that A through J are maxillary, while K through T are mandibular.
The mandibular replacements are 20 for K, 21 for L, 22 for M, 23 for N, 24 for O, 25 for P, 26 for Q, 27 for R, 28 for S, and 29
for T. Remember that permanent molars 17, 18, 19, 30, 31, and 32 do not replace primary teeth.
75. Answer: B (Ref. Wheeler’s Dental Anatomy, Physiology and Occlusion, By Stanley Nelson, 2014, page no. 46)
The primary mandibular first molar is a highly unusual tooth. Although it technically contains four cusps, in typical molar
fashion it has a great variety in the prominence of those cusps. The MB and ML cusp are most prominent, and the mesial
section of the crown comprises two-thirds of the crown area. Both the DB and DL cusps are much reduced. The range of
cusp size, in decreasing order is: MB, ML, DB, DL. The ML cusp is notable for a pointy, cone-like shape.
76. Answer: A
In postural rest position, the mandible stays a comfortable distance from the maxilla, with the teeth slightly separated. The
freeway space between the teeth is often 2–5 mm. There is no tooth contact, and the position is determined and maintained
by the muscles of occlusion and related tendons and ligaments. Centric occlusion involves full tooth contact. Centric
relation is normally a few millimeters distal to centric occlusion and is achieved by moving distal from centric occlusion.
This procedure involves tooth contact. Protruded contact involves contacting guidance of the maxillary and mandibular
teeth during protrusion.
77. Answer: A (Ref. Wheeler’s Dental Anatomy, Physiology and Occlusion, By Major M Ash and Stanley Nelson, 2003, page no. 508)
Mandibular first molars are the largest teeth in the mouth, in mesiodistal dimension. While they are also large buccolingually,
they are invariably larger mesiodistally by about 0.5–1.0 mm. This is in contrast to the maxillary first molar, which is slightly
larger buccolingually than mesiodistally, usually by about 1.0 mm. The mandibular second molar is about equal in both
directions and is thus more symmetrical than the first molar.
78. Answer: A (Ref. Wheeler’s Dental Anatomy, Physiology and Occlusion, By Major M Ash and Stanley Nelson, 2003, page no. 506)
Maxillary first premolars are almost always double rooted. When single rooted, they always have two separate root canals.
The level of division of the two roots varies, from complete division up to the crown, to only a small separation apically. The
mandibular first premolar is most often single rooted, and the mandibular second premolar is almost always single rooted
(double roots are rarer than in the mandibular first). The maxillary second premolar is most often single rooted. The extreme
likelihood that the maxillary first premolar will have two roots has implications for both endodontics and exodontia of this
tooth. It is also reported that some small percentage (5% in one study) are actually triple rooted.
79. Answer: D (Ref. Orban’s Oral Histology & Embryology, By GS Kumar, 2014, page no. 274)
The mesiolingual cusp of a maxillary molar is a holding cusp. The general rule for maxillary holding cusps is as follows: a
maxillary holding cusp contacts the distal marginal ridge of its mandibular counterpart and the mesial marginal ridge of
the mandibular tooth distal to its counterpart, except for the mesiolingual cusps of the molars, which contact the central
fossae of their counterparts. This should be the central fossa of the counterpart, the mandibular first molar.
80. Answer: C (Ref. Wheeler’s Dental Anatomy, Physiology and Occlusion, By Stanley Nelson, 2014, page no. 116)
This is known as an arch trait, as it is true for both incisors of each arch. Both incisors of the maxilla are wider mesiodistally.
Both incisors of the mandible are wider buccolingually. The maxillary central, in particular, is known for the greatest
asymmetry in this regard. It is much wider mesiodistally than buccolingually, and that difference, expressed as a ratio, is
greatest for that incisor. It is also the largest incisor, in both dimensions, in absolute size.
81. Answer: C

https://t.me/DentalBooksWorld
Chapter 10 • Oral Anatomy and Histology 925

The palatal or lingual root of the maxillary first molar is the most massive by far. It is normally straight but palatally inclined,
giving the three roots a tripod-like appearance. The mesiobuccal root (choice A) is second in size, and the distobuccal (choice
B) is smallest. The mesiobuccal is often curved distally and the distobuccal curved mesially. This pliers-like appearance is not
found in the maxillary second and third molars.
82. Answer: C (Ref. Wheeler’s Dental Anatomy, Physiology and Occlusion, By Stanley Nelson, 2014, page no. 214)
The mandibular first premolar is the exception to premolar form in many ways. It is canine-like with a reduced lingual cusp,
resembling a cingulum. Its occlusal table tilts downward from buccal to lingual in a slanted fashion. It is also slightly higher
on the distal side than the mesial, and its mesial marginal ridge is less prominent than its distal marginal ridge. This means
that in a mesial view you can see some of the occlusal table because it slants toward you. From the distal view, this is not true.
The higher distal end blocks any occlusal view from that direction.
83. Answer: D (Ref. Wheeler’s Dental Anatomy, Physiology and Occlusion, By Stanley Nelson, 2014, page nos. 46 and 48)
The mandibular and maxillary canines exhibit similar cross-sectional shapes at the cervical line. The mandibular canine
cross-section is oval and flattened mesiodistally. It is generally slightly wider at the labial end than at the lingual. The pulp
cavity at this point is lens shaped, double convex.
84. Answer: E (Ref. Wheeler’s Dental Anatomy, Physiology and Occlusion, By Stanley Nelson, 2014, page no. 215)
There are two facts or concepts involved in correctly answering this question. The first is that the mandibular first molar is
a five-cusped tooth, named corresponding to the cusps listed in the answer choices. The second is that the pulpal anatomy
reflects the cuspal anatomy (or vice versa). Tall, large cusps are likely to have tall, large pulp horns. Small cusps generally
have small associated pulp horns. In this case, the distal cusp is generally the smallest of the cusps of the mandibular first
molar, so its pulp horn is likely to be smallest as well.
85. Answer: D (Ref. Wheeler’s Dental Anatomy, Physiology and Occlusion, By Major M Ash and Stanley Nelson, 2003,
page no. 93)
The premolars are usually considered to be two-cusped teeth (bicuspids). The exception to the rule, to a small extent, is the
mandibular second premolar. Often, the lingual cusp area is divided into a mesiolingual cusp and a distolingual cusp. This
gives the tooth a three-cusped appearance. A two-cusped variety also exists, with a large lingual cusp instead of the two
smaller ones. In the three-cusped variety, a lingual groove separates the two lingual cusps. Note that the two-cusped and
three-cusped varieties are not grossly different in size, as the single lingual cusp is about the same size as a mesiolingual and
distolingual combined. The other premolars exist primarily in double-cusped varieties only.
86. Answer: A
This can vary considerably, but using average eruptions, all permanent first molars usually erupt at around age 6–7, so four
first molars are likely. Rule out all premolars, which generally erupt between age 9 and 12. Likewise, canines usually erupt
between age 9 and 12. For the incisors, usually the mandibular centrals are in at about age 6, so that makes eight so far.
Mandibular laterals and maxillary centrals and laterals usually erupt between 7 and 9 years. So the average 6-year old has all
first molars and mandibular centrals as erupted permanent teeth.
87. Answer: B
Contacts ensure that food does not get lodged between teeth and into the sulcus. Food, debris, and plaque in that area
will lead to inflammation, tissue destruction, and bone loss. Contacts do not protect the incisal surface, as they are below
(cervical to) that area. They do protect the gingiva (see above), but not especially the alveolar mucosa, which is more cervical
and basically below the tooth region. The alveolar mucosa begins at the mucogingival junction. Contact is not involved in
restoration retention. All restorations must be retained by retention within the individual tooth itself.
88. Answer: D (Ref. Orban’s Oral Histology & Embryology, By GS Kumar, 2003, page no. 140)
We often do not think of anterior teeth as having marginal ridges, as they are less distinct than on the posterior teeth; and in
the posterior teeth, they are often points of occlusal contact. However, the maxillary canine, as well as other anterior teeth,
does have both a mesial and distal marginal ridge. The lingual anatomy of the maxillary canine has a cusp tip, with mesial
and distal marginal ridges sloping down from it. Centrally, a lingual ridge splits the lingual surface in two. In between the
marginal ridge and the lingual ridge, we find two depressions – the mesiolingual fossa and the distolingual fossa

TMJ AND PDL


1. Answer: E (Ref. Orban’s Oral Histology and Embryology, By GS Kumar, 2014, page no. 366)
The lateral pterygoid connects from the greater wing of the sphenoid and the lateral surface of the lateral pterygoid plate to the
mandibular condyle, articular disc, and capsule. It is the only muscle that attaches to these structures. The stylomandibular
ligament, sometimes given a vague protective function, extends from the styloid process to the lingula of the mandible. The
medial pterygoid muscle is an elevator with no direct attachment to the TMJ. The mandibular condyle is separated from the

https://t.me/DentalBooksWorld
926 Triumph’s Complete Review of Dentistry

glenoid (temporal) fossa by the articular (TMJ) disc. The condyle does not pull the disc. The disc is also called the meniscus;
they are the same thing.
2. Answer: B (Ref. Orban’s Oral Histology and Embryology, By GS Kumar, 2014, page no. 367)
The bony origins of the three TMJ ligaments are primarily from three different bones. The stylomandibular ligament
originates from a thin, bony extension of the temporal bone, known as the styloid process. The spine of the sphenoid bone is
the major origin of the sphenomandibular ligament. The lateral ligament (temporomandibular ligament) has its origin from
the zygomatic process. This process is a fusion of the temporal and zygomatic bones.
3. Answer: A (Ref. Orban’s Oral Histology and Embryology, By GS Kumar, 2014, page no. 189)
We can eliminate choice B, as the apex is at the opposite end from the crown. The apical fibers are, of course, most apical,
and run from the apex of the alveolar bone socket to the root tip. The oblique fibers are slightly more coronal. They run from
the cementum near the apical end, obliquely coronally to alveolar bone. The horizontal fibers are more coronal and run
horizontally from the cementum to the alveolar bone around mid-root. The alveolar crestal fibers are most coronal, and run
from the most coronal part of the root to the alveolar crest of the bone.
4. Answer: C (Ref. Orban’s Oral Histology and Embryology, By GS Kumar, 2014, page no. 195)
All of the vessels listed supply blood to the PDL, as all of them are in the vicinity of the PDL and all anastomose or send
branches there. Out of this group, however, the major source is vessels branching from the periosteum surrounding the
alveolar bone. Note that both lymphatics and nerves follow the path of the blood vessels in the PDL.
5. Answer: B
Periodontal fibers must run from the cementum of the tooth into the alveolar bone. The main periodontal fiber types are
alveolar crestal, horizontal, oblique, and apical, named for either their position or alignment. The transseptal fiber is classified
with the gingival group, as it does not enter the alveolar bone. Instead, as the name suggests, it travels from one tooth to
another tooth, crossing over the alveolar crest. It attaches directly from the cementum of one tooth into the cementum of
another, but does not attach to the bone.
6. Answer: C (Ref. Orban’s Oral Histology and Embryology, By GS Kumar, 2014, page no. 189)
The attachment fibers are usually divided into two groups: the gingival group and the periodontal group. Gingival fibers attach
tissues, not including alveolar bone. For example, circular fibers run only within gingiva, encircling the tooth. Dentogingival
fibers run from the cementum to the gingiva. Dentoperiosteal fibers run from the cementum to the periosteum. Periodontal
fibers run from the alveolar bone to the cementum, and include alveolar crestal, oblique, horizontal, and apical.
7. Answer: C (Ref. Orban’s Oral Histology and Embryology, By GS Kumar, 2014, page no. 367)
The origins and insertions of the TMJ ligaments are as follows: The sphenomandibular ligament arises from a spine on the
sphenoid bone and runs forward and downward to insert on the lingula and deep ramus of the mandible. The stylomandibular
ligament arises from the spine of the temporal bone known as the styloid process and inserts on the lower ramus and angle
of the mandible. The lateral ligament is also known as the temporomandibular ligament. It descends from the lower border
and tubercle of the zygoma to the posterior lateral condyle. Its fibers merge with those of the articular capsule.
8. Answer: C (Ref. Orban’s Oral Histology and Embryology, By GS Kumar, 2014, page no. 370)
This may seem counterintuitive, as excess motion of the condyle results in the mandible being locked in a forward and open
position. However, anatomically, what is happening is that the condyle has moved forward, down the surface of the articular
eminence, past the tip of the eminence, and past the bulk of the articular tubercle. It is now past a large, inferiorly projecting
mound of bone (the tubercle). To return the condyle to normal position, a downward (inferior) force is needed to position
the condyle once again below the articular tubercle and eminence. The inferior directed force is also needed to counteract
the elevating forces of the medial pterygoid, masseter, and temporalis muscles. On repositioning, the mandible often snaps
strongly into normal position, due to these strong elevations.
9. Answer: C (Ref. Orban’s Oral Histology and Embryology, By GS Kumar, 2014, page no. 367)
The TMJ has an upper and a lower compartment, separated by the articular disc, which is composed of fibrous connective
tissue. Both the upper and lower compartments are synovial; that is, they produce synovial fluid. Synovial fluid aids in
lubricating the articular surfaces of the joint, and is found in many movable joints. Note that the synovial membrane lines
the inside of the joint but not the actual articulating surfaces. These surfaces are fibrous connective tissue over hyaline
cartilage.
10. Answer: D (Ref. Orban’s Oral Histology and Embryology, By GS Kumar, 2014, page no. 372)
First, eliminate choices A and B, as the disc divides the joint compartment into upper (superior) and lower (inferior)
compartments only. In the upper compartment, the upper surface of the disc acts as a cushion as the mandibular condyle
slides forward. This cushion slides against the articular eminence in the movement of the condyle in a forward and
downward direction. When rotating about an axis during opening, the condyle slides against the lower surface of the disc

https://t.me/DentalBooksWorld
Chapter 10 • Oral Anatomy and Histology 927

which conforms to the shape of the condylar head. So we say that translation occurs in the upper compartment and rotation
in the lower.
11. Answer: B (Ref. Orban’s Oral Histology and Embryology, By GS Kumar, 2014, page no. 372)
The ligaments, being fibrous connective tissue in composition, play no role in the movement of the condyle. Muscular
tissue is necessary for this movement, specifically fibers from the upper head of the lateral pterygoid muscle. There is a
disagreement among anatomy sources as to the function or role of the ligaments. None give the ligaments an extremely
important function, but most claim that they help limit motion of the mandible beyond a certain point, or act as barriers to
excessive forward or downward motion. Others give the ligaments a general strengthening function which adds stability to
the joint. Most importantly, the Dental Boards usually agree with the general notion of adding strength, stability, and limits
of movement to the joint.
12. Answer: C
Odontoblasts form dentin and surround the dental pulp. The dentin forms the bulk of both the tooth crown and the root.
However, dentin is not one of the opposing surfaces within the periodontal ligament. Within the ligament, the alveolar bone
of the tooth socket lies next to the cementum surface of the tooth root. On the cemental side of the ligament, cementum-
forming cementoblasts are found. On the bone side, bone-forming osteoblasts and bone-remodeling osteoclasts are both
found. In addition, numerous collagen fiber bundles insert in both the cementum and the alveolar bone, forming the
connection that keeps the tooth in place in the socket.

ORAL MUCOUS MEMBRANE


1. Answer: B (Ref. Orban’s Oral Histology & Embryology, By GS Kumar, 2014, page no. 262)
INTERDENTAL GINGIVA
• It occupies the gingival embrasure.
• Gingival embrasure is the interproximal space beneath the area of tooth contact.
• It is of two shapes – pyramidal and “col” shape.
• “col” is a valley-like depression that connects a facial and lingual papilla.

“col” in various types of contacts


• The shape of interdental gingiva depends on the contact point between the two adjoining teeth and presence or absence
of some degree of recession.
Interdental col in normal gingiva – mandibular anterior segment, facial and buccolingual views. Interdental col after gingival
recession.
In diastema, the gingiva is firmly bound over the interdental bone and the interdental papilla will be absent.
2. Answer: C (Ref. Orban’s Oral Histology & Embryology, By GS Kumar, 2014, page no. 238)
Melanocyte, a specialized skin cell that produces the protective skin-darkening pigment melanin. Birds and mammals
possess these pigment cells, which are found mainly in the epidermis, though they occur elsewhere – e.g., in the matrix
of the hair. Melanocytes are branched, or dendritic, and their dendrites are used to transfer pigment granules to adjacent
epidermal cells.
All melanocytes, whether resident in the basal epidermis or in the matrix of the hair, have migrated there during embryonic
life from a region known as the neural crest. Each epidermal melanocyte is associated with a group of neighboring
keratinocytes (keratin-synthesizing epidermal cells) into which its dendrites transfer pigment. This structure is known as an
epidermal melanocyte unit. The melanin produced by melanocytes is of two kinds: dark brown eumelanin and pale red or
yellowish pheomelanin. Both are formed within the melanocytes by the initial oxidation of the amino acid tyrosine with the
aid of the enzyme tyrosinase; subsequently their synthetic pathways diverge.
An increase in melanin pigmentati Hypoplasia, concrescence on may be caused by an increased density of melanocytes, by
abnormal packaging of melanin, or by increased melanin production.
3. Answer: B

Cytokeratin 4 • Nonkeratinized squamous epithelium, including cornea and transitional epithelium


Cytokeratin 7 a. A subgroup of glandular epithelia and their tumors
b. Transitional epithelium and transitional carcinoma

https://t.me/DentalBooksWorld
928 Triumph’s Complete Review of Dentistry

Cytokeratin 8 a. Glandular epithelia of the digestive, respiratory, and urogenital tracts, both endocrine and exocrine
cells, as well as mesothelial cells
b. Adenocarcinomas originating from those above
Cytokeratin 10 • Keratinized stratified epithelium
• Differentiated areas of highly differentiated squamous cell carcinomas
Cytokeratin 13 a. Nonkeratinized squamous epithelia, except cornea
Cytokeratin 14 a. Basal layer of stratified and combined epithelia
Cytokeratin 18 a. Glandular epithelia of the digestive, respiratory, and urogenital tracts, both endocrine and exocrine
cells, as well as mesothelial cells
b. Adenocarcinomas originating from those above
Cytokeratin 19 • Glandular-type epithelia
• Carcinomas
Does not react with hepatocytes and hepatocellular carcinoma
Cytokeratin 20 a. Glandular-type epithelia. Signet ring/round clear cells
b. GI stromal tumor (Krukenberg)
Notes about cytokeratin from Carranza
• They are numbered in a sequence that is contrary to their molecular weight. In general, basal cells begin synthesizing lower-
molecular-weight keratins (e.g., K19 [40 kD]), and they express other higher-molecular-weight keratins as they migrate to the
surface. K1 keratin polypeptide (68 kD) is the main component of the stratum corneum.
• Keratins K1, K2, and K10 through K12, which are specific to epidermal-type differentiation, are immunohistochemically
expressed with high intensity in orthokeratinized areas and with less intensity in parakeratinized areas. K6 and K16, which
are characteristic of highly proliferative epithelia, and K5 and K14, which are stratification-specific cytokeratins, are also
present. Parakeratinized areas express K19, which is usually absent from orthokeratinized normal epithelia.
• As with other nonkeratinized epithelia, the sulcular epithelium lacks granulosum and corneum strata and K1, K2, and K10
through K12 cytokeratins, but it contains K4 and K13, the so-called “esophageal-type cytokeratins.” It also expresses K19, and
it normally does not contain Merkel cells.
4. Answer: C (Ref. Orban’s Oral Histology & Embryology, By GS Kumar, 2014, page no. 256)
Very important:
• The hard palate is formed by the palatine processes of the maxillae and the horizontal plates of the palatine bones.
• The hard palate is bounded in front and at the sides by the tooth-bearing alveolus of the upper jaw and is continuous
posteriorly with soft palate. It is covered by thick mucosa covered bound tightly to the underlying periosteum.
• In its more lateral regions, it also possesses a submucosa containing the main neurovascular bundle. The mucosa is covered
by keratinized stratified squamous epithelium, which shows regional variations and may be ortho- or parakeratinized.
• The periphery of the hard palate consists of gingivae; a narrow ridge, palatine raphe, devoid of submucosa, runs
anteroposteriorly in the midline.
• An oval prominence, the incisive papilla, lies at the anterior extremity of the raphe. It covers the incisive fossa at the oral
opening of the incisive canal and also marks the position of the fetal nasopalatine canal.
5. Answer: D
6. Answer: A (Ref. Orban’s Oral Histology & Embryology, By GS Kumar, 2014, page no. 254)
Merkel cells are found in the top layer of the skin. These cells are very close to the nerve endings that receive the sensation
of touch. Merkel cell carcinoma, also called neuroendocrine carcinoma of the skin or trabecular cancer, is a very rare type of
skin cancer that forms when Merkel cells grow out of control.
7. Answer: A (Ref. Orban’s Oral Histology & Embryology, By GS Kumar, 2014, page no. 268)
The taste buds on the tongue sit on raised protrusions of the tongue surface called papillae. There are three types of lingual
papillae that contain taste buds present on the human tongue:
• Fungiform papillae – as the name suggests, these are slightly mushroom-shaped if looked at in longitudinal section. These
are present mostly at the dorsal surface of the tongue, as well as at the sides. Innervated by facial nerve.
• Foliate papillae – these are ridges and grooves toward the posterior part of the tongue found at the lateral borders.
Innervated by facial nerve (anterior papillae) and glossopharyngeal nerve (posterior papillae).

https://t.me/DentalBooksWorld
Chapter 10 • Oral Anatomy and Histology 929

• Circumvallate papillae – there are only about 10 to 14 of these papillae on most people, and they are present at the back of
the oral part of the tongue. They are arranged in a circular-shaped row just in front of the sulcus terminalis of the tongue.
They are associated with ducts of von Ebner’s glands, and are innervated by the glossopharyngeal nerve.
• The fourth type of papillae – the filiform papillae – are the most numerous but do not contain taste buds. They are
characterized by increased keratinization and are involved in the mechanical aspect of providing abrasion.
Salt, sweet, sour, and umami tastes causes depolarization of the taste cells, although different mechanisms are applied.
Bitter causes an internal release of Ca2+, no external Ca2+ is required.
8. Answer: B
9. Answer: A
10. Answer: C
11. Answer: B (Ref. Orban’s Oral Histology & Embryology, By GS Kumar, 2014, page no. 283)
• Junctional epithelium is a collar-like band of stratified squamous nonkeratinizing epithelium. It is three to four layers
thick in early life, but the number of layers increases with age to 10 or even 20 layers.
• In addition, the junctional epithelium tapers from its coronal end, which may be 10–29 cells wide to 1 or 2 cells wide at its
apical termination, which is located at the cementoenamel junction in healthy tissue.
• These cells can be grouped in two strata: the basal layer that faces the connective tissue and the suprabasal layer that
extends to the tooth surface. The length of the junctional epithelium ranges from 0.25 to 1.35 mm.
• The junctional epithelium is formed by the confluence of the oral epithelium and the reduced enamel epithelium during
tooth eruption However, the reduced enamel epithelium is not essential for its formation; in fact, the junctional epithelium
is completely restored after pocket instrumentation or surgery, and it forms around an implant.
• The junctional epithelium is attached to the tooth surface (epithelial attachment) by means of an internal basal lamina.
• It is attached to the gingival connective tissue by an external basal lamina that has the same structure as other epithelial–
connective tissue attachments elsewhere in the body.
• The internal basal lamina consists of a lamina densa (adjacent to the enamel) and a lamina lucida to which
hemidesmosomes are attached. Hemidesmosomes have a decisive role in the firm attachment of the cells to the internal
basal lamina on the tooth surface.
12. Answer: C
13. Answer: A
14. Answer: D
15. Answer: B (Ref. Orban’s Oral Histology & Embryology, By GS Kumar, 2014, page no. 238)
Squamous cells are flattened, pancake-like cells, and when in layers are known as stratified. The stratified squamous cells
lining the mucosa of the oral cavity are similar to those lining the outer skin, although less keratinized. They do, however,
flake off continuously into the mouth. Simple squamous tissue would be too thin and not allow for the constant flaking and
replacement. Columnar epithelium is found in both the respiratory system and other organs of the digestive system.
16. Answer: D (Ref. Orban’s Oral Histology & Embryology, By GS Kumar, 2014, page no. 239)
The gingival masticatory mucosa is characterized by keratinization, thin or absent submucosa, and a firm, direct binding of
the lamina propria to the periosteum. In general, the submucosa is thicker in the lining mucosa. Note that submucosa, when
present, is always found beneath lamina propria, and is never between epithelium and lamina propria. If all four tissues
are present in a digestive mucosa, the order from the outside will be: epithelium, lamina propria, muscularis mucosae,
submucosa.
17. Answer: B (Ref. Orban’s Oral Histology & Embryology, By GS Kumar, 2014, page no. 255)
Masticatory mucosa consists of the gingiva and lining of the hard palate. Most of the other areas of the oral cavity are lined
by lining mucosa. Specialized mucosa is found on the dorsum of the tongue and contains taste buds and various papillae.
The characteristics of masticatory mucosa are keratinization, thin or absent submucosa, and tight binding of the lamina
propria to the underlying periosteum. Lining mucosa is generally nonkeratinized, with a thin lamina propria. Near the teeth,
the gingiva and lining mucosa (alveolar mucosa) meet at the mucogingival junction.

OCCLUSION
1. Answer: D
The MB, ML, and DB cusps of the maxillary molars are the largest, and form the primary cusp triangle, or trigon, of the
tooth. The distolingual cusp is the smallest, and is not part of the trigon. It is sometimes referred to as the talon or talon
cusp. It is most noticeable in the first molar, less noticeable in the second, and often absent in the third molar. The maxillary

https://t.me/DentalBooksWorld
930 Triumph’s Complete Review of Dentistry

third molar therefore often does not have the rhomboid appearance of a maxillary molar, and the occlusal aspect will appear
triangular or heart-shaped instead.
2. Answer: C
Contact is re-established in proximal areas when Class II restorations are completed. This restoration of contact at the
correct contact level prevents food from wedging in between teeth and causing buildup of plaque and debris. This, in turn,
protects the periodontal ligament, because the chief etiology of periodontal bone loss is calculus and plaque accumulation.
Stability of the arch is maintained and tooth drifting is also minimized. Without contact, teeth may drift mesially or distally
into the space left by the faulty restoration. Proximal contact is not considered to be related to filling material retention. This
is because each restoration’s retention is supposed to depend solely on the retentive properties of that individual tooth, tooth
preparation, and material. In other words, retention stands alone within the individual tooth.
3. Answer: B (Ref. Wheeler’s Dental Anatomy, Physiology and Occlusion, Stanley Nelson, 2014, page no. 469)
The mesiolingual cusp of a maxillary third molar is a holding cusp. The general rule for maxillary holding cusps is as follows:
a maxillary holding cusp contacts the distal marginal ridge of its mandibular counterpart and the mesial marginal ridge of
the mandibular tooth distal to its counterpart, except for the mesiolingual cusps of the molars, which contact the central
fossae of their counterparts. This should be the central fossa of the counterpart, the mandibular third molar.
4. Answer: A (Ref. Wheeler’s Dental Anatomy, Physiology and Occlusion, Stanley Nelson, 2014, page no. 44)
Over time, posterior teeth lose small amounts of enamel as the contact points wear against each other during chewing. The
contacts become slightly broader, the teeth themselves become slightly closer, through drifting, and the embrasures between
the teeth become slightly smaller. Diastemas (choice B) are not created, because the teeth drift to close any space created
by the attrition. This process does not affect crown length (choices C and D), which is occluso cervical in direction. It only
affects the mesiodistal dimension.
5. Answer: C (Ref. Wheeler’s Dental Anatomy, Physiology and Occlusion, Stanley Nelson, 2003, page no. 399)
The names for the fossa include mandibular, articular, glenoid, and temporal. The anterior border of the fossa is the articular
eminence of the temporal bone, and the posterior border is the tympanic section of the temporal bone. Slightly more
posterior is the mastoid process and associated styloid process. So the entire eminence is a temporal bone feature.
6. Answer: A
In general, rounded roots are primarily rotated, including the maxillary central incisors and maxillary canines. All double-
and triple-rooted teeth cannot be rotated and must be luxated in the buccal and lingual directions. Mesial–distal motion
is normally ruled out because of the existence of adjacent teeth. It is used in rare cases where there is no mesial and distal
adjacent tooth. The maxillary first premolar is invariably double rooted with a buccal and lingual root, and can never be
rotated without breaking the crown off the root.
7. Answer: D
Because of the shift of the mandibular teeth in a mesial direction due to the smaller size of the anterior teeth, most teeth
have two opposing teeth. That is, they oppose their counterpart in the other arch, and a tooth either mesial or distal to that
counterpart. There are two classes of exceptions. One of the exceptions is the mandibular central incisors. These two teeth
(#24 and #25) oppose only the maxillary centrals (#8 and #9). The other exception is the maxillary third molars (#1 and
#16), which oppose only the mandibular third molars. Note that the mandibular third molars oppose both the maxillary
second and third molars.
8. Answer: A
The masseteric sling is a powerful pair of muscles which wrap underneath the angle and the ramus of the mandible and act
to both support and close (elevate) the mandible. On the lateral surface of the mandible we find the masseter, while on the
medial surface we find the medial pterygoid. Although the temporalis (choice D) also strongly elevates, it is not part of this
structure. The lateral pterygoid (choice B) attaches to the condyle and the TMJ disc, and pulls the condyles forward and
laterally. The lateral pterygoid is not part of the masseteric sling.
9. Answer: A (Ref. Wheeler’s Dental Anatomy, Physiology and Occlusion, Stanley Nelson, 2003, page no. 239)
The mesiobuccal cusp of a mandibular molar is a holding (supporting) cusp. The general rule of occlusion of mandibular
holding cusps is as follows: the holding cusps of the mandibular teeth occlude on the mesial marginal ridge of their maxillary
counterpart, and the distal marginal ridge of the maxillary tooth mesial to their counterpart, except distobuccal cusps of
mandibular molars occlude with central fossae of their counterparts, the distal cusp of the mandibular first molar occludes
with the distal triangular fossa of its counterpart, and the first premolar occludes only with the mesial marginal ridge of
its counterpart (but not the canine). In this case, the maxillary counterpart is the maxillary second molar, and the tooth
immediately mesial to it is the maxillary first molar.
10. Answer: B
11. Answer: A

https://t.me/DentalBooksWorld
Chapter 10 • Oral Anatomy and Histology 931

Note that the holding (supporting, occluding) cusps of the posterior dentition are the lingual (palatal) cusps of the maxillary
teeth and the buccal (facial) cusps of the mandibular teeth. Buccals of maxillary teeth and linguals of mandibular teeth are
guiding cusps.
Choice A is the lingual of a mandibular first molar, so it is not a holding cusp, and is a guiding cusp.
Choice B is a buccal of a mandibular molar and is a holding cusp. Choice C is a lingual cusp of a maxillary molar and is a
holding cusp. Choice D is the lingual of a maxillary premolar and is a holding cusp. Choice E is the palatal (lingual) of a
maxillary premolar and is a holding cusp.
12. Answer: C (Ref. Wheeler’s Dental Anatomy, Physiology and Occlusion, Stanley Nelson, 2003, page no. 464)
The distolingual cusp of a maxillary molar is a holding cusp. The general rule for maxillary holding cusps is as follows: a
maxillary holding cusp contacts the distal marginal ridge of its mandibular counterpart and the mesial marginal ridge of
the mandibular tooth distal to its counterpart, except for the mesiolingual cusps of the molars, which contact the central
fossae of their counterparts. This should be the distal marginal ridge of the mandibular first molar and the mesial marginal
ridge of the second molar.
13. Answer: A (Ref. Wheeler’s Dental Anatomy, Physiology and Occlusion, Stanley Nelson, 2003, page no. 239)
Lingual cusps of mandibular teeth are guiding cusps, not holding cusps. Therefore, they do not occlude on marginal ridges
or central fossae.
14. Answer: D (Ref. Wheeler’s Dental Anatomy, Physiology and Occlusion, Stanley Nelson, 2014, page no. 87)
A small incisal embrasure will be located wherever the contact points are high (incisal) and where proximal–incisal line
angles are almost perpendicular. The best example of this arrangement is the central incisors. Their mesial–incisal line angles
are very square, and the contact is in the incisal third of the tooth. As you go further distally, the line angles become more
rounded, especially for canines. These rounded line angles at the incisal, as well as the more apical contacts, dropping to the
middle third as you reach the distal of the canine, ensure much larger incisal embrasures.
15. Answer: A (Ref. Orban’s Oral Histology and Embryology, By GS Kumar, 2014, page no. 442)
As the tooth crown nears eruption, the ameloblasts produce their final product, known as the inner enamel cuticle. This
material is acellular, and is firmly adherent to the enamel surface as the tooth erupts. Its older name is Nasmyth’s membrane.
This cuticle has on its outside a second acellular layer formed from the keratinized remnants of the dental sac. This adherent
double layer may persist for some time after the tooth erupts but is eventually worn away, leaving the enamel exposed in the
oral cavity.
16. Answer: D (Ref. Wheeler’s Dental Anatomy, Physiology and Occlusion, Stanley Nelson, 2014, page no. 87)
As a general rule, the buccal heights of contour of the premolars are located within the cervical thirds. The lingual heights of
contour will normally be more occlusal. On the mandibular first premolar, it is especially occlusal, within the occlusal third.
17. Answer: B
The general rule for innervation by CN V (the trigeminal) is that it innervates the muscles of mastication (temporalis, lateral
pterygoid, medial pterygoid, masseter), the mylohyoid, the anterior digastric, and the two tensors (tensor tympani and
tensor veli palatini). Note that the buccinator is not included. The buccinator is in the group of muscles of facial expression,
which are all innervated by CN VII (the facial nerve).
18. Answer: A
Mandibular incisors are widest near the incisal tip, and their contacts (both central and lateral) are always near the incisal
edge. The mandibular canine contact with the lateral incisor is thus in the incisal third. In the canine, the contact is also in
the incisal third, but not as incisal as in the lateral. The distal contact of the canine is also in the incisal third, but near the
junction of the incisal and middle thirds.
19. Answer: D (Ref. Wheeler’s Dental Anatomy, Physiology and Occlusion, Stanley Nelson, 2014, page no. 280)
In occlusion, cusps are defined as either holding (supporting) cusps or guiding cusps. Holding cusps, in central occlusion,
make contact with the opposing arch, establish vertical dimension of occlusion, and support the forces of occlusion. They
are the lingual cusps of the maxillary arch and buccal cusps of the mandibular. When the teeth are in centric occlusion, these
cusps are in contact. The other cusps (guiding) are not. The other cusps are either lingual to the contact (lingual cusps of
the mandibular arch) or buccal to the contact (buccal cusps of the maxillary arch). Holding cusps generally occlude in the
marginal ridge and central fossa areas of the opposing arch. Guiding cusps generally lie in embrasures between teeth, or
between cusps of the lingual or buccal surfaces of the opposing arch.
20. Answer: A (Ref. Wheeler’s Dental Anatomy, Physiology and Occlusion, Stanley Nelson, 2014, page no. 238)
The question depends on the fact that the maxillary first premolar has an asymmetric positioning of its two cusps. Rather
than having the buccal and lingual cusps in a straight line, the buccal cusp is displaced slightly distally, and the lingual cusp
is displaced slightly mesially. This gives the occlusal view of the tooth a slightly twisted appearance. Cusp ridges run from the
cusp tip to the proximal edge of the tooth at the marginal ridge. Therefore, if you look facially at the maxillary first premolar,

https://t.me/DentalBooksWorld
932 Triumph’s Complete Review of Dentistry

you will see the buccal cusp shifted toward the distal. This makes the cusp tip farther from the mesial, and thus there will be
a longer mesial cusp ridge on this tooth.
21. Answer: D
22. Answer: A
For the most part, rotation of a tooth during extraction works best on teeth with rounded roots, such as the maxillary central
incisor and the maxillary canine. It can never be used on double-rooted teeth.
23. Answer: D
24. Answer: D
25. Answer: C
The normal (Class I) jaw relationship will result in the mandibular first molar being located one cusp (one-half tooth)
mesial to the maxillary first molar. If the mandibular molar is located distal to the maxillary, it indicates a small mandible
(micrognathy) or another problem causing the maxillary teeth to be too far mesial and the mandibular too far distal. This
distocclusion is Angles Class II. Mesiocclusion is Angles Class III, where the mandibular first molar is more than one-half
tooth mesial to the maxillary first molar.
26. Answer: E (Ref. Wheeler’s Dental Anatomy, Physiology and Occlusion, Stanley Nelson, 2014, page no. 84)
The best way to answer the question is to imagine that the embrasures are spaces surrounding a small point of contact
between two incisors. This small contact point will have space above it, below it, in front of it, and in back of it. These spaces
in dental anatomy will be incisal, cervical (gingival), buccal (facial), and lingual (palatal), respectively. Note that there is no
space mesial or distal to the contact point. Immediately mesial or distal to the contact would be tooth structure.
27. Answer: B
28. Answer: E
The general rule for maxillary buccal cusps is that they occlude in the facial embrasures between their mandibular
counterparts and the teeth distal to their counterparts, except for the mesiobuccal cusps of the molars, which occlude in the
buccal grooves of their mandibular counterparts, and the distobuccal cusp of the first molar, which opposes the distobuccal
groove of the mandibular first molar. In this question, we are not dealing with the exception of the mesiobuccal cusps of the
maxillary molars or the distobuccal cusp of the maxillary first molar, so this cusp should follow the general rule and occlude
with the facial embrasure between its mandibular counterpart (the mandibular second molar) and the mandibular molar
distal to it (the mandibular third molar).
29. Answer: A (Ref. Orban’s Oral Histology and Embryology, By GS Kumar, 2014, page no. 87)
Attrition is the mechanical wearing of teeth due to physiologic processes, including chewing and bruxism. Abrasion (choice
B) is the mechanical wearing away of tooth structure due to some outside object, such as toothbrush abrasion at the cervical
lines of teeth due to hard sideways brushing, or from habitually holding bobby pins or nails with the teeth. Erosion (choice
C) is the chemical dissolving of tooth structure. It can be caused by a number of factors, including sucking on lemons and
other acidic fruit, excessive intake of acidic beverages (cola), or excessive vomiting, as in bulimia. Bulimics often exhibit
normal facial surfaces with severely eroded lingual surfaces.
30. Answer: B
31. Answer: B
In this case, the maxillary counterpart is the maxillary third molar, and the cusp should occlude in the central fossa of this
tooth. Remember that the mandibular third molar is located one-half tooth mesial to the maxillary.
32. Answer: A
As a rule, the mandible will move toward the side of injury when the lateral pterygoid is damaged. In this case, the right
condyle will not move because of the muscle damage. The left condyle moves forward, but because there is no movement
on the right side, the left condyle moves out and rotates to the right, with the nonmoving right condyle acting as a pivot
point. Damage to the lateral pterygoid will not affect elevation (closing), because the lateral pterygoid is not an elevator. The
elevators are the temporalis, medial pterygoid, and masseter.
33. Answer: D
34. Answer: A
In this question, we are not dealing with the exception of the distolingual cusps of mandibular molars, so in this case,
the mandibular third molar mesiolingual cusp contacts the lingual embrasure between its counterpart (the maxillary third
molar) and the tooth mesial to it (the maxillary second molar).
35. Answer: E
36. Answer: C
37. Answer: C

https://t.me/DentalBooksWorld
Chapter 10 • Oral Anatomy and Histology 933

Protrusive movement is defined as moving the mandible outward, away from the head. If you are in centric occlusion,
you will protrude to move toward an edge-to-edge position. The lateral pterygoid moves both condyles forward when it
contracts, so the initial protrusive movement is forward. When the condyle contacts the articular eminence in the glenoid
fossa, it cannot move directly forward anymore and begins to glide downward along the surface of the fossa.
38. Answer: A
Buccal cusps of maxillary teeth are guiding cusps, not holding cusps. Therefore, they do not occlude on marginal ridges
or central fossae. The general rule for maxillary buccal cusps is that they occlude in the facial embrasures between their
mandibular counterparts and the teeth distal to their counterparts, except for the mesiobuccal cusps of the molars, which
occlude in the buccal grooves of their mandibular counterparts, and the distobuccal cusp of the first molar, which opposes
the distobuccal groove of the mandibular first molar. In this question, we are dealing with the exception of the mesiobuccal
cusps of maxillary molars, which occlude with the buccal grooves of their mandibular counterparts. In this case, the
maxillary second molar mesiobuccal cusp contacts the mandibular second molar buccal groove (its counterpart).
39. Answer: E
The distolingual cusp of a maxillary third molar is theoretically a holding cusp; however, it is often missing on this tooth.
The general rule for maxillary holding cusps is as follows: a maxillary holding cusp contacts the distal marginal ridge of
its mandibular counterpart and the mesial marginal ridge of the mandibular tooth distal to its counterpart, except for
the mesiolingual cusps of the molars, which contact the central fossae of their counterparts. This should be the distal
marginal ridge of the mandibular third molar and no mesial marginal ridge, because there is no tooth distal to the third
molar. So the answer is either the cusp does not exist, or it contacts only the distal marginal ridge of the mandibular third
molar.
40. Answer: D
As you move from centric occlusion to edge-to-edge position, the mandibular teeth separate from the maxillary and a space
is created between the two arches. Try this yourself, as you protrude and slide the mandible forward. The anterior teeth
act as a guide as you slowly separate from occlusal contact until only incisal edges of incisors contact. At this point, the
increased space between the arches adds to vertical dimension. There is no vertical overlap (overbite; choice C) nor there
is any horizontal overlap (overjet; choice B) at this point. So protrusion to edge-to edge increases vertical dimension while
decreasing both vertical and horizontal overlap.
41. Answer: D
While theoretically it may appear that the correct answer is choice A, most authors state that there is no contact with the
maxillary canine distal marginal ridge, and the canine remains slightly out of contact but near the area of the buccal cusp
of the mandibular first premolar. The general rule of occlusion of mandibular holding cusps is as follows: the holding cusps
of the mandibular teeth occlude on the mesial marginal ridge of their maxillary counterpart, and the distal marginal ridge
of the maxillary tooth mesial to their counterpart, except distobuccal cusps of mandibular molars occlude with central
fossae of their counterparts, the distal cusp of the mandibular first molar occludes with the distal triangular fossa of its
counterpart, and the first premolar occludes only with the mesial marginal ridge of its counterpart (but not the canine).
42. Answer: D (Ref. Wheeler’s Dental Anatomy, Physiology and Occlusion, Stanley Nelson, 2003, page no. 239)
The buccal cusp of a mandibular canine is not considered to be a holding (supporting) cusp. Therefore, the general rule
of occlusion of mandibular holding cusps does not apply. The canine, as an anterior tooth, will normally be slightly out of
contact between its maxillary counterpart and the tooth immediately mesial to it. In this case, the maxillary counterpart is
the maxillary canine, and the tooth mesial to it is the lateral incisor.
43. Answer: B (Ref. Orban’s Oral Histology and Embryology, By GS Kumar, 2014, page no. 242)
The lamina propria is a loose connective tissue located within the mucosal layer, just underneath the epithelium. In the oral
cavity, it will be found below the outer layer of stratified squamous epithelium. The lamina propria in the oral cavity often
forms wedge-like extensions into concavities in the epithelium, known as connective tissue papillae. The corresponding
epithelial extensions into the connective tissue are known as rete pegs.
44. Answer: C
Increased cementum production (hypercementosis) is a common X-ray finding. Its cause is not completely known, but
it seems to occur more often in teeth that have lost function and/or are supererupted, rather than in teeth with excessive
function (occlusal trauma). Occlusal trauma is recognized clinically by heavy contact (using articulating paper) and high
mobility. It is recognized radiographically by a widened PDL space and possibly with accompanying bone resorption. Note
that in loss of function, teeth often have a narrowed PDL.
45. Answer: D
46. Answer: D

https://t.me/DentalBooksWorld
934 Triumph’s Complete Review of Dentistry

To answer this question, place your own teeth in centric occlusion (maximum intercuspation). Notice that the occlusion is
held in place by the lingual cusps of maxillary teeth and the buccal cusps of mandibular teeth. Mandibular lingual cusps are
not in contact. In protrusion, the anterior teeth act to separate the maxilla and mandible as the mandible moves forward.
The anterior teeth act as guides in retrusion as well (try it). So far the mandibular lingual cusps are not involved. If you make
a left working movement (slide your mandible left), notice that the mandibular lingual cusps on the left drag against the
lingual side of the maxillary lingual cusps. In the nonworking side (in this case, the right side), the mandibular lingual cusps
move away from contact.
47. Answer: C (Ref. Wheeler’s Dental Anatomy, Physiology and Occlusion, Stanley Nelson, 2014, page no. 87)
As is generally the case, the distal contact of this tooth is more cervical than is the mesial contact. On the mesial side, where
it contacts the central incisor, the contact will be near the junction of incisal and middle thirds. However, on the distal side, it
is slightly more cervical, usually around the middle of the middle third. This is the contact with the maxillary canine. On the
canine, this height of contour is at the junction of the incisal and middle third.
48. Answer: E
49. Answer: D
Isomorphy refers to a close similarity in morphology and crown shape between two teeth. The most notable examples of
isomorphy are the primary second molars with the permanent first molars in both arches. Thus, by looking at a primary
second molar in either arch, you can very accurately predict the morphology of the permanent first molar in that same arch.
Isomorphy does not exist between primary first and second molars, as both first molars are very unusual in shape and both
second molars closely resemble permanent first molars.
50. Answer: A (Ref. Wheeler’s Dental Anatomy, Physiology and Occlusion, Stanley Nelson, 2003, page no. 461)
Lingual cusps of mandibular teeth are guiding cusps, not holding cusps. Therefore, they do not occlude on marginal ridges
or central fossae. The general rule for mandibular lingual cusps is that they occlude in the lingual embrasures between
their maxillary counterparts and the teeth mesial to their counterparts, except for the distolingual cusps of the mandibular
molars, which occlude in the lingual grooves of their maxillary counterparts. In this question, we are not dealing with the
exception of the distolingual cusps of mandibular molars, so in this case, the mandibular first molar mesiolingual cusp
contacts the lingual embrasure between its counterpart (the maxillary first molar) and the tooth mesial to it (the maxillary
second premolar).
51. Answer: D
Remember that the mandibular teeth, being smaller, are set one-half tooth mesial to the corresponding maxillary tooth. That
is, the maxillary central contacts the mandibular central and half of the mandibular lateral. The maxillary lateral contacts
both the mandibular lateral and canine. Note that the maxillary canine cusp tip sits in the embrasure between the mandibular
canine and first premolar, although the incisors are the main contacts in protrusive movement.
52. Answer: B
The Carabelli cusp (trait) is a variable cusp or extension of the mesiolingual cusp of the maxillary first molar. Its expression
varies from that of a full cusp, at its largest, to a small protuberance at its smallest. Its expression varies among various races
and ethnic groups. Some anatomists describe it as a cusp, making the maxillary first molar a five-cusped tooth. Others
consider it a variety or trait of a four-cusped tooth. It is sometimes missing, and a groove or pit is found in the normal
Carabelli cusp region of the mesiolingual cusp.
53. Answer: B
The vertical dimension of occlusion (VDO) is a vertical measurement on the front of the face when the teeth are in full
occlusion (centric occlusion). When the face is at rest, the teeth are slightly apart, and the vertical dimension of the front
of the face is slightly longer. This is the vertical dimension of rest (VDR). The distance between the teeth at this point is the
freeway space (FS). So when we take the smaller VDO and add the few millimeters of the FS, we get the slightly longer VDR.
VDR is generally 2–5 mm more than VDO.
54. Answer: D
The temporalis is a strong, broad, flat muscle which can both elevate (close) and retrude (pull back) the mandible. This is
due to the fact that it contains both vertical and nearly horizontal muscle fibers. The vertical fibers pull the mandible straight
upward (elevation), while the horizontal fibers pull straight backward (retrusion). The most anterior fibers of the temporalis
are the vertical. As you move posteriorly, they become first diagonal, and then horizontal. NBDE questions in the past have
referred to this muscle as being an elevator in the anterior and an elevator/retruder in the posterior.
55. Answer: A
The mesiolingual cusp of a maxillary second molar is a holding cusp. The general rule for maxillary holding cusps is as
follows: a maxillary holding cusp contacts the distal marginal ridge of its mandibular counterpart and the mesial marginal

https://t.me/DentalBooksWorld
Chapter 10 • Oral Anatomy and Histology 935

ridge of the mandibular tooth distal to its counterpart, except for the mesiolingual cusps of the molars, which contact the
central fossae of their counterparts. This should be the central fossa of the counterpart, the mandibular second molar.
56. Answer: A (Ref. Wheeler’s Dental Anatomy, Physiology and Occlusion, Stanley Nelson, 2003, page no. 116)
Mandibular central incisors contact at the most incisal point found on any tooth. The contact is located just below the incisal
edge. On the distal surface, it is still incisal and near the edge, but is slightly more cervical than the contact found on the
mesial side of the tooth.
57. Answer: C (Ref. Wheeler’s Dental Anatomy, Physiology and Occlusion, Stanley Nelson, 2003, page no. 268)
The question may be answered on a general principle, in that the third molars are often the most unpredictable teeth in either
arch. The specific patterns referred to here are the pit-groove patterns. In third molars, you will often find supplemental
grooves at right angles to the main grooves, and additional pits and fissures not normally seen in first and second molars.
The crenulated pattern refers to a highly grooved overall occlusal surface with grooves running in all directions, leaving a
lacy, nook-and-cranny occlusal surface.
58 Answer: A
By definition, a working movement occurs when the mandibular teeth on one side move laterally across the surfaces of the
maxillary teeth toward their own side. In other words, in a left working movement, left mandibular cusps move laterally
left across the cusps of the maxillary left teeth. The other side is known as the nonworking (balancing) side. In this case, the
balancing side is the right side. In a left working movement, the balancing side also moves left, as the mandible cannot move
both left and right at the same time.
59. Answer: A (Ref. Wheeler’s Dental Anatomy, Physiology and Occlusion, Stanley Nelson, 2003, page no. 444)
The mesiobuccal cusp of a mandibular molar is a holding (supporting) cusp. The general rule of occlusion of mandibular
holding cusps is as follows: the holding cusps of the mandibular teeth occlude on the mesial marginal ridge of their maxillary
counterpart, and the distal marginal ridge of the maxillary tooth mesial to their counterpart, except distobuccal cusps of
mandibular molars occlude with central fossae of their counterparts, the distal cusp of the mandibular first molar occludes
with the distal triangular fossa of its counterpart, and the first premolar occludes only with the mesial marginal ridge of
its counterpart (but not the canine). In this case, the maxillary counterpart is the maxillary third molar, and the tooth
immediately mesial to it is the maxillary second molar.
60. Answer: D
The central fossa, containing a central pit, is located within the main cusp triangle (trigon) of the maxillary first molar. The
three major cusps: MB, ML, and DB, surround the fossa and form the trigon, or primitive cusp triangle. The DL cusp (talon)
is not part of the trigon, does not surround the central fossa, and is the smallest cusp of the tooth (excluding the Carabelli
cusp or trait).
61. Answer: A
In a left working movement, the mandible shifts to the left. From a central position, the left side of the mandible is moving
away from center (laterally). The right side of the mandible, although also moving left, is moving toward the center
(medially). The left TMJ purely rotates when the right is rotating and translating mechanically. Note that it is impossible for
both sides of the mandible to move either laterally or medially at the same time.
62. Answer: B
In a left working movement, the mandible moves toward the left. The left side is then known as the working side and the
right side as the nonworking (balancing) side. The lateral pterygoids pull the mandibular condyles forward, so the pull of
both together results in protrusion. For only a left-sided movement, the left lateral pterygoid does not contract and the left
condyle stays in a relatively unchanging position (it does rotate slightly). The right lateral pterygoid contracts and pulls the
right side of the mandible outward, and then it turns left, as the left side of the mandible is stationary. Remember that the
right lateral pterygoid moves the mandible left, and the left lateral pterygoid moves the mandible right.
63. Answer: D
64. Answer: A (Ref. Wheeler’s Dental Anatomy, Physiology and Occlusion, Stanley Nelson, 2003, page no. 217)
The mesial marginal groove, extending from the marginal ridge, along the mesial side of the crown into the proximal area
(mesial concavity) and down into the mesial root concavity, is a characteristic of many maxillary first premolars. This has
implications for both calculus removal (scaling and root planing) and restoration (placement of well adapted matrix bands).
It is not found in the other premolars, although they may sometimes have some less pronounced root concavities. Root
concavities are less common and less pronounced in the mandibular premolars.
65. Answer: A

https://t.me/DentalBooksWorld
936 Triumph’s Complete Review of Dentistry

66. Answer: B (Ref. Wheeler’s Dental Anatomy, Physiology and Occlusion, Stanley Nelson, 2003, page no. 399)
The Posselt’s envelope of motion is a tracing of the extreme border movements of the mandible. It shows how far the
mandible can protrude, retrude, open, and close, and all motions connecting these points. Protruded contact position is
caused by protruding the mandible as far as possible. This will normally push the mandible past edge-to-edge, until the
mandible appears to be in a Class III relationship. As the mandible protrudes, anterior tooth contact causes separation of the
teeth out of occlusion
67. Answer: B
68. Answer: E (Ref. Wheeler’s Dental Anatomy, Physiology and Occlusion, Stanley Nelson, 2003, page no. 461)
Lingual cusps of mandibular teeth are guiding cusps, not holding cusps. Therefore, they do not occlude on marginal ridges
or central fossae. The mandibular first premolar lingual cusp contacts the lingual embrasure between its counterpart (the
maxillary first premolar) and the tooth mesial to it (the maxillary canine).

GENES IN TOOTH DEVELOPMENT


1. FGF-8, LHX-6/7, GSC
2. Answer: A
3. SHH
4. MSX gene and DLX gene
5. DLX-1, DLX-2, BARX-1
6. MSX-1, MSX-2, DLX-2
7. LEF-1 in oral epithelium
8. MSX1, MSX2, DLX1, DLX2
9. Microdontia, macrodontia, Peg lateral, dens in dente
10. Hypoplasia, concrescence

Gene Mutation Tooth phenotype Reference


MSX1, MSX2 Double mutant Initiation stage arrest Bei and Maas (1998)
DLX1, DLX2 Double mutant Initiation stage arrest Thomas et al. (1997)
FGF8 FGF8flox Initiation stage arrest Trumpp et al. (1999)
LHX6/LHX7 Double mutant Initiation stage arrest Grigoriou et al., (1998)
APITX2 Null Initiation stage arrest Liu et al. (2003)
GLI2, GLI3 Double mutant Initiation stage arrest Hardcastle et al. (1998),
P63 Null Initiation stage arrest Yang et al. (1999)
DKK1 K14 transgenic Initiation stage arrest Andl T et al. (2002)
PAX9 Null Bud stage arrest Peters et al. (1998)
LEF1 Null Bud stage arrest Van genderen et al. (1994)
MSX1 Null Bud stage arrest Satokata and Maas (1994)
RUNX2 Null Bud stage arrest Aberg et al. (2004)
BARX1 Null Bud stage arrest Tucker et al. (1998)
BMPR1A K14 transgenic Bud stage arrest Andl et al. (2004)
FGFR2B Null Bud stage arrest De Moerlooze et al. (2000)
SHH K14 conditional KO Bud stage arrest Dassule et al. (2000)
Noggin K14 TG Bud stage arrest Plikus et al. (2005)

https://t.me/DentalBooksWorld
Chapter 10 • Oral Anatomy and Histology 937

Activin βA Null Bud stage arrest, lack incisors and Matzuk et al. (1995)
mandibular molars
CTIP2 Null Late bell stage defect Golonzhka et al. (2009)
GLI2 Null Abnormal maxillary incisor Hardcastle et al. (1998)
GLI3 Heterozygous Maxillary incisor development Hardcastle et al. (1998),
arrested as a rudimentary epithelium Mo et al. (1997)
thickening
EDA Tabby encode eda Small enamel knot Tucker et al.(2000)
EDAR Downless Absent enamel knot, disorganized Headon and Overbeek
enamel rope (1999)
FGF10 Null Smaller tooth germ, cervical loops of Harada et al. (2002)
the incisors are hypoplastic
WNT/β catenin K14 conditional KO Misshapen tooth bud, ectopic teeth Liu et al. (2008)
Ectodin/SOSTDC1/ Null Supernumerary teeth, enlarge enamel Kassai et al. (2005)
wise knot, abnormal cusp
APC K-14CRE;APCCKO/CKO Supernumerary teeth Kuraguchi et al. (2006)
SP6 Null Supernumerary teeth Nakamura et al. (2008)
LRP4 Null Supernumerary teeth Johnson et al. (2005)
IFT88/polaris Null Supernumerary teeth Liu et al., (2005)
GAS1 Null Supernumerary teeth Ohazama et al., (2009)
OSR2 Null Supernumerary teeth Zhang et al.(2009)
Sprouty2,4 Null Supernumerary teeth Klein et al. (2006)

Dental Placode Bud Cap Bell Erupting tooth


Secondary
enamel knot
Primary Enamel
enamel knot Dentin
Ameloblast
Cementum
Odontoblast Jaw bone
Pulp

Initiation Morphogenesis Cytodifferentiation Matrix secretion

Dlx1-/-,Dlx2-/- Activin BA-/- Msx2-/- DMP1-/- Shh (ptcmes)


Gli2-/-,Gli3-/- Lef1-/- Sp6-/- Dspp-/- Nfi-c/CTF-/-
Msx1-/-,Msx2-/- Msx1-/- Sp3-/- Cp2-/- Msx2-/-
Pitx2-/- Pax9-/- Gdnf-/- Shh K14cKO sp6-/-
P63-/- Runx2-/- Amelx-/- Smoothened K14cKO Noggin K14 TG
Lhx6-/-,Lhx7-/- Barx1-/- Ameloblastin-/- TGFϐ1 dspp cKO
Dkk1 K14 TG Fgfr2b-/- Lama3-/- Eda K14 TG
Fgf8flax Bmpr1a k14 TG Enamelin-/- Fst K14 TG
Shh K14 cKO Mmp20-/- Wnt K14 TG
Noggin K14 TG Periostin-/- Connexin 43
Tbx1-/- Sprouty4-/-,sprouty2+/-
Noggin K14 TG

https://t.me/DentalBooksWorld
938 Triumph’s Complete Review of Dentistry

Gene Mutation Tooth phenotype Reference


Enamel defect
MSX2 Null Enamel hypoplasia Satokata et al. (2000)
LAMA3 Null Enamel hypoplasia Ryan et al. (1999)
SP3 Null Enamel hypoplasia Bowman et al. (2000)
SP6 Null Enamel hypoplasia Nakamura et al. (2008)
Smoothened K14 conditional KO Enamel hypoplasia Gritli-Linde et al. (2002)
GDNF Null No enamel deVicente et al. (2002)
Periostin Null Incisor enamel defect Rios et al. (2005)
TGFB1 DSPP conditional KO Enamel hypoplasia Haruyama et al. (2006)
EDA K14 transgenic No enamel Mustonen et al. (2004)
Follistatin K14 transgenic No enamel Wang et al. (2004)
Follistatin Null Ectopic enamel Wang et al. (2004)
WNT3 K14 transgenic No enamel Millar et al. (2003)
AMELX Null Enamel hypoplasia Gibson et al. (2001)
Ameloblastin Null No enamel Fukumoto et al. (2005)
TBX1 Null Enamel free teeth Caton et al. (2009)
Enamelin Null Enamel hypoplasia/aplasia Hu et al. (2008)
MMP20 Null Enamel hypoplasia Caterina et al. (2002)
Connexin 43 Dominant negative Enamel hypoplasia Dobrowolski et al. (2008)
Sprouty2,4 Spry2+/−, Spry4−/− Ectopic enamel Klein et al. (2008)
Periostin Null Thinner enamel layer, Rios et al. (2005)
Noggin K14 transgenic Abnormal ameloblast Plikus et al.(2005)
Dentin defect
DSPP Null Dentinogenesis imperfecta Thyagarajan et al. (2001)
DMP1 Null Abnormal dentin tubule system Lu et al. (2007)
MSX2 Null Dentinogenesis imperfecta Aioub et al. (2007)
SP6 Null Abnormal dentin structure Nakamura et al. (2008)
SP3 Null Dentin defect Bowman et al. (2000)
Noggin K14 transgenic Abnormal dentinoblast Plikus et al.(2005)
Root defect
MSX2 Null Root malformation Satokata et al. 2000
mes
SHH Ptc Shorter root Nakatomi et al. (2006)
NFI-C/CTF Null Lacking root Steele-perkins et al. (2003)
SP6 Null Defect in root formation Nakamura et al. (2008)
Noggin K14 transgenic Failed to form multiple root Plikus et al.(2005)

SHORT-ANSWER QUESTIONS
1. 4
Permanent first molars and mandibular 2nd Pm have 3 – cusp type, developing from 5 lobes
2. Mandibular canine
3. Maxillary lateral (lingual pit and lingual groove)
4. Mesial than distal and maxillary than mandibular

https://t.me/DentalBooksWorld
Chapter 10 • Oral Anatomy and Histology 939

5. II, II, IM, MM, MM, MM, MM, M


6. IJ, JM, JM, MM, MM, MM, MM, M
I: Incisal 1/3
J: Junction of incisal and middle 1/3s
M: Middle 1/3
7. Canine to third molar
8. False. The mesial side is usually bigger
9. 97%
10. Maxillary first molar (facial wider) and between mandibular centrals (same)
11. Mandibular anterior teeth
12. Mandibular posterior teeth
13. Trapezoid
14. Anterior: triangle
Max. post: trapezoid
Mand. post: rhomboid
15. Maxillary first Pm
16. Facial: Anterior and posterior at cervical 1/3, except for mandibular 1st molars (junction of cervical and middle 1/3)
Lingual: Anterior: cervical 1/3 Posterior: middle 1/3, except for mandibular 1st Pm (occlusal 1/3)
17. Facially and lingually
18. Maxillary lateral and canine
19. Maxillary canine and first Pm
20. Maxillary canine
21. Centered/lingual/labial to the long axis of the tooth lingual/centered
22. Centered/lingual/labial to the long axis of the tooth lingual
23. Centered/lingual/labial to the long axis of the tooth facial
24. Centered/lingual/labial to the long axis of the tooth centered
25. False. Mandibular teeth usually erupt before maxillary
26. 1) 3rd molars
2) Maxillary lateral
27. Maxillary lateral
28. Mandibular central
29. Mandibular first Pm
30. Mandibular central
31. Maxillary first molar
32. Maxillary first molar
33. Mandibular first Pm
34. Max. central incisor
35. Mandibular canine
36. 1. Maxillary first Pm (mesial)
2. Primary maxillary canine
37. 1
38. 2
39. Mandibular second Pm
40. Mandibular second Pm
41. 2nd
42. Distally
Mandibular canine inclined mesially
43. 1.5
2.5
44. Mandibular lateral and canine
45. Mandibular central

https://t.me/DentalBooksWorld
940 Triumph’s Complete Review of Dentistry

46. Mandibular canine


47. 1) Max. canine
2) Mand. canine
48. True
49. 50%
50. Permanent mandibular molars
51. Max. 1st molar primary
2nd max molar
52. Mandibular first molar
53. 7–9

https://t.me/DentalBooksWorld
11 Dental Materials

SYNOPSIS

ADA SPECIFICATION NUMBERS


What is ADA and ANSI?
• The ADA (American Dental Association) is an ANSI (American National Standards Institute) accredited standards
developing organization. ADA standards have been approved as American National Standards by ANSI.
• The ADA is the accredited dental standards body of the ANSI and also designated the official United States representative
for the International Organization for Standardization (ISO) Technical Committee 106 Dentistry (TC 106).

Why is there a need for these ADA specification numbers?


Dental standards ensure that everyone is on the same page—those who design and manufacture dental products and the dentists
who use them. Through comprehensive analysis, the ADA establishes baseline standards and technical recommendations for
almost every tool of modern dentistry, from radiographic systems to sealants to manual toothbrushes.

ANSI/ADA SPECIFICATION NUMBERS

ADA Specification Numbers


ANSI/ADA Specification No. 1—Alloy for dental amalgam
ANSI/ADA Specification No. 2—Gypsum bonded investment
ANSI/ADA Specification No. 3—Impression compound
ANSI/ADA Specification No. 4—Inlay casting wax
ANSI/ADA Specification No. 5—Casting gold alloys
ANSI/ADA Specification No. 6—Dental mercury
ANSI/ADA Specification No. 7—Wrought gold wire
ANSI/ADA Specification No. 8—Zinc phosphate cement
ANSI/ADA Specification No. 9—Silicate cement
ANSI/ADA Specification No. 11—Agar impression material
ANSI/ADA Specification No. 12—Denture base polymers
ANSI/ADA Specification No. 15—Artificial teeth for dental prostheses
ANSI/ADA Specification No. 17—Denture base temporary relining resins
ADA Specification No. 18—Alginate impression materials
ANSI/ADA Specification No. 19—Dental elastomeric impression materials
ANSI/ADA Specification No. 22—IOPA film
ANSI/ADA Specification No. 23—Dental excavating burs
ANSI/ADA Specification No. 25—Dental gypsum products
ADA Specification No. 26—Dental X-ray equipment

https://t.me/DentalBooksWorld
942 Triumph's Complete Review of Dentistry

ADA Specification No. 27—Resin-based filling materials


ANSI/ADA Specification No. 28—Root canal files and reamers
ANSI/ADA Specification No. 30—Dental zinc oxide – Eugenol and zinc oxide – Non-eugenol cements
ANSI/ADA Specification No. 32—Orthodontic wires
ANSI/ADA Specification No. 33—Dental product standards development vocabulary
ADA Specification No. 34—Dental aspirating syringes
ANSI/ADA Specification No. 37—Dental abrasive powders
ANSI/ADA Specification No. 38—Metal–ceramic dental restorative systems
ANSI/ADA Specification No. 39—Pit and fissure sealants
ANSI/ADA Specification No. 41—Recommended standard practices for biological evaluation of dental materials
ANSI/ADA Specification No. 43—Electrically powered dental amalgamators
ADA Specification No. 44—Dental electrosurgical equipment
ANSI/ADA Specification No. 46—Dental patient chair
ANSI/ADA Specification No. 47—Dental units
ANSI/ADA Specification No. 48—Visible light-curing units
ANSI/ADA Specification No. 48-2—LED curing lights
ANSI/ADA Specification No. 53—Polymer-based crown and bridge materials
ANSI/ADA Specification No. 54—Double-pointed, parenteral, single-use needles for dentistry
ANSI/ADA Specification No. 57—Endodontic sealing material
ANSI/ADA Specification No. 58—Root-canal files, type H (Hedstrom)
ANSI/ADA Specification No. 62—Dental abrasive pastes
ANSI/ADA Specification No. 63—Root-canal barbed broaches and rasps
ANSI/ADA Specification No. 66—Glass ionomer cements
ANSI/ADA Specification No. 69—Dental ceramic
ANSI/ADA Specification No. 70—Dental X-ray protective aprons and accessory devices
ANSI/ADA Specification No. 71—Root-canal filling condensers (pluggers and spreaders)
ANSI/ADA Specification No. 73—Dental absorbent points
ANSI/ADA Specification No. 74—Dental operator’s stool
ANSI/ADA Specification No. 75—Resilient lining materials for removable dentures
ANSI/ADA Specification No. 76—Non-sterile natural rubber latex gloves for dentistry
ANSI/ADA Specification No. 78—Dental obturating cones
ANSI/ADA Specification No. 80—Dental materials – Determination of color stability
ANSI/ADA Specification No. 82—Dental reversible/irreversible hydrocolloid impression material systems
NSI/ADA Specification No. 85-Part 1—Disposable prophy angles
ANSI/ADA Specification No. 87—Dental impression trays
ANSI/ADA Specification No. 88—Dental brazing alloys
ANSI/ADA Specification No. 89—Dental operating lights
ANSI/ADA Specification No. 94—Dental compressed air triturator quality
ANSI/ADA Specification No. 95—Root-canal enlargers
ANSI/ADA Specification No. 96—Dental water-based cements
ANSI/ADA Specification No. 97—Corrosion test methods
ANSI/ADA Specification No. 99—Athletic mouth protectors and materials
ANSI/ADA Specification No. 100—Orthodontic brackets and tubes
ANSI/ADA Specification No. 101—Root-canal instruments

https://t.me/DentalBooksWorld
Chapter 11 • Dental Materials 943

ANSI/ADA Specification No. 102—Non-sterile nitrile gloves


ANSI/ADA Specification No. 103—Non-sterile poly vinyl chloride gloves for dentistry
ANSI/ADA Specification No. 105—Orthodontic elastomeric materials
ANSI/ADA Specification No. 108—Amalgam separators
ANSI/ADA Specification No. 109—Procedures for storing dental amalgam waste and requirements for amalgam waste
storage/shipment containers
ADA Technical Report No. 110—Standard procedures for the assessment of laser-induced effects on oral hard and soft tissue
ANSI/ADA Specification No. 113—Periodontal curettes, dental scalers, and excavators
ANSI/ADA Specification No. 116—Oral rinses
ANSI/ADA Specification No. 119—Manual toothbrushes
ANSI/ADA Specification No. 120—Powered toothbrushes
ANSI/ADA Specification No. 122—Dental casting and baseplate waxes
ANSI/ADA Specification No. 125—Manual interdental brushes
ANSI/ADA Specification No. 126—Casting investments and refractory die materials

TERMINOLOGY
• ANSI/ADA/ISO—Acronyms for organizations that administer or develop national and international standards. ANSI
is the national organization established for the purpose of accrediting and coordinating product standards development
activities in the United States. It is not a US government agency. The ADA is a national standards development organization
accredited by ANSI. ISO is a worldwide federation of national standard bodies. The results of ISO technical work are
published as International Standards. Efforts in the United States directed toward the development of ISO standards are
channeled through ANSI.
• Base—A material that is used to protect the pulp in a prepared cavity by providing thermal insulation; a base may also serve
as a medicament.
• Brittleness—The relative inability of a material to deform plastically before it fractures.
• C-Factor—Configuration factor. This represents the ratio between the bonded surface area of the resin-based composite
restoration to the non-bonded or free surface area. The greater the C-factor, the greater the deleterious effects to the
restoration.
• Cavity liner—A material that coats the bottom of a prepared cavity that protects the pulp.
• Cermet—A glass ionomer cement that is reinforced with filler particles prepared by fusing silver particles to form glass.
• Chroma—Degree of saturation of a particular hue (dominant color).
• Compomer—Resin-based composite consisting of a silicate glass filler phase and a methacrylate-based matrix with
carboxylic acid functional groups; also known as polyacid-modified GIC, a term derived from “composite” and “ionomer.”
• Compressive strength—The maximum stress a material can sustain under crush loading.
• Compressive stress—Compressive force per unit area perpendicular to the direction of the applied force.
• Coring—A microstructure in which a composition gradient exists between the center and the surface of cast dendrites,
grains, or particles.
• Creep—Time-dependent plastic strain of a solid under a static load or constant stress.
• Baseplate wax—Dental wax provided in a sheet form to establish the initial arch form in the construction of complete
dentures. It typically contains approximately 75% paraffin or ceresin wax, beeswax, or other waxes, and resins.
• Bite wax—A wax form used to record the occlusal surfaces of the teeth as an aid in establishing maxillo–mandibular
relationships.
• Boxing wax—A wax sheet form used as a border at the perimeter of an impression to provide an enclosed boundary for the
base of the cast to be made from a poured material such as gypsum or resin.
• Burnout—The process of heating an invested mold to eliminate the embedded wax or plastic pattern.
• Corrective wax (dental impression wax)—A thermoplastic wax that is used to make a type of dental impression.

https://t.me/DentalBooksWorld
944 Triumph's Complete Review of Dentistry

• Dental wax—(1) A low-molecular-weight ester of fatty acids derived from natural or synthetic components, such as
petroleum derivatives, that softens to a plastic state at a relatively low temperature. (2) A mixture of two or more waxes and
additives used as an aid for the production of gypsum casts, production of non-metallic denture bases, registering of jaw
relations, and laboratory work.
• Direct wax technique—A process whereby a wax pattern is prepared in the mouth directly on the prepared teeth.
• Divesting—The process of removing investment from a cast metal or hot-pressed ceramic.
• Elastic memory—The tendency of a solid wax form to partially return to its original shape when it is stored at a higher
temperature than that to which it was cooled.
• Flow—The relative ability of wax to plastically deform when it is heated slightly above the body temperature.
• Hygroscopic expansion—The amount of setting expansion that occurs when a gypsum-bonded casting investment is
immersed in water, which is usually heated to approximately 38°C. (See Chapter 9 for more information on this process.)
• Indirect wax technique—A procedure in which a wax pattern is prepared on a die.
• Inlay wax—A specialized dental wax that can be applied to dies to form direct or indirect patterns for the lost-wax technique,
which is used for the casting of metals or hot pressing of ceramics.
• Refractory—Capacity for sustaining exposure to a high temperature without significant degradation.
• Sprue—The mold channel through which the molten metal or ceramic flows into a mold cavity.
• Sprued wax pattern—A wax form consisting of the prosthesis pattern and the attached sprue network.
• Sticky wax—A type of dental wax that exhibits high adhesion to dry, clean surfaces when it is heated to a plastic condition.
• Amalgam—An alloy containing mercury.
• Amalgamation—The process of mixing liquid mercury with one or more metals or alloys to form an amalgam.
• Delayed expansion—The gradual expansion of a zinc-containing amalgam over a period of weeks to months. This
expansion is associated with the development of hydrogen gas, which is caused by the incorporation of moisture in the
plastic mass during its manipulation in a cavity preparation.
• Dental amalgam—An alloy that is formed by reacting mercury with silver, copper, and tin, and which may also contain
palladium, zinc, and other elements to improve handling characteristics and clinical performance.
• Dental amalgam alloy (alloy for dental amalgam)—An alloy of silver, copper, tin, and other elements that is processed in
the form of powder particles or as a compressed pellet.
• Marginal breakdown—The gradual fracture of the perimeter or margin of a dental amalgam filling, which leads to the
formation of gaps between the amalgam and the tooth.
• Trituration—The mixing of amalgam alloy particles with mercury in a device called a triturator; the term is also used to
describe the reduction of a solid to fine particles by grinding or friction.
• Acid etching—Use of an acidic chemical substance to prepare the tooth enamel and/or dentin surface to provide retention
for bonding.
• Adhesion—A state in which two surfaces are held together by chemical or physical forces or both with or without the aid
of an adhesive. Adhesion is one aspect of bonding.
• Adhesive—Any substance that joins or creates close adherence of two or more surfaces. Intermediate material that causes
two materials to adhere to each other.
• Alloy—Compound combining two or more elements having properties not existing in any of the single constituent elements.
Sometimes used to refer to an amalgam.
• Coping—A thin covering of the coronal portion of the tooth usually without anatomic conformity. It is a custom-made or
pre-fabricated thimble-shaped core or base layer designed to fit over a natural tooth preparation, a post core, or an implant
abutment so as to act as a substructure onto which other components can be added to give a final form to a restoration or
prosthesis. It can be used as a definitive restoration or as part of a transfer procedure.
• Core buildup—The replacement of a part or all of the crown of a tooth whose purpose is to provide a base for the retention
of an indirectly fabricated crown.
• Cosmetic dentistry—Those services provided by dentists solely for the purpose of improving the appearance when form
and function are satisfactory and no pathological conditions exist [Source: ADA policy “Cosmetic Dentistry” (1976, p. 850)].

https://t.me/DentalBooksWorld
Chapter 11 • Dental Materials 945

• Inlay—An intracoronal dental restoration, made outside the oral cavity to conform to the prepared cavity, which restores
some of the occlusal surface of a tooth, but does not restore any cusp tips. It is retained by luting cement (Source: American
College of Prosthodontics; The Glossary of Prosthodontic Terms).
• Laminate veneer—A thin covering of the facial surface of a tooth, usually constructed of tooth-colored material and used
to restore discolored, damaged, misshapen, or misaligned teeth.
• Porcelain/ceramic—Refers to pressed, fired, polished, or milled materials containing predominantly inorganic refractory
compounds including porcelains, glasses, ceramics, and glass-ceramics.
• Composite resin—A material composed of plastic with small glass or ceramic particles that is usually cured with filtered
light or chemical catalyst.
• Dental implant—A dental implant is a titanium cylinder surgically placed in the bone of the upper or lower jaw to provide
support for a dental restoration or appliance.
• Brittleness—The relative inability of a material to deform plastically before it fractures.
• Compressive stress—Compressive force per unit area perpendicular to the direction of the applied force.
• Compressive strength—Compressive stress at fracture.
• Ductility—The relative ability of a material to elongate plastically under a tensile stress. This property is reported
quantitatively as percent elongation.
• Elastic strain—The amount of deformation that is recovered instantaneously when an externally applied force or pressure
is reduced or eliminated.
• Elastic modulus (also modulus of elasticity and Young’s modulus)—The stiffness of a material, calculated as the ratio of
elastic stress to elastic strain.
• Flexural strength (bending strength or modulus of rupture)—Force per unit area at the instant of fracture in a test
specimen subjected to flexural loading.
• Flexural stress (bending stress)—Force per unit area of a material that is subjected to flexural loading.
• Fracture toughness—The critical stress intensity factor at the point of rapid crack propagation in a solid containing a crack
of known shape and size.
• Hardness—The resistance of a material to plastic deformation, which is typically produced by an indentation force.
• Malleability—The ability of a material to be hammered or compressed plastically into thin sheets without fracture.
• Percent elongation—The amount of plastic strain, expressed as a percent of the original length, which a tensile test specimen
sustains at the point of fracture (Ductility).
• Plastic strain—An irreversible deformation that remains when the externally applied force is reduced or eliminated.
• Pressure—Force per unit area acting on the surface of a material (compare with Stress).
• Proportional limit—The magnitude of elastic stress above which plastic deformation occurs.
• Resilience—The amount of elastic energy per unit volume that is sustained on loading and released upon unloading of a
test specimen.
• Shear stress—The ratio of shear force to the original cross-sectional area parallel to the direction of the applied force.
• Shear strength—Shear stress at the point of fracture.
• Stress—Force per unit area within a structure subjected to a force or pressure (see Pressure).
• Stress concentration—The area or point of significantly higher stress that occurs because of a structural discontinuity such
as a crack or pore or a marked change in dimension.
• Strain—Change in dimension per unit initial dimension. For tensile and compressive strain, a change in length is measured
relative to the initial reference length.
• Stress intensity (stress intensity factor)—The relative increase in stress at the tip of a crack of a given shape and size when
the crack surfaces are displaced in the opening mode (also Fracture Toughness).
• Strain hardening (work hardening)—The increase in strength and hardness and decrease in ductility of a metal that result
from plastic deformation.
• Strain rate—Change in strain per unit time during the loading of a structure.

https://t.me/DentalBooksWorld
946 Triumph's Complete Review of Dentistry

• Strength—(1) The maximum stress that a structure can withstand without sustaining a specific amount of plastic strain
(yield strength); (2) the stress at the point of fracture (ultimate strength).
• Tensile stress—The ratio of tensile force to the original cross-sectional area perpendicular to the direction of the applied
force.
• Tensile strength (ultimate tensile strength)—Tensile stress at the instant of fracture.
• Toughness—The ability of a material to absorb elastic energy and to deform plastically before fracturing; measured as the
total area under a plot of tensile stress versus strain.
• True stress—The ratio of applied force to the actual (true) cross-sectional area; however, for convenience, stress is often
calculated as the ratio of applied force to the initial cross-sectional area.
• Yield strength—The stress at which a test specimen exhibits a specific amount of plastic strain.
• Primary bonds—Ionic bonds, covalent bonds, and metallic bonds.
• Secondary bonds—Hydrogen bonds and Van der Waals bonds.
• Density—The measure of the weight of a material compared with its volume.
• Hardness—The resistance of a solid to penetration.
• Ultimate strength—The maximum amount of stress a material can withstand without breaking.
• Elasticity—The ability of a material to recover its shape completely after deformation from an applied force.
• Stiffness—A material’s resistance to deformation.
• Proportional limit—The greatest stress a structure can withstand without permanent deformation.
• Resilience—The resistance of a material to permanent deformation.
• Toughness—The ability of a material to resist fracture.
• Ductility—The ability of an object to be pulled or stretched under tension without rupture.
• Malleability—The ability of a material to be compressed and formed into a thin sheet without rupture.
• Edge strength—The strength of a material at the fine margins.
• Viscosity—The ability of a liquid material to flow.
• Thixotropic—The property of a liquid to flow more readily under mechanical force.
• Direct restorative material—The restorations placed directly into a cavity preparation.
• Indirect restorative material—Materials used to fabricate restorations outside the mouth that are subsequently placed into
the mouth.
• Mixing time—The amount of time allotted to bring the components of a material together in a homogenous mix.
• Working time—The time permitted to manipulate the material in the mouth.
• Initial set time—The time at which the material can no longer be manipulated in the mouth.
• Final set time—The time at which the material has reached its ultimate state.
• Chemical set materials—Materials that set through a timed chemical reaction with the combination of a catalyst and a
base.
• Light-activated materials—Materials that require a blue-light source to initiate a reaction.
• Dual set materials—Materials that polymerize by a chemical reaction when the material is mixed with a catalyst or initiated
by exposure to a blue light, or by a combination of chemical or light reaction.
• Shelf life—The useful life of a material before it deteriorates or changes in quality.

GYPSUM PRODUCTS
INTRODUCTION
• Gypsum is a naturally occurring white powdery mineral mined in various parts of the world, with chemical name calcium
sulfate dihydrate – CaSO4.2H2O.
• Gypsum is derived from a Greek word “Gypsas” (chalk).
Gypsum products used in dentistry are based on calcium sulfate hemihydrate (CaSO4.1/2H2O)

https://t.me/DentalBooksWorld
Chapter 11 • Dental Materials 947

The current ISO standard for dental gypsum products identifies five types of materials as follows:
• Type 1 Dental plaster, impression
• Type 2 Dental plaster, model
• Type 3 Dental stone, die, model
• Type 4 Dental stone, die, high strength, low expansion
• Type 5 Dental stone, die, high strength, high expansion

APPLICATION IN DENTISTRY
• For cast preparation
• Models and dies
• Impression material
• Investment material
• Mounting of casts
• As a mold material for processing of complete dentures

CHEMICAL AND PHYSICAL NATURE OF GYPSUM PRODUCTS


• As gypsum is the dihydrate form of calcium sulfate (CaSO4.2H2O), on heating, it loses 1.5 g mol of its 2 g mol of water and
is converted to calcium sulfate hemihydrate (CaSO4.1/2H2O).
• When calcium sulfate hemihydrate is mixed with water, the reverse reaction takes place and calcium sulfate hemihydrate is
converted back to calcium sulfate dihydrate.
• The reaction is exothermic.

MANUFACTURE OF DENTAL PLASTER AND STONE


• Formed by calcining of gypsum.
• Gypsum is grounded and subjected to heat 110–130°C to drive off a part of water of crystallization.
• As the temperature is raised further, the remaining water of crystallization is also removed and products are formed.
β-Hemihydrate (plaster)
• Gypsum is heated in a kettle, vat, or rotatory kiln open to air.
• Crystals – Spongy and irregular.
α-Hemihydrate (stone)
• Gypsum is heated to 125°C under steam pressure in an autoclave or boiled in a solution of CaCl2.
• Crystals – More dense and prismatic.

CLASSIFICATION
1. Depending on the method of calcination:
• Dental plaster or β-hemihydrate
• Dental stone or α-hemihydrate or hydrocal
• Dental stone, high strength, or densite
2. Other gypsum products:
• Impression plaster
• Dental investments:
–– Gypsum bonded investments
–– Phosphate bonded investments
–– Silica bonded investments

https://t.me/DentalBooksWorld
948 Triumph's Complete Review of Dentistry

FIVE TYPES OF GYPSUM PRODUCTS AND THEIR PROPERTIES


1. Impression Plaster (Type 1)
• They are composed of plaster of Paris to which modifiers have been added to regulate setting time and setting expansion.
• Impression plaster is rarely used any more for dental impression because it has been replaced by less rigid materials, such
as hydrocolloids and elastomers.
Modifiers such as:
• Potassium sulfate – Decreases setting expansion so as to prevent warpage of impression and also decreases setting time
drastically.
• Borax
Advantage:
• Records excellent fine details
Disadvantages:
• Small dimensional changes
• Fracture on removal from undercuts
• Separating media is required
• Non-toxic but causes dryness
2. Model Plaster (Type 2)
• β-Hemihydrate
• Powder particles are porous and irregular.
• It is usually white in color.
Uses:
• For primary cast for complete dentures
• For articulation purposes
• For flasking in denture construction
Advantage:
• Inexpensive
Disadvantages:
• Low strength
• Porosity
3. Dental Stone (Type 3)
• Discovered in 1930
• α-Hemihydrate or hydrocal
• Powder particles are more dense and regular in shape
• Comes in different colors, like yellow and green
Use:
• Making casts for diagnostic purposes and for complete or partial denture construction
Advantage:
• Greater strength and surface hardness
Disadvantage:
• More expensive than plaster
4. Dental Stone, High Strength (Type 4)
• Modified α-hemihydrate, densite, or die stone
• Powder particles are very dense, cuboidal in shape, and have reduced surface area.
Use:
• For making casts or dies for crown, bridge, and inlay fabrication

https://t.me/DentalBooksWorld
Chapter 11 • Dental Materials 949

Advantages:
• High strength
• Surface hardness
• Abrasion resistant
• Minimum setting expansion
5. Dental Stone, High Strength, High Expansion (Type 5)
• Most recent gypsum product
Use:
• When inadequate expansion has been achieved during the fabrication of cast crowns
Advantages:
• Higher compressive strength
• Higher setting expansion (0.10–0.30%)
SYNTHETIC GYPSUM
• α and β-hemihydrates can also be made from the by-products/waste products of the manufacture of phosphoric acid.
• Synthetic product is more expensive than that made from natural gypsum.

SETTING
It follows reversal in reaction of calcium sulfate hemihydrate powder with water to produce gypsum.
• The product of the reaction is gypsum and the heat evolved in the exothermic reaction is equivalent to the heat used
originally in calcinations.
• The products formed during calcination react with water to form gypsum, but at different rates.
• For example, hexagonal anhydrite reacts very rapidly, whereas when orthorhombic anhydrite is mixed with water, the
reaction may require hours since the orthorhombic anhydrite has a more stable and closely packed crystal lattice.
SETTING REACTION
• Gypsum is a unique material.
• Various hydrates have a relatively low solubility, with a distinct difference between the greater solubility of hemihydrate and
dihydrate.
• Dihydrate is too soluble for use in structures exposed to atmosphere.
–– Dissolution of calcium sulfate hemihydrate.
–– Formation of saturated solution of calcium sulfate.
–– Subsequent aggregation of less soluble calcium sulfate dihydrate.
–– Precipitation of the dihydrate crystals.

THEORIES FOR SETTING REACTION


1. Colloidal/gel theory
2. Hydration theory
3. Crystalline theory
4. Dissolution–precipitation theory
1. Colloidal/Gel Theory
• Originated in 1893 by M. Michaelis.
• When plaster is mixed with water, plaster enters into the colloidal state through a sol–gel mechanism.
• In the sol state, hemihydrate particles are hydrated to form dihydrate, thereby entering into an active state.
• As the measured amount of water is consumed, the mass converts to a solid gel.
2. Hydration Theory
• The rehydrated plaster particles join together through hydrogen bonding to the sulfate groups to form the set material.

https://t.me/DentalBooksWorld
950 Triumph's Complete Review of Dentistry

3. Crystalline Theory
• Originated in 1887 by Henry Louis Le Chatelier.
• In 1907, supported by Jacobus Henricus van’t Hoff.
• The difference in the solubilities of calcium sulfate dihydrate and hemihydrate causes setting differences.
• Dissolved CaSO4 precipitates as calcium sulfate dihydrate, since it is less soluble than hemihydrate.
• X-ray diffraction studies – Not all hemihydrate is converted to dihydrate.
In a setting mass of plaster, two types of centers are there:
1. Dissolution center – Around CaSO4 hemihydrate
2. Precipitation center – Around CaSO4 dihydrate.
4. Dissolution–Precipitation Theory
• Based on dissolution of plaster and instant recrystallization of gypsum to interlocking of crystals.

Solubility of Gypsum and Gypsum Products


The setting reaction is as follows:
1. When the hemihydrate is mixed with water, a suspension is formed that is fluid and workable.
2. The hemihydrate dissolves until it forms a saturated solution.
3. This saturated solution of hemihydrate, supersaturated in dihydrate, precipitates out dihydrate.
4. As the dihydrate precipitates, the solution is no longer saturated with the hemihydrate, so it continues to dissolve.
Dissolution of hemihydrate and precipitation of dihydrate as either new crystals or further growth on the already present.
The reaction continues until no further dihydrate precipitates out of solution.
5. As the gypsum forming increases, mass hardens into needle-like clusters called spherulites.
6. The intermeshing and entangling of crystals lead to a strong, solid structure.
W:P ratio:
• The amount of water and hemihydrate should be gauged accurately by weight.
• W:P ratio is an important factor in determining the physical and chemical properties of the final gypsum product.
Temperature:
• Change in temperature causes change in the relative solubilities of hemihydrate and dihydrate, which alter the rate of the
reaction.

SETTING OF GYPSUM PRODUCTS


Mixing time: The time from the addition of powder to the water until the mixing is completed.
• Mechanical mixing – 20–30 sec.
• Hand spatulation – 1 min.
Working time: The time available to use a workable mix.
• Working time – 3 min.
Loss of gloss test for initial set: Some of the excess water is taken up in forming the dihydrate so that the mix loses its gloss.
Initial Gillmore test for initial set: The mixture is spread out, and the needle is lowered onto the surface. The time at which
it no longer leaves an impression is called the “Initial Set.”
• This is marked by a definite increase in strength.
Vicat test for setting time:
• Vicat penetrometer is used.
• The needle with a weighed plunger rod is supported and held just in contact with the mix. After the gloss is lost, the plunger
is released.
• The time elapsed until the needle no longer penetrates to the bottom of the mix is known as the “Setting Time.”
Gillmore test for final setting time:
• A heavier Gillmore needle is used.
• The time elapsed at which this needle leaves only a barely perceptible mark on the surface is called the “Final Setting Time.”
https://t.me/DentalBooksWorld
Chapter 11 • Dental Materials 951

Ready-for-use criterion:
• The subjective measure of the time at which the set material may be safely handled in the usual manner
• Ready-for-use state is reached in approximately 30 min.

CONTROL OF SETTING TIME


The setting time depends on:
1. Temperature • Effect of temperature on the setting time may vary from one plaster or stone to another; little
change occurs between 0°C and 50°C
• If the temperature of plaster water mixture exceeds 50°C, a gradual retardation occurs
• As the temperature approaches 100°C, no reaction takes place
• At a higher temperature range (50–100°C), there is a tendency for any gypsum crystals formed
to be converted back to the hemihydrate form
2. W:P ratio • The more water used for mixing, the fewer nuclei there are per unit volume; consequently, the
setting time is prolonged
3. Fineness • The finer the particle size of the hemihydrate is, the faster the mix hardens, the rate of
hemihydrate dissolution increases, and the gypsum nuclei are also more numerous. Therefore,
a more rapid rate of crystallization occurs
4. Humidity • Increased contamination by moisture produces sufficient dihydrate on hemihydrate powder to
retard the solution of hemihydrate
• Contamination of gypsum with moisture from air during storage increases the setting time
FACTORS THAT CONTROL SETTING TIME
a. Factors controlled by the operator:
1. W:P ratio • The more the W:P ratio, fewer the nuclei per unit volume and so prolonged setting time
2. Mixing time • Within practical limits, longer and rapid mixing leads to a shorter setting time
• Some gypsum crystals form immediately when the plaster comes in contact with water and as
the mixing begins, formation of crystals increases.
• Some crystals are also broken up by using a spatula and are distributed resulting in the formation
of more nuclei of crystallization resulting in a decreased setting time.
b. Factors controlled by the manufacturer:
1. By the addition of accelerators and retarders:
Accelerators: • Gypsum (<20%) – Decreases the setting time.
• The set gypsum used as an accelerator is called “Terra Alba.”
• Potassium sulfate (conc. 2–3%) – reduces the setting time of model plaster from approximately
10 min to 4 min.
• Sodium chloride (<28%)
Retarders: • Organic materials – Glue, gelatin, and some gums
• Potassium citrate, borax, sodium chloride (20%), and sodium citrate.

SETTING EXPANSION
• Phenomenon: Based on the crystallization mechanism.
–– The crystallization process occurs as an outgrowth of crystals from nuclei of crystallization.
–– The dihydrate crystals growing from the nuclei not only intermesh with but also obstruct the growth of adjacent crystals.
–– If this process is repeated by thousands of crystals during growth, an outward stress or thrust develops that produces an
expansion of the entire mass.
–– The crystal impingement and movement result in the formation of micropores.
• Expansion may vary from 0.06% to 0.5%.
• The volume of dihydrate formed is less than or equal to the volume of hemihydrate and water, i.e., actually a volumetric
contraction should occur during the setting reaction, but instead, a setting expansion is observed.
https://t.me/DentalBooksWorld
952 Triumph's Complete Review of Dentistry

CONTROL OF SETTING EXPANSION


1. W:P ratio • Increase in W:P ratio decreases the nuclei of crystallization per unit volume,
so there is less growth of dihydrate crystals which leads to less outward thrust
2. Accelerators and retarders: Chemicals Accelerators:
added by the manufacturer to regulate • Sodium chloride (up to 2% of hemihydrate)
setting expansion
• Sodium sulfate (maximum effect at 3.4%)
• Potassium sulfate (>2%)
• Potassium tartrate
Retarders:
• Chemicals that form a coating on the hemihydrate particles and prevent the
hemihydrate from going into the solution in the normal manner
• Citrates, acetates, and borates

HYGROSCOPIC SETTING EXPANSION


• Setting expansion that occurs under water is called “Hygroscopic Setting Expansion (HSE).”
• Setting expansion without water immersion is called “Normal Setting Expansion (NSE).”
Stages of Hygroscopic Setting Expansion
Stage I – Initial mix stage.
• Represented by three round particles of hemihydrate surrounded by water.
Stage II – Initial crystal growth stage.
• Crystals of dihydrate have started to form.
• In NSE, the water around the particles is reduced by hydration and the particles are drawn closer together by the surface
tension of water.
• In HSE, the setting is taking place under water so that water of hydration is replaced and the distance between the particles
remains the same.
Stage III – Solid phase contact stage.
• As the dihydrate crystals grow, they contact each other and setting expansion begins.
• In NSE, the water around the particles is reduced. The particles with their attached crystals are drawn together as before,
but the contraction is opposed by the outward thrust of the growing crystals.
• In HSE, the crystals are not inhibited, because the water is again replenished from the outside. In fact, the original particles
are now separated further as the crystals grow and setting expansion occurs.
Stages IV and V
• Expansion and termination.
• The effect becomes more marked.
• The crystals that are inhibited in NSE become intermeshed and entangled much sooner than in HSE in which the crystals
grow much more freely during the early stages before the intermeshing.
• The observed setting expansion that occurs when the gypsum product sets under water may be greater than that which
occurs during setting in air.

STRENGTH
• The strength of the gypsum product is expressed in terms of compressive strength.
• The strength of plaster and stone increases rapidly as the material hardens after initial setting time.
• Free water content of the set product affects its strength.
Two strength properties of gypsum are:
1. Wet strength (Green Strength)
• Strength obtained when the water in excess of that required for hydration of the hemihydrate is left in the test specimen.

https://t.me/DentalBooksWorld
Chapter 11 • Dental Materials 953

2. Dry strength
• Strength obtained when the excess water in the specimen has been driven off by drying.
• Dry strength is two or more times as high as wet strength.
Strength depends upon:
1. Addition of accelerators and retarders – Decreases the wet and dry strength of gypsum products.
2. Increase in W:P ratio increases porosity, which decreases dry strength.

IMPRESSION MATERIALS
INTRODUCTION
Construction of a model or cast is an important step in numerous dental procedures. Various types of casts and models can
be made from gypsum products using an impression mold or negative likeness of a dental structure.

TERMINOLOGIES
Diagnostic casts – Positive replicas of the teeth produced from impressions that create a negative representation of the teeth;
commonly called study models and used for diagnostic purposes and numerous chairside and laboratory procedures.
Bite registration – An impression of the occlusal relationship of opposing teeth in centric occlusion (patient’s normal bite).
Colloid – Glue-like material composed of two or more substances in which one substance does not go into solution but is
suspended within another substance; it has at least two phases, a liquid phase called a sol and a semisolid phase called a gel.
Hydrocolloid – A water-based colloid used as an elastic impression material.
Reversible hydrocolloid – An agar impression material that can be heated to change a gel into a fluid sol state that can flow
around the teeth and then cooled to gel again to make an impression of the shapes of the oral structures.
Irreversible hydrocolloid – An alginate impression material that is mixed to a sol state and as it sets converts to a gel by a
chemical reaction that irreversibly changes its nature.
Agar – A powder derived from seaweed that is a major component of reversible hydrocolloid.
Sol – Liquid state in which colloidal particles are suspended; by cooling or chemical reaction, it can change into a gel.
Gel – A semisolid state in which colloidal particles form a framework that traps liquid (e.g., Jell-O).
Hysteresis – The property of a material to have two different temperatures for melting and solidifying, unlike water, which
has one temperature for both.
Syneresis – A characteristic of gels to contract and squeeze out some liquid that then accumulates on the surface.
Alginate – A versatile irreversible hydrocolloid that is the most widely used impression material in the dental office; it lacks
the accuracy and fine surface detail needed for impressions for crown and bridge procedures.
Elastomers – Highly accurate elastic impression materials that have qualities similar to rubber; they are used extensively in
indirect restorative techniques, such as crown and bridge procedures.
Imbibition – The act of absorbing moisture.
Surfactant – A chemical that lowers the surface tension of a substance so that it is more readily wet; for example, oil beads on
the surface of water, but soap acts as a surfactant to allow the oil to spread over the surface.
Polysulfide – An elastic impression material that has sulfur-containing (mercaptan) functional groups; it has also been
referred to as rubber base impression material.
Condensation silicone – A silicone rubber impression material that sets by linking molecules in long chains but produces a
liquid by-product by condensation.
Addition silicone – A silicone rubber impression that also sets by linking molecules in long chains but produces no by-
product; the most commonly used addition silicones are the polyvinyl siloxanes.
Polyvinyl siloxane (PVS) – Very accurate addition silicone elastomer impression material; it is used extensively for crown and
bridge procedures because of its accuracy, dimensional stability, and ease of use.
Polyether – A rubber impression material with ether functional groups; it has high accuracy and is popular for crown and
bridge procedures.

https://t.me/DentalBooksWorld
954 Triumph's Complete Review of Dentistry

Impression compound – An impression material composed of resin and wax with fillers added to make it stronger and more
stable than wax.
Impression plaster – An impression material composed of a gypsum product similar to plaster of Paris.
Zinc oxide eugenol (ZOE) – A hard and brittle impression material used in complete denture procedures.

CLASSIFICATION
Based on the setting mechanism Inelastic/rigid materials Elastic materials
Irreversible/chemical reaction Plaster of Paris and zinc oxide eugenol Alginate
(cannot be used in undercuts) Elastomeric impression materials
Reversible/physical reaction Impression compound Agar (used to reproduce undercuts/
interproximal spaces)

Elastomeric impression material (ADA specification number – 19)


Composition
Materials/ Polysulfide Polyether Addition silicone Condensation silicone
composition
Facts The main chain in the The materials are supplied
copolymer of ethylene oxide as a base paste and a low-
and tetrahydrofuran viscosity liquid catalyst (or
Cross linking and setting are paste catalyst), a two paste-
promoted by an initiator and system, or a two-putty
an aromatic sulfonate ester system
Base paste
Functional Mercaptan Polymethylhydrosiloxane Alpha – omega – hydroxyl-
unit and divinylpolysiloxane terminated polydimethyl
siloxane
Filler Lithopone or Colloidal silica
titanium dioxide
to provide strength
Plasticizer Dibutyl phthalate Glycol ether or phthalate
to confer
appropriate
viscosity
Accelerator Sulfur 0.5%
Catalyst (accelerator) paste
Filler Lead dioxide
(gives brown
color)
Catalyst It contains Stannous octoate
divinylpolysiloxane and
platinum salt
Plasticizer Dibutyl phthalate
to confer
appropriate
viscosity
Retarder Oleic or stearic
acid

https://t.me/DentalBooksWorld
Chapter 11 • Dental Materials 955

By-product Water Low-molecular-weight No by-product, if correct Ethyl alcohol


alcohols proportions are taken.
If not, hydrogen gas is
formed. So to overcome
it, palladium is added as a
scavenger

PROPERTIES
Agar Alginate Polysulfide Condensation Addition Polyether
silicone silicone
Preparation Boil, temper, Powder, water Two pastes Two pastes or Two pastes Two pastes
store paste–liquid
Handling Complicated Simple Simple Simple Simple Simple
Ease of use Technique Good Fair Fair Good Good
sensitive
Ease of removal Easy Easy Easy Moderate Moderate Moderate to
difficult
Working time 7–15 2.5 5–7 3 2–4.5 2.5
Setting time 5 3.5 8–12 6–8 3–7 4.5
(min)
Stability 1 hour at Immediate 1 hour Immediate 1 week 1 week
100% relative pour pour
humidity
Electroplating No No Yes Yes Yes Yes

Disinfection Poor Poor Fair Excellent Excellent Fair


Elastic recovery 98.8 97.3 96.9–94.5 99.6–98.2 99.9–99.0 99.0–98.3
Flexibility 11 12 8.5–20 3.5–7.8 1.3–5.6 1.9–3.3
Flow – – 0.4–1.9 <0.10 <0.05 <0.05
Reproduction 25 75 25 25 25 25
limit
Shrinkage – – 0.4–0.5 0.2–1.0 0.01–0.2 0.2–0.3
Tear strength 700 380–700 2280–7410 2280–4370 1640–5260 1700–4800

MAXIMUM AND MINIMUM


Maximum working time – Polysulfide
Minimum working time – Addition silicone
Maximum setting time – Polysulfide
Minimum setting time – Addition silicone and polyether
Maximum tear strength – Polysulfide
Minimum tear strength – Condensation silicone
Maximum % contraction – Condensation silicone
Minimum % contraction – Addition silicone

https://t.me/DentalBooksWorld
956 Triumph's Complete Review of Dentistry

Maximum contact angle – Condensation and addition silicone


Minimum contact angle – Polyether
Maximum stiffness – Polyether
Minimum stiffness – Polysulfide
Maximum distortion – Polysulfide
Minimum distortion – Addition silicone
Most costly – Polyether
Least costly – Polysulfide
Most hydrophobic – Addition silicone
Least hydrophobic – Polyether

Material Disinfection
Polysulfide 10–30 min immersion
Polyether Chlorine compounds, NOT
More than 10 min immersion or spray is acceptable
Addition silicone 2% Glutaraldehyde for 10 min. Long immersion can cause surfactant in hydrophilic to
leach out and render less hydrophilic
But no adverse effects are observed even to an extended exposure of 18 hours. The
only drawback appears to be its reduced wetting ability for the hydrophilic silicone
impression material
Condensation silicone Shorter duration is recommended to maintain its dimensional stability

Characteristics features Orders


Stiffness (elastic modulus) PS<CS<AS<PE
Thermal contraction CS=AS<PS<PE
Setting shrinkage PE=AS<PS<CS
Reproduction details PS<PE<CS<AS
Tear strength PS>PE>AS>CS
Biocompatibility PS>AS>CS>PE
Dimensional stability AS>PE>PS>CS
Permanent deformation following PS>PE>CS>AS
strain increases in

Advantages and Disadvantages


Advantages Disadvantages
Polysulfides
Good wetting ability High permanent deformation
Good surface detail Unpleasant taste and odor
Easy to remove Must pour within 1 hour
High tear strength
Condensation silicones
Good surface detail (dry surfaces) Hydrophobic
Good dimensional accuracy Shrinks on storage
Low permanent deformation Must pour within 1 hour

https://t.me/DentalBooksWorld
Chapter 11 • Dental Materials 957

Easy to disinfect Low tear strength


High range of viscosities
Polyethers
Hydrophilic High permanent deformation
Good surface detail Swells in disinfectants or moist environment
Good dimensional accuracy Difficult to remove
Good resistance to deformation Low tear strength
Highly acceptable to patients Care needed when disinfecting
Addition silicones
Good surface detail (dry surfaces) Hydrophobic (unless surfactant added)
Good dimensional accuracy Low tear strength
Good storage stability
Low permanent deformation
Easy to disinfect
Highly acceptable to patients

Failures Causes
Rough or uneven surfaces • Premature removal from mouth
on impression • Improper ratio
• Presence of oil/debris
• Increase in temperature
Bubbles Too rapid polymerization
Irregularly spaced voids Debris on teeth
Rough/chalky stone cast • Increased water and wetting agent left over
• Inadequate cleaning
• Failure to delay pour of addition silicone at least 20 min

Other facts and applied MCQs that can be asked


• Setting time – Denotes that the timing from the beginning of mixing to the curing process has advanced sufficiently.
What will happen if waiting for an extra minute after mixing?
• It ensures success and polymerization may continue for a considerable time after setting.
Permanent deformation exhibited by elastomeric impression material should be clinically negligible provided that
A. The material has adequately gelled
B. Negligible pressure is applied to the tray during polymerization
C. Impression is removed rapidly along the path of insertion
D. Undercuts present in the area are minimum
E. All of the above

HYDROCOLLOIDS
• Agar and Alginate
• ADA specification number for duplicating materials: 20
• Size of colloid particles: 1–200 nm

https://t.me/DentalBooksWorld
958 Triumph's Complete Review of Dentistry

COMPOSITION
Agar Alginate
Components Uses Components Uses
Agar Chief active ingredient Filler-Diatomaceous earth – Strength
60%, ZnO2
Water – 80% Predominant Reactor – Calcium sulfate Initiates the reaction
ingredient
Retarder – Borax (0.2–0.5%) It also gives strength Retarder – Sodium phosphate Slows down the reaction
Accelerator – Potassium sulfate Also acts as gypsum Calcium alginate – 15% Chief active ingredient
hardner
Thymol and glycerine Bactericidal and Accelerator – Potassium Accelerates the reaction
plasticizer, respectively titanium fluoride

Three Steps in Manufacturing Agar Impression Material


Step 1 Liquefaction temperature (70–100°C) 10 min at 100°C
Step 2 Tempering For 3–10 min, several days for 65°C
Step 3 Gelation temperature (37–50°C) 3–10 min at 45°C

Alginate: Mixing and setting time


Mixing time: 45 sec to 1 min
Setting time:
Fast setting alginate: 1.5–3 min
Normal setting alginate: 3–4.5 min

Failures
Problems Causes Solutions
Premature set • Too much powder in mixture • Fluff powder in the container; use correct measures
• Prolonged mixing/loading time for powder and water
• Water or room too warm • Use timer to gauge working time
• Use cool water to slow the set
Slow set • Water too cold • Use warmer water
• Too much water • Use correct water/powder measures
Grainy, lack of surface Incomplete mix of powder and water Wet all of powder, and mix to creamy consistency
detail
Incomplete coverage • Tray too small or too short for arch • Select larger tray or extend borders with rope wax
of teeth or tissues • Tray incompletely seated • Use a mouth mirror to check for complete seating of
the tray
Voids on occlusal Trapped air when the tray is seated Wipe alginate on occlusal surfaces before seating the
surfaces tray
Large voids at • Trapped air • Place alginate in vestibule or palate before seating the
vestibule or midpalate • Not enough alginate in tray tray
• Improper seating of the tray • Use adequate amount of alginate
• Lip in the way • Seat the tray in posterior first, allow alginate to flow
forward into vestibule, seat the tray in anterior
• Pull lip out to create room for alginate

https://t.me/DentalBooksWorld
Chapter 11 • Dental Materials 959

Small voids Air trapped in mix during spatulation Press alginate against sides of the bowl when mixing
throughout with wide-blade spatula to force out air
Distortion or double • Impression removed too soon • Check residual alginate in the bowl for set; let stand
imprint • Tray moved while alginate was setting an additional 1 min
• Hold the tray steady until set; do not have patient hold
the tray
Torn alginate • Impression removed too slowly • Remove the impression quickly with a snap
• Thin mix • Use proper proportions of water and powder
Excess alginate at back • Tray seated in anterior first, then • Seat tray in posterior first, forcing alginate anteriorly
of tray posterior, forcing alginate out the back • Load tray level with sides
• Tray overfilled with alginate • Create shallow trough for teeth
• Remove excess alginate from the back of the tray

Other Applied Facts About Agar


• Gelation is accelerated by cooling water for 3–5 min.
• Applied fact – In mouth, waiting for an extra minute increases the strength and tear resistance.
• The lower the temperature is, the more rapidly the gelation occurs.
• If the material is held rigidly in the tray, the impression material shrinks toward the center of its mass, thereby creating
larger dies.
• Cooling water is circulated at 18–21°C.

Criteria for an Acceptable Alginate Impression


• Both maxillary and mandibular impressions
–– All teeth and alveolar processes recorded
–– Peripheral roll and frenums included
–– No large voids and few small bubbles present
–– Good reproduction of detail
–– Free of debris
–– No distortion
–– Alginate firmly attached to tray
• Maxillary impression
–– Palatal vault recorded
–– Hamular notch area included
• Mandibular impression
–– Retromolar areas included
–– Lingual extensions recorded

Other Applied Facts About Alginate


• The setting time is best regulated in Alginate by – Amount of retarder added (not by controlling temperature).
• An increase in temperature shortens the setting time – For 10° rise in temperature, there is 1 min reduction in the setting time.
• Always water is added to the bowl first, because if powder is added first, then the penetration of water is inhibited and greater
mixing time is required.
• A small amount of gypsum left in the bowl can accelerate the set.
• The thickness of alginate impression between the tray and the tissues should be at least 3 mm.
• Compressive strength is doubled in the first 4 min.

https://t.me/DentalBooksWorld
960 Triumph's Complete Review of Dentistry

• The tray should not be removed from the mouth for at least 3 min after gelation has occurred.
• Over mixing decreases the strength.
• Gypsum hardener in agar – Sulfate.
• Gypsum hardener in alginate – Potassium titanium fluoride.

IMPRESSION COMPOUND
ADA specification number – 3
Types (types I and II)
Although the dental compound has fallen into disuse, it can be used for full-crown impressions (type I), impressions of
partially or completely edentulous jaws (type I), and impression trays in which a final impression is taken with another
material (type II). The dental compound cannot be used to record undercuts because it is not elastic. The impression
compound is available in either cakes or sticks in various colors from a number of manufacturers.
Composition
• Principle ingredient – Wax and resin
• To improve plasticity/workability – Shellac, gutta percha, and stearic acid are added
• For a better flow, calcium carbonate is added
• The filler is added to increase viscosity, to increase rigidity, and to overcome tackiness
• Pour as soon as possible or within 1 hour
• Glass transition temperature: 55–60°C
• To reduce distortion – Allow cooling inside the mouth
Modeling plastic wax
Green compound: 50–51.1°C
Grey compound: 53.3–54.4°C
Red compound: 54.4–55.6°C
Disinfectant – 2% glutaraldehyde
Failures
1. Distortion. If the material is not completely cooled, the inner portions of the impression will still be soft when the impression
is removed, resulting in distortion. Also, if water has been incorporated as the result of wet kneading, the material could
have excessive flow at mouth temperature, producing distortion during removal from the mouth. If the tray used to carry
the compound to the mouth is too flexible, distortion can result. It is important to select a tray that is strong and rigid.
A delay in preparing the stone cast may also cause distortion. The cast should be poured as soon as possible after the
impression has been removed from the mouth.
2. Compound is too brittle or grainy. Prolonged immersion in the water bath will cause low-molecular-weight components
to leach out.

ZINC OXIDE EUGENOL IMPRESSION PASTE


ADA specification number – 16
Composition
Powder
• Zinc oxide – 87%
• Mineral/vegetable oil – Plasticizer – 13%
Liquid
• Oil of clove (eugenol) – 70–85%
• Rosin – Speeds up the reaction and renders a more homogeneous product.
• Canada balsam/Peru balsam – Increases flow and improves mixing properties.

https://t.me/DentalBooksWorld
Chapter 11 • Dental Materials 961

• Acetic acid, zinc acetate, water, and calcium chloride – Accelerator.


• The setting reaction is accelerated by the presence of water, high humidity, or heat. A dimensional change of only about
0.1% shrinkage accompanies the setting.
• These impression materials are classified as hard- and soft-set. The hard-set material sets faster (in about 10 min, compared
with 15 min for the soft-set material), although the hard- and soft-set materials both begin to set in about 5 min. The hard-
set material is more fluid before setting than the soft-set material; after setting, it is harder and more brittle.
• Non-eugenol pastes containing carboxylic acids (e.g., lauric or ortho-ethoxybenzoic acid) in place of eugenol are available
to avoid the stinging and burning sensation experienced by some patients.
Failures
• Inadequate working or setting time. An increase in humidity and/or temperature results in decreased working and setting
time. It is important to select a material that provides the required setting time.
• Distortion. If the tray warps on standing, the impression will also become distorted. It is important to select a stable tray
material.
• Loss of detail. If there is loss of detail, the impression material may not be compatible with the stone used to prepare the cast,
and/or there may be adhesion between the impression and the stone.
• Setting reaction is ionic in nature.
• Impression can be preserved indefinitely – Dimensionally stable.
• Zinc oxide eugenol interocclusal record is more stable than one made with wax.

Storage conditions Dimensional change Causes


Air Shrinkage Evaporation of water from the gel
H2O Expansion Imbibition and absorption of water
100% relative humidity Shrinkage Syneresis
Inorganic salt solutions Expansion or shrinkage Depends on the relationship of electrolyte in gel and in solution

DENTAL POLYMERS AND DENTURE BASE RESINS


INTRODUCTION
Various materials have been used to construct dentures, including cellulose products, phenol-formaldehyde, vinyl resins, and
vulcanite. However, they are not used now because of
• Cellulose products suffered from warpage in the mouth, and from a taste of camphor due to its use as a plasticizer. This
camphor leached out of the denture, causing blistering, staining, and loss of color within a few months.
• Phenol-formaldehyde (bakelite) proved to be too difficult to process and also lost its color in the mouth.
• Vinyl resins were found to have a low resistance to fracture, and failures were common, possibly due to fatigue.
• Vulcanite was the first material to be used for the mass production of dentures, but its esthetic qualities are not very good
and it has now been replaced by acrylic resins.
• Acrylic resin (poly methyl methacrylate) is now the material of choice; it has the required esthetic quality, and is cheap and
easy to process.

CRITERIA
Criteria for an Ideal Denture Base Material
• Natural appearance
• High strength, stiffness, hardness, and toughness
• Dimensional stability
• Absence of odor, taste, or toxic products
• Resistance to absorption of oral fluids

https://t.me/DentalBooksWorld
962 Triumph's Complete Review of Dentistry

• Good retention to polymers, porcelain, and metals


• Ease of repair
• Good shelf life
• Ease of manipulation
• Low density
• Accurate reproduction of surface detail
• Resistance to bacterial growth
• Good thermal conductivity
• Radiopacity
• Ease of cleaning
• Inexpensiveness to use

TERMINOLOGIES
Terminologies in Dental Polymers
The backbone—The main chain of a polymer.
Block copolymer—A polymer made of two or more monomer species and identical monomer units (“mers”) occurring in
relatively long sequences along the main polymer chain. See also random copolymer and graft or branched copolymer.
Chain transfer—The stage of polymerization in which the free radical on the growing end of one polymer chain is transferred
to either a monomer or a second polymer chain. This terminates the chain growth in the first chain and initiates the chain
growth in the monomer or second polymer chain.
Curing—Chemical reaction in which low-molecular-weight monomers (or small polymers) are converted into higher-
molecular-weight materials to attain desired properties (see also the closely related terms polymerization and setting).
Crosslink—A difunctional or multifunctional monomer that forms a link between two polymer chains. Crosslinked polymers
have many such crosslinks between neighboring chains such that a three-dimensional interconnected polymer network results.
Denture base—The part of the denture that rests on the soft tissues overlying the maxillary and mandibular jawbone and that
anchors the artificial teeth.
Elastic recovery—Reduction or elimination of elastic strain (deformation per unit length) when an applied force is removed;
elastic solids recover elastic strain immediately on removal of the applied force, whereas viscoelastic materials recover elastic
strain over time. The greater the viscous nature of an elastomer, the more incomplete the recovery.
Final set—The stage at which the curing process is complete.
Free radical—An atom or group of atoms (R) with an unpaired electron (•). R•-producing reactions that initiate and propagate
polymerization and eventually lead to a final set.
Glass transition temperature (Tg)—The temperature at which macromolecule molecular motion begins to force the polymer
chains apart. Thus, polymeric materials soften when heated above this temperature.
Graft or branched copolymer—Polymer in which a sequence of one type of mer unit is attached as a graft (branched) onto
the backbone of a second type of mer unit.
Initial set (of a polymer)—The stage of polymerization during which the polymer retains its shape.
Induction—Activation of free radicals, which in turn initiates growing polymer chains.
Macromolecule—A large high-molecular-weight compound usually consisting of repeating units in a chainlike configuration
(see also polymer).
Mer—The term used to designate the repeating unit or units in a polymer chain; thus, mers are the “links” in the chain.
Monomer—Chemical compound that is capable of reacting to form a polymer.
Plastic flow (of a polymer)—Irreversible deformation that occurs when polymer chains slide over one another and become
relocated within the material.
Polymer—Chemical compound consisting of a large organic molecule (“macromolecule”) formed by the union of many
smaller repeating units (mers).

https://t.me/DentalBooksWorld
Chapter 11 • Dental Materials 963

Polymerization—Chemical reaction in which monomers of a low molecular weight are converted into chains of polymers
with a high molecular weight.
Propagation—Stage of polymerization during which polymer chains continue to grow to high molecular weights.
Random copolymer—A polymer made of two or more monomer species but with no sequential order between the mer units
along the polymer chain. See also block copolymer and graft or branched copolymer.
Resin or synthetic resin—Blend of monomers and/or macromolecules with other components, which form a material with
a set of useful properties.
Resin-based composite—A highly crosslinked resin reinforced by a dispersion of amorphous silica, glass, crystalline, or
organic resin filler particles and/or fibers bonded to the polymer matrix by a coupling agent.
Setting (of a polymer)—Extent to which polymerization has progressed.
Thermoplastic polymer—A macromolecule material made of linear and/or branched chains that softens when heated
above the glass-transition temperature (Tg), at which molecular motion begins to force the chains apart and soften the
polymer. Thermoplastics can be heated above the Tg, molded to a new shape, and then cooled below the Tg to retain the new
configuration.
Thermosetting polymer—A polymeric material that becomes permanently hard when heated above the temperature at
which polymerization occurs and that does not soften again on reheating to the same temperature.
Termination—The stage of polymerization during which polymer chains no longer grow.
Viscoelastic—Term describing a polymer that combines the spring-like behavior of an elastic solid (such as a rubber band)
with that of the puttylike behavior of a viscous, flowable fluid (such as honey).

Heat-Cured Resins
These materials consist of a powder and a liquid, which, on mixing and subsequent heating, form a rigid solid.
Composition of the Powder
Constituent Percentage (%) Reason for inclusion
Polymethylmethacrylate 95–98% Principal component
Benzoyl peroxide 1 Initiator
Titanium dioxide Small amount Increases opacity to match the translucency of the oral
Zinc oxide soft tissues
Inorganic pigments 1 Varies color, respectively:
Mercuric sulfide Red
Cadmium sulfide Yellow
Ferric oxide Brown
Dibutyl phthalate Small amount Plasticizer
Dyed synthetic fibers – Nylon or Small amount Simulate anatomical structures such as capillaries
acrylic within the denture base material
Composition of the Liquid
Principle component Methyl methacrylate – Monomer
Inhibitor Hydroquinone
Cross-linking agent Ethylene glycol dimethacrylate

Cold-Cured Resins
• The chemistry of these resins is identical to that of the heat-cured resins, except that the cure is initiated by a tertiary amine
(e.g., dimethyl-P-toluidine or sulfinic acid) rather than heat.
• This method of curing is not as efficient as the heat curing process, and tends to result in a low-molecular-weight material.

https://t.me/DentalBooksWorld
964 Triumph's Complete Review of Dentistry

• This has an adverse effect on the strength properties of the material and also raises the amount of uncured residual monomer
in the resin.
• The color stability is not as good as for the heat-cured material, and the cold-cured resins are more prone to yellowing.
• The size of the polymer beads is somewhat smaller than in the heat-cured resin (which has a bead size of 150 µm) to ease
dissolution in the monomer to produce a dough.
• The doughy stage has to be reached before the addition curing reaction begins to affect the viscosity of the mix and prevents
the adaptation of the mix to the mold walls.
• The lower molecular weight also results in a lowering of the glass transition temperature, with Tg being typically 75–80°C.

Visible Light Cured Resins


A visible light activated denture base resin was first introduced in the 1980s, on the premise that it did not contain methyl
methacrylate, which is a known sensitizer. Thus, the material was advocated for its biocompatibility, but also because of its
low bacterial adherence, ease of fabrication and manipulation, patient acceptance, ability to bond to other denture resins, and
a lack of requirement for proportioning and mixing.

Other Facts
• The polymerization shrinkage is reduced when compared to using a monomer because most of the material that is being
used (i.e., the beads or granules) has already been polymerized.
• The polymerization reaction is highly exothermic, as a considerable amount of heat energy (80  kJ/mol) is released in
reducing the C=C to –C–C– bonds.
• Since a large proportion of the mixture is already in the form of a polymer, the potential for overheating is reduced.
• The monomer is extremely volatile and highly flammable, so the container must be kept sealed at all times and must be kept
away from naked flames.
• Hydroquinone also extends the shelf life of the monomer by reacting rapidly with any free radicals that may form
spontaneously within the liquid and producing forms of stabilized free radicals that are not able to initiate the polymerization
process.
• Contamination with the polymer beads or granules must be avoided, as these carry the benzoyl peroxide on their surface
and only a tiny amount of the polymer is needed to start the polymerization reaction.
• The polymer powder is very stable and has a virtually indefinite shelf life.

AMALGAM
TERMINOLOGIES
Amalgam—An alloy containing mercury.
Amalgamation—The process of mixing liquid mercury with one or more metals or alloys to form an amalgam.
Creep—The time-dependent strain or deformation that is produced by a stress. The creep process can cause an amalgam
restoration to extend out of the restoration site, thereby increasing its susceptibility to marginal breakdown.
Delayed expansion—The gradual expansion of a zinc-containing amalgam over a period of weeks to months. This expansion
is associated with the development of hydrogen gas, which is caused by the incorporation of moisture in the plastic mass
during its manipulation in a cavity preparation.
Dental amalgam—An alloy that is formed by reacting mercury with silver, copper, and tin, and which may also contain
palladium, zinc, and other elements to improve handling characteristics and clinical performance.
Dental amalgam alloy (alloy for dental amalgam)—An alloy of silver, copper, tin, and other elements that is processed in the
form of powder particles or as a compressed pellet.
Marginal breakdown—The gradual fracture of the perimeter or margin of a dental amalgam filling, which leads to the
formation of gaps between the amalgam and the tooth.

https://t.me/DentalBooksWorld
Chapter 11 • Dental Materials 965

Trituration—The mixing of amalgam alloy particles with mercury in a device called a triturator; the term is also used to
describe the reduction of a solid to fine particles by grinding or friction.
• By definition, an amalgam is an alloy that contains mercury. Mercury, a liquid at room temperature, can dissolve and react
to form an alloy with numerous metals. When metal particles are mixed with mercury, the outer portion of the particles
dissolves into mercury.
• At the same time, mercury diffuses into the metal particles. When the solubility of the metal in mercury is exceeded,
crystals of mercury-containing compounds start to precipitate within the mercury.
• During this period of reaction, the metal particles coexist with the liquid mercury, giving the mix a plastic consistency. This
means that the mixture can be adapted to any shape with a light pressure. As the content of liquid mercury in the mixture
decreases by the formation of precipitates, the mixture hardens. This process is called amalgamation and the material has
been used for restoring the tooth structure.
• The first use of amalgam for tooth filling was recorded in the Chinese medical literature in 659 AD.
ADA specification number – 1

AMALGAM WAR
When amalgam was first introduced in 1833, many dentists were outraged at the suggestion of installing such a highly toxic
metal in their patients’ mouths. Amalgam was called “Quecksilber” and anyone who placed amalgams was called a “Quack.”
This controversy, later termed the first amalgam war.
• First amalgam war – Initiated in 1841
• Second amalgam war – Alfred Stock in 1920
• Third amalgam war – H. A. Huggins in late 1970s

CLASSIFICATION OF AMALGAM
Based on Cu Content
• Conventional or low-copper alloy
• High-copper alloy
–– Low-Cu alloys (less than 6% copper)
–– High-Cu alloys (more than 6% copper)
Based on Zn Content
• Zn containing (more than 0.1% zinc)
• Zn free (less than 0.1% zinc)
Based on the Shape of Alloy Particles
• Lathe cut alloys
–– Regular-cut
–– Fine-cut
–– Micro-fine cut
• Spherical alloys
• Spheroidal alloys
New amalgam alloys

COMPOSITION
Low-Copper Alloys
• Silver–tin alloys are quite brittle and difficult to blend uniformly unless a small amount of copper is substituted for silver.
• Within the limited range of copper solubility, an increased copper content hardens and strengthens the silver–tin alloy.
• The chief function of zinc in an amalgam alloy is to act as a deoxidizer, which is an oxygen scavenger that minimizes the
formation of oxides of other elements in the amalgam alloys during melting. Alloys without zinc are more brittle, and their
amalgams tend to be less plastic during condensation and carving.

https://t.me/DentalBooksWorld
966 Triumph's Complete Review of Dentistry

• The ranges of conventional alloy composition by weight in the early 1980s were 66.7%–71.5% silver, 24.3%–27.6% tin,
1.2%–5.5% copper, 0%–1.5% zinc, and 0%–4.7% mercury.
• The structure of these conventional alloys was dominated by Ag3Sn (γ phase) with some Cu3Sn (ε phase).
High-Copper Alloys
• The first high-copper alloy was formulated by mixing one part of silver–copper, spherical eutectic (Ag–Cu; 71.9% silver and
28.1% copper by weight) particles to two parts of Ag3Sn, provided as lathe-cut particles.
• This modification raises the copper content to 11.8% by weight. This is often called “dispersed-phase alloy” or “admixed
high-copper alloy.”
• A second type of high-copper alloy was made by melting all components of the dispersed phase alloy. This process yields a
single composition system.
• The presence of the higher copper content makes mechanical cutting of ingots into particles difficult. Thus, they are often
provided in a spherical form that is produced by an atomization process.
• The copper content of this group of alloys can be as high as 30% by weight. Various amounts of indium or palladium have
been included in some commercial systems.

Composition of some Composition (mass %)


typical amalgam alloys
Amalgam Silver (Ag) Tin (Sn) Copper (Cu) Zinc (Zn)
Low copper (lathe cut) 70.3 25.9 2.8 0.9
Low copper (spherical) 72 25 3 –
High copper (admix) 69.5 17.7 11.8 1
High copper (spherical) 61 26 13 –
Amalgam alloys contain at least 69% weight silver, 29% weight tin, and <6% copper.

What Are the Advantages and Disadvantages of its Components?


Element Function
Silver Main constituent of the alloy, combines with tin
Tin Combines with silver
Increases mechanical properties, decreases creep, increases corrosion resistance, and
Copper
decreases the amount of the γ2 phase formation
Zinc Acts as a scavenger of oxygen
Mercury Sometimes added to increase the rate of reaction (pre-amalgamation)
Indium, palladium, selenium, All increase corrosion resistance and improve certain mechanical properties of the final
platinum, and gold product. Decrease creep

INDIVIDUAL COMPONENTS
Zinc Indium Palladium Mercury
Increases strength Increases strength Increases strength Decreases setting time
Increases expansion Increases expansion Increases corrosion resistance Decreases delayed expansion
Increases flow Increases flow
Increases setting time Increases setting time
Decreases corrosion resistance Amalgamation more difficult
Increases plasticity
Decreases brittleness
Scavenger

https://t.me/DentalBooksWorld
Chapter 11 • Dental Materials 967

Lathe-Cut Powder versus Spherical Powder


• Amalgams made from lathe-cut powders or admixed powders tend to resist condensation better than amalgams made
entirely from spherical powders.
• Since freshly triturated amalgams from spherical powders are very plastic, one cannot rely on the pressure of condensation
to establish a proximal contour for a class II restoration.
• Spherical alloys require less mercury than typical lathe-cut alloys because the spherical alloy powder has a smaller surface
area per volume ratio than does the lathe-cut powder. Amalgams with low mercury content generally have better properties.

Particle Size
• The average particle sizes of modern powders range between 15  µm and 35  µm. Smaller particles greatly increase the
surface area per unit volume of the powder.
• A powder containing tiny particles requires a greater amount of mercury to form an acceptable amalgam. It is critical to
maintain an optimal particle size and size distribution.
• The particle size distribution can affect the character of the finished surface. When the amalgam has partially hardened, the
tooth anatomy is carved in the amalgam with a sharp instrument.
• During carving, the larger particles may be pulled out of the matrix, producing a rough surface. Such a surface is probably
more susceptible to corrosion than a smooth surface.
• A smaller average particle size tends to produce a more rapid hardening of the amalgam with greater early strength.

SETTING REACTION OF AMALGAM


Conventional Low-Copper Amalgam
This type of amalgam sets by the reaction of silver–tin particles with mercury to produce two reaction product phases.
i. The silver–tin phase gamma 1 phase
Gamma phase
• The major reaction product phases are silver–mercury and tin–mercury (approximately Ag2Hg3 and Sn7-8Hg).
• The original alloy is designated as gamma phase, and the reaction product phases are called gamma–1 and gamma–2.
Gamma 1 phase
• The silver–mercury crystals are generally small and form most of the matrix. This phase has intermediate corrosion
resistance.
Gamma 2 phase
• Tin–mercury reaction product crystals are long and blade-like, penetrating throughout the matrix. Although they constitute
less than 10% of the final composition, they form a penetrating matrix because of intercrystalline contacts between the
blades.
• This phase is prone to corrosion in clinical restorations, a process that proceeds from the outside of the amalgam, along the
crystals, connecting to the new crystals at intercrystalline contacts.
High-Copper Amalgam
High-copper amalgam sets in a manner similar to low-copper amalgams except that tin–mercury reactions are suppressed by
the preferential formation of copper–tin phases instead.
Setting reaction of high-copper admixed amalgam alloy and setting reaction of unicompositional high-copper amalgam
alloy.
Low-copper alloy
Ag3Sn + Hg Ag2Hg3 + Sn7–8Hg + unreacted Ag3Sn

High-copper admix alloy


Ag3Sn + Ag-Cu + Hg Ag2Hg3 + Sn7–8Hg + unreacted Ag-Cu + unreacted Ag3Sn

Secondary Cu6Sn5 + Ag2Hg3 + unreacted Ag-Cu


solid-state reacon
High-copper spherical alloy
[Ag3Sn + Cu3Sn] + Hg Ag2Hg3 + Cu6Sn5 + unreacted [Ag3Sn + Cu3Sn]

https://t.me/DentalBooksWorld
968 Triumph's Complete Review of Dentistry

PHASES
Mercury Hg Most weakest phase (ref – Marzouk)
Gamma Ag3 Sn Formed by peritectic reaction
Gamma 1 Ag2 Hg
Gamma 2 Sn8 Hg Weakest phase
Epsilon Cu3 Sn Occurs in high-copper single-composition
amalgam only
ETA Cu6 Sn5

PROPERTIES AND FACTS


• Copper is added to Ag3 Sn to harden/increase the strength of the silver/tin alloy.
• Zinc in amalgam causes delayed expansion that occurs during the condensation process.
• If there is no zinc in an amalgam alloy, the final product will be less plastic.
• The average particle size of alloy particles ranges between 15 µm and 35 µm.
• Lathe cut obtained from ingot (after that washed with acid to make it more reactive).
• Ag–Sn alloy – Rapid quenching of alloy ingot results in a maximum amount of beta phase and slow cooling results in
gamma phase.
• Stress relief of amalgam done at 100°C.
• Spherical particle → washed with acid + heat treatment—Coarsens/also slows down the reaction with mercury.

Applied Facts About Particle Size


• Very small particle size less than 3 µm → surface area per unit volume—Tiny particle requires a greater amount of mercury.
• Spherical alloys require less mercury than lathe cut alloys because spherical alloy powder particles have a smaller surface
area per unit volume (which means that they have a larger particle size).
• Solubility of Ag in mercury → 0.035 wt %
• Solubility of Sn in mercury →0.60 wt %
• Admix alloy powder contains → 30–55% wt spherical high-copper powder.
• Total copper content in admixed high copper → 9–20 wt %
• Total copper content in single-composition copper → 13–30 wt %
• Admixed = Lathe cut copper alloy + spherical silver copper alloy
• Low copper → gamma 1 + gamma 2 + unconsumed alloy particles (beta + gamma)
• Admixed copper → gamma 1 + eta + unconsumed alloy of both types of particles (to some extent gamma 2)
• Single composition alloy → 60 wt % Ag + 27 wt % + Sn + 13 wt % Cu
• To eliminate gamma 2 phase, at least 12% of copper content is needed

Factors that Lead to Contraction and Expansion


Contraction Expansion
Less mercury in the mix Postoperative sensitivity
Decreased mercury alloy ratio Protrusion of restoration
Increased condensation pressure Pressure on pulp (it causes excessive expansion)
Small particle size Increased mercury in the mix
Longer trituration
Micro leakage
Plaque accumulation and secondary caries

https://t.me/DentalBooksWorld
Chapter 11 • Dental Materials 969

• Amalgam should neither contract nor expand more than 20 µm/cm measured at 37°C between 5 min and 24 hours after
the beginning of trituration.
• Initial contraction is for 20 min from the beginning of trituration.
• Contraction continues as long as the growth of gamma 1 phase continues.
• Expansion due to zinc starts after 3–5 days and may continue for months reaching value up to 400 µm (4%) and this is due
to the reaction of zinc with water.
• More unconsumed Ag–Sn particles that are retained in the final structure strengthen the amalgam.
• Amalgam defects are more common at margins.
• The major cause of secondary caries is plaque accumulation.

Amalgam Compressive strength at 1 hour Tensile strength Creep value


Low copper 145 60 2% (0.8–8%)
Admix 137 48 0.4%
Single composition 262 (it doubles) 64 0.13%
• So the compressive strength should be 310 Mpa and the amalgam cannot withstand high tensile bending stress.
• At the end of 20 min, the compressive strength is 6% and the minimum compressive strength should be 80 Mpa at the end
of 1 hour.
• Should not use much stresses in the restored tooth region for at least 8 hours after the placement – 70% of strength is
reached by the eighth hour.
• Mercury is liquid at room temperature.
• Amalgam becomes stronger to support moderate biting forces within the first hour.
• Creep value <3% is recommended.

Trituration depends up on
• Type of amalgam
• Trituration time
• Speed of the amalgamator
Both under and over trituration and excess mercury (>54%) decrease the strength.

Creep Rate
• Increases with high gamma 1 volume fraction.
• Decreases with larger gamma 1 grain size.
• Gamma 2 increases creep rate.
• Lathe cut/admixed alloys → requires maximum condensation pressure and low mercury:alloy ratio.
• Spherical amalgam requires light condensation pressure.
• Most common corrosion products are oxides/chlorides of tin.
• Gold and amalgam → corrosion of amalgam is because of large difference in EMF (electromotive force).

TARNISH AND CORROSION


Low-copper and high-copper amalgams undergo two types of corrosions:
1. Chemical corrosion
2. Electrochemical corrosion
1. Chemical corrosion
This occurs most notably on the occlusal surface and produces a black film of silver–sulfur (Ag–S) tarnish.
This discoloration is limited to the surface and does not compromise any properties except for esthetics.

https://t.me/DentalBooksWorld
970 Triumph's Complete Review of Dentistry

2. Electrochemical corrosion
The basic components required for electrochemical corrosion to take place are
• Anode (site of corrosion)
• Cathode
• Circuit
• Electrolyte
This kind of corrosion is due to the oxidation of the tin–mercury phase of the amalgam leading to the formation of oxides
and oxychlorides of tin.
Mechanism of Electrochemical Corrosion
• Electrochemical corrosion occurs whenever chemically different sites act as the anode and the cathode.
• This corrosion requires the sites be connected by an electrical circuit in the presence of an electrolyte, typically saliva.
• The anode corrodes, producing soluble and insoluble corrosion reaction products.
Galvanic Corrosion
If an amalgam is in direct contact with an adjacent metallic restoration such as gold crown, the amalgam is the anode in
the circuit. This type of electrochemical corrosion is called galvanic corrosion and is associated with the presence of
macroscopically different reaction products.
Local Galvanic Corrosion
The same process may occur microscopically (local galvanic corrosion) because of the electrochemical differences of different
phases within the amalgam. Residual amalgam alloy particles act as the strongest cathodes. Tin–mercury or copper–tin
reaction product phases are the strongest anodes in low-copper and high-copper amalgams.
Concentration Cell Corrosion or Crevice Corrosion
Local electrochemical cells also may arise whenever a portion of the amalgam is covered by plaque or soft tissue. The covered
area has a locally lowered oxygen or higher hydrogen ion concentration, making it behave more anodically and corrode.
Cracks and crevices produce similar conditions and preferentially corrode.
Stress Corrosion
Regions within the amalgam that are under stress also display a greater propensity for corrosion (stress corrosion).

Clinical Significance
• Amalgam has a linear coefficient of thermal expansion that is 2.5 times greater than the tooth structure, and it does not
bond to the tooth structure. During expansion and contraction, percolation occurs along the external walls. The formation
of corrosion products prevents the fluid ingress and egress along the margins and contributes to the self-sealing ability of
the amalgam.
• Electrochemical corrosion is not a mechanism of mercury liberation from set amalgam.
• Mercury immediately reacts with locally available silver and tin from amalgam alloy particles and is reconsumed to form
more corrosion reaction products.

DIRECT-FILLING GOLD
INTRODUCTION
• Direct gold are those gold restorative materials that are manufactured for directly compacting them in to the prepared
cavities.
• Gold is used for the purpose of restorations.
• High-quality direct-gold restoration can be ensured only when four principal conditions are satisfied.
• An appropriate gold form is used for each specific clinical situation.
• The material is used only where it is indicated.
• A perfectly dry and clean field is provided.
• The material is properly manipulated with the correct instruments.

https://t.me/DentalBooksWorld
Chapter 11 • Dental Materials 971

History of Gold Foil Used as a Tooth-Filling Material


• Gold leaf as a restorative material became popular in the United States in the early 19th century.
• Marcus Bull of Hartford, Connecticut, began producing beaten gold for dental applications in 1812.
• In 1853, sponge gold was introduced in the United States and England to replace gold leaf.
• This was followed by the cohesive, or adhesive, gold introduced by American dentist Robert A. Arthur in 1855.

What are some of the properties of pure gold?


• Most noble metal, rarely tarnishes
• Chemical inactive (not affected by air, moisture, heat, or most solvents)
• Can be cold-welded at room temperature due to metallic bonds
• High thermal conductivity
• Hard to form a dense restoration

FORMS OF DIRECT-FILLING GOLD


1. Gold foil (fibrous gold)
a. Sheets
• Cohesive
• Non-cohesive
b. Pellets (hand rolled and preformed)
c. Cylinders
d. Corrugated foil
e. Platinized foil
f. Laminated foil
g. Ropes
2. Electrolytic precipitate (also called crystalline gold)
a. Mat gold
b. Mat foil (mat gold plus gold foil)
c. Gold–calcium alloy
3. Granular gold (also called encapsulated gold powder/powdered gold)
The second type is misnamed crystalline gold because all three types are crystalline metals.

Gold Foil
• Gold foil is sometimes called fibrous gold; it is often provided in thickness as low as 0.6 µm.
Standard No. 4 gold foil is supplied in 100 × 100 mm (4 × 4 inch) sheets that weigh 4 grains (0.259 g) and about 0.51 µm thick.
• The numbering system refers to the weight of a standard sheet, so it reflects the thickness well. Thus, No. 3 foil weighs
3 grains (0.0194 g) and is about 0.38 µm thick.
• The No. 3 foil is issued in the electrolytic and powder products.
• Gold foil is manufactured by beating pure gold into thin sheets.
• The gold foil is cut into 4 × 4 inch (10 × 10 cm) sheets and sold in books of sheets, separated by pages of thin paper. The
books contain 1/10 oz or 1/20 oz of gold. The sheet of foil that weighs 4 g is termed No. 4 foil; the sheet weighing 3 g is
termed No. 3 foil; and the sheet weighing 2 g is termed No. 2 foil.
• Because the 4 × 4 inch sheets are too large to be used in restorative procedures, they are rolled into cylinders or pellets before
insertion into tooth preparations. (The gold foil referred to in the restorative sections of this chapter is in a pellet form.)

https://t.me/DentalBooksWorld
972 Triumph's Complete Review of Dentistry

• Pellets of gold foil are generally rolled from 1/32-inch, 1/43-inch, 1/64-inch, or 1/128-inch sections cut from a No. 4 sheet
of foil. The book of foil is marked and cut into squares or rectangles.
• Each piece is placed on clean fingertips, and the corners are tucked into the center, and then the foil is lightly rolled into a
pellet form.
• In addition, cylinders of gold foil may be rolled from the segments of a sheet.
• After the pellets of gold are rolled, they may be conveniently stored in a gold foil box, which is divided into labeled sections
for various sizes of pellets.
• Cylinders of foil and selected sizes of other types of gold may also be stored in the box.
• Preferential contamination is suggested by placing a damp cotton pellet dipped into 18% ammonia into each section of the
box.
• This serves to prevent deleterious oxides from forming on the gold until it is used.
• Powdered gold is made by a combination of chemical precipitation and atomization, with an average particle size of 15 mm.
• The atomized particles are mixed together in wax, cut into pieces, and wrapped in No. 4 or No. 3 foil; several sizes of these
pellets are available. This product is marketed as Williams E-Z Gold (Ivoclar-Williams, Amherst, NY, United States).

Cohesive and Non-cohesive Gold


• Forms of gold can be supplied in both cohesive and non-cohesive conditions; only the sheet foil is typically furnished in
either of these two conditions.
• The ammonia-treated gold foil is called non-cohesive foil.

Preformed Gold Foil


• The ropes and cylinders are rolled in specific sizes and cut in various lengths to provide many sizes of increments for filling
the prepared tooth cavity.
• The laminates are not available as preforms, but they can be made in a dental office by placing a number of sheets on top of
one another and then cutting the laminate into pieces of a desired size.

Platinized Gold Foil


This form of gold is a laminated structure that can be produced in one of the two ways:
1. Two sheets of No. 4 pure gold foil and a layer of pure platinum foil sandwiched between them and hammered.
2. Layers of platinum and gold can be bonded together by a cladding process during the rolling operation, and thus the
sandwich is already welded together before the hammering procedure begins.
• This product is available only in No. 4 sheet form.
• The objective of adding platinum to gold foil is to increase the hardness and wear resistance of restorations that are made
from this material.

Electrolytic Precipitate
Another form of gold for direct filling consists of microcrystalline gold powder formed by electrolytic precipitation (also
called crystalline, mat, or sponge).
• It cannot be described as foil because it is not formed by a thickness reduction process such as hammering and rolling.
• The powder, which consists of dendritic crystals approximately 0.1 mm in length, is formed into shapes by sintering at an
elevated temperature well below the melting point of gold, which is 1063°C or 1945°F.

Mat Gold
• Mat gold is an electrolytically precipitated crystalline form that is sandwiched between sheets of gold foil and formed into strips.
• Alloyed electrolytic precipitate.

https://t.me/DentalBooksWorld
Chapter 11 • Dental Materials 973

• One form of electrolytic precipitate is alloyed with calcium.


• The alloy is converted to a mat form by sintering at an elevated temperature.
• The calcium content of the finished product is about 0.1%.
• Its purpose is to produce stronger restorations by dispersion strengthening.

Cohesion and Degassing


Direct gold is inserted into tooth preparations under force. The purpose of the force is to weld the gold into restorations
containing minimal porosity or internal void spaces.
• Welding occurs because pure gold with an absolutely clean surface coheres as a result of metallic bonding.
• As the gold is forced and compressed into a tooth preparation, succeeding increments cohere to those previously place.
• For successful welding to occur during restoration, the gold must be in a cohesive state before compaction, and a suitable,
biologically compatible compacting force must be delivered.
• Direct gold may be either cohesive or non-cohesive. It is non-cohesive in the presence of surface impurities or wax, which
prevents one increment of gold from cohering to another.
• The manufacturer supplies books of gold foil or pre-rolled cylinders in a cohesive or non-cohesive state. E-Z Gold pellets
are supplied with a wax coating that must be burned off before compaction.
• Because gold attracts gases that render it non-cohesive, such gases must be removed from the surface of the gold before dental
compaction. This process usually is referred to as degassing or annealing and is accomplished by the application of heat.
• The term degassing is preferable because the desired result is to remove residual surface contamination (although further
annealing, resulting in additional internal stress relief or recrystallization, may also occur in this process).
• All direct-filling gold products are degassed immediately before use except when non-cohesive foil is specifically desired.
Under-heating during degassing should be avoided because it fails to render the gold surface pure.
• Over-heating also should be avoided because it may cause the gold to become brittle or melt and render it unusable.
• Degassing is accomplished by heating the gold foil on a mica tray over a flame or on an electric annealer or by heating each
piece of gold over a pure ethanol flame.
• The advantage of the technique involving use of the pure ethanol flame is that each piece of gold is selected and heated just
before insertion, and waste of gold is avoided. A careful technique is needed to degas an increment of gold in the flame
correctly.
• The gold is passed into the blue inner core of the flame on the tip of a foil-passing instrument and held just until the gold
becomes dull red, and then the instrument is withdrawn from the flame.
• After a few seconds are allowed for cooling, the gold is placed in the preparation. Although any of the three degassing
procedures is satisfactory for gold foil, this is not the case for E-Z Gold. The E-Z Gold pellet must be heated 0.5–1 inch
above the ethanol flame until a bright flame occurs (caused by the ignition of the wax) and the pellet becomes dull red for
2–3 sec; then it is withdrawn from above the flame.
• Temperatures below 315°C (600°F) are inadequate to attain optimum hardness of the compacted gold.

PRINCIPLES OF COMPACTION
• Cold welding refers to the process of forming atomic bonds between pellets, segments, or layers as a result of condensation.
• Wedging refers to the pressurized adaptation of the gold form within the space between tooth structure walls or corners that
have been slightly deformed elastically.
• Direct-filling gold must be compacted during insertion into tooth preparations.
• With the exception of E-Z Gold, the compaction takes the form of malleting forces that are delivered either by a hand mallet
used by the assistant or by an Electro-Mallet (McShirley Products, Glendale, CA, United States) or a pneumatic mallet used
by the dentist.
• E-Z Gold, because of its powdered form, may be compacted by heavy hand pressure delivered in a rocking motion with
specially designed hand condensers.

https://t.me/DentalBooksWorld
974 Triumph's Complete Review of Dentistry

• Successful malleting of the gold foil may be achieved with any of the currently available equipment. Some operators prefer
the Electro-Mallet or the pneumatic mallet because a dental assistant is not required for the procedure.
• Condensers are designed to deliver forces of compaction to direct gold.
• Condensers used in the handpieces of the Electro-Mallet or pneumatic mallet consist of a nib, or working tip, and a short
shank (approximately 1 inch [2.5 cm] in length) that fits into the malleting handpiece.
• Condensers used with the hand mallet are longer (approximately 6 inches [15 cm]) and have a blunt-ended handle that
receives light blows from the hand mallet.
• Condenser nibs are available in several shapes and sizes.
• All have pyramidal serrations on the nib faces to prevent slipping on the gold.

CONDENSERS
1. The round condensers, which have 0.4–0.55 mm in diameter.
2. The Varney foot condenser, which has a rectangular face that is approximately 1–1.3 mm, and
3. The parallelogram condensers, which are used only for hand pressure compaction and have nib faces that measure
approximately 0.5–1 mm.
• Condenser shanks may be straight, mono-angled, or offset, and their nib faces may be cut perpendicular to the long axis
of the handle or perpendicular to the end portion of the shank.
• A given amount of force is distributed over four times as much area for a 2 mm diameter tip as for a 1 mm tip.
• In other words, the pressure is four times as great with a 1 mm condenser as with a 2 mm condenser.
• It takes four times as much force to fully compact the area under a 2 mm diameter tip as it does for a 1 mm tip.
• The smaller the nib face size (i.e., area), the greater the pounds per square inch delivered (given a constant malleting
force). If the nib diameter is reduced by half, the effective compaction force in pounds per square inch is four times
greater (because the area of a circle is proportional to the square of the diameter).
• For most gold, the 0.4- to 0.55-mm diameter nibs are suitable. Smaller condensers tend to punch holes in the gold,
whereas larger ones are less effective in forcing the gold into angles in the tooth preparation.

COMPACTION TECHNIQUE FOR GOLD FOIL


• Compaction begins when a piece of gold is placed in a tooth preparation.
• The gold is first pressed into place by hand, and then a condenser of suitable size is used to begin malleting in the center of
the mass (often this is done while this first increment is held in position with a holding instrument).
• Each succeeding step of the condenser overlaps (by half) the previous one as the condenser is moved toward the periphery.
• The gold moves under the nib face of the condenser, effecting compaction as malleting proceeds.

Physical Properties of a Compacted Gold


Material and technique Mpa KHN Apparent density (g/cm3)
Mat gold 169 53 14.5
Powdered gold 190 58 14.9
Gold foil 273 (highest) 69 15.8 (highest)
Mat gold and gold foil 227 75 (highest) 15

• Maximum density of gold without any voids is 19.3 g/cm3 (true density of pure gold).
• The restorations made with DFGs do not exhibit the high strength and hardness of those made with dental casting alloys.
• So they cannot be used for large stress bearing areas, such as a cast crown.
• So DFG is limited to areas where they can be used to simply fill a space rather than serve as a high-stress-bearing area.

https://t.me/DentalBooksWorld
Chapter 11 • Dental Materials 975

DENTAL COMPOSITE RESINS


TERMINOLOGIES
Activation—A process by which sufficient energy is provided to induce an initiator to generate free radicals and cause
polymerization to begin.
Activator—A source of energy used to activate an initiator and produce free radicals. Three energy sources are currently used
to dissociate an initiator into free radicals: (1) heat, which supplies thermal energy; (2) an electron-donating chemical such as
a tertiary amine, which forms a complex and reduces the necessary thermal energy to that available at ambient temperature;
and (3) visible light, which supplies energy for photoinitiation in the presence of a photosensitizer such as camphorquinone
(CQ).
C-factor—Configuration factor. This represents the ratio between the bonded surface area of a resin-based composite
restoration to the non-bonded or free surface area. The greater the C-factor, the greater the stress that develops at the restoration
margin, which can lead to gap formation, marginal breakdown and leakage, and other problems, such as secondary caries.
Chemically activated resin—A resin system consisting of two pastes—one containing an initiator (e.g., benzoyl peroxide)
and the other an activator (e.g., an aromatic tertiary amine)—which, when mixed together, release free radicals that initiate
polymerization.
Chemically cured composite—A particle-reinforced resin that is polymerized through a chemical activation process wherein
two components are blended together just prior to placement of the composite. Also known as self-cure composite.
Composite—In materials science, a solid formed from two or more distinct phases (e.g., filler particles dispersed in a polymer
matrix) that have been combined to produce properties superior to or intermediate to those of the individual constituents;
also a term used in dentistry to describe a dental composite or resin-based composite.
Coupling agent—A bonding agent applied to the surfaces of reinforcing particles (filler) to ensure that they are chemically
bonded to the resin matrix. Organo-silane compounds are the more common class of dental composite coupling agents.
Dental composite—Highly crosslinked polymeric materials reinforced by a dispersion of amorphous silica, glass, crystalline,
mineral, or organic resin filler particles and/or short fibers bonded to the matrix by a coupling agent.
Depth of cure—Depth or thickness of a light-cured resin that can be converted from a monomer to a polymer when exposed
to a light source under a specific set of conditions.
Dual-cure resin—Dental composite that contains both chemically activated and light-activated components to initiate
polymerization and potentially overcome the limitations of either the chemical- or light-cure systems when used alone.
Estrogenicity—Potential of synthetic chemicals with a binding affinity for estrogen receptors to cause reproductive alterations.
Bisphenol A, a precursor of certain monomers such as bis-GMA, is a known estrogenic compound that is considered to have
possible effects on fetal and infant brain development and behavior.
Filler—Inorganic, glass, and/or organic-resin particles that are dispersed in a resin matrix to increase rigidity, strength, and
wear resistance, to decrease thermal expansion, due to water sorption, and reduce polymerization shrinkage.
Flowable composite—Hybrid composite with reduced filler level and a narrower particle size distribution that increases flow
and promotes intimate adaptation to prepared tooth surfaces.
Gel point/gelation—The point in the polymerization reaction where sufficient cross-links have formed to produce a rigid,
glassy state in which internal flow among the developing polymer chains has stopped. All but highly local molecular motion
has stopped. After the gel point, stresses cannot be relieved but instead continue to increase and concentrate near the bonded
interfaces. Thus, reducing the polymerization rate provides more time for adjacent polymer chain segments to slip among
themselves, rearrange to lower energy configurations, and relieve developing stresses before the gel point is reached.
Hybrid composite—A particle-filled resin that contains a graded blend of two or more size ranges of filler particles to achieve
an optimal balance among the following properties: ease of manipulation, strength, modulus (relative rigidity), polymerization
shrinkage, wear resistance, appearance, and polishability. Nanohybrids contain at least one dispersed filler with particle sizes
of 100 nm or less (see nanofilled, below).
Inhibitor—A chemical added to resin systems to provide increased working time and extended storage life by minimizing
spontaneous polymerization, usually through free-radical scavenging mechanisms.
Initiator—A free radical-forming chemical used to start the polymerization reaction. It enters into the chemical reaction and
becomes part of the final polymer compound; thus, it is not a catalyst although often incorrectly labeled as such.

https://t.me/DentalBooksWorld
976 Triumph's Complete Review of Dentistry

Light-cured/photocured/photoinitiated composite—A particle-filled resin consisting of a single paste that becomes


polymerized through the use of a photosensitive initiator system (typically camphorquinone and an amine) and a light-source
activator (typically visible blue light). See also activator and initiator.
Matrix—A plastic resin material that forms a continuous phase upon curing and binds the reinforcing filler particles.
Microfilled composite—Composite reinforced with colloidal silica filler particles, approximately 40 nm in size, which can be
polished to a highly smooth surface. Microfillers are non-discrete nanometer-sized particles that are agglomerated into large
three-dimensional chainlike networks that drastically increase the monomer viscosity.
Nanofilled composite/nanocomposite—Composites with the same-size particles as microfilled composites, but the particles
have been surface-treated before they have agglomerated into large three-dimensional chainlike networks. Instead, they form
either isolated and/or loosely bound spheroidal agglomerates (called clusters) of primary nanoparticles. The surface treatment
allows an increase in filler loading by reducing the viscosity that develops when the particles are added to the monomer.
Cluster size may exceed 100 nm. All of the nanocomposites currently being marketed have average primary particle sizes in
the 40-nm range and so are of the same size as microfilled composites.
Ormocer—Ormocer is an acronym for organically modified ceramics, which are inorganic-organic copolymers. The organic,
reactive monomers are bound to an inorganic Si–O–Si network to reinforce the resin, reduce polymerization shrinkage, and
improve abrasion resistance.
Oxygen-inhibited layer—The thin surface region of a polymerized resin containing unreacted methacrylate groups arising
from dissolved oxygen, which acts to inhibit the free-radical polymerization curing reaction; also known as the air-inhibited
layer.
Packable composite—A hybrid resin composite designed for use in posterior areas where a stiffer consistency facilitates
condensation into a cavity form in a manner similar to that used for lathe-cut amalgams, also known as condensable composite.
Polyhedral oligomeric silsesquioxane (POSS)—A molecular-sized hybrid, organic–inorganic resin containing 12-sided
silicate cage structures that impart nanoparticle-like reinforcement.
Siloxane—A tetrafunctional epoxy siloxane monomer that cures via ring-opening polymerization. When the rings open, they
lengthen and occupy more space; the resulting expansion offsets a portion of the polymerization shrinkage.

INTRODUCTION
Dental resin-based composites are structures composed of three major components: a highly crosslinked polymeric matrix
reinforced by a dispersion of glass, mineral, or resin filler particles and/or short fibers bound to the matrix by coupling agents.
Such resins are used to restore and replace dental tissue lost through disease or trauma and to lute and cement crowns and
veneers and other indirectly made or prefabricated dental devices.

CLASSIFICATION
Classification of Resin Composites
1. Conventional or macrofilled resin composites had filler particles with a size of 10–40 μm and their disadvantages were
poor finish and relatively high wear.
• The most common used fillers in composites were quartz and strontium or barium glass.
• Quartz filler had good esthetics and durability but suffered from the absence of radiopacity and high wear of antagonist
teeth.
• Barium and strontium glass particles are radiopaque, but are unfortunately less stable than quartz.
2. Microfilled resin composites were introduced in the late 1970s.
• They contain colloidal silica filler with a particle size of 0.01–0.05 μm.
• The small size made it possible to polish the resin composite to a smooth surface finish.
• A problem was to obtain a high filler load. Compared to macrofilled resin composites, the microfilled did not have as
good physical properties.
• Hybrid resin composites were introduced to solve this and the shrinkage problem of resin composites.
• The first introduced hybrid resin composites contained large filler particles of a size of 15–20 μm as well as colloidal silica
of a particle size of 0.01–0.05 μm.

https://t.me/DentalBooksWorld
Chapter 11 • Dental Materials 977

3. Modern hybrid composites contain reduced submicron fillers.


• These composites are supposed to combine the advantages of macrofilled and microfilled composites, but they do not
have the final finish or translucency of microfilled resin composites.
• Nanocomposites are a recent development on the market.
• They contain filler particles with sizes less than 10 nm (0.01 μm) and are claimed to provide increased esthetics, strength,
and durability.

Activation of Resin Composites


• For the early resin composites, polymerization was initiated by mixing of two pastes.
• One paste contained an activator, such as a tertiary amine, which was used to split the initiator, usually benzoyl peroxide,
which was found in the second paste.
• The UV light, with a wavelength of approximately 365 nm, splits benzoin methyl ether into free radicals, without the
presence of tertiary amines, thus starting the polymerization.
• In this way, only one paste of composite was necessary and polymerization would not start until it was activated by the UV
light.
• In modern light-cured resin composites, the most common photo initiator is camphorquinone.
• It is sensitive to light with a wavelength in the blue range, approximately 420–490 nm with a peak at 468 nm.

APPLICATIONS
Dental applications for resin-based composites include
• cavity and crown restoration materials,
• adhesive bonding agents,
• pit and fissure sealants,
• endodontic sealants,
• of ceramic veneers,
• cementation for crowns,
• bridges, and
• other fixed prostheses

EVOLUTION OF COMPOSITES
Composite Refinements
Reviewing the last 55 years

Non-bonded Dentin-bonded
composites composites
Acid-etching and 3 part, 2 part, 1 part
enamel bonding Dentin bonding systems

1950 1960 1970 1980 1990 2000 2010

Original MACROFILL MACROFILL FLOWABLES NANOFILL and


development Self-cured composites NANOHYBRID composites
PACKABLES
composites
MIDIFILL MIDIFILL CONTROLLED/LOW
composites composites SHRINKAGE composites
MIDIHYBRID MINIHYBRID
Self-cured composites composites

UV-cured

Visible light-cured
(QTH, PAC, Laser, LED)

https://t.me/DentalBooksWorld
978 Triumph's Complete Review of Dentistry

COMPOSITION AND FUNCTION


Dental composites are made up of three major components:
• A highly crosslinked polymeric resin matrix reinforced by a dispersion of glass, silica, crystalline, metal oxide or resin-
reinforcing filler particles or their combinations, and/or short fibers, which are bonded to the matrix by silane coupling
agents.
• In addition, dental composites contain a number of other components, including an activator–initiator system that is
required to convert the resin paste from a soft, moldable filling material to a hard, durable restoration.
Pigments help to match the color of the tooth structure – Ultraviolet (UV) absorbers and other additives improve color
stability, and polymerization inhibitors extend storage life and provide increased working time for chemically activated resins.
Matrix – The resin matrix in most dental composites is based on a blend of aromatic and/or aliphatic dimethacrylate monomers
such as bis-GMA and urethane dimethacrylate to form highly crosslinked, strong, rigid, and durable polymer structures.
Filler – Various transparent mineral fillers are employed to strengthen and reinforce composites as well as to reduce curing
shrinkage and thermal expansion (generally between 30% and 70% by volume or 50% and 85% by weight of a composite).
These include so-called “soft glass” and borosilicate “hard glass,” fused quartz, aluminum silicate, lithium aluminum silicate
(beta-eucryptite, which has a negative coefficient of thermal expansion), ytterbium fluoride, and barium (Ba), strontium (Sr),
zirconium (Zr), and zinc glasses. The latter five types of fillers impart radiopacity because of their heavy metal atoms.
• Quartz had been used extensively as a filler in the early versions of dental composites.
• It has the advantage of being chemically inert but it is also very hard, making it abrasive to opposing teeth or restorations
as well as difficult to grind into very fine particles, so it is also difficult to polish. The so-called amorphous silica has the
same composition and refractive index as quartz; however, it is not crystalline and not as hard, thus, greatly reducing the
abrasiveness of the composite surface structure and improving its polishability.
For acceptable esthetics, the translucency of a composite restoration must be similar to that of tooth structure. Thus, the
index of refraction of the filler must closely match that of the resin. For bis-GMA and TEGDMA, the refractive indices are
approximately 1.55 and 1.46, respectively, and a mixture of the two components in equal proportions by weight yields a
refractive index of approximately 1.50.

Functions of Fillers
Fillers can provide the following benefits:
• Reinforcement: Increased filler loading generally increases physical and mechanical properties that determine clinical
performance and durability, such as compressive strength, tensile strength, modulus of elasticity (i.e., stiffness or rigidity),
and toughness. As the volume fraction of fillers approaches approximately 70%, abrasion and fracture resistance are raised
to levels approaching those of tooth tissue, thereby increasing both clinical performance and durability.
• Reduction of polymerization shrinkage/contraction: Increased filler loading reduces curing shrinkage in proportion to
the filler volume fraction.
• Reduction in thermal expansion and contraction: Increased filler loading decreases the overall coefficient of thermal
expansion of the composite because glass and ceramic fillers thermally expand and contract less than do polymers.
• Control of workability/viscosity: Fluid liquid monomer + filler → a paste. The more the filler is, the thicker the paste is.
• Decreased water sorption: Increased filler loading decreases water sorption. The absorbed water softens the resin and
makes it more prone to abrasive wear and staining.
• Imparting radiopacity: Resins are inherently radiolucent. Radiopacity is most often imparted by adding certain glass filler
particles containing heavy metal atoms, such as Ba, Sr, or Zn, and other heavy-metal/heavy-atom compounds such as YbF3,
which strongly absorb X-rays.
The traditional inorganic filler particles had average diameters of about 8–40  µm. Currently, small particles range from
0.005 µm to 2 µm. Particles larger than the wavelength of visible light cause light scattering.
However, the smaller the filler particle size, the higher the surface-to-volume ratio available to form polar or hydrogen bonds
with monomer molecules to inhibit their flow and increase viscosity (resistance to mixing and manipulation) and thus, the
less filler that can be added.

https://t.me/DentalBooksWorld
Chapter 11 • Dental Materials 979

Filler Particle size


Macrofillers 10–100 µm
Small/fine fillers 0.1–10 µm
Midfillers 1–10 µm
Minifillers 0.1–1 µm
Microfillers 0.01–0.1 µm (agglomerated)
Nanofillers 0.005–0.1 µm
5–100 mm – Nanoagglomerated

Advantages:
• Excellent esthetics
• Conservation of tooth structure
• Good longevity
• Can be repaired
• Bonds to tooth structure
• Complex tooth preparation needed
• Economic restorative material
Disadvantages:
• Technique sensitive
• Placement takes longer time than amalgams and other restorative materials
• Risk of microleakage and secondary caries
• Meticulous oral hygiene maintenance needed
• Lower fracture toughness and cannot be used in areas of high occlusal stress
• Biocompatibility issues with bisphenol A
• Polymerization shrinkage effects

Filler content
Composite classification Weight % Volume % Volume shrinkage (%) Average particle size (μm)
Hybrid 74–87 57–72 1.6–4.7 0.2–3.0
Nanohybrid 72–87 58–71 2.0–3.4 0.4–0.9 (macro)
– – – 0.015–0.05 (nano)
Microfills 35–80 20–59 2–3 0.04–0.75
Flowables 40–60 30–55 4–8 0.6–1.0
Compomers 59–77 43–61 2.6–3.4 0.7–0.8

PROPERTIES
Characteristic/property Unfilled Traditional Hybrid Hybrid (all- Microfilled Flowable Packable
acrylic (small purpose) hybrid hybrid
particle)
Size (µm) – 8–12 0.5–3 0.4–10 0.04–0.4 0.6–1.0 Fibrous
Inorganic filler (vol %) 0 60–70 65–77 60–65 20–59 30–55 48–67
Inorganic filler (wt %) 0 70–80 80–90 75–80 35–67 40–60 65–81
Compressive strength (Mpa) 70 250–300 350–400 300–350 250–350 – –
Tensile strength (Mpa) 24 50–65 75–90 40–50 30–50 – 40–45

https://t.me/DentalBooksWorld
980 Triumph's Complete Review of Dentistry

Elastic modulus (Gpa) 24 8–15 15–20 11–15 3–6 4–8 3–13


Thermal expansion coefficient 92.8 25–35 19–26 30–40 50–60 – –
(ppm/°C)
Water sorption (mg/cm) 1.7 0.5–0.7 0.5–0.6 0.5–0.7 1.4–1.7 – –
Knoop hardness (KHN) 15 55 50–60 50–60 25–35 – –
Curing shrinkage (vol %) 8–10 – 2–3 2–3 2–3 3–5 2–3
Radiopacity (mm Al) 0.1 2–3 2–3 2–4 0.5–2 1–4 2–3

BONDING AND BONDING AGENTS


INTRODUCTION
Bonding and adhesion comprise a complex set of physical, chemical, and mechanical mechanisms that allow the attachment
and binding of one substance to another.

HISTORICAL BACKGROUND
Non-bonded Acid-etching and Dentin-bonded 3 part, 2 part, 1 part Current
composites enamel bonding composites Dentin bonding systems time

1960 1970 1980 1990 2000 2010

Buonocore RL Bowen Fusayama Nakabayashi Kanka Self-etching and


reports acid- introduces NPG- introduces describes resin introduces self-adhesive bonding
etching of GMA as the 1st- Total-Etch infiltration of dentin wet systems developed
enamel generation of dentin denn collagen bonding
dentin bonding and enamel to form a technique
agent “hybrid” layer

Applications for Bonding


• Orthodontic bracket bonding
• Porcelain laminate veneer bonding
• Pit and fissure sealants
• Amalgam bonding
• Both enamel and dentin bonding

Mechanisms of Adhesion
If true adhesion of restorative materials to the tooth structure is to be achieved, three conditions must be satisfied:
1. Sound tooth structure must be conserved
2. Optimal retention must be achieved
3. Microleakage must be prevented
The fundamental mechanism of adhesion to the tooth structure can be regarded simply as an exchange by which the
inorganic tooth material (hydroxyapatite) is replaced by synthetic resins.
This process involves two parts:
1. Removing hydroxyapatite to create micropores
2. Infiltration of resin monomers into the micropores and subsequent polymerization.
As a result, resin tags (extension of resins that have penetrated into the etched enamel or dentin) are formed that
micromechanically interlock or interpenetrate with the hard tissue.

https://t.me/DentalBooksWorld
Chapter 11 • Dental Materials 981

1. Surface energy and wetting


2. Interpenetration (formation of a hybrid zone)
3. Micromechanical interlocking
4. Chemical bonding

Wetting is the essential first step for the success of all adhesion mechanisms. Wettability of a liquid on a solid can be
characterized by the contact angle that forms between a liquid and solid, as measured within the liquid.
Categories of wettability include
“mostly non-wetting” – (>90°),
“absolutely no wetting” – (180°),
“mostly wetting” – (<90°), and
“absolute wetting” – (0°)
• Wettability can be enhanced by increasing the surface energy of the substrates (e.g., dentin, enamel, and synthetic materials).
• Since a clean, microroughened tooth surface has a higher surface energy than unprepared tooth surfaces, organic adhesives
are inherently able to wet and spread over such a surface unless a low surface tension material contaminates it before the
adhesive can be applied.
• The acid-etch technique, by which contaminants are removed and microporosities are created, is widely used to generate
high-energy tooth surfaces and promote wetting by adhesive monomers.
• Enamel and dentin are hydrated, hydrophobic, and permeable to water.

Smear Layer
• Whenever both enamel and dentin tissues are mechanically cut, especially with a rotary instrument, a layer of adherent
grinding debris and organic film known as a smear layer is left on their surfaces and prevents strong bonding.
• Forms – 1–5 µm of thickness.
• Different quantities and qualities of the smear layer are produced by the various cutting and instrumentation techniques, as
occurs, for example, during cavity or root canal preparation.
• In dentin, the smear layer becomes burnished into the underlying dentinal tubules and lowers dentin permeability, which
is a protective effect.
• However, it is also a very weak cohesive material and interferes with strong bonding. Therefore, various cleaning or treatment
agents and procedures are employed to either remove the smear layer or enhance its cohesive strength and other properties.

Acid-Etch Technique
Enamel Etching
• The first meaningful demonstration of intraoral adhesion was reported by Michael Buonocore (1955).
• Buonocore etched enamel surfaces with various acids, placed an acrylic restorative material on the micromechanically
roughened surfaces, and found a great increase in the resin–enamel bond strength (~20 MPa).
• One of the surface conditioning agents he used, phosphoric acid, is still the most widely used etchant today for bonding to
both enamel and dentin.
• Depending on the concentration, phosphoric acid removes the smear layer and about 10 microns of enamel to expose
prisms of enamel rods to create a honeycomb-like, high-energy retentive surface.
• The higher surface energy ensures that resin monomers will readily wet the surface, infiltrate into the micropores, and
polymerize to form resin tags.
The pattern of etching enamel may vary from selective dissolution of either the enamel rod centers:
1. Type I etching – Dissolution of prism cores (honey comb appearance)
2. Type II etching – Dissolution of prism peripheries (cobble stone appearance)

https://t.me/DentalBooksWorld
982 Triumph's Complete Review of Dentistry

3. Type III etching – Combination of type I and II pattern


4. Type IV – Pitted appearance (unfinished puzzle, map-like appearance)
5. Type V – Smooth appearance
Prior to the introduction of enamel acid etching and the use of enamel bonding agents, restorative materials were placed
directly on the smear layer of the prepared tooth. The apparent bond strength is the cohesive strength (5–10 MPa) of the
smear layer, which is not sufficient to withstand the daily mechanical forces experienced in the mouth.
Dentin Etching
• Dentin etching did not gain wide acceptance until Fusayama introduced the total-etch concept in 1979.
• Both dentin and enamel are etched simultaneously, typically using 37% phosphoric acid. His study demonstrated that not
only was restoration retention substantially increased but also pulp damage did not occur as had been generally assumed.
• A subsequent study by Nakabayashi et al. (1984) revealed that hydrophilic resins can infiltrate the surface layer of acid-
demineralized collagen fibers that is produced in etched dentin and it can form a layer of resin-infiltrated dentin with high
cohesive strength.
• Such a hybrid layer structure forms very strong resin bonds through the development of an interpenetrating network of
polymers and dentinal collagens, together with numerous micromechanical interlocks at the resin–hybrid layer interface.
• Since the total-etch technique usually involves etching with an acid followed by rinsing to remove the acid, this technique
is also known as the etch-and-rinse technique.

Etching Time
• The optimal application time for the etchant may vary somewhat, depending on previous exposure of the tooth surface to
fluoride and other factors.
• For example, a permanent tooth with a high fluoride content may require a somewhat longer etching time, as do primary teeth.
• In the latter, increased surface conditioning time is needed to enhance the etching pattern on primary tooth enamel that is
more aprismatic than permanent tooth enamel.
• Currently, the etching time for most etching gels is approximately 15 sec.
• The advantage of such short etching times is that they yield acceptable bond strength in most instances, while conserving
enamel and reducing treatment time.

Rinsing and Drying Stage


Once the tooth is etched, the acid should be rinsed away thoroughly with a stream of water for about 20 sec, and the rinsed
water must be removed.
Cleanness of the Bonding Surfaces
If contamination occurs, the contaminant should be removed, and the surface should be etched again for 10 sec.

Dentin Bonding Agents


Dental bonding agents are designed to provide a sufficiently strong interface between restorative composites and tooth
structure to withstand mechanical forces and shrinkage stress. The success of adhesives is dependent on two types of bonding:
1. Micromechanical interlocking, chemical bonding with enamel and dentin, or both
2. Copolymerization with the resin matrix of composite materials

A successful dentin bonding system must meet several requirements:


• Adequate removal or dissolution of the smear layer from enamel and dentin
• Maintenance or reconstitution of the dentin collagen matrix
• Good wetting
• Efficient monomer diffusion and penetration
• Polymerization within the tooth structure
• Copolymerization with the resin composite matrix

https://t.me/DentalBooksWorld
Chapter 11 • Dental Materials 983

CLASSIFICATION
Classification of Dental Bonding System
Etch–and–rinse Self-etch
Three-step (fourth Two-step (fifth Two-step (sixth generation) One-step (all-in-one)
generation) generation) • Does not involve a separate (seventh generation)
• Combines the primer etching step • This category combines the
and adhesive resin into • In this case, an acidic conditioner, the primer, and
one application monomer, which is not the bonding resin into a
rinsed, is used to condition single step
and prime the tooth at the • There is no need for rinsing
same time or drying of the tooth
• There are two types of structure because of the lack
self-etch adhesives (Van of an etch step
Meerbeek et al., 2001) • Recently, a new self-etch
• Mild and strong varieties adhesive bonding agent
• Strong – pH < 1 has been introduced that
expands this category for use
• Mild – pH = 2 – Only
with total etch procedures
partially dissolve dentin
(Scotchbond Universal
– So some amount of
Adhesive, 3M ESPE, St. Paul,
hydroxyapatite crystals
MN, United States)
remains available within the
hybrid layer…now specific • This system contains
carboxyl or phosphate phosphate monomer and
groups of functional silane and is claimed to offer
monomers can then extended bond durability as
chemically interact with this well as versatility for use in
residual hydroxyapatite various clinical procedures
• Because this layer has some
mineral content, the bond to
dentin is better than that of
etch-and-rinse adhesives
Etch Etch (30–40% phosphoric Etch and Prime Etch, Prime, and Bond
Apply for 15 sec, rinse 15 acid – 37% usually used – If One application without Apply 1–5 layers without
sec, gently air dry while used beyond 50%, it forms rinsing, gently air – dry rinsing, gently air – dry, light
keeping dentin moist monocalcium phosphate cure
monohydrate layer on the
etched surface
Apply for 15 sec, rinse
15 sec, gently air dry while
keeping dentin moist
Primer Prime and bond Bond
It is hydrophilic Apply 1–5 layers, gently Apply one layer, gently air–dry,
Apply 1–5 layers, gently air–dry, light cure light cure
air–dry
Bond
Resin is hydrophobic
Apply one layer, gently air–
dry, light–cure

https://t.me/DentalBooksWorld
984 Triumph's Complete Review of Dentistry

Important Facts
• Enamel – It promotes the dissolution of enamel rods.
• Dentin – Bonding is more difficult and less predictable than in enamel because of the organic characteristics of dentin. In
this case, phosphoric acid treatment exposes a collagen network that is nearly devoid of hydroxyapatite.
• Bonding occurs by diffusion and infiltration of the resin within the collagen mesh, forming a hybrid layer.
• After in situ polymerization, this hybrid layer provides micromechanical retention to the restoration.
• True chemical – Adhesive bonding is unlikely to contribute significantly to bond strength, as the monomer functional
group has only very weak affinities for collagens.
• In the two-step category, hydrophilic and ionic monomers are combined, with the result that the bonded interface does not
develop a hydrophobic resin layer and thus, leaves the bond susceptible to water penetration and subsequent degradation,
which greatly reduces the bond durability.

Measurement of Bond Strength


• Although no universal agreement on the minimal bond strength necessary to provide successful bonding has been
established, a value of 20 Mpa or higher has been proposed as a reasonable goal.
• By using very small specimens (approximately 1 mm square in cross-section), one can prepare multiple specimens from a
single tooth. Even with very high indicated strengths, for example, 30 Mpa or higher, failure occurs at the adhesive interface
rather than within the dentin.

Bond Strength of Various Bonding Agents


First generation 2–3 Mpa
Second generation 4.5–6 Mpa
Third generation 16–26 Mpa
Fourth generation 25 Mpa*
Fifth generation Relatively higher but less than the fourth generation
Sixth generation Lower than the fourth and the fifth generation
Seventh generation Equal to the sixth generation
*Highly effective with shear bond strengths of 25 MPa to both enamel and dentin. Fusayama and Nakabayashi also described
the penetration of resins into dentin as giving dentinal seals with high bond strengths. Kanca also introduced the “wet bonding”
concept with these systems

Aging Effects and Degradation of the Hybrid Layer


• The change of bond strength over time is a very useful indication of relative clinical lifetime and can be evaluated by
accelerated aging tests in vitro.
• Several investigations have found that the bond strength of three-step adhesive systems show little or no decrease in contrast
to two-step adhesive systems that decrease significantly during a 4–5 years span.
• The hybrid layer thickness is 0.5–5 μm.

The Clinical Application Determines the Best Choice of an Adhesive System


Generation Recommendation
Etch-and-rinse (fourth generation) Bonding composite cores
Two-step etch-and-rinse (fifth generation) Bonding anterior and posterior composites and cementation of
veneers with resin cements
Self-etch, two-step (sixth generation) Bonding posterior composites
Dual-cure one-step, self-etch (seventh generation) Esthetic posts and ceramic restorations bonded with resin cement
Light-cured one-step, self-etch (seventh generation) Bonding posterior composite restorations

https://t.me/DentalBooksWorld
Chapter 11 • Dental Materials 985

Other Facts From Articles


It is very important for the clinicians to recognize differences in dentin composition before planning restorations depending
on the dentin bonding.
Duke proposed a visual classification for dentin sclerosis.
• Category 1
If dentin nominally appears light yellow or whitish and is opaque, then there is no sclerosis present.
• Category 2
More than category 1 but <50% of way between 1 and 4.
• Category 3
Less than category 4 >50% of way between categories 1 and 4.
• Category 4
If dentin is dark yellow or discolored and appears glassy or translucent, then there is a significant sclerosis. It is found more in
older individuals.

Facts About Cementation


• For cementation of all-ceramic restorations, resin (can be chemically or light or dual cured) cement is the material of choice
(Anusavice, 12th edition, p. 269).
• Self-adhesive resin cements are dual-cured materials and can be used for cementation of indirect restorations in most clinical
situations.
• For cementation of veneers, light-cured resin cements are preferred.
• Self-adhesives are not recommended by manufactures as luting agents for orthodontic fixed appliances.
• Self-adhesive cements exhibit a lower bond strength to intact enamel in comparison with conventional orthodontic resin
cements that require phosphoric acid etching.
• If doubt exists about whether the pit and fissure is free from caries or not, it is still justified to place a sealant.

Clinical trials in which sealants were intentionally placed in pits and fissures that were diagnosed as having caries have shown
that as long as a sealant is well retained, no caries progression will occur.

DENTAL CASTING ALLOYS


HISTORY
Historical Perspective on Dental Casting Alloys
1907: The lost wax process
Taggart described a method of making gold inlays using the lost wax technique.
1932: Classification of gold-based casting alloys
The dental materials group at the National Bureau of Standards (now National Institute of Standards and Technology)
evaluated the properties of alloys being used and classified them by their Vickers hardness number (VHN):
Type I (soft, VHN, 50–90)
Type II (medium, VHN, 90–120)
Type III (hard, VHN, 120–150), and
Type IV (extra hard, VHN 150, and above)
1933: Cobalt–chromium and nickel–chromium alloys
The advantages of cobalt–chromium and nickel–chromium alloys are their lighter weight, greater stiffness (elastic modulus),
greater strength, and reduced cost.
1959: Porcelain-fused-to-metal prostheses
1971: The end of the Bretton Wood’s system
The Bretton Woods system, implemented after World War II, was an international monetary framework of fixed exchange
rates between gold and currency. The system ended on August 15, 1971, a step that completely abandoned the gold standard.

https://t.me/DentalBooksWorld
986 Triumph's Complete Review of Dentistry

CLASSIFICATION
Classification of Dental Casting Alloys
The carat system specifies the gold content of an alloy based on parts of gold per 24 parts of the alloy.
Fineness is the unit that describes the gold content in noble metal alloys by the number of parts of gold per 1,000 parts of the
alloy. For example, pure gold is 24-carat or 1,000 fine, whereas an 18-carat alloy contains 75% pure gold and is 750 fine.
The terms carat and fineness are rarely used to describe the gold content of current dental alloys. However, fineness is often
used to identify gold alloy solders.
Since the classification of gold alloys was established in 1932 by the National Bureau of Standards, the number of alloy
compositions has increased astronomically.
Dental alloys currently available for dental castings can be classified according to their composition, their intended usage, or
their mechanical properties.

Alloy Classification by Noble Metal Content


In 1984, ADA proposed a simple classification for dental casting alloys based on the content of noble metals.
Three categories were described: high noble (HN), noble (N), and predominantly base metal (PB).
The noble metals and silver are sometimes called precious metals, referring to their high economic values, but the term
precious is not synonymous with noble.
Silver is reactive in the oral cavity and is not considered a noble metal.
Alloy Classification by Noble Metal Content—ADA (1984)
Alloy type Total noble metal content
High noble (HN) Must contain ≥40% Au and ≥60% by weight of noble metal elements*
Noble (N) Must contain ≥25% by weight of noble metal elements
Predominantly base metal (PB) Contains <25% by weight of noble metal elements
*Noble metal elements include Au, Pd, Pt, Rh, Ru, Ir, and Os.

PROPERTIES
Mechanical Property Requirements Proposed in ISO Draft International Standard 1562 for Casting Gold
Alloys (2002)
Type Descriptor Yield strength (MPa) Elongation (%) Examples of applications
1. Low 80 18 Inlays
2. Medium 180 10 Inlays and onlays
3. Hard Onlays, thin cast backings, pontics, full crowns, and
270 5
saddles
4. Extra hard Saddles, bars, claps, crowns, bridges, and partial denture
360 3
frameworks

Desirable Properties of Dental Casting Alloys


1. Biocompatibility
• The alloy must tolerate oral fluids and not release any harmful products into the oral environment.
• Allergic reactions, however, are peculiar to the individual patient, and the practicing dentist has an obligation,
morally and legally, to minimize this risk.
2. Tarnish and corrosion resistance
• Corrosion is the physical dissolution of a material in the oral environment and tarnish is a thin film of a surface
deposit that is adherent to the metal surface.
• Corrosion resistance is derived from the use of noble metals that do not react in the oral environment (e.g., gold and
palladium) or by the ability of one or more of the metallic elements to form an adherent passivating surface film,
which inhibits any subsurface reaction (e.g., chromium and titanium).

https://t.me/DentalBooksWorld
Chapter 11 • Dental Materials 987

3. Thermal properties
• To achieve an accurate fit of cast prostheses, oversized dies for waxing and controlled mold expansion are needed to
compensate for casting shrinkage of the alloy and provide space for the luting cement.
• For metal-ceramic prostheses, the alloys must have closely matching thermal expansion coefficients to be compatible
with given porcelains, and they must tolerate high processing temperatures without deforming via a creep process.
4. Strength requirements
• Alloys for metal-ceramic prostheses are finished in thin sections and require sufficient stiffness to prevent excessive
elastic deflection from functional forces, especially when they are used for long-span frameworks.
5. Fabrication of cast prostheses and frameworks
• The molten alloy should flow freely into the most intricate regions of the investment mold, without any appreciable
interaction with the investment material, and wet the mold surface without forming porosity within the surface or
subsurface regions of the alloy.
• This property is also termed castability, which is measured by percent completion of a cast mesh screen pattern or
other castability patterns.
• Cutting, grinding, finishing, and polishing are the necessary steps in obtaining a prosthesis with a satisfactory surface
finish. The hardness of an alloy is a good primary indicator of the likely difficulty of cutting and grinding it.

Alloys for All-Metal Prostheses


• These alloys are discussed in three main categories:
• Noble (includes high noble), predominantly base metal, and CP Ti and titanium alloys.
• Titanium alloys are discussed separately, as they possess properties that are different from those of other base metal alloys.
• The compositions of selected high noble and noble alloys and their physical and mechanical properties are below.
• Since some noble metals contain no gold and rely on palladium for corrosion resistance, these alloys are discussed in two
categories: Au-based alloys and Ag-Pd alloys.
• Typical compositions of high noble and noble alloys for all-metal prostheses.

Gold-Based Alloys
• These alloys are generally yellow in color.
• Type 1 gold alloys are soft and designed for inlays supported by teeth and not subjected to significant mastication forces.
• Type 2 alloys are widely used for inlays because of their superior mechanical properties, but they have less ductility than
type 1 alloys.
• Type 3 alloys are used for constructing crowns and onlays for high-stress areas. Increasing the Pt or Pd content raises the
melting temperature, which is beneficial when components are to be joined by soldering (or brazing).
• Type 4 gold alloys are used in high-stress areas such as bridges and partial denture frameworks. The cast alloy must be rigid
to resist flexure, possess high yield strength to prevent permanent distortion, and be ductile enough for adjustment if the
clasp of a framework has been distorted or needs adjustment.
• Changes of alloy color caused by the reduction in gold content are compensated for by an increase in copper, silver, and
palladium.
• Higher silver and copper content reduces the corrosion resistance of these alloys.
• These reduced gold alloys have moderate moduli of elasticity but a higher hardness and yield strength than their high noble
counterparts.

Heat Treatment of Gold–Copper Alloys


• To soften the alloy, the casting is placed in a furnace for 10 min at 700°C and then quenched in water.
• All intermediate phases in the alloy are changed to a disordered solid solution at 700°C, and the rapid quenching prevents
ordering from occurring during cooling.
• The tensile strength, proportional limit, and hardness are reduced by such a treatment, and the ductility is increased.

https://t.me/DentalBooksWorld
988 Triumph's Complete Review of Dentistry

• To harden the alloy, the temperature of the furnace is set between 200°C and 450°C and the casting is heated for 15–30 min
before it is quenched in water.
• Before the alloy is age-hardened, it should be subjected to a softening heat treatment to relieve all residual strain hardening
before the alloy is hardened again by heat treatment to produce a disordered solid solution.
• Otherwise the amount of solid-state transformation will not be properly controlled.
• In metallurgical terminology, the softening heat treatment is referred to as a solution heat treatment and the hardening heat
treatment is termed age hardening.
• The softening heat treatment is indicated for structures that are grounded or reshaped plastically to a different form, either
in or out of the mouth.
• Because the proportional limit is increased during age hardening, a considerable increase in the modulus of resilience can
be expected.
• The hardening heat treatment is indicated for metallic partial dentures, saddles, FDPs, and other similar structures where
rigidity of the prosthesis is needed. For small structures, such as inlays, a hardening treatment is not usually required.
• Age hardening reduces the ductility of gold alloys.
• A reasonable amount of ductility is essential if the clinical application requires some permanent deformation of the as-cast
structure, as is needed for clasp and margin adjustment and for burnishing.

Silver–Palladium Alloys
• These alloys are white and predominantly silver in composition, but they contain at least 25% of palladium to provide
nobility and increase the tarnish resistance of the alloy.
• They may also contain copper and a small amount of gold.
• Casting temperatures are in the range of those for yellow gold alloys.
• The copper-free Ag–Pd alloys may have physical properties similar to those of a type 3 gold alloy.
• With 15% or more copper, the alloy may have properties more like those of a type 4 gold alloy.
• Despite reports of poor castability because of the lower density and propensity of dissolving oxygen in the molten state, Ag–Pd
alloys can produce acceptable castings when close attention is paid to precise control of the casting and mold temperatures.
• The major limitation of Ag–Pd alloys in general and in the Ag–Pd–Cu alloys in particular is their greater potential for
tarnish and corrosion.
• The amount of corrosion expected during service is negligible if the palladium content is greater than 25%.
• The tarnish resistance of the alloys is especially dependent upon the composition and the integrity of the casting.
• The colored phase of the Pd–In binary alloy system is hard and brittle and is not a strengthener.
• Silver, copper, and/or gold can be added to increase the ductility and improve the castability of the alloy for dental
applications.

Predominantly Base Metals


• Base metal alloys generally comprise the group of cast metals that rely on chromium for corrosion resistance.
• Chromium on the surface of the alloy rapidly oxidizes to form a thin layer of chromium oxide, which prevents the diffusion
of oxygen into the underlying metals and improves its corrosion resistance.
• Chromium also strengthens the alloy by solution hardening.
• Because of the high cost of noble metals, these base metals have also been adapted for dual applications such as the
production of all-metal and metal-ceramic prostheses.
• Currently, there are two main groups of base metal dental alloys:
–– Nickel–chromium (Ni–Cr)
–– Cobalt–chromium (Co–Cr).

https://t.me/DentalBooksWorld
Chapter 11 • Dental Materials 989

• The Ni–Cr alloys can be further divided into those with and without beryllium, which improves castability and promotes
the formation of a stable metal oxide for porcelain bonding.
• The majority of Ni–Cr alloys are for small castings such as crowns and FDPs, and Co–Cr alloys are primarily used for
casting removable partial dentures in which high elastic modulus and yield strength are needed.
• Some Ni–Cr alloys, which are used for partial denture frameworks, are formulated for their relative ease of finishing and
polishing compared with Co–Cr alloys, which are used for crowns and FDPs in spite of their low ductility.
• Molybdenum increases corrosion resistance and strength and decreases the thermal expansion coefficient of base metal
alloys.
• The latter is beneficial for porcelain bonding and minimizes the risk of porcelain cracking or fracture.

DENTAL CEMENTS
INTRODUCTION
Dental cements are restorative materials which are used to fill the prepared cavities.

Protective Materials
Bases 1–2 mm – Pulpal, thermal, and chemical protection
Liners
  Thick 1–50 µm
  Thin 0.2–1 mm – Pulpal and thermal protection
Thin liners
  Suspensionliners 20–25 μm
  Solution liners 2–5 µm
Cavity varnish Protects pulp by sealing the tubule denying entry of irritants

Highests and Lowests


Highest solubility Calcium hydroxide, GIC, and silicate
Lowest solubility Resin cements; ZnPo4 has low solubility in water
Highest irritant to pulp Silicate and zinc phosphate
Lowest irritant to pulp Zinc oxide eugenol
High compressive strength Silicate, resin, and zinc phosphate
Low compressive strength Calcium hydroxide
High setting time Zinc oxide eugenol, GIC, and Zn polycarboxylate
Low setting time Resin cements
High tensile strength Hybrid ionomer and GIC type 2
Low tensile strength Silicate and zinc oxide eugenol
High pH Zinc oxide eugenol
Low pH Zinc phosphate and silicate
High anticariogenic property Silicate, GIC, and cermet
Low anticariogenic property Zinc phosphate

https://t.me/DentalBooksWorld
990 Triumph's Complete Review of Dentistry

PROPERTIES
General Properties
Film thickness 24-hour compressive 24-hour tensile
Cements Setting time
(max value) strength strength
Zinc phosphate 2.5–8 min (5.5 min) 20 µm 115.5 Mpa 5.5 Mpa
Zinc silico phosphate 4 min 25 µm 145 Mpa 3.5–7.6 Mpa
Zinc polycarboxylate 6–9 min 21 µm 55 Mpa 6.2 Mpa
Zinc oxide eugenol type 1 4–10 min 25 µm 6–28 Mpa –
Zinc oxide eugenol type 2 (EBA) 9.5 min 25 µm 55 Mpa 4.1 Mpa
Resin cements 2–4 min <25 µm 70–172 Mpa –
86 Mpa – Type 1
GIC 7 min 24 µm 6.2 Mpa
150 Mpa – Type 2

pH of Dental Cements
Time Zinc phosphate Zinc silico Zinc polycarboxylate GIC
phosphate
2 min 2.14 1.43 3.42 2.33
1 hour 4.34 3.60 5.08 4.55
24 hours 5.50 5.50 5.94 5.67

POSITIVES AND NEGATIVES OF DENTAL CEMENTS


Zinc Phosphate
Positives:
• Most commonly used cement
• It has an acceptable compressive strength (double than that of GIC)
Negatives:
• Pulpal irritation
• Lack of adhesiveness
• Lack of anti-cariogenic properties
• Not suitable when mechanical retention is poor or when esthetic demand is high

Zinc Oxide Eugenol Cement


Positives:
• Biocompatibility
• Alkaline pH
Negatives:
• Low compressive strength

Zinc Polycarboxylate Cement


Positives:
• Blandness to the pulp
• Adhesive to the tooth structure
It is used in postoperative sensitivity

https://t.me/DentalBooksWorld
Chapter 11 • Dental Materials 991

Negatives:
• Short working time (so limits their use to single-unit or three-unit FPDs)
• No fluoride release

GIC
Positives:
• Greater resistance to degradation in oral cavity
• Fluoride release
Negatives:
• Lower elastic modulus
• Low stiffness (so excessive elastic deformation of ceramic prosthesis which may result in fracture)

Compomers
Negatives:
• Used only for low-stress areas

Resin Cements
Positives:
• Used when esthetic perfection is required
Negatives:
• Has poor handling properties

DESCRIPTION ABOUT INDIVIDUAL DENTAL CEMENT


Zinc Phosphate Cement
Composition:
Powder Liquid
Zinc oxide – 87% Phosphoric acid
Magnesium oxide – 13% Acid content – 33 ± 5%
• Zinc oxide is sintered at 1,000–1,400°C and made into powder
• Reaction is acid base
• The set cement consists of unreacted zinc oxide particles embedded in a cohesive amorphous matrix of zinc aluminum
phosphate
• Only mechanical bonding (no chemical bonding)
• The setting time can be measured at a load of 400 g
• The recommended powder liquid ratio is 1.4 mg powder to 0.5 ml liquid
• The working time of the cement is 5 min
• The setting time can be extended by:
–– Reducing powder liquid ratio
–– Mixing cements in increments and introducing in smaller quantities of powder into liquid
–– Prolonging spatulation time
–– Regulating the setting time of the mixing slab – Cool glass slab prolongs the setting time
–– The smaller the particle size is, the faster the cement will set

https://t.me/DentalBooksWorld
992 Triumph's Complete Review of Dentistry

Zinc Polycarboxylate Cement


Composition:
Powder Liquid
Zinc oxide Polyacrylic acid
Stannous fluoride Acid concentration varies from 45 to 65%
• Fluoride release from this cement contributes to only 15–20% and the main purpose of stannous fluoride is to increase
its setting time and strength
• Bonds chemically to the tooth structure
• It is a pseudoplastic cement and it undergoes thinning at an increased shear rate
• Its main disadvantage is that it has a short working time (only about 2.5 min)
• pH of the cement liquid is 1.7
• Excellent biocompatibility
• Powder liquid ratio – 1.5 parts of powder to 1 part of liquid by weight
• Glossy appearance after the mix indicates that the cement mixture is good and it will bond well where as dull looking
mixture does not bond well to the tooth structure
• Minimal irritation to the pulp

GIC
• Maximum particle size – 50 µm
• Particle size for luting/cementing agents – 15 µm
Composition of glass used in GIC:
• SiO2
• Al2O3
• CaF2
Composition of the liquid used in GIC:
• Polyacrylic acid of 40–50%
• Tartaric acid
• Itaconic and maleic acid
The purpose of tartaric acid is that it improves handling characteristics, increases working time, shortens working time,
decreases viscosity, and increases shelf-life
Classification:
Types Uses
Type 1 Luting and orthodontic brackets
Type 2A Esthetic restoration
Type 2B Reinforced restoration
Type 3 Lining cements and bases
• Reaction is chelation.
• Na and F do not participate in crosslinking of the cement.

GYPSUM-BONDED INVESTMENT
• The most common method used to form metal inlays, onlays, crowns, bridges, and other metal frameworks is to cast
molten alloys by centrifugal force, under pressure, or under vacuum and pressure into a mold cavity.
• The material used for the mold must be sufficiently refractory and thermally stable that it can withstand exposure to the
high temperatures of the molten metal as the metal solidifies and cools to room temperature.
• The mold cavity is produced by eliminating a wax or resin pattern by heating the mold to a specific temperature and for a
specific time. This is called the burnout process.

https://t.me/DentalBooksWorld
Chapter 11 • Dental Materials 993

• To provide a pathway to the mold cavity for the molten metal, the wax or resin pattern must have one or more cylindrical
wax segments attached at the desired point(s) of metal entry; this arrangement is termed a sprued wax pattern.
• A sprue is the channel in a refractory investment mold through which the molten metal flows.
• After the wax pattern has been made, either directly on a prepared tooth or on a replica die of the tooth, a sprue former base
is attached to the sprued wax pattern, an investment ring is pressed into the sprue former base, and an investment slurry is
vibrated into the ring to embed the wax pattern in the investment.
• Generally, two types of investments—gypsum-bonded and phosphate-bonded—are employed, depending on the melting
range of the alloy to be cast.
• The gypsum-based materials represent the type traditionally used for conventional casting of gold alloy inlays, onlays,
crowns, and larger fixed dental prostheses (FDPs).
• Phosphate-based investments are designed primarily for alloys used to produce copings or frameworks for metal-ceramic
prostheses and some base metal alloys.
• It can also be used for pressable ceramics.
• A third type is the ethyl silicate-bonded investment, which is used principally for the casting of removable partial dentures
made from base metals (cobalt-based and nickel-based alloys).
• Commercially pure titanium and titanium alloys require a special investment as well as a controlled atmosphere to achieve
satisfactory castings.
• The type of investment used depends on whether the appliance to be fabricated is fixed or removable and on the method of
obtaining the expansion required to compensate for the contraction of the molten alloy during solidification.
• Type I investments are those employed for the casting of inlays or crowns when the compensation for alloy casting shrinkage
is accomplished principally by thermal expansion of the investment.
• Type II investments are also used for casting inlays, onlays, or crowns, but the major mode of compensation for alloy
shrinkage during solidification is by hygroscopic expansion achieved by immersing the invested ring in a warm water bath.
• Burnout of the investment is performed at a lower temperature than that used for the high-heat burnout technique.
• Type III investments are used rarely in the construction of partial dentures because they are designed for casting gold alloys.
This chapter focuses primarily on type I and type II investments.

COMPOSITION
• The ingredients of dental inlay investments employed with conventional gold casting alloys are α-hemihydrate of gypsum,
and quartz, or cristobalite, which are forms of silica.
• Most investments contain the α-hemihydrate of gypsum because of its greater strength.
• This gypsum product serves as a binder for the other ingredients and to provide rigidity.
• The strength of the investment is dependent on the amount of binder used. The investment powder may contain 25–45% of
calcium sulfate hemihydrate. The remainder consists of silica allotropes and controlling chemicals.

Gypsum-Bonded Investments
• The α-hemihydrate form of gypsum is generally the binder for investments used in casting gold-containing alloys with
melting ranges below 1,000°C.
• When this material is heated at temperatures sufficiently high to completely dehydrate the investment and to ensure
complete castings, it shrinks considerably and occasionally fractures.
• All forms shrink considerably after dehydration between 200°C and 400°C. A slight expansion takes place between 400°C
and approximately 700°C, and a large contraction then occurs.
• This latter shrinkage is most likely caused by decomposition and the release of sulfur dioxide. This decomposition not only causes
shrinkage but also contaminates the castings with the sulfides of the non-noble alloying elements, such as silver and copper.
• Thus, it is imperative that gypsum investments not be heated above 700°C. However, for gypsum products containing
carbon, the maximum temperature is 650°C.
• Usually, castings made from pure gypsum (CaSO4•2H2O) molds are extremely undersized. The α-hemihydrate product,
which requires less mixing water and shrinks less, is the optimal choice as a binder.

https://t.me/DentalBooksWorld
994 Triumph's Complete Review of Dentistry

0.4

0.0
Type IV Stone
W/P - .22
−0.4

−0.8

Expansion (%)
Type III Stone
W/P - .30
−1.2
Plaster
−1.6 W/P - .50

−2.0

−2.4

−2.8
0 200 400 600 800 1000
Temperature (°C)

Dimensional change of three forms of gypsum when heated.

Silica
• When quartz, tridymite, or cristobalite is heated, a change in crystalline form occurs at a transition temperature
characteristic of the particular form of silica.
• For example, when quartz is heated, it inverts (transforms) reversibly from a “low” room-temperature crystal form, known
as α quartz, to a “high” form, called β quartz, at a temperature of 573°C.
• This α-to-β phase transformation is called an inversion, and it is accompanied by a linear expansion of 0.45%. In a similar
manner, cristobalite undergoes an analogous transition between 200°C and 270°C from “low” (α cristobalite) to “high” (β
cristobalite).
• Two inversions of tridymite occur at 117°C and 163°C, respectively.
• The β-allotropic forms are stable only above the transition temperature noted, and an inversion to the lower α form
occurs on cooling in each case.
• In powdered form, the inversions occur over a range of temperature rather than instantaneously at a specific temperature.
• The density decreases as the α form changes to the β form, with a resulting increase in volume that occurs by a rapid
increase in the linear expansion.
2.0
Cristobalite
1.6
Quartz
Thermal Expansion (%)

1.2
Tridmite
0.8

0.4

Fused quartz
0.0
0 100 200 300 400 500 600 700
Temperature (°C)

Thermal expansion of four forms of silica.

https://t.me/DentalBooksWorld
Chapter 11 • Dental Materials 995

Modifiers
• In addition to silica, certain modifying agents, coloring matter, and reducing agents, such as carbon and powdered copper,
are present.
• The reducing agents are used in some investments to provide a non-oxidizing atmosphere in the mold when a gold alloy is
cast.
• Unlike the dental stones, a setting expansion is usually desirable to assist in compensating for the contraction of the alloy.
Some of the added modifiers—such as alkali-earth and transition-metal chlorides, boric acid, and sodium chloride—not
only regulate the setting expansion and the setting time but also prevent most of the shrinkage of gypsum when it is heated
above 300°C.

Setting Time
• The setting time for dental inlay casting investment should not be less than 5 or more than 25 min.
• Usually, the modern inlay investments set initially in 9–18 min.
• Sufficient time should be allowed for mixing and investing the pattern before the investment sets.

Normal Setting Expansion


• A mixture of silica and calcinated gypsum powder (calcium sulfate hemihydrate, CaSO4•1/2H2O) results in setting expansion
greater than that of the gypsum product used alone.
• The silica particles probably interfere with the intermeshing and interlocking of the crystals as they form.
• Thus, the thrust of the crystals is outward during growth, and they increase expansion. Generally, the resulting setting
expansion in such a case is high.
• Type I investments should exhibit a maximum setting expansion in air of 0.6%.
• The purpose of the setting expansion is to aid in enlarging the mold to compensate partially for the casting shrinkage of
the alloy.
• Typically, the setting expansion of these investments is approximately 0.4%. This expansion is controlled by retarders and
accelerators.

Hygroscopic Setting Expansion


• The theory of hygroscopic setting expansion was previously described in connection with the setting of dental plaster and
stone.
• Hygroscopic setting expansion, which is greater in magnitude than normal setting expansion, differs from normal setting
expansion in that it occurs when the gypsum product is allowed to set when placed in contact with heated water.
• The hygroscopic setting expansion may be six or more times greater than the normal setting expansion of a dental
investment. In fact, it may be as high as 5 linear percent.
• The hygroscopic setting expansion is one of the methods for expanding the casting mold to compensate for the casting
shrinkage of gold alloys.
• Type II investments should exhibit a minimum setting expansion in water of 1.2%.

Characteristic features Setting and hygroscopic expansion Thermal expansion


Excess W:P ratio Decreased Decreased
Excess spatulation time Increased No effect
More older the investment Decreased No effect
Delayed immersion Decreased –
High temperature of water bath Increased –
Strength of wax More distortion Less effect

https://t.me/DentalBooksWorld
996 Triumph's Complete Review of Dentistry

Variables That Affect Hygroscopic Expansion


• The magnitude of the hygroscopic setting expansion of a dental investment is generally proportional to the silica content of
the investment, other factors being equal.
• The finer the particle size of the silica is, the greater the hygroscopic expansion is.
• In general, α-hemihydrate is apt to produce a greater hygroscopic expansion in the presence of silica than is the
β-hemihydrate, particularly when the expansion is unrestricted.
• A dental investment should have enough hemihydrate binder with the silica to provide sufficient strength after hygroscopic
expansion.
• Otherwise shrinkage occurs during the subsequent drying of the set investment. At least 15% of binder is necessary to
prevent drying shrinkage.
• With most investments, as the mixing time is reduced, the hygroscopic expansion is decreased. This factor is also important
in the control of the effective setting expansion.
• The older the investment is, the lower its hygroscopic expansion is.
• Immersion in water at room temperature (rather than 37.7°C) reduces the effective expansion.
• The magnitude of the hygroscopic setting expansion can be controlled by the amount of water added to the setting
investment. It has been proved that the magnitude of the hygroscopic expansion is proportional to the amount of water
added during the setting period until maximal expansion occurs. No further expansion is then evident regardless of the
amount of water added.
• Hygroscopic setting expansion is a continuation of ordinary setting expansion because the immersion water replaces the
water of hydration, thus preventing confinement of the growing crystals by the surface tension of the excess water.
• Because of the dilution effect of the quartz particles, the hygroscopic setting expansion in these investments is greater than
that of the gypsum binder when used alone.
• This effect is the same as previously described for normal setting expansion. This phenomenon is purely physical.
• The hemihydrate binder is not necessary for hygroscopic expansion because investments with other binders exhibit a
similar expansion when they are allowed to set under water.
• The greater the amount of the silica or the inert filler is, the more rapidly the added water can diffuse through the setting
material and the greater the expansion. The W:P ratio affects the hygroscopic expansion for the same reason that it affects
the normal setting expansion.

Thermal Expansion of Gypsum-Bonded Investments

A
1.6 Average mix
B
Thinner mix
1.4 C
Less spatulation
1.2 D
Linear expansion (%)

Aged investment
1.0

0.8

0.6

0.4

0.2

0.0
0 0.2 0.4 0.6 0.8 1.0 1.2 1.4 1.6 1.8
Water added (mL)

Relationship of the linear hygroscopic setting expansion and the amount of water added as influenced by certain manipulative
factors.

https://t.me/DentalBooksWorld
Chapter 11 • Dental Materials 997

• The desired magnitude of the thermal expansion of a dental investment depends on its use. If the hygroscopic expansion
is to be used to compensate for the contraction of the gold alloy, as for the type II investments, thermal expansion should
be between 0% and 0.6% at 500°C. However, for type I investments, which rely principally on thermal expansion for
compensation, the thermal expansion should not be less than 1% or greater than 1.6%.

Strength
• The fracture resistance of the investment must be adequate to prevent cracking, bulk fracture, or chipping of the mold
during heating and casting of gold alloys.
• The strength of the investment is affected by the W:P ratio in the same manner as any other gypsum product; the more
the water that is employed in mixing, the lower the compressive strength is. Heating the investment to 700°C may increase
or decrease the strength as much as 65%, depending on the composition. The greatest reduction in strength on heating is
found in investments containing sodium chloride.
• The compressive strength for the inlay investments should not be less than 2.4  MPa when tested 2 hours after setting.
Any investment that meets this requirement should have adequate strength for casting of an inlay. However, when larger,
complicated castings are made, greater strength is necessary, as required for type III partial denture investments.

THERMAL SHRINKAGE
Wax
• A wax pattern prepared directly in a patient’s mouth will shrink about 0.4% when cooled from oral temperature. In the
indirect method of preparing the wax pattern on a die, the wax shrinkage is about 0.2%.
Gold Alloy
• The casting shrinkage takes place as the solidified metal cools to room temperature.
• The values for this shrinkage depend on the geometry of the casting. For example, the gold shrinkage ranges from 1.25% for
a thin three-quarter crown to 1.75% for full crowns and 2% for class 5 restorations.

THERMAL EXPANSION
Gypsum-Bonded Investment
High-Heat Technique
• This method uses cristobalite (a high-expansion form of silica) investment materials.
• After the investment has been mixed according to the manufacturer’s instructions and allowed to set for at least 45 min and
no longer than 60 min, the mold is placed in a 200°C oven for 20–30 min to burn out the wax patter.
• The temperature of the mold is further elevated by transferring the mold to a second oven and holding at 700°C for no
longer than 20–30 min to obtain the maximum thermal expansion (TE) of 1.25%.
• Because this type of investment is weak by nature, a metal ring must be used. To increase the setting expansion (SE) of
0.35%, the inside of the ring should be lined with a dampened liner strip that also acts as a cushion against which expansion
can take place.
• This greater expansion caused by the uptake of water from the liner, referred to as hygroscopic expansion (HE), is double
the normal SE.
Water-Immersion Hygroscopic Technique
• Investments made for water immersion are much stronger than the high-heat types; therefore, a metal ring is not necessary.
• Instead, a rubber ring is used to contain the mixed investment. Maximum HE is obtained by immersing the invested pattern
and rubber ring, allowing the investment to set under water.
• Most of the compensatory expansion is HE (1.50%), which again includes the normal SE of 0.3%.
• This expansion takes place with the pattern present in the mold, which may cause distortion in certain pattern configurations
(e.g., mesio-occlusodistal).

https://t.me/DentalBooksWorld
998 Triumph's Complete Review of Dentistry

Technique Setting expansion (%) Hygroscopic expansion (%) Thermal expansion (%)
High heat (cristobalite) 0.35 0.70* 1.25 (700°C)
Hygroscopic immersion
0.30 1.50 0.55 (480°C)
(Beauty-Cast, Whip Mix)
Hygroscopic water added
0.75 2.00 0.55 (480°C)
(Hygrotrol, Jelenko)
Phosphate–high heat
0.23–0.50 0.35–1.20 1.33–1.58 (700°C)
(Ceramigold, Whip Mix)

SPRUING
• The purpose of spruing the wax pattern is fourfold.
• To form a mount for the wax pattern and fix the pattern in space, so a mold can be made.
• To create a channel for elimination of wax during burnout.
• To form a channel for the ingress of molten alloy during casting.
• To compensate for alloy shrinkage during solidification.
Sprue Size and Design
• The sprue must be large enough so that it remains open until the casting solidifies and short enough to allow rapid filling
of the mold cavity.
• Large and small inlays require sprues that are 14 gauge (4–5 mm long) and 16 gauge (3–4 mm long), respectively.
• Large and small crowns require 10- and 12-gauge sprues, respectively, with an average sprue length of 4–5 mm.

One-surface inlay MOD inlay

Y- sprue of MOD inlay Three-quarter crown

Full crown 3-unit fixed paral denture

Sprue designs for castings

https://t.me/DentalBooksWorld
Chapter 11 • Dental Materials 999

G F
E

C
B
D

Diagrammatic representation of a dental casting mold


A, Crucible former
B, Sprue
C, Cavity formed by wax pattern after burnout
D, Investment
E, Liner
F, Casting ring
G, Recommended maximum investment thickness of approximately 6 mm between the end of the mold cavity and the end of
the invested ring to provide pathways for sufficient gas escape during casting.

Point of Attachment
• Sprue attachment must always be made at the bulkiest portion of the pattern. If two bulky portions of the castings are
separated by a thin cross section (e.g., for a mesio-occlusodistal inlay), a Y-shaped sprue must be used.
• Turbulence of the molten gold as it enters the mold causes porosity, which is due to entrapped gases and an inappropriate
angle of sprue attachment.
• All attachments, both the sprue pattern and sprue/crucible former, must therefore be “trumpeted” or “filleted” to eliminate
all sharp corners, angles, and instrument marks.

Sprue Selection
• The wax sprue is most common.
• Plastic sprues are not recommended because their higher flow temperatures and TE characteristics make it difficult to
eliminate the sprue.
• Because the wax melts at a much lower temperature than the plastic sprue and the TE of the wax is five times that of plastic,
excessive wax pressure may build up in the mold during burnout before the plastic sprue softens.
• A hollow metal sprue pin is preferable to a solid metal pin because of its stronger attachment.
• Sticky wax must be used to fill the hollow sprue core before use.

Orientation in Mold
• The wax pattern is mounted on the sprue pin, which in turn is mounted on a clean sprue/crucible former.
• It is essential that, when the investment ring is placed over the assembly of the pattern and sprue/crucible former, the
pattern be 6 mm from the end of the ring.
• If the pattern is less than 6 mm from the end, there is not enough thickness of investment to keep the molten gold from
breaking through.
• If there is more than 6 mm of space, the gold will solidify before the entrapped air can escape, resulting in rounded margins,
incomplete casting, or mold fracture.

https://t.me/DentalBooksWorld
1000 Triumph's Complete Review of Dentistry

Liner
• A liner is placed inside the ring to allow lateral expansion of the investment, and 3 mm of clearance is allowed at each end
of the ring so the mold is sealed and anchored in place.
• After the liner is placed in the ring, it is dipped in water until saturated, and the excess water is shaken off.
MELTING
New Metal
• Since gold alloys and other alloys change composition during casting, at least one-third of new gold by weight must be used
for each melt.
Contamination
• Clean melting crucibles are essential to prevent alloy contamination.
• Copper-containing gold alloys and non-copper alloys for use with porcelain should not be melted in the same crucible.
• Previously, cast metal must be thoroughly cleaned using appropriate fluxes to remove all gases, oxides, and investment
before remelting.

DEFECTS IN CASTING
1. Distortion
Any marked distortion of the casting is probably related to a distortion of the wax pattern. This type of distortion can be
minimized or prevented by proper manipulation of the wax and handling of the pattern.
The configuration of the pattern, the type of the wax, and the thickness influence the distortion that occurs, as has been
discussed. For example, distortion increases as the thickness of the pattern decreases.
2. Surface Roughness and Irregularities
The surface of a dental casting should be an accurate reproduction of the surface of the wax pattern from which it is made;
excessive roughness or irregularities on the outer surface of the casting necessitate additional finishing and polishing, whereas
irregularities on the cavity surface prevent a proper seating of an otherwise accurate casting.
Air bubble: Rapid heating Under heating: Liquid:powder Prolonged heating:
• Air bubbles cause rates: • Incomplete ratio: • When the high-heat casting
small nodule on a • Rapid heating elimination of • The amount technique is used, a prolonged
casting. results in fins wax residues of water and heating of the mold at the casting
• Such nodules can be or spines on may occur if investment temperature is likely to cause a
removed if they are the casting or the heating should be disintegration of the investment,
not in a critical area. may result as a time is too measured and the walls of the mold are
• However, for characteristic short or if accurately. roughened as a result; furthermore,
nodules on margins surface insufficient The higher the the products of decomposition
or on internal roughness may air is available L:P ratio, the are sulfur compounds that may
surfaces, the removal be evident in the furnace. rougher the contaminate the ally to the extent
of these irregularities because of These casting. that the surface texture is affected.
might alter the fit of flasking of the factors are • When the thermal expansion
the casting. investment particularly technique is employed, the mold
when the water important should be heated to the casting
• The best method to
or steam pours with the low- temperature – never higher than
avoid air bubbles is
into the mold. temperature 7,000°C, and the casting should be
to use the vacuum
investment made immediately.
investing technique.
techniques.
3. Porosity
Porosity may occur both within the interior region of a casting and on the external surface. The latter is a factor in surface
roughness, but it is also generally a manifestation of internal porosity.

https://t.me/DentalBooksWorld
Chapter 11 • Dental Materials 1001

Porosities in noble metal castings may be classified as follows:


I. Solidification defects
a. Localized shrinkage porosity
b. Microporosity
II. Trapped gases
a. Pinhole porosity
b. Gas inclusions
c. Subsurface porosity
III. Residual air
Localized shrinkage: Microporosity: Also occurs Pinhole and Subsurface Entrapped air
Caused by: from the rapid solidification gas inclusion porosity: porosity: On
Is generally caused by but is generally present in porosities: The reasons the inner surface
incomplete feeding of molten fine grain alloy castings Are related to for such voids of the casting,
metal during solidification. when the solidification is too the entrapment have not been sometimes referred
rapid for the micro void to of gas during completely to as backpressure
Therefore, there must be
segregate to the liquid pool. solidification established porosity, can
continual feeding of molten
This premature solidification Caused by: Caused by: be produced
metal through the sprue to
causes the porosity large concave
make up for the shrinkage Gases are They may be
Such a phenomenon can depressions
of metal volume during dissolved by some caused by the
occur from the rapid Caused by:
solidification metals when they simultaneous
solidification if the mold or This is caused by
Eliminated by: are in molten nucleation of
casting temperature is too the inability of the
This problem can be solved in state… Both are solid grains
low. It is unfortunate that air in the mold to
the future simply be attaching characterized and gas bubbles
this type of defect is not escape through
one or more small-gauge sprues by a spherical at the first
detectable unless the casting the pores in the
(e.g., 18 gauge) at the surface contour, but they moment that
is sectioned. In any cast, it is investment or
most distant from the main are decidedly the metal
generally not a serious defect by the pressure
sprue attachment and extending different in size freezes at the
gradient that
the sprue(s) surface most mold walls
displaces the air
distant from the main sprue pocket toward
attachment and extending the the end of the
sprue laterally within 5 mm of investment via the
the edge of the ring molten sprue and
button
Eliminated by:
Proper burnout,
an adequate
mold and casting
temperature, a
sufficiently high
casting pressure,
and proper L:P
ratio can help
to eliminate this
phenomenon
4. Incomplete or Missing Detail
Only a partially complete casting or perhaps no casting at all is found. The obvious cause is that the molten alloy has been
prevented, in some manner, from completely filling the mold.

https://t.me/DentalBooksWorld
1002 Triumph's Complete Review of Dentistry

There are two causes:


1. Insufficient venting is directly related to the back pressure exerted by the air in the mold. If the air cannot be vented quickly,
the molten alloy does not fill the mold before if solidifies. In such a case, the magnitude of the casting pressure should be
suspected. If insufficient casting pressure is employed, the back cannot be overcome.
2. A second common cause for an incomplete casting is incomplete elimination of wax residues from the mold.

DENTAL WAX
KEY TERMS
Baseplate wax—Dental wax provided in sheet form to establish the initial arch form in the construction of complete dentures.
This product typically contains approximately 75% paraffin or ceresin wax, beeswax other waxes, and resins.
Bite wax—A wax form used to record the occlusal surfaces of teeth as an aid in establishing maxillo–mandibular relationships.
Boxing wax—A wax sheet form used as a border at the perimeter of an impression to provide an enclosed boundary for the
base of the cast to be made from a poured material such as gypsum or resin.
Burnout—The process of heating an invested mold to eliminate the embedded wax or plastic pattern.
Corrective wax (dental impression wax)—A thermoplastic wax that is used to make a type of dental impression.
Dental wax—(1) A low-molecular-weight ester of fatty acids derived from natural or synthetic components, such as petroleum
derivatives, that soften to a plastic state at a relatively low temperature. (2) A mixture of two or more waxes and additives
used as an aid for the production of gypsum casts, production of nonmetallic denture bases, registering of jaw relations, and
laboratory work.
Direct wax technique—A process whereby a wax pattern is prepared in the mouth directly on prepared teeth.
Divesting—Process of removing investment from a cast metal or hot-pressed ceramic.
Elastic memory—The tendency of a solid wax form to partially return to its original shape when it is stored at a higher
temperature than that to which it was cooled.
Flow—Relative ability of wax to plastically deform when it is heated slightly above the body temperature.
Hygroscopic expansion—The amount of setting expansion that occurs when a gypsum-bonded casting investment is
immersed in water, which is usually heated to approximately 38°C. (See Chapter 9 for more information on this process.)
Indirect wax technique—The procedure in which a wax pattern is prepared on a die.
Inlay wax—A specialized dental wax that can be applied to dies to form direct or indirect patterns for the lost-wax technique,
which is used for the casting of metals or hot pressing of ceramics.
Refractory—Capable of sustaining exposure to a high temperature without significant degradation.
Sprue—The mold channel through which molten metal or ceramic flows into a mold cavity.
Sprued wax pattern—A wax form consisting of the prosthesis pattern and the attached sprue network.
Sticky wax—A type of dental wax that exhibits high adhesion to dry, clean surfaces when it is heated to a plastic condition.

HISTORY
• Beeswax was derived from secretions that bees use to build honeycombs. Although beeswax is still used today, modern
waxes, such as those used to preserve furniture and automobile surfaces and those designed for dental procedures, are made
from natural plant and animal sources; some types are derived synthetically from petroleum products and distillates.
• Synthetic waxes are typically composed of hydrogen, carbon, oxygen, and chlorine.
• Synthetic waxes are more uniform than natural waxes in their organic structure and more homogeneous in composition.
• Carnauba is one of the hardest and most durable waxes.
• It is derived from the fronds of carnauba palm trees and is one of the main components of dental inlay wax.
• Candelilla wax, a major component of some dental waxes, is obtained from plants growing in Costa Rica, Guatemala,
Mexico, Nicaragua, Panama, and the southwestern United States.
• In comparison to plant-derived carnauba and candelilla waxes, animal-derived beeswax, and mineral-derived paraffin and
ceresin waxes, other dental waxes are produced from the components of fats, gums, oils, and resins.

https://t.me/DentalBooksWorld
Chapter 11 • Dental Materials 1003

TYPES
Types of Inlay Waxes
• The wide variety of dental waxes can be classified into two groups, those used primarily in the clinic and those used in
commercial dental laboratories.
• Clinical products include bite registration wax, disclosing wax (also known as pressure-indicating paste), utility waxes for
altering and adapting impression trays, and low-melting type I inlay waxes used in the mouth for direct-waxing processes
for pattern production.
• Laboratory products include boxing wax, baseplate wax, sticky wax, beading wax, utility wax, and hard, medium, and soft
type II inlay-type waxes for making patterns on patients’ models using the indirect wax technique.
• Type II waxes are required for the lost-wax processing of cast prostheses and frameworks.
• Each of these waxes has a melting range over which the temperature must be adjusted by means of a burner flame to control
the flow properties for each specific application.
• Dental waxes can also be classified into one of three types, pattern wax (inlay, casting, and baseplate types), processing
wax (boxing, utility, and sticky types), and impression wax (bite registration and correction types). Casting wax is used for
partial denture frameworks and other metal frameworks. One of the correction types includes waxes for repairing ceramic
margin defects on all-ceramic inlays and crowns.
• Inlay waxes are used to prepare patterns. These patterns are reproduced in gold via a casting process and in ceramic by hot-
isostatic-pressing procedures. Inlay wax is sometimes referred to as casting wax, although other types of pattern waxes also
fall into this category.
• Inlay wax must exhibit excellent adaptability to model or die surfaces, and it must be free from distortion, flaking, or
chipping during the preparation of patterns.
• It must also be able to disintegrate, volatilize, and be eliminated completely from an investment mold during the burnout
or wax elimination procedure.
• For direct wax techniques, type I inlay wax must soften at a temperature that is not hazardous to the pulp tissue, and it must
harden at a temperature above the mouth temperature.
• The colors of inlay waxes should contrast with the hues of teeth and dies. Dental waxes are supplied in a variety of colors
including blue, green, yellow, red, and ivory.
• The colors are useful to provide a suitable contrast against a die that is an accurate replica of a prepared tooth or dental
arch form.
• Ivory-colored wax is useful for esthetic case presentations to patients. If applied as a veneer in a sufficient thickness, its
opacity must be sufficient to mask colored die stones.
• Inlay waxes may be softened over a flame or in water at 54–60°C to enable their flow in the liquid state and their adaptation
to the prepared tooth or die.
• These waxes are designed to maintain uniform workability over a wide temperature range and to facilitate accurate adaptation
to the tooth or die under pressure. Additive layers and corrections may be applied to produce a relatively homogeneous
pattern.
• These fused layers can be carved easily without chipping or flaking. A regular or soft type of wax is typically used for
indirect work at room temperature or in cool weather. A harder or medium type with a low flow property is indicated for
use in warmer climates.
• The first procedure in the casting of an inlay or crown for the lost-wax process is the preparation of a dental wax pattern.
• The cavity is prepared in the tooth and the pattern is carved directly on a die that is a reproduction of the prepared tooth
and dental tissues (indirect technique).
• The direct technique for producing wax inlay patterns within prepared teeth is rarely used because of the wax’s sensitivity
to changes in pressure, temperature, and heating and cooling rates during manipulation.
• Because the thermal expansion coefficient of wax is extremely high compared with the values for other dental materials, a
wax pattern made in the mouth (direct technique) will shrink appreciably as it is cooled to room temperature.
• A pattern made by the indirect method may not shrink as much, although the amount depends on whether or not the
pattern is allowed to reach room temperature before it is removed from the die.

https://t.me/DentalBooksWorld
1004 Triumph's Complete Review of Dentistry

• Dipping waxes are used occasionally to facilitate the wax pattern preparation process. This wax is kept molten to provide a
station for mass production of patterns.
• Type I is a medium wax employed in direct techniques and type II is a soft wax used in the indirect techniques. No matter
how a pattern is prepared, it should be an accurate reproduction of the missing tooth structure or part of a prosthesis that
is adapted to soft tissues.
• The wax pattern forms the outline of the mold into which an alloy is cast or a ceramic is hot-isostatically pressed.
Consequently, the resulting appliance, device, prosthesis, or framework can be no more accurate than the wax pattern
regardless of the care observed in subsequent procedures.
• Therefore, the pattern should be well adapted to the prepared cavity or replica cavity and properly carved without any
significant distortion.
• Before the adaptation of the wax pattern within a tooth or a die, a separating medium must be used to ensure the complete
separation of the wax pattern without distortion. After the pattern is removed from the prepared cavity, it is encased in a
gypsum- or phosphate-based material or other type of refractory material known as an investment. This process is called
investing the pattern.
• After investing anatomically accurate wax or resin patterns for inlays, onlays, crowns, bridges, and frameworks for removable
partial dentures, the invested material must be eliminated completely before the molten metal is cast or core ceramic is hot-
pressed into the mold cavity.
• Wax patterns are used in the production of several types of complex removable devices or prostheses in addition to single-
tooth restorations.

COMPOSITION
Composition of Dental Waxes
• Natural waxes are derived from mineral, vegetable, and animal origins.
• Synthetic waxes are chemically synthesized from natural wax molecules. Most synthetic waxes are more homogeneous than
pure natural waxes.
• Coloring agents are added for contrast of wax patterns against tooth, die, and model surfaces or to provide an ivory-colored
or other natural tooth color as demonstration models used for educating patients about treatment options.
• Some formulations contain a compatible filler to control expansion and shrinkage of the wax product.
• Most dental waxes contain 40–60% paraffin by weight, which is derived from high-boiling fractions of petroleum.
• They are composed mainly of a complex mixture of hydrocarbons of the methane series together with minor amounts of
amorphous and microcrystalline phases.
• The wax can be obtained in a wide range of melting or softening temperatures depending on the molecular weight and
distribution of the constituents.
• The melting range can be determined by a temperature versus time cooling curve for a paraffin-based inlay wax.
• The temperature–time relationship during cooling indicates the successive solidification of progressively lower-molecular-
weight fractions.
• This condition promotes moldability of the wax below its melting temperature. Paraffin that is used for type I waxes has a
higher melting point than the paraffin used for type II waxes.
• Paraffin wax is likely to flake when it is trimmed, and it does not produce a smooth, glossy surface, which is a desirable
requisite for an inlay wax. Thus, other waxes and natural resins must be added as modifying agents.
• Gum dammar, or dammar resin, is a natural resin. It is added to the paraffin to improve the smoothness in molding and
to render it more resistant to cracking and flaking. It also increases the toughness of the wax and enhances the smoothness
and luster of the surface.
• Carnauba wax occurs as a fine powder on the leaves of certain tropical palms. This wax is very hard, and it has a relatively
high melting point and it has an agreeable odor.
• It is combined with the paraffin to decrease flow at mouth temperature. Carnauba wax contributes greater glossiness to the
wax surface than dammar resin.
• Candelilla wax can also be added partially or entirely to replace carnauba wax.

https://t.me/DentalBooksWorld
Chapter 11 • Dental Materials 1005

• Candelilla wax provides the same general qualities as carnauba wax but its melting point is lower and it is not as hard as
carnauba wax. Ceresin may replace part of the paraffin to modify the toughness and carving characteristics of the wax.
• Ceresin is typically a white wax extracted from ozokerite, a waxy mineral mixture of hydrocarbons that is colorless or white
when pure, but it has a somewhat unpleasant odor.
• Carnauba wax is often replaced in part by certain synthetic waxes that are compatible with paraffin wax.
• At least two waxes of this type can be used. One is a complex nitrogen derivative of the higher fatty acids and the other
contains esters of acids derived from montan wax, a derivative hard wax that is obtained by solvent extraction of certain
types of lignite or brown coal.
• Approximately a third of all montan wax produced is used in automobile polishing pastes.
• For an impression compound, a synthetic wax is preferable to a natural wax because it has greater uniformity. Because of
the high melting point of the synthetic waxes, more paraffin can be incorporated to improve the general working qualities
of the product.

PROPERTIES
Desirable Properties of Wax
• Control of the properties of dental wax is accomplished by a combination of factors.
• For example, certain wax formulations may be based on the amount of carnauba wax, the desired melting range of the
hydrocarbon wax, and the addition of resin to achieve desirable properties
• Depending on the specific application of a given wax, the melting range, viscosity, adaptability, flow, elastic recovery,
carvability, and burnout properties of these materials control the quality and reproducibility of the final prostheses and
restorations. The most important properties of inlay waxes are as follows:
–– The wax should be uniform when softened. It should be compounded with ingredients that blend with each other so that
there are no granules on the surface and no hard spots within the surface when the wax is softened.
–– The color should contrast with die materials or prepared teeth. Since it is necessary to carve the wax margins against
the die surface, the wax must exhibit a definite contrast in color and sufficient opacity in thin layers to facilitate proper
finishing of the margins.
–– The wax should not fragment into flakes or similar surface particles when it is molded after softening. Such flakiness is
likely to be present in paraffin wax, so modifiers must be added to minimize this effect.
–– Once the wax pattern has solidified, it is necessary to carve the original tooth anatomy and the margins so that the pattern
conforms precisely to the surface of the die. The latter procedure sometimes requires that the wax be carved to a very
thin layer. The wax must not be pulled away by the carving instrument or chip as it is carved or such precision cannot be
achieved.
–– For lost-wax casting of metals, an investment mold is formed around a wax pattern. After the mold containing the wax
pattern has been formed, the wax must be eliminated from the mold.
■■ Elimination of the sprued wax pattern is usually accomplished by heating the mold to melt and ignite the wax. If the
wax leaves a residue or an impervious coating on the walls of the mold, the cast metal inlay may be adversely affected.
■■ Consequently, the wax should burn out completely by oxidizing residual carbon to volatile gases. Ideally, when wax
melts and is vaporized at 500°C, it should not leave a solid residue that amounts to more than 0.10% of the original
weight of the specimen.
■■ Expansion and shrinkage of casting wax are extremely sensitive to temperature.
■■ Normally soft wax shrinks more than hard wax. High-shrinkage wax may cause significant pattern distortion when it
solidifies.
■■ It is necessary to avoid excessive shrinkage and expansion caused by a temperature change.
■■ For this reason, an organic filler is added to certain wax formulations.
■■ Such fillers should be completely miscible with the components of the inlay wax during manufacture, and they should
not leave an undesirable residue after burnout.

https://t.me/DentalBooksWorld
1006 Triumph's Complete Review of Dentistry

Flow of Dental Wax


• One of the desirable properties of type I inlay wax is that it should exhibit a marked plasticity or flow at a temperature
slightly above that of the mouth.
• The temperatures at which the wax is plastic are indicated by the time–temperature cooling curve for a typical type I wax.
• The wax begins to harden slowly below 65°C and becomes solid below approximately 48°C; below which it cools rapidly at
a constant rate.
• The flow is measured by subjecting cylindrical specimens to a designated load at the stated temperature and measuring the
percentage of reduction in length.
• The maximum flow permitted for type I waxes at 37°C is 1%.
• Their low flow at this temperature permits carving and removal of the pattern from the prepared cavity at oral temperature
without distortion.
• In addition, both type I and type II waxes at 45°C must have a minimal flow of 70% and a maximum flow of 90%. At this
temperature, the wax is inserted into the prepared cavity.
• If the wax does not have sufficient plasticity, it will not flow into all of the areas in the preparation and reproduce the details
that were established by the invested wax pattern.
• Requirements for inlay wax flow (%)
Type of T = 30°C T = 37°C T = 40°C T = 40°C T = 45°C T = 45°C
Wax Maximum Maximum Minimum Maximum Minimum Maximum
I – 1 – 20 70 90
II 1 – 50 – 70 90

Thermal Properties of Dental Waxes


• Inlay waxes are softened with heat, forced into the prepared tooth cavity in either the tooth or the die, and cooled.
• The wax may expand as much as 0.7% with an increase in temperature of 20°C or contract as much as 0.35% when it is
cooled from 37°C to 25°C. The average linear thermal expansion coefficient over this temperature range is 350 × 10−6/K,
with values ranging from 217 to 512 × 10−6/K.
Thermal Expansion of Inlay Wax
• The expansion rate increases abruptly above approximately 35°C. The temperature at which a change in rate occurs is
known as the glass transition temperature.
• Some constituents of the wax probably change in their crystalline form at this temperature, and the wax is more plastic at
higher temperatures.
• Waxes tend to return partially to their original shape after manipulation. This is known as elastic memory.

Specialty Waxes
• A pattern made of hard wax is less sensitive to temperature conditions than one made of soft wax. The exothermic heat
generated during the setting of an investment affects the pattern selectively.
• A soft wax pattern may result in a slightly larger and relatively rougher casting than a hard wax pattern. This tendency of
softer inlay waxes to expand during setting in a hygroscopic bath at 37.8°C (100°F) may contribute to the phenomenon of
hygroscopic expansion.
• Other types of waxes are employed for different purposes than those described for the inlay waxes. The composition of each
type is adjusted for the particular requirements. One of the most common is baseplate wax.
• Baseplate wax is used to establish the initial arch form in the construction of complete dentures. Supplied in 1- to 2-mm-
thick red or pink sheets, the wax is approximately 75% paraffin or ceresin with additions of beeswax and other resins or
waxes.
• The harder the wax, the less the flow at a given temperature. The difference in flow of the three types may be advantageous
for a particular application.

https://t.me/DentalBooksWorld
Chapter 11 • Dental Materials 1007

• Type I, a soft wax, is used for building veneers.


• Type II, a medium wax, is designed for patterns to be placed in the mouth in normal climatic conditions.
• Type III, a hard wax, is used for trial fitting in the mouth in tropical climates.
• Because residual stress is present within the wax from contouring and manipulating the wax, the finished denture pattern
should be flasked as soon as possible after completion of all adjustments and manipulations.
• The impression waxes, also referred to as bite waxes or corrective waxes, tend to distort if they are withdrawn from undercut
areas.
• Thus, they are limited to use in edentulous sites of the mouth or in occlusal surface areas.

FINISHING AND POLISHING MATERIALS


TERMINOLOGIES
Abrasive—A sharp, hard, natural, or synthetic substance used for grinding, finishing, or polishing a softer surface.
Air-particle abrasion—The process of removing material, contaminants, stain, or carious tissue by use of air pressure and
abrasive particles appropriate for the substrate being treated.
Buffing—The process of producing a lustrous surface through the abrading action of fine abrasives bound to a non-abrasive
carrier with or without a liquid or paste medium.
Bulk reduction—The process of removing the excess material (natural tooth or synthetic structure) by cutting or grinding
with rotary instruments to provide a desired anatomic form.
Contouring—The process of producing a desired anatomic form by cutting or grinding away excess material.
Cutting—The process of removing the material from the substrate by use of a bladed bur or an abrasive embedded in a
binding matrix on a bur or disc.
Finished and polished restoration—A prosthesis or direct restoration whose outer surface has been progressively refined
to a desired state of surface finish.
Finishing—The process of removing surface defects or scratches created during the contouring process through the use of
cutting or grinding instruments or both.
Glaze ceramic—A specially formulated ceramic powder that, when mixed with a liquid, applied to a ceramic surface, and
heated to an appropriate temperature for a sufficient time, forms a smooth glassy layer on a dental ceramic surface (see
natural glaze).
Grinding—The process of removing the material from a substrate by abrasion with relatively coarse particles.
Natural glaze—A vitrified layer that forms on the surface of a dental ceramic containing a glass phase when the ceramic is
heated to a glazing temperature for a specified time.
Overglaze—Thin surface coating of glass formed by fusing a thin layer of glass powder that becomes a viscous liquid at a
lower temperature than that associated with the ceramic substrate.
Polish—Luster or gloss produced on a finished surface.

Polishing—The process of providing luster or gloss on a material surface.


• The intraoral surfaces of virtually every direct and indirect restoration must be contoured by grinding, finishing, and
polishing procedures.
• The goal of these procedures is to produce the smoothest surface possible in a limited time.
• Finishing a restoration involves contouring to create optimal marginal finish, without overhangs or excess material extending
beyond the cavity margin, and establishing an occlusal anatomy in harmony with the rest of the dentition.
• Polishing a restoration involves smoothing the surface with a series of abrasives to create the lowest surface roughness and
a high surface luster or polish.
• Finished and polished restorations provide four benefits of dental care – better gingival health, chewing efficiency, patient
comfort, and esthetics.

https://t.me/DentalBooksWorld
1008 Triumph's Complete Review of Dentistry

The advantages of finishing and polishing include


• Minimizing plaque accumulation at margins and on surfaces of restorations
• Minimizing the risk of surface staining
• Minimizing surface degradation and wear in clinical service
• Maximizing the esthetics of the restoration by creating a high luster or polish
• Enhanced patient and dentist satisfaction
• Reduced likelihood that a dentist will decide to replace the restoration unnecessarily (Oleinisky et al., 1996)
• There are, however, some potential disadvantages of excessive finishing and polishing; a notable problem is increased heat
generation, which may adversely affect the pulp.

Cutting and Grinding


A grinding operation removes small particles of a substrate through the action of bonded or coated abrasive instruments.
Cutting and grinding are both considered predominantly unidirectional in action.
Procedure Burs used
Contouring 12–16 fluted carbide burs or abrasives ranging in size from 30 to 100 µm provide a fine
contouring action
Finishing 18–30 fluted carbide burs, fine and superfine diamond burs, or abrasives that are between
8 µm and 20 µm in size
Polishing Polishing is multidirectional and fine-particle discs and strips are used

Biological Hazards
• Dispersions of solid particles are generated and released into the breathing space of laboratories and dental clinics whenever
finishing operations are performed.
• The air-borne particles may contain the tooth structure, dental materials, and microorganisms.
• Such aerosols have been identified as potential sources of infectious and chronic diseases of the eyes and lungs and present
a hazard to dental personal and their patients.
• Silicosis, also called grinder’s disease, is a major illness caused by inhalation of aerosol particles released from any number
of silica-based materials that are used in the processing and finishing of dental restorations.
• They can remain in air for more than 24 hours before settling.

ABRASION
The outermost particles or surface material of an abrading instrument is referred to as the abrasive. The material being
finished is called the substrate.
Abrasion is further divided into the processes of two-body and three-body wear.
Dental abrasives are supplied in a number of forms
• Two-body abrasives, including
–– Bonded abrasives
–– Coated abrasives
• Three-body abrasives, including
–– Paste abrasives
–– Loose abrasives
• Microparticle (or hard-particle) abrasives, delivered by air pressure
Two-body abrasion occurs when abrasive particles are Three-body abrasion occurs when abrasive particles are free to
firmly bonded to the surface of the abrasive instrument translate and rotate between two surfaces
and no other abrasive particles are used An example of three-body abrasion involves the use of nonbonded
A diamond bur abrading a tooth represents an example of abrasives such as those in dental prophylaxis pastes
two-body wear

https://t.me/DentalBooksWorld
Chapter 11 • Dental Materials 1009

• Diamond burs should always be used in the presence of water spray and at rotational speeds of less than 50,000 revolutions
per minute (rpm).
• Polishing pastes are considered nonbonded abrasives and are primarily used for final polishing.

Color coding for specialized diamond for occlusal reduction of tooth surfaces. The color-coded bands refer to the depth of
the cutting portion
Depth Color coding
1 mm White
1.5 mm Green
1.8 mm Orange
2.0 mm Blue
2.4 mm Red

MATERIALS USED IN ABRASION


• Many types of natural and synthetic (human-made) materials are available for use in dentistry. The following materials are
listed from most to least abrasive.
Diamond
• Diamond is the hardest substance known, rating a 10 on the Mohs hardness scale. It will efficiently abrade any substance.
• Rotary diamonds are expensive and usually are not disposable, so they are found most often bonded in varying degrees of
coarseness to rotary cutting shanks or disks.
• They are sterilizable and can be reused several times before they wear out. Fine-particle diamonds come in a paste for
polishing composite and porcelain restorations.
• They are used to cut crown and bridge preparations and to finish and polish composite restorations.
• Fine-particle diamonds come in a paste for polishing composite and porcelain restorations.
Tungsten Carbide Finishing Burs
• Tungsten carbide is very hard—harder than steel—and carbide tools with cutting edges do not dull quickly.
• Tungsten carbide finishing burs come in several shapes, with designs ranging from 7 to 30 cutting flutes.
• The higher the number of flutes the bur has, the finer the ultimate finish. A bur that contains only seven flutes will have a
very aggressive cutting action.
• These burs rank up to 9 on the Mohs scale and are used primarily for finishing. They are used to cut preparations or to finish
composite restorations.
Silicon Carbide
• Rare in nature, silicon carbide is typically synthesized as an extremely hard and efficient abrasive material (9–10 on the
Mohs scale).
• Silicon carbide-coated disks and bonded rotary devices are used primarily in the beginning steps of finishing procedures
for composites and ceramics.
Aluminum Oxide (Corundum), Emery
• Aluminum oxide (corundum; 9 on the Mohs scale) is a synthetic abrasive that is often manufactured as a white or tan powder.
• The powder form is used in sandblasting restorations in preparation for cementation and air abrasion. It is used in bonded
and coated rotary devices. Aluminum oxide-impregnated rubber wheels are called Burlew wheels.
• This popular abrasive comes in several grit sizes and has largely replaced emery.
• It is used to smooth enamel or to finish metal alloys and ceramic materials, and to polish highly filled and hybrid composite
restorations and porcelain restorations.
Sand
• Sand is a natural abrasive composed of quartz and silica.
• This abrasive rates a 7 on the Mohs scale and is manufactured as coated disks and handheld strips used in the finishing
process.

https://t.me/DentalBooksWorld
1010 Triumph's Complete Review of Dentistry

Silicon Dioxide
• Silicon dioxide has a Mohs ranking of 6–7 and is commonly found in prophylaxis paste for heavy stain removal and on
rubberized cups and points used for finishing and polishing composite restorations.
Pumice
• Pumice is volcanic silica manufactured as a loose abrasive.
• Superfine, flour of pumice (Mohs hardness scale, 6) is extremely fine and a major component of many prophylaxis pastes
used to polish tooth structure, dental amalgam, and acrylic bases.
• Fine, medium, and coarse pumice are primarily used in dental laboratory procedures and should not be used on natural
tooth structures.
Rouge
• Rouge is iron oxide with a Mohs hardness value of 5–6.
• It is frequently found in block form, which then is run onto a rag wheel to polish precious and semiprecious metal alloys in
the laboratory.
• Rouge is not used intraorally.
Tin Oxide
• Tin oxide is an extremely fine abrasive that is used extensively as a final polishing agent for enamel and restorations.
• This abrasive is usually found as a powder that is mixed with water or glycerin.
Calcium Carbonate
• Calcium carbonate, also called chalk or whiting, is a mild abrasive with a low Mohs ranking of 3.
• It is found in prophylaxis paste and dentifrice.
• It is used to polish teeth, metal restorations, and plastic materials.
Sodium Bicarbonate
• Sodium bicarbonate has a very low Mohs ranking of 2.5 and is used as a cleaning agent in toothpaste and in air polishing.
Potassium and Sodium
• Potassium and sodium have very low Mohs rankings of 0.4 and 0.5.
• These agents are nonabrasive and are used in toothpaste and desensitizing agents.
Prophylaxis (Prophy) Paste
• Prophylaxis (prophy) paste is a mixture of 50–60% abrasive materials such as pumice and tin oxide and lubricants.
• Prophy paste may be 20 times more abrasive to dentin and 10 times more abrasive to enamel than commercially prepared
dentifrice.
• Preservatives, flavoring agents, coloring agents, and therapeutic agents are added.
• The abrasive powder is diluted with a lubricant to reduce the rate of abrasion and the amount of frictional heat produced.
• The lubricant also helps keep the preparation in a paste form by preventing hardening on exposure to air.
• Preservatives are included to prolong shelf life, and coloring and flavoring agents are added to increase patient acceptance.
• Fluoride is added to many preparations as a therapeutic agent in the prevention of caries.
• Prophylaxis pastes are supplied as coarse grit (5 μm) to superfine grit (2 μm) commercially prepared pastes for polishing
and cleaning of tooth structures.

Finishing and Polishing Time


Restorations Polishing time
Amalgam After 24 hours
Composite A delay of 10 min or more is recommended after curing to allow complete polymerization
Gold alloys Use slow-speed handpiece, contour with carbide burs and green stones, and finish with
pink stone (aluminum oxide)

https://t.me/DentalBooksWorld
Chapter 11 • Dental Materials 1011

DENTAL CERAMICS
DENTAL PORCELAIN
• Porcelain has been used for denture teeth since 1790. The main advantages of porcelain are its excellent esthetic properties,
durability, and biocompatibility.
• Porcelain is defined as a white, translucent ceramic that is fired to a glazed state.
A ceramic may be defined as a material which is an inorganic non-metal solid produced by the application of heat which is
then cooled. It may be amorphous and partly or wholly crystalline. Dental ceramics need to be translucent and so feldspar
and silica are incorporated into the material to achieve this. Dental ceramics are therefore really glasses called feldspathic
“porcelains.” Pigments are also included to improve and optimize the esthetics.
CAD–CAM ceramic—A partially or fully sintered ceramic blank that is used to produce a dental core or veneer structure
using a computer-aided design (CAD) and computer-aided manufacturing or milling (CAM) process.
Castable ceramic—A glass specially formulated to be cast into a mold and converted by heating to a glass-ceramic as a core
coping or framework for a ceramic prosthesis.
Glass-ceramic—A ceramic that is formed to shape in the glassy state and subsequently heat treated to partially or completely
crystallize the object. Glass-ceramic blanks are also available for CAD–CAM processes.
Green state—The semi-hard, preferred condition of a ceramic object. A green ceramic may be wet, as produced by slip-
casting, or it may be isostatically pressed to shape prior to firing. Green ceramics are always porous. They are too fragile for
use intraorally.

CLASSIFICATION OF CERAMICS
1. According to their application:
• Artificial teeth
• Jacket crown and inlays
• Enamel veneer over cast metal crown
• Abrasive and polishing agents
2. According to their maturing or fusion temperature:
• Low fusing: 871–1,066°C
• Medium fusing: 1,093–1,260°C
• High fusing: 1,288–1,371°C
3. According to their types:
• Felspathic or conventional porcelain
• Leucite-reinforced porcelain
• Aluminous porcelain
• Glass-infiltrated alumina
• Glass ceramic
4. According to their substructure method:
• Cast metal
• Swaged metal
• Glass ceramic
• CAD–CAM porcelain
• Sintered ceramic core
5. According to their method of firing:
• Air fired at atmospheric pressure
• Vacuum fired at reduced pressure

https://t.me/DentalBooksWorld
1012 Triumph's Complete Review of Dentistry

6. Based on the fabrication method:


• Sintered porcelain – Leucite, alumina, and spinel
• Cast porcelain – Alumina and spinel
• Machined porcelain – Zirconia, alumina, and spinel
7. Based on the crystalline phase:
• Glassy (or vitreous) phase
• Crystalline phase
8. Based on the layering technique:
• Incisal or enamel porcelain – Gives the crowns a natural translucent appearance at the incisal edge.
• Body or dentin layer – Opaque porcelain contains approximately 15% tin oxide, zirconium oxide, or titanium dioxide.

CLASSIFICATION BASED ON THEIR FUSION TEMPERATURE


The ultra-low Fired below 850°C Crown and bridge, veneer ceramic
Low fusing “porcelains” fired between 850°C and 1,100°C Crown and bridge, veneer ceramic
Medium fusing ceramics Fired between 1,100°C and 1,260°C Denture teeth, presintered zirconia
High fusing ceramics Fired between 1,315°C and 1,370°C Denture teeth, fully sintered alumina,
zirconia core ceramics

COMPOSITION
Composition Decorative ceramic (%) Dental ceramic (%)
Kaolin 50–70 3–5
Quartz (silica) 15–25 12–25
Feldspar 15–25 70–85
Metallic colorants <1 1
Glass 0 Up to 15 depending on fusing temperature

Metallic Oxides in Ceramic


Metallic oxide Color
Chromium Green
Cobalt Blue
Copper Green
Iron Brown
Manganese Lavender
Nickel Brown
Titanium Yellow brown

Manufacture of the Ceramic Powder


• The ceramic is supplied to the dental laboratory as a powder.
• The manufacturers make this powder by taking the raw materials and grinding them to form fine powders.
• The molten mass thus produced is then rapidly cooled in cold water, which leads to large internal stresses, cracking
and crazing of the mass. The resulting fragments of ceramic are known as frit, with the process called fritting (which
is a pyrochemical reaction).

Basic Structure
• Basically, porcelain is a type of glass – Three dimensional network of silica.
• Since pure glass melts at a too high temperature – modifiers added to lower the fusion temperature – Sodium or potassium.
https://t.me/DentalBooksWorld
Chapter 11 • Dental Materials 1013

Silicon Oxygen Pottasium

Firing or Sintering
• It is to fuse the particles of porcelain powder producing hard mass.
Stages of Firing
• Low bisque stage – Lack complete adhesion, very porous, low shrinkage.
• Medium bisque stage – Water evaporates with better cohesion to the powder and some porosity.
• High bisque stage – Fusion of particles to form a continuous mass, complete cohesion, and no more shrinkage.
Glazing
• The glazing is to obtain a smooth surface that simulates a natural tooth.
• It is done by either:
–– Auto glazing – Rapid heating up to the fusion temperature for 1–2 min to melt the surface particles (preferred).
–– Add on glazing – Applying a glaze to the surface and re-firing.

Strengthening Ceramics
1. Development of residual compressive stresses:

• Ion exchange
• Thermal tempering
• Thermal compatibility
2. Designing components to decrease stress concentration:
Interruption of crack propagation

• Dispersion of crystalline phase


• Transformation toughening

Denture Teeth
The raw materials for porcelain denture teeth are mainly feldspar, about 15% quartz, and, to improve moldability, kaolin (4%).
A plastic mass made from this mixture and additional pigments is formed into metal molds and fired under vacuum to reduce
porosity. During firing, the teeth are glazed by the glass produced from the feldspar. Metal pins or holes are then placed in
the teeth for mechanical attachment to the denture base. Improvements in acrylic denture teeth have increased their use as an
alternative to porcelain.
Advantages:
• Excellent biocompatibility
• Natural appearance
• High resistance to wear and distortion

https://t.me/DentalBooksWorld
1014 Triumph's Complete Review of Dentistry

Disadvantages:
• Brittle
• No bond to acrylic denture bases and requires mechanical attachments
• Produces clicking sound on contact
• Cannot be polished easily after grinding
• Higher density increases the weight of the teeth
• Mismatch in coefficient of thermal expansion (TE) produces stresses in acrylic denture base

METAL CERAMIC RESTORATIONS


COMPOSITION
Dental porcelains are used to bond with metals for a natural-appearing outer layer. These porcelains were developed by
raising the coefficient of TE of feldspar to match the values of gold alloys, which are 13–14 × 10−6 °C. This modification was
accomplished by heating feldspar with alkali metal carbonates (e.g., K2CO3 and Li2CO3) to approximately 1,093°C (2,000°F) to
form a glass and a high-expansion ceramic phase identified as leucite (K2O · Al2O3 · 4SiO2).

Incisal edge
Alloy casting

(Min 0.5 mm) Opaque layer


(1 mm)
(Approx 0.1 mm)
Enamel layer

Cross section of a metal-ceramic crown with full coverage.

ADHESION OF CERAMIC TO METALS


Factors identified as promoting good adhesion or bonding of a porcelain enamel to a metal are wetting, adherent oxide, and
mechanical retention.

Wetting
Good wetting of the porcelain on the metal is indicated by a low contact angle of a drop of the porcelain when fired on the
solid. It promotes penetration of the glass into surface irregularities and, therefore, a greater area of contact. Good wetting also
indicates chemical compatibility between the porcelain and the metal.

Adherent Oxide
The presence of an adherent oxide on the metal surface that is wet by the porcelain provides a beneficial transition layer. The
diffusion of atoms from the metal and porcelain into this oxide is cited as evidence of a chemical bond. A non-adherent oxide,
however, can lead to a weak boundary and failure.

Mechanical Retention
The presence of surface roughness on the metal oxide surface can result in mechanical retention on a microscopic level,
especially if undercuts are present.

https://t.me/DentalBooksWorld
Chapter 11 • Dental Materials 1015

Porcelain Porcelain

Metal
oxide
Metal Metal

Type I: Metal-porcelain Type II: Metal oxide-porcelain

Porcelain
Porcelain

Porcelain Metal oxide


Metal oxide

Metal Metal

Type III: Cohesive within Type IV: Metal-metal oxide


porcelain

Porcelain Porcelain

Metal oxide Metal oxide


Metal
Metal
oxide
Metal Metal

Type V: Metal oxide-metal oxide Type VI: Cohesive within metal

Bond Failure Classification (According to Interfaces Formed)


Type I Metal – Porcelain
Type II Metal oxide – Porcelain
Type III Cohesive within porcelain
Type IV Metal – Metal oxide
Type V Metal oxide – Metal oxide
Type VI Cohesive within a metal

PICTURE-BASED QUESTIONS
1. Fifth generation bonding agent – 3m ESPE Adper single bond plus adhesive is a fifth generation (total etch, single-bottle
agent) combining a dentin primer and adhesive in a single bottle

https://t.me/DentalBooksWorld
1016 Triumph's Complete Review of Dentistry

2. 3M ESPE Adper Easy bond self-etch adhesive is a seventh generation, light cured, self-etch bonding agent

3. 3M ESPE Adper Scotchbond seelf-etch adhesive is a two-bottle, self-etching adhesive system (sixth generation, type 1)

4. Amalgamator

5. Light cure composite

6. Dental automatic dispenser

https://t.me/DentalBooksWorld
Chapter 11 • Dental Materials 1017

7. PENTAMIX – Dental automatic dispenser

MULTIPLE CHOICE QUESTIONS

ADA SPECIFICATION
1. ADA specification number for inlay casting wax is
A. 4 B. 5
C. 112 D. 7
2. ADA specification number for dental ceramic is
A. 69 B. 68
C. 106 D. 38
3. ADA specification number for dental abrasive powder is
A. 33 B. 34
C. 37 D. 38
4. ADA specification number for baseplate wax is
A. 122 B. 4
C. 106 D. 38
5. ADA specification number for casting investments and refractory die materials is
A. 126 B. 125
C. 66 D. 70
6. ADA specification number for pit and fissure sealants is
A. 39 B. 40
C. 38 D. 43
7. ADA specification number for dental amalgam is
A. 1 B. 5
C. 2 D. 7
8. ADA specification number for dental mercury is
A. 6 B. 68
C. 7 D. 10
9. ADA specification number for silicate cement is
A. 8 B. 34
C. 9 D. 38
10. ADA specification number for denture base polymers is
A. 12 B. 1
C. 122 D. 22
11. ADA specification number for agar is
A. 11 B. 1
C. 10 D. 111

https://t.me/DentalBooksWorld
1018 Triumph's Complete Review of Dentistry

12. ADA specification number for alginate is


A. 18 B. 8
C. 28 D. 38
13. ADA specification number for zinc phosphate cement is
A. 8 B. 18
C. 96 D. 99
14. ADA specification number for glass ionomer cements is
A. 60 B. 68
C. 66 D. 76
15. ADA specification number for dental elastomeric impression materials is
A. 33 B. 34
C. 19 D. 38
16. ADA specification number for electrically powered dental amalgamators is
A. 43 B. 4
C. 3 D. 34
17. ADA specification number for dental brazing alloys is
A. 88 B. 125
C. 66 D. 70
18. ADA specification number for IOPA films is
A. 2 B. 22
C. 38 D. 43
19. ADA specification number for orthodontic wires is
A. 4 B. 32
C. 112 D. 7
20. ADA specification number for gypsum materials is
A. 69 B. 68
C. 25 D. 38
21. ADA specification number for amalgam separators is
A. 34 B. 33
C. 108 D. 107
22. ADA specification number for gypsum bonded investments is
A. 2 B. 4
C. 106 D. 38
23. ADA specification number for impression compound is
A. 3 B. 125
C. 66 D. 70
24. ADA specification number for wrought alloy is
A. 39 B. 40
C. 7 D. 13
25. ADA specification number for ethyl silicate bonding casting investment is
A. 91 B. 5
C. 112 D. 7
Refer synopsis for all other ADA/ANSI specification numbers

PROPERTIES OF DENTAL MATERIALS


1. When the wetting is complete, what will be the contact angle?
A. 90 B. 0
C. 0–90 D. >90
2. Modulus of elasticity means
A. Rigidity or stiffness of the material B. Ability to be stretched with permanent deformation
C. Ductility of a material D. Malleability of a metal
https://t.me/DentalBooksWorld
Chapter 11 • Dental Materials 1019

3. KNH of enamel is close to


A. Pure gold B. Amalgam
C. Porcelain D. Composite
4. Sublimation is the conversion of
A. Solid directly to gas B. Gas to liquid
C. Gas directly to liquid D. Solid to liquid
5. Coefficient of thermal expansion of which of the following is most similar to that of tooth?
A. Silicate B. Type II GIC
C. Composites D. Amalgam
6. Viscosity of the liquid increases with the increase in shear stress in case of
A. Dilatant fluid B. Thixotropic fluid
C. Newtonian fluid D. Pseudoplastic fluid
7. Which of the following properties of dental materials is time dependent?
A. Creep B. Yield strength
C. Elastic limit D. Resilience
8. Relative inability of a material to deform plastically before it fractures
A. Fracture toughness B. Brittleness
C. Yield strength D. Proportional limit
9. This property is reported quantitatively as percent elongation
A. Ductility B. Malleability
C. Resilience D. Elastic modulus
10. Regarding toughness
A. It is defined as the ability of a material to absorb elastic energy and to deform plastically before fracturing
B. Measured as the total area under a plot of tensile stress versus strain
C. Both of the above are correct
D. Both of the above are incorrect

AMALGAM
1. Difference between gelation and liquefaction temperature is called
A. Imbibition B. Syneresis
C. Hysteresis D. Adsorption
2. Which of the following impression material requires a hardener?
A. Agar B. Alginate
C. Elastomers D. Impression compound
3. The setting time of elastomeric impression material is defined as
A. The time from the start of the mixing till the material is fully set
B. The time elapsed from the beginning of the mixing until curing has advanced
C. The time from the start of the mixing till just before the elastic properties have fully developed
D. None of the above
4. The function of 2% potassium sulfate in a gypsum product is
A. To increase the setting expansion B. Regulate setting time – reduces it
C. Acts as retarder – increases setting time D. None of the above
5. Tg (glass transition temperature) of impression compound is
A. 39°C B. 43.5°C
C. 65°C D. 100°C
6. The major disadvantage of polysulfide impression material in clinical practice is
A. Poor biocompatibility B. Poor tear strength
C. It is radiolucent D. It stains clothes and has unacceptable odor
7. Which of the following is mucostatic impression material?
A. Agar B. Alginate
C. ZOE paste D. Impression compound

https://t.me/DentalBooksWorld
1020 Triumph's Complete Review of Dentistry

8. Which of the following are characteristic features of high-copper amalgam alloy?


A. Low compressive strength B. High marginal breakdown
C. Less marginal breakdown D. High creep
9. Dimensional stability of elastomeric impression material can be given in descending order as
A. Polysulfide > polyether > condensation silicone > addition silicone
B. Polyether > condensation silicone > polysulfide > addition silicone
C. Addition silicone > polyether > polysulfide > condensation silicone
D. Addition silicone > condensation silicone > polysulfide > polyether
10. Rough surface of elastomeric impression results from
A. Inadequate mixing
B. Air bubbles
C. Too rapid polymerization
D. Incomplete polymerization caused by premature removal from mouth
11. The term condensation polymerization refers to
A. Only one functional group is repeated in polymerization
B. No by-product is formed
C. Elimination of small molecules as by-product occurs
D. Polymerization is accompanied by repeated elimination of small molecules
12. The flow of impression compound at 37°C
A. 2% B. 4%
C. 6% D. 10%
13. Which of the following is a disadvantage of polysulfide impression material in clinical practice?
A. Poor biocompatibility B. Poor tear strength
C. It is radiolucent D. Too long setting time
14. Which of the following is rigid and irreversible impression that sets by chemical reaction?
A. Agar B. Alginate
C. ZOE paste D. Impression compound
15. Rough surface of elastomeric impression results from
A. Inadequate mixing
B. Air bubbles
C. Too rapid polymerization
D. Incomplete polymerization caused by premature removal from the mouth
16. Which of the following is the most flexible impression material?
A. Impression compound B. Impression paste
C. Addition silicone D. Alginate
17. Main disadvantage of polysulfide impression material is
A. Distortion B. Leaching products
C. Cannot be electroplated D. Difficulty in pouring cast
18. The polishing agent that can be used to polish amalgam restorations is
A. Garnet B. Emery
C. Silex D. Alumina
19. The deoxidizing agent added to dental alloys is
A. Palladium B. Silver
C. Copper D. Zinc
20. Gallium substitutes for which of the following in gold alloys have
A. Silver B. Tin
C. Indium D. Zinc
21. Co-efficient of thermal expansion of amalgam is
A. 6.6 a (ppm k–1) B. 11.4 a (ppm k–1)
–1
C. 14.0 a (ppm k ) D. 25.0 a (ppm k–1)

https://t.me/DentalBooksWorld
Chapter 11 • Dental Materials 1021

22. Which of the following increases the viscosity and rigidity of agar impression material?
A. Borax B. Potassium sulfate
C. Diatomaceous earth D. Glycerine
23. The impression material to make impression of teeth and soft tissues with reversible setting mechanism is
A. Alginate hydrocolloid B. Polysulfide
C. Agar hydrocolloid D. Polyether
24. Catalyst present in the elastomeric impression material is
A. Polysulfide polymer B. Titanium dioxide
C. Lead dioxide D. Lithopone
25. At the end of 24 hours, the contraction of impression material for type I and III materials is
A. 0.5% B. 0.05%
C. 0.1% D. 0.01%
26. Reactor used in alginate hydrocolloid is
A. Trisodium phosphate B. Calcium sulfate dihydrate
C. Zinc oxide D. Potassium titanium fluoride
27. Percentage of agar in agar impression material is
A. 15% B. 80%
C. 10% D. 55%
28. Gum dammer or dammer resin is a natural resin added to paraffin to improve its smoothness in molding and will
render it more
A. Toughness B. Resistant
C. Smoothness D. Flow
29. Which component of irreversible hydrocolloid causes allergic reaction?
A. Phenolphthalein B. Sodium fluorotitanate
C. CaSO4 D. MgO
30. Second amalgam was initiated by
A. Alfred Stock B. H. A. Huggins
C. William Taggart D. None of the above
31. Zinc-free alloys contain
A. Less than 1% zinc B. Less than 0.1% zinc
C. Less than 0.01% zinc D. They do not contain zinc
32. Which of the following decreases delayed expansion?
A. Mercury B. Zinc
C. Copper D. Palladium
33. The average particle sizes of modern powders range between
A. 15 µm and 35 µm B. 5 µm and 15 µm
C. 5 µm and 30 µm D. 35 µm and 50 µm
34. Sn8 Hg – this phase is known as
A. GAMMA 2 B. GAMMA 1
C. ETA D. GAMMA
35. Total copper content in single composition copper is
A. 13–30 wt % B. 9–20 wt %
C. Less than 6% D. More than 30%
36. All of the following causes expansion of amalgam except
A. Postoperative sensitivity B. Protrusion of restoration
C. Pressure on pulp D. Longer trituration time
37. Compressive strength at 1 hour is least for
A. Low copper B. Admixed alloy
C. Unicompositional alloy D. Amalgam with more silver

https://t.me/DentalBooksWorld
1022 Triumph's Complete Review of Dentistry

38. Recommended creep value is


A. Less than 1% B. More than 1%
C. Less than 3% D. Less than 0.8%
39. Amalgam has a linear coefficient of thermal expansion that is _____________ times greater than the tooth structure
A. 2 times B. 2.5 times
C. 4 times D. 3.5 times
40. Minimum compressive strength of 80% for amalgam should be achieved at the end of
A. 1 hour B. 30 minutes
C. 6 hours D. 9 hours

DENTAL CEMENTS
1. Which of the following restorative materials demonstrates maximum water solubility?
A. Light cure GIC B. Microfilled composite
C. Conventional composites D. Hybrid composites
2. pH of non-eugenol cement is
A. 6 B. 7
C. 8 D. 9
3. Ionic bonding is found in
A. GIC B. Polycarboxylate cement
C. Zinc phosphate cement D. All of the above
4. Cement that resembles dental caries radiographically is
A. Ca(OH)2 B. Zinc oxide eugenol
C. Amalgam D. Zinc phosphate
5. Restoration that is radiolucent is
A. Amalgam B. Composite
C. Zinc oxide eugenol D. Zinc phosphate
6. Liquid used in polycarboxylate cement is
A. Poly acrylic acid B. Phosphoric acid
C. Eugenol D. Methacrylic acid
7. Resins are incorporated in zinc oxide eugenol cements to
A. Decrease the strength B. Increase the strength
C. Decrease the film thickness D. None of the above
8. Which of the cements has thermal conductivity similar to that of dentine?
A. Calcium hydroxide B. Zinc silicophosphate
C. Zinc oxide eugenol D. IRM
9. Cavity varnish should not be used with resin restorations as
A. It interferes with the adhesion of resin B. Methyl methacrylate dissolves the cavity varnish
C. It affects the strength of the restoration D. It prevents fluoride uptake by the enamel
10. Vitremer is
A. Metal-modified GIC B. Polyacid-modified composite
C. Resin-modified GIC D. Light-cured GIC
11. Important property of pit and fissure sealant is
A. Low viscosity B. Color
C. High filler content D. Fluoride release
12. Which of the following cements provides maximum resistance to enamel decalcification when used with orthodontic
bands?
A. GIC B. Silicate
C. ZnPO4 D. Zinc polycarboxylate
13. Zinc phosphate compared to GIC has
A. Higher compressive strength B. Higher tensile strength
C. Higher modulus of elasticity D. Greater film thickness

https://t.me/DentalBooksWorld
Chapter 11 • Dental Materials 1023

14. Compomer is
A. Polyacid-modified composite B. Polyacid-modified GIC
C. Polyacid-modified resin D. Polyacid-modified acrylic
15. Which of the following has low anticariogenic property?
A. Zn phosphate B. GIC
C. Zinc oxide eugenol D. Zinc polycarboxylate
16. Disadvantages of zinc phosphate cement are
A. Poor biocompatibility B. Acidic pH
C. Low compressive strength D. All of the above
17. The main purpose of stannous fluoride in zinc polycarboxylate cement is
A. Fluoride release B. To increase strength
C. For better handling purpose D. To increase setting time
18. Which of the following cements is truly adhesive to the tooth structure?
A. GIC B. ZOE
C. Calcium hydroxide D. Zinc phosphate
19. Which of the following cements has obtundant property?
A. Zinc phosphate B. GIC
C. Zinc polycarboxylate D. Zinc oxide eugenol
20. Ceramic restorations are cemented using
A. GIC cement B. Resin cement
C. Zinc phosphate cement D. Zinc oxide eugenol cement
21. Which of the following is similar to composite in composition?
A. Zinc oxide eugenol B. Glass ionomer cement
C. Varnish D. Resin cement
22. Permanent cement that is mixed on glass slab is
A. Bonding agents B. Glass ionomers
C. Zinc oxide eugenol D. Zinc phosphate
23. Which of the following is very diverse in its application?
A. Bonding agents B. Composites
C. Resin cements D. Glass ionomer cements
24. Which of the following cements is/are used as an intermediate restorative material?
A. Zinc phosphate B. Polycarbonates
C. Both of the above D. None of the above
25. Thickness of varnishes should be
A. 0.01 mm B. 0.1 mm
C. 1 mm D. 0.25 mm
26. Varnish should not be used under restorations of
A. GIC B. Composite
C. Resin modified GIC D. All of the above
27. Which of the following is not an objective of a base?
A. To protect pulp from various irritants B. To encourage recovery of the injured pulp
C. To provide mechanical support for the pulp D. None of the above
28. Type III ZOE is used in
A. Temporary cementation B. Permanent cementation
C. Temporary filling D. Cavity liner
29. Type of ZOE used in cavity liners is
A. Type I B. Type II
C. Type III D. Type IV
30. Least percentage of which of the following is found in zinc oxide-eugenol cement
A. Zn oxide B. White resin
C. Zn stearate D. Olive oil

https://t.me/DentalBooksWorld
1024 Triumph's Complete Review of Dentistry

31. Which of the following has low pH?


A. Zinc phosphate B. GIC
C. Cermet D. Zinc polycarboxylate
32. Which of the following has highest solubility?
A. Calcium hydroxide B. Zinc phosphate
C. Zinc oxide eugenol D. Resin cements
33. Which of the following cements lacks anticariogenic properties?
A. Zinc phosphate B. Calcium hydroxide
C. GIC D. Zinc silicophosphate
34. Which of the following cements cannot be used when esthetic demand is high?
A. Calcium hydroxide B. Zinc silicophosphate
C. Zinc phosphate D. Cermet
35. Which of the following cements has low modulus of elasticity?
A. Zinc phosphate B. Zinc polycarboxylate
C. GIC D. Resin cements
36. Recommended powder liquid ratio for zinc phosphate cement is
A. 1.4 mg powder to 0.5 ml liquid B. 2.4 mg powder to 1.5 ml liquid
C. 3.4 mg powder to 2.5 ml liquid D. 1.4 mg powder to 1 ml liquid
37. Acid concentration of polyacrylic acid used in zinc polycarboxylate cement is
A. 50% B. 70%
C. 37% D. 20%
38. The purpose of tartaric acid in GIC is to
A. Improve handling characteristics B. Increase working time
C. Decrease viscosity D. All of the above
39. The particle size for luting/cementing agents in GIC cement is
A. 15 micrometers B. 25 micrometers
C. 10 micrometers D. 5 micrometers
40. GIC type used for luting and orthodontic brackets’ bonding purpose is
A. Type 1 B. Type 2A
C. Type 2B D. Type 3
41. Restoration that is radiolucent is
A. Amalgam B. Composite
C. Zinc oxide eugenol D. Zinc phosphate
42. Compressive strength of small particle hybrid composite is (in MPa)
A. 350–400 B. 300–350
C. 250–300 D. 400–450
43. Size of flowable hybrid composite in micrometers is
A. 8–12 B. 0.5–3
C. 0.04–0.4 D. 0.6–1
44. Filler size of mini filler in micrometers is
A. 10–100 B. 0.1–10
C. 0.1–1 D. 0.01–0.1
45. Radiopacity of small particle hybrid composite in mm aluminum is
A. 1–2 mm B. 0.1 mm
C. 2–3 mm D. 2–4 mm
46. Compressive strength is least for
A. Unfilled acrylic resin B. Traditional composites
C. Hybrid composites D. Microfilled composites
47. Water sorption is highest for
A. Microfilled composites B. Hybrid-small particles
C. Traditional D. Hybrid-flowable

https://t.me/DentalBooksWorld
Chapter 11 • Dental Materials 1025

48. Knoop hardness number is least for


A. Unfilled acrylic B. Traditional composites
C. Hybrid D. Microfilled
49. Filler (in volume %) content is highest for
A. Hybrid-small particle B. Microfilled composites
C. Flowable composites D. Packable composites
50. Filler (in volume %) content of microfilled composites is
A. 60–70% B. 20–60%
C. 80–90% D. 65–80%
51. Which component of composites exhibits estrogenicity?
A. Bis – GMA B. TEGDMA
C. UDMA D. Bis – EMA
52. A major problem with microfilled composites is
A. They tend to be sticky and slump
B. They tend to wear off soon (inferior ability to resist wear)
C. They cannot be polished
D. They are more radiolucent
53. A major problem with microfilled composites is
A. They cannot be used much in anterior surfaces as they are more radioopaque
B. They tend to wear off soon (inferior ability to resist wear)
C. They cannot be polished
D. They have less filler content
54. Statement A – Macrofilled composites are easier to handle than micros filled to the same density
Statement B – Greater surface-to-volume ratios give microparticles one advantage over macroparticles
A. Both statements are correct B. Both statements are wrong
C. Statement A is wrong and B is correct D. Statement B is wrong and A is correct

GYPSUM
1. Hydrocal is
A. Type 1 stone B. Type 2 stone
C. Type 3 stone D. Type 4 stone
2. Setting expansion of gypsum products is
A. 0.06–0.5% B. 0.07–0.6%
C. 0.08–0.6% D. 0.09–0.9%
3. Water–powder ratio of dental stone is
A. 0.30 B. 0.40
C. 0.50 D. 0.20
4. Mixtures of low water–powder ratio will have
A. More expansion and more heat release B. Less expansion and less heat release
C. More expansion and less heat release D. Less expansion and more heat release
5. Accelerators in gypsum products are all except
A. Sodium chloride (up to 2% of hemihydrate) B. Sodium sulfate (max. effect at 3.4%)
C. Potassium sulfate (<2%) D. Potassium tartrate
6. In type 2 model plaster
A. Powder particles are porous and irregular B. Powder particles are more dense and regular in shape
C. Powder particles are porous and regular D. Powder particles are less dense and irregular
7. In gypsum products, the effect of temperature on setting time is
A. Retardation occurs if the temperature exceeds 50°C
B. Retardation occurs if the temperature decreases below 50°C
C. No relation between the setting reaction and the temperature
D. No reaction takes place if the temperature exceeds 50°C

https://t.me/DentalBooksWorld
1026 Triumph's Complete Review of Dentistry

8. The gypsum product having the highest setting expansion is


A. Type II plaster B. Type III stone
C. Type IV stone D. Type V stone
9. Theory originated in 1893 by M. Michaelis is
A. Colloidal/gel theory B. Hydration theory
C. Crystalline theory D. Dissolution–precipitation theory
10. Theory originated in 1887 by Henry Louis Le Chatelier is
A. Colloidal/gel theory B. Hydration theory
C. Crystalline theory D. Dissolution–precipitation theory
11. W/P ratio for type 2 gypsum products is
A. 0.45–0.50 B. 0.28–0.30
C. 0.40–0.45 D. 0.20–0.22

INVESTMENTS
1. The function of fluoride flux during soldering of stainless steel is
A. It forms a surface protective layer B. It decreases the melting point
C. It dissolves the chromic oxide layer D. None
2. Which of the following acts as a gypsum hardener?
A. 5% NaCl B. 2% KCl
C. 2% K2SO4 D. 1% Al2SO3
3. The use of quartz in gypsum-bonded investment is
A. Counteracts expansion B. Provides strength
C. Is a retarder D. None of the above
4. In the investment for gold, the strength is provided by
A. Silica B. Gypsum
C. Cristobalite D. Carbon
5. Which part of flame has maximum heat content?
A. Reducing zone B. Combustion zone
C. Oxidizing zone D. Central zone
6. Basic unit of a metal is
A. Grain B. Nucleus
C. Dendrite D. Embryo
7. To minimize distortion, the die should be poured
A. Within 30 minutes B. 40 minutes–1 hour
C. 1 hour–2 hours D. After 2 hours
8. Function of wetting agents used in casting procedure is
A. To facilitate wetting of ring liner B. To facilitate mixing investment
C. To reduce contact angle of a liquid with wax surface D. For better wax elimination
9. Gypsum-bonded investment should not be heated above
A. 700°C B. 750°C
C. 800°C D. 900°C
10. Finer the particle size of silica
A. Slower the hygroscopic expansion B. Normal setting expansion
C. Greater the hygroscopic expansion D. No setting expansion
11. The length of the sprue former should be adjusted for gypsum-bonded investor and the top of the wax pattern is
within
A. 6 mm of the open end of the ring B. 10 mm of the open end of the ring
C. 1 mm of the open end of the ring D. 2 mm of open end of the ring
12. Degassing means
A. Removal of residual surface contamination B. Removal of Grease
C. Addition of gasses D. Disinfecting

https://t.me/DentalBooksWorld
Chapter 11 • Dental Materials 1027

13. The inability of the air in the mold to escape through the pores in the investment leads to
A. Localized shrinkage porosity B. Back pressure porosity
C. Subsurface porosity D. Suck back porosity
14. The premature termination of the flow of molten metal during solidification is termed
A. Gas inclusion porosity B. Subsurface porosity
C. Localized shrinkage porosity D. Back
15. Of the various glass ionomer products, the highest cumulative release of fluoride after 30 days is from
A. Type I glass ionomer B. Type II glass ionomer
C. Glass ionomer liner (conventional) D. Glass ionomer liner (light–cure)
16. Cavity varnish should not be used with resin restorations as
A. It interferes with the adhesion of resin B. Methyl methacrylate dissolves the cavity varnish
C. It effects the strength of the restoration D. It prevents fluoride uptake by the enamel
17. Modern metal casting alloys have equiaxed fine grain microstructure due to incorporation of small amounts of
iridium, ruthenium, or rhenium as grain refining elements for palladium alloy are
A. Less than 1 wt% B. More than 1 wt%
C. Less than 2 wt% D. More than 2 wt%
18. Maximum hardness in traditional high gold types III and IV casting alloys is achieved in 1–4 hours after isothermal
annealing at elevated temperatures of
A. 130–250°C B. 100–230°C
C. 230–350°C D. 330–450°C
19. Inlay waxes may be softened over flame or in water to enable their flow in a liquid state and adaptation to prepared
tooth or die at the temperature of
A. 110–120°F B. 100–120°F
C. 130–140°F D. 140–150°F
20. Gum dammer or dammer resin is a natural resin added to paraffin to improve its smoothness in molding and will
render it more
A. Toughness B. Resistant
C. Smoothness D. Flow
21. Type of investment material used for casting of removable partial denture with base metal alloy is
A. Gypsum-based material B. Phosphate-based material
C. Ethyl silicate bonded D. All of the above
22. Hygroscopic expansion of investment material increases with the increase in
A. Silica content B. Water–powder ratio
C. Particle size D. Shelf life
23. The soldering temperature for orthodontic silver solders is between
A. 550 and 555°C B. 620 and 625°
C. 670 and 675° D. 595 and 600°C
24. Joining of metal surfaces that occurs locally without a filler metal is termed
A. Brazing B. Soldering
C. Welding D. Electroforming

BONDING
1. Which of the following contains primer and bonding agent in a single bottle and is known as “Single Component System”?
A. 3 B. 4
C. 5 D. 6
2. A layer of coating of bonding agent is applied with a brush so that oxygen from air does not pass through and the
bonding agent thickness used to prevent oxygen penetration is
A. 100 µm B. 200 µm
C. 150 µm D. 50 µm
3. Currently, the etching time for most etching gels is approximately
A. 10 seconds B. 15 seconds
C. 20 seconds D. 25 seconds
https://t.me/DentalBooksWorld
1028 Triumph's Complete Review of Dentistry

4. The apparent bond strength that is the cohesive strength of the smear layer is
A. 2 MPa B. 5–10 MPa
C. 15 MPa D. 3–4 MPa
5. Bond strength is highest for
A. Second generation B. Third generation
C. Fourth generation D. Sixth generation
6. Who introduced the total etch concept?
A. Fusayama B. Buonocore
C. Nakabayashi D. All of the above
7. Once the tooth is etched, the acid should be rinsed away thoroughly with a stream of water for about
A. 10 seconds B. 20 seconds
C. 25 seconds D. 15 seconds
8. Dimensions of resin tags – in diameter and length, respectively, are
A. 6 µm in diameter and 10–20 µm in length B. 5 µm in diameter and 20–30 µm in length
C. 7 µm in diameter and 10–20 µm in length D. 6 µm in diameter and 9 µm in length
9. Primer is
A. Hydrophilic B. Hydrophobic
C. Both D. None
10. For cementation of all ceramic restorations (can be chemically or light or dual cured), the cement that is the material
of choice is
A. GIC B. Zinc phosphate
C. Resin cements D. Rm GIC
11. For bonding posterior composite restorations, the recommended generation is
A. Two-step etch-and-rinse (fifth generation) B. Etch-and-rinse (fourth generation)
C. Dual-cure one-step, self-etch (seventh generation) D. Light-cured one-step, self-etch (seventh generation)
12. The hybrid layer thickness is
A. 0.5–5 μm B. Less than <0.5 μm
C. 5–9 μm D. 2–9 μm
13. Hybrid layer was given by
A. Fusayama B. Buonocore
C. Nakabayashi D. All of the above

CERAMICS
1. Low fusing alloys are
A. >1,300°C B. 850–1,000°C
C. Less than 850°C D. 1,000–1,300°C
2. Potash fuses with kaolin and quartz to form glass when heated from
A. 1,250 to 1,500°C B. 1,500 to 1,800°C
C. 1,050 to 1,200°C D. 800 to 1,000°C
3. Modulus of elasticity of dental ceramic is
A. 20 GPa B. 70 GPa
C. 110 GPa D. 50 GPa
4. Compressive strength of dental ceramic is
A. 200 MPa B. 450 MPa
C. 300 MPa D. 500 MPa
5. Porosity in porcelain at condensation is determined by
A. Size an d shape of particles B. Uniform distribution of particle size
C. Number of particles D. None of the above
6. Maximum shrinkage on firing of ceramic occurs during which stage
A. High bisque B. Low bisque
C. During condensation D. All of the above

https://t.me/DentalBooksWorld
Chapter 11 • Dental Materials 1029

7. Maximum shrinkage on firing of ceramic occurs during which stage


A. High bisque B. Low bisque
C. Medium bisque stage D. None of the above
8. Ceramic restorations are cemented using
A. GIC cement B. Resin cement
C. Zinc phosphate cement D. Zinc oxide eugenol cement
9. The first porcelain tooth material was introduced by
A. DeChemant B. Hoe et al.
C. Plateau D. Mc-Clean
10. In which of the following, high fusing porcelains are used?
A. For crowns B. In denture teeth
C. With titanium alloys D. Over aluminous core porcelain
11. Feldspathic porcelain
A. Shows reliable chemical bonding with metals B. Has high tensile strength
C. Is no longer used D. Shows insignificant firing shrinkage
12. One of the purposes of etching beveled enamel surface is to
A. Increase surface energy B. Produce macro under cut
C. Increase convenience D. Prevent caries
13. Tooth preparation for glass ceramic on occlusal and incisal edges must be reduced to a minimum of
A. 2 mm B. 1 mm
C. 1.5 mm D. 2.5 mm
14. The most important cause of porosity in a porcelain jacket crown is
A. Excessive firing temperature B. Moisture contamination
C. Inadequate condensation of the porcelain D. Failure to anneal the platinum matrix
15. Which of the following is incorporated in porcelain during firing?
A. Argon B. Helium
C. Hydrogen D. Oxygen
16. Ormocers are
A. Organically modified glass ionomers B. Organically modified ceramics
C. Composites modified glass ionomers D. Glass ionomer modified composites
17. For PFM (porcelain fused to metal) crown, the porcelain should have
A. High fusion expansion
B. High fusion temperature
C. Linear coefficient of thermal expansion less than that of the metal
D. Linear coefficient of thermal expansion more than that of the metal
18. Which of the following elements in metallo ceramic alloys produces “greening” effect of porcelain?
A. Silver B. Palladium
C. Platinum D. Gold
19. The average thickness of opaque porcelain in a metal ceramic crown is usually
A. 0.3 mm B. 0.7 mm
C. 0.5 mm D. 0.1 mm
20. The absence of shrinkage in shrink-free ceramic (Cerestores) is because
A. It contains silica B. Of injection molded
C. Contains organic silicon which gets converted to silica D. Contains mica crystals which prevent outward thrust
21. Indium is added to metal alloy in porcelain fused to metal
A. To improve bonding B. To increase strength
C. To decrease porosity D. All of the above
22. Crystalline phase of ceramic/porcelain is increased by
A. Transformation toughening B. Glazing
C. Sintering D. Ceramming

https://t.me/DentalBooksWorld
1030 Triumph's Complete Review of Dentistry

23. Chemical strengthening of ceramics is achieved by


A. Aluminosilicate and Na+ ions B. Potassium nitrate and Na+ ions
C. Al2O3 and K+ ions D. Aluminosilicate and K+ ions
24. Material preferably used for cementing ceramic veneers is
A. Resin cement B. GIC
C. Zinc phosphate cement D. Zinc polycarboxylate cement
25. Which of the following has Knoop hardness number close to that of enamel?
A. Pure gold alloy B. Amalgam
C. Composite D. Porcelain
26. Concentricity of a bur gives the static measurements of
A. Symmetry of the blade B. Width of the bur head
C. Symmetry of the bur head D. Shape of the bur head
27. Ceramic that is used in porcelain fused to metal restorations consists of ceramic that is
A. High fusing B. Ultrahigh fusing
C. Low fusing D. Ultralow fusing
28. Compared with dental alloys, ceramics show
A. High ductility B. High hardness
C. Low esthetic value D. Susceptibility to brittle fracture
29. The first commercially available castable ceramic material for dental use is
A. Dicor B. IPS Empress
C. In-ceram D. Captek
30. Greening occurs when porcelains are fired on silver-free alloys which may be attributed to
A. Vaporization of silver from the walls of contaminated furnaces
B. Surface diffusion of silver from the marginal metal
C. More rapid silver diffusion in sodium containing glass
D. Silver ionization by porcelains with high oxygen potential
31. Bonding in porcelain fused to metal restorations is
A. Chemical and mechanical in nature B. Only chemical bonding
C. Only mechanical bonding D. None of the above
32. The first layer of porcelain applied on to a metal in PFM crowns would be
A. Opaque B. Body
C. Incisal D. Both A and B
33. What is incorporated in porcelain during firing?
A. Argon B. Helium
C. Plydrogen D. Oxygen
34. Linear coefficient of thermal expansion of metal used for metal ceramic restoration ranges from
A. 11.5 to 12 .5 × 10–6/°C B. 10. 5 to 11.5 × 10–6/°C
–6
C. 13.5 to 14.5 × 10 /°C D. 16. 5 to 17.5 × 10– 6/°C
35. Feldspathic porcelain is
A. K2O Al2O3 6SiO2 B. MgO Al2O3
C. K2OMgO 6SiO2 D. Al3F6

DENTAL WAX
1. Which of the following cannot be completely burnout while casting?
A. Pattern wax B. Inlay wax
C. Acrylic D. Composite
2. Type I wax is
A. Medium and soft B. Hard wax
C. Only soft D. Medium wax
3. Carnauba wax is added to paraffin wax to
A. Decrease flow at mouth temperature B. Increase flow at mouth temperature
C. Increase strength D. Give good odor
https://t.me/DentalBooksWorld
Chapter 11 • Dental Materials 1031

4. The average linear thermal expansion coefficient of wax is


A. 250 × 10−8/K B. 350 × 108/K
−6
C. 250 × 10 /K D. 350 × 10−6/K
5. Glass transition temperature of wax
A. 35°C B. 45°C
C. 49°C D. 54°C
6. Processing wax are all except
A. Boxing B. Utility
C. Sticky D. Inlay
7. Pattern wax among the following is
A. Boxing B. Baseplate
C. Utility D. Impression wax
8. Both type I and type II waxes at 45°C must have a minimal flow of ______________ and a maximum flow of
______________respectively
A. 70% and 90% B. 90% and 70%
C. 50% and 50% D. 60% and 80%
9. Which is used to establish the initial arch form in the construction of complete dentures?
A. Baseplate wax B. Casting wax
C. Utility wax D. Inlay wax
10. Which wax is used for trial fitting in the mouth in tropical climates?
A. Type I wax B. Type II wax
C. Type III wax D. All of the above
11. Impression wax is also known as
A. Corrective wax B. Bite wax
C. Both of the above D. None of the above
12. Wax can expand and contract as much as
A. 0.70% and 0.35% B. 0.35% and 0.70%
C. 0.20% and 0.50% D. 0.50% and 0.20%
13. The maximum flow permitted for type I waxes at 37°C is
A. 1% B. 2%
C. 0.1% D. 0.5%
14. Korecta number 3 wax is
A. Hard B. Soft
C. Extra hard D. Extra soft
15. Color of Iowa wax is
A. Yellow–white B. Orange
C. White D. Green
16. Carnauba wax is a component of/obtained from
A. Plant source B. It is synthetic
C. Animal source D. Mineral source
17. Modulus ratio for inlay and soft green casting wax is
A. 7:1 B. 1:7
C. 1:9 D. 9:1
18. Ductility _________ with increase in temperature of waxes
A. Increases B. Decreases
C. Remains stable D. Decreases rapidly
19. Waxes are available in all forms except
A. Universal B. Buccal and lingual
C. Occlusal D. Cervical

https://t.me/DentalBooksWorld
1032 Triumph's Complete Review of Dentistry

MISCELLANEOUS
1. The first reaction after mixing of water with alginate is the reaction of sodium phosphate with
A. Insoluble Ca ions B. Soluble Ca ions
C. Soluble K ions D. Insoluble K ions
2. What are the ways to decrease the setting time of alginate impression materials?
A. Increase water temperature B. Rapid mixing
C. Slow mixing D. Both A and B
3. The process of absorbing water which leads to expansion of alginate is known as
A. Syneresis B. Imbibition
C. Sublimation D. Sintering
4. The process of exudation of the liquid component of a gel which leads to shrinkage of alginate is known as
A. Syneresis B. Imbibition
C. Sublimation D. Sintering
5. Major ingredient of alginate impression materials is
A. Sodium alginate B. Calcium sulfate dehydrate
C. Diatomaceous earth D. Potassium sulfate
6. Minor ingredient of alginate impression materials is
A. Sodium alginate B. Calcium sulfate dehydrate
C. Diatomaceous earth D. Sodium phosphate
7. Consistency and flexibility of set impression of alginate are controlled by
A. Sodium alginate B. Calcium sulfate dehydrate
C. Diatomaceous earth D. Potassium sulfate
8. Which of the following ingredients of alginate counteracts inhibiting effect of alginate on stone surface?
A. Sodium alginate B. Calcium sulfate dehydrate
C. Diatomaceous earth D. Potassium sulfate
9. The setting time of alginate impression materials is controlled by
A. Sodium alginate B. Calcium sulfate dehydrate
C. Potassium sulfate D. Sodium phosphate
10. Type of calcium hemihydrate in dental stone is
A. Alpha B. Beta
C. Delta D. Gamma
11. Type of calcium hemihydrate in dental plaster is
A. Alpha B. Beta
C. Delta D. Gamma
12. The principal component of dental gypsum products is
A. Calcium sulfate hemihydrate B. Calcium sulfate dihydrate
C. Potassium sulfate hemihydrate D. Potassium sulfate dihydrate
13. Types of calcium hemihydrate are obtained through the process of
A. Sintering B. Fritting
C. Calcination D. Precipitation
14. The setting reaction of 1 g of calcium hemihydrate yields how much calories
A. 3,100 B. 3,600
C. 3,900 D. 4,100
15. For the impressions of flabby tissues, which of the following impression materials is used?
A. Impression plaster B. Model plaster
C. Alginate D. Impression compound
16. Type 2 gypsum product is
A. Impression plaster B. Model plaster
C. Dental stone D. High-strength dental stone

https://t.me/DentalBooksWorld
Chapter 11 • Dental Materials 1033

17. For diagnostic casts, which of the following materials is used?


A. Impression plaster B. Model plaster
C. Dental stone D. High-strength dental stone
18. For master casts for die fabrication, which of the following materials is used?
A. Impression plaster B. Model plaster
C. Dental stone D. High-strength dental stone
19. Term gummy smile indicates that when gingival display is
A. >5 mm B. >3 mm
C. >4 mm D. <3 mm
20. Gull wing effect is shown by the outline of the incisal edges from
A. Incisor to incisor B. Canine to canine
C. Premolar to premolar D. Molar to molar
21. An elastomeric impression material that has an exothermic setting reaction with water as a by-product is
A. Polyether B. Polysulfide
C. Addition silicon D. Condensation silicon
22. Polysulfide impressions must be poured within
A. 15 minutes B. 30 minutes
C. 45 minutes D. 1 hour
23. Auto-polymerized polymethyl methacrylate shows how much shrinkage during polymerization
A. 0.2% B. 0.5%
C. 1.0% D. 1.5%
24. Heat-polymerized polymethyl methacrylate shows how much shrinkage during polymerization
A. 1–7% B. 2–7%
C. 3–7% D. 2–3%
25. Best provisional restoration material is
A. PMMA B. PEMA
C. Bis-acryl D. All of the above
26. An impression material that sets in a cross-linking polymerization reaction and gives off the by-product ethanol is
A. Polyether B. Polysulfide
C. Condensation silicones D. Addition silicones
27. An impression material that changes from a gel to a sol with the application of heat and is reversible with cooling is
A. Polyether B. Polysulfide
C. Condensation silicones D. Agar
28. The most popular crown and bridge impression material and there are no by-products of reaction is
A. Polyether B. Polysulfide
C. Condensation silicones D. Addition silicones
29. Maximum stress before fracturing is known as
A. Strength B. Ultimate tensile strength
C. Yield strength D. Modulus
30. Rigidity or stiffness is determined by
A. Strength B. Ultimate tensile strength
C. Yield strength D. Modulus
31. Resistance to permanent deformation is known as
A. Strength B. Ultimate tensile strength
C. Yield strength D. Modulus
32. Which of the following is important for wear characteristics and finishing?
A. Strength B. Ultimate tensile strength
C. Yield strength D. Hardness
33. Alginate impression material is
A. Expensive compared to other impression materials B. Easy to use
C. Not affected by gain or loss of water D. Well known for its long-term stability

https://t.me/DentalBooksWorld
1034 Triumph's Complete Review of Dentistry

34. Impression materials that have mechanical properties permitting considerable elastic deformation but that return to
their original form are classified as
A. Thermoplastic B. Elastometric
C. Inelastic D. Resins
35. You mix alginate and take an impression. While measuring the water, you got involved in conversation and did not
notice how warm it was. It will
A. Make the mix unstable B. Lengthen the gelation time
C. Not affect the gelation time D. Shorten the gelation time
36. Which of the following dental materials is an example of an aqueous elastometric?
A. ZOE impression paste B. Polysulfide
C. Irreversible hydrocolloid D. Addition silicone
37. The brown paste used in rubber base is called the
A. Polymer B. Accelerator
C. Base D. Filler
38. Dental impression compound is known as a/an
A. Chemoplastic material B. Irreversible material
C. Hydroelastic material D. Thermoplastic material
39. Which of the following is an example of an inelastic impression material?
A. Polysulfide B. ZOE impression paste
C. Alginate D. Addition silicone
40. The term used for the setting of hydrocolloid impression materials is
A. Crystallization B. Polymerization
C. Curing D. Gelation
41. The popularity of agar impression materials is limited by the
A. High cost B. Need for special equipment
C. Poor reproduction of detail D. Difficulty in pouring the impression
42. When an agar impression slightly contracts and exudes water, it is termed
A. Imbibition B. Gelation
C. Syneresis D. Hysteresis
43. Addition silicones are the most popular type of rubber impression material. The reason for this is cost
A. True, false B. False, true
C. Both true D. Both false
44. Custom impression trays are made on a model of the patient’s arch. Therefore, to make a custom tray, an alginate
impression is also needed
A. True, false B. False, true
C. Both true D. Both false
45. A patient came to the clinic and it was found that she needed extensive restorative treatment to which she agreed. You
were asked to take impressions as the first step of her care. The impression material of choice would be
A. Agar B. Alginate
C. Dental impression compound D. Addition silicone
46. The impression tray that is used to record the prepared tooth, a bite registration, and an impression of the opposing
teeth is a
A. Stock tray B. Custom tray
C. Triple tray D. Bite registration tray
47. Which of the following impression materials sets by physical means?
A. Agar B. ZOE
C. Alginate D. Addition silicone
48. The desirable strength of gypsum materials is ____ related to the amount of water used
A. Directly B. Indirectly
C. Not D. Partially

https://t.me/DentalBooksWorld
Chapter 11 • Dental Materials 1035

49. For gypsum products, a suitable accelerator and retarder, respectively, would be
A. Ethyl alcohol and oleic acid B. Oleic acid and glycerin
C. Borax and potassium sulfate D. Potassium sulfate and borax
50. The gypsum material known as “high strength stone” may also be referred to as
A. Plaster B. Type II stone
C. Type III stone D. Improved stone
51. The final setting time for gypsum products is typically
A. 15–30 min B. 30–45 min
C. 45–90 min D. 90–120 min
52. To make a correct mix for dental stone when using 50 g of powder, the amount of water would be approximately
A. 10–12 ml B. 14–15 ml
C. 28–30 ml D. 45–50 ml
53. At a previous appointment, the orthodontist took impressions of patient’s maxillary and mandibular arches. The
replicas made from each impression to discuss the treatment plan are termed
A. Casts B. Dies
C. Study models D. Molds
54. Quartz in dental porcelain acts as a
A. Binder B. Frit
C. Opacifier D. Strengthener
55. Which of the following resins may be classified as thermosetting?
A. Polystyrenes B. Vinyl resins
C. Epoxy resins D. Impression compound
56. In general, the hygroscopic expansion of gypsum investments will be higher if they contain
A. Coarser silica particles and alpha hemihydrate B. Finer silica particles and beta hemihydrate
C. Coarser silica particles and beta hemihydrate D. Finer silica particles and alpha hemihydrate
57. The cement capable of forming a chemical bond with the tooth structure
A. Reinforced zinc oxide eugenol cement B. Silicophosphate cement
C. Polycarboxylate cement D. Composite resin cement
58. Silicate cements are made up of oxide or fluorides of all of the following except
A. Aluminum B. Tin
C. Calcium D. Sodium
59. Sodium N-lauroyl sarcosinate is used in dentifrice pastes as a
A. Detergent B. Humectant
C. Therapeutic agent D. Binder
60. During the casting of noble metal alloys, the mold should be held at the burn-out temperature for at least
A. 15 minutes B. 60 minutes
C. 3 hours D. 6 hours
61. The principal strengthening phases of cerestore, an injection molded ceramic material, are
A. Aluminum oxide and silicon oxide B. Magnesium silicate and beta-alumina
C. Alpha-alumina and magnesium aluminate spinel D. None of the above
62. A dental restoration may be tarnished because of
A. Deposition of pigment producing bacteria B. Surface discoloration
C. Formation of oxides or sulfides D. All of the above
63. An agar hydrocolloid impression should never be removed from the mouth by a weaving method because this can
result in distortion of the impression material
A. Both the statement and the reason are true B. The statement is true and the reason is false
C. The statement is false but the reason is true D. Both the statement and the reason are false
64. The principal hardener in noble metal casting alloys is
A. Brass B. Iron
C. Silver D. Copper

https://t.me/DentalBooksWorld
1036 Triumph's Complete Review of Dentistry

65. Resins that soften when heated and set on cooling are termed
A. Thermoset resins B. Irreversible resins
C. Thermoplastic resins D. Elastomeric resins
66. A compressive stress is always accompanied by
A. An elastic strain B. A compressive strain
C. A shearing strain D. A tensile strain
67. Which of the following may result in the expansion of an amalgam restoration made with zinc free alloys?
A. Moisture contamination during condensation B. High mercury/alloy ratio
C. Under trituration D. All of the above
68. Self-cured and heat-cured acrylic resins are most similar in respect of which of the following
A. Color stability B. Curing shrinkage
C. Transverse strength D. Hardness
69. During the setting of gypsum, the mass thickens and then hardens into needle-like clusters called
A. Spicules B. Crystallites
C. Spherulites D. Stellites
70. Which of the following colloidal solutions can be termed aerosol?
A. Liquid in solid B. Gas in liquid
C. Liquid in air D. Solid in liquid
71. Which of the following is required for cross-linking for condensation silicone elastomers?
A. Platinum oxide B. Tin octoate
C. Copper dioxide D. Calcium sulfate
72. Zinc phosphate cement powder contains all of the following except
A. Zinc oxide B. Magnesium oxide
C. Bismuth oxide D. Aluminum phosphate
73. Dental porcelains are manufactured by a process termed
A. Fritting B. Fusing
C. Vulcanizing D. Sintering
74. The structure of Dicor, a castable glass ceramic, is essentially composed of
A. Crystalline mica particles B. Microscopic silica particles
C. Interlocking apatite crystals D. Microcrystalline quartz particles
75. Strength of gypsum investments is provided by
A. Quartz B. Dental stone
C. Tridymite and cristobalite together D. Silica
76. Internal porosity is most likely to occur in which portion of a denture
A. On the surface B. In those portions located near the flask periphery
C. In the center of a thick portion D. In the portions having less thickness of resin
77. Invariant transformation is a property of
A. Eutectic and peritectic alloys B. Peritectic alloys only
C. Eutectic alloys only D. Solid solutions only
78. The chemical used as a catalyst for the cross-linking of addition silicones is
A. Sulfinic acid B. Titanium dioxide
C. Platinum salt D. Palladium salt
79. Which of the following is a thermoplastic material?
A. Impression compound B. Acrylic resin
C. Dental porcelain D. Plaster
80. Abrasive used in air polishing agent is
A. NaCl B. 27 μ alumina particles
C. 50 μ alumina particles D. Sodium bicarbonate

https://t.me/DentalBooksWorld
Chapter 11 • Dental Materials 1037

81. Of all the bevels placed on gold inlay preparation, which is the most important bevel for success of restoration?
A. Occlusal B. Gingival
C. Axiopulpal D. Faciopulpal
82. The gingival cavosurface bevel for a class II preparation to receive a cast gold inlay
A. Results in a 300 metal that is burnishable
B. Results in no benefits whatsoever
C. Results in open margin if casting procedures are not accurate
D. Results in difficulty in taking wax patterns
83. Skipping effect is seen in
A. Use of liquid etchant B. Use of viscous etchant
C. Use of GIC as pit and fissure sealant D. Use of composite as pit and fissure sealant
84. What is a ferrule?
A. Pin retained restorative preparation B. Secondary retentive feature for amalgam restoration
C. Band encircling external dimension of tooth D. Preparation design for composite inlay
85. Transformation toughening is found in ceramics containing
A. Octagonal zirconia B. Tetragonal zirconia
C. Decahedron zirconia D. All of the above
86. Similarity in the chemical structure of estrogen is seen with
A. HEMA B. Polyether
C. Bisphenol A D. Gutta-percha
87. Which of the following drug is commonly used to control salivation in operative dentistry?
A. Pilocarpine B. Propranolol
C. Atropine D. Muscarine
88. Best cement for cementation of porcelain laminate is
A. Dual cure resin B. GIC
C. Composite D. Polycarboxylate
89. Glass infiltrated alumina core ceramic is
A. Dicor B. Inceram
C. Captek D. IPS-empress
90. Porcelain bonded to metal is strongest when it is
A. Air fired B. Fired under compression
C. Tempered after firing D. Fired several times before completion
91. Condensation shrinkage of porcelain during firing depends on
A. Rate of arriving at firing temperature B. Uniformity of particle size
C. Shape and size of particle D. Type of investment used
92. To prevent porosity in dental porcelain, it should be baked
A. In the presence of air B. In vacuum
C. For long period D. Under pressure
93. Which of the following polishing agents is called whitening agent?
A. Precipitated chalk B. Iron oxide
C. Chromium oxide D. Rouge
94. Which of the following should not be used to polish (or) finish amalgam?
A. A ball burnisher B. A rubber (Burlew) disk
C. Finishing bur D. A white stone
95. Machineable glass ceramic is
A. Cerestore B. Décor MGC
C. Infusium D. Leucite

https://t.me/DentalBooksWorld
1038 Triumph's Complete Review of Dentistry

PREVIOUS YEAR QUESTIONS


1. Which of the following cements is truly adhesive to the tooth structure?
A. GIC B. ZOE
C. Calcium hydroxide D. Zinc phosphate
2. Which of the following cements has obtundant property?
A. Zinc phosphate B. GIC
C. Zinc polycarboxylate D. Zinc oxide eugenol
3. Ceramic restorations are cemented using
A. GIC cement B. Resin cement
C. Zinc phosphate cement D. Zinc oxide eugenol cement
4. Which of the following is similar to composite in composition?
A. Zinc oxide eugenol B. Glass ionomer cement
C. Varnish D. Resin cement
5. Permanent cement that is mixed on glass slab is
A. Bonding agents B. Glass ionomer
C. Zinc oxide eugenol D. Zinc phosphate
6. Which of the following is very diverse in its application?
A. Bonding agents B. Composites
C. Resin cements D. Glass ionomer cement
7. Which of the following cements is/are used as an intermediate restorative material?
A. Zinc phosphate B. Polycarbonates
C. Both of the above D. None of the above
8. Thickness of varnishes should be
A. 0.01 mm B. 0.1 mm
C. 1 mm D. 0.25 mm
9. Varnish should not be used under restorations of
A. GIC B. Composite
C. Resin modified GIC D. All of the above
10. Which of the following is not an objective of a base?
A. To protect pulp from various irritants B. To encourage recovery of the injured pulp
C. To provide mechanical support for the pulp D. None of the above
11. Type III ZOE is used in
A. Temporary cementation B. Permanent cementation
C. Temporary filling D. Cavity liner
12. Type of ZOE used in cavity liners is
A. Type I B. Type II
C. Type III D. Type IV
13. Least percentage of which of the following is found in zinc oxide–eugenol cement
A. Zn oxide B. White resin
C. Zn stearate D. Balsam
14. Which of the following restorative materials demonstrates maximum water solubility?
A. Light cure GIC B. Microfilled composite
C. Conventional composites live oil D. Hybrid composites
15. pH of non-eugenol cement is
A. 6 B. 7
C. 8 D. 9
16. Which of the following cannot be completely burnout while casting?
A. Pattern wax B. Inlay wax
C. Acrylic D. Composite

https://t.me/DentalBooksWorld
Chapter 11 • Dental Materials 1039

17. Type I wax is


A. Medium and soft B. Hard wax
C. Only soft D. Medium wax
18. Carnauba wax is added to paraffin wax to
A. Decrease flow at mouth temperature B. Increase flow at mouth temperature
C. Increase strength D. Give good odor
19. The average linear thermal expansion coefficient of wax is
A. 250 × 10−8/K B. 350 × 108/K
−6
C. 250 × 10 /K D. 350 × 10−6/K
20. Glass transition temperature of wax is
A. 35°C B. 45°C
B. 49°C C. 54°C
21. Processing wax are all except
A. Boxing B. Utility
C. Sticky D. Inlay
22. Pattern wax among the following is
A. Boxing B. Baseplate
C. Utility D. Impression wax
23. The inability of the air in the mold to escape through the pores in the investment leads to
A. Localized shrinkage porosity B. Back pressure porosity
C. Subsurface porosity D. Suck back porosity
24. The premature termination of the flow of molten metal during solidification is termed
A. Gas inclusion porosity B. Subsurface porosity
C. Localized shrinkage porosity D. Back pressure porosity
25. Of the various glass ionomer products, the highest cumulative release of fluoride after 30 days is from
A. Type I glass ionomer B. Type II glass ionomer
C. Glass ionomer liner (conventional) D. Glass ionomer liner (light cure)
26. The angle between adhesive and adherent is zero. It indicates
A. Complete wetting of surface B. Increase surface roughness
C. The adherent and adhesive molecules are together D. Irregularities are present between each other
27. Slow or soft start polymerization is seen in
A. LED light cure units B. Laser light cure units
C. Quartz–Tungsten–Halogen light cure units D. PAC light cure units
28. The primary impression material used in flabby ridge is
A. Irreversible hydrocolloid B. Putty silicon
C. Heavy body elastomers D. Impression compound
29. Poisson’s ratio is
A. Lateral strain by axial strain within elastic limit B. Equal to flexural strength
C. More in amalgam D. Low for gold alloy
30. Which of the following is not true about elastomeric impression materials?
A. Single mixture materials have higher viscosity
B. Shear thinning is related to viscosity of monophase impression material
C. Improper mixing of material can cause permanent deformation of impression
D. Putty-wash technique of impression reduces dimensional change on setting
31. Impression material of choice in patients with submucous fibrosis is
A. Zinc oxide eugenol B. Addition silicone
C. Condensation silicone D. Plaster of Paris
32. Which of the following is not correct about dental stone?
A. High-strength low-expansion dental stone contain some additive to reduce expansion
B. Dental stone is manufactured by heating at temperature 110–1, 200°C in an open kettle
C. Dental stone is much harder and stronger than beta-hemihydrates
D. Microscopically it is seen as cleavage fragments and crystal in the form of rods and prisms
https://t.me/DentalBooksWorld
1040 Triumph's Complete Review of Dentistry

33. Which of the following is not true about gypsum-bonded investment material?
A. It is used for gold alloy
B. 65–75% of gypsum changes to form alpha hemi-hydrate
C. The investment material is heated at 500–7,000°C
D. Heating above 7,000°C of investment causes formation of sulfur dioxide from copper sulfate
34. The first porcelain tooth material was introduced by
A. DeChemant B. Ash
C. Plateau D. Mc-Clean
35. The relative hardness of elastomers is determined using
A. Rockwell tester B. Barcol indenter
C. Knoop pyramid D. Shore durometer
36. Resistance of a material to permanent deformation is known as
A. Elongation B. Resiliency
C. Ductility D. Fracture strength
37. The commonest type of porosity that results in a fluid resin technique is
A. Polymerization shrinkage porosity B. Porosity due to inadequate pressure
C. Air inclusion porosity D. Porosity due to increased heat
38. Degassing means
A. Removal of residual surface contamination B. Removal of grease
C. Addition of gasses D. Disinfecting
39. The inability of the air in the mold to escape through the pores in the investment leads to
A. Localized shrinkage porosity B. Back pressure porosity
C. Subsurface porosity D. Suck back porosity
40. Coefficient of thermal expansion of amalgam is
A. 6.6 a (ppm k–1) B. 11.4 a (ppm k–1)
–1
C. 14.0 a (ppm k ) D. 25.0 a (ppm k–1)
41. Class II stone, densite, or improved stone is
A. Type III B. Type IV
C. Type V D. None of the above
42. Eames technique for proportioning alloy and mercury is
A. 1:1 by volume B. 8:5 by weight
C. 5:8 by volume D. 1:1 by weight
43. What is the minimum thickness of the elastomeric impression material for an accurate impression?
A. 2 mm B. 2–4 mm
C. Greater than 3 mm D. Maximum in the area of operation
44. Which of the following increases the viscosity and rigidity of agar impression material?
A. Borax B. Potassium sulfate
C. Diatomaceous earth D. Glycerine
45. The most common cause of chalky surface of plaster cast is
A. A delay of more than 20 minutes in pouring the cast B. Incorporation of air during mixing
C. High speed accelerator D. All of the above
46. The zone of the flame used for maximum heating of the alloy is
A. Oxidizing zone B. Reducing zone
C. Combustion zone D. Thermolytic zone
47. The type of gold recommended in occlusal load bearing areas is
A. Annealed gold B. Gold foil
C. Mat gold D. Powdered gold
48. Gamma 2 phase of amalgam causes
A. Delayed expansion B. Tarnish and corrosion
C. Decreases the strength of restoration D. All of the above

https://t.me/DentalBooksWorld
Chapter 11 • Dental Materials 1041

49. Delayed expansion in dental amalgam is due to


A. Copper B. Zinc
C. Silver D. Tin
50. Which of the following is true of impression plaster?
A. It is an alpha hemihydrate
B. It is manufactured by calcination of gypsum under steam heat
C. It is more mucostatic than alginate
D. Borax is used to control setting expansion
51. Maximum hardness in traditional high gold types III and IV casting alloys is achieved in 1–4 hours after isothermal
annealing at elevated temperatures of
A. 130–250°C B. 100–230°C
C. 230–350°C D. 330–450°C
52. Nanoleakage due to resin-hybrid layer is about
A. 0.1 B. <0.1
C. 0.2 D. <0.2
53. The impression material to make impression of teeth and soft tissues with reversible setting mechanism is
A. Alginate hydrocolloid B. Polysulfide
C. Agar hydrocolloid D. Polyether
54. Catalyst present in the elastomeric impression material is
A. Polysulfide polymer B. Titanium dioxide
C. Lead dioxide D. Lithopone
55. At the end of 24 hours, the contraction of impression material for type I and III materials is
A. 0.5% B. 0.05%
C. 0.1% D. 0.01%
56. In agar hydrocolloids, the component used to strengthen the gel is
A. Potassium sulfate B. Thymol
C. Glycerine D. Borax
57. Reactor used in alginate hydrocolloid is
A. Trisodium phosphate B. Calcium sulfate dihydrate
C. Zinc oxide D. Potassium titanium fluoride
58. Which of the following materials is not used for recording of occlusal relationship of natural/artificial teeth?
A. Agar hydrocolloid B. Alginate hydrocolloid
C. Wax D. Impression plaster
59. Dental investment is the plaster mixed with fillers like
A. Silica B. Ceramic
C. Glass D. Chalk powder
60. Gum dammer or dammer resin is a natural resin added to paraffin to improve its smoothness in molding and will
render it more
A. Toughness B. Resistant
C. Smoothness D. Flow
61. Type of investment material used for casting of removable partial denture with base metal alloy is
A. Gypsum-based material B. Phosphate-based material
C. Ethyl silicate bonded D. All of the above
62. Hygroscopic expansion of investment material increases with the increase in
A. Silica content B. Water–powder ratio
C. Particle size D. Shelf life
63. The powder of polymethylmethacrylate consists of polymethylmethacrylate and a small amount of initiator which is
A. Benzyl peroxide B. Hydroquinone
C. Glycol peroxide D. Peroxide

https://t.me/DentalBooksWorld
1042 Triumph's Complete Review of Dentistry

64. When using a polysulfide rubber impression material, greatest accuracy in the cast is produced by
A. Allowing the material to bench cure for 20 minutes B. Adding a catalyst to the impression material
C. Placing the wet cotton in the tray D. Pouring immediately
65. The main purpose of boxing the impression is
A. Economy of material B. To preserve the border width
C. To avoid breakage of impression while pouring D. To give definite shape to the base
66. Which die material is the material of choice with hydrochloride?
A. Silicophosphate B. Silver amalgam
C. Improved stone D. Epoxy resin
67. Corrosion and fractured margins of amalgam fillings may be resulted due to
A. Under trituration of amalgam B. Excessive residual mercury
C. Saliva contamination D. All of the above
68. Which of the following types is used as die type of gypsum products?
A. Type I B. Type II
C. Type III D. Type IV
69. Which of the impression materials undergoes chelation reaction during setting?
A. Alginate B. ZnO
C. Agar agar D. Polysulfide
70. A refractory cast is made up of
A. Die stone B. Investment
C. Plaster D. Stone
71. Which of the following hardness tests uses a 136° diamond tool to test the hardness of a material?
A. Knoop hardness test B. Vickers hardness test
C. Brinnell’s hardness test D. Rockwell hardness test
72. Liquid component of heat cured acrylic has all of the following except
A. Hydroquinone B. Methyl methacrylate
C. Dimethyl-p-toluidine D. Ethyl glycol dimethacrylate
73. Which of the following is the correct glass transition temperature of the impression compound?
A. 43.5°C B. 39°C
C. 65°C D. 100°C

RECENT PREVIOUS YEAR QUESTIONS


1. The silver amalgam alloy which needs least amount of mercury is
A. Lathe cut B. Spherical
C. Unicompositional D. Admixed
2. Which of the following is a solid solution?
A. Cobalt chromium B. Amalgam
C. Silver gallium D. Silver palladium
3. In synthetic resin dentistry, the role of plasticizer is
A. To improve the flow B. To add bulk
C. To reduce softening or fusion temperature D. To prevent polymerization shrinkage
4. Which of the following is not associated with corrosion?
A. Carbonates B. Sulfides
C. Oxides D. Chlorides
5. Retarder used in heat cure acrylic monomer resin to prevent polymerization
A. Benzoyl peroxide B. Polyacrylic acid
C. Hydroquinone D. Dibutyl phthalate
6. Condensation reaction occurs in
A. Agar B. Alginate
C. ZOE D. Polysulfide

https://t.me/DentalBooksWorld
Chapter 11 • Dental Materials 1043

7. The type of strain developed when the force is applied perpendicular to the surface
A. Compressive B. Tensile
C. Shear D. Flexure
8. Time required by alginate for gelation at room temperature is
A. 1–2 minutes B. 3–4 minutes
C. 6–8 minutes D. 12–16 minutes
9. Percentage of carbon in carbon steel hand cutting instruments is
A. 1–1.2% A. 10–12%
B. 0.6–0.8% C. 6–8%
10. Prolonged spatulation of gypsum results in
A. Reduced setting time B. Increased compressive strength
C. Increased hygroscopic expansion D. Increased hardness
11. For a bonding agent to be effective, the wetting angle should be
A. Minimum with hard tissue B. Maximum with both dentin and enamel
C. Minimum with dentin and maximum with enamel D. Minimum with enamel and maximum with dentin
12. In dental materials, the setting time is measured by which test?
A. Vicat needle test B. Vicker’s test
C. Brinell test D. Cold bend test
13. Water:powder ratio for type IV dental stone is
A. 0.65–0.70 B. 0.45–0.50
C. 0.22–0.24 D. 0.28–0.30
14. In mechanical trituration of amalgam, what is adversely affected?
A. Hardness of the filling B. Tarnish and corrosion resistance
C. Working time D. Final gloss of the filling
15. Alginate fillers derived from
A. Potassium alginate B. Calcium sulfate
C. Diatomaceous earth D. Sodium phosphate
16. In an addition silicone impression, which of the following should be done in order to get the best results of the cast?
A. Delay pouring of cast B. Add flavoring agent to prevent bad odor
C. Apply ketone on the base D. Apply chloroform on the base
17. Molecule with permanent dipole
A. Water B. Liquid nitrogen
C. Oxygen D. Helium
18. Amount of heat that is required to change boiling water into vapors is referred to as
A. Latent heat of fusion B. Latent heat of vaporization
C. Latent heat of sublimation D. Melting temperature
19. Metal oxides used in porcelain
A. To improve strength B. To improve bonding with porcelain
C. Impart color D. All of these
20. Which of the following is not an antiflux?
A. Graphite B. Boric acid
C. Iron oxide D. Calcium carbonate in alcohol
21. Lithium disilicate containing crystals ceramic crown is
A. Captek B. In ceram
C. IPS EMPRESS D. IPS EMPRESS 2
22. Shrinkfree ceramic is known as
A. Cerestore B. Dicor
C. IPS in-ceram D. Captek
23. Most common reason for incomplete casting when using a centrifugal casting machine is
A. Narrow sprue B. Increased porosity of the investment
C. Large reservoir D. Hollow sprue

https://t.me/DentalBooksWorld
1044 Triumph's Complete Review of Dentistry

24. Most water soluble cement is


A. Zoe B. Zinc phosphate
C. Zinc polycarboxylate D. Silicophosphate
25. Residual monomer content in heat cure resin is
A. 0.3–0.5% B. 0.5–1%
C. 3–5% D. 1–2%
26. Residual monomer content in chemically cure resin is
A. 0.3–0.5% B. 0.5–1%
C. 3–5% D. 1–2%
27. Chromium when added to steel does not increase
A. Corrosion resistance B. Increase hardness
C. Decrease elastic limit D. Increase tensile strength
28. Zinc phosphate cement pH at the time of mixing
A. 1–2 B. 2.5–4.5
C. 6–7 D. 1.5–3.5
29. Which of the following is not a function of using boric oxide in ceramic restorations?
A. It acts as glass modifier
B. It decreases the viscosity of ceramic restoration
C. It removes the unsintered matrix from the ceramic residue
D. It prevent the unreacted particles from being oxidizing in atmosphere
30. Which is not true about ethyl silicate investment material?
A. MgO increases strength
B. Aqueous solution of colloidal silica forms gel after heating
C. Gel silica formed by heating
D. Most user friendly material
31. Which is false regarding ethyl silicate bonded investment?
A. Silica gel which reverts to silica on heating
B. Magnesium oxide provides strength
C. It is easy to handle and most commonly used
D. On mixing ammonium chloride silica liquid converts to gel
32. The monophase elastomeric material can be used as both tray and syringe materials because it has
A. Long setting time B. It is economical
C. Pseudo plastic properties D. Better recording of tissues
33. Major drawback of porcelain is
A. High compressive strength B. Brittleness
C. Translucency D. Hardness
34. The best bio-compatible dental casting alloy is
A. Co–Cr B. Ni–Cr
C. Titanium alloy D. Gold
35. The unalloyed form of titanium used to form dental implants is in the
A. Beta form B. Alpha form
C. Gamma form D. Delta form
36. Which color of articulating paper is used for diagnosis of premature contacts in eccentric relations?
A. Green B. Blue
C. Red D. Pink
37. An example of rigid reversible impression material is
A. Agar-agar B. Impression compound
C. Alginate D. Impression paste
38. The initiator in the heat cure polymer powder is
A. Hydroquinone B. Sulfinic acid
C. Benzoyl peroxide D. Sulfur dioxide

https://t.me/DentalBooksWorld
Chapter 11 • Dental Materials 1045

39. Impression material which has a high incidence of air porosity is


A. Alginate impression material B. Impression compound
C. Zinc oxide eugenol impression paste D. Polyether impression material
40. Conventional dental amalgam alloy contains
A. Silver, tin, copper, and zinc B. Silver, mercury, copper, and zinc
C. Silver, tin, palladium, and zinc D. Silver, copper, iridium, and mercury
41. The elastomer having the longest curing time is
A. Additional silicone B. Condensation silicone
C. Polyether D. Polysulfide
42. Discoloration of Ag containing alloy is due to
A. Wet corrosion B. Dry corrosion
C. Tarnish D. Both tarnish and corrosion

ANSWERS

ADA SPECIFICATION
1. Answer: A
2. Answer: A
3. Answer: C
4. Answer: A
5. Answer: A
6. Answer: A
7. Answer: A
8. Answer: A
9. Answer: C
10. Answer: A
11. Answer: A
12. Answer: A
13. Answer: A
14. Answer: C
15. Answer: C
16. Answer: A
17. Answer: A
18. Answer: B
19. Answer: B
20. Answer: C
21. Answer: C
22. Answer: A
23. Answer: A
24. Answer: C
25. Answer: A

PROPERTIES OF DENTAL MATERIALS


1. Answer: B (Ref. Basic Dental Materials, By John J. Manappallil, 2015, page no. 14)
• The contact angle is the angle formed by the adhesive with the adherend at their interface. The less the contact angle,
the more the wettability and spreadability.
• If the contact angle is <90°, the result is rise of liquid.
• If the contact angle is >90°, the result is depression of liquid.
• The contact angle between saliva and the acrylic denture base is 75°.
• The contact angle between glass and mercury is 135°.

https://t.me/DentalBooksWorld
1046 Triumph's Complete Review of Dentistry

2. Answer: A (Ref. Phillip’s Science of Dental Materials, By Kenneth J. Anusavice, Chiayi Shen, H. Ralph Rawls, 2014, page no. 54)
Elastic modulus (also modulus of elasticity and Young’s modulus)—Stiffness of a material that is calculated as the ratio
of elastic stress to elastic strain
Flexural strength (bending strength or modulus of rupture)—Force per unit area at the instant of fracture in a test
specimen subjected to flexural loading
Ductility—Relative ability of a material to elongate plastically under a tensile stress. This property is reported quantitatively
as percent elongation
Malleability—Ability to be hammered or compressed plastically into thin sheets without fracture
3. Answer: C (Ref. Phillip’s Science of Dental Materials, By Kenneth J. Anusavice, Chiayi Shen, H. Ralph Rawls, 2014,
page no. 64)

Enamel 340
Amalgam 90
Composite 55
Pure gold 75
Porcelain 412

4. Answer: A (Ref. Phillip’s Science of Dental Materials, By Kenneth J. Anusavice, Chiayi Shen, H. Ralph Rawls, 2014,
page no. 18)
Heat of vaporization—Thermal energy required to convert a solid to a vapor
Latent heat of fusion—Thermal energy required to convert a solid to a liquid. The temperature at which this change
occurs is known as the melting temperature or fusion temperature.
When water boils, energy is needed to transform the liquid to vapor, and this quantity of energy is known as the heat of
vaporization.
It is possible for some solids to change directly to a vapor by a process called sublimation (as seen in dry ice).
5. Answer: B (Ref. Phillip's Science of Dental Materials, By Kenneth J. Anusavice, Chiayi Shen, H. Ralph Rawls, 2014, page
no. 31).

Porcelain 6.6
Dentin 8.3
Enamel 11.4
Silicate 10
Type II GIC 11
Amalgam 25
Denture resins 81
Composites 14–50

6. Answer: A (Ref. Craig’s Restorative Dental Materials, By Ronald L. Sakaguchi, John M. Powers, 2012, page no. 45)
Most dental materials display pseudoplastic behavior. Increasing the shear stress helps the material to flow. An excellent
example is the cementation of a crown onto a prepared tooth.
Ideal behavior is represented by the Newtonian curve that starts at zero shear stress and zero shear strain rate. However, it
is possible for liquids to resist flow until a certain critical shear stress is exceeded. This is called Bingham Body behavior.
There are many advantages for Bingham Body behavior in dentistry. Toothpaste – It only flows out of the tube when we
press hard enough. Dental sealant – When it is painted onto an occlusal surface, it does not run off before it is cured.
(What type of viscosity behavior includes a critical shear stress? (Bingham Body)
7. Answer: A
Creep is defined as time-dependent plastic deformation.
8. Answer: B
9. Answer: A
10. Answer: C

https://t.me/DentalBooksWorld
Chapter 11 • Dental Materials 1047

AMALGAM
1. Answer: C (Ref. Basic Dental Materials, By John J. Manappallil, 2015, 264)
• When an agar impression slightly contracts and exude water, this is termed syneresis.
• Hysteresis is melting and gelling at different temperatures.
• Imbibition is when the hydrocolloid absorbs water, swell, and distort.
2. Answer: A
3. Answer: C (Ref. Craig’s Restorative Dental Materials, By Ronald L. Sakaguchi, John M. Powers, 2012, page no. 293)
Mixing time – The amount of time the auxiliary has to bring the components
Working time – The time permitted to manipulate the material in the mouth
Setting time – The time from the start of mixing till just before the elastic properties have fully developed
Initial set time – The time that begins when the material can no longer be manipulated in the mouth
Final set time – The time when the material has reached its ultimate state
4. Answer: B (Ref. Phillip’s Science of Dental Materials by Kenneth J. Anusavice, Chiayi Shen, H. Ralph Rawls, 2014, page no. 175)
Factors that control setting time:
a. Factors controlled by the operator:
1. W:P ratio
• The more the W:P ratio, the fewer the nuclei per unit volume, so prolonged setting time
2. Mixing time
• Within practical limits, longer and rapid mixing leads to shorter setting time.
• Some gypsum crystals form immediately when the plaster comes in contact with water and as the mixing begins,
formation of crystals increases.
• Some crystals are also broken up by mixing spatula and are distributed resulting in the formation of more nuclei of
crystallization resulting in decreased setting time.
• Effect of W:P ratio and mixing time on the setting time of plaster of Paris.
b. Factors controlled by the manufacturer:
1. By the addition of accelerators and retarders:
• Accelerators:
• Gypsum (<20%) – Decreases setting time
• The set gypsum used as an accelerator is called “Terra Alba”
• Potassium sulfate (conc. 2–3%) reduces the setting time of model plaster from approximately 10 to 4 minutes
• Sodium chloride (<28%)
• Retarders:
• Organic materials – Glue, gelatin, and some gums
• Potassium citrate, borax, sodium chloride (20%), sodium citrate

Characteristics features Orders


Stiffness (elastic modulus) PS<CS<AS<PE
Thermal contraction CS=AS<PS<PE
Setting shrinkage PE=AS<PS<CS
Reproduction details PS<PE<CS<AS
Tear strength PS>PE>AS>CS
Biocompatibility PS>AS>CS>PE
Dimensional stability AS>PE>PS>CS
Permanent deformation following PS>PE>CS>AS
strain increases in

(Ref. Phillip’s Science of Dental Materials, By Kenneth J. Anusavice, Chiayi Shen, H. Ralph Rawls, 2014, page no. 161)

https://t.me/DentalBooksWorld
1048 Triumph's Complete Review of Dentistry

5. Answer: A (Ref. Craig’s Restorative Dental Materials, By Ronald L. Sakaguchi, John M. Powers, 2012, page no. 394)
Glass transition temperature – The temperature below which a material is hard and brittle, and above which it is soft is
called the glass transition temperature.
Glass transition of impression compound is 43.5°C
Fusion temperature of impression compound is 39°C
6. Answer: D (Ref. Phillip’s Science of Dental Materials, By Kenneth J. Anusavice, Chiayi Shen, H. Ralph Rawls)
Advantages and Disadvantages of Elastomeric Impression Materials

Advantages Disadvantages
Polysulfides
Good wetting ability High permanent deformation
Good surface detail Unpleasant taste and odor
Easy to remove Must pour within 1 hour
High tear strength Low tear strength
Condensation silicones
Good surface detail (dry surfaces) Hydrophobic
Good dimensional accuracy Shrinks on storage
Low permanent deformation Must pour within 1 hour
Easy to disinfect Low tear strength
High range of viscosities
Polyethers
Hydrophilic High permanent deformation
Good surface detail Swells in disinfectants or moist environment
Good dimensional accuracy Difficult to remove
Good resistance to deformation Low tear strength
Highly acceptable to patients Care needed when disinfecting
Addition silicones
Good surface detail (dry surfaces) Hydrophobic (unless surfactant added)
Good dimensional accuracy Low tear strength
Good storage stability
Low permanent deformation
Easy to disinfect
Highly acceptable to patients

7. Answer: C (Ref. Phillip’s Science of Dental Materials, By Kenneth J. Anusavice, Chiayi Shen, H. Ralph Rawls, 2014, page
no. 170)
Mucostatic impression materials
• Impression plaster
• Agar agar impression material
• Zinc oxide eugenol impression pastes
• Light body elastomers
Mucocompressive impression materials
• Impression compound
• Viscous alginate
• Heavy and putty consistencies of elastomers
8. Answer: C (Ref. Craig’s Restorative Dental Materials by Ronald L. Sakaguchi, John M. Powers, 2012, page no. 204) High-
copper (new-generation, gamma-2 free) amalgam

https://t.me/DentalBooksWorld
Chapter 11 • Dental Materials 1049

Composition
• Silver: 40–70%
• Tin: 12–30%
• Copper: 12–30%
• Indium: 0–4%Zinc: 0–1%Palladium: 0.5%
High-copper amalgam was developed in 1962 by the addition of silver–copper eutectic particles to traditional silver–tin
lathe-cut particles in an attempt to dispersion strengthen or dispersion harden the alloy.
It is important to note that high-copper alloys must contain at least 12% copper to eliminate the gamma-2 phase. Compared
to their low-copper amalgam counterparts, high-copper alloys exhibit the following physical properties: greater strength,
less tarnish and corrosion, and less creep. Overall, they are also less sensitive to handling variables and produce better
long-term clinical results.
9. Answer: C (Ref. Phillip’s Science of Dental Materials by Kenneth J. Anusavice, Chiayi Shen, H. Ralph Rawls, 2014, page no. 161)

Characteristics features Orders


Stiffness (elastic modulus) PS<CS<AS<PE
hermalcontraction CS=AS<PS<PE
Setting shrinkage PE=AS<PS<CS
Reproduction details PS<PE<CS<AS
Tear strength PS>PE>AS>CS
Biocompatibility PS>AS>CS>PE
Dimensional stability AS>PE>PS>CS
Permanent deformation following strain PS>PE>CS>AS
increases in

10. Answer: D (Ref. Phillip’s Science of Dental Materials, By Kenneth J. Anusavice, Chiayi Shen, H. Ralph Rawls, 2014, page
no. 175)

Failures Causes
Rough or uneven surfaces on impression Premature removal from mouth
Improper ratio
Presence of oil/debris
Increase in temperature
Bubbles Too rapid polymerization
Irregularly spaced voids Debris on teeth
Rough/chalky stone cast Increased water and wetting agent
left over
Inadequate cleaning
Failure to delay pour of addition
silicone at least 20 minutes

11. Answer: C (Ref. Craig’s Restorative Dental Materials by Ronald L. Sakaguchi, John M. Powers, 2012, page no. 204)
Condensation polymers are any kind of polymers formed through a condensation reaction—where molecules join
together—losing small molecules as byproducts such as water or methanol, as opposed to addition polymers which
involve the reaction of unsaturated monomers.
Copolymers
When two or more different types of monomers join together, the polymer formed from them is called a copolymer.
Copolymers are produced to enhance the physical and mechanical properties of the material. They are used in dentures
to make them more resistant to fracture, in soft reline materials to make them soft and pliable, and in mouth guards to
improve their shock-absorbing capacity.

https://t.me/DentalBooksWorld
1050 Triumph's Complete Review of Dentistry

Polymerization
The act of forming polymers is called polymerization. In general, less than 100% of the monomer is used up. The
remaining unused monomer is called the residual monomer. The best clinical results occur when there is little residual
monomer.
Cross-linked polymers
The polymer chains often have short chains of atoms attached to their sides. When the side chains of adjacent polymers
bond together, the polymers are termed cross-linked polymers.
When side chains of adjacent polymers are joined by weak bonds, the polymers are easily manipulated, bent, or stretched.
When adjacent polymers are joined by highly charged side chains, the bond is stronger, and the cross-linked polymers are
stronger and stiffer. They are also more wear resistant and, consequently, can be used in denture teeth. They polish more
easily and are less affected by solvents such as alcohol.
Polymerization reactions
There are two types of polymerization:
• Addition polymerization
• Condensation polymerization
The reactions are the same as for the impression polymers, addition silicones, and condensation silicones.
Addition polymerization
Addition polymerization is the most common form of polymerization for dental materials. It occurs in three stages:
Stage 1: Initiation (or induction)
Stage 2: Propagation
Stage 3: Termination
Unlike condensation polymerization, the reaction does not produce any byproducts. Monomers have a core unit of two
carbon atoms joined by a double bond. One carbon atom has two hydrogen atoms attached, and the other carbon atom
has attached to it one hydrogen atom and one reactive group called a free radical. The free radical is made reactive by the
chemical reaction of organic peroxides, such as benzoyl peroxide, with an activator or accelerator, such as a tertiary amine,
or by heating.
Initiation
The free radical initiates the reaction by opening the bond between the two carbon atoms of the monomer. The broken
carbon bond causes the monomer molecule to bond to another monomer. Each linkage leaves a free radical available for
further reaction.
Propagation
The process of linking monomer units is termed propagation, and it continues until the monomer units are used up, or
until a substance reacts with the free radical to tie it up.
Termination
When the free radical is tied up or destroyed, the process is terminated.
Curing methods
The materials that react by chemical means are called chemical-curing, self-curing, or autopolymerizing. Materials that use
heat to initiate the reaction are called heat-curing polymers. Materials in which the reaction is activated by light are called
photo- or light-cured materials. Whether initiated by chemical means, light, or heat, the polymerization process releases
heat (i.e., it is an exothermic reaction). The heat must be controlled during the process. If the temperature becomes too
great, the monomer will vaporize and produce porosity in the material. Porosity weakens the material, causes it to discolor
as stains are absorbed into the pores, and can lead to retention and growth of oral microorganisms and development of an
unpleasant odor (“denture breath”).
Condensation polymerization
Materials formed by a condensation reaction do not have many uses in dentistry. The condensation silicone impression
materials are the most commonly known, and even they are not used much today. Typically, more than one type of
monomer is used. The reaction itself produces byproducts such as water, hydrogen gas, or alcohol that may compromise
the physical properties or handling characteristics
12. Answer: C
13. Answer: D
14. Answer: C (Ref. Basic Dental Materials, By John J. Manappallil, 2015)

https://t.me/DentalBooksWorld
Chapter 11 • Dental Materials 1051

Based on setting mechanism Inelastic/rigid materials Elastic materials


Irreversible/chemical reaction Plaster of Paris and zinc oxide eugenol Alginate
(cannot be used in undercuts) Elastomeric impression materials
Reversible/physical reaction Impression compound Agar (used to reproduce
undercuts/interproximal spaces)

15. Answer: D
16. Answer: C
17. Answer: A
18. Answer: C (Ref. Dental Materials: Properties and Manipulation, By John M. Powers, John C. Wataha, 2015, page no. 76)
• Amalgam polishing is done 24 hours after the restoration is complete.
• Fine grades of pumice are used for polishing amalgam restorations. It is usually mixed with water (slurry of pumice)
to help reduce the heat created by the friction of the abrasive particles during polishing. Tin oxide or amalgloss is used
as the finest abrasive agent.
• An exception to the normal protocol is high-copper amalgams with high early strength. Restorations of these amalgams
may be polished 8–10 minutes after the start of trituration, to avoid the need for the patient to attend a second
appointment.
19. Answer: D
Zinc acts as a deoxidizing agent or a scavenger.
20. Answer: A
21. Answer: D
22. Answer: C (Ref. Basic Dental Materials, By John J. Manappallil, 2015, page no. 382)

Material Percentage (approximate) Purpose


Sodium or potassium alginate 15–20% Colloidal particles as basis of the gel
Calcium sulfate dihydrate 14–20% Creates irreversible gel with alginate
Potassium sulfate 10% Ensures set of gypsum materials
Trisodium phosphate 2% Retarder to control setting time
Diatomaceous earth 55–60% Filler to increase thickness and strength
Other additives: Very small quantities  
• Organic glycols Reduce dust when powder is handled
• Flavoring agents Improve taste of material
• Coloring agents Provide pleasant colors
• Disinfectants Cause antibacterial action

23. Answer: C
24. Answer: C
25. Answer: A
26. Answer: B
27. Answer: A (Ref. Basic Dental Materials, By John J. Manappallil, 2015, page no. 263)

Material Percentage (approximate) Purpose


Agar 12–15% Colloidal particles as basis of the gel
Potassium sulfate 1% Ensures set of gypsum materials
Borax 0.2% Strengthens gel
Alkyl benzoate 0.1% Antifungal agent
Water 85% Dispersing medium for the colloidal suspension

https://t.me/DentalBooksWorld
1052 Triumph's Complete Review of Dentistry

28. Answer: C
29. Answer: A
30. Answer: A (Ref. Sturdevant’s Arts and Science of Operative Dentistry, By Andre V. Ritter)
First amalgam war – Initiated in 1841
Second amalgam war – Alfred Stock in 1920
Third amalgam war – H. A.Huggins
31. Answer: B (Ref. Phillip’s Science of Dental Materials, By Kenneth J. Anusavice, Chiayi Shen, H. Ralph Rawls, 2014, page
no. 539)
Classification of amalgam
Based on Cu content
• Conventional or low-copper alloy
• High-copper alloy
–– Low-Cu alloys (less than 6% copper)
–– High-Cu alloys (more than 6% copper)
Based on Zn content
• Zn containing (more than 0.1% zinc)
• Zn free (less than 0.1% zinc)
Based on the shape of alloy particles
• Lathe-cut alloys
–– Regular-cut
–– Fine-cut
–– Micro-fine cut
• Spherical alloys 
• Spheroidal alloys
32. Answer: A (Ref. Basic Dental Materials, By John J. Manappallil, 2015, page no. 139)

Zinc Indium Palladium Mercury


• Increases strength • Increases strength • Increases strength • Decreases setting time
• Increases expansion • Increases expansion • Increases corrosion resistance • Decreases delayed expansion
• Increases flow • Increases flow
• Increases setting time • Increases setting time
• Decreases corrosion • Amalgamation more
resistance difficult
• Increases plasticity
• Decreases brittleness

33. Answer: A
34. Answer: A (Ref. Basic Dental Materials, By John J. Manappallil, 2015)

Mercury Hg Most weakest phase (Ref. Marzouk)


Gamma Ag3 Sn Formed by peritectic reaction
Gamma 1 Ag2 Hg
Gamma 2 Sn8 Hg Weakest phase
Epsilon Cu3 Sn Occurs in high-copper single composition
amalgam only
ETA Cu6 Sn5

35. Answer: A Option B is for unicompositional alloys.

https://t.me/DentalBooksWorld
Chapter 11 • Dental Materials 1053

36. Answer: D (Ref. Basic Dental Materials, By John J. Manappallil, 2015)


Factors that lead to contraction and expansion

Less mercury in the mix Postoperative sensitivity


Decreased mercury alloy ratio Protrusion of restoration
Increased condensation pressure Pressure on pulp (it causes
excessive expansion)
Small particle size Increased mercury in the mix
Longer trituration
Micro leakage
Plaque accumulation and
secondary caries

37. Answer: B (Ref. Phillip’s Science of Dental Materials, By Kenneth J. Anusavice, Chiayi Shen, H. Ralph Rawls, 2014)

Amalgam Compressive Tensile Creep value


strength at 1 hour strength
Low copper 145 60 2% (0.8–8%)
Admix 137 (least) 48 0.4%
Single composition 262 (it doubles) 64 0.13%

38. Answer: C (Ref. Phillip’s Science of Dental Materials, By Kenneth J. Anusavice, Chiayi Shen, H. Ralph Rawls, 2014, page no. 33)
Recommended creep value is less than 3%

Low copper 2%
Admixed 0.4%
Unicompositional 0.13%

39. Answer: B (Ref. Phillip’s Science of Dental Materials, By Kenneth J. Anusavice, Chiayi Shen, H. Ralph Rawls, 2014)
• Amalgam has a linear coefficient of thermal expansion that is 2.5 times greater than the tooth structure, and it does
not bond to the tooth structure. During expansion and contraction, percolation occurs along the external walls. The
formation of corrosion products prevents the fluid ingress and egress along the margins and contributes to the self-
sealing ability of amalgam.
• Electrochemical corrosion is not a mechanism of mercury liberation from set amalgam.
• Mercury immediately reacts with locally available silver and tin from amalgam alloy particles and is reconsumed to
form more corrosion reaction products.
40. Answer: A (Ref. Phillip’s Science of Dental Materials, By Kenneth J. Anusavice, Chiayi Shen, H. Ralph Rawls, 2014)
• At the end of 20 minutes, compressive strength is 6% and minimum compressive strength should be 80 Mpa at the end
of 1 hour.

DENTAL CEMENTS
1. Answer: B (Ref. Basic Dental Materials, By John J. Manappallil, 2015, page no. 185)
Maximum water solubility is seen in microfilled composite followed by hybrid composites.
2. Answer: B (Ref. Phillip’s Science of Dental Materials, By Chiayi Shen, H. Ralph Rawls, 2013, page no. 331)
• The freshly mixed zinc phosphate is highly acidic, with a pH between 1 and 2 after mixing, and, even after setting
1 hour, the pH may still be below 4. After 24 hours, the pH is usually between 6 and 7.
• Pain on cementation is due not only to the free acidity of the mix but also to osmotic movement of fluid through the
dentinal tubule.

https://t.me/DentalBooksWorld
1054 Triumph's Complete Review of Dentistry

• Hydraulic pressure developed during seating of the restoration may also contribute to pulpal damage.
• Prolonged pulpal irritation, especially in deep cavities that necessitate some form of pulpal protection, may be associated
with prolongation of the low pH.
• Irritation is minimized by a high powder/liquid ratio and rapid setting.
• A material that has a low acid content and incorporates calcium hydroxide has little effect on pulp when used as a liner.
• Very thin mixes will also lead to etching of the enamel.
3. Answer: C (Ref. Phillip’s Science of Dental Materials, By Chiayi Shen, H. Ralph Rawls, 2014, page no. 316)
• Chemical bonding is seen in GIC and Zinc Polycarboxylate
• Ionic bonding is seen in Zinc Phosphate cement
• Chemical bonding is not seen in Zinc Phosphate cement
4. Answer: A (Ref. Dental Materials Properties and Manipulation, By John M. Powers, John C. Wataha, 2015, page no. 92)
• Calcium hydroxide and composite are radiolucent cements. Nowadays newer products are made radiopaque though.
5. Answer: B (Ref. Phillip's Science of Dental Materials, By Chiayi Shen, H. Ralph Rawls, 2014, page no. 5)
• Metallic restorations (amalgam and gold) – Absorbs X-rays, and very little (if any) radiation comes in contact with the
film. It appears radiopaque on a dental radiograph.
• Nonmetallic restorations (porcelain, composite and acrylic) – May vary from radiolucent to slightly radiopaque,
depending of the density of the material. Porcelain is the most dense and least radiolucent, acrylic is least dense and
most radiolucent.
• Amalgam restorations – Most common, absorbs X-ray beams and appear completely radiopaque. May be seen in a
variety of shapes, sizes, and locations on a dental radiograph.
• One-surface amalgam (pit amalgam) – Appear as a distinct, small round of ovoid radiopacities. May be seen on buccal,
lingual, or occlusal surfaces of the teeth.
• Two-surface and multisurface amalgam – Appear radiopaque and are characterized by their irregular outlines or
borders. May involve any tooth surface.
• Gold restorations – Gold restorations appear completely radiopaque and unlike amalgam restorations, they exhibit a
smooth marginal outline.
• Gold crown and bridges – Appear as a large radiopaque restoration with smooth contour and regular borders.
• Gold inlays and onlays – Exhibit marginal outlines that appear smooth and regular
• Stainless steel and chrome crowns – Usually used as temporary restorations. They are not contoured properly to the
cervical portion of the tooth and do not fit tooth well. They are thin and do not absorb dental X-rays to the extent that
amalgam, gold, and other cast metals do. Both appear radiopaque but not as densely as amalgam or gold. Some areas
may appear “see-through.”
• Post and core restorations – Can be seen in endodontically treated teeth. Cast metal that appears radio dense as
amalgam or gold. It appears radiopaque. The core portion of the restoration resembles the prepped portion of a tooth
crown, and the post portion extends into the pulp canal.
• Porcelain restorations – Unlike metallic restorations which are totally radiopaque, porcelain are slightly radiopaque and
resemble the radiodensity of dentin.
• All-porcelain crowns – Appears slightly radiopaque. A thin radiopaque line that outlines the prepared tooth may be
evident. This thin line represents cement. The radiodensity appears identical to an all-porcelain bridge.
• Porcelain-fused-to-metal crown – Metal component appears completely radiopaque and the porcelain component
appears slightly radiopaque. Radiodensities appear identical to porcelain-fused-to-metal bridge.
• Composite restorations – Varies in radiographic appearance from radiolucent to slightly radiopaque, depending on the
composition of the composite material. Some manufacturers add radiopaque particles to their products in order to help
the viewer differentiate a composite restoration from dental caries.
• Acrylic restorations – Often used as an interim or temporary crown or filling. Of all nonmetallic restorations, acrylic is
the least dense and appears radiolucent or barely visible on a dental radiograph.
• Base materials – Zinc phosphate cement and zinc oxide-eugenol paste, are used as cavity liners to protect the pulp
of the tooth. Base materials are placed on the floor of a cavity preparation, it appears radiopaque. (If compared with
amalgam, base material appears less radio dense.)
• Metallic pins – Used to enhance retention of amalgam or composite, appears as cylindrical or screw-shaped radiopacities
on X-ray. (Holds amalgams in place.)

https://t.me/DentalBooksWorld
Chapter 11 • Dental Materials 1055

• Gutta percha – Clay-like material used in endodontic therapy to fill the pulp canals. Appears radiopaque, similar in
density to that of base materials. (Less radio dense when compared with metallic restorations.)
• Silver points – Are used with root canals to fill the canals. They are metal and appear much like metal restorations.
Appears radiopaque, but appears more radio dense than gutta percha.
6. Answer: A (Ref. Phillip's Science of Dental Materials, By Chiayi Shen, H. Ralph Rawls, 2014, page no. 319)
Invented by Smith in 1968
Main components
• Powder: Similar to that used for zinc phosphate cement, Al2O3, SnF2 are also added to improve its strength, to release
F and improve its manipulation (dried polyacid)
• Liquid: 40–50% aqueous solution of polyacrylic acid or copolymers of acrylic acid with itaconic or maleic acids
(molecular weight approximately 20,000–50,000)
Advantages
• Higher pH than for Zn phosphate cement
• Lower disintegration in the mouth environment
• Very good biological properties
• Adhesion to the tooth tissues
Disadvantages
• Shorter working time, worse manipulation properties too
• High liquid viscosity
• Lower resistance to the mechanical load
• High creep
7. Answer: B This cement features a reinforcing polymer incorporated into the powder.
This gives the cement the strength to resist condensation forces and to ensure adequate life when used as a temporary
filling. By incorporating the reinforcing agent in the powder instead of the liquid, the mixing properties are excellent.
Although the cement is reinforced, it can be easily removed when used in a temporary situation.

Features Benefits
Reinforcing component present Excellent mixing properties
in powder
Radiopaque Shows clearly under X-ray
Long-term temporary restorative Intermediate solution

Product indications
• Temporary filling material
• Intermediate filling material
• Base or lining under restorative materials (amalgam, silicate, silicophosphate, glass ionomer)
• For sealing the coronal portion of teeth undergoing endodontic treatment 
Contraindications
As with other eugenol containing materials, the polymerization of acrylic direct filling materials (acrylic resins and
composite resins) and temporary crown and bridge materials is inhibited and use with these materials must be avoided.
Typical properties

Working Net setting time Compressive Disintegration


time strength
ISO 3107 class I types III and IV – 3–10 minutes 25 MPa minimum <1.50%
Reinforced zinc oxide eugenol 4 minutes 4 minutes 45 seconds 38 MPa 0.50%
cement

Shelf-life
Three years from the date of manufacture when supplied in AHL’s standard packaging.

https://t.me/DentalBooksWorld
1056 Triumph's Complete Review of Dentistry

8. Answer: B (Ref. Basic Dental Materials, By John J. Manappallil, 2015, page no. 85)

Material Coefficient (×106/°C)


Human teeth 8–15
Ceramics 8–14
Glass ionomer base 10–11
Gold alloys 12–15
Dental amalgam 22–28
Composites 25–68
Unfilled acrylics and sealants 70–100
Inlay wax 300–1,000

9. Answer: B (Ref. Essentials of Dental Assisting, By Debbie S. Robinson, Doni L. Bird, 2016, page no. 321)
Cavity varnish is a liner used to seal the dentinal tubules to help prevent microleakage and is placed in a cavity to receive
amalgam alloy after any bases have been placed. Cavity varnish is being used less and less with amalgam restorations, and
dentin bonding agents are replacing cavity varnish as the liner of choice. Cavity varnish has an organic solvent of ether or
chloroform that quickly evaporates, leaving the resin as a thin film over the preparation. This varnish should be slightly
thicker than water. If it becomes very thick, discard it. Cavity varnish is not used with composites since the varnish retards
the set of composites and interferes with the bonding of composites.
The cavity varnish is applied to the pulpal area, walls of the cavity preparation, and onto the edge of the margins of the
preparation. Any excess varnish can be removed from the enamel with a fresh cotton pellet. A second application of cavity
varnish is placed over the first to thoroughly coat the surfaces of the dentin and fill any voids from bubbles created when
the first application dries. After liners and bases are placed into the cavity preparation, the tooth may be restored with
materials, such as amalgam, composite resin, or glass.
Functions of cavity varnish
• Cavity varnish: When placed beneath the fresh restoration it reduces the chances of micro leakage and thus the chances
of having sensitivity are low.
• Another function of cavity varnish is to block the way of irritants from the restorations to the tooth.
• In amalgam restorations the corrosions products that are released by it are also prevented to get penetrate in the tooth
by cavity varnish.
• Fluoride containing cavity varnish also releases some fluoride products that prevent secondary caries to occur.
• Cavity varnishes mainly come in liquid consistency and they are stored in dark colored bottles.
Contents of cavity varnish
The main content is the organic solvent like alcohol or ether, etc. that have some resins like copal resin, natural gum
dissolved in this solvent.
In addition to this there are some medicinal agents like chlorobutanol, thymol, and eugenol present in cavity varnish.
Main properties regarding cavity varnish
Cavity varnish are least soluble and they are also insoluble in distilled water and also the film thickness is very thin and it
ranges between 2 and 400 µm.
Procedure of cavity varnish
Cavity varnish after getting out from the bottle is applied with the help of some brush, cotton pellet, or can be applied with
the help of wire loop. The cavity varnish is to be applied in layers and the way is to apply one layer, let it dry properly and
then apply the next layer. There is total need of about 5–6 layers to complete the procedure and the layer-wise manner is
important to cover up all the voids that appear in the dried layer.
10. Answer: C (Ref. Basic Dental Materials, By John J. Manappallil, 2015, page no. 117)
The conventional glass ionomer systems however suffer from certain disadvantages. These disadvantages are:
• Short working time
• Long set time
• Technique sensitivity
–– Susceptibility to early moisture contamination
–– Prone to desiccation after setting
• Brittleness

https://t.me/DentalBooksWorld
Chapter 11 • Dental Materials 1057

11. Answer: A (Ref. Basic Dental Materials, By John J. Manappallil, 2015, page no. 202)
The American Academy of Pediatric Dentistry’s Pediatric Restorative Dentistry Consensus Conference confirmed support
for sealant use and published these recommendations:
1. Bonded resin sealants, placed by appropriately trained dental personnel, are safe, effective, and underused in
preventing pit and fissure caries on at-risk surfaces. Effectiveness is increased with good technique and appropriate
follow-up and resealing as necessary.
2. Sealant benefit is increased by placement on surfaces judged to be at high risk or surfaces that already exhibit
incipient carious lesions. Placing sealant over minimal-enamel caries has been shown to be effective at inhibiting
lesion progression. As with all dental treatment, appropriate follow-up care is recommended.
3. The best evaluation of risk is made by an experienced clinician using indicators of tooth morphology, clinical
diagnostics, past caries history, past fluoride history, and present oral hygiene.
4. Caries risk, and therefore potential sealant benefit, may exist in any tooth with a pit or fissure, at any age, including
primary teeth of children and permanent teeth of children and adults.
5. Sealant placement methods should include careful cleaning of the pits and fissures without removal of any appreciable
enamel. Some circumstances may indicate use of a minimal-enameloplasty technique.
6. Placement of a low-viscosity, hydrophilic material-bonding layer as part of or under the actual sealant has been
shown to enhance the long-term retention and effectiveness.
7. Glass ionomer materials have been shown to be ineffective as pit and fissure sealants but can be used as transitional
sealants.
8. The profession must be alert to new preventive methods effective against pit and fissure caries. These may include
changes in dental materials or technology.
12. Answer: A
13. Answer: C (Ref. Phillip's Science of Dental Materials, By Chiayi Shen, H. Ralph Rawls, 2014, page no. 320)
Zinc phosphates have been used in clinical practice for many years. Under routine conditions, they can be easily
manipulated, and they are set sharply to a relatively strong mass from a fluid consistency. Although the properties are far
from ideal, they are usually regarded as a standard against which to compare newer cements.
For a given brand, the properties are a function of the powder/liquid ratio. For a given cementing consistency, the
higher the powder/liquid ratio, the better the strength properties and the lower the solubility and free acidity. At room
temperature (21°–23°C)the working time for most brands at luting consistency is 3–6 minutes, and the setting time is 5–14
minutes. Extended working times and shorter setting times can be achieved by use of a cold mixing slab, which permits up
to an approximate 50% increase in the amount of powder, improving both strength and resistance to dissolution.
The cement must have the ability to wet the tooth and restoration, flow into the irregularities on the joining surfaces,
and fill in and seal the gaps between the restoration and the tooth. The minimum value of film thickness is a function of
powder particle size, powder/liquid ratio, and mix viscosity. As measured by ISO and ANSI/ADA specifications, acceptable
cements give film thicknesses of less than 25 µm. In practice, the cement fills in the inaccuracies between the restoration
and the tooth and allows most castings to seat satisfactorily. Unless escape ways or vents are provided with full crowns,
separation of powder and liquid may occur, with marginal defects in the cement film.
At the recommended powder/liquid ratio (2.5–3.5 g/mL), the compressive strength of the set zinc phosphate cement
is 80–110 MPa (11,000–16,000 psi) after 24 hours. The minimum strength for adequate retention of restorations is about
60 MPa (8,500 psi). The strength is strongly and almost linearly dependent on powder/liquid ratio. The tensile strength
is much lower than the compressive strength, 5–7 MPa (700–900 psi), and the cement shows brittle characteristics. The
modulus of elasticity (stiffness) is about 13 GPa (1.8 × 106 psi).
According to the standard method, the solubility and disintegration in distilled water after 23 hours may range from
0.04% to 3.3% for inferior material. The standard limit is 0.2%. The solubility in fluoride-containing cements is about
0.7–1.0% because of the leaching of fluoride. The solubility in organic acid solutions, such as lactic or citric acid, is 20–30
times higher. These data are only a rough guide to solubility under oral conditions. The comparative evaluation of cement
solubility under clinical conditions has shown significant loss but conflicting results. Dissolution contributes to marginal
leakage around restorations and bacterial penetration. This occurrence may be facilitated by dimensional change. The
cement has been found to contract about 0.5% linearly, giving rise to slits at the tooth-cement and cement-restoration
interfaces.
14. Answer: A (Ref Phillip's Science of Dental Materials, By Chiayi Shen, H. Ralph Rawls, 2014, page no. 328)
Compomers are polyacid modified composite resins. These materials derive their name from merging parts of the
descriptors “composite” and “glass-ionomer.” The idea was to suggest that these new materials were a hybrid between
composite resins and glass-ionomer cements, retaining the benefits of both while minimizing their respective disadvantages.
Thus, composite resins have superior strength, fracture toughness, and aesthetics compared with glass-ionomer cements,

https://t.me/DentalBooksWorld
1058 Triumph's Complete Review of Dentistry

but in general lack the ability to bond chemically to tooth substance and release fluoride. Compomers have been a
successful addition to the range of direct restorative materials. Their popularity is largely attributed to their excellent,
nonsticky handling. In appearance and performance, compomers are more closely related to composite resins than glass-
ionomer cements.
15. Answer: A
16. Answer: C (Ref. Phillip's Science of Dental Materials, By Chiayi Shen, H. Ralph Rawls, 2014, page no. 318)
PROPERTIES
Once the powder and liquid are mixed together, heat is produced, i.e., an exothermic reaction takes place. This reaction
speeds up the setting of the material. To control the setting of zinc phosphate, it should always be mixed on a cool, dry glass
slab, and the whole surface area of the slab should be used during the mix to minimize heat production. The manipulation
technique is very important, as a warm slab, mixing too fast, or contamination by water may speed up the setting time of
the material. Incorporating the powder increments too fast or too slow will also affect the setting of zinc phosphate. Zinc
phosphate is fast setting and has a moderate to high solubility and low acidity (once set). The pH is 1–2 but the acidity
decreases over time (about 24 hours).
• Acidic
• Gives off an exothermic reaction (gives out heat when mixed)
• Strong material (reaches two-thirds of strength in less than 1 hour)
• May be used as a base (thicker mix) or a luting cement (thinner mix)
• Mixing times may be extended by mixing the material over a large surface area (dissipates the heat given off as a result
of the exothermic reaction)
• Mixing on a cool glass slab can also extend the working time and allow for greater powder incorporation (gives the
material a higher strength and reduces solubility)
• Care must be taken to not reduce the ratio of powder to liquid as this results in a more soluble, more irritant, and
weaker material
• 50% of the strength of the material is reached after 10 minutes, reaching its final strength after 24 hours
• Moderate solubility when used as a base and high solubility when used as a luting cement
Advantages
• Long shelf-life
• Low thermo-conductivity
• Rapid setting time
• Low cost
• Long clinical history
Disadvantages
• Does not release fluoride
• Freshly mixed material has a high acidity (reduces with setting), but has the potential to cause pulpal irritation
• Moisture sensitive
• Slight shrinkage during setting
• No adhesive properties
• Brittle
Indications
• Permanent cementation of crowns, bridges, inlays, onlays, orthodontic appliances, and orthodontic bands
Contraindications
• Zinc phosphate is acidic at the time of placement, and care should be taken to protect the pulp
17. Answer: B
18. Answer: A
19. Answer: D
20. Answer: B
21. Answer: D
22. Answer: D (Ref. Phillip's Science of Dental Materials, By Chiayi Shen, H. Ralph Rawls, 2014, page no. 316)
• Zinc phosphate is the permanent cement that is mixed on a glass slab
• GIC is a permanent cement which is usually mixed in a paper pad (also by cool glass slab sometimes; GIC has “GLASS”
in its content, mixing in a glass slab, polyacrylic acid can attack the glass slab so a paper pad is used)

https://t.me/DentalBooksWorld
Chapter 11 • Dental Materials 1059

23. Answer: D
24. Answer: C
25. Answer: B (Ref. Basic Dental Materials, By John J. Manappallil, 2015, page no. 81)

Protective Materials
Bases 1–2 mm – Pulpal, thermal and chemical protection
Liners
Thick 1–50 µm
Thin 0.2–1 mm – Pulpal and thermal protection
Thin liners
Suspension liners 20–25 µm
Solution liners 2–5 µm
Cavity varnish Protects pulp by sealing the tubule denying entry of irritants

26. Answer: D
27. Answer: D
A layer of insulating, sometimes medicated cement, placed in the deep portion of the preparation to protect the pulp from
thermal or chemical injury.
28. Answer: C (Ref. (Basic Dental Materials, By John J. Manappallil, 2015, page no. 99)
• Type I: Temporary ZOE luting cement
• Type II: Long-term ZOE luting cement
• Type III: Temporary ZOE restoration
• Type IV: Intermediate ZOE restoration
29. Answer: D
30. Answer: C (Ref. Basic Dental Materials, By John J. Manappallil, 2015, page no. 100)
The chemical composition of ZOE is typically
• Zinc oxide – 69.0%
• White rosin – 29.3%
• Zinc acetate – 1.0% (improves strength)
• Zinc stearate – 0.7% (acts as accelerator) – LEAST
• Liquid (eugenol – 85%, olive oil – 15%)
31. Answer: A

High pH Zinc oxide eugenol


Low pH Zinc phosphate, silicate

32. Answer: A (Ref. Basic Dental Materials, By John J. Manappallil, 2015, page no. 84)

Highest solubility Calcium hydroxide, GIC, silicate


Lowest solubility Resin cements, ZnPo4 has low solubility
in water

33. Answer: A (Ref. Phillip's Science of Dental Materials, By Chiayi Shen, H. Ralph Rawls, 2014, page no. 316)
Zinc phosphate
Positives
• Most commonly used cement
• It has an acceptable compressive strength (double than that of GIC)

https://t.me/DentalBooksWorld
1060 Triumph's Complete Review of Dentistry

Negatives:
• Pulpal irritation
• Lack of adhesiveness
• Lack of anticariogenic properties
• Not suitable when mechanical retention is poor or when aesthetic demand is high
34. Answer: C
35. Answer: C (Ref. Phillip's Science of Dental Materials, By Chiayi Shen, H. Ralph Rawls, 2014, page no. 324)
Modulus of elasticity of GIC is half of that of zinc phosphate cement.
GIC
Positives:
• Greater resistance to degradation in oral cavity
• Fluoride release
Negatives:
• Lower elastic modulus
• Low stiffness (so excessive elastic deformation of ceramic prosthesis which may result in fracture)
36. Answer: A
37. Answer: A (Ref. Phillip's Science of Dental Materials, By Chiayi Shen, H. Ralph Rawls, 2014, page no. 319)
Zinc polycarboxylate cement:
Composition:

Powder Liquid
Zinc oxide Polyacrylic acid
Stannous fluoride Acid concentration varies from 45% to 65%

38. Answer: D (Ref. Phillip’s Science of Dental Materials, By Chiayi Shen, H. Ralph Rawls, 2014, page no. 324)
The purpose of tartaric acid is it
• improves handling characteristics
• increases working time
• shortens working time
• decreases viscosityincreases shelf life
39. Answer: A (Ref. Basic Dental Materials, By John J. Manappallil, 2015, page no. 107)
GIC:
• Maximum particle size – 50 µm
• Particle size for luting/cementing agents – 15 µm
40. Answer: A
Classification:
Types Uses
Type 1 Luting and orthodontic brackets
Type 2a Esthetic restoration
Type 2b Reinforced restoration
Type 3 Lining cements and bases

41. Answer: B
42. Answer: A
43. Answer: D
44. Answer: C
45. Answer: C
46. Answer: A
47. Answer: A
48. Answer: A

https://t.me/DentalBooksWorld
Chapter 11 • Dental Materials 1061

49. Answer: A
50. Answer: B
Reference for questions from 42 to 50 – Below table

Properties

Characteristic/property Unfilled Traditional Hybrid (small Hybrid (all- Microfilled Flowable Packable
acrylic particle) purpose) hybrid hybrid
Size (µm) – 8–12 0.5–3 0.4–10 0.04–0.4 0.6–1.0 Fibrous
Inorganic filler (vol %) 0 60–70 65–77 60–65 20–59 30–55 48–67
Inorganic filler (wt %) 0 70–80 80–90 75–80 35–67 40–60 65–81
Compressive strength (Mpa) 70 250–300 350–400 300–350 250–350 – –
Tensile strength (Mpa) 24 50–65 75–90 40–50 30–50 – 40–45
Elastic modulus (Gpa) 24 8–15 15–20 11–15 3–6 4–8 3–13
Thermal expansion
92.8 25–35 19–26 30–40 50–60 – –
coefficient (ppm/°C)
Water sorption (mg/cm) 1.7 0.5–0.7 0.5–0.6 0.5–0.7 1.4–1.7 – –
Knoop hardness (KHN) 15 55 50–60 50–60 25–35 – –
Curing shrinkage (vol %) 8–10 – 2–3 2–3 2–3 3–5 2–3
Radiopacity (mm Al) 0.1 2–3 2–3 2–4 0.5–2 1–4 2–3

(Ref. Introduction to Dental Materials, By Richard Van Noort, 2014, page no. 84)
Table taken from Van Noort – Very important..!!
51. Answer: A (Ref. Craig’s Restorative Dental Materials, By Ronald L. Sakaguchi, John M. Powes, 2012, page no. 181)
Estrogenicity—Potential of synthetic chemicals with a binding affinity for estrogen receptors to cause reproductive
alterations. Bisphenol-A,a precursor of certain monomers such as bis-GMA,is a known estrogenic compound that is
considered to have possible effects on fetal and infant brain development and behavior.
Bisphenol A (BPA) is, by definition, a major component of Bis-GMA (bisphenol A glycidyl methacrylate or “Bowen’s
resin”), a molecule known to be at the basis of composites and sealants used in dentistry. In the international dental
literature, articles regularly appear arguing that BPA and/or its derivatives might be released into the oral cavity from
composites and sealants in doses which can produce estrogenic effects.
52. Answer: A
53. Answer: A (Ref. Basic Dental Materials, By John J. Manappallil, 2015, page no. 203)
• A major problem with micro-filled composites is that they tend to be sticky and slump.
• Their main advantage is their superior ability to resist wear and polish to a high shine.
• The viscosity of nanocomposites can be adjusted by varying the size and density of the agglomerated nanoclusters.
However, this does little to make the paste more viscous, and adjusting the size and density of the particles only slows
down the slumping.
• Micro-filled composites tend to be more opaque than other forms of resin-glass composites.
• They are less attractive for anterior buildups in spite of their inherent strength.
54. Answer: A (Ref. Phillip's Science of Dental Materials, By Chiayi Shen, H. Ralph Rawls, 2014, page no. 285)
• A small particle has a greater surface area in relationship to its volume than a large particle.
• Because of added surface area, micro-sized particles are disadvantageous compared to macrosized particles.
• Since friction is a function of involved surface area, increased surface area increases internal friction.
• More particles in the paste make the composite stiff. It becomes very difficult to manipulate.
• Colloidal silica particles, because of their extremely small size, have extremely large surface areas ranging from 50 to
400 square meters per gram.
• Macro-filled composites are easier to handle than micros filled to the same density.
• However, greater surface-to-volume ratios give micro-particles one advantage over macro-particles.
• Greater surface area, combined with smaller volume of micro-sized particles, makes micro-particles more difficult to
dislodge from plastic matrix. Moreover, when a micro-sized particle does pop out, it leaves a smaller crater behind and
is more resistant to wear.

https://t.me/DentalBooksWorld
1062 Triumph's Complete Review of Dentistry

GYPSUM
1. Answer: C (Ref. Phillip's Science of Dental Materials, By Kenneth J. Anusavice, Chiayi Shen, H. Ralph Rawls, 2014, page
no. 193)
Type 1 – Dental plaster, impression
Type 2 – Dental plaster, model
Type 3 – Dental stone, die, model
Type 4 – Dental stone, die, high strength, low expansion
Type 5 – Dental stone, die, high strength, high expansion
Dental stone (Type 3)
• Discovered in 1930
• α-Hemihydrate or hydrocal
• Powder particles are more dense and regular in shape
• Comes in different colors, like yellow and green
2. Answer: A (Ref. Craig’s Restorative Dental Materials, By Ronald L. Sakaguchi, John M. Powers, 2012, page no. 105)
Accelerators:
• Gypsum (<20%) – Decreases setting time
• The set gypsum used as an accelerator is called “Terra Alba.”
• Potassium sulfate (conc. 2–3%) reduces the setting time of model plaster from approximately 10 to 4 minutes
• Sodium chloride (<28%)
3. Answer: A
4. Answer: A (Ref. Craig’s Restorative Dental Materials, By Ronald L. Sakaguchi, John M. Powers, 2012, page no. 321)

Water (ml) Powder (g) Water/powder ratio (g/ml)


Plaster 50–60 100 0.55
Stone 20–35 100 0.30
Theoretical ratio 18.6 100 0.186

5. Answer: A (Ref. Craig's Restorative Dental Materials, By Ronald L. Sakaguchi, John M. Powers, 2012, page no. 321)
• The impact of five different water/powder (w/p) ratios in the characterization of high-strength dental stone was
evaluated, since the recommendations of the gypsum manufacturers are not always correctly followed by the dental
prosthesis technicians.
• Fiber Bragg grating (FBG) sensors were used to measure the setting expansion and temperature variation which
occurred during the setting reaction for each w/p ratio, as well as the thermal expansion coefficient. Thick mixtures
with low w/p ratios had more crystals impinging upon each other during crystal growth, resulting in more expansion
and more heat released.
• This thermal behavior was only achieved to w/p ratios within the manufacturer-recommended mixing ratio range.
6. Answer: C (Ref. Phillip’s Science of Dental Materials, By Kenneth J. Anusavice, Chiayi Shen, H. Ralph Rawls, 2014,
page no. 186)
Accelerators:
• Sodium chloride (up to 2% of hemihydrate)
• Sodium sulfate (max. effect at 3.4%)
• Potassium sulfate (>2%)
• Potassium tartrate
7. Answer: A (Ref. Phillip's Science of Dental Materials, By Kenneth J. Anusavice, Chiayi Shen, H. Ralph Rawls, 2014, page
no. 193)
Model plaster (Type 2)
• β-Hemihydrate
• Powder particles are porous and irregular
• It is usually white in color
Use: For primary cast for complete dentures
• For articulation purposes
• For flasking in denture construction

https://t.me/DentalBooksWorld
Chapter 11 • Dental Materials 1063

Advantage:
• Inexpensive
Disadvantages:
• Low strength
• Porosity
Dental stone (Type 3)
• Discovered in 1930
• α-Hemihydrate or hydrocal
• Powder particles are more dense and regular in shape
• Comes in different colors, like yellow and green
Use:
• Making casts for diagnostic purposes and for complete or partial denture construction
Advantage:
• Greater strength and surface hardness
Disadvantage:
• More expensive than plaster
8. Answer: A (Ref. Phillip’s Science of Dental Materials, By Kenneth J. Anusavice, Chiayi Shen, H. Ralph Rawls, 2014, page
no. 185)
Control of setting time
The setting time depends on:
1.  Temperature
–– Effect of temperature on setting time may vary from one plaster or stone to another; little change occurs between
0ºC and 50ºC. If the temperature of plaster–water mixture exceeds 50ºC, a gradual retardation occurs.
–– As the temperature approaches 100º C, no reaction takes place.
–– At a higher temperature range (50º–100ºC), there is a tendency for any gypsum crystals formed to be converted back
to the hemihydrate form.
2.  W:P ratio
–– The more water used for mixing, the fewer nuclei there are per unit volume; consequently, the setting time is
prolonged.
3.  Fineness
–– The finer the particle size of the hemihydrate, the faster the mix hardens, the rate of hemihydrate dissolution
increases and the gypsum nuclei are also more numerous. Therefore, a more rapid rate of crystallization occurs.
4.  Humidity
–– Increased contamination by moisture produces sufficient dihydrate on hemihydrate powder to retard the solution of
hemihydrate.
–– Contamination of gypsum with moisture from air during storage increases setting time.
Factors that control setting time:
a. Factors controlled by the operator:
1.  W:P ratio
–– The more the W:P ratio, the fewer the nuclei per unit volume, so prolonged setting time.
2.  Mixing time
–– Within practical limits, longer and rapid mixing leads to shorter setting time.
–– Some gypsum crystals form immediately when the plaster comes in contact with water and as the mixing begins,
formation of crystals increases.
–– Some crystals are also broken up by mixing spatula and are distributed resulting in the formation of more nuclei of
crystallization resulting in decreased setting time.
Effect of W:P ratio and Mixing time on the Setting time of plaster of Paris.
b. Factors controlled by the manufacturer:
1. By the addition of accelerators and retarders:
Accelerators:
• Gypsum (<20%) – Decreases setting time
–– The set gypsum used as an accelerator is called “Terra Alba.”

https://t.me/DentalBooksWorld
1064 Triumph's Complete Review of Dentistry

• Potassium sulfate (conc. 2–3%) reduces the setting time of model plaster from approximately 10 to 4 minutes
• Sodium chloride (<28%)
Retarders:
• Organic materials – Glue, gelatin, and some gums
• Potassium citrate, borax, sodium chloride (20%), sodium citrate
9. Answer: D
10. Answer: A
11. Answer: C
12. Answer: A (Ref. Craig's Restorative Dental Materials, By Ronald L. Sakaguchi, John M. Powers, 2012, page no. 321)

Type 2 plaster 0.45–0.50


Type 3 stone 0.28–0.30
Type 4 stone 0.22–0.24

INVESTMENTS
1. Answer: C (Ref. Phillip’s Science of Dental Materials, By Kenneth J. Anusavice, Chiayi Shen, H. Ralph Rawls, 2014, page
no. 138)
The word flux means flow, and in the present context means flow of the molten solder being achieved by allowing or
facilitating the wetting of the substrate. The function of fluxes, therefore, is twofold:
1.  To remove any oxides or other compounds present on the surface, and
2.  To prevent further oxidation by excluding oxygen from the site, purely as a physical barrier.
In general, it is not possible to reduce oxides to metal at low temperatures because the oxides are thermodynamically so very
stable. Another chemical approach must be taken, which is to dissolve the oxide. Formerly, in workshop contexts, this was
done by using a solution of hydrochloric acid or similarly acid substance. This, however, did not provide any protective action.
Boron now provides the basis of many commercial fluxes and the constituents for “home-made” recipes in the form of
borax or sodium tetraborate (Na2B4O7.10H2O) and boric acid (B(OH)3).
2. Answer: C
3. Answer: A (Ref. Phillip’s Science of Dental Materials, By Kenneth J. Anusavice, Chiayi Shen, H. Ralph Rawls, 2014, page no. 201)
A dental gypsum bonded investment composition comprising 100 parts by weight of (A) a mixture comprising (a)
α-hemihydrate gypsum as a binder; (b) quartz and/or cristobalite as a refractory material; and (c) one or more aggregates
selected from the group consisting of metallic oxides, metallic carbides, and metallic nitrides, (B) from 0.01 to 1.0 part by
weight of a polytetrafluoroethylene resin, and (C) from 0.001 to 0.05 part by weight of one or more anionic surfactants
selected from the group consisting of alkylbenzenesulfonates and alkylsulfates.
4. Answer: B Gypsum acts as a hardener and it provides strength.
5. Answer: A (Ref. Phillip’s Science of Dental Materials, By Kenneth J. Anusavice, Chiayi Shen, H. Ralph Rawls, 2014, page
no. 392)
• Light the gas-air blowpipe and adjust the knobs so that a cone-shaped flame is formed.
• Closest to the torch is the mixing zone. This zone generates a cool and colorless cone; this cone is called the
mixing zone.
• Outside that cone is a greenish-blue zone, called the combustion zone. Within this zone, partial combustion takes
place, and this zone is partially oxidizing.
• Following the combustion zone is a weak blue cone tip called the reducing zone.
• This zone is the hottest part of the flame and should be used for melting the alloy.
• Outside the reducing zone is an oxidizing zone in which final combustion occurs between the gas and the surrounding air.
6. Answer: A
7. Answer: A
8. Answer: C
9. Answer: A (Ref. Phillip’s Science of Dental Materials, By Kenneth J. Anusavice, Chiayi Shen, H. Ralph Rawls, 2014, page
no. 229)

https://t.me/DentalBooksWorld
Chapter 11 • Dental Materials 1065

Composition
There are three different binders being used in dental investment materials. These binders are
• Gypsum
• Phosphate
• Silicate
Investment materials used for traditional gold castings use gypsum most often because gypsum-bonded investment
materials tend to be able to produce smoother surfaces. However, because of the need of using investment materials
that can resist higher temperatures, phosphate-, and silicate-based investment materials have also been developed. The
phosphate-bonded investment materials are used for casting porcelain-fused-to-metal castings, because the alloys used
in these constructions are melted at higher temperatures than the traditional gold alloys. The silicate bonded investment
materials are primarily used for casting partial denture frameworks in base metal alloys.
The refractory material used in the different investment materials are different SiO2 structures such as cristobalite,
quartz, tridymite, or fused silica.
In addition to the binder and the refractory material, different catalysts are also added depending on the composition of
the binder. These compounds can also affect the setting expansion of the investment material and thereby also affect the
final size of the casting.
It is also common to add reducing agents such as carbon (graphite). Such an addition results in nonoxidized castings.
The carbon (graphite) reacts with oxygen to produce carbon monoxide or carbon dioxide and by doing so, no oxygen will
be available in the investment material for reacting with the metal during the casting process.
Properties
ANSI/ADA Specification No. 2
Types
There are three types of investment materials. Type I and Type II are for gypsum-bonded investment materials. The Type I
investment material relies primarily on thermal expansion, while a Type II material relies on hygroscopic expansion. The
Type III investment material is phosphate bonded.
Type I investment material
Used for thermal expansion
Thermal expansion for these investment materials range from 1.0% to 1.6%, while their setting expansion in air range
from 0.0% to 0.6%. The combined expansions (setting and thermal expansion) should range from 1.3% to 2.2%.
Type II investment material
Used for hygroscopic expansion
Setting expansion under water should be 1.2–2.2% and the thermal expansion 0–0.6%. The total expansion of a Type II
investment material should range from 1.3% to 2.7%.
Type III investment materials
Use of investment material
Setting expansion should be 0.0–0.4%. Thermal expansion should be 1.0–1.5%. The total expansion should range from
1.2% to 1.9%.
10. Answer: C
11. Answer: A (Ref. Phillip’s Science of Dental Materials, By Kenneth J. Anusavice, Chiayi Shen, H. Ralph Rawls, 2014, page
no. 214)
Sprue former length
Depends on the length of the casting ring. Length of the sprue former should be such that it keeps the wax pattern about
6–8 mm away from the casting ring. Sprue former should be no longer than 2 cm. The pattern should be placed as close to
the center of the ring as possible.
Significance
• Short sprue length:
–– The gases cannot be adequately vented to permit the molten alloy to fill the ring completely leading to back pressure
porosity.
• Long sprue length:
–– Fracture of investment, as mold will not withstand the impact force of the entering molten alloy.

https://t.me/DentalBooksWorld
1066 Triumph's Complete Review of Dentistry

• Top of wax should be adjusted for:


–– 6 mm for gypsum-bonded investments
–– 3–4 mm for phosphate-bonded investments
12. Answer: A
13. Answer: B (Ref. Phillip’s Science of Dental Materials, By Kenneth J. Anusavice, Chiayi Shen, H. Ralph Rawls, 2014, page
no. 225)
Porosity
Porosity may occur, both within the interior region of a casting and on the external surface. The latter is a factor in surface
roughness, but generally it is also a manifestation of internal porosity. Not only does the internal porosity weaken the
casting but it also may be a cause for discoloration, if it extends to the surface. If severe, it can produce leakage at the tooth-
restoration interface, and thus may result in secondary caries. Although the porosity in a casting cannot be prevented
entirely, it can be minimized by the use of proper techniques.
Porosities in noble metal castings may be classified as follows:
a. Solidification defects
• Localized shrinkage porosity
• Microporosity
b. Trapped gases
• Pinhole porosity
• Gas inclusions
• Subsurface porosity
c. Residual air
Localized shrinkage: Is generally caused by incomplete feeding of molten metal during solidification. The linear
contraction of noble metal alloys in changing form a liquid to a solid is at least 1.25%.
• Therefore, there must be continual feeding of molten metal through the sprue to make up for the shrinkage of metal
volume during solidification. If the sprue freezes in its cross-section before this feeding is completed to the casting
proper, a localized shrinkage void will occur in the last portion of the casting that solidifies.
• The porosity in the pontic area is caused by the ability of the pontic to retain heat because of its bulk and because it was
located in the heat center of the ring. This problem can be solved in the future simply by attaching one or more small-gauge
sprues (e.g., 18 gauge) at the surface most distant from the main sprue attachment and extending the sprue laterally within
5 mm of the edge of the ring.
• These small chill-set sprues ensure that solidification begins within these sprues and they act as cooling pins to carry
heat away from the pontic.
• Localized shrinkage generally occurs near the sprue–casting junction, but it may occur anywhere between dendrites,
where the last part of the casting that solidified was in the low melting metal that remained as the dendrite branches
develop.
Microporosity: It also occurs from the rapid solidification but is generally present in fine grain alloy castings when the
solidification is too rapid for the micro void to segregate to the liquid pool. This premature solidification causes the porosity.
Such phenomenon can occur from the rapid solidification if the mold or casting temperature is too low. It is unfortunate
that this type of defect is not detectable unless the casting is sectioned. In any cast, it is generally not a serious defect.
Pinhole and gas inclusion porosities: They are related to the entrapment of gas during solidification. Both are
characterized by a spherical contour, but they are decidedly different in size.
The gas inclusion porosities are usually much larger than pinhole porosity. Many metals dissolve or occlude gases while
they are molten. For example, both copper and silver dissolve oxygen in large amounts in the liquid state; molten platinum
and palladium have strong affinity for hydrogen as well as oxygen. On solidification, the absorbed gases are expelled,
resulting in pinhole porosity.
The larger voids may also result from the same cause, but it seems more logical to assume that such voids may be caused
by gas that is mechanically trapped by the molten metal in the mold or that is incorporated during the casing procedure.
All casings probably contain a certain amount of porosity.
However, the porosity should be kept to a minimum because it may adversely affect the physical properties of the casting.
Oxygen is dissolved by some of the metals, such as silver, in the alloy while they are in the molten state. During
solidification, the gas is expelled to form blebs and pores in the metal. As was pointed out earlier, this type of porosity may

https://t.me/DentalBooksWorld
Chapter 11 • Dental Materials 1067

be attributed to abuse of the metal. Castings that are severely contaminated with gases are usually black when they are
removed from the investment and do not clean easily on pickling. The porosity that extends to the surface usually appears
in the form of small pinholes.
Entrapped air porosity: On the inner surface of the casting, sometimes referred to as backpressure porosity, large concave
depressions can be produced.
This is caused by the inability of the air in the mold to escape through the pores in the investment or by the pressure
gradient that displaces the air pocket toward the end of the investment via the molten sprue and button.
The entrapment is frequently found in a “pocket” at the cavity surface of a crown or mesio-occlusal-distal casting.
Occasionally it is found even on the outside surface of the casting when the casting temperature or mold temperature is so
low that solidification occurs before the entrapped air can escape.
The incidence of entrapped air can be increased by the dense modern investments, an increase in mold density produced
by vacuum investing, and the tendency for the mold to clog with residual carbon when the low-heat technique is used.
Each of these factors tends to slow down the venting of gases from the mold during casting.
Proper burnout, an adequate mold and casting temperature, a sufficiently high casting pressure, and proper L:P ratio can
help to eliminate this phenomenon. It is good practice to make sure that the thickness of investment between the tip of the
pattern and the end of the ring is not greater than 6 mm.
14. Answer: C
15. Answer: D
16. Answer: B
17. Answer: A
18. Answer: D
19. Answer: C
20. Answer: C (Ref. Basic Dental Materials, By John J. Manappallil, 2015, page no. 333)
• Paraffin wax is likely to flake when it is trimmed, and it does not produce a smooth, glossy surface, which is a desirable
requisite for an inlay wax. Thus, other waxes and natural resins must be added as modifying agents.
• Gum dammar, or dammar resin, is a natural resin. It is added to the paraffin to improve the smoothness in molding
and to render it more resistant to cracking and flaking. It also increases the toughness of the wax and enhances the
smoothness and luster of the surface.
• Carnauba wax occurs as a fine powder on the leaves of certain tropical palms. This wax is very hard, and it has a
relatively high melting point and it has an agreeable odor.
• It is combined with the paraffin to decrease flow at mouth temperature. Carnauba wax contributes greater glossiness to
the wax surface than the dammar resin.
• Candelilla wax can also be added partially or entirely to replace carnauba wax.
• Candelilla wax provides the same general qualities as carnauba wax but its melting point is lower and it is not as hard as
carnauba wax. Ceresin may replace part of the paraffin to modify the toughness and carving characteristics of the wax.
• Ceresin is typically a white wax extracted from ozokerite, a waxy mineral mixture of hydrocarbons that is colorless or
white when pure, but it has a somewhat unpleasant odor.
• Carnauba wax is often replaced in part by certain synthetic waxes that are compatible with paraffin wax.
• At least two waxes of this type can be used. One is a complex nitrogen derivative of the higher fatty acids and the other
contains esters of acids derived from montan wax, a derivative hard wax that is obtained by solvent extraction of certain
types of lignite or brown coal.
• Approximately a third of all montan wax produced is used in automobile polishing pastes.
21. Answer: C
22. Answer: A (Ref. Phillip’s Science of Dental Materials, By Kenneth J. Anusavice, Chiayi Shen, H. Ralph Rawls, 2014,
page no. 203)
Setting expansion
Expansion may vary from 0.06% to 0.5%
Volume of dihydrate formed is less than equal volume of hemihydrate and water, i.e., actually a volumetric contraction
should occur during setting reaction, but instead a setting expansion is observed.
• Phenomenon: Based on crystallization mechanism.
• The crystallization process occurs as an outgrowth of crystals from nuclei of crystallization.
• The dihydrate crystals growing from the nuclei not only intermesh with but also obstruct the growth of adjacent crystals.

https://t.me/DentalBooksWorld
1068 Triumph's Complete Review of Dentistry

• If this process is repeated by thousands of crystals during growth, an outward stress or thrust develops that produces an
expansion of the entire mass.
• The crystal impingement and movement results in the formation of micropores.
• Result: The gypsum formed is greater in external volume but less in crystalline volume; therefore, the set material must
be porous.
Control of setting expansion
1. W:P ratio:
Increase in w/p ratio, decreases the nuclei of crystallization per unit volume, so there is less growth of dihydrate crystals
which leads to less outward thrust
2. Accelerators and retarders: Chemicals added by the manufacturer to regulate setting expansion
Accelerators:
• Sodium chloride (up to 2% of hemihydrate)
• Sodium sulfate (max. effect at 3.4%)
• Potassium sulfate (>2%)
• Potassium tartrate
Retarders:
• Chemicals that form a coating on the hemihydrate particles and prevent the hemihydrate from going into the solution
in the normal manner
• Citrates, acetates, and borates
Hygroscopic setting expansion
• Setting expansion that occurs under water is known as “Hygroscopic Setting Expansion.”
• Setting expansion without water immersion is known as “Normal Setting Expansion.”
Stages of hygroscopic setting expansion:
Stage I – Initial mix stage
• Represented by three round particles of hemihydrate surrounded by water

Stage II – Initial crystal growth stage


• Crystals of dihydrate have started to form
• In NSE, the water around the particles is reduced by hydration and particles are drawn close together by surface tension
of water.
• In HSE, the setting is taking place under water so that water of hydration is replaced and the distance between the
particles remain the same.
Stage III – Solid phase contact stage
• As the dihydrate crystals grow, they contact each other and setting expansion begins
• In NSE, the water around the particles is reduced. The particles with their attached crystals are drawn together as
before, but the contraction is opposed by the outward thrust of the growing crystals.
• In HSE, the crystals are not inhibited, because the water is again replenished from the outside. In fact, the original
particles are now separated further as the crystals grow and setting expansion occurs.
Stages IV and V
• Expansion and termination
• The effect becomes more marked
• The crystals that are inhibited in NSE become intermeshed and entangled much sooner than in HSE in which the
crystals grow much more freely during the early stages before the intermeshing.
• The observed setting expansion that occurs when the gypsum product sets under water may be greater than that which
occurs during setting in air.
23. Answer: C
24. Answer: C (Ref. Phillip’s Science of Dental Materials, ByKenneth J. Anusavice, Chiayi Shen, H. Ralph Rawls, 2014, page
no. 408)
Welding is the joining of two bodies brought into contact by almost any other means than soldering. For example, two
perfectly clean gold surfaces (or, indeed, many other metals) placed in vacuum where the adsorbed gases and water may
evaporate (perhaps helped by warming), and then placed in contact, will weld together instantly.

https://t.me/DentalBooksWorld
Chapter 11 • Dental Materials 1069

BONDING
1. Answer: A
Classification of Dental Bonding System

Etch-and-rinse Self-etch
Three step (fourth Two step (fifth Two step (sixth generation) One step (all-in-one) (seventh
generation) generation) • Does not involve a separate etching generation)
• Combines the primer step • This category combines the
and adhesive resin into • In this case, an acidic monomer conditioner, primer, and
one application which is not rinsed is used to bonding resin into a single
condition and prime the tooth at step
the same time • There is no need for rinsing
• There are two types of self-etch or drying of the tooth
adhesives (Van Meerbeek et al., structure because of the
2001): lack of an etch step
• There are two types of self-etch • Recently, a new self-etch
adhesives (Van Meerbeek et al., adhesive bonding agent
2001) There are two types of self- has been introduced that
etch adhesives (Van Meerbeek et expands this category
al., 2001): Mild and strong varieties for use with total etch
• Strong: pH < 1 procedures (Scotchbond
• Mild: pH = 2 Universal Adhesive, 3M
– Only partially dissolve dentin ESPE, St. Paul, MN, United
States)
– So some amount of
• This system contains a
hydroxylapatite crystals remains
available within the hybrid layer… phosphate monomer and
now specific carboxyl or phosphate silane, and claims to offer
groups of functional monomers extended bond durability as
can then chemically interact with well as versatility for use in
this residual hydroxyapatite various clinical procedures
• Because this layer has some
mineral content, the bond to
dentin is better than that of etch-
and-rinse adhesives
Etch Etch (30–40%) phosphoric Etch and prime Etch, Prime, and Bond
Apply for 15 seconds, acid – 37% usually used One application without rinsing, Apply 1–5 layers without
rinse 15 seconds, – If used beyond 50%, gently air dry rinsing, gently air-dry, light
gently air dry while it forms monocalcium cure
keeping the dentin phosphate monohydrate
moist layer on the etched surface
Apply for 15 seconds, rinse
15 seconds, gently air dry
while keeping dentin moist
Primer Prime and bond Bond
It is hydrophilic Apply 1–5 layers, gently Apply one layer, gently air-dry, light
Apply 1–5 layers, air-dry, light cure cure
gently air dry
Bond
Resin is hydrophobic
Apply one layer,
gently air-dry,
light-cure

https://t.me/DentalBooksWorld
1070 Triumph's Complete Review of Dentistry

2. Answer: D
3. Answer: B (Ref. Phillip’s Science of Dental Materials, By Kenneth J. Anusavice, Chiayi Shen, H. Ralph Rawls, 2014, page
no. 261)
Etching time
• The optimal application time for the etchant may vary somewhat, depending on previous exposure of the tooth surface
to fluoride and other factors.
• For example, a permanent tooth with a high fluoride content may require a somewhat longer etching time, as do
primary teeth.
• In the latter, increased surface conditioning time is needed to enhance the etching pattern on the primary tooth enamel
that is more aprismatic than the permanent tooth enamel.
• Currently, the etching time for most etching gels is approximately 15 seconds.
• The advantage of such short etching times is that they yield acceptable bond strength in most instances, while conserving
enamel and reducing treatment time.
4. Answer: B (Ref. Phillip’s Science of Dental Materials, By Kenneth J. Anusavice, Chiayi Shen, H. Ralph Rawls, 2014, page
no. 260)
Prior to the introduction of enamel acid etching and the use of enamel bonding agents, restorative materials were placed
directly on the smear layer of the prepared tooth. The apparent bond strength is the cohesive strength (5–10 MPa) of the
smear layer, which is not sufficient to withstand the daily mechanical forces experienced in the mouth.
5. Answer: C
Bond strength of various bonding agents
(Ref. http://www.iosrjournals.org/iosr-jdms/papers/Vol14-issue11/Version-7/S01411797100.pdf)

First generation 2–3 Mpa


Second generation 4.5–6 Mpa
Third generation 16–26 Mpa
Fourth generation 25 Mpa*
Fifth generation Relatively higher but less
than fourth generation
Sixth generation Lower than fourth and
fifth generations
Seventh generation Equal to sixth generation

*Highly effective with shear bond strengths of 25 MPa to both enamel as well as dentin. Fusayama and Nakabayashi also described
the penetration of resins into dentin as giving dentinal seals with high bond strengths. Kanca also introduced the “wet bonding”
concept with these systems.

6. Answer: A
Dentin etching did not gain wide acceptance until Fusayama introduced the total-etch concept in 1979.
7. Answer: B
8. Answer: A
9. Answer: A
Primer is hydrophilic while resin is hydrophobic.
10. Answer: C (Ref. Phillip’s Science of Dental Materials, By Kenneth J. Anusavice, Chiayi Shen, H. Ralph Rawls, 2014, page
no. 329)
• For cementation of all ceramic restorations, resin (can be chemically or light or dual cured) cement is the material of
choice (Ref. Anusavice, 12th edition, Page no. 269).
• Self-adhesive resin cements are dual-cured materials and can be used for cementation of indirect restorations in most
clinical situations.
• For cementation of veneers, light-cured resin cements are preferred.

https://t.me/DentalBooksWorld
Chapter 11 • Dental Materials 1071

• Self-adhesives are not recommended by manufactures as luting agents for orthodontic fixed appliances.
• Self-adhesive cements exhibit a lower bond strength to intact enamel in comparison with conventional orthodontic
resin cements that require phosphoric acid etching.
• If doubt exists about whether the pit and fissure is free from caries or not, it is still justified to place a sealant.
• Clinical trials in which sealants were intentionally placed in pits and fissures that were diagnosed as having caries have
shown that as long as a sealant is well retained, no caries progression will occur.
11. Answer: D
12. Answer: A
13. Answer: C

CERAMICS
1. Answer: B (Ref. Phillip’s Science of Dental Materials, By Kenneth J. Anusavice, Chiayi Shen, H. Ralph Rawls, 2013, page
no. 432)

Porcelain type Fusion temperature range Clinical applications


High fusing >1,300°C Denture teeth
Medium fusing 1,000°–1,300°C Jacket Crowns, Bridges,
and Inlays
Low fusing 850°–1,000° C Veneers over cast metal
crowns
Ultra-low fusing <850° C C Used with titanium
and its alloys

2. Answer: A
• Feldspar is responsible for forming the glass matrix.
• Feldspar is the lowest melting compound and melts first on firing.
• Feldspar is a naturally occurring mineral and composed of two alkali aluminum silicates such as potassium aluminum
silicate (K2O-Al2O3-6SiO2); also known as potash feldspar or orthoclase and soda aluminum silicate (Na2O-Al2O3-
6SiO2); also known as soda feldspar or albite.
• Most of the currently available porcelains contain potash feldspar as it imparts translucency to the fired restoration.
• Potash fuses with kaolin and quartz to form glass when heated from 1,250°C to 1,500°C. Soda feldspar lowers the
fusion temperature of the porcelain that results in pyroplastic flow.
• This material did not attract the porcelain manufacturers as it does not influence the translucency of the porcelain.
3. Answer: B (Ref. Phillip’s Science of Dental Materials, By Kenneth J. Anusavice, Chiayi Shen, H. Ralph Rawls, 2014, page
no. 62)

Compressive strength 330 MPa


Diametral tensile strength 34 MPa
Transverse strength 62–90 MPa
Shear strength 110 MPa
MOE 69 GPa
Surface hardness 460 KHN
Specific gravity 2.2–2.3 g/cm3
Thermal conductivity 0.0030 cal/sec/cm2
Thermal diffusivity 0.64 mm2/sec
Coefficient of thermal expansion 12 × 10-6/°C

https://t.me/DentalBooksWorld
1072 Triumph's Complete Review of Dentistry

4. Answer: C
5. Answer: A
6. Answer: A (Ref. Phillip’s Science of Dental Materials, By Kenneth J. Anusavice, Chiayi Shen, H. Ralph Rawls, 2013, page
no. 432)
Firing or sintering
It is to fuse the particles of porcelain powder producing hard mass.
Stages of firing:
a. Low bisque stage
• Particles lack complete adhesion, low amount of shrinkage occur, and very porous
b. Medium bisque stage
• Water evaporates with better cohesion to the powder particles and some porosity
c. High bisque stage
• Fusion of particles to form a continuous mass, complete cohesion, and no more shrinkage
7. Answer: C
8. Answer: B
9. Answer: A
10. Answer: B
11. Answer: A
12. Answer: A
13. Answer: C
14. Answer: B
15. Answer: B
16. Answer: B
17. Answer: C
18. Answer: A
19. Answer: A
20. Answer: C
21. Answer: A
22. Answer: D
23. Answer: C
24. Answer: A
25. Answer: D
26. Answer: C
27. Answer: C
28. Answer: B
29. Answer: A
30. Answer: A
31. Answer: C
32. Answer: A
33. Answer: B
34. Answer: C
35. Answer: A (Ref. Phillip’s Science of Dental Materials, By Kenneth J. Anusavice, Chiayi Shen, H. Ralph Rawls, 2013, page
no. 550)
• Feldspar is a naturally occurring mineral and composed of two alkali aluminum silicates such as potassium aluminum
silicate (K2O-Al2O3-6SiO2), also known as potash feldspar or orthoclase and soda aluminum silicate (Na2O-Al2O3-
6SiO2), also known as soda feldspar or albite.
• Most of the currently available porcelains contain potash feldspar as it imparts translucency to the fired restoration.
• Potash fuses with kaolin and quartz to form glass when heated from 1,250°C to 1,500°C. Soda feldspar lowers the
fusion temperature of the porcelain that results in pyroplastic flow.
• This material did not attract the porcelain manufacturers as it does not influence the translucency of the porcelain.

https://t.me/DentalBooksWorld
Chapter 11 • Dental Materials 1073

DENTAL WAX
1. Answer: D
2. Answer: C (Ref. Basic Dental Materials, By John J. Manappallil, 2015)
• Type I: A soft wax, is used for building veneers
• Type II: A medium wax, is designed for patterns to be placed in the mouth in normal climatic conditions
• Type III: A hard wax, is used for trial fitting in the mouth in tropical climates
• Because residual stress is present within the wax from contouring and manipulating the wax, the finished denture
pattern should be flasked as soon as possible after completion of all adjustments and manipulations.
3. Answer: A (Ref. Phillip’s Science of Dental Materials, By Kenneth J. Anusavice, Chiayi Shen, H. Ralph Rawls, 2014, page
no. 196)
Flow can be reduced by
• Adding more carnauba wax
• Using higher melting paraffin wax
4. Answer: D
5. Answer: A
6. Answer: D (Ref. Basic Dental Materials, By John J. Manappallil, 2015, page no. 333)

Pattern wax Processing wax Impression wax


1. Inlay wax 1. Boxing wax 1. Corrective wax
2. Casting wax 2. Utility wax 2. Bite registration wax
3. Base plate wax 3. Sticky wax

7. Answer: B
8. Answer: A

Type of Wax T = 30°C T = 37°C T = 40°C T = 40°C T = 45°C T = 45°C


Maximum Maximum Minimum Maximum Minimum Maximum
I — 1 — 20 70 90
II 1 — 50 — 70 90

9. Answer: A
10. Answer: C
11. Answer: C
12. Answer: A (Ref. Phillip’s Science of Dental Materials, By Kenneth J. Anusavice, Chiayi Shen, H. Ralph Rawls, 2014,
page no. 198)
• Inlay waxes are softened with heat, forced into the prepared tooth cavity in either the tooth or the die, and cooled.
• The wax may expand as much as 0.7% with an increase in temperature of 20°C or contract as much as 0.35% when it is
cooled from 37°C to 25°C.The average linear thermal expansion coefficient over this temperature range is 350×10−6/K,
with values ranging from 217 to 512×10−6/K.
13. Answer: A
14. Answer: B (Ref. Basic Dental Materials, By John J. Manappallil, 2015, page no. 342)
Korecta wax is available in four grades, each with a different degree of plastic deformation at mouth temperature. Each
grade is designed for a specific purpose.
1. Extra hard No.1 (pink): A reinforcing material used only on the external surface to support wax extensions beyond
tray margins.
2. Hard No. 2 (yellow): Used in rebasing as a hard foundation for Korecta waxes 3 and 4 when extensive absorption
(alveolar resorption) necessitates a bulk of material. Also, used to restore occlusion in a partial denture or a removable
partial denture, which has settled due to severe tissue change.
3. Soft No. 3 (red): Used for minor tray correction and as an initial lining to stabilize the tray.

https://t.me/DentalBooksWorld
1074 Triumph's Complete Review of Dentistry

4. Extra soft No. 4 (orange): Used to secure a completely adapted impression under natural masticatory pressure. It
leaves a finished surface and registers fine tissue details.

Wax %Flow at 37°C


Korecta No. l (extra hard) 3
Korecta No. 2 (hard) 80
Korecta No. 3 (soft) 85
Korecta No. 4 (extra soft) 90

These waxes can also be used to produce a mucocompressive impression of the edentulous saddles for a lower,
free-end saddle partial denture. This is known as the Applegate technique.
Korecta wax Nos. 1 and 4 are no longer available which were originally used for making edentulous impressions.
However, necessary waxes like extra soft No. 4 (orange) are available again and can be used.
15. Answer: C
Fluid wax technique
• The anterior and posterior in vibrating lines are marked as for the conventional techniques on the final wash impression.
• Impressions made with zinc-oxide eugenol or plaster are preferred over the elastic impression materials as they set
rigid, are slightly resilient and when reseated in the mouth under pressure, it may distort the relationship between
the wax added to the posterior border and the rest of the denture bearing surface. Also, wax will not adhere to elastic
materials. Hence, either the material in the seal area must be removed prior to the wax application or laboratory varnish
must be applied to the elastic material in the seal area before the wax is placed.
Waxes which can be used are:
• IOWA wax (white) developed by Dr. Earl S. Smith.
• Korecta wax No. 4 (orange) developed by Dr. O. C.Applegate.
• H-L physiologic paste (yellow–white) developed by Dr. C S. Harkins.
• Adaptol (green) developed by Nathan G. Kaye.
These waxes are designed to flow at mouth temperature.
16. Answer: A (Ref. Craig’s Restorative Dental Materials, By Ronald L. Sakaguchi, John M. Powers, 2012, page no. 62)

Natural waxes Synthetic waxes Additives


Mineral Fats
a. Paraffin a. Aerosol OT Stearic acid
b. Monton wax b. Castor wax
c. Barnsdahl c. Flexo wax C
d. Ozokerite d. Dura wax
e. Microcrystalline
Plant Natural resins
a. Carnauba a. Copal
b. Ouricury b. Dammar
c. Candelilla c. Sandarac
d. Japan wax d. Shellac
e. Cocoa butter
Insect Synthetic resins
Bees wax a. Polyethylene
b. Polystyrene
Animal
Spermaceti

17. Answer: A

https://t.me/DentalBooksWorld
Chapter 11 • Dental Materials 1075

18. Answer: A (Ref. Fixed Restorative Techniques, By Henry V. Murray, Troy B.Sluder, Roger E. Barton, page no. 184)
Like flow, ductility increases with increase in temperature of waxes
• The lower the melting temperature of a wax, the more will be its ductility
• Waxes made of components having wide melting ranges have more ductility
• With a wide range of melting point of components, the softening point of lowest is approached first on heating
• On further heating this component liquefies, the softening point of next is approached and so on
• Entire wax mass is plasticized and ductility increases
19. Answer: B(Ref. Craig’s Restorative Dental Materials, By Ronald L. Sakaguchi, John M. Powers, 2012, page no. 310)
Available as
• Universal
• Specific purpose like
–– occlusal
–– cervical
–– underlay
Universal
• Ideally suited for quick coverage of large areas with wax
• Low surface tension
• Low melting point
• Small contraction on hardening
• Excellent flow properties
Occlusal
• High strength – No abrading of contact points
• High rigidity
• Can be easily drawn due to its high surface tension
• Opaque appearance for defined contours and permanent control during modeling
Difference between occlusal and universal wax
• Occlusal wax – Greater surface tension and high rigidity
• The surface tension allows ball-shaped drops to form and harden
• This eases work considerably in the waxing-up phase
• The increased strength guarantees precise modeling
• Universal wax – Excellent flooding properties, can be used in many areas
Cervical
• Low shrinkage
• Particularly stable
• Can be adapted precisely and thinly to the preparation border
• Excellent carving properties
Underlay
• For small undercuts and cavity coverage before inlay modeling
• Significantly softer than cervical wax
• Gentle elasticity
• Good adaptation and carving properties
• Very low shrinkage
Dipping wax
• The hotty LED is a wax dipping pot which permits controlled temperature setting and displays the set and the actual value
Advantages
• High precision via low shrinkage
• Optimum viscosity at 89°–91°C (192°–196°F)
• Precise-fitting copings with an even layer thickness
• High stability and elasticity
• Contains no acrylic additives
• Easy to cut off the preparation border

https://t.me/DentalBooksWorld
1076 Triumph's Complete Review of Dentistry

MISCELLANEOUS
1. Answer: A (Ref. Basic Dental Materials, By John J. Manappallil, 2015, page no. 269)
The powder contains sodium alginate, calcium sulfate, trisodium phosphate, diatomaceous earth, zinc oxide, and
potassium titanium fluoride. On mixing the powder with water, a sol is formed, a chemical reaction takes place, and a gel
is formed.
Here, sodium alginate reacts with calcium sulfate, resulting in sodium sulfate and calcium alginate. This reaction occurs
too quickly often during mixing or loading of the impression tray. Hence it is slowed down by the addition of trisodium
phosphate to the powder.
Trisodium phosphate reacts with calcium sulfate to produce calcium phosphate, preventing calcium sulfate from reacting
with sodium alginate to form a gel.
This second reaction occurs in preference to the first reaction until the trisodium phosphate is used up, and then alginate
sets as a gel. There is a well-defined working time during which there is no viscosity change.
2. Answer: D (Ref. Basic Dental Materials,By John J. Manappallil, 2015, page no. 269)
• Setting time is “best” regulated in alginate by – Amount of retarder added (then only by controlling temperature).
• The increase in temperature, the shorter the setting time – For 10° rise in temperature there is 1 minute reduction in
setting time.
• Always water is added to the bowl first, because if powder is added first then the penetration of water is inhibited and
greater mixing time is required.
• Small amount of gypsum left in the bowl can accelerate the set.
3. Answer: B
Imbibition – the act of absorbing moisture
4. Answer: A
Syneresis is the extraction or expulsion of a liquid from a gel.
5. Answer: C (Ref. Phillip’s Science of Dental Materials, By Kenneth J. Anusavice, Chiayi Shen, H. Ralph Rawls, 2014, page
no. 174)

Alginate
Components Uses
Filler-diatomaceous earth – 60%, Strength
ZnO2
Reactor – Calcium sulfate Initiates reaction
Retarder – Sodium phosphate Slows down reaction
Calcium alginate – 15% Chief active ingredient
Accelerator – Potassium titanium Accelerates the reaction
fluoride

6. Answer: D
7. Answer: C
8. Answer: D (Ref. Phillip’s Science of Dental Materials, By Kenneth J. Anusavice, Chiayi Shen, H. Ralph Rawls, 2014, page
no. 174)
a. The alginate-type hydrocolloids are an elastic-type impression material. An alginate is a salt of alginic acid (an extract
from seaweed). Alginate-type hydrocolloids gel by chemical action. Once the gelation process begins, it is irreversible.
b. Composition: The composition of the alginate-type hydrocolloids varies with different manufacturers. The basic
components are a soluble alginate (either potassium alginate or sodium alginate) and a reactor (calcium sulfate), which
causes the alginate to gel. The product also contains a retarder (sodium or potassium sulfate, oxalate, or carbonate) to
prevent gelation from occurring too rapidly. A fluoride is usually added to prevent retardation of the setting time of the
casts. The remainder of the material is composed of fillers that increase the strength and stiffness of the gel.
c. Usage: Alginate-type hydrocolloids are supplied in powder form, either in bulk or in measured portions packaged in foil
envelopes. The powder must be stored in a cool place. The bulk form must be kept in a tightly closed container to protect it
from contamination and to prevent it from absorbing moisture from the air. The containers are agitated to loosen the bulk
powders before they are measured, thus preventing use of an excessive proportion of the powder. The powder is mixed
with a measured amount of water.
https://t.me/DentalBooksWorld
Chapter 11 • Dental Materials 1077

9. Answer: D (Ref. Basic Dental Materials, By John J. Manappallil, 2015, page no. 269)
• Setting time is “best” regulated in Alginate by – Amount of retarder added – Sodium phosphate (then only by
controlling temperature).
• The increase in the temperature, the shorter the setting time – For 10° rise in temperature there is 1 minute reduction
in setting time.
• Always water is added to the bowl first, because if powder is added first then the penetration of water is inhibited and
greater mixing time is required.
• Small amount of gypsum left in the bowl can accelerate the set.
10. Answer: A (Ref. Phillip’s Science of Dental Materials, By Kenneth J. Anusavice, Chiayi Shen, H. Ralph Rawls, 2014, page
no. 190)
Dental stone (Type 3):
• Discovered in 1930
• α-Hemihydrate or hydrocal
• Powder particles are more dense and regular in shape
• Comes in different colors, like yellow and green

Use:
Making casts for diagnostic purposes and for complete or partial denture construction
Advantage:
• Greater strength and surface hardness

Disadvantage:
• More expensive than plaster
11. Answer: B (Ref. Phillip’s Science of Dental Materials, By Kenneth J. Anusavice, Chiayi Shen, H. Ralph Rawls, 2014, page
no. 188)
Model plaster (Type 2):
• β-Hemihydrate
• Powder particles are porous and irregular
• It is usually white in color

Use:
• For primary cast for complete dentures
• For articulation purposes
• For flasking in denture construction

Advantage:
• Inexpensive

Disadvantages:
• Low strength
• Porosity
12. Answer: A (Ref. Phillip’s Science of Dental Materials, By Kenneth J. Anusavice, Chiayi Shen, H. Ralph Rawls, 2014, page
no. 180)
• Gypsum is a naturally occurring white powdery mineral mined in various parts of the world, with chemical name
calcium sulfate dihydrate – CaSO4.2H2O
• Gypsum is derived from a Greek word “Gypsas” (chalk)
Gypsum products used in dentistry are based on calcium sulfate hemihydrate (CaSO4.2H2O)
13. Answer: C (Ref. Phillip’s Science of Dental Materials,By Kenneth J. Anusavice, Chiayi Shen, H. Ralph Rawls, 2014, page
no. 183)
Calcination
All dental gypsum products are produced through a process of heating gypsum and driving off part of the water of
crystallization.
The result is the driving off of 1 1/2 H2O.
Products differ in characteristics due to the type of calcination process used.
14. Answer: C

https://t.me/DentalBooksWorld
1078 Triumph's Complete Review of Dentistry

15. Answer: A
16. Answer: B
The current ISO standard for dental gypsum products identifies five types of materials as follow:
• Type 1 – Dental plaster, impression
• Type 2 – Dental plaster, model
• Type 3 – Dental stone, die, model
• Type 4 – Dental stone, die, high strength, low expansion
• Type 5 – Dental stone, die, high strength, high expansion
17. Answer: C (Ref. Phillip’s Science of Dental Materials, By Kenneth J. Anusavice, Chiayi Shen, H. Ralph Rawls, 2014, page
no. 190)
Dental stone (Type 3)
• Discovered in 1930
• α-Hemihydrate or hydrocal
• Powder particles are more dense and regular in shape
• Comes in different colors, like yellow and green

Use:
Making casts for diagnostic purposes and for complete or partial denture construction
Advantage:
• Greater strength and surface hardness

Disadvantage:
• More expensive than plaster
18. Answer: D (Ref. Phillip’s Science of Dental Materials, By Kenneth J. Anusavice, Chiayi Shen, H. Ralph Rawls, 2014, page
no. 192)
Dental stone, high strength (Type 4):
• Modified α-hemihydrate, densite, or die stone
• Powder particles are very dense, cuboidal in shape, and has reduced surface area

Use:
For making casts or dies for crown, bridge, and inlay fabrication
Advantages:
• High strength
• Surface hardness
• Abrasion resistant
• Minimum setting expansion
19. Answer: B
20. Answer: B
21. Answer: B (Ref. Phillip’s Science of Dental Materials, By Kenneth J. Anusavice, Chiayi Shen, H. Ralph Rawls, 2014,
page no. 164)
The reaction between the mercaptan molecules and the lead peroxide is written as:
2R-SH + PbO2 → R-S-S-R + H2O + PbO
Thus, during the setting reaction, oxygen from the reactor reacts with the -SH groups of the mercaptan chains.
When that reaction occurs, there is an increase in chain length by linear polymerization through the terminal SH groups,
which results in increased viscosity of the impression material. Cross-linking between pendant SH occurs too, a reaction
that is essential for the development of elastic properties of the impression material.
During the above polymerization process, water forms as a byproduct. The formation of a byproduct during polymerization
is often referred to as condensation polymerization.
Because of the presence of water inside the polysulfide structure, the impression will shrink as water evaporates from the
impression.
Recovery from deformation that occurs during removal requires that sufficient time is allowed for the material to recover
sufficiently before the cast is poured. The recovery time for elastomeric impression materials varies from 30 minutes to
2 hours.

https://t.me/DentalBooksWorld
Chapter 11 • Dental Materials 1079

Because of water evaporation that occurs during storage, polysulfide impressions should be poured within 1/2 hour after
they were made.
At the same time, because of slow recovery time of polysulfides, one should wait 30 minutes from impression making
before the impression is poured.
22. Answer: B
23. Answer: B
24. Answer: C
25. Answer: A
26. Answer: C (Ref. Phillip’s Science of Dental Materials, By Kenneth J. Anusavice, Chiayi Shen, H. Ralph Rawls, 2014, page
no. 154)
The base material of c-silicones contains hydroxyl-terminated polydimethyl siloxane with carbonates as a filler in addition
to plasticizers and pigments. The catalyst contains tin octoate and tetraethyl orthosilicate.
The setting reaction of a c-silicone can be described as:
2HOSi(CH3)2---Si(CH3)2OH + H5C2OSi(CH3)2OC2H5 + Sn-based catalyst → HOSi(CH3)---
Si(CH3)2OSi(CH3)2OSi(CH3)2----Si(CH3)2OH + 2 C2H5OH
During the above polymerization reaction, a three-dimensional network is formed and ethanol is released as a byproduct.
It is the evaporation of ethanol that explains why c-silicones shrink faster over time than polysulfides.
These base materials are supplied as pastes or putties, while the catalyst is in the form of a liquid.
A drawback with c-silicones is that the catalyst has a rather short shelf-life time.
27. Answer: D
28. Answer: D (Ref. Phillip’s Science of Dental Materials, By Kenneth J. Anusavice, Chiayi Shen, H. Ralph Rawls, 2014, page
no. 160)
The base material consists of a low molecular polysiloxane, fillers, plasticizers, and pigments. The reactor paste contains
low molecular weight chains with vinyl groups at the chain ends. The reactor paste also contains fillers, plasticizers, and
pigments.
During setting, the vinyl groups react with the silanol groups of the base material. During the reaction, chain transfer of
hydrogen occurs, something that can cause release of hydrogen gas under the worst circumstances. The latter, though, is
rarely seen in modern a-silicones.
Because of the addition polymerization process, shrinkage does not occur after the material has set. A-silicones are
therefore known to be very dimensional stable.
• ANSI/ADA Specification No. 19 - 2003
–– This specification is for non-aqueous elatsic materials.

Classification
The materials covered by this specification are classified according to consistencies determined immediately after
completion of mixing according to manufacturer’s instructions.
• Type 0: Putty consistency
• Type 1: Heavy-bodied consistency
• Type 2: Medium-bodied consistency
• Type 3: Light-bodied consistency
29. Answer: B
30. Answer: D (Ref. Phillip’s Science of Dental Materials, By Kenneth J. Anusavice, Chiayi Shen, H. Ralph Rawls, 2014, page
no. 55)
• Elasticity – The ability of a material to recovers its shape completely after deformation from an applied force
• Stiffness – A material’s resistance to deformation
• Proportional limit – The greatest stress a structure can withstand without permanent deformation
• Resilience – The resistance of a material to permanent deformation
• Toughness – The ability of a material to resist fracture
31. Answer: C
32. Answer: D
33. Answer: B
34. Answer: B

https://t.me/DentalBooksWorld
1080 Triumph's Complete Review of Dentistry

35. Answer: D (Ref. Basic Dental Materials, By John J. Manappallil, 2015, page no. 269)
• Setting time is best regulated in Alginate by – Amount of retarder added (not by controlling temperature).
• The increase in the temperature, the shorter the setting time – For 10° rise in temperature there is 1 minute reduction
in setting time.
• Always water is added to the bowl first, because if powder is added first then the penetration of water is inhibited and
greater mixing time is required.
• Small amount of gypsum left in the bowl can accelerate the set.
36. Answer: C (Ref. Phillip’s Science of Dental Materials, By Kenneth J. Anusavice, Chiayi Shen, H. Ralph Rawls, 2014, page
no. 176)

Based on setting mechanism Inelastic/rigid materials Elastic materials


Irreversible/chemical reaction Plaster of Paris and zinc oxide Alginate
eugenol (cannot be used in undercuts) Elastomeric impression materials
Reversible/physical reaction Impression compound Agar (used to reproduce
undercuts/interproximal spaces)

37. Answer: B
38. Answer: D
39. Answer: B
40. Answer: D
41. Answer: B
42. Answer: C
43. Answer: A
44. Answer: C
45. Answer: B
46. Answer: C (Ref. Phillip’s Science of Dental Materials, By Kenneth J. Anusavice, Chiayi Shen, H. Ralph Rawls, 2014, page
no. 164)
The manual dual-arch (or triple tray) technique is an efficient way of making a definitive impression and making temporary
provisional restorations for crown and bridge procedures. This article demonstrates how a dental assistant can assist a
dentist during triple tray crown and bridge procedures.
This is the first part of a two-part article, and explains mixing impression materials, making preoperative dual-arch
impressions, deciding in what situations to use separate arch impressions instead of the triple tray, and introduces the topic
of making provisional restorations.
In the future, in-office CAD–C AM scanning and milling of restorations, or digital scanning of tooth preparations,
followed by e-mailing the scans to a dental laboratory, may reduce the frequency of use of the manual triple tray impression
technique.
47. Answer: A
48. Answer: B
49. Answer: D (Ref. Phillip’s Science of Dental Materials, By Kenneth J. Anusavice, Chiayi Shen, H. Ralph Rawls, 2014, page
no. 183)
Factors that control setting time:
a. Factors controlled by the operator:
1. W:P ratio
• The more the W:P ratio, the fewer the nuclei per unit volume, so prolonged setting time.
2. Mixing time
• Within practical limits, longer and rapid mixing leads to shorter setting time.
• Some gypsum crystals form immediately when the plaster comes in contact with water and as the mixing begins,
formation of crystals increases.
• Some crystals are also broken up by mixing spatula and are distributed resulting in the formation of more nuclei of
crystallization resulting in decreased setting time.
Effect of W:P ratio and mixing time on the setting time of plaster of Paris

https://t.me/DentalBooksWorld
Chapter 11 • Dental Materials 1081

b. Factors controlled by the manufacturer:


1. Bythe addition of accelerators and retarders:
Accelerators:
• Gypsum (<20%) – Decreases setting time.
• The set gypsum used as an accelerator is called “Terra Alba.”
• Potassium sulfate (conc. 2–3%) and reduces the setting time of model plaster from approximately 10–4 minutes.
• Sodium chloride (<28%)
Retarders:
• Organic materials – Glue, gelatin and some gums.
• Potassium citrate, borax, sodium chloride (20%), sodium citrate.
50. Answer: C
51. Answer: B (Ref. Phillip’s Science of Dental Materials, By Kenneth J. Anusavice, Chiayi Shen, H. Ralph Rawls, 2014, page
no. 194)
Mixing time: The time from the addition of powder to the water until the mixing is completed.
Mechanical mixing – 20–30 seconds.
Hand spatulation – 1 minute.
Working time: The time available to use a workable mix.
Working time – 3 minutes.
Loss of gloss test for initial set: Some of the excess water is taken up in forming the dihydrate so that the mix loses its gloss.
Initial Gillmore test for initial set: The mixture is spread out, and the needle is lowered onto the surface. The time at which
it no longer leaves an impression is called the “Initial Set.”
This is marked by a definite increase in strength.
Vicat test for setting time:
Vicat penetrometer is used.
The needle with a weighed plunger rod is supported and held just in contact with the mix. After the gloss is lost, the
plunger is released.
The time elapsed until the needle no longer penetrates to the bottom of the mix is known as the “Setting Time.”
Gillmore test for final setting time:
Heavier Gillmore needle is used.
The time elapsed at which this needle leaves only a barely perceptible mark on the surface is called the “Final Setting Time”
(30–45 minutes).
52. Answer: C
53. Answer: D
54. Answer: D (Ref. Phillip’s Science of Dental Materials, By Kenneth J. Anusavice, Chiayi Shen, H. Ralph Rawls, 2014,
page no. 424)

Ingredient Functions
Feldspar (naturally occurring minerals composed of It is the lowest fusing component, which melts first and flows
potash [K2O], soda [Na2O], alumina, and silica). during firing, initiating these components into a solid mass

Silica (quartz) • Strengthens the fired porcelain restoration


• Remains unchanged at the temperature normally used in firing
porcelain and thus contribute stability to the mass during
heating by providing framework for the other ingredients
Kaolin (Al2O3.2SiO2.2H2O – Hydrated • Used as a binder
aluminosilicates) • Increases moldability of the unfired porcelain
• Imparts opacity to the finished porcelain product
Glass modifiers, e.g., K, Na, or Ca oxides or basic oxides They interrupt the integrity of silica network and acts as flux
Color pigments or frits, e.g., Fe/Ni oxide, Cu oxide, To provide appropriate shade to the restoration
MgO, TiO2, and Co oxide
Zr/Ce/Sn oxides and uranium oxide To develop the appropriate opacity

https://t.me/DentalBooksWorld
1082 Triumph's Complete Review of Dentistry

55. Answer: C
56. Answer: D
57. Answer: C (Ref. Phillip’s Science of Dental Materials, By Kenneth J. Anusavice, Chiayi Shen, H. Ralph Rawls, 2014, page
no. 319)
The initial low pH that glass ionomers exhibit contributes to postoperative sensitivity.
However, the advantages of chemically bonding to tooth structure, its bacteriostatic effect, fluoride release, and adequate
compressive and tensile strength make this an acceptable cement.
Glass-ionomer cements are still used today, but their use has seen a slight decline because they yield retention rates
comparable to zinc phosphate.
58. Answer: B
59. Answer: A (Ref. Phillip’s Science of Dental Materials, By Kenneth J. Anusavice, Chiayi Shen, H. Ralph Rawls, 2014, page
no. 251)
The exact composition of a particular toothpaste varies with each manufacturer, but a typical formulation is abrasive
10–40%, humectant 20–70%, water 5–30%, binder 1–2%, detergent 1–3%, flavor 1–2%, preservative 0.05–0.5%, and
therapeutic agent 0.1–0.5%.
Humectants are used in dentifrices to prevent loss of water and subsequent hardening of the paste when it is exposed to air.
The most commonly used humectants are glycerol and sorbitol. Binders are hydrophilic colloids which disperse or swell in
the presence of water and are used to stabilize toothpaste formulations by preventing the separation of the solid and liquid
phases. Examples of binding agents used in toothpaste include the natural gums, the seaweed colloids (alginates, Irish
moss extract, and gum carrageenan), and synthetic celluloses (carboxymethyl cellulose, hydroxyethyl cellulose), with the
latter now being used increasingly for economic reasons.
60. Answer: B
61. Answer: C
62. Answer: D
63. Answer: A
64. Answer: D
65. Answer: C
66. Answer: B
67. Answer: C
68. Answer: D
69. Answer: C (Ref. Applied Dental Materials, By John F. McCabe, Angus W. G. Walls, 2013)
As the dihydrate precipitates, the solution is no longer saturated with the hemihydrate, so it continues to dissolve.
Dissolution of hemihydrate and precipitation of dihydrate as either new crystals or further growth on the already present
ones. The reaction continues until no further dihydrate precipitates out of solution.
• As the gypsum forming increases, mass hardens into needle-like clusters called spherulites.
• The intermeshing and entangling of crystals lead to a strong, solid structure.
70. Answer: C
71. Answer: B
72. Answer: D
73. Answer: A
74. Answer: A
75. Answer: B
76. Answer: C
77. Answer: A
78. Answer: C
79. Answer: A
80. Answer: B
81. Answer: B
82. Answer: A
83. Answer: B
84. Answer: C

https://t.me/DentalBooksWorld
Chapter 11 • Dental Materials 1083

85. Answer: B
86. Answer: C (Ref. Phillip’s Science of Dental Materials, By Kenneth J. Anusavice, Chiayi Shen, H. Ralph Rawls, 2014, page
no. 125)
Estrogenicity—Potential of synthetic chemicals with a binding affinity for estrogen receptors to cause reproductive
alterations. Bisphenol- A, a precursor of certain monomers such as bis-GM A, is a known estrogenic compound that is
considered to have possible effects on fetal and infant brain development and behavior.
87. Answer: C
88. Answer: A (Ref. Phillip’s Science of Dental Materials, By Kenneth J. Anusavice, Chiayi Shen, H. Ralph Rawls, 2014, page
no. 291)
When combined with new curing techniques, advanced resin–cement systems that provide an ideal viscosity level, such as
RelyX Veneer cement and Insure cement, allow dentists to initially place the veneers, then remove excess cement from the
gingival and interproximal margins before final polymerization, without the veneers drifting.
89. Answer: B (Ref. Phillip’s Science of Dental Materials, By Kenneth J. Anusavice, Chiayi Shen, H. Ralph Rawls, 2014, page
no. 449)
Infiltrated ceramics are made through a process called slip-casting, which involves the condensation of an aqueous
porcelain slip on a refractory die. This fired porous core is later glass infiltrated, a process by which molten glass is drawn
into the pores by capillary action at high temperatures.
In-Ceram spinell
Spinell (MgAl2O4) is a natural mineral that is normally found together with limestone and dolomite.
It is of dental significance because of its extremely high melting point (2,135° C) combined with its high strength. Spinell
is also chemically inert and has low electrical and thermal conductivity but, most importantly, it has unique optical
properties. It has moderate strength of about 350 MPa and good translucency.
It is more than twice as translucent as In-Ceram alumina due to the refractive index of its crystalline phase being close to
that of glass.
Glass infiltrating in a vacuum environment results in less porosity, ensuring this high level of translucency. Often, however,
this level of translucency can be excessive and can lead to an overly glassy, low-value appearance.
In-Ceram alumina
Aluminum oxide (Al2O3) is most widely known under the term corundum. As a result of the homogeneous framework
structure made of ultrafine Al2O3 particles, whose cavities are filled with a special glass, the degree of tensile bending
strength is significantly higher than that of all other ceramic systems.
With a weight percentage of 10–20%, aluminum oxide is a component of feldspar, which is the starting material for metal–
ceramic veneering materials.
90. Answer: B
91. Answer: C
92. Answer: B
93. Answer: C
94. Answer: B (Ref. Phillip’s Science of Dental Materials, By Kenneth J. Anusavice, Chiayi Shen, H. Ralph Rawls, 2014, page
no. 247)
Finishing and polishing

Restorations Polishing time


Amalgam After 24 hours, by using tin oxide, iron oxide
Composite A delay of 10 minutes or more is recommended
after curing to allow complete polymerization
Gold alloys Use slow speed hand piece, contour with
carbide burs and green stones, finish with
pink stone (aluminum oxide)

95. Answer: B

https://t.me/DentalBooksWorld
1084 Triumph's Complete Review of Dentistry

PREVIOUS YEAR QUESTIONS


1. Answer: A
2. Answer: D
3. Answer: B
4. Answer: D
5. Answer: D
6. Answer: D
7. Answer: C
8. Answer: B
9. Answer: D
10. Answer: D
11. Answer: C
12. Answer: D
13. Answer: C
14. Answer: B
15. Answer: B
16. Answer: D
17. Answer: C
18. Answer: A
19. Answer: D
20. Answer: A
21. Answer: D
22. Answer: B
23. Answer: B
24. Answer: C
25. Answer: D
26. Answer: A
27. Answer: C
28. Answer: A
29. Answer: A
30. Answer: A
31. Answer: B
32. Answer: B
33. Answer: B
34. Answer: A
35. Answer: D
36. Answer: B
37. Answer: C
38. Answer: A
39. Answer: B
40. Answer: D
41. Answer: B
42. Answer: D
43. Answer: B
44. Answer: C
45. Answer: B
46. Answer: B
47. Answer: C
48. Answer: D
49. Answer: B
50. Answer: C

https://t.me/DentalBooksWorld
Chapter 11 • Dental Materials 1085

51. Answer: D
52. Answer: B
53. Answer: D
54. Answer: C
55. Answer: A
56. Answer: D
57. Answer: B
58. Answer: B
59. Answer: A
60. Answer: C
61. Answer: C
62. Answer: A
63. Answer: A
64. Answer: D
65. Answer: B
66. Answer: C
67. Answer: D
68. Answer: D
69. Answer: B
70. Answer: B
71. Answer: B
72. Answer: C
73. Answer: B

RECENT PREVIOUS YEAR QUESTIONS


1. Answer: B (Ref. Phillip’s Science of Dental Materials, By Kenneth J. Anusavice, Chiayi Shen, H. Ralph Rawls, 2014, page
no. 342)
• Spherical alloys require less mercury than typical lathe-cut alloys because spherical alloys have a lower surface area per
volume.
• Spherical amalgam alloys have more surface area and require less mercury for trituration.
• They set faster as mercury content is reduced and least resistant to condensation forces.
• For lathe-cut alloys, greater condensation pressure is used while spherical amalgams are condensed with lighter
pressure.
• In lathe-cut amalgam alloys due to decreased mercury content, plasticity of amalgam decreases and high force of
condensation is required to properly adapt it to cavity walls.
• However, this is not a problem with spherical amalgam alloys.
• Ideally mercury/alloy ratio in lathe-cut alloy is 1:1. Maximum ratio of mercury/alloy is 8:5.
2. Answer: D (Ref. Phillip’s Science of Dental Materials, By Kenneth J. Anusavice, Chiayi Shen, H. Ralph Rawls, 2014, page no. 80)
The majority of noble metal casting alloys for dental restorations are based on solid solutions. An important example is the
palladium–silver (Pd–Ag) alloy system.
Solid solution (metallic)
• A solid crystalline phase containing two or more elements of at least one of which is a metal and whose atoms share the
same crystal lattice.
• The majority of noble metal casting alloys for dental restorations are based on solid solutions.
An important example is the palladium–silver (Pd–Ag) alloy system.
• In palladium–silver (Pd–Ag) alloys, the two metals are completely soluble in all proportions and the same type of
crystal structure occurs throughout the alloy system (i.e., all compositions of the two elements).
• When this alloy is solidified, the silver atoms are distributed randomly in the FCC structure of pure palladium, thereby
forming a solid solution.
• Because the silver atoms become integrated into the crystal structure of palladium, no microstructural features can be
mechanically separated into two or more phases, so only one solid phase is present.

https://t.me/DentalBooksWorld
1086 Triumph's Complete Review of Dentistry

• Most gold alloys used in clinical dentistry are predominantly solid solutions, although they usually contain more than
two metals.
• When two metals are not completely soluble in each other, the solid state is a mixture of two or more phases. Important
examples are the eutectic alloys and peritectic alloys.
3. Answer: C (Ref. Phillip’s Science of Dental Materials, By Kenneth J. Anusavice, Chiayi Shen, H. Ralph Rawls, 2014, page no. 493)
• Plasticizing compounds, known as plasticizers, are often added to resins to reduce their softening or melting/fusion
temperatures.
• It is possible to make a resin that is normally hard and stiff at room temperature, flexible and soft by adding a plasticizer.
• For example, plastic water pipe made from polyvinylchloride (PVC) is hard and rigid and contains very little plasticizer,
whereas PVC water-line tubing is soft and elastic and contains a very high level of plasticizer.
• In recent years certain plasticizers, particularly phthalates, have received public attention as a potential ingestion hazard
in soft plastics used in infants and children’s toys.
• A plasticizer acts to partially neutralize secondary bonds or intermolecular attractions that normally prevent the resin
chains from slipping past one another (i.e., undergoing plastic flow) when the material is stressed.
• In some cases, this action is analogous to that of a solvent, with the plasticizing agent penetrating between the
macromolecules and increasing the intermolecular spacing.
• This type of plasticizer is referred to as an external plasticizer because it is not a part of the polymer’s structure.
• Its molecular attraction to the polymer should be extremely high so that it does not volatilize or leach out during the
fabrication or subsequent use of the resin. Such a condition is seldom realized in practice, so this type of plasticizer is
used sparingly in dental resins.
4. Answer: A (Ref. Phillip’s Science of Dental Materials, By Kenneth J. Anusavice, Chiayi Shen, H. Ralph Rawls, 2014, page no. 41)
• Tin chlorides and oxides are the corrosion products of low-copper alloys whereas copper oxide is the corrosion product
of high-copper alloys.
• It also occurs due to formation of oxides, sulfides, or chlorides.
• Gamma-2 phase is implicated in both marginal failures and active corrosion in traditional alloys.
• Oxides and chlorides of tin are the corrosion products in traditional alloys. These are present at the tooth–amalgam
interface and penetrate the bulk of restoration.
• Corrosion products containing copper are found in high-copper amalgams.
5. Answer: C (Ref. Craig’s Restorative Dental Materials, By Ronald L. Sakaguchi, John M. Powers, 2012, page no. 192)
• Hydroquinone is added as an inhibitor, which prevents undesirable polymerization or “setting” of the liquid during
storage.
• Polymethyl methacrylate resin systems include powder and liquid components.
• The powder consists of prepolymerized spheres of polymethyl methacrylate and a small amount of benzoyl peroxide,
termed the initiator, which is responsible for starting the polymerization process.
• The liquid is predominantly nonpolymerized methyl methacrylate monomer with small amounts of hydroquinone.
Hydroquinone is added as an inhibitor, which prevents undesirable polymerization or “setting” of the liquid during storage.
• Inhibitors also retard the curing process and thereby increase working time.
• A cross-linking agent can also be added to the liquid.
• Glycol dimethacrylate is used commonly as a cross-linking agent in polymethyl methacrylate denture base resins.
• Glycol dimethacrylate is chemically and structurally similar to methylmethacrylate. Therefore, it can be incorporated
into growing polymer chains.
• One should note that methyl methacrylate possesses one carbon–carbon double bond per molecule and glycol
dimethacrylate possesses two double bonds per molecule.
• As a result, an individual molecule of glycol dimethacrylate can participate in the polymerization of two separate
polymer chains that unite the two polymer chains. If sufficient glycol dimethacrylate is included in the mixture, several
interconnections can be formed and solvent swelling may occur, such as that caused by exposure to ethanol in alcoholic
beverages.
• These interconnections yield a net-like structure that provides increased resistance to deformation.
6. Answer: D (Ref. Craig’s Restorative Dental Materials, By Ronald L. Sakaguchi, John M. Powers, 2012, page no. 76)
• Condensation reaction – A polymerization process in which bifunctional or multifunctional monomers react to form
first dimers, then trimers, and eventually long-chain polymers; the reactions may or may not yield byproducts; the
preferred term is step-growth polymerization.
• All condensation impression materials yield byproducts.

https://t.me/DentalBooksWorld
Chapter 11 • Dental Materials 1087

• The reaction starts at the beginning of mixing and reaches its maximum rate soon after spatulation is complete.
• At this stage, a resilient network has started to form.
• During the final set, a material of adequate elasticity and strength is formed that can be removed past undercuts quite
readily.
• Moisture and temperature have a significant effect on course of the reaction.
• In particular, hot and humid conditions will accelerate the setting of polysulfide impression material. The reaction
yields water as a byproduct. Loss of this small molecule from the set material has a significant effect on the dimensional
stability of the impression.
7. Answer: A (Ref. Phillip’s Science of Dental Materials, By Kenneth J. Anusavice, Chiayi Shen, H. Ralph Rawls, 2014, page
no. 51)
To calculate compressive stress, the applied force is divided by the cross-sectional area perpendicular to the axis of the
applied force.
Compressive stress
When a body is placed under a load that tends to compress or shorten it, the internal resistance to such a load is called
a compressive stress. A compressive stress is associated with a compressive strain. To calculate compressive stress, the
applied force is divided by the cross-sectional area perpendicular to the axis of the applied force.
Shear stress
This type of stress tends to resist the sliding or twisting of one portion of a body over another. Shear stress can also be
produced by a twisting or torsional action on a material.
Flexural (bending) stress
Examples of flexural stresses produced in a three-unit fixed dental prosthesis (FDP) and a two-unit cantilever. These
stresses are produced by bending forces in dental appliances in one of two ways: (1) By subjecting a structure such as an
FDP to three-point loading, whereby the endpoints are fixed and a force is applied between these endpoints and (2) By
subjecting a cantilevered structure that is supported at only one end to a load along any part of the unsupported section.
Tensile stress
A tensile stress is always accompanied by tensile strain, but it is very difficult to generate pure tensile stress in a body, that
is a stress caused by a load that tends to stretch or elongate a body.
8. Answer: B (Ref. Phillip’s Science of Dental Materials, By Kenneth J. Anusavice, Chiayi Shen, H. Ralph Rawls, 2014, page
no. 171)
Usually, the manufacturers make both fast-setting alginate (1.5–3 minutes) and normal-setting alginate (3–4.5 minutes) to
give clinicians a choice of the materials that best suit their working style.
9. Answer: A (Ref. Phillip’s Science of Dental Materials, By Kenneth J. Anusavice, Chiayi Shen, H. Ralph Rawls, 2014, page
no. 406)
Carbon s teels can be described simply as iron–carbon binary alloys that contain less than 2.1% of carbon by weight.
– Phillips
Iron–carbon system
Carbon steels can be described simply as iron–carbon binary alloys that contain less than 2.1% of carbon by weight.
At room temperature, pure iron has a body-centered-cubic (bcc) structure. This phase is stable up to 912°C, where it
transforms to a face-centered-cubic (fcc) structure.
The solubility of carbon in bcc is very low and reaches a maximum of 0.02% at 723°C. This material is known as ferrite.
Above 723°C, a solid solution of carbon in an fcc iron matrix called austenite is formed. The maximal solubility of carbon
in fcc matrix is 2.1%.
When a plain carbon steel containing 0.8% carbon is cooled slowly in the austenitic phase to 723°C, it undergoes a solid-
state eutectoid transformation to yield a microstructural constituent called pearlite, which consists of alternating fine-scale
lamellae of ferrite and iron carbide (Fe3C), referred to as cementite, or simply carbide.
10. Answers: C and A (Ref. Phillip’s Science of Dental Materials, By Kenneth J. Anusavice, Chiayi Shen, H. Ralph Rawls, 2014,
page no. 316)
This is a tricky question. This question is specifically asked about which is increased with prolonged spatulation. Only the
hygroscopic expansion is increased with increased spatulation.
The hygroscopic setting expansion is a physical phenomenon and is not caused by a chemical reaction any more than
is the normal setting expansion, which will have the same effect on the hygroscopic setting expansion. For example, a
reduction in the W/P ratio and increased spatulation will result in a higher hygroscopic expansion.
Within practical limits an increase in the amount of spatulation (either speed of spatulation or time or both) shortens the
setting time.

https://t.me/DentalBooksWorld
1088 Triumph's Complete Review of Dentistry

The mixing process, called spatulation, has a definite effect on the setting time and setting expansion of the material.
Within practical limits an increase in the amount of spatulation (either speed of spatulation or time or both) shortens the
setting time.
Obviously when the powder is placed in water, the chemical reactions starts, and some calcium sulfate dehydrate is formed.
During spatulation the newly formed calcium sulfate dehydrate breaks down to smaller crystals and starts new centers of
nucleation, from which the calcium sulfate dehydrate can be precipitated. Because an increased amount of spatulation
causes more nuclei centers to be formed, the conversion of calcium sulfate hemihydrate to dehydrate is accelerated.
Summary of Effect of Manipulative Variables on Properties of Gypsum Products

Manipulative variable Setting time Consistency Setting expansion Compressive strength


Increase water/powder ratio Increase Increase Decrease Decrease
Increase rate of spatulation Decrease Decrease Increase No effect
Increase temperature of
Decrease Decrease Increase No effect
mixing water from 23° to 30°

11. Answer: A (Ref. Phillip’s Science of Dental Materials, By Kenneth J. Anusavice, Chiayi Shen, H. Ralph Rawls, 2014, page
no. 258)
For perfect wetting, which is the ideal situation for adhesion to occur, this angle should be 0°.
Glass 14
Amalgam 77
Acrylic filling material 38
Composite filling material 51

12. Answer: A (Ref. Craig’s Restorative Dental Materials, By Ronald L. Sakaguchi, John M. Powers, 2012, page no. 94)
Vicat test for setting time
The next stage in the reaction is determined by the use of another instrument, the Vicat penetrometer, seen on the left. The
needle with a weighted plunger rod is supported and held just in contact with the mix. Soon after the gloss is lost, the plunger
is released. The time elapsed until the needle no longer penetrates to the bottom of the mix is known as the setting time. In
some cases, the Vicat and initial Gillmore occurs at the same time, whereas in other instances, there is 1’ small difference.
Gillmore test for final setting time
The next stage in the setting process is measured by the use of the heavier Gillmore needle. The elapsed time at which this
needle leaves only a barely perceptible mark on the surface is called the final setting time.
13. Answer: C (Ref. Phillip’s Science of Dental Materials, By Kenneth J. Anusavice, Chiayi Shen, H. Ralph Rawls, 2014, page no. 185)

Gypsum Products
Type I Type II Type III Type IV Type V
Other name Impression Model plaster Dental stone Class II dental stone Dental stone with
plaster or hydrocal improved dental high strength and
or class I stone or densite or high expansion on
stone high strength dental extra hard, improved
stone dental stone
W/P ratio 0.5–0.75 0.45–0.50 0.28–0.30 0.22–0.24 0.18–0.22
Setting time 4–5 minutes 12–15 minutes 12–15 12–15 minutes 12–15 minutes
minutes
Setting expansion 0.15% 0.30% 0.20% 0.10% 0.30%
Compressive 800 1,300 3,000 5,000 7,000
strength (PSI units)
Uses Final with To fill the flask Construction Die material Improve die material
final wash in denture of casts
impressions construction

https://t.me/DentalBooksWorld
Chapter 11 • Dental Materials 1089

14. Answer: C (Ref. Phillip’s Science of Dental Materials, By Kenneth J. Anusavice, Chiayi Shen, H. Ralph Rawls, 2014, page
no. 347)
Originally, the alloy and mercury were mixed by hand with a mortar and pestle. Today mechanical amalgamations save
time and standardize the procedure.
For mechanical trituration in an amalgamator, disposable capsules containing proportional aliquots of mercury and alloy
are now widely used.
They contain alloy either in pellet form or as a preweighed portion of powder in conjunction with appropriate quantity
of mercury. Provided that the same weights of alloy and mercury are used each time and are triturated by the same
amalgamator, attainment of a proper mix can be controlled by timing the trituration.
15. Answer: C
16. Answer: A (Ref. Phillip’s Science of Dental Materials, By Kenneth J. Anusavice, Chiayi Shen, H. Ralph Rawls, 2014, page
no. 180)
Rough or chalky stone cast:
• Inadequate cleaning of impression
• Excess water that is not blown off of the impression
• Excess wetting agent left on impression
• Premature removal of cast; improper powder/water ratio of stone
• Failure to delay pour of addition silicone that does not contain a palladium salt for at least 20 minutes
17. Answer: A (Ref. Phillip’s Science of Dental Materials, By Kenneth J. Anusavice, Chiayi Shen, H. Ralph Rawls, 2014, page
no. 20)
Any molecule with atoms of different electronegativity is joined by a covalent bond. The electrons are drawn toward the
more electronegative of the two atoms giving that part of the molecule a partial –ve charge and leaving the other part of
the molecule with a partial +ve charge. E.g., H2O, HCl, NH3.
Polar molecules are said to be permanent dipoles and have a permanent dipole. Examples of polar molecules with a net
charge of zero include water and carbon dioxide.
18. Answer: B (Ref. Phillip’s Science of Dental Materials, By Kenneth J. Anusavice, Chiayi Shen, H. Ralph Rawls, 2014, page
no. 18)
Normally all the substances exist in solid, liquid, and gaseous states. Any substance can occur in one or more of the three
states and often it is possible to convert from one state to another.
When water boils it is transformed into vapor. This transformation requires energy and this quantity of energy is called
latent heat of vaporization. The latent heat of vaporization is defined as the amount of heat needed to evaporate 1 g of
liquid to vapor state at a given temperature and pressure. An energy of 540 Cal is required to vaporize 1 g of water at 100°C
and a pressure at 1 atmosphere.
19. Answer: C (Ref. Phillip’s Science of Dental Materials, By Kenneth J. Anusavice, Chiayi Shen, H. Ralph Rawls, 2014, page
no. 419)
Pigmenting oxides are added to obtain the various shades needed to simulate natural teeth. These coloring pigments are
produced by fusing metallic oxides together with fine glass and feldspar and then regrinding to a powder. These powders
are blended with the unpigmented powdered frit to provide the proper hue and chroma. Examples of metallic oxides and
their respective color contributions to porcelain include:
Iron or nickel oxide (brown)
Copper oxide (green)
Titanium oxide (yellowish brown)
Manganese oxide (lavender)
Cobalt oxide (blue)
Opacity may be achieved by the addition of cerium oxide, zirconium oxide, titanium oxide, or tin oxide.
20. Answer: B (Ref. Phillip’s Science of Dental Materials, By Kenneth J. Anusavice, Chiayi Shen, H. Ralph Rawls, 2014, page
no. 321)
“Boric acid is a flux, and is hence not an antiflux.”
Antiflux –A substance such as graphite that prevents flow of molten solder on areas coated by the substance.
“Flux for use with noble metal alloys are generally based on boric or borate compounds such as boric acid, boric anhydrate,
and borax; they act as protective fluxes.”
Type I, by forming, is to eliminate any oxide coating on the substrate metal surface when the filler metal is molten and
ready to flow into place.

https://t.me/DentalBooksWorld
1090 Triumph's Complete Review of Dentistry

Classification:
Fluxes may be divided into the following three types, according to their primary purpose.
1. (Type I) Surface protection
Covers the metal surface and prevents access to oxygen so that no oxides can form.
2. (Type II) Reducing agent
Reduces any oxides present and exposes clean metal.
3. (Type III) Solvent dissolves any oxides present and carries them away.
21. Answer: D (Ref. Phillip’s Science of Dental Materials, By Kenneth J. Anusavice, Chiayi Shen, H. Ralph Rawls, 2014, page
no. 460)
The core microstructure of IPS EMPRESS 2 is quite different from that of IPS EMPRESS, as evidenced by elongated
lithia disilicate crystals 0.5–4 mm in length and a smaller concentration of lithium orthophosphate crystals (Li3PO4)
approximately 0.1–0.3 pm in diameter.
22. Answer: A (Ref. Phillip’s Science of Dental Materials, By Kenneth J. Anusavice, Chiayi Shen, H. Ralph Rawls, 2014, page
no. 442)
Cerestore (shrink-free ceramic system)
Indications
• For periodontally compromised patients
• Advantages
• Good flexural strength
• Highly esthetic
• Good marginal fit
23. Answer: A (Ref. Phillip’s Science of Dental Materials, By Kenneth J. Anusavice, Chiayi Shen, H. Ralph Rawls, 2014, page
no. 227)
Incomplete casting is due to incomplete filling of mold by molten alloys. The common causes are
• Insufficient venting
• High viscosity of metal due to insufficient heating
• Incomplete elimination of wax residues from the mold
24. Answer: D (Ref. Phillip’s Science of Dental Materials, By Kenneth J. Anusavice, Chiayi Shen, H. Ralph Rawls, 2014, page
no. 331)
Zinc oxide eugenol cements
• Unmodified ZOE cements are weak and have a long setting time.
• They tend to be absorbed over time because of high water solubility.
• On contact with moisture, this release free eugenol which is responsible for most of the effects caused by zinc oxide
eugenol cements.
Effects of free eugenol
• Competitive – Inhibit prostaglandin synthetase by penetrating biosynthesis of cyclooxygenase.
• Inhibits sensory nerve activity.
• Inhibits mitochondrial respiration.
• Kills a range of natural oral microorganisms
• Can act as an allergen.
25. Answer: A
26. Answer: C
27. Answer: C (Ref. Phillip’s Science of Dental Materials, By Kenneth J. Anusavice, Chiayi Shen, H. Ralph Rawls, 2014, page
no. 74)
Stainless steel resists tarnish and corrosion primarily because of passivating effect of chromium. A film of chromium oxide
also known as passivating film form on the surface and it protects the alloy from further oxidation and makes the alloy
stainless. Chromium also increases tensile strength, elastic limit, and hardness of alloy.
28. Answer: A (Ref. Phillip’s Science of Dental Materials, By Kenneth J. Anusavice, Chiayi Shen, H. Ralph Rawls, 2014, page
no. 316)
The pH of newly mixed zinc phosphate cement is under 2, but rises to 5.9 within 24 hours and is nearly neutral at 48 hours.
The pH of newly mixed zinc phosphate cement is under 2, but rises to 5.9 within 24 hours and is nearly neutral at 48 hours.
In usage tests in deep cavity preparations, moderate-to-severe localized pulpal damage is produced within 3 days, probably
because of the initial low pH (4.2 at 3 minutes). However, the pH of the set cement approaches neutrality after 48 hours.
By 5–8 weeks, only mild chronic inflammation is present and reparative dentin has usually formed.

https://t.me/DentalBooksWorld
Chapter 11 • Dental Materials 1091

pH of Cements for Luting Applications


Time (minutes) Zinc phosphate Zinc silicophosphate Zinc polycarboxylate Glass ionomer
2 2.14 1.43 3.42 2.33 1.76
5 2.55 1.74 3.94 3.26 1.98
10 3.14 2.15 4.42 3.78 3.36
15 3.30 2.46 4.76 3.91 3.88
20 3.62 2.56 4.87 3.98 4.19
30 3.71 2.79 5.03 4.18 4.46
60 4.34 3.60 5.08 4.55 4.84
1,440 5.50 5.55 5.94 5.67 5.98

29. Answer: C
30. Answer: D
31. Answer: C (Ref. Phillip’s Science of Dental Materials, By Kenneth J. Anusavice, Chiayi Shen, H. Ralph Rawls, 2014, page
no. 210)
“The use of ethyl silicate-bonded investment has declined because of the more complex and time-consuming procedures
involved with their use; however, they are still used in the construction of high-fusing base metal partial denture alloys.”
Ethyl silicate-bonded investment
The use of ethyl silicate-bonded investment has declined because of the more complex and time-consuming procedures
involved with their use; however, they are still used in the construction of high-fusing base metal partial denture alloys.
The reaction can be expressed follows:
Si (OC2H5)4 + 4H2O Si(OH)4 + 4 C2H5OH
Since an ethyl silicate polymer is used, a colloidal sol of polysilicic acid is expected instead of the simpler silicic acid sol
shown in the reaction.
The sol is then mixed with quartz or cristobalite, to which a small amount of finely powdered magnesium oxide is added to
render the mixture alkaline. A coherent gel of polysilicic acid is then formed, accompanied by a setting shrinkage.
This soft gel is dried at a temperature below 1,680° C. During the drying process, the gel loses alcohol and water to form a
concentrated, hard gel.
As might be expected, a volumetric contraction accompanies the drying, which reduces the size of the mold. This
contraction is known as green shrinkage, which is additive to the setting shrinkage.
Gelation is likely to slow and is time-consuming. A faster method for the production of the silica gel can be used.
Certain types of amines can be added to the solution of ethyl silicate so that hydrolysis and gelation occurs simultaneously.
The moderate enlargement before casting mostly compensate not only for the casting shrinkage of the metal but also for
the green shrinkage and the setting shrinkage of the investment.
32. Answer: C (Ref. Basic Dental Materials, By John. J. Manappallil, 2015, page no. 27)
Bingham flow: It occurs when shear stress shear rate plot is linear but has an interception in the shear stress axis. This is termed
as the yield stress; below this stress the material will not flow, e.g., clay suspensions in water, composite-filling materials.
Plastic: The liquid behaves like a rigid body until some minimum value of shear stress is reached.
Pseudoplastic liquid:
• In this case the viscosity varies according to the shear exerted.
• It is seen in single mix or monophase elastomeric impression material
33. Answer: B
34. Answer: C
35. Answer: B
36. Answer: B
37. Answer: B
38. Answer: C
39. Answer: A
Alginate has a high incidence of air porosity. This is the best answer to mark.
40. Answer: A
41. Answer: A
42. Answers: D

https://t.me/DentalBooksWorld
12 Conservative Dentistry

SYNOPSIS

INSTRUMENTS USED IN OPERATIVE DENTISTRY


Terminology and Classification
Categories
The hand instruments used in the dental operatory may be categorized as
1. Cutting (excavators, chisels, and others) or
2. Noncutting (amalgam condensers, mirrors, explorers, probes)

Design
Most hand instruments, regardless of use, are composed of three parts:
1. Handle
2. Shank
3. Blade
For many noncutting instruments, the part corresponding to the blade is termed nib. The end of the nib, or working surface,
is known as face.

a b c b a

a. Blade b. Shank c. Handle

https://t.me/DentalBooksWorld
Chapter 12 • Conservative Dentistry 1093

Blade
length Blade angle
(3) (4)
(1) Blade width

(2) Cutting edge


angle

Formula
(1) (2) (3) (4)

9 7.5 7 15

9 7.5 7 15 0.9 mm 7 mm

Optimal Anti-Rotational Design—The blade edge must not be off-axis by more than 1–2 mm. All dental instruments and
equipment need to satisfy this principle of balance.
Balance is accomplished by designing the angles of the shank so that the cutting edge of the blade lies within the projected
diameter of the handle and nearly coincides with the projected axis of the handle

Shank Angles
The functional orientation and length of the blade determine the number of angles in the shank necessary to balance the
instrument.
Black classified instruments on the basis of the number of shank angles as
1. Mon-angle (one)
2. Bin-angle (two)
3. Triple-angle (three)
• Instruments with small, short blades may be easily designed in mon-angle form while confining the cutting edge within
the required limit.
• Instruments with longer blades or more complex orientations may require two or three angles in the shank to bring the
cutting edge close to the long axis of the handle. Such shanks are termed contra-angled.

Black’s classification system of instruments


1. Function (e.g., scaler, excavator)
2. Manner of use (e.g., hand condenser)
3. Design of the working end (e.g., spoon excavator, sickle scaler)
4. Shape of the shank (e.g., mon-angle, bin-angle, contra-angle)

Formulas
Cutting instruments have formulas describing the dimensions and angles of the working end. These are placed on the handle
using a code of three or four numbers separated by dashes or spaces (e.g., 10–8.5–8–14)
The first number indicates the width of the blade or primary cutting edge in tenths of a millimeter (0.1 mm) (e.g., 10 = 1 mm)

https://t.me/DentalBooksWorld
1094 Triumph's Complete Review of Dentistry

The second number of a four-number code indicates the primary cutting edge angle, measured from a line parallel to the long
axis of the instrument handle in clockwise centigrade
The angle is expressed as a percent of 360 degrees (e.g., 85 = 85% × 360 degrees = 306 degrees). The instrument is positioned
so that this number always exceeds 50
If the edge is locally perpendicular to the blade, this number is normally omitted, resulting in a three-number code
The third number (second number of a three-number code) indicates the blade length in millimeters (e.g., 8 = 8 mm)
The fourth number (third number of a three-number code) indicates the blade angle, relative to the long axis of the handle in
clockwise centigrade (e.g., 14 = 50 degrees)

Bevels
Most hand cutting instruments have on the end of the blade a single bevel that forms the primary cutting edge. Two additional
edges, called secondary cutting edges, extend from the primary edge for the length of the blade
Bi-beveled instruments such as ordinary hatchets have two bevels that form the cutting edge

Secondary cutting edge

Primary
cutting
edge

Secondary cutting edge

Excavators
The four subdivisions of excavators are
1. Ordinary hatchets
2. Hoes
3. Angle-formers
4. Spoon

Instrument Uses
Spoon excavators • Used for removing caries and carving amalgam or direct wax pattern
• The blades are slightly curved, and the cutting edges are either circular or claw-like
• The circular edge is known as a discoid, whereas the claw-like blade is termed cleoid
• The shanks may be bin-angled or triple-angled to facilitate accessibility
Angle-former • It is used primarily for sharpening line angles and creating retentive features in dentin in preparation
for gold restorations
• It also may be used in placing a bevel on enamel margins
Hoe excavator • It has the primary cutting edge of the blade perpendicular to the axis of the handle
• This type of instrument is used for planing tooth preparation walls and for forming line angles. It is
commonly used in Class III and V preparations for direct gold restorations
Ordinary hatchet • An ordinary hatchet excavator has the cutting edge of the blade directed in the same plane as that of
the long axis of the handle and is bi-beveled
• These instruments are used primarily on anterior teeth for preparing retentive areas and sharpening
internal line angles, particularly in preparations for direct gold restorations

https://t.me/DentalBooksWorld
Chapter 12 • Conservative Dentistry 1095

Chisels
Chisels are intended primarily for cutting enamel and may be grouped as
1. Straight, slightly curved, or bin-angle
2. Enamel hatchets
3. Gingival margin trimmers
Straight chisel • The straight chisel has a straight shank and blade, with the bevel on only one side. Its primary edge is
perpendicular to the axis of the handle
The enamel • The enamel hatchet is a chisel similar in design to the ordinary hatchet except that the blade is larger,
hatchet heavier, and beveled on only one side
• It has its cutting edges in a plane that is parallel with the axis of the handle
• It is used for cutting enamel and comes as right or left types for use on opposite sides of the preparation
Gingival marginal • The gingival margin trimmer is designed to produce a proper bevel on gingival enamel margins of
trimmer proximo-occlusal preparations
• It is similar in design to the enamel hatchet except the blade is curved (similar to a spoon excavator),
and the primary cutting edge is at an angle (other than perpendicular) to the axis of the blade
• It is made as right and left types. It also is made so that a right and left pair is either a mesial pair or a distal
pair. When the second number in the formula is 90–100, the pair is used on the distal gingival margin
• When this number is 75–85, the pair is used to bevel the mesial margin
• The 100 and 75 pairs are for inlay–onlay preparations with steep gingival bevel
• The 90 and 85 pairs are for amalgam preparations with gingival enamel bevels that decline gingivally
only slightly
• Among other uses for these instruments is the rounding or beveling of the axiopulpal line angle of
two-surface preparations

Hand Instrument Techniques


Four grasps are used with hand instruments
1. Modified
2. Inverted
3. Palm-and-thumb
4. Modified palm-and-thumb
Modified Pen Grasp
• The grasp that permits the greatest delicacy of touch is the modified pen grasp
• As the name implies, it is similar, but not identical, to that used in holding a pen
• The pads of the thumb and of the index and middle fingers contact the instrument, while the tip of the ring finger (or tips
of the ring and little fingers) is placed on a nearby tooth surface of the same arch as a rest. The palm of the hand generally
is facing away from the operator
• The pad of the middle finger is placed near the topside of the instrument; by this finger working with the wrist and the
forearm, cutting or cleaving pressure is generated on the blade. The instrument should not be allowed to rest on or near the
first joint of the middle finger as in the conventional pen grasp
Inverted Pen Grasp
• The finger positions of the inverted pen grasp are the same as for the modified pen grasp
• The hand is rotated, however, so that the palm faces more toward the operator
• This grasp is used mostly for tooth preparations employing the lingual approach on anterior teeth.
Palm-and-Thumb Grasp
The palm-and-thumb grasp is similar to that used for holding a knife while paring an apple. The handle is placed in the palm
of the hand and grasped by all the fingers, while the thumb is free of the instrument, and the rest is provided by supporting
the tip of the thumb on a nearby tooth of the same arch or on a firm, stable structure. For suitable control, this grasp requires
careful use during cutting. An example of an appropriate use is holding a handpiece for cutting incisal retention for a Class
III preparation on a maxillary incisor

https://t.me/DentalBooksWorld
1096 Triumph's Complete Review of Dentistry

Modified Palm-and-Thumb Grasp


• The modified palm-and-thumb grasp may be used when it is feasible to rest the thumb on the tooth being prepared or the
adjacent tooth
• The handle of the instrument is held by all four fingers, whose pads press the handle against the distal area of the palm and
the pad and first joint of the thumb
• Grasping the handle under the first joints of the ring finger and little finger provides stabilization. This grip fosters control
against slippage
• The modified pen grasp and the inverted pen grasp are used practically universally. The modified palm-and-thumb grasp
usually is employed in the area of the maxillary arch and is best adopted when the dentist is operating from a rear-chair
position

Rotary Power Cutting Equipment


• Powered rotary cutting instruments, known as dental handpieces, are the most commonly used instruments in contemporary
dentistry
• The technologies for both air-driven and electric and air-driven systems have both advantages and disadvantages
–– Air-driven systems are less costly on initial startup and are less expensive with regard to replacing turbines compared
with electric handpieces. Air-driven handpieces weigh less than electric handpieces, and this quality may be the most
significant adjustment for clinicians who make the change from air-driven handpieces to electric handpieces
–– Electric handpieces are quieter than air-driven handpieces, they cut with high torque with very little stalling, they
maintain high bur concentricity, and they offer high-precision cutting. Cutting with electric handpieces is smoother and
more like milling, whereas cutting with the air-driven handpiece is more like chopping the tooth with the bur

Rotary Speed Ranges for Different Cutting Applications


The rotational speed of an instrument is measured in revolutions per minute (rpm)
Three speed ranges are generally recognized
• Low or slow speeds (<12,000 rpm)
• Medium or intermediate speeds (12,000–200,000 rpm)
• High or ultra-high speeds (>200,000 rpm)

RESTORATIONS—AMALGAM, GOLD, CAST METAL, COMPOSITES—PRINCIPLES


AND CONCEPTS
Amalgam Restorations
• Amalgam is used for the restoration of many carious or fractured posterior teeth and in the replacement of failed restorations
• Class I restorations restore defects on the occlusal surface of posterior teeth, the occlusal thirds of the facial and lingual
surface of molars, and the lingual surfaces of maxillary anterior teeth
• Class II restorations restore defects that affect one or both of the proximal surfaces of posterior teeth
• Class VI restorations restore rare defects affecting the cusp tips of posterior teeth or the incisal edges of anterior teeth
Indications
Amalgam is indicated for the restoration of a Class I, II, and VI defect when the defect
1. Is not in an area of the mouth where esthetics is highly important
2. Is moderate to large
3. Is in an area that will have heavy occlusal contacts
4. Cannot be well isolated
5. Extends onto the root surface
6. Will become a foundation for a full coverage restoration
7. Is in a tooth that serves as an abutment for a removable partial denture

https://t.me/DentalBooksWorld
Chapter 12 • Conservative Dentistry 1097

Contraindications
Although amalgam has no specific contraindications for use in Class I, II, and VI restorations, relative contraindications for
use include
1. Esthetically prominent areas of posterior teeth
2. Small to moderate Class I and II defects that can be well isolated
3. Small Class VI defects
Advantages
Primary advantages are the ease of use and the simplicity of the procedure. As noted in the following sections, the placing and
contouring of amalgam restorations are generally easier than those for composite restorations
Disadvantages
The primary disadvantages of using amalgam for Class I, II, and VI defects are (1) amalgam use requires more complex and
larger tooth preparations than composite resin, and (2) amalgams may be considered to have a nonesthetic appearance by
some patients

Tooth Preparation for Amalgam restoration


This section describes the specific technique for preparing the tooth for a conservative Class I amalgam restoration. It is
divided into initial and final stages.
Initial Tooth Preparation
• Initial tooth preparation is defined as establishing the outline form by extension of the external walls to sound tooth
structure while maintaining a specified, limited depth (usually just inside the dentinoenamel junction [DEJ]) and providing
resistance and retention forms
• A No. 245 bur with a head length of 3 mm and a tip diameter of 0.8 mm or a smaller No. 330 bur is recommended to prepare
the conservative Class I tooth preparation
• On posterior teeth, the approximate depth of the DEJ is located at 1.5–2 mm from the occlusal surface. As the bur enters the
pit, an initial target depth of 1.5 mm should be established
• This is one-half the length of the cutting portion of the No. 245 bur. The 1.5-mm pulpal depth is measured at the central
fissure
• Depending on the cuspal incline, the depth of the prepared external walls is 1.5–2 mm
• The depth of the preparation is modified as needed so that the pulpal wall is established 0.1–0.2 mm into dentin
• The length of the blades of an unfamiliar entry bur should be measured before it is used as a depth gauge
• A faciolingual width of no more than 1–1.5 mm and a depth of 1.5–2 mm are considered ideal, but this goal is subject to
the extension of the caries
The primary resistance form is provided by the following:
• Sufficient area of relatively flat pulpal floor in sound tooth structure to resist forces directed in the long axis of the tooth and to
provide a strong, stable seat for the restoration
• Minimal extension of external walls, which reduces weakening of the tooth
• Strong, ideal enamel margins
• Sufficient depth (i.e., 1.5 mm) that results in adequate thickness of the restoration, providing resistance to fracture and wear
• The parallelism or slight occlusal convergence of two or more opposing, external walls provides the primary retention form
• If the tooth preparation is of ideal or shallow depth, no liner or base is indicated. In deeper caries excavations (where the
remaining dentin thickness is judged to be 0.5–1 mm), a thin layer (i.e., 0.5–0.75 mm) of a light-activated, resin-modified
glass ionomer (RMGI) material should be placed
• It is important to provide an approximate 90- to 100-degree cavosurface angle, which should result in 80- to 90-degree
amalgam at the margins. This butt-joint margin of enamel and amalgam is the strongest for both. Amalgam is a brittle
material with low edge strength and tends to chip under occlusal stress if its angle at the margins is less than 80 degrees

Minimum 1.6 mm of marginal tooth structure is necessary (marginal width) in case of premolars and minimum of 2 mm of
marginal width is necessary in case of molars.

https://t.me/DentalBooksWorld
1098 Triumph's Complete Review of Dentistry

Other Conservative Class I Amalgam Preparations


• Facial pit of the mandibular molar
• Lingual pit of the maxillary lateral incisor
• Occlusal pits of the mandibular first premolar
• Occlusal pits and fissures of the maxillary first molar
• Occlusal pits and fissures of the mandibular second premolar

Restorative Technique for Class I Amalgam Preparations


Desensitizer Placement
• A dentin desensitizer is placed in the preparation before amalgam condensation
• The dentin desensitizer is applied onto the prepared tooth surface according to manufacturer’s recommendations; excess
moisture is removed without desiccating the dentin; and then the amalgam is condensed into place
Matrix Placement
• Generally, matrices are unnecessary for a conservative Class I amalgam restoration except as specified in later sections
Insertion and Carving of the Amalgam
• Because of its superior clinical performance, high-copper amalgam is recommended. The triturated amalgam is emptied
into an amalgam well
• Correctly mixed amalgam should not be dry and crumbly. It has a minimal, yet sufficient, “wetness” to aid in achieving a
homogeneous and well-adapted restoration
• The principal objectives during the insertion of amalgam are to condense the amalgam to adapt it to the preparation walls
and the matrix (when used) and produce a restoration free of voids. Thorough condensation helps to reduce marginal leakage
• Optimal condensation also is necessary to minimize the mercury content in the restoration to decrease corrosion and to
enhance strength and marginal integrity
• An excessive temperature increase (i.e., >140°F [>60°C]) can cause irreparable damage to the pulp, the restoration, or both.
• When overheated, the amalgam surface appears cloudy, even though it may have a high polish.
• This cloudy appearance indicates that mercury has been brought to the surface, which results in corrosion of the amalgam
and loss of strength

Principles of Tooth Preparation for Direct Gold Restorations


• The preparation must be smoothed and debrided to permit the first increments of gold to be stabilized.
• Resistance form is established by orienting preparation walls to support the integrity of the tooth, such as a pulpal wall that
is flat and perpendicular to occlusal forces.
• The retention form is established by parallelism of some walls and by strategically placed converging walls (as described in
detail for each tooth preparation).
• In addition, walls must be smooth and flat, where possible (to provide resistance to loosening of the gold during compaction),
and internal line angles must be sharp (to resist movement).
• Internal form includes an initial depth into dentin, ranging from 0.5 mm from the dentinoenamel junction (DEJ) in class I
preparations to 0.75 mm from cementum in Class V preparations.
• Optimal convenience form requires suitable access and a dry field provided by the rubber dam. Access additionally may
require the use of a gingival retractor for Class V restorations or a separator to provide a minimal amount of separation
(0.5 mm maximum) between anterior teeth for Class III restorations.
• Sharp internal line and point angles are created in dentin to allow convenient “starting” gold foil as compaction begins.
• Rounded form is permitted when E-Z Gold is used to begin the restorative phase.
• Removal of remaining carious dentin, final planing of cavosurface margins, and debridement complete the tooth preparation
for direct gold.

https://t.me/DentalBooksWorld
Chapter 12 • Conservative Dentistry 1099

Indications and Contraindications


• Class I direct gold restorations are one option for the treatment of small carious lesions in pits and fissures of most posterior
teeth and the lingual surfaces of anterior teeth.
• Direct gold also is indicated for treatment of small, cavitated Class V carious lesions or for the restoration, when indicated,
of abraded, eroded, or abfraction areas on the facial surfaces of teeth (although access to the molars is a limiting factor).
• Class III direct gold restorations can be used on the proximal surfaces of anterior teeth where the lesions are small enough
to be treated with esthetically pleasing results.
• Class II direct gold restorations are an option for restoration of small cavitated proximal surface carious lesions in posterior
teeth in which marginal ridges are not subjected to heavy occlusal forces (e.g., the mesial or distal surfaces of mandibular
first premolars and the mesial surface of some maxillary premolars).
• Class VI direct gold restorations may be used on the incisal edges or cusp tips. A defective margin of an otherwise acceptable
cast gold restoration also may be repaired with direct gold.
• Direct gold restorations are contraindicated in some patients whose teeth have very large pulp chambers, in patients with
severely periodontally weakened teeth with questionable prognosis, in patients for whom economics is a severely limiting
factor, and in handicapped patients who are unable to sit for the long dental appointments required for this procedure.
• Root canal-filled teeth are generally not restored with direct gold because these teeth are brittle, although in some cases gold
may be the material of choice to close access preparations (for root canal therapy) in cast gold restorations.

Tooth Preparations for Direct filling gold


• The preparations described may be restored entirely with pellets of gold foil, or E-Z Gold may be used.
• If powdered gold is selected, heavy hand pressure compaction may be substituted for hand mallet or automatic mallet
techniques.
Instrumentation
• By use of a high-speed handpiece with air-water spray, the No. 330 or No. 329 bur is aligned, and the outline form (which
includes the limited initial depth) is established.
Restoration
• The restorative phase begins with the insertion of a pellet of E-Z Gold or gold foil.
• The gold is first degassed in the alcohol flame, cooled in air for a few moments, and inserted into the preparation with the
passing instrument.
• The gold is pressed into place with the nib of a small round condenser. In larger preparations, a pair of condensers is used for
this initial stabilization of the gold. Next, compaction of the gold begins with a line of force directed against the pulpal wall.
• Hand pressure is used for E-Z Gold; malleting is used for gold foil.
• The gold is compacted into the pulpal line angles and against the external walls, and the line of force is changed to a
45-degree angle to the pulpal and respective external walls (to compact the gold best against the internal walls).
• Additional increments of gold are added, and the procedure is repeated until the preparation is about three quarters full of
compacted gold.
• If E-Z Gold is to be the final restoration surface, compaction is continued until the restoration is slightly overfilled.

CAST METAL RESTORATIONS


• The cast metal restoration is versatile and is especially applicable to Class II onlay preparations
• The Class II inlay involves the occlusal surface and one or more proximal surfaces of a posterior tooth. When cusp tips are
restored, the term onlay is used
• The procedure requires two appointments: the first for preparing the tooth and making an impression, and the second for
delivering the restoration to the patient
• The fabrication process is referred to as an indirect procedure because the casting is made on a replica of the prepared tooth
in a dental laboratory

https://t.me/DentalBooksWorld
1100 Triumph's Complete Review of Dentistry

Material Qualities
Cast metal restorations can be made from a variety of casting alloys
The American Dental Association (ADA) Specification No. 5 for Dental Casting Gold Alloys requires a minimum total gold-
plus-platinum-metals content of 75 weight percent (wt%)
Such traditional high-gold alloys are unreactive in the oral environment and are some of the most biocompatible materials
available to the restorative dentist
At present, four distinct groups of alloys are in use for cast restorations:
1. Traditional high-gold alloys,
2. low-gold alloys,
3. palladium–silver alloys, and
4. base metal alloys
Indications
• Large restorations
• Endodontically treated teeth
• Teeth at risk for fracture
• Dental rehabilitation with cast metal alloys
• Diastema closure and occlusal plane correction
• Removable prosthodontic abutment
Contraindications
• High caries rate
• Young patients
• Esthetics
• Small restorations
Advantages
• Strength
• Biocompatibility
• Low wear
• Control of contours and contacts
Disadvantages
• Number of appointments and higher chair time
• Temporary restorations
• Cost
• Technique sensitivity
• Splitting forces

Tooth Preparation for Class II Cast Metal Inlays


Initial Preparation
• Carbide burs used to develop the vertical internal walls of the preparation for cast metal inlays and onlays are plane cut,
tapered fissure burs
• These burs are plane cut so that the vertical walls are smooth. The side and end surfaces of the bur should be straight to aid
in the development of uniformly tapered walls and smooth pulpal and gingival walls
• As the occlusogingival height increases, the occlusal divergence should increase because lengthy preparations with minimal
divergence (more parallel) may present difficulties during the seating and withdrawal of the restoration
Occlusal Step
With the No. 271 carbide bur held parallel to the long axis of the tooth crown, the dentist enters the fossa or pit closest to the
involved marginal ridge, using a punch cut to a depth of 1.5 mm to establish the depth of the pulpal wall

https://t.me/DentalBooksWorld
Chapter 12 • Conservative Dentistry 1101

Proximal Box
• Continuing with the No. 271 carbide bur, the distal enamel is isolated by cutting a proximal ditch
• The harder enamel should guide the bur. Slight pressure toward enamel is necessary to prevent the bur from cutting only dentin
• If the bur is allowed to cut only dentin, the resulting axial wall would be too deep. The mesiodistal width of the ditch should
be 0.8 mm (the tip diameter of the bur) and prepared approximately two thirds (0.5 mm) at the expense of dentin and one
third (0.3 mm) at the expense of enamel
Preparation of Bevels and Flares
• After the cement base (where indicated) is completed, the slender, flame-shaped, fine-grit diamond instrument is used to
bevel the occlusal and gingival margins and to apply the secondary flare on the distolingual and distofacial walls
• This should result in 30- to 40-degree marginal metal on the inlay
• This cavosurface design helps seal and protect the margins and results in a strong enamel margin with an angle of 140–150 degrees
• A cavosurface enamel angle of more than 150 degrees is incorrect because it results in a less defined enamel margin (finish
line), and the marginal cast metal alloy is too thin and weak if its angle is less than 30 degrees

Class III, IV, and V Direct Composite Restorations


Indications
Class III, IV, and V direct composite restorations are mainly indicated in the restoration of caries lesions (Class III, IV, and V),
anterior enamel and/or dentin crown fractures (Class IV), and noncarious cervical defects (Class V)
Contraindications
The main contraindication for use of composite for Class III, IV, and V restorations is an operating area that cannot be
adequately isolated
Class V composite restorations also may have their durability compromised when the restoration extends onto the root surface
(no marginal enamel). Because bonding to dentin is not as predictable or durable as bonding to enamel, in such situations,
the dentin gingival margin is more prone to microgap formation and marginal microleakage than the enamel coronal margin
Advantages
The advantages of composite as a direct restorative material relative to other restorative materials are:
1. Esthetics
2. Conservative tooth structure removal
3. Easier, less complex tooth preparation
4. Insulation
5. Decreased microleakage
6. Increased short-term strength of remaining tooth structure
Disadvantages
The disadvantages of direct composite restorations are as follows:
1. Polymerization shrinkage effects
2. Lower fracture toughness than most indirect restorations
3. More technique-sensitive than amalgam restorations and some indirect restorations
4. Possible greater localized occlusal wear
5. Unknown biocompatibility of some components (bisphenol A [BPA])

Tooth Preparation-Direct Composite Restorations


In general, the tooth preparation for a Class III direct composite restoration involves
1. Obtaining access to the defect
2. Removing faulty structures
3. Creating the convenience form for the restoration

https://t.me/DentalBooksWorld
1102 Triumph's Complete Review of Dentistry

Indications for a facial approach include the following:


1. The caries lesion is positioned facially, and facial access would significantly conserve the tooth structure
2. Teeth are irregularly aligned, and facial access would significantly conserve the tooth structure
3. An extensive caries lesion extends onto the facial surface
4. A faulty restoration that originally was placed from the facial approach needs to be replaced
Ideal initial axial wall preparation depth
• Axial wall 0.2 mm into dentin
• Facial extension and axial wall following the contour of the tooth
If the preparation outline extends gingivally onto the root surface, the gingival floor should form a cavosurface margin
of 90 degrees, and the depth of the gingivoaxial line angle should be not more than 0.75 mm at this initial stage of tooth
preparation
The bevel is prepared by creating a 45-degree angle to the external surface and to a width of 0.5–2.0 mm, depending on the
size of the preparation, location of the margin, and esthetic requirements of the restoration
If the gingival floor has been extended gingivally to a position where the remaining enamel thickness is minimal or nonexistent,
the bevel is omitted from this area to preserve the remaining enamel margin or maintain 90-degree cavosurface margin in dentin
Class III lesion, however, the operator may decide that retention form should be enhanced by placing groove (at gingival) or
cove (at incisal) retention features in addition to bonding
Restorative Technique
Matrix Application
A matrix is a device that is applied to a prepared tooth before the insertion of the restorative material. Its purposes include
1. Confining the restorative material excess
2. Assisting in the development of the appropriate axial tooth contours. The matrix usually is applied and stabilized with a
wedge before application of the adhesive because it helps contain the adhesive components to the prepared tooth
3. A properly contoured and wedged matrix is a prerequisite for a restoration involving the entire proximal contact area
Placement of the Adhesive
• Proximal surface of the adjacent unprepared tooth should be protected from inadvertent etching by placing a Mylar strip,
if not yet applied, or a Teflon tape
• Phosphoric acid gel etchant is applied to all of the prepared tooth structure, approximately 0.5 mm beyond the prepared
margins onto the adjacent unprepared tooth
• The etchant typically is left undisturbed for 15 seconds

PIN RETAINED RESTORATIONS


When the teeth are decayed and it has to be restored the dentist goes for the filling but sometimes the decay is extensive and
a large part of the tooth is lost. In such cases pins and posts are used as the simple filling material cannot give the adequate
strength to sustain the masticatory forces and fractures
Indications
1. As additional aids of retention in badly broken down or mutilated teeth.
2. In teeth with poor prognosis, i.e., endodontically and periodontically involved teeth.
3. When one or more cusps need capping.
4. Increased resistance and retention form is needed.

CLASSIFICATION
1. Direct/Nonparallel Pins—are inserted into Dentin followed by placement of restorative material directly over them.
Three major categories of direct pins are:
a. Cemented pins—Pins are 0.001–0.002 inch smaller than their pin channels and the difference in diameter provides
space for cementing medium. Are least retentive but virtually place no stress on surrounding dentin during or after
placement.

https://t.me/DentalBooksWorld
Chapter 12 • Conservative Dentistry 1103

Four sizes of TMS pins.


• Regular (0.031 inch [0.78 mm])
• Minim (0.024 inch [0.61 mm])
• Minikin (0.019 inch [0.48 mm])
• Minuta (0.015 inch [0.38 mm])
b. Friction locked pins—Are 0.001 inches larger than their pin channels and hence utilize the elasticity of dentin for
retaining the tapped pins in a vise like grip. Better retention than cemented pins but generates stresses in dentin in the
form of cracks or craze lines.
c. Threaded pins—Are 0.0015–0.002 inches larger than their pin channels and like friction locked pins they are also
retained by elasticity of dentin. Provides maximum retention but at the same time generates excessive stresses in the
form of cracks in dentin.
2. Indirect/Parallel Pins—Are an integral part of Cast restoration. These pins are placed parallel to each other as well as
parallel to the path of insertion of the restoration.

Types of Pins
• The most frequently used pin type is the self-threading pin
• Friction-locked and cemented pins, although still available, are rarely
• The pin-retained amalgam restoration using self-threading pins originally was described by Going in 1966
• The diameter of the prepared pinhole is 0.0015 to 0.004 inch smaller than the diameter of the pin
• The threads engage dentin as the pin is inserted, thus retaining it
• The elasticity (resiliency) of dentin permits insertion of a threaded pin into a hole of smaller diameter
• Although the threads of self-threading pins do not engage dentin for their entire width, self-threading pins are the most
retentive of the three types of pins being three to six times more retentive than cemented pins
• Vertical and horizontal stresses can be generated in dentin when a self-threading pin is inserted
• Craze lines in dentin may be related to the size of the pin
• The insertion of 0.031-inch self-threading pins produces more dentinal craze lines than does the insertion of 0.021-inch
self-threading pins
• Some evidence suggests, however, that self-threading pins may not cause dentinal crazing
• Pulpal stress is maximal when the self-threading pin is inserted perpendicular to the pulp
• The depth of the pinhole varies from 1.3 to 2 mm, depending on the diameter of the pin used
• A general guideline for pinhole depth is 2 mm

The Thread Mate System (TMS) Pins

Name Color Code Pin Diameter Drill Diameter Total Pin Pin Length Extending
(inches/mm) (inches/mm) Length (mm) from Dentin (mm)
Regular (standard) Gold 0.031/0.78 0.027/0.68 7.1 5.1
Regular (self-shearing) Gold 0.031/0.78 0.027/0.68 8.2 3.2
Regular (two-in-one) Gold 0.031/0.78 0.027/0.68 9.5 2.8
Minim (standard) Silver 0.024/0.61 0.021/0.53 6.7 4.7
Minim (two-in-one) Silver 0.024/0.61 0.021/0.53 9.5 2.8
Minikin (self- Red 0.019/0.48 0.017/0.43 7.1 1.5
shearing)
Minuta (self-shearing) Pink 0.015/0.38 0.0135/0.34 6.2 1
*1 mm = 0.03937 inch

https://t.me/DentalBooksWorld
1104 Triumph's Complete Review of Dentistry

3.0 mm
2.0 mm 2.0 mm

2.0 mm
3.0 mm 3.0 mm

A B C

Three types of pins. (A) Cemented, (B) Friction-locked, (C) Self-threading

FACTORS AFFECTING RETENTION OF THE PIN IN DENTIN AND AMALGAM


Type
With regard to the retentiveness of the pin in dentin, the self-threading pin is the most retentive, the friction-locked pin is
intermediate, and the cemented pin is the least retentive
Surface Characteristics
• The number and depth of the elevations (serrations or threads) on the pin influence the retention of the pin in the amalgam
restoration
• The shape of the self-threading pin gives it the greatest retention value
Orientation, Number, and Diameter
• Placing pins in a non-parallel manner increases their retention
• Bending pins to improve their retention in amalgam is not advisable because the bends may interfere with adequate
condensation of amalgam around the pin and decrease amalgam retention
• Bending also may weaken the pin and risk fracturing dentin. Pins should be bent only to provide for an adequate amount of
amalgam (approximately 1 mm) between the pin and the external surface of the finished restoration (on the tip of the pin
and on its lateral surface)
• Only the specific bending tool should be used to bend a pin, not other hand instruments.
• In general, increasing the number of pins increases their retention in dentin and amalgam
• The benefits of increasing the number of pins must be compared with the potential problems
• As the number of pins increases,
(1) the crazing of dentin and the potential for fracture increase,
(2) the amount of available dentin between the pins decreases, and
(3) the strength of the amalgam restoration decreases
• Also, as the diameter of the pin increases, retention in dentin and amalgam generally increases. As the number, depth, and
diameter of pins increase, the danger of perforating into the pulp or the external tooth surface increases
• Numerous long pins also can severely compromise condensation of amalgam and amalgam’s adaptation to the pins. A pin
technique that permits optimal retention with minimal danger to the remaining tooth structure should be used
Extension into Dentin and Amalgam
• Self-threading pin extension into dentin and amalgam should be approximately 1.5 to 2 mm to preserve the strength of
dentin and amalgam
• Extension greater than this is unnecessary for pin retention and is contraindicated

Pin Placement Factors and Techniques


Pin Size
• Two determining factors for selecting the appropriate-sized pin are the amount of dentin available to receive the pin safely
and the amount of retention desired

https://t.me/DentalBooksWorld
Chapter 12 • Conservative Dentistry 1105

• In the TMS system, the pins of choice for severely involved posterior teeth are the Minikin (0.019 inch [0.48 mm]) and,
occasionally, the Minim (0.024 inch [0.61 mm])
• The Minikin pins usually are selected to reduce the risk of dentin crazing, pulpal penetration, and potential perforation
• The Minim pins usually are used as a backup in case the pinhole for the Minikin is over-prepared or the pin threads strip
dentin during placement and the Minikin pin lacks retention
• Larger-diameter pins have the greatest retention
• The Minuta (0.015 inch [0.38 mm]) pin is approximately half as retentive as the Minim and one-third as retentive as the
Minim pin
• It is usually too small to provide adequate retention in posterior teeth
• The Regular (0.031 inch [0.78  mm]), or largest-diameter, pin is rarely used because a significant amount of stress and
crazing, or cracking, in the tooth
Colors of various Pins
K-89—TMS Minuta (0.350×1.3 mm) with depth-limitation pink
K-90—TMS Minikin (0.425×1.5 mm) with depth-limitation red
K-91—TMS Minikin (0.425×1.5 mm) long shank, with depth-limitation red
K-92—TMS Minim (0.525×2.0 mm) with depth-limitation silver
K-93—For TMS Minim (0.525×4.0 mm) without depth-limitation silver
K-96—For TMS Regular (0.675×2.0 mm) with depth-limitation gold
K-97—For TMS Regular (0.675×5.0 mm) without depth-limitation gold
Orientation – Number – Diameter of Pins
• As Number, diameter, and depth of pins in the tooth increases so does retention of both pins and restorations.
• However, so also increases the chance for Pulpal Penetration or External Tooth Perforation.

INFECTION CONTROL
Air-Borne Contamination
• A high-speed handpiece is capable of creating air-borne contaminants from bacterial residents in the dental unit water spray
system and from microbial contaminants from saliva, tissues, blood, plaque, and fine debris cut from carious teeth
• With respect to size, these air-borne contaminants exist in the form of spatter, mists, and aerosols. Aerosols consist of
invisible particles ranging from 5 mm to approximately 50 mm that can remain suspended in the air and breathed for
hours
• Aerosols and larger particles may carry agents of any respiratory infection carried by the patient. No scientific evidence
indicates, however, that fine aerosols have transmitted the blood-borne infection caused by hepatitis B virus (HBV)
• Transmission of human immunodeficiency virus (HIV) by aerosols is even less likely, as evidenced by the extremely low
transmissibility of HIV in dental procedures and in the homes of infected persons
• Mists that become visible in a beam of light consist of droplets estimated to approach or exceed 50 mm
• Heavy mists tend to settle gradually from the air after 5 to 15 minutes
• Aerosols and mists produced by the cough of a patient with unrecognized active pulmonary or pharyngeal tuberculosis are
likely to transmit the infection
• Spatter consists of particles generally larger than 50 mm and even visible splashes. Spatter has a distinct trajectory,
usually falling within 3 feet (ft) of the patient’s mouth, having the potential for coating the face and outer garments of
the attending personnel
• Spatter or splashing of mucosa is considered a potential route of infection for dental personnel by blood-borne pathogens
• Barrier protection of personnel using masks, protective eyewear, gloves, and gowns is now a standard requirement for dental
procedures

https://t.me/DentalBooksWorld
1106 Triumph's Complete Review of Dentistry

• A pretreatment mouthrinse, rubber dam, and high-velocity air evacuation also can reduce microbial exposure
• To help reduce exposure to air-borne particles capable of transmitting respiratory infections, adequate air circulation should
be maintained, and masks should be kept in place until air exchange in the room has occurred or until personnel leave the
operatory

Direct Contamination
• Direct contamination occurs during direct contact with bodily fluids, and this is a major exposure concern for dental personnel
Indirect Contamination
• With saliva-contaminated hands, the hygienist, the dentist, and the assistant could repeatedly contact or handle unprotected
operatory surfaces during treatments
• The invisible trail of saliva left on such contaminated surfaces often defies either awareness or effective cleanup
• Soiled surfaces that are poorly cleaned provide another source of gross environmental contamination and thus potential
contamination of personnel and patients
• Cross-contamination of patients by such contaminated surfaces was documented in a clinical office radiology setting

Cross-Infections
• Most information on cross-infection and infection control concepts has been derived from data collected in hospitals
• Evidence of oral or systemic cross-infections in dentistry is more difficult to obtain because patients may have contracted
infections elsewhere, before or after having a dental treatment
• Infected patients usually are unaware of the source of their infection and go elsewhere for diagnosis and treatment of
nonoral infections
• Infection outbreaks usually are detected in patients or personnel only when they occur in clusters recognized by other
health care providers or are detected by epidemiologic studies and investigative surveys of personnel

Federal and State Regulations to Reduce Exposure Risks from Pathogens in Blood and Other Sources
of Infection
The term infection control program has a long tradition in hospital usage. Infection control programs such as those
recommended by the Centers for Disease Control and Prevention (CDC) and the American Dental Association (ADA) are
designed to protect both patients and personnel
Occupational Safety and Health Administration Office Exposure Control Plan-OSHA Rule
1. Employers must provide HBV immunization to employees
2. Employers must mandate that standard precautions be observed to prevent contact with blood and other potentially
infectious materials. Saliva is considered a blood-contaminated bodily fluid in relation to dental treatments
3. Employers must implement engineering controls to reduce the production of contaminated spatter, mists, and aerosols
4. Employers must implement work practice control precautions to minimize splashing, spatter, or contact of bare hands
with contaminated surfaces
5. Employers must provide facilities and instruction for washing hands after removing gloves and for washing other skin
immediately or as soon as feasible after contact with blood or potentially infectious materials
6. Employers must prescribe safe handling of needles and other sharp items. Needles must not be bent or cut
7. Employers must prescribe disposal of single-use needles, wires, carpules, and sharps as close to the place of use as
possible, as soon as feasible, in hard-walled, leak-proof, red color biohazard labelled containers that are closable, from
which needles cannot be easily spilled
8. Contaminated reusable sharp instruments must not be stored or processed in a manner that requires employees to reach
into containers to retrieve them
9. Employers must prohibit staff from eating, drinking, handling contact lenses, and application (but not wearing) of facial
cosmetics in contaminated environments such as operatories and cleanup areas
10. Blood and contaminated specimens to be shipped, transported, or stored should be placed in suitable closed containers
that prevent leakage

https://t.me/DentalBooksWorld
Chapter 12 • Conservative Dentistry 1107

11. At no cost to employees, employers must provide them with necessary PPE and clear directions for use of appropriate
universal barrier protection in treating all patients and for all other contact with blood or other infectious materials
12. Employers should ensure that employees correctly use and discard PPE or prepare it properly for reuse. Adequate facilities
should be provided to discard gowns or laundry in the location where they are used. A face shield is not a substitute for a mask

MULTIPLE CHOICE QUESTIONS

PRINCIPLES OF CAVITY PREPARATION


1. According to Mount and Hume classification, 1.2 is
A. Pit and fissure moderate B. Pit and fissure enlarged
C. Contact moderate D. Contact enlarged
2. Pit and fissure caries is seen in
A. Class I B. Class I compound
C. Class II D. Class II compound
3. Line angle for class III preparation is
A. 8 B. 11
C. 6 D. 4
4. The image is example for which class of caries?

A. Class I B. Class II
C. Class III D. Class IV
5. In class II cavity for inlay, the cavosurface margin of the facial and lingual walls clears the adjacent tooth by
A. 0.20 ± 0.05 mm B. 0.50 ± 0.20 mm
C. 0.80 ± 0.35 mm D. 1.10 ± 0.45 mm
6. The function of proximal grooves in a class II cavity is
A. Resistance form B. Retention form
C. Increases strength D. Resistance and retention form
7. The cavosurface angle for inlay cavity preparation
A. 90 degrees B. 150 degrees
C. Less than 90 degrees D. 180 degrees
8. Whenever the caries cone enamel is smaller than in dentin, it is known as
A. Residual caries B. Recurrent caries
C. Forward caries D. Backward caries
9. All are types of pit 7 fissure except
A. Type I B. Type V
C. Type K D. Type H

https://t.me/DentalBooksWorld
1108 Triumph's Complete Review of Dentistry

10. This lesion according to Mount and Hume:


A. 1.1 B. 1.2
C. 1.3 D. 1.4
11. Which of the following principles of the cavity preparation is not considered much in modern methods of restorative
Dentistry?
A. Extension for prevention B. Convenient form
C. Removal of the undermined enamel D. Removal of the carious dentine
12. For cast restorations the cavity wall should have a taper of
A. 30–45 B. 5–10
C. 2–5 D. None of the above
13. Gingivally the depth of a class V cavity is
A. 0.5–1 mm B. 0.75–1 mm
C. 1–1.25 mm D. 2–3 mm
14. ICDAS classification for dental caries distinct visual change in enamel is
A. Code 1 B. Code 2
C. Code 3 D. Code 0
15. The marked angle “A” is
A


A. Cavosurface angle B. Minimal restorative material angle
C. Line angle D. Maximal restorative material angle

AMALGAM
1. Retentive grooves of a proximal box in a class II cavity prepared for amalgam should be
A. Sharp and elongated at the dentinoenamel junction of the buccal and lingual walls
B. Sharp and well defined extending from the gingiva to the occlusal cavosurface angle along the buccal and lingual line
angles
C. Elongated and rounded in dentine at the buccoaxial and linguoaxial line angles extending from the gingival wall to the
axiopulpal line angle
D. Short and rounded at the dentinoenamel junction of the buccal and lingual walls
2. The main difference between composite and amalgam as restorative material is
A. Occlusal wear B. Durability
C. Retention D. Manipulation
3. Minimum distance between minim pin is
A. 3 mm B. 4 mm
C. 5 mm D. 6 mm

https://t.me/DentalBooksWorld
Chapter 12 • Conservative Dentistry 1109

4. Creep rate decreases with


A. Increased force of condensation B. Decreased force of condensation
C. Under or overtrituration D. Delay in time between trituration and condensation
5. The pin used in pin restoration of a root canal-treated tooth is
A. Self-thread pin B. Cemented pin
C. Frictional pin D. Any of the above
6. The ideal ratio of pin in pin restoration between dentin and exposed surface of pin is
A. 3:1 B. 2:1
C. 1:1 D. 1:2
7. In amalgam cavity preparation the cavosurface beveled at
A. Less than 90 degrees B. Less than 45 degrees
C. More than 10 degrees D. At an obtuse angle
8. The plane of the axial wall of an MO cavity prepared for silver amalgam should be
A. A flat straight plane buccolingually
B. Parallel to the long axis of the teeth
C. Concave to allow greater bulk of amalgam
D. Parallel to the long axis of the teeth and should follow the buccolingual contour of the tooth
9. Third amalgam was initiated by
A. Alfred Stock B. H.A. Huggins
C. William Taggart D. None of the above
10. One of the following decreases delayed expansion
A. Mercury B. Zinc
C. Copper D. Palladium
11. Undertrituration of the amalgam results in
A. Increased expansion and reduced strength B. Increased contraction and increased strength
C. Increased expansion but also increased strength D. Increased contraction and reduction in strength
12. Which of the following statements is true regarding Lathe cut silver alloy? (AIPG 2014)
A. Requires least amount of mercury
B. Achieves lowest compressive strength at 1 hour
C. Has tensile strength, both at 15 minutes and 7 days is comparable to high copper, unicompositional alloys
D. Has lower creep value

DENTAL CEMENTS
1. Type of ZOE used for luting purposes
A. Type I B. Type II
C. Type III D. Type IV
2. Thickness of varnishes should be
A. 0.01 mm B. 0.1 mm
C. 1 mm D. 0.25 mm
3. Recommended powder liquid ratio for Zinc phosphate cement is
A. 1.4 mg powder to 0.5 mL liquid B. 2.4 mg powder to 1.5 mL liquid
C. 3.4 mg powder to 2.5 mL liquid D. 1.4 mg powder to 1 mL liquid
4. GIC type used for luting and orthodontic brackets bonding purpose is
A. Type 1 B. Type 2A
C. Type 2B D. Type 3
5. The cement which has antibacterial property is
A. Copper oxide cement. B. Glass informer cement
C. Polycarboxylate cement D. Zinc phosphate cement
6. Compressive strength of small particle hybrid composite is (in Mpa)
A. 350–400 B. 300–350
C. 250–300 D. 400–450

https://t.me/DentalBooksWorld
1110 Triumph's Complete Review of Dentistry

7. Filler size of midifilled composite in micrometer


A. 10–100 B. 1–10
C. 0.1–1 D. 0.01–0.1
8. Vitremer is
A. Metal-modified GIC B. Polyacid-modified composite
C. Resin-modified GIC D. Light-cured GIC
9. Important property of pit and fissure sealant is
A. Low viscosity B. Color
C. High filler content D. Adequate working time
10. Ionic bonding is found in
A. GIC B. Polycarboxylate cement
C. Zinc phosphate cement D. All of the above
11. The particle size for calcium aluminate GIC cement is
A. 15 µm B. 25 µm
C. 10 µm D. 5 µm
12. Which of the following cement provides maximum resistance to enamel decalcification when used with orthodontic
bands?
A. GIC B. Silicate
C. ZnPO4 D. Zinc polycarboxylate
13. Which of the following is true about hybrid ionomer?
A. Releases less fluoride than compomer
B. Fluoride release at the same level as conventional GIC
C. More ionic activity compared to conventional GIC
D. More sensitive to water contamination than conventional GIC
14. The main purpose of stannous fluoride in zinc polycarboxylate cement is
A. Fluoride release B. To increase strength
C. For better handling purpose D. To increase setting time
15. Percentage of zinc oxide in zinc oxide—eugenol cement
A. 70% B. 80%
C. 60% D. 90%
16. Which component of composites exhibit estrogenicity?
A. Bis-GMA B. TEGDMA
C. UDMA D. Bis-EMA
17. The pH of ZnPO4 cement reaches to between 6 and 7 after _____ of mixing
A. 1 hour B. 8 hours
C. 24 hours D. 16 hours

DIRECT FILLING GOLD


1. The purpose of heating gold foil before condensing is
A. To improve welding quality B. To melt the gold
C. To remove adsorbed gases on gold surface D. To increase oxidation
2. If the nib diameter is reduced by half, the compaction force is
A. Halved B. Double
C. Increases four times D. Constant
3. The cavosurface margin for direct filling gold are prepared with
A. 90-degree butt joint B. 45 degrees beveled
C. 30–40 degrees bevel D. 10-degree bevel
4. Best marginal integrity is obtained with
A. GIC B. Gold foil
C. Gold Inlay D. Amalgam

https://t.me/DentalBooksWorld
Chapter 12 • Conservative Dentistry 1111

5. Powdered gold is marketed as


A. William’s E-Z gold B. Morgan’s E-Z gold
C. Robert’s E-Z gold D. Taggart’s E-Z gold
6. The most recent direct filling gold which produces the hardest surface on condensation is
A. Mat gold
B. Encapsulated powdered gold
C. Mat gold alloyed with calcium and wrapped in gold foil
D. Platinized gold

INSTRUMENTATION
1. Identify the marked part


A. Light outlet B. Water outlet
C. Windmill D. Bearing housing
2. The free running speed of a turbine is in the order of ________revolutions per minute (rpm)
A. 4 lakhs B. 2 lakhs
C. 1 lakh D. 50K
3. The cutting speed for caries removal is
A. 1,500 rpm B. 16,000 rpm
C. 500 rpm D. 230,000 rpm
4. Identify the marked part:


A. Width B. Length
C. Blade angle D. Cutting edge angle

https://t.me/DentalBooksWorld
1112 Triumph's Complete Review of Dentistry

5. Identify the instrument


A. Ultrasonic scaler B. Air abrasion unit
C. Instrument sharpening unit D. Apex locator
6. Identify the GMT


A. None B. Mesial side
C. Distal side D. Not a GMT
7. Which of the following instruments will have a four-number formula?
A. Ordinary hatchet B. GMT
C. Jeffery hatchet D. All the above
8. When dental bur is sterilized by autoclaving, which of the following chemical is used for protection of the bur?
A. Sodium nitrate B. Sodium nitrite
C. Silver nitrate D. Silver nitrite
9. Cutting and grinding procedures are predominantly
A. Two directional B. Unidirectional
C. Three directional D. Multi directional
10. Identify


A. Positive rake angle and cutting action B. Positive rake angle and scraping action
C. Negative rake angle and scraping action D. Negative rake angle and cutting action

https://t.me/DentalBooksWorld
Chapter 12 • Conservative Dentistry 1113

11. Identify


A. Rubber dam punch B. Rubber dam frame
C. Rubber dam forceps D. Rubber dam retainer
12. Interdental papilla protruding from the rubber dam, most common cause is
A. Inflammation of interdental papillae B. Use of light weight rubber dam
C. Punch are placed too far D. Punch are placed too close
13. In which of the condition “piggy back” wedging is indicated?
A. Proximal caries with gingival recession B. Tooth with fluted surface
C. Class II with wide proximal box D. Marrow class II cavity
14. Sodium nitrite is used as
A. Antiplaque agent B. Antirust agent
C. Desensitizing agent D. Bleaching agent
15. On a carbide bur, a great number of cutting blades results in
A. Less efficient cutting and a smoother surface B. Less efficient cutting and a rougher surface
C. More efficient cutting and a smoother surface D. More efficient cutting and a rougher surface

ANSWERS

PRINCIPLES OF CAVITY PREPARATION


1. Answer: A
Classification (Mount and Hume, 1998)
Size
Minimal Moderate Enlarged Extensive
1 2 3 4
Site
Pit/fissure 1 1.1 1.2 1.3 1.4
Contact area 2 2.1 2.2 2.3 2.4
Cervical 3 3.1 3.2 3.3 3.4
Australian Dental Journal 1998; 43: 3.
2. Answer: A
3. Answer: C

Type of tooth preparation Line angles Point angles


Class I 8 4
Class II 11 6

https://t.me/DentalBooksWorld
1114 Triumph's Complete Review of Dentistry

Class III 6 3
Class IV 11 6
Class V 8 4

4. Answer: A
Class I: Pit and fissure preparations occur on the occlusal surfaces of premolars and molars, the occlusal two-thirds of
buccal and lingual surface of molars, lingual surface of incisors, and any other abnormal position.

5. Answer: A
Note: This is the 1st pulse question in cons…
In class II cavity for inlay, the cavosurface margin of the gingival seat clears the adjacent tooth by
A. 0.20 ± 0.05 mm
B. 0.50 ± 0.20 mm
C. 0.80 ± 0.35 mm
D. 1.10 ± 0.45 mm
6. Answer: D
7. Answer: B
8. Answer: D
9. Answer: D

Pit and Fissure:


• Pit: Small pin point depression located at the junction of developmental grooves.
• Fissure: Deep clefts between adjoining cusps.

Morphology of Fissures: (NANGO-1960)

V – type U – type I – type K – type

shallow & self cleansable deep, narrrow & retentive


(non invasive technique) (invasive technique)

10. Answer: B
11. Answer: A
12. Answer: C
13. Answer: B
14. Answer: B

https://t.me/DentalBooksWorld
Chapter 12 • Conservative Dentistry 1115

Occlusal Protocol***
ICDAS code 0 1 2 3 4 5 6

Definitions Sound tooth First visual Distinct Localized Underlying Distinct Extensive
surface no change in visual change enamel dark cavity with distinct
caries change enamel; seen in enamel; breakdown shadow visible cavity with
after air drying only after seen when with no from dentin; dentin;
(5 seconds); drying or wet, white, visible dentin dentin, with frank cavity
or hypoplasia, colored change or colored, or underlying or without cavitation is deep
wear, erosion “thin” limited “wired ” than shadow; localized involving and wide
and other to the confines the fissure/ discontinuity enamel less than involving
noncaries of the pit and flossa of surface breakdown half of more than
phenomena fissure area enamel a tooth half of the
widening of surface tooth
fissure
Histologic Lesion depth Lesion depth Lesion depth Lesion Lesion Lesion
depth in P/F was 90% in P/F was in P/F with depth in depth in depth in
in the outer 50% inner 77% in P/F with P/F with P/F
enamel with enamel and dentin 88% into 100% 100%
only 10% into 50% into the dentin dentin reaching
domain outer one- inner one-
third dentin third dentin
Sealant/resto- Sealant optional Sealant optional Sealant Sealant or Minimally Minimally Minimally
ration Recom- DIAGNOdent DIAGNOdent optional or minimally invasive invasive invasive
mendation for may be helpful may be helpful caries biopsy if invasive restoration restoration restoration
low risk DIAGNOdent restoration
is 20–30 needed
Sealant/resto- Sealant optional Sealant optional Sealant Sealant or Minimally Minimally Minimally
ration Recom- DIAGNOdent DIAGNOdent optional or minimally invasive invasive invasive
mendation for may be helpful may be helpful caries biopsy if invasive restoration restoration restoration
moderate risk DIAGNOdent restoration
is 20–30 needed
Sealant/resto- Sealant Sealant Sealant Sealant or Minimally Minimally Minimally
ration Recom- Recommended Recommended optional or minimally invasive invasive invasive
mendation for DIAGNOdent DIAGNOdent caries biopsy if invasive restoration restoration restoration
high risk* may be helpful may be helpful DIAGNOdent restoration
is 20–30 needed
Sealant/resto- Sealant Sealant Sealant Sealant or Minimally Minimally Minimally
ration Recom- Recommended Recommended optional or minimally invasive invasive invasive
mendation for DIAGNOdent DIAGNOdent caries biopsy if invasive restoration restoration restoration
extreme risk** may be helpful may be helpful DIAGNOdent restoration
is 20–30 needed
*Patients with one (or more) cavitated lesion(s) are high-risk patients.
**Patients with one (or more) cavitated lesion(s) and xerostomia are extreme-risk patients.
***All sealants and restorations to be done with a minimally invasive philosophy in mind. Sealants are defined as confined
to enamel. Restoration is defined as in dentin. A two-surface restoration is defined as a preparation that has one part of the
preparation in dentin and the preparation extends to a surface (Note: The second surface does not have to be in dentin). A
sealant can be other resin-based or glass ionomer. Resin-based sealants should have the most conservatively prepared fissures
for proper bonding. Glass ionomer should be considered where the enamel is immature, or where fissure preparation is not
desired, or where rubber dam isolation is not possible. Patients should be given a choice in material selection.

https://t.me/DentalBooksWorld
1116 Triumph's Complete Review of Dentistry

15. Answer: B
Cavosurface Angle and Cavosurface Margin
• The cavosurface angle is the angle of tooth structure formed by the junction of a prepared wall and the external surface
of the tooth. The actual junction is referred to as cavosurface margin. The cavosurface angle may differ with the location
on the tooth, the direction of the enamel rods on the prepared wall, or the type of restorative material to be used.
In the image the cavosurface angle (cs) is determined by projecting the prepared wall in an imaginary line (wʹ) and the
unprepared enamel surface in an imaginary line (usʹ) and noting the angle (csʹ) opposite to the cavosurface angle (cs).
For better visualization, these imaginary projections can be formed by using two periodontal probes, one lying on the
unprepared surface and the other on the prepared external tooth wall.'

Minimal restorave
material angle

Cavosurface
angle

AMALGAM
1. Answer: C
2. Answer: A
3. Answer: C
4. Answer: A
5. Answer: B
6. Answer: C
7. Answer: D
8. Answer: D
9. Answer: B (Ref. Sturdevant’s Art and Science of Operative Dentistry, By Andre V.Ritter)
• First amalgam war—Initiated in 1841
• Second amalgam war—Alfred Stock in 1920
• Third amalgam war—H.A. Huggins
10. Answer: A
Zinc
• Increases strength
• Increases expansion
• Increases flow
• Increases setting time
• Decreases corrosion resistance
• Increases plasticity
• Decreases brittleness
Indium
• Increases strength
• Increases expansion
• Increases flow
• Increases setting time
• Amalgamation more difficult
Palladium
• Increases strength
• Increases corrosion resistance

https://t.me/DentalBooksWorld
Chapter 12 • Conservative Dentistry 1117

Mercury
• Decreases setting time
• Decreases delayed expansion
11. Answer: A
12. Answer: B

DENTAL CEMENTS
1. Answer: B (Ref. Basic Dental Materials, By John J. Manappallil, 2015, page no. 99)
• Type I: Temporary ZOE cement
• Type II: Long-term ZOE luting cement
• Type III: Temporary ZOE restoration
• Type IV: Intermediate ZOE restoration/cavity liner
2. Answer:
Protective materials
Bases
1–2 mm—pulpal, thermal, and chemical protection
Liners
Thick—1–50 µm
Thin—0.2–1 mm: Pulpal and thermal protection
Thin liners again classified into two types:
Suspension liners: 2–5 µm
Solution liners: 20–25 µm
Cavity varnish
Protects pulp by sealing the tubule denying entry of irritants (0.1 mm)
3. Answer: A
4. Answer: A
Classification
Types Uses
Type 1 Luting and orthodontic brackets
Type 2A Esthetic restoration
Type 2B Reinforced restoration
Type 3 Lining cements and bases
5. Answer: A
• Colloidal silica particles, because of their extremely small size, have extremely large surface areas ranging from 50–400
square meters per gram
• Macrofilled composites are easier to handle than micros filled to the same density
• However, greater surface-to-volume ratios give microparticles one advantage over macroparticles
• Greater surface area, combined with smaller volume of microsized particles, makes microparticles more difficult to
dislodge from plastic matrix. Moreover, when a microsized particle does pop out, it leaves a smaller crater behind and
is more resistant to wear
6. Answer: A
7. Answer: B
8. Answer: C
The conventional glass ionomer systems however suffer from certain disadvantages. These disadvantages are:
• Short working time
• Long set time
• Technique sensitivity
–– Susceptibility to early moisture contamination
–– Prone to desiccation after setting
• Brittleness

https://t.me/DentalBooksWorld
1118 Triumph's Complete Review of Dentistry

9. Answer: A (Ref. Basic Dental Materials, By John J. Manappallil, 2015, page no. 202)
Note: If the option is low volatile then choose adequate working time as answer … if not, low viscosity will remain as answer
• The American Academy of Pediatric Dentistry’s Pediatric Restorative Dentistry Consensus Conference confirmed
support for sealant use and published these recommendations:
–– Bonded resin sealants, placed by appropriately trained dental personnel, are safe, effective, and underused in
preventing pit and fissure caries on at-risk surfaces. Effectiveness is increased with good technique and appropriate
follow-up and resealing as necessary.
–– Sealant benefit is increased by placement on surfaces judged to be at high risk or surfaces that already exhibit incipient
carious lesions. Placing sealant over minimal-enamel caries has been shown to be effective at inhibiting lesion
progression. As with all dental treatment, appropriate follow-up care is recommended.
–– Sealant placement methods should include careful cleaning of the pits and fissures without removal of any appreciable
enamel. Some circumstances may indicate use of a minimal-enameloplasty technique.
–– Placement of a low-viscosity, hydrophilic material-bonding layer as part of or under the actual sealant has been shown
to enhance the long-term retention and effectiveness.
–– Glass ionomer materials have been shown to be ineffective as pit and fissure sealants but can be used as transitional
sealants.
–– The profession must be alert to new preventive methods effective against pit and fissure caries. These may include
changes in dental materials or technology
10. Answer: C (Ref. Phillip’s Science of Dental Materials, By Chiayi Shen, H. Ralph Rawls, 2014, page no. 316)
Chemical bonding is seen in GIC and zinc polycarboxylate.
Ionic bonding is seen in zinc phosphate cement.
Chemical bonding is not seen in zinc phosphate cement.
11. Answer: A (Ref. Basic Dental Materials, By John J. Manappallil, 2015, page no. 107)
GIC:
• Maximum particle size—50 µm
• Particle size for luting/cementing agents—15 µm
CaOH GIC: 15 µm
12. Answer: A
13. Answer: B
14. Answer: B
Its strength and not fluoride release (Ref. Anusavice)
15. Answer: A (Ref. Basic Dental Materials by John J. Manappallil, 2015, page no. 100)
The chemical composition of ZOE is typically
Zinc oxide: 69.0%
White rosin: 29.3%
Zinc acetate: 1.0% (improves strength)
Zinc stearate: 0.7% (acts as accelerator)—least
Liquid (eugenol: 85%, olive oil: 15%)
16. Answer: A (Ref. Craig’s Restorative Dental Materials, By Ronald L. Sakaguchi, John M.Powes, 2012, page no. 181)
• Estrogenicity—Potential of synthetic chemicals with a binding affinity for estrogen receptors to cause reproductive
alterations. Bisphenol-A, a precursor of certain monomers such as bis-GMA, is a known estrogenic compound that is
considered to have possible effects on fetal and infant brain development and behavior.
• Bisphenol A (BPA) is, by definition, a major component of Bis-GMA (bisphenol A glycidyl methacrylate or “Bowen’s
resin”), a molecule known to be at the basis of composites and sealants used in dentistry. In the international dental
literature articles regularly appear arguing that BPA and/or its derivatives might be released into the oral cavity from
composites and sealants in doses which can produce estrogenic effects.
17. Answer: C (Ref. Phillip’s Science of Dental Materials, By Chiayi Shen, H. Ralph Rawls, 2013, page no. 331)

DIRECT FILLING GOLD


1. Answer: C
2. Answer: C
3. Answer: C

https://t.me/DentalBooksWorld
Chapter 12 • Conservative Dentistry 1119

4. Answer: B
5. Answer: A
6. Answer: C

INSTRUMENTATION
1. Answer: A

Windmill
(rotor or turbine)

Bearing Housing

Light Outlet

Friction
grip bur

2. Answer: A
The free running speed of a turbine is in the order of 300,000–400,000 revolutions per minute (rpm). As the bur is
applied to the tooth the bur slows to a cutting speed of between 180,000 and 200,000 rpm. The optimum cutting speed is
approximately one-half of the free running speed.
3. Answer: A

Procedure Handpiece Gearing ratio Recommended cutting speeds


(rpm; revolutions per minute)
Cavity preparation High-speed/speed-increasing N/A1:5 230,000
Removing existing High-speed/speed-increasing N/A1:5 160,000–230,000
Intracoronal restorations
Caries removal Low-speed/speed-decreasing 2:1,10:1 1,500
Fine finishing and polishing Low-speed/speed-decreasing 2:1 20,000–40,000
Crown preparations High-speed/speed-increasing N/A1:5 160,000–230,000
Mechanical root canal Speed-decreasing 20:1 Nickel titanium 250–500
preparation
Implant placement Speed-decreasing 20:1 800–1000
Bone removal Low-speed/speed-decreasing – 600–2,000
Shaping titanium implant Low-speed/speed-decreasing – 6,000
abutment
Prophylaxis Low-speed/speed-decreasing 4:1 up to 10,000
Extraoral polishing Straight handpiece N/A 6,000
Cutting zirconia Speed-increasing 1:5 100,000
*These speeds are influenced by the type of bur being used. Tungsten carbide instruments require lower cutting speeds
than diamonds.

https://t.me/DentalBooksWorld
1120 Triumph's Complete Review of Dentistry

4. Answer: C
Blade Blade angle
length (4)
(3) (1) Blade width

(2) Cutting edge


angle


5. Answer: B
6. Answer: C

Distal Gingival Mesial Gingival


marginal trimmer marginal trimmer

7. Answer: B
8. Answer: B
9. Answer: B
10. Answer: C
Positive angle Negative angle

Positive and negative rake angles.


Positive angle results in cutting action.
Negative angle results in scraping action.
11. Answer: C
12. Answer: D
13. Answer: A
14. Answer: B
15. Answer: A

https://t.me/DentalBooksWorld
13 Endodontics

SYNOPSIS

DIAGNOSTIC AIDS IN ENDODONTICS


Diagnosis
Diagnostic testing of vital teeth

Diagnostic test Description


Percussion test Percussion refers to gently tapping or pressing the occlusal or lateral surface of a tooth. A painful
response indicates periradicular inflammation
Palpation test Firm pressure is applied to the mucosa above the apex of the root. This helps to determine whether
the inflammation process has extended into the periapical tissue
Cold test Ice, dry ice, or ethyl chloride is used to determine the pulp response to cold temperature (Refer
below for temperature)
Heat test A small piece of heated gutta-percha or the end of an instrument is heated and placed on the
occlusal or incisal surface of the tooth to determine the pulp response
Electric pulp testing Delivers a small electrical stimulus to the pulp
Visual and tactile Visual and tactile examination of hard and soft tissues relies on checking the three C’s – Color,
inspection Contour, and Consistency

Radiographic examination and interpretation


Tuned aperture computed tomography (TACT):
A new method for creating three-dimensional (3-D) radiographic displays based on optical aperture theory known as tuned-aperture
computed tomography (TACT).
TACT shows promise as a supplement to film-based dental radiography and as a digital alternative to conventional tomographic
systems used in dentoalveolar applications.
Pulpal testing:
• A positive response to pulp testing, thermal or electric, only confirms the existence of functional neural tissue
• Conventional pulp testing does not indicate the presence of normal healthy pulpal tissue with a normal blood flow
• Pulp tests are only assessing the pulp sensitivity, rather than pulp vitality
• Cold and electric tests appear to be more dependable tests than heat

Thermal testing • Application of cold or heat that provokes a long-lasting response is one that is stimulating the C fibers,
indicating the presence of active C fibers in inflamed tissue
• Therefore, determines a pulpal diagnosis of irreversible pulpitis
• A response that lingers for more than 15–20 seconds after the thermal stimulus has been removed is
frequently interpreted as indicating an irreversibly inflamed pulp
Cold testing • It can be accomplished by using one of several different cold stimulus including ice sticks (~0°C), ethyl
chloride (~−5°C), frozen carbon dioxide (~−75°C), or a pressurized refrigerant spray containing either
1,1,1,2 tetrafluoroethane, or dichlorodifluoromethane (~−26°C)
• Cold stimulus should be applied to a tooth until the patient definitively responds or for a maximum of
15 seconds

https://t.me/DentalBooksWorld
1122 Triumph's Complete Review of Dentistry

Heat testing • The simplest and most practical way of performing a heat test is to use gutta-percha softened in a flame
or with an electric heat carrier
• The application of the heat stimulus to a tooth should not last for more than 5 seconds in order to prevent
permanent pulpal damage
Test cavity • Is done using high speed bur with water coolant
• Done from posterior to anterior tooth

Electrical pulp test:


• Electric pulp testing (EPT) is based on the stimulation of sensory nerve fibers within the pulp. Electric pulp testing does not
give any information on the degree of health or disease of a pulp.
• A positive response is merely an approximate indicator of the presence of vital neural fibers in the pulp. It entails a subjective
evaluation from the patient.
• The basic principle of the electric pulp tester is to deliver a high-frequency direct electric current through the tooth.
• The electrical stimulus, like other pulp testing techniques, is best applied to the enamel of the buccal surface of the
mesiobuccal cusp.
• A small amount of a conducting agent such as toothpaste should be applied to the tip of the testing probe.

Conditions showing false positive response:


• Moist gangrenous pulp, which requires maximum current to elicit the response
• Presence of partially necrotic pulp in one of root canals in multirooted teeth
Conditions showing false-negative response:
• Recently traumatized teeth
• Recently erupted teeth with incomplete root formation
• Teeth with extensive restoration and pulp protecting base
• Patients under local anesthesia or sedatives
• Pulp stones
• High amounts of reparative dentin in tooth

Laser Doppler Flowmetry (LDF):


• The laser Doppler flowmetry technique was first described in dental literature in 1986 by Gazelius et al.
• This electro-optical technique uses a laser source that is aimed at the pulp, and the laser light travels to the pulp using the
dentinal tubules as guides
• The backscattered reflected light from circulating blood cells is Doppler-shifted and has a different frequency to the static
surrounding tissues
• The laser beam produced is a low power beam ranging from 1–2 mW. Different wavelength lasers can be produced by
different sources: 633 nm through Helium-neon laser, or 780 and 810 nm by semiconductor diode lasers
• The initial study by Gazelius et al. found that a 750 nm laser penetrated deeper but was associated with signal contamination
of nonpulp origin from surrounding tissues
Limitations of LDF:
• Contamination “noise” due to backscattered light from the periodontal tissues is impossible to eliminate in LDF even if with
a covering, such as a PVS splint
• The closer the probe is positioned to the gingival margin, the higher the signal output will be (due to greater pulp tissue
volume), but the potential gingival contamination
• Any obstruction and/or interference of the light pathway can render LDF useless—examples being restorations, especially
if they are full coverage crowns
• Pigmentation of the tooth structure that may also affect the scattering and filtering of light

https://t.me/DentalBooksWorld
Chapter 13 • Endodontics 1123

Pulse oximetry:
Compared to laser Doppler flow meters, pulse oximeters are relatively inexpensive and commonly used in general anesthetic
procedures. The term oximetry is defined as the determination of the percentage of oxygen saturation of the circulating arterial
blood.

It uses a probe containing a diode that emits light in two wavelengths:


• Red light of approximately 660 nm
• Infrared light of approximately 850 nm

Oxygenated hemoglobin and deoxygenated hemoglobin are different in color and therefore absorb different amounts of red and
infrared light.
Other noninvasive experimental tests:
(a) P
 hotoplethysmography is an analysis of the optical property of a selected tissue. It was developed for pulp testing in
an attempt to improve pulse oximetry, by adding a light with a shorter wavelength. The results, while promising, were
nonetheless equivocal.
(b) S
 pectrophotometry, using dual wavelength lights in an effort to ascertain the contents of enclosed spaces such as the pulp
chamber, has been tested with optimistic, but only initial, experimental results
(c) T
 ransmitted laser light (TLL) is an experimental variation to LDF, aimed at eliminating the nonpulp signals. TLL uses
similar sending/receiving probes as conventional LDF, but the probes are separate.

PULPAL AND PERIRADICULAR PATHOLOGY


Pulpal or periradicular inflammation results from irritation or injury usually from bacterial, mechanical, or chemical sources.
Classification of Pulp Disease
WHO Classification of Pulpal diseases
K04.04 Acute apical periodontitis
K04.05 Chronic apical periodontitis (apical granuloma)
K04.06 Periapical abscess with sinus
K04.61 Periapical abscess with sinus to maxillary sinus
K04.62 Periapical abscess with sinus to nasal cavity
K04.63 Periapical abscess with sinus to oral cavity
K04.07 Periapical abscess with sinus to skin
K04.08 Periapical abscess without sinus
K04.80 Radicular cyst
K04.81 Residual cyst
K04.82 Inflammatory paradental cyst

Reversible Pulpitis
Reversible pulpitis is a transient condition that may be precipitated by caries, erosion, attrition, abrasion, operative procedures,
scaling, or mild trauma. The symptoms are usually the following:
• Pain does not linger after the stimulus is removed
• Pain is difficult to localize (as the pulp does not contain proprioceptive fibers)
• Normal periradicular radiographic appearance
• Teeth are not tender to percussion (unless occlusal trauma is present)
Treatment involves covering up exposed dentine, removing the stimulus, or dressing the tooth as appropriate. Reversible pulpitis
may progress to an irreversible situation.

https://t.me/DentalBooksWorld
1124 Triumph's Complete Review of Dentistry

Irreversible Pulpitis
Irreversible pulpitis usually occurs as a result of more severe insults of the type listed above; typically, it may develop as a
progression from a reversible state. The symptoms are, however, different:
• Pain may develop spontaneously or from stimuli
• In the latter stages, heat may be more significant
• Response lasts from minutes to hours
• When the periodontal ligament becomes involved, the pain will be localized
• A widened periodontal ligament may be seen radiographically in the later stages
Treatment involves either root canal therapy or extraction of the tooth.
Hyperplastic Pulpitis
Hyperplastic pulpitis is a form of irreversible pulpitis and is also known as a pulp polyp. It occurs as a result of proliferation of
chronically inflamed young pulp tissue. Treatment involves root canal therapy or extraction.
Pulp Necrosis
Pulp necrosis occurs as the end result of irreversible pulpitis; treatment involves root canal therapy or extraction.
Hard Tissue Changes
Pulp calcification:
• Physiological secondary dentine is formed after tooth eruption and the completion of root development
• It is deposited on the floor and ceiling of the pulp chamber rather than the walls and with time can result in occlusion of the
pulp chamber
• Tertiary dentine is laid down in response to environmental stimuli as reactionary or reparative dentine
• Reactionary dentine is a response to a mild noxious stimulus whereas reparative dentine is deposited directly beneath the
path of injured dentinal tubules as a response to strong noxious stimuli. Treatment is dependent upon the pulpal symptoms
Internal resorption:
• Pulpal inflammation may cause changes that result in dentinoclastic activity. Such changes result in resorption of dentine;
clinically, a pink spot may be seen in the later stages if the lesion is coronal
• Radiographic examination reveals a punched out outline that is seen to be continuous with the rest of the pulp cavity. Root
canal therapy will result in arrest of the resorptive process
Acute apical periodontitis:
• Causes of acute apical periodontitis include occlusal trauma, egress of bacteria from infected pulps, toxins from necrotic
pulps, chemicals, irrigants, or overinstrumentation in root canal therapy
• Clinically, the tooth is tender to biting
• Widening of the periodontal space may be seen on a radiograph. Treatment depends on the pulpal diagnosis; it may range
from occlusal adjustment to root canal therapy or extraction
Chronic apical periodontitis:
• Chronic apical periodontitis occurs as a result of pulp necrosis
• Affected teeth do not respond to pulp sensitivity tests
• Tenderness to biting, if present, is usually mild; however, some tenderness may be noted to palpation over the root apex
• Radiographic appearance is varied, ranging from minimal widening of the periodontal ligament space to a large area of
destruction of periapical tissues
• Treatment involves root canal therapy or extraction
Condensing osteitis:
• Condensing osteitis is a variant of chronic apical periodontitis and represents a diffuse increase in trabecular bone in response
to irritation
• Radiographically, a concentric radio-opaque area is seen around the offending root
• Treatment is only required if symptoms/pulpal diagnosis indicate a need
Acute apical abscess:
• An acute apical abscess is a severe inflammatory response to micro-organisms or their irritants that have leached out into the
periradicular tissues

https://t.me/DentalBooksWorld
Chapter 13 • Endodontics 1125

• Symptoms vary from moderate discomfort or swelling to systemic involvement, such as raised temperature and malaise
• Teeth involved are usually tender to both palpation and percussion
• Radiographic changes are variable depending on the amount of periradicular destruction already present; however, usually
there is a well-defined radiolucent area, as in many situations an acute apical abscess is an acute exacerbation of a chronic
situation
• Phoenix abscess, which refers to an acute exacerbation of a chronic situation during treatment
• Initial treatment of an acute apical abscess involves removal of the cause as soon as possible
• Drainage should be established either by opening the tooth or incision into a dependent swelling
• An antibiotic may need to be prescribed, depending on the patient’s condition
• Once the acute symptoms have subsided, then root canal therapy or extraction may be performed
Chronic apical abscess:
• In a chronic apical abscess, the abscess has formed a communication through which it discharges
• Such communications may be through an intraoral sinus or, less commonly, extraorally
• Alternatively, the discharge may be along the periodontal ligament; such cases mimic a periodontal pocket
• Usually, these communications or tracts heal spontaneously following root canal therapy or extraction

ENDODONTIC MICROBIOLOGY

Bacterial genera represented in endodontic infections


Gram-Negative bacteria Gram-Positive bacteria
Anaerobes Facultatives Anaerobes Facultatives
Rods Rods
• Dialister • Capnocytophaga • Actinomyces • Actinomyces
• Porphyromonas • Eikenella • Pseudoramibacter • Corynebacterium
• Tannerella • Haemophilus • Filifactor • Lactobacillus
• Prevotella • Eubacterium
• Fusobacterium • Mogibacterium
• Campylobacter • Propionibacterium
• Pyramidobacter • Eggerthella
• Catonella • Olsenella
• Selenomonas • Bifidobacterium
• Centipeda • Slackia
• Atopobium
• Solobacterium
• Lactobacillus
Cocci Cocci
• Veillonella • Neisseria • Parvimonas • Streptococcus
• Megasphaera • Peptostreptococcus • Enterococcus
• Finegoldia • Granulicatella
• Peptoniphilus
• Anaerococcus
• Streptococcus
• Gemella
Spirilla
• Treponema

https://t.me/DentalBooksWorld
1126 Triumph's Complete Review of Dentistry

Asymptomatic Apical Periodontitis Symptomatic Apical Periodontitis Acute Apical Abscess


Porphyromonas endodontalis
Dialister invisus
Treponema denticola
Bacteroidetes clone X083
Tannerella forsythia
Pseudpramibacter alactolyticus
Dialister pneumosintes
Filifactor alocis
Porphyromonas gingivalis
Propionibacterium propionicum
Treponema socranskii
Streptococcus species
Parvimonas micra
Catonella morbi
Treponema parvum
Treponema maltophilum
Veillonella parvula
Olsenella uli
Fusobacterium nucleatum
Campylobacter rectus
Campylobacter gracilis
Pyramidobacter piscolens
Eikenella corrodens
Enterococcus faecalis
Prevotella nigrescens
Synergistes clone BH017
Treponema lecithinolyticum
Prevotella intermedia
Prevotella baroniae
0 25 50 75 100 0 25 50 75 100 0 25 50 75 100
% % %

Prevalence of bacteria detected in primary endodontic infections of teeth with different forms of apical periodontitis.

Gram-Negative Bacteria
• Gram-negative bacteria appear to be the most common microorganisms in primary endodontic infections. Species
belonging to several genera of gram-negative bacteria have been consistently found in primary infections associated with
different forms of apical periodontitis, including abscesses.
• These genera include Dialister (e.g., D. invisus and D. pneumosintes), Fusobacterium (e.g., F. nucleatum), Porphyromonas
(e.g., P. endodontalis and P. gingivalis), Prevotella (e.g., P. intermedia, P. nigrescens, P. baroniae, and P. tannerae), Tannerella
(e.g., T. forsythia), and Treponema (e.g., T. denticola and T. socranskii)

Gram-Positive Bacteria
• Even though anaerobic gram-negative bacteria are reported to be the most common microorganisms in primary infections,
several gram-positive bacteria have also been frequently detected in the endodontic mixed consortium, some of them in
prevalence values as high as the most commonly found gram-negative species.
• The genera of gram-positive bacteria often found in primary infections include Actinomyces (e.g., A. israelii), Filifactor
(e.g., F. alocis), Olsenella (e.g., O. uli), Parvimonas (e.g., P. micra), Peptostreptococcus (e.g., P. anaerobius, P. stomatitis),
Pseudoramibacter (e.g., P. alactolyticus), Streptococcus (e.g., S. anginosus group), and Propionibacterium (e.g., P. propionicum
and P. acnes)

ENDOMETRICS – WORKING LENGTH DETERMINATION


Working length is defined as “the distance from a coronal reference point to the point at which canal preparation and obturation
should terminate.”
Anatomy of the apex of the root

Pulp
Dentin
Cementum

Apical constriction
Apical foramen
Anatomical apex

https://t.me/DentalBooksWorld
Chapter 13 • Endodontics 1127

Some Definitions From Glossary of Endodontic Terms


Anatomy of the apical section Definition
The tip or end of the root as determined morphologically. Anatomical apex may
or may not coincide with the apical foramen. In most cases (50–98% of all roots),
Anatomic apex
the foramen deviates from the greater axis of the tooth, the mean distance between
the anatomical apex and the foramen being 0.5–1.0 mm
Apical foramen or major foramen The main apical opening of the root canal
Apical constriction or minor foramen The apical portion of the root canal having the narrowest diameter; position may
(minor apical diameter, minor diameter) vary but is usually 0.5–1.0 mm short of the center of the apical foramen
The region at which the dentin and cementum are united commonly. Its position
Cementodentinal junction (CDJ)
can be in the range 0.5–3.0 mm from the anatomic apex
The tip or end of the root as determined radiographically; its location can
Radiographic apex vary from the anatomical apex due to root morphology and distortion of the
radiographic image

Methods of Working Length Determination


Radiographical method Nonradiographical method
1. Grossman’s formula 1. Digital tactile sense
2. Ingle’s method 2. Apical periodontal sensitivity
3. Weine’s method 3. Paper point method
4. Radiovisiography 4. Electronic apex locator
5. Xeroradiography

Grossman’s method
Clt = kli × alt / ali where, clt = correct length of the tooth
Kli = known length of the instrument in the tooth
Alt = apparent length of the tooth on radiograph
Ali= apparent length of the instrument on radiograph

Ingle’s method
• Tooth length is measured in the preoperative radiograph
• 1 mm “safety allowance” is subtracted for possible image distortion
• The endodontic file is set at this tentative working length, and the instrument is inserted in the canal
• On the radiograph the difference between the end of file and root end is measured and this value is either subtracted or
added to the initial working length measurement depending on whether the file is short of apex or extended beyond apex
• From this adjusted working length 1 mm “safety factor” is subtracted again to confirm with the apical termination of
instrument

Weine’s modification
A. If, radiographically, there is no resorption of the root end or bone, shorten the length by the standard 1.0 mm
B. If periapical bone resorption is apparent, shorten by 1.5 mm
C. If both root and bone resorption are apparent, shorten by 2.0 mm

https://t.me/DentalBooksWorld
1128 Triumph's Complete Review of Dentistry

Classification of Apex Locators


First-generation apex locators • Resistance • Root canal meter, endodontic meter,
dentometer, endo radar
Second-generation apex locators • Measurements of electrical • Sono-explorer, endo cater, digipex
impedance • Exact-a-pex, Formatron IV, Sono-explorer
mark II, Sono-explorer mark II junior
Third-generation apex locators • Using two different frequencies at • Endex, neosono ultima ez, justy II, mark V
the same plus, root zx, apex pointer, root zx
• Time in order to measure the
difference or ratio between two
currents
Fourth-generation apex locator • Using two or more nonsimultaneous • Bingo 1020, raypex4, neosono mc, element
continuous frequencies in order diagnostic unit and apex locator, propex,
to measure the difference or ratio novapex, APEX-NRG XFR
between two currents
Fifth-generation apex locator • Measures the capacitance and • Propex II, Joypex V, I-root, raypex V
resistance of the circuit separately

Paper point method


In a root canal with an immature (wide open) apex, the most reliable means of determining wl is to gently pass the blunt end
of a paper point into the canal after profound anesthesia
The moisture or blood on the portion of the paper point that passes beyond the apex – an estimation of working length or the
junction between the root apex and the bone

ENDODONTIC INSTRUMENTS
Introduction
The principles of root canal treatment consist of thorough cleaning, adequate shaping, and complete filling of the root canal system.
Classification of endodontic instruments
According to function (by Grossman): Instruments are divided into four groups
1. Exploring instruments
To locate the canal orifice or to assist in obtaining patency of the root canal
(i) Smooth barbed broach
(ii) DG-16 explorer
2. Debriding instruments
To extirpate the pulp and to remove debris and other foreign materials – Barbed broach
3. Shaping instruments
To shape root canal apically and laterally
(i) Reamers
(ii) Files
4. Obturating instruments
To pack gutta-percha into root canal
(i) Plugger
(ii) Spreaders
(iii) Lentulo spirals

https://t.me/DentalBooksWorld
Chapter 13 • Endodontics 1129

Harty’s classification
1. Instrument for access cavity preparation
(i) Basic instrument pack
(ii) Burs
(iii) Rubber dam
2. Instruments for root canal preparation
(i) Hand instruments
(ii) Power-assisted root canal instruments
(iii) Electronic canal measuring device
(iv) Measuring instruments, gauges, and stands
(v) Instruments for retrieving broken instruments and posts
3. Instruments for filling root canals
(i) Lateral condensation
(ii) Vertical condensation
(iii) Hybrid technique
(iv) Thermoplasticized gutta-percha
4. Equipment for storing instruments
5. Sterilization of endodontic instruments
6. Equipment for improving visibility

Classification based on 12th edition Grossman:


• Group 1: Hand-operated endodontic instruments, broaches, files, and reamers
• Group 2: Nonrotary endodontic instruments
• Group 3: Rotary endodontic instruments

Rubber dam
• It protects the patient from inhalation or ingestion of instruments, medicaments, and debris.
• It prevents infection by providing a clean, dry, aseptic working field, free from salivary contamination.
• It allows retraction of soft tissues and the tongue so as not to obstruct the operating field and also protect them from injury.
• It enhances access thereby improving the efficiency of treatment.
• It provides better patient comfort without the oral cavity being flooded with water and/or debris.
–– The sheets come in different colors and thickness (thin, medium, heavy, extra heavy, and special heavy).
–– The thicker material has the advantage of a tighter fit around the neck of the tooth, thus providing a more hermetic seal,
so floss ligatures may not be required. It is also less likely to tear and offers better protection for the underlying soft tissues.
Rubber dam punch
• A punch is used to make the required numbers of holes depending on the teeth to be isolated. Usually single tooth isolation
is all that is required for endodontic treatment.
• The size of hole that is punched is also important; the ease of application with a larger hole must be balanced by the quality
of the seal at the cervical margin.
Rubber dam clamp
• Clamps have two uses: First, they anchor the rubber dam to the tooth, and second, they retract the gingivae
• Most clamps are made from stainless steel; some are made from plated steel, which may be more susceptible to corrosion
by sodium hypochlorite
• There are also nonmetallic clamps made of plastic (SoftClamp, KerrHawe, Bioggio, Switzerland) on the market.
• Clamps are winged or wingless, retentive

https://t.me/DentalBooksWorld
1130 Triumph's Complete Review of Dentistry

• A winged clamp allows the attachment of the rubber dam to the clamp so that both clamp and rubber dam may be applied
to the tooth together
• Retentive clamps are designed to make a four-point contact with the tooth; they have narrow, curved, and slightly inverted
jaws, which may displace gingival tissue to grip the tooth below the level of greatest circumference; they are very useful on
partly erupted teeth
• A basic assortment of clamps may consist of the following – Ivory pattern 00, 0, 1, 2A, 9, W8A, 14, and 14A
• The wingless W8A, the winged 14 and 14A are for molars, the 2A and 1 for premolars, and the 9 for incisors
• A range of lettered Ash clamps, which are generally smaller, may also be used; the winged K and E for molars and premolars
respectively, and the wingless EW for incisors and broken down premolars. The EW clamp gives better access than the
9 clamp.
Clamp forceps
The forceps are used to place, adjust, and remove the rubber dam clamp. Some forceps may require adjustment to their
working ends prior to first use.
Rubber dam frame
• Rubber dam frames come in various sizes and designs; they are shaped so that they do not impinge on the patient’s face
• Rubber dam frames are made from either metal or plastic; the latter is lighter, more comfortable and being radiolucent,
removal is not always necessary when taking radiographs
Methods of application
• In the first method, the rubber dam is attached to the clamp, with or without the frame beforehand, and the whole assembly
placed onto the tooth. In this method, only winged clamps can be used. Once the clamp is firmly seated, a plastic instrument
is used to lift the rubber off the wings to fit against the side of the tooth.
• In the second method, winged or wingless clamps may be used. The clamp is placed on the tooth and the dam is then
stretched over the clamp. If this technique is used, as mentioned earlier, the clamp should be wrapped in dental floss as a
precaution against clamp fracture or dislodgement.
• In the third method, the dam is stretched over the tooth and the clamp, winged or wingless, then placed on the tooth. The
assistance of a dental nurse is normally required for this method of application.
If more than one tooth is to be isolated, the rubber is knifed through each succeeding contact point. The rubber is stretched,
positioned vertically above the contact point and gently forced through the point.

Gates-Glidden
• The Gates-Glidden has a slender shank with a cutting bulb and a pilot-tip
• It is designed so that if it fractures, this will occur near the hub rather than between the shank and the cutting bulb
• Gates-Glidden burs are made of stainless steel and the set of six different sizes of burs have cutting bulbs with diameter
ranging 0.5–1.5 mm
• They are also available in different lengths, a standard 32 mm, a shorter 24 mm and a longer 36 mm. The Gates-Glidden
bur is operated at low speed
• It may be used for coronal root canal enlargement but there is a risk of furcal perforation in mandibular molars
• Gates-Glidden burs may also be used to remove gutta-percha in the coronal part of the root canal

INSTRUMENTS FOR ROOT CANAL PREPARATION


Instrument standardization
D1 is an assessment of the diameter of the working part at the tip end, and is its nominal size
D3 represents a point at 16 mm from D1 where the cutting part of the instrument ends
The taper is a constant 0.02 mm per mm of cutting flute; hence also referred to as 0.02 taper. The shape of the tip is variable.
The lengths of instruments available are normally 21, 25, and 31 mm.

https://t.me/DentalBooksWorld
Chapter 13 • Endodontics 1131

Size D1 (mm) Color


006 0.06 Orange
008 0.08 Gray
010 0.10 Purple
015 0.15 White
020 0.20 Yellow
025 0.25 Red
030 0.30 Blue
035 0.35 Green
040 0.40 Black
045 0.45 White
050 0.50 Yellow
055 0.55 Red
060 0.60 Blue
070 0.70 Green
080 0.80 Black
090 0.90 White
100 1.00 Yellow
110 1.10 Red
120 1.20 Blue
130 1.30 Green
140 1.40 Black

HAND INSTRUMENTS
Barbed broaches
• Barbed broaches are used mainly for the removal of pulp tissue from wide root canals, and cotton wool dressings from the
pulp chamber.
• Provided the instrument is loose within the canal and is used to engage soft tissue or dressings, the risk of fracture is minimal.
Barbed broaches are made from soft steel wire.
Reamers
• Reamers are manufactured by twisting a tapered stainless steel blank to form an instrument with sharp cutting edges along
the spiral.
• They are used with a half-turn twist and pull action, which shaves the canal wall, removing dentine chips from the root canal.
• Nominally they have a triangular cross-section, but the smaller sizes may be manufactured from a square blank.
Files
• There are various types of root canal file, and they are mostly made from stainless steel.
• Files are predominantly used with a filing or rasping action, in which there is little or no rotation of the instrument in the root
canal.
• The properties of different files are related to their design features. The common types of files on the market are:
–– K-file
–– K-flex file
–– Flexofile
–– Hedstrom file

https://t.me/DentalBooksWorld
1132 Triumph's Complete Review of Dentistry

K-file
• This file is so named as it was introduced by the Kerr Company.
• These files are made, like reamers, by twisting a triangular or square blank, but into a tighter series of spirals to produce from
0.9–1.9 cutting edges per millimeter length.
• They will work either in a reaming or a push-and-pull filing motion.
K-flex file
• The K-flex file was developed in an effort to improve on the original K-file design. It has a rhomboid-shaped cross-section.
• When the blank is twisted to form the instrument, it has a series of cutting flutes with alternate sharp (<60°) cutting edges and
obtuse noncutting edges.
• The high and low flute configuration is designed to endow the instrument with greater flexibility, and provide a reservoir for
the dentinal debris. A disadvantage of the K-Flex file is that it tends to lose its cutting efficiency quicker.

K-File K-Reamer
Cross-section Square Triangle
Flute number 1.5–2.5/mm 0.5–1/mm
Cutting angle 90 60
Clearance space Less More
Flexibility Less More

*Flex R *K – Flex (small) *K – Flex Hedstrom *Unifile


K – Reamer K – Reamer (large) *S – File
(large) (small)
K – File (large) K – File (small)

Flexofile
The Flexofile has a noncutting (Batt) tip and a triangular cross-section, so the cutting flutes are sharper and there is more room
for debris removal; it was reported to produce good instrumentation results.
The C + files
The C + files are designed with a robust quadrangular which shows greater resistance to deformation compared to corresponding
K-files. They are particularly useful in facilitating the location of the canal orifices and the initial exploration of calcified canals.
The files are available in ISO diameters 8, 10, and 15 with lengths of 18, 21, and 25 mm.
Hedstrom file
The Hedstrom file is made by a milling process from a steel blank of round cross-section to produce elevated cutting edges. The
tapering effect appears to form a series of intersecting cones. Although the design leads to a sharp and flexible instrument,
the file is inherently weaker due to the reduced shaft diameter and is therefore slightly more prone to breakage. It is most
effective when used in a pull motion. With sharp cutting flutes, it is also used to engage and remove retained instruments, gutta-
percha, and silver points.

Rotary NiTi instruments


• ProFile
• System GT
• Quantec

https://t.me/DentalBooksWorld
Chapter 13 • Endodontics 1133

• Lightspeed LSX
• Hero 642
• K3
• RaCe and BioRaCe
• ProTaper Universal
• Twisted Files
Variable taper
• The concept of taper variation is to maximize the cutting efficiency by minimizing the contact area between the surface of
the instrument and the canal wall. Instead of having to flare a canal using different sizes of standard 0.02 taper files to achieve
the desired shape, the preparation is produced by using files of the desired taper straightaway. The larger the taper, the more
conical the shape of the instrument.
• For ease of use, many rotary instrument systems have matching variable taper hand files, paper points, and gutta-percha
points.
Flute design
The shape of the flutes in cross-section determines cutting efficiency and the ability to remove debris. A design incorporating a
reservoir for the dentine debris will help effective evacuation as the debris is transported coronally.
Rake angle
The rake or cutting angle of most conventional instruments is negative, so the cutting blade scrapes rather than cuts the dentine,
and this is inefficient. A positive rake angle results in more effective cutting but if the rake angle is excessively positive, the
cutting blade will dig into the dentine substrate. Therefore, the rake angle should be only slightly positive

Design features of first- to fifth-generation rotary Ni–Ti file systems


Instrument system Cross-sectional design Taper Speed
First generation
Profile • Triple U shape with radial lands Fixed 2, 4, and 6 % 150 rpm
• Triple U shape with radial lands 1,500 rpm
LightSpeed Specific instrument sequences
• Files have short cutting portion
GT Files • Triple U shape with radial lands Fixed 4, 6, 8, 10, and 12 %
Second generation
• Convex triangular shape, sharp cutting edges 350 rpm
Variable taper along length of
ProTaper with no radial lands
instrument
• F3, F4, and F5 U flutes increase flexibility
• Positive rake angle for cutting efficiency 300 rpm
K3 • Three radial lands Fixed 2, 4, and 6 %
• Peripheral blade relief
• Triangular shape except RaCe 15/0.02 and 600 rpm
20/0.02 which have square shape
RaCe Fixed 2, 4, 6, 8, and 10 %
• Two alternate cutting edges
• No radial lands
Third generation
• Triangular shape
Vortex Blue • Manufactured M-Wire Fixed 4% and 6%
• No radial lands
• Triangular shape
Twisted Files Fixed 4% and 6%
• Manufactured R-phase wire twisted not ground

https://t.me/DentalBooksWorld
1134 Triumph's Complete Review of Dentistry

Instrument system Cross-sectional design Taper Speed


Fourth generation
WaveOne • Convex triangular shape Taper 6% or 8%
Fifth generation
ProTaper Next 150 rpm

• The most recent change has been the fifth-generation shaping files designed so that the center of mass and/or the center
of rotation is offset resulting in a mechanical wave of motion that travels the length of the file, minimizing engagement
between file and dentine.
• ProTaper: Next is the successor to the ProTaper Universal featuring M-Wire technology, offset design creating a “swaggering”
effect reducing taper lock and screw effect and minimizing file contact, thereby reducing the risk of file separation.

Cleaning and Shaping


Principles of Cleaning
• Nonsurgical root canal treatment is a predictable method of retaining a tooth that otherwise would require extraction
• Root canal treatment in a tooth with a vital pulp is more often successful than is root canal treatment of a tooth diagnosed
with a necrotic pulp and periradicular pathosis
• Currently, there are no reliable clinical methods to assess cleaning efficacy
• The presence of clean dentinal shavings, the color of the irrigant, and canal enlargement 3 file sizes beyond the first
instrument to bind have been used to assess the adequacy of cleaning; however, these do not correlate well with debridement
• Obtaining glassy smooth walls is a suggested indicator, but this property cannot be assessed beyond the coronal aspect of
the root canal system

Principles of Shaping
• The purpose of shaping is to facilitate cleaning and provide space for placing obturating materials
• The main objective of shaping is to maintain or develop a continuously tapering funnel from the canal orifice to the apex
• This reduces procedural errors during apical enlargement. The degree of enlargement is partly dictated by the method of
obturation. For lateral compaction of gutta-percha, the canal should be enlarged sufficiently to permit placement of the
spreader to within 1–2 mm of the working length
• There is a correlation between the depth of spreader penetration and the quality of the apical seal. For warm vertical compaction
techniques, the coronal enlargement must permit the placement of pluggers to within 3–5 mm of the working length
• As dentin is removed from the canal walls, the root becomes less resistant to fracture
• The degree of shaping is determined by preoperative root dimensions, the obturation technique, and the restorative treatment plan
• Narrow, thin roots (e.g., those of the mandibular incisors) may not be enlarged to the same degree as more bulky roots
(e.g., those of the maxillary central incisors)

Apical Canal Preparation: Termination of Cleaning and Shaping


• The apical point of termination, also known as the working length (WL), has been 1 mm from the radiographic apex. A classic
study described the apical portion of the canal with the major diameter of the foramen and the minor diameter of the constriction.
• The apical constriction is defined as the narrowest portion of the canal; the average distance from the foramen to the constriction
was found to be 0.5 mm. Another study found the classic apical constriction to be present in only 46% of the teeth studied.
• A study found no typical pattern for foraminal openings; it also found that no foramen coincided with the apex of the root.
The same group reported the foramen-to-apex distance to range from 0.2–3.8 mm.
• The foramen-to-constriction distance increases with age, and root resorption may destroy the classic anatomic constriction.
Resorptive processes are common with pulp necrosis and apical bone resorption. Therefore root resorption is an additional
factor to consider in length determination.

https://t.me/DentalBooksWorld
Chapter 13 • Endodontics 1135

Irrigants
Properties of an ideal irrigant
• Organic tissue solvent
• Inorganic tissue solvent
• Antimicrobial action
• Nontoxic
• Low surface tension
• Lubricant
Sodium Hypochlorite
• The most common irrigant is NaOCl, also known as household bleach. The advantages of an NaOCl solution include
mechanical flushing of debris from the canal, the ability to dissolve vital and necrotic tissue, antimicrobial action, and
lubricating action. In addition, NaOCl is inexpensive and readily available.
• Free chlorine in NaOCl dissolves necrotic tissue by breaking down proteins into amino acids. There is no proven appropriate
concentration of NaOCl, but concentrations ranging from 0.5–5.25% have been recommended.
• A common concentration is 2.5%, a concentration at which the potential for toxicity is reduced, yet some tissue dissolving
and antimicrobial activity is maintained.
• Because the action of the irrigant is related to the amount of free chlorine, an increase in volume can compensate for a
decrease in concentration. Warming the solution can also increase its effectiveness. However, NaOCl is limited in its ability
to dissolve canal content because of limited contact with tissues in all areas of the root canal system.
• Because of toxicity, extrusion is to be avoided. The irrigating needle must be placed loosely in the canal.
• Insertion to binding and slight withdrawal minimizes the potential for extrusion and a sodium hypochlorite accident.
• Special care should be taken when irrigating a canal with an open apex. To control the depth of insertion, the needle is bent
slightly at the appropriate length or a rubber stopper is placed on the needle.
• The irrigant does not move apically more than 1 mm beyond the irrigation tip, so deep placement with small-gauge needles
enhances irrigation.
• During rinsing, the needle is moved up and down constantly to produce agitation and prevent binding or wedging of
the needle.
Ethylenediaminetetraacetic acid (EDTA)
• Ethylenediaminetetraacetic acid (EDTA) is another frequently used irrigant; its activity is directed toward removal of the
smear layer.
• Irrigation with 17% EDTA for 1 minute, followed by a final rinse with NaOCl, is a recommended method.
• Chelators such as EDTA remove the inorganic components and leave the organic tissue elements intact. NaOCl is then
necessary for removal of the remaining organic components; however, the use of NaOCl after chelating agents may lead to
excessive demineralization of radicular wall dentin.
• Demineralization results in removal of the smear layer and plugs and enlargement of the tubules. The action is most effective
in the coronal and middle thirds of the canal and is diminished in the apical third.
• Reduced efficacy may be a reflection of canal size or anatomic variations such as irregular or sclerotic tubules. The variable
structure of the apical dentin presents a challenge during endodontic obturation with adhesive materials.
• The recommended time for removal of the smear layer with EDTA is 1 minute. The small particles of the smear layer are
primarily inorganic and have a high surface-to-mass ratio, which facilitates removal by acids and a chelator.
• EDTA exposure over 10 minutes causes excessive removal of both peritubular and intratubular dentin. Citric acid has also
been shown to be an effective means of removing the smear layer.
Chlorhexidine
• Chlorhexidine has a broad spectrum of antimicrobial activity, provides a sustained action, and has little toxicity.
• 2% chlorhexidine has an antimicrobial action similar to that of 5.25% NaOCl, and it is more effective against Enterococcus
faecalis.
• NaOCl and chlorhexidine are synergistic in their ability to eliminate microorganisms. A disadvantage of chlorhexidine is
its inability to dissolve necrotic tissue and remove the smear layer. Moreover, clinical studies do not confirm that the use of
chlorhexidine is associated with better outcomes.

https://t.me/DentalBooksWorld
1136 Triumph's Complete Review of Dentistry

MTAD
• An alternative method for disinfecting while at the same time removing the smear layer uses a mixture composed of a
tetracycline isomer, an acid, and a detergent (MTAD) as a final rinse to remove the smear layer.
• The effectiveness of MTAD in completely removing the smear layer is enhanced when low concentrations of NaOCl are
used as an intracanal irrigant before the use of MTAD. A 1.3% NaOCl concentration was recommended; MTAD may be
superior to NaOCl in antimicrobial action.
• MTAD has been shown to be effective in killing E. faecalis, an organism commonly found in failing treatments, and it may
prove beneficial during retreatment.
• It is biocompatible, does not alter the physical properties of the dentin, and enhances bond strength. Although there is
encouraging in vitro data, MTAD has not been shown to be clinically beneficial at this point.
QMix
• Use of a recently introduced irrigant, marketed as QMix, follows an underlying strategy similar to that for MTAD.
• QMix has the potential not only to remove the smear layer, but also to provide antibiofilm activity.
• QMix consists of a proprietary mix of chlorhexidine, EDTA, and a surface-active agent. Although this is a new material and
nothing is known about its contribution to clinical outcomes, it appears that smear layer removal is similar to that seen with
17% EDTA, and antimicrobial effects are adequate.
Ultrasonics
• Ultrasonic activation is used to enhance irrigation and to remove materials from the canal, including posts and silver cones.
• Ultrasonically powered instruments may also be used for thermoplastic obturation and root-end preparation during
surgery; however, shaping curved root canals with ultrasonic instruments has been shown to create preparation errors and
is no longer recommended.
• The main mechanism of adjunctive cleaning with ultrasonics is acoustic microstreaming, which is described as complex,
steady-state streaming patterns in vortex-like motions or eddy flows that are formed close to the instrument.
• Agitation of the irrigant with an ultrasonically activated instrument after completion of cleaning and shaping has the benefit
of increasing the effectiveness of the solution.

Preparation Errors
• These include loss of working length, apical transportation, apical perforation, instrument fracture, and stripping
perforations.
• Loss of working length has several causes, including failure to have an adequate reference point from which the working
length is determined, packing of tissue and debris in the apical portion of the canal, ledge formation, and inaccurate
measurement of files.
• Apical transportation and zipping occur when relatively inflexible files are used to prepare curved canals. The restoring
force of the file (the tendency to return to the original straight shape of the file) exceeds the threshold for cutting dentin in
a curved canal.
• When this apical transportation continues with larger and larger files, a teardrop shape develops, and apical perforation can
occur on the lateral root surface.
• Transportation in curved canals already begins with a No. 25 file. Enlargement of curved canals at the working length
beyond a No. 25 file can be done only when an adequate coronal flare is developed.
• Instrument fracture occurs with torsional and cyclic fatigue. Locking the flutes of a file in the canal wall while continuing to
rotate the coronal portion of the instrument is an example of torsional fatigue.
• Cyclic fatigue results when strain develops in the metal. File fracture occurs more frequently with rotaries but may also
involve hand instruments such as K-type and Hedstrom files.
• Stripping perforations occur in the furcal region of curved roots and frequently in the mesial roots of maxillary and
mandibular molars.
• The canal in this area of the root is not always centered in cross-sections; before preparation, the average distance to the
furcal wall (danger zone) is less than the distance to the bulky outer wall (safety zone). An additional complicating factor is
the furcal concavity of the root.

https://t.me/DentalBooksWorld
Chapter 13 • Endodontics 1137

OBTURATION
A canal is obturated after preparation to block all the portal of entries into the root canal system through which microorganisms
and their irritants can enter the canal and cause reinfection, also to seal the irritants left out in the canal, which cannot be
removed by cleaning and shaping procedures.
So a perfect seal is needed at the apex, lateral and accessory canals, and the coronal orifice.

Objectives of “Obturation”
The objective of obturation is to create a watertight seal along the length of the RCS from the coronal opening to the apical
termination.
Traditionally the importance of establishing and maintaining a coronal seal has been overlooked; the quality of the coronal
seal was not deemed important.
A watertight coronal seal can prevent residual microbes in the RCS from gaining access to the periodontal ligament, causing
disease.
It also minimizes the entry of new microbes into the RCS from the apical foramen, lateral or accessory canals, coronal opening,
or odontoblastic tubule dead tracts.

Core Obturating Materials


Obturating materials may be introduced into the canals in different forms and may be manipulated by different means once
inside
Desirable Properties of Obturating Materials
Grossman suggested that the ideal obturant would do the following:
• Be easily introduced into the canal
• Seal the canal laterally and apically
• Not shrink after insertion
• Be impervious to moisture
• Be bactericidal or at least discourage bacterial growth
• Be radiopaque
• Not stain the tooth structure
• Not irritate periapical tissues or affect the tooth structure
• Be sterile or easily sterilized
• Be easily removed from the root canal
Currently, no material or combination of materials satisfies all these criteria.

Gutta-Percha
• The primary bulk ingredient of a GP cone is zinc oxide (±75%). GP, which is a congener of rubber, accounts for approximately
20% and gives the cone its unique properties (e.g., plasticity). The remaining ingredients are binders, opaquers, and coloring
agents.

Advantages:
• Because of its plasticity, it adapts with compaction to irregularities in prepared canals, especially when thermoplasticized.
• It is relatively easy to manage and manipulate, even with complex obturation techniques.
• GP is relatively easy to remove from the RCS, either partially to allow post placement or totally for retreatment.
• GP is relatively biocompatible, being nearly inert over time when in contact with connective tissue. If a cone becomes
contaminated, it can be effectively sterilized by immersion in sodium hypochlorite (1% concentration or greater) for
1 minute

https://t.me/DentalBooksWorld
1138 Triumph's Complete Review of Dentistry

Resin
• Synthetic polyester resin-based polymers have been advocated as an obturation material
• The core material, composed of polycaprolactone with fillers of bioactive glass and other components, is used with a dual-
cured Bis-GMA resin sealer and self-etching primer
• This combination is an attempt to form a single entity, or so-called monoblock, in the RCS; it involves a chemical bond
between the sealer and dentin and the sealer and core material
• The material has been reported to be noncytotoxic, biocompatible, and nonmutagenic and has been approved for use by the
U.S. Food and Drug Administration (FDA)
Silver Points
• Silver points were designed to correspond to the last file size used in preparation and presumably to fill the RCS precisely
in all dimensions
• Although the short-term sealability success of silver points seemed comparable to that of GP, silver points are a poor long-
term choice as a routine obturating material
• Their major disadvantages are lack of adaptability and possible toxicity to periapical tissues from corrosion
• Also, because of their tight frictional fit, silver cones are difficult to remove, either totally during retreatment or partially
during post space preparation
• Silver cones are not recommended
Pastes (Semisolids)
Types
Zinc Oxide–Eugenol (ZnOE)
• ZnOE may be used in its pure state in primary teeth because it is resorbable as the tooth is exfoliated. However, it is generally
not advocated in permanent teeth. Other formulations combine ZnOE with various additives. The types known as N2 and
RC2B are most common.
• These are derivations of Sargent’s formula and contain opaquers, metallic oxides (lead) or chlorides (mercuric), steroids
(at times), plasticizers, paraformaldehyde, and various other ingredients.

Sealers
• Sealer, as an adjunct, accomplishes the objective of creating a watertight seal
• Sealer must be used in conjunction with the primary obturating material, regardless of the technique or material used
• This makes the physical properties and placement of the sealer important
Types
The four major types of sealers are ZnOE-based, plastics, glass ionomer, and those containing calcium hydroxide
ZnOE-Based Sealers
The major advantage of ZnOE-based sealers is their long history of successful use. Obviously, their positive qualities outweigh
their negative aspects (staining, a very slow setting time, nonadhesion, and solubility)
Grossman’s Formulation
Grossman’s formula is as follows:
• Powder: Zinc oxide (body), 42 parts; staybelite resin (setting time and consistency), 27 parts; bismuth subcarbonate, 15
parts; barium sulfate (radiopacity), 15 parts; sodium borate, 1 part
• Liquid: Eugenol
Most ZnOE sealers in use today are variations of this original formula. Three problems with this formulation are its very slow
setting time, toxic effects on host tissue, and lack of adhesiveness.
Epoxy Resin
• Epoxy has traditionally been available in a powder–liquid formula (AH26, AH Plus, and ThermaSeal Plus)
• Its advantages include antimicrobial action, adhesion, a long working time, ease of mixing, and very good sealability. Its
disadvantages are staining, relative insolubility in solvents, some toxicity when unset, and some solubility to oral fluids
• There are newer formulations without hexamine tetramine, which has been implicated in postobturation sensitivity. This
formulation is also easier to mix because it is composed of two pastes mixed equally

https://t.me/DentalBooksWorld
Chapter 13 • Endodontics 1139

Calcium Hydroxide
• Calcium hydroxide was originally introduced to the field of endodontics by Herman in 1930 as a pulp-capping agent, but its
uses today are widespread in endodontic therapy.
• It is the most commonly used dressing for treatment of the vital pulp.
• It also plays a major role as an intervisit dressing in the disinfection of the root canal system.
Mode of action
• A calcified barrier may be induced when calcium hydroxide is used as a pulp-capping agent or placed in the root canal in
contact with healthy pulpal or periodontal tissue.
• Because of the high pH of the material, up to 12.5, a superficial layer of necrosis occurs in the pulp to a depth of up to 2 mm.
• Beyond this layer only a mild inflammatory response is seen, and providing the operating field was kept free of bacteria
when the material was placed, a hard tissue barrier may be formed.
Structure
1. Arrangement = amorphous matrix, crystalline fillers
2. Bonding = covalent; ionic
3. Defects = pores, cracks.
4. Setting reaction = acid–base reaction
Physical properties
1. L.C.T.E = low
2. Thermal conductivity = insulator
3. Electrical conductivity = insulator
Chemical properties
1. Solubility: 0.3–0.5
Mechanical properties
1. Elastic mod = 588
2. Compressive strength >24 hour = 138
Biologic properties
1. Biocompatible
Advantages of calcium hydroxide
• Initially bactericidal then bacteriostatic
• Promotes healing and repair
• High pH stimulates fibroblasts
• Neutralizes low pH of acids
• Stops internal resorption
• Inexpensive and easy to use
Disadvantages of calcium hydroxide
• Does not exclusively stimulate dentinogenesis
• Does exclusively stimulate reparative dentin
• Associated with primary tooth resorption
• May dissolve after 1 year with cavosurface dissolution
• May degrade during acid etching
• Degrades upon tooth flexure
• Marginal failure with amalgam condensation
• Does not adhere to dentin or resin restoration

https://t.me/DentalBooksWorld
1140 Triumph's Complete Review of Dentistry

ENDODONTIC MICROSURGERY
Classification

Kim and Kratchman classified periradicular lesions into categories A–F


• Lesion Types A, B, and C represent lesions of endodontic origin and are ranked according to increasing size of periradicular
radiolucency
• Lesion Types D, E, and F represent lesions of combined endodontic–periodontal origin and are ranked according to the
magnitude of periradicular breakdown
• Class A – Represents the absence of a periapical lesion, no mobility, and normal pocket depth, but unresolved symptoms
after nonsurgical approaches have been exhausted. Clinical symptoms are the only reason for the surgery.
• Class B – Represents the presence of a small periapical lesion together with clinical symptoms. The tooth has normal
periodontal probing depth and no mobility. The teeth in this class are ideal candidates for microsurgery.
• Class C – Teeth have a large periapical lesion progressing coronally, but without periodontal pocket and mobility.
• Class D – Are clinically similar to those in Class C, but have deep periodontal pockets.
• Class E – Teeth have a deep periapical lesion with an endodontic–periodontal communication to the apex but no obvious
fracture.
• Class F – Represents a tooth with an apical lesion and complete denudement of the buccal plate but no mobility

• Classes A, B, and C present no significant treatment problems, and the conditions do not adversely affect treatment outcomes
• Classes D, E, and F present serious difficulties

Indications and Contraindications


Indications and contraindications for surgical endodontics

A. Indications
(i)  Periradicular disease associated with a well-treated previous root treatment where retreatment would be deemed
detrimental to the tooth or where no improvement may be gained
(ii) P
 eriradicular disease associated with anatomical deviations such as tortuous roots, sharp angle bifurcations, pulp
stones, and calcifications preventing nonsurgical retreatment to be undertaken
(iii) Periradicular disease associated with procedural errors such as instrument fractures, ledges, blockages, or perforated
canals, which cannot be corrected nonsurgically
(iv) Where a biopsy of the periradicular tissues is indicated
(v) E
 xploratory surgery to visualize the periradicular tissues and tooth/root is required when perforation or fracture is
suspected
B. Contraindications
(i) Anatomical factors
Proximity to neurovascular bundles, unusual bone or root configurations, proximity to maxillary sinus, lower second
molars with thick cortical plate, and lingual inclination of roots. Limited mouth opening resulting in reduced surgical
access
(ii) Periodontal and restorative factors
Poor supporting structures, active moderate–severe periodontal disease, and failing or failed coronal restorations
(iii) Medical factors
Severe systemic disease (ASA III–IV), patients with diseases such as leukemia or severe neutropenia in the active
stage and uncontrolled diabetes or patients who have recently undergone cardiac or cancer therapy
(iv) Surgeon’s skill and ability
The clinician’s surgical skills and knowledge. Where in doubt a referral should be made to an appropriate endodontist.

https://t.me/DentalBooksWorld
Chapter 13 • Endodontics 1141

Differences between traditional and microsurgical approaches in surgical endodontics:


Traditional Microsurgery
Osteotomy size Large 8–10 mm Small 3–4 mm
Bevel angle Long 45–65° Shallow 0–10°
Resected root surface inspection Not possible with standard instruments Always possible
Isthmus identification Impossible Possible
Root-end preparation Difficult to ensure only within the canal Always within the canal
Root-end filling material Amalgam often used MTA or Super EBA
Sutures 3–0 or 4–0 silk 5–0 or 6–0 monofilaments
Suture removal 7 days postoperatively 2–3 days postoperatively
Healing success at 1 year 40–90% 85–97%

Flap Design
• A number of basic flap designs exist including envelope, triangular, rectangular, semilunar, Ochsenbein–Luebke and papillary
base preservation flap
• Maxillary palatal root access requires a palatal flap to be elevated with separate design features
• It is critical that tissue incision, reflection, and retraction are performed in a way that allows for healing by primary intention

Triangular/rectangular flap • The triangular flap design comprises a horizontal incision extending to several teeth and
distal to the involved tooth and one vertical relieving incision placed mesially.
• This type of flap can be used for periapical surgery, root resorption, cervical resorption,
perforation, and resection of short roots.
• The main advantage of this type of flap is minimal disruption of the vascular blood supply
of the reflected tissues and ease of repositioning.
Submarginal flap • Often referred to as the Ochsenbein–Luebke flap, it is similar to the rectangular flap except
the scalloped horizontal incision is placed within the attached gingivae.
• This flap was used to prevent recession in aesthetically demanding cases.
• The disadvantages of this flap include risk of scar formation, possibility of incisions in close
proximity to the bony cavity resulting in wound dehiscence, and loss of attachment.
Papillary preservation flap • For a papillary-based flap a split thickness incision is made; the first is a shallow cut
perpendicular to the gingival margin.
• It is meant to sever the epithelium and connective tissue to a depth of approximately 1.5 mm
from the surface of the gingiva.
Envelope flap • A simple horizontal intrasulcular incision is made following the labial contour of the teeth
• No vertical incisions are made allowing ease of repositioning.
• This type of flap design is only useful in case of cervical resorption defects, cervical
perforations, and periodontal procedures.
• Due to limited access and visibility, this type of flap is not indicated in periradicular surgery.
Semilunar flap • This type of flap has been indicated when carrying out surgical trephination or where
aesthetic crowns are at risk of gingival recession from the proposed surgery.
• The flap itself expedites surgery by reducing incision and reflection times, maintains the
integrity of the gingival attachment and eliminates potential crestal bone loss.
• Disadvantages include limited access and visibility, difficulties repositioning, increased
incidence of postoperative scarring, predisposition to stretching and tearing of the flap, and
difficulties exposing the lesion in its entirety.

https://t.me/DentalBooksWorld
1142 Triumph's Complete Review of Dentistry

General points to consider regarding incisions and flap design


• Firm continuous incisions should be made to sound bone
• An incision should not cross an underlying bony defect
• The vertical incision should be in the concavities between bone eminences
• The vertical incision should not extend into the mucobuccal fold
• The termination of the vertical incision at the gingival crest must be at the mesial or distal line angle of the tooth
• The base of the flap must be at least equal to the width of its free end
• The flap should offer adequate access and have an adequate blood supply
• The flap must be of adequate size and fully reflected
• The edges must lie on the sound bone
• The flap should be protected throughout the surgical procedure with care not to cause further trauma to the soft tissues.

ENDODONTIC–PERIODONTIC RELATIONSHIP
• The pulpal precursor, the dental papilla, is separated from the periodontal tissue precursors by the Hertwig’s epithelial root
sheath
• Both diseases have their terminal effects in the periodontal tissues
• Simring and Goldberg first described this relationship and coined the term “endo-perio”
• Establishment of the correct diagnosis is crucial to provision of the correct treatment
• This is frequently difficult to establish but especially so in multirooted teeth for a number of reasons
• Molar teeth are morphologically complex and are often difficult to image due to their locations and surrounding structures
such as the zygomatic arch on upper teeth and mandibular tori on the lower
• They have multiple canal spaces complicating vitality testing; some canal spaces may maintain vitality while others become
necrotic
• They are associated with more vertical root fractures than anterior and premolar teeth
• A diagnosis is based on taking a thorough history and full clinical exam as stated earlier

Simon et al. Classification


Coding based on the nature of the diseases and described five distinct categories
Type-1: The primary endodontic lesion
Type-2: The primary endodontic lesion with secondary periodontic involvement
Type-3: The primary periodontic lesion
Type-4: The primary periodontic lesion with secondary endodontic involvement
Type-5: The true combined lesion (both lesions present and have joined in periradicular tissues)
• Belk and Gutmann suggested a sixth category and called this the “concomitant lesion”
• This describes the situation where both endodontic and periodontal lesions exist but are wholly independent of each other

Torabinejad and Trope Classification (1996)


Based on the origin of the periodontal pocket:
• Endodontic origin
• Periodontal origin
• Combined endo-perio lesions
• Separate endodontic and periodontal lesions
• Lesions with communication
• Lesions with no communication

https://t.me/DentalBooksWorld
Chapter 13 • Endodontics 1143

Acute or “blow-out” lesions


When a patient presents with a localized swelling that involves the gingival sulcus, it may be difficult to determine whether the
swelling is due to a periodontal abscess or an abscess of endodontic origin. The tooth must be nonvital. The swelling is usually
on the labial side of the tooth but may occasionally be on the lingual side. As the sulcus is probed, there is usually normal
sulcus depth all the way around the tooth until the area of the swelling is probed. At the edge of the swelling the probe drops
significantly to a level near the apex of the tooth and the probing depth remains the full width of the swelling. At the opposite
edge of the swelling, probing is once again within normal limits. The width of the detached gingiva can be as broad as the
entire buccal or lingual surface of the tooth. This swelling can be characterized as having “blown out” the entire attachment
on that side. Endodontic treatment only is indicated. As the result of endodontic management of the swelling, complete
periodontal reattachment occurs within 1 week in most cases.

Sequencing treatment for endo-perio lesions


1. Periodontal treatment
It is a known fact that root canal infection significantly affects periodontal healing. Pocket depth reduction is significantly
lesser in the presence of canal infection.
2. Endodontic treatment
• Early initiation of endodontic treatment ensures that the cementum layer is kept intact until root canal infection is
eliminated.
• Because there would be no exposed dentine on the root surface, there is reduced chance of root resorption and improved
periodontal healing.
3. Treating both lesions concurrently
This would be required when both endodontic and periodontal infection are present simultaneously. Combined endo-
perio lesions that exist separately on the same tooth (meaning that they are not physically merged) have recently gained a
lot of attention.

The protocol for management


Initial management
• Remove existing restorations and caries
• Chemomechanically prepare canals
• Medicate canals (depends on symptoms)
Follow-up management
• Change intracanal dressing after 3–4 weeks
• Provide initial periodontal treatment
• Review healing after 3 months
• Reassess need for further periodontal treatment
If more periodontal treatment (e.g., surgery) is required,
• Change intracanal medication again
If healing response is favorable,
• Complete root canal filling
Longer-term management
• Defer root filling until after
• Need for periodontal surgery assessed
• Surgery completed with satisfactory outcome

https://t.me/DentalBooksWorld
1144 Triumph's Complete Review of Dentistry

LASERS IN ENDODONTICS
A laser is a device that emits light through a process of optical amplification based on the stimulated emission of electromagnetic
radiation. The term “laser” originated as an acronym for “light amplification by stimulated emission of radiation.”
Wavelengths
• Nd:YAG (neodymium:yttrium–aluminum–garnet) laser developed by Meyers
• Nd:YAG laser (1,064 nm)
• Diode lasers (from 810 nm to 1,064 nm)
• Erbium, chromium:YSGG (2,780 nm)
• Erbium:YAG lasers (2,940 nm)
• CO2 lasers (9,300 and 10,600 nm)
The use of light energy creates a new potential for treating the pulp in primary teeth. Studies have investigated the possibility
of using lasers for the pulpotomy procedure on primary teeth.
In 1996, Wilkerson reported in their study that after 60 days, the use of the argon laser for pulpotomies in swine showed that
all pulps appeared to have normal vitality and pulpal healing.
• The use of the Nd:YAG laser in pulpotomy procedures in primary teeth was reported to be successful in 1999 by Liu et al.
• In another study, Liu et al. (2006) compared the effect of Nd:YAG laser at 2 W (100 mJ, 20 Hz) for pulpotomy to formocresol
(1:5), reporting a significantly superior clinical success of the laser group (97%) in comparison with the formocresol (1:5)
group
• The use of CO2 for pulpotomy procedures was investigated by Shoji et al. (1985) which reported clinical success for pulp
therapy
• Elliot et al. (1999), comparing the effects of the CO2 laser technique to formocresol therapy
• The Er:YAG laser has demonstrated a successful alternative to conventional pulp therapy
• Kimura et al. (2003) reported success using the Er:YAG laser for pulpotomy
• Huth et al. (2005) compared four pulpotomy techniques: Diluted formocresol, ferric sulfate

MULTIPLE-CHOICE QUESTIONS

ANATOMY OF THE PULP CAVITY AND ROOT CANAL


1. Cervical cross-section of maxillary first premolar has
A. A round shape B. Elliptical shape
C. Oval shape D. Square shape
2. Of the following permanent teeth, which is least likely to have two roots?
A. Maxillary canine B. Mandibular canine
C. Maxillary first premolar D. Mandibular first premolar
3. Accessory canals are most frequently found in
A. The cervical one-third of the root B. The middle one-third of the root
C. The apical one-third of the root D. With equal frequency in all the above mentioned
4. A divided pulp canal is most likely to occur in the
A. Root of a maxillary canine B. Root of a mandibular canine
C. Root of a maxillary central incisor D. Lingual root of a maxillary first molar
5. Mandibular first molar has
A. 2 roots and 2 canals B. 2 roots and 3 canals
C. 3 roots and 3 canals D. 3 roots and 4 canals
6. Incidence of third root in upper first premolar
A. 6% B. 10%
C. 12% D. 1%

https://t.me/DentalBooksWorld
Chapter 13 • Endodontics 1145

7. Pulp chamber is stained by


A. 1% Methylene blue dye B. 95% Ethanol and 17% EDTA
C. 1% Toluidine blue D. Both A and B
8. Identify the Vertucci type


A. Type 4 B. Type 5
C. Type 6 D. Type 3
9. Tooth that shows all type of Vertucci classification is
A. Maxillary first premolar B. Maxillary second premolar
C. Maxillary first molar D. Mandibular second molar
10. Identify the isthmus type


A. Type 2 B. Type 3
C. Type 1 D. Type 5
11. The point where pulp tissue ends and periodontal tissue begins is
A. Cementodentinal junction (CDJ) B. Apical constriction
C. Apical foramen D. 3 mm from the apical foramen
12. Tooth that shows least size of main apical foramina
A. Mandibular incisor B. Maxillary premolars
C. Distal root of mandibular molars D. Maxillary incisors
13. Tooth which does not obey first law of symmetry
A. Maxillary molars B. Mandibular molars
C. Mandibular incisors D. Mandibular molars
14. Buccal access if for (PGI JUNE 2015 question)
A. Crowded teeth B. Rotated teeth
C. Lingually inclined teeth D. Tooth with recession
15. C-shaped canal most commonly seen in
A. Mandibular first molars B. Mandibular second molars
C. Maxillary second molars D. Maxillary first molars
16. Type 2 C-shaped canal is
A. The shape is an uninterrupted “C” with no separation or division
B. The canal shape resembles a semicolon resulting from a discontinuation of the “C” outline, but either angle alpha or
beta should be no less than 60 degrees
C. Two or three separate canals and both angles, alpha and beta, are less than 60 degrees.
D. Only one round or oval canal is in the cross-section
17. In anterior teeth, the starting location for access cavity is the center of the anatomic crown on lingual surface at
A. Angle to it B. In line to it
C. Perpendicular to it D. All of the above
18. Most common chances of pulpal exposure will be there if pulpal floor is made perpendicular to the long axis of which
tooth?
A. Maxillary first premolar B. Maxillary first molar
C. Mandibular first premolar D. Mandibular second premolar

https://t.me/DentalBooksWorld
1146 Triumph's Complete Review of Dentistry

19. Identify the tooth apex


Root canal
Cem

Dentine
ent

Dentine
um

A. (A) B. (B)
C. (C) D. None
20. Percentage of distal root with two root canals in mandibular molar
A. 10% B. 30%
C. 60% D. 1%
21. Morning glory appearance is present in
A. Floor of pulp chamber B. Between minor and major apical diameter
C. Between walls of root canal D. Tooth root apex and alveolar bone

DISEASES OF PULP AND PERIRADICULAR TISSUES


1. Patient reports with severe throbbing pain in relation to mandibular second molar. The tooth is sensitive to both hot
and cold food and elicits a painful response on percussion. What would be your treatment of choice?
A. Incision and drainage of periapical area B. Endodontic therapy
C. Indirect pulp capping D. Partial pulpectomy
2. The most important aspect of emergency treatment for an acute apical abscess is to
A. Produce sedation B. Obturation
C. Establish drainage D. None of the above
3. In treating a tooth with a nonvital pulp and a sinus, the sinus should be treated as follows
A. No special treatment B. Cauterization
C. Curettage of sinus D. Use of cautery to eliminate the sinus
4. A pulp polyp may arise in connection with
A. Chronic open pulpitis B. Pulp necrosis
C. Acute pulpitis D. Chronic periapical lesion
5. Internal resorption is due to ___________
A. Pulp necrosis B. Acute inflammation of pulp
C. Chronic inflammation of pulp D. None of the above
6. Most common cause of pulp pathology is
A. Microbes
B. Trauma
C. Leakage from filling materials
D. Pressure sensation from condensation of filling materials
7. Phoenix abscess is
A. Reversible pulpitis B. Acute apical periodontitis
C. Acute exacerbation of an existing chronic inflammation D. Chronic abscess
8. Which of the following is the method of treating internal resorption?
A. Extirpation of pulp and calcium hydroxide dressing B. Extirpation of pulp and formocresol dressing
C. Pulpotomy D. Pulp capping

https://t.me/DentalBooksWorld
Chapter 13 • Endodontics 1147

9. Periradicular tissues—Classification by WHO K04.6 stands for


A. Acute apical periodontitis B. Chronic apical periodontitis
C. Periapical abscess with sinus D. Radicular cyst

BIOMECHANICS AND INSTRUMENTS


1. ADA number for endodontic sealers
A. 56 B. 57
C. 63 D. 100
2. Speed of proTaper
A. 500–600 rpm B. 150–350 rpm
C. 800–1,000 rpm D. 100–250 rpm
3. HERO stands for
A. Heavy elastic in rotation B. High elastic in rotation
C. Heavy elastic rotary D. High elastic rotary
4. Ultrasonic devices were first introduced in endodontics by
A. Weller et al. B. Diamond
C. Richman D. Dai et al.
5. All has U-shaped cross-section except
A. Profile B. Light speed
C. Greater taper D. ProTaper
6. File is manufactured from a
A. Triangular blank B. Round blank
C. Square blank D. Diamond-shaped blank
7. The difference between a reamer and a file is in
A. The number of flutes B. The number of flutes and shape of the shaft
C. The number of flutes and the size of the shaft D. The shape and size of the shaft
8. An endodontic reamer is least likely to fracture when inserted in a
A. Dry, clean canal B. Wet, clean canal
C. Wet, debris laden canal D. Dry, debris laden canal
9. To prevent the fracture of an endodontic instrument, the number of turns given inside the canal in a single motion
should be
A. Less than ¼ of a turn B. ¼–½ turn
C. ½ of a turn D. More than ¾
10. First instrument inserted into the root canal through the access cavity is
A. Barbed broach B. H file
C. K file D. Any of the above
11. Difference between physical characteristics of reamers and files is
A. The cross-sections of reamers and files are square and triangular, respectively
B. The number of flutes on the blade are more in files than in reamers
C. The reamers have more flutes in the blade
D. Files have two superficial grooves to produce flutes in a double helix design
12. Gates-Glidden drill is
A. Flexible and used at low speed B. Flexible and used at high speed
C. Inflexible and used at low speed D. Inflexible and used at high speed
13. In balanced force technique, step 3 involves
A. Clockwise 90-degree turn B. Anticlockwise 180-degree turn
C. Anticlockwise 90-degree turn D. Full 360-degree turn

https://t.me/DentalBooksWorld
1148 Triumph's Complete Review of Dentistry

Step 1 Step 2 Step 3 Step 4

≈90° 180–270° ≈90° 360–720°

DISINFECTION AND OBTURATION


1. McSpadden compactor is of __________ shape
A. Reversible blade of K-file B. Reversible blade of H-file
C. Endosonic instrument D. None of the above
2. Activity of all disinfectants is
A. Reduced by organic debris or blood B. Increased with heat
C. Hastened by adding 5% oxygen D. Nullified by ethylene dioxide gas
3. The purpose of a root canal sealer is to
A. To seal the tubules of the dentine
B. Stimulate healing in periapical region
C. Prevent discoloration
D. Fill the space between solid core material and the pulp canal walls
4. An apical radiolucent area present in central incisor after 4 months of RCT is due to
A. Inadequate obturation and leakage from main canal B. Leakage from accessory canal
C. Leakage from gingival crevice D. Leakage from access opening
5. GP is plasticized (softened) by
A. Alcohol B. Chloroform
C. Eugenol D. EDTA
6. Best material for obturating a root canal of a tooth is
A. Thermoplastic GP B. Silver cone
C. Resorbable paste D. GP with sealer
7. The microorganisms previously present in the periapical tissues following obturation
A. Persist and stimulate the formation of a granuloma
B. Are eliminated by the natural defenses
C. Re-enter and reinfect the sterile canal and are removed by surgery
D. Are eliminated by the medicaments used in endodontic treatment
8. The largest component of gutta percha cones is
A. Gutta percha B. Zinc oxide
C. Resins and waxes D. Coloring agents
9. The vertical fracture of root seen with gutta percha obturation is due to
A. The vertical compression of warm GP points B. Lateral condensation of GP points
C. Use of spreader D. Injection molding technique
10. Active GP is
A. GP activated by heat B. GP + GIC impregnated on external surface
C. GP + CaOH impregnated on external surface D. GP + Resin impregnated on external surface
11. AH Plus—Setting time
A. 8 hours B. 6 hours
C. 12 hours D. 24 hours

https://t.me/DentalBooksWorld
Chapter 13 • Endodontics 1149

12. Gutta percha—Sterilization


A. 5.25 % NaOCl for 1 minute B. 5.25 % NaOCl for 1 minute and 70 % alcohol
C. 5.25 % NaOCl for 1 minute and 100 % alcohol D. 2.25 % NaOCl for 1 minute and 70 % alcohol

BLEACHING
1. The most common consequence of bleaching nonvital teeth is
A. Discoloration B. Cervical resorption
C. Apical periodontitis D. Root resorption
2. Superoxol is
A. 30% H2O2 B. Combination of H2O2 + sodium per borate
C. Combination of HCl + H2O2 D. None of the above
3. Home bleach is also known as
A. Walking bleach B. Thermocatalytic bleaching
C. Power bleach D. All of the above
4. In the walking bleach technique
A. It uses a heat treatment
B. It requires the patient to report in 24 hours
C. Can be done with 35% hydrogen peroxide
D. It uses a mixture of sodium perborate and hydrogen peroxide
5. When undertaking bleaching of vital teeth, all of the following are done except
A. Prophylaxis B. Use of protective eye glasses
C. Use of local anesthesia D. Polishing after treatment
6. Tooth discoloration is due to
A. Acute pulpal abscess B. Pulpal hyperemia
C. Pulpal death D. None of the above
7. Carbamide solution used for bleaching degrades into
A. 0.3% sodium perborate B. 30% hydrogen peroxide
C. 3% hydrogen peroxide D. 30% sodium perborate
8. Vital bleaching causes
A. Internal resorption B. Cervical resorption
C. External resorption D. Periapical periodontitis
9. A 12-year-old child comes to your office with a history of long-term use of tetracycline. The anterior teeth are a mild
yellowish brown. What method would you use to remove the stain?
A. Hydrochloric acid pumice microabrasions B. At home bleaching method
C. Superoxol with or without heat D. Composite resin veneers
10. Night guard bleaching refers to
A. Laser-activated bleaching B. Dentist-prescribed home applied technique
C. Thermo bleaching D. Photo bleaching

ENDODONTIC SURGERY
1. Which is true about mucogingival flap designs?
A. Flaps should be wider at the base B. Flaps should be narrower at the base
C. Flap margins should not reset on the bone D. Mucogingival flaps should be avoided
2. Which surgical procedure is indicated after the endodontic treatment is completed of a mandibular molar with
periodontal and carious involvement of the bifurcation?
A. Hemisection B. Fenestration
C. Root amputation D. Apical curettage

https://t.me/DentalBooksWorld
1150 Triumph's Complete Review of Dentistry

3. Apicoectomy is indicated in which of the following cases?


A. Persistent positive results of root canal culture for microorganisms
B. Incomplete obturation in apical part of root canal with persistent infection
C. Fracture of apical one-third area of root in a vital asymptomatic tooth
D. Presence of a minor pulp stone in pulp chamber
4. Which of the following are complications of periapical surgery?
A. Paresthesia B. Ecchymoses
C. Stitch abscess D. All of the above
5. Endodontic surgery should be avoided in
A. Short-rooted tooth B. Teeth with periodontal disease
C. Lingual surface of mandibular molars D. All of above

TRAUMATOLOGY
1. Percentage of NaOCl used in regenerative endodontics is
A. 1.5% B. 2.5%
C. 5.25% D. 8%
2. Splinting time for root fracture
A. 2–4 weeks semirigid B. 2–4 months semirigid
C. 2–4 weeks rigid D. 2–4 months semirigid
3. Least favorable healing pattern in root fracture is
A. Healing with calcified tissue B. Healing with interproximal connective tissue
C. Healing with interproximal bone and connective tissue D. Interproximal inflammatory tissue without healing
4. Etrusive luxation splinting time
A. 2 weeks B. 4 weeks
C. 3 weeks D. 5 weeks
5. Source of Ca+2 in dentinal bridge is
A. Blood B. Ca(OH)2
C. Saliva D. ICF
6. Apexification is the treatment of choice for a permanent tooth with wide open apex when
A. The pulp is necrotic
B. The pulp is vital
C. The pulp and root canals are calcified
D. There is traumatic pulp exposure during cavity preparation
7. An 8-year-old child had fractured his maxillary central incisor 10 months ago. The pulp shows no response. There is
no periapical lesion in the radiograph. The treatment of choice is
A. Ca(OH)2 pulp capping B. Formocresol pulpotomy
C. Conventional root canal treatment D. Complete debridement and apexification
8. A 2-hour-old avulsed tooth is best stored in medium that contains
A. Tetracycline B. Dexamethasone
C. Ampicillin D. Ibuprofen
9. To store avulsed tooth, which kind of milk is shown to be suitable?
A. Hot milk B. Cold milk
C. Low-fat milk D. High-fat milk
10. Crown infarction is
A. Necrosis of pulp following a fracture
B. An incomplete fracture of enamel without loss of tooth structure
C. Fracture of crown in mass
D. Vertical fracture of crown
11. There is a real relationship between the incidences of fracture of anterior teeth and the
A. Caries in those teeth B. Hardness of the enamel
C. Hardness of dentin D. Protrusion of those teeth

https://t.me/DentalBooksWorld
Chapter 13 • Endodontics 1151

ANSWERS

ANATOMY OF THE PULP CAVITY AND ROOT CANAL


1. Answer: B
2. Answer: A
3. Answer: C
4. Answer: B
5. Answer: B
6. Answer: A
7. Answer: A
Note: Option B is for the following question.
What application is recommended for effective cleaning and drying of the pulp chamber before visual inspection?
Answer: 17 % EDTA and 95% Ethanol
8. Answer: B
Vertucci root canal classification
Type I: Single canal is present in the pulp chamber from crown to apex.
Type II: The pulp separates into two near the crown and joins at the apex to form one root canal.
Type III: Starts as one root canal in the pulp chamber and divided into two as it nears the apical foramen and then fuses
again to form a single root near the apical foramen.
Type IV: The root separates into two distinct canals and extends till the root apex separately.
Type V: The root canal is single is a single entity extending till the apex of the root but dividing into two separate canals
right before the apical foramen.
Type VI: Root canals start as two canals from the pulp chamber and join at the middle of the root to form one and extend
till the apex and again divide into two canals just short of the apical foramen.
Type VII: Root canal starts as a single pulp canal till the middle third of the root and then divides into two separate canals
and then rejoins after some distance and then near the apex divides into two canals again just near the apex.
Type VIII: The pulp chamber near the coronal portion divides into three separate root canals extending till the apex of the root.
9. Answer: B
10. Answer: B
Isthmus classification by Kim et al.
Type I: Incomplete isthmus/faint communication between two canals
Type II: Complete isthmus/two canals with a definite connection between them
Type III: Very short, complete isthmus between two canals
Type IV: Complete or incomplete isthmus between two or more canals
Type V: Two or three canal openings without visible connections
11. Answer: A
Cementodentinal junction is located 1 mm from the apical foramen.
12. Answer: B
Size of main apical foramina
Teeth MeanValue (μm)
Maxillary incisors 289.4
Mandibular incisors 262.5
Maxillary premolars 210 ______________________lowest
Mandibular premolars 268.25
Maxillary molars
Palatal 298
Mesiobuccal 235.05
Distobuccal 232.2
Mandibular molars
Mesial 257.5
Distal 392 ______________________highest

https://t.me/DentalBooksWorld
1152 Triumph's Complete Review of Dentistry

13. Answer: A
First law of symmetry: Except for the maxillary molars, canal orifices are equidistant from a line drawn in a mesiodistal
direction through the center of the pulp chamber floor.
Second law of symmetry: Except for the maxillary molars, canal orifices lie on a line perpendicular to a line drawn in a
mesiodistal direction across the center of the pulp chamber floor.
14. Answer: A
15. Answer: B
The “C” shape canal configuration can vary along the root depth so that the appearance of the orifices may not be good
predictors of the actual canal anatomy.
Category I (C1): The shape is an uninterrupted “C” with no separation or division.
Category II (C2): The canal shape resembles a semicolon resulting from a discontinuation of the “C” outline, but either
angle alpha or beta should be no less than 60 degrees.
Category III (C3): Two or three separate canals and both angles, alpha and beta, are less than 60 degrees.
Category IV (C4): Only one round or oval canal is in the cross-section.
Category V (C5): No canal lumen can be observed (is usually seen near the apex only).
16. Answer: B
17. Answer: C
18. Answer: C
19. Answer: B
Root canal
Cem

Dentine
Dentine
ent
um

Apical
constrictor

Apical
foramen
Apex

20. Answer: B
21. Answer: B

DISEASES OF PULP AND PERIRADICULAR TISSUES


1. Answer: B
2. Answer: C
3. Answer: A
4. Answer: A
5. Answer: C
6. Answer: A
7. Answer: C
8. Answer: C
9. Answer: C

BIOMECHANICS AND INSTRUMENTS


1. Answer: B
2. Answer: B
3. Answer: A
4. Answer: C
5. Answer: D
6. Answer: C

https://t.me/DentalBooksWorld
Chapter 13 • Endodontics 1153

7. Answer: B
8. Answer: B
9. Answer: B
10. Answer: C
11. Answer: B
12. Answer: C
13. Answer: C
Step 1 Step 2 Step 3 Step 4

≈90° 180–270° ≈90° 360–720°

DISINFECTION AND OBTURATION


1. Answer: B
2. Answer: A
3. Answer: D
4. Answer: A
5. Answer: B
6. Answer: D
7. Answer: B
8. Answer: B
9. Answer: A
10. Answer: B
11. Answer: A
12. Answer: A

BLEACHING
1. Answer: B
2. Answer: A
3. Answer: A
4. Answer: D
5. Answer: C
6. Answer: C
7. Answer: C
8. Answer: D
9. Answer: B
10. Answer: B

ENDODONTIC SURGERY
1. Answer: A
2. Answer: A
3. Answer: B
4. Answer: D
5. Answer: D

https://t.me/DentalBooksWorld
1154 Triumph's Complete Review of Dentistry

TRAUMATOLOGY
1. Answer: A
2. Answer: A
3. Answer: D
4. Answer: A
5. Answer: A
6. Answer: A
7. Answer: D
8. Answer: B
9. Answer: C
10. Answer: B
11. Answer: D

https://t.me/DentalBooksWorld
14 Prosthodontics

COMPLETE DENTURE

SYNOPSIS

INTRODUCTION
Complete denture prosthodontics or full denture prosthetics is defined as “The replacement of the natural teeth in the arch
and their associated parts by artificial substitutes.” – GPT

Components of a Complete Denture


1. Impression surface – Intaglio surface
• This surface refers to the surface of the denture that will be in contact with the tissues (basal seat area and limiting structures).
• This is the negative replica of the tissue surface of the patient.
2. Polished surface – Cameo surface
• This surface refers to the external surfaces of the lingual, buccal, labial flanges and the external palatal surface of the
denture.
• This surface should be well polished and smooth to avoid collection of food debris.
3. Occlusal surface
• That portion of the surface of a denture or dentition which makes contact or near contact with the corresponding surface
of the opposing denture or dentition.

Types of Teeth
Property Acrylic teeth Porcelain teeth
Abrasion resistance Low High
Adjustability Easy to adjust Difficult to trim
Bonding Chemical Mechanical
Staining Easily stained Does not stain
Percolation Absent Present
Clicking sound Absent Present
Ease of rebasing Difficult to remove Easy to remove
Trauma to denture base area Less High

Morphology of Teeth
1. Anatomic teeth
• These teeth have prominent pointed or rounded cusps on the masticating surfaces and which are designed to occlude
with the teeth of the opposing denture.
• Anatomic teeth have a 33-degree cuspal angulation.
https://t.me/DentalBooksWorld
1156 Triumph's Complete Review of Dentistry

• Balanced occlusion with anatomic teeth can be achieved in eccentric jaw positions (protrusive, right lateral, and left
lateral movements).
• They are more efficient in cutting and grinding food, so less masticatory effort and forces are needed.
2. Semi anatomic teeth
• These teeth have cusp angles ranging between 0 and 30 degrees.
• The cusp angles are usually around 20 degrees. These are also called modified anatomic teeth.
• Victor Sears in 1922 designed the first anatomic teeth, which was called the channel tooth.
• In 1930, Avery Brothers modified the channel tooth to produce what was called the Scissor bite teeth.
3. Nonanatomic or cuspless teeth
• These are the teeth designed without cuspal prominences on the occlusal surface.
• These teeth have 0-degree cuspal angulation.
• These teeth do not provide balance occlusion.
• Hall in 1929, designed the first cuspless tooth and named it “inverted cusp tooth.”
• In 1929, Myerson introduced the “trukusp” teeth.
• In 1934, Nelson described the “Chopping block.”
• In 1939, Swenson designed the “Nonlock tooth.” The occlusal surface was flat with sluiceways or pathways for food clearance.
• Cuspless teeth are used in patients with bruxism, as these teeth decrease the forces acting on basal tissues.
• Greater range of movements is possible.
• In patients with neuromuscular disorders where accurate jaw relation cannot be recorded, cuspless teeth are preferred.
• In cases with highly resorbed ridge, cuspless teeth are preferred as they do not get locked and displace the denture during
lateral movements.
4. Crossbite teeth
• These teeth are used in jaw discrepancy cases leading to posterior crossbite relationship.
• Here the buccal cusps of maxillary teeth are absent.
• Instead there is a large palatal cusp, which rests on the lower tooth.
• This crossbite teeth was designed by Gysi in 1927.
5. Metal insert teeth (VO posteriors)
• Hardy designed the first metal insert tooth and he called it the “Vitallium Occlusal.”
• Here each tooth will look like the fusion of two premolars and one molar.
• The teeth has greater cutting efficiency.

Masticatory Forces Generated


Type of dentition Amount of forces generated
In newton In kg In pounds
Natural dentition 600–800 N 60–80 kg 120–180 pounds
Complete denture teeth 75–100 N 7–10 kg 16–22 pounds
Removable partial denture 100–120 N 10–12 kg 22–25 pounds
Fixed partial denture 250–300 N 25–30 kg 50–65 pounds
Implant-supported CD 200–400 N 4–40 kg 10–100 pounds

Contacts During Swallowing


Actual chewing per meal 450–600 seconds
Four meals per day 1,800–2,400 seconds (30–40 minutes)
Duration of each masticatory cycle 0.8–1 second
Total number of swallows in a day 600 times
Total number of swallows when a patient is awake 800 times (Ref. Profitt)

https://t.me/DentalBooksWorld
Chapter 14 • Prosthodontics 1157

Total period of occlusal contact during chewing per day 500–600 (avg.) or 10 minutes
(entire day)
Duration of each swallow 1 second
Total time of occlusal contact 18–20 minutes
In bruxism patients, occlusal contact may exceed 30–180 min/day

DIAGNOSIS AND TREATMENT PLANNING


Mental attitude of the patients
De Van’s Dictum: “Meet the mind of the patient before meeting the mouth of the patient.”

Based on the mental attitude of the patients, Dr. MM House classified patients into the following classes.
Class I: Philosophical
• Generally these type of patients are easy going, congenial, mentally well adjusted, co-operative, and confident on dentists.
• These patients have excellent prognosis.
• Those who have presented themselves prior to the extraction of their teeth, have had no experience in wearing dentures, and
do not anticipate any special difficulties in that regard.
Class II: Exacting
• These patients are precise, above average in intelligence, concerned in their dress and appearance, usually dissatisfied by
their previous treatment, do not have confidence in the dentist.
• It is very difficult to satisfy them. But once satisfied, they become the dentist’s greatest supporter.
Class III: Hysterical
• These patients do not want to have any treatment done. They come out of compulsion from their relatives and friends.
• They have a highly negative attitude to the dentist and their treatment.
• They have unrealistic expectations and want the dentures to be better than their natural teeth.
• They are the most difficult patients to manage.
• They show poor prognosis.
Class IV: Indifferent
• Those who are unconcerned about their appearance and feel very little or no necessity for teeth for mastication.
• They are therefore uncooperative and will hardly try to become accustomed to dentures.
• They will not maintain the dentures properly and do not appreciate the skills of the dentist.

Clinical Examination of the Patient


Extraoral examination
Features Classified by
Facial form House and Loop, Frush and Fisher
Facial profile Angle
Muscle tone, development House
Condition of the mucosa House
Arch form House
Ridge relationship Angle
Palatal throat forms House
Lateral throat forms Neil
Tongue size House
Tongue positions Wright
Bone quality radiographically Branemark et al.

https://t.me/DentalBooksWorld
1158 Triumph's Complete Review of Dentistry

Facial Profile (Angle)


Class I: Normal or straight profile
Class II: Retrognathic profile
Class III: Prognathic profile

Muscle Tone (House)


Class I: Normal tension, tone, and placement of the muscle
Class II: Normal muscle function but slightly decreased muscle tone
Class III: Decreased muscle tone and function

Lip Mobility
Class I: Normal mobility
Class II: Reduced mobility
Class III: Paralyzed

Intraoral Examination
Condition of the Mucosa
Class I: Healthy mucosa
Class II: Irritated mucosa
Class III: Pathologic mucosa
Arch Form (House)
Class I: Square
Class II: Tapering
Class III: Ovoid
Atwood’s Classification of Bone Resorption
Class I: Tooth bearing alveolus
Class II: Alveolus after extraction
Class III: High alveolar process
Class IV: Knife edge process
Seibert’s Classification of Residual Ridges
Class I: Loss of faciolingual width
Class II: Loss of apico-coronal height
Class III: Loss of both height and width
Class IV: Normal ridge
Ridge Parallelism
Class I: Both ridges are parallel to the occlusal plane
Class II: The mandibular ridge diverts from the occlusal plane anteriorly
Class III: Either the maxillary ridge diverts from the occlusal plane anteriorly or both ridges divert from the occlusal plane
anteriorly

Classification of Soft Palate


Class Type of soft palate Posterior palatal seal
Class I Soft palate is horizontal and extends posteriorly with minimal Broad
muscular activity (10 degrees)
Class II Palatal contour is between class I and class III (45 degrees) Medium
Class III Most acute contour usually seen with a high V-shaped palatal Very narrow
vault (70 degrees)
https://t.me/DentalBooksWorld
Chapter 14 • Prosthodontics 1159

Palatal Throat Forms (House):


• Class I: Large and normal in form, relatively with an immovable band of tissue 5–12 mm distal to a line drawn across the
distal edge of the tuberosities.
• Class II: Medium sized and normal in form, with a relatively immovable resilient band of tissues, 3–5 mm distal to a line
drawn across the distal edges of the tuberosities.
• Class III: Usually accompanies a small maxilla. The curtain of soft tissue turns down abruptly 3–5 mm anterior to a line
drawn across the palate at the distal edge of the tuberosities.

Lateral Throat Form (Neil):


• Class I: Deep lateral throat form
• Class II: Moderate lateral throat form
• Class III: Shallow lateral throat form.

Palatal Sensitivity (House):


• Class I: Normal
• Class II: Subnormal (Hyposensitive)
• Class III: Supernormal (Hypersensitive)

Classification of Tongue Size (House):


• Class I: Normal in size, development, and function. Sufficient teeth are present to maintain this normal form and
function.
• Class II: Teeth have been absent long enough to permit a change in the form and function of tongue.
• Class III: Excessively large tongue. All teeth have been absent for an extended period of time, allowing for abnormal
development of the size of the tongue.

Classification of Tongue Positions (Wright):


• Class I: The tongue lies in the floor of the mouth with the tip forward and slightly below the incisal edges of the mandibular
anterior teeth.
• Class II: The tongue is flattened and broadened but the tip is in a normal position.
• Class III: The tongue is retracted and depressed into the floor of the mouth, with the tip curled upward, downward, or
assimilated into the body of the tongue.

ANATOMICAL LANDMARKS FOR CD


Histology of Mucous Membrane
• The oral cavity has two layers namely, mucosa and submucosa.
• The mucosa has a keratinized, stratified squamous epithelium.
• The mucosa covering the hard palate and the crest of the residual ridge including the residual attached gingiva is called the
masticatory mucosa.
• The submucosa varies in thickness and consistency and it is responsible for supporting the denture.
• When the submucosa is thin, it gets easily traumatized. When it is loosely attached, inflamed, or edematous, it gets easily
displaced.

Anatomical Landmarks in the Maxilla


Limiting Structures
• Labial frenum
• Labial vestibule

https://t.me/DentalBooksWorld
1160 Triumph's Complete Review of Dentistry

• Buccal frenum
• Buccal vestibule
• Hamular notch
• Posterior palatal seal area
Supporting Structures
Primary stress bearing area:
• Hard palate
• The posterolateral slopes residual alveolar ridge
• Maxillary tuberosity (Boucher 13th edition)
Secondary stress bearing area:
• Rugae
• Alveolar tubercle
Relief areas:
• Incisive papilla
• Cuspid eminence
• Mid-palatine raphe
• Fovea palatina

Posterior Palatal Seal Area (Post Dam)


Definition: The soft tissues at or along the junction of the hard and soft palates on which pressure within the physiological
limits of the tissues can be applied by a denture to aid in the retention of the denture.
• It is the area of the soft palate that contacts the posterior surfaces of the denture base.
• It prevents air entry between the denture base and soft palate.
• It is the area between the anterior and posterior vibrating lines.
• It is the primary retentive area.
Functions of PPS
• Aids in retention by maintaining constant contact with the soft palate during functional movements.
• Reduces the tendency for gag reflex as it prevents the formation of the gap between the denture base and the soft palate.
• Prevents food accumulation between the posterior border of the denture and soft palate.
• Compensates for polymerization shrinkage.

Vibrating Line
Definition: The imaginary line across the posterior part of the palate marking the division between the movable and immovable
tissues of the soft palate which can be identified when the movable tissues are moving.
• It is an imaginary line drawn across the palate that marks the beginning of motion in the soft palate, when the individual
says, “ah.”
• It passes about 2 mm in front of the fovea palatina. Fovea palatina acts as a guide to locate the posterior border of the
denture.
• This line should lie on the soft palate.
• The two vibrating lines are:
–– Anterior vibrating line
–– Posterior vibrating line
Anterior Vibrating Line
• It is an imaginary line lying at the junction between the immovable tissues over the hard palate and the slightly movable
tissues of the soft palate.
• It can be located by asking the patient to perform the “Valsalva” maneuver.
• Valsalva maneuver: The patient is asked to close his nostrils firmly and gently blow through his nose.
• The anterior vibrating line is a cupid’s-bow shaped.

https://t.me/DentalBooksWorld
Chapter 14 • Prosthodontics 1161

Posterior Vibrating Line


• It is an imaginary line located at the junction of the soft palate that shows limited movement and the soft plate that shows
marked movement.
• It also represents the junction between the aponeurosis of the tensor veli palatini muscle and the muscular portion of the
soft palate.
• It is recorded by asking the patient to say “ah” in short but normal nonvigorous fashion.
• This is usually a straight line.

Relief Areas
Incisive Papilla
• It is a midline structure situated behind the central incisors.
• It is the exit point of the nasopalatine nerves and vessels.
• It should be relieved; if not, the denture will compress the vessels or nerves and lead to necrosis of the disturbing areas and
paresthesia of anterior palate.
Fovea Palatine
• It is formed by coalescence of the ducts of several mucous glands.
• This acts as an arbitrary guide to locate the posterior border of the denture.
• The denture can extend 1–2 mm beyond the fovea palatine. The secretion of the fovea spreads as a thin film on the denture
thereby aiding in retention.
• In patients with thick, ropy saliva the fovea palatina should be left uncovered or else the thick saliva flowing between the
tissue and the denture can increase the hydrostatic pressure and displace the denture.
Cuspid Eminence
• It is a bony elevation on the residual alveolar ridges formed after extraction of the canine.
• It is located between the canine and the first premolar.

Anatomical Landmarks in the Mandible


Limiting Structures
• Labial frenum
• Labial vestibule
• Buccal frenum
• Buccal vestibule
• Lingual frenum
• Alveololingual sulcus
• Retromolar pads
• Pterygomandibular raphe
Supporting Structures
• Buccal shelf area
• Residual alveolar ridge
Relief Area
• Crest of the residual alveolar ridge
• Mental foramen
• Genial tubercles
• Torus mandibularis

Alveololingual Sulcus
• It extends from the lingual frenum to the retromylohyoid curtain. It is considered in three regions.
1. Anterior region:
• It extends from the lingual frenum to the premylohyoid fossa, where the mylohyoid curves below the sulcus.

https://t.me/DentalBooksWorld
1162 Triumph's Complete Review of Dentistry

2. Middle region:
• It extends from the premylohyoid fossa to the distal end of the mylohyoid ridge.
• This region is shallower than other parts due to the prominence of the mylohyoid ridge and action of the mylohyoid
muscle.
• The lingual flange should slope medially toward the tongue. This sloping helps in three ways:
–– The tongue rests over the flange stabilizing the denture.
–– Provides space for raising the floor of the mouth without displacing the denture.
–– The peripheral seal is maintained during the function.
3. Posterior region:
• The retromylohyoid fossa is present in this region.
• The denture flange should turn laterally in this region toward the ramus of the mandible to fill up the fossa and complete
the typical S-form of the lingual flange.
• This is called lateral throat form.

Retromylohyoid Fossa
• It lies posterior to the mylohyoid muscle. This fossa is bounded:
–– Anteriorly by the retromylohyoid curtain.
–– Posterolaterally by the superior constrictor of the pharynx.
–– Posteromedially by the palatoglossus and lateral surface of the tongue.
–– Inferiorly by the submandibular gland.

Retromolar Pad
• The retromolar pad is an important structure, which forms the posterior seal of the mandibular denture.
• It is a nonkeratinized pad of tissue seen on a posterior continuation of the pear-shaped pad.
• The pear-shaped pad is a triangular keratinized soft pad of tissue at the distal end of the ridge.
• It is bounded posteriorly by the tendons of the temporalis, laterally by the buccinator and medially by the pterygomandibular
raphe and superior constrictor.
• The denture should extend only one-half to two-thirds over the retromolar pad.
• The retromolar pad has a stippled and keratinized mucosa.

Retromolar Papilla
• It is described as a pear-shaped papilla.
• Craddock coined the term retromolar papilla.
• It is nothing but the residual scar formed after the extraction of third molar.
• The denture should terminate at the distal end of the pear-shaped papilla.
• Beading this area improves retention.

Pterygomandibular Raphe
• Pterygomandibular raphe arises from the hamular process of the medial pterygoid plate and gets attached to the mylohyoid
ridge.
• A raphe is a tendinous insertion of two muscles. In this case, the superior constrictor is inserted posteromedially and the
buccinators inserted anterolaterally.

Buccal-Shelf Area
• It is the area between the buccal frenum and anterior border of the masseter. Its boundaries are:
–– Medially the crest of the ridge
–– Distally the retromolar pad
–– Laterally the external oblique ridge

https://t.me/DentalBooksWorld
Chapter 14 • Prosthodontics 1163

• The width of the buccal shelf area increases as alveolar resorption continues.
• It has a thick submucosa overlying a cortical plate.
• As it lies at right angles to the occlusal forces, it serves as a primary stress-bearing area.

Genial Tubercles
• These are a pair of bony tubercles found anteriorly on the lingual side of the body of the mandible.
• The superior one gives attachment to the genioglossus muscle and the inferior tubercle gives attachment to the geniohyoid
muscle.

Mandible
1. Primary stress bearing area Buccal shelf area
2. Secondary stress bearing area Slopes of edentulous ridge
3. Primary retentive and primary peripheral seal area Retromolar pad
4. Secondary peripheral seal area Anterior lingual border
5. Relief areas Crest of the residual ridge
Mental foramen
Mylohyoid ridge

Maxilla
1. Primary stress-bearing area Residual alveolar ridge, maxillary tuberosity (Boucher
13th edition)
2. Secondary stress-bearing area Rugae or anterior hard palate
3. Tertiary stress-bearing area and secondary retentive area Posterolateral part of hard palate
4. Relieving areas Incisive papilla
Mid-palatine raphae
Cuspid eminence
Fovea palatine
5. Primary retentive area Posterior palatal seal area

Influence of Muscles on Complete Denture


Muscle having dual function in relation to complete denture Masseter
Buccal frenum of maxilla contains Caninus (Levator anguli oris), Buccinator (Pulls frenum
backward), Orbicularis oris (Pulls frenum forward)
Buccal frenum of mandible contains Triangularis (depressor anguli oris)
Buccinator
Labial frenum of mandible contains Incisive
Distobuccal flange of the mandibular denture is limited by Masseter
Distobuccal flange of the maxillary denture is limited by Coronoid process, ramus, masseter
Retromolar pad contain the fibers of Temporalis
Buccinator
Superior constrictor
Pterygomandibular raphae
The lingual flange of lower denture is limited
• In the anterior region by Genioglossus
• In the middle region by Mylohyoid
• In the posterior region by Palatoglossus and superior constrictor
Masseteric notch is formed due to Action of masseter on buccinator

https://t.me/DentalBooksWorld
1164 Triumph's Complete Review of Dentistry

ARTICULATORS
A mechanical device which represents the TMJ and the jaw members to which maxillary and mandibular casts may be
attached to stimulate jaw movements.

Classification of Articulators
1. Articulators Based on Theories of Occlusion
a. Bonwill theory articulators
• This articulator was designed by W. G. A. Bonwill.
• Bonwill theory is also known as the theory of equilateral triangle, according to which the distance between the condyles
is equal to the distance between the condyle and the midpoint of the mandibular incisors.
• An equilateral triangle is formed between the two condyles and the incisal point.
• Theoretically, the dimension of the equilateral triangle is 4 inches.
• Bonwill articulators allow lateral movement and permit the movement of the mechanism only in horizontal plane.
b. Conical theory articulators
• Proposed by RE Hall.
• This theory states that, the lower teeth move over the surfaces of the upper teeth as over the surface of a cone, generating
an angle of 45-degrees with the central axis of the cone tipped 45* to the occlusal plane.
c. Spherical theory articulators
• The spherical theory of occlusion proposed that lower teeth move over the surface of upper teeth as over a surface of
sphere with a diameter of 8 inches.
• The center of sphere was located in the region of glabella.
• The articulator devised by G. S. Monson operated on the spherical theory of occlusion.

2. Based on the Ability to Stimulate Jaw Movements


Class I
• Simple holding instruments capable of accepting a single static registration
• E.g., Slab articulators, Hinge Joints, Barndor, Gysi simplex
Class II
• Instruments that permit horizontal as well as vertical motion but do not orient the motion of the TMJ via facebow transfer
II A Eccentric motion permitted is based on average or arbitrary values
E.g., Mean value articulator
II B Limited eccentric motion is possible based on theories of arbitrary motion
E.g., Monson and Hall’s articulators
II C Limited eccentric motion is possible based on engraving records obtained from the patient
E.g., House’s articulator
Class III
• Permit horizontal, vertical positions, and also accept facebow transfer
III A Accept a static protrusive registration and they use equivalents for other types of motion
E.g., Hanau H, Dentatus
III B They accept static lateral registration and they use equivalents for other types of motion
E.g., Ney, Teledyne Hanau University series, Trubyte, Panadent
Class IV
• They accept three-dimensional dynamic registrations
IV A The condylar path registered cannot by modified
E.g., TMJ articulator, stereograph
IV B They allow customization of the condylar path
E.g., Stuart instrument gnathoscope, Pantograph, Denar

https://t.me/DentalBooksWorld
Chapter 14 • Prosthodontics 1165

3. Based on Adjustability of the Articulator


a. Nonadjustable articulator
• They can open and close in a fixed horizontal axis.
• The condylar path is fixed.
b. Semiadjustable articulator
• Have adjustable horizontal condylar paths, adjustable lateral condylar paths, and adjustable intercondylar distance
• E.g., Arcon and Non-Arcon type articulators
Arcon type articulator
• The term Arcon was given by Bergstrom.
• It is the combination of words Articulator and Condyle.
• In this, the condylar guide is located in the upper member.
• The condylar analogue or sphere is located in the lower membrane.
• The upper member is rigid and lower member is movable just as in the patient.
• E.g., Whipmax, Teledyne (Class III), Ney (Class III)
Non-Arcon type articulator
• The condylar analogue or sphere is located in the upper membrane and condylar guide is located in the lower member.
• The upper member is movable and lower member is rigid unlike in the patient.
• E.g., Hanau articulators, Dentatus, Gysi
c. Fully adjustable articulator
• Capable of being adjusted to follow the mandibular movement in all the directions.
• E.g., Stuart instrument gnathoscope
Arcon
• The fossae are on the upper member, the condyles on the lower.
• The condyles are not rigidly held in the fossae and, therefore, can move away from the fossae if required by occlusal contacts.
• The lower cast has been mounted to the upper cast via jaw registration.
Nonarcon
• The condyles are represented by slots on the upper member, the fossae by balls on the lower member.
• The ball is fixed into the slot and therefore cannot move away, and may not able to reproduce the CRCP/IP movement
which is tooth determined.

TEETH SELECTION
Anterior Teeth Selection
Size of an Anterior Teeth
• Anthropological measurements measure certain anatomical dimensions and derive the size of the teeth using certain
formula
1. Width of the upper central incisor: Circumference of the Head/13
2. Total width of the upper anteriors: Bizygomatic width/3.36
3. Total width of lower anterior = 3/4th width of upper anterior
• Berry’s biometric index: This index is used to derive the width of the central incisor using bizygomatic width or the length
of the face
1. The width of the maxillary central incisor = Bizygomatic width/16
2. The width of the maxillary central incisor = Length of the face/20

https://t.me/DentalBooksWorld
1166 Triumph's Complete Review of Dentistry

• Winkler’s concept: According to Winkler, the teeth should be selected based on three different views namely, physiological,
psychological, and biomechanical.
• Typal form theory: Leon Williams (1917): According to Williams, the shape of the teeth should be inverse of the shape of
the face. Leon Williams classification of facial forms include:
–– Square
–– Tapering
–– Ovoid
–– Combination
Dentogenic Concept and Dynesthetics (Sex, Personality, Age, or SPA Factor)
• This was first described by Frush and Fischer.
• According to them, the sex, personality, and the age of the patient determine the form of the anterior teeth.

Color of the Anterior Teeth


1. Hue
• It denotes the specific color produced by a specific wavelength of light.
• It is in harmony with patient skin color.
2. Chroma or saturation
• It is the amount of color per unit area of an object.
• It denotes the intensity of the color.
• Objects with highly saturated color lacks depth.
3. Brilliance or value
• It denotes the lightness or darkness of an object.
• It is actually the dilution of the color with either black or white to produce darker or lighter shades, respectively.
4. Translucency
• It is the property of the object to partially allow passage of light through it.
• Enamel has high brilliance and translucency.
The following factors are considered while selecting the size of the posterior teeth:
• Buccolingual width
• Mesiodistal length
• Occlusogingival height

INTRODUCTION AND CLASSIFICATIONS OF IMPRESSIONS


Definition: A complete denture impression is a negative registration of the entire denture bearing, stabilizing and border seal
areas present in the dentulous mouth.

Classification of Impressions
Mucostatic or Passive Impression
• It was first proposed by Richardson and later popularized by Henry Page.
• In this method, the impression is made with the oral mucous membrane and the jaws in a normal relaxed condition.
• Impression material of choice is Impression Plaster.
• These dentures will have good stability but poor retention.
Mucocompressive Impression (Carole Jones)
• The mucocompressive technique records the oral tissues in a functional and displaced form.
• The materials used for this technique include impression compound, waxes, and soft-liners.
• Dentures made by this technique tend to get displaced due to the tissue rebound at rest.

https://t.me/DentalBooksWorld
Chapter 14 • Prosthodontics 1167

Selective Pressure Impression (Boucher):


• In this technique, the impression is made to extend over as much denture-bearing area as possible without interfering with
the limiting structures at function and rest.
• In this technique, the nonstress-bearing areas are relieved and the stress-bearing areas are allowed to come in contact with
the tray.
Hand Manipulated Functional Movements (Dynamic Impression)
• It is a mucofunctional technique, which involves recording functional movements of the mucosa while making the
impression.
• Border molding or peripheral tracing or muscle trimming is a mucofunctional technique used in CD fabrication.

Diagnostic Impression
• It is made to prepare diagnostic cast, which is used for the following purposes:
–– To survey the undercuts
–– To estimate the amount of preprosthetic surgery required and to perform mock surgeries
–– Articulate the casts in a tentative jaw relation and evaluate the interarch space
–– To determine the ability to establish occlusal balance

Primary Impression
Definition: An impression made for the purpose of diagnosis or for the construction of a tray.
• This is the first step in fabrication of a CD.
• The preliminary impression is made with a Stock tray.
• There should be at least 5 mm clearance between the stock tray and the ridge.
• The maxillary tray should extend over the tuberosity and the hamular notch.
• If the tray is deficient, utility wax can be added along the posterior border of the tray.
• The preliminary impression can be made using impression compound, alginate, or impression plaster.
Secondary Impression or Wash Impression
• This clinical procedure is done to prepare a master cast. This is done after mouth preparation is complete.
• This method makes use of a custom tray or special tray prepared from the primary cast.
• The borders of the tray should end 2 mm short of the peripheral structures.
• The tray can be made of autopolymerizing resin or reinforced shellac base plate.
• The impression material chosen for the secondary impression should be of low viscosity to record the structures
accurately.
• The materials of choice are, zinc oxide eugenol impression paste and medium bodied elastomeric impression materials.

Reversible Hydrocolloid Impression


• This impression makes use of agar (a reversible hydrocolloid).
• The agar is taken from the tempering section, which is at 46°C and loaded onto a water-cooled rim-lock tray.
• It has excellent surface detail reproduction of up to 25 microns.
• Agar has poor dimensional stability due to syneresis and imbibition.
• Generally elastic impression materials are indicated for recording undercuts.
• Agar is an elastic material and can be used to record undercuts.
Irreversible Hydrocolloid Impression
• Alginate is the hydrocolloid used for this type of impression.
• These alginate impressions have a better peripheral seal than other impressions. These do not cause cross-infections as they
are used only once.
• Their main disadvantage is poor dimensional stability due to syneresis and imbibition.
• All hydrocolloid impressions should be poured immediately after they are made.

https://t.me/DentalBooksWorld
1168 Triumph's Complete Review of Dentistry

Modeling Plastic Impression


• Impression compound is a reversible thermoplastic material, which is used for making preliminary impressions.
• It is softened in a water bath at 64°C. This impression is made using a stock tray.
• It has good dimensional stability but excessive water incorporation during wet kneading can cause dimensional change.
• Its disadvantage is its viscosity. As it is highly viscous, it can displace the tissue surface.

Plaster Impression
• Type I plaster (soluble plaster) is used for taking impression.
• This material has potato starch which helps in easy separation of the cast from the impression.

Silicone Impression
1. Condensation polymerizing silicone
• It is available as a reactor paste and an activator paste, which are mixed together in a recommended ratio to produce a
uniform mix.
• The disadvantage is their dimensional instability due to the formation and evaporation of ethyl alcohol from the
impression.
2. Addition polymerizing silicone
• This variety of silicone does not undergo dimensional change.
• The cast can be poured even after a week.
• The material is available as two varieties, namely tubes and cartridges.
3. Thiokol rubber impression
• These are polysulfide impression materials. They are available as base and accelerator pastes.
• Polysulfide materials are hydrophobic. Precautions should be taken to avoid any moisture contamination on the tissue surface.
• Silicone and Thiokol impression materials are used to take secondary impression.

Objectives of Impression Making


• Retention
• Stability
• Support
• Aesthetics
• Preservation of remaining structures

Retention
That quality inherent in the prosthesis which resists the force of gravity, adhesiveness of foods and the forces associated with the
opening of jaws.
• Retention is the ability of the denture to withstand displacement against its path of insertion. The factors that affect retention
can be classified as follows.
1. Anatomical factors
• Retention increases with increase in size of the denture-bearing area.
• The size of the maxillary denture-bearing area is about 24 cm2 and that of mandible is about 14 cm2.
• Hence maxillary dentures have more retention than mandibular dentures.
2. Physiological factors
• The viscosity of saliva determines retention. Thick and ropy saliva gets accumulated between the tissue surface of denture
and the palate leading to loss of retention.
• Thin and watery saliva can also lead to compromised retention.

https://t.me/DentalBooksWorld
Chapter 14 • Prosthodontics 1169

3. Physical factors
Adhesion
• It is the “physical attraction of unlike molecules to one another.”
• The role of saliva is very important for adhesion.
• A thin film of saliva is formed between the denture and the tissue surface. This thin film helps to hold the denture to the
mucosa.
• The amount of adhesion of present is proportional to the denture base area.
Cohesion
• It is the “physical attraction of like molecules for each other.”
• The cohesive force acts within the thin film of saliva. Watery serous saliva can form a thinner film and is more cohesive
than thick mucus saliva.
Interfacial surface tension
• It is “the tension or resistance to separation possessed by the film of liquid between two well-adapted surfaces.”
• It plays a major role in the retention of a maxillary denture. It is totally dependent on the presence of air at the margins of
liquid and solid contact (liquid air interface).
• In mandibular dentures, where there is excess saliva, there is no surface tension and there is no liquid–air interface (minimal
interfacial surface tension).
• Stefan’s law is associated with interfacial surface tension.
Capillarity or capillary attraction
• It is “that quality or state, because of surface tension causes elevation or depression of the surface of a liquid that is in contact
with a solid.”
• Factors that aid in improvement of capillary attraction are
–– Closeness of adaptation of denture base to soft tissue
–– Greater surface of the denture-bearing area
–– Thin film of saliva should be present
Atmospheric pressure and peripheral seal
• The peripheral seal prevents air entry between the denture surface and the soft tissue. Hence, a low pressure is maintained
within the space between the denture and the soft tissues.
• When displacing forces act on the denture, a partial vacuum is produced between the denture and the soft tissues, which
aids in retention. This property is called the natural suction of a denture.
• Hence atmospheric pressure is referred to as emergency-retentive force or temporary restraining force.
• Retention produced by an atmospheric pressure is directly proportional to the denture base area.
Mechanical factors aiding in retention
• Undercuts
–– Unilateral undercuts aid in retention while bilateral undercuts will interfere with denture insertion and require surgical
correction.
• Retentive springs
• Magnetic forces
–– Intramucosal magnets (Cobalt-platinum magnets) aid in increasing retention of highly resorbed ridges.
• Denture adhesives
–– It helps in initial retention of the denture increasing the psychological comfort of the patient.
–– It is available as soluble and insoluble wafers. According to the ADA, a denture adhesive should have the following
characters:
■■ Product composition should be supplied

■■ Should not affect the integrity of the denture

■■ Biologically acceptable

■■ Effective function as adherent

https://t.me/DentalBooksWorld
1170 Triumph's Complete Review of Dentistry

Composition of Denture Adhesives


• Coloring agents: Red dye
• Flavoring agents: Menthol, peppermint, etc.
• Wetting agents
• Preservatives: Sodium borate, methyl paraffin, polyparaffin
• Plasticizers: They are added to improve the handling properties of the material. E.g., Polyethylene, mineral oil, or
petrolatum.
• Dispersion agents: They are used to prevent powders from clumping.
E.g., Magnesium oxide
Sodium phosphate
Calcium stearate
Calcium silicate
Silicon dioxide
Availability
• The amount of material required for efficient retention of a denture is around 0.5–1.5 g.
Mode of Action
• Effectiveness of a denture adhesive depends on both physical and chemical factors of the material.
• The adhesive tends to swell from 50 to 150% by volume in the presence of water.
• Carbonyl groups in the adhesive material provide strong bioadhesive and biocohesive forces, which improve the retention
of the denture. E.g., Poly methyl vinyl ether maleic-anhydride or PVM/MA has a high level of these carbonyl groups.
• PVM/MA and zinc and calcium salts with CMC have superior retention because of the stronger covalent bond that develops
due to its divalent interaction.
• Suction chambers and suction discs.
–– In the past suction chambers in the maxillary dentures were used to aid in retention.
–– The suction chamber creates an area of negative pressure, which increases retention.
–– They are avoided due to the risk of papillary hyperplasia.

Stability
Definition: Stability is the quality of a denture to be firm, steady, or constant; to resist displacement by functional stresses; and
not to be subject to change of position when forces are applied.
• Stability is the ability of the denture to withstand horizontal forces. The various factors affecting stability are
–– Vertical height of the residual ridge: Highly resorbed ridge offers least stability.
–– Quality of soft tissue covering the ridge: Flabby tissues with excessive submucosa offer poor stability.
–– Occlusal plane: The occlusal plane should be parallel to the ridge.
–– Teeth arrangement: The position of the teeth and their occlusion play an important role in the stability of the denture.
Balanced occlusion facilitates the even distribution of forces across the denture.
The teeth in the denture should be arranged in the neutral zone. The neutral zone is defined as, “The potential space between
the lips and cheeks on one side and the tongue on the other side. Natural or artificial teeth in this zone are subject to equal and
opposite forces from the surrounding musculature.”

Support
“The resistance to vertical forces of mastication, occlusal forces, and other forces applied in a direction toward the denture-
bearing area.”
• In order to provide good support, the denture base should cover as much denture-bearing area as possible. This helps to
distribute forces over a wide area.
• This ability of the denture to distribute forces over wide areas due to an increase in the denture-base area is termed the
“Snow-shoe” effect.

https://t.me/DentalBooksWorld
Chapter 14 • Prosthodontics 1171

Esthetics
• Esthetics is one of the prime concerns of the patient in the complete denture treatment.
• The thickness of the denture flanges is one of the important factors that govern esthetics.
• Thicker denture flanges are preferred in long-term edentulous patients to give the required mouth fullness.

Preservation of Remaining Structures


Muller De Van (1952) stated that, “The preservation of that which remains is of utmost importance and not the meticulous
replacement of that which has been lost.”

Recording of Primary Impression


Definition: “An impression made for the purpose of diagnosis or for the construction of a tray.”
• Trays used for primary impression making are called stock trays.
• There should be at least 2–3 mm clearance between the stock tray and the ridge.
• It should have 5–6 mm clearance for impression compound.
• For making alginate impressions, perforated trays are used.
• A sufficient amount of impression compound is softened in a water bath at 140°F.
• The tray is centered over the upper residual ridge by using the labial frenum as a centering guide.

Fabrication of a Special Tray


• Green stick compound (type 1 impression compound) is used to record the sulcus in function.
• A special tray with 2 mm relief at the border is used to load the green stick compound.
• The major function of a special tray is to provide support for the green stick compound and to provide even thickness of
the impression material.
Adapting the relief wax
• In the maxillary cast, the relief wax should be adapted over the incisive papilla and the mid-palatine raphe.
• In the mandibular cast, the relief wax should be adapted over the crest of the alveolar ridge.
Adapting the spacer
• Medium bodied elastomers which require a minimum bulk of 2 mm to elicit their properties require a spacer.
• The spacer should be about 2 mm thick. Spacers should be cut out in 2–4 places so that the special tray touches the ridge
in these areas.
• The part of the special tray that extends into the cut out of the spacer is called stopper.
• Usually four stoppers are placed: two on the canine eminence on either side and two on the posterior parts of the ridge.
• The stopper can be a 2 mm square, a 2 × 4 mm rectangle, or 2 mm mesiodistally, palatally, over the crest of the ridge, and
buccally half way into the sulcus.
Application of separating medium
• The separating medium is applied to avoid the special tray from binding to the cast.
• A surface tension reducing agent can be applied over the spacer to increase the wettability of the separating medium.
• Commonly used separating media are Cold-mold seal, Tin foil, Starch, Vaseline, Cellulose acetate, etc.
Cold-mold seal: It is basically an aqueous solution of sodium alginate.
Composition:
1. Sodium alginate (2% in water)
2. Glycerine
3. Alcohol
4. Sodium phosphate
5. Preservatives
• The separating medium should be applied with a brush using single-sided strokes.

https://t.me/DentalBooksWorld
1172 Triumph's Complete Review of Dentistry

Fabrication of special tray


• Most commonly used materials for the fabrication of special tray are
–– Shellac
–– Cold cure acrylic
–– Vacuum formed vinyl or polystyrene
–– Vacuum formed thermoplastic resin
–– Type II impression compound (tray compound)
• Shellac trays should be fabricated 6 hours prior to impression procedure.

Secondary Impression Procedures


• The materials of choice for a secondary impression are zinc oxide eugenol impression paste or medium bodied elastomeric
impression materials.
• Recording the posterior palatal seal is very important for the retention of the denture.
• The various methods used for marking postdam are as follows.
1. Conventional approach
• This procedure is done after the wash impression is made and the master cast is poured.
• The “T” burnisher is used to locate the hamular notch by palpating posteriorly to the maxillary tuberosity on both sides.
• The full extent of the hamular notch is marked with an indelible pencil.
• The area between the anterior and posterior vibrating line is scrapped in the master cast to a depth of 1–1.5 mm on either
side of the mid-palatine raphe.
• In the region of mid palatine raphe, it should be only 0.5–1 mm in depth.
• The entire border of the post-dam resembles the shape of a Cupid’s bow.
2. Fluid wax technique
• This technique is done immediately after making the wash impression and before pouring the master cast.
• Zinc oxide eugenol and impression plaster are the suitable materials for this technique as fluid wax adheres well to them.
• The wash impression is painted with fluid wax. Commonly used waxes are
Iowa wax White color Dr. Smith
Korecta wax no. 4 Orange color Dr. O. C Applegate
Adaptol wax Green color Nathan. G. Kaye
H-L physiological paste Yellow-white C. S. Harkins
• The patient’s head should be positioned such that the Frankfort’s horizontal plane is 30 degrees below the horizontal plane.
• The wax in the anterior vibrating line should have a knife-edge margin. Blunt margins indicate improper flow and the
impression should be repeated.
• The border molding procedure for the mandibular impression is Fournet and Tuller’s technique.

Preparation of the Master Cast and Special Tray


• Beading is done to preserve the width and height of the sulcus in a cast.
• Boxing is done to obtain a uniform, smooth, well-shaped base for the cast.
• A 12–15 mm strip of modeling wax is used for boxing. The boxing strip should be at least 13 mm measured vertically from
the highest point of the impression.
• The beading wax is adapted 3–4 mm below the height of contour of the impression flanges. The beading should be at least
4 mm wide.
• Caulking compound and paddle boxing method is best suited for zinc oxide eugenol impressions. Beading is done using
caulking compound and boxing is done using a metal strip.
• The boxed impression is attached/supported with a table tennis paddle while pouring the cast.
• Ideal requirements of a temporary denture base was given by Elder (1955).
• Tucker in 1966 stated that the base plate should not abrade the surface of the cast during removal and placement.

https://t.me/DentalBooksWorld
Chapter 14 • Prosthodontics 1173

Different Materials Used for Construction of Base Plate


Heat cure resins Bruver in 1962
Fluid resin base plates Browning in 1973
Thermoplastic resins Terry and Wahlberg in 1966
Shellac base plates Greener, Harcourt, and Cautenschlager in 1972
Wax base plate Boucher, Hickey, and Zarg in 1975

Fabrication of Occlusal Rims


An occlusal rim is defined as, “Occluding surfaces built on temporary or permanent denture bases for the purpose of making
maxilla–mandibular relation records and arranging teeth.”
Dimensions used to fabricate a standard occlusal rim
Maxillary Occlusal Rim
• Maxillary occlusal rim is “U” shaped.
• It should be 22 mm high from the depth of the sulcus at the region of canine eminence.
• The anterior edge of the occlusal rim at the midline should be about 8 mm away from the incisive papilla.
• It should be 4–6 mm wide in the anterior region.
• The occlusal table should be 10–12 mm above the crest of the alveolar ridge in the anterior region.
• The occlusal table should be 18 mm high when measured from the depth of the sulcus in the posterior region.
• The occlusal table should be 5–7 mm high when measured from the crest of the ridge in the posterior region.
• The occlusal table should be 8–12 mm wide in the posterior region.
Mandibular Occlusal Rim
• Mandibular occlusal rim is “V” shaped or less wider “U” shaped.
• It should be 6–8 mm high when measured from the crest of the ridge in the anterior region.
• It should be 18 mm high when measured from the depth of the sulcus in the canine eminence region.
• It should be 3–6 mm high when measured from the crest of the ridge in the posterior region.
• The occlusal plane should flush to two-third height of the retro-molar pad in the posterior region.
• The width of the occlusal table should be 4–6 mm in the anterior region and 8–12 mm in the posterior region.

Clinical Guidance for Determining the Shape of the Occlusal Rim


The standard occlusal rim should be reshaped for the patient according to the following clinical guidelines:
• Maxillary anterior edge should be 0–2 mm below the upper lip at rest.
• The level of the upper lip in relation to occlusal rim shows the lower lip line. But this can vary between patients.
• Maxillary posterior occlusal plane should be one-fourth inch below the opening of the Stenson’s or parotid duct.
• Mandibular incisal edge should be at the level of the lower lip and about 2 mm behind the maxillary incisal edge.
• Canine eminence of the lower occlusal rim should be located at the corner of the mouth.
• Posterior part of the lower occlusal plane should extend to two-third the height of the retromolar pad.
• Anterior maxillary occlusal plane should be parallel to the interpupillary line. The anteroposterior occlusal plane should be
parallel to the ala-tragus line or the Camper’s line or Bromel’s line.

MAXILLO-MANDIBULAR JAW RELATIONS


Jaw relation is defined as, “Any relation of the mandible to the maxilla.”

Orientation Jaw Relation


• This record gives the angulation of the maxilla in relation to the base of the skull.
• It is necessary to do orientation jaw relation before carrying out other jaw relations.
• Orientation jaw relation can be recorded with a facebow.

https://t.me/DentalBooksWorld
1174 Triumph's Complete Review of Dentistry

• Facebow is “A caliper-like device which is used to record the relationship of the maxillae and/or the mandible to the TMJ.”
• The facebow helps to orient the cast in the patient’s terminal hinge axis.
• Hinge axis is the imaginary line around which the condyles can rotate without translation.
• The terminal hinge position is the most retruded hinge position and it is learnable, repeatable, and recordable.
• It coincides with centric relation. A 12–15 degree pure rotational movement of the joint is possible in this position.
• When a patient opens his mouth widely more than 12–15 degrees, then the condylar rods will move anteriorly (Translation).

Parts of a Facebow
• U-shaped frame
• Condylar rods
• Bite-forks (the thickness of the bite fork & the wax together should not be more than 6 mm)
• Locking device
• Orbital pointer with a clamp
Types of Facebows
1. Arbitrary facebow
• Fascia type
• Ear piece type
• Hanau facebow (Spring bow)
• Slidematic (Denar)
• Twirl bow (It does not require any physical attachment to the articular)
• Whip-mix (It has a built-in hinge axis locator)
2. Kinematic or hinge bow

Arbitrary Facebow
• It is the mostly commonly used facebow in complete denture construction. The hinge axis is approximately located.
• The condylar rods are positioned approximately 13 mm anterior to the auditory meatus on a line running from the outer
canthus of the eye to the top of the tragus also called the canthotragal line.
• This is done using a Richey condylar marker.
Kinematic Facebow
• This facebow is generally used for the fabrication of a fixed partial denture and full mouth rehabilitation. It is not generally
used for complete denture fabrication.

Vertical Jaw Relation


“The length of the face as determined by the amount of separation of the jaws.”
Importance of vertical jaw relation
• The vertical jaw relation is the most critical record because errors in this record produce the first sign of discomfort.
• The various effects of altered vertical jaw relation are:
Increased VD Decreased VD
Increased trauma to the denture-bearing area Comparatively lesser trauma to the denture-bearing area
Increased lower facial height Decreased lower facial height
Cheek biting Angular cheilitis due to folding of the corner of the mouth
Difficulty in swallowing and speech Difficulty in swallowing
Pain and clicking in the TMJ Pain, clicking, discomfort of the TMJ accompanied with headache
and neuralgia
Stretching of facial muscles Loss of lip fullness

https://t.me/DentalBooksWorld
Chapter 14 • Prosthodontics 1175

Increased volume or cubical space of the oral cavity Decreased volume or cubical space of the oral cavity
Obstruction of the opening of the Eustachian tube due to the
elevation of the soft palate due to elevation of the tongue/mandible
Corners of the mouth are turned down, thinning of the vermillion
border of the lip
Loss of muscle tone

Recording of Vertical Jaw Relation


1. Vertical dimension at rest position
• It is the length of the face when the mandible is in rest position.
• VD at rest = VD at occlusion + Freeway space
• The following methods can be used to measure the vertical dimension at rest:
–– Facial measurements after swallowing and relaxing
–– Tactile sense
–– Measurement of anatomic landmarks
–– Speech
–– Facial expression
2. Vertical dimension at occlusion
• It is the length of the face when the teeth are in contact and the mandible is in centric relation or the teeth are in centric
relation.

Measurement of Vertical Dimension at Occlusion


Mechanical methods:
A. Ridge relation
a. Distance from the incisive papilla to mandibular incisors
• The incisive papilla is a stable landmark that does not change a lot with the resorption of the alveolar ridge.
• The distance of the papilla to the maxillary incisor edge is 6 mm, usually the vertical overlap between the upper and
lower incisors is 2 mm.
• Hence, the distance between the incisive papilla and the lower incisors will be approximately 4 mm.
b. Ridge parallelism
• The mandible is parallel to the maxilla only at occlusion.
• The mandible of the patient is adjusted to be parallel to maxilla. This position associated with a 5-degree opening of
the jaw in the TMJ gives a correct amount of jaw separation.
B. Pre-extraction records
a. Profile photographs
b. Profile silhouettes (the chin should be at least 2 mm above the level of the lower border of the silhouette)
c. Radiography
d. Articulated casts
e. Facial measurements
C. Measurements from former dentures
a. A
 Boley’s gauge is used to measure the distance between the border of the maxillary and the mandibular denture when
both these dentures are in occlusion
D. Physiological methods
a. Power point: (by BOOS)
• A metal plate (central bearing plate) is attached to the maxillary record base.
• A bimeter is attached to the mandibular record base.
• The bimeter has a dial, which shows the amount of pressure acting on it.
• The highest value is called the Power Point.

https://t.me/DentalBooksWorld
1176 Triumph's Complete Review of Dentistry

b. Using wax occlusal rims


c. Physiological rest position (Niswonger and Thomson in 1934)
d. Phonetics:
• The maxilla and mandible show a characteristic relationship during speech.
E. Skin
If the VD is too high, the skin of the cheeks will appear very stretched and the nasolabial fold will be obliterated, the nasolabial
angle will be increased.

Silverman’s Closest Speaking Space


• It was first described by Silverman.
• This differs from Niswonger’s method in that the Niswonger’s method measures the vertical dimension when the muscles
are at rest or physiological tonus whereas in this method the muscles are active.
• When sounds like ch, s, j are pronounced, the upper and lower teeth reach their closest relation without contact.
• This minimal amount of space between the upper and lower teeth in this position is called the “Silverman’s closest speaking
space.”
• This space indicates the vertical dimension of the patient.
• In an ideal case, the lower incisor should almost touch the palatal surface of the upper incisor.
• Increase in the freeway space between the U and L incisors indicates an inadequate vertical dimension at occlusion.
• A decrease in the closest speaking space will indicate an excessive vertical dimension at occlusion.
The “F” or “V” and “S” speaking anterior tooth relation:
• This was given by Pound and Murrel.
• In this method, the incisal guidance is established by arranging the anterior teeth on the occlusal rim before recording the
vertical dimension at occlusion.
• The position of the anterior teeth is determined by the position of the maxilla when the patient pronounces words beginning
with “F” and “V.”

Horizontal Jaw Relation


It is the relationship of the mandible to the maxilla in a horizontal plane. It can also be described as the relation of the mandible
to maxilla in the anteroposterior direction.
• Centric relation denotes the relationship of the mandible to the maxilla when the mandible is at its posterior most position.
• Centric relation is the most posterior relation of the mandible to the maxilla and the anterosuperior relation of condyle to
the glenoid fossa.
• Eccentric relation denotes the relationship of the mandible to the maxilla when the mandible is at any position other than
the centric relation position.

Significance of Centric Relation


• We know that proprioceptive impulses guide the mandibular movements.
• In dentulous patients the proprioceptive impulses are obtained from the periodontal ligament.
• Edentulous patients do not have any proprioceptive guidance from their teeth to guide their mandibular movements.
• The source of the proprioceptive impulses for an edentulous patient is transferred to the TMJ.
• The CR acts as a proprioceptive center to guide the mandibular movements.

Methods of Recording the Centric Jaw Relation


1. Physiological methods
• Tactile or inter occlusal check records
• Pressureless method
• Pressure method

https://t.me/DentalBooksWorld
Chapter 14 • Prosthodontics 1177

2. Functional method (Chew-in method)


• Needles-House method
• Patterson method
3. Graphic methods
• Intraoral
• Extraoral
4. Radiographic methods
• Profile radiographs
• Facial photographs

LAB PROCEDURES PRIOR TO INSERTION


• Disarticulation is best done in the presence of two people.
• The pattern is invested in a dental flask with dental stone or plaster using a 2-pour or 3-pour technique.
• Improper positioning of the cast in the flask will produce uneven exposure of the artificial teeth.
• The advantage of 3-pour technique is that it facilitates easy removal of the denture after curing.
• Dewaxing is carried out by placing the flask in boiling water (100°C) for 5 minutes.
• Dewaxing for a prolonged time will lead to leaching of the wax into the investing plaster.
• The ridge lap portions of the teeth are trimmed using a round bur to create small retentive pot holes (diatorics).
• For processing of a denture, polymer–monomer proportion is 3:1 by volume and 2:1 by weight.
• The working time is the time elapsing between the stringy stage and the beginning of rubbery stage.
• The working time is affected by temperature. Decrease in temperature increases the working time.
• According to ADA specification number 12, dough should be moldable for at least 5 minutes.
• Over packing leads to excessive thickness of the material and malpositioning of prosthetic teeth.
• Under packing leads to noticeable denture base porosity.
• The flask is closed and the closing force is applied slowly during the trial packing to allow the excess dough known as flash
to flow out between the halves of the flask.
• The flask is opened and the flash is trimmed away with a “Le Cron wax carver.”
• After final closure, the flasks are kept at room temperature for 30–60 minutes. This is known as Bench Curing.

Curing Cycle
• Long curing cycle: Heat the flask in water at 60–70°C for 9 hours.
• Short curing cycle: Heat the flask at 65°C for 90 minutes, then boil for 1 hour for adequate polymerization.
• The flask should be cooled slowly (i.e.) bench cooled. Sudden cooling can result in warpage of the denture due to differential
thermal contraction of the resin and the gypsum mold.
• Cooling overnight is ideal.
• Finishing of the denture:
–– The thickness of the palatal surface is reduced using a large, egg-shaped bur.
–– Reduction should never be carried out on the tissue surface.
–– Uniform thickness of 2–2.5 mm must be maintained.
–– Small irregularities should be removed using a “Paintbrush motion” against a lathe mounted acrylic trimmer.

If too much monomer is used (low polymer monomer ratio), then:


1. There will be greater curing or polymerization shrinkage
2. More time is needed to reach the packing consistency
3. Porosity can occur in the denture

If too little monomer is used (high polymer monomer ratio), then:


1. Cured acrylic will be granular.
2. Dough will be difficult to manage and it may not form a continuous plastic mass.
https://t.me/DentalBooksWorld
1178 Triumph's Complete Review of Dentistry

Evaluation of Denture Function After Insertion


• The retention of the denture can be evaluated by checking for the peripheral seal of the denture.
• One of the most challenging functions that should be reproduced in a denture is speech.
• Dentures play different roles in the production of different sounds:
1. Bilabial sounds (b, p, and m) are controlled by the lip support and become defective due to the absence of lip support or
alteration in the vertical dimension at occlusion.
2. Labiodental sounds (f and v) are governed by the relation of the incisal edges of the upper anterior teeth to the lower lip.
• If the teeth are set too high then “f ” will sound like “v.”
3. Linguodental sounds (th) are governed by the position of the tongue between the upper and lower anteriors.
• Normally the tongue should project 3 mm anteriorly between the teeth.
• If the tongue gets positioned about 6 mm in front of the teeth, then it means the teeth have been set very lingually.
4. Linguoalveolar sounds (t, d, s, z, v, and l) are made when the tongue touches the anterior part of the palate.
• These are the most important of all sounds in a complete denture because it is determined by the thickness of the
denture base.
5. S sound is controlled by the anterior part of the palatal plate of the denture base. It is considered separately because it is
produced in two tongue positions.
• Hence it is also called the dental and alveolar sounds.
• The acoustic character of “s” sound is a strong high frequency sound wave of 3–4 KHz.
• The auditory character of “s” sound is a sharp loud sound.
6. Valar sound (k, g, and kg) are dependent on the posterior part of the palate and are not affected by the prosthesis.

Direct Sequelae of Wearing Complete Denture


Denture stomatitis
• It is the pathological reaction of the palatal portion of the denture-bearing mucosa.
• It is commonly known as, “Denture induced stomatitis,” “Denture sore mouth,” or “Chronic strophic candidiasis.”
• It is seen in 50% of complete denture wearers. According to Newton, denture stomatitis can be classified as
–– Type I: Localized simple infection with pinpoint hyperemia.
–– Type II: Erythematous type, generalized simple type presenting a more diffuse erythema involving a part or the entire
denture covered mucosa.
–– Type III: Granular type, involving the central part of the hard palate and alveolar ridge.
• Type I is usually trauma induced, Types II and III are associated with microbial plaque accumulation.
• CADS (Candida-associated Denture Stomatitis) are often associated with angular cheilitis.
• Surgical management of DS includes the elimination of deep crypts in Type III DS. This is preferably done by Cryosurgery.
Flabby ridge
• Flabby ridges are most commonly seen in the anterior part of maxilla opposing natural mandibular anterior teeth.
• Flabby ridges provide poor support for the denture. Hence they should be surgically removed.
Traumatic ulcers
• They are commonly known as, “Sore Spots.”
• They usually develop within 1–2 days after the placement of new dentures.
• They are small, painful lesions covered with a grey necrotic membrane and surrounded by inflammatory halo with firm,
elevated borders.
Denture irritation hyperplasia
(EPULIS FISSURATUM)
• It is a hyperplastic reaction of the mucosa occurring along the borders of the denture.
• These lesions result from trauma due to unstable dentures with thin denture flanges.
• Deep ulcerations, fissuring, and inflammation may occur at the depth of the sulcus.
• They may produce severe lymphadenopathy mimicking a neoplasm.

https://t.me/DentalBooksWorld
Chapter 14 • Prosthodontics 1179

Burning mouth syndrome (BMS)


• It is characterized by burning sensation in the structures in contact with the dentures without any visible changes in the
mucosa.
• The mucosa is clinically healthy in BMS.
• Pain usually starts in the morning and aggravates during the day.
• Burning sensation is usually associated with dry mouth and persistent altered taste sensation.

Indirect Sequelae of Wearing CD


1. Atrophy of masticatory muscles
• Atrophy of a muscle due to poor usage is called disuse atrophy. It is more common in women and older people.
• Generally CD patients take around 7 times more effort than dentulous patients to obtain the same result.
• In other words, what a dentulous person can do with one chewing cycle, will take 7 chewing cycles in a CD patient.
2. Nutritional deficiencies

Residual Ridge Resorption


• This is the most common and important sequale of wearing complete dentures.
• Residual ridge resorption (RRR) is more common in women due to osteoporotic changes in the bone.
• Alveolar remodeling is more important in areas with thick cortical bone especially in the buccal parts of the maxilla and
lingual parts of the mandible which are load-bearing regions.
• Excessive pressure applied to the non-stress bearing areas can produce RRR.
• RRR more common in patients who wear their dentures continuously overnight.
Pattern of RRR
• Resorption occurs more rapidly in the first 6 months after extraction of teeth and at a slower pace till 12 months.
• The rate of resorption progresses after 65 years of age.
• IN general, RRR occurs more commonly in females than males.
• During the first year after extraction, the amount of RRR is about 2–3 mm in the maxilla and 4–5 mm in the mandible.
• Later the annual rate of reduction of height in mandible is 0.1–0.2 mm and it is four times less in the edentulous maxilla.
• Severe RRR can be related to a small gonial angle.
Clinical features of RRR
• Decreased VD at occlusion
• Reduction of lower facial height
• Anterior rotation of the mandible
• Increase in relative prognathism
• Resorption is centripetal (toward the center) in the maxilla and centrifugal (away from the center) in the mandible
• Hence the size of the maxillary arch will decrease in resorption and the size of the mandibular arch will increase with resorption

RELINING OF A COMPLETE DENTURE


Definition: It refers to the process of adding base material to the tissue surface of the denture in a quantity sufficient to fill the
space which exists between the original denture contour and the altered tissue contour.

Indications for relining:


• Immediate dentures after 3–6 months where maximum residual ridge resorption would have occurred
• Economic reasons where a patient cannot afford a new denture
• Geriatric patients

https://t.me/DentalBooksWorld
1180 Triumph's Complete Review of Dentistry

Contraindications for relining and rebasing:


• When the residual ridge had resorbed excessively
• Abused soft tissues due to an ill-fitting denture
• TMJ problems
• Patient dissatisfied with the appearance of existing dentures
• Unsatisfactory jaw relationships
• Severe osseous undercuts

Relining Procedures
Boucher’s technique:
• Static and open mouth relining technique.
• Both the maxillary and mandibular dentures are relined at the same time.
• Used ZnOE as the impression material.
Winkler’s functional relining method:
• Fluid resins (tissue conditioners) are used impression materials.
Chairside relining method: Makes use of acrylic that is added to the denture

Rebasing of Complete Denture


• Relining is the correction of denture base and tissue relationship without changing the occlusal relation.
• Rebasing is a process of replacing all the base materials of a denture.
• The main disadvantage of relining and rebasing of CD is change in centric relation.
• If the vertical dimension is changes, rebasing is required, otherwise relining would be sufficient.
• Rebasing is similar to relining except that there is extensive replacement of the denture base material.
• In rebasing, a new VD and CR are recorded.
• Generally one must keep in mind that when tissue damage is excessive the treatment shifts from relining to rebasing.
• Another thumb rule is, rebasing should be done if the VD of the patient is changed.
• Rebasing is done properly only in dentures with porcelain teeth.

Special Dentures
Immediate denture: • A complete or removable partial denture constructed for insertion immediately following
Interim immediate denture the removal of natural teeth. These immediate dentures may be of two types
1. Interim immediate denture
2. Conventional immediate denture.
• These are indicated when age, health or lack of time precludes more definitive treatment.
• It is temporary partial denture used temporarily, during the healing period of the patient
to preserve ridge contour, until the permanent denture can be fabricated.
• They are mainly indicated in patients with periodontal disease going in for total
extraction.
Conventional immediate • It is an immediate denture, which can be later modified to serve as the permanent
denture prosthesis.
• It is usually done for patients undergoing total extraction.
Transitional denture • A Transitional denture may become an interim denture when all of the natural teeth have
been removed from the dental arch.
• Usually transitional dentures are used as a supportive therapy when the patient is
expected to transit from the partially edentulous condition to a completely edentulous
condition due to poor periodontal prognosis of the existing teeth.

https://t.me/DentalBooksWorld
Chapter 14 • Prosthodontics 1181

Treatment denture • To establish new occlusal relationship or VD and to condition the soft tissues that have
been abused by ill-fitting prosthesis.
Tooth supported over-dentures • These are the dental prosthesis that replaces the lost or missing natural dentition and
associated structures of the maxilla and/or mandible and receives partial support or
stability from one or more modified natural teeth.
• They are also known as Hybrid dentures or tooth supported CDs.

Combination Syndrome
• This was identified by Kelly in 1972.
• It was observed in patients wearing a maxillary complete denture opposing a mandibular distal extension prosthesis.
• This syndrome is not seen in cases of CD opposing natural mandibular posterior teeth.
Sequence of Combination Syndrome:
• There is over force acting upon anterior portion of the maxillary denture.
• This leads to increased resorption of the anterior part of the maxilla which gets replaced by flabby tissue.
• The occlusal plane gets tilted anteriorly upwards and posteriorly downwards due to lack of anterior support.
• The labial flange will displace and irritate the labial vestibule leading to the formation of epulis fissuratum.
• Posteriorly there will be fibrous overgrowth of the tissues in the maxillary tuberosity.
• Due to the tilt of occlusal plane, the mandible shifts anteriorly during occlusion.
• The VDO is decreased.

QUICK FACTS

COMPLETE DENTURE BASICS


• 1 kg force = 10 Newton (approx.)
• 1 kg force = 2.2 pounds
• The maximum biting force in molar area is 800 N (80 kg) – Conscious effort
• The maximum biting force in incisor area is 200 N (20 kg) – Conscious effort
• Maximal bite forces appear to be 5–6 times less for complete denture wearers compared to natural dentition
• Periodontal mechanoreceptors can precisely discriminate between forces of different magnitude. Incisors can detect forces up
to 0.01 N or less, whereas the threshold for molars is 0.1 N
• Maximum axial forces generated during swallowing are in the range 70–150 N
• The highest masticatory forces are generated when the maxillary and mandibular teeth are in contact
• Loss of teeth mainly affects the alveolar bone
• Residual Ridge Resorption (RRR) is directly proportional to bone resorption factor, pressure, and anatomic factor
• RRR is inversely proportional to bone formation factor and damping effect
• The direction of resorption of maxillary ridge is upwards and palatal direction (maxilla becomes smaller)
• Resorption of mandibular anterior ridge occurs in downward and lingual direction.
• Resorption of mandibular posterior ridge occurs in downward and buccal direction (mandible becomes wider)
• These patterns of resorption results in a wide mandible and narrow maxilla. This makes many patients appear prognathic.
• The mean ratio of anterior maxillary RRR to the anterior mandibular RRR is 1:4
• Patients with large tongue mostly experience poor lower denture stability
• Thick and pasty saliva is due to reflex sympathetic stimulation of salivary glands
• Watery saliva is due to parasympathetic stimulation
• Treatment modality for bilateral soft-tissue tuberosity undercuts is removal of tissue undercut on one side so that the undercut
on the other side helps in retention

https://t.me/DentalBooksWorld
1182 Triumph's Complete Review of Dentistry

• Treatment of choice for bilateral bony tuberosity undercuts is removal of both the undercuts such that no bony undercuts
exists
• The most common site of occurrence of mandibular tori is in the premolar region
• 33% of edentulous mouths have retained root tips
Anatomical Landmarks of Maxilla
• Labial frenum has no muscle fiber attachments, hence it is a passive frenum.
• Orbicularis oris is the main muscle of lip. Its fibers run horizontally and it has an indirect displacing effect on the denture.
• Buccal frenum has the following attachments:
–– Levator anguli oris – Attaches beneath the frenum
–– Orbicularis oris – Pulls the frenum in a forward direction
–– Buccinator – Pulls the frenum in the backward direction
• The distal end of the buccal flange of the denture should be adjusted in such a way that there is no interference to the coronoid
process during mouth opening.
• The distolateral border of denture base rests in the hamular notch.
Anatomical Landmarks in Mandible
• The muscle, incisivus, and orbicularis oris influence the labial frenum.
• Unlike maxillary labial frenum, it is an active muscle.
• Buccal vestibule is influenced by the action of masseter.
• When the masseter contracts, it pushes inward against the buccinator, producing a bulge into the mouth. It is reproduced as a
notch in the denture flange called the masseteric notch.
• The support of the mandibular denture comes from the body of the mandible.
• The available denture-bearing area for an edentulous mandible is 14 cm2 but for maxilla it is 24 cm2 (Hence, the mandible is
less capable of resisting occlusal forces.).

Methods for Recording Centric Relation


1. Static or pressureless method involves, Stapler pin method or the Nick and Notch method.
• The Nick and Notch are placed on the maxillary occlusal rim. The nick prevents the lateral movements and the notch
prevents the anteroposterior movements.
• The trough is placed in the mandibular occlusal rim.
2. Functional method of Needles house involves the fabrication of occlusal rims made of impression compound.
• Four metal beads or styli are embedded into the premolar and the molar areas of the maxillary occlusal rim.
• The maxillary occlusal rim is inserted into the patient’s mouth and the patient is asked to make protrusive, retrusive,
right and left lateral movements of the mandible.
• When all the movements are made, a “diamond shaped marking” pattern is formed on the mandibular occlusal rim.
• The posterior most point of this diamond pattern indicates the centric jaw relation.

Postinsertion Problems
Problem Causes
Soreness on the slopes of the ridge Deflective occlusal contact resulting in shifting of bases
Soreness on the crest of the ridge Increased VD resulting in heavy contacts
Generalized soreness of the basal seat area Increased VD
Fleeting painful ulcers/sores Increase VD
Burning sensation in anterior palate region of Inadequate relief in incisive papilla
patients wearing new dentures
Numbness and tingling sensation in the anterior Overextension of the anterior lingual border
one-third of the palate
Loosening of denture while smiling Due to inadequate relief of the buccal frenum

https://t.me/DentalBooksWorld
Chapter 14 • Prosthodontics 1183

Loosening of upper denture while opening the Excessive thickness of distobuccal flange
mouth Interference with Coronoid process
Difficulty while swallowing Due to over extension of the lingual flange into the lateral throat form
Increased VD
Pain and soreness during chewing Deflective occlusal contacts
Mucosal irritation Overextension of the denture borders
Epulis fissuratum Ill-fitting or over extended dentures
Papillary hyperplasia Results from candida infection and improper relief of the palatal area in
complete dentures
Clicking noise during teeth contacts Increased VD and improper retention
Tight dentures become loose during usage Errors in occlusion
Cheek biting Insufficient horizontal overlap of posterior teeth

Quick Facts in Complete Denture


• The occlusal adjustments in an ill-fitting denture is done after 48 hours, after the swelling and inflammation has subsided.
• The dentures should be relined after calcification of extraction socket, i.e., after 6–8 months.
• Relining is usually done 3–4 months after construction of immediate dentures.
• Transitional dentures are used when the prognosis of the remaining teeth is very poor.
• If a fixed prosthesis is contemplated the temporary removable partial denture is given.
• If a definitive prosthesis is contemplated, interim denture is given.
• Jiffy denture is a type of Interim Immediate Denture (IID)
• After insertion, immediate denture should be left in the mouth for first 24 hours.
• The occlusal corrections are done after 48 hours.
• The main disadvantage of immediate denture is necessity for frequent relining and rebasing.
• The function of surgical splint in immediate denture is that it helps in trimming the bony spicules for proper fitting of the
denture.
• The other disadvantage is that there is no try-in for anterior teeth.
• In overdentures, cuspids, and bicuspids are usually selected as abutments.
• The use of selected initial roots in overdenture therapy is called submucosal vital root retention.
• If porcelain teeth are used in single CD against natural teeth, there will be resorption of natural teeth.
• If acrylic teeth are used against natural teeth, there will be resorption of resin teeth.
• Schubiger is a bar type attachment of overdentures. It consists of Schubiger sleeve, studs placed in the roots, and a Gerber
bar containing this sleeves.
• Minimum number of implants in maxillary overdenture is 4
• Minimum number of implants in mandibular overdenture is 2

FIXED PARTIAL DENTURE

SYNOPSIS

Fixed Prosthodontic Treatment involves the replacement and restoration of teeth with by artificial substitutes that are not
readily removable from the mouth
Fixed Partial Denture (PDF) or Bridge:
• Defined as a prosthetic appliance permanently attached to remaining teeth, which replaces one or more missing teeth.

https://t.me/DentalBooksWorld
1184 Triumph's Complete Review of Dentistry

Indications for FPD:


• Short span edentulous arches
• Presence of sound teeth that can offer sufficient support adjacent to the edentulous spaces
• Cases with ridge resorption where a removable partial denture cannot be stable or retentive
• Patient’s preference
• Mentally compromised and physically handicapped patients who cannot maintain the removable prosthesis
Contraindications for FPD:
• Large amount of bone loss as in trauma
• Very young patients where teeth have large pulp chambers
• Presence of periodontally compromised abutments
• Long span edentulous spaces
• Bilateral edentulous spaces, which require cross arch stabilization
• Congenitally malformed teeth, which do not have adequate tooth structure
• Mentally sensitive patients who cannot co-operate with invasive treatments

Classifications of FPD
1. Depending on the type of connector
• Fixed partial denture
• Fixed removable partial denture
• Removable fixed partial denture
2. Depending on types of material used
• All metal crowns
• Metal ceramic crowns
• All ceramic crowns
• All acrylic crowns
• Ceramic Veneer
• Acrylic Veneer
3. Depending on the length of span
• Short span bridges
• Long span bridges
4. Depending on the duration of use
• Permanent fixed partial dentures
• Long span bridges
–– Interim prosthesis
–– Periodontally weak abutment (Mary-land bridge)
–– Splints
5. Based on type of abutment
• Normal/ideal abutment
• Cantilever abutment
• Pier abutment
• Mesially tilted abutment
–– Mesial half crown
–– Telescopic crown
• Endodontically treated teeth
–– Core: Plastic core material
–– Postcore restorations
–– Periodontally weak teeth
–– Implants abutments

https://t.me/DentalBooksWorld
Chapter 14 • Prosthodontics 1185

Parts of a Fixed Partial Denture


Retainer Unit of FPD that attaches abutment to the suspended unit
Connector Unit of FPD that attaches retainer to the suspended unit
Pontic Suspended portion of FPD that replaces the missing tooth
Abutment Abutment is the natural tooth or a root which retains or supports the bridge
Abutment is not as such a part of fixed prosthesis instead it is the actual natural tooth that supports the
fixed prosthesis

Retainer
The part of a fixed partial denture which unites the abutment to the remainder of the restoration.
Types of retainers
Based on tooth coverage
1. Full veneer crowns
2. Partial veneer crowns
3. Conservative (minimal preparation) retainers
Based on materials being used
1. All metal retainers
2. Metal ceramic retainers
3. All ceramic retainers
4. All acrylic retainers

Full Veneer Crown


Indications • Posterior teeth in nonesthetic areas
• A retainer requiring maximum retention
• Short clinical crowns
Contraindications • Teeth in aesthetic zone
• Extensively restored tooth

Partial Veneer Crown


Indications • Intact or minimally restored teeth
• Teeth with normal anatomic crown form, i.e., without excessive cervical construction
• Teeth with adequate crown length
Contraindications • High caries rate
• Short-teeth
• Bell-shaped teeth
• Teeth with insufficient buccolingual width

Connectors
The portion of a fixed partial denture that unites the retainer and pontic.
They are broadly classified into
1. Rigid connectors
2. Nonrigid connectors
a. Tenon–Mortise connectors
b. Loop connectors
c. Split pontic connectors
d. Cross pin-wing connector

https://t.me/DentalBooksWorld
1186 Triumph's Complete Review of Dentistry

Rigid connectors
They are used to unite retainers and pontic in a fixed–fixed partial denture.
These connectors are used when the entire load on the pontic is to be transferred directly to the abutments.
Nonrigid connectors
These connectors are indicated in case where a single path of insertion cannot be achieved due to nonparallel abutments.
Indications of nonrigid connectors
• Short span FPD replacing one tooth
• Tilted FPD abutment
• An edentulous space on both sides of the tooth (Pier abutment)
Tenon–Mortise connectors
This nonrigid connector consists of a Mortise (female) prepared within the contours of the retainer and a Tenon (male)
attached to the pontic.
Loop connectors
They are used when an existing diastema is to be maintained in a planned fixed prosthesis.
The loop may be cast from a platinum-gold palladium alloy wire.
Split pontic connectors
• They are used only in cases of pier abutments.
• Here the connector is incorporated within the pontic.
• The pontic is split into mesial and distal segments. The mesial segment is fabricated with a shoe/key.
• The distal segment is fabricated with a key-way to fit over the shoe.
Cross pin and Wing connectors
• They are similar to split pontic connectors.
• It can be used for tilted abutments.
• A wing is attached to the distal retainer.
• The wing should be fabricated such that it aligns with the long axis of the mesial abutment.
• The wing along the distal retainer is termed as the “retainer wing component.”
• The pontic is attached to the mesial retainer and it is termed the “retainer pontic component.”

The Law of Beams


• Width of the connector is directly proportional to its resistance to deformation
• Height of the connector is directly proportional to the cube root of strength
• The deflection of an FPD is proportional to the cube of its span length

PONTICS
Pons → Latin word – bridge
Classification
1. Based on Mucosal Contact
a. With mucosal contact
• Ridge lap or saddle pontic
• Modified ridge lap
• Ovate pontic
• Conical pontic
b. Without mucosal contact
• Sanitary/hygienic pontic
• Modified sanitary pontic/perel pontic/arc-shaped pontic

https://t.me/DentalBooksWorld
Chapter 14 • Prosthodontics 1187

2. Based on Materials Used


• All metal pontics
• All ceramic pontics
• Metal ceramic pontics
• Metal with resin facing pontics
• Fiber reinforced composite pontics
3. Based on Method of Fabrication
• Custom made pontics
• Prefabricated
–– Trupontic
–– Interchangeable facing
• Sanitary pontic
• Pin-facing pontic
• Modified pin-facing pontic
• Reverse pin facing
• Harmony pontic
• Porcelain fused to metal pontic
Ridge Lap or Saddle Pontic
• Snug contact
• Over laps both buccal and lingual surfaces of the ridge
• Known as ridge lap
• Forms a large concave contact with the ridge
• Simulates emergence profile of missing tooth
• Must not displace soft tissues or cause blanching
Modified Ridge Lap
• Does not contact lingual aspect of ridge
• Vertical arms end at the crest of the ridge
• Most commonly used in areas that have high visibility
Ovate Pontic
• Esthetically appealing design
• Bluntly rounded tissue contacting surface
• Broad flat ridges
Conical Pontic
• Egg-shaped, bullet-shaped, and heart-shaped pontic
• Design is related to “sanitary dummy”
• Knife edged residual ridge requires flatter contours with a narrow tissue contact area
• Emergence profile compromised
• Small tip
Sanitary/Hygienic Pontic
• Easy cleansing
• No contact with residual ridge, minimal chances of inflammation
• Occlusogingival thickness of pontic greater than 3 mm
• Convex configuration in both faciolingually and mesiodistally
• Conventional type is called “fish belly” design
Modified Sanitary Pontic
• Alternative design with mesiodistal concavity
• This design adds strength to connectors, at the same time allows space for easier cleansing

https://t.me/DentalBooksWorld
1188 Triumph's Complete Review of Dentistry

Based on Material Used


1. All metal pontics
• Indicated in parafunctional habits like bruxism
• High stress bearing teeth like mandibular molars
• Contraindicated when esthetics is of prime concern
2. All ceramic pontics
• Metal-free pontics
• Makes use of alternative high-strength material like alumina or zirconia
• Just opposite to all metal pontics
• Indicated on areas of prime esthetic concern
• Contraindicated on bruxism, reduced interarch pontic space, long-span bridges
3. Metal ceramic pontics
• Pontics have a metal substructure over which ceramic build-up is done
• Most commonly used pontic type
• Can be used in almost all clinical types
• Good esthetics
• Adequate strength
• Biocompatible
• Disadvantage includes more extensive lab procedures than all metal pontics
4. Metal with resin facing pontics
• Instead of ceramic, acrylic/composite resin is veneered to facial or buccal surface under the metallic substructure
• Indicated in long-term provisional restorations
• Contraindicated in definitive restorations
5. Fiber reinforced composite pontics
• Composite pontics fabricated around a resin fiber that provides strength to the prosthesis
• Indicated in resin bonded fixed prosthesis
• Anterior single missing tooth situations
• As temporary situations in young adults till a permanent prosthesis is fabricated
• Contraindicated in replacement of posterior missing tooth
• Also in long-span bridges and in deep bite cases

BASIC PRINCIPLES OF TOOTH PREPARATION


The basic principles of tooth preparation are
• Preservation of tooth structure
• Retention and resistance
• Structural durability
• Marginal integrity
• Preservation of periodontium

Preservation of Tooth Structure


“Muller De Van stated that the perpetual preservation of what is remaining is more important than the meticulous replacement
of what is lost.”

Retention and Resistance Forms


• Retention is defined as the ability of the preparation to prevent displacement of the restoration in a direction opposite to
the path of insertion.
• One of the major sources of retention in a fixed partial denture is tenso-friction.

https://t.me/DentalBooksWorld
Chapter 14 • Prosthodontics 1189

1. Primary retention
• Sleeve retention
• Wedge-type retention
2. Secondary retention
• Pins
• Boxes
• Grooves
Resistance forms
• Resistance is the ability of the prosthesis to resist displacement by forces directed in an apical or oblique direction.
Features determining the balance between resistance and retention forms
1. Taper
• Degree of taper is inversely proportional to the retention form.
• Zero degree taper is the most retentive but it is almost impossible to obtain
• The sum of degree of taper is known as “degree of convergence.”
• For optimum retention, 4–10 degree convergence is sufficient.
• A tapering fissure diamond is ideal to produce the required taper for any preparation.
• This diamond is designed with a three-degree taper; hence, if the operator aligns the diamond parallel to the long axis of
the tooth during tooth preparation, a six-degree convergence will be produced.
2. Freedom of displacement
• Limiting the freedom of displacement from torqueing and twisting forces aid to increase the resistance of the restoration.
3. Length
• The length of the crown improves retention in two ways:
–– The height of the prepared tooth should be greater than the tipping arc of displacement to prevent displacement of
the restoration.
–– Increase in height increases the area of cementation thereby improving the retention.
Structural Durability
• The ability of the restoration to withstand destruction due to external forces is known as structural durability.
• The amount of reduction required for structural durability depends on the type of restorative material being used and the
design of restoration.

Occlusal Reduction
• It is the most vital as most of the forces affecting the restoration, directly act on the occlusal surface.
The amount of occlusal reduction required for commonly used materials is:
Gold alloys Functional cusp – 1.5 mm
Nonfunctional cusp – 1.0 mm
Metal ceramic restorations 1.5–2.0 mm reduction in the functional cusp
1.0–1.5 mm reduction in the nonfunctional cusp
All ceramic restorations 2 mm reduction throughout
• Additional thickness in this region is necessary because the functional cusp of the tooth is the one, which bears the maximum
load during mastication.
• It is prepared on the palatal cusps of maxillary and buccal cusps of mandibular posterior teeth.

Marginal Integrity
• Marginal adaptation and the seating of the restoration affect marginal integrity.
• Casting shrinkage may lead to marginal discrepancy.
• The most accepted discrepancy is around 10 microns.

https://t.me/DentalBooksWorld
1190 Triumph's Complete Review of Dentistry

Preservation of the Periodontium


• Biological width is the distance between base of gingival sulcus and the alveolar crest.
• It is 2 mm in width (1 mm JE + 1 mm connective tissue).
• If the distance between the finish line and the alveolar crest is less than 2 mm, the restoration may lead to gingival
inflammation, loss of alveolar crest, and pocket formation.

Tooth Preparation Steps and Their Functions


Occlusal–planar reduction Structural durability
Functional cusp bevel Structural durability
Axial reduction Structural durability, resistance, and retention
Wing Resistance and retention
Chamfer finish Periodontal preservation and marginal integrity
Radial shoulder Periodontal preservation and structural durability
Gingival bevel Marginal integrity

Finish Lines
Finish line Bur used Indications Advantages Disadvantages
Chamfer Torpedo diamond 1. Finish line of Heavy chamfer is used Improper reduction will
choice for cast to provide 90-degree produce an undesirable fragile
metal restorations cavo-surface angle with piece of enamel (Lipping)
2. Lingual margins large rounded radius.
of metal ceramic Less stress and good
restorations success rate.
Shoulder Flat end tapered 1. All ceramic The wide ledge gives 1. The sharp internal line
diamond bur restorations resistance angle may cause stress
where sufficient concentration and fracture of
thickness of the the tooth.
margin is required 2. Requires more tooth
for structural reduction.
durability.
2. All anterior
restorations (and
facial margins of
metal ceramic
restorations) are
fabricated with a
shoulder where
esthetics is the
primary concern.
Shoulder with bevel Similar to shoulder 1. Labial finish line Superior marginal Requires subgingival extension.
finish line, but of metal ceramics adaptation. Detection of postcementation
an external bevel 2. Proximal boxes of Resists distortion caries is difficult
is created on the inlays and onlays. Facilitates removal of
gingival margin of 3. Occlusal shoulder unsupported enamel
the finish line. of onlays.
Knife edge Extremely thin 1. Young patients Conservative and Does not provide a distinct
2. MOD onlay ideal for marginal finish line.
3. Pinledge adaptation. Waxing and polishing becomes
restorations critical.
4. Inaccessible areas Overcontoured restoration in an
5. Finish lines used attempt to obtain the bulk.
in cementum.
https://t.me/DentalBooksWorld
Chapter 14 • Prosthodontics 1191

Supragingival finish lines • Better periodontal health


• Facilitates accurate impression making
• Allows accurate assessment of the fit
Subgingival finish line • Used when additional retention is needed
• Indicated in anterior zone where esthetics is a prime concern
• In cervical erosion and root hypersensitivity cases

Types of FPD
1. Cantilever FPD
• It is used when support can be obtained only from one side of the edentulous space.
• The abutment teeth on the supporting side should be strong enough to withstand the additional torsional forces.
• Support can be obtained from more than one tooth on the same side of the edentulous space.
2. Spring cantilever FPD
• This is a special cantilever bridge, exclusively designed for replacing maxillary incisors but these dentures can support
only a single point.
• Support is obtained from posterior abutments.
• Can be used in case of diastema.
3. Modified fixed removable partial dentures
• These were developed by Andrew and are known as Andrew’s bridge system.
• These dentures are indicated for edentulous ridges with severe vertical deficit.
• It consists of two fixed retainers attached to their abutments and connected by a rectangular bar that follows the curve of
the ridge under it.
4. Resin-bonded FPDS
A. Rochette bridge
• Rochette was the first person to design resin-bonded prosthesis.
• The wing-like retainers contain conical perforation for retention.
• The resin exposed through the metal perforations is subjected to external stress, abrasion, and marginal leakage.
• He also used silane coupling agents for additional retention.
B. Maryland bridge
• It is developed to overcome the shortcomings of Rochette bridges.
• These were developed by Livaditis and Thompson from the University of Maryland school of dentistry.
• Here retention is developed by the microporosities present on the tissue surface of the retainer.
• The etching is done by electrochemical etching by a 3.5% HNO3.
• Microporosities are created by etching the tissue surface of the retainer.
C. Virginia bridges
• They were first proposed by Moon and Hudgins.
• This resin-bonded FPD uses particle-roughened retainers.
• The retainer wax pattern are fabricated using resin. Salt crystals are sprinkled onto the surface of the resin pattern.
• The salt crystals are dissolved and the resin pattern is invested and casted (Lost-Salt technique).
• The salt crystals provide voids in the resin pattern and these voids will also be reproduced in the cast metal retainer
and they help in mechanical retention.

OCCLUSION
Features of Group Function
• Group function is characterized by contact of all the teeth on working side.
• Group function or unilateral balanced occlusion is widely accepted and used method of tooth arrangement in restorative
dental prosthesis.
• Mutually protected occlusion is also known as canine-protected occlusion or organic occlusion (cuspid protection theory).
• In case of missing canine or in the presence of anterior bone loss, the mouth should be probably restored to group function.
https://t.me/DentalBooksWorld
1192 Triumph's Complete Review of Dentistry

Features of Mutually Protected Occlusion


• Uniform contact of all teeth when the condyles are in their most superior position.
• Stable posterior tooth contacts with vertically directed resultant forces.
• CR = CO
• No contact of posterior teeth in lateral or protrusive movements.
• Anterior tooth contacts harmonize with functional jaw movements.

In an Ideal or Optimum Occlusion


• The load exerted on the dentition should be distributed equally.
• Loading should be parallel to the long axis of the tooth and the horizontal forces on any tooth should be avoided.
• There should be a cusp to fossa occlusal arrangement.
• During excursive movements, there should be no posterior tooth contact (decreased horizontal contact)

ABUTMENTS AND TYPES OF ABUTMENTS IN FIXED PARTIAL DENTURES


An abutment can be defined as “A tooth, a portion of a tooth, or that portion of an implant used for the support of a fixed or
removable prosthesis.”

The Major Criteria for Selecting an Abutment Tooth


1. Location, position, and condition of the tooth
• Teeth adjacent to edentulous spaces
• Vital teeth are predominantly preferred
• Pulp capped teeth should not be used as abutments as they are always under the risk of requiring RCT
• Endodontically treated teeth may be used as an abutments
2. Root configurations
• Roots with greater labiolingual widths are preferred
• Roots with irregular curvatures are preferred
• Teeth with longer roots serve as better abutments
• Teeth with conical roots can be used for short span fixed partial dentures
3. Crown root ratio
Ideal C:R ratio 1:2
Optimum C:R ratio 2:3
Favorable C:R ratio 1:1
4. Root surface area
Average root surface area (mm2)
Tooth Maxilla Mandible
Central incisor 204 154
Lateral incisor 179 168
Cuspid 273 268
1st Bicuspid 234 180
2nd Bicuspid 220 207
1st molar 433 431
2nd molar 431 426

https://t.me/DentalBooksWorld
Chapter 14 • Prosthodontics 1193

• The maxillary first molar has maximum root surface area (433) followed by mandibular first molar (431)
• Among posterior teeth, mandibular first premolars have least root surface area (180)
• Among anterior teeth maxillary canines have maximum root surface area (273) and the mandibular central incisors have
least root surface area (154)
• The principal limitation is mesiodistal tooth inclination, as teeth inclined more than 25–30 degrees make poor bridge
abutment
• For a tooth to be selected as an abutment, it should not have tilted more than 24 degrees
• Johnstone et al. proposed Ante’s law
• A ratio of 1:1 or greater satisfies the Ante’s law

Types of Abutments
1. Cantilever abutments
• More than average bone support should be present.
• Sufficient amount of tooth structure should be available because the final retainer should be more retentive.
• Endodontically treated teeth are not preferred.
• Ideal cantilever situations include:
–– Replacement of lateral incisor with canine support.
–– Replacement of first premolar with second premolar and first molar support.
2. Pier abutments
• A pier abutment is a single tooth with two adjacent edentulous spaces on either side.
• The forces acting on one end of the prosthesis will tend to lift the other end like a lever using abutment as a fulcrum. In
such conditions, the lifespan of the retainer is dramatically reduced.
• In order to prevent trauma to a abutment, a stress breaker should be provided near the pier abutment.
• The stress breaker is a nonrigid connector with a key in a key way.
• The key way is usually placed on the distal surface of the pier abutment.
• The male component or the key is attached to the mesial surface of the mesial pontic of the distal edentulous space.
• If the pier abutment is mobile, then a rigid connector should be used instead of a nonrigid connector.

Mechanical Methods of Fluid Control


• Rubber dam
• Suction devices
• High volume vacuum
• Saliva ejector
• Svedopter
• Cotton rolls
Cotton Rolls
• Simplest method of fluid control and isolation.
1. Maxillary arch isolation
Single cotton roll in the buccal vestibule adjacent to maxillary first molar is sufficient.
2. Mandibular arch isolation
Multiple cotton rolls are placed on buccal and lingual side of prepared tooth.
High Vacuum Suction
• Used for fluid and saliva removal
• Powerful suction equipment used with an assistant
• May also used to retract lip simultaneously
Saliva Ejector
• Low vacuum suction
• Maxillary arch → corner of mouth (opp. To prep side) → during tooth preparation
• Not as effective as high vacuum suction
https://t.me/DentalBooksWorld
1194 Triumph's Complete Review of Dentistry

Svedopter: mandibular arch


• Flange type of saliva ejector made of metal
• For fluid removal and tongue retraction
• Used with patient in a upright position, without any resistance

Anti-sialogogues
Used to provide fluid control by reducing salivary flow
Example:
Anticholinergic drugs → atropine, dicyclomine, propantheline
Drug Dose
Atropine sulfate 0.4 mg
Dicyclomine 10–20 mg
Propantheline bromide 7.5–15 mg
Clonidine 0.2 mg

GINGIVAL DISPLACEMENT
Deflection of marginal gingival away from tooth.
Also called gingival retraction or tissue dilation.
Indicated to provide adequate reproduction of finish lines and also to accurately duplicate subgingival margins.
By providing the best possible condition for impression material, fluid control.
Methods:
1. Mechanical method
2. Mechanical–chemical methods
3. Chemicals

Mechanical Method
Physically displaces the gingiva
1. Copper band
• Carries impression material and displaces gingiva
• Impression compounds and elastomeric materials have been used
• Chemico-mechanical method of gingival retraction (retraction cord)
• It is a method of combining a chemical with pressure packing
2. Rubber dam
• Used when limited number of teeth in one quadrant are being restored
• When preparations do not have to extend sub-gingivally
• Clamp should be blocked out
Note: Addition silicone should be avoided as rubber interferes with its settings.
3. Cotton threads
Plain cotton threads → retraction achieved is physical, without any hemostasis
4. Magic foam
• Consists of “Comprecap” → A hollow cotton
• “Magic foamcord” → polyvinyl siloxane material

https://t.me/DentalBooksWorld
Chapter 14 • Prosthodontics 1195

5. Plain retraction cord


• Causes mechanical opening of the gingival sulcus
• Minimum cord diameter needed for retraction is at least 0.2 mm
• Braided cords are packed using instruments with serrated circular heads
• Twisted cords are packed with smooth, nonserrated circular head instruments
• Time of placement is at least 4 minutes before making the impression

Mechanical–Chemical
• Displacement/retraction cord used for mechanically separating tissue from peripheral margin
• Impregnated with chemical for astringent action and/or hemostasis as impressions are made
• Cord displaces the gingival tissues both laterally and vertically

Chemicals:
1. Ferric sulfate → 13–20%
• Recommended packing time is 1–3 minutes
• Solutions of ferric sulfate above 15% are very acidic and can cause significant tissue irritation and postoperative root
sensitivity
2. Aluminum chloride → 5–25%
• It is one of the most commonly used astringents
• Time of application is 10 minutes
• Advantages include least irritating for gingival tissues and has no systemic effects
3. Vasoconstrictors
• Racemic epinephrine (0.1%, 8%)
• This has systemic effects and may lead to epinephrine syndrome
• There is a risk of inflammation of gingival cuff and rebound hyperemia

QUICK REVIEW OF IMPLANTS


• A notch at the apex of implant is an anti-rotational feature.
• Abutment is fixed to implant with the help of abutment screw.
• Healing abutment forms tissues around the implant known as permucosal attachment.
• It is also known as gingival former or healing cap and is placed during second stage of surgery.

Measurements in Implant Dentistry


Normal biological width 2.04–2.91 mm
Implant biological width 3.08 mm
Normal probing depth 3 mm (metal probe)
Implant probing depth 2.5–5 mm (plastic probe)
Minimum Distance Required
Buccal plate 0.5 mm
Lingual plate 1 mm
Maxillary sinus 1 mm
Nasal cavity 1 mm
Interimplant distance 3 mm from the outer edge
Distance from inferior alveolar canal 2 mm

https://t.me/DentalBooksWorld
1196 Triumph's Complete Review of Dentistry

Distance from mental nerve 5 mm


Distance from inferior border of mandible 1 mm
Distance from adjacent natural tooth 1.5 mm
Bone Values
Minimum implant height 9 mm
Minimum bone height 10 mm
Minimum implant bone width (anterior) 3.2 mm
Minimum implant bone height (posterior) 3.5 mm (up to 6 mm)
Implant Supported Over Dentures
No. of implants in maxilla 4–6 implants
No. of implants in mandible 2 implants

IMPLANT SYNOPSIS
An implant can be defined as “A graft or insert set firmly or deeply into or onto the alveolar process that may be prepared for
its insertion.”
Dahlse (1940) Endosteal implants
Linkow Blade type implants
Per Ingvar Branemark (1980) Osseointegration
Weiss Fibro-osseous integration

Classification of Implants
1. Depending on the placement within the tissues

1. Epiosteal implants This is a type of dental implant that receives its primary bone
support by resting on it.
E.g., Subperiosteal implant.
2. Transosteal implants This type of implant that penetrates both cortical plates and passes
through the entire thickness of the alveolar bone.
3. Endosteal implants It is the dental implant that extends into the basal bone for support.
It transects only one cortical plate.
It can be further classified into:
1. Root form implant
2. Plate form implants
a. Root form implant They are used over a vertical column of bone.
b. Plate form implants They are used for horizontal column of bone.

2. Depending on their reaction with bone


a. Bioactive (hydroxyapatite)
b. Bioinert (metals)
3. Depending on the treatment options

FP-1 Fixed prosthesis; replaces only the crown; looks like a natural tooth
FP-2 Fixed prosthesis; replaces the crown and a portion of the root; crown contour appears normal in the occlusal half
but is elongated or hypercontoured in the gingival half
FP-3 Fixed prosthesis; replaces missing crowns and gingival color and portion of the edentulous site; prosthesis most
often uses denture teeth and acrylic gingiva, but may be made of porcelain or metal

https://t.me/DentalBooksWorld
Chapter 14 • Prosthodontics 1197

RP-4 Removable prosthesis; overdenture supported completely by implant


RP-5 Removable prosthesis; overdenture supported by both soft tissue and implant

Classification of Bone Depending Upon the Density


D1 bone: • Dense compact bone
• Found in the anterior regions of mandible and in the lateral aspects of symphysis of the mandible
• Provides excellent stability to titanium implants
D2 bone: • Dense to thick porous compact and coarse trabecular bone
• Commonly found in the anterior portion of the mandible followed by the posterior mandible
• It provides excellent ridge healing
• Threaded titanium implants and titanium plasma coated press fit cylinder implants are preferred
for such bone
D3 bone: • Porous compact and file trabecular bone
• Found in the anterior or posterior maxilla and posterior regions of the mandible
• This bone is very easy to cut during the surgical procedures
• Implants coated with hydroxyapatite are indicated to increase the bone contact
D4 bone: • Fine trabecular bone
• Found in posterior maxilla in a long-term edentulous patient
• It is the most difficult bone to obtain rigid fixation

Healing Period of Implants


Mandible
• 4 months (posterior mandible)
• 3 months (anterior region)
Maxilla
• 6 months in the anterior and posterior region
Bone graft
• 6–9 months

Materials Used in Dental Implants


1. Metals
• Stainless steel
• Cobalt–Chromium–Molybdenum alloys
• Titanium and its alloys
• Surface coated titanium
• Gold
• Tantalum
2. Ceramics
• Hydroxyapatite
• Bioglass
• Aluminum oxide
3. Polymers and Composites
4. Others
• Carbons

https://t.me/DentalBooksWorld
1198 Triumph's Complete Review of Dentistry

QUICK FACTS

• A widened periodontal ligament space may indicate premature occlusal contact and is often associated with tooth mobility.
• FPD is contraindicated in patients with high smile line because a fixed prosthesis replaces only the missing tooth structure
and not the supporting tissues.
• If the molar tooth is extruded more than 2.5 mm, extraction is to be considered.
• In case of a missing canine, where a cantilever FPD is planned, support should be taken from both central and lateral incisor.
• Complete veneer crown acts as a closed hydraulic chamber preventing easy escape of cement.
• Porcelain jacket crown is contraindicated in cases where there is minimum overjet.
• Porcelain have a good compressive strength of 173 Mpa but a low diametrical tensile strength and hence fracture under tensile stress.
• Porcelain is not ductile; they are considered to be brittle.
Basic facts of crown preparation:
• Thickness of metal collar for porcelain-fused-to-metal crowns – 0.3 mm
• Tooth reduction on non-centric cusps on a cast metal crown is 1 mm
• Tooth reduction on a centric cusps for a cast metal crown is 1.5 mm
• Average thickness of opaque porcelain in a metal-ceramic crown is 0.1 mm
• Recommended angle of convergence for maximum parallelism is 5–10 degrees
• Recommended facial reduction for an anterior PFM crown (base metal) is 1.5 mm
• Recommended facial reduction for an anterior PFM crown (noble metal) is 1.7 – 2 mm
• Recommended incisal reduction for an anterior PFM crown (base metal) is 2 mm.
• Metal coping for an anterior metal-ceramic crown is 0.3 – 0.5 mm.

REMOVABLE PARTIAL DENTURE

SYNOPSIS

Any prosthesis that replaces some teeth in a partially dentate arch, which can be removed from mouth and replaced at will.

REMOVABLE PARTIAL DENTURE TYPES


1. Acrylic partial dentures
• Made of acrylic resins with clasps of wrought wire
• Commonly referred to as flippers
• Entirely tissue supported and cause gingival recession with long-term use
• So, commonly termed as “gum strippers”
• Should be used only as temporary dentures
2. Cast partial dentures
• Clasp retained
• Attachment retained partial dentures
Temporary removable partial denture
• Used where tissue changes are imminent
• May be of three types
1. Interim denture
• Used to enhance esthetics and/function for a limited period of time
• Which is to be definitive for dental prosthesis

https://t.me/DentalBooksWorld
Chapter 14 • Prosthodontics 1199

2. Transitional denture
• Nothing but a serving interim prosthesis to which artificial teeth will be added as natural teeth are lost
• Only to get replaced after postextraction tissue changes have occurred
3. Treatment denture
• Prosthesis used for purpose of treating or conditioning the soft tissues, which have been abused by ill-fitting of soft tissues

Sequelae of Partial Edentulism


Few consequences of partial teeth loss are
1. Esthetics
2. Speech
3. Drifting and tilting
4. Supraerupted teeth
5. Overloading of remaining teeth
6. Loss of masticatory efficiency
7. Loss of vertical dimension
8. Mandibular deviation
9. Loss of alveolar bone
10. Combination syndrome

Supraerupted Tooth
Defect in occlusal plane → when posterior teeth are lost
Affect esthetics → when anterior teeth are lost
Classification of supraerupted tooth
Class 1
• Pose no appreciable problems in positioning teeth in opposing arch
• No potential for creating occlusal trauma
• And so, no treatment needed
Class 2
• Supraerupted tooth poses a definite problem
• But can be successfully managed by enameloplasty to reduce height of crown, incisal edge, and cusp tips
Class 3
• Poses moderately severe problems
• Can be successfully managed by altering teeth to such a degree that enamel is penetrated
• Teeth require a cast restoration
Class 4
• Severely extruded
• Extraction is permissible if tooth is nonessential or useless to success of RPD
• Class 4 (E) → endodontic therapy → for support
• Class 4 (O)→ surgical orthodontics → retention and bracing

Indications for Removable Partial Denture


Extraoral factors Intraoral factors
Age Distal extension situations
Sex Long edentulous span
General health Reduced periodontal support
Occupation and economic status Cross arch stabilization

https://t.me/DentalBooksWorld
1200 Triumph's Complete Review of Dentistry

Patient desires and attitude Excessive residual ridge resorption


Time factor Esthetics
Immediate replacement of teeth
Obturation of intraoral defects
Alteration of vertical dimensions
Poor prognosis for complete denture

CLASSIFICATION OF REMOVABLE PARTIAL DENTURE (RPD)


Cummer’s Classification:
• This was the first professionally recognized classification (Cummer in 1920).
• According to him, partial dentures can be classified into four types based on the position of direct retainers.
He cited the various possible saddle positions in his paper, “A little excursion in partial denture mathematics.”
Diagonal Two direct retainers are diagonally opposite to one another.
Diametric Two direct retainers are diametrically opposite to one another.
Unilateral Two or more direct retainers present on the same side.
Multilateral Three direct retainers in a triangular relationship.

Kennedys Classification (Dr. Edward Kennedy in 1923):


Class I
Bilateral edentulous area located posterior to remaining natural teeth.
Class II
Unilateral edentulous areas located posterior to the remaining natural teeth.
Class III
Unilateral edentulous area with natural teeth both anterior and posterior to it.
Class IV
Single, bilateral edentulous area located anterior to remaining natural teeth.
Sequence of Kennedy’s classification partly based on frequency of occurrence:
Class I – most common
Class IV – least common
Class III – fully tooth supported prosthesis
Class I, Class II, and long-span Class IV – Tooth and tissue supported prosthesis
Based on Applegate – Kennedy classification:
Class V
Edentulous area bounded anteriorly and posteriorly by natural teeth but in which anterior abutment is not suitable for support
Class VI
Edentulous area in which teeth adjacent to space are capable of total support of required of prosthesis. This denture does not
require any tissue support

Applegate’s Rules
Rule 1 Classification should follow rather than precede extractions that might alter the original classification.
Rule 2 If the third molar is missing and not to be replaced, it is not considered in the classification.
Rule 3 If the third molar is present and is to be used as an abutment, it is considered in the classification.
Rule 4 If the second molar is missing and is not to be replaced, it is not considered in the classification.
Rule 5 The most posterior edentulous area or areas always determine the classification.

https://t.me/DentalBooksWorld
Chapter 14 • Prosthodontics 1201

Rule 6 Edentulous areas other than those, which determine the classification are referred to as modification spaces
and are designated by their number.
Rule 7 The extent of the modification is not considered, only the number of additional edentulous areas are
considered.
Rule 8 There can be no modification areas in class IV. Because any additional edentulous space will definitely be
posterior to it and will determine the classification.

Bailyn’s Classification
• Proposed by Bailyn, it was the first classification to give importance to support of partial dentures by remaining tissues.
He used descriptive letters like A and P.
A Anterior restorations, where there are saddle areas anterior to the first bicuspid (premolar)
P Posterior restoration, where there are saddle areas posterior to the canine
Further, they are subclassified as follows:
Class I Bounded saddle (not more than three teeth missing)
Tooth supported
Class II Free end saddle (there is no distal abutment tooth)
Tooth-tissue supported
Class III Bounded saddle (more than three teeth missing)
Tooth-tissue supported

Godfrey’s Classification
• Proposed in 1951, it is based on the location and size of edentulous spaces.
The specialty of this classification is that the main classes have no modifications.
Class A Tooth-borne denture base in the anterior part of the mouth.
Class B Mucosa-borne denture base in the anterior region.
Class C Tooth-borne denture base in the posterior part of the mouth.
Class D Mucosa-borne denture base in the posterior region.

Friedman’s Classification
• He introduced ABC classification in 1953.
A Anterior
B Bounded posterior
C Cantilever

SURVEYING
It is the first step in the design of a RPD.
• It is defined as, “An analysis and comparison of the prominence of intraoral contours associated with the fabrication of
prosthesis.”
Surveyor
• It is an instrument used in the construction of a removable partial denture to locate and delineate the contours and relative
positions of abutment teeth and associated structures.
• It is a parallelometer, an instrument used to determine the relative parallelism of surfaces or other areas in a cast.
• A. J. Fortunati was the first person to use a surveyor.

https://t.me/DentalBooksWorld
1202 Triumph's Complete Review of Dentistry

Objectives of Surveying
• To design an RPD such that it is rigid and flexible components are approximately positioned to obtain good retention and
bracing.
• To determine the path of insertion of a prosthesis such that there is no interference to insertion along its path.
• To mark the height of contour of the area (hard or soft tissues) above the undercut.
• To make survey lines (height of contour of a tooth).
• To mark the undesirable undercuts into which the prosthesis should not extend.

Parts of a Surveyor
1. Level platform
• On which the cast holder is moved
2. Vertical arm
• That supports the super structure
3. Horizontal arm
• In Ney’s surveyor it is fixed, whereas in Will’s surveyor it may revolve around the vertical column
4. Surveying arm
• It is spring loaded in Will’s surveyor and in Ney instrument the arm is completely passive
5. Cast holder
• On which the cast to be studied is attached
6. Analyzing rod
• Paralleling tool, used to locate undercuts
• It is the first tool to be used during surveying
• This tool does not mark anything on the cast
• Used to determine the parallelism of the tooth surfaces before making the survey line
7. Carbon marker
• Used to scribe the height of contour
8. Undercut gauges
• Used to identify the amount and location of desired retentive undercut
9. Wax knife
• Used to eliminate or block out areas of undesirable contours with wax on the cast

Survey Lines
• Survey lines are nothing but the height of contour of the abutment teeth marked by a carbon marker during surveying.
• The height of contour can be defined as “A line encircling a tooth designating its greatest circumference at a selected
position.”
Blatterfein divided the buccal and lingual surfaces of the tooth adjacent to the edentulous spaces into two halves by the line
passing through the center of these surfaces along the vertical axis of the tooth.
• The area closer to the edentulous space is known as the near zone and the other, that lies away from the edentulous space
is called the far zone.
• In the other words, the proximal surface of the tooth near the edentulous space is called the near zone and the proximal
surface of the tooth away from the edentulous space is called the far zone.

Types of Survey Lines


Survey line Pattern of line Clasp used
High survey line • This line passes from the occlusal third in the Wrought wire clasp, which is more flexible
near zone to the occlusal third in the far zone. should be used.
• When high survey line is present, undercut
will be deep.
https://t.me/DentalBooksWorld
Chapter 14 • Prosthodontics 1203

Medium survey line • The line passes from the occlusal third in the Aker’s or Roach Clasp is used.
near zone to the middle third in the far zone.
• During the survey, the cast should be tilted
such that maximum number of teeth have a
medium survey line.
Low survey line • This line is closer to the cervical third of the Modified T clasp is used.
tooth in both near and far zone.
Diagonal survey line • This survey line runs from the occlusal third Reverse circle clasp is used.
of the near zone to the cervical third of the
far zone.
• This is more common on the buccal surfaces
of canines and premolars.

Uses of a Surveyor
• Surveying the diagnostic and primary casts
• Tripoding the cast (recording the cast position)
• Transferring the tripod marks to another cast
• Surveying the master cast
• Contouring casts and crowns
• Surveying the master cast
• Placing internal attachment and rest

Tripoding the Primary Cast


• Recording the special orientation of the cast is done by a procedure called tripoding.
• Tripoding is a very simple procedure, wherein three different widely spaced out points of a single plane are marked on the cast.
• These tripod points are reference points and should not be altered till the end of the treatment.
• The commonly used additional reference points are:
–– Distal marginal ridge of first premolar
–– Incisal edge of lateral incisor
–– Lingual cusp tip of first premolar on the opposite side
• After surveying the diagnostic cast, the mouth preparation is done. After completion of mouth preparation, the master cast
is made.
• After surveying the cast in horizontal direction, the main purpose of tilting the cast is to determine the favorable path of
insertion.
• The most favorable path of insertion of a RPD is one that is perpendicular to occlusal plane.

Types of Surveyors
The surveyors commonly used are:
• Ney surveyor (widely used)
• Jelenko’s or Will’s surveyor
• William’s surveyor
Features Ney surveyor Will’s surveyor (Jelenko) William’s surveyor
Horizontal arm Horizontal arm is fixed The horizontal arm swivels Have revolving horizontal
horizontally around the arm with a joint in the
vertical column middle.
Surveying arm Surveying arm is completely Surveying arm is spring Spring-mounted surveying
passive and is positioned by a loaded arm that could be locked at
locking device any position

https://t.me/DentalBooksWorld
1204 Triumph's Complete Review of Dentistry

Level platform/surveying Universal table Universal table They have a gimbal stage
table table and are used to place
precision attachments
Vertical arm Retained by friction within a Spring mounted and returns
fixed bearing to top position when it is
released
Carbon marker Circular Triangular -
Undercut gauge Circular beaded A fan-shaped bead with each -
wing of the fan measuring
different dimensions

Color code index marking on master cast


Red Rest seats indicated by solid red lines.
Areas that require recontouring are outlined in red and filled with evenly spaced
diagonal lines.
Black (common lead pencil) Used to place survey lines, tissue undercut areas.
Blue Acrylic portion of RPD
Brown Metal framework of RPD

DETERMINING THE PATH OF INSERTION AND GUIDING PLANES


• After surveying, the next step in the treatment plan of partial dentures is determining the path of insertion and guiding plane.
• Path of insertion is defined as, “the direction in which a prosthesis is placed upon and removed from the abutment teeth.”

Factors determining the path of insertion and removal of a removable partial denture are
1. Guiding planes
2. Retentive undercuts
3. Interferences
4. Esthetics
5. Denture base
6. Location of vertical minor connector
7. Point of origin of approach arm

Guiding Plane
• These are two or more parallel vertical surfaces of abutment teeth shaped to guide the prosthesis during placement and
removal without causing undesirable forces against the teeth.
• The path of insertion will always be parallel to guide planes.
• Guiding planes are prepared on the proximal and axial surfaces of primary and secondary abutment teeth.
• The surface of the minor connector that contacts the secondary abutment is known as the proximal plate of the minor
connector.
• The proximal plate on the minor connector should and will contact the guide planes during insertion. It is the only part of
RPD that contacts the proximal plates.
• The guide planes are prepared in enamel surface or in wax patterns for cast restorations.

Structure of a Guide Plane


• Guiding planes are usually 2–3 mm in occlusogingival height parallel to the path of insertion.
• Guide planes do not naturally occur on the teeth; instead it should be prepared by the clinician during prosthetic mouth
preparation.
• The guide plane should be flat and does not contain any undercuts.
https://t.me/DentalBooksWorld
Chapter 14 • Prosthodontics 1205

• As a thumb rule, proximal guiding plane surfaces should be about two-thirds buccolingual width between the buccal and
lingual cusps and about two-thirds the length of the enamel crown portion from the marginal ridge cervically.
• For distal extension (DE) dentures, a guiding plane should involve, approximately one-third the buccolingual width of tooth
and two-thirds of vertical length of enamel crown.

Functions of a Guiding Plane


• It minimizes the wedging stresses on the abutments.
• Makes insertion and removal of the prosthesis easier.
• Aids in stabilization of prosthesis against horizontal forces.
• Reduces the block-out area, contributes to indirect retention, and frictional forces.

Retentive Undercuts
• For a clasp to be retentive, its path of escapement must be other than parallel to the path of removal of the denture itself.
• With the analyzing rod being attached to the vertical arm, each abutment tooth is examined for the presence of retentive
undercuts.
• It is a rule that, retentive undercuts must be present on the abutment teeth at the horizontal tilt.
• If the retentive undercut is not present, it must be created by the use of a full crown or the enamel surfaces may be contoured
to improve the retentive undercuts.
• Ideally, the proposed abutment tooth should have 0.010 inch undercut at the most desired location on either mesiobuccal
or distobuccal line angle and in the gingival third of the clinical crown.
• A 0.010 inch undercut is desired when cast chrome alloy is used for the frame work.
• A 0.020 inch undercut is needed for wrought wire combination clasp because of the greater flexibility of wrought alloys.

Interferences
In maxilla
• Torus palatinus
• Bony exostoses
• Buccally tipped teeth
In mandible
• Lingual tori
• Lingual inclination of remaining teeth
• Bony exostoses

Parts of a Removable Denture Prosthesis


The various components of a removable partial denture are
• Major connector
• Minor connector
• Rest
• Direct retainer
• Indirect retainer
• Denture base
• Artificial tooth

MAJOR CONNECTOR
• “A part of a removable partial denture which connects the components on one side of the arch to the components on the opposite
side of the arch”
• It is the most important and largest component of a removable denture and also helps in indirect retention.

https://t.me/DentalBooksWorld
1206 Triumph's Complete Review of Dentistry

Types of Major Connectors


Maxillary major connectors
• Palatal bar
• Palatal strap
• Anteroposterior or double palatal bar
• Horseshoe-shaped or U-shaped connector
• Closed horseshoe or anteroposterior palatal strap
• Complete palate

Type Shape Advantages Disadvantages Indications


Palatal bar Narrow half oval Minimalism in design, Bulky Interim partial denture
shape. effortlessness in fabrication Little vertical Rarely in Kennedy class
Width less than 8 mm. support III
Poor bony support
from the palate
Palatal strap Wide strap Good rigidity and Tissue reactions Bilateral short span tooth
Less bulky strength. supported prosthesis
Distribute stresses (Kennedy class III strong
Improved retention abutments)
Unilateral DE cases
Anteroposterior Anterior bar is Relieved for bony exostosis Less palatal support When anterior and
palatal bar narrower. Stiffness Discomfort posterior abutment teeth
(Double palatal Posterior bar is half- Decreased bulk of are widely separated.
Bar) oval. metal Cases with inoperable
Most rigid palatal tori.
major connector. Class IV cases.
Anteroposterior Anterior and posterior Structurally, it is the most Distress in speech Indicated in almost any
palatal strap or palatal straps rigid palatal connector. Tissue reactions partial denture design.
closed horse-shoe connected by two Additional effect due to Kennedy class IV
lateral bars. L-beam effect.
The border should be Good palatal support.
6 mm away from the The central opening
margin. provides taste sensation.
Horse-shoe- Thin band extending Beneficial in restoring More lateral forces Used when many anterior
shaped palatal in the anterior and anterior teeth produced. teeth to be replaced.
bar or U-shaped posterior lateral Least desirable In cases of inoperable tori
bar surfaces of palate. major connector. extending to posterior
Anteriorly it is a thin It lacks rigidity. limit of hard palate.
plate that covers the Cannot be used for
cingula of the teeth. DE cases.
Complete palate Uniform metal Rigid simple design Tissue reactions First choice of connector
extending over entire Good support Posterior palatal in distal extension cases.
hard palate. Greatest retention seal cannot be For patients with well-
The posterior border altered developed muscles of
extends to the junction Weight of the mastication.
of hard and soft palate. prosthesis may
dislodge prosthesis

https://t.me/DentalBooksWorld
Chapter 14 • Prosthodontics 1207

Common Indications of Maxillary Major Connector


Condition Major connector design
Periodontal support of the remaining teeth is weak and more • Complete palate
palate should be covered • Wide palatal strap
For long-span distal extension bases • Complete palate
• Closed horse-shoe or A-P palatal strap
If a torus is present and must not be removed • U or horse shoe
• Closed horse shoe
• A-P palatal bar
When several anterior teeth are to be replaced • Horse shoe
• Closed Horse shoe
• Complete palate

Mandibular Major Connectors


Types Shape Advantages Disadvantages Indications Contraindications
Lingual bar Half pear shaped bar Ease of design Food impaction Used in all Lack of sufficient
with superior and and construction Patient discomfort situations where space lingually,
inferior border Less tissue a minimum of Tori
coverage 8 mm space
Decreased required in
decalcification lingual anterior
and tissue region
reactions
Lingual plate Pear shaped bar with Indirect retention Decalcification of All possible Condition where
superior and inferior Good rigidity, tooth. situations lingual bar is used.
border. support and Soft tissue reactions
Superior border stabilization
extends as thin plate
to lingual surface of
anterior teeth
Double Superior and inferior Stabilization to Complex design Crowded lower Ideal situations
lingual bar bar connected by lateral horizontal forces. Food entrapment anterior teeth
or Kennedy’s bars Indirect retention Tongue annoyance Diastemas in
continuous lower anteriors
bar Compromised
Periodontal
support
Labial bar Half pear shaped bar in Labial support Distorts lip support Used when teeth Last choice of
labial or buccal surface Patient discomfort are lingually connector
Modification of labial Difficulty in placed. Tents to distort lip.
bar (Swing lock) a fabrication Inoperable Poor esthetics.
hinge is placed at one mandibular tori.
end and lock at the
other end for closer
adaptation.
Sublingual bar Kidney shaped Less obstructive Sublingual Similar to lingual Similar to lingual
extending sublingually than lingual bar extension is critical bar bar
Increased rigidity
than lingual bar

https://t.me/DentalBooksWorld
1208 Triumph's Complete Review of Dentistry

Common Indications for Mandibular Major Connectors


Condition Major connector indicated
Routinely indicated or first preference major connector • Lingual bar
for tooth supported RPD
High lingual frenum, active tissues of the floor of the • Lingual plate
mouth
Long span edentulous ridges, Class I or II design RPD • Lingual plate
and indirect retention is needed
Anterior teeth having reduced periodontal support and • Lingual plate
need stabilization • Double lingual bar
Large interproximal spaces that could cause esthetic • Double lingual bar
problems by the metal display of a lingual plate
Extreme lingual inclination of premolars and anterior • Labial bar
teeth, inoperable tori

MINOR CONNECTOR
“The connecting link between the major connector or base of a removable partial denture and other units of the prosthesis, such as
clasps, indirect retainers, and occlusal rests.”
• It is also a rigid part of RPD.
• It should form a right angle with the major connector so that the gingival crossing is abrupt and cover little gingival tissues.
• Minor connector is the only part of RPD that contacts the guiding plane of abutment.
Functions of a Minor Connector
• It connects the major connector to other parts like clasps, rests, indirect retainers, and denture bases.
• It transmits stresses evenly to all components so that there is no concentration of load at any single point.
• It transmits the forces acting on the prosthesis to the edentulous ridge and the remaining natural teeth.

Types of Minor Connector


1. Joining the clasp assembly to major connectors
2. Joining the indirect retainer or auxiliary rest to the major connector
3. Joining the denture base to the major connector
4. Approach arm in bar type clasp

Minor Connector
Clasp–assembly minor connector
• Must be rigid with sufficient bulk
• But the bulk, must be concealed
• If located in the proximal surface of teeth adjacent to edentulous area, it should be broad buccolingually and thin
mesiodistally
• Thickest portion should be at lingual line angle of the tooth and should taper evenly to its thinnest portion at buccal line
angle area
• If clasp assembly not placed adjacent to edentulous area, minor connector is placed in the embrasure between two teeth
• Minor connector should never be placed on the convex lingual surface of tooth
Indirect retainer or auxiliary rest minor connector
• Designed to lie in embrasure
• Connects indirect retainer and auxiliary rest to major connector
• Should form a right angle with major connector
• Provided its junction to be gentle curve

https://t.me/DentalBooksWorld
Chapter 14 • Prosthodontics 1209

Denture base minor connector


• Joins the denture base to major connector
• Must possess the sufficient strength and rigidity to anchor the denture base securely and must not interfere with tooth
arrangement
• In maxillary distal extension bases → must extend to cover the tuberosity (ladder or loop design)
• In mandibular distal extension bases → should extend two-thirds the length of residual ridge (open lattice or ladder
type design)
Types are:
1. Lattice work construction
2. Mesh construction
3. Bead wire or nail head construction
Approach arm minor connector
• Serves as an approach arm for bar clasp → vertical projection clasp
• Supports the direct retainer
• Engages the undercut from gingival margin
• Only minor connector not required to be rigid
• Should have a smooth even taper from start to finish
• Must not cross a soft tissue undercut

Rest and Rest Seats


• Rest is the unit of RPD that provides vertical support.
• This component is designed in such a way that the transmitted forces are directed along the long axis of the supporting
tooth.
• In all the tooth supported denture, the rests are capable of transferring 100% of occlusal stresses to the abutment teeth.
• Rests act as a vertical stop and prevents impingement of soft tissues.
• The rest that is a component part of a direct retainer is referred as “Primary rest.”
• Additional rests that are used for indirect retention are known as “auxiliary rest or secondary rest.”
• One of the principle is that rests should be placed on the proximal surfaces of all the teeth adjacent to edentulous spaces, in
order to prevent food impaction between minor connector and the tooth.
• Strategically positioned minor connector and the rest, together can act as the reciprocal clasp arm.
Types of Rests
1. Occlusal rest: It is placed on the occlusal surfaces of posterior teeth.
2. Lingual or cingulum rest: It is placed on the lingual surface of a tooth, mostly the maxillary canine.
3. Incisal rest: Placed on the incisal edges of the teeth, usually a mandibular canine.

Features of Rests and Rest Seats


Outline form of the rest seat Triangular-shaped base on the marginal ridge and rounded apex
toward the center of the tooth
Size of the occlusal rest One-half of the buccolingual width of tooth from cusp tip to cusp tip
and one-third to one-half of the mesiodistal width
Angle between the floor of rest seat and long axis of It should be less than 90 degrees
the abutment
Floor of the rest in posterior teeth Spoon- or saucer-shaped or concave shape with smooth gentle curves.
Floor of the rest on cast restorations Box-shaped
Outline form of the lingual rest seat V-shaped or half-moon shaped
Burs used to prepare occlusal rest on sound amalgam No. 6 and No. 8 round burs
Bur used to prepare lingual and incisal rests One-fourth inch bur
Common tooth preferred for lingual rest Maxillary canine

https://t.me/DentalBooksWorld
1210 Triumph's Complete Review of Dentistry

Common tooth preferred for incisal rest Mandibular canine


Measurements for lingual rests M-D length: 2.5–3 mm
B-L width: 2 mm
Depth: 1.5 mm
Measurements for incisal rests Width: 2.5 mm
Depth: 1.5 mm

Direct Retainers
• Direct retainer is the component of removal prosthesis that engages an abutment tooth in such a manner as to resist dislodgment
of the prosthesis.
• Direct retainer possess the characteristics of vertical support, retention, and stability.
Upper part of the abutment Support
Middle third of the abutment Stability
Gingival third of the abutment Retention
Clasp Assembly Components
• One or more minor connectors
• Principle rest
• Retentive arm or holding arm
• Reciprocal arm or bracing arm

Types of Direct Retainers


1. Intracoronal (Internal attachment/Precision attachment)
2. Extracoronal (Clasp retainer)
• Intracoronal retainer should be indicated in tooth-supported denture bases only.
• It should not be used with tissue supported or DE denture cases unless some forms of stress breaker is used.

Extracoronal (or) Clasp Direct Retainer


• It is the most frequently used direct retainer.
• Prothero’s cone theory is the basis of clasp retention.
• Requirement of clasp assembly includes
–– Retention
–– Support
–– Stability
–– Reciprocation
–– Encirclement
–– Passivity
Retention of a Clasp
• The retentive clasp arm provides retention.
• The terminal third of the retentive arm is flexible and engages the undercut area.
• The proximal third of the retentive arm or the shoulder is rigid and is positioned above the height of contour.
• Factors affecting the retention are
–– The amount of clasp arm that extends below the height of contour
–– The depth of the retentive terminal that extends into the undercut
Support
• The most important property regarding the health of oral tissues is support
• It is the property of a clasp that resists displacement of the clasp in a gingival direction
• The prime support units of a clasp are rests

https://t.me/DentalBooksWorld
Chapter 14 • Prosthodontics 1211

Stability
• Stability is the ability to resist horizontal displacement of the prosthesis.
• Except the retentive clasp terminal, every other components of the clasp contribute to the stability.
• Cast circumferential clasp provides greater stability due to rigid shoulder.
• On the other hand, wrought wire clasp has flexible shoulder and the bar clasp lacks shoulder and hence both offers lesser
stability.
• All the three types of clasps have rigid reciprocal or bracing arm which provide equal stability.
Reciprocation
• Reciprocation is provided by the reciprocal arm or the bracing arm.
• It is positioned on the opposite side of the tooth from the retentive arm at the junction of the gingival and middle thirds of
the abutment tooth.
• This resists the forces exerted by the retentive arm during placement and removal of the denture.
Encirclement
• Clasp should be designed to encircle more than 180 degrees of the abutment tooth.
• It may be continuous as in case of circumferential clasp or may be broken as in case of bar clasps.
Passivity
• A clasp should be completely passive in nature.
• The retentive function is activated only when a dislodging force is applied to the denture.

Component Function Location


Rest Support Occlusal, lingual, and incisal
Minor connector Stability From the marginal ridge to the junction of the middle and gingival third of
the proximal surface of abutment crown.
Clasp arms Stability Apical portion of middle third of crown
Reciprocation Apical portion of middle third of crown
Retention Gingival third of crown in measured undercut.

Types of Circumferential Clasp


1. Simple circlet clasp
• It is the most versatile and widely used clasp.
• It is most often the clasp of choice for tooth supported RPD.
• The clasp usually approaches the undercut on the abutment tooth from the edentulous area and engages the undercut
remote from the dentulous space.
2. Ring clasp
• Encircles nearly all of the tooth from its point of origin
• Indicated in unsupported tipped molars
• It is difficult to repair
• Back action clasp is the modification of ring clasp
3. Embrasure clasp or modified crib clasp
• Indicated on the side of the arch where there is no edentulous space like unmodified class II or class III partial denture.
• It is basically two simple circlet clasps joined at the body. It should always be used with double occlusal rest.
• The tooth structure must be removed on the buccal inclines only.
4. Multiple clasp
• It is essentially two simple circlet clasps joined at the terminal end of the two reciprocal arms.
• It is used when additional retention is needed when the principle abutment tooth has lost its periodontal support.
5. Half and half clasp
• It consists of circumferential retentive arm arising in one direction and a reciprocal arm arising from another direction.

https://t.me/DentalBooksWorld
1212 Triumph's Complete Review of Dentistry

6. Reverse or reverse approach circlet clasp


• It is used when the retentive undercut is located on the adjacent surface (distobuccal) of the edentulous space.
• It is exact opposite of simple circlet clasp.
• Simple circlet clasp engages mesial undercut on the abutment tooth and reverse circlet clasp engages on the distal
undercut.
7. Reverse action or C or fish-hook or hairpin clasp
• It is simple circlet clasp in which the retentive arm after crossing the facial surface of the tooth from its point of origin
loops back in a hairpin turn to engage an undercut below its point of origin.
• It is used only when bar type retentive arm is contraindicated.
8. Combination clasps
• It is an extended occlusal rest with buccal and lingual clasp arms.
• It consists of a wrought round wire retentive clasp arm and a cast reciprocal arm.
• The wrought wire retentive arm is a circumferential clasp arm.
• It is indicated when retentive undercut is present on the mesiobuccal side of abutment tooth adjacent to a DE space.
• It is usually placed in 0.020 inch undercut.

Types of Bar Clasps


1. T-clasp
• The retentive terminal and its opposing encircling finger project laterally from approach arm to form a T.
• The retentive terminal must cross under the height of contour to engage retentive undercut while other finger of T stays
on the suprabulge area of the tooth.
• It is most frequently indicated to engage a distobuccal undercut adjacent to distal extension space.
• Modified T-clasp has increased esthetics but decreased encirclement of abutment.
2. I-bar
• The retentive I bar is normally near the center of the facial surface of tooth.
• In I clasp, the retentive arm is placed on the distobuccal surface of maxillary canine.

Circumferential clasp Bar clasp


Also known as Akers’ clasp Also called Vertical projection or Roach Clasp
It approaches the undercut from above the height of contour It approaches the undercut from below the height of contour
Offers pull type of retention Offers push type of retention
Continuous type of encirclement Broken type of encirclement
Does not permit functional prosthesis movement Permits physiological tooth movement and functional
prosthesis movement
Less esthetic but has more stability and bracing quality More esthetic but has lesser stability and bracing quality
A cast circumferential clasp arm can be used with any type Bar clasp arm can be used with tooth borne partial dentures
of base except to engage a mesiobuccal undercut adjacent to and when retentive under cut is present adjacent (distobuccal)
distal extension space to the distal extension space

Indirect Retainers
• In case of partial dentures not supported by natural teeth at each end of the edentulous space (Class I bilateral DE, Class II
unilateral), the denture is subjected to rotational forces which require additional units to resist these forces. This is achieved
using indirect retainers.
• Indirect retainers controls the movement of the denture base away from the ridge.
• The imaginary line passing through teeth and direct retainers around which the rotation of denture occurs is known as
fulcrum line.
• More than one fulcrum line may be present for the same removable partial denture.
• Class I RPD has three fulcrum lines.
https://t.me/DentalBooksWorld
Chapter 14 • Prosthodontics 1213

Functions of indirect retainers


• Resists the vertical movement of denture base away from the residual ridge.
• Indirect retainer in DE RPD uses the mechanical advantage by moving the fulcrum line farther from the force.
• Contributes to the support and stability of the partial denture.
–– An indirect retainer should be placed as far anterior from the DE base as possible.
–– A line projected at right angles from the fulcrum line indicates the most effective location of the indirect retainers.
Forms of indirect retainers
• Auxiliary occlusal rest
• Canine extension from occlusal rest
• Lingual rest
• Incisal rest
• Continuous bar retainers and lingual plates
• Rugae support

Condition Clasp indicated


Clasp indicated to engage a mesiobuccal undercut adjacent to Combination clasp
a distal extension space
Clasp indicated to engage a distobuccal undercut adjacent to Bar clasp
distal extension space Reverse circlet or reverse approach clasp
“C” clasp or Fish hook or Hair pin clasp
Clasp indicated in tipped molars (maxillary molars tip Ring claps
mesiobuccally and mandibular molars tip mesiolingually)
Clasp indicated when denture replaces an entire half of the Multiple clasp
dental arch or when additional retention is needed
Clasp indicated in nonedentulous arches Embrasure clasp
Isolated rotated abutment tooth Half and half clasp

QUICK FACTS

Few Facts on Indications of Removable Partial Denture:


• Planning of RPD starts at the time of examination itself.
• The main advantage of RPD over FPD in replacing bilateral lost teeth is, cross-arch stabilization.
• The primary consideration in designing bilateral distal extension RPD is load distribution.
• Ante’s law: The pericemental surface area of the abutment teeth to be used for a fixed partial denture must be equal to or
exceed the pericemental surface area of the teeth being replaced.
• Ante’s law was proposed by Johnston et al.

Quick Facts on Surveyor


• Dental surveyor was first introduced by A. J. Fourtunati in 1918.
• Commercially, Ney instrument was marketed in 1923 which is most commonly used.
• Dr. Nobel Wills introduced Horizontal arm.
• Horizontal arm is fixed in Ney surveyor.
• The term height of contour – Coined by Kennedy.
• Infrabulge and suprabulge – De Van.
• Cummer called it as guideline which helps in the placement of clasps.
• Typical survey line – medium survey line.
• Atypical survey line – high, low, diagonal.

https://t.me/DentalBooksWorld
1214 Triumph's Complete Review of Dentistry

Quick Facts on Preparation of Rest Seats


• Reduction of the marginal ridge of approximately 1.5 mm is usually necessary.
• There should be no sharp edges or line angles in the preparation.
• Floor of the rest seat must incline toward the center of the tooth to transmit forces down the long axis of tooth.
• When the angle between the floor of rest seat and long axis of the abutment is greater than 90 degrees, it causes
–– Failure to transmit occlusal forces along the long axis of the abutment tooth
–– Slippage of the prosthesis
–– Orthodontic movement of the abutment tooth
• Improperly prepared rest seats have the effect of inclined planes between tooth and prosthesis.
• For DE prosthesis, the occlusal rest seats are located on the distal side of the distal most abutment next to edentulous space.
• For a DE prosthesis, the rest seat should be shallow, saucer-shaped, and the rest should move like a ball and socket joint,
allowing the horizontal forces to be dissipated.
• The important cause of failure of an occlusal rest is insufficient reduction of the marginal ridge (1.5 mm is usually required).

MULTIPLE CHOICE QUESTIONS: COMPLETE DENTURE

ANATOMICAL LANDMARKS, IMPRESSION PROCEDURES


1. Common cause of repeating impression is
A. Improper position of tray B. Too less or excess material
C. Voids in the impression D. Mixing not good
2. Impressions in which the tissues are recorded with minimum displacement are known as
A. Functional B. Secondary
C. Mucostatic D. Fluid wax
3. Centric relation is
A. Horizontal reference position of mandible
B. Vertical reference position of mandible
C. Both horizontal and vertical reference position of mandible
D. None of these
4. Disadvantage of functional relining method is
A. Fit is lost in the denture after bone resorption
B. The tissue surface of the metal framework cannot be relined after insertion
C. Occlusion is usually effected due to addition of the new layer to the surface of the denture
D. Maintains the relationship of the framework of the abutment teeth while making the impression
5. While making impression of flabby fibrous tissues on maxillary ridge for complete denture prosthesis, care is taken to
A. Use mucostatic impression technique
B. Use selective pressure technique of impression making
C. Use a close fitting tray
D. Follow normal procedures
6. Which of the following are stress-bearing areas?
A. Buccal surface, incisive papilla, palatine rugae
B. Slopes of residual ridge, palatine rugae, and midpalatine raphe
C. Incisive papilla, slope of the residual ridge, and crest of the residual ridge
D. Slope of residual ridge, buccal shelf area, and hard palate
7. The primary stress-bearing area of maxillary complete denture
A. Alveolar ridge B. Buccal flange
C. Posterior palatal seal area D. Palate
8. PPS anatomically is
A. Pterygomandibular notch and fovea palatine B. Pterygomandibular notch and PNS
C. Maxillary tuberosity and pterygomaxillary raphe D. Posterior limit of palatine bone

https://t.me/DentalBooksWorld
Chapter 14 • Prosthodontics 1215

9. The relationship of the denture base that resists dislodgement of denture in the horizontal direction is
A. Stability B. Pressure
C. Support D. Retention
10. Before making final impressions for the new denture, patients must be restricted from wearing old dentures for
A. 1–2 hours B. 2–6 hours
C. 12–24 hours D. 48–72 hours
11. Mouth temperature waxes for recording functional impression
A. Modeling wax B. Iowa
C. Sticky D. Green stick
12. Retromolar pad
A. Should not be covered by lower denture
B. Should be covered one-half to two-thirds of it by lower denture
C. Should be covered less than one-fourth of it by lower denture
D. Should be covered completely by lower denture
13. In mandibular denture, distobuccal flange is mainly influenced by
A. Buccinators B. Masseter
C. Temporalis D. Platysma
14. Buccinator mechanism is opposed by
A. Orbicularis oris B. Tongue
C. Superior constrictor D. All of the above
15. Which of the following structure is present on posterior extent of posterior palatal seal?
A. Vibrating line B. Fovea palatine
C. Junction of hard and soft palate D. Posterior nasal spine
16. Which muscle has a dual function as related to complete denture?
A. Masseter B. Temporalis
C. Lateral pterygoid D. Geniohyoid
17. For taking final impression, tray should be inserted (AIIMS SR Ship question…still it is a doubt whether its option A
or option B. Spectrum says as option B, Pulse as option A)
A. Anteriorly first B. Posteriorly first
C. Both together D. No fixed procedure
18. Which of the following statement is false?
A. The term “pear-shaped” pad was coined by Craddock B. Mucosa overlying pear-shaped pad is stippled
C. Retromolar pad lies posterior to pear-shaped pad D. Mucosa overlying pear-shaped pad is not stippled
19. The retromolar pad must be covered by the denture base because it aids in
A. Retention only B. Retention and support
C. Stability only D. Stability and support
20. The most successful materials for soft liner applications have been
A. Impression plaster B. Silicone rubbers
C. Waxes D. Irreversible hydrocolloids
21. The mean ratio of anterior maxillary RRR to anterior mandibular RRR is
A. 1:4 B. 4:1
C. 2:1 D. 3:1
22. Movement of denture base was the least in patient with poor ridges when
A. 20 degree teeth were used B. 33 degree teeth were used
C. 0 degree teeth were used D. Inverted cusp teeth were used
23. The record of the position of the patients maxillary ridge in relation to the condyles is the
A. Jaw relation B. Articulator
C. Facebow record D. Centric jaw relation
24. The purpose of relieving mid–palatine area in complete dentures is to prevent
A. Pressure on palate B. Midline fractures in dentures
C. Incorrect centric relation D. Resorption of bone

https://t.me/DentalBooksWorld
1216 Triumph's Complete Review of Dentistry

ARTICULATION AND JAW RELATION


1. Before an accurate facebow transfer record can be made, it is necessary to determine
A. The axial center of opening closing rotation B. The inclination of each condyle
C. The physiologic rest position D. Centric relation
2. Five factors of articulation was originally described by
A. Boucher B. Fenn Liddlow and Gimson
C. Vincent Trapuzzano D. Rudolf Hanau
3. Occlusal rim in the posterior region
A. Slightly buccal to the crest of the ridge B. Slightly lingual to the crest of the ridge
C. On the crest of the ridge D. Vestibule
4. Which color of articulating paper is used for the diagnosis of premature contacts in eccentric relations?
A. Green B. Blue
C. Red D. Pink
5. What is the established vertical dimension freeway space in mm?
A. 5–6 mm B. 1–3 mm
C. 2–4 mm D. 6–8 mm
6. Altering of vertical dimensions result as errors in occlusion if
A. Facebow transfer is not done
B. Diagnostic impressions have bubbles on the occlusal surface
C. Diagnostic photographs are not done
D. Facebow transfer is done
7. Hinge axis facebow records
A. Relationship of mandible to cranium
B. Relationship of maxilla to mandible
C. Relationship of maxilla to the axis of rotation of the jaw
D. All of the above
8. Extraoral tracing method is grouped in the following for recording centric relation for complete denture patients
A. Functional method B. Close mouth method
C. Static method D. None of the above
9. Protrusive relation records
A. Incisal guidance B. Condylar guidance
C. Terminal hinge axis opening D. Centric relation
10. During protrusive movement of mandible, interferences are present in
A. Incisal guidance B. Anterior teeth
C. Maxillary cusps (mesial inclines) D. Mandibular cusps (mesial inclines)
11. In a whip mix articulator, the intercondylar distances can vary or can be adjusted from
A. 88–112 mm B. 80–110 mm
C. 90–120 mm D. 75–125 mm
12. After facebow transfer and CR
A. Articulate maxillary cast first
B. Articulate mandibular cast first
C. Simultaneously articulate both casts in CR position
D. Simultaneously articulate both casts at protrusive position
13. False statement of physiological rest
A. It is an unreliable method B. Is used to determine centric relation
C. Taken when muscles are in relaxed state D. It requires patient co-operation
14. Fastest and most appropriate method of recording condylar axis?
A. Kinematic B. Ear piece
C. Fascia type D. Twirl bow

https://t.me/DentalBooksWorld
Chapter 14 • Prosthodontics 1217

15. The most easiest and fastest way to record condylar axis is (another version of the same question)
A. Eye bow B. Facebow (ear piece)
C. Kinematic bow D. None
16. Which is not considered as anterior reference point in facebow transfer? (Not a third point)
A. Orbitale B. Menton
C. Ala of nose D. Nasion

OCCLUSION, SELECTIVE GRINDING, AND TROUBLESHOOTING


1. What is the lesion associated with the following appearance – Red painful mucosa velvet like, tender palate is seen on
removal of denture?
A. Pseudomembranous candidiasis B. Mucocutaneous candidiasis
C. Atrophic candidiasis D. Ulcerative candidiasis
2. Group function occlusion in an existing dentition is
A. No balancing side contacts
B. Working side contacts from canine to third molar without balancing side contacts
C. Canine rise in protrusion
D. Both A and B
3. Wax add-on technique employed in developing occlusion was pioneered by
A. PK. Thomas B. Meyerson
C. E. Payne D. Stuart and Stallard
4. The reason for burning sensation in anterior palate region in a patient wearing new complete denture
A. Overextension in buccal sulcus area B. Occlusal discrepancies
C. Inadequate relief of incisive papilla D. Rough palatal surface
5. When there is a prematurity in centric occlusion but not in eccentric or other movements, then reduce
A. Cusps of opposing teeth B. Mesiodistal position of teeth
C. Opposing fossa or marginal ridge D. None of the above
6. If the inclination of the condylar path during protrusive movement is shallow, the cusp height must be
A. Long B. Flat
C. Short D. Concave
7. When setting the teeth in neurocentric concept of occlusion
A. There is no need to be concerned with sagittal condylar incline
B. Sagittal condylar incline plays a key role
C. There is a projection above the occlusal plane
D. There is a projection below the occlusal plane
8. Main disadvantage of single complete denture against natural teeth is
A. Abrasion of natural teeth B. Abrasion of artificial teeth
C. Difficulty of making of balanced occlusion D. All of the above
9. Discrepancy between centric relation and centric occlusion correlates with
A. 0.15–0.25 mm B. 0.20–0.40 mm
C. 0.50–1.5 mm D. 1–2 mm
10. If deflective occlusal contact is found in centric occlusion and not in other working centric movements, then
A. Grind the cusp and opposing fossa B. Grind the deflective cusp
C. Grind the marginal ridge or opposing fossa D. No need of any t/t as it is normal
11. Bennett shift is
A. Lateral movement of the body of the mandible B. Direct lateral shift of the working side condyle
C. Direct lateral shift of the balancing side condyle D. Forward translation of both the condyles
12. A bilateral balanced occlusion
A. Is dental articulation, which is unobstructed by cuspal interference
B. Is simultaneous contact of the occluding surfaces of the teeth of both sides of the mouth in the retruded jaw relationship
C. Is simultaneous contact of the occluding surfaces of the teeth of both sides of the mouth in the various jaw positions
D. Results in Christensen’s phenomenon

https://t.me/DentalBooksWorld
1218 Triumph's Complete Review of Dentistry

13. Purpose of beading maxillary cast is to


A. Transfer the major connector design with a visible outline
B. Give strength to the casting
C. Provide intimate tissue contact
D. Provide for undercuts
14. Lingual placement of post teeth will not cause
A. Speech problem B. Gagging
C. Tongue movement reduction D. Excessive force on residual ridge
15. Linear curve running from anterior to posterior teeth is
A. Curve of Spee B. Curve of Monson
C. Curve of Anti-Monson D. Bonwill curve
16. Zygoma (or) Key ridge corresponds to
A. Mesiobuccal root of upper 1st molar B. Mesiobuccal root of upper 2nd molar
C. Distobuccal cusp of upper 1st molar D. Distobuccal cusp of 2nd molar
17. Balanced occlusion is achieved in noncuspal posterior teeth by
A. Increasing the incisal guidance B. Increasing the compensative curves
C. Decreasing the condylar guidance D. All of the above
18. The theory of matching teeth to face forms was advanced by
A. Hanau B. Frush and Fischer
C. J Leon Williams D. Schiffman
19. Anatomic teeth should have a cusp angle of
A. 30 degrees B. 31 degrees
C. 32 degrees D. 33 degrees

PHONETICS AND OVERIMMEDIATE DENTURE


1. The purpose of “V” sound in complete denture patient is
A. Anterior posterior relation of maxillary and mandibular teeth
B. Freeway space
C. Closest speaking space
D. None of the above
2. Which one among of the following is not an advantage of overdenture?
A. Maintenance of residual ridge integrity
B. Improved denture retention
C. Perception of natural feeling
D. Conducive plaque – free environment to maintain abutment
3. The advantage of metal base denture is
A. Increase in tissue tolerance B. Easy laboratory procedure
C. Rebasing and relining are quite easy D. Increase in restorative cost
4. Difficulty in pronouncing “T” sounds is due to
A. Thick mandibular space B. Irregular lower incisors
C. Irregular upper incisors D. None of the above
5. During the pronunciation F and V, they sound alike when the anterior teeth are positioned:
A. Too forward B. Too long
C. Too broad D. Too narrow
6. Which of the following measures the vertical dimension when the mandible and muscles involved are in physiologic
function of speech?
A. Freeway space B. Leeway space
C. Closest speaking space D. Primate space
7. The biggest disadvantages of immediate denture is
A. Delayed healing of sockets B. Impossibility of anterior try-in
C. Severe alveolar bone loss D. Less chair side time in fabrication

https://t.me/DentalBooksWorld
Chapter 14 • Prosthodontics 1219

8. Minimum distance between alveolar bone and occlusal plane in implant supported overdenture should be
A. 10 mm B. 12 mm
C. 15 mm D. 18 mm

RELINING, REBASING, AND OTHERS


1. All of the following are requirements of an ideal denture cleanser except
A. Should be difficult to separate from denture fit surface B. Should dissolve all denture deposits
C. Exhibit bactericidal effect D. Nontoxic
2. Procedure which involves adding new layer of processed denture material to the denture base is called
A. Rebasing B. Relining
C. Volcanizing D. Addition polymerization
3. Main component of denture cleanser is
A. H2O2 B. Detergents
C. Perborates D. Alkali agents
4. The mean denture bearing area of edentulous mandible and maxilla is, respectively
A. 14 cm2 and 24 cm2 B. 24 cm2 and 14 cm2
2 2
C. 14 mm and 24 cm D. 24 cm2 and 14 cm2
5. The hinge axis is an imaginary line passing through or near the
A. Center of the external auditory meatus ` B. The coronoid process
C. Center of the condylar fossae D. The center of the condyle
6. The lateral condylar guidance of Hanau is calculated by
A. L = H/8 + 12 B. L = H/12 + 8
C. L = 1/2 H + 12 D. L = H/8 − 12
7. The average denture bearing area in the edentulous mandible is approximately
A. 18.13 cm2 B. 24.25 cm2
2
C. 14 cm D. 16.25 cm2
8. Rebasing of denture involves
A. To refit the denture in which the impression surface and the denture ceases to fit the tissue properly
B. To refit and simultaneously reorient the denture
C. To reorient the denture only
D. None of these
9. The ratio of 1.618 to 1.0 that has been celebrated as the standard of visual esthetics since ancient times is a constant
that is designated as
A. Ф B. ∆
C. Οο D. TT
10. According to Cawood and Holle, cup-shaped depression in the upper part of the alveolar ridge of edentulous mandible
comes under
A. Class III B. Class IV
C. Class V D. Class VI
11. Which of the following is true about remounting?
A. Used for rebasing procedure
B. To maintain stability
C. More accurate for interocclusal correction than in mouth
D. It should not be done
12. A reline for a complete denture is contraindicated when
A. There is resorption of the ridge B. The denture contains a broken tooth
C. Centric occlusion and centric relation do not coincide D. There is excessive overclosure of the vertical dimension
13. Torquing is resisted by denture via
A. Adhesion B. Cohesion
C. Adequate border seal D. Interfacial surface tension

https://t.me/DentalBooksWorld
1220 Triumph's Complete Review of Dentistry

14. In hemi-mandibulectomy, vertical dimension is best determined by


A. Lips closure and unstrained facial expression B. Facial silhouette
C. Facial photography D. All of the above

TEETH SELECTION
1. Squint test is used as a guide for selecting
A. Shade of teeth B. Shape of teeth
C. Length of teeth D. Width of teeth
2. Which of the following is not true for conventional heat activated PMMA denture base resins?
A. Insoluble in oral fluids B. Low cost
C. High abrasion resistance D. Short fatigue life
3. While arranging artificial teeth, the labial surfaces of the maxillary central incisors are usually ___________ in front
of the posterior border of the incisive papilla
A. 12–15 mm B. 5–8 mm
C. 8–10 mm D. 2–3 mm
4. In complete denture, esthetics of tooth selection depends on
A. Density of hue B. Lightness or darkness
C. Metamerism of material D. Value of color

MULTIPLE CHOICE QUESTIONS: FIXED PARTIAL DENTURE

COMPONENTS OF FPD, RESIN MODIFIED BRIDGES


1. Lost salt technique is used to fabricate
A. Maryland bridge B. Rochette bridge
C. Cantilever bridge D. Virginia bridge
2. Extraction of a tilted abutment tooth is indicated when the tilt is more than
A. 25–30 degrees B. 10–15 degrees
C. 5–10 degrees D. 45–50 degrees
3. Laboratory analog that cannot not used as a screw abutment
A. Brass B. Stainless steel
C. Chromium cobalt (Aluminum) D. Ti alloys
4. It was unwise to provide a fixed partial denture when the root surface area of the abutment was less than the root
surface area of the teeth being replaced. This was given by
A. Ante’s law B. Applegate’s law
C. Pick’s first law D. Hooke’s law
5. Pier abutment is a
A. Long-standing abutment B. Free end abutment
C. Mesially tilted abutment D. Distally tilted abutment
6. The optimum crown root ratio is
A. 3:2 B. 2:3
C. 1:2 D. 2:1
7. A 25-year-old male patient complaints of missing tooth #27 with no distal abutment and prefers a fixed form of
prosthetic treatment. An abutment that is present only at one end of the edentulous span is called
A. Cantilever abutment B. Pier abutment
C. Mesially tilted abutment D. Distally tilted abutment
8. Forces acting on the abutment tooth in an FPD can be effectively minimized by
A. Directing the forces perpendicular to long axis of the crown
B. Directing the forces along the long axis of the crown
C. By splinting as many teeth as possible
D. By using proper cementation techniques
https://t.me/DentalBooksWorld
Chapter 14 • Prosthodontics 1221

9. Lateral incisors alone cannot be used as an abutment due to


A. Unesthetic B. Insufficient pericemental area
C. Arch of rotation might be interfered D. None
10. Two or more parallel vertical surfaces of abutment teeth shaped to direct the prosthesis during placement and removal,
is known as
A. Orientation plane B. Parallel block out
C. Survey lines D. Guiding planes
11. Biologically and mechanically acceptable connector of FPD
A. Is thin occluso-gingivally and wide facio-lingually
B. Extends the entire inter-proximal space occluso-gingivally
C. Is circular in form and occupies the region of the contact area
D. Extends into the facial margins of the retainer
12. One of the following is true about a pier abutment
A. Presence of edentulous space mesial to the abutment tooth
B. Presence of edentulous space distal to the abutment tooth
C. Presence of edentulous spaces on both sides of the abutment tooth
D. A periodontally weak abutment
13. The resin bonded bridges have the advantages of
A. Minimal tooth preparation B. Alignment correction
C. Longevity D. Long span bridge
14. The abutment for a proximal half crown has
A. An intact buccal surface B. An intact lingual surface
C. An intact distal surface D. An intact mesial surface
15. Nonrigid connectors are indicated for an FPD when
A. It is not possible to prepare two abutments with a common path of placement
B. The abutments are prepared with a common path of placement
C. The abutments have a poor crown to root ratio
D. The abutments have an ideal crown to root ratio
16. The form of retention utilized in resin retained FPDs or Maryland bridges is
A. Mechanical retention B. Chemical retention
C. Physical retention D. Micro mechanical retention
17. A lone standing mandibular second molar is to be used a distal abutment for a three unit bridge. The best treatment
option is
A. Use a nonrigid connector in the bridge B. Uprighting of the tooth orthodontically
C. Prepare the molar abutment for three fourth crown D. Use of telescopic crown
18. Ante’s law was introduced in the year
A. 1911 B. 1924
C. 1926 D. 1928
19. One of the following is not a component of fixed partial denture prosthesis
A. Nonrigid connector B. Split pontic
C. Abutment D. Partial veneer crown
20. Which of the following tooth is the least desirable to use as an abutment tooth for an FPD?
A. Tooth with pulpal involvement B. Tooth with minimal coronal structure
C. Tooth rotated and tipped out of line D. Tooth with short, tapered root with long clinical crowns

PRINCIPLES OF TOOTH PREPARATION


1. Principles of tooth preparation include all of the following, except
A. Preservation of tooth structure B. Structural durability
C. Marginal integrity D. Contacts and contour
2. Which among the following is not a principle of tooth preparation?
A. Retention and resistance B. Marginal integrity
C. Temporization D. Preservation of tooth structure
https://t.me/DentalBooksWorld
1222 Triumph's Complete Review of Dentistry

3. Functional cusp bevel is given for


A. Marginal integrity B. Structural durability
C. Retention and resistance form D. To improve the geometry of tooth surface
4. Uniform reduction of the tooth surface may be ensured by the placement of
A. Pins B. Depth grooves
C. Ledges D. Bevels
5. In FPD, retention is provided by
A. Over reduction B. Parallel walls
C. Over tapering D. Functional cusp bevel
6. The most favorable restoration for a root canal treated tooth is
A. A full veneer crown B. A three-quartet crown
C. An onlay D. Depends upon the condition of crown
7. Best FPD for single unit crown is
A. Highly polished porcelain B. Highly glazed porcelain
C. Highly polished acrylic D. Highly polished cast restoration

ALL CERAMIC, METAL CERAMIC RESTORATIONS AND FINISH LINES


1. A three-quarter crown preparation is named so based on
A. Number of line angles involved B. Number of retentive grooves placed
C. Number of surfaces involved D. Number of axial walls involved
2. The recommended margin for porcelain laminate veneer is
A. Chamfer B. Long chamfer
C. Feather edge D. Shoulder
3. In full veneer the preparation seating groove is formed with
A. No. 171 L bur B. Round end tapered diamond
C. Short needle diamond D. Torpedo bur
4. Overall optimum degree of taper preparation for maxillary anterior tooth is
A. 6 degrees B. 10 degrees
C. 14 degrees D. 19 degrees
5. The best gingival finish line for porcelain and jacket crown
A. Chamfer B. Knife edge
C. Shoulder D. Shoulder on facial and chamfer on lingual side
6. The ideal crown preparation has a degree of convergence from gingival finish line toward occlusal
A. 2–4 degrees B. 3–5 degrees
C. 4–9 degrees D. 8–11 degrees
7. Amount of occlusal reduction for metal-ceramic crown on functional cusp is
A. 0.8 mm B. 1.5–2.0 mm
C. 2–2.5 mm D. 0.5–0.8 mm
8. The best finish line for an anterior metal ceramic crown is
A. Chamfer with bevel B. Heavy chamfer
C. Shoulder D. Shoulder with bevel
9. Finish line of choice when preparing a tooth for an all ceramic restoration would be
A. Chamfer B. Shoulder
C. Bevel D. None of the above
10. Contraindications for the metal-ceramic crown include
A. Caries B. Untreated periodontal diseases
C. Patients with large pulp chambers D. All of the above
11. When preparing a three-fourths crown on maxillary incisor tooth, the proximal grooves should generally be parallel
to the
A. Long axis of the tooth B. Gingival two-thirds of the labial surface
C. Incisal two-thirds of the labial surface D. Short axis of the tooth

https://t.me/DentalBooksWorld
Chapter 14 • Prosthodontics 1223

12. Disadvantage of ceramic veneer


A. Discoloration B. Lower strength
C. Absorption of oral fluids D. Technique sensitive
13. Chamfer finish line is used in
A. Labial side of all ceramic crown B. Lingual of all ceramic
C. Lingual of PFM D. Labial of PFM
14. The recommended finish line for porcelain jacket crown is
A. Chamfer B. Knife edge
C. Shoulder D. Shoulder with bevel
15. Reverse three-fourths crown is placed on the crown as it is
A. More esthetic than conventional ¾ crown B. Requires less tooth reduction axially
C. More retentive than complete veneer crown D. Easy to prepare compared to conventional ¾ crown
16. Amount of cusp reduction recommended to receive a metal ceramic restoration is
A. 0.75 mm B. 1 mm
C. 1.5 mm D. 2.5 mm
17. The chamfer finish line is used in
A. Lingual surface of metal ceramic crown B. Facial surface of metal ceramic crown
C. Facial surface of porcelain jacket crown D. Lingual surface of porcelain jacket crown
18. Which of the following finish lines can be used for an all-metal crown?
A. Chamfer B. Bevel
C. Feather edge D. All of the above
19. All of the following are advantages of shoulder finish line, except
A. Good crown contour B. Less distortion
C. Provide adequate bulk D. Superior marginal adaptation
20. The ultimate in finish lines that permit an acute margin of metal is
A. Shoulder B. Chamfer
C. Knife edge D. Shoulder with bevel
21. Finish lines of a complete coverage crown preferably should lie
A. Supragingivally B. Subgingivally
C. At gingival margin D. Supragingival on labial and subgingival on lingual side
22. If the destruction is 2 mm beyond the “ac (alveolar crest),” how much extrusion is needed?
A. 5 mm B. 3 mm
C. 4 mm D. 6 mm
23. Reduction done on facial aspect of a tooth for laminate veneer is
A. 0.5 mm B. 1.0 mm
C. 1.5 mm D. 2.0 mm
24. Which of the following does not have slide fit?
A. Shoulder B. Chamfer
C. Shoulder with bevel D. Knife edge
25. The chamfer finish line is used in
A. Lingual surface of metal ceramic crown B. Facial surface of metal ceramic crown
C. Facial surface of porcelain jacket crown D. Lingual surface of porcelain jacket crown
26. The gingival finish line for veneer metal restoration is
A. Radial shoulder B. Chamfer
C. Shoulder D. Shoulder with bevel
27. If the root of the tooth is fractured 3 mm beyond CEJ (Cemento Enamel Junction), how much of extrusion is needed
for surgical crown lengthening?
A. No need. Extrusion is done only if fracture is more than 4 mm
B. 3 mm of extrusion
C. 2 mm of extrusion
D. 4 mm of extrusion

https://t.me/DentalBooksWorld
1224 Triumph's Complete Review of Dentistry

CEMENTATION OF FPD
1. Temporary restoration or crown which match natural teeth and color
A. Celluloid B. Porcelain
C. Gold restoration D. ZOE
2. Which of the following cannot be given as temporary restoration?
A. Zinc-oxide eugenol B. Cellulose crowns
C. Polycarboxylate D. An acrylic crown cemented with zinc oxide-eugenol
3. Factors that decrease the cement space for a complete crown would be
A. Thermal and polymerization shrinkage of the impression material
B. Use of a solid cast with individual stone dies
C. Use of an internal layer of soft wax
D. Use of resin or electroplated dies
4. Luting agent applied in single crown should
A. Fill one-half of the inner volume
B. Fill one-fourth of the inner volume
C. Completely filling to eliminate air
D. Be applied just as liner to the inner surface of the crown
5. Fabrication of crown by CAD-CAM processing occurs within
A. 5 minutes B. 10 minutes
C. 20 minutes D. 30 minutes
6. The grayish discoloration of ceramic veneers looks like a nonvital tooth; this is due to
A. Hue B. Chrome
C. Value D. Brilliance
7. Thickness of the die spacer should be
A. 10–20 μm B. 20–40 μm
C. 30–60 μm D. 25–50 μm
8. Minimum length of the die preparation on the cast should be
A. 0–5 mm B. 5–10 mm
C. 10–15 mm D. 15–20 mm
9. The success of a removable die system depends upon
A. The type of the pin system used B. Abrasion resistance of the die material
C. Precise relocation of the die in the working cast D. Length of the dowel pin
10. The best way to assess the occlusal clearance after an onlay preparation is
A. Articulating paper B. Wax chew-in
C. Depth cuts D. Impression

TISSUE MANAGEMENT AND GINGIVAL RETRACTION


1. Tissue displacement is commonly needed to obtain
A. Point contact
B. Maryland bridge
C. To expose all necessary surfaces, both prepared and not prepared
D. None of the above
2. In doing gingival retraction all of the following are used, except
A. 8% Racemic epinephrine B. 8% Alum
C. 8% Zinc chloride D. Aluminum nitrate
3. Chemico-mechanical gingival retraction is not required
A. For excavating gingival caries B. Expose finish line of abutment for recording impression
C. For subgingival preparation D. Copper band impression

https://t.me/DentalBooksWorld
Chapter 14 • Prosthodontics 1225

MULTIPLE CHOICE QUESTIONS: REMOVABLE PARTIAL DENTURE

RPD PART 1
1. Find the Kennedy classification in reference to the partially edentulous arch and accompanying framework shown


A. Class I, Modification 0 B. Class II, Modification 1
C. Class III, Modification 0 D. Class IV, Modification 0
2. Identify the correct in designing a RPD–acrylic resin portion is marked by
A. Blue B. Brown
C. Pink D. Red
3. Your patient has teeth 20–29 remaining. You plan to use mesial rests and I-bars on both terminal abutments. Because
there are lingual tori present, you will need to use linguoplating in the premolar areas. The illustration is a lingual view
of teeth 27–29. The dotted line on #29 represents the height of contour (survey line). What should be the relationship
of the superior border of the plating and the survey line?
A. The plating should extend above the survey line B. The plating should end at or below the survey line
C. The plating must end exactly at the survey line D. The plating must end below the survey line
4. In the maxillary class III RPD shown either a palatal strap or an anterior–posterior palatal strap major connector
may be used. If an anterior–posterior strap design is to be considered, the opening between the anterior and posterior
palatal straps should be at least _______ mm
A. 5 B. 10
C. 15 D. 20
5. True or False
Because there are no movable tissues on the palate, the borders of maxillary major connectors may be located farther from
the gingival margins than those of mandibular major connectors
A. True B. False
6. On the mandibular class III framework shown, circumferential clasps have been used on the canine, premolar, and
molars. Which one of the following statements regarding the clasping is TRUE?
A. The retentive arms on the canine and premolar should be wrought wire while those on the molars should be cast
B. The lingual arms on the molars are frequently the retentive arms because there are often no usable facial undercuts
C. If the undercut on the second premolar is on the distofacial, the circumferential retentive arm would most likely be
changed to I-bars
D. In order to ensure adequate retention, the tips of both the buccal and lingual arms on the molars should be placed in
undercuts
7. True or False
On the RPD framework shown, the rests on teeth numbers 20 and 28 are indirect retainers and function when the patient
bites down on the distal extension area
A. True B. False
8. True or False
Guide surface preparations should be completed before rest seat preparations
A. True B. False
9. True or False
In the class II mandibular RPD shown to the right, the clasp on tooth #27 should be wrought wire because the cingulum
rest acts like a distal rest and the tip of the retentive arm is in front of the axis of rotation
A. True B. False

https://t.me/DentalBooksWorld
1226 Triumph's Complete Review of Dentistry

10. True or False


When tripoding a cast, the vertical arm of the surveyor must be locked in position
A. True B. False
11. Referring to the maxillary class II RPD design shown to the right, which of the following statements is TRUE?
A. A short thick I-bar clasp could be used on tooth #6
B. A wrought wire circumferential clasp could be used on tooth #6
C. It is sometimes not necessary to use any clasp on tooth #6
D. B and C are both true
12. The following statement refers to the drawing of the class I RPD to the right: The clasp arm on tooth #20 should be
wrought wire because functional forces cause tissueward movement of the denture base, the denture rotates around
the rest (which is on the distal), and the clasp arm engages the tooth, tending to tilt it distally
A. The statement and the reason are true and related B. The statement and the reason are true but are not related
C. The statement is true but the reason is false D. Neither the statement nor the reason is true
13. In the maxillary class I RPD shown at the right, the posterior border of the palatal plate major connector should be
located
A. As far forward as is possible while maintaining enough width for strength
B. Approximately 20 mm behind the anterior border
C. Approximately 10 mm anterior to the fovea palatini
D. At the posterior palatal seal area
14. True or False
All components of a maxillary major connector should cross the palate at right angles to the midline and curves in the
major connector should be located to one side of the midline
A. True B. False
15. In the case shown to the right, which is the most effective indirect retainer?
A. The rest on tooth #19 B. The rest on tooth #21
C. The rest on tooth #27 D. The rest on tooth #28
16. What is the most significant problem with the rest shown on the mandibular canine to the right?
A. The rest should be wider incisogingivally B. The rest should be located more toward the incisal edge
C. The rest should be thicker buccolingually D. The rest should be located in a prepared rest seat
17. True or False
In the RPD shown to the right, the guide surface–guide plate contacts on the distal surfaces of teeth numbers 20 and 29
determine a definite path of displacement/dislodgement
A. True B. False
18. True or False
The all plastic maxillary RPD should be avoided in all partial dentures except where the prosthesis is considered transitional
and loss of the remaining teeth is imminent
A. True B. False
19. True or False
Tooth-supported RPDs require physiologic relief of the guide plates
A. True B. False
20. The superior border of a mandibular lingual bar major connector must be at least _____ mm below the gingival
margins.
A. 1–2 B. 2–3
C. 3–4 D. 4–5
21. For maxillary major connectors, except where plating is used, the border of the major connector should be at least
______ mm away from the gingival margin
A. 1 B. 3
C. 5 D. 6
22. For the anterior–posterior palatal strap maxillary major connector, the anterior, posterior, and lateral straps should
be about ______ mm wide
A. 4–6 B. 6–8
C. 8–10 D. At least 10

https://t.me/DentalBooksWorld
Chapter 14 • Prosthodontics 1227

23. The basic philosophy of dental treatment for a partially edentulous patient is to
A. Preserve what remains and restore what is missing B. Replace the missing teeth
C. Improve the path of insertion D. Reshape rotated teeth
24. It is best not to use a balanced occlusion when mandibular RPDs oppose maxillary complete dentures
A. True B. False
25. The function of the guide plate is to
A. Help establish a definitive path of insertion/dislodgement
B. Stabilize the RPD by controlling its horizontal position
C. Provide contact with the adjacent tooth
D. All of the above
26. What is the design error in the maxillary RPD framework shown to the right?
A. The anterior teeth should have been plated
B. An anterior–posterior palatal strap major connector should have been used
C. There should be a cast circumferential clasp on tooth #6
D. The posterior border of the major connector should cross the palate at right angles to the midline
27. What would be the best denture base connector when there is limited interocclusal space (<3 mm)?
A. Open latticework B. Meshwork
C. All metal base D. All plastic base
28. The first consideration in developing occlusion is the evaluation and establishment of the correct position of the
occlusal plane. This may be compromised by supererupted and malposed teeth
A. True B. False
29. Your patient’s partially edentulous arch is depicted in the illustration to the right. The missing teeth (with the exception
of the third molars) were extracted 3 weeks ago. What would be the best denture base/replacement teeth combination
in this instance?
A. Open latticework
B. Metal bases with beads for attachment of processed tooth colored acrylic resin
C. An all metal base
D. Tube teeth
30. A wrought wire clasp is not used in which of the following situations?
A. On a terminal abutment of an extension RPD B. As an embrasure clasp
C. On a tooth with an indirect retainer on it D. Both A and B above
31. In designing an RPD framework for the partially edentulous arch to the right – if at all possible, circumferential clasps
should be used on the numbers 28 and 30. They would be preferred to infrabulge retainers
A. True B. False
32. True or False
Determining areas for physiologic relief is accomplished by marking the framework intraorally. This process includes
adjusting the casting to allow for functional movement on Class I and II RPDs to relieve stress on the terminal abutment
teeth.
A. True B. False
33. Reciprocation between bracing and retentive components requires
A. I-bars be used
B. Bracing components contact after retentive components
C. Correct timing of contact during seating and removal of the RPD
D. Both B and C
34. Infrabulge clasps originate
A. Above the height of contour B. Below the height of contour
C. Above the 0.01” undercut D. Above the occlusal surface of most premolars
35. True or False
A modified palatal plate is used in maxillary class II cases and may or may not include lingual plating
A. True B. False

https://t.me/DentalBooksWorld
1228 Triumph's Complete Review of Dentistry

36. All of the following clasps are infrabulge clasps except


A. I-bar B. Modified T-bar
C. Roach clasp D. Akers clasp

RPD PART 2
1. In a Class III modification 1 RPD, the rests are usually placed
A. On the canines B. Away from the modification space
C. In the area of the opposing occlusal contact D. Adjacent to the modification space
2. A mandibular lingual bar major connector such as the one shown in the RPD to the right requires a minimum of
_____ mm of vertical height between the gingival margin and the floor of the mouth


A. 4 B. 5
C. 7 D. 8
3. The presence of mandibular lingual tori would indicate the need for
A. A metal base B. Lingual plating
C. Tube teeth D. Extra indirect retainers
4. The illustration to the right shows a framework for a mandibular class II RPD. The indirect retainer on tooth 21 functions
when


A. The patient chews on something hard B. The patient chews on something soft
C. The patient chews on something sticky D. The patient bites the bullet
5. True or False
An anterior–posterior palatal strap maxillary major connector has greater strength and rigidity than a horseshoe design
A. True B. False
6. Your patient has the mandibular arch form shown to the right. A rest on which tooth would be the most effective
indirect retainer?

A. 20 B. 21
C. 22 D. 27

https://t.me/DentalBooksWorld
Chapter 14 • Prosthodontics 1229

7. Your patient exhibits the mandibular class II modification 1 arch shown to the right. Tooth #30 is tilted mesiolingually
and has very little tissue undercut on the lingual. What is the best clasp for this situation?


A. Cast circumferential clasp utilizing a distolingual undercut
B. Cast I-bar utilizing a mesiolingual undercut
C. Cast ring clasp utilizing a mesiolingual undercut
D. Wrought wire circumferential clasp utilizing a distolingual undercut
8. Rigid metal retention is associated with
A. A dual path of insertion B. A class IV RPD
C. The need for excellent esthetics D. All of the above
9. True or False
The palatal strap maxillary major connector is primarily used in short span maxillary class III modification 1 RPDs
A. True B. False
10. Your patient has the class I arch shown to the right. The axis of rotation is most frequently determined by
A. The placement (location) of the primary rests B. The type of clasp arms selected
C. The placement of the minor connectors D. The placement of the indirect retainers
11. In maxillary RPDs, the bead line is approximately _______ mm thick (deep)
A. 4 B. 3
C. 2 D. 1
12. The C + 1 rule
A. Does not apply to class IV RPDs B. Does not apply to class III RPDs
C. Gives a general rule for the number of clasps D. Both A and C
13. Which of the following maxillary major connectors possesses the least strength and rigidity?
A. Anterior–posterior palatal strap B. Horse-shoe
C. Palatal plate D. Modified palatal plate
14. True or False
A cingulum rest is normally placed between the middle and incisal thirds of the maxillary incisors in order to avoid
occlusal interferences
A. True B. False
15. Identify the part of the dental surveyor


A. Vertical arm B. Horizontal arm
C. Surveying arm D. Marking tool

https://t.me/DentalBooksWorld
1230 Triumph's Complete Review of Dentistry

RPD PART 3
1. True or False
When possible, plating on maxillary RPDs should be avoided due to the potential for interfering with speech and occlusion
A. True B. False
2. The survey line (height of contour) of a full contour wax-up for a crown for an RPD abutment tooth can be visualized using
A. Red wax B. Zinc stearate powder
C. Border wax D. Pressure indicating paste
3. Your patient has only teeth numbers 20 through 29 remaining. You have tried in the framework and are border
molding the extension areas in preparation for an altered cast impression. The distobuccal area is shaped by the
A. Buccinator muscle B. Masseter muscle
C. Internal pterygoid muscle D. Mylohyoid muscle
4. Your patient has teeth numbers 20 through 29 remaining. The survey line and undercut (shaded area) for tooth #29
are as shown on the right. There is no contraindication to the use of an infrabulge clasps but there is a very solid
contact in the mesial–occlusal fossa. What would be your choice for rest location and retentive arm?

SL


A. Distal rest and cast circumferential clasp B. Distal rest and cast I-bar
C. Mesial rest and cast I-bar D. Distal rest and WW circumferential clasp
5. Your patient has teeth numbers 20 through 29 remaining. The survey line and undercut (shaded area) for tooth #29
are as shown on the right. There is no contraindication to the use of an infrabulge clasps and there are no occlusal
problems affecting rest location. What would be the best choice for rest location and retentive arm?

SL


A. Mesial rest and I-bar B. Mesial rest and modified T-bar
C. Mesial rest and WW circumferential clasp D. Distal rest and WW circumferential clasp
6. Your patient has teeth numbers 20 through 29 remaining. The survey line and undercut (shaded area) for tooth #29
are as shown on the right. There is no contraindication to the use of an infrabulge clasps. The opposing occlusion is
provided by a complete denture. What would be your choice for rest location and retentive arm?

SL


A. Mesial rest and I-bar B. Mesial rest and modified T-bar
C. Mesial rest and cast circumferential clasp D. Distal rest and WW circumferential clasp
7. Your patient has only teeth numbers 20 through 29 remaining. You have tried in the framework and are border
molding the extension areas in preparation for an altered cast impression. The distolingual border molding is shaped
by the
A. Masseter muscle B. Internal pterygoid muscle
C. Superior constrictor muscle D. Mylohyoid muscle
8. True or False
As a general rule, plating is indicated for the maxillary arch if there are less than three contiguous maxillary incisor teeth
remaining.
A. True B. False

https://t.me/DentalBooksWorld
Chapter 14 • Prosthodontics 1231

9. All of the components listed below may be involved in the 180 degree encirclement rule except
A. I-bar B. Minor connector
C. Guide plate D. Open latticework
10. What is the correct relationship of the foot of an I-bar to the survey line (height of contour) on a terminal abutment
for an extension RPD?
A. The foot should be entirely above the survey line
B. The foot should be partially above and partially below the survey line
C. The foot should be totally below the survey line
D. The relationship of the foot of the I-bar and the survey line is of no consequence
11. True or False
The primary indication for an Oddo clasp occurs when an anterior abutment has excessive labial inclination so that the
retainer (clasp) would be very close to the incisal edge
A. True B. False
12. True or False
The purpose of the altered cast impression procedure is to obtain the maximum support possible from the edentulous
areas of class I and class II RPDs.
A. True B. False
13. True or False
When there are extensive edentulous areas present in both arches and no opposing teeth meet, we should establish the
vertical dimension of occlusion prior to making a centric relation record
A. True B. False
14. True or False
You are fabricating maxillary and mandibular RPDs for your patient. At the framework try-in appointment, both
frameworks should initially be placed in the mouth to check for occlusal interferences
A. True B. False
15. True or False
A protrusive record is made with the mandibular anterior teeth approximately 6 mm forward of centric relation (or with
the mandibular and maxillary anterior teeth in an edge to edge relationship). This record is used to set the horizontal
condylar guidance on the articulator
A. True B. False
16. True or False
The space that opens between the posterior teeth during anterior movement of the mandible is called Christensen’s
phenomenon. This posterior separation is increased if the incisal guidance is increased
A. True B. False
17. True or False
Before trying in a framework, you should inspect the master cast for damage and inspect the framework for sharp fins
A. True B. False
18. True or False
Your RPD framework fits the cast but does not fit in the mouth. One should assume that the impression for the cast was
inaccurate and that a new impression will need to be made
A. True B. False
19. True or False
The fewer teeth that remain, the more like a denture the RPD becomes and the more likely the need for a custom
impression tray
A. True B. False
20. Reason(s) for selecting a mandibular lingual plate major connector is/are
A. The presence of lingual tori
B. Anticipated loss of one or more of the remaining teeth
C. A high lingual frenum
D. A, B, and C above

https://t.me/DentalBooksWorld
1232 Triumph's Complete Review of Dentistry

21. Your patient has teeth 20 through 29 remaining. Tooth #29 exhibits the tooth contours and undercut shown in the
illustration to the right. Which of the clasp assemblies listed has the greatest danger of torquing the abutment during
functional movements of the extension base?

SL


A. Mesial rest and cast I-bar B. Distal rest and WW circumferential clasp
C. RPA clasp assembly with a cast clasp D. All have about the same potential danger
22. Which of the following clasps commonly utilize lingual undercuts?
A. Ring clasp B. Extended arm clasp
C. Half and half clasp D. Both A and C
23. Which of the abutments in the partially edentulous arch to the right has the greatest potential for utilization of a ring clasp?


A. #18
B. #21
C. #28
D. None of the abutments have any potential for the use of a ring clasp
24. True or False
Metal denture bases are most commonly used over well-healed posterior ridges where vertical space is a problem
A. True B. False
25. A cingulum rest should be placed
A. Between the occlusal and middle thirds of the incisor teeth
B. Above the middle third of the incisor teeth
C. At the junction of the gingival and middle thirds of the incisor teeth
D. On the distoincisal edges of the incisor teeth
26. The external finish line is
A. The external junction of framework metal and denture base plastic
B. The external junction of framework metal and supporting tissues
C. The external junction of the framework metal and the natural teeth
D. The external junction of the natural teeth and the denture base plastic

ANSWERS: COMPLETE DENTURE

ANATOMICAL LANDMARKS, IMPRESSION PROCEDURES


1. Answer: C
2. Answer: C (Ref. Textbook of Prosthodontics, By V. Rangarajan, T. V. Padmanabhan, 2017, page no. 228)
Mucocompressive Technique
The mucocompressive technique was initiated by Greene Brothers. They introduced a modeling plastic, a method for
manipulating it and a technique that is said to have been the first to utilize all the denture bearing area for denture retention.
They were the first to teach the closed mouth all modeling plastic technique called the Greene Brothers all compound
impression.
The main objective of this technique was to attain better retention of the dentures.

https://t.me/DentalBooksWorld
Chapter 14 • Prosthodontics 1233

The typical technique by Greene brothers was as follows.


• A preliminary impression was made in impression compound and a custom tray was constructed using baseplate with
its periphery one-eighth inch shorter than the denture outline.
• With this tray another impression with compound was taken.
• Well-fitting rinse with uniform occlusal surface were made and the height of the bite adjusted against a similar bite rim
on the mandibular ridge.
• Areas to be relieved like median raphe was softened on the impression and was again inserted in the mouth and was
held under biting pressure for one/two minutes.
• The peripheral margins of the impression was then softened and border molding was done by asking the patient to give
various cheek and lip movement as in whistling and smiling.
• The posterior palatal seal was obtained by swallowing movements by the patient under biting pressure.
• The claims made by the advocates of this technique was that since border molding was done in their functional
positions, the final dentures would retain well and cannot be dislodged during functional movements of the jaw.
Variations in this technique
• Some used the preliminary impression itself as the tray and impression to be improved by border molding.
• Some preferred to make custom trays in a more stable and stronger material than compound for better results.
• Relief in hard areas was obtained in number of ways. Some custom trays were made with escape holes in areas overlying
the hard tissues and close adaptation provided in those areas covering the soft tissues.
• Some use low fusing compound by softening and adapting it to the soft tissues.
• Some advocate unnatural movement of the mouth along with massaging of the cheeks and lips from outside during
border molding.
• Post dam is obtained in number of ways.
• The addition of soft wax like carding wax or low fusing compound for this purpose is common.
• Scraping of cast is also used.
The amount of pressure applied to the tissues in the mucocompressive technique was not only great but was applied to
the center of the palate and the peripheral tissues which were not well suited to receive the maximum biting load this
interferes with normal blood supply of the tissues resulting in their breakdown.
As soon as this change took place both the peripheral seal and excellent retention were lost. Hence the retention
achieved by these means was transient and harmful to the health of tissues.
Dentures made by this technique would fit well during mastication, i.e., only a short period each day, but would not be
closely adapted to the tissue when the patient was at rest. This is because of the rebounding of tissues.
These disadvantages indicated a need for spacer in the custom tray fabrication.
1930–1948
• Concept of mucostatics was introduced by Harry L. Page in 1938.
• Addison in 1944 also mentioned the same principle of making impressions of displaceable tissue in its passive state and
considered interfacial surface tension as one of the main factors of retention.
• With new materials like zinc oxide eugenol, waxes, elastomers, individual tray construction was emphasized.
Minimal pressure technique based on mucostatic principle
• In a brochure published by Hary L. Page in 1946 he stated that all soft tissues were chiefly fluid and 80% or more of
the tissues are composed of water. According to Pascal’s law, any pressure applied to a confined fluid is transmitted
undiminished and equally in all directions. Page contended that since the soft tissues are confined under a denture, any
pressure applied will be transmitted in all directions.
• The advocates of this principle considered interfacial surface tension as the only important retentive mechanism in
complete dentures. Therefore, they did not resist vertical displacement, which was the only movement capable of
interrupting surface tension. However, Dykins recommended a short lingual flange to resist lateral displacement.
• According to the principle of mucostatics, the impression material had to record without distortion, every detail of the
mucosa so that a completed denture would fit all minute elevations and depressions. So much emphasis was placed on
recording details that separating substances could not be used at any point in the procedure.
• Mucostatics further demanded a metal base. Gold, one of the most accurate metals, was bypassed in favor of chrome
alloy which are not considered to be quite so accurate as gold.
A typical impression method representing this technique was as follows.
• A compound impression was made in a suitable tray and a cast was made.
• On this base plate wax was adapted which acted as a spacer according to denture outline.
• Custom tray was fabricated over this spacer.

https://t.me/DentalBooksWorld
1234 Triumph's Complete Review of Dentistry

• A soft ribbon of carding wax was applied at the posterior margin of the maxillary tray and it was placed in the mouth
under light pressure and patient was asked to do swallowing movements in order to obtain a posterior palatal seal.
• A small amount of impression plaster mixed into a smooth consistency was placed in the tray, introduced in the mouth,
and was slowly raised to position and held with as little pressure as possible.
• No border molding was advocated but the soft plaster was expected to mold itself to the relaxed vestibular tissues.
• The impression was held till the impression hardened and was then removed.
Variations in the technique
• Some techniques use compound instead of wax for obtaining post dam.
• Some techniques advocate post dam over the final impression.
• Zinc oxide eugenol and alginate had also been used for similar results.
• Page’s application of Pascal’s law to the field of denture impressions is only partly correct because the tissues involved
are not wholly incompressible and fluids may escape through the borders of the denture.
• Page’s claim that retention is a function of surface tension alone is also objectionable because this tensile force itself is
dependent upon adhesion and cohesion.
• The elimination of use of separating media results in distortion of the cast.
• The use of chrome cobalt as denture bases results in failure of accurate detail reproduction.
• The mucostatic principle ignores the value of dissipating masticatory forces over as largest possible basal seat area.
Further, the mucostatic denture minimized the retentive role of the musculature as described by Fish in 1948.
The merit of this technique was its high regard for health and preservation of tissue.
1948–1964
• There was an increased emphasis on biologic factors of complete denture impression making.
• Selective pressure concept by Boucher became popular.
• Craddock, Landa et al. advocated use of escape vents.
• More attention was given to esthetics; in the impression techniques used greater emphasis was on flanges, border
molding, posterior palatal seal, and denture extension.
• In 1948, the mucoseal technique – a variation of the mucostatic technique was introduced.
• Vacustatics concept was developed by Milo V. Kubalib and C. Buffington to eliminate the functional limitations of
impressions.
Selective pressure technique based on selective pressure theory
• Advocated by Boucher in 1950, it combines the principles of both pressure and minimal pressure techniques.
• The philosophy of the selective pressure technique is that certain areas of the maxilla and mandible are by nature
better adapted for withstanding extra loads from the forces of mastication. These tissues are recorded under slight
placement of pressure while other tissues are recorded at rest or relieved with minimal pressure in a position that will
offer maximum coverage with the least possible interference with the health of surrounding tissues.
• Here an equilibrium between the resilient and nonresilient tissues is created.
Primary stress bearing areas of maxilla are crest of alveolar ridge and the horizontal plate of palatine bone and in the
mandible it is the buccal shelf area.
Secondary stress bearing areas of the maxillary foundation are rugae area and the slopes of the ridge in the mandibular
foundation.
Areas requiring minimum pressure are incisive papilla, midpalatine suture, tori in the maxilla, and crest of mandibular
residual ridge.
In the maxilla, the tissue underlying the region of posterior palatal seal has glandular and soft tissue between the mucous
membrane lining and the periosteum covering the bone. This tissue can be more readily displaced for the maintenance of
peripheral seal of the maxillary denture.
3. Answer: A (Ref. Textbook of Complete Dentures, By Arthur O. Rahn, John R. Ivanhoe, Kevin D., 2009, page no. 176)
Centric relation: the maxillomandibular relationship in which the condyles articulate with the thinnest avascular portion
of their respective discs with the complex in the anterior superior position against the shapes of the articular eminencies.
This position is independent of tooth contact. This position is clinically discernible when the mandible is directed
superiorly and anteriorly. It is restricted to a purely rotary movement about the transverse horizontal axis (GPT-5).
CR is anatomically determined; it is repeatable and reproducible (Ruth et al.). Okeson describes it as the most orthopedically
and musculoskeletally stable position of the mandible. Others consider it to be the essence of optimal temporomandibular joint
form and function. It is the most reliable reference point for accurately recording the relationship of the mandible to the maxilla.
Therefore, a determination of the CR is a prerequisite for the analyses of dental interarch, condylar position, and skeletal
relationships. A properly aligned condyle–disc assembly in centric relation can resist maximum loading by the elevator

https://t.me/DentalBooksWorld
Chapter 14 • Prosthodontics 1235

muscles with no sign of discomfort. At the most superior position, the condyle disc assembly are braced medially, thus CR
also the midmost position.
4. Answer: C (Ref. Textbook of Prosthodontics, By V. Rangarajan, T. V. Padmanabhan, 2017, page no. 230)
Functional Relining Method
It consists of adding a new surface to the inner or tissue side of the denture base.
The procedure may be accomplished before the insertion of the partial denture, or it may be done later if the denture base
no longer fits the ridge adequately because of bone resorption.
5. Answer: A
6. Answer: D
Mandible:
1. Primary stress-bearing area Buccal shelf area
2. Secondary stress-bearing area Slopes of edentulous ridge
3. Primary retentive and primary peripheral seal area Retromolar pad
4. Secondary peripheral seal area Anterior lingual border
5. Relief areas Crest of the residual ridge
Mental foramen
Mylohyoid ridge
Maxilla:
1. Primary stress-bearing area Residual alveolar ridge, maxillary tuberosity (Boucher 13th edition)
2. Secondary stress-bearing area Rugae or anterior hard palate
3. Tertiary stress-bearing area and Secondary Posterolateral part of hard palate
retentive area
4. Relieving areas Incisive papilla
Mid-palatine raphe
Cuspid eminence
Fovea palatine
5. Primary retentive area Posterior palatal seal area

Muscle having dual function in relation to Masseter


complete denture
Buccal frenum of maxilla contains Caninus (Levator anguli oris), Buccinator (Pulls frenum backward),
Orbicularis oris (Pulls frenum forward)
Buccal frenum of mandible contains Triangularis (depressor anguli oris)
Buccinator
Labial frenum of mandible contains Incisive
Distobuccal flange of the mandibular denture is Masseter
limited by
Distobuccal flange of the maxillary denture is Coronoid process, ramus, masseter
limited by
Retromolar pad contain the fibers of Temporalis
Buccinator
Superior constrictor
Pterygomandibular raphe
The lingual flange of lower denture is limited
• In the anterior region by
• In the middle region by Genioglossus
• In the posterior region by Mylohyoid
Palatoglossus and Superior constrictor
Palatoglossus notch is formed due to Action of masseter on buccinator

https://t.me/DentalBooksWorld
1236 Triumph's Complete Review of Dentistry

7. Answer: A
8. Answer: A (Ref. Textbook of Complete Dentures, By Arthur O. Rahn, John R. Ivanhoe, Kevin D., 2009, page no. 122)
According to Glossary of Prosthodontic Terms-8, posterior palatal seal is the seal at the posterior border of a maxillary prosthesis.
Posterior palatal seal area is the soft tissue area at or beyond the junction of the hard and soft palates on which pressure, with
in physiological limits, can be applied by a denture to aid in retention. Fovea palatinae are the two small pits or depressions in
the posterior aspect of the palate, one on each side of the midline, at or near the attachment of the soft palate to the hard palate.
Pterygomaxillary notch is the palpable notch formed by the junction of the maxilla and the pterygoid hamulus of the sphenoid
bone. The postdam is a raised portion of the denture base at the posterior extent of the upper denture and is located on its
fitting surface. It extends bilaterally from the midline to the pterygomaxillary notch regions and lies on a displaceable portion
of the junction of the hard and soft palates, which appears clinically immobile during phonation. This is its orthodox position.
Posterior palatal seal consist of two components, namely, pterygomaxillary seal area and postpalatal seal. Pterygomaxillary
seal extends through pterygomaxillary notch continuing 3–4 mm anterolaterally, approximating the mucogingival junction.
It occupies entire width of hamular notch (loose connective tissue lying between pterygoid hamulus of the sphenoid bone
and distal portion of maxillary tuberosity). The notch is covered by pterygomaxillary fold (extend from posterior aspect of
tuberosity to retromolar pad). This fold influences the posterior border seal if mouth is wide open during final impression
procedure. The pterygomandibular ligament extends from the hamular process to the lingula of the mandible.
Anterior vibrating line demarcates zone of transition between no movement of the tissue overlying hard palate and
some movement of the tissues of soft palate. It serves as anterior border of posterior palatal seal. It extends laterally into
pterygomaxillary notch. It is not a straight line due to the presence of posterior nasal spine. It always occurs in soft palate.
According to Sear, it is not the junction of hard and soft palates. According to Gerald S. Wintraub, it is usually located in
the junction of hard and soft palates. It can be recorded by Valsalva maneuver (ask patient to blow gently through nose
with nostrils closed using finger) or by Sharry’s method (ask patient to say “ah” with short vigorous bursts). Posterior
vibrating line is an imaginary line at the junction of the aponeurosis of the tensor veli palatini muscle and the muscular
portion of the soft palate. It is elicited by asking the patient to say “ah” in short bursts in a normal, unexaggerated fashion
posterior vibrating line marks the most distal extension of denture base. Fovea palatine is a clinically visible indentation
in the mucosa of the midline of the palate formed by the coalescence of several mucous gland ducts, which is unique to
humans. There is lot of difference of opinion on the location of fovea palatini and anterior vibrating line. According to
Sicher, fovea palatine is located just posterior to location of hard and soft palates. According to Swenson, vibrating line is
2 mm in front of fovea palatine. Silverman concluded that posterior palatal seal can be extended 8.2 mm distal to vibrating
line for retention and stability. In a study by Lye, the mean position of vibrating line is 1.31 mm behind fovea, but posterior
limit of denture can be extended an additional of 2 mm before soft tissue movement is sufficient to break the seal.
9. Answer: A (Ref. Complete Dentures, By Hugh Devlin, 2012)
Stability is “the resistance against horizontal movements and forces that tends to alter the relationships between the
denture base and its supporting foundation in horizontal or rotatory direction.”
10. Answer: D
11. Answer: B
12. Answer: B (Ref. Textbook of Complete Dentures, By Arthur O. Rahn, John R. Ivanhoe, Kevin D., 2009, page no. 192)
The retromolar pad is a nonkeratinized area of tissue and is a posterior continuation/extension of the pear-shaped pad. It
is also known as piriformis papilla. It is a small inclination going up and posteriorly and is bordered by muscles in the back
of the jaw.
13. Answer: C
14. Answer: B
15. Answer: A
16. Answer: A (Ref. Prosthodontic Treatment for Edentulous Patients: Complete Dentures, By Mahesh Verma, Aditi Nanda,
2017, page no. 10)
Masseter
The masseter is a rectangular muscle that originates from the zygomatic arch and extends downward to the lateral aspect
of the lower border of the ramus of the mandible. Its insertion on the mandible extends from the region of the second
molar at the inferior border posteriorly to include the angle. It is made up of two portions or heads: (1) the superficial
portion, which consists of fibers that run downward and slightly backward, and (2) the deep portion, which consists of
fibers that run in a predominantly vertical direction.
As fibers of the masseter contract, the mandible is elevated and the teeth are brought into contact. The masseter is a
powerful muscle that provides the force necessary to chew efficiently. Its superficial portion may also aid in protruding the
mandible. When the mandible is protruded and biting force is applied, the fibers of the deep portion stabilize the condyle
against the articular eminence.

https://t.me/DentalBooksWorld
Chapter 14 • Prosthodontics 1237

17. Answer: A
18. Answer: D
Retromolar pad
• Pear-shaped area found on each side of the distal end of the residual mandibular ridge
• It is important to avoid displacement of the retromolar pad while making the impression
• Used as guide to locate the occlusal plane of the mandibular denture, which must not be higher than half its vertical height
• Must be covered by denture to avoid its move backward
19. Answer: B
20. Answer: B
21. Answer: A
22. Answer: C
23. Answer: C
24. Answer: A

ARTICULATION AND JAW RELATION


1. Answer: A
2. Answer: D (Functional Occlusion in Restorative Dentistry and Prosthodontics By, Iven Klineberg, Steven Eckert, 2015,
page no. 230).
Hanau’s Quint
Rudolph L. Hanau proposed nine factors that govern the articulation of artificial teeth. They are
• Horizontal condylar guidance
• Compensating curve
• Protrusive incisal guidance
• Plane of orientation
• Buccolingual inclination of tooth axis
• Sagittal condylar pathway
• Sagittal incisal guidance
• Tooth alignment
• Relative Cusp Height “Laws of balanced articulation”
Hanau later condensed these nine factors and formulated five factors which are commonly known as Hanau’s Quint.
• Condylar guidance
• Incisal guidance
• Compensating curves
• Relative cusp height
• Plane of orientation of occlusal plane
3. Answer: C (Ref. Textbook of Complete Dentures, By Arthur O. Rahn, John R. Ivanhoe, Kevin D. Plummer, 2009,
page no. 137)
Occlusion rims
Definition: Occlusion rims are occluding surfaces constructed on record bases or permanent denture bases to be used in
recording jaw relations and for arranging teeth.
Requirements
• The position should be in the anticipated position of the artificial teeth
• It must be securely attached to the base
• The occlusal surface must be smooth and flat
• It should be contoured to support the lip and cheeks accurately
• All the surfaces should be smooth
Functions
• The occlusion rims are used to establish
–– The level of the occlusal plane
–– The arch form
–– Proper lips support

https://t.me/DentalBooksWorld
1238 Triumph's Complete Review of Dentistry

• Record the maxillomandibular relations


• For arrangement and setting of the teeth
• Selection of the teeth size (canine lines, midline, high and low lip lines)
Wax Rim characteristics
1. Wax rims are smooth and have a flat occlusal surface. They are about as wide buccolingually as denture teeth – wider in
the posterior, narrower in the anterior
2. The occlusal rim must be centered buccolingually over and parallel to the residual ridge crest.
3. Except for the anterior portion of the maxillary occlusal rim is labially oriented L.
Dimensions A – Maxillary arch
1. The anterior wax rim height is 20–22 mm
2. The posterior wax rim height is 16–18 mm (from the center of the ridge the height would be 8 mm)
3. The width of the anterior rim is approximately 3–5 mm
4. The width of the occlusal rim in the posterior region is approximately 8–10 mm
The occlusal rim is properly sealed to the baseplate without any voids.
The posteriors of the occlusion rims are cut at a 30-degree angle to the occlusal plane
Mandibular arch
l. The anterior wax rim height is 15–18 mm
2. The posterior wax rim height covers two-thirds of the retromolar pad
3. The width of the anterior rim is approximately 3–5 mm
4. The width of the occlusal rim in the posterior region is approximately 8–10 mm
5. The occlusal rim is properly sealed to the baseplate without any voids
4. Answer: B (Ref. Occlusion, By Hamish Thomson, 2016, page no. 220)
• Articulating paper will identify the interferences that prevent the desired working and balancing side contacts
• Harmony of upper working cusps on lower buccal and lingual inclines can be obtained by proper selective grinding
procedure
• The red color is usually used to mark usually the centric contact points, while blue is used for marking eccentric points
(very important)
• Articulation paper is available in a variety of colors (blue, red, green, orange, and black)
• Blue appears to be the most common color used
• Red is useful on highly polished surfaces of gold or composite resin
• Papers are available with different colored surfaces so that one arch is marked in red and the opposing arch is marked
in blue
Thickness of Selected Articulation Papers

XX thin Mynol 25 microns


X thin Mynol 63
Thin Mynol 100
Thick Mynol 350
Premium Mynol 100
Articulation paper Bausch 60
Articulation paper – extra thin Bausch 40
5. Answer: C (Ref. Textbook of Complete Dentures, By Arthur O. Rahn, John R. Ivanhoe, Kevin D. Plummer, 2009, page no. 181)
Using “m” sound: The patient repeats the letter “m.” When the lip just touches ask the patient to hold the jaws still. The
distance between the tip of the nose and the chin is measured (VDR).The occlusion rims are adjusted and again measured.
The second measurement should be 2–4 mm less than the first measurement (VDO).
6. Answer: A
7. Answer: C
8. Answer: A (Ref. Textbook of Complete Dentures, By Arthur O. Rahn, John R. Ivanhoe, Kevin D. Plummer, 2009, page no. 176)
Centric relation – An anteriorly superiorly braced position along the articular eminence of the glenoid fossa, with the
articular disc interposed between the condyle and eminence. The maxillomandibular relationship in which the condyles
articulate with the thinnest avascular portion of their respective discs with the condyle–disc complex in the anterior–
superior position against the articular eminences. It is used as a repeatable reference position for mounting casts in an
articulator; ideally the condylar position is also coincident with maximum intercuspation of the teeth. Centric relation

https://t.me/DentalBooksWorld
Chapter 14 • Prosthodontics 1239

record provides the orientation of mandibular to maxillary teeth in CR in the terminal hinge position, where opening and
closing are purely rotational movements.
Systems for recording centric relation
• Static recording (interocclusal checkout)
• Graphic recording (intraoral or extraoral Gothic arch tracings)
• Physiological/functional wax rims or wax cones during unguided/unassisted patient movement
• Cephalometric recording – cephalometric radiography to determine optimal position of the condyles
Recording technique
Bimanual technique (Dawson) single-handed technique – Accurately mounted casts depend on precise manipulation of
the patient’s mandible by the dentist. The condyles should remain in the same place throughout the opening–closing arc.
Trying to force the mandible backward will lead to downward translation of the condyles, and restorations made to such a
mandibular position will be in supraclusion at the try-in stage. The load-bearing surfaces of the condylar processes, which
face anteriorly, should be manipulated.
9. Answer: B
10. Answer: D
11. Answer: B (Ref. Techniques in Complete Denture Technology, By Duncan J. Wood, 2015, page no. 36)
All whip mix semi-adjustable articulators feature the ARCON design. ARCON is a contraction of the words for articulator
and condyle. In an ARCON-designed articulator, the mechanical fossa is fixed relative to the maxillary cast, making the
instrument more anatomically correct. Because of their ARCON design, Whip mix semi-adjustable articulators are ideal
for the study of occlusion and the movements of the temporomandibular joint.

WHIPMIX 2000 SERIES WHIPMIX 4000 SERIES


The Model #2240 Articulator – an open fossa design, features a fixed intercondylar width
of 110 mm, curved emenitia with a ¾' radius, fixed Bennett angle set at 7½ degrees, has immediate side shift capability,
and is interchangeable within the 2000 series.
The Model #2340 Articulator – an open fossa design, features a fixed intercondylar width of 110 mm, straight condylar
element with progressive side shift capability, and is interchangeable within the 2000 series.
The Model #3040 Articulator – a closed tracking design, features a fixed intercondylar width of 110 mm, progressive side
shift capability, and is interchangeable within the 3000 series.
The Model #3140 Articulator – has a modified tracking design, features a fixed intercondylar width of 110 mm,
progressive side shift capability, and is interchangeable within the 3000 series.
The Model #4640 Articulator – offers the most visibility of any Whip mix articulator available on the market. This is
made possible by a design that eliminated the crossbar between the upright posts. The Model #4640 Articulator features
a closed tracking design, a fixed intercondylar width of 110 mm, progressive side shift capability, and is interchangeable
within the 4000 series.
The Model #4641 Articulator, like the Model #4640, offers the most visibility of any Whip Mix Articulator available
on the market. The Model #4641 Articulator has a modified tracking design, a fixed intercondylar width of 110 mm,
progressive side shift capability, and is interchangeable within the 4000 series.
All whip mix semi-adjustable articulators are also available with the Quick Mount Magnet System.
12. Answer: A
13. Answer: C
14. Answer: A
15. Answer: B
16. Answer: B

https://t.me/DentalBooksWorld
1240 Triumph's Complete Review of Dentistry

OCCLUSION, SELECTIVE GRINDING, AND TROUBLESHOOTING


1. Answer: C (Ref. Removable Partial Dentures: A Practitioners’ Manual, 2015, page no. 60)
Oral candidiasis is a common opportunistic infection of the oral cavity caused by an overgrowth of Candida species,
the commonest being Candida albicans. The incidence varies depending on age and certain predisposing factors. There
are three broad groupings consisting of acute candidiasis, chronic candidiasis, and angular cheilitis. Risk factors include
impaired salivary gland function, drugs, dentures, high carbohydrate diet, and extremes of life, smoking, diabetes mellitus,
Cushing’s syndrome, malignancies, and immunosuppressive conditions. Management involves taking a history, an
examination, and appropriate antifungal treatment with a few requiring samples to be taken for laboratory analysis. In
certain high risk groups, antifungal prophylaxis reduces the incidence and severity of infections. The prognosis is good in
the great majority of cases.
2. Answer: D (Ref. Textbook of Prosthodontics, By Deepak Nallaswamy, 2017, page no. 231)
Group function of working side is indicated whenever the arch relationship does not allow the anterior guidance to do its
job of discluding the nonfunctioning side. The anterior guidance cannot do its job in conditions like Class 1 occlusion with
extreme over jet, Class 3 occlusion with all lower anterior teeth outside of upper, some end-to-end bites and anterior open bite.
Canine disclusion may provide a cuspid protected occlusion in parafunctional lateral movements that may be beneficial if
posterior teeth have significant bone loss, considerable occlusal wear, or number of cracks and if patient clenches or grinds
the teeth. Canine disclusion is preferred to group function the conditions as follows.
• If both canines on one side have little or no bone loss the first condition for the use of canine guidance is met. The
radiographs should show normal sized canine roots that have little evidence of bone loss. If canine occlude in lateral
movements toward the side in which these teeth are present, canine rise is a logical approach. If sound maxillary and
mandibular canines do not occlude in lateral movement, canine disclusion cannot be achieved. If posterior teeth have
little or no bone loss, group function is the objective. If posterior teeth have moderate to severe bone loss and canines
are essentially sound, canines may be restored to create canine disclusion or orthodontic positioning of canines can be
used to permit it.
• If one or both canines on one side have moderate to severe bone loss and guided tissue regeneration is not feasible,
canine disclusion should be discarded as an objective. If adjacent teeth have moderate to severe bone loss as do canines,
splinting to distribute the loading more evenly is necessary. If mobility is minimal selective grinding to smooth out
group function will suffice.
3. Answer: C
4. Answer: C
Please read this table thoroughly!! Very important!!

Symptom(s) Cause(s) Treatment


Discrete painful areas Pearls or sharp ridges of acrylic on Use disclosing material and relieve
impression surface
Pain on insertion and removal Denture is not relieved in the region Use the disclosing material and adjust in the
of undercuts region of “wipe off ”
Areas painful to pressure Faulty impression, damage to master Use disclosing material to accurately locate
cast, warpage of denture base, lack of area to be relieved
relief to active frena, nondisplaceable
mucosa over the bony
Pain on swallowing Over extended lower denture Determine extent and location of
overextension and relieve accordingly
Generalized pain over the denture Underextended denture base due to Extend denture to optimal available denture
bearing area overadjustment to the periphery support area
Lack of relief for frena, muscle Peripheral overextension resulting Relieve with aid of disclosing material
attachments, pinching of tissues from impression stage and design
between the denture base, and error
retromolar pad or tuberosity

https://t.me/DentalBooksWorld
Chapter 14 • Prosthodontics 1241

Symptom(s) Cause(s) Treatment


Sore throat, difficulty in swallowing Posterior palatal area is too deep Removal of existing seal and replacement
with new material is required
Pain on eating in the presence of Anterior or posterior premature Detect occlusal prematurity and adjust by
occlusal imbalance contacts, lack of balanced occlusion selective grinding
If the error is severe take new interocclusal
record and reamount
Pain lingual to lower anterior ridge If no overextension is present, look Detect deflective occlusal inclines of
for protrusive slide from centric posterior teeth and adjust by selective
occlusion grinding
Pain or inflammation of labial aspect Lack of overjet Reduce the overbite. If appearance is
of lower ridge disturbed then rearrange the incisor
Pain in the periphery of dentures Vertical dimension of occlusion is If VDO is less than 1.5 mm then rearrange
more teeth at proper VDO
Cheek and/or lip biting For cheeks: the functional width of For cheeks: Restore functional width of
the sulcus was not restored sulcus
For lips: poor lip support/inadequate For lips: Grind lower incisor to alter the
overjet incisal guidance
Tongue biting Teeth placed in the tongue space Remove lower lingual sulcus or reset teeth
more lingually
Pain at the posterior region of upper Distobuccal border of the upper Use disclosing material to accurately define
denture on opening denture is too thick and constraining area involved; relieve and polish
the coronoid process
Burning sensation over the upper Burning mouth syndrome seen in Correction of any denture faults, may
denture bearing areas middle age or elderly patient require multi-vitamin drugs, nutrition, and
medical advice
Beefy red tongue Vitamin B12/Folate deficiency Seek medical advice
Frictional lesions related to dentures, Xerostomia, side effect of prescribed Advise citrus lozenges or artificial saliva
complain of dry mouth drugs
Tongue thrusting. Empty mouth Neurological or psychological aspect; Difficult to manage ; seek medical advice
chewing seen in elderly can be drug related
Presence of herpetiform ulcers in the Herpes simplex or herpes zoster virus Suggest preventive remedy (e.g. acyclovir)
mouth but with medical advice
Clicking of the TMJ on opening and/ TMJ pain dysfunction syndrome may Careful correction of vertical dimension of
or closing mouth with or without be related to rapid change of VOD the dentures
tenderness
Painless erythema of mucosa related Denture-related stomatitis, ill-fitting Rest to tissue; correct denture problem
to support of upper denture may be denture with candidal infection using tissue conditioners and occlusal
accomplished with angular cheilitis pivots; for angular cheilitis advice antifungal
and antibacterial agents
Lack of peripheral seal Underextended borders in depth and Relining of the dentures
width
Inelasticity of the cheek tissue Consequences of ageing process, Border molding is done in increments using
scleroderma, mucous fibrosis softened tracing compound
Xerostomia – reduces ability to form Side effects of drugs, patient on Dentures designed to maximize retention
adequate seal radiotherapy, salivary gland disease and minimize displacing forces; artificial
saliva can be prescribed

https://t.me/DentalBooksWorld
1242 Triumph's Complete Review of Dentistry

Symptom(s) Cause(s) Treatment


Speech and eating difficulties Lower posterior placed lingually, Correct design faults, denture adhesives
occlusal plane too high, upper may be prescribed
posterior placed too far buccally,
lingual flange of lower convex,
reduced neuromuscular control
Denture rocking, gap between Deficient impression, damaged Reline if design is satisfactory, ensure areas
the periphery of flange and ridge, master cast, warped denture, of heavy contact between the denture and
occlusal errors subsequent to warpage overadjustment of impression surface, tissues and relieved before impression
residual ridge resorption, excessive making
relief
Overextended denture borders in Thickened lingual flange causes Relieve the overextension. Check borders
depth and width, slow rise of lower tongue to lift the denture, thick upper of the record rims and trial dentures at the
denture when the mouth is half open, and lower labial flanges may produce appropriate stages
line of inflammation at reflection of displacement during muscle activity
the sulcus tissues, deep postdam on
upper denture base
Poor fit to supporting tissue Poor impression Reline if design is satisfactory
Denture not in optimal space Molars on lower denture are placed Remove lingual cusp and lingual surface
lingually, posterior occlusal table too and repolishing reset or remake the
broad causes tongue biting, thick dentures, reshape lingual polished surface,
lingual flanges encroaching in tongue thin lower labial flange or remake the
space, excessive lip pressure to lower denture
anterior aspect, excessive pressure
from upper lip to the denture in
anterior aspect
Occlusal errors Uneven tooth contact causing tilting Adjust occlusion until even initial contact in
of the dentures centric position is obtained
Centric relation and centric occlusion Occlusion adjusted to coincide centric
does not coincide occlusion with centric position
Lack of freedom in centric Remount dentures on articulator and adjust
area of occlusal contact, allow freedom of
movement from the centric position, use
nonanatomic teeth if required
Ulceration labial to lower ridge Excessive overbite, lack or balance Reduce height of lower anteriors
and lower anterior tooth contact Remove the posterior teeth from dentures
causes tilting and soreness of lower and reset.
ridge. Usually requires teeth to be reset or
Last molar placed too far over the dentures remade
retromolar pad.
Occlusal plane not oriented
appropriately and masticatory forces
tend to move dentures over the
supporting tissues
Bony prominence covered by this Denture rocks over the prominence Use disclosing paste and relieve the denture
mucosa which may be covered with inflamed accordingly
tissues
Fibrous displaceable tissue Masticatory forces tend to cause Reline using low viscosity material and
the denture to sink and tilt into the provide many vents, maximize posterior
supporting tissues border seal

https://t.me/DentalBooksWorld
Chapter 14 • Prosthodontics 1243

Symptom(s) Cause(s) Treatment


Clicking of dentures Excessive VDO, occlusal interference, Patient education, relieve occlusal
may lack skill with new denture, loose interference adjust vertical or remake the
dentures denture
Difficulty in eating, denture moves on Unstable dentures Construct new dentures
supporting tissues
Jaws close too far Decreased VDO May increase up to 1.5 mm by relining or
else remake
Speech problems, cannot open mouth Excessive VDO Can remove up to 1.5 mm or else remake
widely, facial pain over masseter
Speech problems Cause may not be obvious Check the vertical dimension, check
positioning of the teeth, excessive palatal
contour
Gagging Loose dentures, thick distal border Construct new dentures
of upper denture, low occlusal plane,
palatal placement of upper posterior
teeth
Too much visibility of teeth Level of occlusal plane unacceptable, Reset teeth or remake
poor lip support
Creases at the corners of the mouth VDO decreased, labial fullness and Adjust correct tooth position, re-register
anterior tooth position inaccurate jaw relation
Color of denture base Denture base not characterized to Rebase with suitable material
individual needs
Appearance not satisfied Patient failed to comment during try- Accurate assessment of patient aesthetic
in, change from old denture to new is requirements, ample time with patient
sudden, influenced by relatives during try-in, use available evidence such as
photographs to assist

5. Answer: C
Selective grinding of teeth:
During lateral excursion
1. Balancing side contacts occurs on
• Buccal incline of upper lingual cusp (BUL) and lingual incline of lower buccal cusp (LLB).
• These are called mediotrusive or nonworking inclines.
2. Working side contacts occurs on
• Lingual inclines of upper buccal cusp (LUB) and buccal inclines of Lowe Lingual cusp (BLL).
• These are known as laterotrusive or working side inclines.

Mandible position Interferences


Balancing/Nonworking/Mediotrusive side BUL × LLB
Working/Laterotrusive side LUB × BLL
Centric relation Distal inclines of mandibular teeth and mesial inclines of
maxillary teeth

Deflective contacts Occlusal corrections


Posterior teeth: Reduce fossa or marginal ridge
• Deflective contact only in CR
• Deflective contact in both CR and eccentric relations Reduce both fossa and cusp

https://t.me/DentalBooksWorld
1244 Triumph's Complete Review of Dentistry

Anterior teeth:
• When incisal edge of lower anteriors touching the Reduce the lingual incisal edges
maxillary lingual fossa in CR only
• Incisal edge of lower anteriors touching the maxillary Deepen the lingual fossa of upper teeth and reduce the
lingual fossa both in centric and eccentric relations incisal edges of the lower teeth
• Deflective contacts occurring on working side BULLS law.
Upper Buccal cusp and Lower Lingual cusp are Shorten
without deepening the central fossae.
6. Answer: C
7. Answer: A
8. Answer: C
9. Answer: C
10. Answer: C
11. Answer: C
The transversal movement of the human mandible on the balancing side is characterized by a downward, forward, and
inward movement of the condyle. Projected onto the horizontal plane, an angle between the midsagittal plane and the
curve of the nonworking condyle can be seen. This angle is called the Bennett angle. The rigid connection established by
the body of the mandible results in a simultaneous rotation of the working side condyle. In addition, there is a lateral shift
of the mandible toward the working side called the Bennett movement. A review of the literature demonstrates that there
are controversies regarding a physiological or pathological amount of Bennett movement, the direction and pattern, the
reaction to occlusal equilibration, and to the multitude of influencing factors.
12. Answer: C
13. Answer: C
14. Answer: B Refer table for explanation.
15. Answer: A (Ref. Textbook of Prosthodontics, By Deepak Nallaswamy, 2017, page no. 243)
The Curve of Spee (called also von Spee’s curve or Spee’s curvature) is defined as the curvature of the mandibular occlusal
plane beginning at the tip of the lower incisors and following the buccal cusps of the posterior teeth, continuing to the
terminal molar.
The Curve of Spee is distinct from the Curve of Wilson, which is the upward (U-shaped) curvature of the maxillary and
mandibular occlusal planes in the coronal plane.
The Curve of Spee is basically a part of a circle (8-inch diameter) which has its circumference as the anterior ramus of
mandible. Ideally, it is aligned so that a continuation of this arc would extend through the condyles. The curvature of this
arc would relate, on average, to part of a circle with a 4-inch radius. It is the only anteroposterior curve of occlusion.
16. Answer: A
17. Answer: B
18. Answer: C
19. Answer: D

PHONETICS AND OVER IMMEDIATE DENTURE


1. Answer: A (Ref. Application of the Neutral Zone in Prosthodontics, By Joseph J. Massad, David R. Cagna, Charles J.
Goodacre, Russell A. Wicks, Swati A. Ahuja, 2017, page no. 114)

Voiceless speech sounds (created by air alone)


Fricatives Air is forced by tongue through a narrow aperture and is associated s, sh, th, f
Plosives with friction P, t, k
Affricatives Explosive release of air ch
A combination of the friction and explosive elements
Voice speech sounds (created by laryngeally produced noise)
Vowels Formed from continuous vocal cord vibrations; tongue, and lip a, e, i, o, u
Voiced consonants positions impart structural overtones b, d, j, m, q, r
A combination of air produced sounds and laryngeal tone

https://t.me/DentalBooksWorld
Chapter 14 • Prosthodontics 1245

Classification according to anatomic sound formation


Palatolingual Tongue is positioned just behind the maxillary incisor teeth with s
Tongue and hard palate the sides of tongue in contact with maxillary posterior teeth and t, d, n
Tongue and hard palate alveolar ridge k, g, ng
Tongue and soft palate Tongue is placed firmly against the anterior hard palate th
Linguodental Posterior dorsal tongue is raised to occlude with soft palate f, v
Labiodental Tip of tongue is placed between maxillary and mandibular teeth b, p, m
Bilabial Formed by lower lip contacting the incisal edge of the maxillary
incisor teeth
These sounds are formed between the lips
Lingua velar Consonants
The so-called back consonants or gutturals, /k/, /g/. It is produced by contact of the middle of the tongue with the soft palate.
In the case of the plosives /k/ and /g/, the palatopharyngeal valve is closed; pressure is build up behind the lingua velar contact,
and then released. In case of /ng/ (sing) the palatopharyngeal valve is open and the voiced airstream is nasally resonated.
Bilabial Consonants
Distinctive valving involves the lips: in three, lip closure and in two, lip rounding, for /p/, /b/, and /n/ intraoral breath pressure
is impounded behind closed lips, palatopharyngeal valve is closed, and the impounded air is suddenly exploded orally.
For production of the nasal /m/ the palatopharyngeal valve is open and the voiced air stream is resonated nasally.
Linguopalatal Consonants
The consonant /r/ is a voiced semivowel; the sides of the tongue touch the maxillary teeth. The tip of the tongue is often
pointed to an immediately postdental area, but at other times, it is the tongue blade that is arched highest toward the palate
while the tip points down. The palatopharyngeal valve is closed.
• Whistling when “S” is pronounced: The anterior part of the tongue is obstructed by the upper premolars making a
groove too large for the escape of air
• Lisping when “S” is pronounced: The air space is too small; thus the palatal part of the denture must be made thinner
• TH and T sounds undistinguished: There is inadequate interocclusal space, or the anterior teeth are too far lingual and
must be moved labially
• In the normal F and V sounds; the upper anterior teeth contact the lower lip at its highest point: If these sounds are
undistinguished, the upper incisors must be moved either vertically or horizontally to the proper position
2. Answer: D
3. Answer: A
4. Answer: C
5. Answer: B (Ref. Phonetic consideration in denture prosthesis, By Robert Rothman and Application of the Neutral Zone in
Prosthodontics, By Joseph J. Massad, David R. Cagna, Charles J. Goodacre, Russell A. Wicks, Swati A. Ahuja, 2017,page no.114)
Labiodental Sounds
F and V are representatives of the labiodental group of sounds. Formed by raising the lower lip into contact with the incisal
edge of the maxillary anterior teeth.
Clinical Significance
Upper anterior teeth are too short (set too high up), V sound will be more like an F
If they are too long (set too far down), F will sound more like a V.
6. Answer: C
7. Answer: B
8. Answer: A (Ref. Implants in Clinical Dentistry, By Richard M. Palmer, Leslie C. Howe, Paul J. Palmer, 2nd edition, 2011,
page no. 4)
The following guidelines should be used when selecting implant size and evaluating mesiodistal space for implant placement:
• The implant should be at least 1.5 mm away from the adjacent teeth.
• The implant should be at least 3 mm away from an adjacent implant.
• A wider diameter implant should be selected for molar teeth because of the high occlusal loads.
Implant diameter and length
• The implant diameter depends on the alveolar width, whereas the available bone height determines the implant length
• The implant length should be ≥ 10 mm, and a minimum diameter of 3.3–4.1 mm for the mandibular anterior while 4.1
mm for the maxilla
https://t.me/DentalBooksWorld
1246 Triumph's Complete Review of Dentistry

• The literature provides evidence of an increased failure rate for short implants − 7 and 10 mm
• Narrow diameter implants (2.5–3 mm) can be successfully used to treat narrow bone ridges although more long-term
studies are needed to compare narrow and conventional diameter implant outcomes
• In both the maxilla and the mandible, wide-diameter implants may provide additional support for removable partial
dentures. However, the use of wide-diameter implants for anchorage of removable partial dentures still requires
critical evaluation to assess whether wide-diameter implants affect the suprastructure design. Using standard-diameter
implants, the suprastructure may frequently be better designed and more comfortable for the patient
Implant number
• The two-implant overdenture therapy is a very reliable therapy for patients with an edentulous mandible
• Several authors hypothesize that it is appropriate to use two implants with an interconnector parallel to the hinge axis
and a resilient overdenture on an ovoid or round bar
• The bar’s purpose is to enhance free rotation during dorsal loading with twist-free load transmission to the implants
• Comparative prospective studies validate the benefit of two or four implants in the edentulous mandible
• Survival rates in the two-implant overdenture groups compared with four-implant overdenture groups appear to be
equivalent for patient satisfaction
• One ten-year trial displays no significant clinical and radiographic differences in patients treated with two or four
implants overdenture
• However, a mandibular overdenture with two implants and a bar has fewer complications
• There are no specific guidelines for the number of implants necessary to support a maxillary overdenture
• A minimum of four well-spaced implants is often recommended for an implant-supported and retained-overdenture. The
increased minimum of implants compared to the mandible is due to the softer bone and type of distribution of occlusal
forces in the maxilla. However, the use of only two maxillary implants may not compromise implant survival or patient
satisfaction. The most posterior implant should be inserted as far distally as possible to reduce the extension of cantilever
Implant position
• Although not standard, studies recommend four intraforaminal implants for cases of advanced atrophy or thin
mandibular ridges. For these instances, implants should be equidistant apart, or as an alternative one can mount a
cantilever-fixed prosthesis; in fact, the bar segments may become rather short, and short female bar retainers are
subject to frequent loosening or loss. The length of the bar segments can range from 15 to 25 mm. The total number of
intraforaminal implants distribution should be related to the shape of the ridge
• If a large or V-shaped anterior ridges exists, three to four implants will provide for a more favorable design of the bar
and the prosthesis. In presence of U-shaped mandibular jaw, two anterior implants could provide for a bar of adequate
length. A U-shaped mandible with large curvature allow for an adequate placement of four implants and a connecting
bar. Alignment of the implants in a rather straight line is not favorable for fixed prostheses
• The best anchorage design for the maxilla is four equidistance implants, but six implants for compromised bone.
• Positioning the implants in anterior maxilla, mesial to the first premolars enhances the stability of the overdenture. For
a design without palatal coverage, the consensus favors a minimum of four implants.
• In order to avoid dramatic changes in prosthetic design, one investigator recommends six implants
• Despite this recommendation, others clinicians’ implant prognosis were not compromised with the presence of
compromised quality and quantity of bone, off-ridge relations, or high applied forces, and palatal coverage

RELINING, REBASING, AND OTHERS


1. Answer: B
2. Answer: B
3. Answer: C
4. Answer: A
5. Answer: C
6. Answer: A
7. Answer: C
8. Answer: B
9. Answer: A
10. Answer: D
11. Answer: C
12. Answer: C and D

https://t.me/DentalBooksWorld
Chapter 14 • Prosthodontics 1247

13. Answer: C
14. Answer: A

TEETH SELECTION
1. Answer: A
2. Answer: C
3. Answer: C
4. Answer: B

ANSWERS: FIXED PARTIAL DENTURE

COMPONENTS OF FPD, RESIN MODIFIED BRIDGES


1. Answer: D (Ref. Contemporary Fixed Prosthodontics, By Stephen F. Rosenstiel, Martin F. Land, Junhei Fujimoto, 2015,
page nos. 365 and 712)

Resin bonded bridge is a minimally invasive fixed dental prosthesis that is luted to tooth structures, primarily enamel,
which relies on composite resin cements for retention.
Types
• Cantilever
• Fixed-fixed
• Fixed-movable
• Hybrid
Cantilever
• Involves the use of single retainer
• Abutment tooth maybe either mesial or distal
• Less expensive, but limited to replacing one
• Missing tooth
Fixed-fixed
• One or more retainers are placed on either side of the pontic
• Differential movement of abutments can result in bond failure
• This design of bridge is indicated where excursive movements on pontics cannot be avoided
Fixed-movable
• Design is in two parts, keyed together by a nonrigid attachment
• Connector which may be either ready- or laboratory-made
• Permits movement of the two parts relative to each other in vertical direction mainly
• Provides stress breaking action
• Should be used in short spans and where opposing proximal walls of abutment cannot be prepared parallel
Hybrid
• A combination of a conventional retainer at one end and a resin-bonded retainer at the other end of the pontic
• Indicated where one of the abutments is minimally restored, and a resin-bonded retainer is used at this site to conserve
tooth tissue
• The male part of the joint is often attached to the resin-bonded retainer to simplify maintenance when de-bond occurs
Rochette Bridge
• Wing-like retainers with perforations through them to enhance resin retention
• Macromechanical retention + silane coupling agent to produce adhesion to metal

https://t.me/DentalBooksWorld
1248 Triumph's Complete Review of Dentistry

Maryland Bridge
• An electrolytic etching procedure for nonprecious ceramic bonding alloys to provide a microporous surface that allows
micromechanical interlock with the cement
• Thinner wings and no perforations
Virginia Bridge (Lost Salt Technique)
• Salt crystals (150–250 μm) were incorporated into wax and removed in solution leaving cubic retentive pits
• Produces roughness on the inner surface of the retainer
• This was a time saving method and more retention is achieved compared to the technique of etching
Cast Mesh Fixed Bridge
• A net like nylon mesh is placed over lingual surface of abutment teeth on the cast
• It is then covered by wax, with the undersurface of the retainer becoming mesh like when retainer is cast
Advantages of resin-bonded bridges
• Minimal tooth preparation
• Beneficial in case of decay free teeth, large pulp chambers, slender lower incisors
• Little need to provide temporary crown
• Reduced chairside time; overall cost is minimal convertibility
• Less destructiveness
• Possible to upgrade to a conventional bridge without detriment to health of abutment teeth
• Supragingival margins
• Mandatory for RBB
• Soft tissue is undisturbed, facilitates plaque removal
• Acceptance by the patient
• Technique involves minimum of “injections” and “drilling”
• Rebonding possible
Disadvantages of resin-bonded bridges
• High failure rate
• More frequent debond as compared to conventional
• Plaque may trap underneath this de-bonded retainer, which can result in carious destruction if undetected
• Esthetics
• Problems can occur with incisal shine-through of metal if an opaque cement is not used
• Occlusal interferences
• Mostly retained by lingually placed metal flanges
• Not possible to reduce abutment teeth sufficiently
• Frequent problem when upper arch is involved
• Redistribution of space between pontic and abutment teeth
• Limited tooth replacement
• Small spans tend to be more successful than large ones
• No alignment correction
Indication for nonrigid connector
• The existence of pier abutment which promotes a fulcrum-like situation that can cause the weakest of the terminal
abutments to fail and may cause the intrusion of a pier abutment.
• The existence of the malaligned abutment, where parallel preparation might result in devitalization. Such situation can
be solved by the use of intracoronal attachment as connectors.
• Long span, FPD which can be distort due to shrinkage and pull of porcelain on thin sections of framework and thus,
affect the fitting of the prosthesis on the teeth.
• In the mandibular arch, FPD consisting of anterior and posterior segments, a nonrigid connector is indicated as the
mandible flexes mediolaterally during opening and closing strokes.
• Disparity in retentive capacity of the abutments.
Contraindication for nonrigid connector
• If the abutment presents significant mobility

https://t.me/DentalBooksWorld
Chapter 14 • Prosthodontics 1249

• If the span between the abutments is longer than one tooth, the stresses transferred to the abutment tooth under
soldered retainer would be destructive
• If the posterior retainer and pontic are opposed by a removable partial denture or an edentulous ridge while the two
anterior retainers are opposed by natural dentition
2. Answer: A
3. Answer: A
4. Answer: A (Ref. Contemporary Fixed Prosthodontics, By Stephen F. Rosenstiel, Martin F. Land, Junhei Fujimoto, 2015,
page no. 81)
Ante’s law is a concept given by Irvin Ante in 1926 which helps in determining the amount of support which has to be
taken to replace missing teeth which has been given in terms of pericemental area of the abutment teeth. This has been
adopted and reinforced by other authors – Johnston, Dykema, Shillingburg, and Tylman.
Definition: The sum of pericemental area of abutment teeth should be equal to or surpass that of teeth being replaced.
Definition according to GPT: “In fixed partial denture prosthodontics for the observation that the combined pericemental
area of all the abutment teeth supporting a fixed partial denture should be equal to or greater in pericemental area than
the tooth or teeth being replaced; as formulated for removable partial prosthodontics the combined pericemental area of
the abutment teeth plus the mucosal area of denture base should be equal to or greater than the pericemental area of the
missing teeth”
5. Answer: A
Pier abutment also known as an intermediate abutment is defined as a natural tooth located between terminal abutments
that serve to support a fixed or removable dental prosthesis.
6. Answer: B (Ref. Contemporary Fixed Prosthodontics, By Stephen F. Rosenstiel, Martin F. Land, Junhei Fujimoto, 2015,
page no. 81)
Ratios according to Ante’s law for replacement of missing teeth
• The optimum crown root ratio for the tooth which can be utilized as an FPD abutment is 2:3.
• Ideal crown root ratio should be 1:2 which is the Root Surface area should be double that of the crown surface area.
• But the ratio can be 1:1 which is the Root can be equal in area to that of the Crown present.
• Tooth support varies depending on length and shape of root and not just the surface area. Here are some of the cases
where the shapes of the roots determine the amount of support provided irrespective of the length or surface area it
covers.
• Long irregularly shaped and divergent roots offer great support.
• Roots with greater faciolingual dimension will make it a superior abutment to the roots which are circular in
cross-section.
• Short, conical, and blunted roots offer poor support, for example, a molar with divergent roots will provide better
support than molar with conical roots with little or no inter-radicular bone.
• Single rooted with elliptical cross-section will offer better support than the tooth with a circular cross-section. As
circular teeth will not give enough resistance to forces exerted on the tooth and in the case of an elliptical root the forces
are dispersed more efficiently thus giving better support.
7. Answer: A
8. Answer: B
9. Answer: B
10. Answer: D
11. Answer: C
12. Answer: C
13. Answer: A
14. Answer: C
15. Answer: A (Ref. Contemporary Fixed Prosthodontics, By Stephen F. Rosenstiel, Martin F. Land, Junhei Fujimoto, 2015,
page no. 843)
Connectors are those parts of a fixed partial denture (FPD) that join the individual retainers and pontics together. In most
of the cases stress concentration is found in the connectors of the prosthesis.
Nonrigid Connectors
• Precision attachments
• Key and keyway

https://t.me/DentalBooksWorld
1250 Triumph's Complete Review of Dentistry

• Tube and screw attachment


• Split pontic
• Cross pin and wing
Indications
• Malaligned abutments without common path of insertion
• Existence of pier abutments
• Long span FPDs, which can distort due to shrinkage of porcelain
• Distal abutment with questionable prognosis
• Presence of numerous mobile teeth which need to be splinted by fixed restoration
Contraindications
• Teeth with large pulp chambers
• Abutment with reduced clinical crown height
16. Answer: D
17. Answer: B
18. Answer: C
19. Answer: C
20. Answer: D

PRINCIPLES OF TOOTH PREPARATION


1. Answer: D
2. Answer: C
3. Answer: B
4. Answer: B
5. Answer: B
6. Answer: A
7. Answer: B

ALL CERAMIC, METAL CERAMIC RESTORATIONS AND FINISH LINES


1. Answer: D
2. Answer: (Ref. Contemporary Fixed Prosthodontics - E-Book, By Stephen F. Rosenstiel, Martin F. Land Junhei Fujimoto;
2015, page no. 184)
There are four types of finishing lines for full coverage restorations:
1. Knife edge
2. Chamfer
3. Shoulder
4. Beveled shoulder
Knife-edged Preparation
A knife-edge, or a feather-edge preparation that is basically designed so that as the tooth is prepared for zero cutting
results at the gingival termination.
Summary of knife edge preparations is as follows:
1. Little resistance to marginal distortion during firing of porcelain
2. Margin not always distinct
3. Poor control over placement of subgingival margin
4. Insufficient preparation in cervical area
5. No control over reduction of cervical tooth structure
6. Employed with long clinical crown lengths following periodontal surgery
The Chamfer Preparation
A chamfer, according to Boucher is “a marginal finish either curved or formed by a plane at an obtuse angle to the
external surface of a prepared tooth.”

https://t.me/DentalBooksWorld
Chapter 14 • Prosthodontics 1251

There are three different chamfer types of preparations:


1. Hybrid. Insert the chamfered stone about one-third of the depth of the stone and obtain a hybrid between a chamfer
preparation and an exaggerated knife-edge type of preparation.
2. Ski-sloped. Insert the chamfered stone into the radius of the instrument or half the depth of the stone; then a more
ideal type of chamfer preparation is developed.
3. Rounded shoulder. Insert the chamfered stone into its full diameter, the resulting type of chamfer preparation appears
to approximate a rounded shoulder.
3. Answer: A
The occlusal reduction is performed by removing the tooth portions between the orientation groves with the same bur.
After the functional cusp bevel is made, a no. 17L bur is used to smooth the planes of the occlusal reduction. Three vertical
groves are performed in the vestibular surface with a flat-end tapered diamond.
4. Answer: (Ref. Contemporary Fixed Prosthodontics - E-Book, By Stephen F. Rosenstiel, Martin F. Land Junhei Fujimoto,
2015, page no. 227)

Maxilla
Arch Mesiodistal Faciolingual Overall
Anterior tooth 10 10 10
Premolar 14 14 14
Molar 17 21 19
Isthmus – – 7
Box – – 7
Mandible
Anterior tooth 10 10 10
Premolar 16 12 14
Molar 24 20 22
Isthmus – – 12
Box – – 12

5. Answer: C
6. Answer: B
7. Answer: B
To ensure good
esthetics, substantial
tooth reduction is
necessary.
Buccal
Facial
Lingual Lingual
≥0.3 mm
≥0.5 mm ≥1.2 mm ≥0.6 mm
≥0.3 mm
≥1.2 mm ≥1 mm
≥0.5 mm
≥1.5 mm 1.3–1.7 mm 1.3–1.7 mm
0.8–1.2 mm

8. Answer: D
9. Answer: B
10. Answer: C (Ref. Metal ceramic crown indications, contraindications, advantages and disadvantages, By Varun, 2012)
Metal ceramic crowns or porcelain fused metal crowns are used to get both strength (due to metal) and esthetics (due to
ceramic) which make it an ideal type of crown for every tooth, be it Anterior or Posterior.

https://t.me/DentalBooksWorld
1252 Triumph's Complete Review of Dentistry

Indications of metal ceramic:


• The metal ceramic crown is indicated on teeth that require complete coverage, where significant esthetic demands are
placed on the dentist. E.g., anterior teeth.
• Extensive tooth destruction as a result of caries, trauma of existing previous restorations that precludes the use of more
conservative restoration.
• For superior retention and strength.
• An endodontically treated teeth in conjunction with a suitable structure. E.g., post and core.
• If all ceramic crown is contraindicated.
Contraindications of Metal Ceramic and Complete Ceramic Crowns
• Contraindications for the metal ceramic crown, as for all fixed restoration, include patients with active caries of
untreated periodontal disease.
• In young patients with large pulp chambers.
• Intact buccal wall.
• When more conservative retainer is technically feasible.
11. Answer: C (Ref. Contemporary Fixed Prosthodontics - E-Book, By Stephen F. Rosenstiel, Martin F. Land, Junhei Fujimoto,
2015, page no. 860)
Three quarter (¾) crown:
It is the most commonly used Partial Veneer Crown restoration, generally all the tooth surfaces except the buccal or the
labial surface is not included in the preparation. It preserves more of the coronal surface of tooth structure.
Since it does not cover the entire axial coronal surfaces, it tends to be less retentive and resistance than full veneer crown It
can be used for anterior or posterior teeth. It can be used as single restoration or as a retainer for short span bridge.
Uses
1. As a retainer for short span bridge
2. As a single restoration
3. As a splint in anterior teeth
For posterior teeth
1. Lost moderate amount of tooth structure with intact and well-supported buccal surface
2. Retainer for fixed partial denture
For anterior teeth
1. Suitable for teeth with a sufficient bulk
2. Retainer for FPD or splinting of anterior teeth.
Contraindications
1. Short clinical crown
2. High carries index
3. Extensive destruction
4. Poor alignment
5. Thin teeth
6. Long span bridge
7. Nonvital teeth
Tooth Preparation
• 1.5 mm on functional cusp (lingual), 1.0 mm on nonfunctional cusp (facial)
• Less than 0.5 mm on facial cusp tip if sufficient horizontal overlap
• 1.5 mm clearance
• Follow contours of opposing tooth
• Maintain contours of tooth being prepared
• Extend bevel into lingual embrasure
12. Answer: D
13. Answer: C
14. Answer: C
15. Answer: B
16. Answer: C
17. Answer: A

https://t.me/DentalBooksWorld
Chapter 14 • Prosthodontics 1253

18. Answer: A
19. Answer: D
20. Answer: C
21. Answer: A
22. Answer: A

d - destruction
ac - alveolar crest
fsb - final sulcus bottom
fcm - final crown margin

d – destruction, ac – alveolar crest, fsb – final sulcus bottom, fcm – final crown margin
• Between d and ac – X (distance to be found out)
• Between ac and fsb – biological width (standard = 2 mm)
• Between fsc and fcm – 1 mm (1 mm to prevent placement of crown margin too far subgingivally)
• So extrusion distance if the destruction is 2 mm beyond the ac (alveolar crest), = X + Biological width + 1 mm = 2 mm
+ 2 mm + 1 mm = 5 mm
Also,

If the destruction is Extrusion needed


1 mm 4 mm
3 mm 6 mm
4 mm 7 mm
(Clue – Add 3 mm to the destruction value given in the question)
23. Answer: A
24. Answer: A
25. Answer: A
26. Answer: A
27. Answer: B
Consider a central incisor of 25 mm, where root forms 14 mm and crown forms 11 mm. If the crown is broken and the
root is fractured beyond 3 mm, then if you do not do extrusion, proceed with crown lengthening alone. The new crown
length will be of 14 mm (normal old crown length 11 mm plus fractured compensation 3 mm for the root). This will
produce esthetic problems. So the treatment of choice is always Extrusion followed by Crown lengthening. So extrude the
root by 3 mm (for the fractured root segment 3 mm).
11mm 3mm

14mm
11mm

3mm
11mm
11mm 14mm

CEMENTATION OF FPD
1. Answer: A
2. Answer: C (Ref. Contemporary Fixed Prosthodontics, By Stephen F. Rosenstiel, Martin F. Land, Junhei Fujimoto, 2015,
page no. 774)
Polycarboxylate Cement
Sometimes called a semipermanent cement; polycarboxylate cement is a good choice for longer-term temporary
restorations or temporaries that require greater retention, such as stainless steel crowns, and for cementing some long-
term temporary orthodontic appliances. This type of cement is kind to the tooth tissues as well as the gingival tissues,
while possessing adequate retentive properties to hold the restoration or appliance in place during an extended period. It
has reasonably low-solubility properties and is easy to remove after cementation.

https://t.me/DentalBooksWorld
1254 Triumph's Complete Review of Dentistry

If the dentist is concerned about remaining cement on the preparation interfering with the bond of the permanent
cement, polycarboxylate would prove to be a good choice as it is easy to clean off of the tooth. Polycarboxylate is available
as a powder/liquid form, as well as a paste/paste automix syringe, and must be quickly and accurately mixed to avoid
manipulation past the working time of the material. This luting cement can be mixed to a thinner consistency if being used
for temporary purposes. Because of its pink/opaque color, this cement could show through some esthetic provisionals
and change the shade and the esthetics of the temporary. Some of these polycarboxylate temporary cements are resin
reinforced for added strength and improved properties.
3. Answer: D (Ref. Contemporary Fixed Prosthodontics, By Stephen F. Rosenstiel, Martin F. Land, Junhei Fujimoto, 2015,
page no. 449)
Electroplated Dies
Electroplated dies are the ones that are produced when an impression material is electroplated.
When a die is made in this manner, this process is known as electroforming. The impression materials which can be
electroplated are impression compound and elastomeric materials.
Advantages
No dimensional changes occur during electrodeposition of a metal.
It reproduces the impression accurately.
The die is tough and has good strength.
4. Answer: A
Marginal gaps greater than 75 μm may lead to accelerated cement washout and retention failure. In order to reduce the
cement margin thickness, several approaches have been suggested. A groove may be placed in the preparation or the
casting to act as an additional spacer or vent for the cement. In implant prostheses the casting is often thicker than on
natural teeth. As a result, a groove may be placed inside the casting, from the occlusal (incisal) to a few millimeters above
the margin. The cement seal may be reduced to almost one-half of its thickness with this technique. Another method to
reduce film thickness is the timing of the prosthesis insertion.
5. Answer: C
6. Answer: C (Ref. Contemporary Fixed Prosthodontics, By Stephen F. Rosenstiel, Martin F. Land, Junhei Fujimoto, 2015,
page no. 710)
Hue, Value, and Chroma. Three dimensions of color: The 3D-Master shade guide uses color science to communicate
information about the appearance of teeth with the three dimensions of color: hue, value, and chroma (saturation). Value
(lightness) describes overall intensity to how light or dark a color is.
7. Answer: B
8. Answer: D
9. Answer: C
10. Answer: B

TISSUE MANAGEMENT AND GINGIVAL RETRACTION


1. Answer: C
2. Answer: D (Ref. Contemporary Fixed Prosthodontics, By Stephen F. Rosenstiel, Martin F. Land, Junhei Fujimoto, 2015,
page no. 399)
1. Ferric sulfate → 13–20%
• Recommended packing time is 1–3 minutes
• Solutions of ferric sulfate above 15% are very acidic and can cause significant tissue irritation and postoperative root
sensitivity
2. Aluminum chloride → 5–25%
• It is one of the most commonly used astringents
• Time of application is 10 minutes
• Advantages include, least irritating for gingival tissues and has no systemic effects
3. Vasoconstrictors
• Racemic epinephrine (0.1%, 8%)
• This has systemic effects and may lead to epinephrine syndrome
• There is a risk of inflammation of gingival cuff and rebound hyperemia
3. Answer: D (Ref. Gingival Retraction Methods for Fabrication of Fixed Partial Denture: Literature Review, 2016)

https://t.me/DentalBooksWorld
Chapter 14 • Prosthodontics 1255

Mechanical method
Physically displaces the gingiva
1. Copper Band
• Carries impression material and displaces gingiva
• Impression compounds and elastomeric materials have been used
• Chemico-mechanical method of gingival retraction (Retraction Cord)
• It is a method of combining a chemical with pressure packing
2. Rubber Dam
• Used when limited number of teeth in one quadrant are being restored
• When preparations do not have to extend subgingivally
• Clamp should be blocked out
*Addition silicone should be avoided as rubber interferes with its settings
Mechanical–chemical
• Displacement/retraction cord used for mechanically separating tissue from peripheral margin
• Impregnated with chemical for astringent action and/or hemostasis as impressions are made
• Cord displaces the gingival tissues both laterally and vertically

ANSWERS: REMOVABLE PARTIAL DENTURE

RPD PART 1
1. Answer: A
According to the Kennedy Classification, the partially edentulous arch shown is a Class I, Modification 0. There are
bilateral posterior extension areas with no other modification (edentulous) space(s) present.

All class I RPDs have an axis of rotation. To be very specific, the axis of rotation will pass through the rigid metal closest
to the edentulous space that lies above the survey line (height of contour). If the design is correct and the laboratory and
clinical procedures are carried out properly, the axis of rotation should pass through the most distal rests on each side. The
axis of rotation for this case is indicated by the black line.

Class I and Class II RPDs always have an axis of rotation. Class III RPDs never have an axis of rotation. Class IV RPDs
seldom have an axis of rotation – some people consider that Class IV RPDs have an axis of rotation if all the incisors and
both canines are missing, creating an “anterior extension” area.
2. Answer: A
Color codes
Brown – metallic component
Blue – acrylic resin portion
Red – rest seat, areas of the teeth to be prepared, relieved, or contoured, tripod marks
Black – survey lines, soft tissue undercuts, wrought wire clasp, type of tooth.

https://t.me/DentalBooksWorld
1256 Triumph's Complete Review of Dentistry

3. Answer: C
The plating must end exactly at the survey line (height of contour).
If the plating ends below the survey line, there will be a space between the superior edge of the plating and the tooth
creating a food trap. In addition, the edge of the plating will stand away from the tooth and may be irritating to the tongue.
If the plating ends above the survey line, it will pre-empt the mesial rest. The axis of rotation will pass through the plating
instead of the rest. The I-bar will contact the tooth in front of this “new” axis of rotation and will engage the tooth when
the extension base moves toward the residual ridge. Also, since the plating will act like a rest on an inclined plane, there
will be the tendency to push the tooth toward the facial during function.
4. Answer: C
Although the exact space required is somewhat arbitrary, we generally feel that the opening should be at least 15 mm. If
the opening is less than 15 mm, the benefit gained from uncovering the tissues may be offset by the increased “sensitivity”
of the tongue to the multiple major connector borders.
5. Answer: A
Major connectors on the maxillary arch should be at least 6 mm from the gingival margin. On the mandibular arch, the
proximity of movable tissues will not permit this amount of distance. In general, mandibular major connectors should be
at least 3 mm from the gingival margin.
6. Answer: B
A. Incorrect. All of the clasps should be cast. Since there is no functional movement of a class III RPD, there is no need for
the stress-breaking effects of wrought wire clasps.
B. Correct. Lower molars, particularly second or third molars, frequently have no facial undercuts. In addition, the normal
drift of the teeth is to the lingual, accentuating lingual undercuts and minimizing facial undercuts.
C. Incorrect. Modified T-bars would be the retainers of choice if the undercuts were on the distofacial. I-bars could be
used but the tiny space between the I-bars and the denture base would create food traps and would make the plastic very
difficult to finish properly. In addition, the I-bars would tend to be very short and inflexible.
D. Incorrect. Only one arm of each clasp assembly should extend into an undercut (be retentive). The other arm must be
rigid to provide bracing or reciprocation and thus cannot extend into an undercut and provide retention.
7. Answer: B
Although the rests on teeth numbers 21 and 28 are indirect retainers, they function when the extension base attempts to
move away from the tissues (residual ridge)—not toward the residual ridge as would occur when the patient bites down.
These rests are called indirect retainers because they increase the effectiveness of the direct retainers (clasps) when there is
an attempt to dislodge the prosthesis.
8. Answer: A
The tooth structure removed during guide surface preparation will adversely affect the shape and contour of the rest seat
preparation. Thus, the guide surfaces should be created before rest seats are prepared.
9. Answer: A
Both the statements and the reasons are true. The rigid metal located above the survey line (height of contour) and closest
to the edentulous space is that portion of the rest where it joins the guide plate/minor connector. The tip of the clasp lies in
front of the axis of rotation and the clasp will be activated when the extension base moves toward the residual ridge. Thus,
the clasp must be flexible to provide stress relief for the abutment.
10. Answer: A
The purpose of tripoding is to allow re-orientation of the cast in the same position at a later date. Tripoding is based on
the geometric principle that three points determine a plane. If the vertical arm of the surveyor moves during the tripoding
process, the plane indicated by the marks would not be the same as that currently being used.
11. Answer: D
A. Incorrect. Cast clasps should not be used anterior to the axis of rotation.
B. Incorrect. B is a true statement but not the best answer. The most commonly used solution would be to use a WW
clasp. The WW clasp would provide stress-relief through its increased flexibility: an important attribute in this case since
the abutment tooth is in front of the axis of rotation.
C. Incorrect. C is a true statement but not the best answer. If the patient is an experienced RPD wearer and retention is of
minor importance, one might consider leaving the retentive arm off tooth #6. This would be more likely if esthetics were
a major consideration. A good option might be to include the WW clasp initially and check the need for the clasp after
delivery. This would be done by merely bending the clasp slightly away from the tooth and having the patient wear the
prosthesis for a few days. If the patient managed the RPD without retention from the clasp, it could be cut off.
D. Correct. D is the best answer. Both A and B are true statements.

https://t.me/DentalBooksWorld
Chapter 14 • Prosthodontics 1257

12. Answer: A
The statements are both true and are related. The tip of the clasp arm would lie in front of the axis of rotation and the clasp
would engage the tooth during functional movement of the extension base. Thus, the clasp needs to be flexible to provide
stress relief to the abutment.
13. Answer: D
A. Incorrect. The palatal plate major connector is selected because it provides maximum support and denture style
retention. Moving the posterior border forward negates both of these attributes.
B. Incorrect. There is no such relationship between the anterior and posterior borders of a maxillary palatal plate major
connector.
C. Incorrect. One of the primary reasons a full palate is used is to create “denture style retention.” Termination of the
major connector 10 mm anterior to the fovea palatini would place the bead on the hard palate and a “seal” would be very
difficult to achieve.
D. Correct. The border should be placed in the posterior palatal seal area and should be beaded. One of the primary
reasons a full palate is used is to create “denture style retention.” To do so, a “seal” must be created at the posterior border
by slightly depressing the soft tissues.
14. Answer: A
Crossing the midline at right angles seems to create a situation which is less noticeable to the tongue.
15. Answer: B
The rests on #19 and #28 are not indirect retainers. Rests on #21 and #27 are indirect retainers. The rest on #21 is further
from a line connecting the tips of the retentive arms on teeth numbers 19 and 28 and is thus more effective than the rest
on tooth #27. In reality, the rest on #27 could be omitted without appreciable loss of indirect retention.
16. Answer: D
Rests must be placed in prepared rest seats. This is particularly important on anterior teeth where, without prepared rest
seats, rests lie on inclined planes. Downward pressure on the rest will tend to push the abutment buccally.
Note: The rest could be a little wider incisogingivally and it could also be a little thicker. However, these problems pale in
comparison to the fact that there is no rest preparation.
17. Answer: B
The guide surface–guide plate contacts do not determine a definite path of insertion in this case. If the minor connectors
to the rests on the mesial occlusal surfaces of the premolars are in intimate contact with the abutments, they may help
to ensure a definite path of insertion/dislodgement. However, it is safest to place the tips of the I-bar retentive arms just
in front of the greatest mesiodistal curvature of the facial surface of the abutments. If the tips are placed at or behind
the greatest mesiodistal curvature, it would be possible for the RPD to move slightly upward and backward allowing the
retentive arms to escape the undercuts without flexing. Under these circumstances, the RPD would not exhibit retention.
18. Answer: A
All plastic RPDs lack rigidity and tend to encourage plaque collection. Also, most lack adequate rests. They should only be
used on a temporary basis or when the dentition is “terminal.”
19. Answer: B
Because tooth-supported RPDs do not exhibit movement toward the tissues during function, physiologic relief is not
necessary. Extension RPDs, on the other hand, do exhibit functional movement and do require physiologic relief. This
is accomplished as follows: first coat the guide plates with chloroform and rouge or another disclosing medium, seat the
framework in the mouth, and push toward the tissues over the extension areas; then relieve the guide plate with a fine
textured stone. Repeat as necessary. It is very important that the guide plates do not bind against the abutments during
functional movement – otherwise they will pre-empt the planned mesial rests.
20. Answer: C
3–4 mm is a minimum. Numerous studies have shown that placing the superior border of the major connector less than 3
mm away from the gingival margin leads to increased plaque collection and subsequent increased marginal inflammation.
21. Answer: D
Atleast 6 mm can almost always be obtained since one does not have the space limitations created by the floor of the
mouth – as in the mandibular arch. Depending on the anatomy of the arch and the width and strength requirement of the
elements of the major connector, more than 6 mm can often be obtained.
22. Answer: B
6–8 mm allows for adequate strength while allowing for maximum tissue exposure both over the palate and in the area of
the marginal gingiva.

https://t.me/DentalBooksWorld
1258 Triumph's Complete Review of Dentistry

23. Answer: A
Restoring what is missing is rather obvious. It should be equally obvious that the restoration will ultimately fail if what
remains is not preserved. While answers 2, 3, and 4 play a role in RPD design and treatment, they do not represent a
“philosophy” and are certainly secondary to “preservation and restoration.”
24. Answer: B
It is best to use balanced occlusion when a complete denture opposes a partially edentulous arch restored with a removable
partial denture.
25. Answer: D
Guide plates serve all the functions listed under A, B, and C. Therefore, the best answer is D (all of the above).
26. Answer: D
A. Incorrect. There is no information given that would lead one to believe that the anterior teeth should be plated. In this
particular case, plating the anterior teeth would definitely be the exception rather than the rule.
B. Incorrect. In the situation shown, an A-P palatal strap would be difficult to design. The space between teeth 3 and 6 is
much better suited to the use of a modified palatal plate – if it is done correctly.
C. Incorrect. Tooth #6 lies bodily in front of the axis of rotation and a cast circumferential clasp should definitely not be
used. If a clasp is needed or desired, it should be a wrought wire.
D. Correct. For many people, the tongue would be sensitive to the lack of symmetry of the major connector in the area of
the midline of the palate. It would be better if the posterior border crossed the midline at a right angle and then extended
backward more or less parallel to the residual ridge.
27. Answer: C
The all-metal base would be very strong even with very limited thickness. In order for plastic to extend beneath either
latticework or meshwork, the metal retentive network must be off the ridge by a millimeter or so. This would leave little
space for plastic external to the network. There simply is not enough interocclusal space for adequate thickness of both
plastic and metal. The all plastic base would be very weak because of the limited thickness and is therefore not indicated.
28. Answer: A
Partially edentulous arches often exhibit unusual occlusal planes due to malposed natural teeth. More often than not, the
final occlusal plane is a compromise necessitated by supereruption and drifting.
29. Answer: A
Because of the “green” residual ridges, open latticework is the best choice. Relining is easier after the tissues heal completely.
In fact, the other choices all involve some type of metal base and relining after healing is almost impossible.
30. Answer: B
A. Incorrect. Wrought wire clasps are commonly used on terminal abutments for extension RPDs.
B. Correct. Wrought wire clasps should not be used as embrasure clasps. It is difficult to bend the clasp accurately through
the embrasure and the slightest nick in the wire or wear from occlusion will predispose the clasp to breakage in the area
where it passes over the occlusal surfaces of the teeth.
C. Incorrect. Wrought wire clasps are frequently used on teeth that have indirect retainers on them. A good example would be a
class II modification 1 RPD where the anterior abutment on the tooth-supported side has a rest that acts as an indirect retainer.
If this tooth also has a clasp on it, the clasp should be wrought wire because the tooth lies bodily in front of the axis of rotation.
D. Incorrect. D is incorrect because C is incorrect.
31. Answer: A
When only one tooth is missing on a tooth-supported RPD, infrabulge retainers should be avoided. The descending and
ascending portions of the arms are so close together that they form food traps.
32. Answer: A
The statement is an accurate description of “physiologic adjustment.” Class III and Class IV RPDs do not require
physiologic adjustment since there is no movement of the prosthesis during function.
33. Answer: C
A. Incorrect. The type of clasp (retentive arm) is not an important consideration in determining whether or not
reciprocation occurs.
B. Incorrect. For reciprocation to occur, retentive and bracing components must contact the teeth at the same time.
C. Correct. For reciprocation to occur, retentive and bracing components must contact the teeth at the same time. As
the retentive tips pass over the height of contour, the rigid opposing elements must maintain contact with the abutments.
This issue of timing is critical in the concept of reciprocation. If the rigid elements only contact the teeth when the RPD is
seated, they will function for bracing but not for reciprocation.
D. Incorrect. Since B is incorrect, D is also incorrect.

https://t.me/DentalBooksWorld
Chapter 14 • Prosthodontics 1259

34. Answer: B
A. Incorrect. Clasps originating above the height of contour are suprabulge retainers, “supra” meaning above.
B. Correct. “Infra” means below so infrabulge clasps originate below the height of contour and approach the undercut
from a gingival direction.
C. Incorrect. Infrabulge clasps approach the undercut from below – from a gingival direction. Also, by definition, supra-
or infrabulge categories are based on a relationship to the “bulge” or height of contour, not to the amount of undercut.
D. Incorrect. An approach from above the occlusal surface would constitute “suprabulge.” Such an approach is also
impractical and nonsensical.
35. Answer: A
Modified palatal plate major connectors are used in maxillary Class II RPDs. Plating is not common but can be used.
36. Answer: D
A clasp assembly should have all of the attributes described in A, B, and C.
All the clasps mentioned, except the Akers clasp, approach the undercut from a gingival or infrabulge direction. “Akers” is
the old name for a circumferential clasp which is, of course, a suprabulge clasp.

RPD PART 2
1. Answer: D
A. Incorrect. Although A is possible, it is not the best answer because the stem of the question does not specify that the
canines are adjacent to the edentulous areas.
B. Incorrect. In tooth-supported RPDs, rests are usually placed immediately adjacent to the edentulous space(s).
C. Incorrect. Areas of opposing occlusal contacts should be avoided whenever possible.
D. Correct. In tooth-supported RPDs, rests are usually placed immediately adjacent to the edentulous space(s).
2. Answer: C
Atleast 3 mm is required for the space between the superior border of the major connector and the gingival margin and at
least 4 mm is required for the vertical height of the lingual bar. 3 + 4 = 7 mm minimum.
3. Answer: B
A. Incorrect. There is no relationship between the presence of tori and the selection of a metal base.
B. Correct. The major connector cannot extend below the height of contour of the tori and consequently, there is seldom
the 7 mm minimum vertical space needed for a lingual bar.
C. Incorrect. There is no relationship between tori and the selection of tube teeth.
D. Incorrect. It is certainly not the best answer. However, there is some logic for answer D. Since the tissue over the tori
is very thin and not tolerant to any vertical pressure, one could argue that additional indirect retainers could be used to
safeguard against tissue trauma under the major connector. However, B is really the best answer.
4. Answer: C
A. Incorrect. Indirect retainers do not function when the patient bites down. It does not matter whether the substance is
soft or hard.
B. Incorrect. Same as A.
C. Correct. Indirect retainers come into play after the patient bites down and then begins to open again. The sticky nature
of the material attempts to pull the denture base away from the residual ridge. The indirect retainers prevent the RPD
from simply rotating around a line connecting the tips of the direct retainers (clasps, retentive arms). They assist the direct
retainers in resisting dislodgement of the prosthesis.
D. Incorrect.
5. Answer: A
The anterior–posterior palatal strap has greater strength and rigidity because of its circular shape and because the metal
straps lie in several different planes.
6. Answer: D
A. Incorrect. A rest on #20 would be a primary rest, not an indirect retainer.
B. Incorrect. Tooth #21 would often have a rest on it and this rest would function as an indirect retainer. However, it
would not be the most effective indirect retainer.
C. Incorrect. This tooth would seldom have a rest in the partially edentulous situation shown. Even if it did, it would not
be the most effective indirect retainer.
D. Correct. The rest on #27 would be a very effective indirect retainer because it lies furthest from a line connecting the
tips of the direct retainers (clasps).

https://t.me/DentalBooksWorld
1260 Triumph's Complete Review of Dentistry

7. Answer: B
A. Incorrect. Due to the mesiolingual tilt of the tooth, it is unlikely that there will be a usable distolingual undercut.
B. Correct. The mesiolingual I-bar will work very well. It must be on the mesiolingual corner of the tooth and it must
emanate from the inferior border of the major connector so that it has adequate length to be flexible.
C. Incorrect. This is not the best selection unless there is a severe tissue undercut that will cause the mesiolingual I-bar to
stand out in the floor of the mouth and irritate the tongue.
D. Incorrect. Due to the mesiolingual tilt of the tooth it is unlikely that there will be a usable distolingual undercut. In
addition, there is no reason to use a WW clasp in this situation – no need for increased flexibility or stress-breaking.
8. Answer: D
A. Incorrect. The statement is true but is not the best answer. The dual path of insertion allows rigid metal to be placed in
an undercut.
B. Incorrect. The statement is true but is not the best answer. Rigid metal retention is not used exclusively in class IV
RPDs but certainly is commonly used in such situations.
C. Incorrect. The statement is true but is not the best answer. Rigid metal retention eliminates the need for clasps on
anterior teeth and produces excellent esthetics. This is the reason it is often used on class IV RPDs.
D. Correct. A, B, and C are all true statements.
9. Answer: A
If the edentulous areas of maxillary class III RPDs are larger, one is more apt to select an anterior–posterior palatal strap.
10. Answer: A
A. Correct. The axis of rotation passes through the rigid metal that lies above the height of contour and closest to the
edentulous space. This should be the rests, as all other rigid metal lies on inclined slopes. In the case shown, the axis of
rotation would pass through the rests on teeth numbers 20 and 27.
B. Incorrect. The axis of rotation is not determined by the type or location of the clasps.
C. Incorrect. The location of the minor connectors has no relation to the axis of rotation.
D. Incorrect. The axis of rotation passes through the primary rests and is important when the extension base moves
toward the tissues. The indirect retainers function when the extension base moves away from the tissues and are not
related to the location of the axis of rotation.
11. Answer: D
Because the metal exhibits very little dimensional change in the casting process, the bead can be very light. Its primary
purpose is to provide positive contact with the tissues so food does not get under the major connector. At the posterior
border of a palatal plate major connector, it may also help provide denture style retention.
12. Answer: D
A. Incorrect. Although A is a true statement, it is not the best answer.
B. Incorrect. With very few exceptions, the C+1 rule does apply to class III RPDs.
C. Incorrect. Although C is a true statement, it is not the best answer.
D. Correct. Statements A and C are both correct.
13. Answer: B
The horse-shoe maxillary major connector has the least strength and rigidity unless it is very bulky. For this reason, it is
the least desirable of all the maxillary major connectors.
14. Answer: B
The statement is false. Rests on maxillary incisors generally need to be placed as far gingivally as possible to avoid the
opposing occlusion. This is generally no further incisally than the junction of the gingival and middle thirds
15. Answer: C

RPD PART 3
1. Answer: A
Plating on maxillary RPDs occasionally causes problems with speech and frequently interferes with normal occlusion of
the maxillary and mandibular teeth.
2. Answer: B
After the surface of the wax-up is lightly covered with zinc stearate, the analyzing rod is passed over the surface of the wax.
The height of contour will appear darker than the surrounding powdered surface.

https://t.me/DentalBooksWorld
Chapter 14 • Prosthodontics 1261

3. Answer: B
A. Incorrect. The buccinator muscle is a relatively weak muscle and its fibers run anteriorly–posteriorly. It has very little if
any effect on the shape of the distobuccal corner of the denture base.
B. Correct. The masseter muscle is a powerful muscle whose fibers run more or less superiorly–inferiorly. Upon
contraction, it pushes the buccinator muscle and other tissues into the distobuccal corner of the denture base.
C. Incorrect. The internal pterygoid muscle has no effect on the distobuccal corner of the mandibular denture base.
D. Incorrect. The mylohyoid muscle has no effect on the buccal aspect of the denture base.
4. Answer: D
A. Incorrect. No! Never!
B. Incorrect. The I-bar would lie in front of the axis of rotation and is a cast clasp. It would not release during function and
would have the potential to torque the tooth.
C. Incorrect. Although the undercut is appropriate for an I-bar, a mesial rest cannot be used due to the occlusion and, if a
distal rest is used, the cast I-bar would lie in front of the axis of rotation.
D. Correct. Since the occlusion precludes the use of a mesial rest, the rest must be moved to the distal. Virtually any type
of clasp will place the retentive tip in front of the axis of rotation and flexibility of the clasp is of paramount importance –
thus, wrought wire.
5. Answer: B
A. Incorrect. The undercut is not appropriate for an I-bar. The I-bar must be placed just in front of the greatest mesiodistal
curvature of the facial surface. In this case, most of the undercut is behind the greatest mesiodistal curvature of the facial
surface.
B. Correct. Most of the undercut is behind the greatest mesiodistal curvature of the facial surface and that is where the
retentive tip should be placed. The vertical approach arm must be placed in front of the greatest mesiodistal curvature, so that
the clasp cannot simply move upward and backward without flexing. If this could happen, the clasp would not be retentive.
C. Incorrect. A circumferential clasp cannot be used because the undercut is adjacent to the edentulous area (and on the
same side of the tooth as the origin of the arm).
D. Incorrect. Same reason as C.
6. Answer: A
A. Correct. A mesial rest and I-bar is the best choice. The I-bar lies at or behind the axis of rotation and would release
during functional movement of the denture base. It is our first choice if the undercut is in the appropriate location, if an
infrabulge clasp can be used, and if there is no contraindication to placement of a rest on the mesio-occlusal surface.
B. Incorrect. Although a modified T-bar could work, it is not the first choice, especially when most of the undercut lies on
the mesiofacial surface.
C. Incorrect. This would be the “RPA” concept. The problem is that the originating portion of the clasp is rigid and would
lie above the survey line. It would function like a rest on an inclined plane – not a good idea!
D. Incorrect. Although a distal rest and WW circumferential clasp could work in this situation, it is not our first choice.
We would rather have a clasp arm that releases during functional movement (downward, tissueward) of the denture base.
7. Answer: C
A. Incorrect. The masseter muscle has no effect on the contour of the lingual flange.
B. Incorrect. The internal pterygoid muscle has no effect on the lingual denture flange.
C. Correct. The superior constrictor muscle affects the most distal portion of the lingual flange. If you are overextended in
this area, the patient’s complaint will often be that he/she has a sore throat.
D. Incorrect. The mylohyoid muscle affects the lingual flange but not at its most posterior (distal) aspect.
8. Answer: A
The plating will provide cross-arch stability and will also provide a mechanism for adding artificial teeth to the RPD
should any of the remaining teeth be lost. Also, plating will close up small, one tooth openings in the framework.
9. Answer: D
A. Incorrect. An I-bar (or any type of retentive arm) can be one component that helps establish 180-degree encirclement
of an abutment. Because retentive arms are flexible, they may not be quite as effective as some other components but they
still contribute.
B. Incorrect. Because of their location and their rigidity, minor connectors are very effective contributors to 180-degree
encirclement.
C. Incorrect. Because of their location and rigidity, guide plates are very effective contributors to 180-degree encirclement.
D. Correct. Open latticework (or any type of denture base retentive element) has no relation to the 180-degree encirclement rule.

https://t.me/DentalBooksWorld
1262 Triumph's Complete Review of Dentistry

10. Answer: C
A. Incorrect. The foot cannot extend above the survey line or it will not release during functional movements of the
extension base.
B. Incorrect. Same reasoning as A.
C. Correct. The foot should be totally below the survey line so that it will release during functional movements of the
extension base.
D. Incorrect. It does make a difference.
11. Answer: A
The statement is true – the primary reason for using an Oddo hinge clasp is to improve esthetics by moving the clasp arm
gingivally. Less frequently, Oddo hinge clasps may be used to prevent infrabulge arms from standing out in the vestibule.
While Oddo clasps are more commonly used in the maxillary arch for esthetic reasons, they may be used in either arch to
prevent infrabulge arms from standing out in the vestibule.
12. Answer: A
The altered cast impression captures the tissues of the edentulous ridges in relation to the way the framework fits in
the mouth (not on the cast). Hopefully, the two would be the same but that is not always the case. With an altered cast
impression, one also tends to avoid the overextension that is prevalent with a cast made from an alginate impression in a
stock tray.
13. Answer: A
It is always a good idea to establish the VDO before making a centric relation record. In theory, the VDO could be adjusted
on the articulator after the casts are mounted if a face-bow transfer has been done. However, since the type of face-bow
transfer we do is “arbitrary” and since some inaccuracy is to be expected, it is best to make the centric relation record at
the correct VDO whenever possible.
14. Answer: B
The frameworks should be tried in one at a time. First, each should be checked for fit. Then the occlusion should be
adjusted with each of the frameworks in place without the other. Finally, the occlusion is adjusted with both frameworks
in place.
15. Answer: A
After the protrusive record is made, the casts, prostheses, or record bases and the record are placed on the articulator.
The horizontal guidance controls on each side are loosened and rotated until the record and teeth or keys fit together as
accurately as possible.
16. Answer: A
The amount of posterior separation is affected by both the incisal guidance and the horizontal condylar guidance. The
separation is increased as both IG and HCG increase – the effect of IG is greater anteriorly and the effect of HCG is greater
posteriorly.
17. Answer: A
If the master cast has been damaged in the fitting of the framework to the cast, there is a high probability that the
framework will not fit in the mouth. The areas of damage should be the first areas adjusted if the framework does not fit.
Sharp fins are very prone to damaging the cast and thus also the very suspect if the framework does not fit – but sometimes
the damage to the cast caused by tiny fins is very difficult to see.
18. Answer: A
The impression and/or the cast may be inaccurate and thus be the cause of the problem. However, because of the time and
cost involved in making the framework, all other possible causes should be eliminated before making a new impression.
Scarring of the cast or sharp fins or protuberances on the framework might be correctable by analyzing the metal contacts
with the teeth with some type of disclosing medium and relieving the metal. Inaccuracies from very minor tooth movement
may also be corrected in this manner. If attempts to fit the framework to the mouth are unsuccessful, then one can assume
that either the impression or the cast is ina
19. Answer: A
As fewer teeth remain, the edentulous areas become larger. In general, a stock tray will perform adequately in areas where
teeth remain but relatively poorer in edentulous areas. This is one of the major reasons an altered cast impression is done.
However, when the number of teeth is very few and the edentulous areas are very large, it is sometimes easier to make a
custom tray and border mold before the impression is made. In this way, the altered cast procedure can usually be omitted.

https://t.me/DentalBooksWorld
Chapter 14 • Prosthodontics 1263

20. Answer: D
A, B, and C are all very valid reasons for selecting a linguoplate major connector.
21. Answer: C
A. Incorrect. The I-bar should release and thus, should create little danger of torquing the abutment.
B. Incorrect. Since the WW arm is flexible, it provides some stress-breaking effect and the danger of torquing the abutment
should be minimized.
C. Correct. Even though the RPA clasp has a mesial rest, the originating portion of the circumferential clasp will lie above
the height of contour and will act like a distal rest on an inclined plane. The tip of the cast arm will now lie in front of the
axis of rotation.
D. Incorrect. One of the other choices is much more potentially damaging to the abutment.
22. Answer: D
Both the ring clasp and the half and half clasp use lingual undercuts almost exclusively.
23. Answer: A
Ring clasps are used almost exclusively on lower molars that have drifted mesially and lingually. We would generally prefer
to use a mesiolingual I-bar, but in cases where there is considerable undercut below the abutment, a ring clasp is just about
the only alternative.
24. Answer: A
The primary indication for metal bases is limited vertical space. However, the ridges should be well-healed because relining
is almost impossible.
25. Answer: C
A. Incorrect. Too high. It is difficult to prepare an adequate rest seat and the potential for torquing forces on the abutment
teeth are increased. In the maxillary arch there would also be the increased potential for interference with the opposing
occlusion.
B. Incorrect. Too high. Same reasoning as for answer A.
C. Correct. This location places the rest seat and rest just above the cingulum but as low as possible so that potential
torquing forces on the abutment are minimized.
D. Incorrect. Totally nonsensical.
26. Answer: A
By definition, the external finish line is the external junction of the framework metal and the denture base plastic.

https://t.me/DentalBooksWorld
15 Physiology

SYNOPSIS

CELLULAR PHYSIOLOGY

CELL STRUCTURE
The cell is the functional unit of living organisms. There are approximately 100 trillion cells in a human.

Cell Membrane (Plasma Membrane)


The membrane that surrounds the cell is called cell membrane. It is approximately 7.5–10 nm thick. It is composed almost
entirely of protein and lipids. Carbohydrates are minimal and occurs almost in the form of glycoproteins or glycolipids.

Cell membrane component Percentage


Protein (major constituent) 55%
Lipids 42%
  Phospholipids – phosphoglycerides mainly 25%
  E.g., lecithin (phosphatidylcholine)
 Cholesterol 13%
  Sugar containing lipids – glycolipids (cerebroside, ganglioside) 4%
Carbohydrate – Glycocalyx (glycoprotein and glycolipids) 3%
Glycocalyx:
• Is a layer of carbohydrate on the surface of the plasma membrane of most eukaryotic cells. It is
made up of the oligosaccharide side-chains of the glycolipid and glycoprotein components of the
membrane and may include oligosaccharides secreted by the cell
• They are responsible for the fuzziness seen on an electron microscope
Functions:
• Attachment of cell
• Binding of antigens
• Prevents from cell injury
• Cell to cell recognition
The above percentage is in terms of area occupied. In terms of masses, the cell membrane contains
proteins and lipids in a 50:50 ratio.

LIPIDS IN CELL MEMBRANE


• Plasma membrane consists of a lipid bilayer coated on either side with a layer of protein, i.e., the cell membrane is a double-
layered film of lipids. The features are as follows:
• Major lipids are Phospholipids. Examples are:
–– Phosphoglycerides – Lecithin (phosphatidylcholine), phosphatidylethanolamine, phosphatidylserine, phosphatidylinositol
–– Sphingomyelin – Seen mainly in myelin sheaths

https://t.me/DentalBooksWorld
Chapter 15 • Physiology 1265

• Other lipids are cholesterol and glycolipids, i.e., sugar containing lipids (gangliosides and cerebrosides)
• Triglycerides are not present
• Membrane lipids are amphipathic, i.e., contain both hydrophobic and hydrophilic regions
PROTEINS IN CELL MEMBRANE
Most of the proteins are glycoproteins
Integral (Transmembrane or Intrinsic) Proteins
• These proteins are through and through, i.e., they run the entire length of the lipid bilayer
• Integral proteins are globular and amphipathic, consisting of two hydrophilic ends separated by an intervening hydrophobic
region that traverses the hydrophobic core of the bilayer
Peripheral (Extrinsic) Proteins
• Peripheral proteins are present on the surface of the membrane and do not penetrate all the way through
• They are located on both surface of membrane, outer and inner
• They are bound to the hydrophilic regions of the transmembrane proteins through electrostatic and hydrogen bonds

MARKER ENZYME (PROTEINS) FOR BIOLOGICAL MEMBRANES


Plasma (cell) membrane 5’-Nucleotidase, Adenylyl cyclase, Na, K, ATPase
Endoplasmic reticulum Glucose-6-phoshatase
Golgi apparatus Galactosyltransferase, Golgi mannosidase II, Sialyltransferase, GlcNAc
transferase I
Inner mitochondrial membrane ATP synthase
Biological membranes have self-releasing property, i.e., after an injury they repair themselves by rapid releasing

CYTOPLASM
The intracellular material, i.e., the material which is present inside the cell enclosed by the cell membrane excluding nucleus
is the cytoplasm.
Cytoplasm = Intracellular material (Organelles + Cytosol) – Nucleus
Metabolic pathways that occur in cytosol (outside the organelles):
• Glycolysis
• Glycogenolysis
• HMP Shunt (PPP Shunt)
• Glycogenesis
• Fatty acid synthesis
• Bile acid – Bile salt synthesis
• Cholesterol synthesis

Mitochondria
Mitochondria is known as the “powerhouse of the cell” because it contains enzymes for energy metabolism and ATP synthesis
They are composed of two membranes: An outer membrane and an inner membrane
Metabolic pathways occurring in mitochondria are:
• Fatty acid oxidation
• Electron transport chain
• Citric acid cycle/Krebs cycle/Tricarboxylic (TCA) cycle
• Oxidative phosphorylation
Metabolic pathways that occur initially in mitochondria followed by in cytosol:
Urea synthesis – initial two steps in mitochondria and remaining steps in cytosol
Gluconeogenesis – initially in mitochondria followed by in cytosol

https://t.me/DentalBooksWorld
1266 Triumph’s Complete Review of Dentistry

MARKERS OF CELL ORGANELLES


Organelle Markers
Mitochondria Inner membrane: ATP synthase, succinate dehydrogenase
Intermembrane space: Creatine kinase, adenylate kinase
Matrix: Glutamate dehydrogenase, pyruvate dehydrogenase, enzymes of TCA cycle, and
Beta-oxidation of fatty acids
Plasma membrane 5’-Nucleotidase, Adenyl cyclase, Na+/K+–ATPase
Lysosome Acid phosphatase
Golgi apparatus Galactosyl transferase, Golgi mannosidase II, Sialyltransferase, G1cNAc transferase I
Endoplasmic reticulum Glucose-6-phospahatase
Peroxisome Catalase
Cytosol Lactate dehydrogenase
Nucleus DNA
Ribosome High content of RNA

CYTOSKELETON
These are cellular elements which lend shape and form to the cell. All cells have cytoskeleton, a system of fibers that maintains
the structure of a cell and also permits it to change shape and move. The cytoskeleton is made up primarily of three filaments:
1. Microtubules
2. Microfilaments
3. Intermediate filaments

Microtubules • They are hollow fine tubules made up of a protein called tubulin
(diameter–25 nm) • They are made up of two globular subunits: alpha- and beta-tubulin
• Alpha and beta subunits form heterodimers and those dimers polymerize to form microtubules
• Microtubules are polar
• They function both to determine cell shape (providing structural support) and in a variety
of cell movements
Microfilament (actin): • They are filamentous structures, made of two F-actin (filamentous actin) strands that are
diameter: 7–9.5 nm coiled helically
• Actin is the most abundant protein in mammalian cells
• Actin is most often associated with muscle contraction
Intermediate filaments: • These are elongated fibrous molecules, with a central rod domain, an amino terminal head and
diameter: 10–12 nm a carboxy terminal tail
• Unlike microtubules or microfilaments, intermediate filaments are very stable and remain
mostly polymerized, not undergoing rapid assembly and disassembly and not disappearing
during mitosis, as do actin and many microtubules
At least four classes of intermediate filaments are found:
1. Lamins
2. Keratins
3. Vimentin like
• Desmin – In muscles
• Vimentin – In various mesenchymal cell
• Glial fibrillary acid (GFA) – In glial cells
• Peripherin – In neurons
4. Neurofilaments – In neurons
Force generated for… By...
Microtubules Kinesin, Dynein
Microfilaments Myosin

https://t.me/DentalBooksWorld
Chapter 15 • Physiology 1267

BODY FLUIDS
• Water constitutes about 60% of the body weight. Of this 60%, 40% is present in cell (intracellular fluid, ICF) and 20% is
present outside the cells (extracellular fluid, ECF)
• Again ECF (20%) divided into interstitial fluid (15% of the body weight) and plasma (5% of body weight)
• An adult weighing 70 kg contains 42 l of water. Of these 42 l, 28 l are present in ICF and 14 l are present as ECF
• Again the ECF is present as interstitial fluid (10.5 l) and blood plasma (3.5 l)

Extracellular fluid (ECF):


• Is divided into extravascular and intravascular compartments
• Extravascular = interstitial fluid + transcellular fluid. Transcellular fluids is a relatively small volume which includes CSF,
intraocular fluid, fluids in GIT, and potential spaces (joints, pleura, peritoneum)
• Intravascular = blood plasma
• One-third of total body water is extracellular, the remaining two-thirds are intracellular

Total body water (42 l)/60% of the body weight


Intracellular Fluid (28 l) Extracellular Fluid (14 l)
• 40% of the body weight • 20% of the body weight
• 67% of the total body water • 33% of the total body water
 – Interstitial fluid (10.5 l)
• 15% of the body weight
• 25% of the total body water
• 75% of ECF
 – Plasma (3.5 l)
• 5% of the body weight
• 8% of total body water
• 25% of the ECF

DYES USED FOR VARIOUS MEASUREMENT OF FLUID


Fluid volume Dye/Indicator used
Total body water (TBW) volume Deuterium oxide: D2O (most commonly used), tritium oxide (3H2O),
Aminopyrine, Antipyrine
Extracellular fluid (ECF) Inulin (most accurate), Sucrose, Mannitol, Sodium thiosulfate, Na(22),
I(125)-iothalamate
Plasma volume Evans Blue (T-1824), Serum albumin labeled with radioactive iodine-I (125)
Blood cell volume Cr (51) labeled RBC

TOTAL BLOOD VOLUME


• If you know plasma volume and hematocrit (% of blood volume that is made up by cells), total blood volume can be
calculated
• TBV = plasma volume × (100:(100 – hematocrit))
• E.g., hematocrit = 38, plasma volume = 3,500. Total blood volume is then:
–– 3,500 × (100:(100 – 38)) = 5,645 ml

COMPOSITION OF ECF AND ICF


Component Extracellular fluid Intracellular fluid
Na 142 14
K 5.5 150
Cl 103 4

https://t.me/DentalBooksWorld
1268 Triumph’s Complete Review of Dentistry

HCO3 28 10
Mg 1.2 58
Ca 2.5 0.0001
PO4 4 75
Protein (organic anion) 5 (2 g%) 40 (16 g%)
Predominant ions in ECF: Na, Cl, HCO3, Ca
Predominant ions in ICF: K, Mg, Organic anion (organic phosphate, protein)

Ribosomes 1. Free ribosomes – Not attached to other organelles; synthesize proteins used inside the cell
2. Bound ribosomes – Attached to the endoplasmic reticulum (ER); form rough ER; synthesize
proteins destined for use in the plasma membrane or for export from the cell
Endoplasmic reticulum Network of membranes that form flattened sacs called cisterns; arranged in parallel rows within
the cytoplasm of a cell; contains enzymes involved in a variety of metabolic activities
1. Rough (granular):
• Contains ribosomes
• Site of protein synthesis
2. Smooth (agranular):
• No ribosomes present
• Synthesizes certain lipids and carbohydrates
• Contains enzymes that release glucose into the bloodstream and inactivate or detoxify a variety
of drugs and potentially harmful substances, including alcohol, pesticides, and carcinogens
Golgi complex 1. Stack of 3–20 flattened membranous sacs (cisterns)
2. Within the cisterns, proteins are modified, sorted, and packaged into vesicles for transport to
different destinations
Lysosomes 1. Membrane-enclosed vesicles that form in the Golgi complex
2. Contain digestive enzymes
3. Function in the digestion of worn-out organelles (autophagy) and self (autolysis)

MEMBRANE TRANSPORT
Transport Across The Cell Membrane:
Process Type Description Examples
Simple diffusion Passive Movement of particles through Movement of carbon dioxide
the phospholipid bilayer or out of all cells; movement of
through channels from an area sodium ions into nerve cells as
of high concentration to an they conduct impulse
area of low concentration – that
is, down the concentration
gradient
Channel-mediated Passive Diffusion of particles through a Diffusion of sodium ions into
passive transport membrane by means of channel nerve cells during a nerve
(facilitated diffusion) structures in the membrane impulse
(particles move down their
concentration gradient)
Osmosis Passive Diffusion of water through Diffusion of water molecules
a selectively permeable into and out of cells to
membrane in the presence of at correct imbalances in water
least one impermeable solute concentration

https://t.me/DentalBooksWorld
Chapter 15 • Physiology 1269

Facilitated diffusion Active Diffusion of particles through a Movement of glucose


membrane by means of carrier molecules into most cells
molecules; also called carrier
mediated passive transport

Phagocytosis Active Movement of cells or other Trapping of bacterial cells by


large particles into a cell by phagocytic white blood cells
trapping it in a section of
plasma membrane that pinches
off to form an intracellular
vesicle; type of endocytosis
Pinocytosis Active Movement of fluid and Trapping of large protein
dissolved molecules into a cell molecules by some body cells
by trapping them in a section of
plasma membrane that pinches
off to form an intracellular
vesicle; type of endocytosis
Exocytosis Active Movement of proteins or other Secretion of the hormone
cell products out of a cell by prolactin by pituitary cells
fusing a secretory vesicle with a
plasma membrane

Types of Transport Across Cell Membrane


Carrier protein involved Energy required Concentration gradient
Simple diffusion No No Along (Downhill)
Osmosis (Diffusion of H2O) No No Along (Downhill)
Facilitated diffusion Yes No Along (Downhill)
Active transport Yes Yes Against (Uphill)
Exocytosis No Yes –
Endocytosis
 Pinocytosis No Yes –
 And
 Phagocytosis
 Receptor Yes Yes –
 Mediated
 Endocytosis

CIRCULATORY SYSTEM

COMPOSITION OF BLOOD
• Plasma 55% of volume
• Formed elements 45% (RBCs, WBCs, platelets)

PLASMA
• The liquid part of blood
• Clear straw-colored fluid
• Plasma consists of liquid solvent – Mostly water and solutes without the formed elements
• 93% water and 7% solutes

https://t.me/DentalBooksWorld
1270 Triumph’s Complete Review of Dentistry

Serum = plasma with clotting factors removed


Plasma proteins (8% of blood)
Most proteins in blood do not readily pass through capillaries into interstitial spaces
1. Albumins (with other proteins) contribute to viscosity, osmotic pressure and blood volume helps buffer the blood,
transports many solutes by binding to them: e.g., drugs, penicillin, pigments, fatty acids, bile salts
2. Globulins – some are antibodies, part of immune system, some help transport solutes some involved in clotting
3. Fibrinogen soluble precursor to fibrin = framework for clotting

Formed Elements
About 45% of whole blood erythrocytes (RBCs) – most, 45%, of formed elements leukocytes (WBCs)
Erythrocytes
• Main function is to carry oxygen to cells
• Also deliver some carbon dioxide to the lungs
• Most abundant of the three types of formed elements – 99% of formed elements; ~5.5 mil/μl (μl = mm3)
• Equivalent to 2.5 trillion blood cells in the whole body
Biconcave disc – Thin center, thick edges 7.5 μm diameter, 2.0 μm thick
• High surface/volume ratio
–– Greater efficiency of gas exchange
–– Area of all RBCs in body =>football field for gas exchange
• Flexible
–– Easily deforms to fit through narrow capillaries

RBCs are packed full of hemoglobin molecules


In each RBC are 200–300 million hemoglobin molecules; each hemoglobin molecule can combine with 4O2 = oxyhemoglobin
Therefore, each RBC can carry ~1 billion O2 molecules
Leucocytes
• Slightly larger than RBCs = 8 μm diameter
• Are the only “true cells” of the formed elements
• They retain their organelles, including nucleus
• Large, irregular, lobed nucleus
• Relatively few are found circulating in blood: 4,000–11,000/mm3 or 1% of blood
• Numbers are misleading since they do most of their work outside the blood vessels
• Function as a part of the immune system to protect the body
• Attack and destroy bacteria and pathogens
• Remove dead cells and tissues
• Most WBCs are motile by amoeboid motion (pseudopodia)
• They squeeze out of capillaries into tissue spaces
White blood cells are the cells that help the body fight infection. There are a number of different types and subtypes of white
blood cells, each having different roles to play.
The three major types of white blood cells are:
1. Granulocytes
2. Monocytes
3. Lymphocytes
Granulocytes
There are three different forms of granulocytes:
1. Neutrophils
2. Eosinophils
3. Basophils
https://t.me/DentalBooksWorld
Chapter 15 • Physiology 1271

Granulocytes are phagocytes, that is, they are able to ingest foreign cells such as bacteria, viruses, and other parasites.
Granulocytes are so called because these cells have granules of enzymes which help to digest the invading microbes.
Granulocytes account for about 60% of our white blood cells.
Thrombocytes (Platelets)
• Not whole cells
• Small, irregular shaped cell fragments
• 2–4 μm diameter
• Second most abundant formed elements
• Average 250,000/μL
• Range: 150,000–350,000/μL
• Number varies depending on site of collection
• No gender differences
Diverse Functions
1. Secrete vasoconstrictors in small vessels, vasoconstriction can be maintained by chemicals released by platelets that begin
to accumulate at the site of damage
2. Promote hemostasis – stopping the flow of blood by platelet plug and/or clotting
3. Stimulate formation of clot dissolving enzymes to remove clots no longer needed – clots are not permanent, after repair the
clot is removed by chemicals activated by platelets. Platelets have a short life span: ~1–2 weeks

ABO SYSTEM
Blood type Antigens Antibodies produced Can receive blood from Can donate blood to
A A Anti B A, O A, AB
B B Anti A B, O B, AB
AB A and B Neither A, B, AB, and O AB
(Universal Recipient)
O None Both O A, B, AB, O (Universal Donor)

BLOOD GROUP INHERITANCE


Mother/Father O A B AB
O O O, B O, B AB
A O, A O, A O, A, B, AB A, B, AB
B O, B O, A, B, AB O, B A, B, AB
AB A, B A, B, AB A, B, AB A, B, AB

PLASMA COMPATIBILITY TABLE


Donor/Recipient O A B AB
O + + + +
A + +
B + +
AB +

https://t.me/DentalBooksWorld
1272 Triumph’s Complete Review of Dentistry

CLOTTING FACTORS
Factor Name
I Fibrinogen
II Prothrombin
III Tissue factor or thromboplastin
IV Calcium
V Proaccelerin (Labile factor)
VII Proconvertin (Stable factor)
VIII Antihemophilic factor A Antihemophilic globulin
IX Antihemophilic factor B Plasma thromboplastin component Christmas factor
X Stuart–Prower factor
XI Plasma thromboplastin antecedent Hemophilia C Rosenthal syndrome
XII Hageman factor
XIII Fibrin stabilizing factor Laki–Lorand factor

COAGULATION CASCADE
Factor Other name(s) Pathway Characteristic
Prekallikrein (PK) Fletcher factor Intrinsic Functions with HMWK and factor XII
High molecular weight Contact activation cofactor; Fitzgerald, Intrinsic Co-factor in kallikrein and factor XII
kininogen (HMWK) Flaujeac Williams factor activation, necessary in factor XIIa
activation of XI, precursor for bradykinin
(a potent vasodilator and inducer of
smooth muscle contraction)
Fibrinogen Factor I Both  
Prothrombin Factor II Both Contains N-term. gla segment
Tissue factor Factor III Extrinsic  
Calcium ions Factor IV Both  
V Proaccelerin, labile factor, accelerator Both Protein cofactor
(Ac-) globulin
VI (same as Va) Accelerin Both This is Va, redundant to Factor V
VII Proconvertin, serum prothrombin Extrinsic Endopeptidase with gla residues
conversion accelerator (SPCA),
cothromboplastin
VIII Antihemophilic factor A, antihemophilic Intrinsic Protein cofactor
globulin (AHG)
IX Christmas Factor, antihemophilic factor Intrinsic Endopeptidase with gla residues
B, plasma thromboplastin component
(PTC)
X Stuart–Prower Factor Both Endopeptidase with gla residues
XI Plasma thromboplastin antecedent Intrinsic Endopeptidase
(PTA)
XII Hageman Factor Intrinsic Endopeptidase
XIII Protransglutaminase, fibrin stabilizing Both Transpeptidase
factor (FSF), fibrinoligase

https://t.me/DentalBooksWorld
Chapter 15 • Physiology 1273

CLOTTING CASCADE
Coagulation Cascade

Intrinsic pathway Extrinsic pathway

XII Tissue factor


XI
VII
IX
VIII
Common
aParal pathway Prothrombin
Thromoboplastin Time
Time
X
V
Ca++
Lipids

Prothrombin (II) Thrombin

Fibrinogen (I) Fibrin clot (XIII)

EXCRETORY SYSTEM

Distal Convoluted Tubule (DCT) • This segment of the renal tubule is lined with simple cuboidal epithelium and has no
brush border
• Actively reabsorbs sodium and chloride
• It is relatively impermeable to water, but in the presence of antidiuretic hormone
(ADH) its permeability to water increases, making urine concentrated
• It secretes ammonium ions and hydrogen ions
• Forms part of the juxtaglomerular apparatus
Thick Ascending Loop • In this part of the loop, Na, K and Cl is actively reabsorbed
• It is impermeable to H2O
• This segment makes the urine less concentrated
• The Loop of Henle reabsorbs 10–20% sodium and chloride and 10% of the filtered water
Thin Descending Loop • Reabsorbs water (H2O) passively
• It is impermeable to sodium (Na)
• It allows the urine to be concentrated, the urine in the TDL is hypertonic
Proximal Convoluted Tubule • PCT reabsorbs two-thirds of the filtered Na or (65–80% of Na) and H2O
(PCT) • It reabsorbs all of the glucose and amino acids
• Glucose is reabsorbed via Na+-Glu cotransporter
• It also reabsorbs a fraction of the bicarbonate, potassium, phosphate, and calcium
• It secretes ammonia, which functions as a buffer for secreted H+. It also secretes
creatine, which is used to access the function of the kidney
Collecting Duct • In the presence of ADH, water is reabsorbed
• Reabsorption of Na ions
Bowman’s Capsule Collection of filtrate
Renal Corpuscle Produces a filtrate of blood that must be modified as it passes through the convoluted
tubules and nephron loop

https://t.me/DentalBooksWorld
1274 Triumph’s Complete Review of Dentistry

RESPIRATORY SYSTEM

LUNG VOLUMES
Tidal volume (TV): Volume of air inhaled or exhaled with each breath during normal breathing (0.5 l)
Inspiratory reserve volume (IRV): Maximal volume of air inhaled at the end of a normal inspiration (3 l)
Expiratory reserve volume (ERV): Maximal volume of air exhaled at the end of a tidal volume (1.2 l)
Inspiratory capacity (IC): Maximal volume of air inhaled after a normal expiration (3.6 l) (TV + IRV)
Functional residual capacity (FRC): The volume of gas that remains in the lung at the end of a passive expiration (2–2.5 l or
40 % of the maximal lung volume) (ERV + RV)
Residual volume (RV): The volume of gas remains in the lung after maximal expiration (1–1.2 l)
Total lung capacity (TLC): The maximal lung volume that can be achieved voluntarily (5–6 l) (IRV + ERV + TV + RV)
Vital capacity (VC): The volume of air moved between TLC and RV (4–5 l) (IRV + ERV + TV)
Multiplying the tidal volume at rest by the number of breaths per minute gives the total minute volume (6 l/min). During exercise
the tidal volume and the number of breaths per minute increase to produce a total minute volume as high as 100–200 l/min

Oxygen Hemoglobin Dissociation Curve


• Sigmoid shape of the oxy-Hb dissociation curve results from the allosteric interactions of the globin monomers that make up
the hemoglobin tetramer as each one binds O2.
• Multiple factors can affect the affinity of Hb for oxygen, thus causing the curve to shift to the left (increased oxygen affinity)
or to the right (decreased O2 affinity).
Higher Hb-O₂ affinity (left shift)
100% lower CO₂
higher pH
lower temperature
80%
Percent saturation

60%
Reduced Hb-O₂ affinity (right shift)
higher CO₂
40% higher pH
higher temperature

20%

0%
0 10 20 30 40 50 60 70 80 90 100
P₂ (mm Hg)

• O2 is released in the tissues where the pO2 is low. In this situation Hb has a low affinity for oxygen. Other factors may cause a
further reduction in Hb affinity for oxygen – i.e., a lower oxygen saturation for a given pO2.
• Factors which result in shifting of the oxygen-dissociation curve to the right include increased concentration of pCO2,
acidosis, raised temperature, and high concentrations of 2,3 diphosphoglycerate (2,3 DPG). These factors, in effect, cause the
Hb to give up oxygen more readily.

GASTROINTESTINAL SYSTEM

GASTROINTESTINAL HORMONES
• Enteroendocrine system regulates the digestive process through the secretion of gastrointestinal hormones.
• The lower part of the alimentary canal contains endocrine cells called Paracrine cells or Histocrine cells.
• These cells secrete Gut polypeptides or GI hormones.
• The GI hormones control the motility of the alimentary canal and secretion of digestive glands.

https://t.me/DentalBooksWorld
Chapter 15 • Physiology 1275

Some of the important GI hormones are as follows:


Gastrin: It is secreted by the G cells of the intestinal wall and upper gastric wall. It stimulates the secretion of gastric juice and
insulin.
Cholecystokinin–Pancreozymin: It is secreted from the I cells of the duodenum. It stimulates pancreas for secretion and gall
bladder for bile secretion.
Secretin: It is secreted by the S cells of the small intestine. It stimulates the pancreas for bicarbonate secretion. It is secreted
when the stomach becomes acidic.
Motilin: The EC cells of duodenum secrete motilin to increase intestinal motility.
Vasoactive intestinal polypeptide (VIP): Secreted from the gut to inhibit gastric secretion and motility.
Neurotensin: Secreted from ileum to enhance blood flow in the intestine and to inhibit motility.
Gastric inhibitory peptide (GIP): Secreted by the K cells of the duodenum and jejunum to stimulate insulin secretion and
inhibition of gastric secretion. It is stimulated by the presence of fat and glucose in the intestine.
Somatostatin: Secreted by the D cells of pancreas. Stimulated by acid in the stomach. It is a GH inhibiting hormone.

Hormone Secreted by Target(s) Releasing stimuli Actions


Gastrin G cells of Stomach Histamine secreting Peptides in lumen Increases acid secretion in
and parietal cells of stomach and gastric motility
stomach
Cholecystokinin Endocrine cells of Gall bladder; Partially digested Gallbladder contraction;
(CCK) small intestine pancreas; gastric proteins and fatty acids in inhibits gastric emptying;
muscle duodenum stimulates secretion of
pancreatic enzymes
Secretin Endocrine cells of Pancreas; stomach Acid and partially Stimulates bicarbonate
small intestine digested proteins and secretion and pepsin
fatty acids in duodenum release; inhibits gastric acid
and small intestine secretion, bile ejection
Motilin Endocrine cells of Smooth muscle Fasting Stimulates gastric
upper small intestine of Antrum and contractions (hunger pangs)
duodenum

1. Hormones that affect the stomach: The first 4 (Gastrin, CKK, Secretin, GIP)
2. The inhibitory hormone: Somatostatin
3. Hormones that affect intestinal motility: The bottom 3 (NO, VIP, and Motilin)

SECRETIONS OF THE GIT


Saliva 750–1,500 ml of saliva secreted/day
Gastric juice 1,200–1500 ml/day – acidic in nature
Pancreatic juice 500–800 ml/day – alkaline in nature
Bile juice 800–1,200 ml/day – alkaline in nature
Succus entericus 1,500–1,800 ml/day – alkaline in nature

ENDOCRINE SYSTEM
PITUITARY GLAND
• Pituitary gland is also called hypophysis or master gland; it is located below the hypothalamus in sella turcica, which is a part
of the sphenoid bone in the middle cranial fossa
• It is connected to the hypothalamus by the pituitary stalk
• The gland is small in size and weighs 0.5 g

https://t.me/DentalBooksWorld
1276 Triumph’s Complete Review of Dentistry

Anterior Lobe (Adenohypophysis) Clinical importance


Acidophil cells Somatotrophs (produce growth hormone, GH) Hypersecretion:
Children – Gigantism
Adults – Acromegaly
Hyposecretion:
Children – Dwarfism
Adults – Acromicria
Basophil cells Corticotrophs (release adrenocorticotropic hormone, Hypersecretion – Cushing’s disease
ACTH) Deficiency – Hypoadrenalism
Thyrotrophs (release thyroid stimulating hormone, TSH) Hypersecretion is noted in Hypothyroidism
Hyposecretion is seen in Hyperthyroidism
Mammotrophs (produce prolactin, PRL) Deficiency occurs in Sheehan syndrome
Hypersecretion occurs in prolactinoma
Gonadotrophs (release luteinizing hormone, LH, and Hyposecretion – Hypogonadism
follicle-stimulating hormone, FSH) Hypersecretion – Infertility
Posterior Lobe (Neurohypophysis)
Oxytocin: A hormone that stimulates contractions during Hyposecretion occurs in Asperger syndrome
labor.
Antidiuretic hormone: A hormone that stimulates water Hyposecretion – Diabetes Insipidus
re-absorption in the kidneys. Hypersecretion – SIADH (Syndrome of
inappropriate ADH)
Intermediate Lobe
Melanocyte-stimulating hormones (MSH) Hypersecretion – Pigmentation
Hyposecretion – Obesity

THYROID GLAND
It is located in the anterior region of the neck around the larynx and trachea below the thyroid cartilage of the larynx, and
extend inferiorly to the level of 5th–6th cervical vertebrae
1. Epithelial cells (follicular cells) Follicles produce and secrete the Hyperthyroidism occurs due to adenoma and
thyroid hormones T3 and T4 autoimmune reaction in Grave’s disease
Hypothyroidism occurs in children as Cretinism and
as Myxedema in adults
2. Parafollicular cells (C-cells) Calcium regulatory hormone No significant clinical effects with hyper- and
(calcitonin) hyposecretion

PARATHYROID GLAND
They are four in number and are located behind the thyroid gland and control the amount of calcium in our blood and bones
1. Parathormone (PTH) Hyperparathyroidism results in hypercalcemic states
Hypoparathyroidism causes tetany

PANCREAS
Pancreas produces both exocrine and endocrine secretions
1. Alpha cells Glucagon Glucagon helps insulin maintain normal blood glucose by working in
the opposite way of insulin
2. Beta cells Insulin Decreased secretion causes diabetes mellitus
3. Delta cells Somatostatin Somatostatin is secreted to maintain a balance of glucose and/or salt in
the blood
4. Gamma cells Pancreatic polypeptide Helps control water secretion and absorption from the intestines

https://t.me/DentalBooksWorld
Chapter 15 • Physiology 1277

ADRENAL GLAND
They are two in number and are present above the kidney
Adrenal cortex Mineralocorticoids Aldosterone maintains the body salt and water balance
Glucocorticoids Cortisol regulates body metabolism
Adrenal androgens Dehydroepiandrosterone and Testosterone
Adrenal Medulla Catecholamines Adrenaline, noradrenaline, and dopamine

GASTROINTESTINAL SYSTEM
Regulatory Substances of GIT
GASTROINTESTINAL HORMONES
Gastrin Secreted from G-cells of the GI mucosa Increases H+ secretion and helps in growth of the
gastric mucosa
Cholecystokinin Secreted from I-cells of duodenum and Controls gall bladder sphincter, increases HCO3
jejunum secretion and gastric emptying
Secretin Secreted from S cells of duodenum Increases pancreatic and biliary HCO3 secretion.
Decreases H+ secretion
Gastric Inhibitory Peptide Secreted from duodenum and jejunum Increases insulin secretion and decreases gastric
H+ secretion

GASTROINTESTINAL PARACRINES
Somatostatin Secreted from Cells of GI mucosa Inhibits release of GI hormones and H+ secretion
Histamine Secreted from Mast cells of the gastric mucosa Increases H+ secretion and potentiates gastrin and
vagus stimulation

GASTROINTESTINAL NEUROCRINES
Vasointestinal peptide (VIP) Secreted from mucosal neurons and Relaxes smooth muscle, stimulates
smooth muscles of GI HCO3, and inhibits H+ secretion
Gastrin releasing protein (Bombesin) Secreted from vagus Stimulates gastrin release from G cells
Enkephalins Secreted from mucosal neurons and Inhibit intestinal secretion of fluid and
smooth muscles of GI electrolytes

GASTROINTESTINAL SECRETIONS

Secretions Composition Regulation Functions


Saliva Na+, CL−, K+, HCO3−,α- Parasympathetic (VII and • Lubrication with mucin
Amylase, Lingual Lipase, IX) and sympathetic system • Carbohydrate digestion with salivary
Kallikrein amylase or ptyalin
• Fat digestion with lingual lipase
• Lysozyme provides antibacterial action
Gastric HCl, pepsinogen, Intrinsic Parasympathetic system (X) HCl helps in the breakdown of proteins into
secretion factor polypeptides through activation of pepsin
Pancreatic HCO3−, Pancreatic lipase, Secretin, cholecystokinin, Trypsin – breakdown of protein to peptides
secretion amylase, and proteases and parasympathetic system Chymotrypsin – Catalyzes protein breakdown
(Trypsin, chymotrypsin, Pancreatic lipase – lipolysis of triglycerides to
carboxypeptidase, Elastase, monoglycerides
Nuclease, and collagenase)
Bile Bile salts, bilirubin, Cholecystokinin and Helps in reduction of surface tension of lipids
cholesterol, and phospholipids parasympathetic system by emulsification and aids in fat absorption

https://t.me/DentalBooksWorld
1278 Triumph’s Complete Review of Dentistry

MULTIPLE CHOICE QUESTIONS

CELL PHYSIOLOGY
1. Sequence of vesicle transport is
A. ER – Cis – Trans – Cell membrane B. ER – Trans – Cis – lysosome
C. Cis – ER – Trans – Cell membrane D. ER – lysosome – Trans – Cis
2. Ineffective osmoles is
A. Na+ B. K+
C. Urea D. All of the above
3. Most accurate measurement of extracellular fluid volume (ECF) can be done by using
A. Mannitol B. Sucrose
C. Aminopyrine D. Inulin
4. For sodium–potassium pump, the coupling ratio is
A. 2:3 B. 1:1
C. 1:4 D. 3:2
5. Glycophorin is present in
A. Hepatocyte B. Enterocyte
C. Lymphocyte D. RBC
6. Resting membrane potential is mainly due to
A. K+ B. Na+
++
C. Mg D. Cl−
7. Defect in collagen formation is seen in
A. Scurvy B. Hunter’s syndrome
C. Marfan’s syndrome D. All of the above
8. Which of the following is/are extracellular matrix protein?
A. Collagen B. Fibronectin
C. Laminin D. All of the above
9. The process by which fusion of part of a cell membrane occurs is/are
A. Cell division B. Endocytosis
C. Virus replication D. All of the above
10. Which of the following is not true about microtubules?
A. Charged B. GTP not required
C. Dynamic instability D. Polarity
11. Which parts of the cell cycle are fixed in duration?
A. G2 B. S
C. M D. All of the above
12. Intracellular sorting and packing is done for
A. Golgi apparatus B. ER
C. Cytoplasm D. Ribosome
13. Lipids and proteins interact in membrane by
A. Covalent bonds B. Hydrogen bonds
C. Hydrophobic interactions D. Both hydrophobic and covalent interactions
14. Protein synthesis occurs in
A. Golgi bodies B. Smooth ER
C. Lysosomes D. Rough ER
15. Function of phospholipids in cell membrane is
A. Transduction of signals B. Cell to cell variation
C. DNA replication D. Transmembrane preparation of protein

https://t.me/DentalBooksWorld
Chapter 15 • Physiology 1279

16. Predominant extracellular ions are


A. Na+ B. Cl−

C. PO4 D. All of the above
17. What is the marker for cell membrane?
A. Na+K+ ATPase B. Sialyltransferase
C. LDH D. G6PD
18. Which of the following is true about Na+K+ pump?
A. Hypercalcemia causes arrest in Na+K+ pump B. Increase in intracellular Na+ increase action potential
+
C. K is pumped against the gradient D. 2K+ are exchanged with 5Na+
19. Viscosity of synovial fluid depends upon
A. Glucuronic acid B. Hyaluronic acid
C. N-acetyl galactosamine D. N-acetyl glucosamine
20. Which of the following is true about sodium?
A. Normal serum level is 135–145 mEq/l B. Daily intake is 150 mmol of NaCl
C. Major protection is extra cellular D. All of the above
21. Transition temperature of lipid bilayers of cell membrane is increased by
A. Saturated fatty acids B. Cholesterol
C. Unsaturated fatty acids D. Hydrocarbons
22. Keratin of skin and nail differ because
A. Vander Waal bond B. Hydrogen bond
C. Disulfide bond D. Covalent bond
23. Exocytosis is
A. Extrusion of cell-bound vesicles B. Intrusion of liquid particles
C. Intrusion of solid particles D. All of the above
24. Uronic acid level in urine is elevated in which of the following?
A. Niemann–Pick’s disease B. Mucopolysaccharidosis
C. Tyrosinosis D. Maple syrup urine disease
25. D2O (Deuterium oxide) is used to measure the volume of
A. Total body water B. Blood
C. Intracellular fluid D. Extracellular fluid
26. Which of the following is true about Na+-K+ pump? It is
A. Electrically neutral B. Pumps out one Na+ for one K+
C. Involves ATPase activity D. It can move Na+ in and out of cell
27. Which important component of cell wall has a carbohydrate moiety?
A. Cholesterol B. GM2 Gangliosides
C. Phosphoglyceride D. Triacylglycerol
28. Which of the following is true about body water?
A. ECF volume can be determined by dilution methods B. 10% is intracellular water
C. Water constitutes 60% of the body weight D. Plasma volume constitutes 10% of the body water
29. RBC membrane is maintained by
A. Collagen B. Elastin
C. Spectrin D. Laminin
30. The following is true regarding Golgi apparatus A/E
A. Trans is secretory end B. Nonpolarized structure
C. Situated near nucleus D. All of the above
31. Triple helix is seen in
A. Elastin B. RNA
C. Collagen D. DNA

https://t.me/DentalBooksWorld
1280 Triumph’s Complete Review of Dentistry

32. True statements are


A. Extracellular water constitutes about 40% of body weight
B. Total plasma constitutes about 10% body weight
C. Totally body water constitutes 60% of body weight
D. Most intracellular water is the musculoskeletal system
33. Markers of plasma membrane are
A. Adenylyl cyclase B. Galactosyltransferase
C. Glucokinase D. ATP synthase
34. Binding site present on Beta unit of Na+-K+ pump is
A. ATP B. Glycosylation
C. Na+ D. K+
35. Which of the following membrane has the highest protein content per gram tissue?
A. Plasma membrane B. Myelin sheath
C. Inner mitochondrial membrane D. Outer mitochondrial membrane
36. True regarding Na+-K+ pump is
A. Heterodimer–heterogeneous B. Heterodimer–homogeneous
C. Homodimer–homogeneous D. Homodimer–heterogeneous
37. Osmotic pressure of 1 mol of idea solute relative to pure water is
A. 22.4 atm B. 6.5 atm
C. 2 atm D. 4 atm
38. What is effect on the membrane when extracellular concentration of K+ is decreased?
A. Decrease magnitude of RMP B. Increase negativity of the membrane
C. Decrease negativity of the membrane D. All of the above
39. Addition of PUFA in plasma membrane causes
A. Increase in fluidity of membrane B. Membrane becomes rigid
C. No change in fluidity of membrane D. Decrease in fluidity of membrane
40. The correct sequence of a cell cycle is
A. G0-M-G2-S-G1 B. G0-G1-S-M-G2
C. G0-G1-S-G2-M D. G0-G1-G2-S-M
41. K+ homeostasis in human is
A. Actively secreted in the distal tubule B. It maintains the Na+ K+ ATPase activity
+
C. Most of the K  are intracellular D. Most of the K+ are extra cellular
42. The emeiocytosis or reserve pinocytosis requires which ion?
A. K+ B. Na+
++
C. Mg D. Ca++
43. The enzyme marker for Golgi apparatus is
A. Galactosidase B. Peroxidase
C. Catalase D. Galactosyltransferase
44. Membrane integrity of RBC is due to
A. Hemoglobin B. G-Protein
C. Ankyrin D. Spectrin
45. Regarding transport of Ca++ across a membrane, which of the following are true?
A. It is a passive mechanism B. Ca++ calmodulin binding
C. It is symport D. Requires hydrolysis of ATP
46. Which of the following is true about lipid bilayer of cell wall?
A. Symmetrical arrangement of cell wall components B. Not made up of amphipathic lipids
C. Asymmetrical arrangement of cell wall component D. Lateral diffusion of ions
47. Intracellular binding site on Na+ K+ pump is
A. PO4 B. Ouabain
C. Na+ D. ATP

https://t.me/DentalBooksWorld
Chapter 15 • Physiology 1281

48. In cell membrane, following are true except


A. Lipids are symmetrical B. Lipids are regularly arranged
C. None D. Protein displayed laterally
49. Osmoreceptors are present in the
A. Left atrium B. Ventricle
C. Anterior hypothalamus D. Internal carotid artery
50. Active transport across the cell membrane is mediated by
A. Na+-K+ ATPase B. G-proteins
C. Channel protein D. Carrier protein
51. Marker enzyme of mitochondria is
A. Glutamic dehydrogenase B. Na+-K+ ATPase
C. No specific enzyme D. Lactate dehydrogenase
52. Which of the following is true about mitochondria?
A. Isolation is done by hydrolysis in alkaline pH B. Protein synthesis
C. Fatty acid synthesis D. All of the above
53. Clathrin is used in
A. Cell to cell adhesion B. Plasma membrane
C. Receptor mediated endocytosis D. Exocytosis
54. Which of these is not a part of extracellular matrix?
A. Fibronectin B. Laminin
C. Collagen D. Integrins
55. Osmotic adaptation is A/E
A. In brain cells B. Due to osmolytes
C. Protects against large H2O shift D. Due to urea and glucose mainly
56. Osmoreceptor is located at which site?
A. Left atrium B. Ventricle
C. Anterior hypothalamus D. Internal carotid artery
57. Which of the following is the true statement about Ca2+ transport across membrane?
A. Maintain intracellular Ca2+ 10,000 times higher than the extracellular Ca2+
B. Calcium calmodulin mediated
C. It is symport
D. Requires hydrolysis of ATP
58. Plasma membrane is mainly composed of
A. Phospholipid B. Protein
C. Cholesterol D. Carbohydrate
59. Which of the following is the true statement about ions composition in body?
A. Phosphorous and Mg++ are major ions intercellularly B. Na+, Cl− principal ions in ECF
C. Kidney tightly regulates Na+, K+, Cl− composition D. All of the above
60. True about facilitated diffusion are A/E
A. Does not require energy B. Occur in direction of concentration gradient
C. Facilitated by charge of molecule D. Occur in direction opposite to electrical gradient

GENERAL PHYSIOLOGY
1. The water content of lean body mass is about
A. 30 ml/100 g B. 50 ml/100 g
C. 70 ml/100 g D. Highly variable
2. Body mass index is calculated as
A. Weight in pounds by height in meters
B. Weight in kg by height in meters
C. Weight in kg divided by square of height in meter squared
D. Weight in kg divided by body surface area

https://t.me/DentalBooksWorld
1282 Triumph’s Complete Review of Dentistry

3. The most abundant cation in ICF is


A. Sodium B. Potassium
C. Magnesium D. Calcium
4. The most abundant anion in ECF is
A. Bicarbonate B. Chloride
C. Phosphate D. Protein anion
5. ECF volume is determined by
A. Plasma [Na] B. Plasma protein concentration
C. The amount of sodium in the ECF D. All of the above
6. The body fluid compartment that contains more osmotically active particles (in relation to other fluid compartments
in the same individual) is
A. Intracellular fluid B. Plasma
C. Interstitial fluid D. None
7. What percentage of osmolality of plasma in a healthy, well-hydrated individual is attributable to sodium and its
accompanying anions?
A. 30% B. 50%
C. 70% D. 90%
8. Which one of the following contributes least to the osmolality of plasma?
A. Glucose B. Proteins
C. Sodium D. Urea
9. The osmolar concentration of sodium in normal human plasma is approximately
A. 275–295 mOsm/l B. 135–145 mOsm/l
C. 240–250 mOsm/l D. 95–110 mOsm/l
10. In the steady state, the value of which of the following variables is the same in ICF and ECF?
A. pH B. Osmolality
C. Concentration of proteins D. Number of osmoles
11. Which compartment does the term “sucrose space” refer to?
A. Extracellular fluid (ECF) B. Interstitial fluid (ISF)
C. Intracellular fluid (ICF) D. Plasma
12. The volume of distribution of intravenously administered sucrose in a healthy well-hydrated 70-kg man is about
A. 3.5 l B. 10.5 l
C. 14 l D. 28 l
13. In a healthy adult, ECF volume constitutes what fraction of body weight?
A. 10% B. 20%
C. 30% D. 40%
14. Which of the following substances can be used as a marker for the ECF compartment?
A. Nonmetabolizable sugars B. Glucose
C. Radio-iodinated albumin D. D2O
15. Which of the following markers is incorrectly matched with its volume of distribution?
A. Antipyrine – total body water B. Inulin – ECF volume
C. Evans blue – plasma volume D. I 125 albumin – blood volume
16. What fraction of total body potassium is present in plasma?
A. 0.4% B. 8.2%
C. 12% D. 88%
17. The lipid bilayer per second (without proteins) is most permeable to
A. Sodium ions B. Urea
C. Glucose D. Water
18. Red blood cells from a healthy individual were placed in each of the following solutions and observations were made
after 1 hour. Cells would have most likely have lysed when placed in
A. 0.3% NaCl B. 0.9% NaCl
C. 1.2% NaCl D. All of the above

https://t.me/DentalBooksWorld
Chapter 15 • Physiology 1283

19. Which of the following solutions is hypertonic?


A. 0.9 % NaCl B. 5% dextrose
C. 20% mannitol D. Distilled water
20. ICF volume does not change when dehydration is
A. Isotonic B. Hypertonic
C. Hypotonic D. Osmotonic
21. ICF volume decreases when dehydration is
A. Isotonic B. Hypertonic
C. Hypotonic D. Osmotonic
22. Cell volume and pressure is mainly dependent upon activity of
A. Na glucose cotransporter B. Na-K pump
C. Glucose transporter D. Na-Ca exchanger
23. Which of the following exerts the greatest osmotic effect across capillaries on a mole–mole basis?
A. Sodium B. Chloride
C. Dextran D. Albumin
24. The rate of diffusion of a substance across the cell membrane is inversely proportional to
A. Concentration gradient for the substance B. Diffusion coefficient
C. Surface area available for diffusion D. Thickness of the membrane
25. Which of the following modes of transport across the cell membrane is quantitatively more important for flux of ions?
A. Diffusion B. Filtration
C. Vesicular transport D. None of the above
26. Among plasma proteins, albumin makes the greatest contribution to the colloid osmotic pressure of plasma proteins
because, relative to most plasma proteins, albumin
A. Is the most abundant plasma protein B. Has the longest half-life
C. Has the lowest molar mass D. Exits the capillary fastest
27. Osmotic pressure of crystalloids is not included in the Starling’s equation describing fluid exchange across capillaries
because
A. They were discovered after Starling described the forces
B. Crystalloids carry electric charges
C. Crystalloids diffuse freely across the capillary endothelium
D. The quantity of crystalloids in plasma is negligible compared to plasma proteins
28. The term oncotic pressure is applied to osmotic pressure exerted by
A. Albumin and other plasma proteins across the capillary wall
B. Crystalloids across the cell membrane
C. Hemoglobin across the capillary wall
D. Substances such as urea, lactate, and glucose across the cell membrane
29. Which of the following is matched incorrectly?
A. Microfilaments: actin, myosin B. Intermediate filaments: vimentin, keratin
C. Microtubules: clathrin D. Cytoskeleton: spectrin, ankyrin
30. The mitotic spindle is made up of a protein called
A. Tubulin B. Caveolin
C. Connexin D. Clarthrin
31. The mitochondrial genome is absent from
A. Sperm cells B. Ovum
C. Mature red blood cells D. White blood cells
32. Most of the calcium in the endoplasmic reticulum is sequestered by
A. Calmodulin B. Caldesmon
C. Calbindin D. Calsequestrin

https://t.me/DentalBooksWorld
1284 Triumph’s Complete Review of Dentistry

33. Which of the following statements regarding regulation of cell cycle is correct?
A. This serves to regulate transition of the cell from one phase of the cell cycle to the next.
B. At restriction points, cyclins always promote transition from G0 to S phase.
C. Defects in DNA synthesis normally allow regression from G2 to M phase.
D. Deficiency of p53 allows progression to M phase.
34. Which of the following is an example of primary active transport?
A. Ca extrusion from cells by the Na-Ca exchanger
B. Glucose entry into cells through glucose transporter 2 (GLUT-2)
C. H2O flux across cell membranes through aquaporins
D. Ca sequestration in sarcoplasmic reticulum by Ca-ATPase
35. Sodium–glucose cotransport in the intestine and kidney is an example of
A. Primary active transport B. Secondary active transport
C. Facilitated diffusion D. Passive transport
36. Which of the following is an active transport processes?
A. Intrusion of calcium into ECF B. Efflux of K during repolarization
C. Entry of chloride into neurons D. Transcytosis (vesicular transport)
37. Which of the following processes is ATP dependent?
A. Acidification of lysosomes B. Actin–myosin cross bridge cycling
C. Exocytosis D. All of the above
38. Which of the following transport processes is mediated by a carrier protein in the plasma membrane?
A. Glucose uptake through SGLT-1 B. Na influx through Na channels
C. Na-K ATPase D. Water fluxes through aquaporins
39. Which of the following processes does not exhibit “saturation kinetics”?
A. Facilitated diffusion B. Na+, Ca2+ exchanger
C. Simple diffusion D. Na+ coupled active transport
40. Na-K-2Cl transporter in the apical membrane of the thick ascending limb of the loop of Henle is an example of
A. Primary active transport B. Secondary active transport
C. Passive transport D. Counter transport
41. Which of the following is an example of passive transport?
A. Calcium efflux by calcium pump B. Na–Ca exchanger
C. Potassium efflux through potassium leak channels D. Calcium sequestration in sarcoplasmic reticulum
42. Which of the following ions is not transported across the cell membrane by a primary active transport mechanism?
A. Na B. Cl
C. K D. Ca
43. Emiocytosis requires an increase in the intracellular concentration of
A. Na B. K
C. Ca D. Cl
44. Which of the following hormones does not act via a G-protein coupled receptor in the cell membrane?
A. Thyrotropin releasing hormone B. Angiotensin II
C. Antidiuretic hormone D. Thyroxine
45. Which receptor does not span the cell membrane seven times?
A. Rhodopsin B. Beta-adrenergic receptor
C. Insulin receptor D. M2 receptor
46. Protein kinase C is activated by
A. IP3 B. Diacylglycerol (DAG)
C. cAMP D. Guanylyl cyclase
47. Which of the following is not a second messenger in a signal transduction pathway?
A. cAMP B. Guanylyl cyclase
C. Inositol trisphosphate D. Diacylglycerol

https://t.me/DentalBooksWorld
Chapter 15 • Physiology 1285

48. The smooth muscle relaxing effects of endothelium derived relaxing factor nitric oxide are mediated by an increase in
intracellular level of
A. cAMP B. cGMP
C. Calcium D. Endothelin
49. Which of the following hormone(s) mediate(s) its effects by activating nuclear/cytosolic receptors? (Select all correct
answers)
A. Thyroxine B. Aldosterone
C. Progesterone D. All the above
50. Which intercellular junctions directly allow the passage of small molecules and ions between the cytosol of one cell
and its neighbor without movement into interstitial fluid?
A. Gap junctions B. Focal adhesions
C. Zonula occludens D. Desmosomes
51. Gap junctions are made up of a protein called
A. Connexin B. Clathrin
C. Cadherin D. Calcineurin
52. Connexins do not allow the passage of
A. Polypeptides B. Na ions
C. Ca ions D. Inositol trisphosphate
E. Amino acids
53. The term “homeostasis” was coined by
A. Claude Bernard B. Walter B Cannon
C. Homer Smith D. William Harvey
54. Which of the following statements about negative feedback control systems is incorrect?
A. Output is one of the inputs to the system
B. It is based on a “set-point” for the controlled variable
C. The system corrects “errors”
D. The “set point” of the system cannot be changed by inputs other than the controlled variable.
55. The diameter of a red blood cell is approximately
A. 7 × 10−3 m B. 7 × 10−6 m
−9
C. 7 × 10 m D. 7 × 10−10 m
56. The volume of a red blood cell is approximately
A. 8 × 10−10 liter B. 80 cubic microliter
C. 85 microns D. 90 femtoliter
57. The [H+] of arterial plasma is normally about
A. 20 nmol/L B. 30 nmol/L
C. 40 nmol/L D. 50 nmol/L
58. Normally, the ratio of PaCO2 and plasma HCO3 is
A. 1.2 B. 1.4
C. 1.6 D. 1.8
59. The Henderson–Hasselbalch equation states that when a buffer acid is half dissociated the pH of the solution is
equal to
A. pKa B. 7.0
C. 2 pKa D. 0.5 pKa
60. A buffer is most effective when the pH of the solution is close to
A. 0.5 pKa B. pKa
C. 2 pKa D. All of the above
61. The most abundant protein in mammalian cells is
A. Actin B. Collagen
C. Titin D. Dystrophin
62. The diameter of which of the following cytoskeletal components is the least?
A. Microfilaments B. Intermediate filaments
C. Microtubules D. None of the above
https://t.me/DentalBooksWorld
1286 Triumph’s Complete Review of Dentistry

63. Which of the following is an example of a ligand-gated ion channel?


A. Nicotinic Ach receptor (nAchR) B. Beta-2 adrenergic receptor
C. 5 HT receptor D. Histamine 2 receptor
64. Which of the following is an example of juxtacrine communication?
A. Cell–cell anchors caused by interactions between TGF and TGF receptors on adjacent cells
B. Passage of ions and small molecules between the cytosols of adjacent cells through connexons
C. Attachment of cell membrane to basal lamina via integrins
D. All of the above
65. Which of the following is not known to act via a G-protein coupled receptor in target cell membranes?
A. Acetylcholine B. Aldosterone
C. Epinephrine D. Histamine
66. Dimerization of which of the following hormone receptors is essential for signaling?
A. Acetylcholine receptor B. Beta-2 adrenergic receptor
C. Growth hormone receptor D. Histamine receptor
67. O2 consumption is minimal when temperature gradient between skin and external environment is
A. <1.5° B. >3.5°
C. <0.5° D. <2.5°
68. A/E are the features of exercise
A. Increase stroke volume B. Increase O2 extraction
C. Left shift of Hb-O2 dissociation curve D. Increase blood supply to muscle
69. Low haptoglobin in hemorrhage is masked by concurrent presence of
A. Obstructive biliary disease B. Liver parenchymal disease
C. Malnutrition D. Pregnancy
70. In exercising muscle, true about metabolism is
A. Same in aerobic and anaerobic B. Fatty acid used mainly
C. Glycogen and creatine kinase used aerobically D. All of the above
71. Which of the following does not change in the old age?
A. Glucose tolerance B. GFR
C. Blood pressure D. Hematocrit
72. Which of the following statement is true regarding basal metabolic rate?
A. It is not affected by dietary changes B. Decreased by 50% in starvation
C. Increased in starvation D. It is not influenced by hormonal changes
73. Autoregulation means
A. Regulated by local metabolites B. Well developed in the skin
C. Maintains the blood pressure D. Vary with change in pressure
74. Which of the following does not changes with age?
A. Glucose tolerance test B. Hematocrit
C. FEV1 D. Glomerular filtration test
75. Positive feedback is seen in
A. Entry of Ca into sarcoplasmic reticulum B. LH Surge
C. Thrombolytic activity coagulation cascade D. Stimulation of gastric secretion of histamine and gastrin
76. Physiological changes during severe exercise are
A. Hyperventilation in the beginning B. Hyperkalemia
C. Increased H2CO3 D. All of the above
77. All the following are true about the homeostatic mechanism of the body except
A. Value of controlled variable oscillates near a set point
B. System is stabilized by the positive feedback
C. Values revolve around the mean
D. Value of controlled variable is compared to the reference value
78. Living body control system correct deformity/disturbance
A. Immediately B. Completely
C. With gain D. All of the above
https://t.me/DentalBooksWorld
Chapter 15 • Physiology 1287

NERVE AND MUSCLE PHYSIOLOGY


1. Most of the ATP generated in nerve cells is utilized to energize the
A. Na–Ca exchanger B. H–ATPase in the cell membrane
C. Na–K ATPase D. Synthesis of proteins
2. The normal resting cardiac muscle cell is most permeable to
A. Na B. K
C. Ca D. Cl
3. The membrane potential at which net flux of an ion across the membrane is zero is called
A. Resting membrane potential B. Spike potential
C. Threshold potential D. Equilibrium potential of that ion
4. The resting membrane potential of ventricular cardiomyocytes is closest to the equilibrium potential for
A. Sodium B. Chloride
C. Potassium D. Calcium
5. The Nernst potential (also called equilibrium potential) is positive for
A. Na and Cl B. Na and K
C. Na and Ca D. K and Cl
6. The equilibrium potential of chloride in mammalian spinal motor neurons is typically about
A. + 20 mV B. Minus 40 mV (inside negative)
C. Minus 70 mV (inside negative) D. Minus 90 mV (inside negative)
7. The resting membrane potential of some neurons is equal to the equilibrium potential of
A. Na B. K
C. Cl D. Ca
8. Hypokalemia would be expected to result in
A. Increased neuronal excitability B. A more negative RMP
C. No change in RMP D. A decrease in firing level of neurons
9. The membrane potential of cardiac muscle cells is most affected by even a small change in plasma concentration of
A. Na B. K
C. Cl D. Ca
10. Which of the following would justify excluding sodium conductance from the equation for estimating resting
membrane potential in skeletal muscle fibers?
A. Extracellular [Na] is higher than ICF [Na] B. Na–K pump extrudes 3 Na for 2 K pumped in
C. Negligible Na permeability of the resting membrane D. Unexcited cells are equally permeable to Na and K
11. Excitability is defined as
A. Presence of a resting membrane potential
B. Use of more than 30% of ATP synthesized for powering the Na–K ATPase
C. Response to a threshold stimulus with a propagated action potential
D. Presence of voltage gated ion channels in a tissue
12. Site of origin in motor neurons of conducted impulses is
A. Dendritic zone B. Axon hillock
C. Node of Ranvier D. Terminal buttons
13. In motor neurons, the portion of the cell with the lowest threshold for the production of a full-fledged action
potential is
A. Initial segment B. Soma
C. Dendritic zone D. Node of Ranvier
14. Number of Na channels per square micrometer of membrane in myelinated mammalian neurons is maximum in
A. Cell body B. Dendritic zone
C. Initial segment D. Node of Ranvier
15. Which one of the following statements about electrotonic potentials is incorrect?
A. They are graded responses B. They are local (nonpropagated) responses
C. They may be depolarizing or hyperpolarizing D. They are produced by a threshold stimulus

https://t.me/DentalBooksWorld
1288 Triumph’s Complete Review of Dentistry

16. A stronger than normal stimulus can cause excitation of nerve or muscle during the
A. Absolute refractory period B. Relative refractory period
C. Spike potential D. Overshoot
17. The duration of action potential in a nerve is typically closest to
A. 2 ms B. 20 ms
C. 200 ms D. 2,000 ms
18. A traveling nerve impulse does not depolarize the area immediately behind it because
A. It is hyperpolarized B. The area immediately behind is refractory
C. It is not self-propagating D. The conduction is always orthodromic
19. Which of the following nerve fibers is most susceptible to hypoxia?
A. Somatic motor neurons B. Nociceptive afferents
C. Preganglionic autonomic neurons D. Fibers transmitting touch sensation
20. Which nerve fiber type is most susceptible to conduction block by local anesthetics?
A. Type A B. Type B
C. Type C D. All the above
21. Nociceptors signal through
A. Aα fibers B. Aβ fibers
C. Aγ fibers D. Aδ fibers
22. The duration of action potential in a skeletal muscle fiber is typically
A. 5 ms B. 25 ms
C. 100 ms D. 250 ms
23. In a skeletal muscle, thin filaments do not contain
A. Actin B. Myosin
C. Troponin D. Tropomyosin
24. Actin is tethered to Z-lines in a sarcomere by
A. Actinin B. Titin
C. Nebulin D. Dystrophin
25. Ryanodine receptor is located in the
A. Sarcolemma B. T-tubule
C. Terminal cisterns of sarcoplasmic reticulum D. Cytosol
26. ATPase activity of which of the following proteins is altered to regulate skeletal muscle contraction?
A. Actin B. Myosin
C. Troponin D. Tropomyosin
27. Major source of calcium for contraction of skeletal muscle is
A. ECF B. Cytosol
C. Mitochondria D. Sarcoplasmic reticulum
28. Excitation and contraction of skeletal muscle are coupled by
A. ATP B. Myosin
C. Release of calcium into sarcoplasm D. Calmodulin
29. Rigor mortis is due to
A. Damage to actin and myosin B. Rapid sequestration of Ca in ER
C. Increased myosin ATPase D. ATP depletion
30. Activity of which contractile protein is altered to regulate smooth muscle contraction?
A. Catherin B. Myosin
C. Calmodulin D. Tropomyosin
31. Calcium-binding protein that plays a key role in regulation of smooth muscle cell contraction is
A. Dystrophin B. Calmodulin
C. Troponin C D. Calcineurin

https://t.me/DentalBooksWorld
Chapter 15 • Physiology 1289

32. Which of the following statements is incorrect about visceral smooth muscle?
A. Neighboring cells are electrically coupled by means of gap junctions
B. Contraction can occur in the absence of extrinsic neural innervation
C. Twitch duration is typically longer when compared to skeletal muscle
D. Stretch of smooth muscle consistently increases active tension
33. Type I skeletal muscle fibers
A. They are called slow, oxidative fibers B. They are innervated by slow motor units
C. They have slow myosin ATPase activity D. All the above
34. Type II A skeletal muscle fibers
A. They are called fast oxidative, glycolytic fibers B. They are innervated by fast motor units resistant to fatigue
C. They have fast myosin ATPase activity D. All the above
35. Type II B skeletal muscle fibers
A. They are fast glycolytic fibers B. They have little to no myoglobin and low oxidative capacity
C. innervated by fast motor units that are fatigable. D. All the above
36. Oxidative capacity is highest in
A. Type I muscle fibers B. Type IIA muscle fibers
C. Type IIB muscle fibers D. All the above
37. Which of the following statements is incorrect?
A. Contraction can occur without an appreciable decrease in the length of the muscle
B. Contraction against a constant load with approximation of the ends of the muscle is called isotonic contraction
C. Muscles can lengthen while doing work
D. Isometric contractions do work whereas isotonic contractions do not
38. The total tension generated during skeletal muscle contraction is highest when the muscle
A. Is appreciably shorter than resting length B. Contracts isometrically at resting length
C. Is stretched beyond its resting length D. None of the above
39. Smallest amount of muscle that can contract in response to excitation of a single motor neuron is
A. One muscle fiber B. A muscle fasciculus
C. The entire muscle D. All muscle fibers supplied by that neuron
40. The number of muscle fibers innervated by a motor axon is smallest in
A. Gastrocnemius B. Orbicularis oculi
C. Single-unit smooth muscle D. Orbicularis oris
41. Staircase phenomenon is due to
A. Increased availability of intracellular calcium B. Synthesis of stable troponin C molecules
C. Summation D. Tetanus
42. Force of muscle contraction cannot be increased by
A. Increasing the frequency of activation of motor units B. Increasing the number of motor units activated
C. Recruiting larger motor units D. Varying release of calcium from SR
43. Recruitment and activation of motor units is investigated by
A. Electromyography B. Electroencephalography
C. Nerve action potential recordings D. Clinical examination of tendon jerks
44. Minimum stimulus intensity that produces a compound action potential in nerve or muscle is called
A. Rheobase B. Chronaxie
C. Twice rheobase D. Twice chronaxie
45. Which of the following is incorrect about cardiac muscle?
A. T-system is located at the Z lines
B. Adjacent muscle cells are coupled by gap junctions
C. Intercalated disks are at Z-lines
D. The twitch duration is shorter compared to type II skeletal muscle fibers
46. Regarding ionic basis of action potential in working cardiomyocytes, which of the following is incorrect?
A. Phase 0: Na influx B. Phase 1: K influx
C. Phase 2: Ca influx D. Phase 3: K efflux

https://t.me/DentalBooksWorld
1290 Triumph’s Complete Review of Dentistry

47. The shortest phase of the action potential in ventricular cardiomyocytes is


A. Phase 0 B. Phase 1
C. Phase 2 D. Phase 3
48. Which of the following characteristics is unique to skeletal muscle and is not observed in normal cardiac muscle?
A. Gradation of force production B. Refractoriness
C. Staircase phenomenon D. Tetanizability
49. Which of the following accelerates contraction as well as relaxation of heart muscle?
A. Norepinephrine B. Calcium
C. Digoxin D. Potassium
50. Which is the predominant energy substrate utilized by cardiac muscle?
A. Carbohydrate B. Amino acids
C. Ketones D. Fatty acids
51. Which of the following is not a step in vascular smooth muscle contraction?
A. Ca binding to troponin C
B. Activation of calcium-calmodulin dependent myosin light chain kinase
C. Phosphorylation of myosin
D. Increased myosin ATPase activity
52. Resting membrane potential is most unstable in which of the following muscles?
A. Smooth muscle of the stomach B. Gastrocnemius muscle
C. Orbicularis oculi D. Muscle cells in the left ventricle
53. The latch-bridge mechanism in smooth muscle is responsible for
A. Fast muscle twitch B. Sustained muscle contraction
C. Excitation-contraction coupling D. Unstable membrane potential
54. The repolarizing phase of the action potential in nerve fibers is primarily due to
A. Na–K ATPase
B. Sodium influx via voltage gated sodium channels
C. Potassium efflux via voltage gated potassium channels
D. Calcium efflux mediated by Na–Ca exchanger
55. Which of the following is correct with regard to numbers of each of the following in the CNS?
A. Neurons >> glia > synapses B. Glia >>> synapses >>> neurons
C. Synapses >> glia > neurons D. Synapses >> neurons >> glia
56. Which of the following types of cells work as scavenger cells in the CNS?
A. Microglia B. Oligodendroglia
C. Ependymal cells D. Astrocytes
57. Which of the following cells in the CNS is primarily involved in the reuptake of excitatory neurotransmitters released
by neurons?
A. Astroglial cell B. Microglial cell
C. Oligodendroglial cell D. Ependymal cell
58. Find the faulty statement regarding muscle spindle
A. Peripheral has no actin and myosin
B. Central zone has no active and myosin
C. Type 2 nuclear bag fibers has high level of myosin ATPase activity
D. Type 1 nuclear bag fibers has low myosin ATPase activity
59. Which is the commonest type of synapse in the CNS?
A. Axosomatic synapse B. Axodendritic synapse
C. Axoaxonal synapse D. All the above
60. Which of the following events in postsynaptic neuronal membrane underlie excitatory postsynaptic potentials?
A. Opening of Na channels B. Closure of K channels
C. Both of the above D. Opening of chloride channels

https://t.me/DentalBooksWorld
Chapter 15 • Physiology 1291

61. Which of the following events in postsynaptic neuronal membrane underlie inhibitory postsynaptic potentials?
Opening of
A. Na channels B. Ca channels
C. Cl channels D. All the above
62. Inhibitory postsynaptic potentials (IPSP) are an example of
A. Synaptic inhibition B. Presynaptic inhibition
C. Direct inhibition D. Indirect inhibition
63. The inhibitory amino acid neurotransmitters in the CNS are
A. Glutamate and glycine B. Glutamate and aspartate
C. GABA and glycine D. Aspartate and glycine
64. Which of the following neurotransmitters has both excitatory and inhibitory effects?
A. Aspartate B. GABA
C. Glutamate D. Glycine
65. The first physiologic response to high environmental temperature is
A. Cutaneous vasodilation B. Decreased heat production
C. Nonshivering thermogenesis D. Sweating
66. Central warping is done by
A. Annulospiral ending B. Flower spray ending
C. Type II fiber D. Golgi tendon
67. Lowest most level of integration of stretch reflex is at
A. Lower medulla B. Spinal cord
C. Cerebral cortex D. Medulla
68. Resting membrane potential is close to the isoelectrical potential of
A. Cl B. Na
C. Mg++ D. K+
69. Increased velocity of conduction in a nerve is favored by
A. Increased resistance B. Increased velocity
C. Increased capacitance D. Decreased capacitance
70. Which is true regarding nerve condition?
A. Propagated action potential is generated in dendrites B. Faster in unmyelinated fibers
C. All or none phenomenon D. Condition independent of amplitude
71. Which of the following is an aminoneuro transmitter?
A. GABA B. Acetylcholine
C. Epinephrine D. Lignocaine
72. True for smooth muscle contraction is A/E
A. Slow and prolonged contractile response independent of nerve supply
B. Length tension relationship does not exist
C. Catch bridge are less cycle less ATP bridge
D. Dephosphorylation causes definite relaxation
73. True about Renshaw cell inhibition is
A. Has memory for spinal cord B. Inhibition of feedback propagation
C. Add on collateral sensation D. Increased by local anesthetics
74. Which of the following of is true about propagated nerve action potential?
A. Fastest C fibers B. None
C. Decremental D. Not affected by hypoxia
75. Plate ending are feature of
A. Nuclear bag fibers B. Nuclear chain fibers
C. Golgi tendon D. Extrafusal
76. True about nerve impulse is
A. If current is increased too slow nerves respond fast B. Travels in one direction along axon
C. Travels with speed of electric current D. Travels in one direction at synapse

https://t.me/DentalBooksWorld
1292 Triumph’s Complete Review of Dentistry

77. Orthodromic conduction is


A. The jumping of depolarized from node to node B. The point at which a runaway spike potential
C. An axon can conduct impulse in one direction only D. An axon can conduct impulse in both direction
78. The average number of muscle fibers attached to one Golgi tendon organ are
A. 5–10 B. 1–3
C. 15–75 D. 10–15
79. Group B Nerve fibers are situated in
A. Fibers carrying pain sensation B. Muscle spindles
C. Post ganglionic autonomic fibers D. Preganglionic autonomic fibers
80. Muscle spindle is
A. Receptor for myotatic or stretch receptor reflex
B. Receptor for a variety of multisynaptic reflexes
C. Excited by both stretch and contraction of the muscles in which it is located
D. Occurs only in antigravity extensor muscles
81. A traveling nerve impulse does not depolarize the area immediately behind it, because
A. It is refractory B. It is hyperpolarized
C. The condition is always orthodromic D. It is not self-propagating
82. Nerve fibers involved in proprioception
A. Type C fiber B. Type IV fiber
C. Type A fiber D. Type b fiber
83. Autogenic inhibition is the function of
A. Extrafusal fiber B. Intrafusal fiber
C. Muscle spindle D. Golgi tendon organ
84. In which type of nerve fibers is conduction blocked maximally by pressure?
A. Ab B. Aa
C. C D. Ag
85. Stimulation of post ganglion sympathetic neurons leads to
A. Fast EPSP B. Slow EPSP
C. Slow IPSP D. All of the above
86. True regarding excitation contraction coupling in smooth muscle is
A. Phosphorylation of actin is required for contraction
B. Presence of cellular calcium is essential to cause muscle contraction
C. Presence of troponin is essential
D. Sustained contraction occurs with high calcium concentration
87. Leksell efferents go to which one of the following?
A. Golgi tendon organ B. Spindle
C. Golgi bottle cells D. Interneurons
88. Not true about organ tendon is
A. Protective B. Contracting
C. Inhibitory D. Negative feed back
89. Increased gamma efferent discharge is seen in all except
A. Anxiety B. Jendrassik’s maneuver
C. Stimulation of skin D. Rapid shallow breathing
90. The band which disappears on muscular contraction is
A. M or CM B. A
C. H D. I
91. In muscle contraction all are true except
A. H zone disappears B. A bond remains unchanged
C. Two Z lines come closer D. I band become Wider

https://t.me/DentalBooksWorld
Chapter 15 • Physiology 1293

92. Resting membrane potential of nerve is equal to equilibrium potential of


A. Cl− B. Na+

C. HCO3 D. K+
93. Group II sensory fibers attach to
A. Flower spray ending B. Annulospiral ending
C. Pacinian corpuscle D. Golgi tendon
94. Intensity of sensory stimulation is directly related to
A. Duration of action potential (AP) B. Frequency of AP
C. Amplitude of AP D. All of the above
95. Find the false statement regarding sensory ending
A. Primary ending conduct Ia fibers B. Annulospiral warp the ends
C. Flower spray is secondary D. Primary ending is annulospiral
96. In intersegmental reflex, the afferents come from
A. IIb fibers B. Muscle spindles
C. Golgi tendon D. Ia fibers
97. Which of the following is true about Nissl granule?
A. Present in dendrite B. Involves in protein synthesis
C. Structurally they are endoplasmic reticulum D. All of the above
98. Action potential is when
A. K+ goes from ECF to ICF B. Threshold stimulation is required
C. Decremental phenomenon D. Does not obey all or none phenomenon
99. Tendon organ, correct is A/E
A. Regulates tension B. Ramified nobby nerve ending
C. Myelinated Ib fibers D. Weak stretch activates reflex
100. The afferent fibers which are most sensitive to local anesthetic belong to group
A. B B. A
C. D D. C
101. Resting membrane potential in nerve fiber
A. Increases as extracellular K+ increase B. Depends upon K+ equilibrium
C. Is equal to the potential of ventricular muscle fiber D. Can be measured by surface electrodes
102. Closed extensor reflex is a
A. Monosynaptic reflex B. Sympathetic
C. Withdrawal reflex D. Postural reflex
103. Action of calmodulin is
A. Through calmodulin dependent kinase B. Through CAPM dependent kinase
C. Through CGPM dependent kinase D. All of the above
104. Which of the following is true about Nerve fiber “C”?
A. Unmyelinated B. Most susceptible to Hypoxia
C. Not for temp. and pain sense D. Preganglionic autonomic
105. The intrafusal fibers of the skeletal muscles are innervated by one of the following type of motor neurons. Choose the
correct answer
A. Beta B. Alpha
C. Delta D. Gamma
106. Synaptic potentials can be recorded by
A. Voltage clamp technique B. Patch clamp technique
C. EEG D. Microelectrode
107. Golgi tendon organ true is A/E
A. Activated by both stretch and contraction of muscle B. Static and dynamic response
C. Transmit to Ib fibers D. Responsible for stretch reflex

https://t.me/DentalBooksWorld
1294 Triumph’s Complete Review of Dentistry

108. Increase in cytosolic calcium from intracellular storage, during smooth muscle contraction is due to
A. CCMP B. IP3-DAG
C. CAMP D. CGMP
109. The first change occurring in a sharp cut nerve is
A. Degeneration of myelin sheath B. Schwann cells proliferation
C. Degeneration of neurilemma D. Chromatolysis
110. Calmodulin activates
A. 2, 3 DPG B. Glucokinase
C. Muscle phosphorylase D. Protein kinase
111. What is true regarding the gamma efferent neuron?
A. An “A” group
B. Motor neuron with a smaller diameter than that of alpha efferent neurons
C. Innervates intrafusal fibers
D. All of the above
112. Contractile part of intrafusal fiber
A. Both B. None
C. Ends D. Center
113. All of the following are true regarding intrafusal fibers, except
A. Nuclear bag fibers are lesser B. Nuclear chain fibers are shorter and thinner
C. Secondary ending excited by chain fibers only D. Primary ending excited by bag fibers only
114. “Nerve terminals release chemicals” – discovered by
A. Domagk B. Loewi
C. Dale D. Withering
115. Which of the following is the true statement about Golgi tendon organ?
A. Detects length change B. 3–25 muscle fibers
C. High threshold D. Dynamic response only

CVS
1. Maximum difference of BP occurs between
A. Femoral artery and femoral vein B. Descending Aorta and common iliac artery
C. Capillaries and venules D. Arterial end and venous end
2. True about blood flow in various organs
A. Kidney >Brain > Heart >Liver B. Liver > Heart > Brain > Kidney
C. Liver > Kidney >Brain > Heart D. Liver >Brain >kidney > Heart
3. All of the following is true about lung circulation except
A. V/P ratio at rest is 0.8 B. In apex ventilation is less than at base
C. Decreased vital capacity in supine position D. All of the above
4. Effectiveness of blood brain barrier is by
A. Thick basement membrane B. Tight arrangement of astrocytes
C. Tight endothelial function D. Microglial cell
5. Palpable enlargement of the liver in an individual with heart failure is most closely related to
A. A decrease in pulmonary venous pressure B. An increase in left ventricular compliance
C. An increase in mean arterial pressure D. An increase in mean right atrial pressure
6. Activity of factor VIII procoagulant is deficient in
A. ITP B. Sickle cell anemia
C. Hemophilia D. Von Willebrand’s disease
7. Plasma level of brain natriuretic peptide is least likely to be elevated in
A. Acute mitral regurgitation B. Cardiac tamponade due to chest trauma
C. Heart failure due to dilated cardiomyopathy D. Heart failure due to acute aortic regurgitation

https://t.me/DentalBooksWorld
Chapter 15 • Physiology 1295

8. Wedge hepatic venous pressure represents pressure in


A. Main hepatic vein B. Main portal vein
C. Central vein radicles D. Sinusoids
9. Which of the following is the order of activation after stimulation of Purkinje fibers?
A. Epicardium ≥ Septum ≥ Endocardium B. Septum ≥ Epicardium ≥ Endocardium
C. Septum ≥ Endocardium ≥ Epicardium D. Endocardium ≥ Septum ≥ Epicardium
10. Volume determining preload is
A. Volume of blood aorta B. Ventricular ejection volume
C. End diastolic volume of ventricles D. Endosystolic volume
11. Which of the following situations will lead to increase in viscosity of blood?
A. Hypoglycemia B. Fasting state
C. Amyloidogenesis D. Multiple myeloma
12. Function of lymphatics
A. Carries protein B. Fat absorption
C. Prevention role in infection D. All of the above
13. Which of the following is true about pulmonary circulation?
A. Hypoxia causes vasoconstriction B. Blood volume in the lungs is 450 ml
C. It as low resistance D. All of the above
14. P.G. increasing the renal blood flow is
A. PGI2 B. TXA2
C. PGF2 D. All of the above
15. Least conduction velocity is seen in
A. Bundle of His B. Ventricular myocardial fibers
C. AV node D. Purkinje fibers
16. Thrombosthenin is a/an
A. Contractile protein B. Thrombosis preventing protein
C. Protein for regulating platelets production D. Coagulation protein
17. Absolute monocytosis is seen in
A. Kala-azar B. TB
C. Brucellosis D. All of the above
18. Orthopnea in heart failure occurs when the reservoir function of which of the following is overwhelmed?
A. Pulmonary veins B. Pulmonary arteries
C. Right atrium D. Systemic veins
19. The cardiac output can be determined by all except
A. V/Q ratio B. Fick’s principle
C. Thermodilution D. Echocardiography
20. Cardiac index ratio is determined by
A. Surface area only B. Peripheral resistance
C. CO and surface area D. SV and surface area
21. Erythropoiesis is promoted by all of the following except
A. Thyroxine B. ACTII
C. Prolactin D. Estrogen
22. The pressure volume curve is shifted to the left in
A. Mitral stenosis B. Aortic stenosis
C. Mitral regurgitation D. Aortic regurgitation
23. What is true of iron?
A. Spleen in major storage organ B. Fe++ is excreted in urine
C. It is stored in ferritin D. It is absorbed by transferrin in the intestine
24. All are vitamin K dependent clotting factors of hepatic origin except
A. VII B. II
C. X D. VIII

https://t.me/DentalBooksWorld
1296 Triumph’s Complete Review of Dentistry

25. Shape of the arterial pulse is influenced by


A. Velocity of blood B. Viscosity of blood
C. Cross-sectional area of artery D. Arterial wall expansion
26. Neutrophil secretes
A. Myeloperoxidase B. Superoxide dismutase
C. Catalase D. Lysosomal enzyme
27. In which of the following organs is the flow least under sympathetic control?
A. Brain B. Heart
C. Viscera D. Skin
28. Which of the following is not a Vitamin K dependent factor?
A. IX B. X
C. II D. VII
29. Max. blood supply to the liver is by
A. Splenic A B. Mesenteric A
C. Portal vein D. Hepatic A
30. Spuriously high BP is seen in all the following, except
A. Thick calcified vessels B. Obesity
C. Auscultatory gap D. Small cuff
31. The best method to access the adequacy of replacement of fluid in case of shock
A. Increased PaO2 B. Decrease in thirst
C. Blood pressure D. Increase in urine output
32. Helper and cytotoxic cells belong to
A. Monocytes B. Macrophages
C. T cells D. B cells
33. ADT test is for
A. Hg F B. Hg A
C. Hg A2 D. H B
34. Central lymphoid organs are
A. Bone marrow B. Lymph node
C. Tonsil D. All of the above
35. What is not true for extrasystole in ventricle?
A. Hints at serious heart ailment B. Falls to produce radial pulse
C. Tendency to be followed by a compensatory pause D. Associated with abnormal QRS complex
36. Of the following which one correlates with isovolumic contraction phase?
A. AV closure and aortic pulmonary valve opening B. AV opening and aortic and pulmonary valve closure
C. Both valves are open D. Both valves are closed
37. ABO blood group system remains the most important because
A. ABO(H) antigens are present in most body tissues and fluids
B. ABO(H) antibodies are invariably present in plasma when persons RBC lacks the corresponding antigen
C. It was the first blood group system to be discovered
D. It has four different blood groups A, B, AB, O(H)
38. Baroreceptor stimulations produce
A. Decreased heart rate and BP B. Increased heart rate and BP
C. Increased cardiac contractility D. All of the above
39. Physiological dead space in the lung is
A. Zone 3 B. Zone 4
C. Zone 1 D. Zone 2
40. A shift of posture from supine to upright posture is associated with cardiovascular adjustments. Which of the following
is not true in the context?
A. Decreased in cardiac output B. Decreased in stroke volume
C. Rise in central venous pressure D. Rise in heart rate

https://t.me/DentalBooksWorld
Chapter 15 • Physiology 1297

41. True regarding vascularity of the lung is


A. Perfusion is more in the apical lob than in base
B. Distended pulmonary veins in the lower lob
C. Hypoxia causes vasodilation
D. Pulmonary resistance is half of the systemic vascular resistance
42. Which of these mediate the precapillary sphincter relaxation?
A. Catecholamines B. Capillary filling
C. Local hormones D. Sympathetic stimulations
43. Reticulocytes are stained with
A. Brilliant cresyl blue B. Methyl violet
C. Indigo carmine D. Sudan Black
44. Venous return (VR) is given by formula
A. VR = (MFSP − RVR)/RAP B. VR = (RVR − RAP)/MFSP
C. None D. VR = (MFSP − RAP)/RVR
45. Blood pressure is defined as the product of
A. Pulse pressure x pulse rate B. Cardiac output x peripheral resistance
C. Systolic pressure x pulse D. Diastolic pressure x pulse rate
46. Albumin is an important factor in maintaining osmotic pressure, it has
A. High molecular weight and low blood concentration B. High molecular weight and high blood concentration
C. Low molecular weight and high blood concentration D. Low molecular weight and low blood concentration
47. Prostaglandin causing renal vasodilation
A. PGD2 B. PGE2
C. PGF2 D. All of the above
48. Cardiac index is defined as
A. C.O.P. per unit body surface area B. Stroke volume M2 /BSA
C. End diastolic volume D. Syst press/M2 BSA
49. Nitric Oxide is produced in
A. Platelets B. Serum
C. Endothelium D. Plasma
50. All are true regarding circulation except
A. Transfused blood is mainly distributed in arteries B. Hematocrit markedly change peripheral resistance
C. Nonpulsatile flow is recommended D. All of the above
51. Myocardial oxygen demand depend upon
A. Preload B. Afterload
C. Intramyocardial tension D. All of the above
52. Repolarization in isolated muscle piece fiber proceeds from
A. Endocardium to Epicardium B. Epicardium to Endocardium
C. Right to left D. Left to right
53. Life span of neutrophils is
A. 10 days B. 15 days
C. 6 hours D. 6 days
54. True about cytokines are all except
A. Involve in leukocytic movements B. It takes part in intrinsic enzymatic reaction
C. It is polypeptide D. Have autocrine and paracrine effects
55. Blood supply in splanchnic vessels decrease due to
A. Venodilation with increased blood flow B. Venodilation with normal blood flow
C. Venodilation with decrease blood flow D. Venoconstriction with decrease blood flow
56. A procoagulant not normally circulating in the plasma is
A. Prothrombin B. Fibrinogen
C. Factor V D. None of the above

https://t.me/DentalBooksWorld
1298 Triumph’s Complete Review of Dentistry

57. Cardiac index in a normal person is


A. 3.2 B. 2.1
C. 5.9 D. 4.6
58. Source of TNF are
A. Neutrophil B. Macrophage
C. Basophils D. All of the above
59. Pressure on carotid sinus cause
A. Reflex bradycardia B. Hyperapnea
C. Dyspnea D. Tachycardia
60. During homeostasis, platelet affects the coagulation area
A. Platelet adhesion to exposed endothelium B. Clot reaction
C. Activation of prothrombinase D. All of the above
61. A patient with increased BP and decreased heart rate is likely to have
A. Increased ICT B. Brain tumor
C. Head trauma D. All of the above
62. Lysozymes are found in
A. Eukaryotes B. Plants
C. Virus D. All of the above
63. Protein filtration across cerebral capillaries in limited by
A. Foot process of astrocytes B. Fibrous tissue
C. High CSF pressure D. Low BP
64. Ringer lactate contains all except
A. Sodium B. Chloride
C. Potassium D. Bicarbonate
65. Speed of conduction is faster in
A. Purkinje system B. Ventricular muscle
C. AV node D. SA node
66. Memory cell in immune system are long lived and escape apoptosis because of
A. Platelet-derived growth factor B. Nerve growth factor
C. Insulin-like growth factor D. Fibroblast growth factor
67. The most abundant protein in blood is
A. Albumin B. Hemoglobin
C. Fibrinogen D. Beta-1 globulin
68. The wheal in the triple response is due to
A. Contraction of precapillary sphincters B. Increased capillary permeability
C. Axon reflex D. Decreased absorption of fluid
69. Macrocytes have a mean corpuscular volume greater than
A. 70 fL B. 80 fL
C. 90 fL D. 100 fL
70. The amount of hemoglobin contained in normocytes is approximately
A. 20 pg B. 25 pg
C. 30 pg D. 35 pg
71. The amount of hemoglobin present in 100 ml of red blood cells is defined as
A. MCH B. MCHC
C. Hemoglobin index D. RDW
72. Capillaries empty when mechanically stimulated. This is called the
A. Axon reflex B. White reaction
C. Flare response D. Red reaction
73. Primary hemostasis refers to cessation of bleeding due to
A. Formation of a definitive clot B. Clot retraction
C. Formation of a temporary platelet plug

https://t.me/DentalBooksWorld
Chapter 15 • Physiology 1299

74. The adhesion of platelets to subendothelial collagen is impaired in the absence of


A. Von Willebrand factor B. Plasmin
C. Heparin D. Antithrombin III
75. Which of the following clotting factors is not vitamin K dependent?
A. Factor II B. Factor V
C. Factor VII D. Factor IX
76. The extrinsic pathway is triggered by the release of
A. Factor VII B. Tissue factor
C. Tissue factor pathway inhibitor D. Contact factor
77. The extrinsic pathway is inhibited by
A. Tissue factor B. Thromboplastin
C. Tissue factor pathway inhibitor (TFPI) D. Contact factor
78. The test that screens the extrinsic pathway is
A. Prothrombin time (PT) B. Activated partial thromboplastin time (aPTT)
C. Thrombin time D. Urea solubility test
E. Clot lysis time
79. The enzyme that ultimately lyses fibrin is
A. Plasminogen B. TPA
C. Urokinase D. Plasmin
80. Prolongation of prothrombin time does not occur when there is a deficiency of only
A. Factor VIII B. Factor IX
C. Factor X D. Vitamin K
81. Red cell antigens A and B are chemically
A. Phospholipids B. Glycosphingolipids
C. Glycopeptides D. Polypeptides
82. Select all correct answers. Red blood cell antigens A and B are also present in
A. Saliva B. Semen
C. Amniotic fluid and pancreas D. All the above
83. The red blood cells of a person with the Bombay blood group do not have
A. GLUT B. H substance
C. Spectrin D. Ankyrin
84. Most of the iron in the body is present in
A. Hemoglobin B. Myoglobin
C. Ferritin D. Transferrin
85. In an Rh-negative mother not previously sensitized by the Rh antigen, Rh incompatibility does not usually have a
serious consequence during the first pregnancy because
A. Antibodies are not able to cross placenta
B. The titer of IgG is low during the primary immune response
C. IgG is ineffective against fetal red cells
D. Massive hemolysis in the fetus is compensated by increased erythropoiesis
86. In the context of blood transfusions, ABO compatibility is important because
A. There are 3 antigens in this system
B. The A and B antigens are present in all cells
C. When an individual’s RBC lacks the A or B antigen, the corresponding antibody is invariably present in serum
D. O is a strong antigen
87. Osmotic fragility of red blood cells is decreased in
A. Sickle cell anemia B. Hereditary spherocytosis
C. Macrocytic hypochromic anemia D. Macrocytic anemia
88. Hereditary spherocytosis occurs due to mutations in genes coding for
A. Spectrin and ankyrin B. Na-K ATPase
C. Glucose-6-phosphate dehydrogenase D. Pyruvate kinase

https://t.me/DentalBooksWorld
1300 Triumph’s Complete Review of Dentistry

89. CO is formed as an end product of


A. Heme metabolism B. Arginine metabolism
C. Oxidation of acetoacetate D. All of the above
90. Heme is converted to bilirubin mainly in the
A. Kidneys B. Liver
C. Spleen D. Bone marrow
91. The protein that binds extracorpuscular hemoglobin is
A. Hemin B. Haptoglobin
C. Hemopexin D. Haptopexin
92. When a serum sample is electrophoresed, which of the following bands is normally absent?
A. Albumin B. α1 globulin
C. α2 globulin D. Fibrinogen
93. Which of the following is not synthesized in the liver?
A. IgG B. α2 macroglobulin
C. Albumin D. Angiotensinogen
94. Which of the following plasma proteins are protease inhibitors?
A. α1 antitrypsin B. Transferrin
C. C-reactive protein D. Antithrombin II
95. Which of the following is a “negative” acute phase reactant?
A. Albumin B. C-reactive protein
C. α2 macroglobulin D. Transferrin
96. ESR is increased in
A. Anemia B. Hypofibrinogenemia
C. Spherocytosis D. Polycythemia
97. The average half-life of neutrophils in the circulation is
A. 6 hours B. 5 days
C. 2 weeks D. 1 month
98. The protein content of lymph draining from the __________ is highest
A. Choroid plexus B. Skeletal muscle
C. Liver D. Gastrointestinal tract
99. The heart continues to beat even after all nerves to it are sectioned. This property is called
A. Excitability B. Conductivity
C. Automaticity D. Contractility
100. What is the primary ionic basis of the prepotential in the SA node?
A. Ca influx through transient T Ca channels B. Inwardly directed long-lasting Ca current
C. Inward Na current D. Potassium efflux through leak channels
101. Normally, the impulse that excites the left ventricular myocardium originates in the
A. SA node B. Purkinje system
C. Left bundle branch D. Ventricle
102. Conduction speed is slowest in the
A. SA node B. Atrial pathways
C. Bundle of His D. Purkinje system
103. What is the effect of vagal stimulation on the membrane potential of the SA node?
A. It increases an inward calcium current B. It increases the slope of the prepotential
C. It activates a hyperpolarizing potassium current D. It increases intracellular cAMP
104. Activation of beta-adrenergic receptors in the heart is normally associated with which of the following?
A. Decrease in the slope of phase 4 depolarization in SA nodal cells
B. Decrease in conduction speed through AV node
C. Inhibition of Ca induced Ca release following depolarization in ventricular myocytes
D. Accelerated sequestration of Ca in the sarcoplasmic reticulum by the Ca-ATPase

https://t.me/DentalBooksWorld
Chapter 15 • Physiology 1301

105. Prepotentials are normally absent from


A. P cells in the SA node B. AV nodal cells
C. Purkinje fibers D. All the above
106. Cardiac muscle cannot be tetanized because of
A. Accommodation
B. Its slow rate of repolarization
C. Calcium influx during phase II
D. Voltage inactivation of Na channels at membrane potentials < 80 mV
107. The ability of the AV node to generate its own impulses when the sinus node is “sick” is due to
A. A constant phase 4 membrane potential B. Slow calcium entry during phase zero
C. Spontaneous diastolic depolarization D. The absence of prepotentials
108. The propagation of repolarization from the ventricular epicardium to endocardium is represented by the
A. QRS complex B. QT interval
C. T wave D. TP period
109. T wave inversion occurs when ventricular repolarization occurs from
A. endocardium to epicardium B. epicardium to endocardium
C. apex to base of the heart D. base to apex of the heart
110. Which of the following is the shortest event in a cardiac cycle?
A. QRS interval B. ST segment
C. ST interval D. RR interval
111. Electrical activity in which region of the heart does not result in deflections on the surface electrocardiogram?
A. Atria B. Bundle of His
C. Free wall of the left ventricle D. Free wall of the right ventricle
112. In sinus rhythm, the last portion of the ventricle to depolarize is
A. Interventricular septum from left to right
B. Anteroseptal region of the myocardium
C. Most of the myocardium from endocardium to epicardium
D. Posterobasal portion of left ventricle and the pulmonary conus
113. The most reliable index of AV nodal delay is
A. AH interval B. PA interval
C. PR interval D. PR segment
114. Hypocalcemia is associated with QT prolongation because
A. It is invariably associated with bundle branch block
B. It increases ventricular activation time
C. It lengthens the duration of ventricular repolarization
D. It accelerates opening of potassium channels
115. Stimulation of sympathetic nerves to the heart decreases
A. Heart rate B. Force of cardiac contraction
C. Speed of conduction D. Refractory period
116. Right axis deviation may occur in
A. Deep inspiration B. Emphysema
C. Dextrocardia D. Pulmonary hypertension
117. The AV node does not conduct more than
A. 180 impulses per minute B. 230 impulses per minute
C. 280 impulses per minute D. 330 impulses per minute
118. The ST segment is elevated in acute myocardial infarction because of
A. Flow of current into the infarct during diastole B. TP segment depression, early repolarization of infarct
C. Late depolarization of infarct D. All the above
119. During the cardiac cycle, aortic valve closes at the end of
A. Isovolumetric systole B. Rapid ejection
C. Diastasis D. Protodiastole

https://t.me/DentalBooksWorld
1302 Triumph’s Complete Review of Dentistry

120. The maximum pressure rise in the ventricle occurs during


A. Ejection B. Isovolumetric contraction
C. Protodiastole D. Diastasis
121. Normally, which of the following events in the cardiac cycle occurs at some point between S1 and the following S2?
A. Onset of ventricular diastole B. Atrial systole
C. Rapid ventricular filling D. The “a” wave of the JVP
122. Comparing left and right atria, the left atrium normally has a taller v wave than the right atrium because
A. Left atrial filling pressure is high B. Pulmonary vessels empty into the left atrium
C. Left atrium is more complaint D. Right ventricle is more compliant
123. In which of the following states is isovolumetric ventricular relaxation abbreviated, assuming that the prevailing
heart rate is identical in each?
A. Aortic regurgitation B. Mitral regurgitation
C. Mitral stenosis D. Tricuspid regurgitation
124. In postnatal life, steady state outputs of the right and left ventricle are matched in vivo by
A. The Frank–Starling mechanism B. Sympathetic influences on the SA node
C. Vagal influences on the SA node D. Varying the afterload for each ventricle
125. Which of the following need to be recorded in order to determine systolic time intervals?
A. ECG, ECHO, and phonocardiogram (PCG) B. ECG, PCG, and carotid artery pulse (CAP)
C. ECHO, CAP, and PCG D. ECG, CAP, and apex cardiogram
126. The dicrotic notch is absent from
A. Radial arterial pulse tracing B. Pulmonary arterial pulse tracing
C. Aortic pulse tracing D. None of the above
127. The most recent technique for noninvasive measurement of cardiac output is
A. Pulmonary artery catheterization B. Thermodilution
C. Echocardiography D. Impedance cardiography
128. The best measure of left ventricular preload is
A. Left ventricular end-diastolic volume B. Left atrial pressure
C. Pulmonary capillary wedge pressure D. Right atrial pressure
129. The most appropriate index of left ventricular afterload is
A. Systolic arterial pressure B. Mean arterial pressure
C. Systemic vascular resistance D. Left ventricular systolic pressure
130. Right ventricular preload typically exceeds left ventricular preload during
A. Deep inspiration B. Deep expiration
C. Valsalva maneuver (for 15 seconds) D. Quiet standing
131. LV preload is increased by a/an
A. Increase in intrapericardial pressure B. Decrease in left ventricular compliance
C. Sympathetic stimulation of veins and the heart D. Inhibition of Na-K ATPase in cardiomyocytes
132. An increase in whole body oxygen demand is met chiefly by
A. Increasing cardiac output B. Increasing oxygen content of arterial blood
C. Increasing oxygen extraction from arterial blood D. Increasing blood pressure
133. Windkessel vessels are represented by the
A. Aorta B. Internal jugular vein.
C. Arterioles D. Muscular arteries
134. The stopcocks of the circulation are
A. Arterioles B. Capillaries
C. Valves D. Venules
135. Across which site in the circulation is the pressure drop maximum?
A. Arterioles B. Venules
C. Capillaries D. Aortic valve

https://t.me/DentalBooksWorld
Chapter 15 • Physiology 1303

136. At any time, the greatest fraction of blood volume is present in the
A. Heart B. Arteries
C. Veins D. Capillaries
137. The term “capacitance vessels” is applied to
A. Pulmonary capillaries B. Thoroughfare channels
C. Shunts D. Veins and venules
138. Hydraulic conductivity of capillaries is highest in
A. Glomeruli B. Intestinal villi
C. Skin D. Brain
139. What would be the change in blood flow to a tissue if radius of the arterioles in that tissue is doubled and perfusion
pressure is halved?
A. Increases 8 times B. Increases 16 times
C. Increases 4 times D. Decreases 4 times
140. The pressure in a blood vessel at which flow ceases is called
A. Mean arterial pressure B. Pulse pressure
C. Critical closing pressure D. Perfusion pressure
141. Thin walled capillaries do not burst when intracapillary pressure is increased within physiologic limits because
A. They lack smooth muscle cells
B. The blood flow rate is less
C. They have a small radius
D. Capillary hematocrit is less than whole-body hematocrit
142. That capillaries can withstand high internal pressures without bursting is explained using
A. Bernoulli’s principle B. Laplace’s law
C. Poiseuille–Hagen law D. Fahraeus–Lindqvist effect
143. Which of the following is usually associated with turbulence in blood flow?
A. Reynolds number less than 2000 B. Decrease in blood flow velocity
C. Decrease in density of blood D. Increase in diameter of blood vessel
144. Which of the following statements is/are correct?
A. Pulse pressure is directly proportional to stroke volume
B. Pulse pressure is inversely proportional to compliance of large arteries
C. Reflected arterial pulse waves normally serve to increase coronary perfusion during diastole
D. All the above
145. Filtration at the arterial end of capillary occurs mainly due to
A. Hydrostatic pressure in capillaries B. Hydrostatic pressure in interstitium
C. Oncotic pressure in capillaries D. Oncotic pressure in interstitium
146. Patients with acute cardiac failure may not have edema if
A. Oncotic pressure of plasma proteins is high B. Renal compensation occurs
C. Cardiac output is decreased D. There is a fall in systemic capillary hydrostatic pressure
147. Venous return is transiently increased during
A. Strain phase of Valsalva maneuver B. Positive end-expiratory pressure
C. Intravenous bolus of frusemide D. Deep inspiration
148. What is the chemical identity of endothelium-derived relaxing factor (EDRF)?
A. Nitrous oxide B. Nitric oxide
C. Potassium D. Carbon monoxide
149. Which of the following has a direct vasodilator effect on smooth muscle in arterioles in the presence of endothelial
dysfunction?
A. Acetylcholine B. Angiotensin II
C. Nitric oxide D. Norepinephrine

https://t.me/DentalBooksWorld
1304 Triumph’s Complete Review of Dentistry

150. Which of the following does not have vasodilator actions?


A. NO B. CO
C. Potassium D. Angiotensin III
151. Blood flow to exercising skeletal muscle is decreased by
A. Accumulation of K+ B. Norepinephrine
C. ↑ in PCO2 in muscle D. Adenosine
152. The most potent vasoconstrictor is
A. Endothelin 1 B. Angiotensin II
C. Norepinephrine D. Vasopressin
153. Loss of 500 ml of blood over 30 minutes will lead to
A. An increase in HR, fall in BP B. A fall in BP and HR
C. A prominent increase in HR and BP D. A slight increase in HR
154. The mechanism that regulates cerebral blood flow during cerebral compression is the
A. CNS ischemic response B. Cushing’s reflex
C. Bezold–Jarisch reflex D. Both A and B
155. Heart rate is slowed by
A. Deep inspiration B. Bainbridge reflex
C. Increased intracranial tension D. Carotid massage
156. Atrial natriuretic peptide is metabolized by
A. Dicarboxypeptidase B. Kininase II
C. Neutral endopeptidase D. Kininase I
157. Acute effect of bilateral clamping of the carotid arteries proximal to carotid sinuses is
A. Increase in heart rate and mean arterial pressure
B. Decrease in heart rate and mean arterial pressure
C. Decrease in heart rate and an increase in mean arterial pressure
D. Increase in heart rate and a decrease in mean arterial pressure
158. Normally, in an adult at rest, total blood flow to the brain is about
A. 250 ml/min B. 500 ml/min
C. 750 ml/min D. 1,200 ml/min
159. Which of the following is least likely to aggravate insult in an injured brain?
A. Hypercapnia B. Hypoxia
C. Hypotension D. Hypothermia
160. Areas where the blood–brain barrier is leaky include
A. Posterior pituitary B. Median eminence
C. Subfornical organ and pineal D. All the above
161. The blood–brain barrier is formed by
A. Tight junctions between vascular endothelial cells in the cerebral capillaries
B. Choroidal epithelial cells
C. Ependymal cells
D. Foot processes of oligodendroglia
162. The arteriovenous O2 concentration difference is highest across the
A. Brain B. Heart
C. Kidneys D. Liver
163. If the noradrenergic nerves to the heart are stimulated after giving a β-blocker, then what would be the effect on
coronary blood flow?
A. Coronary vasodilation B. Coronary vasoconstriction
C. No change D. Unpredictable

https://t.me/DentalBooksWorld
Chapter 15 • Physiology 1305

CNS
1. “No matter where a particular sensory pathway is stimulated along its course to the cortex, the conscious sensation
produced is referred to the location of the receptor.” This is
A. The law of projection B. Weber–Fechner law
C. Muller’s law of specific nerve energies D. Bell–Magendie law
2. The phantom limb phenomenon exemplifies
A. Bell–Magendie law B. The law of projection
C. Muller’s doctrine of specific nerve energies D. Weber–Fechner law
3. A single sensory axon and all of its peripheral branches constitute a
A. Receptive field B. Sensory unit
C. Dermatome D. Sensory nerve
4. Intensity of a sensory stimulus (whether threshold or subthreshold) is not encoded by
A. Size of generator (receptor) potentials B. Frequency of action potentials in sensory neurons
C. Recruitment of sensory units D. Size of action potentials
5. Which of the following is a rapidly adapting sensory receptor?
A. Muscle spindle B. Carotid sinus
C. Pain receptor D. Pacinian corpuscle
6. Which of the following fibers has the greatest threshold?
A. Touch B. Pain
C. Pressure D. Cold
7. The gate theory of pain was proposed by
A. Cannon and Bard B. Charles Sherrington
C. Wall and Melzack D. Weber and Fechner
8. Anterolateral cordotomy does not interfere with perception of
A. Fine touch B. Pain
C. Pressure D. Temperature
9. Ablation of somatosensory area (SI) does not significantly impair
A. Joint position sense B. Touch localization
C. Two-point discrimination D. Pain perception
10. Which sensory modality is transduced by free nerve endings?
A. Vision B. Taste
C. Smell D. Sound
11. A modified neuroepithelial cell is not the sensory receptor in the
A. Visual pathway B. Olfactory pathway
C. Auditory pathway D. Gustatory pathway
12. In which sensory system does excitation of the sensory receptor by an adequate stimulus result in hyperpolarization
of receptor cells?
A. Visual pathway B. Auditory pathway
C. Taste pathway D. Olfactory signaling
13. Axons of ganglion cells in the retina terminate in
A. Lateral geniculate nucleus B. Pretectal nucleus
C. Suprachiasmatic nucleus D. All the above
14. The light reflex is integrated in the
A. Midbrain B. Frontal eye field
C. Medulla D. Primary visual area
15. Which of the following is not a component of the near response?
A. Pupillary constriction B. Convergence of the visual axes
C. Increase in convexity of the lens D. Ciliary muscle relaxation
16. Which of the following is most sensitive to light?
A. Rods B. Cones maximally sensitive to light at 440 nm
C. Cones maximally sensitive to light at 535 nm D. Cones maximally sensitive to light at 565 nm
https://t.me/DentalBooksWorld
1306 Triumph’s Complete Review of Dentistry

17. Rod cell excitation by photons results in the conversion of


A. 11-cis-retinal to 11-trans-retinal B. 11-cis-retinal to all trans-retinal
C. All trans retinal to all cis-retinal D. 11-trans-retinal to 11-cis-retinal
18. During the dark phase of the visual cycle, which form of vitamin A combines with opsin to make rhodopsin?
A. All trans retinaldehyde B. All trans retinol
C. 11-cis retinaldehyde D. 11-cis retinol
19. Which of the following changes occur in rod cells when rhodopsin is activated by light?
A. Increase in cGMP B. Deactivation of phosphodiesterase
C. Depolarization of rod cells D. Decreased release of neurotransmitter
20. Which of the following reflexes is always absent in individuals who are totally blind?
A. Vestibulo-ocular reflex B. Nystagmus
C. Visual accommodation D. Light reflex
21. In the visual system, the term blobs refers to
A. Parvocellular pathway from the lateral geniculate nucleus
B. Clusters of axons of ganglion cells in the retina
C. Clusters of amacrine cells in the retina
D. Clusters of cells in layers 2 and 3 of the visual cortex that have high levels of cytochrome oxidase
22. What is the amplification provided by the lever action of the auditory ossicles and the large size of the tympanic
membrane compared to the oval window?
A. 5 times B. 10 times
C. 22 times D. 45 times
23. Which of the following functions as a frequency analyzer in hearing?
A. Reissner’s membrane B. Tectorial membrane
C. Basilar membrane D. Middle ear ossicles
24. Which of the following structures is most involved in localization of sound stimuli?
A. Superior olivary complex B. Inferior colliculus
C. Medial geniculate nucleus D. Auditory cortex
25. The fifth taste modality is
A. Umami B. Imami
C. Himami D. Emami
E. Mami
26. The receptor for the stretch reflex is the
A. Muscle spindle
B. Golgi tendon organ
C. Pacinian corpuscle
27. In the stretch reflex, the afferents are
A. I a fibers B. I b fibers
C. III fibers D. A delta fibers
28. In the stretch reflex, relaxation of antagonist muscles is due to
A. Reciprocal innervation B. Presynaptic inhibition
C. Ia fiber stimulation D. Antidromic conduction
29. The term “small motor nerve system” refers to
A. Ia afferents from muscle spindle B. α-Motor neurons
C. γ-Motor neurons D. None of the above
30. Physiologically, an intact innervated muscle can normally be made to contract by stimulation of which of the following
neurons innervating that muscle?
A. α-Motor neurons B. γ-Motor neurons
C. Either one of the above D. None of the above
31. The muscle spindle is not innervated by
A. Aα fibers B. Aγ fibers
C. Ia fibers D. II fibers

https://t.me/DentalBooksWorld
Chapter 15 • Physiology 1307

32. The principal receptor mediating proprioception is


A. Muscle spindle B. Golgi tendon organ
C. Mechanoreceptor D. Joint capsule receptor
33. The force of skeletal muscle contraction is sensed by
A. Nuclear bag fiber B. Nuclear chain fiber
C. Golgi tendon organ D. All the above
34. Tetanospasmin inhibits the release of
A. GABA B. Glutamate
C. Glycine D. Acetylcholine
35. All neural influences affecting muscle contraction ultimately funnel through
A. α-Motor neurons B. γ-Motor neurons
C. Corticospinal tract D. Basal ganglia
36. Which of the following is not a feature of lower motor neuron lesions?
A. Muscle atrophy B. Fasciculations
C. Fibrillations D. Spasticity
37. Righting reflexes are integrated for the most part in the
A. Medulla B. Midbrain
C. Spinal cord D. Cerebral cortex
38. The cerebellum receives sensory input from the
A. Labyrinth B. Proprioceptors
C. Eyes D. All of the above
39. Hypothalamus does not play a prominent role in the regulation of
A. Food and water intake B. Temperature
C. Respiration D. Osmolality
40. Exposure to darkness leads to increased melatonin secretion. It is brought about by
A. Decreasing the activity of suprachiasmatic nucleus
B. Increasing activity of serotonin N-acyl transferase
C. Decreasing the hydroxy-indole-O-methyl transferase activity.
D. Blocking the release of norepinephrine from sympathetic nerve terminals
41. The first physiologic response to high environmental temperature is
A. Cutaneous vasodilation B. Decreased heat production
C. Nonshivering thermogenesis D. Sweating

ENDOCRINOLOGY
1. The term “neurohormone” is applied to
A. Oxytocin and vasopressin B. NO and CO
C. Glycine and glutamate D. FSH and LH
2. Which of the following hormones does not act through G-protein coupled receptors?
A. Dopamine B. Epinephrine
C. Angiotensin II D. Retinoic acid
3. The group of chemical messengers whose actions are known to be mediated by receptor tyrosine kinases includes
A. Angiotensin II, ANP, and ET 1 B. EDRF, ANP, and AVP
C. FSH, LH, and GHRH D. Insulin, EGF, IGF 1, PDGF
4. JAK–STAT pathways mediate the effects of
A. Transducin B. Aquaporins
C. Gustducins D. Growth hormone
5. Which of the following hormones is not a glycoprotein?
A. TRH B. FSH
C. LH D. hCG

https://t.me/DentalBooksWorld
1308 Triumph’s Complete Review of Dentistry

6. The three glycoprotein hormones each containing 2 subunits (an alpha and a beta subunit) produced by the
pituitary are
A. FSH, LH, and TRH B. TRH, ACTH, and LH
C. TSH, LH, and FSH D. GH, TSH, and LH
7. Histamine released from mast cells in the stomach stimulates the secretion of HCl by parietal cells. What type of
signaling is this?
A. Endocrine B. Paracrine
C. Autocrine D. Juxtacrine
8. Which of the following has the longest biologic half-life?
A. Insulin B. Angiotensin II
C. Glucagon D. Thyroxine
9. Selective section of the pituitary stalk usually increases the secretion of which of the following hormones?
A. Growth hormone B. FSH
C. LH D. Prolactin
10. GH secretion is increased by all of the following except
A. Pharmacologic doses of hydrocortisone B. Hypoglycemia
C. Exercise D. Protein meal
11. Which of the following hormones has intrinsic lactogenic activity?
A. TSH B. MSH
C. GH D. Dopamine
12. The commonest cell type in the anterior pituitary is
A. Somatotroph B. Lactotroph
C. Corticotroph D. Thyrotroph
13. Check all correct statements regarding oxytocin
A. It is essential for milk ejection B. It increases uterine contractility
C. It facilitates ejaculation of semen D. All the above
14. ADH circulates in plasma primarily
A. Bound to neurophysin I B. Bound to neurophysin II
C. Bound to plasma albumin D. Not bound to plasma proteins
15. Normally, the thyroid gland secretes
A. CGRP B. PTH related peptide
C. TSH D. Calcitonin
16. Which of the following hormones lowers plasma level of ionized calcium?
A. Calcineurin B. Calcitonin
C. Parathyroid hormone D. PTH related peptide
17. Select all correct answers. TSH secretion is inhibited by, except
A. Dopamine B. Somatostatin
C. T3 and T4 D. TRH
18. For a euthyroid woman who is neither pregnant nor nursing, and taking no medication, the recommended daily
dietary allowance of iodine is
A. 75 µg B. 150 µg
C. 225 µg D. 300 µg
19. In a healthy euthyroid adult, thyroid gland predominantly secretes
A. Thyroxine B. Triiodothyronine
C. Reverse T3 D. All the above
20. Iodine is concentrated in thyroid follicular epithelial cells by
A. Primary active transport B. Secondary active transport
C. Simple diffusion D. Facilitated diffusion
21. Most of the T3 in the liver is formed from circulating T4 by the action of
A. Type 1 deiodinase (D1) B. Type 2 deiodinase (D2)
C. Type 3 deiodinase (D3) D. None of the above

https://t.me/DentalBooksWorld
Chapter 15 • Physiology 1309

22. Which of the following enzymes protects the fetus from hyperthyroidism when the mother is hyperthyroid?
A. Type I deiodinase; outer ring deiodinase B. Type 2 deiodinase (3’ deiodinase)
C. Type 3 deiodinase (D3); inner ring deiodinase D. Iodotyrosine deiodinase
23. Most of the circulating T4 is bound to
A. Prealbumin B. Albumin
C. Thyroxine-binding globulin D. All the above
24. Hyperthyroidism after administration of large amounts of iodine (iodine-induced hyperthyroidism; Jod-Basedow
mechanism) is least likely in an individual with which of the following?
A. Panhypopituitarism B. Endemic (iodine deficiency) goiter
C. Hashimoto’s thyroiditis D. Graves’ disease
25. The most abundant cell type in the islets of Langerhans is
A. A cells B. B cells
C. D cells D. F cells
26. Insulin secretion from B-cells in pancreas is inhibited by
A. Acetylcholine B. Activation of beta-adrenergic receptors
C. Activation of alpha-adrenergic receptors D. Glucagon
27. Which of the following is a potent stimulator of insulin secretion?
A. Somatostatin B. VIP
C. Glucagon like polypeptide-1 (GLP-1) D. Leptin
28. Insulin dependent glucose uptake into skeletal muscle and adipose tissue is mainly mediated by
A. GLUT 1 B. GLUT 2
C. GLUT 3 D. GLUT 4
29. Insulin secretion from beta cells of pancreas in response to a glucose load is mediated by
A. GLUT 1 B. GLUT 2
C. GLUT 3 D. GLUT 4
30. Which of the following conditions increases the risk of fasting hypoglycemia?
A. Adrenocortical sufficiency B. Hyperglucagonemia
C. Severe hypothyroidism D. Acromegaly
31. Which tissues do not require insulin for glucose uptake?
A. White blood cells B. Liver
C. Adipose tissue D. Skeletal muscle
32. The hormone of energy storage is
A. Growth hormone B. Thyroxine
C. Insulin D. Glucagon
33. Which of the following hormones is diabetogenic?
A. Epinephrine B. Cortisol
C. Growth hormone D. All the above
34. Glucagon, when present in high concentrations such as in a type I diabetic, facilitates all of the following except
A. Lipolysis B. Gluconeogenesis
C. Hepatic glycogenolysis D. Muscle glycogenolysis
35. Which of the following is the most sensitive test of pancreatic B-cell insulin secretory reserve?
A. Oral glucose tolerance test B. Fasting plasma glucose
C. Urine glucose excretion D. Random blood glucose
36. Which of the following increases insulin/glucagon molar ratio the most?
A. A large carbohydrate meal B. Intravenous glucose
C. Overnight fast D. A small protein meal
37. Hypoglycemia does not stimulate the secretion of
A. Epinephrine B. Cortisol
C. Growth hormone D. Insulin

https://t.me/DentalBooksWorld
1310 Triumph’s Complete Review of Dentistry

38. The satiety-producing hormone is


A. Orexin B. Resistin
C. Adiponectin D. Leptin
39. Insulin sensitivity is not reduced in
A. Glucagon-secreting tumors B. Tumors of adrenal medulla
C. Tumors of adrenal cortex D. Hypopituitarism
40. Somatostatin inhibits the secretion of
A. Insulin and gastrin B. Glucagon
C. Growth hormone D. All the above
41. Which of the following organelles is a major site of steroidogenesis?
A. Peroxisomes B. Ribosomes
C. Smooth endoplasmic reticulum D. Rough endoplasmic reticulum
42. In which zones is corticosterone converted to aldosterone?
A. Zona glomerulosa B. Zona fasciculata
C. Zona reticularis D. None of the above
43. The hormone that is synthesized only in the zona glomerulosa is
A. DHEA B. Aldosterone
C. Corticosterone D. Deoxycorticosterone
44. Which of the following hormones has the highest mineralocorticoid activity?
A. Cortisol B. Corticosterone
C. Deoxycorticosterone D. Cortisone
45. Which of the following hormones has maximum glucocorticoid activity?
A. Cortisol B. Corticosterone
C. Deoxycorticosterone D. Cortisone
46. Glucocorticoids decrease the number of circulating
A. Eosinophils B. Insulin
C. Platelets D. Red blood cells
47. The release of androgens from the adrenal cortex is stimulated mainly by
A. LH B. FSH
C. ACTH D. GnRH
48. Hormone that is mainly secreted by the adrenal medulla is
A. Epinephrine B. Norepinephrine
C. Dopamine D. Adrenomedullin
49. Adrenaline, noradrenaline, and dopamine act upon membrane receptors that span the membrane
A. 2 times B. 5 times
C. 7 times D. 10 times
50. The lipolytic effect of catecholamines is mediated by
A. ß1-Adrenergic receptors B. ß2-Adrenergic receptors
C. ß3-Adrenergic receptors D. ß1- and ß3-Adrenergic receptors
51. Which of the following hormones is critical for acute regulation of a falling serum ionized calcium?
A. Calbindin B. Calcitonin
C. Calcitriol D. Parathyroid hormone
52. The major stimulator of parathyroid hormone release from the parathyroid gland is
A. Vitamin D B. A fall in plasma ionized calcium
C. Calcitonin D. A fall in plasma phosphate concentration
53. Deficiency of which of the following hormones is not known to result in a clinically significant disorder of regulation
of ionized calcium in plasma?
A. Parathyroid hormone B. Calcitriol
C. Calcitonin D. Calcium sensing receptor

https://t.me/DentalBooksWorld
Chapter 15 • Physiology 1311

54. Calcitonin inhibits the activity of


A. Osteoblasts B. Osteoclasts
C. Parafollicular cells D. Thyroid
55. Bone resorption is induced by abnormally elevated levels of
A. IGF-1 B. Insulin
C. Estrogens D. Glucocorticoids
56. The biologically most active androgen is
A. Testosterone B. Dehydroepiandrosterone
C. Androstenedione D. Dihydrotestosterone
57. Which of the following cells undergo meiotic division?
A. Primordial germ cells B. Primary spermatocytes
C. Secondary spermatocytes D. Spermatids
58. Which of the following hormonal combinations in plasma is associated with normal lactation?
A. Cortisol + prolactin + oxytocin B. Estrogen (E) + progesterone (P) + prolactin
C. E + P + insulin + cortisol + prolactin D. Prolactin + oxytocin + E + P
59. Which of the following has an inhibitory influence on erythropoiesis?
A. Cortisol B. Growth hormone
C. Estradiol D. Thyroxine
60. Insulin increases actives of all the following enzymes, except
A. Pyruvate carboxylase B. Glucokinase
C. Acetyl-CoA carboxylase D. Glycogen synthase
61. Ovulation is associated with sudden rise in
A. Testosterone B. Prolactin
C. Oxytocin D. LH
62. Which of the following is true about milk secretion?
A. Neuroendocrine part of post pituitary is involved B. Secretion by contraction of lactiferous sinus
C. Oxytocin hormone is involved D. All of the above
63. Glucagon is secreted by
A. Gama cell B. C cell
C. Alpha cell D. Beta cell
64. Iodine uptake is seen in the following organs
A. Thyroid B. Salivary gland
C. Mammary gland D. All of the above
65. Insulin secrets in
A. Glucose B. Adrenaline
C. Calmodulin D. All of the above
66. In tetany, hyperexcitability is due to
A. Prevent Na+ and K+ release B. Decrease Ca++ produce generation of AP
C. Low Ca++ causes increase permeability to Na+ D. Prevent K+ release
67. Hypocalcemia due to calcitonin is by
A. Decreased bone resorption B. Increased excretion in kidney -
C. Decreased renal reabsorption D. Decreased intestinal absorption
68. Which of the following is the true statement about calcium?
A. Absorbed in upper small intestine B. Absorbed in lower small intestine
C. Absorption increased by alkaline pH D. All of the above
69. CAPM acts through
A. Increase Ca2+ release B. PIP3 pathway
C. Activation of protein kinase D. Activation of adenylate cyclase
70. What is the role of insulin in lipid metabolism?
A. Active lipoprotein lipase B. Active Acetyl CoA carboxylase
C. Decrease free fatty acid level D. All of the above

https://t.me/DentalBooksWorld
1312 Triumph’s Complete Review of Dentistry

71. Insulin is secreted along with the following molecule in a 1:1 ratio
A. Somatostatin B. C-peptide
C. Pancreatic polypeptide D. Glucagon
72. Hormone secreted by adenohypophysis are
A. TSH B. Gonadotropins
C. ACTH D. All of the above
73. Minimum fasting BGL for diagnosis of diabetes mellitus is
A. 120 mg/dl B. 114 mg/dl
C. 132 mg/dl D. 126 mg/dl
74. To synthesize insulin on a large basis, the most suitable starting material obtained from the beta cells of the
pancreas is
A. cDNA of insulin B. mRNA of insulin
C. Genomic DNA D. Total cellular RNA
75. All of these cause hyperglycemia except
A. Cortisol B. GH
C. Glucagon D. Insulin
76. Function of phospholamban is
A. Bins actin with myosin B. Collects calcium into the sarcoplasmic reticulum
C. Regulates Na K pump D. Transport calcium out of the mitochondria
77. Which of the following is True about intracellular receptor?
A. Mainly on nuclear surface B. Estrogen does not act on it
C. GH act on it D. All of the above
78. Sodium channels are specifically blocked by
A. Tetrodotoxin B. Nifedipine
C. Choline D. Tetraethyl lead
79. Hormone synthesized as peptide precursor is
A. Insulin B. PTH
C. Renin D. All of the above
80. Epinephrine action in the liver
A. Glycolysis B. Lipolysis
C. Glycogenolysis D. Gluconeogenesis
81. Parathyroid hormone is responsible for all actions except
A. Mobilizes calcium from bone B. Increase intestinal absorption of calcium
C. Absorption of phosphorous increase D. Vitamin D absorption increases
82. Which of the following act through tyrosine kinase receptor?
A. GH B. FSH
C. Insulin D. Glucagons
83. What is effect of cortisol on metabolism?
A. Increase neoglucogenesis B. Increase proteolysis
C. Increase protein anabolism in the liver D. All of the above
84. In stress which hormone is increased?
A. None B. Both A and B
C. Vasopressin D. Adrenaline
85. ACTH level is highest during
A. Afternoon B. Night
C. Early morning D. Evening
86. Secondary messengers include
A. cAPM B. IP3
C. Diacylglycerol D. All of the above
87. FSH acts on which of the following?
A. Endometrium B. Myometrium
C. Granulosa cell D. Theca interna

https://t.me/DentalBooksWorld
Chapter 15 • Physiology 1313

88. Intracellular receptors are found in


A. Corticosteroids B. Androgen
C. Thyroxine D. All of the above
89. Acromegaly is due to excess of
A. Growth hormone B. Somatomedin
C. Insulin D. Somatostatin
90. Stress-induced hyperglycemia is due to
A. Glucocorticoids B. GH
C. Epinephrine D. All of the above
91. Insulin secretion is/are increase by all except
A. VIP B. Gastrin
C. CCK D. GIP
92. Stress-induced hyperglycemia is mediated through which hormone
A. Glucagon B. Epinephrine
C. Cortisol D. All of the above
93. Which set of hormones have nuclear receptor?
A. Estrogen, TSH, and Gonadotropin releasing hormone (GnRH)
B. Testosterone, cortisol, and estrogen
C. Estrogen, Thyroxin, and Glucagon
D. Estrogen, Thyroxin, and TSH
94. All except which of the following changes are seen on the fifth day of fasting?
A. Decreased glucose tolerance B. Increase in FFA levels
C. Decreased level of insulin D. Decreased growth hormone
95. Exposure to darkness leads to increased melatonin secretion. It is brought about by
A. Increasing the serotonin N-acetyl transferase
B. Decreasing the activity of suprachiasmatic nuclei
C. Blocking the release of norepinephrine from sympathetic nerve terminals
D. Decreasing the hydroxy-indole-o-methyl transferase activity

GIT
1. In jaundice, there is an unconjugated hyperbilirubinemia which is most likely due to
A. Hepatitis B. Cirrhosis
C. Obstruction of bile canaliculi D. Increased breakdown of red cells
2. Bilirubin is conjugated with which of the following?
A. Glycine B. Glutamine
C. Acetyl CoA D. Glucuronic acid
3. Which of the following body secretions is maximum?
A. Salivary B. Gastric
C. Sweat D. Lacrimal
4. Histamine stimulates the secretion of
A. Gastrin by stomach B. Pancreatic enzymes
C. HCl by stomach D. Amylase by salivary gland
5. Bile acids are derived from
A. Bile salts B. Bile pigments
C. A and B D. Cholesterol
6. The most important function of hydrochloric acid in the stomach is
A. Destruction of bacteria B. Neutralization of chyme
C. Activation of pepsinogen D. Stimulation of pancreatic secretion
7. Pancreas produce
A. Pepsinogen B. Chymotrypsinogen
C. Hydrochloric acid D. All of the above

https://t.me/DentalBooksWorld
1314 Triumph’s Complete Review of Dentistry

8. Production of bile takes place in


A. The gall bladder B. Kupffer cells
C. Hepatic duct D. Hepatocytes
9. Intrinsic factor, which helps in absorption of Vit. B12 is produced by
A. Parietal cells of stomach B. Chief cells of stomach
C. Beta cells of pancreas D. Goblet cells
10. Pharyngeal phase of deglutition
A. Is a reflex mechanism B. Vocal cords are closed
C. Is a voluntary mechanism D. A and B
11. Nutrients are mainly absorbed in
A. Small intestine B. Large intestine
C. Liver D. Stomach
12. Pepsinogen is activated by
A. Enterokinase B. Low pH
C. Trypsin D. Chymotrypsin
13. Best stimuli for secretin secretion is
A. Protein B. Acid
C. Fat D. Bile
14. Maximum secretory glands in stomach are
A. Fundic glands B. Pyloric glands
C. Gastric glands D. Brunner’s glands
15. All are actions of CCK, except
A. Relax lower esophageal sphincter B. Increased pancreatic secretion
C. Increased gastric secretion D. Causes GB contraction
16. Vagal stimulation following intake of food does not affect the secretion of
A. Stomach B. Pancreas
C. Parotid D. Gall bladder
17. Gastrin is produced by
A. Pancreas B. Gastric antral cells
C. Pituitary D. All
18. Small intestinal peristalsis is controlled by
A. Myenteric plexus B. Meissner’s plexus
C. Vagus nerve D. Parasympathetic
19. The duodenum secretes a hormone which has following effects except
A. Causes copious pancreatic juice rich in bicarbonate and poor in enzymes
B. Increases gastric motility
C. Causes gall bladder to contract and sphincter of Oddi to relax
D. Leads to meagre flow of pancreatic juice rich in enzymes
20. Cholagogues are the substances, which cause
A. Contraction of gall bladder B. Increase concentration of bile
C. Increase secretion of bile D. Favors acidification of bile
21. Parietal cells of gastric mucosa secrete
A. HCl B. Gastrin
C. Mucin D. All of the above
22. Most potent stimulus for secretin is
A. Dilatation of intestine B. Acid chyme
C. Protein D. Fat
23. The only sugar normally absorbed in the intestine against a concentration gradient is
A. Xylose B. Mannose
C. Glucose D. Galactose

https://t.me/DentalBooksWorld
Chapter 15 • Physiology 1315

24. Cephalic phase of gastric secretion is mediated by


A. Neurohormones B. Vagus
C. Hormones D. Gastric
25. Secretin does not cause
A. Bicarbonate secretion B. Augments the action of CCK
C. Contraction of pyloric sphincter D. Gastric secretion increase
26. The final sugars in intestinal chyme are
A. Glucose and Fructose B. Ribose and Mannose
C. Ribose and Xylulose D. Xylulose and Fructose
27. Gastric secretion is stimulated by all the following, except
A. Secretin B. Gastric distension
C. Gastrin D. Vagal stimulus
28. Peristalsis in the gut is due to
A. Pre-peristaltic intestinal secretion
B. Mechanical distension
C. Simultaneous action of circular and longitudinal muscles
D. Extrinsic nervous influence
29. Maximum absorption of bile occurs at
A. Jejunum B. Duodenum
C. Ileum D. Colon
30. Gallbladder contraction is controlled primarily by the following hormone
A. Pancreatozymine B. Cholecystokinin-pancreozymin
C. Secretin D. Glucagon
31. Pancreatic juice rich in water and electrolytes but poor in enzymes is secreted in response to
A. Pancreatozymine B. Cholecystokinin
C. Secretin D. Proteins
32. Secretin is secreted by
A. Duodenum B. Pancreas
C. Liver D. Stomach
33. Urine urobilinogen is absent in
A. Obstructive jaundice B. Viral hepatitis
C. Hemolytic jaundice D. All of the above
34. Which of the following is a reflex mediated by vagus?
A. Bile flow from the liver B. Pancreatic secretion of bicarbonate
C. Cephalic phase of gastric secretion D. Mucous secretion from the Brunner’s glands
35. Fats absorbed with the help of bile salts are
A. Free fatty acids B. Glycerol
C. Higher fatty acids, diglycerides, and monoglycerides D. All of the above
36. Which of the following secretions has a very high pH?
A. Gastric juice B. Pancreatic juice
C. Bile in gall bladder D. Saliva
37. Cephalic phase of gastric secretion can be demonstrated by the following experiment
A. Heidenhain’s pouch B. Condition reflex
C. Pavlov’s pouch D. None of the above
38. Trypsin is an activator of all of the following enzymes, except
A. Chymotrypsinogen B. Pepsinogen
C. Proelastase D. Procolipase
39. Gastrointestinal hormone in the following is
A. Pepsin B. Ptyalin
C. Cholecystokinin D. Trypsin

https://t.me/DentalBooksWorld
1316 Triumph’s Complete Review of Dentistry

40. The most important action of secretin is to


A. Neutralize the acid from the stomach B. Increase secretion of bicarbonates by pancreas
C. Decrease gastric secretion D. Cause contraction of pyloric sphincter
41. Within which parts of a gastric gland are chief cells located?
A. Fundus B. Isthmus
C. Neck D. Gastric pit
42. All of the following causes the secretion of gastric juice during cephalic phase, except
A. Food in the mouth B. Sight of food
C. Food in the stomach D. Thought of food
43. Which is not produced enteroendocrinally?
A. Intrinsic factor B. Secretin
C. Motilin D. GIP
44. Peristalsis involves coordinated contraction and relaxation
A. Above and below the food bolus B. Lateral and medial to food bolus
C. Dorsal and ventral to food bolus D. None of the above
45. Stomach accommodates the meal by
A. Deceptive relaxation B. Receptive relaxation
C. Reactive relaxation D. None of the above
46. The intrinsic factor for vitamins B12 absorption is produced in the
A. Liver B. Stomach
C. Pancreas D. Duodenum
47. Bile salt repeats its cycle in
A. 4 times a day B. 2 times in between meals
C. 8 times a day D. 10 times a day in fasting
48. Bile color is due to
A. Creatinine B. Bilirubin
C. Globulin D. 5 mercaptan purine
49. Which of the following is not a salivary antibacterial substance?
A. Amylase B. Lactoperoxidase
C. Lysozyme D. Lactoferrin
50. Gastric secretions stimulated by all of the following except
A. Vagal stimulus B. Gastrin
C. Secretin D. Gastric distension
51. Most potent stimulus for secretin is
A. Acid chyme B. Dilatation of intestine
C. Fat D. Protein
52. Pacemaker of the GIT is located in
A. Long muscle of small intestine B. Cardiac end of stomach
C. Funds of stomach D. Pyloric end of stomach
53. Bilirubin is derived from
A. Cholesterol B. Amino acids
C. Myoglobin D. Muscle
54. All of the following are secreted proenzyme from except
A. Pepsis B. Ribonuclease
C. Trypsin D. Chymotrypsin
55. Glossitis and cheilitis are seen with which vitamin deficiency?
A. Vit. K B. Vit. D
C. Vit. B2 D. Vit. B12
56. Stimulation for gastric emptying
A. Secretin B. CCK
C. Gastrin D. All of the above

https://t.me/DentalBooksWorld
Chapter 15 • Physiology 1317

57. Intrinsic factor is secreted from


A. Chief cells B. Oxyntic cells
C. Mucous secreting cells D. All of the above
58. Which of the following is absorbed in the colon?
A. Bile salts B. Electrolytes
C. Iron D. Proteins
59. Physiological gastrectomy is
A. Upper one-third of stoma resected B. Ligation of four out of five arteries
C. Ligate all major arteries D. Antrectomy
60. In the following food items, which has the highest “Glycemic Index”?
A. Ice-cream B. Whole white bread
C. Corn-flakes D. Brown rice
61. Factors responsible for causing diarrhea after vagotomy are, except
A. Irregular peristalsis B. None
C. Rapid gastric emptying D. Hypoacidity in duodenum
62. Which of the following does not simulate enterogastric reflex?
A. H+ ions bathing duodenal mucosa B. Hormones
C. Products of protein digestion in the duodenum D. Duodenal distension
63. True about dietary fiber
A. Increase GI transit time B. Not prevention against colonic cancer
C. Increase bulk of stool D. Only soluble fibers are included in diet
64. Gastrin is produced by
A. Pancreas B. Gastric antral cells
C. Pituitary D. All of the above
65. Best stimuli for secretin is
A. Fat B. Protein
C. Bile D. Acid
66. Gastro-colic reflex is related to
A. Pendular movement B. Colonic cases
C. Mass peristalsis D. Segmental movement
67. The amount of water absorbed in the intestine in a day is
A. 10 l B. 8 l
C. 5 l D. 1 l
68. Fat is maximally absorbed in
A. Stomach B. Jejunum
C. Ileum D. Colon
69. Lower esophageal sphincter
A. Relaxes on increasing abdominal pressure B. Relaxes ahead on the peristaltic wave
C. Has no tonic activity D. Has no tone which is provided by the sympathetic system
70. Function of peritoneum are, except
A. Hormone release B. Lubrication
C. Enzymatic digestion D. Pain sensitive

RENAL
1. The “gold standard” for estimation of glomerular filtration rate is the estimation of urinary clearance of
A. Inulin B. Creatinine
C. Urea D. Mannitol
2. In clinical practice, the urinary clearance of which substance is most frequently estimated as a surrogate of GFR?
A. Inulin B. Creatinine
C. Urea D. Mannitol

https://t.me/DentalBooksWorld
1318 Triumph’s Complete Review of Dentistry

3. Which of the following substances should not be used to measure GFR?


A. Inulin B. Creatinine
C. Iothalamate D. Glucose
4. The term filtration fraction is used to refer to
A. RPF/GFR B. GFR/RPF
C. RPF × GFR D. All the above
5. What fraction of renal plasma flow is normally filtered in the glomerular capillaries?
A. 0.1 B. 0.2
C. 0.3 D. 0.4
6. When mean arterial pressure is held constant, selective constriction of glomerular efferent arterioles
A. Increases GFR B. Decreases filtration fraction
C. Decreases renal plasma flow D. All the above
7. Mesangial cells are similar to
A. Pericytes B. Fibrocytes
C. Mast cells D. Ependymal cells
8. What percentage of nephrons has long loops of Henle?
A. 5 B. 15
C. 25 D. 85
9. The macula densa is located in the
A. Juxtaglomerular cells B. Extraglomerular mesangium
C. Beginning of distal tubule D. Peritubular capillaries
10. The kidneys do not synthesize or secrete
A. Calcitriol B. Renin
C. Erythropoietin D. PGE2
11. The brush border on the luminal membrane is most extensive in cells of the
A. PCT B. Loop of Henle
C. DCT D. Collecting duct
12. Which of the following increases renal cortical blood flow?
A. Prostaglandin E2 B. Norepinephrine
C. Angiotensin II D. Vasopressin
13. Type I renal medullary interstitial cells secrete
A. Renin B. PGE2
C. Aquaporins D. Aldosterone
14. Which of the following substances is cleared the most by the kidneys?
A. Inulin B. PAH
C. Creatinine D. Urea
15. Which of the following substances normally has a “clearance ratio” of about 1.2?
A. Inulin B. Creatinine
C. Urea D. PAH
16. Which of the following substances has a clearance ratio close to 1?
A. Creatinine B. Mannitol
C. Urea D. Glucose
17. Arrange the following substances in the descending order of their clearance [Cr: creatinine; PAH: para-aminohippuric
acid]
A. PAH > Inulin > Urea > Cr > Glucose B. Inulin > PAH > Cr > Urea > Glucose
C. PAH > Cr > Inulin > Urea > Glucose D. Cr > PAH > Inulin > Glucose > Urea
18. In a healthy individual with a normal renal plasma flow, in which segment of the nephron would the concentration of
PAH be the least?
A. Proximal convoluted tubule B. Thin descending limb of loop of Henle
C. Thick ascending limb of loop of Henle D. Cortical collecting duct

https://t.me/DentalBooksWorld
Chapter 15 • Physiology 1319

19. Autoregulation of GFR is achieved through changes in


A. Renal perfusion pressure B. Renal blood flow
C. Glomerular afferent arteriolar resistance D. Glomerular efferent ventricular resistance
20. The rate-limiting step in the biosynthesis of angiotensin II is the secretion of
A. Renin B. Angiotensinogen
C. Angiotensin I D. Angiotensin II
21. Renin in the circulation originates mainly from
A. Macula densa B. Juxtaglomerular cells
C. Type I medullary interstitial cells D. Peritubular capillaries
22. The substrate for renin is
A. Angiotensinogen B. Angiotensin I
C. Angiotensin II D. Bradykinin
23. Angiotensin converting enzyme is a/an
A. Dipeptidyl carboxypeptidase B. Aminopeptidase
C. Aspartyl protease D. None of the above
24. Physiologically, important stimulators of aldosterone production by the zona glomerulosa cells include
A. A rise in plasma [K+] B. Angiotensin II
C. Angiotensin III D. All the above
25. Which hormone directly induces proton secretion by alpha-intercalated cells in the collecting ducts?
A. Aldosterone B. Atrial natriuretic peptide
C. Vasopressin D. Parathyroid hormone
26. In the nephron, glucose reabsorption occurs mainly in the
A. Proximal tubule B. Loop of Henle
C. Distal convoluted tubule D. Collecting duct
27. In the presence of vasopressin, most of the water filtered by the kidneys is reabsorbed in the
A. Proximal tubule B. Thin descending limb
C. Thick ascending limb D. Collecting ducts
28. The tubular fluid at the end of the PT is ____ with respect to plasma
A. Isotonic B. Hypotonic
C. Hypertonic D. Isometric
29. Which is the “concentrating segment” of the nephron?
A. Descending limb of loop of Henle B. Thin ascending limb of loop of Henle
C. Thick ascending limb of loop of Henle D. Collecting ducts
30. Which portion of the tubule is least permeable to water?
A. Proximal tubule B. Thin descending limb of Henle’s loop
C. Thick ascending limb of Henle’s loop D. Cortical collecting duct
31. Which is the fundamental mechanism generating hypertonicity in the medullary interstitium?
A. Active transport of NaCl in the thick ascending limb
B. Action of ADH in the collecting ducts
C. Passive recirculation of NaCl in the medullary interstitium
D. Urea permeability of collecting ducts
32. What does the macula densa sense?
A. Na concentration of fluid delivered to it B. Cl concentration of the fluid delivered to it
C. Volume of fluid delivered to it D. Both B and C
33. The “diluting segment” of the nephron is
A. Proximal tubule B. Descending limb of loop of Henle
C. Thick ascending limb of loop of Henle D. Cortical collecting duct
34. “Free water” is formed mainly in the
A. Descending limb of loop of Henle B. Thin ascending limb of loop of Henle
C. Thick ascending limb of loop of Henle D. Collecting duct

https://t.me/DentalBooksWorld
1320 Triumph’s Complete Review of Dentistry

35. What fraction of filtered water is reabsorbed in the loop of Henle?


A. 15% B. 25%
C. 35% D. 5%
36. What fraction of filtered Na is reabsorbed in the loop of Henle?
A. 15% B. 25%
C. 35% D. 5%
37. In the absence of medication or disease processes affecting transport in the nephron, fluid entering the early distal
tubule is
A. Always isotonic B. Always hypotonic
C. Always hypertonic D. Isotonic or hypotonic
38. Principal cells (P cells) in the cortical collecting duct of the nephron are mainly involved in
A. Secretion of protons B. Potassium reabsorption
C. Reabsorption of sodium and chloride D. All the above
39. Vasa recta receive blood from
A. Afferent arterioles of cortical nephrons B. Afferent arterioles of juxtamedullary nephrons
C. Efferent arterioles of cortical nephrons D. Efferent arterioles of juxtamedullary nephrons
40. Hypertonicity in the inner medulla is maintained because
A. vasa recta receive 50% of renal blood flow
B. Of the low blood flow rate through vasa recta
C. NaCl passively moves into the medullary interstitium from the thin ascending limb
D. None of the above
41. The system for the formation of concentrated or dilute urine does not include the
A. Proximal convoluted tubule B. Loop of Henle
C. Collecting ducts D. Vasa recta
42. Urea is reabsorbed from
A. Thick ascending limb of loop of Henle B. Distal convoluted tubule
C. Cortical collecting ducts D. Inner medullary collecting ducts
43. The reabsorption of urea by the inner medullary collecting ducts is dependent on facilitation by
A. Angiotensin II B. Aldosterone
C. Antidiuretic hormone D. Atrial natriuretic peptide
44. Which segment of the nephron is permeable to urea?
A. Descending limb of loop of Henle B. Thin ascending limb of loop of Henle
C. Thick ascending limb of loop of Henle D. Proximal convoluted tubule
45. Plasma potassium concentration is chiefly regulated by
A. Aldosterone B. Vasopressin (ADH)
C. Renin D. ANP
46. Which ion is both secreted and absorbed by the nephron?
A. Na B. K
C. Cl D. Ca
47. Potassium secretion occurs mainly in
A. PT B. Early distal tubule
C. Late distal tubule D. Cortical collecting ducts
48. Which of the following inhibits ADH secretion?
A. Hyperosmolality B. Exercise
C. Angiotensin II D. Ethanol
49. In a healthy adult, the urge to void urine is first felt at a bladder volume of about
A. 50 ml B. 150 ml
C. 250 ml D. 350 ml
50. In a healthy adult, the volume of urine in the bladder that initiates reflex contraction of the bladder is about
A. 50–100 ml B. 100–200 ml
C. 200–300 ml D. 300–400 ml

https://t.me/DentalBooksWorld
Chapter 15 • Physiology 1321

51. The most abundant buffer in proximal tubular fluid is


A. Bicarbonate B. Phosphate
C. Ammonia D. None of the above
52. Most of the filtered HCO3 is reabsorbed from
A. PT B. Distal tubule
C. cortical collecting duct D. Loop of Henle
53. In the nephron, the highest amount of H ions is secreted into tubular lumen in the
A. PCT B. DCT
C. Collecting ducts D. Loop of Henle
54. The principal mechanism of H+ secretion in the proximal convoluted tubule is via
A. Na–H exchanger B. H–ATPase
C. H–K ATPase D. H–Ca exchanger
55. The principal site of ammoniagenesis in the nephron is
A. PT B. Loop of Henle
C. Early distal tubule D. Late distal tubule
56. The principle site of acidification of tubular fluid is
A. Proximal convoluted tubule B. Loop of Henle
C. The aldosterone sensitive distal nephron D. Urinary bladder
57. The principal mechanism for acid secretion by type A intercalated cells in the nephron is
A. H–ATPase B. Na–H exchanger
C. H–Ca exchanger D. H–K ATPase
58. Normally, in urine, most of the H+ is tied up with
A. Bicarbonate B. Phosphate
C. Ammonia D. All of the above
59. Normally, most of the titratable acidity of urine is attributable to acid buffered by
A. Bicarbonate B. Phosphate
C. Ammonia D. Uric acid
60. Type B intercalated cells in the collecting duct are concerned with
A. Secretion of protons B. Controlled bicarbonate secretion
C. ADH-stimulated water reabsorption D. Na reabsorption
61. In arterial blood gas analysis, which of the following is calculated?
A. Arterial pH B. Plasma bicarbonate
C. PaCO2 D. PaO2
62. The most common acid–base disturbance in patients who are on mechanical ventilators is
A. Metabolic acidosis B. Metabolic alkalosis
C. Respiratory acidosis D. Respiratory alkalosis
63. Which of the following is not elevated in ethylene glycol poisoning?
A. Anion gap B. Base excess
C. Osmolar gap D. Serum osmolality
64. The severity of acidosis is related to
A. Anion gap B. pH of arterial plasma
C. pH of ICF and plasma D. Plasma HCO3
65. The high sodium content of the filtrate in renal medulla is because
A. Increased excretion of Na+ from PCT
B. Increased absorption of Na+ from PCT
C. At the top Henle, there is counter current mechanism
D. Increased blood flow to vasa recta
66. Transport maximum (TM) means
A. Substance cleared from plasma/min B. Amount of toxic substances excreted/min
C. Maximum reabsorption and secretion D. Maximum amount of glomerular filtration/min

https://t.me/DentalBooksWorld
1322 Triumph’s Complete Review of Dentistry

67. Several hormones regulate the tubular reabsorption of water and electrolytes at different sites in the nephron. Which
of the following combination is correct?
A. Aldosterone in collecting ducts B. Angiotensin in distal tubule
C. ANP in loop of Henle D. ADH in proximal tubule
68. The length of distal convoluted tubule is
A. 5 mm B. 2 mm
C. 12 mm D. 8 mm
69. Vasopressin acts by
A. Water secretion at loop Henle B. Water transport at PCT
C. Water transport across collecting duct D. Water absorption at medullary ducts
70. Which of the following has no Tm value?
A. Sulfate, Uric acid B. Urea
C. Albumin, arginine D. Glucose hemoglobin, phosphate
71. GFR increases if
A. Afferent arteriole constricts B. Afferent arteriole dilates
C. Efferent arteriole constricts D. Efferent arteriole dilates
72. JG cells (Juxtaglomerular apparatus) are
A. Macula densa B. Smooth muscular cells of afferent arteriole
C. Islets of epithelial cells D. All of the above
73. Substance involved in countercurrent mechanism for maintaining medullary gradient
A. NaCl, urea B. NaCl, urea, water
C. NaCl D. Urea
74. In renal glycosuria, the renal threshold for glucose is
A. Same B. Greatly increased
C. Low D. High
75. Relaxation of mesangial cells of kidney is brought about by
A. PGF2 B. Vasopressin
C. cAMP D. Endothelin
76. Which is true about rennin?
A. Increase GFR, which cause increase in secretion of rennin
B. Increase plasma Na+ and H2O
C. It helps to convert angiotensinogen to Angiotensin-I
D. Secreted by PCT
77. All of the following are true regarding rennin angiotensin system, except
A. Aspartic acid is essential for rennin activity B. Angiotonin is an octapeptide
C. Catalysis site of ACE contain Zn++ D. PAR is better than RPC
78. Of the following, all except one, result in increased secretion of Renin
A. Decreased amount of Na+ in DCT B. Renal ischemia
C. Narrowing of afferent arterioles D. Decreased amount of Na+ in PCT
79. Renal medullary hyperosmolarity is due to
A. Increased Na+ B. Increased urea
C. Increased potassium D. All of the above
80. Which of the following is true about function of Angiotensin II?
A. Constriction afferent arteriole B. Autoregulation of GFR
C. Release aldosterone D. All of the above
81. In a normal person at resting condition, GFR is
A. 60 ml/min B. 150 ml/min
C. 125 ml/min D. 90 ml/min
82. Metabolic alkalosis is seen in all except
A. Ureterosigmoidostomy B. Thiazide
C. Systemic antacid therapy D. Prolonged vomiting

https://t.me/DentalBooksWorld
Chapter 15 • Physiology 1323

83. Increase anion gap is seen in


A. DKA B. Cardiac failure
C. Lactic acidosis D. All of the above
84. Which of the following is freely filtered by kidney across glomerular capillaries?
A. Globulin B. Albumin (across glomerular capillaries)
C. HCO3 D. Creatinine
85. The urge for micturition is left when the bladder is filled with
A. 500–700 cc of urine B. 700–800 cc of urine
C. 100–200 cc of urine D. 200–300 cc of urine
86. Daily loss of water from skin in absence of sweating is
A. 1.5 l B. 1 l
C. 500–700 ml D. 200–300 ml
87. Best test for GFR is with
A. Creatinine B. PAH
C. Inulin D. Hippuric acid
88. Inulin clearance is equal to
A. 625 ml/min B. 55 ml/min
C. 40 ml/min D. 125 ml/min
89. Inulin clearance closely resembles
A. Creatinine clearance B. PAH clearance
C. GFR D. Renal plasma flow
90. Erythropoietin is secreted by all, except
A. Renal cell carcinoma B. Adrenocortical tumors
C. Hemangioblastoma D. Hepatoma
91. Which one of the following is not responsible for concentration of urine in the kidney?
A. Vasopressin B. Epinephrine
C. Aldosterone D. Angiotensin II
92. Renal medullary hyperosmolarity is due to
A. Increased Na B. Increased K
C. Urea D. All of the above
93. Which of the following cells are responsible for acid secretion in kidney?
A. Mesangial cell B. Pericytes cell
C. I cells D. P cells
94. Physiologically inactive form is
A. Angiotensin III B. Angiotensin IV
C. Angiotensin I D. Angiotensin II
95. Tubular maximum for kidney in practice is actually less than the calculated value because
A. Depends on renal blood flow
B. Depends on blood pressure
C. Different nephrons have different transport maximum
D. Depends on GFR
96. The most sensitive index for renal tubular function is
A. GFR B. Creatinine clearance
C. Specific gravity of urine D. Blood urea
97. Normal anion gap metabolic acidosis is seen in-
A. Lactic acidosis B. Diabetic ketoacidosis
C. Diarrhea D. Renal failure
98. Two substances that can probably be used to determine filtration fraction are
A. PAH and phenol red B. Insulin and PAH
C. Insulin and mannitol D. Urea and diodrast

https://t.me/DentalBooksWorld
1324 Triumph’s Complete Review of Dentistry

99. Active reabsorption of glucose occurs in the


A. Proximal tubule B. Distal tubule
C. Collecting ducts D. Loop of Henle
100. Each kidney contains about ______ nephrons
A. Four million B. Five million
C. One million D. Two million
101. Filtration pressure in glomeruli of kidney
A. 6 mm of Hg B. 10 mm of Hg
C. 2.0 mm of Hg D. 15 mm of Hg
102. What is implied, if a drug has more renal clearance than the GFR?
A. Drug is secreted in the tubules B. Drug is reabsorbed in the tubules
C. Drug is neither secreted, nor resorbed D. Drug is excreted in bile

ANSWERS

CELL PHYSIOLOGY
1. Answer: A (Ref. Guyton and Hall Textbook of Medical Physiology, By John E. Hall, 13th edition, page no. 22)
2. Answer: C (Ref. Guyton and Hall Textbook of Medical Physiology, By John E. Hall, 13th edition, page no. 376)
3. Answer: D (Ref. Guyton and Hall Textbook of Medical Physiology, By John E. Hall, 13th edition, page no. 381)
4. Answer: D (Ref. Guyton and Hall Textbook of Medical Physiology, By John E. Hall, 13th edition, page no. 421)
5. Answer: D (Ref. Guyton and Hall Textbook of Medical Physiology, By John E. Hall, 13th edition, page no. 452)
• Glycophorin is a sialoglycoprotein of the membrane of a RBC.
• Glycophorins are rich in sialic acid, which gives the red blood cells a very hydrophilic-charged coat.
• It is also a membrane-spanning protein and carries sugar molecules. It is heavily glycosylated (60%). 
6. Answer: A (Ref. Guyton and Hall Textbook of Medical Physiology, By John E. Hall, 13th edition, page no. 64)
7. Answer: A (Ref. Guyton and Hall Textbook of Medical Physiology, By John E. Hall, 13th edition, page no. 900)
8. Answer: D (Ref. Guyton and Hall Textbook of Medical Physiology, By John E. Hall, 13th edition, page no. 21)
9. Answer: D (Ref. Guyton and Hall Textbook of Medical Physiology, By John E. Hall, 13th edition, page no. 19)
10. Answer: B (Ref. Guyton and Hall Textbook of Medical Physiology, By John E. Hall, 13th edition, page no. 17)
11. Answer: D (Ref. Guyton and Hall Textbook of Medical Physiology, By John E. Hall, 13th edition, page no. 37)
12. Answer: A (Ref. Guyton and Hall Textbook of Medical Physiology, By John E. Hall, 13th edition, page no. 22)
13. Answer: B (Ref. Guyton and Hall Textbook of Medical Physiology, By John E. Hall, 13th edition, page no. 48)
14. Answer: D (Ref. Guyton and Hall Textbook of Medical Physiology, By John E. Hall, 13th edition, page no. 15)
15. Answer: A (Ref. Guyton and Hall Textbook of Medical Physiology, By John E. Hall, 13th edition, page no. 12)
16. Answer: D (Ref. Guyton and Hall Textbook of Medical Physiology, By John E. Hall, 13th edition, page no. 112)
17. Answer: B (Ref. Guyton and Hall Textbook of Medical Physiology, By John E. Hall, 13th edition, page no. 468)
18. Answer: C (Ref. Guyton and Hall Textbook of Medical Physiology, By John E. Hall, 13th edition, page no. 56)
19. Answer: B (Ref. Guyton and Hall Textbook of Medical Physiology, By John E. Hall, 13th edition, page no. 21)
20. Answer: D (Ref. Guyton and Hall Textbook of Medical Physiology, By John E. Hall, 13th edition, page no. 68)
21. Answer: A (Ref. Guyton and Hall Textbook of Medical Physiology, By John E. Hall, 13th edition, page no. 47)
22. Answer: C (Ref. Guyton and Hall Textbook of Medical Physiology, By John E. Hall, 13th edition, page no. 1016)
23. Answer: A (Ref. Guyton and Hall Textbook of Medical Physiology, By John E. Hall, 13th edition, page no. 22)
24. Answer: B
• Mucopolysaccharidoses are a group of metabolic disorders caused by the absence or malfunctioning of lysosomal
enzymes needed to break down molecules called glycosaminoglycans.
• Mucopolysaccharidoses are part of the lysosomal storage diseases, a group of more than 40 genetic disorders that result
when the lysosome organelle malfunctions.
25. Answer: A (Ref. Guyton and Hall Textbook of Medical Physiology, By John E. Hall, 13th edition, page no. 309)
26. Answer: C (Ref. Guyton and Hall Textbook of Medical Physiology, By John E. Hall, 13th edition, page no. 101)

https://t.me/DentalBooksWorld
Chapter 15 • Physiology 1325

27. Answer: B (Ref. Guyton and Hall Textbook of Medical Physiology, By John E. Hall, 13th edition, page no. 20)
28. Answer: C (Ref. “Previous question”)
29. Answer: C (Ref. Guyton and Hall Textbook of Medical Physiology, By John E. Hall, 13th edition, page no. 448)
30. Answer: B (Ref. “Previous question”)
31. Answer: C (Ref. Guyton and Hall Textbook of Medical Physiology, By John E. Hall, 13th edition, page no. 38)
32. Answer: C (Ref. “Previous question”)
33. Answer: A (Ref. Guyton and Hall Textbook of Medical Physiology, By John E. Hall, 13th edition, page no. 956)
34. Answer: B (Ref. “Previous question”)
35. Answer: C (Ref. “Previous question”)
36. Answer: A (Ref. “Previous question”)
37. Answer: A (Ref. Guyton and Hall Textbook of Medical Physiology, By John E. Hall, 13th edition, page no. 338)
38. Answer: B (Ref. “Previous question”)
39. Answer: A (Ref. “Previous question”)
40. Answer: C (Ref. Guyton and Hall Textbook of Medical Physiology, By John E. Hall, 13th edition, page no. 5)
41. Answer: C (Ref. “Previous question”)
42. Answer: D (Ref. Guyton and Hall Textbook of Medical Physiology, By John E. Hall, 13th edition, page no. 19)
43. Answer: D (Ref. “Previous question”)
44. Answer: D (Ref. “Previous question”)
45. Answer: D (Ref. Guyton and Hall Textbook of Medical Physiology, By John E. Hall, 13th edition, page no. 24)
46. Answer: C (Ref. “Previous question”)
47. Answer: B (Ref. “Previous question”)
48. Answer: A (Ref. “Previous question”)
49. Answer: C (Ref. Guyton and Hall Textbook of Medical Physiology, By John E. Hall, 13th edition, page no. 382)
50. Answer: D (Ref. “Previous question”)
51. Answer: A (Ref. “Previous question”)
52. Answer: B (Ref. “Previous question”)
53. Answer: C (Ref. Guyton and Hall Textbook of Medical Physiology, By John E. Hall, 13th edition, page no. 19)
54. Answer: D (Ref. Guyton and Hall Textbook of Medical Physiology, By John E. Hall, 13th edition, page no. 211)
55. Answer: D (Ref. Guyton and Hall Textbook of Medical Physiology, By John E. Hall, 13th edition, page no. 315)
56. Answer: C (Ref. “Previous question”)
57. Answer: D (Ref. “Previous question”)
58. Answer: B (Ref. “Previous question”)
59. Answer: D (Ref. “Previous question”)
60. Answer: D (Ref. Guyton and Hall Textbook of Medical Physiology By John E. Hall, 13th edition, page no. 51)

GENERAL PHYSIOLOGY
1. Answer: C (Ref. A Guyton and Hall Textbook of Medical Physiology, By John Edward Hall, 13th edition, page no. 943)
2. Answer: C (Ref. A Guyton and Hall Textbook of Medical Physiology, By John Edward Hall, 13th edition, page no. 894)
3. Answer: B (Ref. A Guyton and Hall Textbook of Medical Physiology, By John Edward Hall, 13th edition, page no. 392)
4. Answer: B (Ref. A Guyton and Hall Textbook of Medical Physiology, By John Edward Hall, 13th edition, page no. 352)
5. Answer: C (Ref. A Guyton and Hall Textbook of Medical Physiology, By John Edward Hall, 13th edition, page no. 969)
Children and adults have the same concentration of sodium in plasma, even though ECF volumes are greatly different.
So, ECF volume is proportional to the amount of sodium contained in ECF, not to the concentration of this ion.
6. Answer: A (Ref. A Guyton and Hall Textbook of Medical Physiology, By John Edward Hall, 13th edition, page no. 312)
ICF contains more osmoles than ECF because it is the larger of the two compartments.
7. Answer: D (Ref. A Guyton and Hall Textbook of Medical Physiology, By John Edward Hall, 13th edition, page no. 426)
8. Answer: B (Ref. A Guyton and Hall Textbook of Medical Physiology, By John Edward Hall, 13th edition, page no. 54)
• Osmolality of a solution depends upon the number of osmoles per unit volume of the solution.
• One Na ion and one albumin molecule exert the same osmotic effect.
• The concentration of Na in plasma is 140 mmol/l.
• The concentration of albumin in plasma is much less.

https://t.me/DentalBooksWorld
1326 Triumph’s Complete Review of Dentistry

• Hence, the contribution of the proteins to osmolality of plasma is negligible.


• It is why they are not included in the equation used to calculate serum osmolality.
9. Answer: B (Ref. A Guyton and Hall Textbook of Medical Physiology, By John Edward Hall, 13th edition, page no. 54)
• Osmolality of normal human plasma: 275–295 mOsm/l (approx.)
• Osmolar concentration of sodium: 135–145 mOsm/l.
10. Answer: B (Ref. A Guyton and Hall Textbook of Medical Physiology, By John Edward Hall, 13th edition, page no. 54)
11. Answer: A (Ref. A Guyton and Hall Textbook of Medical Physiology, By John Edward Hall, 13th edition, page no. 192)
12. Answer: C (Ref. A Guyton and Hall Textbook of Medical Physiology, By John Edward Hall, 13th edition, page no. 196)
13. Answer: B (Ref. “Previous question”)
14. Answer: A
ECF include inulin, mannitol, and sucrose.
• Isotonic solutions of inulin, mannitol, and sucrose can all be used to determine ECF volume.
• Radio-iodinated albumin may be used for estimating plasma volume.
15. Answer: D (Ref. A Guyton and Hall Textbook of Medical Physiology, By John Edward Hall, 13th edition, page no. 309)
16. Answer: A (Ref. A Guyton and Hall Textbook of Medical Physiology, By John Edward Hall, 13th edition, page no. 389)
• TBW = ICF + ECF volumes = 28 + 14 = 42 lConcentration of potassium in ICF = 140 mM (K is the major intracellular
cation and its typical concentration is worth remembering)
• Concentration of potassium in ECF = 5 mM
17. Answer: D (Ref. A Guyton and Hall Textbook of Medical Physiology, By John Edward Hall, 13th edition, page no. 47)
18. Answer: A (Ref. A Guyton and Hall Textbook of Medical Physiology, By John Edward Hall, 13th edition, page no. 372)
19. Answer: C (Ref. A Guyton and Hall Textbook of Medical Physiology, By John Edward Hall, 13th edition, page no. 312)
20. Answer: A (Ref. A Guyton and Hall Textbook of Medical Physiology, By John Edward Hall, 13th edition, page no. 312)
• Isotonic dehydration is the result of proportional loss of NaCl and water from ECF, and there is no change in ECF
osmolality.
• There is thus no drive for movement of water between ICF and ECF →no change in ICF volume.
• In hypertonic dehydration, steady state ECF osmolality increases → leads to intracellular dehydration.
• In hypotonic dehydration, steady state osmolality of ECF is decreased → some of this water will enter cells.
21. Answer: B (Ref. A Guyton and Hall Textbook of Medical Physiology, By John Edward Hall, 13th edition, page no. 312)
22. Answer: B (Ref. A Guyton and Hall Textbook of Medical Physiology, By John Edward Hall, 13th edition, page no. 55)
• By pumping 3 Na ions out of the cell for every 2 K ions pumped into the cell, the Na-K ATPase effectively removes one
osmole from the cell.
• Thus, inhibition of Na-K pump would cause cells to swell.
23. Answer: D (Ref. A Guyton and Hall Textbook of Medical Physiology, By John Edward Hall, 13th edition, page no. 196)
• Starling’s law of filtration: Starling described forces that affect fluid flux across capillaries.
• Albumin is quantitatively the most important contributor to the colloid osmotic pressure of plasma.
• 1 g of albumin in 100 ml of plasma exerts an osmotic effect of 6 mm Hg; the same concentration of globulins exert a
pressure of only about 1.5 mm Hg.
24. Answer: D (Ref. A Guyton and Hall Textbook of Medical Physiology, By John Edward Hall, 13th edition, page no. 51)
According to Fick’s law of diffusion,
Diffusion rate (J) = DA dc/dx
D – Diffusion coefficient
A – Area available for diffusion
dc – concentration gradient
dx – thickness of membrane
25. Answer: A (Ref. A Guyton and Hall Textbook of Medical Physiology, By John Edward Hall, 13th edition, page no. 51)
26. Answer: A (Ref. A Guyton and Hall Textbook of Medical Physiology, By John Edward Hall, 13th edition, page no. 489)
• Osmotic pressure is dependent upon the number of particles and not the mass of a substance.
• Albumin does not make a significant contribution to the osmolality and osmotic pressure of plasma because they are
found in concentrations much lower than Na, Cl – the most effective osmoles in ECF.
• Albumin is quantitatively the most important contributor to the colloid osmotic pressure of plasma (oncotic pressure)
because it is restricted to plasma and does not exit the capillary as fast.
• Among plasma proteins, albumin is the most abundant and hence it contributes the most to colloid osmotic pressure of
plasma.
https://t.me/DentalBooksWorld
Chapter 15 • Physiology 1327

27. Answer: C (Ref. A Guyton and Hall Textbook of Medical Physiology, By John Edward Hall, 13th edition, page no. 190)
Crystalloids include NaCl, Ringer’s lactate – they diffuse freely across the capillary so they do not cause an osmotic pressure
difference across the capillary membrane.
28. Answer: A (Ref. “Previous question”)
• Actin – most abundant in muscle.
• Collagen – most abundant protein in extracellular matrix.
• Dystrophin is a glycoprotein in the muscle cell membrane; it
• is involved in transmitting the tension generated by shortening of sarcomeres eventually to the tendon.
• Titin – largest protein in the human body.
29. Answer: C (Ref. A Guyton and Hall Textbook of Medical Physiology, By John Edward Hall, 13th edition, page no. 19)
30. Answer: A (Ref. A Guyton and Hall Textbook of Medical Physiology, By John Edward Hall, 13th edition, page no. 17)
Microtubules have a 9 + 2 fibrillar structure.
Made of tubulin, dynein, and kinesin.
The mitotic spindle is made up of microtubules.
Colchicine arrests cells in metaphase by inhibiting polymerization of microtubules.
31. Answer: C (Ref. A Guyton and Hall Textbook of Medical Physiology, By John Edward Hall, 13th edition, page no. 535)
Mature RBCs do not have mitochondria and exclusively use anaerobic glycolysis to generate ATP.
Although sperm cells contain mitochondria, mitochondria from sperm cells do not enter the secondary oocyte during
fertilization.
32. Answer: D (Ref. A Guyton and Hall Textbook of Medical Physiology, By John Edward Hall, 13th edition, page no. 15)
33. Answer: D (Ref. A Guyton and Hall Textbook of Medical Physiology, By John Edward Hall, 13th edition, page no. 42)
Deficiency of the tumor suppressor protein p53 allows entry into M phase and facilitates the growth of tumors, and a
number of cancers are associated with deficiency of p53.
34. Answer: D (Ref. A Guyton and Hall Textbook of Medical Physiology, By John Edward Hall, 13th edition, page no. 55)
35. Answer: B (Ref. A Guyton and Hall Textbook of Medical Physiology, By John Edward Hall, 13th edition, page no. 350)
36. Answer: D (Ref. A Guyton and Hall Textbook of Medical Physiology, By John Edward Hall, 13th edition, page no. 201)
• Proteins are translocated from capillaries into the interstitium by transcytosis (vesicular transport).
• This involves endocytosis of the protein molecules into vesicles in endothelial cells and exocytosis of these vesicles into
the interstitium.
37. Answer: D (Ref. A Guyton and Hall Textbook of Medical Physiology, By John Edward Hall, 13th edition, page no. 87)
There are proton translocating ATPases in lysosomes.
38. Answer: D (Ref. A Guyton and Hall Textbook of Medical Physiology, By John Edward Hall, 13th edition, page no. 51)
• “Carrier-mediated transport” emphasizes the fact that the transported species binds to a transport protein.
• Glucose attaches to the GLUT molecule and a conformational change in GLUT shuttles glucose to the interior of the cell.
• The transport of Na and K by Na-K ATPase is also an example of carrier-mediated transport because both ions bind to
the transporter.
39. Answer: C (Ref. A Guyton and Hall Textbook of Medical Physiology, By John Edward Hall, 13th edition, page no. 51)
40. Answer: B (Ref. “Previous question”)
41. Answer: C
42. Answer: B (Ref. “Previous question”)
Chloride transport across the cell membrane occurs by secondary active transport mechanisms or diffusion.
43. Answer: C (Ref. A Guyton and Hall Textbook of Medical Physiology, By John Edward Hall, 13th edition, page no. 22)
Emiocytosis (exocytosis) of secretory granules requires a rise in intracellular calcium.
EXAMPLE – release of insulin by B cells of the pancreas involves depolarization of the B cell by closure of K channels
followed by influx of calcium through voltage-dependent calcium channels.
44. Answer: D (Ref. A Guyton and Hall Textbook of Medical Physiology, By John Edward Hall, 13th edition, page no. 854)
45. Answer: C (Ref. A Guyton and Hall Textbook of Medical Physiology, By John Edward Hall, 13th edition, page no. 985)
46. Answer: B (Ref. A Guyton and Hall Textbook of Medical Physiology, By John Edward Hall, 13th edition, page no. 935)
47. Answer: B
48. Answer: B (Ref. “Previous question”)
49. Answer: D (Ref. “Previous question”)
50. Answer: A (Ref. A Guyton and Hall Textbook of Medical Physiology, By John Edward Hall, 13th edition, page no. 102)

https://t.me/DentalBooksWorld
1328 Triumph’s Complete Review of Dentistry

• Gap junctions (electrical synapses; connexons) are made up of a protein called connexin; these are large diameter
channels that allow the passage of ions and other small molecules such as amino acids and glucose.
• They electrically couple adjacent cells.
• Found in visceral smooth muscle and heart and between some neurons.
• The ventricle of Heart contracts as one unit because gap junctions electrically couple all muscle fibers in the ventricle.
• Connexins do not allow large molecules such as proteins to pass between cells.
51. Answer: A (Ref. “Previous question”)
• Clathrin is a protein involved in receptor-mediated endocytosis.Clathrin is the protein that coats pits that are the sites of
aggregation of cell-surface receptors involved in receptor-mediated endocytosis.
• Cadherin is a cell adhesion molecule.
• Calcineurin, like calmodulin, is a calcium binding protein found in cells.
52. Answer: A (Ref. “Previous question”)
53. Answer: B (Ref. A Guyton and Hall Textbook of Medical Physiology, By John Edward Hall, 13th edition, page no. 4)
54. Answer: D (Ref. A Guyton and Hall Textbook of Medical Physiology, By John Edward Hall, 13th edition, page no. 9)
The “central controller” or “integrator” of all relevant sensory input is the hypothalamus.
The fact that a rise in body temperature leads to a fall in temperature back toward 98.6°F suggests that temperature is the
controlled variable, and this operates as a negative feedback control system.
55. Answer: B (Ref. A Guyton and Hall Textbook of Medical Physiology, By John Edward Hall, 13th edition, page no. 535)
56. Answer: D (Ref. “Previous question”)
57. Answer: C (Ref. A Guyton and Hall Textbook of Medical Physiology, By John Edward Hall, 13th edition, page no. 425)
58. Answer: C (Ref. A Guyton and Hall Textbook of Medical Physiology, By John Edward Hall, 13th edition, page no. 502)
59. Answer: A (Ref. A Guyton and Hall Textbook of Medical Physiology, By John Edward Hall, 13th edition, page no. 412)
60. Answer: B (Ref. A Guyton and Hall Textbook of Medical Physiology, By John Edward Hall, 13th edition, page no. 549)
61. Answer: A (Ref. A Guyton and Hall Textbook of Medical Physiology, By John Edward Hall, 13th edition, page no. 79)
62. Answer: A (Ref. A Guyton and Hall Textbook of Medical Physiology, By John Edward Hall, 13th edition, page no. 39)
63. Answer: A (Ref. A Guyton and Hall Textbook of Medical Physiology, By John Edward Hall, 13th edition, page no. 777)
64. Answer: A (Ref. A Guyton and Hall Textbook of Medical Physiology, By John Edward Hall, 13th edition, page no. 55)
65. Answer: B (Ref. A Guyton and Hall Textbook of Medical Physiology, By John Edward Hall, 13th edition, page no. 393)
66. Answer: C (Ref. A Guyton and Hall Textbook of Medical Physiology, By John Edward Hall, 13th edition, page no. 930)
67. Answer: C (Ref. A Guyton and Hall Textbook of Medical Physiology, By John Edward Hall, 13th edition, page no. 340)
68. Answer: C (Ref. “Previous question”)
69. Answer: B (Ref. A Guyton and Hall Textbook of Medical Physiology, By John Edward Hall, 13th edition, page no. 490)
70. Answer: C (Ref. A Guyton and Hall Textbook of Medical Physiology, By John Edward Hall, 13th edition, page no. 259)
71. Answer: D
72. Answer: B (Ref. A Guyton and Hall Textbook of Medical Physiology, By John Edward Hall, 13th edition, page no. 957)
73. Answer: C (Ref. A Guyton and Hall Textbook of Medical Physiology, By John Edward Hall, 13th edition, page no. 343)
74. Answer: B
75. Answer: D (Ref. A Guyton and Hall Textbook of Medical Physiology, By John Edward Hall, 13th edition, page no. 9)
76. Answer: D (Ref. A Guyton and Hall Textbook of Medical Physiology, By John Edward Hall, 13th edition, page no. 545)
77. Answer: B (Ref. “Previous question”)
78. Answer: D (Ref. A Guyton and Hall Textbook of Medical Physiology, By John Edward Hall, 13th edition, page no. 4)

NERVE AND MUSCLE PHYSIOLOGY


1. Answer: C (Ref. Guyton and Hall Textbook of Medical Physiology, By John E. Hall, 13th edition, page no. 956)
2. Answer: B (Ref. Guyton and Hall Textbook of Medical Physiology, By John E. Hall, 13th edition, page no. 111)
3. Answer: D (Ref. Guyton and Hall Textbook of Medical Physiology, By John E. Hall, 13th edition, page no. 651)
4. Answer: C (Ref. Guyton and Hall Textbook of Medical Physiology, By John E. Hall, 13th edition, page no. 111)
5. Answer: C (Ref. Guyton and Hall Textbook of Medical Physiology, By John E. Hall, 13th edition, page no. 62)
6. Answer: C (Ref. Guyton and Hall Textbook of Medical Physiology, By John E. Hall, 13th edition, page no. 696)
7. Answer: C (Ref. Guyton and Hall Textbook of Medical Physiology, By John E. Hall, 13th edition, page no. 587)
8. Answer: B (Ref. Guyton and Hall Textbook of Medical Physiology, By John E. Hall, 13th edition, page no. 389)

https://t.me/DentalBooksWorld
Chapter 15 • Physiology 1329

9. Answer: B (Ref. “Previous Question”)


10. Answer: C (Ref. “Previous Question”)
• Resting nerve cell membrane is 50 times more permeable to K than NaK flux through potassium channels determines
RMP
11. Answer: C (Ref. Guyton and Hall Textbook of Medical Physiology, By John E. Hall, 13th edition, page no. 73)
12. Answer: B (Ref. Guyton and Hall Textbook of Medical Physiology, By John E. Hall, 13th edition, page no. 147)
13. Answer: A (Ref. Guyton and Hall Textbook of Medical Physiology, By John E. Hall, 13th edition, page no. 696)
14. Answer: D (Ref. Guyton and Hall Textbook of Medical Physiology, By John E. Hall, 13th edition, page no. 72)
15. Answer: D (Ref. “Previous Question”)
16. Answer: B (Ref. Guyton and Hall Textbook of Medical Physiology, By John E. Hall, 13th edition, page no. 728)
17. Answer: A (Ref. Guyton and Hall Textbook of Medical Physiology, By John E. Hall, 13th edition, page no. 65)
18. Answer: B (Ref. Guyton and Hall Textbook of Medical Physiology, By John E. Hall, 13th edition, page no. 89)
19. Answer: C (Ref. Guyton and Hall Textbook of Medical Physiology, By John E. Hall, 13th edition, page no. 785)
Type B fibers are most susceptible to hypoxia.
20. Answer: C (Ref. Guyton and Hall Textbook of Medical Physiology, By John E. Hall, 13th edition, page no. 631)
21. Answer: D (Ref. Guyton and Hall Textbook of Medical Physiology, By John E. Hall, 13th edition, page no. 623)
• Nociceptors – nerve endings signaling pain
• “Fast pain” is transmitted through myelinated A-delta fibers
• “Slow pain” is the result of transmission through unmyelinated C fibers.
22. Answer: A (Ref. Guyton and Hall Textbook of Medical Physiology, By John E. Hall, 13th edition, page no. 93)
23. Answer: B (Ref. Guyton and Hall Textbook of Medical Physiology, By John E. Hall, 13th edition, page no. 78)
24. Answer: A (Ref. “Previous Question”)
25. Answer: C (Ref. Guyton and Hall Textbook of Medical Physiology, By John E. Hall, 13th edition, page no. 1136)
26. Answer: B (Ref. Guyton and Hall Textbook of Medical Physiology, By John E. Hall, 13th edition, page no. 78)
27. Answer: D (Ref. Guyton and Hall Textbook of Medical Physiology, By John E. Hall, 13th edition, page no. 112)
• Skeletal muscle depends on the sarcoplasmic reticulum (SR) as the major source of calcium.
• Cardiac muscle and smooth muscle depend on influx of calcium from ECF and Ca-induced calcium release (from SR)
for contraction
28. Answer: C (Ref. Guyton and Hall Textbook of Medical Physiology, By John E. Hall, 13th edition, page no. 112)
29. Answer: D (Ref. Guyton and Hall Textbook of Medical Physiology, By John E. Hall, 13th edition, page no. 88)
30. Answer: B (Ref. “Previous Question”)
• Actin and myosin are contractile proteins
• Calmodulin is not a contractile protein, it is a
• calcium binding protein that activates myosin
• light chain kinase upon binding calcium.
31. Answer: B (Ref. Guyton and Hall Textbook of Medical Physiology, By John E. Hall, 13th edition, page no. 935)
32. Answer: D (Ref. Guyton and Hall Textbook of Medical Physiology, By John E. Hall, 13th edition, page no. 104)
33. Answer: D (Ref. Guyton and Hall Textbook of Medical Physiology, By John E. Hall, 13th edition, page no. 97)
34. Answer: D (Ref. Guyton and Hall Textbook of Medical Physiology, By John E. Hall, 13th edition, page no. 84)
35. Answer: D (Ref. “Previous Question”)
36. Answer: A (Ref. “Previous Question”)
37. Answer: D (Ref. “Previous Question”)
Isotonic contraction Isometric contraction
Tension develop is constant; muscle shortens Tension develop; muscle does not shorten
*External work is done External work is not done
Example: Walking, bending, running Trying to lift a heavy weight, sustained handgrip
38. Answer: B (Ref. Guyton and Hall Textbook of Medical Physiology, By John E. Hall, 13th edition, page no. 99)
39. Answer: D (Ref. Guyton and Hall Textbook of Medical Physiology, By John E. Hall, 13th edition, page no. 85)
Motor unit is an alpha motor neuron and all muscle fibers innervated by it but smallest amount of muscle that can contract
in response to excitation of a single motor neuron.
40. Answer: B

https://t.me/DentalBooksWorld
1330 Triumph’s Complete Review of Dentistry

41. Answer: A (Ref. Guyton and Hall Textbook of Medical Physiology, By John E. Hall, 13th edition, page no. 85)
• Staircase phenomenon or Treppe or Bowditch effect
• There is complete relaxation following each contraction effectIt occurs in skeletal as well as cardiac muscleIt is attributed
to increased availability of intracellular calcium (beneficial effect of previous contractions)
42. Answer: D (Ref. “Previous Question”)
Gradation of force in skeletal muscle is achieved by:
• Recruitment of motor units
•  ↑ The frequency of stimulation of motor units
• Asynchronous firing of motor units
• Variations in preload
Gradation of force is also not achieved by varying calcium release from the sarcoplasmic reticulum because each action
potential that enters the T tubule releases a constant amount of calcium from the terminal cisterns of the SR.
43. Answer: A
44. Answer: A
Chronaxie has been defined as the minimum stimulus duration at which twice the rheobasic current will produce a
responseIn a fatigued muscle, the chronaxie is longer
45. Answer: D (Ref. “Previous Question”)
46. Answer: B (Ref. “Previous Question”)
47. Answer: A (Ref. Guyton and Hall Textbook of Medical Physiology, By John E. Hall, 13th edition, page no. 111)
48. Answer: D
49. Answer: A (Ref. “Previous Question”)
• Norepinephrine has a positive chronotropic (increases rate of contraction)
• Inotropic (increases force of contraction)
• Lusitropic effect (it accelerates relaxation) on the heart
50. Answer: D (Ref. Guyton and Hall Textbook of Medical Physiology, By John E. Hall, 13th edition, page no. 264)
51. Answer: A (Ref. Guyton and Hall Textbook of Medical Physiology, By John E. Hall, 13th edition, page no. 935)
52. Answer: A (Ref. “Previous Question”)
53. Answer: B (Ref. Guyton and Hall Textbook of Medical Physiology, By John E. Hall, 13th edition, page no. 99)
• Latch bridges are actomyosin complexes that detach slowly
• This is made possible by a slow myosin ATPase and enables maintenance of tension for longer periods without
consumption of more ATP
54. Answer: C (Ref. Guyton and Hall Textbook of Medical Physiology, By John E. Hall, 13th edition, page no. 66)
55. Answer: D (Ref. Guyton and Hall Textbook of Medical Physiology, By John E. Hall, 13th edition, page no. 958)
Ratio of synapses to neurons in the brain is about 1,000; glia outnumber neurons by about 50 times.
56. Answer: A (Ref. Guyton and Hall Textbook of Medical Physiology, By John E. Hall, 13th edition, page no. 461)
57. Answer: A (Ref. Guyton and Hall Textbook of Medical Physiology, By John E. Hall, 13th edition, page no. 770)
Reduced reuptake of excitatory neurotransmitters such as glutamate by astrocytes in ischemic zones is said to contribute to
excitotoxicity in stroke.
58. Answer: A (Ref: A Guyton and Hall Textbook of Medical Physiology, By John E. Hall, 13th edition, page no. 897)
59. Answer: B
98% synapses in the CNS are axodendritic.
60. Answer: C (Ref. A Guyton and Hall Textbook of Medical Physiology, By John E. Hall, 13th edition, page no. 582)
Opening of chloride ion channels in postsynaptic membrane would produce hyperpolarization of postsynaptic neurons
61. Answer: C (Ref. A Guyton and Hall Textbook of Medical Physiology, By John E. Hall, 13th edition, page no. 582)
62. Answer: C (Ref. A Guyton and Hall Textbook of Medical Physiology, By John E. Hall, 13th edition, page no. 977)
Direct inhibition involves direct inhibition of the postsynaptic neuron by producing IPSPs on it. It is also called postsynaptic
inhibition.
63. Answer: B (Ref. A Guyton and Hall Textbook of Medical Physiology, By John E. Hall, 13th edition, page no. 577)
64. Answer: D (Ref. “Previous Question”)
Glycine is an inhibitory neurotransmitter in the spinal cord. In the absence of glycine, glutamate cannot exert its excitatory
effects
65. Answer: A (Ref. A Guyton and Hall Textbook of Medical Physiology, By John E. Hall, 13th edition, page no. 920)

https://t.me/DentalBooksWorld
Chapter 15 • Physiology 1331

66. Answer: A (Ref. A Guyton and Hall Textbook of Medical Physiology, By John E. Hall, 13th edition, page no. 697)
67. Answer: B (Ref. A Guyton and Hall Textbook of Medical Physiology, By John E. Hall, 13th edition, page no. 705)
68. Answer: A (Ref. “Previous Question”)
69. Answer: C (Ref. A Guyton and Hall Textbook of Medical Physiology, By John E. Hall, 13th edition, page no. 337)
70. Answer: C (Ref. A Guyton and Hall Textbook of Medical Physiology, By John E. Hall, 13th edition, page no. 656)
71. Answer: A (Ref. A Guyton and Hall Textbook of Medical Physiology, By John E. Hall, 13th edition, page no. 733)
72. Answer: B (Ref. A Guyton and Hall Textbook of Medical Physiology, By John E. Hall, 13th edition, page no. 99)
73. Answer: B (Ref. A Guyton and Hall Textbook of Medical Physiology, By John E. Hall, 13th edition, page no. 696)
74. Answer: B (Ref. A Guyton and Hall Textbook of Medical Physiology, By John E. Hall, 13th edition, page no. 69)
75. Answer: A (Ref. A Guyton and Hall Textbook of Medical Physiology, By John E. Hall, 13th edition, page no. 92)
76. Answer: D (Ref. A Guyton and Hall Textbook of Medical Physiology, By John E. Hall, 13th edition, page no. 92)
77. Answer: C
78. Answer: D
79. Answer: D (Ref. A Guyton and Hall Textbook of Medical Physiology, By John E. Hall, 13th edition, page no. 899)
80. Answer: A (Ref. A Guyton and Hall Textbook of Medical Physiology, By John E. Hall, 13th edition, page no. 330)
81. Answer: A (Ref. A Guyton and Hall Textbook of Medical Physiology, By John E. Hall, 13th edition, page no. 676)
82. Answer: C (Ref. “Previous question”)
83. Answer: D (Ref. A Guyton and Hall Textbook of Medical Physiology, By John E. Hall, 13th edition, page no. 701)
84. Answer: A (Ref. A Guyton and Hall Textbook of Medical Physiology, By John E. Hall, 13th edition, page no. 157)
85. Answer: D (Ref. “Previous question”)
86. Answer: B (Ref. “Previous question”)
87. Answer: B
88. Answer: B (Ref. A Guyton and Hall Textbook of Medical Physiology, By John E. Hall, 13th edition, page no. 701)
89. Answer: D (Ref. A Guyton and Hall Textbook of Medical Physiology, By John E. Hall, 13th edition, page no. 700)
90. Answer: C (Ref. A Guyton and Hall Textbook of Medical Physiology, By John E. Hall, 13th edition, page no. 76)
91. Answer: D (Ref. A Guyton and Hall Textbook of Medical Physiology, By John E. Hall, 13th edition, page no. 83)
92. Answer: A (Ref. “Previous question”)
93. Answer: A (Ref. “Previous question”)
94. Answer: B
95. Answer: B
96. Answer: B
97. Answer: D
98. Answer: B (Ref. A Guyton and Hall Textbook of Medical Physiology, By John E. Hall, 13th edition, page no. 73)
99. Answer: D (Ref. A Guyton and Hall Textbook of Medical Physiology, By John E. Hall, 13th edition, page no. 701)
100. Answer: D
101. Answer: B (Ref. “Previous question”)
102. Answer: C (Ref. A Guyton and Hall Textbook of Medical Physiology, By John E. Hall, 13th edition, page no. 703)
103. Answer: A (Ref. A Guyton and Hall Textbook of Medical Physiology, By John E. Hall, 13th edition, page no. 935)
104. Answer: A (Ref. A Guyton and Hall Textbook of Medical Physiology, By John E. Hall, 13th edition, page no. 71)
105. Answer: D (Ref. “Previous question”)
106. Answer: D (Ref. A Guyton and Hall Textbook of Medical Physiology, By John E. Hall, 13th edition, page no. 589)
107. Answer: D (Ref. “Previous question”)
108. Answer: B (Ref. A Guyton and Hall Textbook of Medical Physiology, By John E. Hall, 13th edition, page no. 934)
109. Answer: A
110. Answer: C (Ref. A Guyton and Hall Textbook of Medical Physiology, By John E. Hall, 13th edition, page no. 986)
111. Answer: D (Ref. “Previous question”)
112. Answer: C
113. Answer: D (Ref. A Guyton and Hall Textbook of Medical Physiology, By John E. Hall, 13th edition, page no. 697)
114. Answer: B
115. Answer: B (Ref. “Previous question”)

https://t.me/DentalBooksWorld
1332 Triumph’s Complete Review of Dentistry

CVS
1. Answer: A
2. Answer: C
3. Answer: C
4. Answer: C
5. Answer: D
6. Answer: C
7. Answer: B
8. Answer: D
9. Answer: C
10. Answer: C
11. Answer: D
12. Answer: D
13. Answer: D
14. Answer: A
15. Answer: C
16. Answer: A
17. Answer: D
18. Answer: A
19. Answer: A
20. Answer: C
21. Answer: D
22. Answer: B
23. Answer: C
24. Answer: D
25. Answer: D
26. Answer: A
27. Answer: A
28. Answer: B
29. Answer: C
30. Answer: C
31. Answer: D
32. Answer: C
33. Answer: A
34. Answer: A
35. Answer: A
36. Answer: D
37. Answer: B
38. Answer: A
39. Answer: C
40. Answer: C
41. Answer: B
42. Answer: C
43. Answer: A
44. Answer: D
45. Answer: B
46. Answer: C
47. Answer: B
48. Answer: A
49. Answer: C
50. Answer: D

https://t.me/DentalBooksWorld
Chapter 15 • Physiology 1333

51. Answer: D
52. Answer: A
53. Answer: C
54. Answer: B
55. Answer: D
56. Answer: D
57. Answer: A
58. Answer: D
59. Answer: A
60. Answer: D
61. Answer: D
62. Answer: D
63. Answer: A
64. Answer: D
65. Answer: B
66. Answer: B
67. Answer: B
68. Answer: B
69. Answer: D
70. Answer: C
71. Answer: B
72. Answer: B
73. Answer: C
74. Answer: A
75. Answer: B
76. Answer: B
77. Answer: C
78. Answer: A
79. Answer: D
80. Answer: A
81. Answer: B
82. Answer: D
83. Answer: B
84. Answer: A
85. Answer: B
86. Answer: C
87. Answer: A
88. Answer: A
89. Answer: A
90. Answer: C
91. Answer: B
92. Answer: D
93. Answer: A
94. Answer: A
95. Answer: A
96. Answer: A
97. Answer: A
98. Answer: C
99. Answer: C
100. Answer: D
101. Answer: A
102. Answer: A

https://t.me/DentalBooksWorld
1334 Triumph’s Complete Review of Dentistry

103. Answer: C
104. Answer: D
105. Answer: C
106. Answer: B
107. Answer: D
108. Answer: C
109. Answer: A
110. Answer: A
111. Answer: B
112. Answer: D
113. Answer: A
114. Answer: C
115. Answer: D
116. Answer: A
117. Answer: B
118. Answer: D
119. Answer: D
120. Answer: B
121. Answer: A
122. Answer: A
123. Answer: A
124. Answer: A
125. Answer: B
126. Answer: D
127. Answer: D
128. Answer: A
129. Answer: D
130. Answer: A
131. Answer: C
132. Answer: A
133. Answer: A
134. Answer: A
135. Answer: A
136. Answer: C
137. Answer: D
138. Answer: A
139. Answer: A
140. Answer: C
141. Answer: C
142. Answer: B
143. Answer: D
144. Answer: D
145. Answer: A
146. Answer: D
147. Answer: D
148. Answer: B
149. Answer: C
150. Answer: D
151. Answer: B
152. Answer: A
153. Answer: D
154. Answer: D

https://t.me/DentalBooksWorld
Chapter 15 • Physiology 1335

155. Answer: D
156. Answer: C
157. Answer: A
158. Answer: C
159. Answer: D
160. Answer: D
161. Answer: A
162. Answer: B
163. Answer: B

CNS
1. Answer: A
2. Answer: B
3. Answer: B
4. Answer: D
5. Answer: D
6. Answer: B
7. Answer: C
8. Answer: A
9. Answer: D
10. Answer: C
11. Answer: B
12. Answer: A
13. Answer: D
14. Answer: A
15. Answer: D
16. Answer: A
17. Answer: B
18. Answer: C
19. Answer: D
20. Answer: C
21. Answer: D
22. Answer: C
23. Answer: C
24. Answer: D
25. Answer: A
26. Answer: A
27. Answer: A
28. Answer: A
29. Answer: C
30. Answer: C
31. Answer: A
32. Answer: A
33. Answer: C
34. Answer: C
35. Answer: A
36. Answer: D
37. Answer: B
38. Answer: D
39. Answer: C
40. Answer: B
41. Answer: A

https://t.me/DentalBooksWorld
1336 Triumph’s Complete Review of Dentistry

ENDOCRINE
1. Answer: A (Ref. Guyton and Hall Textbook of Medical Physiology, By John E. Hall, 13th edition, page no. 752)
Neurohormones are hormones synthesized and secreted by neurons into the general circulation.
2. Answer: D (Ref. Guyton and Hall Textbook of Medical Physiology, By John E. Hall, 13th edition, page no. 955)
Retinoic acid, thyroxine, and steroid hormones act mainly through receptors located in the cytosol or nucleus of target
cells.
3. Answer: D (Ref. Guyton and Hall Textbook of Medical Physiology, By John E. Hall, 13th edition, page no. 925)
4. Answer: D (Ref. Guyton and Hall Textbook of Medical Physiology, By John E. Hall, 13th edition, page no. 945)
• Janus tyrosine kinases (JAK) are enzymes that phosphorylate signal transducers and activators of transcription (STAT)
• Growth hormone, prolactin, and erythropoietin act via JAK–STAT pathways
5. Answer: A (Ref. Guyton and Hall Textbook of Medical Physiology, By John E. Hall, 13th edition, page no. 959)
TRH is a tripeptide.
6. Answer: C (Ref. Guyton and Hall Textbook of Medical Physiology, By John E. Hall, 13th edition, page no. 942)
7. Answer: B (Ref. Guyton and Hall Textbook of Medical Physiology, By John E. Hall, 13th edition, page no. 410)
8. Answer: D (Ref. Guyton and Hall Textbook of Medical Physiology, By John E. Hall, 13th edition, page no. 954)
9. Answer: D (Ref. Guyton and Hall Textbook of Medical Physiology, By John E. Hall, 13th edition, page no. 1068)
Dopamine from the hypothalamus inhibits the release of prolactin.
10. Answer: A (Ref. Guyton and Hall Textbook of Medical Physiology, By John E. Hall, 13th edition, page no. 1068)
11. Answer: C (Ref. Guyton and Hall Textbook of Medical Physiology, By John E. Hall, 13th edition, page no. 1068)
12. Answer: A (Ref. Guyton and Hall Textbook of Medical Physiology, By John E. Hall, 13th edition, page no. 959)
50% of somatotroph cells in the anterior pituitary secrete growth hormone.
13. Answer: D (Ref. Guyton and Hall Textbook of Medical Physiology, By John E. Hall, 13th edition, page no. 1068)
14. Answer: D (Ref. Guyton and Hall Textbook of Medical Physiology, By John E. Hall, 13th edition, page no. 1383)
• ADH is stored in the posterior pituitary bound to neurophysins.
• ADH circulates free in plasma.
15. Answer: D (Ref. Guyton and Hall Textbook of Medical Physiology, By John E. Hall, 13th edition, page no. 1013)
16. Answer: B (Ref. “Previous question”)
17. Answer: D (Ref. Guyton and Hall Textbook of Medical Physiology, By John E. Hall, 13th edition, page no. 959)
18. Answer: B (Ref. “Previous question”)
19. Answer: A (Ref. “Previous question”)
20. Answer: B (Ref. Guyton and Hall Textbook of Medical Physiology, By John E. Hall, 13th edition, page no. 951)
21. Answer: A (Ref. Guyton and Hall Textbook of Medical Physiology, By John E. Hall, 13th edition, page no. 952)
• Type 1 deiodinase – Found in the liver, kidney, and in the thyroid.
• It converts T4 to T3.
• Extrathyroidal T3 is generated as a result of the action of D1 spills over into the circulation for entry into target cells.
22. Answer: C (Ref. “Previous question”)
• Type 3 deiodinase (D3) is present in the placenta, brain.
• This enzyme inactivates T4 converting it to rT3, and also inactivates T3.
• Placental D3 thus protects the fetus from an excess of T4 and T3 should the mother be hyperthyroid.
• Some T4 from the mother does cross the placenta (via specific transporters) and supports growth and development in
the fetus before the fetal thyroid begins making T4.
23. Answer: C (Ref. “Previous question”)
24. Answer: A (Ref. Guyton and Hall Textbook of Medical Physiology, By John E. Hall, 13th edition, page no. 960)
In individuals ingesting an “excess” of iodide, hyperthyroidism may develop if “autoregulatory mechanisms” in the thyroid
fail to prevent an iodine-induced increase in thyroid hormone synthesis. This is called iodide-induced hyperthyroidism or
the Jod-Basedow phenomenon.
The Jod-Basedow phenomenon is likely when TSH levels are high or when there are antibodies that stimulate TSH receptor.
25. Answer: B (Ref. Guyton and Hall Textbook of Medical Physiology, By John E. Hall, 13th edition, page no. 983)
26. Answer: C (Ref. Guyton and Hall Textbook of Medical Physiology, By John E. Hall, 13th edition, page no. 1099)
Alpha 2-adrenergic receptors: epinephrine/norepinephrine inhibit insulin secretion.
Beta-adrenergic receptors: stimulate insulin secretion.

https://t.me/DentalBooksWorld
Chapter 15 • Physiology 1337

27. Answer: C (Ref. Guyton and Hall Textbook of Medical Physiology, By John E. Hall, 13th edition, page no. 990)
• GLP-1 (7-36) is glucagon-like polypeptide
• It is a GI hormone that is a potent stimulator of insulin secretion
28. Answer: D (Ref. “Previous question”)
29. Answer: B (Ref. “Previous question”)
• Insulin increases K uptake by muscle probably by stimulating Na-K ATPase.
• Insulin increases the oxidation of ketone bodies → treatment of diabetic ketoacidosis.
30. Answer: C (Ref. Guyton and Hall Textbook of Medical Physiology, By John E. Hall, 13th edition, page no. 998)
31. Answer: B (Ref. “Previous question”)
Red blood cells – most regions of the brain with the exception of cells in the satiety centerLiver cells do not require insulin
for glucose uptake
32. Answer: C (Ref. “Previous question”)
33. Answer: D (Ref. Guyton and Hall Textbook of Medical Physiology, By John E. Hall, 13th edition, page no. 944)
34. Answer: D (Ref. Guyton and Hall Textbook of Medical Physiology, By John E. Hall, 13th edition, page no. 1117)
35. Answer: A (Ref. Guyton and Hall Textbook of Medical Physiology, By John E. Hall, 13th edition, page no. 982)
36. Answer: A (Ref. Guyton and Hall Textbook of Medical Physiology, By John E. Hall, 13th edition, page no. 985)
37. Answer: D(Ref. “Previous question”)
38. Answer: D (Ref. Guyton and Hall Textbook of Medical Physiology, By John E. Hall, 13th edition, page no. 893)
• Leptin is a hormone secreted by adipocytes → inhibit food intake (anorexigenic hormone).
• It signals the amount of fat to the CNS.
• Leptin levels increase after a meal.
• Leptin deficiency as well as deficiency of functional leptin receptors have been implicated in the pathogenesis of obesity.
39. Answer: D (Ref. “Previous question”)
Diabetogenic hormones, which when present in excess reduce insulin sensitivity and thus increase plasma glucose levels,
particularly in individuals otherwise predisposed to impaired beta-cell function.
They are:
• Epinephrine
• Norepinephrine
• Glucagon
• Growth hormone
• Cortisol
• Thyroxine
40. Answer: D (Ref. Guyton and Hall Textbook of Medical Physiology, By John E. Hall, 13th edition, page no. 993)
41. Answer: C (Ref. Guyton and Hall Textbook of Medical Physiology, By John E. Hall, 13th edition, page no. 980)
42. Answer: A (Ref. Guyton and Hall Textbook of Medical Physiology, By John E. Hall, 13th edition, page no. 966)
43. Answer: B (Ref. Guyton and Hall Textbook of Medical Physiology, By John E. Hall, 13th edition, page no. 393)
44. Answer: C (Ref. Guyton and Hall Textbook of Medical Physiology, By John E. Hall, 13th edition, page no. 975)
• Aldosterone is the most potent endogenous mineralocorticoid hormone.
• Next to aldosterone, deoxycorticosterone has the highest mineralocorticoid activity.
45. Answer: A (Ref. Guyton and Hall Textbook of Medical Physiology, By John E. Hall, 13th edition, page no. 974)
46. Answer: A (Ref. “Previous question”)
• Glucocorticoids decrease the number of eosinophils, basophils, and lymphocytes in the circulation.
• Glucocorticoids, particularly in pharmacologic doses suppress cell-mediated immunity by multiple mechanisms
including:
–– Inhibiting IL-2 gene transcription
–– Inhibiting T cell proliferation
–– Inducing apoptosis in lymphocytes
47. Answer: C (Ref. Guyton and Hall Textbook of Medical Physiology, By John E. Hall, 13th edition, page no. 976)
48. Answer: A (Ref. Guyton and Hall Textbook of Medical Physiology, By John E. Hall, 13th edition, page no. 781)
49. Answer: C (Ref. Guyton and Hall Textbook of Medical Physiology, By John E. Hall, 13th edition, page no. 771)
50. Answer: D (Ref. Guyton and Hall Textbook of Medical Physiology, By John E. Hall, 13th edition, page no. 928)

https://t.me/DentalBooksWorld
1338 Triumph’s Complete Review of Dentistry

51. Answer: D (Ref. Guyton and Hall Textbook of Medical Physiology, By John E. Hall, 13th edition, page no. 1009)
52. Answer: B (Ref. “Previous question”)
53. Answer: C (Ref. “Previous question”)
54. Answer: B (Ref. “Previous question”)
55. Answer: D (Ref. “Previous question”)
56. Answer: D (Ref. Guyton and Hall Textbook of Medical Physiology, By John E. Hall, 13th edition, page no. 1028)
57. Answer: B (Ref. Guyton and Hall Textbook of Medical Physiology, By John E. Hall, 13th edition, page no. 1022)
58. Answer: A (Ref. Guyton and Hall Textbook of Medical Physiology, By John E. Hall, 13th edition, page no. 1068)
59. Answer: C (Ref. Guyton and Hall Textbook of Medical Physiology, By John E. Hall, 13th edition, page no. 1043)
• Androgens stimulate erythropoiesis.
• Estrogens have an inhibitory influence on erythropoiesis.
• Cortisol stimulates erythropoiesis and polycythemia may be a striking feature of Cushing’s syndrome.
• Thyroxine also has a stimulatory influence on erythropoiesis.
60. Answer: A (Ref. “Previous question”)
61. Answer: D (Ref. Guyton and Hall Textbook of Medical Physiology, By John E. Hall, 13th edition, page no. 1032)
62. Answer: D (Ref. “Previous question”)
63. Answer: C (Ref. Guyton and Hall Textbook of Medical Physiology, By John E. Hall, 13th edition, page no. 993)
64. Answer: D (Ref. Guyton and Hall Textbook of Medical Physiology, By John E. Hall, 13th edition, page no. 962)
65. Answer: A (Ref. “Previous question”)
66. Answer: C (Ref. Guyton and Hall Textbook of Medical Physiology, By John E. Hall, 13th edition, page no. 759)
67. Answer: A (Ref. Guyton and Hall Textbook of Medical Physiology, By John E. Hall, 13th edition, page no. 396)
68. Answer: A (Ref. “Previous question”)
69. Answer: C (Ref. Guyton and Hall Textbook of Medical Physiology, By John E. Hall, 13th edition, page no. 935)
70. Answer: D (Ref. Guyton and Hall Textbook of Medical Physiology, By John E. Hall, 13th edition, page no. 984)
71. Answer: B (Ref. Guyton and Hall Textbook of Medical Physiology, By John E. Hall, 13th edition, page no. 984)
72. Answer: D (Ref. Guyton and Hall Textbook of Medical Physiology, By John E. Hall, 13th edition, page no. 862)
73. Answer: D (Ref. Guyton and Hall Textbook of Medical Physiology, By John E. Hall, 13th edition, page no. 997)
74. Answer: B (Ref. “Previous question”)
75. Answer: D (Ref. “Previous question”)
76. Answer: B (Ref. “Previous question”)
Phospholamban, also known as PLN or PLB, is a protein that, in humans, is encoded by the PLN gene. Phospholamban is a
52-amino acid integral membrane protein that regulates the Ca2+ pump in cardiac muscle cells.
77. Answer: A (Ref. Guyton and Hall Textbook of Medical Physiology, By John E. Hall, 13th edition, page no. 597)
78. Answer: A (Ref. Guyton and Hall Textbook of Medical Physiology, By John E. Hall, 13th edition, page no. 1141)
79. Answer: D (Ref. “Previous question”)
80. Answer: C (Ref. Guyton and Hall Textbook of Medical Physiology, By John E. Hall, 13th edition, page no. 781)
81. Answer: C (Ref. Guyton and Hall Textbook of Medical Physiology, By John E. Hall, 13th edition, page no. 1009)
82. Answer: C (Ref. “Previous question”)
83. Answer: D (Ref. “Previous question”)
84. Answer: B (Ref. “Previous question”)
85. Answer: C (Ref. Guyton and Hall Textbook of Medical Physiology, By John E. Hall, 13th edition, page no. 976)
86. Answer: D (Ref. Guyton and Hall Textbook of Medical Physiology, By John E. Hall, 13th edition, page no. 934)
87. Answer: C (Ref. Guyton and Hall Textbook of Medical Physiology, By John E. Hall, 13th edition, page no. 1032)
88. Answer: D (Ref. “Previous question”)
89. Answer: A (Ref. Guyton and Hall Textbook of Medical Physiology, By John E. Hall, 13th edition, page no. 945)
90. Answer: D (Ref. “Previous question”)
91. Answer: A (Ref. “Previous question”)
92. Answer: D (Ref. “Previous question”)
93. Answer: B (Ref. “Previous question”)
94. Answer: D (Ref. “Previous question”)
95. Answer: A (Ref. Guyton and Hall Textbook of Medical Physiology, By John E. Hall, 13th edition, page no. 1035)

https://t.me/DentalBooksWorld
Chapter 15 • Physiology 1339

GIT
1. Answer: D (Ref. Guyton and Hall Textbook of Medical Physiology, By John E. Hall, 13th edition, page no. 886)
2. Answer: D (Ref. Guyton and Hall Textbook of Medical Physiology, By John E. Hall, 13th edition, page no. 885)
3. Answer: B (Ref. Guyton and Hall Textbook of Medical Physiology, By John E. Hall, 13th edition, page no. 824)
4. Answer: C (Ref. Guyton and Hall Textbook of Medical Physiology, By John E. Hall, 13th edition, page no. 827)
5. Answer: D (Ref. Guyton and Hall Textbook of Medical Physiology, By John E. Hall, 13th edition, page no. 871)
6. Answer: C (Ref. Guyton and Hall Textbook of Medical Physiology, By John E. Hall, 13th edition, page no. 822)
7. Answer: B (Ref. Guyton and Hall Textbook of Medical Physiology, By John E. Hall, 13th edition, page no. 822)
8. Answer: D (Ref. Guyton and Hall Textbook of Medical Physiology, By John E. Hall, 13th edition, page no. 882)
9. Answer: A (Ref. “Previous question”)
10. Answer: D (Ref. Guyton and Hall Textbook of Medical Physiology, By John E. Hall, 13th edition, page no. 808)
11. Answer: A (Ref. Guyton and Hall Textbook of Medical Physiology, By John E. Hall, 13th edition, page no. 813)
12. Answer: B (Ref. “Previous question”)
13. Answer: B (Ref. Guyton and Hall Textbook of Medical Physiology, By John E. Hall, 13th edition, page no. 827)
14. Answer: A (Ref. Guyton and Hall Textbook of Medical Physiology, By John E. Hall, 13th edition, page no. 834)
15. Answer: C (Ref. Guyton and Hall Textbook of Medical Physiology, By John E. Hall, 13th edition, page no. 824)
16. Answer: C (Ref. Guyton and Hall Textbook of Medical Physiology, By John E. Hall, 13th edition, page no. 819)
17. Answer: D (Ref. Guyton and Hall Textbook of Medical Physiology, By John E. Hall, 13th edition, page no. 823)
18. Answer: A (Ref. Guyton and Hall Textbook of Medical Physiology, By John E. Hall, 13th edition, page no. 800)
19. Answer: B (Ref. Guyton and Hall Textbook of Medical Physiology, By John E. Hall, 13th edition, page no. 802)
20. Answer: A (Ref. Guyton and Hall Textbook of Medical Physiology, By John E. Hall, 13th edition, page no. 829)
21. Answer: A (Ref. “Previous question”)
22. Answer: B (Ref. Guyton and Hall Textbook of Medical Physiology, By John E. Hall, 13th edition, page no. 824)
23. Answer: C (Ref. Guyton and Hall Textbook of Medical Physiology, By John E. Hall, 13th edition, page no. 813)
24. Answer: B (Ref. Guyton and Hall Textbook of Medical Physiology, By John E. Hall, 13th edition, page no. 823)
25. Answer: D (Ref. Guyton and Hall Textbook of Medical Physiology, By John E. Hall, 13th edition, page no. 827)
26. Answer: A (Ref. Guyton and Hall Textbook of Medical Physiology, By John E. Hall, 13th edition, page no. 824)
27. Answer: A (Ref. Guyton and Hall Textbook of Medical Physiology, By John E. Hall, 13th edition, page no. 824)
28. Answer: B (Ref. Guyton and Hall Textbook of Medical Physiology, By John E. Hall, 13th edition, page no. 803)
29. Answer: C (Ref. Guyton and Hall Textbook of Medical Physiology, By John E. Hall, 13th edition, page no. 814)
30. Answer: B (Ref. Guyton and Hall Textbook of Medical Physiology, By John E. Hall, 13th edition, page no. 829)
31. Answer: C (Ref. Guyton and Hall Textbook of Medical Physiology, By John E. Hall, 13th edition, page no. 824)
32. Answer: A (Ref. Guyton and Hall Textbook of Medical Physiology, By John E. Hall, 13th edition, page no. 811)
33. Answer: A (Ref. Guyton and Hall Textbook of Medical Physiology, By John E. Hall, 13th edition, page no. 866)
34. Answer: C (Ref. Guyton and Hall Textbook of Medical Physiology, By John E. Hall, 13th edition, page no. 824)
35. Answer: C (Ref. “Previous question”)
36. Answer: B (Ref. Guyton and Hall Textbook of Medical Physiology, By John E. Hall, 13th edition, page no. 983)
37. Answer: C (Ref. “Previous question”)
38. Answer: B (Ref. “Previous question”)
39. Answer: C (Ref. Guyton and Hall Textbook of Medical Physiology, By John E. Hall, 13th edition, page no. 824)
40. Answer: B (Ref. “Previous question”)
41. Answer: A (Ref. “Previous question”)
42. Answer: C (Ref. “Previous question”)
43. Answer: A (Ref. Guyton and Hall Textbook of Medical Physiology, By John E. Hall, 13th edition, page no. 844)
44. Answer: A (Ref. Guyton and Hall Textbook of Medical Physiology, By John E. Hall, 13th edition, page no. 803)
45. Answer: B (Ref. Guyton and Hall Textbook of Medical Physiology, By John E. Hall, 13th edition, page no. 809)
46. Answer: B (Ref. Guyton and Hall Textbook of Medical Physiology, By John E. Hall, 13th edition, page no. 449)
47. Answer: C (Ref. “Previous question”)
48. Answer: B (Ref. Guyton and Hall Textbook of Medical Physiology, By John E. Hall, 13th edition, page no. 886)
49. Answer: A (Ref. Guyton and Hall Textbook of Medical Physiology, By John E. Hall, 13th edition, page no. 834)
50. Answer: C (Ref. Guyton and Hall Textbook of Medical Physiology, By John E. Hall, 13th edition, page no. 827)

https://t.me/DentalBooksWorld
1340 Triumph’s Complete Review of Dentistry

51. Answer: A (Ref. “Previous question”)


52. Answer: D (Ref. Guyton and Hall Textbook of Medical Physiology, By John E. Hall, 13th edition, page no. 811)
53. Answer: C (Ref. Guyton and Hall Textbook of Medical Physiology, By John E. Hall, 13th edition, page no. 84)
54. Answer: B (Ref. Guyton and Hall Textbook of Medical Physiology, By John E. Hall, 13th edition, page no. 16)
55. Answer: C (Ref. Guyton and Hall Textbook of Medical Physiology, By John E. Hall, 13th edition, page no. 899)
56. Answer: C (Ref. “Previous question”)
57. Answer: B (Ref. Guyton and Hall Textbook of Medical Physiology, By John E. Hall, 13th edition, page no. 844)
58. Answer: B (Ref. Guyton and Hall Textbook of Medical Physiology, By John E. Hall, 13th edition, page no. 815)
59. Answer: C (Ref. Guyton and Hall Textbook of Medical Physiology, By John E. Hall, 13th edition, page no. 452)
60. Answer: C (Ref. “Previous question”)
61. Answer: B (Ref. Guyton and Hall Textbook of Medical Physiology, By John E. Hall, 13th edition, page no. 782)
62. Answer: B (Ref. Guyton and Hall Textbook of Medical Physiology, By John E. Hall, 13th edition, page no. 824)
63. Answer: C (Ref. “Previous question”)
64. Answer: D (Ref. “Previous question”)
65. Answer: D (Ref. “Previous question”)
66. Answer: C (Ref. Guyton and Hall Textbook of Medical Physiology, By John E. Hall, 13th edition, page no. 815)
67. Answer: B (Ref. “Previous question”)
68. Answer: B (Ref. Guyton and Hall Textbook of Medical Physiology, By John E. Hall, 13th edition, page no. 802)
69. Answer: B (Ref. Guyton and Hall Textbook of Medical Physiology, By John E. Hall, 13th edition, page no. 809)
70. Answer: A (Ref. “Previous question”)

RENAL
1. Answer: A (Ref. A Guyton and Hall Textbook of Medical Physiology, By John E. Hall, 13th edition, page no. 366)
• Inulin clearance gives the best estimate of GFR since it is freely filtered and neither reabsorbed nor secreted by the
nephron.
• Further, inulin does not affect hemodynamics.
• Despite being the gold standard for measurement of GFR, inulin clearance measurement is invasive and is not feasible
in clinical practice.
2. Answer: B (Ref. A Guyton and Hall Textbook of Medical Physiology, By John E. Hall, 13th edition, page no. 339)
• Endogenous creatinine clearance is slightly higher than inulin clearance since some creatinine is also secreted by the tubules.
• Despite this shortcoming, creatinine clearance remains the most common clinical method to estimate GFR when
accurate measurements are needed.
• However, for routine practice, creatinine clearance is estimated.
• Inputting age, sex, ethnicity, and serum creatinine, deploying regression equations such as the MDRD equation or the
Cockcroft–Gault equation
3. Answer: D (Ref. “Previous question”)
• Iothalamate is handled like inulin, and its clearance has been used to measure GFR.
• Glucose is normally not excreted by the nephron. So it cannot be used to measure GFR.
• Phenol red has been used to estimate renal plasma flow – Tubular cells secrete it.
4. Answer: B (Ref. A Guyton and Hall Textbook of Medical Physiology, By John E. Hall, 13th edition, page no. 336)
5. Answer: B (Ref. A Guyton and Hall Textbook of Medical Physiology, By John E. Hall, 13th edition, page no. 338)
6. Answer: D (Ref. “Previous question”)
7. Answer: A (Ref. A Guyton and Hall Textbook of Medical Physiology, By John E. Hall, 13th edition, page no. 431)
Mesangial cells contract, surface area available for filtration reduces.
8. Answer: B (Ref. A Guyton and Hall Textbook of Medical Physiology, By John E. Hall, 13th edition, page no. 374)
9. Answer: C (Ref. A Guyton and Hall Textbook of Medical Physiology, By John E. Hall, 13th edition, page no. 356)
10. Answer: B (Ref. A Guyton and Hall Textbook of Medical Physiology, By John E. Hall, 13th edition, page no. 234)
11. Answer: A (Ref. A Guyton and Hall Textbook of Medical Physiology, By John E. Hall, 13th edition, page no. 354)
12. Answer: A (Ref. A Guyton and Hall Textbook of Medical Physiology, By John E. Hall, 13th edition, page no. 342)
13. Answer: B (Ref. A Guyton and Hall Textbook of Medical Physiology, By John E. Hall, 13th edition, page no. 342)
In the kidneys, PGE2 has vasodilator effects

https://t.me/DentalBooksWorld
Chapter 15 • Physiology 1341

14. Answer: B (Ref. “Previous question”)


• Clearance ratio is the ratio of the clearance of a substance by the kidneys to the clearance of inulin.
• The clearance of substances secreted by the nephron is greater than inulin clearance.
• In contrast, the clearance of substances (such as glucose) that are reabsorbed by the nephron is less than inulin clearance.
15. Answer: B (Ref. A Guyton and Hall Textbook of Medical Physiology, By John E. Hall, 13th edition, page no. 366)
16. Answer: B (Ref. A Guyton and Hall Textbook of Medical Physiology, By John E. Hall, 13th edition, page no. 366)
17. Answer: C (Ref. “Previous question”)
18. Answer: A (Ref. “Previous question”)
19. Answer: C (Ref. A Guyton and Hall Textbook of Medical Physiology, By John E. Hall, 13th edition, page no. 339)
Renal autoregulation defends changes in GFR and renal blood flow in the face of changes in renal perfusion pressure.
20. Answer: A (Ref. A Guyton and Hall Textbook of Medical Physiology, By John E. Hall, 13th edition, page no. 234)
Synthesis and secretion of renin by juxtaglomerular cells is regulated by renal blood flow as well as by sympathetic
innervation of JG cells.
The synthesis of ANG II is what is manipulated pharmacologically by ACE inhibitors but physiologically, this is not the
rate-limiting step.
21. Answer: B (Ref. A Guyton and Hall Textbook of Medical Physiology, By John E. Hall, 13th edition, page no. 343)
22. Answer: A (Ref. A Guyton and Hall Textbook of Medical Physiology, By John E. Hall, 13th edition, page no. 234)
23. Answer: A (Ref. “Previous question”)
24. Answer: D (Ref. “Previous question”)
• Angiotensin II (and not ACTH) is the most important trophic factor for the zona glomerulosa and one of the two most
important stimulators of aldosterone release.
• Even a 0.5 mM increase in plasma [K] stimulates aldosterone release.
• In turn, aldosterone regulates plasma [K] by promoting K secretion in the collecting ducts.
• Angiotensin III has 100% aldosterone releasing activity but only 40% of the pressor activity of aldosterone.ACTH as
well as a decrease in plasma [Na] stimulate aldosterone release but the stimulatory effects of acute administration of
pharmacologic doses of ACTH on aldosterone release are transient if at all.
25. Answer: A
26. Answer: A (Ref. “Previous question”)
27. Answer: A (Ref. “Previous question”)
28. Answer: A (Ref. “Previous question”)
29. Answer: A (Ref. “Previous question”)
• Luminal fluid becomes more and more concentrated in the thin descending limb because this portion of the nephron is
relatively impermeable to solute and freely permeable to water.
• Water exits the descending limb into the hypertonic medullary interstitium. Thus osmolality of tubular fluid is greatest
at the tip (apices) of the loops of Henle.
30. Answer: C (Ref. “Previous question”)
31. Answer: A (Ref. A Guyton and Hall Textbook of Medical Physiology, By John E. Hall, 13th edition, page no. 375)
32. Answer: D (Ref. A Guyton and Hall Textbook of Medical Physiology, By John E. Hall, 13th edition, page no. 345)
33. Answer: C (Ref. “Previous question”)
The ascending limb of the loop of Henle is impermeable to water but actively reabsorbs salt, and is called the diluting
segment of the nephron.
As long as active transport of salt occurs in the thick ascending limb, fluid emerging from the ascending limb of the loop of
Henle is always hypotonic.
34. Answer: C (Ref. “Previous question”)
35. Answer: A (Ref. “Previous question”)
36. Answer: B (Ref. “Previous question”)
37. Answer: B (Ref. “Previous question”)
38. Answer: C (Ref. A Guyton and Hall Textbook of Medical Physiology, By John E. Hall, 13th edition, page no. 391)
39. Answer: D (Ref. A Guyton and Hall Textbook of Medical Physiology, By John E. Hall, 13th edition, page no. 378)
40. Answer: B (Ref. “Previous question”)
41. Answer: A (Ref. “Previous question”)
42. Answer: D (Ref. “Previous question”)

https://t.me/DentalBooksWorld
1342 Triumph’s Complete Review of Dentistry

43. Answer: C (Ref. A Guyton and Hall Textbook of Medical Physiology, By John E. Hall, 13th edition, page no. 949)
44. Answer: B (Ref. “Previous question”)
45. Answer: A (Ref. A Guyton and Hall Textbook of Medical Physiology, By John E. Hall, 13th edition, page no. 393)
Aldosterone is the hormone regulating secretion of potassium in the collecting ducts, and hyperaldosteronism is typically
associated with hypokalemia though most patients with primary hyperaldosteronism are normokalemic.
46. Answer: B (Ref. A Guyton and Hall Textbook of Medical Physiology, By John E. Hall, 13th edition, page no. 326)
47. Answer: D (Ref. A Guyton and Hall Textbook of Medical Physiology, By John E. Hall, 13th edition, page no. 358)
48. Answer: D (Ref. A Guyton and Hall Textbook of Medical Physiology, By John E. Hall, 13th edition, page no. 383)
49. Answer: B (Ref. A Guyton and Hall Textbook of Medical Physiology, By John E. Hall, 13th edition, page no. 324)
50. Answer: D (Ref. “Previous question”)
51. Answer: A (Ref. A Guyton and Hall Textbook of Medical Physiology, By John E. Hall, 13th edition, page no. 419)
52. Answer: A (Ref. “Previous question”)
53. Answer: A (Ref. “Previous question”)
54. Answer: A (Ref. “Previous question”)
55. Answer: A (Ref. A Guyton and Hall Textbook of Medical Physiology, By John E. Hall, 13th edition, page no. 419)
56. Answer: C
• pH of proximal tubular fluid drops from 7.4 to 6.8 whereas in the “classic distal tubule” it can drop to as low as 4.4
• Although the amount of protons secreted is much higher in the PT, acidification of tubular fluid occurs to a greater
extent in the aldosterone sensitive distal nephron
57. Answer: A (Ref. A Guyton and Hall Textbook of Medical Physiology, By John E. Hall, 13th edition, page no. 368)
58. Answer: C (Ref. “Previous question”)
59. Answer: B (Ref. “Previous question”)
The amount of alkali required to titrate acidic urine to the pH of arterial plasma is called titratable acidity; normally, most
of this is due to protons buffered by phosphate as H2PO4.
In patients with chronic renal failure, protons buffered by creatinine and uric acid constitute about 20% of titratable acid.
60. Answer: B (Ref. A Guyton and Hall Textbook of Medical Physiology, By John E. Hall, 13th edition, page no. 357)
61. Answer: B (Ref. A Guyton and Hall Textbook of Medical Physiology, By John E. Hall, 13th edition, page no. 426)
• Type A intercalated cells secrete protons (and conserve bicarbonate); acid-secretion by intercalated cells is stimulated by
aldosterone.
• Proton secretion is also affected by transepithelial voltage; a lumen negative transepithelial voltage favors secretion of
protons and potassium ions in the collecting duct.
• Type B intercalated cells have been observed in the context of metabolic alkalosis.
• They express pendrin, a Cl–HCO3 exchanger, on the luminal membrane, and secrete bicarbonate in exchange for
luminal chloride.
62. Answer: D (Ref. A Guyton and Hall Textbook of Medical Physiology, By John E. Hall, 13th edition, page no. 422)
As long as alveolar ventilation and pulmonary blood flow are adequate and ventilation–perfusion matching is optimal, CO2
elimination is not a concern even if there is a reduction in diffusion capacity of the lungs for oxygen because CO2 rapidly
equilibrates across the alveoli.
In contrast → oxygenation of blood is more readily compromised by a reduction in diffusion capacity of lungs even when
ventilation–perfusion balance and pulmonary blood flow are optimal.
63. Answer: B (Ref. A Guyton and Hall Textbook of Medical Physiology, By John E. Hall, 13th edition, page no. 426)
Base excess = Observed buffer base – normal buffer base
Base excess is present in metabolic alkalosis, and it is negative in metabolic acidosis.
64. Answer: C (Ref. A Guyton and Hall Textbook of Medical Physiology, By John E. Hall, 13th edition, page no. 423)
• If bicarbonate and PaCO2 are abnormal and the pH of arterial plasma appears to be WNL, this suggests the possibility of
a mixed acid–base disturbance,
• such as mixed metabolic acidosis and respiratory alkalosis; or mixed respiratory acidosis and metabolic alkalosis
65. Answer: C (Ref. A Guyton and Hall Textbook of Medical Physiology, By John E. Hall, 13th edition, page no. 378)
66. Answer: C (Ref. A Guyton and Hall Textbook of Medical Physiology, By John E. Hall, 13th edition, page no. 351)
67. Answer: A (Ref. A Guyton and Hall Textbook of Medical Physiology, By John E. Hall, 13th edition, page no. 969)
68. Answer: A (Ref. A Guyton and Hall Textbook of Medical Physiology, By John E. Hall, 13th edition, page no. 372)
69. Answer: C (Ref. A Guyton and Hall Textbook of Medical Physiology, By John E. Hall, 13th edition, page no. 212)

https://t.me/DentalBooksWorld
Chapter 15 • Physiology 1343

70. Answer: B (Ref. A Guyton and Hall Textbook of Medical Physiology, By John E. Hall, 13th edition, page no. 351)
71. Answer: B (Ref. “Previous question”)
72. Answer: B (Ref. “Previous question”)
73. Answer: B (Ref. “Previous question”)
74. Answer: C (Ref. A Guyton and Hall Textbook of Medical Physiology, By John E. Hall, 13th edition, page no. 439)
75. Answer: C (Ref. A Guyton and Hall Textbook of Medical Physiology, By John E. Hall, 13th edition, page no. 431)
76. Answer: C (Ref. A Guyton and Hall Textbook of Medical Physiology, By John E. Hall, 13th edition, page no. 235)
77. Answer: D (Ref. A Guyton and Hall Textbook of Medical Physiology, By John E. Hall, 13th edition, page no. 431)
78. Answer: D (Ref. “Previous question”)
79. Answer: D (Ref. “Previous question”)
80. Answer: D (Ref. “Previous question”)
81. Answer: C (Ref. “Previous question”)
82. Answer: A (Ref. A Guyton and Hall Textbook of Medical Physiology, By John E. Hall, 13th edition, page no. 422)
83. Answer: D (Ref. A Guyton and Hall Textbook of Medical Physiology, By John E. Hall, 13th edition, page no. 426)
84. Answer: D (Ref. A Guyton and Hall Textbook of Medical Physiology, By John E. Hall, 13th edition, page no. 366)
85. Answer: C (Ref. A Guyton and Hall Textbook of Medical Physiology, By John E. Hall, 13th edition, page no. 330)
86. Answer: D (Ref. A Guyton and Hall Textbook of Medical Physiology, By John E. Hall, 13th edition, page no. 305)
87. Answer: C (Ref. “Previous question”)
88. Answer: D (Ref. A Guyton and Hall Textbook of Medical Physiology, By John E. Hall, 13th edition, page no. 366)
89. Answer: C (Ref. “Previous question”)
90. Answer: B (Ref. A Guyton and Hall Textbook of Medical Physiology, By John E. Hall, 13th edition, page no. 448)
91. Answer: B (Ref. A Guyton and Hall Textbook of Medical Physiology, By John E. Hall, 13th edition, page no. 781)
92. Answer: D (Ref. A Guyton and Hall Textbook of Medical Physiology, By John E. Hall, 13th edition, page no. 378)
93. Answer: C (Ref. A Guyton and Hall Textbook of Medical Physiology, By John E. Hall, 13th edition, page no. 420)
94. Answer: C (Ref. A Guyton and Hall Textbook of Medical Physiology, By John E. Hall, 13th edition, page no. 236)
95. Answer: C (Ref. A Guyton and Hall Textbook of Medical Physiology, By John E. Hall, 13th edition, page no. 351)
96. Answer: C (Ref. A Guyton and Hall Textbook of Medical Physiology, By John E. Hall, 13th edition, page no. 423)
97. Answer: C (Ref. “Previous question”)
98. Answer: B (Ref. “Previous question”)
99. Answer: A (Ref. “Previous question”)
100. Answer: C (Ref. “Previous question”)
101. Answer: D (Ref. A Guyton and Hall Textbook of Medical Physiology, By John E. Hall, 13th edition, page no. 339)
102. Answer: A (Ref. A Guyton and Hall Textbook of Medical Physiology, By John E. Hall, 13th edition, page no. 365)

https://t.me/DentalBooksWorld
16 General Medicine

SYNOPSIS

HEMATOLOGY
General Presenting Complaints
General Weakness and orthostasis
Musculoskeletal (joints) Tingling, cracking, warmth, pain, stiffness, refusal to use the joint (young children)
Central nervous system (CNS) Headache, stiff neck, vomiting, lethargy, irritability, spinal cord syndromes
Gastrointestinal (GI) Hematemesis, melena, frank red blood per rectum, abdominal pain
Genitourinary Hematuria, renal colic, postcircumcision bleeding
Other Epistaxis, oral mucosal hemorrhage, hemoptysis, dyspnea (hematoma leading to
airway obstruction), compartment syndrome symptoms, contusions, excessive
bleeding with routine dental procedures
Systemic signs of hemorrhage:
• Tachycardia
• Tachypnea
• Hypotension
• Orthostasis

Terminologies:
• Erythrocytes that have a normal size or volume (normal MCV) are called normocytic
• When the MCV is high, they are called macrocytic
• When the MCV is low, they are termed microcytic Erythrocytes containing the normal amount of hemoglobin (normal
MCHC) are called normochromic
• When the MCHC is abnormally low they are called hypochromic, and when the MCHC is abnormally high they are called
hyperchromic

Normal Values:
Serum iron 50–150 µg/dl
Ferritin 50–200 µg/l
Total iron binding capacity 300–360 µg/dl
MCV 79–93
MCH 27–31 pg/cell
MCHC 32–35
% Saturation 30–50%
Red cell distribution width <14.5%
Free erythrocytic porphyrin Less than 30 µg/dl for men, and less than 40 µg/dl levels for women

https://t.me/DentalBooksWorld
Chapter 16 • General Medicine 1345

MCV
To calculate the MCV expressed in femtoliters (fl or 10-15L), the following formula is used:
Hematocrit (%) × 10
MCV =
RBC count (millions/mm3 blood)
MCHC
To calculate the MCHC, expressed as grams of hemoglobin per 100 ml packed cells, the following formula is used:
Hemoglobin (g/100ml) × 100
MCHC =
Hematocrit (%)

MCV MCHC
Female 80–95 fl 30–34 gHb/100 ml
Male 80–95 fl 30–34 gHb/100 ml

ANEMIA
Definition:
It is defined as a decrease in the total amount of red blood cells (RBCs) or hemoglobin in the blood.
Classification:
Microcytic hypochromic Normocytic normochromic Macrocytic hypochromic
Iron deficiency anemia Sickle cell anemia Vitamin B12 deficiency
Lead poisoning Hemolytic anemia Thiamine deficiency
Thalassemia Hemangioma Folic acid deficiency
Sideroblastic anemia DIC
Blood loss
Anemia of chronic condition
Anemia due to renal failure
Fanconi’s anemia
Malignancy
Red cell aplasia

Tests for Anemia


Test Normal IDA Chronic disease Thalassemia Sideroblastic Renal disease
anemia
Serum iron 50–150 µg% D D N (or) I N (or) I N
TIBC 250–400 µg% I D N N N
% of 15–50% D D N (or) I N (or) I N
saturation of
iron
Serum 30–150 µg/L D I N (or) I N –
ferritin
Marrow iron + to ++ Zero I – Sideroblasts I
stores
RWD 12–15% I – N (or) D N (or) D –
Reticular <2% D – I – –
count

https://t.me/DentalBooksWorld
1346 Triumph's Complete Review of Dentistry

SICKLE CELL ANEMIA


• HbS: Replacement of glutamate by valine at position 6 Beta chain of HbA
• HbC: Replacement of glutamate by lysine at position 6 Beta chin of HbA
• HbE: Replacement of glutamate by lysine at position 26 Beta chain of HbA
Caused by a mutation in a Beta globin gene that changes the sixth amino acid from glutamic acid to valine in the beta chain
of HbA (Alpha 2 Beta 2)

Clinical Features:
• Increased mechanical fragility/hemolysis
• Decreased osmotic fragility
• Cardiomegaly/heart failure
• Hepatomegaly
• Stunted growth
• Bossing of skull
• Fish mouth vertebra
• Chronic leg ulcer
• Osteomyelitis
• Priapism
• Hyposplenism
• Renal papillary necrosis
• Retinal hemorrhage
• Pulmonary hypertension
• Increased blood viscosity
• Icterus/cholelithiasis
• Infarction/painful crisis → Severe skeletal pain but no change in Hb%

Factors Favoring Crisis:


• Decreased pH
• Increased Hb concentration
• Combination of HbS with other Hb
• Hypoxic (2,3 Bisphosphoglycerate increases polymerization)

THALASSEMIA
• Are characterized by decreased rate of synthesis of Hb (Beta) chains, which are structurally normal
• Most common mutation in Beta Thalassemia is the intervening sequence 1 (IVS-1) or INTRON
TYPES:
THALASSEMIA HbA2 HbF
Alpha thalassemia trait Normal Normal
Beta thalassemia trait Increased Increased
Thalassemia minor Increased Normal

Evan’s Syndrome → Thrombocytopenia + Immune hemolysis


**Does not present with microangiopathic features or fragmented RBCs

https://t.me/DentalBooksWorld
Chapter 16 • General Medicine 1347

PAROXYSMAL NOCTURNAL HEMOGLOBINURIA


• It is an acquired, intravascular disorder, acquired at Stem Cell Level
• Acidification enhances activity of complement
• Brown urine in the morning
• Three common manifestations:
–– Hemolytic anemia
–– Venous thrombosis
–– Deficient hematopoiesis

LEUKOCYTE ALKALINE PHOSPHATASE: Leukocyte alkaline phosphatase (LAP) is the term for alkaline phosphatase
that is found in leukocytes. LAP test is used to check for signs of CML or other conditions.
Conditions with increased LAP score Conditions with decreased LAP score
• Polycythemia • PNH
• Leukemoid reaction • Chronic myeloid leukemia
• Infection
• Essential thrombocytosis

FANCONI’S ANEMIA
• Autosomal recessive
• Radial ray anomaly
• Microcephaly
• Solid tumors
• Lead to AML
• Pancytopenia with hypocellular marrow
• Normochromic normocytic anemia – Aplastic anemia
• Premalignant state
• Positive family history
• Inherited chromosomal stability syndrome
• Hyperpigmentation (Cafe au lait spots)
• Hypopigmentation

HODGKIN’S LYMPHOMA
Classical Histologically distinct type
• Mixed cellularity (MC) • Lymphocyte predominant (LD)
• Nodular sclerosis (NS)
• Lymphocyte rich (LR)
• Lymphocyte depleted (LD)

REED–STERNBERG CELL
Reticular variant Lymphohistiocytic variant Mononuclear variant Lacunar variant
LD type LP type MC and LR type NS type

https://t.me/DentalBooksWorld
1348 Triumph's Complete Review of Dentistry

FACTS ABOUT LYMPHOMA


• Most common type all over the world – NS
• Most common type in India – MC
• Least common type – LD
• Best prognosis – LP
• Worst prognosis – LD
• Association with EBV – MC
• Popcorn cells seen in – LP
• Lacunate cells seen in – NS
• Mononuclear variant seen in – MC/LR
• Reticular variant seen in – LD
• Classic Reed–Sternberg cells seen in – MC
• CD 15, 30 positive/CD 45 negative – NS/MC/LR/LD
• CD 15, 30 negative/CD 45 positive – LP
• Rye’s classification is for – Hodgkin’s lymphoma
• Rx for classical HD (Limited) – CT + RT
(Advanced) – CT (+/− RT)
• Rx for nodular/LP Hodgkin’s disease (Limited) – RT
(Advanced) – CT (+/− RT)
• Most common type – NS > MC > LP > LD
• Prognosis (good to worst) – LP > NS > MC > LD
• Association with EBV – MC (+/− LD)
• Variant more common in females – NS
• Mediastinal involvement particularly common in – NS
• Most common lymph node involved in Hodgkin’s disease – cervical and supraclavicular
• CNS involvement is an uncommon feature in Hodgkin’s Disease
• Hodgkin’s Disease – paraneoplastic syndrome – cerebellar degenerative disease – anti-Tr antibodies
• Absolute lymphocyte count <600/µl is a poor prognostic factor for Hodgkin’s disease

ANN ARBOR STAGING FOR HODGKIN’S DISEASE


• Stage 1: Single lymph node
• Stage 2: 2 or 3 lymph nodes on same side of diaphragm
• Stage 3:
– 1–Both sides – Spleen
– 2–Both sides – Iliac
• Stage 4: Extranodal site
– Liver or bone marrow
– A–No symptoms
– B–Unexplained weight loss, fever
– C–Localized extralymphatic tissue (without liver/bone marrow)
• MOPP regimen – Main adv effect – Infertility/sterility
• International prognostic index for non-Hodgkin’s lymphoma – Age (>60 years)/stage of disease (3 or 4)/LDH levels/
performance status/number of extranodal sites involved (>1)

https://t.me/DentalBooksWorld
Chapter 16 • General Medicine 1349

NON-HODGKIN’S DISEASE
B-CELL NEOPLASMS T-CELL NEOPLASMS
• Burkitt’s lymphoma • Mycosis fungoides
• Hairy cell leukemia • Adult T-cell lymphoma
• Mantle cell lymphoma • Angiocentric lymphoma
• Follicular lymphoma
• Multiple myeloma

POLYCYTHEMIA
• Polycythemia is an increased number of red blood cells in the blood. In polycythemia, the levels of Hemoglobin (Hb), hematocrit,
or the red blood cell (RBC) count may be elevated when measured in the complete blood count, as compared to normal.
• Hemoglobin levels greater than 16.5 g/dl in women and greater than 18.5 g/dl in men suggest polycythemia. In terms of
hematocrit, a value greater than 48 in women and 52 in men is indicative of polycythemia.
• Production of red blood cells (erythropoiesis) occurs in the bone marrow and is regulated in a series of specific steps.
• One of the important enzymes regulating this process is called erythropoietin (epo). The majority of Epo is produced and
released by the kidneys, and a smaller portion is released by the liver.
• Polycythemia can result from internal problems with the production of red blood cells. This is termed primary polycythemia.
If polycythemia is caused due to another underlying medical problem, it is referred to as secondary polycythemia.
• Most cases of polycythemia are secondary and are caused by another medical condition. Primary polycythemias are
relatively rare.
• 1–5% of newborns can have polycythemia (neonatal polycythemia).
• Polycythemia vera (PV) is related to a genetic mutation in the JAK2 gene, which is thought to increase the sensitivity of bone
marrow cells to Erythropoietin.
• Primary familial and congenital polycythemia (PFCP) is a condition related to a mutation in the EPOR gene and causes
increased production of red blood cells in response to Erythropoietin.

BURKITT’S LYMPHOMA
• Translocation in Burkitt’s lymphoma:
–– C myc gene on chromosome 8
t (8;14) Heavy chain
t (8;22) Light chain
t (2;8) Light chain

Hairy cell leukemia:- (Large B cells)


Presence of tartrate-resistant acid phosphatase “TRAP” in neoplastic B cells

MYCOSIS FUNGOIDES (misnomer):


• Cutaneous T-cell Lymphoma
• **Pautrier’s microabscess
• Sezary–Lutzner cells → atypical lymphocytes with grooved or cerebriform nucleus seen both in tissue and cells

MULTIPLE MYELOMA
• Anemia (normocytic normochromic)
• Infections
• Bleeding tendency

https://t.me/DentalBooksWorld
1350 Triumph's Complete Review of Dentistry

• Lytic lesions – Bone pain


–– Pathological fracture
–– Cord compression
• Classic triad hyperviscosity syndrome (infrequent)
–– Marrow plasmacytosis >10%
–– Lytic bone lesions
–– Serum or urine M component
• Hypercalcemia (metastatic calcification)
• Cryoglobulinemia (Raymond’s phenomenon)
• Osteoporosis
• Amyloidosis
• Proteinuria
• Visual disturbance
• Renal failure
• IgG – most common
• IgD/IgE – least common
• Hypogammaglobulinemia

The Durie–Salmon myeloma diagnostic criteria


Major
• Plasmacytoma on tissue biopsy
• BM Plasmacytomas >30% plasma cells
• Monoclonal M spike on electrophoresis
–– IgG – >3.5 g/dl
–– IgA – >2.0 g/dl

LEUKOPENIA
The leukopenia definition states that it is a decrease in the white blood cell (WBC) count of the body and can directly affect
the body’s ability to fight infections.
ETIOLOGY
1. General Causes
• These include viral or bacterial infections, like HIV, malaria, influenza, typhoid, dengue, tuberculosis, sepsis, psittacosis,
Lyme disease, and rickettsial infections. These infections can disrupt the bone marrow function of the body and decrease
the WBC count.
• In addition to infections, some autoimmune diseases like myelokathexis and diseases like aplastic anemia and systemic
lupus erythematosus (SLE) also cause decrease in WBCs.
• Besides, certain types of cancer, like Hodgkin’s lymphoma and leukemia, can decrease the total white blood count in the
body.
• Malnutrition, along with deficiencies in the essential nutrients like copper and zinc in the body, can cause low WBC.
2. Medications
• An antipsychotic drug, clozapine, causes total eradication of all granulocytes in the blood.
• Bupropion HCl, which is an antidepressant and a drug used for smoking addiction treatment, also leads to decrease in
leukocytes after prolonged use.
• Drugs used for mania, migraine, and epilepsy – valproic acid and lamotrigine (antiepileptic) – also cause leukopenia.
• A common antibacterial drug, metronidazole, also leads to decreased WBCs.

https://t.me/DentalBooksWorld
Chapter 16 • General Medicine 1351

• Many immunosuppressive drugs like sirolimus, mycophenolate mofetil, tacrolimus, cyclosporine, leflunomide, and
many TNF inhibitors can cause leukopenia.
• Interferon proteins that are used commonly in multiple sclerosis treatment, e.g., Rebif, Avonex, and Betaseron, can lead
to leukopenia.
• Chemotherapy and radiation therapy for treating cancer can severely decrease the leukocytes in the body. The major
treatment for cancer is the decrease in the total count of neutrophils or neutropenia.
• Arsenic poisoning can, to some extent, cause leukopenia.

LEUKEMIA
Feature Acute lymphocytic Acute Chronic lymphocytic Chronic
myelogenous myelogenous
Peak age of incidence Childhood Any age Middle and old age Young adulthood
WBC count High in 50% High in 60% High in 98% High in 100%
Normal or low in 50% Normal or low in 40% Normal or low in 2%
Differential WBC Many lymphoblasts Many myeloblasts Small lymphocytes Entire myeloid series
count
Anemia Severe in >90% Severe in >90% Mild in about 50% Mild in 80%
Platelets Low in >80% Low in >90% Low in 20–30% High in 60%
Low in 10%
Lymphadenopathy Common Occasional Common Infrequent
Splenomegaly In 60% In 50% Usual and moderate Usual and severe
Other features Without prophylaxis, CNS rarely involved Occasionally Philadelphia
CNS commonly Sometimes Auer rods hemolytic anemia and chromosome
involved hypogammaglobulinemia Positive in >90%
leukocyte alkaline
phosphatase

CHRONIC MYELOID LEUKEMIA


• Reciprocal translocation between long arm of chromosomes 9 and 22
• Break point cluster region (BCR) located on chromosome 22
• ABL gene located on chromosome 9
• Philadelphia chromosome with oncogenic BCR–ABL gene is found on chromosome 9

Clinical Features:
• Leukocytosis
• Elevated basophils
• Blasts <5%
• Thrombocytosis
• Promyelocytes <10%
• Mildly anemic
• Myeloid: Erythroid is increased
• Bone Marrow is hypercellular
• Leukocytosis, mature forms and intermediate, immature forms are less common blasts <5% and promyelocytes <10%,
thrombocytosis, no anemia, M:E ratio is increased, decreased LAP scores, vitamin B12 proteins in increased binding
• Marker for B lymphocyte – CD 19,10,20

https://t.me/DentalBooksWorld
1352 Triumph's Complete Review of Dentistry

Treatment:
• Only curative Rx of CML is allogenic stem cell
• Massive splenomegaly
• Transplantation
• Drug of choice – Imatinib

ACUTE MYELOID LEUKEMIA: A cancer of the myeloid line of blood cells, characterized by the rapid growth of
abnormal cells that build up in the bone marrow and blood and interfere with normal blood cells.
Classification – FAB (French American British)
Subtype Name
M0 Undifferentiated acute myeloblastic leukemia
M1 Acute myeloblastic leukemia with minimal maturation
M2 Acute myeloblastic leukemia with maturation
M3 Acute promyelocytic leukemia (APL)
M4 Acute myelomonocytic leukemia
M4 Acute myelomonocytic leukemia with eosinophilia
M5 Acute monocytic leukemia
M6 Acute erythroid leukemia
M7 Acute megakaryoblastic leukemia
Auer rods are seen in AML (clumps of azurophilic granular material). Most common in M3 and absent in M0.

ACUTE LYMPHOID LEUKEMIA


• Most common type
• pre B ALL
–– presents as mediastinal enlargement (T-cell ALL)
• Rx – L Asparaginase

CONDITIONS WITH INCREASED ESR


• Pregnancy
• Menstruation
• Multiple myeloma
• Anemia other than sickle cell
• Leukocytosis
• New born
• DLE
CONDITIONS WITH DECREASED ESR
• Polycythemia → due to increase in zeta potential
• Congestive heart failure
• Sickle cell anemia
• Afibrinogenemia

https://t.me/DentalBooksWorld
Chapter 16 • General Medicine 1353

IMMUNOGLOBULIN DISEASE
• Franklin’s disease – gamma heavy chain disease
• Seligmann’s disease – alpha chain disease (most common)
• Heavy chain disease with kappa light chain – Mu chain disease

BLEEDING AND COAGULATION DISEASE


Vitamin C deficiency
• Increased bleeding time
• Normal CT/PT/aPTT/TT

Vitamin K deficiency
• Normal bleeding time
• Increased CT/PT/aPTT/TT
• The effect of aspirin on platelet function occurs within 1 hour and lasts for the duration of the affected platelets life span,
that is, 1 week

Plasma t1/2 for clotting factors:


• Fibrinogen – 2–4 days
• Prothrombin – 3–4 days
• Factor – 1–1½ days
• Factor 7 – 4–6 hours (least)
• Factor 8 – 8–12 hours
• Factor 9 – 18–24 hours
• Factor 10 – 40–60 hours
• Factor 11 – 40–70 hours
• Factor 12 – 60 hours (2–3 days)
• High molecular weight kininogen – 150 hours (5–6 days)
• Prekallikrein – 35 hours (1–1½) days
• Incidence of hemophilia A – 1:5,000 (factor 8)
• Incidence of hemophilia B – 1:30,000 (factor 9)
• aPTT is prolonged with deficiency of factors – 12, 11, 8, 9, 2, 1, and thrombin

DISORDERS OF PLATELET FUNCTION


I. Platelet – vessel wall interaction (adhesion)
1. Von Willebrand disease (deficiency of vWF)
2. Bernard–Soulier syndrome (deficiency of Gp1b receptor)
II. Platelet – platelet interaction (aggregation)
1. Congenital afibrinogenemia (deficiency of plasma fibrinogen)
2. Glanzmann thrombasthenia (deficiency of Gp2b–3a receptor)
III. Platelet secretion and signal transduction
1. Storage pool deficiency
2. Quebec platelet disorder
3. Chediak–Higashi syndrome
4. Grey platelet syndrome – Alpha Granules
5. Wiskott–Aldrich syndrome
IV. Disorders of platelet coagulation – protein interaction
1. Scott Syndrome (defect in factor 5a–10a interaction on platelets)
https://t.me/DentalBooksWorld
1354 Triumph's Complete Review of Dentistry

VON WILLEBRAND FACTOR DISEASE


Property vWF Factor 8
Gene Located on chromosome 12 Located on chromosome 10
Inheritance AD Sex linked
Synthesis Endothelial/platelet/megakaryocytes Liver cells
Function Adhesion of platelets to vessel wall Activation of factor 10
Disease caused vWF disease Hemophilia
Ristocetin-induced platelet aggregation is decreased in vWF
vessel wall contains (endothelium) → vWF and platelet contains → Gp 1b/9 receptors

WISKOTT–ALDRICH SYNDROME
• X-linked recessive syndrome
• xp11.23
• Triad – Eczema/Thrombocytopenia/Immunodeficiency
• Decreased platelet size and count
• Impaired platelet aggregation response
• Decreased IgM, normal IgG and IgA, decreased CD-8 T-cell count

BERNARD–SOULIER SYNDROME GLANZMANN THROMBASTHENIA


Autosomal recessive Autosomal recessive
Platelets cannot adhere Platelets cannot aggregate
Deficiency of Gp 1b/9 receptor Deficiency of Gp 2b/3a complex
Aggregation with collagen/ADP is normal, only abnormal with Aggregation with collagen/ADP/thrombin is normal, only
ristocetin abnormal with ristocetin
Decreased platelet count (Thrombocytopenia) Normal platelet count
Large platelets Normal platelet size
Prolonged bleeding time Prolonged bleeding time
vWF is normal vWF is normal
Episodes of mucosal hemorrhage is seen Episodes of mucosal hemorrhage is seen

CAUSES OF THROMBOCYTOPENIA
ITP/TTP/SLE/Post-transfusion/HUS/DIC/ Drugs – Gold, Heparin, Quinine, Sulfonamides, Aplastic anemia (it specifically
causes nonmegakaryocytic thrombocytopenia), Vitamin B12, and folic acid deficiency/leukemia/metastasis/radiation/
Wiskott–Aldrich syndrome
Idiopathic thrombocytopenic purpura:
• Thrombocytopenia
• Autoimmune antibodies against platelets
• Prolonged bleeding time
• Petechiae/ecchymosis
• Acute → Children/often preceded by viral infection
• Chronic → Adults/Females
• No splenomegaly
Thrombotic thrombocytopenic purpura:
• Antibodies to ADAMTS 13

https://t.me/DentalBooksWorld
Chapter 16 • General Medicine 1355

Intravascular hemolysis:
• Microangiopathic hemolytic anemia
• No hepatosplenomegaly
• Thrombocytopenia
• Decreased renal function
• Disturbed neurological function – not seen in HUS
• Fever
• Increased LDH levels
• Hemolysis
• Normal coagulation tests
• Coombs test negative and not mediated by auto antibodies

DISSEMINATED INTRAVASCULAR COAGULATION


• Intravascular activation of coagulation by both intrinsic and extrinsic mechanism
• Two stages – Thrombotic and consumption phase
• Stage of secondary fibrinolysis, at site of intravascular coagulation
• aPTT/PT/BT/TT/Plasmin levels/Fibrin degradation products – increased
• Presence of schistocytes/Burr cells/Spherocytes/Helmet cells in the peripheral fibers
• Most common site for thrombin formation in DIC is brain > heart > lung > kidney > adrenal > liver
• Two important endocrine manifestations
1. Adrenals – Friedrich Hausen syndrome
2. Pituitary – Sheehan’s syndrome
• Most sensitive test for DIC is increased FDP levels
• Most sensitive test for DIC is that it predicts tendency of bleeding in serum fibrinogen levels
• Thrombocytopenia is present
• Hypofibrinogenemia

SHEEHAN’S SYNDROME
• Sheehan’s syndrome is a condition that affects women who lose a life-threatening amount of blood in childbirth or who have
severe low blood pressure during or after childbirth, which can deprive the body of oxygen. In Sheehan’s syndrome, the lack
of oxygen can damage your pituitary
• Sheehan’s syndrome causes the pituitary gland to not produce enough pituitary hormones (hypopituitarism). Also called
postpartum hypopituitarism, Sheehan’s syndrome is rare in industrialized nations, largely due to improved obstetrical care.
But it is a major threat to women in developing countries.
Causes:
• Sheehan’s syndrome is caused by severe blood loss or extremely low blood pressure during or after childbirth. These factors
can be particularly damaging to the pituitary gland, which enlarges during pregnancy, destroying hormone-producing
tissue so that the gland cannot function normally.
Signs and Symptoms:
• Difficulty breast-feeding or an inability to breast-feed
• No menstrual periods (amenorrhea) or infrequent menstruation (oligomenorrhea)
• Inability to regrow shaved pubic hair

https://t.me/DentalBooksWorld
1356 Triumph's Complete Review of Dentistry

• Slowed mental function, weight gain, and difficulty staying warm as a result of an underactive thyroid (hypothyroidism)
• Low blood pressure (hypotension)
• Low blood sugar (hypoglycemia)
• Fatigue
• Irregular heartbeat
• Breast shrinkage
Treatment of Sheehan’s syndrome involves lifelong hormone replacement therapy

CARDIOVASCULAR SYSTEM
CORONARY HEART DISEASE (CHD)
Coronary heart disease is the most common form of heart disease and the single most important cause of premature death
in America and European countries.

CORONARY HEART DISEASE – CLINICAL MANIFESTATIONS AND PATHOLOGY


Clinical problem Pathology
Stable angina Ischemia due to fixed atheromatous stenosis of one or more
coronary arteries
It describes the features of “stable” angina pectoris which
occurs when coronary perfusion is impaired by fixed or stable
atheroma of the coronary arteries
Unstable angina (a component of acute coronary syndrome – Ischemia is caused by dynamic obstruction of a coronary
explained below) artery due to plaque rupture with superimposed thrombosis
and spasm
Myocardial infarction Myocardial necrosis is caused by acute occlusion of a coronary
artery due to plaque rupture and thrombosis
Heart failure Myocardial dysfunction due to infarction or ischemia
Arrhythmia Altered conduction due to ischemia or infarction
Sudden death Ventricular arrhythmia, asystole, or massive myocardial
infarction

RISK FACTORS OF CORONARY HEART DISEASE


Nonmodifiable factors:
• Increasing age
• Having family history of heart disease
Modifiable risk factors:
• Smoking – both active and passive
• High blood cholesterol
• High blood pressure
• Diabetes
• Physical inactivity
• Obesity
• Depression

https://t.me/DentalBooksWorld
Chapter 16 • General Medicine 1357

ANGINA PECTORIS
Angina pectoris is the symptom complex caused by transient myocardial ischemia and constitutes a clinical syndrome rather
than a disease; it may occur whenever there is an imbalance between myocardial oxygen supply and demand. Coronary
atheroma is the most common cause of angina.
UNSTABLE ANGINA
Unstable angina belongs to the spectrum of clinical presentations referred to collectively as acute coronary syndromes (ACSs),
which range from ST-segment elevation myocardial infarction (STEMI) to non-STEMI (NSTEMI). Unstable angina is considered
to be an ACS in which there is no detectable release of the enzymes and biomarkers of myocardial necrosis. See the image below.
PRINZMETAL ANGINA
Unlike typical angina – which is often triggered by exertion or emotional stress – Prinzmetal’s angina almost always occurs
when a person is at rest, usually between midnight and early morning. These attacks can be very painful.
Prinzmetal angina may also be referred to as:
• Variant angina
• Prinzmetal’s variant angina
• Angina inversa
Prinzmetal’s angina is rare, representing about two out of 100 cases of angina, and usually occurs in younger patients than
those who have other kinds of angina.
Causes of Variant (Prinzmetal) Angina: The pain from variant angina is caused by a spasm in the coronary arteries (which
supply blood to the heart muscle).
Symptoms of Variant (Prinzmetal) Angina:
The pain or discomfort:
• Usually occurs while resting and during the night or early morning hours
• Are usually severe
• Can be relieved by taking medication
Important feature – ST segment is elevated
Treatment of Variant Angina – Prinzmetal’s Angina
Medicines can help control the spasms. Drugs such as calcium antagonists and nitrates are the mainstays of treatment.
The spasms tend to come in cycles – appearing for a time, then going away. After 6–12 months of treatment, doctors may
gradually reduce the medication.
Prinzmetal’s angina is a chronic condition that will need to be followed by your healthcare provider even though the prognosis
is generally good.
SYNDROME X:
Cardiac syndrome X is angina (chest pain) with signs associated with decreased blood flow to the heart tissue but with
normal coronary arteries. Cardiac syndrome X is sometimes referred to as “microvascular angina” when there are findings
of microvascular dysfunction.

ACUTE CORONARY SYNDROME


Acute coronary syndrome (ACS) refers to a spectrum of clinical presentations ranging from those for ST segment elevation
myocardial infarction (STEMI) to presentations found in non-ST segment elevation myocardial infarction (NSTEMI) or in
unstable angina. It is almost always associated with rupture of an atherosclerotic plaque and partial or complete thrombosis
of the infarct related artery
Common signs of an acute coronary syndrome:
• Chest pain or discomfort, which may involve pressure, tightness, or fullness
• Pain or discomfort in one or both arms, the jaw, neck, back, or stomach
• Shortness of breath
• Feeling dizzy or lightheaded

https://t.me/DentalBooksWorld
1358 Triumph's Complete Review of Dentistry

• Nausea
• Sweating
Typically retrosternal, ill localized, compressing pain (Levine’s sign)

Pathogenesis

Adventitia

Lumen
Media
Asymptomatic
Intima
atherosclerotic
plaque

Stable fixed
atherosclerotic
plaque

Stable angina
Plaque disruption
and platelet aggregation
Thrombus

Unstable
plaque

Unstable angina Non−ST−segment ST−segment


elevation MI elevation MI

Acute coronary syndromes

MYOCARDIAL INFARCTION
A heart attack (also known as a myocardial infarction) is the death of heart muscle from the sudden blockage of a coronary
artery by a blood clot. Coronary arteries are blood vessels that supply the heart muscle with blood and oxygen. Blockage
of a coronary artery deprives the heart muscle of blood and oxygen, causing injury to the heart muscle. Injury to the heart
muscle causes chest pain and chest pressure sensation. If blood flow is not restored to the heart muscle within 20–40 minutes,
irreversible death of the heart muscle will begin to occur. Muscle continues to die for 6–8 hours at which time the heart attack
usually is “complete.” The dead heart muscle is eventually replaced by scar tissue.
Signs and symptoms
• Pain, fullness, and/or squeezing sensation of the chest
–– Jaw pain, toothache, headache
–– Shortness of breath
Typical chest pain in acute MI has the following characteristics:
• Intense and unremitting for 30–60 minutes
• Substernal, and often radiates up to the neck, shoulder, and jaw, and down the left arm
• Usually described as a substernal pressure sensation that also may be characterized as squeezing, aching, burning, or even sharp
• In some patients, the symptom is epigastric, with a feeling of indigestion or of fullness and gas

https://t.me/DentalBooksWorld
Chapter 16 • General Medicine 1359

PATHOPHYSIOLOGY OF MYOCARDIAL INFARCTION

Plaque disrupon or erosion

Thrombus formaon with or without embolizaton

Acute cardiac ischemia

Non-ST segment elevation ST segment elevaon

Markers of myocardial Elevated markers of Myocardial necrosis


necrosis not elevated myocardial necrosis markers elevated

Unstable angina Non-ST segment elevaon ST segment elevation


myocardial infarcon myocardial infarction
(Q waves usually absent) (Q waves usually present)

So the above three features unstable, Non-ST segment elevaon myocardial infarcon, and
ST segment elevaon myocardial infarcon – together FORMS ACUTE CORONARY SYNDROME

Cardiac Markers for Myocardial Infarction:


• Located in the myocardium
• Released in cardiac injury
–– Myocardial infarction
–– Non-Q-wave infarction
–– Unstable angina pectoris
–– Other conditions affecting cardiac muscle (trauma, cardiac surgery, myocarditis, etc.)
• Can be measured in blood samples
1. CREATININE KINASE–MB ISOENZYMES
• CK–MB isoenzymes have been the biochemical indicator of choice for the diagnosis of AMI
• Cardiospecificity of CK–MB is not 100%
• They are present in both skeletal and cardiac muscle
• False positive elevation occurs in trauma, heavy exertion, and myopathies
• First appears 4–6 hours after symptoms onset
• Peaks at 24 hours
• Returns to normal in 48–72 hours
2. MYOGLOBIN
• Myoglobin is a heme protein found in skeletal and cardiac muscles
• It is currently the earliest marker
• Rises 2–4 hours after onset of infarction
• Peaks at 6–12 hours
• Returns to normal within 24–36 hours
https://t.me/DentalBooksWorld
1360 Triumph's Complete Review of Dentistry

• Important for ruling out MI rather than ruling in


• Troponin complex: Three distinct subunits on thin filament: TnC, TnT, and TnI, which regulate myosin–actin Ca-dependent
interaction
• Troponins have high specificity for myocardial injury
–– Sensitive to minor myocardial damage
–– Appear 4–8 hours after symptoms onset
–– Remain elevated for up to 14 days post MI
–– Useful in risk stratification of patients with ACS
BEST MARKER FOR CARDIAC DISEASE DIAGNOSIS
• The best marker depends on the time from onset of symptoms
• The earliest marker is myoglobin and CK-MB isoforms
–– In the intermediate period (6–24 hours) CK–MB and Troponin
–– More than 24 hours troponins are recommended

Summary of Cardiac Markers:


Marker Detection Peak Disappearance
Myoglobin 1–4 hours 6–7 hours 24 hours
CK–MB 3–12 hours 12–18 hours 2–3 days
Troponin T (cTnT) 4–12 hours 12–48 hours 5–15 days
Troponin I (cTnI) 4–12 hours 12–24 hours 5–7 days
• Myoglobin first comes and first goes (rises first, disappears first)
• LDH is called “lazy enzyme” because it comes late, and goes late (rises after 24 hours and remain till 14 days)
• Troponin-I has the greatest sensitivity and specificity
• **The investigation of choice within first hour of MI is “Creatinine Kinase–MB”
• “CKMB” is useful for confirmation of reinfarction as this enzyme disappears after 72 hours of MI
• So, if a patient comes after 72 hours of initial infarction, and you are suspecting a reinfarction, go for “CKMB”
• Myoglobin disappears after 24–48 hours
• CKMB disappears after 72 hours
• Trop I disappears after 7–10 days
• LDH disappears after 14 days

CONGESTIVE HEART FAILURE


A chronic condition which affects the pumping power of heart muscles. Decrease in cardiac output, the amount of blood that
the heart pumps, is not adequate to circulate the blood returning to the heart from the body and lungs, causing fluid to leak
from capillary blood vessels. This leads to the symptoms that may include shortness of breath, weakness, and swelling.
NYHA Functional Classification for Congestive Heart Failure
The New York Heart Association (NYHA) Functional Classification provides a simple way of classifying heart disease, useful
for preoperative assessment. It places patients in one of the four categories, based on how much they are limited during
physical activity:
Class I: Patients with no limitation of activities; they suffer no symptoms from ordinary activities.
Class II: Patients with slight, mild limitation of activity; they are comfortable with rest or with mild exertion.
Class III: Patients with marked limitation of activity; they are comfortable only at rest.
Class IV: Patients who should be at complete rest, confined to bed or chair; any physical activity brings on discomfort and
symptoms occur at rest.

https://t.me/DentalBooksWorld
Chapter 16 • General Medicine 1361

Difference Between Left and Right Heart Failure


Left heart failure Right heart failure
Left ventricle cannot pump blood to the body Right ventricle cannot adequately pump blood to the lungs

Fluid backs up and leaks into the lungs Blood fluid back up in the veins that deliver to the heart

Causing shortness of breath Can cause fluid to leak into tissues and organs

Symptoms of Left Heart Failure


• The hallmark and most common symptom of left heart failure is shortness of breath and may occur:
–– while at rest
–– with activity or exertion
–– while lying flat (orthopnea)
–– while awakening the person from sleep (paroxysmal nocturnal dyspnea)
• Death occurs within the first hour or immediately → ventricular fibrillation or asystole
• Death occurs after (infarction) surviving first few hours → cardiac failure

CARDIAC MURMURS
They are abnormal sounds heard in between heartbeats.

I. MURMURS HEARD IN VARIOUS CARDIAC DISORDERS


ASD:
• Systolic ejection murmur
• Mid-diastolic murmur
• Holosystolic murmur
VSD:
• Holosystolic murmur
• Mid-diastolic apical murmur
PS:
• Ejection systolic murmur
AS:
• Ejection systolic murmur
AR:
• Austin flint murmur
• Early diastolic murmur
• Systolic murmur
MS:
• S1 → accentuated
• S2 → normally split S2
• Mid-diastolic murmur with pan systolic accentuation

https://t.me/DentalBooksWorld
1362 Triumph's Complete Review of Dentistry

• Heard best at apex in left lateral recumbent position


• Increases in expiration and exercise
MR:
• Pan systolic murmur
TR:
• Pan systolic murmur
• Early systolic murmur
TS:
• Mid-diastolic murmur
AV block:
• Rytand’s murmur

II. SYSTOLIC AND DIASTOLIC MURMURS


Systolic murmurs:
• MR
• TR
• ASD/VSD/TOF
• AS
• PS
Diastolic murmurs:
• MS
• AR
• TS
• ASD/VSD

CONGENITAL HEART DISEASE


These are anomalies that present at birth which occur due to developmental abnormalities.
Pathogenesis of Congenital Heart Disease: Types and typical locations of ASDs. In this view, the free walls of the RA and RV
have been removed, looking leftward toward the septal surface. MPA indicates main pulmonary artery; 1, sinus venosus ASD
at the junction of the superior vena cava (SVC) and RA; 2, ostium secundum ASD; and 3, ostium primum/AV canal type ASD.

SVC

Sinus venosus

Osum
Atrium
secundum
Sinus
venosus Osum primum
AV septum
Coronary
sinus Ventricle
IVC

https://t.me/DentalBooksWorld
Chapter 16 • General Medicine 1363

Types and typical locations of VSDs. In this view, the free walls of the RA and RV have been removed, looking leftward toward
the septal surface. SVC indicates superior vena cava; 1, perimembranous VSD; 2, muscular VSD; 3, inlet/AV canal type VSD;
and 4, subpulmonary VSD.

Aorta

Pulmonary artery
SVC
Supracristal
Membranous
ventricular Cristae
supraventricularis
septum
Infundibular

Right
atrium Infracristal
Trabecular

Inlet

IVC

Classification of Congenital Heart Disease


Classification of Congenital Heart Disease

Acyanotic Cyanotic

Increased pulmonary Obstruction to blood Decreased pulmonary Mixed blood


blood flow flow from ventricles blood flow flow

Atrial septal defect Coarctation of aorta Tetralogy of Fallot Transposition of


great arteries
Ventricular septal Aortic stenosis Tricuspid atresia
defect Truncus arteriosus
Pulmonic stenosis
Patent ductus Hypoplastic left
arteriosus heart syndrome

Atrioventricular canal

TETRALOGY OF FALLOT
• Clubbing/Cyanosis/Normal atrial pulse/Normal JVP/Normal first heart sound/S2
• Single/Flow murmur → ejection systolic murmur located on third left intercostal space/right to left shunt
• Tetralogy of Fallot – VSD + RVH + over riding of aorta + pulmonary stenosis
• Trilogy of Fallot – ASD + RVH + pulmonary stenosis
• Pentalogy of Fallot – VSD + RVH + over riding of aorta + pulmonary stenosis + ASD (or) patent foramen ovale

https://t.me/DentalBooksWorld
1364 Triumph's Complete Review of Dentistry

Shunt Anastomosis
Blalock–Taussig Subclavian artery to pulmonary artery
Pott’s Descending aorta to pulmonary artery
Waterson Ascending aorta to pulmonary artery

Pulse:
• Normalcy → regular intervals
• Rate between 60 and 100 per minute
Normally has two waves:
• Small anacrotic wave on upstroke → not felt
• Big tidal or percussion wave → felt by palpating finger
1. Pulsus alternans
• Characterized by a strong and weak beat occurring alternately.
Causes:
• Left ventricular failure
• Toxic myocarditis
• Paroxysmal tachycardia
2. Pulsus bisferiens
• Rapid rising, twice beating pulse
• Both the waves are felt during systole
• Percussion wave is felt first followed by small wave
Causes:
• Aortic regurgitation
• Hypertrophic cardiomyopathy
3. Pulsus tardus
• Seen in aortic stenosis
4. Pulsus paradoxus
• Normally systolic blood pressure falls by 3–10 mm during inspiration
• Here systolic blood pressure falls more than 10 mmHg
Causes:
• Lung conditions
• Asthma
• Superior vena cava obstruction
• Emphysema
• Airway obstruction
• Cardiac conditions
• Pericardial effusion
• Constrictive pericarditis
• Congestive cardiac failure

CYANOSIS
• Bluish discoloration of nails due to increased amount of reduced hemoglobin in capillary blood
• Also occurs due to increased amount of sulfhemoglobin (0.5 mg%) and methemoglobin (1.5 mg%)

https://t.me/DentalBooksWorld
Chapter 16 • General Medicine 1365

ELECTROCARDIOGRAM (ECG)
It is the process of recording the electrical activity of the heart by using electrodes placed on chest.
ECG Components

• A deflections from the base line represent


Waves cardiac events
• P - QRS - T waves

• A specific portion of the complex


Segment • PR − ST

• Distance measured as time between two


Interval cardiac events
• QT-PR-TP

Waves in ECG
P wave
Originates from the Sinoatrial Node, SA node represents atrial depolarization.
Normal P wave:
1. Height is <2.5 mm (2.5 small squares)
2. Width is <0.08
Significance of normal P wave
1. Impulse originating in SA node
2. Normal atrial conduction and a normal atrium
Abnormality of P waves
1. Tall P wave: >2.5 mm – seen in Right Atrial Enlargement. “P pulmonale” tall and tented P wave as seen in Right Atrial
enlargement. E.g., in Cor pulmonale
2. Wide P: >0.08 seconds – Left Atrial Enlargement. “P Mitrale” – broad and bifid P wave as seen in Left Atrial enlargement.
E.g., in Mitral stenosis
3. Inverted: AV Junctional Rhythm, Normal in AVR, Arm lead reversal, Coronary sinus rhythm, Dextrocardia, Left arterial
rhythm
4. Not followed by QRS: Mobitz type I and II AV block, Third degree AV block
5. Occurring on T wave: Atrial ectopic, AV reentry tachycardia, AV nodal reentry tachycardia, AV junction
6. Rhythm, Ventricular ectopic with retrograde conduction
7. Absent: Atrial fibrillation, Atrial flutter, Hyperkalemia, Mild AV junction rhythm, Sinus arrest or sinoatrial block
8. Abnormal shape: Atrial ectopic, Multifocal atrial tachycardia, Wandering atrial pacemaker
Q wave
Q wave is normally seen in lead V5, V6.
It is produced due to septal depolarization.
Height >25% of R wave, Width <0.04 (1 small squares).
Pathological Q
1. If seen in lead II, V1, V2 or if >5 mm in V5, V6. Pathological Q as seen in old MI.
QRS Complex
Represents depolarization of ventricular muscles and is most prominent wave in ECG.
R wave has a gradual normal increase in height through lead V1 to V6.
Width <0.12 (3 small squares)
Abnormality of QRS complex
1. Large QRS: Calibration set to 20 mV, Dextrocardia, LVH, RVH, MI, WPW syndrome
2. Small QRS: Calibration set to 5 mV, Dextrocardia, Emphysema, Obesity, Pericardial effusion

https://t.me/DentalBooksWorld
1366 Triumph's Complete Review of Dentistry

3. Absent: Ventricular stand still, Asystole during cardiac arrest


4. Wide: Calibration set to 50 mm/second, Brugada syndrome, Hyperkalemia, LBB, RBB, WPW syndrome
5. Varying size: Pericardial effusion
6. Poor progression R wave: Left Ventricular Hypertrophy, Anteroseptal MI
 J point
The J point is the junction between the termination of the QRS complex and the beginning of the ST segment. Abnormal J
(slurring elevation) is seen in Hypothermia – “Osborn Wave.”
T wave
Represents end of repolarization of the ventricles
Normally it is less than 2/3 height of R wave
Height Abnormality of T waves
1. Tall: Hyperkalemia – Tented (concave outside) T wave, MI (convex outside)
2. Small: Hypokalemia, Hypothyroidism, Pericardial effusion
3. Inverted: Hyperthyroidism, Hyperventilation, Left Bundle Branch Block, Right Bundle Branch Block, Mitral valve
prolapsed, Myocardial Infarction, Subarachnoid hemorrhage.
Intervals and Segments of ECG
PR Interval 
Represents conduction of impulse from atrium to ventricles. Height 0.08 seconds and 1 mm elevation in limb lead, >0.08
seconds and 2 mm elevation in chest lead
1. Elevated: Acute MI, Pericarditis, Left Bundle Branch Block, Left Ventricular aneurysm, Prinzmetal’s angina Brugada
syndrome
2. Depressed: Myocardial ischemia, MI (NSTEMI), Right Ventricular Hypertrophy, Left Ventricular Hypertrophy, Drugs
(Antiarrhythmic)
3. Flat, Downsloping, Depressed: MI
ST Segment
Isoelectric segment in ECG originates with beginning of ventricular repolarization.
Normally 0.08 seconds in duration, slightly upward concavity
1. ST Elevation:>0.08 seconds and 1 mm elevation in limb lead, >0.08 seconds and 2 mm elevation in chest lead
2. Elevated: Acute MI, Pericarditis, LBBB, LV aneurysm, Prinzmetal’s angina Brugada syndrome
3. Depressed: Myocardial ischemia, MI (NSTEMI), RVH, LVH, Drugs (Antiarrhythmic)
4. Flat, Downsloping, Depressed: MI
QT Interval
Normal:0.42 seconds, Corrected: (QTc) = QT/√RR
1. Long: Cardiomyopathy, Drugs (antiarrhythmic), Hypocalcaemia, Hypothyroidism, Acute MI, Myocarditis
2. Short: Hypercalcemia, Drugs (Digoxin)

P‒R S‒T
T
P segment segment
U
J

P T
Q
P‒R S‒T
interval S interval
QRS
Q‒T
interval

https://t.me/DentalBooksWorld
Chapter 16 • General Medicine 1367

ECG CHANGES IN VARIOUS CONDITIONS


ST Elevation is seen in
• MI
• Coronary artery spasm
• Ventricular aneurysm
• Prinzmetal’s angina
QT Prolongation is seen in
• Hypothermia
• Hypocalcemia
• Romano–Ward syndrome
• Hypokalemia
• Hypomagnesemia
• Erythromycin
• MVP
• MI
• Rheumatic carditis
QT Interval is shortened in
• Hypercalcemia
• Hyperthermia
• Digitalis toxicity
• Vagal stimulation
ECG Changes In Hypokalemia
• U Wave
• Prolonged Q-T interval
• **Wide QRS Complex
• ST segment depression
• T-Wave flattening or inversion
• Increased PR interval with ST depression
ECG Changes In Hyperkalemia
• Prolonged PR interval
• Early change seen is increased T wave amplitude (Peaked T waves)
• Sin wave pattern
• Loss of P waves
• Prolonged (wide) QRS interval
• Ventricular asystole
• AV conduction delay
• Ventricular fibrillation or asystole
• QT interval decreased

HEART SOUNDS
The first heart sound (S1) represents closure of the atrioventricular (mitral and tricuspid) valves as the ventricular pressure
exceeds atrial pressure at the beginning of systole.
S1 is normally a single sound because mitral and tricuspid valve closure occurs almost simultaneously.
The second heart sound (S2) represents closure of the aortic and pulmonary valves. S2 is normally split.
The more muscular, and less compliant left ventricle (LV) empties earlier than the right ventricle, so the aortic component
(A2) precedes the pulmonary component (P2) by a discernible interval.
https://t.me/DentalBooksWorld
1368 Triumph's Complete Review of Dentistry

Venous return to the right ventricle (RV) increases during inspiration and P2 is even more delayed, so it is normal for the split
of the second heart sound to widen during inspiration and narrow during expiration.
Abnormally wide splitting of S2 may be found in conditions associated with:
1. RV volume overload, such as atrial septal defect (ASD). In the presence of an ASD, the split may be “fixed” with no change
during inspiration and expiration.
2. RV outflow obstruction, such as pulmonary stenosis.
3. Delayed RV depolarization such as complete right bundle branch block.
S2 is single if one of the semilunar valves is missing, as in pulmonary and aortic atresia or truncus arteriosus, or if both valves
close simultaneously, as in pulmonary hypertension with equal ventricular pressures.
The third heart sound (S3) represents a transition from rapid to slow ventricular filling in early diastole. S3 may be heard in
normal children.
The fourth heart sound (S4) is an abnormal late diastolic sound caused by forcible atrial contraction in the presence of
decreased ventricular compliance.

• Cardiopulmonary Resuscitation (CPR)


• CPR Sequence – CAB (CIRCULATION – AIRWAY – BREATHING)
• Compression rate – 100/minute
• Compression depth – 2 inches (5 cm)
• Compression to Ventilation Ratio
–– 3:2 (one or two rescuers – Adults)
–– 30:2 (single rescuer – Children)

IMPORTANT CRITERIA
Framingham Criteria for Congestive Heart Failure
Diagnosis of CHF requires the simultaneous presence of at least two major criteria or one major criterion in conjunction with
two minor criteria.
Major Criteria:
• Paroxysmal nocturnal dyspnea
• Neck vein distention
• Rales
• Radiographic cardiomegaly (increasing heart size on chest radiography)
• Acute pulmonary edema
• S3 gallop
• Increased central venous pressure (>16 cm H2O at right atrium)
• Hepatojugular reflux
• Weight loss > 4.5 kg in 5 days in response to treatment
Minor Criteria:
• Bilateral ankle edema
• Nocturnal cough
• Dyspnea on ordinary exertion
• Hepatomegaly
• Pleural effusion
• Decrease in vital capacity by one third from maximum recorded
• Tachycardia (heart rate >120 beats/minute)

https://t.me/DentalBooksWorld
Chapter 16 • General Medicine 1369

**Duke Criteria for Infective Endocarditis (IE)


Major Criteria:
A. Positive blood culture for Infective Endocarditis
1. Typical microorganism consistent with IE from two separate blood cultures, as noted below:
• Viridans streptococci, Streptococcus bovis, or HACEK* group, or
• Community-acquired Staphylococcus aureus or enterococci, in the absence of a primary focus
2. Microorganisms consistent with IE from persistently positive blood cultures defined as:
• Two positive cultures of blood samples drawn >12 hours apart, or
• All of three or a majority of four separate cultures of blood (with first and last sample drawn 1 hour apart)
B. Evidence of endocardial involvement
1. Positive echocardiogram for IE defined as:
Oscillating intracardiac mass on valve or supporting structures, in the path of regurgitant jets, or on implanted material in the
absence of an alternative anatomic explanation, or abscess, or new partial dehiscence of prosthetic valve
2. New valvular regurgitation (worsening or changing of preexisting murmur not sufficient)
Minor Criteria:
• Predisposition: predisposing heart condition or intravenous drug use
• Fever: temperature >38.0°C (100.4°F)
• Vascular phenomena: major arterial emboli, septic pulmonary infarcts, mycotic aneurysm, intracranial hemorrhage,
conjunctival hemorrhages, and Janeway lesions
• Immunologic phenomena: glomerulonephritis, Osler’s nodes, Roth spots, and rheumatoid factor
• Microbiological evidence: positive blood culture but does not meet a major criterion as noted above or serological evidence
of active infection with organism consistent with IE
• Echocardiographic findings: consistent with IE but do not meet a major criterion as noted above
Clinical Criteria for Infective Endocarditis Requires:
• Two major criteria, or
• One major and three minor criteria, or
• Five minor criteria.

Revised Jones Criteria for Acute Rheumatic Fever


A firm diagnosis requires that two major or one major and two minor criteria are satisfied, in addition to evidence of recent
streptococcal infection.

Major Criteria:
1. Carditis: All layers of cardiac tissue are affected (pericardium, epicardium, myocardium, endocardium).
• The patient may have a new or changing murmur, with mitral regurgitation being the most common followed by aortic
insufficiency.
2. Polyarthritis: Migrating arthritis that typically affects the knees, ankles, elbows, and wrists.
• The joints are very painful and symptoms are very responsive to anti-inflammatory medicines.
3. Chorea: Also known as Sydenham’s chorea, or “St. Vitus’s dance.”
• There are abrupt, purposeless movements.
• This may be the only manifestation of ARF and its presence is diagnostic.
• May also include emotional disturbances and inappropriate behavior.
4. Erythema marginatum: A nonpruritic rash that commonly affects the trunk and proximal extremities, but spares the face.
• The rash typically migrates from central areas to periphery, and has well-defined borders.
5. Subcutaneous nodules: Usually located over bones or tendons; these nodules are painless and firm.

https://t.me/DentalBooksWorld
1370 Triumph's Complete Review of Dentistry

Minor Criteria:
1. Fever
2. Arthralgia
3. Previous rheumatic fever or rheumatic heart disease
4. Acute phase reactants: Leukocytosis, elevated erythrocyte sedimentation rate (ESR) and C-reactive protein (CRP)
5. Prolonged P-R interval on electrocardiogram (ECG)

Diagnostic Criteria for Thromboangiitis Obliterans (Buerger’s Disease)


Several different criteria have been proposed for the diagnosis of thromboangiitis obliterans.

Diagnostic Criteria of Shionoya (he who proposed the criteria):


• Smoking history
• Onset before the age of 50 years
• Infrapopliteal arterial occlusions
• Either arm involvement or phlebitis migrans
• Absence of atherosclerotic risk factors other than smoking
Diagnostic Criteria of Olin (he who proposed it):
• Age younger than 45 years
• Current or recent history of tobacco use
• Presence of distal extremity ischemia indicated by claudication, pain at rest, ischemic ulcers or gangrenes, and documented
by noninvasive vascular testing
• Exclusion of autoimmune diseases, hypercoagulable states, and diabetes mellitus
• Exclusion of a proximal source of embolization by echocardiography and arteriography
• Consistent arteriographic findings in the clinically involved and noninvolved limb

INFECTIVE ENDOCARDITIS
• Due to microbial infection of heart valve lining of cardiac chamber or blood vessel
• Mostly affects areas of endocardial damage due to high pressure jet of blood such as VSD, MR, and AR
• Greater risk in high pressure regions like VSD, MR, and AR
• Risk of endocarditis is less in low pressure lesions like ASD
Clinical Features:
• ROTH spots in fundi
• Osler’s nodes → painful tender swellings at the finger tips
• Positive blood culture
• Cerebral emboli
• Murmurs, arrhythmias, and cardiac failure
• Cerebral emboli
• Splenomegaly
• Hematuria
• Petechial hemorrhage of skin and mucous membrane
• Splinter hemorrhage and clubbing of nails

https://t.me/DentalBooksWorld
Chapter 16 • General Medicine 1371

Type of endocarditis Causative organisms


Acute endocarditis Staphylococcus aureus
Postoperative endocarditis Streptococcus pneumoniae
Subacute endocarditis Staphylococcus epidermidis
Streptococcus mitis
Streptococcus sanguinis
Alpha hemolytic streptococci
Staphylococcus aureus – Common cause of acute endocarditis.
Streptococcus sanguinis – Causative agent for bacterial endocarditis in persons with preexisting cardiac lesions.

PROPHYLAXIS FOR ENDOCARDITIS DURING DENTAL PROCEDURE


Procedure Antibiotic regimen
Dental or upper respiratory tract procedure under local Amoxicillin 3 g orally 1 hour before procedure
anesthesia
If allergic to penicillin Clindamycin 600 mg orally 1 hour before procedure
Special risk patients, i.e., prosthetic valve or previous Amoxicillin 1 g IV plus gentamicin 120 mg IV at induction plus
endocarditis Amoxicillin 0.5 g orally 6 hours later
If allergic to penicillin Vancomycin 1 g IV infusion plus gentamicin 120 mg IV
induction

RESPIRATORY SYSTEM
CLASSIFICATION
I. Infectious Diseases:
1. Upper respiratory tract infections
• Common cold
• Sinusitis
2. Lower respiratory tract infections
• Tuberculosis
• Pneumonia
• Obstructive lung disease
• Asthma
• Chronic obstructive lung disease
–– Chronic bronchitis
–– Emphysema

II. Interstitial Lung Disease


1. Occupational
• Asbestosis
• Silicosis
• Pneumoconiosis
2. Connective tissue disorders
• Scleroderma
• Systemic lupus erythematosus

https://t.me/DentalBooksWorld
1372 Triumph's Complete Review of Dentistry

3. Idiopathic
• Idiopathic interstitial pneumonia

III. Granulomatous Diseases


• E.g., Sarcoidosis

Respiratory Failure
Respiratory failure is a syndrome in which the respiratory system fails in one or both of its gas exchange functions: oxygenation
and carbon dioxide elimination
Two types – type 1 (failure of oxygenation) and type 2 (defect in ventilation)
COMPONENTS
TYPE 1 TYPE 2
PaO2 D D
PaCO2 D (or) N I
PA-a O2 I N

ALKALOSIS VERSUS ACIDOSIS


Directions of pH and HCO3
ROME Concept:
• Respiratory = Opposite
–– pH is high, PCO2 is low (Alkalosis)
–– pH is low, PCO2 is high (Acidosis)
• Metabolic = Equivalent
–– pH is high, HCO3 is high (Alkalosis)
–– pH is low, HCO3 is low (Acidosis)
Acetone breath → Diabetes mellitus
Musty breath → Hepatic coma
Uremic breath → Renal disease
Alcoholic breath → Alcoholics

OBSTRUCTIVE LUNG DISEASE


People with obstructive lung disease have shortness of breath due to difficulty exhaling all the air from the lungs. Because of
damage to the lungs or narrowing of the airways inside the lungs, exhaled air comes out more slowly than normal. At the end
of a full exhalation, an abnormally high amount of air may still linger in the lungs.
The most common causes of obstructive lung disease are:
• COPD, which includes emphysema and chronic bronchitis
• ASTHMA
• Bronchiectasis
• Cystic fibrosis

RESTRICTIVE LUNG DISEASE


People with restrictive lung disease cannot fully fill their lungs with air. Their lungs are restricted from fully expanding.
Restrictive lung disease most often results from a condition causing stiffness in the lungs themselves. In other cases, stiffness
of the chest wall, weak muscles, or damaged nerves may cause the restriction in lung expansion.
Some conditions causing restrictive lung disease are:

https://t.me/DentalBooksWorld
Chapter 16 • General Medicine 1373

Interstitial lung disease, such as idiopathic lung fibrosis


• Sarcoidosis
• Obesity
• Scoliosis
• Neuromuscular disease, such as muscular dystrophy

DIFFERENCES BETWEEN OBSTRUCTIVE AND RESTRICTIVE LUNG DISEASE


Components Obstructive disease Restrictive disease
FEV1 D N
FVC D D
FEV1/FEC (FEV1%) D N OR I
TLC and RV I D
DLCO N (decreased in emphysema) D
VC N OR D D

CHRONIC OBSTRUCTIVE PULMONARY DISEASE (COPD) – Barrel Shaped Chest


Definition and Classification
• COPD is a progressive, obstructive airway disease that is not fully reversible.
• It results from disease of the airways and parenchyma in the form of chronic bronchitis and emphysema.
• Three million people in the UK are diagnosed with COPD and a suspected two million more may be undiagnosed. COPD
is the fifth most common cause of death in the UK.
Aetiology
1. Smoking – Smoking is by far the most important aetiological factor. 90% of cases of COPD are associated with smoking.
However, only 10% of smokers will develop it, indicating the presence of a co-factor such as a genetic predisposition.
Emphysema is characteristically centriacinar. Smoking increases the risk of these patients developing emphysema.
Emphysema is characteristically panlobular with a lower zone predominance.
2. Alpha-1 antitrypsin deficiency
Alpha-1 antitrypsin deficiency is an autosomal codominant condition characterized by a deficiency in alpha-1 antitrypsin. It
affects around 1 in 5,000 in the UK.
Alpha-1 antitrypsin is a protease inhibitor that is synthesized by the liver. It acts in the lung parenchyma to oppose the action
of elastase. Elastase is a protease that causes the breakdown of elastin, a protein important to the structural integrity of the
alveoli. This causes emphysema. 
Pathophysiology – COPD is a disease of both the airways and the alveoli.

Airways
Chronic bronchitis refers to inflammation of the bronchi, defined as a chronic productive cough for 3 (or more) months in
2 consecutive years where other causes are excluded.
Chronic bronchitis leads to:
• Goblet cell hyperplasia
• Mucus hypersecretion
• Chronic inflammation and fibrosis
• Narrowing of small airways

Alveoli
• Emphysema is the permanent enlargement of airspaces distal to the terminal bronchiole when interstitial pneumonias
are excluded.

https://t.me/DentalBooksWorld
1374 Triumph's Complete Review of Dentistry

• Inflammatory processes lead to the production of proteases by inflammatory cells such as macrophages. The protease
elastase causes the destruction of elastin, a protein important to the structural integrity of the alveoli.
• Loss of elastin has two effects:
–– Collapse: the alveoli are prone to collapse.
–– Dilation and bullae formation: alveoli dilate and may eventually join with neighboring alveoli forming bullae. 

CHRONIC BRONCHITIS EMPHYSEMA


Blue bloaters – predominantly bronchitis Pink puffers – predominantly
Pathological process of permanent destructive enlargement Condition in a patient coughing up sputum for at least
of the spaces distal to terminal bronchioles 3 consecutive months for more than 2 successive years

COR PULMONALE
• Cor pulmonale refers to right ventricular impairment secondary to pulmonary disease. In the developed world COPD is
the most common cause.
• Clinical features are those of right-sided heart failure.

Clinical Features
• Chronic productive cough and dyspnea are the hallmarks of COPD.

Symptoms
• Productive cough
• SOB
• Orthopnea
Signs
• Dyspnea
• Pursed lip breathing (prevents alveolar collapse by increasing the positive end expiratory pressure)
• Wheeze
• Coarse crackles
• Loss of cardiac dullness
• Downward displacement of the liver
• Signs of CO2 retention
–– Drowsy
–– Asterixis
–– Confusion
• Signs of cor pulmonale
–– Peripheral edema
–– Left parasternal heave (caused by right ventricular hypertrophy)
–– Raised JVP
–– Hepatomegaly 

Management
Beta-2 agonists, muscarinic antagonists, and steroids offer symptomatic relief. Smoking cessation and oxygen at home offer
decreases in mortality.

https://t.me/DentalBooksWorld
Chapter 16 • General Medicine 1375

Medical management
Two types of bronchodilators are used in COPD: Beta-2 agonists (BA) and muscarinic antagonists (MA). They may be short-
acting (SA) or long-acting (LA) in nature.
For diseases in which these inhalers do not control symptoms, inhaled corticosteroids may be prescribed.

ASTHMA
• Asthma is a chronic disease involving the airways in the lungs. These airways, or bronchial tubes, allow air to come in and
out of the lungs.
• Especially affects terminal bronchioles.

Asthma Symptoms:
• Asthma triad → Samter’s triad
• Hyper sensitivity to aspirin + nasal allergy/polyposis + bronchial asthma
• The most common symptom is wheezing. This is a scratchy or whistling sound when you breathe.
• Other symptoms include:
–– Shortness of breath
–– Chest tightness or pain
–– Chronic coughing
–– Trouble sleeping due to coughing or wheezing
• Histological findings in sputum – Creola bodies and Curschmann’s spirals

Features of Acute Severe Asthma:


• Silent chest
• Bradycardia
• Pulsus paradoxus
• Exhaustion, confusion, and reduced conscious level
Asthma symptoms, also called asthma flare-ups or asthma attacks, are often caused by allergies and exposure to allergens
such as pet dander, dust mites, pollen, or mold. Nonallergic triggers include smoke, pollution, or cold air or changes in weather.
Asthma symptoms may be worse during exercise, when you have a cold or during times of high stress.
Children with asthma may show the same symptoms as adults with asthma: coughing, wheezing, and shortness of breath. In
some children chronic cough may be the only symptom.

TREATMENT:
• Controller medications are taken daily and include inhaled corticosteroids: ciclesonide, flunisolide, beclomethasone.
• Leukotriene antagonists are oral medications that include montelukast, zafirlukast, and zileuton.
• Quick-relief or rescue medications are used to quickly relax and open the airways and relieve symptoms during an asthma
flare-up, or are taken before exercising if prescribed.
• These include short-acting beta agonists.
• Oral and intravenous corticosteroids may be required for acute asthma flare-ups or for severe symptoms. Examples
include prednisone and methylprednisolone.

CYSTIC FIBROSIS
• Autosomal recessive disease
• Defect in chloride channel

https://t.me/DentalBooksWorld
1376 Triumph's Complete Review of Dentistry

• Increased sodium content in sweat


• Increased viscosity of secretions in the lung and other organs
• It causes ciliary dysfunction and chronic bronchial infections
• Generalized calculus formation
• Malabsorption

PNEUMONIA
It is an infection of the lungs with a range of possible causes. It can be a serious and life-threatening disease.
PNEUMOCOCCAL PNEUMONIA
• Lobar pneumonia

Clinical Features:
• Flushing
• Tachycardia
• Tachypnea
• Males affected more
• Sputum is rusty
Austrian triad: Pneumococcus pneumonia + infective endocarditis + meningitis
STAPHYLOCOCCAL PNEUMONIA
Bronchopneumonia or lobular pneumonia
Clinical Features:
• Shaggy thin-walled cavities seen in chest X-ray
• Pneumatoceles are characteristic empyema in infants
KLEBSIELLA PNEUMONIA
Causes community-acquired lobar pneumonia aka Friedlander’s pneumonia
Clinical Features:
• Commonly affects alcoholic
• Greater than 40 year
• Diabetic
• COPD patients
• Current jelly sputum is seen.
• Upper lobe involvement is seen
Pneumocystis carinii
Seen in HIV positive/immunocompromised or low CMI patients or Plasma cell or interstitial pneumonia
Clinical Features:
• Fever
• Dyspnea
• Dry cough
• Retrosternal pain worsening on inspiration
• Bilateral infiltrates are seen
• X-ray finding is normal film
• Mononuclear infiltration

https://t.me/DentalBooksWorld
Chapter 16 • General Medicine 1377

HEPATOLOGY
HEPATITIS
Hepatitis A Hepatitis B Hepatitis C Hepatitis D Hepatitis E
What is hepatitis virus disease?
HAV mainly affects HBV is the second most HCV is the most HDV occurs HEV is the most
children. It does not has common cause of chronic common cause of as coinfectious common hepatitis
a fatal course. It is the hepatitis. It causes liver cell chronic hepatitis. It disease along with all over the world in
most common hepatitis damage, leading to cirrhosis causes cirrhosis and HBV and it causes adults and it has no
in children all over the and cancer. cancer. superinfection chronicity.
world. It does not lead Most common cause of of those patients Most common
to chronic disease. fulminant hepatitis. who were infected cause of fulminant
Most common hepatitis with HBV. hepatitis in
all over India is Highest incidence pregnancy.
Hepatitis A. of fulminant
Chronic hepatitis and hepatic failure.
carriers are seen in all
except hepatitis A and E.
Single-stranded RNA Double-stranded DNA RNA RNA RNA
42 nm in size
What is its incubation period?
15–50 days. 3–6 months. Approximately 2–25 weeks. 2–8 weeks. 2–9 weeks.
Approximately 30 days. 120 days. Approximately 7–9 Approximately 40
weeks. days.
How is it spread?
Transmitted by fecal/ Mainly via Heterosexual. Contact with infected Contact with Transmitted through
oral route, through Contact with infected blood, contaminated infected blood, fecal/oral route.
close person-to-person blood, seminal fluid, vaginal IV needles, razors, contaminated Outbreaks associated
contact or ingestion of secretions, contaminated and tattoo/body needles. Sexual with contaminated
contaminated food and needles, including tattoo/ piercing tools. contact with water supply in other
water. body piercing tools. Infected Infected mother to HDV-infected countries. Not spread
mother to newborn. newborn. Not easily person. via blood transfusion
spread through sex and sexual contact.
and via breast milk.
Symptoms
May have none. Adults May have none. Some persons Even fewer acute cases Same as HBV. Same as HBV.
may have light stools, have mild flu-like symptoms, seen than any other
dark urine, fatigue, dark urine, light stools, hepatitis. Otherwise
fever, and jaundice. jaundice, fatigue, and fever. same as HBV
Treatment of Chronic Disease
No treatment. Antivirals with varying Interferon and Interferon with No treatment.
success. combination therapies varying success.
with varying success.
Vaccine
Two doses of vaccine Three doses may be given to None. HBV vaccine None.
to anyone over the age persons of any age. prevents HDV
of 2. infection.

https://t.me/DentalBooksWorld
1378 Triumph's Complete Review of Dentistry

Who is at risk?
Household or sexual Infant born to infected Anyone who had a IV drug users, Travelers to
contact with an infected mother, having sex with blood transfusion homosexual men, developing countries,
person or living in infected person or multiple before 1992; health and those having especially pregnant
an area with HAV partners, IV drug users, care workers, IV drug sex with an HDV- women.
outbreak. Travelers to emergency responders, users, hemodialysis infected person.
developing countries, health care workers, patients, infants born
homosexual men, and homosexual men, and to infected mother, and
IV drug users. hemodialysis patients. multiple sex partners.
Prevention other than vaccine
Immune globulin within Immune globulin within Safe sex. Clean up Hepatitis B Avoid drinking or
2 weeks of exposure. 2 weeks of exposure. spilled blood with vaccine to prevent using potentially
Vaccination. Washing Vaccination provides bleach. Wear gloves HBV infection. contaminated water.
hands with soap and protection for 18 years. when touching blood. Safe sex.
water after going to the Safe sex. Clean up infected Do not share razors or
toilet. Use household blood with bleach and wear toothbrushes.
bleach to clean surfaces protective gloves. Do not
contaminated with share razors, toothbrushes,
feces, such as changing and needles.
tables. Safe sex.
Prognosis – Excellent Poor Poor Good Poor

POST-EXPOSURE PROPHYLAXIS

If mother is positive with HbsAg in 3rd trimester → 90% chances for transmission

To prevent this do elective C-Section (Cesarean section), this is done at 39th week (ideally)

Give passive immunoglobulin within 24 hours after birth

Source HIV HBV HCV


Vertical transmission 30% 90% 5%
Needle prick 0.3% 30% 3%

SIGNIFICANCE OF HEPATITIS ANTIGENS


HbsAg First to appear in blood
HbcAg Never appears in blood
HbeAg Denotes replication of virus or infectivity
HbxAg Hepatocellular cancer (HBV > HCV)
HBV DNA If viral load is >2×10 to the power of 4 viral copies IU/ml (or)
10 to the power of 5 viral copies of DNA/ml – indicative of
high viral load

https://t.me/DentalBooksWorld
Chapter 16 • General Medicine 1379

Chronic HBV Infection with Persistent HBeAg


HBV DNA
HBsAg
HBeAg
Total anti-HBc
IgM anti-HBc

Titer

0 4 8 12 16 20 24 28 32 36
Weeks After Exposure Years

INVESTIGATION HBsAg Anti HBs Anti HBc HBeAg Anti HBc


Immunization with HBsAg − + − _ _
Recovery from hepatitis B − + IgG _ +/−
Acute hepatitis infectivity + − IgM + _
Chronic hepatitis (infectivity) + − IgG + _
Chronic hepatitis (low infectivity) + − IgG _ +

JAUNDICE
Jaundice is a yellowish discoloration of the skin and mucous membranes caused by hyperbilirubinemia. Jaundice becomes
visible when the bilirubin level is about 2–3 mg/dl (34–51 μmol/ml)
Types of Jaundice:
There are three types of jaundice, depending on what is affecting the movement of bilirubin out of the body.
1. Prehepatic jaundice
Prehepatic jaundice occurs when a condition or infection speeds up the breakdown of red blood cells. This causes bilirubin
levels in the blood to increase, triggering jaundice.
Causes of prehepatic jaundice include:
• Malaria – a blood-borne infection spread by mosquitoes
• Sickle cell anemia – an inherited blood disorder where the red blood cells develop abnormally; it is most common among
black Caribbean, black African, and black British people
• Thalassemia – similar to sickle cell; it is most common in the people of Mediterranean, Middle Eastern, and, in particular,
South Asian descent
• Crigler–Najjar syndrome – a genetic syndrome where an enzyme needed to help move bilirubin out of the blood and into
the liver is missing
• Hereditary spherocytosis – a genetic condition that causes red blood cells to have a much shorter life span than normal
2. Intrahepatic jaundice
Intrahepatic jaundice happens when a problem in the liver – for example, damage due to infection or alcohol, disrupts the
liver’s ability to process bilirubin.
Causes of intrahepatic jaundice include:
• The viral hepatitis group of infections – hepatitis A, B, and C
• Alcohol liver disease where the liver is damaged as a result of intake of too much of alcohol
• Leptospirosis – a bacterial infection that is spread by animals, particularly rats
• Glandular fever – a viral infection caused by the Epstein–Barr virus
• Drug misuse – leading causes are ecstasy and overdoses of paracetamol
• Primary biliary cirrhosis – a rare condition that causes progressive liver damage
• Gilbert’s syndrome– a common genetic syndrome where the liver has problems breaking down bilirubin at a normal rate
• Liver cancer – a rare and usually incurable cancer that develops inside the liver

https://t.me/DentalBooksWorld
1380 Triumph's Complete Review of Dentistry

• Exposure to substances known to be harmful to the liver – such as phenol (used in the manufacture of plastic) or carbon
tetrachloride (widely used in the past in processes such as refrigeration, although now its use is strictly controlled)
• Autoimmune hepatitis – a rare condition where the immune system starts to attack the liver
• Primary sclerosing cholangitis – a rare type of liver disease that causes long-lasting (chronic) inflammation of the liver
• Dubin–Johnson syndrome – a rare genetic syndrome where the liver is unable to move bilirubin out of the liver
3. Posthepatic jaundice
Posthepatic jaundice is triggered when the bile duct system is damaged, inflamed, or obstructed, which results in the gallbladder
being unable to move bile into the digestive system.
Causes of posthepatic jaundice include:
• Gallstones – obstructing the bile duct system
• Pancreatic cancer
• Gall bladder cancer or bile duct cancer
• Pancreatitis – inflammation of the pancreas, which can either be acute or chronic pancreatitis

Causes Causes
- Sickle cell crisis - Trauma
- Blood transfusion - Extensive burns
- Hemolytic drugs - Recent surgery
- Hemolytic anemia - Prolonged immobility & fasting
- Pronlonged total parenteral nutrition
- Diabetes
New Excess hemolysis of RBC / - Infection
Destruction of erythrocyte - Chemical Irritants
- Drugs
- Neoplasms
Hemolytic / Prehepatic

Carcinoma of head of
Unconjugated pancreas Acalculous cholecystitis
hyperbilirubinemia
Edema of pancreatitis
Gilbert Syndrome Jaundice / Icterus Compression

Extrahepatic Sclerosing cholangities


Cannot conjugate bilirubin Conjugated Cholestasis
Hyperbilirubinemia
Increase unconjugated bilirubin hyperbilirubinemia Obstruction
of common bile duct
Hepatocellular / Hepatic Obstructive / post-hepatic
Causes Calculus cholecystitis
Occulusion
- Hepatitis Cholelithiasis
- Cirrhosis Damaged hepatocytes Intrahepatic
Causes
- Hepatic carcinoma
- Cirrhosis
- Hepatitis
Leaks conjugated bilirubin Swelling, fibrosis, or - Cholangitis
Initially increase conjugated bilirubin obstruction of liver canaliculi - Pancreatic carcinoma

Differential Diagnosis of Jaundice


Pathology Serum bilirubin Urine Urine AST, ALT alk.phos., γGT
bilirubin urobilinogen
Total (T) Direct (D) D/T*
Hemolysis – < 0.2 – – –
Liver Disease
0.6 ± 0.1 –
hepatocellular
Obstructive
partial – – – –
0.7 ± 0.1
complete –

Cirrhosis

Neonatal
( ) – < 0.2 – – – –
jaundice

https://t.me/DentalBooksWorld
Chapter 16 • General Medicine 1381

WILSON’S DISEASE
It is a rare autosomal recessive inherited disorder of copper metabolism that is characterized by excessive deposition of copper
in the liver, brain, and other tissues. Wilson disease is often fatal if not recognized and treated when symptomatic.
Gene involved – ATP7B gene
Signs and Symptoms:
• Jaundice
• Muscle pain
• Fatigue
• Hepatobiliary dysfunction
Test Typical finding False "negative" False "positive"
Serum ceruloplasmin Decreased by 50% of Normal levels in patients with Low levels in:
lower normal value marked hepatic inflammation – malabsorption
Overestimation by – aceruloplasminemia
immunologic assay Pregnancy,
– heterozygotes
estrogen therapy
24-hour urinary copper >1.6 µmol/24 h Normal: Increased:
>0.64 µmol/24 h in – incorrect collection – hepatocellular necrosis
children – children without liver disease – cholestasis
– contamination
Serum "free" copper >1.6 µmol/L Normal if ceruloplasmin
overestimated by immunologic
assay
Hepatic copper >4 µmol/g dry weight Due to regional variation Cholestatic syndromes
– in patients with active liver
disease
– in patients with regenerative
nodules
Kayser-Fleischer Present Absent Primary biliary cirrhosis
rings by slit lamp – in up to 50% of patients with
examination hepatic Wilson's disease
– in most asymptomatic
siblings
Treatment:
• The mainstay of therapy for Wilson’s disease is pharmacologic treatment with chelating agents such as D-penicillamine
and trientine.
• Other agents include sodium dimercaptosuccinate, dimercaptosuccinic acid, zinc, and tetrathiomolybdate.
• Zinc salts act as inductors of metallothioneins, which favor a negative copper balance and a reduction of free plasmatic copper.
• The use of surgical decompression or transjugular intrahepatic shunting (TIPS) in the treatment of portal hypertension is
reserved for individuals with recurrent or uncontrolled variceal bleeding that is unresponsive to standard conservative measures.

ENDOCRINOLOGY
GENERAL PRESENTING COMPLAINTS
Alimentary Changes
• Weight loss (thyrotoxicosis, DM) versus gain (Cushing’s, hypothalamic dz)
• Wasting
• Appetite: loss (Addison’s) versus increased (thyrotoxicosis)

https://t.me/DentalBooksWorld
1382 Triumph's Complete Review of Dentistry

• Diarrhea (hyperthyroid, Addison’s)


• Constipation (hypothyroidism, hypercalcemia)
• Polydipsia [excess drinking] (DM, renal dz, hypercalcemia)
Integumental Changes
• Pigmentation (hypopituitarism, Cushing’s, acromegaly, Addison’s)
• Dryness (hypothyroidism, hypoparathyroidism)
• Sweating (hyperthyroidism, pheochromocytoma, acromegaly)
• Overgrowth (acromegaly)
Nervous Changes
• Nervousness, irritability (thyrotoxicosis)
• Fatigue (hypothyroid, DM, Addison’s, acromegaly)
• Headaches (hypoglycemia)
• Seizures (hypoglycemia)
• Visual loss (acromegaly, DM)
Rheumatoid Changes
• Shorter stature
• Gigantism (Marfan’s, Klinefelter’s, growth hormone)
• Hand, skull bony growth (acromegaly)
Urogenital Changes
• Polyuria (DM, DI, polydipsia)
• Menstrual changes (polycystic ovary, pituitary dz)
• Impotence (hyperprolactinemia, hypogonadism, acromegaly)
Past Medical, Surgical History
• Congenital problems
• Goiter, thyroid problems
• Thyroidectomy
• Thyroid surgery, radiation
• Parathyroid, pituitary surgery
Family History
• Condition in a family member (Marfan’s, DM)

MULTIPLE ENDOCRINE NEOPLASIAS


Types Name Glands affected Genes affected
MEN 1 Wermer’s syndrome Parathyroid, pancreatic, and Menin gene Chr 11
pituitary
MEN 2A Sipple syndrome MTC, pheochromocytoma, RET GENE Chr 10
parathyroid, amyloidosis,
Hirschsprung’s disease
MEN 2B MEN Type 3 MTC, pheochromocytoma, RET GENE Chr 10
GIT neuroma, Marfanoid
features
MEN 4 MEN Type 4 Parathyroid and anterior CDKN-1B
pituitary tumors

https://t.me/DentalBooksWorld
Chapter 16 • General Medicine 1383

DISORDERS OF ADRENAL CORTEX

Cushing’s Syndrome
The most common cause of Cushing’s syndrome is iatrogenic administration of steroids.
Endogenous cause – Bilateral adrenal hyperplasia/Pituitary microadenoma
Cushing’ syndrome results from an excessive amount of corticosteroids secreted from the adrenal cortex (outer part) of the gland.
Clinical Features:
• Upper body obesity
• Round, red, full face (Moon face)
• Slow growth rate in children
• Acne or skin infections
• Striae on the skin of the abdomen, thighs, upper arms, and breasts
• Bone pain or tenderness
• Collection of fat between the shoulders – buffalo hump
• Weakening of the bones, which leads to rib and spine fractures
• Weak muscles
• Mental changes, such as depression, anxiety, or changes in behavior
• Fatigue
• Frequent infections
• Headache
• Increased thirst and urination
• High blood pressure
• Diabetes
• Females – Excess hair growth on the face, neck, chest, abdomen, and thighs. Menstrual cycle that becomes irregular or stops
• Males – Low libido, erectile dysfunction

Difference between Cushing’s Syndrome and Cushing’s Disease


Cushing’s disease is caused by a benign pituitary gland tumor causing oversecretion of ACTH which increases adrenal gland
cortisol production.
Cushing’s syndrome refers to the signs and symptoms associated with excess cortisol in the body.

Addison’s Disease (Adrenal Insufficiency)


• Addison’s disease or primary adrenocortical insufficiency is characterized primarily by inability of adrenals to elaborate
sufficient quantities of adrenal cortical hormones.
• Most common cause in India is – Tuberculosis
• Most common cause in developed countries – Autoimmune disease
Symptoms:
• Weight loss and decreased appetite
• Hyperpigmentation
• Low blood pressure, even fainting
• Salt craving
• Hypoglycemia
• Nausea, diarrhea, or vomiting
• Abdominal pain
• Muscle or joint pains
https://t.me/DentalBooksWorld
1384 Triumph's Complete Review of Dentistry

• Irritability
• Depression
• Body hair loss or sexual dysfunction in women
Addisonian Crisis
Sudden appearance of signs and symptoms of Addison’s disease is called Addisonian crisis or Acute adrenal failure

Symptoms:
• Pain in your lower back, abdomen, or legs
• Severe vomiting and diarrhea, leading to dehydration
• Low blood pressure
• Loss of consciousness
• Hyperkalemia and hyponatremia

DIABETES MELLITUS
Two types: Type 1 and Type 2 Diabetes Mellitus
Type 1 Diabetes Mellitus
It is a chronic condition in which the pancreas produce little or no insulin. Insulin is a hormone needed to allow sugar
(glucose) to enter cells to produce energy.
Type 2 Diabetes Mellitus
• Inheritance – Polygenic/Heterogeneous
• C-peptide levels are increased
• Insulin levels are also increased
• No autoantibodies
• No HLA association
• Asymptomatic
• >40 years/Obese
• Insulin levels are actually normal/high – Insulin resistance cause Type 2 DM
Maturity Onset Diabetes of Young (MODY)
• Predominantly young
• Positive family history
• Inheritance – Monogenic
• Autosomal dominant
• Risk of ketoacidosis – Low
• Noninsulin dependent
• Obesity – Uncommon
• Pathophysiology
–– Beta cell dysfunction
–– No insulin resistance
• Fasting C-peptide
–– Low
• No antibodies
• No hypertension
• No hyperlipidemia
• No insulin resistance
• Insulinopenia

https://t.me/DentalBooksWorld
Chapter 16 • General Medicine 1385

Manifestations of Diabetes Ketoacidosis


• Tachycardia
• Hypotension
• Dehydration
• Tachypnea
• Kussmaul respiration
• Cerebral edema
• Metabolic acidosis
Causes of early morning hyperglycemia in diabetes patients using insulin
• Insufficient insulin (most common)
• Dawn phenomenon (common)
–– Early morning hyperglycemia
• Somogyi phenomenon (Least common)
–– Rebound hyperglycemia after an incident of hypoglycemia

DISORDERS OF CALCIUM METABOLISM


Hyperparathyroidism:
• Most common cause – Parathyroid adenoma
• Adenomas are more commonly located in the Inferior parathyroid gland
Primary Secondary Tertiary
Elevated PTH Elevated PTH because of lowered Ectopic PTH
serum calcium
E.g., Single adenoma E.g., chronic renal failure and E.g., SCC
Vitamin D deficiency
CLINICAL FEATURES (Bone, Serum, and X-Ray)
• Recurrent nephrolithiasis
• Peptic ulceration
• Extensive bone resorption
• Mental changes
• Elevated PTH
• Asymptomatic hypercalcemia
• Osteitis fibrosa cystica
• Absence of lamina dura
• Pinhead stippling of skull
• Subperiosteal bone resorption of pharyngeal clefts
• Brown tumor – Osteoporosis/Deformity/Pathological fracture
• Salt and pepper/Pepperpot appearance
Rugger jersey spine: Band like osteosclerosis of superior/inferior margins of the vertical body. Seen in CRF due to secondary
hyperparathyroidism or renal osteodystrophy.

Hyperparathyroidism Serum calcium Serum phosphate Alkaline phosphatase PTH


Primary Increase Decrease Increase Increase
Secondary Decrease Increase or Decrease Increase Increase
Tertiary Increase Decrease Increase Increase

https://t.me/DentalBooksWorld
1386 Triumph's Complete Review of Dentistry

HYPOPARATHYROIDISM
• Low calcium/High phosphate (hyperphosphatemia)
• 25 hydroxycholecalciferol → 25(OH)-D-1Hydroxylase
• 1,25 Dihydroxycholecalciferol (active Vitamin D)
• So if PTH is decreased, there will be no conversion
• Albright’s osteodystrophy is not seen here
• If PTH is decreased, then there will be decreased mobilization of calcium from bone – resulting in hypocalcemia
• Decreased PTH/Normal PTH infusion response
PSEUDOHYPOPARATHYROIDISM
Hereditary disorder associated with signs and symptoms of hypoparathyroidism (i.e., decreased calcium + increased
phosphate) but with elevated PTH levels
Clinical Features:
• Increased PTH
• Decreased calcium
• Increased phosphate
• Decreased response of urinary cAMP to PTH
• Round face
• Short stature
• Cataract
• Brachydactyly
• Short 4th, 5th metacarpals
• Exostosis
• Impairment in olfaction and taste
• Obesity
PSEUDOPSEUDOHYPOPARATHYROIDISM (PPHP)
It refers to the subset of patients who carry the abnormal GNAS1 mutation (GS ALPHA subunit deficiency, with Albright’s
hereditary osteodystrophy, but no endocrine/biochemical changes or altercations)

HYPERCALCEMIA
Causes
• Overactive parathyroid glands. The most common cause of hypercalcemia, overactive parathyroid glands
(hyperparathyroidism) may stem from a small noncancerous tumor on one or more of the four parathyroid glands.
• Cancer. Lung cancer and breast cancer, as well as some cancers of the blood, can increase your risk of hypercalcemia. Spread
of cancer (metastasis) to your bones also increases your risk of hypercalcemia.
• Other diseases. Certain diseases, such as tuberculosis and sarcoidosis, may raise blood levels of Vitamin D, which stimulates
your digestive tract to absorb more calcium.
• Immobility. People with cancer or other diseases that cause them to spend a great deal of time sitting or lying down may
develop hypercalcemia. Over time, bones that do not bear weight release calcium into the blood.
• Medications. Certain drugs – such as lithium, which is used to treat bipolar disorder – may increase the release of parathyroid
hormone.
• Supplements. Taking excessive amounts of calcium or vitamin D supplements over time can raise calcium levels in your
blood above normal.
• Hereditary factors. A rare genetic disorder known as familial hypocalciuric hypercalcemia causes an increase of calcium in
your blood because of faulty calcium receptors in your body.
• Dehydration. A common cause of mild or transient hypercalcemia is dehydration, because when there is less fluid in your
blood, calcium concentrations rise.

https://t.me/DentalBooksWorld
Chapter 16 • General Medicine 1387

Management
• First line of Rx – Hydration
• Increased salt intake/saline
• Diuresis – Furosemide
• Bisphosphonate
• Glucocorticoids
• Plicamycin
• Calcitonin
• Dialysis
• Do not use thiazide diuretics

Conditions in Which Alkaline Phosphatase is Increased and Decreased


Increased Conditions:
• Paget’s disease
• Rickets
• Osteomalacia
• Hyperparathyroidism
• Bone disease in renal failure
• Lytic bone neoplasms
• Hereditary hypophosphatasia
Decreased Conditions:
• Hypoparathyroidism
• Multiple myeloma
• Hypothyroidism
• Achondroplasia
• Pernicious anemia
• Magnesium deficiency
• Chronic myeloid leukemia
• Wilson’s disease
Normal in – Osteoporosis

DISORDERS OF THYROID GLAND


THYROIDITIS
This is a category of thyroid diseases that involves inflammation of your thyroid gland. Thyroiditis can be caused by antibody
attacks on your gland, viruses, and bacteria. The types of thyroiditis include:
• Hashimoto’s thyroiditis
• Subacute granulomatous thyroiditis/De Quervain’s thyroiditis/painful subacute thyroiditis
• Acute infectious thyroiditis
• Painless thyroiditis/silent thyroiditis 
• Riedel’s thyroiditis/invasive thyroiditis
• Postpartum thyroiditis
Treatment depends on the type of thyroiditis and ranges from monitoring and nonsteroidal anti-inflammatory medications to
antibiotics and thyroid hormone replacement medication.

https://t.me/DentalBooksWorld
1388 Triumph's Complete Review of Dentistry

TOXIC MULTINODULAR GOITER


This is a disease where the thyroid gland develops multiple nodules and enlarges (goiter). Frequently, the nodules are
“functional,” meaning that apart from the thyroid gland’s hormone production, the nodules themselves also produce and
release thyroid hormone. Toxic multinodular goiter frequently causes hyperthyroidism.
The key risk factors for thyroid disease include:
• Iodine deficiency or excess
• Exposure to radioactivity or radioactive fallout
• Overconsumption of soy and goitrogenic foods
• Surgery or trauma to the neck area
• Pregnancy or recent childbirth
• Female gender
• Personal or family history of autoimmune disease
• Cigarette smoking

HYPOTHYROIDISM
The most common thyroid condition is hypothyroidism, which refers to a condition where you do not have enough thyroid
hormone. It has a number of causes:
• Hashimoto’s thyroiditis – An autoimmune condition that causes impairment or destruction of the thyroid gland.
• Postsurgical hypothyroidism – After all or part of the thyroid gland has been surgically removed.
• Postablation hypothyroidism – Results after radioactive iodine treatment (RAI) used for thyroid cancer, Graves’ disease, and
in some cases of hypothyroidism and nodules.
• Congenital hypothyroidism – When a baby is born without a thyroid gland or with a malformed gland that is not capable of
producing enough thyroid hormone.
• Iodine-deficiency hypothyroidism – Due to a deficiency of iodine in the diet.
• Drug- and supplement-induced hypothyroidism – Prescription medications and supplements that have the ability to cause
hypothyroidism.
• Goitrogen-induced hypothyroidism – Very high consumption of raw goitrogens – Foods that have chemicals in them that
slow the thyroid.
• Secondary/central hypothyroidism – Due to a defect in the functioning/communications of the pituitary gland and the
hypothalamus.
• Traumatic hypothyroidism – Serious trauma to the neck, such as whiplash or breaking the neck, has been linked to the onset
of hypothyroidism.
• Hypothyroidism of unknown origin/idiopathic hypothyroidism – There are cases where the thyroid becomes underactive
and no other underlying causes or diseases have been identified.

HYPERTHYROIDISM
This refers to a condition where the thyroid gland is overproducing thyroid hormone. It also has a number of causes:
• Autoimmune Graves’ disease – the most common cause of hyperthyroidism
• Autoimmune Hashimoto’s disease – sometimes causes periods of temporary hyperthyroidism
• Iodine-excess hyperthyroidism – results from overexposure or overconsumption of iodine
• Drug- and supplement-induced hyperthyroidism – results from several prescription drugs as well as certain over-the-
counter supplements
• Toxic multinodular disease – a condition that frequently causes overproduction of thyroid hormone
• Thyroiditis – certain forms can cause periods of hyperthyroidism
• Pituitary-induced hyperthyroidism – where the thyroid gland can become overstimulated by the pituitary gland and produce
excessive amounts of thyroid hormone
• Thyroid nodules – in some cases they can trigger overactivity of the surrounding thyroid gland, causing hyperthyroidism.
They can also, on their own, produce thyroid hormone.

https://t.me/DentalBooksWorld
Chapter 16 • General Medicine 1389

GOITER
This refers to a condition where the thyroid gland is enlarged in size. Goiter can result from autoimmune Hashimoto’s and
Graves’ diseases, iodine excess and deficiency, and nodular thyroid disease.
Symptoms
The symptoms of thyroid disease tend to reflect the gland’s underactivity (hypothyroidism), overactivity (hyperthyroidism),
autoimmune activation, and/or inflammation/enlargement/tenderness in the neck area (thyroiditis, nodules, cancer).
Common Symptoms of Hypothyroidism
• Fatigue
• Weight gain
• Inability to lose weight with diet and exercise
• Constipation 
• Infertility
• Feeling cold
• Hair loss (including the outer edge of the eyebrows) 
• Brain fog
• Muscle and joint pains/aches
Common Symptoms of Hyperthyroidism
• Anxiety
• Insomnia
• Panicky feeling 
• Tremors
• Exaggerated reflexes
• Elevated heart rate
• Diarrhea or loose stools
• Feeling overheated 
• Unexplained weight loss
In some cases, thyroid diseases and conditions can have no symptoms at all, such as thyroid cancer or certain types of thyroiditis.
Evaluation and Diagnosis
• A clinical examination
• Blood testing: Typically, this includes the thyroid stimulating hormone (TSH) test, free thyroxine (Free T4), and free
triiodothyronine (Free T3) antibodies testing to diagnose Hashimoto’s and Graves’ disease, and testing for reverse T3 levels.
• Imaging tests: A variety of imaging tests are done to further evaluate the size, shape, and function of the thyroid gland.
These tests also look at nodules or lumps and evaluate whether or not they are suspicious for thyroid cancer and require
further evaluation. These tests include the radioactive iodine uptake (RAI-U), CT scans, magnetic resonance imaging
(MRI), and ultrasound.
• Fine needle aspiration biopsy: Fine needle aspiration (FNA) biopsy is done when thyroid nodules are considered suspicious
and need to be tested for possible thyroid cancer.
Treatment
Hypothyroidism
This condition is treated with thyroid hormone replacement drugs. These are prescription medications that replace the
missing thyroid hormone in the body. 
• The most commonly prescribed thyroid hormone replacement drug is known generically as levothyroxine, a synthetic form
of the thyroid hormone thyroxine.
• There is also a synthetic form of the T3 hormone, known as liothyronine, which is sometimes added to levothyroxine for
T4/T3 combination treatment.

https://t.me/DentalBooksWorld
1390 Triumph's Complete Review of Dentistry

• Also, there is a hormone replacement drug called natural desiccated thyroid, sometimes abbreviated NDT or called “thyroid
extract.” While it has been available for more than a century and is still in use today, NDT is considered controversial by the
mainstream medical community.
Graves’ Disease and Hyperthyroidism
There are three key ways that Graves’ disease and chronic hyperthyroidism are treated:
1. Antithyroid drug treatment – Antithyroid drugs, including methimazole (Tapazole), carbimazole (Neo-Mercazole), and
propylthiouracil (PTU), can slow down the thyroid’s production of thyroid hormone.
2. Radioactive iodine treatment (RAI) – Given in a single dose, either in a capsule or drink, it enters the thyroid, radiates
thyroid cells, and damages and kills them. This shrinks the thyroid, slows down its function, and reverses hyperthyroidism.
3. Thyroid surgery/thyroidectomy – In some cases, this is performed as a treatment for Graves’ disease and hyperthyroidism,
especially for people who cannot tolerate antithyroid drugs or are pregnant and RAI is not an option.

Assessment of Thyroid Gland:


Disorder T4 T3 TSH
Primary hypothyroidism Decreased Decreased Increased
Primary hyperthyroidism Increased Increased Decreased
Subclinical hypothyroidism Normal Normal Increased
Subclinical hyperthyroidism Normal Normal Decreased
TSH secreting pituitary tumor Increased Increased Increased
Sick euthyroid/Secondary Decreased Decreased Decreased
hypothyroidism

SYNDROME OF INAPPROPRIATE ADH – SIADH


• Antidiuretic hormone secretion caused by excess vasopressin release
• Increased vasopressin release → increased absorption of water producing hyperosmolar
• Findings are opposite to DI (here less loss of water)

Cardinal Features:
• Plasma → hyponatremia, decreased osmolality
• Hypouricemia**
• Urine → increased sodium, increased osmolality

Treatment:
• Restrict fluid intake
• Lithium and demeclocycline → drugs for SIADH
• **Demeclocycline antagonizes ADH may lead to DI

PHEOCHROMOCYTOMA
Most common site of origin:
• Adrenal medulla
Most common extra adrenal site:
• Paravertebral sympathetic ganglions in organ of Zuckerkandl
**near aortic bifurcation
Hormones secreted in this condition are:
• Norepinephrine predominantly

https://t.me/DentalBooksWorld
Chapter 16 • General Medicine 1391

• Epinephrine
• Dopamine
Vanillylmandelic acid in urine
CT scan is the investigation of choice for adrenal pheochromocytoma
MIBG scan is the investigation of choice for locally recurrent, metastatic, ectopic, and extra-adrenal pheochromocytoma

GENETICS
AUTOSOMAL DOMINANT
• Appears in both sexes with equal frequency
• Both sexes transmit the trait to their offspring
• Does not skip generations
• Affected offspring must have an affected parent, unless they possess a new mutation
• When one parent is affected (heterozygous) and the other parent is unaffected, approximately half of the parents will be affected
• Unaffected parents do not transmit the trait
AUTOSOMAL RECESSIVE
• Appears in both sexes with equal frequency
• Trait tends to skip generations
• Affected offspring are equally born to unaffected parents
• When both parents are heterozygous, approximately one-fourth of the offspring will be affected
• Appears more frequently among the children of consanguine marriages

Autosomal Dominant Autosomal Recessive


Cannot be recessive as affected parents Cannot be dominant as unaffected parents
could not have an unaffected offspring could not have an affected offspring

Parents MUST be heterozygous Parents MUST be heterozygous

X-Linked Dominant X-Linked Recessive


Mode of inheritance cannot be confirmed Sex linkage cannot be confirmed
However, 100% incidence of affected However, high incidence of affected sons
daughters from an affected father suggests X-linked recessive inheritance
suggests X-linked dominant inheritance

https://t.me/DentalBooksWorld
1392 Triumph's Complete Review of Dentistry

AUTOSOMAL DOMINANT DISEASES


• Huntington’s disease: CAG triplet, chorea, anticipation, delayed age of onset, alteration of gene expression due to
hypermethylation of histones
• Hereditary spherocytosis: Membrane defect – ankyrin and spectrin
• Hereditary hemorrhagic telangiectasia (Osler–Weber–Rendu syndrome): mucosal telangiectasias, recurrent epistaxis,
skin discoloration, AVM, GI bleeding (hematochezia)
• Hypertrophic obstructive cardiomyopathy: Most common cause of sudden cardiac death, beta-myosin heavy chain
mutation is the commonly observed genetic defect to cause the disease
• Hypokalemic periodic paralysis: episodes of muscle weakness, severe paralysis, channelopathy, ↓K+ serum (goes into
muscle cells)
• Achondroplasia: Fibroblast growth factor Receptor 3 gene (FGFR-3), chromosome 4
• Tuberous sclerosis: T-S genes; TSC-1 (hamartin), TSC-2 (tuberin), chromosome 16. Kidney angiolipomas, <3
rhabdomyomas, tubers in brain (nonmalignant), astrocytomas, seizures, developmental delay, MR, behavioral problems,
hamartomas, skin abnormalities (Shagreen patches, ash-leaf spots, sebaceous adenomas, facial angiofibromas)
• Osteogenesis imperfecta (except type VII): type 1 collagen
• Familial hypercholesterolemia: type IIa, LDL receptor deficiency, chromosome 19
• Familial Adenomatous Polyposis (FAP) and Peutz–Jeghers: APC gene, chromosome 5
• Marfan’s Syndrome: defective fibrillin, chromosome 15
• Myotonic Dystrophy: CTG triplet, anticipation. DMPK gene, abnormal expression of Myotonin Protein Kinase. Muscle
loss, <3 arrhythmia, testicular atrophy, frontal baldness, cataracts.
• Acute Intermittent Porphyria: porphobilinogen deaminase deficiency, dark, port-wine urine, delayed age of onset.
• Neurofibromatosis type I: NF1 gene, Von Recklinghausen, chromosome 17, sphenoid dysplasia, congenital pseudarthrosis,
scoliosis, meningiomas, gliomas, pheochromocytoma.
• Polydactyly: Ulnar, radial, central
• Polycystic Kidney Disease (ZD – adults): PCKD1 gene (85%) – chromosome 16; PCKD2 gene (15%) – chromosome 4;
large, round cysts. Assoc. w/ cysts in the liver, pancreas, berry aneurisms in circle of Willis, colonic diverticula, MVP
• Osteopetrosis Type II – adult type: thick, dense bones (marble bones), osteoclast dysfunction
• Waardenburg Syndrome: deafness, lateral displacement of inner canthi, hypertelorism, white patches of skin in ventral midline
• Ehlers–Danlos syndrome: it can be AR too. Collagen cross linking defect
• Retinoblastoma: Rb gene, chromosome 13
• Von Hippel Lindau: VHL tumor suppressor gene, chromosome 3, benign/malignant tumors (CNS), capillary
hemangioblastomas, clear cell carcinoma of kidney, pheochromocytoma, pancreatic neuroendocrine tumors
• Von Willebrand Disease: vWF deficiency
• Li–Fraumeni syndrome: p53 mutation, young age onset of malignancy, <45 years, family hx, cancer everywhere (breast,
brain, leukemia, adrenal cortex)

AUTOSOMAL RECESSIVE DISEASES


• A beta lipoproteinemia: decrease ApoB-48, Apo B-100; pigmentary degeneration of retina, acanthocytes, steatorrhea,
cerebellar ataxia
• Acute Fatty Liver of Pregnancy: microvesicular steatosis in the liver, mitochondrial dysfunction in the oxidation of fatty
acids leading to an accumulation in hepatocytes
• Alkaptonuria: homogentisate oxidase deficiency, increase homogentisic acid, ochronosis, dark blue urine
• Acyl CoA Dehydrogenase deficiency (MCAD): fasting hypoglycemia, no ketone bodies, dicarboxylic acidemia
• Bernard–Soulier Syndrome: gp1b deficiency, prolonged bleeding time
• Bloom Syndrome: chromosome 15, Ashkenazi Jews, BLM gene
• Carpenter Syndrome: craniosynostosis, acrocephaly, craniofacial asymmetry, increased ICP, cutaneous syndactyly,
polydactyly, mild-profound MR

https://t.me/DentalBooksWorld
Chapter 16 • General Medicine 1393

• Chediak–Higashi Syndrome: LYST gene mutation, microtubule polymerization defect, no phagolysosome formation,
albinism
• Chondrodystrophy: normal-sized trunk and abnormally short limbs and extremities (dwarfism)
• Congenital Adrenal Hyperplasia: 17 alpha or 21 beta, or 11 beta hydroxylase deficiency; enlargement of adrenal glands
due to increased ACTH
• Congenital Hepatic Fibrosis: hepatic (periportal) fibrosis, irregularly shaped proliferating bile duct, portal hypertension,
renal cystic disease
• Cystic Fibrosis: CFTR gene, Phe508, defective chloride channel, chromosome 7
• Dubin–Johnson Syndrome: direct hyperbilirubinemia, cMOAT deficiency, black liver
• Endocardial Fibroelastosis: restrictive/infiltrative cardiomyopathy, thick fibroelastic tissue in endocardium of young
children, <2 years old
• Familial Mediterranean fever: chromosome 16, recurrent autoinflammatory disease, characterized by F, PMN disfx,
sudden attacks pain/inflammation (7 types of attacks – abdominal, joints, chest, scrotal, myalgias, erysipeloid, fever).
Complication: AA-amyloidosis
• Fanconi Anemia: genetic loss of DNA crosslink repair, often progresses to AML, short stature, ↑incidence of tumors/
leukemia, aplastic anemia
• Friedreich’s Ataxia: GAA triplet repeat, chromosome 9, neuronal degeneration, progressive gait and limb ataxia, areflexia,
hypertrophic cardiomyopathy, axonal sensory neuropathy, kyphoscoliosis, dysarthria, hand clumsiness, loss of sense of
position, impaired vibratory sensation.
• Gaucher’s disease: glucocerebrosidase deficiency, glucocerebroside accumulation, femur necrosis, crumpled paper
inclusions in macrophages.
• Ganzmann’s thrombasthenia: gpII bIIIa deficiency, deficient platelet aggregation.
• Hartnup Disease: tryptophan deficiency, leads to niacin deficiency, pellagra-like dermatosis
• Hemochromatosis: HFE gene, C282Y MC mutation, chromosome 6, unrestricted reabsorption of Fe+ in SI, iron deposits
in organs, bronze diabetes, DM1, malabsorption, cardiomyopathy, joint degeneration, increased iron, ferritin, TIBC.
Complications: liver cirrhosis, hepatocellular carcinoma
• Homocystinuria: due to B6 deficiency (defective cystathionine synthase) or due to B9,B12 deficiency (defective
homocysteine Methyltransferase), dislocated lenses (in and down), DVT, stroke, atherosclerosis, MR
• Krabbe’s disease: Galactocerebrosidase deficiency, galactocerebroside accumulation, globoid cells, optic atrophy, peripheral
neuropathy
• Leukocyte Adhesion Defect (LAD): CD-18+ deficiency, omphalitis in newborns, chronic recurrent bacterial infections,
increased WBC count, no abscess or pus formation
• Metachromatic Leukodystrophy: Arylsulfatase A deficiency, sulfatide accumulation, demyelination (central and
peripheral), ataxia, Dementia (DAD)
• Niemann–Pick Disease: sphingomyelinase deficiency, sphingomyelin accumulation, HSM, cherry-red macula, foam cells
• Phenylketonuria (PKU): phenylalanine hydroxylase deficiency, Phe accumulation, MR, microcephaly, diet low in Phe!!!
also in pregnancy, avoid aspartame, musty odor
• Polycystic Kidney Disease (children): ARPKD, progressive and fatal renal failure, multiple enlarged cysts perpendicular to
renal capsule, association with liver cysts. Bilateral palpable mass.
• Rotor Syndrome: direct hyperbilirubinemia, cMOAT deficiency, no black liver
• SCID: ADA def. and rag-1, rag-2 def., bubble-boy
• Shwachman–Diamond Syndrome: exocrine pancreatic insufficiency (2 MCC in children after CF), bone marrow
dysfunction, skeletal abnormalities, short stature
• Sickle Cell Disease and Trait: Hb S, beta globin chain, chromosome 11, position 6, nucleotide codon change (glutamic
acid to valine), vaso-occlusive crisis (pain), autosplenectomy, acute chest pain syndrome, priapism, hand–foot syndrome,
leg ulcers, aplastic crisis, drepanocytes and Howell–Jolly bodies, hemolytic anemia, jaundice, bone marrow hyperplasia
• Tay–Sachs Disease: Hexosaminidase A deficiency, GM2 accumulation, cherry-red macula, onion skin lysosomes
• Thalassemia: alpha (chromosome 16, gene deletion), beta (chromosome 11, point mutation)
• Werner’s Disease: adult progeria

https://t.me/DentalBooksWorld
1394 Triumph's Complete Review of Dentistry

Wilson’s Disease: Chromosome 13, WD gene, ATP7B gene (encodes for Copper transporting ATPase), copper accumulation
in liver, brain (putamen), eyes (Descemet’s membrane: Kayser–Fleischer ring), decreased ceruloplasmin
Xeroderma Pigmentosa: defective excision endonuclease, no repair of thymine dimers caused by UV radiation, excessive
freckling, multiple skin cancers

X-Linked Dominant Diseases


• Alport’s Syndrome: “hereditary nephritis,” type IV collagen deficiency, alternating thickening and thinning of GBM,
COL4A5 mutation, hearing loss, ocular abnormalities (lens and cornea), hematuria (gross or micro) since childhood
• Bruton’s Agammaglobulinemia: btk gene defect, no mature B cells or plasma cells, low lymphoid tissue, hepatitis,
enterovirus infections, first 6 months protected by maternal Ab (no symptoms)
• Becker’s Muscular Dystrophy: altered dystrophin gene, later onset than Duchenne’s, slow progression, relatively normal
life span, less severe, rare cardiac involvement
• Chronic Granulomatous Disease (CGD): NAPDH oxidase deficiency, recurrent catalase (+) infections, nitro blue
tetrazolium test negative (yellow)
• Congenital Aqueductal Stenosis: MCC of congenital obstructive hydrocephalus
• Color blindness (red-green): cannot distinguish shades of red and green (usually blue-green)
• Duchenne’s Muscular Dystrophy: dystrophin gene mutation (Xp21), absent dystrophin protein, MC and severe of muscular
dystrophies, normal until 5 years old, short life span (<30 years old), progressive muscle weakness, calf pseudohypertrophy,
<3 failure, arrhythmias, respiratory insufficiency and infections (decreased mucociliary clearance). Pneumonias CC of
death.
• Fabry’s Disease: alpha galactosidase A, Ceramide trihexoside accumulation, angiokeratomas, renal failure, peripheral
neuropathy
• Glucose 6-P Dehydrogenase (G6PD) Deficiency: chronic hemolytic anemia, MCC of enzymatic deficiency HA, Heinz
bodies, bite cells. Triggers are infections, drugs (antimalarial), fava beans
• Hemophilia A and B: factor VIII and IX deficiency, respectively. PTT prolongation
• Hunter Disease: iduronate sulfatase deficiency, heparan sulfate accumulation, no corneal clouding, aggressive behavior
• Inherited Nephrogenic Diabetes Insipidus: V2 receptors in collecting duct do not respond to ADH
• Lesch–Nyhan Syndrome: HGPRT1 deficiency, spastic cerebral palsy, self-mutilation, hyperuricemia, oral crystals in
diapers, early death
• Menkes Disease: ATP7A gene mutation (copper efflux protein), Cu+ is lysyl oxidase cofactor, Cu+ accumulates in intestine
and kidneys; deficient in other tissues = deficient collagen cross linking; steely “kinky” hair, MR, arterial tortuosity,
hypotonia
• Ornithine Transcarbamoylase Deficiency: urea cycle, orotic aciduria + hyperammonemia (no megaloblastic anemia),
orotic acid accumulation, increased glutamine. Cerebral edema, lethargy, vomiting, hyperventilation, convulsions, coma,
death.
• SCID: IL-receptor, Gamma chain deficiency
• Wiskott–Aldrich Syndrome: combined partial B and T immunodeficiency, IgM deficiency, thrombocytopenia, eczema

X-Linked Recessive Diseases


• Red–Green Color Blindness: relatively benign in nature, also known as Daltonism, cannot distinguish certain shades of
red and green
• Hemophilia A: a blood clotting disorder, mutation of the Factor VIII gene, deficiency of Factor VIII, blood in the urine and
stool, deep bruises, large, unexplained bruises, excessive bleeding, bleeding gums, frequent nosebleeds, pain in the joints,
tight joints

https://t.me/DentalBooksWorld
Chapter 16 • General Medicine 1395

• Hemophilia B (also known as Christmas disease): blood clotting disorder, mutation of the Factor IX gene, deficiency of
Factor IX, symptoms similar to hemophilia A
• Duchenne Muscular Dystrophy: mutation in the dystrophin gene, rapid progression of muscle degeneration, loss of
skeletal muscle control, respiratory failure, and death
• Becker’s Muscular Dystrophy: milder form of Duchenne, slow progressive muscle weakness of the legs and pelvis
• X-Linked Ichthyosis: hereditary deficiency of the steroid sulfatase enzyme, scaling of the skin, particularly on the neck,
trunk, and lower extremities, extensor surfaces are typically the most severely affected areas
• X-Linked Agammaglobulinemia (XLA): do not generate mature B cells, affects the body’s ability to fight infection,
untreated XLA patients are prone to develop serious and even fatal infections
• Glucose-6-Phosphate Dehydrogenase Deficiency: causes nonimmune hemolytic anemia due to multiple causes which
include infection, exposure to certain medications, chemicals, or foods. Also known as “favism,” as it can be triggered by
chemicals existing naturally in broad (or fava) beans

QUICK FACTS

CARDIOVASCULAR SYSTEM
• In ASD pulmonary flow is greater than systemic flow
• ASD is a defect in the atrial septum and not in the patent foramen ovale
• Anatomic closure of Patent ductus arteriosus → 1–3 months
• Most common mode of inheritance of congenital heart disease → Multifactorial
• Cardiac abnormality seen in Noonan’s syndrome → ASD
• Cardiac abnormality seen in Marfan’s syndrome → MR, MVP, AR, Aortic aneurysm
• Recurrent pulmonary infection is not seen in which congenital heart disease? → TOF
• Ellis–van Creveld syndrome → ASD
• Turner’s syndrome → Coarctation of aorta
• MC heart valve involved in IV drug user → Tricuspid valve
• Source of infecting microorganism → Skin (MC agent – S. aureus)
• Cardiac murmurs are almost always present in IE except in patients with early acute endocarditis/IV drug abuse

RESPIRATORY SYSTEM
Respiratory diseases causing clubbing are:
• Fibrosing alveolitis
• Empyema
• Bronchiectasis
• Lung abscess
• Bronchogenic carcinoma

https://t.me/DentalBooksWorld
1396 Triumph's Complete Review of Dentistry

Primary pulmonary hypertension:


• Increased jugular venous pressure
• Loud P2
• Parasternal heave
• Nitric oxide used in treatment
• Interstitial pneumonia is most frequently due to influenza virus
Most common cause of community-acquired pneumonia:
Streptococcus pneumoniae > mycoplasma
Most common cause of lobar pneumonia:
Streptococcus pneumoniae (>90%)
Most common cause of hospital-acquired pneumonia:
Enteric Gram-negative bacilli – E. coli, Pseudomonas, Klebsiella
CURB 65 Criteria:
Used for predicting mortality in community-acquired pneumonia

HEPATITIS ANTIGEN CONCEPTS


Serology:
Acute Infection: Antigens and a “new” (acute) antibody 
• HBsAg (+): the first to appear, this measures viral particles
• HBeAg (+): this means active viral replication
• IgM HBcAb (+)
• HBV-DNA (+)
Window period: This is the period between the end of the detection of HBsAg and the beginning of HBsAb – “new” antibody
• IgM HBc Antibody (+): the only one positive
Resolved Infection: “old” (chronic) antibodies
• IgG HBc Antibody (+): this means you got the infection, but cleared it!
• IgG HBs Antibody (+)
Immunization: “old” antibody
• IgG HBs Antibody (+): the only one positive
Chronic Infection: Everything positive, except for the immunization Ab, similar to acute state, but the core antibody is IgG
• HBsAg (+)
• HBeAg (+)
• IgG HBc Antibody (+)
• HBV-DNA (+)

MEN TYPE 1
• Most common manifestation – Hyperparathyroidism
• Second most common manifestation – Pancreatic tumors
• Third most common manifestation – Pituitary tumors
• So, most commonly manifestation is Hyperparathyroidism > Pancreatic polypeptide (Gastrinoma) > Prolactinoma

https://t.me/DentalBooksWorld
Chapter 16 • General Medicine 1397

MULTIPLE CHOICE QUESTIONS

HEMATOLOGY
1. A blood profile showing raised MCV and normal MCHC is found in
A. Sideroblastic anemia B. Lead toxicity
C. Vitamin B12 deficiency D. Thalassemia
2. Half-life of a normal RBC is
A. 60 days B. 90 days
C. 100 days D. 120 days
3. Pancytopenia with plasmacytosis in marrow is seen in
A. Fanconi’s anemia B. Aplastic anemia
C. IDA D. Myelodysplastic syndrome
4. Mentzer’s index ratio <13 indicates
A. IDA B. Sickle cell anemia
C. Thalassemia D. Liver diseases
5. Most sensitive index of iron deficiency anemia is
A. Decreased serum iron and increased TIBC B. Increased serum iron and decreased TIBC
C. Increased serum iron and increased TIBC D. Decreased serum iron and decreased TIBC
6. All of the following diseases will have decreased Osmotic Fragility (OF) except
A. Beta-thalassemia B. Megaloblastic anemia
C. Hemoglobinopathies D. Hereditary spherocytosis
7. Cold hemolysins are?
A. IgM B. IgG
C. IgA D. IgE
8. CLL transformation into diffuse large B-cell lymphoma is known as
A. Evan’s syndrome B. HELLP syndrome
C. Sezary syndrome D. Richter syndrome
9. Poor clot retraction is a diagnostic feature of
A. DIC B. ITP
C. Hemophilia D. VwB disease
10. Among the clotting factors, one of the following factor’s deficiency does not produce bleeding. Pick it out
A. Factor VII B. Factor X
C. Factor XI D. Factor XII
11. Which of the following does not need treatment?
A. Neuroblastoma B. ALL
C. CLL D. T-cell leukemia
12. Sezary syndrome is associated with
A. T-cell lymphocytic leukemia B. B-cell lymphoma
C. Mycosis Fungoides D. Follicular lymphoma
13. Abnormality of platelet aggregation results in
A. Glanzmann’s thrombasthenia B. Bernard–Soulier’s syndrome
C. TAR syndrome D. Wiskott–Aldrich syndrome
14. Abnormality of platelet secretion (alpha-granules) will cause
A. Chediak–Higashi syndrome B. vWD
C. TAR syndrome D. Grey platelet syndrome
15. Most common infection transmitted by blood transfusion is
A. EBV B. CMV
C. Hepatitis D. HIV

https://t.me/DentalBooksWorld
1398 Triumph's Complete Review of Dentistry

16. Which of the following virus is related to post-transfusion hepatitis?


A. HBV B. HCV
C. HDV D. HEV
17. Cryoprecipitate contains all except
A. Factor 1 B. Factor 8
C. Factor 9 D. Factor 13
18. Survival rate of transfused platelets is
A. 1–7 days B. 2–5 days
C. 2 weeks D. 24 hours
19. Platelet transfusion in ITP is indicated if platelet counts are
A. <40,000 B. <30,000
C. <20,000 D. <10,000
20. Pyridoxine deficiency causes
A. Sideroblastic anemia B. Aplastic anemia
C. Sickle cell anemia D. Megaloblastic anemia
21. A newborn baby presented with profuse bleeding from the umbilical cord stump after birth. The other clinical
examinations, PT, APTT are found to be within normal range. The most probable diagnosis is
A. Factor VII deficiency B. Glanzmann’s thrombasthenia
C. VwB disease D. Bernard–Soulier disease
22. A patient with previously normal hemoglobin suffered a sudden massive acute hemorrhage. He is most likely to show
all of the following, except
A. High reticulocyte count B. High neutrophil count
C. Low PCV D. Low MCV
23. Which of the following indicates bad prognosis in AML?
A. Deletion of X or Y chromosome B. T (8:12)
C. Monosomy D. Nucleophosmin mutation
24. Which of the following is not seen in Sickle cell anemia?
A. Thrombocytosis B. Leukocytosis
C. Leukopenia D. None of the above
25. Which of the following is the major criteria for diagnosis of polycythemia vera, according to the proposed new WHO
criteria?
A. Low erythropoietin vera B. Presence of JAK–2 mutation
C. Thrombocytosis D. Increased LAP
26. To best confirm a nonmatched blood transfusion, which of the following investigation should be done immediately?
A. Indirect Coombs test B. Antibody in patient’s serum
C. Antibody in donor serum D. Direct Coombs test
27. A 23-year-old asymptomatic female patient has MCV – 70, ferritin – 100 g/L, Hb – 10 g%. What is the most likely
cause?
A. Vitamin B12 deficiency B. Folate deficiency
C. Thalassemia trait D. IDA
28. DIC is associated with which one of the following types of AML?
A. M3 B. M4
C. M6 D. M1
29. Drug of choice for CML is
A. Infliximab B. Hydroxyurea
C. Imatinib D. IFN
30. Ring sideroblasts are characteristically seen in
A. Acute lymphoid leukemia B. Acute myeloid leukemia
C. Myelodysplastic syndrome D. Anemia of chronic disease

https://t.me/DentalBooksWorld
Chapter 16 • General Medicine 1399

31. The malignancy of which cell lineage is related with autoimmune hemolytic anemia?
A. Pre B-cell B. Pre T-cell
C. T-cell D. B-cell
32. Bone marrow biopsy is mandatory to establish the diagnosis of
A. Acute leukemia B. Hairy cell leukemia
C. Thalassemia D. Megaloblastic anemia
33. Ingestion of aspirin and other NSAID drugs predominantly shows
A. Prolonged BT B. Prolonged PT
C. Prolonged APTT D. Prolonged CT
34. Which one of the following is not a major criteria for diagnosis of multiple myeloma?
A. Lytic bone lesions B. Plasmacytoma on tissue biopsy
C. Bone marrow plasmacytosis D. M spike >3 g% for IgG, >2% for IgA
35. Rappaport’s classification is used for
A. Hodgkin’s lymphoma B. Non-Hodgkin’s lymphoma
C. Burkitt’s lymphoma D. Multiple myeloma

CARDIOVASCULAR SYSTEM
1. Which of the following about atherosclerosis is true?
A. Intake of unsaturated fatty acid associated with decreased risk
B. Thoracic aorta involvement is more severe than abdominal aorta involvement
C. Extent of lesion in veins is same as that in arteries
D. Hypercholesterolemia does not always increase the risk of atherosclerosis perse
2. A 14-year-old girl on exposure to cold has pallor of extremities followed by pain and cyanosis. In later ages of life she
is prone to develop
A. SLE B. Scleroderma
C. Rheumatoid arthritis D. Histiocytosis
3. A patient develops sudden palpitation with HR 150 beats/min, regular. What could be the cause?
A. PSVT B. Sinus tachycardia
C. Ventricular tachycardia D. Atrial flutter with block
4. Which of the following is least likely to be associated with infective endocarditis?
A. ASD B. VSD
C. Coarctation of aorta D. Aortic stenosis
5. Which of the following is true about endocarditis?
A. It tends to occur in high pressure areas
B. It tends to occur in right side of the heart
C. ASD patients are highly vulnerable to endocarditis
D. Endocarditis occurs more frequently in patients with pure stenosis than those with valvular incompetence
6. CPKMB rises within or seen at ___________ hours of acute MI
A. 2–3 hours B. 4–8 hours
C. Within first half an hour D. 8–10 hours
7. CPKMB enzyme peaks at ___________ hours after MI?
A. 48 B. 72
C. 24 D. 36
8. Cardiac marker Troponin-I returns to normal after
A. 3–4 days B. 4–7 days
C. 7–10 days D. After 10 days
9. Which cardiac marker is accepted as a sensitive marker for diagnosis for acute MI in healthy individuals?
A. Troponin-T B. Troponin-I
C. CK–MB D. CPKMB

https://t.me/DentalBooksWorld
1400 Triumph's Complete Review of Dentistry

10. All of the following statements about third heart sound are true except
A. Occurs due to rapid filling of the ventricles during atrial systole
B. Seen in constrictive pericarditis
C. Seen in Atrial septal defect
D. Seen in Ventricular septal defect
11. All of the following congenital heart diseases are associated with loud S3 except
A. VSD B. PDA
C. ASD D. TOF
12. Atrial heart sound is
A. 1st heart sound B. 2nd heart sound
C. 3rd heart sound D. 4th heart sound
13. Beck’s triad is seen in
A. Constrictive pericarditis B. Cardiac tamponade
C. Restrictive cardiomyopathy D. Tetralogy of Fallot
14. Beck’s triad of cardiac tamponade consists of all except
A. Hypotension
B. Absent heart sounds
C. Increased JVP with a prominent x descent and absent y descent
D. Arrhythmia
15. Floppy valve syndrome – find the wrong statement
A. Also known as Barlow’s syndrome B. Most common in females
C. Patients are usually symptomatic D. Investigation of choice is ECG
16. Best predictor for future risk of cardiovascular events is
A. C-reactive protein B. Lipoprotein A
C. LDL Cholesterol D. Homocysteine
17. Cardiovascular complications of HIV infection include all of the following except
A. Infective endocarditis B. Kaposi’s sarcoma
C. Pericardial effusion D. Aortic aneurysm
18. Earliest ECG change seen in Hyperkalemia is
A. Peaked T waves B. Prolonged PR interval
C. Prolonged QRS duration D. Loss of P waves
19. Congestive heart failure is associated with
A. Hyponatremia B. Decreased serum urea levels
C. Levels of circulating norepinephrine is decreased D. Decreased ADH and Aldosterone
20. Most common cause of prosthetic valve endocarditis is
A. S. albus B. S. viridans
C. S. aureus D. Enterococci
21. Which is not true about juvenile rheumatoid arthritis?
A. Fever B. Arthritis
C. Anemia D. Leukopenia
22. Loud first heart sound is seen in
A. Mitral stenosis B. Mitral regurgitation
C. MV prolapse D. All of the above
23. Most commonest cause of pulsus paradoxus is
A. Pericardial effusion B. Adhesive pericarditis
C. Constrictive pericarditis D. Chylopericardium
24. Pulsus bisferiens may be seen in all except
A. Combined AS + AR B. Hypertrophic subaortic stenosis
C. Normal individuals D. None of the above

https://t.me/DentalBooksWorld
Chapter 16 • General Medicine 1401

25. Pulsus paradoxus is associated with


A. Cardiac tamponade B. Patent ductus arteriosus
C. Hypertension D. ASD
26. Pulses bisferiens is seen in
A. Aortic stenosis B. Aortic regurgitation
C. Mitral regurgitation D. Mitral stenosis
27. Anacrotic pulse is seen in
A. AS B. AR
C. MS D. MR
28. Cyanosis is not seen in
A. Anemia B. Polycythemia vera
C. Tetralogy of Fallot D. Acute attack of asthma
29. Blood pressure is defined as the product of
A. Systolic pressure × pulse rate B. Cardiac output/peripheral resistance
C. Cardiac output × stroke volume D. Cardiac output × peripheral resistance
30. The most commonest mode of inheritance of congenital heart disease is
A. Polygenic B. Multifactorial
C. Monogenic D. Autosomal dominant
31. RACE protocol is for
A. Atrial fibrillation B. Paroxysmal supraventricular tachycardia
C. Heart failure D. Diabetes mellitus
32. Atrial fibrillation is seen in all except
A. Primary hyperthyroidism B. Mitral stenosis
C. Mitral regurgitation D. Dilated cardiomyopathy
33. Boot-shaped heart is seen in
A. Ebstein’s anomaly B. Tetralogy of Fallot
C. Transposition of greater arteries D. All of the above
34. Area of mitral valve orifice is
A. 4–6 cm sq. B. 2–4 cm sq.
C. 6–8 cm sq. D. 1–2 cm sq.
35. Duroziez’s sign is seen in?
A. AR B. AS
C. MR D. MS
36. Half-life of alteplase is
A. 1 minute B. 3 minutes
C. 6 minutes D. 10 minutes
37. Which of the following is a polygenic disorder?
A. Prolonged QT syndrome B. Marfan’s syndrome
C. Hypertrophic cardiomyopathy D. Coronary artery disease

ENDOCRINOLOGY
1. A girl presented with severe hyperkalemia and peaked T waves on ECG. Most rapid way to decreased serum potassium
level is
A. Calcium gluconate IV B. Oral resins
C. Insulin + glucose D. Sodium bicarbonate
2. Drug effectively used in treatment of Cushing’s syndrome is
A. Insulin B. Prednisolone
C. Ketoconazole D. High dose multivitami
3. Corticosteroid hormones are responsible for all of the following functions, except
A. Mobilization of glucose from the liver B. Control of total body water
C. Inhibition of lymphocyte proliferation D. Increase in muscle mass
https://t.me/DentalBooksWorld
1402 Triumph's Complete Review of Dentistry

4. Addison’s disease is associated with serum levels of


A. Elevated potassium and low sodium B. Elevated Sodium and low potassium
C. Elevated sodium and potassium D. Decreased sodium and potassium
5. Adrenal cortex does not produce
A. Glucocorticoid hormone B. Adrenocorticotropic hormone
C. Androgen D. Aldosterone
6. Nelson syndrome is
A. ACTH producing tumor in pituitary B. Ectopic ACTH producing tumor
C. Unilateral adrenal hyperplasia D. Bilateral adrenal hyperplasia
7. Hormone responsible for stimulating appetite is
A. Ghrelin B. Obestatin
C. Motilin D. Leptin
8. Choose a statement that is not true about gigantism
A. In most cases it is because of pituitary tumor
B. It is diagnosed by measurement of fasting growth hormone level
C. It occurs only if high growth hormone is present before fusion of bone epiphysis
D. Growth hormone is inhibited by increase in blood glucose
9. The most common hormonal disturbance of pituitary gland is, increased secretion of
A. Prolactin B. Growth hormone
C. Antidiuretic hormone D. Thyroid secreting hormone
10. Which of the following endocrine tumors is most commonly seen in MEN I?
A. Insulinoma B. Gastrinoma
C. Glucagonoma D. Somatostatinoma
11. Cortisol is secreted from
A. Adrenal medulla B. Adrenal cortex – zona fasciculata
C. Adrenal cortex – zona reticularis D. Adrenal cortex – zona glomerulus
12. Diabetes insipidus is due to deficiency of which hormone?
A. Atrial natriuretic peptide B. Vasopressin
C. Aldosterone D. Insulin
13. Antidiuretic hormone has greatest influence on kidney at
A. Cortex B. Medulla
C. DCT D. PCT
14. Long-term management of hypercalcemia does not include
A. Bisphosphonates B. Calcitonin
C. Hydration D. Loop diuretics
15. Which medication should be avoided in thyroid storm?
A. Beta blockers B. Glucocorticoids
C. Insulin D. Aspirin
16. Growth hormone –
A. Directly stimulates growth of cartilage and bone B. Levels are subnormal in acromegaly
C. Promotes lipolysis in adipose tissue D. Enhance protein breakdown
17. What are the major effects of excess cortisol?
A. Catabolic + immunosuppression B. Catabolic + anabolic
C. Catabolic + immunostimulation D. Anabolic + immunosuppression
18. Insulin acts/signal via
A. JAK/STAT pathway B. Tyrosine kinase pathway
C. IP3 D. cGMP
19. Adrenal hemorrhage and insufficiency due to Neisseria meningitidis known as
A. Addison’s disease B. Cushing’s syndrome
C. Waterhouse–Friderichsen syndrome D. Simmonds disease

https://t.me/DentalBooksWorld
Chapter 16 • General Medicine 1403

20. The following drug acts by increasing the insulin sensitivity in peripheral tissue
A. Sitagliptin B. Metformin
C. Exenatide D. Rosiglitazone
21. Which one of the following is an intermediate-acting insulin?
A. Lispro B. Aspart
C. Regular D. NPH
22. Subclinical hypothyroidism is characterized by
A. Increased T4, Increased T3, and Normal TSH B. Increased T4, Normal T3, and Normal TSH
C. Normal T4, Normal T3, and Increased TSH D. Normal T4, Increased T3, and Increased TSH
23. Hurthle cells are seen in all except
A. Hashimoto thyroiditis B. Hurthle cell thyroid adenoma
C. Follicular cell carcinoma D. Papillary adenoma
24. HLA associated with diabetes mellitus are
A. DR 3 and 4 B. DR 34 and 7
C. DR 6 and 9 D. DR 5
25. Thyroiditis that is associated with viral etiology is
A. Hashimoto thyroiditis B. Riedel’s thyroiditis
C. Subacute thyroiditis D. None of the above
26. Most dangerous complication of diabetes ketoacidosis is
A. Cerebral edema B. Venous thrombosis
C. ARDS D. Myocardial infarction
27. Jod-Basedow disease is
A. Iodine-induced hyperthyroidism
B. Iodine-dependent transient suppression of thyroid gland
C. Excess increase in thyroid hormone
D. Decreased synthesis of thyroid hormone
28. Which one among the following is false about Maturity Onset Diabetes of Young (MODY)?
A. Inheritance is monogenic B. Positive family history – autosomal dominant
C. Obesity is an uncommon feature D. Insulin-dependent diabetes
29. Cushing’s syndrome is characterized by all except
A. Hypokalemia B. Hypernatremia
C. Hypertension without edema D. Increased plasma renin activity
30. Rebound hyperglycemia after an incident of hypoglycemia is known as
A. Dawn phenomenon B. Somogyi phenomenon
C. Insufficient insulin D. Diabetic ketoacidosis
31. Causes of early morning hyperglycemia in diabetes patients using insulin is
A. Insufficient insulin B. Dawn phenomenon
C. Somogyi phenomenon D. All of the above
32. Gene involved in MEN TYPE 2A is
A. Menin gene chr 11 B. RET gene chr 10
C. CDKN-1B D. Menin gene chr 10
33. Metabolic syndrome – all among the following are true except
A. Also known as insulin resistance syndrome B. Hypertension is seen
C. Fasting blood sugar level will be >100 mg% D. Metabolic alkalosis is seen
34. Which thyroiditis mimics anaplastic carcinoma?
A. Hashimoto thyroiditis B. Riedel’s thyroiditis
C. De Quervain’s thyroiditis D. All of the above
35. Treatment for myxedema coma is
A. Liothyronine B. Levothyroxine
C. Amiodarone D. Iodide

https://t.me/DentalBooksWorld
1404 Triumph's Complete Review of Dentistry

CENTRAL NERVOUS SYSTEM


1. Resting (static) tremor is seen in
A. Cerebellar lesion B. Thyrotoxicosis
C. Parkinson’s disease D. Hepatic encephalopathy
2. A patient reports with speaking difficulty. On examination, it is found that his comprehension is lost. There is
impaired naming and repetition, but fluency is preserved. What is the possible diagnosis?
A. Broca’s aphasia B. Wernicke’s aphasia
C. Anomic aphasia D. None of the above
3. Which of the following drug is ineffective in the treatment of status epilepticus?
A. Carbamazepine B. Lorazepam
C. Fosphenytoin D. Diazepam
4. Patients with Alzheimer’s disease show a significant depletion of
A. Dopamine B. Ach
C. Serotonin D. Both A and C
5. Ash-leaf spots, Shagreen patch of skin are found in
A. Tubular sclerosis B. Multiple sclerosis
C. Neurofibromatosis D. Sturge–Weber syndrome
6. Cerebral Ischemia occur when cerebral blood flow is less than
A. 10 ml/100 g/min B. 15 ml/100 g/min
C. 20 ml/100 g/min D. 25 ml/100 g/min
7. Which of the following drug is NOT used in the prophylaxis of migraine?
A. Flunarizine B. Levetiracetam
C. Propranolol D. Topiramate
8. Bilateral Babinski sign is a characteristic of
A. Subarachnoid hemorrhage B. Basal ganglia and thalamic hemorrhage
C. Pontine hemorrhage D. Cerebellar hemorrhage
9. Identify the side involved in the below given image


A. Right-sided upper motor neuron lesion B. Left-sided upper motor neuron lesion
C. Right-sided lower motor neuron lesion D. Left-sided lower motor neuron lesion
10. In which of the following conditions neurons are exclusively affected?
A. Spinocerebellar ataxia B. Supranuclear palsy
C. Corticobasal degeneration D. Multisystem atrophy
11. Neurofibrillary tangles are seen in all of the following except
A. Alzheimer’s disease B. Progressive supranuclear palsy
C. Senile dementia D. Huntington’s disease

INFECTIONS
1. Tissues surrounding an infection by _____ show crepitus
A. Staphylococcus B. Streptococcus
C. Clostridium tetani D. C. perfringens
2. Congenital rubella syndrome may cause all the following, except
A. Patent ductus arteriosus B. Spontaneous abortion
C. Sensory nerve deafness D. None of the above

https://t.me/DentalBooksWorld
Chapter 16 • General Medicine 1405

3. Hepatitis E infection spreads by


A. Feco-oral route B. Parenteral
C. Blood transfusion D. Rectal
4. All of the following statements are true for Hepatitis E, except:
A. Commonest cause for epidemic in India
B. Affects children more than adults
C. Causes fulminant hepatic failure
D. Effects pregnant females and responsible for their mortality
5. Class II of HIV associated disease is descriptive of
A. Acute infections B. Asymptomatic infections
C. Persistent generalized lymphadenopathy D. All of the above
6. Secondary infections in HIV infected patient are classified under
A. Class I B. Class II
C. Class III D. Class IV
7. EB virus is associated with carcinoma of
A. Carcinoma of larynx B. Carcinoma of bladder
C. Nasopharyngeal carcinoma D. Chronic lymphocytic leukemia
8. Leukopenia is seen in the following except
A. Measles B. Viral hepatitis
C. Typhoid D. Bacterial endocarditis
9. Increased transaminase is seen in
A. Hepatitis A B. Subacute bacterial endocarditis
C. Myocardial infarction D. Jaundice
10. Herpes virus is an example of
A. DNA virus B. RNA virus
C. RNA and DNA virus D. Partially DNA and partially RNA
11. The most serious complication of urticaria is
A. Anaphylactic shock B. Heart block
C. Cerebral edema D. Septicemia
12. Miliary tuberculosis is seen when tubercle bacteria spread via
A. Lymphatics B. Waldeyer’s ring
C. The bloodstream D. The urinary system
13. Usual incubation period for hepatitis B infection is
A. 1–2 days B. 1–2 weeks
C. 1–6 months D. 1 year
14. Among the following, which can be known as an early manifestation of a patient infected with HIV virus?
A. Elevation of p16 B. Unexplained fear and weight loss
C. Kaposi’s sarcoma D. Hairy cell leukoplakia
15. The serum of a patient who has received recombinant Hepatitis B vaccine is positive for:
A. Hepatitis B surface antigen (HbsAg) B. Hepatitis B core antibody (Anti-HbC)
C. Hepatitis B surface antibody (Anti-HBs) D. Anti-HbC and Anti-HBs
16. CSF glucose levels are normal in
A. Viral meningitis B. Pyogenic meningitis
C. Tubercular meningitis D. Carcinomatous meningitis
17. Which among the following is not an occupational hazard for a dentist?
A. AIDS B. Enteric fever
C. Hepatitis B D. Herpes
18. Which of the following can be used for treatment of Pseudomonas infection?
A. Nafcillin B. Cloxacillin
C. Methicillin D. Piperacillin

https://t.me/DentalBooksWorld
1406 Triumph's Complete Review of Dentistry

19. Acute epiglottis is most commonly caused by


A. H. influenzae B. Cloxacillin
C. S. pneumoniae D. RSV
20. Scrofula is a variant of TB affecting
A. Skin B. Lymph node
C. Spine D. Lungs
21. Serological test for HIV becomes positive in ____________ weeks after infection
A. 4 weeks B. 8 weeks
C. 12 weeks D. 36 weeks
22. All the following are characteristics of congenital syphilis, except
A. Ghon’s complex B. Interstitial keratitis
C. Mulberry molars D. Notched incisors
23. The part of the spinal cord that is involved in poliomyelitis is
A. Posterior horn B. Lateral column
C. Posterior column D. Anterior horn
24. The most dependable definitive diagnosis of typhoid is by
A. Blood culture B. Widal test
C. Bone marrow culture D. Stool culture
25. In AIDS, there is a deficiency of
A. CD2 B. CD3
C. CD4 D. CD8
26. Tertiary syphilis most frequently involves the
A. CNS B. Liver
C. Testes D. Aorta
27. A primary syphilitic lesion of the male genitalia is characterized by
A. Soft, ragged ulcer B. Punched out painless ulcer with indurated edges
C. Raised, flat grayish, most confluent lesions D. Necrotic rubbery lesion
28. Flask-shaped ulcers are associated with
A. Bacillary dysentery B. Amoebiasis
C. Typhoid D. Tuberculosis
29. Most common cause of multiple fetal anomalies is
A. Syphilis B. Tetracycline
C. Rubella D. Rubeola
30. Homologous serum jaundice is caused by
A. Virus A B. Virus B
C. Virus C D. None of these
31. In Human Immunodeficiency Virus (HIV) infection, diffuse lymphadenopathy in a person who is clinically well is
usually a sign of which of the following?
A. Lymphoma B. Kaposi’s sarcoma
C. Tuberculosis D. Persistent generalized lymphadenopathy
32. Immunoglobulin administration is life saving in one of the following conditions
A. Rabies B. Clostridium
C. Poliomyelitis D. Typhoid
33. HIV is a
A. Retrovirus B. Flavivirus
C. Oncovirus D. Arbovirus
34. Not a common sequel of mumps is
A. Sterility B. Diab

You might also like